Você está na página 1de 2692

CONTEÚDO

APRESENTAÇÃO 2

A NOVA OLIMPÍADA BRASILEIRA DE MATEMÁTICA 5


Introdução

OLIMPÍADA BRASILEIRA DE MATEMÁTICA 7


Problemas de treinamento

OLIMPÍADA BRASILEIRA DE MATEMÁTICA 12


Problemas das provas das primeiras fases Júnior e Sênior 1997

A OLIMPÍADA DE MAIO 22
Introdução

III OLIMPÍADA DE MAIO 23


Primeiro nível

III OLIMPÍADA DE MAIO 29


Segundo nível

9a. OLIMPÍADA DO CONE SUL 35


Introdução

8a. OLIMPÍADA DO CONE SUL 36


Problemas

ARTIGOS
NÚMEROS MÁGICOS E CONTAS DE DIVIDIR 38
Carlos Gustavo Tamm de Araújo Moreira

COMO PERDER AMIGOS E ENGANAR PESSOAS 41


Nicolau C. Saldanha

DOIS PROBLEMAS SOBRE GRAFOS 51


Paulo Cezar Pinto Carvalho

PROBLEMAS PROPOSTOS 58

AGENDA OLÍMPICA 60
Sociedade Brasileira de Matemática

COORDENADORES REGIONAIS 61
APRESENTAÇÃO
EUREKA!, a revista da Olimpíada Brasileira de Matemática faz
parte de um grande projeto que tem como objetivo principal contribuir
decisivamente para a melhoria de ensino de Matemática em nosso país.
O que planejamos realizar é descrito (de forma resumida), nesta
apresentação.

DOS OBJETIVOS

O ensino de Matemática hoje no Brasil difere pouco do ensino


praticado há 20 anos. A cada ano, livros novos são editados repetindo quase
sempre o mesmo estilo e os mesmos conteúdos dos anteriores. Existem
hoje no Brasil bons livros de Matemática dedicados aos alunos tanto do
ensino fundamental quanto do ensino médio. Entretanto, o que lhes falta é
um ingrediente que, no mundo de hoje, é fundamental: o estímulo à
criatividade. Entendemos que não é suficiente para a formação do futuro
cidadão um aprendizado burocrático da Matemática e percebemos a
importância de estimular os alunos desde tenra idade a resolver problemas
novos e desafiantes, propiciando o desenvolvimento da imaginação e da
criatividade.

O programa de Olimpíadas de Matemática é reconhecido em todos


os países do mundo desenvolvido como o mais eficiente instrumento para
atingir esse objetivo. Aproveitando o natural gosto dos jovens pelas
competições, as Olimpíadas de Matemática têm conseguido estimular
alunos a estudar conteúdos além do currículo escolar e, também, por outro
lado, aumentar e desenvolver a competência dos professores.

DO PROJETO

O programa de Olimpíadas de Matemática existe no país há 19


anos. Sempre foi pequeno e dedicado a encontrar jovens talentos para a
Matemática ou para ciências afins e, neste aspecto, cumpriu sua finalidade.
Temos hoje brilhantes matemáticos e cientistas de renome mundial que
tiveram origem nas Olimpíadas de Matemática. Entretanto, reconhecemos

EUREKA! N 1, 1998

2
Sociedade Brasileira de Matemática

que, com esta atividade, pode-se fazer muito mais. Com parceria do IMPA
(Instituto de Matemática Pura e Aplicada) e com a SBM (Sociedade
Brasileira de Matemática), foi submetido ao CNPq um projeto que
pretende contribuir para a melhoria do ensino de Matemática no Brasil
utilizando as Olimpíadas de Matemática como mecanismo propagador.
Este projeto teve boa acolhida e neste momento estamos iniciando um
trabalho de grandes dimensões que, para ter seus objetivos cumpridos,
necessitará também (e principalmente) do apoio e da ajuda de diversos
segmentos da sociedade: alunos, professores, escolas, universidades,
secretarias de educação etc.

Nossa atividade estará centrada na resolução de problemas e


atingirá alunos desde a 5 a. série do ensino fundamental até a 3 a. série do
ensino médio e, naturalmente, seus professores. Para a divulgação deste
material, utilizaremos esta revista, cartazes mensais com diversas
informações sobre atividades olímpicas e um site na Internet.

Para movimentar os jovens realizar-se-à anualmente uma nova


Olimpíada Brasileira de Matemática, que estará dividida em níveis de
acordo com a escolaridade do aluno. Além disso, estaremos apoiando a
realização de competições de Matemática em nível regional.

Para os professores, estão sendo planejados cursos de


aperfeiçoamento em diversas regiões do país, também colocaremos à
disposição, através do site da Internet, um vasto banco de problemas e uma
biblioteca especializada localizados na nossa sede no IMPA.

DA REVISTA

EUREKA!, a revista da Olimpíada de Matemática é uma publicação


dedicada principalmente aos alunos e professores da escola secundária a
qual será editada quatro vezes ao ano e terá basicamente a seguinte
estrutura:

a) Seção de problemas de treinamento com soluções, dividida, em


três níveis: para os alunos de 5 a. e 6a. séries, para os alunos de 7 a. e
8a. séries e para os alunos de ensino médio. Esta seção pretende

EUREKA! N 1, 1998

3
Sociedade Brasileira de Matemática

fornecer aos alunos material para estudo e pesquisa dirigidos à


Olimpíada Brasileira, que será realizada nesses mesmos três níveis.

b) Seção de artigos de Matemática elementar, tratando de assuntos


que complementem o currículo escolar e que também abordem
novos conteúdos. Estes artigos estarão classificados em iniciante,
intermediário ou avançado, de acordo com o estágio de
desenvolvimento dos leitores aos quais se destinem os artigos.

c) Seção de Problemas de diversos níveis, sem solução, para que os


leitores possam pesquisar e enviar suas soluções para a revista,
sendo as melhores publicadas nos números seguintes.

d) Seção de Cartas dos Leitores, em que alunos e professores terão


possibilidade de fazer quaisquer perguntas. Todas as cartas serão
respondidas e as mais relevantes serão publicadas.

e) Agenda, para informarmos todas as atividades ligadas às


Olimpíadas de Matemática no Brasil e no exterior.

DOS CARTAZES

Para que nossa atividade permaneça viva durante o ano,


enviaremos todos os meses para as escolas cadastradas um cartaz da
Olimpíada Brasileira de Matemática. Esse cartaz conterá todas as
informações sobre as atividades olímpicas e também o Problema do Mês,
em cada um dos três níveis. Contamos com que muitos alunos fiquem
interessados nesse desafio e que nos enviem soluções.

Todos os colégios cadastrados receberão gratuitamente a revista


EUREKA! e os cartazes mensais. Para cadastrar um colégio, basta entrar em
contato conosco dando o nome do colégio, endereço completo e o nome de
um professor responsável para receber a correspondência.

EUREKA! N 1, 1998

4
Sociedade Brasileira de Matemática

Rio de Janeiro, abril de 1998


A NOVA OLIMPÍADA BRASILEIRA DE MATEMÁTICA
A Olimpíada Brasileira de Matemática será realizada a partir deste
ano de 1998 de forma bastante diferente da que vinha sendo praticada nos
últimos anos. Isto porque agora passa a atingir os alunos desde a 5 a. série
do ensino fundamental. Antes, a Olimpíada Brasileira de Matemática era
principalmente um instrumento para detectar talentos e desenvolvê-los,
mas, agora, tem também por objetivo promover em âmbito nacional a
melhoria do ensino de Matemática nas escolas, com o desenvolvimento
conjunto de alunos e professores.
A Olimpíada Brasileira de Matemática, a partir deste ano, não será
apenas uma competição. Para a preparação dos alunos e para o
aperfeiçoamento dos professores, a OBM distribuirá aos colégios revistas e
cartazes contendo farto material para estudo e pesquisa, dedicados a cada
faixa de escolaridade e desenvolvimento dos alunos. A realização das
provas é uma finalização (sempre parcial) dessa atividade.
A Olimpíada Brasileira de Matemática será realizada em três fases
e em três níveis. São eles:

Nível 1 - para alunos da 5ª. e 6ª. séries do ensino fundamental.


Nível 2 - para alunos da 7ª. e 8ª. séries do ensino fundamental.
Nível 3 - para alunos do ensino médio (antigo 2º. grau).

Para cada um dos níveis, a OBM terá três fases. Na primeira,


qualquer aluno interessado poderá participar. Para participar das outras,
existirá um critério de promoção.

- A prova da primeira fase será de múltipla escolha, contendo de 20


a 25 questões sobre conteúdo adequado a cada um dos níveis de
escolaridade. Nestas questões, serão incluídas algumas que
dependam de alguma criatividade, porém não fugindo dos
conteúdos tradicionais das escolas.

- A prova da segunda fase será discursiva e constará de 6 problemas,


exigindo uma maior dose de iniciativa e criatividade.

- A prova da terceira fase será também discursiva.

EUREKA! N 1, 1998

5
Sociedade Brasileira de Matemática

- As provas da primeira e segunda fases da OBM serão realizadas


nas escolas que desejarem participar dessa atividade. A correção
das provas também será realizada nas escolas, com o salutar
envolvimento de seus professores, de acordo com critérios
determinados pela organização. Os coordenadores oferecerão
locais alternativos aos alunos que desejarem participar da
Olimpíada, caso o colégio onde realizam seus estudos não venha a
organizar a atividade.

- A prova da terceira fase será realizada em um local central


designado pelo coordenador local e corrigida pelo comitê
organizador da OBM.

Para tornar viável a realização de uma competição de Matemática


em âmbito nacional, foi criada uma estrutura operativa. As atividades de
elaboração das provas, edição da revista, publicação dos cartazes etc. serão
centralizadas na Secretaria da Olimpíada Brasileira de Matemática,
localizada no IMPA (Rio de Janeiro). Para apoiar as atividades no país,
existem hoje cerca de 30 coordenadores regionais que darão assistência às
escolas de sua área de atuação. Cada colégio participante da OBM ficará,
portanto, ligado ao coordenador regional mais próximo, que fornecerá toda
a assistência necessária.
Em 1998, a Olimpíada Brasileira de Matemática será realizada nas
seguintes datas:

Primeira fase: Sábado, 6 de junho


Segunda fase: Sábado, 12 de setembro
Terceira fase: Sábado, 24 de outubro (níveis 1, 2 e 3) e
Domingo, 25 de outubro (nível 3)

A Olimpíada Brasileira de Matemática não é de forma alguma uma


competição entre colégios. Ela pretende essencialmente despertar nos
alunos o gosto pelo estudo da Matemática através da resolução de
problemas novos, estimulando o desenvolvimento da imaginação e da
criatividade. O aspecto da competição naturalmente existe, mas jamais
estará ligado a grupos, equipes, colégios, cidades ou regiões. Desejamos
deixar bem claro que uma medalha oferecida pela Olimpíada Brasileira de
Matemática é um reconhecimento ao esforço individual do aluno premiado,
mas representa também o coroamento de um trabalho em que centenas ou

EUREKA! N 1, 1998

6
Sociedade Brasileira de Matemática

milhares de anônimos alunos também se desenvolveram. E isto, no fundo,


é o que importa.
OLIMPÍADA BRASILEIRA DE MATEMÁTICA
Problemas de treinamento para a Primeira fase

Primeiro nível

1) Num quadrado formado por 9 quadrados menores e do mesmo


tamanho, queremos escrever um X e um O, de forma que eles não
fiquem vizinhos, isto é, os quadrados em que se encontram não
podem ter um lado ou um vértice comum. O desenho abaixo
mostra uma dessas possibilidades:

De quantas maneiras podemos localizar os dois sinais, respeitadas


as condições apresentadas?

a) 32 b) 20 c) 64 d) 18 e) 12

2) Jacira consegue datilografar 20 páginas de um manuscrito em 4


horas e Joana o faz em 5 horas. Ainda restam 900 páginas do
manuscrito para datilografar. Se as duas começarem a datilografar
no mesmo instante essas páginas, quantas páginas deverá pegar a
mais lenta, de forma que ambas terminem juntas?

a) 225 b) 500 c) 400 d) 450 e) 180

3) O professor Epaminondas, no primeiro dia de aula, apostou que,


entre os alunos daquela classe, pelo menos dois fariam aniversário
no mesmo dia do mês. O professor tinha certeza de que ganharia a
aposta, pois naquela classe o número de alunos era maior ou igual
a:

EUREKA! N 1, 1998

7
Sociedade Brasileira de Matemática

a) 15 b) 32 c) 28 d) 31 e) 30
4) Seu Pedro possui três lotes quadrados: um deles tem lado de 10
metros, e os outros dois têm lados de 20 metros cada. Seu Pedro
quer trocar os três lotes por um outro lote quadrado, cuja área seja
a soma das áreas daqueles três lotes. O novo lote deverá ter lado de
medida:

a) impossível de obter b) 24 metros c) 25 metros


d) 40 metros e) 30 metros

5) Um jogo consiste em partir da casa 1 à casa 36 numa trilha com


casas numeradas de 1 a 36. Os dois jogadores começam na casa 1 e
o avanço de casas depende do lançamento de dois dados cúbicos
comuns.

- Se a soma dos pontos for par, o jogador avança 3 casas.


- Se a soma dos pontos for ímpar, o jogador avança 1 casa.
- Se o jogador ultrapassar a última casa, retorna à casa 1.
- A ordem com que os jogadores iniciam suas jogadas é definida por
alguma forma de sorteio.

Ganha quem parar primeiro na casa 36.


O menor número de jogadas que alguém pode fazer e ganhar é

a) 37 b) 13 c) 12 d) 14 e) 17

Segundo nível

1) A equação do 2. grau ax2 + bx – 3 = 0 tem –1 como uma de suas


raízes. Sabendo que os coeficientes a e b são números primos
positivos, podemos afirmar que a2 + b2 é igual a:

a) 29 b) 89 c) 17 d) 13 e) 53

EUREKA! N 1, 1998

8
Sociedade Brasileira de Matemática

2) Você já conhece o quadrado mágico de ordem 3: a soma dos


números das linhas, das colunas e das diagonais é 15. A figura a
seguir mostra uma das oito possibilidades de escrever os números
no quadrado:

O único número que não pode mudar de posição em todos esses


quadrados mágicos é:

a) 1 b) 3 c) 5 d) 7 e) 9

3) No modo SP, o aparelho de videocassete grava exatamente duas


horas e, no modo EP, grava quatro horas de filme, com menor
qualidade. Carlinhos quer gravar um filme de 136 minutos, com a
melhor qualidade possível. Ele decidiu começar no modo EP e
terminar no modo SP. Após quantos minutos de gravação no modo
EP ele deve passar ao modo SP ?

a) 20 b) 16 c) 8 d) 32 e) 68

4) Os pontos A, B e C são vértices de um triângulo cujos lados


medem 3, 4 e 5 cm e pertencem ao interior de uma circunferência,
da qual estão a uma distância de 1 cm. O raio da circunferência,
em centímetros, é:

a) 5 b) 7 c) 2,5 d) 4,2 e) 3,5

5) Um número de dois algarismos não nulos é igual ao dobro do


produto desses algarismos. Esse número pertence ao conjunto:

a){11,12,..., 30} b){31,32,..., 50}


c){51,52,..., 70} d){71,72,..., 90}

EUREKA! N 1, 1998

9
Sociedade Brasileira de Matemática

e){91,92,..., 99}

Terceiro nível

1) Considere três circunferências concêntricas ( mesmo centro T ) de


raios 1, 2 e 3, respectivamente. Considere um triângulo cujos
vértices pertencem, um a cada uma das circunferências. Sabendo
que o triângulo tem área máxima sob essas condicões, podemos
afirmar que, para este triângulo, o ponto T é o:

a) baricentro b) incentro c) circuncentro d) ortocentro


e) ex-incentro

2) Dada a função f: Z Z ( Z é o conjunto dos números inteiros)


definida por

x – 1 se x é ímpar
f(x) = e
x + 1 se x é par,

podemos afirmar que o número de soluções da equação


f(x) = f(2x) é:

a) 1 b) 2 c) 3 d) 4 e) 0

3) Na seqüência de inteiros positivos a1, a2,..., ak,…, para 1 i  k,o


termo ai é o i-ésimo ímpar positivo; para i > k, o termo ai é a média
aritmética dos termos anteriores. Podemos concluir que a2k é igual
a:

a) k2 b)k c)2k d) 0 e) k

4) Os vértices de um triângulo têm coordenadas (0,0), (3,1) e (1,7),


respectivamente. As retas que passam pelos vértices e por um
ponto T no interior do triângulo dividem-no em 6 triângulos de
mesma área. Então:

EUREKA! N 1, 1998

10
Sociedade Brasileira de Matemática

a) T = (3,6) b) T = (2,4) c) T = (4/3, 8/3)


d) T = (2,8) e) T = (2/3, 4/3)
5) Para quantos valores reais de p a equação x3 – px2 + px –1 = 0 tem
todas as raízes reais e inteiras ?

a) 1 b) 2 c) 3 d) 4 e) 5 ou mais

6) Considere o conjunto P dos pontos (x,y) do R2 tais que x e y sejam


inteiros. Por exemplo, (1,1)  P. Tome agora uma circunferência
de diâmetro igual a 5, de forma que em seu interior haja o maior
número possível de pontos de P. Esse número é:

a) 10 b) 16 c) 20 d) 14 e) 21

Nota: Veja as respostas na página 21.

Você sabia… que os antigos egípcios não usavam frações com


numerador maior que 1 nem somavam frações iguais de
numerador 1 ?
Assim por exemplo, eles se referiam ao número 2/5 como 1/3 +
1/15. Veja o problema 9 na página 60.

EUREKA! N 1, 1998

11
Sociedade Brasileira de Matemática

  
OLIMPÍADA BRASILEIRA DE MATEMÁTICA
Provas Júnior e Sênior 1997

 Até o ano passado a Olimpíada Brasileira de Matemática era realizada


em apenas dois níveis.

PRIMEIRA FASE JÚNIOR

PROBLEMA 1

O número N tem três algarismos. O produto dos algarismos de N é 126 e a


soma dos dois últimos algarismos de N é 11. O algarismo das centenas de
N é:

a)2 b) 3 c)6 d)7 e)9

PROBLEMA 2

A fortuna de João foi dividida da seguinte forma: um quinto para seu irmão
mais velho, um sexto do restante para seu irmão mais novo e partes iguais
do restante para cada um de seus 12 filhos. Que fração da fortuna cada
filho recebeu?

1 1 1 1 1
a) b) c) d) e)
20 18 16 15 14

PROBLEMA 3

No alvo abaixo, uma certa pontuação é dada para a flecha que cai na região
A e outra para a flecha que cai na região B. Alberto lançou 3 flechas: uma
caiu em B e duas em A, e obteve 17 pontos. Carlos também lançou 3
flechas: uma caiu em A e duas em B, e obteve 22 pontos. Quantos pontos
são atribuídos para uma flecha que cai na região A?

EUREKA! N 1, 1998

12
Sociedade Brasileira de Matemática

A
B

a) 2
b) 3
c) 4
d) 5 e) 6
PROBLEMA 4

Seja f uma função definida para todo x real, satisfazendo as condições:

 f(3) = 2

 f(x + 3) = f(x) f(3)
Então, f(–3) vale:
1
a)–6 b)0 c) d)2 e)–1
2

PROBLEMA 5

Quatro carros, de cores amarela, verde, azul e preta, estão em fila. Sabe-se
que o carro que está imediatamente antes do carro azul é menor do que o
que está imediatamente depois do carro azul; que o carro verde é o menor
de todos; que o carro verde está depois do carro azul; e que o carro amarelo
está depois do preto. O primeiro carro da fila:

a)é amarelo. b)é azul. c)é preto. d)é verde.


e) não pode ser determinado apenas com esses dados.

OBS: O primeiro da fila é o que vem antes de todos os outros.

PROBLEMA 6

EUREKA! N 1, 1998

13
Sociedade Brasileira de Matemática

64 jogadores de habilidades diferentes disputam um torneio de tênis. Na


primeira rodada, são feitos 32 jogos (os emparelhamentos são por sorteio),
e os perdedores são eliminados. Na segunda rodada, são feitos 16 jogos, os
perdedores são eliminados, e assim por diante. Se os emparelhamentos são
feitos por sorteio e não há surpresas ( se A é melhor que B, A vence B),
qual o número máximo de jogos que o décimo melhor jogador consegue
jogar?

a)2 b)3 c)4 d)5 e)6

EUREKA! N 1, 1998

14
Sociedade Brasileira de Matemática

PROBLEMA 7

O número de pares (x, y) de reais que satisfazem o sistema de equações

 x 2  xy  y 2  1 0
3 2 2 é igual a:

 x  x y  xy  x  y  2 0
a)0 b)1 c)2 d)3 e)4

PROBLEMA 8

Seja y = x2  x  1  x –3 . Se 1  x < 2, então y é igual a:

a)x + 4 b)3x – 2 c)x – 4 d)3x + 2 e)x – 2

PROBLEMA 9

Um gramado tem a forma de um quadrado com 10m de lado. Uma corda


tem um dos extremos fixado em um dos vértices, e no outro extremo está
amarrado um bode. Se o bode consegue comer metade da grama, então o
comprimento da corda é de aproximadamente:

a)8m b)7,5m c)7m d)6,5m e)6m

PROBLEMA 10
7 p 11
Se p e q são inteiros positivos tais que   , o menor valor que q
10 q 15
pode ter é:

a)6 b)7 c)25 d)30 e)60

EUREKA! N 1, 1998

15
Sociedade Brasileira de Matemática

PROBLEMA 11

A equação x  10  2 x  3  1  3x :

a) não tem solução.


b) tem uma única solução positiva.
c) tem uma única solução negativa.
d) tem duas soluções, uma positiva e outra negativa.
e) tem duas soluções, ambas negativas.

PROBLEMA 12

Como o médico me recomendou caminhadas, todo dia de manhã dou uma


volta (com velocidade constante) na quadra em que resido. Minha mulher
aproveita para correr (com velocidade constante) em volta do quarteirão.
Saímos juntos e chegamos juntos. Ela percorre a quadra no mesmo sentido
que eu e me ultrapassa duas vezes durante o percurso. Se ela corresse no
sentido contrário ao meu, quantas vezes ela cruzaria comigo?

a)2 b)3 c)4 d)5 e)6

PROBLEMA 13

Em uma urna há 28 bolas azuis, 20 bolas verdes, 12 bolas amarelas, 10


bolas pretas e 8 bolas brancas. Qual é o número mínimo de bolas que
devemos sacar dessa urna para termos certeza de que sacaremos pelo
menos 15 bolas da mesma cor?

a)58 b)59 c)60 d)71 e)72

PROBLEMA 14

Um ladrilho, em forma de polígono regular, foi retirado do lugar que


ocupava em um painel. Observou-se, então, que esse ladrilho, se sofresse
uma rotação de 40o ou de 600 em torno de seu centro, poderia ser encaixado
perfeitamente no lugar que ficou vago no painel. O menor número de lados
que pode ter esse ladrilho é:

EUREKA! N 1, 1998

16
Sociedade Brasileira de Matemática

a)6 b)9 c)12 d)15 e)18

PROBLEMA 15

No triângulo retângulo ABC da figura abaixo, está inscrito um quadrado.


Se AB = 20 e AC = 5, que porcentagem a área do quadrado representa da
área do triângulo ABC?
C

A B

a) 25%
b) 30%
c) 32%
d) 36%
e) 40%

PROBLEMA 16

Em certo país, a unidade monetária é o pau. Há notas de 1 pau e moedas de


meio pau, um terço de pau, um quarto de pau e um quinto de pau. Qual a
maior quantia, em paus, que um cidadão pode ter em moedas sem que
possa juntar algumas delas para formar exatamente um pau?

11 5 7 13
a) b) 1 c) 2 d) 2 e)
12 12 15 60
43
2
60

PROBLEMA 17

EUREKA! N 1, 1998

17
Sociedade Brasileira de Matemática

João e Pedro são vendedores e ganham R$ 1000,00 de salário e comissão


de 8% sobre as vendas. Em setembro, João ganhou R$ 2000,00 e Pedro
ganhou R$ 2 500,00. Nesse mês, as vendas de Pedro superaram as de João
em:

a) 20% b) 25% c) 30% d) 40% e) 50%

PROBLEMA 18

Um triângulo ABC, de lados AB = c, AC = b e BC = a, tem perímetro 2p.


Uma circunferência tangencia o lado BC e os prolongamentos dos lados
AB e AC nos pontos P, Q e R, respectivamente. O comprimento AR é igual
a:

a)p  a b)p  b c)p  c d)p e) 2p

PROBLEMA 19

P é um ponto interior a um quadrado ABCD. As distâncias de P aos


vértices A e D e ao lado BC são iguais a 10cm. O lado do quadrado mede:

a)10cm b)12cm c)14cm d)16cm e)18cm

PROBLEMA 20

A figura ao lado mostra três dados iguais. O número da face que é a


base inferior da coluna de dados:
a) é 1.
b) é 2.
c) é 4.
d) é 6.
e) pode ser 1 ou 4.

PRIMEIRA FASE SÊNIOR (*)

(*) Na prova da primeira fase sênior apareceram os problemas 5, 6, 7, 10,


12, 13, 14, 16, 17, 18, 19 e 20 da primeira fase júnior.

EUREKA! N 1, 1998

18
Sociedade Brasileira de Matemática

PROBLEMA 1

Quantos são os pares não-ordenados de inteiros positivos tais que, em cada


par, a soma do produto dos números do par com a soma dos números do
par com o módulo da diferença dos números do par seja igual a 20?

a)1 b)2 c)3 d)4 e)5


PROBLEMA 2

O número de pares (x, y) de inteiros que satisfazem a equação


x + y + xy = 120 é:

a)1 b)2 c)3 d)4 e)6

PROBLEMA 3
1 1
O conjunto-solução da inequação  é o conjunto:
x x 1
a) dos reais diferentes de 0 e de 1.
b) dos reais positivos diferentes de 1.
c) dos reais diferentes de zero e menores que 1.
d) dos reais entre 0 e 1.
e) vazio.

PROBLEMA 4

O número de valores inteiros de m para os quais as raízes da equação


x2 – (m + m2)x + m3 – 1 = 0 são inteiras é igual a:

a)0 b)1 c)2 d)3 e)4

PROBLEMA 5

Os vértices de um decágono regular convexo ABC...J devem ser coloridos


usando-se apenas as cores verde, amarela e azul. De quantos modos isso
pode ser feito se vértices adjacentes não podem receber a mesma cor?

a)1022 b)1024 c)1026 d)1524 e)1536

EUREKA! N 1, 1998

19
Sociedade Brasileira de Matemática

PROBLEMA 6

Uma das soluções inteiras e positivas da equação 19x + 97y = 1997 é,


evidentemente, (x0, y0) = (100,1). Além desse, há apenas mais um par de
números inteiros e positivos, (x1, y1), satisfazendo a equação. O valor de
x1+y1 é:

a)23 b)52 c)54 d)101 e)1997


PROBLEMA 7

Selecionam-se 3 vértices de um cubo. Qual é a probabilidade de eles


pertencerem a uma mesma face?
1 1 1 2 3
a) b) c) d) e)
5 6 7 7 7

PROBLEMA 8
k
Sendo k inteiro, o número de valores distintos de sen é igual a:
9
a)5 b)8 c)9 d)10 e)18

PROBLEMA 9

Para cobrir um terraço em forma de um retângulo ABCD, usa-se uma placa


plana de alumínio apoiada em quatro estacas verticais fixadas nos vértices
do retângulo. A placa fica inclinada em relação ao chão para escoar a água
das chuvas. Se as estacas que partem dos vértices A, B e C têm
comprimentos respectivamente iguais a 3, 4 e 5 metros, o comprimento da
que parte de D é:

a)3m b)4m c)5m d)6m e)8m

PROBLEMA 10

Se seu salário sobe 26% e os preços sobem 20%, de quanto aumenta o seu
poder aquisitivo?
a)5% b)6% c)7% d)8% e)9%

PROBLEMA 11

EUREKA! N 1, 1998

20
Sociedade Brasileira de Matemática

O reservatório de um caminhão-tanque tem a forma de um cilindro de


3
revolução com eixo horizontal e está cheio até da altura. A fração da
4
capacidade total do resevatório que está ocupada é de aproximadamente:

a)80% b)75% c)68% d)60% e)56%

PROBLEMA 12

O preço de um estacionamento é formado por um valor fixo para as duas


primeiras horas e um adicional por cada hora subseqüente. Se o
estacionamento por 3 horas custa R$ 5,00 e por 5 horas custa R$ 6,00,
quanto custa o estacionamento por 8 horas?

a)R$ 7,00 b)R$ 7,50 c)R$ 9,60 d)R$ 12,00


e)R$ 13,33

PROBLEMA 13

O número de soluções reais da equação x2 = 2x é:

a)0 b)1 c)2 d)3 e)4

 Você sabia… que  é aproximadamente:


3,141592653589793238462643383279502884197169399375105820974944592
30781640628620899862803482534211706798214808651328230664709384460
95505822317253594081284811174502841027019385211055596446229489549
30381964428810975665933446128475648233786783165271201909145648566
9234603486104543266482133936072602491412737245870066…?

EUREKA! N 1, 1998

21
Sociedade Brasileira de Matemática

Respostas dos problemas de treinamento


Primeira fase da Olimpíada Brasileira de Matemática.

Primeiro nível
1) a 2) c 3) b 4) e 5) b

Segundo nível
1) a 2) c 3) d 4) e 5) b

Terceiro nível
1) d 2) a 3) b 4) c 5) b 6) e

RESPOSTAS DA PRIMEIRA FASE -OBM JÚNIOR- 1997

1) d 8) a 15) c
2) b 9) a 16) d
3) c 10) b 17) e
4) c 11) e 18) d
5) c 12) c 19) d
6) e 13) b 20) c
7) c 14) e

RESPOSTAS DA PRIMEIRA FASE -OBM SÊNIOR- 1997

1) b 6) a 11) a
2) e 7) e 12) b
3) d 8) c 13) d
4) c 9) b
5) c 10) a

Você sabia que o erro da


aproximação
de  por 355/113 é menor que 3×10-7?

EUREKA! N 1, 1998

22
Sociedade Brasileira de Matemática

A OLIMPÍADA DE MAIO
Introdução

A Federação Iberoamericana de Competições de Matemática


organizou pela primeira vez a Olimpíada de Maio no ano de 1995.

A competição está dividida em dois níveis: estudantes menores de


13 anos e estudantes menores de 15 anos. O concurso se realiza por
correspondência e está baseado no modelo que segue a Olimpíada de
Matemática do Pacífico (APMO), concurso de longa distância com grande
tradição.

En maio deste ano se realizará a IV Olimpíada de maio, seguindo o


calendário seguinte:

 Limite para o envio dos problemas. 31 de janeiro

 Envio dos enunciados das provas


aos delegados de cada país: 11 de abril

 Prova: 09 de maio,
14h
 Limite da chegada dos listados e provas
para cada país: 13 de junho

 Envio dos resultados e diplomas 27 de junho


de honra aos delegados de cada país:

A seguir apresentamos as provas da III Olimpíada de Maio, realizada em


maio de 1997, com as respectivas soluções.

EUREKA! N 1, 1998

23
Sociedade Brasileira de Matemática

III OLIMPÍADA DE MAIO


Primeiro nível

Duração da prova: 3 horas.


Cada problema vale 10 pontos.
Não se pode usar máquina de calcular.
Não se pode consultar livros nem notas.

1) Num tabuleiro quadrado de 9 casas (de três por três), deve-se


colocar nove elementos do conjunto S = {0, 1, 2, 3, 4, 5, 6, 7,8,9 },
distintos um do outro, de modo que cada um deles fique numa casa
e se verifiquem as seguintes condições:

 As somas dos números da segunda e terceira fileira sejam,


respectivamente, o dobro e o triplo da soma dos números da
primeira fileira.
 As somas dos números da segunda e terceira coluna sejam,
respectivamente, o dobro e o triplo da soma dos números da
primeira coluna.

Mostre todas as formas possíveis de colocar elementos de S no


tabuleiro,cumprindo com as condições indicadas.

2)
A M B
No retângulo ABCD, M, N, P e Q
são os pontos médios dos lados.
Q N Se a área do triângulo sombreado
é 1, calcular a área do retângulo
ABCD.
D P C

EUREKA! N 1, 1998

24
Sociedade Brasileira de Matemática

3) Num tabuleiro de 8 por 8, colocam-se 10 fichas que ocupam, cada


uma, uma casa. Em cada casa sem ficha está escrito um número
entre 0 e 8, que é igual à quantidade de fichas colocadas nas casas
vizinhas. Casas vizinhas são as que têm um lado ou um vértice em
comum.Mostre uma distribuição das fichas que faça que a soma
dos números escritos no tabuleiro seja a maior possível.

4) Joaquín e seu irmão Andrés vão todos os dias para a aula no ônibus
da linha 62. Joaquín paga sempre as passagens.

Cada passagem tem impresso um número de 5 dígitos. Um dia,


Joaquín observa que os números das passagens, além de
consecutivos, são tais que a soma dos dez dígitos é precisamente
62.

Andrés pergunta para ele se a soma dos dígitos de algum dos boletos é 35
e, ao saber a resposta, pôde dizer corretamente o número de cada boleto.

Quais são estes números?

5) Quando Pablo fez 15 anos, fez uma festa convidando 43 amigos.


Ele tem uma torta com forma de polígono regular de 15 lados e
sobre ela coloca 15 velas.

As velas são colocadas de modo que entre velas e vértices nunca há três
alinhados (três velas quaisquer não estão alinhadas, nem duas velas
quaisquer com um vértice do polígono, nem dois vértices quaisquer do
polígono com uma vela).

Logo depois, Pablo divide a torta em pedaços triangulares, mediante cortes


que unem velas entre si ou velas e vértices, mas nunca se cruzam com
outros já realizados. Por que, ao fazer isto, Pablo consegue distribuir um
pedaço para cada um de seus amigos mas ele fica sem comer?

EUREKA! N 1, 1998

25
Sociedade Brasileira de Matemática

SOLUÇÕES

1) Nas condições do problema, a soma de todos os elementos do


quadrado deve ser um múltiplo de 6. Como 0 + 1 +2 +3 + 4 + 5 + 6 + 7 + 8
+ 9 = 45, que deixa resto 3 quando dividido por 6, as únicas possibilidades
para o conjunto dos números que aparecem no quadrado são 0, 1, 2, 3, 4,
5, 6, 7, 8 e 0, 1, 2, 4, 5, 6, 7, 8, 9 (note que retiramos respectivamente 3 e 9
que são os elementos que deixam resto 3 quando divididos por 6). No
primeiro caso a soma dos elementos da primeira linha (e da primeira
coluna) deve ser :

45  9 45  3
6 E no segundo: 7 
6 6

No primeiro caso, as possibilidades para o conjunto resp. C1 elementos da


primeira linha (resp. da primeira coluna) são:{0,1,5}, {0,2,4} e {1,2,3}

1.a) Se L1 = {0,1,5} e C1 = {0,2,4}, temos a única solução

 015   024 
   
 236  , e, por simetria, se L = {0,2,4} e C = {0,1,5}, temos 138 
1 1

 487   567 
   
a) Se L1 {0,1,5} e C1 = {1,2,3} ou L1 = {1,2,3} e C1 = {0,1,5},
temos

EUREKA! N 1, 1998

26
Sociedade Brasileira de Matemática

105  123 
   
 246  ou  048 
 387   567 
   
b) Se L1 = {0,2,4} e C1 = {1,2,3} ou L1 = {1,2,3} e C1 = {0,1,5},
temos

 204   213 
   
156  ou  057 
 378   468 
   
No segundo caso, as possibilidades para L1 (ou C1) são {0,1,6}, {0,2,5} e
{1,2,4} (não pode aparecer o 3).

Para cada escolha de L1 e C1 temos uma única possibilidade de solução, e


as soluções são:

EUREKA! N 1, 1998

27
Sociedade Brasileira de Matemática

01625 14205 

248,19570,16e8
5976847 895
EUREKA! N 1, 1998

28
Sociedade Brasileira de Matemática

2)

Sejam O o centro do retângulo e T


A M B a interseção de ON com BP.
Os triângulos OTP e OTB
são de áreas iguais, pois têm a
Q N mesma base e igual altura
O T (OP = NB). Como T é o ponto
médio, os triângulos OTP e
NTB são iguais, ambos são de
D P C área 1.Então, a área do OTP é
2 e, como é a metade da área de
ONB, a área de ABCD é 16.

3)

Cada ficha soma 1 em cada uma das casas vizinhas que estão livres de
ficha. Uma casa tem como máximo 8 vizinhas ( perde vizinhas se está
numa borda do tabuleiro). Vejamos que é impossível colocar as 10 fichas
em 10 casas isoladas, tais que nenhuma fique na borda do tabuleiro.
Podemos pensar que temos um tabuleiro de 6 por 6 –pois as casas das
bordas não interessam– ou dividimos em 9 setores 2 por 2, mediante
paralelas aos lados. Se queremos selecionar casas isoladas, em cada setor
podemos escolher ao máximo 1. São, em total, no máximo 9 casas isoladas.

Se uma casa fica na borda do tabuleiro, terá como máximo 5 vizinhas. Ou


seja ao colocar uma ficha ali, somará no máximo 5. Por outro lado, podem-
se colocar 8 fichas isoladas mais 2 nas quais as casas se toquem num
vértice; neste caso se perde só uma casa vizinha por cada uma delas. A
soma total é: 8 × 8 + 2 × 7 = 78. Duas possíveis distribuições são as
seguintes:

1 1 2 1 2 1 1 0
1 * 2 * 2 * 1 0
2 2 4 2 4 2 2 0
1 * 2 * 2 * 1 0
2 2 4 2 4 2 2 0
1 * 2 * 2 * 2 1

EUREKA! N 1, 1998

29
Sociedade Brasileira de Matemática

1 1 2 1 2 2 * 1
0 0 0 0 0 1 1 1

0 1 1 2 1 2 1 1
0 1 * 2 * 2 * 1
1 2 3 3 3 3 2 1
1 * 2 * 2 * 1 0
1 2 3 3 3 3 2 1
1 2 * 2 * 2 * 1
1 * 2 2 1 2 1 1
1 1 1 0 0 0 0 0

4) Se o número menor é abcde, e deve ser 9, pois caso contrário o


maior seria abcd (e + 1), e a soma dos dez dígitos é 2 ( a + b + c +
d + e ) + 1, que é ímpar e não poderia ser nunca 62.

Além disso, se o número menor acaba num número par de noves


(99 ou 9999), a soma dos dez dígitos também seria um número
ímpar.

Assim, o número menor é abcd9 (d não é 9) ou ab999 (b não é 9).

No primeiro caso, o outro número será abc(d + 1)0, e a soma dos dez
dígitos 2( a + b + c + d) + 10 = 62, ou seja, a + b + c + d = 26, e os
dígitos do número menor somam a + b + c + d +9 = 35. Haveria mais de
um número de cinco dígitos nessas condições 85859, 77669, etc.,) pelo que
a resposta que deu Joaquín à pergunta do seu irmão foi "não". Assim, os
números serão:

ab999 e a(b + 1)000, a soma dos dez dígitos é 2(a + b) + 28 = 62; assim:
a + b = 17, e como b não é 9, a = 9 e b = 8.

Os números dos boletos são: 98999 e 99000

5) Seja n o número de triângulos em que se pode dividir a torta com


as condições dadas.

Somaremos os ângulos interiores destes n triângulos de duas


formas:

EUREKA! N 1, 1998

30
Sociedade Brasileira de Matemática

(1) 180 n
(2) 360 × 15 + 180 (15 – 2)

Cada ponto interior (vela) contribui com 360, e a soma dos ângulos
interiores de um polígono convexo de L lados, é 180 (L – 2).
Portanto: 180 n = 360 × 15 + 180 × 13, onde n = 43.
Também pode utilizar-se a relação de Euler de um mapa plano:
R + V = L + 1 ( R = regiões, V = vértices e L = lados).
Então V = 30, 3R = 2L – 15 ( Todos os lados são comuns a duas regiões,
exceto os 15 lados do contorno do polígono). Assim, pois:
2 L  15
 30  L  1 Portanto: L = 72 e R = 43
3

Você sabi@
que a página web da Olimpíada Brasileira de Matemática é
http://www.obm.org.br
III OLIMPÍADA DE MAIO

Segundo nível

Duração da prova: 3 horas.


Cada problema vale 10 pontos.
Não se pode usar máquina de calcular.
Não se pode consultar livros nem notas.

1) Quantos são os números de sete algarismos que são múltiplos de


388 e terminam em 388?

2)

Em um quadrado ABCD de lado k,


A B colocam-se os pontos P e Q sobre os
lados BC e CD , respectivamente, de
P forma que PC = 3PB e QD = 2QC.
Sendo M o ponto de interseção de AQ e
M
EUREKA! N 1, 1998

D Q C 31
Sociedade Brasileira de Matemática

PD, determine a área do triângulo QMD


em função de k.

3) Temos 10000 fichas iguais com a forma de um triângulo


equilátero. Com esses pequenos triângulos se podem formar
hexágonos regulares sem superposições de fichas ou vazios.

Considere agora o hexágono regular que desperdiça a menor


quantidade possível de triângulos. Quantos triângulos sobram?

4) Nas figuras, assinalam-se os vértices com um círculo. Chamam-se


caminhos aos segmentos que unem os vértices. Distribuem-se
números inteiros não negativos nos vértices, e nos caminhos se
assinalam as diferenças entre os números de seus extremos.

Diremos que uma distribuição é elegante se aparecem nos caminhos todos


os números de 1 a n, em que n é o número de caminhos.

4 7 6
Veja um exemplo de 11 5 1
distribuição elegante: 9 1
11 2 8
2 7
0 10 9
12 3
12

Dar –se possível– uma distribuição elegante para as seguintes figuras. Em


caso de não ser possível, mostrar por quê.



  
  

EUREKA! N 1, 1998

32
Sociedade Brasileira de Matemática

5) Quais são as possíveis áreas de um hexágono com todos os ângulos


iguais e cujos lados medem 1,2,3,4,5 e 6 em alguma ordem?

SOLUÇÕES

1)

Solução A

O número se expressa como: n. 103 + 388, em que n é um número de


quatro cifras.

n. 103 + 388 = k. 388.


n. 103 = (k – 1).388.
Mas 388 = 22.97, então o número n de quatro cifras deve ser múltiplo de
97.
N = t.97, com 11 t  103. São 93 números.

Solução B

Para que um número multiplicado por 388 termine em 388, as últimas


cifras devem ser 001, 501, 251 ou 751.

O menor múltiplo de 388 que tem sete cifras é 388  2578, e o maior é 388
 25773.
Entre 2578 e 25773 temos:

23 números terminados en 001, desde 3001 até 25001


23 números terminados en 501, desde 3501 até 25501
23 números terminados en 251, desde 3251 até 25251
24 números terminados en 751, desde 2751 até 25751

São em total: 23 + 23 + 23 + 24 = 93

EUREKA! N 1, 1998

33
Sociedade Brasileira de Matemática

Você sabia que o maior número primo conhecido é


23021377-1, que tem 909529 dígitos e foi descoberto com a
ajuda de um
computador pessoal?

 Consulte na Internet a página

http://www.mersenne.org/prime.htm

2) Sejam D = (0,0), C = (k,0), B = (k,k) e A = (0,k), temos


3k 
  2k 
 e Q 
P  k , ,0 
4 
  3 
3 3
A equação de PD é y  x, e de AQ é y  k x . Se M= (x0,y0), temos
4 2
3 3 9 4k 3 k
x k  x  x k  x   y  x  , que é a altura de
4 0 2 0 4 0 0 9 0 4 0 3
M em relação a BQ, donde a área do QMD é
DQ  k / 3  2k / 3 k / 3 k 2
 
2 2 9

3)

Um hexágono é a união de 6 triângulos equiláteros iguais. Cada um destes


triângulos, se tem lado n, decompõe-se em n2 triângulos pequenos.

Lado 1 1 triângulo pequeno


Lado 2 4 triângulos pequenos
Lado 3 9 triângulos pequenos
Lado n n2 triângulos pequenos

EUREKA! N 1, 1998

34
Sociedade Brasileira de Matemática

O hexágono de lado n contem 6n2 destes triângulos pequenos.

100 
Busca-se o maior n tais que 6n2  10000 n     40
 6

Usam-se 6 × 402 triângulos pequenos. Perdem-se 400 = (1000 – 6 × 402)


triângulos.
4)
1
0 1
4 5
6 3

6 4
2

No segundo caso, não é possível, pois devem aparecer nas arestas os


números 1,2,3,…,10 (5 pares e 5 ímpares).

Se quatro ou cinco vértices recebem números com a mesma paridade,


temos pelo menos 6 arestas pares, portanto a numeração não será elegante.
Nos outros casos, teríamos seis arestas ímpares.

5) Sejam x, y, z, u, v, w os lados consecutivos do hexágono.


Prolongamos os lados y, u e w e obtemos um triângulo
equilátero. A área é igual à área deste triângulo equilátero
menos as áreas de três triângulos equiláteros de lados x, z e
v.

3
Área do hexágono:   x  y  z  2  x 2  v 2  z 2 
4  

EUREKA! N 1, 1998

35
Sociedade Brasileira de Matemática

Vejamos quais são os possíveis valores de x, y, z, u, v, w.

Seja x = 1, temos w + x + v = y + x + z
w + x + v = v + u + z (pois o triângulo de fora é
equilátero)

Donde temos
w+v = y+z
w+x = u+z

E temos v + x = y – u. Não pode ser v – x = 5, porque os únicos dois


números que têm diferença 5 são 1 e 6.

 Se v – x = 4, temos v = 5, y = 6, u = 2.
De w + 6 = z + 7, resulta, além disso, w = 4, z = 3.

 Se v – x = 3, então v = 4. Pode ser y = 5, u = 2 ou y = 6, u = 3.


O primeiro caso é impossível, porque não quedam valores de w, z
tais que w + 5 = z + 7. O segundo também é impossível, pois não
restam valores de w, z tais que w + 4 = z +6

 Se v – x = 2, temos v = 3 e pode ser y = 6, u = 4 ou y = 4, u = 2


No primeiro caso, w + 4 = z + 7, donde w = 5, z = 2.

 Se v – x = 1, temos v = 2 e pode ser y = 4, u = 3 ou y = 5, u = 4


ou y = 6, u = 5.
O primeiro e terceiro casos são impossíveis.
No segundo caso, w + 3 = 6 + z, onde w = 6, z = 3.

3 3
Os possíveis valores da área são: 100  1  25  9   65 
4 4

3 3
 81  1  4  9   67
4 4

Os hexágonos são:
3
5
3
6
2 4
1
1
EUREKA! N 1, 1998

4 5 36
2 6
Sociedade Brasileira de Matemática

9a. OLIMPÍADA DE MATEMÁTICA DO CONE SUL


Salvador - BA, 13 a 21 de junho de 1998

A 9ª. Olimpíada de Matemática dos países do Cone Sul será


realizada em Salvador, BA, no período de 13 a 21 de junho de 1998. Esta
Olimpíada será realizada pela segunda vez no país (a primeira foi em 1993,
em Petrópolis, RJ). Dela participam alunos de até 15 anos dos seguintes
países: Argentina, Brasil, Bolívia, Chile, Paraguai, Peru e Uruguai. A
organização da Olimpíada está a cargo da Professora Luzinalva Amorim,
da Universidade Federal da Bahia.
A equipe brasileira será selecionada através de provas realizadas
em março e maio deste ano e será liderada pelos professores Paulo Cezar
Pinto Carvalho, do IMPA, e Florêncio Ferreira Guimarães, da UFES.
A competição consta de duas provas, realizadas em dois dias, cada
uma com três problemas, valendo 10 pontos cada. Veja abaixo as provas
da última Olimpíada de Matemática do Cone Sul, realizada em Assunção
(Paraguai), em 1997, e os resultados obtidos pela equipe brasileira.

EUREKA! N 1, 1998

37
Sociedade Brasileira de Matemática

Você sabi@ que a


Olimpíada Brasileira de Matemática
já tem página web??

Visite-nos no endereço eletrônico http://www.obm.org.br

8a. OLIMPÍADA DO CONE SUL


21 a 25 de Abril de 1997. Assunção, Paraguai.

Primeiro dia.
Tempo: três horas.

PROBLEMA 1

De cada número inteiro positivo n, n  99, subtraímos a soma dos


quadrados de seus algarismos. Para que valores de n esta diferença é a
maior possível?

PROBLEMA 2

Seja C uma circunferência de centro O, AB um diâmetro dela e R um ponto


qualquer em C distinto de A e de B. Seja P a interseção da perpendicular
traçada por O a AR. Sobre a reta OP se marca o ponto Q, de maneira que
QP é a metade de PO e Q não pertence ao segmento OP. Por Q traçamos a
paralela a AB que corta a reta AR em T.
Chamamos de H o ponto de interseção das retas AQ e OT.
Provar que H, R e B são colineares.

PROBLEMA 3

EUREKA! N 1, 1998

38
Sociedade Brasileira de Matemática

Demonstrar que existem infinitos ternos (a, b, c), com a, b, c números


naturais, que satisfazem a relação: 2a2 + 3b2 – 5c2 = 1997.

Segundo dia.
Tempo: três horas.

PROBLEMA 4

Considere um tabuleiro de n linhas e 4 colunas.


Na 1a. linha são escritos 4 zeros (um em cada casa). A seguir, cada linha é
obtida a partir da linha anterior realizando a seguinte operação: uma das
casas, a escolher, é mantida como na linha anterior; as outras três são
trocadas: se na linha anterior havia um 0, coloca-se 1; se havia 1, coloca-se
2; e se havia 2, coloca-se 0.
Construa o maior tabuleiro possível com todas as suas linhas distintas e
demonstre que é impossível construir um maior.

PROBLEMA 5

Seja n um número natural, n > 3.


Demonstrar que entre os múltiplos de 9 menores que 10 n há mais números
com a soma de seus dígitos igual a 9(n-2) que números com a soma de seus
dígitos igual a 9(n-1).

PROBLEMA 6

Considere un triângulo acutângulo ABC, e seja X um ponto do plano do


triângulo.

Sejam M, N e P as projeções ortogonais de X sobre as retas que contêm as


alturas do triângulo ABC. Determinar para que posições de X o triângulo
MNP é congruente a ABC.

Nota: a projeção ortogonal de um ponto X sobre uma reta l é a interseção de


l com a perpendicular a ela que passa por X.

EUREKA! N 1, 1998

39
Sociedade Brasileira de Matemática

RESULTADOS OBTIDOS PELA EQUIPE BRASILEIRA

BRA 1 Murali Srinivasan Vajapeyam OURO


BRA 2 Rui Lopes Viana Filho OURO
BRA 3 Christian Iveson BRONZE
BRA 4 Daniele Véras de Andrade BRONZE

NÚMEROS MÁGICOS E CONTAS DE DIVIDIR


Carlos Gustavo Tamm de Araújo Moreira
 Nível Iniciante.

Temas muito inocentes de aritmética básica, como contas de multiplicar,


podem gerar resultados bastante interessantes e surprendentes, como ao
multiplicar o número 142857 por 2, 3, 4, 5, 6 e 7:

142857 × 2 = 285714
142857 × 3 = 428571
142857 × 4 = 571428
142857 × 5 = 714285
142857 × 6 = 857142

Por que razão acontece essa repetição dos dígitos de 142857 ao multiplicá-
lo por 2, 3, 4, 5 e 6, sempre com a mesma ordem circular? Será mera
coincidência? Será possível obter outros exemplos desse tipo?
A resposta tem a ver com o resultado de 142857 × 7, que é 999999. Isso
quer dizer que o período da representação decimal de 1/7 é exatamente
142857. Vamos examinar com cuidado a conta de divisão de 1 por 7:

10 7
30 0,142857
20
60
40
50
1

EUREKA! N 1, 1998

40
Sociedade Brasileira de Matemática

repetindo o resto 1, o que quer dizer que todo o processo se repete e o


resultado da divisão é 1/7 = 0,142857142857142857…
Podemos reescrever o processo assim:

1 =0×7+1
10 =1×7+3
30 =4×7+2
20 =2×7+6
60 =8×7+4
40 =5×7+5
50 = 7 × 7 + 1. Daí temos:
10 – 7 × 1 = 3, e portanto 100-7 × 10 = 30, e como 30 – 7 × 4 = 2 temos:
100 – 7 (10 + 4) = 2, e analogamente obtemos:
1000 – 7 (100 + 40 + 2) = 6
10000 – 7 (1000 + 400 + 20 +8) = 4
100000 – 7 (10000 + 4000 + 200 + 80 + 5) = 5
1000000 – 7 (100000 + 40000 + 2000 + 800 + 50 + 7 ) = 1

( A última igualdade diz que 142857 × 7 = 999999)

Desta forma, os restos sucessivos que aparecem na divisão de 1 por 7, que


são 3, 2, 6, 4, 5, 1 são, respectivamente, os restos na divisão por 7 de 10,
100, 1000, 10000, 100000 e 1000000. Estes restos assumem todos os
valores possíveis entre 1 e 6 e isso equivale ao fato de o período de 1/7 ter
6 casas. Desta forma, temos:

2 × 0,142857142857142857… = 2/7 = 100/7–14 = 100 × 0, 14285714


2857142857… – 14 = 0,285714285714285714…, e, portanto, temos 2 ×
142857 = 285714

Da mesma maneira temos que 3/7 = 10/7 – 1 implica 3 × 142857 =


428571, e as outras igualdades seguem de modo análogo.

Notemos agora que sempre que o período da representação decimal de 1/n


tiver n –1 casas decimais (que é o máximo possível), o período (que será
igual a (10n-1 –1) / n ) terá as mesmas propiedades de 142857. O primeiro
valor de n maior que 7 para o qual isso acontece é 17, e o período de 1/17

EUREKA! N 1, 1998

41
Sociedade Brasileira de Matemática

é 0588235294117647. Multiplique esse número por 2, 3, 4, 5, 6, 7, 8, 9,


10, 11, 12, 13, 14, 15, 16 e 17 para conferir.

Observe que, para que isso aconteça, n deve ser um número primo, pois se
n = p × b, com b maior que 1 e p um número primo diferente de 2 e 5,
então p nunca aparecerá como resto na divisão de 1 por n, pois em geral
um fator primo comum de n e de um resto que aparece na divisão de 1 por
n só pode ser 2 ou 5 ( de fato, um resto que aparece na divisão de 1 por n
é resto da divisão de alguma potência de 10 por n ). Por outro lado, se os
únicos fatores primos de n são 2 e 5, então 1/n tem representação decimal
finita.

Conclusão: Se o período de 1/n tiver n–1 casas decimais, ele terá


propiedades análogas às de 142857: os dígitos de seus produtos por 1, 2, 3,
4, …, n–1 serão sempre os mesmos, na mesma ordem circular. Para que
isso aconteça, n deve ser primo e a menor potência de 10 que deixa resto 1
quando dividida por n deve ser 10n–1. Dizemos que, nesse caso, 10 é raiz
primitiva módulo n. Não se sabe se existem infinitos primos n com essa
propriedade. Isso seguiria de uma famosa conjectura de teoria dos
números, a conjectura de Artin (vide [V]).

Os números primos n menores que 100 tais que o período de 1/n


na base 10 tem n –1 casas são 7, 17, 19, 23, 29, 47, 59, 61 e 97.

Por outro lado, para todo número primo n existem números


naturais B entre 2 e n – 1 tais que o período de 1/n na base B tem
exatamente n – 1 casas (nesses casos B é raiz primitiva módulo n). Se um
número B tem essa propriedade, todas as bases da forma kn + B com k
natural também têm. Nesses casos, o período de 1/n na base B ( ou seja, o
número (Bn-1–1)/n ), quando multiplicado por 1, 2, 3, …, n – 1 terá
representações na base B que serão permutações uma da outra com a
mesma ordem circular.

Por exemplo, com n = 5 e B = 8, temos que a representação de 1/5 na base


8 é 0,146314631463… Na base 8 temos:
2 × (1463)8 =(3146)8 , 3 × (1463)8 = (4631)8 ,
4 × (1463)8 = (6314)8 , 5 × (1463)8 = (7777)8

EUREKA! N 1, 1998

42
Sociedade Brasileira de Matemática

Referências:
[L] Lima, Elon L., Meu Professor de Matemática e outras histórias, pp. 158-170 –
SBM, 1991.
[T] Tahan, Malba, O homen que calculava, Ed. Record.
[V] Voloch, José Felipe, Raizes Primitivas e a Conjectura de Artin, Revista
Matemática Universitária Nº9/10, dezembro de 1989, pp. 153-158.
COMO PERDER AMIGOS E ENGANAR PESSOAS
Nicolau C. Saldanha
 Nível Avançado.

Neste artigo apresentaremos quatro situações simples em que


probabilidades enganam. Em alguns casos a probabilidade de certos
eventos tem um valor diferente daquele que a maioria das pessoas parece
julgar razoável, pelo menos de início; em um exemplo mostraremos como
é facil chegar a conclusões absurdas. Para que o leitor possa pensar
sozinho, apresentaremos primeiro quatro "enunciados", em que lançamos
cada situação, e depois quatro "desenvolvimentos" em que voltamos a
discutir as quatro situações na mesma ordem. Qualquer um pode usar estes
exemplos para divertir-se às custas de seus amigos, mas em nenhum caso o
autor tem responsabilidade pela integridade física daqueles que usarem a
Matemática para o mal.

ENUNCIADOS

1. Em um programa de auditório, o convidado deve escolher uma dentre


três portas. Atrás de uma das portas há um carro e atrás de cada uma das
outras duas há um bode. O convidado ganhará o que estiver atrás da porta;
devemos supor neste problema que o convidado prefere ganhar o carro. O
procedimento para escolha da porta é o seguinte: o convidado escolhe
inicialmente, em caráter provisório, uma das três portas. O apresentador do
programa, que sabe o que há atrás de cada porta, abre neste momento uma
das outras duas portas, sempre revelando um dos dois bodes. O convidado
agora tem a opção de ficar com a primeira porta que ele escolheu ou trocar

EUREKA! N 1, 1998

43
Sociedade Brasileira de Matemática

pela outra porta fechada. Que estratégia deve o convidado adotar? Com
uma boa estratégia, que probabilidade tem o convidado de ganhar o carro?

2. Um móvel tem três gavetas iguais. Em uma gaveta há duas bolas


brancas, em outra há duas bolas pretas, e na terceira há uma bola branca e
outra preta. Abrimos uma gaveta ao acaso e tiramos uma bola ao acaso sem
olhar a segunda bola que está na gaveta. A bola que tiramos é branca. Qual
é a probabilidade de que a segunda bola que ficou sozinha na gaveta seja
também branca?

EUREKA! N 1, 1998

44
Sociedade Brasileira de Matemática

3. Dois amigos querem decidir quem pagará a conta do restaurante com


uma aposta. Cada um deles escolhe uma seqüência de três caras ou coroas,
e eles jogam uma moeda até que saia uma das duas seqüências: aquele que
tiver escolhido a primeira seqüência a sair ganhou a aposta. Por exemplo,
André (por ser o primeiro em ordem alfabética) é o primeiro a escolher e
fica com a seqüência ckc (em que c representa cara e k coroa) enquanto
Bernardo responde com cck. Eles jogam a moeda obtendo
kckkckkkkccck, e neste momento Bernardo declara-se o vencedor. Esta
aposta é justa? André leva vantagem ou desvantagem por ser o primeiro a
escolher? Quais são as probabilidades de vitória de cada um?

4. Aqui novamente devemos nos imaginar em um programa de auditório.


Eugênio foi sorteado e tem direito a um prêmio, mas ele deve escolher
entre dois envelopes lacrados aparentemente iguais. O apresentador
informa que cada envelope tem um cheque e que o valor de um cheque é o
dobro do outro, mas não diz nada sobre o valor dos cheques, nem indica
qual envelope contém o cheque de maior valor. Eugênio escolhe e abre um
envelope que contém um cheque de, digamos, R$ 100. Neste momento, o
apresentador sempre faz uma proposta ao convidado: ele pode trocar de
envelope mediante uma multa de 5% do valor do cheque que ele tem em
mãos, no caso, R$ 5. Assim, se Eugênio aceitar, ele pode ganhar R$ 45 (se
o cheque no segundo envelope for de R$ 50) ou R$ 195 (se o outro cheque
for de R$ 200). Suponhamos que Eugênio (que fez um curso de Introdução
à Probabilidade no período anterior) queira maximizar o valor esperado de
seu prêmio. Deve ele aceitar a troca? E se o valor do primeiro cheque
tivesse sido outro, de que forma deveria isto influenciar a decisão de
Eugênio? Se Eugênio trocar de envelope independentemente do valor do
cheque, não vale mais a pena para ele trocar de envelope antes de abrir,
evitando, assim, a multa?

DESENVOLVIMENTOS

1. A resposta correta é que, trocando de porta, a probabilidade de ganhar o


carro é 2/3, enquanto não trocando a probabilidade é apenas 1/3. Uma
forma simples de ver isto é a seguinte: trocando de porta, o convidado
ganha, desde que a primeira porta que ele escolher esconda um dos dois
bodes, como se pode facilmente perceber. A melhor estratégia para o

EUREKA! N 1, 1998

45
Sociedade Brasileira de Matemática

convidado é, portanto, trocar sempre, e assim sua probabilidade de ganhar


fica sendo 2/3.

O erro comum aqui é achar que, após a eliminação de uma porta (que foi
aberta pelo apresentador, revelando um bode), há uma simetria entre as
duas outras portas e a probabilidade de cada uma esconder o carro é 1/2.
Não existe, entretanto, tal simetria, pois a porta escolhida pelo convidado
não poderia, pelas regras, ser trocada pelo apresentador, enquanto a outra
poderia ter sido aberta, mas não foi.

Este processo de fato era seguido em um programa nos Estados Unidos.


Uma longa e áspera discussão ocorreu na imprensa quanto a qual era o
valor correto da probabilidade, e pessoas que deveriam ser capazes de
resolver um problema trivial como este passaram pela vergonha de publicar
soluções erradas. Julgamos melhor esquecer os detalhes deste episódio
deprimente.

2. A resposta correta é 2/3 (e não 1/2). As seis bolas seriam de início


igualmente prováveis, mas sabemos que a primeira bola escolhida foi
branca: assim, as três bolas brancas têm igual probabilidade. Estamos
interessados em saber a cor da companheira de gaveta de cada bola branca:
em dois casos é branca, em um caso é preta. Assim, a probabilidade de que
a segunda bola seja branca é 2/3, como já afirmamos.

Um raciocínio comum, mas errado, é dizer: as gavetas são igualmente


prováveis, mas obviamente não escolhemos a gaveta que contém duas
bolas pretas. Portanto, teríamos probabilidade 1/2 de termos escolhido a
gaveta com duas bolas brancas e 1/2 de termos escolhido a gaveta com
uma bola de cada cor; no primeiro caso, a segunda bola é branca e, no
segundo caso, a bola é preta. Assim, a resposta seria 1/2.

O que há de errado neste raciocínio? O erro está em dizer que as duas


gavetas possíveis são igualmente prováveis. Inicialmente a probabilidade
de cada gaveta é de fato a mesma (inclusive para a gaveta com duas bolas
pretas), mas, ao tirarmos uma bola e constatarmos que ela é branca, isto
deixa de ser verdade. Isto é bem óbvio para a gaveta com duas bolas pretas:
passou a ser impossível termos escolhido esta gaveta. Entre as duas outras
gavetas, entretanto, há uma diferença que está sendo ignorada no raciocínio
do parágrafo anterior. Se pré-escolhermos a gaveta com duas bolas brancas,

EUREKA! N 1, 1998

46
Sociedade Brasileira de Matemática

temos certeza de passar no teste: uma bola escolhida ao acaso nesta gaveta
será sempre branca. Por outro lado, se pré-escolhermos a gaveta com uma
bola de cada cor, ainda temos probabilidade 1/2 de sacarmos uma bola
preta, o que estaria em contradição com o enunciado. Assim, a
probabilidade de termos escolhido cada uma destas duas gavetas é 2/3 e
1/3, respectivamente. Podemos, a partir deste ponto facilmente deduzir a
resposta correta de 2/3.

É fato empírico desencorajador que muitas pessoas teimam em dizer que a


probabilidade é 1/2 mesmo após esta explicação. O seguinte exemplo serve
como exercício para aqueles que entenderam a explicação e é uma espécie
de redução ao absurdo do raciocínio "rival". Temos novamente três
gavetas, uma com vinte bolas brancas, uma com vinte bolas pretas e a
terceira com dez bolas de cada cor. Abrimos uma gaveta e, sem olhar,
retiramos ao acaso dez bolas: elas são todas brancas. Qual a probabilidade
de que as dez bolas restantes sejam também brancas?

3. No nosso exemplo, Bernardo tinha probabilidade 2/3 de ganhar. Em


geral, o segundo a jogar leva uma vantagem considerável e, se escolher
bem sua resposta, pode garantir uma probabilidade de vitória de pelo
menos 2/3, mas às vezes até 7/8, dependendo da primeira jogada. A Tabela
1 dá a probabilidade de vitória de Bernardo para cada par de jogadas (a
coluna é a jogada de André e a linha a de Bernardo).

ccc cck ckc ckk kcc kck kkc kkk

ccc _ 1/2 2/5 2/5 1/8 5/12 3/10 1/2


ccc 1/2 _ 2/3 2/3 1/4 5/8 1/2 7/10
ckc 3/5 1/3 _ 1/2 1/2 1/2 3/8 7/12
ckk 3/5 1/3 1/2 _ 1/2 1/2 3/4 7/8
kcc 7/8 3/4 1/2 1/2 _ 1/2 1/3 3/5
kck 7/12 3/8 1/2 1/2 1/2 _ 1/3 3/5
kkc 7/10 1/2 5/8 1/4 2/3 2/3 _ 1/2
kkk 1/2 3/10 5/12 1/8 2/5 2/5 1/2 _

Tabela 1

Não reconstruiremos aqui toda a tabela: apresentaremos apenas como


exemplo a situação descrita no enunciado. O leitor que estiver interessado
em aprender mais sobre este problema pode consultar nosso Precisa-se de
alguém para ganhar muito dinheiro, a ser publicado na Revista do

EUREKA! N 1, 1998

47
Sociedade Brasileira de Matemática

Professor de Matemática do Chile, mas já disponível na home page do


autor: http://www.mat.puc-rio.br/~nicolau/. O Diagrama 2 descreve bem a
situação. Os seis vértices indicam as seis situações possíveis durante o
processo de jogar a moeda. O ponto indica que nenhum jogador tem como
esperar fazer uso das jogadas já feitas, ou seja, ou nenhum lance ainda foi
feito, ou foi lançado apenas um k, ou os dois últimos lances foram kk;
como o jogo sempre começa nesta situação, chamaremos este vértice de
inicial. O c indica que o último lance foi um c mas o anterior ou não existiu
ou foi um k. Os vértices cc e ck indicam que estes foram os dois últimos
lances. Finalmente, os vértices cck e ckc indicam que o jogo terminou;
chamaremos estes vértices de finais.

cc cck
 c
ck ckc

Diagrama 2

As duas setas partindo de cada vértice (exceto os finais) indicam como a


situação se modifica a cada lance de moeda: elas correspondem às
possibilidades de tirar c ou k em um dado momento. Queremos agora
calcular a probabilidade de vitória de Bernardo, dado que o jogo chegou a
uma certa situação. Temos, assim, quatro probabilidades a serem
calculadas:
p., pc, pcc e pck; consideramos naturalmente pcck = 1 e pckc = 0. Como a partir
de cada vértice não final as probabilidades associadas às duas setas são
iguais, temos as seguintes equações:

p. = 1/2 (p. + pc)


pc = 1/2 (pcc + pck)
pcc= 1/2 (pcc + 1)
pck= 1/2 p. .

Resolvendo o sistema, temos p. = 2/3, conforme afirmamos.

O erro mais natural aqui é achar que todas as seqüências são igualmente
boas: isto não é verdade, pois os dois últimos lances em geral serviram,

EUREKA! N 1, 1998

48
Sociedade Brasileira de Matemática

sem sucesso, para tentar finalizar as seqüências e servirão agora para tentar
iniciá-las. Mais surpreendente ainda é o fato de que o segundo jogador
sempre tem uma boa resposta: este jogo é um pouco como jogar par-ou-
ímpar ou pedra-papel-tesoura com um dos jogadores tendo o direito de
escolher sua jogada só depois de ver a jogada do adversário.

4. Antes de mais nada gostaríamos de lembrar que Eugênio deseja, por


hipótese, maximizar o valor esperado do prêmio. Este critério é razoável
em algumas situações e em outras não. Outro convidado poderia precisar
desesperadamente de uma certa quantia, talvez R$ 100, e gostaria,
portanto, de maximizar a probabilidade de ganhar pelo menos este valor
crítico. Ainda outro convidado pode ser tão curioso que deseja saber quanto
há em cada envelope mais do maximizar seu prêmio. O leitor, se fosse o
convidado, talvez julgasse interessante considerar ainda outros aspectos.
Podemos imaginar inúmeros critérios diferentes e em princípio cada
critério gera um novo problema. Nós nos propomos aqui a estudar o
problema na forma em que foi proposto e não a discutir se Eugênio, com
sua opção pelo valor esperado, é um homem verdadeiramente sábio.

Neste problema, ao contrário dos outros, apresentaremos inicialmente um


raciocínio falho e vamos segui-lo até chegarmos a um absurdo deixando a
análise dos erros deste raciocínio para o final. Para tornar a discussão toda
mais viva, acompanharemos o pensamento de Eugênio.

Ao receber a proposta de troca, Eugênio pensa: Se ficar com este cheque,


meu prêmio será de R$ 100. Se trocar de cheque, tenho probabilidade 1/2
de ganhar R$ 45 e probabilidade 1/2 de ganhar R$ 195: o valor esperado é
de (1/2)  45 + (1/2)  195 = 120 reais. Como 120 é maior que 100, a troca é
vantajosa. Eugênio troca de cheque e fica felicíssimo ao ver que o outro
cheque é de R$ 200: ele ganhou R$ 195!

Ao voltar para seu lugar no auditório, Eugênio continua pensando: Na


verdade vale a pena trocar qualquer que seja o valor do primeiro cheque.
Se chamarmos este valor de x, temos por um lado a opção de ficar com x e
por outro lado a opção de arriscar, com probabilidade 1/2 de ganhar
0.45x e probabilidade 1/2 de ganhar 1.95x. No primeiro caso, o valor
esperado é x e, no segundo caso, o valor esperado é 1.2x. Assim, como x >

EUREKA! N 1, 1998

49
Sociedade Brasileira de Matemática

0, vale sempre a pena trocar. Eugênio fica feliz com sua conclusão e pensa
como seu curso de Probabilidade foi útil.

Mas um pouco mais tarde Eugênio começa a ter dúvidas quanto a suas
conclusões: Se vale a pena trocar de envelope sempre, então não é
necessário abrir o envelope e ler o valor do cheque para tomar a decisão
de trocar. Neste caso, eu poderia ter trocado de envelope um minuto antes
e ter evitado a multa. Eugênio fica irritado, pensando que poderia ter
ganhado 5 reais a mais se apenas tivesse pensado mais rápido. Mas ele
continua pensando: Ei, espere, há algo errado! Um minuto antes os dois
envelopes estavam lacrados e pareciam iguais para mim: trocar
significaria apenas escolher o outro. Mas, então, cada vez que eu penso
em um envelope tenho que trocar e nunca posso escolher nada!
Assim, ao invés de aproveitar seu prêmio, Eugênio passa a noite
angustiado com seu paradoxo. Na manhã seguinte, Eugênio procura seus
colegas do curso de Probabilidade com a pergunta: o que exatamente há de
errado com este raciocínio?

O erro de Eugênio está logo no início, quando aceita, sem aliás sequer
questionar, que a probabilidade do segundo cheque ser maior é 1/2. O leitor
deve estar muito surpreso: é quase como se de repente disséssemos que
cara e coroa têm probabilidades diferentes. Por isso daremos uma
explicação relativamente longa para tentar convencer.

Começaremos fazendo algumas digressões considerando o que um outro


convidado, o João, que nunca estudou probabilidade, mas que tem bom
senso, faria em algumas situações extremas. João não acompanha todos os
sorteios, mas mesmo assim ele certamente tem alguma noção, por vaga que
seja, de qual a faixa dos prêmios. Assim, se o valor do primeiro cheque
fosse muito baixo, João certamente pensaria: Não é possível, ou pelo
menos não é provável, que o segundo cheque seja ainda menor. Assim,
quase certamente eu peguei o envelope com o cheque de menor valor (além
de ter tido o azar de vir em um dia em que os prêmios foram baixos) e
aposto que o outro cheque é maior: vou trocar. Por outro lado, se o valor
do primeiro cheque fosse muito alto, seu pensamento seria: Que sorte, hoje
os prêmios estão ótimos! E é muito improvável que o segundo cheque seja
ainda maior! Vou ficar com este cheque mesmo! Assim, João não atribui
probabilidades iguais às duas possibilidades (o segundo cheque ser maior

EUREKA! N 1, 1998

50
Sociedade Brasileira de Matemática

ou menor), e as probabilidades que ele atribui (inconscientemente) a estes


dois eventos dependem do valor do primeiro cheque.

Bem, este era o João e não o Eugênio: ao considerá-lo, desviamo-nos


temporariamente do problema original e do contexto que nos impusemos
no primeiro parágrafo deste desenvolvimento, pois João nem sabe o que é
o valor esperado e seus critérios não são os de Eugênio. João atribuiu
subjetivamente probabilidades diferentes aos dois eventos; Eugênio (que
aliás não se defrontou com situações extremas) atribuiu probabilidades
iguais. Será que em algum sentido é errado atribuir sempre probabilidades
iguais?

Sim, atribuir probabilidades sempre iguais é não apenas errado, mas


contraditório com a Teoria da Probabilidade que Eugênio tenta usar. Para
entender isto, vamos representar cada configuração inicial de envelopes por
um par ordenado (x1, x2) de números reais positivos: x1 é o valor do cheque
no primeiro envelope escolhido pelo convidado, e x2 é o valor do segundo
cheque. Assim, o espaço amostral   R2 é a união de duas semi-retas
abertas partindo da origem, como mostrado na Figura 3. A história que
contamos envolvendo Eugênio corresponde ao ponto (100, 200), também
indicado. Ao abrir o primeiro envelope, definimos o valor de x1 e ficamos
restritos à interseção de  com uma reta vertical, ou seja, aos dois pontos
(x1, x2 = 2x1) e (x1, x2 = x1/2).

Eugênio implicitamente aceita que a probabilidade condicional a um valor


qualquer fixo para x1 destes dois pontos é 1/2. Assim, ele deve aceitar que:

P({(t, 2t); t  [T, 2T)}) = P({(t, t/2); t  [T, 2T)})

para qualquer número positivo T, em que P(C), C  , denota a


probabilidade de que (x1, x2) esteja em C. Por outro lado, a simetria inicial
entre os envelopes diz que

P({(t, 2t); t  [T, 2T)}) = P({(2t, t); t  [T, 2T)}).


Sejam

An = {(t, 2t); t  [2n, 2n+1)},


Bn = {(2t, t); t  [2n ,2n+1)},

EUREKA! N 1, 1998

51
Sociedade Brasileira de Matemática

em que n é um inteiro qualquer; as identidades acima nos dão P(An) =


P(Bn-1) e P(An) = P(Bn), respectivamente. Por indução, P(An) = P(Bn) =
P(A0) para todo n. Observemos desde já que esta conclusão é no mínimo
estranha: ela diz que a probabilidade de o valor de menor cheque estar
entre 64 e 128 é igual à probabilidade de o menor cheque estar entre 2 64 e
265, ou entre 24199021 e 24199022; no próximo parágrafo veremos que esta
conclusão é não apenas estranha, mas realmente absurda, mesmo
ignorando o fato de que um prêmio de R$ 2 4199021 é uma impossibilidade
prática.

X2

300

200 (100,200)

100

(100,50)

100 200 300 X1

Figura 3

Observemos que os conjuntos An e Bn são dois a dois disjuntos e sua união


é . Se P(A0)>0, podemos tomar N  N tal que N P(A0)>1 e temos

  An 
P
 0 n  N   ,1
 
EUREKA! N 1, 1998

52
Sociedade Brasileira de Matemática

o que é absurdo. Por outro lado, se P(A0) = 0 temos

EUREKA! N 1, 1998

53
Sociedade Brasileira de Matemática

  (An  Bn )
P
  N  n  N  0
 
para todo N, o que também é um absurdo, pois, quando N cresce, este
conjunto também cresce, tendendo no limite para , donde teríamos
P()= 0, contradizendo P() = 1. Assim, em qualquer caso, temos um
absurdo.

Esta explicação é um pouco técnica, mas coincide perfeitamente com o


"bom senso" de João: não podemos ignorar o primeiro cheque. Se seu valor
for muito baixo, a probabilidade de que o segundo cheque seja maior deve
em geral ser muito maior do que 1/2, pois P(An) deve tender a zero quando
n tende a –. Por outro lado, se o seu valor for muito alto, a probabilidade
de que o segundo cheque seja ainda maior deve ser muito menor do que
1/2, pois P(An) também deve tender a zero quando n tende a +. E
Eugênio, afinal de contas, precisa fazer uma avaliação sutil, dependendo de
que valores são plausíveis como prêmio: até um certo valor-limite vale a
pena trocar, acima deste valor não.

EUREKA! N 1, 1998

54
Sociedade Brasileira de Matemática

Nicolau C. Saldanha
Departamento de Matemática, PUC-RIO
Gávea, Rio de Janeiro, RJ 22453-900, BRASIL
nicolau@mat.puc-rio.br, http://www.mat.puc-rio.br/~nicolau/
DOIS PROBLEMAS SOBRE GRAFOS
Paulo Cezar Pinto Carvalho
IMPA
 Nível Intermediario.

INTRODUÇÃO

A figura abaixo mostra um mapa rodoviário de um país fictício.


Neste artigo vamos examinar dois problemas relativos a este mapa:

1. Um funcionário, encarregado de verificar, periodicamente, o estado das


estradas, deseja planejar a sua rota de inspeção. Idealmente, esta rota
deveria se iniciar na capital e percorrer cada estrada exatamente uma vez,
voltando, então, ao ponto de partida. Existe tal rota?

2. Um representante de vendas de uma companhia deseja planejar uma


rota na qual ele visite cada cidade exatamente uma vez, voltando ao ponto
de partida. Existe tal rota?

Fig. 1 - Mapa rodoviário de um país fictício

Há vários pontos em comum entre os dois problemas. Por


exemplo: em ambos se deseja verificar a existência de um circuito (ou
ciclo) no grafo determinado pelo mapa (um grafo é um par (V, A), em que
V é o conjunto de vértices do grafo, e A é um conjunto de pares de vértices
– os arcos do grafo). No primeiro problema, este circuito deve incluir
exatamente uma vez cada arco do grafo. No segundo problema, o circuito
deve incluir exatamente uma vez cada vértice do grafo. Embora os dois
problemas sejam aparentemente semelhantes, há algumas diferenças

EUREKA! N 1, 1998

55
Sociedade Brasileira de Matemática

fundamentais entre eles. Convidamos os leitores a refletir um pouco sobre


cada um deles antes de prosseguir.

CIRCUITOS EULERIANOS

O primeiro problema – o do inspetor de estradas – foi estudado


pela primeira vez por Euler (1707-1783). Por esta razão, um circuito que
percorre cada arco de um grafo exatamente uma vez é chamado de circuito
euleriano e um grafo que possui um tal circuito é chamado de grafo
euleriano. A situação estudada por Euler ficou imortalizada como o
Problema das Pontes de Könisberg, ilustrado na figura abaixo, e que
possivelmente já é conhecido por muitos dos leitores. O objetivo é
percorrer exatamente uma vez todas as sete pontes da cidade (hoje
Kaliningrado), que conectam as duas ilhas entre si e com as margens do
rio, voltando ao ponto de partida.

Fig. 2 – O Problema das Pontes de Könisberg

Em linguagem de grafos, trata-se de encontrar um circuito euleriano


no grafo da figura acima, no qual os vértices representam as ilhas e as
margens e os arcos são as pontes1. Euler mostrou a não-existência de tal
circuito através de um argumento extremamente simples. Consideremos,
por exemplo, a ilha da direita. Um circuito qualquer deve chegar à ilha e
sair dela o mesmo número de vezes. Logo, para que exista um circuito
euleriano, deve haver um número par de pontes com extremidade nesta
ilha. Como existem três pontes nessas condições, concluímos que não é
possível encontrar um circuito euleriano. De modo mais geral, temos o
seguinte:

Teorema: Existe um circuito euleriano em um grafo se e somente se o


grafo é conexo (isto é, existe um caminho ligando qualquer par de
1
A rigor, neste caso temos um multi-grafo, já que certos pares de vértices são
ligados por mais de um arco.

EUREKA! N 1, 1998

56
Sociedade Brasileira de Matemática

vértices) e cada vértice tem grau par (ou seja, o número de arcos que nele
incidem é par).
O argumento acima mostra a necessidade de se ter grau em cada
vértice para existir um circuito euleriano. É também óbvio que o grafo
precisa ser conexo. A prova de que essas duas condições implicam na
existência de um circuito euleriano pode ser feita por indução finita no
número de arcos do grafo e é deixada como um exercício para o leitor.

[Sugestão: suponha a propriedade verdadeira para grafos com menos de n


arcos e considere um grafo com n arcos, satisfazendo às duas condições.
Começando em um vértice qualquer, percorra arcos do grafo, até voltar a
um vértice já visitado (o caminho gerado possui, então, um ciclo).
Retirando do grafo os arcos desse ciclo, obtém-se um ou mais grafos
satisfazendo as duas condições e com menor número de arcos (portanto,
com circuitos eulerianos, de acordo com a hipótese de indução). Basta
explicar como “costurar” esses circuitos eulerianos ao ciclo descrito
acima].

Podemos aplicar este teorema ao nosso problema de inspeção de


estradas. Da mesma forma como no Problema das Pontes de Könisberg,
não existe qualquer circuito euleriano no grafo determinado pelo mapa
rodoviário, já que o vértice correspondente à capital tem grau 3. Assim, se
o nosso inspetor de estradas recebesse de seu chefe a incumbência de
elaborar um trajeto nas condições do problema 1, ele poderia facilmente
convencê-lo da impossibilidade de fazê-lo. Como veremos a seguir, a
situação do seu colega representante de vendas é bem pior...

CIRCUITOS HAMILTONIANOS

Um circuito passando exatamente uma vez por cada vértice de um


grafo é chamado de circuito hamiltoniano, em homenagem ao matemático
irlandês William Rowan Hamilton (1805-1865), que estudou este problema
no grafo determinado pelas arestas de um dodecaedro regular (existe ou
não um circuito hamiltoniano neste caso?). Um grafo que possui um
circuito hamiltoniano é chamado de grafo hamiltoniano.

A situação do problema de verificar se um grafo é hamiltoniano é


bem diferente da do problema anterior. Apesar de terem sido estudados por

EUREKA! N 1, 1998

57
Sociedade Brasileira de Matemática

vários séculos, não há uma boa caracterização dos grafos hamiltonianos.


Há diversas famílias de grafos para os quais existe um circuito
hamiltoniano (um exemplo trivial é um grafo completo, em que cada
vértice é ligado a todos os outros); também é possível estabelecer certas
condições que implicam na não-existência de um circuito. Mas uma
caracterização geral não foi encontrada e, à luz de certos avanços em teoria
da computação das últimas décadas, parece improvável que ela seja
encontrada algum dia.

O problema de decidir se um grafo é hamiltoniano está na


companhia de diversos problemas ilustres, com as seguintes características
em comum:

 O problema possui uma assimetria fundamental: é muito fácil


convencer alguém da existência de um circuito hamiltoniano em um
grafo: basta exibir tal caminho. No entanto, é difícil, em geral,
convencer alguém da não-existência de um tal circuito. Por exemplo, o
grafo da figura abaixo (o leitor é capaz de reconhecê-lo?) tem um
circuito hamiltoniano, de cuja existência o leitor fica imediatamente
convencido pela figura. Já o grafo dado no início do artigo não tem
circuito hamiltoniano, mas não existe um argumento simples e geral
para demonstrar esse fato (assim, nosso amigo representante de vendas
certamente terá mais trabalho para convencer seu chefe da
impossibilidade de elaborar uma rota nas condições do problema 2 do
que seu colega inspetor de estradas).

Fig. 3 – Um grafo hamiltoniano

 Não se conhece um algoritmo eficiente para verificar se um grafo é


hamiltoniano (por eficiente, entendemos aqui um algoritmo em que o
número de passos seja limitado por um polinômio no número de

EUREKA! N 1, 1998

58
Sociedade Brasileira de Matemática

vértices do grafo). Além disso, parece improvável que um tal algoritmo


possa algum dia ser encontrado, porque sua existência implicaria na
existência de algoritmos eficientes para um grande número de outros
problemas, para os quais também não se conhecem algoritmos
eficientes. Estes problemas (incluindo o de verificar a existência de
circuito hamiltoniano) formam uma classe de problemas chamados de
NP-completos. Um outro problema famoso da classe é o de
determinar o número mínimo de cores que podem ser usadas para
colorir os vértices de um grafo de modo que vértices de mesma cor não
sejam ligados por um arco.

O leitor poderá estar pensando assim: mas será que esta história de
algoritmos eficientes tem relevância, numa era de computadores cada vez
mais velozes? Afinal de contas, existe um algoritmo extremamente simples
para verificar se um grafo possui um circuito hamiltoniano. Se existir um
tal circuito, ele corresponderá a uma permutação (circular) dos vértices
com a propriedade de que vértices consecutivos sejam ligados por um arco
do grafo. Ora, para verificar a existência de circuito hamiltoniano basta
gerar todas as permutações circulares dos vértices e testar se uma delas
corresponde a um percurso no grafo.
É claro que este algoritmo funciona para grafos de tamanho
moderado (ele poderia ser o recurso usado pelo nosso vendedor: como são
apenas 9 cidades, ele teria que testar “apenas” 8! = 40.320 caminhos, o que
seria feito com rapidez em um computador). Mas o que ocorre com grafos
maiores? Vejamos, por exemplo, uma situação em que o número de
cidades cresce para 50 (o que representaria um tamanho ainda bastante
razoável para uma situação real). Neste caso, o computador deveria
examinar 49! circuitos potenciais. Tentemos estimar a magnitude deste
número. A forma mais simples é usar a fórmula de Stirling, que fornece a
n
 n
estimativa n!  2n   . Mas, neste caso, podemos usar estimativas
e
mais elementares. Por exemplo, podemos usar apenas potências de 2.
Temos:

49! = 1 × 2 × 3 × 4 × 5 × 6 × 7 × 8 × ... × 15 × 16 × ... × 31 × 32 × … ×


49 > 1 × 2 × 2 × 4 × 4 × 4 × 4 × 8 × ... × 8 × 16 × ... × 16 × 32 ×
… × 32 = 22 x 44 x 88 x 1616 x 3218 = 22+8+64+90 = 2164.

Mas 210 = 1024 >103. Logo 49! > 16. 1048.

EUREKA! N 1, 1998

59
Sociedade Brasileira de Matemática

Ora, um computador moderno pode realizar cerca de 200 milhões de


operações por segundo. Se em cada operação ele conseguir testar um
circuito, ele ainda assim precisará de mais de 16. 10 48 / 2. 106 = 8 × 1042
segundos, o que corresponde a aproximadamente a 2 × 10 35 anos. Assim,
trata-se claramente de uma missão impossível para o algoritmo de força
bruta baseado na análise de cada permutação de vértices.

PROBLEMAS DIFÍCEIS QUE TAMBÉM SÃO ÚTEIS

O resultado da discussão acima pode parecer bastante desanimador:


não parece haver bons métodos para verificar a existência de um circuito
hamiltoniano e algoritmos de força bruta só funcionam para problemas
com pequeno número de vértices (é bom que se diga que existe um meio
termo: há estratégias que permitem resolver o problema acima para valores
razoáveis de n, reduzindo substancialmente o número de possibilidades a
serem examinadas; mesmo estes algoritmos, no entanto, tornam-se
impráticos a partir de um certo ponto). O mesmo ocorre com todos os
chamados problemas NP-completos.

No entanto, ao invés de ficarmos deprimidos com esta característica


desses problemas, podemos explorá-la para uma importante finalidade em
criptografia, que é a parte da Matemática que estuda métodos para criar e
decifrar códigos. Para tal, é também muito importante a assimetria
apontada acima (e que ocorre em todos os problemas NP-completos): é
difícil encontrar um circuito hamiltoniano (ou mostrar que não existe um),
mas é fácil testar se uma seqüência de vértices forma um circuito
hamiltoniano.

Suponhamos que você seja cliente de um banco. Para ter acesso aos
serviços, você usa o número de sua conta (que é público) e uma senha, que
em princípio deve ser conhecida apenas por você. O procedimento mais
simples seria ter a sua senha armazenada no sistema do banco. Mas aí
você correria o risco de que ela fosse descoberta, por exemplo, por um
funcionário desonesto. Em lugar disto, o sistema do banco armazena uma
versão codificada da senha, que não precisa ficar em segredo. Esta
codificação deve ser feita de tal forma que seja simples verificar se sua
senha está correta (para que você seja autorizado a retirar dinheiro do caixa

EUREKA! N 1, 1998

60
Sociedade Brasileira de Matemática

eletrônico), mas seja praticamente impossível recuperar a senha a partir da


versão codificada.
Problemas NP-completos servem como uma luva para esta tarefa.
Se quiséssemos usar o problema do circuito hamiltoniano, poderíamos agir
mais ou menos da formadescrita a seguir. O cliente poderia escolher uma
permutação dos números de 1 a 50, conhecida apenas por ele. A partir
dessa informação, seria gerado um grafo, contendo necessariamente os
arcos correspondentes ao circuito (os demais poderiam, por exemplo, ser
gerados por um método aleatório, em que cada um dos possíveis arcos teria
uma certa probabilidade de sere escolhido). Este grafo seria armazenado no
sistema. A figura a seguir mostra uma representação de uma permutação
dos números de 1 a 50 e um grafo, gerado aleatoriamente, que possui um
ciclo hamiltoniano dado por esta permutação.

Fig. 4 – Um ciclo hamiltoniano e um grafo gerado a partir dele

Quando o cliente fosse utilizar sua conta, o sistema simplesmente


verificaria se a permutação apresentada corresponde a um caminho no
grafo. Como é improvável que um tal ciclo pudesse ser encontrado para
um grafo deste tamanho, dificilmente um impostor conseguiria se fazer
passar pelo cliente, ainda que conhecesse o grafo-problema. Na prática, são
utilizados outros problemas NP-completos para se fazer codificação de
senhas, mas a idéia é exatamente a mesma acima.

PALAVRAS FINAIS

Grafos são uma fonte inesgotável de problemas com enunciado


simples mas que escondem, muitas vezes, uma sofisticada estrutura
matemática. Neste artigo abordamos apenas alguns aspectos de dois desses
problemas. Certamente voltaremos a falar em grafos em outros artigos
desta revista. Para o leitor que deseja saber mais sobre o assunto,
recomendamos os livros a seguir:

 Jaime Luiz Szwarcfiter. Grafos e Algoritmos Computacionais.


Editora Campus.

EUREKA! N 1, 1998

61
Sociedade Brasileira de Matemática

 Oynstein Ore. Graphs and Their Uses. The Mathematical


Association of America.
PROBLEMAS PROPOSTOS
 Convidamos o leitor a enviar
soluções dos problemas propostos
e sugestões de novos
problemas para os próximos números.

1) Mostre que, dado um conjunto de n pessoas, existem duas que


possuem o mesmo número de amigos entre as pessoas do conjunto.

2) Em uma pista circular há postos de gasolina, e o total de gasolina


que há nos postos é exatamente o suficiente para um carro dar uma
volta. Prove que existe um posto de onde um carro com o tanque
inicialmente vazio pode partir e conseguir dar uma volta completa
na pista (parando para reabastecer nos postos).

3) Prove que existe n  N tal que os últimos 1000 dígitos de n1998 são
iguais a 1.

4) Escreva 1998 como soma de (um número arbitrário de ) parcelas


de modo que o produto das parcelas seja o maior possível.

5) Sejam a  0 e P1P2P3P4P5 uma poligonal aberta contida em um dos


semi-planos determinados pela reta P1 P5 . Prove que é possível
escolher pontos P6 e P7 no plano com P5 P6 = a de modo que é
possível ladrilhar o plano com infinitos ladrilhos congruentes ao
heptágono P1P2P3P4P5P6P7.

6) Mostre que toda seqüência com n2 + 1 elementos possui uma


subseqüência crescente com n + 1 elementos ou uma subseqüência
decrescente com n + 1 elementos.

7) Prove que 1 2 3  ...  1998 2

8) Considere um torneio de xadrez envolvendo brasileiros e


argentinos em que cada jogador joga contra todos os outros

EUREKA! N 1, 1998

62
Sociedade Brasileira de Matemática

exatamente uma vez. Ao final do torneio, cada jogador obteve


metade dos pontos que conquistou jogando contra brasileiros e
metade jogando contra argentinos. Prove que o número total de
jogadores do torneio é um quadrado perfeito (obs: cada vitória vale
1 ponto, empate 1/2 ponto e derrota 0 ponto).

9) Prove que todo número racional positivo pode ser escrito como
soma de um certo número de frações distintas de numerador 1.

Você sabia… que

cos
2
5

5  1
4
e 
2  1 17  34  2 17  2 17  3 17  34  2 17  2 34  2 17
cos 
17 16

mas não é possível escrever cos


2
7
e cos
2
9
usando radicais reais ?

EUREKA! N 1, 1998

63
Sociedade Brasileira de Matemática

AGENDA OLÍMPICA

IV OLIMPÍADA DE MAIO
09 de maio, 14 h

OLIMPÍADA DO CONE SUL


13 a 21 de junho de 1998
Salvador – BA.

OLIMPÍADA BRASILEIRA DE MATEMÁTICA


Primeira Fase – Sábado, 6 de junho
Segunda Fase – Sábado, 12 de setembro
Terceira Fase – Sábado, 24 de outubro (níveis 1,2 e 3)
Domingo, 25 de outubro (nível 3).

39 a. OLIMPÍADA INTERNACIONAL DE MATEMÁTICA


10 a 21 de julho
Taiwan.

OLIMPÍADA IBEROAMERICANA DE MATEMÁTICA


13 a 20 de setembro de 1998
República Dominicana.

EUREKA! N 1, 1998

64
Sociedade Brasileira de Matemática

COORDENADORES REGIONAIS

Alberto Hassen Raad (UFJF) Juiz de Fora-MG


Antônio C. Rodrigues Monteiro (UFPE) Recife-PE
Amarísio da Silva Araújo (UFV) Viçosa-MG
Angela Camargo (Esc. Tec. Hermann
Hering) Blumenau-SC
Antônio C. do Patrocínio (IMECC/UNICAMP) Campinas-SP
Benedito T. Vasconcelos Freire (UFRGDN) Natal-RN
Carlos A. Bandeira Braga (UFPB) João Pessoa-PB
Claudio Arconcher (Col. Leonardo da Vinci) Jundiaí-SP
Élio Mega (Col. ETAPA) São Paulo-SP
Florêncio F. Guimarães F. (UFES) Vitória-ES
Francisco Dutenhefner (UFMG ) BH-MG
Gisele de A. Prateado G. (UFGO) Goiânia-GO
João B. de Melo Neto (UFPI) Teresina-PI
José Carlos Pinto Leivas (URG) Rio Grande-RS
José Paulo Carneiro (USU) Rio de Janeiro-RJ
José Vieira Alves (UFPB) Campina Grande-PB
Leonardo Matteo D'orio (Parque de Material
Aeronáutico de Belém) Belém-PA
Luzinalva M. de Amorim (UFBA) L. de Freitas-BA
Marco Polo (Colégio Singular) Santo André-SP
Marcondes Cavalcante França (UF Ceará) Fortaleza-CE
Mario Jorge Dias Carneiro (UFMG) BH-MG
Ma-To-Fú (UEM) Maringá-PR
Pablo Rodrigo Ganassim (L. Albert Einstein) Rio das Pedras-SP
Paulo H. Cruz Neiva de L. Jr. (Esc. Tec.Everardo
Passos) Piracicaba-SP
Reinaldo Gen Ichiro Arakaki (INPE) S.J.Campos-SP
Ricardo Amorim (Centro Educ. Logos) Nova Iguaçu-RJ
Sergio Claudio Ramos (IM-UFRGS) Porto Alegre-RS

EUREKA! N 1, 1998

65
Sociedade Brasileira de Matemática

EUREKA! N 1, 1998

66
CONTEÚDO

AOS LEITORES 2

OLIMPÍADA BRASILEIRA DE MATEMÁTICA 4


Problemas de treinamento para a Segunda Fase

XIX OLIMPÍADA BRASILEIRA DE MATEMÁTICA 10


Problemas Júnior Segunda Fase e Soluções

IV OLIMPÍADA DE MAIO 16
Resultados

IV OLIMPÍADA DE MAIO 17
Prova

9a. OLIMPÍADA DE MATEMÁTICA DO CONE SUL 21

9a. OLIMPÍADA DE MATEMÁTICA DO CONE SUL 22


Problemas e soluções

39a. OLIMPÍADA INTERNACIONAL DE MATEMÁTICA 30


Resultados e problemas

ARTIGOS
PARIDADE 32
Eduardo Wagner

OS PROBLEMAS DO VISITANTE MATEMÁTICO 39

DIVISIBILIDADE, CONGRUÊNCIAS E ARITMÉTICA MÓDULO n 41


Carlos Gustavo Moreira

SOLUÇÕES DE PROBLEMAS PROPOSTOS EUREKA N1 53

PROBLEMAS PROPOSTOS 59

AGENDA OLÍMPICA 61

COORDENADORES REGIONAIS 62
Sociedade Brasileira de Matemática

AOS LEITORES

Iniciamos este segundo número da revista EUREKA! transmitindo


aos leitores nossa satisfação pela acolhida do primeiro número por alunos e
professores. A comunidade estudantil e os professores das escolas passam a
ter, de forma que esperamos permanente, uma publicação específica que,
além de fornecer material para tornar as aulas mais ricas e interessantes, é
um veículo de contato entre todos para expor experiências, dirimir dúvidas
e nos aproximarmos cada vez mais.

Já estamos recebendo correspondência de muitos alunos e alguns


professores com respeito às soluções dos problemas propostos. Isto muito
nos alegra e temos a certeza de que nos próximos números essa
correspondência só tenderá a crescer. Entretanto, gostaríamos de pedir aos
professores que nos enviem também colaborações para os números
seguintes da revista: problemas interessantes com soluções, pequenos
artigos, experiências em sala de aula, olimpíadas ou torneios regionais,
enfim, material que seja adequado aos alunos da 5ª série do ensino
fundamental à última série do ensino médio. Estas colaborações serão
fundamentais para que nossa revista permaneça viva e seja sobretudo útil a
toda a comunidade.

A Olimpíada Brasileira de Matemática de 1998

Realizamos a primeira fase da Olimpíada Brasileira de Matemática


em mais de mil colégios do nosso país. Em nosso projeto pretendíamos
atingir, nesta primeira etapa dessa nova atividade, cerca de 20 000 alunos
mas, para nossa surpresa, esse número já superou o dobro do pretendido.
Através dos relatórios enviados pelas escolas aos Coordenadores
Regionais, estabelecemos as notas de corte para a promoção dos alunos à
segunda fase que se realizará em setembro. A terceira fase, já mais
centralizada, será feita em outubro e esperamos que no final de novembro
possamos divulgar a lista dos alunos premiados.

Como em toda competição, é natural que o número de premiados


seja relativamente pequeno em relação ao número inicial de participantes.
Porém, aqui não há perdedores. Todos são de alguma forma ganhadores: de

EUREKA! N 2, 1998

2
Sociedade Brasileira de Matemática

uma experiência nova, de um estímulo para estudar mais e crescer, ou da


possibilidade de ver que objetivos que pareciam longínquos realmente
podem ser atingidos.

Devemos ainda relatar que alguns colégios não participaram da


Olimpíada Brasileira de Matemática com receio de que, sem uma
preparação adequada, seus alunos não tivessem um resultado satisfatório.
Especialmente para estes colégios enviamos nossa mensagem final:

A Olimpíada Brasileira de Matemática não é uma competição


entre colégios. A OBM tem como objetivo principal estimular o estudo de
Matemática entre os jovens, desenvolver professores e propiciar uma
melhoria do ensino e do aprendizado desta matéria nas escolas
brasileiras.

Comitê Editorial.

EUREKA! N 2, 1998

3
Sociedade Brasileira de Matemática

OLIMPÍADA BRASILEIRA DE MATEMÁTICA


Problemas de treinamento para a Segunda Fase

Primeiro Nível

1) Determine o menor inteiro cuja representação decimal consiste


somente de 1's e que é divisível pelo número 333…333 formado
por 100 algarismos iguais a 3. (Problema proposto por Antonio
Luiz Santos.)

2) Numa gaveta há 6 meias pretas e 6 meias brancas. Qual é o número


mínimo de meias a se retirar (no escuro) para garantir que:

a) As meias retiradas contenham um par da mesma cor?


b) As meias retiradas contenham um par de cor branca?

3) Quando se escrevem os números 1, 2, 3, 4, 5, 6, 7, 8, 9, 10, 11, 12,


…1998, qual é o dígito que ocupa o lugar 1998?

Segundo Nível

1) Determine com quantos zeros consecutivos termina a


representação decimal do número 1  2  3  … 1998.

2) Suponha que desejamos saber de qual janela de um prédio de 36


andares é seguro jogarmos ovos para baixo, de modo que os ovos
não se quebrem ao atingirem o chão. Para tal, admitimos que:

 Um ovo que sobrevive a uma queda pode ser usado novamente.


 Um ovo quebrado deve ser descartado.
 O efeito da queda é o mesmo para todos os ovos.
 Se um ovo se quebra quando jogado de uma certa janela então ele
quebrará se jogado de uma altura superior.
 Se um ovo sobrevive a uma queda então ele sobreviverá a uma
queda menor.

EUREKA! N 2, 1998

4
Sociedade Brasileira de Matemática

 Não se sabe se da janela do primeiro andar os ovos quebram, e


também não se sabe se da janela do último andar os ovos quebram.

Se temos apenas 1 ovo e queremos ter certeza de obter um resultado


correto, o experimento deve ser guiado apenas por um único caminho:
jogue o ovo pela janela do primeiro andar; se não se quebrar, jogue o ovo
pela janela do segundo andar. Continue até que o ovo se quebre. Na pior
das hipóteses, este método necessitará de 36 lançamentos para ser
concluído. Suponha que 2 ovos estão disponíveis. Qual é o menor número
de lançamentos de ovos necessários para garantir todos os casos?

3) Considere cinco pontos quaisquer P1, P2, …, P5 no interior de um


quadrado de lado 1. Mostre que pelo menos uma das distâncias dij
entre Pi e Pj é menor que 2 / 2 .

Terceiro Nível

1) Determine quantos números naturais menores que 1998 têm um


número ímpar de divisores positivos.

2) Mostre que, dados 5 pontos do plano em posição geral (isto é três


pontos quaisquer nunca estão em linha reta), há 4 que formam um
quadrilátero convexo.

3) Dois discos A e B são divididos em 2n setores iguais. No disco A,


n setores são pintados de azul e n de vermelho. No disco B, os
setores são pintados de azul ou vermelho de forma completamente
arbitrária. Mostre que A e B podem ser superpostos de modo que
pelo menos n setores tenham cores coincidentes.

EUREKA! N 2, 1998

5
Sociedade Brasileira de Matemática

Soluções do Primeiro Nível

1)
333...333  
É claro que d = 100 três =3  111 ...111 
 100 uns  = 3n . Portanto, o número
 
111 ...111
procurado N = k uns deve ser divisível por n e por 3 (n não é
divisível por 3 porque a soma dos seus algarismos é igual a 100 que não é
divisível por 3). Se k é um número da forma k = 100q + r onde 0  r < 100
111 ...111 000...00 111 ...11  M  R
então obviamente N = 100q uns r zeros r uns . Como
M é divisível por n então, N é divisível por n se, e somente se, R = 0 ou
seja, se r = 0 e conseqüentemente se, e somente se, k for divisível por 100.
Se k = 100q então a soma dos algarismos de N é igual a 100q e esta soma
será divisível por 3 (e consequentemente também o número N)se, e
somente se, q for divisivel por 3. Portanto, o menor número
N 111 ...111 divisível por d consiste em 300 uns.
k uns

2)

a) 3 meias (necessariamente teremos 2 meias brancas ou 2 meias


pretas; se tirarmos apenas duas pode ser que uma seja branca e a
outra preta).

b) 8 meias (se tirassemos apenas 7 meias poderiam ser 6 pretas e


apenas uma branca).

3)

Quando se escrevem os números do 1 ao 99, usam-se 9 + 2 (99 – 9) = 189


dígitos. Ficam por preencher 1809 (1998 – 189) lugares. Para cada uma das
centenas que seguem usam-se 300 dígitos. Como 1809 = 300  6 + 9, ao
terminar de escrever os 1998 dígitos se escrevem todos os números desde
o número 1 até completar 7 centenas (do número 1 até 699) e 9 dígitos
mais: 700, 701 e 702. Portanto o dígito que ocupa o lugar 1998 é o número
2.
Soluções do Segundo Nível

EUREKA! N 2, 1998

6
Sociedade Brasileira de Matemática

1)

A resposta é 496. Se a decomposição de 1  2  … 1998 em fatores


primos é 2a  3b  5c…, temos necessariamente c < a, pois para todo r
natural há mais múltiplos de 2 r que de 5r entre 1 e 1998. Assim, o número
de zeros do final de 1  2  … 1998 é igual a c. Para determinar c,
observamos que entre 1 e 1998 há 399 múltiplos de 5 (pois 399  5 < 1998
< 400  5), 79 múltiplos de 25, 15 múltiplos de 125, 3 múltiplos de 625
mas nenhum múltiplo de 3125, e portanto temos c = 399 + 79 + 15 + 3 =
496. (De fato, ao contar os múltiplos de 5, que são 399, já contamos os
múltiplos de 25, mas estes devem ser contados pelo menos em dobro para
calcular o exponente de 5, por isso somamos 79, mas é preciso contar os
múltiplos de 125 pelo menos 3 vezes e só foram contados 2 vezes, por isso
somamos 15. E assim por diante.)

2)

8 lançamentos. Jogamos o primeiro ovo do oitavo andar. Se quebrar, basta


testar os 7 primeiros com o segundo ovo. Se não quebrar, o jogamos do
15., depois do 21., depois do 26., depois do 30., depois do 33., depois
do 35. e finalmente do 36.. Se ele quebrar por exemplo quando jogado do
26. andar, basta testar o segundo ovo nos andares 22, 23, 24 e 25, para o
que gastamos 4 + 4 = 8 lançamentos. A escolha dos andares se devem a 8 +
7 = 15, 8 + 7 + 6 = 21, 8 + 7 + 6 + 5 = 26, 8 + 7 + 6 + 5 + 4 = 30, 8 + 7 + 6
+ 5 + 4 + 3 + 2 + 1 = 36. O resultado não pode ser melhorado, pois se o
primeiro ovo quebra no n-ésimo lançamento, devemos testar com o ovo
restante todos os andares entre os usados nos (n – 1)-ésimo e n-ésimo
lançamentos, no pior caso.

Tente generalizar este problema fazendo variar o número de ovos


disponíveis e o número de andares do prédio.

3)

EUREKA! N 2, 1998

7
Sociedade Brasileira de Matemática

Dividimos o quadrado em 4 quadrados de lado 1/2. Necessariamente dois


desses pontos, digamos Pi e Pj, estarão num mesmo quadradinho, e sua
distância dij será menor que a diagonal do quadradinho (que é a maior
2
distância possível entre dois de seus pontos), ou seja .
2

Soluções do Terceiro Nível

1)

  
Se n  p1 1 p 2 2 ... p k k é a fatoração em primos de n, os divisores positivos
  
de n são todos os números da forma p1 1 p 2 2 ... p k k com 0  1  1, 0  2
 2, …, 0  k  k, i  N, i. Assim , o número de divisores positivos
de n é (1 + 1) (1 + 2)…(1 + k ). Para que este número seja ímpar é
necessário e suficiente que todos os i sejam pares, ou seja, que n seja
quadrado perfeito. Como 442 = 1936 < 1998 < 2025 = 45 2, há 44 quadrados
perfeitos entre 1 e 1998, portanto há 44 naturais menores que 1998 com um
número ímpar de divisores positivos.

2)

Se o menor polígono convexo que contém os 5 pontos tiver mais de 3 lados


o problema é trivial. Caso contrário, dois dentre os 5 pontos (digamos D e
E), estão dentro do triângulo cujos vértices são os outros 3. Ao prolongar a
reta que une esses dois pontos cortamos dois dos lados do triângulo,
digamos AB e AC. Nesse caso, é fácil ver que o quadrilátero BDEC é
convexo.

3)

EUREKA! N 2, 1998

8
Sociedade Brasileira de Matemática

Sejam S1, S2, …, S2n os setores do disco B. Tentamos colocar o disco A


sobre o disco B nas 2n posições possíveis (com os setores coincidindo).
Para cada i com 1  i  2n , em exatamente n das posições do disco A o
setor Si terá cor coincidente com o setor do disco A que está sobre ele.
Assim, o número médio de setores com cores coincidentes nos dois discos
para as 2n posições do disco A é 2n  n/2n = n, e necessariamente há
posições do disco A para as quais há pelo menos n setores com cores
coincidentes.

2fisdufiows
uhf Você sabia… que há tantos números racionais
quanto números naturais, mas há estritamente mais
números reais que racionais (isto é, existe uma bijeção
f : N  Q mas não existe nenhuma bijeção g : Q  R) ?
E que é impossível decidir se existe algum conjunto
com estritamente mais elementos que os naturais mas
estritamente menos elementos que os reais ??

XIX OLIMPIADA BRASILEIRA DE MATEMÁTICA


Problemas Júnior Segunda Fase e Soluções

EUREKA! N 2, 1998

9
Sociedade Brasileira de Matemática

A Olimpíada Brasileira Júnior correspondia aproximadamente aos


atuais níveis 1 e 2 da OBM. Estamos publicando a prova da segunda fase
júnior do ano passado com soluções, a qual acreditamos ser bom material
de treinamento tanto para a segunda fase da OBM quanto para a terceira
nos níveis 1 e 2. No próximo número da EUREKA! publicaremos a
segunda fase da Olimpíada Brasileira sênior do ano passado.

PROBLEMA 1

No edifício mais alto de Terra Brasilis moram Eduardo e Augusto. O


número do andar do apartamento de Eduardo coincide com o número do
apartamento de Augusto. A soma dos números dos apartamentos dos dois é
2164. Calcule o número do apartamento de Eduardo sabendo que há 12
apartamentos por andar. (Por exemplo, no primeiro andar estão os
apartamentos de 1 a 12, no segundo, de 13 a 24, e assim por diante.)

PROBLEMA 2

A professora de Matemática propôs o seguinte problema para seus alunos:

"Marquem 6 pontos sobre uma circunferência. Eu quero que vocês pintem


o maior número de cordas determinadas por estes pontos, de modo que não
existam quatro dos pontos sobre a circunferência determinando um
quadrilátero com todos os lados e diagonais coloridos."

a) Edmilson encontrou uma solução correta colorindo 12 cordas. Exiba


uma maneira de como fazer isto.

b) Gustavo afirmou ter encontrado uma solução na qual pintara 13 cordas.


Mostre que a solução de Gustavo não está correta.

PROBLEMA 3

EUREKA! N 2, 1998

10
Sociedade Brasileira de Matemática

Sejam ABCD um quadrado, M o ponto médio de AD e E um ponto sobre o


lado AB. P é a interseção de EC e MB. Mostre que a reta DP divide o
segmento EB em dois segmentos de mesma medida.

PROBLEMA 4

Mostre que existem infinitos inteiros positivos n satisfazendo


simultaneamente as seguintes condições:
i. n é ímpar;
ii. n possui exatamente 1200 divisores positivos;
iii. existem exatamente 1997 triângulos retângulos, dois a dois não
congruentes, de lados inteiros e n como medida de um dos catetos.

PROBLEMA 5

Seja n  1 um inteiro. Temos n lâmpadas alinhadas e numeradas, da


esquerda para a direita, de 1 a n. Cada lâmpada pode estar acesa ou
apagada. A cada segundo, determina-se a lâmpada apagada de maior
número e inverte-se o estado desta (de acesa para apagada ou de apagada
para acesa) e das lâmpadas posteriores (as lâmpadas de maior número).

a) Mostre que em algum momento todas as lâmpadas estarão acesas


(e o processo se encerrará).

b) Suponha que inicialmente todas as lâmpadas estejam apagadas.


Determine depois de quantos segundos todas as lâmpadas estarão
acesas.

c) Suponha agora n = 11 e que no início somente as lâmpadas de


números 6, 7 e 10 estejam acesas. Mostre que após exatamente
1997 segundos todas as lâmpadas estarão acesas.

SOLUÇÕES

EUREKA! N 2, 1998

11
Sociedade Brasileira de Matemática

1)

Seja a o andar do apartamento de Eduardo. Então o número de seu


apartamento é 12 (a – 1) + b, com 1  b  12. Daí,

a + 12 ( a – 1 ) + b = 2164,
b = 2176 – 13a
1  2176 – 13a  12
a = 167, b = 5

Portanto, o número do apartamento de Eduardo é:


12 (a – 1) + b = 12  166 + 5 = 1997.

2)

a) Uma maneira é mostrada abaixo:

b) Suponha que a solução de Gustavo esteja correta. Sejam A, B, C, D, E,


F os pontos. Então, como os 6 pontos determinam 15 cordas, somente
dois segmentos não foram coloridos. Estes dois segmentos incidem em
3 ou 4 vértices.

i.) Se A é vértice comum de dois segmentos não coloridos, AB e AF,


então caso existem 6 quadriláteros totalmente coloridos: ACDE,
BCDE, BCDF, BCEF, BDEF e CDEF.
ii.) Se os segmentos AB e EF não foram coloridos então existem 4
quadriláteros coloridos: CDAE, CDAF, CDBE, CDBF.

3)

EUREKA! N 2, 1998

12
Sociedade Brasileira de Matemática

A E B

P
M

N D C

Prolongue BM até encontrar o prolongamento do lado CD no ponto N.


Claramente, AMB DMN , donde segue que AB = DN . Portanto, D
é o ponto médio de CN. O resultado segue observando que os triângulos
CPN e EPB são semelhantes e, como PD é mediana do triângulo CPN,
conclui-se que o prolongamento de DP encontra EB em seu ponto médio.

4)

Seja n um número natural ímpar. Vamos calcular o número de triângulos


retângulos de lados inteiros nos quais n é medida de um dos catetos. Para
isso, devemos ter

n2  x2  y2 ,
n 2 ( y  x )( y  x ),

com x e y inteiros positivos, x < y. Observe que (y – x) < (y + x). Se


fizermos (y – x) = d, com d um divisor de n2, d será menor que n e
(y + x) = n2/d será maior que n. Para qualquer d satisfazendo estas
condições, podemos encontrar uma solução:

EUREKA! N 2, 1998

13
Sociedade Brasileira de Matemática

 1  n2 
x    d
 2  d 
 y  x d 
 
 n  2 
 y  x   1  n2 
 d  y    d 
 2 d 


Estas soluções são inteiras e positivas, pois n é ímpar (logo d também), e
d  n . Portanto, o número de triângulos retângulos é o número de
divisores de n2 menor que n. Mas para cada divisor de n2 menor que n,
corresponde um divisor maior que n. Lembrando que n é também um
divisor, concluímos que o número procurado é 1/2 (d(n2) – 1), onde d(n) é
o número de divisores positivos de n. Portanto, é necessário e suficiente
que n2 seja um número ímpar com d(n2) = 2  1997 + 1 = 3995 divisores.
Uma das várias possibilidades para n2 ter 3995 divisores é ser da forma
p4q798, com p e q primos distintos. Neste caso, n = p2q399, possui
d(n) = (2 +1)  (399 +1) = 1200 divisores.

5)

Vamos representar por 1 uma lâmpada acesa, e por 0 uma lâmpada apagada
e interpretar o número obtido na base 2.

Veja que se, em algum passo, o último dígito for 0, ele será o único dígito
alterado no próximo passo. Isto significa que o número aumentará 1
unidade.
Caso contrário, o número terminará com um bloco de 1's antecipado por
um 0:…011…1. No próximo passo, o número será …100…0. Mas observe
que (…011…1) + 1 = …100…0. Portanto, em qualquer caso, o número k é
sucedido pelo número k + 1.

EUREKA! N 2, 1998

14
Sociedade Brasileira de Matemática

a) Dada qualquer disposição inicial das lâmpadas, ou seja, qualquer


número binário de no máximo n dígitos, em algum momento todos os
dígitos serão iguais a 1, pois este é o maior número de n dígitos na base
2.
b) Existem 2n números de no máximo n dígitos na base 2. Começando
com 0, devemos chegar a 2 n–1, passando por todos os naturais
intermediários. São necessários, então, 2n–1 segundos.
c) Observe que a configuração inicial representa o número 2 5 + 24 + 2 =
50. Para n = 11, todas as lâmpadas estarão acesas depois de (211–1) –
50 = 1997 segundos.

Você sabia… Que um polígono regular com


um número ímpar de lados só pode ser construído exatamente
com régua e compasso se o número de lados for um produto
k
de primos distintos da forma 2 2  1 (esses primos são chamados primos de Fermat) ?
E que só são conhecidos 5 primos de Fermat: 3, 5, 17, 257 e 65537,
k
apesar de Fermat ter conjecturado que todo número da forma 22 1 é primo

EUREKA! N 2, 1998

15
Sociedade Brasileira de Matemática

(isso já é falso para k = 5 : 232 + 1 é divisível por 641.) ??

IV OLIMPÍADA DE MAIO
Resultados

Primeiro nível

Fabio Dias Moreira Ouro Coord. Est. Rio de Janeiro-RJ


Davi M. Alexandrino Nogueira Prata Militar Fortaleza-CE
Lyussei Abe Prata Etapa São Paulo-SP
Cibele Norie Sakai Uyhara Prata Integrado Itatiba-SP
Pedro Davoli Ometto Bronze Koelle Rio Claro-SP
Kelly Correa de Paula Bronze M.Schledorn Jundiaí-SP
Marcelo Kenji Honda Bronze Pioneiro São Paulo-SP
Rafael Martins Gomes Nascimento Bronze S. Dumont Fortaleza-CE
Priscila Carrara Menção Cass. Ricardo S. J. Campos-SP
Thiago Pimentel Nykiel Menção Militar Juiz de Fora-MG
Rodrigo Evangelista Delgado Menção Militar Juiz de Fora-MG
Luiz Eduardo de Godoi Menção Cass. Ricardo S. J. Campos-SP

Segundo nível

Hugo Pinto Iwata Ouro SETA S.J.Rio Preto-SP


Ulisses Medeiros de Albuquerque Prata Militar Fortaleza-CE
Afonso de Paula P. Rocha Prata S. Dumont Fortaleza-CE
Artur D. Nelmi Bronze Bandeirantes São Paulo-SP
Luiz Fernando Mendes Correa Bronze Militar Juiz de Fora-MG
Andre de Almeira Bosso Bronze Progresso Araraquara-SP
Fabricio Siqueira Benevides Bronze 7 de setembro Fortaleza-CE
Luiz Brizeno Firmeza Neto Menção Evolutivo Fortaleza-CE
Daniel Nobuo Uno Menção Etapa São Paulo-SP
Juliana Regina C. Zucare Menção Bandeirantes São Paulo-SP

Os alunos Fabio Dias Moreira (Rio de Janeiro, RJ) e Hugo Pinto


Iwata (São José do Rio Preto, SP) receberam medalha de ouro na
Olimpíada e com isso, ganharam uma viagem de uma semana para a
Argentina onde se reunirão com os ouros dos outros países para diversas

EUREKA! N 2, 1998

16
Sociedade Brasileira de Matemática

atividades turísticas e culturais. Esta viagem será realizada em outubro, em


data que ainda será marcada.

A seguir, publicamos a prova da IV Olimpíada de maio, com as


respostas dos problemas.

IV OLIMPÍADA DE MAIO
Primeiro nível

Duração da prova: 3 horas.


Cada problema vale 10 pontos.
Não se pode usar máquina de calcular.
Não se pode consultar livros nem notas.

PROBLEMA 1

Com seis varetas se construiu uma peça como a da


figura. As três varetas exteriores são iguais entre si.
As três varetas interiores são iguais entre si. Deseja-
se pintar cada vareta de uma cor só de modo que, em
cada ponto de união, as três varetas que chegam
tenham cores diferentes.As varetas só podem ser
pintadas de azul, branco, vermelho ou verde.
De quantas maneiras pode-se pintar a peça?

PROBLEMA 2

Têm-se 1998 peças retangulares de 2cm de altura e 3cm de comprimento e


com elas se armam quadrados (sem superposições nem buracos). Qual é a
maior quantidade de quadrados diferentes que se pode ter ao mesmo
tempo?

PROBLEMA 3

Existem quatro botes numa margem de um rio; seus nomes são Oito,
Quatro, Dois e Um, porque essas são as quantidades de horas que cada um

EUREKA! N 2, 1998

17
Sociedade Brasileira de Matemática

deles demora para cruzar o rio. Pode-se atar um bote a outro, porém não
mais de um, e então o tempo que demoram em cruzar é igual ao do mais
lento dos botes. Um só marinheiro deve levar todos os botes até à outra
margem do rio. Qual é o menor tempo necessário para completar o
translado?

PROBLEMA 4

ABCD é um quadrado de centro O.


Sobre os lados DC e AD foram E
construidos os triângulos equiláteros
DAF e DCE. Decida se a área do
triângulo EDF é maior do que, D C
menor do que ou igual à área do
triângulo DOC.
F O

A B

PROBLEMA 5

Escolha um número de quatro dígitos ( nenhum deles zero) e começando


com ele construa uma lista de 21 números distintos, de quatro dígitos cada
um, que satisfaça a seguinte regra: depois de escrever cada novo número da
lista devem-se calcular todas as médias entre dois dígitos desse número,
descartando-se as médias que não dão um número inteiro, e com os que
restam se forma um número de quatro dígitos que ocupará o lugar seguinte
na lista. Por exemplo, se na lista se escreveu o número 2946, o seguinte
pode ser 3333 ou 3434 ou 5345 ou qualquer outro número armado com os
dígitos 3, 4 ou 5.

Segundo nível

PROBLEMA 1

EUREKA! N 2, 1998

18
Sociedade Brasileira de Matemática

Inês escolheu quatro dígitos distintos do conjunto {1,2,3,4,5,6,7,8,9}.


Formou com eles todos os possíveis números de quatro dígitos distintos e
somou todos eses números de quatro dígitos. O resultado é 193314.
Encontre os quatro dígitos que Inês escolheu.

PROBLEMA 2

ABC é um triângulo equilátero. N é um ponto do lado AC tal que


AC  7.AN , M é um ponto do lado AB tal que MN é paralelo a BC e P é
um ponto do lado BC tal que MP é paralelo a AC. Encontre a fração
área ( MNP )
.
área ( ABC )

PROBLEMA 3

Dado um tabuleiro quadriculado de 4  4, com cada casa pintada de uma


cor distinta, deseja-se cortá-lo em dois pedaços de igual área mediante um
só corte, que siga os lados das casas do tabuleiro. De quantas maneiras se
pode fazer isto?

Obs. Os pedaços em que se divide o tabuleiro devem ser peças inteiras; não
devem ser desconectados pelo corte.

PROBLEMA 4

O chão do pátio tem desenhado um octógono regular.


Emiliano escreve nos vértices deste os números de 1 a 8 em qualquer ordem.
Deixa uma pedra no ponto 1.
Caminha em direção ao ponto 2 e, havendo percorrido 1/2 do caminho, se
detém e deixa a segunda pedra.
Daí caminha em direção ao ponto 3 e, havendo percorrido 1/3 do caminho,
se detém e deixa a terceira pedra.
Daí caminha em direção ao ponto 4 e, havendo percorrido 1/4 do caminho,
se detém e deixa a quarta pedra.
Deste modo segue até que, depois de deixar a sétima pedra, caminha em
direção ao ponto 8, e havendo percorrido 1/8 do caminho, deixa a oitava
pedra.

EUREKA! N 2, 1998

19
Sociedade Brasileira de Matemática

A quantidade de pedras que ficarem no centro do octógono depende da


ordem em que ele escreveu os números nos vértices. Qual é a maior
quantidade de pedras que podem ficar no centro?

PROBLEMA 5

O planeta X31 tem só dois tipos de notas, mas o sistema não é tão mau já
que só há quinze preços inteiros para os quais o pagamento não pode ser
feito de forma exata (nesses casos deve-se pagar a mais e receber o troco).
Se 18 é um dos preços para os quais não se pode fazer pagamento exato,
encontre o valor de cada tipo de nota.

RESPOSTAS

IV OLIMPÍADA DE MAIO Primeiro nível 1998

PROBLEMA 1: 16 formas.
PROBLEMA 2 : 9
PROBLEMA 3: 15h
PROBLEMA 4 : As áreas são iguais.
PROBLEMA 5 : Há muitas soluções.

IV OLIMPÍADA DE MAIO Segundo nível 1998

PROBLEMA 1 : 5, 7, 8, e 9
PROBLEMA 2 : 6/49
PROBLEMA 3: 70 maneiras.
PROBLEMA 4 : 4 pedras.
PROBLEMA 5: 4 e 11

EUREKA! N 2, 1998

20
Sociedade Brasileira de Matemática

Você sabia… Que a todo momento há dois


pontos antípodas na terra com a mesma
temperatura e a mesma pressão (admitindo que
temperatura e pressão dependem
continuamente do ponto)??

9a. OLIMPÍADA DE MATEMÁTICA DO CONE SUL

A 9ª. Olimpíada de Matemática do Cone Sul foi realizada em


Salvador, BA, no período de 13 a 21 de junho de 1998. Esta Olimpíada foi
realizada pela segunda vez no país (a primeira foi em 1993, em Petrópolis,
RJ). Dela participaram alunos de até 15 anos dos seguintes países:
Argentina, Brasil, Bolívia, Chile, Paraguai, Peru e Uruguai. A organização
da Olimpíada esteve a cargo da Professora Luzinalva Amorim, da
Universidade Federal da Bahia.
A equipe brasileira foi selecionada através de provas realizadas em
março e maio deste ano e foi liderada pelos professores Paulo Cezar Pinto
Carvalho, do IMPA, e Florêncio Ferreira Guimarães, da UFES.
A competição constou de duas provas, realizadas em dois dias,
cada uma com três problemas, valendo 10 pontos cada. Veja a seguir os
resultados obtidos pela equipe brasileira e as provas da 9 a. Olimpíada de
Matemática do Cone Sul.

RESULTADOS DA EQUIPE BRASILEIRA

BRA 1 Mila Lopes Viana Bronze


BRA 2 Pedro Paulo Gouveia Prata
BRA 3 Fabricio Siqueira Benevides Prata
BRA 4 Jônathas Diógenes Castelo Branco Bronze

EUREKA! N 2, 1998

21
Sociedade Brasileira de Matemática

Você sabi@ que a


Olimpíada Brasileira de Matemática
tem página web??

Visite-nos no endereço eletrônico


http://www.obm.org.br

9a. OLIMPÍADA DE MATEMÁTICA DO CONE SUL


Problemas e soluções

Primeiro dia.
Tempo: 4 horas 30 min.

PROBLEMA 1

São dados 98 cartões. Em cada um deles está escrito um dos números 1, 2,


3, …, 98 (não existem números repetidos). Pode-se ordenar os 98 cartões
de tal modo que ao considerar dois cartões consecutivos a diferença entre o
número maior e o número menor escritos neles seja sempre maior que 48.
Indicar como e de quantas formas é possível efetuar a ordenação.

PROBLEMA 2

Sejam H o ortocentro (interseção das alturas) do triângulo acutângulo ABC


e M o ponto médio do lado BC. Seja X o ponto em que a reta HM intersecta
o arco BC (que não contém A) da circunsferência circunscrita a ABC. Seja
Y o ponto de interseção da reta BH com a circunsferência, distinto de B.
Demonstre que XY = BC.

PROBLEMA 3

Prove que, pelo menos para 30% dos naturais n entre 1 e 1.000.000, o
primeiro dígito de 2n é 1.

Segundo dia.
Tempo: 4 horas 30 minutos.

EUREKA! N 2, 1998

22
Sociedade Brasileira de Matemática

PROBLEMA 4

Determine todas as funções f tais que


f ( x 2 )  f ( y 2 )  2 x  1  f ( x  y ) f ( x  y )

quaisquer que sejam os números reais x, y.

PROBLEMA 5

Em Terra Brasilis existem n casas onde vivem n duendes, cada um em uma


casa. Existem estradas de mão única de tal modo que:

 cada estrada liga duas casas;


 em cada casa começa exatamente uma estrada;
 em cada casa termina exatamente uma estrada.

Todos os dias, a partir do dia 1, cada duende sai da casa onde está e chega à
casa vizinha. Uma lenda de Terra Brasilis diz que, quando todos os
duendes regressarem à posição original, o mundo acabará.

(a) Demonstre que o mundo acabará.


(b) Se n = 98, demonstre que é possível que os duendes construam e
orientem as estradas de modo que o mundo não se acabe antes de
300.000 anos.

PROBLEMA 6

O Prefeito de uma cidade deseja estabelecer um sistema de transportes com


pelo menos uma linha de ônibus, no qual:

(i) cada linha passe exatamente por três paradas;


(ii) cada duas linhas distintas tenham exatamente uma parada em
comum;
(iii) para cada duas paradas de ônibus distintas exista exatamente uma
linha que passe por ambas.

Determine o número de paradas de ônibus da cidade.

EUREKA! N 2, 1998

23
Sociedade Brasileira de Matemática

SOLUÇÕES

1)

Vamos provar uma versão um pouco mais geral do problema:

Seja k um número natural. Encontrar todas as permutações a1, a2, …a2k dos
números 1, 2, …, 2k que verificam ai  a i 1 k para todo i = 1, 2, …,
2k –1.

Solução
Em primeiro lugar observamos que, se dois números entre 1, 2, …, 2k
diferem de pelo menos k, então o maior dos números está entre k + 1, k + 2,
… 2k e o menor, entre 1, 2, …, k . Chamemos simplesmente os números
destes dois conjuntos de "grandes" e "pequenos", respectivamente.
Suponhamos que a1, a2, …a2k é uma permutação com a propriedade em
questão. Pelo que dissemos acima, seus termos com índice ímpar (par)
devem ser todos grandes ou todos pequenos. Sejam, por exemplo, a1, a3, …
a2k – 1 pequenos e a2, a4, …a2k grandes. Consideremos a soma

S  a1  a 2  a 2  a 3  ...  a 2 k  2  a 2 k  1  a 2 k  1  a 2 k

Como cada termo de índice par é maior do que seus vizinhos,

S (a 2  a1 )  (a 2  a3 )  ...  (a 2 k  2  a 2 k  1)  (a 2 k  a 2 k  1 )
2(a 2  a 4  ...  a 2 k )  2(a1  a3  ...a 2 k  1 )  a 2 k  a1

2((k  1)  (k  2)  ...  2k )  2(1  2  ...  k )  a 2 k  a1


2k 2  (a 2 k  a1 )

Notemos que a condição ai  a i 1 k determina a escolha de a1 e a2k .


Os únicos vizinhos possíveis de k e k +1 são 2k e 1, respectivamente. Logo

EUREKA! N 2, 1998

24
Sociedade Brasileira de Matemática

k e k+1 devem ser o primeiro e o último termos da permutação. E como


escolhemos começar com a1 pequeno, a1 = k, a2k = k +1.
Então a2 = 2k, a2k –1 = 1.
Regressando à soma S, vemos que ela é igual a 2k2 – (( k +1) – k ) = 2k2 – 1.
Por outro lado, cada dois somandos da forma ai  1  a i  a i  a i 1
contribui com pelo menos k + ( k + 1) = 2k + 1. Isto se deve ao fato de ser
impossível que a i  1  a i  a i  a i 1 k , pois teríamos, neste caso, ai – 1
= ai + 1. Assim, temos

S (2k  1)  (2k  1)  ...  (2k  1)  k 2k 2  1


                 
k1

Então ai  1  ai  a i  a i 1 2k  1 para todo i = 1, 3, … 2k – 1. Isto é,


as diferenças consecutivas são k, k + 1, k, k + 1, …, k. Começando com
a1 = k, a2 = 2k (que diferem em k ), podemos determinar todos os ai da
seqüência:

a 3  2k  ( k  1)  k  1
a 4  ( k  1)  k  2k  1
a 5  ( 2k  1)  ( k  1)  k  2

Portanto a1, a2 , …, a2k é

k, 2k, k – 1, 2k – 1, k – 2, …, 2, k + 2, 1, k + 1

Por simetria, existe exatamente uma solução além desta: a que obtemos
tomando a solução acima na ordem inversa.

EUREKA! N 2, 1998

25
Sociedade Brasileira de Matemática

2)

Seja {L} = PM  AH. Mostraremos que L coincide com H. Inicialmente,


observe que  NPA = 90 (pois AM é diâmetro). Prolonguemos PM até
encontrar a circunsferência circunscrita no ponto N, diametralmente oposto
ao vértice A (pois  NPA = 90.)
Logo o circuncentro O é o ponto médio de AN e, como OM  AL, segue
que M é o ponto médio de LN; como m é ponto médio de BC , segue que
LBNC é um paralelogramo, de modo que BL  NC . Mas  NCA = 90
(pois AN é diâmetro), ou seja, NC  AC. Daí segue que BL  AC e, como
AL  BC, concluimos que L  H.

3)

Vamos provar que para cada inteiro positivo k existe uma potência de 2
com exatamente k dígitos (na base 10) e cujo primeiro dígito é 1. De fato,
se considerarmos a menor potência de 2 maior que 10 k + 1, devemos ter
2 n < 10 k + 1  2n + 1,
ou 10 k + 1  2n + 1 < 2  10 k + 1 .
6
Portanto basta calcular quantos dígitos possui 210 . Mas, de 103 < 210,
5 6 6
obtemos 10 310  210 , donde segue que 210 tem mais de 300.000
algarismos e segue que no mínimo 300.000/1.000.000 = 30% de tais
potências começam com o algarismo 1.

EUREKA! N 2, 1998

26
Sociedade Brasileira de Matemática

Observações:

1. Utilizamos somente que existe uma potência de 2 que começa com o


dígito 1 e possui exatamente k dígitos. Como verifica-se
imediatamente, existe exatamente uma potência de 2 com k dígitos que
começa com 1.
6 t 6 t+1
2. 210 possui exatamente 301.030 algarismos, pois se 10 < 210 < 10 ,
aplicando logaritmos, vem t < 106 log 2 < t + 1, donde t + 1 = 301.030.

3. Utilizando as idéias de 1 e 2, é possível mostrar que a probabilidade de


uma potência de 2 começar com o algarismo 1 é log 2. Mais
precisamente, se f (n) é o número de inteiros k (1  k  n ) tais que 2k
que iniciam com o algarismo 1, então

f (n)
lim  log 2  0,301029995664.
n  n

4)

Fazendo x = y temos
f (2x)  f (0)= 2x + 1

Logo, para x = 0, (f (0))2 = 1  f (0) = 1

Assim, f (2x) = (2x +1) e, portanto, f (x) = x + 1 ou f (x) = – (x + 1).


Substituindo as funções encontradas na equação funcional original,
verificamos que apenas f (x) = x + 1 satisfaz as condições do problema.

5)

(a) Numere os duendes de 1 a n e seja f(i) o vizinho do duende número


i. A função f é claramente uma bijeção. Em algum momento cada
duende retornará a sua casa pois a seqüência f (i), f ( f (i) ),
f ( f ( f (i))),…assume um número finito de valores, donde existirão
inteiros positivos r < s tais que f s (i) = f r(i), portanto
f s – r (i) = i (pois f é bijetora). Seja g(i) o menor inteiro positivo tal
que o duende i retorna à sua casa depois de g(i) dias. Depois de

EUREKA! N 2, 1998

27
Sociedade Brasileira de Matemática

mmc(g(1), g (2),…,g(n)) dias, todos os duendes retornarão à


posição original e o mundo acabará.

(b) Divida os 98 duendes em 8 ciclos de tamanhos 3, 5, 7, 11, 13, 17,


19, 23 (98 = 3 + 5 + 7 + 11 + 13 + 17 + 19 + 23). Os duendes
retornarão à posição inicial depois de 3  5  7  11  13  17 
19  23 = 111546435 > 366  300.000.
Alternativamente, podemos dividir os duendes em ciclos de
tamanhos 3, 8, 9, 5, 7, 11, 13, 19 e 23, e eles retornarão à posição
original em mmc (3, 8, 9, 5, 7, 11, 13, 19, 23) = 8  9  5  7  11
 13  19  23 = 157.477.320

6)

Um exemplo de tal sistema é aquele que tem uma só linha com exatamente
3 pontos. Para o que segue, suponhamos que haja pelo menos 4 pontos 1, 2,
3, 4 e que uma das linhas é R1 = 123 (aqui, e no que segue, R = abc
significa que a linha R passa pelos pontos a, b, c, não importando a ordem.
Assim, por exemplo, R = bca é a mesma linha.)
Por (iii), devem existir linhas R2, R3 e R4 que passam pelos pares de pontos
{1, 4}, {2, 4} e {3, 4}, respectivamente. Notemos que R2 , R3 e R4 devem
ser distintas. De fato, se, digamos, os pares {2, 4} e {3, 4} estão na mesma
linha R2 , então R2 = 234, logo R1 e R2 têm duas paradas em comum e isto é
impossível por (ii). Novamente por (ii), cada uma das linhas R2 , R3 e R4
tem exatamente um ponto em comum com R1 = 123. Como não podem
haver dois pontos entre 1, 2, 3, que estão em R2 , R3 e R4 (novamente por
(ii)), devemos ter R2 =14a, R3 = 24b, R4 = 34c para pontos distintos a, b, c
que são por sua vez distintos de 1, 2, 3, 4. Para manter uma notação
consistente, sejam a = 5, b = 6 e c = 7. Logo R2 = 145, R3 = 246 e R4 = 347.
Com isso, provamos que há pelo menos 7 pontos.
Agora, suponhamos que exista pelo menos um ponto a mais, digamos 8.
Por (iii), existe uma linha S que passa por 1 e 8. Como S tem uma parada
em comum com R3 = 246, concluímos que S = 128, S = 148 ou S = 168.
Nenhuma destas é possível, pois

1. as linhas 128 e 148 têm dois pontos em comum com R1 = 123 e


R2 = 145, respectivamente.
2. 168 não tem ponto em comum com R4 = 347.

EUREKA! N 2, 1998

28
Sociedade Brasileira de Matemática

Esta contradição é devida a termos suposto que existem mais de 7 pontos.


Completamos a construção do sistema de transportes com 7 pontos de
ônibus. Devem haver linhas R5, R6, R7 por {1, 6}, {2, 5}, {3, 5},
respectivamente (pois as linhas não estão entre as já existentes R1 , R2 , R2 ,
R3 e R4).
Pode-se verificar que a escolha R5 = 167, R6 = 257 e R7 = 356 funciona.
As 7 linhas 123, 145, 246, 347, 167, 257, 356 formam um exemplo de tal
sistema.
Concluímos, então, que a cidade pode ter exatamente 3 ou exatamente 7
pontos de ônibus.

Observação:

Este problema é equivalente a particionar as arestas de um grafo completo


Kn em triângulos de modo que quaisquer dois triângulos tenham
exatamente um vértice em comum.
Observando que, satisfeitas as condições do problema, cada vértice de um
triângulo é comum a (n – 3)/2 outros triângulos, o total de triângulos em Kn
é
n  3 1 n( n  1)
13  , o que só se verifica quando n = 3 ou n = 7.
2 3 2
Para concluir a resolução, basta obter as partições nestes casos.

39a. OLIMPÍADA INTERNACIONAL DE MATEMÁTICA

EUREKA! N 2, 1998

29
Sociedade Brasileira de Matemática

Resultados e problemas

No mês de tantas expectativas dos brasileiros, o Brasil consegue


uma medalha de ouro na 39a. Olimpíada Internacional de Matemática
realizada com a presença de 76 países em Taiwan nos dias 10 a 21 de julho
último.
O estudante Rui Lopes Viana Filho (SP) foi ganhador de uma
medalha de ouro. Também foram premiados os estudantes Emanuel
Carneiro (CE) medalha de bronze, Murali Vajapeyam (PB) menção
honrosa e Mauricio Carrari (SP) menção honrosa. Trata-se de feito muito
importante, visto que países como Alemanha, Inglaterra, Israel, Suécia,
Australia e muitos outros não conquistaram medalhas de ouro.

Merece também elogios o fato da equipe brasileira ter sido uma


das que tiveram melhor desempenho na questão 5 da prova, superando por
exemplo, as equipes dos EUA e da Rússia. Veja a seguir as questões da
39a.Olimpíada Internacional de Matemática.

Primeiro dia
Duração da Prova: 4 horas 30 min.

PROBLEMA 1

No quadrilátero convexo ABCD, as diagonais AC e BD são


perpendiculares e os lados opostos AB e DC não são paralelos. Sabemos
que o ponto P, onde se intersectam as mediatrizes de AB e DC, está no
interior de ABCD. Prove que ABCD é um quadrilátero inscritível se, e
somente se, os triângulos ABP e CDP têm áreas iguais.

PROBLEMA 2

Numa competição, existem a concorrentes e b juízes, onde b  3 é um


inteiro ímpar. Cada juiz avalia cada um dos concorrentes, classificando-o
como "aprovado" ou "reprovado". Suponha que k é um número tal que as

EUREKA! N 2, 1998

30
Sociedade Brasileira de Matemática

classificações dadas por dois juízes quaisquer coincidem no máximo para k


k b 1
concorrentes. Prove que  .
a 2b

PROBLEMA 3

Para qualquer inteiro positivo n, seja d(n) o número de divisores positivos


de n (incluindo 1 e n).
d (n 2 )
Determine todos os inteiros positivos k tais que k para algum n.
d ( n)

Segundo dia
Duração da Prova: 4 horas 30 min.

PROBLEMA 4

Determine todos os pares (a, b) de inteiros positivos tais que ab2 + b + 7


divide a2b + a + b.

PROBLEMA 5

Seja I o incentro do triângulo ABC. A circunferência inscrita no triângulo


ABC é tangente aos lados BC, CA e AB nos pontos K, L e M,
respectivamente. A reta que passa por B, paralela ao segmento MK,
intersecta as retas LM e LK nos pontos R e S, respectivamente. Prove que o
ângulo RIS é agudo.

PROBLEMA 6

Considere todas as funções f definidas no conjunto N dos inteiros


positivos, com valores no mesmo conjunto, que satisfazem
f (t 2 f ( s ))  s ( f (t )) 2 , para todos s e t em N. Determine o menor valor
possível de f(1998)
PARIDADE
Eduardo Wagner

EUREKA! N 2, 1998

31
Sociedade Brasileira de Matemática

 Nível Iniciante.

Todo número natural é par ou ímpar.

Elementar, não? A afirmação acima, que é uma das mais simples e


óbvias da Matemática, é também uma ferramenta de grande utilidade na
resolução de muitos problemas envolvendo números naturais. Vamos
comentar neste artigo alguns deles, em graus diferentes de dificuldade, mas
inicialmente precisamos recordar três importantes propriedades:

a) a soma de dois números pares é par.


b) a soma de dois números ímpares é par.
c) a soma de um número par com um número ímpar é ímpar.

Dizemos que dois números inteiros têm mesma paridade, quando


são ambos pares ou ambos ímpares. Assim, podemos dizer que a soma de
dois números inteiros é par se, e somente se, eles têm mesma paridade.
Vamos aos problemas.

PROBLEMA 1
Em um quartel existem 100 soldados e, todas as noites, três deles são
escolhidos para trabalhar de sentinela. É possível que após certo tempo um
dos soldados tenha trabalhado com cada um dos outros exatamente uma
vez?

RESPOSTA : Não.
Escolha um soldado. Em cada noite em que trabalha, ele está em
companhia de dois outros. Como 99 é um número ímpar, não podemos
formar pares de soldados sempre diferentes para trabalhar com o escolhido.

PROBLEMA 2
Um jogo consiste de 9 botões luminosos (de cor verde ou vermelha)
dispostos da seguinte forma:

EUREKA! N 2, 1998

32
Sociedade Brasileira de Matemática

1 2 3

4 5 6

7 8 9

Apertando um botão do bordo do retângulo, trocam de cor ele e


seus vizinhos (do lado ou em diagonal). Apertando o botão do centro,
trocam de cor todos os seus 8 vizinhos porém ele não.

Exemplos:

Apertando 1, trocam de cor 1, 2, 4 e 5.


Apertando 2, trocam de cor 1, 2, 3, 4, 5 e 6.
Apertando 5, trocam de cor 1, 2, 3, 4, 6, 7, 8 e 9.

Inicialmente todos os botões estão verdes. É possível, apertando


sucessivamente alguns botões, torná-los todos vermelhos?

RESPOSTA : Não é possível.


Observe que apertando um botão do vértice do retângulo, trocam de cor 4
botões. Apertando um botão do meio de um lado, trocam de cor 6 botões e
apertando um botão do centro trocam de cor 8 botões. Assim, cada vez que
apertamos um botão trocam de cor um número par de botões. Como
existem 9 botões, não é possível que todos troquem de cor.

PROBLEMA 3
Escrevemos abaixo os números naturais de 1 a 10.

1 2 3 4 5 6 7 8 9 10.

Antes de cada um deles, coloque sinais “+” ou “–” de forma que a soma de
todos seja zero.

SOLUÇÃO: Não é possível fazer isto.


Imaginando que fosse possível, deveríamos separar os números dados em
dois grupos com a mesma soma. Então colocaríamos sinais negativos nos

EUREKA! N 2, 1998

33
Sociedade Brasileira de Matemática

números de um dos grupos e sinais positivos nos números do outro.


Teríamos então uma soma igual a zero. Acontece que a soma dos números
naturais de 1 a 10 é igual a 55. Como este número é ímpar, não podemos
separar os números dados em dois grupos que tenham a mesma soma.

Como o leitor deve estar percebendo, os argumentos utilizados


permitiram concluir que as respostas dos três problemas propostos foram
iguais: “não é possível fazer tal coisa”. Na maioria das vezes, um
argumento de paridade serve exatamente para isto. Mostrar que um
determinado fato não pode ocorrer e isto não é desanimador, muito pelo
contrário. Serve para nos convencer que não adianta ficar gastando tempo
demais fazendo tentativas inúteis. As experiências são valiosas no sentido
de nos abrir os olhos para a possibilidade do problema não ter solução e, a
partir daí, buscar um argumento que resolva definitivamente a questão.

É muito importante também explorar um problema, ou seja,


imaginar pequenas modificações no enunciado e verificar o que ocorre com
sua resposta. Por exemplo, o problema 3 não tem solução porque a soma
dos naturais de 1 até 10 é 55 (ímpar). O que ocorreria se a soma fosse par?
Este é um novo e atrativo problema. Vamos enunciá-lo:

PROBLEMA 3A:
Escrevemos abaixo os números naturais de 1 a 11.

1 2 3 4 5 6 7 8 9 10 11

Antes de cada um deles, coloque sinais “+” ou “–” de forma que a soma de
todos seja zero.

SOLUÇÃO:
A soma dos números naturais de 1 a 11 é 66. Como podemos separá-los em
dois grupos de soma 33? Começando pelos maiores observe que 11 + 10 +
9 = 30. Logo, 11 + 10 + 9 + 3 = 33. O problema 3A tem como uma solução
possível:

+1 + 2 – 3 + 4 + 5 + 6 + 7 + 8 – 9 – 10 – 11 = 0

EUREKA! N 2, 1998

34
Sociedade Brasileira de Matemática

Fica ao encargo do leitor mostrar que sempre que a soma dos


naturais de 1 até n é par então podemos separá-los em dois grupos de igual
soma. Você pode utilizar o caminho que utilizamos acima, ou buscar uma
outra forma.

Para saber mais e intrigar seus colegas

Você pode propor aos seus amigos os problemas 3 ou 3A com uma lista
grande de números naturais consecutivos. O problema terá ou não solução
caso a soma desses números seja par ou ímpar, respectivamente.
Entretanto, é possível encontrar o resultado desta soma rapidamente, sem
precisar somar todas as parcelas. A soma de todos os naturais de 1 até n é
(1  n ) n
igual a . Por exemplo, a soma de todos os naturais de 1 até 10 é
2
(1  10)10 11 10
 55 . Procure demonstrar este fato e, se não
2 2
conseguir, pergunte ao seu professor ou escreva para a EUREKA!

PROBLEMA 4
Mostre que se a, b e c são inteiros ímpares, a equação ax 2  bx  c 0 não
tem raiz racional.

Comentários:

1) Um número é raiz de uma equação dada se quando for substituído no


2
lugar do “x” a igualdade ficar correta. Por exemplo, x  é raiz (ou
3
2
solução) da equação 3 x  2  0 porque 3   2  0 . Ainda, x 2 é
3
solução da equação x 4  x 3  x  10 0 porque 2 4  2 3  2  10 0 .
Freqüentemente não sabemos como resolver uma equação mas, em geral,
podemos verificar se um certo valor de x é ou não uma de suas raízes.

2) Um número é racional quando puder ser escrito como uma fração de


2 4
numerador e denominador inteiros. Por exemplo, e são exemplos
7 1
de números racionais.

EUREKA! N 2, 1998

35
Sociedade Brasileira de Matemática

3) Quando desejamos demonstrar que certo fato é impossível utilizamos


freqüentemente o método da redução ao absurdo. Este método consiste em
imaginar o contrário, ou seja, que tal fato seja possível. A partir daí
procuramos chegar a uma contradição, a um absurdo. Conseguindo isso,
teremos mostrado que nossa hipótese (a do contrário) é falsa e
conseqüentemente, que a afirmação inicial é verdadeira.

Vamos ver tudo isso na solução do problema. Não se preocupe se você


ainda não sabe resolver uma equação do segundo grau. Isto não será
necessário. Tudo o que precisamos é verificar se um número racional pode
ser uma raiz.

Solução do problema 4
p
Imaginemos que o número racional seja raiz da equação
q
ax 2  bx  c 0 onde a, b e c são inteiros ímpares. Logo, fazendo a
substituição, devemos ter,

2
 p  p
a   b   c  0
q q

p2 p
a 2
 b  c 0
q q

ap 2  bpq  cq 2 0

Vamos acrescentar agora uma hipótese importante para facilitar nosso


p
trabalho. Vamos supor que a nossa fração seja irredutível, ou seja, que
q
4
ela já foi simplificada ao máximo. Por exemplo, no lugar de estaremos
6
2
considerando o que é a mesma coisa. Consideramos então, para a
3
solução do problema, que p e q não são ambos pares.
2 2
Observe agora a equação ap  bpq  cq 0 nos seguintes casos:

EUREKA! N 2, 1998

36
Sociedade Brasileira de Matemática

2 2
a) p e q são ímpares: neste caso, ap é ímpar, bpq é ímpar e cq é
ímpar. Como a soma de três números ímpares é ímpar, o resultado não
pode ser zero.
2 2
b) p é par e q é ímpar: neste caso, ap é par, bpq é par e cq é ímpar.
Como a soma de dois números pares e um ímpar é ímpar, o resultado não
pode ser zero.

c) p é ímpar e q é par: vale o mesmo argumento do caso b).

Demonstramos então que nenhuma fração de numerador e denominador


inteiros pode ser raiz da equação ax 2  bx  c 0 onde a, b e c são inteiros
ímpares.

PROBLEMA 5
Um tabuleiro 6  6 está coberto com dominós 2  1. Mostre que existe uma
reta que separa as peças do tabuleiro sem cortar nenhum dominó.

SOLUÇÃO:
Cada dominó é formado por dois quadrados e portanto, se o tabuleiro está
inteiramente coberto, 18 dominós foram utilizados. Imagine agora uma reta
(horizontal, por exemplo) que separe o tabuleiro em duas partes. Se ela não
corta nenhum dominó, está resolvido o problema. Suponha então que ela
corte ao meio um dominó. Neste caso, acima desta reta teremos n dominós
inteiros mais meio dominó, ou seja, teremos acima desta reta 2n + 1
quadrados, que é um número ímpar. Mas isto é impossível porque se o
tabuleiro tem 6 unidades de largura, qualquer reta o dividirá em partes que
contém números pares de quadrados acima e abaixo dela. Assim, se uma
reta corta um dominó, deverá cortar um outro dominó. Para a divisão do
tabuleiro, existem 10 retas possíveis e, se cada uma delas cortar dois
dominós, deveríamos ter 20 dominós no tabuleiro. Como eles são apenas
18 então existe uma reta (pelo menos) que não corta nenhum dominó.

Problemas para pesquisa

PROBLEMA 6

EUREKA! N 2, 1998

37
Sociedade Brasileira de Matemática

Os números naturais de 1 até 1998 são escritos em um imenso quadro


negro. Em seguida, um aluno apaga dois quaisquer colocando no lugar sua
diferença (não negativa). Depois de muitas operações, um único número
ficará escrito no quadro. É possível que esse número seja zero?

PROBLEMA 7
Em uma ilha plana existem 11 cidades numeradas de 1 a 11. Estradas retas
ligam 1 a 2, 2 a 3, 3 a 4, ..., 10 a 11 e 11 a 1. É possível que uma reta corte
todas as estradas?

OS PROBLEMAS DO VISITANTE MATEMÁTICO


The Mathematical Visitor foi um periódico que existiu nos Estados
Unidos entre 1877 e 1896. Era uma revista destinada aos amantes da arte
de resolver problemas de Matemática. Publicava problemas propostos pelo
seu abnegado editor ou leitores e, em números subseqüentes, trazia as
melhores soluções apresentadas. Procurava fortalecer entre os
norteamericanos, na época em que sua nação lutava para se inserir no rol
dos países mais desenvolvidos, uma tradição há muito existente na Europa:

EUREKA! N 2, 1998

38
Sociedade Brasileira de Matemática

a prática das saudáveis competições matemáticas públicas, instituídas por


revistas como a famosa Ladies Diary, da Inglaterra.

Os problemas do The Mathematical Visitor eram, em sua grande


maioria, de nível elementar, embora alguns deles exigissem o uso de
integrais em sua resolução. Quanto à criatividade e à elegância das
questões propostas, a qualidade variava bastante. Num período em que
faltavam calculadoras eletrônicas e sobrava lazer para as pessoas, eram
muito freqüentes problemas cuja solução requeria muito mais paciência e
tempo disponível de que engenhosidade e talento.

Um exemplo de problemas desse tipo é o seguinte, que foi


proposto em 1887:

1) Considere a seqüência dos triângulos pitagóricos (triângulos


retângulos de lados inteiros) nos quais os catetos são inteiros
consecutivos. Ache a expressão geral para os lados n-ésimo
triângulo e calcule explicitamente os lados do centésimo. (A
resposta da segunda parte envolve números com 76 algarismos.)

Outros problemas computacionais são:


4
2) Calcular 4 4 .

3) Obter a raiz cúbica de 2 com 100 algarismos decimais!

Mas não se pense que The Mathematical Visitor só trazia perguntas


sem graça. Alguns problemas bem elementares lá propostos ainda guardam
interesse a são apresentados aqui como desafio aos nossos leitores.

4) Com apenas dois cortes retilíneos e recolagem, transforme um


retângulo num quadrado de mesma área, supondo que a razão entre
o maior e o menor lado do retângulo é menor do que ou igual a 4.

5) Comprei na feira um queijo que pesou 9 quilos. Desconfiei da


pesagem e o vendedor propôs, como compensação, vender-me um
queijo igual, desta vez pesado no outro prato da balança. O peso
foi de 4 quilos. Ganhei ou perdi na transação? Qual é o verdadeiro
peso do queijo?

EUREKA! N 2, 1998

39
Sociedade Brasileira de Matemática

6) Ache três números inteiros cuja soma é um cubo e a soma de dois


quaisquer deles também é um cubo.

7) O doutor A mata 3 pacientes em cada 7 que trata; o doutor B mata


4 em cada 13 e o doutor C mata 5 em cada 19. Qual é a
probabilidade de um doente sobreviver se for tratado por esses 3
médicos ao mesmo tempo?

8) Ache quatro inteiros que são quadrados e a soma de dois quaisquer


deles ainda é um quadrado. (Observação: nem a redação da revista
nem o autor do problema sabiam como resolvê-lo.)

Um dos problemas mais interessantes, propostos em 1881, foi o


seguinte:

9) Um vaso de vinho está suspenso sobre outro, de igual capacidade


(digamos 1 litro), cheio de água. Por um orifício no fundo de cada,
o vinho escorre sobre o vaso de água e a mistura se esvai na
mesma velocidade. Quando o vaso de vinho estiver vazio, qual é o
volume de água no vaso inferior?

A coleção de problemas do The Mathematical Visitor foi re-editada


em 1996 pela Math Pro Press, Westford, Mass., sob o título "Problems and
Solutions from The Mathematical Visitor".

Nossa revista aguarda respostas de nossos leitores para os


problemas acima propostos, especialmente os de números 4, 5 e 9.

DIVISIBILIDADE, CONGRUÊNCIAS E ARITMÉTICA MÓDULO n


Carlos Gustavo Moreira

 Nível Avançado

INTRODUÇÃO

Este artigo se propõe a ser uma referência sobre os temas citados


no título, que aparecem naturalmente em diversos problemas de
Matemática elementar, alguns dos quais serão explicitamente tratados aqui.
O estilo é mais conciso do que a maioria dos outros artigos desta revista, o

EUREKA! N 2, 1998

40
Sociedade Brasileira de Matemática

que pode tornar a leitura mais difícil, mas não desanime! Procure entender
os enunciados das proposições e os problemas resolvidos e buscar sua
propria solução para eles, além de pensar nos problemas propostos e
enviar-nos suas soluções. Em caso de qualquer dúvida não deixe de
escrever-nos.

Seção 1: Divisão euclidiana e o teorema fundamental da aritmética


Os resultados que seguem têm como base o seguinte fato sobre os inteiros:
Dados a  Z, b  N* existem q, r  Z com 0  r < b e a = bq + r. Tais q e
r estão unicamente determinados. De fato, q = [a/b] e r = a – bq (aqui [x]
denota o único inteiro k tal que k  x < k + 1). Como conseqüência temos a

Proposição 0 (Divisão Euclidiana): Dados a  Z, b  Z* existem q, r 


Z unicamente determinados tais que 0  r < be a = bq + r 

Definição: Dados dois inteiros a e b , com a  0 dizemos que a divide b


(denotamos ab) se existe c inteiro tal que b = ac.

Proposição 1: Dados a, b  Z não ambos nulos existe d  N* tal que da,


db e, para todo c  N*, ca, cb  cd. Além disso existem x, y  Z
com d = ax + by. (Esse d é chamado o máximo divisor comum entre
a e b : d = mdc (a, b). )

Demonstração: Seja A = {k > 0 x, y  Z tais que k = ax + by} e seja


d = ax0 + by0 o menor elemento de A. Mostraremos que da. Como
d  N*, existem q, r  Z com a = dq + r e 0  r < d. Queremos mostrar
que r = 0. De fato, se r > 0, r = a – dq = a (1 – qx0) + b(– qy0)
 A, contradizendo o fato de d ser o menor elemento de A.
Portanto, r = 0 e
a = dq  da. Do mesmo modo prova-se que db. Suponha agora que
ca e cb. Então cax0 + by0 = d, como queríamos provar 

Lema: Se mdc (q, n) = 1 e nqk então nk.


Prova do Lema: Como mdc(q, n) = 1, existem x, y  Z com qx + ny = 1,
logo qkx + nky = k, portanto nk (pois qkx e nnky) 

EUREKA! N 2, 1998

41
Sociedade Brasileira de Matemática

Corolário: Sejam p um número primo e a, b  Z. Se pab então pa ou


pb 

Teorema fundamental da aritmética: Todo número natural n  2 possui


uma única fatoração (a menos da ordem dos fatores), como
produto de primos.

Demonstração: n = 2 é primo. Vamos mostrar a existencia da fatoração por


primos por indução: Se n é primo não há o que provar. Se n é
composto, n = ab, a, b  N, a < n, b < n e, por hipótese de
indução, a e b se decompõem como produto de primos,
portanto n se decompõe como produto de primos.
Vamos agora mostrar a unicidade, também por indução: Suponha
que n admita duas fatorações n = p1p2…pr e n = q1q2…qs como produto de
primos. O Corolário acima mostra que, como p1q1q2…qr, p1 deve dividir
algum qi e portanto p1 = qi (pois são ambos números primos) e, como
n/p1 = n/qi < n admite uma única fatoração prima, por hipótese de indução,
concluímos que a fatoração de n é única 
Proposição 2: O conjunto dos números primos é infinito.

Demonstração: Suponha que o conjunto dos números primos seja finito,


digamos { p1, p2,…, pn}. Nesse caso, o número N = p1p2…pn
+1 seria maior que todos os primos, mas não divisível por
nenhum deles, pois
pi( p1p2…pn + 1)  pi 1, absurdo. Teríamos então um natural N > 2 que
não seria múltiplo de nenhum primo, contradizendo o teorema
fundamental
da aritmética 
Obs.: As idéias desta seção podem ser utilizadas em situações mais gerais,
como no estudo de polinômios (por exemplo com coeficientes racionais),
onde existe um algoritmo de divisão, a partir do qual pode-se provar de
modo análogo resultados correspondentes aos aqui apresentados sobre
máximo divisor comum, existência e unicidade de fatoração.

EUREKA! N 2, 1998

42
Sociedade Brasileira de Matemática

Seção 2: Congruências
Definição: Sejam a, b, n  Z, n > 0. Dizemos que a é congruente a b
(módulo n) (denota-se a  b (módulo n)) se n(b – a)
Obs: a  a (módulo n), a  b (módulo n)  b  a (módulo n),
a  b (módulo n), b  c (módulo n)  a  c (módulo n), ou seja,
congruência (módulo n) é uma relação de equivalência.

Proposição: Se a  b (módulo n) e c  d (módulo n) então a + c  b + d


(módulo n) e ac  bd (módulo n).

Demonstração: n(b – a), n (d – c)  n  (b + d) – (a + c)  (a + c) 


(b + d) (módulo n), e bd – ac = b(d – c) + ((b – a)  n(bd – ac)  bd 
ac (módulo n) 

Definição: Dados n, a  Z n > 0, definimos a =a (módulo n) =


= {k  Zk  a (módulo n)}.
Dados a, b  Z definimos a +b = a  b ea b = ab ( estas operações
de soma e produto estão bem definidas pela proposição
anterior).
Definimos ainda Z/nZ = {a (módulo n), a  Z}={0, 1, 2,… n  1 }.
Cada a é chamada uma classe de congruência módulo n.

Definição: Sejam n, a  Z, n > 0. Dizemos que a é invertível módulo n se


existe b  Z com ab  1(módulo n) (ou seja, tal que a b =
1). Dizemos que b é o inverso de a em Z/nZ.

Definição: (Z/nZ)* ={a a  Z e a é invertível (módulo n)}. Obs. a é


invertível (módulo n)  mdc (a, n) =1. De fato, mdc (a, n) =
1   x, y  Z tais que ax + ny = 1a x = 1 (módulo
n).

Notação: Dado um conjunto finito X, escrevemos # X para significar o


número de elementos de X.

EUREKA! N 2, 1998

43
Sociedade Brasileira de Matemática

Definição: A função  de Euler, : N  N é definida por


(n) =  (Z/nZ)* =  {k  Z  0  k < n e mdc (k, n) = 1}.

Notemos que se p é um número primo e k  N então (pk) = pk – pk–1 =


pk (1–1/p). De fato, mdc (r, pk ) = 1 se e só se p não divide r. Logo (pk) =
{r  Z0  r < pk e mdc (r, pk ) =1} =  {r  Z 0  r < pk }

 {r  Z 0  r < pk e pr} = pk – pk – 1.

Definição: n números inteiros a1, a2,…an formam um sistema completo de


resíduos (s.c.r.) módulo n se {a1 , a2,…, an}= Z/nZ isto é,
se os a representam todas as classes de congruência módulo n
( por exemplo, 0,1,2,…n –1 formam um s.c.r. (módulo n)).
(n) números inteiros b1, b2,…b(n) formam um sistema completo de
invertíveis (s.c.i.) módulo n se {b1, b2,…b(n)} = (Z/nZ)*, isto é, se os bi
representam todas as classes de congruências invertíveis módulo n.

Proposição: Sejam q, r, n  Z, n > 0, q invertível módulo n, a1, a2,…,an um


s.c.r. (módulo n) e b1, b2,…,b(n) um s.c.i. (módulo n).
Então qa1 + r, qa2 + r,…, qan + r formam um s.c.r. (módulo n) e
qb1, qb2,…qb(n) formam um s.c.i. (módulo n).

Demonstração: Vamos provar que se a1, …an formam um s.c.r. (módulo n)


então qa1 + r, …qan + r formam um s.c.r. (módulo n). Basta
provar que
qai + r  qaj + r (módulo n)  i = j, pois nesse caso teremos n classes de
congruências distintas módulo n, que devem ser todas as
classes de Z/nZ.
Seja y  Z tal que qy  1 (módulo n).
Temos qai = qai + r – r  qaj + r – r = qaj (módulo n)  qyai  qyaj
(módulo n)  ai  aj (módulo n)  i = j.
Seja agora b1, b2,…b(n) um s.c.r. (módulo n). Temos que qbi é invertível
módulo n. para todo i,1  i  (n), pois se xi é tal que bi xi  1 (módulo n).
então (qbi) (xiy) = (qy) (bixi)  1 (módulo n). Por outro lado, se qbi  qbj

EUREKA! N 2, 1998

44
Sociedade Brasileira de Matemática

(módulo n) então bi  yqbi  yqbj  bj (módulo n)  i = j, e portanto qb1,


qb2,…qb(n) é um s.c.i. (módulo n) 

Teorema (Euler): Sejam a, n  Z, n > 0, tais que mdc (a, n) = 1. Então


a(n) 1 (módulo n).

Demonstração: Seja b1, b2,…b(n) um s.c.i. (módulo n) Pela proposição


anterior, (ab1), (ab2),…,(ab(n)) formam um s.c.i. (módulo n), e temos
{b1, b2,…,b(n)} = { ab 1 , ab 2 ,… ab (n)} = (Z/nZ)*b1 b2.…b(n)
= ab 1  ab 2 ….ab(n) =a(n) b1b2 …b(n)  b1 b2 …b(n) (a(n) –1)
= 0 a(n) =1 pois b1, b2,…b(n) são invertíveis (módulo n)  a(n)  1
(módulo n) 

Corolário: (Pequeno Teorema de Fermat): Se a  Z e p é primo então


ap  a (módulo p).
Prova : Se pa, então ap  a  0 (módulo p).
Se p não divide a, então mdc (a, p) =1  a p–1  1 (módulo p)  ap  a
(módulo p) 

Exercício resolvido: Exiba n  N tal que 2n tenha mais de duas mil casas
decimais e tenha entre suas 2000 últimas casas decimais 1000 zeros
consecutivos.
2000 2000
Solução: 2 ( 5 )
1 (módulo 5 ), pelo teorema de Euler. Portanto,

existe b  N com 2 ( 5 ) = 52000 b + 1, e teremos 2 2000 ( 5 ) = 102000 b


2000 2000

2000
+ 22000, e portanto os 2000 últimos dígitos de 2 2000 ( 5 ) coincidem com
a representação decimal de 22000, que tem no máximo 667 dígitos, pois
2000
23 < 10  22000 < 23.667 < 10667. Desta forma , 2 2000 ( 5 ) tem pelo menos
2000 – 667 = 1333 zeros consecutivos dentre as 2000 últimas casas
decimais, de modo que n = 451999 + 2000 satisfaz as condições do
enunciado (pois (52000) = 45 1999) 

EUREKA! N 2, 1998

45
Sociedade Brasileira de Matemática

Teorema Chinês dos restos: Se mdc (m, n) = 1 então


f: Z/mnZ Z/mZ  Z/nZ,
f (a (módulo n)) = (a (módulo m), a (módulo n))
é uma bijeção.

Demonstração: f está bem definida, pois se a = b (módulo mn) então a  b


(módulo m) e a  b (módulo n). Como Z/mnZ e Z/mZ 
Z/nZ têm mn elementos cada, é suficiente verificar que f é
injetiva. E, de fato, se a  b (módulo m) e a  b (módulo n)
então m  (b – a) e n(b – a)  b – a =
= mk, nmk  nk, pois mdc (m, n) = 1  mn(b – a)  a  b (módulo
mn) 

Corolário: Se m1, m2,…, mr  1 são inteiros, e mdc (mi, mj) = 1 para i  j


então f: Z/m1 m2,…mrZ  Z/m1Z  Z/m2Z … Z/mrZ,
f (a (módulo m1  m2 . …. mr)) = (a (módulo m1), … , a (módulo mr))
é uma bijeção 
Notemos que este Corolário mostra que, dados inteiros a1, a2,…ar, existe
um inteiro n com n = a1 (módulo m1), n  a2 (módulo m2), …, n  ar
(módulo mr).

Proposição: Temos f ((Z/mnZ)*) = (Z/mZ)*  (Z/nZ)* para a função f


definida acima.

Demonstração: Isto segue do fato de que a é primo com mn se e só se a é


primo com m e a é primo com n 

Corolário: mdc (m, n) = 1   (mn) =  (m)   (n) 


  
Como conseqüência, se n = p1 1 , p 2 2 ... p k k onde p1, p2,… pk são primos
distintos, 1, 2,… k  N* então (n) = n (1–1/p1) (1–1/p2)
…(1–1/pk).
Em particular, se n  3 então (n) é par 
Vamos mostrar um problema cuja solução usa de modo não trivial o
teorema chinês dos restos:

EUREKA! N 2, 1998

46
Sociedade Brasileira de Matemática

Problema: Prove que dado n  N existe um conjunto de n elementos


A  N tal que para todo B  A, B  ,  x é uma potência não trivial
xB
(isto é, um número da forma mk, onde m, k são inteiros maiores ou iguais a
2), ou seja, A = {x1, x2,… xn} tal que x1, x2,…xn, x1 + x2, x1 +x3,…, x n  1  x n
,
…,x1 + x2 +…xn são todos potências não triviais.

Solução: A = {4} é solução para n = 1, A = {9,16} é solução para n = 2.


Vamos provar a existencia de um tal conjunto por indução em n. Suponha
que A={x1,…, xn} é um conjunto com n elementos e para todo B  A, B 
kB
,  x m
xB
B Vamos mostrar que existe c  N tal que o conjunto
à ={cx1, cx2, …, cxn, c} satisfaz o enunciado.
Seja  = mmc {kB, B  A, B  } o mínimo múltiplo comum de todos os
exponentes kB.
Para cada B  A, B   associamos um número primo pB >  , de forma
que B1  B2  p B1  p B2 , e associamos um natural r com rB  0 (módulo
px ), X  B,  rB + 1  0 (módulo pB) (tal rB existe pelo teorema chinês
dos restos), e tomamos

c (1  mBkB ) rB


BA
B
Como c é uma potência  -ésima, c é uma potência kB-ésima para todo
B  A, B  , portanto, para B’ {cx1, cx2,…,cxn}, B’, teremos
B’= {cxx  B} para algum B  A, B  . Logo  x será uma
xB '
potência
kB-ésima.

Além disso,

EUREKA! N 2, 1998

47
Sociedade Brasileira de Matemática

 
 K X rX 
x c(1  m )  (1  m X ) (1  mBK B ) rB 1 ,
KB

X B 'U {c}
B
 X A 
 X ,B 
que é uma potência pB-ésima, pois rX é múltiplo de pB para X  B e  rB + 1
é múltiplo de pB 

Seção 3: Ordens e raízes primitivas.


Dados n  N* e a  Z com mcd (a, n) = 1, definimos a ordem de a
módulo n, ordn a: = min {t  N*at  1(módulo n)}. Dado a  (Z/nZ)*
definidos
orda = ord n a.

Proposição: {t  N*a t  1(módulo n)}={k.ord n a, k  N*}.

Demonstração: Como a ord n a 1 (módulo n), para todo k  N tem-se


a k .ord n a (a ord n a ) k 1k 1 (módulo n). Por outro lado, se t  n, at  1
(módulo n), existe k  N com
t k ord n a  r ,0 r  ord n a  a t a k .ord n a a r 1.a r a r (módulo n) 
a r  1 (módulo n), portanto r = 0 ( pois 0 < r < ord n a contradiria a
minimalidade de ord n a ), e t = k. ord n a 

Corolário: ord n a (n) 

Definição: Se ord n a = (n), dizemos que a é raiz primitiva módulo n.


Exemplos: 2 é raiz primitiva módulo 5, pois 2 1 = 2, 22 = 4, 23 = 8, 24 = 16,
que é a primeira potência de 2 congruente a 1 módulo 5 e 4 = (5).
 1 é raiz primitiva módulo 2, pois ord2 1 =1 = (2).
 3 é raiz primitiva módulo 4, pois ord4 3 = 2 = (4).

Proposição 3.1: a é raiz primitiva módulo n  {a t, t  N} = (Z/nZ)*.

Demonstração: Para todo a  Z com mdc (a, n) = 1 temos {a t, t  N} 


(Z/nZ)*. Se a é raiz primitiva módulo n então os números 1, a, a2,…a(n)–1

EUREKA! N 2, 1998

48
Sociedade Brasileira de Matemática

são distintos (módulo n) pois ai = aj (módulo n), com 0  i < j < (n)  aj–i
 1 (módulo n) com 0 < j – i < (n), absurdo  {at, t  N} = (Z/nZ)*.
Por outro lado, #{a t, t  N} ord n a (o argumento acima mostra que de
fato vale a igualdade), e portanto
{a t, t  N} = (Z/nZ)*  ord n a = (n) 

Corolário 1: Se m divide n e a é a raiz primitiva módulo n então a é raiz


primitiva módulo m 
Corolário 2: Se k  3, então não existe nenhuma raiz primitiva módulo 2k.

Prova: Pelo corolário anterior, basta provar que não existe raiz primitiva
módulo 8, e isso segue do fato que se a é ímpar,
a = 2r + 1, r  Z  a2 = 4r ( r + 1) + 1  1 (módulo 8) 

Proposição 3.2: Sejam p um número primo, e a  Z raiz primitiva módulo


p. Então a ou a + p é raiz primitiva módulo p2.

Demonstração: Por hipótese, ord p a = ordp(a+ p) = (p) = p – 1.


Portanto
p – 1 ord p a (pois at  1 (módulo p2)  at  1(módulo p)), e, como
2

ord p 2 a (p2) = p( p – 1), devemos ter ord p 2 a = p – 1 ou


ord p 2 a = p ( p – 1) = (p2). Do mesmo modo, ord p 2 (a + p) = p – 1 ou
ord p 2 (a + p) = p(p – 1) = (p2).

Basta provar, portanto, que ord p a  p – 1 ou ord p ( a + p)  p – 1.


2 2

Suponha que ord p a = p – 1. Portanto, a p–1  1 (módulo p2), e então


2

2
(a + p) p–1 = a p–1 + (p – 1) pa p–2 + C p  1 ap–3. p2 +…  1 + (p – 1) pa p–2
(módulo p2), portanto (a + p) p–1 não é congruente a 1(módulo p2), pois p2
não divide (p – 1) pa p –2, donde ord p (a + p)  p – 1 
2

Proposição 3.3: Se p é um número primo ímpar e a é raiz primitiva


módulo p2 então a é raiz primitiva módulo pk para todo k  N.

EUREKA! N 2, 1998

49
Sociedade Brasileira de Matemática

Demonstração: temos ap–1  1 (módulo p), mas ap–1 não é congruente a 1


(módulo p2), portanto ap–1 = 1 + b1 p, onde p não divide b1. Vamos mostrar
k1
por indução que a p ( p  1) 1  bk p k , onde p não divide bk, para todo
k1
k  1: Temos a p ( p  1) = ( a p ( p  1) ) p (1  bk p k  1 ) p 
k

1  bk p k 1  C p2 bk2 p 2 k  ... 1  bk p k 1 (módulo pk + 2). Logo


k
a p ( p  1) 1  bk p k 1 , com bk +1  bk (módulo p). Segue-se que p não
divide bk +1.
Vamos agora mostrar por indução que a é raiz primitiva módulo pk para
todo k  2. Suponha que a seja raiz primitiva módulo pk. Então temos
pk –1( p – 1) = (pk) = ord p a  ord p a (pk +1) = pk( p – 1). Portanto,
k k 1

ord p k 1 a = p k –1
(p – 1) ou ord p k 1 a = pk (p – 1) = (pk+1), mas o primeiro
k1
caso é impossível, pois a p ( p  1)
1  bk p k ,que não é congruente a 1
módulo pk+1, pois p não divide bk. Portanto ord p k 1 a = (pk+1) e a é raiz

primitiva módulo p k+1 


Exemplo: 2 é raiz primitiva módulo 5k, k  N. De fato, 2 é raiz primitiva
módulo 5, e, como 24 = 16  1 (módulo 25), 2 é raiz primitiva módulo
25 = 52 (como na proposição 3.2). Portanto, pela proposição 3.3, 2 é raiz
primitiva módulo 5k, k  N.

Exercício resolvido: Mostre que existe n natural tal que os mil últimos
dígitos de 2n pertencem a {1, 2}.

Solução: Observamos inicialmente que para todo k  N existe um número


mk de k algarismos, todos 1 ou 2, divisível por 2k.
De fato, m1 = 2 e m2 = 12 satisfazem o enunciado.
Seja mk = 2 k rk , rk  N. Se rk é par, tome mk+1 = 210k + mk =
2k+1 (5k + rk /2), e se rk é ímpar, tome mk+1 = 10k + mk=2k+1(5k + rk)/2.
Como m1000  2 (módulo 10), 5 não divide r1000 = m1000/21000. Como 2 é raiz
primitiva módulo 51000, existe k  N com 2k  r1000 (módulo 51000). Logo
2k = b  51000 + r1000, para algum b  N. Portanto, 2k+1000 = b  101000 +
21000  r1000 = b  101000 + m1000, e as 1000 últimas casas de 2k+1000 são as 1000
casas de m1000, que pertencem todas a {1, 2} 

EUREKA! N 2, 1998

50
Sociedade Brasileira de Matemática

Observação: Se p é primo ímpar, k  N e a é um inteiro ímpar tal que a é


raiz primitiva módulo pk então a é raiz primitiva módulo 2pk, pois (pk)=
ord p a  ord 2 p a (2pk) =  (pk)  ord 2 p a = (2pk). Isso implica
k k k

que se a é raiz primitiva módulo pk então a ou a + pk é raiz primitiva


módulo 2pk ( pois a e a + pk são raízes primitivas módulo pk e um deles é
ímpar.) 

Proposição 3.4: Se n = ab, com a  3 e b  3 inteiros tais que mdc(a, b) =


1, então não existe raiz primitiva módulo n.

Demonstração: Temos (n) = (a) (b) e, como a  3 e b  3 , (a) e


(b) são pares. Se mdc (k, n) = 1 então temos k(n)/2 = (k(b)/2)(a)  1 (módulo
a), e k(n)/2 = (k(a)/2)(b)  1 (módulo b). Assim, k(n)/2 = 1(módulo n), e
portanto
ord n k  (n)/2 < (n) 

Teorema: Existe alguma raiz primitiva módulo n se, e só se,


n = 2, n = 4, n = pk ou n = 2pk onde p é primo ímpar.

Prova: Pelos resultados anteriores, basta provar que se p é primo ímpar


então existe raiz primitiva módulo p, ou seja, existe a  (Z/pZ)* com
ordp a = p – 1.
Para cada a  (Z/pZ)*, tem-se ordp a( p – 1). Seja d um divisor de p – 1.
Definimos N(d) =  {a  (Z/pZ)*ordp a = d}.
Temos portanto p – 1 = 
d p 1
N(d). O resultado seguirá dos dois lemas
seguintes:

Lema 1: N(d)  (d) para todo d divisor de p – 1.

Prova: Se N(d) > 0 então existe a  (Z/pZ)* com ordp a. Se ordp a = d,


entãoad = 1 e, para 0  k < d, as classes de ak são todas distintas módulo
p, e (ak)d =1. Como a equação xd –1 = 0 tem no máximo d raízes
distintas em Z/pZ (pois Z/pZ é um corpo), suas raízes são exatamente ak,
0  k < d. Por outro lado, ordp ak = d  mcd(k, d) = 1, pois se r > 1 é tal
que rk e rd então (ak)d/r = (ad)k/r  1(módulo p), logo ord p(ak)  d/r <

EUREKA! N 2, 1998

51
Sociedade Brasileira de Matemática

d. Desta forma, b  (Z/pZ)* ord pb = d}  {ak, 0  k < d e mcd (k,d)


= 1}, portanto N(d)  (d) 
Lema 2: d n
(d) = n, para todo n  N.
Prova do Lema 2: Considere os n números racionais 1/n, 2/n, …,n/n. Ao
simplificá-los, aparecem exatamente (d) deles com denominador d, para
cada divisor d de n. Portanto, 
d n
(d) = n 
Fim da prova do teorema:
Do Lema 2 segue que 
d p 1
(d) = p – 1 e, como p – 1= d
p 1
N(d) e
N(d)  (d) para todo d, devemos ter N(d) = (d) para todo d. Em
particular, N(p – 1) = (p – 1) > 0  existem raízes primitivas módulo p 
PROBLEMAS

1) Prove que existem infinitos números primos congruentes a 3


módulo 4.
2) Determine todos os n naturais tais que (2n – 1)/n é inteiro.
3) Determine todos os n naturais tais que (2n + 1)/n2 é inteiro.
4) Prove que se a e b são naturais e (a2 + b2) / (ab + 1) é inteiro então
(a2 + b2) / (ab + 1) é quadrado perfeito.
5) Sejam a, n  N*. Considere a sequência (xn) definida por x1 = a,
xk+1 = a xk ,  k  N. Mostre que existe N  N tal que xk+1  xk
(módulo n), para todo k  N.

Obs.: Os problemas 3 e 4 foram propostos na 31 a. e na 29a. Olimpíada


Internacional de Matemática (1990 e 1988) respectivamente.
SOLUÇÕES DE PROBLEMAS PROPOSTOS EUREKA! N1

 Publicamos aqui algumas


das respostas enviadas por nossos leitores.

EUREKA! N 2, 1998

52
Sociedade Brasileira de Matemática

1) Mostre que, dado um conjunto de n pessoas, existem duas que


possuem o mesmo número de amigos entre as pessoas do conjunto.

SOLUÇÃO
Primeira Hipótese: há apenas uma única pessoa sem amigos; logo entre as
n – 1 pessoas restantes, cada pessoa é amiga de no mínimo uma pessoa e
no máximo n – 2 pessoas. Seja f : P  Q onde P = conjunto das pessoas
restantes e Q = conjunto dos possíveis números de amigos de uma
determinada pessoa em P, ou seja:
P = { p1, p2, …, pn–1}
Q = { 1, 2, 3, …, n – 2}
Observe que há n – 2 valores no conjunto Q para n – 1 valores em P ; isto
quer dizer que  n1, n2  P tais que f ( n1 ) = f ( n2 ).

Segunda Hipótese: Suponha que todas as n pessoas tenham amigos entre


si, ou seja:
P = { p1, p2, …, pn} e Q = { 1, 2, 3, …, n – 1}
Observe que agora o conjunto Q possui n – 1 valores, pois cada pessoa de
P possui no mínimo 1 amigo e no máximo (n – 1) amigos entre as (n – 1)
pessoas restantes. Pelo mesmo motivo da primeira hipótese  n1, n2  P
tais que f ( n1 ) = f ( n2 ).

Conclusão: há pelo menos duas pessoas com a mesma quantidade de


amigos.

2) Em uma pista circular há postos de gasolina, e o total de gasolina


que há nos postos é exatamente o suficiente para um carro dar uma
volta. Prove que existe um posto de onde um carro com o tanque
inicialmente vazio pode partir e conseguir dar uma volta completa
na pista (parando para reabastecer nos postos).

3) O Professor Carlos Alberto da Silva Victor observou que o problema


3 estava com o enunciado errado (de fato, n1998 é um quadrado
perfeito e portanto deve ser congruente a 0 ou a 1 módulo 4, não
podendo pois terminar por 11 na representação decimal.)
O enunciado correto é:
Prove que existe n  N tal que os 1000 primeiros dígitos de n1998
são iguais a 1.

EUREKA! N 2, 1998

53
Sociedade Brasileira de Matemática

4) Escreva 1998 como soma de (um número arbitrário de ) parcelas


de modo que o produto das parcelas seja o maior possível.

SOLUÇÃO
Observe inicialmente que, dado n  N,
n n
(i) se n (n > 4) é par, temos  n
2 2
 n  1  n 1
(ii) se n (n > 3) é ímpar, temos    n
 2   2 
Sejam 1998 = n1 + n2 + n3 + … nk e
P = n1  n 2  n 3  … n k
Com as observações (i) e (ii) devemos ter ni  { 1, 2, 3, 4} e como 4 = 2  2
podemos substituir 4 por "2 + 2" e teremos ni  { 1, 2, 3}; logo P = 1  2
 3 . É evidente que  = 0; pois se  = 1, "1 + 2" pode ser substituído por
um 3 e "1 + 3" pode ser substituído por "2 + 2". Também   2, pois
"2 + 2 + 2" pode ser substituído por "3 +3" ( 3  3 > 2  2  2) e
conseqüentemente P = 2  3 com ( = 1 ou 2 ). Como 1998 = 3  666 + 0,
P = 3666 e S = 3 33  ...  3
666 vezes

5) Sejam a  0 e P1P2P3P4P5 uma poligonal aberta contida em um dos


semiplanos determinados pela reta P1 P5 . Prove que existem
pontos P6 e P7 no plano, com P5 P6 = a, de modo que é possível
ladrilhar o plano com infinitos ladrilhos congruentes ao heptágono
P1P2P3P4P5P6P7.

6) Mostre que toda seqüência com n2 + 1 elementos possui uma


subseqüência crescente com n + 1 elementos ou uma subseqüência
decrescente com n +1 elementos.

7) Prove que 1 2 3  ...  1998 2

SOLUÇÃO
Definamos a função  : N – {0}  Z tal que
 (1) = 2
 (n + 1) =  (n)2 – n, n  1

EUREKA! N 2, 1998

54
Sociedade Brasileira de Matemática

Temos que 1<  (1) = 2


2<  (2) =  (1)2 – 1 = 22 – 1 = 3

Mostraremos que agora por indução que n <  (n) para todo n  3

  (3) =  (2)2 – 2 = 32 – 2 = 7. Logo, 3 <  (3)


 (Hipótese de indução) suponhamos que n <  (n)

como 0 < n <  (n), segue que n2 <  (n)2 isto é, n2 <  (n +1) + n. Dai,
n2 – n <  (n +1)
Mas n + 1 < n2 – n se e somente se 0 < n2 – 2n – 1 se e somente se
0 < n2 – 2n + 1 – 2 se e somente se 0 < (n – 1)2 – 2. Esta última
desigualdade é verdadeira se n  3
Portanto, se n  3, n +1 < n2 – n <  (n + 1) e dai n +1 <  (n + 1). Pelo
princípio de indução, segue que n <  (n) para todo n  3 como n  n
para todo n  N –{0} e daí, n   (n) para todo n  N –{0}
Portanto, 1998 <  (1998)
1998 <  (1997)2 – 1997
1997 + 1998 <  (1997)2
1997  1998 <  (1997) pois 0 < 1997 <  (1997).

Prosseguindo desta maneira, chegaremos a


1  2  3  ...  1998 <  (1) = 2

8) Considere um torneio de xadrez envolvendo brasileiros e


argentinos em que cada jogador joga contra todos os outros
exatamente uma vez. Ao final do torneio, cada jogador obteve
metade dos pontos que conquistou jogando contra brasileiros e
metade jogando contra argentinos. Prove que o número total de
jogadores do torneio é um quadrado perfeito (obs: cada vitória vale
1 ponto, empate 1/2 ponto e derrota 0 ponto).

SOLUÇÃO
Sejam k o número de brasileiros e n o número de argentinos no torneio.
Cada jogador brasileiro jogou k – 1 partidas contra brasileiros. Observe que

EUREKA! N 2, 1998

55
Sociedade Brasileira de Matemática

o número de vitórias, o número de empates e o número de derrotas (de


cada jogador brasileiro contra jogadores brasileiros) somadas deve ser
igual a k – 1.

(i) Seja s o número total de vitórias ocorridas entre brasileiros e E o


número de empates, logo:
2s + E = k ( k – 1); pois o número de vitórias é igual ao número de
derrotas.

(ii) Usando a mesma idéia do item (i) para os argentinos, temos:


2s' + E' = n ( n – 1); onde s' é o número total de vitorias entre
argentinos e E' o número total de empates entre argentinos.

E E'
(iii) Sejam P = s + e P' = s + , os totais de pontos obtidos
2 2
nos itens (i) e (ii) entre brasileiros e entre argentinos, respectivamente.

(iv) Suponha agora que os jogos entre brasileiros e argentinos; logo


cada brasileiro joga n partidas com os argentinos e cada argentino
jogou k partidas com os brasileiros.
Seja p o total de vitórias que os brasileiros obtiveram com os
argentinos e q o total de empates que os brasileiros obtiveram com
os argentinos, logo 2p + q = nk.
Como o total de pontos de cada brasileiro, metade foi contra
q
brasileiros e outra metade entre argentinos, temos P = p +
2
Sejam p' o total de vitórias que os argentinos obtiveram contra os
brasileiros e q' o total de empates que os argentinos obtiveram
contra os brasileiros, logo:
q'
2p' + q' = n k e também P' = p' + .
2
De (i), (ii), (iii) e (iv) temos:

EUREKA! N 2, 1998

56
Sociedade Brasileira de Matemática

 E q
 P  s   p   2s  E 2 p  q
2 2
 (v)
 P'  s ' E '  p' q'  2 s'E  2 p'q'
 2 2
Somando (v) teremos:

2
 sE  2
 s'E '  2 p  q  2 p 'q '
     
   
k(k–1) + n(n –1) = nk + nk

n + k = (n – k)2 ou seja o total de jogadores é um quadrado perfeito:

Nota: Para cada n, k com n + k = (n – k)2 é possível construir torneios com


k brasileiros e n argentinos satisfazendo as condições do enunciado. Note
t2 t t2  t
também que se n + k = t2 então n  e k .
2 2

9) Prove que todo número racional positivo pode ser escrito como
soma de um certo número de frações distintas de numerador 1.

SOLUÇÃO
p
(i) Seja inicialmente a fração 1, logo  n  N tal que
q
1 p 1 p 1 np  q
n

q

n 1
, observe que para n  2, temos: q

n

nq
.
np  q
Nós podemos repetir o processo inicial para a fração nq
até
encontrarmos a fração inicial como uma soma de frações com
numeradores iguais a 1; observe também que np – q < p, ou seja o
np  q
numerador da fração é menor do que o numerador da
nq
fração original e já que os numeradores dessas frações não podem
decrescer indefinidamente, este procedimento deverá terminar com

EUREKA! N 2, 1998

57
Sociedade Brasileira de Matemática

um número finito de frações com numeradores iguais a 1. Resta


então mostrar que essas frações são todas distintas; se não
vejamos:

np  q p 1 1 1 1 1
      (n  2);
nq q n n  1 n n(n  1) n

np  q
então quando nq
é escrita como uma soma de frações de
numeradores iguais a 1, todos os denominadores dessas frações são
maiores do que n, mostrando portanto que essas frações são todas
distintas.

p
(ii) Seja  1 , então  n  N tal que:
q
1 1 1 p 1 1 1
1   ...   1   ... 
2 3 n q 2 3 n 1

p 1 1 1 1  1
logo: 1    ,  ...   com   1,
q 2 3 4 n   n 1

usando o ítem (i) podemos expandir  como uma soma finita de
frações unitárias cujos denominadores são maiores que "n + 1".

Soluções dos problemas 1, 4, 8 e 9 enviadas por Carlos Alberto da Silva Victor, Nilópolis,
Rio de Janeiro-RJ. Solução do problema 7 enviada por Manuel João de Jesus Almeida,
Rio de Janeiro-RJ. Agradecemos também a participação de Carlos Eduardo Cardoso
Borges, Wayne L. Silva de Paula, Marco Rogério Vieira e Vicente Wilson Moura Gaeta.
Continuamos esperando as soluções dos problema 2, 3, 5 e 6.

PROBLEMAS PROPOSTOS
 Convidamos o leitor a enviar
soluções dos problemas propostos
e sugestões de novos
problemas para os próximos números.

EUREKA! N 2, 1998

58
Sociedade Brasileira de Matemática

10) Suponha que temos k moedas, todas iguais exceto por uma que tem
peso ligeiramente diferente das anteriores (não se sabe se maior ou
menor), e uma balança de dois pratos.

3n  3
a) Mostre que se k  é possível determinar com n pesagens
2
qual é a moeda diferente, e se ela é mais leve ou mais pesada que
as outras.
3n  1
b) Mostre que se k  é possível determinar com n pesagens
2
qual é a moeda diferente, mas nem sempre é possível dizer se ela é
mais leve ou mais pesada que as outras.

3n  1
c) Mostre que se k  não é sempre possível determinar qual é a
2
moeda diferente.

11) Determine todas as soluções de xy = yx com x e y racionais


positivos.

12) a) Prove que se n  N e 2n + 1 é um número primo então n é uma


potência de 2.
b) Prove que se a, n  N, n  2 e an –1 é primo, então a = 2 e n é
primo.

13) Dado n  N determine determine o maior k  N tal que existam


conjuntos A1, A2,…, Ak contidos em {1, 2, …, n} de forma que
Ai  Aj para todo i  j.

14) (Problema proposto por Antonio Luiz Santos): Determine o número


1 1 1
de soluções de   com x e y inteiros positivos.
x y 1998

15) Considere uma seqüência de triângulos retângulos AnBnCn no plano


cuja hipotenusa seja BnCn, com as seguintes condições:

i) A1 B1  A1C1 1
ii) Bn + 1 = Bn e An + 1 = Cn para todo n  N.

EUREKA! N 2, 1998

59
Sociedade Brasileira de Matemática

iii) An+1 Cn+1 é congruente à altura de An em relação a BnCn.

Mostre que qualquer ponto do plano pertence a infinitos triângulos


AnBnCn .

Você sabia…
Que há 6 poliedros regulares no espaço euclidiano
de 4 dimensões mas apenas 3 em 5 ou mais
dimensões ??
Em dimensão n há o simplexo, com n + 1 "faces"
(que são simplexos) de dimensão n – 1, o hipercubo,
com 2n "faces" ( que são hipercubos) de dimensão n
– 1 e o hiperoctaedro, dual do hipercubo, com 2n
"faces" (que são simplexos) de dimensão n – 1. Em
dimensão 4, além desses há o C24, que tem 24
"faces" octaédricas, o C120, que tem 120
"faces"dodecaédricas e o C600, que tem 600 "faces"
tetraédricas.
Lembre-se que em 3 dimensões há 5 poliedros
regulares: o tetraedro (caso particular do simplexo),
o cubo (caso particular do hipercubo), o octaedro
(caso particular do hiperoctaedro), o dodecaedro,
que tem 12 faces pentagonais, e o icosaedro, que
tem 20 faces triangulares.

AGENDA OLÍMPICA

OLIMPÍADA BRASILEIRA DE MATEMÁTICA

Primeira Fase – Sábado, 6 de junho


Segunda Fase – Sábado, 12 de setembro
Terceira Fase – Sábado, 24 de outubro (níveis 1, 2 e 3)
Domingo, 25 de outubro (nível 3).

EUREKA! N 2, 1998

60
Sociedade Brasileira de Matemática

OLIMPÍADA IBEROAMERICANA DE MATEMÁTICA

13 a 20 de setembro de 1998
República Dominicana.

Você sabia… Que é possível pentear


um toro (superfície em forma de pneu) cabeludo
mas não uma esfera cabeluda sem deixar
rodamoinhos??

COORDENADORES REGIONAIS

Alberto Hassen Raad (UFJF) Juiz de Fora-MG


Antônio C. Rodrigues Monteiro (UFPE) Recife-PE
Amarísio da Silva Araújo (UFV) Viçosa-MG
Angela Camargo (Centro de Educação
de Adultos CEA) Blumenau-SC
Antônio C. do Patrocínio (IMECC/UNICAMP) Campinas-SP
Ariosto de Oliveira Lima (UFPI) Parnaíba-PI
Benedito T. Vasconcelos Freire (UFRGDN) Natal-RN
Carlos A. Bandeira Braga (UFPB) João Pessoa-PB
Claudio Arconcher (Col. Leonardo da Vinci) Jundiaí-SP

EUREKA! N 2, 1998

61
Sociedade Brasileira de Matemática

Egnilson Miranda de Moura (Col. Agrícola do


Bom Jesus) Bom Jesus-PI
Élio Mega (Col. ETAPA) São Paulo-SP
Florêncio F. Guimarães F. (UFES) Vitória-ES
Francisco Dutenhefner (UFMG ) BH-MG
Gisele de A. Prateado G. (UFGO) Goiânia-GO
Ivanilde H. Fernandes Saad (U. Católica Dom Bosco) Campo Grande-MS
João B. de Melo Neto (UFPI) Teresina-PI
João F. Melo Libonati (Grupo Educ. IDEAL) Belém-PA
José Carlos Pinto Leivas (URG) Rio Grande-RS
José Luis Rosas Pinho (UFSC) Florianópolis-SC
José Paulo Carneiro (USU) Rio de Janeiro-RJ
José Vieira Alves (UFPB) Campina Grande-PB
Leonardo Matteo D'orio (Parque de Material
Aeronáutico de Belém) Belém-PA
Licio Hernandes Bezerra (UFSC) Florianópolis-SC
Luzinalva M. de Amorim (UFBA) L. de Freitas-BA
Marco Polo (Colégio Singular) Santo André-SP
Marcondes Cavalcante França (UF Ceará) Fortaleza-CE
Mario Jorge Dias Carneiro (UFMG) BH-MG
Ma-To-Fú (UEM) Maringá-PR
Pablo Rodrigo Ganassim (L. Albert Einstein) Rio das Pedras-SP
Paulo H. Cruz Neiva de L. Jr. (Esc. Tec.Everardo
Passos) Piracicaba-SP
Reinaldo Gen Ichiro Arakaki (INPE) S.J.Campos-SP
Ricardo Amorim (Centro Educ. Logos) Nova Iguaçu-RJ
Sergio Claudio Ramos (IM-UFRGS) Porto Alegre-RS
Tadeu Ferreira Gomes (U. do Estado da Bahia) Juazeiro-BA
Wagner Pereira Lopes (Esc. Tec. Fed. de Goiás) Jataí-GO

EUREKA! N 2, 1998

62
CONTEÚDO

AOS LEITORES 2

OLIMPÍADA BRASILEIRA DE MATEMÁTICA 3


Problemas de treinamento para a terceira fase

XIX OLIMPÍADA BRASILEIRA DE MATEMÁTICA (1997) 8


Problemas e soluções da segunda fase sênior

13a. OLIMPÍADA IBEROAMERICANA DE MATEMÁTICA 18


República Dominicana 1998 - Problemas e resultados

39a. OLIMPÍADA INTERNACIONAL DE MATEMÁTICA 20


Soluções

ARTIGOS

O PRINCÍPIO DA INDUÇÃO 26
Elon Lages Lima

FRAÇÕES CONTÍNUAS, REPRESENTAÇÕES DE NÚMEROS E APROXIMAÇÕES 44


Carlos Gustavo Moreira

SOLUÇÕES DE PROBLEMAS PROPOSTOS EUREKA Nos. 1 e 2 56

PROBLEMAS PROPOSTOS 61

COORDENADORES REGIONAIS 62
Sociedade Brasileira de Matemática

AOS LEITORES

1998 tem sido um bom ano para o programa brasileiro de


Olimpíadas de Matemática. Tivemos em torno de 40.000 participantes na
primeira fase, ganhamos mais uma medalha de ouro na Olimpíada
Internacional de Matemática e fomos o país com maior soma de pontos na
Olimpíada Iberoamericana de Matemática. Esperamos concluí-lo com uma
terceira fase da Olimpíada Brasileira de Matemática que faça jus aos
resultados até agora obtidos, estimulando ainda mais a imaginação criativa
dos jovens competidores, propiciando a descoberta de novos talentos para a
matemática e, em particular, criando as bases para as equipes brasileiras que
participarão nas olimpíadas internacionais do ano que vem. Esperamos que
os números da revista Eureka! que apresentamos este ano sejam úteis para
aumentar o número de participantes da Olimpíada Brasileira de Matemática
e que permitam a todos os classificados chegarem bem preparados à terceira
fase, além de contribuir para o enriquecimento da cultura matemática de
nossa comunidade acadêmica e escolar.

Esta Eureka! 3 está mais difícil que as anteriores, entre outras


razões, por ter boa parte de seu material dedicado à preparação para a
terceira fase do terceiro nível. Grande parte do material das Eureka! 1 e 2 é
adequada à preparação para a terceira fase dos primeiros dois níveis, mas no
terceiro nível a prova (como mostra a segunda fase sênior da Olimpíada
Brasileira de Matemática do ano passado, aqui resolvida) costuma ser mais
técnica, de modo que resolvemos usar a Eureka! 3 para oferecer aos
participantes da terceira fase uma preparação adequada, com problemas mais
difíceis e bem diferentes dos que usualmente se estudam nas escolas.

A terceira fase será realizada nas seguintes datas.

Sábado 24 de outubro 1o. nível


2o. nível
3o. nível (primeira prova).

Sábado 14 de novembro 3o. nível (segunda prova).

EUREKA! N 3, 1998

2
Sociedade Brasileira de Matemática

Comitê Editorial.
OLIMPÍADA BRASILEIRA DE MATEMÁTICA
Problemas de treinamento para a terceira fase

1) Sejam três pontos A, B e C pertencentes a uma circunferência de


 
centro O tais que AOB < BOC. Seja D o ponto médio do arco AC
que contém o ponto B. Seja K o pé da perpendicular a BC por D.
Prove que AB  BK KC .

2) Prove que existe uma seqüência a0, a1, …, ak, …, onde cada ai é um
algarismo (ou seja, ai  {0, 1, 2, 3, 4, 5, 6, 7, 8, 9}) com a0 = 6, tal
que para cada inteiro positivo n o número xn = a0 + 10a1 + 100a2 +
… + 10n–1 an–1 (cuja representação decimal é an–1 an–2 …a1a0) é tal que
x n2  x n é divisível por 10n.

3) Seja A = {x1 < x2 <…< xn} um conjunto de números inteiros


positivos tal que se x e y são dois números naturais que não
pertencem a A então x + y não pertence a A. Provar que xi  2i – 1
para i = 1, 2,…, n.

4) Considere a seqüência (xn) nN definida por x1 = 19, x2 = 98 e, para

 1
 xn  , se xn1 0
todo n  N, x n 2  xn1
 0, se x 0.
 n1
Prove que existe n  N tal que xn = 0 e encontre o menor n com
essa propriedade.

5) Sejam ABC um triângulo, M o pé da bissetriz interna do ângulo A e


N o pé da bissetriz interna do ângulo B. Suponha que MN seja
bissetriz do ângulo AMC. Calcule o ângulo A.

EUREKA! N 3, 1998

3
Sociedade Brasileira de Matemática

6) Ache todas as soluções reais de  x    1998 x  1998


(  y  denota o único inteiro tal que  y   y   y   1) .

7) Mostre que o produto de todos os números da forma


 1  2  3 ...  100 é o quadrado de um número inteiro.

Soluções

1)

Sejam AB = x, BD = y; marcamos D' tal que D'C = y. Então D'D = x por ser
D ponto médio de AC e resulta DD' // BC. Se K' é o pé da perpendicular a
BC por D', então temos
AB = DD' = KK' e BK = K'C
AB + BK = KK'+ K'C = KC.

2)
O primeiro termo é a0 = 6; então x1 = 6 e x12  x1 36  6 30 , que é
divisível por 101.
Seja n  1. Suponhamos que existem a0, a1,…,an–1 tais que
x n a 0  10a1  10 2 a 2  ...  10 n  1 a n  1
verifica que x n2  x n é divisível por 10n (ou seja
x n2 n
 x n 10 r , com r  N)
Temos que encontrar an tal que
x n 1 a 0  10a1  10 2 a 2  ...  10 n  1 a n  1  10 n a n x n  10 n a n

EUREKA! N 3, 1998

4
Sociedade Brasileira de Matemática

seja tal que x n21  x n 1 é divisível por 10 n 1.


x n21  x n 1 ( x n  10 n a n ) 2  ( x n  10 n a n )  x n2  2 x n 10 n a n  10 2 n a n2  x n  10 n a n 
( x n2  x n )  10 n (2 x n a n  a n )  10 2 n a n2 10 n r  10 n (2 x n a n  a n )  10 2 n a n2 
10 n (r  2 x n a n  a n )  10 2 n a n2 . Assim,
x n21  x n 1 e divisível por 10 n 1  r  2 x n a n  a n é divisível por 10 
r  (2 x n  1)a n é divisível por 10.
Dado que a0 = 6, temos que xn = 10t + 6 com t  N; então 2xn = 10h + 2.
(com h = 2t + 1). Logo r + (2xn – 1)an é divisível por 10  r + (10h + 1)an é
divisível por 10  r + an é divisível por 10.
Sempre existe um único inteiro an entre 0 e 9 de modo tal que isto se
verifique.

Obs: A seqüência (an) começa por 6, 7, 3, 9, 0, 1, 7, 8, 7, 1, 8, 0, 0, 4, 7, 3…


Assim, por exemplo, x10 = 1787109376.

Problema extra:
Prove que a seqüência (an) obtida não é periódica nem pré-periódica.

3)
Suponhamos que o enunciado é falso, ou seja que existe k tal que

xk > 2k – 1, 1  k  n.
Então os conjuntos

B1 = {1, xk – 1}, B2 = {2, xk – 2}, …, Bk = {k, xk – k}

são disjuntos dois a dois e seus elementos são menores que xk.
Além disso, para cada j, 1  j  k, j  A ou xk – j  A, pois no caso
contrario, ou seja, se j  A e xk – j  A, teríamos que xk = j + (xk – j)  A.
Portanto, para cada j, 1  j  k, A  Bj  , donde A tem pelo menos k
elementos menores que xk , absurdo.

4)
Se xn+1  0, temos xn+2 xn+1 = xn+1 xn –1. Definindo yn = xn xn+1 temos

EUREKA! N 3, 1998

5
Sociedade Brasileira de Matemática

yn+1 = yn–1 para todo n tal que xn+1  0. Como y 1 = x1 x2 = 19  98 = 1862,


temos yk = 1863 – k enquanto yk –1 for diferente de 0, e portanto y1862 = 1 e
y1863 = 0  x1862 x1863 = 1 e x1863 x1864 = 0. Assim, x1863  0 e x1864 = 0, donde
1864 é o menor n tal que xn = 0.

5)
AN c BM c ab
Pelo teorema das bissetrizes,  e   MC  , e como
NC a MC d bc
MA AN c
MN é bissetriz de ˆ C devemos
AM ter   , donde
MC CN a
bc ab
MA  (pois MC  pela lei dos senos aplicada aos triângulos
bc b 1
ac
senA BMa sen( A / 2)
ABC e ABM temos bc    , e portanto sen
senB b bc MA senB
bc
 A 1 A  2
(A/2) = sen A = 2 sen (A/2) cos (A/2)  cos       A .
2 2 2 3 3

6)
 x   
é sempre inteiro. Seja x0 a solução de x  1998 x 1998,
1998 x
ou seja x0 = 999 (3 – 5 )  763,1... e
1998x 0 1998  x 0 999( 5  1) 1234,8... Temos
x   
0
1998 x
0
 1997. A função f ( x )  x  
aumenta de uma 1998 x 
unidade quando x ou torna-se inteiro. Os próximos valores de x
1998 x
maiores que x0 para os quais x e 1998 x são inteiros são respectivamente
764 e 12352 / 1998 < 764.
Assim, f (12352 /1998) = 763 + 1235 = 1998 e f (764) = 764 + 1235 = 1999
(de fato 1998 764  1236). Como f (x) é não-decrescente, o conjunto das
soluções é o intervalo
1235 2 
   763,3758758758...,764 .
,764 
 1998 

EUREKA! N 3, 1998

6
Sociedade Brasileira de Matemática

7)
O número referido no enunciado é o quadrado do produto de todos os 2 99
números da forma 1  2  3 ...  100 (no produto do enunciado cada
um desses números aparece uma vez, assim como seu simétrico). Neste
último produto, obtemos uma soma de termos do tipo
( , a , )( a )...( a ), com m 2 99 , a , a ,...a  {2,3,...,100} e
1 2 2 m m 1 2 m

 j  {-1,1}, j.
Fixamos  2 ,  3 ,...,  100  N com  2   3  ...   100 2 99 , e
consideramos todos os termos como acima que têm exatamente  k valores
de a j k , para 2 k 100 . Se todos os  j são pares esses termos são
todos inteiros. Se algum deles (digamos  r ) é ímpar, podemos associar de
forma bijetiva a cada termo desses o termo obtido trocando os sinais de
todos os  j para os quais a j r. Assim, a cada termo associamos o seu
simétrico, e portanto, nesse caso a soma dos termos considerados é 0. Assim,
o produto de todos os números da forma 1  2  3 ...  100 é um
inteiro, e portanto o produto do enunciado é um quadrado perfeito.

Você sabia… que são conhecidos

51539600000 casas decimais de , calculadas


por Y. Kamada e D. Takahashi, da Universidade
de Tokio em 1997? E que em 21/8/1998 foi
calculada pelo projeto Pihex a 5000000000000 a.

casa binária de ?
Consulte a home-page
http://www.cecm.sfu.ca/pi

XIX OLIMPIADA BRASILEIRA DE MATEMÁTICA (1997)


Problemas e soluções da segunda fase sênior

EUREKA! N 3, 1998

7
Sociedade Brasileira de Matemática

PROBLEMA 1

Duas circunferências de raios R e r e centros O e O', respectivamente,


intersectam-se nos pontos P e P'. Seja l a reta que passa por P e P'.
Determine em função de R e r, o menor valor que pode assumir a soma das
distâncias de l a O e O'.

PROBLEMA 2

Dizemos que um conjunto A  N satisfaz a propriedade P(n) se A tem n


n ( n  1)
elementos e A + A = {x + y tal que x  A e y  A} tem elementos.
2
Dado A  N finito definimos o diâmetro de A como sendo a diferença entre
o maior e o menor elemento de A. Seja f (n) o menor diâmetro que um
2
n
conjunto A satisfazendo P(n) pode ter. Mostre que  f (n)  n
3
para todo n
4
 2.

(Se o seu tempo de prova não estiver esgotado, tente melhorar esta
estimativa. Por exemplo, tente mostrar que f (p) < 2p2, para todo número
primo p.)

PROBLEMA 3

a) Prove que não existem funções f : R  R e g : R  R


satisfazendo g ( f (x) ) = x3 e f ( g(x) ) = x2 para todo x  R.

b) Exiba funções f : ( 1,  )  ( 1,  ) e g :( 1,  )  (1,  ) tais que

g ( f(x) ) = x3 e f ( g(x) ) = x2 , para todo x  (1,  ).

PROBLEMA 4

EUREKA! N 3, 1998

8
Sociedade Brasileira de Matemática

Seja Fn definido por F1 = 1, F2 = 1 e Fn+2 = Fn+1 + Fn , para todo n  1. Seja


Vn  Fn2  Fn22 , n 1. Mostre que, para todo n inteiro positivo, Vn ,

Vn+1 e Vn+2 são lados de um triângulo de área 1/2.

PROBLEMA 5

Sejam c  Q, f (x) = x2 + c. Definimos f 0 (x) = x , f n+1 (x) = f ( f n (x)),


n  N . Dizemos que x  R é pré-periódico se { f n (x), n  N } é finito.
Mostre que { x  Q  x é pré-periódico } é finito.

PROBLEMA 6

Seja f uma função do plano no plano que satisfaz d (P,Q) = 1 


d (f (P), f(Q)) = 1 para todos os pontos P e Q do plano. Mostre que
d (f (P), f(Q)) = d (P, Q) para todos os pontos P e Q do plano.
(d (X,Y) denota a distância entre X e Y).

SOLUÇÕES

1)
Considere a circunferência de raio R fixa, cujo centro O está sobre uma reta
s. O problema se resume a determinar a posição de O' em s que minimiza a
soma d das distâncias de O e O' a  . Claramente,  é perpendicular a s.
Seja I o ponto de intersecção de s com  . Temos dois casos a considerar:

(i) OO'  OI. Neste caso, d = OO' e d é mínimo quando I = O'.


(ii) OO'  OI. Neste caso, considere O''  O' em s tal que O'I = IO''
( O'' é simétrico de O' em relação à  ). Assim, temos que
d = OO''  OI (primeiro caso) e d é mínimo quando O' = I = O''.
Em ambos casos, temos que d é mínimo quando O' = I . Por Pitágoras, este
mínimo é igual a R 2  r 2 .

2)
Dado um conjunto finito A  N , denotaremos por d(A) o diâmetro de A.
Temos duas desigualdades a provar:

EUREKA! N 3, 1998

9
Sociedade Brasileira de Matemática

n2
(i) f (n)  , para todo n  2.
4
Vamos supor, por absurdo, que exista um conjunto A = {a1, a2, …,an}, n  2,
n2
tal que A satisfaz P(n), a1 < a2 < …< an e d(A) < . Como A satisfaz P(n),
4
n( n  1)
A + A = {a1 + a2, a1 + a2, …,an + an} tem elementos. Como
2
a1 + a1 < a1 + a2 < … < an + an, temos que (an + an) – (a1 + a1) + 1 
n ( n  1) n2 n 2 n2 n2
 an – a1     d ( A)  , o que é uma
2 4 4 4 4
contradição. Isto demonstra (i).

(ii) f ( n)  n 3 , para todo n  2.


Como {0, 1} satisfaz P(2), temos que f(2)  1 < 23. Agora, vamos supor que
f ( n)  n 3 para algum n  2. Seja An = {a1, a2, …, an}  N tal que An
satizfaz P(n) e d(An) = f(n) < n3. Sem perda de generalidade, podemos supor
que 0 = a1 < a2 < … < an = d(An), bastando para isto subtrair de cada
elemento de An o menor de seus elementos. Agora, queremos achar
an + 1  N – An tal que An + 1 = {a1, a2, …, an + 1} satisfaça P(n +1) e
n ( n  1)
d (An + 1) < (n + 1)3. Como An + An tem elementos e
2
An 1  An 1 ( An  An )  {ai  a n 1 1 i n  1}, temos que
a n 1  N  An e An+1 P (n  1) se
satisfaz e somente se,
ai  a j
a n 1  P {ai  a j  a k 1 i, j , k n}  { 1 i, j n}. Como
2
n(n  1) n( n  1)
P n 3  , temos que a n 1 n 3  , pois basta escolher
2 2
an+1 como o menor natural que não está em P. Assim,
f ( n  1) d ( An 1 )  ( n  1) 3 . Por indução finita em n, temos que (ii) é
verdade, o que completa nossa demonstração.

Vamos ainda, verificar que, para p primo ímpar, f ( p )  2 p 2 . Para isto,


construímos o conjunto A {k  2 pg ( k ),0  k  p  1}, onde
g ( k ) k (mod p ), 0  g(k)  p – 1.
2

EUREKA! N 3, 1998

10
Sociedade Brasileira de Matemática

Temos d ( A)  p  1  2 p ( p  1) = 2 p 2  p  1  2 p 2 e se tivéssemos
i  2 pg (i )  j  2 pg (i )  r  2 pg ( r )  s  2 pg ( s ), então
i  j  2 p ( g (i )  g (i ))  r  s  2 p ( g (r )  g ( s ))  i  j 
 r  s, g (i )  g ( j )  g ( r )  g ( s )

Assim, i  r s  j e i 2  j 2 r 2  s 2 (mod p), logo

(i  r )(i  r )  ( s  j )( s  j )(mod p )  i  r  s  j  0(mod p ) ou


i  r  s  j (mod p ) . Portanto i r e s = j ou i = s e r = j.
Com um pouco de teoria dos Corpos, é possível provar, utilizando um
elegante argumento devido a Bose-Chowla, que, de fato, temos f ( p) < p2
para p primo. Seja K = Z/pZ o corpo com p elementos e L  K um corpo
com p2 elementos. Seja  um gerador do grupo (cíclico) multiplicativo de L,

ou seja, tal que { k  , k  Z} L  {0}. Para cada m  K,  + m  L – {0, 1}, e,


portanto, existe am  Z, 0 < am < p2 – 1 tal que  am   m. O conjunto
A {a m ,0 m  p  1} tem diâmetro no máximo p 2  3  p 2 e
a a  a  a  (  i )(  j ) (  r )(  s )  (i  j  r  s )  (ij  rs) 0.
i j r s

Como   K, temos i  j r  s e ij  rs  {i, j} {r , s}.

3)

a) Vamos supor, por absurdo, que existam funções f, g : R  R


satisfazendo, para todo x  R,

( I ) g ( f ( x ))  x 3 e
( II ) f ( g ( x ))  x 2

Agora x, y  R , f ( x)  f ( y )  g ( f ( x))  g ( f ( y ))  x 3  y 3  x  y. Logo f


é injetora. Ainda, de (I) e (II), temos
2 3 2 2
( f ( x)  f ( g ( f ( x ))  f ( x )  f (0)  f (0), f (  1)  f (  1) e f (1) 2  f (1),
logo { f (0), f (1), f (  1)}  {0,1} o que é um contradição (pois f é injetora e,

portanto,
{ f (0), f (1), f (–1)} tem 3 elementos).

EUREKA! N 3, 1998

11
Sociedade Brasileira de Matemática

(b) Vamos supor, por enquanto, que existam funções f , g : (1,)  (1,)
tais que g ( f ( x ))  x 3 e f ( g ( x))  x 2 , para todo x  (1, ). Agora,
considere as funções

x
( x) log 2 (log 2 g (2 2 ))
x
 ( x) log 2 (log 2 f (2 2 ))

Temos
x
log 2 (log2 f ( 2 2 )) x
  ( x) log 2 (log 2 g (2 2 )) log 2 (log 2 g ( f (2 2 ))) 
x
log 2 (log 2 (2 2 ) 3 ) log 2 3 2 x  x  log 2 3

x
log 2 (log2 g ( 2 2 )) x
  ( x) log 2 (log 2 f (2 2 )) log 2 (log 2 f ( g (2 2 ))) 
x
log 2 (log 2 (2 2 ) 2 ) log 2 2 x 1  x  1

Supondo que ( x ) ax  b e  ( x) cx  d , devemos ter, para todo


x  R,
   ( x) acx  ad  b  x  log 2 3
  ( x) acx  d  bc  x  1
Podemos escolher, por exemplo, a log 2 3, b 0, c log 3 2 e d = 1.
(ou seja,  ( x)  x log 2 3 e  ( x) x log 3 2  1
De (A), temos
x ( x)  (log2 (log 2 x )) log 2 3. log 2 log 2 x log 2 3
g (2 2 ) 2 2  g ( x) 2 2 2 2 2log 2 x

e de (B)
 ( x) 1log2 log2 x . log3 2
x  (log2 (log2 x )) log3 2
f (22 ) 2 2  f ( x) 2 2 22 22 log2 x
É fácil verificar que as funções acima estão definidas em (1, ) e
satisfazem as condições do enunciado. Elas fornecem, portanto, uma
possível solução para o item b).

EUREKA! N 3, 1998

12
Sociedade Brasileira de Matemática

4)
2 n 1
Primeiramente, notemos que, para n 0, Fn Fn 2  Fn 1 (  1) . De
2 2 2
fato, F1 F3  F2 1 2  1 (  1) e por indução supondo que
Fn Fn  2  Fn21 (  1) n 1 temos que
Fn 1 Fn 3  Fn2 2  Fn 1 ( Fn  2  Fn 1 )  Fn22  Fn21  Fn  2 ( Fn  2  Fn 1 ) 
 ( Fn Fn  2  Fn21 )  ( 1) n 1 ( 1) n  2 .
Dividimos o problema em dois casos; indicados pelas seguintes figuras:
(i)

Fn Vn

Vn+2
Fn+2

Fn+1
Vn+1

Fn+2 Fn+3
Fn+4

Se A é a área do triângulo sombreado, de lados Vn, Vn+1 e Vn+2, temos

1 Fn 2 Fn 4 1 Fn Fn 2 Fn 1 Fn 3
A     Fn 1 Fn 2  
2 2 2 2 2

Fn 2 Fn 4 1  Fn 2 ( Fn  2 Fn 1 )  Fn 1 Fn 3 1  Fn 2 ( Fn 2  Fn 1 )  Fn 1 Fn 3 

1  Fn 2 Fn 3  Fn 1 Fn 3 1  Fn23  Fn 2 Fn 4  Fn2 3 1, o que


ocorre sempre que n é ímpar.
(ii)

EUREKA! N 3, 1998

13
Sociedade Brasileira de Matemática

Vn
Fn

Vn+1
Fn+2
Vn+2
Fn+1

Fn+2 Fn+3
Fn+4

Se A é a área do triângulo sombreado, de lados Vn, Vn+1 e V n+2, temos


analogamente que

1 F F 1 FF F F
A   n 2 n 4   n n 2  Fn 1 Fn 2  n 1 n 3 
2 2 2 2 2
2
Fn 2 Fn 4  Fn 3  1, o que ocorre sempre que n é ímpar.
Em qualquer dos casos, temos que a área do triângulo de lados V n, Vn+1 e
1
Vn+2 é .
2

5)

Se x  c  1, então
2
x 2  x  x ( x  1)  c e, portanto,
 c  c  x 2  c x 2  c  x
f n 1
( x)  f n
( x)  c  1 para todo n  0. Logo, se x é pré-períodico,
então x  c  1 (*).
r p
Agora, sejam c  , onde ( r , s ) 1, e x 
q
, onde ( p , q ) 1,
s
com p, q, r , s  Z e q, s  0. Temos
sp 2
s( x 2  c)  r
q2
u
Se x 2  c  , u , v  Z, v 0, então
v
su sp 2 q2
 2  r  svp 2 q 2 ( su  rv)  q 2 svp 2  q 2 sv  sv q 2  v  .
v q s

EUREKA! N 3, 1998

14
Sociedade Brasileira de Matemática

q2
Se q > s, o denominador v de x 2  c é maior ou igual a  q, que é o
s
denominador de x isto é, o denominador de f n1 ( x ) é maior que o
denominador de f n ( x), n 0, e, portanto, se x é pré-periódico, então seu
denominador é no máximo s (**).
De (*) e (**), segue que há apenas um número finito de pontos pré-
periódicos racionais.

6)

Em primeiro lugar, observe que as imagens dos vértices de um triângulo


equilátero de lado 1 formam também um triângulo eqüilátero de lado 1.
Assim, dados dois triângulos eqüiláteros de lado 1 com um lado em comum,
os vértices opostos ao lado comum podem ter mesma imagem ou imagens
diferentes distando 3. Em outras palavras, se A e A' são pontos tais que
AA' = 3. então d ( f ( A), f ( A' ))  {0, 3}. Vamos mostrar que, de fato,
d ( f ( A), f ( A' ))  3. Se f ( A)  f ( A' ), então tomando B com AB = 1 e
A' B = 3 , teríamos d ( f ( A), f ( B )) 1  d ( f ( A' ), f ( B )) 1, o que seria
absurdo. Assim, d(A,A' ) = d ( f ( A), f ( A' ))  3  d ( f ( A), f ( A' ))  3.
Desta forma qualquer reticulado triangular formado por vértices de
triângulos eqüiláteros de lado 1 de interiores disjuntos e cobrindo o plano é
preservado por f, no seguinte sentido: a imagem deste reticulado também
será outro reticulado do mesmo tipo. Em particular, pontos a distância n são
levados em pontos também à distância n, n  N.
Este último fato mostra que triângulos de lados 1, n 2  n  1 e n 2  n  1
que têm área 3 / 4 são preservados pela função f , já que seus vértices
estão em reticulado triangular de lado 1.

B 1 C

1 1

A
n

EUREKA! N 3, 1998

15
Sociedade Brasileira de Matemática

AB  n 2  n  1

AC  n 2  n  1

Utilizando um procedimento análogo ao anterior, vamos agora considerar a


imagem dos vértices de dois triângulos deste tipo com o lado de medida
n 2  n  1 em comum. Sendo X e Y os vértices destes triângulos opostos ao
lado comum, temos novamente que XY  n  d ( f ( X ), f (Y )) 0 ou
d ( f ( X ), f (Y ))  XY  n , onde
3
n  2
n  n 1
é o dobro da altura dos triângulos considerados em relação ao lado comum.
Vamos demonstrar que os pontos à distância  n têm, de fato, imagens
distintas. Seja kn tal que k n  n  1 ( k n  1) n .
Sendo d ( A0 , A1 )  n , considere pontos Ai , 2 i k n  1 tais que
d ( Ai , Ai 1 )  n para 0 i k n e d ( A0 , Akn 1 ) 1 Temos
d ( f ( A0 , f ( Ak n 1 )) 1 e, portanto,
kn
1  d ( f ( Ai ), f ( Ai 1 )) d ( f ( A0 ), f ( A1 ))  k n  n ,
i 0
Se d ( f ( A0 ), f ( A1 )) fosse 0, então 1 k n  n  1, o que seria absurdo
assim, XY  n  d ( f ( X ), f (Y ))  n . Como antes, temos que XY =
k n  d ( f ( X ), f (Y )) k n para k  N.

Agora, suponha que existam X e Y tais que d ( f ( X ), f (Y ))  d ( X , Y ).


Sejam n  N tal que 4 n  d ( f ( X ), f (Y ))  d ( X , Y ) e P  R 2 com
d ( P, X )
 N, d ( P, Y )  2 n .
n
Tome Q  R 2 com d ( P, Q) d (Y , Q)  n  d ( f ( P ), f (Q)) 
d ( f (Y ), f (Q))  n  d ( f ( P ), f (Y )) 2 n
e como d ( P, X ) d ( f ( P), f ( X )), temos
d ( f ( X ), f (Y ))  d ( X , Y )  d ( f ( X ), f (Y ))  d ( f ( X ), f ( P)) 
d ( X , P)  d ( X , Y ) d ( f (Y ), f ( P ))  d ( P, Y )  4 n , absurdo.

EUREKA! N 3, 1998

16
Sociedade Brasileira de Matemática

Obs: As funções f : R2  R2 que satisfazem as condições do enunciado são


chamadas isometrias, e são composições de translações com rotações e / ou
reflexões.

Você sabia… Que o número de pessoas que


já apertaram a mão de outras pessoas um
número ímpar de vezes é par ??

13a. OLIMPÍADA IBEROAMERICANA DE MATEMÁTICA


República Dominicana 1998 - Problemas e resultados

EUREKA! N 3, 1998

17
Sociedade Brasileira de Matemática

Primeiro dia
Duração da Prova: 4 h e 30 minutos.

PROBLEMA 1

São dados 98 pontos sobre uma circunferência. Maria e José jogam


alternadamente da seguinte maneira: cada um deles traça um segmento
unindo dois dos pontos dados que não tenham sido unidos entre si
anteriormente. O jogo termina quando os 98 pontos tenham sido usados
como extremos de um segmento pelo menos uma vez. O vencedor é a pessoa
que faz o último traço. Se o José começa o jogo, quem pode garantir a sua
própria vitória?

PROBLEMA 2

A circunferência inscrita no triângulo ABC é tangente aos lados BC, CA e AB


nos pontos D, E e F, respectivamente. AD corta a circunferência num
segundo ponto Q. Demonstrar que a reta EQ passa pelo ponto médio de AF
se e somente se AC = BC .

PROBLEMA 3

Encontrar o menor número natural n com a seguinte propriedade: entre


quaisquer n números distintos do conjunto {1, 2, …, 999} pode-se escolher
quatro números diferentes a, b, c, d, tais que a + 2b + 3c = d.

Segundo dia
Duração da Prova: 4 h e 30 minutos.

PROBLEMA 4

Em volta de uma mesa redonda estão sentados representantes de n


países (n  2), satisfazendo a seguinte condição: se duas pessoas são do
mesmo país, então, seus respectivos vizinhos da direita não podem ser de um
mesmo país. Determinar, para cada n, o número máximo de pessoas que
pode haver em volta da mesa.
PROBLEMA 5

EUREKA! N 3, 1998

18
Sociedade Brasileira de Matemática

Encontrar o maior valor possível n para que existam pontos distintos P1, P2,
P3, … , Pn no plano, e números reais r1, r2, … , rn de modo que a distância
entre quaisquer dois pontos diferentes Pi e Pj seja ri  rj.
PROBLEMA 6

Seja  a raiz positiva da equação t2  1998t  1 = 0. Define-se a sucessão x0,


x1, x2, … , xn , … por:

 xo 1

 xn1  xn , para n 0,1,2,...
Encontrar o resto da divisão de x1998 por 1998.

Nota: [x] indica a parte inteira de x, ou seja, [x] é o único inteiro k tal que
k  x  k + 1.

RESULTADOS

A equipe Brasileira teve uma excelente participação na 13 a.


Olímpíada Iberoamericana de Matemática realizada em República
Dominicana de 18 a 27 de setembro na qual participaram 18 países.
Os países que obtiveram maior soma de pontos foram:

BRASIL 132 pontos


CHILE 127 pontos
ARGENTINA 120 pontos
PERU 117 pontos
MÉXICO 115 pontos
ESPANHA 112 pontos

O Resultado da Equipe Brasileira

BRA 1 Murali Srinivasam Vajapeyam Prata - 32 pontos


BRA 2 Emanuel Augusto de Souza Carneiro Ouro - 37 pontos
BRA 3 Fabricio Shigueu Catae Ouro - 35 pontos
BRA 4 Mauricio Pereira Carrari Bronze - 28 pontos

Cada um dos seis problemas da prova vale 7 pontos.

EUREKA! N 3, 1998

19
Sociedade Brasileira de Matemática

39a. OLIMPÍADA INTERNACIONAL DE MATEMÁTICA


Soluções

PROBLEMA 1
No quadrilátero convexo ABCD, as diagonais AC e BD são perpendiculares
e os lados opostos AB e DC não são paralelos. Sabemos que o ponto P,
onde se intersectam as mediatrizes de AB e DC, está no interior de ABCD.
Prove que ABCD é um quadrilátero inscritível se, e somente se, os triângulos
ABP e CDP têm áreas iguais.

SOLUÇÃO
Suponha primeiro que ABCD seja inscritivel. Como AC  BD temos
 
AB  CD  . Claramente o centro O do círculo circunscrito pertence às
mediatrizes de AB e DC, logo P = O, e como área de
1  1 
OAB  r 2 sen AB  r 2 sen CD  área de OCD (onde r é o raio do
2 2
círculo), vale a primeira implicação.
Suponha agora que ABCD não seja inscritível. Suponha sem perda de
generalidade que PC < PA. Seja Q o ponto de interseção de AC e BD.
Prolongamos QC e QD até intersectarmos o círculo de centro p e raio PA =
PB em novos pontos C’ e D’ . Como AC’ e BD’ são perpendiculares, pela
primeira implicação sabemos que área de PAB = área de PC’D’, mas
C’D’ > CD ( C’D’ é hipotenusa do triângulo retângulo QC’D’, de catetos
maiores que o triângulo retângulo QCD, do qual CD é hipotenusa), e
d(P, C’D’) > d (P, CD) (de fato, C’ e D’ estão no mesmo semiplano
determinado pela reta CD , distinto do semiplano ao qual pertence P,
e
d (P, C’D’) = d(P, M), onde M é o ponto médio de C’D’, e portanto pertence
ao mesmo semiplano que C’ e D’ , logo d(P, CD) < d(P, M) =
= d (P, C‘D’ )). Portanto área de PC’D’ > área de PCD, absurdo, pois
estamos supondo que área de PAB = área de PCD.

PROBLEMA 2
Numa competição, existem a concorrentes e b juízes, onde b  3 é um inteiro
ímpar. Cada juiz avalia cada um dos concorrentes, classificando-o como
"aprovado" ou "reprovado". Suponha que k é um número tal que as

EUREKA! N 3, 1998

20
Sociedade Brasileira de Matemática

classificações dadas por dois juízes quaisquer coincidem no máximo para k


k b 1
concorrentes. Prove que  .
a 2b

SOLUÇÃO
Para cada um dos candidatos, se j é o número de juizes que o aprovam, o
número de pares de juízes que tem julgamentos coincidentes em relação a
2 2 2 2 (b  1) 2
C
ele é j  C b j C b 1  C b 1  , de modo que o número total de
2 2
4
a (b  1) 2
pares de julgamentos coincidentes é no máximo , que, por outro
4
2 b(b  1)
lado, por hipótese, deve ser no máximo k C b k . Assim,
2
(b  1) a (b  1) 2 k b 1
devemos ter kb    .
2 4 a 2b

PROBLEMA 3
Para qualquer inteiro positivo n, seja d(n) o número de divisores positivos de
n (incluindo 1 e n).
d (n 2 )
Determine todos os inteiros positivos k tais que k para algum n.
d (n)

SOLUÇÃO
  
Obsevemos inicialmente que se n  p1 1 p 2 2 ... p k k ( pi primos distintos)
então d (n) (1   1 )(1   2 )...(1   k ).
2 (1  2 1 )(1  2 2 )...(1  2 k )
Assim, d ( n ) / d ( n)  . Como o numerador é
(1   1 )(1   2 )...(1   k )
ímpar, se o resultado for inteiro deve ser ímpar (e todos os i devem ser
pares).
Vamos mostrar que qualquer número natural ímpar é da forma desejada. Para
isso, devemos mostrar que todo número ímpar pode ser escrito como produto
2r  1
de frações da forma , r  N, não necessariamente distintas. Faremos
r 1
isso por indução. Seja m um número ímpar, e seja 2 s a maior potência de 2
que divide m + 1. Temos portanto m 2 s 1 q  2 s  1 para algum q  N,
donde

EUREKA! N 3, 1998

21
Sociedade Brasileira de Matemática

m( 2 s  1) 2 2 s (2q  1)  2 s 1 ( q  1)  1 2 2 s ( 2q  1)  2 s 1 ( q  1)  1
m   2s 1 
2s  1 2s  1 2 ( 2q  1)  2 s ( q  1)  1
2 2 s  1 (2q  1)  2 s (q  1)  1 2 2 s 1 (2q  1)  4( q  1)  1
  ( 2q  1).
2 2 s  2 (2q  1)  2 s  1 (q  1)  1 2 s (2q  1)  (2q  1)
Como 2q + 1 < 2s + 1 q + 2s – 1 = m, por hipótese de indução, 2q + 1 se
2r  1
escreve como produto de frações da forma , e portanto m também.
r 1

PROBLEMA 4
Determine todos os pares (a, b) de inteiros positivos tais que ab2 + b + 7
divide a2b + a + b.

SOLUÇÃO

a 2b  a  b
Se é inteiro então
ab 2  b  7
b(a 2 b  a  b)  a (ab 2  b  7) b 2  7a
 é inteiro. Como
ab 2  b  7 ab 2  b  7
b 2  7a b 2  7a
b 2  7 a  b 2  ab 2  b  7 temos que  1 . Se 0
ab 2  b  7 ab 2  b  7
teremos b2 = 7a, donde b é múltiplo de 7 (digamos
b = 7t ), e (7t)2 = 7a nos dá a = 7t2. É fácil ver que (a, b) = (7t2, 7t) satisfaz as
condições do enunciado para todo t inteiro positivo (temos nesse caso
a 2b  a  b
t ).
ab 2  b  7
b 2  7a 2 b 2  7a
Se  0 devemos ter b < 7a e  1 (pois é inteiro), e
ab 2  b  7 ab 2  b  7
portanto 7a  7a  b 2 ab2  b  7  7a  ab2  b2  7  b 1 ou b = 2.
b 2  7a 1  7a 57
Se b 1, 2   7  , e devemos ter que a + 8 divide
ab  b  7 a  8 a 8
57, com a inteiro positivo  a + 8 = 19 ou a + 8 = 57  a = 11 ou
a 2 b  a  b 133
a = 49. Para a = 11 e b = 1 temos   7, e para a = 49 e
ab 2  b  7 19

EUREKA! N 3, 1998

22
Sociedade Brasileira de Matemática

a 2 b  a  b 2451
b = 1 temos   43.
ab 2  b  7 57
b 2  7a 4  7a
Se b = 2, 2
 . Como 4 – 7a > – 18 – 8a = – 2 (4a + 9), se
ab  b  7 4a  9
4  7a
é inteiro negativo, devemos ter
4a  9
4  7a 13
  1  4  7 a   4a  9  a   N.
4a  9 3
Assim, as soluções são dadas por
( a, b) (7t ,7t ), t  N; ( a, b) ( 11,1 ) e (a, b) = (49,1).
2

PROBLEMA 5
Seja I o incentro do triângulo ABC. A circunferência inscrita no triângulo
ABC é tangente aos lados BC, CA e AB nos pontos K, L e M,
respectivamente. A reta que passa por B, paralela ao segmento MK,
intersecta as retas LM e LK nos pontos R e S, respectivamente. Prove que o
ângulo RIS é agudo.

SOLUÇÃO
__ __ __
Como BM  BK ˆ K temos BI  MK , e
e BI é bissetriz de MB
__ __
portanto BI  RS . Queremos mostrar que RIˆS é agudo, o que é
2 2 2
equivalente a RI  SI  RS , o que equivale a
2 2 2 2 2 2
BR  BI  BS  BI  ( BR  BS ) 2  BR  2 BR BS  BS , e
2
portanto devemos provar que BI  BR BS .

ˆ  BA
ˆ C , B  AB ˆ A, temos KBˆ S  MBˆ R 
ˆ C e C  BC   Bˆ
Se A ,
2
ˆ
  A   Cˆ   Cˆ
KSˆB  (e portanto SKˆ B  ) e MRˆ B  (e portanto
2 2 2
ˆ
ˆ R    A ). Assim, os triângulos MBR e SBK são semelhantes e
BM
2
BR BK
, donde (pois BI é
2 2
 BR BS  BM BK  BM  BI
BM BS
hipotenusa do triângulo retângulo BMI ).

EUREKA! N 3, 1998

23
Sociedade Brasileira de Matemática

PROBLEMA 6
Considere todas as funções f definidas no conjunto N dos inteiros positivos,
2 2
com valores no mesmo conjunto, que satisfazem f (t f ( s ))  s ( f (t )) ,
para todos s e t em N. Determine o menor valor possível de f(1998)

SOLUÇÃO
Dizemos que h : N  N é estritamente multiplicativa se h(xy) = h(x) h(y),
para quaisquer x, y  N, e dizemos que h é uma involução se h(h(x)) = x para
todo x  N. É facil ver que se f satisfaz a involução estritamente
multiplicativa então f satisfaz a condição do enunciado: f (t2 f (s)) = (f (t)2
f (f (s)) = s (f (t))2. Podemos definir f : N  N estritamente multiplicativa por
f ( p11 p 2 2 ... p k k )  f ( p1 )1 ... f ( p k ) k ( pi primos distintos), onde f (2) =
3,
f (3) = 2, f (37) = 5, f (5) = 37 e f (p) = p, para todo p primo não pertencente a
{2, 3, 5, 37}, e teremos f (1998) = f ( 2  33  37 ) = f (2) f (3)3 f (37) = 3  23 
5 = 120. Vamos provar que 120 é menor valor possível para f (1998).

Fazendo t = 1 temos f (f (s )) = s f (1)2,  s  N. Em particular, f é injetiva,


pois f (s) = f (u)  f (f (s)) = f ( f (u))  s f (1)2 = u f (1)2  s = u.
Temos ainda f (t 2 f (1)) = f (t)2 para todo t  N ( fazendo s = 1), e portanto
temos f (t2 f (s) 2) = f (t2 f (s2 f (1))) = s2 f (1) f (t)2, e fazendo s = f (u) temos
f (t2 ( f ( f (u))2)= f (u)2 f (1) f (t)2. Assim, provamos que f (t 2 u2 f (1)4) =
= ( f (u) f (t))2 f (1), para quaisquer u, t  N.
Portanto, se ut = xy, f (t 2 u2 f (1)4) = f (x2 y2 f (1)4), logo ( f (u) f (t))2 f (1) =
( f (x) f (y))2 f (1)  f (u) f (t) = f (x) f (y). Como x2  1 = x  x, f (x2) f (1) =
f (x)2, x  N. Se pk é uma potência de primo que divide f (1), e p r é a maior
potência de p que divide f (x) para todo x  N, temos que f (x)2 é múltiplo de
 r k 
pr  pk  f (x) é múltiplo de p  2  onde    denota o menor inteiro que é
maior ou igual a , para todo x  N, o que é absurdo se r < k (pois teríamos
r  k 
 2   r ). Logo p divide f (x) para todo x  N, e portanto f (1) divide
k

EUREKA! N 3, 1998

24
Sociedade Brasileira de Matemática

f (x), para todo x  N. Como xy  1 = x  y , f (xy) f (1) = f (x) f (y) 


f ( xy) f ( x) f ( y ) f ( x)
  . Definindo g : N  N, g (x) = temos que g é
f (1) f (1) f (1) f (1)
estritamente multiplicativa, g é injetiva, g (1) = 1 e g (x)  f (x) para todo
x  N. Temos g(1998) = g(2  33  37) = g(2) g(3)3 g(37). Observemos agora
que g(2), g(3) e g(37) devem ser naturais distintos maiores que 1, e não
podemos ter {2, 4}  {g(2), g(3), g(37)}, pois se g( p) = 2 e g(q) = 4
com
{p, q}  {2, 3, 37} teríamos g(p2) = g(p)2 = g(q)  p2 = q, absurdo. Assim
g(1998) = g(2) g(3)3 g(37) = g(2) g(3) g(37) g(3)2  2  3  5  g(3)2 
2  3  5  22 = 120, logo f (1998)  120, como afirmamos 

Você sabia… Que o matemático Harald


Bohr (irmão do físico Niels Bohr), famoso por sua
teoria das funções quase-periódicas, era um
consagrado jogador de futebol? Ele jogou no
meio-de-campo da seleção da Dinamarca, que
ganhou a medalha de prata nos jogos Olímpicos
de Londres, em 1908, quando seu time derrotou
a França pela contagem de 17 a 1 (!).

O PRINCÍPIO DA INDUÇÃO
Elon Lages Lima
 Nível Avançado.

INTRODUÇÃO

EUREKA! N 3, 1998

25
Sociedade Brasileira de Matemática

O Princípio da Indução é um eficiente instrumento para a


demonstração de fatos referentes aos números naturais. Por isso deve-se
adquirir prática em sua utilização. Por outro lado, é importante também
conhecer seu significado e sua posição dentro do arcabouço da Matemática.
Entender o Princípio da Indução é praticamente o mesmo que entender os
números naturais.
Apresentamos abaixo uma breve exposição sobre os números
naturais, onde o Princípio da Indução se insere adequadamente e mostra sua
força teórica antes de ser utilizado na lista de exercícios propostos ao final.

1. A SEQÜÊNCIA DOS NÚMEROS NATURAIS

Os números naturais constituem um modelo matemático, uma escala


padrão, que nos permite a operação de contagem. A seqüência desses
números é uma livre e antiga criação do espírito humano. Comparar
conjuntos de objetos com essa escala abstrata ideal é o processo que torna
mais precisa a noção de quantidade; esse processo (a contagem) pressupõe
portanto o conhecimento da seqüência numérica. Sabemos que os números
naturais são 1, 2, 3, 4, 5,… A totalidade desses números constitui um
conjunto, que indicaremos com o símbolo N e que chamaremos de conjunto
dos naturais. Portanto N = {1, 2, 3, 4, 5,…}.
Evidentemente, o que acabamos de dizer só faz sentido quando já se
sabe o que é um número natural. Façamos de conta que esse conceito nos é
desconhecido e procuremos investigar o que há de essencial na seqüência 1,
2, 3, 4, 5… .
Deve-se a Giussepe Peano (1858-1932) a constatação de que se
pode elaborar toda a teoria dos números naturais a partir de quatro fatos
básicos, conhecidos atualmente como os axiomas de Peano. Noutras
palavras, o conjunto N dos números naturais possui quatro propriedades
fundamentais, das quais resultam, como conseqüências lógicas, todas as
afirmações verdadeiras que se podem fazer sobre esses números.
Começaremos com o enunciado e a apreciação do significado dessas quatro
proposições fundamentais a respeito dos números naturais.
2. OS AXIOMAS DE PEANO

Um matemático profissional, em sua linguagem direta e objetiva,


diria que o conjunto N dos números naturais é caracterizado pelas seguintes
propriedades:

EUREKA! N 3, 1998

26
Sociedade Brasileira de Matemática

A. Existe uma função s : N  N, que associa a cada n  N um


elemento s(n)  N, chamado o sucessor de n.
B. A função s : N  N é injetiva.
C. Existe um único elemento 1 no conjunto N, tal que 1  s(n) para
todo n  N.
D. Se um subconjunto X  N é tal que 1  N e s(X)  X
(isto é, n  X  s(n)  X), então X = N.

Observe que, como estamos chamando de N o conjunto dos números


naturais, a notação n  N significa que n é um número natural.
As afirmações A, B, C e D são os axiomas de Peano. A notação s(n) é
provisória. Depois de definirmos adição, escreveremos n + 1 em vez de s(n).

Como concessão à fraqueza humana, nosso matemático nos faria a


gentileza de reformular os axiomas de Peano em linguagem corrente, livre
de notação matemática. E nos diria então que as afirmações acima
significam exatamente o mesmo que estas outras:

A'. Todo número natural possui um único sucessor, que também é um


número natural.
B'. Números naturais diferentes possuem sucessores diferentes. (Ou
ainda: números que têm o mesmo sucessor são iguais.)
C'. Existe um único número natural que não é sucessor de nenhum
outro. Este número é representado pelo símbolo 1 e chamado de
"número um".
D'. Se um conjunto de números naturais contém o número 1 e, além
disso, contém o sucessor de cada um de seus elementos, então esse
conjunto coincide com N, isto é, contém todos os números naturais.

A partir daí, retomamos a palavra para dizer que o sucessor de 1


chama-se "dois", o sucessor de dois chama-se "três", etc. Nossa civilização
progrediu ao ponto em que temos um sistema de numeração, o qual nos
permite representar, mediante o uso apropriado dos símbolos 0, 1, 2, 3, 4, 5,
6, 7, 8 e 9, todos os números naturais. Além disso, nossa linguagem também
fornece nomes para os primeiros termos da seqüência dos números naturais.
(Números muito grandes não têm nomes específicos, ao contrário dos
menores como "mil novecentos e noventa e oito". Quem sabe, por exemplo,
o nome do número de átomos do universo?)

EUREKA! N 3, 1998

27
Sociedade Brasileira de Matemática

Voltando a usar a notação s(n) para o sucessor do número natural n,


teremos então 2 = s(1), 3 = s(2), 4 = s(3), 5 = s(4), etc. Assim, por exemplo,
a igualdade 2 = s(1) significa apenas que estamos usando o símbolo 2 para
representar o sucessor de 1. A seqüência dos números naturais pode ser
indicada assim:
1 s 2 s 3 s 4 s 5 s 
As flechas ligam cada número ao seu sucessor.
Nenhuma flecha aponta para 1, pois este número não é sucessor de
nenhum outro. O diagrama acima diz muito sobre a estrutura do conjunto N
dos números naturais.

3. O AXIOMA DA INDUÇÃO

Um dos axiomas de Peano, o último, possui claramente uma


natureza mais elaborada do que os demais. Ele é conhecido como o axioma
da indução. Faremos dele uma análise detida, acompanhada de comentários.
O significado informal do axioma D é que todo número natural pode
ser obtido a partir de 1 por meio de repetidas aplicações da operação de
tomar o sucessor. Assim, por exemplo, 2 é o sucessor de 1, 3 é o sucessor do
sucessor de 1, etc. Para se entender melhor o axioma da indução é util
examinar o exemplo, no qual N = {1, 2, 3,…} mas a função s : N  N é
modificada, pondo-se s(n) = n + 2. Então, se começarmos com 1 e a este
número aplicarmos repetidamente a operação de tomar o "sucessor" (nesta
nova acepção) obteremos s(1) = 3, s(3) = 5, s(5) = 7, etc., e nunca
chegaremos a qualquer número par. Portanto, o diagrama

1 s 3 s 5 s 2 s 4 s 6 s 


exibe uma função injetiva s : N  N para a qual não é verdade que todo
número natural n pode ser obtido, a partir de 1, mediante repetidas
aplicações da operação de passar de k para s(k).

Dentro de um ponto de vista estritamente matemático, podemos


reformular o axioma da indução do seguinte modo: Um subconjunto X  N
chama-se indutivo quando s(X)  X, ou seja, quando n  X  s(n)  X, ou
ainda, quando o sucessor de qualquer elemento de X também pertence a X.

EUREKA! N 3, 1998

28
Sociedade Brasileira de Matemática

Dito isto, o axioma da indução afirma que o único subconjunto indutivo de


N que contém o número 1 é o proprio N.
No exemplo acima, os números ímpares 1, 3, 5, … formam um
conjunto indutivo que contém o elemento 1 mas não é igual a N.
O papel fundamental do axioma da indução na teoria dos números
naturais e, mais geralmente, em toda a Matemática, resulta do fato de que ele
pode ser visto como um método de demonstração, chamado o Método de
Indução Matemática, ou Princípio da Indução Finita, ou Princípio da
Indução, conforme explicaremos agora.
Seja P uma propriedade que se refere a números naturais. Um dado
número natural pode gozar ou não da propriedade P.
Por exemplo, seja P a propriedade de um número natural n ser
sucessor de outro número natural. Então 1 não goza da propriedade P, mas
todos os demais números gozam de P.

O Princípio da Indução diz o seguinte:

Princípio da Indução: Seja P uma propriedade referente a números


naturais. Se 1 goza de P e se, além disso, o fato de o número natural n gozar
de P implica que seu sucessor s(n) também goza, então todos os números
naturais gozam da propriedade P.
Para ver que o Princípio da Indução é verdadeiro (uma vez
admitidos os axiomas de Peano) basta observar que, dada a propriedade P
cumprindo as condições estipuladas no enunciado do Princípio, o conjunto X
dos números naturais que gozam da propriedade P contém o número 1 e é
indutivo. Logo X = N, isto é, todo número natural goza da propriedade P. As
propriedades básicas dos números naturais são demonstradas por indução.
Comecemos com um exemplo bem simples.

Exemplo 1. Entre os axiomas de Peano não consta explicitamente a


afirmação de que todo número é diferente do seu sucessor, a qual
provaremos agora. Seja P esta propriedade. Mais precisamente, dado o
número natural n, escrevamos P(n) para significar, abreviadamente, a
afirmação n  s(n). Então P(1) é verdadeira, pois 1  s(1), já que 1 não é
sucessor de número algum; em particular, 1 não é sucessor de si próprio.
Além disso, se supusermos P(n) verdadeira, isto é, se admitimos que
n  s(n), então s(n)  s(s(n)), pois a função s : N  N é injetiva. Mas a
afirmação s(n)  s(s(n) significa que P(s(n)) é verdadeira. Assim, a
verdade de P(n) acarreta a verdade de P(s(n)). Pelo Princípio da

EUREKA! N 3, 1998

29
Sociedade Brasileira de Matemática

Indução, todos os números naturais gozam da propriedade P, ou seja, são


diferentes de seus sucessores.

Nas demonstrações por indução, a hipótese de que a propriedade P é


válida para o número natural n (da qual deve decorrer que P vale também
para s(n)) chama-se hipótese de indução.
O Princípio da Indução não é utilizado somente como método de
demonstração. Ele serve também para definir funções f: N  Y que têm
como dominio o conjunto N dos números naturais.
Para se definir uma função f : X  Y exige-se em geral que seja dada
uma regra bem determinada, a qual mostre como se deve associar a cada
elemento x  X um único elemento y = f(x)  Y.
Entretanto, no caso particular em que o domínio da função é o
conjunto N dos números naturais, a fim de definir uma função f : N  Y não
é necessário dizer, de uma só vez, qual é a receita que dá o valor f(n) para
todo n  N. Basta que se tenha conhecimento dos seguintes dados:
(1) O valor f (1);
(2) Uma regra que permita calcular f (s(n)) quando se conhece f (n).
Esses dois dados permitem que se conheça f (n) para todo número
natural n. (Diz-se então que a função f foi definida por recorrência.) Com
efeito, se chamarmos de X o conjunto dos números naturais n para os quais
se pode determinar f (n), o dado (1) acima diz que 1  X e o dado (2)
assegura que n  X  s(n)  X. Logo, pelo axioma da indução, tem-se
X = N.
Obs. : Uma função f : N  Y cujo domínio é o conjunto dos números
naturais chama-se uma seqüência ou sucessão de elementos de Y. A notação
usada para uma tal seqüência é (y1, y2,…,yn,…), onde se usa yn em vez de f(n)
para indicar o valor da função f no número n. O elemento yn .

4. ADIÇÃO E MULTIPLICAÇÃO DE NÚMEROS NATURAIS

A adição e a multiplicação de números naturais são exemplos de


funções definidas por recorrência.
Para definir a adição, fixaremos um número natural arbitrário k e
definiremos a soma k + n para todo n  N.
Fixado k, a correspondência n  k + n será uma função f: N N,
f(n) = k + n, chamada "somar k". Ela se define por recorrência, a partir dos
seguintes dados:
(S1) k + 1 = s(k)

EUREKA! N 3, 1998

30
Sociedade Brasileira de Matemática

(S2) k + s(n) = s(k + n).


Portanto, k + 1 é, por definição, o sucessor de k. E, se conhecermos k + n,
saberemos o valor de k + s(n): por definição, tem-se k + s(n) = s(k + n). Isto
nos permite conhecer k + n para todo n  N (e todo k  N).
Usando as notações definitivas n + 1 em vez de s(n) e (k + n) + 1 em
vez de s(k + n), a igualdade (S2) se escreve assim:
(S2') k + (n + 1) = (k + n) +1.
Assim, as igualdades (S1) e (S2) ou, equivalentemente, (S1) e (S2')
definem por recorrência a soma k + n de dois números naturais quaisquer k e n.
A multiplicação de números naturais se define de modo análogo à
adição. Fixado arbitrariamente um número natural k, a multiplicação por k
associa a todo número mnatural n o produto n  k, definido por indução da
seguinte maneira:
(P1) 1 k = k.
(P2) (n + 1) k = nk + k.
O produto nk escreve-se também nk e lê-se "n vezes k". A definição acima
diz portanto que uma vez k é igual a k e n + 1 vezes k é igual a n vezes k
mais (uma vez) k . Assim, por definição, 2  k = k + k, 3  k = k + k + k, etc.
Usa-se indução para provar as propriedades básicas da adição e da
multiplicação de números naturais. Entre elas, destacam-se as seguintes,
válidas para quaisquer k, n, p  N:
Associatividade: k + (n + p) = (k + n) + p e k  (n  p) = (k  n) p
Comutatividade: k+n=n+k e kn=nk
Lei do Corte: k+n=k+pn=p e kn=kpn=p
Distributividade: k (n + p) = k  n + k  p.
Omitiremos as demonstrações destes fatos. O leitor pode considerá-
las como exercícios sobre o método da indução.

5. ORDEM

A adição de números naturais permite introduzir uma relação de


ordem em N. Dados os números naturais m, n diremos que m é menor do que
n, e escreveremos m < n, para significar que existe p  N tal que n = m + p.
Neste caso, diz-se também que n é maior do que m e escreve-se n > m para
exprimir que se tem m < n. A notação m  n significa que m < n ou m = n.
Por definição, tem-se portanto m < m + p para quaisquer m, p  N. Em

EUREKA! N 3, 1998

31
Sociedade Brasileira de Matemática

particular, m < m + 1. Segue-se também da definição que 1 < n para todo


número natural n  1.
Com efeito, pelo axioma C, n  1 implica que n é sucessor de algum
número natural m, ou seja, n = m + 1 = 1 + m, logo n > 1. Assim, 1 é o
menor dos números naturais.
Provaremos a seguir as propriedades básicas da relação de ordem
m < n que definimos. A primeira delas é a transitividade.

Teorema 1. (Transitividade.) Se m < n e n < p, então m < p.


Demonstração: Se m < n, n < p então n = m + k, p = n + r, logo p = (m +
k) + r = m + (k + r), portanto m < p.
Outra importante propriedade de relação de ordem é que, dados dois
números naturais diferentes m, n, ou se tem m < n ou então n < m. Esta
propriedade pode ser reformulada de outra maneira, como segue.
Diremos que os números naturais m, n são comparáveis quando se
tem m = n, m < n ou n < m. Podemos então enunciar o seguinte teorema.

Teorema 2. (Comparabilidade.) Todo número natural n é comparável com


qualquer número natural m.
Demonstração: Isto se prova por indução. O número 1 é comparável com
qualquer outro número natural pois já sabemos que 1 < m para todo m  1.
Suponhamos agora que o número n seja comparável com todos os números
naturais. Mostremos, a partir daí, que n + 1 também tem essa propriedade.
Com efeito, seja m  N tomado arbitrariamente. Sabemos que se tem
m < n, m = n ou n < m. Examinemos cada uma dessas possibilidades:
Se for m < n então m < n + 1 por transitividade, pois sabemos que n < n + 1.
Se for m = n, então m < n + 1.
Se for n < m então m = n + p. Neste caso, há duas possibilidades. Ou se tem
p = 1, donde m = n + 1, ou então p > 1, logo p = 1 + p', e daí m = (n + 1) +
p' e concluímos que n + 1 < m. Em qualquer hipótese, vemos que n + 1 é
comparável com qualquer número natural m. Por indução, fica provada a
comparabilidade de quaisquer números naturais m, n.
A comparabilidade dos números naturais é complementada pela proposição
abaixo.

Teorema 3. (Tricotomia.) Dados m, n  N, qualquer das afirmações m < n,


m = n, n < m exclui as outras duas.
Demonstração: Se tivéssemos m < n e m = n, então seria m = m + p,
donde m + 1 = m + p + 1 e, cortando m, concluiríamos que 1 = p + 1, um

EUREKA! N 3, 1998

32
Sociedade Brasileira de Matemática

absurdo, pois 1 não é sucessor de p. Portanto m < n (e analogamente, n < m)


é incompatível com m = n.
Do mesmo modo, se tivéssemos m < n e n < m, então teríamos n = m + p e
m = n + k, do que resultaria n = n + k + p, logo n + 1 = n + k + p + 1 e,
cortando n, concluiríamos que 1 = k + p + 1, um absurdo.
O teorema seguinte mostra que n e n + 1 são números consecutivos.

Teorema 4. Não existem números naturais entre n e n + 1.


Demonstração: Se fosse possível ter n < p < n + 1, teríamos p = n + k e n
+ 1 = p + r, logo n + 1 = n + k + r. Cortando n, obteríamos 1 = k + r. Por
definição, isto significaria k < 1, o que é absurdo, pois já vimos que k  1 
k > 1.
A conexão entre a relação de ordem e as operações de adição e multiplicação
é dada pelo seguinte teorema:

Teorema 5. (Monotonicidade.) Se m < n, então m + p < n + p e mp < np.


Demonstração: Usando a definição de <, temos que m < n  n = m + k 
n + p = (m + k) + p  m + p < n + p. Analogamente, m < n  n = m + k 
np = mp + kp  np >mp.
A recíproca da monotonicidade é a Lei do Corte para desigualdades: m + p <
n + p  m < n e mp < np  m < n. O leitor poderá prová-la por absurdo,
usando a tricotomia e a própria monotonicidade.

6. BOA ORDENAÇÃO

Dado o subconjunto A  N, diz-se que o número natural a é o menor


(ou primeiro) elemento de a quando a  A e, além disso, a  x, para todos os
elementos x  A.
Por exemplo, 1 é o menor elemento de N.
De agora em diante, dado n  N, indicaremos com In o conjunto dos
números naturais p tais que 1  p  n. Assim, I1 = {1}, I2 = {1, 2}, I3 = {1,
2, 3} etc.
As propriedades da relação de ordem m < n, demonstradas na seção
anterior para os números naturais (exceto o Teorema 4 que vale apenas para
números inteiros), são igualmente válidas para os números inteiros, racionais
e, mais geralmente, para números reais quaisquer. Existe, porém, uma
propriedade de suma importância que é válida para a ordem entre os

EUREKA! N 3, 1998

33
Sociedade Brasileira de Matemática

números naturais, mas sem equivalente para números inteiros, racionais ou


reais.

Teorema 6. (Princípio da Boa Ordenação.) Todo subconjunto não-vazio A 


N possui um menor elemento.
Demonstração: Sem perda de generalidade, podemos admitir que 1  A,
pois caso contrário 1 seria evidentemente o menor elemento de A. O menor
elemento de A, cuja existência queremos provar, deverá ser da forma n + 1.
Devemos pois encontrar um número natural n tal que n +1  A e, além
disso, todos os elementos de A são maiores do que n, logo maiores do que 1,
2, …, n. Noutras palavras, procuramos um número natural n tal que In  N –
A e n + 1  A. Com esse objetivo, consideramos o conjunto
X = {n  N; In  N – A}.
Portanto, X é o conjunto dos números naturais n tais que todos os elementos
de A são maiores do que n. Como estamos supondo que 1  A, sabemos que
1  X. Por outro lado, como A não é vazio, nem todos os números naturais
pertencem a X, ou seja, temos X  N. Pelo axioma D, vemos que o conjunto
X não é indutivo, isto é, deve existir algum n  X tal que n + 1  X Isto
significa que todos os elementos de A são maiores do que n mas nem todos
são maiores do que n + 1. Como não há números naturais entre n e n + 1,
concluímos que n + 1 pertence a A e é o menor elemento de A.

O Princípio da Boa Ordenação pode muitas vezes ser usado em


demonstrações, substituindo o Princípio da Indução. Vejamos um exemplo.
Dissemos anteriormente que um subconjunto X  N chama-se
indutivo quando n  X  n + 1  X, ou seja, quando X contém o sucessor
de cada um dos seus elementos. O Princípio da Indução afirma que se um
conjunto indutivo X contém o número 1 então X contém todos os números
naturais.
Vamos usar o Princípio da Boa Ordenação para provar que se um
conjunto indutivo X contém o número a, então X contém todos os números
naturais maiores do que a.
A prova desta afirmação se faz por absurdo, como ocorre em geral
quando se usa a boa ordenação. Suponhamos então que existam números
naturais, maiores do que a, não pertencentes ao conjunto indutivo X. Seja b o
menor desses números. Como b > a, podemos escrever b = c + 1, onde, pela

EUREKA! N 3, 1998

34
Sociedade Brasileira de Matemática

definição de b, tem-se necessariamente c  X. Mas, como X é indutivo, isto


obriga que b = c + 1  X, uma contradição.
A proposição qua acabamos de demonstrar pode ser enunciada da
seguinte forma:

Teorema 7: (Princípio da Indução Generalizado.) Seja P uma propriedade


referente a números naturais, cumprindo as seguintes condições:
(1) O número natural a goza da propriedade P;
(2) Se um número natural n goza da propriedade P então seu sucessor n + 1
também goza de P.
Então todos os números naturais maiores do que ou iguais a a gozam da
propriedade P.

Exemplo 2. Vejamos uma situação simples onde se emprega o Princípio da


Indução Generalizado. Trata-se de provar que 2n + 1 < 2n, para todo n  3.
Esta afirmação, (que é falsa para n = 1 ou n = 2), vale quando n = 3.
Supondo-a válida para um certo n  3, mostremos que daí decorre sua
validez para n + 1. Com efeito, 2(n + 1) + 1 = (2n + 1) + 2 < 2n + 2 < 2n + 2n
= 2n + 1. (Na primeira desigualdade, usamos a hipótese de indução.)

Exemplo 3. Usando a desigualdade 2n + 1 < 2n, qua acabamos de provar


para n  3, podemos demonstrar que n2 < 2n para todo n  5, empregando
novamente o Princípio da Indução Generalizado. Com efeito, vale 5 2 < 25
pois 25 < 32. Supondo válida a desigualdade n2 < 2n para um certo valor de n
 5, daí segue-se que (n + 1)2 = n2 + 2n + 1 < 2n + 2n + 1 (pela hipótese de
indução) < 2n + 2n (pelo exemplo anterior) = 2n + 1. Portanto P(n)  P(n + 1).
Pelo Princípio de Indução Generalizado, segue-se que P(n) vale para todo
n  5. Evidentemente, a desigualdade n2 < 2n é falsa para n = 1, 2, 3, 4.
O teorema abaixo contém outra aplicação do Princípio da Boa Ordenação.

Teorema 8. Toda função monótona não-crescente f: N  N é constante a


partir de um certo ponto. ( Isto é, existe n0  N tal que f(n) = f(n0), para todo
n  n0.)
Demonstração: Seja n0 o menor elemento do conjunto X = {f(1), f(2), …,
f(n),…}. Então n > n0  f(n)  f(n0) (porque a função f é não-crescente) o
que acarreta que f(n) = f(n0) (porque f(n0) é o menor elemento de X).

EUREKA! N 3, 1998

35
Sociedade Brasileira de Matemática

Corolário: Toda seqüência decrescente n1 > n2 > … de números naturais é


finita. Com efeito, do contrário, pondo f(k) = nk, obteríamos uma função
estritamente decrescente f : N  N.

7. SEGUNDO PRINCÍPIO DA INDUÇÃO


Em algumas situações, ao tentarmos fazer uma demonstração por
indução, na passagem de n para n + 1, sentimos necessidade de admitir que a
proposição valha não apenas para n e sim para todos os números naturais
menores do que ou iguais a n. A justificativa de um raciocínio desse tipo se
encontra no
Teorema 9: (Segundo Princípio da Indução.) Seja X  N um conjunto com
a seguinte propriedade:
(I) Dado n  N, se todos os números naturais menores do que n
pertencem a X, então n  X.
O segundo Princípio da Indução afirma que um conjunto X  N
com a propriedade (I) coincide com N.
Demonstração: Com efeito, supondo, por absurdo, que X  N, isto é,
que N – X  , seja n o menor elemento do conjunto N – X, ou seja, o menor
número natural que não pertence a X. Isto quer dizer que todos os números
naturais menores do que n pertencem a X. Mas então, pela propriedade (I), n
pertence a X, uma contradição. Segue-se que N – X =  e X = N.

Obs. : Se um conjunto X  N goza da propriedade (I), para que um número


natural n não pertencesse a X seria necessário que existisse algum número
natural r < n tal que r  X. Em particular, se n = 1, como não existe número
natural menor do que 1, a hipótese 1  X não pode ser cumprida. Noutras
palavras, (I) já contém implicitamente a afirmação de que 1  X. Assim, ao
utilizar o Segundo Princípio da Indução, não é preciso estipular que X
contém o número 1.

Toda propriedade P que se refira a números naturais define um subconjunto


X  N, a saber, o conjunto dos números naturais que gozam da propriedade
P. (E reciprocamente, todo conjunto X  N define uma propriedade referente
a números naturais, a saber, a propriedade de pertencer a X.) Deste modo,
"propriedade" e "conjunto" são noções equivalentes. Por isso, é natural que o
Segundo Princípio da Indução possua a formulação seguinte, onde ele
aparece como o

EUREKA! N 3, 1998

36
Sociedade Brasileira de Matemática

Teorema 10: (Segundo método de demonstração por indução.) Seja P uma


propriedade referente a números naturais. Dado n  N, se a validade de P
para todo número natural menor do que n implicar que P é verdadeira para
n, então P é verdadeira para todos os números naturais.
Demonstração: Com efeito, nas condições do enunciado, o conjunto X dos
números naturais que gozam da propriedade P satisfaz a condição (I) do
Segundo Princípio da Indução, logo X = N e P vale para todos os números
naturais.
Aplicaremos agora o Segundo Princípio da Indução para demonstrar
um fato geométrico. No exemplo a seguir, usamos os números naturais como
instrumento de contagem, isto é, como números cardinais, pois empregamos
expressões do tipo um polígono de n lados". (Vide seção 6.)
Sabe-se que, traçando diagonais internas que não se cortam, pode-se
decompor qualquer polígono em triângulos justapostos. Isto é evidente
quando o polígono é convexo: basta fixar um vértice e traçar as diagonais a
partir dele. Se o polígono não é convexo, a prova requer mais cuidados.
(Vide "Meu Professor de Matemática", pag. 109.)
O leitor pode experimentar com um polígono não-convexo e
verificar qua há muitas maneiras diferentes de decompô-lo em triângulos
justapostos mediante diagonais internas. Mas vale o resultado seguinte, no
qual usaremos o Segundo Princípio da Indução.

Exemplo 4. Qualquer que seja a maneira de decompor um polígono P, de n


lados, em triângulos justapostos por meio de diagonais internas que não se
intersectam, o número de diagonais utilizadas é sempre n – 3.

Com efeito, dado n, suponhamos que a proposição acima seja


verdadeira para todo polígono com menos de n lados. Seja então dada uma
decomposição do polígono P, de n lados, em triângulos justapostos,
mediante diagonais internas. Fixemos uma dessas diagonais. Ela decompõe
P como reunião de dois polígonos justapostos P1, de n1 lados, e P2, de n2
lados, onde n1 < n e n2 < n, logo a proposição vale para os polígonos P1 e P2.
Evidentemente, n1 + n2 = n + 2.

EUREKA! N 3, 1998

37
Sociedade Brasileira de Matemática

P2
P1

As d diagonais que efetuam a decomposição de P se agrupam assim:


n1 – 3 delas decompõem P1, n2 – 3 decompõem P2 e uma foi usada para
separar P1 de P2. Portanto d = n1 – 3 + n2 – 3 + 1 = n1 + n2 – 5. Como n1 + n2
= n + 2, resulta que d = n – 3. Isto completa a demonstração.

Observações:

1. Para habituar-se com o método de demonstração por indução é


preciso praticá-lo muitas vezes, a fim de perder aquela vaga
sensação de desonestidade que o principiante tem quando admite
que o fato a ser provado é verdadeiro para n, antes de demonstrá-lo
para n + 1.

2. Pratique também (com moderação) o exercício de descobrir o erro


em paradoxos que resultam do uso inadequado do método de
indução. Vejamos dois desses sofismas:

Exemplo 5. Todo número natural é pequeno.


Ora, 1 certamente é pequeno. E se n é pequeno, n + 1 não vai
subitamente tornar-se grande, logo também é pequeno. (O erro aqui consiste
em que a noção "número pequeno" não é bem definida.)
Exemplo 6. Toda função f : X  Y, cujo domínio é um conjunto finito X, é
constante.
Isto é obviamente verdadeiro se X tem apenas 1 elemento. Supondo
a afirmação verdadeira para todos os conjuntos com n elementos,
seja f : X  Y definida num conjunto X com n + 1 elementos. Considere um
elemento a  X. Como X' = X – {a} tem n elementos, f assume o mesmo
valor c  Y em todos os elementos de X'. Agora troque a por um outro

EUREKA! N 3, 1998

38
Sociedade Brasileira de Matemática

elemento b  X'. Obtém-se X'' = X – {b} um conjunto com n elementos


(entre os quais a). Novamente pela hipótese de indução, f é constante e igual
a c em X''. Logo f (a) = c e daí f : X  Y é constante. (Aqui o erro reside no
uso inadequado da hipótese de indução. O raciocínio empregado supõe
implicitamente que X tem pelo menos 3 elementos. Na realidade, não vale a
implicação P(1) P(2).)

O perigo de fazer generalizações apressadas relativamente a


asserções sobre números naturais fica evidenciado com o seguinte exemplo:

Exemplo 7. Considere o polinômio p(n) = n2 – n + 41 e a afirmação "o valor


de p(n) é sempre um primo para n = 0, 1, 2, 3, …". Embora isso seja
verdadeiro para n = 0, 1, 2, …, 40, temos p(41) = 412 – 41 + 41 = 412 não é
primo, logo a afirmação não é verdadeira.

Semelhantemente, a expressão q(n) = n2 – 79n + 1601 fornece


primos para n = 1, 2, …, 79, mas q(80) = 802 – 79  80 + 1601 = 1681 não é
primo, pois é divisível por 41. A moral da história é: Só aceite que uma
afirmação sobre os números naturais é realmente verdadeira para todos os
naturais se isso houver de fato sido demonstrado!

8. NÚMEROS CARDINAIS
Vamos agora mostrar como se usam os números naturais para contar
os elementos de um conjunto finito. O Princípio da Indução será essencial.
Lembremos que, dado n  N, escrevemos In = {p  N; p  n}, portanto
In = {1, 2, …, n}.
Uma contagem dos elementos de um conjunto não-vazio X é uma bijeção
f : In  X. Podemos pôr x1 = f(1), x2 = f(2),…, xn = f(n) e escrever
X = {x1, x2,…xn}. Diz-se então que X possui n elementos. O conjunto X
chama-se um conjunto finito quando existe n  N tal que X possui n
elementos.
Um exemplo óbvio de conjunto finito é In. Evidentemente, a função
identidade f: In  In é uma contagem dos elementos de In.
Um exemplo de conjunto infinito é o proprio conjunto N dos
números naturais, pois nenhuma função f : In  N pode ser sobrejetiva, não
importa qual n se tome. De fato, dada f, tomamos k = f(1) + f(2) +…+ f(n) e
vemos que k > f(x) para todo x  In, logo k  f(In), e f não é sobrejetiva.

EUREKA! N 3, 1998

39
Sociedade Brasileira de Matemática

A fim de que não haja ambigüidade quando se falar do número de


elementos de um conjunto finito X, é necessário provar que todas as
contagens de X fornecem o mesmo resultado. Noutras palavras, dado o
conjunto X, os números naturais m, n e as bijeções f : Im  X, g : In  X,
devemos mostrar que se tem m = n. Começamos observando que se f e g são
bijeções, então  = g–1  f : Im  In também é uma bijeção. Basta portanto
provar o seguinte:

Teorema 11. Dados m, n  N, se  : Im  In é uma bijeção, então m = n.


Demonstração. Com efeito, chamemos de X o conjunto dos números
naturais n que têm a seguinte propriedade: só existe uma bijeção  : Im  In
quando m = n. Evidentemente, 1  X. Suponhamos agora que n  X.
Dada uma bijeção : Im+1  In+1, duas coisas podem acontecer. Primeira:
(m + 1) = n + 1. Neste caso, a restrição Im : Im  In é uma bijeção, logo m
= n, donde m + 1 = n + 1. Segunda: (m + 1) = b, com b < n + 1. Neste
caso, consideramos
a =  –1(n + 1) e definimos uma nova bijeção  : Im + 1  In + 1, pondo  (m +
1) = n + 1, (a) = b e (x) = (x) para os demais elementos x  Im + 1. Então
recaímos no caso anterior e novamente concluímos que m + 1 = n + 1. Isto
mostra que n  X  n + 1  X, logo X = N e a unicidade do número cardinal
de um conjunto finito fica demonstrada.

Agora os números naturais não são apenas elementos do conjunto-


padrão N, mas servem também para responder perguntas do tipo "quantos
elementos tem o conjunto X?,"ou seja, podem ser usados também como
números cardinais.

A adição de números naturais se relaciona com a cardinalidade dos


conjuntos por meio da seguinte proposição.

Teorema 12: Sejam X, Y conjuntos finitos disjuntos. Se X tem m elementos e


Y tem n elementos, então X Y tem m + n elementos.
Demonstração: Com efeito, se f : Im  X e g : In  Y são bijeções,
definimos uma bijeção h : Im+n  X Y por h (x) = f (x) se 1  x  m e
h(x) = g(x) + m se m + 1  x  m + n, o que conclui a demonstração.
Prova-se, por indução, que todo subconjunto de um conjunto finito X é
também finito e seu número de elementos é menor do que ou igual ao de X
(Veja E.L.Lima, "Análise Real", vol 1, pag. 5.)

EUREKA! N 3, 1998

40
Sociedade Brasileira de Matemática

E conveniente incluir, por definição, o conjunto vazio entre os


conjuntos finitos e dizer que o seu número de elementos é zero. Embora zero
não seja um número natural, ele passa a ser o número cardinal do conjunto
vazio.
Seguem-se algumas proposições que devem ser demonstradas por
indução ou boa ordenação. Os dez últimos exercícios foram sugeridos pelo
Professor A. C. Morgado.

Exercícios:

1. Construa um esquema de setas começando com os números ímpares,


seguidos dos números pares divisíveis por 4 em ordem decrescente e,
por fim, os pares não divisíveis por 4 em ordem crescente. Noutras
palavras, tome X = N e defina s : X  X pondo s(n) = n + 2 se n não é
divisível por 4, s(n) = n – 2 se n for múltiplo de 4. Mostre que s : X  X
cumpre os axiomas A, B, C mas não D.
2. Defina, por recorrência, uma função f : N  N estipulando que f (1) = 3
e f (n + 1) = 5. f (n) + 1. Dê uma formula explícita para f (n).
3. Dê uma fórmula explícita para f : N  N sabendo que f(1) = 1, f(2) = 5
e f (n + 2) = 3f (n + 1) – 2f (n).
4. Seja X  N um conjunto indutivo não-vazio. Mostre que existe a  N tal
que X = {n  N; n  a}.
n(n  1)(2n  1)
5. Prove, por indução, que 12  2 2  ...  n 2  .
6
6. Num polígono com n  6 lados, o número de diagonais é maior do que n.
7. Prove, por indução que [(n + 1)/n]n < n, para todo n  3. (Sugestão:
Observe que (n + 2)/(n + 1) < ( n + 1)/n e eleve ambos os membros desta
desigualdade à potência n + 1.) Conclua daí que a seqüência
1, 2 , 3 3 , 4 4 , 5 5 ,... é decrescente a partir do terceiro termo.
8. Prove, por indução a desigualdade de Bernoulli: (1 + a)n > 1 + na
quando 1 + a > 0.

EUREKA! N 3, 1998

41
Sociedade Brasileira de Matemática

n
 ( n  1) 2 
9. Para todo n  N, ponha x n   e prove, por indução que se
 n( n  2) 
n2
tem x n  . Conclua, a partir daí, que a seqüência de termo geral
n 1
n
 n 1
  é crescente.
 n 
3
 n 2 n
Sugestão: observe que x n 1    x n .
 n 1  n  3

10. Use a distributividade de duas maneiras diferentes para calcular


(m + n )(1 + 1) e aplique em seguida a Lei do Corte para obter uma nova
prova de que m + n = n + m.

11. Um conjunto S  N, não-vazio, é limitado superiormente, se existe um


natural k tal que para todo natural x  S, então x  k. Mostre que S
possui um maior elemento. (Isto é, existe m  S tal que x  m, para todo
x  S.)

12. Demonstre que a soma dos n primeiros números ímpares é n2, ou seja,
que 1 + 3 + 5 +…+ (2n – 1) = n2.

13. Prove que 2n – 1 é múltiplo de 3, para todo número natural n par.

14. Demonstre que, para todo número natural n, vale


 1  1  1  1
 1  1   1  ... 1   n  1.
 1  2  3  n

15. Demonstre que


1 1 1 1 1 1 1 1
1     ..      ...  .
2 3 4 199 200 101 102 200

EUREKA! N 3, 1998

42
Sociedade Brasileira de Matemática

1 2 
16. Determine An se A =  
 2 4

p 1 pn 1


17. Demonstre, usando o Princípio da Indução Finita, que

  .  .


p  p 
Este resultado é comumente conhecido por Teorema das Colunas. (Por
quê?).

EUREKA! N 3, 1998

43
Sociedade Brasileira de Matemática

1 3 7 p
18. Considere a seqüência , , ,..., n ,..., onde
1 2 5 qn
p n 1  p n  2q n e q n 1  p n  q n . Demonstre que
a) m.d.c (pn, qn) = 1;
(1  2 ) n
b) pn é o inteiro mais próximo de e qn é o inteiro mais próximo
2
2
de (1  2 ) n .
4

19. [A Torre de Hanói.] São dados três suportes A, B e C. No suporte A estão


encaixados n discos cujos diâmetros, de baixo para cima, estão em
ordem estritamente decrescente. Mostre que é possível, com 2 n – 1
movimentos, transferir todos os discos para o suporte B, usando o
suporte C como auxiliar, de modo que jamais, durante a operação, um
disco maior fique sobre um disco menor.

20. Demonstre que 2n < n!, para n  4.

21. Demonstre que 2n3 > 3n2 + 3n + 1 para n  3.

22. Considere n retas em um plano. Mostre que o "mapa" determinado por


elas pode ser colorido com apenas duas cores sem que duas regiões
vizinhas tenham a mesma cor.

FRAÇÕES CONTÍNUAS,
REPRESENTAÇÕES DE NÚMEROS E APROXIMAÇÕES
EUREKA! N 3, 1998

44
Sociedade Brasileira de Matemática

Carlos Gustavo Moreira


 Nível Avançado.

INTRODUÇÃO

A teoria de frações contínuas é um dos mais belos temas da


matemática elementar, sendo ainda hoje assunto de pesquisa recente
(incluindo a do autor destas linhas). O objetivo deste artigo é servir como
referência didática em português a nível secundário sobre o assunto.

Nas inclusões N  Z  Q  R a passagem de Q para R é sem


dúvida a mais complicada conceitualmente, e a representação de um número
real está diretamente ligada à propria noção de número real.

De fato, o conceito de número natural é quase um conceito primitivo


no ensino secundário. Já um número inteiro é um número natural com um
sinal que pode ser + ou –, e um número racional é a razão entre um número
inteiro e um natural não nulo. Por outro lado, dizer o que é um número real é
tarefa bem mais complicada, mas há coisas que podemos dizer sobre eles.
Uma propriedade essencial de R é que todo número real pode ser bem
aproximado por números racionais. Efetivamente, dado x  R, existe k  Z
(k = [x]) tal que 0  x – k < 1. Podemos escrever a representação decimal de
x – k = 0, a1a2…an…, ai  {0, 1, …, 9}, o que significa que se
rn = an + 10.an–1 + 100.an–2 +…+ 10n–1 . a1, então
rn rn  1 rn
n
x  k  n
, e portanto k  é uma boa aproximação racional de
10 10 10 n
rn 1
x, no sentido que o erro x   k   é menor que , que é um
10 n
10 n
número bem pequeno se n for grande. A representação decimal de um
número real fornece pois uma seqüência de aproximações por racionais
cujos denominadores são potências de 10.

p p 1
Dado qualquer x  R e q natural não nulo existe p  Z tal que q x  q
p 1 p 1 1
, e portanto x
q

q
e x
q

q
. Em particular há aproximações
1
de x por racionais com denominador q com erro menor que . A
q

EUREKA! N 3, 1998

45
Sociedade Brasileira de Matemática

representação decimal de x equivale a dar essas aproximações para os


denominadores q que são potências de 10, e tem méritos como sua
praticidade para efetuar cálculos que a fazem a mais popular das
representações dos números reais. Por outro lado, envolve a escolha
arbitrária da base 10, e oculta freqüentemente aproximações racionais de x
muito mais eficientes do que as que exibe. Por exemplo,

2 1 314 35 1 3 141 592


     e     
7 70 10 1 3 30 0 0 10 0 0
22 355
mostram que e são melhores aproximações de  que aproximações
7 113
decimais com denominadores muito maiores, e de fato são aproximações
muito mais espectaculares do que se podia esperar.

O objetivo deste artigo é apresentar uma outra maneira de


representar números reais, que sempre fornece aproximações racionais
surpreendentemente boas, e de fato fornece todas essas aproximações
excepcionalmente boas, além de ser natural e conceitualmente simples: a
representação por frações contínuas.

Dado x  R definimos [x] como o único inteiro tal que [x]  x < [x] + 1).
Definimos recursivamente
1
 0 x, a n [ n ], e, se n  Z ,  n 1  , para todo n  N .
 n  an
Se, para algum n, n = an temos

EUREKA! N 3, 1998

46
Sociedade Brasileira de Matemática

1
x  0 a 0  : [a ; a1 , a 2 ..., a n ].
1 0
a1 
1
a 2  ... 
an
Se não denotamos

1
x a0  : [ a 0 ; a1 , a 2 ...].
1
a1 
a 2  ...

O sentido dessa última notação ficará claro mais tarde. A representação


acima se chama a representação por frações contínuas de x.

Curiosidade: O denominador da n-ésima aproximação em base B de um


número real é Bn. Já o denominador qn da n-ésima aproximação por fração
contínua de x depende de x. Apesar disso, para quase todo real x, n q n
2
converge a e  / 12 ln 2
3,27582291872... (meu número real preferido!) e

pn
n x
2
converge a e   / 6 ln 2
0,093187822954...

qn
Observação: Os n (como funções de x) são funções distintas do tipo
ax  b
com a, b, c, d inteiros. Se a fração contínua de x é periódica, ou seja,
cx  d
se n + k = n, n  N, k  N*, então x será raiz de uma equação do segundo
grau com coeficientes inteiros, ou seja, será um irracional da forma
r + s , r , s  Q. A recíproca é verdadeira (de fato já foi enunciada no
artigo de José Paulo Carneiro na RPM, ver referências), mas sua prova é
mais difícil, e será apresentada no Apêndice.
Se x  Q, sua representação será finita, e seus coeficientes an vêm do
algoritmo de Euclides:

EUREKA! N 3, 1998

47
Sociedade Brasileira de Matemática

p
x , q 0 p a 0 q  r0 0 r0  q
q
q a1 r0  r1 0 r1  r0
r0 a 2 r1  r2 0 r2  r1
 
rn  2 a n rn  1

Isso já é uma vantagem da representação por frações contínuas (além de não


depender de escolhas artificiais de base), pois o reconhecimento de racionais
é mais simples que na representação decimal.

Seção 1: Reduzidas e boas aproximações.

Seja x = [a0; a1, a2, …]. Sejam pn  Z, qn  N* primos entre si tais que
pn
= [a0; a1, a2, …, an], n  0. O seguinte resultado será fundamental no
qn
que seguirá.

Proposição: (pn) e (qn) satisfazem a recorrência pn+2 = an+2 pn+1+ pn e qn+2 =


an+2 qn+1 +qn, para todo n  0. Temos ainda p0 = a0, p1 = a0a1 +1, q0 = 1,q1 =
a1. Além disso, pn+1 qn – pnqn+1 = (–1)n, n  0.

Prova: Por indução em n, provaremos que se tk > 0, para k > 1 então

xk
[t0; t1, t2, …, tk] = onde as seqüências (xm) e (ym) são definidas por
yk

x0 = t0, y0 = 1, x1 = t0t1 + 1, y1 = t0, xn+2 = tn+2 xn+1 + xn, yn+2 = tn+2 yn+1 + yn, n.
Suponha que a afirmação seja válida para k = n. Para k = n+1 temos

1
[t0; t1, t2, …, tn, tn+1] = [t0; t1, t2, …, tn + t ]=
n 1

EUREKA! N 3, 1998

48
Sociedade Brasileira de Matemática

1
(t n  ) x n 1  x n  2
t n 1 t (t x  x n  2 )  x n  1 t n 1 x n  x n  1
 n 1 n n  1  .
1 t n 1 (t n y n  1  y n  2 )  y n  1 t n 1 y n  y n  1
(t n  ) y n 1  y n 2
t n 1

Por outro lado as igualdades

 p1q0 – p0q1 = (a0a1 +1) – a0a1 = 1


 pn+2 qn+1 – pn+1 qn+2 = (an+2 pn+1 + pn) qn+1 – (an+2 qn+1 + qn) pn+1 =
= – ( pn+1 qn – pnqn+1)
mostram que pn+1 qn – pnqn+1 = (–1)n, n  N, o que implica em particular
que os pn, qn dados pelas recorrências acima são primos entre si.

 n p n 1  p n 2 p n 2  q 
Corolário: x  e n  n 2
, n  N.
 n q n 1  q n 2 q n  1  p n  1

Prova: A primeira igualdade é conseqüência direta da prova, e a segunda é


conseqüência direta da primeira pois x = [a0; a1, a2, …, an–1, n].
Note que as reduzidas de ordem par são menores e as de ordem
ímpar maiores que x = [a0; a1,…].

pn 1 1
Teorema 1: x    2 , n  N.

qn qn qn1 qn

EUREKA! N 3, 1998

49
Sociedade Brasileira de Matemática

pn 1 pn1 1
Além disso,
x   2 ou x   2 ,n N.

qn 2qn qn1 2qn1


pn p n 1
Prova: x sempre pertence ao segmento de extremos e cujo
qn q n 1
comprimento é:

n
pn1 pn ( 1) 1 pn 1 1
    x   2 .
qn1 qn qnqn1 qnqn1 qn qnqn1 qn
Além disso, se

EUREKA! N 3, 1998

50
Sociedade Brasileira de Matemática

pn 1 pn1 1 1 pn pn1
x  2 xe   2 então x x 
1
2q n2

1
2q n2 1
 q n 1 q n ,

qn 2qn qn1 2qn1 q nn 1 qn qn1


absurdo 

pn 1 1
Observação: De fato x   2 . Quanto maior for

qn qn qn1 an1qn
pn
an+1 melhor será a aproximação de x. O próximo resultado nos dá
qn
pn
explicitamente o erro da aproximação de x por .
qn
pn (  1) n
Proposição: x  , onde
q n ( n 1   n 1 )q n2
q n 1
 n 1  [0; a n , a n  1 , a n  2 ,..., a1 ].
qn

EUREKA! N 3, 1998

51
Sociedade Brasileira de Matemática

p n 1  q n 1 x
Demonstração: Temos  n 1  . Portanto,
qn x  pn

pn  1  qn  1 x qn  1 p q  pn qn  1 ( 1) n p
 n 1   n 1    n 1 n   x n 
qn x  pn qn qn (qn x  pn ) qn (qn x  pn ) qn
q n (q n x  p n ) ( 1) n
 
q n2 ( n 1   n 1 )q n2

Como aplicação podemos provar o seguinte.

Teorema (Hurwitz, Markov): Para todo  irracional, n  1 temos

p  pn1 pn pn1 

p
q

1
5q 2
para pelo menos um racional   , , . Em

q  qn1 qn qn1 
p 1
particular    tem infinitas soluções racionais p/q.
q 5q 2

Demonstração: Suponha que o teorema seja falso. Então existe  irracional,


n  1 com  n   n  5, n 1   n 1  5 e  n 2   n  2 5. Devemos
portanto ter an = an+1 = an+2 = 1 (todos são claramente no máximo 2, e se
algum ak é igual a 2 com k  {n, n + 1, n + 2}, teríamos
1
 k   k 2   5 , absurdo.)
3
Seja x = 1/n+2 e y = n+1. As desigualdades acima se traduzem em
1 1 1 1
  5, 1  x  y  5 e   5. Temos
1 x y x 1 y
1 1 1 1 5
1 x  y  5  1 x  5  y      e
1 x y 5 y y y ( 5  y ),
51
portanto y ( 5  y ) 1  y  . Por outro lado temos
2

EUREKA! N 3, 1998

52
Sociedade Brasileira de Matemática

1 1 1 1 5
x  5  1 y      e portanto
x 1 4 5  1  y 1  4 (1  y )( 5  1  y )
51 51
(1  4)( 5  1  y ) 1  y  , e portanto devemos ter y  , o
2 2
qn 1
que é absurdo pois y  n 1   Q.
qn
p 1
Obs: em particular provamos que    tem infinitas soluções
q 5q 2
p
racionais q , para todo  irracional. 5 é o maior número com essa
propriedade, De fato, se
1 5 p 1
 > 0,   e  q  , temos
2 ( 5   )q 2
1 5 p

 1 5  1  1 5    2 q
q  p   q   p q 1  5   p  ,
2  ( 5   )q 2   2  5 
     

2 2 1 5 p
ou seja , p  pq  q  2

q
 5 ( 5   ). Se q é grande, 1/q2

1 5 p
é pequeno, e  é muito próximo de 0, donde
2 q
1 5 p 5
  5 ( 5 ) é muito próximo de  1, absurdo, pois
2 q 5 
p2  pq  q 2 1 (de fato p2 – pq – q2 é um inteiro não nulo, pois se
2
 p  p p  1  5 1  5 
p2 – pq – q2 = 0 teríamos       1 0    , ,
q q q  2 2 
p
absurdo, pois  Q .)
q

1 5 p 1
Outra maneira de ver que, para todo  > 0, 2

q
 tem
( 5   )q 2
p
apenas um número finito de soluções q  Q é observar que as melhores
p
1 5 n
aproximações racionais de são as reduzidas q de sua fração
2 n

contínua [1, 1, 1, 1, …] (ver seção 2 e exemplos), para as quais temos

EUREKA! N 3, 1998

53
Sociedade Brasileira de Matemática

1  5 pn 1
  ,
2 com

n 1

n  1 se aproximando cada
2 q (   ) q
n n 1 n 1 n

vez mais de
1 5 5 1
[1;1,1,1...]  [0;1,1,1,...]    5.
2 2

Exemplos:

  = [3; 7, 15, 1, 292, 1, 1, 1, 2, 1, 3, 1, 14, 2, 1,…], portanto


p0 p 22 p 333 p 3 355
3, 1  , 2  ,  ,...
q0 q1 7 q 2 106 q 3 113
 e = [2; 1, 2, 1, 1, 4, 1, 1, 6, 1, 1, 8,…, 1, 1, 2n, …], (isso não é fácil
de provar.)
 2 [1;2,2,2,...] pois

1 1 1
2 1  1  1  ...
2 1 1 1
2 2
2 1 1
2
2 1
1 5 1 1
1  1  ...
1 5 2 1 5 1
[1;1,1,1,...] pois 1 
2 2 1 5
2
1 5
Isso prova em particular que 2 e são irracionais, pois sua fração
2
contínua é infinita.

Seção 2: Boas aproximações são reduzidas.

O próximo teorema (e seu Corolário 2) caracteriza as reduzidas em termo do


erro reduzido da aproximação de x por p/q, o qual é, por definição, a razão
entre x  p / q e o erro máximo da aproximação por falta com
denominador q, que é 1/q. Assim, o erro reduzido da aproximação de x por
p/q é qx  p .

EUREKA! N 3, 1998

54
Sociedade Brasileira de Matemática

p pn
Teorema 2: q n x  p n  qx  p , p, q  Z,0  q q n ,  .
q qn
Além disso, q n x  p n  qx  p , p, q  Z,0  q  q n 1 .

p pn 1 1 p
Prova:    se q < qn+1, e assim está fora do
q qn qq n q n q n 1 q

 p n p n 1 
intervalo  ,  . Portanto,
 q n q n 1 

p  p p p p  1 1
x min   n ,  n 1   qx  p   qn x  pn .
q  q q n q q n 1  qq n  1 q n 1
p n 1
Além disso, se vale a igualdade, então x  , donde an+1  2, e qn+1 >
q n 1
2qn, pois numa fração contínua finita, como no algaritmo de Euclides, o
último coeficiente an é sempre maior que 1. Nesse caso, se q  qn , teremos
p p p p 1 1 q  q 1 1
x  x  n  n 1  n    n 1   qx  p   qn x  pn .
q qn qn 1 qn qqn qn qn 1 qqn qn 1 qqn 1 qn 1
pn p
Corolário 1: x   x , q  q n .
qn q

p q'
Corolário 2: Se qx  p  q ' x  p ' , q ' q, q  q ' então p/q é uma
reduzida da fração contínua de x.

Prova: Tome n tal que q n q  q n 1 .


Teremos q n x  p n  qx  p , e portanto p/q  p n /q n 

p 1 p
Teorema 3: Se x   então é uma reduzida da fração contínua de
q 2q 2 q
x.

EUREKA! N 3, 1998

55
Sociedade Brasileira de Matemática

p p n 1
Prova: Seja n tal que qn < q  qn+1. Suponha que  . Então, temos
q q n 1
duas possibilidades:
q p 1 1
a) q  n 1  x    2.
2 q qq n 1 2q

b)
qn 1 p p p p p 1 1
q  qn 1  2qn  x   n  nm  n   
2 q qn q qnm qn qqn qn qn 1
qn 1  q 1 1
   2.
qqn qn 1 2qqn 2q

Apêndice: Frações contínuas periódicas

Nesta seção provaremos que os números reais com fração contínua periódica
são exatamente as raízes de equações do segundo grau com coeficientes
inteiros.
Lembramos que na representação de x por fração contínua, an, n são
definidos por recursão por

1
 0 x, a n [ n ],  n 1  .
 n  an
e temos
pn 2  q n 2 x
n  , n  N .
q n 1 x  p n 1

Isso dá uma prova explícita do fato de que se a fração contínua de x é


periódica, então x é raiz de uma equação do segundo grau com coeficientes
inteiros. De fato, se n + k = n , n  N, k  N* então

EUREKA! N 3, 1998

56
Sociedade Brasileira de Matemática

p n  2  q n  2 x p n k  2  q n k  2 x
 
q n 1 x  p n 1 q n k  1 x  p n k  1
(q n  1 q n  k  2  q n  2 q n  k  1 ) x 2  ( p n  k  1 q n  2  p n  2 q n  k  1  p n  k  2 q n  1  p n  1 q n  k  2 )
x  p n  1 p n  k  2  p n  2 p n  k  1 0.
qn 1
Note que o coeficiente de x2 é não-nulo, pois é uma fração irredutível
q n 2
q n k  1
(de fato p n  1 q n  2  p n  2 q n  1 (  1) n ) de denominador qn–2 e é
q n k  2
uma fração irredutível de denominador qn+k–2 > qn–2 , donde
qn  1 q n k  1
  q n  1 q n k  2  q n  2 q n k  1 0.
qn  2 q n k  2
Vamos provar agora um resultado devido a Lagrange segundo o qual se x é
uma irracionalidade quadrática, isto é, se x é um irracional do tipo r +
s , r , s  Q, s  0 então a fração contínua de x é periódica, i. e,
existem
n  N, k  N* com n + k = n . Neste caso, existem a, b, c inteiros tais que
ax 2  bx  c 0 , com b 2  4ac  0 e b 2  4ac irracional. Como vimos
na seção 1,
p   pn 2
x  n 1 n , e portanto
q n  1 n  q n  2
2
2  p   p n 2   p   p n 2 
ax  bx  c 0  a n  1 n   b n  1 n c 0
 q  q 
 q n  1 n  q n  2   n 1 n n 2 
 An n2  Bn n  C n 0,
onde

An ap n2 1  bp n  1 q n  1  cq n2 1


B n 2ap n  1 p n  2  b( p n  1 q n  2  p n  2 q n  1 )  2cq n  1 q n  2
C n ap n2 2  bp n  2 q n  2  cq n2 2.

Note que Cn = An–1. Vamos provar que existe M > 0 tal que 0 < A n M
para todo n  N, e portanto 0  C n M , n  N :

EUREKA! N 3, 1998

57
Sociedade Brasileira de Matemática

 p   p 
An ap n2 1  bp n  1 q n  1  cq n2 1 aq n2 1  x  n  1   x  n  1 ,
 q n  1  q n 1
  
onde x e x são as raízes de a, X2 + bX + c = 0, mas
p n 1 1 p p n 1  p 
x  2 1  An aq n2 1 x  n  1 x a  x  x  x  n  1 
q n 1 qn 1 q n 1 q n 1  q 
 n 1 

a x  x  1 : M . 
Notemos agora que B n2  4 An C n b 2  4ac, n  N. De fato,
Bn2  4 An C n ( p n  1 q n  2  p n  2 q n  1 ) 2 (b 2  4ac) b 2  4ac. Portanto,
Bn2  4 AnCn  b 2  4ac  4M 2  b 2  4ac  Bn  M '  4M 2  b 2  4ac,n  N.
Provamos assim que An, Bn e Cn estão uniformemente limitados, donde há
apenas um número finito de possíveis equações An X2 + BnX + Cn = 0, e
portanto de possíveis valores de n. Assim, necessariamente n+k = n para
alguma escolha de n  N, k  N*.

Referências:
 N. Beskin - Frações contínuas - Iniciação à Matemática - Editora Mir.
 José Paulo Q. Carneiro - Um processo finito para a raiz quadrada – Revista do
Professor de Matemática 34, 1997, pp. 36-44.
 C.D. Olds - Continued Fractions - New Mathematical Library - Random House.
 A. M. Rockett, P. Szüsz - Continued Fractions - World Scientific.

SOLUÇÕES DE PROBLEMAS PROPOSTOS EUREKA! Nos. 1 e 2

 Publicamos aqui algumas


das respostas enviadas por nossos leitores.

2) Em uma pista circular há postos de gasolina, e o total de gasolina que


há nos postos é exatamente o suficiente para um carro dar uma volta.
Prove que existe um posto de onde um carro com o tanque

EUREKA! N 3, 1998

58
Sociedade Brasileira de Matemática

inicialmente vazio pode partir e conseguir dar uma volta completa na


pista (parando para reabastecer nos postos).

Solução

Sejam P1, P2,…,Pn os postos de gasolina, li a quantidade de gasolina no posto


Pi e ci a quantidade de gasolina necessária para ir de Pi a Pi+1, para
i = 1, 2,…, n (convenção: para 1  k  n, Pn+k : = Pk ). Por hipótese,
n n k k
 l  c .
i 1
i
i 1
i Suponha que exista k com 1  k  n e  l  c
i 1
i
i 1
i (se

não existe tal k podemos dar a volta começando em P1). Tome k0 com 1 k0 
k0
n tal que  (l
i 1
i  c i ) seja o menor possível. Afirmamos que podemos dar a

volta começando em Pk 0 1 . De fato, se não for assim, existe r com 1 r  n


k0 r k0 r k0
e 
i k 0 1
(l i  c i )  0, mas então teríamos  (l i  c i )   (l i  ci ), o que é
i 1 i 1
k 0 r k o r  n
um absurdo (se k0+r > n temos i 1
(l i  c i )   (l
i 1
i  c i ), pois
n

 (l
i 1
i  c i ) 0).

3) Prove que existe n  N tal que os 1000 primeiros dígitos de n1998 são
iguais a 1.

Solução

Seja n  N tal que


111 ..11  0 1 2 ... p ; onde  i é tal que 0  i 9;
n1998 =   
1000 algarismos
i = 1, 2, 3, …, p. Seja também k = 111…11, daí:
k 9999...9,
k .10s  n1998     logo k . 10s  n1998 < (k + 1) . 10s,
s algarismos

Precisamos garantir que há algum n  N que satisfaça a desigualdade acima;

EUREKA! N 3, 1998

59
Sociedade Brasileira de Matemática

seja então s = 1998 . p :

k . 101998 . p  n1998 < (k + 1) . 101998 . p 


n
1998
1998
k .10 p n  1998 k  1.10 p  k  p  1998 k  1.
10
observe que se tomarmos n = 10 . 
p 1998

k  1; onde  z  = maior inteiro
menor
ou igual a z, e p suficientemente grande satisfaremos a condição do enuncia-
do.

Conclusão: n  N tal que n1998 é escrito como no enunciado.

5) Sejam a  0 e P1P2P3P4P5 uma poligonal aberta contida em um dos


semiplanos determinados pela reta P1 P5 . Prove que existem
pontos P6 e P7 no plano, com P5 P6 = a, de modo que é possível
ladrilhar o plano com infinitos ladrilhos congruentes ao heptágono
P1P2P3P4P5P6P7.

Solução

Traçe a paralela a P3P2 passando por P1. O ponto P7 pertencerá a essa reta e
 
teremos P1 P7  P3 P2 . O ponto P6 pertencerá à paralela a P3P4 passando
 a 
por P5 e satisfará P5 P6 = a, ou seja, P5 P6  P3 P4 .
P3 P4
Rodando o heptágono H = P1P2P3P4P5P6P7 de 180 em torno do ponto médio
de P1P2 obtemos o heptágono H' = P1'P2'P3'P4'P5'P6'P7' com
P1' = P2 , P2' = P1 , P3' = P7 , P7' = P3. Transladando infinitas vezes os

heptágonos H e H' por k . P3 P6 , k  Z, cobrimos uma faixa dentada, que,

transladada infinitas vezes por m . P4 ' P5 , m  Z, nos permite cobrir o
plano.
Desta forma, cobrimos o plano com os heptágonos
   
H  K . P3 P6  m . P4 ' P5 e H' + k . P3 P6  m . P4 ' P5 , k  Z, de
interiores disjuntos e todos congruentes a H.

EUREKA! N 3, 1998

60
Sociedade Brasileira de Matemática

6) Mostre que toda seqüência com n2+1 elementos possui uma


subseqüência crescente com n+1 elementos ou uma subseqüência
decrescente com n+1 elementos.

Solução

Dada uma seqüência a1, a2,…, a n 2 1 de números reais, definimos para 1  i


 n2+1 o número f (i) como sendo o número máximo de termos de uma
subseqüência decrescente de a1, a2,…, a n 2 1 começando em ai. Suponha que
não exista nenhuma subseqüência decrescente de n +1 elementos. Então
f (i)  n para todo i, e portanto f (i) só pode assumir os n valores 1, 2, …, n.
Assim, existem 1  i1 < i2 <…< in+1 com f (i1) = f (i2) = … = f (in+1), mas nesse
caso devemos ter a i1 a i2 ... a n 2 1 , com n + 1 termos.
Obs. 1: Mostra-se com um argumento análogo que toda seqüência
com mn+1 elementos possui uma subseqüência crescente de m+1 elementos
ou uma subseqüência decrecente de n+1 elementos (de fato que existe uma
seqüência crescente de m+1 elementos ou uma seqüência estritamente
decrescente de n+1 elementos.)

Obs. 2: O resultado (e sua generalização na obs. 1) é o melhor possível. De


fato, dados m, n  N, a seqüência de mn termos n, n–1, n–2, …, 1, 2n,
2n–1, 2n–2, …, n+1, 3n, …, 2n+1, …, mn, mn–1, …, (m–1) n+1 não contém
nenhuma seqüência crescente de mais de m elementos nem nenhuma
seqüência decrescente de mais de n elementos.

12) a) Prove que se n  N e 2n + 1 é um número primo então n é


uma potência de 2.
b) Prove que se a, n  N, n  2 e an –1 é primo, então a = 2 e n é
primo.

Solução

a) Sabemos que n  N pode ser escrito da seguinte forma: n 2 k p onde


k  N e p é ímpar.

EUREKA! N 3, 1998

61
Sociedade Brasileira de Matemática

Seja n = 2n + 1, logo n = 2 2 p  1  2 2
k
  k p k
 1, fazendo  2 2  x  p  1.
Se p é im ímpar maior do que 1, teremos:
x  p  1 (  1)( p  1   p  2   p  3  ...  1) e, como x é primo, ele não
poderá ser fatorável em um produto de fatores diferentes de 1. Basta então
observar que o segundo fator da multiplicação acima não é igual a 1 com p
ímpar maior do que 1, mas isso segue de  p     p  1   1. Logo
k
devemos ter necessariamente x 2 2  1, ou seja n = 2k.

b) Seja y = an – 1 = (a – 1)(an–1 + an–2 +…+ a +1) primo:

i) Vamos verificar inicialmente que a deve ser igual a 2. De fato


a – 1 = 1, já que o segundo fator não pode ser igual a 1 (a  1).
ii) Suponha que n não seja primo, n = k1 . k2 com k1  2 e k2  2, Logo

( k 2  1)
y 2 k1 k2  1 (2 k1 ) k2  1 (2 k1  1) (2 k1  2 k1 ( k2  2)  ...  2 k1  1), obser-ve
que 2 k1  1 3 e 2 k1 ( k 2  1)  ...  2 k1  1  3 e conseqüentemente não
teremos y primo, logo n não pode ser escrito como acima; donde n é primo.

1 1 1
14) Determine o número de soluções de   com x e y inteiros
x y 1998
positivos.

Solução

Temos 1998x + 1998y = xy .


Somando 19982 dos dois lados temos
xy – 1998x – 1998y + 19982 = 19982, logo
x(y – 1998) – 1998(y – 1998) = 19982, donde
(x – 1998) (y – 1998) = 19982.

Desta forma o número de soluções é o mesmo que a quantidade de sistemas


da forma abaixo que possamos obter:

EUREKA! N 3, 1998

62
Sociedade Brasileira de Matemática

 x  1998 a

 y  1998 b
 2
 ab 1998
com a observação de que os pares (x, y) solução devam ser inteiros e
positivos, devemos ter

a 19 80 a  19 8
 
b 19 80 b  19 8
logo, só servem a e b positivos, já que se –1998< a < 0 e –1998 < b < 0
implica ab < 19982. O número de soluções é, portanto, o número de divisores
positivos de 19982 = 22. 36. 372, que é dado por
(2 + 1) (6 + 1) (2 + 1) = 63.

Soluções dos problemas 2, 5 e 6 enviadas por Zoroastro Azambuja Neto.


Soluções dos problemas 3 e 12 enviadas por Carlos Alberto da Silva Victor.

EUREKA! N 3, 1998

63
Sociedade Brasileira de Matemática

Solução do problema 14 enviada por Vicente Wilson Moura Gaete e André Luiz
Arruda Marques. Continuamos esperando as soluções dos problemas 10, 11,
12, 13 e 15.

PROBLEMAS PROPOSTOS
 Convidamos o leitor a enviar soluções dos problemas propostos e sugestões de novos
problemas para os próximos números.

2
16) Seja l a reta {( x, y )  R y 0}, C1 o círculo centrado em
1 1 1 1
(0, ) de raio e C 2 o círculo centrado em (1, ) de raio .
2 2 2 2
Seja F o conjunto de círculos em R2 com as seguintes propriedades:
i) {C1, C2}  F
ii) Se C e C’ pertencem a F, são tangentes entre si e tangentes a l então
~
todo círculo C tangente aos dois círculos C e C’ e à reta l pertence
a F.
~
iii) Se F é um conjunto de círculos satisfazendo as propriedades i) e ii)
~
então F  F . Determine o conjunto dos pontos de tangência dos
círculos C  F com a reta l.

17) Dado n  N, uma partição  de n é uma lista ordenada


 (a1 , a2 ,...,ar ) , r , a1 , a2 ,..., ar  N * com a1  a2 ...  ar e a  a  ...  a  n.
1 2 r

Seja Pn o conjunto das partições de n. Para   Pn , definimos A()


como o número de termos iguais a 1 em 
( ou seja , A( ) 
 {i  {1,2,..., r} ai 1}), e B() como o número de termos
distintos na partição  (ou seja, B () =  {a1, a2, …, ar}).
Prove que  A( )   B ( ) para todo n  N.
 Pn  Pn
18) Seja  a maior raiz real da equação x3 – 3x2 + 1 = 0.
Prove que [2004] é divisível por 17.
Obs: [y] é o único inteiro tal que [y]  y < [y] + 1.

EUREKA! N 3, 1998

64
Sociedade Brasileira de Matemática

19) a) Determine o número máximo de regiões em que n retas podem


dividir o plano.

b) Determine o número máximo de regiões em que n planos podem


dividir o espaço.
COORDENADORES REGIONAIS

Alberto Hassen Raad (UFJF) Juiz de Fora-MG


Antônio C. Rodrigues Monteiro (UFPE) Recife-PE
Amarísio da Silva Araújo (UFV) Viçosa-MG
Angela Camargo (Centro de Educação
de Adultos CEA) Blumenau-SC
Antônio C. do Patrocínio (IMECC/UNICAMP) Campinas-SP
Ariosto de Oliveira Lima (UFPI) Parnaíba-PI
Benedito T. Vasconcelos Freire (UFRN) Natal-RN
Carlos A. Bandeira Braga (UFPB) João Pessoa-PB
Claudio Arconcher (Col. Leonardo da Vinci) Jundiaí-SP
Egnilson Miranda de Moura (Col. Agrícola do Bom Jesus) Bom Jesus-PI
Élio Mega (Col. ETAPA) São Paulo-SP
Florêncio F. Guimarães F. (UFES) Vitória-ES
Francisco Dutenhefner (UFMG ) BH-MG
Gisele de A. Prateado G. (UFGO) Goiânia-GO
Ivanilde H. Fernandes Saad (U. Católica Dom Bosco) Campo Grande-MS
João B. de Melo Neto (UFPI) Teresina-PI
João F. Melo Libonati (Grupo Educ. IDEAL) Belém-PA
Jorge Ferreira (UEM) Maringá-PR
José Carlos Pinto Leivas (URG) Rio Grande-RS
José Luis Rosas Pinho (UFSC) Florianópolis-SC
José Paulo Carneiro (USU) Rio de Janeiro-RJ
José Vieira Alves (UFPB) Campina Grande-PB
Leonardo Matteo D'orio (Parque de Material
Aeronáutico de Belém) Belém-PA
Licio Hernandes Bezerra (UFSC) Florianópolis-SC
Luzinalva M. de Amorim (UFBA) L. de Freitas-BA
Marco Polo (Colégio Singular) Santo André-SP
Marcondes Cavalcante França (UF Ceará) Fortaleza-CE
Mario Jorge Dias Carneiro (UFMG) BH-MG
Pablo Rodrigo Ganassim (L. Albert Einstein) Piracicaba-SP
Paulo H. Cruz Neiva de L. Jr. (Esc. Tec.Everardo Passos) S. J.Campos-SP
Reinaldo Gen Ichiro Arakaki (INPE) S.J.Campos-SP
Ricardo Amorim (Centro Educ. Logos) Nova Iguaçu-RJ
Sergio Claudio Ramos (IM-UFRGS) Porto Alegre-RS

EUREKA! N 3, 1998

65
Sociedade Brasileira de Matemática

Tadeu Ferreira Gomes (U. do Estado da Bahia) Juazeiro-BA


Valdenberg Araújo da Silva (U. Federal de Sergipe) São Cristovão-SE
Wagner Pereira Lopes (Esc. Tec. Fed. de Goiás) Jataí-GO

EUREKA! N 3, 1998

66
CONTEÚDO

AOS LEITORES 2

XX OLIMPÍADA BRASILEIRA DE MATEMÁTICA 3


Problemas e soluções da Primeira Fase

XX OLIMPÍADA BRASILEIRA DE MATEMÁTICA 14


Problemas e soluções da Segunda Fase

XX OLIMPÍADA BRASILEIRA DE MATEMÁTICA 22


Problemas e melhores soluções da Terceira Fase

XX OLIMPÍADA BRASILEIRA DE MATEMÁTICA 34


Resultados

ARTIGOS

PROBLEMAS ANTIGOS 37
Eduardo Wagner

O LOGOTIPO DA OLIMPÍADA BRASILEIRA DE MATEMÁTICA 42


Paulo Cezar Pinto Carvalho

SOLUÇÕES DE PROBLEMAS PROPOSTOS 47

PROBLEMAS PROPOSTOS 56

ERRATA 57

CARTAS DOS LEITORES 58

COMO ASSINAR A EUREKA! 59

AGENDA OLÍMPICA 60

COORDENADORES REGIONAIS 61
Sociedade Brasileira de Matemática

AOS LEITORES

Iniciamos este segundo ano da revista EUREKA! transmitindo nossa


satisfação pela acolhida no primeiro ano de vida da revista, de toda a
comunidade estudantil e dos diretores e professores dos colégios envolvidos.
Agradecemos a todos os que têm apoiado esta iniciativa e esperamos
continuar apoiando, através desta publicação, o trabalho dos professores.

Nesta nova edição da revista EUREKA! aproveitamos para registrar a


XX Olimpíada Brasileira de Matemática, da qual publicamos as provas da
primeira, segunda e terceira fases com soluções (que, esperamos, serão úteis
para a preparação para a XXI OBM), bem como as listas de premiados nos
três níveis. Nesta edição, também publicamos artigos de dificuldade
intermediária e material enviado por numerosos professores e alunos.
Esperamos seguir recebendo colaborações dos nossos leitores: soluções dos
problemas propostos, pequenos artigos e curiosidades matemáticas.

Devido ao interesse manifestado por diversas pessoas, criamos


recentemente as assinaturas individuais da revista EUREKA!. Para maiores
informações, veja página 59 desta edição. Por outro lado, estamos
planejando criar, nos próximos números, um pequeno espaço publicitário
ligado ao ensino da matemática, para o qual aguardamos propostas de
leitores, editoras e instituições de ensino. Desta forma, estaremos gerando
recursos que ajudarão a manter a publicação da revista.

Aproveitamos, por fim, para registrar que foi realizada em janeiro de


1999 a segunda Semana Olímpica, que reuniu premiados na XX Olimpíada
Brasileira de Matemática nos 3 níveis e professores de vários estados. A
atividade foi realizada em Maracanaú, Ceará, no centro de treinamento do
Colégio 7 de Setembro, ao qual gostaríamos de agradecer pelo apoio.

Comitê Editorial.

EUREKA! N°4, 1999


2
Sociedade Brasileira de Matemática

XX Olimpíada Brasileira de Matemática


Primeira Fase - Nível 1

01. Qual dos números a seguir é o maior?


A) 345 B) 920 C) 2714 D) 2439 E) 8112

02. Um menino joga três dados e soma os números que aparecem nas faces
voltadas para cima. O número dos diferentes resultados dessa adição é:
A)12 B) 18 C) 216 D) 16 E) 15

03. Renata digitou um número em sua calculadora, multiplicou-o por 3,


somou 12, dividiu o resultado por 7 e obteve o número 15. O número
digitado foi:
A) 31 B) 7 C) 39 D) 279 E) 27

04. Numa competição de ciclismo, Carlinhos dá uma volta completa na pista


em 30 segundos, enquanto que Paulinho leva 32 segundos para completar
uma volta. Quando Carlinhos completar a volta número 80, Paulinho estará
completando a volta número:
A) 79 B) 78 C) 76 D) 77 E) 75

05. Elevei um número positivo ao quadrado, subtrai do resultado o mesmo


número e o que restou dividi ainda pelo mesmo número. O resultado que
achei foi igual:
A) Ao próprio número B) Ao dobro do número
C) Ao número mais 1 D) À raiz quadrada do número
E) Ao número menos 1

06. Quantos números de 3 algarismos existem cuja soma dos algarismos é 25 ?


A) 2 B) 4 C) 6 D) 8 E) 10

07. João é mais velho que Pedro, que é mais novo que Carlos; Antônio é
mais velho do que Carlos, que é mais novo do que João. Antônio não é mais
novo do que João e todos os quatro meninos têm idades diferentes. O mais
jovem deles é:
A) João B) Antônio C) Pedro D) Carlos
E) impossível de ser identificado a partir dos dados apresentados

EUREKA! N°4, 1999


3
Sociedade Brasileira de Matemática

08. Escreva um número em cada círculo da fila abaixo, de modo que a soma
de três números quaisquer vizinhos (consecutivos) seja 12.

3 5

No último círculo à direita deve estar escrito o número:


A) 3 B) 2 C) 1 D) 4 E) 7

09. Dezesseis cubos de 1cm de lado são colocados juntos, formando o


paralelepípedo representado abaixo.

A superfície do mesmo foi pintada de verde e, em seguida, os cubos foram


separados. O número de cubos com exatamente duas faces verdes é:
A) 2 B) 6 C) 4 D) 8 E) 10

10. Uma fazenda retangular que possui 10 km de largura por 20 km de


comprimento foi desapropriada para reforma agrária. Se a fazenda deve ser
dividida para 200 famílias de modo que todas as famílias recebam a mesma
área, então cada família deve receber:
A) 1.000.000 m2 B) 100.000 m2 C) 5.000 m2 D) 1.000 m2
2
E) 10.000 m

11. Um estacionamento para carros cobra 1 real pela primeira hora e 75


centavos a cada hora ou fração de hora seguinte. André estacionou seu carro
às 11h 20min e saiu às 15h 40min. Quantos reais ele deve pagar pelo
estacionamento?
A) 2,50 B) 4,00 C) 5,00 D) 4,75 E) 3,75

12. Para fazer 12 bolinhos, preciso exatamente de 100g de açúcar, 50g de


manteiga, meio litro de leite e 400g de farinha. A maior quantidade desses
bolinhos que serei capaz de fazer com 500g de açúcar, 300g de manteiga, 4
litros de leite e 5 quilogramas de farinha é:
A) 48 B) 60 C) 72 D) 54 E) 42

EUREKA! N°4, 1999


4
Sociedade Brasileira de Matemática

13. Joãozinho brinca de formar quadrados com palitos de fósforo como na


figura a seguir.

A quantidade de palitos necessária para fazer 100 quadrados é:


A) 296 B) 293 C) 297 D) 301 E) 28

14. A soma de todos os números ímpares de dois algarismos menos a soma


de todos os números pares de dois algarismos é:
A) 50 B) 46 C) 45 D) 49 E) 48

15. O número que devemos somar ao numerador e subtrair do denominador


1478
da fração para transformá-la na sua inversa é:
5394
A) 3.916 B) 3.913 C) 3.915 D) 3.912 E) 3.917

16. O alfabeto usado no planeta X tem somente duas letras: X e x. O


sobrenome (nome de família) de cada um de seus habitantes é uma
seqüência formada por 4 letras. Por exemplo, xXxx é um possível sobrenome
utilizado nesse planeta. O maior número de sobrenomes diferentes que
podem ser dados no planeta X é:
A) 12 B) 14 C) 15 D) 16 E) 18

17. João quer desfazer-se de sua coleção de 1.000 bolinhas. Para tanto
escolhe dez garotos da rua onde mora. Dá ao primeiro garoto x bolinhas, ao
segundo x + 1 bolinhas. Assim faz até chegar ao décimo garoto. Sempre dá
uma bolinha a mais para o próximo garoto. No final, João ainda fica com um
resto de bolinhas. Sendo x o número que deixa João com o menor resto
possível, x é igual a:
A) 94 B) 95 C) 96 D) 97 E) 98

18. No planeta Z todos os habitantes possuem 3 pernas e cada carro possui 5


rodas. Em uma pequena cidade desse planeta, existem ao todo 97 pernas e
rodas. Então podemos afirmar:

A) É possível que existam 19 carros nessa cidade


B) Existem no máximo 16 carros nessa cidade

EUREKA! N°4, 1999


5
Sociedade Brasileira de Matemática

C) Essa cidade tem 9 habitantes e 14 carros


D) Essa cidade possui no máximo 17 carros
E) Nessa cidade existem mais carros do que pessoas

19. São dados um tabuleiro e uma peça, como mostra a figura.

De quantas maneiras diferentes podemos colocar a peça no tabuleiro, de


modo que cubra completamente 3 casas?
A) 16 B) 24 C) 36 D) 48 E) 60

20. Pedro e Maria formam um estranho casal. Pedro mente às quartas,


quintas e sextas-feiras, dizendo a verdade no resto da semana. Maria mente
aos domingos, segundas e terças-feiras, dizendo a verdade no resto da
semana. Certo dia, ambos dizem: ''Amanhã é dia de mentir''. O dia em que
foi feita essa afirmação era:
A) segunda-feira B) terça-feira C) sexta-feira D) sábado
E) domingo

Nível 2

1. Quantos são os números inteiros x que satisfazem à inequação


3 x  7?
A) 13 B) 26 C) 38 D) 39 E) 40

02. Hoje é sábado. Que dia da semana será daqui a 99 dias?


A) segunda-feira B) sábado C) domingo
D) sexta-feira E) quinta feira

03. Anulada.

04. Um pai tem 33 anos e seu filho, 7 anos. Depois de quantos anos a idade
do pai será o triplo da idade do filho?
A) 3 B) 7 C) 6 D) 9 E) 13

EUREKA! N°4, 1999


6
Sociedade Brasileira de Matemática

05. O quadrilátero ABCD é um quadrado de A B


área 4m2. Os pontos M e N estão no meio dos
lados a que pertencem. Podemos afirmar que a
área do triângulo em destaque é, em m2, N

D M C

A) 2 B) 1,5 C) 2,5 D) 3 E) 3,5

06. Qual é o dígito das unidades do número 31998?


A) 1 B) 3 C) 5 D) 7 E) 9

07. Num código secreto, as 10 primeiras letras do nosso alfabeto


representam os algarismos de 0 a 9, sendo que a cada letra corresponde um
único algarismo e vice-versa. Sabe-se que d + d = f, d . d = f, c + c = d, c + d
= a e a – a = b. Podemos concluir que a + b + c + d é igual a:
A) 0 B) 2 C) 4 D) 6 E) 8

08. O número 1234a6 é divisível por 7. O algarismo a vale:


A) 0 B) 2 C) 5 D) 6 E) 8

09. No trapézio abaixo, têm-se: AB paralelo a CD, AD = 10 cm e CD = 15


cm. O ângulo C mede 75º e o ângulo D, 30º . Quanto mede o lado AB, em
centímetros?

A) 5 B) 7,5 C) 10 D) 12,5 E) 5 3

10. No quadrado mágico abaixo, a soma dos números em cada linha, coluna
e diagonal é sempre a mesma. Por isso, no lugar do X devemos colocar o
número:

EUREKA! N°4, 1999


7
Sociedade Brasileira de Matemática

A)30 B) 20 C) 35 D) 45 E) 40

11. Passarinhos brincam em volta de uma velha árvore. Se dois passarinhos


pousam em cada galho, um passarinho fica voando. Se todos os passarinhos
pousam, com três em um mesmo galho, um galho fica vazio. Quantos são os
passarinhos?
A) 6 B) 9 C) 10 D) 12 E) 15

12. Pelo menos quantos metros de barbante são necessários para amarrar 15
pacotes, conforme a figura, sabendo que cada pacote mede 10cm  20cm 
40cm, sendo reservados 20cm para o laço?

A) 39 B) 36 C) 48 D) 56 E) 42

13. Para assistir ao filme Central do Brasil, cada um dos x alunos de uma
turma deveria pagar y reais pelo frete do ônibus. Como faltaram 3 alunos,
cada um dos alunos presentes teve que pagar 2 reais a mais para cobrir o
preço do frete. Qual foi esse preço?
A) (x + 3)(y – 2) B) (x – 3) y + 2 C) x(y + 2) – 3
D) xy – 6 E) (x – 3)(y + 2)

14. Seu Horácio resolveu incrementar a venda de CDs em sua loja e


anunciou uma liquidação para um certo dia, com descontos de 30% sobre o
preço das etiquetas. Acontece que, no dia anterior à liquidação, seu Horário
aumentou o preço marcado nas etiquetas, de forma que o desconto
verdadeiro fosse de apenas 9%. De quanto foi o aumento aplicado por seu
Horácio?
A) 30% B) 39% C) 21% D) 40% E) 31%

15. Um fabricante de brinquedos embala bolas de pingue-pongue em dois


tipos de caixas. Num dos tipos ele coloca 10 bolas e no outro coloca 24

EUREKA! N°4, 1999


8
Sociedade Brasileira de Matemática

bolas. Num certo dia foram embaladas 198 bolas e usadas mais de 10 caixas.
Quantas caixas foram feitas nesse dia?
A)14 B) 16 C) 15 D) 17 E) 11

16. Coloque em cada quadradinho, no desenho a seguir, os algarismos 1, 2,


3, 4 ou 5, de forma que cada um deles apareça pelo menos uma vez e que o
número formado seja o maior possível e múltiplo de 9.

No número que você construiu, o algarismo mais repetido apareceu:


A) 6 vezes B) 5 vezes C) 4 vezes D) 3 vezes E) 2 vezes

17. Observe as igualdades a seguir:


3 2  4 2 5 2
5 2  12 2 13 2
7 2  24 2 25 2
9 2  40 2 412
...
Considere a igualdade 17  x  y 2 com base nos exemplos anteriores,
2 2

procure determinar os números naturais x e y. Podemos concluir que x + y é


igual a:
A) 289 B) 121 C) 81 D) 144 E) 196

18. Você vai pintar a bandeira abaixo utilizando 4 cores: azul, verde, amarelo
e vermelho, uma em cada região.

Se o vermelho e o amarelo não podem ficar juntos, de quantas maneiras


pode ser pintada a bandeira?
A) 12 B) 4 C) 18 D) 20 E) 16

19. Um crime é cometido por uma pessoa e há quatro suspeitos: André,


Eduardo, Rafael e João. Interrogados, eles fazem as seguintes declarações:

•André: Eduardo é o culpado. •Eduardo: João é o culpado.


EUREKA! N°4, 1999
9
Sociedade Brasileira de Matemática

•Rafael: Eu não sou culpado. •João: Eduardo mente quando diz que eu
sou culpado.
Sabendo que apenas um dos quatro disse a verdade, quem é o culpado?
A) André. B) Eduardo. C) Rafael. D) João.
E) Não se pode saber.

20. Anulada.

Nível 3

01. Veja Problema 1 Nível 2. 02. Veja Problema 2 do Nível 2.


03. Veja Problema 5 do Nível 2. 04. Veja Problema 6 do Nível 2.
05. Veja Problema 15 do Nível 2.

06.- 0,4444... 
A) 0,2222… B) 0,3333… C) 0,4444… D) 0,5555… E) 0,6666…

07.- Veja Problema 8 do Nível 2.

08.- Todos os ângulos internos de um polígono convexo são menores que


(não podendo ser iguais a) 160. O número de lados desse polígono é, no
máximo, igual a:
A) 12 B) 14 C) 15 D) 17 E) 18

09.- A média aritmética de seis números é 4. Quando acrescentamos um


sétimo número, a nova média é 5. O número que foi acrescentado é:
A) 5 B) 6 C) 8 D) 10 E) 11

10. Veja Problema 19 do Nível 2.

11.- Em uma calculadora, a tecla A transforma o número x que está no visor


1
em e a tecla B multiplica por 2 o número que está no visor. Se o número
x
2 está no visor e digitamos a seqüência ABABABAB...AB (total de
digitações: 998), obteremos no visor um número que é igual a:
A) 1 B) 2-498 C) 2-500 D) 2499 E) 2500

EUREKA! N°4, 1999


10
Sociedade Brasileira de Matemática

12.- Um número inteiro n é bom quando 4n + 1 é um múltiplo de 5. Quantos


números bons há entre 500 e 1.000?
A) 50 B) 51 C)100 D) 101 E) 102

13.- Em um conjunto de pontos do espaço, a distância entre dois pontos


diferentes quaisquer é igual a 1. O número máximo de pontos que pode
haver nesse conjunto é:
A) 2 B) 3 C) 4 D) 6 E) 8

14.- Se x homens fazem x embrulhos em x segundos, em quantos segundos y


homens farão y embrulhos?
x2 y2 y
A) y B) x C) D) E)
y x x

15.- Você entra em um restaurante para comer pizza e espera pagar uma
quantia proporcional à quantidade de comida pedida. Se uma pizza com 20
cm de diâmetro custa R$ 3,60, quanto você espera pagar por uma outra do
mesmo sabor com 30cm de diâmetro?
A) R$ 5,40 B) R$ 5,80 C) R$ 6,60 D) R$ 7,50 E) R$ 8,10

16.- A função f associa a cada real x o menor elemento do conjunto


 15  x 
 x  1,  . O valor máximo de f(x) é:
 2 
A) 4 B) 5 C) 11/2 D) 16/3 E) 19/4

17.- Vendi dois rádios por preços iguais. Em um deles tive lucro de 25%
sobre o preço de compra e no outro tive prejuízo de 25%. Em relação ao
capital investido:
A) não tive lucro nem prejuízo B) lucrei 6,25%
C) lucrei 16% D) tive prejuízo de 6,25%
E) tive prejuízo de 16%

18.- A respeito da resposta de um problema, Maurício, Paulo, Eduardo e


Carlos fizeram as seguintes afirmações:
– Maurício: É maior que 5. – Paulo: É menor que 10.
– Eduardo: É um número primo. – Carlos: É maior que 12.
Entre as afirmações acima, quantas, no máximo, podem ser verdadeiras?
A) 0 B) 1 C) 2 D) 3 E) 4

EUREKA! N°4, 1999


11
Sociedade Brasileira de Matemática

1
19.- Os valores reais de x que satisfazem a inequação x 2 são:
x
A) –1  x  1 B) x = 1 C) x  1 D) x  1
E) x  2

20.- De quantos modos se pode colocar na tabela abaixo duas letras A, duas
letras B e duas letras C, uma em cada casa, de modo que não haja duas letras
iguais na mesma coluna?

A) 12 B) 24 C) 36 D) 48 E) 64

21.- Um viajante deveria caminhar durante uma hora num sentido entre o
norte e o leste, fazendo 30 0 com o norte. Atrapalhou-se e caminhou uma hora
num sentido entre o norte e o oeste, formando 30 0 com o norte. Para chegar
ao seu destino, ele deve agora tomar um rumo que faça com o norte um
ângulo de:
A) 00 B) 300 C) 450 D) 600 E) 900

22.- Barcas vão do Rio a Niterói em 25 minutos e lanchas fazem a viagem


em 15 minutos. A que horas a barca que partiu do Rio às 10h 01min é
alcançada pela lancha que saiu do Rio às 10h 07min?

A) 10h 15min B) 10h 16min C) 10h 17min D) 10h 18min


E)10h 20min

23.- Veja Problema 17 do Nível 2.

24.- A soma das raízes reais de x 3  3x 2  3x  1 0 é:


A) –3 B) 1  3 2 C) 1 D) 3 2  1 E) 3

25.- Dado um cubo, considere o conjunto de 27 pontos formado pelos


vértices desse cubo, pelos pontos médios de suas arestas, pelos centros de
suas faces e pelo centro do cubo. Quantas são as retas que passam por três
desses pontos?
A) 49 B) 54 C) 63 D) 81 E) 108

EUREKA! N°4, 1999


12
Sociedade Brasileira de Matemática

Respostas Nível 1:
01.- E 06.- C 11.- B 16.- D
02.- D 07.- C 12.- B 17.- B
03.- A 08.- A 13.- D 18.- D
04.- E 09.- D 14.- C 19.- C
05.- E 10.- A 15.- A 20.- B

Respostas Nível 2:
01.- D 06.- E 11.- B 16.- B
02.- C 07.- D 12.- B 17.- A
03.- Anulada 08.- D 13.- E 18.- A
04.- C 09.- A 14.- A 19.- C
05.- B 10.- B 15.- D 20.- Anulada

Respostas Nível 3:
01.- D 06.- E 11.- A 16.- D 21.- E
02.- C 07.- D 12.- C 17.- D 22.- B
03.- B 08.- D 13.- C 18.- D 23.- A
04.- E 09.- E 14.- B 19.- B 24.- D
05.- D 10.- C 15.- E 20.- D 25.- A

Você sabi@…
Que a Olimpíada Brasileira de Matemática tem uma lista eletrônica de
discussão de problemas de Matemática, aberta a todos os alunos e
professores interessados? Entre em contato conosco!!!


e-mail:obm@impa.br

EUREKA! N°4, 1999


13
Sociedade Brasileira de Matemática

XX OLIMPÍADA BRASILEIRA DE MATEMÁTICA


Segunda Fase - Nível 1

PROBLEMA 1
João comprou um livro e reparou que ele tinha 200 páginas. Seu irmão mais
novo arrancou ao acaso 25 folhas e somou os números das 50 páginas.
Explique porque o resultado desta soma não pode ser igual a 1998.
Atenção: cada folha tem duas páginas. A primeira folha tem as páginas 1 e 2,
a segunda folha tem as páginas 3 e 4, e assim por diante.

Solução
Como cada folha contém duas páginas tais que a soma dos seus respectivos
números é ímpar, ao adicionarmos todos esses 25 números, obteremos
necessariamente uma soma ímpar que, portanto, não pode ser igual a 1998.

PROBLEMA 2
1 1 1 1 1 1
Que frações devem ser retiradas da soma      para que a
2 4 6 8 10 12
soma das restantes seja igual a 1?

Solução
1 1 1 1 1 1 60 30 20 15 12 10
           (*)
2 4 6 8 10 12 120 120 120 120 120 120

Uma vez que 60 + 30 + 20 + 10 = 120, é claro que podemos remover


15 1 12 1
 e  . (além disso, vê-se claramente no lado direito da
120 8 120 10
27
igualdade (*) que não existem outros termos cuja soma seja igual a )
120
1 1
Assim, devemos remover e .
8 10

PROBLEMA 3
Encontre dois números de três algarismos cada um, usando cada um dos
dígitos 1, 2, 3, 4, 5, 6 exatamente uma vez, de forma que a diferença entre
eles (o maior menos o menor) seja a menor possível.

Solução
Para que a diferença seja a menor possível, os números devem ser os mais
próximos possíveis. Assim, os algarismos das centenas devem ser

EUREKA! N°4, 1999


14
Sociedade Brasileira de Matemática

consecutivos. A melhor escolha é aquela em que as dezenas formadas pelos


algarismos restantes tenham a maior diferença possível, o que ocorre para as
dezenas 65 e 12. Assim, os algarismos das centenas devem ser 3 e 4. O
menor número começado por 4 é 412 e o maior começado por 3 é 365, cuja
diferença é 47.

PROBLEMA 4
Existem casas em volta de uma praça. João e Pedro dão uma volta na praça,
caminhando no mesmo sentido e contando as casas. Como não começaram a
contar da mesma casa, a 5ª. casa de João é a 12ª. de Pedro e a 5ª. casa de
Pedro é a 30ª. de João. Quantas casas existem em volta da praça?

Solução
Sejam Jn e Pn respectivamente as n-ésimas casas de João e Pedro. De J5 a J30
exclusive, existem 30 – 5 – 1 = 24 casas. De P5 a P12 exclusive existem 12 –
5 – 1 = 6. Logo, no total existem 24 + 6 + 2 = 32 casas.

PROBLEMA 5
Existem 20 balas sobre uma mesa e duas crianças começam a comê-las, uma
criança de cada vez. Em cada vez, cada criança deve comer pelo menos uma
bala e está proibida de comer mais que a metade das balas que existem sobre
a mesa. Nesta brincadeira, ganha a criança que deixar apenas uma bala sobre
a mesa. Qual das duas crianças pode sempre ganhar na brincadeira: a
primeira ou a segunda a jogar? Como deve fazer para ganhar?

Solução
Ganha a primeira criança. No início ele deve comer 5 balas, deixando 15
balas sobre a mesa. A segunda criança deve comer no mínimo uma e no
máximo 7 balas, sobrando entre 8 e 14 balas sobre a mesa. Em qualquer caso
a primeira criança pode comer algumas balas, deixando exatamente 7 sobre a
mesa. A segunda criança agora deve comer entre uma e três balas, deixando
de 4 a 6 balas sobre a mesa. A primeira criança agora come algumas delas,
deixando exatamente 3 balas, forçando a segunda criança a comer uma.
Comendo mais uma após isso, a primeira criança acaba deixando apenas
uma bala no final e ganhando o jogo. De um modo mais geral, a estratégia
ganhadora consiste em deixar o adversário com 2 k – 1 balas, para algum k 
N. O adversário é obrigado a comer de 1 a ( 2 k  1  1) balas, deixando sobre
a mesa um número de balas que está sempre entre 2 k–1 e 2k – 2. O primeiro
jogador pode, então, jogar novamente de modo a deixar o adversário com

EUREKA! N°4, 1999


15
Sociedade Brasileira de Matemática

2 k  1  1 balas. O processo prossegue até o adversário ser reduzido a 2 1 – 1 =


1 bala.

PROBLEMA 6
Pintam-se de preto todas as faces de um cubo de madeira cujas arestas
medem 10 centímetros. Por cortes paralelos às faces, o cubo é dividido em
1.000 cubos pequenos, cada um com arestas medindo 1 centímetro.
Determine:
a) O número de cubos que não possuem nenhuma face pintada de preto.
b) O número de cubos que possuem uma única face pintada de preto.
c) O número de cubos que possuem exatamente duas faces pintadas de preto.
d) O número de cubos que possuem três faces pintadas de preto.

Solução
Estão sem nenhuma face pintada, os cubos interiores ao cubo maior. Portanto
devem ser retiradas uma fila de cima e uma fila de baixo, uma da frente e
outra de trás, e uma de cada lado, ficando assim com um cubo de aresta 8
que contém 83 = 512 cubos pequenos.

a) Estão com uma face pintada aqueles que pertencem a uma face mas
não possuem lado comum com a aresta do cubo maior, isto é, 8 2 = 64
em cada face. Como são seis faces, temos 6  64 = 384 cubos
pequenos.
b) Estão com duas faces pintadas aqueles que estão ao longo de uma
aresta mas não no vértice do cubo maior, isto é, 8 cubos em cada
aresta. Como são 12 arestas, temos 8  12 = 96 cubos pequenos.
c) Estão com 3 faces pintadas aqueles que estão nos vértices do cubo
maior, ou seja, 8 cubos pequenos.

Nível 2
PROBLEMA 1
1 1 1 1 1 1 1
Que frações devem ser retiradas da soma       para
2 3 4 6 8 10 12
que a soma das restantes seja igual a 1? Dê todas as soluções.

Solução

EUREKA! N°4, 1999


16
Sociedade Brasileira de Matemática

1 1 1 1 1 1 1 60 40 30 20 15 12 10
             (*)
2 3 4 6 8 10 12 120 120 120 120 120 120 120

Devemos escrever 120 como soma de algumas parcelas 60, 40, 30, 20, 15,
12, 10. As soluções possíveis são 60 + 40 + 20 = 120
60 + 30 + 20 + 10 = 120.
1 1 1 1 1 1 1
Assim, podemos remover , , e ou , e .
4 8 10 12 3 8 10
Evidentemente 15 e 12 não podem aparecer, pois a soma não seria múltipla
de 10 nesse caso.

PROBLEMA 2
Veja Problema 3 do Nível 1.

PROBLEMA 3
Cinco cartões numerados com 3, 4, 5, 6 e 7, respectivamente, são colocados
em uma caixa. Os cartões são retirados da caixa, um de cada vez e colocados
sobre a mesa. Se o número de um cartão retirado é menor do que o número
do cartão imediatamente anterior, então este cartão imediatamente anterior é
colocado de volta na caixa. O procedimento continua até que todos os
cartões estejam sobre a mesa. Qual é o número máximo de vezes que
retiramos cartões da caixa?

Solução
O número máximo de vezes que retiramos cartões da caixa é 15, o que
corresponde à seqüência de cartões retirados 7, 6, 5, 4, 3, 7, 6, 5, 4, 7, 6, 5, 7,
6, 7. De fato, dentre os primeiros 5 cartões há necessariamente um que é
menor que o cartão seguinte, e que portanto não voltará mais para a caixa, o
mesmo acontecendo para pelo menos um cartão dentre os 4 seguintes, depois
para pelo menos um dentre os 3 seguintes, depois para pelo menos um dentre
os dois seguintes, sobrando no máximo um cartão, que será o último a ser
retirado da caixa.

PROBLEMA 4
Em um triângulo acutângulo ABC o ângulo interno de vértice A mede 300.
Os pontos B1 e C1 são os pés das alturas traçadas por B e C, respectivamente
e os pontos B2 e C2 são médios dos lados AC e AB, respectivamente. Mostre
que os segmentos B1C2 e B2C1 são perpendiculares.

Solução

EUREKA! N°4, 1999


17
Sociedade Brasileira de Matemática

O segmento B1 C2 é uma mediana do A


triângulo AB1 B e portanto AC2 = B1 C2 e
C2 B̂1 A = B Â C = 30. 30

Daí BCˆ 2 B1  C 2 Bˆ1 A  BAˆ C  60 .


Analogamente, ACˆ 1 B 2  30 . Finalmente C2
C1Oˆ C 2 180  BCˆ 2 B1  ACˆ 1 B 2  90
 O  B2

C1 
 B1

B C
PROBLEMA 5
Veja Problema 5 do Nível 1.

PROBLEMA 6
Pintam-se de preto todas as faces de um cubo de madeira cujas arestas
medem n centímetros onde n  3. Por cortes paralelos às faces, o cubo é
dividido em n3 cubos pequenos, cada um com arestas medindo 1 centímetro.
Sabendo que o número total de cubos pequenos com exatamente uma face
pintada de preto é igual ao número de cubos pequenos apresentando todas as
faces sem pintura, determine o valor de n.

Solução
Um cubo pequeno que não possui qualquer face pintada provém do interior
do cubo grande. Isto significa que esse cubo pequeno é parte de um cubo de
lado n – 2, obtido quando retiramos uma unidade de cada face do cubo
original. Assim, existem (n – 2)3 cubos pequenos não pintados. Por outro
lado, um cubo pequeno com uma face pintada provém da face do cubo
original, mas não tendo qualquer parte da aresta deste cubo. Assim, existem
6(n – 2)2 cubos pequenos com face pintada. Portanto, (n – 2)3 = 6(n – 2)2,
com n > 2. Logo, n – 2 = 6, ou seja, n = 8.

Nível 3

PROBLEMA 1
Veja Problema 3 do Nível 2.

PROBLEMA 2
Veja problema 5 do Nível 1.

EUREKA! N°4, 1999


18
Sociedade Brasileira de Matemática

PROBLEMA 3
Uma reta que passa pelos pontos médios de dois lados opostos de um
quadrilátero convexo forma ângulos iguais com ambas as diagonais. Mostre
que as duas diagonais têm o mesmo comprimento.

Solução
Sejam ABCD o quadrilátero, M,N,P e Q os pontos médios dos lados AB,
BC, CD e DA, respectivamente. MN e PQ são paralelos à diagonal AC e
medem a metade de seu comprimento, enquanto NP e QM são paralelos à
diagonal BD e medem a metade de seu comprimento. Assim, MNPQ é um
paralelogramo. As condições do problema dizem que a reta que passa pelos
pontos médios de dois lados opostos de ABCD (digamos MP , sem perda de
generalidade) formam ângulos iguais com AC e BD , portanto com PQ e
NP , donde MP é bissetriz de NP ˆ Q . Logo MNPQ deve ser um losango,

donde MN  NP , e portanto AC BD (pois MN  AC / 2 e NP  BD / 2 ).

PROBLEMA 4
Sobre os lados AB e AC de um triângulo acutângulo ABC são construídos,
exteriormente ao triângulo, semicírculos tendo estes lados como diâmetros.
As retas contendo as alturas relativas aos lados AB e AC cortam esses
semicírculos nos pontos P e Q. Prove que AP = AQ.

Solução
Sejam M o pé da altura relativa ao lado AB. Como o triângulo APB é
retângulo em P, e PM é a altura de P em relação a AB temos
2
ˆ C.
AP  AB AM  AB AC cos BA Analogamente mostra-se que
2
ˆ C.
AQ  AC AB cos BA Portanto, AP  AQ.
PROBLEMA 5
Seja f : N  R uma função tal que
f(1) = 999 e f(1) + f(2) + ... + f(n) = n2f(n) para todo n inteiro positivo.
Determine o valor de f(1998).

Solução
Calculemos alguns valores de f(n):
2
f (1)  999; f (1)  f ( 2)  2  f ( 2)  3 f ( 2)  999  f ( 2)  333

EUREKA! N°4, 1999


19
Sociedade Brasileira de Matemática

2 333
f (1)  f ( 2)  f (3)  3  f (3)  8 f (3)  999  333  f (3) 
2
2 333 999
f (1)  f ( 2)  f (3)  f ( 4)  4  f ( 4)  15 f ( 4)  999  333   f ( 4)  Assi
2 10
999 999 999 999
m, temos f (1)  , f ( 2)  , f (3)  , f ( 4)  , e é razoável
1 3 6 10
999 1998
conjecturar que f (n) 
1  2  ...  n

n( n  1)
Para todo n  N. Vamos provar
esse fato: Para n  2 temos
f (1)  f (2)  ...  f ( n) n 2 f ( n)  ( n 2  1) f ( n)  f (1)  f ( 2)  ...  f (n  1) 
f (1)  f ( 2)  ...  f ( n  1) Por
f ( n)  .
n2  1
hipótese de indução,
1998 1998 1998
f (k ) 
k ( k  1)

k

k 1
, para k = 1, 2, …, n – 1, e portanto
1998
1998 1998 1998 1998 1998 1998
f (1)  f ( 2)  ... f ( n  1)      ...    
2 2 1 3 n 1 n 1
1998 1998( n  1) 1998( n  1) 1998
  f (n)   pois n 2  1 ( n  1)( n  1)),
n n 2
n ( n  1) n ( n  1)
como queríamos demonstrar.
1998 1
Fazendo n = 1998 temos f(1998) =  .
1998 1999 1999

PROBLEMA 6
O menor múltiplo de 1998 que possui apenas os algarismos 0 e 9 é 9990.
Qual é o menor múltiplo de 1998 que possui apenas os algarismos 0 e 3?

Solução
1998 = 2  999 = 2  33  37. Um número formado apenas pelos algarismos
0 e 3 é múltiplo de 33 se e somente se o número de algarismos 3 é múltiplo
de 9 (pois ao dividi-lo por 3 obtemos um número que possui apenas os
algarismos 0 e 1 que deve ser múltiplo de 9, o que ocorre se e só se o número
de algarismos 1 é múltiplo de 9 ). Assim, o número desejado deve ter pelo
menos 9 algarismos 3, e deve terminar por 0, por ser par. O menor número
com essas propriedades é 3333333330, que é múltiplo de 1998 pois é par, é
múltiplo de 33 e é múltiplo de 37 por ser múltiplo de 111 (é igual a 111 
30030030)).

EUREKA! N°4, 1999


20
Sociedade Brasileira de Matemática

Você sabia…
Que é possível (teoricamente) dividir uma bolinha de gude num
número finito de pedaços, remontá-los com movimentos rígidos e
obter uma bola (sem buracos) do tamanho da terra com uma
manada de elefantes em
cima ? Isso é conseqüência do chamado Paradoxo de Banach-
Tarski.

Você sabia…
Que o conjunto de valores positivos assumidos pelo polinômio
(k + 2){1– ( [wz + h + j – q]2 + [ (gk + 2g + k + 1)  (h + j) + h – z]2 +
[16(k + 1)3  (k + 2) (n + 1)2 + 1 – f 2]2 + [2n + p + q + z – e]2 +
[e3  (e + 2)  (a + 1)2 + 1 – o2]2 + [(a2 – 1) y2 + 1 – x2]2 +
[16r2y4  (a2– 1) + 1 – u2]2 + [((a + u2  (u2 – a))2 – 1)  (n + 4dy)2 +
1 – (x + cu)2]2 + [(a2 – 1)l 2 + 1 – m2]2 +[ai + k + 1 – l – i]2 + [n + l + v – y]2 +
[p + l (a – n – 1) + b (2an +2a – n2 – 2n – 2) – m]2 + [q + y (a – p – 1) +
s  (2ap + 2a – p2 – 2p – 2) – x]2 + [z +pl (a – p) + t (2ap – p2 – 1) – pm]2)}
quando as variáveis a, b, c, …, z assumem valores naturais é o conjunto
dos números primos ?

XX OLIMPIADA BRASILEIRA DE MATEMÁTICA


Problemas e melhores soluções da Terceira Fase

 Nível 1
PROBLEMA 1
Considere a tabela 3  3 abaixo, onde todas as casas, inicialmente, contém
zeros:

EUREKA! N°4, 1999


21
Sociedade Brasileira de Matemática

0 0 0

0 0 0

0 0 0

Para alterar os números da tabela, é permitida a seguinte operação: escolher


uma sub-tabela 2  2 formada por casas adjacentes, e somar 1 a todos os
seus números.

a) Diga se é possível, após uma seqüência de operações permitidas,


chegar à tabela abaixo:
7 9 2

15 25 12

8 18 10

b) Complete o quadro abaixo, sabendo que foi obtido por uma


seqüência de operações permitidas:
14

19 36

14

Solução de Fábio Dias Moreira.


Antes de resolver o problema, é preciso notar que existem quatro quadrados
2  2 no quadrado 3  3. Analisando-os, percebemos que os quadrados do
canto são afetados por apenas um deles. Com isso, deduzimos que cada
número nos quadradinhos do canto indica o número de vezes que a operação
permitida foi utilizada com o quadrado 2  2 que continha o quadradinho do
canto. Os quadradinhos do lado são afetados por dois quadrados diferentes,
assim como no diagrama abaixo.

EUREKA! N°4, 1999


22
Sociedade Brasileira de Matemática

1 3

1 1,3B 3
1 3
1,2A 3,4C
2 4
2 2,4D 4

4
2
Como a cada utilização da operação permitida, se for utilizada uma das sub-
tabelas escritas em um dos quadrados laterais, o quadradinho lateral
aumentará em um, concluímos que o valor do quadradinho lateral é igual à
soma dos dois quadradinhos da ponta adjacentes. O quadradinho central,
como é afetado por todos os quatro quadrados, é fácil deduzir que ele nada
mais é que o número de operações aplicados na tabela.

a) Como os números dos quadradinhos do canto representam quantas vezes


a sub-tabela que contém aquele quadradinho foi utilizada para fazer uma
operação e o quadradinho central o número de operações feitas,
podemos concluir que de acordo com o diagrama, A + B + C + D = E*.
Mas no nosso caso, temos 7 + 2 + 8 + 10 = 25, ou 27 = 25. Absurdo.
Concluímos então que é impossível obter esta tabela. Outra prova é que
a cada operação, aumentamos em um o valor de quatro casos, mantendo
o resultado divisível por quatro. No nosso caso, a soma dos números é
106, que não é divisível por 4.
A F B

G E I

C H D

b) No diagrama acima, com os raciocínios antes do problema, temos: G =


A + C, F = A + B, I = B + D e H = C + D. Portanto, no problema, 19 =
14 + C, onde C = 5. Com isso, 14 = 5 + D, onde D = 9. Prosseguindo
com base em *, 36 = 5 + 9 + 14 + B, onde B = 8. F = 8 + 14  F = 22.
Finalizando, 1 = 8 + 9 = 17. A tabela é

14 22 8

19 36 17

5 14 9

EUREKA! N°4, 1999


23
Sociedade Brasileira de Matemática

PROBLEMA 2
Encontre uma maneira de se escrever os algarismos de 1 a 9 em seqüência,
de forma que os números determinados por quaisquer dois algarismos
consecutivos sejam divisíveis ou por 7 ou por 13.

Solução de Andressa Rissetti Paim.


Múltiplos de 7 : 7, 14, 21, 28, 35, 42, 49, 56, 63, 70, 77, 84, 91, 98.
Múltiplos de 13 : 13, 26, 39, 52, 65, 78, 91.

Como nenhum dos múltiplos de 7 e 13, a não ser 77, terminava em 7, na


seqüência, 7 deveria ser o 1. número, então, o número formado foi
784913526.

5*
3
9 – 1*
6
5* 5 – 6*
2
1–3
7–8–4 9*
2–6
9–1–3–5
6*
Nota do editor: * significa que não dá para continuar sem repetir um número já
usado.

PROBLEMA 3
Em um jogo existem 20 buracos vazios em fila e o jogador deve colocar um
pino em cada buraco de acordo com as seguintes regras:

a) Se colocar um pino em um buraco e se os dois buracos vizinhos


estiverem vazios, o pino permanece.
b) Se colocar um pino em um buraco e se um dos buracos vizinhos
estiver ocupado, o pino deste buraco vizinho deve ser retirado.
c) Se colocar um pino em um buraco e se os dois buracos vizinhos
estiverem ocupados, então um dos pinos vizinhos deve ser retirado.

Determine qual é o número máximo de pinos que podem ser colocados.

EUREKA! N°4, 1999


24
Sociedade Brasileira de Matemática

Solução de Caio Magno Castro.


O número máximo é 19. Veja a explicação abaixo:
Começo colocando um pino no primeiro buraco da esquerda, pulo um
buraco e coloco outro pino. Depois eu coloco um pino no buraco que está
entre os dois e retiro o da direita. Depois pulo uma casa do segundo pino e
coloco um terceiro pino. Depois coloco um pino entre o terceiro e o segundo
pino e retiro o terceiro pino, o da direita. E faço essa operação
sucessivamente até chegar ao último pino.

PROBLEMA 4
Sete números naturais são escritos em círculo. Sabe-se que, em cada par de
números vizinhos, um deles divide o outro. Mostre que há dois números não
vizinhos com a mesma propriedade (isto é: um deles divide o outro).

Solução de Márcio Jun Hisamoto.

a
b
g
c
f
d
e

Em cada dois números adjacentes pelo menos um é múltiplo do outro; desse


modo é impossível fechar o círculo sem que algum número divida um outro
número que não seja adjacente a ele, pois se a for múltiplo de b e b for
múltiplo de c, então c divide a e já haverá dois números não vizinhos com a
propriedade. Se a for múltiplo de b e c for múltiplo de b e d, e for múltiplo
de d e q e o g não existisse, poderia não haver dois números não vizinhos
com a mesma propriedade mas como o g existe ele terá de ser múltiplo de f e
divisor de a. Desse modo f terá de ser divisor de a, sendo que com isso
haverá a propriedade.

 Nível 2
PROBLEMA 1
Prove que em qualquer pentágono convexo existem dois ângulos internos
consecutivos cuja soma é maior ou igual a 216.

Solução de Thiago Barros Rodrigues Costa.

EUREKA! N°4, 1999


25
Sociedade Brasileira de Matemática

Considere A, B, C, D e E os vértices do pentágono.


Suponha que não existam dois ângulos consecutivos cuja soma seja maior
ou igual a 216. Assim:
 
A B  216
  A
B  C  216 B
 
C  D  216
  E
C
D  E  216
 
E  A  216 D

        
Somando membro a membro: A B  B  C  C  D  D  E  A  5216
         
2 ( A  B  C  D  E )  1080  A  B  C  D  E  540 , mas a soma dos
    
ângulos internos de um pentágono é 540*. Logo A B  C  D  E 540
é absurdo. Então pelo menos 2 ângulos consecutivos tem soma maior ou
igual a 216.
*Lema: A soma dos ângulos de um pentágono é 540.
Prova: Seja P um ponto interior a ABCDE, logo P vai formar 5 triângulos no
pentágono, a soma de todos os ângulos P dos triângulos é 360. E a soma
dos ângulos que restam é justamente a soma dos ângulos de ABCDE. A soma
dos ângulos do pentágono é 5180 – 360 = 540.

PROBLEMA 2
No triângulo ABC, D é o ponto médio de AB e E o ponto do lado BC tal que
 
BE = 2  EC. Dado que os ângulos ADC e BAE são iguais, encontre o

ângulo BAC .
Solução de Daniel Pinheiro Sobreira.

B
x
y P
x
D
 W E
O
 x
y 2
 W W
90 –  90 – 
A C

EUREKA! N°4, 1999


26
Sociedade Brasileira de Matemática

Chamarei de P o ponto médio de BE . E chamarei BP = PE = EC = X. e


BD = DA = y.
A interseção de AE com DC , chamarei de O. O triângulo DOA é
isósceles, portanto AO  DO . Chamarei AO de W.
O segmento DP é a base média do triângulo ABE, pois D é o ponto médio de
AB e P é o ponto médio de BE , então DP // AE. Conseqüentemente os
triângulos OCE e DCP são semelhantes, na razão de 1/2. Então temos
CO
1 / 2 = D , 2 CO  CO  W D , W = CO.
CO  W
Chamarei o ângulo OÂD de ,  ADO, também será , e o  ADC, será 2,
pois é externo ao triângulo DOA. Como AO = W e OC = W o triângulo
ADC é isósceles e o ângulo da base é 90 – .
O ângulo BAC =  + 90 – , BÂC = 90.

PROBLEMA 3
Veja problema 3 do Nível 1.

PROBLEMA 4
São dados 15 números naturais maiores que 1 e menores que 1998 tais que
dois quaisquer são primos entre si. Mostre que pelo menos um desses 15
números é primo.

Solução de Humberto Silva Naves.

Teorema: Dado um número n, composto, então ele possui um fator (1)


menor ou igual à raiz quadrada deste número.
Prova: Se n = a  b, podemos ter ou a < n , a = n ou a > n :
1.- a= n
2.- a< n
n n
3.- a> n  b   n
a n
Em qualquer caso, temos um fator menor ou igual a n e diferente de 1.

Resposta: Dado 1 < n < 1998, se ele não for primo, ele tem que ter um fator
primo menor que 1998 , ou seja, um fator primo, menor que 45. Como só
existem 14 primos menores que 45, e são 15 números, então um desses não

EUREKA! N°4, 1999


27
Sociedade Brasileira de Matemática

terá fator primo menor que 45, logo será primo. (Pelo Corolário do teorema
anterior.)

   Nível 3
Primeira Prova.

PROBLEMA 1
Veja Problema 4 do Nível 2.

PROBLEMA 2
No triângulo ABC, D é o ponto médio de AB e E o ponto do lado BC tal que
BE = 2  EC. Dado que os ângulos AD ˆ C e BA ˆ E são iguais, encontre o
ˆ
ângulo BAC .

Veja a solução do Problema 2 do Nível 2.

Solução alternativa de Fabricio Siqueira Benevides.

A


y

D 
F
y

A 2x E x C

Seja F = AE  CD. Denotamos por XYZ a área do triângulo XYZ.


Veja que ABE = 2  ACE pois possuem a mesma altura relativa a BE e EC
respectivamente e DE = 2  EC. Analogamente BFE = 2  FEC. Temos:
ABE = 2  ACE (I)
BFE = 2  FEC (II)
I – II  ABE – BFE = 2  (ACE – FEC)  ABF = 2  AFC.

EUREKA! N°4, 1999


28
Sociedade Brasileira de Matemática

Só que: ABF = 2  ADF (mesma altura relativa a BA e DA


respectivamente e BA = 2  DA) Donde 2  ADF = 2  AFC  ADF =
AFC.
Como esses possuem a mesma altura relativa a DF e FC respectivamente,
temos: DF = FC   
F A C  A C F 
 
Mas  = A D C B A E  DF  AF . Daí, DF = AF = FC e CDA é

retângulo em A .

Prova: ADC :  +  +  +  +  = 180   +  = 90  DÂC = 90


Logo BÂC = 90 é o que queríamos achar.

PROBLEMA 3
Duas pessoas disputam um jogo da maneira descrita a seguir.
Inicialmente escolhem dois números naturais: n  2 (o número de rodadas) e
t  1 (o incremento máximo).
Na primeira rodada o jogador A escolhe um natural m1  0 e, posteriormente,
o jogador B escolhe um natural positivo n1  m1.
Para 2  k n, na rodada k o jogador A escolhe um natural mk com mk – 1  mk
 mk – 1  t e posteriormente o jogador B escolhe um natural nk com nk – 1  nk
 nk – 1  t. Após essas escolhas, nessa k-ésima rodada, o jogador A ganha
mdc (mk , nk – 1) pontos e o jogador B ganha mdc (mk , nk) pontos. Ganha o
jogo o jogador com maior pontuação total ao fim das n rodadas. Em caso de
pontuações totais iguais, o jogador A é considerado vencedor.
Para cada escolha de n e t, determine qual dos jogadores possui estratégia
vencedora.

Solução de Rui Lopes Viana Filho.

Nota: A solução usa a notação abreviada (m, n) = mcd(m,n).


Seja S um super-número. Esse super-número é divisível por todos os
naturais.
Para B ganhar sempre, basta fazer nk = S + mk. Pois, na rodada k, B ganha
(nk, mk) = mk pontos e A ganha (S + mk–1, mk) pontos. Assim A ganha no
máximo mk pontos, temos que mkS + mk–1  mkmk–1, mas mk > mk–1. Assim

EUREKA! N°4, 1999


29
Sociedade Brasileira de Matemática

A nunca ganhará mk pontos. Ganhará sempre menos que mk. Portanto B no


total ficará com mais pontos que A, já que ganha mais em todas as rodadas.
Obs. É claro que não existe um super-número, mais para cada n, t e m1, B
pode criar um número, que não chega a ser super, mas também serve. Basta
fazer:
S = ( 235711…p)M, com  e M suficientemente grandes!, Assim B
sempre ganha!
Explicando melhor:
Após A escolher m1, B sabe que o maior número que A pode escolher é
m1 + (n – 1) t
B só precisa fazer
m1 ( n  1)t
S j m1
j e nk S  m k
Justificativa:
m1 < mi  m1 + (n – 1) t  miS
Na rodada k:
 B ganha (mk, nk) = mk, já que mkS + mk

 A ganha (mk, nk – 1) = (mk, S + mk – 1)

Como

mk  mk1  mk não divide mk1


 
   mk não divide (mk1  S)  mk  (mk ,nk1) Portanto, a cada rodada B

m S m S
k k
ganha mais pontos que A, e portanto B ganha o jogo.
Obs. mk – 1 < mk  mk – 1 + t  S + mk – 1 < S + mk  S + mk – 1 + t 
 nk –1 < nk  nk – 1 + t
Resposta: B sempre ganha (se for esperto)!!!

EUREKA! N°4, 1999


30
Sociedade Brasileira de Matemática

   Nível 3
Segunda Prova.

PROBLEMA 4
Dois meninos jogam o seguinte jogo. O primeiro escolhe dois números
inteiros diferentes de zero e o segundo monta uma equação do segundo grau
usando como coeficientes os dois números escolhidos pelo primeiro jogador
e 1998, na ordem que quiser (ou seja, se o primeiro jogador escolhe a e b o
segundo jogador pode montar a equação 1998x2 + ax + b = 0, ou
bx2 + 1998x + a = 0, etc.) O primeiro jogador é considerado vencedor se a
equação tiver duas raízes racionais diferentes.
Mostre que o primeiro jogador pode ganhar sempre.

Solução de Fabricio Siqueira Benevides.


Inicialmente veja que, se num polinômio p(x) = anxn + … + a1x + a0, tivermos
an + … a1 + a0 = 0, teremos p (1) = 0 e 1 é raiz de p. Dessa forma se o
primeiro jogador escolhe b = – (1998 + a), 1 será raiz da equação do
segundo grau que o seu oponente irá montar.
Se um polinômio tem coeficientes inteiros (na verdade vale para coeficientes
racionais) e possui uma raiz irracional do tipo a + b r (r não é quadrado
perfeito), então a – b r também é raiz. Ou seja, as raizes irracionais vêm
aos pares. No caso de uma equação de segundo grau, e coeficientes inteiros,
ambas as raizes são irracionais, ou ambas são racionais.
No nosso caso, como 1 já é raiz, a outra raiz será racional. Basta ver então,
apenas se 1 não é raiz multipla (pois queremos que as raizes sejam distintas).
Para isso basta escolher a adequadamente.
Se o primeiro jogador escolher os números a = n1998, e b = – (n + 1) 1998,
n  2  N, ele ganha. (1 não será raiz múltipla e a equação terá duas raizes
racionais distintas.)

Obs. É possivel obter soluções com a + b + 1998  0, por exemplo com


{a, b} = {2040, – 5478} (solução obtida com o auxílio de um computador.)

PROBLEMA 5
Determine todas as funções f : N  N que satisfazem
f (2 f ( x)) x  1998 para todo x  N {0,1,2,...}.

Solução de Emanuel Augusto de Souza Carneiro.

EUREKA! N°4, 1999


31
Sociedade Brasileira de Matemática

1° Passo. f é injetiva.
Prova: Suponha que exista k  N tal que f (x) = f (y) = k para x, y  N e
x  y. Daí f(x) = f(y)  2f(x) = 2 f(y)  f(2f(x)) = f (2f(y))  x + 1998 =
y + 1998  x = y. Absurdo! Logo f é injetiva.

2° Passo. f e sobrejetiva no contradomínio{k N  k  1998}, isto é, para


todo k  N, k  1998    N tal que f() = k.
Prova: Se k  1998 faça k = x + 1998 para x  0 e x  N, logo
f(2f(x)) = x + 1998 = k
Tome então 2f(x) =   N.
Logo f é sobrejetiva no contradomínio acima.
3° Passo. Então para todo k  A = {k Nk  1998} existe um único
natural x (que chamaremos f ' (k) = x) tal que f(x) = k.
Obviamente existe um natural i ímpar tal que f (i) = w  1998 pois não
podemos ter 0  f(i)  1997 para todos os naturais ímpares porque a função f
é injetiva e de N em N (princípio de casa de pombos). Para esse w teremos
f '(w) = i, pois f (i) = w. Por outro lado faça w = a + 1998 com a  N Pela
definição: f (2f(a)) = a + 1998 = w. Logo f '(w) = 2f (a).
Como f '(w) é único, pela conclusão 1  i = 2f(a) como f(a)  N  i é par,
absurdo!
Conclusão 2. Não existe função desse tipo.

PROBLEMA 6
Dois matemáticos, perdidos em Berlim, chegam à esquina da rua Barbarossa
com a rua Martin Luther, e precisam chegar à esquina da rua Meininger com
a rua Martin Luther. Infelizmente eles não sabem para que lado fica a rua
Meininger, nem a que distância ela está, logo são obrigados a ir e voltar ao
longo da rua Martin Luther até chegaram à esquina desejada.
Qual é o menor valor para o número positivo K tal que eles podem ter
certeza de que se há N quarteirões (ou quadras) entre as ruas Barbarossa e
Meininger então eles conseguem chegar ao destino andando no máximo KN
quarteirões (ou quadras)?

Solução (dos matemáticos perdidos em Berlim)


Este problema é baseado numa situação real, ocorrida com os matemáticos
Nicolau Saldanha e Carlos Gustavo Moreira, que se encontravam em
Berlim por ocasião do Congresso Internacional de Matemática de 1998. Era
de noite, não havia ninguém na Martin-Luther-Strasse a quem pedir
informações e eles queriam chegar rápido ao destino. A idéia do problema

EUREKA! N°4, 1999


32
Sociedade Brasileira de Matemática

era que eles andassem juntos. Apareceram soluções em que os matemáticos


se separavam (o que não havia sido previsto), as quais foram avaliadas caso
a caso. Como os matemáticos não sabiam para que lado nem a que distância
estava a Meiningerstrasse, deviam adotar uma estratégia do seguinte tipo:
andar a1 quarteirões para um lado (digamos o direito), depois voltar ao ponto
inicial e andar a2 quarteirões para a esquerda, depois a3 para a direita, depois
a4 para a esquerda e assim sucessivamente, onde a1, a2, a7…são números
inteiros positivos com a1 < a3 < a5 < … e a2 < a4 < a6 <… até encontrar a
Meiningerstrasse. Os piores casos são quando a Meininger está a a2k + 1 + 1
quarteirões à direita ou a2k + 1 à esquerda da Barbarossastrasse, com k
natural (convencionamos a0 = 0).
Nesses casos, temos que entre o ponto inicial e o destino há an + 1
quarteirões, e os matemáticos andam no total 2a1 + 2a2 + …+ 2an + 2an + 1 +
an + 1 quarteirões até chegarem ao destino (com n = 2k+1 ou n = 2k). Assim,
devemos ter 2a1 + 2a2 + … + 2an + 2an + 1 + an + 1  k (an + 1), ou seja,
 k  1
S n 1   a n  1 para todo n  N, onde Sn = a1 + a2 + … + an.
 2 
Para k = 9 existem estratégias que satisfazem as condições do problema, por
exemplo tomando am = 2m para todo m  N. De fato, teremos Sn + 1 = 21 + 22
k1
+… + 2n + 1 = 2n + 2 – 2 < 4 (2 n  1)  (a n  1) para todo n  N.
2
k 1
Mostraremos que 9 é o menor k possível. Seja k < 9. Então c = é
2
menor que 4. Se k satisfaz as condições do problema, deve haver uma
sequência (an) como acima com Sn + 1  c (an + 1) para todo n  N.
Como an = Sn – Sn –1 teremos Sn + 1  c (Sn – Sn – 1 + 1) para todo n  N.
Definimos Un = Sn – c, temos Un + 1  c (Un – Un – 1) para todo n  N. Como c
< 4, Un > 0 para todo n  3, e, definindo Vn = Un + 1 /Un para todo n  3,
teremos Vn  c (1 – 1/Vn – 1) para todo n  4, onde Vn > 0 para todo n  3.
Entretanto, Vn  c (1 – 1/Vn – 1) implica Vn – Vn – 1  c ( 1 – 1/Vn –1) – Vn – 1 =
2
c2 c 
2  c    Vn 1 
cVn  1  c  V n 1 4 2  c(c  4)
   0  Vn  V n  1
Vn  1 Vn  1 4Vn  1
para todo n  4. Por outro lado, para todo n  4 temos
c ( c  4) c ( c  4)
Vn  Vn  1   Vn  Vn  1  para todo n  4, donde
4V n  1 4V3

EUREKA! N°4, 1999


33
Sociedade Brasileira de Matemática

( n  3) c (c  4)
V n V3  para todo n  4, absurdo, pois o lado direito é
4V3
2
4V3
negativo para n > 3 + 
c (4  c)
XX OLIMPÍADA BRASILEIRA DE MATEMÁTICA
Resultado - Primeiro Nível (5a. e 6a. Séries)

Nome Cidade - Estado Prêmio


Márcio Jun Hisamoto São Paulo - SP Ouro
Fábio Dias Moreira R. de Janeiro - RJ Ouro
Andressa Rissetti Paim Santa Maria - RS Prata
Henry Wei Cheng Hsu São Paulo - SP Prata
Natália Argene Lovate Pereira Jundiaí - SP Prata
Daniel Cherobini Santa Maria - RS Prata
Luis Eduardo de Godoi SJ dos Campos - SP Prata
Milton Eiji Kato São Paulo - SP Prata
Fabrício Henrique de Faria São Paulo - SP Prata
Davi Máximo Alexandrino Nogueira Fortaleza - CE Prata
Caio Magno Castro de Paula Fortaleza - CE Prata
Bruno Moreira de Souza Días SJ dos Campos - SP Prata
Patrícia Akemi Komura São Paulo - SP Bronze
Marcelo Li Koga São Paulo - SP Bronze
Alberto Hikaru Shintani São Paulo - SP Bronze
Diego Gomes Gripp Vitória - ES Bronze
Renato Mendes Coutinho Americana - SP Bronze
Antônio Monteiro Guimarães Jr. Campina G - PB Bronze
Leonardo Luis Desideri Freitas Vitória - ES Bronze
Helder Seiji Kato São Paulo - SP Bronze
Aline Galvão São Paulo - SP Bronze
Rodrigo Miyashiro Nunes dos Santos São Paulo - SP Bronze
João Marcos da Cunha Silva Fortaleza - CE Bronze
Thiago Mizuta São Paulo - SP Bronze
Oliveiro Ribeiro Barbosa Jr. Teresina - PI Bronze
Jorge Peixoto Goiânia - GO Menção Honrosa
Andréia Lúcio de Castro Goiânia - GO Menção Honrosa
Léo Jaime Zandonai BentoGonçalves - RS Menção Honrosa
Pedro Junqueira de Barros São Paulo - SP Menção Honrosa
Lucas Ikeda França São Paulo - SP Menção Honrosa
Paulo Roberto Sampaio Santiago Salvador - BA Menção Honrosa

EUREKA! N°4, 1999


34
Sociedade Brasileira de Matemática

Rafael Marini Silva Vila Velha - ES Menção Honrosa


Eduardo Souza Cruz Vitória - ES Menção Honrosa
Breno Ignácio da Silva Sertãozinho - SP Menção Honrosa
Diogo dos Santos Suyama Belo H. - MG Menção Honrosa
Flavio Schiavini Abe Vitória - ES Menção Honrosa

Resultado - Segundo Nível (7a. e 8a. Séries)

Nome Cidade - Estado Prêmio


Thiago Barros Rodrigues Costa Fortaleza - CE Ouro
Humberto Silva Naves Goiânia - GO Ouro
Daniel Pinheiro Sobreira Fortaleza - CE Ouro
Afonso de Paula Pinheiro Rocha Fortaleza - CE Prata
Hugo Pinto Iwata SJ de Rio Preto - SP Prata
Thiago da Silva Sobral Fortaleza - CE Prata
João Alfredo Castellani Fajardo Freire Salvador - BA Prata
Artur Duarte Nehmi São Paulo - SP Prata
Einstein do Nascimento Jr. Fortaleza - CE Prata
Daniel Pessôa Martins Cunha Fortaleza - CE Prata
Gustavo Alonso Daud Patavino Santos - SP Prata
Rafael de Holanda Barroso Fortaleza - CE Bronze
Eduardo Kunio Kuroda Abe São Paulo - SP Bronze
Renata Lourenção Delamanha Jundiaí - SP Bronze
Ricardo de Castro Palácio Fortaleza - CE Bronze
Helen Wei Ling Hsu São Paulo - SP Bronze
Frederico Pinto São Paulo - SP Bronze
Eduardo Famini Silva Salvador - BA Bronze
Victor Marchesini Ferreira Salvador - BA Bronze
Thalita Basso Jundiaí - SP Bronze
Eduardo Suaiden Klein R. de Janeiro - RJ Menção Honrosa
Mauricio Massao Soares Matsumoto São Paulo - SP Menção Honrosa
Claudia Giacomin Bof Aracruz - ES Menção Honrosa
Guilherme Silveira Barrozo Netto R. de Janeiro - RJ Menção Honrosa
Thiago Araújo Fiorio Fortaleza - CE Menção Honrosa
Pedro Fernando Almeida Di Donato SJ dos Campos - SP Menção Honrosa
Gustavo Modenesi São Paulo - SP Menção Honrosa
Jaquellyne Gurgel Penaforte Fortaleza - CE Menção Honrosa
Jordan Guimarães Lombardi SJ dos Campos - SP Menção Honrosa
Tiago Monteiro Fernandes Piracicaba - SP Menção Honrosa
Marcelo Loula Novais de Paula Salvador - BA Menção Honrosa

EUREKA! N°4, 1999


35
Sociedade Brasileira de Matemática

Resultado - Terceiro Nível (Ensino Médio)

Nome Cidade - Estado Prêmio


Emanuel Augusto de Souza Carneiro Fortaleza - CE Ouro Prêmio especial
Mauricio Pereira Carrari São Paulo - SP Ouro
Rui Lopes Viana Filho São Paulo - SP Ouro
Fernando Paz Cardoso São Paulo - SP Ouro
Fabricio Siqueira Benevides Fortaleza - CE Ouro
Tony Calleri França Fortaleza - CE Prata
Jônathas Diógenes Castello Branco Fortaleza - CE Prata
Glauf Sidney Duarte Moreira Jr. Fortaleza - CE Prata
Sergio Alvarez Araújo Correia Fortaleza - CE Prata
Sérgio Tadao Martins São Paulo - SP Prata
Lucas Heitzmann Gabrielli São Paulo - SP Bronze
Christian Iveson São Paulo - SP Bronze
Bruno Gurgel Fernandes Távora Fortaleza - CE Bronze
Mila Lopes Viana São Paulo - SP Bronze
Daniel Massaki Yamamoto São Paulo - SP Bronze
Daniele Véras de Andrade Fortaleza - CE Bronze
Mauricio Masayuki Honda São Paulo - SP Bronze
Leonardo Cardoso Souza Angra dos Reis - RJ Bronze
Daniel Nobuo Uno São Paulo - SP Bronze
Daniel Mourão Martins Fortaleza - CE Bronze
João Paulo de Tarso Ferreira Angra dos Reis - RJ Bronze
Evandro Makiyama de Melo São Paulo - SP Bronze
Christian Lyoiti Watanabe Angra dos Reis - RJ Bronze
Fred Olavo A. Carneiro Fortaleza - CE Menção Honrosa
Bruno Da Cunha Raymundo Rio de Janeiro - RJ Menção Honrosa
Pedro Paulo de Simoni Gouveia Fortaleza - CE Menção Honrosa
Carlos Alexandre Rolim Fernandes Fortaleza - CE Menção Honrosa
Márcio Afonso Assad Cohen Rio de Janeiro - RJ Menção Honrosa
Matheus de Lima Faheina Fortaleza - CE Menção Honrosa
Rodrigo M. Gorgoll São Paulo - SP Menção Honrosa

EUREKA! N°4, 1999


36
Sociedade Brasileira de Matemática

Vítor Menezes Santana Goiânia - GO Menção Honrosa


Alexandre Ferreira Terezan Rio de Janeiro - RJ Menção Honrosa
Ricardo Sallai Viciana São Paulo - SP Menção Honrosa
Remo H. de M. Furtado Fortaleza - CE Menção Honrosa
Wayne Leonardo Silva de Paula Belém - PA Menção Honrosa
Thiago Steiner Alfeu Fortaleza - CE Menção Honrosa
Márcio Reis Lopes Salvador - BA Menção Honrosa
Seong Ho Lee Santo André-SP Menção Honrosa

PROBLEMAS ANTIGOS
Eduardo Wagner
 Nível Intermediário.

Você sabe quando foi realizada a primeira Olimpíada de


Matemática?
Foi no ano de 1894, na Hungria. Neste ano, a Sociedade de
Matemática e Física da Hungria promoveu uma competição de Matemática,
envolvendo todos os alunos dos últimos anos das escolas, para homenagear
seu presidente Loránd Eötvös, eleito ministro da educação do país. O evento
foi um sucesso, e passou a ser realizado todos os anos.
Vamos mostrar neste artigo alguns problemas dessas competições
com suas soluções resumidas. Os problemas escolhidos não são muito
difíceis, mas são bastante interessantes. Recomendo aos leitores pensar um
pouco em cada um deles antes de ver a solução. As ferramentas exigidas são
elementares (apenas no problema 2 a noção de congruência é adequada) mas
as soluções necessitam de uma certa dose de criatividade.
Aproveitem!

PROBLEMA 1 – Olimpíada de 1894


Prove que as expressões 2x + 3y e 9x + 5y são divisíveis por 17 para os
mesmos pares de valores dos inteiros x e y.

PROBLEMA 2 – Olimpíada de 1898


Determine todos os valores do natural n, para os quais 2 n  1 é múltiplo de
3.

PROBLEMA 3 – Olimpíada de 1905


Na figura a seguir, AM, BN e CP são paralelos.

EUREKA! N°4, 1999


37
Sociedade Brasileira de Matemática

A C B

Prove que
1 1 1
 
AM BN CP

PROBLEMA 4 – Olimpíada de 1906


A seqüência a1, a2, a3, …, an representa uma arrumação arbitrária dos
números 1, 2, 3, …, n. Prove que se n é um número ímpar o produto

(a1 – 1)(a2 – 2)(a3 – 3) … (an – n)


é um número par.

PROBLEMA 5 – Olimpíada de 1910


Se a, b, c são números reais tais que a 2  b 2  c 2 1 , prove que

1
  ab  bc  ca 1
2

PROBLEMA 6 – Olimpíada de 1913


Prove que para todo natural n > 2, tem-se 1 2 3 n  2  n n .

PROBLEMA 7 – Olimpíada de 1916


No triângulo ABC, AD é a bissetriz do ângulo A. Prove que AD  AB AC
.

PROBLEMA 8 – Olimpíada de 1916


Divida os números 1, 2, 3, 4, 5 em dois conjuntos quaisquer. Prove que um
dos conjuntos contém dois números e sua diferença.

SOLUÇÕES

EUREKA! N°4, 1999


38
Sociedade Brasileira de Matemática

PROBLEMA 1
Observe que 4(2x + 3y) + (9x + 5y) = 17(x + y). Portanto, se 2x + 3y for
múltiplo de 17, então 9x + 5y também será, e vice versa.

OBS: Esta aparente “mágica” não é a única forma de resolver este problema.
Os leitores que conseguirem outra solução (para este ou para qualquer
problema deste artigo) podem enviar suas descobertas para publicação nos
próximos números da EUREKA!
PROBLEMA 2
A solução mais natural para este problema utiliza congruências. Observe que
2  (–1) mód.3. Logo, 2n  (–1)n mód.3 e, portanto, 2n + 1  (–1)n + 1 mód.3.
Concluimos então que , 2n + 1  0 mód.3 se, e somente se, n é ímpar.

OBS: O leitor familiarizado com indução pode conseguir outra solução.

PROBLEMA 3
N

M
P

A C B

Utilizando semelhança de triângulos na figura acima temos:

CP CB

AM AB

CP AC

BN AB
Somando temos:
CP CP AC  CB
  1
AM BN AB
Daí,
1 1 1
 
AM BN CP

EUREKA! N°4, 1999


39
Sociedade Brasileira de Matemática

PROBLEMA 4
O produto (a1 – 1)(a2 – 2)(a3 – 3) … (an – n) possui um número ímpar de
termos porque n é ímpar. Mas, a soma desses termos é zero, que é par. Como
a soma de uma quantidade ímpar de números ímpares não pode ser par,
concluímos que um dos termos é par e, conseqüentemente, o produto é um
número par.

PROBLEMA 5
Primeira parte:
 a  b  c 2 0
2 2 2
a  b  c  2ab  2bc  2ac 0
1  2ab  bc  ca  0
1
ab  bc  ca 
2
Segunda parte:
 a  b  2   b  c  2   c  a  2 0
2 a 2  b 2  c 2   2 ab  bc  ca  0
1   ab  bc  ca  0
ab  bc  ca 1
PROBLEMA 6
A expressão do lado esquerdo da desigualdade pode ser escrita assim:

1.n.2.(n – 1).3.(n – 2). … .(n – 2).3.(n – 1).2.n.1

Considere agora separadamente os produtos:

1.n, 2.(n – 1), 3.(n – 2), … , (n – 2).3, (n – 1).2, n.1

O primeiro e o último são iguais a n, mas afirmamos que qualquer um dos


outros é maior que n. De fato, os produtos “do meio” são da forma (k + 1)(n
– k) onde k assume os valores: 0, 1, 2, …, n – 1. Como para eles, n – k é
maior que 1, temos que

(k + 1)(n – k) = k(n – k) + (n – k) > k.1 + (n – k) = n

EUREKA! N°4, 1999


40
Sociedade Brasileira de Matemática

Logo, como n é maior que 2, o produto do lado esquerdo é maior que n.n.n.
… .n = nn.

PROBLEMA 7
Considere a circunferência circunscrita ao triângulo ABC.
A

D
B C

E
A bissetriz AD encontra a circunferência em E, ponto médio do arco BC.
Como os ângulos ABC e AEC são iguais (cada um deles vale a metade do
arco AC) e como os ângulos BAE e EAC são também iguais (porque AD é
uma bissetriz), concluimos que os triângulos ABD e AEC são semelhantes.
Daí,
AB AD

AE AC
ou seja,
AD AE  AB AC
Como AD é menor que AE temos que
AD AD  AB AC
ou seja,
AD  AB AC

PROBLEMA 8
Vamos tentar dividir 1, 2, 3, 4, 5, em dois conjuntos tais que nenhum deles
contém a diferença de dois de seus elementos. O 2 não pode estar no mesmo
conjunto que o 1 ou o 4 porque 2 – 1 = 1 e 4 – 2 = 2. Portanto, vamos
colocar o 2 em um conjunto e o 1 e o 4 no outro. O 3 não pode ficar no
segundo conjunto porque 4 – 3 = 1. Logo, o 3 deve ficar no primeiro
conjunto, junto com o 2. Agora, o 5 não pode ficar no primeiro conjunto
porque 5 – 3 = 2, e nem pode ficar no segundo porque 5 – 4 = 1.

EUREKA! N°4, 1999


41
Sociedade Brasileira de Matemática

A divisão proposta é portanto impossível.

O LOGOTIPO DA OLIMPÍADA BRASILEIRA DE MATEMÁTICA


Paulo Cezar Pinto Carvalho
IMPA
 Nível Intermediário.

Você já prestou atenção ao logotipo da Olimpíada Brasileira de


Matemática, presente na capa da EUREKA! e (em sua versão animada) na
página da Internet da OBM? Os círculos coloridos são uma referência ao
símbolo dos Jogos Olímpicos, que é formado por 5 anéis entrelaçados
representando os continentes. No logotipo da OBM, porém, estes anéis
estão dispostos de um modo tal que conhecimentos matemáticos são
essenciais para sua construção. O que existe de difícil em dispor cinco anéis
de modo que cada um seja tangente a dois outros e, além disso, tangente a
dois círculos adicionais, um interior e outro exterior? Vejamos.

Tomemos dois círculos arbitrários, um contido no outro e


posicionemos um novo círculo, de modo a ser tangente a ambos. A partir
daí, os demais círculos estão definidos e a Fig. 1 mostra o que ocorre no caso
geral: quando tentamos colocar o último círculo, vemos que a figura não
fecha, ou seja, não é possível colocar um quinto círculo tangente a dois dos
quatro círculos já colocados e aos dois iniciais.

Fig. 1 - O quinto círculo não se encaixa

Será que é possível colocar o primeiro círculo colorido em outra


posição, de modo a fazer com que a figura se feche exatamente? Pode-se ter
uma idéia da resposta a esta pergunta observando a versão animada do

EUREKA! N°4, 1999


42
Sociedade Brasileira de Matemática

logotipo. Observe que os círculos interno e externo são fixos, mas os


coloridos assumem tamanhos e posições variáveis e parecem girar em torno
deles (veja a Fig. 2 a seguir). Ou seja, a animação sugere que o fechamento
da figura não depende da posição ou tamanho do primeiro círculo colorido,
dependendo somente do tamanho e posição relativas dos círculos interno e
externo!

Fig. 2 - Os cinco círculos se encaixam para qualquer posição do primeiro

A explicação para estes fatos está em uma transformação geométrica


dos pontos do plano chamada de inversão e definida do seguinte modo.

Definição: Seja O um ponto do plano e k um número real positivo. A


inversão de centro O e constante k associa a cada ponto P do plano, distinto
de O, o ponto P’ (chamado de inverso de P) sobre a semi-reta OP tal que
OP. OP’ = k.
A Fig. 3 a seguir ilustra o resultado de se aplicar uma transformação
de inversão a um conjunto de pontos do plano. Como o produto OP. OP’
deve ser constante, quanto mais próximo um ponto estiver de O, mais
distante o seu inverso estará.

Q'
Q

P P'
O

R R'

Fig. 3 - Inversão

O logotipo da OBM é construído com o auxílio deste tipo de


transformação, explorando dois fatos fundamentais.

EUREKA! N°4, 1999


43
Sociedade Brasileira de Matemática

a) Inversões são tranformações injetivas (isto é, pontos distintos


possuem inversos distintos).
Para verificar este fato, basta observar que o ponto P cujo inverso é um
certo ponto P’ está univocamente determinado e é justamente o inverso de
P’ (ou seja, a transformação inversa de uma inversão é ela mesma).

b) O inverso de um círculo que não passa pelo centro de inversão é um


outro círculo.
Consideremos uma inversão de centro O e constante k e tomemos um círculo
C que não passa por O. Seja P um ponto de C, P’ o seu inverso e Q o outro
ponto em que a reta OP corta C.

P P'
Q
O

Fig. 4 - O inverso de um círculo

Uma propriedade fundamental do círculo é que o produto OP. OQ é


igual a uma constante p (a potência de O em relação a C) para qualquer
posição de P. Assim,
OP ' OP.OP ' k
 
OQ OP.OQ p
Portanto, o inverso de C pode ser obtido assim: para cada ponto Q
de C, tomamos o ponto P' sobre a semi-reta OQ tal que OP' = (k/p) OQ.
Este tipo de transformação é chamado de homotetia e sempre transforma
uma figura em outra semelhante (ela faz uma ampliação ou redução da
figura, conforme k/p seja maior ou menor que 1). Em particular, o
transformado de um círculo por homotetia é sempre um outro círculo. Em
resumo: o inverso de um círculo (que não passa pelo centro de inversão O) é
um outro círculo, obtido através de uma homotetia de centro O (para você
pensar: como será o inverso de um círculo que passa por O?).
Agora estamos em condições de entender como é construído o
logotipo da OBM. O ponto de partida é a figura abaixo: dois círculos
concêntricos, com cinco círculos de raios iguais encaixados entre eles.

EUREKA! N°4, 1999


44
Sociedade Brasileira de Matemática

Fig. 5 – O ponto de partida: cinco círculos iguais entre círculos concêntricos


Só é possível encaixar estes 5 círculos para um determinado valor da
razão R/r entre os raios dos círculos externo e interno. De modo mais geral,
vejamos qual deve ser esta razão para que n círculos possam ser encaixados
entre os dois círculos concêntricos. O diâmetro de cada um dos círculos
iguais é a diferença R  r entre os raios dos círculos concêntricos. Por outro
lado, seus centros formam um polígono regular de n lados, inscrito em um
R r
círculo de raio concêntrico aos dois círculos iniciais, como mostra a
2
figura abaixo.

A O

Fig. 6 - Quando é possível encaixar n círculos entre círculos concêntricos?

Rr
No triângulo retângulo OAM , a hipotenusa OA mede eo
2
R r
cateto AM mede e é oposto a um ângulo igual a 180°/n. Assim:
2
R r Rr
 sen(180 / n)
2 2
ou, desenvolvendo:

EUREKA! N°4, 1999


45
Sociedade Brasileira de Matemática

R 1  sen(180 / n)

r 1  sen(180 / n)
No nosso caso, em que n = 5, devemos ter
R 1  sen( 36 )

r 1  sen(36 )
Quando os raios R e r estão nessa proporção, é possível encaixar cinco
círculos iguais entre os dois círculos concêntricos.
Para terminar de formar o logotipo, tomamos o conjunto formado
pelos dois círculos concêntricos e pelos cinco círculos de raios iguais
encaixados entre eles e aplicamos uma transformação de inversão.

Fig. 7 – O logotipo da OBM, obtido por inversão

A menos que o centro de inversão seja o seu centro comum, os


inversos dos círculos concêntricos não são concêntricos. Além disso, os
inversos dos cinco círculos iguais não tem mais raios iguais, dando o aspecto
irregular do logotipo. Os raios dos círculos tornam-se mais desiguais quanto
mais o centro de inversão se afasta do centro dos círculos concêntricos.
Note, no entanto, que as propriedades de tangência são preservadas, em
virtude da injetividade da inversão, que faz com que o número de pontos de
interseção de figuras seja preservado através da transformação. Para
produzir a animação do logotipo, basta girar o conjunto de cinco círculos de
raios iguais na figura original: seus transformados por inversão mudarão de
tamanho e posição à medida que isto ocorre.
Se você quiser, pode experimentar com as propriedades dessa
transformação visitando a página da OBM na Internet. Basta clicar sobre o
símbolo animado da OBM, ou ir diretamente ao seguinte endereço:
http: //www.obm.org.br/logotipo.htm. Você encontrará uma página
interativa que permite variar as proporções do símbolo através da escolha do
centro de inversão. Você até poderá criar símbolos diferentes mudando o
número de anéis! Na verdade, os "designers" que criaram o logotipo da

EUREKA! N°4, 1999


46
Sociedade Brasileira de Matemática

OBM utilizaram um programa parecido, para ajustar o tamanho e posição


relativa dos anéis de modo a produzir uma figura agradável do ponto de vista
visual. Este é um bom exemplo do emprego da Matemática em artes visuais.
Há casos notáveis de artistas, como Escher, que usaram a Matemática como
ferramenta essencial em seu processo criativo.
Em futuros números da EUREKA! voltaremos a falar de inversão,
estudando suas propriedades em mais detalhe e mostrando outras aplicações.
Aguardem!
SOLUÇÕES DE PROBLEMAS PROPOSTOS

 Publicamos aqui algumas das respostas enviadas por nossos leitores.


11) Determine todas as soluções de xy = yx com x e y racionais
positivos.

Solução de Carlos Alberto da Silva Victor:

i) É evidente que x = y  Q+ satisfaz a equação.


x
ii) Suponha x  y, seja então a (a  Q+), logo:
y
1 p pq
ay  y a  y a a  1 e, fazendo a  1  , a  , temos :
q q
q


 y  p  q  p

  q 
  
 q p
  p q p
 x  
  q 
Já que p e q são primos entre si, p + q e q também são primos entre
si, e portanto devemos garantir que p p  q e p q sejam inteiros.
Necessariamente devemos ter p = 1; se não vejamos: suponha que p 2 e
que q = sp para algum inteiro positivo s; daí s p  p  q  ( s  1) p
(pois (s + 1)p – sp > p . sp – 1  p) e não teremos p p  q sendo um inteiro, o
que nos obriga fazer p = 1.

EUREKA! N°4, 1999


47
Sociedade Brasileira de Matemática

Conclusão:
  1  q  q 1
 x  
  q 
Para x  y, teremos:  q
 1 q 
 y  
  q 
onde q é um inteiro maior do que ou igual a 1. É fácil verificar que tais x e y
são racionais e são soluções do problema, para todo q  1 natural.

13) Dado n  N determine determine o maior k  N tal que existam


conjuntos A1, A2,…, Ak contidos em {1, 2, …, n} de forma que
Ai  Aj para todo i  j.

Solução de Zoroastro Azambuja Neto:


n!
k
Observemos inicialmente que C n  C n n / 2  para todo k com
k! ( n  k )!
0  k  n (onde [n/2] é o único inteiro tal que [n/2]  n/2  [n/2] + 1). De fato,
C nk 1 n k
se k  n / 2,  1 pois n > 2k e portanto
C nk k 1
n  2k + 1, ou seja n – k  k + 1. Portanto, se k  n / 2, C nk C n n / 2  e se
k  n / 2, C nk C nn  k C n n / 2  , pois n  k  n / 2. Seja A  {1, 2,…, n} e
P = (a1, a2, …, an) uma permutação de {1, 2, …, n}. Dizemos que P passa por
A se existe m  n com A = {a1, a2, …, am}. Se A tem m elementos existem
exatamente m! (n – m)! permutações que passam por A. Como
n! n!
C nm C n n / 2   ,
m!( n  m)!  n / 2 !(n   n / 2 )! donde
m! (n – m)!   n / 2 !(n   n / 2 )! para todo m  n.
Note agora que se A1, A2, …, Ak  {1, 2, …, n} são tais que, para todo i  j
Ai  Aj , então, se i  j, nenhuma permutação passa por Ai e Aj ao mesmo
tempo.
Se mi é o número de elementos do conjunto Ai (1  i  k ), podemos concluir
k
que  m !(n  m)!n! k. n / 2 ! (n   n / 2 )!n! k C 
i 1
i n
n / 2
.

EUREKA! N°4, 1999


48
Sociedade Brasileira de Matemática

Por outro lado, há C n n / 2  subconjuntos de {1, 2, …, n} com


exatamente  n / 2 elementos e, obviamente, se A e B são dois subconjuntos
distintos de  n / 2 elementos de {1, 2,…, n} então A  B, de modo que o
maior k que satisfaz as condições do enunciado é C n n / 2  .

15) Considere uma seqüência de triângulos retângulos AnBnCn no plano


cuja hipotenusa seja BnCn, com as seguintes condições:

i) A1B1 = A1C1 = 1;
ii) Bn+1 = Bn e An+1 = Cn para todo n  N;
iii) An+1Cn+1 é congruente à altura de An em relação a BnCn.

Mostre que qualquer ponto do plano pertence a infinitos triângulos AnBnCn.

Adaptada de uma solução enviada por Onofre Campos da Silva Farias:

Sejam an, bn e cn os comprimentos dos lados BnCn, AnCn e AnBn do triângulo


AnBnCn e n o ângulo An Bˆ n C n .
É suficiente mostrarmos que as hipotenusas dos triângulos crescem
infinitamente e que os triângulos dão infinitas voltas em torno do ponto B1.
Em outras palavras, devemos mostrar que a seqüência (an) é ilimitada e que
n
Sn  
k 1
k também cresce ilimitadamente.

Seja hn a altura relativa à hipotenusa BnCn. É imediato que


anhn = bncn.
Além disso, pelas condições (ii) e (iii), segue que
an = cn+1 e bn+1 = hn,
donde concluímos que
bn+1cn+1 = bncn = ... = b1c1 = 1.
Daí, anhn = 1, para todo natural n, e como an+12 = bn+12 + cn+12, vem
1
a n21 a n2  2 , a12 = 2.
an
2
Seja pn = an . Então, para mostrar que an é ilimitada, é suficiente mostrarmos
que (pn) é ilimitada. Note que an+1 > an, para todo natural n, de modo que (pn)
é crescente. Temos

EUREKA! N°4, 1999


49
Sociedade Brasileira de Matemática

1
p n 1  p n  , p1 2.
pn
Vamos mostrar por indução que sempre temos Pn  n  1. De fato,
1 1 n2
P1 2  2 , e Pn  n  1 implica Pn 1  Pn   n 1    n  2.
Pn n 1 n 1
Assim, Pn (e portanto an) é ilimitada. Vamos agora provar que (Sn) é
bn 2 1
ilimitada. Temos  n  sen n  bn bn 1  bn 1 (pois bn  decresce
an cn
quando n cresce). Como
bn21 a n21  a n2 ,  1   2  ...   n  (a 22  a12 )  (a32  a 22 )  ...  (a n21  a n2 ) 
n
a n21  a12  n  2  2, e portanto S n  
k 1
k é ilimitado.

18) Seja  a maior raiz real da equação x3 – 3x2 + 1 = 0.


Prove que [2004] é divisível por 17.
Obs: [y] é o único inteiro tal que [y]  y < [y] + 1.

Solução de Luiz Antonio Ponce Alonso:

Considere ,  e  as raízes de f(x) = x3 – 3x2 + 1,  a maior delas,


e S(n) = n + n + n .
(Obs. É fácil de se ver que todas as raízes são reais. De fato,
f ( 1)  0  f (0), e f (1)  0  f (3)).
Com estas considerações e as relações de Girard para f(x)= 0, tem-se que:

(I) S(0) = 0 +  0 +  0 = 3, S(1) = 1 + 1 + 1 = 3 e


S(2) =  +  +  = ( +  +  ) – 2 ( +  +  ) = 9 – 0 = 9
2 2 2 1 1 1 2

(II) f(x) = x2 . ( x – 3 ) +1 > 0 para todo x  3.


Como f é contínua e f( 2,87 )  – 0,07 < 0, segue-se que
2,87 <  < 3.
Por outro lado, de (I) tem-se  2 +  2 +  2 = 9 .
Assim,  2 +  2 < 9 – (2,87)2 < 1.
Sendo f(0) = 1, conclui-se que  e  são não nulas . Logo,
0 <  2 +  2 < 1 e conseqüentemente ,

EUREKA! N°4, 1999


50
Sociedade Brasileira de Matemática

0 <  2 < 1 e 0 <  2 < 1.

(III) P(x) = xn – 3 . f(x) = x n – 3x n – 1 + x n – 3 ( n  3) é um polinômio, tal que:

 P( )  n  3 n 1  n 30
 n n 1 n 3
 P(  )   3   0
 n n 1 n 3
 P()   3   0

Adicionando-se membro a membro , obtém-se para n  3, a seguinte


relação de recorrência:
S( n ) – 3. S ( n –1 ) + S ( n – 3 ) = 0 , ou, melhor ainda

S(n) = 3. S (n –1) – S (n – 3) (n  3)

(IV) Sendo S(0), S(1) e S(2) inteiros por (I), podemos concluir, através da
relação de recorrência acima e indução sobre n, que S(n) será um
inteiro para qualquer natural n.
Assim,
S(2004) =  2004 +  2004 +  2004 é um inteiro.

Como 0 <  2 < 1 e 0 <  2 < 1 ....... por (II), segue-se que
0 < 2004 + 2004 = ( 2) 1002 + (2 )1002 <  2 +  2 < 1.
Logo, S(2004) – 1 <  2004 < S(2004), ou seja,
  2004 = S(2004) – 1.

(V) A relação de recorrência obtida em (III) implica em particular que


S (n) 3S (n  1)  S ( n  3) ( mod 17) para todo n 3, o que
permite construir a tabela seguinte de S(n) (mod 17):
S(0)  3 (mod 17), S(1)  3 (mod 17), S(2)  9 (mod 17),
S(3)  7 (mod 17), S(4)  1 (mod 17), S(5)  11(mod 17),
S(6)  9 (mod 17), S(7)  9 (mod 17), S(8)  16 (mod 17),
S(9)  5 (mod 17), S(10)  6 (mod 17), S(11)  2 (mod 17),
S(12)  1 (mod 17), S(13)  14 (mod 17), S(14)  6 (mod 17),
S(15)  0 (mod 17), S(16)  3 (mod 17), S(17)  3 (mod 17),

EUREKA! N°4, 1999


51
Sociedade Brasileira de Matemática

S(18)  9 (mod 17).

(VI) Note que S(16)  S(0) (mod 17), S(17)  S(1) (mod 17) e
S(18)  S(2) (mod 17). Isso permite mostrar que S(n +16)  S(n)
(mod 17) para todo n  N. De fato, por indução, S(n + 16) =
3S(n + 15) – S (n + 13)  3S(n – 1) – S(n – 3) (mod 17) =
S(n)( mod 17), se n  3 (usamos como base de indução os casos
n = 0, n = 1 e n = 2). Como conseqüência, concluimos que
S(n)  S(n + 16p) (mod 17), para todo p  N.

(VII) Como 2004 = 4 + 16.p , com p = 125, temos de (IV), (V) e (VI):
S(2004)  S(4)  1 (mod 17).
Portanto,
  2004 = S(2004) – 1 é divisível por 17.

Nota: A demonstração acima foi baseada na resolução de um problema de


enunciado similar a este, proposto pela França e não utilizado na IMO de
1988.

19) a) Determine o número máximo de regiões em que n retas podem


dividir o plano.
b) Determine o número máximo de regiões em que n planos podem
dividir o espaço.

Solução de Carlos Alberto da Silva Victor:


a) Observe inicialmente os casos abaixo:

i) Uma reta divide o plano em duas regiões: 1 + 1.


ii) Uma segunda reta é dividida pela anterior no máximo em duas
partes e mais duas regiões são acrescentadas, ou seja: com 2 retas
temos: ( 1 + 1 + 2) regiões (no máximo.)
iii) Uma terceira reta é dividida pelas duas retas anteriores no máximo
em três partes e acrescentando então mais três regiões, ou seja: com
3 retas temos: (1 +1 +2 +3) regiões (no máximo.)
iv) Suponha agora que tenhamos n retas; a n-ésima reta é dividida pelas
(n – 1) outras retas no máximo em n partes e evidentemente

EUREKA! N°4, 1999


52
Sociedade Brasileira de Matemática

acrescentando n regiões, o que nos dará: (1 + 1 + 2 + 3 +…+ n)


n2  n
regiões ou  1 regiões.
2
Obs: Se as retas estão em posição geral todas as desigualdades acima são
igualdades.

b) Observe que:

i) Quando temos dois planos, o segundo plano intersecta o primeiro


plano no máximo através de uma reta e o segundo plano é dividido
em duas partes.
ii) Um terceiro plano intersecta os planos anteriores em no máximo
duas retas e o terceiro plano é dividido em 4 partes.
iii) Um quarto plano intersecta os planos anteriores em no máximo 3
retas e o quarto plano é dividido em 7 partes.

Em geral, o k-ésimo plano intersecta os anteriores em no máximo k – 1 retas,

que o dividem em no máximo


 k  1 2  (k  1)  1  k 2  k  2 regiões
2 2
(pelo ítem a), ou seja, ao ser acrescentado o k-ésimo plano são criadas no
k2  k 2
máximo novas regiões do espaço.
2
Como um plano divide o espaço em duas regiões temos no máximo
n
k2  k 2
2 
k 2 2
regiões em que k planos dividem o espaço.

n( n  1)(2n  1)
Sabendo que 12  2 2  3 2  ...  n 2  , temos um total de
6
n 3  5n  6
regiões (no máximo).
6
Obs: Se os planos estão em posição geral todas as desigualdades acima são
igualdades.

Continuamos esperando as soluções dos problemas 10, 16 e 17 das edições


anteriores.

Problemas Propostos no Cartaz


Nível Iniciante

EUREKA! N°4, 1999


53
Sociedade Brasileira de Matemática

Maria convidou 9 garotos e 8 garotas para sua festa de aniversário. Ela


preparou camisetas com os números de 1 a 18 e ficou com a de número 1 e
distribuiu as demais para seus convidados. Em determinado momento, em
que todos estavam dançando, a soma dos números de cada casal era um
quadrado perfeito. Quais pares estavam dançando?

Solução:
A maior soma possível dos números de um casal é 18 + 17, que é menor que
62. Assim, os valores das somas dos números de cada casal devem valer 4, 9,
16 ou 25, de modo que os pares de 18, 17 e 16 devem ser 7, 8 e 9,
respectivamente. Portanto o par de 2 deve ser 14 (pois não pode ser o próprio
2 e o 7 já é par do 18), o do 11 deve ser 5 (pois 14 já é par de 2), o de 4 deve
deve ser 12 (pois 5 já é par de 11), o de 13 deve ser 3 (pois 12 já é par de 4),
o de 1 deve ser 15 (pois 3 já é par de 13 e 8 já é par de 17) e o de 10 deve ser
6 (pois 15 já é par de 1). Assim, os pares são 18 e 7, 17 e 8, 16 e 9, 15 e 1, 14
e 2, 13 e 3, 12 e 4, 11 e 5, 10 e 6.

Nota: Ana Maria B. Guimarães e Karina de F. M. Silva da Escola Estadual


Professor Fábregas enviaram soluções corretas deste problema.

Nível Intermediário
AD é a bissetriz interna do ângulo A do triângulo ABC, com D sobre o lado
BC. As bissetrizes dos ângulos ADB e ACB concorrem em E, com E sobre o
lado AB. Determine a medida do ângulo BAC.

O problema é equivalente ao problema 5 da página 3 da EUREKA! No.3,


resolvido na página 6 da mesma.

Nível Avançado
São dadas 13 moedas, das quais 12 têm o mesmo peso. Não se sabe se a
décima terceira moeda é mais leve ou mais pesada que as demais. Mostre
que é possível determinar a moeda diferente empregando três pesagens em
uma balança de braços. Isto ainda seria possível com 14 moedas?

Solução:
Das 13 moedas selecionaremos dois grupos de 4 moedas A1, A2, A3, A4 e
B1, B2, B3, B4 e as pesamos. Sobram 5 moedas, C1, C2, C3, C4 e C5. Temos
duas possibilidades:

EUREKA! N°4, 1999


54
Sociedade Brasileira de Matemática

i) A balança fica equilibrada. Neste caso a moeda diferente está entre


as 5 restantes. Pesamos agora A1, A2, A3 e C1, C2, C3 (A1, A2 e A3 são
padrão). Temos mais duas possibilidades
i.1) Equilíbrio. A moeda diferente é C4 ou C5. Pesamos A1 e C4. Se der
diferente a moeda diferente é C4, e se houver equilíbrio é C5.
i.2) Desequilíbrio. Vamos supor sem perda de generalidade que o grupo
C1C2C3 é mais pesado que A1A2A3. Nesse caso a moeda diferente é
C1, C2 ou C3 e é mais pesada que as outras. Pesamos C1 e C2. Se
houver desequilíbrio a mais pesada é a diferente. Se houver
equilíbrio é a C3.
ii) Desequilíbrio. Vamos supor sem perda de generalidade que o grupo
A1A2A3A4 é mais pesado que B1B2B3B4. Pesamos agora A1A2B1 e
A3B2C1. Temos três possibilidades:
ii.1) Equilíbrio. Nesse caso a moeda diferente é A4, B3 ou B4. Se for A4 é
mais pesada e se for B3 ou B4 é mais leve. Pesamos B3 e B4. Se
houver equilíbrio a diferente é A4. Se não a mais leve das duas é a
diferente.
ii.2) O grupo A1A2B1 é mais pesado. Nesse caso a moeda diferente é A1,
A2 ou B2. Pesamos A1 e A2. Se houver equilíbrio a diferente é B2, se
não é a mais pesada das duas.
ii.3) O grupo A1A2B1 é mais leve. Nesse caso a moeda diferente é B1 ou
A3. Pesamos B1 e C1. Se houver equilíbrio a moeda diferente é A3, se
não é B1.

Se tivermos 14 moedas não é possível determinar sempre a moeda


diferente. Se na primeira pesagem pesamos dois grupos de 5 ou mais moedas
e não houver equilíbrio, a moeda diferente pode ser qualquer uma das pelo
menos 10 envolvidas na pesagem. Como cada pesagem tem apenas 3
resultados possíveis, as duas últimas pesagens dão no total no máximo 9
resultados diferentes, que não permitem distinguir todas as (pelo menos 10)
possibilidades de moeda diferente.
Se pesarmos dois grupos de 4 ou menos moedas na primeira
pesagem e houver equilíbrio sobram pelo menos 6 moedas para análise. Se
na segunda pesagem usarmos 4 ou mais dessas moedas de situação
desconhecida e não houver equilíbrio, qualquer uma delas pode ser a
diferente e a última pesagem (que só tem 3 resultados possíveis) não pode
determiná-la com segurança.
Se na pesagem usamos 3 ou menos das moedas de situação
desconhecida sobram pelo menos 3 em situação desconhecida. Se na terceira
pesagem usamos duas delas ou todas as 3 e houver desequilibrio qualquer

EUREKA! N°4, 1999


55
Sociedade Brasileira de Matemática

uma dessas moedas pode ser a diferente, e não concluímos nossa tarefa. Se
usamos uma ou nemhuma e houver equilíbrio sobram pelo menos duas de
situação desconhecida, e em qualquer caso não é sempre possível determinar
a moeda diferente.

Nota: O problema 10, proposto na página 59 da EUREKA! No. 2 generaliza


este problema. Tente resolvê-lo agora, adaptando para a situação geral os
argumentos desta solução!

PROBLEMAS PROPOSTOS
 Convidamos o leitor a enviar soluções dos problemas propostos e sugestões de novos
problemas para os próximos números.

20) Diga se existe uma função polinomial de R2 em R cuja imagem seja


o intervalo (0,) {x  R x  0}.
21) a) Encontre todas as soluções inteiras da equação a  b  c .
b) Encontre todas as soluções inteiras da equação 3 a  3 b 3 c .

22) Sejam  ,  ,  ,  os ângulos de um quadrilátero, nessa ordem.


Prove que esse quadrilátero é inscritível se, e somente se, a relação
        2 ocorre.

23) Seja ABC um triângulo qualquer de ortocentro H e sejam ha, hb, hc os


comprimentos das alturas relativas a A, B, C respectivamente. Prove
____ ____ ____
1
que ha . AH  hb . BH  hc CH  (a 2  b 2  c 2 ).
2
24) Na loteria de Truchilândia, cada bilhete tem um número de três
algarismos que usa somente os algarismos 1, 2, 3, 4 (é permitido
repetir os dígitos). Um bilhete é ganhador se coincide em pelo
menos duas posições com o número sorteado.
Um apostador quer compar vários bilhetes, de maneira que um deles
ganhe com certeza, mas gastando o mínimo possível. Determinar
quantos bilhetes deve comprar e quais bilhetes deve comprar.

Obs. Se o bilhete sorteado for o 423 então 123 é um bilhete


ganhador, mas 243 não é.

EUREKA! N°4, 1999


56
Sociedade Brasileira de Matemática

25) Durante o ano de 1998, uma pequena livraria, que abria nos sete dias
da semana, vendeu no mínimo um livro por dia e um total de 600
livros no ano todo. Diga, justificando, se existiu, obrigatóriamente,
um período de dias consecutivos onde foram vendidos exatamente
129 livros.

Nota: Os problemas 21, 22, 23 e 24 foram propostos na 1a. lista de preparação


para a X Olimpíada de Matemática do Cone Sul. O problema 25 foi
proposto na IX Olimpíada de Matemática do Rio Grande do Norte.
ERRATA
A solução do problema 2 da 9a. Olimpíada de Matemática do Cone
Sul, (cujo enunciado está na página 22 da EUREKA! No.2) está errada. Na
verdade publicamos a solução de outro problema do banco, cujo enunciado
era:

Sejam H o ortocentro do triângulo ABC , não retângulo, e M o ponto


médio do lado BC. A circunferência circunscrita em P. Mostre que
P, H, M são colineares.

A solução correta do problema 2 da 9a. Olimpíada de Matemática do Cone


Sul é como segue:

PROBLEMA 2:
Sejam H o ortocentro (interseção das alturas) do triângulo acutângulo ABC e
M o ponto médio do lado BC. Seja X o ponto em que a reta HM intersecta o
arco BC (que não contém A) da circunsferência circunscrita a ABC. Seja Y o
ponto de interseção da reta BH com a circunsferência, distinto de B.
Demonstre que XY = BC.

Solução:

EUREKA! N°4, 1999


57
Sociedade Brasileira de Matemática

Seja X' o simétrico de H em relação ao ponto M. Vamos mostrar que X  X'.


O quadrilátero HBX'C é um paralelogramo, pois os pontos médios de suas
diagonais coincidem. Então
 BX'C = BHC = 180 –  BAC.
Segue que X' deve pertencer ao arco BC que não contém A, donde X  X'.
Observe agora que o quadrilátero BYCX é um trapézio inscritível (pois
BY X'C = XC ), donde BC = XY.

CARTAS DOS LEITORES


 Publicamos aqui algumas cartas enviadas por nossos leitores.

UMA ESCOLA PÚBLICA NA EUREKA!

Eureka! Lembrei-me de Arquimedes quando tive a luz para a idéia


que mexeu comigo e um punhado de alunos que até então estavam inertes
face à necessidade de aprender Matemática.

Descobri! Diria Arquimedes, radiante de felicidade, no bom


português. E tal foi a emoção que vivi quando atinei para a solução que
revolucionaria o ensino da Matemática numa tão carente escola pública. Tal
luz, que me veio à mente, iluminou alguns alunos que poderiam hoje estar
mergulhados, como muitos, nas trevas do descaso e do abandono do ensino
público.
Descobri! Que o saber dos números independe das classes sociais. É
democrático! E nasce no espirito daqueles a quem seja dada a fagulha e uma
palavra de confiança, de incentivo.

A função da Matemática é profunda, é humanística, pois o bem estar


da coletividade é o fim de tudo. Cabe, à Matemática a criação de espíritos
disciplinados, mentes sadias e aprimoradas. E esse é o nosso escopo:
estimulando, propiciando e melhorando o ensino dessa matéria nas Escolas
Brasileiras.

Prof. Paulo Araripe


Fortaleza-CE

EUREKA! N°4, 1999


58
Sociedade Brasileira de Matemática

COMO ASSINAR A EUREKA!

Se você é fanático por Matemática e deseja receber na sua casa a


revista EUREKA!, faça o seu pedido escrevendo para: Secretaria da
Olimpíada Brasileira de Matemática, Estrada Dona Castorina, 110 Jardim
Botânico - Rio de Janeiro, RJ - CEP: 22460-320. O custo de cada exemplar
avulso ou atrasado é de R$4,00. Você pode fazer uma assinatura anual o que
dará direito a receber as publicações do referido ano (mínimo 3 exemplares)
por um valor promocional de R$10,00. Para isso, faça um depósito no Banco
do Brasil - Agência 0598-3 - Conta N52208-2 em nome do professor
Eduardo Wagner. Envie-nos a fotocopia do depósito e faça referência aos
números desejados. Não esqueça de colocar seu nome e endereço completos
e nós remeteremos a(s) revista(s) pelo correio. Pedidos podem ser feitos
também por e-mail e comprovantes de depósito poderão ser enviados pelo
fax.

Se tiver qualquer dúvida entre em contato conosco.


Telefone: 021-5295077 / Fax: 021-5295023
e-mail: obm@impa.br
Home-Page: http//www.obm.org.br/

EUREKA! N°4, 1999


59
Sociedade Brasileira de Matemática

Você sabia…
Que o livro 10 Olimpíadas Iberoamericanas de
Matemática (em espanhol) já pode ser adquirido na
Secretaria da Olimpíada Brasileira de Matemática ?
Entre em contato conosco.

AGENDA OLÍMPICA
5a. OLIMPÍADA DE MAIO
8 de maio (sábado)

10a. OLIMPÍADA DO CONE SUL


17 a 24 de maio
Argentina

21a. OLIMPÍADA BRASILEIRA DE MATEMÁTICA


Primeira Fase – 12 de junho (sábado)
Segunda Fase – 28 de agosto (sábado)
Terceira Fase – 23 de outubro (sábado) e 24 de outubro (domingo)

40a. OLIMPÍADA INTERNACIONAL DE MATEMÁTICA


10 a 22 de julho
Bucharest, Romênia.

14a. OLIMPÍADA IBEROAMERICANA DE MATEMÁTICA


setembro
Cuba.

EUREKA! N°4, 1999


60
Sociedade Brasileira de Matemática

Você sabia… Que a II Olimpíada Brasileira de


Astronomia deverá realizar-se em junho próximo entre
alunos do ensino fundamental e médio de todo o Brasil
?
Mais esclarecimentos podem ser obtidos através do
seguinte endereço:
Prof. João Batista Garcia Canalle
Coordenador da II OBA
Instituto de Física - UERJ
Tel/Fax: 021-5877150 e 5877447
e-mail: canalle@uerj.br

COORDENADORES REGIONAIS

Amarisio da Silva Araújo (UFV) Viçosa - MG


Alberto Hassen Raad (UFJF) Juiz de Fora - MG
Antônio C. Rodrigues Monteiro (UFPE) Recife - PE
Angela Camargo (Centro de Educação
de Adultos CEA) Blumenau - SC
Ariosto de Oliveira Lima (UFPI) Parnaíba - PI
Benedito T. Vasconcelos Freire (UFRN) Natal - RN
Claudio Arconcher (Col. Leonardo da Vinci) Jundiaí - SP
Egnilson Miranda de Moura (Col. Agrícola do Bom Jesus) Bom Jesus - PI
Élio Mega (Col. ETAPA) São Paulo - SP
Florêncio F. Guimarães F. (UFES) Vitória - ES
Francisco Dutenhefner (UFMG) Belo Horizonte - MG
Gisele de A. Prateado G. (UFGO) Goiânia - GO
Ivanilde H. Fernandes Saad (U. Católica Dom Bosco) Campo Grande - MS
João B. de Melo Neto (UFPI) Teresina - PI
João F. Melo Libonati (Grupo Educ. IDEAL) Belém - PA
Jorge Ferreira (UEM) Maringá - PR
José Carlos Pinto Leivas (URG) Rio Grande - RS
José Luis Rosas Pinho (UFSC) Florianópolis - SC
José Paulo Carneiro (USU) Rio de Janeiro - RJ
José Vieira Alves (UFPB) Campina Grande - PB
Leonardo Matteo D'orio (Parque de Material
Aeronáutico de Belém) Belém - PA
Licio Hernandes Bezerra (UFSC) Florianópolis - SC
Luzinalva M. de Amorim (UFBA) Salvador - BA
Marco Polo (Colégio Singular) Santo André - SP

EUREKA! N°4, 1999


61
Sociedade Brasileira de Matemática

Marcondes Cavalcante França (UF Ceará) Fortaleza - CE


Mario Jorge Dias Carneiro (UFMG) Belo Horizonte - MG
Pablo Rodrigo Ganassim (L. Albert Einstein) Piracicaba - SP
Paulo H. Cruz Neiva de L. Jr. (Esc. Tec.Everardo Passos) SJ dos Campos - SP
Reinaldo Gen Ichiro Arakaki (INPE) SJ dos Campos - SP
Ricardo Amorim (Centro Educ. Logos) Nova Iguaçu - RJ
Roberto Vizeu Barros (Colégio ACAE) Volta Redonda - RJ
Sergio Claudio Ramos (IM-UFRGS) Porto Alegre - RS
Seme Gebara Neto (UFMG) Belo Horizonte - MG
Tadeu Ferreira Gomes (U. do Estado da Bahia) Juazeiro - BA
Valdenberg Araújo da Silva (U. Federal de Sergipe) São Cristovão - SE
Wagner Pereira Lopes (Esc. Tec. Fed. de Goiás) Jataí - GO
Waldemar M. Canalli (P.M. S. João de Meriti) S. João de Meriti -
RJ

EUREKA! N°4, 1999


62
CONTEÚDO

AOS LEITORES 2

5a. OLIMPÍADA DE MAIO 3


Problemas

5a. OLIMPÍADA DE MAIO 5


Resultados

10a. OLIMPÍADA DE MATEMÁTICA DO CONE SUL 7


Problemas e Soluções

40a. OLIMPÍADA INTERNACIONAL DE MATEMÁTICA 15


Problemas e Resultados

ARTIGOS
ADEDANHA OU "DE COMO OS DEUSES MATEMÁTICOS
TROUXERAM A PAZ AO MUNDO" 17
Pablo Emanuel

QUADRILÁTEROS E TRIÂNGULOS 23
Marcelo Mendes

O PRINCÍPIO DAS GAVETAS 27


Paulo Cezar Pinto Carvalho

DESIGUALDADES ELEMENTARES 34
Antonio Caminha Muniz Neto

40a. OLIMPÍADA INTERNACIONAL E 14a. OLIMPÍADA IBEROAMERICANA 50


DE MATEMÁTICA
Primeiro teste de Seleção

SOLUÇÕES DE PROBLEMAS PROPOSTOS 51

PROBLEMAS PROPOSTOS 57

COMO ASSINAR A EUREKA! 59

AGENDA OLÍMPICA 60

COORDENADORES REGIONAIS 61
Sociedade Brasileira de Matemática

AOS LEITORES

Realizamos a primeira fase da XXI Olimpíada Brasileira de Matemática


em mais de 3.000 colégios do nosso país. Através dos relatórios enviados pelas
escolas aos Coordenadores Regionais, estabelecemos as notas de corte para a
promoção dos alunos à segunda fase que se realizará em agosto.

Notas de corte para promoção a segunda fase:

Primeiro nível (5a. e 6a. séries) : 09 acertos ou mais.


Segundo nível (7a. e 8a. séries) : 10 acertos ou mais.
Terceiro nível (Ensino médio) : 12 acertos ou mais.

A prova da segunda fase será discursiva e ainda será aplicada nos


colégios. Os enunciados, as soluções e os criterios de pontuação serão enviados a
todas as escolas participantes.

A Comissão de Olimpíadas agradece a participação de alunos e


professores na primeira fase e deseja sucesso a todos nas fases seguintes.

Agradecemos às pessoas que colaboraram com este número


propondo problemas e enviando soluções de problemas propostos. Aproveitamos
para continuar estimulando nossos leitores a contribuir para nossa revista com
artigos, problemas e soluções.

Lembramos da existência da lista de discussão de problemas de


matemática da OBM, cujo endereço eletrônico mudou para:
obm-l@mat.puc-rio.br . Para maiores informações escreva para obm@impa.br ou
para o Prof. Nicolau Saldanha, administrador desta lista em nicolau@mat.puc-
rio.br .

Comitê Editorial.

EUREKA! N°5, 1999


2
Sociedade Brasileira de Matemática

5a. OLIMPÍADA DE MAIO


Primeiro Nível

PROBLEMA 1
São escolhidos 2 números inteiros entre 1 e 100 inclusive, tais que a diferença é 7
e o produto é múltiplo de 5. De quantas maneiras pode ser feita a escolha?

PROBLEMA 2
Num paralelogramo ABCD, BD é a diagonal maior.
Ao fazer coincidir B com D, mediante uma dobra, se forma um pentágono
regular.
Calcular as medidas dos ângulos que a diagonal BD forma com cada um dos
lados do paralelogramo.

PROBLEMA 3
Em cada um dos 10 degraus de uma escada existe uma rã.
Cada rã pode, de um pulo, colocar-se em outro degrau, mas quando uma rã faz
isso, ao mesmo tempo, uma outra rã pula a mesma quantidade de degraus em
sentido contrário: uma sobe e outra desce.
Conseguirão as rãs colocar-se todas juntas num mesmo degrau?

PROBLEMA 4
2 1
Dez cartões quadrados de 3 centímetros de lado são
3 cortados por uma línha, como mostra a figura.
Depois dos cortes tem-se 20 peças: 10 triângulos e
10 trapézios. Forme um quadrado que utilize as 20
peças sem superposições nem espaços.

PROBLEMA 5
Ana, Beatriz, Carlos, Diego e Emilia jogam um torneio de xadrez.
Cada jogador enfrenta uma vez só cada um dos outros quatro jogadores.
Cada jogador consegue 2 pontos se ganha a partida, 1 ponto se empata e 0 pontos
se perde a partida.
Ao finalizar o torneio, as pontuações dos 5 jogadores são todas diferentes.
Encontre o máximo número de empates que pode ter tido o torneio e justifique
por que não pode ter havido um número maior de empates.

EUREKA! N°5, 1999


3
Sociedade Brasileira de Matemática

SEGUNDO NÍVEL
PROBLEMA 1
Um número natural de três algarismos é chamado de tricúbico se é igual à soma
dos cubos dos seus dígitos. Encontre todos os pares de números consecutivos tais
que ambos sejam tricúbicos.

PROBLEMA 2
A figura representa a quarta parte de um círculo de B

raio 1. No arco AB, se consideram pontos P e Q de


forma tal que a reta PQ seja paralela à reta AB.
Sejam X e Y os pontos de interseção da reta PQ com as
_____2 ____ 2
retas OA e OB respectivamente. Calcular PX  PY . O A

PROBLEMA 3
A primeira fileira da tabela ao lado é preenchida com
os números de 1 a 10, em ordem crescente. A segunda
fileira é preenchida com os números de 1 a 10, em
qualquer ordem.
Em cada casa da terceira fileira se escreve a soma dos
dois números escritos nas casas acima. Existe alguma
maneira de preencher a segunda fileira de modo que os
algarismos das unidades dos números da terceira fileira
sejam todos distintos?
PROBLEMA 4
Seja ABC um triângulo equilátero. M é o ponto médio do segmento AB e N é o
ponto médio do segmento BC. Seja P o ponto exterior a ABC tal que o triângulo
ACP é isósceles e retângulo em P. PM e AN cortam-se em I. Prove que CI é a
bissetriz do ângulo MCA.

PROBLEMA 5
São dados 12 pontos que são os vértices de um polígono regular de 12 lados.
Rafael deve traçar segmentos que tenham seus dois extremos em dois dos pontos
desenhados.
É permitido que cada ponto seja extremo de mais de um segmento e que os
segmentos se cruzem, mas é proibido traçar três segmentos que sejam os três
lados de um triângulo em que cada vértice é um dos 12 pontos iniciais.
Encontre o número máximo de segmentos que pode traçar Rafael e justifique por
que não é possivel traçar um número maior de segmentos.

EUREKA! N°5, 1999


4
Sociedade Brasileira de Matemática

5a. OLIMPÍADA DE MAIO


Resultado Brasileiro

Primeiro Nível

Fábio Dias Moreira Ouro Rio de Janeiro - RJ


André de Carvalho Amaro Prata São Paulo - SP
Diego Costa de Almeida Prata Fortaleza - CE
Daniel Haanwinckel Junqueira Bronze Salvador - BA
Raúl M. Alexandrino Nogueira Bronze Fortaleza - CE
Bruna Griguol Bronze Cafelândia - PR
Felipe Oliveira de Sousa Bronze Fortaleza - CE
Guilherme Finkelfarb Lichand Menção Honrosa São Paulo - SP
Leonardo Luis Desideri Freitas Menção Honrosa Vitória - ES
Zilma K. Barbosa Bezerra Menção Honrosa Fortaleza - CE
Thiago Augusto Caldas Bello Menção Honrosa Salvador - BA
Mateus Gomes Filgueiras Menção Honrosa Fortaleza - CE

Segundo Nível

João Alfredo Castellani F. Freire Ouro Salvador - BA


Arthur Duarte Nehmi Prata São Paulo - SP
Davi M. Alexandrino Nogueira Prata Fortaleza - CE
Luiz Brizeno Firmeza Neto Bronze Fortaleza - CE
Luciana Andretta do Nascimento Bronze Cafelândia - PR
Thiago Barros Rodrigues Costa Bronze Fortaleza - CE
Daniel Pinheiro Sobreira Bronze Fortaleza - CE
Maurício Massão Soares Matsumoto Menção Honrosa São Paulo - SP
Fabio S. Toniolo Menção Honrosa São Paulo - SP
Einstein do Nascimento Jr. Menção Honrosa Fortaleza - CE
Hugo Pinto Iwata Menção Honrosa S J do Rio Preto - SP
Rodrigo Roque Dias Menção Honrosa São Paulo - SP

O Brasil teve uma excelente participação na 5 a. Olimpíada de Maio na


qual participaram 13 paises, sendo o país com maior pontuação nos dois níveis
em que é realizada esta competição.

EUREKA! N°5, 1999


5
Sociedade Brasileira de Matemática

5a. OLIMPÍADA DE MAIO


Clasificação por paises

PRIMEIRO NÍVEL SEGUNDO NÍVEL

País Pontuação País Pontuação


Brasil 286 Brasil 405
Argentina 272 Peru 341
Espanha 206 Argentina 264
México 168 Cuba 219
Peru 157 Colombia 188
Colombia 154 México 175
Cuba 152 Espanha 136
Costa Rica 143 Uruguai 125
Uruguai 142 Chile 123
Chile 121 Bolívia 108
Bolívia 118 Costa Rica 57
Venezuela 95 Venezuela 57
Equador 55 Equador 22

!!!!

Você sabia… Que o maior número primo


conhecido é 26972593–1, que tem 2.098.960
dígitos e foi descoberto em 1/6/99 por Nayan
Hafratwala, um participante do GIMPS, um
projeto cooperativo para procurar primos de
mersenne?

Consulte na Internet a página


Http://www.mersenne.org/prime.htm

EUREKA! N°5, 1999


6
Sociedade Brasileira de Matemática

10a. OLIMPÍADA DE MATEMÁTICA DO CONE SUL


Problemas e Soluções

A 10a. Olimpíada de Matemática do Cone Sul foi realizada na cidade de


Córdoba, Argentina no período de 17 a 24 de maio de 1999. Dela participaram
alunos de até 15 anos dos seguintes países: Argentina, Brasil, Bolívia, Chile,
Paraguai, Peru e Uruguai.
A equipe brasileira foi selecionada através de provas realizadas em março
e maio deste ano e foi liderada pelos professores Florêncio Ferreira Guimarães
Filho da UFES, e Antônio Caminha Muniz Neto, da UFCE.

O Resultado da Equipe Brasileira

BRA 1 Daniel Massaki Yamamoto BRONZE


BRA 2 Daniel Pinheiro Sobreira BRONZE
BRA 3 Fabrício Siqueira Benevides PRATA
BRA 4 Humberto Silva Naves PRATA

Primeiro Dia
Duração da prova: 4 horas

PROBLEMA 1
Achar o menor inteiro positivo n tal que as 73 frações

19 20 21 91
, , ,.........,
n  21 n  22 n  23 n  93

sejam todas irredutíveis.

SOLUÇÃO
a a
A fração é irredutível se e só se é irredutível ( se a e b tem um fator
b b a
comum, então a e b – a têm um fator comum, e reciprocamente).
O problema se transforma em achar o menor valor de n tais que as frações
19 20 91
, ,...,
n2 n2 n2
sejam todas irredutíveis.

EUREKA! N°5, 1999


7
Sociedade Brasileira de Matemática

Se n + 2 é primo, maior que 91, todas as frações são irredutíveis. Assim, um valor
possível de n é 95. Verifiquemos que é o menor possível.
20
Se n + 2 < 97 e n + 2 é par (n é par) há frações redutíveis, por exemplo .
n2
Se 19  n + 2  91, obviamente há uma fração redutível.
Se n + 2 < 19, então n + 2 tem um múltiplo entre 19 e 91, e portanto, há uma
fração redutível.
31
Se n + 2 = 93 = 3 . 31, então é redutível.
n2
19
Se n + 2 = 95 = 5 . 19, então é redutível.
n2
Então, o valor mínimo de n + 2 é 97, que corresponde a n = 95.

PROBLEMA 2
Seja ABC um triângulo retângulo em A. Construir o ponto P sobre a hipotenusa
BC, tal que se Q for o pé da perpendicular traçada desde P ao cateto AC, então a
área do quadrado de lado PQ é igual à área do retângulo de lados iguais a PB e
PC. Mostrar os passos da construção.

SOLUÇÃO

B
H

A Q C

____ ____ ____ ____


P  BC satisfaz as condições do enunciado se e só se PQ  PB PC (*).
2

____ ___ ___


____ ____
PQ AB AB
Como PQC BAC , vem que  ____ . Daí, PQ PC ____ , e segue
____
PC BC BC

EUREKA! N°5, 1999


8
Sociedade Brasileira de Matemática

____ ____
PC BC 2 ____
de (*) que ____
 ____
. Sendo H o pé da altura relativa a BC , temos
PB AB 2

____ ____ ____


BH BC  AB , donde 2

____ ____
PC BC
 ____ .
____
PB BH

Temos então a seguinte construção:

B'

B
P

H' H

A C
r

i) Traçe, por C, a reta r tal que r r AC .
____ ____
ii) Marque em r o ponto B' tal que B ' C BC e B, B' estejam num mesmo

semi-plano dos determinados por BC .
iii) Trace a altura AH relativa à hipotenusa BC.
iv) Marque H'  AB tal que BH' = BH.
v) BC  B'H' = {P}

PROBLEMA 3
Há 1999 bolinhas em uma reta; algumas são vermelhas e as demais azuis
(poderiam ser todas vermelhas ou todas azuis). Debaixo de cada bolinha
escrevemos o número igual à soma da quantidade de bolinhas vermelhas à direita
dela mais a quantidade de bolinhas azuis à esquerda dela. Se, na sequência de

EUREKA! N°5, 1999


9
Sociedade Brasileira de Matemática

números assim obtida, houver exatamente três números que aparecem uma
quantidade ímpar de vezes, quais podem ser estes três números?

SOLUÇÃO

Se as 1999 bolinhas são de uma mesma cor, a sucessão de números é crescente ou


decrescente. Cada número aparece uma vez só e há 1999 (portanto, não há
exatamente 3 números que se repetem um número ímpar de vezes (1 é ímpar).
Logo, há bolinhas das duas cores.
Dada uma distribuição das bolinhas que tem em certa posição uma bolinha azul A
e na posição seguinte uma bolinha vermelha R, se há a bolinhas azuis à esquerda
de A e r bolinhas vermelhas à sua direita, então há a + 1 bolinhas azuis à esquerda
de R e r – 1 bolinhas vermelhas à sua direita. O número escrito embaixo de A é
n = a + r e o número escrito embaixo de R é a + 1 + r – 1 = n.
Se trocamos de lugar A e R, e não mexemos em nenhuma outra bolinha, na nova
distribuição há a bolinhas azuis à esquerda de R e r – 1 bolinhas vermelhas à sua
direita, enquanto que à esquerda de A há a bolinhas azuis e, à sua direita, r – 1
bolinhas vermelhas. Os números escritos embaixo de R e A são a + r – 1= n – 1 e
a + r – 1 = n – 1. Os números escritos embaixo das outras bolinhas não mudam.
Então, depois da troca, o número n se repete duas vezes menos e o número n – 1
se repete duas vezes mais. Os números que se repetem uma quantidade ímpar de
vezes serão os mesmos em ambas configurações.
Portanto, basta estudar a configuração na qual todas as bolinhas vermelhas são
consecutivas, a partir da primeira, e todas as azuis são consecutivas, a partir da
última vermelha.
Sejam , , as quantidades de bolinhas vermelhas e azuis, respectivamente; então
 +  = 1999. Embaixo da primeira bolinha (é vermelha) está o número  – 1, na
seguinte,  – 2, depois  – 3, e assim por diante, até ter 0 na última bolinha
vermelha (na posição ). Então, embaixo da primeira bolinha azul há 0, na
segunda 1 e assim por diante, até a última, que tem  – 1 embaixo.
Se  < , os números 0, 1, 2, …,  – 1 aparecem duas vezes (quantidade par) e os
números ,  + 1,  + 2, …,  – 1 aparecem uma vez (quantidade ímpar). Se há
exatamente 3 números que aparecem uma quantidade ímpar de vezes, estes são ,
 + 1 e  + 2 =  – 1. Portanto,  +  = 2 + 3, donde  = 998, e os três números
que se repetem uma quantidade ímpar de vezes são 998, 999 e 1000.
Se  > , os três números que aparecem uma quantidade ímpar de vezes são ,
 +1 e  + 2 =  – 1, donde  +  = 2 + 3 e os tres números são, novamente,
998, 999 e 1000.

EUREKA! N°5, 1999


10
Sociedade Brasileira de Matemática

Segundo Dia
Duração da prova 4 horas.

PROBLEMA 4
Seja A um número de seis algarismos, três dos quais estão coloridos e são iguais a
1, 2 e 4.
Demonstrar que é sempre possível obter um número que é múltiplo de 7,
efetuando uma só das seguintes operações: ou suprimir os três algarismos
coloridos, ou escrever todos os algarismos de A em alguma ordem.

SOLUÇÃO

Provaremos o seguinte resultado mais geral: Seja A um número de mais de 3


algarismos, três dos quais são 1, 2, 4. Prove que é sempre possível permutarmos
os algarismos de A de modo que o número resultante seja um múltiplo de 7.

Prova: Seja B = (ak…a1)10, k  1, o número obtido a partir de A ao suprimirmos


uma ocorrência de cada um dos algarismos 1, 2, 4 e C o número que queremos
obter a partir de A.

i) B = 7 : tome C = 2471
7......7 : C 7
 ...
 72471
  . Analogamente tratamos o caso
ii) B =    tome
k 1 k1
em que só há algarismos 0 e 7 em B.
Suponhamos, de agora em diante, que nem todos os algarismos de B
sejam iguais a 7 ou zero.
iii) B não é equivalente a 0 (mod 7):
Como {0, 124, 142, 214, 241, 412, 421} é um sistema completo de
restos, módulo 7, (isto é, esses números, quando divididos por 7 deixam
todos os restos possíveis: 0, 1, 2, 3, 4, 5 e 6) obtemos C justapondo, à
direita de B, uma permutação conveniente de 124.
iv) B  0 (mod. 7):
Se a1  7,0 então B' = (ak…a2 0 a1)10 não é múltiplo de 7, pois 10B –B' =
9a1. Como {0, 1024, 1042, 2014, 2041, 4012, 4021} também é um
sistema completo de restos, módulo 7, obtemos C como em (iii) (isto é,
somando 100B' a um dos seis números 1024,…,4021).

PROBLEMA 5

EUREKA! N°5, 1999


11
Sociedade Brasileira de Matemática

É dado um quadrado de lado 1. Demonstrar que, para cada conjunto finito de


pontos no bordo do quadrado, é possível achar um vértice do quadrado com a
seguinte propriedade: a média aritmética dos quadrados das distâncias de tal
3
vértice aos pontos do conjunto é maior ou igual a .
4

SOLUÇÃO

Sejam A1A2A3A4 o quadrado e X1, X2, …, Xn os pontos do perímetro. Devemos


_____ 2 _____ 2 _____ 2
provar que uma das quatro somas A X  A X  ...  A X , i 1, 2, 3, 4, é
i 1 i 2 i n
3n
maior ou igual que .
4
Somamos entre si estas quatro somas
_____ 2 _____ 2 _____ 2 _____ 2 _____ 2 _____ 2
( A1 X 1  A1 X 2  ...  A1 X n )  ( A2 X 1  A2 X 2  ...  A2 X n ) 
______ 2 ______ 2 ______ 2 _____ 2 _____ 2 _____ 2
( A3 X 1  A3 X 2  ...  A3 X n )  ( A4 X 1  A4 X 2  ...  A4 X n )
e reagrupamos os somandos em n grupos, um por cada ponto Xi
_____ 2 _____ 2 _____ 2 _____ 2 _____ 2 ______ 2 _____ 2 _____ 2
( A1 X 1  A2 X 1  A3 X 1  A4 X 1 )  ( A1 X 2  A2 X 2  A3 X 2  A4 X 2 )  ... 
______ 2 ______ 2 ______ 2 _____ 2
( A1 X n  A2 X n  A3 X n  A4 X n ).
Demostraremos que, se X é um ponto do perímetro do quadrado, então
_____ 2 _____ 2 _____ 2 _____ 2
( A1 X  A2 X  A3 X  A4 X ) 3.
Sejam x e 1 – x as distâncias X aos extremos do lado ao que pertencem. Então as
distâncias de X aos outros dois vértices do quadrado são, em alguma ordem,
1  x 2 e 1  (1  x ) 2 , e temos
_____ 2 _____ 2 ______ 2 ______ 2
A1 X  A2 X  A3 X  A4 X  x 2  (1  x) 2  (1  x 2 )  (1  (1  x) 2 ) 4( x 2  x  1).
Devemos provar que, para todo x   0,1 ,
1 2 3 3
4( x 2  x  1) 3, mas isto equivale a ( x  )   , que é claramente
2 4 4
verdadeira.
Temos assim que

EUREKA! N°5, 1999


12
Sociedade Brasileira de Matemática

_____ 2 _____ 2 _____ 2 _____ 2 _____ 2 _____ 2


( A1 X 1  A1 X 2  ...  A1 X n )  ( A2 X 1  A2 X 2  ...  A2 X n ) 
_____ 2 ______ 2 _____ 2 ____ 2 _____ 2 _____ 2
 ( A3 X 1  A3 X 2  ...  A3 X n )  ( A4 X 1  A4 X 2  ...  A4 X n ) 3n,
3n
portanto, um dos quatro somandos é maior ou igual que , c.q.d.
4

PROBLEMA 6
Uma formiga caminha pelo piso de um pátio circular de raio r e avança em linha
reta, mas às vezes se detém. Cada vez que se detém, antes de continuar a
caminhar, gira 60o alternando o sentido (se da última vez ela girou 60 o para a
direita da próxima vez gira 60o para a esquerda, e vice-versa). Achar o maior
comprimento possível do caminho percorrido pela formiga. Demonstrar que o
comprimento assim obtido é efetivamente, o máximo possível.

60o

Giro de 60o à direita.


SOLUÇÃO

Podemos supor que a formiga só se detém uma vez, pois, caso se detenha mais
vezes podemos substituir seu caminho por outro de mesmo comprimento onde ela
só se detém uma vez, como na figura abaixo

EUREKA! N°5, 1999


13
Sociedade Brasileira de Matemática

Sejam A e B respectivamente os pontos inicial e


C final do caminho percorrido pela formiga. Se AB
C'
não é diâmetro, traçamos o diâmetro A'B' // AB.
B Se C' é tal que A'C'B'  ACB, temos C' dentro
A B' do círculo (pois A'C'B' = 120) e A'C' + C'B' =
O A' B '
A'
( AC  CB )  AC  CB.
AB
Logo, para o caminho ser máximo AB deve ser
diâmetro, ou seja, AB = 2r
Temos agora que maximizar AC + CB sobre todos os triângulos ACB. tais que

que ACB 120 e AB = 2r. Seguem duas demonstrações de que AC + CB é
máxima quando AC = CB:

1a. demonstração
a 2  b 2  ab 4r 2  ( a  b) 2  ab 4r 2 
C
( a  b) 2
120 a ( a  b) 2  ( a  b) 2  ab 4r 2 
b
4
4r
B a b , com igualdade se e só se a b.
2r 3
A

2a. demonstração
Seja C ' ' sobre AC tal que C ' ' C CB e C ' '  A.
C'' Temos AC ' '  AC  CB e AC ' ' B 60 .

C
120

A B

EUREKA! N°5, 1999


14
Sociedade Brasileira de Matemática

Então devemos determinar a maior corda


C'' AC'', com C'' sobre o arco capaz do ângulo
60 sobre AB.
C O' Se O' é o centro da circunferência do arco
capaz, a corda é o diâmetro por O', Daí,
C  O' e AC = CC'' = CB

A B

4r
Assim, a resposta do problema é .
3

40a. OLIMPÍADA INTERNACIONAL DE MATEMÁTICA


Problemas e Resultados

A 40a. Olimpíada Internacional de Matemática foi realizada na cidade de


Bucharest, Romênia, no período de 10 a 22 de julho de 1999. A equipe brasileira
foi liderada pelos professores Nicolau Corção Saldanha, da PUC - Rio, e Carlos
Gustavo T. de A. Moreira, do IMPA.

O Resultado da Equipe Brasileira

BRA 1 Fabrício Siqueira Benevides 13 pontos (Medalha de Bronze)


BRA 2 Pedro Paulo de Simoni Gouveia 12 pontos (Medalha de Bronze)
BRA 3 Daniel Massaki Yamamoto 08 pontos
BRA 4 Sérgio Tadao Martins 14 pontos (Medalha de Bronze)
BRA 5 Daniel Nobuo Uno 11 pontos
BRA 6 Humberto Silva Naves 17 pontos (Medalha de Bronze)

A prova deste ano foi considerada difícil. Pela primeira vez em muitos
anos nenhum participante obteve a pontuação máxima (42 pontos). A maior
pontuação obtida este ano foi de 39 pontos.

Primeiro Dia
Duração da prova: 4 horas e 30 minutos

EUREKA! N°5, 1999


15
Sociedade Brasileira de Matemática

PROBLEMA 1
Determine todos os conjuntos finitos S de pontos do plano com pelo menos três
elementos que satisfazem a seguinte condição:

Para quaisquer dois pontos distintos A e B de S, a mediatriz do segmento


AB é um eixo de simetria de S.

PROBLEMA 2
Seja n  2 um inteiro fixo.
a) Determinar a menor constante C para a qual a desigualdade


1i  j n
x i x j ( x i2  x 2j ) C ( x
1i n
i )4

é válida para quaisquer números reais x1, …, xn  0.


b) Para esta constante C, determine quando ocorre a igualdade.
PROBLEMA 3
Considere um tabuleiro quadrado n  n, onde n é um inteiro positivo par fixo. O
tabuleiro está dividido em n2 quadrados unitários. Dizemos que dois quadrados
distintos do tabuleiro são adjacentes se eles têm um lado comum.
Marcam-se N quadrados unitários do tabuleiro de tal forma que qualquer
quadrado (marcado ou não) é adjacente a pelo menos um quadrado marcado.
Determine o menor valor possível para N.

Segundo Dia
Duração da prova: 4 horas e 30 minutos

PROBLEMA 4
Determine todos os pares (n, p) de inteiros estritamente positivos tais que

 p é primo,
 n  2p, e
 (p – 1)n + 1 é divisível por np–1.

PROBLEMA 5
Duas circunferências 1 e 2 estão contidas no interior de uma circunferência
 e são tangentes a  em pontos distintos M e N, respectivamente. A

circunferência 1 passa pelo centro de 2 . A reta que passa pelos dois pontos

EUREKA! N°5, 1999


16
Sociedade Brasileira de Matemática

de interseção de 1 e 2 intersecta  em A e B. As retas MA e MB


intersectam 1 respectivamente em C e D.
Prove que CD é tangente a 2 .

PROBLEMA 6
Determine todas as funções f : R  R tais que

f ( x  f ( y ))  f ( f ( y ))  x f ( y )  f ( x )  1

para quaisquer x, y  R.

ADEDANHA OU “DE COMO OS DEUSES MATEMÁTICOS


TROUXERAM A PAZ AO MUNDO”
Pablo Emanuel - IMPA
 Nível Iniciante

Diz a lenda que, há muitos milênios, o mundo vivia em guerra constante,


pois as pessoas não sabiam como resolver as suas discordâncias, a não ser pela
força bruta. Um dia, os deuses (que são exímios matemáticos), para resolver esta
situação, enviaram um mensageiro à Terra, com a missão de ensinar os homens a
resolverem as suas disputas. O anjo se dirigiu então aos homens, dizendo:

- Quando dois entre vós precisarem chegar a um acordo, que se faça


como vos digo: que um escolha par e o outro escolha ímpar, então que ambos
mostrem ao mesmo tempo a mão exibindo uma certa quantidade de dedos. Serão
então somadas estas quantidades. Se a soma for um número par declara-se
vencedor o jogador que escolheu par e, caso contrário, declara-se vencedor
aquele que escolheu ímpar.

Os homens ficaram maravilhados com a sabedoria dos deuses e, deste dia


em diante, houve um grande período de paz, pois todas as questões eram
resolvidas com o jogo que eles haviam aprendido dos deuses.
Um dia, porém, esta paz foi abalada. Três reis disputavam um pedaço de
terra, que ficava exatamente na divisa entre os três países. Eles estavam prontos a

EUREKA! N°5, 1999


17
Sociedade Brasileira de Matemática

utilizar o jogo divino do par-ou-ímpar, mas o rei que sabia mais matemática entre
os três se levantou e disse:

- Caros colegas, nós todos sabemos que um número só pode ser par ou
ímpar, não existindo uma terceira opção. Como somos três, algum de nós não vai
ter opção alguma.
Este era realmente um problema muito sério. Para resolvê-lo, foi
chamado o melhor matemático da Terra na época, chamado Zerinhoum. Ele
pensou durante várias semanas em como resolver o problema dos reis, e
finalmente chegou a uma solução:

- Majestades, encontrei a solução para o vosso problema. Ao mesmo


tempo, vós estendereis vossas mãos, mantendo-as ou com a palma para cima ou
com a palma para baixo. Aquele dentre vós que tiver a mão em posição diferente
dos demais ganha a disputa.

- E se todos nós tivermos as palmas das mãos viradas para o mesmo


lado? -indagaram os reis.
- Neste caso, majestades, vós jogareis novamente, até que algum entre
vós vença.
Como a disputa era muito urgente, os reis aceitaram a sugestão do
eminente matemático. Houve mais um período de paz, desta vez muito mais
curto. Em pouco tempo, as pessoas perceberam que o jogo de Zerinhoum podia se
alongar indefinidamente, e que era possível se fazer alianças para prejudicar
adversários políticos.

Então as pessoas rezaram aos deuses, pedindo um novo jogo, que


trouxesse de novo a paz à Terra. Os deuses então enviaram novamente um
mensageiro. Quando ele chegou, os homens lhe cercaram dizendo:

- Mensageiro dos deuses, atendeste as nossas preces. Vivíamos em


guerra, e os deuses nos enviaram o sagrado jogo do par-ou-ímpar, que nos trouxe
a paz. Mas este jogo só podia ser jogado por dois jogadores, e as trevas se
abateram de novo sobre nós. Então um grande homem nos ensinou um novo jogo,
que chamamos Zerinhoum em sua homenagem. Mas este jogo tinha problemas, e
a guerra voltou a nos assolar. Por favor, ó grande sábio, que vem em nome dos
deuses, ensina-nos um novo jogo, que possa nos trazer de volta nossa paz.
E o anjo assim respondeu:

EUREKA! N°5, 1999


18
Sociedade Brasileira de Matemática

- Eu vos ensinarei um novo jogo. Zerinhoum era um grande matemático,


mas não conhecia os segredos dos deuses. Eu vos revelarei estes segredos.
Para isto, o melhor é começar pelo antigo jogo do par-ou-ímpar. Como se
decide se um número é par ou é ímpar? Basta dividi-lo por 2. Se o resto for igual
a 0, o número será par, se for igual a 1, o número será ímpar. Estas são as únicas
duas opções, porque o resto sempre é menor do que o dividendo (2). Reparai que
se dividirmos o número por 3, passam a existir 3 opções para o resto, pois ele
pode ser 0, 1 ou 2. Na divisão por 4, existem 4 restos possíveis ( 0, 1, 2 e 3). Em
geral, quando dividimos um número por n , existem n restos possíveis ( 0, 1, 2,
…, n – 2 e n – 1 ).
E o que isto tem a ver com o jogo? Tudo, eu vos digo. Se n pessoas
estiverem em uma disputa, vós fareis como eu vos digo: As pessoas escolherão,
cada uma, um número entre 0 e n – 1 diferente. Depois, ao mesmo tempo, elas
mostrarão as mãos, exibindo uma quantidade qualquer de dedos. As quantidades
serão somadas, e o número resultante será dividido por n. A pessoa que escolheu
o resto desta divisão será a vencedora.

Esta é a forma que os deuses jogam. Mas vós da Terra sois muito
desorganizados para poder escolher tantos números de forma tranqüila. Portanto,
eu vos ensinarei uma forma alternativa de jogar este jogo. Vós vos arrumareis em
um círculo. Uma pessoa será designada a contar. Então vós gritareis a palavra
mágica “Adedanha” e todos mostrarão as mãos. Os resultados serão somados, e
aquele que havia sido designado fará o seguinte procedimento: Em primeiro lugar
falará “Um”, e apontará para o céu, para que nunca vos esqueçais de que foram os
deuses que vos ensinaram este jogo. Então apontará para si mesmo e falará
“Dois”. Depois apontará para o jogador à sua esquerda e falará “Três”, e depois
seguirá apontando para o jogador à esquerda deste e assim por diante, sempre
acrescentando um ao número que havia falado anteriormente, até chegar à soma
que havia sido calculada. O jogador que estiver sendo apontado neste momento
será o vencedor. Se a soma for 1, o jogador que estiver à direita do que estiver
contando será declarado vencedor. Se for 0, será o que estiver à direita deste.

Os homens entenderam as determinações do mensageiro, mas ainda não


entendiam porque o segundo jogo era equivalente ao primeiro. O anjo então lhes
explicou:

- A pessoa que está contando vai apontar para si mesma quando estiver
falando “2”. Depois vai dar uma volta completa no círculo e vai apontar para si
mesma novamente quando estiver no “2 + n”, e novamente no “2 + 2n”. Ou seja,
ela vai estar apontando para si mesma se e somente se estiver falando um número

EUREKA! N°5, 1999


19
Sociedade Brasileira de Matemática

cujo resto na divisão por n seja 2. Da mesma forma, vai estar apontando para o
jogador à sua esquerda se e somente se estiver falando um número que deixa
resto 3 ao ser dividido por n. E assim por diante, de forma que cada jogador terá
associado a si um número entre 0 e n – 1 tal que ele é o vencedor se e somente se
o resultado da soma deixa aquele resto quando dividido por n.

Os homens estavam maravilhados com a explicação do mensageiro, mas


um sábio ancião levantou uma questão:

- Ó, mensageiro divino, sem dúvida és sábio e sagaz. No entanto, uma


dúvida me corrói o espírito. Tendo cada jogador 10 dedos, esta soma pode atingir
números muito elevados, fazendo com que o responsável pela contagem passe um
tempo enorme falando e apontando até que se descubra o vencedor.

- Tens toda a razão, sábio homem. Mas em verdade vos digo que é tolice
que um jogador exiba uma quantidade de dedos maior ou igual à quantidade de
jogadores. Com efeito, suponde que um jogador coloque um número maior ou
igual a n. Os primeiros n dedos só vão ter o efeito de fazer com que a contagem
dê uma volta completa no círculo, sem alterar em nada quem será o vencedor.
Portanto, ele pode subtrair n da sua quantidade sem que isto altere o resultado. Se
o número persistir maior ou igual a n, basta voltar a subtrair, até que o número
fique entre 0 e n – 1.

- Isto de fato diminui sobremaneira o esforço requerido- replicou o


ancião. Mas ainda assim o resultado pode chegar a n(n – 1), que ainda é bastante
grande.

- És de fato perspicaz, meu nobre homem. Mas não penseis que a


sabedoria dos deuses possui limite. O mesmo processo que foi aplicado a cada
número individualmente pode ser aplicado à soma. Por exemplo, considerai um
jogo com 4 jogadores. Suponde que um dos jogadores exibe 3 dedos e outro exibe
2 dedos. Por que considerar a sua soma como sendo 5, se o efeito de somar 4 é
apenas fazer com que o responsável pela contagem dê uma volta a mais? Em vez
disto, é muito mais sensato considerar a sua soma como sendo 5 – 4 = 1. Mais
geralmente, considere um jogo com n jogadores. Em primeiro lugar diminui-se n
dos valores jogados por cada um, de forma que todos eles estejam entre 0 e n – 1
(se todos os jogadores dessem ouvidos às palavras dos deuses, não jogariam além
destes limites). Depois procede-se a soma, da seguinte forma. Soma-se o primeiro
valor com o segundo. Caso esta soma seja um valor maior ou igual a n, subtrai-se
n do resultado ( Sede espertos e sabereis que fazendo isto sempre obtereis um

EUREKA! N°5, 1999


20
Sociedade Brasileira de Matemática

número entre 0 e n – 1 ). Depois, a este resultado, soma-se o terceiro valor,


tomando-se o cuidado de se subtrair n caso a soma exceda n – 1. Prossegue-se
desta forma até que todos os valores tenham sido somados. Se seguistes o meu
raciocínio até este ponto, não deveria ser-vos surpresa o fato que o resultado de
uma tal operação está sempre entre 0 e n – 1, e portanto o jogador responsável
pela contagem nunca precisará dar mais de uma volta.

E então todos os habitantes se ajoelharam aos pés do anjo, reconhecendo


a sua suprema sabedoria, e o mundo conheceu enfim a paz. Até hoje os homens
jogam os jogos de par-ou-ímpar e adedanha da forma como foram ensinados
pelos deuses, embora, infelizmente, a maioria tenha se esquecido da lição final e
continue se extenuando em uma interminável contagem que dá voltas e mais
voltas.



E foi assim que a lenda me foi contada pela minha avó, que ouviu de sua
avó, que ouviu de sua própria avó, e assim por diante, até o princípio dos tempos.

Você deve estar achando meio esquisita a maneira de somar que foi
ensinada pelos deuses. No entanto, eles a usaram em várias outras coisas que nos
são muito familiares. Se você não acredita, responda rápido a estas perguntas:

a) Se uma coisa começa em uma segunda-feira e dura 7 dias, em que


dia ela termina? E se durar 14 dias? E se durar 701 dias?

b) Se uma coisa começa às 8 horas da manhã e dura 24 horas, a que


horas ela acaba? E se durar 48 horas? E se durar 4804 horas?

c) Se o ponteiro dos minutos de um relógio está apontando 23


minutos, para onde ele estará apontando daqui a 60 minutos? e daqui a
120 minutos? e daqui a 66681 minutos?

Garanto que, se você respondeu à terceira pergunta dos 3 ítens, não


contou de um em um (ou então já estamos no terceiro milênio ). Você percebeu
que os dias da semana se repetem de 7 em 7 dias, que as horas do dia se repetem
de 24 em 24 horas e que o ponteiro do relógio volta a apontar para o mesmo
ponto de 60 em 60 minutos. Garanto também que você, sem se dar conta, já

EUREKA! N°5, 1999


21
Sociedade Brasileira de Matemática

pensou várias vezes coisas como “5 horas depois das 21 horas são 2 horas da
manhã”, ou seja, fez a conta 21 + 5 = 2 ! E, por incrível que pareça, esta conta
está certa!!! Está certa, porque você está pensando a menos de múltiplos de 24
(ou, como preferem os matemáticos, módulo 24) , ou seja:

21 + 5 = 2 ( + um múltiplo de 24 ) ,

ou, como preferem os matemáticos,

21 + 5 = 2 (mod 24) .

Desta forma, a terceira pergunta do item c) pode ser reescrita como


“Quanto é 23 + 66681 (mod 60)” . Se você foi esperto(a) o suficiente para
responder àquela pergunta, você já deve ter percebido que 66681 = 21 (mod 60),
e que 23 + 66681 = 23 + 21 (mod 60), ou seja, 23 + 66681 = 44 (mod 60), logo o
ponteiro estará apontando para o minuto 44. Só para ver se você entendeu até
agora, preencha estas lacunas:

2 + 2 = 1 (mod __ )
2 +__ = 0 (mod 17)
26 = 3 (mod __ )

Não se esqueça que a expressão “mod n” é só uma forma abreviada de “+


um múltiplo de n”. Lembrando-se disto, veja quantas coisas você sabia, mas não
sabia que sabia:

3  3 = 1 (mod 4)
1 = – 1 (mod 2)
2  2  2  2 = 1 (mod 5)
3  3  3  3 = 1 (mod 5)

(esta talvez você não saiba, mas n  n  n  n = 1 (mod 5), sempre que n
não é múltiplo de 5. Você pode ver isto e muito mais no artigo do professor
Carlos Gustavo Moreira, na EUREKA! No. 2. Pergunta: se n é múltiplo de 5,
quanto é n  n  n  n (mod 5)? )

Agora que você já sabe o segredo dos deuses matemáticos, já pode jogar
adedanha da forma original, como os deuses a conceberam, e manter a paz no
mundo sem fazer esforço.

EUREKA! N°5, 1999


22
Sociedade Brasileira de Matemática

QUADRILÁTEROS E TRIÂNGULOS
Marcelo Mendes
 Nível Intermediário

Apresentamos a seguir alguns resultados que servem de ferramenta para


resolução de problemas de geometria elementar envolvendo quadriláteros e
triângulos, bastante freqüentes em problemas de olimpíada.

QUADRILÁTEROS INSCRITÍVEIS

Os ângulos opostos de um quadrilátero inscritível são suplementares.


Reciprocamente, se os ângulos opostos de um quadrilátero são suplementares,
então esse quadrilátero é inscritível (cíclico).

EUREKA! N°5, 1999


23
Sociedade Brasileira de Matemática

A B

Além disso, se ocorrer uma situação onde dois ângulos iguais “olham” para um mesmo
segmento, então os extremos desse segmento e os vértices dos dois ângulos formam um
quadrilátero inscritível.

D
 C

A B

Exemplo: Seja AB o diâmetro de um semicírculo. Um ponto M é marcado no


semicírculo e um ponto K é marcado sobre AB. Um círculo com o centro P passa
por A, M, K e um círculo com centro Q passa por M, K, B. Prove que M, K, P, Q
pertencem a um mesmo círculo.

Solução: No círculo circunscrito de AMK, MPK = 2MAK; e no círculo


circunscrito de BMK, MQK = 2MBK. Como AB é diâmetro do semicírculo,
AMB = 90o e MAK+MBK = 90o. Daí, MPK+MQK = 180o e MPKQ é
inscritível.

TEOREMA DE PTOLOMEU

Se ABCD é um quadrilátero inscritível de diagonais AC e BD,


então:
AB  CD + AD  BC = AC  BD.

EUREKA! N°5, 1999


24
Sociedade Brasileira de Matemática

A
d D

a
c
E

B b C

Prova: Seja x = BD e y = AC e a, b, c, d, os comprimentos dos lados. Construa


CDE = ABD, E  AC. Daí, CDE  ADB e ADE  BCD, dando,
respectivamente, ECx = ac e AEx = bd. Somando essas duas últimas equações,
temos xy = ac + bd, como queríamos provar 

Há também uma extensão para esse teorema que vale para quadriláteros não
inscritíveis: AB  CD + AD  BC > AC  BD, isto é, numa situação geral vale
AB  CD + AD  BC  AC  BD.

Exemplo: Prove que, se ABCDEFG é um heptágono regular convexo, então:

1 1 1
  .
AB AC AD

Aplicando o Teorema de Ptolomeu no quadrilátero inscritível ACDE, onde


CD = DE = a = AB, AC = CE = b e AD = AE = c, temos bc = ac + ab. Dividindo
essa última equação por abc, segue o resultado.

A RELAÇÃO ENTRE A DISTÂNCIA DO ORTOCENTRO A UM VÉRTICE


E DO CIRCUNCENTRO AO LADO OPOSTO

Sejam H e O respectivamente o ortocentro e o circuncentro, do ABC e


M, o ponto médio do lado BC. Então AH = 2OM.

EUREKA! N°5, 1999


25
Sociedade Brasileira de Matemática

A
Y
N
H O

B X M C

Prova: Sejam AX e BY alturas e N, o ponto médio de AC. Como MN é base


média, MN // AB e MN = ½AB. Daí, ABH  OMN pois têm lados paralelos
entre si (e razão 2:1). Portanto, AH = 2OM 

Exemplo: Prove que o ortocentro, o baricentro e o circuncentro de um triângulo


qualquer são colineares. (Reta de Euler)

Seja G a interseção de AM e HO (na figura acima). Então, AHG  GOM na


razão 2:1. Daí, AG = 2GM. Portanto, G é o baricentro e pertence à reta HO.

PROBLEMAS

1. Seja P um ponto sobre o menor arco AC da circunferência circunscrita a


um triângulo equilátero ABC. Calcule a medida do ângulo APC.

2. Prove que um trapézio é inscritível se, e somente se, ele for isósceles
(lados não paralelos iguais).

3. Sejam AX e BY alturas de um triângulo isósceles ABC (AC = BC) de


ortocentro H. Prove que 2HXXC = XYHC.

4. Seja ABCD um losango inscritível de lado 1 e P, um ponto sobre o menor


arco CD. Prove que PD2 + PCPA = 1.

5. Seja P um ponto sobre o menor arco AC da circunferência circunscrita a


um triângulo equilátero ABC. Prove que PB = PA + PC.

6. Seja H o ortocentro de um triângulo ABC e P, o ponto diametralmente


oposto a B na circunferência circunscrita a ABC. Prove que AHCP é um
paralelogramo.

EUREKA! N°5, 1999


26
Sociedade Brasileira de Matemática

7. ABCD é um paralelogramo. H é o ortocentro do ABC e O, o


circuncentro do ACD. Prove que H, O, D são colineares.
8. Seja A1A2…An um polígono regular de n lados. Se
1 1 1
  , calcule n.
A1 A2 A1 A3 A1 A4

9. Sejam M, N, P os pontos médios dos lados de um ABC acutângulo de


circuncentro O. Prolongue MO, NO, PO, a partir de O, até X, Y, Z,
respectivamente, tais que MX 2 OM , NY 2 ON , PZ 2 OP.
Prove que XYZ é semelhante ao ABC .

10. Sejam M, N, P os pontos médios dos lados de um ABC acutângulo de


circuncentro O. Prolongue MO, NO, PO, a partir de O, até X, Y, Z,
respectivamente, tais que MX, NY, PZ tenham comprimentos
respectivamente iguais às metades das alturas do triângulo a partir dos
vértices A, B, C. Prove que XYZ é semelhante ao triângulo órtico de
ABC (triângulo formado pelos pés das alturas do ABC).

O PRINCÍPIO DAS GAVETAS


Paulo Cezar Pinto Carvalho - IMPA
 Nível Intermediário
Muitos problemas atraentes de matemática elementar exploram relações entre
conjuntos finitos, expressas em linguagem coloquial. Parte de sua atração vem
justamente do fato de que podem ser formulados e, muitas vezes, resolvidos sem
recorrer a fórmulas ou a técnicas complicadas. Vejamos um exemplo simples.

EUREKA! N°5, 1999


27
Sociedade Brasileira de Matemática

Exemplo 1. Qual é o número mínimo de pessoas que devemos reunir para que
tenhamos certeza de que entre elas há duas que fazem aniversário no mesmo
mês?

Solução: A resposta é 13. Se houvesse apenas 12 pessoas, seria possível que


cada uma delas fizesse aniversário em um mês diferente. Com 13 pessoas, há ,
obrigatoriamente, pelo menos um mês com mais de um aniversário (se houvesse,
no máximo, um aniversário por mês, o número de pessoas presentes seria, no
máximo, 12).

O argumento empregado acima é conhecido como Princípio das Gavetas de


Dirichlet ou Princípio das Casas do Pombos. Um possível enunciado para este
princípio é o seguinte:

Se n objetos forem colocados em, no máximo, n – 1 gavetas, então pelo menos


uma delas conterá pelo menos dois objetos.

(Uma maneira um pouco mais formal de dizer o mesmo é: se o número de


elementos de um conjunto finito A é maior do que o número de elementos de um
outro conjunto B, então uma função de A em B não pode ser injetiva.)

Embora trate-se de um fato extremamente elementar, ele é útil para resolver


problemas que, pelo menos à primeira vista, não são imediatos. Para aplicá-lo,
devemos identificar, na situação dada, quem faz o papel dos objetos e quem faz o
papel das gavetas.

Exemplo 2. Uma prova de concurso possui 10 questões de múltipla escolha, com


cinco alternativas cada. Qual é o menor número de candidatos para o qual
podemos garantir que pelo menos dois deles deram exatamente as mesmas
respostas para todas as questões?
Solução: Neste caso, os objetos são os alunos e as gavetas são as possíveis
seqüências de respostas . Como cada questão pode ser respondida de 5 modos, a
prova pode ser preenchida de 5  5  5  … 5 = 510 = 9 765 625 modos. Logo, só
se pode ter a certeza de que dois candidatos fornecem exatamente as mesmas
respostas se houver pelo menos 9 765 626 candidatos.

Exemplo 3. Em uma reuniao há n pessoas. Mostre que existem duas pessoas que
conhecem exatamente o mesmo número de outros participantes (admitimos que
“conhecer”seja uma relação simétrica, ou seja, se a conhece b, então b conhece
a).

EUREKA! N°5, 1999


28
Sociedade Brasileira de Matemática

Solução: Os objetos são as pessoas. As gavetas, naturalmente, são as quantidades


de pessoas conhecidas. Temos, no entanto, uma dificuldade: as possíveis
quantidades de conhecidos são 0, 1, 2, …, n – 1. Assim, à primeira vista, temos n
gavetas para n objetos, o que nos impede de usar o princípio das gavetas. Note,
porém, que as gavetas 0 e n – 1 não podem ser usadas simultaneamente: se existir
uma pessoa que não conhece nenhum participante, então não pode existir um
participante que conheça todos! Assim, uma das gavetas 0 ou n – 1 permanece
desocupada e os n objetos devem ser, portanto, distribuídos em n – 1 gavetas.
Portanto, uma delas será ocupada por pelo menos dois objetos, o que mostra que
há duas pessoas que conhecem exatamente o mesmo número de participantes.

Nos casos anteriores, foi bastante simples identificar as gavetas. Nem sempre
é assim. Os exemplos a seguir ilustram situações em que é necessário “construir”
as gavetas a serem usadas.

Exemplo 4: Escolhem-se 5 pontos ao acaso sobre a superfície de um quadrado de


lado 2. Mostre que pelo menos um dos segmentos que eles determinam tem
comprimento menor ou igual a 2 .

Solução: Neste caso, está claro que os objetos são os 5 pontos. O ponto chave da
resolução está na identificação das gavetas. Devemos subdividir o quadrado dado
em 4 partes de modo tal que a distância entre dois pontos situados em uma destas
partes nunca seja maior que 2 . A Fig. 1 mostra como fazê-lo: basta dividi-lo
nos quatro quadrados determinados pelas retas que unem os pontos médios dos
lados opostos. Em cada uma destas quatro “gavetas”, a distância máxima entre
dois pontos é igual à sua diagonal, que mede 2 . Portanto, dados 5 pontos, pelo
menos 2 estarão em uma mesma “gaveta” e, assim, determinam um segmento de
comprimento menor ou igual a 2 .
1 1

1 2

Figura 1

EUREKA! N°5, 1999


29
Sociedade Brasileira de Matemática

Exemplo 5. Escolha 101 números dentre os elementos do conjunto {1, 2, 3, …,


200}. Mostre que, dentre os números escolhidos, há sempre dois números tais
que um divide o outro.

Solução: Antes de mais nada, observe que podemos escolher 100 números do
conjunto sem que exista um par onde um número divide o outro: basta tomar os
números 101, 102, …, 200. É claro que se acrescentamos mais um número p
(obrigatoriamente menor ou igual a 100) a essa coleção, um múltiplo seu já estará
lá. Na verdade, podemos garantir que esse múltiplo é da forma 2 rp (basta tomar p
e multiplicá-lo sucessivamente por 2 até que ele se torne maior do que 100).
Mostraremos que isso ocorre para qualquer conjunto de 101 elementos. Ou seja,
todo subconjunto com 101 dos números de 1 a 200 sempre contém um número e
um múltiplo seu obtido através de multiplicação por uma potência de 2. Note
que esta afirmativa é mais forte do que a dada do enunciado, mas, como veremos,
nos permite estruturar uma demonstração. Isto ocorre com frequência nos
problemas envolvendo o Princípio das Gavetas: parte do sucesso nas soluções
depende da habilidade em perceber o que deve ser demonstrado. Voltando à
solução, observemos que todo inteiro n se escreve, de modo único, na forma n =
2rb, onde r é um inteiro não negativo e b é um número ímpar. Por exemplo, 18 =
21.9 , 36 = 22.9 e 125 = 20.125. Para os números de 1 a 200, os valores possíveis
de b são os ímpares de 1 a 199, que são 100. Aqui estão nossas gavetas! Já que
há 100 valores possíveis de b, qualquer coleção de 101 números de 1 a 200 possui
dois números x = 2rb e y = 2sb com o mesmo b (isto é, temos dois objetos que
serão colocados na mesma gaveta). Se r < s, então x divide y; senão, y divide x, o
que conclui a demonstração.

O último exemplo requer um argumento um pouco mais sofisticado.

Exemplo 6: Em uma reunião, há 6 pessoas. Mostre que necessariamente existem


3 pessoas que se conhecem mutuamente ou 3 pessoas que não se conhecem
mutuamente (como no exemplo 3 admitimos que, se a conhece b, então b
conhece a).
Solução: Usaremos o diagrama da Fig. 2 abaixo para ilustrar a situção. Cada
pessoa é representada por um vértice do hexágono. Quando duas pessoas se
conhecem, ligamos os vértices correspondentes por um segmento contínuo;
senão, usamos um segmento tracejado. O que devemos mostrar é que, nesta
figura, necessariamente existe um triângulo formado por linhas contínuas ou um
triângulo formado por linhas tracejadas.

EUREKA! N°5, 1999


30
Sociedade Brasileira de Matemática

Figura 2

Consideremos os segmentos que incidem em um dos vértices p1. Como eles são
5, há pelo menos 3 deles que são contínuos ou pelo menos 3 que são tracejados.
Admitamos que haja 3 contínuos (o argumento seria análogo no outro caso).
Denotemos por p2, p3 e p4 vértices ligados a p1 por segmentos contínuos (veja a
Fig. 3). Se algum dos segmentos p2p3, p2p4 ou p3p4 é contínuo, este segmento,
juntamente com os que ligam seus extremos a p1, formam um triângulo contínuo.
Por outro lado, se nenhum deles é contínuo, eles formam um triângulo tracejado,
o que completa a demonstração.

p2

p1 p3

p4

Figura 3

Este exemplo é um caso particular de um teorema mais geral, chamado de


Teorema de Ramsey. Dado qualquer inteiro k  3, existe um inteiro R(k) tal que,
em uma reunião de R(k) pessoas, sempre existem k que se conhecem
mutuamente ou k que não se conhecem mutuamente. Este resultado normalmente
é expresso usando a linguagem de grafos: ao se colorir, com duas cores, as
arestas de um grafo completo com R(k) vértices, há sempre um subgrafo
completo com k vértices onde todas as arestas têm a mesma cor. (Na realidade, o
Teorema de Ramsey aborda situações mais gerais; veja, por exemplo os
problemas 8 e 9 abaixo).

EUREKA! N°5, 1999


31
Sociedade Brasileira de Matemática

Aproveitamos para mencionar a seguinte generalização do princípio das


gavetas: Se n objetos são colocados em m gavetas e n > mk (onde m, n e k são
números naturais) então alguma gaveta conterá pelo menos k + 1 objetos.

Terminamos com uma lista de problemas que podem ser resolvidos com as
técnicas aqui ilustradas. As soluções serão publicadas nos próximos números da
EUREKA!.

PROBLEMAS

1) Numa gaveta há 6 meias pretas e 6 meias brancas. Qual é o número mínimo


de meias a se retirar (no escuro) para garantir que:

a) As meias retiradas contenham um par da mesma cor?


b) As meias retiradas contenham um para de cor branca?

2) Sejam n um natural ímpar e A uma matriz simétrica em que cada linha e


coluna seja uma permutação dos inteiros 1, 2,…, n.
Mostre que cada um destes números aparece uma vez na diagonal de A.

3) Mostre que se um subconjunto com n + 1 elementos é escolhido do conjunto


{1, 2, 3,…, 2n} então este subconjunto necessariamente contém um par de
números primos entre si.

4) Considere 9 pontos de coordenadas inteiras no R3. Mostre que o ponto médio


de um dos segmentos de reta definidos por estes pontos também tem
coordenadas inteiras.

5) Mostre que se n é ímpar e a1, a2,…,an é uma permutação de 1, 2,…,n, então o


produto (a1 – 1) (a2 – 2)…(an – n) é par.

6) Mostre que em qualquer coleção de n inteiros há um subconjunto cuja soma


dos elementos é divisível por n.

7) Mostre que em qualquer coleção de n inteiros existe um par cuja soma ou


diferença é divisível por n.

EUREKA! N°5, 1999


32
Sociedade Brasileira de Matemática

8) Mostre que em toda reunião com 10 pessoas existem 3 que se conhecem


mutuamente ou 4 que se desconhecem mutuamente. Mostre que, na realidade,
o resultado vale mesmo que na reunião só existam 9 pessoas.

9) Dados inteiros a, b  2, seja N (a, b) o menor número para o qual, dado um


conjunto com N (a, b) pessoas, sempre existam a que se conheçam
mutuamente ou b que se desconheçam mutuamente (se existir tal número). Os
problemas anteriores implicam que N (3, 3)  6 e N (3, 4)  9. Mostre que:

a) N(a, 2) = a;
b) N(a, b) = N (b, a);
c) N(a, b)  N (a – 1, b) + N (a, b – 1); observe que, em consequência,
N(a, b) existe para todo par (a, b).

10) Dois discos A e B são divididos em 2n setores iguais. No disco A, n setores


são pintados de azul e n de vermelho. No disco B, os setores são pintados de
azul ou vermelho de forma completamente arbitrária.
Mostre que A e B podem ser superpostos de modo que pelo menos n setores
tenham cores coincidentes.

11) Sejam A1, A2,…, A100 subconjuntos distintos de um conjunto X satisfazendo a


propriedade de que cada Ai possua mais da metade dos elementos de X.
Mostre que existem 6 elementos x1, x2,…x6 de X tais que cada Ai contenha
pelo menos um destes 6 elementos.

12) Considere um conjunto A com n elementos. Seja F uma família de


subconjuntos de A tal que:

- Quaisquer dois elementos de F têm interseção não vazia.


- Nenhum outro subconjunto de A intersecta todos os elementos de F.

a) Dê exemplo de uma família F satisfazendo a estas condições.


b) Mostre que F possui 2n – 1 elementos.

13) Uma fábrica produz pelo menos uma unidade de um produto X por dia e no
máximo 10 unidades deste produto por semana. Mostre que dado qualquer
inteiro positivo n existe um conjunto de dias consecutivos em que a produção
total é igual a n [ Sugestão: mostre que existe um número k (dependente de n)
suficientemente grande para o qual os conjuntos {S1, S2,…Sk} e {S1 + n, S2 +

EUREKA! N°5, 1999


33
Sociedade Brasileira de Matemática

n, …, Sk + n} tem pelo menos um elemento comum, onde Si é a soma das


produções nos dias 1, 2, …, i.].

14) Mostre que toda sequência com n2 + 1 elementos possui uma subsequência
crescente com n + 1 elementos ou uma subsequência decrescente com n + 1
elementos.

15) Sejam mn + 1 elementos tais que a1 < a2 < …< amn + 1. Mostre que ou existem
m + 1 destes números tais que nenhum é divisor de um outro ou existem n + 1
deles tais que cada um é divisor do seguinte.

16) Prove que se o conjunto {1, 2, 3, …, 1978} é partido em 6 subconjuntos, em


algum destes subconjuntos existe um elemento que é igual à soma de dois
elementos, não necessariamente distintos, do mesmo subconjunto.

17) Considere um conjunto com 2n pontos.

a) Mostre que é possível conectar estes pontos com n2 segmentos de reta sem
que um triângulo de vértices nos pontos dados seja formado.
b) Mostre que se os pontos são conectados por n2 + 1 segmentos de reta, então
pelo menos um triângulo é formado.

18) Considere um conjunto de n pontos 1, 2, …, n. Para cada par de pontos é


escolhida uma orientação para o segmento de reta que os une. Se o segmento
ij é orientado de i para j dizemos que i  j. Mostre que existe uma
permutação a1, a2, … an de 1, 2, …, n tais que a1  a2  …  an.

19) São dados n pontos azuis e n pontos vermelhos no plano. Mostre que é
possível formar n pares de pontos (um azul e um vermelho em cada par) de
modo que os n segmentos de reta definidos por estes pares não se cruzem.

20) Mostre que dados 5 pontos do plano em posição geral há 4 que formam um
quadrilátero convexo.

DESIGUALDADES ELEMENTARES
Antonio Caminha Muniz Neto

EUREKA! N°5, 1999


34
Sociedade Brasileira de Matemática

 Nível Avançado.

Pretendemos neste artigo desenvolver ferramentas básicas a fim de que o


leitor se torne apto a resolver uma vasta gama de problemas de competições
matemáticas que envolvam desigualdades. Tentamos tornar nossa exposição a
mais auto-suficiente possível. Em certas passagens, contudo, algum
conhecimento de cálculo é útil, ainda que não imprescindível. Em tais ocasiões
indicamos ao leitor a referência [3] como bibliografia auxiliar.

Antes de discutirmos qualquer desigualdade em especial, consideremos um


exemplo preliminar.

Exemplo 1: Para todo inteiro positivo n, prove que


1 1 1 1 1
1  2  3  4 ...  n  2 log 2 n  1 .  
Solução : Veja que, para todo inteiro k > 1,

1
 1  ...  1k  1 1 1
k  k  ... k  12
2 k  1 1 2 k  1  2 2 2 2
     2
2 k  1 vezes
Portanto, sendo 2 k a maior potência de 2 menor ou igual a n, temos
n 2k k k
1 1
2   1
j 1  1
2   1j 1  12    2 j 1
1
1
 ...  1
2j  1  1
2   12 1  k
2
j 3 j 3 j 2 j 2


Mas 2 k n  2 k 1  k log 2 n  k  1  1  k2  12 log 2 n  1 , e a desigualdade do 
enunciado é imediata. 

O exemplo acima foi colocado de propósito. Ele chama atenção para o fato de
que nem sempre precisamos de algo mais que raciocínio para resolver problemas
envolvendo desigualdades. A proposição abaixo mostra um pouco mais sobre
como podemos derivar desigualdades interessantes com muito pouca matemática.

EUREKA! N°5, 1999


35
Sociedade Brasileira de Matemática

Proposição 1 (Desigualdade do Rearranjo): Sejam a1  a 2  ...  a n reais


positivos dados, e considere a expressão S  a1b1  a 2 b2  ...a n bn , onde
( b1 , b2 ,..., bn ) é uma reordenação de ( a1 , a2 ,..., a n ) . Então
a1a n  a2 a n  1  ...a n a1  S a12  a22  ...a n2

Prova : Vamos primeiro tornar S a maior possível. Como só há um número finito


(n fatorial) de possíveis reordenações ( b1 , b2 ,..., bn ) , há uma delas que torna S
máxima. Suponha então que estamos com a reordenação ( b1 , b2 ,..., bn ) que
torna S máxima.

Queremos mostrar que essa reordenação é exatamente ( a1 , a2 ,..., a n ) . Para


isso, basta mostrarmos que deve ser b1  b2  ...  bn . Suponha o contrário, i.e.,
que existam índices i < j tais que bi  b j . Trocando as posições de bi e b j
(i.e., pondo bi ao lado de a j e b j ao lado de a i ), S varia de
ai b j  a j bi  ( ai bi  a j b j ) ( ai  a j )( b j  bi )  0 , quer dizer, S aumenta. Mas
isso contraria o fato de ser a reordenação ( b1 , b2 ,..., bn ) aquela que torna S
máxima. Logo, b1  b2  ...  bn e daí bi a i para todo i, donde o maior valor
possível de S é a12  a 22  ...a n2 .
O raciocínio para minimizar S é análogo. 
Passemos agora a nosso principal objetivo, o estudo de desigualdades especiais. A
mais importante destas é a dada pela proposição 2 abaixo. Antes, uma definição.

Definição 1 : Dados n > 1 reais positivos a1 , a 2 ,..., a n , definimos

a  a  ...a n
i. A média aritmética de a1 , a 2 ,..., a n como o número 1 2 .
n
ii. A média geométrica de a1 , a 2 ,..., a n como o número n a1a 2 ... a n .

Proposição 2 (Desigualdade Entre as Médias Aritmética e Geométrica) : Dados


n > 1 reais positivos a1 , a 2 ,..., a n , sua média aritmética é sempre maior ou
igual que a média geométrica, ocorrendo a igualdade se e só se a1 , a 2 ,..., a n
forem todos iguais. Em símbolos:
a1  a 2  ...a n
n  n a1a2 ... a n

EUREKA! N°5, 1999


36
Sociedade Brasileira de Matemática

Prova : Façamos a prova em dois passos:

i. A desigualdade é verdadeira quando n for uma potência de 2, ocorrendo a


igualdade se e só se todos os números forem iguais.
ii. A desigualdade é verdadeira em geral, e a igualdade ocorre se e só se os
números forem todos iguais.

i. Façamos indução sobre k  1, sendo n 2 k : Para k = 1, temos


a1  a 2
2
 a1a2  a1  2 a1a2  a2 0  ( a1  a2 )2 0 , o que é verdade. Há
a2 ) 2  0 , i.e., se e só se a1  a 2 .
igualdade se e só se ( a1 
a  a  ... a n n
Se já provamos que 1 2  a1a2 ... a n , com igualdade se e só se
n
a1 ...  a n para n 2 k , então

 
( a1 ... a n )  ( a n 1 ... a 2 n ) a1 ... a n a n 1 ... a 2 n n a1 ...a n  n a n 1 ...a 2 n
2n  12 n  n  2 
 n a1... a n n a n 1... a2 n  2 n a1... a n a n 1... a2 n

Para haver igualdade, devemos ter igualdade em todas as passagens. Então, deve
ser

a1 ... a n a n 1 ...a 2 n
n  n a1... a n , n
 n a n 1... a2 n e

n a1 ...a n  n a n 1 ...a 2 n
2  2 n a1... a n a n 1... a 2 n
Para as duas primeiras igualdades, segue da hipóteses de indução que deve
ser
a1 ... a n e a n 1 ...  a2 n
A última igualdade ocorre se e só se n a1... a n  n a n 1... a 2 n . Estas duas
condições juntas implicam que devemos ter a1 ...  a n  a n 1 ...  a 2 n . É
também evidente que se os números forem todos iguais a igualdade ocorre.

EUREKA! N°5, 1999


37
Sociedade Brasileira de Matemática

ii. Seja agora n > 1 um natural qualquer e a1 ,a 2 ,...,a n reais positivos.


Tome k natural tal que 2 k  n. Usando a desigualdade entre as médias para os
2 k números a1 , a 2 ,..., a n e 2 k  n cópias de a  n a1a2 ... a n , obtemos

a1  ...a n  a  ... a k k 2k k 2k k
k 2 a1... a n a 2 n
 a na 2 n
 a 2 a ,
2

a1  ...a n
e daí a1  ...a n  ( 2 k  n )a 2 k a , ou ainda a  n a1 ... a n , que era a
n
desigualdade desejada.

Para haver igualdade, segue do item i que deve ser a1 ...  a n  a ...  a . Em
particular, todos os números a1 , a 2 ,..., a n devem ser iguais. É fácil ver que se
esses números forem todos iguais então há igualdade. 
Corolário 2.1 : Dados n > 1 reais positivos a1 , a 2 ,..., a n , temos

( a1  a2  ...a n )  1
a1  a1  ... a1  n 2 ,
2 n

com igualdade se e só se a1 , a 2 ,..., a n forem todos iguais.

Prova : Basta aplicarmos duas vezes a proposição 2 e multiplicarmos os


resultados:
1 1 1 1
a1  a 2  ... a n n a1a 2 ...a n
a1  a2  ...a n  n n a1a2 ... a n e n


Exemplo 2 : (Olimpíada Israelense) Sejam k e n inteiros positivos, n > 1. Prove
que
1
kn  1
kn 1  ... kn 1n  1  n  n k 1
k 1 
Prova : Basta ver que

 n 1  n 1 n 1 n 1
 1   n 

 j 0
kn  j 
 j 0
 1
kn  j 
1  
j 0
kn  j 1
kn  j  nn 
j 0
kn  j 1
kn  j n n k 1
k ,

EUREKA! N°5, 1999


38
Sociedade Brasileira de Matemática

onde aplicamos a desigualdade entre as médias aritmética e geométrica uma vez.


kn  j 1
Note que, como os números kn  j são dois a dois distintos, não há igualdade,
razão do sinal > acima. 

Dentre todas as desigualdades especiais que temos oportunidade de usar em


problemas de competições matemáticas, a desigualdade a seguir se constitui,
juntamente com a desigualdade entre as médias aritmética e geométrica, num dos
dois mais importantes resultados a serem guardados.

Proposição 3 (Desigualdade de Cauchy): Sejam a1 , ..., a n , b1 , ..., bn reais


dados, não todos nulos (n > 1). Então

| a1b1  ...  a n bn |  a12 ...  a n2 b12 ...  bn2

Além disso, teremos a igualdade se e só se os a i e os bi forem proporcionais,


i.e., se e só se existir um real positivo  tal que bi  ai para todo i.

Prova : Considere o seguinte trinômio do segundo grau


f ( x ) ( a1 x  b1 )2  ( a 2 x  b2 ) 2  ...( a n x  bn ) 2

Desenvolvendo os parênteses, chegamos a

f ( x ) ( a12  a 22  ...a n2 ) x 2  2( a1b1  a 2 b2  ...a n bn ) x  ( b12  b22  ...bn2 )


Por ser uma soma de quadrados, temos f ( x ) 0 para todo real x, e daí deve ser
 0 , i.e.,
4( a1b1  a 2 b2  ...a n bn ) 2 4( a12  a 22  ...a n2 )( b12  b22  ...bn2 )
Cancelando o fator 4 e extraindo a raiz quadrada de ambos os membros,
chegamos na desigualdade de Cauchy. Examinemos agora a igualdade. Se houver
igualdade, quer dizer, se for  0 , então o trinômio tem uma raiz real  :
( a1  b1 )2  ( a 2   b2 )2  ...( a n   bn )2 0
Mas aí todos os parênteses devem ser nulos, i.e., bi  ai para todo i. Então,
havendo igualdade os a i e bi devem ser proporcionais. É evidente que se eles
forem proporcionais a igualdade ocorre. 
Temos a seguir alguns corolários importantes.

EUREKA! N°5, 1999


39
Sociedade Brasileira de Matemática

Corolário 3.1 (Desigualdade entre as Médias Quadrática e Aritmética) : Dados


a12  a 22  ... a n2 a1  a 2 .. a n
reais positivos a1 , ..., a n , temos  , com igualdade
n n
se e só se a1  ... a n .

Prova Fazendo b1  b2 ...  bn 1 na desigualdade de Cauchy, obtemos

a1  a 2  ...  a n  a12 ...  a n2 n,

com igualdade se e só se existir um real positivo  tal que ai  para todo i,


quer dizer, se e só se os a i forem todos iguais. Para obter a desigualdade do
enunciado, basta dividir ambos os membros da desigualdade acima por n. 
Corolário 3.2 : Se n > 1 é inteiro e a1 , ..., a n , b1 , ..., bn são reais positivos,
então

 a1
b1
a

 ... bn  a1b1 ...  a n bn   a1 ...  a n 
n
2
,

com igualdade se e só se b1  ...  bn .

a
Prova : Faça x i  bi , y i  ai bi e aplique a desigualdade de Cauchy para os
i

números x1 ,..., x n , y1 ,..., y n . 


Exemplo 3 : (Teste de Seleção da Romênia para IMO) Sejam x1 , x 2 ,..., x n 1
reais positivos tais que x1  x 2  ... x n x n 1 . Prove que

x1 ( x n 1  x1 )  ... x n ( x n 1  x n )  x n 1 ( x n 1  x1 )  ... x n 1 ( x n 1  x n )
Prova : Para 1  j  n , seja y j  x n 1  x j . Pela desigualdade de Cauchy
temos

x1 y1  ... x n y n  x1  ... x n y1  ... y n 

 x n 1 ( x n 1  x1 )  ... ( x n 1  x n ) 

Temos mais duas desigualdades importantes.

EUREKA! N°5, 1999


40
Sociedade Brasileira de Matemática

Proposição (Chebychev): Sejam a1 , ..., a n , b1 ,..., bn


4 reais, com
a1 a2 ... a n e b1 b2 ... bn . Então

 a1  a 2 ... a n
n  b1  b2 ... bn
n  a1b1  a 2 b2  ... a n bn
n
,

com igualdade se e só se a1 a2 ... a n ou b1 b2  ... bn .

Prova :
a1b1  a 2 b2  ...a n bn
n   a1  a 2 ...a n
n  b1  b2 ...bn
n 
 1
n2
 n a1b1  a2b2  ...an bn    a1  a2  ...an  b1  b2  ...bn   
n
1
= n2
 ( ai  a j )( bi  b j ) 0 ,
i , j 1

já que os ai , bi são igualmente ordenados.

Note que a condição do enunciado é suficiente para haver igualdade. Por outro
lado, suponha que tenhamos a igualdade. Como ( a i  a j )( bi  b j ) 0 para todos
i, j, devemos ter ( a i  a j )( bi  b j ) 0 para todos os i, j. Suponha que existisse
um índice k com bk  bk 1 . Então b1 ... bk  bk 1 ... bn , e de
( ai  a k 1 )( bi  bk 1 )  0 segue que a i  a k 1 para i  k. Portanto
a1 a2 ... a k a k 1 . De ( a i  a k )( bi  bk ) 0 e i  k concluímos que
a k 1 ... a n . Logo, todos os a i devem ser iguais. 
Corolário 4.1 : Sejam a1 , a 2 ,..., a n reais positivos e k um natural. Então
a1  a 2  ... a n k
a1k  a 2k  ... a nk
n   n  ,

com igualdade se e só se todos os a i forem iguais ou k  {0, 1}.

Prova : Por indução, o resultado acima é trivialmente verdadeiro para k = 1.


Suponha k > 1 e o resultado válido para k - 1. Como ambos os membros da
desigualdade acima são invariantes por permutações dos índices 1, 2, ..., n,
podemos supor sem perda de generalidade que a1 a 2 ... a n . Daí,

EUREKA! N°5, 1999


41
Sociedade Brasileira de Matemática

a1k  1 a 2k  1 ... a nk  1 , e da desigualdade de Chebychev obtemos


a1k  a 2k  ... a nk
n   a1  a 2  ... a n
n   a1k  1  a 2k  1  ...  a nk  1 
n
.

a1  a 2  ...  a n k  1
Pela hipótese de indução, vem que
a1k  1  a 2k  1  ...  a nk  1
n 
 n  .

Combinando as duas desigualdades acima segue o resultado. A condição de


igualdade é óbvia a partir da desigualdade de Chebychev. 

Por fim, vejamos algo um pouco mais sofisticado.

Definição 2 : Seja I um intervalo da reta e f : I  R uma função. A função f é


dita
i. Convexa se f  x y
2
f ( x) f ( y)
2 para todos os x, y em I.
f  x y f ( x ) f ( y )
ii. Côncava se 2 2 para todos os x, y em I.
Nas aplicações, quase sempre lidamos com funções contínuas (se você não sabe o
que vem a ser uma função contínua, pense na mesma como uma função cujo
gráfico não sofre interrupções ou saltos ao longo de seu domínio). Se f for
contínua a proposição a seguir é geometricamente evidente. A partir de agora,
sempre que nos referirmos a uma função estaremos sempre supondo ser seu
domínio um intervalo da reta e a função contínua nesse intervalo.

Proposição 5 : Sejam f : I  R uma função. Então:

i. f é convexa se e só se, para todos x, y em I e todo t  [0, 1] tivermos


f   1  t  x  ty   1  t  f  x   tf  y 
ii. f é convexa se e só se, para todos x, y em I e todo t  [0,1] tivermos
f   1  t  x  ty   1  t  f  x   tf  y 
Façamos o caso em que f é convexa. O outro caso é análogo. Observe que
 1  t  x  ty  [ x , y ]  I , e que, no trapézio abaixo,  1  t  f  x   tf  y 
é o comprimento da paralela às bases pelo ponto  1  t  x  ty

EUREKA! N°5, 1999


42
Sociedade Brasileira de Matemática

f (z)

x z = ( 1 – t ) x + ty y

Prova : Suponha primeiro que f satisfaz a condição do item i. Tomando t  12


concluímos que f é convexa. Reciprocamente, suponha que f seja convexa. Dados
x, y em I, temos

x y f    f  y
x y f  x  f  y

f  x 3 y
4   f  2 y
2 


2
2
 2
2
 f  y
 14 f  x   3
4
f  y  Trocando x

por y e raciocinando como acima segue que, para



t  0, 14 , 12 , 43 ,1 , 
f   1  t  x  ty   1  t  f  x   tf  y  (*)

Por indução sobre k inteiro positivo podemos concluir de maneira análoga que (*)
m
continua válida para todo t da forma , onde 0  m 2 k é inteiro. Como todo
2k
real em [0,1] é limite de uma seqüência de números dessa forma, segue que (*)
continua válida para todo t em [0, 1]. 

As afirmações a seguir são agora bastante evidentes, e vão ser nosso principal
guia quando quisermos decidir se uma dada função é ou não convexa ou côncava.

i. Se para todos a < b em I o gráfico de f entre as retas x = a e x = b estiver


abaixo da reta que passa por ( a , f ( a )), ( b, f ( b )) , então f é convexa, e
reciprocamente.

EUREKA! N°5, 1999


43
Sociedade Brasileira de Matemática

ii. Se para todos a < b em I o gráfico de f entre as retas x = a e x = b estiver


acima da reta que passa por ( a , f ( a )), ( b, f ( b )) , então f é côncava, e
reciprocamente.

A figura abaixo para se convencer da validez desse resultado no caso em que f é


convexa.

y = f(x)

e (c, e)

d (c, d)

a c b x

Nele, c  a 2 b . É evidente que d  f ( c )  f  a 2 b  e e


f (a ) f (b)
2
. Daí,
f  a 2 b   f (a )2 f (b) e f é convexa.
Para nós, a importância dessa discussão sobre funções côncavas e convexas
reside na seguinte:

Proposição 6 (Desigualdade de Jensen): Sejam I um intervalo da reta e


f : I  R uma função. Se x1 ,..., x n  I e t1 ,..., t n  [0,1] , com t1  ...  t n 1 ,
então t1 x1  ...  t n x n  I e

i. f convexa  f  t1 x1  ...  t n x n  t1 f  x1   ...  t n f  x n 

ii. f côncava  f  t1 x1  ...  t n x n  t1 f  x1   ...  t n f  x n 

EUREKA! N°5, 1999


44
Sociedade Brasileira de Matemática

Prova : Façamos a prova, por indução sobre n > 1, para o caso em que f é
convexa, sendo o outro caso análogo. O caso n = 2 é nossa hipótese. Suponha
agora que para um certo n > 1 e todos x1 ,..., x n  I e t1 ,..., t n  [0,1] , com
t1  ...  t n 1 , tenhamos
t1 x1 ... t n x n  I e f  t1 x1  ...  t n x n  t1 f  x1   ...  t n f  x n 
Considere agora x1 ,..., x n 1  I e t1 ,..., t n 1  [0,1] , com t1  ...  t n 1 1 .
Se t n 1 1 então t1 ... t n  0 e nada há a fazer. Senão, defina
t x1  ...  t n x n
y1 1 t n 1
 s1 x1  ...  sn x n ,
j t
onde s j  1 t . Como s1  ... sn 1 , segue da hipótese de indução que
n 1
y  I . Daí,

f  t1x1  ...  t n 1x n 1   f  1  t n 1   t1 x1  ...  t n x n


1 t n 1  t n 1 x n 1 


 
 f 1  t n 1  y  t n 1 x n 1 1  t n 1  f ( y )  t n 1 f ( x n 1 ) ,

já que f é convexa. Aplicando a outra metade da hipótese de indução, obtemos

t t
f  y   f  s1x1  ...  sn x n  s1 f  x1   ...sn f  x n   1 t1 f  x1   ... 1 tn f  xn 
n 1 n 1
Juntando essas duas desigualdades, obtemos a desigualdade de Jensen. 
Vejamos agora um exemplo de como aplicar a desigualdade de Jensen.

Exemplo 5: (Olimpíada Balcânica) Sejam n > 1 e a1 ,..., a n reais positivos com


n
soma 1. Para cada i, seja bi   aj . Prove que
j 1, j 1

a1 a a
1 b1  1 2b  ... 1 bn  2 nn 1 .
2 n

Prova : Veja que b j 1  1  a j  2  a j , e então temos de provar que

a1 a a
2  a1
 2  2a  ... 2  na  2 nn 1
2 n

EUREKA! N°5, 1999


45
Sociedade Brasileira de Matemática

Afirmamos que a função f :   ,2   R dada por f  x   2 x x é convexa.


Para ver isso, basta escrever f  x   2 2 x  1 , e esboçar o gráfico de f, como
abaixo.

y f

2 x

Portanto, temos pela desigualdade de Jensen que

n  n 
 
 f a j  n f  1n

 a j  nf  1n 

 2 nn 1 
j 1 j 1

Exemplo 6 : Utilizando a função logaritmo natural e a desigualdade de Jensen,


vamos dar outra prova da desigualdade entre as médias aritmética e geométrica.

Prova : Sejam a1 ,..., a n reais positivos. Existem reais x1 ,..., x n tais que
a j ln x j para todo j. Como f  x  ln x é uma função côncava, vem que

f  x1   ...  f  x n 
n
f  x1  ...  x n
n 
,

ou seja,
ln n x1 ... x n  ln  x1  ... x n
n 
Como f é crescente, chegamos ao resultado desejado. 

EUREKA! N°5, 1999


46
Sociedade Brasileira de Matemática

Vale notar, para quem tem familiaridade com derivadas, que é possível provar
que, se f ' ' existe, então f é convexa se e só se f ' ' ( x ) 0 para todo x em I e f é
côncava se e só se f ' ' ( x ) 0 para todo x em I.

Finalizamos este artigo com alguns problemas onde procuramos oferecer


oportunidade de exercitar o que foi aprendido no texto, além de desenvolver um
pouco mais a teoria. É bom salientar que em alguns deles mais de uma
desigualdade pode ser usada.

Problemas onde não precisamos das desigualdades acima, mas de criatividade

1. (Olimpíada Americana): Prove que, para todos a, b, c reais positivos, temos


1
3 3  3 13  3 13 1
 abc
a  b  abc b  c  abc c  a  abc

2. (Desigualdade de Abel): Sejam a1 , ..., a n , b1 ,..., bn reais dados (n > 1),


com a1 a2 ... a n 0 . Se M e m são respectivamente o máximo e o mínimo do
conjunto {b1 , b1  b2 ,..., b1  ...  bn } , prove que

ma1 a1b1  a 2 b2  ...a n bn  Ma1 ,


com igualdade se e só se a1  a 2  ... a n .

3. (Teste de Seleção de Singapura para IMO): Prove que, quaisquer que


sejam os reais positivos a, b e c, tem-se
c a 2  ab  b 2  a b 2  bc  c 2 b a 2  ac  c 2 .

4. (Banco IMO): Sejam n > 1 um inteiro dado. Determine o maior valor


possível da soma  
xi x j xi  x j
1i  j n
 sobre todas as n-uplas x1 ,..., x n de
reais não negativos cuja soma é 1.

Desigualdade entre as médias aritmética e geométrica

EUREKA! N°5, 1999


47
Sociedade Brasileira de Matemática

5. (Torneio das Cidades) Sejam a, b e c reais positivos dados. Prove que


a3 b3 c3
2 2  2 2  2  a  3b  c
a  ab  b b  bc  c c  ca  a 2

6. Dados os reais positivos a1 , a2 , a3 , b1 , b2 , b3 , prove que

3  a1  b1  a2  b2  a3  b3   3 a1a 2 a3  3 b1b2 b3

Desigualdades de Chebychev, Jensen e Cauchy

7. (Olimpíada Turca) Sejam n > 1 inteiro e x1 ,..., x n reais positivos tais


n
que  x i2 1 . Determine o valor mínimo de
i 1
x15 x 25 x n5
x 2  x 3  ... x n  x1  x 3  ... x n  ... x1  x 2  ... x n  1
.

8. (Olimpíada Romena): Seja h a altura de um tetraedro regular e


h1 , h2 , h3 , h4 as distâncias desde um ponto P em seu interior às faces do
tetraedro. Prove que
h  h1 h h h h h h
h  h1  h  h2  h  h3  h  h4 125
2 3 4

9. (Banco IMO) : Sejam a, b, c, d reais não negativos tais que


ab  bc  cd  da 1 . Prove que
a3 3 3 3
bc d  c bd  a  d ca  b  a db  c  13

10. Sejam n > 1 e x1 , x 2 ,..., x n reais positivos cuja soma é 1. Prove que

x1 xn n x1  ... x n
 ...  n 1

1 x1 1 x n n 1

11. Sejam x1 , x 2 ,..., x n reais pertencentes ao intervalo [0, 1] e tais que


x1  x 2  ...  x n  a , com 0  a  1 . Prove que

EUREKA! N°5, 1999


48
Sociedade Brasileira de Matemática

1 a
1 a
1 x 1 x
 1 x1 1 x 2 ... 1 x n 
1 2
1 x
n
 nn aa  n
Outras Desigualdades

12. (Desigualdade de Bernoulli): Sejam n um inteiro positivo e x  1 um


real. Prove que  1  x  n 1  nx .

13. (Desigualdade entre as Médias de Potências): Sejam    reais


positivos. Então, para todos a1 , a 2 ,..., a n reais positivos, vale

1 1
 
 a1  a 2  ...  a n    a1  a 2  ...  a n   ,
 n
  n 
   
com igualdade se e só se a1 , a 2 ,..., a n forem todos iguais.

14. (Desigualdade de Giroux): Sejam I1 ,..., I n intervalos fechados da reta


e considere a função f :I 1  ...I n  R de n variáveis, convexa
separadamente em relação a cada variável. Então, se I j [a j , b j ] , f atinge seu
valor máximo em um dos 2 n pontos da forma ( c1 ,..., c n ) , com ci  a i ou
bi para cada i. Prove isto e enuncie um resultado análogo à desigualdade de
Giroux para uma função de n variáveis f, côncava separadamente em relação a
cada variável.

15. (Olimpíada Búlgara): Sejam n  2 um inteiro e 0  x i 1 para 1 i  n.


Prove que

n
( x1  x 2 ... x n )  ( x1 x 2  x 2 x3 ... x n  1 x n  x n x1 )   2 
16. Os três itens a seguir visam derivar uma desigualdade difícil.
1 1
i. (Desigualdade de Young): Sejam p e q reais positivos tais que p  q 1 . Prove
que
p yq
xy  xp  q
,  x , y 0
ii. (Desigualdade de Holder): Sejam a1 , a2 ,..., a n , b1 , b2 ,..., bn reais

EUREKA! N°5, 1999


49
Sociedade Brasileira de Matemática

1 1
positivos e p, q reais positivos tais que p  q 1 . Prove que
n n 1/ p n 1/ q
   

i 1
a i bi 


i 1
a ip 






i 1
biq 

iii. (Desigualdade de Minkowsky): Sejam a1 , a2 ,..., a n , b1 , b2 ,..., bn reais


positivos e p um real maior que 1. Prove que

p
 a1  b1 
p
... a n  bn 
p
 p a1p ... a np  p b1p ... bnp

Sugestão: Faça ci = ( a i  bi ) p  1 e use o ítem anterior para (ai) e (ci) e para (bi) e
(ci).

Referências:

[1] Shklarsky, D. O., Chentzov, N. N. e Yaglom, I. M. The USSR Olympiad


Problem Book. Dover. Toronto, 1993.
[2] Rousseau, C. e Lozansky, E. Winning Solutions. Springer-Verlag, 1996.
[3] Lima, Elon L., Análise Real, vol. 1. IMPA, 1995.

EUREKA! N°5, 1999


50
Sociedade Brasileira de Matemática

40a. OLIMPÍADA INTERNACIONAL E 14a. OLIMPÍADA


IBEROAMERICANA DE MATEMÁTICA
Primeiro teste de Seleção
PROBLEMA 1
Determine todos os inteiros positivos n > 1 para os quais existem um inteiro
positivo k inteiros x1, x2, …xn dois a dois distintos tais que o conjunto
{x i  x j ; 1 i  j n}
Seja um conjunto de potências distintas de k. Observação: x1, x2, …xn não são
necessariamente positivos.

PROBLEMA 2
Sejam a, b, c, d números reais tais que
a 4 5  a ,b  4  5  b ,c  4  5c e d  4 5d. Calcule
abcd.

PROBLEMA 3
Considere um triângulo ABC e BD e CE as bissetrizes dos ângulos B e C,
respectivamente ( D  AC e E  AB). A circunferência circunscrita a ABC tem
centro O e a circunferência ex-inscrita tangente ao lado BC tem centro Ia. Estas
duas circunferências intersectam-se nos pontos P e Q.
(i) Mostre que PQ é paralelo a DE.
(ii) Prove que IaO é perpendicular a DE.

PROBLEMA 4
Sejam Q+ e Z o conjunto dos racionais extritamente positivos e o conjunto dos
inteiros. Determine todas as funções f : Q+  Z satisfazendo as seguintes
condições:
(1) f (1999) = 1
(2) f(ab) = f (a) + f(b), para quaisquer a, b  Q+.
(3) f(a + b)  min{f(a), f(b)}, para quaisquer a, b  Q+.
A notação min{x, y} denota o menor dentre os inteiros x e y. Por exemplo,
min{3, 4} = 4 e min{3, 3} = 3.

PROBLEMA 5
(i) Se m, n são inteiros positivos tais que 2 n – 1 divide m2 + 9, prove que n é
uma potência de 2.
(ii) Se n é uma potência de 2, prove que existe um inteiro positivo m tal que
2n – 1 divide m2 + 9.

EUREKA! N°5, 1999


51
Sociedade Brasileira de Matemática

SOLUÇÕES DE PROBLEMAS PROPOSTOS

 Publicamos aqui algumas das respostas enviadas por nossos leitores.

21) a) Encontre todas as soluções inteiras da equação a b  c.


3
b) Encontre todas as soluções inteiras da equação a  b 3 c .
3

Solução de Carlos Alberto da Silva Victor.

a) Vamos separar em 2 etapas:

i) Tomando a = k2 ; b = t2 teremos c = (k + t)2 consequentemente


(k2, t2, (k + t)2) é solução da equação para todo k, t  N.

ii) Suponha que a não seja um quadrado perfeito. Neste caso, a pode ser
escrito na forma a = k2s, onde s é um produto de primos distintos. Como
c a  b  2 ab , devemos ter b = t2 . s (t  N) já que ab deverá ser um
quadrado perfeito (observe que se tomarmos s com exponente ímpar
diferente de 1, podemos incluí-lo em t2); Daí:
c a  b  2 ab k 2 s  t 2 s  2kts
c s (k 2  t 2  2kt )
c s. ( k  t ) 2 e neste caso ( k 2 s, t 2 s , s ( k  t ) 2 ) é solução da
equação onde k , s, t ,  N.

Nota: observe que o ítem (ii) representa a situação genérica e inclui as soluções
de (i), fazendo s = 1.

3
b) a  3 b 3 c , logo c a  b  3(3 a 2 b  3 ab 2 ). Vamos separar também
em 3 etapas:
i) Tomando a = k3; b = t3 teremos c = (k + t)3 com k, t  N e
consequentemente (k 3 , t 3 , ( k  t ) 3 ) é solução da equação.

ii) Suponha que a não seja um cubo perfeito. Observe que a pode ser escrito
na forma: a t 3 .  .  2 onde  e  são produtos de primos distintos. Já
que c a  b  3 (3 a 2 b  3 ab 2 ), devemos tomar

EUREKA! N°5, 1999


52
Sociedade Brasileira de Matemática

b t 3 .  .  2 pois a 2 k 6 .  2  4 (de fato, 3


a 2 b  3 ab 2 é racional, e,
elevando ao quadrado, obtemos que a 3 ab 2  2ab  b3 a 2 b (e logo
a 3 ab 2  b3 a 2 b ) também é racional, donde, assumindo a  b,
3
ab 2 e 3
a 2 b são racionais e inteiros). daí:
c k 3 .  .  2  t 3 .  2  3(k 2 t  .  2  k . t 2 .  .  2 ) 
c  .  2 ( k 3  t 3  3k 2 t  3kt 2 )
Logo ( k 3 .  .  2 , t 3 .  2 ,  .  2 (k  t ) 3 ) é solução da equação dada.
(Novamente, as soluções em (i) são obtidas de (ii) com  =  = 1.

22) Sejam  ,  ,  ,  os ângulos de um quadrilátero, nessa ordem. Prove


que esse quadrilátero é inscritível se, e somente se, a relação
        2 ocorre.

Solução de André Luiz Arruda Marques.


A

D 
 B

1a. parte: Hipótese: O quadrilátero é inscritível.


Tese: É válida a relação         2

EUREKA! N°5, 1999


53
Sociedade Brasileira de Matemática

 
 BCD  BAD 360

Sabe-se que :  
 BCD 2  2  2 360 2      ( )1

 BAD 2
 
 ABC  ADC 360

 ABC 2
  2  2 360 2      (2)

 ADC 2
de (1) e (2)  (   )(    )  2          2 (c.q.d)

2a. parte: Hipótese: É válida a relação         2


Tese: O quadrilátero é inscritível

Têm-se a relação:         2
Fatorando vem: (   )(    )  2  
Sabe-se que:        360 2

 xy  2
Seja:   x e   y    x y 
 x  y 2
EUREKA! N°5, 1999
54
Sociedade Brasileira de Matemática

   
Logo:
  o quadrilátero é inscritível (c.q.d.).

    
23) Seja ABC um triângulo qualquer de ortocentro H e sejam ha, hb, hc os
comprimentos das alturas relativas a A, B, C respectivamente. Prove que
____ ____ ____
1
ha . AH  hb . BH  hc CH  ( a 2  b 2  c 2 ).
2

Solução de Maria Ivete Caetano Rodrigues.


A

z
O
y
N
H
M
C x B
____
____ ____
ha  AM CM x AB c
____ ____ ____
Notações: h BN BO  y BC a
b
____ ____ ____
hc CO AN z AC b

___ ____
Temos que : CMH é retângulo  (CH ) 2 ( h  AH ) 2  x 2
a
I
ACM é retângulo  b 2 ha2  x 2  x 2 b 2  ha2 II
___ ____ ____
substituindo II em I, temos (CH ) 2 h 2  2h AH  ( AH ) 2  b 2  h 2
a a a
___ ____ ____
( CH ) 2 b 2  ( AH ) 2  2ha AH (1)

____ ____ ____


Analogamente temos: ( BH ) 2 ( CH ) 2  a 2 - 2h CH (2) e
c

EUREKA! N°5, 1999


55
Sociedade Brasileira de Matemática

____ ____ ____


( AH ) 2 ( BH ) 2  c 2 - 2hb BH (3)

somando (1), (2) e (3), obtemos:


___ ____ ____
2ha AH  2hc CH  2hb BH a 2  b 2  c 2
___ ____ ____
2(ha AH  2hb BH  hc CH ) a 2  b 2  c 2
___ ____ ____
a2  b2  c2
ha AH  hb BH  hc CH  (c.q.d.)
2

25) Durante o ano de 1998, uma pequena livraria, que abria nos sete dias da
semana, vendeu no mínimo um livro por dia e um total de 600 livros no
ano todo. Diga, justificando, se existiu, obrigatoriamente, um período de
dias consecutivos onde foram vendidos exatamente 129 livros.

Solução de Marcelo Rufino de Oliveira.

Seja ai o total acumulado de livros vendidos até o final do i-ésimo dia.


Por exemplo: a5 = total vendidos até o quinto dia a35 – a31 = total de livros
vendidos entre os dias 35 e 32 (inclusive).
Então: a1 < a2 < a3 < … < a364 < a365 = 600 (1)
Resta agora analisar se existem ai, aj (i, j  N menores que 365, j > i) tais que
aj – ai = 129
Somemos agora a cada termo de (1) o valor 129:
a1 + 129 < a2 + 129 < a3 + 129 < … < a364 + 129 < a365 + 129 = 729
Chamemos estes termos de bi (bi = ai + 129): b1 < b2 < b3 < … < b364 < b365 = 729
Temos então 730 termos, 365 termos ai e 365 termos bi, com ai  aj (i  j) e bp 
bk (p  k).
Notemos que estes 730 termos naturais estão entre 1 e 729, ou seja, existem dois
valores iguais (princípios das casa dos pombos ou Princípio das Gavetas de
Dirichet) entre estes 730 termos. Como cada ai é distinto e cada bi é distinto,
então existe um am que é igual a um bn:
am = bn => am = an + 129 => am – an = 129
Que prova que existe um período de dias consecutivos (m – n dias) onde foram
vendidos exatamente 129 livros.

PROBLEMA No. 9 DO VISITANTE MATEMÁTICO (Revista EUREKA! No. 2)

EUREKA! N°5, 1999


56
Sociedade Brasileira de Matemática

Um vaso de vinho está suspenso sobre outro, de igual capacidade (digamos 1


litro), cheio de água. Por um orifício no fundo de cada vaso, o vinho escorre
sobre o vaso de água e a mistura se esvai na mesma velocidade. Quando o vaso
de vinho estiver vazio, qual é o volume de água no vaso inferior?

Solução de Carlos Frederico Borges Palmeira.

Aqui está uma solução, que usa apenas a noção de limite e o fato que
1 1
lim(1  ) n  .
n e
Vamos discretizar o problema, supondo que primeiro deixamos passar uma fração
1
de um vaso para o outro, a água e o vinho se misturam, e depois retiramos
n
1
do novo volume do segundo vaso. Após n passos o primeiro vaso estará vazio.
n

Seja v( k  1) a quantidade de vinho no segundo vaso após o k-ésimo passo.


1 1
Temos v (k  1) (1  ) (v ( k )  ), sendo v (0) = 0.
n n
Agora, a equação de diferenças v (k  1) av (k )  b tem solução
1 an
v( k ) a n v(0)  ( ) b (nada além de progressões geométricas aqui).
1 a
1
(1  (1  ) n
1 n n ) 1 (1  1 ),
Segue que v( n) (1  ) v(0)  ( e como v (0) =
n 1 n n
n
1 1
0, temos apenas o segundo termo, que fica (1  ) (1  (1  ) n ) , e passando
n n
1
ao limite quando n tende ao infinito, obtemos 1  ( ). Esta é a fração de vinho
e
que resta no segundo vaso quando o primeiro fica vazio.

EUREKA! N°5, 1999


57
Sociedade Brasileira de Matemática

Você sabia… que todo poliedro convexo


(com as faces rígidas) é rígido (isto é, não pode
ser deformado) mas existem poliedros não -
convexos flexíveis?

Errata: No Você sabia… da EUREKA! No. 4, página 21, sobre um polinômio


que assume apenas valores negativos ou primos, há um erro tipográfico na
terceira linha da expressão do polinômio: onde está "…[ e3 . (e + 2) . (a +1)2 +
…" leia-se "…[e3 . (e + 2) . (a + 1) 2 + …".

Agradecemos também o envio das soluções da EUREKA! N o.4 a: Osvaldo Ribeiro


da Silva Júnior, Robério Bacelar da Silva, Manuel João de Jesus Almeida, Vicente
Wilson Moura Gaeta, Rubens Henriques, Raul Rabello Mesquita, Otávio Braga,
Antonio Caminha Neto, Alexandre Celestino Leite de Almeida, Luciana Rocha
Pedro, Francisco Dutenhefner, Seme Gebara Neto. Continuamos esperando as
soluções dos problemas 10, 16, 17, 20 e 24.

PROBLEMAS PROPOSTOS

 Convidamos o leitor a enviar soluções dos problemas propostos e sugestões de novos


problemas para os próximos números.

26) Sejam as funções fo (x) = xn e fi (x) = fi – 1(x + 1) – fi – 1(x) onde x, n e i são


inteiros positivos. Prove que, para todo x, fn(x) = n!

27) O triângulo equilátero ABC possui um ponto interno P tal que em P


chegam três segmentos de reta (PA, PB, PC) onde PA = 6, PB = 8 e
PC = 7. Com esses dados descubra qual é a área do
triângulo.

EUREKA! N°5, 1999


58
Sociedade Brasileira de Matemática

B C

28) Seja n  2 um número inteiro. Prove que n e n + 2 são ambos primos se e

4((n  1)!  1)  n
somente se é inteiro.
n( n  2)

29) Seja n > 1 um número inteiro. Existem n lâmpadas L0, L1, ... , Ln–1
colocadas em um círculo. Cada lâmpada está ACESA ou APAGADA.
Uma seqüência de passos S0, S1, ... , Si, ... é executada. O passo Sj afeta
apenas o estado da lâmpada Lj (deixando o estado de todas as outras
inalterado) da seguinte forma:

Se Lj–1 está ACESA, Sj muda o estado de Lj de ACESA para APAGADA,


ou de APAGADA para ACESA;

Se Lj–1 está APAGADA, Sj deixa o estado de Lj inalterado.

As lâmpadas são rotuladas mod n, ou seja,

L – 1 = Ln –1, L0 = Ln, L1 = Ln + 1, etc.

Inicialmente todas as lâmpadas estão ACESAS. Mostre que:

a. Existe um inteiro positivo M(n) tal que depois de M(n) passos todas as
lâmpadas estão ACESAS de novo;

b. Se n é da forma 2k então todas as lâmpadas estão ACESAS depois de


n2 –1 passos;

EUREKA! N°5, 1999


59
Sociedade Brasileira de Matemática

c. Se n tem a forma 2k + 1 então todas as lâmpadas estão ACESAS depois


de n2 – n + 1 passos.



O problema 6 da Olimpíada Internacional de Matemática de 1994 pedia que fosse


mostrada a existência de um conjunto A de inteiros positivos com a seguinte
propriedade: para todo conjunto infinito de números primos S existem um inteiro
positivo k  2 e dois inteiros positivos m  A e n  A, cada um dos quais é um
produto de k elementos distintos de S.

Os leitores que enviarem soluções corretas (até o dia 15 de novembro de 1999)


para o seguinte problema concorrerão a um exemplar do livro "10 Olimpíadas
Iberoamericanas de Matemática".

PROBLEMA "CUÁTICO":

Prove que no problema acima k não pode ser escolhido independentemente do


conjunto S. Mais precisamente:

Prove que para qualquer conjunto de inteiros positivos A e para todo inteiro
positivo k existe um conjunto infinito de números primos S tal que o produto de k
elementos distintos de S está sempre em A ou o produto de k elementos distintos
de S nunca pertence a A.

Nota: Os problemas 26 e 27 foram propostos por Christian Lyoiti Watanabe e Roberto


Gomides respectivamente.
COMO ASSINAR A EUREKA!

Se você é fanático por Matemática e deseja receber na sua casa a revista


EUREKA!, faça o seu pedido escrevendo para: Secretaria da Olimpíada
Brasileira de Matemática, Estrada Dona Castorina, 110 Jardim Botânico - Rio
de Janeiro, RJ - CEP: 22460-320. O custo de cada exemplar avulso ou atrasado é
de R$4,00. Você pode fazer uma assinatura anual o que dará direito a receber as
publicações do ano (mínimo 3 exemplares) por um valor promocional de
R$10,00. Para isso, faça um depósito no Banco do Brasil - Agência 0598-3 -
Conta N52208-2 em nome do professor Eduardo Wagner. Envie-nos a fotocopia

EUREKA! N°5, 1999


60
Sociedade Brasileira de Matemática

do depósito e faça referência aos números desejados. Não esqueça de colocar seu
nome e endereço completos e nós remeteremos a(s) revista(s) pelo correio.
Pedidos podem ser feitos também por e-mail e comprovantes de depósito poderão
ser enviados pelo fax.

Se tiver qualquer dúvida entre em contato conosco.


Telefone: 021-5295077 / Fax: 021-5295023
e-mail: obm@impa.br
Home-Page: http//www.obm.org.br/

R e v is t a d o
P ro fe s s o r d e
M a te m á tic a
A RPM é uma publicação da SBM
destinada aos professores de Matemática
do ensino médio, que pretende ser um
veículo de circulação e intercâmbio de
idéias através de seus artigos e seções.
Estamos comemorando o número 40
Revista do Professor de Matemática da RPM e 17 anos de publicação sem
Caixa Postal 66281
CEP 05315-970 São Paulo – SP
interrupção, com capa nova e com
e-mail: rpm@ime.usp.br renovado estímulo para continuar
Fone/Fax: (11) 818-6124 auxiliando o professor de Matemática.

AGENDA OLÍMPICA

21a. OLIMPÍADA BRASILEIRA DE MATEMÁTICA


Primeira Fase – 12 de junho (sábado)
Segunda Fase – 28 de agosto (sábado)
Terceira Fase – 23 de outubro (sábado) e 24 de outubro (domingo)

EUREKA! N°5, 1999


61
Sociedade Brasileira de Matemática

14a. OLIMPÍADA IBEROAMERICANA DE MATEMÁTICA


12 a 19 de setembro
La Habana, Cuba.

2a. OLIMPÍADA IBEROAMERICANA DE MATEMÁTICA UNIVERSITÁRIA


16 de setembro

Você sabia…
Que se an + 1 é primo (com a e n naturais) então n = 2k para algum k
natural? Tente provar isso!, (primos desta forma são conhecidos como
primos de
Fermat generalizados.) E que se a = 2 só se conhecem 5 primos
desta forma (3, 5, 17, 257 e 65537), os chamados primos de Fermat?
E que, por outro lado, se conhecem muitos primos de Fermat
generalizados grandes (como 10183016384 + 1 e 6723416384 + 1,
respectivamente o 16o e
18o. maiores primos conhecidos em 27/7/99)?

COORDENADORES REGIONAIS

Amarisio da Silva Araújo (UFV) Viçosa - MG


Alberto Hassen Raad (UFJF) Juiz de Fora - MG
Antônio C. Rodrigues Monteiro (UFPE) Recife - PE
Angela Camargo (Centro de Educação
de Adultos - CEA) Blumenau - SC
Benedito T. Vasconcelos Freire (UFRN) Natal - RN
Claudio Arconcher (Col. Leonardo da Vinci) Jundiaí - SP

EUREKA! N°5, 1999


62
Sociedade Brasileira de Matemática

Élio Mega (Col. ETAPA) São Paulo - SP


Florêncio F. Guimarães F. (UFES) Vitória - ES
Francisco Dutenhefner (UFMG) Belo Horizonte - MG
Gisele de A. Prateado G. (UFGO) Goiânia - GO
Ivanilde H. Fernandes Saad (U. Católica Dom Bosco) Campo Grande - MS
João B. de Melo Neto (UFPI) Teresina - PI
João F. Melo Libonati (Grupo Educ. IDEAL) Belém - PA
Jorge Ferreira (UEM) Maringá - PR
José Carlos Pinto Leivas (UFRG) Rio Grande - RS
José Clovis Saraiba (UFMA) São Luis - MA
José Luis Rosas Pinho (UFSC) Florianópolis - SC
José Paulo Carneiro (USU) Rio de Janeiro - RJ
José Vieira Alves (UFPB) Campina Grande - PB
Leonardo Matteo D'orio (Sistema Titular de Ensino)Belém - PA
Licio Hernandes Bezerra (UFSC) Florianópolis - SC
Luzinalva M. de Amorim (UFBA) Salvador - BA
Marco Polo (Colégio Singular) Santo André - SP
Marcondes Cavalcante França (UF Ceará) Fortaleza - CE
Pablo Rodrigo Ganassim (L. Albert Einstein) Piracicaba - SP
Paulo H. Cruz Neiva de L. Jr. (Esc. Tec.Everardo Passos) SJ dos Campos - SP
Reinaldo Gen Ichiro Arakaki (INPE) SJ dos Campos - SP
Ricardo Amorim (Centro Educ. Logos) Nova Iguaçu - RJ
Roberto Vizeu Barros (Colégio ACAE) Volta Redonda - RJ
Sergio Claudio Ramos (IM-UFRGS) Porto Alegre - RS
Seme Gebara Neto (UFMG) Belo Horizonte - MG
Tadeu Ferreira Gomes (U. do Estado da Bahia) Juazeiro - BA
Tomás Menéndez Rodrigues (U. Federal de Rondonia) Porto Velho - RO
Valdenberg Araújo da Silva (U. Federal de Sergipe) São Cristovão - SE
Wagner Pereira Lopes (Esc. Tec. Fed. de Goiás) Jataí - GO
Waldemar M. Canalli (P.M. S. João de Meriti) S. João de Meriti - RJ

EUREKA! N°5, 1999


63
CONTEÚDO

40a. OLIMPÍADA INTERNACIONAL DE MATEMÁTICA 2


Problemas e Soluções

14a. OLIMPÍADA IBEROAMERICANA DE MATEMÁTICA 13


Prova

ARTIGOS

CONTAR DE DUAS MANEIRAS, PARA GENERALIZAR 15


José Paulo Q. Carneiro - Universidade Santa Úrsula

A MATEMÁTICA DAS ABELHAS 18


José Cloves Saraiva - Universidade Federal de Maranhão

O TEOREMA DE RAMSEY 23
Carlos Gustavo T. de A. Moreira - IMPA

APLICAÇÕES DOS NÚMEROS COMPLEXOS À GEOMETRIA 30


Edmilson Motta - Colégio Etapa

40a. OLIMPÍADA INTERNACIONAL E 14a. OLIMPÍADA IBEROAMERICANA 39


DE MATEMÁTICA
Segundo teste de Seleção

10a. OLIMPÍADA DE MATEMÁTICA DO CONE SUL 40


Primeiro teste de Seleção

10a. OLIMPÍADA DE MATEMÁTICA DO CONE SUL 41


Segundo teste de Seleção

SOLUÇÕES DE PROBLEMAS PROPOSTOS 42

PROBLEMAS PROPOSTOS 57

ASSINATURA DA REVISTA EUREKA! 2000 59

FICHA DE CADASTRAMENTO ASSINATURA DA REVISTA EUREKA! 60

COORDENADORES REGIONAIS 61
Sociedade Brasileira de Matemática

40a. OLIMPÍADA INTERNACIONAL DE MATEMÁTICA


Problemas e Soluções

Primeiro Dia
Duração da prova: 4 horas e 30 minutos

PROBLEMA 1
Determine todos os conjuntos finitos S de pontos do plano com pelo menos três
elementos que satisfazem a seguinte condição:

Para quaisquer dois pontos distintos A e B de S, a mediatriz do segmento AB é um


eixo de simetria de S.

Solução de Fabrício Siqueira Benevides (Fortaleza - CE):

Seja  o fecho convexo dos pontos de S, isto é conjunto de pontos da periferia de


S que formam um polígono convexo que contém todos os pontos de S. Sejam
P1 , P2 ,..., Pn os vértices do polígono formado pelo fecho convexo.

Pn Pn'
P3' P3

P1 P2
______
Afirmação 1) P3 é o simétrico de Pn em relação à mediatriz de P P que
1 2
______
denotaremos por md( P P ).
1 2
______
Prova: Suponha o contrário. Seja Pn' o simétrico de Pn a md( P P ). Pn' deve
1 2
P ,
estar no interior do polígono 1 2P ,..., Pn , pois este é o fecho convexo.

EUREKA! N°6, 1999


2
Sociedade Brasileira de Matemática

  ______
Daí, P1 P2 Pn ' < P1 P2 P3 , e se P3 ' é o simétrico de P3 à md( P P )
1 2
 
teremos P1 P2 P3 ' > P2 P1 Pn (pela simetria), aí P3 ' pertenceria a S e estaria
fora do fecho convexo. Absurdo. Se Pn' está sobre o segmento P2P3, teremos
   ______ ______
Pn P1 P2 P1 P2 P3  P3 '  P1 Pn e P1 P3 '  P1 Pn  P3 ' está fora do fecho
convexo (Absurdo).

Se Pn' está sobre o prolongamento de P2P3, já temos um absurdo pois Pn' estaria
fora do fecho convexo.
Logo, P3 é realmente o simétrico de Pn.
______
Assim P3 é o simétrico de Pn em relação à md( P P ), donde
1 2
______ _____  
P1 Pn P 2 P 3 ( e Pn P1 P2 P1 P2 P3 ).
______ ______
Analogamente, se olharmos para a mediatriz de P P concluimos que P P =
2 3 1 2
______  
P3 P4 ( e P1 P2 P3 P2 P3 P4 ).
Se continuarmos a olhar para as mediatrizes de Pi Pi 1 , i 1,..., n  1, é facil
concluir que: todos os ângulos internos de P1 , P2 ,..., Pn são iguais e:
______ _____ _____ ______ _____ _____
P1 P2 P 3 P 4 P5 P6 ... P2 k 1 P2( k 1) e P2 P3 P 4 P 5 P6 P7 ... P2 q P2 q 1
(ver indices módulo n.)

Temos agora 2 casos:

i) n é ímpar n = 2k + 1, teremos:
______ _____ ________ ______ _____ ________
P1 P2 P 3 P 4 ... P2 k 1 P1 P2 P3 P 4 P5 ... P2 k P2 k 1 ; ou
seja o fecho convexo é um polígono regular

ii) n é par: n = 2k.


______ _____ _________ ______ _____ ____________
P1 P2 P 3 P 4  P2 k 1 P2 k e P2 P3 P 4 P 5 ... P2 k  2 P2 k  1
mas md (P1P3) também é eixo de simetria e é fácil ver que o simétrico de
______
P2 está sobre a mediatriz de P P (e seu simétrico será ele mesmo).
1 3

EUREKA! N°6, 1999


3
Sociedade Brasileira de Matemática

______ ______
Nesse caso P P = P P , donde novamente o fecho convexo forma
1 2 2 3
um polígono regular.

Portanto, o fecho convexo é um polígono regular.


Para finalizar, provaremos que não pode haver qualquer ponto em S além de
P1 , P2 ,..., Pn .
Já vimos que P1 P2 ...Pn é regular. Seja O o centro da circunferência
circunscrita a P1 P2 ...Pn . (pode ser O  S ou O  S). Seja P um ponto de S
diferente de P1 ...Pn .
(P esta sobre algum lado ou no interior de P1 ...Pn ).
Seja Pi o vértice do nosso polígono cuja distância a P seja mínima. E seja
_____
r md ( PPi ). É claro que r não passa por O, (que é o centro de simetria de
P1 ...Pn . ), assim se d1 é a maior distância de um vértice Pj que está no semi-
plano esquerdo de r a r; e d2 é a maior distância de um vértice Pk que está no
semi-plano direito de r a r; então d1  d2. Suponha d1 > d2. Então o simétrico de Pj
deveria pertencer a S mas está fora do fecho convexo. Absurdo.
Na verdade, quero dizer que se r não é um eixo de simetria de P1 ...Pn ,
tomamos P1 ' ,..., Pn ' os simétricos de P1 ,..., Pn em relação a r, um dos Pi'
estará fora do polígono P1 ...Pn .

Conclusão:
Um conjunto finito de pontos S satisfaz o problema somente se e só se é o
conjunto dos vértices de um dado polígono regular.

PROBLEMA 2
Seja n  2 um inteiro fixo.
a) Determinar a menor constante C para a qual a desigualdade


1i  j n
x i x j ( x i2  x 2j ) C ( x
1i n
i )4

é válida para quaisquer números reais x1, …, xn  0.


b) Para esta constante C, determine quando ocorre a igualdade.

Solução:

EUREKA! N°6, 1999


4
Sociedade Brasileira de Matemática

Consideremos inicialmente o caso n = 2. Queremos achar a menor constante C tal


que x1 x 2 ( x12  x 22 ) C ( x1  x 2 ) 4 para quaisquer x1 , x 2 0.
1
Para x1 x 2 1 temos x1 x 2 ( x12  x 22 )  ( x1  x 2 ) 4 , donde necessariamente
8
1 1
C  . Vamos mostrar que C  . Para isso, basta mostrar que
8 8
1
x1 x 2 ( x12  x 22 )  ( x1  x 2 ) 4 para x1 , x 2 0.
8
Como ( x1  x 2 ) x14  4 x1 x 2 ( x12  x 22 )  6 x12 x 22  x 24 , isso equivale a mostrar
4

que x14  x 24  6 x12 x 22 4 x1 x 2 ( x12  x 22 ),


mas isso segue diretamente de
1
( x1  x 2 ) 4 0. Para n > 2 temos ainda C  , pois fazendo x1 x 2 1 e
8
1
x k 0 para k 3 vale a igualdade com C  .
8
1
Mostraremos que ainda vale a desigualdade do enunciado para C  . Para isso,
8
mostraremos o seguinte:

Lema: Se x1 ,..., x n 0 e o número de termos xi  0 com 1 i n é m 3


~ ~ ~
então podemos substituir x1 ,..., x n por x , x ,..., x tais que
1 2 n
~ ~ ~ ~ ~


1i n
xi  
1i n
xi , 
1i j n
x i x j ( x i2  x 2j )  x
1i j n
i x j ( xi2  x 2j ) e o número de

~
termos x  0 com 1 i n é m  1.
i

Prova do Lema: Sejam x r e x s , r s os dois menores termos não nulos dentre


~ ~ ~
x1 , x 2 ..., x n . Definimos x x  x , x 0 e x x para k {r , s}.
r r s s k k
~
~
Obviamente temos x
1i n
i  x
1i n
i e há m – 1 termos x 0. Note agora que
i

y
1i j n
i y 2
j ( yi  y 2j ) y r y s ( y r2  y s2 )  ( y r3  y s3 ) 
k{r , s}
yk  ( yr  ys ) y
k{ r , s}
3
k,

EUREKA! N°6, 1999


5
Sociedade Brasileira de Matemática

donde
~ ~ ~ ~

x
1i j n
i x j ( x 2
i  x 2j )  x x
1i  j n
i j ( x i2  x 2j )  x r x s ( x r2  x s2 )  (( x r  x s ) 3  ( x r3  x s3 )) x
k{r , s}
k 

 x r x s (3( x r  x s ) x
k{r , s}
k  ( x r2  x s2 )),
mas como xr e xs são os dois menores
termos não nulos dentre os xi (e há pelo menos 3), temos
x  xs 3
k{r , s}
xk  r
2
 3( x r  x s )
k{r , s} 2

x k  ( x r  x s ) 2  x r2  x s2 .

Para concluir observamos que:

i) Se no máximo um termo dentre os xi é não nulo então



1i  j n
x i x j ( x12  x 2j ) 0,
e vale a igualdade se e só se todos os termos
são nulos.
ii) Se exatamente dois termos, digamos xr e xs, são não nulos, caímos no
4
1 
caso n = 2, ou seja, temos 
1i  j n
x i x j ( x12  x 2j )   
8  1i n
xi  ,


valendo a igualdade se e só se xr = xs.

iii) Finalmente, se há m termos não nulos com m  3, aplicamos o lema m – 2


vezes e obtemos x1 ' ,..., x n ' onde exatamente dois dos x i ' são não
nulos e vale
4 4
1  1 

1i  j n
x i x j ( x12  x 2j )   x i ' x j ' ( x i'2  x 'j2 )  
1i  j n 8  
 1i n
xi ' 


 
8

 1i n
xi  ,


onde a segunda desigualdade segue de ii).
1
Assim , temos sempre C  , ocorrendo a igualdade se e só se há exatamente
8
dois termos xr e xs não nulos e xr = xs.

PROBLEMA 3
Considere um tabuleiro quadrado n  n, onde n é um inteiro positivo par fixo. O
tabuleiro está dividido em n2 quadrados unitários. Dizemos que dois quadrados
distintos do tabuleiro são adjacentes se eles têm um lado comum.
Marcam-se N quadrados unitários do tabuleiro de tal forma que qualquer
quadrado (marcado ou não) é adjacente a pelo menos um quadrado marcado.
Determine o menor valor possível para N.

EUREKA! N°6, 1999


6
Sociedade Brasileira de Matemática

Adaptação da solução de Humberto Silva Naves (Goiânia - GO):

Vamos criar um algorítmo para preencher estes quadrados n  n .


Quando pintarmos um dos quadradinhos temos que pintar pelo menos outro
quadradinho adjacente.

Procedemos da seguinte maneira:

Pintamos os dois quadradinhos do canto superior esquerdo (1,1) e (1,2).


"Caminhamos" no sentido horário no bordo do quadrado, de modo que pintamos
2 quadradinhos e pulamos outros 2. Isso é possível pois o número de
quadradinhos no bordo é múltiplo de quatro. Agora formamos outro quadrado
menor de canto superior (3,3) e continuamos o mesmo procedimento para este
quadrado,que também possui lado de medida par e assim sucessivamente.
O número de quadrados pintados vai ser:
N(n  n) = 2n – 2 + N((n – 4)  (n – 4))
Este algorítmo determina o menor N, pois cada quadradinho só possui um, e
somente um "vizinho" pintado.

n2 n
Essa construção mostra por indução que é possível marcar  quadrados e
4 2
obter uma solução.
Para provar que esse número de quadrados marcados é o menor possível
associamos a cada quadrado coordenadas (x, y) com 1  x  n e 1  y  n. Se
considerarmos os quadrados {(x, y)  (x e y são ímpares e x + y  2 (mod 4)) ou (x
é ímpar, y é par e x + y  n + 1 (mod 4))} verificamos que nenhuma peça é

EUREKA! N°6, 1999


7
Sociedade Brasileira de Matemática

adjacente a mais de um dos quadrados deste conjunto. Como este conjunto tem
n2 n
 elementos, necessitamos no mínimo esta quantidade de peças 
4 2

Segundo Dia
Duração da prova: 4 horas e 30 minutos

PROBLEMA 4
Determine todos os pares (n, p) de inteiros estritamente positivos tais que

 p é primo,
 n  2p, e
 (p – 1)n + 1 é divisível por np–1.

Solução de Humberto Silva Naves (Goiânia - GO):

Analiçando os casos:
p = 2: temos que as únicas soluções são para n = 1 e n = 2.
p = 3: temos que as únicas soluções são para n = 1 e n = 3.
p >3: se n < p – 1, temos: (Caso trivial n = 1  p é divisível por 1).
n é ímpar, pois, (p – 1)n + 1 é ímpar, logo n não pode ser par.

Lema:
Seja a > b  3, então: ab < ba
b a
Prova: Basta provar que: b ln a  a ln b   , mas isto é obvio para a,
ln b ln a
x ln x  1
b  3, pois a função t ( x )  é crescente a partir de e. (t'(x) = 0
ln x (ln x) 2
para
x  e.)
Afirmação: Se p –1 > n  3 ( pois n é ímpar), então:
( p  1) n  n p  1  ( p  1) n  1 n p  1 ou ( p  1) n  1  n p  1 . Como
( p  1) n  1 n p  1 (pois ( p  1) n  1 é múltiplo de p e n p – 1 não é, temos
( p  1) n  1  n p  1 . Portanto n não pode ser menor que p – 1.
Afirmação: Se n > p, então n não é primo:

EUREKA! N°6, 1999


8
Sociedade Brasileira de Matemática

Prova: se n for primo temos:


( p  1) n  1 0(mod n)  [ a n a (mod n)]
( p  1)  1 0(mod n)  p 0(mod n) (um absurdo!).
Logo, n é composto:
n = a  b (a é o menor fator primo de n). Então:
( p  1) ab  1 0(mod(ab) p  1 )  ( p  1) ab  1 0(mod a p  1 )  ( p  1) ab  1(mod a p  1 )
Seja K o menor natural tal que ( p  1) K 1(mod a p  1 ).
Devemos ter K 2ab ( pois 2 ab = K  q + r  ( p – 1)r  1 (mod ap–1)  r = 0).
Além disso devemos ter K   (ap–1)  K ap–2(a – 1).
Então K = 2  ac, onde   {0, 1} (pois se K tivesse um outro fator primo além
do 2 que dividisse (a – 1), esse fator também teria que dividir 2ab que implica
que esse fator dividiria ab, mas como o menor fator primo de n = ab é o a,
teriamos absurdo).
K é par, pois se não fosse teriamos K  ab  (p – 1)ab  1  –1 (mod ap–1), um
absurdo pois é ímpar. Logo K = 2  ac.
Temos
c c
( p  1) 2a 1(mod a p  1 )  (( p  1) a ) 2 1(mod a p  1 ) 
c c
( p  1) a  1(mod a p  1 )  ( p  1) a  1(mod a ) 
p  1  1(mod a ), pois b a b(mod a ))  p 0(mod a ), um absurdo pois
p  a, porque p < n < 2p).
Logo não podemos ter n > p.
As únicas possibilidades que restam é n = p – 1 ou n = p.
n = p – 1 é falsa pois p – 1 é par. Se n = p temos: ( p  1) p  1 0(mod p p  1 ) e
como p > 3, temos:
[ p p  ( ip ) p p  1 ( 1)1  ...  ( 3p ) p 3 ( 1) p  3  ( 2p ) p 2 ( 1) p  2  ( 1p ) p 1 ( 1) p  1  ( 1) p ] 
 1 0(mod p 3 )  p 2 0(mod p 3 ).
(Um absurdo pois p3 não divide p2).
Logo as soluções são:
S = {(1, 2); (2, 2); (1, 3); (3, 3)}  { (1, p )  p é primo}

PROBLEMA 5
Duas circunferências 1 e 2 estão contidas no interior de uma circunferência
 e são tangentes a  em pontos distintos M e N, respectivamente. A

circunferência 1 passa pelo centro de 2 . A reta que passa pelos dois pontos

EUREKA! N°6, 1999


9
Sociedade Brasileira de Matemática

de interseção de 1 e 2 intersecta  em A e B. As retas MA e MB


intersectam 1 respectivamente em C e D.
Prove que CD é tangente a 2 .

Solução de Pedro Paulo de Simoni Gouveia (Fortaleza - CE):

Considere a figura relativa ao problema:

1

M

O
D
2
2
C
W
Z

N''
O2 A

N'
N

X 2

Sejam O e O2 os centros de 1 e 2 respectivamente: Tome uma inversão  =


inversão de centro em A e razão igual à potência do ponto A em relação a 1 e
2 (que é a mesma pois A  ao eixo radical de 1 e 2 ). Assim
 ( M ) C e  ( N ) N '. Então a imagem de  pela inversão será uma reta

passando por C e N ', como  é tangente à 1 e 2  CN ' é tangente
também à 1 e 2 .
Tome agora uma inversão  com centro em B e razão igual à potencia de B em
relação à 1 e 2 (B  eixo radical).
 ( M ) D e  ( N ) N ". Essa inversão leva  na reta

que será tangente comum à 1 e 2 .
DN "
Temos a seguinte situação.

EUREKA! N°6, 1999


10
Sociedade Brasileira de Matemática

Temos DW
= ZC, e como
 
DWC DZC que DCZW é um
trapézio isósceles
D C  AB // CD  CD  OO2 .
______
Basta-nos provar agora que O D é
2
bissetriz de
W Z

N'' O2 N" D C
N' pois aí, O2 será o incentro do triângulo
formado por D, C e o encontro de

e
DN "

CN ' .
Prova:
 
N " D O2    D O1
.
Logo
 
N " O O2 C D O2 
é bissetriz  2 é o incírculo  2 é tangente a CD 

PROBLEMA 6
Determine todas as funções f : R  R tais que

f ( x  f ( y ))  f ( f ( y ))  x f ( y )  f ( x )  1
para quaisquer x, y  R.

Solução:
Fazendo x  f (z ), obtemos
f ( f ( z )  f ( y ))  f ( f ( z ))  f ( f ( y ))  f ( z ) f ( y )  1.
Fazendo x  f ( y ), obtemos f (0) 2 f ( f ( y ))  f ( y ) 2  1, donde
1  f (0)  f ( y ) 2
f ( f ( y ))  . Portanto,
2

EUREKA! N°6, 1999


11
Sociedade Brasileira de Matemática

1  f (0)  f ( z ) 2 1  f (0)  f ( y ) 2
f ( f ( z )  f ( y ))  f ( f ( z ))  f ( f ( y ))  f ( z ) f ( y )  1   
2 2
( f ( z )  f ( y ))2
 f ( z ) f ( y )  1  f (0)  . Provamos assim que
2
w2
f ( w)  f (0)  sempre que w puder ser escrito como f ( z )  f ( y ). Vamos
2
ver que qualquer
w  R pode ser escrito desta forma:
f ( x  f (0))  f ( x )  f ( f (0))  xf (0)  1. Se tivéssemos f (0) 0 o lado
esquerdo seria sempre 0 e o lado direito seria igual a – 1, absurdo, portanto
f (0) 0 e f ( f (0))  xf (0)  1 pode assumir qualquer valor real w (basta
1  w  f ( f (0))
tomar x  ) , mas então w  f ( z )  f ( y )
f (0)
com z  x  f (0) e y = x.
w2
Mostramos assim que f ( w)  f (0)  para todo w  R.
2
Em particular f ( f ( y ))  f (0)  f ( y ) 2 / 2. Por outro lado
1  f (0 )  f ( y ) 2 1  f (0)
f ( f ( y ))  , donde  f (0)  f (0) 1, e
2 2
w2
f ( w) 1  para todo w  R. É fácil verificar que f satisfaz a condição do
2
enunciado 

EUREKA! N°6, 1999


12
Sociedade Brasileira de Matemática

Você sabia… que o livro


"Problemas de las Olimpíadas
Matemáticas del Cono Sur
(I a IV)" já está a venda na Secretaria
da Olimpíada Brasileira de Matemática?
Informações:
Tel: 21-5295077
e-mail:obm@impa.br

14a. OLIMPÍADA IBEROAMERICANA DE MATEMÁTICA


12 a 19 de setembro, La Havana, Cuba

O Resultado da equipe Brasileira

BRA1 Christian Iveson PRATA


BRA2 Fabrício Siqueira Benevides BRONZE
BRA3 Fernando Paz Cardoso PRATA
BRA4 Rui Lopes Viana Filho OURO

Primeiro Dia

PROBLEMA 1:
Encontre todos os inteiros positivos que são menores que 1000 e cumprem
a seguinte condição: o cubo da soma dos seus dígitos é igual ao quadrado
do referido inteiro.

PROBLEMA 2:
Dadas duas circunferências M e N, dizemos que M bissecta N se a corda
comum é um diâmetro de N.
Considere duas circunferências fixas C1 e C2 não-concêntricas.
a) Prove que existem infinitas circunferências B tais que B bissecta C1 e B
bissecta C2.
b) Determine o lugar geométrico dos centros das circunferências B.

PROBLEMA 3:
Sejam P1, P2,…, Pn n pontos distintos sobre uma reta do plano (n  2).

EUREKA! N°6, 1999


13
Sociedade Brasileira de Matemática

_____
Consideram-se as circunferências de diâmetro Pi Pj (1  i < j  n) e
colorimos cada circunferência com uma cor escolhida entre k cores dadas.
Chamamos (n, k)-nuvem a esta configuração.
Para cada inteiro positivo k, determine todos os n para os quais se verifica
que qualquer (n,k)-nuvem contém duas circunferências tangentes
exteriormente da mesma cor.

Nota: Para evitar ambiguidades, os pontos que pertencem a mais de uma


circunferência não são coloridos.
Segundo Dia

PROBLEMA 4:
Seja B um inteiro maior que 10 tal que cada um dos seus dígitos pertence
ao conjunto {1, 3, 7, 9}. Demonstre que B tem fator primo maior ou igual
a 11.

PROBLEMA 5:
Um triângulo acutângulo ABC está inscrito numa circunferência de centro
O. As alturas do triângulo são AD, BE, e CF. A reta EF intersecta a
circunferência em P e Q.
a) Prove que OA é perpendicular a PQ.
b) Se M é o ponto médio de BC, prove que AP 2 2 . AD. OM .

PROBLEMA 6:
Sejam A e B pontos do plano e C um ponto da mediatriz de AB.Constrói-se
uma sucessão C1, C2,…, Cn ,…., da seguinte maneira:
C1 = C e, para n  1, se Cn não pertence ao segmento AB, Cn+1 é o
circuncentro do triângulo ABCn.
Determine todos os pontos C tais que a sucessão C1, C2,…, Cn,…está
definida para todo n e é periódica a partir de um certo ponto.

EUREKA! N°6, 1999


14
Sociedade Brasileira de Matemática

Nota: Uma sucessão C1, C2, …, Cn,… é periódica a partir de um certo


ponto se existem inteiros positivos k e p tais que C n  p C n para todo
n k .

CONTAR DE DUAS MANEIRAS, PARA GENERALIZAR


José Paulo Q. Carneiro - Universidade Santa Úrsula

 Nível Intermediário
1. O seguinte problema é muito conhecido: quantas partidas há em um
campeonato de futebol de um só turno com n times?

Uma maneira de raciocinar é: cada time vai jogar n  1 partidas (com os outros
n  1 times). Portanto, o número de partidas parece ser n( n  1) . Mas nessa
contagem, cada partida foi contada duas vezes (por exemplo, o jogo Vasco 
Flamengo foi contado entre os jogos do Vasco e também entre os jogos do
n( n  1)
Flamengo). Logo, o número correto é: .
2
Por outro lado, pode-se raciocinar também de outra maneira: O time T1 vai
jogar n  1 partidas. Colocando essas partidas de fora, o time T2 vai jogar
n  2 partidas contra os clubes restantes (agora já estamos evitando a repetição
desde o início!). Além das que já jogou nessa enumeração, o time T3 vai jogar
n  3 partidas, e assim por diante. Então o número total de partidas é:
( n  1)  (n  2)    1 .
( n  1) n
Comparando as duas contagens, conclui-se que: 1  2    ( n  1)  ,
2
que é a célebre fórmula da soma dos n  1 primeiros inteiros positivos. O
interessante é que, ao resolver um problema de contagem por dois métodos
diferentes, chegamos a uma fórmula de caráter geral.

EUREKA! N°6, 1999


15
Sociedade Brasileira de Matemática

Deve ser notado também que esta fórmula é básica para deduzir a fórmula geral
da soma dos termos de uma progressão aritmética, pois:
a1  (a1  r )  (a1  2r )    (a1  (n  1)r ) na1  (1  2    (n  1))r 
(n  1)n n(a  a1  (n  1)r ) n(a1  a n )
na1  r  1  .
2 2 2
2. Vamos agora pensar no número de partidas de um torneio usual de tênis,
onde todos os jogos são eliminatórios. Observe primeiramente o torneio do fim
para o início. A final é jogada pelos 2 vencedores das 2 semifinais (jogadas por 4
jogadores), que vieram de 4 quartas de final (jogadas por 8 jogadores), e assim
por diante. De um modo geral, um torneio desse tipo começa com 2 n jogadores,
que jogam 2 n  1 partidas entre si de forma eliminatória. Na fase seguinte,
sobram 2 n  1 jogadores, que jogam 2 n  2 partidas, e assim sucessivamente, até
a final. O número total de partidas é, portanto: 2 n  1  2 n  2    2  1 .

Vamos agora contar o número de partidas de outra forma. Cada partida tem um
perdedor (não há empate em tênis). Por outro lado, como o torneio é eliminatório,
todos os 2 n jogadores iniciais, exceto o vencedor final, perdem exatamente uma
partida. Logo, o número total de partidas do torneio é 2 n  1 .
Comparando os resultados, conclui-se que 1  2    2 n  2  2 n  1 2 n  1 .

3. A última fórmula é um caso particular da fórmula da soma dos termos de


a (1  q n )
uma progressão geométrica: a  aq  aq 2    aq n 1  .
1 q
Será que esta fórmula também pode ser deduzida por um raciocínio análogo aos
anteriores? Para isto, imaginemos um torneio de tênis também eliminatório, mas
com q n jogadores iniciais (onde q é um natural  1 ), e onde, a partir de um
ranqueamento prévio, os jogadores são grupados em q n  1 grupos (ou “chaves”)
de q jogadores. Dentro de cada grupo, os jogadores previamente ordenados
J 1 ,  , J q jogam entre si da seguinte forma: J 1 contra J 2 ; o vencedor desta
partida joga contra J 3 ; e assim por diante, até J q . Portanto, dentro de cada
grupo, há q  1 partidas, e como são q n  1 grupos, há, nesta primeira fase,
 q  1 q n  1 partidas. De cada grupo sai um vencedor, e os q n  1 vencedores
são novamente grupados em q n  2 grupos de q jogadores. O processo se repete,

EUREKA! N°6, 1999


16
Sociedade Brasileira de Matemática

até que se atinge um único grupo de q jogadores, e desse grupo sai o vencedor do
torneio.
O número total de partidas é  q  1 q n  1   q  1 q n  2    (q  1) . Por outro
lado, por ser o torneio eliminatório, pelo mesmo princípio anterior, o número total
de partidas é também igual ao número inicial de jogadores menos um, isto é:
q n  1.
Comparando as duas contagens, conclui-se que:
( q  1)(1  q  q 2    q n  1 ) q n  1 , que é a fórmula procurada.

Deve-se notar que esta dedução só vale para q natural  1 . Mas a fórmula obtida
serve também como uma sugestão para qualquer q real (ou complexo!). Para
comprovar a sugestão, basta multiplicar os dois fatores do lado esquerdo e
simplificar, obtendo o lado direito (ou observer que dois polinômios cujos valores
coincidem em infinitos pontos são necessariamente iguais).

4. É curioso que se obtenham as fórmulas de soma dos termos de


progressões aritméticas e geométricas através de contagens por dois métodos
diferentes, porém mais importante talvez é o próprio método, que é muito
fecundo e muito usado, por exemplo, em Análise Combinatória.
 n
Só para dar um exemplo concreto, o símbolo   representa o número de
k  
subconjuntos de k elementos que se pode extrair de um conjunto com n elementos
n
(também se usa C k ), e pode ser calculado, como é conhecido, por:
 n  n( n  1) 1
   .
k 1 2  k

Suponha que se deseje calcular quantas comissões de p pessoas se podem formar


a partir de um certo conjunto que consiste de n homens e n mulheres (supõe-se
 2n 
p n ). É claro que este número é igual a   . Por outro lado, se contarmos
 p
separadamente as comissões que são formadas por p homens, mais as que são
formadas por p  1 homens e 1 mulher, mais as que são formadas por p  2
homens e 2 mulheres, etc., concluímos que este número também é igual a

EUREKA! N°6, 1999


17
Sociedade Brasileira de Matemática

 n  n   n  n   n  n 
             . Daí se deduz, portanto, a interessante
 0  p   1  p  1  p  0 
p
 n  n   2n 
fórmula:   k  p  k   p  .
k 0

Para dar um exemplo concreto, tomemos n 6 e p 3 , obtendo:


 6  6   6   6   6  6   6   6 
               1 20  6 15  6 15  1 20 220 
 0  3   1   2   2  1   3   0 
 12 
  .
 3
Espero que o leitor tenha ficado estimulado a criar outros exemplos análogos.

A MATEMÁTICA DAS ABELHAS


José Cloves Saraiva - UFMA
 Nível Avançado
As abelhas constróem colméias para reservar o mel e para o
desenvolvimento e reprodução da sua própria espécie. Cada alvéolo tem a forma
de um prisma reto hexagonal fechado numa das extremidades por uma cobertura
de três losângos de forma que o volume é máximo enquanto a área lateral total
(incluindo a cobertura) é fixada. Com isso as abelhas fazem o menor gasto de
cera na construção das paredes peliculares que o constituem. Nesta construção
econômica cada parede serve para dois alvéolos contíguos, o que motiva a
escolha de prismas retos com seção transversal poligonal regular, como nas
figuras abaixo:

É fácil ver que as formas prismáticas com seção transversal um polígono


regular de n lados que podem ser usadas para cobrir o espaçõ determinadas pela

EUREKA! N°6, 1999


18
Sociedade Brasileira de Matemática

360
condição de que é um número inteiro positivo, onde In é o ângulo interno
In
de um polígono regular.

360 360 2n 2n  4  4 4
   2 
Podemos escrever In   n  2 180  n  2 ( n  2) ( n  2)
 
 n 
que é um inteiro positivo se e só se n – 2 é um divisor de 4, logo devemos ter n =
3,
n = 4 ou n = 6 como mostra a figura acima.

Por um cálculo volumétrico simples das três posibilidades acima


observando o mesmo volume, ou escolhendo a forma mais convexa, das três
acima indicadas a escolha das abelhas pela forma hexagonal é de fato a mais
econômica (dentre um triângulo, um quadrado e um hexágono regulares de
mesmo perímetro o hexágono tem a maior área.)

Eis como as abelhas colocam os seus alvéolos hexagonais. Esses


alvéolos, para maior economia de material , são fechados por três losangos iguais.
O valor constante do melhor ângulo agudo de um losango de fechamento causou
sério debate entre teólogos, naturalistas e matemáticos.

EUREKA! N°6, 1999


19
Sociedade Brasileira de Matemática

O problema que as abelhas resolvem pode ser abordado, sem grande


dificuldade, com os recursos da Matemática elementar.

Como mostra a figura acima, as abelhas fecham uma das bases do prisma
com três losangos congruentes cujo ângulo agudo do losango, , mede
aproximadamente 7032', o que dá origem ao problema do ângulo de fechamento
da cobertura rômbica, que é o seguinte:

De todas as coberturas rômbicas de um prisma reto hexagonal, a que


faz volume máximo para área lateral total fixada é formada por três
losangos congruentes com ângulo agudo   7032'.

Para uma solução deste problema observe primeiro a comparação do


prisma hexagonal com a cobertura rômbica:

E
D A
C B
B E
____
BB' h  r
B
D A
B' r r
r C
A figura também ajuda a obtermos as relações métricas necessárias para
deduzir o volume V e a área lateral total S em função do raio r da base hexagonal
e do ângulo , com vértice no ponto D, e os itens seguintes são facilmente
comprovados:

____ ____
1) O trapézio ACDE é retângulo com bases CD e AE perpendiculares ao
____
plano que contém o triângulo equilátero ABC, de lado AB medindo r .
____ ____
2) EA 2 DC

EUREKA! N°6, 1999


20
Sociedade Brasileira de Matemática

____ 2
____ 2 ____ 2
3) 2 EA
BD  DE r 
4
____ 2 ____ 2
___ 2  
BE 2   2 EA  
4)  BD sen  r  sen 2
4 2  4  2
 
5)
____ 2 ___ 2 ___ 2 1/ 2
 ___ 2 2 ___ 2
  
BE  EA  r 2  (4r 2  EA ) sen 2  EA  r  EA r  3tg 2  1
2  2 
6) A área lateral total S do prisma é dada por:

1/ 2
3  r2  2   
S 6  r 2 tg   3tg  1  rh 
 4 2 4  2  
BD DE sen
Usamos aqui que a área de BDE é 
2
___ 2 
2tg
1 2 EA 3 2  3  2 ).
 (r  ) sen  r (1  tg 2 ) sen  r 2 tg (pois sen 
2 4 8 2 4 2 2 
1  tg
2
7) O volume V do prisma é dado por:

1/ 2
3 3 2 r  2   
V  r   3tg  1  h
2  2  2  
Fixe V, e obtemos a seguinte expressão para S em função de .

1/ 2
3       4V
S  r 2 3tg   3tg 2  1  
2  2  2   3r
Observando a expressão acima, vemos que ela torna-se mínima se
1/ 2
   2   
3tg   3tg  1  for mínimo para valores de  com a condição
 2  2  
 3
tg  . A seguinte desigualdade trigonométrica, que provaremos a seguir,
2 3
garante o resultado do problema do ângulo (agudo) de fechamento da cobertura
do prisma:

EUREKA! N°6, 1999


21
Sociedade Brasileira de Matemática

1/ 2
 3    
Se S tg  então 3tg   3tg 2  1  2 , valendo a igualdade se e
2 3 2  2 
 2
só se tg  .
2 2

Demonstração: Para todo  vale:


2
   2  
3 2 tg  1 0  6 tg  6 2tg  3 0
 2  2 2

Somando-se 3tg 2  1 aos dois membros da desigualdade obtemos:
2
2
      
9tg 2  6 2 tg  2 3tg 2  1   3tg  2  3tg 2  1 
2 2 2  2  2
1/ 2
     3
3tg  2  3tg 2  1 , e como tg  ,
2  2  2 3

3tg  2  0, logo:
2
1/ 2 1/ 2
       
3tg  2  3tg 2  1  3tg   3tg 2  1  2
2  2  2  2 
Daí, é claro que, o menor valor de S é determinado pela igualdade:
1/ 2 2
  2     
3tg   3tg  1  2   2 tg  1 0 
2  2   2 
 2 2
tg    2arc tg 70 32'
2 2 2

Finalizando estas notas, sugerimos que o leitor faça a prova de que, o


ângulo  é o mesmo ângulo agudo entre diagonais internas de um cubo.

EUREKA! N°6, 1999


22
Sociedade Brasileira de Matemática

Referência:
- Malba Tahan, As Maravilhas da Matemática, Bloch Editores.

O TEOREMA DE RAMSEY
Carlos Gustavo T. de A. Moreira - IMPA

 Nível Avançado
1. O teorema de Ramsey para grafos.
Vamos começar este artigo lembrando do exemplo 6 do artigo "O Princípio das
Gavetas", de Paulo Cezar Pinto Carvalho, publicado na EUREKA! No. 5: se há 6
pessoas numa reunião então há necessariamente 3 pessoas que se conhecem
mutuamente ou 3 pessoas que não se conhecem mutuamente na reunião (onde
admitimos que, se a conhece b, então b conhece a). Este exemplo equivale ao
seguinte: se tomamos 6 pontos, e pintamos cada segmento que une dois desses
pontos de preto ou vermelho então necessariamente existe um triângulo cujos
vértices são três desses pontos e cujos 3 lados são da mesma cor.

Nos exercícios 8 e 9 do mesmo artigo é proposta uma generalização:

Proposição 0: Dados os inteiros a, b  2 sempre existe um número N inteiro


positivo tal que, em qualquer conjunto de N pessoas, sempre existem a pessoas
que se conhecem mutuamente ou b pessoas que se desconhecem mutuamente. Em

EUREKA! N°6, 1999


23
Sociedade Brasileira de Matemática

homenagem a F. P. Ramsey, que provou este e outros resultados deste artigo,


chamamos o menor número N com esta propriedade de R(a, b).

Vamos provar este resultado seguindo os passos propostos do problema 9 do


artigo do Prof. Paulo Cezar:

i) R(a, 2) = a para todo a  2.


De fato, num conjunto de a pessoas, ou todas se conhecem ou existem
duas que se desconhecem.

ii) R(a, b) = R(b, a).


De fato, conhecer e desconhecer desempenham um papel simétrico no
enunciado.

iii) R ( a, b) R ( a  1, b)  R (a, b  1), para a, b 3.

Para provar isto, fixemos uma pessoa P. Se temos R ( a  1, b)  R ( a, b  1) ou


mais pessoas na reunião, há pelo menos R ( a  1, b)  R ( a, b  1)  1 outras
pessoas, e um dos seguintes casos se verificará:
1) P conhece pelo menos R ( a  1, b) pessoas.
Neste caso, por definição de R ( a  1, b) , dentre essas R ( a  1, b)
pessoas há b que se desconhecem mutuamente (e não temos mais o que
provar), ou a – 1 que se conhecem mutuamente, e, juntando P a essas a –
1 pessoas, obtemos a pessoas que se conhecem mutuamente.

2) P desconhece pelo menos R ( a, b  1) pessoas.


A análise deste caso é análoga à do caso anterior, trocando os papeis de
conhecer e desconhecer no argumento 
Já mencionamos que podemos enunciar resultados deste tipo em termos de
colorações de segmentos (ou mais tecnicamente, de arestas de grafos completos)
usando duas cores. O resultado acima pode ser generalizado aumentando o
número de cores:

Proposição 1: Dados inteiros k 1, a1 , a 2 ,...a k 2 existe n  N tal que dados


n pontos, se pintarmos cada segmento que une dois desses pontos de uma dentre
k cores possíveis então haverá a1 pontos tais que todo segmento que une dois
desses pontos é da primeira cor, ou a2 pontos tais que todos os segmentos que
unem dois desses pontos são da segunda cor, ou… ou ak pontos tais que todos os

EUREKA! N°6, 1999


24
Sociedade Brasileira de Matemática

segmentos que unem dois desses pontos são da k-ésima cor. Chamamos de
R (a1 , a 2 ,...a k ) o menor número com essa propriedade.

Demonstração: Esta proposição pode ser provada de modo análogo à anterior. A


existência dos números R ( a1 ,...a k ) segue dos seguintes fatos, cuja prova
deixamos como exercício:

i) R ( a1 ,...a k  1 , 2) R ( a1 ,...a k  1 ) para a1 ,...a k  1 2.


ii) R (a (1) , a ( 2) ,...a ( k ) ) R( a1 , a 2 ,...a k ) para a1 ,...a k 2 e
qualquer permutação  de {1, 2, …, k}.

R (a1 ,...a k ) R (a1  1, a 2 ,..., a k )  R (a1 , a 2  1, a 3 ,..., a k )  ... 


iii)
 R (a1 , a 2 ,...a k  1 , a k  1)  k  2.

2. Estimativas de números de Ramsey.


As demonstrações dos resultados anteriores fornecem estimativas superiores
para os números de Ramsey R ( a, b) e R ( a1 ,...a k ) , por exemplo:

( a  b  2)!
i) R( a, b) C aa b1 2  , para a, b 2
( a  1)!(b  1)!
De fato vale a igualdade para b = 2 (temos os dois lados iguais a a), e se
a, b > 2 temos, por indução,
R ( a, b) R( a  1, b)  R ( a, b  1) C aa b2 3  C aa b1 3 C aa b1 2 

ii)
(a1  ...  a k  2k  2)!
R(a1 ,..., a k )  : C aa11 ...1,a2ak 1,2a3k 22,...ak  2
(a1  1)!(a 2  1)!(a 3  2)!(a 4  2)!...(a k  2)!
De fato, isso vale quando todos os ai exceto dois são iguais a 2, e o caso
geral segue, por indução da identidade
b1 ,..., bk  1,bk  1
C bb11,..., bk b1  1,b2 ,..., bk b1 ,b2  1,..., bk
b2 ...bk C b1 ...bk  1  C b1 ...bk  1  ...  C b1 ...bk  1 

EUREKA! N°6, 1999


25
Sociedade Brasileira de Matemática

É menos trivial, entretanto, dar boas estimativas inferiores para números de


Ramsey. Para provar, por exemplo, que R (3,3) 6, é necessário mostrar que há
exemplos de grafos completos bicoloridos de 5 vértices sem triângulos monocromáticos,
o que pode ser feito explicitamente:

Se o número de pontos, cresce, entretanto, é bastante difícil construir exemplos


explícitos. As melhores estimativas conhecidas para R(k, k) se devem ao método
probabilistico introduzido pelo grande matemático húngaro Pál Erdös, que se
tornou uma das técnicas mais poderosas da teoria dos grafos:

Proposição 2: R(k, k) > 2k/2 para todo k  N.


Demonstração: Dados n pontos, pintamos aleatoriamente as arestas que ligam
dois desses pontos de vermelho ou preto, com probabilidade 1/2. Dado um
k ( k  1)
subconjunto de k desses pontos, a probabilidade de que todas as
2
k ( k  1) / 2
1
arestas que unem dois desses pontos sejam da mesma cor é 2   .
 2
k n!
Como há C n  subconjuntos de k pontos do conjunto inicial de n
k! ( n  k )!
pontos, a probabilidade de que em algum deles todas as arestas sejam da mesma
k ( k  1) / 2 k ( k  1) / 2
1 nk  1 
cor é no máximo 2C nk   2   que é menor ou igual a
 2 k!  2 
k / 2 1
2
se n 2 k / 2 , mas 2 k / 2 1  k! para k  4, donde há probabilidade
k!
positiva de que em nenhum subconjunto de k pontos todas as arestas sejam da
mesma cor, e em particular há exemplos desta situação, donde R(k, k) é
necessariamente maior que 2k/2 para k  4.
Para k = 3, R(k, k) = 6 > 23/2, e para k =2, R(k, k) = 2 > 21/2, o que conclui a
demonstração 
Vamos discutir agora como estimar a função f ( n) R (3,3,...,3), com n
termos iguais a 3, ou seja, o menor número de vértices de um grafo completo tal

EUREKA! N°6, 1999


26
Sociedade Brasileira de Matemática

que ao pintarmos suas arestas usando n cores necessariamente obtemos um


triângulo monocromático.
A demonstração da proposição 1 nos fornece f ( n) nf ( n  1)  n  2, o que
implica, por exemplo, f (n) 3n! para todo n  2. (lembremos que
f (3) R (3,3) 6).
3n  1
Suponha agora que sejam dados pontos, aos quais atribuímos índices 0,
2
3n  1
1, 2, … . Vamos descrever uma forma de, a cada par desses pontos,
2
atribuir um número entre 0 e n – 1 (o que equivale a colorir as arestas do grafo
3n  1
completo de vértices usando n cores), sem que haja 3 pontos tais que a
2
cada par desses pontos é atribuido o mesmo número (ou seja, sem que haja
triângulos monocromáticos).
Ao par de pontos de índices i e j atribuímos um número da seguinte maneira:
n 1
escrevemos i j como 
r 0
r 3 r onde  r { 1,0,1} para todo r, e

atribuimos a {i, j} o menor k com  k 1 (sempre existe um tal k pois i  j >


0).
Deixamos como exercício para o leitor verificar que não há triângulos
monocromáticos nesta configuração (ver problema proposto N o. 32 pág 55).
3n  1
Obtemos assim a estimativa f ( n)  .
2
Determinar exatamente os valores de números de Ramsey clássicos R(a, b) com
3  a  b que são conhecidos são: R(3, 3) = 6, R(3, 4) = 9, R(3, 5) = 14,
R (3, 6) = 18, R(3, 7) = 23, R(3, 8) = 28, R (3, 9) = 36, R(4, 4) = 18,
e R(4, 5) = 25.
O único número Ramsey com mais de duas cores cujo valor é conhecido é
R(3, 3, 3) = 17.

3. O Teorema de Ramsey para multigrafos:


Até agora estivemos falando sobre colorações de arestas de grafos, ou seja, a cada
conjunto de dois vértices associamos uma cor. Uma maneira de generalizar este
resultado é associar cores não a pares de vértices, mas a conjuntos de m vértices,
onde m é um inteiro positivo fixo (que pode ser maior que 2). Como as
configurações que aparecem são um pouco mais complicadas, vamos introduzir
notações um pouco mais formais:

EUREKA! N°6, 1999


27
Sociedade Brasileira de Matemática

Dado um conjunto A e um inteiro positivo k denotamos por [A]m o conjunto dos


m
subconjuntos de m elementos de A, ou seja [ A] {B  A # B k} .
Dado j inteiro positivo, definimos Ij = {1, 2, …, j}.
A versão m-dimensional (ou para m-hipergrafos) do teorema de Ramsey é dada
pelo seguinte teorema (do qual os resultados da seção 1 são casos particulares):

Teorema (Ramsey): Sejam m, k inteiros positivos. Dados a1, a2,…, ak inteiros


positivos existe um inteiro positivo, que denotaremos por R m ( a1 ,..., a k ) tal
que para todo n R m ( a1 ,..., a k ) e para qualquer função f : [ I n ] m  I k existem
j  I k e A  I n com # A a j tal que f ([ A] m ) :{ f ( x), x  [ A] m }  { j}.
Demonstração: Para m = 1 o resultado é uma aplicação simples, do princípio das
gavetas: basta tomar R1 ( a1 ,..., a k ) a1  ...  a k  k  1. Vamos provar o
resultado geral por indução em m. De fato, provaremos que podemos tomar
R m ( a1 ,..., a k ) 1  R m  1 ( R m ( a1  1,..., a k ),
R m ( a1 , a 2  1..., a k ),..., Rm ( a1 , a 2 ,...a k  1))
(e se algum dos ai é menor que m podemos tomar
Rm ( a1 ,..., a k ) min{a1 , a 2 ,..., a k }). Note que isto fornece exatamente a
recursão de proposição 1 da seção 1 no caso m = 2.
Se n 1  R m  1 ( Rm ( a1  1, a 2 ,..., a k ),..., R m (a1 , a 2 ,...a k  1)), dada uma
~
função f : [ I n ] m  I k , definimos uma função f : [ I ] m  1  I da seguinte
n 1 k
~
forma: dado A  [ I n  1 ] m  1 , definimos f ( A) : f ( A  {n}).
Como n  1 R m  1 ( Rm ( a1  1,..., a k ),..., R m ( a1 ,...a k  1)), existem
j  Ik
e B  I n  1 com # B R m ( a1 ,..., a j  1,..., a k ) tal que f ([ B ] m  1 ) { j}.
Agora, por definição de Rm (a1 ,..., a j  1,..., a k ) existe i  I k com i  j e

A  B com# A ai e f ([ A] m ) {i}, caso em que já conseguimos o que queríamos,

ou existe A  B com # A a j  1 e f ([ A] m ) { j}. Nesse caso, teremos


# ( A  {n}) a j e f ([ A  {n}] m ) { j}, pois A  B , e novamente
conseguimos nosso objetivo .

EUREKA! N°6, 1999


28
Sociedade Brasileira de Matemática

Obs.: O único número de Ramsey cujo valor é conhecido com


m  2 (e a j  m,  j ) é R3 ( 4,4) 13 .
O exercício 20 de [PC] pede para mostrar que dados 5 pontos no plano em
posição geral há 4 que formam um quadrilátero convexo. O leitor poderia
perguntar: o que faz um problema geométrico como este num artigo de
combinatória? A resposta está ligada a uma generalização deste resultado,
descoberta por Erdös e Szekeres:
Dado um inteiro positivo n 4 existe um inteiro positivo f(n) tal que dados f(n)
pontos no plano em posição geral há n deles que são vértices de um n-ágono
convexo.
Para provar isso, mostraremos que podemos tomar f ( n)  R4 ( n,5). Para isso,
dados R4 ( n,5) pontos no plano, e um conjunto de 4 desses pontos,
associamos a esse conjunto o número 1 se eles formam um quadrilátero
convexo, e 2 em caso contrário.Não é possível que haja 5 pontos tais que a
cada 4 deles é associado o número 2, pelo resultado do exercício 20 de [PC],
donde, por definição de R4 ( n,5) , necessariamente há n desses pontos tais
que cada 4 desses pontos formam um quadrilátero convexo, mas isso implica
que esses n pontos são vértices de um n-ágono convexo.

5. O teorema de Ramsey infinito:


Teorema: Sejam m, k inteiros positivos e A um conjunto infinito.
Para qualquer função f: [ A] m  I k existem j  I k e um conjunto infinito
B  A tal que f ([ B] m ) { f ( x) x  [ B] m } { j}

Demonstração: Vamos provar o resultado por indução em m. Para m = 1 o


resultado segue do fato de que se X é infinito e C é finito então para toda função
f : X  C existe C  C tal que f  1 (c) {x  X f ( x ) c} é infinito.
Seja agora m  2 e f : [ A] m  I k , onde A é infinito.Fixamos xo  A, e

definimos Ao  A \ {xo } e g : [ A ]
o
m 1
 I k por g (C ) : f (C  {x o }), onde C é
um subconjunto de m –1 elementos de Ao . Pela hipótese de indução existe um
conjunto infinito Bo  Ao e j o  I k tal que g o ([ Bo ] m 1 ) { j o }. A partir daí
repetimos o processo recursivamente: dado n  0 fixamos x n 1  Bn e definimos

An1 Bn \ {xn1} e g n1 [ An 1 ] m 1  I k por g n 1(C )  f (C  {x n 1 })

EUREKA! N°6, 1999


29
Sociedade Brasileira de Matemática

para C  An 1 com m –1 elementos. Pela hipótese de indução existe


Bn 1  An 1 infinito e j n 1  I k tal que g ([ Bn 1 ] m  1 ) { j n 1 }.
Podemos agora tomar D = {x0, x1, x2,…}, que é um conjunto infinito e definir
h : D  I k por h( x r )  j r . Como Ik é finito, existe jIk tal que
h  1 ( j ) {x  D h( x )  j} é infinito. Afirmamos que B  h  1 ( j ) satisfaz a

condição do enunciado. De fato, dado um subconjunto X {xi1 , xi2 ,. , xim } de B com

m elementos, temos f (X ) gi1 ({xi2 ,. ., xim })  ji1 h(xi1 )  j )

Sugestão: Tente usar os resultados deste artigo para resolver o problema


“Cuático”da Eureka! No5. Pág. 58. (ver também pág. 56 desta EUREKA!).

Referências:

-[R]- Ramsey, F.P., On a Problem of Formal Logic, Proc. London Math. Soc. 30
(1930) P P. 264-286.
-[PC]- Paulo Cezar Pinto Carvalho, Princípio das Gavetas – Eureka! 5 PP.27-33.
-[GRS]- R.L. Graham, , B.L. Rothscild, e J.H. Spencer, Ramsey Theory. Wiley.
Interscience, 1990.
-[ES]- Erdös e G. Szekeres – A Combinatorial Problem in Geometry – Compositio
Math.2 (1935), PP-464-470.
-[Rad]- Stanislaw P. Radziszowski – “Smal “Ramsey Numbers – Dinamic Surveys-
Electronic Journal of Combinatorics. http://www.combinatorics.org
-[Er]- Pál Erdös, Some Remarks on The Theory Of Graphs, Bull. Amer. Math. Soc. 53
(1947), PP.292-294.
APLICAÇÕES DOS NÚMEROS COMPLEXOS À GEOMETRIA
Edmilson Motta - Colégio Etapa
 Nível Avançado
É importante ter em mente que os números complexos não são apenas
vetores; eles podem ser multiplicados. Nas aplicações à Geometria, nós faremos
uso extensivo desta propriedade. Números Complexos são particularmente

EUREKA! N°6, 1999


30
Sociedade Brasileira de Matemática

eficientes para certos tipos de problemas, mas podem gerar dificuldades artificiais
em problemas que admitem soluções mais diretas utilizando outros métodos.
Na Geometria Elementar, os triângulos são as peças básicas e a
congruência e a semelhança de triângulos, os conceitos fundamentais. Nós
começaremos caraterizando a semelhança de triângulos em termos de números
complexos. Inicialmente, vamos estabelecer algumas convenções. Sejam z1, z2, z3,
w1, w2, w3 números complexos. Nós dizemos que z1z2z3 e w1w2w3 são
semelhantes, e escrevemos  z1 z2 z3 ~  w1w2w3, se e somente se, o ângulo em zk
é igual ao ângulo em wk , k = 1, 2, 3, e têm mesma orientação, isto é, ambos anti-
horários ou ambos horários (Veja figura a seguir.)
z1
w2

w3

z3
w1

z2

Se os triângulos tem orientações distintas (um horário, o outro anti-


horário), então escrevemos
z1 z2 z3 ~ w1w2w3 (reverso)

Como para complexos ,  e  distintos,

 
arg arg (    )  arg (   )  medida do ângulo orientado
 
entre  e  , então , ,  são colineares  a medida do ângulo orientado
 

 
entre  e  é múltipla de 
___________ __ __
           
 R      .
          __ __
 
Exercício 1:

EUREKA! N°6, 1999


31
Sociedade Brasileira de Matemática

__ __
  
    
Mostre que       é imaginário puro   0.
   __ __
 
Generalizando, para quatro pontos distintos , , ,   C,
__ __
       
 ||   R   ,
    __ __
 
   
além disso,  e  tem a mesma direção se, e somente se,    é um real
__ __
 

    
positivo;     é imaginário puro   0.
      __ __
 
Exercício 2:

Mostre que, se   0, então          0.

z1 w1 1
z2  z1 w2  w1
Teorema: z1 z2 z3 ~ w1w2w3    z2 w2 1  .0
z3  z1 w3  w1
z 3 w2 1
Demonstração: Dois triângulos são semelhantes se, e somente se, (caso LAL) as
razões entre as medidas de dois pares de lados correspondentes são iguais e os
ângulos entre estes lados são iguais (incluindo a orientação).
Assim,
z 2  z1 w2  w1 z  z1 w  w1
z1 z2 z3 ~ w1w2w3   e arg 2  arg 2
z 3  z1 w3  w1 z 3  z1 w3  w1

EUREKA! N°6, 1999


32
Sociedade Brasileira de Matemática

z1 w1 1
z2  z1 w2  w1
   z2 w2 1  .0
z3  z1 w3  w1
z3 w3 1
Corolário:

__
z1 w 1 1
__ __
z2  z1 w2  w1 __
z z z ~ w w w (reverso) 
1 2 3 1 2 3  __ __  z2 w2 1 0.
z 3  z1
w3  w1 __
z3 w 3 1
__ __ __
Demonstração: Temos que  w w w ~ w1 w2 w3 (reverso). Logo
1 2 3
__ __ __
z1 z2 z3 ~ w1w2w3 (reverso)  z1 z2 z3 ~  w w w
1 2 3

Exercício 3:
__ __ __
Complete a demonstração acima, verificando que  w w w ~ w1w2w3
1 2 3
(reverso).

Vejamos agora alguns exemplos de aplicações mais específicas.

EUREKA! N°6, 1999


33
Sociedade Brasileira de Matemática

 CARACTERIZAÇÕES DOS TRIÂNGULOS EQUILÁTEROS.


Sendo  2    1 0,

z1 z 3 1
z1 z2 z3 é equilátero z1 z2 z3 ~ z3 z1 z2  z 2 z1 1 0
z3 z 2 1
 z12  z 22  z 32  z 2 z 3  z 3 z1  z1 z 2 0
 ( z1   z 2   2 z 3 ) ( z1   2 z 2   z 3 ) 0
 ( z1   z 2   2 z 3 ) 0 ou ( z1   2 z 2   z 3 ) 0

z1 1 1 z1 1 1
2
 z2  10 ou z2  10
z3  1 z3  1
2
z1 z2 z3 ~ 1 2 ou z1 z2 z3 ~12
(geometricamente, esta última caracterização é bastante intuitiva).

EUREKA! N°6, 1999


34
Sociedade Brasileira de Matemática

x
0 1

2

 O TEOREMA DE NAPOLEÃO.
Sobre cada lado de um triângulo arbitrário, desenhe um triângulo equilátero
(no exterior). Temos então que os baricentros desses três triângulos
equiláteros são os vértices de um quarto triângulo equilátero.
w3 w2
z1
2
3

z3
z2

1

w1
Demonstração:
Sejam z1z2z3 o triângulo dado; w1z3z2, z3w2z1, z2z1w3 triângulos equiláteros
com a mesma orientação que 12, digamos; e 1, 2, 3, os baricentros desses
triângulos. Então
w1   z 3   2 z 2 0
z 3   w2   2 z1 0
z 2   z1   2 w3 0
Para provarmos que 123 é equilátero, calculamos
1  2
 1   2   2 3  ( w1  z 3  z 2 )  ( z 3  w2  z1 )  ( z 2  z1  w3 )
3 3 3

EUREKA! N°6, 1999


35
Sociedade Brasileira de Matemática

=
1
(( w1   z 3   2 z 2 )  ( z 3   w 2   2 z1 )  ( z 2   z1   2 w3 )) 0.
3
Portanto123 é um triângulo equilátero.

Exercício 4:
Sendo , , ,  números complexos, temos que
(   ).(   )  (   ) (    ) (   ) (    )
(a) A partir da identidade acima, mostre que
                      .
(b) Demonstre o teorema de Ptolomeu-Euler:

Para quaisquer pontos A, B, C, D no plano, AB  CD + BC  DA  AC  BD,


com igualdade se, e somente se, estes quatro pontos são concíclicos ou
colineares.

Exercício 5:
(a) Sejam a, b, c, d, e, f números complexos. Prove que

a b c d e f b c a e f  d
.  1    1.
c b e d a f a c f  e b d
(b) Utilizando o item anterior, resolva o seguinte problema do banco da IMO 98:
Seja ABCDEF um hexágono convexo tal que
AB CD EF
 B   D  F 360  e   1.
BC DE FA
BC AE FD
Prove que   1.
CA EF DB

MAIS EXEMPLOS
PROBLEMA (BANCO / IMO 98)
Seja ABC um triângulo, H o seu ortocentro, O o seu circuncentro e R o seu
circunraio. Seja D o simétrico de A com relação a BC, E o simétrico de B com
relação a AC e F o simétrico de C com relação a AB.
Prove que D, E e F são colineares se, e somente se, OH = 2R.

RESOLUÇÃO:

EUREKA! N°6, 1999


36
Sociedade Brasileira de Matemática

Sejam a, b, c, h e 0 as coordenadas complexas do A, B, C, H e O,


__ __ __
respectivamente. Conseqüentemente, a a b b c c  R 2 e h = a + b + c.
Como D é o simétrico de A com relação a BC, d satisfaz
_________
d  b  a  b    __ __ __
  __ __ 
b c   d  b  c a  b c  b c   0. (1)
     
c b c b
Temos que
__ __
R 2 (b  c ) __ __
R 2 (b 2  c 2 )
b  c  e b c bc  ,
bc bc
substituindo em (1), obtemos
 bc  ca  ab k  2bc
d   ,
a a
R 2 ( a  b  c) R 2 ( h  2a )
d   ,
bc bc

onde k = bc + ca + ab. Analogamente


k  2ca __ R 2 (h  2b) k  2ab __
R 2 ( h  2c )
e , e  ,f  e f  .
b ca c ab
Como

EUREKA! N°6, 1999


37
Sociedade Brasileira de Matemática

_
dd1 2
(b a)(k  2ab) R (a b)(h c)2
__
_ e d e d
ab a b c
  ee 1  _ _  2
_ f  d f d ( c  a )( k  2 ca ) R ( a  c )( h  b )2
ff 1 ca abc

EUREKA! N°6, 1999


38
Sociedade Brasileira de Matemática

2
R (c-a)(a  b)  (ck-2abc) (h  2c)
 222 
a b c (bk  2abc)  (h  2b)
 R 2 (b  c)(c  a )(a  b)(hk  4abc)

a 2b 2c 2
__
e h  R 2 k / abc, segue que D, E e F são colineares

  0
 hk  4abc 0
__
 h h 4R 2
 OH  2 R.

PROBLEMA (Olimpíada Chinesa 98)


Seja D um ponto no interior de um triângulo acutângulo ABC, com
DA  DB  AB + DB  DC  BC + DC  DA  CA = AB  BC  CA. Determine quais
são as possíveis posições que D pode ocupar.

RESOLUÇÃO:
Sejam a, b, c, e 0 as coordenadas complexas de A, B, C e D, respectivamente.
Temos, então que DA  DB  AB + DB  DC  BC + DC  DA  CA = AB  BC  CA
 a b (b  a )  b c (c  b)  c a (a  c)  (b  a )(c  b)( a  c ) (*)
Como
ab(b  a )  bc(c  b)  ca(a  c)  (b  a )(c  b)(a  c) , sendo
w1 ab(b  a),
w2 bc(c  b), w3 ca (a  c), (*)  w1  w2  w3  w1  w2  w3 e
portanto, w1, w2, w3 estão alinhados.
Assim, existem reais positivos  e  tais que

EUREKA! N°6, 1999


39
Sociedade Brasileira de Matemática

w1 w2 ab(b  a) bc(c  b) a(b  a) c(c  b)


a  b c
   b
 
 a c
,

w1 w3 ab(b  a) ca(a  c) b(b  a) c(a  c)


 
isto é,  A C B 180   A DB e, analogamente,
   
 A B C 180   A DC e  B A C 180   B DC. O único ponto D no
interior de um triângulo acutângulo que satisfaz essas condições é o ortocentro.

PROBLEMA (Olimpíada Universitária Húngara 1995)


São dados n pontos na circunferência unitária de modo que o produto das
distâncias de qualquer ponto da circunferência a estes pontos é menor ou igual a
2. Prove que os pontos são vértices de um n–ágono regular.

RESOLUÇÃO:
Considere a circunferência centrada na origem e sejam z1, z2, …, zn os números
complexos que representam os pontos. Podemos assumir que
(  1) n z1 z 2 ...z n 1 (verifique! ).
Considere ainda o seguinte polinômio
p ( w) ( w  z1 )( w  z 2 )...(w  z n ) 
w n  a1 w n  1  ...  a n  1 w  1 w n  Q ( w)  1
Então p(z) é o produto das distâncias do ponto representado pelo número
complexo z aos pontos dados .
Logo, se z é um número complexo de módulo 1, então  p(z)|  2.
Sejam w1, w2,… wn as raízes n-ésimas da unidade.
Sabe-se que w1k  w2k  ...  wnk 0 para todo k = 1, 2,…,n – 1. Portanto
Q( w1 )  Q ( w2 )  ...  Q( wn ) 0. Se Q(w) não é identicamente nulo, então,
para algum j, Q (wj) é diferente de zero e tem parte real não negativa, pois Q(0) =
0 e Q tem no máximo n – 1 raízes. Consequentemente,
p ( w j )  2  Q ( w j )  2 , uma contradição.
Desta forma o polinômio Q é identicamente nulo e p(z) = zn + 1. As raízes z1, z2,
…, zn do polinômio p(z) formam um n-ágono regular.

Mais alguns exercícios relacionados.

EUREKA! N°6, 1999


40
Sociedade Brasileira de Matemática

PROBLEMA 1 (IMO 63)


Todos os ângulos internos de um n-ágono são iguais e seus lados satisfazem a
relação a1 a 2 ... a n . Prove que a1 a 2 ... a n .

PROBLEMA 2 (Treinamento para IMO 92)


Prove que para todo inteiro positivo n, ( 2  i ) n ( 2  i ) n . Conclua que os
ângulos agudos do triângulo de lados 3, 4 e 5 são irracionais quando expressos
em graus.

PROBLEMA 3 (IMO 75)


Determine se existem ou não 1975 pontos sobre a circunferência unitária tais que
a distância entre quaisquer dois é um número racional.

PROBLEMA 4 (Olimpíada Romênia 92)


Sejam p, q  C, q  0. Se as raízes da equação x 2  px  q 0 têm o mesmo
p
módulo, mostre que q é um número real.

PROBLEMA 5 (Seleção para IMO 92)


A área do polígono A1 A2 ... An é S. São dados um ângulo  e um ponto Q.
Rodemos Q de um ângulo  no sentido anti-horário ao redor de Ai para encontrar
um ponto Pi. Ache a área de P1 P2 ...Pn .

PROBLEMA 6 (Putnam 67)


Seja ABCDEF um hexágono inscrito em uma circunferência de raio r. Mostre que
____ ____ ___
se AB = CD = EF = r, então os pontos médios de BC , DE , e FA são os
vértices de um triângulo equilátero.

PROBLEMA 7 (IMO 86)


São dados um triângulo A1 A2 A3 e um ponto P0 no plano. Definimos
As  As  3 para todo s 4 e construímos uma seqüência de pontos
P0 , P1 , P2 ,... tais que Pk 1 é a imagem de Pk sob a rotação com centro

EUREKA! N°6, 1999


41
Sociedade Brasileira de Matemática

Ak 1 e ângulo de 120 (sentido horário). Prove que se P1986 = P0 , então o


triângulo A1 A2 A3 é equilátero.

PROBLEMA 8 (Olimpíada Hungria 1899)


A0 , A1 , A2 , A3 , A4 dividem a circunferência unitária em cinco partes iguais.
Prove que ( A0 A1 A3 A2 ) 2 5.

PROBLEMA 9 (Putnam 55)


A1 , A2 ,... An é um polígono regular inscrito em uma circunferência de raio r e
n

centro O. P é um ponto sobre OA1 . Mostre que  PA
k 1
k OP n  r n .

PROBLEMA 10
Dados um ponto P sobre uma circunferência unitária e os vértices
A1 , A2 ,... An de um n-ágono regular inscrito, prove que
PA 12  PA 22  ...  PA n2 e PA 14  PA 24  ...  PA n4
são constantes.

Bibliografia:

- Complex numbers & Geometry, The Mathematical Association of America


Liang - Shin Hahn, 1994.
40a. OLIMPÍADA INTERNACIONAL E 14a. OLIMPÍADA
IBEROAMERICANA DE MATEMÁTICA
Segundo teste de Seleção
PROBLEMA 1
Encontre todos os inteiros positivos l, m, n, primos dois a dois, tais que
(l + m + n)(1/l + 1/m + 1/n) é um inteiro positivo.

PROBLEMA 2
Considere ABC inscrito em uma circunferência fixa. Sejam D, E e F pontos
____ ____ ___
arbitrários distintos dos vértices, pertencentes aos lados BC , CA e AB,
respectivamente. Sejam P, Q e R os pontos de intersecção das retas AD, BE e CF
com a circunferência, respectivamente. Mostre que

EUREKA! N°6, 1999


42
Sociedade Brasileira de Matemática

AD BE CF
  9.
PD QE RF
Determine também as condições sobre o ABC e os pontos D, E e F para que a
igualdade ocorra.

PROBLEMA 3
Para n natural, seja (n) a quantidade de números naturais que são primos com n
e (n) o número de fatores primos de n. Mostre que se (n) é um divisor de n – 1
e (n)  3, então n é primo.

PROBLEMA 4
Para reais positivos satisfazendo a + b + c = abc, mostre que
1 1 1 3
   , e determine quando a igualdade ocorre.
1  a2 1  b2 1  c2 2

PROBLEMA 5
Seja I o incentro de ABC, O1 uma circunferência passando por B e tangente à
reta CI e O2 uma circunferência passando por C e tangente à reta BI. Mostre que a
circunferência circunscrita ao ABC e as circunferências O1 e O2 interceptam-se
em um único ponto.

PROBLEMA 6
Seja Fn o conjunto de todas as bijeções f de {1, …, n} em {1, …, n} satisfazendo
(i) f ( k ) k  1 para k = 1, 2, …n e (ii) f (k )  k para k = 2, …,
n.
Determine a probabilidade de que f (1)  1 para um f arbitrário em Fn
10a. OLIMPÍADA DE MATEMÁTICA DO CONE SUL
Primeiro teste de Seleção

PROBLEMA 1
Em um tabuleiro 1999  1999 encontra-se um certo número de torres (torre é uma
peça que se move horizontalmente ou verticalmente). Prove que é possível colorir
as torres utilizando três cores de modo que nenhuma torre ataque outra de mesma
cor (uma torre ataca outra quando ambas estão na mesma linha ou coluna sem
peças intermediárias).

EUREKA! N°6, 1999


43
Sociedade Brasileira de Matemática

PROBLEMA 2
Encontre todas as soluções reais de x [ x [ x [ x ]]] 88 , onde [x] é o inteiro
satisfazendo [ x]  x  x  1 (por exemplo, [3, 7] = 3, [4] = 4 e [–] = – 4.

PROBLEMA 3
A bissetriz do ângulo B em um triângulo ABC intercepta o lado AC no ponto D.
 
Seja E um ponto sobre o lado BC tal que 3C A E  2 B A E. Os segmentos BD
e AE interceptam-se no ponto P. Se ED = AD = AP, determine os ângulos do
triângulo.

PROBLEMA 4
Mostre que há infinitos naturais n tais que n 21 divide n!, onde
n! n ( n  1)...2 1 por exemplo, 4! 4 3 2 1  24).

PROBLEMA 5
Considere um polígono convexo com 2000 lados no plano. Prove que é possível
escolher 1998 pontos no plano tais que qualquer triângulo formado por vértices do
polígono tenha exatamente um dos pontos escolhidos em seu interior.

10a. OLIMPÍADA DE MATEMÁTICA DO CONE SUL


Segundo teste de Seleção

PROBLEMA 1
Sejam p, q, r, s inteiros não negativos tais que
( p  q ) 2  q ( r  s ) 2  s
Prove que p = r e q = s.

PROBLEMA 2

EUREKA! N°6, 1999


44
Sociedade Brasileira de Matemática

As mn casas de um tabuleiro m  n (m, n > 1 inteiros) são pintadas


alternadamente de branco e preto, como em um tabuleiro de xadrez. Em seguida,
colocamos em cada casa um número inteiro, de modo que a soma dos números de
cada linha e a soma dos números de cada coluna sejam pares. Prove que a soma
dos números escritos nas casas pretas é par.

PROBLEMA 3
Seja ABCD um paralelogramo, H o ortocentro do triângulo ABD e O o
circuncentro do triângulo BCD. Prove que os pontos H, O e C são colineares.

PROBLEMA 4
Determine todos os ternos (a, b, c) de inteiros positivos tais que a e b são pares e
a b  b a  2c.



Você sabia…
Que o record de maior primo de Fermat
n
generalizado (i. e., da forma a 2  1 ) conhecido
foi batido em 2/11/1999?
Este primo é 11402416384 + 1, tem 82854 dígitos
e foi descoberto por Kimmo Herranen, usando
um programa desenvolvido por Yves Gallot
(veja
http://perso.wanadoo.fr/yves.gallot/primes/gfn.html)

SOLUÇÕES DE PROBLEMAS PROPOSTOS


 Publicamos aqui algumas das respostas enviadas por nossos leitores.

10) Suponha que temos k moedas, todas iguais exceto por uma que tem peso
ligeiramente diferente das anteriores (não se sabe se maior ou menor), e uma
balança de dois pratos.

EUREKA! N°6, 1999


45
Sociedade Brasileira de Matemática

3n  3
a) Mostre que se k  é possível determinar com n pesagens qual
2
é a moeda diferente, e se ela é mais leve ou mais pesada que as
outras.
3n  1
b) Mostre que se k  é possível determinar com n pesagens qual
2
é a moeda diferente, mas nem sempre é possível dizer se ela é mais
leve ou mais pesada que as outras.
3n  1
c) Mostre que se k  não é sempre possível determinar qual é a
2
moeda diferente.

Solução do Editor:

Fato 1: Se tivermos 3n ou menos moedas com apenas uma diferente, e que seja
sabido se a moeda diferente é mais leve ou mais pesada então é possível
determinar a moeda diferente com apenas n pesagens.
Prova : Se n = 0 a afirmação é óbvia. Vamos provar o fato por indução. Se
tivermos 3 n 1 ou menos moedas de modo que uma delas é, digamos,
mais pesada que as outras, dividimos as moedas em três grupos, A, B e C,
de 3 n ou menos moedas cada, de modo que A e B tenham o mesmo
número de moedas. Pesamos o grupo A com o grupo B. Se os pesos
forem diferentes, o grupo mais pesado tem 3 n ou menos moedas e
contém a moeda diferente, e portanto podemos , com mais n pesagens
(por hipótese de indução), determiná-la. Se os pesos forem iguais a
moeda mais pesada estará no grupo C, e , como antes, podemos
determiná-la em mais n pesagens.

Corolário 1:
3n  1
Se tivermos k  moedas das quais uma é diferente, e tivermos um
2
 2k  1  n 1
grupo de pelo menos   3 moedas padrão então é possível
 3 
determinar a moeda diferente e se ela é mais leve ou mais pesada com n
pesagens.

EUREKA! N°6, 1999


46
Sociedade Brasileira de Matemática

 2k  1 
3 n  1 moedas dentre as
3n  1
Prova: Separamos um grupo A de   ,e
 3  2
 2k  1  n 1
pesamos com um grupo de 
3  3 moedas padrão. Se der
 
diferente saberemos que a moeda diferente está no grupo A e se ela é
mais leve ou mais pesada, e, usando o Fato 1, podemos determiná-la com
mais n – 1 pesagens.
Se der igual a moeda diferente está no grupo das
n
 2k  1  3  1 n 1 3 n 1
k     3  moedas restantes, o que prova o
 3  2 2
corolário por indução, dado que temos uma quantidade suficiente de
moedas padrão e, para n = 1, se tivermos uma moeda padrão é só pesá-la
com a moeda diferente para descobrir se esta é mais leve ou mais pesada.

Corolário 2:
3n  1
Se tivermos k  moedas das quais uma é diferente e tivermos pelo
2
 2k  1 
menos  3 n  1 moedas padrão então é possível determinar com
 3 
n pesagens a moeda diferente.

Prova: Para n = 1 temos 2 moedas. Pesamos uma delas com uma moeda padrão.
Se der diferente é ela a diferente, se der igual é a outra. O resto da prova
(por indução) é igual à do corolário 1.

3n  1
Fato 2: Se tivermos dois grupos A e B de k  moedas cada uma das quais
2
é diferente tais que o peso total de A é maior que o peso total de B e pelo
 2k  1

menos 2  9  2 3 n  2 moedas padrão então é possível determinar
 
com n pesagens a moeda diferente e se ela é mais leve ou mais pesada.

 2k  1  n 2
Prova: Seja r   3 . Pesamos de um lado 2r moedas do grupo A, r
 9 
do grupo B e do outro r moedas do grupo A, r do grupo B e 2r moedas
padrão. Se o primeiro lado tiver peso maior dividimos suas 2r moedas do
grupo A em dois grupos de r e os pesamos. Se der igual a moeda
diferente será mais leve e estará entre as r 3 n  2 moedas do grupo B

EUREKA! N°6, 1999


47
Sociedade Brasileira de Matemática

que estavam do outro lado. Se der diferente a moeda diferente será mais
pesada e estará no lado mais pesado nessa última pesagem. Nesses casos
o Fato 1 implica o resultado pois com mais n – 2 pesagens descobrimos a
moeda diferente. Se o primeiro lado tiver peso menor a análise é análoga,
trocando os papeis dos grupos A e B.
Finalmente, se der igual a moeda diferente estará nos grupos A'  A ou
n 1
 2k  1  3  1
B '  B de k  3   moedas que não foram pesadas, e
 9  2
o resultado segue por indução ( O caso n = 1 é trivial) 

Vamos agora resolver os ítens originais do problema:


3n  3
a) Se temos k moedas pesamos dois grupos de
2
n
 k  1 3  1
m    moedas cada. Se der diferente usamos o Fato 2, e se
 3  2
3 n 1  1
der igual usamos o Corolário 1 para as k  2m  moedas
2
restantes.

3n  1 3n 1  1
b) Se temos k  pesamos dois grupos de moedas cada. Se
2 2
der diferente usamos o Fato 2, e se der igual usamos o Corolário 2 para as
3n 1  1
moedas restantes.
2
Para ver que não é possível descobrir a moeda diferente e se ela é mais
leve ou mais pesada com n pesagens, observamos que se na primeira
3n 1  1
pesagem pesamos mais que moedas em cada prato e der
2
 3n  1  1 
2  3n  1  1  3n  1
diferente, teremos no mínimo  2  possibilidades
 
para a moeda diferente, e não podemos distingui-las com
n – 1 pesagens (que dão no máximo 3 n  1 possíveis resultados finais).
3n 1  1
Assim, devemos ter no máximo moedas em cada prato na
2

EUREKA! N°6, 1999


48
Sociedade Brasileira de Matemática

3n 1  1
primeira pesagem, e se der igual sobram pelo menos moedas.
2
O resultado seguirá do
3k  1
Lema: Se temos moedas das quais sabemos apenas que uma é diferente,
2
e algumas moedas padrão não podemos distinguir a moeda diferente e se
ela é mais leve ou mais pesada com k pesagens.

Prova: Pelo argumento acima, não podemos usar mais de 3 k  1 moedas


desconhecidas na primeira pesagem, e se esta der equilíbrio sobram pelo
3k  1  1
menos moedas, e o resultado segue por indução (o caso k = 0 é
2
trivial).
3n  1
c) Como antes não podemos usar mais que moedas em cada prato
2
3n 1  3
na primeira pesagem, e se esta der igual sobrarão pelo menos
2
moedas, caso em que não é possível determinar a moeda diferente com
mais n – 1 pesagens, o que pode ser provado do mesmo modo que o lema
anterior 
2
16) Seja l a reta {( x, y )  R y 0}, C1 o círculo centrado em
1 1 1 1
(0, ) de raio e C 2 o círculo centrado em (1, ) de raio .
2 2 2 2
Seja F o conjunto de círculos em R2 com as seguintes propriedades:

i) {C1, C2}  F
ii) Se C e C’ pertencem a F, são tangentes entre si e tangentes a l
~
então todo círculo C tangente aos dois círculos C e C’ e à reta l
pertence a F.
~
iii) Se F é um conjunto de círculos satisfazendo as propriedades i) e
~
ii) então F  F . Determine o conjunto dos pontos de tangência
dos círculos C  F com a reta l.

EUREKA! N°6, 1999


49
Sociedade Brasileira de Matemática

Solução de Zoroastro Azambuja Neto (Rio de Janeiro - RJ):

O conjunto dos pontos de tangência será o conjunto {( x,0), x  Q  [0,1]}.


p
Cada racional x  [0,1] será representado por q onde p é inteiro, q é inteiro
positivo e mdc( p, q ) 1. Para provar isso mostraremos os seguintes fatos por
indução:
p 1
i) O círculo tangente em ( q , 0) terá raio .
2q 2
p r
ii) Se os círculos tangentes em q e são tangentes entre si então
s
ps  qr 1
Para isso, notemos que se dois círculos centrados em (x, r1) e (y, r2) são tangentes
à reta l e tangentes entre si então
( r1  r2 ) 2  d 2 (r1  r2 ) 2  d 2 4r1 r2  d 2 r1 r2 ,
onde d  x y.
r1+ r2

r1 r2

As afirmações i) e ii) são verdadeiras para os círculos iniciais C1 e C2. Se o


 p 1 
círculo C é tangente a l e tem centro  , 2  , o círculo C ' é tangente a l e tem
 q 2q 
r 1  ~
centro  , 2  e qr – ps = 1 então, se o círculo C tangente a C e C ' e à reta l
s
 2s 
p r p r
tem centro (x, y) com q  x  s então, se d ' = x  q e d ' '   x,
s
devemos
2 y 2 y
ter d '  e d "  , e
q 2 s 2
r p 1
d ' d "    ,
s q qs

EUREKA! N°6, 1999


50
Sociedade Brasileira de Matemática

2( q  s ) y 1 1
donde   y e
qs 2 qs 2( q  s) 2
1 p p(q  s)  1
d'  x   d' 
q(q  s) q q(q  s)
pr ~  pr 
= q  s (pois ps = qr – 1). Assim, C é tangente em  ,0  e tem raio
 qs 
1
.
2(q  s ) 2
Como q ( p  r )  p( q  s ) qr  ps 1 e (q  s )r  ( p  r ) s qr  ps 1
~
vemos que C satisfaz i) e ii).
Esses fatos implicam que todos os círculos criados terão centro em pontos
p  p 
racionais. Basta provar agora que para todo racional q  [0,1], o ponto  , 0 
q 
é ponto de tangência de algum dos círculos. Faremos isto por indução em q (para
q = 1 o resultado é óbvio): basta mostrar se mdc(p, q) = 1 e q  2 que e possível
escrever p = p' + p" e q = q' + q" com p', p", q', q" inteiros, q', p"  0 e
q'p"– p'q" = 1.
Estas equações podem ser escritas como q ' ( p  p ' )  p ' ( q  q ' ) 1, ou seja,
q ' p  p ' q 1, onde 0  q '  q e 0  p '  p. Como mdc(p, q) = 1 existem x, y
 Z com px  qy 1, e teremos para todo k  Z, p ( x  kq)  q ( y  kp) 1.
Certamente podemos escolher k de modo que 0  x  kq  q (note que x não é
múltiplo de q, senão 1  px  qy também seria), e então tomamos q ' x  kq e
pq ' 1
p ' kp  y (temos p '  , mas 1 q '  q, donde 0  p '  p ).
q

17) Dado n  N, uma partição  de n é uma lista ordenada


 (a1 , a2 ,...,ar ) , r , a1 , a2 ,..., ar  N * com a1  a2 ...  ar e a  a  ...  a  n.
1 2 r

Seja Pn o conjunto das partições de n. Para   Pn , definimos A() como o


número de termos iguais a 1 em  ( ou seja , A( ) 
 {i  {1,2,..., r} a i 1}), e B() como o número de termos distintos na
partição  (ou seja, B () =  {a1, a2, …, ar}).
Prove que  A( )   B ( ) para todo n  N.
 Pn  Pn

Solução de Zoroastro Azambuja Neto (Rio de Janeiro - RJ):

EUREKA! N°6, 1999


51
Sociedade Brasileira de Matemática

Provaremos o resultado por indução em n. Para n = 1 o resultado é óbvio, pois a


~
única partição de 1 é (1). As partições   ( a ,..., a )
1 n
de n + 1 com a1 1
~
(isto é, com A( )  0) são obtidas das partições  ( a2 ,..., ar ) de n
acrescentando um 1 na primeira posição. Esta operação acrescenta 1 ao A() e
~
acrescenta 1 ao B() caso A() = 0, e 0 caso contrário (ou seja, caso A( )  2).
~ ~ ~ ~ ~

Admitindo o resultado para n, o resultado para n + 1 segue então de ~


 ~ ~

A )(  B )( #{ partiçãoden1 A )(| 2} B().
Pn1 Pn1 Pn1
~
A )( 0
O lado direito é igual a 0, pois, considerando
~ ~ ~ ~ ~
f {  Pn 1 | A( ) 2}  {  Pn 1 | A( ) 0}, tal que se  (a1 , a 2 ,...a r )
~
~
então os elementos da partição f ( ) são A( ), a ~ ,..., a r (somamos
A ( ) 1
~ ~
todos os A( ) 2 termos de  iguais a 1), é fácil ver que f é sobrejetiva, e ,
dado
~ ~ ~ ~ ~
  Pn 1 com A( ) 0, # { ' Pn 1 | A( ) 2 e f ( ' )  } B( ),
(escolheremos um elemento de  e o decompomos como soma de uns) donde
~ ~ ~

#{  Pn1 | A( ) 2}  B( ).
~
Pn1
~
A( )0

20) Diga se existe uma função polinomial de R2 em R cuja imagem seja


o intervalo (0,) {x  R x  0}.

Solução de Zoroastro Azambuja Neto (Rio de Janeiro - RJ):

Considere a função f ( x, y ) x 2  ( xy  1) 2

EUREKA! N°6, 1999


52
Sociedade Brasileira de Matemática

É fácil ver que f ( x, y ) 0, ( x, y )  R 2 , e se f ( x ) 0 então x = 0 e xy – 1 =


 1 
0, o que é impossível. Dado a > 0, f  a ,  a, donde Im (f) = (0, + ).
 a

24) Na loteria de Truchilândia, cada bilhete tem um número de três algarismos


que usa somente os algarismos 1, 2, 3, 4 (é permitido repetir os dígitos). Um
bilhete é ganhador se coincide em pelo menos duas posições com o número
sorteado.
Um apostador quer compar vários bilhetes, de maneira que um deles ganhe
com certeza, mas gastando o mínimo possível. Determinar quantos bilhetes
deve comprar e quais bilhetes deve comprar.

Obs. Se o bilhete sorteado for o 423 então 123 é um bilhete ganhador, mas
243 não é.

Solução de Flávio Wagner Rodrigues (São Paulo - SP):

1. Vamos observar inicialmente que o sorteio dessa loteria admite 64


resultados possíveis.
2. Com um único bilhete o apostador tem 10 resultados favoráveis. De fato,
para cada par de posições 1 e 2, 1 e 3 e 2 e 3 existem quatro bilhetes
favoráveis o que nos dá um total de 12 mas, o número de seu bilhete é
contado 3 vezes e portanto são apenas 10 resultados favoráveis distintos.

3. Segue-se de 1 e 2 que 6 bilhetes não são suficientes para garantir a vitória


do apostador. .Vamos mostrar agora que 7 bilhetes também não são
suficientes. Como são quatro algarismos e 7 jogos existe pelo menos um
algarismo que aparece uma única vez na primeira posição. Vamos supor
que existe um único jogo que tenha o 1 na primeira posição. Esse jogo
garante a vitória do jogador em 7 jogos começados com 1. Como existem
16 jogos começados por 1, a vitória nos outros 9 deve ser garantida pelos
elementos que aparecem nas posições 2 e 3 dos demais jogos. Como
existem apenas 6 outros jogos existirão 3 jogos começados por 1 que se
forem sorteados não darão vitória ao jogador.

4. Existem vários conjuntos de oito jogos que resolvem o problema. Vamos


dar dois exemplos e mostrar em seguida como esses conjuntos podem ser
construídos.

EUREKA! N°6, 1999


53
Sociedade Brasileira de Matemática

5. Exemplo 1 : 124, 131, 212, 313, 342, 243, 421, 434


Exemplo 2 : 123, 111, 232, 334, 342, 244, 413, 421
Exemplo 3 : 111, 122, 212, 221, 333, 344, 434, 443

Roteiro para construir um conjunto de oito jogos que resolve o problema.

Inicialmente vamos observar que o conjunto deverá satisfazer duas condições


importantes:

1- Cada algarismo deve aparecer em cada posição exatamente duas


vezes.
2- É necessário que existam jogos que contenham elementos repetidos
como 11, 22, 33, 44 não importando em que posições.

1- Escolha dois jogos que tenham o 1 na primeira posição e algarismos


distintos nas outras posições. Por exemplo 1 3 4 e 1 4 2. Não é difícil
mostrar que dos 16 jogos que começam por 1 esses dois jogos garantem a
vitória do apostador em 12. Restam 4 jogos para os quais será necessário
que a vitória seja garantida pelas posições 2 e 3 dos demais jogos. No
nosso exemplo, vamos necessitar de jogos terminando em 11, 13, 21 e 23

2- A primeira observação acima e a conveniência de não repetir ligações nos


permitem dizer que os dois jogos que faltam devem terminar por 32 e 44.
A distribuição dos algarismos que devem ocupar as primeiras posições
deve ser feita levando em conta as condições iniciais evitando sempre
que possível a repetição de ligações. Uma possibilidade seria :

134, 142, 211, 223, 313, 321, 432, 444

26) Sejam as funções fo (x) = xn e fi (x) = fi – 1(x + 1) – fi –1 (x) onde x, n e i são


inteiros positivos. Prove que, para todo x, fn(x) = n!

Solução de Bruno Fernandes Cerqueira Leite (São Paulo - SP):

Antes de começar, é importante deduzir (pelo Binômio de Newton) que:

EUREKA! N°6, 1999


54
Sociedade Brasileira de Matemática

a a a 1  a  a 2
(x 1)  x a x   x  . . (isto será muito usado na demonstração).

 2
Agora temos f 0 ( x) x n e f i 1 ( x)  f i ( x  1)  f i ( x)

n n n 1  n  n  2 f 1 ( x)
f1(x)  f0 (x 1)  f0 (x) (x 1)  x n x   x . . 
 2 n
x n  1  a1 x n  2  a 2 x n  3  ..., onde a1, a2, …, etc são coeficientes racionais (que
não nos interessam).

f 2 ( x ) f 1 ( x  1) f 1 ( x)
f 2 ( x )  f 1 ( x  1)  f 1 ( x ) 
n

n

n
 
 ( x  1) n  1  a1 ( x  1) n  2  ... 
f ( x)

 x n  1  a1 x n  2     
 ...  ( x  1) n  1  x n  1  a1 ( x  1) n  2  x n  2  ...  2
n

 n 2  n  1  n 3    n  2  n 4 
(n  1) x  x  ...  a1  (n  2) x n  3   x  ...  ...
  2     2  

f 2 ( x)
  x n  2  b1 x n  3  b2 x n  4  ..., onde b1, b2, …, etc, são coeficientes
n(n  1)
racionais (cuja determinação não nos interessa).
Continuando com esses procedimentos, podemos chegar a expressões análogas
para f3(x), f4(x), etc.
A semelhança entre essas expressões nos leva a "desconfiar"que

EUREKA! N°6, 1999


55
Sociedade Brasileira de Matemática

f b ( x)
 x n  b   1 x n  b  1   2 x n  b  2  ..., onde
An,b

 B j , j 1,2, etc; são coeficientes (não nos interssam)



n!
 An,b n(n  1)(n  2)... 
 (n  b)!
        
 b fatores

Vamos provar isto por indução. Suponha que a fórmula valha para f1(x), …,
fb–1(x). Teremos:
f ( x)  f b  1 ( x  1)   f b  1 ( x) 
f b ( x)  f b  1 ( x  1)  f b  1 ( x)  b     e por
An ,b  1  An,b  1   An ,b  1 
hipótese de indução,
f b ( x)
An ,b  1
    
 ( x  1) n  b 1   1 ( x  1) n  b  ...  x n  b 1   1 x n  b  ...  ( x  1) n  b 1  x n  b 1 

nb nb nb nb1 nb1   nb1 nb nb2 



1(x 1)  x .(. n 1)xb   x . 1n b)(. x  x .
2      2  
fb ( x) f b ( x) f b ( x)
 ... 
An, b  1

(n  b  1) x n  b  1 x n  b  1   2 x n  b  2  ...  
An ,b  1 (n  b  1)

n!
(n  b 
(n  b  1)!

EUREKA! N°6, 1999


56
Sociedade Brasileira de Matemática

fb ( x) f ( x)
  x n  b  1 x n  b  1   2 x n  b  2  ...  b  x n  b  1 x n  b  1   2 x n  b  2  ...,
n! An,b
( n  b)!
confirmando o que foi "desconfiado".
f n ( x)
Agora vamos usar a fórmula para  x n  n 1  f n ( x)  An , n  n !
An , n

27) O triângulo equilátero ABC possui um ponto interno P tal que em P chegam
três segmentos de reta (PA, PB, PC) onde PA = 6, PB = 8 e PC = 7. Com
esses dados descubra qual é a área do triângulo.

B C

Solução de Ricardo Klein Hoffmann (Porto Alegre - RS):


A

P
8 7

B C
8 6
D

1. Seja BCD o triângulo congruente ao triângulo APB onde:

EUREKA! N°6, 1999


57
Sociedade Brasileira de Matemática

 ___ ___
 PB BD 8
 ___ ___
 AP CD 6
 ___ ___
 AB BC x

 
2. Temos que A B C P B D 60 . Então, o PBD é equilátero com
___
PD 8.

3. Seja C P D 
4.  (PDC )  Lei dos cossenos
11 135
6 2 7 2  8 2  2 7 8 cos   cos   daqui, sen 
16 16
5.  (BPC )  Lei dos cossenos
149  63 5
x 2 8 2  7 2  2 8 7 cos(  60 )  x 2 
2
2
x 3
6. Área  ( ABC )  S
4
3  149  63 5 
Então, S   

4  2 
3
Portanto, a área do  ( ABC )  (149  63 5 ).
8

28) Seja n  2 um número inteiro. Prove que n e n + 2 são ambos primos se e

4((n  1)!  1)  n
somente se é inteiro.
n( n  2)

Solução de Marcelo Rufino de Oliveira (São José dos Campos - SP):

Antes de iniciar a resolução, vou enunciar e provar os seguintes Teoremas, que


serão de grande utilidade:

1o Teorema (Teorema de Wilson):

EUREKA! N°6, 1999


58
Sociedade Brasileira de Matemática

“Se p é um número primo, então o número (p – 1)! + 1 é divisível por p.”


Prova: Consideremos a congruência linear: ax  1 (mod p) onde a é qualquer dos
p – 1 inteiros positivos da seqüência
1, 2, 3, …, p – 1 (1)
Então, o mdc (a, p) = 1 e é sabido que esta congruência admite uma única solução
módulo p, isto é, existe um único inteiro a1, com 1  a1  p – 1, tal que aa1  1
(mod p)
Se p é primo, temos que a = a1 se e somente se a = 1 ou a = p – 1.
De fato, a congruência quadrática a2  1 (mod p) é equivalente à seguinte
(a – 1)(a + 1)  0 (mod p) de modo que p | (a – 1) ou p | (a + 1), o que implica
que:
a – 1  0 (mod p) e a = 1 ou a + 1  0 (mod p) e a = p – 1
Então para cada a distinto da seqüência (1) existe somente um a1 correspondente
na seqüência (1) que satisfaz aa1  1 (mod p). Omitindo os pares 1 e p – 1, com
os p – 3 inteiros restantes: 2, 3, …, p – 3, podemos formar
(p – 3)/2 pares a, a1, com a  a1, e tais que aa1  1 (mod p). Multiplicando todas
essas (p – 3)/2 congruências, obtemos:
2.3.4…(p – 3)(p – 2)  1 (mod p) => (p – 2)!  1 (mod p)
Multiplicando por p – 1: (p – 1)!  p – 1 (mod p) => (p – 1)!  – 1 (mod p)

2o Teorema:
“Uma condição necessária e suficiente para que um número natural p > 1 seja
primo é que o número (p – 1)! + 1 seja divisível por p.”
Prova: Suponhamos que p não seja primo. Então existe um divisor q de p tal que
1 < q < p. O número (p – 1)! + 1 sendo divisível por p, também deve ser divisível
por q. Desde que q < p, então q  p – 1, implicando que q divide algum inteiro
positivo entre 2 e p – 1. Assim q divide (p – 1)!. Como q divide (p – 1)! + 1 então
q também divide 1, que é uma contradição, pois 1 < q < p.

1a parte (ida)
I) Suponhamos que n e n + 2 são números primos.
De acordo com o Teorema de Wilson nós temos que (n – 1)! + 1 é divisível por n
e (n + 1)! + 1 é divisível por n + 2.
Como n divide 4((n – 1)! + 1) e, evidentemente, n também divide n, então divide
a sua soma, ou seja, 4((n – 1)! + 1) + n é divisível por n.
Basta analisar agora se 4((n – 1)! + 1) + n também é divisível por n + 2.
x = 4((n – 1)! + 1) + n = 4((n – 1)! + 1) + n + 2.(n + 1)! – 2.(n + 1)! 
x = 2.(n + 1)! + n + 4 – 2[(n + 1)! – 2.(n – 1)!] 

EUREKA! N°6, 1999


59
Sociedade Brasileira de Matemática

x = 2[(n + 1)! + 1] + (n + 2) + [(n – 1)!n(n + 1) – 2.(n – 1)!] 


x = 2[(n + 1)! + 1] + (n + 2) + (n – 1)![n2 + n – 2] 
x = 2[(n + 1)! + 1] + (n + 2) + (n – 1)!(n + 2)(n – 1)
Como n + 2 divide tanto 2[(n + 1)! + 1], como (n + 2) e também
(n – 1)!(n + 2)(n – 1), então divide a sua soma, que é x = 4((n – 1)! + 1) + n.
Como n e n + 2 são ambos primos e dividem 4((n – 1)! + 1) + n, então o seu
produto também divide 4((n – 1)! + 1) + n.

2a parte (volta)
II) Suponhamos agora que para um número natural n > 1, o valor
4((n – 1)! + 1) + n é divisível pelo produto n(n + 2).
Se n for par, isto é, se n = 2k, onde k é um número natural, então n – 1  k.
Isto implica que (n – 1)! é divisível por k e também que 4.(n – 1)! é divisível por
2k.
Consequentemente 4.(n – 1)! é divisível por n, e como
4((n – 1)! + 1) + n = 4.(n – 1)! + n + 4 é divisível por n, então 4 é divisível por n.
Como n é par então n = 2 ou n = 4.
Entretanto é fácil verificar que 4((n – 1)! + 1) + n não é divisível por n(n + 2)
caso n = 2 ou n = 4.
Assim, o fato de que 4((n – 1)! + 1) + n é divisível por n implica que
(n – 1)! + 1 é divisível por n. Pelo 2o Teorema temos que n é um número primo.
Fazendo as mesmas operações da 1a parte da solução:
4((n – 1)! + 1) + n = 2[(n + 1)! + 1] + (n + 2) + (n – 1)!(n + 2)(n – 1)
Como 4((n – 1)! + 1) + n, n + 2 e (n – 1)!(n + 2)(n – 1) são todos termos
divisíveis por n + 2, então (n + 1)! + 1 também é divisível por n + 2.
Pelo 2o Teorema temos que n + 2 também é um número primo.

Agradecemos também o envio das soluções a: Carlos Alberto da Silva Victor (Rio
de Janeiro - RJ), Tertuliano Franco (Salvador - BA), Manuel João de Jesus Almeida -
Rio de Janeiro - RJ) e Daniel Pessôa Martins Cunha (Fortaleza - CE). Seguimos
esperando a solução do problema 29.
O prazo para recebimento das soluções do problema "Cuático" publicado na revista
EUREKA! No. 5 (que concorre a prêmio!) foi prorrogado até dia 15 de fevereiro.

Problema "Cuático"
Prove que para qualquer conjunto de inteiros positivos A e para todo inteiro
positivo k existe um conjunto infinito de números primos S tal que o produto de k

EUREKA! N°6, 1999


60
Sociedade Brasileira de Matemática

elementos distintos de S está sempre em A ou o produto de k elementos distintos


de S nunca pertence a A.

Errata:
No primeiro teste de seleção para a 40a. IMO e a 14a. OIM, publicado na
EUREKA! No. 5, pág 50, no problema 4, onde diz:
"Por exemplo, min{3, 4} = 4…" deve dizer "Por exemplo, min {3, 4} = 3…"

PROBLEMAS PROPOSTOS

 Convidamos o leitor a enviar soluções dos problemas propostos e sugestões de novos


problemas para os próximos números.

30) Determine todas as funções f : R  R que satisfazem as condições:


 1  f ( x)
(i) f(– x) = – f(x), (ii) f(x + 1) = f(x) + 1, (iii) f    2 para x  0.
 x x
31) Seja x1, x2, x3, … uma seqüência de números reais não negativos satisfazendo
x x
x n  n  2 n  1 para n = 3, 4, 5, … Estabeleça condições necessárias e
2 x n 2  x n 1
suficientes em x1 e x2 para xn ser inteiro para infinitos valores de n.

32) a) Prove que todo número inteiro não nulo m admite uma única representação
n 1
da forma m  
k 0
k 3 k , onde n é um inteiro positivo e  k  { 1,0,1} para

todo k, com  n  1 0.


3n  1 V {P0 , P1 ,..., P3n  1 }
b) Dado um conjunto de
pontos ,
2 2

escrevemos em cada aresta que une dois desses pontos Pi e Pj (i  j) um


número pertencente a {0, 1, …, n – 1} da seguinte forma: escreveremos
n 1
i j  
k 0
k 3 k , com  k  { 1,0,1}, k , e associamos à aresta Pi

Pj o número m min{k 0  k 1}.

EUREKA! N°6, 1999


61
Sociedade Brasileira de Matemática

Prove que não existe nenhum triângulo cujos vértices pertençam a V com
o mesmo número escrito em seus três lados.

33) Na parede interna de um vaso cilíndrico de cristal existe uma gota de mel
num ponto B situado a três centímetros do seu bordo superior. Na parede
externa, num ponto A diametralmente oposto ao da gota, está uma formiga.
Sabendo que a altura do vaso é de 20cm e o seu diâmetro é 10 cm. Indicar o
caminho mais curto para que a formiga atinja a gota de mel.

34) ABC é um triângulo, tal que AB = c, AC = b e BC = a.


Por um ponto interior P deste triângulo, são traçadas paralelas aos seus lados.
Sabe-se que as intersecções, da paralela ao lado de medida a, com os lados
deste triângulo, determinam um segmento de medida a'.
Analogamente, as paralelas aos lados de medidas b e c, determinam com os
lados do triângulo, segmentos de medidas b' e c' respectivamente.
a ' b' c '
Nestas condições demonstre que   2.
a b c

35) Sabendo que num triângulo ABC a altura relativa ao vértice A mede 12cm. e a
altura relativa ao vértice B mede 20cm., determine todos os valores possíveis
para a altura relativa ao vértice C.

Problemas 30 e 31 propostos por Marcelo Rufino de Oliveira (São José dos Campos
- SP), problema 33 proposto por Gleydson Chaves Ricarte (Fortaleza - CE),
problema 34 proposto por Paulo Jacob (Lista de discussão de problemas),
problema 35 proposto por Carlos Alexandre Gomes da Silva (Natal - RN).

EUREKA! N°6, 1999


62
Sociedade Brasileira de Matemática

Você sabia…
Que ninguém sabe se existem infinitos primos p tais que
p + 2 também seja primo (mas todo mundo acredita nisso)?
Um para de primos p, p + 2 é chamado um par de primos
gêmeos.
Os maiores primos gêmeos conhecidos são
361700055  239020  1, que têm 11755 dígitos e foram
descobertos este ano por Henri Lifchitz.
Também não se sabe se todo número par maior ou igual a 4
pode ser escrito como soma de dois primos
(esta é a conjectura de Golbach).

ASSINATURA DA REVISTA EUREKA! 2000

Se você deseja seguir recebendo na sua casa a revista EUREKA!, faça o


seu pedido escrevendo para: Secretaria da Olimpíada Brasileira de
Matemática, Estrada Dona Castorina, 110 Jardim Botânico - Rio de Janeiro, RJ -
CEP: 22460-320. Você pode fazer uma assinatura anual o que dará direito a
receber as publicações do ano 2000 (mínimo 3 exemplares) por um valor de
R$10,00. Para isso, faça um depósito no Banco do Brasil - Agência 0598-3 Conta
N52208-2 em nome do professor Eduardo Wagner. Envie-nos a fotocopia da
ficha do verso preenchida e uma cópia do recibo do depósito. Nós remeteremos
a(s) revista(s) pelo correio. Pedidos podem ser feitos também por e-mail e
comprovantes de depósito poderão ser enviados pelo fax.
Se tiver qualquer dúvida entre em contato conosco.
Telefone: 21-5295077 / Fax: 21-5295023
e-mail: obm@impa.br
Home-Page: http//www.obm.org.br/

LISTA DE DISCUSSÃO DE
PROBLEMAS
DE MATEMÁTICA OLÍMPICA
Está aberta uma lista de discussão de
problemas de Matemática Olímpica.

EUREKA! N°6, 1999


63
Sociedade Brasileira de Matemática

A lista é aberta à todos.


O endereço da lista é:
Obm-l@mat.puc-rio.br
Para assinar, envie uma mensagem para:
Majordomo@mat.puc-rio.br
Com texto
Subscribe obm-l
    
Quem assim proceder deve receber um pedido de confirmação auto-explicativo
(automaticamente gerado pelo majordomo em inglês).
Em caso de problemas, escreva para o Professor Nicolau Saldanha (administrador
desta lista): e-mail: nicolau@mat.puc-rio.br
Desta lista estão participando muitos alunos e professores e nela são discutidos
problemas e aspectos teóricos de matemática. Trata-se de um meio informal e
eficiente de preparação para as olimpíadas.

ASSINATURA 2000 DA REVISTA EUREKA!


Atenção! Leia atentamente as instruções da página anterior.
Por favor preencha esta ficha com letra de forma.

Nome:

Endereço:
Bairro:
Cidade: Estado:
Cep:
Telefone: ( )
Fax: ( )
e-mail:

Números atrasados podem ser comprados utilizando a mesma forma de


pagamento da assinatura, sendo os preços:

 Números 1, 2 e 3 R$10,00
 Números 4, 5 e 6 R$10,00
 Números por separado R$4,00 (cada)

EUREKA! N°6, 1999


64
Sociedade Brasileira de Matemática

Você sabia… que ninguém sabe


se para todo n  N existe k  N tal que
a k (n) 1 , onde a1 (n) n e

 n / 2, se n é par
ak 1 (n)  ?
 3n  1, se n é ímpar
(Isto é conhecido como o problema
3 x  1, e a resposta é afirmativa para
todo n 2 40 )

COORDENADORES REGIONAIS

Amarisio da Silva Araújo (UFV) Viçosa - MG


Alberto Hassen Raad (UFJF) Juiz de Fora - MG
Antônio C. Rodrigues Monteiro (UFPE) Recife - PE
Angela Camargo (Centro de Educação
de Adultos - CEA) Blumenau - SC
Benedito T. Vasconcelos Freire (UFRN) Natal - RN
Claudio Arconcher (Col. Leonardo da Vinci) Jundiaí - SP
Clauss Haetinger (UNIVATES) Lajeado - RS
Crescêncio das Neves (UFAM) Manaus-AM
Élio Mega (Col. ETAPA) São Paulo - SP
Enzo Marcom Takara (Col. Singular) Santo André - SP
Flávia Jerônimo Barbosa (UFPB Campus I) João Pessoa - PB
Florêncio F. Guimarães Filho (UFES) Vitória - ES
Francisco Dutenhefner (UFMG) Belo Horizonte - MG
Gisele de A. Prateado Gusmão (UFGO) Goiânia - GO
Ivanilde H. Fernandes Saad (U. Católica Dom Bosco) Campo Grande - MS
João Benício de Melo Neto (UFPI) Teresina - PI
João F. Melo Libonati (Grupo Educ. IDEAL) Belém - PA
Jorge Ferreira (UEM) Maringá - PR
José Carlos Pinto Leivas (UFRG) Rio Grande - RS

EUREKA! N°6, 1999


65
Sociedade Brasileira de Matemática

José Cloves Saraiva (UFMA) São Luis - MA


José Gaspar Ruas Filho (ICMC-USP) São Carlos - SP
José Luis Rosas Pinho (UFSC) Florianópolis - SC
José Paulo Carneiro (Univ. Santa Úrsula) Rio de Janeiro - RJ
José Vieira Alves (UFPB) Campina Grande - PB
Leonardo Matteo D'orio (Sistema Titular de Ensino)Belém - PA
Licio Hernandes Bezerra (UFSC) Florianópolis - SC
Luzinalva M. de Amorim (UFBA) Salvador - BA
Marcondes Cavalcante França (UF Ceará) Fortaleza - CE
Pablo Rodrigo Ganassim (L. Albert Einstein) Piracicaba - SP
Paulo H. Cruz Neiva de L. Jr. (Esc. Tec.Everardo Passos) SJ dos Campos - SP
Reinaldo Gen Ichiro Arakaki (INPE) SJ dos Campos - SP
Ricardo Amorim (Centro Educ. Logos) Nova Iguaçu - RJ
Roberto Vizeu Barros (Colégio ACAE) Volta Redonda - RJ
Sergio Claudio Ramos (IM-UFRGS) Porto Alegre - RS
Seme Gebara Neto (UFMG) Belo Horizonte - MG
Tadeu Ferreira Gomes (U. do Estado da Bahia) Juazeiro - BA
Tomás Menéndez Rodrigues (U. Federal de Rondonia) Porto Velho - RO
Valdenberg Araújo da Silva (U. Federal de Sergipe) São Cristovão - SE
Wagner Pereira Lopes (Esc. Tec. Fed. de Goiás) Jataí - GO
Waldemar M. Canalli (P.M. S. João de Meriti) S. João de Meriti - RJ

EUREKA! N°6, 1999


66
CONTEÚDO

AOS LEITORES 2

XXI OLIMPÍADA BRASILEIRA DE MATEMÁTICA 3


Problemas e soluções da Primeira Fase

XXI OLIMPÍADA BRASILEIRA DE MATEMÁTICA 13


Problemas e soluções da Segunda Fase

XXI OLIMPÍADA BRASILEIRA DE MATEMÁTICA 21


Problemas e soluções da Terceira Fase

XXI OLIMPÍADA BRASILEIRA DE MATEMÁTICA 36


Resultados

ARTIGOS

EQUAÇÕES DIOFANTINAS 39
Antonio Caminha Muniz Neto

SOLUÇÕES DE PROBLEMAS PROPOSTOS 49

PROBLEMAS PROPOSTOS 59

AGENDA OLÍMPICA 61

COORDENADORES REGIONAIS 62
Sociedade Brasileira de Matemática

AOS LEITORES
Realizamos durante 1999 a XXI Olimpíada Brasileira de Matemática em
mais de 2.500 colégios de nosso país, atingindo na realização da primeira etapa
cerca de 60.000 alunos. Este ano a Olimpíada se realizará nas seguintes datas:

Primeira Fase – Sábado, 10 de junho


Segunda Fase – Sábado, 02 de setembro
Terceira Fase – Sábado, 21 de outubro (níveis 1,2 e 3)
Domingo, 22 de outubro (nível 3 - segundo dia).

A Comissão Nacional de Olimpíadas entende que todo aluno que desejar


participar da OBM deve poder fazê-lo sem restrições. A comissão oferece
inclusive a alunos de escolas que não participam da OBM a possibilidade de fazer
as provas sob supervisão direta do Coordenador Regional. As escolas podem
naturalmente aconselhar seus alunos a participar ou não da Olimpíada de acordo
com seus próprios critérios, mas a escola nunca deve impedir um aluno de
participar se este for o seu desejo.

Lembramos que a Olimpíada Brasileira de Matemática é uma competição


entre alunos e não entre colégios. A OBM divulga apenas os nomes e pontuações
dos alunos premiados; a OBM nunca divulgou nem divulgará comparações entre
colégios. Nosso objetivo é estimular o estudo de Matemática entre os jovens,
contribuir para o aprimoramento dos professores e propiciar uma melhoria do
ensino e do aprendizado desta matéria nas escolas brasileiras e não comparar
desempenhos de escolas.
O Regulamento da OBM foi atualizado. Leia o novo regulamento no site:
http://www.obm.org.br/regulamento.htm 

Finalmente, aproveitamos, para registrar a realização da Semana


Olímpica 2000, atividade que vem sendo realizada desde 1998. Nesta
oportunidade o evento teve lugar na Universidade Metodista de Piracicaba
(UNIMEP) entre os dias 21 a 27 de janeiro de 2000. Durante a Semana Olímpica
2000, reunimos alunos ganhadores da XXI Olímpiada Brasileira de Matemática
nos seus três níveis de competição. Estes alunos participaram de um treinamento
intensivo com professores de diversas partes do país como preparação para a
futura formação das equipes que representarão o Brasil em Olimpíadas
Internacionais. Além disso eles tiveram a oportunidade de conquistar novas

EUREKA! N°7, 2000


2
Sociedade Brasileira de Matemática

amizades, iniciando um relacionamento extremamente proveitoso com outros


jovens da mesma faixa de idade e com interesses semelhantes.
XXI OLIMPÍADA BRASILEIRA DE MATEMÁTICA
Primeira Fase - Nível 1

01. Um pequeno caminhão pode carregar 50 sacos de areia ou 400 tijolos. Se


foram colocados no caminhão 32 sacos de areia, quantos tijolos pode ainda ele
carregar?
A) 132 B) 144 C) 146 D) 148 E) 152

02. A calculadora de Juliana é bem diferente. Ela tem uma tecla D, que duplica o
número escrito no visor e a tecla T, que apaga o algarismo das unidades do
número escrito no visor.Assim, por exemplo, se estiver escrito 123 no visor e
apertarmos D, teremos 246; depois, apertando T, teremos 24. Suponha que esteja
escrito 1999. Se apertamos D depois T, em seguida D, depois T, teremos o
número:
A) 96 B) 98 C) 123 D) 79 E) 99

3. O gráfico abaixo mostra o valor aproximado do dólar em reais no dia 15 dos


últimos 6 meses.

2 ,0

1 ,5

1 ,0

D J F M A M J

Marcelo comprou um carro usando um sistema de financiamento chamado


leasing corrigido pela variação do dólar e suas prestações vencem exatamente
no dia 15 de cada mês. Em dezembro, Marcelo pagou R$ 600,00 de prestação.
Com base na tabela, podemos dizer que em maio a prestação foi de:
A) R$ 700,00 B) R$ 850,00 C) R$ 650,00 D) R$ 900,00
E) R$ 800,00

4. Numa certa cidade, o metrô tem todas suas 12 estações em linha reta. A
distância entre duas estações vizinhas é sempre a mesma. Sabe-se que a distância
entre a terceira e a sexta estações é igual a 3 300 metros. Qual é o comprimento
dessa linha?

EUREKA! N°7, 2000


3
Sociedade Brasileira de Matemática

A) 8,4 km B) 12,1 km C) 9,9 km D) 13,2 km E) 9,075 km

5. A metade do número 211  4 8 é igual a:


A) 2 5  4 4 B) 2 5  2 8 C) 110  2 8 D) 215  4 5 E) 2 9  4 7

6. Quantos números de dois algarismos são primos e têm como antecessor um


quadrado perfeito ?
A) 2 B) nenhum C) 1 D) 3 E) 6

07. Quantas vezes num dia (24 horas) os ponteiros de um relógio formam ângulo
reto ?
A) 48 B) 44 C) 24 D) 22 E) 23

8. Dona Zizi comprou 2 balas para cada aluno de uma 5 a série. Mas como os
meninos andavam meio barulhentos, ela resolveu redistribuir essas balas, dando 5
para cada menina e apenas 1 para cada menino. Podemos concluir que na 5 a série
A) 20% são meninos B) 30% são meninas C) 75% são meninos
D) 50% são meninas E) 66,6...% são meninos

9. Vários caixotes cúbicos de plástico


Azul ficaram armazenados ao ar livre, na
posição indicada na figura ao lado, na qual
apenas um dos caixotes não é visível. Com
o tempo, o plástico exposto ao ar perdeu
sua cor, tornando-se cinza. Ao desfazer a
pilha, verificaremos que o número de
caixotes com três faces azuis e três
cinzentas será:
A) 4 B) 5 C) 3 D) 2
E) 1

10. Ronaldo, sempre que pode, guarda moedas de 50 centavos ou 1 real.


Atualmente, ele tem 100 moedas, num total de 76 reais. Quantas moedas de um
valor ele tem a mais do que a de outro valor ?
A) 48 B) 4 C) 8 D) 52 E) 96

EUREKA! N°7, 2000


4
Sociedade Brasileira de Matemática

11. Juntando três quadrados congruentes e fazendo coincidir


lados comuns, sem superposição, podemos formar duas figuras
diferentes, como mostra a figura ao lado. Observe que uma
figura obtida de outra por rotação, deslocamento ou reflexão, é
congruente à mesma figura (muda apenas a posição). Por
exemplo, temos abaixo figura iguais em 4 posições diferentes:

Vamos agora pegar três losangos congruentes, um deles


120 o representado ao lado.
o
Quantas figuras diferentes podemos formar com os três
60 losangos, fazendo coincidir lados comuns ?

A) 1 B) 2 C) 3 D) 5 E) 9

12. Renata digitou um trabalho de 100 páginas numerados de 1 a 100 e o


imprimiu. Ao folhear o trabalho, percebeu que sua impressora estava com defeito,
pois trocava o zero pelo um e o um pelo zero na numeração das páginas. Depois
de consertar a impressora, quantas páginas teve que reimprimir, no mínimo ?
A) 18 B) 20 C) 22 D) 30 E) 28

13. Letícia vendeu todos seus CDs de videogames para três amigos, que lhe
pagaram, respectivamente, R$ 240,00, R$ 180,00 e R$ 320,00. Todos os CDs
tinham o mesmo preço. Quantos CDs tinha Letícia no mínimo?
A) 20 B) 37 C) 28 D) 21 E) 25

14. 6 cartões com números somente numa das faces


8 2 4
são colocados sobre uma mesa conforme a ilustração. Os
cartões X e Y estão com a face numerada voltada para X 6 Y
baixo. A média aritmética dos números de todos os cartões
é 5. A média aritmética dos números do cartão Y e seus três
vizinhos é 3. Qual é o número escrito no cartão X ?
A) – 4 B) 12 C) 0 D) 15 E) 10

EUREKA! N°7, 2000


5
Sociedade Brasileira de Matemática

2
15. Rafael tem da idade de Roberto e é 2 anos mais jovem que Reinaldo.
3
A
4
idade de Roberto representa da idade de Reinaldo. Em anos, a soma das
3
idades dos três é:
A) 48 B) 72 C) 58 D) 60 E) 34

16. Marcos quer pesar três maças numa balança de


Dois pratos, mas ele dispõe de apenas um bloco de
200 gramas. Observando o equilíbrio na balança, ele
observa que a maçã maior tem o mesmo peso que as
outras duas maçãs juntas; o bloco e a maçã menor
pesam tanto quanto as outras duas maçãs juntas; a
maçã maior junto com a menor pesam tanto quanto
bloco. O peso total das três maçãs é:
A) 250 g B) 300 g C) 350 g D) 400 g E) 450 g
100 20
17. No desenho ao lado estão representados 12 16
80
Quatro triângulos retângulos e um retângulo, 60

bem como suas medidas. 16 60


88
Juntando todas essas figuras, podemos construir 20
80

um quadrado. O lado desse quadrado irá medir: 12 16 100

A) 88 cm B) 100 cm C) 60 cm
D) 96 cm E) 80 cm

18. Numa certa cidade, foi adotado o seguinte sistema de rodízio de carros: duas
vezes por semana, de segunda a sexta, cada carro fica proibido de circular, de
acordo com o final de sua placa (algarismo das unidades). O número médio de
finais de placa proibidos diferentes para cada dia de proibição é:
A) 4 B) 1 C) 3 D) 2 E) indefinido

19. Alexandre, consultando a programação de filmes, decidiu gravar


Contato, cuja duração é de 150 minutos. Para gravar numa única fita, ele
começou com velocidade menor (modo EP, que permite gravar 6 horas) e, num
dado momento, mudou para a velocidade maior (modo SP, que permite gravar 2
horas), de forma que a fita acabou exatamente no fim do filme. Do início do filme
até o momento da mudança do modo de gravação, quantos minutos se passaram?
A) 60 B) 30 C) 15 D) 45 E) 105

EUREKA! N°7, 2000


6
Sociedade Brasileira de Matemática

20. Você sabe que existem 9 números de um algarismo, 90 números de dois


algarismos, 900 números de três algarismos, etc. Considere agora cada número
cujo último algarismo à direita representa o número de algarismos desse número.
Por exemplo, o número 9 074 é um deles, pois 4 é o número de seus algarismos.
Quantos números desse tipo existem ?
A) 99 999 999 B) 99 999 992 C) 100 000 000 D) 10 000 000
E) 1 000 000 000
XXI OLIMPÍADA BRASILEIRA DE MATEMÁTICA
Primeira Fase - Nível 2

01. Veja problema 01 do Nível 1.

02. Em um hotel há 100 pessoas. 30 comem porco, 60 comem galinha e 80


comem alface. Qual é o maior número possível de pessoas que não comem
nenhum desses dois tipos de carne?
A) 10 B) 20 C) 30 D) 40 E) 50

3. Uma folha quadrada foi dobrada duas vezes ao longo de suas diagonais
conforme ilustração abaixo, obtendo-se um triângulo isósceles. Foi feito um corte
na folha dobrada, paralelo à base desse triângulo, pelos pontos médios dos outros
lados. A área do buraco na folha corresponde a que fração da área da folha
original ?

1 1 3 3 1
A) B) C) D) E)
2 6 8 4 4

4. Veja problema 9 do Nível 1.


5. Veja problema 17 do Nível 1.

06. Contando-se os alunos de uma classe de 4 em 4 sobram 2 e contando-se de 5


em 5 sobra 1. Sabendo-se que 15 alunos são meninas e que nesta classe o número
de meninas é maior que o número de meninos, o número de meninos nesta classe
é igual a :
A) 7 B) 8 C) 9 D) 10 E) 11

07. O quociente de 5050 por 2525 é igual a :

EUREKA! N°7, 2000


7
Sociedade Brasileira de Matemática

A) 2525 B) 1025 C) 10025 D) 225 E) 2  2525


4
08. Qual o 1999o algarismo após a vírgula na representação decimal de ?
37
A) 0 B) 1 C) 2 D) 7 E) 8

09. Um retângulo ABCD está dividido em quatro retângulos menores. As áreas de


três deles estão na figura abaixo. Qual é a área do retângulo ABCD?
A D

16

12 27

B C

A) 80 B) 84 C) 86 D) 88 E) 91

10. Em um aquário há peixes amarelos e vermelhos: 90% são amarelos e 10% são
vermelhos. Uma misteriosa doença matou muitos peixes amarelos, mas nenhum
vermelho. Depois que a doença foi controlada verificou-se que no aquário, 75%
dos peixes vivos eram amarelos. Aproximadamente, que porcentagem dos peixes
amarelos morreram?
A) 15% B) 37% C) 50% D) 67% E) 84%

11. Pedro saiu de casa e fez compras em quatro lojas, cada uma num bairro
diferente. Em cada uma gastou a metade do que possuía e a seguir, ainda pagou
R$ 2,00 de estacionamento. Se no final ainda tinha R$ 8,00, que quantia tinha
Pedro ao sair de casa?
A) R$ 220,00 B) R$ 204,00 C) R$ 196,00 D) R$ 188,00
E) R$ 180,00
x  99
12. Quantos são os possíveis valores inteiros de x para que seja um
x  19
número inteiro?
A) 5 B) 10 C) 20 D) 30 E) 40

13. A diferença entre a maior raiz e a menor raiz da equação


 2 x  45 2   x  21 2 0 é:
A) 2 B) 3 C) 4 D) 5 E) 6

EUREKA! N°7, 2000


8
Sociedade Brasileira de Matemática

14. Uma bola de futebol é feita com 32 peças de couro. 12 delas são pentágonos
regulares e as outras 20 são hexágonos também regulares. Os lados dos
pentágonos são iguais aos dos hexágonos de forma que possam ser costurados.
Cada costura une dois lados de duas dessas peças.
Quantas são as costuras feitas na fabricação de uma bola de futebol?
A) 60 B) 64 C) 90 D) 120 E) 180
15. Hoje, 12/6/1999, Pedro e Maria fazem aniversário. No mesmo dia em 1996, a
idade de Pedro era 3/4 da idade de Maria. No mesmo dia em 2002, a idade de
Pedro será igual à de Maria quando ele tinha 20 anos. Quantos anos Maria está
fazendo hoje?
A) 30 B) 31 C) 32 D) 33 E) 34

16. Uma caixa contém 100 bolas de cores distintas. Destas, 30 são vermelhas, 30
são verdes, 30 são azuis e entre as 10 restantes, algumas são brancas e outras são
pretas. O menor número de bolas que devemos tirar da caixa, sem lhes ver a cor,
para termos a certeza de haver pelo menos 10 bolas da mesma cor é:
A) 31 B) 33 C) 35 D) 37 E) 38

17. Quantos são os triângulos que possuem medidas dos seus lados expressas por
números inteiros e tais que a medida do maior lado seja igual a 11 ?
A) 10 B) 11 C) 12 D) 24 E) 36

18. Os pontos S, T e U são os pontos de tangência do círculo inscrito no triângulo


PQR sobre os lados RQ, RP e PQ respectivamente. Sabendo que os
comprimentos dos arcos TU, ST e US estão na razão TU : ST : US = 5 : 8 : 11, a
razão TPU : SRT : UQS é igual a :
A) 7 : 4 : 1 B) 8 : 5 : 2 C) 7 : 3 : 2 D) 11 : 8 : 5 E) 9 : 5 : 1

19. Aos vértices de um cubo são atribuídos os números de 1 a 8 de modo que os


conjuntos dos números correspondentes aos vértices das seis faces são
{1, 2, 6, 7}, {1, 4, 6, 8}, {1, 2, 5, 8}, {2, 3, 5, 7}, {3, 4, 6, 7} e {3, 4, 5, 8}. O
vértice atribuído ao número 6 está mais longe do vértice de número
A) 1 B) 3 C) 4 D) 5 E) 7

20. Com os 5 números ímpares entre –5 e 4 e com os 5 números pares entre –5 e


4 são formados 5 pares de números. Se N é a soma dos produtos, obtidos em cada
par de números, o valor mínimo possível de N é igual a :
A) – 41 B) – 40 C) – 28 D) –10 E) 0

EUREKA! N°7, 2000


9
Sociedade Brasileira de Matemática

XXI OLIMPÍADA BRASILEIRA DE MATEMÁTICA


Primeira Fase - Nível 3

1. Veja problema 01 do Nível 1.


02. Veja problema 02 do Nível 2.

03. Um gafanhoto pula exatamente 1 metro. Ele está em um ponto A de uma reta,
só pula sobre ela, e deseja atingir um ponto B dessa mesma reta que está a 5
metros de distância de A com exatamente 9 pulos. De quantas maneiras ele pode
fazer isso?
A) 16 B) 18 C) 24 D) 36 E) 48

4. Sendo a  b e b  0, sabe-se que as raízes da equação x 2  ax  b 0 são


exatamente a e b. Então, a – b é igual a:
A) 0 B) 1 C) 2 D) 3 E) 4

05. Veja problema 09 do Nível 2.


06. Veja problema 14 do Nível 2.

07. A diferença entre a maior raiz e a menor raiz da equação


 2 x  45 2   x  21 2 0 é:
A) 2 B) 3 C) 4 D) 5 E) 6

08. Veja problema 12 do Nível 2.

1
09. Se 00 < x < 900 e cos x  então x está entre:
4
A) 00 e 300 B) 300 e 450 C) 450 e 600 D) 600 e 750 E) 750 e 900

10. Veja problema 11 do Nível 2.


n
11. Para todo n natural definimos a função f por: f  n se n é par,
2
f  n  3n  1 se n é ímpar. O número de soluções da equação f  f  f  n    16 é:

EUREKA! N°7, 2000


10
Sociedade Brasileira de Matemática

A) 2 B) 3 C) 4 D) 5 E) 6

12. O número N = 11111 . . . 11 possui 1999 dígitos, todos iguais a 1. O resto da


divisão de N por 7 é:
A) 1 B) 2 C) 4 D) 5 E) 6
13. Um quadrado ABCD possui lado 40cm. Uma circunferência contém os
vértices A e B e é tangente ao lado CD. O raio desta circunferência é:
A) 20cm B) 22cm C) 24cm D) 25cm E) 28cm

14. Veja problema 18 do Nível 2.


15. Para quantos valores inteiros de x existe um triângulo acutângulo de lados 12,
10 e x?
A) 9 B) 10 C) 12 D) 16 E) 18

16. A circunferência abaixo tem raio 1, o arco AB mede 70 0 e o arco BC mede


400. A área da região limitada pelas cordas AB e AC e pelo arco BC mede:

C
A

A) /8 B) /9 C) /10 D) /12 E) /14

17. A reta r contém os pontos (0, 4) e (7, 7). Dos pontos abaixo, qual é o mais
próximo da reta r?
A) (1999, 858) B) (1999, 859) C) (1999, 860)
D) (1999, 861) E) (1999, 862)

18. Quantos são os pares (x, y) de inteiros positivos que satisfazem a equação
2x +3y = 101 ?
A) 13 B) 14 C) 15 D) 16 E) 17

19. Quantos números inteiros entre 10 e 1000 possuem seus dígitos em ordem
estritamente crescente? (Por exemplo, 47 e 126 são números deste tipo; 52 e 566
não).
A) 90 B) 98 C) 112 D) 118 E) 120

20. Veja problema 10 do Nível 2.


21. Veja problema 15 do Nível 2.

EUREKA! N°7, 2000


11
Sociedade Brasileira de Matemática

22. No quadrado ABCD o ponto E é médio de BC e o ponto F do lado CD é tal


que o ângulo AEF é reto. Aproximadamente, que porcentagem a área do triângulo
AEF representa da área do quadrado?
A) 28% B) 31% C) 34% D) 36% E) 39%
23. Dois irmãos herdaram o terreno ABC com a forma de um triângulo retângulo
em A, e com o cateto AB de 84m de comprimento. Eles resolveram dividir o
terreno em duas partes de mesma área, por um muro MN paralelo a AC como
mostra a figura abaixo. Assinale a opção que contém o valor mais aproximado do
segmento BM.
B

M N

A C

A) 55m B) 57m C) 59m D) 61m E) 63m

24. As representações decimais dos números 21999 e 51999 são escritas lado a
lado. O número de algarismos escritos é igual a :
A) 1999 B) 2000 C) 2001 D) 3998 E) 3999

25. Veja problema 16 do Nível 2.

GABARITO

Primeiro Nível (5a. e 6a. séries)


1) B 6) A 11) E 16) B
2) D 7) B 12) E 17) E
3) B 8) C 13) B 18) A
4) B 9) A 14) E 19) D
5) D 10) B 15) C 20) C

Segundo Nível (7a. e 8a. séries)


1) B 6) E 11) D 16) E
2) D 7) C 12) C 17) E
3) E 8) B 13) A 18) A
4) A 9) E 14) C 19) D
5) E 10) D 15) B 20) B

Terceiro Nível (Ensino Médio)


1) B 6) C 11) C 16) B 21) B
2) D 7) A 12) A 17) D 22) B

EUREKA! N°7, 2000


12
Sociedade Brasileira de Matemática

3) D 8) C 13) D 18) E 23) C


4) D 9) E 14) A 19) E 24) B
5) E 10) D 15) A 20) D 25) E

XXI OLIMPÍADA BRASILEIRA DE MATEMÁTICA


Segunda Fase - Nível 1

PROBLEMA 1
Corte 10 algarismos do número 1234512345123451234512345, para que
o número restante seja o maior possível.

PROBLEMA 2
Sabe-se que três meses consecutivos de um determinado ano, não bissexto,
possuem cada um exatamente quatro domingos.
a) Estes meses podem ser janeiro, fevereiro e março?
b) Podem ser agosto, setembro e outubro?

PROBLEMA 3
Na figura, os triângulos ABC e EGF são equiláteros. O perímetro do triângulo
ABC é 132cm e, além disso,
AE = EC B
BD = DC
EF = FC
DG = GE D
a) Qual o perímetro da área sombreada?
G
b) Que fração da área do triângulo ABC
representa a área sombreada?
A C
E F
PROBLEMA 4
Pedro distribuiu 127 moedas de 1 real em sete caixas e colocou em cada uma
delas uma etiqueta dizendo o número de moedas da caixa. Essa distribuição foi
feita de forma que qualquer quantia de R$1,00 a R$127,00 pudesse ser paga
entregando-se apenas caixas fechadas. De que maneira Pedro fez essa
distribuição?

PROBLEMA 5
Um edifício muito alto possui 1000 andares, excluindo-se o térreo. Do andar
térreo partem 5 elevadores:
O elevador A pára em todos os andares.

EUREKA! N°7, 2000


13
Sociedade Brasileira de Matemática

O elevador B pára nos andares múltiplos de 5, isto é, 0, 5, 10, 15, …


O elevador C pára nos andares múltiplos de 7, isto é, 0, 7, 14, 21, …
O elevador D pára nos andares múltiplos de 17, isto é, 0, 17, 34, 51, …
O elevador E pára nos andares múltiplos de 23, isto é, 0, 23, 46, 69, …
a) Mostre que, excetuando-se o andar térreo, não existe nenhum andar onde
param os 5 elevadores.
b) Determine todos os andares onde param 4 elevadores.

PROBLEMA 6
Encontre o menor tabuleiro quadrado que pode ser ladrilhado usando peças com o
seguinte formato:

Obs: Ladrilhado significa completamente coberto, sem superposição de peças, e


de modo que nenhum ponto fora do tabuleiro seja coberto por alguma peça.

SOLUÇÕES SEGUNDA FASE - NÍVEL 1

SOLUÇÃO PROBLEMA 1
O maior número restante é 553451234512345. Para ver isto, podemos supor que
os cortes são feitos da esquerda para a direita. Se deixarmos de cortar todos os
quatro primeiros algarismos, o número que resta começará por 1, 2, 3 ou 4. Logo,
menor que o número acima. Feito isto, se deixarmos de cortar a segunda
seqüência 1234, o número que resta terá na primeira ou segunda casa, da
esquerda para a direita, 1, 2, 3 ou 4. Ainda menor que o número acima. Os dois
primeiros 5 devem permanecer, pois retirando-se um deles, completamos 9
retiradas e aí algum algarismo da terceira seqüência 1234 aparecerá na 1 a ou na 2a
casa. Finalmente devemos cortar a seqüência 12, que ocupa a 11 a e 12a posição.

SOLUÇÃO PROBLEMA 2
Se o dia primeiro de janeiro for Segunda-feira, e o ano não for bissexto, então os
meses de janeiro, fevereiro e março terão 4 domingos cada.

SOLUÇÃO PROBLEMA 3 (Solução resumida)


3 1 1 13
a) Perímetro 2  44  3121 . b) S '  S . S S
4 4 4 16

SOLUÇÃO PROBLEMA 4

EUREKA! N°7, 2000


14
Sociedade Brasileira de Matemática

Basta distribuir as moedas em 7 caixas contendo respectivamente 1, 2, 4, 8, 16,


32 e 64 moedas. Para outros pagamentos Pedro pode fazer 3 = 1 + 2, 5 = 1 + 4, 6
= 2 + 4, 7 = 1 + 2 + 4. Assim já pode pagar as quantias de 1 a 7 reais com o
conteúdo das caixas. Somando-se a parcela de 8 a estas somas chega-se nas
somas de 9 até 15. Somando-se a parcela de 16 às 15 somas assim formadas
obtém-se somas de 17 a 31. A estas acrescenta-se a parcela de 32. E finalmente a
parcela de 64, obtendo-se assim todas as somas de 1 a 127 = 1+ 2 + 4 + 8 + 16 +
32 + 64.

SOLUÇÃO PROBLEMA 5
a) O elevador B pára nos múltiplos de 5.
O elevador C pára nos múltiplos de 7.
O elevador D pára nos múltiplos de 17.
O elevador E pára nos múltiplos de 23.

Como 5, 7, 17 e 23 são números primos, para que todos parem num mesmo
andar, este tem que ser múltiplo de 5  7  17  23 = 13685 e o prédio só tem
1000 andares.

b) Para que num andar parem exatamente quatro elevadores, devem parar A,
que pára em todos, e três dos restantes.
B, C e D param nos múltiplos de 5  7  17 = 595
B, C e E param nos múltiplos de 5  7  23 = 805
B, D e E param nos múltiplos de 5  17  23 = 1955
C, D e E param nos múltiplos de 7  17  23 = 2737
Logo, os andares onde param 4 elevadores são o 595 e o 805.

SOLUÇÃO PROBLEMA 6
O menor tabuleiro é do tipo 10  10 coberto com 20 peças, como mostrado, por
exemplo, pela figura abaixo, à esquerda.
Com efeito, o número de casas do
tabuleiro é um quadrado perfeito
múltiplo de 5. Logo é 25, 100, 225
ou ... etc. Mas um tabuleiro 5  5 não
pode ser coberto com peças deste tipo,
pois ao tentarmos completar uma
lateral do tabuleiro, seremos
conduzidos a uma das duas figuras à
direita, as quais não se deixam

EUREKA! N°7, 2000


15
Sociedade Brasileira de Matemática

completar pelas peças para formar todo


o tabuleiro.

XXI OLIMPÍADA BRASILEIRA DE MATEMÁTICA


Segunda Fase - Nível 2

PROBLEMA 1
Três meses consecutivos de um determinado ano, não bissexto, possuem
exatamente quatro domingos cada um. Prove que um destes meses é fevereiro.

PROBLEMA 2
Num quadro-negro são escritos três inteiros. Começa-se, então, uma sequência de
movimentos onde, em cada passo, apaga-se um deles e escreve-se em seu lugar a
soma dos outros dois diminuída de uma unidade. Após vários movimentos, estão
escritos no quadro os números 17, 75 e 91. É possível que no início estejam
escritos no quadro :

a) 2, 2, 2 ?
b) 3, 3, 3 ?

PROBLEMA 3
Seja ABCD um quadrado. Escolhemos pontos M, N, P, Q respectivamente sobre
AB, BC, CD e DA, de modo que as circunferências circunscritas aos triângulos
MBN e PDQ sejam tangentes exteriormente. Mostre que MN +PQ  AC.

PROBLEMA 4
Determine o maior natural n para o qual existe uma reordenação (a, b, c, d) de (3,
6, 9, 12) (isto é, {a, b, c, d} = {3, 6, 9, 12}) tal que o número n 3a 6 b9 c12 d seja
inteiro. Justifique sua resposta.

PROBLEMA 5

EUREKA! N°7, 2000


16
Sociedade Brasileira de Matemática

A C
Um professor de matemática passou aos seus alunos a adição  onde A, B,
B D
C e D são inteiros positivos, as frações estão simplificadas ao máximo e os
denominadores são números primos entre si. Os alunos adicionaram as frações
tirando o mínimo múltiplo comum dos denominadores das parcelas e escrevendo
este como o denominador do resultado. Mostre que a fração que os alunos
encontraram como resultado está simplificada.

PROBLEMA 6
Determine todos os inteiros positivos n para os quais é possível montarmos
um retângulo 9  10 usando peças 1  n.

SOLUÇÕES SEGUNDA FASE - NÍVEL 2

SOLUÇÃO PROBLEMA 1
Se nenhum destes meses for fevereiro, o número total de dias não pode ser
menor que 91 = 7. 13 e portanto o número total de domingos não poderia ser
menor do que 13.

SOLUÇÃO PROBLEMA 2

a) Estão escritos inicialmente 3 números pares. Quando um deles é apagado, é


escrito em seu lugar um número ímpar. Após o 1º movimento ficam então
dois números pares e um número ímpar. Se apagarmos agora o número ímpar,
surgirá em seu lugar outro númro ímpar e se apagarmos um número par
aparecerá em seu lugar outro número par. Deste modo, após qualquer número
de movimentos restarão dois números pares e um número ímpar e portanto,
não é possível termos no final os três números ímpares 17, 75 e 91.
b) Sim, uma possível sequência de movimentos é : 3, 3, 3  5, 3, 3  5, 3, 7
 5, 11, 7  17, 11, 7  17 , 11, 27  17, 43, 27  17, 43, 59 17, 75,
59  17, 75, 91.

SOLUÇÃO PROBLEMA 3
A figura abaixo representa a situação, onde X e Y são os pontos médios dos
segmentos MN e PQ e Z é o ponto de tangência das circunferências. Então, como
 MBN PDQ 90 , segue que BX = MX = NX = XZ e DY = QY = YP =
YZ. Assim, MN + PQ = BX + XZ + ZY + YD  BD = AC .

EUREKA! N°7, 2000


17
Sociedade Brasileira de Matemática

A Q D

M P
Z
X

B N C

SOLUÇÃO PROBLEMA 4
Temos 3 a 6 b 9 c 12 d 2 b 2 d 3 a b 2 c d . Para (a, b, c, d) dados, o maior n
possível é mdc{b  2d , a  b  2c  d } b  2d . Note que b + 2d é máximo
(com b e d elementos distintos de {3, 6, 9, 12}) quando d = 12 e b = 9. Neste
caso, b + 2d = 33, e a + b + 2c + d = 21 + a + 2c. Tomando a = 6 e c = 3, temos
também a + b + 2c + d = 33, que é obviamente o maior valor possível para n,
obtido para (a, b, c, d) = (6, 9, 3, 12).
SOLUÇÃO PROBLEMA 5
Como os denominadores das frações são primos entre si, seu MMC é BD e assim,
AD  CB
a fração resultante é . Suponhamos que esta fração não seja irredutível
BD
isto é, que exista algum número primo p que divida o numerador e o denominador
desta fração. Como o produto BD é divisível por p, um dos seus termos, digamos
B sem perda de generalidade o seja. Entretanto, uma das parcelas da soma AD +
CB é divisível por p e como a soma, por hipótese, é divisível por p a parcela AD
é também divisível por p. Portanto A ou D é divisível por p. No primeiro caso
A
temos uma contradição com o fato da fração ser irredutível, no outro casos a
B
contradição está no fato de que os denominadores das frações iniciais sempre são
primos entre si.

SOLUÇÃO PROBLEMA 6
É claro que n deve ser no máximo 10 e dividir 90. Assim, restam para n as
possibilidades 1, 2, 3, 5, 6, 9, 10. Fora n = 6, é imediato que n pode assumir
qualquer um dos outros valores acima. Começando a tentar montar o retângulo
com peças 1  6 a partir de um canto, concluímos prontamente que a tarefa não é
possível.

EUREKA! N°7, 2000


18
Sociedade Brasileira de Matemática

XXI OLIMPÍADA BRASILEIRA DE MATEMÁTICA


Segunda Fase - Nível 3

PROBLEMA 1
Nos extremos de um diâmetro de um círculo, escreve-se o número 1 (primeiro
passo) . A seguir, cada semicírculo é dividido ao meio e em cada um dos seus
pontos médios escreve-se a soma dos números que estão nos extremos do
semicírculo (segundo passo) . A seguir, cada quarto de círculo é dividido ao meio
e em cada um dos seus pontos médios coloca-se a soma dos números que estão
nos extremos de cada arco (terceiro passo). Procede-se, assim, sucessivamente:
sempre cada arco é dividido ao meio e em seu ponto médio é escrita a soma dos
números que estão em seus extremos. Determinar a soma de todos os números
escritos após 1999 passos.

PROBLEMA 2 Veja problema 3 do nível 2.


PROBLEMA 3 Veja problema 4 do nível 2.

PROBLEMA 4
Determine todos os inteiros positivos n para os quais é possível montarmos um
retângulo 9  10 usando peças 1  n.

PROBLEMA 5
José tem três pares de óculos, um magenta, um amarelo e um ciano. Todo dia de
manhã ele escolhe um ao acaso, tendo apenas o cuidado de nunca usar o mesmo

EUREKA! N°7, 2000


19
Sociedade Brasileira de Matemática

que usou no dia anterior. Se dia primeiro de agosto ele usou o magenta, qual a
probabilidade de que dia 31 de agosto ele volte a usar o magenta?

PROBLEMA 6
Encontre as soluções inteiras de x 3  y 3 999 .

SOLUÇÕES SEGUNDA FASE - NÍVEL 3

SOLUÇÃO PROBLEMA 1

Seja S(n) a soma dos termos em cada passo em um dos semicírculos. Observemos
que S(1) = 2, S(2) = 4, e S(3) = 10. Deste modo, nos parece razoável conjecturar
que S(n) = 3n  1 +1. Claramente, S(1) = 31  1 + 1. Os novos termos adicionados
para formar Ln +1 representam somas de dois termos consecutivos de L n e cada
termo de Ln, excetuando-se o primeiro e o último, aparece em exatamente duas
destas somas. Daí, S(n +1) = S(n) + 2(S(n)  1) = 3S(n) – 2 = 3(3n  1 + 1) – 2 =
3(n + 1)  1 + 1. Levando em consideração o outro semicírculo, temos que a
soma após os 1999 passos é igual a 2.(31999  1 + 1)  2 = 2. 31998
SOLUÇÃO PROBLEMA 2 Veja solução do problema 3 do nível 2.
SOLUÇÃO PROBLEMA 3 Veja solução do problema 4 do nível 2.

SOLUÇÃO PROBLEMA 4
É claro que n deve ser no máximo 10 e dividir 90. Assim, restam para n as
possibilidades 1, 2, 3, 5, 6, 9, 10. Fora n = 6, é imediato que n pode assumir
qualquer um dos outros valores acima. Começando a tentar montar o retângulo
com peças 1  6 a partir de um canto, concluímos prontamente que a tarefa não é
possível.

SOLUÇÃO PROBLEMA 5
Sejam mn , an e cn as probabilidades de que no dia n ele use óculos magenta,
an  cn
amarelo e ciano, respectivamente. Temos m1 = 1, a1 = c1 = 0 e mn 1  ,
2
mn  cn mn  an
a n1  , e c n1  Como an + cn + mn = 1, temos
2 2

EUREKA! N°7, 2000


20
Sociedade Brasileira de Matemática

1  mn 1  (  2) 2  n
m n 1  . Assim, m n  , e em 31 de agosto a probabilidade
2 3
1  2  29
de que ele volte a usar o magenta é m31  .
3

SOLUÇÃO PROBLEMA 6
Temos ( x  y ) ( x 2  xy  y 2 ) 33 37 . Suponhamos x > y. Assim, os possíveis
valores de a = x – y são 1, 3, 9, 27, 37, 3  37, 9  37, 27  37 e cada valor permite
fazer y = x – a e precisamos apenas verificar se as raízes de
999
x 2  x( x  a)  ( x  a) 2  são inteiras. Na verdade, alguns destes valores
a
são obviamente inapropriados: a x  y x 3  y 3 0 (mod 3) , donde os
valores 1 e 37 podem ser descartados. Por outro lado, se x  y 3b temos
( x 3  y 3 ) 3b 3 , donde podemos descartar a  27. Os dois valores restantes, 3 e
9, são de fato possíveis e dão as quatro soluções:
(10,1), (  1, 10), (12,9) e (  9, 12).
XXI OLIMPÍADA BRASILEIRA DE MATEMÁTICA
Terceira Fase - Nível 1
PROBLEMA 1
Diga como dividir um cubo em 1999 cubinhos. A figura mostra uma maneira de dividir
um cubo em 15 cubinhos.

PROBLEMA 2
Emanuela, Marta e Isabel são três nadadoras que gostam de competir e por isso
resolveram organizar um desafio de natação entre elas. Ficou combinado o total
de pontos para o primeiro, o segundo e o terceiro lugares em cada prova. A
pontuação para primeiro lugar é maior que a para o segundo e esta é maior que a
pontuação para o terceiro. As pontuações são números inteiros positivos. O
desafio consistiu de várias provas e ao final observou-se que Emanuela fez 20
pontos, Marta 9 pontos e Isabel 10. A primeira prova foi vencida por Isabel.

EUREKA! N°7, 2000


21
Sociedade Brasileira de Matemática

(a) Quantas provas foram disputadas?


(b) Determine o total de pontos para o primeiro, segundo e terceiro lugares.

PROBLEMA 3
Um reino é formado por dez cidades. Um cidadão muito chato foi exilado da
cidade A para cidade B, que é a cidade do reino mais longe de A. Após um
tempo, ele foi expulso da cidade B para a cidade C do reino mais longe de B.
Sabe-se que a cidade C não é a mesma cidade A. Se ele continuar sendo exilado
dessa maneira, é possível que ele retorne à cidade A?

Nota: as distâncias entre as cidades são todas diferentes.

PROBLEMA 4
Adriano, Bruno e Carlos disputaram uma série de partidas de tênis de mesa. Cada
vez que um jogador perdia, era substituído pelo que estava a esperar. A primeira
partida foi disputada por Adriano e Bruno. Sabe-se que Adriano venceu 12
partidas e Bruno 21. Quantas vezes Adriano e Bruno se enfrentaram?
SOLUÇÕES TERCEIRA FASE - NÍVEL 1

PROBLEMA 1
SOLUÇÃO DE MARIANA DE MORAES SILVEIRA (Belo Horizonte - MG)
O cubo deve ser dividido em 1000 cubinhos, ou seja 10  10  10, depois, deve-
se pegar um deles e dividí-lo novamente em 1000 cubinhos para que obtenhamos
1999 cubinhos. Assim teremos 1000 – 1 (que será dividido) + 1000 = 1999
cubinhos.

PROBLEMA 2
SOLUÇÃO DE DIOGO DOS SANTOS SUYAMA (Belo Horizonte - MG)
a) Foram disputadas 3 provas. Como 20 + 10 + 9 = 39, o número de pontos
distribuidos por prova só poderia ser 3 ou 13, pois estes são os únicos
divisores de 39, a não ser o mesmo e o 1. Em consequências, o número de
provas também será um desses números. Porém, se forem disputadas 13
provas, só há uma maneira de se distribuir os pontos: 2 para o primeiro, 1
para o segundo e 0 para o terceiro. Entretanto, 0 não é positivo, sendo assim
descartada essa hipótese.
b) Já sabendo que são 3 provas, é impossível que a vencedora ganhe menos que
8 pontos, pois assim, Emanuela só conseguiria os 20 pontos fazendo 7, 7 e 6
pontos em cada prova. Para isso, seria preciso que a vencedora fizesse 7
pontos, a segunda colocada 6 e a última 0, mas como vimos, 0 não é positivo.
É impossível, também que a vencedora faça mais de 10 pontos, pois não seria

EUREKA! N°7, 2000


22
Sociedade Brasileira de Matemática

possível que a segunda fizesse mais pontos que a última, ou que esta não
fizesse 0 pontos. Então, as únicas possibilidades são: 1 a.  10, 2a.  2, 3a. 
1; 1a.  9, 2a.  3, 3a.  1; 1a.  8, 2a.  4, 3a.  1; e 1a.  8, 2a.  3, 3a.
 2. A primeira opção é incorreta, pois Isabel, que venceu uma das provas,
não poderia ter feito pontos nas outras. A segunda opção também não é
correta, pois Isabel teria que marcar apenas um ponto em duas provas. A
última opção é incorreta também, pois Isabel teria que marcar 2 pontos em
duas provas. Terceira opção: 1a.  8, 2a.  4, 3a.  1 é a correta. Veja o
quadro abaixo:

1a. Prova 2a. Prova 3a. Prova Total


Emanuela 4 8 8 20
Marta 1 4 4 9
Isabel 8 1 1 10

PROBLEMA 3 Veja solução do problema 2 do nível 2.


PROBLEMA 4 Veja solução do problema 3 do nível 2
XXI OLIMPÍADA BRASILEIRA DE MATEMÁTICA
Terceira Fase - Nível 2

PROBLEMA 1
Seja ABCDE um pentágono regular tal que a estrela ACEBD tem área 1. Sejam P
interseção entre AC e BE e Q a interseção entre BD e CE. Determine a área de
APQD.
D

Q
E C

A B

PROBLEMA 2
Um reino é formado por dez cidades. Um cidadão muito chato foi exilado da
cidade A para a cidade B, que é a cidade do reino mais longe de A. Após um

EUREKA! N°7, 2000


23
Sociedade Brasileira de Matemática

tempo, ele foi expulso da cidade B para a cidade C do reino mais longe de B.
Sabe-se que a cidade C não é a mesma cidade A. Se ele continuar sendo exilado
dessa maneira, é possível que ele retorne à cidade A?

Nota: as distâncias entre as cidades são todas diferentes.

PROBLEMA 3
Adriano, Bruno e Carlos disputaram uma série de partidas de tênis de mesa. Cada
vez que um jogador perdia, era substituído pelo que estava a esperar. A primeira
partida foi disputada por Adriano e Bruno. Sabe-se que Adriano venceu 12
partidas e Bruno 21. Quantas vezes Adriano e Bruno se enfrentaram?

PROBLEMA 4
Prove que há pelo menos um algarismo diferente de zero entre a 1.000.000 a. e a
3.000.000a. casa decimal de 2 após a vírgula.

SOLUÇÕES TERCEIRA FASE - NÍVEL 2

PROBLEMA 1 Veja solução do problema 1 do nível 3.

PROBLEMA 2
SOLUÇÃO DE EINSTEIN DO NASCIMENTO JÚNIOR ( Fortaleza - CE )
Há dez cidades A, B, C, D, E, F, G, H, I, J.
Um chato da cidade A foi exilado para a cidade mais longe de A, a cidade B.
Como B é a cidade mais longe de A, pode-se dizer que se tomarmos A como
sendo o centro de uma circunferência de raio AB, todas as cidades estarão dentro
dos limites da circunferência, exceto a cidade B que estará em cima dela.

A B

E D

EUREKA! N°7, 2000


24
Sociedade Brasileira de Matemática

Como as distâncias entre as cidades não são iguais e o chato foi exilado para a
cidade C que é a mais longe de B então BC > AB.
Da cidade C ele será exilado para a cidade D que é a mais longe de C e assim
sucessivamente até chegar na cidade J onde teremos a seguinte verdade:
AB < BC < CD < . . . < HI < IJ.
Ao chegar nesse ponto vemos que A com certeza não é a cidade mais longe de J
pois
AB = raio
AJ < raio
AJ < AB
AB < IJ
AJ < IJ
Logo ele irá para uma cidade diferente de A, e nunca retornará à cidade A.

PROBLEMA 3
SOLUÇÃO DE FÁBIO DIAS MOREIRA (Rio de Janeiro - RJ)
Quando começa a série, já ocorre um encontro entre Adriano (A) e Bruno (B).
Vamos chamar de VA, VB e VC o número de vitórias de Adriano, Bruno e Carlos,
respectivamente. Então ao final da série VA + VB = 33 e depois do 1o. jogo VA + VB
= 1. Suponhamos que o segundo jogo seja x  C. Chamemos de E o número de
jogos A  B.
Então no 2o. jogo E = 1. Enquanto C ganhar, VA + VB e E permanecem constantes.
Quando C perder, VA + VB aumenta uma unidade. O próximo jogo será A  B,
aumentando VA + VB e E em uma unidade. Após este jogo, o próximo será x  C.
Ou seja, para que E aumente uma unidade, VA + VB aumenta duas, e o aumento de
um em E. Como no 2o. jogo E = 1 e falta que VA + VB aumente 32 unidades,
ocorrem 1 + 16 = 17 jogos A  B.

PROBLEMA 4
SOLUÇÃO DE HENRIQUE CHOCIAY (Pinhais - PR)
Para começar a desenvolver 2 , utilizei o processo de extração que não utiliza
tentativas (processo prático por aproximação).

EUREKA! N°7, 2000


25
Sociedade Brasileira de Matemática

2
– 1 1,414
1 .0 0 
–96 1  2 = 24  4  100
4 .0 0 96 
281
1 .1 9 0 0 14  2 = 281  1  400
1 .1 2 9 6
0060400 141  2 = 2824  4  11900
11296 
Deste lado, o número de casas sempre aumenta em 1 casa,
nunca mais. (mesmo se houvesse um caso de 99999  9 =
899991 (só aumenta 1 casa) (entre 1.000.000 e 3.000.000)

Quando estivermos no número 1.000.000 de casas no multiplicador, teremos


999.999 casas decimais. Supondo que haja só 1 casa no resto nesta situação,
depois de 1.000.000 de operações, teremos 1.999.999 casas decimais (1 milhão
de zeros), 2.000.000 no multiplicador e 2.000.001 no resto, podendo obter
número diferente de zero.
Em geral o fato de, não podendo haver divisão, com o aumento das casas
divisoras em 1 e do resto em 2 e as casas decimais serem menores que as
divisoras em 1 torna impossível a obtenção desta seqüência de zeros entre as
casas de 1.000.000 e 3.000.000.

XXI OLIMPÍADA BRASILEIRA DE MATEMÁTICA


Terceira Fase - Nível 3
PRIMEIRO DIA

PROBLEMA 1 Veja problema 1 do nível 2.


PROBLEMA 2 Veja problema 4 do nível 2.

PROBLEMA 3
Temos um tabuleiro quadrado 10  10.
Desejamos colocar n peças em casas do tabuleiro de tal forma que não existam 4
peças formando em retângulo de lados paralelos aos lados do tabuleiro.

EUREKA! N°7, 2000


26
Sociedade Brasileira de Matemática

Determine o maior valor de n para o qual é possível fazer esta construção.

SEGUNDO DIA

PROBLEMA 4
O planeta Zork é esférico e tem várias cidades. Dada qualquer cidade existe uma
cidade antípoda (i.e., simétrica em relação ao centro do planeta).
Existem em Zork estradas ligando pares de cidades. Se existe uma estrada
ligando as cidades P e Q então também existe uma estrada ligando as cidades P'
e Q', onde P' é a antípoda de P e Q' é a antípoda de Q. Além disso, estradas não
se cruzam e dadas duas cidades P e Q sempre é possível viajar de P a Q usando
alguma seqüência de estradas.
O preço da Kriptonita em Urghs (a moeda planetária) em duas cidades ligadas
por uma estrada difere por no máximo 100 Urghs. Prove que existem duas
cidades antípodas onde o preço da Kriptonita difere por no máximo 100 Urghs.

PROBLEMA 5
Em Tumbólia existem n times de futebol .
Deseja-se organizar um campeonato em que cada time joga exatamente uma vez
com cada um dos outros. Todos os jogos ocorrem aos domingos, e um time não
pode jogar mais de uma vez no mesmo dia.
Determine o menor inteiro positivo m para o qual é possível realizar um tal
campeonato em m domingos.

PROBLEMA 6
Dado triângulo ABC mostre como construir com régua e compasso um triângulo
A’B’C’de área mínima com C ' AC , A' AB e B ' BC tal que
   
B ' A' C '  B A C e A' C ' B '  A C B.

EUREKA! N°7, 2000


27
Sociedade Brasileira de Matemática

C' B'

A B
A'

SOLUÇÕES TERCEIRA FASE - NÍVEL 3

PROBLEMA 1
SOLUÇÃO DE HUGO PINTO IWATA (São José do Rio Preto - SP)

R Q
E C

S
P

A B

Como o pentágono e a estrela são regulares, o quadrilátero APQD é um trapézio.


A área do trapézio APQD é igual à área do triângulo APD somada à do triângulo
____
PQD. Como BDRP também é um trapézio, RP// QD, então a área de PQD é
igual à de RQD. Como a estrela é regular, a área de RQD é igual à de ERS, então,
a área de PQD é igual à de ERS. Assim a área do trapézio APQD é igual à soma
das áreas dos triângulos APD e ERS, que é igual à figura APDRES, que é
exatamente metade da estrela toda.
Resposta: A área de APQD é 0,5.

PROBLEMA 2

EUREKA! N°7, 2000


28
Sociedade Brasileira de Matemática

SOLUÇÃO DE HUMBERTO SILVA NAVES (Goiânia - GO)

Suponhamos, por absurdo, que todos os algarismos das casas decimais entre a
1.000.000a. casa decimal e a 3.000.000a. casa decimal de 2 fossem zero, então:
   
6 6 6
10 210 1010 2  10 310 2 (onde  x   Z e  x   x   x   1)
   
6 6 6 6 6 6
210 310
10 K  10 2 (onde K  1010 2 )  10 210 K 10 310 2  10 210 K  1,
6 6 6
mas como 10 210 K 10 310 2 , (pois se não fosse teríamos 2  K / 1010 ,
um absurdo, pois 2 é irracional!) então:
K K 1
6
10 210 K  10 310
6
2  10 210 K  1 
6
6
 106
 2 10 6

10 310 10 10
K2 K2 2K 1 2 2106 2K 1
2106
 2  210 6
 4106
 6106
 K  2 10  K 2  2106  4106 ,
10 10 10 10 10 10
mas como K  10 2   Z, temos (pela definição de  x  ):
6
10

K K 2 1 2K 1
6
K 1010 2  K  1  10  2  210  10   210 6
 ,
10 10
6
10 4 10 2 6 6

logo:
6 2K 1 1 1
K 2  2 10 210  K 2  210 6
 4106
 K 2   4106  K 2  1 
10 10 2 10
 K 2  1, um absurdo, pois não existe
6
2 2106 2 210
K  2 10  K  1  0  2 10
nenhum inteiro maior que 0 e menor que 1, disto concluímos que há um
algarismo diferente de 0 nestas casas decimais. (Poderíamos ter uma aproximação
6
melhor pois 2K é bem menor que 10 210 ).
Obs:  x  denota a função do "maior inteiro": é o único inteiro tal que
 x   x   x   1.
PROBLEMA 3
SOLUÇÃO DA BANCA
O problema é equivalente a encontrar subconjuntos A1, A2, …, A10 do conjunto {1,
2, 3, …, 10} cuja soma do número de elementos seja a maior possível tais que a
interseção de dois quaisquer deles tenha no máximo um elemento (Ai é o
conjunto das posições das peças na i-ésima linha do tabuleiro). Se Ai tem ki
k (k  1)
elementos então há C k2i  i i subconjuntos de 2 elementos não pode
2
2
pertencer a dois dos conjuntos Ai, e há no total C10 45 subconjuntos
de 2 elementos de

EUREKA! N°7, 2000


29
Sociedade Brasileira de Matemática

10
k i ( k i  1)
{1, 2,…,10}. Assim, devemos ter 
i 1 2
45.

Por outro lado, se existem i, j com kj > ki + 1, temos


k (k  1) (k j  1)( k j  2) k i (k i  1) k j (k j  1)
C k2i 1  C k2j  1  i i     k i  1  k j  C k2i  C k2j .
2 2 2 2
10
k i ( k i  1) 10
Assim para minimizar 
i 1 2
mantendo 
i 1
k i fixo devemos ter

k i  k j 1 para todo i, j. Se observamos que 5C 42  5C 32 5 6  5 3 45,


concluímos que se
10
k i (k i  1) 10


i 1 2
45 então k i 1
i 5 4  5 3 35, valendo a igualdade se e só

se 5 dos ki são iguais a 4 e os outros 5 iguais a 3. Para que a contrução seja


possível nesse caso precisamos de que cada par de elementos apareça em
exatamente um dos conjuntos Ai . Nesse caso, cada elemento de {1, 2, 3…, 10}
deve aparecer em 3 conjuntos com 4 elementos ou em um conjunto com 4
elementos e 3 conjuntos com 3 elementos (pois cada um dos outros 9 elementos
aparece exatamente uma vez junto com ele). Como haveria 5 conjuntos com 4
elementos, o número médio de conjuntos com 4 elementos aos quais cada
elemento pertence é 2, donde há elementos que pertencem a 3 conjuntos com 4
elementos (pois um elemento não pode pertencer a exatamente 2 conjuntos com 4
elemetos). Assim, podemos supor sem perda de generalidade que A1 = {1, 2, 3,
4}, A2 = {1, 5, 6, 7} e A3 = { 1, 8, 9, 10}, mais então qualquer outro conjunto de 4
elementos deve estar contido em {2, 3, …, 10}, e portanto deve intersectar um
dos conjunto A1, A2, A3, A4, em pelo menos 2 elementos. Portanto, não é possível
10
que k i 1
i seja igual a 35. Por outro lado é possível construir exemplos com
10

k
i 1
i 34, como abaixo:

A1 = {1, 2, 3, 4}, A2 = {1, 5, 6, 7}, A3 = { 2, 5, 8, 9}, A4 = {3, 6, 8, 10},


A5 = {1, 9, 10}, A6 = { 2, 7, 10}, A7 = {3, 7, 9}, A8 = {4, 5, 10}, A9 = { 4, 6, 9} e
A10 = {4, 7, 8}.

   
   
   

EUREKA! N°7, 2000


30
Sociedade Brasileira de Matemática

   
  
  
  
  
  

  

PROBLEMA 4
SOLUÇÃO DE GILBERTO SANTOS DO NASCIMENTO (São Paulo - SP)
Seja C1' o antípoda de C1 .
Vamos ligar C1 a C1' e vice-versa, formando uma linha fechada. Abaixo
C 'j é o antípoda de C j para todo j.

C3 …
Cn
C2
C'1
C1

C'n
… C'2
C'3

Agora, supondo que a diferença da Kriptonita de C1' para C1 seja maior que
100. Então, vamos supor que (C2 – C1) + (C3 – C2) +…+ (C1' – Cn) > 100. Como
ao percorrer o caminho, temos de ter uma diferença zero ao chegarmos em C1
novamente, somando (C2' – C1') + (C3' – C2') +…+ (C1 – Cn') < – 100.
Agora, supondo que o Superman trace uma linha de C1 a C1' (esta linha não
poderia ser uma estrada, pois | C1' – C1| > 100) a soma das diferenças na parte de cima
da linha deve ser maior que 100 e embaixo menor que –100.
> 100

Parte de cima (p.c.)
C1 C'1
Parte de baixo (p.b.)

Agora, supondo que esta linha percorra a figura, ligando todas as cidades antípodas, na
parte de cima, a soma deve continuar sendo maior que 100 e embaixo menor que 100. Em

EUREKA! N°7, 2000


31
Sociedade Brasileira de Matemática

p.c. (parte de cima), a soma não pode passar bruscamente de > 100 para < –100, pois são
somadas apenas duas diferenças de cada vez (menores que 200 no total!). Assim, para que
p.c. fique negativo < –100 e p.b. fique positivo > 100, teríamos de ter duas cidades
antípodas com diferença > 100 em módulo.

> 100 Ck



p.c. C1'
C1
p.b. < –100 p.b. p.c. > 100 C1'

… C2'
< –100 … …
Ck'

Continuando o percurso, ao chegarmos em C1', teremos de ligá-lo a C1. No


entanto, p.c. estará em baixo e a soma das diferenças na direção de C1' para C1
terá de ser positivo > 100. Mas essa soma era negativa e < –100 quando
começamos () Contradição.
O mesmo ocorre analogamente com p.b. Logo, em algum par da cidades (uma
cidade e sua antípoda), a diferença do preço da Kriptonita deverá ser menor ou
igual a 100.
Viva o Superman!.

PROBLEMA 4
SOLUÇÃO DE HUMBERTO SILVA NAVES (Goiânia - GO)
Suponhamos, por absurdo, que os preços diferem por mais de 100 Urghs em
todas as cidades antípodas, então:

| x0 – y0| > 100  M0 – m0 > 100 (onde xn e yn são antípodas e representam o preço
da Kriptonita).
| x1 – y1| > 100  M1 – m1 > 100

| xn – yn| > 100  Mn – mn > 100 (Onde Mn = máx (xn, yn) e mn = min (xn, yn))

Como sabemos que existe um caminho de estradas que leva de M0 até m0, então
deve existir uma estrada que liga (para certo i, j  N; i, j  n) Mi  mj.

EUREKA! N°7, 2000


32
Sociedade Brasileira de Matemática

Como existe uma estrada ligando Mi  mj, também existe uma estrada ligando
mj  Mi (antípodas). Pode acontecer i = j, caso em que se conclui facilmente
que Mi – mi > 100, um absurdo pois mi e Mi são "vizinhas", logo o preço da
Kriptonita difere por no máximo 100 Urghs.
Se i  j, então:
| Mj – mi |  100 (são "vizinhas")
| Mi – mj |  100, mas como
Mi – mi > 100 e Mj – mj > 100, então:
Mi + Mj – mi – mj > 200
Mi – mj + Mj – mi > 200  | Mi – mj + Mj – mi| > 200  | Mi – mj| + | Mj – mi| >
200  200  | Mi – mj| + | Mj – mi| > 200, um absurdo, logo existem cidades
antípodas cujo preço difere no máximo em 100 Urghs.

PROBLEMA 5
SOLUÇÃO DE FABRÍCIO SIQUEIRA BENEVIDES (Fortaleza - CE)

Façamos 2 casos, n par e n ímpar.

i) n par.

Cada time tem que jogar com cada um dos outros. Se os times são: T1, T2, …, Tn;
temos que um time Ti tem que jogar (n – 1) vezes e para isso precisará de pelo
menos (n – 1) domingos. (pois só pode jogar 1 vez por domingo). Mostraremos
que é possível realizar o campeonato em (n – 1) domingos. Para isso basta que o
jogo entre Ti e Tj (i  j) ocorra no seguinte domingo.

1) dij  i + j (mod n – 1), 1  dij  n – 1 para  i  n, j  n


2) din  2i (mod n – 1), 1  din  n – 1 para todo i  n, j  n
(se um dos times for Tn).

Podemos observar isso numa tabela que indique o dia entre Ti e Tj


Exemplo: para n = 6

dij T1 T2 T3 T4 T5 T6
T1 3 4 5 1 2
T2 3 5 1 2 4
T3 4 5 2 3 1
T4 5 1 2 4 3

EUREKA! N°7, 2000


33
Sociedade Brasileira de Matemática

T5 1 2 3 4 5
T6 2 3 4 5 1

O campeonato organizado assim satisfaz o problema pois: é fácil ver que um time
i joga com cada um dos outros times (no domingo dij, j  i). E cada time só joga
uma vez num mesmo dia, caso contrário teríamos: um time Ti que joga contra Tj e
Tk no mesmo domingo, ou seja dij = dki

1) Se i = n: djn = dkn  2j  2k (mod n – 1) como (2, n – 1) = 1 teriamos


j  k (mod n – 1), {j, k}  {1, 2, …, n – 1}  j = k, uma contradição.

2) Se i  n.

2.1) j e k  n : dik = dij  i + k  i + j (mod n – 1)  j  k (mod n – 1) e k = j.


uma contradição.
2.2) j = n, k  n, sem perda de generalidade:
din = dik  i + i  i + k (mod n – 1) i  k (mod n – 1),
{i, j}{1, 2, …, n – 1}  i = j, uma contradição.

Agora se n for ímpar, como cada time tem que jogar com todos os outros seria
necessário pelo menos (n – 1) domingos.
Só que (n – 1) domingos não são suficientes pois em cada dia há um time que fica
sem jogar. Assim, se no primeiro dia Ti foi o time que não jogou, ele ainda
precisará de mais ( n – 1) domingos para jogar contra os outros. De modo que são
necessários pelo menos n domingos.
Para ver que n domingos são suficientes, basta que o campeonato se organize
assim: Sejam T1, T2, …, Tn os times. Criamos um time virtual chamado Tn + 1 onde
jogar contra Tn + 1 um certo dia, significa não jogar naquele dia.
Temos então n + 1 = x times, organizamos então como no caso anterior o
campeonato. Como x é par isso pode ser feito em x – 1 = n dias.
Obs: O exemplo para (2k – 1) times é obtido do de (2k) times esquecendo-se um
dos times.

Resposta: Se n é par m = n – 1.
Se n é ímpar m = n.

PROBLEMA 6
SOLUÇÃO DA BANCA

EUREKA! N°7, 2000


34
Sociedade Brasileira de Matemática

 c

B'
C'
c

D


a a 

A A' B

Sejam A, B, C os ângulos internos do triângulo ABC, sejam A', B', C'
os ângulos internos do triângulo A'B'C' e consideremos A' = A e C' = C.
Seja D o ponto de interseção das circunferências circuscritas aos triângulos AA'C'
e CC'B'. Nos quadriláteros inscritíveis AA'DC' e CC'DB' temos A'DC' = π – A
e C'DB' = π – C. Logo, A'DB' = 2π – (π – A) – (π – C) = π – B, e
portanto, a circunferência circunscrita ao triângulo BB'A' passa por D.

No quadrilátero inscritível AA'DC', DAA' = DC'A' =  e DA'C' = DAC' =


a. Como A = A' concluimos que DA'B' = . Logo, no quadrilátero inscritível
BB'DA' temos que DBB' = . No quadrilátero inscritível CC'DB' temos que
DCB' = DC'B' = c, e como C = C' concluímos que DCC' = .
O ponto D está então associado ao triângulo ABC pela propriedade:

DAB = DBC = CDA

e portanto não depende da posição de A', B' e C'. O ponto D é fixo e sua
construção será mostrada no final da solução.

Como os ângulos A'DB', B'DC' e C'DA' são constantes, a menor área possível do
triângulo A'B'C' é obtida quando os segmentos DA', DB' e DC' forem os menores
possíveis. Logo, DA', DB' e DC' são respectivamente perpendiculares aos lados
AB, BC e CA.

EUREKA! N°7, 2000


35
Sociedade Brasileira de Matemática

Construção do ponto D

Seja E a interseção da mediatriz de AB com a perpendicular a BC traçada por B. A


circunferência de centro E e raio EA = EB é tangente em B à reta BC. Logo, para
qualquer ponto X do menor arco AB tem-se que XAB = XBC.

Seja F a interseção da mediatriz de BC com a perpendicular a CA traçada por C.


A circunferência de centro F e raio FB = FC é tangente em C à reta CA. Logo,
para qualquer ponto X do menor arco BC tem-se que XBC = XCA.

O ponto D, interseção desses dois arcos é tal que DAB = DBC = DCA.
(Note que qualquer ponto D com esta propriedade deve pertencer a cada um dos
lugares geométricos descritos acima, o que nos dá a unicidade).

XXI OLIMPÍADA BRASILEIRA DE MATEMÁTICA


Resultado - Primeiro Nível (5a. e 6a. séries)

NOME CIDADE – ESTADO PRÊMIO


Henry Wei Cheng Hsu São Paulo – SP Ouro
Diogo dos Santos Suyama Belo Horizonte – MG Ouro
Sergio Santos do Nascimento São Paulo – SP Ouro
Gustavo Eufrásio Farias Fortaleza – CE Ouro
Luciano Lacerda Silveira Campo Grande – MS Prata
Emanuel Augusto Varussa Padovan Rio Claro – SP Prata
Fabrício Henrique de Faria São Paulo – SP Prata
Thiago Jorge Marinho Vieira Fortaleza – CE Prata
Paulo Roberto Sampaio Santiago Salvador – BA Prata
Mariana de Moraes Silveira Belo Horizonte – MG Prata
Gabriel Vieira Lana Belo Horizonte – MG Bronze
João Cláudio Telles Vianna Rio de Janeiro – RJ Bronze

EUREKA! N°7, 2000


36
Sociedade Brasileira de Matemática

Rafael Daigo Hirama Campinas – SP Bronze


Ana Cláudia de Franco Suzuki São Paulo – SP Bronze
Luiza de Almeida Aoki S. J. dos Campos – SP Bronze
Bruno Leonardo Schneider São José – SC Bronze
Paulo Rebello Bortolini Jundiaí – SP Bronze
Victor Mesquita Barbosa Fortaleza – CE Bronze
Thiago Augusto Caldas Bello Salvador – BA Bronze
Sinuhe Djin Maschio Shin São Paulo – SP Bronze
Raul Máximo Alexandrino Nogueira Fortaleza – CE Bronze
Bruno Fiorio Fortaleza – CE Bronze
Pedro H. Milet Pinheiro Pereira Rio de Janeiro – RJ Bronze
Bernardo Melo Sobreira Fortaleza – CE Bronze
Mário Luiz Aranha da Silva Salvador – BA Bronze
Conrado F. Paulo da Costa Rio de Janeiro – RJ Bronze
Rodrigo Aguiar Pinheiro Fortaleza – CE Bronze
Daniel Medeiros de Albuqerque Fortaleza – CE Bronze
Gabriela Duarte Costa Constantino Timóteo – MG Bronze
Tiago Porto Barbosa Fortaleza – CE Menção Honrosa
Vitor Henrique Gonçalves São Carlos – SP Menção Honrosa
Gabriel Tomé de Lima Mogi das Cruzes – SP Menção Honrosa
Gustavo Pinheiro Melo Fortaleza – CE Menção Honrosa
Túlio Ivo Cordeiro Fulálio Campina Grande – PB Menção Honrosa
Leonardo Lucas Rentz Maceió – AL Menção Honrosa
Daniela Satie Kondo São Paulo – SP Menção Honrosa
Rafael Santos Correia de Araujo Salvador – BA Menção Honrosa
Felipe Paupitz Schlichting Florianópolis – SC Menção Honrosa
Álinson Santos Xavier Fortaleza – CE Menção Honrosa
Antonia Taline de Souza Mendonça Fortaleza – CE Menção Honrosa
Gustavo Hübner Campina Grande – PB Menção Honrosa
Leonardo Deeke Boguszewski Curitiba – PR Menção Honrosa
Paola Valente Giorgini Rio de Janeiro – RJ Menção Honrosa
Roberta Pieroni Visconti São Paulo – SP Menção Honrosa
Alan Hideki Uchida São Paulo – SP Menção Honrosa
Cincinato Furtado Leite Neto Fortaleza – CE Menção Honrosa
Marcus Edson Barreto Brito Fortaleza – CE Menção Honrosa
Thiago de Sá Jorge Curitiba – PR Menção Honrosa
Vento Inte Nunes Vieira Curitiba – PR Menção Honrosa

Resultado - Segundo Nível (7a. e 8a. séries)

NOME CIDADE – ESTADO PRÊMIO


Henrique Chociay Pinhais – PR Ouro
Davi Máximo Alexandrino Nogueira Fortaleza – CE Ouro
Maurício Massao Soares Matsumoto São Paulo – SP Ouro
Fábio Dias Moreira Rio de Janeiro – RJ Ouro
Eduardo Kunio Kuroda Abe São Paulo – SP Ouro
Larissa de Lima Fortaleza – CE Prata
Einstein do Nascimento Júnior Fortaleza – CE Prata
Diego Cortez Gutierrez S. J. dos Campos – SP Prata
Bernardo Freitas Paulo da Costa Rio de Janeiro – RJ Prata
Bruno Koga Fortaleza – CE Prata
Rafael Tajra Fonteles Teresina – PI Prata
André Luis Hirschfeld Danila São Paulo – SP Prata
Rodrigo Barbosa dos Santos Stein Vitória – ES Prata
Daniel Pessôa Martins Cunha Fortaleza – CE Bronze
Jaquelyne Gurgel Penaforte Fortaleza – CE Bronze
Thiago Braga Cavalcante Fortaleza – CE Bronze
Henrique Cortada Barbieri São Paulo – SP Bronze

EUREKA! N°7, 2000


37
Sociedade Brasileira de Matemática

Vinícius Piovesan de Toledo Jundiaí – SP Bronze


Lucas Gabriel Maltoni Romano Jundiaí – SP Bronze
Danilo Vieira Castejon Goiânia – GO Bronze
Thiago da Silva Sobral Fortaleza – CE Bronze
Guilherme Oliveira Campos Bauru – SP Bronze
Eduardo Barbosa Araújo Fortaleza – CE Bronze
Tatyana Zabanova Campinas – SP Bronze
Rafael Montorfano Franco Maringá – PR Bronze
Otacílio Torres Vilas Boas Salvador – BA Bronze
Henrique Fernandes Macedo Juiz de Fora – MG Menção Honrosa
Vinicius de Aguiar Furvie São Paulo – SP Menção Honrosa
Renato R. Sinohara da S. Souza S. J. dos Campos – SP Menção Honrosa
Daniel Teixeira Brasília – DF Menção Honrosa
Jefferson Ho Yun Lee São Paulo – SP Menção Honrosa
Kiyoshi Horie Filho Ourinho – SP Menção Honrosa
Fábio Eiji Arimura São Paulo – SP Menção Honrosa
Toni Chenson Wang São Paulo – SP Menção Honrosa
Guilherme Tosi Nova Venécia – ES Menção Honrosa
Yuri Gomes Lima Fortaleza – CE Menção Honrosa
Thiago Mizuta São Paulo – SP Menção Honrosa
Lucas Sáber Rocha Macaé – RJ Menção Honrosa
Daniel Nascimento Duplat Salvador – BA Menção Honrosa
Tiago Monteiro Fernandes Rio Claro – SP Menção Honrosa
Caio Ribeiro de Souza Rio de Janeiro – RJ Menção Honrosa
Adalberto Studart Neto Fortaleza – CE Menção Honrosa
Renato Araújo Barbosa Sete Lagoas – MG Menção Honrosa
Cibele Ferreira de Souza Mineiros – GO Menção Honrosa
Marina Lima Medeiros Fortaleza – CE Menção Honrosa
Carolina Nunes Nery Belo Horizonte – MG Menção Honrosa
Germanna Oliveira Queiroz Fortaleza – CE Menção Honrosa
Luciana Akemi Nishimaru São Paulo – SP Menção Honrosa
Fabiano Siggelkow Linhares São Paulo – SP Menção Honrosa
SandraTie Nishibe Minamoto Mogi das Cruzes – SP Menção Honrosa
Daniel Haanwinckel Junqueira Salvador – BA Menção Honrosa
Solleon Natus Tavares de Menezes Fortaleza – CE Menção Honrosa
Anna Laura Sfredo São Paulo – SP Menção Honrosa
Bruno Gomes Coelho São Paulo – SP Menção Honrosa
Daniel Bréscia dos Reis Belo Horizonte – MG Menção Honrosa
André Bastos Veras Teresina – PI Menção Honrosa
LincolnYoshyiti Hamaji São Paulo – SP Menção Honrosa
João Paulo Aguiar Santos Juiz de Fora – MG Menção Honrosa
Bruno Bozon Furlan São Paulo – SP Menção Honrosa
Márcio Antonio Ferreira Belo Filho Goiânia – GO Menção Honrosa
João Felipe Almeida Destri Florianópolis – SC Menção Honrosa
Caio Bória de Oliveira S. J. dos Campos – SP Menção Honrosa
Patrick Gonçalves Jaguaré – ES Menção Honrosa
Larissa Goulart Rodrigues Goiânia – GO Menção Honrosa
Eduardo Horai São Paulo – SP Menção Honrosa
Resultado - Terceiro Nível (Ensino Médio)

NOME CIDADE – ESTADO PRÊMIO


Daniel Massaki Yamamoto São Paulo – SP Ouro
Daniel Nobuo Uno São Paulo – SP Ouro
Ulisses Medeiros Albuquerque Fortaleza – CE Ouro
Humberto Silva Naves Goiânia – GO Ouro
Carlos Stein Naves de Brito Goiânia – GO Prata
Lucas Heitzmann Gabrielli São Paulo – SP Prata
Fabrício Siqueira Benevides Fortaleza – CE Prata
Giuliano Boava Criciúma – SC Prata
Jônathas Diógenes Castello Branco Fortaleza – CE Prata
Ronaldo Ikaro Farias Araújo Fortaleza – CE Prata
Carlos Emanuel Rodrigues Nogueira Fortaleza – CE Bronze
Daniel Mourão Martins Fortaleza – CE Bronze
Gilberto Santos do Nascimento São Paulo – SP Bronze
Rogério Uhlmann Yamauti São Paulo – SP Bronze

EUREKA! N°7, 2000


38
Sociedade Brasileira de Matemática

Fernando Silva Barros C. Lafaiete – MG Bronze


Leandro dos Santos de Jesus Rio de Janeiro – RJ Bronze
Hugo Pinto Iwata S. José do Rio Preto – SP Bronze
Leandro de Mattos Ferreira Rio de Janeiro – RJ Bronze
Bruno Fernandes Cerqueira Leite São Paulo – SP Bronze
Adenilson Pereira Bonfim Belém – PA Bronze
Mônica Mitiko Soares Matsumoto São Paulo – SP Bronze
Leonardo da Costa Linhares Rio de Janeiro – RJ Bronze
Tertuliano Franco Santos Franco Salvador – BA Bronze
Arthur Duarte Nehmi São Paulo – SP Bronze
Paulo César de Melo Hanaoka Campo Grande – MS Bronze
João Alfredo Castellani F. Freire Salvador – BA Menção Honrosa
Eduardo Famini Silva Salvador – BA Menção Honrosa
Lívia Camargos Rodrigues Oliveira Belo Horizonte – MG Menção Honrosa
Roberto Tiburcio Canito Frota Fortaleza – CE Menção Honrosa
Rui Facundo Vigelis Fortaleza – CE Menção Honrosa
Carlos Yuji Hatae São Paulo – SP Menção Honrosa
Daniel Pinheiro Sobreira Fortaleza – CE Menção Honrosa
Pedro Paulo de Simoni Gouvéia Fortaleza – CE Menção Honrosa
Thiago Barros Rodrigues Costa Fortaleza – CE Menção Honrosa
Mauricio Masayuki Honda São Paulo – SP Menção Honrosa
Christian Lyoiti Watanabe Itaguaí – RJ Menção Honrosa
Guilherme Goettems Schneider São Leopoldo – RS Menção Honrosa
Danilo Castello Branco A. Bessa São Paulo – SP Menção Honrosa
Bruno Woltzenlogel Paleo Piracicaba – SP Menção Honrosa
Camila Shirota Piracicaba – SP Menção Honrosa
Miriam Ou São Paulo – SP Menção Honrosa
Diêgo Veloso Uchôa Teresina – PI Menção Honrosa
Pedro Ferreira Fortaleza – CE Menção Honrosa
Celio Hira São Paulo – SP Menção Honrosa
Gustavo Maltez Lengler Rio de Janeiro – RJ Menção Honrosa
Fernando Duarte Menezes Fortaleza – CE Menção Honrosa
Fernando Prado Rocha Goiânia – GO Menção Honrosa
Paulo Henrique Jacob Silva São Paulo – SP Menção Honrosa
Zhang He São Paulo – SP Menção Honrosa
Renato Takamatsu São Paulo – SP Menção Honrosa
Ulisses Duarte Nehmi São Paulo – SP Menção Honrosa
Eduardo Moraes de Morais São Paulo – SP Menção Honrosa
Humberto Vinhais São Paulo – SP Menção Honrosa
Ilan Felts Almog São Paulo – SP Menção Honrosa
Pietro Kreitlon Carolino Salvador – BA Menção Honrosa
Ivo Almino Gondim Fortaleza – CE Menção Honrosa

EQUAÇÕES DIOFANTINAS
Antonio Caminha Muniz Neto

 Nível Intermediário

Denominaremos equação diofantina (em homenagem ao matemático


grego Diofanto de Alexandria) uma equação em números inteiros. Nosso objetivo
será estudar dois tipos particulares de equações diofantinas, a equação de
Pitágoras e a de Pell, e determinar suas soluções. Também estudaremos o método
da descida, que nos permitirá mostrar que algumas equações diofantinas não
possuem soluções não triviais, num sentido a ser precisado.

EUREKA! N°7, 2000


39
Sociedade Brasileira de Matemática

Ternos Pitagóricos
Queremos estudar as soluções (x, y, z) da equação x 2  y 2  z 2 , com x, y, z
inteiros não nulos. Após determinar tais soluções, vamos ver como podemos
utilizar as informações obtidas para resolver outras equações em números
inteiros. O resultado fundamental é o seguinte

Teorema 1: As soluções (x, y, z) da equação x 2  y 2  z 2 , com x, y, z inteiros não


nulos, são dadas por: ( x , y , z ) ( 2uvd , ( u 2  v 2 )d , ( u 2  v 2 )d )
ou ( x, y, z ) ((u 2  v 2 )d ,2uvd ,(u 2  v 2 )d ) onde d, u, v são inteiros não
nulos, com u  v, mdc(u, v) = 1 e u e v de paridades distintas.

Prova: Sejam x, y, z inteiros positivos quaisquer satisfazendo a equação acima


(os demais casos são análogos), e d o mdc de x e y. Então d 2 divide z 2 , e daí d
divide z. Existem portanto inteiros não nulos a, b, c, com mdc(a, b) = 1, tais que
(x, y, z) = (da, db, dc). Ademais, como
x 2  y 2  z 2  a 2  b2 c 2 ,
basta determinarmos as soluções (a, b, c) da equação, sujeitas à condição
mdc(a, b) = 1 (que por sua vez implica mdc(a, c) = 1 e mdc(b, c) = 1).
Note agora que, dado um inteiro qualquer t, temos que t 2 deixa resto 0 ou 1 na
divisão por 4, quando t for respectivamente par ou ímpar. Assim, se fossem a e b
ímpares, teríamos a 2 e b 2 deixando resto 1 na divisão por 4, e daí c 2  a 2  b 2
deixaria resto 2 quando dividido por 4, o que é um absurdo. Como a e b são
primos entre si, não podem ser ambos pares. Há então dois casos: a ímpar e b par,
a par e b ímpar. Analisemos o primeiro caso (o segundo é análogo).
Se a for ímpar e b par, então c também é ímpar. De a 2  b 2 c 2 obtemos
b 2 ( c  a )( c  a ) , e não é difícil concluir que mdc(c – a, c + a) = 2. Podemos
então escrever  b2  2  c2a  c2a  . Note que  c 2a  e  c 2a  são primos entre
si.
Mas se o produto de dois naturais primos entre si  c 2a e c a
2
 é um quadrado
perfeito, então cada um deles deve ser um quadrado perfeito. Existem então
inteiros positivos primos entre si u e v, tais que c  a 2 v 2 , c  a 2u 2 , e daí
( a, b, c ) ( u 2  v 2 , 2uv , u 2  v 2 ) .
Note ainda que, como u 2  v 2 c é ímpar, u e v devem ter paridades distintas.
Por substituição na equação original, concluímos que os ternos acima são
realmente soluções da equação, de modo que nada mais há a fazer. 
EUREKA! N°7, 2000
40
Sociedade Brasileira de Matemática

Vemos então que há uma quantidade infinita de ternos (x, y, z)


satisfazendo a equação acima. Por exemplo, fazendo d = v = 1 e u = 2n, n inteiro
positivo, obtemos o terno ( x , y , z ) ( 4n, 4n 2  1, 4n 2  1)
Um terno de inteiros positivos (x, y, z) tais que x 2  y 2  z 2 é denominado um
terno Pitagórico, em alusão ao matemático grego Pitágoras e seu famoso teorema
sobre triângulos retângulos. De fato, um tal terno (x, y, z) determina um triângulo
retângulo de catetos x e y e hipotenusa z inteiros.

z
x

y
Vejamos em que a equação acima pode ajudar na solução de outros problemas.
Consideremos a tarefa de determinar as soluções inteiras não nulas da equação
x 2  y 2  2 z 2 , com x  y. Em uma qualquer dessas soluções, devemos ter x e y
com a mesma paridade, pois caso contrário x 2  y 2 seria um número ímpar.
Assim, existem inteiros a e b tais que
x a  b, y a  b
Basta tomarmos a  12 ( x
 y ) e b  12 ( x  y ) , notando que x + y e x – y são
números pares. Substituindo as expressões acima para x e y na equação original,
concluímos que
x 2  y 2 2 z 2  a 2  b 2  z 2
Mas essa última equação é a nossa já conhecida equação de Pitágoras. Então, de
acordo com o teorema acima, podemos escrever
( a, b, z ) ( 2uvd , ( u 2  v 2 )d , ( u 2  v 2 ) d ) ou
( a , b, z ) (( u 2  v 2 )d , 2uvd , ( u 2  v 2 )d )
onde d, u, v são inteiros não nulos, com u  v, mdc(u, v) = 1 e u e v de paridades
distintas.
Segue daí que as soluções (x, y, z) de nossa equação são de um dos tipos abaixo,
onde d, u, v satisfazem as mesmas condições do teorema acima.
( x, y , z ) ( 2uvd  ( u 2  v 2 )d , 2uvd  ( u 2  v 2 )d , ( u 2  v 2 )d )
ou
( x, y, z ) ((u 2  v 2 )d  2uvd ,(u 2  v 2 )d  2uvd ,(u 2  v 2 )d )

EUREKA! N°7, 2000


41
Sociedade Brasileira de Matemática

Descida de Fermat e Equações sem Soluções


As equações analisadas acima são, em um certo sentido, privilegiadas, pois
possuem uma infinidade de soluções. Nosso próximo exemplo será o de uma
equação que só admite a solução inteira x = y = z = 0. Ela ilustra um método que
pode ser estendido a outras equações, a fim de provar que elas não possuem
soluções inteiras não nulas.

Exemplo 1: A equação 3x 2  y 2  2 z 2 não possui soluções inteiras não nulas.

Prova: Suponha o contrário. Então a equação possui uma solução (x, y, z) em


inteiros positivos. Então, dentre todas as soluções (x, y, z), com x, y e z inteiros
positivos, existe uma (x, y, z) = (a, b, c) para a qual z = c é o menor possível.
Trabalhemos tal solução.

Vamos usar o seguinte fato, que você pode provar facilmente: se um inteiro u não
for múltiplo de 3, então u 2 deixa resto 1, quando dividido por 3. Então, se b não
for múltiplo de 3, teremos de 3a 2  b 2 2c 2 que c também não será múltiplo de
3. Olhando os restos de cada termo da equação por 3, teremos que 3a 2  b 2
deixa resto 1 e 2c 2 deixa resto 2.1 = 2. Logo, não poderia ser 3a 2  b 2 2c 2 .
Assim, b deve ser múltiplo de 3, digamos b 3b1 . Daí vem que
3a 2  9b12  2c 2 , e c também é múltiplo de 3, digamos c 3c1 . Substituindo
na equação, chegamos a 3b12  a 2  6c12 .
Então, a também é múltiplo de 3. Sendo a 3a1 , a equação acima nos dá
b12  3a12  2c12 , e ( b1 , a1 , c1 ) é uma outra solução de nossa equação original,
com c1  c3  c . Mas isso é uma contradição, pois partimos de uma solução na
qual o valor de z era c, mínimo possível. Logo, nossa equação não possui
soluções não nulas. 

Esquematicamente, o método da descida (devido ao matemático francês Pierre


Simon de Fermat) consiste então no seguinte:

i. Supor que uma dada equação possui uma solução em inteiros não nulos.
ii. Concluir daí que ela possui uma solução em inteiros positivos que seja, em
algum sentido, mínima.
iii. Deduzir a existência de uma solução positiva menor que a mínima,
chegando a uma contradição.

EUREKA! N°7, 2000


42
Sociedade Brasileira de Matemática

Já que determinamos acima as soluções da equação de Pitágoras, nada mais


natural que tentar estudar a equação mais geral abaixo, denominada equação de
Fermat. Aqui, n > 2 é um inteiro fixado.
x n  y n z n ,
Por cerca de três séculos os matemáticos defrontaram-se com o problema de
decidir sobre a existência de soluções não nulas (x, y, z) dessa equação, problema
que somente foi resolvido na década de noventa, utilizando métodos muitíssimo
complexos.
Vamos aproveitar o método da descida para analisar um caso simples dessa
equação, aquele em que n é um múltiplo de 4. O leitor interessado em saber mais
sobre essa equação pode consultar uma das referências [2] ou [3] da bibliografia,
onde o caso n = 3 é discutido.

Teorema 2: Se n for múltiplo de 4 então não existem inteiros não nulos x, y, z tais
que x n  y n  z n .

Prova: Seja n = 4k, k natural. Se x n  y n  z n , então teremos


( x k ) 4  ( y k ) 4 ( z 2 k ) 2 , ou seja, ( x k , y k , z 2 k ) será uma solução da equação
a 4  b 4  c 2 . Assim, basta mostrarmos que essa última equação não admite
soluções não nulas. Por absurdo, suponhamos que existam inteiros positivos a, b,
c tais que a 4  b 4  c 2 . Podemos também supor que a, b e c foram escolhidos de
tal modo que não há outra solução positiva a ' , b' , c' com c '  c (aqui vamos
usar o método da descida). Então a e b são primos entre si, e o teorema 1 garante
a existência de inteiros positivos primos entre si u e v tais que
a 2  u 2  v 2 , b 2  2uv , c  u 2  v 2 . Como a 2  v 2  u 2 , segue novamente do
teorema 1 a existência de inteiros positivos primos entre si p e q tais que
2 2 2
a  p 2  q 2 , v  2 pq, u  p 2  q 2 . Mas aí b 2uv 4 pq( p  q )
Como p e q são primos entre si, temos que ambos são também primos com
p 2  q 2 . Portanto, sendo 4 pq( p 2  q 2 ) um quadrado devemos ter p, q e
p 2  q 2 quadrados, digamos p  2 , q  2 , p 2  q 2  2 , com , , 
positivos. Por fim, segue que  4   4  2 , com
2 2 2 2 2
c u  v  u  p  q  , contrariando a minimalidade de c. Logo, não há
soluções não nulas de x n  y n  z n quando n for múltiplo de 4. 

A Equação de Pell

EUREKA! N°7, 2000


43
Sociedade Brasileira de Matemática

Nem sempre é fácil, ou mesmo possível, determinar todas as soluções em inteiros


de uma dada equação. Por exemplo, para a equação x 2  2 y 2 1 , é bem mais
fácil mostrar que ela possui uma infinidade de soluções do que determinar todas
elas. Podemos gerar infinitas soluções dessa equação a partir de uma só solução
não nula.
Uma vez que a 2  2b 2 1 , teremos ( a  b 2 )( a  b 2 ) 1 , e daí
( a  b 2 )2 ( a  b 2 )2 1
Desenvolvendo os binômios, chegamos a

(a 2  2b 2  2ab 2 )(a 2  2b 2  2ab 2 ) 1 , e daí a a 2  2b 2  2
 2 2ab  1
2

 
Portanto, se (a, b) for uma solução, a 2  2b 2 ,2ab será outra solução. Sendo
a e b positivos, temos a  a 2  2b 2 , e desse modo determinamos uma infinidade
de soluções da equação (contanto que tenhamos uma solução não nula). Veja que
(3, 2) é uma solução não nula de nossa equação.
É fácil ver que o método acima utilizado também garante que, quando d for um
2 2
inteiro tal que d é irracional, a equação x  dy 1 admite infinitas
soluções não nulas, desde que admita uma solução não nula. Também, com
poucas modificações podemos tratar a equação x 2  dy 2  1 (veja o exercício
6).
Observe que, apesar de determinarmos facilmente infinitas soluções da equação
acima, não sabemos se há outras. Vamos agora começar a responder essa
pergunta, para uma classe mais ampla de equações.

Definição 1 (Equação de Pell): Seja d um inteiro positivo que não seja um


quadrado. Nesse caso, sabemos que d é irracional. Chamamos equação de
Pell à equação x 2  dy 2  m , onde m é um inteiro qualquer.
É claro que no caso m = 0 a equação não admite soluções além da trivial
x = y = 0, pois se esse fosse o caso teríamos x e y não nulos, e daí d  xy , um
racional.
x
Lema 1: Seja  um irracional qualquer. Existem infinitos racionais y , com x e
x 1
y inteiros não nulos primos entre si, tais que y   
y2
.
Prova: Seja n > 1 um natural qualquer, e considere os números j , com
j = 0, 1, ..., n. Seja { j }  j  [ j ]  [0,1). Como
   
0, 1  0, 1n   1n , 2n   ...  nn 1 , 1 ,
segue do princípio de Dirichlet que existem 0 k  j  n tais que { j} e {k}
pertencem a um mesmo intervalo dos que aparecem no lado direito da igualdade

EUREKA! N°7, 2000


44
Sociedade Brasileira de Matemática

acima. Então | { j} - {k} | < 1


n Daí,  j  k      j   k   1n , e segue que


j  k
   j  k    ( j 1k ) n  1
( j  k )2

Existe então um par (x, y) de inteiros, x   j   k , y  j  k  n , tais que


x1 1 1
x
   1
. Se x  dx1 , y  dy1 , com d > 1, então y1     , de
y2 y12
y y2

modo que podemos supor que x e y são primos entre si.


Para garantirmos a existência de infinitos tais pares, suponha que achamos x e y
x 1
primos entre si e tais que y   
y2
. Escolha agora um natural n tal que
1 x
n  y   . Repetindo o argumento acima, chegamos a um par de inteiros primos
x1
entre si x1 , y1 , com y1    ny
1
1
e y1 n . Portanto,

x1 x1
y1    ny
1
1
 x
y  e y1    ny11  y12 ,
1

donde ( x1 , y1 ) ( x , y ) satisfaz o lema. 

Lema 2: Seja d um inteiro positivo que não seja um quadrado. Existe um inteiro
m para o qual a equação x 2  dy 2  m admite infinitas soluções inteiras.

Prova: Sabemos que d é irracional. Assim, o conjunto S dos pares (x, y) de


x 1
inteiros primos entre si tais que y  d  é infinito. Mas se x e y forem
y2

inteiros satisfazendo essa desigualdade, então

x 2  dy 2  x  dy x d y  1y x   d y 2 d y  1y   1
y 2 
d y 2
Segue que algum inteiro não nulo m entre  ( 2 d  1) e 2 d  1 se repete um
número infinito de vezes entre os valores de x 2  dy 2 , com (x, y) em S. Mas isto
2 2
é o mesmo que dizer que a equação x  dy  m admite infinitas soluções. 

Teorema 3 (Soluções da Equação de Pell): Seja d um inteiro positivo que não


seja um quadrado. A equação x 2  dy 2 1 admite infinitas soluções em inteiros
positivos x, y. Ademais, existe uma solução em inteiros positivos x1 , y1 tal que
todas as demais soluções dessa equação são da forma x n  y n d ( x1  y1 d ) n
, onde n é um número natural.

EUREKA! N°7, 2000


45
Sociedade Brasileira de Matemática

Prova: Admitamos por enquanto que nossa equação tenha uma solução em
inteiros positivos x, y. Dentre todas essas soluções, escolha aquela x1 , y1 tal que
  x1  y1 d seja o menor possível.
Dado um natural qualquer n, sabemos que existem inteiros positivos x n , y n tais
que ( x1  y1 d ) n  x n  y n d . Daí, sabemos que
( x1  y1 d ) n  x n  y n d , e assim
1 ( x12  dy12 ) n ( x1  y1 d ) n ( x1  y1 d ) n 
( x n  y n d )( x n  y n d )  x n2  dy n2
Então todos os pares ( x n , y n ) são soluções da equação.
Seja agora (x, y) uma solução qualquer em inteiros positivos. Para terminar, basta
mostrarmos que existe um natural n tal que x  y d  n . Suponha o contrário.
Então existe um natural n tal que  n  x  y d   n1 . Daí, vem que
1    n ( x  y d )   . Mas
  n ( x  y d ) ( x1  y1 d )  n ( x  y d ) ( x n  y n d )  1 ( x  y d ) 
( x n  y n d )( x  y d ) ( xx n  dyy n )  ( x n y  y n x ) d
e ocorre que
( xx n  dyy n ) 2  d ( x n y  y n x )2  x n2 ( x 2  dy 2 )  y n2 ( dy 2  x 2 )  x n2  dy n2 1 ,
de modo que   n ( x  y d ) ( xx n  dyy n , x n y  y n x ) também é solução.
Como 1    n ( x  y d )   , basta mostrarmos que xx n  dyy n , x n y  y n x  0
para chegarmos numa contradição. Sejam a  xx n  dyy n , b  x n y  y n x .
Temos a  b d  0 e a 2  db 2 1 , donde a  b d ( a  b d )  1  0 .
Então, 2a ( a  b d )  ( a  b d )  0 . Por outro lado, a  b d  1 implica
a  b d ( a  b d )  1  1 , e daí b d  a  1 0 . Logo, b > 0.
Para terminar, basta mostrarmos que a equação x 2  dy 2 1 admite uma
solução. Tome, de acordo com o lema 2, um inteiro (não nulo) m tal que
x 2  dy 2  m admita uma infinidade de soluções. Podemos escolher duas dessas
soluções, ( x1 , y1 ), ( x 2 , y 2 ) digamos, tais que | x1 | | x 2 | mas x1  x 2 e y1  y2
, módulo m. Então
( x1  y1 d )( x 2  y 2 d ) ( x1 x 2  dy1 y 2 )  ( x 2 y1  x1 y 2 ) d (*)
Mas x1 x 2  dy1 y 2  x12  dy12  0 (mod m) e x 2 y1  x1 y 2 (mod m) , donde
existem inteiros u e v tais que x1 x 2  dy1 y 2  mu, x 2 y1  x1 y 2  mv
Segue de (*) que ( x1  y1 d )( x 2  y 2 d )  m( u  v d ) , e daí

EUREKA! N°7, 2000


46
Sociedade Brasileira de Matemática

( x1  y1 d )( x 2  y 2 d )  m( u  v d ) .
Multiplicando ordenadamente essas duas igualdades, chegamos a
m2 ( x12  dy12 )( x22  dy 22 )  m2 ( u 2  dv 2 ) ,
ou seja, u 2  dv 2 1 . Resta mostrarmos que u e v são não nulos. Se u = 0
teríamos  dv 2 1 , um absurdo. Se v = 0, viria u = 1 ou –1. De (*) seguiria que
( x1  y1 d )( x 2  y 2 d ) m , e assim ( x1  y1 d ) ( x 2  y 2 d ) , donde
por fim | x1 | | x 2 | , o que é um absurdo. 

Exemplo 2: Agora podemos determinar todas as soluções inteiras não nulas da


equação x 2  2 y 2 1 . O teorema 3 ensina que as soluções positivas dessa
equação são da forma ( x n , y n ) , onde x n e y n são os únicos inteiros para os
quais x n  y n 2 ( x1  y1 2 ) n , sendo ( x1 , y1 ) a solução positiva para a qual
x1  y1 2 é o menor possível. Como os pares (x, y) = (1, 1), (1, 2), (2, 1), (2, 2),
(2, 3) não são soluções da equação e (3, 2) é, é fácil nos convencermos de que
( x1 , y1 ) = (3, 2). Desse modo, temos os pares ( x n , y n ) dados pela igualdade
x n  y n 2 (3  2 2 ) n
Determine agora as demais soluções não nulas da equação acima. O exercício 7
discute mais alguns aspectos dessa equação.

Exercícios:
1. Seguindo os passos da prova do teorema 1, mostre que as soluções em
inteiros não nulos da equação x2  2 y2 z2 são da forma
2 2 2 2
x ( u  2 v )d , y  2uvd , z ( u  2 v )d , onde d, u, v são inteiros
não nulos, com u e 2v primos entre si.
2. Mostre que as equações a seguir não possuem soluções inteiras não nulas:
i. x 4  4 y 4  z 2
ii. x 4  2 y 4  z 2
iii. x 2  y 2  3z 2
O item i do exercício a seguir tem a ver com o exemplo 1 do texto.
3. i. Mostre que não existem racionais x e y tais que x 2  xy  y 2  2 .
ii. Determine todas as soluções racionais da equação x 2  xy  y 2 1 .
Para resolver os próximos dois exercícios utilizamos o teorema 1. Eles são mais
difíceis que os anteriores, e no primeiro deles você pode achar útil o seguinte
resultado, conhecido como Teorema de Ptolomeu: dado um quadrilátero convexo
inscritível ABCD, tem-se

EUREKA! N°7, 2000


47
Sociedade Brasileira de Matemática

AB. CD  AD. BC  AC. BD

D
B

C
Para uma prova do Teorema de Ptolomeu, você pode consultar a referência [4].
4. Temos no plano uma circunferência de raio 1. Mostre que podemos
escolher em tal circunferência 2000 pontos A1 , A2 ,..., A2000 tais que Ai A j é
racional, quaisquer que sejam 1 i  j 2000 .
5. Seja r um inteiro positivo dado. Queremos determinar o número de
triângulos ABC, dois a dois não congruentes, satisfazendo as seguintes condições:
i. O raio da circunferência inscrita em ABC mede r.
ii. Os comprimentos dos lados de ABC são números inteiros, primos entre si.
Mostre que o número de tais triângulos é 2 k , onde k é o número de fatores
primos distintos de r.
6. Prove, sem apelar para o teorema 2, que a equação x 2  2 y 2  1
admite uma infinidade de soluções inteiras.
7. Prove que as soluções positivas ( x n , y n ) da equação do exemplo 2 são
dadas pelas seqüências ( x1 , y1 ) (3, 2) e x n 1 3x n  4 y n ,
y n1 2 x n  3 y n
8. Prove que há infinitos inteiros n tais que n 2  ( n  1) 2 seja quadrado.

Bibliografia

[1] Introdução à Teoria dos Números. Plínio O. dos Santos. Coleção Matemática Universitária.
IMPA. 1999.
[2] An Introduction to the Theory of Numbers. I. Niven, H. Zuckermann. John Wiley & Sons.
New York. 1980.
[3] A Classical Introduction to Modern Number Theory. K. Ireland & M. Rosen. Springer-Verlag.

EUREKA! N°7, 2000


48
Sociedade Brasileira de Matemática

New York. 1990.


[4] Quadriláteros e Triângulos. M. Mendes. Eureka! No5. OBM 1999
SOLUÇÕES DE PROBLEMAS PROPOSTOS
 Publicamos aqui algumas das respostas enviadas por nossos leitores.
29) Seja n > 1 um número inteiro. Existem n lâmpadas L0, L1, ... , Ln–1
colocadas em um círculo. Cada lâmpada está ACESA ou APAGADA.
Uma seqüência de passos S0, S1, ... , Si, ... é executada. O passo Sj afeta
apenas o estado da lâmpada Lj (deixando o estado de todas as outras
inalterado) da seguinte forma:

Se Lj–1 está ACESA, Sj muda o estado de Lj de ACESA para APAGADA, ou de


APAGADA para ACESA;

Se Lj–1 está APAGADA, Sj deixa o estado de Lj inalterado.

As lâmpadas são rotuladas mod n, ou seja, L – 1 = Ln –1, L0 = Ln, L1 = Ln + 1, etc.

Inicialmente todas as lâmpadas estão ACESAS. Mostre que:

a) Existe um inteiro positivo M(n) tal que depois de M(n) passos todas as
lâmpadas estão ACESAS de novo;
b) Se n é da forma 2k então todas as lâmpadas estão ACESAS depois de
n2 –1 passos;

c) Se n tem a forma 2k + 1 então todas as lâmpadas estão ACESAS depois


de n2 – n + 1 passos.

Solução de Frank Castro (São Paulo - SP):

a) Vamos inicialmente representar o estado das lâmpadas L0, L1, L2, ..., Ln–1 por
uma n-upla u = (u0, u1, u2, ..., un–1), onde ui = 0 se Li está apagada e ui = 1 se Li
está acesa.
Evidentemente o estado inicial das lâmpadas é dado pela n-upla e = (1, 1, 1, ...,1).
Nessas condições a operação Sj tranforma a n-upla (u0, u1, u2, ..., un–1) na nova
n-upla (u0, u1,..., uj–1, uj–1 + uj,..., uj+1,..., un–1), onde a soma uj–1 + uj é tomada
módulo 2 ( e j é tomada módulo n).
Assim sendo, nosso problema consiste em determinar um valor natural r, tal que:
S r ( S r  1 (...( S1 ( S 0 (e)))...)) e

EUREKA! N°7, 2000


49
Sociedade Brasileira de Matemática

Para tanto, denotemos por Rj a operação que transforma a n-upla (u0, u1,..., uj–1, uj,
uj+1,..., un–1) na n-upla (u j mod n , u ( j 1) mod n ,...u ( j  n  1) mod n ). Observe que R–j é a
operação inversa de Rj e R0 deixa a n-upla inalterada.
Nesses termos temos para uma n-upla qualquer S j  R j ( S 0 ( R j )) .
__
Agora para uma n-upla qualquer u podemos escrever:
__ __
S r ( S r  1 ...( S1 ( S 0 ( u )))...) S r S r  1 ...S1 S 0 ( u ) ( R r S 0 Rr )( R r 1 S 0 Rr  1 )...
__ __ __
...( R 1 S 0 R1 ) S 0 ( u ) R r S 0 ( RS 0 ) r ( u ) R  r  1 ( RS 0 ) r 1 ( u ) onde R R1 .
Consequentemente, S r S r  1 ...S1 S 0 (e) e  ( RS 0 ) r 1 (e)  Rr 1 (e) e como
existe apenas um número finito de estados das lâmpadas (2 n precisamente) que
equivale ao número total de n-uplas, em algum estágio a seqüência
e, ( RS 0 )1 (e), ( RS 0 ) 2 (e),... deve repetir algum de seus elementos. Nessas
condições para algum m e n ( m < n ) teremos : ( RS 0 ) m (e) ( RS 0 ) n (e). Sendo
RS 0 uma bijeção (verifique!) temos ( RS 0 ) n  m (e) e, o que conclui o ítem a.

b) Primeiramente associaremos à n-upla u = (u0, u1, u2, ..., un–1) o polinômio P(x)
2 n 2 n 1
da forma : P( x) u n  2  u n  3 x  u n  4 x  ...  u 0 x  u n  1 x , onde os
coeficientes serão olhados módulo 2 (isto é, P(x)  Z / 2Z [ x ]).
Chamemos tal polinômio de polinômio de posição. Observe agora que para a
n-upla ( RS 0 )1 (u ) temos:
( RS 0 )1 (u ) R1 ( S 0 (u )) R1 (u n  1  u 0 , u1 , u 2 ,..., u n  1 ) (u1 , u 2 ,..., u n  1 , u n  1  u 0 )
e seu polinômio de posição é dado por
Q( x) u n  1  u n  2 x  u n  3 x 2  ...  u1 x n  2  (u n  1  u 0 ) x n  1 Não se esqueça
que a adição u n 1  u0 é tomada módulo 2. Assim,
Q ( x) x P ( x) mod( x n  x n  1  1). Assim sendo, queremos encontrar r tal
que x r 1 mod( x n  x n  1  1). Note que quando x r 1 mod( x n  x n  1  1).
o polinômio de posição associado a ( RS 0 ) r será congruente a P(x) que
representará a n-upla e = (1, 1, 1,...1) no estado inicial das lâmpadas. Suponha
agora n = 2k.
2
Então x n ( x n ) n ( x n  1  1) n x n( n  1)  1 mod( x n  x n  1  1) pois se n é uma
potência de 2, todos os coeficientes, exceto o primeiro e o último da expansão
binomial ( x n  1  1) n são pares, logo congruentes a zero módulo 2.
Finalmente temos:

EUREKA! N°7, 2000


50
Sociedade Brasileira de Matemática

2 2 2
x n  x n ( n  1) 1 mod( x n  x n  1  1)  x n  x n n
1 mod( x n  x n  1  1) 
2 2
xn n
( x n  1) 1 mod( x n  x n  1  1)  x n 1
1 mod( x n  x n  1  1)
(pois x n  1 x n  1 (mod x n  x n  1  1)).
Assim, após ( n 2  1) etapas todas as lâmpadas estarão acesas novamente.
c) Suponha n 2 k  1
2
Assim x n  1 ( x n 1 ) n  1 ( x n  x) n  1 x n( n  1)  x n  1 onde todas as congruências
foram tomadas mod( x n  x n  1  1). Como no ítem anterior n – 1 é potência de
2, logo todos os coeficientes, exceto o primeiro e o último da expansão binômial
( x n  x ) n  1 são pares, consequentemente congruentes a zero módulo 2.
Finalmente temos:
2 2
x n  1  x n( n  1) x n  1 mod( x n  x n  1  1)  x n  n ( x n  1  1)  x n  1 mod( x n  x n  1  1) 
2 2
(*) x n n
( x n )  x n  1 mod( x n  x n  1  1)  x n  x n  1 mod( x n  x n  1  1) 
2
 xn  n 1
1 mod( x n  x n  1  1). Observe que, como estamos trabalhando
módulo 2, x n  x n  1  1  x n  1  1 mod(x n  x n  1  1), e isso justifica a
congruência (*). Assim sendo, após n 2  n  1 etapas, todas as lâmpadas estarão
acesas novamente.

30) Determine todas as funções f : R  R que satisfazem as condições:


 1  f ( x)
(i) f(– x) = – f(x), (ii) f(x + 1) = f(x) + 1, (iii) f    2 para x  0.
 x x

Solução de Carlos Alberto da Silva Victor (Rio de Janeiro - RJ):

Do ítem (ii) : f(x + 1) = f(x) temos que f(x) = 1 + f( x – 1) e usando (i) 


1  1
f(x) + f (1 – x) = 1. Sejam x  0 e x  1, logo: f    f 1   1 e usando (iii) 
 x  x
 
1  1 
f ( x)  f   1 e usando novamente (iii) 
x2  x 
 x  1
1 1  x 
f ( x)  2
f   1
x2  x   x  1 
 
 x  1

EUREKA! N°7, 2000


51
Sociedade Brasileira de Matemática

1 ( x  1) 2  1 
2
f ( x )  2
f 1   1 
x x  x  1
1 ( x  1) 2  1 
2
f ( x )  2 1 2
f ( x  1) 1 
x x  ( x  1) 
1 ( x  1) 2 1
 f ( x )   2 f ( x  1) 1
x2 x2 x
f ( x)  ( x  1) 2  f ( x  1)  x 2  f ( x)  f ( x  1) 2 x  1

 f (x)  f (1  x) 1
donde:
  f ( x)  x
 f (x)  f (1 x) 2x  1
para x 0  f (1)  f (0)  1

 f (1)  f (0) 1
x  1  f (0)  f ( 1)  1  f (1)  1   f (1) 1 e f (0) 0.
 f (0)  f (1) 1
Conclusão: x  R, teremos f(x) = x como sendo a única solução.

31) Seja x1, x2, x3, … uma seqüência de números reais não negativos satisfazendo
x x
x n  n  2 n  1 para n = 3, 4, 5, … Estabeleça condições necessárias e
2 x n 2  x n 1
suficientes em x1 e x2 para xn ser inteiro para infinitos valores de n.

Solução de Davi Máximo Alexandrino Nogueira (Fortaleza - CE):

Afirmação: x1  x 2 , x1 e x 2 inteiros.

xn 2 xn 1
Prova: Se, x n  teremos,
2 xn 2  xn 1

EUREKA! N°7, 2000


52
Sociedade Brasileira de Matemática

1 2 x  xn 1 1 2 1
xn (2 x n 2  xn 1 )  xn 2 xn 1   n 2    
xn xn 2 xn 1 xn xn 1 xn 2
1 1 1 1 1
    tome y n  logo, a seqüência y1, y2, …, é uma
xn x n 1 x n 1 x n 2 xn
x1  x 2
P.A., de razão r desse modo,
x1 x 2
yn  y1  (n  1)r  x1  xn  (n  1)rx1 xn 
x1
 x1  xn (1  (n  1)rx1 )  x1  xn  . Suponha r  0.
1  (n  1) x1r
x1  x 2 x1 x 2
Como r  , temos x n  fazendo x1 – x2 = a,
x1 x 2 x 2  (n  1)( x1  x 2 )
teremos
x1 x 2
xn  . Porém, para algum k, tal que x1 x 2  x 2  (k  1) a
x 2  (n  1)a
teremos x n  1 , para todo n  k. Logo, devemos ter r = 0, o que conclui a
demonstração.

32) a) Prove que todo número inteiro não nulo m admite uma única representação
n 1
da forma m  
k 0
k 3 k , onde n é um inteiro positivo e  k  { 1,0,1} para

todo k, com  n  1 0.


3n  1 V {P0 , P1 ,..., P3n  1 }
Dado um conjunto de pontos , escrevemos em
2 2

cada aresta que une dois desses pontos Pi e Pj (i  j) um número


pertencente a
n 1
{0, 1, …, n – 1} da seguinte forma: escreveremos i  j  
k 0
k 3 k , com

 k  { 1,0,1}, k , e associamos à aresta Pi Pj o número


m min{k 0  k 1}.
Prove que não existe nenhum triângulo cujos vértices pertençam a V com o
mesmo número escrito em seus três lados.

EUREKA! N°7, 2000


53
Sociedade Brasileira de Matemática

Solução de Carlos Alberto da Silva Victor (Rio de Janeiro - RJ):

a) Sabendo que m  0 (mod 3) ; m  1 (mod 3) ou m  – 1 (mod 3),


teremos:
m = 3k0 + r0
k0 = 3k1 +r1
k1 = 3k2 + r2



kn–2 = 3kn–1 + rn–1
com kn–1 = 0 e rn–1  0, onde ri {–1, 0, 1} (note que os ri estão unicamente
determinados, e existe n com kn–1 = 0 pois m  k0  k1  ...  ki enquanto
tivermos k i  1  0. ) Substituindo na primeira igualdade k0 pela sua igualdade,
obteremos m como função de k1, r0. Tomando novamente k1 = 3 k2 + r2 e fazendo
as substituições sucesivas de k2, k3, ..., kn-2 ; obtemos:
m 3 n k n  1  rn  1 3 n  1  rn  2 3 n 2  ...  r2 3 2  r1 31  r0 3 0 e já que
n 1
k n  1 0 , teremos m  r k 3 k com rn  1 0.
k 0

b) Sejam Pi , Pj e Ps três pontos quaisquer de V e por hipótese i < j < s;


n 1 t 1 ~
portanto podemos escrever: j  i   k 3 k e s  i   p 3 p , com
k 0 p 0

~ ~
 k ,  p  { 1,0,1};  n  1 1 e  t-1 1.
Vamos também fazer a hipótese de que para as arestas Pi Pj e Pi Ps ,
~
tenhamos o
número  min{k 0 /  k 1} min{ p 0 /  p 1} onde
0   n  1 e 0   t  1. Podemos então escrever:
 1 n 1
 k k


j  i  
k 0
 k .3  
k  1
 k . 3  1. 3 
(1)
  1 t 1
 s  i   .3 p   3 p  1.3 (2)
 p0 p p 1 pk.

EUREKA! N°7, 2000


54
Sociedade Brasileira de Matemática

De (2) – (1) obtemos


1 ~ t 1 ~ 1 n 1
 
s  j    p .3 p    p .3 p     k .3k    k .3k 
p 0 p  1  k 0 k  1 
~ ~
Para 0 k   1,  k e  k pertencem a {–1, 0}, donde  k   k  { 1,0,1},
e os somatórios com p   1 e k   1 são múltiplos de 3  1 , e portanto,
'
ao escrever s  j    3 , com    { 1,0,1}, não aparece o termo 13  ,

'
 0
'
o que garante que o número associado à aresta Pj Ps, que é min{ 0 /   1}
em ( s  j ) não será igual a .
Conclusão: não existe triângulo cujos vértices pertençam a V com o mesmo
número escrito em seus três lados.
33) Na parede interna de um vaso cilíndrico de cristal existe uma gota de mel
num ponto B situado a três centímetros do seu bordo superior. Na parede externa,
num ponto A diametralmente oposto ao da gota, está uma formiga. Sabendo que a
altura do vaso é de 20cm e o seu diâmetro é 10cm. Indicar o caminho mais curto
para que a formiga atinja a gota de mel.

Solução de Daniel Pessôa Martins Cunha (Fortaleza - CE):

Cobrindo o vaso com papel por dentro e por fora, e marcando nele a localização da
formiga, da gota de mel e da borda, poderemos ver que ao desamassar o papel ficarão as
seguintes impressões, com as seguintes medidas:
B Gota

Parte de dentro

3 cm

 r = 5

Borda A' B'


5 /2 5 /2
Parte de fora

3 cm

A Formiga

EUREKA! N°7, 2000


55
Sociedade Brasileira de Matemática

Como a menor distância entre 2 pontos é a medida do segmento que os une, o


____
menor caminho é o segmento AB de medida : 36  25 2 (Teorema de
Pitágoras).
Ao colocar o papel de volta ao vaso veremos o menor caminho a ser percorrido

pela formiga. Que é subir em diagonal até o ponto médio do arco A' B ' ,
determinado pelo diâmetro na borda. Depois descer em diagonal até a gota de
mel.

34) ABC é um triângulo, tal que AB = c, AC = b e BC = a.


Por um ponto interior P deste triângulo, são traçadas paralelas aos seus lados.
Sabe-se que as intersecções, da paralela ao lado de medida a, com os lados deste
triângulo, determinam um segmento de medida a'.
Analogamente, as paralelas aos lados de medidas b e c, determinam com os lados
do triângulo, segmentos de medidas b' e c' respectivamente.
a ' b' c '
Nestas condições demonstre que   2.
a b c

Solução de Francisco Antonio Martins de Paiva (Fortaleza - CE):

De acordo com o problema temos:


A

t
H
s
G

s
t
P
I F
y x

w z z
w

y
B
D E C

EUREKA! N°7, 2000


56
Sociedade Brasileira de Matemática

Como as retas traçadas são paralelas aos lados então os quadriláteros PFCE,
PIBD, PHAG são paralelogramos, e com isso concluímos que seus lados opostos
são congruentes. Daí temos:
BC = a, AC = b, AB = c, IF = x + y = a' , GD = t + w = c', HE = s + z = b'
___ ____
Os triângulos ABC e AIF são semelhantes pois IF // BC , de onde temos:

xy c w x y b z
 e 
a c a b

a' w a' z
1  1 
a c a b
___ ____
Os triângulos ACB e GCD são semelhantes pois AB// GD , de donde temos:

t w b s t w a y
 e 
c b c a
c' s c' y
1  1 
c b c a
___ ____
Os triângulos ACB e HEB são semelhantes pois AC// HE , de onde temos:

sz a x sz c t
 e 
b a b c

b' x b' t
1  1 
b a b c

Daí temos que:

 a ' b' c'  x y z s t w


2    6      
a b c a a b b c c
 a ' b ' c '  x  y zs tw
2    6   
a b c a b c
 a ' b' c '  a ' b' c '
2    6   
a b c a b c
 a ' b' c ' 
3    6
a b c

EUREKA! N°7, 2000


57
Sociedade Brasileira de Matemática

a ' b' c '


  2.
a b c

35) Sabendo que num triângulo ABC a altura relativa ao vértice A mede 12cm. e a
altura relativa ao vértice B mede 20cm, determine todos os valores possíveis para
a altura relativa ao vértice C.

Solução de Frank Castro (São Paulo - SP):

Temos: ha = 12, hb = 20.


Sendo a, b e c os lados do triângulo e S sua área, valem as seguintes relações:
c  b a (I)
2 S 2 S 2 S
a , b e c Substituindo as três últimas igualdades em (I)
ha hb hc

Vem que:
1 1 1 1 1 1
     , assim hc = 30. Agora, sabemos que: a + b > c
hc hb ha 20 12 30
2 S 2 S 2 S
a ,b e c . Substituindo temos:
ha hb hc
1 1 1 1 1 1
      hc  7,5
ha hb hc 12 20 hc
Resposta: 7,5cm < hc < 30 cm.

Agradecemos também o envio das soluções a: Ricardo Klein Hoffmann (Porto


Alegre - RS), Geraldo Perlino Júnior (São Paulo - SP), José Heleno Faro (Cachoeiro
de Itapemirim - ES).

EUREKA! N°7, 2000


58
Sociedade Brasileira de Matemática

Você sabia…
Que há novos records de primos grandes descobertos em
2000?

Maior par de primos gêmeos conhecido:


2409110779845260000±1.
Esses primos têm 18075 dígitos, e foram descobertos por
Wassing, Járai e Indlekofer.

Maior primo de Fermat generalizado conhecido: 16717632768 +


1, que tem 171153 dígitos e foi descoberto por Yves Gallot
(este é o oitavo maior primo conhecido atualmente, e maior
primo conhecido que não é de Mersenne).

PROBLEMAS PROPOSTOS

 Convidamos o leitor a enviar soluções dos problemas propostos e sugestões de novos


problemas para os próximos números.

36) Na figura abaixo o triângulo DEF tem área de medida S. Sabendo-se que o
triângulo DEF está inscrito num triângulo arbitrário ABC, mostre que as
medidas Si ( i = 1, 2, 3) das áreas dos outros triângulos formados satisfazem a
3
S
desigualdade 1 1 1 e que a igualdade ocorre se e só se os pontos
 
S1 S 2 S 3
DEF são os pontos médios dos lados do triângulo, ABC.
A

S1
F
E
S
S2
S3
B C
D

37) Cinco quadrados são dispostos conforme ilustra o diagrama abaixo.


Mostre que a medida da área do quadrado S é igual a medida da área do
triângulo T.

EUREKA! N°7, 2000


59
Sociedade Brasileira de Matemática

38) Os lados e diagonais de um polígono regular de n lados são coloridos em k


cores tais que:
i) para cada cor a e dois vértices A e B do polígono, o segmento AB é colorido
de a ou existe um vértice C tal que AC e BC são coloridos de a.

ii) os lados de qualquer triângulo com vértices entre os vértices do polígono são
coloridos usando no máximo 2 cores.
Prove que k  2.

39) Sejam x, y e z os ângulos de um triângulo de lados opostos a, b e c


respectivamente. Prove que,
 1 1 1 1 1 1 a b c
a    b    c   2   .
 y z  z x  x y  x y z
40) a) Calcular a soma dos divisores positivos de um número natural em termos
de sua fatoração prima.
b) Dizemos que n  1 é abundante se a soma de seus divisores é maior que 2n.
Prove que se n é abundante então kn é abundante para todo inteiro k  1.
c) Prove que existe n0  N tal que todo inteiro n  n0 pode ser escrito como
soma de dois números abundantes.

Problemas 36 e 37 propostos por Carlos Alexandre Gomes da Silva (Natal - RN),


problema 38 proposto na Olimpíada Búlgara - 1998, problema 39 proposto por Aldo
Trajano Louredo, problema 40 proposto por Gleydson Chaves Ricarte (Fortaleza -
CE) e Zoroastro Azambuja Neto (Rio de Janeiro - RJ).

Errata:

Eureka!No. 6, pág 40: O enunciado do problema 4 deve dizer:

EUREKA! N°7, 2000


60
Sociedade Brasileira de Matemática

Problema 4: Mostre que há infinitos naturais n tais que n 2  1 divide n!, onde
n! = n  (n–1)..21 (por exemplo, 4! = 4  3  2  1 = 24).

Eureka! No. 6, pág 27: o segundo parágrafo está truncado. A versão correta é:
Determinar exatamente os valores de números de Ramsey clássicos R(a, b) é, em
geral, um problema computacionalmente muito difícil.
Os únicos valores de R(a, b) com 3  a  b que são conhecidos são: R(3, 3) = 6,
R(3, 4) = 9, R(3, 5) = 14, R (3, 6) = 18, R(3, 7) = 23, R(3, 8) = 28, R (3, 9) = 36,
R(4, 4) = 18, e R(4, 5) = 25.
O único número Ramsey com mais de duas cores cujo valor é conhecido é
R(3, 3, 3) = 17.

AGENDA OLÍMPICA

XI OLIMPÍADA DO CONE SUL


14 a 19 de abril de 2000
Montevideu – Uruguai

VI OLIMPÍADA DE MAIO
13 de maio de 2000

XXII OLIMPÍADA BRASILEIRA DE MATEMÁTICA


Primeira Fase – Sábado, 10 de junho
Segunda Fase – Sábado, 02 de setembro
Terceira Fase – Sábado, 21 de outubro (níveis 1,2 e 3)
Domingo, 22 de outubro (nível 3 - segundo dia).

XLI OLIMPÍADA INTERNACIONAL DE MATEMÁTICA


13 a 25 de julho
Taejon, Coreia do Sul.

EUREKA! N°7, 2000


61
Sociedade Brasileira de Matemática

XV OLIMPÍADA IBEROAMERICANA DE MATEMÁTICA


16 a 24 de setembro de 1998
Caracas, Venezuela

III OLIMPÍADA IBEROAMERICANA DE MATEMÁTICA UNIVERSITÁRIA


outubro de 2000

COORDENADORES REGIONAIS

Amarisio da Silva Araújo (UFV) Viçosa - MG


Alberto Hassen Raad (UFJF) Juiz de Fora - MG
Angela Camargo (Centro de Educ.de Adultos - CEA) Blumenau - SC
Benedito T. Vasconcelos Freire (UFRN) Natal - RN
Claudio Arconcher (Col. Leonardo da Vinci) Jundiaí - SP
Claus Haetinger (UNIVATES) Lajeado - RS
Crescêncio das Neves (UFAM) Manaus-AM
Élio Mega (Col. ETAPA) São Paulo - SP
Enzo Marcom Takara (Col. Singular) Santo André - SP
Flávia Jerônimo Barbosa (UFPB Campus I) João Pessoa - PB
Florêncio F. Guimarães Filho (UFES) Vitória - ES
Francisco Dutenhefner (UFMG) Belo Horizonte - MG
Gisele de A. Prateado Gusmão (UFGO) Goiânia - GO
Ivanilde H. Fernandes Saad (U. Católica Dom Bosco) Campo Grande - MS
João Benício de Melo Neto (UFPI) Teresina - PI
João F. Melo Libonati (Grupo Educ. IDEAL) Belém - PA
Jorge Ferreira (UEM) Maringá - PR
José Carlos Pinto Leivas (UFRG) Rio Grande - RS
José Cloves Saraiva (UFMA) São Luis - MA
José Gaspar Ruas Filho (ICMC-USP) São Carlos - SP
José Luis Rosas Pinho (UFSC) Florianópolis - SC
José Paulo Carneiro (Univ. Santa Úrsula) Rio de Janeiro - RJ
José Vieira Alves (UFPB) Campina Grande - PB
Leonardo Matteo D'orio (Sistema Titular de Ensino)Belém - PA
Licio Hernandes Bezerra (UFSC) Florianópolis - SC

EUREKA! N°7, 2000


62
Sociedade Brasileira de Matemática

Luzinalva M. de Amorim (UFBA) Salvador - BA


Marcondes Cavalcante França (UF Ceará) Fortaleza - CE
Pablo Rodrigo Ganassim (L. Albert Einstein) Piracicaba - SP
Paulo H. Cruz Neiva de L. Jr. (Esc. Tec.Everardo Passos) SJ dos Campos - SP
Reinaldo Gen Ichiro Arakaki (INPE) SJ dos Campos - SP
Ricardo Amorim (Centro Educ. Logos) Nova Iguaçu - RJ
Roberto Vizeu Barros (Colégio ACAE) Volta Redonda - RJ
Sergio Claudio Ramos (IM-UFRGS) Porto Alegre - RS
Seme Gebara Neto (UFMG) Belo Horizonte - MG
Silvio de Barros Melo (UFPE) Recife - PE
Tadeu Ferreira Gomes (U. do Estado da Bahia) Juazeiro - BA
Tomás Menéndez Rodrigues (U. Federal de Rondonia) Porto Velho - RO
Valdenberg Araújo da Silva (U. Federal de Sergipe) São Cristovão - SE
Wagner Pereira Lopes (Esc. Tec. Fed. de Goiás) Jataí - GO
Waldemar M. Canalli (P.M. S. João de Meriti) S. João de Meriti - RJ

EUREKA! N°7, 2000


63
CONTEÚDO

AOS LEITORES 2

XI OLIMPÍADA DE MATEMÁTICA DO CONE SUL 3


Problemas e soluções

VI OLIMPÍADA DE MAIO 11
Problemas

VI OLIMPÍADA DE MAIO 13
Resultados

XLI OLIMPÍADA INTERNACIONAL DE MATEMÁTICA 14


Problemas e Resultados

ARTIGOS

INTRODUÇÃO À GEOMETRIA PROJETIVA 16


Luciano G. M. Castro

CONTAR DUAS VEZES PARA GENERALIZAR (O RETORNO) 28


José Paulo Carneiro, Universidade Santa Úrsula

O PRINCÍPIO DO ELEMENTO EXTREMO 33


José Rosales Ortega, Escola de Matemática - Instituto Tecnológico de Costa Rica

FUNÇÕES MULTIPLICATIVAS E A FUNÇÃO DE MÖBIUS 43


Carlos Gustavo T. de A. Moreira, IMPA  Nicolau Corção Saldanha, PUC-Rio

OLIMPÍADAS AO REDOR DO MUNDO 47

SOLUÇÕES DE PROBLEMAS PROPOSTOS 51

PROBLEMAS PROPOSTOS 60

AGENDA OLÍMPICA 61
Sociedade Brasileira de Matemática

COORDENADORES REGIONAIS 62
AOS LEITORES

Este número da Eureka! contém as provas das competições internacionais


de que participamos na primeira parte do ano 2000: a Olimpíada de Maio, a
Olimpíada do Cone Sul e a Olimpíada Internacional de Matemática. Estas provas
fornecem material que pode (e deve) ser usado na preparação para a Terceira Fase
da Olimpíada Brasileira de Matemática.

Na seção de artigos, é com prazer que publicamos artigos de novos


colaboradores da Eureka!. Destacamos o artigo do Prof. José Rosales Ortega, da
Costa Rica, que esperamos dê início a uma colaboração intensa com professores
de outros países, igualmente dedicados à disseminação da matemática entre os
jovens.

Neste número inauguramos uma nova seção, “Olimpíadas ao Redor do


Mundo”, organizada pelo Prof. Antônio Luiz Santos, que trará problemas de
Olimpíadas realizadas em outros países. Esta seção se junta à de problemas
propostos no objetivo de fornecer ainda mais material para treinamento e
desenvolvimento individual.

Aproveitamos para registrar, com satisfação, um grau cada vez maior de


participação de nossos leitores. Temos recebido um número crescente de soluções
para os problemas propostos, além de sugestões de novos problemas. Obrigado a
todos que têm colaborado!

Comitê Editorial

EUREKA! N°8, 2000


2
Sociedade Brasileira de Matemática

XI OLIMPÍADA DE MATEMÁTICA DO CONE SUL


14 a 19 de abril, Montevidéu - Uruguai

A XI Olimpíada de Matemática do Cone Sul foi realizada em


Montevidéu, Uruguai, no período de 14 a 19 de abril de 2000. A equipe brasileira
foi liderada pelos professores Paulo José Bonfim Gomes Rodrigues e Marcelo
Mendes, ambos de Fortaleza - CE. Nesta oportunidade a equipe brasileira obteve
a maior pontuação entre os países participantes e a única medalha de ouro da
competição.

RESULTADOS DA EQUIPE BRASILEIRA

BRA1 Carlos Stein Naves de Brito Prata


BRA2 Davi Máximo Alexandrino Nogueira Bronze
BRA3 Humberto Silva Naves Ouro
BRA4 Larissa Cavalcante Queiroz de Lima Prata

PROBLEMA 1
Dizemos que um número é descendente se cada um de seus dígitos é menor do
que ou igual ao dígito anterior, da esquerda para a direita. Por exemplo, 4221 e
751 são números descendentes, enquanto 476 e 455 não são descendentes.
Determine se existem inteiros positivos n para os quais 16n é descendente.

SOLUÇÃO DE CARLOS STEIN NAVES DE BRITO (GOIÂNIA - GO)


Sabemos que 16 n 6(mod 10), pois 6 n 6(mod 10).
Assim o dígito das unidades será sempre 6.
Temos então:
2 4 n 6(mod 10)
2 4 n 2 4 k (mod10.000) pois (10.000,2 4 n ) 2 4 .
Temos que 2 4 k 6(mod 10)  k 5q  1.
2 4 n 2 4 (5q  1)(mod10.000)
2 4 n 10(8q  1)  6(mod10.000)

EUREKA! N°8, 2000


3
Sociedade Brasileira de Matemática

Temos que 8q + 1 deve ter dígitos maiores ou iguais a 6. Em particular, 8q + 1


termina por 7 ou 9. Temos então as seguintes possibilidades para os seus últimos
3 dígitos:
999, 997, 987, 977, 887, 877, 777.
Os únicos que são da forma 8q + 1 são 977 e 777.
Como 25 divide 7776, 16n não termina em 77776 nem em 97776.
16 n 87776(mod10 5 )  16 n 987776(mod 10 6 ). Como 27 divide 987776,
16n
não termina em 9987776. Como 26 divide 99776, 16n não termina em 999776 
16n tem no máximo 6 dígitos, e basta verificar os casos. Como para nenhum caso
haverá solução, 16 n nunca é descendente.

PROBLEMA 2
Em um tabuleiro 8  8 distribuímos os inteiros de 1 a 64, um em cada casa.
A seguir, colocam-se sobre o tabuleiro fichas quadradas 22, que cobrem
exatamente quatro casas (sem superposição) e de modo que os quatro números
cobertos por cada ficha determinem uma soma menor que 100.

Mostrar uma distribuição desses inteiros que permita colocar o maior número de
fichas, e demonstrar que não é possível obter uma distribuição que permita
colocar mais fichas.

SOLUÇÃO DE CARLOS STEIN NAVES DE BRITO (GOIÂNIA - GO)


Sabemos que o somatório dos números sobre os quais colocamos fichas dividido
pelo número de fichas deve ser menor que 100.
Logo se preenchessemos todo o tabuleiro (com 16 fichas):

(64  1)64
1  2  3  ...  64 2 32 65
100  100  100  130 100. Absurdo!
16 16 16
Então a cada ficha a menos que colocamos devemos tirar o maior somatório de
números sem estar preenchidos, pois assim a razão anterior vai ser mínima.
A cada ficha que retiramos tiraremos 64, 63, 62, 61, depois 60, 59, 58, 57... até a
razão do somatório dos números preenchidos dividido pelo número de fichas ser
menor que 100. Disso temos:
Dado: o somatório inicial é 2080 e o número inicial de fichas é 16 e sendo n o
número de fichas retiradas que deve ser mínimo

EUREKA! N°8, 2000


4
Sociedade Brasileira de Matemática

2080   64  (64  1)  (64  2)  ...  (64  ( 4n  1))


100 
16  n
  ( 4n  1  0) 4 n  
2080   4n 64   
  2 
100 
16  n
2080  256n  8n 2  2n 1600  100n  4n 2  79n  240 0

 2 79 49 n116 
 4 n  79 n  240 0  n  n 3,75 
 8 2 

+ +

3,75 16

Como se quer o n mínimo, que satisfaça a desigualdade, n é 4 e teremos 12 fichas


no máximo.
Para n = 3, com 13 fichas:
Podemos colocar 12 fichas, do seguinte modo:
Vamos ter os números de 1 até 48. É agrupamos eles de 4 em 4 para a soma ser
menor que 100.
Esses grupos são {1, 24, 25, 48}, {2, 23, 26, 47},...,{12, 13, 36, 37}.
Da forma {1+ n, 24 – n, 25 + n, 48 – n} com n {0, 1, ..., 11}.
Então colocaremos esses números em espaços 2  2:

1 24 2 23 3 22 4 21
25 45 26 47 27 46 28 45

Faremos isso com todos os grupos, sobrando ainda um espaço 2  8, que não
terão ficha, onde colocaremos aleatóriamente os números {49, 50,...,64}.
Sendo essa uma solução com cada ficha sob um grupo daqueles citados.

EUREKA! N°8, 2000


5
Sociedade Brasileira de Matemática

Exemplo completo:

1 24 2 23 3 22 4 21
25 48 26 47 27 46 28 45
5 20 6 19 7 18 8 17
29 44 30 43 31 42 32 41
9 16 10 15 11 14 12 13
33 40 34 39 35 38 36 37
49 50 51 52 53 54 55 56
57 58 59 60 61 62 63 64

PROBLEMA 3
Um quadrado de lado 2 é dividido em retângulos mediante várias retas paralelas
aos lados (algumas horizontais e outras verticais). Os retângulos são coloridos
alternadamente de preto e branco, como se fosse um tabuleiro de xadrez. Se deste
modo a área branca resultou igual a área preta, demonstrar que ao recortar os
retângulos pretos ao longo de seus bordos, é possível formar com estes (sem
superposição) um retângulo preto 1  2.

SOLUÇÃO DE HUMBERTO SILVA NAVES (SÃO PAULO - SP)


Seja x1 ; x 2 ; x3 ;...; x n as distâncias entre as retas verticais ( xi é distância entre
a i-ésima reta e a (i – 1)-ésima reta) e y1 ; y 2 ;...; y p as distâncias entre as retas
horizontais: ( y i é a distância entre a i-ésima reta vertical e a (i – 1)-ésima reta).
Por simetria, podemos considerar:

 xi yj   xi yj   xi yj
Área sombreada = i e j de i e j pares i e j ímpares
mesma
paridade
Logo, área sombreada =
     
2   xi y j   xi y j   xi   yi     x j   y j 
i e j pares i e j
ímpares
 i par  i par   j ímpar  j ímpar 
e denotamos:

EUREKA! N°8, 2000


6
Sociedade Brasileira de Matemática

A  xi e B   yi
"i" par "i "par

mas  xi "i
" par
xi   x j 2 
" j " ímpar
 xj
" j " ímpar
2  A,
e de mesmo modo

concluímos que:  yj 2  B.
" j " ímpar

Logo: Área sombreada = 2 = A B  (2  A)(2  B) 


2 2 AB  4  2( A  B)  2( A  B)  2 AB 2 
A  B  AB 1  A(1  B ) 1  B  ( A  1)(1  B) 0

 A  1 0
Logo devemos ter
 ou
1  B 0
 A 1 ou B 1 Agora o problema fica fácil, pois se A 1 (por simetria),
temos:  xi   x j , logo basta juntar os "quadradinhos" de cada linha, aí
i par j ímpar

vai formar um retângulo de base 1, e se juntarmos todos esses retângulos de base


1, vamos formar outro retângulo, cujos lados medem: 2 e 1.

PROBLEMA 4
Sejam ABCD um quadrado (sentido horário) e P um ponto qualquer pertencente
ao interior do segmento BC. Constrói-se o quadrado APRS (sentido horário).
Demonstrar que a reta CR é tangente a circunferência circunscrita ao triângulo
ABC.

SOLUÇÃO DE LARISSA CAVALCANTE QUEIROZ DE LIMA (FORTALEZA-CE)

EUREKA! N°8, 2000


7
Sociedade Brasileira de Matemática

A

B O ABC é retângulo, portanto o
 
centro da circunferência circunscrita
 45 –
está no ponto médio de sua
 hipotenusa: AC  centro da
P
M
 circunferência é o ponto M
45 +
*ABCD é um quadrado  as
diagonais se cortam ao meio, e as
S  diagonais são iguais  AM = BM =
D C
MC = MD
45

Seja BA ˆ P  e ˆ A 
BP ;    90 (  retângulo ABP);
    90 180
Note que RPˆ C 180  90    RPˆ C 
ˆ C 45 ( ABC é retângulo é isósceles)
* BA
 PA ˆ C BA ˆ C  BA
ˆ P 45  

 AP PR ˆ
*  APR é isósceles e retângulo   PAR PRˆA 45
 APˆR 90
 RAˆ C PAˆ R  PAˆ C 45  ( 45   ) 45  45   
 RAˆ C RPˆ C   APCR é um quadrilátero inscritível
 APˆ R  ACˆ R 90
 CR é perpendicular a AC e que é o diâmetro da circunferência circunscrita a
ABC  CR é tangente.

PROBLEMA 5
No plano cartesiano, considere os pontos de coordenadas inteiras. Uma operação
consiste em:
Escolher um destes pontos e realizar uma rotação de 90 o. no sentido anti-horário,
com centro neste ponto.
É possível, através de uma seqüência dessas operações, levar o triângulo de
vértices (0, 0), (1, 0), e (0, 1) no triângulo de vértices (0, 0), (1, 0) e (1, 1)?

EUREKA! N°8, 2000


8
Sociedade Brasileira de Matemática

SOLUÇÃO ADAPTADA DA SOLUÇÃO DE DAVI MÁXIMO ALEXANDRINO NOGUEIRA


(FORTALEZA - CE)
Considere a figura relativa a demonstração:
Y
B
A
B
A
B
A B A B A B A
B A B A B A

A (0,0) B A B A B A X

Considere duas cores A e B. Pinte o ponto (0,0) de A. A partir daí, pinte todos os
outros pontos (coordenadas inteiras) do plano com as cores A e B,
alternadamente. Isto é, pintamos (a, b) de A se a + b é par, e de B se a + b é
ímpar. Vamos provar que um ponto e sua imagem possuem a mesma cor.
De fato, se P = (x, y), a imagem de (a, b) pela rotação de 90o no sentido
antihorário com centro em P e (x + y – b, x + y + a), cuja soma das coordenadas é
2x + 2y + a – b  a + b (mod 2).
Como o primero triângulo tem um ponto da cor A e dois da cor B e o segundo tem
dois pontos da cor A e um da cor B não é possível tal coisa.

PROBLEMA 6
Existe um inteiro positivo divisível pelo produto de seus algarismos e tal que esse
produto é maior que 102000?

SOLUÇÃO DE HUMBERTO SILVA NAVES (SÃO PAULO - SP)


Primeiramente vamos provar que 10 é raiz primitiva no módulo 7 n.
*Sabemos que quando n = 1 ou n = 2, isto é verdadeiro.
** Suponhamos que 10 seja uma raiz primitiva no módulo 7 n (n 2)
Seja "a" uma raiz primitiva no módulo 7 n 1 (ela existe pois 7 n 1 é uma
potência de um primo), isto é: a j percorre todas as classes de congruência que
são primas com 7, no módulo 7 n 1 , consequentemente "a" também é raiz
primitiva no módulo 7 n .
Pela definição de "a", existe um x  N e um y  N , tais que:
a x 10(mod 7 n )
a y 10(mod 7 n 1 )

EUREKA! N°8, 2000


9
Sociedade Brasileira de Matemática

Temos que mdc ( x;  (7 n )) 1, pois 10 também é raiz primitiva no módulo


7 n.
Se mdc ( y;  (7 n 1 )) d 1, teríamos:

y n
n1 
 a 10 (mod 7 ) x y n
a 10(mod 7 )   x n  a a (mod 7 )  x y(mod (7n ) 
y

 a 10(mod 7 ) Chegamos a uma contradição, pois

 y é primo com  (7n ) (pois x também é)


mdc ( y;  (7 n 1 )) d e
mdc ( y;  (7 n )) 1, isto quer dizer:
mdc ( y;6 7 n ) 1 e mdc ( y;6 7 n  1 ) 1 (com n 2 ), que é um absurdo.
Daí concluímos que mdc ( y;  (7 n 1 )) 1.
Logo 10 também é uma raiz primitiva no módulo 7 n 1 , e por indução
concluímos que:
n  N; 10 é raiz primitiva no módulo 7 n .
Agora vamos achar um exemplo:
Considere a, tal que: 7 a  10 2000
E como 10 é raiz primitiva no módulo 7 a , considere b > a, tal que:
10 b 7  6 10 a (mod 7 a ), temos que:
 10 a  1  10 b  10 a
 x  7   0 (mod 7 a )  x 0(mod 7 a )
 9  9
mas:
 b adígitos
  a dígitos
 
x  1111111 ...1 000
        ...0  777 ...
   7 1111111 ...1 777...7
total de: "b  a " "1"'s "a" 0 ' s "a" 7's

Ou seja x é divisível pelo produto de seus dígitos.

EUREKA! N°8, 2000


10
Sociedade Brasileira de Matemática

Você sabia…
Que foi novamente batido o record de maior primo de Fermat
generalizado conhecido?
É o número 4859465536 + 1 descoberto este ano por Scott e
Gallot, que é o 6o. maior primo conhecido (e o único primo
conhecido com mais de um milhão de bits que não é de
Mersenne). Com isso, os
9 maiores primos conhecidos são de Mersenne ou de Fermat
generalizados. São eles: 26972593 – 1, 23021377 – 1, 22976221 – 1,
21398269 – 1, 21257787 – 1, 4859465536 + 1, 2859433 – 1, 2756839 – 1 e
16717632768 + 1, os quais têm, respectivamente, 2098960,
909526, 895932, 420921, 378632, 307140, 258716, 227832 e
171153 dígitos.

VI OLIMPÍADA DE MAIO
13 de maio de 2000

PRIMEIRO NÍVEL
Duração da prova: 3 horas

PROBLEMA 1
Encontre todos os números naturais de quatro algarismos formados por dois
dígitos pares e dois dígitos ímpares tais que, ao multiplicá-los por 2, se obtém
números de quatro algarismos com todos os seus dígitos pares e, ao dividí-los por
2, se obtém números naturais de quatro algarismos com todos os seus dígitos
ímpares.

PROBLEMA 2
Seja ABC um triângulo retângulo em A, cujo cateto AC mede 1cm. A bissetriz do
ângulo BA ˆ C corta a hipotenusa em R; a perpendicular a AR traçada por R corta
o lado AB em seu ponto médio. Encontre a medida do lado AB.

PROBLEMA 3
Para escrever todos os números naturais consecutivos desde 1ab até ab2 inclusive
foram utilizados 1ab1 algarismos. Determine quantos algarismos a mais

EUREKA! N°8, 2000


11
Sociedade Brasileira de Matemática

precisam-se para escrever os números naturais até o aab inclusive. Diga todas as
possibilidades. (a e b representam dígitos).

PROBLEMA 4
Temos peças com forma de triângulo equilátero de lados 1; 2; 3; 4; 5 e 6 (50
peças de cada medida).
Precisa-se armar um triângulo equilátero de lado 7 utilizando algumas destas
peças, sem buracos nem superposições. Qual é o menor número de peças
necessárias?

PROBLEMA 5
Numa fileira temos 12 cartas que podem ser de três tipos: com as duas faces
brancas, com as duas faces pretas ou com uma face branca e a outra preta.
Inicialmente temos 9 cartas com a face preta voltada para cima.
Viram-se as seis primeiras cartas da esquerda e ficam 9 cartas com a face preta
voltada para cima.
Continuando, viram-se as seis cartas centrais, ficando 8 cartas com a face preta
voltada para cima.
Finalmente, viram-se seis cartas: as três primeiras da esquerda e as três últimas da
direita, ficando 3 cartas com a face preta voltada para cima.
Diga se com esta informação se pode saber com certeza quantas cartas de cada
tipo existem na fileira.

SEGUNDO NÍVEL

PROBLEMA 1
O conjunto {1, 2, 3, 4} pode ser dividido em dois subconjuntos A = {1, 4} e B =
{3, 2} sem elementos comuns e tais que a soma dos elementos de A seja igual a
soma dos elementos de B. Essa divisão é impossível para o conjunto {1, 2, 3, 4,
5} e também para o conjunto {1, 2, 3, 4, 5, 6}.
Determine todos os valores de n para os quais o conjunto dos primeiros n
números naturais pode ser dividido em dois subconjuntos sem elementos comuns
tais que a soma dos elementos de cada subconjunto seja a mesma.

PROBLEMA 2
Num paralelogramo de área 1 são traçadas retas que unem cada vértice com o
ponto médio de cada lado não adjacente a ele. As oito retas traçadas determinam
um octógono no interior do paralelogramo. Calcule a área do octógono.

PROBLEMA 3

EUREKA! N°8, 2000


12
Sociedade Brasileira de Matemática

Sejam S uma circunferência de raio 2; S1 uma circunferência de raio 1 tangente


interiormente a S em B e S2 uma circunferência de raio 1 tangente a S1 no ponto
A, mas que não é tangente a S. Se K é o ponto de interseção da reta AB com a
circunferência S, demonstre que K pertence a circunferência S2.

PROBLEMA 4
Temos um cubo de 3  3  3 formado pela união de 27 cubinhos 1  1  1.
Retiramos alguns cubinhos de tal modo que os que permanecem seguem
formando um sólido constituído por cubinhos que estão unidos pelo menos por
uma face ao resto do sólido. Quando um cubinho é retirado, os que permanecem
ficam no mesmo lugar em que estavam inicialmente.
Qual é o máximo número de cubinhos que podem ser retirados de modo que a
área do sólido que resulte seja igual à área do cubo original?

PROBLEMA 5
Um retângulo pode ser dividido em n quadrados iguais e também pode ser
dividido em n + 98 quadrados iguais. Se a área do retângulo é n, com n inteiro,
encontre os lados do retângulo. Diga todas as possibilidades.
VI OLIMPÍADA DE MAIO
Resultados

PRIMEIRO NÍVEL

Fabio Dias Moreira Medalha de Ouro Rio de Janeiro - RJ


Guilherme Salermo Santos Medalha de Prata Goiânia - GO
Raul M. Alexandrino Nogueira Medalha de Prata Fortaleza - CE
Alex Correa Abreu Medalha de Bronze Niteroi - RJ
Iuri Lima Ribeiro Medalha de Bronze Fortaleza - CE
Antônia Taline de Souza Mendonça Medalha de Bronze Fortaleza - CE
Cincinato Furtado Leite Neto Medalha de Bronze Fortaleza - CE
Alan Hideki Uchida Menção Honrosa São Paulo - SP
Rodrigo Aguiar Pinheiro Menção Honrosa Fortaleza - CE
Luty Rodrigues Ribeiro Menção Honrosa Fortaleza - CE

SEGUNDO NÍVEL

Marcio Antonio F. Belo Medalha de Ouro Goiânia - GO


Henrique Chociay Medalha de Prata Curitiba - PR

EUREKA! N°8, 2000


13
Sociedade Brasileira de Matemática

Davi M. Alexandrino Nogueira Medalha de Prata Fortaleza - CE


Larissa Goulart Rodrigues Medalha de Bronze Goiânia - GO
Andreia Lucio dos Santos Medalha de Bronze Goiânia - GO
Thiago da Silva Sobral Medalha de Bronze Fortaleza - CE
Luis Gustavo Bastos Pinho Medalha de Bronze Fortaleza - CE
Samuel Barbosa Feitosa Menção Honrosa Fortaleza - CE
Adriano Arantes Paterlini Menção Honrosa Tatuí - SP
Germanna de Oliveira Queiroz Menção Honrosa Fortaleza - CE

XLI OLIMPÍADA INTERNACIONAL DE MATEMÁTICA


13 a 25 de julho, Taejon - Coréia do Sul

A XLI Olimpíada Internacional de Matemática foi realizada em Taejon,


Coréia do Sul, no período de 13 a 25 de julho de 2000. A equipe brasileira foi
liderada pelos professores Élio Mega e Edmilson Motta, ambos de São Paulo -
SP.

RESULTADOS DA EQUIPE BRASILEIRA

BRA1 Daniel Nobuo Uno Bronze


BRA2 Daniel Massaki Yamamoto Bronze
BRA3 Fabrício Siqueira Benevides Bronze
BRA4 Humberto Silva Naves ----------
BRA5 Sergio Tadao Martins ----------
BRA6 Ulisses Medeiros de Albuquerque Menção Honrosa

PROBLEMA 1
Duas circunferências 1 e 2 intersectam-se em M e N.

EUREKA! N°8, 2000


14
Sociedade Brasileira de Matemática

Seja l a tangente comum a 1 e 2 que está mais próxima de M do que de N.


A reta l é tangente a 1 em A e a 2 em B. A reta paralela a l que passa por M
intersecta novamente a circunferência 1 em C e novamente a circunferência
2 em D.
As retas CA e DB intersectam-se em E; as retas AN e CD intersectam-se em P; as
retas BN e CD intersectam-se em Q.
Mostre que EP = EQ.

PROBLEMA 2
Sejam a, b, c números reais positivos tais que abc = 1. Prove que

 1  1  1
 a  1   b  1   c  1   1.
 b  c  a

PROBLEMA 3
Seja n 2 um número inteiro positivo. No início existem n pulgas numa reta
horizontal, nem todas no mesmo ponto.
Para um número real positivo  , define-se um salto da seguinte maneira:
 Escolhem-se duas pulgas quaisquer nos pontos A e B com o ponto A à esquerda
do ponto B;
 A pulga que está em A salta até o ponto C da reta, à direita de B, tal que
BC
 .
AB
Determine todos os valores de  para os quais, para qualquer ponto M na reta e
quaisquer posições iniciais das n pulgas, existe uma sucessão finita de saltos que
levam todas as pulgas para pontos à direita de M.

PROBLEMA 4
Um mágico tem cem cartões numerados de 1 a 100. Coloca-os em três caixas,
uma vermelha, uma branca e uma azul, de modo que cada caixa contém pelo
menos um cartão.
Uma pessoa da platéia escolhe duas das três caixas, seleciona um cartão de cada
caixa e anuncia a soma dos números dos dois cartões que escolheu. Ao saber esta
soma, o mágico identifica a caixa da qual não se retirou nenhum cartão.

EUREKA! N°8, 2000


15
Sociedade Brasileira de Matemática

De quantas maneiras podem ser colocados todos os cartões nas caixas de modo de
que este truque sempre funcione? (Duas maneiras consideram-se diferentes se
pelo menos um cartão é colocado numa caixa diferente).

PROBLEMA 5
Verifique se existe um inteiro positivo n tal que n é divisível por exatamente 2000
números primos diferentes e 2 n  1 é divisível por n.

PROBLEMA 6
Sejam AH 1 , BH 2 , CH 3 as alturas de um triângulo acutângulo ABC. A
circunferência inscrita no triângulo ABC é tangente aos lados BC, CA, AB em
T1 , T2 , T3, respectivamente. Seja l1 a reta simétrica da reta H 2 H 3
relativamente à reta T2 T3 , l 2 a reta simétrica da reta H 3 H 1 relativamente à
reta T3T1 e l 3 a reta simétrica da reta H 1 H 2 relativamente à reta T1T2 .
Prove que l1 , l 2 , l 3 determinam um triângulo cujos vértices pertencem à
circunferência inscrita no triângulo ABC.

INTRODUÇÃO À GEOMETRIA PROJETIVA


Luciano G. M. Castro

 Nível Avançado

Artigo baseado em aula ministrada na III Semana Olímpica


Piracicaba - SP

Começamos com um problema de Geometria Euclidiana:

Problema Inicial:

As tangentes a uma circunferência de centro O, traçadas por um ponto


exterior C, tocam a circunferência nos pontos A e B. Seja S um ponto qualquer da
circunferência. As retas SA, SB e SC cortam o diâmetro perpendicular a OS
nos pontos A', B' e C ', respectivamente.
Prove que C' é o ponto médio de A'B'.
Encorajamos o leitor a resolver este problema utilizando métodos da Geometria
Euclidiana, antes de prosseguir.

EUREKA! N°8, 2000


16
Sociedade Brasileira de Matemática

Nossa principal meta é desenvolver ferramentas da Geometria Projetiva que nos


permitam resolver este e outros problemas similares de forma direta e natural.

1. POLARIDADE

Dada uma circunferência  , de centro O e raio R, vamos criar uma


associação entre pontos e retas do plano, da seguinte maneira: Para cada ponto A
distinto de O, seja A' o ponto da semi-reta OA tal que OA OA'  R 2 . (A' é
chamado inverso de A em relação a  . A transformação A  A' é a inversão
relativa a  ). Seja a a reta perpendicular a OA passando por A'. Dizemos que a
é a reta polar de A em relação a  , e que A é o pólo de a em relação a  .

a
R

O A A'

A transformação do plano que leva cada ponto em sua polar e cada reta em seu
pólo é chamada de polaridade. Para simplificar a notação, usaremos a mesma
letra para designar um ponto (maiúscula) e sua polar (minúscula).

Teorema 1: Sejam A e B dois pontos do plano, a e b suas respectivas polares. Se


B  a, então A  b. Neste caso, dizemos que A e B são conjugados.

EUREKA! N°8, 2000


17
Sociedade Brasileira de Matemática

a Considere um ponto B  a.
Seja B'  OB tal que AB '  OB. Os
b triângulos OAB' e OBA' são retângulos e têm
um ângulo comum ( AÔB'  BÔA' ), logo
são semelhantes. Assim,
OA OB'
A A'   OB OB' OA OA'  R 2 .
OB OA'
O Logo B' é o inverso de B, de onde AB ' b e
B' A  b.

Assim, se imaginarmos o ponto B variando ao longo da reta a, sua polar, b,


variará ao longo do feixe de retas que passam pelo ponto A.
Diremos que um ponto e uma reta são incidentes quando o ponto pertence à reta,
o que é o mesmo que dizer que a reta passa pelo ponto.
A polaridade, portanto, é uma transformação que preserva incidências.

Exercício 1: Se um ponto é conjugado a si mesmo, então ele pertence à


circunferência e sua polar é a tangente à circunferência por ele.

Este resultado nos permite desenvolver a seguinte construção para a reta polar de
um ponto A exterior à circunferência:

Exercício 2: Se A é exterior à circunferência, sejam B e C os pontos de contato das


duas tangentes à circunferência traçadas por A. A reta BC é a polar de A.

EUREKA! N°8, 2000


18
Sociedade Brasileira de Matemática

Solução:
B Como A pertence às polares de B e C, então
B e C pertencem à polar de A. Logo a  BC

2. O PLANO PROJETIVO

A polaridade definida anteriormente sugere que pontos e retas têm


comportamentos parecidos em relação à incidência. Há algumas falhas, porém. A
transformação não está definida para o ponto O, centro da circunferência, nem
tampouco para as retas que passam por O.

Podemos resolver este problema ampliando o plano euclidiano,


acrescentando-lhe uma nova reta que chamaremos de "reta do infinito", que
representaremos por o. Esta nova reta será a polar do ponto O.

Formalmente, os pontos da nova reta do infinito estão em


correspondência biunívoca com os feixes de retas paralelas no plano euclidiano.

Vejamos como a polaridade nos leva naturalmente a esta definição para


os pontos do infinito.

Por exemplo, vamos identificar o pólo de uma reta r que passa por O.
Sejam A e B os pontos de contato de r com a circunferência. Como A e B estão
sobre a reta r, suas retas polares a e b passam pelo pólo R. Logo R é o ponto de
encontro das duas retas a e b, que no plano Euclidiano seriam paralelas. De fato,
a reta polar de qualquer ponto de r será perpendicular a r no plano euclidiano.
Estas retas passam a ser, no plano projetivo, um feixe de retas concorrentes (no
ponto do infinito R).

EUREKA! N°8, 2000


19
Sociedade Brasileira de Matemática

Esta é a maneira de trabalhar com a reta do infinito: cada um de seus pontos


corresponde a um único feixe de retas paralelas no plano euclidiano. E vice-versa: a cada
feixe de retas paralelas no plano euclidiano corresponde um único ponto da reta do
infinito.

A a ...

O ... R

...
B b

3. O PRINCÍPIO DA DUALIDADE

Os pontos e retas do plano projetivo têm exatamente o mesmo


comportamento em relação a incidência. Assim, qualquer propriedade
envolvendo pontos, retas e incidência permenece válida ao trocarmos pontos por
retas e retas por pontos. A nova propriedade assim obtida é denominada "dual" da
primeira.

Em outras palavras, para todo teorema da Geometria Projetiva recebemos


outro grátis, oferecido pelo Princípio da Dualidade. Basta trocar a palavra "ponto"
pela palavra "reta" e vice versa.
Exemplos:
Propriedade Dual

Dada uma reta, sempre existe um Dado um ponto, sempre existe uma reta
ponto não incidente a ela. não incidente a ele.

Cada reta é incidente a pelo menos Cada ponto é incidente a pelo menos três
três pontos distintos. retas distintas.

Dois pontos distintos determinam Duas retas distintas determinam um único


uma única reta a eles incidente. ponto a elas incidente.

EUREKA! N°8, 2000


20
Sociedade Brasileira de Matemática

Observação:
Apesar de termos definido o plano projetivo como uma extensão do plano
euclidiano, isto não é necessário. O plano projetivo existe de forma independente,
podendo ser caracterizado a partir de um conjunto de axiomas, entre os quais
estão as propriedades duais citadas anteriormente.

4. QUÁDRUPLAS HARMÔNICAS

No plano euclidiano, se quatro pontos A, B, C e D de uma reta são tais


que:
AC AD
 ,
BC BD

dizemos que C e D "dividem harmonicamente" o segmento AB .


Observe que, de acordo com a definição, isto também implica que A e B dividem
harmonicamente o segmento CD . Representaremos esta situação com o símbolo
H( AB, CD ). Também diremos que A, B, C e D , nesta ordem, formam uma
"quádrupla harmônica".
Dados os pontos A, B e C sobre uma reta, o ponto D tal que H( AB; CD ) é
chamado "conjugado harmônico" de C em relação a AB .
Surpreendentemente, apesar da definição utilizar a noção de distância (que não faz
sentido no plano projetivo), o conceito de quádruplas harmônicas faz sentido no Plano
Projetivo, por meio da seguinte construção para o conjugado harmônico:

G
Figura 1
a1
H
b1
c
b2
a2

A C B D

Dados os pontos A, B e C sobre uma reta r, traçamos duas retas quaisquer a1 e a2


passando por A e uma reta c passando por C. Unindo a B os pontos de incidência

EUREKA! N°8, 2000


21
Sociedade Brasileira de Matemática

de c com a1 e a2 , respectivamente, obtemos as retas b1 e b2. Fica então formado


um quadrilátero (EFHG, na figura) tal que os lados opostos concorrem em A e B,
e tal que uma de suas diagonais passa por C. Seja D o ponto de encontro de r com
a outra diagonal do quadrilátero. Então D é o conjugado harmônico de C em
relação a AB .

Esta construção é a definição de quádruplas harmônicas no plano projetivo.


Vejamos que ela coincide, no plano Euclidiano, com a definição usual. Sejam os
pontos E, F, G como na figura 1.
Aplicando o Teorema de Menelaus* no ABE , secante DGF, temos:

AD BG EF
  1. (1)
BD EG AF

No ABE , aplicamos o Teorema de Ceva* para as cevianas concorrentes


EC , BF e AG :
AC BG EF
  1. ( 2)
BC EG AF

AC AD
De (1) e (2) temos  .
BC BD

*Ver apêndice.

5. PONTO MÉDIO E CONJUGAÇÃO HARMÔNICA

O principal indício de que quádruplas harmônicas são uma noção


projetiva é o fato de, no plano euclidiano, o ponto médio de um segmento não
possuir conjugado harmônico.
Porém, no plano projetivo, sejam A e B pontos sobre a reta r e C o ponto
médio de AB . Ao realizarmos a construção da figura 1, verificamos que FC é
paralelo a r. No plano projetivo, o conjugado harmônico D é o ponto do infinito
correspondente ao feixe de retas paralelas a r.

6. FEIXES HARMÔNICOS

Vamos agora dualizar a definição de quádrupla harmônica. Dadas 3 retas


a, b e c concorrentes em um ponto R, podemos dualizar, passo a passo, a
construção do conjugado harmônico:

EUREKA! N°8, 2000


22
Sociedade Brasileira de Matemática

Sobre a reta a tomamos dois pontos distintos A1 e A2 e sobre a reta


c tomamos um ponto C. Sejam B1 e B2 os pontos de intersecção da reta b
com as retas CA1 e CA2 , respectivamente. Seja d a reta determinada
pelos pontos R e A1 B2  A2 B1 . Chamamos d de conjugado harmônico de
c em relação a a e b. Dizemos que as quatro retas concorrentes a, b, c, e d
formam um "feixe harmônico". Representamos esta situação com o
símbolo H( ab, cd ).
R

A2
Figura 2
B1

C
d

A1 B2 D
a c
b

Teorema 2: Uma reta qualquer do plano corta um feixe harmônico em quatro


pontos que formam uma quádrupla harmônica.
Se você percebeu a semelhança entre as figuras 1 e 2 deve ter
desconfiado deste fato. A demonstração é imediata.
Na construção da figura 2, os pontos A1 e B2 podem ser escolhidos sobre uma reta
s arbitrária (que não passe por R), e o ponto C fora de s. As retas A1C e B2 C
determinam os pontos B1 e A2. Sendo c  s C ' e d  s  D , vemos que o
quadrilátero RA2 CB1 possui dois lados opostos concorrendo em A1 e B2, com
suas diagonais passando por C' e D . Portanto H( A1 B2 ; C ' D ) , como
queríamos demonstrar.

Exercício 3:
Escreva o dual doTeorema anterior.

7. O TEOREMA DE PASCAL

EUREKA! N°8, 2000


23
Sociedade Brasileira de Matemática

Sem dúvida, é um dos mais belos teoremas da Geometria Projetiva. É válido para
qualquer cônica, apesar de que aqui só veremos a demonstração para a
circunferência, no plano euclidiano. É importante mencionar, no entanto, que no
Plano Projetivo não há qualquer diferença entre uma circunferência e qualquer
outra cônica não-degenerada.

Teorema 3: Os pontos de encontro entre os 3 pares de lados opostos de um


hexágono ABCDEF (convexo ou não) inscrito em uma circunferência são colineares.
X Consideremos o triângulo XYZ indicado na
figura. Aplicamos o Teorema de Menelaus
três vezes:
XYZ , secante PDE:
E PX DY EZ
  1
A
PY DZ EX
C
P
Z XYZ , secante QBC:
Q QZ BX CY
R   1
QX BY CZ
D Y F
XYZ , secante RAF:
B RY AX FZ
  1
RZ AY FX
Multiplicando essas três últimas equações e lembrando que
XA  XB = XE  XF,
YA  YB = YC  YD e
ZC  ZD = ZE  ZF (potência dos pontos x, y, z em relação à
PX QZ RY
circunferência), obtemos PY QX RZ 1.

Logo, pelo recíproco do Teorema de Menelaus no triângulo XYZ, secante


PQR, temos que P, Q e R são colineares.

Fazendo coincidir certos pares de pontos no hexágono ABCDEF, podemos


deduzir teoremas análogos ao de Pascal para pentágonos, quadriláteros e até
triângulos inscritos na circunferência. Por exemplo, fazendo coincidir A com B e
D com E, as retas AB e DE tornam-se tangentes à circunferência, e obtemos a
seguinte configuração:

EUREKA! N°8, 2000


24
Sociedade Brasileira de Matemática

A B

Q P
A
R

D E
C

Figura 3

Exercício 4:
Na figura anterior, verifique que o ponto comum às tangentes em C e F também
pertence à reta PQR.

8. MAIS POLARIDADES

Agora estamos prontos para retomar nosso estudo das polaridade. Aproveitando
tudo o que vimos até aqui, vamos deduzir algumas propriedades mais avançadas.

Teorema 4: (Construção da reta polar usando apenas régua)


Seja  uma circunferência e A um ponto exterior a ela.
Consideremos duas retas distintas passando por A e cortando  nos pontos B, C,
D e E (figura). Então a reta polar de A em relação a  é a reta que une os pontos
BD  EC e BE  CD .

EUREKA! N°8, 2000


25
Sociedade Brasileira de Matemática

P Demonstração:

As polares de B, C, D e E são as retas b,


B
R
c, d, e e tangentes a  em seus
respectivos pólos.
C Sendo R b  c e S d  e, temos
que as polares de R e S são as retas
Q
r  BC e s  DE.
A Como A r  s, sua polar é a reta
E a  RS .
Sendo P  BE  CD e Q  BD  EC ,
um dos corolários do Teorema de Pascal
garante que P, Q, R e S são colineares,
logo a  PQ, como queríamos
demonstrar.
S

Teorema 5: (Relação entre reta polar e quádruplas harmônicas)


Dados uma circunferência  e um ponto exterior A, qualquer reta secante à
circunferência passando por A corta a polar a no conjugado harmônico do ponto
A em relação ao segmento com extremos nos dois pontos de corte da secante com
a circunferência.

Demonstração: Exercício 5
(Dica: na figura anterior, use o quadrilátero PBQC para encontrar o conjugado
harmônico de A em relação a ED ).

9. RESOLUÇÃO DO PROBLEMA INICIAL:

Podemos agora apresentar uma solução simples e elegante para o problema


proposto no início deste artigo.

EUREKA! N°8, 2000


26
Sociedade Brasileira de Matemática

A'
P
O C

C'
S
B'
B
s
c
d

D

Seja d o diâmetro perpendicular a OS .


Seja D o ponto do infinito correspondente ao feixe de retas paralelas a d.
Queremos provar que H( A' B ' , C ' D ) . Para isto, basta provar que as retas
SA , SB , SC e SD formam um feixe harmônico. Parece natural tentar
verificar que a reta AB corta o feixe em uma quádrupla harmônica. Mas isso
equivale a provar que SC é a reta polar do ponto SD  AB.
Isto é simples:

 C é a intersecção das polares de A e B, logo sua polar é c  AB.


 SD é tangente à circunferência no ponto S, logo é a polar de S ( SD s) .
Assim, SD  AB  s  c, e sua polar é, portanto, SC , como queríamos
demonstrar.

APÊNDICE:
TEOREMA DE CEVA:

EUREKA! N°8, 2000


27
Sociedade Brasileira de Matemática

A
Suponha que as cevianas AM, BN e
CP de um triângulo ABC se
encontrem em um ponto Q. Então
P N AN CM BP
  1.
Q CN BM AP

B C
M

Prova:
Suponha que Q t1 A  t 2 B  t 3 C com t1  t 2  t 3 1.
t 2 B  t 3C t1 A  t 3 C t1 A  t 2 B
Então teremos M  , N e P .
t 2  t3 t1  t 3 t1  t 2
AN CM BP t 3 t 2 t1
Assim,      1 
CN BM AP t1 t 3 t 2
TEOREMA DE MENELAUS:
A
Suponha que X  AB, Y  BC e Z  AC
sejam colineares. Então
X
AX BY CZ
  1.
BX CY AZ
Prova:
Suponha que X tA  (1  t ) B e
C
B
Y
Y sB  (1  s )C.
Então Z uX  (1  u )Y , onde u é tal que
(1  t )u  s (1  u ) 0, ou seja,
Z st (1  s )(1  t )
Z A C. Assim,
s t  1 s t  1
AX BY CZ 1  t 1  s st
     1
BX CY AZ t s (1  s )(1  t )

CONTAR DUAS VEZES PARA GENERALIZAR


(O RETORNO)
EUREKA! N°8, 2000
28
Sociedade Brasileira de Matemática

José Paulo Q. Carneiro, Universidade Santa Úrsula

 Nível Avançado
( 2) 2 2 2
1. A fórmula que dá diretamente a soma dos quadrados S n 1  2    n
dos n primeiros inteiros positivos pode ser deduzida de várias maneiras (por
exemplo, [3]). Uma das mais comuns é partir da identidade:
 k  1 3  k 3 3k 2  3k  1 , escrevê-la para k variando de 1 até n:
23  13 3 12  3 1  1
33  23 3 22  3 2  1
.........................................
 n  1 3  n3 3n 2  3n  1
e somar termo a termo estas n igualdades, obtendo:
 n  1 3  13 3Sn( 2)  3Sn(1)  n
(1) n(n  1)
onde S n 1  2    n  , como é bem conhecido (ver [1]).
2
Substituindo este valor e fazendo as contas, chega-se a :
n(n  1)(2n  1)
S n( 2 ) 12  22    n 2 
6
Esta dedução é bastante eficiente e rápida, mas, quando apresentada pela primeira
vez a um estudante, costuma deixar aquela sensação de “coelho tirado da
cartola”, devido ao aparecimento súbito de uma identidade cuja motivação não se
sabe de onde veio. Este tipo de sensação desperta admiração em uns, mas em
outros inspira uma frustração, proveniente da reflexão: “eu nunca vou conseguir
bolar um artifício destes!”. Coloca-se, portanto, a questão: há algum problema
onde a soma dos quadrados apareça naturalmente? E, para este problema, há
alguma outra maneira de resolvê-lo, por meio da qual possamos deduzir a
fórmula da soma dos quadrados?

p
2. Tradicionalmente, em problemas de contagem, o símbolo Cn ( “combinação
de n, p a p”) representa o número de subconjuntos de p elementos contidos em
2
um conjunto de n elementos. Se, por exemplo, fizermos p  2 , então Cn é o
número de pares (não ordenados) que se pode extrair de um conjunto com n
elementos. Exemplos: o número de apertos de mão dados por n pessoas quando
cada uma cumprimenta todas as outras somente uma vez, ou ainda o número de
partidas de futebol em um campeonato com um só turno e n equipes. Em [1], um

EUREKA! N°8, 2000


29
Sociedade Brasileira de Matemática

artigo com o mesmo título que o presente aproveitava justamente o último


exemplo citado para mostrar como, resolvendo um mesmo problema de contagem
2
por dois métodos diferentes, era possível deduzir que: Cn
(n  1)n
1  2    (n  1)  .
2
2
3. Os pitagóricos (sec.VI a.C.) chamavam os números Cn de números
triangulares. O motivo é que eles podem ser vistos como “triângulos” nas figuras:

T1 1 T2 1  2 3 T3 1  2  3 6 T4 1  2  3  4 10
2
Deste modo: Cn Tn  1 , para n  1 .
Além dos números triangulares, os pitagóricos consideravam também os
números quadrados Q1 12 1 , Q2  22 4 , etc., que podem ser
visualizados como quadrados (daí seu nome).

Estas figuras pitagóricas sugerem também uma relação interessante entre os


números triangulares e os números quadrados. Se você partir o quadrado usando a
diagonal sudoeste-nordeste, e incluindo esta diagonal na parte de baixo, você
poderá olhar cada número quadrado como a soma de dois números triangulares
consecutivos; mais especificamente: Qn Tn  1  Tn .

2 2 1  3 32 3  6 42 6  10

Esta relação pode, é claro, ser confirmada algebricamente, já que:


(n  1)n n(n  1)
Tn  1  Tn    n 2 Qn .
2 2

EUREKA! N°8, 2000


30
Sociedade Brasileira de Matemática

4. A observação precedente pode ser usada para calcular a soma dos quadrados
dos n primeiros números naturais. De fato:
Q1  T1
Q2 T1  T2
Q3 T2  T3
...................
Qn Tn  1  Tn
( 2)
Somando termo a termo, temos: S n Q1    Qn  2(T1    Tn  1 )  Tn . Só
resta agora calcular T1    Tn  1 , isto é, a soma dos n  1 primeiros números
2 2 2
triangulares. Para isto, lembremos que esta soma é o mesmo que C2  C3   Cn
, a qual vamos calcular pelo artifício de resolver um mesmo problema por duas
contagens diferentes (ver [1]).

O número de subconjuntos de 3 elementos contidos em um conjunto A de n  1


elementos é representado, como já se sabe, por C n31 . Vamos contar estes
subconjuntos.

Para formar um subconjunto de A com 3 elementos, primeiramente escolhemos


um elemento a  A . Para isto, temos n  1 escolhas. Uma vez escolhido a,
temos n escolhas possíveis para tomar um segundo elemento b; e para cada
escolha de a e b, temos n  1 escolhas possíveis para selecionar o terceiro
elemento c. Isto dá então um total de (n  1)n( n  1) escolhas. Mas é claro que
esta contagem inclui repetições. Para cada a, b, c escolhidos, houve 6
repetições, correspondentes às 6 permutações destes elementos, a saber: a, b, c ;
a , c, b ; b, a, c ; b, c, a ; c, a , b ; c , b, a . Portanto:
( n  1) n ( n  1 )
C n31  .
6
Por outro lado, se quisermos evitar desde o início as repetições, podemos contar
do seguinte modo. Primeiramente, fixamos o elemento a; o número de
subconjuntos de A com 3 elementos e que possuem a é o mesmo que o de
2
subconjuntos de A  {a} com 2 elementos, isto é: Cn . Tomemos agora um
segundo elemento b  a . O número subconjuntos de A com 3 elementos, que
possuem b mas não a, é o mesmo que o de subconjuntos de A  {a; b} com 2
2
elementos, isto é: Cn  1 . Analogamente, o número subconjuntos de A com 3
elementos, que contêm c, mas não intersectam {a, b} , é o mesmo que o de

EUREKA! N°8, 2000


31
Sociedade Brasileira de Matemática

2
subconjuntos de A  {a; b; c} com 2 elementos, isto é: Cn  2 . E assim por
diante, até que cheguemos ao antepenúltimo elemento, quando já teremos
3
contado todos os subconjuntos A com 3 elementos. Logo: Cn 1
Cn2  Cn2 1    C22 .

Deste modo, concluímos que:


(n  1)n(n  1)
T1    Tn  1 C22  C32   Cn2 Cn31  . Conseguimos,
6
portanto, calcular a soma dos n  1 primeiros números triangulares. Daí
concluímos que:
(n  1)n( n  1) ( n  1)n
S n( 2) Q1    Qn  2(T1    Tn  1 )  Tn  
3 2
n(n  1)(2n  1)
 .
6
Podemos generalizar as fórmulas acima, calculando de duas maneiras diferentes o
número de subconjuntos de k + 1 elementos contidos em um conjunto A de n + 1
elementos, que é representado por C nk11 .
A primeira expressão para C nk11 é clássica e pode ser provada do mesmo modo
que foi feito para k + 1 = 3: temos
( n  1) n( n  1)(n  2)...(n  k  1) ( n  1)!
C nk11  
( k  1)! ( k  1)!(n  k )!

(lembremos que m! = 1 . 2 . ... . m).


Seja agora A {a1 , a 2 ,..., a n 1 }. O número de subconjuntos de k + 1
elementos de A que contêm a1 é C nk (escolhemos os k elementos de A
diferentes de a1 ). O número de subconjuntos de k + 1 elementos de A que
contêm a 2 mas não contêm a1 é C nk 1 , e assim sucessivamente, o que mostra
a igualdade
C nk11 C nk  C nk 1  ...  C kk .
( n  k  1)(n  k  2)...( n  1)n
Se Pk (n) C nkk  1  é o "polinômio
k!
triangular generalizado de dimensão k", temos que Pk (n) é um polinômio em n
de grau k, e, pela fórmula acima, temos
Pk (1)  Pk ( 2)  ...  Pk ( m) C kk  C kk1  ...  C mk k  1 C mk 1k .

EUREKA! N°8, 2000


32
Sociedade Brasileira de Matemática

Podemos, como antes, escrever n k como uma combinação linear dos polinômios
Pj ( n),0  j k , e usar a fórmula acima para obter uma fórmula para
S n( k ) 1k  2 k  ...  n k (essa fórmula será a combinação correspondente dos
j 1
termos C n  j , com 0  j k ).
Tal fórmula também pode ser obtida recursivamente como no início do artigo,
k
k 1 k 1
somando as identidades ( j  1)  j  C kr 1  j r , desde j = 1 até j = n,
r 0
ficando o lado esquerdo igual a ( n  1) k 1  1 e o direito igual a
k1
(k  1) S n( k )   C kr 1 S n( r ) , o que dá
r 0
k 1
1  
S n( k )   (n  1) k 1  1   C kr1 S n(r ) .
k 1  r 0 

Referências Bibliográficas:
[1] Carneiro, J.P., Contar duas vezes para generalizar, Eureka!, nº6, pp.15-17, 1999.
[2] Eves, H., Introdução à História da Matemática, Editora da UNICAMP, 1995
[3] Valadares, E.C., e Wagner, E., Usando geometria para somar, Revista do Professor de
Matemática, nº39, pp.1-8, 1999.
O PRINCÍPIO DO ELEMENTO EXTREMO
José Rosales Ortega
Escola de Matemática, Instituto Tecnológico de Costa Rica

EUREKA! N°8, 2000


33
Sociedade Brasileira de Matemática

 Nível Avançado

Resumo

O artigo expõe um princípio de natureza heurística chamado o "princípio do


extremo", que permite resolver problemas matemáticos de nível olímpico de
maneira simples.

1.- Introdução.
Muitos matemáticos profissionais desejam contribuir para tornar a Matemática
mais atrativa aos estudantes com talento. Uma forma de seguir este objetivo é criar
problemas que requeiram uma grande dose de sentido comum, imaginação, e
muitas vezes, uma estratégia específica de resolução de problemas. Este artigo
introduz uma dessas estratégias, o "princípio do elemento extremo". Ainda que este
nome não seja amplamente usado, este princípio pode lhe ajudar a resolver
problemas matemáticos que aparecem freqüentemente em olimpíadas. O material é
baseado na experiência pessoal ganha ao trabalhar com estudantes com talento em
matemática e na minha participação como organizador de várias competições
olímpicas.

2.- A idéia do princípio.


Considere uma fileira de estudantes ordenada em forma decrescente segundo a
altura. A maioria deles tem dois vizinhos. Dois "elementos extremos", o mais alto e
o mais baixo, tem somente um vizinho, porém estes dois elementos extremos
possuem outras propriedades muito úteis. Por exemplo, quando contamos os
estudantes na fileira, a melhor maneira é começar com um destes elementos
extremos.
Em matemática, algumas vezes trabalhamos com conjuntos cujos elementos
parecem ser equivalentes e cujas propriedades conhecidas são poucas. Uma
estratégia poderosíssima em tais casos é considerar o elemento, ou os elementos,
que de alguma forma são elementos extremos. Por exemplo, quando consideramos
um conjunto infinito de números naturais, o elemento extremo é seu elemento
menor. Para um conjunto finito de números reais os elementos extremos são o
máximo e o mínimo do conjunto.
Em muitos casos o elemento extremo é atrativo devido a que suas propriedades
adicionais nos permitem obter concluir sobre o mesmo elemento, ou sobre o do
conjunto como um todo. Por exemplo, em um triângulo o lado maior se opõe ao
ângulo maior e vice-versa. Na continuação apresentamos mais exemplos.

EUREKA! N°8, 2000


34
Sociedade Brasileira de Matemática

Exemplo 1. Sejam     os ângulos de um triângulo. Como  é o ângulo


maior, então   / 3, já que, caso contrário, teríamos    / 3,    / 3 e
   / 3 , o que condradiz o fato de que       . Da mesma forma podemos
concluir que   / 3.
Também não é difícil obter que, se  é o menor ângulo de um polígono convexo
com n lados (n > 3), então  ( n  2) / n. Para provar isto assuma o contrário, e
use o resultado que estabelece que a soma dos ângulos no polígono é igual a
( n  2) .
Exemplo 2. Considere três raios com origem comum num mesmo plano, formando
três ângulos a b c, tal que a + b + c = 2  . É fácil ver que c 2 / 3 e
a 2 / 3. Expressões similares podem ser encontradas se, em lugar de três raios
considerarmos n raios com um origem comum.

a b

Exemplo 3. Os exemplos anteriores podem ser generalizados se considerarmos


uma sucessão a1 a 2 ... a n de números reais tais que s a1  a 2  ...  a n .
Então a1  s / n e a n  s / n.
Estes exemplos são elementares, mas eles preparam o caminho para resolver o
primeiro exemplo não trivial.

Exemplo 4. Seis pontos em um plano são tais que quaisquer três deles não são
colineares. Prove que três desses seis pontos formam um triângulo que possui um
2
ângulo interno maior ou igual a .
3
Solução: Denote os pontos por A1 . A2 ,..., A6 , e seja M seu fecho convexo.
Podem ocorrer dois casos:

 M possui seis vértices. Aplicando o resultado da segunda parte do exemplo


1, para n = 6, vemos que o maior dos ângulos  de M satisfaz a
desigualdade  2 / 3. Se denotarmos por Ai o vértice de  , e por

EUREKA! N°8, 2000


35
Sociedade Brasileira de Matemática

A j e Ak os vértices adjacentes a  , então o triângulo Ai A j Ak tem


a propriedade requerida.

 M possui menos de seis vértices. Neste caso existem três vértices


Ai , A j e Ak de M, e um ponto Al dentro do triângulo Ai A j Ak .

Ai

Al


Aj Ak

Aplicando o resultado do exemplo 2 aos raios AlAi, AlAj, e AlAk, segue-se que o
maior dos ângulos  satisfaz a desigualdade  2 / 3. Então o triângulo
Al A j Ak possui a propriedade requerida.

3.- Aplicações Geométricas.


As aplicações nesta seção estão relacionadas com objetos geométricos. Em cada
caso o problema é resolvido ao encontrar a maior ou a menor distância, ângulo ou
área.

Problema 1.- Em certo país existem 100 cidades. As distâncias entre cada par de
cidades estão especificadas, e todas são diferentes.
Uma estrada conecta duas cidades A e B se, e somente se, B é a cidade mais
próxima de A ou A é a cidade mais próxima de B.

 Prove que existem no máximo 5 estradas que saem de cada cidade.


 É possível que algumas das estradas formem um polígono?

Solução: Provemos a primeira parte. Considere uma cidade X e duas estradas XA e


XB que ligam X a A e a B, respectivamente.

EUREKA! N°8, 2000


36
Sociedade Brasileira de Matemática

A B

Segue-se que AB é o maior lado do triângulo ABX . Isto é verdade, pois, se (por
exemplo) AX for o maior lado do triângulo ABX , então nem A é a cidade mais
próxima para X, nem X é a cidade mais próxima para A e portanto a estrada X não
deveria existir. Portanto, o ângulo AX ˆB é o maior ângulo no triângulo ABX .
ˆ
Segue-se (exemplo 1) que AXB 60 porque ABX é escaleno.
Suponha que exista uma cidade X e que seis estradas vão desde X até outras
cidades. Então a soma dos seis ângulos em volta de X deveria ser maior que
6 60 360 , o que é impossível.
Mostremos a segunda parte. Para isto suponhamos que existam estradas que
formam um polígono.
A estrada AB foi construída por um dos seguintes motivos:
A B

 B é a cidade mais próxima de A, ou


 A é a cidade mais próxima de B.

Considere ainda sem perda de generalidade, que AB é o maior lado do polígono.


Então CA < AB e BD < AB. Portanto, B não é a cidade mais próxima de A e A não é
a cidade mais próxima de B. Logo, a estrada AB não deveria existir. Segue-se que
tal polígono não existe.

Problema 2.- Os comprimentos das bissetrizes de um triângulo ABC são menores


ou iguais a 1. Prove que a área do triângulo é menor ou igual a 3 / 3.
Solução: Seja  BAC o menor ângulo do triângulo, e seja AD a sua bissetriz.
AB e AC não podem ser ambos maiores que AD / cos( / 2). Para demonstrar isso,
veja a figura:

EUREKA! N°8, 2000


37
Sociedade Brasileira de Matemática

AD
y Ax  Ay 
cos( / 2)
B

/2
A x C

Suponhamos que AB  AD / cos( / 2). . Como  60 , então

AD AD 2
AB    .
 cos 30 3
cos
2
h l
Denotemos por c e c a altura e a bissetriz do vértice C do triângulo ABC,
respectivamente. Então, a área do triângulo é:
1 1 1
( ABC )  hc AB  l c AB  .
2 2 3
Problema 3.- Sejam n( n  3) pontos num plano tais que a área de qualquer
triângulo com três desses pontos como vértices não seja maior que 1. Prove que
todos os pontos estão contidos num triângulo, cuja área é menor ou igual a 4.

Solução: Este problema tem o aspecto de ser muito difícil. A idéia para resolvê-lo
está baseada no seguinte: se você tem um triângulo de área 4, como poderia
relacioná-lo com um triângulo de área 1? Uma boa idéia é conectar os pontos
médios dos lados do triângulo de área 4. A área do triângulo obtido é 1.
Raciocinando inversamente, se temos um triângulo de área 1 e se traçarmos
paralelas m, n, p aos lados AB, BC e CA (de modo que C  m, A n e B  p ),
respectivamente, obteremos um triângulo de área 4.
Agora não é difícil completar a solução. Considere todos os triângulos cujos
vértices são três quaisquer dos n pontos dados. Seja ABC o triângulo de maior área.
Trace as retas m, n, p como foi descrito anteriormente. Se o ponto A e o ponto X
onde X é um dos n pontos dados estão em diferentes lados de m, então
( ABX )  ( ABC ). Os outros casos são análogos. Segue-se que nenhum dos pontos
dados está fora do triângulo MNP triângulo formado pelas interseçcões de m, n, p.
Como ( ABC ) 1, então o triângulo MNP contém os pontos, e ( MNP ) 4.

4.- Aplicações Algébricas.

EUREKA! N°8, 2000


38
Sociedade Brasileira de Matemática

Problema 4.- Em cada quadrado de um tabuleiro com infinitas fileiras e colunas,


se escreve um número natural. O número escrito em cada quadrado é igual à média
dos números escritos em todos seus quadrados vizinhos (Dois quadrados são
vizinhos se eles compartilham um lado em comum.) Prove que todos os números
escritos são iguais.

Solução: Aqui aplicaremos o famoso resultado sobre conjuntos não vazios de


números naturais, o qual estabelece que sempre há um elemento mínimo. Seja f o
menor dos números naturais escritos no tabuleiro, e sejam a, b, c, d os números
escritos nos quatro quadrados vizinhos de f. Então
a b c d
f  ,
4
quer dizer a  b  c  d 4 f . Como f é o elemento mínimo, segue-se que
a  f , b  f , c  f e d  f . Se uma destas quatro desigualdades não é uma
igualdade, então teríamos a  b  c  d  4 f , o que é um absurdo. Portanto, se x é
um número escrito em uma casa da mesma coluna da casa na qual está escrito o
número f, então x = f. O mesmo resultado é válido para as linhas. Logo, todos os
números escritos são iguais.

Problema 5.- Em cada quadrado de um tabuleiro de m fileiras por n colunas, se


escreve um número real. O número em cada quadrado é igual a média dos números
escritos em todos seus quadrados vizinhos. (Dois quadrados são vizinhos se eles
compartilham um lado comum.) Prove que todos os números escritos são iguais.
A solução deste exercício é um pouco diferente da do exercício prévio.

Solução: Há duas coisas diferentes. Primeiro, alguns quadrados tem menos que
quatro quadrados vizinhos. O leitor pode adaptar facilmente a situação ao
raciocínio da solução do exemplo 4. Segundo, a existência do número menor se
baseia em uma razão diferente: cada conjunto finito de números possui um
elemento menor. Este é um assunto importante. Se estamos tratando com elementos
extremos, devemos estar certos de que existem, qualquer que seja a razão.

Problema 6.- Prove que não existem inteiros positivos x, y, z e t tais que
x 2  y 2 3( z 2  t 2 ).
Solução: Algumas vezes não é fácil imaginar como introduzir um elemento
extremo. Uma boa ideia nestes casos é assumir a negação da proposição, e ver onde
se pode encontrar uma contradição.

EUREKA! N°8, 2000


39
Sociedade Brasileira de Matemática

Assuma que existem inteiros positivos x, y, z e t tais que x 2  y 2 3( z 2  t 2 ). Já


que x 2  y 2 é divisível por 3, então x, y também são divisíveis por 3 (Prove).
Portanto, x = 3m e y = 3n, onde m, n são inteiros positivos. Depois de substituir 3m
por x e 3n por y na equação, e dividindo por 3, obtemos que
z 2  t 2 3(m 2  n 2 ).
Pela mesma razão que antes se conclui que z = 3p e t =3q, onde p, q são inteiros
positivos. Logo, a equação original é equivalente a
m 2  n 2 3( p 2  q 2 ).
Portanto, obtivemos inteiros positivos m, n, p e q, que satisfazem a equação, e tais
que m < x, n < y, p < z e q < t. O argumento anterior pode ser usado
indefinidamente para obter sucessões decrescentes de números inteiros positivos, o
que é impossível. Logo, a idéia é considerar o menor elemento, em algum sentido.
Sejam x, y, z e t inteiros positivos tais que x 2  y 2 3( z 2  t 2 ) e a soma ( x 2  y 2 )
é a menor entre todas as soluções da equação. Seguindo o raciocínio de antes
obteremos os números m, n, p e q, que satisfazem a equação, com m < x e n < y.
Portanto,
m2  n2  x2  y 2.
Esta é uma contradição.

5.- Aplicações Variadas.

A pergunta "Como começar a solução?" parece ser a principal pergunta nas


soluções dos problemas deste artigo. Espera-se que, quanto maior a quantidade de
exemplos que o leitor vir, maior será a experiencia ganha. Portanto exporemos
mais exemplos que ajudem a exemplificar o princípio do extremo.

Problema 7.- Existirá uma função f : N*  N*; onde N* é o conjunto dos inteiros
positivos tais que se cumpra a seguinte igualdade para cada número natural n > 1:
f (n)  f ( f (n  1))  f ( f ( n  1)) ?
Solução: A resposta é NÃO. Para ver isto observe que entre os valores
f ( 2), f (3),..., f ( n),...,
deve haver um elemento mínimo, digamos que seja f(n0), onde n0 > 1.
Observe que
f (n0  1)  f (n0 )  f ( f (n0  1))  f ( f (n0  1)) 1  1  1.

Como
f ( n0  1)  1, então f ( f (n 0  1))  { f ( 2), f (3),...}

EUREKA! N°8, 2000


40
Sociedade Brasileira de Matemática

Portanto, f ( f (n0  1))  f (n0 ), o que implica que


f (n0 )  f ( f (n0  1))  f ( f (n0  1)) 1  f ( n0 ),
o que é impossível.

Problema 8.- Cada quadrado de um tabuleiro de dimensões 8  8 contém ou um 0


ou um 1. Para cada quadrado A que contém um 0, a soma dos números na mesma
fileira de A e os números na mesma coluna de A é maior ou igual a 8. Prove que a
soma de todos os números no tabuleiro é maior ou igual a 32.

Solução: Considere a soma dos números em cada fileira e em cada coluna. Escolha
a menor destas somas. Suponha que tal soma corresponda à fileira L. Denote por k
o número de números 1 que aparecem em L. Podem ocorrer os seguintes casos:

 k  4. Então cada fileira contém ao menos quatro números 1. Portanto, a


soma de todos os números no tabuleiro é maior ou igual a 4  8 = 32.

 k < 4. Então existem 8 – k zeros em L. Cada coluna que cruza L em um


quadrado com um 0 contém não menos que 8 – k uns. Portanto, a soma de
todos os números no tabuleiro é maior ou igual a
(8  k ) 2  k 2 2(32  8k  k 2 ) 2((k  4) 2  16) 2 16 32.

Uma extensão do princípio do extremo é a seguinte regra: "ordene os elementos


segundo o seu tamanho (valor)". Esta regra é usada na solução do seguinte
problema.

Problema 9.- A soma de 17 inteiros positivos distintos é igual a 1000. Prove que
podem ser escolhidos 8 destes inteiros de tal forma que a sua soma é maior ou
igual a 500.

Solução: Ordene os inteiros em uma fileira a1  a 2  ...  a17 . Considere o


número a 9 que é a metade da fileira, e o valor médio da soma, parte inteira de
1000/17, que é 58. Podem ocorrer os seguintes casos:

 a9  58. Então a10  59, a11  60,..., a17  66. Portanto,


a10 + a11 +...+ a17  59 + 60 +...+ 66 = 500.

EUREKA! N°8, 2000


41
Sociedade Brasileira de Matemática

 a9 < 58. Então a9  57, a8  56,..., a1  49. Portanto,


a1 + a2 +...+ a9  49 + 50 +...+ 57 = 477.

Segue que a10 + a11 +...+ a17  1000 – 477 > 500.

Problema 10.- Encontre todas as soluções positivas do sistema


x1  x 2  x32 , x 2  x3  x 42 , x3  x 4  x52 , x 4  x5  x12 , x5  x1  x 22

Solução: Sejam x e y o maior e o menor dos números x1 ,..., x5 ,


respectivamente.
Observe que temos que x 2 2 x e y 2 2 y. Como x > 0 e y > 0 segue-se que
x 2 e que y 2, logo se conclui que
2  y  x  2.
Portanto, segue-se que a única solução do sistema é dada por
x1  x 2 ...  x5 2.

6.- Exercícios.

Nesta seção voce encontrará alguns problemas que são resolvidos por meio do
princípio do extremo, estando claro que pode haver outras soluções que não usem
este princípio. Mas pede-se ao leitor que faça todo o esforço possível para
resolver os seguintes exercícios usando unicamente o princípio do extremo.

1. Os números positivos x, y e z são tais que


2y 2z 2x
x ,y  ,z  .
1 y 1 z 1 x
Prove que x = y = z.

2. Seis círculos iguais num mesmo plano possuem um ponto em comum. Prove
que um dos círculos contém o centro de outro dos círculos.

3. Oito pontos são escolhidos dentro de um círculo de raio um. Prove que existem
dois pontos cuja distância é menor que 1.

4. A soma de vários números reais não negativos é 3, e a soma de seus quadrados


é estritamente maior que um. Prove que podem ser escolhidos três destes
números cuja soma é estritamente maior que um.

EUREKA! N°8, 2000


42
Sociedade Brasileira de Matemática

Referências

[1] María Falk de Losada, Problemas y Soluciones 1987-1991, Nivel


Superior, Universidad Antonio Nariño, Colombia, 1994.
[2] Eduardo Wagner, Carlos Gustavo T. de A. Moreira et al, 10 Olimpíadas
Iberoamericanas de Matemática, OEI, Madrid, 1996.
[3] Loren Larson, Problem -Solving Through Problems, Springer - Verlag,
New York, 1983.
[4] G. Polya, How to solve it, Princeton University Press, USA, 1965.
[5] D. O. Shklarsky, N.N Chentzov e I.M. Yaglom, The USSR Olympiad
Problem Book, Dover Publications, New-York, 1993.
[6] Ravi Vakil, A Mathematical Mosaic: patterns and problem-solving,
Bredan Kelly Publishing, Burlington, Ontario, 1996.

FUNÇÕES MULTIPLICATIVAS E A FUNÇÃO DE MÖBIUS*


Carlos Gustavo. T. de A. Moreira, IMPA  Nicolau Saldanha, PUC-Rio

 Nível Avançado

EUREKA! N°8, 2000


43
Sociedade Brasileira de Matemática

Recordamos inicialmente uma propriedade da função  de Euler, provada em [2]


(Lema 2, página 52). Lembremos que, para n inteiro positivo,
 (n) :# {a  Z / nZ | a é invertívelmódulo n } #{k  Z | 0 k  n e mdc (m, n) 1}.
Teorema 1: Para todo natural n,
  (d ) n.
d |n

Prova: Considere as n frações


0 1 n 1
, ,...,
n n n

e simplifique cada uma delas: obtemos assim, para cada d|n, (d) frações com
denominador d, donde segue a identidade do enunciado.

Mais formalmente, dado a  Z / nZ , sejam d = n/(n, a) e a' = a/(n, a).


Claramente a '  (Z / dZ)*, e definimos assim uma função de Z /(n) para a
união disjunta dos conjuntos (Z / dZ) * , onde d varia sobre os divisores de n. A
inversa desta função leva a '  (Z / dZ) * em a , com a na' / d , donde a
função é uma bijeção 

O processo de construir g a partir de f como


g ( n)  f ( d )
d |n

é bastante comum em teoria dos números Um fato interessante sobre este tipo de
construção é ligado à noção de funções multiplicativas. Dizemos que f : N  C
é multiplicativa se mdc(m, n) 1  f (mn)  f (m) f (n). A função  de Euler,
por exemplo, é multiplicativa (ver o corolário da página 47 de [2]). Se f é uma
função multiplicativa e n  p11 p 2 1 ... p k k é a fatoração prima de n, então

* Adaptado do livro Primos de Mersenne (e outros primos muito grandes), dos mesmos
autores([1]).
k
f ( n)  f ( p ). Além disso, vale a seguinte
i 1
i
i

EUREKA! N°8, 2000


44
Sociedade Brasileira de Matemática

Proposição: Se f : N  C é multiplicativa então g : N  C , g (n) 


d |n
f (d )
é
multiplicativa.

mdc(m, n) 1,g(mn)   f (d )  f (d1d 2 )  f (d1 ) f (d 2 ) 


Prova: Se d |mn d1 | m d1 | m
d 2 |n d 2 |n
  
  f ( d 1 )   f ( d 2 )   g ( m) g ( n ) 
 d |m  d | n 
 1  2 

Note que esta proposição fornece uma nova prova do Teorema 1: pela
multiplicidade de   ( d ) , basta provar que   (d ) n se n é potência de
d |n d |n

primo, mas se p é primo


k k k
  (d )   ( p j ) 1   ( p j ) 1  ( p j
k
 p j 1 )  p k .
d| p j 0 j 1 j 1

Seria interessante poder inverter em geral identidades do tipo g (n) 


d |n
f (d )

para escrever f a partir de g. O teorema anterior nos mostra que se fazemos f = 


na equação acima temos g(n) = n; invertendo esta identidade teríamos uma
fórmula para . Vamos mostrar como fazer este tipo de inversão.

Definimos a função de Möbius  : N  Z por

 ( 1) m , se n  p1 p2 ... pm , com p1 , p2 ,..., pm primos distintos,


 (n) 
 0, se n tem algum fator primo repetido em sua fatoração.
Assim, (1) = (6) = (10) = 1, (2) = (3) = (5) = (7) = – 1 e (4) = (8) =
(9) = 0. Note que  é uma função multiplicativa.

EUREKA! N°8, 2000


45
Sociedade Brasileira de Matemática

Lema: Para todo inteiro positivo n temos

1, se n  ,1
 (d)  ,0 se n  .1
d |n 

Dem: Como  é multiplicativa, h(n) 


d |n
 ( d ) é multiplicativa.

Temos h(1) = 1 e, para cada p primo e k  1 inteiro,


k
h( p k )   ( p j ) 1  ( 1) 0,
j 0

donde, se n > 1, n  p11 ... p k k  h(n) h( p11 )h( p 2 2 )h( p k k ) 0 

Teorema 2: (Fórmula de inversão de Möbius) Se para todo n > 0 temos


g ( n)  f ( d )
d |n

então
g (n)   ( n / d ) g ( d ).
d |n

Dem: Basta provar que


 
f ( n)   ( n / d ) 
  f ( d ' ) .
d |n  d '|d 

Mas, escrevendo d'' = n/d e m = n/d' temos

   
  (n / d )  f (d ' ) 
 
   ( d ' ' )  f (n / m)  f ( n) 
 
d |n  d '|d  m|n  d ''|m 

 (d )
Corolário: Para todo natural n,  (n)   ( n / d ) d n .
d |n d |n d
Teorema 1.22: (Segunda fórmula de inversão de Möbius) Sejam f e g funções
reais com domínio (0, +) tais que f(t) = g(t) = 0 para todo t < 1. Se

EUREKA! N°8, 2000


46
Sociedade Brasileira de Matemática

x

 x    x
g ( x)  f    f 
k 1  k  k 1 k

para todo x então, para todo x,


x

 x    x
f ( x)   (k ) g     ( k ) g   .
k 1  k  k 1 k
Prova: Basta provar que

   x 
f ( x )   ( k )  f    ,
k 1  r 1  kr  
mas, tomando m = kr a última soma é igual a
    x 
     (k )  f  m   ,
m 1   k |m  
que pelo lema é igual a f(x) 

Apesar de não estar relacionada com o resto da nossa discussão, não podemos
deixar de mencionar a seguinte conjectura.

Conjectura (Hipótese de Riemann): Se   1 / 2 então


n
1
lim
n  n 
  (m) 0.
m 1

Esta é uma das formulações da famosa hipótese de Riemann, um dos problemas


em aberto mais importantes da matemática.
Podemos reenunciar esta conjectura assim: seja f : (0,)  R definida por
f (t ) 0 se t < 1 e

 f (t / k ) 1, se t 1.
k 1
Então, para todo   1 / 2,
f (t )
lim 0.
n  t 

De fato, pela segunda fórmula de inversão de Möbius temos


t
f (t )   ( m).
m 1

EUREKA! N°8, 2000


47
Sociedade Brasileira de Matemática

[1] Carlos Gustavo T. de A. Moreira e Nicolau Saldanha, Primos de Mersenne (e outros


primos muito grandes), 22o. Colóquio Brasilerio de Matemática IMPA, 1999.

[2] Carlos Gustavo T. de A. Moreira, Divisibilidade, congruências e aritmética módulo n,


Eureka! No. 2, pp. 41-52.
OLIMPÍADAS AO REDOR DO MUNDO

 A partir deste número da EUREKA! apresentaremos esta nova seção cujo


objetivo é contemplar os leitores que não têm facilidade de acesso a problemas de
competições de outras nações com alguns exemplos de problemas, de tais
competições.

Nações distintas possuem culturas matemáticas distintas, portanto o leitor


pode achar alguns problemas extremamente fáceis e outros extremamente
difíceis. Tentaremos apresentar uma grande variedade de problemas
principalmente daqueles países que costumam ter um bom desempenho na
Olimpíada Internacional de Matemática. Divirtam-se e mandem suas soluções.

Antonio Luiz Santos


PROBLEMAS:

1. (Bulgária-1998) Seja f  x   x 3  3x  1 . Determine o número de soluções reais


distintas da equação f  f  x   0 .
2. (Repúblicas Tcheca e Eslovaca-1998) Determine todos os números reais x tais que
x  x  x  x    88 .
3. (Áustria/Polônia-1998) Considere todos os pares ordenados  a, b  de números
naturais tais que o produto a a b b , escrito na base 10, termina com
exatamente 98 zeros. Determine o par  a, b  para o qual o produto ab é o
menor possível.
4. (Reino Unido-1998) Em um triângulo ABC, D é o ponto médio de AB e E é um
ponto do lado BC tal que BE = 2EC. Sabendo que ADC = BAE determine
a medida do ângulo BAC.
5. (Turquia-1998) Seja  a n  uma seqüência de números reais definida por a1 t
e a n 1 4a n 1  a n  para n 1 . Para quantos valores distintos de t
teremos a1998 0 ?

EUREKA! N°8, 2000


48
Sociedade Brasileira de Matemática

6. (Rússia-1998) Um número de 10 algarismos é dito interessante se todos os seus


algarismos são distintos e ele é um múltiplo de 11111 . Quantos números
interessantes existem?
7. (Rússia-1998) Existem números de 5 algarismos M e N onde todos os algarismos
de M sejam pares, todos os algarismos de N sejam ímpares, cada um dos
algarismos de 0 a 9 ocorrendo exatamente uma vez entre M e N e tais que M
divide N?
8. (Romênia-1998) O volume de um paralelepípedo é 216cm 3 e a sua área total é
216cm 2 . Mostre que o paralelepípedo é um cubo.
9. (Irlanda-1998) Um triângulo ABC possui medidas dos lados expressas por
números inteiros, A = 2B e C > 90. Determine o valor mínimo do
perímetro deste triângulo.
10. (Canadá-1998) Em um triângulo ABC tem-se que BAC = 40 e ABC = 60.
Sejam D e E pontos sobre os lados AC e AB respectivamente tais que CBD
= 40 e BCE = 70. Mostre que a reta que contém AF é perpendicular à que
contém BC.
11. (China-1999) A base de uma pirâmide é um polígono convexo de 9 lados. Pinta-
se cada uma das diagonais da base e cada uma de suas arestas laterais de
preto ou de branco (observe que os lados da base não estão coloridos). Mostre
que existem três segmentos coloridos com a mesma cor que formam um
triângulo.
12. (Irlanda-1999) Três números a  b  c estão em progressão aritmética se
c  b b  a . Definamos a seqüência  u n  , n 0,1,2,3,... da seguinte
maneira : u 0 0 , u1 1 e para cada n 1 , u n 1 é o menor inteiro
positivo tal que u n 1  u n e  u 0 , u1 ,..., u n , u n 1  não possui três elementos
em progressão aritmética. Determine u100 .
13. (Irlanda-1999) Uma função f : N N satisfaz às condições :
f  ab   f  a  f  b  se o máximo divisor comum de a e b é 1,
f  p  q   f  p   f  q  para todos os números primos p e q.
Mostre que f  2  2, f  3 3 e f 1999 1999 .
14. (Suíça-1999) Determine todas as funções f : R\  0  R satisfazendo
1 1
f   x   f    x para todo x  R\  0 .
x  x
15. (Suíça-1999) Dois círculos intersectam-se em dois pontos M e N. Um ponto A
qualquer do primeiro círculo, distinto de M e N, é unido aos pontos M e N de
modo que as retas AM e AN intersectam novamente o segundo círculo nos

EUREKA! N°8, 2000


49
Sociedade Brasileira de Matemática

pontos B e C. Mostre que a tangente ao primeiro círculo em A é paralela a


BC.

16. (Estônia-1999) Mostre que o segmento que une o ortocentro e o baricentro de um


triângulo acutângulo ABC é paralelo ao lado AB se, e somente se,
tg A tgB 3 .

17. (Ucrânia-1999) Mostre que o número 9999999  1999000 é composto.

1 1
18. (Armênia-1999) Resolva a equação x 2  1
4  3x  2

19. (Lituânia-1999) Duas cordas AB e CD de um círculo intersectam-se no ponto K.


O ponto A divide o arco CAD em duas partes iguais. Se AK = a, KB = b,
determine a medida da corda AD.

20. (Espanha-1999) Mostre que existe uma seqüência de inteiros positivos


 a1 , a 2 ,..., a n ,... tal que a12  a 22  ...  a n2 é um quadrado perfeito para
todo inteiro positivo n.
21. (Estônia-1999) Determine o valor da expressão
 1   2   1999   2000   2000   2000 
f  f    f   f  f    f  
 2000   2000   2000   2000   1999   1 
x2
supondo que f  x   .
1  x2
1 1 1 1
22. (Eslovênia-1999) Inicialmente os números 1, , , ..., , são
2 3 1998 1999
escritos em um quadro negro.
Em cada passo, escolhemos dois destes números, digamos a e b, e os
substituímos pelo número a  b  ab . Continuamos desta maneira até que
reste um único número no quadro negro. É possível que este número seja
2000? Justifique sua resposta.
23. (Rússia-1999) A soma dos algarismos de um inteiro positivo n escrito no
sistema de numeração decimal é igual a 100 e a soma dos algarismos do
número 44n é 800. Determine a soma dos algarismos do número 3n .

EUREKA! N°8, 2000


50
Sociedade Brasileira de Matemática

24. (Rússia-1999) Um círculo que passa pelos vértices A e B de um triângulo ABC é


tangente ao lado BC, e o círculo que passa pelos vértices B e C e é tangente
ao lado AB intersecta o primeiro círculo no ponto K, K  B . Se O é o centro

do círculo circunscrito ao triângulo ABC, mostre que BKO  .
2
25. (Espanha-2000) Determine o maior número inteiro N que satisfaz as seguintes
condições :
N
(a)   possui seus três algarismos iguais.
3
N
(b)   é igual à soma de n números naturais consecutivos a partir de
3
1.
26. (Espanha-2000) A figura mostra um plano com ruas que
A delimitam 12 quadras quadradas. Uma
pessoa P caminha de A até B e outra Q
caminha de B até A.
Ambas partem ao mesmo tempo
seguindo caminhos de comprimento
mínimo com a mesma velocidade
constante.
Em cada ponto com duas possíveis
direções a tomar, ambas possuem a
B mesma probabilidade.
Determine a probabilidade de que P e
Q se cruzem.
27. (Polonia-2000) Prove ou disprove a seguinte afirmativa :
a 2  b3
Todo número racional positivo pode ser escrito sob a forma onde a,
c5  d 7
b, c e d são inteiros positivos.

28. (Polonia-2000) Seja I o incentro de um triângulo ABC com AB  AC. As retas


suportes dos segmentos BI e CI intersectam os lados AC e AB nos pontos D e
E respectivamente. Determine todos os ângulos BAC para os quais a
igualdade DI = EI pode ser satisfeita.

EUREKA! N°8, 2000


51
Sociedade Brasileira de Matemática

29. (Áustria/Polonia-1999) Determine todos os pares de inteiros positivos  x, y  tais


que x x  y  y y  x .
30. (Polonia-1998) Determine todos os pares de inteiros positivos  x, y  tais que
y x  x 50 .
31. (Baltic Way-1999) As bissetrizes dos ângulos A e B do triângulo ABC
intersectam os lados BC e CA nos pontos D e E respectivamente. Supondo
que AE  BD  AB , determine a medida do ângulo C.

SOLUÇÕES DE PROBLEMAS PROPOSTOS


 Publicamos aqui algumas das respostas enviadas por nossos leitores.

36) Na figura abaixo o triângulo DEF tem área de medida S. Sabendo-se que o
triângulo DEF está inscrito num triângulo arbitrário ABC, mostre que as
medidas Si ( i = 1, 2, 3) das áreas dos outros triângulos formados satisfazem a
3
S
desigualdade 1 1 1 e que a igualdade ocorre se e só se os pontos
 
S1 S 2 S 3
DEF são os pontos médios dos lados do triângulo, ABC.
A

S2
F
E
S
S3
S1
B C
D

Solução de Carlos Alberto da Silva Victor (Rio de Janeiro - RJ):

EUREKA! N°8, 2000


52
Sociedade Brasileira de Matemática

A ACb

z ABc
b–x BCa

F S2
E

c–z S x
S3 S1
C
B y
a–y D

3
S S S S
Provar que 1 1 1 é idêntico a mostrar que   3.
  S1 S 2 S 3
S1 S 2 S 3

Se k a área de ABC, temos então:

k x y k z (b  x) k (c  z ) (a  y )
S1  ; S2  ; S3  e
a b b c a c
S k  S1  S 2  S 3 .

Façamos também:
x z y
m1  ; m 2  e m3  e evidentemente teremos 0  m1  1 ; 0  m 2  1 ;
b c a
0  m3  1 ; e consequentemente: S1 k m1 m3 ; S 2 km 2 (1  m1 ) e
 1 1 1 
S 3 k (1  m 2 ) (1  m3 ). Seja  ( k  S1  S 2  S 3 )   .
 S1 S 2 S 3 

1 m3 (1m2 )


Portanto m m 1  m 2 1  m2  m1  m2  m3  1  m3
 2 3   3
m3 m 2 m1 (1  m 2 )(1  m3 ) 1  m1

ou seja:

EUREKA! N°8, 2000


53
Sociedade Brasileira de Matemática

m m 1  m2 m1 1  m3 1  m1
 2  3    3
m3 m 2 m1 1  m 2 1  m1 1  m3
             
1 2 3

como a soma de qualquer número positivo x com o seu inverso é sempre maior
do que 2 ou igual a 2, valendo a igualdade se e só se x = 1 ( de fato,
2
1  1 
x  2  x  0), teremos:  2  2  2  3 3. observe também
x  x
que só temos  1 2 quando m 2 m3 ;  2 2 quando m1 1  m 2 e que
1
 3 2 quando 1  m3 1  m1 ; ou seja m1 m 2 m3  , o que garante que
2
os pontos D, E e F são médios dos lados correspondentes e como consequência
teremos o mínimo de  3.
Conclusão:  3 e a igualdade ocorre para os pontos médios.

37) Cinco quadrados são dispostos conforme ilustra o diagrama abaixo.


Mostre que a medida da área do quadrado S é igual a medida da área do
triângulo T.

Solução de Geraldo Perlino (Itapecerica da Serra - SP):

EUREKA! N°8, 2000


54
Sociedade Brasileira de Matemática

S2 e
d
 


e
d a
b 

  b–a

 a
c b
c S1
 
a b a b

Prove : S1 = S2 S1 = c2 e a 2  b 2 c 2 (Pitágoras)
dh d
S2  h e sen(   )  S 2  e sen(   )
2 2
d
S 2  e ( sen cos   sen cos  ) ; d 2 b 2  4a 2 e e 2 a 2  4b 2 .
2

 b a
 sen d ; sen e d  2b2 2a2 
  S2  e   
cos  2a ;cos 2b 2  de de 
 d e
 S 2 a 2  b 2 c 2 .

38) Os lados e diagonais de um polígono regular de n lados são coloridos em k


cores tais que:
i) para cada cor a e dois vértices A e B do polígono, o segmento AB é colorido
de a ou existe um vértice C tal que AC e BC são coloridos de a.

EUREKA! N°8, 2000


55
Sociedade Brasileira de Matemática

ii) os lados de qualquer triângulo com vértices entre os vértices do polígono são
coloridos usando no máximo 2 cores.
Prove que k  2.

Solução:
Suponha que haja pelo menos 3 cores a, b e c. Vamos construir um subconjunto
infinito de vértices de X, o que é uma contradição.
Fixemos um vértice Z  X. Existe um vértice A1 tal que A1 Z tem a cor a, e um
vértice B1 tal que a cor de B1 A1 e de B1 Z é b. Existe um vértice C tal que as cores
de C1 Z e C1B1 são C. Considerando os triângulos C1 A1 Z e C1 A1 B1, e usando a
condição ii), concluímos que a cor de C1A1 tem que ser C.
Vamos construir por indução vértices An , B n , C n para cada inteiro positivo n,
todos distintos tais que, para todo i < n, as cores de ZAn , Ai An , Bi An e C i An
são a, as cores de ZBn , Ai Bn , Bi Bn , C i Bn e An Bn são b e as cores de
ZC n , Ai C n , Bi C n , C i C n , An C n e B n C n são c.
Suponhamos contruídos A j , B j , C j para 1  j n.
Por i), existe An 1 tal que as cores de An 1 B n e An 1C n são a.
Considerando os triângulos An 1 B n P e An1C n P, (e usando a condição ii),
concluímos que a cor de An 1 B n P tem que ser a, para cada ponto P criado
anteriormente. Do mesmo modo, existe B n 1 tal que as cores de B n 1 An 1 e
B n 1C n são b. Considerando os triângulos B n 1 An 1 P e B n1C n P, para
cada ponto P criado anteriormente, concluímos que a cor de B n 1 P tem que ser
b. Por fim, existe C n 1 tal que as cores de C n 1 B n 1 e C n 1 An 1 são c, e,
considerando os triângulos C n 1 B n 1 P e C n 1 An 1 P, para cada ponto P
criado anteriormente, concluímos que a cor de C n 1 P tem que ser c. É fácil ver
que os pontos criados são todos distintos. Por exemplo: como a cor de An Z é a,
temos An Bk e An C k para todo k. Como a cor de An Bn  1 é a, An Z ,
e como a cor de An C n  1 é a, An  A j , para todo j  n.

39) Sejam x, y e z os ângulos de um triângulo de lados opostos a, b e c


respectivamente. Prove que
 1 1 1 1 1 1 a b c
a    b    c   2    .
 y z  z x x y x y z

EUREKA! N°8, 2000


56
Sociedade Brasileira de Matemática

Solução:

c b

y a z

Suponha sem parda de generalidade que a b c.


1 1 1
Teremos por tanto x  y z , logo x  y  z .
1 1 1 1 1 1
Temos então (a  b)   0, (b  c)   0 e (c  a )   0.
y x z y x z
Somando essas 3 desigualdades obtemos a desigualdade do enunciado.

40) a) Calcular a soma dos divisores positivos de um número natural em termos


de sua fatoração prima.
b) Dizemos que n  1 é abundante se a soma de seus divisores é maior que 2n.
Prove que se n é abundante então kn é abundante para todo inteiro k  1.
c) Prove que existe n0  N tal que todo inteiro n  n0 pode ser escrito como
soma de dois números abundantes.

Solução de Marcio Afonso Assad Cohen (Rio de Janeiro - RJ):


a) Seja k  p11 p 2 2 p 3 3 ... p n n
Todo divisor de k é da forma p1a1 p 2a2 ... p nan com 0 ai  i , e
reciprocamente.
Portanto a soma de todos os divisores é:
1 2 n 1 n 1 n
an  1
S (k )    ...  ( p1a1 p 2a2 ... p nan )  ... ( p1a1
  p 2a2 ... p n  1 )  p nan
 
a1 0 a2 0 an 0 a1 0 an  1 0 an 0

(pois p1a1 ... p nan 11 é constante para o somatório em an).

EUREKA! N°8, 2000


57
Sociedade Brasileira de Matemática

1 n 1  p n n 1  1 
  ...  p1a1 p 2a2 ... p nan 11  
 p  1  (soma da P.G.)
a1 0 an  1  n 
 p n n 1  1  1 n 1  n 1
   ...  p1a1 ... p nan 11 . (pois p n  1 é constante em relação
 p  1  a 0 pn  1
 n 1 an  1

às variáveis a1 , a2 ,..., an  1 ).
Procedendo de maneira análoga, agora para o termo p n  1 , e assim por diante
obtemos:
 p11 1  1   p 2 2 1  1   p n n  1 
S (k )     ...  
 p 1   p 1   p  1  , que é o que queríamos.
 1   2   n 
S (k )
b) Vamos analisar a razão para k  p11 ... p n n
k
Temos que
 1
S (k ) 1  p1 1  1   p n n 1  1  (1  p1  p12  ...  p11 ) (1  p n  ...  p n n )
  ...    ...  
k k  p1  1   p 1  p11 p n n
 n 
 1 1 1   1 1 
1   2  ...    ... 1   ...   .
 p1 p 2 p1 1   pn p n n 
 
Agora se multiplicarmos k por m q1k1 q 2k 2 ... q sk s , duas coisas podem
acontecer:
i) Para cada primo p i que aparece na fatoração de k e de m , o fator
S ( kn) 1 1
referente a ele no produtório aumenta, pois,  i 1  0,...,  i  r  0
kn pi pi
 1 1   1 1 
e portanto, 1   ...     1   ...   r .

 pi pi  
i
pi pi i 
ii) Para cada primo q j que só aparece na fatoração de k, vemos que ao
S ( km)  1 1 
calcularmos , aparecerá um novo fator  1   ...  k j   1, de modo

km qj qj
 
que
S ( km) S ( k )
 .
km k

EUREKA! N°8, 2000


58
Sociedade Brasileira de Matemática

S ( km) S ( k )
Em qualquer caso portanto, vale  , m  N * . Em particular, k
km k
S (k ) S (km)
abundante  2  2.
k km

S ( km) S ( k )
c) Note que na letra b), vale a desigualdade estrita  para
km k
todo m 2.
S (6)  1  1 3 4
 Em particular, como  1   1     2,
6  2   3 2 3
vemos que todo múltiplo de 6, maior do que 6 é abundante. (pois
S (n) S (6t ) S (6)
n 6t ; t  1    2)
n 6t 6
 Logo, se para um natural N, existe N1 abundante tal que
N  N1 mod 6, e N  N1  6, então
N  N1 6t ; t  N  N  N1  6t;t  1  N pode ser escrito como soma de dois
números abundantes.

 Nosso problema se resume então a descobrir 6 números abundantes, dois


a dois distintos módulo 6: mas para isso é suficiente achar N abundante tal que
N 1 mod 6 (pois nesse caso 2 N 2 mod 6 e 2N é abundante pela letra b); e
analogamente, 3 N 3 mod 6,4 N 4;5 N 5 e 6 N 0 mod 6 são todos
abundantes e distintos módulo 6).
Também seria suficiente achar algum T abundante tal que T 5 mod 6, pois
nesse caso, 5T é congruente a 1 mod 6 e recaímos no caso anterior.

 Note agora, que todo número da forma


N 5 7  2 11 3 . (  1) 1 1 2 (  1)  3 ... mod 6  N 1 mod 6 ou
1

N  1 5 mod 6.
Obs: todo primo p > 3 é congruente a 1 ou –1 mod 6, pois do contrário teríamos:
p 2,4,0(mod 6)  p é par ou p 3(mod 6)  p é múltiplo de 3.
 O problema então fica sendo o de encontrar um número da forma

N 5 7 7 ... p p abundante.
5

Isso é possível mesmo se nos restringirmos apenas a números em que


 5  7 ...  p 1.

EUREKA! N°8, 2000


59
Sociedade Brasileira de Matemática

Basta ver que nesse caso,


S(N )  1  1  1  1   1
1    1    1   1  ...1   
N  5  7  11   13   p
S ( N ) 6 8 12 14 p 1
     ...  .
N 5 7 11 13 p
Em particular,
6 8 12 14 18 20 24 30 32
S (5 7 11 13 17 19 23 29 31)          
5 7 11 13 17 19 23 29 31
 6 8 12 14 18   20 24 30 32  145152 460800
      1,7059 1,1729  2,0008  2.
 5 7 11 13 17   19 23 29 31  85085 392863
Logo, N 5 7 11 13 17 19 23 29 31 é abundante.
(temos N (  1) 1 (  1) 1 (  1) 1 (  1) (  1) 1 5 mod 6)
Tomando portanto N 0 5 N , temos N 0 ,2 N 0 ,3 N 0 ,4 N 0 ,5 N 0 ,6 N 0 são
abundantes distintos módulo 6. Fazendo então n0 6 N 0  6, vemos pelas
observações anteriores que n  n0 tem-se que n pode ser escrito como soma
de dois números abundantes!.

PROBLEMA "CUÁTICO" (Publicado na Eureka! No. 5):


Prove que para qualquer conjunto de inteiros positivos A e para todo inteiro
positivo k existe um conjunto infinito de números primos S tal que o produto de k
elementos distintos de S está sempre em A ou o produto de k elementos distintos
de S nunca pertence a A.

Solução de Daniel Massaki Yamamoto (São Paulo - SP):


Considere o Conjunto P formado por todos os primos.
Para todo subconjunto de P com k elementos, pinte-o de azul se o produto destes
pertencer a A e de vermelho caso contrário. Pelo Teorema de Ramsey Infinito,
existe um subconjunto infinito de P tal que todos os seus subconjuntos de k
elementos são da mesma cor, ou seja os produtos de seus elementos sempre
pertencem ou nunca a A. Chamando-o de S, acabamos o problema.

Agradecemos também o envio das soluções e a colaboração de:

Alex Correa Abreu (Niteroi - RJ)


Carlos A. Gomes (Natal - RN)
Diego Alvarez Araújo Correia (Fortaleza - CE)

EUREKA! N°8, 2000


60
Sociedade Brasileira de Matemática

Estillac Lins Maciel Borges Filho (Belém - PA)


Fabrício Siqueira Benevides (Fortaleza - CE)
Fernando Carvalho Ramos (Santa Maria - RS)
Geraldo Perlino Júnior (São Paulo - SP)
José Clovis Adão Macedo (Matão - SP)
José Guilherme Moreira Pinto (Juiz de Fora - MG)
Luciano Marinho Filho (Recife - PE)
Luiz Fernando Athayde Júnior (Rio de Janeiro - RJ)
Marcelo Rufino de Oliveira (Belém - PA)
Nijair Araújo Pinto (Fortaleza - CE)
Osvaldo Mello Sponquiado (Olímpia - SP)
Paulo de Sousa Sobrinho (Natal - RN)

Errata:
O problema No. 4 (Olimpíada Romênia 92) publicado Na Eureka! N o. 6, pág 37,
deveria dizer: Sejam p, q  C, q  0. Se as raízes da equação x 2  px  q 0 têm
p2
o mesmo módulo, mostre que é um número real.
q
O problema No. 8 (Olimpíada Hungria 1899) publicado Na Eureka! N o. 6, pág 38,
deveria dizer: A0 , A1 , A2 , A3 , A4 dividem a circunferência unitária em cinco
partes iguais. Prove que ( A0 A1 A2 A4 ) 2 5.

Você sabia…
Que foram recentemente batidos os recordes de maior par de
primos gêmeos (p, p +2) conhecido? São eles 4648619711505·
260000 ± 1, descobertos este ano por Wassing, Jarai e Indlekofer, e
têm 18075 dígitos cada. Também tem 18075 dígitos o maior
primo conhecido p tal que 2p + 1 também é primo (tais primos
são conhecidos como primos de Sophie Germain). É o número
3714089895285 · 260000 – 1, descoberto pelos mesmos Wassing,
Jarai e Indlekofer. Este é o maior primo conhecido p tal que o
número de Mersenne 2p – 1 é composto (de fato é divisível por
2p +1; veja o problema 43 proposto na página 60).

PROBLEMAS PROPOSTOS

 Convidamos o leitor a enviar soluções dos problemas propostos e sugestões de novos


problemas para os próximos números.

EUREKA! N°8, 2000


61
Sociedade Brasileira de Matemática

41) Se a e b são números reais positivos, então a b  b a  1.

42) Suponha que a, b e c são as medidas dos lados de um triângulo ABC, com
semi-perímetro p e área S, verifique que
1 1 1 3 p
   
a b c 2 s
e mais ainda: verifique que a igualdade acima ocorre apenas se o triângulo
for equilátero.

43) Prove que se p é um primo da forma 4k + 3, então 2p + 1 também é primo se


e somente se 2p + 1 divide 2p – 1.

44) O produto de dois inteiros positivos consecutivos pode ser igual ao produto
de dois inteiros positivos consecutivos pares?

45) Existe uma seqüência infinita de:


a) Números reais
b) Números inteiros
Tais que a soma de quaisquer dez termos consecutivos é positiva, enquanto
que para todo n a soma dos primeiros 10n + 1 termos consecutivos é
negativa?

46) (Baltic Way, 1997)


i) Prove a existência de dois conjuntos infinitos A e B, não necessariamente
disjuntos, de inteiros não negativos tais que cada inteiro não negativo pode
ser representado de uma única forma como a + b, com a  A e b  B.

ii) Prove que em cada tal par (A, B), ou A ou B contém apenas múltiplos de
algum inteiro k > 1.

Problemas 41 e 42 propostos por Carlos Alexandre Gomes da Silva (Natal - RN), problemas
44 e 45 obtidos do 21o. Torneio das Cidades - Primaveira 2000.

AGENDA OLÍMPICA

XI OLIMPÍADA DO CONE SUL


14 a 19 de abril de 2000

EUREKA! N°8, 2000


62
Sociedade Brasileira de Matemática

Montevidéu – Uruguai

VI OLIMPÍADA DE MAIO
13 de maio de 2000

XXII OLIMPÍADA BRASILEIRA DE MATEMÁTICA - 2000


Primeira Fase – Sábado, 10 de junho
Segunda Fase – Sábado, 02 de setembro
Terceira Fase – Sábado, 21 de outubro (níveis 1,2 e 3)
Domingo, 22 de outubro (nível 3 - segundo dia).

XLI OLIMPÍADA INTERNACIONAL DE MATEMÁTICA


13 a 25 de julho de 2000
Taejon, Coréia do Sul.

XV OLIMPÍADA IBEROAMERICANA DE MATEMÁTICA


16 a 24 de setembro de 2000
Caracas, Venezuela

III OLIMPÍADA IBEROAMERICANA DE MATEMÁTICA UNIVERSITÁRIA


7 de outubro de 2000

COORDENADORES REGIONAIS

Amarisio da Silva Araújo (UFV) Viçosa - MG


Alberto Hassen Raad (UFJF) Juiz de Fora - MG

EUREKA! N°8, 2000


63
Sociedade Brasileira de Matemática

Angela Camargo (Centro de Educ.de Adultos - CEA) Blumenau - SC


Benedito T. Vasconcelos Freire (UFRN) Natal - RN
Claudio Arconcher (Col. Leonardo da Vinci) Jundiaí - SP
Claus Haetinger (UNIVATES) Lajeado - RS
Crescêncio das Neves (UFAM) Manaus-AM
Élio Mega (Col. ETAPA) São Paulo - SP
Enzo Marcom Takara (Col. Singular) Santo André - SP
Florêncio F. Guimarães Filho (UFES) Vitória - ES
Francisco Dutenhefner (UFMG) Belo Horizonte - MG
Gisele de A. Prateado Gusmão (UFGO) Goiânia - GO
Ivanilde H. Fernandes Saad (U. Católica Dom Bosco) Campo Grande - MS
Jacqueline F. Rojas Arancibia (UFPB) João Pessoa - PB
João Benício de Melo Neto (UFPI) Teresina - PI
João F. Melo Libonati (Grupo Educ. IDEAL) Belém - PA
Irene Nakaoka (UEM) Maringá - PR
José Carlos Pinto Leivas (UFRG) Rio Grande - RS
José Cloves Saraiva (UFMA) São Luis - MA
José Gaspar Ruas Filho (ICMC-USP) São Carlos - SP
José Luis Rosas Pinho (UFSC) Florianópolis - SC
José Paulo Carneiro (Univ. Santa Úrsula) Rio de Janeiro - RJ
José Vieira Alves (UFPB) Campina Grande - PB
Leonardo Matteo D'orio (Sistema Titular de Ensino)Belém - PA
Licio Hernandes Bezerra (UFSC) Florianópolis - SC
Luzinalva M. de Amorim (UFBA) Salvador - BA
Marcondes Cavalcante França (UF Ceará) Fortaleza - CE
Pablo Rodrigo Ganassim (L. Albert Einstein) Piracicaba - SP
Paulo H. Cruz Neiva de L. Jr. (Esc. Tec.Everardo Passos) SJ dos Campos - SP
Reinaldo Gen Ichiro Arakaki (INPE) SJ dos Campos - SP
Ricardo Amorim (Centro Educ. Logos) Nova Iguaçu - RJ
Roberto Vizeu Barros (Colégio ACAE) Volta Redonda - RJ
Sergio Claudio Ramos (IM-UFRGS) Porto Alegre - RS
Seme Gebara Neto (UFMG) Belo Horizonte - MG
Silvio de Barros Melo (UFPE) Recife - PE
Tadeu Ferreira Gomes (U. do Estado da Bahia) Juazeiro - BA
Tomás Menéndez Rodrigues (U. Federal de Rondonia) Porto Velho - RO
Valdenberg Araújo da Silva (U. Federal de Sergipe) São Cristovão - SE
Wagner Pereira Lopes (Esc. Tec. Fed. de Goiás) Jataí - GO
Waldemar M. Canalli (P.M. S. João de Meriti) S. João de Meriti - RJ

EUREKA! N°8, 2000


64
CONTEÚDO

AOS LEITORES 2

XV OLIMPÍADA IBEROAMERICANA DE MATEMÁTICA 3


Problemas e Soluções

XLI OLIMPÍADA INTERNACIONAL DE MATEMÁTICA 16


Problemas e Soluções

ARTIGOS

BRAHMAGUPTA PARA TODOS 28


José Cloves Verde Saraiva

EQUAÇÕES DE RECORRÊNCIA 33
Héctor Soza Pollman

EQUAÇÕES FUNCIONAIS 41
Eduardo Tengan

OLIMPÍADAS AO REDOR DO MUNDO 45

SOLUÇÕES DE PROBLEMAS PROPOSTOS 55

PROBLEMAS PROPOSTOS 59

COORDENADORES REGIONAIS 60

RECADASTRAMENTO PARA COLÉGIOS 2001 61


Sociedade Brasileira de Matemática

AOS LEITORES
Países de grande tradição nas olimpíadas de Matemática têm o seu
excelente rendimento em muito baseado na existência de revistas de divulgação
para estudantes pré-universitários. Apenas citando alguns exemplos, a húngara
KöMaL, a russa (agora também americana) Kvant (Quantum), a romena Gazeta
Matematica e a canadense (lida por alguns, poucos, brasileiros) Crux
Mathematicorum desenvolvem em seus países o ambiente ideal para o
aparecimento e desenvolvimento de jovens talentosos. Nós queremos que a
Eureka! tenha o mesmo sucesso que essas revistas.
Para isso estamos tentando trazer artigos cada vez mais interessantes e
elucidativos, e problemas cada vez mais bonitos e desafiantes para todos os
leitores da Eureka! (inclusive os dos Níveis 1 e 2), porém tal sucesso não depende
exclusivamente de nós. É fundamental a participação (muito) ativa de todos os
leitores. Ficamos bastante contentes com a repercussão da seção "Olimpíadas ao
Redor do Mundo", com vários leitores enviando resoluções dos problemas
propostos. Mas queremos que todas as outras seções e artigos tenham a mesma
repercussão. Por exemplo, vários artigos trazem problemas propostos porém
raramente recebemos resolução desses problemas. Ficaremos contentes em
recebê-las e poderemos saber se o público-alvo os entendeu.
Assim, faça-nos saber todo o estudo(!) e a diversão(!!) que cada uma das
Eureka! proporcionou (não se esqueça dos números antigos).

INSTRUÇÕES PARA AUTORES


Serão publicados na revistas Eureka! artigos relevantes na preparação dos
estudantes para a Olimpíada Brasileira de Matemática em seus diversos níveis e
para várias olimpíadas de caráter internacional das quais o Brasil participa.
Como para a grande maioria dos tópicos e técnicas explorados nas
olimpíadas não existem publicações expositórias adequadas em língua
portuguesa, nosso objetivo inicial é abordá-los todos em artigos auto-suficientes.
Assim, daremos preferência àqueles que tratem de assuntos ainda não explorados
nos números anteriores da Eureka!. Como a deficiência em artigos adequados
para estudantes do Ensino Fundamental (Níveis 1 e 2 da OBM) é ainda mais
grave, estes terão primazia na sua publicação. Vale a pena observar que, quando
um tema é importante para os estudantes de diversos níveis, ele deve aparecer em
artigos adequados para cada um desses níveis, separadamente.
É recomendável que os artigos tragam alguns problemas resolvidos
detalhadamente e referências que o complementem ou aprofundem.

EUREKA! N°9, 2000


2
Sociedade Brasileira de Matemática

Os Editores.
XV OLIMPÍADA IBEROAMERICANA DE MATEMÁTICA
16 a 24 de setembro, Caracas - Venezuela

A XV Olimpíada Iberoamericana de Matemática foi realizada em


Caracas, Venezuela no período de 16 a 24 de setembro de 2000. A equipe
brasileira foi liderada pelos professores Ralph Costa Teixeira (Rio de Janeiro -
RJ) e Eduardo Tengan (São Paulo - SP). Nesta oportunidade a equipe brasileira
obteve a maior pontuação entre os países participantes além da Copa Puerto Rico,
prêmio entregue ao país com maior progresso nos últimos dois anos na
competição.

RESULTADOS DA EQUIPE BRASILEIRA

BRA1 Daniel Nobuo Uno Prata


BRA2 Daniel Massaki Yamamoto Ouro
BRA3 Fabrício Siqueira Benevides Ouro
BRA4 Humberto Silva Naves Ouro

PRIMEIRO DIA

PROBLEMA 1
Constrói-se um polígono regular de n lados ( n  3 ) e enumeram-se os vértices de
1 a n. Traçam-se todas as diagonais do polígono. Demonstrar que se n é ímpar, é
possível associar a cada lado e a cada diagonal um número inteiro de 1 a n, tal
que se verifiquem simultaneamente as seguintes condições:

1. O número associado a cada lado ou diagonal seja diferente dos números dos
seus vértices.
2. Para cada vértice, todos os lados e diagonais que nele se intersectem tenham
números diferentes.

SOLUÇÃO DE HUMBERTO SILVA NAVES (SÃO PAULO - SP)

Existe um jeito simples de pintar os lados e as diagonais do polígono regular.


Escolhe-se um ponto do polígono regular com o número "i" associado à este
ponto, que chamaremos de Pi.

EUREKA! N°9, 2000


3
Sociedade Brasileira de Matemática

Seja O o centro da circunferência circunscrita ao polígono regular.

i Pi

x1 i x1'

O
x2 i x2'

Fig. 1

Sabemos que a reta Pi O é eixo de simetria dos pontos do polígono regular e


nenhum ponto do polígono está sobre a reta Pi O (pois n é ímpar). Digamos
esses pares de pontos simétricos e numeramos com o número "i", ou seja
pegamos um outro ponto qualquer (diferente de Pi) do polígono regular, ligamos
com o seu respectivo simétrico e numeramos com o número "i" (fazemos isso
para todos os outros pontos). Vide fig. 1.
Este é um exemplo do processo para o ponto Pi.
Repetimos esse processo para todos os pontos do polígono regular.
Desta forma pintamos todos os lados e todas as diagonais, pois:
De uma forma geral (e mais simples):
Se quisermos saber o número de um segmento Pi P j do polígono regular, basta
traçarmos a mediatriz desse segmento, que vai certamente encontrar um outro
vértice X do polígono regular (pois n é ímpar) e o segmento Pi P j vai ter o
mesmo número de X.
Esta forma de numeração claramente satisfaz as condições do enunciado pois:

1) Não existe nenhum segmento com o mesmo número de um de seus


vértices, pois a mediatriz desse segmento não passa pelos vértices do
segmento.

2) Para cada vértice, todos os lados e todas as diagonais que incidem neste
vértice tem números diferentes, pois se existissem dois segmentos de
mesmo número, teríamos:

EUREKA! N°9, 2000


4
Sociedade Brasileira de Matemática

Pj
O ponto Pw (que deve estar na
mediatriz de Pi P j e na mediatriz de
w Pi Pt ) é o próprio O, um absurdo.
Pw= O Pi
Logo vale a propriedade 2 e isso acaba
w
o problema.

Pt

PROBLEMA 2
Sejam S1 e S2 duas circunferências de centros O1 e O2, respectivamente, secantes
em M e N. A reta t é a tangente comum a S1 e S2, mais próxima de M. Os pontos
A e B são os pontos de tangência de t com S1 e S2, respectivamente, C é o ponto
diametralmente oposto a B e D é o ponto de interseção da reta O1O2 com a reta
perpendicular à reta AM que passa por B. Demonstrar que M, D e C são
colineares.

SOLUÇÃO DE DANIEL MASSAKI YAMAMOTO (SÃO PAULO - SP)

BC é diâmetro  BMˆ C 90


ˆ D BM
Para provar [MDC ], basta provar que BM ˆ C 90

B

A 90 –
 M
2
1

O1 D O2

EUREKA! N°9, 2000


5
Sociedade Brasileira de Matemática

Geometria Analítica: C (n, 22)


Dados:
M = (0, 1)
A = (–m, 0)
N
O1 = (–m, y1)
B = (n, 0) O2
O2 = (n, y2) D
Quero determinar D: O1 2

1
M
1
A m0 n B
= (–m, 0) = (n, 0)

1
AM : y  ( x  m)
m
BD :  AM
y
BD : y  m( x  n)  x  n
m
y 2  y1
O1O2 : y  y 2  ( x  n)
mn
BD  O1O2 {D}
y 2  y1  y 
y  y2   
mn  m
 y  y1 
y 1  2  y2
 ( m  n)m 
 m( m  n) 
y D  y 2  
 m( m  n)  y 2  y 1 
y 2 ( m  n)
x 0  n
m( m  n )  y 2  y 1
 y 2 m(m  n)  m( m  n)  y 2  y1 
 
y D  1  m(m  n)  y 2  y1 
coef. angular de DM : 
xD  mn(m  n)  n( y 2  y1 )  y 2 (m  n) 
 
 m(m  n) y 2  y1 

EUREKA! N°9, 2000


6
Sociedade Brasileira de Matemática

 n2 1  n2 1  m2 1


 2  m m  n   m  mn 
2
     
 2   2 
     
 n2  m2   n2 1
mn m  n   n 

 
 2  ( m  n)
 2   

n2 2 m 2  mn  n2 m2 n m n
2

m  mn   
2  2 
2
  m  mn 
2

2
n  (m 2  mn) 
2

2 2 
  
 n( n  m) n 2 1   mn  1 
 m  n  mn     ( m  n) 
 2 2 2  2 
n
 mn(m  n)  (m  n) n(m  n)(mn  1)
 2  n
 mn  1  ( m  n)(mn  1)
( m  n) 
 2 

1
coef. ang. de BM : 
n
 1
n     1 , logo
 n
BM  DM .

PROBLEMA 3
Encontrar todas as soluções da equação

 x  1 y  x z 1
para x, y, z inteiros maiores que 1.

SOLUÇÃO
Considerando a equação módulo x  1 , obtemos
  1 z 1(mod x  1),
o que implica que z é ímpar. Logo
(1)
 x  1 y  x z 1  ( x  1) y x z  1  ( x  1) y ( x  1)( x z  1  x z  2  ...  x  1) 
(2)  x  1 y  1  x z  1  x z  2  ...  x  1 e, então, x é par pois, caso contrário, os
dois lados de (2) seriam de paridade oposta.

EUREKA! N°9, 2000


7
Sociedade Brasileira de Matemática

Analogamente, escrevendo (1) na forma


 x  1 y  1  ( x  1) y  2  ...  ( x  1)  1  x z  1 ,
nós vemos que y é par também.
Sejam, portanto, x 2x1 e y 2 y1 Então por (1),
(3) (( x  1) y1  1)(( x  1) y1  1)  x z .
Temos ainda que x | ( x  1) y1  1 e, como x é par, mdc
(( x  1) y1  1, ( x  1) y  1) 2.
Assim de (3),
( x  1) y1  1 2 x1z
(4).
( x  1) y1  1 2 z  1
Conseqüentemente, 2 z  1  2 x1z , ou seja, x1 1  x = 2 e, de (4), y 2 e
z 3.
P.S. É gratificante saber que tal equação estudada por grandes matemáticos desse
século como W.J. Leveque e A. Schinzel, pode ser resolvida por brilhantes
estudantes pré-universitários em um tempo tão limitado quando o de uma prova
de olimpíada.

SEGUNDO DIA

PROBLEMA 4
De uma progressão aritmética infinita 1, a1 , a 2 ,  de números reais,
eliminam-se termos, obtendo-se uma progressão geométrica infinita
1, a n1 , a n2 ,  de razão q. Determinar os possíveis valores de q.

SOLUÇÃO DE DANIEL NOBUO UNO (SÃO PAULO - SP)

PG = 1, 1  m1 r ;1  m 2 r ;1  m3 r... com mi  Z  ; apenas


1  m1 r 1  m 2 r 1  mi 1 r
m0 0  q   ...  .
1 1  m1 r 1  mi r
 (1  m2 r ) 1 (1  m1 r ) 2  1  m 2 r 1  m12 r 2  2m1 r.

EUREKA! N°9, 2000


8
Sociedade Brasileira de Matemática

Supondo
m2
 2
2 m2 m1
r 0; m 2 m1 r  2m1  m 2 m1 (m1 r  2)  m1 r  2  r
m1 m1
a
Como m1 ; m2  Z  , então r é racional; Seja r  ; onde a; b  Z ; b 0, a
b
e b primos entre si.
Daí: q 1  m1 r .
m1 r  Q  q  Q.
c
Seja q  ; c; d  Z; d 0 c e d primos entre si.
d
Daí teremos
a ki a  b
PA : 1  k i  
b b
j
 c  cj
PG:    j
d  d
Análise do denominador:

ki a  b
PA: ; k i a  b  Z  o máximo do denominador é b.
b
cj
PG: como mdc(c; d) = 1  o denominador é d j .
j
d
Podemos pegar um j suficientemente grande tal que d j  b
Logo, a PG não pode ser infinita para d  1.
Logo; como d  Z  d 1 ou d = –1.
Se d 1  q c  razão inteira, e para isso, é só pegar r = 1 e tirar os termos
desnecessários.
A PA será sempre crescente ou sempre decrescente  p r 0  se d = –1.

PROBLEMA 5
Dois jogadores, alternadamente, retiram pedras de um conjunto de 2000 pedras,
de acordo com as seguintes regras:

1. Em cada jogada pode-se retirar 1, 2, 3, 4 ou 5 pedras.

EUREKA! N°9, 2000


9
Sociedade Brasileira de Matemática

2. Em cada jogada, um jogador não pode retirar o mesmo número de pedras que
o seu adversário retirou na jogada anterior.

O jogador que, na sua vez, não puder jogar de maneira válida, perde. Determinar
que jogador tem uma estratégia que lhe garanta a vitória e encontrar essa
estratégia.

SOLUÇÃO DE DANIEL MASSAKI YAMAMOTO (SÃO PAULO - SP)

n\ m 1 2 3 4 5 n\ m 1 2 3 4 5
0 0 0 0 0 0 24 1 1 1 0 1
1 0 1 1 1 1 25 1 1 1 1 0
2 1 1 1 1 1 26 0 0 0 0 0
3 1 1 0 1 1 27 0 1 1 1 1
4 1 1 1 0 1 28 1 1 1 1 1
5 1 1 1 1 0 29 1 1 0 1 1
6 1 1 0 1 1 30 1 1 1 1 1
7 0 0 0 0 0 31 1 1 1 1 0
8 1 1 1 1 1 32 1 1 0 1 1
9 1 1 1 1 1 33 0 0 0 0 0
10 1 1 1 1 1 34 0 1 1 1 1
11 1 1 1 0 1 35 1 1 1 1 1
12 1 1 1 1 0 36 1 1 1 1 1
13 0 0 0 0 0 37 1 1 1 0 1
14 0 1 1 1 1 38 1 1 1 1 0
15 1 1 1 1 1 39 0 0 0 0 0
16 1 1 0 1 1 40 0 1 1 1 1
17 1 1 1 1 1 41 1 1 1 1 1
18 1 1 1 1 0 42 1 1 0 1 1
19 1 1 0 1 1 43 1 1 1 1 1
20 0 0 0 0 0 44 1 1 1 1 0
21 0 1 1 1 1 45 1 1 0 1 1
22 1 1 1 1 1 46 0 0 0 0 0
23 1 1 1 1 1

 0 se quem vai jogar agora perde (com ambos jogando o melhor possível).
G(n, m) 
1 se ganha.
n = atual
m = movimento anterior

G (1, 1) = 0

EUREKA! N°9, 2000


10
Sociedade Brasileira de Matemática

A fórmula recursiva é

 0 se para todo 1 i 5, n i, i m, G(n  i, i) 1 (isto é, se o outro ganha



G(n, m)  para qualquer movimento)
1 caso contrário.

Para 1 n 5, G ( n, m) 1 se m n
Podemos definir G (0, m) = 0

Visualizando:
Como preencher a linha n: veja os quadrados com (*)

(*)
(*)
(*)
(*)
(*)
n

Se foram todos 1's, a linha n terá só zeros.


Se tiver dois ou mais zeros, a linha n terá todos 1's.
Se for zero, todos serão 1's, a menos da que está na mesma coluna do zero.

Sempre perde:
7, 13, 20, 26, 33, 39, 46

Parece que para n = 7 ou 13 (mód 13) sempre perde (G(n, m) = 0)


Basta ver que há um bloco 5  5 da tabela que se repete, (tirando as 6 primeiras
linhas).
Agora vamos provar a fórmula recursiva.
Primeiras 5 linhas:

- G (n, m) = 1 se m  n, pois ele pode tirar n pedras e ganhar.


- G (1, 1) = 0, pois não pode jogar.
- G (2, 2) = 1, pois tirando uma pedra, forçará o outro a ficar com G (1, 1),...

EUREKA! N°9, 2000


11
Sociedade Brasileira de Matemática

- G (i, i) = 0, i = 3, 4 ou 5, pois não importa como jogue o outro pode ganhar.

Agora, a fórmula nos outros casos:

Tirando i pedras, se você está com G(n, m), o adversário vai ficar com G(n – i, i).
Se algum deles for 0, o adversário vai perder e você ganha. Se todos forem 1, o
adversário sempre ganha.
Note que você não pode tirar m pedras, então não pode deixar o adversário com
G(n – m, m).
Na tabela, o bloco que se repete é 13 n 17, igual a 26 n 30.

Então o primeiro jogador ganha, pois se ele tirar 4 pedras na primeira jogada, vai
deixar o outro com 1996 pedras.
1996 7(mod 13)  o segundo perde.

PROBLEMA 6
Um hexágono convexo é bonito se tem quatro diagonais de comprimento 1 cujos
extremos contêm todos os vértices do hexágono.

(a) Para cada número k maior que 0 e menor ou igual a 1, encontrar um


hexágono bonito de área k.
3
(b) Demonstrar que a área de um hexágono bonito qualquer é menor que .
2

SOLUÇÃO

Consideremos todos os hexágonos bonitos ABCDEF com lados opostos paralelos


tais que os quatro segmentos dados de comprimento 1 são AC , CE , BF e
FD.
Seja   BFD ACE (figura (a)).

A B A' A B

F  C F C' C

E D E' E D

(a) (b)

EUREKA! N°9, 2000


12
Sociedade Brasileira de Matemática

Translademos os segmentos AC e CE na direção de AB para obter A' C '


e C ' E ' respectivamente de forma que A' , F , E ' sejam colineares do mesmo
modo que B, C ' , D. A área do hexágono ABCDEF é igual a área do retângulo
A'BDE' que por sua vez é igual a sen  (figura (b)). Dado que quaisquer
k  (0,1] é igual a sen  para algum  com 0 <   90 então temos um
hexágono bonito que cumpre o pedido.

b) Para demonstrar o resultado pedido, enunciamos os seguintes lemas sem


demonstração:
1- A soma dos comprimentos de um par de lados opostos de um quadrilátero
convexo é menor que a soma dos comprimentos das diagonais.
2- A área de um quadrilátero convexo é menor ou igual que a metade do produto
das suas diagonais.
Utilizaremos por simplicidade a seguinte terminología:
Sabemos que no hexágono há quatro diagonais de comprimento 1 cujos extremos
contém todos os vértices do mesmo. Chamaremos diagonais principais as
diagonais em questão. Definamos o grau de um vértice de um hexágono bonito
como o número de diagonais principais que passam por ele. Além disso
denotaremos por diagonal que divida ao hexágono nos quadriláteros.
Dado que a soma dos graus dos vértices é 8 e o grau de cada um é pelo menos 1,
temos dois casos:
a) Um vértice tem grau 3 e os restantes têm grau 1.
b) Dois vértices têm grau 2 e os restantes têm grau 1.
No primeiro caso, dado que há exatamente três diagonais que saem de um mesmo
vértice num hexágono, só há um possível tipo de configuração (salvo
"issomorfismo" de grafos):
1
Pelo lema 2 a área do hexágono é menor ou igual que ( AB  AC ) e isto é
2
1
menor que ( 2 AX  BC ) pela desigualdade triangular. Como
2
3
AX 1, BC 1, a área é menor que .
2

EUREKA! N°9, 2000


13
Sociedade Brasileira de Matemática

A
M N

X
B C
O

Para o segundo caso, sejam P e Q os vértices de grau máximo. Consideremos as


possibilidades:
c) Os vértices P, Q estão conectados por uma diagonal principal. Nesse caso se a
diagonal que os conecta não é central, as outras diagonais principais que saem
destes dois vértices devem estar num mesmo lado de PQ . Além disso, alguma
dessas diagonais deve ser central, pois caso contrário, chegariam a um mesmo
ponto e nesse caso se formaria um triângulo equilátero com diagonais principais
como lados e sabemos que isso não pode ocorrer pois haveria três vértices de
grau 2. Portanto, dependendo se ambas diagonais são centrais ou só uma é, se
apresentam as seguintes configurações:
C
No desenho à direita, AB 1 pelo lema
B
1. Logo a área do hexágono é menor ou A
igual que
1 1
( AB CQ  PQ AD )  (CQ  AD ) P Q
2 2
e isto é menor que 1 pelo lema 1. D

C
No desenho à esquerda, AB < 2 pelo
B
A lema 1. Logo, a área do hexágono é
menor ou igual que
1 1 3
( AB PC  PQ BD)  (2  1) 
P Q 2 2 2
D
Se a diagonal que os conecta é central, as outras diagonais principais que saem de
P e Q devem estar em lados distintos de PQ , pois caso contrário, se utilizarão já
três diagonais sem haver unido dois vértices consecutivos que ficam a um lado de
PQ e estes não podem ser conectados pela diagonal que falta. Então, só há uma
configuração possível:

EUREKA! N°9, 2000


14
Sociedade Brasileira de Matemática

P
Aqui a área é menor ou igual que
1 1
( PX  QY )  (2) 1, pelos lemas 2 Y
2 2
e 1 respectivamente.
X
Q

d) Os vértices P e Q não estão conectados por uma diagonal principal.


Nesse caso só há três configurações possíveis:

Pelo lema 2, neste caso a área é menor ou P


1
igual que (1 1  1 1) 1.
2

Neste caso a área é menor ou igual que P


X Q
1 1 3
( XY  1)  ( 2  1)  , onde o
2 2 2
último se da por desigualdade triangular.
Y

Pelo lema 1, algum dos segmentos PX


ou QY mede menos que 1, logo esta P
Q
configuração pode ser reduzida à
configuração de um vértice de grau três,
que já foi estudada (se bem não é a X
mesma, o raciocínio é idêntico).
Y
Esgotadas as possibilidades, se resolve
completamente o problema.

EUREKA! N°9, 2000


15
Sociedade Brasileira de Matemática

Você sabia…
Que existe um algoritmo que, dada uma soma envolvendo uma
função hipergeométrica (por exemplo: binomiais, exponenciais,
logarítmicas, trigonométricas), expressa-a como uma soma
telescópica. Com isso, podemos encontrar uma recursão
satisfeita por tal soma ou mesmo, quando possível, obter uma
fórmula fechada. O mais impressionante é que o algoritmo só
não obtém uma fórmula fechada se esta não existir (mais
precisamente, quando a soma não puder ser escrita como uma
função hipergeométrica).
Os autores deste algoritmo - Marko Petkovsek, Herbert Wilf e
Doron Zeilberger - publicaram um livro descrevendo sua
descoberta, com o inusitado título "A =B ".
Este livro encontra-se disponível na rede. Veja :

http://www.cis.upenn.edu/~wilf/AeqB.html

XLI OLIMPÍADA INTERNACIONAL DE MATEMÁTICA


Problemas e Soluções

PROBLEMA 1
Duas circunferências 1 e 2 intersectam-se em M e N.
Seja l a tangente comum a 1 e 2 que está mais próxima de M do que de N.
A reta l é tangente a 1 em A e a 2 em B. A reta paralela a l que passa por M
intersecta novamente a circunferência 1 em C e novamente a circunferência
2 em D.
As retas CA e DB intersectam-se em E; as retas AN e CD intersectam-se em P; as
retas BN e CD intersectam-se em Q.
Mostre que EP = EQ.

SOLUÇÃO DE ONOFRE CAMPOS DA SILVA FARIAS (FORTALEZA - CE) e


DANIEL MASSAKI YAMAMOTO (SÃO PAULO - SP)

EUREKA! N°9, 2000


16
Sociedade Brasileira de Matemática

E l
B
 D
 

A  M
Q

P
C

1 N
2

Inicialmente, vamos mostrar que AE = AM e que BE = BM. De fato, como AB ||


CD e AB é tangente a 1, segue-se que
EAB =ACM = BAM = 
e
EBA = BDM = ABM = 

Daí, é imediato que os triângulos EAB e MAB são congruentes (caso ALA), de
modo que
AE = AM e BE = BM.
Além disso, segue da congruência que EM  AB, e portanto, EM  CD. Agora,
basta mostrarmos que PM = MQ, porque dessa forma EM será mediatriz de PQ,
tal que EP = EQ. Para isto, note que o prolongamento de MN passa pelo ponto
médio de AB, pois MN é o eixo radical das duas circunferências e AB é tangente
comum. Assim, como PQ || AB, concluímos que MN também passa pelo ponto
médio de PQ. Logo, EM  PQ e EM passa pelo ponto médio de PQ, de modo que
EM é mediatriz de PQ e EP = EQ.

SOLUÇÃO DE DANIEL NOBUO UNO (SÃO PAULO - SP)

EUREKA! N°9, 2000


17
Sociedade Brasileira de Matemática

E
B Y
  D e


A +
X  M Q 2
+
 P
C

  
 
1 

[ XABY ]
Temos CA ˆ N CM ˆ N e NB ˆ D  NM ˆ D . Como CM ˆ N  NM ˆ D  
EAˆ N  EB ˆ N   CA ˆ N    NBˆ D  ; temos que AEBN é cíclico.
ˆ ˆ
Seja EAB   ECD   ANˆ M  ; XA ˆ C   CN ˆ A  .
Seja EBA  , analogamente, BDM BNM YBD DNˆ B 
ˆ ˆ ˆ ˆ
Como AEBN é cíclico, EA ˆ B ENˆ B  ; EB ˆ A EN ˆ A 
ˆ ˆ ˆ
ECQ ENQ  ECNQ é cíclico  EQC ENC    ˆ

ED ˆ P ENˆ P  EDNP é cíclico  EPˆ D ENˆ D    .


Logo, EPQ é isósceles.

PROBLEMA 2
Sejam a, b, c números reais positivos tais que abc = 1. Prove que

 1  1  1
 a  1   b  1   c  1   1.
 b  c  a

SOLUÇÃO
x y z
Fazendo a  y , b  z , c  x

EUREKA! N°9, 2000


18
Sociedade Brasileira de Matemática

1
(podemos tomar, por exemplo, y 1, x a, z  ac) , a desigualdade é
b
equivalente a ( x  z  y )( x  y  z )( y  z  x) xyz. Fazendo u x  z  y,
v x  y  z e w  y  z  x, temos u  v 2 x, u  w 2 z e v  w 2 y,
donde a desigualdade é equivalente a 8uvw (u  v)(u  w)(v  w). Como
u  v, u  w e v  w são todos positivos, no máximo um dentre os números u, v
e w não é positivo. Se houver um tal número, 8uvw será negativo ou nulo,
enquanto (u  v)(u  w)(v  w) é positivo, e a desigualdade acima é trivial. Se
u, v e w são positivos, a desigualdade acima é conseqüência direta da
desigualdade entre as médias aritmética e geométrica, pois
(u  v )(u  w)(v  w) u 2 v  uv 2  u 2 w   uw 2  v 2 w  vw 2  2uvw , donde
a desigualdade é equivalente a u 2 v  uv 2  u 2 w  uw 2  v 2 w  vw 2 6uvw, que
segue das desigualdades u 2 v  vw 2 2uvw, uv 2  uw 2 2uvw e
u 2 w  v 2 w 2uvw.

PROBLEMA 3
Seja n 2 um número inteiro positivo. No início existem n pulgas numa reta
horizontal, nem todas no mesmo ponto.
Para um número real positivo  , define-se um salto da seguinte maneira:
 Escolhem-se duas pulgas quaisquer nos pontos A e B com o ponto A à esquerda
do ponto B;
 A pulga que está em A salta até o ponto C da reta, à direita de B, tal que
BC
 .
AB
Determine todos os valores de  para os quais, para qualquer ponto M na reta e
quaisquer posições iniciais das n pulgas, existe uma sucessão finita de saltos que
levam todas as pulgas para pontos à direita de M.
SOLUÇÃO:
1
A resposta é para l 
( n  1)
Devemos demonstrar duas coisas:
1
a) que, para l  , existe uma seqüência infinita de movimentos que vai
( n  1)
levando as pulgas cada vez mais para a direita, ultrapassando qualquer ponto
prefixado M;

EUREKA! N°9, 2000


19
Sociedade Brasileira de Matemática

1
b) que, para l  e para qualquer posição inicial dada as pulgas, existe um
( n  1)
ponto M tal que as pulgas em um número finito de movimentos jamais alcançam
ou ultrapassam M.

Começaremos pelo item b). Sejam x1 , x 2 , ..., x n as posições iniciais das


pulgas, com x1 x 2 ... x n , de tal forma que x n é a posição da pulga mais à
direita.
 1 
Seja P =    x n  l x1  l x 2  ...  l x n  1 .
 (1  ( n  1) l ) 

O ponto P claramente está à direita de todas as pulgas.


Afirmamos que se após alguns movimentos as novas posições são x '1 ,..., x ' n e
definimos
 1 
P '    x ' n  l x'1  l x ' 2  ...  l x' n  1 .
 (1  ( n  1) l ) 

Se P ' P, isto conclui a demonstração.


Basta considerar o que ocorre após um movimento.
Se a pulga que estava em x i pula sobre a pulga que estava em x n então
x ' n  x n l ( x n  x i ) e x' n  l x n x n  l x i e P ' P.
Qualquer outro caso é ainda mais favorável. De fato, se a pulga que pulou
continua atrás de x n , temos x n '  x n e x1 '...  x n  1 '  x1  ...  x n  1 , donde
P '  P. Se ela passa de x n , teremos x n '  x j  ( x j  xi ) 
x n ' x n  x n ' x j 
x j  x i  x n  xi .

Item a) Se P x n  ( x1  x 2  ...  x n  1 ) se, em cada movimento, a pulga mais


à esquerda pula sobre a pulga mais à direita, temos x n ' x n  ( x n  x1 ) 
x n ' x n x n  x1 e P ' P , donde P é uma constante positiva (escolhendo a
origem, por exemplo, em x n ) . Temos então
1 n 1 1

n  1 j 1
( x n  x j ) x n 
n 1
( x1  ...  x n  1 ) x n  ( x1  ...  x n  1 ) P 

EUREKA! N°9, 2000


20
Sociedade Brasileira de Matemática

1 n 1 P
x n  x1  
n  1 j 1
( x n  x j ) P  x n ' x n ( x n  x1 ) 
n 1
, donde o ponto

P
mais à direita caminha pelo menos para a direita a cada passo, logo tende a
n 1
infinito. Como o ponto mais a direita, após n – 1 passos será o ponto mais à
esquerda, todos os pontos tendem a infinito (para a direita).

Nota: Na estratégia descrita na solução do item a), o ponto mais à esquerda se


torna sempre o mais à direita, donde podemos definir
x n 1  x n ' x n  ( x n  x1 ), e teriamos simplesmente x j '  x j 1 , j.
Reduzimos então a análise dessa estratégia ao estudo da recorrência linear
x n 1 (1  ) x n  x1 , cujo polinômio característico é
P ( x )  x n 1  (1  ) x n  ,
P( x)
do qual 1 é raiz, donde, como x n  ( x n  1  x n  2  ...  x  1), a
x 1
expressão
y m x m  ( x m  1  x n  2  ...  x m  n 2  x m  n 1 ) é um invariante da recorrência,
isto é, y m 1  y m m, donde y m é constante. Daí vem nossa fórmula para P.
Veja o artigo sobre equações de recorrência nesta Eureka.

PROBLEMA 4
Um mágico tem cem cartões numerados de 1 a 100. Coloca-os em três caixas,
uma vermelha, uma branca e uma azul, de modo que cada caixa contém pelo
menos um cartão.
Uma pessoa da platéia escolhe duas das três caixas, seleciona um cartão de cada
caixa e anuncia a soma dos números dos dois cartões que escolheu. Ao saber esta
soma, o mágico identifica a caixa da qual não se retirou nenhum cartão.
De quantas maneiras podem ser colocados todos os cartões nas caixas de modo de
que este truque sempre funcione? (Duas maneiras consideram-se diferentes se
pelo menos um cartão é colocado numa caixa diferente).

SOLUÇÃO DE FABRÍCIO SIQUEIRA BENEVIDES (FORTALEZA - CE)

Seja f (n) o número de maneiras de se colocarem os cartões de 1 a n , mas sem


contar a ordem das caixas. No final, basta multiplicar f (100) por 6 = 3!.
Achamos facilmente que f ( 4) 2, onde as duas únicas maneiras são:

EUREKA! N°9, 2000


21
Sociedade Brasileira de Matemática

1, 4 1
2 e 2, 3
3 4

Vamos provar por indução que f (n  1)  f (n) 2 para n 4, e que as únicas
maneiras são das formas:

A) 1, 4, 7,... B) 1
2, 5,... e 2, 3, ..., n – 1
3, 6,... n

Há duas possibilidades para o cartão n + 1:

1) Ele aparece junto com outros cartões. Nesse caso, os cartões de 1 a n


estarão na configuração A ou B.
Se for A): n  2, n  1 e n estão em caixas diferentes. n + 1 não pode estar na
mesma caixa de n ou n – 1, pois (n  2)  (n  1) (n  1)  (n)
Se colocarmos n + 1 na mesma caixa de n – 2, a mágica funcionará. Basta ver a
soma módulo 3.
No caso B): n  1 não pode entrar em nenhuma caixa, pois
1  (n  1) 2  (n) e ( n)  3 (n  1)  2

2) ( n  1) está em uma caixa isolada.


Como ( n  1)  (1) ( 2)  ( n), 2 e n devem estar na mesma caixa.
Temos a seguinte configuração:

n 1 Se k está junto com 2 e k + 1 está na outra caixa,


2, n, k (n  1)  k n  (k  1)  .
k 1 Então k e k + 1 estão na mesma caixa.

Fazendo isso para k = 2, 3,..., n – 2, temos que os cartões 2, 3,..., n estão na


mesma caixa . Como a outra caixa não pode ficar vazia, 1 está nela.
Para ver que esta configuração funciona, basta ver os intervalos das possíveis
somas para cada par de caixas.
o número total é 6 f (100) 12.

PROBLEMA 5

EUREKA! N°9, 2000


22
Sociedade Brasileira de Matemática

Verifique se existe um inteiro positivo n tal que n é divisível por exatamente 2000
números primos diferentes e 2 n  1 é divisível por n.

SOLUÇÃO DE HUMBERTO SILVA NAVES (SÃO PAULO - SP)

Vamos, primeiramente, provar o seguinte "super-lema":


n n
Lema: 3 n | 2 3  1 e 2 3  1 tem pelo menos "n" fatores primos distintos:
Esse lema será provado por indução:
Quando n = 1 ou n = 2, o lema é verdadeiro pois
2 3  1 9 3 2 e
2
2 3  1 513 33 19

Suponha que seja válido, também, para um certo "p" ( p 2), Vamos provar que
também é verdadeiro para " p  1":
p p p p
* 3 p | 2 3  1  k  Z | 2 3  1 k 3 p  2 3  1  k 3 p  (2 3 ) 3 
p 1 p 1
( 1  k 3 p ) 3  2 3  1  k 3 p 1  k 2 3 2 p 1  k 3 3 3 p  2 3 1
p 1
(k  k 2 3 p  k 3 3 2 p  1 ) 3 p 1  3 p 1 | 2 3  1.
3 p 1 3p 3 3 3p 3p p
* *2  1 ( 2 )  1 (2  1)[(2 )  ( 2 3 )  1], pois
2

a, b  R temos : a 3  b 3 ( a  b)(a 2  ab  b 2 )


p p p p p p p
Só que 2 23  2 3  1 ( 2 3  1) 2  3( 2 3  1)  3  mdc ( 2 3  1;2 23  2 3  1) 
p p p
mdc( 2 3  1;3) 3 pois 3 p | 2 3  1  3 | 2 3  1. Logo deve existir um fator primo
p p P
"j" tal que j | ( 2 23  2 3  1) e j | (2 3  1), logo
p 1 p p p
23  1 ( 2 3  1)(2 23  2 3  1)
p p 1
tem pelo menos 1 fator primo a mais que 2 3  1, logo 2 3  1 tem pelo menos
"p + 1" fatores primos em sua decomposição.
Logo o lema é válido para "p + 1", também.
Pelo princípio da indução finita, provamos que o lema é verdadeiro para todo
natural "n".
Pronto, o "super-lema" veio te salvar nessa hora de sufoco!
2000
Sabemos que ímpar 2 3  1 tem pelo menos 2000 fatores primos e pegamos
1999 fatores primos que são diferentes de 3 e os chamamos de
p1 ; p 2 ; p 3 ;...; p1999 . Temos que
2000
3 2000 | 2 3 1 e

EUREKA! N°9, 2000


23
Sociedade Brasileira de Matemática

2000
p1 p 2 p 3 ... p1999 | 2 3 1
2000
Basta escolhermos o inteiro ímpar n 3 p1 ... p1999 , pois:
2000
3 2000 | n  2 3  1 | 2 n  1 e como
2000
n | 23 1  n | 2n 1
Logo existe um "n" de 2000 fatores primos, tal que n | 2 n  1 .

PROBLEMA 6
Sejam AH 1 , BH 2 , CH 3 as alturas de um triângulo acutângulo ABC. A
circunferência inscrita no triângulo ABC é tangente aos lados BC, CA, AB em
T1 , T2 , T3, respectivamente. Seja l1 a reta simétrica da reta H 2 H 3
relativamente à reta T2 T3 , l 2 a reta simétrica da reta H 3 H 1 relativamente à
reta T3T1 e l 3 a reta simétrica da reta H 1 H 2 relativamente à reta T1T2 .
Prove que l1 , l 2 , l 3 determinam um triângulo cujos vértices pertencem à
circunferência inscrita no triângulo ABC.

SOLUÇÃO DE ONOFRE CAMPOS DA SILVA FARIAS (FORTALEZA - CE)


A
H2

T2
H3

T3
N
P L M l1

B T1 C

fig. 01
Inicialmente, vamos mostrar que as retas l1, l2 e l3 são paralelas aos lados
BC, CA e AB, respectivamente.

Sejam P H 2 H 3  T2 T3 , M e N os pontos de interseção da reta l1 com


a circunferência inscrita. Supondo AB < AC, temos a fig. 01 acima. Veja que
PH3T3 = AH3H2 = C ,

EUREKA! N°9, 2000


24
Sociedade Brasileira de Matemática

B C
 AT 3T2 AT 2T3  ,
2
de modo que
B C
H 3 PT3  e H3PM = 2.H3PT3 = B – C,
2
e, portanto,
H3LM = H3PM + LH3P = (B – C) + C = B.

Logo, l1 || BC e, analogamente, concluímos que l2 || AC e l3 || BA.


Se AB = AC, então, l1 || BC por simetria. Neste caso, teremos H2H3 || T2T3
|| l1 || BC.
Agora, vamos calcular a distância entre as retas l1 e BC. Como PT2 é a
bissetriz do ângulo H2PN, então

d(T2, PH2) = d(T2, l1) e d(l1, BC) = d(T2, BC) – d(T2, PH2) (1)
Mas,
d(T2, BC) = (p – c).sen C

e como AH2H3 = B, segue que d(T2, PH2) = H2T2.sen B. (2)

Também,
H2T2 = CH2 – CT2 = a.cos C – (p – c).
Em (2), obtemos:
d(T2, PH2) = (a.cos C – (p – c))sen B. (3)

a2  b2  c2 b c
Usando que a. cos C  , sen B  , sen C  , obtemos:
2b 2R 2R

EUREKA! N°9, 2000


25
Sociedade Brasileira de Matemática

d (l1 , BC ) ( p  c ).senC  ( p  c ).senB  senB.(a. cos C )


c b b a2  b2  c2
( p  c )(
 ) .
2R 2R 2R 2b
( a  b  c ).(b  c ) a  b  c 2
2 2
 
4R 4R
2
ab  ac  a

4R
a bc a

2R 2
( p  a ).senA.
Dessa forma, acabamos de mostrar que

d(l1, BC) = d(T2, AB) = d(T3, AC) = (p – a).sen A. (*)

Vamos mostra agora que BT2 = BN e, analogamente, mostraremos que


CT3 = CM.
Sejam U e V as projeções ortogonais de T2 e N sobre os lados AB e BC
respectivamente. Então,
NV = T2U = (p – a).sen A.

Sejam I o incentro e E a projeção ortogonal de I sobre l1 (fig. 02). Veja


que
EN = T1V e EI = |(p – a).sen A – r|.
A
U
T2

T3
M E N
P r l1
I
r

B T1 V C
fig. 02

EUREKA! N°9, 2000


26
Sociedade Brasileira de Matemática

i ) T1V 2 EN 2 IN 2  EI 2 r 2  (( p  a ). sen A  r ) 2


2r.( p  a ) sen A  ( p  a ) 2 . sen 2 A

ii ) T3U 2 T2T3 2  T2U 2


T2T3 2  ( p  a) 2 . sen 2 A

Agora, o quadrilátero AT3IT2 é inscritível na circunferência de diâmetro AI.


Logo, temos duas relações:

I) T2T3 = AI.sen A;
II) T2T3.AI = 2r.AT2 = 2r.(p – a) (pelo teorema de Ptolomeu).

De (I) e (II), obtemos T2T32 = 2r.(p – a).sen A. Portanto, em (i) e (ii) ficamos
com T1V 2 T3U 2  T1V T3U , e como BT1 = BT3, por último segue que

BVN  BUT2 (**)

de modo que BT2 = BN, como queríamos demonstrar.

T2

T3
r
r
I

B C
fig. 03

De (**), ainda podemos concluir que ABT2 = CBN, e como ABI = CBI,
temos T2BI = NBI.

EUREKA! N°9, 2000


27
Sociedade Brasileira de Matemática

Finalmente, como T2B = NB, podemos concluir que N é o simétrico de


T2 em relação à bissetriz do ângulo B. Analogamente, M é o simétrico de T3 em
relação à bissetriz interna do ângulo C.

Dessa forma, podemos redefinir as retas l1, l2 e l3 da seguinte forma:

Sejam S1, S2 e S3, respectivamente, os simétricos de T1, T2 e T3 em relação


às bissetrizes dos ângulos A, B e C, respectivamente. L1 é a reta que passa
por S2 e S3, l2 é a reta que passa por S1 e S3 e l3 é a reta que passa por S1 e S2.

Claramente, l1, l2 e l3 determinam um triângulo inscrito na circunferência


inscrita no triângulo ABC.

EUREKA! N°9, 2000


28
Sociedade Brasileira de Matemática

BRAHMAGUPTA PARA TODOS


José Cloves Verde Saraiva - UEMA

 Nível Intermediário

Nestas notas apresentaremos as fórmulas da geometria plana para o


cálculo da área de um triângulo e de um quadrilátero cíclico (inscrito numa
circunferência) em função do comprimento de seus lados e de seu semi-
perímetro. Para o triângulo esta é conhecida como a fórmula de Heron de
Alexandria, embora escritores árabes afirmem que esta foi descoberta por
Arquimedes. Para os quadriláteros cíclicos há uma generalização natural da
fórmula de Heron, tão importante, que é considerada como a mais notável
descoberta da geometria hindu, feita por BRAHMAGUPTA. Originalmente a
prova de Brahmagupta faz uso do conhecido Teorema de Ptolomeu para
quadriláteros cíclicos. Aqui, estas duas fórmulas são deduzidas, elementarmente,
da Lei dos co-senos para um triângulo.

1. A FÓRMULA DE HERON

Seja ABC um triângulo cujos lados medem a, b e c indicados na figura


abaixo, então a medida da área deste triângulo é dada pela fórmula:

S  p ( p  a )( p  b)( p  c)
1
onde p  ( a  b  c) é o semi-perimetro do ABC .
2

a c
h

C b A

________________________________________________________________________________
O autor agradece o apoio à pesquisa da Universidade Estadual de Maranhão (UEMA) e o saudável
ambiente de trabalho do DEMATI-CECEN

EUREKA! N°9, 2000


29
Sociedade Brasileira de Matemática

Prova: Todos sabem que a área de um triângulo é calculada pela fórmula


1
S  bh. Nas condições aqui tratadas sabemos que a altura relativa ao lado
2
CA é dada por h a senĈ , donde podemos escrever para a área:

1 ˆ (*)
S  ab senC
2

É também muito conhecida a Lei dos co-senos para um triângulo. E, pela figura
acima, esta pode ser escrita como:

c 2 a 2  b 2  2ab cos Cˆ (**)

Com efeito, elevando ao quadrado S na expressão (*) temos que:


4S 2
a 2
b 2
sen2 ˆ a 2 b 2 (1 
C cos 2 Cˆ ) ab(1  ˆ ) 
cos C ab(
( ab  ˆ )( ab  ab cos C
ab cos C ˆ )

Multiplicando este último resultado por 4, obtemos:


ˆ )(2ab  2ab cos C
16 S 2 ( 2ab  2ab cos C ˆ)

no qual podemos completar quadrados e adequar os fatores para o uso da Lei dos
co-senos (**) temos:
16S 2
[  a 2
 b 2
 2ab  a 2
 b 2
 2ab  ˆ ][a
cos C 2
 b 2

2 2 2 2 2 2
[  ( a  b)  c ][(a  b)  c ] [c  (a  b) ][(a

fatorando as diferenças de quadrados, podemos escrever:


16 S 2 (c  a  b)(c  a  b)( a  b  c)(a  b  c)
(a  b  c)( a  b  c )( a  b  c )(a  b  c)

Aonde, dividindo por 16:


1 1 1 1
S 2  (a  b  c) (a  b  c  2a ) (a  b  c  2b) (a  b  c  2c)
2 2 2 2

Portanto:
1 1  1  1 
S 2  ( a  b  c)  ( a  b  c ) 
a   (a  b  c )  b   ( a  b  c)  c
2 2  2  2 
1
substituindo o semi-perímetro p  ( a  b  c), concluímos que:
2

EUREKA! N°9, 2000


30
Sociedade Brasileira de Matemática

S  , a conhecida fórmula de Heron.


p ( p  a )( p  b)( p  c)
De uma certa analogia da fórmula acima demonstrada deduzimos, a seguir, o
resultado principal destas notas.

2. A FÓRMULA DE BRAHMAGUPTA

A medida da área de um quadrilátero cíclico de lados a, b, c, d cujo semi-


perímetro denotado por p é a seguinte:
K  ( p  a )( p  b)( p  c)( p  d )

1
D onde p  ( a  b  c  d )
2
1
d ( p  a)  ( a  b  c  d )
c 2
1
A x ( p  b)  ( a  b  c  d )
2
a C 1
( p  c)  (a  b  c  d )
b 2
1
B ( p  d )  (a  b  c  d )
2

Os seguintes fatos elementares são considerados na prova:

I. Seja um quadrilátero tal que seus ângulos opostos internos sejam


suplementares: na figura temos Bˆ  D
ˆ 180 (esta relação vale para os
quadriláteros cíclicos).

II. As áreas dos triângulos S1 e S 2 são dadas pelas relações:


1 ˆ e S  1 cd senD ˆ
S1  ab senB 2
2 2
III. A diagonal x, indicada na figura anterior, pela Lei dos co-senos aplicada
aos triângulos S1 e S 2 verifica a relação:
a 2  b 2  2ab cos Bˆ x 2 c 2  d 2  2cd cos D
ˆ

Prova: De I. obtemos que cos( Bˆ  D


ˆ )  1, que implica a igualdade
senB ˆ senDˆ 1  cos B ˆ cos D ˆ (*). Com efeito, seja K a medida

EUREKA! N°9, 2000


31
Sociedade Brasileira de Matemática

da área do quadrilátero cíclico dada por: K S1  S 2 , Substituindo II. Obtemos a


igualdade 2 K ab senB ˆ  cd senD ˆ . Elevando ao quadrado obtemos:
4 K 2 a 2 b 2 sen 2 Bˆ  2 abcd senBˆ senD
ˆ  c 2 d 2 sen 2 D
ˆ

substituindo (*) no produto de senos temos:

ˆ )  2abcd (1  cos B
4 K 2 a 2 b 2 (1  cos 2 B ˆ cos D
ˆ )  c 2 d 2 (1  cos 2 D
ˆ)

ou melhor,

2 2 2 2 ˆ ˆ cos D
ˆ )  abcd (1  cos B ˆ )  c 2 d 2 (1  cos 2 D
ˆ cos D ˆ)
4K  a b (1  cos B )  abcd (1  cos B

4 K 2  a 2 b 2 (1  cos B
ˆ )(1  cos B
ˆ )  ab(1  cos D ˆ )  c 2 d 2 (1  cos D
ˆ )cd (1  cos B ˆ )(1  c

Multiplicando, adequadamente, por 4 os dois membros,

16 K 2 2[ ab (1  cos Bˆ )  cd (1  cos D


ˆ )] 2 [ ab (1  cos B
ˆ )  cd (1  cos D
ˆ )]

16 K 2  [ 2ab cos Bˆ  2ab  2cd cos D ˆ  2cd ][ 2ab cos B ˆ  2ab  2cd cos D ˆ  2cd co
Substituindo a relação III. obtemos:

16K 2  [a 2  b 2  2ab  (c 2  d 2 )  2cd ][a 2  b 2  2ab  (c 2  d 2 )  2cd ]


16 K 2  [(a  b) 2  (c  d ) 2 ][(a  b) 2  (c  d ) 2 ]
16 K 2  [(a  b)  (c  d )][(a  b)  (c  d )][(a  b)  (c  d )][(a  b)  (c  d )]

por tanto temos que:


1 1 1 1
K 2  ( a  b  c  d ) ( a  b  c  d ) ( a  b  c  d ) (a  b  c  d )
2 2 2 2
daí, é imediato ver que:
K 2 ( p  a )( p  b)( p  c )( p  d )
permitindo concluir que,
K  ( p  a )( p  b )( p  c )( p  d )
concluindo a prova.

Como observação final, a fórmula acima demonstrada não pode ser mais geral do
que foi provado por BRAHMAGUPTA. De fato, a fórmula vale exatamente para
os quadriláteros cíclicos. Em geral, se o quadrilátero não for cíclico, sua área é
estritamente menor que ( p  a )( p  b)( p  c )( p  d ) , pois nesse caso

EUREKA! N°9, 2000


32
Sociedade Brasileira de Matemática

ˆ senD
senB ˆ  1  cos B
ˆ cosD
ˆ , o que pode ser usado como na
prova acima para provar nossa afirmação.

REFERÊNCIA
E.W. Hobson, A Treatise on Plane Trigonometry (NY); Macmillan Company, 4a ed.,
(1902), pg. 204.
EQUAÇÕES DE RECORRÊNCIA
Héctor Soza Pollman - Universidade Católica do Norte - Antofagasta, Chile

 Nível Avançado

Frenqüentemente em teoria da Computação (ver exemplo [2]), ao analisar


o tempo de execução de um algoritmo (ou o espaço ocupado na memória pelos
dados), obtemos uma (ou mais) equações discretas, chamadas de Equações de
Recorrência, cuja incógnita é uma função inteira f(n), que geralmente é uma
função do tamanho n do problema (por exemplo: a quantidade de dados a ordenar

EUREKA! N°9, 2000


33
Sociedade Brasileira de Matemática

se é um algoritmo de ordenamento). Esta equação resulta ser uma relação entre


f(n) e seus valores prévios, como são f(n – 1), f(n / 2), ou outro. Além disso se
conhecemos o algoritmo analisado com detalhe, podemos estabelecer um valor de
bordo num ponto dado (como f(0) por exemplo). Neste artigo são apresentados
alguns dos métodos desenvolvidos para resolver este tipo de equações, as quais
aparecem em ordem de dificuldade.

Os tipos de equações de recorrência a serem consideradas são as


seguintes, em que a incógnita é a sucessão x n com n 0 :

1. EQUAÇÃO LINEAR DE PRIMEIRA ORDEM COM COEFICIENTES DE VALOR INTEIRO 1.

O tipo mais simples de equação de recorrência de primeira ordem é:


x n 1 x n  bn , n 0
x b
em que 0 e a sucessão n são dados do problema. Sua resolução faz uso da
propriedade telescópica da soma obtendo:
n 1
x n x 0   bi , n 1
i 0

Exemplo: Para a equação: x n 1 x n  2 n , n 0, com x 0 1, obtemos


n
x n 2 , n 0.

2. EQUAÇÃO LINEAR DE PRIMEIRA ORDEM COM COEFICIENTES CONSTANTES.

A equação linear de primeira ordem com coeficientes constantes é:


a n x n bn x n  1  c n , n 0
em que x 0 e as sucessões numéricas a n , bn e c n são dados do problema (as
sucessões a n e bn não devem ser nulas). Para resolver esta equação ela deve ser
multiplicada pelo fator S n (chamado fator somante):
S n a n x n S n bn x n  1  S n c n
Impõe-se a condição:
S n bn S n  1 a n  1 (1)
com o qual obtemos:
S n a n x n  S n 1a n 1 x n 1  S n cn

Observa-se que a equação anterior se reduz a uma de primeiro tipo, e que sua
solução é:

EUREKA! N°9, 2000


34
Sociedade Brasileira de Matemática

n
1
xn  ( S 0 a 0 x 0   S i ci ), n 0
S n an i 1

O fator somante é obtido a partir da condição (1) e considerando que S 0 1 :


a a ...a a
S n  n  1 n  2 1 0 , n 1
bn bn  1 ...b2 b1

EXEMPLO: AS TORRES DE HANOI.


Dadas três varetas e n discos de distintos tamanhos colocados na primeira vareta
em ordem de tamanho (do menor ao maior), mover estes n discos desde a vareta
inicial até a terceira usando a segunda como auxiliar, sem colocar um disco de
tamanho maior sobre um de tamanho menor (para maiores explicações ver [4]).
Se x n é a quantidade de movimentos para levar os n discos da primeira a terceira
vareta, podemos provar, ao analisar como são distribuídos os movimentos, que, se
x n é a quantidade de movimentos para mover os n discos desde a primeira à
terceira vareta (com n 0), então:
x n 2 x n  1  1, n 1
com x 0 0. De fato, dada uma solução do problema de Hanoi com n – 1 discos
em x n  1 movimentos, podemos mover os n – 1 primeiros discos para a segunda
vareta, depois mover o último disco para a terceira vareta e por fim mover os
n 1 primeiros discos para a terceira vareta, gastando
x n  1  1  x n  1 2 x n  1  1 movimentos. Neste caso temos que o fator somante
1
resulta ser: S n  n . Logo, a solução da equação das torres de Hanoi é:
2
x n 2 n  1, n 0
Observamos, por exemplo, que para n = 3 devem ser realizados 7 movimentos.
Deixamos como exercício para o leitor provar que é impossível resolver este
problema usando uma quantidade menor de movimentos.

3. EQUAÇÕES HOMOGÊNEAS DE PRIMEIRA ORDEM COM COEFICIENTES CONSTANTES.

Considere a equação:
a k x n k  a k  1 x n k  1  ...  a 0 x n 0, n 0 (2)
em que a 0 ,..., a k são sucessões independentes de n, e os valores de x i são
conhecidos para i = 0, ..., k – 1 (correspondem aos valores de bordo). Supondo

EUREKA! N°9, 2000


35
Sociedade Brasileira de Matemática

que a equação (2) admite uma solução do tipo: x n  n , em que  é um


parâmetro inteiro, e substituindo em (2) temos:
a k n  k  a k  1 n k  1  ...  a 0 n 0
Se   0 então obtemos a equação característica associada a equação (2):
a k k  a k  1k  1  ...  a 0 0 0
Vamos mostrar que se esta equação tem as raízes complexas 1 ,...,  r com
multiplicidades 1 , 2 ,..., r  N , respectivamente, então as soluções de (2)
são exatamente as seqüências (x n ) da forma
xn Q1 (n)1n  Q2 (n)n2  ...  Qr ( n)nr ,
onde Q1 ,..., Qr são polinômios com grau (Qi )   i , 1 i r (em particular,
se  i é uma raíz simples então Qi é constante).
Seja P ( x) a k x k  a k  1 x k  1  ...  a 0 um polinômio.
Dizemos que uma seqüência ( x n ) nN satisfaz a propriedade Rec ( P ( x )) se
a k x n  k  a k  1 x n  k  1  ...  a 0 x n 0, n  N . Não é difícil verificar os
seguintes fatos:

i) Se ( X n ) e (Yn ) satisfazem Rec ( P ( x )) e c  C então ( Z n )  X n  cYn


satisfaz Rec ( P ( x )) .

ii) Se Q( x ) br X r  br  1 X r  1  ...  b0 e ( X n ) satisfaz Rec ( P ( x )) então


( X n ) satisfaz Rec ( P ( x )Q ( x ))
r
(isso segue de  b j (a k X n  j k  a k  1 X n  j k  1  ...  a 0 X n  j ) 0, n  N )
j 0

iii) ( X n ) satisfaz Rec ( P ( x )) se e só se (Yn )  X n  n   satisfaz Rec


k
( P (X )) (substitua X n  j  n  j Yn  j em  a j X n j 0).
j 0
n
iv) Se S n  x k então ( x n ) satisfaz Rec ( P ( x )) se e só se ( S n ) satisfaz
k 0
Rec (( x  1) P ( x )) (escreva x n  j 1 S n  j 1  S n  j e substitua em
n
 a j x n  j 1 0).
j 0

EUREKA! N°9, 2000


36
Sociedade Brasileira de Matemática

Por iii), para ver que, para todo polinômio Q( x) de grau menor que m,
X n Q (n) n satisfaz Rec (( x   ) m ), basta ver que (Yn )  Q ( n)  satisfaz
Rec (( x  1) m ), o que faremos por indução. Isso é claro se m = 1, e em geral, se
~
Z n Yn 1  Yn Q ( n  1)  Q( n), como Q ( x ) Q ( x  1)  Q ( x ) tem grau
menor que m – 1, ( Z n ) satisfaz Rec (( x  1) m  1 ) (por hipótese de indução), e
logo, por (iv), (Yn ) satisfaz Rec (( x  1) m ). Essa observação, combinada com
ii), mostra que se ( P ( x) ( x  1 ) 1 ( x   2 )  2 ...( x   r )  r , e grau (Qi )   i
r
n
para 1 i r então x n  Qi (n) i satisfaz Rec ( P ( x )) .
i 1
Para ver que se ( x n ) satisfaz Rec ( P ( x )) então x n é da forma acima,
usaremos indução novamente.

Supomos 1 0 e tomamos Yn  X n 1n , Z n Yn 1  Yn (com Z 0 Y0 ).


Por iii) e iv), Z n satisfaz Rec ( P (1 x ) ( x  1)) e, portanto por hipótese de
~ ~ ~
indução, Z n Q1 ( x )  Q2 ( x)( 2 1 ) n  ...  Qr ( x )( r 1 ) n , onde grau
~ ~
Qi   i para 2 i r e grau Q1   1  1.
Para terminar a prova, vamos mostrar que se existem polinômios P1 , P2 ,..., Pk
tais que Yn 1  Yn P1 (n)  P2 ( n)  2n  ...  Pk (n)  kn (onde 1,  2 ,...,  k são
complexos distintos e Pi 0, i 2 ) então
~ ~ n ~ n ~ ~
Yn P1 ( n)  P2 ( n)  2  ...  Pk ( n)  k , onde P1 ,..., Pk são polinômios com
~ ~
grau Pi grau Pi para i 2 e grau P1 grau P1  1, por indução na
soma dos graus dos polinômios Pi , onde convencionamos que o grau do
polinômio nulo é –1.
(no nosso caso temos  i  i 1 , e como X n 1n Yn o resultado segue
imediatamente).

Para provar essa afirmação observamos inicialmente que, se a soma dos graus de
Pi é –1, então Yn 1  Yn 0, n, e logo, Yn é constante e, em geral,
consideramos 2 casos:

EUREKA! N°9, 2000


37
Sociedade Brasileira de Matemática

a) P1 ( x) c m x m  c m  1 x m  1  ...  c 0 , c m 0. Nesse caso definimos


~ c n m 1 ~ ~
Yn Yn  m , e temos Yn 1  Yn Q1 (n)  P2 (n)  1n  ...  Pk ( n)  kn ,
m 1
~
com grau Q  m. Por hipótese de indução, Yn (e logo Yn ) é da forma
desejada.

b) P2 ( x) d s x s  d s  1 x s  1  ...  d 0 , d s 0. Nesse caso, definimos


~ d n s n2
Yn Yn  s , e temos
2  1
~ ~
Yn 1  Yn P1 (n)  Q(n)  2n  P3 (n)  3n  ...  Pk (n)  kn ,
~
com grau Q  s. Por hipótese de indução, Yn (e logo Yn ) é da forma
desejada.

Exemplo: x n sen( n ) satisfaz uma recorrência linear. De fato,


x n 1 sen(n   ) sen(n ) cos   cos(n ) sen 
x n 2 sen( n  2 ) sen( n ) cos 2  cos(n ) sen2 
sen2 sen2
 x n 2  x n 1 (cos 2  cos  ) x n , ou seja,
sen sen
x n 2 2 cos X n 1  X n . Note que x n não parece ser da forma geral descrita
nesta seção, mas de fato,

e in  e  in 1 1  i n 1 1
xn   (e i ) n  (e )  (cos   isen ) n  (cos   isen ) n
2i 2i 2i 2i 2i
Obs. Se ( x n ) satisfaz Rec (( x  1) P ( x )) , onde
P ( x) a n x k  a k  1 x k  1  ...  a 0 , então, se definirmos
Yn a k x n k  a k  1 x n k  1  ...  a 0 x n , teremos Yn 1 Yn , n  N , ou seja,
Yn é constante. Assim, a k x n k  ...  a 0 x n é um invariante da seqüência x n , o
que é uma observação útil para muitos problemas olímpicos. Veja o problema 3
da IMO.

EXEMPLO: OS NÚMEROS DE FIBONACCI.

A sucessão que lhes dá origem é: f n 2  f n 1  f n , n 0

EUREKA! N°9, 2000


38
Sociedade Brasileira de Matemática

em que f 0 e f 1 são dados. Ao aplicar o método analisado, considerando f 0 =


0 e f 1 1, obtemos o polinômio característico  2    1 0, cujas soluções
são:
1 5 1 5
1  , 2 
2 2
Considerando as condições do bordo a solução geral da equação de Fibonacci é
(ver [3]):
1
fn  (1n   n2 ), n 0
5

Observa-se que os valores associados a esta sucessão são todos inteiros. Por
exemplo: f 3 2, f 4 3, etc. Podemos comprovar que, se n converge a infinito
então  2 converge a zero, portanto, f n é da ordem de 1n , e a fração f n 1 f n
converge a 1 .

4. EQUAÇÕES NÃO HOMOGÊNEAS DE PRIMEIRA ORDEM COM COEFICIENTES


CONSTANTES.

A equação mencionada é do tipo:


a k x n k  a k  1  ...  a 0 x n  y n ,
onde a 0 , a1 ,..., a k são constantes e y n satisfaz uma equação homogênea de
primeira ordem com coeficientes constantes.
Supondo que y n satisfaça
b y n   b 1 y n  1  ...  b0 y n 0,
onde b0 , b1 ,..., b são constantes, observamos que
b (a k x m k  ...  a 0 x m  ) 
b 1 (a k x m  1k  ...  a 0 x m  1 )  ...
b0 (a k x m  k  ...  a 0 x m ) 0,
ou seja, temos uma equação homogênea de primeira ordem com coeficientes
constantes.
Pode-se demonstrar que a equação característica da recorrência é:
(a k x k  a k  1 x k  1  ...  a 0 ) (b x   b 1 x  1  ...  b0 ) 0.

Exemplo 1: Considere a seguinte equação de recorrência

EUREKA! N°9, 2000


39
Sociedade Brasileira de Matemática

1, se n é multiplo de 4, n 1;


xn  xn 1 
 0, caso contrário
x 0 0.

1, se n é multiplo de 4, satisfaz a recorrência.


y n 
 0, caso contrário
y n 4  y n  y n 4  y n 0, n 0. Assim,
( x n  4  x n 3 )  ( x n  x n  1 ) 0 
x n 4  x n 3  x n  x n  1 0 e
x 0 0; x1  x 0 0  x1 0;
x 2  x1 0  x 2 0;
x 3  x 2 0  x 3 0;
x 4  x 3 1  x 4 1.
A equação característica é x 5  x 4  x  1 0  ( x 4  1) ( x  1) 0 a qual
possui as raízes 1 (raiz dupla); –1; i ; – i. Ou seja,
x n ( An  B) 1n  C ( 1) n  D i n  E ( 1) n ,
A, B, C, D, E constantes.
De fato, considerando as condições de bordo,
n 3 ( 1) n (1  i ) n (1  i )
xn     i  ( i ) n 
4 8 8 8 8
n i n ((1  i )  (  1) n (1  i ))  (  1) n  3
xn  
4 8
n
(É interessante notar que, na verdade, x n  .)
 4

Exemplo 2: Seja a seguinte equação de recorrência, que considera logaritmos em


base 2:

f ( n) 2 f ( n )  log 2 log 2 n, n 3
f (2) 1

EUREKA! N°9, 2000


40
Sociedade Brasileira de Matemática

Neste caso aplicamos uma troca de variável para ir desta equação a uma equação
linear, e poder resolvê-la, o qual significa que haverá solução só para os valores
de n que tome com este cambio. Este é:
k k1
n 2 2 , n 2 2 , log 2 log 2 n k

Ao efetuar essas substituições na equação obtemos:


x k  2 x k  1 k (3)
onde:
k
x k  f ( n)  f ( 2 2 )
0
x 0  f ( 2 2 ) 1

A equação (3) é uma equação não homogênea. Procedendo como acima 


obtemos:
x k 2  4 x k 1  5 x k  2 x k  1 0

cuja solução considerando a condição do bordo é:


x k  2  k  3 2 k , k 0

Logo, voltando a variável n original, a solução final é:


k
f (n) 3 log 2 n  log 2 log 2 n  2, n 2 2 , k 0

A solução só tem resultados inteiros para os valores de n mencionados. Por


exemplo: f (4) 3, f (16) 8, etc. Deixamos a prova deste fato como exercício
para o leitor.

EUREKA! N°9, 2000


41
Sociedade Brasileira de Matemática

EQUAÇÕES FUNCIONAIS
Eduardo Tengan - Colégio Etapa

 Nível Avançado

Uma das técnicas básicas para a resolução de equações com funções é


perceber quando ela é injetora, isto é, quando f (a )  f (b)  a b. Isto é
particularmente freqüente em problemas em que temos equações do tipo
f ( f ( x)) kx, k 0. De fato,
f ( a)  f (b)  f ( f ( a))  f ( f (b))  ka kb  a b.
"Sabendo que f é injetora, podemos provar novas relações aplicando f dos dois
lados da equação". Por exemplo, considere o seguinte problema:

(IMO) Seja Q  o conjunto dos racionais positivos. Construa uma função


f : Q   Q  tal que f ( xf ( y ))  f ( x) y para todo x, y  Q  .
Para x = 1, temos f ( f ( y ))  f (1) y e daí temos que f é injetora:
f (a )  f (b)  f ( f (a ))  f ( f (b))  f (1) a  f (1) b  a b. ( f (1)  Q  ,
logo f (1) 0).
Agora, vamos provar que a função é multiplicativa, isto é, que
f ( ab)  f ( a ) f (b). Aplicamos f a cada membro da equação,
f (1)
f ( f (ab)) 
ab
f ( f (a )) f (1)
f ( f (a ) f (b))  
b ab
Como os resultados são iguais e f é injetora, concluímos que
f ( ab)  f ( a ) f (b).
Daí temos:
f (1 1)  f (1) f (1)  f (1) 1
 1 1 1 1 1
f  a    f (a ) f    1  f ( a ) f    f   
 a a a  a  f (a)
a  1  f (a )
f    f a   
b  b  f (b)

EUREKA! N°9, 2000


42
Sociedade Brasileira de Matemática

Assim, basta construir a função para os inteiros positivos. Mais ainda, basta
defini-la para os primos. Devemos ter f ( f ( p))  f (1) p 1 p . Pensando um
pouco, sendo p1 , p 2 , p 3 ,... todos os primos, podemos tomar

 pn 1 se n é par
f ( pn ) 
1 pn1 se n é ímpar
e verificar que a condição inicial é satisfeita.

EXERCÍCIO 1
(IMO) Determine o menor valor possível de f(1998), onde f é uma função do
conjunto N dos inteiros positivos nele mesmo, tal que, para todo m, n  N :
f ( n 2 f ( m)) m( f ( n)) 2 .

Para funções de domínio real, podemos utilizar desigualdades para obter


igualdades. Por exemplo, considere o seguinte problema.

Determine todas as funções f : R 2  R tais que f ( a; a ) a para todo a  R e


a  b  c  d  f (a, b)  f (c, d ) para quaisquer a, b, c, d  R .
Observe, em primeiro lugar, que, para   0,
 a b a b  a b a b 
f  ,     f ( a , b)  f   , 
 2 2   2 2 
a b a b
    f ( a , b)    (*)
2 2
Logo é razoável que f ( a, b) ( a  b) 2 . Suponha que existam a 0 e b0 tais
que f (a 0 , b0 ) (a 0  b0 ) 2 . Se f (a0 , b0 )  ( a0  b0 ) 2 , então
f (a0 , b0 ) (a0  b0 ) 2  p, p  0 . Mas f ( a 0 , b0 ) (a 0  b0 ) 2  p 2 por (*),
ou seja,
(a 0  b0 ) 2  p (a 0  b0 ) 2  p 2  p 0, absurdo.
Analogamente f ( a 0 , b0 )  ( a 0  b0 ) 2 é impossível. Logo f (a, b) (a  b) 2
para todo a, b  R.
Podemos utilizar um raciocínio semelhante em diversos problemas que envolvem
funções crescentes. Às vezes, é necessário obter a desigualdade a partir das
 
condições do problema, muitas vezes, utilizamos relações como f x 2  f  x   2

EUREKA! N°9, 2000


43
Sociedade Brasileira de Matemática

para concluir que x 0  f ( x) 0 (basta substituir x no lugar de x na relação


anterior).
Observe o exercício a seguir.
Seja f uma função de R em R tal que f(1) = 1, f (a + b) = f(a) +f(b) para todo a,
b e f ( x ) f (1 x ) 1 para todo x 0. Prove que f ( x ) x para todo número
real.
É fácil ver que f ( n) n para todo n inteiro positivo e de
f (0)  f (0)  f (0)  f (0) 0 e f (0)  f (1)  f (  1)  f ( 1)  1, que
f ( x )  x para todo x  Z . Para verificar este resultado para x  Q,
basta
utilizar f (1 x) 1 f ( x). Observamos ainda que f é injetora: temos
f ( y )  f ( x  y )  f ( x)  f ( x  y )  f ( x)  f ( y ).
Como f ( x)  f (1 x) 1  f ( x) 0, então f ( x) 0  x 0 e
f ( x)  f ( y )  f ( x)  f ( y ) 0  f ( x  y ) 0  x  y 0  x  y.
Para estender o resultado para R, precisamos obter uma desigualdade (na
verdade, é só desta forma que poderemos distinguir o conjunto dos racionais do
conjunto dos reais. No jargão matemático, dizemos que R é um corpo ordenado
completo). Utilizando a observação que precede o exercício, vamos tentar
calcular f ( x 2 ).
Se a a 2 , f (a  a 2 ) 0 (pois f é injetora), logo
1 1  1  1 1 
2
 2
f  2
 f   
f (a )  f ( a ) f ( a  a ) a a   a 1 a 
1  1  1 1
f    f     f (a 2 ) ( f (a )) 2 ,
a  1  a  f (a ) f (1  a )
que vale também quando a a 2  a 0 ou a 1.
Agora, observando que a  b  a  b  0  f ( a  b)  0  f ( a )  f (b),
concluímos verificando que, por exemplo, se f ( x 0 )  x 0 para algum x 0  R ,
que se f ( x 0 )  x 0 então existe um q  Q tal que f ( x 0 )  q  x 0 . Porém
q  x 0  f (q )  f ( x 0 )  q  f ( x 0 ), o que é absurdo. O caso f ( x 0 )  x 0 é
análogo, o que termina o problema.

EXERCÍCIO 2
(IMO) Encontre todas as funções f : R  R tais que f ( x 2  f ( y ))  y  f ( x) 2 .
Dica: Prove que f ( x 2 ) ( f ( x)) 2 e que f ( x  y )  f ( x )  f ( y ), para x 0 e
y  R , então conclua. Se voce não conseguir concluir, puxa!! Você passou muito
perto da resolução.
EUREKA! N°9, 2000
44
Sociedade Brasileira de Matemática

EXERCÍCIO 3
(IMO) Encontre todas as funções f, definidas no conjunto dos reais não negativos
e assumindo valores reais não negativos, tais que:
i) f ( xf ( y )) f ( y )  f ( x  y ) para todo x, y 0
ii) f (2) 0
iii) f ( x ) 0 para 0 x  2
Dica: x 2 y  x 2 f ( y ). Incrível, não?
PONTO FIXO
Muitas vezes, é útil considerarmos os pontos fixos de uma função, isto é, pontos
x tais que f ( x ) x. Para mostrar que esta simples consideração leva, muitas
vezes, à solução do problema, observe abaixo o seguinte exemplo:

(IMO) Seja S o conjunto dos reais maiores que –1. Encontre todas as funções
f : S  S satisfazendo as condições
i) f ( x  f ( y )  xf ( y ))  y  f ( x )  yf ( x), x, y  S
ii) f ( x) x é estritamente crescente para – 1 < x < 0 e x > 0.
Para x = 0, temos f ( f ( y ))  y (1  f (0))  f (0), donde concluímos que f é
injetora. De f ( f (0))  f (0) e da injetividade de f, concluímos que f (0) 0.
Seja x 0 um ponto fixo de f. Sabemos da condição ii) que há no máximo um
ponto em cada um dos intervalos (–1; 0) e (0; + ). Substituindo x  y x 0 em
 
i), encontramos f x 02  2 x 0 x 02  2 x 0 .
Se x 0  (  1;0), x 02 2 x 0  (  1;0), logo x 02  2 x 0 x 0 , absurdo.
Analogamente, não há pontos fixos em ( 0;  ).
Assim, 0 é o único ponto fixo de f. Substituindo x = y em i), temos
f ( x  f ( x)  xf ( x)) x  f ( x)  xf ( x), ou seja xf (x ) é ponto fixo e,
portanto, igual a 0, logo f ( x)  x (1  x), que satisfaz i) e ii).
EXERCÍCIO 4
(IMO) Encontre todas as funções f definidas no conjunto dos reais positivos e
assumindo valores neste conjunto e que satisfaz as condições:
i) f ( xf ( y ))  yf ( x) para todo x, y  R * ;
ii) f ( x)  0 quando x  .

EXERCÍCIO 5
(Torneio das Cidades) Mostre que não existem funções f : R  R tais
f ( f ( x)) x 2  1996. Dica: utilize pontos fixos, mas utilize mesmo!
EXERCÍCIO 6

EUREKA! N°9, 2000


45
Sociedade Brasileira de Matemática

(IMO) Seja N 0 o conjunto dos inteiros não negativos. Encontre todas as funções
f : N 0  N 0 tais que f ( m  f ( n))  f ( f (m))  f ( n), m, n  N 0 .
Dica: considere o menor ponto fixo da função.
OLIMPÍADAS AO REDOR DO MUNDO
 O comitê editorial da EUREKA! sente-se gratificado pela acolhida desta
nova seção por parte dos seus leitores.
Aproveitamos a oportunidade para agradecer aqueles que nos enviaram
sugestões, opiniões, críticas e principalmente soluções para os problemas.
Cumpre informar, aos leitores, que por uma questão de espaço físico as soluções
de todos os problemas propostos, em um exemplar de EUREKA!, não poderão ser
apresentadas no número posterior ao daquele em que foram publicados visto que
a revista possui outras seções de grande interesse do público em geral.
Entretanto, as mesmas serão divulgadas nos números posteriores à
medida que os leitores as enviarem. Se houver interesse “mais urgente” na
solução de algum problema específico, solicitamos contactar a OBM, seção
OLIMPÍADAS AO REDOR DO MUNDO, através de carta ou e-mail.

Antonio Luiz Santos




Primeiramente vamos aos problemas propostos deste número


32. (Moldávia-1998) A seqüência  a n  , n  N* verifica as relações a 1 
1
2
e
a n 1
an  para todo número natural n  1 . Calcule a1  a 2   a1998 .
2na n  1  1
33. (Moldávia-1999) Seja n um número natural tal que 2n 2 possui 28 divisores
distintos e o número 3n 2 possui 30 divisores distintos . Qual o número de
divisores do número 6n 2 ?

34. (Ucrânia-1996) A seqüência  a n  , n 0 é tal que a 0 1 , a100 0 , e para


todo n 1 , tem-se que a n 1 2a1 a n  a n  1 .
a) Mostre que a1 1 .
b) Determine a1996

EUREKA! N°9, 2000


46
Sociedade Brasileira de Matemática

35. (Ucrânia-1997) Seja d  n  o maior divisor ímpar de um número natural n .


 2n 
Definamos uma função f : N  N tal que f  2n  1 2 n e f  2n  n   
 d  n 
para todo n  N. Determine todos os valores de k tais que
f  f ... f 1...  1997 onde f é iterada k vezes.

36. (China-1999) Seja PQRS um quadrilátero inscrito num círculo e cuja medida do
ângulo PSR seja igual a 90 o . Se H e K são os pés das perpendiculares
baixadas de Q sobre PR e PS respectivamente (convenientemente
prolongados se necessário). Mostre que HK divide QS ao meio.

37. (Rússia-1999) Os algarismos de um inteiro positivo A em sua representação


no sistema de numeração decimal crescem da esquerda para a direita. Determine
a soma dos algarismos do número 9 A .

38. (Japão-1999) Para um hexágono convexo ABCDEF cujos lados possuem


todos medidas iguais a 1 , determine o valor máximo M e o valor mínimo m
das diagonais AD , BE e CF e seus possíveis conjunto de valores.

39. (Irlanda-1999) Determine todos os inteiros positivos m tais que a quarta


potência do número de seus divisores positivos é igual a m .

40. (Irlanda-1999) Mostre que existe um número inteiro positivo na seqüência de


Fibonacci que é divisível por 1000 .

41. (Taiwan-1999) Seja P * o conjunto de todos os números primos ímpares


menores do que 10000 . Determine todos os números primos p  P * tal que
para cada subconjunto S de P * , digamos, S  p1 , p 2 , ... , p k  , com
k 2 , sempre que p  S , existe algum q em P * , mas não em S tal que
q  1 é um divisor de  p1  1 p 2  1  p k  1.
42. (Taiwan-1999) As alturas de um triângulo acutângulo ABC onde AB  AC
intersectam os lados BC , AC e BC nos pontos D , E e F

EUREKA! N°9, 2000


47
Sociedade Brasileira de Matemática

respectivamente. Se EF intersecta BC no ponto P e a reta que passa por D


e é paralela a EF intersecta AC e AB em Q e R respectivamente, seja N
um ponto sobre o lado BC tal que NQP  NRP  180o . Prove que
BN  CN .

43. (Bulgária-1999) Seja p um parâmetro real tal que a equação x 2  3 px  p 0


possui duas raízes reais distintas x1 e x 2 .
a) Prove que 3 px1  x 22  p  0 .
b) Determine o menor valor possível de
p 2
3 px 2  x12 3p
A  . Quando ocorre a igualdade ?
3 px1  x 22 3p p 2

44. (Bulgária-1999) Determine o menor número natural n tal que a soma dos
quadrados de seus divisores (incluindo 1 e n ) é igual a  n  3 2 .

45. (Bulgária-1999) Seja M o ponto médio do lado BC de um triângulo ABC


no qual CAB 45 o e ABC 30 o .
a) Determine AMC
AB BC
b) Prove que AM 
2 AC

46. (Bulgária-1999) Sejam M um ponto do interior de um quadrado ABCD e


A1 , B1 , C1 e D1 os pontos de interseção de AM , BM , CM e DM
respectivamente com o círculo circunscrito ao quadrado ABCD . Mostre que
A1 B1 C1 D1  A1 D1 B1C1 .

47. (Irã-1999) Determine todas as funções f : R  R que satisfazem a


 
f  f  x   y   f x 2  y  4 f  x  y para todos os números reais x e y .

48. (Irã-1999) Em um triângulo ABC a bissetriz do ângulo BAC intersecta o


lado BC no ponto D . Seja  um círculo tangente a BC no ponto D e que
passa pelo ponto A . Se M é o segundo ponto de interseção de AC com  e
se BM intersecta o círculo em P , mostre que AP é uma mediana do
triângulo ABD .

EUREKA! N°9, 2000


48
Sociedade Brasileira de Matemática

49. (Repúblicas Tcheca e Eslovaca-1999) Determine o menor número natural que pode
ser obtido colocando-se parêntesis na expressão
15 : 14 : 13 : 12 : 11 : 10 : 9 : 8 : 7 : 6 : 5 : 4 : 3 : 2

50. (Repúblicas Tcheca e Eslovaca-1999) A média aritmética de uma quantidade de


números primos distintos é igual a 27 . Determine o maior número primo que
aparece entre eles.

51. (Repúblicas Tcheca e Eslovaca-1999) Mostre que para todo número natural n o
produto
 2  2  2  2
 4   4   4    4  
 1  2  3  n
é um inteiro.

52. (Espanha-1998) As tangentes dos ângulos de um triângulo são inteiros


positivos. Determine estes números.

53. (Espanha-1998) Determine todas as funções estritamente crescentes


f : N*  N* tais que f  n  f  n    2 f  n 

54. (Eslovênia-1999) Seja O o centro do círculo circunscrito ao triângulo ABC .


Se P e Q são os pontos médios de AO e BC respectivamente, determine a
medida do ângulo OPQ se  CBA 4 OPQ e  ACB 6  OPQ .

55. (Eslovênia-1999) Determine todos os inteiros x e y que satisfazem à equação


x 3  9 xy  127  y 3 .

56. (Estônia-1999) Determine todos os valores de a tais que o valor absoluto de


uma das raízes da equação x 2   a  2  x  2a 2  5a  3 0 seja igual a duas
vezes o valor absoluto da outra raiz.

57. (Estônia-1999) Sejam O1 e O2 os centros de dois círculos que não se


intersectam e de mesmo raio. Se s é a reta que passa pelos seus centros e t é
sua tangente comum externa, considere um círculo tangente aos dois círculos nos
pontos K e L e também tangente às retas s e t nos pontos M e P
respectivamente. Determine a medida de O1O 2 . Mostre ainda que os pontos
M , K e N estão alinhados onde N é o ponto de tangência da reta t com o
primeiro círculo.

EUREKA! N°9, 2000


49
Sociedade Brasileira de Matemática

58. (St.Petersburg-1999) 150 bolas de encher (bexigas) vermelhas , 150 azuis e


150 verdes flutuam sob o teto de um circo. Existem exatamente 13 bolas
verdes dentro de cada bola azul e exatamente 5 bolas azuis e 19 bolas verdes
dentro de cada bola vermelha. Mostre que algumas bolas verdes não estão
contidas no interior de nenhuma das outras 449 bolas.

59. (St.Petersburg-1999) Todos os números inteiros positivos não superiores a 100


são escritos em ambos os lados de 50 cartas (cada número é escrito exatamente
uma vez). Estas cartas são postas sobre uma mesa de modo que somente os
números que estejam virados para cima podem ser vistos. Gustavo pode escolher
várias cartas, virá-las e então calcular a soma de todos os 50 números que
aparecem agora. Qual é o valor máximo da soma S tal que Gustavo pode com
certeza obter uma soma não inferior a S ?

60. (St.Petersburg-1999) Três mágicos apresentam um truque entregando a uma


pessoa da platéia um maço de cartas numeradas com 1,2,...,2n  1( n  6). O
espectador fica com uma das cartas e aleatoriamente distribui as restantes entre o
primeiro e o segundo mágicos (cada um deles fica com n cartas) . Estes olham
suas cartas (sem se comunicar um ao outro) e cada um escolhe duas cartas
formando um maço (ordenado) com estas cartas e as entrega ao terceiro mágico.
O terceiro mágico olha estas quatro cartas e anuncia a carta que ficou com o
espectador. Explique como este truque pode funcionar.



Agora vamos aos comentários e soluções dos leitores para alguns dos
problemas apresentados no número anterior de EUREKA!. O critério por nós
adotado para este número foi apresentar as soluções dos problemas que foram,
até o presente momento, resolvidos pelo maior número de leitores.

4. (Reino Unido-1998) Em um triângulo ABC , D é o ponto médio de AB e E


é um ponto do lado BC tal que BE  2 EC . Sabendo que  ADC BAE
determine a medida do ângulo BAC .

Enviaram soluções Einstein do Nascimento Júnior (Fortaleza-CE), Geraldo


Perlino Júnior (SP) e Diego Alvarez Araújo Correia (Fortaleza-CE).

Solução de Einstein do Nascimento Júnior:

EUREKA! N°9, 2000


50
Sociedade Brasileira de Matemática

Sejam   ADC BAE e P  AE  CD e tracemos pelo ponto D uma


reta paralela a AE e que intersecta o lado BC no ponto Q . Como EC  EQ
e DQ // PE então AP  PD . Daí, PD  PA  PC e seja
 PCA PAC  logo,
 PCA   PAC   PAB  PDA 180 o  2  2 180 o     90 o .

7. (Rússia-1998) Existem números de n algarismos M e N onde todos os algarismos


de M sejam pares, todos os algarismos de N sejam ímpares, cada um dos
algarismos de 0 a 9 ocorrendo exatamente uma vez entre M e N e tais que M
divide N ?
Enviaram soluções com comentários sobre um possível erro no enunciado: Diego
Alvarez Araújo Correia (Fortaleza-CE) e Marcílio Miranda de Carvalho (Teresina
– PI).

Solução de Marcílio Miranda de Carvalho:

M |N M
  K  N absurdo!
par ímpar

Conclusão : Não existem M e N que satisfaçam às condições do problema


Marcílio também observou que como a Rússia tem grande tradição em IMO’s
deveria haver um erro no enunciado e que o mesmo possivelmente deva ser : “ ...
e tais que N divide M ” apresentando a seguinte solução para o novo
enunciado :
M 0  2  4  6  8 2 mod 9
N 1  3  5  7  9 7 mod 9 

N | M  N K  M  7 K 2 mod 9  K 8 mod 9   K 8.


Mas N 13579  N K  105  M possui mais de 5 algarismos.
Conclusão : Não existem M e N que satisfaçam ás condições do problema.

8. (Romênia-1998) O volume de um paralelepípedo é 216cm 3 e a sua área total é


216cm 2 . Mostre que o paralelepípedo é um cubo.

Enviaram soluções Einstein do Nascimento Júnior (Fortaleza-CE), Geraldo


Perlino Júnior (SP), José Guilherme Moreira Pinto (Juiz de Fora - MG) e Diego
Alvarez Araújo Correia (Fortaleza-CE).

EUREKA! N°9, 2000


51
Sociedade Brasileira de Matemática

Solução de Diego Alvarez Araújo Correia:


Sejam a , b e c as medidas das dimensões do paralelepípedo. Pelo enunciado
tem-se : abc 216 e ab  ac  bc 108 .
ab  bc  ac 3
Como MA  MG ,  ab bc ac então
3
108 3 2 2
  abc  3  216 36  36 36 . Como ocorre a igualdade, temos que
3
ab bc ca  a b c .

13. (Irlanda-1999) Uma função f : N  N satisfaz às condições :

f  ab   f  a  f  b  se o máximo divisor comum de a e b é 1,


f  p  q   f  p   f  q  para todos os números primos p e q.
Mostre que f  2  2, f  3 3 e f 1999 1999 .

Enviaram soluções Marcílio Miranda de Carvalho (Teresina – PI) e Geraldo


Perlino Júnior (SP).

Solução de Marcílio Miranda de Carvalho:


Seja p um número primo ímpar, então f  2 p   f  2   f  p  . Como,
f  2 p   f  p   f  p   2 f  p   f  2  2
Além disso, f  4   f  2   f  2  4  f 12 4 f  3 .

Por outro lado

f 12  f  7   f  5  f 12 2 f  2   f  3  f  2   f  3 6  2 f  3  f  3 3 .

Finalmente,

f  5  f  2   f  3 5  f 15 15  f 13 13  f  26  26  f  23 23.


Mas, f 13 13  f 11 11  f  33 33  f  31 31  f  29 29 . Logo,
f  2001  f  3 f  23 f  29 2001  f 1999 1999

14. (Suíça-1999) Determine todas as funções f : R\  0  R satisfazendo a


1 1
f   x   f    x para todos x  R\  0 .
x  x

EUREKA! N°9, 2000


52
Sociedade Brasileira de Matemática

Enviaram soluções Marcílio Miranda de Carvalho (Teresina – PI) e Geraldo


Perlino Júnior (SP).
Solução de Geraldo Perlino Júnior:
Seja a 1
R com a 0 . Fazendo x  a e x  na equação dada temos
a
1  1  1 1
 f  a   f      a e a f     f  a   . Resolvendo-se o sistema
a  a   a  a
1 2 1
formado por estas duas equações chegamos a f  a    a   e portanto,
2 a
1 1
f  x   x 2   .
2 x

15. (Suíça-1999) Dois círculos intersectam-se em dois pontos M e N . Um


ponto A qualquer do primeiro círculo, distinto de M e N , é unido aos pontos
M e N de modo que as retas AM e AN intersectam novamente o segundo
círculo nos pontos B e C . Mostre que a tangente ao primeiro círculo em A é
paralela a BC .

Enviaram soluções Einstein do Nascimento Júnior (Fortaleza-CE), Geraldo


Perlino Júnior (SP) e Diego Alvarez Araújo Correia (Fortaleza-CE) .

Solução de Einstein do Nascimento Júnior:


Seja MNA  e P um ponto sobre a tangente ao primeiro círculo em A .
Temos então que  MAP  MNA e como o quadrilátero MNBC é
inscritível então MNB 180 o   e daí MCB 
resultando em  ACB CAP  AP // BC .

17. (Ucrânia-1999) Mostre que o número 9999999  1999000 é composto.

Solução de Marcílio Miranda de Carvalho:


9999999  1999000 10 7  1  2 10 3  1 10 3  
6 6 3 3
 
9 10  10  1  2 10  10 3 10 3 10 3  1  3 10 6  10 3  3 10 3  1 
6

         
3 10 3 3 10 3  1  3 10 3 10 3  1  10 3  1 3 10 3 3 10 3  1  3 10 3  1 10 3  1 
 3
 3 3
 3 10  1 3 10  10  1 2999 4001 

EUREKA! N°9, 2000


53
Sociedade Brasileira de Matemática

19. (Lituânia-1999) Duas cordas AB e CD de um círculo intersectam-se no


ponto K . O ponto A divide o arco CAD em duas partes iguais. Se AK  a
e KB b , determine a medida da corda AD .

Enviaram soluções Marcílio Miranda de Carvalho (Teresina – PI), Einstein do


Nascimento Júnior (Fortaleza-CE), e Geraldo Perlino (SP) e Geraldo Perlino
Júnior (SP).

Solução de Marcílio Miranda de Carvalho:

Seja  ABD ADK  então tem-se que os triângulos ADK e ABD são
semelhantes logo,
AD a  b 2
   AD  a a  b   AD  a a  b 
a AD

21. (Estônia-1999) Determine o valor da expressão


 1   2   1999   2000   2000   2000 
f  f    f   f  f    f  
 2000   2000   2000   2000   1999   1 
x2
supondo que f  x   .
1  x2

Enviaram soluções Marcílio Miranda de Carvalho (Teresina – PI), Einstein do


Nascimento Júnior (Fortaleza-CE), Geraldo Perlino Júnior (SP), Diego Alvarez
Araújo Correia (Fortaleza-CE) e Gibran Medeiros de Souza (Natal-RN).

Solução de Marcílio Miranda de Carvalho:


2
1 1
 
1 x2  x x2 2
x 2  x  1 1
f  x  f    2
 2
  . Logo o valor da
 x  1 x 1 1  x2 x2  1 x2  1
1  
 x x2
expressão é
 2000  3999
1999  f   1999  f 1 
 2000  2

EUREKA! N°9, 2000


54
Sociedade Brasileira de Matemática

1 1 1 1
22. (Eslovênia-1999) Inicialmente os números 1, , , ..., , são
2 3 1998 1999
escritos em um quadro negro. Em cada passo, escolhemos dois destes números,
digamos a e b, e os substituímos pelo número a  b  ab . Continuamos
desta maneira até que reste um único número no quadro negro. É possível que
este número seja 2000 ? Justifique sua resposta.

Enviaram soluções Einstein do Nascimento Júnior (Fortaleza-CE) e Geraldo


Perlino Júnior (SP).

Resumo da solução de ambos com adaptações:

Seja a  b a  b  ab . É fácil ver que a  b b  a e


a   b  c   a  b   c e portanto podemos escolher os números em qualquer
ordem. Calculemos os resultados então na ordem dada :
1 1 1
1  2, 2  3,...,1998  1999
2 3 1999
Deste modo vemos que nós sempre alcançaremos 1999 não podendo então
alcançar 2000 .

EUREKA! N°9, 2000


55
Sociedade Brasileira de Matemática

SOLUÇÕES DE PROBLEMAS PROPOSTOS


 Publicamos aqui algumas das respostas enviadas por nossos leitores.
42) Suponha que a, b e c são as medidas dos lados de um triângulo ABC, com
semi-perímetro p e área S, verifique que
1 1 1 3 p
   
a b c 2 s
e mais ainda: verifique que a igualdade acima ocorre apenas se o triângulo
for equilátero.

Solução de Marcelo Rufino de Oliveira (Belém-PA):

Sejam x = a + c – b y = a + b – c z = b + c – a
Pela Desigualdade Triangular temos que x > 0, y > 0 e z > 0.
Assim, isoloando a, b e c: a = x + y b = y + z c = z + x
Pela Desigualdade entre as Médias Aritmética e Geométrica temos:
x  y 2 xy , y  z 2 yz e z  x 2 zx (1)
2
 1 1 1
Vamos desenvolver agora o valor de     em função de x, y e z, usando
a b c
para isso o resultado (1):
2 2 2
2
 1 1 1  1 1 1   1 1 1  1 1 1 1 
               
a b c  x  y y  z z  x   2 xy 2 yz 2 zx


 4  xy yz zx 
Pela Desigualdade de Cauchy podemos afirmar que
(a1  a 2  a3 ) 2 3(a12  a 22  a 32 ) .
1 1 1
Fazendo então a1  , a2  e a3  concluímos que:
xy yz zx
2
 1 1 1 3 1 1 1 
         (2)
 a b c  4  xy yz zx 
p
Calculando em função de x, y e z obtemos:
s
p xyz xyz 1 1 1
    
s ( x  y  z ) xyz xyz xy yz zx

Assim, usando o resultado (2):


EUREKA! N°9, 2000
56
Sociedade Brasileira de Matemática

2
 1 1 1 3 1 1 1  3 p2 1 1 1 3 p
             (3)
a b c 4  xy yz zx  4 s 2 a b c 2 s

Como nas duas desigualdade usadas a igualdade vale se e só se os termos são


iguais, então temos a igualdade na desigualdade (3) sé e só se a = b = c.

43) Prove que se p é um primo da forma 4k + 3, então 2p + 1 também é primo se


e somente se 2p + 1 divide 2p – 1.

Solução de Alex Corrêa Abreu (Niterói - RJ):

Primeira parte:

Se 2p + 1 também é primo, temos:

2  ( 2 p 1) 1(mod 2 p  1)  2 2 p 1(mod 2 p  1) 


2 p 1(mod 2 p  1), só que 2 p  1 é da forma 8k  7, logo
2 p 1(mod 2 p  1)  2 p  1 2 p  1

Segunda parte:

Se 2 p 1(mod 2 p  1), como p é primo então :


p ord 2 p 1 2, só que (2,2 p  1 ) 1 
 ( 2 p  1) kp, com k 2.
Não podemos ter k = 1, pois  (n) é par para todo n 3. Assim,
 ( 2 p  1) 2 p  2 p  1 é primo.

44) O produto de dois inteiros positivos consecutivos pode ser igual ao produto
de dois inteiros positivos consecutivos pares?

Solução de Daniel Pessôa Martins Cunha (Fortaleza - CE):

EUREKA! N°9, 2000


57
Sociedade Brasileira de Matemática

Seja A o produto de dois números inteiros positivos consecutivos. Isto implica


que A x( x  1) onde x  Z .
Observe que:
- Caso x seja par temos:
( x  2) x  x( x  1)  x( x  2)  ( x  2) x  A  x ( x  2)
(Esta desigualdade é fácil de ser vista)
- Caso x seja ímpar temos: (Logo x + 1 é par)
( x  1)( x  1)  x( x  1)  ( x  1)( x  3)  ( x  1)( x  1)  A  ( x  1)( x  3)
(desigualdade fácil de ser vista)
Analizando os casos vemos que A está entre dois produtos consecutivos de dois
inteiros pares consecutivos positivos.

Logo conclui-se que não é possível que o produto de dois inteiros positivos seja
igual ao produto de dois inteiros positivos consecutivos pares.

46) (Baltic Way, 1997)


i) Prove a existência de dois conjuntos infinitos A e B, não necessariamente
disjuntos, de inteiros não negativos tais que cada inteiro não negativo pode
ser representado de uma única forma como a + b, com a  A e b  B.

ii) Prove que em cada tal par (A, B), ou A ou B contém apenas múltiplos de
algum inteiro k > 1.

Solução de Humberto Silva Naves (São Paulo - SP):

i) Todo natural se escreve da maneira única como soma de potências de 2

distintas, donde os conjuntos A {nN | n se escreve como soma de potências de 2

distintas com expoente ímpar} e B {nN | n se escreve como soma de potências de

2 distintas com expoente par} satisfazem as condições do enunciado (note que


0  A e 0  B ).
ii) 0 deve pertencer aos dois conjuntos, e 1 a exatamente um deles (senão 1 = 0 +
1 = 1 + 0 seria representado de 2 formas distintas), digamos ao conjunto B. Seja k
o menor elemento positivo do conjunto A. É fácil ver que {0, 1, ..., k – 1}  B.

EUREKA! N°9, 2000


58
Sociedade Brasileira de Matemática

Vamos provar que o mdc dos elementos de A é k.


Para isso, vamos mostrar por indução que para cada inteiro positivo m, existem
inteiros r1 ,..., rs tais que {x  B | x  mk} U {ri k , ri k  1,..., ri k  k  1}, e
i 1
todos os elementos de A menores que mk são múltiplos de k. Consideremos o
inteiro mk.
Ele deve ser escrito de maneira única como soma de um elemento de A com um
elemento de B. Se mk  A, podemos escrever os elementos
mk , mk  1,..., mk  k  1 Como soma de um elemento de A(mk) com um
elemento de B (pertencente a {0,1,..., k  1}. Se mk  A, podemos escrever
mk m  m( k  ), com m A e m( k  )  B.
Devemos ter   k. Se  1, por hipótese de indução,
m( k  ), m( k  ) ,..., m(k  )  k  1 pertencem a B , donde, para
O  j k  1, mk  j ml  ( m( k  )  j ) é soma de elementos de A e de B
menores que mk, donde nenhum dos mk  j , 0  j k  1 pertencem a A nem a
B (pela unicidade da representação, senão poderíamos escrevê-los como
0  ( mk  j ) ou ( mk  j )  0), o que prova a afirmação para m + 1. Se
 0, mk  B. Queremos mostrar que para 0  j k  1, mk  j pertence a B (e
logo não pertence a A), provando a afirmação para m  1. Suponha o contrário, e
considere o menor j com 0  j k  1 tal que mk + j não pertence a B. Devemos
ter mk + j = x + y, com x A \ {0} e y  B. Se x  mk , x rk e
y ( m  r ) k  j , donde por hipótese de indução, (m  r ) k  B e
mk 0  mk rk  ( m  r )k , contradizendo a unicidade. Se x mk , como
mk , mk  1,..., mk  j  1 pertencem a B (e portanto não pertencem a A),
devemos ter x mk  j e y 0, mas nesse caso teríamos
(m  1)k k  mk (mk  j )  (k  j ), contradizendo novamente a unicidade.

Agradecemos também o envio das soluções e a colaboração de:

José Guilherme Moreira Pinto (Juiz de Fora - MG)


Osvaldo Melo Sponquiado (Olímpia - SP)
Diêgo Veloso Uchoa (Teresina - PI)
Nijair Araújo Pinto (Fortaleza - CE)
Gibran M. de Souza (Natal - RN)
Carlos Alberto da Silva Victor (Nilópolis - RJ)
Einstein do Nascimento Júnior (Fortaleza - CE)
Samuel Barbosa Feitosa (Fortaleza - CE)
Geraldo Perlino Jr. (São Paulo - SP)

EUREKA! N°9, 2000


59
Sociedade Brasileira de Matemática

PROBLEMAS PROPOSTOS
 Convidamos o leitor a enviar soluções dos problemas propostos e sugestões de novos
problemas para os próximos números.

47) Dada uma circunferência  , trace as tangentes a ela por um ponto exterior,
A, tocando-a em M e N. Trace a reta r passando por A e tocando  em B e C. Se
D é o ponto médio de MN , prove que MN é a bissetriz de BDC .

48) Doze pintores vivem em doze casas construídas ao longo de uma rua circular
e são pintadas ou de branco ou de azul. Cada mês um dos pintores, pegando
consigo bastante tinta branca e azul, deixa sua casa e caminha ao longo da rua no
sentido anti-horário. Desta forma, ele repinta cada casa (iniciando na sua) com a
cor oposta. Finaliza o trabalho tão longo repinte alguma casa branca de azul. Em
um ano, cada casa estará pintada com a sua cor original sabendo que, no começo
do ano, ao menos uma casa estava pintada de azul.

49) Dado um polígono regular de n lados.


Assinale aleatoriamente, no seu interior, um ponto M. Sendo x1 , x 2 ,..., x n as
distâncias de M a cada um dos lados, verifique que:
1 1 1 2
  ...  , onde a é a medida do lado do poligono.
x1 x 2 xn a

50) Calcule o determinante:

MDC (1, 1) MDC (1,2)  MDC (1, n)


MDC (2,1) MDC (2,2)  MDC (2, n)
  
MDC (n,1) MDC (n,2)  MDC (n, n)
Onde MDC (a, b) é o máximo divisor comum de a e b.

51) Três feirantes foram vender melancias. Um levou 10; outro 16; o terceiro,
26. Todos venderam algumas melancias pelo mesmo preço até o meio dia. Depois
disso, os três baixaram o preço, mas continuaram vendendo por preços iguais.

EUREKA! N°9, 2000


60
Sociedade Brasileira de Matemática

Quando voltaram para casa, após venderem todas as melancias, cada um tinha a
mesma quantia de dinheiro; 35 mil cruzeiros. Por quanto foi vendida cada
melancia antes e após o meio-dia?

Problema 47 proposto por Carlos Lucas de Melo Pontes e Silva (Fortaleza - CE), problemas
48 e 51 propostos por Jorge Luis Rodrigues Costa (Fortaleza - CE) e problemas 49 e 50
propostos por Carlos A. Gomes (Natal - RN).
COORDENADORES REGIONAIS

Amarísio da Silva Araújo (UFV) Viçosa - MG


Alberto Hassen Raad (UFJF) Juiz de Fora - MG
Angela Camargo (Centro de Educ.de Adultos - CEA) Blumenau - SC
Benedito T. Vasconcelos Freire (UFRN) Natal - RN
Claudio Arconcher (Col. Leonardo da Vinci) Jundiaí - SP
Claus Haetinger (UNIVATES) Lajeado - RS
Cleonor Crescêncio das Neves (UTAM) Manaus-AM
Élio Mega (Col. ETAPA) São Paulo - SP
Kátia Gonçalves de Faria (Col. Singular) Santo André - SP
Florêncio F. Guimarães Filho (UFES) Vitória - ES
Francisco Dutenhefner (UFMG) Belo Horizonte - MG
Gisele de A. Prateado Gusmão (UFGO) Goiânia - GO
Ivanilde H. Fernandes Saad (U. Católica Dom Bosco) Campo Grande - MS
Jacqueline F. Rojas Arancibia (UFPB) João Pessoa - PB
João Benício de Melo Neto (UFPI) Teresina - PI
João F. Melo Libonati (Grupo Educ. IDEAL) Belém - PA
Irene Nakaoka (UEM) Maringá - PR
José Carlos Pinto Leivas (UFRG) Rio Grande - RS
José Cloves Saraiva (UFMA) São Luis - MA
José Gaspar Ruas Filho (ICMC-USP) São Carlos - SP
José Luis Rosas Pinho (UFSC) Florianópolis - SC
José Paulo Carneiro (Univ. Santa Úrsula) Rio de Janeiro - RJ
José Vieira Alves (UFPB) Campina Grande - PB
Marcelo Rufino de Oliveira (Sistema Titular de Ensino)Belém - PA
Licio Hernandes Bezerra (UFSC) Florianópolis - SC
Luzinalva M. de Amorim (UFBA) Salvador - BA
Marcondes Cavalcante França (UF Ceará) Fortaleza - CE
Pablo Rodrigo Ganassim (L. Albert Einstein) Piracicaba - SP
Paulo H. Cruz Neiva de L. Jr. (Esc. Tec.Everardo Passos) SJ dos Campos - SP
Reinaldo Gen Ichiro Arakaki (INPE) SJ dos Campos - SP
Ricardo Amorim (Centro Educ. Logos) Nova Iguaçu - RJ
Roberto Vizeu Barros (Colégio ACAE) Volta Redonda - RJ
Sérgio Cláudio Ramos (IM-UFRGS) Porto Alegre - RS
Seme Gebara Neto (UFMG) Belo Horizonte -MG

EUREKA! N°9, 2000


61
Sociedade Brasileira de Matemática

Silvio de Barros Melo (UFPE) Recife - PE


Tadeu Ferreira Gomes (U. do Estado da Bahia) Juazeiro - BA
Tomás Menéndez Rodrigues (U. Federal de Rondonia) Porto Velho - RO
Valdenberg Araújo da Silva (U. Federal de Sergipe) São Cristovão - SE
Wagner Pereira Lopes (Esc. Tec. Fed. de Goiás) Jataí - GO
Waldemar M. Canalli (P.M. S. João de Meriti) S. João de Meriti - RJ

CADASTRAMENTO 2001

Colégios
(Preencher com letra de forma)

Instituição:
Pública  Privada 
Diretor:
Endereço:
Bairro:
Cidade: Estado:
Cep:
Telefone: ( )
Fax: ( )
e-mail:

Professor Responsável:

Endereço:
Bairro:
Cidade: Estado:
Cep:
Telefone: ( )

EUREKA! N°9, 2000


62
Sociedade Brasileira de Matemática

Fax: ( )
e-mail:

Para seguir participando Olimpíada Brasileira de Matemática, uma cópia desta ficha
deve ser preenchida e enviada para a Secretaria da Olimpíada Brasileira de Matemática
pelos colégios ainda não recadastrados.

EUREKA! N°9, 2000


63
CONTEÚDO

AOS LEITORES 2

XXII OLIMPÍADA BRASILEIRA DE MATEMÁTICA 3


Problemas e soluções da primeira fase

XXII OLIMPÍADA BRASILEIRA DE MATEMÁTICA 13


Problemas e soluções da segunda fase

XXII OLIMPÍADA BRASILEIRA DE MATEMÁTICA 21


Problemas e soluções da terceira fase

XXII OLIMPÍADA BRASILEIRA DE MATEMÁTICA 34


Resultados

ARTIGOS

OS NÚMEROS IRRACIONAIS 37
Hermano Frid

OLIMPÍADAS AO REDOR DO MUNDO 47

SOLUÇÕES DE PROBLEMAS PROPOSTOS 52

PROBLEMAS PROPOSTOS 59

AGENDA OLÍMPICA 60

COORDENADORES REGIONAIS 61
Sociedade Brasileira de Matemática

AOS LEITORES

Realizamos durante o ano 2000 a XXII Olimpíada Brasileira de


Matemática, atingindo na realização da Primeira Fase cerca de 80.000 alunos.
Este ano a Olimpíada Brasileira de Matemática terá importantes modificações em
relação à aplicação das provas da segunda e terceira fases (para maiores
esclarecimentos leia com atenção o nosso novo regulamento publicado no site:
http://www.obm.org.br/).  Além disso a competição contará pela
primeira vez com a participação de alunos de ensino superior, para os quais foi
criado o nível Universitário. Assim, a partir deste ano, a OBM passa a ser
realizada em 4 níveis de competição. O calendário para este ano é o seguinte:

NÍVEIS 1, 2 e 3

Primeira Fase – Sábado, 09 de junho


Segunda Fase – Sábado, 01 de setembro
Terceira Fase – Sábado, 20 de outubro (níveis 1, 2 e 3)
Domingo, 21 de outubro (Níveis 2 e 3 – Segundo dia de prova).

NÍVEL UNIVERSITÁRIO

Primeira Fase – Sábado, 01 de setembro


Segunda Fase – Sábado, 20 de outubro e Domingo, 21 de outubro

Gostaríamos de registrar a realização da IV Semana Olímpica. Neste ano


o evento teve lugar no Colégio Militar de Salvador (Salvador – BA) entre os dias
19 a 26 de janeiro. Aproveitamos a oportunidade para expressar o nosso
agradecimento pela calorosa acolhida.

Durante a IV Semana Olímpica, reunimos os alunos premiados na XXII


OBM nos três níveis de competição. Um arquivo com as aulas ministradas
durante o evento pode ser consultado no seguinte endereço eletrônico:
http://www.obm.org.br/semana.htm

Por fim, queremos agradecer aos alunos que têm nos ajudado com a
revisão da revista EUREKA!

EUREKA! N°10, 2001


2
Sociedade Brasileira de Matemática

Os editores.
XXII OLIMPÍADA BRASILEIRA DE MATEMÁTICA
Problemas e soluções da primeira fase – nível 1

1. Observe as multiplicações a seguir:

12 345 679  18 = 222 222 222


12 345 679  27 = 333 333 333
12 345 679  54 = 666 666 666

Para obter 999 999 999 devemos multiplicar 12 345 679 por:
A) 29 B) 99 C) 72 D) 41 E) 81

2. Outro dia ganhei 250 reais, incluindo o pagamento de horas extras. O


salário (sem horas extras) excede em 200 reais o que recebi pelas horas extras.
Qual é o meu salário sem horas extras?
A) 200 reais B) 150 reais C) 225 reais D) 175 reais E) 180 reais

3. Num relógio digital, que marca de 0:00 até 23:59, quantas vezes por dia
o mostrador apresenta todos os algarismos iguais?
A) 10 B) 8 C) 6 D) 7 E) 9

4. A prefeitura de uma certa cidade fez uma campanha que permite trocar 4
garrafas de 1 litro vazias por uma garrafa de 1 litro cheia de leite. Até quantos
litros de leite pode obter uma pessoa que possua 43 dessas garrafas vazias?
A) 11 B) 12 C) 13 D) 14 E) 15

5. Numa caixa havia várias bolas, sendo 5 azuis, 4 amarelas, 3 vermelhas,


2 brancas e 1 preta. Renato retirou 3 bolas da caixa. Sabendo que nenhuma delas
era azul, nem amarela, nem preta, podemos afirmar a respeito dessas 3 bolas que:
A) são da mesma cor. B) são vermelhas. C) uma é vermelha e duas são
brancas.
D) uma é branca e duas são vermelhas. E) pelo menos uma é vermelha.

6. Se a área do retângulo dado é 12, qual é a área da figura sombreada?

EUREKA! N°10, 2001


3
Sociedade Brasileira de Matemática

A) 3 B) 4 C) 5 D) 6 E) 8

7. O número 10 pode ser escrito de duas formas como soma de dois números
primos: 10 = 5 + 5 e 10 = 7 + 3. De quantas maneiras podemos expressar o
número 25 como uma soma de dois números primos?
A) 4 B) 1 C) 2 D) 3 E) nenhuma

8. 1 litro de álcool custa R$0,75. O carro de Henrique percorre 25 km com 3


litros de álcool. Quantos reais serão gastos em álcool para percorrer 600 km?
A) 54 B) 72 C) 50 D) 52 E) 45

9. Um certo número N de dois algarismos é o quadrado de um número natural.


Invertendo-se a ordem dos algarismos desse número, obtém-se um número ímpar.
A diferença entre os dois números é o cubo de um número natural. Podemos
afirmar que a soma dos algarismos de N é:
A) 7 B) 10 C) 13 D) 9 E) 11

10. Juliano colou uma bandeirinha cinza em cada engrenagem, como mostra a
figura abaixo:

As engrenagens são iguais e quando a engrenagem da esquerda girou um


pouco, a sua bandeirinha ficou na posição indicada com a bandeirinha branca
pontilhada. Nesta condição, podemos afirmar que a posição da ban deirinha na
engrenagem da direita é:
a) b) c) d) e)

A) B) C) D) E)

11. Uma fábrica embala 8 latas de palmito em caixas de papelão cúbicas de 20


cm de lado. Para que possam ser melhor transportadas, essas caixas são
colocadas, da melhor maneira possível, em caixotes de madeira de 80 cm de
largura por 120 cm de comprimento por 60 cm de altura. O número de latas de
palmito em cada caixote é
A) 576 B) 4.608 C) 2.304 D) 720 E) 144

EUREKA! N°10, 2001


4
Sociedade Brasileira de Matemática

12. Há 18 anos Hélio tinha precisamente três vezes a idade de seu filho. Agora
tem o dobro da idade desse filho. Quantos anos têm Hélio e seu filho?
A) 72 anos e 36 anos. B) 36 anos e 18 anos. C) 40 anos e 20 anos.
D) 50 anos e 25 anos. E) 38 anos e 19 anos.

13. Se os números naturais são colocados em colunas, como se mostra


abaixo, debaixo de que letra aparecerá o número 2000?
A B C D E F G H I
1 2 3 4 5
9 8 7 6
10 11 12 13 14
18 17 16 15
19 20 21 ... ...

A) F B) B C) C D) I E) A

14. O emir Abdel Azir ficou famoso por vários motivos. Ele teve mais
de 39 filhos, incluindo muitos gêmeos. De fato, o historiador Ahmed Aab afirma
num dos seus escritos que todos os filhos do emir eram gêmeos duplos, exceto
39; todos eram gêmeos triplos, exceto 39; todos eram gêmeos quádruplos, exceto
39. O numero de filhos do emir é:
A) 111 B) 48 C) 51 D) 78 E) 75

15. Quatro amigos vão visitar um museu e um deles resolve entrar sem pagar.
Aparece um fiscal que quer saber qual deles entrou sem pagar.

– Eu não fui, diz o Benjamim. – Foi o Carlos, diz o Mário.


– Foi o Pedro, diz o Carlos. – O Mário não tem razão, diz o Pedro.

Só um deles mentiu. Quem não pagou a entrada do museu?

A) Mário B) Pedro C) Benjamim D) Carlos


E) não é possível saber, pois faltam dados

16. Em um jogo de duas pessoas, os jogadores tiram, alternadamente, 1, 2, 3, 4


ou 5 palitos de uma pilha que inicialmente tem 1000 palitos. Ganha o jogador que
tirar o último palito da pilha. Quantos palitos o jogador que começa deve tirar na
sua jogada inicial de modo a assegurar sua vitória?
A) 1 B) 2 C) 3 D) 4 E) 5

17. Quantos números inteiros e positivos menores do que 1.000.000 existem

EUREKA! N°10, 2001


5
Sociedade Brasileira de Matemática

cujos cubos terminam em 1?


A) 1.000 B) 10.000 C) 50.000 D) 100.000 E) 500.000
18. Os 61 aprovados em um concurso, cujas notas foram todas
distintas, foram distribuídos em duas turmas, de acordo com a nota obtida no
concurso: os 31 primeiros foram colocados na turma A e os 30 seguintes na turma
B. As médias das duas turmas no concurso foram calculadas. Depois, no entanto,
decidiu-se passar o último colocado da turma A para a turma B. Com isso:

A) A média da turma A melhorou, mas a da B piorou.


B) A média da turma A piorou, mas a da B melhorou.
C) As médias de ambas as turmas melhoraram.
D) As médias de ambas as turmas pioraram.
E) As médias das turmas podem melhorar ou piorar, dependendo das notas
dos candidatos.

19. Escrevem-se, em ordem crescente, os números inteiros e positivos que sejam


múltiplos de 7 ou de 8 (ou de ambos), obtendo-se 7, 8, 14, 16, ... . O 100 o número
escrito é:
A) 406 B) 376 C) 392 D) 384 E) 400

20. A figura abaixo foi desenhada em cartolina e dobrada de modo a formar um


cubo.

Qual das alternativas mostra o cubo assim formado?

A) C) D) E)
B)

GABARITO
NÍVEL 1

1) E 6) D 11) A 16) D
2) C 7) B 12) A 17) D
3) B 8) A 13) C 18) C

EUREKA! N°10, 2001


6
Sociedade Brasileira de Matemática

4) D 9) D 14) C 19) E
5) E 10) A 15) B 20) B
XXII OLIMPÍADA BRASILEIRA DE MATEMÁTICA
Problemas e soluções da primeira fase – nível 2

1. Veja o problema 17 do nível 1.


2. Veja o problema 11 do nível 1.
3. Veja o problema 10 do nível 1.
4. Veja o problema 15 do nível 1.
5. Veja o problema 18 do nível 1.

6. No triângulo ABC representado ao lado, a B A


Medida do ângulo Ĉ é 60° e a bissetriz do
ângulo B̂ forma 70° com a altura relativa ao
vértice A. A medida do ângulo  é:

A) 50° B) 30° C) 40° D) 80° E) 70° C

7. Veja o problema 6 do nível 1.

8. Alberto, Beatriz e Carlos correm numa pista circular. Todos saem ao


mesmo tempo e do mesmo lugar, cada um desenvolvendo velocidade constante.
Alberto e Beatriz correm no mesmo sentido. Correndo no sentido oposto, Carlos
encontra Alberto, pela primeira vez, exatamente 90 segundos após o início da
corrida e encontra Beatriz exatamente 15 segundos depois. Quantos segundos
são necessários para que Alberto ultrapasse Beatriz pela primeira vez?
A) 105 B) 630 C) 900 D) 1.050
E) não pode ser determinado

9. DEFG é um quadrado no exterior do pentágono regular ABCDE. Quanto


mede o ângulo EÂF?
A) 9o B) 12o C) 15o D) 18o E) 21o

10. Quantos são os números inteiros de 2 algarismos que são iguais ao


dobro
do produto de seus algarismos?
A) 0 B) 1 C) 2 D) 3 E) 4

11. Veja o problema 19 do nível 1.

EUREKA! N°10, 2001


7
Sociedade Brasileira de Matemática

12. Uma caixa contém 900 cartões, numerados de 100 a 999. Retiram-se
ao acaso (sem reposição) cartões da caixa e anotamos a soma dos seus
algarismos. Qual é a menor quantidade de cartões que devem ser retirados da
caixa, para garantirmos que pelo menos três destas somas sejam iguais?
A) 51 B) 52 C) 53 D) 54 E) 55

13. Se x e y são números reais positivos, qual dos números a seguir é o maior?
x3  y3
A) xy B) x2 + y2 C) (x + y)2 D) x2 + y(x + y) E)
x y
14. Na figura, as distâncias entre dois pontos horizontais consecutivos e as
distâncias entre dois pontos verticais consecutivos são iguais a 1. A região comum ao
triângulo e ao quadrado tem área:

9 15 8 1114
A) B) C) D) E)
10 16 9 1215
a a 1
15. Sejam a e b números reais positivos tais que < 1. Então
b b 1
a a a
A) é igual a + 1. B) é igual a . C) é menor que .
b b b
a
D) é maior que mas menor que 1. E) pode ser maior que 1.
b
16. Veja o problema 16 do nível 1.

17. Quantos são os retângulos que têm os pontos A e B como vértices, e


cujos vértices estão entre os pontos de interseção das 9 retas horizontais com as 9 retas
verticais da figura abaixo?

A) 3 B) 4 C) 7 D) 2 E) 5

EUREKA! N°10, 2001


8
Sociedade Brasileira de Matemática

18. Veja o problema 14 do nível 1.

19. De Itacimirim a Salvador, pela estrada do Coco, são 60 km. Às 11 horas,


a 15 km de Salvador, dá-se um acidente que provoca um engarrafamento, que
cresce à velocidade de 4 km/h, no sentido de Itacimirim. A que horas,
aproximadamente, devemos sair de Itacimirim para chegar a Salvador ao meio-
dia, sabendo que viajamos a 60 km/h, exceto na zona de engarrafamento, onde a
velocidade é 6 km/h?
A) 10h43min B) 10h17min C) 10h48min D) 10h53min E) 11h01min

20. Colocamos em ordem crescente os números escritos nas casas brancas


do tabuleiro a seguir (estamos mostrando apenas as suas quatro primeiras linhas).
Assim, por exemplo, o nono número da nossa lista é 14. Qual é o 2000 o número
da nossa lista?

1
2 3 4
5 6 7 8 9
10 11 12 13 14 15 16
… … … … … … … … …

A) 3931 B) 3933 C) 3935 D) 3937 E) 3939

GABARITO
NÍVEL 2

1) D 6) D 11) E 16) D
2) A 7) D 12) C 17) E
3) A 8) B 13) C 18) C
4) B 9) A 14) D 19) A

EUREKA! N°10, 2001


9
Sociedade Brasileira de Matemática

5) C 10) B 15) D 20) D


XXII OLIMPÍADA BRASILEIRA DE MATEMÁTICA
Problemas e soluções da primeira fase – nível 3

1. Veja o problema 13 do nível 2.


2. Veja o problema 9 do nível 2.
3. Veja o problema 14 do nível 2.

4. Escrevemos uma lista com todos os números inteiros de 1 a 30, inclusive.


Em seguida, eliminamos alguns destes números de forma que não sobrem dois
números tais que um seja o dobro do outro. Qual é a quantidade máxima de
inteiros que podem permanecer na lista?
A) 15 B) 18 C) 19 D) 20 E) 21

5. Veja o problema 15 do nível 2.

6. Seja f uma função real que tem as seguintes propriedades:


i) Para todos x, y reais, f(x + y) = x + f(y);
ii) f(0) = 2.
Quanto vale f(2000)?
A) 0 B) 2 C) 1998 D) 2000 E) 2002

7. Há três cartas viradas sobre uma mesa. Sabe-se que em cada uma delas está
escrito um número inteiro positivo. São dadas a Carlos, Samuel e Tomás as
seguintes informações:

i) todos os números escritos nas cartas são diferentes;


ii) a soma dos números é 13;
iii) os números estão em ordem crescente, da esquerda para a direita.

Primeiro, Carlos olha o número na carta da esquerda e diz: “Não tenho


informações suficientes para determinar os outros dois números.” Em seguida,
Tomás olha o número na carta da direita e diz: “Não tenho informações
suficientes para determinar os outros dois números.” Por fim, Samuel olha o
número na carta do meio e diz: “Não tenho informações suficientes para
determinar os outros dois números.” Sabendo que cada um deles sabe que os
outros dois são inteligentes e escuta os comentários dos outros, qual é o número
da carta do meio?

A) 2 B) 3 C) 4 D) 5

EUREKA! N°10, 2001


10
Sociedade Brasileira de Matemática

E) Não há informações suficientes para determinar o número.


8. Veja o problema 16 do nível 2.
9. Veja o problema 12 do nível 2.

10. A notação x significa o maior inteiro que não supera x. Por exemplo,
3,5 = 3 e 5 = 5. O número de inteiros positivos x para os quais
 x    x  10 é:
1 1
2 3

A) 11 B) 12 C) 13 D) 14 E) 15

11. Veja o problema 20 do nível 2.


12. Veja o problema 18 do nível 1.

13. A figura abaixo mostra o logotipo de uma empresa, formado por


dois círculos concêntricos e por quatro círculos de mesmo raio, cada um deles tangente a
dois dos outros e aos dois círculos concêntricos. O raio do círculo interno mede 1 cm.
Então o raio do círculo externo deverá medir, em cm:

A) 2 2 + 3 B) 2 +2 C) 4 2 + 1 D) 3 2 E) 2 +1

14. Veja o problema 8 do nível 2.


15. Veja o problema 10 do nível 2.

16. Dois nadadores, inicialmente em lados opostos de uma piscina, começam


simultaneamente a nadar um em direção ao outro. Um deles vai de um lado a
outro da piscina em 45 segundos e o outro em 30 segundos. Eles nadam de um
lado para outro por 12 minutos, sem perder qualquer tempo nas viradas. Quantas
vezes eles passam um pelo outro (indo no mesmo sentido ou em sentidos
opostos) durante este tempo, contando as vezes em que se encontram nos
extremos da piscina.
A) 10 B) 12 C) 15 D) 18 E) 20

17. A soma de dois números naturais é 29. O mínimo valor para a soma de seus
quadrados é:
A) 785 B) 733 C) 647 D) 421 E) 334

18. Veja o problema 1 do nível 2.

EUREKA! N°10, 2001


11
Sociedade Brasileira de Matemática

19. Veja o problema 17 do nível 2.


20. Veja o problema 10 do nível 1.

21. Na figura temos que os triângulos ABC e A’B’C’ são equiláteros e a região
destacada é um hexágono regular. A razão entre a área da região destacada e a
área do triângulo ABC é igual a:
A
C ’

B ’
O

C
A’
2 4 2 3
A) 1 B) C) D) E)
3 5 2 2
22. Veja o problema 14 do nível 1.
23. Veja o problema 19 do nível 2.

24. Seja P(x) = a2000x2000 + a1999x1999 + a1998x1998 + … + a1x + a0. Então a2000 + a1998 +
a1996 +
… + a0 é igual a
P (1)  P (  1) P (1)  P ( 1)
A) B) C) P(2000) + P(1998) +
2 2
… + P(0)
D) P(0) P(1) E) P(–1)  P(1)

25. Quantos números de três algarismos (que não começam com 0) possuem um
algarismo que é a média aritmética dos outros dois?
A) 121 B) 117 C) 112 D) 115 E) 105

GABARITO
NÍVEL 3

1) C 6) E 11) D 16) E 21) B


2) A 7) C 12) C 17) D 22) C

EUREKA! N°10, 2001


12
Sociedade Brasileira de Matemática

3) D 8) D 13) A 18) D 23) A


4) D 9) C 14) B 19) E 24) B
5) D 10) E 15) B 20) A 25) A
XXII OLIMPÍADA BRASILEIRA DE MATEMÁTICA
Problemas e soluções da segunda fase – nível 1

PROBLEMA 1:
De quantas maneiras diferentes podemos construir um paralelepípedo usando
exatamente 24 blocos cúbicos de medidas 1  1  1?
Obs: Blocos de dimensões 2  3  4 e 2  4  3 devem ser considerados iguais.

PROBLEMA 2:
O retângulo ao lado está dividido em 9
D
quadrados, A, B, C, D, E, F, G, H e I. O I

quadrado A tem lado 1 e o quadrado B tem


lado 9. G
C
Qual é o lado do quadrado I? F
H
A
B E

PROBLEMA 3:
Pintamos de vermelho ou azul 100 pontos em uma reta. Se dois pontos vizinhos
são vermelhos, pintamos o segmento que os une de vermelho. Se dois pontos
vizinhos são azuis, pintamos o segmento de azul. Finalmente, se dois pontos
vizinhos têm cores distintas, pintamos o segmento de verde. Feito isto, existem
exatamente 20 segmentos verdes. O ponto na ponta esquerda é vermelho.
É possível determinar com estes dados a cor do ponto na ponta direita?
Em caso afirmativo, qual a cor deste ponto?
PROBLEMA 4:
Desejamos escrever os inteiros de 1 a 10 nas casas do
desenho ao lado de tal forma que quaisquer quatro números
alinhados aparecem em ordem crescente ou decrescente.
a) Mostre uma maneira de dispor os números respeitando
estas condições.
b) Quais números podem aparecer nas pontas da estrela?
c) Quais números podem aparecer nas outras cinco posições?

PROBLEMA 5:

EUREKA! N°10, 2001


13
Sociedade Brasileira de Matemática

Qual é o menor inteiro positivo que é o dobro de um cubo e o quíntuplo de um


quadrado?

PROBLEMA 6:
Qual é o maior inteiro positivo n tal que os restos das divisões de 154, 238 e 334
por n são iguais?

SOLUÇÕES DA SEGUNDA FASE - NÍVEL 1

SOLUÇÃO DO PROBLEMA 1:
Sejam a b c as dimensões do paralelepípedo. Temos que a, b, c Ν * e
abc 24. Como abc a.a.a  a 3 24, temos a 2, ou seja
a 1 ou a 2 .
Se a 1, bc 24. As possibilidades para b e c são
b 1 e c 24; b 2 e c 12; b 3 e c 8; b 4 e c 6. Se
a 2, bc 12. As possibilidades para b e c com b 2 são b 2 e c 6;
b 3 e c 4. Assim, há 6 maneiras de construirmos o paralelepípedo.

SOLUÇÃO DO PROBLEMA 2:
O quadrado A medida de lado 1cm enquanto que o quadrado B tem medida de
lado 9cm. Daí que as longitudes dos lados dos quadrados restantes são:
C = 10cm E = 8cm.
F = 7cm G = 4cm.
D = 14cm. I = 18cm.

SOLUÇÃO DO PROBLEMA 3:
Temos que os segmentos verdes dividem os pontos da reta em conjuntos de
pontos com cores iguais, sendo que o primeiro conjunto à esquerda contém
pontos vermelhos, o segundo conjunto contém pontos azuis, o terceiro conjunto
contém pontos vermelhos, e assim por diante. Como há 20 segmentos verdes,
temos 21 conjuntos de pontos.
Assim, como o 21º conjunto contém pontos vermelhos, o ponto na ponta direita é
vermelho.

SOLUÇÃO DO PROBLEMA 4:

EUREKA! N°10, 2001


14
Sociedade Brasileira de Matemática

6 5

1 4 7 10 1 4 7 10

3 8 3 8
5 6

2 9 2 9

1) 1 e 2 ocupam pontas vizinhas. É fácil ver que colocando o 2 no meio ou em


uma ponta "oposta"a 1 o problema não tem solução.
2) 9 e 10 ocupam pontas vizinhas. Pelo mesmo raciocinio anterior.
3) Uma vez que 1 e 2 estão colocados o 3 está no meio, entre o 1 e o 2. Observe
que colocar o 3 em qualquer outra posição leva a um absurdo.
4) Uma vez que 1, 2 e 3 estão colocados, fica claro que o 4 é vizinho ao 3.
5) Se 1, 2, 3 e 4 já estão colocados, 5 pode estar no meio ou em uma ponta, e o
mesmo ocorre com o 6. (ver figuras) Quando um deles está numa ponta, o outro
está no meio.
6) O 7 está no meio.

Respostas:
a) Ver figuras
b) 1, 2, 9 e 10 obrigatórios mais 5 ou 6.
c) 3, 4, 7, 8 obrigatórios mais 5 ou 6.

SOLUÇÃO DO PROBLEMA 5:
Decomponha N em primos = 2 a2 3 a3 ...
Dobro de um cubo quer dizer que todos os ai são múltiplos de 3 exceto a2 que
deixa resto 1 na divisão por 3.
Quíntuplo de um quadrado quer dizer que todos são pares exceto a5.
Os menores expoentes possíveis são então a2 = 4; a5 = 3 e os outros a3 = a7 =...=
0.
Resposta: N = 24 53 = 2000.

SOLUÇÃO DO PROBLEMA 6:
Dois números deixam o mesmo resto quando divididos por n se e só se sua
diferença é múltipla de n. Logo, as diferenças 238 – 154 = 84 e 334 – 238 = 96
são ambas múltiplas de n. Como n é o maior possível, concluímos que n deve ser
o maior divisor comum de 84 e 96, que é 12.

EUREKA! N°10, 2001


15
Sociedade Brasileira de Matemática

XXII OLIMPÍADA BRASILEIRA DE MATEMÁTICA


Problemas e soluções da segunda fase – nível 2
PROBLEMA 1:
Qual é o menor inteiro positivo que é o dobro de um cubo e o quíntuplo de um
quadrado?

PROBLEMA 2:
De quantas maneiras diferentes podemos construir um paralelepípedo usando
exatamente 216 blocos cúbicos de medidas 1  1  1? Obs: Blocos de dimensões
2  3  36 e 2  36  3 devem ser considerados iguais.

PROBLEMA 3: D
F
C
No retângulo ABCD, E é o ponto médio do lado
BC e F é o ponto médio do lado CD. A interseção G
E

de DE com FB é G. O ângulo EAF mede 20o.

Quanto vale o ângulo EGB ?
A B

PROBLEMA 4:
O retângulo ao lado está dividido em 9
D
quadrados, A, B, C, D, E, F, G, H e I. O quadrado I

A tem lado 1.
G
Qual é o lado do quadrado I? C
F
H
A
B E

PROBLEMA 5:

EUREKA! N°10, 2001


16
Sociedade Brasileira de Matemática

Listamos os inteiros de 1 a n. Desta lista apagamos o inteiro m. A média dos n – 1


134
números restantes é . Determine n e m.
11

PROBLEMA 6:
O campeonato Venusiano de futebol é disputado por 10 times, em dois turnos. Em
cada turno cada equipe joga uma vez contra cada uma das outras. Suponha que o
Vulcano FC vença todas as partidas do 1 o. turno. Caso não vença o 2 o. turno, o
Vulcano FC jogará uma final contra o vencedor do 2 o. turno, na qual terá
vantagem caso faça mais pontos que o adversário durante todo o campeonato
(vitória vale 3 pontos, empate vale 1 ponto e derrota 0 pontos).
a) Determine o menor n tal que, se o Vulcano FC fizer exatamente n pontos no
segundo turno, garantirá pelo menos a vantagem na final (independente de contra
quem e com que placares conquiste os n pontos).
b) Determine o menor n tal que, se o Vulcano FC fizer pelo menos n pontos no
segundo turno, garantirá pelo menos a vantagem na final (independente de contra
quem e com que placares conquiste os n pontos).

SOLUÇÕES DA SEGUNDA FASE - NÍVEL 2

SOLUÇÃO DO PROBLEMA 1: Veja a solução do problema 5 do nível 1.


SOLUÇÃO DO PROBLEMA 2:
Sejam a b c as medidas do paralelepípedo. Temos então que a, b e c são
inteiros positivos e abc 216 . Como a b c a a a  a 6 e a | 216,
temos
a 1, a 2, a 3, a 4 ou a 6. Se a 1, temos b c 216. As
possibilidades neste caso são b 1 e c 216; b 2 e c 108; b = 3 e
c = 72; b = 4 e c = 54;
b 6 e c 36; b 8 e c 27; b 9 e c 24; b 12 e c 18. Se
a 2, temos b c 108, com b 2. Temos então as possibilidades
b 2 e c 54; b 3 e c 36;
b 4 e c 27; b 6 e c 18; b 9 e c 12.
Se a 3, temos b c 72, com b 3. Temos então as possibilidades
b 3 e c 24; b 4 e c 18; b 6 e c 12; b 8 e c 9. Se a 6,
a única solução é b c 6. Temos, assim, 19 maneiras de construirmos o
paralelepípedo.

EUREKA! N°10, 2001


17
Sociedade Brasileira de Matemática

Observação: pode-se verificar que o número de soluções de b.c  r, com


 d  n 
b c naturais, é   , onde  x  denota o menor número inteiro maior ou
 2 
igual a x e d  n  é o número de divisores de n. Assim, b c 216 tem
 d  216   d 108 
 2  8 soluções; b c 108 com b 2 tem  2   1 5
   
soluções (descontamos aqui a solução b  1 e c 108 ); b c 72 com
 d  72  
b 3 tem  2   2 4 soluções (eliminamos
 
b 5 e c 72 e b 2 e c 36 ); b c 54 com b 4 tem
 d  54  
 2   3 1 solução (eliminamos b 1, b 2 e b 3 ) e b c 36
 
 d  36  
com b 6 tem    4 1 solução (elimina-se b = 1, 2 ,3 ou 4).
 2 

SOLUÇÃO DO PROBLEMA 3:
D y
F C
^ ^
G

1) F A D F B C x
E

x x
^ ^
2) E A B  E D C  y
o
20
y
A B
 x  y 70
^
D E C 90  y   x
90  ( x  y )    20
SOLUÇÃO DO PROBLEMA 4:
Seja x o lado de B. O lado de C = x – 1, D = x + 5, E = x – 1, F = x – 2,
G = 4, H = 2x – 3, I = x + 9 (=D + G) mas também é 3x – 9 (=F + H – G). Assim
x + 9 = 3x – 9 e x = 9. Assim, o lado de I é 18.

SOLUÇÃO DO PROBLEMA 5:
A média aritmética dos inteiros de 1 a n é (n + 1)/2. Quando se apaga um destes
números, a menor média possível é a dos números de 1 a (n – 1), que é n / 2, e a
maior é a dos números de 2 a n, que é n/2 +1. Logo, deve-se ter

EUREKA! N°10, 2001


18
Sociedade Brasileira de Matemática

n 2 n 4 4
 12   1 o que fornece 22 n 24 e, portanto, n é igual a 23 ou
2 11 2 11 11
24. Mas a média dos números restantes é uma fração de denominador 11. Logo,
a quantidade de números que restam no quadro deve ser múltipla de 11. Portanto,
n só pode ser igual a 23. Finalmente, a soma dos números que restam é 22 x 12
2/11 = 268.A soma dos números de 1 a 23 é 23 x 12 = 276. Logo, o número
apagado foi m = 276 – 268 = 8.

SOLUÇÃO DO PROBLEMA 6:
No pior caso, o 2o. colocado do 1o. turno faz 24 pontos no 1 o. turno. Se o Vulcano
FC fizer 23 pontos no 2 o. turno, ele ganhará 7 jogos e empatará 2, e o 2 o.
colocado no 1o. turno chegará a um máximo de 25 pontos (pois no máximo
empatará com o Vulcano FC) no segundo turno. Assim, o Vulcano FC terá
vantagem na decisão, nesse caso. Note que se o Vulcano FC fizer 24 pontos no 2o.
turno perdendo para o 2o. colocado do 1o. turno, este pode fazer 27 pontos no 2 o.
turno e ganhar a vantagem para a decisão. Se o Vulcano FC fizer 22 pontos ou
menos e o Klingon FC tiver feito 24 pontos no 1o. turno poderá fazer 27 pontos
no 2o. turno, somando 51 pontos, mais que os 49 (ou menos) pontos do Vulcano
FC. Assim, a resposta da segunda pergunta é n = 25, enquanto a resposta da 1a.
pergunta é n = 23.

XXII OLIMPÍADA BRASILEIRA DE MATEMÁTICA


Problemas e soluções da segunda fase – nível 3

PROBLEMA 1: Veja o problema 1 do nível 2.


PROBLEMA 2: Veja o problema 4 do nível 2.

PROBLEMA 3:
O trapézio ABCD tem bases AB e CD. O lado DA mede x e o lado BC mede 2x. A
  
soma dos ângulos DAB e ABC é 120o. Determine o ângulo DAB .

PROBLEMA 4: Veja o problema 6 do nível 2.

PROBLEMA 5:
1 1 1 1 1 1
O número 1 2
 2  1  2  2  ...  1  2
 é racional;
1 2 2 3 2000 20012
p
escreva-o na forma q , p e q inteiros.

EUREKA! N°10, 2001


19
Sociedade Brasileira de Matemática

PROBLEMA 6:
Para efetuar um sorteio entre os n alunos de uma escola (n > 1) se adota o
seguinte procedimento. Os alunos são colocados em roda e inicia-se uma
contagem da forma "um, DOIS, um, DOIS,...". Cada vez que se diz DOIS o aluno
correspondente é eliminado e sai da roda. A contagem prossegue até que sobre
um único aluno, que é o escolhido.
a) Para que valores de n o aluno escolhido é aquele por quem começou o
sorteio?
b) Se há 192 alunos na roda inicial, qual é a posição na roda do aluno
escolhido?

SOLUÇÕES DA SEGUNDA FASE - NÍVEL 3

SOLUÇÃO DO PROBLEMA 1: Veja a solução do problema 1 do nível 2.


SOLUÇÃO DO PROBLEMA 2: Veja a solução do problema 4 do nível 2.

SOLUÇÃO DO PROBLEMA 3:
Tracemos DM // BC (vide figura abaixo). Como AMD = ABC e DAM
+ AMD = DAM + ABC = 120 tem-se que ADM = 60. Como
AD = x e BC = 2x, sendo P o ponto médio de DM, então, AD = DP =
x e ADP é um triângulo equilátero, isto é, AP = x. Portanto APM é
um triângulo isósceles com PAM = AMP e como DPA é um
ângulo externo do triângulo APM temos 60 = DPA = PAM +
AMP = 2. AMP = 2. ABC. Portanto, ABC =30 e DAB
=120 – ABC = 90.
D C

A M B

SOLUÇÃO DO PROBLEMA 4: Veja a solução do problema 6 do nível 2.

SOLUÇÃO DO PROBLEMA 5:

EUREKA! N°10, 2001


20
Sociedade Brasileira de Matemática

2000
1 1 2000
a 4  2a 3  3a 2  2a  1
S  1    
a 1 a 2  a  1 2 a 1 a 2  a  1
2

2000
a 2  a  1 2000  1 
 2
 1  2 
a 1 a a a 1  a a
2000
1 1   1  1 1  1 1 
2000      2000  1        ...    
a 1  a a 1  2  2 3  2000 2001 
1 2000
2000  1  2000 
2001 2001

SOLUÇÃO DO PROBLEMA 6:
a) Para que o primeiro da fila seja o escolhido é preciso, antes de mais nada, que
haja um número par de alunos (caso contrário, ele será eliminado quando
começar a segunda rodada). Mais precisamente, o primeiro da fila é o escolhido
se e só se, a cada rodada, a fila tem um número par de alunos. Portanto, o
primeiro da fila é escolhido se e só se o número de alunos é uma potência de 2.

b) Como 192 = 26 . 3, nas primeiras 6 rodadas a fila tem um número par de


alunos. Após estas 6 rodadas, a fila se reduz a três alunos e é fácil verificar que o
escolhido é o terceiro deles. Resta, portanto, determinar quem são os alunos que
restam após as primeiras 6 rodadas. Na primeira rodada, sobrevivem 1, 3, 5, 7, ...,
191. De um modo geral, sobrevivem à rodada de ordem n (n = 1, 2, ..., 6) os
números da forma 2n . k + 1. Portanto, após 6 rodadas os sobreviventes são 1, 65
e 129 e o aluno escolhido é o de número 129.
XXII OLIMPÍADA BRASILEIRA DE MATEMÁTICA
Problemas e soluções da terceira fase – nível 1

PROBLEMA 1:
Paulo tem três dados comuns idênticos nos quais a soma dos números em duas faces
opostas é sempre igual a 7. Ele cola os dados, de modo que cada par de faces coladas
tenha o mesmo número, e depois os coloca sobre uma mesa não transparente, conforme
indica a figura. A soma dos números em todas as onze faces visíveis é 36. Qual é a soma
dos números das três faces que estão em contato com a mesa?

PROBLEMA 2:
Um triângulo equilátero pode ser recortado em triângulos equiláteros menores. A
figura abaixo mostra como recortar um triângulo equilátero em 7 triângulos

EUREKA! N°10, 2001


21
Sociedade Brasileira de Matemática

equiláteros. Mostre como recortar um triângulo equilátero em 20 triângulos


equiláteros menores.

PROBLEMA 3:
Isabel tem dois baralhos, cada um com 50 cartas. Em cada um dos baralhos estão
escritos os números de 1 a 100 (em cada carta estão escritos dois números, um em
cada face da carta). Por um defeito de fabricação, a distribuição dos números nas
cartas não é a mesma nos dois baralhos (por exemplo, em um dos baralhos o 1
aparece na mesma carta do 2; no outro, o 1 aparece com o 76).
Mostre como Isabel deve fazer para que, ao colocar as 100 cartas sobre uma
mesa, as faces voltadas para cima mostrem todos os números de 1 a 100.

PROBLEMA 4:
Considere a seguinte tabela 5  5, preenchida com os números de 1 a 25.

1 2 3 4 5
6 7 8 9 10
11 12 13 14 15
16 17 18 19 20
21 22 23 24 25

Em cada fileira horizontal e em cada fileira vertical, trocamos o sinal de 2


números, de forma que, em cada fileira horizontal e em cada fileira vertical, haja
3 números positivos e 2 números negativos. Somamos, então, todos os números
da tabela. Calcule os possíveis valores dessa soma.

SOLUÇÕES DA TERCEIRA FASE – NÍVEL 1

PROBLEMA 1: Veja a solução do problema 1 do nível 2.

PROBLEMA 2: SOLUÇÃO DE BÁRBARA ÁRABE SARAIVA (SANTOS - SP)

Esta é uma forma de dividir um triângulo equilátero


em 20 triângulos equiláteros menores.
Dividindo o maior triângulo em 4 partes (triângulos
equiláteros menores), dois de esses quatro menores
triângulos em nove outros menores triângulos
equiláteros, obteremos 20 triângulos equiláteros:

EUREKA! N°10, 2001


22
Sociedade Brasileira de Matemática

2 maiores e 18 menores.

PROBLEMA 3: Veja a solução do problema 2 do nível 2.

PROBLEMA 4: SOLUÇÃO DA BANCA


Escrevemos os números da tabela na seguinte forma:

0+1 0+2 0+3 0+4 0+5


5+1 5+2 5+3 5+4 5+5
10 + 1 10 + 2 10 + 3 10 + 4 10 + 5
15 + 1 15 + 2 15 + 3 15 + 4 15 + 5
20 + 1 20 + 2 20 + 3 20 + 4 20 + 5

Cada número é da forma 5a  b, com 0 a 4 e 1 b 5.


Depois de trocar de sinal temos que em cada linha há dois números negativos, se
em cada linha fazemos a soma só das partes 5a temos que a soma dessa linha
é 5a (já que há três 5a e dois  5a ) e a soma de todas as linhas considerando
somente os números 0 +5 +10 + 15 + 20 = 50.
Agora consideremos os números b . Em cada coluna há dois números b que
trocaram de sinal e três que não, portanto a soma dos números dessa coluna é b e
a soma das colunas considerando somente as partes b , é: 1 + 2 + 3 + 4 + 5 =
15. Logo a soma total é sempre 65.
XXII OLIMPÍADA BRASILEIRA DE MATEMÁTICA
Problemas e soluções da terceira fase – nível 2

PROBLEMA 1: Veja o problema 1 do nível 1.

PROBLEMA 2:
Isabel tem dois baralhos, cada um com 50 cartas. Em cada um dos baralhos estão
escritos os números de 1 a 100 (em cada carta estão escritos dois números, um em
cada face da carta). Por um defeito de fabricação, a distribuição dos números nas
cartas não é a mesma nos dois baralhos (por exemplo, em um dos baralhos o 1
aparece na mesma carta do 2; no outro, o 1 aparece com o 76).
Mostre como Isabel deve fazer para que, ao colocar as 100 cartas sobre uma
mesa, as faces voltadas para cima mostrem todos os números de 1 a 100.

PROBLEMA 3:
Em uma folha de papel a reta r passa pelo canto A da folha e forma um ângulo 
com a borda horizontal, como na figura 1. Para dividir este ângulo  em três
partes iguais, executaremos as seguintes construções:

EUREKA! N°10, 2001


23
Sociedade Brasileira de Matemática

a) inicialmente, marcamos dois pontos B e C sobre a borda vertical de modo que


AB = BC; pelo ponto B traçamos a reta s paralela à borda (figura 2);
b) a seguir, dobramos o papel, ajustando-o de modo que o ponto C coincida com
um ponto C’ sobre a reta r e o ponto A coincida com um ponto A’ sobre a reta s
(figura 3); chamamos de B’ o ponto com o qual B coincide.
Mostre que as retas AA’ e AB’ dividem o ângulo  em três partes iguais.

r r r

C’

C C B’
s A’
B B
  
A A A

Figura 1 Figura 2 Figura 3

PROBLEMA 4:
É possível encontrar duas potências de 2, distintas e com o mesmo número de
algarismos, tais que uma possa ser obtida através de uma reordenação dos dígitos
da outra?
SOLUÇÕES DA TERCEIRA FASE – NÍVEL 2
PROBLEMA 1: SOLUÇÃO DE ALEX CORRÊA ABREU (NITERÓI - RJ)
Sejam a e b os números das faces coladas. Como a é o número da face oposta à
face de b, no dado central, temos que a + b = 7.
A soma dos números das faces de cada dado é 21, então a soma dos números das
faces de todos os dados é 63, mas a soma das faces coladas é 2 (a + b) = 14, e a
soma das faces visíveis é 36, temos, então, que a soma de números das faces em
contato com a mesa é: 63 – 36 – 14 = 13. Resposta: a soma é 13.

PROBLEMA 2: SOLUÇÃO DE VITOR GABRIEL KLEINE (MOGI DAS CRUZES - SP )


Podemos fazer isso pegando qualquer carta de qualquer baralho, colocando sobre a mesa
e vendo seu verso. Depois disso procuramos a carta de mesmo número do verso
(procurando no outro baralho, já que foi usada no primeiro baralho). Fazemos com esta
carta o mesmo que foi feito com a primeira carta. Continua-se a fazer isso até fechar um
ciclo (um mesmo número que já saiu em um baralho sair no outro). Exemplo:

EUREKA! N°10, 2001


24
Sociedade Brasileira de Matemática

baralho: 1 2 1 2

x y z w Ciclo fechado.

verso: y z w x

Quando um ciclo for fechado pega-se outra carta e começa um novo ciclo.
Fazendo isso até o final das cartas as faces voltadas para cima mostrarão todos os
números de 1 a 100.

PROBLEMA 3: Veja solução do problema 1 do nível 3.

PROBLEMA 4: SOLUÇÃO DE SAMUEL BARBOSA FEITOSA (FORTALEZA - CE)


Sejam A 2 m e A' 2 n onde A' é uma reordenação dos dígitos de A,
suponha sem perda de generalidade que A  A' , daí A é um múltiplo de A' pois
possui os mesmos fatores primos. Então temos que A  A'k , onde k  Z ,
k  1 (pois caso não fosse  A'  A , o que seria um absurdo, pois A e A' são
distintos.) e k é uma potência de dois, pois A só possui fatores primos iguais a 2,
daí k 2,4,8  A 2 A' ou A 4 A' ou A 8A'.
Como a soma de seus dígitos é a mesma, A e A' deixam o mesmo resto (mod 9) e
sua diferença é divisível por 9, mas A – A' só pode se: A', 3A', 7A', onde nenhuma
desas diferenças é divisível por 9. Daí não existem tais números. Nenhum número
é divisível por 9, pois cada um desses números não possui pelo menos dois
fatores primos iguais a 3.
XXII OLIMPÍADA BRASILEIRA DE MATEMÁTICA
Problemas e soluções da terceira fase – nível 3

PROBLEMA 1: Veja o problema 3 do nível 2.

PROBLEMA 2:
Seja  (n) a soma de todos os divisores positivos de n, onde n é um inteiro
positivo (por exemplo,  (6) 12 e  (11) 12). Dizemos que n é quase
perfeito se  ( n) 2n  1 (por exemplo, 4 é quase perfeito, pois  ( 4) 7).
n
Sejam n mod k o resto da divisão de n por k e s (n)  n mod k (por
k 1
exemplo: s(6) = 0 + 0 + 0 + 2 + 1 + 0 = 3 e s(11) = 0 + 1 + 2 + 3 + 1 + 5 + 4 + 3 +
2 + 1 + 0 = 22).
Prove que n é quase perfeito se, e somente se, s (n) s (n  1).

PROBLEMA 3:

EUREKA! N°10, 2001


25
Sociedade Brasileira de Matemática

Seja f uma função definida nos inteiros positivos da seguinte forma:


Dado n, escrevemos n 2 a ( 2b  1), com a e b inteiros e definimos
f (n) a 2  a  1.
Determine o menor inteiro positivo n tal que f (1)  f ( 2)  ...  f ( n) 123456.

PROBLEMA 4:
A avenida Providência tem infinitos semáforos igualmente espaçados e
sincronizados.
A distância entre dois semáforos consecutivos é de 1.500m. Os semáforos ficam
abertos por 1 min 30s, depois fechados por 1 min, depois abertos por 1 min 30s e
assim sucessivamente.
Suponha que um carro trafegue com velocidade constante igual a v, em m/s, pela
avenida Providência.
Para quais valores de v é possível que o carro passe por uma quantidade
arbitrariamente grande de semáforos sem parar em qualquer um deles?

PROBLEMA 5:
Seja X o conjunto de todas as seqüências a ( a1 , a 2 ,..., a 2000 ) tais que
a i {0,1,2} se 1 i 1000 e a i  {0,1} se 1001 i 2000. Dados a e b
em X, definimos a distância d ( a, b) entre a e b como sendo o número de
valores de i, 1 i 2000, tais que a i bi . Determine o número de funções
f : X  X que preservam distância, isto é, tais que d ( f ( a ), f (b)) d ( a, b),
para quaisquer a e b em X.

PROBLEMA 6:
Seja C um cubo de madeira. Para cada um dos 28 pares de vértices de C cortamos
o cubo C pelo plano mediador dos dois vértices do par. Em quantos pedaços fica
dividido o cubo?

Nota: Dados dois pontos A e B no espaço, o plano mediador de A e B é o


conjunto dos pontos do espaço cujas distâncias a A e B são iguais. Em outras
palavras: é o plano perpendicular ao segmento AB passando pelo ponto médio de
AB.

SOLUÇÕES DA TERCEIRA FASE – NÍVEL 3

PROBLEMA 1: SOLUÇÃO DE MARTHA P. ARAÚJO DE MORAES (FORTALEZA - CE)

EUREKA! N°10, 2001


26
Sociedade Brasileira de Matemática

C’

B’

B A’
X

A P K
ˆ A'  então
Veja que AP  A' P , então AA' P é isósceles. Seja PA
ˆ ' B  ( BA' // AP ).
AA
Note que APX A' PX ,
Daí:
XAˆ P  XA ˆ' P
ˆ
  XAA' 2
XA ˆ A' 
Agora observe que:
BA ˆ X 90  2  BXˆA 2
ˆ
BAX 90  2  B ' XˆA'  2
Donde segue que os pontos A, X, B' são colineares. Como
CB C ' B ' , AB  A' B ' e CB  AB, temos que C ' B '  A' B ' .
Então AB' é mediana e altura do C ' AA' , sendo, conseqüentemente, bissetriz
do C ' AA'. Daí: C ' A ˆ B ' B ' A
ˆ A' PAˆ A'  .

PROBLEMA 2: SOLUÇÃO DE FABRÍCIO SIQUEIRA BENEVIDES (FORTALEZA - CE)

Fixe n. Seja a i n mod i e bi (n  1) mod i


n n
Temos s (n)  a i e s ( n  1)  bi
i 1 i 1
Veja que se ,
d n por definição, a d 0, e que
n 0(mod d )  n  1  1(mod d )  bd d  1
(já que 0 bd d  1) (inclusive se d = 1)
Além disso se t \ n, at  0 e é fácil ver que b t a t  1.

EUREKA! N°10, 2001


27
Sociedade Brasileira de Matemática

Sendo assim:
n n 1
s(n) s(n  1)   ai  bi   ad   at  an  bd   bt   (bd  ad ) (a  b )   (d  1) 1
t t
t | n d |n t | n
i 1 i 1 d |n t \ n d |n t \ n d|n d n
d n d n d n
Seja f(n) o número de divisores de n. Temos:
(d 1)1d 11((n)n  nf )(() 1)n nf )( (n) n (nf )1 n  f (n)   (n) 2n  1.
d|n t|n d|n d|n t|n
dn dn dn

De modo que s ( n) s (n  1)   (n) 2n  1.

PROBLEMA 3: SOLUÇÃO DE ULISSES MEDEIROS DE ALBUQUERQUE (FORTALEZA - CE)

Considere as representações binárias dos números, ex: 17 = (10001); 24 =


(11000) e 5 = (101)
Seja n na base 2 igual a (...a i ...a 3 a 2 a1 a 0 ), onde a i 0 ou a i 1, i  Z 
j
se 2  n  a j 0.
n 2 a ( 2b  1)  a é a quantidade de zeros à direita na sua representação
binária. Exemplo:
a para o 24 é 3, já a = 0 para o 17 e o 5. Isto vem exatamente do que significa a
representação de um número em uma dada base. (*)
Seja S k  f (1)  f (2)  f (3)  ...  f (2 k )
Como a só depende da quantidade de zeros no final (*), temos que se 2 j  n,
n 1 então f (2 j  n)  f ( n), pois terão a mesma quantidade de zeros à direita
na base 2.
Assim,
S k  f (1)  f (2)  ... f (2 k  1  1)  f (2 k  1 )  f (2 k  1  1)  f (2 k  1  2)  ...  f (2 k )
S k ( f (1)  f (2)  ...  f (2 k  1  1)  f (2 k  1 ))  ( f (1)  f (2)  ...  f (2 k  1  1))  f (2 k )
S k ( S k  1 )  ( S k  1  f ( 2 k  1 ))  f (2 k )

S k S k  1  S k  1   (k  1) 2  (k  1)  1  k 2  k  1   
S k 2 S k  1  2 k
S k 2 ( S k  1  k ).
Primeiros Sk's:
S0 = 1, S1 = 4, S2 = 12, S3 = 30, S4 = 68, S5 = 146, S6 = 304, S7 = 622, S8 = 1260, S9 =
2538,
S10 = 5096, S11 = 10214, S12 = 20452, S13 = 40930, S14 = 81888, S15 = 163806

EUREKA! N°10, 2001


28
Sociedade Brasileira de Matemática


Seja g (n)  f (1)  f (2)  ...  f (n), provaremos que g (n)  a i S i , onde
i 1
n (...a j ...a 5 a 4 a 3 a 2 a1 a 0 ).
j j 1 0
Seja j o maior possível tal que a j 1. n 2  a j  1 2  ...  a1 2  a 0 2
g (n) ( f (1)  f (2)  ...  f (2 j ))  ( f (2 j  1)  f (2 j  2)  ...  f (2 j  a j  1 2 j  1  ...a 0 ))
g (n) ( S j )  f (1)  f (2)  ...  f ( a j  1 2 j  1  ...  a 0 ))
De modo análogo, tomamos o maior j 0 , tal que j  j 0 e a j0 1.
g (n) S j  ( f (1)  f (2)  f (3)  ...  f (2 j0 ))  ( f (2 j0  1)  ...  f (2 j0  a j0  1 2 j0  1  ...  a0 ))
g (n) S j  ( S j0 )  ( f (1)  f (2)  ...  f (a j0  1 2 j0  1  ...  a 0 ))
De maneira análoga, fazemos (vamos baixando) para todos os ai's = 1.

Como a i 1 ou a i 0, podemos escrever g (n)  a i S i
i 1

Para termos o menor n, tal que g ( n) 123456


Temos que conseguir uma soma de S k 's 123456, com os menores k's
possíveis, pois isto se refletirá em (...a i ...a 2 a1 a 0 ) com os menores i's
possíveis. Mas isto é uma tarefa fácil se tomarmos os Sk's calculados na página
anterior e também sabendo que:
S k  2 S k  1  S k  1  2 S k  2 ...  S k  1  S k  2  S k  3  ...  S 0
Daí, temos que a soma procurada é:
S14  S13  S 7  S 2  S1 81888  40930  622  12  4 1234456
Assim, o menor n tal que g ( n) 123456 é (110000010000110)2
n 214  213  2 7  2 2  21 16384  8192  128  4  2
n = 24710
O menor inteiro positivo tal que f (1)  ...  f ( n) 123456 é 24710.

PROBLEMA 4: SOLUÇÃO DA BANCA


Suponha que no tempo 0 os sinais se abram e que o carro passe pela primeira vez
por um sinal no tempo t 0 0 (mediremos o tempo sempre em segundos). Os
sinais estarão abertos entre os tempos 150k e 150k + 90 e fechados entre os
tempos 150k + 90 e 150(k + 1), para todo inteiro k. O carro passará pelos sinais
1500
nos tempos t 0  r , para todo inteiro não negativo r . Assim, a condição
v
necessária e suficiente para que o carro encontre sempre o sinal aberto é que

EUREKA! N°10, 2001


29
Sociedade Brasileira de Matemática

t0 10 3
 r seja igual a um inteiro mais um número entre 0 e para todo r
150 v 5
10
inteiro. Isso é claramente possível se é inteiro (com qualquer t 0 entre 0 e
v
10
90) e se é a metade de um inteiro ímpar (com qualquer t 0 entre 0 e 15).
v
Vamos mostrar que esses são os únicos casos possíveis.
10r
Primeiro mostraremos que é igual a um inteiro mais um número
v
 2 10
pertencente a  0,  para algum r0: seja  j   , com j inteiro e    0,1).
 5  v
2
Se 0    , tomamos r0 1.
5
1 3 1
Se    ,2 1   , com 0    , e tomamos r0 2.
2 5 5
3
Se    1, tomamos  1   e k inteiro tal que k  1  (k  1)  . Como
5
2 3
  , temos k  , e podemos tomar r0 k .
5 5
2 1 3 4
Se   , temos  2  1, e podemos proceder como no caso
5 2 5 5
anterior, tomando  1  2 e r0 2k .
Para finalizar, vamos mostrar que, nesses casos, existe k inteiro positivo tal que
t0 10 3 
 k é igual a um inteiro mais um elemento de  ,1.
150 v 5 
10 2
De fato, existe m inteiro tal que r0 m   , com 0    , e existem  e j
v 5
t0 t
inteiros com    j  0  (  1)  , donde
150 150
t 0 10 t0 3
 r0 lm    (m  j  1)  , onde  1      1.
150 v 150 5
20
Assim as possíveis velocidades são v  m / s, para cada inteiro positivo k.
k

PROBLEMA 5: SOLUÇÃO DE HUMBERTO SILVA NAVES (SÃO PAULO - SP)

EUREKA! N°10, 2001


30
Sociedade Brasileira de Matemática

Vamos observar um caso particular primeiro:


Sabemos que:
d ( f (0,0,0,...,0), f (1,0,0,...,0)) 1
e d ( f (1,0,0,...,0), f ( 2,0,0,0,...,0)) 1
e d ( f ( 2,0,0,...,0), f (0,0,0,...,0)) 1
Seja A  f (0,0,...,0), B  f (1,0,0,...,0)
e C  f ( 2,0,0,...,0)
A ( a1 , a 2 ,..., a 2000 ) e B (b1 , b2 ,..., b2000 ) e C (c1 , c 2 ,..., c 2000 )
Deve existir um único i1  N e 1 i1 2000, tal que:
a i1 bi1 , vamos provar que i1 1000.
Deve existir um único i1 ' tal que bi1 ' ci1 ' e se fosse i1 i1 ' teríamos que
d ( A, C ) 2, um absurdo, logo i1 i1 '
Logo temos:
A (..., a i1 ,...)
B (..., bi1 ,...) e como ai1 bi1 c i1 ai1 logo i1 1000.
Vamos provar que se:
x 0  f (0, a 2 ' , a 3 ' ,..., a 2000 ' ) ( x1 , x 2 ,..., x 2000 ) então x i1 a i1 .
Suponhamos por absurdo que xi1 a i1 (por simetria, consideramos x i1 bi1 )
Se d ( A; x 0 ) m, então d ( B; x 0 ) m  1, pois B  f (1,0,0,0,0,...,0) e
A  f (0,0,0,...,0) x 0  f (0, a 2 ' , a 3 ' ,..., a 2000 ' ) mas d ( B, x 0 ) m  1
(pois x i1 bi1 ) que é um absurdo, logo xi1 a i1
Analogamente verificamos que se
x1  f (1, b2 ' , b3 ' ,..., b2000 ' ) ( y1 , y 2 ,..., y 2000 ) então y i1 bi1
Vamos generalizar o argumento (nós só fizemos para o 1 o. termo):

Teorema 1: Seja
At  f (0,0,...,0,0,...) ( a1 , a 2 ,..., a 2000 )
Bt  f (0,0,...,1,0,...) (b1 , b2 ,..., b2000 )
C t  f (0,0,...,2,0,...,0) (c1 , c 2 ,..., c 2000 )
onde t 1000 : e alteramos apenas o t-ésimo termo no domínio.
Então se x '  f ( x1 ,..., x 2000 ) ( y1 , y 2 ,..., y 2000 )
y it a it se x t 0 (onde it é posição que muda de At para Bt)

EUREKA! N°10, 2001


31
Sociedade Brasileira de Matemática

y it bit se xt 1
y it cit se xt 2

Demonstração: Análoga à anterior (basta trocar algumas variáveis e copiar a


demonstração acima).
É claro que i1 , i 2 ,..., i1000 são todos distintos. Na verdade (i1 ,..., i1000 ) é uma
permutação de (1,2,...,1000).
Consideramos agora as seguintes 2000-uplas.

A j  f (0,0,...,0) ( a1 , a 2 ,..., a 2000 )


B j  f (0,0,...,1,0,0,0,...,0) (b1 ,..., b2000 ) onde j  1000 e colocamos o 1
na
j-ésima posição.

Sabemos que d ( A j , B j ) 1  t  N tal que:


a t bt e esse t é único!
É claro que t  1000 (pois se fosse t < 1000, existiria
w 1000 tal que i w t , um absurdo, pois o valor de posição wi da imagem
é determinado exclusivamente pelo valor da posição w da 2000-upla do domínio
da função t (devido ao teorema 1).

Vamos chamar esse t de i j , assim como fizemos anteriormente.


Seja x '  f ( x1 , x j ,..., x 2000 ) ( y1 ,..., y j ,..., y 2000 )
De forma semelhante à anterior, mostramos que:
y i j a i j se x j 0
y i j bi j se x j 1

Para contar o número de funções f : X  X , basta contar o número de


permutações de {1,2,...,1000} vezes o número de permutações de
{1001,...,2000} (3!)1000 ( 2!)1000 que é 1000! 1000! 121000 pois
para determinarmos uma função f : X  X basta escolher:
(i1 ,..., i 2000 ) que é uma permutação de (1,2,...,1000) e (i1001 ,..., i 2000 )
que é uma permutação de (1001,..., 2000) e escolher os valores apropriados

EUREKA! N°10, 2001


32
Sociedade Brasileira de Matemática

de ( a it , bit , c it ), para 1 t 1000 (1000 permutações de {0,1,2}) e de


(a it , bit ), para 1001 t 2000 (1000 permutações de {0, 1}).

PROBLEMA 6: SOLUÇÃO DE CHRISTIAN WATANABE (ITAGUAÍ - RJ)

Plano mediador de dois vértices adjacentes (PMVA).

Existem 12 arestas, logo são 12 pares de vértices


adjacentes, mas 4 pares possuem o mesmo plano
mediador. Portanto são 12 : 4 = 3 planos.

Plano mediador de dois vértices opostos de uma


face (PMVOF).

Plano mediador de dois vértices opostos (PMVO).

Repare que todos os planos mediadores juntos determina em cada face a seguinte
figura:

EUREKA! N°10, 2001


33
Sociedade Brasileira de Matemática

Como o centro do cubo é interseção de


todos os PMs e todas as interseções
PMVO
entre retas da figura ao lado são
extremidades das interseções entre PMs,
ao ligarmos as interseções entre PMs,
teremos várias pirâmides cujo vértice
comum é o centro do cubo e as bases são
PMVA os triângulos da face. Como são 16  6 =
PMVOF 96 triângulos no total, o cubo fica
dividido em 96 pirâmides.

XXII OLIMPÍADA BRASILEIRA DE MATEMÁTICA


Resultado – nível 1 (5a. e 6a. séries)
NOME CIDADE – ESTADO PRÊMIO
Maricy Miki Hisamoto São Paulo – SP Ouro
Yuriy Thallickson Bincovsky Curitiba – PR Ouro
Guilherme Rohden Echelmeier Itajaí – SC Ouro
Gabriel Tavares Bujokas São Paulo – SP Ouro
Heitor Silva Lima Lacerda Fortaleza – CE Ouro
Camila Santos Costa Salvador – BA Prata
Thomás Yoiti Sasaki Hoshina Rio de Janeiro – RJ Prata
Júlia Ribeiro Lamardo São Paulo – SP Prata
Augusto Ossamu Shitani São Paulo – SP Prata
Thiago de Paula Garcia Caixeta Colatina – ES Prata
Vitor Rezende Faria Goiânia – GO Prata
Vitor Humia Fontoura Salvador – BA Prata
Adriano Jorge Braun Vieira Neto Fortaleza – CE Prata
Mariana de Camargo Penteado São Paulo – SP Prata
Luiz Müller Vitória – ES Prata
Floris Uyttenhove Vitória – ES Bronze
Raul Máximo Alexandrino Nogueira Fortaleza – CE Bronze
Rudá Moreira de Lima e Silva Unaí – MG Bronze
Fabio Eigi Imada S.J. dos Campos – SP Bronze
Blandina Lavor Barbosa Bezerra Vila Velha – ES Bronze
Felipe Sanches Varroni São Paulo – SP Bronze
Nicoli Gavassa São Paulo – SP Bronze
Rafael Fonseca de Campos Atibaia – SP Bronze
Bruna Aguilar Trotta Belo Horizonte – MG Bronze
Daniel Folador Rossi São Mateus – ES Bronze
Max Douglas Peixoto da Silva Fortaleza – CE Bronze
Tiago Abreu Tavares de Sousa Campina Grande – PB Bronze
Paulo André Carvalho de Melo Rio de Janeiro – RJ Bronze
Eduardo Martins de Figueiredo Vitória – ES Bronze
João Marcos Nobuo Umetsu Hansen Jundiaí – SP Bronze
Gabriel Salvagno Jundiaí – SP Bronze
Adalberto Delgado Neto Fortaleza – CE Bronze
Gilberto Marques Arsiolli Três Lagoas – MS Bronze
Felipe Leon Peres Camargo Shalders Vitória – ES Bronze
Martin Alexander Barrios Gundelach Rio de Janeiro – RJ Menção Honrosa
Louise Nagashima Omi São Paulo – SP Menção Honrosa
Édipo Martins Sípoli Curitiba – PR Menção Honrosa
André Ikeda Cantão Curitiba – PR Menção Honrosa
Paulo Henrique Macera S.J. dos Campos – SP Menção Honrosa
Lucas de Barros Navarro Salvador – BA Menção Honrosa
Ana Paula Seno Pinheiro Ourinhos – SP Menção Honrosa
Eduardo Fischer Encantado – RS Menção Honrosa
Germano Bezerra de Menezes Pinho Fortaleza – CE Menção Honrosa
Wagner Silveira Aniceto Campo Grande – MS Menção Honrosa
Gabryel Melo Lutz Goiânia – GO Menção Honrosa

EUREKA! N°10, 2001


34
Sociedade Brasileira de Matemática

Diego Frade Bernardes Fortaleza – CE Menção Honrosa


Átila Pereira Ricarte Fortaleza – CE Menção Honrosa
Frederico de Souza Frydman Salvador – BA Menção Honrosa
Pedro Thiago Ezequiel de Andrade Fortaleza – CE Menção Honrosa
Hugo Siqueira Robert Pinto Fortaleza – CE Menção Honrosa
Bárbara Gomes Arabe Saraiva Santos – SP Menção Honrosa
Domingos Gomes de Aguiar Neto Fortaleza – CE Menção Honrosa
Fernanda Mary Sonoki São Paulo – SP Menção Honrosa
Carolina Lisboa Borgo Montanha – ES Menção Honrosa
Raphael Rodrigues Mata Salvador – BA Menção Honrosa
Gil Henriques Vassouras – RJ Menção Honrosa
Gustavo Schmidt Joau e Silva Juiz de Fora – MG Menção Honrosa
Anderson Cipriano de Lima Jaboatão dos G.– PE Menção Honrosa
Rafael Santos Correia de Araújo Salvador – BA Menção Honrosa
Paulo Henrique Gonçalves dos Santos São Paulo – SP Menção Honrosa
Hanna Kirihara e Silva Florianópolis – SC Menção Honrosa
Luciana Salomão Vilar São Carlos – SP Menção Honrosa
Thaís Viveiro São Paulo – SP Menção Honrosa
Raffaello Couto Caser Vitória – ES Menção Honrosa

XXII OLIMPÍADA BRASILEIRA DE MATEMÁTICA


Resultado – nível 2 (7a. e 8a. séries)
NOME CIDADE – ESTADO PRÊMIO
Alex Corrêa Abreu Niterói – RJ Ouro
Fabio Dias Moreira Rio de Janeiro – RJ Ouro
Larissa Cavalcante Queiroz de Lima Fortaleza – CE Ouro
Rafael Daigo Hirama Campinas – SP Ouro
Samuel Barbosa Feitosa Fortaleza – CE Prata
Jorge Peixoto de Morais Neto Goiânia – GO Prata
Israel Franklim Dourado Carrah Fortaleza – CE Prata
Davi Máximo Alexandrino Nogueira Fortaleza – CE Prata
Vitor Gabriel Kleine Mogi das Cruzes – SP Prata
Juliana Gomes Varela Fortaleza – CE Prata
Ayran Ayres Barbosa Loriato Vitória – ES Prata
Daniel Haanwickel Junqueira Salvador – BA Prata
Raquel Lamboglia Guimarães Fortaleza – CE Prata
Paulo Roberto Sampaio Santiago Salvador – BA Prata
Telmo Luis Correa Jr. Santo André – SP Prata
Adalberto Studart Neto Fortaleza – CE Bronze
João Marcos da Cunha Silva Fortaleza – CE Bronze
Thiago Costa Leite Santos São Paulo – SP Bronze
Kiyoshi Horie Filho Ourinhos – SP Bronze
Antônio Monteiro Guimarães Jr. Campina Grande – PB Bronze
Adriano Brasileiro Silva Fortaleza – CE Bronze
Renato Mendes Coutinho Americana – SP Bronze
Diogo dos Santos Suyama Belo Horizonte – MG Bronze
Gustavo Ferruzzi Martucci Piracicaba – SP Bronze
Henry Wei Cheng Hsu São Paulo – SP Bronze
Thiago Pinheiro Faury São Paulo – SP Bronze
Otacílio Torres Vilas Boas Salvador – BA Bronze
Vitor Sarmento Mesquita Fortaleza – CE Bronze
Felipe Netto de Santana Rio de Janeiro – RJ Bronze
Patricia Akemi Komura São Paulo – SP Menção Honrosa
Regiane Cristina Yamanari Guararapes – SP Menção Honrosa
Guilherme Honda Saito São Paulo – SP Menção Honrosa
Milton Eiji Kato São Paulo – SP Menção Honrosa
Vinicius Antonio Batagello Araçatuba – SP Menção Honrosa
Thiago Mizuta São Paulo – SP Menção Honrosa
Daniel Costa Garcia Goiânia – GO Menção Honrosa
Henrique Castro Noronha Valinhos – SP Menção Honrosa

EUREKA! N°10, 2001


35
Sociedade Brasileira de Matemática

Raphael Henrique Ribas Curitiba – PR Menção Honrosa


Dafne de Albuquerque Simão Fortaleza – CE Menção Honrosa
Michel Renato Manzolli Ballestero Araraquara – SP Menção Honrosa
Fernanda Ramos Correia Salvador – BA Menção Honrosa
Fernando Santos Simões Ferreira Vitória – ES Menção Honrosa
Lucas Lolli Saui Florianópolis – SC Menção Honrosa
Luciano Lacerda Silveira Campo Grande – MS Menção Honrosa
Samara Anny Maia Fava Fortaleza – CE Menção Honrosa
Gustavo Eufrásio Farias Fortaleza – CE Menção Honrosa
Vinicius Figueiredo de Castro Rio de Janeiro – RJ Menção Honrosa
Luís Eduardo de Godoi S.J. dos Campos – SP Menção Honrosa
Helder Seiji Kato São Paulo – SP Menção Honrosa
Márcio Jun Hisamoto São Paulo – SP Menção Honrosa
Vinicius Augusto Paccola Matão – SP Menção Honrosa
Erika Famini Silva Salvador – BA Menção Honrosa

XXII OLIMPÍADA BRASILEIRA DE MATEMÁTICA


Resultado – nível 3 (Ensino médio)
NOME CIDADE – ESTADO PRÊMIO
Ulisses Medeiros de Albuquerque Fortaleza – CE Ouro
Sérgio Tadao Martins São Paulo – SP Ouro
Daniel Massaki Yamamoto São Paulo – SP Ouro
Humberto Silva Naves São Paulo – SP Ouro
Carlos Stein Naves de Brito Goiânia – GO Ouro
Lucas Heitzmann Gabrielli São Paulo – SP Prata
Daniel Mourão Martins Fortaleza – CE Prata
Leonardo Augusto Zão Nilópolis – RJ Prata
Fabrício Siqueira Benevides Fortaleza – CE Prata
Christian Lyoiti Watanabe Itaguaí – RJ Prata
Rodrigo Roque Dias São Paulo – SP Prata
Rui Facundo Vigelis Fortaleza – CE Prata
Thiago Barros Rodrigues Costa Fortaleza – CE Prata
Ronaldo Ikaro Farias Araújo Fortaleza – CE Prata
Daniel Nobuo Uno São Paulo – SP Prata
Daniel Pinheiro Sobreira Fortaleza – CE Prata
João Alfredo Castellani Fajardo Freire Salvador – BA Bronze
Daniel Pessoa Martins Cunha Fortaleza – CE Bronze
Guilherme Fujiwara São Paulo – SP Bronze
Gilberto Kirk Rodrigues Rio de Janeiro – RJ Bronze
Artur Duarte Nehmi São Paulo – SP Bronze
Rodrigo Villard Milet Rio de Janeiro – RJ Bronze
Eduardo Moraes de Morais São Paulo – SP Bronze
Carlos Sartori Ferreira Filho Rio de Janeiro – RJ Bronze
Thiago da Silva Sobral Fortaleza – CE Bronze
Silvano José Gomes Flumignan P. Prudente – SP Bronze
Mateus Ymanaka Barretto São Paulo – SP Bronze
Hugo Pinto Iwata S.J. do Rio Preto – SP Bronze
Paulo Ribeiro de Almeida Neto Ananindeua – PA Bronze
Diêgo Veloso Uchôa Teresina – PI Bronze
Artur Radoman de Oliveira Rio de Janeiro – RJ Bronze
Yuri Gomes Lima Fortaleza – CE Menção Honrosa
Rafael Tajra Fonteles Teresina – PI Menção Honrosa
Maurício de Carvalho Paiva Belém – PA Menção Honrosa
Bernardo Freitas Paulo da Costa Rio de Janeiro – RJ Menção Honrosa
Eduardo Barbosa Araújo Fortaleza – CE Menção Honrosa
Antônio Davi Macêdo de Castro Fortaleza – CE Menção Honrosa

EUREKA! N°10, 2001


36
Sociedade Brasileira de Matemática

Eduardo Famini Silva Salvador – BA Menção Honrosa


Arnaldo João do Nascimento Júnior Duque de Caxias – RJ Menção Honrosa
Augusto Quadros Teixeira Belo Horizonte – MG Menção Honrosa
Einstein do Nascimento Jr. Fortaleza – CE Menção Honrosa
Marcos Soares de Souza Rio de Janeiro – RJ Menção Honrosa
Caio Augusto P. del Bianco Licciardi Atibaia – SP Menção Honrosa
Luiz Antonio Felinto Cruz Fortaleza – CE Menção Honrosa
Diego Alvarez Araujo Correia Fortaleza – CE Menção Honrosa
Afonso de Paula P. Rocha Fortaleza – CE Menção Honrosa
Tibério Bittencourt de Oliveira Goiânia - GO Mençao Honrosa

OS NÚMEROS IRRACIONAIS
Hermano Frid
Instituto de Matemática Pura e Aplicada - IMPA

 Nível Intermediário.

No texto a seguir fazemos uma breve introdução ao conceito de


número irracional. Na sua maior parte o texto será acessível a alunos da última
série do primeiro grau. As duas últimas seções talvez requeiram um pouco mais
de maturidade embora não exijam nenhum conhecimento prévio adicional. Para
simplificar a exposição nos restringiremos a números positivos. A extensão dos
fatos abordados ao contexto geral de números positivos, negativos e 0 não requer
nenhuma dificuldade adicional.
Pode-se imaginar que a idéia de número inteiro positivo tenha surgido num
estágio primário da civilização, juntamente com a necessidade da prática da
contagem. Por exemplo, era necessário a um pastor saber contar de algum modo o
número de animais no seu rebanho. A maneira de representar o resultado dessa
contagem era no início bastante diferente da que usamos agora e é provável que
no começo cada pessoa tivesse sua maneira própria de fazê-lo. Contar significa
estabelecer um modo de comparar quantidades de elementos de conjuntos
distintos. Por exemplo, a quantidade de pedrinhas em um saco com a quantidade
de animais num rebanho, ou a quantidade de alimentos conseguidos em uma
caçada ou em colheita com a quantidade de membros da tribo. Também não é
difícil imaginar que a ideia de fração tenha surgido na evolução da civilização
humana, primeiramente e de forma mais elementar, com a ocorrência usual da
necessidade de um determinado grupo de pessoas partilhar um ou mais bens de
propriedade comum entre seus membros. E num estágio mais avançado, dentre
outras motivações possíveis, com a necessidade de as pessoas trocarem entre si
bens de tipos distintos. Por exemplo, um pastor deseja trocar com um agricultor

EUREKA! N°10, 2001


37
Sociedade Brasileira de Matemática

peles de carneiro por sacos de milho numa razão de 3 peles de carneiro para cada
grupo de 7 sacos de milho. Por outro lado, a idéia de um “número” que não seja
nem inteiro nem fração é, em princípio, muito menos natural que a daqueles e
surge num estágio muito mais avançado da civilização com a necessidade da
prática da medição. Por exemplo, medir as dimensões ou a área de um terreno,
comparar as distâncias entre pares de pontos distintos, etc. Procuraremos, a
seguir, mostrar as propriedades básicas destes números “estranhos” em contraste
com as propriedades, na maior parte já bem conhecidas, daqueles mais intuitivos,
os inteiros e as frações.

1. BASE DECIMAL; DÍZIMAS

Os números reais positivos podem ser representados no sistema decimal por uma
seqüência de algarismos – elementos do conjunto {0, 1, 2, 3, 4, 5, 6, 7, 8, 9} –
Separados por uma vírgula. Assim, se a N , a N  1 ,..., a 0 , a  1 , a  2 , a  3 ,..., são
algarismos quaisquer, um número real positivo representado no sistema decimal
tem a forma a N a N  1 a N  2 ...a1 a 0 , a  1 a  2 a  3 ..., (1)
onde a N  0. Nessa representação, à esquerda da vírgula temos sempre um
número finito de algarismos, porém à direita podemos ter uma infinidade de
algarismos. Por exemplo, 783,5231 representa o número obtido como resultado
da expressão
7 10 2  8 101  3 10 0  5 10  1  2 10  2  3 10  3  1 10  4.
(2)

154
Por outro lado, a fração tem representação decimal 0, 1545454… com uma
999
infinidade de algarismos à direita. Essa representação se traduz como resultado de
uma expressão com infinitas parcelas
1 10  1  5 10  2  4 10  3  5 10  4  4 10  5  5 10  6  ...
(3)
154
Essa expressão significa exatamente que se quisermos aproximar no
999
sistema decimal com “precisão de 8 casas decimais, por exemplo, devemos tomar
como aproximação o número 0,15454545 que é resultado da expressão
1 10  1  5 10  2  4 10  3  5 10  4  4 10  5  5 10  6
 4 10  7  5 10  8.
(4)

EUREKA! N°10, 2001


38
Sociedade Brasileira de Matemática

Claro, o número 0, 1545454… é o que chamamos de uma dízima periódica e por


154
isso pode ser obtido como uma fração .
999

O QUE ACONTECE NO CASO DE UMA DÍZIMA NÃO-PERIÓDICA?

Neste caso, assim como no periódico, temos uma infinidade de algarismos à


direita da vírgula e assim só nos é possível escrever a representação decimal até
uma certa casa decimal, porém, diferentemente do que acontece no caso
periódico, não há repetição indefinidamente de um determinado grupo de
algarismos e, assim, o número em questão não pode ser obtido como uma fração
p
com e e q diferente de 0. Os números que podem ser obtidos como frações
q
são chamados racionais; os que não podem ser obtidos como frações são
chamados irracionais.
2. POR QUE PRECISAMOS DOS NÚMEROS IRRACIONAIS?
Responderemos esta pregunta através de um exemplo. Euclides provou que o
número positivo cujo quadrado é 2, isto é, o número positivo x que satisfaz a
equação x 2 2, (5)
não é racional. Euclides argumentou da seguinte forma: Suponhamos que o
número x satisfazendo (5) seja racional. Então existem inteiros positivos p e q,
p2
primos entre si, tais que  2. ou seja p 2 2q 2 .
q2
(6)
Portanto p 2 é par e p também é par; p pode ser escrito na forma p = 2k. Assim,

( 2k ) 2 2q 2  2 k 2 q 2  (7)
Pela mesma razão que acabamos de expor, concluímos que q também deve ser
par. Mas isto nos leva a uma contradição pois p e q são primos entre si por
p
hipótese! Assim, a suposição de que x  nos leva a uma contradição e,
q
portanto, deve ser descartada, considerada falsa.
Chegamos à conclusão que 2 , que é como representamos o número positivo
cujo quadrado é 2, é um número irracional!!
3. COMO OBTER APROXIMAÇÕES RACIONAIS PARA 2
Podemos obter aproximações cada vez melhores de 2 (o número x que satisfaz
(5)) através do seguinte procedimento que é um caso particular de um esquema
inventado por Newton conhecido como método de Newton. (Com base nesse

EUREKA! N°10, 2001


39
Sociedade Brasileira de Matemática

método podemos programar as máquinas de calcular para produzirem


aproximações de 2 tão precisas quanto o avanço da eletrônica nos permitir).
primeiro “chutamos” um número x 0 como uma primeira aproximação de x que
nos pareça razoável; por exemplo, x 0 1. Em seguida observamos que
x 2  x02 ( x  x0 )( x  x0 )  2 x 0 ( x  x0 ),
onde o símbolo  significa “é aproximadamente igual a”. Assim,
x 2  x02  2 x0 ( x  x0 ),

EUREKA! N°10, 2001


40
Sociedade Brasileira de Matemática

e, portanto, dividindo a “equação aproximada” por 2x 0 e arranjando os termos,


obtemos
x 2  x 02
x  x0 . (8)
2 x0
2 1 3
substituindo x 2 2 e x 0 1 em (8), obtemos x  1  .
2 2
3
Assim temos uma segunda aproximação x1  . Encontramos também x 2 :
2
9
2
x2  4  3  x   1 1  3  x   1  3  x 2 17 . Da mesma forma,
2 2
3 2 4 3 2 12 2 12
podemos obter uma quarta aproximação x3 , fazendo
x 2  x 22 2  (17 / 12) 2 17
x3   x2  
2 x2 17 / 6 12
288  289 17 288  289  2 289 577
    .
2 12 17 12 2 12 17 408
577
Assim, x3  seria a aproximação seguinte: Sua representação decimal é a
408
x 1,4142156862745
      098039
   21568627
    ...
 9....
dízima periódica 3
período

Agora se você pegar uma máquina de calcular e pedir (através dos devidos
comandos) que ela calcule 2 , você obterá, se sua máquina puder exibir 33
dígitos (incluindo a vírgula ou ponto), a expressão decimal

1,4142135623730950488016887242097.

Horrível, não é? Você obterá uma expressão ainda maior se sua máquina puder
exibir mais dígitos. Repare como nossas aproximações x1 , x 2 e x3 estão cada
vez mais próximas desse número!

4. OS NÚMEROS RACIONAIS PODEM SER ENUMERADOS

EUREKA! N°10, 2001


41
Sociedade Brasileira de Matemática

Isto significa que podemos dispor os números racionais numa sucessão da forma
r1 , r2 , r3 ,..., com uma infinidade de elementos. Podemos interpretar este fato
como significando que a quantidade de números racionais, embora sendo infinita,
é de uma “ordem de infinitude” equivalente a dos números naturais 1, 2, 3…. O
argumento para a demonstração desse fato é devido a Georg Cantor.
p
Como todo racional tem uma representação única como fração com p e q
q
inteiros positivos primos entre si, basta que saibamos enumerar os pares
ordenados (p, q) de naturais primos entre si. A forma de obter essa enumeração
está descrita pela figura abaixo:

EUREKA! N°10, 2001


42
Sociedade Brasileira de Matemática

EUREKA! N°10, 2001


43
Sociedade Brasileira de Matemática

A enumeração é obtida seguindo-se o caminho indicado pelas flechas, iniciando a


partir de (1,1), tendo o cuidado de descartar os pares de naturais que não são
primos entre si, como, por exemplo, (2,2), (4,2), (3,3) etc.. Com isso, teríamos

1 1 2 3 1
r1  1, r2  , r3  2, r4  3, r5  , etc.
1 2 1 1 3

5. REPRESENTAÇÃO DECIMAL DOS RACIONAIS

Há pouco dissemos que não era possível pôr uma dízima não periódica em forma
p
de fração com p e q naturais primos entre si. Vamos dar uma explicação para
q
este fato. Fixemos um natural q. Quando dividimos um número qualquer N > q
pelo número q. Obtemos como resto da divisão um elemento do conjunto (finito)
{0, 1, 2,…, q – 1}. Tomemos como exemplo q = 7 e N = 17; nesse caso os restos
possíveis pertencem ao conjunto {0, 1, 2, 3, 4, 5, 6}. Agora vamos recordar o
algoritmo da divisão com esse exemplo específico:

EUREKA! N°10, 2001


44
Sociedade Brasileira de Matemática

17 7
1 4 2, 4 2 8 5 7 1 4 2 8 5 7 1…
30
28
20
14
60
56
40
35
50
49
10
7
3 0

EUREKA! N°10, 2001


45
Sociedade Brasileira de Matemática

O que acontece é que os restos possíveis são elementos do conjunto finito de q


elementos {0, 1,…, q – 1}(no exemplo acima q = 7). Assim, em no máximo q
iterações do algoritmo ou acabamos repetindo um elemento do conjunto de restos
possíveis (no exemplo acima o primeiro a se repetir foi o 3), ou o 0 ocorre como
resto e o processo termina. No primeiro caso, a partir daí passamos a repetir os
restos ocorridos anteriormente na mesma ordem (3, 2, 6, 4, 5, 1, no exemplo
acima). As casas decimais no quociente por sua vez também se repetem o
obtemos então uma dízima periódica. No segundo caso, obtemos simplesmente
um número finito de casas decimais.

6. REPRESENTAÇÃO DECIMAL DOS IRRACIONAIS

Todo número irracional positivo possui uma representação decimal única por
meio de uma dízima não periódica. Para simplificar vamos nos restringir aos
números entre 0 e 1. Já sabemos que um número cuja representação decimal
possui uma quantidade finita de casas decimais pertence ao conjunto dos
racionais. Da mesma forma aprendemos que um número cuja representação
decimal é uma dízima periódica é também um número racional. Por outro lado,
vimos no item anterior que as representações decimais de um racional são
necessariamente de um dos dois tipos: ou possuem uma quantidade finita de casas
decimais, ou “terminam” em uma dízima periódica. Logo, uma representação
decimal para um número irracional tem necessariamente que ser uma dízima não-
periódica. Afirmamos que essa representação é única. Repare que isso não ocorre
em geral com os racionais. Por exemplo, 0, 21 e 0, 20999… representam o
21
mesmo racional . Suponhamos que um irracional x entre 0 e 1 possua duas
100
representações decimais distintas:
x 0, a  1 a  2 a  3 ..., (10)
x 0, b 1b 2 b 3 ..., (11)
Se essas representações são distintas certamente existe um p  N tal que
a  k b k , para k 0,..., p  1, e a  p b p . Para fixar idéias vamos assumir
então que a  p b p  1 e por (10) e (11)
x 0, a  1 a  2 ...a  p , (12)
x 0, a  1 a  2 ...b p 999... 0, a  1 a  2 ...(b p  1), (13)
já que b k a  k se k  0 ,..., p  1 e b k é sempre menor ou igual a 9. Mas
(12) e (13) implicam que  p a  b p  1 e x 0, a  1 a  2 ...a  p .
Porém nesse caso x é racional e chegamos a uma contradição! Chegaríamos a
uma contradição semelhante também se tivéssemos assumido b p  a  p ,

EUREKA! N°10, 2001


46
Sociedade Brasileira de Matemática

argumentando da mesma forma apenas trocando os papéis dos a  k e b k . A


contridição tem origem no fato de termos suposto que havia duas representações
decimais distintas para o mesmo irracional x. Logo essa possibilidade tem que ser
descartada, considerada falsa, e assim concluímos que todo irracional possui uma
representação decima única como dízima não-periódica.

7. OS IRRACIONAIS NÃO PODEM SER ENUMERADOS

Isto significa que não podemos dispor os números irracionais numa sucessão
s1 , s 2 , s 3 ,..., mesmo admitindo uma infinidade de elementos. Quer dizer,
diferentemente dos racionais, a “ordem de infinitude” da quantidade dos números
irracionais é maior que a dos números naturais. Concluímos daí que existem
muito mais números irracionais do que racionais!
Vamos tentar justificar nossa afirmação sobre a não-enumerabilidade dos
irracionais. O argumento é uma adaptação de uma idéia também devida a G.
Cantor. Suponhamos que fosse possível dispor os irracionais numa sucessão
s1 , s 2 , s 3 ,..., . Basta considerarmos apenas os irracionais entre 0 e 1. Criamos
um número irracional x, também entre 0 e 1, através de uma representação
decimal (portanto, uma dízima não periódica) da seguinte forma. O número x tem
representação decimal dada por x 0, x  1 x  2 x  3 ... onde x  p é escolhido
dentro do conjunto {0, 1, …, 9} de modo que x  p é diferente de ( s p )  p onde
este último é o algarismo que aparece na casa decimal de ordem p do irracional
s p (p-ésima elemento da sucessão s1 , s 2 ,...s p ,...). A escolha de cada x p
também deve atender a condição de não permitir que nenhum grupo de
algarismos dentre os já escolhidos x  1 , x  2 ,..., x  ( p  1) possa se tornar o
gerador de uma dízima periódica. Desta forma obtemos uma dízima não periódica
representando um único irracional que, no entanto, não pode constar na lista
s1 , s 2 , s 3 ,..., . De fato, se x  s r , para algum r  N, então como
x  r ( s r )  r teríamos um absurdo (uma contradição)!.

8. ESTUDO SUPLEMENTAR: O IRRACIONAL 

O número  é definido como sendo a área limitada por um círculo de raio 1. Ele é
certamente o irracional transcendente mais conhecido. A expressão transcendente
significa, neste contexto, um número irracional que não é raiz de nenhuma
equação polinomial com coeficientes inteiros. Por exemplo, os irracionais
2 ,1  3 não são transcendentes pois são raízes das equações polinomiais

EUREKA! N°10, 2001


47
Sociedade Brasileira de Matemática

x 2 2, x 2  2 x  2 0, respectivamente. Neste último caso dizemos que os


números são algébricos. A demonstração de que  é um número irracional, apesar
de não ser trivial, pode ser feita usando-se apenas o cálculo diferencial elementar
que é ensinado no primeiro período dos cursos de ciências exatas. A primeira
demonstração de que  é irracional só foi obtida em 1766 por J. H. Lambert, de
forma não completamente rigorosa, tendo sido finalmente (re)obtida de modo
rigoroso pelo famoso matemático A. M. Legendre e publicada em 1855. A prova
de que  é transcendente é muito mais complexa e só foi obtida em 1882 por F.
Lindermann.
O fabuloso matemática grego Arquimedes foi o primeiro a obter uma
aproximação razoável de  por numeros racionais. Ele provou que
10 1
3   3 ,
71 7
usando dois polígonos regulares de 96 lados, um inscrito e outro circunscrito a
um círculo de raio 1.
Podemos obter aproximações cada vez melhores de , com o auxílio de uma
máquina de calcular bastante rudimentar, capaz apenas de fazer as operações
básicas (+, –,  ) e mais a operação de extrair raiz quadrada, da seguinte forma. A
idéia é aproximarmos o círculo de raio 1 por polígonos regulares de 2 n lados
inscritos neste círculo. Primeiramente, é fácil verificar que para a área e o
perímetro do polígono regular de 2 n lados inscritos num círculo de raio 1 temos
1
Área = Perímetro  4  l 2 ,
4
onde l é o comprimento do lado do polígono. Como l se aproxima mais e mais de
0 a medida que n cresce, vemos que para o círculo de raio 1 devemos ter (fazendo
l = 0 na fórmula acima)
1
Área = Perímetro
4
Assim, podemos também definir  como sendo a metade do perímetro do círculo
de raio 1. Por outro lado, usando o teorema de Pitágoras que diz que em um
triângulo retângulo o quadrado da hipotenusa é a soma dos quadrados dos catetos,
se l n denota o comprimento do lado do polígono regular de 2 n lados, é fácil
mostrar que
l n 1  2  4  l n2 . (14)
Para n = 2 temos o polígono regular de 4 lados, quadrado, inscrito no círculo de
raio 1, cujo lado, facilmente obtido usando-se o teorema de Pitágoras, é
l 2  2.

EUREKA! N°10, 2001


48
Sociedade Brasileira de Matemática

Por meio de (14) obtemos sucessivamente

l3  2  2,

l4  2  2 2,

l5  2  2 2 2,

l6  2  2 2 2 2 ,

l7  2  2 2 2 2 2 ,

l8  2  2 2 2 2 2 2 ,
 

Para obter uma boa aproximação de  calculemos, por exemplo, o valor da


metade do perímetro do polígono de 2 8 256 lados, inscrito no círculo de raio
1, cujo lado tem comprimento igual a l8 . Podemos obter um valor aproximado
para l8 executando a seguinte seqüência de operações numa calculadora
2 sqrt + 2 = sqrt + 2 = sqrt + 2 = sqrt
+ 2 = sqrt + 2 = sqrt + / – + 2 = sqrt,
e obtemos
l8 = 0.0245430765714398521588165239020064.
Agora, multiplicaremos o resultado obtido para l 8 por 256, que é o número de
lados da polígono em questão, e em seguida dividimos por 2 o que nos dá

 ~ 3.14151380114430107632851505945682
o que fornece uma aproximação com erro menor que 0, 0001 já que é sabido que

3, 1415 <  < 3, 1416.

Considerações finais: Exceto pelas duas últimas seções, o texto acima foi
elaborado a partir de um “pedido” de minha filha, Marina, atualmente na 8a. série
do primeiro grau, urgida por um trabalho de casa em grupo passado por sua
professora. O referido trabalho, felizmente, resultou bastante diferente do que foi
exposto acima, que acabou servindo apenas como uma entre várias referências
usadas pelo grupo. No entanto, as 7 primeiras seções foram bem compreendidas

EUREKA! N°10, 2001


49
Sociedade Brasileira de Matemática

por ela e seu grupo; as duas últimas foram escritas depois que o prazo para a
entrega do trabalho havia esgotado e, portanto, não chegaram a ser “testadas”.
Para concluir gostaria de deixar aqui meus agradecimentos ao estimado professor
e colega Elon Lages Lima pelas sugestões sobre uma versão preliminar destas
notas.

EXERCÍCIOS:

1) Usando o mesmo argumento de Euclides descrito em 2. prove que


3 , 5 e 7 são irracionais.
2) Usando o método de Newton, descrito em 3, obtenha aproximações
correspondentes ao x3 do texto para os irracionais 3 , 5 , 7 e compare
com o resultado fornecido pela máquina de calcular.
3) Pesquise sobre a vida e a obra dos grandes matemáticos mencionados no
texto: Arquimedes, Pitágoras, Euclides, Isaac Newton e Georg Cantor.
4) Prove a fórmula (14).
OLIMPÍADAS AO REDOR DO MUNDO

 O comitê editorial de EUREKA! agradece aos inúmeros elogios


recebidos pela criação desta seção bem como ao crescente número de leitores que
nos enviam soluções para os problemas da mesma.
A realização de inúmeras olimpíadas nesta época do ano torna mais
importante a preparação para as mesmas e por isto resolvemos apresentar neste
número somente novos problemas acompanhados dos nomes dos leitores que nos
enviaram soluções de problemas anteriores. No próximo número de EUREKA!
voltaremos ao normal apresentando problemas e soluções.
Continuamos salientando que estamos à disposição na OBM para aqueles
que estiverem interessados na solução de algum problema particular. Para tanto,
basta contactar a OBM, seção OLIMPÍADAS AO REDOR DO MUNDO, através de
carta ou e-mail.

Antonio Luiz Santos

Primeiramente vamos aos problemas propostos deste número

EUREKA! N°10, 2001


50
Sociedade Brasileira de Matemática

Sejam a, b, c números reais tais que as equações


61. (Rússia-2000)
x  ax  1 0 e x 2  bx  c 0 possuem exatamente uma raiz real
2

comum e as equações x 2  x  a 0 e x 2  cx  b 0 também possuem


exatamente uma raiz comum. Determine a soma a  b  c .

2 3
1  2   2   2   21000 
62. (Rússia-2000) Determine a soma        
      
 3  3   3   3   3 
onde, como usual,  x  é o maior inteiro que não supera x.
63. (Rússia-2000) Sejam a e b números reais não nulos que satisfazem à
equação
  
a 2 b 2 a 2 b 2  4 2 a 6  b 6 
Mostre que pelo menos um deles não é racional.

64. (Rússia-2000) Seja M o conjunto que consiste dos 2000 números


11, 101, 1001, ... . Mostre que pelo menos 99% dos elementos de M
não são primos.
65. (Rússia-2000) Sejam ABCD um paralelogramo com A 60º e O o
circuncentro do triângulo ABD . A reta AO intersecta a bissetriz externa
do ângulo C do paralelogramo em K . Determine AO OK .

66. (Estônia-2000) Sejam ABC um triângulo acutângulo com ACB 60º ,


AD e BE duas de suas alturas que se intersectam no ponto H . Mostre
que o circuncentro do triângulo ABC pertence à bissetriz dos ângulos
AHE e BHD .

67. (Estônia-2000) Sobre o lado AC do triângulo ABC , toma-se um ponto D


distinto de A e C . Sejam O1 e O2 os centros dos círculos circunscritos
aos triângulos ABD e CBD respectivamente. Mostre que os triângulos
O1 DO2 e ABC são semelhantes.

68. (Eslovênia-2000) Determine todos os inteiros n para os quais o valor da


expressão abaixo é inteira.
25 625 25 625
  n   n
2 4 2 4

EUREKA! N°10, 2001


51
Sociedade Brasileira de Matemática

69. (Eslovênia-2000) Determine todas as funções f : Z  Z tais que f  f  n   n e


f  f  n  2  2  n para todo inteiro n e f  0  1 .

70. (Belarus-2000) Os triângulos equiláteros ABF e CAG são construídos


externamente sobre a hipotenusa AB e sobre o cateto CA do triângulo
retângulo ABC . Se M é o ponto médio de BC , determine BC se
MF 11 e MG 7 .

71. (Belarus-2000) Determine todos os pares de inteiros positivos  m, n  que


satisfazem à equação
 m  n  2  n 2  m  4 m 2 n
72. (Belarus-2000) Quantos pares  n, q  satisfazem a igualdade
n! 
 q   2000
2

 
com n inteiro positivo e q um número racional não inteiro tal que
0  q  2000 , onde  x  x   x  ?
73. (Moldávia-2000) O ortocentro H de um triângulo ABC não pertence a
nenhum de seus lados. Sabendo que a medida de AH é igual ao raio do
círculo circunscrito ao triângulo ABC , determine a medida do ângulo
A .

74. (Moldávia-2000) Resolva em R a equação


x 2
 3x  2  2
 
 3 x 2  3 x  2  2  x 0

75. (Moldávia-2000) Dado o número 2000 , calcule a soma das décimas potências
dos algarismos deste número e continue fazendo o mesmo com o número
obtido e assim sucessivamente. Mostre que entre os números obtidos existem
pelo menos dois números iguais.

76. (Moldávia-2000) Os números inteiros a, b, c satisfazem à relação


a  b  c 0 . Mostre que o número 2a 4  2b 4  2c 4 é um quadrado
perfeito.

77. (Hungria-2000) Se 
A  1000  1000 2  1 1000
, determine o 2000–ésimo
algarismo após a vírgula da sua representação decimal.

EUREKA! N°10, 2001


52
Sociedade Brasileira de Matemática

78. (Hungria-2000)Determine todos os inteiros n para os quais


n  4n  14n 2  20n  10 é um quadrado perfeito.
4 3

79. (Repúblicas Tcheca e Eslovaca-2000) Uma função f : N  N é tal que f  n  1


se n é ímpar, e f  n  k para todo inteiro par n 2 k l , onde k é um
número natural e l um número ímpar . Determine o maior número natural n
para o qual
f 1  f  2    f  n  123456

80. (Repúblicas Tcheca e Eslovaca-2000) Se n


é número natural, mostre que
2n
4 3  3 4 2n é divisível por 13 se, e somente se, n é par.

81. (Irã-2000) Dois círculos se intersectam nos pontos A e B . Uma reta l que
passa por A , intersecta estes círculos nos pontos C e D . Se M e N
são os pontos médios dos arcos BC e BD que não contém A e K é o
ponto médio de CD , mostre que MKN 90º .

82. (Irlanda-2000) Sejam x 0 , y 0 números reais tais que x  y 2 . Mostre



que x 2 y 2 x 2  y 2 2 . 
83. (Iugoslávia-2000) Em um tubo de ensaio há exatamente uma ameba. A cada
segundo algumas das amebas dividem-se em sete novas amebas ou morre
exatamente uma das amebas. Determine o período mínimo de tempo após o
qual o número de amebas no tubo de ensaio será igual a 2000 .

84. (Iugoslávia-2000) Mostre que todo número racional positivo pode ser
representado sob a forma
a3  b3
r
c3  d 3
onde a, b, c e d são inteiros positivos.

Os lados AC e BC de um triângulo ABC possuem a


85. (Polônia-2000)
mesma medida. Sejam P um ponto do interior do triângulo tal que
 PAB PBC e M o ponto médio de AB Mostre que
 APM  BPC 180º .

EUREKA! N°10, 2001


53
Sociedade Brasileira de Matemática

86. (Inglaterra-2000) Dois círculos C1 e C 2 se intersectam nos pontos M e N


e possuem uma tangente comum sendo P e Q respectivamente os pontos
de tangência com os círculos. Se N é o ponto mais próximo de PQ e a
reta determinada por PN intersecta C 2 novamente em R , mostre que
MQ é a bissetriz do ângulo PMR .

87. (Inglaterra-2000) Para cada inteiro positivo k , definamos a seqüência  an 


por a 0 1 e a n kn    1 a n  1 para n 1 . Determine todos os valores
n

de k para os quais 2000 é um termo da seqüência.

88. (Inglaterra-2000) Sejam x, y , z números reais positivos tais que xyz 32 .
Determine o valor mínimo de
x 2  4 xy  4 y 2  2 z 2
89. (Balcânica-2000) Determine todas as funções f : R  R que possuem a
propriedade :
f  xf  x   f  y    f  x    y
2

para todos os números reais x e y .

90. (Baltic Way-2000) Seja ABC um triângulo tal que


BC AB  BC

AB  BC AC
Determine a razão  A : C .



Enviaram soluções de problemas anteriores os seguintes leitores da EUREKA!

Cláudio Pamplona dos Santos Dias Rio de Janeiro – RJ Prob. 36


Daniel Pinheiro Sobreira Fortaleza – CE Prob. 32
Davi Máximo Alexandrino Nogueira Fortaleza – CE Prob. 28, 47, 48
Diego Alvarez Araújo Correia Fortaleza – CE Prob. 20, 32, 36, 43, 48, 51, 52
Diêgo Veloso Uchôa Teresina – PI Prob. 1, 26, 29, 31, 28
Evandro Makiyama São Paulo – SP Prob. 35, 39, 45, 46, 47, 52, 56
Formiga Atibaia – SP Prob. 32, 43, 45, 56
Geraldo Perlino Jr. São Paulo – SP Prob. 32 ao 59
Juliana Gomes Varela Fortaleza – CE Prob. 37, 43, 45
Lucas de Melo Pontes Fortaleza – CE Prob. 32, 35, 36, 43, 45, 47, 51, 52, 54
Luciano Prudente Rio de Janeiro – RJ Prob. 32, 43, 44, 47, 51, 52
Marcelo Ribeiro de Souza Rio de Janeiro - RJ Prob. 32, 44
Marcílio Miranda de Carvalho Teresina – PI Prob. 8, 37, 45, 51

EUREKA! N°10, 2001


54
Sociedade Brasileira de Matemática

Márcio Miranda de Carvalho Teresina – PI Prob. 26, 27, 44, 49, 50


Marina Lima Medeiros Fortaleza – CE Prob. 47
Rodrigo Roque Dias São Paulo – SP Prob. 20, 43
Rodrigo Villard Milet Rio de Janeiro – RJ Prob. 32, 36, 43, 45, 51, 53, 54, 56, 57
Thiago da Silva Sobral Fortaleza – CE Prob. 32 ao 56
Wallace Rodrigues de Holanda Miranda Teresina – PI Prob. 32, 33, 49, 51, 52, 56, 59
Yuri Gomes Lima Fortaleza – CE Prob. 32 ao 56

Você sabia…
Que para calcular as raízes quadradas de 100, 121, 144,
169, 400, 441, 484, 900, 961, basta calcular as raízes dos
algarísmos situados nos extremos?
Há outros números de 3 dígitos com essa propriedade?

Colaboração de Rildo Alves do Nascimento (Sta. Maria da Boa


Vista – PE)

SOLUÇÕES DE PROBLEMAS PROPOSTOS


 Publicamos aqui algumas das respostas enviadas por nossos leitores.

41) Se a e b são números reais positivos, então a b  b a  1.

Solução dos alunos do CEMPI (Teresina – PI)


Observe que se “a” ou “b” for maior ou igual a 1 a desigualdade é imediata.
Então analisaremos o caso em que 0  a, b  1.
1 1
Seja a  ,b , u  0 e v  0.
(1  u ) (1  v)
1 1 1 1 1 v
ab  b
*   
Então (1  u ) 1  u b 1  u b  1  1 u  v e
1  u  
1 v 
a 1 u
Analogamente b  e portanto:
1u  v
1 v 1 u 1
ab  ba   1   1.
1 u  v 1 u  v 1u  v
*Agora o problema se reduz a mostrarmos que (1  u ) b  1  ub para 0  b  1.
Para isso basta termos uma noção de cálculo diferencial

EUREKA! N°10, 2001


55
Sociedade Brasileira de Matemática

 1 
f (u ) 1  bu  (1  u ) b  f ' (u ) b  b(1  u ) b  1 b 1  1 b 
 0.
 (1  u ) 
Como f é crescente no intervalo (0,1)  f (u )  f (0) 0 e portanto segue
imediatamente a demonstração.

45) Existe uma seqüência infinita de:


a) Números reais
b) Números inteiros
Tais que a soma de quaisquer dez termos consecutivos é positiva, enquanto que
para todo n a soma dos primeiros 10n + 1 termos consecutivos é negativa?

Solução de Zoroastro Azambuja Neto (Rio de Janeiro – RJ)


1 1
a) Sim. Tome a10 k 1  k
para todo inteiro positivo k e a n   se n não é
10 9
múltiplo de 10.
1 1 1 1 1
Temos a1  ...  a10n 1    ...  n    0 e, se
10 100 10 9 9 10 n
1
10k  n 10(k  1), a n  a n 1  ...  a n 9  k 1  0.
10
b) Não. Se n  a1 , a1  a 2  ...  a10 n 1 a1  (a 2  ...  a11 )  (a12  ...  a 21 ) 
 (a10 n  g  ...  a10 n 1 )  a1  1  1  ...  1  a1  n  0.

47) Dada uma circunferência , trace as tangentes a ela por um ponto exterior,
A, tocando-a em M e N. Trace a reta r passando por A e tocando  em B e C.
Se D é o ponto médio de MN , prove que MN é a bissetriz de BDC .

Solução de Cláudio Arconcher (Jundiaí – SP)


L
r*

O’

N
r
P C
B
A
D O

EUREKA! N°10, 2001


56
Sociedade Brasileira de Matemática

Consideremos a inversão na circunferência  de centro O.


Notemos que o ponto D, pela sua definição, é o inverso do ponto A em relação a
 , B e C coincidem com seus inversos em relação a  . Assim a figura inversa
da reta r é a circunferência r* que contém O. Assim sendo as circunferências 
e r* compartilham a corda BC . A reta perpendicular a BC pelo ponto médio P
passa por O e O’ (centro de r*). Por esse motivo os arcos RL e CL indicados
___
na figura acima são congruentes, ( OL é um diâmetro de r*).
Notemos agora que o fato de ser reto o ângulo ND ˆ O implica que a reta pelos
pontos N e D passa por L. Isso completa nossa demonstração: os ângulos inscritos
BD ˆ L e CD ˆ L ”enxergam” arcos congruentes do mesmo círculo, melhor,
circunferência, r*, são congruentes por esse motivo.

49) É dado um polígono regular de n lados.


Assinale aleatoriamente, no seu interior, um ponto M. Sendo x1 , x 2 ,..., x n as
distâncias de M a cada um dos lados, verifique que:
1 1 1 2
  ...  , onde a é a medida do lado do poligono.
x1 x 2 xn a

Solução de Antonio Caminha Neto (Fortaleza – CE)


Sejam O o centro e A1 , A2 , ...., An os vértices do polígono. Podemos supor,
sem perda de generalidade, que x i é a distância de M ao lado Ai Ai 1 (aqui,
An 1  A1 ). Então
n n
axi a
A( A1 A2 ... An )  A( Ai MAi 1 )   ( x1  x 2  ....  x n ) (*)
i 1 i 1 2 2
Por outro lado, sendo r o raio do círculo  inscrito no polígono, temos também
n n
ar ar
A( A1 A2 ... An )  A( Ai OAi 1 )  n  (**)
i 1 i 1 2 2
Comparando (*) e (**), concluímos que x1  x 2  ....  x n nr . Agora, note que
o perímetro de  é menor que o do polígono, o que nos dá 2 πr  na . Então
n2a
x1  x 2  ....  x n nr 

EUREKA! N°10, 2001


57
Sociedade Brasileira de Matemática

Finalmente, sabemos (ver artigo “Desigualdades Elementares, A. Caminha”,


Eureka 5) que para os reais positivos x1 , x 2 , ...., x n tem-se
2
1 1 1 n
  ....   , e daí
x1 x 2 x n x1  x 2  ....  x n
1 1 1 n2 n2 2π
  ....    2 2
 .
x1 x 2 x n x1  x 2  ....  x n n /(n a / 2 ) a

50) Calcule o determinante:

MDC (1, 1) MDC (1,2)  MDC (1, n)


MDC (2,1) MDC (2,2)  MDC (2, n)
  
MDC (n,1) MDC (n,2)  MDC (n, n)
Onde MDC (a, b) é o máximo divisor comum de a e b.

Solução de Márcio A. Assad Cohen e Fábio de Oliveira Costa (Rio de Janeiro – RJ)
Convenção: det (n) é igual ao determinante desejado quando este tem n linhas e
colunas.
Inicialmente escreveremos o determinante com uma quantidade de linhas
suficiente para percebermos como o problema se comporta ao mudarmos o valor
de n.
1 1 1 1 1 1 1 1
1 2 1 2 1 2 1 2
1 1 3 1 1 3 1 1
1 2 1 4 1 2 1 4
Determinante quando n = 8
1 1 1 1 5 1 1 1
1 2 3 2 1 6 1 2
1 1 1 1 1 1 7 1
1 2 1 4 1 2 1 8
Pode-se perceber que em cada linha k 1 k n  do determinante, os elementos
dessa linha considerada repetem-se com período k, pois mdc(x,k) = mdc(x + k, k)
Como mdc(1,n) = 1 n  N , a primeira linha é 1 1 1 ... 1

EUREKA! N°10, 2001


58
Sociedade Brasileira de Matemática

Além disso, para todo p primo, mdc(n, p) = 1 se n < p, logo a p-ésima linha é da
forma 1 1 1 ... 1 p, e a p-ésima coluna idem quando tomamos n = p
Logo, fazendo Lp = Lp – L1 vemos que det(p) = (p-1) det(p-1), sendo det(p) o
determinante desejado quando o número de linhas é primo.
Obs: Fazer Lp = Lp – L1 significa trocar a linha p do determinante pela
combinação linear (linha p – linha 1). Essa notação será usada outras vezes no
decorrer dessa resolução.
Calculemos então o determinante para alguns valores pequenos de n:
det(2) = 1.det(1) = 1;
det(3) = 2.det(2) = 2;
Para calcular det(4), note que podemos fazer L4 = L4 – L2 para concluir que det(4)
= 2.det(3) = 4, e portanto det(5) = 4.det(4) = 16; det(6) é um pouco mais difícil de
calcular, mas notando a similaridade da L6 com L2 e L3, pode-se perceber que
fazendo L6 = L6 – L3 + L1 – L2 reduzimos a ordem do determinante de forma que
det(6) = 2.det(5) ( pois a6,6 vira 6 – 3 + 1 – 2 = 2 e para i < 6, a6,i = 0). Daí, det(7)
= 6.det(6) também fica determinado.
Para det(8) é fácil ver que a combinação linear a ser utilizada é L8 = L8 – L4 e para
det(9), a combinação linear é L9 = L9 – L3.
É razoável supor que sempre é possível encontrar uma combinação linear que
reduza de 1 a ordem do determinante. Note que os números das linhas utilizadas
nas combinações lineares sempre foram divisores do número de linhas do
determinante correspondente ( por exemplo, no caso n = 6 usamos as linhas 1, 2,
3 e 6 ).
Mais do que isso, os sinais que antecedem as linhas nas combinações lineares
parecem estar ligados à função  de Möbius!
De fato, no caso do 6, temos: L6 =  (6) L6 +  (3)L3 +  (2)L2 +  (1)L1.
Entretanto, essa combinação linear falha para L8. No caso, entre os divisores de 8,
apenas 1 e 2 têm  não nulo. Mas estamos usando exatamente L4 e L8. Uma
maneira de satisfazer esse caso e o do 6 é fazer:
8 8
L8 =    L8 +    L4 = L8 – L4. Note que para n = 6 temos:
8  4
 6  6 6  6
L6 =    L6 +    L3 +    L2 +    L1 = L6 – L3 – L2 + L1, como
 6  3  2 1
desejávamos.
n
Tentaremos então provar que fazer a substituição Ln =   Ld é suficiente
d |n d
para reduzir a ordem ( note que para d = n isso dá Ln).
Temos então que as entradas da última linha zeram, com exceção daquele que
também está na última coluna do determinante.

EUREKA! N°10, 2001


59
Sociedade Brasileira de Matemática

De fato, consideramos a coluna x, x n. ( que na última linha equivale à entrada


mdc(x, n))
n
Quero calcular    .mdc ( x,d ).
d |n d
Vamos tentar usar a fórmula da inversão de Möbius (no caso, g(d) = mdc(x, d)).
Se acharmos uma função “f” tal que g ( n )  f  d  n  0 , o
d |n

n
problema termina porque aí f ( n )   .g  d  .
d |n d

 f  d  0 se d não divide x
Tomando então f : N  N, , sendo  a função de Euler

 f  d    d  se d divide x
temos:
d  =
 f d   
d |n d |n
 d 
d |mdc ( x ,n )
(***)

mdc(x, n) = g(n)  n .
d |x
E se x < n, temos que n não divide x, e portanto f  n  0 . Logo,
n
   .mdc( x , d )  f  n  0 para x < n. Se, entretanto, x = n, então
d |n d
n
f  n    n  ( pois n divide x = n ) e   d .mdc( n , d )   n 
d |n
portanto,

a última linha passa a ser da forma 0 0 0 0 ...   n  e então


det  n    n  . det  n  1 Logo,
n
det  n     i   1 .  2  .  3 ...  n  .
i 1

(***) pois temos como conseqüência da fórmula da inversão de Möbius,


   d  n .
d |n

51) Três feirantes foram vender melancias. Um levou 10; outro 16; o terceiro,
26. Todos venderam algumas melancias pelo mesmo preço até o meio dia. Depois
disso, os três baixaram o preço, mas continuaram vendendo por preços iguais.
Quando voltaram para casa, após venderem todas as melancias, cada um tinha a

EUREKA! N°10, 2001


60
Sociedade Brasileira de Matemática

mesma quantia de dinheiro; 35 mil cruzeiros. Por quanto foi vendida cada
melancia antes e após o meio-dia?

Solução de Diêgo Veloso Uchôa (Teresina – PI)


Sejam A1 , A2 e A3 os vendedores que venderam 10, 16 e 26 melancias
respectivamente, x i a quantidade de melancias que o vendedor Ai vendeu antes
do meio dia e y o preço antes do meio dia e y1 o preço após o meio dia.
Portanto, teremos:

 I) y x1  (10  x1 ) y1 35000



 II) y x2  (16  x2 ) y1 35000
 III) y x  (26  x ) y 35000
 3 3 1
De I) e II)
 y ( x1 )  (10  x1 ) y1  y x 2  (16  x 2 ) y1  y ( x1  x 2 ) (6  x1  x 2 ) y1
Analogamente de II), III) e I), III) teremos y ( x1  x3 ) (16  x1  x3 ) y1 e
y ( x 2  x3 ) (10  x 2  x3 ) y1

x1  x2 w1 y w1 (6  w1 ) y1


Faça x1  x3 w2  y w2 (16  w2 ) y1 
x2  x3 w3 y w3 (10  w3 ) y1
y w1 (6  w1 ) y1 16  w2 6  w1 16 w2 6 w1
       
y w2 (16  w2 ) y1 w2 w1 w2 w2 w1 w1
16 6 8 3
   ; Imagine, agora, essas duas frações nas suas formas
w2 w1 w2 w1
irredutíveis e como os seus numeradores deverão ser iguais e o número 8 não tem
o fator “3” então w1 é múltiplo de "3"  w1 3k ( com k  N) se
k 1  w1 6  w2 16 (Absurdo! pois w2  x1  x 2  10, pois x1  10) logo
k = 1 e w1 3  w2 8 e w3 5 . Como todos os vendedores venderam

EUREKA! N°10, 2001


61
Sociedade Brasileira de Matemática

algumas melancias antes e após o meio dia temos que 1 x1 9 e 1 x3 25
mas w3  x1  x3 8  x3 1 e x1 9  x 2 6.
Lembrando que y w1 (6  w1 ) y1  y 3 y1 e assim substituindo em I)
3 y1 x1  (10  9) y1 35000  28 y1 35000  y1 1250 cruzeiros e
y 3750 cruzeiros.

Seguimos aguardando a solução do problema 48 publicado na EUREKA! No. 9


Agradecemos também o envio das soluções e a colaboração de:

Carlos A.Gomes (Natal – RN)


Carlos Alberto da Silva Victor (Nilópolis – RJ)
Carlos Rodrigo Mendes Lima (Rio de Janeiro – RJ)
Diego Alvarez Araújo Correia (Fortaleza – CE)
Einstein do Nascimento Júnir (Fortaleza – CE)
Fernando Carvalho Ramos (Santa Maria – RS)
Formiga (São Paulo – SP)
Geraldo Perlino (Itaparica da Serra – SP)
Geraldo Perlino Júnior (São Paulo – SP)
Gilbert R. Johnson (EUA)
Luciano Nunes Prudente (Rio de Janeiro – RJ)
Manuel Murillo Tsijli (Costa Rica)
Osvaldo Melo Sponquiado (Olímpia – SP)
Paulo de Souza Sobrinho (Natal – RN)
Rodrigo Roque Dias (São Paulo – SP)
Vicente Wilson Moura Gaeta (Manaus – AM)
PROBLEMAS PROPOSTOS
 Convidamos o leitor a enviar soluções dos problemas propostos e sugestões de novos
problemas para os próximos números.

52) Quatro retas se interceptam formando quatro triângulos conforme a


figura abaixo.

EUREKA! N°10, 2001


62
Sociedade Brasileira de Matemática

F
C
B

a) Prove que as circunferências circunscritas aos quatro triângulos possuem um


ponto em comum.
b) Prove que os centros dessas quatro circunferências são concíclicos (i.e. existe
uma circunferência que passa por todos eles).

53) Prove que num círculo convexo dado e para o mesmo número de lados. O
polígono regular inscrito é aquele cuja superfície é máxima.

x
54) Sejam ( x n ) a seqüência definida por x1 2, x n 1 2 n , n 1, e ( y n ) a
2001
seqüência definida por y1 2001, y n 1 2001( yn )
, n 1. Prove que
existe c natural tal que y n  x n c para todo n  N e determine o menor c
com essa propriedade.

55) Seja S o conjunto de pontos interiores de uma esfera de raio 1 e C o conjunto


de pontos interiores de um círculo também de raio 1. Existe alguma função
f : S  C tal que d ( A, B ) d ( f ( A), f ( B )) para quaisquer pontos
A, B  S? (d (A, B) denota a distância euclidiana entre A e B).

Problema 52 proposto por Carlos Alexandre Gomes da Silva (Natal – RN), problema 53
proposto por Luis Farias Maia (Fortaleza – CE)
AGENDA OLÍMPICA

XXIII OLIMPÍADA BRASILEIRA DE MATEMÁTICA

EUREKA! N°10, 2001


63
Sociedade Brasileira de Matemática

NÍVEIS 1, 2 e 3
Primeira Fase – Sábado, 9 de junho
Segunda Fase – Sábado, 1 de setembro
Terceira Fase – Sábado, 20 de outubro (níveis 1, 2 e 3)
Domingo, 21 de outubro (níveis 2 e 3 - segundo dia de prova).

NÍVEL UNIVERSITÁRIO
Primeira Fase – Sábado, 1 de setembro
Segunda Fase – Sábado, 20 e Domingo, 21 de outubro

VII OLIMPÍADA DE MAIO
Sábado, 12 de maio

XLII OLIMPÍADA INTERNACIONAL DE MATEMÁTICA
1 a 14 de julho
Washington, Estados Unidos

IV OLIMPÍADA IBEROAMERICANA DE MATEMÁTICA UNIVERSITÁRIA
Sábado, 6 de outubro



Errata: O leitor Diego Alvarez Araújo Correia, de Fortaleza – CE, notou um erro
tipográfico na página 45 da revista Eureka N o. 8. O enunciado correto do Teorema
2 (Fórmula de inversão de Möbius) é o seguinte:
Se para todo n > 0 temos g ( n)  f ( d ) então, para todo n,
d |n

f ( n)   ( n / d ) g (d ).
d |n

COORDENADORES REGIONAIS

Amarísio da Silva Araújo (UFV) Viçosa – MG


Alberto Hassen Raad (UFJF) Juiz de Fora – MG

EUREKA! N°10, 2001


64
Sociedade Brasileira de Matemática

Angela Camargo (Centro de Educação de Adultos – CEA) Blumenau – SC


Benedito Tadeu Vasconcelos Freire (UFRN) Natal – RN
Carlos Frederico Borges Palmeira (PUC-Rio) Rio de Janeiro – RJ
Claudio Arconcher (Colégio Leonardo da Vinci) Jundiaí – SP
Claus Haetinger (UNIVATES) Lajeado – RS
Cleonor Crescêncio das Neves (UTAM) Manaus – AM
Élio Mega (Colégio Etapa) São Paulo – SP
Kátia Gonçalves de Faria (Colégio Singular) Santo André – SP
Florêncio Ferreira Guimarães Filho (UFES) Vitória – ES
Gisele de Araújo Prateado Gusmão (UFGO) Goiânia – GO
Ivanilde Fernandes Saad (UC. Dom Bosco) Campo Grande– MS
Jacqueline Fabiola Rojas Arancibia (UFPB) João Pessoa – PB
João Benício de Melo Neto (UFPI) Teresina – PI
João Francisco Melo Libonati (Grupo Educacional Ideal) Belém – PA
Irene Nakaoka (UEM) Maringá – PR
José Carlos Pinto Leivas (UFRG) Rio Grande – RS
José Cloves Saraiva (UFMA) São Luis – MA
José Gaspar Ruas Filho (ICMC-USP) São Carlos – SP
José Luiz Rosas Pinho (UFSC) Florianópolis – SC
José Vieira Alves (UFPB) Campina Grande – PB
Marcelo Rufino de Oliveira (Sistema Titular de Ensino) Belém – PA
Licio Hernandes Bezerra (UFSC) Florianópolis – SC
Luzinalva Miranda de Amorim (UFBA) Salvador – BA
Marcondes Cavalcante França (UFC) Fortaleza – CE
Pablo Rodrigo Ganassim (Liceu Terras do Engenho) Piracicaba – SP
Paulo Henrique Cruz Neiva de Lima Jr. (Escola Técnica Everardo Passos) SJ dos Campos – SP
Reinaldo Gen Ichiro Arakaki (INPE) SJ dos Campos – SP
Ricardo Amorim (Centro Educacional Logos) Nova Iguaçu – RJ
Roberto Vizeu Barros (Colégio Acae) Volta Redonda – RJ
Sérgio Cláudio Ramos (IM-UFRGS) Porto Alegre – RS
Silvio de Barros Melo (UFPE) Recife – PE
Tadeu Ferreira Gomes (UEBA) Juazeiro – BA
Tomás Menéndez Rodrigues (U. Federal de Rondônia) Porto Velho – RO
Valdenberg Araújo da Silva (U. Federal de Sergipe) São Cristovão – SE
Wagner Pereira Lopes (Escola Técnica Federal de Goiás) Jataí – GO
Waldemar M. Canalli (Prefeitura Municipal de S. João de Meriti) SJ de Meriti – RJ

EUREKA! N°10, 2001


65

 

  
  !" $# #$
%   &
'()*(*+-, .*/*01
  !" $# #$
%   2
3415)*6 7 .*/*0!89:.15, 6 4*, 9;0
<=  !" $# #$
% >?A@ B? #$CBD$EF G
'()*(*+-, .*/*014%34*15)6 7 .*/0%89;.*1H, 6 4, 9:0
<= *> %" ?#$  D>?D $B D I#! >%@>? B J*K
'()*(*+-, .*/*014%34*15)6 7 .*/0%89;.*1H, 6 4, 9:0
LM=N$O:PQR
SE#ET ?D$U J*G
V .9:6 01XW*)*Y-0AZ[*, (4
? \D%* E#?] \F$ # #$^`_! $#$ !" $# $ *a
3.b .*46dcH.feg9:.!h0*(*7 4*6 4*1
ijk?% j &*a
'/)*.*9;/*0
cl4(*m.*(
> !" #? $ n?]#?#C!FD$# ao
>Fp?q`E#E`_?>! $E`*` K J
_r! $$`* K s
\D$# >" %jB 2 o
B#$D$ ?#$\ D ?  2 J
Sociedade Brasileira de Matemática
t uwvxy zu

É com grande satisfação que informamos a excelente participação do


Brasil na IMO. Pela primeira vez, os seis alunos da equipe ganharam medalhas,
sendo 4 de prata e 2 de bronze. Na soma dos pontos, o Brasil ficou à frente de
mais de 80% das nações representadas, incluindo muitos países de grande
tradição olímpica. Veja a prova nesta edição e mais detalhes sobre a IMO no site

http://imo.wolfram.com/

Isso só nos estimula ainda mais a fazer uma Eureka cada vez melhor e
que atinja um público cada vez maior. Gostaríamos de agradecer mais uma vez o
envio de grande número de problemas e soluções pelos leitores, o que é
importante para a Eureka!, e nos anima a continuar o trabalho de proporcionar
diversão e material de treinamento à nossa comunidade olímpica. Gostaríamos
ainda de dizer que continuamos contando com o envio de artigos pelos leitores.
Artigos para iniciantes são prioritários. São também prioritários artigos sobre
temas olímpicos que até hoje não apareceram na Eureka!, como Contagem,
Grafos (nível avançado), Funções (nível intermediário), Trigonometria aplicada à
Geometria (pode ser uma compilação de problemas). Traduções (devidamente
autorizadas, é claro) de artigos de boas revistas (com o mesmo perfil, é claro)
serão muito bem recebidas.

Finalmente agradecemos a valiosa ajuda dos professores Eduardo Tengan


e Carlos Shine e dos estudantes Alex Cardoso Lopes, Guilherme I. C. Fujiwara e
Rodrigo K. Yamashita na revisão da revista.

{x|}>~€ƒ‚„…}>|†

EUREKA! N°11, 2001

2
Sociedade Brasileira de Matemática
‡ yˆyvAyˆ‰‹Š
Œ t t w‰ t yˆ
$Žg‘$Ž {‹’`“” ‘$•
_r! –
Sara escreveu no quadro negro um número inteiro de menos de trinta algarismos
e que termina em 2.
Célia apaga o 2 do fim e escreve-o no início.
O número que fica é igual ao dobro do número que tinha escrito Sara.
Qual é o número que Sara escreveu?
_r! —
Vamos pegar um retângulo ABCD de papel; o lado AB mede 5 cm e o lado BC
mede 9 cm.
Fazemos três dobras:
1- Levamos o lado AB sobre o lado BC e chamamos de P o ponto do lado BC
que coincide com A. Forma-se então um trapézio retângulo BCDQ.
2- Dobramos de forma que B e Q coincidam. Forma-se um polígono de 5 lados
RPCDQ.
3- Dobramos de novo fazendo coincidir D com C e Q com P. Forma-se um
novo trapézio retângulo RPCS.
Após fazer estas dobras, fazemos um corte perpendicular a SC pelo seu ponto
médio T, obtendo o trapézio retângulo RUTS.
Calcule a área da figura que aparece ao desdobrarmos o último trapézio RUTS.
_r! ˜
Temos três caixas, uma azul, uma branca e uma vermelha, e 8 bolinhas. Cada
bolinha tem um número de 1 a 8, sem repetições. Distribuímos as 8 bolinhas nas
caixas, de maneira que há pelo menos duas bolinhas em cada caixa. Logo, em
cada caixa, somam-se todos os números escritos nas bolinhas contidas na caixa.
Os três resultados denominam-se soma azul, soma branca e soma vermelha,
segundo a cor da caixa correspondente. Encontre todas as possíveis distribuições
das bolinhas tais que a soma vermelha seja igual ao dobro da soma azul, e a soma
vermelha menos a soma branca seja igual à soma branca menos a soma azul.
_r! ™
Utilizando exclusivamente números primos forma-se um conjunto com as
seguintes condições:
1- Qualquer número primo de um algarismo pode estar no conjunto.
2- Para que um número primo de mais de um algarismo esteja no conjunto,
devem estar no conjunto o número que se obtém ao suprimir-lhe só o

EUREKA! N°11, 2001

3
Sociedade Brasileira de Matemática

primeiro algarismo e também o número que se obtém ao suprimir-lhe só o último


algarismo.

Determine, entre conjuntos que cumpram estas condições, aquele que tem maior
quantidade de elementos. Justifique por que não pode haver um com mais
elementos.
Lembre-se de que o número 1 não é primo.
_r! š
Num tabuleiro de 8 casas, como na figura abaixo, há inicialmente uma ficha em
cada casa.
Uma jogada consiste em escolher duas fichas e mover uma delas uma casa à
direita e a outra, uma casa à esquerda.
Se depois de 4 jogadas as 8 fichas estão distribuídas somente em 2 casas,
determine quais podem ser estas casas e quantas fichas há em cada uma delas.

› ‘$œx ’`ž`{‹’`“” ‘$•


_r! –
Na minha calculadora, uma das teclas de 1 a 9 está com defeito: ao pressioná-la
aparece na tela um dígito entre 1 e 9 que não é o correspondente.
Quando tentei escrever o número 987654321, apareceu na tela um número
divisível por 11 e que deixa resto 3 ao ser dividido por 9.
Qual é a tecla defeituosa? Qual é o número que apareceu na tela?
_r! —
No trapézio ABCD, o lado DA é perpendicular às bases AB e CD. A base AB
mede 45, a base CD mede 20 e o lado BC mede 65. Seja P no lado BC tal que BP
mede 45 e seja M o ponto médio de DA.
Calcule a medida do segmento PM.
_r! ˜
Num tabuleiro de 3 fileiras e 555 colunas, pintam-se de vermelho 3 casas, uma
em cada uma das 3 fileiras.
Se escrevemos nas casas, ordenadamente por fileiras, da esquerda para a direita,
os números de 1 a 1665 (na primeira fileira de 1 a 555, na segunda de 556 a 1110

EUREKA! N°11, 2001

4
Sociedade Brasileira de Matemática

e na terceira de 1111 a 1665) há 3 números que ficam escritos nas casas


vermelhas.
Escrevemos nas casas, ordenadamente por colunas, de cima para baixo, os
números de 1 a 1665 (na primeira coluna de 1 a 3, na segunda de 4 a 6, na terceira
de 7 a 9,…, e na última de 1663 a 1665) há 3 números que ficam escritos nas
casas vermelhas.
Chamamos números vermelhos aos que em alguma das duas distribuições ficam
escritos nas casas vermelhas.
Diga quais são as 3 casas que devemos pintar de vermelho para que existam só 3
números vermelhos.
Mostre todas as possibilidades.
_r! ™
Em volta de um círculo situam-se dez moedas de 1 cm de raio como indicado na
figura abaixo. Cada moeda é tangente ao círculo e às duas moedas vizinhas.
Demonstre que a soma das áreas das dez moedas é o dobro da área do círculo.

_r! š
No quadro negro estão escritos os números naturais desde 1 até 2001, inclusive.
Temos que apagar alguns números de modo que entre os que ficam sem apagar
seja impossível escolher dois números distintos tais que o resultado de sua
multiplicação seja igual a algum dos números que ficam sem apagar.
Qual é a quantidade mínima de números que devem ser apagados? Para esta
quantidade, apresente um exemplo que mostre quais números são apagados.
Justifique por que não obtemos a propriedade desejada se apagarmos menos
números.

EUREKA! N°11, 2001

5
Sociedade Brasileira de Matemática
‡ yŸy‹ vAyˆ‰‹Š
Œ t t ¡¢‰ t yˆ
£]¤>¥§¦l¨:© ª>«d¬
­$®ƒª¥*¯ ¨ ¤>¯ ®ƒ¬

$Ž`‘$gŽ {‹’`“” ‘$•


° ± ²-³*´:µ¶%µ·%°?·*¸5·¹*µ*·%º¶*»¸5³ ¼%·µ*³*½ ¾³%µ·%¿»*´;¶ À¶*± Á¹*± ³!Â>À¿
°¶µ*´;± Ã*¶!Ä*Å*²-± ¶%µ·%ƳǟÈÉ´;¶ %¼ ·µ*³*½ ¾³%µ·%Ê´;³fÈɳ À¶*± Á¹*± ³!Â>À¿
Ë?¾*± ³Ã*¶!µ*·!ʳ*»*½ ³!À³´:²5± ³!Ƴ̱ ÍηfÈɳ ¼%·µ*³*½ ¾³%µ·%Ê´;³fÈɳ Æ?¶*½ ³fȃ± ¹*³!Â>Ϻ
°»µ*Ð!¼%¶´:·± ´:³!µ*·!Ä*± Ñ!³%·!º± ½ Òd³ ¼!·*µ*³½ ¾*³!µ*·!Ó´:¶¹*¸5· Ô?¹*³*ÕÂ>¼!À
Ë?·*½ Ñ!¶%Ä»*± Çƶ*´;´:·*± ³!Ö5Ź*± ¶*´ ¼!·*µ*³½ ¾*³!µ*·!Ó´:¶¹*¸5· º³¹fÈɶ$×=¹*µ´:Ø!Â>ºÊ
°³Ù*¾³*·*½-°¶*µ´:± Ã*»·*Ǽ%³ÚÈɳ ¼!·*µ*³½ ¾*³!µ*·!Ó´:¶¹*¸5· º³½ Òd³*µ¶*´Â>ÓÌ×
Ë?¾*¶Ñ%ÐÇÛ± Üd¶Ç5¾*±Hº³Ç5³*Ý-±5Û¶ÌÇ5¾± ¹*³ ¼!·*µ*³½ ¾*³!µ*·!Ó´:¶¹*¸5· °?± ¶%µ·%Ö5³¹*·± ´:¶!Â>°Ö
°¶µ*¶½ Þɶ%º³¹fÈɶÇ`ƶǟÈɳ!¼%³ß5³*´;³*¹*µ»*à³ ¼%·¹*ß5á¶%Û?¶*¹*´;¶*Ç5³ À¶*± Á¹*± ³!Â>À¿
ij*± ÇÔÜd·*µ³â×± Òd³¸5¶*ý ¶fÒ ¼!·*¹*ß-á*¶!Û¶*¹´:¶*Ç-³ ºá¶%Ö-¶*Ç5Ø!µ*¶Ç`Ƴ*Ñ!Ù*¶ÇºÊ
ã·*½ ± Ù·%À¶*¹*ß-³*½ Òd·Çä×=Ç5Ç5± Ç ¼%·¹*ß5á¶%Û¶¹*´;¶*Ç5³ Æ?³*Ñ!Ù*± ¹*³!À´;³*¹µ*·!Â>ÊÓ

› ‘$œx ’`ž`{‹’`“” ‘$•


×=½ ·ÚÍXƶ´:´:å³â×=à´:·» ¼%·µ*³*½ ¾³%µ·%¿»*´;¶ æ?± ÈÉ·*´;ç*±5°Ö
ij*´:± Ç-Ç5³%Æ?³fÒd³½ ²5³*¹ÚÈÉ·%èé*µ·%Ä̱ Ñ!³ %¼ ·µ*³*½ ¾³%µ·%Ê´;³fÈɳ ã¶*´ Èɳ*½ ·¸5³!Â>ÆÏ
À»*± ½ ¾*·´:Ñ!·%°?¶*µ*´;± Ã*»·*Ǻ³½ ·*´;Ñ%¶ ¼%·µ*³*½ ¾³%µ·%Ê´;³fÈɳ À¶*± Á¹*± ³!Â>À¿
¿Èɳ*²5Õ ½ ± ¶âË?¶*´;´:·*Çê± ½ ³*Ç`Ó¶³*Ç ¼!·*µ*³½ ¾*³!µ*·!Ó´:¶¹*¸5· º³½ Òd³*µ¶*´Â>ÓÌ×
ë Ç-´:³*·½5ã´:³*¹Ý5½ ± Ñíì¶*»´:³µ*¶ ¼!·*µ*³½ ¾*³!µ*·!Ó´:¶¹*¸5· ã¶*´ Èɳ*½ ·¸5³!Â>ÆÏ
Ä»*± Çϵ»*³´:µ*¶!µ*·!À¶µ*¶± ¼!·*µ*³½ ¾*³!µ*·!Ó´:¶¹*¸5· ºá¶%Ö-¶*Ç5Ø!µ*¶Ç`Ƴ*Ñ!Ù*¶ÇºÊ
ãÐ*à± ¶%ì± ³Ç¼!¶*´;·*± ´:³ ¼!·*µ*³½ ¾*³!µ*·!Ó´:¶¹*¸5· °?± ¶%µ·%Ö5³¹*·± ´:¶!Â>°Ö
×=¹µ*´:ر ³%ÄÅ*²5± ¶!µ*·!ƳǟÈÉ´:¶ ¼%·¹*ß5á¶%Û¶¹*´;¶*Ç5³ À¶*± Á¹*± ³!Â>À¿
Û·¹*´ Üîï·±5ƾ*·¹*Ã%ÛÇ5» ¼!·*¹*ß-á*¶!Û¶*¹´:¶*Ç-³ ºá¶%ʳ»*½ ¶!ÂrºÊ
ì³ÚÞɹ*·!µ*·$×=½ à*»ð*»·*´:ð»*·%º± Ñ%ᶠ¼!·*¹*ß-á*¶!Û¶*¹´:¶*Ç-³ ã¶*´ Èɳ*½ ·¸5³!Â>ÆÏ

EUREKA! N°11, 2001

6
Sociedade Brasileira de Matemática
ñ yˆyv%yŸ‰‹ŠŒ t t w‰ t E
‰nòEyˆ t ó ¢uô¡v
Enunciados e Resultado Brasileiro

A XII Olimpíada de Matemática do Cone Sul foi realizada na cidade de


Santiago do Chile no período de 1 a 6 de julho de 2001. Dela participaram alunos
de até 15 anos dos seguintes países: Argentina, Brasil, Chile, Equador, Peru e
Uruguai.
A equipe brasileira foi selecionada através de provas realizadas em março
e maio deste ano e foi liderada pelos professores Élio Mega e Carlos Yuzo Shine
de São Paulo - SP.
{ Ž }|õ]ö€ƒ÷~‚¡~÷ ‘$ø õù}
ú`„û÷>|ög}d„…÷
ú Žüý ‘ þ|>€ƒ}þ¡~‚ÿ’`÷|ÿ}þ€ƒ‚„…† 
ú Ž ü  œ õö
}>„ ÿ} õjx÷„…÷ $Ž`ü ]ü 
ú Ž ü  • ÷„…|>| ÷ `÷ ÷>ö d÷>þ€ƒ} Aõ}>„…‚ x~} • ÿ÷ $Ž`ü ]ü
ú Ž ü  Ž ÷ ƒ÷}>öž`÷ ‚ `„…÷ ¡÷ { Ž {
$Ž`‘$gŽ {‹ž ü
ìÔ?°r!× "¿ # & ¾*¶´:³Ç·!Ñ%·± ³*é
_r! –
Em cada casa de um tabuleiro quadriculado 2000 × 2000 deve-se escrever um dos
três números: –1, 0 ou 1. Se, em seguida, somam-se os números escritos em cada
linha e cada coluna, obtêm-se 4000 resultados. Mostre que é possível preencher o
tabuleiro de modo que os 4000 resultados assim obtidos sejam todos distintos.
_r! —
Tem-se uma sucessão a1, a2, a3, ..., an, ... de números inteiros positivos, com as
seguintes propriedades:
i) Todo número inteiro positivo aparece uma ou mais vezes na sucessão.
ii) a1 = 1
iii) a3n+1 = 2an + 1
iv) an+1 ≥ an
v) a2001 = 200

Calcule o valor de a1000.

Obs. O enunciado deste problema está incorreto, pois na verdade não existe tal
sequência (tente demonstrar isto!). Entretanto, se suprimirmos a condição i) é
possível resolver o problema (tente fazer isto também!).

EUREKA! N°11, 2001

7
Sociedade Brasileira de Matemática

_r! ˜
Três triângulos acutângulos estão inscritos em uma mesma circunferência, de
modo que seus vértices são nove pontos distintos. Demonstre que se pode
escolher um vértice de cada triângulo de maneira que os três pontos escolhidos
determinem um triângulo cujos ângulos sejam menores que ou iguais a 90o.

› ‘$œx ’ž`{‹ž ü


ìÔ°r×!"¿$# & ¾*¶´:³*Ç·%Ñ!·*± ³é

_r! ™
Um polígono de área S está contido no interior de um quadrado de lado a.
Demonstre que há pelo menos dois pontos do polígono que estão separados por
uma distância maior que ou igual a Sa .

_r! š
Ache todos os números inteiros positivos m tais que m + 2001⋅S(m) = 2m onde
S(m) representa a soma dos algarismos de m.

_r! &%
Seja g uma função definida para todo inteiro positivo n, que satisfaz

i) g(1) = 1
ii) g(n + 1) = g(n) + 1 ou g(n + 1) = g(n) – 1 para todo n ≥ 1
iii) g(3n) = g(n) para todo n ≥ 1
iv) g(k) = 2001 para algum inteiro positivo k.

Ache o menor valor possível de k entre todas as funções g que cumprem as


condições anteriores e demonstre que é o menor.

EUREKA! N°11, 2001

8
Sociedade Brasileira de Matemática
>Fp('$ # `_! –
Antes de procurarmos uma maneira de preencher um tabuleiro 2000 × 2000,
vamos preencher um tabuleiro menor, digamos, 2 × 2.
Uma maneira de preenchê-lo é:

1 –1 → 1 soma: 0

1 0 → 1 soma: 1

↓ ↓
soma: soma:
2 –1

Vamos tentar preencher agora um tabuleiro 4 × 4. Para isso, aproveitamos o


tabuleiro 2 × 2. Para que as somas obtidos continuem iguais, colocamos 1's e –1's
desta forma:
1 –1 1 –1 → soma: 0

1 0 1 – 1 → soma: 1

1 1

–1 –1
↓ ↓
soma: soma:
2 –1

Para completar o trabalho, basta preencher o subtabuleiro 2 × 2 do canto inferior


direito:
1 –1 1 –1 → soma: 0

1 0 1 – 1 → soma: 1

1 1 1 0 → soma: 3

1 –1 → soma: –2
–1 –1
↓ ↓ ↓ ↓
soma: soma: soma: soma:
2 –1 4 –3

EUREKA! N°11, 2001

9
Sociedade Brasileira de Matemática

Observe que podemos preencher um tabuleiro (2k + 2) × (2k + 2) a partir de um


tabuleiro 2k × 2k. Digamos que no tabuleiro 2k × 2k as somas sejam iguais a
– (2k – 1), – (2k – 2), …2k. Para preencher um tabuleiro (2k + 2) × (2k + 2),
colocamos o tabuleiro 2k × 2k no canto superior esquerdo, preenchemos as 2k
primerias casas da 2k + 1-ésima linha e da 2k + 1-ésima coluna con 1's e as 2k
primeiras casas da 2k +-ésima linha e da 2k + 2-ésima coluna com –1's. No canto
interior direito preenchemos da mesma forma que anteriormente:
1 –1
1 –1
2k × 2k ) ) (A) Nas linhas e colunas em (A) e (B)
temos as somas de –(2k – 1) a 2k
1 –1
1 1 … 1 1 0 → soma:2k + 1
–1 –1 … –1 1 –1 → soma: –2k

↓ ↓
(B) soma soma
2k + 2 –(2k + 1)

Assim, temos todas as somas de (2k + 1) a 2k + 2. Logo é possível preencher um


tabuleiro 2n × 2n para todo n natural não nulo; em particular, o tabuleiro 2000 ×
2000.
>Fp('$ # `_! —
O enunciado do problema está incorreto: Vamos mostrar que não existe uma tal
sucessão.
Temos que a n +1 = a n ou a n +1 = a n + 1 (observe que, se a n +1 ≥ a n + 2, o número
inteiro positivo a n + 1 não apareceria na sucessão).
O número da forma 3n + 1 mais próximo de 2001 é 2002 = 3⋅ 667 + 1.
Além disso, a 2002 = a 2001 = 200 ou a 2002 = a 2001 + 1 = 201.
Sendo a 2002 = 2a 667 + 1, temos que a2002 é ímpar, logo a2002 = 201. Assim, 2a667 +
1 = 201 ⇔ a667 = 100. Mas 667 = 3 ⋅ 222 +1, e temos 2a222 + 1 = 100 ⇔ a222 =
99
, que não é inteiro. Contradição.
2
Três alunos da delegação brasileira deram uma solução equivalente à anterior.
Alguns alunos argentinos resolveram o problema ignorando a condição (i).
Mostremos que é possível calcular a2000 sem utilizar a condição (i).

EUREKA! N°11, 2001

10
Sociedade Brasileira de Matemática

De (iv), indutivamente mostra-se que a m ≥ a n para m ≥ n.


Aplicando (iii) temos:

a 4 = a 3⋅1+1 = 2a1 + 1 = 2 ⋅ 1 + 1 = 3
a13 = a 3⋅4 +1 = 2a 3 + 1 = 2 ⋅ 3 + 1 = 7
a 40 = a 3⋅13+1 = 2a13 + 1 = 2 ⋅ 7 + 1 = 15
a121 = a 3⋅40 +1 = 2a 40 + 1 = 2 ⋅ 15 + 1 = 31
a 364 = a 3⋅121+1 = 2a121 + 1 = 2 ⋅ 31 + 1 = 63
a1093 = a 3⋅364 +1 = 2a 364 + 1 = 2 ⋅ 63 + 1 = 127

Logo a1000 ≤ a1093 ⇔ a1000 ≤ 127 (I)

Agora estudemos a3. Seja a 3 = k . Temos:


a10 = a 3⋅3+1 = 2a 3 + 1 = 2k + 1
a 31 = a 3⋅10 +1 = 2a10 + 1 = 2(2k + 1) + 1 = 4k + 3
a 94 = a 3⋅31+1 = 2a 31 + 1 = 2(4k + 3) + 1 = 8k + 7
a 203 = a 3⋅94 +1 = 2a 94 + 1 = 2(9k + 7) + 1 = 16k + 15
a 650 = a 3⋅283+1 = 2a 283 + 1 = 2(26k + 15) + 1 = 32k + 31
a 2561 = a 3⋅850 +1 = 2a850 + 1 = 2(32 k + 31) + 1 = 64k + 63

Como a 2551 ≥ a 2001 , temos 64k + 63 ≥ 200 ⇔ k ≥ 3. Mas a 3 ≤ a 4 ⇔ k ≤ 3. Logo


k = 3 e, portanto, a850 = 32 ⋅ 3 + 31 = 127. Desta forma a1000 ≥ a850 ⇔ a100 ≥ 127(II).
De (I) e (II), temos a1000 = 127.

>Fp('$ # `_! ˜


Sejam A1 A2 A3, B1 B2 B3 e C1 C2 C3 os triângulos. Tome o ponto A1 e trace o
diâmetro AA1 que passa por A1. Podemos supor, sem perda de generalidade, que,
dentre os pontos B1, B2, B3, C1, C2 e C3, o mais próximo de A é B1 e que, dentre os

pontos C1, C2 e C3, o mais próximo de A, contido no arco AA1 que contém B1 é
C1.

EUREKA! N°11, 2001

11
Sociedade Brasileira de Matemática

C1
B1

A1 A
O

O diâmetro C1 C que passa por C1 divide a circunferência em dois arcos. Se C1,


C2 e C3 estivessem no arco que contém A1, teríamos que o maior ângulo do
180°
triângulo C1C2C3 seria maior ou igual a = 90° , o que não é possível. Logo
2
existe um ponto, Ci no arco que contém B1. Sendo C1 o mais próximo de A, e
∩ ∩
como AC1 que contém B1, temos que Ci pertence ao arco CA .
Temos então que o triângulo A1B1Ci satisfaz as condições do enunciado, pois
∩ ∩ ∩ ∩
 ∩  m (B A ) m ( AA1 ) ∩ m(Ci A1 ) m( A1 A)
m A1 C i B1  = a 1 1 < a = 90°, m( A1 B1 Ci ) = ≤ = 90°
  2 2 2 2
∩ ∩
∩ m(C i B1 ) m(CC1 )
e m(C i A1 B1 ) = < = 90°. Observe que o triângulo é retângulo
2 2
se, e somente se, Ci = A.

Obs. Como o número total de pontos é 9, que é ímpar, podemos escolher um


deles (que será o A1) cujo antípoda não é nenhum dos outros pontos.
Nossa solução mostra então que é possível obter um triângulo acutângulo AiBjCk.

>Fp('$ # `_! ™


Suponha, por absurdo, que quaisquer dois pontos do polígono estejam separados
por uma distância menor que S/a.

EUREKA! N°11, 2001

12
Sociedade Brasileira de Matemática

y
a x

Consideramos o ponto mais à esquerda e mais à direita x e y do nosso polígono.


Se b é a diferença de suas abscissas, o polígono está contido num retângulo de
lados b e a.
Temos claramente S ≤ ab, donde b ≥ S/a.
Como claramente xy ≥ b, temos um absurdo e o resultado está provado.

>Fp('$ # `_! š


Temos m + 2001 S(m) = 2m ⇔ 2001 S(m) = m. Assim, m é divisível por 3 e
conseqüentemente S(m) também o é. Logo S(m) = 3k, para algum k inteiro, e
m = 2001 ⋅ 3k = 9 ⋅ 667k é divisível por 9. Desta forma, 9 divide S(m).
Seja n o número de algarismos de m. temos que S(m) ≤ 9n (cada algarismo é
menor ou igual a 9), assim 2001S(m) ≤ 18009n ⇔ m ≤ 18009n.
Mas m ≥ 100
, + ...0 = 10 , logo 10 ≤ 18009n.
n n

*
n −1 zeros
Esta última desigualdade só é válida para n ≤ 6. Assim, S(m) ≤ 9 ⋅ 6.
Como S(m) é divisível por 9, temos S(m) = 9, 18, 27, 36, 45 ou 54. Temos
S(m) = 9 ⇒ m = 9⋅2001 = 18009, o que não é possível pois 1+8+0+0+9 = 18.
S(m) = 18 ⇒ n = 18⋅2001 = 36018.
S(m) = 27 ⇒ m = 27⋅2001 = 54027, o que não é possível pois 5+ 4+0+2+7 = 18
S(m) = 36 ⇒ m = 36⋅2001 = 72036, o que não é possível pois 7+2+0+3+6=18
S(m) = 45 ⇒ m = 45⋅2001 = 90045, o que não é possível pois 9+0+0+0+4+5=18
S(m) = 54 ⇒ m = 54⋅2001 = 108054, o que não é possível pois 1+0+8+0+5+4=18

Logo a única solução é n = 36018.

EUREKA! N°11, 2001

13
Sociedade Brasileira de Matemática
>Fp('$ # `_! &%
Queremos encontrar o menor valor de k tal que g(k) = 2001. Assim, seja ak o
menor valor tal que g(ak) = n. Desta forma, devemos calcular a2001.
Temos g(1) = 1, logo a1 = 1. Podemos tomar g(2) = 2, logo a2 = 2. Observe que
nesse caso g(3) = g(1) = 1 e g(6) = g(2) = 2. Podemos tomar g(4) = 2 e g(5) = 3,
logo a3 = 5.
Considere g(k) e g(k + 1). Temos g(3k) = g(k) e g(3k + 3 ) = g(k + 1). Assim,
g(3k + 3) = g(3k) + 1 ou g(3k + 3) = g(3k) – 1. Desta forma, g(3k +1) e g(3k + 2)
são no máximo iguais a g(k) + 1 ou g(k + 1) + 1. Por exemplo, se g(4) = 2 e
g(5) = 3, temos g(12) = 2 e g(15) = 3. Tomando g(13) = 3 e g(14) = 4, temos que
a4 = 14.
Note que, como g(k) ≤ 2 para k ≤ 4, temos g(k) ≤ 3 para k ≤ 12.
Assim, podemos encontrar an + 1 em função de an. Temos que g(an – 1) = n – 1
pois g(an – 1 ) < n (se g(an – 1) > n, existiria k < an tal que g(k) = n, o que
contradiz a hipótese de an ser mínimo. Assim, g(3(an – 1) = n – 1 e g(3an) = n.
Para k ≤ an – 1, temos g(k) ≤ n – 1, logo g(k) ≤ n para k ≤ 3(an – 1). Podemos
tomar g(3an – 2) = n e g(3an – 1) = n + 1. Logo an + 1 = 3an – 1.
Note que temos "quase" uma progressão geométrica. Se somarmos um número x
 x −1
de cada lado da igual dado, temos a n +1 + x = 3a n −1 + x = 3 a n +  . Se
 3 
x −1 1  1  1 1
fizermos x = ⇔ x = − , temos  a n +1 −  = 3 a n −  . Sendo bn = a n − ,
3 2  2  2 2
temos bn+1 = 3bn, ou seja , bn é uma progressão geométrica de razão 3. Logo bn =
1 1
b1 ⋅ 3n – 1. Como b1 = a1 − = , temos
2 2
n −1 n−1
3 1 3 3n−1 + 1 3 2000 + 1
bn = ⇔ an − = ⇔ an = . Para n = 2001, temos a n = .
2 2 2 2 2

Obs. A função g que construímos é tal que g(k) é o maior possível, para todo k, e
é obtida da seguinte forma: Primeiro observamos que todo natural n pode ser
escrito de maneira única como n = 3 k + ε 1 3 k −1 + ε 2 3 k − 2 + ... + ε k −1 ⋅ 3 + ε k , com
3k + 1 3 k +1 + 1
ε j ∈{−1,0,1} para 0 ≤ j < k (onde k é tal que ≤n< ) . Temos então
2 2
k −1
g (n) = 1 + ∑ε
j =0
i ,ou seja, g(n) é o número de termos não nulos na representação

acima.

EUREKA! N°11, 2001

14
Sociedade Brasileira de Matemática
ñ vxyÉy` vAyˆ‰‹Š
Œ t t yŸz t yŸ t v w‰ t E‰nò yˆ t
Enunciados e Resultado Brasileiro

A XLII Olimpíada Internacional de Matemática foi realizada na cidade


de Washington – DC, USA no período de 1 a 14 de julho de 2001 e teve a
participação de 85 países.
A equipe brasileira foi selecionada através de provas realizadas em março
e maio deste ano e foi liderada pelos professores Nicolau C. Saldanha (Rio de
Janeiro – RJ) e Antonio Caminha Muniz Neto (Fortaleza - CE).
{ Ž }|õ]ö€ƒ÷~‚¡~÷ ‘$ø õù}
ú`„û÷>|ög}d„…÷
ú Žüý ü ö }- `‚„…„/.>÷ ü10 „…}>õ  „…÷€ƒ÷
ú Ž ü  `÷„…ö‚|
›$€ƒ}þ¡’`÷}>|~]}ú`„…€ƒ‚  „…÷€ƒ÷
ú Ž ü  
÷ ‚ÿú`÷>„û„…‚| Ž ‚~„… õ]} | ‚]|€ƒ÷ ú`„û‚]þ }
ú Ž ü  ` õ  0 }>„…€ƒ‚ÿ›$ö >÷’`÷ }|  „…÷€ƒ÷
ú Ž ü 2 ž`÷   3 - ¡‚ ü ö} ->÷þ~]„ûgþ‚ÿ’`‚ õ]}„…÷  „…÷€ƒ÷
ú Ž ü 4 ž`÷þ}>ö  5þ
}„…‚¡›$‚ 0 „…}„…÷ ú`„û‚]þ }

$Ž`‘$gŽ {‹ž  ü
ìÔ?°r!× "¿# a ¾*¶ ´:³Ç·!Ñ%·± ³*é

_r! –
Seja ABC um triângulo acutângulo com circuncentro O. Seja PA uma altura do
triângulo com P no lado BC.
Considere que BCˆ A ≥ ABˆ C + 30° .
Prove que CAˆ B + COˆ P < 90° .

_r! —
Prove que
a b c
+ + ≥1
a + 8bc
2
b + 8ca
2
c + 8ab
2

para quaisquer números reais positivos a, b, e c.

EUREKA! N°11, 2001

15
Sociedade Brasileira de Matemática
_r! ˜
Vinte e uma meninas e vinte e um meninos participaram numa competição
matemática.
• Cada participante resolveu no máximo seis problemas.
• Para cada menina e cada menino, existe pelo menos um problema que foi
resolvido por ambos.
Prove que existe um problema que foi resolvido por pelo menos três meninas e
pelo menos três meninos.
› ‘$œx ’ž`{‹ž ü
ìÔ°r×!"¿$# a ¾*¶´:³*Ç·%Ñ!·*± ³é
_r! ™
Seja n um inteiro ímpar maior do que 1 e sejam k1 , k 2 ,..., k n inteiros dados. Para
cada uma das n! permutações a = (a1 , a 2 ,..., a n ) de {1,2,..., n}, defina
n
S (a) = ∑k a .
i =1
i i

Prove que existem duas permutações b e c, b ≠ c, tais que n! é um divisor de


S (b) − S (c).

_r! š
Num triângulo ABC, seja AP a bissectriz de BAˆ C com P no lado BC, e seja BQ a
bissectriz de ABˆ C com Q no lado CA.
Sabemos que BAˆ C = 60° e que AB + BP = AQ + QB.
Quais são os possíveis valores dos ângulos do triângulo ABC?

_r! &%
Sejam a, b, c, d inteiros com a > b > c > d > 0. Considere que

ac + bd = (b + d + a − c)(b + d − a + c).

Prove que ab + cd é um número primo.

EUREKA! N°11, 2001

16
Sociedade Brasileira de Matemática
t Ezz ¢ 76 t 8y
9 ª>®ƒ¨ ¬>¥;:$¦=< ¬?>A@¯ B¤?C 9 ¬>¨ DE¯ ¬?F$© ªG>ª
$®f© ¯ E>¬I>ªd¥*¤ª«>¬I¤J ª¦5¨ ª JI¯ B¯ ¥© ®ƒª>«ªKB>ªML/NO>r¤J
ªPBª?Q=¨SR J Gd¯ T*ªU(>rª¨SVfªd«>¬®$CÌ­WH
H

♦ XZY []\^`_ ab cdb ePaf\(g

A Torre de Hanói é um dos quebra-cabeças matemáticos mais populares. Ele foi


inventado por Edouard Lucas em 1883.
ý †  }h>÷|
As peças são n discos de tamanhos diferentes e todos com um furo em seu centro
e três pinos onde são colocados os discos. Certamente podem ser encontrados em
qualquer loja de brinquedos.
 † Ž }„…÷|
}
‚ 0 }>€ƒ‚|
~‚g‚‚
Inicialmente os discos formam uma torre onde todos são colocados em um dos
pinos em ordem decrescente de tamanho.

Devemos transferir toda a torre para um dos outros pinos de modo que cada
movimento é feito somente com um disco, nunca havendo um disco maior sobre
um disco menor.
 † ü }„ õþ]€ƒ÷ ø õ]}|>}„ 3 >÷dö÷>~÷
Queremos saber qual é o menor número de movimentos necessários para resolver
uma torre de Hanói com n discos.

Há uma história (imaginada pelo próprio Edouard Lucas) sobre a torre de Hanói:

No começo dos tempos, Deus criou a Torre de Brahma, que contém três pinos de
diamante e colocou no primeiro pino 64 discos de ouro maciço. Deus então
chamou seus saserdotes e ordenou-lhes que transferissem todos os discos para o
terceiro pino, seguindo as regras acima. Os sacerdotes então obedeceram e
começaram o seu trabalho, dia e noite. Quando eles terminarem, a Torre de
Brahma irá ruir e o mundo acabará.

EUREKA! N°11, 2001

17
Sociedade Brasileira de Matemática
 † ‘ |€ƒõ~]÷þ~‚¡‚ÿù„…‚ 0 ö }¡÷
Para resolver um problema (não só este, mas vários outros problemas na
matemática) que envolve n coisas, ajuda ver o que acontece para valores
pequenos de n. Vejamos alguns casos.
• n = 1. Fazemos

1 movimento foi suficiente.

• n = 2. Fazemos

3 movimentos deram.

• n = 3. Fazemos

7 movimentos deram.

Mas é claro que não podemos fazer só isso. Não podemos ficar observando o que
acontece para todos os valores de n! Então temos que começar a tirar algumas
conclusões.

EUREKA! N°11, 2001

18
Sociedade Brasileira de Matemática
2 † ` ‚ÿ‚¡„û}>|‚ö}„$‚¡ù„…‚ 0 ö }÷ ‚ i ~|‚|j
Vamos olhar o caso n = 3 mais perto. Observe os três primeiros movimentos:

Note que o que fizemos foi mesmo para resolver o caso n = 2. O próximo
movimento foi

Isto é, passamos o disco maior para o pino sem discos.


Agora, veja os três últimos movimentos:

Novamente fizemos o mesmo que foi feito para o caso n = 2, só que transferindo
agora a "subtorre" para o pino onde estava o disco maior.
Agora, imaginemos uma torre com n discos. Imagine também que sabemos
resolver o problema com n – 1 discos.

k n discos

Podemos transferir os n – 1 discos de cima para um pino vazio:


vários movimentos

n–1
k discos
k

EUREKA! N°11, 2001

19
Sociedade Brasileira de Matemática

Depois passamos o disco maior para o outro pino vazio:

Por fim, colocamos os n – 1 discos menores sobre o disco maior:

vários movimentos

n–1
discos m
m

Assim, podemos resolver o problema com n discos. Por exemplo, para resolver o
problema com 4 discos, transferimos os 4 – 1 = 3 discos de cima para um pino
vazio (já sabemos fazer isso!), depois passamos o disco maior para o outro pino
vazio e por fim colocamos os 3 discos sobre o disco maior. Para resolver o
problema com 5 discos, transferimos os 5 – 1 = 4 discos de cima para um pino
vazio (acabamos de aprender a fazer isso!), e assim por diante.
4 † ž`÷þ~‚¡þ‚¡}÷>‚| 0 ]‚ |
Voltemos à pergunta que será calada: queremos saber o número mínimo de
movimentos necessários para resolver uma torre de Hanói com n discos. Vamos
dar um nome para este número, digamos Tn. Assim, o número mínimo de
movimentos necessários para resolver um problema com 1 disco é T1, com 2
discos é T2, com 2001 discos é T2001, com ♥ discos é T♥, e, em especial,
com n – 1 discos é Tn – 1.

n † ”$‚ö€ƒ÷>þ~‚ÿ÷>‚ÿù„…‚ 0 ö }÷
Já vimos que podemos resolver o problema da seguinte forma:

vários movimentos

n–1
o discos
o

EUREKA! N°11, 2001

20
Sociedade Brasileira de Matemática

vários movimentos

n–1
discos p
p

Vamos ver quantos movimentos são necessários neste modo de resolver o


problema. Precisamos de Tn – 1 movimentos para movimentar os n – 1 primeiros
discos, mais um para movimentar o disco maior e mais Tn – 1 para colocar os n – 1
discos sobre o disco maior. Assim, precisamos de Tn – 1 + 1 + Tn – 1 = 2Tn – 1 + 1
movimentos. Mas não sabemos se este modo de resolver o problema usa o menor
número de movimentos; poderia haver outro modo que use menos movimentos.
Como o menor número de movimentos é Tn, temos:
Tn ≤ 2Tn −1 + 1 (I)
Provemos que na verdade Tn = 2Tn −1 + 1. Para isso, mostraremos que
Tn ≥ 2Tn −1 + 1 (lembre-se de que se a ≤ b e a ≥ b então a = b). Esta
aparentemente estranha maneira de se demonstrar que uma coisa é igual a outra é
na verdade bem comum em vários problemas. Muitas igualdades podem ser
obtidas a partir de desigualdades.
Considere agora, então, o disco maior. Ele vai ter que sair da torre inicial uma
hora. Mas para ele sair, é preciso que os outros n – 1 discos saiam de cima dele! E
mais, se quisermos mudá-lo de lugar ele vai ter que ir para um pino vazio, pois
ele não pode ficar sobre nenhum dos outros discos por ser o maior (que trabalho
esse disco dá!)! Logo precisamos transferir os n – 1 discos para um pino só, o que
requer no mínimo Tn – 1 movimentos. Para mudarmos ele de lugar, precisamos, é
claro, de mais um movimento. E depois, para colocarmos os n – 1 discos sobre o
disco maior precisamos no mínimo mais Tn – 1 movimentos. Assim, para resolver
o problema precisamos na verdade de no mínimo Tn −1 + 1 + Tn −1 = 2Tn −1 + 1
movimentos. Logo

EUREKA! N°11, 2001

21
Sociedade Brasileira de Matemática

Tn ≥ 2Tn −1 + 1 (II)
Assim, de (I) e (II),
Tn = 2Tn −1 + 1 (*)
Assim, como T1 = 1 (é só ver o caso n = 1), podemos, fazendo n = 2, concluir que
T2 = 2T1 + 1 = 2 ⋅ 1 + 1 = 3 (exatamente como achamos antes!!) e, fazendo n = 3,
descobriríamos que T3 = 2T2 + 1 = 2 ⋅ 3 + 1 = 7 (que coisa!). Para n = 4, acharíamos
T4 = 2T3 + 1 = 2 ⋅ 7 + 1 = 15. Se quiséssemos então Tn para um valor qualquer de
n, devemos ter todos os valores de Tk para k = 1, 2, …, n – 1, mas com certeza é
possível calcular. Uma seqüência deste tipo (isto é, tal que para calcular um dos
valores usamos os valores anteriores) é chamada recorrente e a equação que
relaciona os termos da seqüência é chamada de relação de recorrência (no caso,
temos que (*) é uma equação de recorrência).1
Poderíamos parar por aqui (pois já sabemos como calcular os valores de Tn ), mas
encontraremos uma fórmula para Tn que não depende de seus valores anteriores
(tal fórmula é costumeiramente chamada fórmula fechada). Nem sempre se pode
(e quando se pode, pode ser bem difícil) fazer isso com uma relação de
recorrência, mas com esta em particular pode ser feita.
Observe que temos "quase" Tn = 2Tn −1 . Vamos ver se podemos acertar isso. Se
somarmos um número x aos dois lados da equação (*), temos
 1+ x 
Tn + x = 2Tn −1 + 1 + x ⇔ Tn + x = 2 ⋅  Tn −1 + 
 2 
Se fizermos x = (1 + x) / 2 ⇔ x = 1 e sendo An = Tn + 1, temos
An = 2 An −1 = 2 ⋅ 2 An − 2 = 2 2 An − 2 = 2 2 ⋅ 2 An −3 = 2 3 An −3 = ... = 2 n −1 A1
Como A1 = T1 + 1 = 1 + 1 = 2, temos An = 2 n . Assim,
An = Tn + 1 ⇔ 2 n = Tn + 1 ⇔ Tn = 2 n − 1
Assim, precisamos de 2 n − 1 movimentos para resolver o problema da torre de
Hanói com n discos. Ou seja, os sacerdotes precisarão de 2 64 − 1 movimentos.
Mesmo se eles fizessem um movimento por segundo, eles precisariam de mais de
500 bilhões de anos!! Podemos ficar tranqüilos por enquanto.

1
Para outros comentários e resultados sobre recorrência veja o artigo "Equações de Recorrência",
de Héctor Soza Pollman, publicado na revista Eureka! No. 9

EUREKA! N°11, 2001

22
Sociedade Brasileira de Matemática
q † { 0 | }>„/>÷hrd‚ÿ¡ù‚„…€ƒ÷þ]€ƒ}
Os alunos mais observadores devem ter notado de antemão que Tn = 2 n − 1
bem antes, quando calculamos Tn para valores pequenos de n. Ter essa percepção
é bom, mas só perceber que Tn = 2 n − 1 não é suficiente.
É preciso provar que esta relação realmente é verdadeira. As aparências podem
enganar!! Por exemplo, considere a seqüência

n(n − 1)(n − 2)...(n − 2000)


an = +n
2001!
(lembre-se : 2001! = 1⋅ 2 ⋅ 3 ⋅ …⋅ 2001)
Temos a1 = 1, a 2 = 2,..., a 2000 = 2000. Isto poderia nos levar a crer que a n = n,
não? Pois veja quanto vale a 2001 e você terá uma bela surpresa!
‘ - }>„/sg‚|
01. Encontre uma fórmula fechada para cada uma das relações de recorrência a
seguir:
a) a n = 3a n −1 + 4, a1 = 0
b) bn = 2bn −1 + 3 , b1 = 5

02. (Prova de Seleção para a IMO e Olimpíada Iberoamericana 2001, adaptada)


Seja f uma função de t em t tal que, para todos x, y, z reais,
f ( x + y ) + f ( y + z ) + f ( z + x) ≥ 3 f ( x + 2 y + 3 z )
a) Mostre que f (a) ≥ f (0) para todo a real.
uv
Mostre que f (a) ≤ f (0) para todo a real e conclua que as funções f onde
f (a) = f (0) são as únicas soluções do problema.
{ 0 | }>„/>÷hrd‚$w
A grosso modo, uma função f de um conjunto A em um outro conjunto B, é uma
relação que toma cada elemento x de A e o transforma em um elemento f(x) de B.
As equações de recorrência que acabamos de estudar são exemplos de funções de
N em R.

03. Na torre de Hanói, suponha que em vez de transferir a torre para um dos
pinos, você tenha que transferir a torre para cada um dos outros pinos uma
vez. Encontre o número mínimo de movimentos para resolver esse problema.

EUREKA! N°11, 2001

23
Sociedade Brasileira de Matemática
zyyx n‰‹!zy t w uyzxô t vx t u
‰ Šz  { vx
‰ t u¢ wnvxyŸ‰ ŠŒ t t u
£ä(ª | ª>¤~¨ }̪€ ® ªMÌ¬B?© ¤¨ ¤¥

♦ XZY []\^`_ af\‚Sƒ \„b …‚b †(g

O que desejamos mostrar com esse texto é o potencial significativo da


trigonometria para resolver problemas de olimpíadas de matemática,
principalmente quando combinada com algumas desigualdades.
Esse texto, após a resolução de cada exemplo, apresenta um esquema da mesma,
um Guia de Resolução, o qual busca facilitar o entendimento geral do que foi
feito. Caso o leitor queira tentar resolver tais exemplos antes de conhecer a
resolução descrita aqui, poderá recorrer a este guia, como instrumento auxiliar.

Vamos aos exemplos.


*<!>C– :
(Seleção para IMO 99 – Brasil ) Para reais positivos satisfazendo
1 1 1 3
a + b + c = abc, mostre que + + ≤ , e determine
1+ a 2
1+ b 2
1+ c 2 2
quando a igualdade ocorre.

>Fp('$‡
A idéia básica para resolver esse problema é fazer uso da transformação de um
número real em tangente de outro. Isso vem do simples fato de que qualquer
número real a pode ser representado pela tangente de outro número real α
pertencente ao intervalo (–π/2, π/2), sendo tal α único – isso é explicado pelo fato
da função tangente, nesse intervalo, ser bijetora e ter como imagem todo o
conjunto dos números reais. E sendo ainda a um real positivo, podemos fazer
a = tg α, α ∈ (0, π/2).
Agora, podemos perguntar: por que essa transformação nos seria útil? Isso é
respondido se percebermos que a partir da conhecida identidade trigonométrica
1+ tg²α = sec²α, obtemos o seguinte resultado: 1 / 1 + tg 2α = cos α , ∀α ∈ ˆ ,
π
α≠ + kπ , com o qual podemos simplificar a desigualdade a ser provada. É
2
claro que se o estudante não tem o devido costume com essas fórmulas, ele,
provavelmente, não as reconheceria e nem pensaria em utilizar a transformação

EUREKA! N°11, 2001

24
Sociedade Brasileira de Matemática

para tangente. Mas é aí que entra a relevância da trigonometria . Agora podemos


prosseguir com a resolução.
Façamos a = tg α, b = tg β e c = tg γ, onde α, β, γ ∈ (0, π/2). Temos então que
tg α + tg β + tg γ = tg α.tg β.tg γ (1) . Como 1 / 1 + tg 2α = cos α , ∀α ∈ ‰ ,
π
α≠ + kπ , então o que devemos mostrar agora é que cos α + cos β + cos γ
2
≤ 3/2 para quaisquer α, β, γ ∈ (0, π/2) que satisfaçam a condição (1). Mas de (1)
vem que tg (α + β + γ ) = 0 (verifique! Dica: use a fórmula da tangente da soma
de três termos). Logo, como α , β , γ ∈ (0, π/2), temos que α + β + γ = π.
Para finalizarmos a demonstração, usaremos a seguinte forma especial da
desigualdade de Jensen (ver [4]): se uma função f é estritamente côncava (ver
observação abaixo) num dado intervalo (a,b), então

 a + ... + a n  f (a1 ) + ... + f (a n )


f  1  ≥ ,
 n  n

para quaisquer ai ∈ (a, b), ocorrendo a igualdade se e somente se os ai ' s forem


todos iguais. E caso a função seja estritamente convexa (ver observação abaixo)
em um determinado intervalo a desigualdade muda de sinal.

Continuando, como a função cosseno é estritamente côncava no intervalo (0,π/2),


temos que:

cos α + cos β + cos γ α + β +γ  π  1 3


≤ cos  = cos  = ⇔ cos α + cos β + cos γ ≤ ,
3  3  3 2 2

para quaisquer α, β, γ ∈ (0,π/2), ocorrendo a igualdade se e somente se α = β = γ


= π/3 ⇔ a = b = c = tg π/3 = 3 , concluindo a demonstração.

\F #$
$F$pŠ'‡
• A transformação de um número real em tangente de outro e o uso da fórmula
1+ tg²α =sec²α .
• Uso da propriedade dada no enunciado, para encontrar outra de melhor
proveito.
• Uso de uma forma especial da desigualdade de Jensen.

EUREKA! N°11, 2001

25
Sociedade Brasileira de Matemática

{ 0 | †z‹ Formalmente, uma função f : I → Œ , onde I ⊂ Œ é um intervalo, é


 x + y  f ( x) + f ( y )
estritamente côncava se f  > , para quaisquer x, y distintos
 2  2
 x + y  f ( x) + f ( y )
em I. E é estritamente convexa se f  < , para quaisquer x, y
 2  2
distintos em I. Uma maneira geométrica de identificar funções estritamente
côncavas ou convexas é observar a forma do gráfico das mesmas: se o gráfico for
uma curva com concavidade voltada para baixo, a função é estritamente côncava,
e se a concavidade for voltada para cima, é estritamente convexa. Como
exemplos de funções estritamente côncavas, temos a função f(x) = cos x no
domínio (0,π/2) e as funções logarítmicas cujas bases são maiores do que 1. E de
funções estritamente convexas temos a função f(x) = tg x no domínio (0,π/2) e a
função f(x)=1/sen x, com x em (0,π). (Como exercício, classifique outras funções
conhecidas em estritamente convexas ou côncavas).
É importante o leitor ver as referências [2] e [4], onde encontram-se definições e
resultados mais precisos e genéricos, além das demonstrações.

O próximo exemplo, da IMO de 1996, é tido por alguns matemáticos interessados


em olimpíadas (ver [1]), como o problema mais difícil já proposto em IMO’s. É
um problema de geometria associado a desigualdade (algo bastante explorado em
olimpíadas).
*<!> — : (IMO 96) Seja ABCDEF um hexágono convexo tal que AB é paralelo
a DE, BC é paralelo a EF e CD é paralelo a FA. Sejam RA, RC, RE os raios das
circunferências circunscritas aos triângulos FAB, BCD, DEF respectivamente, e
seja p o perímetro do hexágono. Prove que
p
R A + RC + R E ≥ .
>Fp('$‡ 2

Algo nesse problema já nos insinua a usar a trigonometria, você percebe? O fato
dele relacionar raio de circunferência circunscrita com lado (que tem a ver com o
perímetro) faz-nos lembrar da conhecida lei dos senos, que afirma: dado um
BC AC AB
triângulo ABC, temos que = = = 2 R , onde R é o raio da
senA senB senC
circunferência circunscrita ao triângulo dado (como exercício, prove-a). Daí,
portanto, podemos agora não mais trabalhar com os raios dos triângulos citados,
mas sim com algumas diagonais do hexágono. Isso porque, pela lei dos senos,

EUREKA! N°11, 2001

26
Sociedade Brasileira de Matemática

∧ ∧ ∧
obtemos que BF = 2RA.sen F A B , BD = 2RC.sen B C D , FD = 2RE.sen D E F (ou
seja, encontramos uma relação entre os raios citados no problema e algumas
diagonais do hexágono, o que facilitará o nosso trabalho).
A próxima parte da resolução do problema é a que exige uma maior dose de
criatividade por parte do estudante. Vejamos. Prolonguemos os lados paralelos
BC e EF do hexágono (vide figura 1). Por A e D tracemos perpendiculares aos
lados prolongados, obtendo o retângulo de vértices M, N, P e Q, ilustrados na
figura 1. Como MN e PQ são as menores distâncias entre pontos das retas
paralelas BC e EF (pois esses segmentos são perpendiculares às mesmas), temos
que BF ≥ MN e BF ≥ PQ ⇒ 2BF ≥ MN + PQ ⇒ 2BF ≥ AM + NA + DP + DQ ⇒

2BF ≥ AB.senB + AF.senF + CD.senC + DE.senE (1),

onde sen X denota o seno do ângulo interno de vértice X do hexágono, o qual é


igual ao seno do respectivo ângulo externo, pois os mesmos são suplementares.
B C
M P

A D

N F E Q

figura 1

Pela lei dos senos, nós já sabemos que BF/senA = 2RA . Então dividindo ambos os
lados da desigualdade (1) por senA, obtemos:

senB senF senC senE


4 R A ≥ AB. + AF . + CD. + DE. (I)
senA senA senA senA

E de forma análoga, seguindo os mesmos passos com as diagonais BD e DF do


hexágono, obtemos:

EUREKA! N°11, 2001

27
Sociedade Brasileira de Matemática

senB senD senA senE


4 RC ≥ BC. + CD. + AF . + EF . (ii)
senC senC senC senC

senA senC senD senF


4 R E ≥ AB. + BC. + DE. + EF . (iii)
senE senE senE senE

E agora, somando (I), (ii) e (iii), obtemos

 senA senB   senC senB   senC senD 


4( R A + RC + R E ) ≥ AB. +  + BC. +  + CD. + +
 senE senA   senE senC   senA senC 
 senE senD   senE senF   senA senF 
+ DE. +  + EF . +  + FA. + .
 senA senE   senC senE   senC senA 

Agora observe que como os lados opostos do hexágono convexo são paralelos,
nós temos que os ângulos opostos do mesmo são congruentes. Assim, nós
obtemos: senA = senD; senB = senE; senC = senF.
Por conseguinte, nós temos que os fatores que estão multiplicando os lados do
hexágono na última desigualdade acima são da forma (z + 1/z), sendo z positivo
(pois o seno de um ângulo maior que 0º e menor que 180º é sempre positivo). E é
fácil verificar que z + 1/z ≥ 2, para todo z positivo. Assim nós obtemos:

4( RA + RC + RE ) ≥ 2( AB + BC + CD + DE + EF + FA) ⇒ 4( RA + RC + RE ) ≥ 2 p ⇒
p
⇒ RA + RC + RE ≥ , concluindo a demonstração.
$ŽŠ &‘W’ “W’~”~•–ŽW—Š˜Š• ‡ 2

• Uso da Lei dos Senos.


• Uso das construções: prolongamento de dois lados opostos e traçado de
perpendiculares pelos dois vértices restantes.
• Congruência dos ângulos opostos do hexágono convexo.
• Uso da desigualdade: z + 1/z ≥ 2, z > 0.

’d™$’~šœ›~–•ž Ÿ
(IMO 91) Seja ABC um triângulo e X um ponto interior do mesmo.
Prove que pelo menos um dos ângulos ∠XAB, ∠XBC, ∠XCA é menor ou igual a
30º.

EUREKA! N°11, 2001

28
Sociedade Brasileira de Matemática
“Š’~”•$–ŽŠ—(˜W•Ÿ
Geralmente, em questões que envolvem um ponto num interior de um triângulo, é
útil traçarmos perpendiculares a partir desse ponto aos lados do triângulo. Vamos
utilizar isso.
Sejam P, Q, R os pés das perpendiculares traçadas por X aos lados BC, CA e AB,
respectivamente. Para facilitar, denotaremos por α , β , γ os ângulos do triângulo
(∠BAC, ∠CBA, ∠ACB) e por α’ , β ’ , γ’ os ângulos ∠XAB, ∠XBC, ∠XCA.

R Q

X
B C
P

figura 2

Nós temos que PX = BX.senβ ’ = CX.sen(γ – γ’); QX = CX.senγ’ = AX.sen(α – α’);


RX = AX.senα’ = BX.sen(β – β ’). Multiplicando essas três igualdades, nós
obtemos:
sen(α − α ' ).sen( β − β ' ).sen(γ − γ ' ) = senα '.senβ '.senγ ' ⇔

sen(α − α ' ) sen( β − β ' ) sen(γ − γ ' )


= 1.
senα ' senβ senγ '

sen( A − x)
Agora observe que a função f ( x) =
= senA. cot x − cos A é
senx
estritamente decrescente no intervalo (0, π), visto que a função cotangente é
estritamente decrescente nesse intervalo. Assim, se α’ , β ’ , γ’ forem todos
maiores que 30º, teremos que:

sen(α − α ' ) sen( β − β ' ) sen(γ − γ ' ) sen(α − 30º ) sen( β − 30º ) sen(γ − 30º )
1= < ⇔
senα ' senβ senγ ' sen30º sen30º sen30º
1
sen(α − 30º ) sen( β − 30º ) sen(γ − 30º ) > sen30º sen30º sen30º = (1)
8

EUREKA! N°11, 2001

29
Sociedade Brasileira de Matemática

Mas, nós temos que:

sen(α − 30º ).sen(β − 30º ) =


1
(cos(α − β ) − cos(α + β − 60º )) ≤ 1 (1 − cos(α + β − 60º )) =
2 2
=
1
(1 − sen(γ − 30º ))
2

Observe que essa última igualdade decorre do fato de (γ – 30º) ser complementar
a (α + β – 60º). Continuando, nós temos que:

sen(α − 30º ) sen( β − 30º ) sen(γ − 30º ) ≤


1
(1 − sen(γ − 30º )).sen(γ − 30º ) =
2
11   1
2
1
=  −  sen(γ − 30º ) −  ≤ .

24  2  8

Mas esta última desigualdade obtida contradiz (1), logo α’ , β ’ , γ’ não podem ser
todos maiores do que 30º, o que encerra a nossa demonstração.

$ŽŠ &‘W’ “W’~”~•–ŽW—Š˜Š•Ÿ

• Construção das perpendiculares a partir de X aos lados do triângulo e


obtenção da igualdade:
sen(α − α ' ).sen( β − β ' ).sen(γ − γ ' ) = senα '.senβ '.senγ ' . Esses ângulos são
identificados no início da resolução.
sen( A − x)
• Observar que a função f ( x) = é estritamente decrescente.
senx
• Supor, por absurdo, que todos os três ângulos são maiores do que 30º, e
chegar a uma contradição.

’d™$’~šœ›~–•¢¡dŸ
Prove que, dentre quaisquer cinco reais y1 , y2 , y3 , y4 , y5 , existem
dois, que satisfazem:
yi − y j
0≤ ≤ 1.
1 + yi y j

EUREKA! N°11, 2001

30
Sociedade Brasileira de Matemática
“Š’~”•$–ŽŠ—(˜W•Ÿ
Olhando para o termo do meio da desigualdade acima, o que ele nos faz lembrar?
Sem muita dificuldade, associamo-lo logo à formula da tangente da diferença.
Então mais uma vez façamos uso da transformação para tangente. Isto é, façamos
yi = tg xi, i = 1, 2, 3, 4, 5 , xi ∈ (–π/2, π/2).
Como tg 0 = 0 e tg π/4 = 1, devemos ter agora:

tgx i − tgx j π π
tg 0 ≤ ≤ tg ⇔ tg 0 ≤ tg (x i − x j ) ≤ tg .
1 + tgx i .tgx j 4 4

E ainda, como no intervalo (–π/2, π/2) a função tangente é sempre crescente,


π
obtemos 0 ≤ xi − x j ≤ . Agora o que temos que provar é que existem dois
4
dentre os cinco xi’s que satisfazem esta última desigualdade. Para isso, usamos o
conhecido Princípio da Casa dos Pombos. Dividamos o intervalo (–π/2, π/2) de
tamanho π em outros quatro intervalos de tamanho π/4. Assim, pelo princípio
citado, dois dentro os cinco xi’s estarão no mesmo intervalo, os quais vão
satisfazer a desigualdade pedida.

$ŽŠ &‘W’ “W’~”~•–ŽW—Š˜Š•Ÿ

• Uso da transformação para tangente.


• Aplicação da tangente da diferença.
• Uso do Princípio da Casa dos Pombos.

Finalizamos esse texto com alguns problemas para o leitor exercitar o que foi
mostrado. É lógico que existe mais de uma solução para cada problema, mas
pede-se que o leitor tente resolvê-los utilizando a trigonometria e as
desigualdades mostradas ou outras conhecidas.

’d™$’~“Š£Š¤¥£( •”~Ÿ

¦ §
. (IMO 61) Prove que, para qualquer triângulo de lados a, b, c e área A, temos
que: a 2 + b 2 + c 2 ≥ 4 3 A .

EUREKA! N°11, 2001

31
Sociedade Brasileira de Matemática

¦ ¨d©
Prove que, dentre 13 números reais, existem dois, x e y, tais que:

( )
x − y ≤ 2 − 3 .1 + xy .
¦  ©
(OBM – 85) Um quadrilátero convexo está inscrito em uma circunferência de
raio unitário. Demonstre que a diferença entre seu perímetro e a soma de suas
diagonais é maior do que zero e menor do que 2.
¦ ¡ ©
(Ibero-Americana 88) As medidas dos ângulos de um triângulo estão em
progressão aritmética e as medidas das alturas do mesmo também. Prove que o
triângulo é equilátero.
¦ ªd©
(Putnam 78) Encontre a área de um octógono convexo que está inscrito em
uma circunferência e que tem que quatro lados consecutivos medindo 3 unidades
e os lados restantes medindo 2 unidades. Dê a resposta na forma r + s t , com r,
s e t inteiros positivos.
¦ «d©
(Grã-Bretanha 84) O quadrilátero ABCD tem uma circunferência inscrita.

Para o lado AB nós associamos a expressão f(AB) = p1.(sen D A B ) + p2.

(sen A B C ), onde p1 e p2 são as medidas das perpendiculares traçadas de A e B,
respectivamente, até o lado oposto CD. Definimos f(BC), f(CD) e f(DA)
similarmente, usando para cada um as perpendiculares ao lado oposto. Mostre
que f(AB) = f(BC) = f(CD) = f(DA).
¦ ¬d©
(IMO 91) Em um triângulo ABC, as bissetrizes AD, BE, CF encontram-se no
ponto I. Mostre que:
1 IA IB IC 8
≤ . . ≤ .
4 AD BE CF 27
¦ ­d©
Mostre que se um quadrilátero de lados a, b, c, e d é inscritível e
circunscritível então sua área é abcd .

EUREKA! N°11, 2001

32
Sociedade Brasileira de Matemática
¦ ®d©
Uma função d(x, y) de dois reais x, y é chamada distância se d(x, y) = d(y, x);
d(x, x) = 0 ; e d(x, y) + d(y, z) ≥ d(x, z), para quaisquer reais x, y, z. Prove que a
seguinte função é uma distância:

x− y
d ( x, y ) = .
1+ x2 1+ y2
¯ §d¦ ©
Sejam x, y, z reais positivos tais que xy + yz + zx = 1. Prove que:

( )
2x 1 − x 2
+
(
2y 1− y2
+
) (
2z 1 − z 2

x )+
y
+
z
(1 + x )
2 2
(1 + y )
2 2
(1 + z )
2 2
1+ x 2
1+ y 2
1+ z2

Rafael Tajra Fonteles cursa a 2ª. Série do Ensino Médio no Instituto Dom
Barreto de Teresina – PI.
O Prof. José Nazareno Cardeal Fonteles da Universidade Federal do Piauí e
coordenador de matemática do Instituto Dom Barreto colaborou com o artigo,
fazendo a revisão do mesmo.

°  ° – •“(Š± ŠŸ


[1] – ENGEL, Arthur. Problem-Solving Strategies. Springer-Verlag, New York, 1998.
[2] – LARSON, Loren C. Problem-Solving Through Problems. Springer-Verlag, New
York, 1983.
[3] – MEGA, Élio & WATANABE, Renate. Olimpíadas Brasileiras de Matemática – 1ª
a 8ª. Comissão de Olimpíadas da SBM. Núcleo, São Paulo, 1988.
[4] – MUNIZ NETO, Antônio Caminha. Desigualdade Elementares. Eureka! Nº 5. OBM,
1999.
[5] – Página da Web mantida por Jonh Scholes: www.kalva.demon.co.uk/

EUREKA! N°11, 2001

33
Sociedade Brasileira de Matemática
²O³O´&µz¶O²¸·º¹ ´M»½¼¾µy²
¿AÀ(Á=ÂÃÀÄMÅÇÆÈPÉÂÈMÊ5Ë5ÄÌ ÍÉÎ Ä?¿WÏ ÂÐÂ

♦ ÑZÒ Ó]ÔÕ ÖWÓd×PØÙ ×ÚPÛ(Ü

Um polinômio é uma expressão da forma


p( x) = a 0 + a1 x + a 2 x 2 + ... + a n x n .
Uma série formal é uma expressão ainda mais simples - basta apagar o último
termo:
p( x) = a 0 + a1 x + a 2 x 2 + ...
Somas e produtos são definidos de maneira análoga às operações correspondentes
com polinômios. Assim, por exemplo,
(1 − 3x + 3 2 x 2 − 3 3 x 3 + ...)(1 + 3x) = 1 − 3x + 3 2 x 2 − 33 x 3 + ...
+ 3 x − 3 2 x 2 + 3 3 x 3 − ...
=1
de modo que podemos escrever 1 − 3 x + 3 2 x 2 − 33 x 3 + ... = 1 /(1 + 3 x). De
maneira geral, podemos "compactar" uma série formal 1 + ax + a2x2 +… na
forma 1/(1 – ax), que certamente ocupa bem menos espaço que uma série
infinita…
Vejamos uma primeira aplicação das séries formais. Vamos determinar uma
"formula fechada" para a seqüência definida por a 0 = 0 , a1 = 1 e
a n + 2 = 5a n +1 − 6a n para n ≥ 0. A idéia é considerar a série formal
f ( x) = a 0 + a1 x + a 2 x 2 + ... e tentar "compactá-la" e depois "descompactá-la".
Para alcançar o primeiro objetivo, observe que
f ( x) = a 0 + a1 x + a 2 x 2 + a 3 x 3 + ...
− 5 xf ( x) = −5a 0 x − 5a1 x 2 − 5a 2 x 3 + ...
6 x 2 f ( x) = 6a 0 x 2 + 6a1 x 3 + ...
Somando as equações acima, os coeficientes de x n , n ≥ 2, anulam-se e ficamos
com
x
(1 − 5 x + 6 x 2 ) f ( x) = a 0 + ( a1 − 5a 0 ) x ⇔ f ( x) =
1 − 5x + 6 x 2
Agora, como descompactar f(x)? O truque aqui é "quebrá-lo" em pedaços que
sabemos como descompactar.
Observe que 1 − 5 x + 6 x 2 = (1 − 2 x)(1 − 3x) e que é razoável procurar constantes a
e b tais que

EUREKA! N°11, 2001

34
Sociedade Brasileira de Matemática

a b x (a + b) − (3a + 2b) x x
+ = ⇔ =
1 − 2 x 1 − 3x 1 − 5 x + 6 x 2
(1 − 2 x)(1 − 3 x) 1 − 5x + 6 x 2

a + b = 0
⇔ ⇔ a = −1 e b = 1
3a + 2b = −1
Logo
x 1 1
f (x) = = − = (1 + 3x + 32 x2 + 33 x3 + ...)− (1 + 2x + 22 x2 + 23 x3 + ...)
1 − 5x + 6x 1 − 3x 1 − 2x
2

= (3 0 − 2 0 ) + (31 − 21 ) x + (3 2 − 2 2 ) x 2 + (3 3 − 2 3 ) x 3 + ...
Assim, o coeficiente de x n em f(x) (denotado por x n f (x)) é 3 n − 2 n . Mas [ ]
[x ]f ( x) = a
n
n , pela definição de f(x), logo a n = 3 − 2 . n n

›~“Š• ° –A’~šœ §
Utilizando séries formais, encontre "formulas fechadas" para as seguintes
seqüências:

a) F0 = 0, F1 = 1, Fn + 2 = Fn +1 + Fn para n ≥ 0 (esta é a famosa seqüência de


Fibonacci)
b) t 0 = t1 = 1, t n + 2 = −2t n +1 − 4t n para n ≥ 0
c) p 0 = p1 = 1, p 2 = 0, p n + 3 = 7 p n +1 − 6 p n para todo n ≥ 0

(As outras seqüências não tem nome. Alguma sugestão?)


›~“Š• ° –A’~šœ ¨
Calcule
F0 F1 F2 F3
0
+ 1
+ 2
+ + ...
10 10 10 10 3

em que Fn denota a seqüência de Fibonacci.

Outra aplicação das séries formais é ajudar a contar. Neste contexto, as séries
formais recebem o nome de funções geratrizes. O esquema geral é o seguinte: o
exponente de x quantifica alguma propriedade em que estamos interessados,

EUREKA! N°11, 2001

35
Sociedade Brasileira de Matemática

como o comprimento de uma seqüência, o número de conjuntos em uma partição,


a quantidade de duendes verdes em um jardim, etc. Se para cada objeto
associarmos tal potência de x e somarmos estas potências, o coeficiente de x n
será, respectivamente, o número de objetos com comprimento n, o número de
partições com n conjuntos, o número de jardins com n duendes verdes, etc.
Por exemplo, considere o problema de determinar o número de maneiras de se
escrever n como soma de termos 1, 2, 3, sem levar em conta a ordem dos termos.
A idéia aqui não é tentar obter este número para um valor particular de n. Somos
mais ousados: vamos obter todos estes números de uma só vez. Para isto,
escrevemos a função geratriz f(x) que é a soma das potências xs para cada soma s:

f ( x) = x 0 + x 1 + x 1+1 + x 2 + x 1+1+1 + x 1+ 2 + x 3 + ... = 1 + x + 2 x 2 + 3x 3 + ...

( x 0 corresponde à soma sem nenhum termo), de modo que, por exemplo, o


coeficiente de x 3 é o número de maneiras de escrever 3 como soma não ordenada
de termos 1, 2, 3. Aparentemente, obter f (x) é uma tarefa mais difícil do que a
inicial. Mas observe que cada termo de f (x) é o produto de um termo da forma
xsomas de 1's , um termo da forma xsomas de 2's e um termo da forma xsomas de 3's, logo

f ( x) = ( x0 + x1 + x1+1 + x1+1+1 + ...)(x0 + x2 + x2+2 + x2+2+2 + ...)(x0 + x3 + x3+3 + x3+3+3 + ...)


⇔ f ( x ) = (1 + x + x 2 + x 3 + ...)( 1 + x 2 + x 4 + x 6 + ...)( 1 + x 3 + x 6 + x 9 + ...)
1 1 1
⇔ f ( x) =
1− x 1 − x 1 − x3
2

[ ]
O problema agora é encontrar x n f ( x ), o que pode ser feito utilizando-se as
técnicas já vistas.

›~“Š• ° –A’~šœ&
Mostre que o número de partições não ordenadas de n com exatamente k termos
distintos é

xn yk [ ]∏
1 + x j ( y − 1)
1− x j
j ≥1

EUREKA! N°11, 2001

36
Sociedade Brasileira de Matemática
›~“Š• ° –A’~šœ&¡
a) Encontre constantes a, b, c, d, e, f tais que
1 1 1 a b c d e f
f ( x) = = + + + + +
1− x 1− x 1− x
2 3
(1 − x ) 3
(1 − x ) 2
1 − x 1 + x 1 − ωx 1 − ω 2 x
em que ω = ( − 1 + i 3 ) / 2 .

[ ]
b) Mostre que x n f ( x) =
(n + 3) 2

7 (−1) n ω n + ω 2 n  (n + 3) 2 1 
+ + = + 
12 72 8 9  12 2
em que x  denota o maior inteiro menor ou igual a x.

›~“Š• ° –A’~šœ ª
a) Determine a função geratriz do número de soluções da equação
x1 + x 2 + ... + x k = n , em que x i são inteiros positivos, 1 ≤ x i ≤ k .
b) Determine a função geratriz do número de partições ordenadas de n. (por
exemplo, 4 = 1 + 3 = 3 + 1 = 2 + 2 = 2 + 1 + 1 = 1 + 2 + 1 = 1 + 1 + 2 = 1 + 1
+ 1 + 1, de modo que há 8 partições ordenadas de 4)
c) Determine o número de partições ordenadas de n.

Os próximos problemas mostram uma técnica muito importante chamada


convolução. Ela se baseia no seguinte fato:
se
f ( x ) = f 0 + f 1 x + f 2 x 2 + f 3 x 3 + ...
g ( x ) = g 0 + g 1 x + g 2 x 2 + g 3 x 3 + ...

então h ( x ) = f ( x ) g ( x ) é a série formal associada à seqüência h 0 = f 0 g 0 ,


h1 = f 0 g 1 + f 1 g 0 , h 2 = f 0 g 2 + f 1 g 1 + f 2 g 0 , ..., h k = ∑fg
i+ j=k
i j ,...

›~“Š• ° –A’~šœ «

a) Mostre que se f(x) é a função geratriz da seqüência ( a n ) n ≥ 0 , então


f ( x ) /(1 − x ) é a função geratriz da seqüência b n = a 0 + a1 + ... + a n .
b) Prove que F0 + F1 + ... + Fn = F n + 2 − 1, em que Fn denota a seqüência de
Fibonacci.
(Este resultado pode também ser provado facilmente sem o uso de séries formais.
Tente!)

EUREKA! N°11, 2001

37
Sociedade Brasileira de Matemática
›~“Š• ° –A’~šœ ¬
Prove que
n  n   n  n
  F0 +   F1 +   F 2 + ... +   Fn = F2 n
n  n − 1  n − 2 0

em que, adivinhe, Fn denota a seqüência de Fibonacci!

›~“Š• ° –A’~šœ ­
a) Mostre que o número de triangulações Tn (por diagonais que não se
interceptam fora dos vértices) de um polígono convexo de n vértices satisfaz
T n +1 = T 2 T n + T3 T n −1 + ... + T n T 2
em que T2 = 1.

1 − (1 − 4 x ) 1 / 2
b) Prove que T ( x ) = T 2 + T3 x + T 4 x 2 + ... =
2x
1  2n − 2 
c) Mostre que T n =  .
n − 1  n − 1 

(T n +1 é o assim chamado n-ésimo número de Catalan). Para isto, lembre-se da


fórmula do binômio de Newton generalizada:

r   r  r ( r − 1)( r − 2 )...( r − k + 1) Þ Ý
(1 + x ) r = ∑  k x
k ≥0
k
,   =
k  k!
,k ∈ e r∈

Muitas vezes, as funções geratrizes são utilizadas não para calcular o


número exato de maneiras de se fazer isto ou aquilo, mas para mostrar que duas
quantidade são iguais. Vamos mostrar que o número de partições (não ordenadas)
de n em naturais distintos é igual ao número de partições (também não ordenadas)
de n em naturais ímpares. Por exemplo, 7 = 5 + 1 + 1 = 3 + 3 + 1 = 3 +1 + 1 + 1 +
1 = 1 + 1 + 1 + 1 + 1 + 1 + 1 e 7 = 6 + 1 = 5 + 2 = 4 + 3 = 4 + 2 + 1, de modo
que em ambos os casos o número de partições em naturais distintos é:

(1 + x )(1 + x 2 )(1 + x 3 )(1 + x 4 )...

enquanto que o número de partições em naturais ímpares é dado por

EUREKA! N°11, 2001

38
Sociedade Brasileira de Matemática

(1 + x + x1+1 + x1+1+1 + ...)(1 + x 3 + x 3+3 + x 3+3+3 + ...)...(1 + x 5 + x 5+5 + x 5+ 5+5 + ...)...


= (1 + x + x 2 + x 3 + ...)(1 + x 3 + x 6 + x 9 + ...)...(1 + x 5 + x 10 + x 15 + ...)...
1 1 1
= ...
1 − x 1 − x 1 − x5
3

Observe que as expressões acima são iguais! De fato, para se convencer disto,
basta multiplicar as igualdades

1− x2 1− x4 1 − x6
= 1 + x, = 1 + x 2
, = 1 + x 3 ,...
1− x 1− x 2
1− x 3

Isto completa a demonstração.

›~“Š• ° –A’~šœ ®
a) Escreva a função geratriz do número de maneiras de escrever um número n
como soma de potências distintas de 2.
b) Verifique que a função acima é igual a 1/(1 – x).
c) Utilizando os resultados acima, mostre que todo número pode ser escrito de
maneira única em base 2.
›~“Š• ° –A’~šœ § ¦
Prove que o número de partições de n em que apenas as partes ímpares podem ser
repetidas é igual ao número de partições de n em que nenhuma parte aparece mais
do que três vezes.
›~“Š• ° –A’~šœ § §
Prove que o número de partições de n com uma única parte menor (ela ocorre
uma única vez) e parte maior no máximo duas vezes a parte menor é igual ao
número de partições de n em que a maior parte é ímpar e a menor parte é maior
do que metade da parte maior.
›~“Š• ° –A’~šœ § ¨
Mostre que o número total de 1's nas partições de n é igual à soma dos números
de partes distintas em cada partição de n.

EUREKA! N°11, 2001

39
Sociedade Brasileira de Matemática
¹ ßàµá»žâO㥼¾ä¼ ²å¼¾¹æ´M¶OäM¹ ´çäM¹æ»žèMé&äM¹

ëê
O comitê editorial de ìAíZîìAï Ö ! agradece aos admiradores da seção
Olimpíadas ao redor do Mundo o envio de críticas, soluções e possíveis erros nos
enunciados dos problemas. Diversos leitores apontaram um possível erro no
enunciado de:
  © œ š ñò óôöõ`÷ ødù §d® ®d®dú
ð Seja n um número natural tal que 2n 2 possui 28 divisores
distintos e o número 3n 2 possui 30 divisores distintos. Qual o número de
divisores do número 6n 2 ?

Na realidade o problema está errado mesmo, apesar de corresponder à versão


publicada em : http://www.olsedim.com/olympiad/99.html

De fato, se 2n2 tem 28 divisores, 3n2 só pode ter 24, 42 ou 54 divisores (prove!).
O problema é parecido com a 29ª. questão da ûPüýþ7ÿ
onde os
números que possuíam 28 e 30 divisores eram respectivamente 2n e 3n, e pede-
se o número de divisores de 6n (Esse tem solução!).

Continuamos salientando que estamos à disposição na OBM para aqueles


que estiverem interessados na solução de algum problema particular. Para tanto,
basta contactar a OBM, através de carta ou e-mail.
ÿ5ý 5ý

Primeiramente vamos aos problemas propostos deste número
® §d© ÷ ø ù ¨ ¦d¦d¦ ú
!#"
$%$ Os coeficientes a e b da equação x 2 + ax + b = 0 e suas raízes
são quatro números distintos. É possível determinar a equação usando estes
quatro números?
® ¨d© ÷ ø ù ¨ ¦d¦d¦ ú
!#"
$%$ Um inteiro positivo n é chamado perfeito se a soma de todos os
seus divisores, excluindo n, é igual a n. Prove que se um número perfeito
maior do que 28 é divisível por 7 então ele é divisível por 49.
®  © ÷ ø ù ¨ ¦d¦d¦ ú
!#"
$%$ Prove a existência de números reais distintos a1 , a 2 , ... , a10 tais
que a equação:
(x − a1 )⋅ (x − a 2 )⋅ ... ⋅ (x − a10 ) = (x + a1 )⋅ (x + a 2 )⋅ ... ⋅ (x + a10 )
possua exatamente 5 raízes reais distintas.

EUREKA! N°11, 2001

40
Sociedade Brasileira de Matemática
® ¡ © ¨ ¦d¦d¦%*
!#"
$%$'& (%)
O círculo inscrito no triângulo ABC possui centro O e tangencia
o lado AC no ponto K. Um segundo círculo S com centro no mesmo ponto O
intersecta todos os lados do triângulo ABC. Sejam E e F os pontos de
interseção de S com os lados AB e BC que estão mais próximos do vértice B;
B1 e B2 são os pontos de interseção de S com o lado AC com B1 mais
próximo de A. Se P é o ponto de interseção dos segmentos B 2 E e B1 F ,
mostre que os pontos B, K e P são colineares.
® ªd© ¨ ¦d¦d¦%*
+!,#-
.%. & (%) A seqüência de números reais (a1 , a 2 ,..., a 2000 ) satisfaz a
+ + ⋅ ⋅ ⋅ + = (a1 + a 2 + ⋅ ⋅ ⋅ + a n ) para todo n , 1 ≤ n ≤ 2000 .
2
condição: a13 a 23 a n3
Mostre que todo elemento da seqüência é um número inteiro.
® «d© ¨ ¦d¦d¦%*
+!,#-
.%. & (%)
Cada um dos números 1, 2, 3, ..., N é preto ou branco. É
permitido mudar simultaneamente as cores de quaisquer três dos números se
um deles é a média aritmética dos outros dois. Determine os valores de N
para os quais é possível fazer com que todos os números fiquem brancos.
® ¬d© ¨d¦d¦ ¦'*
+!/0.2143 5 & (')
Determine todos os restos possíveis da divisão do quadrado de
um número primo com 120 por 120.
® ­d© ¨d¦d¦ ¦'*
+!/0.2143 5 & (')
Em um inteiro positivo M, de três algarismos, o algarismo das
centenas é menor do que o algarismo das dezenas e o algarismo das dezenas é
menor que o algarismo da unidades simples. A média aritmética de M com
todos os números de três algarismos obtidos pela reordenação dos algarismos
de M termina em 5. Determine tais números M.
® ®d© ¨d¦d¦ ¦'*
+!/0.2143 5 & (')
Gustavo convidou um número ímpar de pessoas para a festa de
seu aniversário e os dispôs em torno de uma mesa circular de modo que os
vizinhos de cada menina fossem meninos, os vizinhos de cada menino,
exceto Gustavo cujos vizinhos eram ambas meninas, fossem uma menina e
um menino. Mostre que :
a) o número de meninos convidados para a festa é divisível por 4.
b) na direção diametralmente oposta a Gustavo está sentada uma menina .
§ ¦d¦d© ¨ ¦d¦d¦%*
+!/0.'6 7980:%5 & (%)
Seja H o ortocentro de um triângulo acutângulo ABC com
AC ≠ BC . A reta que passa pelos pontos médios de AB e HC intersecta a
bissetriz do ângulo ∠ACB no ponto D. A reta HD passa pelo circuncentro do
triângulo ABC. Determine a medida do ângulo ∠ACB .

EUREKA! N°11, 2001

41
Sociedade Brasileira de Matemática
§ ¦d§d© ¨ ¦d¦d¦%*
+!/0.'6 7980:%5 & (%)
Sobre uma mesa existem pilhas de moedas. Um movimento
consiste em escolher uma das pilhas com pelo menos três moedas, pegar uma
moeda desta pilha, retirá-la da mesa e finalmente dividir as moedas restantes
desta pilha em duas outras pilhas (não necessariamente do mesmo tamanho).
É possível obtermos somente pilhas com três moedas após vários
movimentos se começarmos com uma única pilha com 2000 moedas?
§ ¦d¨d© ¨ ¦d¦d¦%*
+!/0.'6 7980:%5 & (%)
Determine todas as funções f : ; → ; que satisfazem a
condição f (x − f ( y )) = 1 − x − y para todos x, y ∈ ; .
§ ¦  © ¨ ¦d¦d¦%*
+!/0.'6 7980:%5 & (%)
Sobre uma mesa estão três caixas com pelo menos uma bola
em cada uma delas. Um movimento consiste em duplicar o número de bolas
de uma das caixas de modo que o número de bolas necessário para tal seja
retirado de uma das outras duas caixas. É possível que após vários
movimentos uma das caixas esteja vazia?
§ ¦ ¡ © ° ¨ ¦d¦ ¦'*
+ & < 6 7
=>- .'. & (')
Determine todos os pares de números inteiros (x, y) que
( ) ( )
satisfazem a equação: y x 2 + 36 + x y 2 − 36 + y 2 ( y − 12 ) = 0
§ ¦dªd© ° ¨ ¦d¦ ¦'*
+ & < 6 7
=>- .'. & (')
Seja M o ponto de interseção das diagonais AC e BD de um
quadrilátero convexo ABCD. Seja K o ponto de interseção do prolongamento
do lado AB, no sentido de B para A, com a bissetriz do ângulo ∠ACD .
Sabendo que MA ⋅ MC + MA ⋅ CD = MB ⋅ MD , prove que ∠BKC = ∠CDB .
§ ¦d«d© š ¨ ¦d¦ ¦'*
+ 7 6 ? @28 & (')Para cada subconjunto não vazio X do conjunto
M = {1,2,...,2000}, seja a X a soma do menor com o maior elemento de X.
Determine a média aritmética de todos tais números a X assim obtidos.
§ ¦d¬d© š ¨ ¦d¦ ¦'*
+ 7 6 ? @28 & (')
Determine o valor máximo do produto xy se os números reais x
e y satisfazem a relação: y (1 + x 2 ) = x 1 − 4 y 2 − 1 .
 
§ ¦d­d© š ¨ ¦d¦ ¦'*
+ 7 6 ? @28 & (')
Dois círculos se intersectam nos pontos M e N. A reta que
passa por M intersecta os círculos nos pontos A e B de modo que M ∈ (AB ) .
Os pontos C e D são os pontos médios dos arcos AN e BN respectivamente e
que não contém o ponto M e os pontos K e L são os pontos médios dos
segmentos AB e CD respectivamente. Prove que CL = KL.

EUREKA! N°11, 2001

42
Sociedade Brasileira de Matemática
§ ¦d®d© ¨d¦d¦ ¦'*
+!A0B
C D (')
Seja q(n) a soma dos algarismos de n. Qual o valor de
(((
q q q 2000 ? 2000
)))
§ §d¦d© ¨d¦d¦ ¦'*
+!EF>G'H & (')
Determine o número primo p para o qual o número
1 + p + p + p 3 + p 4 é um quadrado perfeito.
2

§ §d§d© ¨ ¦d¦ ¦'*
Sejam P (x ) e Q(x ) dois trinômios
+!,#I'J -
K 6 & H ( .MLNH'O I'H ( IP/0.%6 7Q8 ( H (')
quadráticos tais que três das raízes da equação P (Q (x )) = 0 são os números
− 22 , 7 e 13 . Determine a quarta raiz desta equação.
§ §d¨d© ¨ ¦d¦ ¦'*
+!,#I'J -
K 6 & H ( .RLNH'O I'H ( IS/0.%6 7Q8 ( H (')
Sabendo que a , b e c são números reais
positivos, resolva o sistema no conjunto dos números reais positivos :
 xy + xz − x = a

 yz + yx − y = b
 zx + zy − z = c

§ §  © › ¨d¦d¦ ¦'*
( 7
6 3 5 & (') Uma seqüência ( p1 , p 2 ,..., p n ) de números primos satisfaz à
seguinte condição: para n ≥ 3 , p n é o maior divisor primo de
p n −1 + p n − 2 + 2000. Mostre que a seqüência ( p n ) é limitada.
§ § ¡ © ¨ ¦d¦ ¦'*
+!T .'F ( I%6 )
ABC é um triângulo do Plano Cartesiano cujos vértices são pontos
de coordenadas inteiras. As medidas de dois dos lados AB, BC e CA
{
pertencem ao conjunto 17 , 1999 , 2000 . Qual o valor máximo possível }
da área de ABC ?
§ §dªd© ¨d¦ ¦d¦'*
+!T 5
U 6 ( 14I'F>F (')
Sejam P e Q os pontos de tangência da tangente comum a dois
círculos C1 e C 2 que se intersectam nos pontos M e N sendo N mais próximo
de PQ do que M. Mostre que os triângulos MNP e MNQ possuem áreas
iguais.
§ §d«d© ¨d¦ ¦d¦'*
+!T 5
U 6 ( 14I'F>F (') Quais os inteiros positivos a e b tais que
( a+
3 3
) 2
b − 1 = 49 + 203 6 .
§ §d¬d© ¨d¦d¦ ¦'*
+!V#6 I'W ( 5
O (') Determine os números reais x tais que:
 x 2 − x − 1 − 3 − 5 − 7 − 9 − 11 − 13 = x 2 − 2 x − 48

EUREKA! N°11, 2001

43
Sociedade Brasileira de Matemática
§ §d­d© ¨d¦d¦ ¦'*
+!V#6 I'W ( 5
O (')
Um quadrilátero convexo ABCD está inscrito em um
semicírculo de diâmetro AB. Sejam S o ponto de interseção de AC e BD e T o
pé da perpendicular baixada de S a AB. Mostre que ST divide o ângulo
∠CTD ao meio.
§ §d®d© §d® ¬d¦'*
+!T 5
U 6 ( 14I'F>F (')
As medidas dos ângulos ∠B e ∠C de um triângulo isósceles
ABC são iguais a 50º . Sejam D e E pontos sobre os lados BC e AC
respectivamente de modo que ∠BAD = 50º e ∠ABE = 30º . Determine a
medida do ângulo ∠BED .
§ ¨d¦d© › ¨d¦d¦ ¦
+ 7
6 3 5 & (')
) Seja X o conjunto de todos os inteiros positivos. Prove ou
disprove a seguinte afirmação :
“Existe uma função f : X → X tal que a igualdade f ( f (n )) = 2n é
verdadeira para todo n ∈ Y ”.

Z[Z\Z

Agora vamos aos comentários e soluções dos leitores para alguns dos
problemas apresentados em nossa seção nos números anteriores da ]^_]a`cbed f

§ © ° § ®d® ­'*
Seja f (x ) = x 3 − 3 x + 1 . Determine o número de soluções reais
+ B
6 U
g%F & (')
e distintas da equação f ( f (x )) = 0 .

hijkl
m inoqpo rts
jnivuo
rtjwxiy{zxwicr|hm ivus
kji}hu~2
A resposta é 7. Utilizando cálculo, a derivada de f (x ) é 3x 2 − 3 , e então seus
pontos críticos ocorrem em (− 1,3) e (1,−1) , respectivamente máximo e mínimo.
Mas f (− 2 ) = −1 , f (0) = 1 e f (2 ) = 3 assim, as raízes de f pertencem aos
intervalos (− 2,−1) , (0,1) e (1,2) . Se contarmos o número de vezes que f atravessa
completamente cada intervalo obteremos a nossa resposta. Com efeito, f atravessa
(− 2,−1) uma vez (quando x < −2 ), atravessa o intervalo (0,1) três vezes (quando
− 2 < x < −1 , 0 < x < 1 e 1 < x < 2 ); atravessa o intervalo (1,2) três vezes
também e portanto a equação f ( f (x )) = 0 possui 7 raízes.

EUREKA! N°11, 2001

44
Sociedade Brasileira de Matemática
¨ © §d®d® ­
+!,#I'J -
K 6 & H ( .L#H%O
I%H ( IR/0.%6 798 ( H (') ) Determine todos os números reais x tais que
x x x x  = 88 .
€‚ƒ„
… †‡ˆŠ‰ ‹tŒ ‡{‚
‹tŒ Ž
‚Ž‘cƒ‹tŒ †’Œ
‹t‹“Œ•”‹t–Œ
‚—‡ ˜'Œ™’š›2œ
22
A resposta é . Com efeito,
7
88 88
x x x  = ⇒ x é da forma com k ≤ x .
3

x k
1) Se x ≥ 0 então x ≥ 3 pois, se x < 3 teríamos
x x x x  < 3 4 = 81
88 1
Como = 29 tem-se que 29 ≥ x x x  ≥ 27 e daí :
3 3
88
x x x  = 27 ⇒ x = 27 (não serve)
88 22
x x x  = 28 ⇒ x = =
28 7
88
x x x  = 29 ⇒ x = (não serve)
29
2) Se x ≤ 0 então
x < −3 ⇒ x  ≤ −4 ⇒ x x x x  ≥ 3 3 ⋅ 4 > 88
por outro lado,
x ≥ −3 ⇒ x  ≥ −3 ⇒ x x x x  ≤ 3 4 = 81
e portanto, não há soluções negativas.

¨ ¬d© ¨d¦d¦ ¦
+!07
6 3 5 & (') ) Prove ou disprove a seguinte afirmativa :
a 2 + b3
Todo número racional positivo pode ser escrito sob a forma onde a , b ,
c5 + d 7
c e d são inteiros positivos.

€‚ƒ„
… †‡žŸ ‹t  Švž•‹tŒ Ž†0Œ•†‡•’Œ
‹t¡ Œ ‚Š¢v”¤£‡
‹“‡
‰ ŽcŒ™¥ˆŠ›2œ
A afirmativa é verdadeira.
Fazendo a = x 3 y 2 , b = x 5 y 2 , c = xy e d = x 2 y para quaisquer inteiros positivos
x e y , temos:

EUREKA! N°11, 2001

45
Sociedade Brasileira de Matemática

a 2 + b3
=
( ) ( )
2
x3 y 2 + x5 y 2
3

=
x 6 y 4 + x 15 y 6 x
=
c5 + d 7 ( )
(xy )5 + x 2 y
7
x 5 y 5 + x 14 y 7 y

Como todo número racional pode ser obtido pelo quociente de dois números
inteiros positivos, a afirmação está provada.

¦%§ ¦
¨!©#ª
«%«'¬ ­%®°¯%±'±
) Um número de 10 algarismos é dito interessante se todos os
seus algarismos são distintos e ele é um múltiplo de 11111. Quantos números
interessantes existem ?

€‚ƒ„
… †‡žŒ ‹t  ²Š‚vž•‹tŒ{Ž†cŒ†‡•’Œ ‹t¡ Œ
‚—¢v”£c‡ ‹t‡ ‰
ŽŒq™c¥ˆ›2œ
A resposta é 3456 . Seja I um número interessante então
I ≡ 0 + 1 + 2 + ⋅ ⋅ ⋅ + 9 ≡ 0(mod 9 )
( )
Logo, I = 99999 ⋅ N = 10 5 − 1 ⋅ N para algum número natural N de 5 algarismos.
Digamos, N = a1 a 2 a 3 a 4 a 5 logo,
I = 10 9 a1 + ⋅ ⋅ ⋅ + 10 5 a5 − 10 4 a1 − ⋅ ⋅ ⋅ − 10a 4 − a5
I = 10 9 a1 + ⋅ ⋅ ⋅ + 10 6 a 4 + 10 5 (a 5 − 1) + 10 4 (9 − a1 ) + ⋅ ⋅ ⋅ + 10(9 − a 4 ) + 10 − a 5 .
Sejam d1 , d 2 ,..., d 9 , d10 os dígitos de I, nesta ordem, então d1 + d 6 = 9 ,
d 2 + d 7 = 9 , d 3 + d 8 = 9 , d 4 + d 9 = 9 e d 5 + d10 = 9 . Como os únicos pares de
dígitos cuja soma é 9 são (0,9), (1,8), (2,7), (3,6) e (4,5) o número de
possibilidades para d1 , d 2 ,..., d 9 , d10 é 9 ⋅ 8 ⋅ 6 ⋅ 4 ⋅ 2 ⋅ 1 ⋅ 1 ⋅ 1 ⋅ 1 ⋅ 1 = 3456

¦'§ 1 1
¯ ¨!³#´>µ·¶'¸ ¬ ­'®!¯'±'±%±'¹ Resolva a equação + =1
x 2
(
4 − 3x ) 2

º»¼½¾
¿ »ÀÁqÂÁ ÃtÄ
¼À»v¥Á
Ãt¼ÅƻǺ¿ »v¥#Ä
½¼»vÈcº¥É2Ê

As soluções da equação são


2 3 2 3
, (1 − 2 cos 20º ) , 2 3 (1 + 2 cos 40º ) e
3 3 3
2 3
(1 + 2 cos 80º ) .
3

EUREKA! N°11, 2001

46
Sociedade Brasileira de Matemática

2− y
Com efeito fazendo-se 4 − 3 x = y + 2 temos que x = para x ≠ 0 e
3
4
x≠ . Fazendo as devidas substituições e simplificando chegamos a
3

( )
y y 3 − 12 y − 8 = 0 o que implica em y = 0 ⇒ x =
2 3
3
e y 3 − 12 y − 8 = 0 .
Aplicando-se a fórmula da equação do terceiro grau nesta última temos:

2 3 2 3
−8  − 8   − 12  −8  − 8   − 12 
y=3 − +   +  +3 − −   + 
2  2   3  2  2   3 
ou
 θ  θ 
y = 3 4  3 1 + 3i + 3 1 − 3i  = 3 4  3 2cis + 3 2cis −  
   3  3 
π π (1 + 6k )
onde θ = 2kπ + . Daí, y = 4 cos para k = 0,1,2 e assim obtemos as
3 9
outras raízes.

Obs: Também é possível chegar às soluções fazendo


1 1 3 1
= cosθ , = senθ ⇔ + =4⇔
x 4 − 3x cosθ senθ

senθ + cos θ = 2 senθ cos θ ⇔ sen(θ + 30°) = sen(2θ ) . Podemos fazer


3 1

2 2
θ = 30°,θ = 50° , θ =170° ou θ = 290° , e obtemos as mesmas
soluções (ainda que escritas numa outra forma:

2 3 1 2 3 1 1
= , (1 + 2 cos 40°) = = ,
3 cos 30° 3 cos 290° cos 70°
2 3 1 1 2 3 1
(1 − 2 cos 20°) = =− e (1 + 2 cos 80°) = ).
3 cos170° cos10° 3 cos 50°

EUREKA! N°11, 2001

47
Sociedade Brasileira de Matemática
Ë%Ì'§ Ë'Ì%Ì'Ì
¨!Í0«'Î ­'¸ Ï
­%® ¹ Mostre que existe uma seqüência de inteiros positivos
(a 1 , a 2 ,..., a n ,...) tal que a12 + a 22 + ... + a n2 é um quadrado perfeito para todo
inteiro positivo n .
º»¼½¾
¿ »ÀÁÐÄ ÃtÑ ²Š¼Å»vЕÅÃtÄ{ÆÀcÄÀÁ•ÒÄ ÃtÓ Ä
¼—Ô»vÇÕcÁ ÃtÁ Ö
ÅÆÄqÈc×ØÉ2Ê
Consideremos a seguinte seqüência:
n −1

∑ (a ) k
2
−1
a1 = 3 e a n = k =1
, ∀n ∈ {2,3,4,...}
2
Desta forma teremos :
n
 n −1 2
∑ (a )
k =1
k
2

=  (a k )  + (a n ) = (2a n + 1) + (a n ) = (a n + 1)
 k =1 
2 2 2

o que conclui a demonstração. (note que an é sempre par, e também é igual a


a n −1 (a n −1 + 2 )
, para todo n ≥ 3).
2

Ë%Ù'§ Ë'Ì%Ì'Ì
¨!Í0«'Î ­'¸ Ï
­%® ¹ Determine o maior número inteiro N que satisfaz as seguintes
condições :
N 
(a)   possui seus três algarismos iguais.
3
N 
(b)   é igual à soma de n números naturais consecutivos a partir de 1.
3
º»¼½¾
¿ »ÀÁÚÛÄ ¼¼—Ä
Ñ Á•Ü»cÀÃtÅݽÁ
ÖqÀÁ•Þ»¼Ä ÆcÀÄqЕÅÃtÄ ÆÀcÄÇÕ0Á
ÓÁ
Ö ÅÆcÄÈ×؊É2Ê

De acordo com as condições (a) e (b), tem-se que


 N  n(n + 1) ß
3= = 111 ⋅ k , k ∈ ; 1 ≤ k ≤ 9
  2
ou seja,

− 1 + 1 + 8·111·k
n=
2

EUREKA! N°11, 2001

48
Sociedade Brasileira de Matemática

Como n é natural, o radicando deve ser um quadrado perfeito o que ocorre


somente para k = 6 que substituído na expressão anterior nos fornece n = 36 e
N  N
daí,   = 111 ⋅ 6 = 666 ⇒ 667 > ≥ 666 ⇒ 2001 > N ≥ 1998
3 3
e portanto o maior N que satisfaz às condições dadas é 2000.

Atendendo a um pedido especial do leitor José Renato Carneiro e Carneiro


antecipamos a solução de:

Ù%Ù'§
¨!Í0«'à á9â0¶%¸
¬ ­%®°¯%±'±'±%¹ Determine todos os inteiros x e y que satisfazem à equação
x 3 + 9 xy + 127 = y 3 .

º»¼½¾
¿ »ÀÁãÄ
ÆÅÁ ¼ ×ÅÆÔÁ
ÅÃt»vº»äÃtÁ
ÅÃtÄÇå»cÓæÄ
¼Á{ç'ÄÈcÒèÉ2Ê

As soluções da equação são (3,7 ) e (− 7,−3) .


Fazendo-se y = x + a , substituindo-se e simplificando a equação proposta
(
chegamos a: (9 − 3a )x 2 + 9a − 3a 2 x + 127 − a 3 = 0 (*) )
Esta equação deve possuir soluções reais para possuir raízes inteiras. Seu
discriminante é:
( )
D = (9 − 3a ) − 4 ⋅ (9 − 3a ) 127 − a 3 = (9 − 3a ) a 3 + 9 a 2 − 508
2
( )
Se a ≥ 6 então a 3 + 9a 2 ≥ 540 e D < 0 . Para − 2 ≤ a ≤ 2 temos 9 − 3a > 0 e
a 3 + 9a 2 = a 2 (a + 9) ≤ 4 ⋅ 11 < 508 . Para − 8 ≤ a ≤ −3, a 2 (a + 9) ≤ 64 ⋅ 6 < 508, e
para a ≤ −9, a 2 (a + 9) ≤ 0 < 508. Assim, D ≥ 0 somente para a ∈ {3,4,5}. Para
a = 3 obtemos uma contradição em (*) . Para a = 4 , a equação x 2 + 4 x − 21 = 0
é satisfeita com x = 3 e x = −7 . Para a = 5 a equação 3x 2 + 15 x − 1 = 0 não tem
nenhuma solução inteira. As únicas soluções da equação proposta são portanto
(3,7 ) e (− 7,−3) .

ééé

EUREKA! N°11, 2001

49
Sociedade Brasileira de Matemática

Enviaram soluções de problemas os seguintes leitores da êìë·íêìîï !


ðcñòôó%õ öø÷úùû÷ýü4÷#öþ÷ ÿþó   ò
÷         ö ó ó ù ñ ÷ !ö ö ÷ ÿ
    'ó 0÷ ÿ    !"
ðcò>ö  ñ  ÷ $ò #ý÷ ñŠñ  % &  '% 4ü ÷ tó øõ ÷ ñ÷
 ()*+,*+-*.+
/ 0ñ øò>÷#ö % ÷
1 ó  ó  ÷  2 
3   ó%õ ò þ÷ ù ñ÷
 (.*+"*+,*."*.*(.+
 ó%ñõ ÷  4 ð 5 ÷  ÿýó ó'õ ÿ
 6 ñ÷
 (.*+*+.*.)
ýó ò   õ°ö 4%  ÷ 71 ó  ó  ÷    ÷#ö  ù4ó ò  ñ ÷   ù ñ÷
 (*,*+*.)*.8*(!"
94ö  ÷ ò  °ö  ðò ö ñ ó4ö 4%: õ ;ûó <ô÷ ñ ó%õ  1ôó 2 9 ñ÷
 ()**+*+"*+,*(.*.8*.*!"
<ô= ó ý?ñ >  A÷ @ ó  ó <ô÷ ñ ó%õ  1ôó 2 9 ñ÷ (..
5  ñ ó%õ öøA ÷   ñõ  ò ÷ % ü4& ÷ tó øõ ÷ '%  ñ÷  BC.*+"
5  ñ ó%õ öøA ÷   ñõ  ò ÷N=ù Døò  ÷{ñ % ü4& ÷ tó øõ ÷ '%  ñ ÷ !ö ö ÷ÿ
    ó 0÷ ÿ
 =   (!"
5  øñ ó E ò @  ö  ñ ÷  ö % ÷
1 ó ÿýó ó'õ 6 ÿ ñ÷  ()*+-
F  òøñ   HG  ó % ó
ò ó ò ó % ó%õ ;ûó4öø÷{ñ  /þð ñ÷  (,*-*!*+"*+*(+.*."*.=*.)*.8*..
I ÷ 1   õ @ Jó ÷  ÷{ññ K4ó=ù Døò  ÷ ñ % ó%õ ;ûó4öø÷{ñ  /þð ñ÷  (8)*)*+=J*+-*+*(+.
ùû÷  L5E  õ M  ñ  @'÷{ñ  ñ Nó  
ò ÷ ù  1 ö  < ÷ ñ ó  @ 5 ñ÷  (-
@ ó%ñ  õ ÷  °ö % ÷
1 ó   ÷#ö  ù4ó ò  ñ ÷   ù ñ÷  (*+,
@ ó%ñ  õ ÷ 
O  òô÷Nö P õ  ;  ñ ó /  õ Q ûð ñ÷  (*)*,*-*+8*(+*.)*.-*..*.!
@ ó%ñ  > õ  A ÷ @  ñ ó%ò>öýóö  ó%ñ ;ûó'õ Mô÷ #  ñ  òôó  =I ñ÷  ()*!*+*+!*."*(.)
@6R%ñ  & ÷ @  ñ ó ò öýóö  ó%ñ ;4ó%õ M ÷ #  ñ  òôó  =I ñ÷  ()*-*.*+)
@ = ó øñ &÷ <  õ  0ö 4% 6÷
1ôó   ÷#ö  ù4ó ò  ñ ÷   ù ñ÷  (*+-
@ øñ  õ E ÷ SQó  ÷{ò  õ ÷  ðcò>ö{ñ ó4ö  @ ó  TE ð G ñ÷  ()*,
P$JU ó%õ öý& ÷ @  õ õ ÷ %$V ÷{7ò   óûöý÷ P õ > EV ó 2%  ñ÷  BC)*)8*,)*-,*)*(.*+"*+8*.)*.,
4= ó øõ ÷#ðcõ  4ó òôö ñ  ðcñ =ó D
W ÷ % 6÷
1 ó #  ñ  òôó  =I ñ÷  (,,*-*-+*8=*8)*(8
 ÷þö{ñ  3ø÷N ð 36  ó%=ñ   ò M  ñ ÷ <ô÷ ñ ó%õ  1ôó 2 9 ñ÷  (*+*.*+"*+,*(.)*.
 ÷
ò X  ó%ñŠõ ÷ % 6 ÷  Hó  ó  <ô÷ ñ ó%õ  1ôó 2 9 ñ÷  ()*.*+-
Y ó%õ õ ó JZ °ö  @4@  ñ ó òôöþó #  ñ  òôó  =I ñ÷  (*)*,*-*.*(!*+)*+*+.*."*.)*.+

[&\$]_^a` bEcZd bfe


gih$jlkmEn m_k o:p_qEmlrsout vxw_jy m$vzjr {4|Z|&}~jLp_k$m:p$yo4r€m$voEn ‚4h$rƒo_v
„…:† ‡Lˆ…_‰:Š_‹_ŒŽ$† ‹ˆ ‹LŒ Œ …_:†‹:ŒL‘

’”“f“f“f“f“–•Q—™˜Zš_›f›f›f›Z›œŸž ¢¡:£¤ ¥$¦§£¨ ©¦£Eª ¦:«


’”“f“f“f“f“Z“f“&•Q—Ž˜fš_›f›Z›f›f›f››œ¬žBœŸ¡:£¤ ¥$¦§£Žœ­¦$£uª ¦$«
®”¯f¯f¯f¯f¯Z¯f¯f¯&°²±™³f´_µZµfµfµfµµfµZµ¶Ÿ·J¸­¹:º» ¼$½§º¾¸¿½$ºuÀ ½$Á
®”¯f¯f¯f¯f¯Z¯f¯f¯f¯f¯–°Q±™³Z´_µfµfµµfµZµfµfµfµfµE¶¬·2°J¯¬¹_º» ¼$½§º­°'¯i½$ºuÀ ½:Á

Âuà Ã$Ä:çÅLÆ_ÇÈÄÉ Ê$ÃÌËÊLÇÍÇÈÎÏ Ð Ê$ÃÌÑ­Ã Ò:ÊÓÐ Ê_ÔEÊÃÓÕ_É_ϐǀÊ$Ã_Ö

×fØLـØÌÚ$ÛÜÝÌÞLß:ێÛ_Ú Ú àÓÚ$Û:Þ4áâ_ãäå à¿ä$Ø$ã:æ_å ãLß à_ç

EUREKA! N°11, 2001

50
Sociedade Brasileira de Matemática
èÈéiêìëÓíÓîiïÈèñð™ïóòÈôÌéiõ™êìï÷ö©ø¢èóòÈôÓéiò÷éièQùúéiè

û
üLýþÿ  
ý ÿ ý         Hÿ  

48) Doze pintores vivem em doze casas construídas ao longo de uma rua circular
e são pintadas ou de branco ou de azul. Cada mês um dos pintores, pegando
consigo bastante tinta branca e azul, deixa sua casa e caminha ao longo da rua
no sentido anti-horário. Desta forma, ele repinta cada casa (iniciando na sua)
com a cor oposta. Finaliza o trabalho tão longo repinte alguma casa branca de
azul. Em um ano, cada casa estará pintada com a sua cor original sabendo
que, no começo do ano, ao menos uma casa estava pintada de azul.
º»¼½¾
¿ »ÀÁ c »Ãt»Ä Ö
æÓ»"!ç Ä#ä½%$Ä'&Á æ»ÇÜŊ» ÀÁ'( Ä ÆcÁ{ÅÃt»vÈÜ)(
É2Ê

Dizemos que uma pintura de uma casa é induzida se não é feita pelo dono
da casa. Isso só acontece quando o pintor acabou de mudar a cor da casa
imediatamente anterior de azul para branca.
Vamos mostrar que cada casa muda de cor exatamente duas vezes, sendo
uma induzida e outra não, o que claramente implica o resultado.
Primeiro vamos ver que uma casa não pode mudar de cor três vezes. Para
isso, supomos por absurdo que alguma casa muda de cor pelo menos 3 vezes, e
consideramos o primeiro momento em que uma casa muda de cor pela terceira
vez. Nesse caso, pelo menos duas das 3 pinturas são induzidas, e portanto a casa
imediatamente anterior muda duas vezes de azul para branca antes disso, e
portanto muda de cor pelo menos 3 vezes antes desse momento, o que é uma
contradição.

Para terminar, vamos agora ver que cada casa muda de cor duas vezes, o
que equivale a dizer que muda (pelo menos) uma vez de forma induzida. Para
isso, basta ver que toda casa muda alguma vez de azul para branca (pois uma casa
muda de forma induzida quando a casa anterior muda de azul para branca ela
muda de forma induzida). Isso é obvio para casas azuis. Vamos provar isso por
indução no número de casas imediatamente anteriores que são inicialmente
brancas. Para a casa imediatamente anterior esse número é menor (lembre que há
pelo menos uma casa azul no início), e portanto em algum momento ela muda de
azul para branca, e nossa casa muda de forma induzida, donde muda de cor pelo
menos duas vezes, e portanto em algum momento muda de azul para branca ❏

EUREKA! N°11, 2001

51
Sociedade Brasileira de Matemática

Obs. Note que essa solução não depende da ordem em que os pintores executam
seu trabalho (depois de um pintor acabar de pintar, o próximo não precisa ser seu
vizinho, mas é importante que cada pintor faça o processo uma vez).

52) Quatro retas se interceptam formando quatro triângulos conforme a figura


abaixo.
A

F B C

a) Prove que as circunferências circunscritas aos quatro triângulos possuem um


ponto em comum.
b) Prove que os centros dessas quatro circunferências são concíclicos (i.e. existe
uma circunferência que passa por todos eles).
º»¼½¾
¿ »ÀÁ * ÃtÆÄ
¼À»+(
»0¿ »À»+&Ä Ö
Ñ Å-,Á Ææ»Çã½/.½Á•À0ÁÒÄ0
ÅÄ Ö•ÈÜ)( É2Ê

P
D

F C
B

Seja P o outro ponto de interseção dos circuncírculos aos triângulos AED e FEB.
Temos que #FBEP é inscritível (# denota quadrilátero nesta solução), logo
EFˆB = BPˆ E . Da mesma forma EPˆ D ≡ Aˆ

EUREKA! N°11, 2001

52
Sociedade Brasileira de Matemática

(#ADEP é inscritível) e APˆ D ≡ AEˆ D ≡ FEˆ B , mas no triângulo FBE temos


FBˆ E ≡ Aˆ + Cˆ e FEˆ B ≡ 180° − ( Aˆ + Cˆ + EFˆB ) ⇒ FEˆ B + EFˆB ≡ 180° − ( Aˆ + Cˆ ),
mas como BPˆA ≡ APˆD + EPˆD + BPˆE ≡ FEˆB + EFˆB + Aˆ ≡ 180° − Aˆ − Cˆ + Aˆ ≡ 180° − Cˆ ,
portanto #APBC é inscritível o que implica que o circuncírculo ao triângulo ABC
passa por P. Temos ainda que FPˆ B ≡ FEˆ B (# FBEP é inscritível), logo
FPˆ D ≡ BPˆ A ≡ 180° − Cˆ , o que nos dá que o #FPDC também é inscritível, com
isso os circuncírculos aos triângulos ABC, FDC, FEB e AED possuem o ponto P
comum.
b)
A

P
O2

O1 I1
D
I2
O3
E

F B C

O4
Sejam:
O1: Centro do circuncírculo a ∆BEF ;
O2: centro do circuncírculo a ∆ADE ;
O3: centro do circuncírculo a ∆ABC ;
O4: centro do circuncírculo a ∆CDF ;

Lema: O segmento determinado pelos pontos de intersecção de dois círculos é


perpendicular ao segmento determinado pelos centros dos círculos:

C1 C2

EUREKA! N°11, 2001

53
Sociedade Brasileira de Matemática

∆C1 MC 2 ≡ ∆C1 NC 2 (L.L.L), logo MCˆ 2 C1 ≡ NCˆ 2 C1 o que significa que C1C 2 é
bissetriz de ∆MC 2 N , mas como este triângulo é isósceles então C1C 2 também é
altura e C1C 2 ⊥ MN (c.q.d).

∧ ∧
Do lema acima temos PB ⊥ O1O3 e PE ⊥ O1O2 , e portanto B P E = O2 O1 O3 e
BPˆ E ≡ BFˆE (subentendem o mesmo arco BE no círculo de centro O1).
Analogamente DPˆ C ≡ O Oˆ O , mas DPˆ C ≡ DFˆC (subtendem o mesmo arco
2 4 3

DC no círculo de centro O4). Como DFˆC ≡ BFˆE temos


DPˆ C ≡ BFˆE ≡ BPˆ E ≡ O2 Oˆ 1O3 ⇒ O2 Oˆ 4 O3 ≡ O2 Oˆ 1O3 .
Com isso provamos que #O1O2O3O4 é inscritível.

53) Prove que num círculo convexo dado e para o mesmo número de lados, o
polígono regular inscrito é aquele cuja superfície é máxima.

º»¼½¾
¿ »ÀÁÒÄ
Ó¼Š»Ö * ¼äcÁ Ãtæ»vÀĺżÓ
Ä 1 ÅÑ
æ»ÃÇ32Å5456%7/8%4—Å֕ÈÜ)9 É2Ê

a) Observe inicialmente o polígono convexo A1 A2 A3…An no círculo de centro


O.
B2
A2
B1 A1 B3
A3

O
Bn A4
An
: ;

Tracemos os segmentos OA1 , OA 2 , OA 3 ,..., OA n que encontram a circunferência


nos pontos B1, B2, B3,…,Bn; respectivamente.
Observe que o polígono B1B2B3…Bn tem superfície maior que o polígono
A1A2…An; isto nos garante que o polígono de área máxima deve ser inscrito na
circunferência.

EUREKA! N°11, 2001

54
Sociedade Brasileira de Matemática

b) Considere agora, o polígono convexo inscrito de n lados como na figura


abaixo:
B2

B1 B3
θ1 θ2
θn θ3
O
Bn
B4

< =

R2
A área S do polígono é dado por S = (senθ 1 + senθ 2 + ... + senθ n )
2
n n
onde ∑θ i = 2π . S será máximo quando
i =1
∑ senθ
i =1
i for máximo.

Para este problema, vamos utilizar a "desigualdade de Jensen" que diz: suponha
que f com concavidade para baixo (côncava) no intervalo I; então para todo
x1 , x 2 ,..., x n ∈ I teremos:
f ( x1 ) + f ( x 2 ) + ... + f ( x n )  x + x 2 + ... + x n 
≤ f 1 
n  n 
e a igualdade ocorrerá se e somente se x1 = x 2 = ... = x n . Para o nosso problema
temos f ( x) = senx e I = ]0, π [ onde θ 1 ,θ 2 ,...,θ n ∈ I ; portanto

senθ 1 + senθ 2 + ... + senθ n  θ + θ 2 + ... + θ n   2π 


≤ sen 1  = sen 
n  n   n 

e para o caso máximo: θ 1 = θ 2 = θ 3 = ... = θ n = ; ou seja, o polígono é regular
n
inscrito.

EUREKA! N°11, 2001

55
Sociedade Brasileira de Matemática

54) Sejam ( x n ) a seqüência definida por x1 = 2, x n +1 = 2 n , ∀n ≥ 1, e ( y n ) a


x

( y n2001 )
seqüência definida por y1 = 2001, y n +1 = 2001 , ∀n ≥ 1. Prove que
existe c natural tal que y n ≤ x n + c para todo n ∈ > e determine o menor c
com essa propriedade.

?@8%45A%BDC8"E%F *)45FD0Ò)8%GHGHIJ'*ä%GHFA"K32LŠæ3FGH6%LMcÜ9DNPO

Provaremos não que x n + c ≥ y n mas sim x n + c ≥ 44022 ⋅ y n2001 = 22 ⋅ 2001 ⋅ y n


2001

pois é fácil ver que só com o fato xn + c ≥ yn não se pode fazer indução.
Escolheremos c de tal modo que x1+ c ≥ 22 ⋅ 2001 ⋅ y12001 . Suponha válida a
desigualdade para um certo k então
xk +c ≥ 22 ⋅ 2001 ⋅ yk2001 ⇒ xk +1+c = 2 xk +c > 2 22⋅2001⋅ yk ,
2001

mas 211 ≥ 2001 ⇒ xk +1+ c ≥ (2001y k ) 2⋅ 2001 = yk2+⋅2001


2001

1 = yk2001
+1 ⋅ yk +1 ,
2001

mas y1 = 2001 ⇒ y k2001 ≥ 20012001 > 22 ⋅ 2001 ⇒ x k +1+ c ≥ 22 ⋅ 2001 ⋅ y k2001


+1

o que completa a indução.

Vemos que c = 4 satisfaz a condição pois x5 = 265536 > 22 ⋅ 20012002 mas


2001
x5 = 2 65536 < y2 = 20012001 já que 65536 < 20012001 < 2 < 2001, portanto c > 3 e
para c = 4 funciona ⇒ c = 4 é o menor valor.

55) Seja S o conjunto de pontos interiores de uma esfera de raio 1 e C o conjunto


de pontos interiores de um círculo também de raio 1. Existe alguma função
f : S → C tal que d ( A, B) ≤ d ( f ( A), f ( B)) para quaisquer pontos
A, B ∈ S? (d (A, B) denota a distância euclidiana entre A e B).

?@8%45A%BDC8"E%FRQSA%L54UT%FGH,+F'V%AXW5LUYZJDGHJ K3?@CD8\[@JA%4U8+M%?@[@NPO

Seja O o centro da esfera e C o da circunferência.


Tome A, B ∈ S tal que A, B e O sejam colineares,

EUREKA! N°11, 2001

56
Sociedade Brasileira de Matemática

d ( A; O) = d ( B; O), d ( A; B ) = 2 − ε , ε → 0, ε > 0.

Tome também P, Q ∈ S tal que P, Q e O sejam colineares,


d ( P; O) = d (Q; O), PQ⊥ AB e d ( P; Q) = 2 − ε .


Agora note que d ( f ( A); f ( B)) ≥ 2 − ε ⇒ ∠f ( A) C f ( B ) → 0, analogamente

d ( f ( P ); f (Q)) ≥ 2 − ε ⇒ ∠f ( P ) C f (Q) → 0.
 ε
Como d ( A; P ) = d ( B; P) = 2 1 −  então
 2
 ε ∧
d ( f ( A); f ( P)), d ( f ( B ); f ( P )) ≥ 2 1 −  ⇒ f ( P) C f ( A) → 90°.
 2

Agora vamos tomar P' e Q' do mesmo modo que tomamos P e Q, porém em um
plano perpendicular ao plano de A, B e P. Temos que
 ε
d ( P ' ; P) = d ( P' ; Q) = 2 1 − .
 2
∧ ∧
Mas como vimos antes, f ( P ' ) C f (Q' ) → 0 e f ( P ' ) C f ( A) → 90°,
logo f ( P ' ) → f ( P ) ou f ( P ' ) → f (Q) ⇒ d ( f ( P ' ); f ( P )) → 0 ou
d ( f ( P' ); f (Q)) → 0, o que é um absurdo, pois

 ε
d ( f ( P ' ); f ( P )), d ( f ( P ' ); f (Q)) ≥ 2 1 −  → 2 ❏
 2

*)]%^H_`%aDba,"8%c'd3_,+eXfD,g8"ah%iDj-8+`%_Dc c8%45k%lDm%ac a _'b8%4-_De%8%^H_lDn8"`Xapo

q r s tuv wyx zD{|}z| ~y{Dv€{ z€ ‚ƒX„y…


†‡ ˆ‡‰ uvu Š
©
u{zDtz‹wŒ{‚ƒX„y…
† v€u s z ‡Ž zŠ 
© r zDux  |Du r vz‘ x
©
r s Dz…ys ’Xu}zR“  }€ r ”u|‹wŒz
©ŽX ©
|‹w †• v–r   •— z ˜ r }Pw  ‘ ‰ ƒ
© zv–r ™Dz ˜ r s s  v r zDux  |Du r vz‘ x
© © ©

EUREKA! N°11, 2001

57
Sociedade Brasileira de Matemática
òÈô™éiõ™êìï÷ö©ø¢èóòÈôÓéiò÷éièQùúéiè

š
›yœŸž  ¡¢ £•œ ¤"œ¦¥ §  ¨ œ©ª¢¦§Xž–  ¢©ª¤œ¥ « ¬­§¤"¡ œ¤¯®X©œ °¥ § £•¢ ¤®Ÿ©œP®Ÿœ¤¨ œ¤+§±¤«²§ ¤¨ ­ §¤³¡ §´ œXžœ ¤
®Ÿ©œ° ¥ §£¢¤%®X¢©¢•œ ¤\®Ÿ©µ·¶¸  £•œ ¤ ¹£§©œ ¤ .

56) Para cada número n, seja f(n) a quantidade de maneiras que se pode expressar
n como a soma de números iguais a 1, 3 ou 4.
Por exemplo, f(4) = 4, pois todas as maneiras possíveis são 4 = 1 + 1 + 1 + 1,
4 = 1 + 3, 4 = 3 + 1, 4 = 4. Demonstrar que se n é par, f(n) é um quadrado
perfeito.

57) Dado n números reais x1, x2, …, xn satisfazendo as condições


x1 + ... + x n = 0 e x12 + ... + x n2 = 1 , prove que existem i e j tais que
1
xi x j ≤ − .
n
2 p −1 − 1
58) Determine todos os primos p para os quais o número é o quadrado
p
de um inteiro.

59) Um pedestal de altura a sustenta uma coluna de altura b (b > a). A que
distância do monumento se deve colocar um observador para ver o pedestal e
a coluna sob ângulos iguais?

60) Se num triângulo ABC , A = 2B, provar que a 2 = b(b + c).


Obs.: a, b e c são, respectivamente, os lados opostos aos ângulos A, B e C.

61) Na figura abaixo um quadrado EFGH foi colocado no interior do quadrado


ABCD, determinando 4 quadriláteros. Se a, b, c, e d denotam os medidas das
áreas dos quadriláteros, mostre que a + b = c + d .

c
d

EUREKA! N°11, 2001

58
Sociedade Brasileira de Matemática

62) Se ABCD é um quadrilátero convexo tal que os lados AB, BC , CD e DA


medem respectivamente a, b, c e d e que α, β, σ e γ são as medidas dos seus
ângulos internos, mostre que a medida da área desse quadrilátero, denotada
por (ABCD), é dada por:
D

γ c
d

d1
A α σ C

a
b

(ABCD) = ( p − a )( p − b)( p − c)( p − d ) − abcd cos 2 δ onde:


a+b+c+d
p=
2
α +σ β +γ
δ= (a fórmula também vale se fizermos δ = )
2 2

‰ v ztDs u ”  }\º»\u\º¼\½v–z½zD}·wŒz}+½zv¾~y¿ €u vzÁÀp{r  ™z †• ™  s {D u}+ÃÄzv-w  s u Š  ‘ ~yƒXÅÆ ½vzDts u ”  ºÇ
½vzD½zD}·wŒz±½Dzv †• v€us z ‹ÈŒr |Dz±Du"És r ’Xur v  à — u s ʔˑ ‰ y
… ÅÆ)½DvztDs u”  }̺ ÍÌuÌ» Î̽Dvz½Dz}ŒwŒz}̽Dzv
É }P’  s z † us s z Ž ½Dz|DÏ© r  ¸zÐÃÉs ¿ ”½r  ‘ ŽX‰ ÅƯ½DvztDs u ”  }Ñ»ÒÑuÑ» ÓѽDvz½Dz}PwHzD}ѽ¸zv'~  vs zD}х ‡
q zD”u}Ó  w  s‘ “yÅ ‡
©

EUREKA! N°11, 2001

59
Sociedade Brasileira de Matemática
øÕÔiï×ÖÓð÷ø é­êSØö ò×ٖÚ÷ø

Û@Û@ÜUÜ5ÜÝSÞ%Ü5ß [@à5*)á)*Ñâ)ã*)ä Ü5Þ/å@Ü ã)*Ñá å±ß *)æ å@ß'ç æ Ü5è *

2)à51 å@Ü ä¦éDêë'a ì


[@^Hj5,+aDj5^H_ V/_ca í'î@ï¸ðñDòópôöõRòD÷ªøùDúûDó
ä@aD]%k%h%`/_ V%_Dca í î@ïDðñòDóüôýò÷ þ ÷Dÿ5÷'ð3ó
æ%a^HbaDj5^H_ V/_ca í î@ïDðñDòóüô'ò÷RóDùÿ5ùDð ó3ú
÷ þÁýô ÷ 
 ó úDópôyýªòD÷RóùDÿ5ùð3ó ú ÷ þR÷ /þ ÷DùúDò¸ó ò ñ ò÷3ó ñ

2)à51 å@Þ 2 Ü 1 å ã)ä Ü æ ç ã Ü5Ý


[@^Hj5,+aDj5^H_ V/_ca í'î@ï¸ðñDòópôDý¾òD÷ þ ÷ÿ5÷Rð3ó
ä@a]/k%h%`%_'V%_cDa í'î@ïðDñòópô'÷  ó ú¸óyôpýªò÷'óùDÿ5ù¸ð3ó


1 Ü5ÜDÝSÞ%Ü5ß [@à5*)á*Ñá å¦ß * Ü5Ý
î@ïðñDòóyôý ò÷Rñ ó


Û@Ü5ÜDÝZÞ/Ü5ß [@àU*á)*Ñá å¦ß *)æ å@ß ç æ ÜUè *Ñá Ý è)Ý 2 å ä )Þ
 ñ'ò÷¾ø ù û¸ó
î@ñúDÿ ñópô)û  ÷


Û@Þ%Ü5ÜÝSÞ%ÜUß [@à5*)á)* Ü 2)æ å ã)2)* èÜUÝ 2)* Þ á å¦ß *)æ å@ß'ç æ Ü5è *
ýªñ ý' ! ò÷ªøù ûó
" ñþû úÿ5óDúpô# þ ÿ5ñòDó¸þ$)ú òDó¸þ


Û 1 ÜÝSÞ%Ü5ß [@à5*)á)* Ü â å ã Ý * ß'å ã Ü5è *2)*Ñá å¦ß *)æ å@ß ç æ ÜUè *
'ñ  ò÷ þ÷ÿ-÷ ð3ó
% úDñþ ô$&3ùùDñ


Ü1 ÝZÞ/Ü5ß [@àU*á)* Ü â å ã Ý * ß å ã Ü5è *)2)*Ñá å±ß *)æ å@ß'ç æ Ü5è *  2 Ü 1 å ã)ä Ü æ ç ã Ü *
î@ïðñDòóyô'ò¸÷ óDùÿ5ùð3ó

♦♦♦
♦♦ ♦

EUREKA! N°11, 2001

60
Sociedade Brasileira de Matemática
ÚÓéiéiô™ð™ï×Ö÷øiðÓéiô™ïÈèóô™ï ÔÑÙéÑÖ÷øÕÙ7è

')(*,+- .0/ 123*)45/ 6 78*9')+ *0:<; 1 =?>A@<B0C BED FHG8I<JLK9MLN


')6 OQP,+ R 1TS3*8.U.8PEVW3*U*82 =?>A@<X@HC XHY8D Z[8\]@QG0^ J9KLMLN
')VQ_P06 *L`a*0(*0+ _b1 = cA\Udef^ GL[U\]gh[8Y8i<J8FQj8G9[8\9k0[UY8l e GIKLcLghk0C m l Y8n9\Ud8JUYLK9o0c
pbPEV<PU2A/ R 1
qb*U2PErs*8.UtU1VHt8P06 1.vu+ P0/ + P =?>A@waxbC xbJ8e J8lHK]w,x
`A*0+6 1b.yu+ P82bP0+/ tU1Tpb1+ _bPU.]zh*06 (P0/ + * =?{0>bcA|}wbD G8C wbD GL[U\)XHJUd8\8D ^ G9K]wbX
`)6 *Er<2A/ 1')+ tU1VHtE~HP,+ = cAGl U€8D Gv?\UGUd8J0^ [8G9[8J9BED d8iHD C XHY8d[8D J0‚}KLo5{
`)6 *Er<.yS3*UP8R?/ V<_bP0+ =?>AxAƒ BEkE„AgUoEC ?J8… \JU[8G9Kyw3o
`)6 PU1VQ1+h`)+ PU.tU†EVHt,/ 12b*8.y‡3PU78PU. =?>b„kM)C MLJUd8JUY8IKLkM
ˆ 6 / 1T‰,P8_b* = cAGl U€8D GvgUe JUŠ8J8C oEjUGy{UJ8Y8l GLK)o{
Wb13.‹EVQ_P06 *L401ar<ŒH* = cAGl U€8D G9oED dU€Y8l J0^ C oEJUd8e G)kdU[0^ LK9o5{
u6 1A+ †EVQt0/ 1TuHP0++ P,/ + *L
r/ (*0+ ŽUP8.vu/ 6 ~<1 =?>A@gho0C BED e 0^ D JLKvgUo
/ .P06 PL23P)')+ *E:<; 1&z0+ *8R P*U2b1
rQ.0(Ž81 =?>A@<N‘C NGD ’Ud8D J)KLN‘
“ 78*EV/ 6 2PvuHP,+”VH*EV<2bPU.L4*8*U2 =?>bc)•U–3G8n—mhG8IHi<G8C cAJUnLŠUGLN^ JUdU[\UKLM9o
˜8*Ut8™ArQP,6 / VHPvu<*EO/ 16 *yW31b; *U.)')+ *EVQt,/ Ob/ * =?>A@{m8C XHG8jGv{U\8I<IHGUJLKv{0m
˜81bŽ81&pbPEV- t0/ 12bPy‰,P06 1T‡3P8R 1 =?>A@{0ƒ C „\0^ \8I<D d8J9K]{Eƒ
˜81bŽ81&u+ *EV<t0/ .8tU1T‰,P06 1›š/ OQ1VH*8Rœ/ = N^ Y8ŠUGygh[8YiQJ8iHD G8dUJ8lƒ [U\8J8l C mU\8l 8nKv{Uk
“ + P0V<Pv‡b*Ež?*U1Až?* =?>Ag8MLC MLJ0^ D dU€8Ÿ9K]{w
˜81b.8 9`A*0+6 1b.yzE/ V<R 1&š<P0/ 78*U. =?>A@w3N5C wbD GLN^ JdU[8\9Kyw3o
˜81b.8 9`)6 137PU.L4*0+ *,/ 7* =?>A@<M9k0C oEjUGy}YD IK)MLk
˜81b.8 9*U.,¡Q*0+EW,r<*U.yu/ 6 ~H1 =?ƒ cAM)cA|?>3o{8C oEjUGLcaJ0^ l GIKLo{
˜81b.8 všrb/ ŒW31b.U*U.]zE/ Vb~H1 =?>A@<ocAC @<l G0^ D JUd8UŠUGl D IKLoEc
˜81b.8 9s5/ P0/ + *9')6 7UP8. =?>A@{m8C cAJ nLŠ8D dJLN^ J8dU[U\LKy{Em
‰a*0+ tUP,6 1&WAr<¢œ/ V<1
2PL£6 / 78P,/ + * = o0D I<e \8nLJ9„D e Y8l J0^œ[\vgUd8I<D dUG8C mU\8l 8nKv{Uk
š/ t0/ 1&SbP,+”VH*EV<2bPU.yp3P8ŒHP,++ * =?>A@<ocAC @<l G0^ D JUd8UŠUGl D IKLoEc
šrHŒ/ VQ*,6 7U*y‰L/ + *EV<2b*L23P)'9(1+/ ( =?>A@mhk0C oEJ8l ¤HJU[8G,^œKym8k
‰a*0+ tU1AV<23P.9`A*U78*06 t8*EV<R Pvu+ *EV<¥U* =?>A@<cAC @<G0^ e Jl \8ZHJ)K9c)g
zU*EO6 1TW3132+/ _1
*EV<*U.U.0/ ( =?œD iH\YL„3\,^ ^ J8I[UG]ghd8€U\Ud8¦UG8C {UD ^ J8i<D i<J8§UJLKLo{
zU*Er6 1TS3PEVb+f/ ™ArHPL`)+”r<Œ
‡bP,/ 7*92bPvš/ (*9˜0+¨ =?g8IHi<G8l JL„b8i<d8D i<JygU¤Q\,^ J0^ [UGy{hJIHIHGIHC oEX[8G8IcaJ8n9ŠG8IK9o©{
WbP0/ V<*06 2b1PEV
“ t,~b/ + 1ª')+ *Ežœ*Ež8/ =?ƒ xA{0gUC oEX[8G8IcaJ8n9ŠG8IK9o©{
WA/ tU*0+ 21
')(1+/ ( = cA\Udef^ GygU[8Y8iHJiHD G8dUJlb}G8€UG8I<C xbG8¤HJ]ƒ €YUJ8FHYLKvwbX
Wb1aO<P0+ R 1
s5/ Œ<P0rªpb*,++ 1b. = cAGl U€8D G9kEiHJU\C BEG8l e Jvwb\U[UG8dU[8J9KywbX
4E 0+ _A/ 1`)6 «0rH2A/ 1›Wb*E(13. =?ƒ M)|?>A@bwbNo0C {UG0^ e G9kEl \U€0^ \LKvwbo
45/ 6 70/ 1
2Pyp3*0++ 1b.y‰,P06 1 =?>A@{g8C wb\8iHD ¬ \LKv{Eg
q*U2bPEruHP0++ P,/ + *L1(PU. =?>Ag0mhkEC XHY8JZH\8D ^ G9KymUk
q1a(«U.y‰,P0VH 0VH23PŒ
Wb1b2A+f/ _ar<PU. =?>A•U@<\ [8\,^ J8l<[U\ywbGUdU[8­Ud8D J8C {UG0^ e G9BE\l ¦8G9Kywb‘
sE*06 2PEVOQP,+ _ª')+ *E:H; 12b*L45/ 6 7U* =?>A•U@<\U[8\,^ J8l<[U\LoE\0^ €8D ŠU\C oEjUGLc)^ D I<e G8¤HjUG)KLog
®v*U_VHP0+zUP,+ P0/ + *yšH1a¡<PU. =?g8IHi<G8l JL„b8i<d8D i<Jy@Q\U[\0^ J8l<[8\9N5G8D Ÿ8I<C XHJ8e J,‚}KLN‘
®v*06 2bP0(*0+‰L¨}`a*0VH*06 6 / =?{0^ \¬ \8D e Y0^ JLM9Y8d8D iHD ŠUJ8lH[U\Lo©•}XHG8jUG)[U\LM9\0^ D e D C oEX[8\LM9\0^ D e D<KywbX

EUREKA! N°11, 2001

61
CONTEÚDO

AOS LEITORES 2

XLII OLIMPÍADA INTERNACIONAL DE MATEMÁTICA 3


Problemas e Soluções

XVI OLIMPÍADA IBEROAMERICANA DE MATEMÁTICA 13


Problemas e Soluções

ARTIGOS
COMO FERMAT E BÉZOUT PODEM SALVAR O DIA 25
Antonio Caminha Muniz Neto

GRAFOS E CONTAGEM DUPLA 31


Carlos Yuzo Shine

OLIMPÍADAS AO REDOR DO MUNDO 40

SOLUÇÕES DE PROBLEMAS PROPOSTOS 49

PROBLEMAS PROPOSTOS 59

AGENDA OLÍMPICA 60

COORDENADORES REGIONAIS 61
Sociedade Brasileira de Matemática

AOS LEITORES

Nesta última edição de 2001 publicamos soluções dos problemas da


Olimpíada Internacional e da Olimpíada Iberoamericana deste ano, nas quais o
Brasil obteve ótimos resultados, que deixaram toda a nossa comunidade olímpica
muito contente.

Por falar na comunidade olímpica, lembramos que neste ano foi criado o nível
universitário da OBM, cujas provas e soluções serão publicadas no próximo
número da Eureka!. Aproveitamos para saudar os novos (e antigos) olímpicos
universitários.

Mais uma vez agradecemos o grande número de soluções e problemas


propostos que são enviados pelos leitores, e que nos ajudam a fazer a Eureka!.
Como um estímulo adicional a nossos colaboradores, vamos dar um
reconhecimento especial aos leitores que resolverem mais problemas
propostos,ou aqueles problemas propostos que considerarmos mais difíceis, ou
que estiverem há bastante tempo sem solução. Aguardem e continuem
colaborando!

Finalmente, agradecemos a colaboração de Carlos Yuzo Shine, Eduardo


Tengan e Pablo Ganassim, que ajudaram na revisão deste número, e desejamos
um ótimo 2002 a todos.

Os editores.

EUREKA! N°12, 2001

2
Sociedade Brasileira de Matemática

XLII OLIMPÍADA INTERNACIONAL DE MATEMÁTICA


Problemas e Soluções

PROBLEMA 1
Seja ABC um triângulo acutângulo com circuncentro O. Seja PA uma altura do
triângulo com P no lado BC.
Considere que BCˆ A ≥ ABˆ C + 30° .
Prove que CAˆ B + COˆ P < 90° .

SOLUÇÃO DE ALEX CORRÊA ABREU (NITERÓI – RJ):


A

90 – Ĉ 90 – B

O
α A–α


B
C P M

Sejam ACˆ B = C , ABˆ C = B, CAˆ O = A e COˆ P = α ⇒ CP = b cos C . Seja M o


ponto médio de BC.
CM a
Como COˆ M = Aˆ e COˆ P = α ⇒ POˆ M = A − α ⇒ tg Aˆ = = e
OM 2OM
PM
tg ( Aˆ − α ) = onde BC = a , AC = b , AB = c e R = OA = OB = OC ⇒
OM
CM − PM
( (
tgα = tg Aˆ − Aˆ − α = )) ˆ ˆ (
tg A − tg A − α
=
)
OM =
CP ⋅ OM
=
1 + tgA ⋅ tg A − α
ˆ ˆ ( 1+
)
CM ⋅ PM OM 2 + CM ⋅ PM
2
OM
b cos Cˆ ⋅ R cos Aˆ
= já que CP = b cos Cˆ OM = R cos Aˆ ,
a 2 ab
R cos A +
2 2 ˆ
− cos Cˆ
4 2
EUREKA! N°12, 2001

3
Sociedade Brasileira de Matemática

a a a
PM = − CP = − b cos Cˆ e CM = e, pela lei dos senos no triângulo
2 2 2
ABC ⇒ b = 2 RsenBˆ e a = 2RsenAˆ ⇒
2R 2 senBˆ cos Cˆ cos Aˆ 2senBˆ cos Cˆ cos Aˆ
⇒ tgα = 2 = , mas
R cos 2 Aˆ + R 2 sen 2 Aˆ − 2 R 2 senAˆ senBˆ cos Cˆ 1 − 2senAˆ senBˆ cos Cˆ
2senBˆ cos Cˆ cos Aˆ senAˆ
como tgα > 0 ⇒ 1 − 2senAˆ senBˆ cos Cˆ > 0 ⇒ tgα ⋅ tgAˆ = ⋅ =
1 − 2senAˆ senBˆ cos Cˆ cos Aˆ
2 senBˆ senAˆ cos Cˆ  1 
= e como 2x 2 − x −1 < 0, ∀x ∈ − ,1 ⇒ 2 sen 2 Aˆ − sen Aˆ < 1
1 − 2 senBˆ senAˆ cos Cˆ  2 
∀Aˆ ∈ (0,90°), mas sen Cˆ − Bˆ ≥ ( 1
2
)
pois 90° > Cˆ − Bˆ ≥ 30° e a função seno é

( )
ˆ
crescente no intervalo (0,90°) ⇒ senAˆ ⋅ sen Cˆ − Bˆ ≥ senA ⇒
2
( ˆ ) ˆ ˆ( )
⇒ − senA ≥ −2 senAsen C − B ⇒ 2 senA − 2senAsen C − B ≤ 2 sen A − senA < 1 ⇒
ˆ ˆ ˆ ˆ ˆ 2 ˆ ˆ

⇒ 2 senAˆ (senAˆ − sen(Cˆ − Bˆ ))< 1 ⇒ 2 senAˆ (sen(Bˆ + Cˆ ) + sen(Bˆ − Cˆ ))< 1 ⇒


2 senAˆ senBˆ cos Cˆ
⇒ 4 senAˆ senBˆ cos Cˆ < 1 ⇒ pois
1 − 2 senAˆ senBˆ cos Cˆ
( )
1 − 2 senAˆ senBˆ cos Cˆ > 0 ⇒ tgAˆ ⋅ tgα < 1 ⇒ tg Aˆ + α = tgα + tgAˆ > 0 ⇒ Aˆ + α < 90°.

PROBLEMA 2
Prove que
a b c
+ + ≥1
a 2 + 8bc b 2 + 8ca c 2 + 8ab

para quaisquer números reais positivos a, b, e c.

SOLUÇÃO DE ALEX CORRÊA ABREU (NITERÓI – RJ):


a b c
+ + ≥ 1 ⇔ a 2 (b 2 + 8ca)(c 2 + 8ba) + b 2 (a 2 + 8cb)(c 2 + 8ba) +
a 2 + 8bc b 2 + 8ac c 2 + 8ba
c 2 (b 2 + 8ca)(a 2 + 8bc) ≥ 513a 2b 2 c 2 + 8(a 3b 3 + a 3c 3 + c 3b 3 ) + 64abc(a 3 + b 3 + c 3 )
Elevando ao quadrado, obtemos

EUREKA! N°12, 2001

4
Sociedade Brasileira de Matemática

(I)
2 513a 2 b 2 c 2 + 8(a 3 b 3 + a 3 c 3 + c 3 b 3 ) + 64abc(a 3 + b 3 + c 3 ) ⋅
⋅ (ab c 2 + 8ab + ac b 2 + 8ac + bc a 2 + 8bc ) ≥ 510a 2 b 2 c 2 − 8(a 3 b 3 + a 3 c 3 + b 3 c 3 )
mas pela desigualdade das médias vale:
(II) 8(a 3b3 + a 3c 3 + c 3b3 ) ≥ 24a 2b 2c 2 e 64 abc ( a 3 + b 3 + c 3 ) ≥ 192 a 2 b 2 c 2
donde:
(III) 2 513a 2 b 2 c 2 + 8( a 3 b 3 + a 3 c 3 + c 3 b 3 ) + 64abc(a 3 + b 3 + c 3 ) ≥ 27abc
portanto basta mostrar que:
(IV) (ab c 2 + 8ab + ac b 2 + 8ac + bc a 2 + 8bc ) ≥ 9abc . É fácil ver que (II),
(III) e (IV) implicam (I).
Dividindo (IV) por abc, obtemos 8 ab2 + 1 + 8 ac2 + 1 + 8 bc2 + 1 ≥ 9 , e fazendo
c b a
ab ac bc
2
= u , 2 = v, 2 = w temos que provar que: 8u + 1 + 8v + 1 + 8 w + 1 ≥ 9 ,
c b a
dado que uvw = 1.
Usando várias vezes a desigualdade entre as médias aritmética e geométrica,
obtemos
8u + 1 + 8v + 1 + 8w + 1 ≥ 3 ⋅ 6 512uvw + 64(uv + uw + vw) + 8(u + v + w) + 1 ≥

≥ 3 ⋅ 6 512 uvw + 192 3 u 2 v 2 w 2 + 24 3 uvw + 1 = 3 ⋅ 6 729 = 9 , o que prova


(IV), e de (II), (III) e (IV) obtemos (I), c.q.d..

PROBLEMA 3
Vinte e uma meninas e vinte e um meninos participaram numa competição
matemática.

• Cada participante resolveu no máximo seis problemas.


• Para cada menina e cada menino, existe pelo menos um problema que foi
resolvido por ambos.

Prove que existe um problema que foi resolvido por pelo menos três meninas e
pelo menos três meninos.

EUREKA! N°12, 2001

5
Sociedade Brasileira de Matemática

SOLUÇÃO DE CARLOS STEIN NAVES DE BRITO (GOIÂNIA – GO):


Considere a tabela:
O
A O1 O2 O3 ... O21
A1 X1,1 X1,2 X1,3 X1,21

A2 X2,1 X2,2 X2,3 X2,21

A3 X3,1 X3,2 X3,3 X3,21

. .
. .
. .

A 21 X 21,1 X21,2 X21,3 ... X 21,21

Cada intersecção Xi,j é a questão que a menina Ai e o menino Oj fizeram em


comum (se fizeram mais de uma em comum, escolha uma delas qualquer)

Lema: em cada linha i da tabela temos pelo menos 11 Xi,k's com a propriedade (I):
esse Xi,k aparece pelo menos 3 vezes nesta linha (contando ele mesmo).

Prova: Se por absurdo há menos de 11, então há pelo menos 11 que não têm essa
propriedade (são 21 no total). Então cada um desses se refere a uma questão que
aparece no máximo 2 vezes na linha.
Mas se há pelo menos 11 Xi,k’s e para cada questão no máximo dois Xi,k’s se
referem a ela, então há no mínimo 6 questões referidas nesse grupo.
Mas uma linha só tem no máximo 6 questões referidas, pois a menina i fez no
máximo 6 questões. Assim todas 6 questões feitas pela menina aparecem no
máximo 2 vezes. Absurdo, pois 6 × 2 = 12, mas na linha aparecem 21 (21 > 12)
questões. Assim o lema é verdadeiro.
Há o lema análogo de que em cada coluna j há pelo menos 11 Xkj’s com a
propriedade (II): cada um se refere a uma questão que aparece pelo menos 3
vezes na coluna. A prova é análoga.
Contando os Xi,j’s com a propriedade (I) temos pelo menos 11 × 21 (21 linhas).
Analogamente pelo menos 11 × 21 com a propriedade (II).
Se não houvesse nenhum Xi,j que tem ambas propriedades, teríamos pelo menos
11 × 21 com propriedade (I), outros 11 × 21 com propriedade (II), somando
22 × 21 Xi,j’s, absurdo, pois há só 21 × 21 Xi,j’s. Assim há um Xi,j que tem ambas
propriedades. Assim ele se refere a uma questão y que aparece 3 vezes em sua
coluna e 3 vezes em sua linha. Como aparece 3 vezes em sua coluna (em 3 linhas
distintas), as 3 meninas relativas a essas 3 linhas fizeram a questão y (pois tem ela
em comum).

EUREKA! N°12, 2001

6
Sociedade Brasileira de Matemática

Analogamente 3 meninos fizeram a questão y, analisando as 3 colunas distintas.


Assim 3 meninos e 3 meninas fizeram a questão y, o que sempre ocorre para certa
questão y.

SEGUNDO DIA
DURAÇÃO: 4 horas e meia.

PROBLEMA 4
Seja n um inteiro ímpar maior do que 1 e sejam k1 , k 2 ,..., k n inteiros dados. Para
cada uma das n! permutações a = (a1 , a 2 ,..., a n ) de {1,2,..., n}, defina
n
S (a) = ∑k a .
i =1
i i

Prove que existem duas permutações b e c, b ≠ c, tais que n! é um divisor de


S (b) − S (c).

SOLUÇÃO DE CARLOS STEIN NAVES DE BRITO (GOIÂNIA – GO):


Se não há S(b) – S(c) tal que n! S (b) − S (c) , e há n! permutações, cada
permutação é de uma classe módulo n!. Se não,existem b e c tais que
S (b) ≡ S (c)(mod n) ⇒ S (b) − S (c) ≡ 0(mod n). Absurdo.
Seja ∑ S (a ), o somatório de todas n! permutações possíveis.
i

Cálculo de ∑ S ( a ) : i

Cada ki aparece em (n – 1)! permutações multiplicando cada coeficiente (de 1 a


n). Logo no total temos (n − 1)!(1 + 2 + ... + n ) de cada ki. Assim,
(n + 1)n ⋅ (n + 1) ⋅ n! ⋅
* ∑ S (a ) = (n − 1)!
i
2
∑k i =
2
∑k i ≡ 0(mod n!)

(pois n é ímpar).Temos também que cada permutação é de uma classe, logo

n!(n! + 1)
∑ S (a ) ≡ 1 + 2 + ... + n!≡
i
2
(mod n!).

Como n > 1, 2 n! , e
n!(n!+1)  n! 
∑ S (a ) ≡
n! n!
** i ≡   ⋅ n! + ≡ (mod n!) .
2 2 2 2
n!
De * e ** temos que 0 ≡ (mod n!) , absurdo.
2

EUREKA! N°12, 2001

7
Sociedade Brasileira de Matemática

Logo há S(b) e S(c) tais que S(b) ≡ S(c) (mod n!) ⇒ n! S (b) − S (c).

PROBLEMA 5
Num triângulo ABC, seja AP a bissectriz de BAˆ C com P no lado BC, e seja BQ a
bissectriz de ABˆ C com Q no lado CA.
Sabemos que BAˆ C = 60° e que AB + BP = AQ + QB.
Quais são os possíveis valores dos ângulos do triângulo ABC?

SOLUÇÃO DE THIAGO BARROS RODRIGUES COSTA (FORTALEZA – CE):


A

Q
c
b

α
α
B P c

Nomenclatura:
a = BC
b = AC
c = AB
α = ABˆ Q = CBˆ Q
2ac cos α
Lema 1: BQ =
(a + c)
Prova: note que a área do triângulo ABC é igual a soma das áreas dos triângulos
ABQ e BQC. Logo
a ⋅ c ⋅ sen2α c ⋅ BQ ⋅ senα a ⋅ BQ ⋅ senα
= + ⇒
2 2 2
a ⋅ c ⋅ 2 senα ⋅ cos α = (a + c )BQ ⋅ senα ⇒
2ac cos α
BQ =
(a + c)

EUREKA! N°12, 2001

8
Sociedade Brasileira de Matemática

bc
Lema 2: AQ =
(a + c)

Prova: Pelo teorema das bissetrizes internas temos que:


CQ a CQ + AQ (a + c) b (a + c) bc
= ⇒ = ⇒ = ⇒ AQ = .
AQ c AQ c AQ c (a + c)

ac
Lema 3: BP =
(b + c )

Prova: análogo ao lema 2.

Do problema, temos que AB + BP = AQ + QB. Então, pelos lemas:


ac bc 2ac cos α a + b + c b + 2a cos α
+c= + ⇒ = ⇒
b+c a+c a+c b+c a+c
⇒ (a + c )(a + b + c ) = (b + c )(b + 2a cos α ) ⇒
a 2 + c 2 + 2ac + ab + bc = b 2 + bc + 2a (b + c )cos α ⇒
a 2 + c 2 − b 2 + 2ac + ab = 2a(b + c )cos α . (I)
Agora, pela lei dos cossenos no ∆ABC :
a 2 + c 2 − 2ac cos 2α = b 2 ⇒ a 2 + c 2 − b 2 = 2ac cos 2α
Assim, de (I):
2ac cos 2α + 2ac + ab = 2a(b + c )cos α ⇒
2c(cos 2α + 1) + b = 2(b + c )cos α ⇒
( )
2c cos 2 α − sen 2α + cos 2 α + sen 2α + b = 2(b + c )cos α ⇒
4c cos α − 2(b + c )cos α + b = 0.
2

Resolvendo a equação:
1 b
cos α = ou cos α = .
2 2c
1
Se cos α = , o único valor que ele poderia assumir era 60°, nesse caso
2
ABC = 2α = 120° = 180° − 60° = 180° = BAˆ C , absurdo.
ˆ
b
Então cos α = . Pela lei dos senos no triângulo
2c

EUREKA! N°12, 2001

9
Sociedade Brasileira de Matemática

b sen2α sen2α 2senα cos α


ABC : = ⇒ cos α = = ⇒
c sen(120° − 2α ) 2 sen(120° − 2α ) 2 sen(120° − 2α )
sen(120° − 2α ) = senα (note que cosα ≠ 0) ⇒
Veja que α deve pertencer ao intervalo (0, 60°) para que o triângulo possa existir.
Assim, o único valor possível para α seria fazendo α = 120° − 2α ⇒ α = 40° ⇒
ABC = 80° ⇒ BCA = 40°
Logo os ângulos do triângulo são:
ABˆ C = 80°, BCˆ A = 40° e BAˆ C = 60° .

PROBLEMA 6
Sejam a, b, c, d inteiros com a > b > c > d > 0. Considere que

ac + bd = (b + d + a − c)(b + d − a + c).

Prove que ab + cd é um número primo.

SOLUÇÃO DE ANTONIO CAMINHA MUNIZ NETO (FORTALEZA – CE):


Simplificando a condição do enunciado obtemos a2 + c2 – ac = b2 + d2 + bd, ou
ainda (2a – c)2 + 3c2 = (b + 2d)2 + 3b2 (*). Suponha, por contradição, que ab + cd
= p, p primo. A condição a > b > c > d > 0 garante que p = ab + cd ≥ 4×3 + 2×1 =
14, de modo que p ≥ 17. Por outro lado,

2p = 2ab + 2cd = (2a – c)b + (b + 2d)c,

de modo que mdc(2a – c, b + 2d) divide 2p. Para mdc(2a – c, b + 2d) ser par,
deveríamos ter b e c pares, donde p = ab + cd seria par, o que é um absurdo.
Logo,

mdc(2a – c, b + 2d) = 1 ou p

Afirmação: mdc(2a – c, b + 2d) = 1:

Suponha que mdc(2a – c, b + 2d) = p. Então (*) nos daria que p23(b2 –
c ), e daí p2(b2 – c2), uma vez que p ≠ 3. Porém, p = ab + cd > b, de modo que
2

0 < b2 – c2 < p2,


um absurdo.

EUREKA! N°12, 2001

10
Sociedade Brasileira de Matemática

Agora, sejam x = 2a – c, y = b + 2d. Então mdc(x, y) = 1 e segue de (*)


que x2 – y2 = 3(b2 – c2), ou ainda

(x – y)(x + y) = 3(b – c)(b + c)

Consideraremos dois casos separadamente:

i. b ≡/ c (mod 2): o fato de ser p = ab + cd garante que mdc(b, c) = 1. Como b + c


e b – c são ímpares, isto implica em mdc(b + c, b – c) = 1. Um argumento análogo
implica em mdc(x + y, x – y) = 1 também. Se 3  (x + y) então

x – y = mdc(x – y, (b + c)(b – c)) = mdc(x – y, b + c) mdc(x – y, b – c)


e
x + y = 3mdc(x + y, (b + c)(b – c)) = 3mdc(x + y, b + c) mdc(x + y, b – c)

Escrevendo α = mdc(x – y, b – c), β = mdc(x – y, b + c), γ = mdc(x + y, b – c) e


finalmente δ = mdc(x + y, b + c), temos x – y = αβ e x + y = 3γδ. Por outro lado,

b – c = mdc(b – c, x – y)mdc(b – c, x + y) = αγ e b + c = βδ,

analogamente. Resolvendo para a, b, c e d obtemos 4a = αβ + βδ + 3γδ – αγ, 2b


= αγ + βδ, 2c = – αγ + βδ e 4d = – αβ – βδ + 3γδ – αγ. Daí

4p = 4(ab + cd) = βγ (α2 + 3δ2)

Mas b ≡/ c (mod 2) implica em b + c e b – c ímpares, de modo que α, β, γ


e δ são ímpares. Assim, temos os seguintes casos:

a) βγ = p: nesse caso, α 2 + 3δ 2 = 4 e daí α = δ = 1. Portanto, a = b, um absurdo.

b) βγ = 1: neste caso, β = γ = 1 e daí c = d, um novo absurdo.

ii. b ≡ c (mod 2): Nesse caso b e c devem ser ímpares, pois do contrário 2
dividiria ab + cd = p. Assim, nas notações acima, temos x e y também ímpares.
Segue que

mdc(b + c, b – c) = mdc(x + y, x – y) = 2

EUREKA! N°12, 2001

11
Sociedade Brasileira de Matemática

(pois já temos mdc(x, y) = 1). Se 3(x + y) (o outro caso é novamente análogo),


escrevendo
 x − y  x + y   b − c  b + c 
   = 3  
 2  2   2  2 
e pondo
 x− y b−c x− y b+c
α = mdc  ,  , β = mdc  , ,
 2 2   2 2 

 x+ y b−c x+ y b+c


γ = mdc  ,  , δ = mdc  , 
 2 2   2 2 

chegamos, como acima, a 2a = αβ + βδ + 3γδ – αγ, b = αγ + βδ, c = – αγ + βδ e


2d = – αβ – βδ + 3γδ – αγ. Daí,
p = ab + cd = βγ(α 2 + 3δ 2)

Nem β nem γ são iguais a p, pois do contrário teríamos b > p, o que contradiz ab
+ cd = p. Logo β = γ = 1 e daí c = d, um novo absurdo.

EUREKA! N°12, 2001

12
Sociedade Brasileira de Matemática

XVI OLIMPÍADA IBEROAMERICANA DE MATEMÁTICA


Enunciados e Resultado Brasileiro

A XVI Olimpíada Iberoamericana de Matemática foi realizada na cidade


de Minas, Uruguai no período de 24 a 29 de setembro de 2001.
A equipe brasileira foi liderada pelos professores Carlos Gustavo Tamm
de Araujo Moreira, do Rio de Janeiro – RJ e Pablo Rodrigo Ganassim, de São
Paulo – SP.

O Resultado da Equipe Brasileira

BRA 1 Carlos Stein Naves de Brito Ouro


BRA 2 Daniel Massaki Yamamoto Prata
BRA 3 Daniel Pinheiro Sobreira Ouro
BRA 4 Thiago Barros Rodrigues Costa Prata

PRIMEIRO DIA
DURAÇÃO: 4 horas e meia.

PROBLEMA 1
Dizemos que um número natural n é "charrua" se satisfaz simultaneamente as
seguintes condições:

- Todos os algarismos de n são maiores que 1


- Sempre que se multiplicam quatro algarismos de n, obtém-se um divisor de n.

Demonstrar que para cada número natural k existe um número "charrua" com
mais de k algarismos.

SOLUÇÃO DE DANIEL MASSAKI YAMAMOTO (SÃO PAULO – SP):


Vou construir números “charruas” (o que, afinal, significa isso?), utilizando os
algarismos 2 e 3.
Multiplicando-se 4 algarismos, obtemos um número da forma d = 3. . 2±., onde
0 ^ . ^ 
Vou fixar o final do número em “3232” e completar o resto apenas com 2’s.
Então 0 ^ . ^ 
3 2 n (I)
Para que dGn, basta garantirmos que:  4
2 n (II)

EUREKA! N°12, 2001

13
Sociedade Brasileira de Matemática

É fácil ver que 2 4 3232 ⇒ 2 4 M ⋅ 10 4 + 3232, ∀M ∈  ⇒ 2 4 n , logo (II) está


satisfeito.
Para garantirmos a divisibilidade por 9 , basta que a soma dos algarismos seja
divisível por 9.
Pegue n0 = 22223232 . Note que 9G
Podemos colocar blocos de nove 2’s à esquerda de n0 que não afetará a
divisibilidade por 9.
k  k 
Adicionando   blocos, o número terá mais de 9 ⋅   + 8 > k dígitos.
9 9

PROBLEMA 2
A circunferência inscrita no triângulo ABC tem o centro O e é tangente aos lados
BC, AC e AB nos pontos X, Y e Z, respectivamente. As rectas BO e CO
intersectam a recta YZ nos pontos P e Q, respectivamente.

Demonstrar que se os segmentos XP e XQ têm o mesmo comprimento, então o


triângulo ABC é isósceles.

SOLUÇÃO DE THIAGO BARROS RODRIGUES COSTA (FORTALEZA – CE):


A

Y
βQ
Z

O
.

α σ’
α σ
B X C

Pelo teorema do bico, o triângulo BXZ é isósceles. Como OB é bissetriz do


ângulo B , OB é mediatriz do segmento XZ .
Considere o triângulo ZXP. O ponto P pertence a mediatriz de XZ . Assim, a
altura relativa ao vértice P e a mediana relativa ao mesmo são iguais ⇒ ∆XZP é
isósceles ⇒ XP = PZ . (*)
De maneira análoga, XQ = QY .(**) .
Suponha agora que XP = XQ.

EUREKA! N°12, 2001

14
Sociedade Brasileira de Matemática

Então, por (*) e (**), QY = PZ (* * *) .


Seja β = AZˆY = AYˆZ (O ∆AZY é isósceles, pelo teorema do bico).
Como O é incentro OZ ⊥ AZ ⇒ YZˆO = 90 − β .
Da mesma forma, OY ⊥ AY ⇒ OYˆZ = 90 − β ⇒
OYˆZ = YZˆO . (1)
Note que OZ e OY são raios ⇒ OZ = OY . (2)
Por (***), (1) e (2) temos que os triângulos OZP e OYQ são congruentes ⇒
OQˆ Y = OPˆ Z . (3)
Se α = OBˆ Z e σ = OCˆ Y , temos que (no ∆ZPB )
α + ZPˆ O = β = σ + OQˆ Y mas, por (3), OQˆ Y = OPˆ Z ⇒
α = σ , mas ABˆ C = 2α = 2σ = ACˆ B ⇒ ∆ABC é isosceles.

PROBLEMA 3
Sejam S um conjunto de n elementos e S1, S2,…,Sk subconjuntos de S (k ≥ 2),
cada um deles com pelo menos r elementos.

Demonstrar que existem i e j, com 1 ≤ i < j ≤ k , tais que o número de elementos


nk
comuns a Si e Sj é maior ou igual a r − .
4(k − 1)

SOLUÇÃO DE CARLOS STEIN NAVES DE BRITO (GOIÂNIA – GO):


Chamaremos os n elementos de S de b1, b2, ...,bn .Se o número de subconjuntos Si
 ai 
que contêm {bi} é ai,   é o número de pares de subconjuntos que
2 
compartilham bi.
n
Temos que ∑a
i =1
i ≥ kr , pois é a soma de quantos elementos há em cada um dos

subconjuntos Si.
 x + k   x   x + k − 1  x + 1
Temos que, para k ≥ 1,   +   ≥   +  , pois:
 2  2  2   2 
( )( ) ( )( )
x 2 + 2xk + k 2 − k − x + x 2 − x ≥ x 2 + k 2 +1+ 2kx − 2x − 2k − x − k +1 + x 2 + x ⇔ k ≥ 1.

EUREKA! N°12, 2001

15
Sociedade Brasileira de Matemática

n
 ai 
Logo em ∑  2  você pode ir reduzindo a diferença entre os a ’s que a soma só
i =1
i

diminui até que você tenha número com diferença máxima, dois a dois, de um.
Seja kr = yn + q, 0 ≤ q ≤ n − 1. Assim teremos no fim q ai’s sendo (y + 1) e
n
 ai 
(n – q) ai’s sendo y ( para minimizar ∑  2  ).
i =1

 ai   y + 1  y  y
Temos então: ∑  2  ≥ q 2 
 + (n − q )  = qy + n .
2 2 
Vamos mostrar que
 kr 
kr  − 1 (yn + q ) y + q − 1
 y  ⇔ 2qy + ny − ny ≥
2
qy + n  ≥  
n n 

 
2 2 2 2
q2 q2
⇔ 2qy + ny 2 − ny ≥ ny 2 + + 2 yq − ny − q ⇔ q ≥ , o que segue de n ≥ q.
n n
Logo temos:
 kr 
kr  − 1
n
 ai    .

n
  ≥
i =1  2  2
 ai 
Note agora que ∑  2  conta com multiplicidade o número de subconjuntos S i

 k  k (k − 1)
com algum elemento em comum. Dividindo por   = , que é o número
2 2
de pares de subconjuntos Si , obtemos o tamanho médio das interseções de dois
 kr 
kr  − 1
desses subconjuntos, que é portanto maior ou igual a   ⋅2⋅ 1 ⋅ 1 .
n
2 k k −1
Basta mostrar que esse mínimo para esse certo subconjunto é maior ou igual a
nk
r− e acaba o problema.Para isso,note que
4(k − 1)
 kr 
r  − 1
 n  ≥ r − nk ⇔
k −1 4(k − 1)

EUREKA! N°12, 2001

16
Sociedade Brasileira de Matemática

 kr 
r  − 1
n  ≥ 4r (k − 1) − nk ⇔
k −1 4(k − 1)
4r 2 k − 4n ≥ 4nrk − 4rn − n 2 k ⇔
4r 2 k ≥ 4nrk − n 2 k ⇔ 4r 2 ≥ 4nr − n 2 ⇔ n 2 − 4nr + 4r 2 ≥ 0 ⇔
⇔ (n − 2r ) ≥ 0 (sempre verdade).
2

SEGUNDO DIA
DURAÇÃO: 4 horas e meia.

PROBLEMA 4
Determinar o número máximo de progressões aritméticas crescente de três termos
que pode ter uma sucessão a1 < a 2 < ... < a n de n ≥ 3 números reais.
Nota: três termos a i , a j , a k de uma sucessão de números reais formam uma
progressão aritmética crescente se a i < a j < a k e a j − a i = a k − a j .

SOLUÇÃO DE DANIEL PINHEIRO SOBREIRA (FORTALEZA – CE):


Primeiro vejamos que cada P.A. tem um termo central, pois são formadas de três
termos. Vamos estimar o número máximo de posibilidades para o termo central.

Lema: Se em uma seqüência temos um termo ak, e temos x números menores que
ak e y números maiores que ak , vamos ver que ak é termo central de no máximo
min{x, y} P.A.’s.
Para cada P.A. onde ak é o termo central, um dos outros termos é maior e o outro
menor, logo devo ter a mesma quantidade de termos maiores e menores que ak
que participam das P.A.’s de que ak é termo central.
Então se ak for termo central de j > min{x, y} P.A.’s teríamos que ter no mínimo j
números menores que ak e no mínimo j números maiores que ak, ou seja, x _ j e y
_ j, logo min{x, y}_ j. Então o número de P.A.’s com ak como termo central é no
máximo min{x, y}.

Agora vamos dividir em 2 casos:

Caso 1: n ímpar. Temos:


a1 < a 2 < a 3 < ... < a n +1 < a n + 3 < ... < a n −1 < a n
2 2
É lógico que a1 e a n não podem ser termos centrais.

EUREKA! N°12, 2001

17
Sociedade Brasileira de Matemática

Se pegarmos o número a j , temos ( j − 1) números menores que ele


(a1 , a 2 ,..., a j −1 ) e (n − j ) números maiores que ele (a j +1 , a j + 2 ,...a n ).
Logo a j é termo central de no máximo min{ j − 1, n − j} P.A.’s.
n −1
Então temos que o número de P.A.’s é menor ou igual a ∑ min{ j − 1, n − j}.
j =2

n +1
Mas para 2 ≤ j < temos j − 1 < n − j ⇒ 2 j < n + 1.
2
n +1 n +1
Para j = temos j − 1 = n − j e para < j ≤ n − 1 temos j − 1 > n − j.
2 2
Portanto, o número de P.A.’s é menor ou igual a
n −1
2
n −1 n −1


j =2
( j − 1) +
2
+ ∑ (n − j ) =
n +3
j=
2
n −3 n −3
2
n −1 2
 n − 3  n −1
= ∑i + + ∑ i = 21 + 2 + ... + + =
i =1 2 i =1  2  2
 n − 3  n − 3  
 1 +  
  2  2   n − 1  2 + n − 3  n − 3  n − 1
= 2  + 2 =   + =
2 2  2  2
 
 
 n − 1  n − 3   n − 1   n − 1  n − 3   n − 1  n − 1   n − 1 
2

=  + =  + 1 =   =  .


 2  2   2   2  2   2  2   2 
2
 n −1
É fácil ver que a seqüência (1, 2, 3,..., n) tem   P.A.’s, pois acontece
 2 
justamente o que queremos: 2 é termo central de uma P.A. o 3 de duas P.A.’s e
assim é exatamente como ocorre a igualdade.

Caso 2: n par. Temos:


a1 < a 2 < a3 < ... < a n < a n < ... < a n −1 < a n
+1
2 2
Da mesma forma a1 e a n não podem ser termos centrais.

EUREKA! N°12, 2001

18
Sociedade Brasileira de Matemática

Se pegarmos o número a j , temos ( j − 1) números menores que ele


(a1 , a 2 ,..., a j −1 ) e (n − j ) números maiores que ele (a j +1 , a j + 2 ,...a n ).
Logo a j é termo central de no máximo min{ j − 1, n − j} P.A.’s.
n n
Mas para 2≤ j≤ , temos j − 1 < n − j e para + 1 ≤ j ≤ n − 1, temos
2 2
j − 1 > n − j , logo o número de P.A.’s é menor ou igual a
n
n −1 2 n −1

∑ min{ j − 1, n − j} = ∑ ( j − 1) +
j =2 j =2
∑ (n − j ) =
n
j = +1
2
n n
−1 −1
2 2
  n 
= ∑ i + ∑ (i ) = 21 + 2 + ... +  − 1  =
i =1 i =1   2 
n  n  
  − 1 + 1 − 1 
= 2 
2  2   =  n  n − 1 = n(n − 2) .
 2   2  2  4
 
 
n(n − 2 )
Mas é fácil ver que a seqüência (1, 2, ..., n) tem P.A.’s, porque ela
4
satisfaz todas as igualdades.
Logo o número máximo de P.A.’s é:
2
 n −1
Para n ímpar:  
 2 
n(n − 2)
Para n par: .
4

PROBLEMA 5
Num tabuleiro de 2000 × 2001 as casas têm coordenadas (x, y) com x, y inteiros,
0 ≤ x ≤ 1999 e 0 ≤ y ≤ 2000. Uma nave no tabuleiro move-se da seguinte
maneira:
antes de cada movimento, a nave está numa posição (x, y) e tem uma velocidade
(h, v) onde h e v são inteiros. A nave escolhe uma nova velocidade (h', v') de
forma que h' – h seja igual a –1, 0 ou 1 e v'– v seja igual a –1, 0 ou 1. A nova
posição da nave será (x', y') onde x' é o resto da divisão de x + h' por 2000 e y' é o
resto da divisão de y + v' por 2001.

EUREKA! N°12, 2001

19
Sociedade Brasileira de Matemática

Há duas naves no tabuleiro: a marciana e a terrestre que quer capturar a marciana.


Inicialmente cada nave está numa casa do tabuleiro e tem velocidade (0, 0).
Move-se primeiro a nave terrestre e continuam movendo-se alternadamente.
Existe uma estratégia que permita sempre à nave terrestre capturar a nave
marciana, quaisquer que sejam as posições inicias?
Nota: a nave terrestre, que sempre vê a marciana, captura a marciana se depois
de um movimento seu cai na mesma posição da marciana.

SOLUÇÃO DE CARLOS STEIN NAVES DE BRITO (GOIÂNIA – GO):


A estratégia existe!
A nave terrestre deve fazer uma estratégia para ficar na mesma coordenada x,
independente do movimento da nave marciana e também na mesma coordenada
y, até que isso ocorra simultaneamente e a terrestre pegue a nave marciana.
( )
Para ficar no mesmo x : x mi , y mi é a coordenada depois de i rodadas da nave
( )
marciana e xti , y ti é a coordenada da terrestre.
Seja 0 ≤ p ≤ 1999, p ≡ x m − xt (mod 2000).
0 0

Se o primeiro h’ escolhido for 1, xt0 aumenta 1 (mod 2000) .

A partir daí se a nave marciana fizer sua h’ tal que hm − hm = a, a ≤ 1, a nave


'

terrestre fará o mesmo: ht − ht = a.


'

A partir daí temos ht − hm = 1 ( ht e hm são as velocidades momentâneas da nave


terrestre e marciana, respectivamente), isso porque a terrestre já tem ht = 1 por
causa da primeira jogada. Esse ht − hm = 1 , se manterá sempre depois da jogada
terrestre, pois se nm' − nm = a e
ht' − ht = a; ht − hm = 1 ⇒ (ht' − a) − (hm' − a) = 1 ⇒ ht' − hm' = 1, logo a indução é
óbvia. Assim a cada rodada a nave marciana “anda” hm e a terrestre hm + 1. Logo
a cada rodada da nave terrestre temos (tudo mod 2000):
   
0 0  0 1   0 ' 
( ) (
x − x = p;  x m + hm  −  xt + hm + 1  ≡ x m0 − x10 − 1 ≡ p − 1; )

 

m t

 x1m   x1t 
( ) ( ( ))
x m2 − xt2 ≡ x m' + n m2 − xt' + n m2 + 1 ≡ x m' − xt' − 1 ≡ ( p − 1) − 1 ≡ p − 2,
Assim por indução finita: ( se x mk −1 − xtk −1 ≡ p − (k − 1) )
x −x ≡ x
k
m
k
t ( k −1
m +h k
m )− (x k −1
t + h +1 ≡ x
k
m ) ( k −1
m −x k −1
t )− 1 ≡ p − k.
EUREKA! N°12, 2001

20
Sociedade Brasileira de Matemática

Logo depois k jogadas terrestres a diferença entre as coordenadas


x m e xt é p − k (mod 2000), assim para k x = 2000a + p, para qualquer
inteiro a ≥ 0, p − k ≡ 0(mod 2000), assim estarão na mesma coordenada x.
O estudo para as coordenadas y m e y t é análogo, como a nave terráquea tendo
v1 = 1 , e depois alternando sua velocidade igual à marciana, mas agora tudo
módulo 2001, logo se a diferença inicial x m − y t = q temos que depois de k’
0 0

jogadas a diferença será. q − k ' (mod 2001), logo, para k y = 2001b + q, estarão no
mesmo y.
Então a terrestre capturará a marciana quando está no mesmo x e no mesmo y, o
que acontece quando o número de rodadas k satisfaz k x e k y :
k = k x = k y ⇒ 2000a + p = 2001b + q, a, b ∈ 1 ⇒ 2001b − 2000a = p − q.
Achar solução para isso é fácil: é só adotar a = b = p – q, que é obviamente
solução da equação e se p – q < 0, é só pegar b = p – q + 2000 ≥ 0 e
a = p − q + 2001 ≥ 0, que também são soluções e agora positivas. Logo na jogada
terrestre de número k = 2001 p − 2000q + 2000 ⋅ 2001, ela terá o mesmo x e y da
marciana, capturando-a.

PROBLEMA 6
Demonstrar que é impossível cobrir um quadrado de lado 1 com cinco quadrados
1
iguais de lado menor que .
2

SOLUÇÃO OFICIAL:
D C

v
A u B

EUREKA! N°12, 2001

21
Sociedade Brasileira de Matemática

Seja ABCD um quadrado unitário e suponhamos que é possível cobri-lo


1
utilizando cinco quadrados iguais de lado a < . Então, como o diâmetro de cada
2
2
um dos quadrados pequenos é a 2 < , ou seja, menor que a metade da
2
diagonal de ABCD, os vértices A, B, C, D e o centro O de ABCD devem pertencer
a quadrados distintos. Chamaremos de QA, QB, QC, QD e QO estes quadrados.
Para obter uma contradição, demonstraremos primeiro que a parte do perímetro
que cobre cada um dos quadrados QA, QB, QC, QD tem comprimento menor ou
igual a 2a.
Consideremos, por exemplo, QA; como contém a A, o quadrado intersecta ABCD
tal como mostra a figura.
Sejam u, v as porções dos lados AB, AD, contidas em QA. Então
u + v ≤ 2(u 2 + v 2 ) e u 2 + v 2 é a hipotenusa de um triângulo retângulo contido
em QA. Logo, u 2 + v 2 ≤ a 2 e, em conseqüência, u + v ≤ 2 .a 2 = 2a.
Segue que QA, QB, QC, QD em conjunto cobrem uma porção do perímetro de
ABCD de comprimento total menor ou igual a 8a < 4. Então, o quinto quadrado,
QO, deve ter interseção não vazia com o perímetro de ABCD. Digamos, por
exemplo, que intersecte AB.
D C

E F
O

A B

Vamos ver que os cinco quadrados em conjunto não podem cobrir


simultaneamente os segmentos AB, CD, e EF, onde E e F são os pontos médios
de AD e BC.
A demonstração se apoia no lema a seguir:
1 1
Dadas duas retas paralelas l e m a distância e um quadrado de lado a <
2 2
que tem interseção não vazia com cada uma das retas l e m, a soma dos

EUREKA! N°12, 2001

22
Sociedade Brasileira de Matemática

comprimentos dos segmentos da interseção do quadrado com as retas é menor


que a.
Antes de demonstrar o lema, vejamos que é suficiente para completar a solução.
De fato, do lema se conclui que ao menos k ≥ 2 dos quadrados QA, QB, QC, QD
devem ter interseção com EF (um destes quadrados mais QO cobrirão em
conjunto uma porção menor que 2a < 1 do comprimento do segmento EF).
Então, novamente pelo lema, esses k quadrados e QO cobrem uma porção do
comprimento menor que (k + 1)a dos segmentos AB, CD, e EF.
Os restantes (4 – k) quadrados intersectam exatamente um dos segmentos AB e
CD, e cada um deles pode cobrir uma porção de comprimento no máximo
a 2 do segmento correspondente. Assim, a porção de AB, CD e EF coberta
pelos cinco quadrados é menor que:
(k + 1)a + (4 − k )a 2 = ka(1 − 2 ) + a (1 + 4 2 ).
1
Como 1 − 2 < 0, k ≥ 2 e a < , observamos que
2

( ) ( ) ( ) (
ka 1 − 2 + a 1 + 4 2 ≤ 2a 1 − 2 + a 1 + 4 2 = a 3 + 2 2 <) ( )3+ 2 2
2
<3
o que contradiz que AB, CD e EF estão cobertos.

M
h1
l

m
h2
K

1
Falta demonstrar o lema. Se o quadrado KLMN de lado a < intersecta as retas
2
1
paralelas l e m que estão a distância, então dois vértices de KLMN devem estar
2
em lados distintos da banda determinada por l e m (também podem estar em l e
1
m). A condição a < implica que esses dois vértices de KLMN são opostos,
2
digamos K e M.
Então, pela mesma condição, os restantes dois vértices do quadrado devem estar
contidos na faixa comprendida entre l e m. Sejam h1 e h2 as alturas

EUREKA! N°12, 2001

23
Sociedade Brasileira de Matemática

correspondentes às hipotenusas dos triângulos retângulos semelhantes que são as


duas porções do quadrado que estão fora da faixa comprendida entre l e m, e
denotamos ϕ ao menor dos ângulos agudos dos triângulos. Então, o ângulo
comprendido entre KM e qualquer reta perpendicular a l e m é 45° − ϕ e a
projeção de KLMN sobre uma reta mede a 2 cos(45° − ϕ ). Logo,

h1 + h2 = a 2 cos(45° − ϕ ) − = a (senϕ + cos ϕ ) − .


1 1
2 2
A soma das hipotenusas consideradas é:
a (senϕ + cos ϕ ) −
1
h1 + h2
S = (h1tgϕ + h1 cot ϕ ) + (h2 tgϕ + h2 cot ϕ ) = = 2.
senϕ cos ϕ senϕ cos ϕ
Como senϕ + cos ϕ < 1 + senϕ cos ϕ equivale à desigualdade evidente

(1 − senϕ )(1 − cos ϕ ) > 0 e a < 1 , obtemos


2
a(senϕ + cos ϕ ) − a(1 + senϕ cos ϕ ) −
1 1 1
a−
S= 2< 2 =a+ 2 < a.
senϕ cos ϕ senϕ cos ϕ senϕ cos ϕ
A demonstração está completa.

EUREKA! N°12, 2001

24
Sociedade Brasileira de Matemática

COMO FERMAT E BÉZOUT PODEM SALVAR O DIA


Antonio Caminha Muniz Neto, Fortaleza - CE

♦ Nível Avançado.

Certamente você, leitor, tem alguma familiaridade com os fatos mais


básicos da teoria elementar dos números. Portanto, não objetivo desenvolvê-los
aqui de modo sistemático. Para isso você pode consultar [1], [2] ou [3] nas
referências. O que vou fazer é mostrar como dois resultados particulares, os
teoremas de Bézout e Fermat, podem ser usados para abordar com sucesso alguns
problemas interessantes. Para facilitar a leitura, vamos relembrar alguns conceitos
e provar os resultados mais centrais para nós.

Definição 1: Dados dois inteiros não nulos a e b, definimos o máximo divisor


comum (mdc) como o maior inteiro d que divide ambos a e b.

A definição acima certamente faz sentido, uma vez que a e b têm divisores
comuns (1, por exemplo) e qualquer inteiro que divida a e b deve ser, em
particular, menor ou igual a a. Assim, realmente existe um maior inteiro que
divide a e b. Caso esse maior inteiro seja igual a 1, dizemos que a e b são primos
entre si, ou ainda relativamente primos. Para nossos propósitos, o seguinte
resultado sobre o mdc de dois inteiros será suficiente:

Teorema 1 (Bézout): Sejam a e b inteiros não nulos dados e d seu mdc. Então
existem inteiros x e y tais que d = ax0 + by0. Mais ainda, se a e b são positivos,
podemos escolher x0 > 0 e y0 < 0, ou vice-versa.

Prova: Seja S o conjunto dos inteiros da forma ax + by, com x e y inteiros.


Escrevendo a= a×(±1) + b×0, concluímos quea∈ S, e portanto S contém
inteiros positivos. Podemos então escolher o menor inteiro positivo pertencente a
S, o qual vamos denotar por d. Afirmamos que tal menor elemento positivo d é o
mdc de a e b.

Desde que d está em S, devem existir inteiros x0 e y0 tais que d = ax0 + by0. Para
provar que d divide a, dividamos a por d: a = dq + r, com 0 ≤ r < d.
r = a – dq = a – (ax0 + by0)q = a(1 – x0q) + b(– y0q),
que por definição está em S. O fato de ser d o menor inteiro positivo em S,
juntamente com 0 ≤ r < d, implica que r = 0, e assim d divide a. Analogamente
mostramos que d divide b. Por outro lado, se d’ for qualquer outro divisor comum
de a e b, segue que d’ divide ax + by, quaisquer que sejam os inteiros x e y. Em

EUREKA! N°12, 2001

25
Sociedade Brasileira de Matemática

particular, d’ divide d = ax0 + by0, de modo que d’ ≤ d. Isso prova ser d o mdc de
a e b.
Para o que falta, analisemos somente o caso em que a, b > 0 (a análise dos demais
casos é totalmente análoga). Como d = ax0 + by0 = a(x0 + tb) + b(y0 – ta),
x0 y 0
escolhendo t > − , obtemos x0 + tb > 0 > y0 – ta.
b a

Corolário 1.1: Dois inteiros não nulos a e b são primos entre si se e só se existirem
inteiros x e y tais que ax + by = 1.

O outro resultado que usaremos nos problemas a seguir é o pequeno teorema de


Fermat. Recordemos seu enunciado e prova:

Teorema 2 (Fermat): Dados inteiros a > 1 e p primo, tem-se a ≡ a (mod p).


p

Prova: Se a for múltiplo de p nada há a fazer. Senão, como p é primo temos que a
e p são primos entre si.
Considere agora os números a, 2a, 3a, ...., (p – 1)a. Dados 1 ≤ i < j ≤ p – 1, como
ja – ia = (j – i)a é um produto de dois números não divisíveis por p, temos que ja
– ia não é divisível por p. Em linguagem de congruências isso é o mesmo que
ja ≡/ ia (mod p ) . Também, nenhum dos números ja é múltiplo de p, e portanto
os restos dos números a, 2a, 3a, ...., (p – 1)a na divisão por p formam uma
permutação de 1, 2, 3, ..., p – 1. Voltando às congruências, isso implica que
a×2a×3a× .... ×(p – 1)a ≡ 1×2×3× .... ×(p – 1) (mod p),
ou ainda
(p – 1)!ap – 1 ≡ (p – 1)! (mod p)
Mas (p – 1)! é relativamente primo com p, e portanto pode ser cancelado em
ambos os membros da última congruência acima, dando o resultado desejado.

Após essa breve revisão(?), vamos aos problemas!

PROBLEMAS RESOLVIDOS

Problema 1: Sejam a e b inteiros positivos primos entre si. Então todo inteiro c
maior ou igual que o número (a – 1)(b – 1) pode ser escrito da forma c = ar + bs,
com r, s ≥ 0. Mais ainda, o menor inteiro com essa propriedade é (a – 1)(b – 1).

Solução: Dado c inteiro, o fato de serem a e b primos entre si garante que existem
inteiros x e y tais que c = ax + by (você entendeu por quê?). Seja agora y = da +

EUREKA! N°12, 2001

26
Sociedade Brasileira de Matemática

s, onde 0 ≤ s < a. Temos c = ax + b(da + s) = a(x + bd) + bs. Seja r = x + bd. Se c


≥ (a – 1)(b – 1) então (a – 1)(b – 1) ≤ c = ar + bs ≤ ar + b(a – 1), de modo que ar
≥ – (a – 1) , e portanto r ≥ 0.

Resta mostrarmos que (a – 1)(b – 1) – 1 = ab – a – b não pode ser escrito da


forma ar + bs, com r, s ≥ 0. Supondo o contrário, seja ab – a – b = ar + bs, onde
r, s ≥ 0. Então temos a (b – 1 – r) = b(s + 1). Como a e b são primos entre si,
segue que a divide s + 1 e b divide b – 1 – r. Como b – 1 – r < b, deve ser b – 1 –
r ≤ 0, ou ainda r ≥ b – 1. Também, como s + 1 > 0 e a divide s + 1, deve ser s + 1
≥ a, ou s ≥ a – 1. Mas aí, ar + bs ≥ a(b – 1) + b(a – 1) = 2ab – a – b > ab – a – b
= (a – 1)(b – 1) – 1, uma contradição.

Problema 2: (Seleção da Romênia para IMO) Prove que não existe um inteiro n >
1 tal que n divida 3n – 2n.
n n
Prova: Suponha o contrário, isto é, que para algum inteiro n > 1 tenhamos 3 – 2
≡ 0 (mod n). Obviamente 2 e 3 não dividem n. Seja agora p o menor fator primo
de n e n = pm (aqui é que usamos ser n > 1, para garantir que n tem fator primo).
Nossa hipótese, juntamente com o pequeno teorema de Fermat, nos dão:
3n ≡ 2n (mod n) ⇒ 3mp ≡ 2mp (mod p) ⇒ 3m ≡ 2m (mod p) (*)
Se d = mdc(m, p – 1), temos em particular que d divide n. Portanto, o fato de ser p
o menor divisor primo de n implica que d = 1. Tome então inteiros positivos x e y
satisfazendo mx = (p – 1)y + 1. O pequeno teorema de Fermat de novo,
juntamente com (*), nos dão
3 ≡ 3(p – 1)y + 1 = 3mx ≡ 2mx = 2(p – 1)y + 1 ≡ 2 (mod p),
o que é um absurdo.

Problema 3: Sejam m e n inteiros positivos. Determine o polinômio mônico p, de


maior grau possível, que divide simultaneamente os polinômios xm – 1 e xn – 1 .
d
Solução: Primeiro, não é difícil vermos que, sendo d o mdc de m e n, então x – 1
m n
divide ambos x – 1 e x – 1. De fato, seja por exemplo m = dk, com k > 0 inteiro.
Então
xm – 1 = (xd)k – 1 = (xd – 1)(xd(k – 1) + xd(k – 2) + .... + xd + 1)

Mostrar que xd – 1 divide xn – 1 é análogo. A parte mais difícil é mostrar que xd –


1 é o polinômio mônico p de maior grau que divide ambos xm – 1 e xn – 1, e é para
isso que precisamos de Bézout. Seja p um polinômio mônico que divide ambos xm
– 1 e xn – 1 e z uma raiz complexa de p. Como p divide xm – 1 e xn – 1, temos que
z é raiz de ambos esses polinômios. Em outras palavras, zm = zn = 1. Mas o

EUREKA! N°12, 2001

27
Sociedade Brasileira de Matemática

teorema de Bézout garante que existem inteiros u e v tais que mu + nv = 1. Isso


nos dá
zd = zmu + nv = (zm)u×(zn)v = 1u×1v = 1,

e assim z é raiz de xd – 1. Como toda raiz de p é também raiz de xd – 1 e como xd


– 1 só tem raízes simples, segue que p divide xd – 1. Portanto, xd – 1 é o
polinômio mônico de maior grau que divide ambos xm – 1 e xn – 1.

Problema 4: (The William Lowell Putnam Competition) Sejam m e n inteiros


mdc (m, n)  m 
positivos, com m ≥ n. Prove que   é inteiro.
m n

Prova: Para esse problema usamos o teorema de Bézout de um modo bastante


x m
elegante. Seja S o conjunto dos inteiros x tais que   seja inteiro. Veja que m
mn
está em S já que números binomiais são inteiros. Também n está em S, pois
n  m n m!  m − 1
  = =  
m   m (m − n)! n!  n − 1 
n
Por outro lado note que, se x e y estiverem em S, então ux + vy também estará em
S, quaisquer que sejam u e v inteiros. De fato,
ux + vy  m  x  m y  m
  = u ×   + v ×   ,
m n mn mn
que é um inteiro. Como o mdc de m e n pode ser escrito da forma mu + nv, para
algum par de inteiros u e v, segue que mdc(m, n) está em S.

Problema 5: (Seleção do Brasil para a IMO) Determine todas as funções


f : 4 + →= satisfazendo as seguintes condições:
*

i. f (1999) = 1.
ii. f (ab) = f (a) + f (b), para todos os racionais positivos a, b.
iii. f (a + b) ≥ min {f (a), f (b)}, para todos os racionais positivos a, b.

Solução: Fazendo a = b = 1 em ii. Vem que f (1) = 2f (1), donde f (1) = 0. Daí,
dados inteiros positivos m e n, temos
m 1 1
f   = f (m) + f   e f   + f (n) = f (1) = 0,
n n n
donde

EUREKA! N°12, 2001

28
Sociedade Brasileira de Matemática

m
f   = f (m) – f (n)
n
Assim, basta calcularmos os valores de f (n), com n inteiro positivo. Seja n =
a a
p1 1 .... p k k a decomposição de n em fatores primos. Usando ii. várias vezes, vem
que f (n) = a1 f (p1) + .... + ak f (pk), de modo que basta calcularmos f (p), com p
primo. Afirmamos que f (n) ≥ 0 para todo inteiro positivo n. Para provar esse fato,
façamos indução sobre n. Já vimos que f (1) = 0. Suponhamos agora que f (n) ≥ 0
para algum inteiro positivo n. Então f (n + 1) ≥ min {f (n), f (1)} ≥ 0, e isso
termina nossa indução. Afirmamos agora que f (2) = 0 ou f (3) = 0. De fato, como
1999 = 3×9 + 493×4, temos que
1 = f (1999) ≥ min {f (3) + f (9), f (493) + f (4)} ≥

≥ min {3f (3), 2f (2)},


e daí segue o afirmado. Suponhamos que f (2) = 0 ( f (3) = 0 é análogo), e seja p
um primo diferente de 2 e de 1999. Tomando k inteiro suficientemente grande, o
problema 1 garante a existência de inteiros positivos x e y tais que px + 1999y =
2k. Então
0 = f (2k) ≥ min { f (p) + f (x), f (1999) + f (y)}

Como f (1999) + f (y) ≥ 1, segue que f (p) = 0. Então, sendo n = 1999am, com
1999 e m primos entre si, segue que f (n) = a. Mas é imediato verificar que tal
função satisfaz as condições impostas no enunciado.

PROBLEMAS PROPOSTOS

Problema 6: Generalizando a teoria desenvolvida acima e o problema 1, sejam a1,


a2, ...., an (n ≥ 2) inteiros positivos tais que mdc(a1, a2, ...., an) = d (a definição de
mdc que demos no início se aplica nesse caso ipsis literis). Prove que:

i. Existem inteiros x1, x2, ...., xn tais que a1x1 + a2x2 + .... + anxn = d.
ii. Se d = 1, mostre que existe um inteiro positivo m0 tal que todo inteiro m ≥ m0
pode ser escrito da forma a1y1 + a2y2 + .... + anyn, com y1, y2, ...., yn ≥ 0.
(sugestão: para o item i. imite a prova do texto acima. Para ii., que tal usar
indução sobre n > 1? Para uma cota mais precisa para m0, veja [2]).

Problema 7: Considere duas progressões aritméticas infinitas e não constantes de


inteiros positivos. Prove que existem infinitos naturais termos de ambas as

EUREKA! N°12, 2001

29
Sociedade Brasileira de Matemática

seqüências se e só se o mdc de suas razões dividir a diferença entre seus termos


iniciais.
(sugestão: use Bézout. É fácil!).

Problema 8: (The William Lowell Putnam Competition) Prove que não existe
inteiro n > 1 tal que n divida 2n – 1.

(sugestão: use as idéias que apareceram na prova do problema 2).

Problema 9: (Olimpíada Búlgara) Determine todos os primos p, q tais que pq


divida o número (5p – 2p)(5q – 2q)
(sugestão: se q dividir 5p – 2p e p ≥ q, então mdc(p, q – 1) = 1. Como antes, use
Bézout e Fermat).

Problema 10: (Seleção da Romênia para IMO) Sejam p, q números primos. Se q


dividir 2p + 3p, prove que q > p ou q = 5.
(sugestão: se p > 3 e q ≤ p, então q – 1 < p, donde q – 1 e p são primos entre si.
Mais uma vez use Bézout e Fermat).

REFERÊNCIAS

[1] Introdução à Teoria dos Números. Antônio Plínio dos Santos. IMPA. Rio de Janeiro,
1998.

[2] Olimpíadas de Matemática, um Curso de Introdução, vol. 2. Antonio Caminha.


Editora Ipiranga. Fortaleza, 2001 (a ser publicado).

[3] Divisibilidade,Congrências e Aritmética módulo n. Carlos Gustavo Moreira, Eureka!


no.2 (1998),pp. 41-52.

EUREKA! N°12, 2001

30
Sociedade Brasileira de Matemática

GRAFOS E CONTAGEM DUPLA


Carlos Yuzo Shine, Colégio Etapa

♦ Nível Intermediário.

1. GRAFOS

1.1 O que são e para que servem grafos?

Define-se grafo como o par (V, A) onde V = {v1, v2,...,vn} é um conjunto de


vértices e A ⊂ {{vi, vj}t.q. vi, vj ∈ V, i≠ j} é um conjunto de arestas (na verdade,
uma aresta é um par não-orientado de vértices).

A representação mais comum de grafos é associar os vértices a pontos e as arestas


a linhas que ligam os pares de vértices que as formam.

Mas o mais importante é que os grafos podem representar inúmeras situações. Por
exemplo, quando você brinca de ligar os pontos, no fundo você está traçando
arestas em um grafo onde os vértices são dados (em vez de “ligue os pontos”,
poderíamos escrever “areste o grafo”...).

Embora pareçam simples, os grafos têm muito mais utilidades, como veremos.
Na verdade, a Teoria dos Grafos é uma das partes mais importantes da
Matemática, e é muito utilizada principalmente em computação.

Exemplo 1.1
Podemos construir um grafo que represente pessoas apertando mãos. Os vértices
seriam as pessoas. Ligamos dois vértices (formando assim uma aresta) se duas
pessoas se cumprimentaram.
Edmilson
Gustavo

Carlos Eduardo

Onofre Paulo
Emanuel

EUREKA! N°12, 2001

31
Sociedade Brasileira de Matemática

Exemplo 1.2
É possível que os cavalos do tabuleiro (I) fiquem na posição do tabuleiro (II) ?

(I) (II)
Observação: um cavalo, no xadrez, se movimenta da seguinte forma: ele se move
duas casas na vertical ou horizontal e depois se move uma casa na direção
perpendicular à direção em que havia se movimentado antes.

Resolução
Vamos numerar as casas do tabuleiro da seguinte forma:

1 2 3

4 5 6

7 8 9

Vamos construir um grafo onde os vértices são as casas do tabuleiro. Ligaremos


dois vértices i e j se é possível um cavalo ir da casa i à casa j. Temos então o
seguinte grafo (verifique!).

EUREKA! N°12, 2001

32
Sociedade Brasileira de Matemática

1
6 8

5 3
7

2 4

Coloquemos agora os cavalos nas situações inicial e final, respectivamente:


1 1
6 8 6 8

5 3 5 3
7 7

2 4 2 4

9 9
Inicial Final
Observe que não podemos ter dois cavalos na mesma casa, assim os cavalos
devem sempre estar na mesma ordem no ciclo. Logo não é possível chegar na
posição final.

Exercícios
01. (IMO) Considere um inteiro positivo r e um retângulo de dimensões
AB = 20, BC = 12. O retângulo é dividido em uma grade de 20 × 12 quadrados
unitarios. Uma moeda pode ser movida de um quadrado a outro se, e somente se,
a distância entre os centros dos quadrados é r . A tarefa é encontrar uma
seqüência de movimentos que levem uma moeda do quadrado que tem A como
vértice ao quadrado que tem B como vértice.

a) Mostre que a tarefa não pode ser feita se r é divisível por 2 ou 3.


b) Prove que a tarefa pode ser feita se r = 73.
c) Pode a tarefa ser feita quando r = 97?

Dicas: Para o item a), use o fato de que um quadrado perfeiro pode deixar
somente os restos 0 ou 1 quando divididos por 3 ou 4. Para os itens b) e c),

EUREKA! N°12, 2001

33
Sociedade Brasileira de Matemática

construa dois grafos: um que considera a posição da moeda na horizontal e outro


na vertical.

1.2. Grau de vértice


Definimos grau de vértice vi como o número de arestas que contêm vi e
denotamos d(vi). No último exemplo, o grau de um vértice seria o número de
apertos de mão que a pessoa correspondente deu.

Exemplo 1.3.
Na cidade de Micrópolis, há 7 telefones. Um candidato a prefeito prometeu que
ampliaria a rede de telefonia de modo que cada um dos 7 telefones esteja
conectado diretamente a exatamente 5 outros telefones. É possível que ele
cumpra sua promessa?

Resolução
Se imaginarmos um grafo onde os vértices são os telefones e as arestas, as
conexões, teríamos que o grau de cada vértice seria 5.

Vamos contar o número de conexões entre dois telefones (ou seja, o número de
arestas do grafo). Como de cada telefone sairiam 5 conexões, teríamos a princípio
5.7 = 35 conexões; mas contamos cada conexão duas vezes, uma vez em cada um
dos dois telefones a que ele está conectado. Assim, deveríamos ter na verdade
35/2 conexões, o que seria um absurdo. Assim, o candidato a prefeito não pode
cumprir sua promessa (não votem nele!!).

Este exemplo mostra

1.3. Um teorema importante

Teorema. Em um grafo, a soma dos graus de todos os vértices é igual ao dobro do


número de arestas. Em símbolos: no grafo (V, A),

d (v i ) = 2 A
vi ∈V

( X denota o número de elementos do conjunto X.)

Demonstração
De cada vértice v saem d(v) arestas. Assim, se somarmos os graus de todos os
vértices, obtemos o número de arestas multiplicado por dois, pois contamos cada
aresta duas vezes (lembre-se de que cada aresta está associada a dois vértices).

EUREKA! N°12, 2001

34
Sociedade Brasileira de Matemática

2. Contagem Dupla
O que acabamos de fazer foi contar algo de duas maneiras diferentes, no caso o
número de arestas (na verdade, o seu dobro). Esta idéia de contar duas vezes é às
vezes muito útil para demonstrar algumas relações.

Exemplo 2.1.
(Combinações) De quantos modos podemos escolher k elementos dentre n
disponíveis?
Importante: Tal número é representado por  n  – lê-se n escolhe k ou
 
k 
combinação de n k a k.

Resolução
Vamos contar de duas maneiras o número de filas com os k elementos escolhidos.
Podemos (i) primeiro escolher os k elementos e colocá-los em filas ou (ii)
escolher diretamente os elementos e irmos colocando na fila.

Fazendo como em (i), temos  n  maneiras de escolhermos os elementos;


 
k 
podemos escolher o primeiro da fila de k maneiras, o segundo de k – 1 maneiras,
e assim por diante. Assim temos
n n
  ⋅ k ⋅ (k − 1) ⋅ ... ⋅ 1 =   ⋅ k!
k k 
maneiras de formar a fila. (lembrete: k ⋅ (k − 1) ⋅ ... ⋅ 1 = k! – lê-se k fatorial).
Por outro lado, fazendo como em (ii), temos n maneiras de escolher o primeiro da
fila, n – 1 maneiras de escolher o segundo e assim por diante, até o último, que
pode ser escolhido de n – k + 1 maneiras. Assim, temos n ⋅ (n − 1) ⋅ ... ⋅ (n − k + 1)
maneiras de formar a fila. Logo, de (i) e (ii), concluímos que
n
  ⋅ k! = número de filas = n ⋅ (n − 1) ⋅ ... ⋅ (n − k + 1)
k 
 n  n ⋅ (n − 1) ⋅ ... ⋅ (n − k + 1) (n − k )! n!
⇒   = ⋅ =
k  k! (n − k )! k!(n − k )!

Exemplo 2.2.
(Lema de Sperner) Dividimos um triângulo grande 123 em triângulos menores de
modo que quaisquer dois dentre os triângulos menores ou não têm ponto em
comum, ou têm um vértice em comum ou tem um lado (completo) em comum.

EUREKA! N°12, 2001

35
Sociedade Brasileira de Matemática

Os vértices dos triângulos são numerados: 1, 2 ou 3. A numeração é arbitrária,


exceto que os vértices sobre os vértices do triângulo maior oposto ao vértice i não
podem receber o número i. Mostre que entre os triângulos menores existe um
com os vértices 1, 2, 3.
1

1 1 3

1
1 2 1
1

2
2 2 2 3 3

Resolução
Contaremos o número de segmentos 12 (com algumas repetições). Eles
aparecem nos triângulos
1 1 1

3
2 1 2
2 2

Digamos que há x triângulos 123, y triângulos 122 e z triângulos 112. Observe


que os segmentos 12 internos ao triângulo grande são contados duas vezes (eles
são comuns a dois triângulos) e os segmentos do lado do triângulo grande,
somente uma vez. Notemos também que os segmentos 12 aparecem duas vezes
nos triângulos 122 e 112 e uma vez nos triângulos 123. Assim,

2 segmentos interiores + segmentos nos lados = número de segmentos = x + 2 y + 2 z

Mostraremos um fato mais forte que o lema: provaremos que x é ímpar e portanto
não pode ser zero.
Observando a equação acima, vemos que basta provarmos que o número de
segmentos 12 sobre os lados do triângulo grande é ímpar.
Como não podemos ter pontos 1 no lado 23 nem pontos 2 no lado 13 , todos os
segmentos 12 estão sobre o lado 12 do triângulo grande. Provemos que o número
de segmentos sobre o lado é ímpar. Para isso, vamos “colocar” vértices 1 ou 2 no
lado 12 . Assim, no começo, temos somente o lado 12 :

EUREKA! N°12, 2001

36
Sociedade Brasileira de Matemática

1 2

Na hora de colocar vértices, considere o menor segmento em cujo interior


colocaremos o vértice. Poderemos estar em uma das seguintes situações:
• Este segmento é do tipo 11 :
1 ou 2
1 ↓ 1 2

Se colocarmos 1, o número de segmentos 12 não muda; se colocarmos 2,


aumenta de 2. De qualquer forma, a paridade do número de segmentos 12 não
muda.
• Este segmento é do tipo 22 :
1 ou 2
1 2 ↓ 2

Se colocarmos 1, o número de segmentos 12 aumenta de 2; se colocarmos 2, não


muda. De qualquer forma a paridade do número de segmentos 12 não muda.
• Este segmento é do tipo 12 :
1 ou 2
1 1 ↓ 2

Se colocarmos 1 ou 2, o número de segmentos 12 não muda e é claro que a


paridade desse número não muda também.
Logo a paridade do número de segmentos 12 nunca muda (ou seja, é invariante).
Como no começo temos um segmento 12 (o próprio lado 12 ), temos que o
número de segmentos 12 no lado do triângulo grande é sempre ímpar, o que
completa nossa demonstração.
Contar algo de duas maneiras também nos ajuda a demonstrar desigualdades.

Exemplo 2.3.
Na terra de Oz há n castelos e várias estradas, sendo que cada uma liga dois
castelos e não há mais do que uma estrada ligando diretamente dois castelos. Diz
a lenda que se houver quatro castelos ligados em ciclo (ou seja, se existirem
quatro castelos A, B, C e D tais que A e B, B e C, C e D e D e A estão ligados), um
dragão aparecerá do centro dos castelos e destruirá a Terra de Oz. Mostre que
para esta desgraça não acontecer o número de estradas deve ser menor ou igual a
( )
1 + 4n − 3 n / 4.

EUREKA! N°12, 2001

37
Sociedade Brasileira de Matemática

Resolução
Considere um castelo ligado a outros dois.

 d (v) 
Para cada castelo v do conjunto V dos castelos temos   pares de estradas.
 2 
Para a desgraça não ocorrer, observemos que devemos Ter no máximo um par de
estradas asociado a um mesmo par de castelos. Assim, a quantidade de pares de
estradas é menor ou igual à quantidade de pares de castelos. Logo
 d (v )  n
∑ 
v∈V  2 
 = pares de estradas ≤ pares de castelos =  
2
⇒ ∑ ((d (v ))
v∈V
2
)
− d (v ) ≤ n 2 − n

⇔ ∑ (d (v )) − ∑ d (v ) ≤ n 2 − n
2
(*)
v∈V v∈V

Sabemos que a soma ∑ v∈V


d (v) é igual ao dobro do número de estradas
2 A . Além disso, pode-se mostrar (usando a desigualdade entre as médias
quadrática e aritmética, ou mesmo Cauchy-Schwarz) que
(∑ d (v) )
2
4A
2

∑ (d (v))
v∈V
2
≥ v∈V
n
=
n
Assim
2

(*) ⇒
n
4A 2
− 2 A ≤ n 2 − n ⇔ 4 A − 2n A − n n 2 − n ≤ 0 (**) ( )
Resolvendo (**) em A , obtemos
n − n 4n − 3
4
≤ A≤
n + n 4n − 3
4
n
⇒ A ≤ 1 + 4n − 3 .
4
( )

EUREKA! N°12, 2001

38
Sociedade Brasileira de Matemática

Exercícios
02. Dizemos que dois poliedros P e Q são equidecomponíveis se é possível cortar
o poliedro P em vários poliedros menores e montar, sem deixar espaços vazios e
sem sobrar poliedros, o poliedro Q. Sejam α 1 , α 2 ,..., α m os ângulos diédricos
(ângulos entre faces adjacentes) de P e β 1 , β 2 ,..., β m os ângulos diédricos de Q.
Mostre que se P e Q são equidecomponíveis então existem números inteiros
positivos a1 , a 2 ,..., a m , b1 , b2 ,..., bn e um número inteiro k tais que
a1α 1 + a 2α 2 + ... + a mα m − (b1 β 1 + b2 β 2 + ... + bn β m ) = kπ
A partir desta relação podemos mostrar (isto é um pouco mais difícil!!) que um
cubo e um tetraedro de mesmo volume não são equidecomponíveis.
03. Dado n inteiro, seja d(n) o número de divisores de n. Seja d (n) o número
médio de divisores dos números entre 1 e n, ou seja,
1 n
∑ d (n) =
n j =1
d ( j)

Mostre que
n n

∑ ∑i
1 1
≤ d ( n) ≤
i=2 i i =1

Esta desigualdade nos mostra que d (n) ≅ ln n, e que a diferença d (n) − ln n é no


máximo 1.
04. (IMO) Num concurso, há m candidatos e n juízes, onde n ≥ 3 é ímpar. Cada
candidato é avaliado por cada juiz, podendo ser aprovado ou reprovado. Sabe-se
que os julgamentos de cada par de juízes coincidem em no máximo k candidatos.
Prove que
k n −1

m 2n

Referências Bibliográficas
A parte de grafos foi baseada em um dos capítulos do livro Mathematical Circles – A
Russian Experience, que aborda o treinamento russo para as olimpíadas.
Muitos dos exercícios de contagem dupla foram extraídos do livro Proofs From The
Book, que contém as demonstrações consideradas pelos autores (e também por muitos
leitores!) as mais elegantes.

EUREKA! N°12, 2001

39
Sociedade Brasileira de Matemática

OLIMPÍADAS AO REDOR DO MUNDO


Þ O comitê editorial de EUREKA! agradece aos leitores o crescente envio de
soluções e aos inúmeros elogios para a seção OLIMPÍADAS AO REDOR DO MUNDO.
Continuamos salientando que estamos à disposição na OBM para aqueles
que estiverem interessados na solução de algum problema particular. Para tanto,
basta contactar a OBM, através de carta ou e-mail.
Antonio Luiz Santos

Primeiramente vamos aos problemas propostos deste número

121. (Rússia-2001) Os valores da função quadrática f (x ) = x + ax + b para dois


2

inteiros consecutivos são os quadrados de dois inteiros também


consecutivos. Mostre que os valores da função quadrática são quadrados
perfeitos para todos os inteiros.

122. (Rússia-2001) Determine todos os números inteiros positivos que podem ser
x2 + y
representados de maneira única sob a forma onde x e y são
xy + 1
inteiros positivos.

123. (Rússia-2001) Os senos dos ângulos de um triângulo são números racionais.


Mostre que os seus cossenos são tambem racionais.

124. (Rússia-2001) Dois círculos s1 e s 2 de centros O1 e O2 intersectam-se nos


pontos A e B . Seja M um ponto qualquer do círculo s1 tal que MA
intersecta s 2 no ponto P e MB intersecta s 2 no ponto Q . Mostre que se o
quadrilátero AO1 BO2 é cíclico (inscritível) então AQ e BP intersectam-
se em s1 .
o
125. (Rússia-2001) Eliminando-se o 2000 . algarismo da expansão decimal da
1
fração (onde p é um número primo maior que 5) obtemos a fração
p
a
irredutível . Mostre que b é divisível por p .
b

EUREKA! N°12, 2001

40
Sociedade Brasileira de Matemática

126. (Rússia-2001) Cinco números, um dos quais é 2000, são escritos em um


quadro negro. É permitido apagar qualquer um destes números e o
substituirmos pelo número a + b − c , onde a , b e c são três quaisquer dos
números restantes. É possível com estas operações obter cinco números
iguais a 2000?

127. (Estônia-2001) Quantos números inteiros positivos menores que 20002001


não contém outros algarismos distintos de 0 e 2?

128. (Estônia-2001) Em um triângulo ABC , as medidas dos seus lados são inteiros
consecutivos e a mediana relativa ao lado BC é perpendicular à bissetriz
interna do ângulo ∠ABC . Determine as medidas dos lados do triângulo
ABC .

129. (Estônia-2001) Considere todos os produtos por 2, 4, 6,...,2000 dos elementos


1 1 1 1 1 
do conjunto A =  , , ,..., ,  . Determine a soma de todos
2 3 4 2000 2001
estes produtos.

130. (Eslovênia-2001) Para os inteiros positivos x e y é verdadeira a igualdade


3x + x = 4 y + y . Mostre que x − y é um quadrado perfeito.
2 2

131. (Eslovênia-2001) Determine todos os números primos da forma 101010...101 .

132. (Eslovênia-2001) Gustavo tentou escrever alguns números utilizando somente


o algarismo 1 e o sinal de “mais”. Ele percebeu , por exemplo, que existem
apenas dois inteiros positivos n (13 e 4) para os quais o número 13 pode ser
escrito utilizando n 1’s e o sinal de “mais”. De fato, o número 13 pode ser
escrito como a soma de 13 1’s ou 11 + 1 + 1 onde 4 1’s foram utilizados.
Determine quntos são os inteiros positivos n tais que o número 125 seja
escrito utilizando-se n 1’s e o sinal de “mais”.

133. (Eslovênia-2001) Seja ABC um triângulo retângulo de hipotenusa AC .


Sabendo que sobre o lado BC existem pontos D e E tais que
∠BAD = ∠DAE = ∠EAC e EC = 2 ⋅ BD . Determine os ângulos do
triângulo.

EUREKA! N°12, 2001

41
Sociedade Brasileira de Matemática

134. (Croácia-2001) Determine todas as funções f : 5\{-1, 1} → 5 que


satisfazem à equação :
 x −3 3+ x
f + f =x
 x +1   1− x 

135. (Croácia-2001) Se x + y + z = 0 , simplifique


x7 + y7 + z 7
(
xyz x 4 + y 4 + z 4 )
Sugestão : calcule (x + y ) e (x + y )
4 6

136. (Croácia-2001). Dado o número n = p1 p 2 p 3 p 4 onde p1 , p 2 , p 3 e p 4 são


quatro números primos distintos. Sejam
d1 = 1 < d 2 < d 3 < ⋅ ⋅ ⋅ < d15 < d16 = n
os divisores positivos de n . Determine todos os n < 2001 tais que
d 9 − d 8 = 22 .

137. (Albânia-2001) Mostre as igualdades:


 3π 2π  6π 1 2π
(i)  cos 2 − sen 2  ⋅ sen = ⋅ sen
 11 11  11 4 11
3π 2π
(ii) tg + 4 ⋅ sen = 11
11 11

138. (Bielorússia-2001) Em um triângulo isósceles ABC , no qual AB = AC e


∠BAC = 30º , marcam-se os pontos Q e P sobre o lado AB e sobre a
mediana AD respectivamente, de modo que PC = PQ ( Q ≠ B ). Determine
a medida do ângulo ∠PQC .

139. (Bielorússia-2001) Eduardo escreveu todos os produtos, todas as somas e todos


os valores absolutos das diferenças dos inteiros positivos a1 , a 2 ,..., a100
tomados dois a dois. Qual o maior número de inteiros ímpares obtidos por
Eduardo?

140. (Bielorússia-2001). Um inteiro positivo k é chamado bom se existe um inteiro


positivo N com k algarismos em sua representação decimal e tal que o
i − ésimo algarismo da esquerda para a direita do número 3N é igual ao

EUREKA! N°12, 2001

42
Sociedade Brasileira de Matemática

(k − i + 1) − ésimo algarismo da direita para a esquerda do número 2N,


1 ≤ i ≤ k . Determine a soma de todos os números bons existentes entre 1 e
100.

141. (Bielorússia-2001) Mostre que


1  1 
an + − 2 ≥ n2  a + − 2
 
n
a a
para todo inteiro positivo n e todo real positivo a .

142. (Finlândia-2001) Determine n ∈ N tais que n + 2 divida 2 + 2001n .


2

143. (Itália-2001) Em um hexágono equiângulo, as medidas de quatro lados


consecutivos são, nesta ordem, 5, 3, 6 e 7. Determine as medidas dos outros
dois lados.

144. (Itália-2001) Dada a equação x = yx ,


2001

(i) determine todos os pares de soluções (x, y) tais que x seja um número
primo e y um inteiro positivo.
(ii) determine todos os pares de soluções (x, y) tais que x e y são inteiros
positivos.

145. (Israel-2001) As medidas dos lados de um triângulo ABC são 4, 5 e 6. Por um


ponto D qualquer de um de seus lados, traçamos as perpendiculares DP e
DQ sobre os outros lados. Determine o valor mínimo de PQ.

146. (Israel-2001) Dados 2001 números reais x1 , x 2 ,..., x 2001 tais que 0 ≤ x n ≤ 1
para cada n = 1,2,...,2001 determine o valor máximo de
2
 1 2001 2   1 2001 

 2001 ∑
x n  −  ∑ x n 
 n =1   2001 n =1 
e onde este máximo é atingido.

147. (Grécia-2000) Seja f : 1 → 5 uma função tal que f (1) = 3 e


f (2m ) + f (2n )
f (m + n ) + f (m − n ) − m + n − 1 =
2
para todos os inteiros não negativos m e n com m ≥ n . Determine a
expressão de f(m).

EUREKA! N°12, 2001

43
Sociedade Brasileira de Matemática

148. (Grécia-2001) Fatore a expressão


A = x 4 + y 4 + z 4 − 2x 2 y 2 − 2 y 2 z 2 − 2z 2 x 2
e mostre que a equação A = 2000 não possui solução no conjunto dos
números inteiros.

149. (Hungria-2000) O produto de 2001 inteiros positivos distintos possui


exatamente 2000 divisores primos distintos. Mostre que podemos escolher
alguns destes 2001 números de modo que seu produto seja um quadrado
perfeito.

150. (Hungria-2000) Seja S o número de subconjuntos com 77 elementos de H = {1,


2, 3,...2000, 2001} para os quais a soma dos elementos de cada subconjunto
seja par e seja N o número de subconjuntos com 77 elementos de H para os
quais a soma dos elementos de cada subconjunto seja ímpar. Determine qual
dos dois S ou N é maior e o quanto?

ÞÞÞ
Agora vamos aos comentários e soluções dos leitores para alguns dos
problemas apresentados em nossa seção nos números anteriores de EUREKA!.

9. (Irlanda-1998) Um triângulo ABC possui medidas dos lados expressas por


números inteiros, ∠A = 2 ⋅ ∠B e ∠C > 90º . Determine o valor mínimo do
perímetro deste triângulo.

SOLUÇÃO DE EINSTEIN DO NASCIMENTO JÚNIOR (FORTALEZA – CE) (com adaptações):


A resposta é 77. De fato, fazendo-se ∠B = x temos ∠A = 2 x , sejam a, b e c as
medidas dos lados opostos aos ângulos ∠A , ∠B e ∠C respectivamente.
Prolongando-se o lado CA no sentido de C para A até o ponto D de um
comprimento igual a c, vemos que o triângulo BCD é semelhante ao triângulo
ABC logo,
ou a 2 = b(b + c )
b a
=
a b+c

Como estamos procurando um triângulo com o menor perímetro possível,


podemos supor que a, b e c não possuem fatores primos comuns pois, de outra
forma um outro exemplo menor haveria. De fato, b e c devem ser primos entre si
porque qualquer fator primo comum a b e c seria também um fator de a. Alem
disso, como o produto b(b + c) é um quadrado perfeito que é igual ao produto de
dois números primos entre si tanto b quanto b + c devem eles mesmos serem

EUREKA! N°12, 2001

44
Sociedade Brasileira de Matemática

quadrados perfeitos. Deste modo, para m e n inteiros positivos quaisquer primos


entre si temos b = m2, b + c = n2, a = mn e daí
n a
= = 2 cos ∠B (pela lei dos senos)
m b
π
O ângulo ∠C = π − 3 ⋅ ∠B é obtuso assim, 0 < ∠B < o que implica em
6
3 n
< cos ∠B < 1 e portanto, 3 < < 2 . É facil ver que esta desigualdade não
2 m
possui soluções inteiras com m = 1, 2 ou 3 daí m ≥ 4 , n ≥ 7 e
a + b + c = mn + n 2 ≥ 4 ⋅ 7 + 7 2 = 77
De fato, o par (m, n) = (4, 7) gera o triângulo (a, b, c) = (28, 16, 33) que satisfaz
todas as condições do problema.

10. (Canadá-1998) Em um triângulo ABC tem-se que ∠BAC = 40º e


∠ABC = 60º . Sejam D e E pontos sobre os lados AC e AB respectivamente
tais que ∠CBD = 40º e ∠BCE = 70º . Seja F o ponto de interseção de BD e
CE. Mostre que a reta que que contém AF é perpendicular à que contém BC.

SOLUÇÃO DE LUCAS DE MELO PONTES E SILVA (FORTALEZA – CE):


Sejam X o ponto de interseção de AF com BC e H o pé da altura que parte de A.
Então pela lei dos senos temos nos triângulos CBD e BDA :
CD BD DA BD
= e =
sen40º sen80º sen20º sen40º
CD sen40º⋅sen40º
ou = .
DA sen80º⋅sen20º
EA sen60º⋅sen10º
Analogamente = . Pelo Teorema de Ceva, se AX, BD e CE
EB sen70º⋅sen40º
são cevianas concorrentes no triângulo ABC tem-se que:

CD EA BX sen40º⋅sen40º⋅sen60º⋅sen10º BX
⋅ ⋅ =1⇒ ⋅ = 1 ou
DA EB XC sen20º⋅sen80º⋅sen70º⋅sen40º XC

XC 2 sen 20º⋅ cos 20º⋅sen60º⋅ cos 80º sen60º tg 60º


= = =
BX sen 20º⋅sen80º⋅ cos 20º 1 ⋅ tg 80º tg 80º
2
Como AH temos nos triângulos AHB e AHC respectivamente:

EUREKA! N°12, 2001

45
Sociedade Brasileira de Matemática

AH AH tg 60º HC
tg 60º = e tg 80º = logo =
HB HC tg 80º HB
HC XC HC + HB XC + XB
Logo = ⇒ = e como ∠BAC é agudo então H está
HB XB HB XB
entre B e C e portanto,
BC BC
= ⇒ XB = HB ⇒ X coincide com H.
HB XB

13. (Irlanda-1999). Uma função f : 1 → 1 satisfaz às seguintes condições:


(i) f (ab ) = f (a ) ⋅ f (b ) se o máximo divisor comum de a e b é 1.
(ii) f ( p + q ) = f ( p ) + f (q ) para todos os números primos p e q.
Mostre que f (2 ) = 2, f (3) = 3 e f (1999 ) = 1999 .

SOLUÇÃO DE MARCÍLIO MIRANDA DE CARVALHO (TERESINA – PI):


Seja p um número primo ímpar. Então,
f (2 p ) = f ( p + p ) = f ( p ) + f ( p ) = 2 ⋅ f ( p )
enquanto que
f (2 p ) = f (2) ⋅ f ( p )
e daí, f (2 ) = 2.
Agora, f (4 ) = f (2) + f (2 ) = 4 ⇒ f (12 ) = 4 ⋅ f (3) . Por outro lado,
f (12 ) = f (7 ) + f (5) ⇒ f (12 ) = 2 ⋅ f (2) + f (3) + f (2 ) + f (3)
⇒ f (3) = 3.
Finalmente,
f (5) = f (2 ) + f (3) = 5 ⇒ f (15) = 15 ⇒ f (13) = 13
⇒ f (26) = 26 ⇒ f (23) = 23.
Por outro lado,
f (13) = 13 ⇒ f (11) = 11 ⇒ f (33) = 33 ⇒ f (31) = 31
⇒ f (29) = 29.
Logo,
f (2001) = f (3)⋅ f (23) ⋅ f (29 ) = 2001 ⇒ f (1999 ) = 1999

EUREKA! N°12, 2001

46
Sociedade Brasileira de Matemática

16. (Estônia-1999) Mostre que o segmento que une o ortocentro e o baricentro de


um triângulo acutângulo ABC é paralelo ao lado AB se, e somente se,
tg∠A ⋅ tg∠B = 3 .

SOLUÇÃO DE GERALDO PERLINO JÚNIOR (SÃO PAULO – SP) (com adaptações):


Sejam D e E os pés das alturas traçadas dos vértices A e C respectivamente. Se H
é o ortocentro do triângulo, observe que H pertence à uma reta paralela a AB e
CE
passando pelo baricentro se, e só se = 3 . Deste modo, basta mostrar que
EH
CE
= tg∠A ⋅ tg∠B . Como ∠AHE = ∠CHD temos também que
EH
∠EAH = ∠BCE , isto é os triângulos retângulos CEB e AEH são semelhantes e
CE AE CE
tg∠B = = . Observando que tg∠A = , obtemos a relação necessária.
EB EH AE

23. (Rússia-1999) A soma dos algarismos de um inteiro positivo n escrito no


sistema de numeração decimal é igual a 100 e a soma dos algarismos do
número 44n é 800. Determine a soma dos algarismos do número 3n.

SOLUÇÃO DE MARCÍLIO MIRANDA DE CARVALHO (TERESINA – PI):


Sejam a1 , a2 , a3 ,..., ak os algarismos de n e S (n ) a soma de seus algarismos.
Então,
44n = 40(a1 a 2 ...a k ) + 4(a1 a 2 ...a k ) = 4(a1 a 2 ...a k ) ⋅ (10 + 1)
Logo, se os algarismos de 44 n forem :
4a1 , (4 a2 + 4 a1 ) , (4a 3 + 4a 2 ) , ... , (4a k + 4a k −1 ) e 4a k
teríamos S (44n ) = 8 ⋅ (a1 + a 2 + ⋅ ⋅ ⋅ + a k ) = 800 . Assim, se houvesse algum “vai-
um”, a soma dos algarismos de 44n cairia . Deste modo, todos os algarismos de
n são menores ou iguais a 2 donde os algarismos de 3n são 3a1 ,3a 2 ,3a3 ,...,3a k .
Assim,
S (3n) = 3a1 + 3a 2 + 3a3 + ... + 3a k = 3.S (n) = 300 .

EUREKA! N°12, 2001

47
Sociedade Brasileira de Matemática

Acusamos o recebimento de soluções de problemas anteriores dos seguintes


leitores de EUREKA!:

Anderson Torres São Paulo – SP Prob.16, 52, 67, 69, 70, 76, 102, 105, 118, 120.
Geraldo Perlino Itapec. da Serra – SP Prob. 115.
Ioziel Matos Corrêa Júnior Rio de Janeiro – RJ Prob. 52, 62, 71, 74, 76, 78.
Jorge Silva Júnior Cachoeiro do Itapemirim – ES Prob. 96, 98, 99, 101, 103, 110.
Marcelo R. De Souza Rio de Janeiro – RJ Prob. 95, 115.
Marcelo Rufino de Oliveira Belém – PA Prob. 91, 92, 93, 95, 97, 98, 99, 101, 102, 104,
107, 109, 110, 111, 112, 114, 115, 118.
Mauro Félix de Souza Cordovil – RJ Prob. 98.
Osvaldo Mello Sponquiado Olímpia – SP Prob. 29, 30, 38, 56, 57, 59, 63, 66, 67, 69, 74, 98,
107, 114, 118.
Paulo Alexandre Araújo Sousa Teresina – PI Prob. 62, 78, 82, 88, 76, 70.
Renato Francisco Lopes Mello Jaboatão dos Guararapes – PE Prob. 98, 99, 101, 109, 117.
Wallace Rodrigues de Holanda Miranda Teresina – PI Prob. 97, 98, 99, 101, 102, 104, 106, 109, 112,
114, 116, 117, 120.
Wilson Carlos de Silva Ramos Belém – PA Prob. 43, 68, 74, 82.

9RFrVDELD«
4XH IRL GHVFREHUWR XP QRYR SULPR GH 0HUVHQQH HP
 " e R Q~PHUR  
 TXH WHP  GtJLWRV H

p DR ODGR GH   XP GRV GRLV SULPRV FRQKHFLGRV FRP
PDLV GH XP PLOKmR GH GtJLWRV 2 GHVFREULGRU 0LFKDHO &DPHURQ
GH  DQRV p XP SDUWLFLSDQWH GR *,036 XP SURMHWR
FRRSHUDWLYR SDUD SURFXUDU SULPRV GH 0HUVHQQH 
&RQVXOWH QD LQWHUQHW D SiJLQD

KWWSZZZPHUVHQQHRUJSULPHKWP

EUREKA! N°12, 2001

48
Sociedade Brasileira de Matemática

SOLUÇÕES DE PROBLEMAS PROPOSTOS


 Publicamos aqui algumas das respostas enviadas por nossos leitores.
56) Para cada número n, seja f(n) a quantidade de maneiras que se pode expressar
n como a soma de números iguais a 1, 3 ou 4.
Por exemplo, f(4) = 4, pois todas as maneiras possíveis são 4 = 1 + 1 + 1 + 1,
4 = 1 + 3, 4 = 3 + 1, 4 = 4. Demonstrar que se n é par, f(n) é um quadrado
perfeito.

SOLUÇÃO DE MÁRCIO ASSAD COHEN (RIO DE JANEIRO – RJ):


O coeficiente de xn em (x + x3 + x4)k conta exatamente uma vez cada modo de
representar o número n na forma α + 3β + 4γ com α + β + γ = k , ou seja, conta
os modos de representar n utilizando 1’s, 3’s e 4’s com um total de k números.
Variando k, segue que f(n) é o coeficiente de xn na série formal


g ( x) = ∑ (x + x
k =0
3
+ x4 )k .

Somando a PG, fatorando, e decompondo em frações parciais tem-se:


1
g ( x) =
1 − x − x3 − x4

1 1 x+2 1 x+3
g ( x) = = ⋅ + ⋅
(1 + x )(1 − x − x ) 5 1 + x
2 2 2
5 1− x − x2

Utilizando respectivamente soma de PG e o método desenvolvido para séries


formais no artigo da Revista Eureka! No. 11 tem-se:
1
= 1 − x 2 + x 4 − x 6 + ...
1+ x2
1
= F0 + F1 x + F2 x 2 + F3 x 3 + ...
1− x − x 2

onde Fn é o n-ésimo número de Fibonacci, com F0 = F1 = 1, Fn+2 = Fn+1 + Fn.


Se n é par, n = 2t, o coeficiente de xn no desenvolvimento de g(x) será então:

[x ]g ( x ) = 2 ⋅ ( − 1) + F2 t −1 + 3 F2 t
t
n
5

EUREKA! N°12, 2001

49
Sociedade Brasileira de Matemática

1+ 5 1− 5
Utilizando que Fn =
1
(α n +1
)
− β n +1 , onde α =
2
,β =
2
:
5
2 ⋅ (−1) + F2t −1 + 3F2t =
t

2 ⋅ (−1) t + [( F2t −1 + F2t ) + F2t ] + F2t =


2 ⋅ (−1) t + F2t + 2 + F2t =

2 ⋅ (−1) t +
1
(α 2t + 3
− β 2t +3 + ) 1
(α 2 t +1
)
− β 2t +1 =
5 5

2 ⋅ (−1) t +
1
[(α + α )α −1 2t + 2
(
− β + β −1 β 2t + 2 ) ]
5

Notando que α ⋅ β = −1 :
1+ 5 1− 5
α + α −1 = − = 5
2 2
β + β −1 = −α −1 − α = − 5

Juntando tudo:
[x ]g ( x ) = 15 [2 ⋅ ( − 1)
n t
+ α 2t + 2 + β 2t + 2 = ]
2
α 2 t + 2 + β 2 t + 2 − 2 ⋅ (α ⋅ β ) t +1  α t +1 − β t +1 
=   = Ft 2

5  5 

E, em particular, f(n) é quadrado perfeito sempre que n é par.

2 p −1 − 1
58) Determine todos os primos p para os quais o número é o quadrado
p
de um inteiro.

SOLUÇÃO DE RODRIGO VILLARD MILET (RIO DE JANEIRO – RJ):

Resposta: p = 3 ou p = 7.

EUREKA! N°12, 2001

50
Sociedade Brasileira de Matemática

2 p −1 − 1
Primeiramente, note que é inteiro, para p > 2 (pelo pequeno Teorema de
p
Fermat), e p = 2 não serve. Daí, como p é ímpar, 2 p −1 − 1 ≡ 0(mod 3). Se p = 3,
2 3−1 − 1 2p−1 −1
= 1 que é quadrado. Se p = 3, como é quadrado,
3 p
9 2 p −1 − 1 ⇔ 2 p −1 ≡ 1(mod 9) ⇔ ord 9 2 = 6 p − 1 ⇒ p ≡ 1(mod 6).
2 p −1 − 1 2 6 k − 1 ( 2 3 k − 1)( 2 3 k + 1)
∴ ∃ k ∈ ; p = 1 + 6k ⇒ = =
p p p
mdc{2 3 k − 1, 2 3 k + 1} = mdc{2, 2 3 k + 1} = 1 como p é primo ( p > 3), p 2 3 k − 1

ou p 2 3 k + 1.

(i) p 2 3 k − 1.
Daí, como 2 3 k − 1 e 2 3 k + 1 não tem fatores em comum, 2 3 k + 1 é quadrado
∴ ∃ q ∈ N ;2 3 k + 1 = q 2 ⇔ 2 3 k = ( q − 1)( q + 1) ⇒ q = 3 e k = 1 (de fato, q-1
e q+1 devem ser potências de 2,o que só é possível se forem 2 e 4). Nesse caso,
2 7 −1 − 1
= 9 que é quadrado.
7

(ii) p 2 3 k + 1. (Assuma k > 1, pois k = 1 já é solução)


( )(
Daí, 2 3 k − 1 é quadrado ∴ ∃ m ∈ N ;2 3 k − 1 = m 2 ⇔ 2 k − 1 2 2 k + 2 k + 1 = m 2 . )
Seja
d = mdc{2 k − 1, 2 2 k + 2 k + 1} . d 2 k − 1 ⇒ d ( 2 k − 1) = ( 2 − 2 ⋅ 2 k + 1),
2 2k

mas d 2 2 k + 2 k + 1, logo d 3.2 k . Como d 2 k − 1, d não divide


2 k ⇒ d | 3 ⇒ d = 1 ou 3 .
• d = 1 ⇒ 2 2k + 2 k + 1 é quadrado. No entanto, temos
(2 ) < 2
k 2 2k k
(
+ 2 +1< 2 +1 , k
)2
então não é possível 2 2k
+ 2 +1
k
ser
quadrado.

EUREKA! N°12, 2001

51
Sociedade Brasileira de Matemática

2k −1
•d =3⇒ é quadrado.
3
2k − 1
Como é ímpar,
3
2k −1
≡ 1(mod 8) ⇒ 2 k ≡ 4 mod 8 ⇒ k = 2 ⇒ p = 13 , que não é solução.
3

59) Um pedestal de altura a sustenta uma coluna de altura b (b > a). A que
distância do monumento se deve colocar um observador para ver o pedestal
e a coluna sob ângulos iguais?

SOLUÇÃO DE CARLOS ALBERTO DA SILVA VÍCTOR (NILÓPOLIS – RJ):


Supondo a altura do observador desprezível, teremos:

b (b > a)

a
θ
θ

x
a a+b
tgθ = ; tg 2θ =
x x
2tgθ a+b 2.a/x a + b
= ∴ = ⇒
1 − tg θ
2
x a2 x
1− 2
x
2
a+b
⇒ 2ax 2 = (a + b )x 2 − a 2 (a + b) ⇒ (b − a )x 2 = a 2 (a + b ),
2a.x
⇒ =
x(a − a )
2 2
x
logo
a+b
x = a. .
b−a

EUREKA! N°12, 2001

52
Sociedade Brasileira de Matemática

60) Se num triângulo ABC , A = 2B, provar que a 2 = b(b + c).


Obs.: a, b e c são, respectivamente, os lados opostos aos ângulos A, B e C.

SOLUÇÃO DE MARCELO RIBEIRO DE SOUZA (RIO DE JANEIRO – RJ):


A

2x
c b

x
B a C
Pela lei dos senos, temos:

senx sen2 x
I) =
b a
senx 2 senx ⋅ cos x
= , obviamente x ≠ kπ , k ∈ =
b a
a = 2b ⋅ cos x (1)

II) Usando lei dos cossenos: b 2 = a 2 + c 2 − 2ac ⋅ cos x


2ac ⋅ cos x = a 2 + c 2 − b 2
a2 + c2 − b2
cos x = (2)
2ac
 a2 + c2 − b2 
III) Substituindo (2) em (1): a = 2b ⋅  

 2 ac 
⇔ a 2 c = a 2 b + bc 2 − b 3
⇔ a 2 (c − b ) = b ( c 2 − b 2 )
⇔ a 2 (c − b) = b(c − b)(c + b).
Se b ≠ c, temos a 2 = b(b + c).
Se b=c, temos b=c=x, donde 4 x = π ⇒ x = π / 4 e
a = b 2 ⇒ a 2 = 2b 2 = b(b + c).

EUREKA! N°12, 2001

53
Sociedade Brasileira de Matemática

61) Na figura abaixo um quadrado EFGH foi colocado no interior do quadrado


ABCD, determinando 4 quadriláteros. Se a, b, c, e d denotam os medidas
das áreas dos quadriláteros, mostre que a + b = c + d .

c
d

SOLUÇÃO DE MÁRCIO ASSAD COHEN (RIO DE JANEIRO – RJ):

Lema: A área de um triângulo cujos vértices no plano complexo são z1, z2, z3

(sentido horário) é dada por S =


1
2
{ }
Im ( z 2 − z1 )( z 3 − z1 ) .

Prova: Se z 2 − z1 = a ⋅ cisθ 1 ; z 3 − z1 = b ⋅ cisθ 2 :


1
2
{ }
Im ( z 2 − z 1 ) ( z 3 − z 1 ) = Im {a ⋅ cis θ 1 b ⋅ cis ( −θ 2 )}=
1
2
absen θ
Im {cis (θ 1 − θ 2 )}=
ab
=S
2 2
Problema:
i 1

w+iv

w+v

w–v
w–iv

-1 -i

EUREKA! N°12, 2001

54
Sociedade Brasileira de Matemática

Coloque os eixos de modo que os vértices do quadrado maior sejam 1, i, –1, – i


(basta adotar uma unidade de medida tal que 2u.m = diagonal).
Seja w o centro do quadrado menor, e w + v um de seus vértices. Então, os outros
vértices do quadrado menor serão w + iv, w – i, w – iv (pois rodar 90o é
multiplicar por i).
Dividindo cada quadrilátero em triângulos como mostra a figura temos:
1
{
a = Im ( w + iv − 1)(i − 1) + (1 − w − iv)v(1 − i )
2
}
1
{
c = Im ( w − iv − 1)(−i + 1) + (−1 − w + iv)v(i − 1)
2
}
1
{ [ ]}
a + c = Im (1 + i) (− w − iv + 1) + v(1 − w − iv) + ( w − iv + 1) + v(1 + w − iv)
2

Note que w é cancelado e a + c independe do centro do quadrado menor:

{
a + c = Im (1 + i)(1 − iv)(1 + v) }
Da mesma forma, obtemos:
1
{ }
b + d = Im (w + v + i)(1 + i) + (−i − w − v)v(−i − 1) + (w − v − i)(−1 − i) + (1 − w + v)v(1 + i) =
2
1
2
{ [
Im (1 − i) (w + v + i) + (i + w + v)v + (−w + v + i) + (−w + v + i)v = ]}
{
Im (1 − i)(i + v)(v + 1) }
Fatorando, obtemos

{ } { }
b + d = Im − i(i + 1)(i + v)(v + 1) = Im (i + 1)(1 − iv)(v + 1) = a + c .

62) Se ABCD é um quadrilátero convexo tal que os lados AB, BC , CD e DA


medem respectivamente a, b, c e d e que α, β , σ e γ são as medidas dos seus
ângulos internos, mostre que a medida da área desse quadrilátero, denotada
por (ABCD), é dada por:

EUREKA! N°12, 2001

55
Sociedade Brasileira de Matemática

γ c
d

d1
A α σ C

a
b

(ABCD) = ( p − a )( p − b)( p − c)( p − d ) − abcd cos 2 δ onde:


a+b+c+d
p=
2
α +σ β +γ
δ= (a fórmula também vale se fizermos δ = ).
2 2

SOLUÇÃO DE WALLACE RODRIGUES DE HOLANDA MIRANDA (TERESINA – PI):


Antes da resolução do problema precisa-se ressaltar algumas noções
fundamentais:

1) cos(α + σ ) = cos α ⋅ cos σ − senα ⋅ senσ


cos(2δ ) = cos α ⋅ cos σ − senα ⋅ senσ
cos 2 δ − sen 2δ = cos α ⋅ cos σ − senα ⋅ senσ
( )
cos 2 δ − 1 − cos 2 δ = cos α ⋅ cos σ − senα ⋅ senσ
senα ⋅ senσ = cos α ⋅ cos σ − 2 cos 2 δ + 1

2) (ABD ) = 1 ⋅ a ⋅ d ⋅ senα (BCD ) = 1 ⋅ b ⋅ c ⋅ senσ


2 2
3) Pela lei dos cossenos nos triângulos ABD e BCD:
a 2 + d 2 − 2ad ⋅ cos α = d 12 = b 2 + c 2 − 2bc ⋅ cos σ
a 2 + d 2 − b 2 − c 2 = 2ad ⋅ cos α − 2bc ⋅ cos σ

Agora vamos à resolução do problema:


(ABCD) = (ABD) + (BCD)

EUREKA! N°12, 2001

56
Sociedade Brasileira de Matemática

(ABCD ) = 1 ⋅ a ⋅ d ⋅ senα + 1 ⋅ b ⋅ c ⋅ senσ


2 2
2(ABCD ) = a ⋅ d ⋅ senα + b ⋅ c ⋅ senσ
(2(ABCD ))2 = (a ⋅ d ⋅ senα + b ⋅ c ⋅ senσ )2
4(ABCD ) = a 2 ⋅ d 2 ⋅ sen 2α + 2abcd ⋅ senα ⋅ senσ + b 2 ⋅ c 2 ⋅ sen 2σ
2

Substituindo (sen 2 ) por (1 − cos 2 ) e (senα ⋅ senσ ) por


(cos α ⋅ cos σ − 2 cos 2
)
δ + 1 de 1) fica:
( )
4(ABCD) = a ⋅ d ⋅ 1 − cos 2 α + 2abcd ⋅ 1 + cos α ⋅ cos σ − 2 cos 2 δ +
2 2 2
( )
(
+ b 2 ⋅ c 2 ⋅ 1 − cos 2 σ )
(
4(ABCD ) = a ⋅ d 2 ⋅ 1 − cos 2 α + abcd ⋅ (2 + 2 cos α ⋅ cos σ ) +
2 2
)
2 2
(
+ b ⋅ c 1 − cos σ − 4abcd ⋅ cos 2 δ 2
)
2 2 2
( )
4(ABCD ) = a ⋅ d ⋅ 1 − cos 2 α + abcd (1 − cos α − cos σ + cos α ⋅ cos σ + 1 +
+ cos α + cos σ + cos α ⋅ cos σ ) + b 2 c 2 1 − cos 2 σ − 4 abcd ⋅ cos 2 δ ( )
2 2 2
( 2
)
4( ABCD) = a ⋅ d 1 − cos α + abcd [(1 − cos α )(1 − cos σ ) + (1 + cos α )(1 + cos σ )] +
( )
b 2 c 2 1 − cos 2 σ − 4abcd ⋅ cos 2 δ
4( ABCD) 2 = ad (1 + cos α )ad (1 − cos α ) + ad (1 − cos α )bc(1 − cos σ ) +
+ ad (1 + cos α )bc(1 + cos σ ) + bc(1 + cos σ )bc(1 − cos σ ) − 4abcd ⋅ cos 2 δ
4( ABCD) 2 = ad (1 + cosα )[ad (1 − cosα ) + bc(1 + cosσ )] + bc(1 − cosσ )[ad (1 − cosα ) +
+ bc(1 + cosσ )] − 4abcd ⋅ cos2 δ
4( ABCD) 2 = [ad (1 − cosα ) + bc(1 + cosσ )][ad (1 + cosα ) + bc(1 − cosσ )] − 4abcd ⋅ cos2 δ
16( ABCD) 2 = 2(ad − ad ⋅ cosα + bc + bc ⋅ cos σ ) ⋅ 2 ⋅ (ad + ad ⋅ cosα + bc − bc ⋅ cos σ )
− 16abcd ⋅ cos 2 δ
16( ABCD) 2 = −(2ad ⋅ cos α − 2bc ⋅ cos σ − 2ad − 2bc)(2ad ⋅ cos α − 2bc ⋅ cos σ +
+ 2ad + 2bc) − 16abcd ⋅ cos 2 δ
Substituindo (2ad ⋅ cos α − 2bc ⋅ cos σ ) por (a 2 + d 2 − b 2 − c 2 ) de 3) fica:
16( ABCD) 2 = −(a 2 − 2ad + d 2 − b 2 − 2bc − c 2 )(a 2 + 2ad + d 2 − b 2 + 2bc − c 2 )
− 16abcd ⋅ cos 2 δ
[ ][
16 ( ABCD ) 2 = − ( a − d ) 2 − (b + c ) 2 ( a + d ) − (b − c ) 2 − 16 abcd ⋅ cos 2 δ ]

EUREKA! N°12, 2001

57
Sociedade Brasileira de Matemática

16( ABCD) 2 = −(a − d + b + c)(a − d − b − c)(a + d − b + c)(a + d + b − c) − 16abcd ⋅ cos2 δ


16( ABCD) 2 = (a + b + c − d )(−a + b + c + d )(a − b + c + d )(a + b − c + d ) −16abcd⋅ cos2 δ
16( ABCD) 2 = (−a + b + c + d )(a − b + c + d )(a + b − c + d )(a + b + c − d ) − 16abcd ⋅ cos2 δ
(−a + b + c + d) (a − b + c + d) (a + b − c + d ) (a + b + c − d)
( ABCD) 2 = ⋅ ⋅ ⋅ − abcd⋅ cos2 δ
2 2 2 2
( ABCD) 2 = ( p − a )( p − b)( p − c)( p − d ) − abcd ⋅ cos 2 δ
( ABCD) = ( p − a )( p − b)( p − c)( p − d ) − abcd ⋅ cos 2 δ .

Agradecemos também o envio das soluções e a colaboração de:

Bruno de Souza Ramos Realengo – RJ


Everaldo de Melo Bonotto Guararapes – SP
Fernando Carvalho Ramos Santa María – RS
Geraldo Perlino Itapecerica da Serra – SP
Jorge Silva Junior Cachoeiro de Itapemerim – ES
Marcelo Ribeiro Rio de Janeiro – RJ
Marcelo Rufino de Oliveira Belém – PA
Marcílio Miranda de Carvalho Teresina – PI
Mateus Queiroz Guilherme de Oliveira Fortaleza – CE
Oswaldo Melo Sponquiado Olímpia – SP

Seguimos aguardando o envio de soluções do problema proposto No. 57 publicado na


revista Eureka! No. 11.

EUREKA! N°12, 2001

58
Sociedade Brasileira de Matemática

PROBLEMAS PROPOSTOS
Convidamos o leitor a enviar soluções dos problemas propostos e sugestões de novos
problemas para os próximos números.

63) Prove que existem infinitos números naturais múltiplos de 51000 sem
nenhum 0 na representação decimal.

64) Iniciando de um certo inteiro positivo, é permitido fazer apenas uma


operação: o dígito das unidades é separado e multiplicado por 4, e então
este valor é somado ao restante do número. Por exemplo, o número 1997 é
transformado para 7.4 + 199 = 227. A operação é feita repetidamente. Prove
que se a seqüência de números obtida contém 1001, então nenhum dos
números na seqüência pode ser um número primo.

65) Determine todos os inteiros N tais que, em base 10, os dígitos de 9N são os
mesmos dígitos de N na ordem inversa, e N possui no máximo um dígito
igual a 0.

66) Prove que, dados um inteiro n ≥ 1 e um conjunto A ⊂ = / n = com n


2

elementos existe B ⊂ = / n 2 = com n elementos tal que


{ }
A + B = x + y x ∈ A, y ∈ B ⊂ = / n 2 = tem mais de n 2 / 2 elementos.

67) Seja ABCD um quadrilátero tal que os círculos circunscritos aos triângulos
ABC e BCD são ortogonais.Prove que os círculos circunscritos aos
triângulos BCD e DAB também são ortogonais.

Problema 63 proposto por Wallace Rodrigues de Holanda Miranda (Teresina – PI);


Problemas 64 e 65 propostos por Marcelo Rufino de Oliveira (Belém – PA); Problema 67
proposto por Luciano Castro (Rio de Janeiro – RJ).

EUREKA! N°12, 2001

59
Sociedade Brasileira de Matemática

AGENDA OLÍMPICA

XXIV OLIMPÍADA BRASILEIRA DE MATEMÁTICA

NÍVEIS 1, 2 e 3
Primeira Fase – Sábado, 8 de junho de 2002
Segunda Fase – Sábado, 14 de setembro de 2002
Terceira Fase – Sábado, 19 de outubro de 2002 (níveis 1, 2 e 3)
Domingo, 20 de outubro de 2002 (níveis 2 e 3 - segundo dia de prova).

NÍVEL UNIVERSITÁRIO
Primeira Fase – Sábado, 14 de setembro de 2002
Segunda Fase – Sábado, 19 e Domingo, 20 de outubro de 2002

VIII OLIMPÍADA DE MAIO
maio de 2002

XIII OLIMPÍADA DE MATEMÁTICA DO CONE SUL
junho de 2002
Fortaleza – CE, Brasil

XLIII OLIMPÍADA INTERNACIONAL DE MATEMÁTICA
julho de 2002
Glasgow, Reino Unido

XVII OLIMPÍADA IBEROAMERICANA DE MATEMÁTICA
setembro de 2002
El Salvador

V OLIMPÍADA IBEROAMERICANA DE MATEMÁTICA UNIVERSITÁRIA
outubro de 2002

♦♦♦

EUREKA! N°12, 2001

60
Sociedade Brasileira de Matemática

COORDENADORES REGIONAIS
Amarísio da Silva Araújo (UFV) Viçosa – MG
Alberto Hassen Raad (UFJF) Juiz de Fora – MG
Angela Camargo (Centro de Educação de Adultos – CEA) Blumenau – SC
Benedito Tadeu Vasconcelos Freire (UFRN) Natal – RN
Carlos Frederico Borges Palmeira (PUC-Rio) Rio de Janeiro – RJ
Claudio Arconcher (Colégio Leonardo da Vinci) Jundiaí – SP
Claus Haetinger (UNIVATES) Lajeado – RS
Cleonor Crescêncio das Neves (UTAM) Manaus – AM
Élio Mega (Colégio Etapa) São Paulo – SP
Rosângela Souza (Colégio Singular) Santo André – SP
Florêncio Ferreira Guimarães Filho (UFES) Vitória – ES
Gisele de Araújo Prateado Gusmão (UFGO) Goiânia – GO
Ivanilde Fernandes Saad (UC. Dom Bosco) Campo Grande– MS
Jacqueline Fabiola Rojas Arancibia (UFPB) João Pessoa – PB
João Benício de Melo Neto (UFPI) Teresina – PI
João Francisco Melo Libonati (Grupo Educacional Ideal) Belém – PA
Irene Nakaoka (UEM) Maringá – PR
José Carlos Pinto Leivas (UFRG) Rio Grande – RS
José Cloves Saraiva (UFMA) São Luis – MA
José Gaspar Ruas Filho (ICMC-USP) São Carlos – SP
José Luiz Rosas Pinho (UFSC) Florianópolis – SC
José Vieira Alves (UFPB) Campina Grande – PB
Marcelo Rufino de Oliveira (Sistema Titular de Ensino) Belém – PA
Licio Hernandes Bezerra (UFSC) Florianópolis – SC
Luzinalva Miranda de Amorim (UFBA) Salvador – BA
Marcondes Cavalcante França (UFC) Fortaleza – CE
Pablo Rodrigo Ganassim (Liceu Terras do Engenho) Piracicaba – SP
Paulo Henrique Cruz Neiva de Lima Jr. (Escola Técnica Everardo Passos) SJ dos Campos – SP
Reinaldo Gen Ichiro Arakaki (INPE) SJ dos Campos – SP
Ricardo Amorim (Centro Educacional Logos) Nova Iguaçu – RJ
Roberto Vizeu Barros (Colégio Acae) Volta Redonda – RJ
Sérgio Cláudio Ramos (IM-UFRGS) Porto Alegre – RS
Silvio de Barros Melo (UFPE) Recife – PE
Tadeu Ferreira Gomes (UEBA) Juazeiro – BA
Tomás Menéndez Rodrigues (U. Federal de Rondônia) Porto Velho – RO
Valdenberg Araújo da Silva (U. Federal de Sergipe) São Cristovão – SE
Wagner Pereira Lopes (Escola Técnica Federal de Goiás) Jataí – GO

EUREKA! N°12, 2001

61
CONTEÚDO

XXIII OLIMPÍADA BRASILEIRA DE MATEMÁTICA 2


Problemas e Soluções da Primeira Fase

XXIII OLIMPÍADA BRASILEIRA DE MATEMÁTICA 11


Problemas e Soluções da Segunda Fase

XXIII OLIMPÍADA BRASILEIRA DE MATEMÁTICA 21


Problemas e Soluções da Terceira Fase

XXIII OLIMPÍADA BRASILEIRA DE MATEMÁTICA 41


Problemas e Soluções da Primeira Fase - Nível Universitário

XXIII OLIMPÍADA BRASILEIRA DE MATEMÁTICA 46


Problemas e Soluções da Segunda Fase - Nível Universitário

XXIII OLIMPÍADA BRASILEIRA DE MATEMÁTICA 57


Premiados

AGENDA OLÍMPICA 61

COORDENADORES REGIONAIS 62
Sociedade Brasileira de Matemática

XXIII OLIMPÍADA BRASILEIRA DE MATEMÁTICA


Problemas e Soluções da Primeira Fase

PROBLEMAS – NÍVEL 1

1. Considere dois números naturais, cada um deles com três algarismos diferentes.
O maior deles só tem algarismos pares e o menor só tem algarismos ímpares. O
menor valor possível para a diferença entre eles é:
A) 111 B) 49 C) 29 D) 69 E) 5

2. Na figura abaixo, temos 4 circunferências e alguns pontos destacados no interior


dessas circunferências. Escolhendo exatamente um desses pontos dentro de cada
uma das circunferências, e unindo-os por segmentos de reta que não se cruzam,
formamos um quadrilátero. Quantos quadriláteros diferentes seremos capazes de
desenhar nessas condições?

A) 4 B) 14 C) 60 D) 120 E) 24

3. Joana escreve a seqüência de números naturais 1, 6, 11,..., onde cada número,


com exceção do primeiro, é igual ao anterior mais cinco. Joana pára quando
encontra o primeiro número de três algarismos. Esse número é:
A) 100 B) 104 C) 101 D) 103 E) 102

4. Quantos números de dois algarismos não são primos nem múltiplos de 2, 3 ou 5?


A) 1 B) 3 C) 2 D) 4 E) mais de 4

5. No conjunto {101, 1 001, 10 001,..., 1 000 000 000 001} cada elemento é um
número formado pelo algarismo 1 nas extremidades e por algarismos 0 entre
eles. Alguns desses elementos são números primos e outros são compostos.
Sobre a quantidade de números compostos podemos afirmar que:
A) é igual 11
B) é igual a 4
C) é menor do que 3
D) é maior do que 4 e menor do que 11
E) é 3

EUREKA! N°13, 2002

2
Sociedade Brasileira de Matemática

6. Uma pêra tem cerca de 90% de água e 10% de matéria sólida. Um produtor
coloca 100 quilogramas de pêra para desidratar até o ponto em que a água
represente 60% da massa total. Quantos litros de água serão evaporados?
(lembre-se: 1 litro de água tem massa de 1 quilograma).
A) 15 litros B) 45 litros C) 75 litros D) 80 litros
E) 30 litros

7. O triângulo equilátero T à direita tem lado 1. Juntando


triângulos congruentes a esse, podemos formar outros
triângulos equiláteros maiores, conforme indicado no desenho
abaixo.

Qual é o lado do triângulo equilátero formado por 49 dos triângulos T?


A) 7 B) 49 C) 13 D) 21
E) é impossível formar um triângulo equilátero com esse número de triângulos T

8. Os números inteiros positivos de 1 a 1000 são escritos lado a lado, em ordem


crescente, formando a seqüência 123456789101112131415... 9991000. Nesta
seqüência, quantas vezes aparece o grupo “89” ?
A) 98 B) 32 C) 22 D) 89 E) 21

9. Um serralheiro tem 10 pedaços de 3 elos de ferro cada um, mostrados abaixo.

Ele quer fazer uma única corrente de 30 elos. Para abrir e depois soldar um elo o
serralheiro leva 5 minutos. Quantos minutos no mínimo ele levará para fazer a
corrente?
A) 30 B) 35 C) 40 D) 45 E) 50

10. Escrevem-se os números naturais numa faixa decorativa, da seguinte maneira:


1 3 5 7

2 4 6 8
Assinale a figura correta:

EUREKA! N°13, 2002

3
Sociedade Brasileira de Matemática

2 00 1 2 00 1 2 00 0
a)A) B)
b) C)c)

2 00 0 2 00 0 2 00 1

d) 2 00 1 e) 2 00 1
D) E)

2 00 0 2 00 0

11. 2 melancias custam o mesmo que 9 laranjas mais 6 bananas; além disso, meia
dúzia de bananas custa a metade de uma melancia. Portanto, o preço pago por
uma dúzia de laranjas e uma dúzia de bananas é igual ao preço de:
A) 3 melancias B) 4 melancias C) 6 melancias D) 5 melancias
E) 2 melancias

12. Qual é o último algarismo da soma de 70 números inteiros positivos


consecutivos?
A) 4 B) 0 C) 7 D) 5 E) Faltam dados

13. Em Tumbólia, um quilograma de moedas de 50 centavos equivale em dinheiro a


dois quilogramas de moedas de 20 centavos. Sendo 8 gramas o peso de uma
moeda de 20 centavos, uma moeda de 50 centavos pesará:
A) 15 gramas B) 10 gramas C) 12 gramas D) 20 gramas
E) 22 gramas

14. As medidas dos lados de um retângulo são números inteiros distintos. O


perímetro e a área do retângulo se exprimem pelo mesmo número. Determine
esse número.
A) 18 B) 12 C) 24 D) 9 E) 36

15. O número N de três algarismos multiplicado por 7 deu como resultado um


número que termina em 171.
A soma dos algarismos de N é:
A) 10 B) 11 C) 12 D) 13 E) 14

16. Em um tabuleiro retangular com 6 linhas e 9 colunas, 32 casas estão ocupadas.


Podemos afirmar que:

A) Todas as colunas têm pelo menos 3 casas ocupadas.


B) Nenhuma coluna tem mais de 3 casas ocupadas.
C) Alguma coluna não tem casas ocupadas.

EUREKA! N°13, 2002

4
Sociedade Brasileira de Matemática

D) Alguma linha tem pelo menos 6 casas ocupadas.


E) Todas as linhas têm pelo menos 4 casas ocupadas.

17. Contando-se os alunos de uma classe de 4 em 4 sobram 2, e contando-se de 5 em


5 sobra 1. Sabendo-se que 15 alunos são meninas e que nesta classe o número de
meninas é maior que o número de meninos, o número de meninos nesta classe é:
A) 7 B) 8 C) 9 D) 10 E) 11

18. São escritos todos os números de 1 a 999 nos quais o algarismo 1 aparece
exatamente 2 vezes (tais como, 11, 121, 411, etc). A soma de todos estes
números é:
A) 6882 B) 5994 C) 4668 D) 7224 E) 3448

19. Cinco animais A, B, C, D, e E, são cães ou são lobos. Cães sempre contam a
verdade e lobos sempre mentem. A diz que B é um cão. B diz que C é um lobo. C
diz que D é um lobo. D diz que B e E são animais de espécies diferentes. E diz
que A é um cão. Quantos lobos há entre os cinco animais?
A) 1 B) 2 C) 3 D) 4 E) 5

20. Com azulejos quadrados brancos e pretos todos do mesmo tamanho, construímos
os seguintes mosaicos.

A regra para se construir estes mosaicos é a seguinte: inicialmente formamos um


quadrado com 1 azulejo branco cercado por azulejos pretos; e em seguida, outro
quadrado, este com 4 azulejos brancos, também cercado por azulejos pretos; e assim
sucessivamente.
Com 80 azulejos pretos, quantos azulejos brancos serão necessários para se fazer
uma seqüência de mosaicos como esta?
A) 55 B) 65 C) 75 D) 85 E) 100

PROBLEMAS – NÍVEL 2
1. Veja o problema 4 do Nível 1.

EUREKA! N°13, 2002

5
Sociedade Brasileira de Matemática

2. O triângulo CDE pode ser obtido pela rotação do triângulo ABC de 90o no
sentido anti-horário ao redor de C, conforme mostrado no desenho abaixo.
Podemos afirmar que α é igual a:
B

α
A C

D 60
O

O
40

E
A) 75o B) 65o C) 70o D) 45o E) 55o

3. Veja o problema 5 do Nível 1. 4. Veja o problema 6 do Nível 1.


5. Veja o problema 8 do Nível 1. 6. Veja o problema 9 do Nível 1.
7. Veja o problema 11 do Nível 1. 8. Veja o problema 12 do Nível 1.
9. Veja o problema 14 do Nível 1. 10. Veja o problema 15 do Nível 1.

11. Os pontos P1, P2, P3, … estão nesta ordem sobre uma circunferência e são tais
que o arco que une cada ponto ao seguinte mede 35°. O menor valor de n > 1 tal
que Pn coincide com P1 é:
A) 37 B) 73 C) 109 D) 141 E) 361

12. Veja o problema 16 do Nível 1.

13. ABCDE é um pentágono regular e ABF é um triângulo equilátero interior. O


ângulo FCD mede:
A) 38° B) 40° C) 42° D) 44° E) 46°

14. Veja o problema 19 do Nível 1.

15. Um círculo é dividido, por 2n + 1 raios, em 2n + 1 setores congruentes. Qual é o


número máximo de regiões do círculo determinadas por estes raios e por uma
reta?
A) 3n B) 3n + 1 C) 3n + 2 D) 3n + 3 E) 4n

16. Paulo e Cezar têm algum dinheiro. Paulo dá a Cezar R$5,00 e, em seguida,
1
Cezar dá a Paulo do que possui. Assim, ambos ficam com R$18,00. A
3
diferença entre as quantias que cada um tinha inicialmente é:
A) R$7,00 B) R$8,00 C) R$9,00 D) R$10,00 E) R$11,00

EUREKA! N°13, 2002

6
Sociedade Brasileira de Matemática

17. Um fazendeiro tinha 24 vacas e ração para alimentá-las por 60 dias. Entretanto,
10 dias depois, ele comprou mais 6 vacas e 10 dias depois dessa compra ele
vendeu 20 vacas. Por mais quantos dias após esta última compra ele pode
alimentar o gado com a ração restante?
A) 50 B) 60 C) 70 D) 80 E) 90

18. Veja o problema 18 do Nível 1.

19. Uma mesa retangular, cujos pés têm rodas, deve ser empurrada por um corredor
de largura constante, que forma um ângulo reto.
b

Se as dimensões da mesa são a e b (com 2a < b), qual deve ser a largura mínima
do corredor para que a mesa possa ser empurrada através dele?
2 2 2 2
A) a + b B) (a + b) C) (a + b) D) (2a + b) E) (a + 2b)
2 4 4 4

20. Somente uma das figuras a seguir representa a planificação de um cubo na qual
está destacada a sua interseção com um plano. Qual?
A) B) C) D) E)

21. Quantos dígitos tem o menor quadrado perfeito cujos quatro últimos dígitos são
2001?
A) 9 B) 5 C) 6 D) 7 E) 8

22. Papa-Léguas participou de uma corrida (junto com o Ligeirinho e o Flash), que
consistia em dar 100 voltas em um circuito. Como sempre, o Coiote queria pegar
o Papa-Léguas e colocou um monte de alpiste no meio da pista. É claro que o
Coiote não conseguiu pegar o Papa-Léguas, mas ele fez com que a velocidade
média dele na primeira volta fosse de apenas 200 km/h. Sabendo disso, a
velocidade média do Papa-Léguas na corrida:
A) Não ultrapassa 200 km/h.
B) Não ultrapassa 250 km/h, mas pode ultrapassar 200km/h.

EUREKA! N°13, 2002

7
Sociedade Brasileira de Matemática

C) Não ultrapassa 2000 km/h, mas pode ultrapassar 250km/h.


D) Não ultrapassa 20000 km/h, mas pode ultrapassar os 2000km/h.
E) Pode ultrapassar 20000 km/h.

23. Veja o problema 20 do Nível 1.


24. Veja o problema 19 do Nível 1.

25. O hexágono ABCDEF é circunscritível. Se AB = 1, BC = 2, CD = 3, DE = 4 e EF


= 5, quanto mede FA?
B
2
C 1
A

3 ?

D F
4 5
E
A) 1 B) 3 C) 15/8 D) 6 E) 9

PROBLEMAS – NÍVEL 3

1. Veja o problema 4 do Nível 1. 2. Veja o problema 2 do Nível 2.


3. Veja o problema 5 do Nível 1. 4. Veja o problema 6 do Nível 1.
5. Veja o problema 8 do Nível 1. 6. Veja o problema 9 do Nível 1.
7. Veja o problema 15 do Nível 1. 8. Veja o problema 11 do Nível 2.
9. Veja o problema 13 do Nível 2. 10. Veja o problema 15 do Nível 2.
11. Veja o problema 22 do Nível 2.

12. O número de soluções inteiras distintas da equação (−6 x 2 + 12x − 2) x −2 x+2 = 4 é:


2

A) 0 B) 1 C) 2 D) 3 E) 4

13. Uma rifa foi organizada entre os 30 alunos da turma do Pedro. Para tal, 30
bolinhas numeradas de 1 a 30 foram colocadas em uma urna. Uma delas foi,
então, retirada da urna. No entanto, a bola caiu no chão e se perdeu e uma
segunda bola teve que ser sorteada entre as 29 restantes. Qual a probabilidade de
que o número de Pedro tenha sido o sorteado desta segunda vez?
A) 1/29 B) 1/30 C) 1/31 D) 1/60 E) 2/31

14. Cinco animais A, B, C, D, e E, são cães ou são lobos. Cães sempre contam a
verdade e lobos sempre mentem. A diz que B é um cão. B diz que C é um lobo. C

EUREKA! N°13, 2002

8
Sociedade Brasileira de Matemática

diz que D é um lobo. D diz que B e E são animais de espécies diferentes. E diz
que A é um cão. Quantos lobos há entre os cinco animais?
A) 1 B) 2 C) 3 D) 4 E) 5

15. São escritos todos os números de 1 a 999 nos quais o algarismo 1 aparece
exatamente 2 vezes (tais como, 11, 121, 411, etc). A soma de todos estes
números é:
A) 6882 B) 5994 C) 4668 D) 7224 E) 3448

16. Veja o problema 19 do Nível 2.


17. Veja o problema 20 do Nível 2.

18. Seja f(x) = x2 – 3x + 4. Quantas soluções reais tem a equação f(f(f(...f(x)))) = 2


(onde f é aplicada 2001 vezes)?
A) 0 B) 1 C) 2 D) 2001 E) 22001

19. Veja o problema 21 do Nível 2.

20. Seja ABCD um trapézio retângulo cujos únicos ângulos retos são  e B̂ . M e N
são os pontos médios de AB e CD, respectivamente. A respeito dos
ângulos α = ANˆ B e β = CMˆ D , podemos dizer que:
A) α < β
B) α > β
C) α = β
D) pode ocorrer qualquer uma das situações das alternativas A), B) e C).
E) o ângulo α é reto

21. A soma dos valores reais de x tais que x2 + x + 1 = 156/(x2 + x) é:


A) 13 B) 6 C) –1 D) –2 E) –6

22. Para cada ponto pertencente ao interior e aos lados de um triângulo acutângulo
ABC, considere a soma de suas distâncias aos três lados do triângulo. O valor
máximo desta soma é igual
A) à média aritmética das 3 alturas do triângulo.
B) ao maior lado do triângulo.
C) à maior altura do triângulo
D) ao triplo do raio do círculo inscrito no triângulo.
E) ao diâmetro do círculo circunscrito ao triângulo.

EUREKA! N°13, 2002

9
Sociedade Brasileira de Matemática

23. Seja f uma função de Z em Z definida como f(x) = x/10 se x é divisível


por 10 e
f(x) = x + 1 caso contrário. Se a0 = 2001 e an+1 = f(an), qual é o menor valor de n
para o qual an = 1?
A) 20
B) 38
C) 93
D) 2000
E) an nunca é igual a 1

24. Veja o problema 25 do Nível 2.

25. No triângulo ABC, AB = 5 e BC = 6. Qual é a área do triângulo ABC, sabendo


que o ângulo Ĉ tem a maior medida possível?
A) 15 B) 5 7 C) 7 7 / 2 D) 3 11
E) 5 11 / 2
GABARITO
NÍVEL 1 (5a. e 6a. séries)
1) E 6) C 11) A 16) D
2) D 7) A 12) D 17) E
3) C 8) B 13) B 18) A
4) B 9) B 14) A 19) D
5) D 10) D 15) C 20) A

NÍVEL 2 (7a. e 8a. séries)


1) B 6) B 11) B 16) B 21) B
2) E 7) A 12) D 17) E 22) D
3) D 8) D 13) C 18) A 23) A
4) C 9) Anulada 14) E 19) D 24) D
5) B 10) C 15) D 20) B 25) B

NÍVEL 3 (Ensino Médio)


1) B 6) B 11) D 16) D 21) C
2) E 7) C 12) D 17) B 22) C
3) D 8) B 13) B 18) C 23) B
4) C 9) C 14) D 19) B 24) B
5) B 10) D 15) A 20) A 25) E

EUREKA! N°13, 2002

10
Sociedade Brasileira de Matemática

XXIII OLIMPÍADA BRASILEIRA DE MATEMÁTICA


Problemas e Soluções da Segunda Fase

PROBLEMAS – NÍVEL 1

PROBLEMA 1
O jogo de dominó é formado por 28 peças retangulares distintas, cada uma com duas
partes, com cada parte contendo de 0 a 6 pontinhos. Por exemplo, veja três dessas
peças:

Qual é o número total de pontinhos de todas as peças?

PROBLEMA 2
As peças de um jogo chamado Tangram são construídas cortando-se um quadrado
em sete partes, como mostra o desenho: dois triângulos retângulos grandes, um
triângulo retângulo médio, dois triângulos retângulos pequenos, um quadrado e um
paralelogramo. Se a área do quadrado grande é 1, qual é a área do paralelogramo?

PROBLEMA 3
Carlinhos faz um furo numa folha de papel retangular. Dobra a folha ao meio e fura
o papel dobrado; em seguida, dobra e fura novamente o papel dobrado. Ele pode
repetir esse procedimento quantas vezes quiser, evitando furar onde já havia furos.
Ao desdobrar a folha, ele conta o número total de furos feitos. No mínimo, quantas
dobras deverá fazer para obter mais de 100 furos na folha?

PROBLEMA 4
Os pontos da rede quadriculada abaixo são numerados a partir do vértice inferior
esquerdo seguindo o caminho poligonal sugerido no desenho. Considere o ponto
correspondente ao número 2001. Quais são os números dos pontos situados
imediatamente abaixo e imediatamente à esquerda dele?

EUREKA! N°13, 2002

11
Sociedade Brasileira de Matemática

13

5 6 7 12

4 3 8 11

1 2 9 10

PROBLEMA 5
Apresente todos os números inteiros positivos menores do que 1000 que têm
exatamente três divisores positivos. Por exemplo: o número 4 tem exatamente três
divisores positivos: 1, 2 e 4.

PROBLEMA 6
Seja N o número inteiro positivo dado por N = 12 + 22 + 32 + 42 +…+ (196883)2 .
Qual é o algarismo das unidades de N ?

PROBLEMAS – NÍVEL 2

PROBLEMA 1: Veja o problema 2 do Nível 1.


PROBLEMA 2: Veja o problema 4 do Nível 1.

PROBLEMA 3
Se a n-ésima OBM é realizada em um ano que é divisível por n, dizemos que esse
ano é super-olímpico. Por exemplo, o ano 2001, em que está sendo realizada a 23a
OBM, é super-olímpico pois 2001 = 87 ⋅ 23 é divisível por 23. Determine todos os
anos super-olímpicos, sabendo que a OBM nunca deixou de ser realizada desde sua
primeira edição, em 1979, e supondo que continuará sendo realizada todo ano.

PROBLEMA 4
As medidas dos ângulos do triângulo ABC são tais que Aˆ < Bˆ < 90 , < Cˆ . As
bissetrizes externas dos ângulos  e Ĉ cortam os prolongamentos dos lados
opostos BC e AB nos pontos P e Q, respectivamente. Sabendo que
AP = CQ = AC , determine os ângulos de ABC.

PROBLEMA 5
Dizemos que um conjunto A formado por 4 algarismos distintos e não nulos é
intercambiável se podemos formar dois pares de números, cada um com 2
EUREKA! N°13, 2002

12
Sociedade Brasileira de Matemática

algarismos de A, de modo que o produto dos números de cada par seja o mesmo e
que, em cada par, todos os dígitos de A sejam utilizados.
Por exemplo, o conjunto {1;2;3;6} é intercambiável pois 21 ⋅ 36 = 12 ⋅ 63.
Determine todos os conjuntos intercambiáveis.

PROBLEMA 6
O matemático excêntrico Jones, especialista em Teoria dos Nós, tem uma bota com 5
pares de furos pelos quais o cadarço deve passar. Para não se aborrecer, ele gosta de
diversificar as maneiras de passar o cadarço pelos furos, obedecendo sempre às
seguintes regras:
• o cadarço deve formar um padrão simétrico em relação ao eixo vertical;
• o cadarço deve passar exatamente uma vez por cada furo, sendo indiferente se
ele o faz por cima ou por baixo;
• o cadarço deve começar e terminar nos dois furos superiores e deve ligar
diretamente (isto é, sem passar por outros furos) os dois furos inferiores.
Representamos a seguir algumas possibilidades.

Qual é o número total de possibilidades que o matemático tem para amarrar seu
cadarço, obedecendo às regras acima?
Observação: Maneiras como as exibidas a seguir devem ser consideradas iguais (isto
é, deve ser levada em conta apenas a ordem na qual o cadarço passa pelos furos).

PROBLEMAS – NÍVEL 3

PROBLEMA 1: Veja o problema 3 do Nível 2.

PROBLEMA 2
No triângulo ABC, a mediana e a altura relativas ao vértice A dividem o ângulo BÂC
em três ângulos de mesma medida. Determine as medidas dos ângulos do triângulo
ABC.

EUREKA! N°13, 2002

13
Sociedade Brasileira de Matemática

PROBLEMA 3
Determine todas as funções f: 5 → 5 tais que f(x) = f(–x) e f(x + y) = f(x) + f(y) + 8xy
+ 115 para todos os reais x e y.

PROBLEMA 4: Veja o problema 5 do Nível 2.

PROBLEMA 5
O matemático excêntrico Jones, especialista em Teoria dos Nós, tem uma bota com n
pares de furos pelos quais o cadarço deve passar. Para não se aborrecer, ele gosta de
diversificar as maneiras de passar o cadarço pelos furos, obedecendo sempre às
seguintes regras:
• o cadarço deve formar um padrão simétrico em relação ao eixo vertical;
• o cadarço deve passar exatamente uma vez por cada furo, sendo indiferente se
ele o faz por cima ou por baixo;
• o cadarço deve começar e terminar nos dois furos superiores e deve ligar
diretamente (isto é, sem passar por outros furos) os dois furos inferiores.
Por exemplo, para n = 4, representamos a seguir algumas possibilidades.

Determine, em função de n ≥ 2, o número total de maneiras de passar o cadarço


pelos furos obedecendo às regras acima.
Observação: Maneiras como as exibidas a seguir devem ser consideradas iguais.

PROBLEMA 6
x2
Seja f(x) = . Calcule
1+ x2
1 2 3 n
f   + f   + f   + ... + f  
1 1 1 1
1 2 3 n
+ f   + f   + f   + ... + f  
2 2 2 2

EUREKA! N°13, 2002

14
Sociedade Brasileira de Matemática

1 2 3 n


+ f   + f   + f   + ... + f  
3 3 3 3
+ ...
1 2 3 n
+ f   + f   + f   + ... + f   ,
n n n n
sendo n inteiro positivo.

SOLUÇÕES – NÍVEL 1

SOLUÇÃO DO PROBLEMA 1:
Cada tipo de pontuação aparece 8 vezes dentre as 28 peças do dominó. Portanto o
número total de pontos é: 8.(0 + 1 + 2 + 3 + 4 + 5 + 6) = 168.

SOLUÇÃO DO PROBLEMA 2:
Traçando a menor diagonal do paralelogramo, observamos que metade do mesmo
equivale a um triângulo retângulo pequeno, cuja área é ¼ da área do triângulo
retângulo grande, que, por sua vez, é ¼ da área do quadrado. Logo a área do
paralelogramo é igual a 2 × 1/16 = 1/8.

SOLUÇÃO DO PROBLEMA 3:
Ao furar após a primeira dobra, Carlinhos faz 2 furos; após a segunda dobra, faz 4


furos, após a terceira dobra, faz 8 furos, etc. Assim, ao desdobrar a folha, ele irá
contar 1 + 2 + 4 + 8 + furos. Notando que:
1 + 2 = 22 − 1 (após a primeira dobra)
1+2+4=2 −1 3
(após a segunda dobra)
1 + 2 + 4 + 8 = 24 − 1 (após a terceira dobra), etc

Basta encontrar o menor k tal que 2k – 1 é maior ou igual a 100


2 k − 1 ≥ 100 ⇔ k ≥ 7
Assim, o menor k vale 7. Isso corresponde a 6 dobras.

SOLUÇÃO DO PROBLEMA 4:
Os pontos correspondentes aos quadrados perfeitos pares e ímpares estão sobre os
lados vertical e horizontal do quadriculado, respectivamente. Os quadrados perfeitos
mais próximos de 2001 são 1936 = 442 e 2025 = 452. Como 2001 está mais
próximo de 2025, o ponto correspondente está no segmento vertical descendente que
termina em 2025. Logo o ponto imediatamente abaixo dele corresponde ao número
2002. Para achar o número do ponto imediatamente à esquerda, consideramos o

EUREKA! N°13, 2002

15
Sociedade Brasileira de Matemática

quadrado perfeito ímpar anterior, que é 432 = 1849. O ponto desejado está no
segmento ascendente que começa em 1850 e situado à mesma distância que o ponto
2001 está de 2025. Logo o número correspondente é: 1850 + (2025–2001) = 1850 +
24 = 1874.

1874 2001

24 24

1849 1850 2025

SOLUÇÃO DO PROBLEMA 5:
Sabemos que todos os números inteiros maiores do que 1 admitem pelo menos um
divisor (ou fator) primo. Dessa forma,
• se n tem dois divisores primos p e q então 1, p, q e pq são divisores de n; logo n
tem mais que três divisores;
• se n é primo, então tem somente dois divisores: 1 e n;
• se n é uma potência de um primo p, ou seja, é da forma ps, então 1, p, p2, ..., ps
são os divisores positivos de n. Para que n tenha três divisores s deverá ser igual
a 2, isto é, n = p2. Assim, os inteiros menores que 1000 com três divisores são:
4, 9, 25, 49, 121, 169, 289, 361, 529, 841, 961.

SOLUÇÃO DO PROBLEMA 6:
Os algarismos das unidades dos quadrados dos números de 1 a 10 são,
respectivamente, 1, 4, 9, 6, 5, 6, 9, 4, 1 e 0. Ora, a soma dos números formados por
esses algarismos é 45. Portanto, a soma 12 + 22 + 32 + 42 + …+ 102 tem como
algarismo das unidades o número 5. De 11 a 20, os algarismos das unidades dos
números se repetem na mesma ordem; portanto, o algarismo das unidades da soma
de seus quadrados também é 5. Conseqüentemente, a soma dos quadrados dos
números de 1 a 20 tem 0 como algarismo das unidades. Logo a soma 12 + 22 + 32 +
42 + ... + n2 tem zero como algarismo das unidades se N é múltiplo de 20. Como N =
12 + 22 + 32 + 42 + ... + 1968832 = 12 + 22 + 32 + 42 + … + 1968802 + 1968812 +
1968822 + 1968832, concluímos que o algarismo das unidades de N é o mesmo do
número 0 + 1 + 4 + 9 = 14, ou seja, 4.

EUREKA! N°13, 2002

16
Sociedade Brasileira de Matemática

SOLUÇÕES – NÍVEL 2

SOLUÇÃO DO PROBLEMA 1: Veja a solução do problema 2 do Nível 1.


SOLUÇÃO DO PROBLEMA 2: Veja a solução do problema 4 do Nível 1.

SOLUÇÃO DO PROBLEMA 3:
Observando que no ano n é realizada a (n – 1978)-ésima OBM, temos que o ano n é
super-olímpico se, e somente se, n – 1978 divide n. Assim, n – 1978 divide n – (n –
1978) = 1978. Como os divisores positivos de 1978 são 1, 2, 23, 43, 46, 86, 989 e
1978, os anos super-olímpicos são 1979, 1980, 2001, 2021, 2024, 2064, 2967 e
3956.

SOLUÇÃO DO PROBLEMA 4:
Os triângulos ACQ e PAC são isósceles. No triângulo ACQ, temos: CAˆ Q = AQˆ C = Aˆ
A Ĉ Q = Ĉ + (180° – Ĉ )/2 = 90° + Ĉ /2
Logo 2Aˆ + (90° + Ĉ /2) = 180° (1)
No triângulo PAC, temos:
C Â P = (180° – Â )/2
A Ĉ P = A P̂ C = 180° – Ĉ
Logo (180° – Â )/2 + 2(180° – Ĉ ) = 180° (2)
Resolvendo o sistema formado pelas equações (1) e (2), obtemos  = 12° e Ĉ = 132°;
daí, B̂ = 180° – 12° – 132° = 36°.
Q


90 , +
2
Â
A 180 , − Aˆ C
180 , − Ĉ
2

EUREKA! N°13, 2002

17
Sociedade Brasileira de Matemática

SOLUÇÃO DO PROBLEMA 5:
Seja A = {x ;y ;t ;z} um conjunto intercambiável. Então podemos supor, sem perda
de generalidade, que
(10x + y)(10t + z) = (10y + x)(10z + t) ⇔ xt = yz (1)
Por (1), temos que 5 e 7 não podem aparecer em A. Se o maior dos elementos de A
fosse menor ou igual a 4, teríamos A = {1;2;3;4}, que não é intercambiável. Logo A
possui pelo menos um dos dígitos 6, 8 ou 9.
Se o maior elemento de A é 9, temos por (1) que 3 e 6 também pertencem a A. Neste
caso temos o conjunto intercambiável A = {2;3;6;9}.
Se o maior elemento de A é 8, temos que 4 e outro algarismo par estão em A. Assim,
temos A = {1;2;4;8} ou A = {3;4;6;8}.
Se o maior elemento de A é 6, temos que 3 e outro algarismo par estão em A. Desta
forma, A = {1;2;3;6} ou A = {2;3;4;6}.
Assim, temos no total 5 conjuntos intercambiáveis: {2;3;6;9}, {1;2;4;8}, {3;4;6;8},
{1;2;3;6} e {2;3;4;6}.
Obs. O enunciado não deixaria claro que as outras possibilidades, por exemplo:
(10 x + y ) ⋅ (10t + z ) = (10 x + z ) ⋅ (10 y + t ) , não deveriam ser consideradas. A análise
dessas possibilidades torna o problema bem mais complicado, porém não acrescenta
novos conjuntos intercambiáveis aos listados acima.

SOLUÇÃO DO PROBLEMA 6:
Como o padrão deve ser simétrico, basta decidir os primeiros 5 furos pelos quais o
cadarço deve passar. A partir daí, os furos ficam determinados pela simetria. Por
exemplo, o 7° furo deve ser o outro furo da mesma linha visitada no 4° furo. Note,
ainda, que a simetria implica em que as linhas visitadas nos 5 primeiros furos são
todas distintas. Além disso, a primeira destas linhas é obrigatoriamente a de cima e a
5ª é obrigatoriamente a de baixo, já que os furos da linha de baixo são visitados
consecutivamente.
Assim, para obter um padrão para o cadarço, podemos iniciar pelo furo da esquerda
da linha superior e devemos decidir:
• em que ordem as 3 linhas intermediárias são visitadas
• de que lado queremos passar nestas 3 linhas e na linha de baixo.
Para escolher a ordem das 3 linhas, observamos que a primeira pode ser escolhida de
3 modos; a seguir, a segunda pode ser escolhida de 2 modos, ficando a terceira
determinada. Logo há 6 possibilidades de escolha para a ordem das linhas.
Para escolher o lado por onde passar nas 4 linhas, temos duas opções para cada uma
delas, para um total de
2 × 2 × 2 × 2 = 16 possibilidades. Logo o número total de modos de amarrar o
cadarço é 6 × 16 = 96.

EUREKA! N°13, 2002

18
Sociedade Brasileira de Matemática

Outra solução:
Começando do lado esquerdo da linha superior, o segundo furo pode ser escolhido
de 6 modos (qualquer um das linhas intermediárias); o terceiro de 4 modos (nas duas
intermediárias restantes) e o quarto e quinto de 2 modos cada (suas linhas estão
determinadas, bastando escolher o lado). Logo há um total de 6 × 4 × 2 × 2 = 96
possibilidades.

SOLUÇÕES – NÍVEL 3

SOLUÇÃO DO PROBLEMA 1: Veja a solução do problema 3 do nível 2.


SOLUÇÃO DO PROBLEMA 2:
A
α
α
α

B H M C

Seja M o ponto médio de BC e H o pé da altura relativa a A. Temos que AH é comum


aos triângulos AHM e AHB, AHˆ B ≅ AHˆ M (retos) e HÂM ≅ HÂB, logo, pelo caso
ALA, os triângulos AHM e AHB são congruentes. Assim, BH = HM = MC/2, pois
MC = MB. Como AM é bissetriz de HÂC, pelo teorema das bissetrizes AH/AC =
HM/MC ⇔ AH/AC = 1/2 ⇔ cos2α = 1/2. Como 0 < 2α  o, 2α = 60o ⇔ α = 30o.
Portanto os ângulos do triângulo ABC são m(BÂC) = 3α = 90o, m( ABˆ C ) = 90o – α 
60o e m( ACˆ B ) = 90o – 2α  o.

SOLUÇÃO DO PROBLEMA 3:
Fazendo y = –x, temos f(x + (–x)) = f(x) + f(–x) + 8x(–x) + 115 ⇔ f(0) = 2f(x) – 8x2 +
115 ⇔ f(x) = 4x2 + (f(0) – 115)/2. Fazendo x = 0 nesta última igualdade, temos f(0) =
4 ⋅ 02 + (f(0) – 115)/2 ⇔ f(0) = –115. Logo f(x) = 4x2 + (f(0) – 115)/2 ⇔ f(x) = 4x2 –
115 e verificamos de fato que esta função satisfaz as condições do enunciado: f(–x) =
4(–x)2 – 115 = 4x2 – 115 = f(x) e f(x) + f(y) + 8xy + 115 = 4x2 – 115 + 4y2 – 115 +
8xy + 115 = 4(x + y)2 – 115 = f(x + y). Assim, f(x) = 4x2 – 115 é a única função que
satisfaz todas as condições do enunciado.

SOLUÇÃO DO PROBLEMA 4: Veja a solução do problema 5 do Nível 2.

SOLUÇÃO DO PROBLEMA 5:
Numere os furos superiores com o número 1, os furos imediatamente abaixo com o
número 2 e assim por diante, até os furos inferiores, que recebem o número n.
EUREKA! N°13, 2002

19
Sociedade Brasileira de Matemática

Observe que basta estabelecermos os primeiros n furos onde o cadarço irá passar (o
padrão é simétrico). Uma maneira pode ser definida por uma seqüência indicando os
números dos primeiros n furos onde o laço passa (observe que tal seqüência tem
todos os números de 1 a n, começa com 1 e termina com n) e por uma outra
seqüência de comprimento n – 1 cujo k-ésimo termo indica se o cadarço muda de
lado ao passarmos do k-ésimo para o (k + 1)-ésimo termo da primeira seqüência. Por
exemplo, (1, 3, 2, 4) e (muda, não muda, muda) representa
1 1
2 2
3 3
4 4
Assim, como há (n – 2)! seqüências com os números de 1 a n começando com 1 e
terminando com n e 2n – 1 seqüências indicando se o cadarço muda de lado ou não, há
(n – 2)! ⋅ 2n – 1 maneiras.

SOLUÇÃO DO PROBLEMA 6:
x2 (1 / x) 2
Seja S a soma pedida. Como f(x) + f(1/x) = + = 1, podemos
1 + x 2 1 + (1 / x) 2
escrever
1 2 3
2S = f   + f   + f   +
1 1 1
 + f  1n 
1 2
+ f + f + f +
2 2
3
2
 + f  n2 
+ 
1 2 3 n
+ f + f + f + + f 
n n n n

 
1  
2  
3 n
+ f + f + f + + f 
 
1  
1  
1 1

1 2 3 n
+ f + f + f + + f 
2 2 2 2

+ 
1 2 3
+ f + f + f + + f 
n n n
n
n

 1 1   1   2    3   1 
⇔ 2 S =  f   + f    +  f   + f    +  f   + f   
 1 1   2   1    1   3 

+  +  f  nn  + f  nn   (n 2


pares de parcelas)

⇔ 2S = n 2
n2
⇔ S=
2

EUREKA! N°13, 2002

20
Sociedade Brasileira de Matemática

XXIII OLIMPÍADA BRASILEIRA DE MATEMÁTICA


Problemas e Soluções da Terceira Fase

PROBLEMAS – NÍVEL 1
PROBLEMA 1:
Numa famosa joalheria estão armazenadas várias pedras preciosas dos seguintes
tipos: esmeraldas; safiras e rubis. Todas as pedras do mesmo tipo têm o mesmo
valor. Além disso, 24 esmeraldas valem tanto quanto 12 rubis e também valem tanto
quanto 8 safiras.
Com R$350.000,00 um príncipe comprou um conjunto com 4 esmeraldas, 6 rubis e 4
safiras. Quanto custa cada tipo de pedra?

PROBLEMA 2:
Um cubinho foi colocado no canto de uma sala, conforme a Figura 1.
Empilharam-se outros cubinhos iguais ao primeiro, de forma a cobrir as faces
visíveis do mesmo, usando-se o menor número possível de peças. Como se pode ver
na Figura 2, após a colocação dos novos cubinhos, restam 9 faces visíveis desses
cubinhos.

Figura 1 Figura 2
a) Quantos cubinhos iguais a esses, no mínimo, seria necessário empilhar, de forma
a cobrir aquelas 9 faces visíveis?
b) Continua-se a fazer essa pilha, repetindo-se o procedimento descrito. Quando a
pilha tiver um total de 56 cubinhos, quantas faces poderão ser vistas?

PROBLEMA 3:
No triângulo ABC tem-se que M é o ponto médio do lado AB (isto é, os segmentos
AM e MB têm o mesmo comprimento). N é o ponto médio de MC e R é o ponto
médio de NA.
O triângulo ABC tem área 2000. Determine a área do triângulo AMR.
C

N
R

A M B

EUREKA! N°13, 2002

21
Sociedade Brasileira de Matemática

PROBLEMA 4:
Dizemos que um número natural é legal quando for soma de dois naturais
consecutivos e também for soma de três naturais consecutivos.
a) Mostre que 2001 é legal, mas 1999 e 2002 não são legais.
b) Mostre que 20012001 é legal.

PROBLEMA 5:
As 42 crianças de uma escola infantil deram as mãos formando uma fila e cada uma
delas recebeu um número da seguinte maneira: a primeira delas ficou com o número
1, a segunda ficou com o número 2 e, assim sucessivamente, até a última, que ficou
com o número 42. Continuando de mãos dadas, foram para um pátio, onde cada uma
delas ficou sobre uma lajota quadrada; duas crianças com números consecutivos
ficaram em lajotas vizinhas com um lado comum (ou seja, do lado esquerdo, do lado
direito, na frente ou atrás, mas nunca em diagonal).

Ao relatar esse fato para a diretora, a inspetora Maria fez o desenho à esquerda,
mostrando a posição de três crianças sobre o retângulo formado pelas 42 lajotas,
sobre as quais estavam as crianças. Num outro comunicado, a inspetora Célia fez
outro desenho, mostrado à direita, com a posição das mesmas crianças sobre o
mesmo retângulo. Ao receber os dois desenhos a diretora disse a uma das inspetoras:
"O seu desenho está errado".

i) Com qual das duas inspetoras a diretora falou? Qual foi o raciocínio da
diretora?

ii) Complete o desenho correto satisfazendo as condições do enunciado.

11 20 11 20

31

31

(Desenho de Maria) (Desenho de Célia)

EUREKA! N°13, 2002

22
Sociedade Brasileira de Matemática

PROBLEMAS – NÍVEL 2

PROBLEMA 1:
Uma folha de papel retangular ABCD, de área 1, é dobrada em sua diagonal AC e,
em seguida, desdobrada; depois é dobrada de forma que o vértice A coincida com o
vértice C e, em seguida, desdobrada, deixando o vinco MN, conforme desenho
abaixo.
A M B

D N C
a) Mostre que o quadrilátero AMCN é um losango.
b) Se a diagonal AC é o dobro da largura AD, qual é a área do losango AMCN?

PROBLEMA 2: Veja o problema 5 do Nível 2.

PROBLEMA 3:
Dado um inteiro positivo h demonstre que existe um número finito de triângulos de
lados inteiros a, b, c e altura relativa ao lado c igual a h .

a b
h

.
c
PROBLEMA 4:
Mostre que não existem dois números inteiros a e b tais que (a + b) (a2 + b2) = 2001.

PROBLEMA 5:
Sejam a, b e c números reais não nulos tais que a + b + c = 0.
(a 3 + b 3 + c 3 ) 2 (a 4 + b 4 + c 4 )
Calcule os possíveis valores de .
(a 5 + b 5 + c 5 ) 2
PROBLEMA 6:
Em um quadrilátero convexo, a altura em relação a um lado é definida como a
perpendicular a esse lado passando pelo ponto médio do lado oposto. Prove que as
quatro alturas têm um ponto comum se e somente se o quadrilátero é inscritível, isto
é, se e somente se existe uma circunferência que contém seus quatro vértices.

EUREKA! N°13, 2002

23
Sociedade Brasileira de Matemática

PROBLEMAS – NÍVEL 3
PROBLEMA 1:
Prove que (a + b)(a + c) ≥ 2 abc (a + b + c) para quaisquer números reais positivos
a, b e c.

PROBLEMA 2:
Dado um inteiro a 0 > 1 definimos uma seqüência (a n ) n ≥0 da seguinte forma; para
cada k ≥ 0 , a k +1 é o menor inteiro a k +1 > a k tal que mdc (a k +1 , a 0 ⋅ a1 ⋅ ... ⋅ a k ) = 1.
Diga para quais valores de a 0 temos que todos os termos ak da seqüência são
primos ou potências de primos.

PROBLEMA 3:
E e F são pontos do lado AB, do triângulo ABC, tais que AE = EF = FB. D é ponto
da reta BC tal que BC é perpendicular a ED. AD é perpendicular a CF. Os ângulos
BDF e CFA medem x e 3x, respectivamente. Calcule a razão (DB) / (DC).

PROBLEMA 4:
Uma calculadora tem o número 1 na tela. Devemos efetuar 2001 operações, cada
uma das quais consistindo em pressionar a tecla sen ou a tecla cos. Essas operações
calculam respectivamente o seno e o cosseno com argumentos em radianos. Qual é o
maior resultado possível depois das 2001 operações?

PROBLEMA 5: Veja o problema 6 do Nível 2.

PROBLEMA 6:
Temos uma fileira longa de copos e n pedras no copo central (copo 0). Os seguintes
movimentos são permitidos:

Movimento tipo A


i–1 i i+1 i+2 i–1 i i+1 i+2
Se há pelo menos uma pedra no copo i e pelo menos uma no copo i + 1 podemos
fazer uma pedra que está no copo i + 1 pular para o copo i – 1 eliminando uma pedra
do copo i.

Movimento tipo B.

EUREKA! N°13, 2002

24
Sociedade Brasileira de Matemática


i–1 i i+1 i+2 i–1 i i+1 i+2

Se há pelo menos duas pedras no copo i podemos pular uma para o copo i + 2 e uma
outra para o copo i – 1.
Demonstre o seguinte fato: fazendo os movimentos tipo A ou B durante um tempo
suficientemente longo sempre chegaremos a uma configuração a partir da qual não é
mais possível fazer nenhum desses dois tipos de movimento. Além disso essa
configuração final não depende da escolha de movimentos durante o processo.

SOLUÇÕES – NÍVEL 1
PROBLEMA 1: SOLUÇÃO DE RAPHAEL RODRIGUES MATA (SALVADOR – BA)
Se 24 esmeraldas equivalem a 12 rubis, significa que 1 rubi equivale a duas
esmeraldas, e se 24 esmeraldas equivalem a 8 safiras, uma safira equivale a 3
esmeraldas. Assim, se o príncipe comprar 6 rubis, é o mesmo que ele comprar 12
esmeraldas, e se ele comprar 4 safiras, é o mesmo que ele comprar 12 esmeraldas.
Assim, o conjunto comprado pelo príncipe tem o mesmo valor de 28 esmeraldas (4 +
12 + 12 = 28).
350000
Para se descobrir o valor de cada esmeralda, basta efetuar = 12500.
28
Sabemos que o rubi vale o dobro da esmeralda, assim, temos 12500 × 2 = 25000.
Por fim, sendo a safira o triplo do valor da esmeralda, temos 12500 × 3 = 37500.
Finalmente, descobrimos que a esmeralda custa R$12500,00; o rubi custa
R$25000,00; e cada safira tem o valor de R$37500,00.

PROBLEMA 2: SOLUÇÃO DE EDUARDO FISCHER (ENCANTADO - RS)


a) São 6 cubos; chego a esta solução apenas olhando. Na fileira de baixo se
acrescentam 3 cubos, na do meio 2 e na de cima 1 (a de cima antes estava vazia).
Note que primeiro foi botado um cubo, depois 3, que é um mais dois, agora bota
6, que é 1 + 2 + 3. Depois acrescentarei 10 (1 + 2 + 3 + 4) e depois 15 (1 + 2 + 3
+ 4 + 5). Isso se deve ao fato que, ao quadricular o chão, na figura 1 se
acrescenta um cubo ao nada. Depois se acrescenta 2 cubos (no chão) e 1 em
cima. Após termos 4 cubos, se acrescenta 3 (no chão), 2 para cobrir os 2 que
antes estão no chão (os mais distantes da parede) e 1 para cobrir lá em cima. E
assim segue.
b) Para chegar a 56, vou somando: 1, 3 = 1 + 2, 6 = 1 + 2 + 3, 10 = 1 + 2 + 3 + 4,
15 = 1 + 2 + 3 + 4 + 5 e 21 = 1 + 2 + 3 + 4 + 5 + 6. A parede terá a altura de 6
cubos, quando isso acontecer. Vamos listar as faces e cubos à mostra:

EUREKA! N°13, 2002

25
Sociedade Brasileira de Matemática

No andar de cima há 1 cubo e 3 faces.


No segundo andar há 6 faces e 2 cubos.
No terceiro andar há 9 faces e 3 cubos.
No quarto andar há 12 faces e 4 cubos.
No quinto andar há 15 faces e 5 cubos.
No andar de baixo há 18 faces e 6 cubos.
Para cada cubo à mostra, há 3 faces vistas. São 21 cubos à mostra, 63 faces no total.

PROBLEMA 3: SOLUÇÃO DE FELIPE GONÇALVES ASSIS (CAMPINA GRANDE – PB)


O triângulo ABC tem área 2000. Ao ser cortado pelo segmento de reta MC, divide-se
em outros 2 triângulos menores, de mesma área, ACM e CMB. Pode-se perceber que
eles têm áreas iguais pois:
• A base de ambos tem o mesmo comprimento, pois AM = MB, já que M é o ponto
médio de AB.
• A altura dos dois também é a mesma.
• Duas medidas que determinam a área de um triângulo são, justamente, base e
altura. Assim descobrimos uma propriedade dos triângulos:

Se um triângulo for cortado por um segmento de reta que parte do ponto médio de
um dos segmentos que o compõem até o vértice formado pelas outras duas retas,
obter-se-ão 2 novos triângulos, de mesma área, correspondente a metade da área do
primeiro triângulo.
Isto é o que ocorre com o triângulo ABC que forma os triângulos ACM e CMB, cada
um com área 1000 (= 2000:2).
Ocorre isto também com ACM, cortado pelo segmento AN, ele forma AMN e ACN,
ambos com área 500 (= 1000:2).
Acontece o mesmo com AMN que é cortado por RM, originando AMR e RMN, cada
qual com área 250(= 500:2).
Resposta: A área de AMR é de 250.

PROBLEMA 4: SOLUÇÃO DE PAULO ANDRÉ CARVALHO DE MELO (RIO DE JANEIRO – RJ)


Para um número ser a soma de 2 naturais consecutivos ele tem que ser:
x + x + 1 = 2x + 1.
Ou seja, ímpar.
Para um número ser a soma de 3 naturais consecutivos ele tem que ser:
x + x + 1 + x + 2 = 3x + 3.
Ou seja, múltiplo de 3.
Portanto um número legal é aquele que é múltiplo de 3 e ímpar
a) O 2001 é múltiplo de 3 e é ímpar, mas o 2002 e o 1999 não são.

EUREKA! N°13, 2002

26
Sociedade Brasileira de Matemática

b) Já que 2001 é múltiplo de 3, 20012001 terá 32001 como um de seus fatores primos e
será ímpar pois um número ímpar multiplicado por outro número ímpar é igual a
número ímpar.
Portanto 20012001 é legal, já que respeita as condições para um número ser legal.

PROBLEMA 5: Veja a solução do Problema 2 do Nível 2.

SOLUÇÕES – NÍVEL 2

PROBLEMA 1: a) SOLUÇÃO DE ELTON GOMES CORIOLANO (FORTALEZA – CE)


Se fizermos a segunda dobradura, teremos a seguinte figura:
B

C
A M

D N

Logo percebemos que AN = CN e que AM = CM . Temos, então, dois triângulos


isósceles: o triângulo ANC e o triângulo AMC. Então os ângulos CAˆ N e ACˆ N são
congruentes e os ângulos CAˆ M e ACˆ M são congruentes também. Sabendo que AM é
paralelo a NC, pode-se dizer que os ângulos CAˆ M e ACˆ N são congruentes, pois estes
são ângulos alternos internos. Assim, CAˆ N = ACˆ N = CAˆ M = ACˆ M . Portanto, os
ângulos MAˆ N e MCˆ N são congruentes. Logo, ACˆ N é congruente a AMˆ C . Assim,
AMCN é paralelogramo, pois seus ângulos opostos são congruentes. Por este motivo,
os lados opostos também serão iguais, ou seja, AM = CN e AN = CM. Dessa forma,
AM = CN = AN = CM. Logo, o quadrilátero AMCN é um paralelogramo com todos
os lados iguais, ou seja, AMCN é um losango.
A M B

D N C

b) SOLUÇÃO DE THIAGO COSTA LEITE SANTOS (SÃO PAULO – SP)


Seja T o centro do retângulo. Como AT é metade de AC , AT = AD , os triângulos
ATN e ADN são retângulos, logo ∆ ATN ≡ ∆ ADN , pelo caso especial cateto-

EUREKA! N°13, 2002

27
Sociedade Brasileira de Matemática

hipotenusa (a hipotenusa comum aos dois triângulos). Analogamente, temos


∆CBM ≡ ∆CTM e portanto o retângulo ABCD está dividido em 6 triângulos
( ABCD ) 1
congruentes. Portanto a área de cada triângulo é igual a = e como o
6 6
1 2
losango AMCN possui 4 dos 6 triângulos, sua área será igual a 4 ⋅ = .
6 3
PROBLEMA 2: SOLUÇÃO DE RAFAEL DAIGO HIRAMA (CAMPINAS – SP)
i) Vamos pintar o retângulo igual a um tabuleiro de damas. Pelo enunciado o número
anterior (e o posterior) de um número está acima, abaixo, a esquerda ou a direita.
Portanto, um número tem sua cor diferente de seus dois vizinhos.
Então, se o primeiro não é pintado, o segundo é, o
terceiro não é, o quarto é, etc.
Ou seja, os números pares tem uma cor e os
impares outra.
Pintando os dois tabuleiros percebemos que no de Célia o 20 e o 31 estão da mesma
cor. Logo o desenho de Célia está errado.
11 20 11 20

31

31

Desenho da Maria
ii) Para completar o desenho, vejamos que o 10 e o 12 devem estar no quadrado à
esquerda e no abaixo do 11 pois este não tem mais vizinhos. Igualmente o 19 e o 21
estão à direita ou abaixo do 20.
O número 12 deve estar abaixo do 11 pois, se não o 9 teria que ficar no lugar
marcado com um círculo, e não haveria lugar para o 8 (pois o único modo de
conectar o 12 com o 20 seria como na figura).
12 11 20
Logo o 12 está abaixo do 11.
13 10 19

14 9 18

15 16 17

Vamos provar que o 19 está abaixo de 20. Se o 19 estivesse a direita de 20 travaria a


saída para o 21 formando uma barreira sem saída. Logo o 19 está abaixo do 20.
Com isso, podemos montar algo obrigatório (tudo por falta de outras opções).

EUREKA! N°13, 2002

28
Sociedade Brasileira de Matemática

10 11 20 21

9 12 19
1) O 9 tem que estar abaixo do 10.
2) O 13 abaixo do 12.
8 13
3) O 8 abaixo do 9.
7
4) O 7 abaixo do 8.
31

Temos que o 21 deve alcançar o 31 em 9 espaços, o 13 alcançar o 19 em 5 espaços e


temos mais 6 espaços para fazer do 6 ao 1.
Se colocarmos o 6 à direita do 7 teríamos problemas, pois o caminho entre o 21 e o
31 (o 30 não pode ficar embaixo pois senão isolaria o 31 de uma parte em branco
pois o cordão 29 a 31 impediria e nenhuma parte do cordão pois senão também faria
uma área sem alcance).
10 11 20 21 22 23

9 12 19 24
Daria pouco espaço para 11 espaços (14 → 18 e 1
8 13 25
→ 6) (O 30 deve ficar na direita do 31 neste caso
pois abaixo e à esquerda ocorreria o espaço sem
7 6 26
alcance), 8 no máximo.
31 30 29 28 27

Então o 6 é abaixo do 7, o 5 abaixo do 6, o 4 à direita do 5, o 3 à direita do 4 (estes 3


por falta de opção) e o 2 à direita do 3 (pois à esquerda do 31 tem o 6 e abaixo tem o
4 sobrando os outros dois (acima e à direita para o 30 e o 32).
Logo, teremos:
10 11 20 21 22 23 Agora, o cordão 21 → 31 deve ter o 30 acima do 31
9 12 19 22 23 24 pois senão forma a mesma área sem alcance como os
8 13 28 27 24 25 exemplos. Então o 32 fica à direita do 31 e o 29 à
7 29 26 25 26 direita do 30, o 14 à direita do 13, o 28 à direita do
6 31 30 29 28 27 29, o 33 à direita do 32, o 1 à direita do 2, o 34 à
5 4 3 2 direita do 33, o 15 à direita do 14, o 18 à direita do
19, o 22, 17, 16, 27 à direita do 21, 18, 15 e 28
10 11 20 21 22 23
respectivamente. O 23, 26 e 35 devem ficar à direita
9 12 19 18 17 24
do 22, 27 e 34, respectivamente. Como o 23 deve
8 13 14 15 16 25
chegar ao 26 em 2 espaços, o 24 fica embaixo do 23
7 30 29 28 27 26
e o 25, embaixo do 24. Então é só completar com o
6 31 32 33 34 35
35 a 42 do único modo possível.
5 4 3 2 1

EUREKA! N°13, 2002

29
Sociedade Brasileira de Matemática

Respostas: i) A diretora falou com Célia. A direitora percebeu que o 20 e o 31 não


poderiam estar na mesma cor se o tabuleiro fosse pintado como o de damas.
ii)
10 11 20 21 22 23 42
9 12 19 18 17 24 41

8 13 14 15 16 25 40

7 30 29 28 27 26 39
6 31 32 33 34 35 38

5 4 3 2 1 36 37

Obs. Estendendo-se esse raciocínio é possível demonstrar que esta é a única maneira
de se completar o desenho.

PROBLEMA 3: SOLUÇÃO DE THIAGO COSTA LEITE SANTOS (SÃO PAULO – SP)

h b
a

m . n α
c

a2 = h2 + m2
Temos
b2 = h2 + n2
Para uma equação Pitagórica:
x2 = y2 + z2, resolvida em inteiros positivos, temos:
z 2 ≥ 2 y + 1 (pois senão y 2 < x 2 < ( y + 1) 2 )
y 2 ≥ 2 z + 1 (análoga a de cima)
z2 ≥ 2y + 1 z2 ≥ 2y +1 2
 y2 −1
⇔ y2 −1 ⇔  y 2 −12 ⇔   ≥ z2 ≥ 2y +1

≥z   ≥z 2
 2 
 
2  2 
Mas se substituirmos y por h e z por m ou n, teremos que estes estarão limitados a
certos valores, logo a, b, e c estão limitados a certos valores e, portanto acabou!!!
Mas supomos que m e n são inteiros positivos e eles poderiam ser irracionais e a
soma de dois irracionais dar um inteiro.
Para isto podemos aplicar a lei dos cossenos.

EUREKA! N°13, 2002

30
Sociedade Brasileira de Matemática

n
cos α = ; n = (b 2 − h 2 )
b
a 2 = b 2 + c 2 − 2bc cos α ⇔ a 2 = b 2 + c 2 − 2c ⋅ b 2 − h 2 . Assim, se n é irracional,
a2 também será, absurdo. Portanto m e n são inteiros.

Obs.: m e n não poderiam ser fracionários, observando as equações iniciais, que


mostram que m2 e n2 são inteiros. Note que se n = p/q é um racional não inteiro,
então n2 = p2/q2 também não será inteiro, pois se q não divide p então q2 não divide
p2.

PROBLEMA 4: SOLUÇÃO DE HENRY WEI CHENG HSU (SÃO PAULO – SP)


2001 = 3. 23 . 29
Temos que a2 + b2 é sempre ≥ 0, então, para o produto (a + b) (a2 + b2) valer 2001,
a + b não pode ser negativo.
Como a e b são inteiros, a2 ≥ a e b2 ≥ b, assim a2 +b2 ≥ a + b (a2 < a somente quando
0 < a < 1)
Os valores possíveis para (a + b) e (a2 + b2) são:
1) a + b = 1 e a2 + b2 = 2001
2) a + b = 3 e a2 + b2 = 667
3) a + b = 23 e a2 + b2 = 87
4) a + b = 29 e a2 + b2 = 69
Vamos analisar os casos 2) e 3).
Em ambos temos a + b ≡ 3(mod 4) e a2 + b2 ≡ 3 (mod 4)
Para a + b ≡ 3 (mod 4), podemos ter os seguintes casos:
a ≡ 0 e b ≡ 3 ⇒ a 2 ≡ 0 e b2 ≡ 1
a ≡ 1 e b ≡ 2 ⇒ a 2 ≡ 1 e b2 ≡ 0
a ≡ 2 e b ≡ 1 ⇒ a 2 ≡ 0 e b2 ≡ 1
a ≡ 3 e b ≡ 0 ⇒ a 2 ≡ 1 e b2 ≡ 0
Então, quando a + b ≡ 3 (mod 4), a2 + b2 será congruente a 1 (mod 4). Portanto os
casos 2) e 3) não podem existir. Vamos analisar o caso 1)
Para a = 1 e b = 0 e a = 0 e b = 1, a2 + b2 sempre valerá 1.
Quando um dos dois for negativo: a = n + 1 e b = – n ou vice-versa
a+b=n+1–n=1
a2 + b2 = n2 + 2n + 1 + n2 = 2n2 + 2n + 1
a2 + b2 = 2001 ⇔ 2n2 + 2n + 1 = 2001 ⇔ 2n2 + 2n – 2000 = 0 ⇔ n2 + n – 1000 = 0
n2 + n – 1000 = 0
∆ = 1 + 4000 = 4001 como 4001 não é quadrado perfeito, n não é inteiro.
Outra maneira: a + b = 1 ⇔ a = 1 – b
a2 + b2 = b2 – 2b + 1 + b2 = 2b2 – 2b + 1

EUREKA! N°13, 2002

31
Sociedade Brasileira de Matemática

2b2 – 2b + 1 = 2001 ⇔ 2b2 – 2b – 2000 = 0 ⇔ b2 – b – 1000 = 0


∆ = 1 + 4000 = 4001 (não é quadrado perfeito).
Assim, o caso 1) não existe. Vamos analisar o caso 4).
a + b = 29 ⇔ b = 29 – a
a2 + b2 = a2 + 841 – 58a + a2 = 2a2 – 58a + 841
a2 + b2 = 69 ⇔ 2a2 – 58a + 772 = 0 ⇔ a2 – 29a + 386 = 0
∆ = 841 – 1544 < 0
Como ∆ < 0, a não é inteiro. Assim, o caso 4) não pode ocorrer.
Como nenhum caso pode ocorrer, não existem dois números a e b tais que
(a + b) (a2 + b2) = 2001.

PROBLEMA 5: SOLUÇÃO DE MARCELA SOBRINHO PEREIRA (FORTALEZA – CE)


Pelo enunciado, temos: a + b + c = 0
a = – (b + c) ⇒ a3 = – b3 – c3 –3b2c –3bc2 = – b3 – c3 –3bc(b + c),
a4 = b4 + c4 + 4b3c + 4c3b + 6c2b2 e a5 = – b5 – c5 –5b4c –5c4b – 10b3c2 –10c3b2.
Substituindo na expressão, temos:
( )
(− 3bc(b + c))2 2b 4 + 2c 4 + 4b 3 c + 4bc 3 + 6c 2 b 2 =
( ( ))
5bc b 3 + c 3 + 2b 2 c + 2bc 2
2

9b c (b + c) (2b + 2c + 4b c + 4bc 3 + 6c 2 b 2 )
2 2 2 4 4 3
=
25b 2 c 2 (b 3 + c 3 + 2b 2 c + 2bc 2 ) 2
9 2(b 6 + c 6 + 8b 2 c 4 + 4b 5 c + 10b 3 c 3 + 8b 4 c 2 + 4bc 5 ) 18
⋅ = .
( )
25 b 6 + c 6 + 8b 2 c 4 + 4b 5 c + 10b 3 c 3 + 8b 4 c 2 + 4bc 5 25

PROBLEMA 6: Veja a solução do problema 5 do nível 3.

SOLUÇÕES – NÍVEL 3

PROBLEMA 1: SOLUÇÃO DE CARLOS STEIN NAVES DE BRITO (GOIÂNIA - GO)


Seja a + b = x, a + c = y e b + c = z. (x, y, z > 0)
Temos um sistema linear de variáveis a, b e c e
 x+ y−z
a + b = x → a = 2
a + b = x a + b = x 
   x+z− y
Resolvendo a + c = y ⇔ − b + c = y − x ⇔ − b + c = y − x → b =
b + c = z b + c = z  2
   z+ y−x
c =
 2

EUREKA! N°13, 2002

32
Sociedade Brasileira de Matemática

Chamando x + y + z = 2p. Temos a = p – z, b = p – y e c = p – x.


Logo a desigualdade vira:
(a + b)(a + c) ≥ abc(a + b + c) ⇔
  
 p − z + p − y   p − z + p − x  ≥ 2 ( p − z )( p − y )( p − x)( p − x + p − y + p − z ) ⇔




x
 
 y




x ⋅ y ≥ 2 ( p − z )( p − y )( p − x) p . Basta provar isso.


Temos que:
 ( a + b) + ( a + c ) > b + c ⇔   (a + c) + (b + c) > ( a + b) ⇔ 
   
x+ y>z  2a > 0 ⇔  ;y+z>x  2c > 0 ⇔ 
 a>0   c>0 
   
 (a + b) + (b + c) > (a + c) ⇔ 
 
e x+z> y  2b > 0 ⇔ .
 b>0 
 

Assim x, y e z podem ser lados de um triângulo, sendo p o semiperímetro.


Assim seja α o ângulo desse triângulo entre x e y.
x ⋅ ysenα x ⋅ y ⋅ 1 x ⋅ ysenα
Logo a área A dele é: A = ; como senα ≤ 1, temos ≥ =A
2 2 2
Mas A em função dos lados é ( p − z )( p − y )( p − x) p , logo
xy
≥ A = ( p − z )( p − y )( p − x) p ⇔ s xy ≥ 2 ( p − z )( p − y )( p − x) p , cqd.
2

PROBLEMA 2: SOLUÇÃO DE DAVI MÁXIMO ALEXANDRINO NOGUEIRA (FORTALEZA - CE)


Notação: mdc (x, y) = (x, y).
Já que a0 faz parte de (an)n ≥ 0, o próprio deve ser potência de primo.
Suponha a0 = pm (p primo). Considere primeiro p = 2: a0 = 2m.
a1 é o menor inteiro que satisfaz a1 > a0 e (a1, a0) = 1. Portanto, temos
a1 = a0 + 1 = 2m + 1. Se m for ímpar, a1 = 2m + 1 ≡ 0 (mod 3)
⇒ 3|a1 ⇒ a1 = 3n, pois todo termo de (an)n ≥ 0 deve ser potência de primo.
Logo 2m + 1 = 3n.
Se m = 1, 3 = 3n ⇒ n = 1, a1 = 3 e a0 = 2 (I)
Se m ≥ 2, segue que 4|2m, e 2m +1 ≡ 3n (mod 4) ⇒ 1 ≡ (–1)n (mod 4) ⇒ n par,
digamos n = 2n0. Logo, ficamos com:

EUREKA! N°13, 2002

33
Sociedade Brasileira de Matemática

3n0 −1 = 2α
2m +1 = 32n0 ⇒ 2m = (3n0 −1)(3n0 +1) ⇒  n ⇒ 2 = 2β − 2α ⇒ 2 = 2α (2β −α −1) ⇒α = 1
3 0 +1 = 2β
e β = α + 1 = 2 ⇒ m = 3 e n = 2 ⇒ a 0 = 8 a1 = 9 (II)
Agora observemos:
(I) a 0 = 2 a1 = 3
Então, a 2 = 5 , a 3 = 7 , a 4 = 11. Provaremos por indução:
ai = (i + 1)-ésimo primo
Supondo a hipótese válida para j, olhemos o passo indutivo j → j + 1:
a j +1 > a j ; (a j +1 , a 0 a1 ...a j ) = 1 . Seja p o (j + 2)-ésimo primo.
Por hipótese, ai = (i + 1)-ésimo primo. Seja x ∈{a j + 1, a j + 2,..., p − 1} ⇒ x > a j
porém, ( x, a 0 a1 ...a j ) ≠ 1 pois os fatores primos de x pertencem ao produto a 0 a1 ...a j
logo, a j +1 = p pois p é o menor inteiro tal que p > aj e (p, a0…aj) = 1 e o resultado
segue.
(II): a0 = 8 a1 = 9 ⇒ a2 = 11, a3 = 13, a4 = 17, a5 = 19, a6 = 23, a7 = 25 = 52, a8 = 29,
a9 = 31, a10 = 37, a11 = 41, a12 = 43, a13 = 47, a14 = 49 = 72, a15 = 53.
Prova analogamente por indução ("mesmo" passo indutivo anterior) que a partir de
i ≥ 15, ai = (i + 1)-ésimo primo (indução feita no anexo).
Sendo assim, suponha a0 = 2m com m par (m > 0). Temos:
a0 = 2m ⇒ a1 = 2m + 1 ⇒ a2 = 2m + 3 (pois, (a2, a1) = (a2, a0) = 1) ⇒ a3 = 2m + 5.
Porém, como m é par, a3 ≡ 2m + 5 ≡ (–1)m + 5 ≡ 0 (mod 3) ⇒ 2m + 5 = 3n
Resolvamos então 2m + 5 = 3n.
Como 2m = a0 > 1 ⇒ m > 0. Mas como estamos supondo m par ⇒ m ≥ 2 ⇒ 4 | 2 m .
Logo,
2m + 5 = 3n ⇒ 2m + 5 ≡ 3n (mod 4) ⇒ 3n ≡ 1 (mod 4) ⇒ n par, digamos n = 2n0.
Logo,
22m0 + 5 = 32n0 ⇒ 32n0 − 22m0 = 5 ⇒ (3n0 − 2 m0 )(3n0 + 2 m0 ) = 5 ⇒
3n0 − 2 m0 = 1
⇒ n (pois 3n0 − 2m0 < 3n0 + 2m0 )
3 0 + 2m0 = 5
⇒ 2 m0 +1 = 4 ⇒ m0 = 1 ⇒ m = n = 2 ⇒a 0 = 4
⇒ a1 = 5, a 2 = 7, a 3 = 9
Prova-se novamente por indução (mesmo passo indutivo da anterior) que para todo
i ≥ 4, ai = (i + 1)-ésimo primo.

EUREKA! N°13, 2002

34
Sociedade Brasileira de Matemática

Agora, suponha a 0 = p m , p ≠ 2. ⇒ a1 = p m + 1 = 2 n pois 2 | p m + 1. Se n for par,


digamos n = 2n0, veja (mod 3):
p m + 1 ≡ 2 2 n0 (mod 3) ⇒ p m + 1 ≡ 1(mod 3) ⇒ p = 3
sendo assim; resolvemos
3 m + 1 = 2 2 n0
2 n0 + 1 = 3α
⇒ 3 m = (2 n0 − 1)(2 n0 + 1) ⇒  n
2 0 − 1 = 3 β
⇒ 3α − 3 β = 2 ⇒ 2 = 3 β (3α − β − 1) ⇒ β = 0, α = 1 ⇒ m = 1
e n = 2 ⇒ a 0 = 3, a1 = 4 ⇒ a 2 = 5 e para i ≥ 2 ⇒ a i = (i + 1) -ésimo primo (pelo
mesmo argumento indutivo).
Sendo assim, suponha n ímpar: a 0 = p m , a1 = p m + 1 = 2 n e a 2 = p m + 2 = 2 n + 1 (já
que (pm, pm + 2) = (pm + 2, pm + 1) = 1) ⇒ 3 | a 2 (já que n é ímpar) ⇒ a 2 = 3 t logo,
3 t − 2 n = 1 (*)
mas já é sabido nosso que as soluções de (*) se dão para:
t = 1 e n = 1 ⇒ a2 = 3 e a1 = 2 ⇒ a0 = 1 (Absurdo!)
t = 2 e n = 3 ⇒ a2 = 9 e a1 = 8 ⇒ a0 = 7 (III)
(III): a0 = 7, a1 = 8, a2 = 9, a3 = 11, a4 = 13, a5 = 17, a6 = 19, a7 = 23, a8 = 25 = 52,
a9 = 29 e, para i ≥ 9, ai = (i + 1)-ésimo primo (como antes).
Resposta: a0 pode ser 2, 3, 4, 7 ou 8.

Anexo:
Para um j > 1, suponha a0, a1, …, aj definidos como disse e a hipótese valendo.
Basta ver que no conjunto {a0, a1,…,aj} aparecem todos os j + 1 primeiros fatores
primos. Chame p = (j + 2)-ésimo primo. Os candidatos a aj + 1 antes de p seriam
aj + 1, aj + 2,…, p – 1. Porém, os fatores primos de qualquer um desses caras
aparecem no produto a0 a1,…, aj ⇒ se x ∈ { aj + 1,…, p – 1},
mdc (x, a0 a1,…, aj) ≠ 1. Logo, x = p (veja que p >aj e mdc (p, a0 a1,… ,aj) = 1).
Os outros passos de indução são totalmente análogos.

EUREKA! N°13, 2002

35
Sociedade Brasileira de Matemática

PROBLEMA 3: SOLUÇÃO DE LARISSA CAVALCANTE QUEIROZ DE LIMA (FORTALEZA - CE)


E F
A 2x x B
(m + n)
x
m
n M
x
m C
x n
x
D
G

ED ⊥ BC ⇒ ∆ EDB é um triângulo retângulo.


Como EF = FB, F é ponto médio de EB ⇒ EF = FB = DF
⇒ Se FDˆ B = x , temos FBˆ D = x ⇒ DFˆE = FDˆ B + DBˆ F = x + x = 2 x < 3x = CFˆA
portanto D não está dentro do segmento BC ⇒ H, pé da altura relativa ao lado BC
está fora do segmento BC (pois AH // ED e H, D, B estão na reta nessa ordem ⇒ H
∉ DB e como D ∉ CB , temos H ∉ CB ) ⇒ ACˆ B é um ângulo obtuso.
* CFˆD = CFˆA − DFˆA = 3 x − 2 x = x ⇒ CFˆD = FDˆ C ⇒ ∆CFD é isósceles com CD =
CF = m.
Seja M o ponto médio de DB ⇒ MF é base média de ∆BDE ⇒ MF // ED ⇒
FMˆ B = EDˆ B = 90° ( DGˆ F = 90° pois AD ⊥ CF e G = AD ∩ CF)
⇒ DGMF é um quadrilátero inscritível ⇒ FDˆ M = FGˆ M = x e DFˆG = DMˆ G = x
⇒ CGˆ M = CMˆ G = x ⇒ ∆CMG é isósceles com CM = CG = n
DM = MB ⇒ MB = DC + CM = m + n e DB = DM + MB = 2(m + n)
GF = GC + CF = m + n
Menelaus ∆ FCB e reta ADG ⇒
AF DB GC 2 2(m + n) n
⋅ ⋅ =1⇒ ⋅ ⋅ = 1 ⇒ 4n = 3m
AB DC GF 3 m m+n
3m 4m + 3m 7
2m + m⋅
DB 2(m + n) 2m + 2n 2 = 2 2=7
= = = =
DC m m m m m 2
DB 7
∴ = .
DC 2
Obs. Ĉ não obtuso ⇒ H ∈ CB e portanto
 AFˆD = AFˆC + CFˆD = 3x + y = 2 x ⇒ y = − x < 0 
D ∈ CB ⇒   contradição.
CFˆD ≥ 0. 

EUREKA! N°13, 2002

36
Sociedade Brasileira de Matemática

PROBLEMA 4: SOLUÇÃO DE ALEX CORRÊA ABREU (NITERÓI - RJ)


Definimos as seqüências (xk), (yk) como sendo respectivamente o máximo e o
mínimo depois da operação feita k vezes ⇒ x k +1 = senx k ou cos y k pois a função
seno é crescente e a co-seno decrescente no intervalo considerado e y k +1 = seny k ou
π  π
cos x k , mas senx > cos y ⇔ senx > sen − y  ⇔ x + y > ⇒
2  2
π
se x k + y k > , temos x k +1 = senx k e y k +1 = cos x k . Analogamente, se
2
π
x k + y k < , temos x k +1 = cos y k e y k +1 = seny k ⇒ x k2 + y k2 = 1, ∀k ≥ 1. mas
2
π
( xk + y k ) 2 = ( xk2 + y k2 + 2xk y k ) ≤ 2( xk2 + y k2 ) = 2 ⇒ xk + y k ≤ 2 < ⇒
2
xk +1 = cos y k e y k +1 = senyk . Assim, temos


x 2001 = cos y 2000 = cos seny1999 = cos sensen...seny1 = cos sensen...sen cos 1 já que
1999 vezes

π π
cos 1 < sen 1, pois <1< .
4 2

PROBLEMA 5: SOLUÇÃO DE EINSTEIN DO NASCIMENTO JUNIOR (FORTALEZA - CE)

Lema 1: Sabemos que um quadrilátero é inscritível se e somente se as mediatrizes


dos lados desse quadrilátero são concorrentes.
Lema 2: Os pontos médios dos lados de um quadrilátero qualquer, convexo ou não,
formam um paralelogramo.

Tome agora um quadrilátero convexo ABCD, com lados opostos não paralelos.

C Sejam M, N, P, Q os pontos médios de AB,


B N
BC, CD, DA.
Trace agora as mediatrizes de BC e AD, que
O'1
se encontram em O1.
M P Chame NQ ∩ MP = E.
O1
Provaremos que as alturas relativas a BC e
a AD se encontram no ponto simétrico a O1
A Q D em relação a E.
Chame O'1 o simétrico de O1 em relação a
E.

EUREKA! N°13, 2002

37
Sociedade Brasileira de Matemática

Pelo Lema 2, E é ponto médio de NQ e E é ponto médio de MP e além disso por


definição de O'1, E é ponto médio de O'1O1.
Então temos que NO1QO'1 é um paralelogramo!
Daí: NO'1 // O1Q ⇒ NO'1 ⊥ AD ⇒ NO'1 é a altura em relação a AD.
QO'1 // NO1 ⇒ QO'1 ⊥ BC ⇒ QO'1 é a altura em relação a BC.
Logo O'1 é o encontro das alturas relativas a BC e AD.
Fazendo o mesmo para os lados AB e CD podemos concluir que:
O simétrico em relação a E do encontro de mediatrizes de lados opostos é igual à
interseção das alturas destes lados opostos.
Chame O2 o encontro das mediatrizes de AB e CD.
O'2 será o simétrico em relação a E de O2 e consequentemente o encontro das alturas
relativas a AB e CD.
Note que: O'1 ≡ O'2 ⇔ O1 ≡ O2.
Então: O'1 ≡ O'2 ⇔ O1 ≡ O2 ⇔ ABCD é inscritível.
Segue que O'1 ≡ O'2 ⇔ ABCD é inscritível.
Logo as 4 alturas tem um ponto em comum se e somente se o quadrado for
inscritível.

PROBLEMA 6: SOLUÇÃO DE HUMBERTO SILVA NAVES (SÃO PAULO - SP)

Vamos introduzir o conceito de energia para as pedras:


pos ( x )
3
E= ∑
 
x 4
, onde pos(x) é a posição de x.

O que acontece se realizarmos um movimento? Vamos mostrar que a energia sempre


diminui a cada momento:
Movimento tipo A:


p–1 p p+1 p–1 p p+1
p p +1 p −1 p
 3 3  3  3 5
E' = E −   −   +  = E ⋅  −   < E
 4 4  4  4 12 
Movimento tipo B:


p–1 p p+1 p+2 p–1 p p+1 p+2

EUREKA! N°13, 2002

38
Sociedade Brasileira de Matemática

p p −1 p+2 p
3 3 3  3  5 
E ' = E − 2  +   +   =E −  ⋅ < E
4 4 4  4   48 
3
Obs. O número não foi escolhido ao acaso, foi escolhido um número q tal que:
4
1 < q + q 2 e 1 + q 3 < 2q
Considere um copo de posição p, onde p é tal que
p
3
  > E 0 , onde E0 é a energia inicial do sistema.
4
Como a energia, a cada movimento, sempre diminui, qualquer que sejam os
movimentos que se faça, nenhuma pedra ficará numa posição menor ou igual a p. Ou
seja, existe uma "barreira" à esquerda para as pedras.
Estamos agora capazes de resolver o problema (a primeira parte):
* Dada uma configuração inicial das "n" pedras, é impossível realizar uma seqüência
infinita de movimentos.
Demonstração: Vamos demonstrar (*) por indução:
Base: Para n = 1 é verdadeiro!
Passo indutivo: Suponha, por absurdo, que seja possível realizar uma seqüência
infinita de movimentos. Sabemos que a posição da pedra mais à esquerda não
aumenta a cada movimento e como existe uma barreira à esquerda, então a partir de
um certo ponto a pedra mais a esquerda não mais será movimentada, e só com os
restantes (o número de pedras restantes é no máximo n – 1) é impossível realizar
uma seqüência infinita de movimentos, o que é um absurdo!
Logo por indução, (*) é verdadeiro para todo n ∈ 1*.
Vamos resolver a segunda parte do problema:
Sabemos que dada uma configuração inicial, independente das escolhas dos
movimentos sempre chegamos a uma configuração onde é impossível mover
(configuração parada). Suponha por absurdo que a partir de uma configuração inicial
se chegue a duas configurações paradas distintas A e B.
Seja k' a posição da pedra mais à direita das configurações A e B e k = k' + 2.
Considere o seguinte invariante: (não varia a cada movimento)
I= ∑
Fk − pos ( x ) , onde Fn é o n-ésimo número de Fibonacci.
x
Lembramos que F1 = 1, F2 = 1 e Fn + 2 = Fn + 1 + Fn, para todo n ≥ 1)
Sabemos que I A = I B , pois I é invariante, isto é, permanece o mesmo depois de cada
movimento.
De fato, Fk – (p – 1) = Fk – p + 1 = Fk – p + Fk – p – 1 = Fk – p + Fk – (p + 1), donde I não muda
após um movimento do tipo A, e Fk − ( p −1) + Fk − ( p + 2) = Fk − p + Fk − p −1 + Fk − p − 2 =

EUREKA! N°13, 2002

39
Sociedade Brasileira de Matemática

= 2 Fk − p , donde I não muda após um movimento do tipo B.


Algoritmo:
1- Seja x a pedra mais à esquerda de A e y a pedra mais à esquerda de B.
Devemos ter pos(x) = pos(y), pois se fosse pos(x) ≠ pos(y) (assumimos sem perda de
generalidade que pos(x) > pos(y)), teríamos:
IA = ∑
Fk − pos (t ) ≤ Fk − pos ( x ) + Fk − pos ( x )− 2 + Fk − pos ( x ) − 4 + ... + F2
t∈ A
se k − pos (x) for par e
IA = ∑F
t∈ A
k − pos ( t ) ≤ Fk − pos ( x ) + Fk − pos ( x ) − 2 + ... + F3 caso contrário:

Mas F2 + ... + F2 k = F2 k +1 − 1 < F2 k +1 e F3 + ... + F2 k +1 = F2 k + 2 − 1 < F2 k + 2 (como se


prova facilmente por indução).
logo I A ≤ Fk − pos ( x ) +1 − 1 < Fk − pos ( x ) +1 ≤ Fk − pos ( y ) ≤ I B , um absurdo!
2- Seja A: = A – {x} e B: = {y}.
3- Vá para o 1.
Pronto! Demonstramos que A e B são a mesma configuração, o que é um absurdo! A
configuração final independe da escolha dos movimentos.

EUREKA! N°13, 2002

40
Sociedade Brasileira de Matemática

XXIII OLIMPÍADA BRASILEIRA DE MATEMÁTICA


Problemas e Soluções da Primeira Fase – Nível Universitário
PROBLEMA 1
Seja f ( x) = e − x sen x. Calcule f (2001)(0). (Denotamos por f (n)(x) a derivada de ordem
n no ponto x; assim, f (2)(x) = f '' (x).)
PROBLEMA 2
Seja s(n) a soma dos algarismos de n. Assim, por exemplo, s(77) = 14 e s(2001) = 3.
Diga se existe um inteiro positivo n com s(n) = 10 e s(n2) = 100. Se não existir,
demonstre este fato. Se existir, dê um exemplo.
PROBLEMA 3
O centro de massa de uma lata cilíndrica de refrigerante tem a mesma posição
quando a lata está vazia ou cheia. Se a massa da lata vazia é m e a massa do
refrigerante dentro da lata cheia é M, determine a fração de refrigerante que deve ser
deixado na lata para que seu centro de massa fique o mais baixo possível.
PROBLEMA 4
Um ratinho ocupa inicialmente a gaiola A e é treinado para mudar da gaiola
atravessando um túnel sempre que soa um alarme. Cada vez que soa o alarme o
ratinho escolhe qualquer um dos túneis incidentes a sua gaiola com igual
probabilidade e sem ser afetado por escolhas anteriores. Qual a probabilidade de que
após o alarme soar 23 vezes o ratinho ocupe a gaiola B?
A B C

D E F

PROBLEMA 5
Seja A uma matriz n × n com a1, j = ai, 1 = 1 (para quaisquer i e j, 1 ≤ i, j ≤ n) e
a i +1, j +1 = a i , j + a i +1, j + a i , j +1 (para quaisquer i e j, 1 ≤ i, j < n). Assim,
1 
1 1 1

1   . Calcule det(A).
3 5 7
A = 1

 
5 13 25
1 
7 25 63
   


PROBLEMA 6
xn
Seja xn uma seqüência de números reais definida por x n +1 = x n2 − , n ≥ 0.
2
Para quais valores de x0 a seqüência converge? Para que valor?

EUREKA! N°13, 2002

41
Sociedade Brasileira de Matemática

SOLUÇÕES – NÍVEL UNIVERSITÁRIO

SOLUÇÃO DO PROBLEMA 1
 π
f ' ( x) = −e − x ( senx − cos x) = − 2e − x sen x − 
 4
Em geral, se u = x + a, onde a é uma constante, a derivada de e − x sen u é igual
π
a − 2e − x sen u − .
 4
 π π  π
Logo f ' ' ( x) = (− 2 ) 2 e − x sen x − −  = 2e − x sen x − .
 4 4  2
 kπ 
Se f ( k ) ( x) = (− 2 ) k e − x sen x − , para k ∈ 1, teremos
 4 
  kπ π    (k + 1)π 
f ( k +1) = (− 2 ) k ⋅  − 2e − x sen x − −   = (− 2 ) k +1 e − x sen x − .
  4 4   4 
 nπ 
Logo, por indução, f ( n ) ( x) = (− 2 ) n e − x sen x −  (n ∈ 1) e
 4 
 2001π   π
f ( x) = (− 2 ) 2001 e − x sen x −
( 2001) 1000 − x
 = − 2 2 e sen x − 
 4   4
1000 − x
= 2 e (cos x − sen x).

SOLUÇÃO DO PROBLEMA 2
Existem muitos inteiros n com as propriedades pedidas. A menor solução é
1101111211. Algumas outras são 10111111111, 11011111111, 200220000202 e
9
n= ∑10
j =0
2j
.

A única condição necessária é que, ao calcular n2 pelo algoritmo usual não deve
ocorrer nenhum 'vai um'. Mais precisamente, se a expansão decimal de n é
n = p (10), p ( x) = a k x k + ... + a 0 com 0 ≤ a i < 10 então temos p(1) = s (n) = 10.
Temos q ( x) = ( p( x)) 2 = b2 k x 2 k + ... + b0 com
b j = a j a 0 + a j −1 a1 + ... + a 0 a j , q (10) = n 2 e q (1) = 100.
Para que s (n 2 ) = q (1) precisamos apenas que cada bj seja menor do que 10.
Exemplos de soluções podem ser facilmente obtidos tomando os algarismos de n
pequenos e espalhados.

EUREKA! N°13, 2002

42
Sociedade Brasileira de Matemática

SOLUÇÃO DO PROBLEMA 3
Se há fração λ de refrigerante na lata, λ ∈ [0,1] , a massa total de refrigerante será
λ
λM e o centro de massa do refrigerante (sem contar a lata) tem altura h , onde h é
2
h
a altura total da lata. Como o centro de massa da lata vazia tem altura , a altura do
2
 λh h
centro de massa é f (λ ) =  λM ⋅ + m ⋅  / (λM + m ).
 2 2
Temos
h
f ' (λ ) = ((λM + m) ⋅ 2λM − (λ 2 M + m) M )
2(λM + m) 2

h
= (λ 2 M 2 + 2λMm − mM ).
2(λM + m) 2

− Mm − M 2 m 2 + M 3 m
As raízes de λ 2 M 2 + 2λMm − mM são λ1 = <0 e
M2
− Mm + M 2 m 2 + M 3 m m 2 + Mm − m m
λ2 = 2
= = < 1. Assim,
M M m + m 2 + Mm
f ' (λ ) < 0 para 0 ≤ λ < λ 2 e f ' (λ ) > 0 para λ 2 < λ ≤ 1, e portanto f (λ ) é mínimo
m 2 + Mm − m
para λ = λ 2 = .
M

SOLUÇÃO DO PROBLEMA 4
Seja an a probabilidade de que após n apitos o ratinho esteja na coluna central (B ou
E). Temos a0 = 0 (o ratinho não começa na coluna central). Claramente após o apito
soar um número par de vezes o ratinho estará em A, C ou E e após um número ímpar
de vezes em B, D ou F. Assim, queremos calcular a23.
Se, antes de soar o alarme, o ratinho está na coluna central ele tem 1/3 de
probabilidade de permanecer lá (independentemente da gaiola onde o ratinho estava
ser B ou E). Por outro lado, se ele não está na coluna central ele tem probabilidade
1/2 de ir para lá (novamente independentemente da gaiola onde o ratinho começou).
Assim,
1 1 1 1 3 1 3
a n +1 = a n + (1 − a n ) = − a n ou a n +1 − = − (a n − ).
3 2 2 6 7 6 7

EUREKA! N°13, 2002

43
Sociedade Brasileira de Matemática

3 1
A seqüência bn = a n − é portanto uma progressão geométrica de razão − com
7 6
3 3 3 3 3(6 + 1)
23
b0 = − . Assim b23 = e a 23 = + = .
7 7⋅6 23
7 7⋅6 23
7 ⋅ 6 23

SOLUÇÃO DO PROBLEMA 5
Fazendo operações em linhas (subtraindo a primeira linha da segunda, a segunda da
terceira e assim por diante) temos
1 1 1 1  

0 2 4 6 
det(A) = det 0 2 8 18  
 
0 2 12 38 

      
Fazendo agora operações em colunas (subtraindo a primeira coluna da segunda, a
segunda da terceira e assim por diante) temos
1 0 0 0   
2 2 2 
 1 1 1
0
det( A) = det 02 6 10   = 2

1 3 5 .
 
n −1
det
 
2 10 26 
1 5 13

0

          



Esta última matriz é a versão (n – 1) × (n – 1) de A. De fato, sua entrada (i, j) é
1
((a i +1, j +1 − a i +1, j ) − (a i , j +1 − a i , j )) = a i , j .
2
Chamando o valor de det(A) para matrizes n × n de bn temos portanto
n ( n −1)
b1 = 1, bn +1 = 2 bn donde bn =
n
2 2 .

SOLUÇÃO DO PROBLEMA 6
x 3 3
Seja f ( x) = x 2 − . Temos f ( x) = x ⇔ x = 0 ou x = . Para x > temos
2 2 2
3 3
f ( x) > x. Assim, se x 0 > temos x n ≥ x 0 > para todo n e, por indução,
2 2
x n +1 = f ( x n ) > x n para todo n. Assim, nesse caso, (xn) é crescente, e portanto
lim n →∞ x n = L para algum L ∈ # ou lim n →∞ x n = +∞. No primeiro caso, teríamos

EUREKA! N°13, 2002

44
Sociedade Brasileira de Matemática

3
L = lim n →∞ x n +1 = lim n →∞ f ( x n ) = f (lim n →∞ x n ) = f ( L), mas L = lim x n ≥ x 0 > ,
2
3
donde f ( L) > L , absurdo. Assim, se x 0 > , lim n→∞ x n = +∞.
2
3
Se x 0 < −1, x1 = f ( x 0 ) > , donde também temos lim n →∞ x n = +∞.
2
< x < , com x ≠ 0, temos f (x ) < x . Por outro lado, para todo
1 3
Se −
2 2
1 1 1 1
n ≥ 1, x n = f ( x n −1 ) = ( x n −1 − ) 2 − ≥ − > − . Assim, se
4 16 16 2
3
− 1 < x 0 < , x n +1 = f ( x n ) ≤ x n para todo n ≥ 1. Portanto, existe
2
3
c = lim n →∞ x n < . Temos portanto
2
0 ≤ c = lim n→∞ x n +1 = lim n →∞ f ( x n ) ∈{ f (c) , f (−c) }. Temos que f (−c) = c, com
1 1 1
c > 0, implica c = , e, como x n ≥ − para todo n ≥ 1, se lim n →∞ x n = teríamos
2 16 2
1 1 1
lim n →∞ x n = , donde = lim n →∞ x n +1 = lim n→∞ f ( x n ) = f   = 0, absurdo. Como
2 2 2
3
f (c) < c se 0 < c < , temos necessariamente c = 0, o portanto lim n →∞ x n = 0.
2
3 3 3
Se x 0 = −1 ou x 0 = temos x n = para todo n ≥ 1, donde lim n →∞ x n = .
2 2 2
3  3
Assim, xn converge se − 1 < x 0 < , quando lim n →∞ x n = 0 e se x 0 ∈ − 1, ,
2  2
3
quando lim n →∞ x n = . Em qualquer outro caso, lim n →∞ x n = +∞.
2

EUREKA! N°13, 2002

45
Sociedade Brasileira de Matemática

XXIII OLIMPÍADA BRASILEIRA DE MATEMÁTICA


Problemas e Soluções da Segunda Fase – Nível Universitário
PROBLEMA 1:
São dados um ponto O e uma reta r no plano. Para cada ponto P de r, seja rp a reta
perpendicular a OP passando por P. Prove que o conjunto {r p P ∈ r} é o conjunto de
todas as retas tangentes a uma parábola.
PROBLEMA 2:
Seja ε um número real positivo arbitrário. Com centro em todos os pontos do plano
com coordenadas inteiras, traça-se um círculo de raio ε . Prove que toda reta
passando pela origem intercepta uma infinidade desses círculos.
PROBLEMA 3:
Definimos SL(2, =) como o conjunto das matrizes 2 × 2 com coeficientes inteiros e
determinante 1. Seja A ∈ SL(2, =) uma matriz tal que existe n > 0 inteiro com An = I.
Prove que existe X ∈ SL(2, =) tal que X–1AX é igual a uma das matrizes:
1 0   0 − 1  0 − 1 0 − 1
±   ; ±   ; ±   ; ±   .
 0 1 1 1  1 0  1 − 1
PROBLEMA 4:
Caminhando sobre os segmentos unitários da figura abaixo, determine quantos
percursos distintos existem de A até B sem passar duas vezes por um mesmo ponto.

A B
1
PROBLEMA 5:
π


Para todo real u, seja I (u ) = ln (1 − 2u cos x + u 2 )dx.
0

b) Calcule I(u) para todo u ∈ 5.


1
a) Prove que I (u ) = I (−u ) = I (u 2 ).
2
PROBLEMA 6:
Seja D o conjunto dos pontos p em 52 com p ≤ 1. Seja f : D → D uma função
sobrejetora satisfazendo f ( p ) − f (q ) ≤ p − q para quaisquer p, q ∈ D. Prove que f
é uma isometria, isto é, que f ( p ) − f (q ) = p − q para quaisquer p, q ∈ D.
(Observação: ( x, y ) denota x 2 + y 2 .)

EUREKA! N°13, 2002

46
Sociedade Brasileira de Matemática

SOLUÇÕES – NÍVEL UNIVERSITÁRIO


PROBLEMA 1: SOLUÇÃO DE EMANUEL DE SOUZA CARNEIRO (FORTALEZA - CE)
Suponha, sem perda de generalidade, que o ponto O seja (0,1) e a reta r seja o eixo x.
Seja P um ponto sobre r, P(p, 0), logo:
(x, y) ∈ rp ⇔ (x – p, y) ⋅ (p, – 1) = 0 ⇔ xp – p2 – y = 0 (Equação da reta rp).
Buscamos agora uma parábola do tipo y = ax2 + bx + c, de modo que estas retas rp
sejam tangentes a ela.
Derivando, obtemos a equação da reta tangente a essa parábola no ponto (x0, y0).
y − y0
= 2ax 0 + b
x − x0
ou seja: y − (ax 02 + bx 0 + c) = ( x − x 0 )(2ax0 + b)
⇔ x(2ax 0 + b) − ax 02 + c − y = 0
tome a = 1/4, b = c = 0, daí teremos: x 1 x0  − 1 x 02 − y = 0.
2  4
Esta será uma idéia: em {rp | P ∈ r} basta tomar p = x0 /2.
1 2
Reciprocamente, cada rp ∈ {rp | P ∈ r} é tangente à parábola y = x no ponto
4
(2p, p2), pois a equação da reta tangente a essa parábola nesse ponto é:
y − p2
= f ' (2 p ) = p ⇔ xp − p 2 − y = 0 , que é a equação da reta rp.
x − 2p
2
Logo o conjunto{rp | P ∈ r} é o conjunto das retas tangentes à parábola y = x .
4

PROBLEMA 2: SOLUÇÃO DE EMANUEL DE SOUZA CARNEIRO (FORTALEZA - CE)


Para provar a questão, mostraremos uma lema equivalente ao lema de Kronecker.

Lema: Seja δ um número irracional. Dado ε > 0 arbitrário, existe a ∈ =* tal que
{aδ } < ε , onde {x}= x – [x] indica a parte fracionária de x.
Prova: Seja n ∈ = tal que nε < 1 ≤ (n + 1)ε . Divida o intervalo [0, 1) em (n + 1)
intervalos do tipo [iε , (i + 1)ε ), 0 ≤ i ≤ n; o último intervalo é [nε, 1).

0 ε 2ε nε 1 (n + 1) ε

Observe agora os números δ, 2δ, 3δ,…;(n + 2)δ . Como a função parte fracionária
vai dos reais em [0,1), pelo princípio da casa dos pombos, dois dentre os (n + 2)
números {δ }, {2δ }, {3δ },…,{(n +2) δ }, cairão num mesmo intervalo [iε, (i + 1)ε).

EUREKA! N°13, 2002

47
Sociedade Brasileira de Matemática

Sejam j, ) os dois números tais que:


{jδ }, { ) δ } ∈ [i ε, (i + 1) ε).
Suponha, sem perda de generalidade, que {jδ} ≥ { ) δ}
Assim:
jδ = [ jδ ] + { jδ }
A A A
δ = [ δ ]+ { δ }
A 
A A
⇒ ( j − )δ = [ jδ ] − [ δ ] + ({ jδ } − { δ })

Logo {( j − A)δ } = { jδ } − {Aδ } < (i + 1)ε − iε = ε e j − A é o inteiro procurado


inteiro

Obs: Não podemos ter {kδ} = 0, k ∈ =*, pois isso implicaria kδ = n ∈ = ⇒ δ = n/k
∈ 4, absurdo!
Vamos agora resolver o problema.
Seja dado ε > 0. Uma reta passando pela origem tem a forma y = mx

Primeiro caso: Se m ∈ 4, nesse caso a reta passa por infinitos "Lattice points" (pontos
de coordenadas inteiras), pois se m = p/q os pontos da forma (x, y) = (kq, kq . p/q) =
(kq, kp), k = 1, 2, 3… estão todos na reta, então ela interceptará todos os círculos que
têm centros nesses pontos.

Segundo caso: Se m for irracional.


Os pontos da reta são da forma (x, mx), logo, pela observação feita após o lema, esta
reta não contém nenhum ponto de coordenadas inteiras.

Suponha que ela intersecte somente uma quantidade finita de círculos, digamos C1,
C2,…, Cn de centros P1, P2, …, Pn
Observe que para qualquer outro ponto no "Lattice" P (fora os centros) teremos:
d(P, r) > ε.
Seja d a menor das distâncias, d1(P1, r), d2(P2, r),…, dn(Pn , r)
onde di(Pi, r) é a distância do centro Pi à reta r.
(Note que di (Pi, r) > 0 ⇒ d > 0).
Logo teríamos dentro todos os pontos de coordenadas inteiras no plano, um deles Pi
que minimizaria a distância à reta r, com d(Pi, r) = d > 0.
Isso contradiz o lema, vejamos:
Pelo lema existe um inteiro a, de modo que {am} < d/2.
Seja b = [am] , o ponto Q = (a, am) ∈ r e sua distância a (a, b) é menor ou igual que
a distância de (a, b) a r.

EUREKA! N°13, 2002

48
Sociedade Brasileira de Matemática

r Logo d' ≤ d((a, b), (a; am)) = |am – b| =


y {am} < d/2.
(a, am) Absurdo, pois d era a distância mínima ⇒
(a, b) a reta corta uma infinidade de círculos.
d'

PROBLEMA 3: SOLUÇÃO DE CARLOS YUZO SHINE (SÃO PAULO – SP)


a b 
Seja A =   uma matriz de SL(2,=) tal que An = I para algum inteiro positivo n.
 c d 
Seja m o menor inteiro positivo tal que Am = I. Sejam λ1 e λ2 os autovalores de A, ou
seja, as raízes da equação det(A – xI) = 0 ⇔ x2 – (a + d)x + 1 = 0. (I)
Se λ1 = λ2, temos que o discriminante da equação (I) é nulo, logo a + d = 2 ou
a + d = –2.
Se a + d = 2, temos
a b   a 2 + bc b(a + d )   2a − 1 2b 
A =   ⇒ A 2 =   =   = 2A − I
c d 

 c(a + d ) d + bc   2c
2
2d − 1
Por indução, pode-se mostrar que Ak = kA – (k – 1)I.
Desta forma, Am = I ⇔ mA – (m – 1)I = I ⇔ A = I.
Se a + d = –2, temos, analogamente, que Ak = (–1)k + 1(kA + (k – 1)I) e portanto Am = I
⇔ (–1)m + 1(mA + (m – 1)I) = I ⇔ A = –I e m é par (na verdade, m = 2).
Se λ1 ≠ λ2, então a matriz A é diagonalizável em &, ou seja, existe uma matriz P tal
 λ1 0 
que A = PDP–1, sendo D =   . Neste caso, temos que Ak = PDkP–1, com
 0 λ 2

 λk 0 
D k =  1 .
k 
 0 λ2 
Assim, Am = I ⇔ PDmP–1 = I ⇔ Dm = I ⇔ λ1m = λ2m = 1, ou seja, λ1 e λ2 são raízes
m-ésimas da unidade. Assim, como a + d = λ1 + λ2, –1 ≤ a + d ≤ 1. Como a + d é
inteiro, temos os seguintes casos:

(i) a + d = –1. Neste caso, (I) ⇔ x2 + x + 1 = 0 e portanto λ1 e λ2 são raízes


cúbicas da unidade. Portanto m = 3.
(ii) a + d = 0. Neste caso, (I) ⇔ x2 + 1 = 0 e portanto λ1 e λ2 são raízes quartas
da unidade. Portanto m = 4.

EUREKA! N°13, 2002

49
Sociedade Brasileira de Matemática

(iii) a + d = 1. Neste caso, (I) ⇔ x2 – x + 1 = 0 e portanto λ1 e λ2 são raízes sextas


da unidade. Portanto m = 6.
Observando as matrizes dadas, temos
3 4 6
 0 − 1  0 − 1  0 − 1
  = I ;   = I ;   = I
 1 − 1 1 0  1 1 
Assim, basta provarmos que existem inteiros x, y, z e w com xw – yz = 1 e
 x y   w − y
A = ± W   ,
 z w  − z x 

sendo W uma das três matrizes acima. Note que


 w − y  x y  x y
   = det  ⋅ I
 − z x  z w   z w

 0 − 1  yw + xz + yz − x 2 − y 2 − xy 
Caso (i) Tomamos W =   . Assim A = ± 2 .

 1 − 1  z + w + zw − yw − xz − xw 
2

Veja que a + d = – 1 é equivalente a xw – yz = 1. A condição det A = 1 é equivalente


a ad – bc = 1 ⇔ a(–1 – a) – bc = 1 ⇔ bc = –(a2 + a + 1). Assim, devemos ter
a = ±( yw + xz + yz )
b = ± (− x 2 − y 2 − xy ) (II)
c = ± ( z 2 + w 2 + zw)

Isso pode ser verificado fatorando a2 + a + 1 = (a – ω)(a – ω ) em inteiros de


Eisenstein. Considere um fator x – yω de a – ω e seja z – wω o fator de a – ω tal que
(x – yω)(z – wω) = a – ω. Note que (x – y ω )(z – w ω ) = a – ω . As normas dos
fatores são x2 + y2 + xy e z2 + w2 + zw e portanto sempre existem x, y, z e w tais que
|b| = x2 + y2 + xy e |c| = z2 + w2 + zw. Substituindo o valor de a de (II) na equação
original verifica-se que de fato bc = – (a2 + a + 1). Reciprocamente, a outra raiz
desta última equação é –1 – a = d = –xz – yw – xw. Mas para resolver isso basta
trocar x e y por z e w, respectivamente, e trocar o sinal.

Obs. Usamos implicitamente a existência e unicidade da fatoração para inteiros de


Eisenstein para obter x, y, z, w como acima.

EUREKA! N°13, 2002

50
Sociedade Brasileira de Matemática

 0 − 1  yw + xz − x 2 − y 2 
Caso (ii) Agora tomamos W =   , com A = ± 2 .

1 0   z + w 2
− yw − xz 
Novamente, é óbvio que a condição a + d = 0 é satisfeita. A condição det A = 1 é
equivalente a ad – bc = 1 ⇔ bc = –(a2 + 1). O sistema correspondente agora é
a = ±( yw + xz )
b = ±(− x 2 − y 2 ) (III)
c = ±( z 2 + w 2 )
Agora usamos inteiros de Gauss! Considere a fatoração a2 + 1 = (a + i)(a – i) e sejam
x + yi e z + wi fatores de a + i com (x + yi)(z + wi) = a + i. Note que (x – yi)(z – wi) =
a – i. Então existem sempre x, y, z e w tais que |b| = x2 + y2 e |c| = z2 + w2.
Novamente, substituindo o valor de a de (III) na equação original vemos que de fato
bc = – (a2 + 1). A outra raiz é oposta ao valor de a de (III), mas é só trocar o sinal.

Obs. Aqui usamos implicitamente a existência e unicidade de fatoração para inteiros


de Gauss para obter x, y, z, w como acima.

 0 − 1
Caso (iii) Tomamos dessa vez W =   . Temos nesse caso
1 1 
 yw + xz − yz − x 2 − y 2 + xy 
A = ± 2  . Mas esse caso é análogo ao caso (i)!

 z + w − zw − yw − xz + xw 
2

Tome y = –y’ e w = –w’ e obtemos a mesma matriz do caso (i).

Logo existe sempre uma matriz X tal que A = XWX–1, onde W é uma das matrizes
dadas no enunciado.

PROBLEMA 4: SOLUÇÃO DE JULIANA ABRANTES FREIRE (RIO DE JANEIRO – RJ)


Vamos numerar os pontos assim:
1 3 5 7 9

A (6) C (6)
0=A B = 10
2 4 6 8
B (6)

F(n) = O número de percursos que chegam em n, sem passar por n + 2.


A(n) = O número de percursos que chegam em n vindos de n – 1.
B(n) = O número de percursos que chegam em n vindos de n – 2.
C(n) = O número de percursos que chegam em n vindos de n + 1.
A(6), B(6) e C(6) estão ilustrados na figura acima.

EUREKA! N°13, 2002

51
Sociedade Brasileira de Matemática

Note que não há mais caminhos relevantes para o problema além destes: Qualquer
caminho indo a 6 vindo de 8, passou obrigatoriamente por 6 ou 7 ou ambos. Se
passou por 6, este caminho não pode porque passaria por 6 duas vezes. Mesmo se
não passou por 6, este caminho não pode: 7 e 8 já foram usados, então este caminho
não pode chegar até B.
Então F(n) = A(n) + B(n) + C(n).
B(n) = F (n – 2), porque nenhum caminho chegando a n – 2 passou por n, como no
exemplo descrito anteriormente. Então
B(n) = A(n – 2) + B(n – 2) + C(n – 2).
A(n) = B(n – 1) + A(n – 1):

B (n – 1) Porque todos os C(n – 1) passaram por n (mais que isto,


C (n – 1)
eles são os caminhos que vieram para n – 1 diretamente
A (n – 1) A(n) de n) e nenhum A(n – 1) ou B(n – 1) passou por n porque
eles não teriam como passar por n e voltar para n – 2 e
n
n – 3, respectivamente, sem repetir pontos e mantendo a possibilidade do caminho
chegar a B.
C(n) = B (n – 1) + A(n – 1), porque estes são os caminhos que chegam a n + 1 sem
passar por n: eles vão direto de n – 1 para n + 1.
Para chegar ao ponto 1:
B (1) 1 3 B(n) = 0, não existe caminho.
A(1) = 1 (caminho direto )
A (1) C(1) C(1) = 1 (caminho ).
0 2

Para chegar ao ponto 2:


1 3
Os caminhos possíveis são:

A (2) C(2)

0 B(2) 2 4
Ou seja A(2) = B(2) = C(2) = 1.
A(3) = A(2) + B(2) = 2 A(5) = A(4) + B(4) = 7
B(3) = A(1) + B(1) + C(1) = 2 B(5) = A(3) + B(3) + C(3) = 6
C(3) = A(2) + B(2) = 2 C(5) = A(4) + B(4) = 7

A(4) = A(3) + B(3) = 4 A(6) = A(5) + B(5) = 13


B(4) = A(2) + B(2) + C(2) = 3 B(6) = A(4) + B(4) + C(4) = 11
C(4) = A(3) + B(3) = 4 C(6) = A(5) + B(5) = 13

EUREKA! N°13, 2002

52
Sociedade Brasileira de Matemática

A(7) = A(6) + B(6) = 24 A(8) = A(7) + B(7) = 44


B(7) = A(5) + B(5) + C(5) = 20 B(8) = A(6) + B(6) + C(6) = 37
C(7) = A(6) + B(6) = 24 C(8) = A(7) + B(7) = 44

A(9) = A(8) + B(8) = 81


B(9) = A(7) + B(7) + C(7) = 68
C(9) = A(8) + B(8) = 81

C(10) = 0 porque não existe ponto 11.


Mas ainda vale
A(10) = A(9) + B(9) = 149
B(10) = A(8) + B(8) + C(8) = 125
F(10) = A(10) + B(10) + C(10) = 149 + 125 + 0 = 274.
PROBLEMA 5: SOLUÇÃO DE MARCIO ASSAD COHEN (RIO DE JANEIRO – RJ)
π
I (u ) = ∫ 0
ln(1 − 2u cos x + u 2 )dx
a) fazendo t = π − x ; dt = – dx:
0 π π

I (u) = − ln(1− 2u cos(π − t) + u2 )dt = ∫ ln(1+ 2ucos t + u )dt = ∫ ln(1− 2(−u)cosx + (−u) )dx =
2 2
π 0 0
I(– u). Logo, I(u) = I(–u).
Note agora que, como cosx = 2cos2x/2 –1:
1 − 2u 2 cos x + u 4 = 1 + 2u 2 + u 4 − 4u 2 cos 2 x / 2 = (1 + u 2 ) 2 − 4u 2 cos 2 x / 2 =
(1 + 2u cos x / 2 + u 2 )(1 − 2u cos x / 2 + u 2 )
π π

Logo, I (u 2 ) = ln (1 + 2u cos x / 2 + u 2 )dx +
0 ∫0
ln(1 − 2u cos x / 2 + u 2 ) dx
Fazendo t = x/2 nessas últimas integrais vem:
I (u 2 ) = 2  ln(1 − 2u cos t + u 2 )dt .
π /2 π /2

 0 ∫
ln(1 + 2u cos t + u 2 )dt +
0 ∫ 
(II)
π 0
Agora, note que ∫
π /2
ln(1 + 2u cos t + u 2 )dt =
x = t-Œ ∫-π/2
ln(1 − 2u cos x + u 2 )dx =
0 π /2
= −
t =− x ∫
π /2
ln(1 − 2u cos x + u 2 )dx = ∫0
ln(1 − 2u cos x + u 2 )dx (III)
De (II) e (III) vem
 π /2 π 
I (u 2 ) = 2 
 0 ∫
ln(1 + 2u cos x + u 2 )dx + ∫ π /2
ln(1 + 2u cos x + u 2 )dx  = 2 I (u )

1
Logo, I(u) = I(u2).
2

EUREKA! N°13, 2002

53
Sociedade Brasileira de Matemática

1 k
b) Fazendo u = 1, vem I(1) = 2I(1) ∴ I(1) = 0. Em geral, I(u) = k
⋅ I (u 2 ), ∀x :
2
1 1 1 1 1
I(u) = I (u 2 ) = ⋅ I (u 4 ) = ⋅ I (u 8 )... (é uma indução simples).
2 2 2 4 2
Supondo 0 ≤ u ≤ 1 inicialmente:
π π
cos x ≤ 1 ⇒ I (u ) = ∫0
ln(1 − 2u cos x + u 2 )dx ≤ ∫
0
ln(1 + u ) 2 dx = 2π ln(1 + u )
k
2π ln(1 + u 2 ) 1 k 1 k
Logo, I (u ) ≤ k k
I (u 2 ) ≤ k ⋅ 2π ln(1 + u 2 ) .
, ∀x pois
2 2 2
 2π ln(1 + u t ) 

Fazendo k → ∞ e substituindo t = 2 , deve-se ter I (u ) ≤  lim
k
 = 0.
 t →∞ t
 
Logo, I(u) = 0 se u ∈ [ –1, 1] (lembrando já que I(u) = I(– u)).
1
Se |u| > 1, faço a substituição u = : Então ( ω < 1) :
ω
 1  π  2 cos x 1  π  ω 2 − 2ω cos x + 1 
ω  0 

I   = ln1 −
ω
+ 2 dx = ln
ω  0

∫ ω2
dx =


1 π π
∫ ∫
I   = ln(1 − 2ω cos x + ω 2 )dx − ln(ω 2 )dx = I (ω ) − 2π ln ω = −2π ln ω .
ω  0 0

1
Logo, se u > 1, temos: I(u) = − 2π ln ∴ I (u ) = 2π ln u
u
zero, se u ≤ 1
Concluindo: I (u ) = 
2π ln u , se u > 1.

PROBLEMA 6: SOLUÇÃO DE BRUNO FERNANDES CERQUEIRA LEITE (SÃO PAULO – SP)


Fato 1: Seja 0 = (0, 0). Então f(0) = 0.
Prova: ∀x ∈ D , temos x − 0 ≤ 1. Logo devemos ter f ( x) − f (0) ≤ 1, ∀x ∈ D! Como
a função é sobrejetora, ∀y ∈ D ∃x 0 ∈ D com f ( x 0 ) = y. Logo, ∀y ∈ D ,
y − f (0) ≤ 1, ou seja, f(0) dista no máximo 1 de qualquer ponto do disco D. Logo
f(0) = 0.
{ } {
Fato 2: Sejam Br = ( x, y ) x + y < r , Br = ( x, y ) x + y ≤ r . Então, se p ∈ Br,
2 2 2 2
}
f ( p ) ∈ B r . Se p ∈ Br , f ( p ) ∈ Br .

EUREKA! N°13, 2002

54
Sociedade Brasileira de Matemática

[ ]
Prova: p ∈ Br ∈ Br ⇔ p − 0 < r [≤ r ]. Mas f ( p) − f (0) = f ( p) − 0 ≤ p − 0 < r[≤ r ].
Logo f ( p ) ∈ B r (∈ B r ).
Fato 3: Se f(p) está no bordo de D ( | f (p) – 0| = 1) então p está no bordo de D.
Prova: f(p) e – f(p) são diametralmente opostos, logo f ( p) − (− f ( p )) = 2. Se p não
estivesse no bordo de D, ∀q ∈ D, p − q < 2, absurdo pois deveríamos ter
f ( p ) − f (q ) ≤ p − q , ∀p, q ∈ D.
Fato 4: Se f(p) e f(q) são diametralmente opostos, então p e q também são.
Prova: Se p e q não fossem diametralmente opostos, p − q < 2 = f ( p) − f (q ) ,
absurdo.
Fato 5: Se p e q são opostos tais que f(p) e f(q) são diametralmente opostos, e se x está
entre p e q então f(x) está entre f(p) e f(q). Além disso, nesse caso
f ( x) f ( q) = xq, f ( x) f ( p ) = xp e f ( x) f (0) = x0
Se A, B, C são pontos distintos, dizemos que B está entre A e C (e denotamos
A → B → C) se AB + BC = AC ) ( onde AB = A − B ).
Prova: Sabemos, pelo fato 4, que p e q são diametralmente opostos. Se x = 0, f(x) = 0
e o lema fica trivial. Suponhamos, sem perda de generalidade, x ≠ 0 e p → x → 0.
Então px + x0 = p 0. Como f ( p ) f ( x) ≤ px e f ( x) f (0) ≤ x0,
f ( p ) f ( x) + f ( x) f (0) ≤ px + x0 = 1 , por outro lado, pela desigualdade triangular,
f ( p ) f ( x) + f ( x) f (0) ≥ f ( p ) f (0) = 1. Logo f(x) está entre f(p) e 0 e portanto está
entre f(p) e f(q), e temos f ( p ) f ( x) = px, f (0) f ( x) = 0 x e f (q) f ( x) = qx.
Fato 6: Se p está no bordo de D ( |p – 0| = 1) então f(p) está no bordo de D (o bordo
de D é B̂ , só para facilitar a notação). Além disso, a restrição de f a B̂ (que tem
imagem B̂ ) é uma composição de rotação com espelhamento.
Prova: Seja p ∈ D tal que f(p) = (1, 0). Pelo fato 3, p ∈ B̂ . A imagem inversa de
(–1, 0), pelo fato 4, é – p, isto é, f(– p) = (–1, 0). (– p ∈ B ). Sejam q e – q as imagens
inversas de (0, 1) e (– 1, 0).
Então q e – q ∈ B̂ e é bem fácil ver que p, q, – p e – q formam um quadrado (ou não
teríamos f (m) − f (n) ≤ m − n , ∀m, n ∈ D).
~
Seja f : D → D a composição de rotação com espelhamento que coincide com f nos
~
pontos p, q, – p e – q. Vamos mostrar que f ( x) = f ( x), ∀x ∈ Dˆ .
~ ~
Note que f é uma bijeção, e que f −1 : Dˆ → Dˆ também é uma composição de
rotação com espelhamento.

EUREKA! N°13, 2002

55
Sociedade Brasileira de Matemática

Sejam m, – m ∈ D̂ . Existem n, – n ∈ B̂ tais que f(n) = m e f(– n) = – m. Devemos


ter m − (1,0) = f (n) − f ( p) ≤ n − p , m − (−1,0) = f (n) − f (− p ) ≤ n + p . Isso já
~ ~
implica que n = f −1 (m) ou n = − f −1 (m) , pois {x ∈ Dˆ | x − p | ≥ | m − (1,0) | e
~ ~ ~ ~
| x + p | ≥ | m − (−1,0) |} = { f −1 (m),− f −1 (m)} (de fato | f (m) − p | = | m − f ( p) | = | m − (1,0) |
~ ~
e | f −1 (m) + p | = | m − f (− p) = | m − (−1,0) | . Como, além disso,
| m − (0,1) | = | f (n) − f (q ) | ≤ | n − q | e | m − (0,−1) | = | f (n) − f (− q) | ≤ | n + q | , sobra
~ ~
apenas a possibilidade n = f −1 (m). Como f −1 é sobrejetiva, dado x ∈ Dˆ existe
~ ~
m ∈ Dˆ tal que x = f −1 (m) , e portanto teremos f ( x) = m = f ( x).
Agora estamos em condições de terminar a prova: de fato segue dos fatos 5 e 6 que
~
f = f e logo é uma composição de rotação com espelhamento, e portanto preserva
~ ~
distâncias. Com efeito, f = f em D , e f leva diâmetros pq com q = − p ∈ Dˆ em
~ ~ ~ ~ ~
diâmetros f ( p ) f (q) (e f ( p ), f (q) ∈ Dˆ ), e tanto f quanto f restritos ao diâmetro
pq preservam as distâncias aos extremos, e portanto preservam distâncias, logo
coincidem.

EUREKA! N°13, 2002

56
Sociedade Brasileira de Matemática

XXIII OLIMPÍADA BRASILEIRA DE MATEMÁTICA


Resultado – Nível 1 (5a. e 6a. Séries)
NOME CIDADE – ESTADO PRÊMIO
Eduardo Fischer Encantado – RS Ouro
Raphael Rodrigues Mata Salvador – BA Ouro
Guilherme R. Nogueira de Souza São Paulo – SP Ouro
André Linhares Rodrigues Fortaleza – CE Ouro
André Martins Costa Aranha Rio de Janeiro – RJ Ouro
Luiz Müller Vitória – ES Ouro
Rafael Bandeira Lages Teresina – PI Prata
Felipe Gonçalves Assis Campina Grande – PB Prata
Renato Rebouças de Medeiros Fortaleza – CE Prata
Thaís Viveiro São Paulo – SP Prata
Adriano Jorge Braun Vieira Neto Fortaleza – CE Prata
Jaques Deivinson da Silva Castello Serra – ES Prata
Enzo Haruo Hiraoka Moriyama São Paulo – SP Prata
Priscilla Yu Chen Kashiwakura São Paulo – SP Prata
Jefferson Quesado Neto Fortaleza – CE Prata
William Vasconcelos de Morais Fortaleza – CE Prata
Sophia Cherem Lopes Belo Horizonte – MG Bronze
Arthur Rodrigues de Oliveira Sobral S. J. dos Campos – SP Bronze
Pedro Paulo Gondim Cardoso Salvador – BA Bronze
Regina Reis da Costa Alves Rio de Janeiro – RJ Bronze
Weslen Costa Timóteo Paulista – PE Bronze
André Rodrigues Salerno Goiânia – GO Bronze
Caroline Goulart Campos Rio de Janeiro – RJ Bronze
Edson Augusto Bezerra Lopes Fortaleza – CE Bronze
Luiz Felipe Bruzzi Curi Belo Horizonte – MG Bronze
Bernardo de Oliveira Veiga Rio de Janeiro – RJ Bronze
Felipe Alves Tomé Fortaleza – CE Bronze
Luiza Cristina Maia e Silva Recife – PE Bronze
Igor Ribeiro Azevedo Belo Horizonte – MG Bronze
Mariana Nasser Brolezzi Santo André – SP Menção Honrosa
Paulo Alexandre Pavoni Curitiba – PR Menção Honrosa
Paulo André Carvalho de Melo Rio de Janeiro – RJ Menção Honrosa
Guilherme Pereira Barbosa Belo Horizonte – MG Menção Honrosa
Gustavo Sampaio Sousa Fortaleza – CE Menção Honrosa
Dennis G. de Macedo Bragagnolo Curitiba – PR Menção Honrosa
Pedro Nogueira Machado Rio de Janeiro – RJ Menção Honrosa
Yuriy Thallickson Bincovski Curitiba – PR Menção Honrosa
Gil Henriques Vassouras – RJ Menção Honrosa
Cássio Kendi Takamori S. J. dos Campos – SP Menção Honrosa
Iuri Lima Ribeiro Fortaleza – CE Menção Honrosa
Franco Veronez Ribeiro Vitória – ES Menção Honrosa
Mateos Kruchelski Tschá Curitiba – PR Menção Honrosa
Kleber Varela dos Santos Jaboatão dos Guararapes – PE Menção Honrosa
Núbia Martins Domingues Belo Horizonte – MG Menção Honrosa
Eduardo Tadafumi Sato Mogi das Cruzes – SP Menção Honrosa
Marco Aurélio Buono Carone Belo Horizonte – MG Menção Honrosa
Thalles Melo de Oliveira Lopes Goiânia – GO Menção Honrosa
Michel Ricardo Nigri Rio de Janeiro – RJ Menção Honrosa
Raquel Pereira Martins Rio de Janeiro – RJ Menção Honrosa

EUREKA! N°13, 2002

57
Sociedade Brasileira de Matemática

Resultado – Nível 2 (7a. e 8a. Séries)


NOME CIDADE – ESTADO PRÊMIO
Thiago Costa Leite Santos São Paulo – SP Ouro
Henry Wei Cheng Hsu São Paulo – SP Ouro
Rafael Daigo Hirama Campinas – SP Ouro
Rodrigo Aguiar Pinheiro Fortaleza – CE Ouro
Marcus Edson Barreto Brito Fortaleza – CE Prata
Daniela Satie Kondo São Paulo – SP Prata
Telmo Luis Correa Junior Santo André – SP Prata
Alan Hideki Uchida São Paulo – SP Prata
Felipe Rodrigues Nogueira de Souza São Paulo – SP Prata
Diogo dos Santos Suyama Belo Horizonte – MG Prata
Ricardo Mizoguchi Gorgoll São Paulo – SP Prata
Karoline Matias Morais Fortaleza – CE Prata
Paulo Roberto Sampaio Santiago Salvador – BA Prata
Marcela Sobrinho Pereira Fortaleza – CE Prata
Thomás Yoiti Sasaki Hoshina Rio de Janeiro – RJ Prata
Guilherme Rodrigues Salerno Goiânia – GO Bronze
Hector Kenzo Horiuti Kitahara São Paulo – SP Bronze
Mauro Cardoso Lopes São Paulo – SP Bronze
André Lucas Ribeiro dos Santos Pindamonhangaba – SP Bronze
Renata Sayuri Takehara S. J. dos Campos – SP Bronze
Henrique Castro Noronha Valinhos – SP Bronze
Matheus Migliolo Coelho Limeira – SP Bronze
Lucas de Freitas Frenay Santo André – SP Bronze
Rafael Marini Silva Vila Velha – ES Bronze
André Slepetys São Paulo – SP Bronze
Luiza Fontana Barbosa Curitiba – PR Bronze
Jefferson Fonlin Tsai São Paulo – SP Bronze
Deborah Regina Fujisawa Okuno São Paulo – SP Bronze
Felipe Paupitz Schlichting Florianópolis – SC Bronze
Elton Gomes Coriolano Fortaleza – CE Bronze
Álison Santos Xavier Fortaleza – CE Menção Honrosa
Marcus Vinícius Martins da Costa Belo Horizonte – MG Menção Honrosa
Rodrigo Viana Soares Fortaleza – CE Menção Honrosa
Lucas M. Pereira Castello Branco Fortaleza – CE Menção Honrosa
Larissa Rodrigues Ribeiro Fortaleza – CE Menção Honrosa
Thiago Jorge Marinho Vieira Fortaleza – CE Menção Honrosa
Cincinato Furtado Leite Neto Fortaleza – CE Menção Honrosa
Rafael Kitayama Shiraiwa São Paulo – SP Menção Honrosa
Anderson Hoshiko Aiziro São Paulo – SP Menção Honrosa
Guilherme Alonso Daud Patavino Santos – SP Menção Honrosa
Vitor Humia Fontoura Salvador – BA Menção Honrosa
André Schultz Santa Bárbara D'Oeste – SP Menção Honrosa
Gabriel Tavares Bujokas São Paulo – SP Menção Honrosa
Francisco Bruno de Lima Holanda Fortaleza – CE Menção Honrosa
Gustavo Eufrásio Farias Fortaleza – CE Menção Honrosa
Katja Stephanie Ried Valinhos – SP Menção Honrosa
Raul Máximo Alexandrino Nogueira Fortaleza – CE Menção Honrosa
Marcos Vainer Loeff São Paulo – SP Menção Honrosa
Antonia Taline de Souza Mendonça Fortaleza – CE Menção Honrosa
Júlio Vitório dos Santos Ferreira Rio de Janeiro – RJ Menção Honrosa

EUREKA! N°13, 2002

58
Sociedade Brasileira de Matemática

Resultado – Nível 3 (Ensino Médio)


NOME CIDADE – ESTADO PRÊMIO
Humberto Silva Naves São José dos Campos – SP Ouro
Davi Máximo Alexandrino Nogueira Fortaleza – CE Ouro
Larissa Cavalcante Queiroz de Lima Fortaleza – CE Ouro
Carlos Stein Naves de Brito Goiânia – GO Ouro
Alex Corrêa Abreu Niterói – RJ Ouro
Daniel Pinheiro Sobreira Fortaleza – CE Prata
Einstein do Nascimento Júnior Fortaleza – CE Prata
Guilherme Fujiwara São Paulo – SP Prata
Thiago Barros Rodrigues Costa Fortaleza – CE Prata
Rafael Tajra Fonteles Teresina – PI Prata
Eduardo Famini Silva Salvador – BA Prata
Rodrigo Roque Dias São Paulo – SP Prata
Fábio Dias Moreira Rio de Janeiro – RJ Prata
Daniel Pessôa Martins Cunha Fortaleza – CE Bronze
Yuri Gomes Lima Fortaleza – CE Bronze
Paulo Ribeiro de Almeida Neto Ananindeua – PA Bronze
Thiago da Silva Sobral Fortaleza – CE Bronze
Germanna de Oliveira Queiroz Fortaleza – CE Bronze
Bernardo Freitas Paulo da Costa Rio de Janeiro – RJ Bronze
Samuel Barbosa Feitosa Fortaleza – CE Bronze
Isaac Newton Ferreira Santa Rita Nova Iguaçu – RJ Bronze
José Luiz Gomes Junior Belém – PA Bronze
Ayran Ayres Barbosa Loriato Vitória – ES Bronze
Fernanda Maria de Oliveira Nicacio Fortaleza – CE Bronze
Henrique Chociay Pinhais – PR Bronze
João Alfredo Castellani Fajardo Freire Salvador – BA Bronze
Rafael da Silva Faria Rio de Janeiro – RJ Bronze
Israel Franklim Dourado Carrah Fortaleza – CE Menção Honrosa
Artur Duarte Nehmi São Paulo – SP Menção Honrosa
Lucas de Melo Pontes e Silva Fortaleza – CE Menção Honrosa
Diogo Luiz Duarte Rio de Janeiro – RJ Menção Honrosa
Estillac B. Filho Belém – PA Menção Honrosa
Alex Cardoso Lopes São Paulo – SP Menção Honrosa
Arthur M. Rocha de Azevedo Scalercio Belém – PA Menção Honrosa
Diego Silva Dias Belém – PA Menção Honrosa
Martha Priscilla Araújo de Moraes Fortaleza – CE Menção Honrosa
Ricardo Monteiro da Silva Lanna Belo Horizonte – MG Menção Honrosa
Fernando Souza Martins S. J. dos Campos – SP Menção Honrosa
Maurício Richartz Curitiba – PR Menção Honrosa
Léo Tsukui Belém – PA Menção Honrosa
Vitor Gabriel Kleine Mogi das Cruzes – SP Menção Honrosa

EUREKA! N°13, 2002

59
Sociedade Brasileira de Matemática

Resultado – Nível Universitário


NOME CIDADE – ESTADO PRÊMIO
Emanuel Augusto de Souza Carneiro Fortaleza – CE Ouro
Carlos Yuzo Shine São Paulo – SP Ouro
Daniel Massaki Yamamoto São Paulo – SP Ouro
Fabrício Siqueira Benevides Fortaleza – CE Prata
Diêgo Veloso Uchôa Teresina – PI Prata
Frederico Vale Girão Fortaleza – CE Prata
Bruno Fernandes Cerqueira Leite São Paulo – SP Prata
Marcio Afonso Assad Cohen Rio de Janeiro – RJ Prata
Lucas Heitzmann Gabrielli São Paulo – SP Prata
Christian Iveson São Paulo – SP Bronze
Daniel Nobuo Uno São Paulo – SP Bronze
Giuliano Boava Florianópolis – SC Bronze
Vinícius José Fortuna Campinas – SP Bronze
Leonardo Augusto Zão Nilópolis – RJ Bronze
Leandro de Mattos Ferreira Rio de Janeiro – RJ Bronze
Rodrigo Villard Milet Rio de Janeiro – RJ Bronze
Tertuliano Franco Santos Franco Salvador – BA Bronze
Aleksander Medella Campos da Silva Rio de Janeiro – RJ Bronze
Arnaldo João do Nascimento Junior Duque de Caxias – RJ Bronze
Artur Radoman de Oliveira Rio de Janeiro – RJ Bronze
Bruno Germano Borics Rio de Janeiro – RJ Bronze
Thiago Afonso de André São Paulo – SP Bronze
Juliana Abrantes Freire Rio de Janeiro – RJ Bronze
Felipe Duarte Cardozo de Pina Rio de Janeiro – RJ Bronze
Fernando Prado Rocha Goiânia – GO Menção Honrosa
Camilo Marcantonio Junior Rio de Janeiro – RJ Menção Honrosa
Dúlio Matos Leite de Carvalho e Silva Rio de Janeiro – RJ Menção Honrosa
Diogo Diniz Pereira da Silva e Silva Campina Grande – PB Menção Honrosa
Ilan Lobel Rio de Janeiro – RJ Menção Honrosa
Rafael Pellizzer Soares Jundiaí – SP Menção Honrosa
Anderson Rodrigues Ferreira Rio de Janeiro – RJ Menção Honrosa
Rafael de Freitas Lemos S. J. dos Campos – SP Menção Honrosa
Bruno Martins Reboredo Rio de Janeiro – RJ Menção Honrosa
Daniele Véras de Andrade Rio de Janeiro – RJ Menção Honrosa

EUREKA! N°13, 2002

60
Sociedade Brasileira de Matemática

AGENDA OLÍMPICA

XXIV OLIMPÍADA BRASILEIRA DE MATEMÁTICA

NÍVEIS 1, 2 e 3
Primeira Fase – Sábado, 8 de junho de 2002
Segunda Fase – Sábado, 14 de setembro de 2002
Terceira Fase – Sábado, 19 de outubro de 2002 (níveis 1, 2 e 3)
Domingo, 20 de outubro de 2002 (níveis 2 e 3 - segundo dia de prova).

NÍVEL UNIVERSITÁRIO
Primeira Fase – Sábado, 14 de setembro de 2002
Segunda Fase – Sábado, 19 e Domingo, 20 de outubro de 2002

VIII OLIMPÍADA DE MAIO
maio de 2002

XIII OLIMPÍADA DE MATEMÁTICA DO CONE SUL
22 a 28 de junho de 2002
Fortaleza – CE, Brasil

XLIII OLIMPÍADA INTERNACIONAL DE MATEMÁTICA
18 a 31 de julho de 2002
Glasgow, Reino Unido

XVII OLIMPÍADA IBEROAMERICANA DE MATEMÁTICA
30 de setembro a 5 de outubro de 2002
El Salvador

V OLIMPÍADA IBEROAMERICANA DE MATEMÁTICA UNIVERSITÁRIA
outubro de 2002

♦♦♦

EUREKA! N°13, 2002

61
Sociedade Brasileira de Matemática

COORDENADORES REGIONAIS
Amarísio da Silva Araújo (UFV) Viçosa – MG
Alberto Hassen Raad (UFJF) Juiz de Fora – MG
Benedito Tadeu Vasconcelos Freire (UFRN) Natal – RN
Carlos Frederico Borges Palmeira (PUC-Rio) Rio de Janeiro – RJ
Claudio Arconcher (Colégio Leonardo da Vinci) Jundiaí – SP
Claus Haetinger (UNIVATES) Lajeado – RS
Cleonor Crescêncio das Neves (UTAM) Manaus – AM
Élio Mega (Colégio Etapa) São Paulo – SP
Rosângela Souza (Colégio Singular) Santo André – SP
Florêncio Ferreira Guimarães Filho (UFES) Vitória – ES
Gisele de Araújo Prateado Gusmão (UFGO) Goiânia – GO
Ivanilde Fernandes Saad (UC. Dom Bosco) Campo Grande– MS
Jacqueline Fabiola Rojas Arancibia (UFPB) João Pessoa – PB
João Benício de Melo Neto (UFPI) Teresina – PI
João Francisco Melo Libonati (Grupo Educacional Ideal) Belém – PA
Irene Nakaoka (UEM) Maringá – PR
José Carlos Pinto Leivas (UFRG) Rio Grande – RS
José Cloves Saraiva (UFMA) São Luis – MA
José Gaspar Ruas Filho (ICMC-USP) São Carlos – SP
José Luiz Rosas Pinho (UFSC) Florianópolis – SC
José Vieira Alves (UFPB) Campina Grande – PB
Marcelo Rufino de Oliveira (Grupo Educacional Ideal) Belém – PA
Licio Hernandes Bezerra (UFSC) Florianópolis – SC
Luzinalva Miranda de Amorim (UFBA) Salvador – BA
Marcondes Cavalcante França (UFC) Fortaleza – CE
Pablo Rodrigo Ganassim (Liceu Terras do Engenho) Piracicaba – SP
Reinaldo Gen Ichiro Arakaki (INPE) SJ dos Campos – SP
Ricardo Amorim (Centro Educacional Logos) Nova Iguaçu – RJ
Roberto Vizeu Barros (Colégio Acae) Volta Redonda – RJ
Sérgio Cláudio Ramos (IM-UFRGS) Porto Alegre – RS
Silvio de Barros Melo (UFPE) Recife – PE
Tadeu Ferreira Gomes (UEBA) Juazeiro – BA
Tomás Menéndez Rodrigues (U. Federal de Rondônia) Porto Velho – RO
Valdenberg Araújo da Silva (U. Federal de Sergipe) São Cristovão – SE
Wagner Pereira Lopes (Escola Técnica Federal de Goiás) Jataí – GO

EUREKA! N°13, 2002

62
CONTEÚDO

AOS LEITORES 2

VIII OLIMPÍADA DE MAIO 3


Enunciados e Resultado Brasileiro

XIII OLIMPÍADA DE MATEMÁTICA DO CONE SUL 6


Enunciados, Soluções e Resultado Brasileiro

XLIII OLIMPÍADA INTERNACIONAL DE MATEMÁTICA 17


Enunciados e Resultado Brasileiro

ARTIGOS
MUROS, PRÉDIOS E ESCADAS 19
Cícero de Oliveira Holmer

INTEIROS DE GAUSS E INTEIROS DE EISENSTEIN 23


Guilherme Fujiwara

SEQÜÊNCIAS ARITMÉTICO-GEOMÉTRICAS 32
José Paulo Carneiro & Carlos Gustavo Moreira

O PRINCÍPIO DA INVARIÂNCIA 35
Marcelo Rufino de Oliveira

TORNEIO DAS CIDADES 43


Provas

OLIMPÍADAS AO REDOR DO MUNDO 48

SOLUÇÕES DE PROBLEMAS PROPOSTOS 53

PROBLEMAS PROPOSTOS 59

AGENDA OLÍMPICA 61

COORDENADORES REGIONAIS 62
Sociedade Brasileira de Matemática

AOS LEITORES

É com grande alegria que anunciamos que, mais uma vez, os 6 estudantes da
equipe brasileira obtiveram medalhas na IMO. Isto mostra a nossa grande evolução
com a nova OBM e as atividades extras que apareceram com ela: a Eureka, a
Semana Olímpica, as semanas de treinamento antes das competições internacionais.
Mas o que nos traz maior satisfação é saber que esses 6 jovens são a ponta de um
iceberg. Basta observar o número de pessoas que resolvem os vários problemas que
trazemos a cada nova edição da Eureka. São estudantes, professores, profissionais
liberais, enfim, amantes da Matemática que provam que no nosso país há muitas
pessoas de boa vontade e de grande competência.

Falando mais um pouco sobre problemas, nesse número publicamos algumas


provas do Torneio das Cidades, uma competição que se caracteriza pela
originalidade de suas questões, algumas das quais já entraram para o folclore
matemático. Ela possui duas modalidades, Sênior (2a e 3a séries EM) e Júnior (8a
série EF e 1a série EM); níveis O (iniciante) e A (avançado).

Para encerrar, os agradecimentos. Mais uma vez, o professor Carlos Shine e


os estudantes Alex Lopes, Felipe de Souza, Henry Hsu, Rodrigo Yamashita e
Guilherme Fujiwara fizeram uma leitura cuidadosa das versões prévias desta edição.

Os editores.

EUREKA! N°14, 2002

2
Sociedade Brasileira de Matemática

VIII OLIMPÍADA DE MAIO


Enunciados e Resultado Brasileiro

PRIMEIRO NÍVEL

PROBLEMA 1
Um grupo de homens, alguns dos quais acompanhados pelas esposas, gastaram 1000
dólares num hotel. Cada homem gastou 19 dólares e cada mulher, 13 dólares.
Determine quantas mulheres e quantos homens estavam no hotel.

PROBLEMA 2
Uma folha de papel retangular (branca de um lado e cinza do outro) foi dobrada três
vezes, como mostra a figura abaixo:
2 3
1

O retângulo 1, que ficou da cor branca após a primeira dobra, tem 20cm a mais de
perímetro que o retângulo 2, que ficou branco após a segunda dobra, e este por sua
vez tem 16cm a mais de perímetro que o retângulo 3, que ficou branco após a
terceira dobra. Determine a área da folha.

PROBLEMA 3
Mustafá comprou um tapete. O vendedor mediu o tapete com uma régua que
supostamente media um metro. Como o resultado foi que o tapete tinha 30 metros de
largura e 20 metros de comprimento, o vendedor cobrou 120000 rupias. Quando
Mustafá chegou a sua casa mediu novamente o tapete e percebeu que o vendedor
tinha cobrado 9408 rupias a mais. Quantos centímetros mede a régua usada pelo
vendedor?

PROBLEMA 4
Num banco só o diretor conhece o segredo do cofre, que é um número de cinco
dígitos. Para proteger este segredo são dados a cada um dos dez empregados do
banco um número de cinco dígitos. Cada um destes números tem numa das cinco
posições o mesmo dígito que o segredo e nas outras quatro posições um dígito
diferente do que tem o segredo nesse lugar. Os números de proteção são:

07344, 14098, 27356, 36429, 45374, 52207, 63822, 70558, 85237, 97665.

Qual é o segredo do cofre?

EUREKA! N°14, 2002

3
Sociedade Brasileira de Matemática

PROBLEMA 5
Encontre o máximo número de caixinhas de 3 × 5 × 7 que podem ser colocadas
dentro de uma caixa de 11 × 35 × 39. Para o número encontrado, indique como
colocar essa quantidade de caixinhas dentro da caixa.

SEGUNDO NÍVEL

PROBLEMA 1
Utilizando cubinhos brancos de lado 1 foi montado um prisma (sem buracos).
As faces do prisma foram pintadas de preto. Sabe-se que os cubinhos que ficaram
com exatamente 4 faces brancas são 20 no total. Determine quais podem ser as
dimensões do prisma. Encontre todas as possibilidades.

PROBLEMA 2
Seja k um número inteiro positivo fixo, k ≤ 10.
Dada uma lista de dez números, a operação permitida é: escolher k números da lista,
e somar 1 a cada um deles. Obtém-se assim uma nova lista de dez números.
Se inicialmente temos a lista 1, 2, 3, 4, 5, 6, 7, 8, 9, 10, determine os valores de k
para os quais é possível, mediante uma seqüência de operações permitidas, obter
uma lista que tenha os dez números iguais. Indique a seqüência para cada caso.

PROBLEMA 3
Num triângulo ABC, retângulo em A e isósceles, seja D um ponto do lado AC (D ≠ A
e D ≠ C) e seja E o ponto do prolongamento do lado BA tal que o triângulo ADE é
isósceles. Se P é o ponto médio do segmento BD, R é o ponto médio do segmento
CE e Q o ponto onde se cortam as retas ED e BC, demonstre que o quadrilátero
ARQP é um quadrado.

PROBLEMA 4
Os vértices de um polígono regular de 2002 lados estão numerados de 1 a 2002, no
sentido horário. Dado um inteiro n, 1 ≤ n ≤ 2002, pinta-se de azul o vértice n, logo,
seguindo o sentido horário, contam-se n vértices começando no seguinte de n, e
pinta-se de azul o número n. E assim sucessivamente, a partir do vértice que segue
ao último vértice que há sido pintado, contam-se n vértices, pintados ou sem pintar, e
o número n é pintado de azul. Quando o vértice que tem que ser pintado já é azul, o
processo pára. Denotamos P(n) ao conjunto de vértices azuis que se obtém com este
procedimento quando se começa pelo vértice n. Por exemplo, P(364) está formado
pelos vértices 364, 728, 1092, 1456, 1820, 182, 546, 910, 1274, 1638 e 2002.
Determine todos os inteiros n, 1 ≤ n ≤ 2002, tais que P(n) tem exatamente 14
vértices.

EUREKA! N°14, 2002

4
Sociedade Brasileira de Matemática

PROBLEMA 5
Dados x e y inteiros positivos, consideramos um quadriculado de x × y, que tem
pintados de vermelho os (x + 1) ⋅ (y + 1) pontos que são vértices de quadradinhos.
Inicialmente há uma formiga em cada um dos pontos vermelhos. Num instante dado,
todas as formigas começam a caminhar pelas linhas do quadriculado, todas com a
mesma velocidade. Cada vez que chegam num ponto vermelho, giram 90° em
alguma direção.
Determine todos os valores de x e y para os quais é possível que as formigas
continuem movendo-se indefinidamente de maneira que em nenhum momento há
duas ou mais formigas num mesmo ponto vermelho. (Não interessam as possíveis
coincidências em pontos das linhas do quadriculado que não são vermelhos.)

RESULTADO BRASILEIRO

PRIMEIRO NÍVEL (ATÉ 13 Anos)


Eduardo Fischer Encantado – RS Medalha de Ouro
Pedro Nogueira Machado Rio de Janeiro – RJ Medalha de Prata
André Márcio de Lima Curvello Goiânia – GO Medalha de Prata
Katja Stephanie Ried Valinhos – SP Medalha de Bronze
Enzo Haruo Hiraoka Moriyama São Paulo – SP Medalha de Bronze
Anderson Gleryston Silva Sousa Campina Grande – PB Medalha de Bronze
Mariana Nasser Brolezzi Santo André – SP Medalha de Bronze
Arthur Rodrigues de Oliveira Sobral S. José dos Campos – SP Menção Honrosa
Cássio Kendi Takamori S. José dos Campos – SP Menção Honrosa
Diogo Bonfim Moraes Morant de Holanda Rio de Janeiro – RJ Menção Honrosa

SEGUNDO NÍVEL (ATÉ 15 Anos)


Fábio Dias Moreira Rio de Janeiro – RJ Medalha de Ouro
Guilherme Rodrigues Salerno Goiânia – GO Medalha de Prata
Telmo Luis Correa Júnior Santo André – SP Medalha de Prata
Thiago Costa Leite Santos São Paulo – SP Medalha de Bronze
André Rodrigues Salerno Goiânia – GO Medalha de Bronze
Henry Wei Cheng Hsu São Paulo – SP Medalha de Bronze
Rodrigo Aguiar Pinheiro Fortaleza – CE Medalha de Bronze
Rafael Marini Silva Vila Velha – ES Menção Honrosa
Larissa Rodrigues Ribeiro Fortaleza – CE Menção Honrosa
Douglas Bokliang Ang Cunha S. José dos Campos – SP Menção Honrosa

EUREKA! N°14, 2002

5
Sociedade Brasileira de Matemática

XIII OLIMPÍADA DE MATEMÁTICA DO CONE SUL


Enunciados, Soluções e Resultado Brasileiro

A XIII Olimpíada de Matemática do Cone Sul foi realizada na cidade de


Fortaleza, Ceará no período de 22 a 28 de junho de 2002.
A equipe brasileira foi liderada pelos professores Yoshiharu Kohayakawa
(São Paulo – SP) e Luciano Guimarães Castro (Rio de Janeiro – RJ).

O Resultado da Equipe Brasileira

BRA 1 Alex Corrêa Abreu Ouro


BRA 2 Israel Dourado Carrah Bronze
BRA 3 Larissa Cavalcante Queiroz de Lima Ouro
BRA 4 Rafael Daigo Hirama Ouro

PRIMEIRO DIA
DURAÇÃO: 4 horas e meia.

PROBLEMA 1:
Os alunos da turma de Pedro praticam a soma e a multiplicação de números inteiros.
A professora escreve os números de 1 a 9 em nove fichas, uma para cada número, e
as coloca em uma urna. Pedro retira três fichas e deve calcular a soma e o produto
dos três números correspondentes. Ana e Julián fazem o mesmo, esvaziando a urna.
Pedro informa à professora que retirou três números consecutivos cujo produto é 5
vezes a soma. Ana informa que não tem nenhum número primo, mas sim dois
consecutivos e que o produto desses três números é 4 vezes a soma dos mesmos.
Quais números retirou Julián?

SOLUÇÃO DE ISRAEL FRANKLIM DOURADO CARRAH (FORTALEZA – CE):


Diremos que Pedro escolheu os números P1, P1 + 1 e P1 + 2, Ana retirou os números
A1, A2 e A3. Logo, temos que:
P1 ⋅ ( P1 + 1) ⋅ ( P1 + 2) = 5 ⋅ ( P1 + P1 + 1 + P1 + 2) = 5 ⋅ (3P1 + 3) = 3 ⋅ 5 ⋅ ( P1 + 1) e como
(P1 + 1) é um número positivo ⇒ P1 ⋅ ( P1 + 2) = 3 ⋅ 5 = 15 ⇒ P1 e ( P1 + 2) são
divisores de 15 (obviamente P1 + 2 > P1). Assim, temos duas possibilidades:
 P1 = 1
1a . Absurdo!
 P1 + 2 = 15 → P1 = 13 = 1
 P1 = 3
2 a . Ok!
 P1 + 2 = 5 → P1 = 3
Portanto, Pedro escolheu os números 3, 4 e 5.

EUREKA! N°14, 2002

6
Sociedade Brasileira de Matemática

Como Ana escolheu 3 números que não são primos ⇒ os possíveis números
retirados por Ana são 1, 6, 8 e 9. (pois 2 e 7 são números primos e 3, 4 e 5 são
números que já foram retirados por Pedro.)
Mas temos também que Ana retirou dois números consecutivos e dentre (1, 6, 8 e 9)
os únicos dois números consecutivos são 8 e 9 ⇒ Ana escolheu os números 8 e 9.
Assim: A2 = 8, A3 = 9 e A1 ⋅ 8 ⋅ 9 = 4 ⋅ (A1 + 8 + 9) ⇒ A1 ⋅ 18 = A1 + 17 ⇒ 17A1 = 17
⇒ A1 = 1. Logo, Ana escolheu os números 1, 8 e 9.
Portanto, Julián retirou os números restantes: 2, 6 e 7.

PROBLEMA 2:
De um triângulo ABC, retângulo em A, conhecemos: o ponto T de tangência da
circunferência inscrita em ABC com a hipotenusa BC, o ponto D de interseção da
bissetriz interna do ângulo B̂ com o lado AC e o ponto E de interseção da bissetriz
interna do ângulo Ĉ com o lado AB. Descreva uma construção com régua e
compasso para obter os pontos A, B e C. Justifique.

SOLUÇÃO DE LARISSA CAVALCANTE QUEIROZ DE LIMA (FORTALEZA – CE):


A a +b+c
. BC = a, AC = b, AB = c; p =
2
E
D N ⇒ CT = p – c; BT = p – b
sejam Y tal que EY ⊥ BC e W tal que
θ DW ⊥ BC
θ .
C . β
β
EC comum
W T Y B 
* ∆EYC ≡ ∆AEC ACˆ E = ECˆ Y (CE bissetriz)
 ˆ
CAE = EYC = 90°
ˆ

 AE = EY
⇒
 AC = CY = b
BD comum

* ∆DWB ≡ ∆ADB BWˆ D = BAˆ D = 90°
 ˆ
 ABD = DBW ( DB bissetriz)
ˆ
 DW = AD
⇒
 AB = WB = c

EUREKA! N°14, 2002

7
Sociedade Brasileira de Matemática

* WT = WB – TB = c – (p – b) = (c + b + a) – a – p = 2p – p – a = p – a
* YT = YC – CT = b – (p – c) = c + b – p = p – a
(
⇒ T é ponto médio de WY. *DEYW é um trapézio retângulo DWˆ Y = WYˆE = 90° . )
Construção do ∆ ABC.
Dados D e E, é fácil obter com régua e compasso o ponto N, ponto médio de DE.
Sabemos que os pontos D, E, Y e W do ∆ABC formam um trapézio retângulo, sendo
T o ponto médio de YW ⇒ NT será base média e NT//EY//DW ⇒
NTˆY = 90° ⇒ NT⊥BC ⇒ Conhecemos já a reta NT, com compasso, marcamos
N '∈ NT ; N ' T = TN .
A mediatriz de NN'é perpendicular a NN' em T, portanto coincide com a reta BC ⇒
conhecemos agora a reta BC .
* EYˆT = 90° ⇒ para encontrar Y, encontramos M1, ponto médio de ET e contruímos
com compasso a circunferência Γ1 de centro M1 passando por E e T. Onde Γ1
encontrar a reta BC será o ponto Y (note que será no segundo ponto de encontro
com BC , o primeiro é T). Analogamente, encontramos o ponto W no encontro da
circunfrência Γ2 de diâmetro DT e da reta BC .
Dessa maneira, encontramos EY e DW . Com o compasso, encontramos Γ3 de raio
EY e centro E, e Γ4 de raio DW e centro D. Γ3 ∩ Γ4 = { A, A' } A será o ponto que está
"acima"de DE ( supondo o ponto T "abaixo"de DE).
Encontramos então as retas AE e AD .
B = AE ∩ BC e C = AD ∩ BC
∴Encontramos então ∆ABC
obs: Se Y coincidir com T, temos que W coincidirá com T e p – a = 0 ou seja,
a+b+c
− a = 0 ⇔ a + b + c − 2a = 0 ⇔ b + c = a Absurdo!
2
⇒ Y ≠ T e W ≠ T.

PROBLEMA 3:
Arnaldo e Bernardo jogam uma Super Batalha Naval. Cada um tem um tabuleiro n x
n. Arnaldo coloca barcos em seu tabuleiro (pelo menos um mas não se sabe
quantos). Cada barco ocupa as n casas de uma linha ou de uma coluna e os barcos
não podem se superpor nem ter um lado comum. Bernardo marca m casas
(representando tiros) em seu tabuleiro. Depois que Bernardo marcou as m casas,
Arnaldo diz quais dentre elas correspondem a posições ocupadas por barcos.
Bernardo ganha se, a seguir, descobre quais são as posições de todos os barcos de

EUREKA! N°14, 2002

8
Sociedade Brasileira de Matemática

Arnaldo. Determine o menor valor de m para o qual Bernardo pode garantir sua
vitória.

SOLUÇÃO DE ALEX CORRÊA ABREU (NITERÓI – RJ):


Suponha n ≥ 4
(I) Primeiro vemos que obviamente tem que marcar casas em todas as linhas e
colunas, pois se uma linha não tiver nenhuma casa marcada obviamente podemos ter
um barco em uma linha não adjacente portanto Bernardo não saberá se ali tem ou
não um barco. De modo análogo para as colunas.

(II) Agora suponha que uma casa está marcada, se na linha e na coluna dessa casa
não tiver mais nenhuma outra casa marcada, pode vir a calhar de arnaldo ter
colocado apenas um barco no tabuleiro e ser exatamente nessa linha, portanto
Bernardo não sabe se o barco está na vertical ou na horizontal pois só vai ter uma
casa onde sabe que o barco está. Então para cada casa marcada existe outra na linha
ou na coluna.

Considere agora que Arnaldo marcou um barco na linha i


⇒ se uma casa é marcada na linha i, para identificar se o
barco está na horizontal ou na vertical precisamos de mais
uma casa adjacente. Se esta estiver pintada o barco está
na vertical (baseado no desenho) e na horizontal caso
contrário.

O problema consiste em pintar blocos 1 × k tais que cada


linha e cada coluna tem interseção com um deles, agora
k1 se tivermos um 1 × k se k > 3 podemos dividir e de fato
k apenas melhoramos as coisas então os blocos são 1 × 2 e
1 × 3.
k2

M(n) é o mínimo procurado para um tabuleiro n × n.

EUREKA! N°14, 2002

9
Sociedade Brasileira de Matemática

3 Se n ≥ 3 retire as 3 primeiras linas e as 3 primeiras


colunas então m(n) = m(3) + m (n – 3) pois precisamos de
1 A m(n – 3) pois se não tiver nenhum barco nas 6 fileiras que
saíram e m(3) se não tiver nas que ficaram pois se
A considerarmos só a região A.
temos que colocar 3 em cada no mínimo, o que dá mais 6
contra 4 de m(3) ⇒ (também porque podemos diminuir as
intersecções) ⇒

m(3k ) = km(3) = 4k

m(3k + 1) = (k − 1)m(3) + m(4) = 4k − 4 + 6 = 4k + 2
m(3k + 2) = 4(k − 1) + m(5) = 4k + 3

é fácil ver que m(3) = 4 pois 3 obviamente não é e m(4) = 6, m (5) = 7 como ao lado.

SEGUNDO DIA
DURAÇÃO: 4 horas e meia.

PROBLEMA 4:
Seja ABCD um quadrilátero convexo tal que suas diagonais AC e BD são
perpendiculares. Seja P a interseção de AC e BD e seja M o ponto médio de AB.
Mostre que o quadrilátero ABCD é inscritível se, e somente se, as retas PM e CD são
perpendiculares.

SOLUÇÃO DE ISRAEL FRANKLIM DOURADO CARRAH (FORTALEZA – CE):


Primeiramente vejamos quando PM e CD são perpendiculares
B
M θ

A
90° – θ

90° – θ θ

P
90° – θ

D .. θ
K C

EUREKA! N°14, 2002

10
Sociedade Brasileira de Matemática

Seja MP ∩ CD = {K }. Como no ∆ABP , retângulo em P, M é o ponto médio da


hipotenusa AB ⇒ PM = MA = MB. Assim, seja
ABˆ D = θ ⇒ MPˆ B = θ ⇒ AMˆ P = 2θ ⇒
MPˆ A = 90° − θ ⇒ CPˆ K = APˆ M = 90° − θ e como
PKˆ C = 90° ⇒ PCˆ D = θ . Logo, ABˆ D = ACˆ D = θ ⇒ O quadrilátero ABCD é
inscritível!
Vejamos agora se ABCD é inscritível:

B
M θ
A
90° – θ

90° – θ

P
90° – θ

D θ
K C

Do mesmo modo como M é o ponto médio da hipotenusa AB do triângulo retângulo


APB ⇒ PM = MA = MB.
Logo, se ABˆ D = θ ⇒ BAˆ P = MPˆ A = 90° − θ ⇒ CPˆ K = 90° − θ e como ABCD é
inscritível ⇒ ACˆ D = ABˆ D = θ ⇒ PKˆ C = 180° − (90° − θ + θ ) = 90° ⇒ MP⊥CD .
Portanto, ABCD é inscritível ⇔ PM⊥CD.

PROBLEMA 5:
Considere o conjunto A = {1, 2,…, n}. Para cada inteiro k, seja rk a maior quantidade
de elementos distintos de A que podemos escolher de maneira que a diferença entre
dois números escolhidos seja sempre diferente de k. Determine o maior valor
n
possível de rk, onde 1 ≤ k ≤ .
2

EUREKA! N°14, 2002

11
Sociedade Brasileira de Matemática

SOLUÇÃO DE RAFAEL DAIGO HIRAMA (CAMPINAS – SP):


Vamos analizar casos pequenos:
1, 2, 3, 4, 5, 6, 7, 8 k = 1 rk = 4 1, 2, 3, 4, 5, 6, 7, 8, 9 k = 1 rk = 5
k = 2 rk = 4 k=2 rk = 5
k = 3 rk = 5 k=3 rk = 6
k = 4 rk = 4 k=4 rk = 5

1, 2, 3, 4, 5, 6, 7, 8, 9, 10
k = 1 rk = 5
k = 2 rk = 6
k = 3 rk = 6
k = 4 rk = 6
k = 5 rk = 5
Isso me deu idéia para um lema!

 m + 1
Lema: Para n = m ⋅ k (m inteiro positivo maior que 1) temos que rk = k ⋅  
 2 
Prova: Podemos dividir os números em casas de congruências módulo k. Por
exemplo o 0.
Seus componentes serão k, 2k, 3k,...,mk. Como a diferença entre dois deles deve ser
diferente de k, não podemos escolher dois números consecutivos nessa seqüência.
Sempre deve haver um "ausente" ou mais entre dois "presentes". Para m par temos
m m
que só poderemos ter escolhidos pois caso tenhamos + 1 escolhidos teremos
2 2
m
− 1 não escolhidos, mas para separar os escolhidos (para não serem consecutivos)
2
m
deveríamos ter pelo menos não escolhidos. Absurdo.
2
m +1 m +1
Para m ímpar teremos escolhidos que podem ser espaçados pelos − 1 não
2 2
m +1 m +1
escolhidos. Do mesmo modo, para + 1 deveríamos ter espaços mas só
2 2
 m +1  m − 3
teriam sobrado m −  + 1 = não escolhidos que não são suficientes.
 2  2

EUREKA! N°14, 2002

12
Sociedade Brasileira de Matemática

Como temos k casas de congruência com m números cada e pelo fato de


m +1
se m é ímpar
 m + 1   2  m + 1
 2  = m temos que rk = k ⋅ 
2 
.
   
 2 se m é par
Agora precisamos ver como transformar o lema em algo que seja mais versátil ao
nosso problema, ou seja, não devemos ter de usar o fato n = m ⋅ k.
 2n 
Analizando mais casos pequenos estou conjecturando que o rk máximo é  .
3
2n
Vamos provar que rk ≤ para todo k.
3
2n
Suponha o contrário, que há rk > . Logo vamos provar primeiro que em uma casa
3
2
de congruência módulo k com j termos o aproveitamento máximo de termos é do
3
total j de termos. Temos a seguinte regra: se x foram escolhidos então pelo menos x –
x 2
1 não podem ter sido. Logo devemos provar que ≤ ⇒ 2 j ≥ 3 x. Sabemos que
j 3
j ≥ x + x − 1, portanto vale 2 j ≥ 3x ⇐ 2[x + ( x − 1)] ≥ 3x ⇔ 4x − 2 ≥ 3x ⇔ x ≥ 2.
Para x ≥ 2 já está provado. Mas se escolhermos x = 1 necessariamente j = 2 pois se j
= 1, ou seja, se há somente um número entre 1 e n com congruência módulo k
n
significa que 2k > n, então k > o que contradiz o enunciado.
2
1 2
Nesse caso o aproveitamento é que é menor que .
2 3
Chamando o aproveitamento para a casa de congruência i de ai e o número de termos
nessa casa de congruência de bi temos
2 2
rk = a1b1 + a 2 b2 + ... + a k bk ≤ (b1 + b2 + b3 + ... + bk ) = n
3 3
2
( pois b1 + b2 + ... + bk = n) portanto rk ≤ n
3
2 
como rk é inteiro rk máximo é  n .
3 
Falta provar a existência de tal rk. Se faz assim:

EUREKA! N°14, 2002

13
Sociedade Brasileira de Matemática

n
Divide-se n por 3 e arredonda-o para cima. Esse é o nosso k. k =  . É obvio que
3
n
para n ≥ 2, k ≤ . ( se n =1 k não existe)
2
2
Se n = 3k então rk = n e é máximo
3
Se n = 3k – 1
n + 1 = 3k então teríamos, em relação ao caso acima a perda de um termo, a
2(n + 1) 2 1 2 
escolher. O 3k rk máximo é rk = − 1 = n − =  n
3 3 3 3 
Se n = 3k – 2
n + 2 = 3k do mesmo modo perdemos 2 em relação ao primeiro caso
2(n + 2) 2 2 2 
rk = − 2 = n − =  n .
3 3 3 3 
Portanto ao montar o caso n = 3k escolhe-se:
2
1, 2, 3,..., k, 2k, 2k + 1,..., 3k totalizando 2k termos n
3
O caso n = 3k – 1 e n = 3k – 2 retira-se o 3k; e o 3k e o 3k – 1 respectivamente.
 1
n = 1 → não podemos ter k 1 ≤ k ≤ 2
Resposta: 
n ≠ 1  2n 
rk máximo é  
 3
Obs: x  maior inteiro menor ou igual a x
x −1 < x  ≤ x
x  inteiro
x  menor inteiro maior ou igual a x
x ≤ x  < x + 1
x  inteiro.
PROBLEMA 6:
Dizemos que um inteiro n, n, > 1, é ensolarado se ele é divisível pela soma dos seus
fatores primos. Por exemplo, 90 é ensolarado pois 90 = 2 ⋅ 3 2 ⋅ 5 e 2 + 3 + 5 = 10
divide 90. Mostre que existe um número ensolarado com pelo menos 10 2002 fatores
primos distintos.

EUREKA! N°14, 2002

14
Sociedade Brasileira de Matemática

SOLUÇÃO DE RAFAEL DAIGO HIRAMA (CAMPINAS – SP):


Vamos ver casos pequenos:
2 ⋅ 3 ⋅ 5 = 30 e é ensolarado pois 2 + 3 + 5 = 10|30
Olha só que interessante: se escolhemos alguns números primos e a soma deles
puder ser escrita como um produto qualquer deles o produto de todos esses primos
vezes a soma deles é um número ensolarado, aliás o mmc é ensolarado.
Vamos ver até onde isso vai:
2 + 3 + 5 + 7 = 17 droga! 17 é primo, vamos adaptar
2 + 3 + 5 + 7 + 17 = 34 e 34 = 2 ⋅ 17
mmc (34, 2 ⋅ 3 ⋅ 5 ⋅ 7 ⋅ 17) = 2 ⋅ 3 ⋅ 5 ⋅ 7 ⋅ 17 que é ensolarado
2 + 3 + 5 + 7 + 11 = 28 e 28 = 22 ⋅ 7
mmc ( 28, 2 ⋅ 3 ⋅ 5 ⋅ 7 ⋅ 11) = 22 ⋅ 3 ⋅ 5 ⋅ 7 ⋅ 11 que é ensolarado
2 + 3 + 5 + 7 + 11 + 13 = 41
2 + 3 + 5 + 7 + 11 + 13 + 41 = 82 e 82 = 2 ⋅ 41
2 ⋅ 3 ⋅ 5 ⋅ 7 ⋅ 11 ⋅ 13 ⋅ 41 é ensolarado.
Prova geral: p1 + p2 + p3 + p4 + ... + pn = x, x = p1α1 p 2α 2 ... p nα n (ou seja, não tem fatores
primos além dos p1 a pn, mas α i pode ser 0)
mmc(p1p2p3...pn), x) = p1β1 p 2β 2 ... p nβ n = y. Como x | y e y só tem fatores p1, p2, ..., pn,
y é ensolarado.
Percebendo os meus testes podemos ver um modo de adaptar se tivermos a soma
desses primos um número primo diferente dos anteriores. E se tivermos mais?
p1 < p 2 < p3 < ... < p n e p1 + p 2 + ... + p n = p1α1 p 2α 2 p3α 3 p 4α 4 ... p nα n p nα+n1+1 p nα+n +22 p nα+n +33 ...

p1 + p 2 + ... + p n + p n +1 = p n +1 ( p1α1 p 2α 2 p3α 3 ... p nα n p nα+n1+1 −1 p nα+n +22 ... + 1)


este número é com certeza menor que a soma inicial logo terá um limite para seus
fatores primos.
Fazendo:
p1 + p2 + ... + pn + pn +1 = p1β1 p2β 2 p3β 3 ... pnβ+n1+1 pnβ+n+22 pnβ+n+33 ...
p1 + p2 + ... + pn + pn +1 + pn + 2 = pn + 2 ( p1β 1 p2β 2 p3β 3 p4β 4 ... pnβ+n1+1 pnβ+n+22 −1 pnβ+n+33 ... + 1)
Calma, podemos evitar tudo isso se escolhermos os n primeiros primos:
2 + 3 + 5 +...+ pn = x ≤ n ⋅ pn. Se tivesem dois primos pi e pj em x tal que i, j > n
teremos pipj ≤ x ≤ n ⋅ pn.
Mas perceba que pk > k para todo k ∈ 1 (isso acontece porque a seqüência dos k
percorre todos os naturais enquanto a dos pk "pula" vários naturais).
Continuando
Vamos provar que n ⋅ pn < pi ⋅ pj, não o contrário. pi ⋅ p j > p n ⋅ p n

EUREKA! N°14, 2002

15
Sociedade Brasileira de Matemática

∴ pi ⋅ p j > n ⋅ p n
Logo em x só pode haver um fator primo diferente dos 2, 3, 5,..., pn. Usaremos pi e
do mesmo modo vemos que x não é divisível duas vezes por pi (é só fazer j = i)
2 + 3 + 5 +...+pn = m ⋅ pi ≤ n ⋅ pn
n ⋅ pn
m≤
pi
pn
Olhe só, como < 1 porque pn < pi porque n < i, m < n só que n < pn ⇒ m < pn
pi
Como m e pn são inteiros m + 1 ≤ pn
Agora pronto:
2 + 3 + 5 +...+ pn = m ⋅ pi
2 + 3 + 5 +...+ pn + pi = (m + 1) ⋅ pi
mas como m + 1 ≤ pn , m + 1 pode ser escrito como produto dos primos 2, 3, 5,..., pn,
ou seja 2 + 3 + 5 + ... + p n + pi = 2α1 3α 2 ... p nα n pi .
α α α
Como 2 + 3 + 5 + ... + p n + pi = 2 1 3 2 ... p n n pi
αn
mmc (2α1 3α 2 ... p nα n pi ,2 ⋅ 3 ⋅ 5 ⋅ ... ⋅ pn ⋅ pi ) = 2 max(α1 ,1) ⋅ 3max(α 2 ,1)... ⋅ p n ⋅ pi aliás,
max( ,1)

como m + 1 ≤ pn αn= 0 ou 1, ou seja max(αn , 1) = 1




Como queremos 102002 primos distintos, se 2 + 3 + ... + p10 2002 não for fatorável nos
os 10 2002 primeiros primos

primos 2,3,..., p102002 ele será da forma m ⋅ pi , i > 10 2002

m < p102002.
Com isso 2 + 3 + ... + p10 2002 + pi = (m + 1) ⋅ pi , que é fatorável em 2,3,5,..., p102002 , pi ,
já que m + 1 ≤ p102002 .
α β
Logo pelo menos um entre 2α1 3α 2 ⋅ ... ⋅ p10102002 ou 2 β1 3 β 2 ⋅ ... ⋅ p10102002 ⋅ pi , com
2002 2002

pi 2+3+...+ 10 2002 , com α i e β i suficientemente grandes são ensolarados, ou seja, há um


p

número ensolarado com 102002 ou 102002 + 1(ou ambos) fatores primos distintos
Alias, esse método prova que para todo t inteiro positivo existe pelo menos um
número ensolarado com t fatores primos ou pelo menos um número ensolarado com t
+ 1 fatores primos (ou ambos).
 x, se x ≥ y
Obs. max( x, y ) = 
 y, se y ≥ x

EUREKA! N°14, 2002

16
Sociedade Brasileira de Matemática

XLIII OLIMPÍADA INTERNACIONAL DE MATEMÁTICA


Enunciados e Resultado Brasileiro

A LXIII Olimpíada Internacional de Matemática foi realizada na cidade de


Glasgow, Reino Unido no período de 18 a 31 de julho de 2002.
A equipe brasileira foi liderada pelos professores Edmilson Motta (São
Paulo – SP) e Ralph Costa Teixeira (Niterói – RJ).

O Resultado da Equipe Brasileira

BRA 1 Alex Corrêa Abreu Bronze


BRA 2 Larissa Cavalcante Queiroz de Lima Prata
BRA 3 Guilherme Issao Camarinha Fujiwara Bronze
BRA 4 Yuri Gomes Lima Bronze
BRA 5 Davi Máximo Alexandrino Nogueira Bronze
BRA 6 Thiago da Silva Sobral Bronze

PRIMEIRO DIA
DURAÇÃO: 4 horas e meia.

PROBLEMA 1
Seja n um inteiro positivo. Seja T o conjunto de pontos (x; y) no plano onde x e y são
inteiros não negativos e x + y < n. Cada ponto de T é pintado de vermelho ou azul.
Se um ponto (x; y) é vermelho, então todos os pontos (x'; y') com x' ≤ x e y' ≤ y
também são. Um conjunto X é um conjunto de n pontos azuis com abcissas todas
distintas, e um conjunto Y é um conjunto de n pontos azuis com ordenadas todas
distintas. Prove que o número de conjuntos X é igual ao número de conjuntos Y.

PROBLEMA 2
Seja BC um diâmetro do círculo Γ de centro O. Seja A um ponto de Γ tal que
0 0 < ∠AOB < 120 0 . Seja D o ponto médio do arco AB que não contém C. A reta que
passa por O e é paralela a DA encontra a reta AC em J. A mediatriz de OA corta Γ
em E e F. Prove que J é o incentro do triângulo CEF.

PROBLEMA 3
Encontre todos os pares de inteiros m, n ≥ 3 tais que há infinitos inteiros positivos a
am + a −1
para os quais n é inteiro.
a + a2 −1

EUREKA! N°14, 2002

17
Sociedade Brasileira de Matemática

SEGUNDO DIA
DURAÇÃO: 4 horas e meia.

PROBLEMA 4
Seja n inteiro maior que 1. Os divisores positivos de n são d1, d2,…,dk , onde

1 = d 1 < d 2 < ... < d k = n

Seja D = d1d2 + d2d3 +…+dk – 1 dk.


(a) Prove que D < n2.
(b) Encontre todos os valores de n para os quais D é um divisor de n2.

PROBLEMA 5
Encontre todas as funções f de 5 em 5 tais que

( f ( x) + f ( z ))( f ( y ) + f (t )) = f ( xy − zt ) + f ( xt + yz )

para todo x, y, z, t ∈ 5.

PROBLEMA 6
Sejam Γ1 , Γ2 ,..., Γn círculos de raio 1 no plano, onde n ≥ 3. Seus centros são O1,
O2,…,On, respectivamente.
Suponha que não exista reta que intercepte mais que dois dos círculos. Prove que
(n − 1)π

1
≤ .
1≤i < j ≤ n Oi O j 4

EUREKA! N°14, 2002

18
Sociedade Brasileira de Matemática

MUROS, PRÉDIOS E ESCADAS


Cícero de Oliveira Holmer, São Paulo – SP

♦ Nível Avançado.

Há um clássico problema de máximos e mínimos cujo enunciado envolve um prédio


(tão alto quanto se queira) e um muro de altura h, à uma distância d deste prédio.
Pretende-se colocar uma escada, apoiada no muro, a partir do solo e alcançando o
prédio, conforme o esquema:

Pergunta-se então o seguinte: Qual é o comprimento mínimo da escada?


Vamos montar um modelo, considerando um triângulo retângulo ABC e um
retângulo APQR inscrito neste triângulo:
C

Q R

θ
B
P A
Sendo PQ = a, QR = b e m( ABˆ C ) = θ , temos:
a b a. cos θ b ⋅ senθ
BC = BQ + QC = + . Assim, BC = f (θ ) e f ' (θ ) = − +
senθ cos θ sen 2θ cos 2 θ
a ⋅ cos θ b ⋅ senθ
Para termos BC mínimo, é preciso que f ' (θ ) = 0, isto é, − + =0
sen 2θ cos 2 θ
b ⋅ senθ a ⋅ cos θ sen 3θ a a
⇔ = ⇔ = ⇔ tgθ = 3 (I).
cos θ
2
sen θ2
cos θ b
3
b
Pelo teorema de Pitágoras, BC 2 = BA2 + AC 2 ⇔ BC 2 = (BP + PA) 2 + ( AR + RC) 2 =

EUREKA! N°14, 2002

19
Sociedade Brasileira de Matemática

2
 a 
=  + b  + (a + b ⋅ tgθ ) 2 . Como BC deve ser mínimo, de (I), temos:
 tgθ 
2
 
  2
    2 2
+ b  +  a + b ⋅ 3  =  3 a 2 ⋅ b + b  +  a + 3 a ⋅ b 2  =
a a
BC = 
2
 b     
 3 a  
 b 
= a ⋅ 3 a ⋅ b2 + 2 ⋅ b ⋅ 3 a2 ⋅ b + b2 + a2 + 2 ⋅ a ⋅ 3 a ⋅ b2 + b ⋅ 3 a2 ⋅ b =
3 3
= a 2 + 3 ⋅ a ⋅ 3 a ⋅ b 2 + 3 ⋅ b ⋅ 3 a 2 ⋅ b + b 2 =  3 a 2 + 3 b 2  ⇔ BC =  3 a 2 + 3 b 2  .
   
3
Portanto, o comprimento mínimo da escada deve ser  3 h 2 + 3 d 2  .
 
Vamos agora considerar uma situação com valores numéricos (talvez você possa
aproveitar melhor o que vem a seguir tendo em mãos papel, caneta e, se possível,
uma boa calculadora).
A partir de um triângulo retângulo bem conhecido, de lados 3, 4 e 5, e outro
triângulo semelhante, por exemplo o de lados 9, 12 e 15, podemos montar a figura:
C

10 m

9m
Q

5m
3m

B 4m P 8m A

Formulamos, então, o seguinte problema: Se o muro tem 3 metros de altura, a


distância do muro ao prédio é igual a 8 metros e a escada tem 15 metros de
comprimento, poderíamos afirmar que a distância do pé da escada ao muro é igual a
4 metros?

EUREKA! N°14, 2002

20
Sociedade Brasileira de Matemática

Vejamos:

O menor comprimento possível da escada é de


3
32 + 3 82 = ( 9 + 4)
3
3

metros.

Pode-se verificar que 3


( )
3
9 + 4 < 15, e isto quer dizer que há duas maneiras
distintas de posicionarmos a escada e, portanto, existem duas distâncias possíveis do
pé da escada ao muro. Vamos então, novamente, montar um modelo:

15

B X P 8 A

Uma solução possível, claro, é x = 4 metros. Busquemos a outra solução:


BP BA x x+8 15 ⋅ x
∆PBQ ~ ∆ABC , logo = ⇔ = ⇔ BQ =
BQ BC BQ 15 x+8
No ∆BPQ temos
2
 15 ⋅ x 
x = BQ − 3 ⇔ x = 
2 2 2 2
 − 9 ⇔ x + 16 ⋅ x − 152 ⋅ x + 144 ⋅ x + 576 = 0
4 3 2

 x+8
Aplicando-se o algoritmo de Briot-Ruffini, obtemos:

1 16 –152 144 576 4


1 20 –72 –144 0

Assim, a outra solução é raiz de x 3 + 20 x 2 − 72 x − 144 = 0.


É possível mostrar que essa equação tem duas raízes reais negativas e uma raiz real
positiva, que é aproximadamente 4,3274534… e pode ser escrita como

EUREKA! N°14, 2002

21
Sociedade Brasileira de Matemática

  − 1567  
 arccos  
4  154 154  −  (ver por exemplo [3] para um método de
 154 ⋅ cos 5
3 3 
 
 
resolução de equações do terceiro e quarto graus).

Assim, as possíveis distâncias do pé da escada ao muro, são de 4 metros e de


  1567  
 arccos −  
4   154 ⋅ 154  −  =
⋅  154 . cos 5  4,3274534... metros.
3  3 
 
 

Referências Bibliográficas

[1] Piskunov N., Cálculo Diferencial e Integral, Tomos I e II, Ed. Mir 1977
[2] Demidovitch B., Problemas e Exercícios de Análise Matemática, Ed. Mir 1978
[3] Moreira, C.G., Uma solução das equações do terceiro e quarto graus, RPM 25,
pp. 23-28.

EUREKA! N°14, 2002

22
Sociedade Brasileira de Matemática

INTEIROS DE GAUSS E INTEIROS DE EISENSTEIN


Guilherme Fujiwara, São Paulo – SP

♦ Nível Avançado.

Vamos abordar nesse artigo a aritmética de dois conjuntos de inteiros algébricos: os


Inteiros de Gauss e os Inteiros de Eisenstein.

1. INTEIROS DE GAUSS
Definimos o conjunto =[i] dos inteiros de Gauss como =[i] = {a + bi | a, b ∈ =},
onde (i2 = –1). A seguir veremos as duas coisas mais importantes de sua aritmética, o
teorema da fatoração única e os primos.

1.1 Norma
Vamos definir uma função N: =[i] → =+ chamada norma, tal que ∀z ∈ =[i], N(z) = z
⋅ z sendo z o conjugado complexo de z. Observe que como ab = a ⋅ b, então
N (a ) ⋅ N (b) = a a ⋅ bb = a ⋅ b ⋅ a ⋅ b = ab ⋅ ab = N (ab) , ou seja, a norma é
multiplicativa.

1.2. Unidades
As unidades em =[i], analogamente a =, são todos os elementos z ∈ =[i] que
possuem inverso, ou seja, que ∃z '∈ =[i ] tal que z ⋅ z ' = 1. Segue que se z = a + bi é
uma unidade, então 1 = N ( z ⋅ z ' ) = N ( z ) ⋅ N ( z ' ) ⇒ N ( z ) = 1 ⇔ a 2 + b 2 = 1 ⇔
⇔ a = ±1, b = 0 ou a = 0, b = ±1 ⇔ = = ±1 ou = = ±i , e como esses quatro tem
inverso, todas as unidades são ± 1 e ± i. Observe então que x ∈ =[i] é unidade
⇔ N ( x) = 1.

1.3. Divisibilidade
Dizemos que para a, b ∈ =[i], a|b (lê-se a divide b) se ∃c ∈ =[i ] tal que b = ac.

1.4. Divisão Euclidiana


Vamos ver como funciona a divisão euclidiana. A divisão Euclidiana é a existência
de q, r ∈ =[i], ∀a, b ∈ =[i], b ≠ 0 tal que a = bq + r, sendo 0 ≤ N (r ) < N (b). Para
demonstrá-la, basta dividir:

a = x + yi, b = z + wi, onde x, y, z, w ∈ =.

EUREKA! N°14, 2002

23
Sociedade Brasileira de Matemática

a x + yi x + yi z − wi xz − xwi + yzi − ywi 2 xz + yw yz − xw


= = ⋅ ⇔ = 2 + i
b z + wi z + wi z − wi z 2 + w2 z + w2 z 2 + w2

xz + yw yz − xw
Tomamos m e n como os inteiros mais próximos de e 2 ,
z +w
2 2
z + w2
xz + yw yz − xw 1
respectivamente. Note que m − ,n− ≤ Se q = (m + ni), então:
z +w 2 2
z +w 2 2
2
a   yz − xw  yz − xw  
r = a − bq = b − q  = b 2 −m+ 2 − n i  ⇒
  z +w z +w 
2 2
b 
 1  2
 1   N (b)
2
⇒ N (r ) ≤ N (b)   +    = < N (b)
 2   2   2
 

1.5. Lema de Euclides


A partir da divisão euclidiana podemos demonstrar o lema de Euclides, ou seja, se p
é um primo de Gauss (ou seja, não pode ser escrito como o produto de dois inteiros
de Gauss cujas normas são maiores que 1), então sendo a, b ∈ =[i], p|ab ⇒ p|a ou
p|b. Para demonstrá-lo, vamos fazer sucessivas divisões euclidianas, sendo a0 = a e
a1 = p. Seja ak + 2 o resto da divisão euclidiana de ak por ak+1. Temos então as
divisões:
a 0 = q1 a1 + a 2
a1 = q 2 a 2 + a 3
a 2 = q3 a3 + a 4

a n − 2 = q n −1 a n −1 + a n
a n −1 = q n a n + a n +1

Observe que como a k ≠ 0 ⇒ N (a k +1 ) < N (a k ) , podemos tomar n tal que N(an +1) =
0, ou seja, an + 1 = 0.
Logo an|an – 1. Observe que a n | a k +1 e a n | a k ⇒ a n | a k −1 . Logo a n | a n e a n | a n −1 ,
então indutivamente, a n | a k , ∀k 0 ≤ k ≤ n, particularmente an | a0 = a e
a n | a1 = p. Tomando as j + 1 primeiras equações e realizando substituições
adequadas, temos que aj = xj a1 + yj a0 = xj p +yj a; particularmente a n = x n p + y n a.

EUREKA! N°14, 2002

24
Sociedade Brasileira de Matemática

Voltando ao lema, veja que se p|a então o lema está certo. Se p não divide a, então,
como a n | p, an|a e an = xnp + yna, então an ∈ {1; – 1; i; – i} e temos:
a n = x n p + y n a ⇔ b = a n−1 ( px n b + aby n ) ⇒ p | b, pois p | ab, o que conclui a
demonstração.

1.6. Fatoração única


A fatoração única é uma das propriedades mais usadas em problemas envolvendo
números inteiros. Vamos prová-la para os inteiros de Gauss. Primeiramente
provaremos que todo inteiro z de Gauss com norma maior que 1 pode ser escrito
como o produto de um ou mais primos de Gauss. Se N(z) = 2, como 2 é primo e a
norma é multiplicativa, então z é primo, portanto está provado. Considere N(z) > 2.
Se z é primo a fatoração é imediata. Se z não é primo, então z = a ⋅ b ⇒ N(z) = N(a) ⋅
N(b), onde N(a), N(b) > 1, portanto N(a), N(b) < N(z). Podemos supor, por indução,
que se N(x) < N(z), então x é fatorável. Logo a e b são fatoráveis, e portanto z.
Para provar que esta fatoração é unica, basta considerar as duas fatorações p1p2…pn e
q1q2…qm . Suponha, por indução, que p1p2…pn = εq1q2…qm, sendo ε uma unidade,
implica que a seqüência (pi) é uma permutação (a menos que sejam multiplicações
por unidades) da (qi). Se max{n; m} = 1, então o resultado é imediato. Supondo que
ele vale se max{n'; m'}< max{n; m}, pelo lema de Euclides, vemos que para algum i,
pn|qi. Sem perda de generalidade, i = m. Como pn e qm são primos, então qm = ε'pn,
onde ε' é uma unidade. Logo p1p2…pn = εq1q2…qm
⇔ p1p2…pn – 1 = εε ' q1 q 2 ...q m −1 . Por indução, p1, p2,...,pn-1 é uma permutação (a
menos que sejam multiplicações por unidades) de q1, q2, …, qm, portanto a fatoração
única está provada.

1.7. Números primos


Vamos agora ver quem são os números primos em Z[i]. Observe que se N(π) é primo
em =, então π é um primo de Gauss (pois se π fatora então N(π) fatora).
Observe que todo primo π divide N(π), portanto ele deve dividir ao menos um fator
primo em = de N(π). Se π dividir ao menos dois números distintos (absolutamente) x
e y primos em =, como sempre é possível tomar a, b ∈ = tal que ax + by = 1,
teríamos π|1, um absurdo. Logo todo primo de Gauss divide exatamente um primo
inteiro positivo (e seu oposto negativo) em =. Seja esse primo inteiro positivo p.
Temos três casos:
Se p é par, então p = 2. Sendo π = a + bi, então a2 + b2 = 2 ⇔ π = ± 1 ± i , e obtemos
os quatro primos 1 + i, 1 – i, –1 + i e –1 –i. Observe que eles são dois a dois um a
multiplicação por uma unidade do outro.

EUREKA! N°14, 2002

25
Sociedade Brasileira de Matemática

Se p ≡ 3 (mód. 4), como x ∈ = ⇒ x2 ≡ 0 ou 1 (mód. 4), então, se existisse π = c +


di, c, d ∈ Z, 1 < N(π) < p2 tal que p = πϕ, é facil ver que, como p é um primo inteiro
ϕ = c – di , logo p = c2 + d2 ≡ 0, 1 ou 2 (mod.4), absurdo, pois p = 4k + 3. Logo p é
um primo de Gauss.
Se p ≡ 1 (mód. 4), então, sendo x = 1 × 2 ×…× ( p – 1)/2, então:
( p − 1) ( p − 1)
x 2 ≡ 1 × 2 × ... × × 1 × 2 × ... ×
2 2
( p − 1) ( p + 1)
≡ 1 × 2 × ... × × × ... × ( p − 2) × ( p − 1) ≡ 1 × ( p − 1)
2 2
≡ −1(mód . p )
Logo p | x 2 + 1 = ( x + i)( x − i). Como π é um primo de Gauss que divide p, então π
∈ Z, π|x + i ou π|x – i ⇒ π|1, absurdo. Portanto π ∉ =[i] tal que p = πϕ. Seja π = a +
bi e ϕ = c + di, a, b, c, d ∈ Z. Como p é primo em z, então mdc(a; b) = mdc(c;d) = 1.
Temos p = (a + bi)(c + di) = ac – bd + (bc + ad)i. Como p ∈ Z, então bc = –ad ⇒ (a
= c e b = – d) ou (a = –c e b = d) ⇔ ϕ = ± π . Como p > 0, então ϕ = π ⇒ N (π ) = p,
logo π é primo (e π e seu conjugado são únicos primos de Gauss que dividem p).
Portanto vimos que os números primos em =[i] são:

(1) O primo 1 + i e seus produtos pelas unidades.


(2) Os primos p em = tal que p ≡ 3 (mod. 4) e seus produtos pelas unidades.
(3) Para cada primo p em =+ tal que p ≡ 1 (mod. 4), os primos a + bi, a – bi e seus
produtos pelas unidades, sendo a2 + b2 = p.

1.8. Ternas pitágoricas


Agora que já vimos a aritmética básica dos inteiros de Gauss, vamos começar com
um resultado simples e interessante. Vamos achar as soluções da equação a2 + b2 =
c2, sendo a, b, c ∈ =. Seja m = mdc(a; b), a' = a/m e b' = b/m. Temos então m2 (a2 +
b2) = c2 ⇒ m|c. Seja então c' = c/m, temos a'2 + b'2 = c'2, mdc(a';b';c') = 1.
Note que a'2 + b'2 = c'2 ⇔ (a'+ b'i)(a'– b'i) = c'2. Observe que se d = mdc(a' + b'i; a'–
b'i), então d|2a' e d|2b' ⇒ d|2. Se d não divide 1, então d|a'2 + b'2 ⇒ a' e b' são
ímpares, o que é um absurdo, basta ver congruência módulo 4. Portanto d|1 ⇔ a' +
b'i e a' + b'i são primos entre si, logo ambos são quadrados perfeitos. Observe
também que quaisquer a'e b' primos entre si tais que a' + b'i e a'– b'i são quadrados
perfeitos são soluções da equação. Portanto a'e b' formam uma solução se e somente
se existem x, y, z, w ∈ = tal que:

EUREKA! N°14, 2002

26
Sociedade Brasileira de Matemática

a '+b' i = ( x + yi) 2 a '+b' i = ( x + yi) 2 a' = x 2 − y 2


⇔ ⇔ a '+b' i = ( x + yi) 2 ⇔
a '−b' i = ( z + wi ) 2 a '−b' i = ( x − yi) 2 b' = 2 xy

Veja então que a' e b' são primos entre si se e só se x e y são primos entre si. Logo as
soluções são a = (x2 – y2) ⋅ d, b = 2xy ⋅ d, ou vice-versa, e conseqüentemente c = (x2 +
y2) ⋅ d, para x, y, d ∈ =, sendo x e y primos entre si.

1.9. O número de representações de um inteiro como a soma de dois quadrados


Provaremos agora o seguinte

Teorema. Dado n ∈ 1 , o número de pares a, b ∈ = tais que n = a2 + b2 é igual a


quatro vezes a diferença entre o número de divisores da forma 4k + 1 de n e o
número de divisores da forma 4k + 3 de n.
Podemos expressar n da forma:
n = 2α p1α ... p αk k × q1β1 (q i ) β1 ...q mβ m (q m ) β m
Sendo pi primos de Gauss inteiros (da forma 4k + 3) e os pares de conjugados qi ≠
q i primos de Gauss (N(qi) da forma 4k + 1) e esses primos diferem dois a dois por
mais do que uma multiplicação por uma unidade.
Sendo n = a2 + b2 = (a + bi)(a – bi), então, pelo teorema da fatoração única e a
multiplicidade do conjugado, temos:
α1 αk
a + bi = ε (1 + i )α p12 ... p k2 × q1γ 1 (q 1 ) β1 −γ 1 ...q mγ m (q m ) β m −γ m , onde 0 ≤ γ i ≤ β i e ε é
uma unidade.
Portanto o número de representações de m como a soma de dois quadrados será 0 se
algum α i for ímpar e será 4(β1 + 1)…(β m + 1) se todos αi forem pares, sendo o fator
4 pois há 4 escolhas possíveis para a unidade.
Observe agora que a fatoração de n em primos inteiros será:
n = 2α p1α1 ... p kα k × N (q1 ) β1 ...N (q m ) β m
Onde pi serão primos da forma 4k + 3 e N(qi) serão primos da forma 4k + 1. Observe
agora que um divisor ímpar de n será da forma:
d = p1a1 ... p kak × N (q1 ) b1 ...N (q m ) bm , onde a1 ≤ α 1 ,..., a k ≤ α k , b1 ≤ β 1 ,..., bm ≤ β m
Note que se a1 + … + ak é par, então d é da forma 4k + 1, se for ímpar é da forma
4k + 3. Portanto, conseguimos verificar que se algum αi for ímpar, o número de d’s
da forma 4k + 3 será igual ao número de d’s da forma 4k + 1, e se todos os αi forem
pares, a diferença entre esses números será (β 1 + 1)…(β m + 1), o que termina a
demonstração do teorema.

EUREKA! N°14, 2002

27
Sociedade Brasileira de Matemática

1.10. Problemas

Problema 1. Determine todos os pares x, y ∈ = tal que y3 = x2 + 1

Problema 2. Sejam x, y, z ∈ 1 tais que xy = z2 + 1. Prove que existem inteiros a, b, c,


d tais que x = a2 + b2, y = c2 + d2 e z = ac + bd.

Problema 3. Prove que existem duas seqüências inteiras (an) e (bn) infinitas e
estritamente crescentes tais que ak(ak + 1) divide bk2 + 1 para todo natural k.

2. INTEIROS DE EISENSTEIN
Vamos agora ver os Inteiros de Eisenstein. Definimos o conjunto =[ω] dos inteiros

de Eisenstein como =[ω] = {a + bω| a, b ∈ =}, sendo ω = −


1 i 3
+ , donde ω2 + ω
2 2
+ 1 = 0. Para ζ = a + bω ∈ =[ω] definiremos a norma como N(ζ) = ζ ⋅ ζ = a2 – ab +
b2. Observe que essa norma segue as mesmas propriedades da norma dos inteiros de
Gauss (é inteira não negativa e multiplicativa).

2.1. Unidades
As unidades em =[ω] são definidas como os seus elementos que possuem inverso,
ou seja u, tal que ∃u – 1 tal que u ⋅ u – 1 = 1 ⇒ N(u) = 1 ⇔ u = ±1, ±ω, ±(1 + ω), e
verificamos que esses quatro números tem inverso, portanto u é unidade se, e só se
N(u) = 1.
Obs. Note que ± (1 + ω ) = ± ω .
2

2.2. Divisão Euclidiana


Para provar a existência de divisão Euclidiana entre a, b ∈ =[ω], b ≠ 0. Sejam α e β
tais que:
a
= α + βω
b
Tomando q = c + dω, tais que c e d são respectivamente os inteiros mais próximos de
α e β. Portanto:
r = a − bq = b(α + βω − q ) ⇒ N (r ) =
1 1 1
= N (b)((α − c) 2 − (α − c)( β − d ) + ( β − d ) 2 ) ≤ N (b) + +  < N (b)
4 4 4
Portanto existe a divisão Euclidiana.

EUREKA! N°14, 2002

28
Sociedade Brasileira de Matemática

2.3. Teorema da fatoração Única


Note que, para os inteiros de Gauss, provamos o lema de Euclides e a fatoração
única, usando somente o fato de que existe divisão Euclidiana, portanto, seguindo os
mesmos passos para provar o lema de Euclides e a fatoração única, provaremos a
fatoração única para os inteiros de Eisenstein.

2.4. Primos
Tudo é muito parecido com os inteiros de Gauss: N(π) é primo em = ⇒ π é primo
em =[ω]; todo primo π em =[ω] divide exatamente um primo inteiro positivo. A
demonstração desses dois fatos é exatamente igual que foi dada na seção de inteiros
de Gauss. Seja p o inteiro positivo primo que o primo, π em Z[ω] divide. Temos três
casos:
• Se p é da forma 3k, então p = 3, e obtemos π = ±(1 – ω) ou ±(2 + ω).
• Se p é da forma 3k + 2, como a2 – ab + b2 só é da forma 3k ou 3k + 1 (verifique
você mesmo), então p é um primo de em Z[ω] tal que N(π) = p2.
• Se p é da forma 3k + 1, pela lei da reprocidade quadrática*:
p −1  −3−1 
 p  − 3  ⋅   − 3
   = (−1) = 1 ⇒   = 1
2  2 

 3  p   p 
Portanto existe x inteiro tal que p | ( x − 1) 2 + 3 = x 2 − 2 x + 2 2 = ( x − 2ω )(x − 2ω 2 ) , e
como p não divide 2, então p não é primo em Z[ω] e existem π e ψ tal que πψ = p.
Como p é um primo inteiro, então ψ = π , logo π e π = ψ são primos em =[ω] e
π ⋅π = p .
Portanto os primos em =[ω] são:
(1) O primo 1 – ω e suas multiplicações por unidades.
(2) Os primos inteiros da forma 3k + 2 e seus produtos pelas unidades, que também
são primos em =[ω].
(3) Para todo primo inteiro p da forma 3k + 1, os primos π e π tal que π π = p e seus
produtos pelas unidades são primos em =[ω].

*A lei de reciprocidade quadrática de Gauss diz o seguinte: dados a ∈ = e p ∈ = primo que não divide
a, definimos  a  = 1, se a é resíduo quadrático mod.p. Para p, q ∈ = primos ímpares com p > 0
 p  0, caso contrário.
  
 p −1   q −1 
vale sempre  p  q  = ( −1) 
  
2  2 
 q  p  .
  

EUREKA! N°14, 2002

29
Sociedade Brasileira de Matemática

2.5. Exemplo
Ache todos os a, b, c ∈ = + lados de um triângulo com um ângulo de 60o.
*

Vamos supor, sem perda de generalidade, que o ângulo de 60o é entre os lados de
medidas a e b. Pela lei dos co-senos, temos:
c 2 = a 2 + b 2 − 2ab cos(60°) = a 2 + b 2 − ab = (a + bω )(a − bω )
Observe a semelhança deste problema com o das ternas pitagóricas.
Seja m = mdc(a; b), a = a'm, b = b'm. Segue que m 2 c 2 ⇔ m c , e teremos c = c'm.
Logo c 2 = a 2 + b 2 − ab ⇔ c' 2 = a' 2 +b' 2 − a' b ', e temos mdc(a'; b'; c') = 1. Seja d tal
que d a '+b' ω e d a '−b' ω . Segue que d 2a ' e d 2b' , logo d 2 . Se d não divide 1,
então 2 d ⇒ 2 a' 2 −a ' b'+b' 2 ⇒ 2 a ' e 2 b' , absurdo, logo d 1 , e portanto:

a' = x 2 − y 2
a '+b' ω = ( x + yω ) 2 = x 2 − y 2 + (2 xy − y 2 )ω ⇔
b' = 2 xy − y 2
Portanto as soluções são a = ( x 2 − y 2 )m, b = (2 xy − y 2 )m e c = ( x 2 − xy + y 2 )m ,
para todo x, y, m ∈ =+ com x > y, e as permutações de a, b e c.

Outro bom exemplo de aplicação dos inteiros de Eisenstein é o problema 6 da IMO


de 2001:
Sejam a, b, c, d inteiros com a > b > c > d > 0. Considere que
ac + bd = (b + d + a − c)(b + d − a + c)
Prove que ab + cd não é primo.
Primeiramente vamos mostrar por que usar inteiros de Eisenstein:
ac + bd = (b + d + a − c)(b + d − a + c) ⇔
⇔ ac + bd = b 2 + bd − ab + bc + bd + d 2 − ad + cd + ab + ad − a 2 + ac − bc − cd + ac − c 2 ⇔
⇔ b 2 + bd + d 2 = a 2 − ac + c 2

Aí vemos por que usar inteiros de Eisenstein.


(b − dω )(b − dω 2 ) = (a + cω )(a + cω 2 )
Observe que como a, b, c > 1, então mdc(b; d) > 1 ⇒ ab + cd não é primo, logo
podemos supor que b e d são primos entre si. Analogamente supomos que mdc(a; c)
= mdc(a; d) = mdc(b; c) =1.

EUREKA! N°14, 2002

30
Sociedade Brasileira de Matemática

Seja π um primo em =[ω ] tal que π b − dω . Vamos provar que π não divide
a + cω ⇔ π não divide a + cω , e segue que b − dω a + cω . Suponha então que
2

π a + cω . Veja que N (π ) (b − dω )(a + cω ), e temos


(b − dω )(a + cω ) = ab + cd + (bc + cd − ad )ω
Como N (π ) ∈ =, e N (π ) (b − dω )(a + cω ) , então N (π ) ab + cd e, supondo ab +
cd primo, teríamos ab + cd = N (π ) . Mas nesse caso segue que
k k
b − dω = επ k ⇒ b − dω 2 = ε ⋅ π ⇒ a + cω = ε ' π , sendo ε e ε ' unidades.
Se µ = ε ' / ε , então b + d + dω = b − dω 2 = µ (a + cω ). Considerando o fato de
mdc(b; d) = mdc(a; c) = mdc(b; c) = 1 e que a > b > c > d > 0, temos que isto é um
absurdo (a verificação fica para o leitor, basta considerar as 6 possibilidades para
µ ) . Logo b − dω a + cω , e analogamente a + cω b − dω . Portanto
b − dω = υ (a + cω ) , onde υ é uma unidade. Novamente, basta verificar todas as
possibilidades para υ e verificar que isto é um absurdo. Portanto ab + cd não é
primo.

2.6. Problemas
Deixamos aqui mais alguns problemas para o leitor:

Problema 1.
(a) Prove que, para cada inteiro n, o número de soluções inteiras de x2 – xy + y2 = n é
finito e divisível por 6.
(b) Determine todas as soluções inteiras de x2 – xy + y2 = 727.

Problema 2.
Mostre que a equação diofantina x3 + y3 + z3 = 0 só tem soluções triviais, ou seja, tais
que xyz = 0.

Problema 3.
Prove que se n é um inteiro positivo tal que a equação x3 – 3xy2 + y3 = n tem soluções
em inteiros (x; y), então ela tem pelo menos três soluções.

EUREKA! N°14, 2002

31
Sociedade Brasileira de Matemática

SEQÜÊNCIAS ARITMÉTICO-GEOMÉTRICAS
José Paulo Carneiro & Carlos Gustavo Moreira

♦ Nível Intermediário.

A) Seqüências Aritmético-Geométricas ( à la Zé Paulo)


Uma progressão aritmética (PA) é uma seqüência tal que cada termo é igual ao
anterior adicionado de uma constante (a razão), isto é, seu termo geral (an) satisfaz à
relação de recorrência a n = a n −1 + r. Daí se deduz, como é conhecido, que
a n = a1 + (n − 1)r .
Analogamente, uma progressão geométrica (PG) é uma seqüência tal que cada termo
é igual ao anterior multiplicado por uma constante (a razão), isto é, seu termo geral
(an) satisfaz à relação de recorrência a n = q a n −1 . Daí se deduz, como é conhecido,
que a n = a1 q n −1 .
Existe um outro tipo de seqüência que aparece freqüentemente, que é uma espécie de
mistura de uma PA e uma PG. É aquela cujo termo geral (an) satisfaz a relação de
recorrência a n = q a n −1 + r , e que vamos chamar de seqüência aritmético-
geométrica (à la Zé Paulo) (SAG), de "razão geométrica" q e "razão aritmética" r.
(só vamos considerar os casos em que r ≠ 0 e q ≠ 1, para não recair numa PA ou
numa PG.)
Observemos que, uma vez conhecido o primeiro termo a1 e a relação de recorrência
a n = q a n −1 + r , conhecem-se sucessivamente a2, a3, etc., e, em princípio, todos os
an. Mas permanece o interesse em determinar uma expressão explícita para o termo
geral de uma SAG em função de n, uma vez conhecidos r, q e a1.
Um exemplo de SAG aparece na solução do célebre problema da Torre de Hanói
(ver [2]), onde são dados três pinos A, B e C, e se quer determinar o número mínimo
de movimentos necessários para se mover do pino A para um dos dois outros pinos
uma pilha de n discos de tamanhos desiguais, de modo que nunca um disco maior
fique em cima de um disco menor. Se an for o número procurado, podemos
raciocinar que, inicialmente, vão ser necessários an – 1 movimentos para mover os n –
1 discos superiores para o pino B, digamos; em seguida, um movimento para mover
o maior de todos os discos para o pino C; e finalmente, mais an – 1 movimentos para
mover a pilha restante para C e completar a operação. Portanto, an satisfaz à relação
de recorrência an = an – 1 + 1 + an – 1 = 2an – 1 + 1, além da condição inicial a1 = 1. A
partir daí, podem ser determinados sucessivamente:
a2 = 2 × 1 + 1 = 3, a3 = 2 × 3 + 1 = 7, e assim por diante.
Uma seqüência constante pode ser considerada uma PA de razão 0 ou uma PG de
razão 1.

EUREKA! N°14, 2002

32
Sociedade Brasileira de Matemática

Uma pergunta interessante é: uma SAG pode ser uma seqüência constante? É claro
que, se c for este valor constante, isto ocorrerá se e só se: c = qc + r, ou seja:
r
c= .
1− q
Vamos agora determinar uma expressão explícita para o termo geral de uma SAG
em função de n. Para isto, consideremos duas SAGs quaisquer, de termos gerais,
respectivamente, an e bn, que tenham a mesma razão aritmética r e a mesma razão
geométrica q, e consideremos a sua diferença dn = an – bn . Como an = q an – 1 + r e
bn = q bn – 1 + r, então dn = an – bn = q (an – 1 – bn – 1) = q dn – 1. Mas isto mostra que
(dn) é uma PG de razão q e, portanto: dn = d1 qn – 1, ou seja: an – bn = (a1 – b1)qn – 1 , ou
ainda: an = bn + (a1 – b1)qn – 1. Como an e bn eram SAGs quaisquer, esta fórmula
indica como qualquer SAG pode ser obtida de outra que tenha a mesma razão
aritmética e a mesma razão geométrica q, podemos tomar bn constante e igual a
r r  r  n −1
, obtendo: a n = +  a1 − q ,
1− q 1− q  1 − q 
que é a expressão que se procurava para an.
Por exemplo, no caso da Torre de Hanói, an = 2an – 1 + 1, com a1 = 1. Portanto:
1  1  n −1
+ 1 −  2 = 2 − 1.
n
1− 2  1− 2 
Vale a pena acrescentar que, se a SAG tiver uma infinidade de termos e |q| < 1, então
r
qn – 1 tende a zero, quando n tende a infinito. Portanto, an tende a .
1− q
2 + a n −1
Por exemplo: a seqüência infinita definida por: a n = , com a1 = 1 tem
5
3 13 63
termos: 1; ; ; ;..., ou, em decimais: 1,000; 0,600; 0,520; 0,504;….
5 25 125
1 2 2 1
Como a n = a n −1 + , esta seqüência é uma SAG com r = e q = , de modo
5 5 5 5
1 1 1 1
que seu termo geral é a n = + , e o seu limite é = 0,5.
2 2 5 n −1 2

B) Seqüências Aritmético-Geométricas ( à la Gugu )


Na seção anterior definimos seqüências aritmético-geométricas (à la Zé Paulo)
generalizando as recorrências de PA's e PG's. Vamos adotar agora um ponto de vista
alternativo, generalizando a fórmula do termo geral de PA's e PG's.

EUREKA! N°14, 2002

33
Sociedade Brasileira de Matemática

Lembremos que em uma PA de termo inicial a0 = a e razão r temos an = a + nr,


∀n ∈  e numa PG de termo inicial a0 = a e razão q, temos an = aqn. Fazemos,
inspirados nessas fórmulas, a seguinte definição:

Definição: Uma progressão Aritmético-Geométrica (PAG) (à la Gugu) é uma


seqüência (an) cujo termo geral satisfaz a fórmula an = (a + nr)qn, ∀n ∈  .
Note que se r = 0 nossa PAG é uma PG, e se q = 1 nossa PAG é uma PA.
É interessante obter uma fórmula para a soma dos n primeiros termos de uma PAG
n −1
(sn = ∑a
j =0
n ), a qual generalizaria as fórmulas para a soma dos n primeiros termos de

PG's (a princípio generalizaria também de PA's, mas suporemos q ≠ 1. Se fizermos q


tender a 1 a fórmula tenderá à fórmula da soma dos n primeiros termos de uma PA).
n −1 s − a n−1 n −2
Temos sn = ∑ (a + jr )q j . Então n = ∑ (a + jr)q j −1 = ∑ (a + ( + 1)r )q  =
j =0 q j =1  =0

∑ "
n−2 n− 2
( q n −1 − 1)
= ∑
 =0
rq  +
 =0
(a + r )q  = r
q −1
+ s n − (a + (n − 1)r )q n −1 e, portanto,

 q −1
sn   = (a + (n −1)r )q n−1 − − r
a q n−1 −1(⇒ sn =
)q 
 (a + (n −1)r −
r a
)q n−1 − +
r 
=
 q  q q −1 q −1  q −1 q q −1
q  qr  n −1 (r − a )q + a
=  a + nr − q + .
q −1 q − 1  (q − 1) 2
aq n a ⋅ (1 − q ) a (q n − 1)
Note que se r = 0 a expressão acima se reduz a + = , que é a
q − 1 (q − 1) 2 q −1
fórmula da soma dos n primeiros termos de uma PG.
Notemos finalmente que SAG's e PAG's satisfazem recorrências lineares
homogêneas (ver [1]): Para SAG's, temos a n + 2 − qa n +1 = r = a n +1 − qa n , logo
a n + 2 = (q + 1) a n +1 − qa n , e para PAG's, notamos que se bn = a n / q n , temos bn =
a n+ 2 2a n −1 an
a + nr, e bn + 2 − bn +1 = r = bn +1 − bn ⇒ bn + 2 = 2bn +1 − bn ⇒ n+2
= n +1
− ⇒ a n+ 2 =
q q qn
= 2qa n +1 − q 2 a n . Os polinômios característicos dessas recorrências são
respectivamente x 2 − (q + 1) x + q = ( x − q )( x − 1) e x2 – 2qx + q2 = (x – q)2.

Referência:
[1] Héctor Soza Pollman, Equações de Recorrência, Eureka! No. 9, pp. 33-40.
[2] Carlos Yuzo Shine, A torre de Hanói, Eureka! No. 11, pp. 17-23.

EUREKA! N°14, 2002

34
Sociedade Brasileira de Matemática

O PRINCÍPIO DA INVARIÂNCIA
Marcelo Rufino de Oliveira, Belém – PA

Artigo baseado em aula ministrada no II Teorema, Fortaleza – CE

♦ Nível Intermediário.

Uma das principais estratégias para resolução de problemas de olimpíadas é a


procura por invariantes. O fundamento do Princípio da Invariância é simples: busca
pelo que se mantém constante quando uma operação permitida é realizada. Entre as
principais formas de invariantes destacam-se três, que serão apresentadas a seguir
através de exemplos resolvidos.

1. Expressões ou Valores Numéricos Invariantes

Exemplo 1.1: Começando com o conjunto {3, 4, 12}, é permitido apagar dois números
a e b e escrever em seus lugares 0,6.a – 0,8.b e 0,8.a + 0,6.b. É possível chegar ao
conjunto {4, 6, 12}?

Resolução:
Repare que (0,6.a – 0,8.b)2 + (0,8.a + 0,6.b)2 = a2 + b2, implicando que a soma dos
quadrados dos números dos conjuntos obtidos é invariante. Como 32 + 42 + 122 = 132
e 42 + 62 + 122 = 142 então não é possível chegar ao conjunto {4, 6, 12}.

Exemplo 1.2: (Maio-99) Em cada um dos 10 degraus de uma escada existe uma rã.
Cada rã pode, de um pulo, colocar-se em outro degrau, mas quando uma rã faz isso,
ao mesmo tempo, uma outra rã pulará a mesma quantidade de degraus em sentido
contrário: uma sobe e outra desce. Conseguirão as rãs colocar-se todas juntas num
mesmo degrau?

Resolução:
Numeremos os degraus de 1 a 10 e associemos a cada rã o número do degrau que
ocupam. O somatório inicial destes valores é S = 1 + 2 + … + 10 = 55.
Perceba agora que este somatório é invariante, pois quando uma rã sobe uma certa
quantidade x de degraus, temos outra rã que desce x, fazendo com que a soma das
numerações destas duas rãs não se altere. Caso todas as rãs ocupem um mesmo
degrau (digamos y), então todas as suas numerações são iguais a deste degrau, ou
seja, teremos 10y = 55, que não possui solução inteira. Deste modo, é impossível que
todas as rãs ocupem um mesmo degrau.

EUREKA! N°14, 2002

35
Sociedade Brasileira de Matemática

Exemplo 1.3: As seguintes operações são permitidas com a equação quadrática


ax2 + bx + c: a) trocar a e c; b) trocar x por x + t, onde t é um número real. Repetindo
estas transformações é possível transformar x2 – x – 2 em x2 – x – 1?

Resolução:
Mostraremos que é invariante o valor do discriminante de todas as equações obtidas
pela aplicação das operações permitidas.
Inicialmente temos ∆0 = b2 – 4ac.
Aplicando a primeira operação: ax2 + bx + c → cx2 + bx + a (1)
Para esta equação temos ∆1 = b2 – 4ca = ∆0
Aplicando a segunda operação:
ax2 + bx + c → a(x + t)2 + b(x + t) + c = ax2 + (b + 2at)x + at2 + bt + c (2)
∆2 = (b + 2at)2 – 4a(at2 + bt + c) = b2 + 4abt + 4a2t2 – 4a2t2 – 4abt – 4ac ⇒
∆2 = b2 – 4ac = ∆0 = ∆1
Como o discriminante de x2 – x – 2 é 9 e o discriminante de x2 – x – 1 é 5, a
transformação é impossível.

2. Restos de Divisões Invariantes

Exemplo 2.1: (Torneio das Cidades-85) Todo membro de uma seqüência, iniciando do
segundo, é igual a soma entre o termo precedente e a soma dos seus dígitos. O
primeiro número é 1. É possível que 123456 pertença à seqüência?

Resolução:
Perceba que: a2 = 2 = 3.0 + 2 a3 = 4 = 3.1 + 1 a4 = 8 = 3.2 + 2
a5 = 16 = 3.5 + 1 a6 = 23 = 3.7 + 2 a7 = 28 = 3.9 + 1 ...
Aparentemente os restos da divisão por 3 dos termos são alternadamente 1 e 2.
Vamos demonstrar isto. Seja S(n) a soma dos dígitos de n. Sabemos que n e S(n)
deixam o mesmo resto na divisão por 3:
i) Se an = 3k1 + 1 ⇒ an + 1 = an + S(an) = 3k1 + 1 + 3k2 + 1 = 3k3 + 2.
ii) Se an = 3k1 + 2 ⇒ an + 1 = an + S(an) = 3k1 + 2 + 3k2 + 2 = 3k3 + 1.
Deste modo, se n é par então an = 3k + 2 e se n é ímpar então an = 3k + 1
(invariante). Como 123456 é divisível por 3 então não pertence à seqüência.

Exemplo 2.2: (Leningrado-85) Três cangurus estão alinhados em uma estrada. A cada
segundo um dos cangurus salta. É permitido que um canguru salte por cima de um
outro canguru, mas não de dois cangurus de uma só vez. Prove que depois de 1985
segundos, os cangurus não podem voltar a ocupar a posição relativa inicial.

EUREKA! N°14, 2002

36
Sociedade Brasileira de Matemática

Resolução:
Existem seis posições para os cangurus: 123 132 312 321 231 213
Note que as posições sublinhadas somente podem ser alcançadas através de um
posição anterior em negrito e vice-versa (invariante). Assim, depois de um número
ímpar de pulos somente as posições sublinhadas (132, 321 e 213) podem ser
alcançadas, fazendo com que depois de 1985 pulos não seja possível que os cangurus
voltem a ocupar a posição inicial (123).

Exemplo 2.3: (Pará-2001) Um tabuleiro 4x4 possui, inicialmente, todas as casas


pintadas de branco. Uma operação permitida é escolher um retângulo consistindo de
3 casas e pintar cada uma das casas da seguinte forma:
– se a casa é branca então pinta-se de preto;
– se a casa é preta então pinta-se de branco.
Prove que, aplicando várias vezes a operação permitida, é impossível conseguirmos
que todo o tabuleiro fique pintado de preto.

Resolução:
Distribuimos as letras a, b e c no tabuleiro da seguinte forma:

a b c a
c a b c
b c a b
a b c a

Note que as letras estão alternadas tanto nas linhas quanto nas colunas. Esta
alternância faz com que toda vez que um retângulo com 3 casas seja selecionado,
então exatamente uma letra a, uma letra b e uma letra c são selecionadas.
Sejam: A a quantidade de casas brancas com a letra a, B a quantidade de casas
brancas com a letra b e C a quantidade de casas brancas com a letra c. No início
temos: A = 6 B = 5 C = 5. Toda vez que selecionamos um retângulo formado de
três casas, estamos somando a cada valor de A, B e C os valores + 1 ou – 1. Perceba
que se todas casas ficarem pretas, então teremos A = 0, B = 0 e C = 0. Entretanto,
note que iniciando de A = 6, B = 5 e C = 5, e alterando simultaneamente por + 1 ou –
1 estes valores, sempre teremos entre os valores de A, B e C dois números ímpares e
um par ou dois pares e um ímpar (invariante), fazendo com que a situação A = 0, B
= 0 e C = 0 seja impossível.

Exemplo 2.4: (OBM Jr.-95) Temos um tabuleiro 1995 × 1995. A cada uma de suas
19952 casas associamos um dos números + 1 ou – 1. Em seguida, associamos a cada

EUREKA! N°14, 2002

37
Sociedade Brasileira de Matemática

linha o produto dos números das casas desta linha, e a cada coluna o produto dos
números das casas de cada coluna.
i) Se T é a soma dos números associados às linhas, colunas e casas, prove que T é
diferente de 0.
ii) Se S é a soma dos números associados às linhas e às colunas, prove que S é
diferente de 0.

Resolução:
i) Como temos 1995 colunas e 1995 linhas, então as somas dos números associados
às linhas e colunas são números ímpares, pois são soma ou subtração de 1995
números ímpares. Em relação às casas temos o mesmo raciocínio, pois são ao todo
19952 casas, cujo valor de cada casa pode ser igual a 1 ou – 1. Somando todos estes
valores teremos também um valor ímpar. Assim, T é a soma de três valores ímpares,
sendo também ímpar, nunca podendo assumir o valor zero.
ii) Inicialmente notemos que qualquer disposição no tabuleiro pode ser alcançada
partindo de uma configuração inicial na qual todas as casas possuem valor 1 e
alterando-se os sinais desejados. Com o tabuleiro contendo somente 1's, temos que
as somas das linhas e colunas é 1995, fazendo uma soma total de 3990, que
obviamente é o maior valor possível. Quando trocamos um 1 por um – 1, notamos
que as somas das linhas e colunas passam a ser 1993, fazendo S = 3986. Assim, fica
evidente que uma alteração de um sinal em uma casa do tabuleiro altera o valor da
soma da linha e da coluna a qual pertence esta casa em ± 2, e por conseqüência altera
a soma total em 0 ou ± 4 (invariante).
Deste modo, a soma total pode ser escrita da forma S = 3990 – 4k.
Caso S = 0, teríamos 4k = 3990, que não possui solução inteira, absurdo.

Exemplo 2.5: (Hong Kong-97) Cinco números 1, 2, 3, 4, 5 estão escritos em um


quadro negro. Um estudante pode apagar dois dos números a e b no quadro e
escrever os números a + b e ab nos seus lugares. Se esta operação é repetida
indefinidamente, podem os números 21, 27, 64, 180, 540 aparecer no quadro negro
ao mesmo tempo?

Resolução:
Não é possível. Note que no início existe somente um número divisível por 3 e no
final existem quatro números divisíveis por 3. Observemos o que acontece com os
restos da divisão por 3 dos números no quadro quando fazemos uma operação:
i) se a = 3x e b = 3y ⇒ a + b = 3z e ab = 3w
ii) se a = 3x + 1 e b = 3y ⇒ a + b = 3z + 1 e ab = 3w
iii) se a = 3x + 2 e b = 3y ⇒ a + b = 3z + 2 e ab = 3w
iv) se a = 3x + 1 e b = 3y + 1 ⇒ a + b = 3z + 2 e ab = 3w + 1

EUREKA! N°14, 2002

38
Sociedade Brasileira de Matemática

v) se a = 3x + 1 e b = 3y + 2 ⇒ a + b = 3z e ab = 3w + 2
vi) se a = 3x + 2 e b = 3y + 2 ⇒ a + b = 3z + 1 e ab = 3w + 1
Portanto, a única forma de aumentar os divisíveis por 3 é escolher a = 3x + 1 e b =
3y + 2. Consequentemente também acrescentamos um número da forma 3k + 2. Por
outro lado, na situação final o único número que não é divisível por 3 é 64, que é da
forma 3k + 1, contradição, pois este número deveria ser da forma 3k + 2 (O caso iii)
mostra que sempre haverá um número da forma 3k + 2 após termos 4 deles da forma
3k).

3. Tendências de Crescimento ou Decrescimento Invariantes

Exemplo 3.1: Um total de 2000 pessoas estão divididas entre os 115 quartos de uma
mansão. A cada minuto, até que todas não estejam em um mesmo quarto, uma
pessoa anda para um quarto com um número igual ou maior de pessoas do que o
quarto que ocupava. Prove que eventualmente todas as pessoas vão estar em um
mesmo quarto.

Resolução:
Seja ai a quantidade de pessoas no quarto i, 1 ≤ i ≤ 115.
Considere a expressão I = a12 + a22 + ... + a115
2
.
Digamos que uma pessoa sai de um quarto que possui n pessoas e vai para um quarto
que possui m pessoas (m ≥ n). A variação de I é:
∆I = ((m + 1)2 + (n – 1)2) – (m2 + n2) = 2(m – n + 1) > 0
Assim, toda vez que uma pessoa troca de quarto o valor de I cresce (tendência de
crescimento invariante). Entretanto note que o valor de I não pode crescer
indefinidamente, uma vez que o número total de pessoas é finito, implicando que
uma hora todas as pessoas estarão em um mesmo quarto.
a
Exemplo 3.2: (2 Lista de Preparação para a Cone Sul-2001) Existem inicialmente n
números 1 em um quadro negro. Em cada passo é permitido apagar quaisquer dois
números a e b e escrever o número ab 2 . Esta operação é feita n – 1 vezes. Prove
a+b
que o último número não é menor que 1 .
n
Resolução:
Suponha que depois de k operações temos os seguintes números escritos no quadro:
a1, a2, ..., an – k. Considere a expressão Ik = 12 + 12 + ... + 21 .
a1 a2 an − k

EUREKA! N°14, 2002

39
Sociedade Brasileira de Matemática

Depois de uma operação a variação de I vale:


2
  2
∆I =  a + b  − 12 − 12 = 1  1 + 1  − 12 − 12 ⇒ ∆I = 1 − 1 2 − 1 2
 2 ab  a b 2a b a b ab 2a 2b
Pela desigualdade entre as médias aritmética e geométrica:
1 + 1
a 2 b2 ≥ 1 1 = 1 ⇒ 1 − 1 − 1 ≤ 0
2 a 2 b 2 ab ab 2a 2 2b 2
Desta forma ∆I ≤ 0, ou seja, I nunca cresce.
Como I0 = n e In – 1 = 12 então In – 1 ≤ I0 ⇒ 12 ≤ n ⇒ a1 ≥ 1 .
a1 a1 n

Exemplo 3.3: (St. Petersburg-96) Vários inteiros positivos distintos estão escritos em
um quadro negro. Uma operação permitida é apagar dois inteiros distintos e escrever
em seus lugares o máximo divisor comum e o mínimo múltiplo comum destes
números. Prove que, depois da aplicação de operações permitidas várias vezes, os
números eventualmente vão parar de mudar.

Resolução:
Suponha que a e b (a > b) são dois números escritos no quadro. Sejam
D = mdc (a, b) e L = mmc (a, b). Consequentemente: D < a, D ≤ b, L ≥ a, L > b.
Como ab = DL ⇒ ab + b2 = DL + b2 (1)
Como (L – b)(D – b) ≤ 0 ⇒ DL – Lb – bD + b2 ≤ 0 ⇒
DL + b2 ≤ bL + bD (2)
Aplicando (2) em (1): ab + b2 ≤ bL + bD ⇒ a + b ≤ D + L (3)
Assim, a soma S dos números escritos no quadro nunca decresce. Repare também
que a igualdade em (3) implica (L – b)(D – b) = 0, ou seja, D = b ⇒ L = a.
Por outro lado, como mdc (x1, x2, ..., xm) ≤ mmc (x1, x2, ..., xm) ≤ x1x2...xm, então,
depois de um número qualquer de operações realizadas, cada número no quadro é
menor ou igual que o produto de todos os números inicialmente escritos. Portanto, S
é menor ou igual a n vezes o produto dos números inicialmente escritos. Como pode-
se fazer um número infinito de operações e S possui ao mesmo tempo as
propriedades de ser limitada e nunca decrescer, então a partir de um certo momento
S fica constante, fazendo com que D = b e L = a, implicando que eventualmente os
números vão parar de mudar.

EUREKA! N°14, 2002

40
Sociedade Brasileira de Matemática

Exercícios:

01. Um círculo é dividido em seis setores. Os números 1, 0, 1, 0, 0, 0 são escritos em


sentido horário. É permitido aumentar em 1 dois números vizinhos. É possível que
em algum momento todos os números sejam iguais?

02. Divide-se um círculo em 10 setores e coloca-se uma ficha em cada setor. Um


movimento consiste em selecionar duas fichas e mover cada uma para um setor
adjacente. Prove que, depois de uma seqüência arbitrária de movimentos, é
impossível que todas as fichas localizem-se em um mesmo setor.

03. (Rio Grande do Norte-99) A professora desafia André e Thiago com o seguinte
jogo, em que eles jogam alternadamente. Ela escreve no quadro-negro os inteiros de
1 a 50. Uma jogada consiste em escolher dois dos números escritos, apagar esses
números, substituindo-os pela soma (Por exemplo, se André escolheu 8 e 23, apaga-
os e escreve 31). Depois de algum tempo, vai restar no quadro negro um único
número. Se esse número é par, o ganhador é André, caso contrário, o ganhador é
Thiago. Quem vence o jogo: André ou Thiago?

04. (Torneio das Cidades-93) Três pilhas de caroços são dadas sobre uma mesa. É
permitido adicionar ou remover de uma pilha um número de caroços que é igual a
soma do número de caroços das outras duas pilhas. Por exemplo [12, 3, 5] pode
tornar-se [12, 20, 5] pela adição de 17 = 12 + 5 para a pilha de 3 ou tornar-se [4, 3, 5]
pela remoção de 8 = 3 + 5 caroços da pilha com 12. É possível, iniciando com pilhas
possuindo 1993, 199 e 19 caroços, conseguir uma pilha vazia depois de uma
seqüência de operações permitidas?

05. (Torneio das Cidades-85) Na ilha de Camelot vivem 13 camaleões roxos, 15


verdes e 17 amarelos. Quando dois de cores distintas se encontram, mudam
simultaneamente para a terceira cor. Poderia dar-se a situação na qual todos tenham
a mesma cor?

06. (Rússia-78) São dadas 3 máquinas que produzem cartões com pares de números
naturais. A primeira, sendo dado o cartão com (a, b), produz um novo cartão com
(a + 1, b + 1); a segunda, sendo dado o cartão com (a, b), produz novo cartão
com (a/2, b/2), se ambos a e b são pares e nada no caso oposto e a terceira máquina,
sendo dados os cartões (a, b) e (b, c), produz um novo cartão com (a, c). Todas as
máquinas retornam também os cartões iniciais. Suponha que foi dado o cartão inicial
(5, 19). É possível obter:
a) (1, 50)?

EUREKA! N°14, 2002

41
Sociedade Brasileira de Matemática

b) (1, 100)?
c) Suponha que foi dado o cartão inicial (a, b) (a < b). Nós queremos obter o cartão
(1, n). Para quais n isto é possível?

07. Em um quadro negro estão escritos n números. A cada minuto apaga-se dois
números a e b e escreve-se o número (a + b)/4. Repetindo esta operação n – 1 vezes,
existirá somente um número no final. Prove que se inicialmente existirem n 1's no
quadro, então o último número não é menor que 1/n.

08. (Leningrado-89) Vários (mas não menos que 2) números não nulos são escritos
em um quadro negro. É possível apagar dois dos números, A e B, e então escrever
nos seus lugares os números A + B/2 e B – A/2. Prove que o conjunto de números
no quadro negro, depois de um número qualquer de operações, não pode coincidir
com o conjunto inicial.
a
09. (2 Lista de Preparação para a Cone Sul-96) 119 anões vivem em uma aldeia com
120 pequenas casas. Uma casa é dita super-habitada se 15 anões ou mais vivem lá.
Todo dia, os anões de uma casa super-habitada têm uma discussão e se mudam para
outras (distintas) casas da aldeia. Algum dia, necessariamente, esse processo se
encerrará?

Bibliografia:

[1] A. Engel, Problem-Solving Strategies, Springer-Verlag, 1998.


[2] P. J. Taylor, Tournament of the Towns 1984-1989, Australian International
Centre for Mathematics Enrichment, 1992.
[3] D. Fomin, A. Kirichenko, Leningrad Mathematical Olympiads 1987-1991, MathPro
Press, 1994.

EUREKA! N°14, 2002

42
Sociedade Brasileira de Matemática

XXII TORNEIO DAS CIDADES


Primavera 2001 – Nível O – Júnior

PROBLEMA 1
O número natural n pode ser trocado por ab se a + b = n, onde a e b também são
números naturais. O número 2001 pode ser obtido a partir de 22 através destas
trocas?

PROBLEMA 2
Um dos segmentos que liga os pontos médios dos lados de um triângulo é maior que
uma das medianas do triângulo. Prove que o triângulo é obtusângulo.

PROBLEMA 3
Vinte quilogramas de queijo estão à venda em uma mercearia e os fregueses estão
em fila para comprar esse queijo. Após algum tempo, tendo acabado de atender um
dos fregueses, a vendedora calcula (corretamente) a quantidade total de queijo já
vendida e anuncia o número de fregueses para os quais há queijo, na quantia exata,
se cada freguês comprar uma porção cuja massa é exatamente igual a quantidade
média comprada pelos anteriores.
Pode ocorrer de a vendedora poder declarar, após cada um dos 10 primeiros
fregueses ter feito sua compra, que há queijo na quantia exata para os próximos 10?
Se isso puder ocorrer, quanto queijo haverá ainda na mercearia após os primeiros 10
fregueses terem feito suas compras?
(A quantidade média de uma seqüência de compras é a massa total de queijo vendida
dividida pelo número de vendas.)

PROBLEMA 4
a) Há 5 triângulos de papel idênticos sobre uma mesa. Cada um pode ser transladado
em qualquer direção. É verdade, então, que qualquer um deles pode ser coberto pelos
outros 4?
b) Há 5 triângulos equiláteros de papel idênticos sobre uma mesa. Cada um pode ser
transladado em qualquer direção. Prove que, então, qualquer um deles pode ser
coberto pelos outros 4.

PROBLEMA 5
Sobre um tabuleiro 15 × 15 são colocados quinze cavalos de modo que em cada
fileira (linha ou coluna) do tabuleiro haja exatamente um cavalo. Então,
simultaneamente, cada cavalo faz um movimento segundo as regras do xadrez.
Prove que após os movimentos haverá dois cavalos em uma mesma fileira do
tabuleiro.

EUREKA! N°14, 2002

43
Sociedade Brasileira de Matemática

Primavera 2001 – Nível O – Sênior

PROBLEMA 1
Um ônibus que percorre um trajeto de 100 km é equipado com um computador, que
prevê quanto tempo falta para chegar ao destino final. Esta previsão é feita
assumindo que a velocidade média do ônibus na parte restante do trajeto será a
mesma da parte já percorrida. Quarenta minutos após a partida do ônibus, o
computador previu que o tempo restante de viagem seria de uma hora. E este tempo
previsto permaneceu inalterado pelas próximas 5 horas.

Isso pode de fato ocorrer? Caso possa, quantos quilômetros o ônibus percorreu
nestas 5 horas?
(A velocidade média do ônibus é o número de quilômetros percorridos dividido pelo
tempo gasto para percorrê-los.)

PROBLEMA 2
A representação decimal do número natural a consiste de n dígitos, enquanto que a
representação decimal de a3 consiste de m dígitos. Pode n + m ser igual a 2001?

PROBLEMA 3
No triângulo ABC o ponto X está sobre o lado AB, enquanto o ponto Y está sobre o
lado BC. Os segmentos AY e CX interceptam-se no ponto Z. Sabe-se que AY = YC e
AB = ZC. Prove que os pontos B, X, Z e Y estão sobre uma circunferência.

PROBLEMA 4
Duas pessoas jogam sobre um tabuleiro 3 × 100. Elas jogam alternadamente: a
primeira coloca dominós 1 × 2 sobre o tabuleiro, a segunda coloca dominós 2 × 1. A
perdedora é aquela que, na sua vez, não puder colocar dominó.
Qual dos dois jogadores têm estratégia vencedora? Descreva-a.

PROBLEMA 5
Nove pontos são desenhados sobre a superfície de um tetraedro regular de aresta 1
cm. Prove que entre estes pontos existem dois cuja distância (no espaço) é menor ou
igual a 0.5 cm.

EUREKA! N°14, 2002

44
Sociedade Brasileira de Matemática

XXIII TORNEIO DAS CIDADES


Outono 2002 – Nível O – Júnior

PROBLEMA 1
É dado um número suficientemente grande de cartões retangulares a cm × b cm,
onde a e b são inteiros positivos e a é menor do que b. Sabe-se que com esses cartões
podemos montar (sem sobrepor cartões e sem buracos) um retângulo 49 cm × 51 cm
e um 99 cm × 101 cm.
Os valores de a e b estão determinados unicamente a partir desses dados?

PROBLEMA 2
Dado um triângulo qualquer, é possível cortá-lo em quatro conjuntos convexos: um
triângulo, um quadrilátero, um pentágono e um hexágono?

PROBLEMA 3
O número x 2 + xy + y 2 , no qual x e y são inteiros positivos, escrito na notação
decimal termina em zero. Prove que ele termina em pelo menos dois zeros.

PROBLEMA 4
Os lados AB, BC, CD e DA do quadrilátero ABCD são tangentes a uma
circunferência nos pontos K, L, M e N, respectivamente; S é o ponto de intersecção
dos segmentos KM e LN. Sabe-se que o quadrilátero SKLB é inscritível.
Prove que o quadrilátero SNDM também é inscritível.

PROBLEMA 5
a) São dadas 128 moedas de duas massas distintas, 64 de cada tipo. Como obter duas
moedas de massas distintas fazendo não mais de 7 pesagens um uma balança com
dois braços (e sem pesos auxiliares)?

b) São dadas oito moedas de duas massas distintas, 4 de cada tipo. Como obter duas
moedas de massas distintas fazendo não mais de duas pesagens em uma balança com
dois braços (e sem pesos auxiliares)?

EUREKA! N°14, 2002

45
Sociedade Brasileira de Matemática

Outono 2002 – Nível A – Júnior

PROBLEMA 1
Sejam a, b, e c as medidas dos lados de um triângulo. Prove a desigualdade
a 3 + b 3 + 3abc >c 3

PROBLEMA 2
Quatro peças (duas brancas e duas pretas) são colocadas em um tabuleiro
quadriculado 23 × 23: as peças brancas são colocadas no canto superior esquerdo e
no canto inferior direito; as peças pretas são colocadas no canto inferior esquerdo e
no canto superior direito. As brancas e pretas se movem alternadamente, sendo que
as brancas começam. Em cada movimento, uma peça é movida para qualquer casa
vizinha (isto é, que tem uma lado em comum) que não tenha peça. O objetivo das
peças brancas é ocupar duas casas vizinhas. As pretas podem evitar que isso
aconteça?

PROBLEMA 3
Sejam E e F os pontos médios dos lados BC e CD, respectivamente, do quadrilátero
convexo ABCD. Os segmentos AE, AF e EF dividem o quadrilátero em 4 triângulos
cujas áreas (em alguma ordem) são inteiros positivos consecutivos.
Qual é a maior área que o triângulo ABC pode ter?

PROBLEMA 4
N lâmpadas estão enfileiradas. Inicialmente, algumas delas estão acesas. A cada
minuto, todas as lâmpadas acesas são apagadas e acendemos cada lâmpada apagada
que for vizinha a exatamente uma lâmpada que estava acesa. Para que valores de n é
possível escolher uma configuração inicial de lâmpadas de modo que em qualquer
momento pelo menos uma lâmpada esteja acessa?

PROBLEMA 5
Um triângulo acutângulo é cortado por uma reta em dois pedaços (não
necessariamente triangulares). Em seguida, um dos pedaços é cortado por uma reta
em dois pedaços, e assim por diante: a cada passo um dos pedaços obtido em
qualquer passo anterior é escolhido e cortado por uma reta em dois novos pedaços.
Após um certo número de passos, todos os pedaços são triangulares. É possível que
todos tenham um ângulo obtuso?

EUREKA! N°14, 2002

46
Sociedade Brasileira de Matemática

PROBLEMA 6
Em uma seqüência crescente de inteiros positivos, cada termo, a partir do 2002-
ésimo, divide a soma de todos os termos anteriores. Prove que cada termo da
seqüência, a partir de um certo ponto, é igual à soma de todos os anteriores.

PROBLEMA 7
Dada uma cadeia de dominós, montada de acordo com as regras usuais, é permitida a
seguinte operação: escolha uma subcadeia contida nela cujas extremidades são iguais
(têm o mesmo número de pontos marcados), retire-a, inverta sua ordem e recoloque.
Mostre que, dadas duas cadeias montadas a partir de um mesmo conjunto de
dominós e com extremidades respectivamente iguais (isto é, os inícios são iguais
entre si, bem como os finais), é possível transformar, através de uma seqüência de
operações como a descrita, uma das cadeias na outra.

EUREKA! N°14, 2002

47
Sociedade Brasileira de Matemática

OLIMPÍADAS AO REDOR DO MUNDO


Þ Como sempre acontece no período em que é realizada a maior parte das
competições internacionais o comitê editorial da revista EUREKA! se preocupa em
mostrar as resoluções das competições nacionais dos anos anteriores para o
treinamento dos nossos atuais e futuros competidores. Por isto, estivemos ausentes
da edição anterior.
Continuamos a disposição na OBM para aqueles que estiverem interessados
na solução de algum problema particular. Para tanto, basta contactar a OBM, através
de carta ou e-mail.
Antonio Luiz Santos

151. (Irlanda-2001) Seja ABC um triângulo de lados BC , CA, AB cujas medidas são
respectivamente iguais a a, b, c. Se D e E são os pontos médios de AC e AB
respectivamente, mostre que a mediana BD é perpendicular a CE se, e somente
se, b 2 + c 2 = 5a 2 .

152. (Irlanda-2001) Mostre que se um número primo ímpar p pode ser colocado sob a
forma x 5 − y 5 para alguns inteiros x e y então
4 p + 1 v2 + 1
=
5 2
para algum inteiro ímpar v.

153. (Irlanda-2001) Determine os números reais x não negativos para os quais


3
13 + x + 3 13 − x
é um número inteiro.

154. (Irlanda-2001) Determine todas as funções f : 1 → 1 que satisfazem


f (x + f ( y )) = f (x ) + y, ∀x, y ∈ N.

155. (Rússia-2001) Os números de 1 a 999 999 são divididos em dois grupos: no


primeiro, cada número nele colocado é tal que o quadrado perfeito mais próximo
dele é o quadrado de um número ímpar. No segundo, os números estão mais
próximos de quadrados perfeitos de números pares. Determine em qual dos
grupos a soma dos números a ele pertencentes é maior.

EUREKA! N°14, 2002

48
Sociedade Brasileira de Matemática

156. (Rússia-2001) Sobre o maior lado AC de um triângulo ABC , toma-se um ponto


N de modo que as mediatrizes dos segmentos AN e NC intersectam os lados
AB e BC nos pontos K e M, respectivamente. Prove que o circuncentro O do
triângulo ABC pertence ao círculo circunscrito ao triângulo KBM.

157. (Rússia-2001) Dois círculos são tangentes internamente no ponto N . Uma


tangente traçada de um ponto K do círculo interno intersecta o círculo externo
nos pontos A e B . Se M é o ponto médio do arco AB que não contém o ponto
N , mostre que o raio do círculo circunscrito ao triângulo BMK não depende da
escolha do ponto K do círculo interior.

158. (Rússia-2001) A seqüência (xn ) é tal que x1 = 1 , xn +1 = n ⋅ sen xn + 1 . Mostre que


esta seqüência não é periódica.

159. (Eslovênia-2001) Sejam a1 , a 2 , a3 , a 4 e a5 números reais distintos. Denotando por


m o número de valores distintos das somas ai + a j , onde 1 ≤ i ≤ j ≤ 5 ,
determine o menor valor possível de m .

160. (Eslovênia-2001) Sejam a, b, c, d , e e f números reais positivos tais que a


seqüência (a, b, c, d ) seja aritmética e a seqüência (a, e, f , d ) seja geométrica.
Mostre que bc ≥ ef .

161. (Eslovênia-2001) Seja D o pé da altura relativa ao lado BC do triângulo ABC .


Sabendo que a bissetriz interna do ângulo ∠C intersecta o lado oposto no ponto
π
E e que ∠CEA = , determine a medida do ângulo ∠EDB .
4

162. (Eslovênia-2001) (a) Mostre que a desigualdade


1
n +1 − n < < n − n −1
2 n
é verdadeira para todo inteiro positivo n .

(b) Mostre que a parte inteira da expressão


1 1 1 1
1+ + + ⋅⋅⋅ + +
2 3 m −1
2
m2
onde m é um inteiro positivo é igual a 2m − 2 ou 2m − 1 .

EUREKA! N°14, 2002

49
Sociedade Brasileira de Matemática

163. (Croácia-2001) Resolva a inequação


x1+ log a x > a 2 x, a > 0, a ≠ 1

164. (Croácia-2001) A partir dos pontos médios dos lados de um triângulo acutângulo
são traçadas perpendiculares aos outros dois lados. Mostre que a área do
hexágono definido por esses segmentos é igual à metade da área do triângulo.

165. (Croácia-2001) Determine todos os pares ordenados de números reais x, y que


satisfazem à equação
 
2
 1
log 2 2 cos 2 (xy ) +
1
 − 1 = − y − 
 2 cos 2 (xy )  2

166. (Croácia-2001) Quatro esferas de raios iguais a R são mutuamente tangentes entre
si. Determine o raio r da maior esfera que pode ser inserida entre elas.

167. (Bielorússia-2001) No gráfico da parábola y = x no plano cartesiano marcamos os


2

pontos A, B e C (com A entre B e C ). No segmento BC marca-se o ponto


N de modo que AN seja paralelo ao eixo das ordenadas. Se S1 e S 2 são as
áreas dos triângulos ABN e ACN , respectivamente, determine a medida do
segmento AN.

168. (Bielorússia-2001) A comissão organizadora da OBM preparou algumas variantes


de uma competição com 4 problemas cada uma. Duas variantes distintas podem
conter um mesmo problema mas não mais do que um. Qual o menor número de
problemas necessários para que a comissão organizadora prepare 10 variantes
da competição?

169. (Bielorússia-2001) O quadrilátero ABCD está inscrito num círculo S1 ; um outro


círculo S 2 passa pelo ponto D , pelo ponto O , de interseção das diagonais do
quadrilátero, e intersecta AD e CD em M e N, respectivamente. Sabendo que
OM e AB intersectam-se em R enquanto que ON e BC intersectam-se em
T ( R , T e A pertencem ao mesmo semiplano em relação a BC ), mostre que
O, R, T e B pertencem ao mesmo círculo.

EUREKA! N°14, 2002

50
Sociedade Brasileira de Matemática

170. (Bielorússia-2001) Determine o resto da divisão de


( )
1! ⋅ 5 + 2! ⋅ 11 + ⋅ ⋅ ⋅ + k! ⋅ k 2 + 3k + 1 + ⋅ ⋅ ⋅ + 200! ⋅ 40601
por 2004 .

171. (Inglaterra-2001) Determine todos os números naturais N de dois algarismos para


os quais a soma dos algarismos de 10 N − N é divisível por 170 .

172. (Inglaterra-2001) Um círculo S é interior a um círculo T e o tangencia no ponto


A . De um ponto P ≠ A sobre T , traçam-se as cordas PQ e PT de T que
tangenciam S em X e Y, respectivamente. Mostre que ∠QAR = 2∠XAY .

173. (Inglaterra-2001) Uma seqüência (an ) é tal que a n = n + n , onde n é um


inteiro positivo e x é o inteiro mais próximo a x , sendo as metades
arredondadas para cima se for necessário. Determine o menor inteiro k para os
quais os termos ak , ak +1 ,..., ak + 2000 formam uma seqüência de 2001 inteiros
consecutivos.

174. (Inglaterra-2001) As medidas dos lados de um triângulo são a, b, c e a medida do


raio do círculo circunscrito ao triângulo é R . Mostre que o triângulo é retângulo
se, e somente se, a 2 + b 2 + c 2 = 8R 2 .

175. (Torneio das Cidades-2001) Diga se existem inteiros positivos a1 < a2 < ... < a100 tais
que para todo k , 2 ≤ k < 100 , o mínimo múltiplo comum de ak −1 e ak é maior
que o mínimo múltiplo comum de ak e ak +1 .

176. (Torneio das Cidades-2001) Os vértices de um triângulo têm coordenadas (x1 , y1 ) ,


(x2 , y2 ) e (x3 , y3 ). Para quaisquer inteiros h e k , o triângulo cujos vértices têm
coordenadas (x1 + h, y1 + k ) , (x2 + h, y2 + k ) e (x3 + h, y3 + k ) é disjunto do
triângulo original.
1
(i) É possível que a área deste triângulo seja maior que ?
2
(ii) Qual a área máxima deste triângulo?

EUREKA! N°14, 2002

51
Sociedade Brasileira de Matemática

177. (Torneio das Cidades-2001) Sejam a e d inteiros positivos tais que, para qualquer
inteiro positivo n , a expansão decimal de a + nd contém um bloco de
algarismos consecutivos igual à expansão decimal de n. Prove que d é uma
potência de 10.

178. (Áustria-Polônia-2001) Determine o número de inteiros positivos a para os quais


existem inteiros não negativos x0 , x1 ,..., x2001 que satisfazem a
2001
a x0 = ∑a
k =1
xk

179. (Estônia-2001) Os ângulos de um n − ágono convexo são α, 2α, ... , nα .


Determine todos os valores possíveis de n e os valores correspondentes de α .

180. (Estônia-2000) Mostre que para todo inteiro a > 1 , existe um número primo p tal
que 1 + a + a 2 + ⋅ ⋅ ⋅ + a p −1 é composto.

ÞÞÞ
Acusamos o recebimento de soluções de problemas anteriores dos seguintes leitores
de EUREKA!:. No próximo número publicaremos algumas delas.

Alessandra A. da Gama Gomes de A. Rio de Janeiro – RJ Prob. 119.


Anderson Torres São Paulo – SP Prob. 4, 16, 28, 31, 36, 46, 47, 53, 57, 59, 65,66, 68, 78, 80, 85, 88,
89, 95, 104, 109, 121, 123, 130,135,137, 145.
Bruno Borges de Souza Lima Goiânia – GO Prob. 102, 107, 147.
Bruno de Souza Ramos Rio de Janeiro – RJ Prob. 127, 128, 131, 132, 134, 138, 139, 143, 144.
Carlos José Amorim da Silva Rio de Janeiro – RJ Prob. 133, 138, 143.
Diego Alonso Teixeira Santos – SP Prob. 121.
Evandro Makiyama de Melo São Paulo – SP Prob. 51, 121, 123, 128, 131, 133, 135, 142, 147.
Filipe Rodrigues de Souza Moreira Rio de Janeiro – RJ Prob. 64, 121, 128, 131, 133, 135, 138, 145, 147.
Geraldo Perlino Jr. Cotia – SP Prob. 121, 122, 125 a 139, 141 a 144, 146 a 148.
Gibran Medeiros de Souza Natal – RN Prob. 138.
Helainy Ignácio de Almeida Torres Rio de Janeiro – RJ Prob. 135, 137, 147.
Helder Oliveira de Castro Mogi das Cruzes – SP Prob. 32, 57, 68, 69, 76, 87, 98, 121,127, 132, 138, 139.
Karla Detagne Santos Rio de Janeiro – RJ Prob. 128, 138.
Leonardo Freitas de Lima Rio de Janeiro – RJ Prob. 138.
Luiz Sérgio Carvalho de Mello Rio de Janeiro – RJ Prob. 8, 47, 69, 127, 128, 132, 143.
Marcelo Ribeiro de Souza Rio de Janeiro – RJ Prob. 121, 122, 124, 126, 127, 128, 130, 132, 133, 134, 138, 143,
147.
Marcelo Rufino de Oliveira Belém – PA Prob. 121 a 150.
Marcílio Miranda de Carvalho Teresina – PI Prob. 66, 87, 106.
Mauro Félix de Souza Rio de Janeiro – RJ Prob. 121, 128, 134.
Okakamo Matsubachi São Paulo – SP Prob. 130.
Raquel Teresa de Souza Gomes Rio de Janeiro – RJ Prob. 128, 133, 143.
Renato Francisco Lopes Mello J. dos Guararapes – PE Prob. 127, 132, 143.
Wallace Alves Martins Rio de Janeiro – RJ Prob. 121, 128, 130, 133, 138.

EUREKA! N°14, 2002

52
Sociedade Brasileira de Matemática

SOLUÇÕES DE PROBLEMAS PROPOSTOS


 Publicamos aqui algumas das respostas enviadas por nossos leitores.

57) Dado n números reais x1, x2 …, xn satisfazendo as condições x1 +…+ xn = 0 e


1
x12 + ... + x n2 = 1 , prove que existem i e j tais que xi x j ≤ − .
n

SOLUÇÃO DE ZOROASTRO AZAMBUJA NETO (RIO DE JANEIRO – RJ):


Podemos supor que x1 ,..., x k ≥ 0, x k +1 ,..., x n < 0.
k n
n−k k
Temos ∑ i =1
xi2 + ∑x
j = k +1
2
j =1=
n
+ .
n
k
n−k n

∑ ∑x
k
Assim, xi2 ≥ ou 2
j ≥ . Supomos sem perda de generalidade que
i =1 n j = k +1 n
k
n−k
∑x
i =1
2
i ≥
n
(o outro caso é análogo). Assim, se xi ≤ ε para 1 ≤ i ≤ k , temos

k
n−k k k n k


i =1
xi ≥
εn
, pois ∑
i =1
xi2 ≤ ε ∑
i =1
xi . Como ∑
j = k +1
(− x j ) = ∑x ,
i =1
i temos

n
n−k
∑ (− x
1
)≥ , donde existe k + 1 ≤ j 0 ≤ n com − x j0 ≥ . Tomando
εn εn
j
j = k +1

1
ε = max x i , temos que existe i0 ≤ k com xi0 = ε , donde − x i0 x j0 = ε (− x j0 ) ≥ ,e
1≤ i ≤ k n
1
logo xi0 x j0 ≤ − .
n

63) Prove que existem infinitos números naturais múltiplos de 51000 sem nenhum 0
na representação decimal.

SOLUÇÃO DE EDUARDO CASAGRANDE STABEL (PORTO ALEGRE – RS):


Provaremos por indução que existe um número de k algarismos, todos diferentes de
0 (zero), divisível por 5k. Para k = 1, tome a1 = 5; para k = 2, tome a2a1 = 25. Agora
suponhamos que seja n = ak – 1ak – 2…a2a1 divisível por 5k – 1.

EUREKA! N°14, 2002

53
Sociedade Brasileira de Matemática

a k a k −1 ...a 2 a1 a k 10 k −1 + a k −1 ...a 2 a1 n
Temos k −1
= k −1
= a k 2 k −1 + k −1
. É preciso que
5 5 5
n
a k 2 k −1 + k −1
≡ 0 módulo 5 para que ak ak – 1…a2a1 seja múltiplo de 5k. A equação é
5
n
equivalente a a k ≡ a k 2 k −13 k −1 ≡ − k −1
3 k −1 módulo 5.
5
Podemos escolher um tal ak no conjunto {1, 2, 3, 4, 5}, pois aí temos um conjunto de
restos da divisão por 5. Para concluir, basta tomar k = 1000, 1001, 1002,…

64) Iniciando de um certo inteiro positivo, é permitido fazer apenas uma


operação: o dígito das unidades é separado e multiplicado por 4, e então este
valor é somado ao restante do número. Por exemplo, o número 1997 é
transformado para 7.4 + 199 = 227. A operação é feita repetidamente. Prove
que se a seqüência de números obtida contém 1001, então nenhum dos
números na seqüência pode ser um número primo.

ADAPTAÇÃO DA SOLUÇÃO DE MARCELO RIBEIRO DE SOUZA (RIO DE JANEIRO – RJ):


Vamos ver que todos os termos da seqüência são múltiplos de 13, caso a seqüência
contenha 1001 (que é múltiplo de 13). De fato, se bn = 10kn + rn , com 0 ≤ rn ≤ 9, o
próximo termo será bn + 1 = kn + 4rn, ou seja, bn = 10(bn +1 − 4rn ) + rn = 10bn +1 − 39rn .
Assim, bn é múltiplo de 13 ⇔ bn + 1 é múltiplo de 13, ou nenhum deles é. Como os
termos antes de 1001 são maiores que 1001 e, partindo de 1001, obtemos a
seqüência 1001 → 104 → 26 → 26 → …, todos os termos da seqüência são
múltiplos de 13, mas nenhum é igual a 13, donde nenhum é primo.

65) Determine todos os inteiros N tais que, em base 10, os dígitos de 9N são os
mesmos dígitos de N na ordem inversa, e N possui no máximo um dígito igual
a 0.

SOLUÇÃO DE DANIEL DE SOUZA RAMOS (PIRASSUNUNGA – SP):


Seja N um número de n dígitos na base 10, com no máximo um dígito igual a 0.
Sabemos pelo enunciado definição do problema que 9N tem n dígitos. Daí temos que
ao multiplicarmos N por 9, 9N só terá n dígitos se o primeiro dígito de N for 1.
Isto implica que o último dígito de 9N é 1. Mas tal fato só será possível se o último
dígito de N for 9. Assim N = 1…9 e 9N = 9…1.
Imaginemos a multiplicação por 9. Para que o primeiro algarismo de 9N seja 9 é
necessário que o dígito anterior ao dígito 1 de N seja 0 ou 1, uma vez que qualquer
dígito maior do que 1, em tal posição, fará com que 9N tenha um número de dígitos
diferente do de N.

EUREKA! N°14, 2002

54
Sociedade Brasileira de Matemática

Analisemos então os 2 casos:


1o. Caso) N = 11…9 (9N = 9…11); 0 ≤ k ≤ 9, k ∈ 1
8
11... k 9 9k + 8 ≡ 1(mod 10)
×9 k ≡ 7(mod 10) ⇒ k = 7

9k...11

Daí: N = 11…79(9N = 97…11), 0 ≤ k', k'' ≤ 9, k', k'' ∈ 1


k '' k' 8
1 1 ... .7 9 9 ⋅ 1 + k ' ≡ 7(mod 10)
×9 k ' ≡ −2 ≡ 8(mod 10) ⇒ k ' = 8 ⇒ k ' ' = 1

9 7....1 1

Porém com k'' = 1 não será possível que 9N seja da forma 97…11. Portanto o caso N
= 11…9 não satisfaz.

2o. Caso) N = 10…9 (9N = 9…01); 0 ≤ k ≤ 9, k ∈ 1


8
10... k 9 9k + 8 ≡ 0(mod 10)
×9 k ≡ 8(mod 10) ⇒ k = 8

9k...01

Daí: N = 10…89(9N = 98…01), 0 ≤ k', k'' ≤ 9, k', k'' ∈ 1 (4)


8* 8 8
1 0 k ....k ' 8 9 Para que esta situação aconteça, k = 8
×9 ou k = 9 (pois apenas 9 ⋅ 8 pode, somado a 8 chegar a 80,

9 8 k '....k 0 1 e 9 ⋅ 9 = 81).

Para o caso k = 8 [k = 8 ⇒ k' = 0 pois 9k' + 8 ≡ 8 (mod 10) ∴ k' ≡ 0(mod 10)]
8 8 8 8
1 0 8 ... .0 8 9 Neste caso o número N possuiria mais de um dígito
×9 0, não condizendo com a situação proposta.

9 8 0....8 0 1

Para o caso k = 9:

EUREKA! N°14, 2002

55
Sociedade Brasileira de Matemática

8 8 8
1 0 9... .k ' 8 9 9k '+8 ≡ 9(mod 10)
×9 k ' ≡ 9(mod 10) ⇒ k ' = 9

9 8 k '....9 0 1

Temos agora que N = 1 0 9…9 8 9. Desenvolvendo:


8 8 8 8
1 0 9 k ... k ' 9 8 9
Para que esta situação aconteça
×9
⇒ k = 8 ou k = 9
9 8 0k '....k 9 0 1

Para o caso k = 8.
8 8 8
1 0 9 8... k ' 9 8 9
9k' + 8 ≡ 8(mod 10)
×9
⇒ k ' ≡ 0(mod 10) ⇒ k ' = 0
9 8 9 k '....8 9 0 1

Não satisfaz o problema pois possui mais de um dígito 0.

Para o caso k = 9.
8 8 8 8 8
1 0 9 9... k ' 9 8 9
9k' + 8 ≡ 9(mod 10)
×9
⇒ k ' ≡ 9(mod 10) ⇒ k ' = 9
9 8 9 k '....9 9 0 1

Temos agora que N = 1 0 9 9 … 9 9 8 9. Desenvolvendo:


8 8 8 8 8 8 8
1 0 9 9 k ... k ' 9 9 8 9
×9
9 9 8 9 k '....k 9 9 0 1
Procedendo de forma análoga ao momento em que sabia-se que N = 1 0 9 … 9 8 9,
teremos N = 1 0 9 9 9… 9 9 9 8 9.
Conclui-se então que todos os inteiros N que satisfazem a condição do problema são
da forma:
N = 10 9, ...9 89, n ∈ 1 ∧ n ≥ 4 .
( n − 4 ) dígitos

EUREKA! N°14, 2002

56
Sociedade Brasileira de Matemática

67) Este problema saiu com o enunciado errado, devido a um erro tipográfico,
como observaram Rodrigo Villard Milet, Anderson Torres e Carlos da Silva
Victor, que enviaram versões corrigidas, com soluções. Pedimos desculpas
aos leitores pelo erro e publicamos as versões corrigidas a seguir:

VERSÃO DE RODRIGO VILLARD MILET (RIO DE JANEIRO – RJ) e


ANDERSON TORRES (SÃO PAULO – SP):
Seja ABCD um quadrilátero tal que os círculos circunscritos aos triângulos ABC e
BCD são ortogonais. Prove que os círculos circunscritos aos triângulos ACD e ABD
também são ortogonais.

SOLUÇÃO:
Vamos denotar por (XYZ) o círculo que passa por X, Y e Z e por ϑ(w1, w2) e ângulo
entre as curvas w1 e w2.
Considere uma inversão ψ com centro em B e de razão k. Como (ABC) e (BCD)
passam por B, ψ((ABC)) = A' C' e ψ((BCD)) = C'D'. Como ϑ((ABC), (BCD)) = 90º,
segue que ∠A'C'D' = 90º.
Temos ψ (( ACD)) = ( A' C ' D' ). Daí, A'D' é diâmetro de (A'C'D'), pois ∠A'C'D' =
90º. E assim, ϑ((A'C'D'), A'D') = 90º, logo ϑ(ψ ((A'C'D')), ψ (A'D')) = ϑ((ACD),
(ABD)) = 90º, pois ψ (A'D') = (ABD)(A'D' não passa por B). Então os círculos (ACD)
e (ABD) são ortogonais.

VERSÃO DE CARLOS ALBERTO DA SILVA VICTOR (NILÓPOLIS – RJ):


Seja ABCD um quadrilátero tal que os círculos circunscritos aos triângulos ABC e
ACD são ortogonais. Prove que os círculos circunscritos aos triângulos BCD e DAB
também são ortogonais.

SOLUÇÃO:
Sejam O1, O2, O3 e O4 os centros dos círculos circunscritos aos triângulos ABC,
ADC, ADB e BDC respectivamente.
Sejam: ∠O1CB = a ; ∠O2 CD = b ; ∠O1CA = θ ; ∠DBC = e ; ∠DBA = d ;
∠BDA = c e ∠BDC = f . Note que, pela hipótese do problema,
∠O2 AC = ∠O2 CA = 90° − θ . Podemos concluir que
Cˆ = ∠O1CB + ∠O1CA + ∠O2 CA + O2 CD = a + θ + 90° − θ 1b = a + b + 90° .
Além disso, é fácil ver que ∠O1 AB = 90° − a ⋅ θ e ∠O2 AD = θ − b, portanto
∠O1 AB + ∠O1 AC + ∠O2 AC + ∠O2 AD = (90° − a − θ ) + θ + (90° − θ ) + θ − b =
= 180° − a − b. Veja também que ∠DO4 O3 = e + f e ∠DO3 O 4 = c + d . Observe
agora no triângulo DO3 O4 em que e + f + c + d = 180° – ∠O3 DO4 e, pelo

EUREKA! N°14, 2002

57
Sociedade Brasileira de Matemática

quadrilátero ABCD temos Aˆ + Bˆ + Cˆ + Dˆ = 360° , ou seja e + f + c + d = 360° –


(Cˆ + Aˆ ) = 360° – [90° + a + b + 180° – a – b] ∴ e + f + c + d = 360° – 270° = 90°.
Conclusão: ∠O3 DO4 = 180° − ∠DO4 O3 − ∠DO3O4 = 180° − (e + f + c + d ) = 90°
e os círculos circunscritos aos triângulos BCD e DAB também são ortogonais.

Agradecemos também o envio das soluções e a colaboração de:

Fábio Nunes Ribeiro Maia Rio de Janeiro – RJ


Anderson Torres São Paulo – SP
Fábio Dias Moreira Rio de Janeiro – RJ
Marcelo Rufino de Oliveira Belém – PA
Helder Oliveira de Castro Mogi das Cruzes – SP
Oswaldo Mello Sponquiado Olimpia – SP
Renato Francisco Lopes Mello Jaboatão dos Guararapes – PE
Marcílio Miranda de Carvalho Teresina – PI
Evandro Makiyama de Melo São Paulo – SP
Gibran Medeiros de Souza Natal – RN
Carlos A. Gomes Natal – RN
Jorge Silva Júnior Cachoeiro de Itapemirim – ES

Seguimos aguardando o envio de soluções do problema proposto Nº. 66 publicado na revista


Eureka! Nº. 12

9RFrVDELD«
&RQVLGHUH XP ELOKmR GH Q~PHURV GLVWLQWRV HVFULWRV FDGD XP HP XP GH XP
 
   


   
   
 

 

            
     

!" # 
         !  



   

$ 

  

 

%


  
 &


  

 
 

  





 

  '()&    

   V  
  V 
*

  Q (  + ,-'.,./ 
 
  0  

     *   % 
   
 
  

   *
 '(H 



1  

 %   
 
    
2345
1  
   %  6
   

*      




 %   7
"  
     
    
    %8 
 
  '6    9    
 
 
   

* 
*

   
          


*
  
8 :



      
 

&

$ SUREDELOLGDGH GH DFHUWDU p DOWD PDLRU TXH   &DOFXOHD

EUREKA! N°14, 2002

58
Sociedade Brasileira de Matemática

PROBLEMAS PROPOSTOS
Convidamos o leitor a enviar soluções dos problemas propostos e sugestões de novos
problemas para os próximos números.

68) Seja ABC um triângulo de lados inteiros e área racional. Prove que existem
pontos X, Y, Z com coordenadas inteiras no plano #2 tais que o triângulo XYZ
é congruente ao triângulo ABC.

69) Sejam a e b inteiros positivos tais que an – 1 divide bn – 1 para todo inteiro
positivo n.
Prove que existe k ∈  tal que b = ak.

70)
P1

X1
X5

P5 P2
Q5 Q1

Q4 Q2
X4 X2
Q3

X3
P4 P3
Na figura acima, para 1 ≤ j ≤ 5, X j é o ponto de interseção dos círculos
circunscritos aos triângulos Qj –1 PjQj e Qj Pj + 1 Qj + 1 distintos de Qj (os índices
são tomados módulo 5). Prove que o pentágono X1X2X3 X4X5 é inscritível.
Obs: O pentâgono P1P2P3P4P5 não é necessariamente regular.

EUREKA! N°14, 2002

59
Sociedade Brasileira de Matemática

71) Considere três circunferências, tangentes duas a duas. Prove que há apenas
duas circunferências tangentes às três simultaneamente, e mostre como
construí-las.

72) Ache todas as funções f : # → # tais que f ( x 4 + y ) = x 3 f ( x) + f ( f ( y )),


∀x, y ∈ #.

73) Prove que, dado um inteiro positivo n, existe uma progressão aritmética
crescente formada por n inteiros positivos cujas somas dos dígitos também
formam uma progressão aritmética crescente, mas não existe uma progressão
aritmética infinita de inteiros positivos cujas somas dos dígitos formem uma
progressão aritmética crescente.

Problema 70 proposto por Jiang Zemin, presidente da China, a membros da direção da União
Internacional de Matemática, durante uma reunião preparatória do Congresso Internacional de
Matemática (ICM), realizado em agosto de 2002, em Beijing, China; Problema 71 proposto por
Marcelo Ribeiro de Souza (Rio de Janeiro – RJ); Problema 72 adaptado da 14a. Asian Pacific
Mathematical Olympiad; Problema 73 adaptado do 17o.Torneio das Cidades, 1995.

EUREKA! N°14, 2002

60
Sociedade Brasileira de Matemática

AGENDA OLÍMPICA

XXIV OLIMPÍADA BRASILEIRA DE MATEMÁTICA

NÍVEIS 1, 2 e 3
Primeira Fase – Sábado, 8 de junho de 2002
Segunda Fase – Sábado, 14 de setembro de 2002
Terceira Fase – Sábado, 19 de outubro de 2002 (níveis 1, 2 e 3)
Domingo, 20 de outubro de 2002 (níveis 2 e 3 - segundo dia de prova).

NÍVEL UNIVERSITÁRIO
Primeira Fase – Sábado, 14 de setembro de 2002
Segunda Fase – Sábado, 19 e Domingo, 20 de outubro de 2002

VIII OLIMPÍADA DE MAIO
11 de maio de 2002

XIII OLIMPÍADA DE MATEMÁTICA DO CONE SUL
22 a 28 de junho de 2002
Fortaleza – CE, Brasil

XLIII OLIMPÍADA INTERNACIONAL DE MATEMÁTICA
18 a 31 de julho de 2002
Glasgow, Reino Unido

XVII OLIMPÍADA IBEROAMERICANA DE MATEMÁTICA
30 de setembro a 5 de outubro de 2002
El Salvador

V OLIMPÍADA IBEROAMERICANA DE MATEMÁTICA UNIVERSITÁRIA
9 de novembro de 2002

♦♦♦

EUREKA! N°14, 2002

61
Sociedade Brasileira de Matemática

COORDENADORES REGIONAIS

Alberto Hassen Raad (UFJF) Juiz de Fora – MG


Amarísio da Silva Araújo (UFV) Viçosa – MG
Benedito Tadeu Vasconcelos Freire (UFRN) Natal – RN
Carlos Frederico Borges Palmeira (PUC-Rio) Rio de Janeiro – RJ
Claudio Arconcher (Colégio Leonardo da Vinci) Jundiaí – SP
Claus Haetinger (UNIVATES) Lajeado – RS
Cleonor Crescêncio das Neves (UTAM) Manaus – AM
Élio Mega (Colégio Etapa) São Paulo – SP
Florêncio Ferreira Guimarães Filho (UFES) Vitória – ES
Gisele de Araújo Prateado Gusmão (UFGO) Goiânia – GO
Irene Nakaoka (UEM) Maringá – PR
Ivanilde Fernandes Saad (UC. Dom Bosco) Campo Grande– MS
Jacqueline Fabiola Rojas Arancibia (UFPB) João Pessoa – PB
João Benício de Melo Neto (UFPI) Teresina – PI
João Francisco Melo Libonati (Grupo Educacional Ideal) Belém – PA
José Carlos Pinto Leivas (UFRG) Rio Grande – RS
José Cloves Saraiva (UFMA) São Luis – MA
José Gaspar Ruas Filho (ICMC-USP) São Carlos – SP
José Luiz Rosas Pinho (UFSC) Florianópolis – SC
José Vieira Alves (UFPB) Campina Grande – PB
Licio Hernandes Bezerra (UFSC) Florianópolis – SC
Luzinalva Miranda de Amorim (UFBA) Salvador – BA
Marcelo Rufino de Oliveira (Grupo Educacional Ideal) Belém – PA
Marcondes Cavalcante França (UFC) Fortaleza – CE
Pablo Rodrigo Ganassim (Liceu Terras do Engenho) Piracicaba – SP
Ramón Mendoza (UFPE) Recife – PE
Reinaldo Gen Ichiro Arakaki (INPE) SJ dos Campos – SP
Ricardo Amorim (Centro Educacional Logos) Nova Iguaçu – RJ
Roberto Vizeu Barros (Colégio Acae) Volta Redonda – RJ
Rosângela Souza (Colégio Singular) Santo André – SP
Sérgio Cláudio Ramos (IM-UFRGS) Porto Alegre – RS
Tadeu Ferreira Gomes (UEBA) Juazeiro – BA
Tomás Menéndez Rodrigues (U. Federal de Rondônia) Porto Velho – RO
Valdenberg Araújo da Silva (U. Federal de Sergipe) São Cristovão – SE
Wagner Pereira Lopes (CEFET – GO) Jataí – GO

EUREKA! N°14, 2002

62
CONTEÚDO

AOS LEITORES 2

XLIII OLIMPÍADA INTERNACIONAL DE MATEMÁTICA 3


Enunciados, Soluções e Resultado Brasileiro

XVII OLIMPÍADA IBEROAMERICANA DE MATEMÁTICA 15


Enunciados, Soluções e Resultado Brasileiro

ARTIGOS

A FÓRMULA DE CARDANO ALÉM DAS CÚBICAS 24


José Cloves Verde Saraiva, São Luis - MA

RECIPROCIDADE QUADRÁTICA 27
Carlos Gustavo T. de A. Moreira & Nicolau Corção Saldanha, Rio de Janeiro - RJ

APLICAÇÕES DE PLANOS PROJETIVOS EM TEORIA DOS NÚMEROS E COMBINATÓRIA 31


Carlos Yuzo Shine, São Paulo - SP

OLIMPÍADAS AO REDOR DO MUNDO 43

SOLUÇÕES DE PROBLEMAS PROPOSTOS 54

PROBLEMAS PROPOSTOS 59

COORDENADORES REGIONAIS 61
Sociedade Brasileira de Matemática

AOS LEITORES

Chegamos a esta última edição do ano 2002 muito contentes com o


desempenho olímpico do Brasil: Pelo segundo ano consecutivo todos os integrantes
da equipe brasileira ganharam medalha na Olimpíada Internacional de Matemática, e
além disso tivemos excelentes resultados na Olimpíada de Matemática do Cone Sul e
na Olimpíada Iberoamericana de Matemática, onde ganhamos a maioria das
medalhas de ouro em disputa. Publicamos aqui as soluções dos problemas da IMO e
da Ibero, sendo a maioria delas dos membros das equipes brasileiras.
Agradecemos mais uma vez a crescente colaboração dos leitores, enviando
problemas propostos e soluções, e pedindo que publiquemos soluções de problemas
de várias fontes, como das Olimpíadas ao Redor do Mundo.
Esse intercâmbio é fundamental para nós, e ajuda a manter a revista Eureka!
viva e interessante.

Abraços e feliz 2003 para todos!

Os editores.

EUREKA! N°15, 2002

2
Sociedade Brasileira de Matemática

XLIII OLIMPÍADA INTERNACIONAL DE MATEMÁTICA


Enunciados, Soluções e Resultado Brasileiro

A XLIII Olimpíada Internacional de Matemática foi realizada na cidade de


Glasgow, Reino Unido no período de 18 a 31 de julho de 2002.
A equipe brasileira foi liderada pelos professores Edmilson Motta (São
Paulo – SP) e Ralph Costa Teixeira (Niterói – RJ).

O Resultado da Equipe Brasileira

BRA 1 Alex Corrêa Abreu Bronze


BRA 2 Larissa Cavalcante Queiroz de Lima Prata
BRA 3 Guilherme Issao Camarinha Fujiwara Bronze
BRA 4 Yuri Gomes Lima Bronze
BRA 5 Davi Máximo Alexandrino Nogueira Bronze
BRA 6 Thiago da Silva Sobral Bronze

PRIMEIRO DIA
DURAÇÃO: 4 horas e meia.

PROBLEMA 1
Seja n um inteiro positivo. Seja T o conjunto de pontos (x; y) no plano onde x e y são
inteiros não negativos e x + y < n. Cada ponto de T é pintado de vermelho ou azul.
Se um ponto (x; y) é vermelho, então todos os pontos (x'; y') com x' ≤ x e y' ≤ y
também são. Um conjunto X é um conjunto de n pontos azuis com abcissas todas
distintas, e um conjunto Y é um conjunto de n pontos azuis com ordenadas todas
distintas. Prove que o número de conjuntos X é igual ao número de conjuntos Y.

SOLUÇÃO DE GUILHERME FUJIWARA (SÃO PAULO – SP)


Primeiramente, seja x(i) o número de pontos azuis cuja ordenada é i, e y(i) o número
de pontos azuis de abscissa i.
n −1 n −1
Veja que o número de X-conjuntos é ∏i=0
y (i) , e o número de Y-conjuntos é ∏ x(i) .
i=0
Para provar que o número de X-conjuntos é igual ao número de Y-conjuntos, é
suficiente provar que os números x(0), x(1), x(2),...,x(n – 1) são uma permutação dos
números y(0), y(1), y(2),..., y(n – 1). Provaremos este lema por indução no n.
Se n = 1, temos que x(0) = y(0) = 1 ou 0 dependendo se (0; 0) é azul ou não.
Suponhamos que o lema é verdadeiro para n < k, provaremos para n = k.
Vamos olhar para a última diagonal de T (reta x + y = k – 1):
Se nela não houver pontos vermelhos, então tome T' como os conjuntos de pontos de

EUREKA! N°15, 2002

3
Sociedade Brasileira de Matemática

T que não estão na última diagonal. Temos que o lema vale para os x'(i) e y'(i) de T',
e como x(i) = x'(i) + 1 e y(i) = y'(i) + 1, então x'(a) = y'(b) ⇔ x(a) = y(b), e além
disso x(k – 1) = y(k – 1) = 1, portanto o lema vale para T (vide fig. 1).

Y (ordenada) Fig. 1

X (abscissa)
Só pontos azuis na reta y + x = k – 1

Se nela houver algum ponto vermelho, digamos (a; k – 1 – a), então aplicamos a
hipótese de indução nos dois conjuntos T' formados acima e à direita de T, que são
menores que T, e assim demonstramos o lema para T (vide fig. 2).
Provamos então o nosso lema e, como já foi visto anteriormente, segue o que é
pedido no enunciado.

Y (ordenada) Fig. 2

2 conjuntos T' menores que T, no


qual aplicamos a hipótese de
indução.

X (abscissa)

PROBLEMA 2

EUREKA! N°15, 2002

4
Sociedade Brasileira de Matemática

Seja BC um diâmetro do círculo Γ de centro O. Seja A um ponto de Γ tal que


0 0 < ∠AOB < 120 0 . Seja D o ponto médio do arco AB que não contém C. A reta que
passa por O e é paralela a DA encontra a reta AC em J. A mediatriz de OA corta Γ
em E e F. Prove que J é o incentro do triângulo CEF.

SOLUÇÃO DE YURI GOMES LIMA (FORTALEZA – CE)


Como EF é perpendicular ao raio OA, os arcos EA e AF são iguais, donde ∠ECA =
∠ACF ⇒ ∠ECJ = ∠JCF. Portanto, J já pertence à bissetriz de ∠ECF. Se
mostrarmos então que AJ = AE, acabou (pois sabemos que o médio do arco EF
eqüidista de E, de F e do incentro de CEF).
Mas, se l é a paralela a DA por O, temos que l // DA e DO // AC ( pois ∠DOB =
∠AOB/2 = ∠ACB ) ⇒ DAJO paralelogramo ⇒ AJ = DO. Também, sendo EF
mediatriz de OA, segue que EA = EO. Mas EO = raio de Γ = DO , donde AJ = EA =
AF.

OBS: é necessário ∠AOB < 120°, pois caso contrário teríamos ∠AOC ≤ 60° e
∠BOD ≥ 60°, o que implicaria J = l ∩ AC fora de Γ, não podendo então ser este o
incentro de CEF.

PROBLEMA 3
Encontre todos os pares de inteiros m, n ≥ 3 tais que há infinitos inteiros positivos a
am + a −1
para os quais n é inteiro.
a + a2 −1

SOLUÇÃO DE ALEX CORRÊA ABREU (NITERÓI – RJ)


Seja P = P ( x ) = x m + x − 1 e Q( x) = x n + x 2 − 1 ⇒ ∃T , R ∈ =[ x] tq
P( x) = T ( x) ⋅ Q( x) + R( x),
P (a ) R (a ) R (a ) P (a )
com deg R < deg Q ou R ≡ 0 ⇒ = T ( a) + ⇒ = − T ( a) é inteiro
Q (a ) Q(a ) Q (a ) Q (a )
R( x)
para infinitos a's mas se R ≠ 0 ⇒ ∃n tq x > n ⇒ 0 < < 1 pois
Q( x)
R( a)
deg R < deg Q ⇒ < 1 se a > n ⇒ existe apenas um número finito de a's tq
Q( a)
R(a) é inteiro ⇒ R ≡ 0 ⇒ P( x) = T ( x) ⋅ Q( x) ⇒ Q( x) P( x) ⇒ Q( x) P( x) ⋅ ( x + 1) − Q( x) =
Q( a)
= x m +1 + x m + x 2 − 1 − x n − x 2 + 1 = x n ( x m − n +1 + x m − n − 1) ; note que temos, m ≥ n

EUREKA! N°15, 2002

5
Sociedade Brasileira de Matemática

(pois Q ( x ) | P( x)) mas mdc (Q( x), x n ) = mdc( x 2 −1, x n ) = 1 ⇒ Q( x) x m−n +1 + x m−n −1 ⇒
por raciocínio análogo ao anterior, m − n + 1 ≥ n ⇒ 2n ≤ m + 1( I ).
Agora, como Q (0) = −1 , Q (1) = 1 ⇒ ∃0 < α < 1 tq Q (α ) = 0 ⇒ P (α ) = T (α )Q (α ) = 0
⇒ α n + α 2 − 1 = 0 = α m + α − 1( II ) ⇒
de (I), α 2 n ≥ α m+1 ( III ) ⇒ por (II), (1 − α 2 ) 2 ≥ α ⋅ α m = α (1 − α ) ⇒
(1 − α ) 2 (1 + α ) 2 ≥ α (1 − α ) ⇒ (1 − α )(1 + α ) 2 ≥ α pois α < 1 ⇒
α 3 + α 2 − 1 ≤ 0 ⇒ α 3 ≤ 1 − α 2 = α n ⇒ n ≤ 3 mas n ≥ 3 ⇒ n = 3 e a desigualdade só
não é estrita em (III) se m + 1 = 2n ⇒ m = 5 logo se existir uma solução será (5, 3),
que de fato é uma solução, pois obviamente
( )( )
x 5 + x − 1 = x 2 − x + 1 x 3 + x 2 − 1 ⇒ P ( x ) = ( x 2 − x + 1) ⋅ Q ( x ).

SEGUNDO DIA
DURAÇÃO: 4 horas e meia.

PROBLEMA 4
Seja n inteiro maior que 1. Os divisores positivos de n são d1, d2,…,dk , onde

1 = d1 < d 2 < ... < d k = n

Seja D = d1d2 + d2d3 +…+dk – 1 dk.


(a) Prove que D < n2.
(b) Encontre todos os valores de n para os quais D é um divisor de n2.

SOLUÇÃO DE THIAGO DA SILVA SOBRAL (FORTALEZA – CE)


n
Lema: d i =
d k +1−i
Prova:
Observando que se d é divisor de n então n/d também o é, podemos agrupar os
divisores aos pares, donde concluímos que d i d k +1−i = n , 0 ≤ i ≤ k . Esse fato
também vale para n sendo quadrado perfeito, pois d k +1 = d k +1 = n
k +1−
2 2

n
Corolário: d i ≤
k +1− i

EUREKA! N°15, 2002

6
Sociedade Brasileira de Matemática

Prova:
De fato, sendo d k +1−i o (k + 1 – i)-ésimo divisor de n, temos d k +1−i ≥ k + 1 − i , e
segue o resultado pelo lema.
Pelo corolário,

 
k −1


n n n n nn 1 1
D = d1 d 2 + d 2 d 3 + + d k −1 d k ≤ + + = n2 =
k k −1 k −1 k − 2 21 j =1 j j + 1
k −1
1 1  2 1
=n 2
∑  j − j + 1  = n
j =1
1 −  < n
 k
2

b) Veja que se n é primo, D = 1 × n , e assim temos que D | n 2 . Suponha n


composto, e seja p o seu menor fator primo. Pelo lema,

 n
D = d1 d 2 + d 2 d 3 + + d k −1 d k = 1 × p + + × n >
p

n2
p

n2 n2
Veja então que < D < n 2 , e como é o maior divisor de n 2 menor que n 2 ,
p p
concluímos que D /| n 2

Por fim, concluímos que D | n 2 ⇔ n é primo.

PROBLEMA 5
Encontre todas as funções f de # em # tais que

( f ( x ) + f ( z ))( f ( y ) + f (t )) = f ( xy − zt ) + f ( xt + yz )

para todo x, y, z, t ∈ #.

SOLUÇÃO DE LARISSA CAVALCANTE QUEIROZ DE LIMA (FORTALEZA – CE)


( f ( x ) + f ( z ))( f ( y ) + f (t )) = f ( xy − zt ) + f ( xt + yz )
x, z, y , t ← 0 : (2 ⋅ f (0)) ⋅ ( 2 ⋅ f (0)) = 2 ⋅ f (0) ⇒ 2 ⋅ f (0) 2 = f (0)
1
f (0) ≠ 0 ⇒ 2 ⋅ f (0) = 1 ⇒ f (0) =
2
z, y , t ← 0 : ( f ( x ) + f (0))( 2 ⋅ f (0)) = 2 ⋅ f (0)
1 1 1
se f (0) ≠ 0 (ou seja f (0) = ) , f ( x ) + f (0) = 1 ⇒ f ( x) = 1 − = ∀x ∈ 5
2 2 2

EUREKA! N°15, 2002

7
Sociedade Brasileira de Matemática

1 1
⇒ Se f (0) ≠ 0, então f (0) = e f ( x) = , ∀x ∈ 5 (o que é uma solução).
2 2
Suponha então f (0) = 0
z ← y; t ← x ⇒ [ f ( x ) + f ( y ) ] = f ( xy − xy ) + f ( x 2 + y 2 )
2

⇒ [ f ( x ) + f ( y )]2 = f ( x 2 + y 2 ) = f ( x) 2 + 2 f ( x ) f ( y ) + f ( y ) 2
z ← x; t ← y ⇒ ( 2 ⋅ f ( x))( 2 ⋅ f ( y )) = f ( xy − xy ) + f ( xy + xy )
⇒ 4 f ( x ) ⋅ f ( y ) = f ( 2 xy )
t , z ← 0 ⇒ ( f ( x) + f (0))( f ( y ) + f (0)) = f ( xy − 0) + f (0 + 0)
⇒ f ( x ) ⋅ f ( y ) = f ( xy ) f é multiplicativa.
⇒ 4 ⋅ f ( xy ) = f (2 xy )
y ← t; t ← y ⇒ ( f ( x ) + f ( z ))( f (t ) + f ( y )) = f ( xt − zy ) + f ( xy + tz )
⇒ f ( xt − zy ) + f ( xy + tz ) = f ( xt + zy ) + f ( xy − zt )
Suponhamos que f não seja identicamente nula (note que f(x) ≡ 0 é uma solução).
Suponha f (a ) < 0 e que ∃x, y : x 2 + y 2 = a
⇒ f (a ) = f ( x 2 + y 2 ) = [ f ( x ) + f ( y ) ]2 ≥ 0 , contradição!
Assim, f (a ) ≥ 0, ∀a ≥ 0.
z ← − z ⇒ ( f ( x ) + f (− z ))( f ( y ) + f (t )) = f ( xy + zt ) + f ( xt − zy )
= ( f ( x) + f ( z ))( f (t ) + f ( y ))
⇒ f ( x ) + f (− z ) = f ( x ) + f ( z )
f (− z ) = f ( z )
f (− z ) = f ( −1) ⋅ f ( z ) ⇒ f ( −1) = 1 = f (1).
z, y , t ← 1 : ( f ( x ) + 1) ⋅ 2 = f ( x − 1) + f ( x + 1)
⇒ 2 f ( x ) + 2 = f ( x − 1) + f ( x + 1) .
* 4 ⋅ f ( xy ) = f ( 2 xy ) = f ( 2) ⋅ f ( xy ) ⇒ f (2) = 4.
* conjectura: f ( x) = x 2
( x 2 + z 2 )( y 2 + t 2 ) = ( xy − zt ) 2 + ( xt + yz ) 2
⇔ x 2 y 2 + x 2 t 2 + z 2 y 2 + z 2 t 2 = x 2 y 2 + z 2 t 2 − 2 xyzt + x 2 t 2 + 2 xtyz + y 2 z 2
Ok!! A função f ( x) = x 2 funciona!!!
2 f ( x ) + 2 = f ( x − 1) + f ( x + 1) ( f (1) = 1; f (2) = 4)
f (m) = m 2 ∀m ≤ n, m ∈ 1.
2 ⋅ f (n) + 2 = f (n − 1) + f (n + 1)
⇒ f (n + 1) = 2 ⋅ n 2 + 2 − (n − 1) 2 = n 2 + 2n + 1 = ( n + 1) 2

EUREKA! N°15, 2002

8
Sociedade Brasileira de Matemática

⇒ f (n) = n 2 , ∀n ∈ 1 .
como f (− n) = f (n); temos f ( z ) = z 2 , ∀z ∈ = .
p
Tome r ∈ 4 ; r = , mdc( p, q) = 1; p, q ∈ =, q ≠ 0
q
 p 1 f (1) f ( p) p 2
f (r ) = f   = f ( p) ⋅ f   = f ( p) ⋅ = = 2 = (r) 2
q q f (q ) f (q ) q
⇒ f (r ) = r 2 , ∀r ∈ 4 .
( f ( x ) + f ( r ))2 ⋅ f ( r ) = f ( x ⋅ r + r 2 ) + f ( x ⋅ r − r 2 )
2 f ( x ) f ( r ) + 2 f ( r ) 2 = f ( x ⋅ r + r 2 ) + f ( xr − r 2 )
2 f ( xr ) + 2r 4 = f ( xr + r 2 ) + f ( xr − r 2 ) ∀x ∈ 5 ,
r ∈ 4 ⇒ 2 f ( y ) + 2r 4 = f ( y + r 2 ) + f ( y − r 2 ), ∀y ∈ 5, r ∈ 4.
Suponha que ∃y, r tais que y ≥ 0 , r > 0 e
f ( y + r 2 ) < f ( y)
[ ( y ) + f (r ) ] < f ( y )
⇒ f
2 2

≥0
,
⇒ f ( y ) + 2 f ( y )⋅ f ( r ) + f ( r ) < f ( y ) , Absurdo!!!


>0 >0
2 2

⇒ f ( y + r ) > f ( y ) ∀y ∈ 5 ⇒ f é crescente em #+.


2

Vamos mostrar que f é contínua em #+: note que se y, r ≥ 0, f(y +r2) – f(y) =
⇒ f ( y ) 2 + 2 f ( y ) f (r ) + f (r ) 2 − f ( y ) = f ( r )(2 f ( y ) + f ( r )) = r 2 ( 2 f ( y ) + r 2 ),
se r ∈ 4 . Assim, dado ε > 0 , para r0 ∈ 4 suficientemente pequeno, temos
f ( y + r02 ) − f ( y ) < ε .
e se r < r0 , temos f ( y + r 2 ) − f ( y ) < f ( y + r02 ) − f ( y ) < ε
⇒ se x − y < r02 temos f ( x ) − f ( y ) < ε ( x > y ) .
* f ( y ) − f ( y − r02 ) = f ( y + r02 ) − f ( y ) − 2r04 < ε − 2r04 < ε
Se r < r0 então y − r 2 > y − r02 ⇒ f ( y − r 2 ) > f ( y − r02 )
⇒ f ( y ) − f ( y − r 2 ) < f ( y ) − f ( y − r02 ) < ε
⇒ para x − y < r02 temos f ( x ) − f ( y ) < ε
portanto f é contínua.
Suponha f ( w) = w 2 + θ ,θ ≠ 0

EUREKA! N°15, 2002

9
Sociedade Brasileira de Matemática

1) θ > 0
Temos que ∃r0 > 0 tal que
x − w < r02 ⇒ f ( x ) − f ( w) < θ
ou seja x 2 − w 2 − θ < θ caso x ∈ 4
mas se x < w temos x 2 < w 2 e portanto w 2 − x 2 > 0
( )
⇒ x 2 − w 2 − θ = w 2 + θ − x 2 = w 2 − x 2 + θ > θ , contradição!!
(existe x ∈ 4 tal que 0 < w − x < r02 ).
2) θ < 0 ⇒ f ( w) = w 2 − θ
temos que ∃r0 > 0 tal que
x − w < r02 ⇒ f ( x) − f ( w) < θ
ou seja x 2 − w 2 + θ < θ caso x ∈ 4
Note que ∃x ∈ 4 tal que 0 < x − w < r02
⇒ x > w ⇒ x 2 > w 2 ⇒ x 2 − w 2 + θ = x 2 − w 2 + θ > θ contradição!!
⇒ f ( x ) = x 2 , ∀x ∈ 5 .
As únicas funções são:
1
f ( x ) ≡ ; f ( x) ≡ 0 ou f ( x) = x 2 , ∀x ∈ 5 .
2

PROBLEMA 6
Sejam Γ1 , Γ2 ,..., Γn círculos de raio 1 no plano, onde n ≥ 3. Seus centros são O1,
O2,…,On, respectivamente.
Suponha que não exista reta que intercepte mais que dois dos círculos. Prove que
(n − 1)π

1
≤ .
1≤ i < j ≤ n Oi O j 4

SOLUÇÃO DE LUCIANO GUIMARÃES CASTRO (RIO DE JANEIRO – RJ)


Seja αij a medida, em radianos, do ângulo agudo formado pela reta OiOj com uma
reta tangente a Γj passando por Oi. As circunferências têm raio 1, e sen α ij ≤ α ij ,
∀i, j ∈ {1, 2, , n} . Assim,
∑ ∑ ∑
1
= sen α ij ≤ α ij .
1≤ i < j ≤ n Oi O j 1≤i < j ≤ n 1≤ i < j ≤ n

EUREKA! N°15, 2002

10
Sociedade Brasileira de Matemática

(n − 1)π
É suficiente, portanto, provar que ∑α
1≤ i < j ≤ n
ij ≤
4
, o que parece menos assustador

que o enunciado original.

Vejamos de que maneira a condição de nenhuma reta cortar mais de duas


circunferências limita a soma dos αij. Fixemos i. Para cada j, a união de todas as
retas que passam por Oi e cortam a circunferência Γj forma dois ângulos opostos pelo
vértice Oi, cada um medindo 2αij. Como essas retas não cortam outra circunferência
além de Γi e Γj, variando j obtemos ângulos disjuntos com vértice Oi, de forma que a
soma de suas medidas não pode ultrapassar 2π, ou seja,

∑ 4α
1≤ j ≤ n
ij ≤ 2π . (1)
j ≠i
Γ2

Γ1

2 α i2

2 α i1

2 α ij
Oi
Γj

Agora, somando estas desigualdades para i ∈ {1, 2, , n} e observando que


α ij = α ji , obtemos

∑ ∑ 4α
1≤i ≤ n 1≤ j ≤ n
ij ≤ ∑ 2π
1≤i ≤ n
⇔ 2 ∑ 4α
1≤ i < j ≤ n
ij ≤ 2 nπ ⇔ ∑α
1≤i < j ≤ n
ij ≤
4
.
j ≠i

EUREKA! N°15, 2002

11
Sociedade Brasileira de Matemática

Hummmm... quase! De fato, este resultado já é assintoticamente equivalente ao


desejado. Apesar de que conseguir um n − 1 no lugar daquele n é a parte difícil deste
problema, você vai perceber que se trata apenas de ir adaptando esta primeira idéia.

Intuitivamente, o que acabamos de fazer foi “girar” uma reta 180o em torno de cada
ponto Oi, sabendo que neste percurso ela cortará todas as outras circunferências, mas
nunca duas ao mesmo tempo. Para melhorar a estimativa, precisamos encontrar uma
forma de girar menos que 180o e, ainda assim, encontrar todas as demais
circunferências. Isto não é possível para todos os Oi, mas podemos fazê-lo com os
“mais afastados”.

Mais precisamente, nossa idéia é trabalhar com o fecho convexo do conjunto {O1,
O2, ..., On}, ou seja, o menor conjunto convexo que contém {O1, O2, ..., On}. Sem
perda de generalidade, podemos supor que esse fecho convexo é o polígono
O1O2O3... Om (m ≤ n) . Desta forma, os pontos Om+1, Om+2, ..., On são interiores ao
polígono.
Vamos separar a soma ∑α
1≤i < j ≤ n
ij em quatro partes:

L= ∑α
1≤ i ≤ m −1
i ,i +1 + α 1m (soma dos αij tais que OiOj é lado do polígono);

D= ∑α
1≤i < j ≤ m
ij − L (soma dos αij tais que OiOj é diagonal do polígono);

T= ∑α
1≤ i ≤ m
ij (soma dos αij tais que Oi é vértice do polígono e Oj é interior);
m< j ≤n

I= ∑α
m<i< j ≤ n
ij (soma dos αij tais que Oi e Oj são interiores ao polígono).

Observe que L + D + T + I = ∑α
1≤ i < j ≤ n
ij .

Vamos usar os ângulos externos do polígono para limitar L. Sejam ai e ei,


respectivamente, as medidas dos ângulos interno e externo do polígono no vértice Oi.
Para simplificar a notação, trabalharemos com o vértice O2. Seja t a tangente comum
exterior a Γ1 e Γ3, mais próxima de Γ2. Então Γ2 está totalmente contida no
semiplano determinado por t que não contém Γ1 e Γ3 (caso contrário existiria uma
reta cortando Γ1, Γ2 e Γ3). Sejam r a paralela a t por O2 e P1 o ponto de r tal que
∠P1O2O1 é agudo e sua bissetriz é perpendicular à reta O1P1. Então a distância de O1
OP
a essa bissetriz é igual a 1 1 . Esta distância é maior que 1, pois P1 pertence a r e a
2
EUREKA! N°15, 2002

12
Sociedade Brasileira de Matemática

distância de O1 a r é maior que 2. Isto significa que a bissetriz de ∠P1O2O1 é exterior


m(∠P1O2O1 )
a Γ1, do que concluímos que α12 < . Procedendo de forma análoga com
2
a circunferência Γ3 e somando as duas desigualdades, temos
π − a 2 e2
α 12 + α 23 < = .
2 2

P1
r
O2

α
12

O1 O3

Fazendo o mesmo para os demais vértices do polígono e somando as desigualdades


obtidas, concluímos que


ei
2L < ⇔ 2L < π. (2)
1≤ i ≤ m 2
Agora utilizaremos os ângulos internos do polígono em um procedimento parecido
ao que fizemos para descobrir a desigualdade (1). Salvo Γ1, Γ2 e Γ3, todas as demais
circunferências estão completamente contidas no ângulo interno Ô2 = ∠O1O2O3.
Para cada j ≠ 2, considere o conjunto união das semi-retas com origem O2, interiores
a Ô2, que cortam a circunferência Γj. Este conjunto forma um ângulo de medida α2j,
para j = 1 e j = 3, e um ângulo de medida 2α2j para os demais valores de j. Como
cada semi-reta pode cortar apenas uma circunferência além de Γ2, os conjuntos
correspondentes a distintos valores de j são disjuntos. Assim,

α 12 + α 23 + 2 α 2 j ≤ a 2 .
4≤ i ≤ n
Procedendo de forma análoga com os demais vértices do polígono e somando as
desigualdades obtidas concluímos que
2 L + 4 D + 2T ≤ ∑
ai = ( m − 2)π .
1≤ i ≤ m
(3)

EUREKA! N°15, 2002

13
Sociedade Brasileira de Matemática

Para concluir, seja Oi um ponto interior ao polígono, ou seja, i > m. Já provamos a


desigualdade (1): ∑
4α ij ≤ 2π .
1≤ j ≤ n
j ≠i

Somando essas desigualdades para todos os pontos Oi interiores temos


∑ ∑ 4α ij ≤
m < i ≤ n 1≤ j ≤ n

2π ⇔ 4T + 8I ≤ ( n − m) 2π ⇔ 2T + 4 I ≤ ( n − m)π .
m<i ≤n
(4)
j ≠i

Agora basta somar as desigualdades (2), (3) e (4), obtendo


2 L + 2 L + 4 D + 2T + 2T + 4 I ≤ π + ( m − 2)π + (n − m)π
( n − 1)π
⇔ 4 L + 4 D + 4T + 4 I ≤ (n − 1)π ⇔ L + D + T + I ≤ .
4
Portanto
(n − 1)π
∑ ∑ ∑
1
= sen α ij ≤ α ij = L + D + T + I ≤ ,
1≤i < j ≤ n Oi O j 1≤ i < j ≤ n 1≤ i < j ≤ n 4
como queríamos demonstrar.

EUREKA! N°15, 2002

14
Sociedade Brasileira de Matemática

XVII OLIMPÍADA IBEROAMERICANA DE MATEMÁTICA


Enunciados, Soluções e Resultado Brasileiro

A XVII Olimpíada Iberoamericana de Matemática foi realizada na cidade de


São Salvador, El Salvador no período de 30 de setembro a 5 de outubro de 2002.
A equipe brasileira foi liderada pelos professores Eduardo Wagner (Rio de
Janeiro – RJ) e Onofre da Silva Farias (Fortaleza – CE).

O Resultado da Equipe Brasileira

BRA 1 Guilherme Camarinha Fujiwara Ouro


BRA 2 Humberto Silva Naves Ouro
BRA 3 Larissa Cavalcante Queiroz de Lima Ouro
BRA 4 Yuri Gomes Lima Prata

PRIMEIRO DIA
DURAÇÃO: 4 horas e meia.

PROBLEMA 1
Os números inteiros desde 1 até 2002, ambos incluídos, escrevem-se num quadro por
ordem crescente 1, 2,..., 2001, 2002. Em seguida apagam-se os que ocupam o
primeiro lugar, quarto lugar, sétimo lugar, etc, ou seja, os que ocupam os lugares da
forma 3k + 1.
Na nova lista apagam-se os números que estão nos lugares da forma 3k + 1. Repete-
se este processo até que se apagam todos os números da lista. Qual foi o último
número que se apagou?

SOLUÇÃO DE GUILHERME CAMARINHA FUJIWARA (SÃO PAULO - SP)


Considere uma sequência infinita ao invés de uma sequência até 2002. Seja então
R(k) o primeiro número a ser apagado na k-ésima seção de apagamento.
Temos então R(1) = 1, R(2) = 2, R(3) = 3, R(4) = 5, R(5) = 8, R(6) = 12.
Queremos então achar o maior R(k) menor ou igual à 2002.
3 
Vamos provar que R(k + 1) =  ⋅ R( k )  (onde k  é o menor inteiro maior ou igual
2 
a x).
Para tal, basta ver que se considerarmos a seqüência que sobra após a primeira série
de apagamentos, teremos que o R(k)-ésimo termo será o primeiro a ser apagado na
k-ésima próxima série de apagamento. Considerando também o primeiro
apagamento, temos que o R(k)-ésimo número dessa lista sérá o primeiro a ser
apagado na (k + 1)-ésima série de apagamento, logo será o R(k + 1).

EUREKA! N°15, 2002

15
Sociedade Brasileira de Matemática

É fácil ver que o n-ésimo termo da sequência que sobra após a primeira série de
3 
apagamentos será o  ⋅ n (basta ver os dois casos de paridade de n), logo temos
2 
3 
que R(k +1) =  ⋅ R( k )  .
 2 
Fazendo as contas, temos então R(6) = 12, R(7) = 18, R(8) = 27, R(9) = 41, R(10) =
62, R(11) = 93, R(12) = 140, R(13) = 210, R(14) = 315, R(15) = 473, R(16) = 710,
R(17) = 1065, R(18) = 1598 e finalmente R(19) = 2397.
Como R(18) = 1598 e 2002 < R(19) = 2397, então o último número apagado foi
1598.

PROBLEMA 2
Dado qualquer conjunto de 9 pontos no plano entre os quais não existem três
colineares, demonstre que para cada ponto P do conjunto, o número de triângulos
que têm como vértices três dos oito pontos restantes e P no seu interior é par.

SOLUÇÃO DE HUMBERTO SILVA NAVES (SÃO JOSÉ DOS CAMPOS - SP)


Seja S o conjunto dos 9 pontos.
Se um ponto P está no interior do triângulo ABC com A, B, C e P ∈ S, temos:
A α

B' C'
P
γ β
C
A' B

Sejam A’ = AP ∩ BC ; B’ = BP ∩ ΑC e C’ = CP ∩ ΑB. As semi-retas PA’, PB’ e


PC’ dividem o plano em três regiões: α, β e γ (A ∈ α, Β ∈ β e C ∈ γ).
Vamos construir um grafo G cujos vértices representam os triângulos (com vértices
em S) com o ponto P em seu interior. Ligaremos 2 vértices deste grafo se e somente
se os triângulos correspondentes tiverem um lado em comum. Vamos agora provar
que o grau de cada vértice é 5.
Seja Q ∈ S – {A ; B ; C ; P}. Temos 3 possibilidades:
EUREKA! N°15, 2002

16
Sociedade Brasileira de Matemática

1) Se Q ∈ α: Temos que o ponto P está no triângulo QBC, logo ∆QBC ∈ V(G).


Como os triângulos ∆ΑBC e ∆QBC tem um lado em comum, eles estão ligados por
uma aresta em G. Obs.: Claro que P não está no ∆QAB e nem no ∆QAC.
2) Se Q ∈ β: ∆QAC ∈ V(G) e está ligado à ∆ABC em G.
3) Se Q ∈ γ : ∆QAB ∈ V(G) e está ligado à ∆ABC em G.

Logo deg (∆ABC) = #(S – {A ; B ; C ; P}) = 5. E como Σ deg(∆) = 2 × #E(G)


⇒ #V(G) é par. Portanto o número de triângulos com P em seu interior é par.

PROBLEMA 3
Um ponto P é interior ao triângulo equilátero ABC e é tal que ∠APC = 120° . Sejam
M a intersecção de CP com AB e N a intersecção de AP com BC. Encontrar o lugar
geométrico do circuncentro do triângulo MBN quando P varia.

SOLUÇÃO DE YURI GOMES LIMA (FORTALEZA – CE)

B

Γ

X Y
O1 N

M
O
P

A C

Vamos mostrar que tal L.G. está contido na reta  mediatriz do raio BO , onde O é o
centro de ABC. Para isso, seja O1 o circuncentro de BMN. Daí, como APˆ C = 120° ,

EUREKA! N°15, 2002

17
Sociedade Brasileira de Matemática

temos NPˆ M = 120° ⇒ BNPM inscritível. Para O1 pertencer a  , devemos ter


BO1 = OO1 , ou seja, O também deve pertencer à circunferência Γ circunscrita em
BMPN . Vamos mostrar que O é o médio do arco MN .
Agora, veja que, como APˆ C = 120° , temos então que BAˆ N = 60° − NAˆ C = ACˆ M .
Assim, ∆ABN ≡ ∆CAM . Logo, a rotação com centro em O e ângulo 120° que leva B
em A e A em C também leva N em M.
Assim, NOˆ M = 120° e NO = MO . Mas sendo NOˆ M = 120° , segue que BMON é
inscritível. Assim, provamos o que queríamos e O1 ∈ .
Mas o LG não é a reta toda. De fato, devemos ter O Bˆ N < 90° , pois ∆O BN é
1 1
isósceles, daí, se X e Y são os pontos de interseção de  com a circunferência
Ω circunscrita a ABC, teremos que O1 pertence ao INTERIOR do segmento XY ,
pois XBˆ C = YBˆ A = 90° (para ver isto, observe que XBYO é losango com
XO = BO ⇒ XOˆ B = 60° ⇒ X é o médio do arco AB).
Agora, dado O1 pertencente ao interior do segmento XY , trace a circunferência de
centro O1 e raio O1 B (que passa por O).
Ela determinará dois pontos M e N sobre AB , BC tais que
OMˆ N = OBˆ N = OBˆ M = ONˆ M e MOˆ N = 120° . Daí, a mesma rotação considerada
antes levará N em M, levando então o ∆ABN no ∆CAM ⇒ AN = CM e
BAˆ N = ACˆM ⇒ 60° − BAˆ N = 60° − ACˆM ⇒ PAˆ C + ACˆP = 60° , onde P = AN ∩CM ,
donde APˆ C = 120°.
Logo, o L.G. procurado é o interior do segmento XY .

SEGUNDO DIA
DURAÇÃO: 4 horas e meia.

PROBLEMA 4
Num triângulo escaleno ABC traça-se a bissectriz interna BD, com D sobre AC.
Sejam E e F, respectivamente, os pés das perpendiculares traçadas desde A e C até à
recta BD, e seja M o ponto sobre o lado BC tal que DM é perpendicular a BC.
Demonstre que ∠EMD = ∠DMF .

EUREKA! N°15, 2002

18
Sociedade Brasileira de Matemática

SOLUÇÃO DE YURI GOMES LIMA (FORTALEZA – CE)


B

M
A'
E

A D C

Seja A' a interseção de AE com BC . Então, como BE é bissetriz, segue que


AE = EA' . Mas então os triângulos ADE e A'DE são congruentes, donde
DAˆ E = DAˆ ' E (I). Como AE e CF são perpendiculares a BD , então
AE // CF ⇒ EAˆ D = DCˆ F (II)
Também:
DFˆ C = DMˆ C = 90° ⇒ o quadrilátero MCFD é inscritível ⇒ DCˆ F = DMˆ F .
A' Mˆ D = A' Eˆ D = 90° ⇒ o quadrilátero A'MED é inscritível ⇒ EMˆ D = EAˆ ' D.
Mas, por (I) e (II), temos que EAˆ ' D = DCˆ F ⇒
⇒ EMˆ D = DMˆ F .
Obs. Por ABC ser escaleno, temos que BD não é perpendicular a AC , i.e,
E ≠ D ≠ F ≠ E.

PROBLEMA 5
A sucessão de números reais a1 , a 2 ,... define-se como:
1
a1 = 56 e a n +1 = a n − para cada inteiro n ≥ 1.
an
Demonstre que existe um inteiro k , 1 ≤ k ≤ 2002, tal que a k < 0.

EUREKA! N°15, 2002

19
Sociedade Brasileira de Matemática

SOLUÇÃO DE LARISSA CAVALCANTE QUEIROZ DE LIMA (FORTALEZA – CE)


Lema: a k < m; m, ak > 0
1
⇒ a k +1 < m −
m
1 1 1 1
Prova: a k < m ⇒ < ⇒− <−
m ak ak m
1 1 1
⇒ ak − < m − ∴ a k +1 < m −
ak m m
1 1 1
* a k +1 = a k − ⇒ a k +1 2 = a k 2 − 2a k ⋅ + 2
ak ak ak
1
⇒ a k +1 2 = a k 2 + 2
−2
ak
Soma telescópica
1
* a 2 2 = a1 2 + 2 − 2
a1
1
a3 = a 2 + 2 − 2
2 2

a2
1
(+) a k +1 2 = a k 2 + 2 − 2
ak
1 1 1
⇒ a k +1 2 = a1 2 + 2 + 2 + ... + 2 − 2 ⋅ k
a1 a2 ak
1) Suponha que ∃ i ≤ 1999 tal que ai < 2( ai > 0)
1 3 3 2 9−4 5
Isso implica: ai +1 < 2 − = . Caso a i + 1 > 0 , ai + 2 < − = = < 1.
2 2 2 3 6 6
1
Se ai + 2 > 0 ⇒ ai +3 < 1 − = 0; i + 3 ≤ 1999 + 3 = 2002
1
⇒ ∃k ,1 ≤ k ≤ 2002 tal que a k < 0.

2) Suponha que ∃/ i ≤ 1999 tal que ai < 2


1 1 1 1
⇒ ai ≥ 2 ∀ 1 ≤ i ≤ 1999 ⇒ ≥ ⇒ ≥ 2
2 ai 4 ai

EUREKA! N°15, 2002

20
Sociedade Brasileira de Matemática

1 1 1
⇒ a 2000 2 = 56 2 + 2
+ ... + 2
− 2(1999) ≤ 56 2 + 1999 ⋅ − 2 ⋅ 1999
a1 a1999 4
 1 7 7
⇒ a 2000 2 ≤ 56 2 − 1999 2 −  = 3136 − (2000 − 1) = 3136 − 3500 + < 0
 4 4 4
Absurdo!
Portanto ∃ i ≤ 1999 tal que ai < 2
⇒ temos que ai , ai +1 , a i + 2 ou ai + 3 é menor que zero (por 1)
∴ ∃k ,1 ≤ k ≤ 2002 tal que a k < 0.

PROBLEMA 6
Um polícia tenta capturar um ladrão num tabuleiro de 2001× 2001. Eles jogam
alternadamente. Cada jogador, na sua vez, deve mover-se uma casa num dos três
seguintes sentidos:

(abaixo); (direita); (diagonal superior esquerda).


Se o polícia se encontra na casa da esquina inferior direita, pode usar a sua jogada
para passar directamente para a casa da esquina superior esquerda (o ladrão não pode
fazer esta jogada). Inicialmente o polícia está na casa central e o ladrão está na casa
vizinha diagonal superior direita do polícia. O polícia começa o jogo.
Demonstre que:
a) O ladrão consegue mover-se pelo menos 10000 vezes sem ser capturado.
b) O polícia possui uma estratégia para capturar o ladrão.
Nota: O polícia captura o ladrão quando entra na casa em que está o ladrão. Se o
ladrão entra na casa do polícia, não há captura.

SOLUÇÃO OFICIAL: Pinte o tabuleiro com 3 cores da seguinte forma:


Figura1: Movimentos
1 a b c a c
2 b c a b c a
3 c a b c c a b
4  

c
1001 b c a … c …

a
b 

2001 c a b b
1 2 3… 2001

EUREKA! N°15, 2002

21
Sociedade Brasileira de Matemática

Observe que os movimentos nos dão o seguinte ciclo:


Quer dizer: de uma casa a só se vai para b, de uma b
a b só se vai para c e de uma c só se vai para a.

c
Inicialmente o polícia começa numa casa c e o ladrão também.
(casas hachuradas na figura)
Assim teremos as seqüências
Polícia: c → a → b → c O polícia nunca poderia entrar numa
casa de mesma cor do ladrão.
Ladrão: c → a → b → c
Para sua sorte existe o túnel.
Se pensarmos um pouco, veremos que o polícia deve atravessar o túnel 2 vezes para
poder tornar compatível seu ciclo com o do ladrão, ou seja, jogar e cair numa casa de
mesma cor do ladrão (podendo pegá-lo).
Seja X = casa superior esquerda
Y = casa inferior direita.
Logo o polícia precisa de 2000 movimentos para chegar até Y, cruza o túnel (1
movimento), mais 4000 para chegar de novo até Y, cruza o túnel (1 movimento).
Neste momento o ladrão deve estar próximo de Y e o polícia precisará de mais 3999
movimentos (pelo menos) para capturar o ladrão (que ficará rodando no quadrado 2
× 2 inferior esquerdo, totalizando
2000 + 4000 + 3999 + 2 = 10001 movimentos do polícia, ou seja, 10000
movimentos do ladrão.
b) Vejamos agora uma estratégia para que o polícia prenda o ladrão.
Suponha que ele já tenha passado 2 vezes pelo túnel.
Numere as linhas do tabuleiro de 1 a 2001, de cima para baixo e as colunas de 1 a
2001, da esquerda para a direita.
Após sair do túnel, o polícia se encontra na casa da linha 1 e coluna 1.
A estratégia é a seguinte:

i) O polícia deve se mover para a direita até que o ladrão fique na mesma
diagonal (inferior direita) do polícia ou uma casa à direita.
ii) Em seguida, o polícia deve fazer o seguinte movimento:

EUREKA! N°15, 2002

22
Sociedade Brasileira de Matemática

Ladrão Joga Polícia Joga

iii) O polícia deve repetir os passos i) e ii) Note que a diferença entre os
números das linhas do polícia e do ladrão sempre diminui e o ladrão sempre
fica na região à direita da diagonal inferior direita do polícia.
Assim, após repetirmos i) e ii) um número finito de vezes, chegamos na situação (na
vez do ladrão)

Polícia
Ladrão

(note que não pode ocorrer outra situação na qual a diferença entre linhas é 1, pois,
na vez do ladrão, os dois devem estar em casas do mesmo tipo).
Seguindo o que é feito em ii), o ladrão, em algum momento, se move para a casa à
direita do polícia. Assim, na próxima jogada, o polícia prende o ladrão.

EUREKA! N°15, 2002

23
Sociedade Brasileira de Matemática

A FÓRMULA DE CARDANO ALÉM DAS CÚBICAS


José Cloves Verde Saraiva, São Luis – MA

♦ Nível Avançado.

INTRODUÇÃO:
Motivado pela leitura do trabalho Equação do Terceiro Grau do Professor Alberto de
Azevedo [1], ocorreu-me a curiosidade de saber as fórmulas das raízes calculadas
por radicais de uma equação polinomial do 5º grau, solúvel, que não fosse a trivial
x 5 + a = 0 , que todos conhecem. Daí então, seguindo os mesmos passos da dedução
da fórmula de Cardano para as equações polinomiais cúbicas foi possível provar que
1
a x 5 − px 3 + p 2 x − r = 0 , já estudada por DE MOIVRE, tem raízes dadas por uma
5
fórmula análoga a de Cardano. Além desta, outras fórmulas semelhantes são
possíveis deduzir para graus maiores que o quinto.
Deixamos para o leitor essa generalização!.

A FÓRMULA DE CARDANO:
É fascinante toda a história da resolução das equações polinomiais do 3º grau. Em
resumo a referência [2] apresenta o seguinte:
"O descobridor do método foi Scipione del Ferro (1465 - ≈ 1562), matemático
italiano, que antes de morrer o revelou aos discípulos Antônio Maria Fior e Annibale
Della Nave".
"Houve uma disputa matemática entre Fior contra Niccolo Fontana (1500 – 1557),
conhecido pelo apelido de Tartaglia (gago, em italiano). A vitória deste último,
muito divulgada, foi do conhecimento do médico e professor Girolano Cardano
(1501 – 1576) que conseguiu lhe atrair para ensinar a regra de resolução sob o
juramento de jamais publicá-la. Cardano procurou a demonstração da regra - e achou
- e ainda motivou seu discípulo Ludovico Ferrari (1522 – 1565) a descobrir solução
para as equações do quarto grau."
"Cardano, numa visita a Della Nave, soube do manuscrito de Del Ferro contendo a
regra de Tartaglia que já existia há 30 anos. Motivo que o levou quebrar o juramento.
Publicou os métodos no seu famoso livro Ars Magna, em 1545, onde não deixou de
fazer referência aos descobridores, embora a contragosto de Tartaglia que se
considerou traído."
Podemos representar a equação geral do 3º grau na forma x 3 + a1 x 2 + a 2 x + a3 = 0 e
a
por uma mudança de variável x = y − 1 a equação fica mais simples na forma
3
EUREKA! N°15, 2002

24
Sociedade Brasileira de Matemática

y 3 − py − q = 0. Calculando o cubo de um binômio (u + v) 3 = u 3 + 3u 2 v + 3uv2 + v 3 , e


pondo em evidência 3uv, temos:
(*) (u + v ) 3 = 3uv(u + v ) + (u 3 + v 3 )
ou melhor,
(u + v ) 3 − 3uv (u + v) − (u 3 + v 3 ) = 0
isto é, y = u + v é uma raiz para valores de p = 3uv e q = u3 + v3, onde podemos
p3
elevar ao cubo a primeira e ter = u 3 v 3 e q = u 3 + v 3 e cair num problema onde
27
u3 e v3 são as raízes de uma equação do 2º grau conhecendo a soma e o produto das
raízes, cuja solução é conhecida:
q q2 p3 q q 2 p3
u3 =
+ − e v3 = − −
2 4 27 2 4 27
donde obtemos a famosa fórmula de Cardano:
q q2 p3 3 q q2 p3
y =u+v= 3 + − + − −
2 4 27 2 4 27

A FÓRMULA DE CARDANO ALÉM DAS CÚBICAS:


O resultado principal destas notas foi motivado por uma analogia da dedução na
1
fórmula de Cardano. Vamos provar que: a equação x 5 − px 3 + p 2 x − r = 0 tem
5
uma de suas raízes dada pela fórmula:
r r2 p5 r r2 p5
x=u +v=5 + + +5 − +
2 4 3125 2 4 3125

DEMONSTRAÇÃO:
Considere x 5 − px 3 − qx − r = 0 . Calculemos polinômios do binômio:
(u + v ) 5 = u 5 + 5u 4 v + 10u 3 v 2 + 10u 2 v 3 + 5uv 4 + v 5
pondo em evidência obtemos:
(u + v ) 5 = 5uv(u 3 + v 3 ) + 10u 2 v 2 (u + v) + (u 5 + v 5 )
da igualdade (*) obtemos que:
(u 3 + v 3 ) = (u + v) 3 − 3uv(u + v)
para substituir no desenvolvimento donde obtemos que:
(u + v ) 5 = 5uv(u + v ) 3 − 15u 2 v 2 (u + v ) + 10u 2 v 2 (u + v) + (u 5 + v 5 ) ,
isto é,

EUREKA! N°15, 2002

25
Sociedade Brasileira de Matemática

(u + v ) 5 = 5uv(u + v ) 3 − 5u 2 v 2 (u + v ) + (u 5 + v 5 )
o que permite obter as igualdades: p = 5uv; q = –5u2v2 e r = u5 + v5.
p2
Estabelecemos p 2 = 25u 2 v 2 , logo temos que q = − faz com que a equação seja
5
1
da forma x 5 − px 3 + p 2 x − r = 0, se x = u + v for uma raiz.
5
r = u 5 + v 5
Verificar as relações  nos leva ao já estudado na dedução da fórmula de
 p = 5uv
5
 p
Cardano r = u 5 + v 5 e   = u 5 v 5 , da mesma forma as raízes são:
5
r r 2 p5 r r2 p5
u5 =
+ − 5 e v5 = − −
2 4 5 2 4 55
de onde concluímos que a raiz x = u + v é dada pela fórmula:
r r2 p5 r r2 p5 .
x = 5 + − + 5 − −
2 4 3125 2 4 3125

OBSERVAÇÕES FINAIS:
Esta fórmula torna mais fácil a determinação das raízes do que a indicada por De
Moivre estudada na referência [3], onde uma análise completa das raízes e o estudo
dos Grupos de Galois nos diversos casos é feito.
Como exercício estude a sétima x 7 + px 5 − qx 3 + rx + s = 0 e generalize.
Finalizando, seria interessante o leitor paciente calcular todas as raízes da equação
abaixo estudada por Adriaan van Roomen (1561 – 1615) por polinômios
trigonométricos.
x 45 − 45 x 43 + 945 x 41 − 12300 x 39 + 111150 x 37 − 740259 x 35 + 3764565x 33 −
− 14945040 x 31 + 46955700 x 29 − 117679100 x 27 + 236030652 x 25 − 378658800 x 23 +
+ 483841800 x 21 − 488494125 x19 + 384942375 x17 − 232676280 x15 + 105306075 x13 −
− 34512075x 11 + 7811375 x 9 − 1138500 x 7 + 95634 x 5 − 3795 x 3 + 45 x = 2 + 2
(Ver referência [4], pp 154).

REFERÊNCIAS:
[1] Alberto de Azevedo, Equação do 3º grau, Depto. Matemática, UNB, 2002.
[2] César Polcino Milies, A Resolução das equações de terceiro e quarto graus, Notas de aula, IME-
USP, 2000.
[3] R.L. Borger, On De Moivre' s Quintic, American Math. Monthly, pp. 171 - 174, vol. 15, 1908.
[4] Paulo A. Martin, Introdução à Teoria dos Grupos e a Teoria de Galois, IME-USP, 1996.

EUREKA! N°15, 2002

26
Sociedade Brasileira de Matemática

RECIPROCIDADE QUADRÁTICA
Carlos Gustavo T. de A. Moreira & Nicolau C. Saldanha, Rio de Janeiro - RJ

♦ Nível Avançado.

A lei de Gauss de reciprocidade quadrática afirma que se p e q são primos há


uma relação direta entre p ser quadrado módulo q e q ser quadrado módulo p. Este
teorema fornece um rápido algoritmo para determinar se a é quadrado módulo p
onde a é um inteiro e p um número primo. Lembramos que a é quadrado módulo n
se existe x ∈ = com x 2 ≡ a(mod n). Este artigo foi adaptado de [3].
a
Definição: Seja p um primo e a um inteiro. Definimos o símbolo de Lagrange  
 p
por
0 se p divide a
a 
  = − 1 se a não é quadrado módulo p
 p  1 se p não divide a e a é quadrado módulo p.

Proposição: Seja p um primo ímpar e a ∈ = tal que p não divide a.
p −1
a
Então   ≡ a 2 (mod p).
 p
Demonstração: Sabemos que se p não divide a então a p −1 ≡ 1(mod p) , ou seja,
x p −1 − 1 tem como raízes 1, 2,…, p – 1 em =/p=. Por outro lado,
 p −1
 p −1

x p −1 − 1 =  x

2 − 1 x

2 + 1. Se existe b ∈

= tal que a ≡ b 2 (mod p) então
  
p −1
a
p−1
a 2 ≡ b p −1 ≡ 1(mod p ); ou seja,   = 1 ≡ a 2 (mod p ).
 p
p −1
Como x 2 ≡ y 2 (mod p) ⇔ x ≡ ± y (mod p) , há pelo menos quadrados em
2
p −1
(=/p=)*, logo os quadrados são exatamente as raízes de x 2 − 1 em =/p=, donde os
p −1
b
não quadrados são exatamente as raízes de x 2 + 1 , ou seja, se   = −1 então
 p
p −1

b 2 ≡ −1(mod p ).

EUREKA! N°15, 2002

27
Sociedade Brasileira de Matemática

p −1
 −1
Corolário: Se p é primo ímpar então   = (− 1) .
2

 p
Vamos agora reinterpretar a proposição. Seja a ∈ (= / p=)* . Para cada
p −1  p − 1
j = 1, 2,...,escrevemos a ⋅ j como ε j m j com ε j ∈ {−1,1} e m j ∈ 1,2,..., .
2  2 
Se mi = m j temos a ⋅ i = a ⋅ j ou a ⋅ i = −a ⋅ j; a primeira possibilidade implica
i = j e a segunda é impossível. Assim, se i ≠ j temos mi ≠ m j donde
   p − 1
m1 ; m2 ;...; m p −1  = 1, 2,..., . Assim,
 2    2 

p −1 (a ⋅ 1)(a ⋅ 2)... a ⋅ p − 1 
a  2 
  ≡ a 2 = ≡
 p p − 1
1 ⋅ 2 ⋅ ... ⋅
2
ε 1ε 2 ...ε p −1 m1 ⋅ m 2 ⋅ ... ⋅ m p −1
≡ 2 2
= ε 1ε 2 ...ε p−1 (mod p)
p −1
1 ⋅ 2 ⋅ ... ⋅ 2
2
a a
donde   = ε 1ε 2 ...ε p −1 , pois ambos pertencem a { – 1, 1}. Assim,   = (− 1) ,
m

 p 2  p
 p − 1
onde m é o número de elementos j de 1, 2,...,  tais que ε j = −1. Como
 2 
primeira conseqüência deste fato temos o seguinte resultado.

Proposição: Se p é um primo ímpar então


2 p 2 −1 1, se p ≡ ±1 (mod 8),
  = (− 1) 8 = 
 p − 1, se p ≡ ±3 (mod 8).
p −1
Demonstração: Se p ≡ 1(mod 4) , digamos p = 4k + 1, temos = 2k . Como
2
p −1 p −1
1≤ 2 j ≤ para j ≤ k e < 2 j ≤ p − 1 para k + 1 ≤ j ≤ 2k , temos
2 2
a 1, se p ≡ 1 (mod 8),
  = (− 1)k = 
 p − 1, se p ≡ 5 (mod 8).

EUREKA! N°15, 2002

28
Sociedade Brasileira de Matemática

p −1
Se p ≡ 3(mod 4) , digamos p = 4k + 3, temos = 2k + 1. Para 1 ≤ j ≤ k temos
2
p −1 p −1
1≤ 2 j ≤ e para k + 1 ≤ j ≤ 2k + 1 temos < 2 j ≤ p − 1 , donde
2 2
a − 1, se p ≡ 3 (mod 8),
  = (− 1)k +1 = 
 p 1, se p ≡ 7 (mod 8).
Teorema: (Lei de reciprocidade quadrática) Sejam p e q primos ímpares.
 p q
Então   = (− 1)( p −1)( q −1) / 4  .
q  p
Demonstração: Na notação acima, com a = q, para cada j ∈ P, onde
P = {1,2,..., ( p − 1) / 2},
temos que ε j = −1 se e só se existe y ∈ = tal que − ( p − 1) / 2 ≤ qj − py < 0. Tal y
deve pertencer a Q, onde Q = {1,2,..., (q − 1) / 2}.
q
Assim, temos que   = (−1) m onde m = X e
 p
X = {( x, y ) ∈ P × Q − ( p − 1) / 2 ≤ qx − py < 0};
 p
note que qx – py nunca assume o valor 0. Analogamente,   = (− 1)n , onde n = Y
q
e Y = {( x, y ) ∈ P × Q 0 < qx − py ≤ (q − 1) / 2}.
 p  q 
Daí segue que    = (−1) k onde k = m + n = Z onde
 q  p 
Z = {( x, y ) ∈ P × Q − ( p − 1) / 2 ≤ qx − py ≤ ( q − 1) / 2}
pois qx – py nunca assume o valor 0. Temos k = C − A − B onde C = P × Q,
A = {( x, y ) ∈ C qx − py < −( p − 1) / 2},
B = {( x, y ) ∈ C qx − py > (q − 1) / 2}.
Como C = ( p − 1)(q − 1) / 4 , basta mostrar que A = B . Mas f : C → C definida
por f ( x, y ) = ((( p + 1) / 2) − x, (( q + 1) / 2) − y ) define uma bijeção entre A e B [

EUREKA! N°15, 2002

29
Sociedade Brasileira de Matemática

Exemplo: Se n ≥ 1 e p = 2 2 + 1 é primo, então 3 não é quadrado módulo p (e logo 3


n

é raiz primitiva módulo p; ver [1]).


 3  p
De fato, como p ≡ 1(mod 4),   =   , mas 2 2 ≡ 1(mod 3) , como pode ser
n

 p  3
 p 2
facilmente mostrado por indução, donde p = 2 2 + 1 ≡ 2(mod 3) , e   =   = −1.
n

 3  3

REFERÊNCIAS:

[1] Carlos Gustavo T. de A. Moreira, Divisibilidade, Congruências e Aritmética módulo n, Eureka! No.
2, pp. 41-52, 1998.
[2] Guilherme Camarinha Fujiwara, Inteiros de Gauss e Inteiros de Eisenstein, Eureka! No. 14, pp. 23-
31, 2002.
[3] Carlos Gustavo T. de A. Moreira e Nicolau C. Saldanha, Primos de Mersenne (e outros primos
muito grandes), 22o. Colóquio Brasileiro de Matemática, IMPA, 1999.

EUREKA! N°15, 2002

30
Sociedade Brasileira de Matemática

APLICAÇÕES DE PLANOS PROJETIVOS


EM TEORIA DOS NÚMEROS E COMBINATÓRIA
Carlos Yuzo Shine, São Paulo - SP

♦ Nível Avançado.

1. Definição de plano projetivo


A definição dada aqui é a mais geral. Dizemos que um conjunto S é um plano
projetivo se existem subconjuntos  1 ,  2 ,... de S que satisfazem as seguintes
propriedades:
(i) Se P e Q pertencem a S, um e somente um dos subconjuntos  i contém P e
Q.
(ii) A interseção de  i e  j consiste sempre de um único elemento, para todo
i ≠ j.
(iii) Existem pelo menos quatro elementos de S tais que, entre eles não haja três
contidos em um dos conjuntos  i .
Os elementos de S são normalmente chamados pontos e os subconjuntos  i , retas.
Note que a propriedade (i) pode ser entendida como "por dois pontos passa uma
única reta" e a propriedade (iii) nos diz que "existem quatro pontos, três a três não
colineares".

Exercícios
01. Um torneio de tênis é disputado entre duas equipes. Cada membro de uma
equipe joga com um ou mais membros da outra equipe, de modo que
(i) Dois membros de uma mesma equipe têm exatamente um oponente em
comum;
(ii) Não existem dois membros de uma equipe que enfrentam, juntos, todos os
membros da outra equipe.
Prove que cada jogador deve jogar um mesmo número de partidas.

02. Mostre que as seguintes proposições são equivalentes em planos projetivos:


(1) Existe uma reta que passa por exatamente n + 1 pontos;
(2) Existe um ponto que está contido em exatamente n + 1 retas;
(3) Todas as retas passam por exatamente n + 1 pontos;
(4) Todos os pontos estão contidos em exatamente n + 1 retas;
(5) Há exatamente n2 + n + 1 retas;
(6) O plano projetivo tem exatamente n2 + n + 1 pontos (diz-se nesse caso que o
plano projetivo tem ordem n).

EUREKA! N°15, 2002

31
Sociedade Brasileira de Matemática

03. (Cone Sul 1998) O prefeito de uma cidade deseja estabelecer um sistema de
transportes com pelo menos uma linha de ônibus, no qual:
(i) cada linha passe exatamente por três paradas;
(ii) cada duas linhas distintas tenham exatamente uma parada em comum;
(iii) para cada duas paradas de ônibus distintas exista exatamente uma linha que
passe por ambas.
Determine o número de paradas de ônibus da cidade.
Dica: com o que parecem as condições (ii) e (iii)? Para encontrar os possíveis
valores, faça uma contagem dupla.

2. Construção de planos projetivos baseados em corpos


Seja K um corpo (ou seja, um conjunto onde estão definidas duas operações, + e ⋅, tal
que todo elemento admite oposto e todo elemento não nulo admite inverso). Então o
conjunto P2(K) de ternas (x, y, z) ≠ (0, 0, 0), x, y, z ∈ K, onde ternas da forma (x, y, z)
e (kx, ky, kz) devem ser consideradas iguais, é um plano projetivo na qual a reta
correspondente ao ponto (a, b, c), reta dual de (abc), é o conjunto de pontos (x, y, z)
que satisfazem
ax + by + cz = 0

Demonstração
Basta mostrar que o conjunto dado tem as propriedades (i), (ii) e (iii).
(i) Sejam (x1, y1, z1) e (x2, y2, z2) pontos distintos, ou seja, tais que não existe
λ ∈ K tal que ( x1 , y1 , z1 ) = λ ( x2 , y2 , z2 ) . Temos que mostrar que existe
somente uma reta que contém ambos os pontos, ou seja, que existe uma
única terna (a, b, c) (que são os coeficientes da reta) tal que
ax1 + by1 + cz1 = 0
(*)
ax 2 + by 2 + cz 2 = 0
Vejamos (*) como um sistema em a, b e c.
Como ( x1 , y1 , z1 ) ≠ λ ( x 2 , y 2 , z 2 ) para todo λ ∈ K , a matriz completa
 x1 y1 z1 
C =   do sistema (*) tem posto 2. Suponhamos, sem perda de
 x2 y2 z 2 
x1 y1
generalidade, que ≠ 0. Logo, resolvendo (*) em a e b, obtemos (verifique!)
x2 y2
a = mc e b = nc, onde m e n são constantes de K. Logo, as soluções de (*) são da
forma (mc, nc, c) = c(m, n, 1). Note que c ≠ 0 pois c = 0 implicaria a = b = c = 0.
Assim, no plano projetivo P2(K), todas as soluções são equivalentes, ou seja, é única.
Logo existe somente uma reta que passa por esses dois pontos.

EUREKA! N°15, 2002

32
Sociedade Brasileira de Matemática

(ii) Análogo ao item (i). Tente você fazer!


(iii) Observe que (1, 1, 1), (1, 0, 0), (0, 1, 0) e (0, 0, 1) satisfazem essas
condições. Aqui, 0 e 1 são os elementos neutros da soma e produto em K,
respectivamente.

3. Dualidade
Observe que se r e s são retas duais dos pontos R = (a, b, c) e S = (d, e, f) de P2(K),
respectivamente, então
R ∈ s ⇔ da + eb + fc = 0 ⇔ ad + be + cf = 0 ⇔ S ∈ r
A propriedade R ∈ s ⇔ S ∈ r é a chave do princípio da dualidade. Por isso, pontos
e retas no plano projetivo se comportam de maneira semelhante quando se fala de
incidência.

4. O caso K = 5
Quando K = 5, temos o plano projetivo P2(5) visto na referência [2]. Vamos
entender a semelhança entre o plano definido aqui e o plano estudado em [2] (se
você não leu essa referência, você pode pular essa seção; mas leia o artigo, ele
realmente vale muito a pena!!).
Primeiro, vamos explicar a primeira "estranheza" do plano projetivo P2(5). Por que
ele tem três coordenadas, e não duas? A coordenada adicional é que faz aparecerem
pontos e retas no infinito. Podemos fazer a seguinte correspondência: o ponto (x, y)
do plano 52 corresponde ao ponto (x, y, 1) de P2(5). Sobraram os pontos do tipo
(x, y, 0) de P2(5), que são os pontos do infinito.
A reta ax + by + c = 0 de 52 pode ser transformada na reta ax + by + cz = 0 de P2(5)
substituindo-se x e y na reta de 52 por x/z e y/z. Aqui, nós homogeneizamos a
equação ax + by + c = 0.
Você pode visualizar a correspondência que fizemos da seguinte forma: se
imaginarmos as ternas (x, y, z) como pontos no espaço, percebemos que os pontos de
P2(5) correspondem a retas que passam pela origem. Se tomarmos o plano α : z = 1
de 53, por exemplo, fazemos corresponder um ponto de P2(5) com o traço da reta
correspondente nesse plano.

EUREKA! N°15, 2002

33
Sociedade Brasileira de Matemática

A(a, b, 1) α

y λ ( a, b,1)

A reta dual de (a, b, c) corresponderia ao traço do plano ax + by + cz = 0 em π.


Com alguns cálculos verifica-se que se R = (a, b, c) e r: ax + by + cz = 0, então,
sendo O = (0, 0, 1), R' e r'os respectivos correspondentes ao ponto R e à reta r em α,
então OR' ⊥ r'.

4.1. O plano projetivo e curvas elípticas


Considere a curva cúbica de R2
ax 3 + bx 2 y + cxy 2 + dy 3 + ex 2 + fxy + gy 2 + hx + iy + j = 0, (**)
onde todas as letras de a a j são números racionais.
Gigantesco, não? Estamos interessados em saber sobre os seus pontos racionais (isto
é, pontos cujas coordenadas são ambas racionais). Para isso, é feita a seguinte
operação: tomamos um ponto racional da curva, denominado O. Dados dois pontos
P e Q racionais da curva, encontre o terceiro ponto P * Q de interseção de PQ com a
curva. Defina P + Q (isso mesmo, estamos somando pontos!) como o terceiro ponto
de interseção da reta que passa por O e P * Q com a curva (isto é, P + Q = O * (P *
Q). Pode-se provar que P + Q também é um ponto racional.

P O
Q
P*Q

P+Q

Só que a gigantesca equação (**) pode ser simplificada para a forma


y2 = x3 + ax2 + bx + c, (✯)

EUREKA! N°15, 2002

34
Sociedade Brasileira de Matemática

que é bem mais simples. Esta é a forma de Weierstrass. Para isso, usamos a
geometria projetiva.
Vamos fazer um caso particular. Considere a curva u3 + 2v3 = 2uv + 1. Um de seus
pontos racionais é (1; 1).
Primeiro devemos homogeneizar a curva. Fazendo u = U/W, obtemos U3 + 2V3 =
2UVW + W3. Note que agora vamos trabalhar no plano projetivo P2(5).
Depois, escolhemos um ponto racional O na curva. Uma escolha é O = (1; 1; 1).
Agora, nós vamos mudar as coordenadas. A tangente à curva pelo ponto O é o nosso
eixo de equação Z = 0. Se a tangente encontra a curva novamente em P (isso sempre
ocorre se O não for ponto de inflexão), o eixo de equação X = 0 é a tangente à curva
que passa por P. O eixo de equação Y = 0 pode ser qualquer reta que passe por O. Se
O for ponto de inflexão, podemos escolher qualquer reta que não passa por O como
o eixo X = 0. Observe que como P2(5) tem três coordenadas, precisamos de três
eixos.

Y= 0

X=0
Z=0

Fazendo algumas contas (use um pouquinho de cálculo; para trabalhar só com duas
variáveis, "desomogenize" - essa palavra existe? - a equação), obtemos que a
tangente a O = (1; 1; 1) é a reta U + 4V – 5W = 0. Esta reta corta novamente a curva
em (–3; 2; 1). A tangente por esse ponto (use cálculo novamente) é 23U + 30V + 9W
= 0. Por fim, tomamos a reta U – V = 0 como o eixo Y = 0. Assim, queremos que
5 X + 3Y + 9Z
U =
U + 4V − 5W =Z 310
5 X − 2Y + 9Z
62U − 62V =Y ⇔ V =
310
23U + 30V + 9W =X
5 X − Y − 53Z
W =
310
(usamos Y = 62U – 62V para simplificar um pouco as contas).
Substituindo U, V e W (veja que o 310 dos denominadores vai cancelar), temos

EUREKA! N°15, 2002

35
Sociedade Brasileira de Matemática

(5 X + 3Y + 9 Z ) 3 + 2(5 X − 2Y + 9 Z ) 3 = 2(5 X + 3Y + 9Z ) ⋅ (5 X − 2Y + 9 Z ) ⋅
(5 X − Y − 53Z ) + (5 X − Y − 53Z ) 3
A equação parece mais gigantesca que antes, mas após "desomogeneizar" (basta
substituir Z = 1) e abrir tudo (eu colocaria as contas, mas a margem neste artigo é
muito pequena para isso) obtemos
xy 2 − ( 4 x − 30) y = −25x 2 + 96 x − 515
Em geral, quando fazemos essa mudança de eixos (a transformação que fizemos ao
resolver o sistema é uma transformação projetiva) a equação se reduz à forma
xy 2 + ( ax + b) y = cx 2 + dx + e
Multiplicando por x nos dois membros, temos (xy )2 + ( ax + b) xy = cx 3 + dx 2 + ex
Agora substitua z = xy: z 2 + ( ax + b) z = cx 3 + dx 2 + ex
1
Completando quadrado no primeiro membro e substituindo z + ( ax + b) por t
2
chegamos finalmente em t 2 = cúbica em x
Para a cúbica ser mônica (ou seja, ter coeficiente dominante unitário e não ser
dentuça), como o coeficiente dominante é c, basta multiplicar ambos os membros
por c2 e fazer as substituições (são as últimas!) t = m/c e x = n/c. Vamos fazer isso no
nosso exemplo.
Multiplicando por x, fazendo z = xy e completando quadrado:
z 2 − ( 4 x − 30) z + ( 2 x − 15) 2 = −25 x 3 + 96 x 2 − 515x + (2 x − 15) 2
⇔ ( z − 2 x + 15) 2 = −25 x 3 + 100 x 2 − 575 x + 225
Agora, t = z – 2x + 15: t 2 = −25 x 3 + 100 x 2 − 575 x + 225
m n
Por fim, t = − e x=− e chegamos à forma de Weierstrass:
25 25
m 2 = n 3 + 100n 2 + 14375n + 140625

Se você escrever m e n em função de u e v (e vice-versa), vai obter funções racionais


(isto é, o quociente de dois polinômios) com todos os coeficientes racionais. Logo
pontos racionais são transformados em pontos racionais, de modo que achar os
pontos racionais da curva original é a mesma coisa que achar os pontos racionais da
curva na forma de Weierstrass (fora, é claro, os pontos que anulam os
denominadores das funções racionais, que não são muitos e são fáceis de achar).
Qual a vantagem da forma de Weierstrass? A vantagem é que podemos normalizar a
curva para a forma (✯), além da soma de pontos. O ponto O pode ser um ponto
infinito, por exemplo. Homogeneizando (✯), obtemos Y 2 Z = X 3 + aX 2 Z + bXZ2 + cZ3 ,

EUREKA! N°15, 2002

36
Sociedade Brasileira de Matemática

e não é difícil ver que o ponto do infinito (0; 1; 0) pertence à curva (e além disso, é o
único!). Então podemos fazer O = (0; 1; 0). Isso facilita um pouco as contas para
adição de pontos. Se P * Q = (a; b)((a; b; 1) no plano projetivo), a reta que passa por
O e P * Q é Y = bZ, ou y = b. Conseqüentemente, como a curva é simétrica em
relação ao eixo Ox, temos P + Q = (a; –b).

P
Q
P*Q
x
P+Q

Exercícios
04. Prove que se P e Q são pontos racionais numa cúbica então P + Q também é
racional.

05. Transforme a cúbica u3 + v3 = α na forma de Weierstrass (um ponto racional da


curva homogeneizada é (1; –1; 0) e a conta não é tão terrível assim).
06. (OBM 2002, nível U) Considere a curva C = {( x; y ) ∈ R 2 | y 2 = x 3 − 43x + 166}.
(a) Seja Q = (a; b) um ponto de C. Suponha que a reta tangente a C no ponto Q
intersecte C num único outro ponto, Q'. Determine as coordenadas de Q'.
(b) Seja P0 = (3; 8). Para cada inteiro não negativo n, definimos Pn + 1 = P'n, o ponto
de interseção de C com a reta tangente a C em Pn. Determine P2002.

07. A adição de pontos é definida somente quando a cúbica no segundo membro de


(✯) tem três raízes distintas (não necessariamente todas reais). Os outros casos
são mais fáceis!

(a) Encontre todos os pontos racionais da curva y2 = x2 (x – 1).


(b) Encontre todos os pontos racionais da curva y2 = x3.

EUREKA! N°15, 2002

37
Sociedade Brasileira de Matemática

5. O caso K = =/p=
Sendo K qualquer corpo, podemos tomar K = =/p= (os inteiros vistos mod p), em
que p é um primo. Nesse caso, sendo =/p= um corpo finito (com p elementos) o
plano projetivo P2(=/p=) é finito.
Por exemplo, fazendo p = 2, obtemos o plano de Fano (rimou!), como pode ser visto
na figura.
(0, 0, 1)

(0, 1, 1) (1, 1, 1)

(1, 0, 1)

(0, 1, 0) (1, 0, 0) (1, 1, 0)

Vamos fazer algumas continhas.

6. Algumas contagens

6.1. Número de elementos de P2(=/p=)


Temos p3 – 1 ternas (a, b, c) ≠ (0, 0, 0). Como (a, b, c) é equivalente a λ (a, b, c) e
λ pode assumir p – 1 valores (1 a p – 1), cada ponto está sendo contado p – 1 vezes.
p3 −1
Logo P2(=/p=) tem = p 2 + p + 1 pontos.
p −1
Observe que, pelo princípio da dualidade, há também p2 + p + 1 retas.

6.2. Número de pontos em cada reta


Fixados a, b, e c, não todos nulos, queremos contar o número de soluções não
equivalentes (x, y, z) ≠ (0, 0, 0) da congruência
ax + by + cz ≡ 0 (mod p) (***)
Suponhamos, sem perda de generalidade, que c ≠ 0 (mod p). Temos então
(***) ⇔ z ≡ − ac −1 x − bc −1 y (mod p)

EUREKA! N°15, 2002

38
Sociedade Brasileira de Matemática

Podemos escolher x e y de p2 maneiras. Porém, não podemos escolher x = y = 0, pois


isso implicaria z = 0. Logo, considerando que cada elemento de P2(=/p=) tem p – 1
p 2 −1
equivalentes, temos que cada reta tem = p + 1 pontos.
p −1

6.3. Número de retas que passam por um ponto


Pelo princípio da dualidade, há p + 1 retas que passam por um ponto dado.

7. Um problema
A seguir, o problema 3 da Olimpíada Iberoamericana de 1996, realizada na Costa
Rica.
Temos um tabuleiro quadriculado de k 2 − k + 1 linhas e k 2 − k + 1 colunas, onde
k = p + 1 e p é um número primo. Para cada primo p, dê um método para distribuir
números 0 e 1, um número em cada casa do tabuleiro, de modo que em cada linha
haja exatamente k números 0, em cada coluna haja exatamente k números 0 e, além
disso, não haja nenhum retângulo, de lados paralelos aos lados do tabuleiro, com
números 0 em seus quatro vértices.

Resolução
Para k = p + 1, k 2 − k + 1 = ( p + 1) 2 − ( p + 1) + 1 = p 2 + p + 1 (coincidência? destino?
ou puramente sorte?). Assim, devemos preencher um tabuleiro (p2 + p + 1) ×
(p2 + p + 1) com 0 e 1 de modo que haja p + 1 (que coisa…) 0 em cada linha e
coluna, e sem retângulos com 0 vértices.
Considere o plano projetivo P2(=/p=) (por que será?) e a cada linha associe um
ponto e a cada coluna associe uma reta. Coloque 0 na casa (i, j) se, e somente se, o
ponto i pertence à reta j. Nas demais casas, coloque 1.
Há claramente p + 1 zeros em cada coluna. Pelo princípio da dualidade, também há
p + 1 zeros em cada linha.
Agora, suponha que exista um retângulo de vértices (a, c), (a, d), (b, c) e (b, d), a ≠ b
e c ≠ d, todos com 0. Logo, pela nossa construção, os pontos a e b pertencem a
ambas as retas c e d. Absurdo, pois a interseção de duas retas é exatamente um
ponto.
É claro que na prova você teria que demonstrar todas as propriedades que
demonstramos antes.

8. Outros fatos sobre planos projetivos finitos


A construção baseada em corpos vale para qualquer corpo. Mas infelizmente, o
número de elementos de um corpo finito deve ser potência de primo (um corpo com

EUREKA! N°15, 2002

39
Sociedade Brasileira de Matemática

pn elementos, p primo, n inteiro positivo, é o conjunto dos polinômios com


coeficientes em =/p=, vistos módulo um polinômio P(x) de grau n e irredutível em
=/p=[x]). Assim, tal construção só nos permite construir planos projetivos de ordem
potência de primo. Será que existem planos projetivos com outra ordem?
Conjectura-se que não, porém esse problema continua em aberto. O teorema de
Bruck-Ryser-Chowla ajuda um pouquinho, dizendo que se n ≡ 1 (mod 4) ou n ≡ 2
(mod 4) e n é ordem de um plano projetivo então n deve ser soma de dois quadrados.
Pensando nos casos pequenos, esse teorema elimina o caso n = 6. A demonstração de
que não existe plano projetivo de ordem n = 10 foi obtida em 1989 por C. W. H.
Lam (com o auxílio de um computador!) e a história da prova está disponível em [7].
O caso n = 12 está em aberto.
Planos projetivos também servem para construir alguns block designs. Um block
design consiste num sistema de incidência (v, k, λ , r, b) na qual um conjunto X de v
pontos é particionado numa família A de b subconjuntos (chamados blocks) de modo
que dois pontos quaisquer determinam λ blocks com k pontos em cada block, e cada
ponto está contido em r blocks. Na verdade, os cinco parâmetros não são
independentes. Fica como exercício para você mostrar que vr = bk e λ (v – 1) =
r (k – 1) (são duas contagens duplas). Assim, pode-se representar o block design
simplesmente como (v, k, λ ). Se b = v (e, conseqüentemente, r = k) o block design é
dito simétrico. Note que um plano projetivo de ordem n é block design
(n2 + n + 1, n, 1). Outros exemplos e mais informações podem ser encontrados no
site: http://mathworld.wolfram.com/BlockDesign.html.

Exercícios
08. (OPM 2001) No condado Heptaprojetivo havia 7 castelos, batizados segundo
grandes personagens: Arnold (A), Borcherds(B), Conway (C), Dilbert (D), Erdös (E),
Faltings (F), Gowers (G). Havia também 7 ruas, cada uma com 3 castelos, como
mostra o mapa a seguir (uma rua é circular):
A

B G

D F C

EUREKA! N°15, 2002

40
Sociedade Brasileira de Matemática

Um belo dia, o conde Steiner decidiu retirar as placas que identificavam os castelos
para fazer uma limpeza. Na hora de recolocá-las, ninguém se lembrava do lugar
correto de cada uma, nem mesmo os moradores dos castelos! Os arquivos do
condado só indicavam os castelos que ficavam numa mesma rua, mas não a ordem
em que eles estavam. Assim, o conde sabia que havia uma rua com os castelos {A, B,
D}, outra com {B, C, E}, outra com {B, F, G}, etc.
Frente aos fatos, Steiner resolveu determinar todas as maneiras de recolocar as
placas respeitando os arquivos do condado, isto é, todas as maneiras nas quais placas
que estavam juntas em uma mesma rua continuassem juntas em uma rua,
possivelmente em outra ordem. Duas destas maneiras estão representadas a seguir:
A B

F C G A

B E

E D G F C D

Chamaremos essas maneiras de válidas.


(a) Prove que o total de maneiras válidas é 7 vezes o número de maneiras válidas
nas quais a placa A é colocada no castelo A.
(b) Prove que o total de maneiras válidas nas quais a placa A é colocada no castelo
A é 6 vezes o número de maneiras válidas nas quais a placa A é colocada no
castelo A e a placa B é colocada no castelo B.
(c) Determine o número de maneiras válidas.

09. No reino da Alândia há n cidades, assim como no reino da Belândia. Foram


construídas m estradas, sendo que cada estrada passa por exatamente duas
cidades, uma de cada reino. Mostre que se não existem quatro cidades ligadas
n
por um ciclo de estradas, então m ≤ (1 + 4n − 3 )
2
Mostre que a igualdade pode ocorrer para infinitos valores de n.
Dicas: a igualdade pode ser demonstrada com uma injeção e contagem dupla - veja
o artigo Grafos e Contagem Dupla, na Eureka! 12; para mostrar que a igualdade
pode ocorrer para infinitos valores de n, encontre primeiro os valores de n para os
quais 4n – 3 é um quadrado perfeito - você vai se surpreender!

EUREKA! N°15, 2002

41
Sociedade Brasileira de Matemática

10. (Extensão do exemplo do artigo Grafos e Contagem Dupla) Na Terra de Oz há n


castelos e várias estradas, sendo que cada uma liga dois castelos. Diz a lenda que
se houver quatro castelos ligados em ciclo (ou seja, se existirem quatro castelos
A, B, C, e D tais que A e B, B e C, C e D e D e A estão ligados), um dragão
aparecerá do centro dos castelos e destruirá a Terra de Oz. Mostre que para esta
desgraça não acontecer o número de estradas deve ser menor ou igual a
(
1 + 4n − 3
n
4
) é mostre que para infinitos n é possível construir

(
1 + 4n − 3
4
)
n −1
estradas. (para mostrar isto você vai precisar saber um pouco
de Álgebra Linear).

REFERÊNCIAS:

[1] D. Pedoe, Geometry: A Comprehensive Course. Este livro tem um bom texto não somente
introdutório sobre geometria projetiva e contém mais aplicações geométricas. O problema 1 e os
axiomas de planos projetivos foram retirados deste livro, assim como a construção de planos projetivos
com coordenadas.
[2] L. Castro, Introdução à Geometria Projetiva, Revista Eureka! 8. A melhor referência que conheço
para começar a estudar geometria projetiva aplicada a problemas de geometria.
[3] O problema 9 foi adaptado de um exemplo retirado do livro Graph Theory: An Introductory Course,
de B. Bollobás. Ele é um caso particular do problema de Zaranckiewsky: qual é o número máximo de
arestas de um grafo bipartido com m vértices em uma classe V1 e n vértices na outra classe V2, de modo
que não haja nenhum subgrafo bipartido completo com r vértices de V1 e s vértices de V2? Tal número é
normalmente representado por z(m; n; r; s).
[4] O problema 10 é uma extensão de um exemplo de meu artigo Grafos e Contagem Dupla, que está
na Revista Eureka! 12.
[5] A construção do exemplo do problema 10 pode ser encontrada no livro Proofs From The Book, de
Martin Aigner e Günter M. Ziegler. Mas tente você mesmo fazê-la antes!
[6] Curvas elípticas e a redução para a forma de Weierstrass podem ser encontradas em Rational Points
on Elliptic Curves, de Joseph H. Silverman e Jonh Tate.
[7] A história da demonstração de que não existe plano projetivo de ordem 10 está em
http://www.cecm.sfu.ca/organics/papers/lam/paper/html/paper.html
[8] Um glosário de termos da Matemática e principais fatos relacionados está em
http://mathworld.wolfram.com/
[9] Uma discussão muito interessante sobre o assunto está em
http://www.mathematik.uni-bielefeld.de/~sillke/PUZZLES/projective_plane
(o grande matemático John Conway inclusive participa desta discussão).

EUREKA! N°15, 2002

42
Sociedade Brasileira de Matemática

OLIMPÍADAS AO REDOR DO MUNDO


Þ Chegamos ao último número de 2002. Apresentamos, como sempre,
questões que não são encontradas facilmente na Internet. Feliz 2003, divirtam-se e
enviem as suas soluções.
Continuamos à disposição na OBM para aqueles que estiverem interessados
na solução de algum problema particular. Para tanto, basta contactar a OBM, através
de carta ou e-mail.
Antonio Luiz Santos

Primeiramente vamos aos problemas propostos deste número

181.(Rússia-2002) Um número de quatro algarismos é escrito em um quadro negro. É


permitido adicionar 1 a quaisquer dois algarismos adjacentes se nenhum deles for
igual a 9 ou subtrair 1 de quaisquer dois algarismos adjacentes se nenhum deles for
igual a 0. Partindo de 1234 é possível obtermos 2002 após efetuarmos estas
operações diversas vezes ?

182.(Rússia-2002) Seja ABC um triângulo cujas medidas dos lados são distintas duas a
duas. Exteriormente ao triângulo, são construídos sobre os seus lados os triângulos
eqüiláteros ABC1, BCA1 e CAB1. Mostre que o triângulo A1B1C1 não é equilátero.

183.(Rússia-2002) Um polinômio quadrático de coeficientes inteiros e coeficiente do


segundo grau igual a 1 assume valores primos em três valores inteiros e
consecutivos. Mostre que ele assume um valor primo em pelo menos mais um valor
inteiro.

184.(Rússia-2002) Seja O o circuncentro de um triângulo isósceles ABC(AB = BC). Um


ponto M pertence ao segmento BO e o ponto M' é o simétrico de M em relação ao
ponto médio de AB. Sejam ainda K o ponto de interseção de M'O com AB e L um
ponto sobre o lado BC tal que ∠ CLO = ∠ BLM . Mostre que os pontos O, K, B, L
pertencem a um mesmo círculo.

185.(Rússia-2002) Qual o maior número de termos possível de uma progressão


aritmética de números inteiros positivos (a1, a2, a3,…,an) de razão igual a 2 tal que o
número a k2 + 1 seja primo para todo k = 1, 2, 3,…,n?

186.(Rússia-2002) Sobre o eixo x são escolhidos pontos x1, x2,.., xn com n ≥ 3 distintos
dois a dois. Traçam-se então todas as parábolas de coeficientes do segundo grau
igual a 1 e que cortam o eixo dos x somente nos pontos escolhidos. Sejam
EUREKA! N°15, 2002

43
Sociedade Brasileira de Matemática

y = f1 , ... , y = f m as equações destas parábolas. Mostre que a parábola


y = f1 + ⋅ ⋅ ⋅ + f m corta o eixo x em exatamente dois pontos.

187.(Rússia -2002) Uma seqüência de números (an) é tal que a0 = 0 e a n +1 ≥ a n + 1 .


2
n
 n 
Mostre a desigualdade : ∑k =1
a k3 ∑
≥  a k 
 k =1 

n−
188.(Rússia-2002) No intervalo (22n, 32n) são escolhidos 2 2 1 + 1 números ímpares.
Mostre que podemos encontrar entre estes números dois números tais que o
quadrado de cada um deles não é divisível pelo outro.

189.(Albânia-2002) Sendo f : 5 ∗ → 5 uma função definida por

1 
1+ x
f (x ) =  2 x
− 2x 
3  

Determine o valor da soma :
 1   2   2002   2002   2002   2002 
f + f  + ⋅⋅⋅ + f  +2f +2f  + ⋅⋅ ⋅ + 2 f  
 2002   2002   2002   2001   2000   1 

190.(Albânia-2002) Mostre como dividir um triângulo qualquer em :


(i) 2002 triângulos semelhantes a ele.
(ii) 2003 triângulos semelhantes a ele.

191.(Irlanda-2002) Em um triângulo ABC tem-se que AB = 20, AC = 21 e BC = 29. Se


D e E são pontos do segmento BC tais que BD = 8 e EC = 9, determine a medida do
ângulo ∠DAE .

192.(Irlanda-2002) Suponha que n seja o produto de quatro números primos distintos a,


b, c, d tais que :
(i) a + c = d
(ii) a(a + b + c + d) = c (d – b)
(iii) 1 + bc + d = bd
Determine n.

EUREKA! N°15, 2002

44
Sociedade Brasileira de Matemática

193.(Irlanda-2002) Seja (an) uma seqüência definida por a1 = 1, a2 = 1, a3 = 1 e


a n +1 a n − 2 − a n an −1 = 2 para todo n ≥ 3 . Prove que an é um inteiro positivo para todo
n ≥ 1.

194.(Eslovênia-2002) 38 é o menor inteiro positivo tal que o seu quadrado termina com
três quatros (382 = 1444). Qual o próximo inteiro positivo com esta propriedade ?

195.(Eslovênia-2002) Sejam M o ponto médio da base AB do trapézio ABCD; E um


ponto interior ao segmento AC tal que BC e ME intersectam-se em F; G o ponto de
interseção de FD e AB; H o ponto de interseção de DE e AB. Mostre que M é o
ponto médio do segmento GH.

196.(Eslovênia-2002) Sejam k um círculo do plano euclidiano, k1 e k2 dois círculos


disjuntos interiores a k e que o tangenciam nos pontos A e B respectivamente. Seja
ainda (t) a tangente comum aos círculos k1 e k2 nos pontos C e D respectivamente tal
que k1 e k2 estejam situados num mesmo semiplano determinado pela reta (t)
enquanto que o centro do círculo k pertence ao outro semiplano. Se E é o ponto de
interseção de AC e BD, mostre que o ponto E pertence ao círculo k.

197.(Eslovênia-2002) Determine o menor inteiro positivo que pode ser escrito como
uma soma de 9, 10 e 11 inteiros positivos consecutivos.

198.(Estônia-2002) Em um triângulo ABC tem-se que ∠ B = 2 ⋅ ∠ C e a bissetriz do


ângulo ∠ A intersecta o lado BC no ponto D tal que AB = CD. Determine a medida
do ângulo ∠ A .

199.(Estônia-2002) Um número natural de 10 algarismos distintos é dito mágico se ele


for múltiplo de 99999. Quantos números mágicos não começados por zero existem ?

200. (Grécia-2002) Determine todos os inteiros positivos N tais que :


(i) N possui exatamente16 divisores 1 = d1 < d 2 < ... < d16 = N
(ii) d 5 = (d 2 + d 4 )d 6 .

201. (Inglaterra-2002) Um quadrilátero ABCD está inscrito em um círculo. As diagonais


AC e BD intersectam-se no ponto Q. Os prolongamentos dos lados DA e CB, a partir
de A e B respectivamente, intersectam-se em P. Sabendo que CD = CP = DQ, mostre
que ∠ CAD = 60º .

EUREKA! N°15, 2002

45
Sociedade Brasileira de Matemática

202.(Inglaterra-2002) Doze pessoas estão sentadas em torno de uma mesa circular. De


quantos modos seis pares de pessoas podem trocar apertos de mão de modo que não
haja cruzamentos de braços?
(Não é permitido que uma pessoa troque apertos de mão com mais de uma pessoa de cada vez)

203.(Inglaterra-2002) Seja f uma função de 1 em 1, onde 1 é o conjunto dos números


inteiros não negativos, que possui as seguintes propriedades :
(i) f (n + 1) > f (n ), para cada n ∈ 1 .
(ii) f (n + f (m )) = f (n ) + m + 1 , para todos m, n ∈ 1 .
Determine todos os valores possíveis de f(2001).

204.(Inglaterra-2002) A altura traçada de um dos vértices de um triângulo ABC


intersecta o lado oposto no ponto D. A partir do ponto D são traçadas
perpendiculares aos outros dois lados intersectando-os nos pontos E e F. Mostre que
a medida do segmento EF independe do vértice do qual a altura é traçada.

205.(Áustria-2002) Determine o menor inteiro positivo x para o qual todas as frações :


3 x + 9 3x + 10 3x + 11 3 x + 49
, , , ... ,
8 9 10 48
são irredutíveis.

206.(Áustria-2002) Determine o maior número real C tal que


((x + y ) 2
)(
− 6 (x − y ) + 8
2
)≥ C
(x − y ) 2

é verdadeira para todos os números reais x e y (x ≠ y ) com xy = 2. Para quais


pares ordenados (x, y) a igualdade é verdadeira ?

9x
207.(Áustria-2002) Seja f (x ) =
9x + 3
 k 
Determine a soma de todas as expressões da forma f   onde k é um número
 2002 
k
inteiro entre 0 e 2002 tal que a fração seja irredutível.
2002
208. (Bielorússia-2002) Determine o maior número possível de grupos que podem ser
extraídos do conjunto {1, 2, 3,…,19, 20}, de modo que o produto dos números de
cada um dos grupos seja um quadrado perfeito .
(O grupo pode conter apenas um número e, neste caso, o produto é igual a este número; além disso, cada número
deve fazer parte de apenas um grupo)

EUREKA! N°15, 2002

46
Sociedade Brasileira de Matemática

209. (Bielorússia-2002) A altura CH de um triângulo retângulo ABC, com ∠C = 90º


intersecta as bissetrizes AM e BN nos pontos P e Q respectivamente. Prove que a
reta que passa pelos pontos médios de QN e PM é paralela à hipotenusa de ABC.

210. (Bielorússia-2002) Um conjunto de números naturais de três algarismos formados


com 1, 2, 3, 4, 5, 6 é chamado agradável se ele satisfaz à seguinte condição: para
quaisquer dois algarismos distintos de 1, 2, 3, 4, 5, 6, existe um número deste
conjunto que contém ambos. Se, para qualquer conjunto agradável calcularmos a
soma de seus elementos, determine o menor valor possível destas somas.
ÞÞÞ
Agora vamos aos comentários e soluções dos leitores para alguns dos
problemas apresentados em números anteriores de EUREKA!. Como sempre, o
critério por nós adotado para este número foi apresentar as soluções dos problemas
que foram, até o presente momento, resolvidos pelo maior número de leitores.

32. (Moldávia-1998) A seqüência (a n ) , n ∈ 1* verifica as relações a1 =


1
e
2
a n −1
an = para todo número natural n > 1 . Calcule a1 + a 2 + ⋅ ⋅ ⋅ + a1998 .
2na n −1 + 1

Resumo das soluções de diversos leitores:


Escrevamos a lei de recorrência de uma forma ligeiramente diferente:
1 1 1 1
= 2n + ⇔ − = 2n
an a n −1 a n a n −1
Logo,
n
 1 1  n

 −  = (2k )

k =2  a k a k −1  k =2

O primeiro membro desta última igualdade é uma soma telescópica e igual a


1 1
− enquanto que o segundo membro é igual a
a n a1

4 + 6 + 8 + ⋅ ⋅ ⋅ + 2n =
(4 + 2n )(n − 1) . Daí,
2

EUREKA! N°15, 2002

47
Sociedade Brasileira de Matemática

− 2 = (n + 2)(n − 1) ⇔ = n(n + 1) ⇔ an =
1 1 1 1 1
= −
an an n(n + 1) n n + 1
Finalmente,
1998 1998
1 1 
∑ ∑
1 1998
ai =  −  =1− =
i =1 n =1  n n + 1 1999 1999

36.(China-1999) Seja PQRS um quadrilátero inscrito num círculo e cuja medida do


ângulo ∠PSR seja igual a 90 o . Se H e K são os pés das perpendiculares baixadas
de Q sobre PR e PS respectivamente (convenientemente prolongados se necessário).
Mostre que HK divide QS ao meio.

Resumo das soluções de diversos leitores:


Seja G o ponto de interseção de KH e SR. Como P, Q, R e S são cíclicos (pertencem
a um mesmo círculo), assim como K, Q, H e P tem-se que
∠ QKG = ∠ QKH = ∠ QPR = ∠ QSG . Daí K, Q, G e S são cíclicos com
∠ KSG = Œ = ∠ SKQ . Desta forma, KQGS é um retângulo e KH divide QS ao
2
meio.

39. (Irlanda-1999) Determine todos os inteiros positivos m tais que a quarta potência do
número de seus divisores positivos é igual a m.

Resumo das soluções de diversos leitores:


Observemos em primeiro lugar que m = 1 possui a referida propriedade.
Suponhamos agora que m ≥ 2. Como m é uma quarta potência ele pode ser escrito
univocamente como
m = p14 1 × p 24 2 × ⋅ ⋅ ⋅ × p n4 n
onde p1 < p 2 < ⋅ ⋅ ⋅ < p n são números primos e i > 0, i = 1, 2, ... , n . O número de
divisores de m é dado por
(4  1 + 1)(4  2 + 1) ⋅ ⋅ ⋅ (4  n + 1)
A requerida propriedade é
(4 1 + 1)(4  2 + 1)⋅ ⋅ ⋅ (4  n + 1) = p11 × p22 × ⋅ ⋅ ⋅ × p n n
Como o lado esquerdo é ímpar então p1 ≥ 3 . Observe que 3 > 4  + 1 se β ≥ 3 ;
também, 5 > 4  + 1 se β ≥ 2 ; finalmente p > 4  + 1 se β ≥ 1 e p ≥ 7 . Além disso,
5 ≥ 4  + 1 se β ≥ 1 , assim, se p β < 4 β + 1 para algum primo ímpar p e algum
β ≥ 1 então p = 3 . Deste modo, devemos ter p1 = 3 ou p1 = 5 .

EUREKA! N°15, 2002

48
Sociedade Brasileira de Matemática

1º Caso : p1 = 5
Neste caso, β 1 = 1 e assim
5(4  2 + 1)⋅ ⋅ ⋅ (4  n + 1) = 5 × p 2 2 × ⋅ ⋅ ⋅ × p n n
Pela terceira desigualdade acima não podem existir outros fatores primos em m
distintos de 5 e portanto a única solução é m = 5 4 .
2º Caso : p1 = 3 . Claramente β 1 = 1 ou β 1 = 2 uma vez que 33 > 4(3) + 1 .
Se β 1 = 1 , então 5(4  2 + 1) ⋅ ⋅ ⋅ (4  n + 1) = 3 × p 2 2 × ⋅ ⋅ ⋅ × p nn . Assim p 2 = 5 . Se

β 2 > 2 então 52 > (4  2 + 1) o que nos leva a uma contradição. Deste modo,
5
3
β 2 = 1 ou β 2 = 2 .
Se β 2 = 1 então 25(4  3 + 1)⋅ ⋅ ⋅ (4  n + 1) = 15 × p33 × ⋅ ⋅ ⋅ × p n n o que é uma contradição
já que 25 não divide o lado direito.
Se β 2 = 2 , então 45(4  3 + 1)⋅ ⋅ ⋅ (4  n + 1) = 75 × p33 × ⋅ ⋅ ⋅ × p n n o que é uma
contradição já que 9 não divide o lado direito.
Se β1 = 2 então 9(4  2 + 1)⋅ ⋅ ⋅ (4  n + 1) = 9 × p 2 2 × ⋅ ⋅ ⋅ × pnn .
Não podemos ter p 2 ≥ 7 logo, n = 1 ou ( n = 2 e p 2 = 5 e β 2 = 1 ). O primeiro caso
nos dá m = 38 ; o último caso nos dá m = 38 × 5 4 . Concluindo existem quatro
soluções a saber,
1, 5 4 , 38 e 38 × 5 4

40. (Irlanda-1999) Mostre que existe um número inteiro positivo na seqüência de


Fibonacci que é divisível por 1000 .

Resumo das soluções de diversos leitores:


Considere o conjunto de 106 + 1 pares
{(F , F
i i +1 )| i = 1,2, ... ,10 6 }
( )
tomados mod 10 3 . Como o conjunto {(a, b ) | 0 ≤ a, b ≤ 999} com a e b inteiros
possui apenas 10 6 elementos, o Princípio da Casa dos Pombos nos diz que existem
inteiros i > j tais que Fi + 1 − F j + 1 e Fi − F j são ambos divisíveis por 1000. Mas
(
Fi −1 − F j −1 = Fi +1 − F j +1 + F j − Fi) ( )
EUREKA! N°15, 2002

49
Sociedade Brasileira de Matemática

Logo, Fi −1 − F j −1 é também divisível por 1000. Argumentando desta maneira de


frente para trás vemos que Fi − j + 1 − F1 e Fi − j − F0 são divisíveis por 1000. Mas,
F0 = 0 daí, Fi − j é divisível por 1000.

60. (St.Petersburg-1999) Três mágicos apresentam um truque entregando a uma pessoa


da platéia um maço de cartas numeradas com 1, 2,...,2 n + 1 (n > 6). O espectador
fica com uma das cartas e aleatoriamente distribui as restantes entre o primeiro e o
segundo mágicos (cada um deles fica com n cartas) . Estes olham suas cartas (sem se
comunicar um ao outro) e cada um escolhe duas cartas formando um maço
(ordenado) com estas cartas e as entrega ao terceiro mágico. O terceiro mágico olha
estas quatro cartas e anuncia a carta que ficou com o espectador. Explique como este
truque pode funcionar.

SOLUÇÃO DE JEAN-CHRISTOPHE YOCCOZ (PARIS - FRANÇA):


A ideia é que cada mágico transmita de algum jeito a soma de suas n cartas módulo
2n + 1 (e então o outro mágico descobre a carta que foi retirada, que é o simétrico da
soma dos dois resultados módulo 2n + 1).
Assim, basta definir uma função f : = = → =
( 2n + 1)= (2n + 1)= (2n + 1)=
× tal

= e um subconjunto arbitrário A de =
(2n + 1)= (2n + 1)=
que, dado c em com n
elementos eles satisfazem a propriedade P(A, c): existem elementos distintos x e y de
A com f(x, y) = c.
A idéia é mostrar a existência de uma função f(x, y) como acima que seja
equivariante, isto é, que satisfaça f(x + c, y + c) = f(x, y) + c, para quaisquer x, y e c.
É fácil ver que para uma tal função f vale P(A, c) se e somente se vale P(A – c, 0),
onde A – c = {x – c | x ∈ A}. Assim, basta ver que todo A com n elementos satisfaz
P(A, 0). Agora, em vez de construir f, vamos construir a pré-imagem de 0 pela f (ou
um subconjunto conveniente dela).
Pela equivariância, se (x, y) e (u, v) são pares distintos com f(x, y) = f(u, v) = 0 então
y – x tem que ser diferente de v – u, senão, para c = u – x, temos (u, v) = (x+ c, y + c),
donde f(u, v) = c. Por outro lado, dado um conjunto de pares{(x1, y1),...,(xk, yk)} com
yj – xj distinto de yi – xi para todo i distinto de j, existe uma função equivariante f com
f(xi, yi) = 0 para todo i. Nosso problema agora se reduz a encontrar um conjunto de
pares {(x1, y1),...,(xk, yk)} com os yi – xi todos distintos (e distintos de 0) tal que para
todo A com n elementos exista i tal que xi e yi estejam em A. Fazemos com que esse
conjunto de pares contenha os pares (0, 1), (2, 1), (0, 2), (3, 6), (9, 6), (3, 9), (4, 8),
(12, 8) e (4, 12). Note que as diferenças yi – xi são 1, –1, 2, 3, – 3, 6, 4, – 4 e 8, e que
um A ruim só pode conter um elemento de cada uma das triplas {0, 1, 2}, {3, 6, 9} e
EUREKA! N°15, 2002

50
Sociedade Brasileira de Matemática

{4, 8, 12}. Vamos completar nosso conjunto de pares com n – 4 pares que formarão
=
(2n + 1)=
uma partição de \{0, 1, 2, 3, 6, 9, 4, 8, 12}. Um conjunto ruim só pode
ter um elemento de cada par, tendo pois no máximo n – 4 + 3 = n – 1 elementos,
como queríamos. Para isso considere alguns casos:

I) n par, n ≥ 12: Escrevemos n = 2k. Os pares serão (– k + 6, k + 6),


(– k + 7, k + 5),...,(–1, 13) (diferenças n, n – 2 ,...,14); (k + 7, 3k + 6),
(k + 8, 3k + 5),...,(2k + 4, 2k + 9) (diferenças n –1, n – 3,...,5); e
(2k + 5, 5), (2k + 6, 7), (2k + 7, 11) e (2k + 8, 10)
(diferenças – n, 1 – n, 4 – n e 2 – n).

II) n ímpar, m ≥ 13: Escrevemos n = 2k + 1. Os pares serão (–1, 13),...,


(– k + 6, k + 6) (diferenças 14 ,..., n –1); (k + 7, 3k + 8),...,(2k + 5, 2k + 10)
(diferenças n ,..., 5); e (2k + 6, 5), (2k + 7, 7), (2k + 8, 11) e (2k + 9, 10)
(diferenças – n, 1– n, 4 – n e 2 – n).

Casos particulares anteriores:


n = 7: Pares (5, 10), (13, 11) e (7, 14) (diferenças 5, – 2 e 7).
n = 8: Pares (5, –1), (7, 14), (15, 10) e (13, 11) (diferenças – 6, 7, – 5 e – 2).
n = 9: Pares (5, 14), (16, 7), (10, 17), (18, 11) e (15, 13) (diferenças 9, –9, 7, –7 e
–2).
n = 10: Pares (20, 10),(19, 11),(18, 13),(5, 14),(16, 7) e (17, 15) (diferenças –10, – 8,
– 5, 9, – 9 e – 2).
n = 11: Pares (16, 5), (17, 7), (20, 11), (18, 10), (19, 13), (21, 14) e (15, 22)
(diferenças – 11, – 10 , – 9, – 8, – 6, – 7 e 7).
ÞÞÞ
Publicamos a seguir, a pedidos, uma solução do problema 12, proposto na
Eureka! No. 8.

12. (Irlanda-1999) Três números a < b < c estão em progressão aritmética se c – b = b –


a. Definamos a seqüência (un), n = 0, 1, 2, 3,… da seguinte maneira: u0 = 0, u1 = 1 e
para cada n ≥ 1, un + 1 é o menor inteiro positivo tal que u n +1 > u n e
{u 0 , u1 ,..., u n , u n +1 } não possui três elementos em progressão aritmética. Determine
u100.
EUREKA! N°15, 2002

51
Sociedade Brasileira de Matemática

k
Vamos mostrar por indução que se n = ( ak ak −1 ...a0 ) 2 , i.e, n = ∑a
j =0
j ⋅ 2 j , com

k
a j ∈ {0,1}, ∀j , então u n = (a k a k −1 ...a0 ) 3 = ∑a
j =0
j ⋅ 3 j . Suponha que isso seja válido

para 0 ≤ n ≤ 2 − 1 (isso é verdade para k = 1).


k

Como {u 0 , u1 ,..., u 2 k −1 } não possui três elementos em progressão aritmética e


k −1
u 2 k −1 = ∑ 1⋅ 3 j =
3k − 1
2
{
, então u 0 , u1 ,..., u 2 k −1 ,3k + u 0 ,3 k + u1 ,...,3 k + u 2 k −1 também}
j =0

não possui três elementos em progressão aritmética, pois se dois termos de uma
3k − 1
progressão aritmética estão entre 0 e , então o terceiro termo é menor ou igual
2
a 3 k − 1 < 3k , e se dois termos de uma progressão aritmética estão entre 3k e 3k +
3k − 1 3k − 1 3 k + 1 3 k − 1
então o termo anterior é maior ou igual a 3 k − = > .
2 2 2 2
Vamos ver que, de fato, u 2 k = 3 k , donde u 2 k + j ≥ 3 k + u j para 0 ≤ j ≤ 2 k − 1 , e logo,
pelo fato acima, u 2 k + j = 3k + u j , para 0 ≤ j ≤ 2 k − 1. Para isso, basta ver que se
3k − 1
< u < 3 k então {u 0 , u1 ,..., u 2 k −1 , u} contém uma progressão aritmética de três
2
k −1
termos. Seja u = ∑bj =0
j ⋅ 3 j , com b j ∈ {0,1,2}, ∀j. Então u = 2v 2 − v1 , onde

k −1 k −1
v2 = ∑d
j =0
j ⋅ 3 j , v1 = ∑c
j =0
j ⋅ 3 j , e, se b j = 0 , então (c j , d j ) = (0,0); se b j = 1 então

(c j , d j ) = (1,1) e, se bj = 2 então (c j , d j ) = (0,1).


Em particular, {c j , d j } ⊂ {0,1}, ∀j , donde v1 e v2 pertencem a {u 0 , u1 ,..., u 2 k −1 } , e
u − v 2 = v 2 − v1 , logo v1 < v 2 < u estão em progressão aritmética.
Finalmente, 100 = 26 + 25 + 22, donde u100 = 36 + 35 + 32 = 981.
ÞÞÞ

EUREKA! N°15, 2002

52
Sociedade Brasileira de Matemática

Acusamos o recebimento de soluções de problemas anteriores dos seguintes leitores


de EUREKA!:

Anderson Torres São Paulo - SP Prob. 24, 32, 35, 37, 40, 43, 45, 51, 54, 56, 63, 88, 103, 114, 124,
143, 151, 154, 158, 160, 164, 168, 169, 170, 171, 172.
Carlos A. Gomes Natal - RN Prob. 151, 162, 174.
Erasmo de Souza Dias Goiânia - GO Prob. 151.
Fernando Carvalho Ramos Santa Maria - RS Prob. 151, 160, 163, 174.
Francisco B. de Lima Holanda Fortaleza - CE Prob. 36, 45, 47, 69, 87, 88, 102, 104, 109, 169, 170.
Geraldo Perlino Itapecerica da Serra - SP Prob. 151, 160, 161, 164, 167.
Geraldo Perlino Júnior Cotia - SP Prob. 151, 152, 153, 154, 158, 159, 160, 161, 162, 163, 165, 166,
167, 168, 170, 171, 172, 173, 174, 178, 179,
Gibran Medeiros de Souza Natal - RN Prob. 163, 166.
Helder Oliveira de Castro Mogi das Cruzes - SP Prob. 43, 45, 46, 54, 66, 63, 67, 70, 74, 86, 90, 99, 101, 102, 105,
115, 128, 133, 143.
Luiz Sérgio Carvalho de Mello Rio de Janeiro - RJ Prob. 138, 151, 166.
Marcelo Ribeiro de Souza Rio de Janeiro - RJ Prob. 151, 153, 154, 160, 162.
Marcelo Rufino de Oliveira Belém - PA Prob. 151, 152, 153, 154, 158, 160, 162, 163, 164, 165, 166, 170,
171, 172, 173, 174, 179, 180.
Okakama Matsubaxi São Paulo - Sp Prob. 130.
Renato F. L. Mello J. dos Guararapes - PE Prob. 154, 160, 162, 166, 173, 179.
Tertuliano C. de Souza Neto Salvador - BA Prob. 61, 62, 68, 69, 70, 72, 74, 76, 82, 83, 87, 88, 90.

EUREKA! N°15, 2002

53
Sociedade Brasileira de Matemática


SOLUÇÕES DE PROBLEMAS PROPOSTOS
Publicamos aqui algumas das respostas enviadas por nossos leitores.

66) Prove que, dados um inteiro n ≥ 1 e um conjunto A ⊂ = / n 2 = com n


elementos existe B ⊂ = / n 2= com n elementos tal que
A + B = {x + y | x ∈ A, y ∈ B} ⊂ = / n = tem mais de n /2 elementos.
2 2

SOLUÇÃO DE ZOROASTRO AZAMBUJA NETO (RIO DE JANEIRO - RJ):


Vamos mostrar que, dado um conjunto X ⊂ = / n 2 = , existe t ∈ = / n 2 = tal que
( A + {t}) ∩ X tem pelo menos X / n elementos, onde X é o número de elementos
de X. De fato, ∑ (A + {t}) ∩ X = ∑ ∑ {t ∈ = / n = a + t = x} = A ⋅ X
2
= n⋅ X ,
= =
t∈ / n 2 a∈A x∈ X

pois, para cada a ∈ A e x ∈ X , existe um único t ∈ = / n 2 = tal que a + t = x. Assim,


1
( A + {t}) ∩ X = X / n , donde o número médio de elementos de
n 2 t∈= / n 2 =
(A + {t}) ∩ X é X / n , o que claramente implica a nossa afirmação.
Agora, dado k ≥0 existem t1 , t 2 ,..., t k ∈ = / n 2 = tais que
1
n 2 − A + {t1 , t 2 ,..., t k } ≤ n 2 ⋅ (1 − ) k . De fato, por indução, dados tais t1 , t 2 ,..., t k ,
n
pela afirmação acima existe t k +1 ∈ = / n 2 = com

( ) ( )
(A + {t k +1}) ∩ (= / n 2 =) \ ( A + {t1 , t 2 ,..., t k }) ≥ 1 ⋅ = / n 2 = \ ( A + {t1 , t 2 ,...,t k }) ,
n
donde ( A + {t1 , t 2 ,..., t k +1 }) = (( A + {t1 , t 2 ,..., t k }) ∪ (A + {t k +1 }) ) C ≤
C

k k +1
 1  1 2  1 2 1
1 − . ( A + {t1 , t 2 ,..., t k }) ≤ 1 −  ⋅ n ⋅ 1 −  = n 1 − 
C
(aqui XC
 n   n   n   n
denota (= / n =)\ X ; temos
2
X C = n2 – X ).
n
 1
Assim, existe B = {t1 , t 2 ,..., t n } tal que n 2 − A + B ≤ n 2 ⋅ 1 −  < n 2 / 2 , donde
 n
A + B > n 2 / 2.

EUREKA! N°15, 2002

54
Sociedade Brasileira de Matemática

−1
 1 n   n 
n
 1
n
 n 1
Obs. Para n ≥ 1,  1 −   =  > 1 +  ≥ 1 +   ⋅ = 2, donde
 n    n −1  n 1  n
 
n
 n  1
  < .
 n −1 2

70)
P1

X1
X5

P5 P2
Q5 Q1

Q4 Q2
X4 X2
Q3

X3
P4 P3

Na figura acima, para 1 ≤ j ≤ 5, X j é o ponto de interseção dos círculos


circunscritos aos triângulos Qj –1 PjQj e Qj Pj + 1 Qj + 1 distintos de Qj (os índices são
tomados módulo 5). Prove que o pentágono X1X2X3 X4X5 é inscritível.

Obs: O pentágono P1P2P3P4P5 não é necessariamente regular.

EUREKA! N°15, 2002

55
Sociedade Brasileira de Matemática

SOLUÇÃO DE ALEX CORRÊA ABREU (NITERÓI – RJ):

B
Y1
.
D
Y2 .
Y
F
.
Y3
A
.
E Y4 C

Lema: As circunferencias circunscritas a BDF, CEF, CDA, BEA, tem um ponto em


comum.

Prova: Seja X a intersecção das circunferencias circunscritas a BDF, CEF.


Y1, Y2, Y3, Y4, o pé das perpendiculares por Y a BD, BE , CD, CE respectivamente ⇒
Y1 , Y2 , Y3 são colineares pela reta de Simson já que Y pertence a circunferência
circunscrita a BDF e analogamente Y2 , Y3 , Y4 também são logo Y1 , Y2 , Y3 , Y4 são
colineares. Portanto pela recíproca da reta de Simson, Y pertence à circunferência
circunscrita a ABE, ( Y1 , Y2 , Y4 colineares) e à circunferência circunscrita a ADC
(Y1 , Y3 , Y4 colineares).
P1

X1
X5

P5 P2
Q5 Q1

Q4 Q2
X4 X2
Q3

X3
P4 P3

EUREKA! N°15, 2002

56
Sociedade Brasileira de Matemática

Agora X 2 Xˆ 1Q2 = Q2 Pˆ2 X 2 pois X 1 X 2 P2Q2 é inscritível e Q2 Xˆ 1 X 3 = Q2 Pˆ4 X 3 pois


X 1 ∈ circunferência circunscrita a P1Q2 P4 pelo lema e X 3 também.
Analogamente Q Xˆ X = Q Pˆ X e Q Xˆ X = X Pˆ Q ⇒ X Xˆ Q = Q Xˆ X e
3 4 3 3 4 3 3 4 2 2 2 2 2 1 2 3 4 2

Q2 Xˆ 1 X 3 = Q3 Xˆ 4 X 3 ⇒ X 2 Xˆ 1 X 3 = X 2 Xˆ 4 X 3 ⇒ X1 X 2 X 3 X 4 é inscritível; analogamente
X 2 X 3 X 4 X 5 também é, logo X 1 X 2 X 3 X 4 X 5 é inscritível.

72) Ache todas as funções f : 5 → 5 tais que f ( x 4 + y ) = x 3 f ( x ) + f ( f ( y )),


∀x, y ∈ 5.

SOLUÇÃO DE RODRIGO VILLARD MILET (RIO DE JANEIRO – RJ):


f ( x 4 + y ) = x 3 f ( x) + f ( f ( y )) (I)
Em (I) , x = 0 ⇒ f ( y ) = f ( f ( y )), logo f ( x 4 + y ) = x 3 f ( x) + f ( y ) (I'). Agora
fazendo x = 1 e y = 0, segue que f(0) = 0. Portanto, fazendo y = 0, temos
f ( x 4 ) = x 3 f ( x ) (II) (Em particular, f é ímpar). Em (I'), f ( x 4 + y ) = f ( x 4 ) + f ( y ) ,
donde f ( x + y ) = f ( x ) + f ( y ) , para todo x > 0. Mas como f é ímpar, é fácil ver que
vale f ( x + y ) = f ( x ) + f ( y ) , para todos x, y reais.

Lema: Todo x real pode ser escrito (de forma única) como x = u + v, onde f(u) = u e
f(v) = 0.

Prova: x = f ( x) + ( x − f ( x)) . Tome u = f (x ) e v = x − f (x) . Para a unicidade,


suponha que x = u + v = u '+v ' . Daí u − u ' = v '−v e aplicando f, segue que u = u' e
assim, v = v'.
Agora, em (II), escreva x = u + v, como no lema. Então:
f ((u + v) 4 ) = (u + v) 3 f (u + v) ⇒ 4 f (u 3 v) + 6 f (u 2 v 2 ) + 4 f (uv3 ) = 3u 2 v(u + v) (III)
Podemos trocar v por –v, pois f(–v) = 0, logo:
− 4 f (u 3 v) + 6 f (u 2 v 2 ) − 4 f (uv 3 ) = −3u 2 v(u − v) (IV)
u 2v2
Fazendo (III) + (IV): f (u 2 v 2 ) = . Pelo lema, u 2 v 2 = u '+v ' , com f(u') = u' e f(v')
2
u '+ v'
= 0. Daí segue que u ' = f (u '+v ' ) = ⇒ u ' = v ' . Aplicando f, segue que u' = v' = 0,
2
ou seja u ≡ 0 ou v ≡ 0 , o que nos dá respectivamente as soluções f ( x) ≡ 0 e
f ( x ) ≡ x , que claramente satisfazem a equação (I).

EUREKA! N°15, 2002

57
Sociedade Brasileira de Matemática

Agradecemos também o envio das soluções e a colaboração de:

Rildo Alves do Nascimento Santa Maria da Boa Vista – PE


Leno Silva Rocha Goiânia – GO
Minh Perez Rehoboth, EUA
Carlos Alberto da Silva Victor Nilópolis – RJ

Seguimos aguardando o envio de soluções dos problemas propostos Nº. 68, 69, 71 e 73
publicado na revista Eureka! Nº. 14

9RFr VDELD« TXH SDUD FRPSOHWDU XP


iOEXP FRPSRVWR SRU n ILJXULQKDV VXSRQGR
TXH YRFr FRPSUH  ILJXULQKD SRU GLD R
WHPSR HVSHUDGR PpGLR p n ⋅ ln n GLDV"
&RQVLGHUDQGR XPD VLWXDomR QD TXDO YRFr
WHP XP iOEXP FRP  ILJXULQKDV H
FRPSUH XP SDFRWH GH  ILJXULQKDV SRU GLD
YRFr GHYH GHPRUDU XP DQR SDUD FRPSOHWDU
R iOEXP

9RFr VDELD« TXH IRL GHVFREHUWR SRU


0DQLQGUD $JUDZDO 1LWLQ 6D[HQD H 1HHUDM
.D\DO XP DOJRULWPR GHWHUPLQtVWLFR TXH
FKHFD D SULPDOLGDGH GH Q~PHURV QDWXUDLV n
HP WHPSR SROLQRPLDO HP ORJ n"
3DUD PDLV LQIRUPDo}HV YHMD http://www.cse.iitk.ac.in/news/primality.html 4:
http://www.utm.edu/research/primes/prove/prove4_3.html

EUREKA! N°15, 2002

58
Sociedade Brasileira de Matemática

PROBLEMAS PROPOSTOS
Convidamos o leitor a enviar soluções dos problemas propostos e sugestões de novos
problemas para os próximos números.

74) Ache todas as funções f : 5 → 5 tais que: f ( x + y ) + f ( x − y ) = 2 f ( x) ⋅ cos y,


∀x, y ∈ 5.

75) Seja Tn um triângulo retângulo cujos lados medem (4 ⋅ n 2 ,4 ⋅ n 4 − 1,4 ⋅ n 4 + 1) ,


onde n é um número inteiro positivo. Seja α n a medida do ângulo oposto ao
lado de medida 4 ⋅ n 2 . Mostre que, se n varia dentro dos inteiros positivos,
α 1 + α 2 + α 3 + ... = 90 0 .

76) Mostre que um polígono qualquer pode ser recortado e os recortes


reorganizados, sem superposição, de tal jeito que formem um quadrado.

77) Prove que as distâncias entre um ponto sobre uma circunferência e os quatro
vértices de um quadrado nesta inscrita não podem ser todos números racionais.

78) Seja ABCD um quadrilátero convexo não trapézio, de diagonais AC e BD iguais.


Tomamos sobre os lados AB e CD, respectivamente, pontos P e Q tais que:
AP DQ AD
= =
PB QC BC
Mostre que os pontos P e Q são colinares com o ponto de interseção das
mediatrizes dos lados AD e BC.

79) Temos uma fileira infinita de copos, cada um deles associado a um inteiro k, e
um número finito de pedras distribuídas de alguma maneira por esses copos. Se
há pelo menos duas pedras no copo k podemos pular uma pedra para o copo
k – 1 e outra para o copo k + 1.
Prove que fazendo movimentos desse tipo um número suficientemente grande de
vezes, chega-se necessariamente a uma situação onde não é possível fazer
nenhum movimento desse tipo (i.e., onde há no máximo uma pedra em cada
copo), e que a configuração final não depende da escolha dos movimentos
durante o processo.

EUREKA! N°15, 2002

59
Sociedade Brasileira de Matemática

80) Sejam α =
1+ 5
2
{ }
, A = {nα , n ∈ 1 *} e B = nα 2 , n ∈ 1 * . Prove que
A ∩ B = ∅ e A∪ B = 1*.

Obs. x  é o inteiro tal que x  ≤ x < x  + 1.

Problema 74 proposto por Gibran M. de Souza (Natal - RN); Problema 75 proposto por Carlos A.
Gomes (Natal - RN); Problema 76 proposto por Eduardo Casagrande Stabel (Porto Alegre - RS);
Problemas 77 e 78 propostos por Evandro Makiyama de Melo (São Paulo - SP).

EUREKA! N°15, 2002

60
Sociedade Brasileira de Matemática

COORDENADORES REGIONAIS

Alberto Hassen Raad (UFJF) Juiz de Fora – MG


Amarísio da Silva Araújo (UFV) Viçosa – MG
Benedito Tadeu Vasconcelos Freire (UFRN) Natal – RN
Carlos Frederico Borges Palmeira (PUC-Rio) Rio de Janeiro – RJ
Claudio Arconcher (Colégio Leonardo da Vinci) Jundiaí – SP
Claus Haetinger (UNIVATES) Lajeado – RS
Cleonor Crescêncio das Neves (UTAM) Manaus – AM
Élio Mega (Colégio Etapa) São Paulo – SP
Florêncio Ferreira Guimarães Filho (UFES) Vitória – ES
Ronaldo Alves Garcia (UFGO) Goiânia – GO
Alexandra de Oliveira Abdala Cousin (UEM) Maringá – PR
Ivanilde Fernandes Saad (UC. Dom Bosco) Campo Grande– MS
Jacqueline Fabiola Rojas Arancibia (UFPB) João Pessoa – PB
João Benício de Melo Neto (UFPI) Teresina – PI
João Francisco Melo Libonati (Grupo Educacional Ideal) Belém – PA
José Carlos Pinto Leivas (UFRG) Rio Grande – RS
José Cloves Saraiva (UFMA) São Luis – MA
José Gaspar Ruas Filho (ICMC-USP) São Carlos – SP
José Luiz Rosas Pinho (UFSC) Florianópolis – SC
José Vieira Alves (UFPB) Campina Grande – PB
Licio Hernandes Bezerra (UFSC) Florianópolis – SC
Luzinalva Miranda de Amorim (UFBA) Salvador – BA
Marcelo Rufino de Oliveira (Grupo Educacional Ideal) Belém – PA
Marcondes Cavalcante França (UFC) Fortaleza – CE
Pablo Rodrigo Ganassim (Liceu Terras do Engenho) Piracicaba – SP
Ramón Mendoza (UFPE) Recife – PE
Reinaldo Gen Ichiro Arakaki (INPE) SJ dos Campos – SP
Ricardo Amorim (Centro Educacional Logos) Nova Iguaçu – RJ
Roberto Vizeu Barros (Colégio Acae) Volta Redonda – RJ
Rosângela Souza (Colégio Singular) Santo André – SP
Sérgio Cláudio Ramos (IM-UFRGS) Porto Alegre – RS
Tadeu Ferreira Gomes (UEBA) Juazeiro – BA
Tomás Menéndez Rodrigues (U. Federal de Rondônia) Porto Velho – RO
Valdenberg Araújo da Silva (U. Federal de Sergipe) São Cristovão – SE
Wagner Pereira Lopes (CEFET – GO) Jataí – GO

EUREKA! N°15, 2002

61
CONTEÚDO

XIV OLIMPÍADA BRASILEIRA DE MATEMÁTICA 2


Problemas e Soluções da Primeira Fase

XIV OLIMPÍADA BRASILEIRA DE MATEMÁTICA 14


Problemas e Soluções da Segunda Fase

XIV OLIMPÍADA BRASILEIRA DE MATEMÁTICA 25


Problemas e Soluções da Terceira Fase

XIV OLIMPÍADA BRASILEIRA DE MATEMÁTICA 44


Problemas e Soluções da Primeira Fase - Nível Universitário

XIV OLIMPÍADA BRASILEIRA DE MATEMÁTICA 49


Problemas e Soluções da Segunda Fase - Nível Universitário

XIV OLIMPÍADA BRASILEIRA DE MATEMÁTICA 58


Premiados

AGENDA OLÍMPICA 62
Sociedade Brasileira de Matemática

XXIV OLIMPÍADA BRASILEIRA DE MATEMÁTICA


Problemas e Soluções da Primeira Fase
PROBLEMAS – NÍVEL 1
4 8
1. A razão (2 ) é igual a:
(48 ) 2
A) 1 B) 1 C) 1 D) 2 E) 8
4 2

2. Num armazém foram empilhadas embalagens cúbicas conforme mostra a figura


a seguir. Se cada caixa pesa 25 kg, quanto pesa toda a pilha?

A) 300 kg B) 325 kg C) 350 kg D) 375 kg E) 400 kg

3. Na balança a seguir temos pesadas bolas de chumbo, todas iguais, e leves


saquinhos de plástico, todos com a mesma quantidade de bolinhas, iguais às que
estão fora dos mesmos. Quantas bolinhas há em cada saquinho?

a
a a
a
a
a a

A) 1 B) 2 C) 3 D) 5 E) 6

4. Escreva os números inteiros de 1 a 9 nos nove quadradinhos, de forma que as


somas dos quatro números em cada uma das pás da “hélice” sejam iguais e de
maior valor possível. Esse valor é:

A) 23 B) 22 C) 21 D) 20 E) 19

EUREKA! N°16, 2003

2
Sociedade Brasileira de Matemática

5. Qual é a quantidade total de letras de todas as respostas incorretas desta questão?


A) Quarenta e oito. B) Quarenta e nove. C) Cinqüenta.
D) Cinqüenta e um. E) Cinqüenta e quatro.

6. Toda a produção mensal de latas de refrigerante de uma certa fábrica foi vendida
a três lojas. Para a loja A, foi vendida metade da produção; para a loja B, foram
vendidos 2 da produção e para a loja C, foram vendidas 2500 unidades. Qual
5
foi a produção mensal dessa fábrica?
A) 4166 latas B) 10000 latas C) 20000 latas D) 25000 latas
E) 30000 latas

7. Um quadrado de área 1 foi dividido em 4 retângulos congruentes, conforme


indicado no desenho à esquerda. Em seguida, os quatro retângulos foram
reagrupados de maneira a formar um quadrado, com um buraco quadrado no
centro, conforme indica o desenho à direita.

A área do buraco é igual a:


A) 1 B) 9 C) 16 D) 3 E) 1
2 16 25 4

8. A linha poligonal AB é desenhada mantendo-se sempre o mesmo padrão


mostrado na figura. Seu comprimento total é igual a:

2
1
A B
1 2 3 4 5 6 7 8 9 30 31

A) 31 B) 88 C) 90 D) 97 E) 105

9. A diferença entre os quadrados de dois números inteiros positivos consecutivos é


sempre:
A) um número primo.
B) um múltiplo de 3.
C) igual à soma desses números.
D) um número par.
E) um quadrado perfeito.

EUREKA! N°16, 2003

3
Sociedade Brasileira de Matemática

10. Marcelo leva exatamente 20 minutos para ir de sua casa até a escola. Uma certa
vez, durante o caminho, percebeu que esquecera em casa a revista Eureka! que ia
mostrar para a classe; ele sabia que se continuasse a andar, chegaria à escola 8
minutos antes do sinal, mas se voltasse para pegar a revista, no mesmo passo,
chegaria atrasado 10 minutos. Que fração do caminho já tinha percorrido neste
ponto?
A) 2 B) 9 C) 1 D) 2 E) 9
5 20 2 3 10

11. O gráfico abaixo mostra o faturamento mensal das empresas A e B no segundo


semestre de 2001.

2 00
m ilhões de rea is

1 80
1 60

1 40
1 20 
1 00
ago

nov
jul

set
out

dez
Com base nesse gráfico, podemos afirmar que:
A) houve um mês em que o faturamento da empresa A foi o dobro do
faturamento da empresa B.
B) no mês de julho, a diferença de faturamentos foi maior que nos demais
meses.
C) a empresa B foi a que sofreu a maior queda de faturamento entre dois meses
consecutivos.
D) no semestre, o faturamento total de A foi maior que o de B.
E) a diferença entre os faturamentos totais do semestre excedeu os 20 milhões
de reais.

12. Patrícia mora em São Paulo e quer visitar o Rio de Janeiro num feriado
prolongado. A viagem de ida e volta, de ônibus, custa 80 reais, mas Patrícia está
querendo ir com seu carro, que faz, em média, 12 quilômetros com um litro de
gasolina. O litro da gasolina custa, em média, R$1,60 e Patrícia calcula que terá
de rodar cerca de 900 quilômetros com seu carro e pagar 48 reais de pedágio. Ela
irá de carro e para reduzir suas despesas, chama duas amigas, que irão repartir
com ela todos os gastos. Dessa forma, não levando em conta o desgaste do carro
e outras despesas inesperadas, Patrícia irá:
A) economizar R$20,00.
B) gastar apenas R$2,00 a mais.

EUREKA! N°16, 2003

4
Sociedade Brasileira de Matemática

C) economizar R$24,00.
D) gastar o mesmo que se fosse de ônibus.
E) gastar R$14,00 a mais.

13. Uma escola vai organizar um passeio ao zoológico. Há duas opções de


transporte. A primeira opção é alugar "vans": cada van pode levar até 6 crianças
e seu aluguel custa R$60,00. A segunda opção é contratar uma empresa para
fazer o serviço: a empresa usa ônibus com capacidade para 48 crianças e cobra
R$237,00, mais R$120,00 por ônibus utilizado. A escola deve preferir a empresa
de ônibus se forem ao passeio pelo menos N crianças. O valor de N é:
A) 28 B) 31 C) 32 D) 33 E) 36

14. O produto de um milhão de números naturais, não necessariamente distintos, é


igual a um milhão. Qual é o maior valor possível para a soma desses números?

A) 1 000 000 B) 1 250 002 C) 1 501 999 D) 1 999 999


E) 13 999 432

15. Se você tiver uma mesa de bilhar retangular cuja razão entre a largura e o
comprimento seja 5/7 e bater em uma bola que está em um canto, de modo que
ela saia na direção da bissetriz do ângulo desse canto, quantas vezes ela baterá
nos lados antes de bater em um dos cantos?
A) 10 vezes B) 12 vezes C) 13 vezes D) 14 vezes E) 15 vezes

16. Na malha quadriculada a seguir, todas as circunferências têm centro em M.


Então pode-se concluir que a área preta é:

A) dois quintos da área do círculo maior.


B) três sétimos da área do círculo maior.
C) metade da área do círculo maior.
D) quatro sétimos da área do círculo maior.
E) três quintos da área do círculo maior.

EUREKA! N°16, 2003

5
Sociedade Brasileira de Matemática

17. As figuras a seguir são construídas com palitos pretos e brancos. Para construir
as figuras, os palitos pretos foram colocados apenas nas bordas e os brancos
apenas no interior. A figura de número n corresponde a um retângulo 3 por n.
Continuando esse procedimento, quantos palitos brancos teremos na figura
2002?
1 2 3

A) 2001 B) 4004 C) 12006 D) 10007 E) 10010

18. Um produtor de leite engarrafa diariamente toda a produção de leite de sua


fazenda. Depois de tirado, o leite segue para um tanque de forma cilíndrica e
então é engarrafado, conforme vemos na figura a seguir. Na tabela vemos a
quantidade de garrafas que foram enchidas e o nível do leite dentro do tanque.
Depois de quantas garrafas serem enchidas o tanque ficará vazio?

Quantidade de garrafas
0 200 400 600
enchidas
Nível do tanque (cm) 210 170 130 90

A) 1000 B) 1050 C) 1100 D) 1150 E) 1200

19. Escrevendo todos os números inteiros de 100 a 999, quantas vezes escrevemos o
algarismo 5?
A) 250 B) 270 C) 271 D) 280 E) 292

20. Uma usina comprou 2000 litros de leite puro e então retirou certo volume V
desse leite para produção de iogurte e substituiu esse volume por água. Em
seguida, retirou novamente o mesmo volume V da mistura e novamente
substituiu por água. Na mistura final existem 1125 litros de leite. O volume V é:
A) 500 litros B) 600 litros C) 700 litros D) 800 litros E) 900 litros

EUREKA! N°16, 2003

6
Sociedade Brasileira de Matemática

PROBLEMAS – NÍVEL 2

1. Um comerciante comprou dois carros por um total de R$ 27.000,00. Vendeu o


primeiro com lucro de 10% e o segundo com prejuízo de 5%. No total ganhou
R$ 750,00. Os preços de compra foram, respectivamente,
A) R$ 10.000,00 e R$ 17.000,00
B) R$ 13.000,00 e R$ 14.000,00
C) R$ 14.000,00 e R$ 13.000,00
D) R$ 15.000,00 e R$ 12.000,00
E) R$ 18.000,00 e R$ 9.000,00
2. Veja o problema No. 15 do Nível 1.

3. Dizer que uma tela de televisão tem 20 polegadas significa que a diagonal da tela
mede 20 polegadas. Quantas telas de televisão de 20 polegadas cabem numa de
60 polegadas?
A) 9 B) 10 C) 18 D) 20 E) 30
4. Veja o problema No. 20 do Nível 1.

5. Dois irmãos, Pedro e João, decidiram brincar de pega-pega. Como Pedro é mais
velho, enquanto João dá 6 passos, Pedro dá apenas 5. No entanto, 2 passos de
Pedro equivalem à distância que João percorre com 3 passos. Para começar a
brincadeira, João dá 60 passos antes de Pedro começar a persegui-lo. Depois de
quantos passos Pedro alcança João?
A) 90 passos B) 120 passos C) 150 passos D) 180 passos
E) 200 passos
6. Veja o problema No. 9 do Nível 1.
7. Veja o problema No. 10 do Nível 1.
8. Veja o problema No. 4 do Nível 1.
9. Veja o problema No. 12 do Nível 1.

10. Traçando segmentos, podemos dividir um quadrado em dois quadradinhos


congruentes, quatro trapézios congruentes e dois triângulos congruentes,
conforme indica o desenho abaixo, à esquerda. Eliminando algumas dessas
partes, podemos montar o octógono representado à direita. Que fração da área do
quadrado foi eliminada?

EUREKA! N°16, 2003

7
Sociedade Brasileira de Matemática

A) 1 B) 2 C) 1 D) 1 E) 3
9 9 4 3 8

11. Veja o problema No. 11 do Nível 1.


12. Veja o problema No. 14 do Nível 1.

13. O lava-rápido "Lave Bem" faz uma promoção:


Lavagem simples R$5,00
Lavagem completa R$7,00

No dia da promoção, o faturamento do lava-rápido foi de R$176,00. Nesse dia,


qual o menor número possível de clientes que foram atendidos?
A) 23 B) 24 C) 26 D) 28 E) 30

14. Veja o problema No. 7 do Nível 1.

15. Quantos números inteiros positivos menores que 900 são múltiplos de 7 e
terminam em 7?
A) 10 B) 11 C) 12 D) 13 E) 14

16. Dado um triângulo ABC onde  = 80° e Ĉ = 40° , a medida do ângulo agudo
formado pelas bissetrizes dos ângulos  e B̂ é:
A) 40° B) 60° C) 70o D) 80° E) 110o

17. Na malha quadrada abaixo, há 6 quadrados de lado 30 cm. A área do triângulo


ABC é:

C
A
B

A) 150 cm2 B) 100 cm2 C) 75 cm2 D) 50 cm2 E) 25 cm2


o
18. Veja o problema N . 8 do Nível 1.
19. Veja o problema No. 19 do Nível 1.

EUREKA! N°16, 2003

8
Sociedade Brasileira de Matemática

2 2
20. Se xy = 2 e x2 + y2 = 5, então x 2 + y 2 + 2 vale:
y x
A) 5 B) 25 C) 5 D) 1 E) 1
2 4 4 2

21. Veja o problema No. 13 do Nível 1.

22. Durante sua viagem ao país das Maravilhas a altura de Alice sofreu quatro
mudanças sucessivas da seguinte forma: primeiro ela tomou um gole de um
líquido que estava numa garrafa em cujo rótulo se lia: "beba-me e fique 25%
mais alta". A seguir, comeu um pedaço de uma torta onde estava escrito: "prove-
me e fique 10% mais baixa"; logo após tomou um gole do líquido de outra
garrafa cujo rótulo estampava a mensagem: "beba-me e fique 10% mais alta".
Finalmente, comeu um pedaço de outra torta na qual estava escrito:"prove-me e
fique 20% mais baixa". Após a viagem de Alice, podemos afirmar que ela:
A) ficou 1% mais baixa
B) ficou 1% mais alta
C) ficou 5% mais baixa
D) ficou 5% mais alta
E) ficou 10% mais alta

23. Vamos provar que 4 é maior que 4.


Sejam a e b dois números tais que a > 4 e a = b.
1) Vamos subtrair 4 dos dois termos desta equação:
a=b
a–4=b–4
2) Colocamos –1 em evidência no segundo membro da equação:
a – 4 = –1 (– b + 4)
a – 4 = –1 (4 – b)
3) Elevamos ambos os termos da equação ao quadrado:
(a − 4) 2 = [ −1 ⋅ ( 4 − b)] 2
(a − 4) 2 = (−1) 2 (4 − b) 2
(a − 4) 2 = 1 ⋅ (4 − b) 2 (a − 4) 2 = ( 4 − b) 2
4) Extraímos a raiz quadrada dos dois membros da equação:
( a − 4) 2 = ( 4 − b) 2
a–4=4–b
5) Como a = b, substituímos b por a
a–4=4–a

EUREKA! N°16, 2003

9
Sociedade Brasileira de Matemática

6) Resolvemos a equação:
a–4=4–a
2a = 8
a=4
Como escolhemos a tal que a > 4, chegamos à inacreditável conclusão de que 4 > 4.
Onde está o erro no argumento acima?
A) Na passagem 2. B) Na passagem 3. C) Na passagem 4.
D) Na passagem 5. E) Na passagem 6.

24. Veja o problema No. 5 do Nível 1.

25. O resto da divisão por 9 de 1111111111− 22222 é:


A) 0 B) 1 C) 3 D) 6 E) 8

PROBLEMAS – NÍVEL 3

1. Veja o problema No. 11 do Nível 1.


p 6,888...
2. Se é a fração irredutível equivalente a o valor de p + q é igual a:
q 2,444...
A) 38 B) 39 C) 40 D) 41 E) 42

3. Veja o problema No. 1 do Nível 2.

4. A seguir vemos quatro vasos, os quais Angela vai encher com água, numa
torneira cuja vazão é constante.

1 2 3 4
Os gráficos A e B a seguir representam o nível da água (eixo vertical), em dois
dos vasos, de acordo com o tempo (eixo horizontal).

A B

Qual dos vasos corresponde ao gráfico A e qual ao gráfico B, respectivamente?


A) 3 e 4 B) 2 e 4 C) 1 e 3 D) 2 e 3 E) 1 e 4

EUREKA! N°16, 2003

10
Sociedade Brasileira de Matemática

5. Veja o problema No. 13 do Nível 1.


6. Veja o problema No. 22 do Nível 2.
7. Veja o problema No. 10 do Nível 1.
8. Veja o problema No. 8 do Nível 1.
9. Veja o problema No. 10 do Nível 2.
10. Veja o problema No. 20 do Nível 2.

11. A média aritmética das idades de um grupo de médicos e advogados é 40 anos.


A média aritmética das idades dos médicos é 35 anos e a dos advogados é 50
anos. Pode-se, então, afirmar que:
A) O número de advogados é o dobro do número de médicos no grupo.
B) O número de médicos é o dobro do número de advogados no grupo.
C) Há um médico a mais no grupo.
D) Há um advogado a mais no grupo.
E) Existem as mesmas quantidades de médicos e advogados no grupo.

1 1
12. Os valores de x, y e z que satisfazem às equações x + = 5 , y + =1 e
y z
1
z+ = 2 são tais que x + 3 y + 2 z é igual a:
x
A) 5 B) 6 C) 7 D) 8 E) 9

13. Veja o problema No. 23 do Nível 2.


14. Veja o problema No. 5 do Nível 1.

15. Sejam x, y, z números inteiros tais que x + y + z = 0. Sobre x 3 + y 3 + z 3 são


feitas as seguintes afirmativas:
i) É necessariamente múltiplo de 2.
ii) É necessariamente múltiplo de 3.
iii) É necessariamente múltiplo de 5.
Podemos afirmar que:

A) somente i) é correta.
B) somente ii) é correta.
C) somente i) e ii) são corretas.
D) somente i) e iii) são corretas.
E) i), ii) e iii) são corretas.

EUREKA! N°16, 2003

11
Sociedade Brasileira de Matemática

16. Seja f uma função real de variável real que satisfaz a condição:
 2002 
f ( x) + 2 f   = 3x
 x 
para x > 0. O valor de f(2) é igual a:
A) 1000 B) 2000 C) 3000 D) 4000
E) 6000

17. Veja o problema No. 25 do Nível 2.

18. Na circunferência abaixo, temos que: AB = 4, BC = 2, AC é diâmetro e os


ângulos ABˆ D e CBˆ D são iguais. Qual é o valor de BD?
A

9
A) 2 3 + 1 B) C) 3 2 D) 2 + 5 E) 4
5

19. Seja α a maior raiz de x2 + x – 1 = 0. O valor de α5 – 5α é :


A) – 1 B) – 2 C) – 3 D) 1 E) 2
... 7
77
20. Qual é o dígito das unidades de 7 7 , onde aparecem 2002 setes?
A) 7 B) 9 C) 3 D) 1 E) 5.

21. Em um trapézio ABCD de área 1, a base BC mede a metade da base AD. Seja K
o ponto médio da diagonal AC. A reta DK corta o lado AB no ponto L. A área do
quadrilátero BCKL é igual a:
3 2 1 2 1
A) B) C) D) E)
4 3 3 9 9

22. N = ❑539984❑ é um número inteiro positivo com oito algarismos, sendo o


primeiro e o último desconhecidos. Sabendo que N é um múltiplo de 198,
encontre o algarismo das unidades de N / 198.
A) 5 B) 6 C) 7 D) 8 E) 9

EUREKA! N°16, 2003

12
Sociedade Brasileira de Matemática

23. No triminó marciano, as peças têm 3 números cada (diferente do dominó da


terra, onde cada peça tem apenas 2 números). Os números no triminó marciano
também variam de 0 a 6, e para cada escolha de 3 números (não necessariamente
distintos) existe uma e somente uma peça que contém esses 3 números. Qual é a
soma dos números de todas as peças do triminó marciano?
A) 756 B) 1512 C) 84 D) 315 E) 900

24. No triângulo ABC, o ângulo  mede 60° e o ângulo B mede 50°. Sejam M o
ponto médio do lado AB e P o ponto sobre o lado BC tal que AC + CP = BP.
Qual a medida do ângulo MPC?
A) 120° B) 125° C) 130° D) 135° E) 145°

25. Duas pessoas vão disputar uma partida de par ou ímpar. Elas não gostam do
zero e, assim, cada uma coloca 1, 2, 3, 4 ou 5 dedos com igual probabilidade.
A probabilidade de que a pessoa que escolheu par ganhe é:
A) 1/2 B) 2/5 C) 3/5 D) 12/25 E) 13/25

GABARITO
NÍVEL 1 (5a. e 6a. Séries)
1) C 6) D 11) D 16) C
2) C 7) B 12) C 17) D
3) B 8) D 13) B 18) B
4) B 9) C 14) D 19) D
5) D 10) B 15) A 20) A

NÍVEL 2 (7a. e 8a. Séries)


1) C 6) C 11) D 16) C 21) B
2) A 7) B 12) D 17) C 22) A
3) A 8) B 13) C 18) D 23) C
4) A 9) C 14) B 19) D 24) D
5) E 10) B 15) D 20) B 25) D

NÍVEL 3 (Ensino Médio)


1) D 6) A 11) B 16) B 21) D
2) E 7) B 12) B 17) D 22) C
3) C 8) D 13) C 18) C 23) A
4) C 9) B 14) D 19) C 24) E
5) B 10) B 15) C 20) C 25) E

EUREKA! N°16, 2003

13
Sociedade Brasileira de Matemática

XXIV OLIMPÍADA BRASILEIRA DE MATEMÁTICA


Problemas e Soluções da Segunda Fase
PROBLEMAS – NÍVEL 1

PROBLEMA 1
O ano 2002 é palíndromo, ou seja, continua o mesmo se lido da direita para a
esquerda.

a) Depois de 2002, quais serão os próximos quatro anos palíndromos?


b) O último ano palíndromo, 1991, era ímpar. Quando será o próximo ano
palíndromo ímpar?

PROBLEMA 2
Um fazendeiro resolveu repartir sua fazenda A
para seus cinco filhos. O desenho ao lado (fora
E
de escala) representa a fazenda e as partes dos
herdeiros, que são da forma triangular, de
BC AC DC G
modo que BD = , AE = , DF = e
4 3 2
EG = GC. O filho mais novo recebeu o terreno C
B F
representado pelo triângulo escuro, de 40 D
alqueires. Quantos alqueires tinha a
propriedade original?

PROBLEMA 3
Dado um número, pode-se escrever o seu dobro ou suprimir o seu algarismo das
unidades. Apresente uma seqüência que começa com 2002 e termina com 13,
usando somente essas duas operações.

PROBLEMA 4
Três amigas foram para uma festa com vestidos azul,
preto e branco, respectivamente. Seus pares de sapato
apresentavam essas mesmas três cores, mas somente Ana
usava vestido e sapatos de mesma cor. Nem o vestido
nem os sapatos de Júlia eram brancos. Marisa usava
sapatos azuis. Descreva a cor do vestido de cada uma das
moças.

EUREKA! N°16, 2003

14
Sociedade Brasileira de Matemática

PROBLEMA 5
No jogo pega-varetas, as varetas verdes valem 5 pontos cada uma, as azuis valem 10
pontos, as amarelas valem 15, as vermelhas, 20 e a preta, 50. Existem 5 varetas
verdes, 5 azuis, 10 amarelas, 10 vermelhas e 1 preta. Carlinhos conseguiu fazer 40
pontos numa jogada. Levando em conta apenas a quantidade de varetas e suas cores,
de quantas maneiras diferentes ele poderia ter conseguido essa pontuação, supondo
que em cada caso fosse possível pegar as varetas necessárias?

PROBLEMA 6
Nas casas de um tabuleiro 8 × 8 foram escritos números inteiros positivos de forma
que a diferença entre números escritos em casas vizinhas (quadrados com um lado
comum) é 1. Sabe-se que numa das casas está escrito 17 e, em outra, está escrito 3.
Desenhe um tabuleiro 8 × 8, preencha-o segundo essas regras e calcule a soma dos
números escritos nas duas diagonais do tabuleiro.

PROBLEMAS – NÍVEL 2

PROBLEMA 1
Geraldinho e Magrão saíram de suas casas no
mesmo instante com a intenção de um visitar o
outro, caminhando pelo mesmo percurso.
Geraldinho ia pensando num problema de
olimpíada e Magrão ia refletindo sobre
questões filosóficas e nem perceberam quando
se cruzaram. Dez minutos depois, Geraldinho
chegava à casa de Magrão e meia hora mais
tarde, Magrão chegava à casa de Geraldinho.
Quanto tempo cada um deles andou?

Observação: Cada um deles anda com


velocidade constante.

PROBLEMA 2
Um grande painel na forma de um quarto de círculo foi
YHUGH composto com 4 cores, conforme indicado na figura ao
D] X O lado, onde o segmento divide o setor em duas partes
4
3.
,
iguais e o arco interno é uma semicircunferência. Qual é
- 7 DPDUHOR a cor que cobre a maior área?

EUREKA! N°16, 2003

15
Sociedade Brasileira de Matemática

PROBLEMA 3
Nas casas de um tabuleiro 8 × 8 foram escritos números inteiros positivos de forma
que a diferença entre números escritos em casas vizinhas (quadrados com um lado
comum) é 1. Sabe-se que numa das casas está escrito 17 e, em outra, está escrito 3.
Calcule a soma dos números escritos nas duas diagonais do tabuleiro.

PROBLEMA 4
O professor Pardal está estudando o comportamento
C familiar de uma espécie de pássaro. Os pontos A, B, C e
B D da figura ao lado, representam a disposição de quatro
ninhos desses pássaros. O professor construiu um posto
D de observação equidistante dos quatro ninhos.
A
Todos os ninhos e o posto de observação estão em um
mesmo nível de altura a partir do solo, a distância de B
a D é de 16 metros e BAˆ D = 45° . Determine a distância
que o posto guarda de cada ninho.

PROBLEMA 5
O primeiro número de uma seqüência é 7. O próximo é obtido da seguinte maneira:
Calculamos o quadrado do número anterior 72 = 49 e a seguir efetuamos a soma de
seus algarismos e adicionamos 1, isto é, o segundo número é 4 + 9 + 1 = 14.
Repetimos este processo, obtendo 142 = 196 e o terceiro número da seqüência é 1 + 9
+ 6 + 1 = 17 e assim sucessivamente. Qual o 2002o elemento desta seqüência?

PROBLEMA 6
O ano 2002 é palíndromo, ou seja, continua o mesmo se lido da direita para a
esquerda.
a) Depois de 2002, quais serão os próximos quatro anos palíndromos?
b) O último ano palíndromo, 1991, era ímpar. Quando será o próximo ano
palíndromo ímpar?
c) O último ano palíndromo primo aconteceu há mais de 1000 anos, em 929.
Determine qual será o próximo ano palíndromo primo.

PROBLEMAS – NÍVEL 3

PROBLEMA 1
Veja o problema No. 5 do Nível 2.

PROBLEMA 2
Para quais inteiros positivos n existe um polígono não regular de n lados, inscrito em
uma circunferência, e com todos os ângulos internos de mesma medida?

EUREKA! N°16, 2003

16
Sociedade Brasileira de Matemática

PROBLEMA 3
Determine o maior natural k para o qual existe um inteiro n tal que 3k divide
n3 – 3n2 + 22.

PROBLEMA 4
Quantos dados devem ser lançados ao mesmo tempo para maximizar a probabilidade
de se obter exatamente um 2?

PROBLEMA 5
Em um quadrilátero convexo ABCD, os lados opostos AD e BC são congruentes e os
pontos médios das diagonais AC e BD são distintos.
Prove que a reta determinada pelos pontos médios das diagonais forma ângulos
iguais com AD e BC.

PROBLEMA 6
Colocamos vários palitos sobre uma mesa de modo a formar um retângulo m × n,
como mostra a figura.
Devemos pintar cada palito de azul, vermelho ou preto de modo que cada um dos
quadradinhos da figura seja delimitado por exatamente dois palitos de uma cor e dois
de outra cor. De quantas formas podemos realizar esta pintura?

...

...
m

   
...

EUREKA! N°16, 2003

17
Sociedade Brasileira de Matemática

SOLUÇÕES – NÍVEL 1

SOLUÇÃO DO PROBLEMA 1
a) Os palíndromos entre 2000 e 3000 são da forma 2aa2, onde a é um algarismo.
Logo os próximos quatro serão 2112, 2222, 2332 e 2442.

b) Como o primeiro algarismo é igual ao último, um palíndromo ímpar maior que


2002 deve começar e terminar por um número ímpar maior ou igual a 3. Logo o
próximo será 3003.

SOLUÇÃO DO PROBLEMA 2
Seja S a área do triângulo ABC.
BC
Se BD = , então ( ABD) = S .
4 4
S 3S
AC S−
Se AE = , então ( AED ) = ( ADC ) = 4 = 4 =S.
3 3 3 3 4
S S
S − + 
DC 4 4 S.
Se DF = , então ( DEF ) = ( DEC ) = =
2 2 2 4
 
3S
S − 
Se EG = EC, então (GFC ) = ( EFC )
=  4  = S.
2 2 8
S
Como (GFC) = 40 temos = 40 ⇔ S = 320 alqueires.
8

SOLUÇÃO DO PROBLEMA 3
Uma possível solução é:
2002, 200, 20, 2, 4, 8, 16, 32, 64, 128, 256, 512, 51, 102, 204, 408, 816, 1632, 163,
326, 652, 1304, 130, 13.

SOLUÇÃO DO PROBLEMA 4
Como os sapatos de Marisa eram azuis, e nem o vestido nem os sapatos de Júlia
eram brancos, conclui-se que os sapatos de Júlia eram pretos e portanto os sapatos de
Ana eram brancos.
O vestido de Ana era branco, pois era a única que usava vestido e sapatos da mesma
cor; conseqüentemente, o vestido de Júlia era azul e o de Marisa era preto.

SOLUÇÃO DO PROBLEMA 5
A soma dos pontos é 40. Segundo as regras do jogo, as possibilidades são:

EUREKA! N°16, 2003

18
Sociedade Brasileira de Matemática

20 20 + 20 (1)

20 15 – 5 20 + 15 + 5 (2)
10
20 + 10 + 10 (3)
10
5–5 20 + 10 + 5 + 5 (4)
5–5–5–5 20 + 5 + 5 + 5 + 5 (5)

15 10 15 + 15 + 10 (6)
5–5 15 + 15 + 5 + 5 (7)

15 10 10 5 15 + 10 + 10 + 5 (8)
5 5–5 15 + 10 + 5 + 5 + 5 (9)

5–5–5–5–5 15 + 5 + 5 + 5 + 5 + 5 (10)

10 10 + 10 + 10 + 10 (11)
10
10 5–5 10 + 10 + 10 + 5 + 5 (12)
5–5–5–5 10 + 10 + 5 + 5 + 5 + 5 (13)
10 – 5 – 5 – 5 – 5 – 5 – 5 não dá, pois há apenas 5 varetas verdes.

A resposta é portanto: de 13 maneiras diferentes.

SOLUÇÃO DO PROBLEMA 6
Como a diferença entre o 17 e o 3 é 14, esses números devem estar em posições
afastadas de 14 casas, contadas na horizontal ou vertical.
Portanto 17 e 3 devem ocupar as extremidades de uma das diagonais do tabuleiro.
A partir disso, o preenchimento das diagonais é feito de maneira única. E uma
maneira de se preencher o tabuleiro é a seguinte:

17 16 15 14 13 12 11 10
16 15 14 13 12 11 10 9
15 14 13 12 11 10 9 8
14 13 12 11 10 9 8 7
13 12 11 10 9 8 7 6
12 11 10 9 8 7 6 5
11 10 9 8 7 6 5 4
10 9 8 7 6 5 4 3

a soma dos números escritos nas diagonais é: 8 × 10 + (3 + 5 +...+ 17) = 160.

EUREKA! N°16, 2003

19
Sociedade Brasileira de Matemática

SOLUÇÕES – NÍVEL 2

SOLUÇÃO DO PROBLEMA 1
Seja t > 0 o tempo, em minutos, decorrido desde a saída de Geraldinho e Magrão até
o instante do encontro.
Sejam g e m as distâncias entre o ponto de encontro e as casas de Geraldinho e
Magrão, respectivamente. Como Geraldino percorre a distância g em t minutos e a
distância m em 10 minutos, temos g = t .
m 10
g 40 t 40
Analogamente, = . Logo = ⇔ t 2 = 400 ⇔ t = 20 (pois t > 0). Logo
m t 10 t
Geraldinho andou 10 + 20 = 30 minutos e Magrão andou 40 + 20 = 60 minutos.

SOLUÇÃO DO PROBLEMA 2 Sejam x, y, z e w as áreas das regiões branca, amarela,


azul e verde, respectivamente.
Seja R o raio do semicírculo. Temos x + y = πR
2

w 2
z 1 πR 2
x e y + z = x + w = π (2 R ) 2 =
8 2
y Assim, x + y = y + z = x + w, logo x = z e y = w.
Como se x é a área de um segmento circular de ângulo
πR 2 R 2  π − 2  2
90° e raio R, x= − = R e
4 2  4 
 π + 2  2 Assim x = z < y = w.
y = R .
 4 
SOLUÇÃO DO PROBLEMA 3
Como a diferença entre o 17 e o 3 é 14, esses números devem estar em posições
afastadas de 14 casas, contadas na horizontal ou vertical.
Portanto 17 e 3 devem ocupar as extremidades de uma das diagonais do tabuleiro.
A partir disso, o preenchimento das diagonais é feito de maneira única. E uma
maneira de se preencher o tabuleiro é a seguinte:
17 16 15 14 13 12 11 10
16 15 14 13 12 11 10 9
15 14 13 12 11 10 9 8
14 13 12 11 10 9 8 7
13 12 11 10 9 8 7 6
12 11 10 9 8 7 6 5
11 10 9 8 7 6 5 4
10 9 8 7 6 5 4 3

EUREKA! N°16, 2003

20
Sociedade Brasileira de Matemática

a soma dos números escritos nas diagonais é: 8 × 10 + (3 + 5 +...+ 17) = 160.

SOLUÇÃO DO PROBLEMA 4
Observe que o posto do observador coincide com o centro do círculo circunscrito ao
quadrilátero ABCD. Como BD = 16 , sendo O o centro do círculo circunscrito, temos
ˆ = 90° e BO = OD = r , donde 16 2 = r 2 + r 2 , pelo teorema de
ˆ = 2 ⋅ BAD
BOD
Pitágoras, e logo r = 128 = 8 2 . Assim, a distância do posto (que deve ficar em O)
aos ninhos será de 8 2 metros.

SOLUÇÃO DO PROBLEMA 5
Os primeiros números da seqüência são (7, 14, 17, 20, 5, 8, 11, 5...) donde vemos
que, exceto pelos 4 primeiros termos, a seqüência é periódica com período 3. Como
2002 deixa resto 1 quando dividido por 3, o número procurado coincide com aquele
que ocupa o 7o. lugar na seqüência, a saber, 11.

Observação:
Para qualquer termo inicial, a seqüência construída de acordo com método descrito
no enunciado do problema será eventualmente periódica, (isto é teremos an + k = ak
para todo k ≥ m, para certos valores positivos de m e n).

SOLUÇÃO DO PROBLEMA 6
a) Os palíndromos entre 2000 e 3000 são da forma 2aa2, onde a é um algarismo.
Logo os próximos quatro serão 2112, 2222, 2332 e 2442.

b) Como o primeiro algarismo é igual ao último, um palíndromo ímpar maior que


2002 deve começar e terminar por um número ímpar maior ou igual a 3. Logo o
próximo será 3003.

c) Um palíndromo de quatro algarismos é da forma abba = a + 10b + 100b + 1000a


= 1001a + 110b, que é múltiplo de 11, já que 110 e 1001 são múltiplos de 11. Logo
o próximo ano palíndromo primo tem no mínimo 5 algarismos.
Os menores palíndromos de 5 algarismos são 10001, que é múltiplo de 73 e 10101,
que é múltiplo de 3. O próximo é 10201 = 1012, divisível por 101. O seguinte,
10301, é primo, pois não é divisível por qualquer primo menor que 10301 < 102.

EUREKA! N°16, 2003

21
Sociedade Brasileira de Matemática

SOLUÇÕES – NÍVEL 3

SOLUÇÃO DO PROBLEMA 1
Veja a solução do problema No. 5 do Nível 2.

SOLUÇÃO DO PROBLEMA 2
Seja C a circunferência de centro O circunscrita ao polígono A1A2...An. Os triângulos
AiAi + 1 O (com An + 1 = A1) são isósceles. Seja α i = OAˆ i Ai +1 .
Então
(1) α 1 + α 2 = α 2 + α 3 = α 3 + α 4 = ... = α n + α 1 .
Portanto.
α 1 = α 3 = α 5 = ..., A2


α 2 = α 4 = α 6 = ... α1 α2
A1 A3
α1 α2
α = α α3
 n 2 αn

αn α3
O

Se n for ímpar, então α 1 = α 2 = ... = α n , logo todos os ângulos Ai Oˆ Ai +1 serão iguais


e o polígono será regular.
Para n par, não é necessário que todos os ângulos sejam iguais.
180(n − 2)
Escolhendo x ≠ y de modo que x + y = ângulo interno = e fazendo
n
x = α 1 = α 3 = ... = α n −1 ,
y = α 2 = α 4 = ... = α n , obtemos um polígono inscritível não regular com todos os
ângulos de mesma medida.
Portanto, para n par ≥ 4, existe um polígono de n lados satisfazendo as condições do
problema.

SOLUÇÃO DO PROBLEMA 3
Se n = 3r, então n 3 − 3n 2 + 22 = (3r ) 3 − 3 ⋅ (3r ) 2 + 22 é a soma de um múltiplo de 3
com 22, logo não é múltiplo de 3.
Se n = 3r + 1, então

EUREKA! N°16, 2003

22
Sociedade Brasileira de Matemática

n3 − 3n2 + 22= (3r +1)3 − 3(3r +1)2 + 22= (3r)3 + 3⋅ (3r)2 + 3⋅ (3r) +1− 3⋅ (3r)2 −
3 ⋅ 2 ⋅ (3r ) − 3 + 22 = (3r )3 − 3 ⋅ (3r ) + 20 , que também não é múltiplo de 3.
Finalmente, se n = 3r – 1, então n 3 − 3n 2 + 22 = (3r − 1) 3 − 3(3r − 1) 2 + 22 =
= (3r) 3 − 3 ⋅ (3r) 2 + 3 ⋅ (3r) − 1 − 3 ⋅ (3r) 2 + 3 ⋅ 2 ⋅ (3r) − 3 + 22 = (3r) 3 − 6 ⋅ (3r) 2 + 9 ⋅ 3r + 18,
que é a soma de um múltiplo de 27 com 18, e portanto é múltiplo de 9 mas não de
27, logo a maior potência de 3 que divide um número da forma n 3 − 3n 2 + 22 é 32 =
9. Assim, k é no máximo 2.

SOLUÇÃO DO PROBLEMA 4
Suponha que os dados estão numerados de 1 a n. A probabilidade de que somente o
dado No. 1 resulte em 2 é:
1 5 5 5 5 n−1
× × ... = .
6 6 6 6 6n
Analogamente, a probabilidade de que somente o dado k, (1 ≤ k ≤ n) resulte em 2 é
5 5 5 1 5 5 5 n−1
× ... × × × ... × = n .
6 6 6 6 6 6 6
Portanto, a probabilidade de obter exatamente um 2 é
5 n −1 5 n −1 5 n −1 5 n −1
Pn = n + n + ... + n = n ⋅ n .
6 6 6 6
n −1 n
5 5
Agora observe que Pn ≥ Pn +1 ⇔ n ⋅ n ≥ (n + 1) ⋅ n +1 ⇔ 6n ≥ 5( n + 1) ⇔ n ≥ 5.
6 6
Para n = 5, ocorre a igualdade (P5 = P6), P5 = P6 > P7 > P8 > P9 >... e
P1 < P2 < P3 < P4 < P5 = P6
E a probabilidade é máxima para n = 5 ou n = 6.

SOLUÇÃO DO PROBLEMA 5
Sejam M e N os pontos médios de AC e BD e P o ponto médio do lado AB. Então
PM é base média do ∆ABC e PN base média do ∆ABD. Segue que
BC AD
PM = = = PN .
2 2
Sendo X e Y as interseções da reta MN com BC e AD, temos então
BXˆM = PMˆ N = PNˆ M = AYˆN ou BXˆM = π − PMˆ N = π − PNˆ M = AYˆN .

EUREKA! N°16, 2003

23
Sociedade Brasileira de Matemática

B
P
A
X

Y M
N

C
D
SOLUÇÃO ALTERNATIVA:
A+C B+D
Provaremos que se, M = e N= então o vetor MN faz ângulos iguais
2 2
com AD e BC . Para isso, como AD = BC , basta ver que os produtos internos

MN ⋅ AD e MN ⋅ BC têm o mesmo módulo.


Temos
2
 A+C − B−D (C − B ) ⋅ ( D − A ) − D − A
MN ⋅ AD = ( N − M ) ⋅ ( D − A ) =   ⋅ (D − A ) = =
 2  2
2
(C − B ) ⋅ ( D − A ) − C − B ( D + B − A − C ) ⋅ (C − B )
= = = ( M − N ) ⋅ ( C − B ) = − MN ⋅ BC
2 2

SOLUÇÃO DO PROBLEMA 6
Há 3n maneiras de colorir a fileira horizontal superior de palitos. O palito vertical
mais à esquerda da primeira linha também pode ser colorido de 3 maneiras.
n
...
. . .  . . .  . . . . . .  . . .

Uma vez definidas as cores dos palitos superior e mais à esquerda de um


quadradinho, há duas maneiras de completá-lo segundo as condições do enunciado:
se ambos têm mesma cor, há duas escolhas para a cor dos dois palitos restantes; se
ambos têm cores diferentes, há duas maneiras de colorir os dois palitos restantes com
estas cores.
Assim, para completar a primeira linha de quadrados há 3n ⋅ 3 ⋅ 2n maneiras
Da mesma forma, a cor do palito vertical mais à esquerda da segunda linha de
quadrados pode ser escolhido de 3 maneiras, e há 2n maneiras de colorir os demais
palitos desta linha. Assim, para m = 2, há 3n ⋅ 3 ⋅ 2n⋅ 3 ⋅ 2n colorações possíveis.
Analogamente, no caso geral, há 3 n ⋅ (3 ⋅ 2 n ) m = 3 n + m ⋅ 2 nm maneiras de realizar a
pintura pedida.

EUREKA! N°16, 2003

24
Sociedade Brasileira de Matemática

XXIV OLIMPÍADA BRASILEIRA DE MATEMÁTICA


Problemas e Soluções da Terceira Fase
PROBLEMAS – NÍVEL 1

PROBLEMA 1
No quadriculado ao lado estão escritos todos os
2 13 16 11 23
inteiros de 1 a 25. Considere todos os conjuntos
formados por cinco desses números, de modo que, 15 1 9 7 10
para cada conjunto, não existem dois números que
estão na mesma linha ou na mesma coluna. 14 12 21 24 8
a) Apresente um conjunto cujo maior elemento é o 3 25 22 18 4
23.
b) Apresente um conjunto cujo maior elemento é o 20 19 6 5 17
menor possível.

PROBLEMA 2
No desenho ao lado, a reta t é perpendicular ao
segmento AB e passa pelo seu ponto médio M. Q
Dizemos que A é o simétrico de B em relação à reta t A
(ou em relação ao segmento PQ ).
Seja XYZ um triângulo retângulo de área 1m2. P
Considere o triângulo X'Y'Z' tal que X' é o simétrico
de X em relação ao lado YZ , Y' é o simétrico de Y M
em relação ao lado XZ e Z' é o simétrico de Z em B
relação ao lado XY .
t
Calcule a área do triângulo X'Y'Z'.

PROBLEMA 3
Um parque tem a forma de um quadrilátero e possui
oito portões de entrada: um em cada vértice do
quadrilátero e um no meio de cada lado. Os portões
foram numerados de 1 a 8, de forma que a soma T dos
números em cada lado é a mesma para os quatro lados.
Apresente um exemplo de numeração dos pontos para
cada um dos possíveis valores de T.

EUREKA! N°16, 2003

25
Sociedade Brasileira de Matemática

PROBLEMA 4
Sete moedas estão dispostas em círculo, com a coroa
visível.

a) Mostre que é possível, virando-se cinco moedas


consecutivas de cada vez, fazer com que todas
fiquem com a cara visível.
b) Mostre que não é possível, virando-se quatro moedas
consecutivas de cada vez, fazer com que todas
fiquem com a cara visível.

PROBLEMA 5
São dados um tabuleiro de xadrez (8 × 8) e palitinhos do tamanho dos lados das
casas. Dois jogadores jogam alternadamente e, em cada jogada, um dos jogadores
coloca um palitinho sobre um lado de uma casa do tabuleiro, sendo proibido
sobrepor palitinhos.
Vence o jogador que conseguir completar primeiro um quadrado 1 × 1 de palitinhos.
Supondo que nenhum jogador cometa erros, qual dos dois jogadores tem a estratégia
vencedora, ou seja, consegue vencer independentemente de como jogue seu
adversário?

PROBLEMAS – NÍVEL 2

PROBLEMA 1
Veja o problema No. 2 do Nível 1.

PROBLEMA 2
Mostre que, entre dezoito inteiros consecutivos de três algarismos, sempre existe
algum que é divisível pela soma de seus algarismos.

PROBLEMA 3
São dados um tabuleiro quadriculado m × n e palitinhos do tamanho dos lados das
casas. Dois jogadores jogam alternadamente e, em cada jogada, um dos jogadores
coloca um palitinho sobre um lado de uma casa do tabuleiro, sendo proibido
sobrepor palitinhos.
Vence o jogador que conseguir completar primeiro um quadrado 1 × 1 de palitinhos.
Supondo que nenhum jogador cometa erros, qual dos dois jogadores tem a estratégia
vencedora, ou seja, consegue vencer independentemente de como jogue seu
adversário?

EUREKA! N°16, 2003

26
Sociedade Brasileira de Matemática

PROBLEMA 4
Uma mistura possui os componentes A e B na razão 3 : 5, uma segunda mistura
possui os componentes B e C na razão 1 : 2 e uma terceira mistura possui os
componentes A e C na razão 2 : 3. Em que razão devemos combinar a 1a, 2a e 3a
misturas para que os componentes A, B e C apareçam na razão 3 : 5 : 2?

PROBLEMA 5
Seja ABC um triângulo inscrito em uma circunferência de centro O e P um ponto
sobre o arco AB que não contém C. A perpendicular traçada por P à reta BO
intersecta AB em S e BC em T. A perpendicular traçada por P a AO intersecta AB em
Q e AC em R.
Prove as duas afirmações a seguir:

a) PQS é um triângulo isósceles


b) PQ 2 = QR ⋅ ST

PROBLEMA 6
 1   1   1 
Seja n um inteiro positivo. Definimos ϕ ( n) = 1 −  ⋅ 1 −  ⋅ ... ⋅ 1 −
  ⋅ n ,
 p1   p2   p k 
onde p1 , p 2 ,..., p k são os fatores primos distintos de n. Prove que para todo m ≥ 1,
existe n tal que ϕ ( n) = m! .

Obs: m! = 1 ⋅ 2 ⋅ ... ⋅ m .

PROBLEMAS – NÍVEL 3

PROBLEMA 1
Mostre que existe um conjunto A formado por inteiros positivos tendo as seguintes
propriedades:
a) A tem 2002 elementos.
b) A soma de qualquer quantidade de elementos distintos de A (pelo menos um)
nunca é uma potência perfeita.

Obs: Uma potência perfeita é um número da forma ab, onde a e b são inteiros
positivos e b ≥ 2.

EUREKA! N°16, 2003

27
Sociedade Brasileira de Matemática

PROBLEMA 2
ABCD é um quadrilátero convexo e inscritível e M é um ponto sobre o lado CD, tal
que o triângulo ADM e o quadrilátero ABCM têm a mesma área e o mesmo
perímetro. Prove que ABCD tem dois lados de comprimentos iguais.

PROBLEMA 3
Numeramos as casas de um tabuleiro quadriculado m × n, onde m, n ≥ 2, com os
inteiros 1, 2, 3,...,mn de modo que, para todo i ≤ mn – 1, as casas i e i + 1 tenham
um lado em comum.
Prove que existe i ≤ mn – 3 tal que as casas i e i + 3 têm um lado em comum.

PROBLEMA 4
Definimos o diâmetro de um subconjunto não vazio de {1, 2,..., n} como a diferença
entre seu maior elemento e seu menor elemento (em módulo).
Calcule a soma dos diâmetros de todos os subconjuntos não vazios de {1, 2,..., n}.

PROBLEMA 5
Temos um número finito de quadrados, de área total 4. Prove que é possível arranjá-
los de modo a cobrir um quadrado de lado 1.
Obs: É permitido sobrepor quadrados e parte deles pode ultrapassar os limites do
quadrado a ser coberto.

PROBLEMA 6
Arnaldo e Beatriz se comunicam durante um acampamento usando sinais de fumaça,
às vezes usando uma nuvem grande, às vezes uma pequena.
No tempo disponível antes do café da manhã, Arnaldo consegue enviar uma
seqüência de 24 nuvens. Como Beatriz nem sempre consegue distinguir uma nuvem
pequena de uma grande, ela e Arnaldo fizeram um dicionário antes de ir para o
acampamento. No dicionário aparecem N seqüências de 24 tamanhos de nuvem
(como por exemplo a seqüência PGPGPGPGPGPGGPGPGPGPGPGP, onde G
significa nuvem grande e P significa nuvem pequena). Para cada uma das N
seqüências, o dicionário indica seu significado. Para evitar interpretações erradas,
Arnaldo e Beatriz evitaram incluir no dicionário seqüências parecidas. Mais
precisamente, duas seqüências no dicionário sempre diferem em pelo menos 8 das 24
posições.
Demonstre que N ≤ 4096 .

EUREKA! N°16, 2003

28
Sociedade Brasileira de Matemática

SOLUÇÕES – NÍVEL 1

PROBLEMA 1: SOLUÇÃO DE MÁRCIO H. MORAES FERNANDES (RIO DE JANEIRO - RJ)


Das informações dadas pelo problema conclui-se a seguinte propriedade:
Propriedade:
Como dois números não podem ficar na mesma coluna ou na mesma linha, sendo
que para formar um conjunto são precisos 5 números e sabendo que o quadriculado
possui 5 linhas e 5 colunas, cada número do conjunto tem que ocupar uma linha e
uma coluna e conseqüentemente, cada linha e cada coluna estarão ocupadas por um
número do conjunto a ser formado.

A) A resolução mais simples para que dois números não se encontrem na mesma
linha ou na mesma coluna são as diagonais. Na diagonal do número 23, apenas o
número 25 é maior que este. Assim peguei todos os números da diagonal menos
o 25 que tive que substituir pelo 3 (que estava numa coluna que ainda não usara)
e assim não pude utilizar o 20 se não repetiria a coluna, dessa forma o último
número foi o 19 que estava em linha e coluna que não utilizei. Conjunto
A = {23, 7, 21, 3, 19}. O conjunto A é a solução para o item a).

B) O menor número que pode ser maior no conjunto de 5 números é o 5. Assim, fui
eliminando os números na seqüência. O número 5 pode ser descartado porque o
4 e o 3 estão na mesma linha e para fazer o conjunto sendo 5 o maior, os dois
teriam que ser utilizados. O número 6 pode ser descartado porque na 4a. coluna
da esquerda para a direita, o único número menor que 6 está na mesma linha que
ele. Se o número 7 for usado, o único número menor que ele na segunda coluna,
estará na mesma linha dele. Com o 8, na quarta coluna poderá ser escolhido o 7
ou 5, escolhendo qualquer um, outros números não poderão ser utilizados: O 1
(na segunda coluna) ou o 6 (na terceira coluna). Com o 9 e o 10, se forem
escolhidos, o número 1 não poderá ser utilizado por estar na mesma linha, não
restando outro número na segunda coluna a ser utilizado. E com o 11 pode se
fazer um conjunto obedecendo a propriedade. Conjunto B = {1, 3, 6, 8, 11}
sendo 11 o menor número possível.

PROBLEMA 2
Veja a solução do problema No. 1 do Nível 2.

PROBLEMA 3: SOLUÇÃO DE LUCIO ASSAOKA HOSSAKA (CURITIBA – PR)


Os possíveis valores de T são 12, 13, 14 e 15 pois 8 < T < 16 = 1 + 8 + 7 (note que
4 + 5 + 6 < 16 é a maior soma possível de números fora de {1, 7, 8}).
9 não é o valor de T pois 8 deve existir e, usando 3 números, é impossível fazer com
que a soma de seu lado seja 9. O mesmo acontece com 10, já que não se pode usar 1

EUREKA! N°16, 2003

29
Sociedade Brasileira de Matemática

e 1. Não pode, igualmente, ser 11 o valor de T pois 7 e 8 devem existir. O único jeito
de 7 chegar a 11 com mais dois números, é 7, 1, 3, pois não se pode repetir números.
O único jeito de somar dois números a 8 com o resultado 11 é 8, 1 e 2. Fazemos
então uma ilustração:
1 8 2

Restam para colocar, os números 4, 5 e 6. É impossível somar 2 desses números


com 3 resultando 11. Os únicos valores para T, são: 12, 13, 14 e 15.

3 7 2 5 2 6

12 ⇒ 8 4 13 ⇒ 7 3

1 5 6 1 8 4

7 6 1 7 2 6

14 ⇒ 3 5 15 ⇒ 5 1

4 2 8 3 4 8

PROBLEMA 4: SOLUÇÃO DE DEBORAH DE SÁ PEREIRA BELFORT (RECIFE – PE)


a) Para conseguir desviar as sete moedas, foi preciso desvirar as cinco primeiras
moedas, e depois desvira-se as próximas cinco, e algumas voltarão a estar
viradas no lado Coroa. Continuo com este ciclo até chegar o resultado:
= coroa = cara

1 2 3 4

EUREKA! N°16, 2003

30
Sociedade Brasileira de Matemática

5 6 7

b) De 4 em 4, que é um número par, não se consegue as sete moedas viradas.


Virando as moedas de 4 em 4, a quantidade de caras vai ser sempre número par; e 7
é ímpar.

PROBLEMA 5
Veja a solução do problema No. 3 do Nível 2.

SOLUÇÕES – NÍVEL 2

PROBLEMA 1: SOLUÇÃO DE ANDRÉ L. RIBEIRO DOS SANTOS (PINDAMONHANGABA - SP)


X
Y'

B
.A 1

Z' α ..
Y α
Z

. A2

X'

+ XYZ ≅+ X ' YZ ( LAL) ⇒ YZˆ ' X ' = YZX


ˆ =α
⇒ XZ = X ' Z '
Logo XZ // X ' Z ' (olhe os ângulos formados pela transversal ZZ ' ).
Marque os pontos B e C no segmento XZ , como mostra a figura.
Seja Y ' A1 a altura do + BY ' C , em relação a Y ' . Prolongue A1 até encontrar o
segmento X ' Z ' , formando 90° em A2.

EUREKA! N°16, 2003

31
Sociedade Brasileira de Matemática

Agora, note que Y ' A2 é a altura do +Z ' Y ' X ' , em relação a Y ' .
Chame a medida de XZ de y ⇒ med( XZ ) = y = med( X ' Z ') .
Chame a medida de YA1 de h ⇒ med(YA1 ) = h
YA1 é a altura do +ZYX , em relação a Y; portanto h = YA1 = YA2 que é a altura
correspondente no +Z ' YX '.
Como Y é simétrico a Y' em relação a XZ , então YA1 = Y ' A1 = h
Assim Y ' A1 = YA1 = YA2 = h,

Área do + XYZ = b ⋅ h = XZ ⋅ YA1 = yh


2 2 2
yh
Logo = 1.
2
Área do + X ' Y ' Z ' = b ⋅ h = X ' Z ' ⋅ Y ' A 2 =
2 2

=
(
X Z ⋅ Y ' A1 + Y A1 + Y A 2 )= y ⋅ (h + h + h)
=
3 yh
2 2 2
De yh = 1 ⇒ 3 yh = 3
2 2
Área do + X ' Y ' Z ' = 3m 2 .

PROBLEMA 2: SOLUÇÃO DE EDUARDO FISCHER (ENCANTADO - RS)


Um número divisível por 18 cumpre a condição. Um número assim possui a soma
dos algarismos igual a 9 ou a 18 (27 só com o 999, que não é par). Qualquer número
divisível por 18 é divisível por 9 e 18. Como em cada 18 números inteiros
consecutivos um é divisível por 18 o problema está resolvido.
Resp. Entre quaisquer 18 inteiros consecutivos, um é divisível por 18. A soma dos
algarismos de um múltiplo de 18 (com 3 algarismos) é 18 ou 9.
Em qualquer caso, o número é divisível pela soma dos algarismos.

PROBLEMA 3: SOLUÇÃO DE ANDRÉ L. RIBEIRO DOS SANTOS (PINDAMONHANGABA - SP)


Para preencher todos os quadrados do tabuleiro, precisamos de um número ímpar de
palitos, se as paridades de m e n forem diferentes; ou de um número par de palitos, se
as paridades forem iguais:

i) m e n são de paridades diferentes: o primeiro jogador coloca o primeiro palito na


posiçào central do tabuleiro e imita espelhadamente *(em relação ao palito) as

EUREKA! N°16, 2003

32
Sociedade Brasileira de Matemática

jogadas do adversário. Haverá uma hora em que todos os quadrados serão ocupados
com 2 palitos e será a vez do segundo jogador. Este por sua vez preenche um dos
quadradinhos com o terceiro palito e o primeiro jogador o completa em seguida,
vencendo o jogo.

ii) m e n são de paridades iguais: o segundo jogador copia as jogadas do primeiro,


espelhadamente*, quando sobram todos os quadrados preenchidos com 2 palitos é a
vez do primeiro jogador, este preenche um quadrado com o terceiro palito, e o
segundo jogador o completa ganhando o jogo.
*espelhadamente: como se estivesse olhando para um espelho, tem a mesma
profundidade mas é invertido lateralmente. Exemplos:
par x par ímpar x ímpar ímpar x par

A A
C C A C central

D
B central

B
D B D

Em todos os casos A está espelhando a B e C está espelhando D.


Se m e n tem a mesma paridade o segundo jogador ganha, se tem paridades
diferentes o primeiro ganha.

PROBLEMA 4: SOLUÇÃO DE THOMÁS YOITI SASAKI HOSHINA (RIO DE JANEIRO – RJ)


Temos na mistura:
3 5
I⇒ A e⇒ B
8 8
1 2
II ⇒ B e ⇒ C
3 3
2 3
III ⇒ A e ⇒ C
5 5
Queremos que na mistura
3 1 1
IV ⇒ A, B e C
10 2 5
Se pegarmos x da I, y da II e z da III teremos:

EUREKA! N°16, 2003

33
Sociedade Brasileira de Matemática

3 2 3
A⇒ x+ z = ∴
8 5 10 15 x + 16 z = 12
1 5 1
B ⇒ y + x = ∴ 15 x + 8 y = 12 y = 2z
3 8 2
10 y + 9 z = 3
2 3 1
C ⇒ y + z = ∴
3 5 5

6 3
y= z=
29 29
6 3 20
Teriamos que y = e z= , logo x =
29 29 29
x : y : z = 20 : 6 : 3.

PROBLEMA 5: SOLUÇÃO DE THOMÁS YOITI SASAKI HOSHINA (RIO DE JANEIRO – RJ)


A
P
α –γ γ
90° –α
α–β
γ α α
Q
α
MR
δ S α–γ
E
α
α–γ O
90°– α N
B β

C
γ

X
ˆ = 90° − α e sendo + ABO isósceles
ˆ de α, logo QAM
a) Chamemos PQS
ABOˆ = 90° − α , então PSQ
ˆ =α .
Logo + PSQ é isósceles.
b) Agora chamemos PAQ ˆ = γ e OBC
ˆ = β teremos então que, como

ˆ = BXP
BPS ˆ = γ , + APQ ≅+PSB , logo AQ = PQ ∴ PQ ⋅ PS = AQ ⋅ BS
ˆ = PAB
PS BS

EUREKA! N°16, 2003

34
Sociedade Brasileira de Matemática

2
Como queremos provar que PQ = QR ⋅ ST , e PQ = PS ,
AQ ST
Basta apenas provar AQ ⋅ BS = QR ⋅ ST ou =
QR BS
AM AM
Pelo + AQM , senα = ∴ AQ =
AQ senα
QR AR AR ⋅ cos β
Pelo + AQR, = ∴ QR =
sen(90 − β ) senα senα
AQ AM
Logo =
QR AR ⋅ cos β
AM
cos(α − β ) =
AR
ST BT BT cos(α − β )
Pelo + BST , = ∴ ST =
sen(90° − α + β ) senα senα
BN BN
Pelo + BSN , senα = ∴ BS =
BS senα
ST BT cos(α − β ) BN
Logo = ; cos β =
BS BN BT

AM BT cos(α − β ) AM BT cos β
= ⇔ = ⇔
AR ⋅ cos β BN AR cos(α − β ) BN
cos(α − β ) cos β
= ⇔ 1 = 1.
cos(α − β ) cos β

PROBLEMA 6: SOLUÇÃO ADAPTADA DE GABRIEL BUJOKAS (SÃO PAULO - SP)


Seja pi o i-ésimo primo positivo.
ϕ ( p1e1 ⋅ p2e2 ⋅ ... ⋅ pnen ) = p1e1 −1 ⋅ p2e2 n −1 ⋅ pnen −1 ( p1 − 1)...( pn − 1); com n, e1 ∈ ] +* (isso
vem diretamente da fórmula). Então basta escrever M! da forma ao lado direito da
igualdade. Para M pequeno é fácil.
1! = 20 ⋅ (2 − 1) = ϕ (2)
2! = 2 ⋅ (2 − 1) = ϕ (4)
3! = 20 ⋅ 31 (2 − 1)(3 − 1) = ϕ (18)

EUREKA! N°16, 2003

35
Sociedade Brasileira de Matemática

4! = 22 ⋅ 31 (2 − 1)(3 − 1) = ϕ (72)
Agora utilizarei indução. Seja pn ≥ 5 o n-ésimo primo. Suponha que para todo
k < pn , k ! possa ser escrito na forma acima utilizando apenas primos menores
que pn na fatoração. Então ( pn − 2)! = ϕ ( p1e1 ⋅ p2e2 ⋅ ... ⋅ pnen−−11 ) implica
pn ! = pn ( pn − 1)( pn − 2)! = ϕ ( pn2 ) ⋅ ( pn − 2)! = ϕ ( p1e1 ⋅ p2e2 ... ⋅ pnen−−11 ⋅ pn2 ). Para os m
com pn < m < pn +1 , m é um produto de primos menores ou iguais a pn , donde
m ! = m ⋅ (m − 1)! também é da forma acima. Conclusão: Para todo M existe um N tal
que M ! = ϕ ( N ).

SOLUÇÕES – NÍVEL 3

PROBLEMA 1: SOLUÇÃO DE THIAGO MORELLO PERES (RIO DE JANEIRO - RJ)


Por absurdo, suponhamos a inexistência da seqüência satisfazendo o item b.
Seja p um número primo maior que 2005003. Seja uma seqüência a progressão
aritmética de primeiro termo p e a razão p:
A = { p, 2 p,3 p..., 2002 p}
Assim qualquer soma é do tipo n ⋅ p com n < p até mesmo para a soma total:
(1 + 2002) ⋅ 2002
p⋅ = p ⋅ 2005003
2
Garante-se assim, que a soma não é potência perfeita, quaisquer que sejam as
parcelas desta.
Como este exemplo não confere com a suposição, esta é um absurdo e, portanto
existem seqüências satisfazendo os itens a e b simultaneamente. cqd.

PROBLEMA 2: SOLUÇÃO DE ELDER RODRIGO B. CAMPOS (RIO DE JANEIRO - RJ)

C e M d D
θ

k
a
c

π–θ
A
B b

EUREKA! N°16, 2003

36
Sociedade Brasileira de Matemática

• 2 p (∆ADM ) = 2 p(, ABCM ) ⇔ a + b + e + k = c + d + k ⇔ c + d = a + b + e (I)


(d + e) c sen θ ab sen θ
• S (, ABCD ) = + .
2 2
cd sen θ
• S ( ∆ADM ) = ⋅ ora, se S (∆ADM ) = S (, ABCM ) e
2
S (∆ADM ) + S (, ABCM ) = S (, ABCD ) ⇒ S (, ABCD ) = 2S (∆ADM ) ⇔
(d + e)c sen θ + ab sen θ = 2cd sen θ ⇔ ec + ab = cd ⇔
(II) ab = c(d – e). De (I): b + a – c = d – e.
(I) em (II) ⇒ ab = c(a + b − c) ⇔ ab = ac + bc − c 2 ⇔
a(b − c) = c(b − c) ⇒ b = c ou a = c
Logo, , ABCD tem dois lados de mesmo comprimento. cqd.

PROBLEMA 3: SOLUÇÃO DE HENRIQUE CHOCIAY (PINHAIS - PR)


A numeração da tabela pode ser comparada com o preenchimento de uma malha de
pontos, observe:
Ex.: Tabela 3 × 4

11 12 3 4

10 1 2 5

9 8 7 6

Malha 3 × 4
Fim

Início

O preenchimento da tabela é análogo à tarefa de passar por todos os pontos da malha


com uma linha única (sem "quebras" ou bifurcações).
A ocorrência de i ao lado de (i + 3), por sua vez, é análoga às figuras:

EUREKA! N°16, 2003

37
Sociedade Brasileira de Matemática

i i+1

i+3 i+2 , , ,

na malha.
O problema torna-se, então, provar que é impossível preencher a tabela sem realizar
uma dessas figuras. A malha é formada por (m – 1)(n – 1) quadrados de 4 pontos
próximos, os quais terão alguns de seus lados preenchidos ao fim do preenchimento.
Se houver quadrado com 3 lados pintados, haverá

i i+3

Ou i ao lado de i + 3.
O total de lados dos quadrados (com multiplicidade) é 4(m – 1) ⋅ (n – 1) = t)
Para fazer a linha, efetuamos (mn – 1) riscos, que podem preencher lados de 1 ou de
2 quadrados.
• Se o risco for feito na lateral da malha, preencherá apenas 1 lado de quadrado.
Exemplo:

• Se o risco for feito no "miolo" da malha, preencherá dois lados de quadrado.


Exemplo:

Supondo a distribuição mais homogênea de lados preenchidos, cada quadrado tem o


mesmo número de seus lados preenchidos.
Se o número de lados preenchidos por riscos for maior que a metade do total de
lados de quadrados, haverá com certeza um quadrado com 3 lados riscados.

EUREKA! N°16, 2003

38
Sociedade Brasileira de Matemática

t)
No >
2
O número de lados preenchidos por p riscos laterais é ( p ⋅ 1) = p
O número de lados preenchidos por (mn – 1) – p riscos no meio é 2(mn – 1) – 2p
O número total de riscos é: [2(mn – 1) – 2p] + (p) = 2mn – p – 2 = No.
O número máximo de riscos laterais é:

n pontos 2(m – 1) + 2(n – 1) – 1

Pmax = 2m + 2n – 5
n–1
riscos/lados

m – 1 riscos

O número mínimo de lados preenchidos é 2mn – Pmax – 2 = 2mn – 2m – 2n + 3 =


= No min.

t)
Se N o min > , fica provado que há (ou similar) e i ao lado de i + 3
2
4(m − 1)(n − 1)
2mn − 2m − 2n + 3 >
2
2mn − 2m − 2n + 3 > 2mn − 2m − 2n + 2
3>2
O número de lados preenchido é maior que a metade do total de lados.

Há e portanto há i ao lado de i + 3 para qualquer tabela.

PROBLEMA 4: SOLUÇÃO ADAPTADA DE RODRIGO KENDY YAMASHITA (SÃO PAULO - SP)


Sejam m e M as somas dos elementos mínimos e máximos dos subconjuntos. Como
o diâmetro de um conjunto é definido como a diferença entre seu máximo e seu
mínimo, a soma desejada é igual a M – m. Note que podemos incluir os subconjuntos
unitários, já que seus máximos e mínimos coincidem.

EUREKA! N°16, 2003

39
Sociedade Brasileira de Matemática

O número k , 1 ≤ k ≤ n, é elemento mínimo dos subconjuntos da forma


{k} ∪ A, sendo A um subconjunto de {k + 1; k + 2;...; n}. Logo k é elemento mínimo
−k
de 2n subconjuntos.
n n −1
Conseqüentemente, m = ∑k ⋅2
k =1
n− k
= ∑ ( n − k ) ⋅ 2k .
k =0

Contemos o número de subconjuntos de diâmetro k. Seja a o mínimo de um desses


subconjuntos. O seu máximo é, então, a + k. Assim, a + k ≤ n ⇔ a ≤ n − k . Logo
podemos escolher a de n – k maneiras. Como há k – 1 números entre a e a + k,

podemos escolher os demais elementos do subconjunto de 2k 1 maneiras. Logo há
(n − k ) ⋅ 2k −1 subconjuntos de diâmetro k. Como há, no total, 2n − n − 1 subconjuntos
não vazios e não unitários,
n −1 n −1 n −1

∑ (n − k ) ⋅ 2
k =1
k −1
= 2 − n − 1 ⇔ 2∑ (n − k ) ⋅ 2
n

k =1
k −1
=2 n +1
− 2n − 2 ⇔ ∑ (n − k ) ⋅ 2k =
k =1
n −1
= 2n +1 − 2n − 2 ⇔ ∑ (n − k ) ⋅ 2k = 2n +1 − 2n − 2 + n = 2n +1 − n − 2
k =0
n +1
Logo m = 2 − n − 2.
Para calcular M, basta observar que podemos associar cada conjunto
A = {a1 ; a2 ;...; an } ao conjunto f ( A) = {n + 1 − a1; n + 1 − a2 ;...; n + 1 − an }, de
modo que se a = mín A então n + 1 − a = max f ( A). A função f é claramente uma
bijeção. Logo, 2n − 1
como subconjuntos
há não vazios,
M = (n + 1) ⋅ (2 − 1) − m ⇔ M − m = (n + 1) ⋅ 2 − n − 1 − 2m ⇔ M − m =
n n

= (n + 1) ⋅ 2n − n − 1 − 2 ⋅ (2n +1 − n − 2) = (n − 3) ⋅ 2n + n + 3.

PROBLEMA 5: SOLUÇÃO DE RAFAEL DAIGO HIRAMA (CAMPINAS – SP)


Podemos supor então que os quadrados têm lado menor que 1, caso contrário é só
posicionar o quadrado de 1 ou mais sobre o quadrado a se cobrir.
Vamos classificar os quadrados como do tipo Qk tal que o lado do quadrado seja
1 1
menor que k −1
e maior ou igual a .
2 2k
Se tivermos um Q0 acabou, pois ele terá lado maior ou igual a 1 e pronto.
Caso contrário vamos dividir o tabuleiro em 4 partes iguais. Cada uma tem lado 1/2,
ou seja, é satisfatoriamente coberto por um Q1 cada um. Então se posiciona todos os
disponíveis. Se tiver pelo menos 4 Q1 acabou.

EUREKA! N°16, 2003

40
Sociedade Brasileira de Matemática

Caso contrário os que sobraram devem ser divididos em 4 e preenchidos por quantos
Q2 tiverem. E assim sucessivamente até preencher o tabuleiro.
Exemplo:

Q2 Q2 Q2 Q2
Q1

Q3 Q1
Q3

Q2 Q2 Q2
Q3 Q3

Agora, para provar que isso sempre é possível basta provar que a área total dos
quadrados usados é menor que 4. Assim, já que o modo de preenchimento pede "use
tantos do Qk quanto existirem", se sobrar buraco ou esqueceu-se de usar um
quadrado em um passo anterior ou falta usar os quadrados menores.
Para isso vamos ver o desperdício de cada quadrado, ou seja, quanto do quadrado
não usamos para preencher a área de interesse (por exemplo, o desperdício de um
quadrado Q3 ao ser colocado sobre um tabuleiro de lado 1/8 é o quanto do quadrado
ficará de fora desse tabuleiro, mesmo que esse resto esteja sobre outra parte do
tabuleiro total ele vai ser contado como desperdício).
1
Vejamos, como sempre usamos Qk para cobrir um tabuleiro de lado , a área do
2k
1 1 1
Qk é 2k − 2
no máximo e 2k
no mínimo e a do tabuleiro é 2 k , logo o desperdício
2 2 2
1 1 3
é de 2k − 2
− 2k
= 2 k no máximo, isso prova que o desperdício não passa de
2 2 2
3
2 2 k = 3 vezes da área preenchida, ou seja, é desperdiçado no total no máximo 3/4
1
2 2k
da área dos quadrados utilizados, ou seja, 1/4 pelo menos é utilizado. Como o total
da área dos quadrados é 4, a área utilizada é pelo menos 1, o que termina o
problema.

EUREKA! N°16, 2003

41
Sociedade Brasileira de Matemática

PROBLEMA 6: SOLUÇÃO DE FÁBIO DIAS MOREIRA (RIO DE JANEIRO - RJ)


Definição: A distância entre duas palavras p e q é o número de posições em que duas
palavras diferem (símbolo: d(p, q)).

Teorema 1: d(p, q) + d(q, r) ≥ d(p, r).


Prova: Seja ∆αβ o conjunto das posições em que α e β diferem. Então o teorema
equivale a #(∆ pq ) + #(∆ qr ) ≥ #(∆ pr ). Mas ∆ pr = (∆ pq ∪ ∆ qr ) − (∆ pq ∩ ∆ qr ) , pois
só há dois tipos de nuvens, logo p e r são iguais nas posições onde ambos diferem de
q. Mas ∆ pq ∩ ∆ qr ⊂ ∆ pq ∪ ∆ qr , logo #(∆ pr ) = #(∆ pq ∪ ∆ qr ) − #(∆ pq ∩ ∆ qr ) =
= #(∆ pq ) + #(∆qr ) − 2#(∆ pq ∩ ∆qr ) , e nossa afirmação equivale a #(∆ pq ∩ ∆qr ) ≥ 0,
obviamente verdadeiro.

Definição: A palavra real mais próxima a uma palavra q é a palavra p que:


i) Pertence ao dicionário.
ii) Minimiza a distância entre p e q.
(se existir mais de uma palavra que atende i) e ii) todas elas são mais próximas).

Definição: A vizinhança de uma palavra p pertencente ao dicionário é o conjunto de


todas as palavras mais próximas a p (símbolo: ε p ).
Teorema 2: Toda vizinhança de uma palavra p contém todas as palavras cujas
distâncias até p sejam menores ou iguais a 4.
Prova: Seja q tal que d ( p, q) ≤ 4 mas q ∉ ε p . Então q ∈ ε r para r pertencente ao
dicionário. Isso implica d(q, r) < d( p, q) ≤ 4, logo d( p, r) ≤ d( p, q) + d(q, r) < 4+ 4 = 8 ,
absurdo, pois p e r não poderiam estar simultaneamente no dicionário.
Teorema 3: Toda palavra p pertence a no máximo seis vizinhanças.
Prova: Suponha que p ∈ ε q1 ∩ ε q2 ∩ ε q3 ∩ ε q4 ∩ ε q5 ∩ ε q6 ∩ ε q7 . Como
d (qi , q j ) ≤ d ( p, qi ) + d ( p, q j ), d ( qi , q j ) ≤ 8. Como d (qi , q j ) ≥ 8, d ( qi , q j ) = 8.
Como d ( p, qi ) = d ( p, q j ), d ( p, qi ) = 4, ∀i ∈ {1,..., 7}. Como cada palavra só tem
24 nuvens, existem duas palavras (q1 e q2, sem perda de generalidade) tais que
∆ pq1 ∩ ∆ pq2 ≠ φ . Mas então, pelos argumentos do teorema 1,
d (q1, q2 ) = d ( p, q1) + d ( p, q2 ) − 2#(∆pq1 ∩∆pq2 ) ⇒ d (q1, q2 ) = 8 − 2#(∆pq1 ∩∆pq2 ) ≤ 8 − 2 ⋅1 = 6
absurdo, pois q1 e q2 não poderiam pertencer simultaneamente ao dicionário.

N é máximo quando todas as palavras distam no máximo quatro da palavra do


dicionário mais próxima a ela e todas as palavras que distam exatamente quatro da

EUREKA! N°16, 2003

42
Sociedade Brasileira de Matemática

palavra do dicionário mais próxima pertencem a seis vizinhanças, já que isso


caracteriza a formação mais densa possível, devido ao seguinte teorema:

Teorema 4: p ∈ ε q ∩ ε r , q ≠ r ⇒ d ( p, q) = d ( p, r ) = 4.
Prova: Suponha que d ( p, q) = d ( p, r ) < 4 (a igualdade é obrigatória pela definição
de vizinhança). Então d (q, r ) ≤ d ( p, q) + d ( q, r ) < 4 + 4 = 8 , absurdo, pois as duas
palavras não poderiam pertencer simultaneamente ao dicionário.
Porquê isso valida nossa afirmação acima? Porque nenhum ponto que dista três ou
menos ao ponto mais próximo pertence a mais de uma vizinhança. Assim, o arranjo
descrito acima é o mais denso, pois todas as palavras que não pertencem ao
dicionário estão sempre cercadas por palavras do dicionário (pertencem sempre ao
número máximo de vizinhanças).
Nas circunstancias acima descritas #(ε p ) = #(ε q ) para todo p e q (pois
C240 + C24
1
+ C242 + C24
3
+ C244 é constante e igual a #(ε p ) por (1)). Além disso,
∑ #(ε
p∈D
p ) , onde D é o dicionário, seria 224 , mas não, é pois as palavras que distam

quatro de uma palavra no dicionário são contadas seis vezes. Vamos achar então n'p ,
com um fator de correção apropriado:

n 'p = C24
0
+ C24
1
+ C242 + C24
3
+ C24
4
d =4
6
d=0 d =1 d =2 d =3
(própria Vamos contar só uma vez!
palavra)

10626
n'p = 1 + 24 + 276 + 2024 + = 4096 (que coincidência!)
6
Mas até agora consideramos o melhor caso – há algum desperdício de palavras
envolvido. Logo algumas vizinhanças são maiores do que são no caso ideal. Por isso,
n'p ≥ 4096 em geral.
Assim 224 ≥ 4096 N = 212 N ⇒ N ≤ 212 = 4096.

EUREKA! N°16, 2003

43
Sociedade Brasileira de Matemática

XXIV OLIMPÍADA BRASILEIRA DE MATEMÁTICA


Problemas e Soluções da Primeira Fase – Nível Universitário

PROBLEMA 1
A função f : (−1,+∞) → # é contínua e derivável.
Sabe-se que f(0) = 0, f '(0) = a e que f(x + 1) = e f(x) para todo x > – 1.
Calcule f '(3).

PROBLEMA 2
Seja A a matriz real n × n
x + y x ... x 
 
x x + y ... x 
A=

    .

x
 x  x + y 

Diga para que valores de x e y a matriz A é inversível e calcule A– 1.

PROBLEMA 3
x2 +1 + x −1

1
Calcule dx.
−1
x2 + 1 + x + 1

PROBLEMA 4
Determine todos os valores inteiros positivos de m para os quais o polinômio
(x + 1)m + xm + 1 é divisível por (x2 + x + 1)2.

PROBLEMA 5
Jogamos 10 dados comuns (com 6 faces equiprováveis numeradas de 1 a 6).
Calcule a probabilidade de que a soma dos 10 resultados seja igual a 20.

PROBLEMA 6
Considere a curva C = {( x, y ) ∈ # 2 y 2 = x 3 − 43x + 166}.
a) Seja Q = (a, b) um ponto de C. Suponha que a reta tangente a C no ponto Q
intersecte C num único outro ponto, Q'. Determine as coordenadas de Q'.

b) Seja P0 = (3, 8). Para cada inteiro não negativo n, definimos Pn +1 = P' n , o
ponto de interseção de C com a reta tangente a C em Pn. Determine P2002.

EUREKA! N°16, 2003

44
Sociedade Brasileira de Matemática

SOLUÇÃO DO PROBLEMA 1
Derivando a equação f ( x + 1) = e f ( x ) temos f ' ( x + 1) = f ' ( x) ⋅ e f ( x ) .
Assim f (1) = e 0 = 1, f ' (1) = f ' (0) ⋅ e f (0 ) = a
f (2) = e1 = e, f ' (2) = f ' (1) ⋅ e f (1) = ae
f '(3) = f '(2) ⋅ e f (2) = ae e+1 .

SOLUÇÃO DO PROBLEMA 2
Se y = 0 todas as linhas são iguais e a matriz não é inversível. Se nx + y = 0 a soma
das n linhas é 0 e portanto a matriz novamente é não inversível. Vamos mostrar que
se nenhuma destas duas condições ocorre a matriz é inversível.

1 1 ... 1 
 
1 1 ... 1 
Se B =  temos B2 = nB e A = yI + xB.
     
 
1 1  1
1 x
Tome C = I− B.
y y ( nx + y )
1 
AC = (yI + xB ) I −
x
B  = I
y y (nx + y ) 

Comentário (não faz parte da solução)


Encontramos C conjecturando que A −1 = uI + vB.
E resolvendo um sistema para encontrar u e v. Pode-se demonstrar antes de tentar
resolver o sistema que A–1, se existir, deve ter a forma acima: A– 1 é uma função
analítica de A, logo um polinômio em A, logo um polinômio em B. Como
observamos que B2 é um múltiplo escalar de B segue que todo polinômio em B é da
forma uI + vB.

SOLUÇÃO ALTERNATIVA
Vamos resolver o sistema
( x + y )a1 + x ⋅ a 2 + ... + x ⋅ a n = b1
 x a ( x y ) a ... x a b
 ⋅ 1+ + 2 + + ⋅ n= 2

#
 x ⋅ a1 + x ⋅ a 2 + ... + ( x + y ) ⋅ a n = bn

EUREKA! N°16, 2003

45
Sociedade Brasileira de Matemática

Somando todas as equações, obtemos ( nx + y )(a1 + ... + a n ) = (b1 + ... + bn ), donde


x
x(a1 + ... + an ) = (b1 + ... + bn ) , caso nx + y ≠ 0.
nx + y
Diminuindo essa igualdade da j-ésima equação, obtemos
x
y ⋅ a j = bj − (b1 + ... + bn ) e, caso
nx + y
−x (n −1) x + y x
y ≠ 0, a j = b1 − ...+ bj − ... − bn.
y(nx + y) y(nx + y) y(nx + y)
 (n − 1) x + y −x −x 
 ... 
 y (nx + y ) y (nx + y ) y( nx + y ) 
 −x ( n − 1) x + y −x 
Assim,  ... 
A −1 =  y (nx + y ) y (nx + y ) y( nx + y ) .


   

 −x
 y (nx + y )
−x
y (nx + y )
 (n − 1) x + y 
y (nx + y ) 

Note que, se nx + y = 0, o sistema não tem solução se b1 +...+ bn ≠ 0, e, se y = 0, o
x
sistema não tem solução se b j − (b1 + ... + bn ) ≠ 0 para algum j . Em nenhum
nx + y
desses casos A é invertível.

SOLUÇÃO DO PROBLEMA 3
Seja f ( x) = x + 1 + x − 1 . Racionalizando, temos
2

x2 +1 + x +1
2
( x + 1 + x − 1) x + 1 − x − 1  x + 1 − 1 − x
2 2 2 2
 = 
f ( x) = , logo f (− x ) = − f ( x), para
( x + 1) − ( x + 1)
2 2
− 2x


1
todo x, e portanto, f ( x) dx = 0.
−1

SOLUÇÃO ALTERNATIVA
x 2 +1 + x −1
Vamos achar uma primitiva de f: Em ∫
x 2 +1 + x +1
dx fazemos x = tan θ, dx =

sec2θ dθ , e, como tan 2 θ + 1 = sec θ (para − π < θ < π ), obtemos


2 2
sec θ + tan θ − 1
∫ sec θ + tanθ + 1 ⋅ sec
2
θ dθ =

EUREKA! N°16, 2003

46
Sociedade Brasileira de Matemática

1 + senθ − cos θ
∫ cos 2
θ (1 + senθ + cos θ )
dθ .

θ 2dz 2z 1− z 2
Fazendo tan = z, dθ = , senθ = , cos θ = , obtemos
2 1+ z 2 1+ z 2 1+ z 2
2
2 z 2 + 2 z  1 + z 2  2dz 2 z (1 + z 2 )dz
∫ 2 + 2 z ⋅  1 − z 2  1 + z 2 = ∫ (1 − z 2 ) 2 .
A B C D 2 z (1 + z 2 )
Buscando A, B, C, D tais que + + + = , obtemos (A
1 + z (1 + z) 2 1 − z (1 − z ) 2 (1 − z 2 ) 2
+ B + Az)(1 – z)2 + (C + D – Cz)(1+z)2 = 2z(1 + z2), donde A – C = 2, B – A + D – C
= 0, –A –2B + C + 2D = 2, A + B + C + D = 0. Assim, D = –B, C = – A, logo A = 1,
C = –1, D = 1, B = –1.
2 z (1 + z 2 ) 1 1 2
Assim, ∫ (1 − z 2 2
)
dz = ln(1 + z) +
1+ z
+ ln(1 − z ) +
1− z
= ln(1 − z 2 ) +
1− z 2
.

−π π π 
Quando x varia entre – 1 e 1, θ varia entre e , donde z varia entre − tan 
4 4 8
π 
π  sen  
e tan   . Temos tan  π  = 4 = 2
= 2 − 1, donde z varia entre 1 − 2 e
8   1 + cos
8  π  2+ 2
 
4

2 − 1. Assim, a integral é ln(1 − z 2 ) + 2 2 −1 pois ( 2 − 1) 2 = (1 − 2 ) 2 .


= 0,
1 − z 1− 2
2

SOLUÇÃO DO PROBLEMA 4
−1 i 3
Seja ω = + uma raiz de x2 + x + 1. Para que (x2 + x + 1)2 divida
2 2
(x + 1)m + xm + 1 = P(x), devemos ter P(ω) = 0 e P'(ω) = 0.
Assim, (ω + 1) m + ω m = −1 e m((ω + 1) m−1 + ω m −1 ) = 0.
1 i 3
Temos que ω + 1 = + é tal que (ω + 1) 2 = ω e (ω + 1) 3 = −1 . Como ω e ω + 1
2 2
são raízes sextas da unidade, o comportamento se repetirá com período 6.
Assim, (ω + 1) m −1 + ω m −1 = 0 equivale a (ω + 1) m −1 = −ω m −1 = (ω + 1) 3+ 2 ( m −1) , ou seja
(ω + 1) m + 2 = 1, o que equivale a m ≡ –2 (mod 6). Nesse caso, temos
(ω + 1) m + ω m = (ω + 1) 4 + ω = ω 2 + ω = −1, donde as duas condições são
satisfeitas. Assim, os números que satisfazem o enunciado são os inteiros positivos
da forma 6k – 2.

EUREKA! N°16, 2003

47
Sociedade Brasileira de Matemática

SOLUÇÃO DO PROBLEMA 5
Devemos encontrar o número de soluções de a1 + a 2 + ... + a10 = 20, 1 ≤ a i ≤ 6.
19 
O número de soluções de a1 + a 2 + ... + a10 = 20, ai ≥ 1 é claramente   .
 9
Devemos agora descontar as soluções para as quais apenas um dentre os ai é ≥ 7 pois
caso contrário tal soma seria ≥ 7 + 7 + 8 ⋅ 1 = 22. Assim, basta descontar 10 vezes o
número de soluções de a1 + a 2 + ... + a10 = 20, a1 ≥ 7 , ai ≥ 1 ,
ou de a~ + a + ... + a = 14, a , a ≥ 1, que é 13  . Assim 19  13  e a
1 2 10 1 i   N =   − 10 
 9 9  9
19  13 
  − 10 
probabilidade pedida é p =   10  9  .
9
6
SOLUÇÃO DO PROBLEMA 6
dy
= 3x 2 − 43, donde dy = 3 x − 43 .
2
a) De y 2 = x 3 − 43x + 166, temos 2 y
dx dx 2y
A equação da reta tangente a C passando por (a, b) é
 3a 2 − 43    3a 2 − 43  
x + b −    Substituindo em y = x − 43 x + 166 temos
2 3
y =     2b  ⋅ a .
 2b     
2
  3a 2 − 43  2b 2 − 3a 3 + 43a  que terá uma raiz dupla em x = a,
x − 43 x + 166 =  
3
x +
 ,
 
  2b  2b 
2
 2 
e cuja soma das raízes é  3a − 43  . Assim, o outro ponto terá primeira coordenada
 
 2b 
2
 
igual a  3a − 43  − 2a , e, substituindo na equação da reta, segunda coordenada
2

 
 2b 
3 3
   2   2   2 
igual a  3a − 43  − 2a 3a − 43  + 2b − 3a + 43a =  3a − 43  +  2b − 9a + 129a  .
2 2 3 3

 2b   2b     
   2b  2b 2b    

b) Usando a fórmula acima, obtemos P1 = (–5, 16), P2 = (11, 32), P3 = (3, – 8), P4 =
(–5, – 16), P5 = (11, – 32) e P6 = (3, 8). Assim, a seqüência (Pn) é periódica de
período 6, logo P2002 = P4 = ( −5, −16) .

Observação: No item b), o fato de P3 diferir de P0 apenas por uma troca de sinal da
segunda coordenada já é suficiente para concluir que a seqüência é periódica de
período 6.

EUREKA! N°16, 2003

48
Sociedade Brasileira de Matemática

XXIV OLIMPÍADA BRASILEIRA DE MATEMÁTICA


Problemas e Soluções da Segunda Fase – Nível Universitário
PROBLEMA 1
Seja y = P(x) um polinômio de grau 4. Mostre que se existe uma reta (em #2) que
corta o gráfico de P em 4 pontos então existe uma reta que corta o gráfico em 4
pontos igualmente espaçados.

PROBLEMA 2
n
A = (a ij ) uma matriz real simétrica n × n tal que aii = 1 e ∑a
j =1
ij < 2 , para todo

i ∈{1,2,..., n} . Prove que 0 < det A ≤ 1.

PROBLEMA 3
n n
Sejam A1 , A2 ,..., Ak ⊂ {1, 2,..., n} conjuntos com A i ≥ e Ai ∩ A j ≤ para
2 4

* A ≥ k k+ 1 ⋅ n.
k
todo i, j com i ≠ j . Prove que i
i =1

PROBLEMA 4

Determine todas as soluções reais da equação x = 2 + 2 − 2 + x.

PROBLEMA 5
Dado x ∈ #, definimos ln 0 ( x) = x e, para cada k ∈ , se ln k ( x) > 0 , definimos
ln k +1 ( x) = ln(ln k ( x)), onde ln é o logaritmo natural.
Dado n inteiro positivo, definimos k(n) como o maior k tal que ln k ( n) ≥ 1 , e an como
k (n)

∏ ln
j =0
j (n) = n ⋅ ln(n) ⋅ ln ln(n) ⋅ ... ⋅ ln k ( n) (n).

1
Diga se a série ∑a
n =1
converge ou diverge.
n

PROBLEMA 6
Considere duas elipses no plano #2 que se intersectam em 4 pontos. Nestes 4 pontos
trace as retas tangentes às duas elipses, obtendo assim 8 retas.
Prove que existe uma elipse (ou circunferência) tangente a estas 8 retas.

EUREKA! N°16, 2003

49
Sociedade Brasileira de Matemática

SOLUÇÕES – NÍVEL UNIVERSITÁRIO

PROBLEMA 1: SOLUÇÃO DE FABRÍCIO SIQUEIRA BENEVIDES (FORTALEZA – CE)


Seja P ( x ) = ax 4 + bx 3 + cx 2 + dx + e e seja A( x) = kx + q a reta que o intersecta
em 4 pontos.
Ou seja, Q( x) = P( x) − A( x) tem quatro raizes.
Queremos mostrar que existe r ( x) = tx + s tal que S ( x ) = P( x ) − r ( x) tem
quatro raizes igualmente espaçadas.
S ( x ) = ax 4 + bx 3 + cx 2 + (d − t ) x + (e − s )
= ax 4 + bx 3 + cx 2 + d ' x + e '.
Note que nosso problema é equivalente a dados a, b, c achar d', e' tais que
S(x) acima tenha 4 raizes igualmente espaçadas.
Primeiro, mostraremos que é possível escolher d' de modo que S(x) seja
−b  −b   −b 
simétrico em relação à reta x = , isto é, S  −k = S  + k  , ∀k ∈ \ .
4a  4a   4a 
−b
Para escrever menos seja u = .
4a
S (u − k ) = S (u + k ) ⇔ a (u − k ) 4 + b(u − k )3 + c(u − k ) 2 + d '(u − k ) + e ' =
= a(u + k )4 + b(u + k )3 + c(u + k )2 + d '(u + k ) + e ' ⇔
⇔ a(u4 − 4u3k + 6u2k 2 − 4uk3 + k 4 ) + b(u3 − 3u2k + 3uk 2 − k3 ) + c(u2 − 2uk + k 2 ) − dk =
a(u4 + 4u3k + 6u2k 2 + 4uk 3 + k 4 ) + b(u3 + 3u2k + 3uk 2 + k 3 ) + c(u2 + 2uk + k 2 ) + d ' k =
⇔ 8au 3 k + 8auk 3 + 6bu 2 k + 2bk 3 + 4cuk + 2d ' k = 0
 −b 
u = ⇒ 8auk 3 = −2bk 3 
 4a 
⇔ (4au + 3bu + 2cu + d ') ⋅ k = 0
3 2

Basta então tomar d ' = −4au 3 − 3bu 2 − 2cu


O fato de já existir uma reta que intersecta o P(x) inicial em 4 pontos, nos diz
que a, b, c nos foram dados de modo que S(x) tenha 3 pontos de máx/mín
locais. (Senão o gráfico de S(x) seria convexo ou côncavo, e qualquer reta o
intersectaria em no máximo 2 pontos).
Logo, o gráfico de S(x) é algo do tipo:

EUREKA! N°16, 2003

50
Sociedade Brasileira de Matemática

ou

−b
x=
4a

Finalmente mudar e' significa transladar o gráfico de S para cima ou para baixo.
Claramente podemos escolher um e tal que S tenha 4 raízes x1e , x2e , x3e , x4e .

Para cada e' desses considere as funções

f (e) = x2e − x1e

s f para r ≤ e ≤ s
g (e) = x3e − x2 e ,
(ver gráfico)
r
h(e) = x4 e − x3 e
Pela escolha de d', S é simétrico e f(e) = h (e).
f (r ) = 0, g (r ) > 0
f ( s ) > 0, g (s ) = 0
Pelo T.V.I. existe e ' ∈ (r , s ) tal que f (e ') = g (e ').
Neste caso, f (e ') = g (e ') = h(e ') , e este é o nosso tão procurado e'.

PROBLEMA 2: SOLUÇÃO DE HUMBERTO SILVA NAVES (S.J. DOS CAMPOS - SP)


Temos que a matriz A é diagonalizável, pois é simétrica, ou seja:
A = H ⋅ D ⋅ H –1 onde H T = H –1 e D é uma matriz diagonal formada pelos auto-
valores de A.
Obs. As matrizes D e H são reais.
Claramente det. A = det. D.
Primeiramente vamos provar que todos os auto-valores de A são positivos:
Seja X um auto-vetor de A, i.e., X é uma matriz n × 1, não nula, tal que

EUREKA! N°16, 2003

51
Sociedade Brasileira de Matemática

 x1 
AX = λ X  
X = #
 xn 

Seja i, tal que 0 < xi = max{ x1 ; x 2 ;...; xn } . Se fosse λ ≤ 0


teríamos: λ xi = ai1 x1 + ai 2 x2 + ... + ain xn ⇔
(λ − 1) xi = ai1 x1 + ai 2 x2 + ... + aii −1 xi −1 + aii +1 xi +1 + ... + ain xn
Temos (λ − 1) xi ≥ xi e ∑
j ≠i
aij < 1 ⇒ ∑
j ≠i
aij x j < xi , pois
1≤ j ≤ n 1≤ j ≤ n

xi = max{ x1 ;...; xn }, logo xi ≤ ( λ − 1) xi = ∑ax


j ≠i
ij j ≤ ∑
j ≠i
aij x j < xi um
1≤ j ≤n 1≤ j ≤ n

absurdo! Logo det D > 0 ⇒ det A > 0.


Claramente um auto-valor de A é uma raiz de P(x) = det (A – xI).
O coeficiente de x n −1 de P(x) é a soma da diagonal principal de A multiplicada por
( −1) , ou seja, ( −1)
n−1 n −1
n . Logo a soma das raízes de P(x) (com suas respectivas
− (−1) n
n −1

multiplicidades) é = n.
(−1) n
Temos: λ1 + λ2 + ... + λn = n onde os λi's são os auto-valores de A, com λi ≥ 0 ∀i ;
1 ≤ i ≤ n. Pela desigualdade das médias, temos:
λ1 + λ2 + ... + λn n
1= ≥ λ1λ2 ...λn ⇒ λ1λ2 λ3 ...λn ≤ 1
n
Mas det D = λ1λ2 ...λn ≤ 1 ⇒ det A ≤ 1.

PROBLEMA 3: SOLUÇÃO DE CARLOS STEIN NAVES DE BRITO (S.J. DOS CAMPOS - SP)

*A =U
k
Seja i
i =1
k
Seja S = ∑ Ai ≥ k ⋅
n
(I)
i =1 2
k
Por absurdo, suponha que U < ⋅ n (II)
k +1
EUREKA! N°16, 2003

52
Sociedade Brasileira de Matemática

Seja ai quantas vezes os elementos de Ai aparecem nos outros A j ( j ≠ i ) , por


exemplo, se A1 = {1, 2,3} , A2 = {1, 2, 5} e A3 = {1,3, 6} , temos que a1 = 4. Temos
k
ai = ∑ Ai ∩ A j
j =1
j ≠i

* A , o seja b o número de A 's que contém t.


k
Para cada t ∈ i
t i
i =1
k
Lema: ∑a i = ∑ b j (b j − 1).
i =1
*A
k
j∈ i
i =1

Prova: Temos que ai é quantas vezes aparece cada t ∈ Ai em outros A j ( j ≠ i ) , logo


cada t aparece em (bt − 1) outros Aj, pois bt conjuntos contém t, tirando o Ai, logo
k k
temos (bt − 1) outros que contém t. Assim ai = ∑ (b −1) ⇒ ∑ a = ∑(∑ (b −1)).
t∈Ai
t
i =1
i
i =1 t∈Ai
t

* A , existem b conjuntos que contém t, logo cada parcela b − 1


k
Para cada t ∈ i t t
i =1
k  
∑  ∑ (b − 1)  , logo ∑ a
k
aparece bt vezes em t i = ∑ b (b − 1). cqd.
t t
i =1  t∈ Ai  i =1
*A
k
t∈ i
i =1

k k
n
Logo ∑ ai = ∑ bt2 − bt = ∑ bt2 − ∑ bt (note que ∑ bt = ∑ Ai ≥ k ⋅ , pois
2
* * * *A
i =1 k k k k i =1
t∈ Ai t∈ Ai t∈ Ai t∈ i
i =1 i =1 i =1 i=1

estamos contando quantas vezes aparece cada elemento). Por Cauchy-Schwarz:


2
 
 
 ∑ bt  2
 k
 t∈ A   ∑ Ai   k 2 ⋅ n2 k 2 ⋅ n2
*
k

(k +1)k ⋅ n
∑k (bt2 ) ≥  i=k1  =  i=1U  = U ≥ U4 > k4 = 4
2
i
S

t∈*Ai
i =1
i =1
Ai * k +1
⋅n

k
(k +1)kn kn kn k ⋅ n(k −1)
→ ∑ai = ∑bt2 − ∑bt > − = (k +1− 2) =
i =1 4 2 4 4

EUREKA! N°16, 2003

53
Sociedade Brasileira de Matemática

k ⋅ n ⋅ (k − 1)
Pelo princípio das casas dos pombos, ∃a j , tal que a j > 4 (senão
k
kn(k − 1) , absurdo).
∑a i

4
k
n(k − 1)
Logo ∃a j , a j >
4
. De novo pela casa dos pombos, como aj =
i=1
Aj ∩ Ai , ∑
i≠ j

n(k − 1)
4aj n n
existe um p tal que A j ∩ Ap ≥ > = ⇒ A j ∩ Ap > , absurdo.
k −1 k −1 4 4
k
Logo U ≥ ⋅n.
k +1
PROBLEMA 4: SOLUÇÃO DE MÁRCIO AFONSO ASSAD COHEN (RIO DE JANEIRO - RJ)
É, tá difícil…vamos tentar uma idéia:
Seja x = 2cos α , com α ∈ (0, π / 3) (Ok, pois já sei que 1 < 2≤ x ≤ 2 + 2 < 2).
Obs: cos 2θ = 2 cos 2 θ − 1 = 1 − 2sen 2θ
2 + x = 2(1 + cos α ) = 4 cos 2 (α / 2) → 2 + x = 2 cos(α / 2)
(pois α / 2 ∈ (0, π / 6) logo cos(α / 2) > 0 ).
2 + x = 2 cos(α / 2) → 2 − 2 + x = 2 − 2 cos(α / 2) = 2(1 − cos(α / 2)) =
= 2 ⋅ 2 sen 2 (α / 4) → 2 − 2 + x = 2 sen(α / 4)
(pois sen(α / 4) > 0 ).
Logo, x = 2 + 2sen(α / 4) → Obs.: sen(α / 4) = cos(π / 2 − α / 4)
2 + 2sen(α / 4) = 2 + 2cos(π / 2 −α / 4) = 2⋅ [1+ cos(π / 2 −α / 4)] = 2 2⋅ cos2 (π / 4 −α /8)
Portanto, tirando raiz: x = 2cos(π / 4 − α / 8), i.e.
2 cos α = 2 cos(π / 4 − α / 8) → cos α = cos(π / 4 − α / 8)
→ α = π / 4 − α / 8 → 9α / 8 = π / 4 → α = 2π / 9
Logo, x = 2cos 2π / 9 é a única solução real da equação.

EUREKA! N°16, 2003

54
Sociedade Brasileira de Matemática

PROBLEMA 5: SOLUÇÃO DA BANCA


Sejam b0 = 1, bk +1 = e bk para todo k ∈ .
Para bk ≤ n < bk +1 temos k(n) = k.
k
1
Como a derivada de ln k +1 ( x) é ∏ ln ( )
=: g k ( x), temos, para cada n,
j =0 j x
n +1
ln k +1 ( n + 1) − ln k +1 ( n) = ∫ g k ( x) dx ≤ g k ( n ), para todo n ≥ bk, pois gk é decrescente.
n

= ∑ g k (n ) ≥ ln k +1 ( bk +1  ) − ln k +1 ( bk  ) .
1
Assim, ∑
bk ≤ n < bk +1 an bk ≤ n < bk +1

1 1
Como ln k +1 ( bk  ) < ln k +1 (bk + 1) < ln k +1 (bk ) + = para todo k ≥ 1 e
2 2
ln k +1 ( bk +1  ) ≥ ln k +1 (bk +1 ) = 1 , temos ∑
1 1
> para todo k, donde
bk ≤ n < bk +1 a n 2
∞ ∞
 1

n =1
a n = ∑  ∑  diverge.
k = 0  bk ≤ n < bk +1 an 

PROBLEMA 6: SOLUÇÃO DA BANCA


Vamos utilizar coordenadas projetivas (Ref.: "Aplicações de planos projetivos em
Teoria dos Números e Combinatória" de Carlos Yuzo Shine - Eureka! No. 15).
Consideremos as duas cônicas do problema inseridas no plano projetivo 2 . O fato
de serem elipses significa que essas cônicas não cortam a "reta do infinito"
z = 0.
Lema: Se, A, B, C e D são tais que 3 quaisquer não são colineares, existe
um sistema de coordenadas projetivas no qual A = [1, 0, 0], B = [0, 1, 0],
C = [0, 0, 1] e D = [1, 1, 1]
G GG G
Demonstração: Sejam a , b , c , d vetores de  3 representando as classes de
equivalência de A, B, C e D respectivamente. Como A, B e C não são
G G G
colineares, a , b e c são l.i., logo existem reais k, l, m tais que
G G G G
d = ka + lb + mc . Como d não pertence às retas AB, BC ou AC, k, l, m são
G G G
diferentes de zero. Assim, se considerarmos a base ka, lb , mc de  3 teremos
A = [1, 0, 0], B = [0, 1, 0], C = [0, 0, 1], D = [1, 1, 1], o que prova o lema.
EUREKA! N°16, 2003

55
Sociedade Brasileira de Matemática

Sejam A, B, C e D os pontos de interseção entre as duas elipses. Usando o


lema, podemos realizar uma mudança de coordenadas que leve os pontos
[–1, 1, 1], [–1, –1, –1], [1, –1, 1] e [1, 1, 1] em [1, 0, 0], [0, 1, 0], [0, 0, 1] e
[–1, 1, 1] como toda mudança de coordenadas é invertível, usando o lema
podemos realizar uma mudança de coordenadas que leve os pontos A, B, C e
D em [1, 1, 1], [–1, 1, 1], [–1, –1, 1], [1, –1, 1].
Podemos escolher esse novo sistema de coordenadas de modo que as duas
cônicas continuem sendo elipses, em relação à reta do infinito z = 0.
De fato a família de cônicas no plano que passam por (–1, –1), (–1, 1), (1, –1)
e (1, 1) é dada pelas equações tx 2 + (1 − t ) y 2 = 1 , t ∈  ( se t ∈{0,1} a cônica se
degenera num par de retas). No novo sistema de coordenadas temos duas
cônicas dessa família. Se uma delas é uma hipérbole, digamos
tx 2 + (1 − t ) y 2 = 1 , com t > 1, podemos aplicar a mudança de coordenadas
 y 1
projetivas que leva [X, Y, Z] em [Y, Z, X] (e, no plano, leva (x, y) em  ,  ):
 x x
a imagem de Q = {(–1, –1), (–1, 1), (1, –1), (1, 1)} ainda é Q e a imagem da
 (t − 1) x 2 y 2 
hiperbole ( tx 2 + (1 − t ) y 2 = 1 ) é a elipse  + = 1 .
 t t 
Assim, temos agora duas cônicas que passam pelos pontos de Q tais que, se
uma delas é uma hiperbole então a outra é uma elipse. Assim, ou essas
cônicas são duas elipses ou qualquer reta no plano intersecta uma dessas
cônicas. O segundo caso não é possível, pois nesse caso as cônicas não
poderiam ser imagem de duas elipses por uma mudança de coordenadas
projetivas, dado que a imagem da reta do infinito, que continua sendo uma
reta, sempre intersecta uma dessas cônicas.

Agora, temos duas elipses que passam pelos pontos de Q. Suponhamos que
suas tangentes no ponto (1, 1) sejam as retas (ax + by = 1) e (cx + dy = 1),
com a, b, c, d > 0, a + b = c + d = 1. Após aplicarmos uma mudança de
coordenadas afim do tipo T(x, y) = ( x / λ , y ) , com λ > 0, obtemos
duas outras elipses cujas retas tangentes em T (1, 1) = (1/ λ ,1) são
(aλ x + by = 1) e (cλ x + dy = 1). As distâncias dessas retas à origem são,
1 1
respectivamente, e .
a 2λ 2 + b 2 c2λ 2 + d 2

EUREKA! N°16, 2003

56
Sociedade Brasileira de Matemática

d 2 − b2
Temos a λ + b = c λ + d ⇔ λ = 2 2 ,
2 2 2 2 2 2 2
que é positivo, pois
a −c
a + b = c + d ⇒ ( a − c > 0 ⇔ d − b > 0).
d 2 − b2
Assim, tomando λ = , e aplicando T ( x, y ) = ( x / λ , y ) às nossas duas
a2 − c2
elipses, obtemos duas elipses que se intersectam em quatro pontos de modo
que todas as 8 retas tangentes às duas elipses nesses pontos estão a uma
mesma distância da origem (por simetria), e logo existe uma círculo tangente
a todas elas, o qual está contido na união dos interiores dessas elipses, e
portanto não intersecta a imagem da reta do infinito pela mudança de
coordenadas projetivas que leva as elipses originais nestas, e logo é imagem
de uma elipse por essa mudança de coordenadas. Essa elipse é tangente às 8
retas do enunciado. Isso resolve o problema.

Nota: Os enunciados dos problemas 3 e 5 da segunda fase do Nível Universitário

*A
n
saíram com alguns erros na prova: no problema 3, aparecia i em vez de
i =1

*
k k (n)

i =1
Ai , e , no problema 5, aparecia ∏ln (x) = x ⋅ ln(x) ⋅ lnln(x) ⋅...⋅ ln
j =0
j k (n) (x)

k ( n)
em vez de ∏ln (n) = n⋅ ln(n) ⋅ lnln(n) ⋅...⋅ ln
j =0
j k ( n) (n).

♦♦♦

Agradecemos a Okakamo Kokobongo Matsubashi pela revisão deste número.

♦♦♦
Errata: No artigo "Reciprocidade Quadrática", de Carlos Gustavo Moreira e Nicolau Saldanha
(publicado na Eureka! No. 15), onde está "símbolo de Lagrange" deveria ser "símbolo de
Legendre".

EUREKA! N°16, 2003

57
Sociedade Brasileira de Matemática

XXIV OLIMPÍADA BRASILEIRA DE MATEMÁTICA


Resultado – Nível 1 (5a. e 6a. Séries)
MEDALHA DE OURO
Henrique Ponde de Oliveira Pinto Salvador - BA
Camila Alves Pereira Gloria do Goitá - PE
Cássio Kendi Takamori São José dos Campos - SP
Jéssica Guerra Caldato Santo André - SP
Vinícius Marques Regitano Piracicaba - SP
MEDALHA DE PRATA
Mário Henrique Mendonça Castilho São João da Boa Vista - SP
Bernardo de Oliveira Veiga Rio de Janeiro - RJ
Rafael Tupynambá Dutra Belo Horizonte - MG
Gabrielle Collato Marcelino Santo André - SP
Maria Fernanda Petri Beto São Paulo - SP
Guilherme Philippe Figueiredo Fortaleza - CE
Cristiano Peres Guimarães Mendonça- SP
Gustavo Henrique dos Santos Figueiredo Santo André - SP
Larissa Lais de Sá São Paulo - SP
Rafael Augusto da Silva Gonçalves Salvador - BA
MEDALHA DE BRONZE
Diogo Bonfim Moraes Morant de Holanda Rio de Janeiro - RJ
Fernanda Sá Leal de Moura Teresina - PI
Luísa Dias Barbosa Alves Recife - PE
David Francisco dos Santos Serra - ES
Marcos Coppa Gomes Filho Natal - RN
Anderson Vasconcelos Maciel Fortaleza - CE
Samuel Carvalho Lima Holanda Fortaleza - CE
Rodolfo de Andrade Marinho Silva Campina Grande - PB
Franz Biondi Siemon Vitória - ES
Rafael Sampaio de Rezende Fortaleza - CE
André Vasconcelos Barros Natal - RN
Guilherme Silva Moura Jequié - BA
MENÇÃO HONROSA
Daniel Luna de Menezes João Pessoa - PB
Laís Moutinho Medeiros Recife - PE
Rafael Moura e Sucupira Fortaleza - CE
Vinicius de Souza Lima e Oliveira Rio de Janeiro - RJ
Weslen Costa Timoteo Paulista - PE
Filipe Alves Tomé Fortaleza - CE
Lays Cardoso Tatagiba Itaperuna - RJ
Marlon Vieira de Lima Júnior Fortaleza - CE
Lukas Carmona Macedo de Souza São Paulo - SP
Alessandro Wagner Palmeira Guarulhos - SP
Nathália Pereira Gonçalves Rio de Janeiro - RJ
Renan Magri Itaporã - PR
Rafael Alcoforado Domingues João Pessoa - PB
Letícia Duarte Ferrari Rio de Janeiro - RJ
Lucio Eiji Assaoka Hossaka Curitiba - PR
Flavia Contartesi São Carlos - SP
Odair Dutra Santana Júnior Botuporanga - SP
Mayara Franco Rodrigues Araraquara - SP
Artur de Almeida Losnak São Paulo - SP
Thaísa Giorno Dantas Rabaneda Lopes Atibaia - SP
Cássio dos Santos Araújo Recife - PE
Amanda Yumi Iseri Uberaba - MG
Tiago Madeira Itajaí - SC
Matheus Mello Asunção Belém - PA
Luísa Castro Noronha Valinhos - SP
Rafael Ellis Reuben São Paulo - SP

EUREKA! N°16, 2003

58
Sociedade Brasileira de Matemática

Resultado – Nível 2 (7a. e 8a. Séries)


MEDALHA DE OURO
Thomás Yoiti Sasaki Hoshina Rio de Janeiro - RJ
André Lucas Ribeiro dos Santos Pindamonhangaba - SP
Vitor Humia Fontoura Salvador - BA
Gabriel Tavares Bujokas São Paulo - SP
MEDALHA DE PRATA
Guilherme Rodrigues Nogueira de Souza São Paulo - SP
Douglas Bokliang Ang Cunha São José dos Campos - SP
Hector Kenzo Horiuti Kitahara São Paulo - SP
Guilherme Rohden Echelmeier Itajaí - SC
Enzo Haruo Hiraoka Moriyama São Paulo - SP
Luty Rodrigues Ribeiro Fortaleza - CE
Eduardo Fischer Encantado - RS
Rafael Kitayama Shiraiwa São Paulo - SP
Thaís Viveiro São Paulo - SP
MEDALHA DE BRONZE
Caio dos Santos Pereira Gazzola Belo Horizonte - MG
Rodrigo Augusto Santana Belém - PA
Rodrigo Viana Soares Fortaleza - CE
André Linhares Rodrigues Fortaleza - CE
Fábio Eigi Imada São José dos Campos - SP
Rafael Montezuma Pinheiro Cabral Fortaleza - CE
Pedro Paulo Gondim Cardoso Salvador - BA
Rhamon Barroso de Sousa Fortaleza - CE
Lucas Magalhães Pereira Castello Branco Fortaleza - CE
Max Douglas Peixoto da Silva Fortaleza - CE
Renata Mayer Gukovas São Paulo - SP
Milena Pinheiro Martins Teresina - PI
Anderson Hoshiko Aiziro São Carlos - SP
MENÇÃO HONROSA
Daniel Yoshio Futenma da Silva São Paulo - SP
Landerson Bezerra Santiago Maracanaú - CE
José Armando Barbosa Filho Fortaleza - CE
Danilo Eiki Yokoyama São Paulo - SP
Fernando Mizoguchi Gorgoll São Paulo - SP
Pedro Thiago Ezequiel de Andrade Fortaleza - CE
José Robério Xavier dos Santos Júnior Fortaleza - CE
Erick Vizolli Curitiba - PR
Camila Vasconcelos de Oliveira Fortaleza - CE
Raphael Rodrigues Mata Salvador - BA
Adriano César Braga Borges Contagem - MG
Gustavo Eidji Camarinha Fujiwara São Paulo - SP
Henrique Kenji Formagio Noguchi São Paulo - SP
André Ikeda Cantão Curitiba - PR
Paulo André Carvalho de Melo Rio de Janeiro - RJ
Fábio Queiroz Vasconcelos Cunha Salvador - BA
Flaviano Ramos Pereira Junior Belém - PA
Mauro Cardoso Lopes São Paulo - SP
Luiz Müller Vitória - ES
Tiago Nery Vasconcelos São Paulo - SP
Thiago de Azevedo Pinheiro Hoshino São Paulo - SP

EUREKA! N°16, 2003

59
Sociedade Brasileira de Matemática

Resultado – Nível 3 (Ensino Médio)


MEDALHA DE OURO
Guilherme Issao Camarinha Fujiwara São Paulo - SP
Fábio Dias Moreira Rio de Janeiro - RJ
Rafael Daigo Hirama Campinas - SP
MEDALHA DE PRATA
Yuri Gomes Lima Fortaleza - CE
Thiago da Silva Sobral Fortaleza - CE
Alex Corrêa Abreu Niterói - RJ
Henrique Chociay Curitiba - PR
Antonio Carlos Maldonado Silveira A. Munhoz Rio de Janeiro - RJ
Henry Wei Cheng Hsu São Paulo - SP
Samuel Barbosa Feitosa Fortaleza - CE
Larissa Cavalcante Queiroz de Lima Fortaleza - CE
Bernardo Freitas Paulo da Costa Rio de Janeiro - RJ
Davi Máximo Alexandrino Nogueira Fortaleza - CE
Thiago Costa Leite Santos São Paulo - SP
MEDALHA DE BRONZE
Einstein do Nascimento Júnior Fortaleza - CE
Eduardo de Moraes Rodrigues Poço São Paulo - SP
Rafael Tajra Fonteles Teresina - PI
Felipe Rodrigues Nogueira de Souza São Paulo - SP
Murilo Vasconcelos Andrade Maceió - AL
Thiago Braga Cavalcante Fortaleza - CE
Paulo Ribeiro de Almeida Neto Ananindeua - PA
Germanna de Oliveira Queiroz Fortaleza - CE
Juliana Gomes Varela Fortaleza - CE
Rodrigo Aguiar Pinheiro Fortaleza - CE
Israel Franklim Dourado Carrah Fortaleza - CE
Daniel Pessoa Martins Cunha Fortaleza - CE
Renato Seiji Tavares São Paulo - SP
Carlos Augusto David Ribeiro Fortaleza - CE
Letícia Rosa dos Santos Rio de Janeiro - RJ
MENÇÃO HONROSA
Rafael Marini Silva Vila Velha - ES
Telmo Luis Correa Junior São Paulo - SP
Diego Alvarez Araujo Correia Fortaleza - CE
Vitor Gabriel Kleine Mogi das Cruzes - SP
Francisco Bruno de Lima Holanda Fortaleza - CE
Diogo dos Santos Suyama Belo Horizonte - MG
Anderson Torres São Paulo - SP
Larissa Rodrigues Ribeiro Fortaleza - CE
Marina Lima Medeiros Fortaleza - CE
Antonia Taline de Souza Mendonça Fortaleza - CE
Rodrigo Angelo Muniz Cariacica - ES
Eduardo Paiva Costa Teresina - PI
Eduardo Monteiro Nicodemos Rio de Janeiro - RJ
Thiago Morello Peres Rio de Janeiro - RJ
Elder Rodrigo Barbosa Campos Rio de Janeiro - RJ
Thiago Luís Viana de Santana Rio de Janeiro - RJ
Filipe Rodrigues de Souza Moreira Rio de Janeiro - RJ
Rodrigo Kendy Yamashita São Paulo - SP
João Marcos da Cunha Silva Fortaleza - CE
Lyussei Abe São Paulo - SP

EUREKA! N°16, 2003

60
Sociedade Brasileira de Matemática

Resultado – Nível Universitário


MEDALHA DE OURO
Carlos Yuzo Shine São Paulo - SP
Humberto Silva Naves São José dos Campos - SP
Marcio Afonso Assad Cohen Rio de Janeiro - RJ
MEDALHA DE PRATA
Thiago Barros Rodrigues Costa Fortaleza - CE
Carlos Stein Naves de Brito São José dos Campos - SP
Rodrigo Villard Milet Rio de Janeiro - RJ
Daniel Massaki Yamamoto São Paulo - SP
Giuliano Boava Florianópolis - SC
Fabrício Siqueira Benevides Fortaleza - CE
Eduardo Famini Silva Rio de Janeiro - RJ
MEDALHA DE BRONZE
Tertuliano Franco Santos Franco Salvador - BA
Rodrigo Roque Dias São Paulo - SP
Lucas de Melo Pontes e Silva São Paulo - SP
Thiago Afonso de André São Paulo - SP
Sergio Alvarez Araujo Correia Fortaleza - CE
Daniel Nobuo Uno São Paulo - SP
Evandro Makiyama de Melo São Paulo - SP
Leonardo Augusto Zão Nilópolis - RJ
Bruno Fernandes Cerqueira Leite São Paulo - SP
Daniel Mourão Martins Rio de Janeiro - RJ
Daniele Véras de Andrade Rio de Janeiro - RJ
Lucas Heitzmann Gabrielli São Paulo - SP
Diogo Diniz P.S. Silva Campina Grande - PB
Diêgo Veloso Uchôa Teresina - PI
Marcelo Handro Maia São José dos Campos - SP
MENÇÃO HONROSA
Gilberto Kirk Rodrigues Rio de Janeiro - RJ
Diogo Luiz Duarte Rio de Janeiro - RJ
Camilo Marcantonio Junior Rio de Janeiro - RJ
Marcio Miranda de Carvalho Teresina - PI
Marcio Paiva Reis Vitória - ES
Arnaldo João do Nascimento Junior Duque de Caxias - RJ

9HMD D OLVWD GH FRRUGHQDGRUHV UHJLRQDLV QD QRVVD


SiJLQD QD LQWHUQHW
ZZZREPRUJEUFRRUGUHJKWP

EUREKA! N°16, 2003

61
Sociedade Brasileira de Matemática

AGENDA OLÍMPICA
XXV OLIMPÍADA BRASILEIRA DE MATEMÁTICA

NÍVEIS 1, 2 e 3
Primeira Fase – Sábado, 7 de junho de 2003
Segunda Fase – Sábado, 13 de setembro de 2003
Terceira Fase – Sábado, 18 de outubro de 2003 (níveis 1, 2 e 3)
Domingo, 19 de outubro de 2003 (níveis 2 e 3 - segundo dia de prova).

NÍVEL UNIVERSITÁRIO
Primeira Fase – Sábado, 13 de setembro de 2003
Segunda Fase – Sábado, 18 e Domingo, 19 de outubro de 2003

IX OLIMPÍADA DE MAIO
10 de maio de 2003

XIV OLIMPÍADA DE MATEMÁTICA DO CONE SUL
23 a 30 de maio de 2003
Ica – Peru

XLIV OLIMPÍADA INTERNACIONAL DE MATEMÁTICA
07 a 19 de julho de 2003
Tóquio – Japão

X OLIMPÍADA INTERNACIONAL DE MATEMÁTICA UNIVERSITÁRIA
25 a 31 de julho de 2003
Universidade Babes-Bolyai, Cluj-Napoca, Romênia

XVIII OLIMPÍADA IBEROAMERICANA DE MATEMÁTICA
13 a 20 de setembro de 2003
Argentina

VI OLIMPÍADA IBEROAMERICANA DE MATEMÁTICA UNIVERSITÁRIA
8 de novembro de 2003

♦♦♦

EUREKA! N°16, 2003

62
CONTEÚDO

AOS LEITORES 2

IX OLIMPÍADA DE MAIO 3
Enunciados e Resultado Brasileiro

XIV OLIMPÍADA DE MATEMÁTICA DO CONE SUL 6


Enunciados e Resultado Brasileiro

XLIV OLIMPÍADA INTERNACIONAL DE MATEMÁTICA 9


Enunciados e Resultado Brasileiro

X OLIMPÍADA INTERNACIONAL DE MATEMÁTICA PARA ESTUDANTES UNIVERSITÁRIOS 11


Enunciados e Resultado Brasileiro

XVIII OLIMPÍADA IBERO-AMERICANA DE MATEMÁTICA 14


Enunciados e Resultado Brasileiro

GEOMETRIA COM CONTAS 17


Carlos Yuzo Shine

A ENUMERABILIDADE DE  x  E O CHÃO TRIANGULAR 36


José Paulo Carneiro

COMO É QUE FAZ? 41

SOLUÇÕES DE PROBLEMAS PROPOSTOS 45

PROBLEMAS PROPOSTOS 59

AGENDA OLÍMPICA 61

COORDENADORES REGIONAIS 62
Sociedade Brasileira de Matemática

AOS LEITORES

Caros leitores,

Neste número a Eureka! publica várias provas de 2003 de competições


internacionais nas quais o Brasil participa: Olimpíada de Maio, Olimpíada do
Cone Sul, Olimpíada Internacional, Olimpíada Ibero-americana e Olimpíada
Internacional Uhniversitária (IMC). Este foi o primeiro ano em que uma delegação
brasileira participa desta última competição, resultado de uma parceria da OBM
com as Universidades em que estudam os membros da equipe brasileira.
Começamos bem: obtivemos três segundos prêmios, três terceiros prêmios e duas
menções honrosas.
Neste número excepcionalmente não publicamos a seção "Olimpíadas ao redor do
mundo", que volta no próximo. Por outro lado, criamos a seção "Como é que
faz?", onde vamos resolver problemas sugeridos pelos leitores.
A idéia é que os leitores enviem problemas que não conseguem resolver, que
tenham aparecido em alguma Eureka! (por exemplo em algum artigo ou prova
publicada) ou alguma competição (ou de qualquer outra origem), que tentaremos
resolver e publicar os mais interessantes na nossa seção.

Os editores

EUREKA! N°17, 2003

2
Sociedade Brasileira de Matemática

IX OLIMPÍADA DE MAIO
Enunciados e Resultado Brasileiro

PRIMEIRO NÍVEL
Duração da prova: 3 horas

PROBLEMA 1
Pedro escreve todos os números de quatro algarismos diferentes que podem ser
armados com dígitos a, b, c, d que cumprem as seguintes condições:
a ≠ 0 ; b = a + 2; c = b + 2 ; d = c + 2.
Calcule a soma de todos os números que Pedro escreveu.

PROBLEMA 2
O triângulo ABC é retângulo em A e R é o ponto médio da hipotenusa BC. Sobre
o cateto maior AB se marca o ponto P tal que CP = BP e sobre o segmento BP se
marca o ponto Q tal que o triângulo PQR é equilátero. Se a área do triângulo ABC
é 27, calcule a área do triângulo PQR.

PROBLEMA 3
Determine o menor número inteiro positivo que termina em 56, é múltiplo de 56 e
tem a soma de seus dígitos igual a 56.

PROBLEMA 4
Célia escolhe um número n e escreve a lista dos números naturais de 1 até n:
1, 2, 3, 4, …, n – 1, n.
Em cada passo, troca a lista: copia o primeiro número ao final e apaga os dois
primeiros.
Depois de n – 1 passos ficará escrito um único número.
Por exemplo, para n = 6 os cinco passos são:
1, 2, 3, 4, 5, 6 → 3, 4, 5, 6, 1 → 5, 6, 1, 3 → 1, 3, 5 → 5, 1 → 5
e ficará escrito o número 5.
Célia escolheu um número n entre 1000 e 3000 e depois de n – 1 passos ficou o
número 1.

EUREKA! N°17, 2003

3
Sociedade Brasileira de Matemática

Determine todos os valores de n que Célia pode ter escolhido.


Justifique porque estes valores servem e os demais não.

PROBLEMA 5
Temos um tabuleiro quadriculado 4 × 4. Definimos a separação entre duas casas
como o menor número de movimentos que deve empregar um cavalo de xadrez
para ir de uma casa a outra (utilizando movimentos do cavalo). Três casas A, B, C
formam um trio bom se as três separações entre A e B, entre A e C e entre B e C
são iguais. Determine um número de trios bons que se formam no tabuleiro.

OBSERVAÇÃO:
Em cada movimento o cavalo se desloca 2 casas em direção horizontal mais uma
casa em direção vertical ou se desloca 2 casas em direção vertical mais uma casa
em direção horizontal.

SEGUNDO NÍVEL
Duração da prova: 3 horas

PROBLEMA 1
São escolhidos quatro dígitos a, b, c, d diferentes entre si e diferentes de zero e se
escreve a lista de todos os números de quatro algarismos que se obtém trocando de
lugar os dígitos a, b, c, d.
Que dígitos deve-se escolher para que a lista tenha a maior quantidade possível de
números de quatro algarismos que sejam múltiplos de 36?

PROBLEMA 2
Seja ABCD um retângulo de lados AB = 4 e BC =3. A perpendicular à diagonal
BD traçada por A corta BD no ponto H. Chamamos de M o ponto médio de BH e
de N o ponto médio de CD. Calcule a medida do segmento MN.

PROBLEMA 3
Encontre todos os pares de números inteiros positivos (a, b) tais que 8b + 1 é
múltiplo de a e 8a + 1 é múltiplo de b.

EUREKA! N°17, 2003

4
Sociedade Brasileira de Matemática

PROBLEMA 4
Beto marcou 2003 pontos verdes no plano, de maneira que todos os triângulos
com seus três vértices verdes têm área menor que 1.
Demonstre que os 2003 pontos verdes estão contidos num triângulo T de área
menor que 4.

PROBLEMA 5
Uma formiga, que está numa aresta de um cubo de lado 8, deve realizar um
percurso pela superfície do cubo e regressar ao ponto de partida. Seu caminho
deve conter pontos interiores das seis faces do cubo e deve visitar só uma vez cada
face do cubo. Encontre o comprimento do caminho mais curto que a formiga pode
realizar e justifique porque é o caminho mais curto.

RESULTADOS

PRIMEIRO NÍVEL

Lucio Eiji Assaoka Hossaka Medalha de Ouro Curitiba - PR


Marlon Vieira de Lima Junior Medalha de Prata Fortaleza - CE
Henrique Pondé de Oliveira Pinto Medalha de Prata Salvador - BA
Guilherme Philippe Figueiredo Medalha de Bronze Fortaleza - CE
Fernanda Sá Leal de Moura Medalha de Bronze Teresina - PI
Régis Prado Barbosa Medalha de Bronze Fortaleza - CE
Tiago Madeira Medalha de Bronze Itajaí - SC
Mateus Faitanin Yin Menção Honrosa Vitória - ES
Diogo Bonfim Moraes Morant de Holanda Menção Honrosa Rio de Janeiro - RJ
Ronaldo Rozenbaum Paiva Menção Honrosa Rio de Janeiro - RJ

SEGUNDO NÍVEL

Telmo Luis Correa Junior Medalha de Ouro São Paulo - SP


Luty Rodrigues Ribeiro Medalha de Prata Fortaleza - CE
Pedro Thiago Ezequiel de Andrade Medalha de Prata Fortaleza - CE
Gabriel Tavares Bujokas Medalha de Bronze São Paulo - SP
Juliana G. Cavalcante Medalha de Bronze Fortaleza - CE
Leandro Farias Lima Medalha de Bronze Fortaleza - CE
Eduardo Fischer Medalha de Bronze Encantado - RS
André Linhares Rodrigues Menção Honrosa Fortaleza - CE
Rodrigo Viana Soares Menção Honrosa Fortaleza - CE
Raphael Rodrigues Viana Menção Honrosa Salvador - BA

EUREKA! N°17, 2003

5
Sociedade Brasileira de Matemática

XIV OLIMPÍADA DE MATEMÁTICA DO CONE SUL


Enunciados e Resultado Brasileiro

A XIV Olimpíada de Matemática do Cone Sul foi realizada na cidade de


Ica, Peru, no período de 23 a 30 de maio de 2003. A equipe brasileira foi liderada
pelos professores Paulo José Bonfim Gomes Rodrigues e Emanuel de Souza
Carneiro, ambos de Fortaleza – CE. Novamente a equipe brasileira obteve a maior
pontuação entre os países participantes.

RESULTADOS DA EQUIPE BRASILEIRA

BRA1 Fábio Dias Moreira Prata


BRA2 Henry Wei Cheng Hsu Prata
BRA3 Thiago Costa Leite Santos Prata
BRA4 Rodrigo Aguiar Pinheiro Ouro

PROBLEMA 1
Em um torneio de futebol entre quatro equipes, A, B, C e D, cada equipe joga com
cada uma das outras exatamente uma vez.

a) Decidir se é possível que, ao finalizar o torneio, as quantidades de gols


marcados e sofridos pelas equipes sejam:

A B C D
Gols marcados 1 3 6 7
Gols sofridos 4 4 4 5

Se a resposta é afirmativa, dê um exemplo com os resultados das seis partidas; em


caso contrário, justifique.

b) Decidir se é possível que, ao finalizar o torneio, as quantidades de gols


marcados e sofridos pelas equipes sejam:

A B C D
Gols marcados 1 3 6 13
Gols sofridos 4 4 4 11

EUREKA! N°17, 2003

6
Sociedade Brasileira de Matemática

Se a resposta é afirmativa, dê um exemplo com os resultados das seis partidas; em


caso contrário, justifique.

PROBLEMA 2
Considere a seqüência {an} definida da seguinte maneira:
a1 = 1
a2 = 3
an+2 = 2an+1 an + 1, para todo inteiro n ≥ 1.

Provar que a máxima potência de 2 que divide a4006 − a4005 é 22003.

PROBLEMA 3
Seja ABC um triângulo acutângulo tal que o ângulo B mede 60o . A circunferência
de diâmetro AC intersecta as bissetrizes internas de A e C nos pontos M e N
respectivamente (M ≠ A, N ≠ C). A bissetriz interna do ângulo B intersecta MN e
AC nos pontos R e S, respectivamente. Demonstrar que BR ≤ RS.

PROBLEMA 4
No triângulo acutângulo ABC, os pontos H, G e M encontram-se sobre o lado BC,
de modo que AH, AG e AM são altura, bissetriz e mediana do triângulo,
respectivamente. Sabe-se que HG = GM, AB = 10 e AC = 14. Determinar a área do
triângulo ABC.

PROBLEMA 5
Seja n = 3k + 1 , onde k é um inteiro, k ≥ 1 . Constrói-se um arranjo triangular
de lado n formado por círculos de mesmo raio como o mostrado na figura para
n = 7.

EUREKA! N°17, 2003

7
Sociedade Brasileira de Matemática

Determinar, para cada k, o maior número de círculos que podem ser coloridos de
vermelho de tal modo que não existam dois círculos vermelhos tangentes entre si.

PROBLEMA 6
Demonstrar que existe uma seqüência de inteiros positivos x1 , x2 ,..., xn ,... que
satisfaz as duas condições seguintes:
i) contém exatamente uma vez cada um dos inteiros positivos,
para cada n = 1,2,... a soma parcial x1 + x2 + ... + xn é divisível por n .
n
ii)

EUREKA! N°17, 2003

8
Sociedade Brasileira de Matemática

XLIV OLIMPÍADA INTERNACIONAL DE MATEMÁTICA


07 a 19 de julho, Tóquio - Japão

A XLIV Olimpíada Internacional de Matemática foi realizada em Tóquio, Japão,


no período de 07 a 19 de julho de 2003. A equipe brasileira foi liderada pelos
professores Nicolau Saldanha, do Rio de Janeiro – RJ e Élio Mega, de São Paulo –
SP.

RESULTADOS DA EQUIPE BRASILEIRA

BRA1 Alex Corrêa Abreu Medalha de Bronze


BRA2 Samuel Barbosa Feitosa Medalha de Bronze
BRA3 Rafael Daigo Hirama Menção Honrosa
BRA4 Larissa Cavalcante Queiroz de Lima Menção Honrosa
BRA5 Fábio Dias Moreira Medalha de Prata
BRA6 Davi Máximo Alexandrino Nogueira Medalha de Bronze

PROBLEMA 1
Seja A um subconjunto do conjunto S = {1,2, ,1000000} com exatamente 101
!
elementos. Demonstre que existem números t1 , t 2 , , t100 em S tais que os

!
conjuntos
A j = {x + t j | x ∈ A} , para j = 1, 2, ,100,
são disjuntos dois a dois.

PROBLEMA 2
Determine todos os pares de inteiros positivos (a, b) tais que
a2
2ab − b3 + 1
2

é um inteiro positivo.

PROBLEMA 3
Considere um hexágono convexo tal que para cada quaisquer dois lados opostos
verifica-se a seguinte propriedade: a distância entre os seus pontos médios é igual
a 3 2 vezes a soma dos seus comprimentos. Demonstre que todos os ângulos do
hexágono são iguais.

EUREKA! N°17, 2003

9
Sociedade Brasileira de Matemática

(Um hexágono convexo ABCDEF tem três pares de lados opostos: AB e DE ,


BC e EF , CD e FA ).

PROBLEMA 4
Seja ABCD um quadrilátero convexo cujos vértices estão sobre uma
circunferência. Sejam P , Q e R os pés das perpendiculares às retas BC , CA e
AB , respectivamente, passando por D . Demonstre que PQ = QR se e só se as
bissetrizes dos ângulos ∠ABC e ∠ADC se intersectam sobre a reta AC .

!, x
PROBLEMA 5
Sejam n um inteiro positivo e x1 , x2 , números reais tais que

n
x1 ≤ x2 ≤ ≤ xn .

(a) Demonstre que

2
 n n  2(n 2 − 1) n n
 ∑∑ i x − x j  ≤ ∑∑ ( xi − x j ) 2 .
 i =1 j =1  3 i =1 j =1


(b) Demonstre que a igualdade é válida se e só se x1 , x2 , , xn formam uma
progressão aritmética.

PROBLEMA 6
Seja p um número primo. Demonstre que existe um número primo q tal que,
p
para todo inteiro n, o número n − p não é divisível por q.

EUREKA! N°17, 2003

10
Sociedade Brasileira de Matemática

X OLIMPÍADA INTERNACIONAL DE MATEMÁTICA PARA


ESTUDANTES UNIVERSITÁRIOS
25 a 31 de Julho, Cluj - Napoca, Romênia

A X Olimpíada Internacional de Matemática para estudantes


universitários foi realizada na cidade de Cluj-Napoca, Romênia, no período de 25
a 31 de Julho de 2003. A equipe brasileira foi liderada pelo professor Luciano
Castro, do Rio de Janeiro – RJ.

RESULTADOS DA EQUIPE BRASILEIRA

Márcio Afonso Assad Cohen IME Medalha de Prata


Humberto Silva Naves ITA Medalha de Prata
Rodrigo Villard Milet UFRJ Medalha de Prata
Carlos Stein Naves de Brito ITA Medalha de Bronze
Daniel Yamamoto ITA Medalha de Bronze
Giuliano Boava UFSC Medalha de Bronze
Eduardo Famini Silva IME Menção Honrosa
Thiago Barros Rodrigues Costa UNICAMP Menção Honrosa

PRIMEIRO DIA

, a ,... uma seqüência de números reais tais que


PROBLEMA 1
a) Seja a1 , a2 , n a1 = 1 e
3
an+1 > an , ∀n.
2
an
Prove que a seqüência n −1
tem um limite finito ou tende a infinito.
3
 
2
b) Prove que para todo α > 1 existe uma seqüência a1 , a2 , , a ,... com as
n
an
mesmas propriedades, tal que lim n −1
= α.
n →∞
 3
 
 2

EUREKA! N°17, 2003

11
Sociedade Brasileira de Matemática


PROBLEMA 2
Sejam a1 , a2 , , a51 elementos não nulos de um corpo. Simultaneamente


trocamos cada elemento pela soma dos outros 50. Desta forma a nova seqüência
b1 , b2 , , b51 é uma permutação da anterior. Quais são os possíveis valores da
característica do corpo?

PROBLEMA 3
Seja A uma matriz quadrada n × n tal que 3A3 = A2 + A + I. Prove que (Ak )k ∈ 
converge a uma matriz idempotente B (i.e., a uma matriz B tal que B2 = B).

PROBLEMA 4
Determine o conjunto de todos os pares (a, b) de inteiros positivos para os quais o
conjunto dos inteiros positivos pode ser decomposto em dois conjuntos A e B tais
que a ⋅ A = b ⋅ B.

PROBLEMA 5
Sejam g :[0,1] → # uma função contínua e f n : (0,1] → # a seqüência de
1 x
x ∫0
funções definida por f 0 ( x) = g ( x) e f n+1 ( x) = f n (t )dt, ∀x ∈ (0,1], n ≥ 0.

Determine lim f n ( x) para todo x ∈ (0,1] .


n →∞

PROBLEMA 6
n −1
Seja f ( z ) = an z + an −1 z + ... + a1 z + a0 um polinômio com coeficientes
n

reais. Prove que se as raízes de f estão no semi-plano esquerdo


{z ∈  | Re( z ) < 0} então ak ak + 3 < ak +1ak + 2 para todo k = 0, 1,…, n – 3.

EUREKA! N°17, 2003

12
Sociedade Brasileira de Matemática

SEGUNDO DIA

PROBLEMA 1
Sejam A e B matrizes reais n × n tais que AB + A + B = 0. Prove que AB = BA.

PROBLEMA 2
2x sen m t
Calcule o seguinte limite: lim+ ∫x dt (m, n naturais dados).
x →0 tn
PROBLEMA 3
Seja A um subconjunto fechado de 5n e seja B o conjunto de todos os pontos b de
5n tais que existe exatamente um ponto a0 em A tal que a0 − b = inf
a∈ A
a −b .
Prove que B é denso em 5n.

PROBLEMA 4
Encontre todos os inteiros positivos n para os quais existe uma família F de
subconjuntos de três elementos de S ={1, 2, …,n} que satisfaz as seguintes
condições:
(i) para quaisquer elementos distintos a, b ∈ S existe exatamente um A ∈ F
tal que a, b ∈ A.
(ii) Se a, b, c, x, y, z são tais que {a, b, x}, {a, c, y}, {b, c, z} ∈ F então
{x, y, z} ∈ F.

PROBLEMA 5
a) Mostre que para toda função f : " × " → # existe uma função g : " → # tal
"
que f ( x, y ) ≤ g ( x) + g ( y), ∀x, y ∈ .
b) Encontre uma função f : × → # # # para a qual não existe g : # → # tal
que f(x, y) ≤ g(x) + g(y), ∀x, y ∈ 5.

PROBLEMA 6
1 n ak
Seja a0 , a1 ,..., an ,... a seqüência definida por a0 = 1 , an+1 = ∑
n + 1 k =0 n − k + 2
.

ak
Calcule ∑2
k =0
k (se existir).

EUREKA! N°17, 2003

13
Sociedade Brasileira de Matemática

XVIII OLIMPÍADA IBEROAMERICANA DE MATEMÁTICA


13 a 20 de setembro, Mar del Plata - Argentina
A XVIII Olimpíada Iberoamericana de Matemática foi realizada na cidade de Mar
del Plata, Argentina, no período de 13 a 20 de setembro de 2003. A equipe
brasileira foi liderada pelos professores Augusto C. de Oliveira Morgado, do Rio
de Janeiro – RJ e Luzinalva Miranda de Amorim, de Salvador – BA.

RESULTADOS DA EQUIPE BRASILEIRA

BRA1 Alex Corrêa Abreu Medalha de Ouro


BRA2 Davi Máximo Alexandrino Nogueira Medalha de Prata
BRA3 Fábio Dias Moreira Medalha de Ouro
BRA4 Samuel Barbosa Feitosa Medalha de Bronze

PRIMEIRO DIA

PROBLEMA 1

a) Têm-se duas sucessões, cada uma de 2003 inteiros consecutivos, e um


tabuleiro de 2 linhas e 2003 colunas

………
………

Decida se é sempre possível distribuir os números da primeira sucessão na


primeira linha e os da segunda sucessão na segunda linha, de modo que os
resultados obtidos ao somar os dois números de cada coluna formem uma nova
sucessão de 2003 números consecutivos.

b) E se trocássemos 2003 por 2004?


Tanto em a) como em b), se a resposta for afirmativa, explique como distribuiria
os números, e se for negativa, justifique o porquê.

EUREKA! N°17, 2003

14
Sociedade Brasileira de Matemática

PROBLEMA 2
Sejam C e D dois pontos da semicircunferência de diâmetro AB tais que B e C
estão em semiplanos distintos em relação à reta AD. Denotemos por M, N e P os
pontos médios de AC, DB e CD, respectivamente. Sejam OA e OB os circuncentros
dos triângulos ACP e BDP. Demonstre que as retas OA OB e MN são paralelas.

PROBLEMA 3
Pablo copia o seguinte problema:
Considere todas as sucessões de 2004 números reais ( x0 , x1 , x2 ,..., x2003 ),
tais que
x0 = 1,
0 ≤ x1 ≤ 2 x0 ,
0 ≤ x2 ≤ 2 x1 ,

0 ≤ x2003 ≤ 2 x2002 .
Entre todas estas sucessões, determine aquela para a qual a expressão seguinte
assume o seu maior valor: S = … .
Quando Pablo ia copiar a expressão S, apagaram o quadro. Só conseguia lembrar-
se de que S era da forma
S = ± x1 ± x2 ± ... ± x2002 + x2003 ,
onde o último termo, x2003 , tinha coeficiente +1, e os anteriores tinham coeficiente
+1 ou –1. Demonstre que Pablo, apesar de não ter o enunciado completo, pode
determinar com certeza a solução do problema.

SEGUNDO DIA
PROBLEMA 4
Seja M ={1, 2,…,49} o conjunto dos primeiros 49 inteiros positivos. Determine o
maior inteiro k tal que o conjunto M tenha um subconjunto de k elementos em que
não haja 6 números consecutivos. Para esse valor máximo de k, encontre a
quantidade de subconjuntos de m, de k elementos, que tenham a propriedade
mencionada.

EUREKA! N°17, 2003

15
Sociedade Brasileira de Matemática

PROBLEMA 5
No quadrado ABCD, sejam P e Q pontos pertencentes aos lados BC e CD
respectivemante, distintos dos extremos, tais que BP = CQ. Consideram-se pontos
X e Y, X ≠ Y, pertencentes aos segmentos AP e AQ respectivamente. Demonstre
que, quaisquer que sejam X e Y, existe um triângulo cujos lados têm os
comprimentos dos segmentos BX, XY e DY.

PROBLEMA 6
Definen-se as sucessões (an )n ≥0 ,(bn ) n ≥0 por:
a0 = 1, b0 = 4 e
an+1 = an2001 + bn , bn+1 = bn2001 + a n para n ≥ 0.

Demonstre que 2003 não divide nenhum dos termos destas sucessões.

EUREKA! N°17, 2003

16
Sociedade Brasileira de Matemática

GEOMETRIA COM CONTAS


Carlos Yuzo Shine, Colégio Etapa

♦ Nível Avançado

Às vezes precisamos de mais elementos para resolver problemas de


geometria. Pode-se traçar novos elementos na figura que possam ajudar ou fazer
algumas contas. Mostraremos algumas técnicas para fazer algumas contas que
ajudam (e até resolvem!).

Em geral, pode-se pensar em problemas de geometria seguindo esses passos:

(i) Faça a figura do problema (praticamente nenhum problema vem com


figura), bem grande e com certa precisão (ou seja, use a régua e o
compasso, mas não é necessário muito rigor).
(ii) Mexa um pouco com os elementos da figura. Algo que é sempre útil é
fixar um certo número de ângulos (de preferência, o menor número
possível, de modo que os ângulos marcados determinem a figura - a não
ser, é claro, que acrescentar algum outro ângulo adicione alguma simetria
algébrica útil) e calcular todos os outros ângulos possíveis (se os ângulos
que você escolheu determinam a figura, é possível calcular todos os
outros, de um jeito ou de outro). Procure quadriláteros inscritíveis para
ajudar. Se necessário, faça conjecturas (é para isso que você fez um
desenho bem feito!). Alguns problemas de geometria já são resolvidos
nesse passo!
(iii) Se o problema ainda não foi resolvido, é hora de elaborar uma estratégia
para resolver o problema, ou seja, determinar quais cálculos devem ser
feitos. Nada de fazer cálculos sem planejá-los!
(iv) Execute sua estratégia. Lembre-se sempre de ter uma meta em mente
(algo do tipo "precisamos calcular tal ângulo") e, se você estiver numa
prova, de controlar seu tempo e o tamanho da conta (não deixe a conta
crescer muito; a falta de controle é um fermento muito poderoso para
contas.)

É claro que esses passos não são precisos e que, para dominá-los, é preciso muito
treino e, por que não, aprender algumas técnicas.

EUREKA! N°17, 2003

17
Sociedade Brasileira de Matemática

TRIGONOMETRIA
Muitos problemas de geometria podem ser resolvidos com o auxílio da
trigonometria. As fórmulas que você deve saber são basicamente essas quatro:

sen(a + b) = sena cos b + senb cos a


sen(a − b) = sena cos b − senb cos a
cos(a + b) = cos a cos b − senasena
cos(a − b) = cos a cos b + senasenb

A partir dessas você pode deduzir essas outras, que na verdade são as mais úteis
para nós e que tornam a trigonometria tão poderosa.

Transformando produtos em somas Transformando somas em produtos

1  x+ y  x− y
senasenb = (cos(a − b) − cos(a + b)) senx + seny = 2sen   cos  
2  2   2 
1  x− y  x+ y
cos a cos b = (cos(a − b) + cos(a + b)) senx − seny = 2sen   cos  
2  2   2 
1 + −
sena cos b = (sen(a − b) + sen(a + b)) cos x + cos y = 2 cos  x y  cos  x y 
2  2   2 
 x+ y  x− y
cos x − cos y = −2sen   sen  
 2   2 
Por fim, relembramos a lei dos senos e a lei dos co-senos. No triângulo ABC, seja
AB = c, AC = b, BC = a, ∠A = α, ∠ B = β e ∠C = γ. O circunraio de ABC é R.

a b c
= = = 2R
senα senβ senγ
a 2 = b 2 + c 2 − 2bc cos α
b 2 = a 2 + c 2 − 2 ac cos β
c 2 = a 2 + b 2 − 2ab cos γ

A lei dos senos, por envolver proporções (que são mais simples) e elementos
adicionais do triângulo (o circunraio), é particularmente útil.
Vamos resolver alguns problemas e mostrar algumas técnicas de cálculo.

EUREKA! N°17, 2003

18
Sociedade Brasileira de Matemática

CONVENÇÃO
Sempre que houver um triângulo ABC, α, β e γ são as medidas dos ângulos
∠BAC, ∠ABC e ∠ACB, respectivamente.

UM COMEÇO E O TRUQUE DA CO-TANGENTE

Exemplo
(Prova de Seleção para a IMO) Seja Γ uma circunferência de centro O tangente
aos lados AB e AC do triângulo ABC nos pontos E e F. A reta perpendicular ao
lado BC por O intercepta EF no ponto D. Mostre que A, D e M (ponto médio de
BC) são colineares.

Resolução
Primeiro, um bom desenho, com todos os ângulos que pudermos marcar (a técnica
do arrastão é bastante útil - é por isso que você deve fazer um desenho grande!!).
Note que os ângulos do triângulo ABC já determinam os ângulos toda a figura
(para perceber isso, note que se construir ABC todos os outros ângulos da figura já
estão determinados).
A

π α
2 2
π α F
D
2 2 γ α
E β 2
α
2 O

β γ
B P M C

EUREKA! N°17, 2003

19
Sociedade Brasileira de Matemática

É sempre bom justificar os cálculos. Seja P a interseção de BC e da reta


perpendicular a BC por O. Como ∠BEO e ∠BPO são retos, o quadrilátero
BPOE é inscritível, de modo que ∠DOE = ∠EBM = β . Analogamente,
∠DOF = γ .
A reta AO é bissetriz de  e AOEF é inscritível, logo ∠OEF = ∠OFE = α / 2.
Mas, como provar que A, D e M estão alinhados? Uma maneira é provar que
∠BAD = ∠BAM , por exemplo. Para isso, é só calcular os dois ângulos.
Como calcularemos φ = ∠BAD ? Veja o triângulo ADE. Sendo r o raio de Γ, com
uma lei dos senos calculamos DE. AE pode ser facilmente calculado. Como já
conhecemos ∠AED (viu como é bom fazer o arrastão?), temos elementos
suficientes para calcular φ.
Para calcular θ = ∠BAM , usaremos o triângulo BAM, da qual conhecemos BM,
AB, e ∠ABM .
Já temos uma estratégia. Vamos executar o plano!
A

π α
2 2 D
E α β
2
r
O
No triângulo ODE,
DE r rsenβ
= ⇔ DE =
senβ  α  α
sen  β +  sen  β + 
 2  2
(note que ∠ODE = π − ( β + α / 2) - utilizamos o fato de que
senx = sen(π − x) para todo x real; utilizaremos bastante esse fato e o fato
sen(π / 2 − x) = cos x)
Sendo o triângulo AEO retângulo em E, obtemos AE = rcotg(α / 2).

EUREKA! N°17, 2003

20
Sociedade Brasileira de Matemática

No triângulo ADE,
DE AE
= (*)
senφ  α
cos  φ + 
 2
Quando temos uma equação do tipo
a b
= ,
senx sen ( x + δ )
e queremos determinar x, utilizamos o truque da co-tangente:
a b sen( x + δ ) b senx cos δ + senδ cos x
= ⇔ = ⇔ =
senx sen( x + δ ) senx a senx
b b
= ⇔ cos δ + senδ cotgx =
a a
e podemos isolar cotg x.
Voltemos a (*). Substituindo DE e AE e utilizando o truque da co-tangente, temos
α   α 
cotg  sen  β + 
α  α   2  2
cos   cotgφ + sen   = ⇔
 2  2 senβ
α   α α 
cos   sen  β +  − sen 2   senβ
⇔ cotgφ = 2  2 2
α  α 
sen   senβ cos  
2 2
 α  α  α 
2sen  β +  cos   − 2sen2   senβ
⇔ cotgφ =  2 2 2
senαsenβ
α 
sen(α + β ) + senβ − 2sen2   senβ
⇔ cotgφ = 2
senαsenβ
 α 
sen(α + β ) + senβ 1 − 2sen 2   
  2 
⇔ cotgφ =
senαsenβ

EUREKA! N°17, 2003

21
Sociedade Brasileira de Matemática

sen(α + β ) + senβ cosα


⇔ cotgφ =
senαsenβ

Calculemos θ. Uma prática normal em trigonometria é adotar o circunraio de


algum triângulo igual a 1/2, de modo que, pela lei dos senos, seus lados sejam
iguais aos senos dos seus respectivos ângulos opostos. Podemos fazer isso porque
estamos só fixando o tamanho da figura. É claro que só podemos fazer isso uma
vez só em cada problema.
A

sen γ

B sen α M
2

1
Nesse caso, façamos isso com ∆ABC. Temos BM = BC / 2 = senα e
2
AB = senγ = sen(α + β ). No triângulo ABM,
BM AB 2sen(α + β )
= ⇔ senβ cotgθ + cos β = ⇔ cotgθ =
senθ sen(θ + β ) senα
2sen(α + β ) − senα cos β
=
senαsenβ
Puxa, os resultados de cotgφ e cotgθ são diferentes! Na verdade, não são. Nunca
perca a fé!

EUREKA! N°17, 2003

22
Sociedade Brasileira de Matemática

cotgφ = cotgθ ⇔ sen(α + β ) + senβ cos α = 2sen(α + β ) − senα cos β


⇔ sen(α + β ) = senα cos β + senβ cos α,
que é sempre verdade.

ALGUMAS IDENTIDADES
Suponha que o circunraio do triângulo ABC é R = 1/2. Então, c = AB = senγ, b =
AC = senβ e a = BC = senα.
Além disso, por exemplo,

α β γ
• O perímetro do triângulo é 2 p = 4cos   cos   cos   ;
     2
2 2
• A área do triângulo é S = senα senβ senγ / 2;
• O inraio do triângulo é r = 2sen  α  sen  β  sen  γ  ;
 2   2  2
• cos α + cos β + cos γ = 1 + r / R;
• α   β   γ 
p − a = 2cos   sen   sen   .
2  2  2

Exercício: Prove todas as identidades acima.

Exemplo:
(IMO) Sejam AH1, BH2 e CH3 as alturas de um triângulo acutângulo ABC. A
circunferência inscrita no triângulo ABC é tangente aos lados BC, CA, AB em T1,
T2 e T3, respectivamente. Considere a reta simétrica da reta H1H2 relativamente à
reta T1T2, a reta simétrica da reta H2H3 relativamente à reta T2T3, a reta simétrica
da reta H1H3 relativamente à reta T1T3. Prove que estas retas simétricas
determinam um triângulo cujos vértices pertencem à circunferência inscrita no
triângulo ABC.

Resolução:
Esse é o problema 6 da IMO de 2001.
Primeiro, uma boa, e bem grande, figura. Vamos só desenhar a reta simétrica
relacionada a T2T3. H é o ortocentro de ABC.

EUREKA! N°17, 2003

23
Sociedade Brasileira de Matemática

A
π
−β
2
π
−γ
2

β H2
γ π
−β π α
− T2
H3 2 2 2
π α
− −γ T3
2 2
)1
P
π α π–α – γ =β X3
− −γ
2 2 H

β β γ
B H1 C

Façamos o arrastão: veja que AH 2 HH 3 é inscritível, logo ∠AH3 H2 = γ . Seja P a


interseção de T2T3 e H2 H3 (só não podemos escolher duas retas TT
i j e Hi H j
concorrentes quando o triângulo ABC é equilátero; tal caso é trivial). Como
AT2 = AT3 , os ângulos ∠AT2T3 e ∠AT3T2 medem ambos π / 2 − α / 2. Assim,
∠H3 PT3 = ∠AT3T2 − ∠PH3T3 = π − α / 2 − γ e, sendo l1 a reta simétrica da reta
H2 H3 relativamente à reta T2T3 , o ângulo entre l1 e T2T3 é igual também a
π − α / 2 − γ . Logo o ângulo entre l1 e AB é 2(π / 2 −α / 2 − γ ) + γ = π −α − γ = β ,
ou seja, l1 e BC são paralelos.
Definindo analogamente l2 e l3 , temos l2 // AC e l3 // AB.
Com isso, já sabemos que o triângulo determinado por l1 , l2 e l3 é semelhante a
ABC, e com lados homólogos paralelos. Temos, então, dois candidatos a tal
triângulo:

EUREKA! N°17, 2003

24
Sociedade Brasileira de Matemática

A
A

A'

C' B'

B' C'

B C B A' C

Estudando um caso particular (o triângulo equilátero, por exemplo), vemos que o


candidato mais indicado é o da direita. Podemos, então calcular a distância entre
lados homólogos nessa situação e compararmos com a distância entre BC e l1.
Assuma que o circunraio de ABC é 1/2, para termos BC = senα , CA = senβ e
BC = senα.
Vamos calcular a distância entre BC e l1 . Seja X 3 a interseção de l1 e AB. A
distância de A a l1 é AX 3senβ . E a distância desejada é AH1 − AX 3senβ . Bom,
AH1 é fácil de calcular: AH1 = ABsenβ = senγ senβ . E AX 3 ? AH3 é fácil de
calcular, AT3 também. Podemos calcular H3T3 = AT3 − AH3 e usar a lei dos senos
no triângulo PH3 X 3 , com a ceviana PT3 . Mãos à obra!!
Para começar, AH3 = AC cos α = senβ cos α e AT3 = p − senα , sendo p o
semiperímetro de ABC. Portanto H3T3 = p − senα − senβ cos α.
Pela lei dos senos no triângulo PH3T3 ,
PT3 H 3T3
=
senγ sen∠H3 PT3
No triângulo PT3 X 3 ,
PT3 X 3T3
=
senβ sen∠X 3PT3
Dividindo as duas últimas equações e tendo em vista que ∠H3 PT3 = ∠X 3 PT3 ,
senγ psenγ − senγ senα − senγ senβ cosα
obtemos X 3T3 = H3T3 =
senβ senβ

EUREKA! N°17, 2003

25
Sociedade Brasileira de Matemática

Da lei dos co-senos no ∆ABC (ela também é útil de vez em quando!),


sen 2 β + sen 2γ − sen2α
senγ senβ cos α =
2
senα + senβ + senγ
Logo, substituindo p = ,
2
−senαsenγ + senβ senγ + sen 2γ − sen 2 β − sen 2γ + sen 2α
X 3T3 =
2senβ
−senαsenγ + senβ senγ − sen 2 β + sen 2α
=
2senβ
Enfim, podemos calcular AX 3 = AT3 + X 3T3 . Veja que
−senα + senβ + senγ
AT3 = p − senα = .
2
senβ (−senα + senβ + senγ ) − senαsenγ + senβ senγ − sen 2 β + sen 2α
AX 3 =
2senβ
−senαsenβ + 2senβ senγ − senαsenγ + sen 2α
=
2senβ
Enfim, a distância entre l1 e BC é
−senαsenβ + 2senβ senγ − senαsenλ + sen 2α
AH1 − AX 3senβ = senβ senγ −
2
senα (−senα + senβ + senγ )
=
2
Na seção de identidades, você deve provar que
α   β   γ 
p − a = 2cos   sen   sen  
 2  2   2
Logo a distância entre l1 e BC é (ufa!)
α   β   γ 
d = 2cos   sen   sen   senα
 2  2   2

Agora calculemos a distância entre os lados homólogos dos triângulos ABC e o de


lados respectivamente paralelos aos lados de ABC.

EUREKA! N°17, 2003

26
Sociedade Brasileira de Matemática

C' B'
α
r
I
r
α

B A' C

Seja I o incentro do triângulo ABC. A distância de I a BC é igual ao inraio r e a


distância de I a B'C' é r cosα.
Assim, a distância entre BC e B'C' é:
α 
d ' = r + r cos α = r (1 + cos α ) = 2r cos 2  
2
Você tem outra identidade para provar:

α   β   γ 
r = 2sen   sen   sen  
 2  2  2
Logo
α   β   γ  α 
d ' = 2 ⋅ 2sen   sen   sen   cos 2  
 2  2   2 2
α  α   β   γ  α 
= 2 ⋅ 2sen   cos   ⋅ sen   sen   cos  
2 2  2   2  2
α   β   γ 
= 2cos   sen   sen   senα = d
 2  2   2

EUREKA! N°17, 2003

27
Sociedade Brasileira de Matemática

Conseqüentemente, l1 contém B' C'. Analogamente (ou você acha que eu faria
todas as contas de novo?), l2 contém A' C' e l3 contém A'B'.

Às vezes traçar novos elementos na figura também ajuda.

Exemplo
(IMO) Seja P um ponto interior ao triângulo ABC tal que
∠APC − ∠ABC = ∠APB − ∠ACB
Sejam D e E os incentros dos triângulos APB e APC, respectivamente. Prove que
as retas BD, CE e AP passam por um ponto em comum.

Resolução
Seja θ = ∠APC − ∠ABC = ∠APB − ∠ACB.
A

α1 α2

γ+θ
β+θ
P

B C

Veja que podemos "separar" θ de β e γ. Note que se θ ficar "para baixo" obtemos
um quadrilátero inscritível, então faremos isso.

EUREKA! N°17, 2003

28
Sociedade Brasileira de Matemática

α1
α2

β γ
F G

α1 + γ γ β α1 + β

θ θ
P

B C
O quadrilátero AFPG é inscritível, logo ∠AFG = β , ou seja, FG // BC.
O problema pede, na verdade, para provarmos que as bissetrizes de ∠ACP e
∠ABP se encontram sobre AP. Sejam Q e R as interseções de BD e CE com AP.
Devemos ter Q = R. Do teorema das bissetrizes,
AQ AB AR AC
= e =
QP BP RP CP
Como
AQ AR AQ AR
Q = R ⇔ AQ = AR ⇔ = ⇔ = ,
AP − AQ AP − AR QP RP
é suficiente demonstrarmos que
AB AC
=
BP CP

Vamos, então, calcular BP e CP. Sendo FG paralela a BC, temos FB = k ⋅ AB.


Aplicando a lei dos senos ao triângulo BFP, temos
BP FB k ⋅ ABsen(α1 + γ ) AB senθ
= ⇔ BP = ⇔ =
sen (α1 + γ ) senθ senθ BP ksen (α1 + γ )

EUREKA! N°17, 2003

29
Sociedade Brasileira de Matemática

Analogamente,
AC senθ
=
CP ksen (α2 + β )
Como α1 + γ e α2 + β somam π , o resultado está demonstrado.

GEOMETRIA ANALÍTICA
Quando aparecem problemas com muitos ângulos retos e que envolvam só retas,
geometria analítica às vezes é indicada.

Exemplo
(IMO) No quadrilátero convexo ABCD, as diagonais AC e BD são perpendiculares
e os lados opostos AB e CD não são paralelos. Sabemos que o ponto P, onde se
intersectam as mediatrizes de AB e CD, está no interior de ABCD. Prove que
ABCD é um quadrilátero cíclico se, e somente se, os triângulos ABP e CDP têm
áreas iguais.

Resolução
Esse problema é perfeito para se resolver com geometria analítica: é muito fácil
colocar as coisas nos eixos (tome como eixos as diagonais); tudo é muito fácil de
calcular analiticamente (mediatrizes e áreas); e , por fim, a única condição que
poderia complicar, que é saber quando ABCD é cíclico, pode ser facilmente
transformada na potência da interseção das diagonais em relação ao seu
circuncírculo.
y

(0 ; b) B

C O A
(c ; 0) P (a ; 0) x

(0 ; d) D

EUREKA! N°17, 2003

30
Sociedade Brasileira de Matemática

Sejam, então, A = (a; 0), B = (0, b), C = (c, 0) e D = (0; d). O quadrilátero ABCD é
inscritível se, e somente se, OA ⋅ OC = OB ⋅ OD ⇔ ac = bd. Fácil não?

Seja P = (x; y). Como P pertence às mediatrizes de AB e CD, temos PA = PB e PC


= PD.

PA = PB ⇔ (x – a)2 + (y – 0)2 = (x – 0)2 + (y – b)2 ⇔ 2ax – a2 = 2by – b2

Analogamente, PC = PD ⇔ 2cx – c2 = 2by – b2. Resolvendo o sistema obtido,


temos

(a2 − b2 )d − (c2 − d 2 )b
x=
2ax − 2by = a 2 − b2 2(ad − bc)

2cx − 2dy = c2 − d 2 ( a 2 − b2 ) c − ( c 2 − d 2 ) a
y=
2(ad − bc)

Tudo bem com os denominadores pois, como AB e CD não são paralelos,


OA / OB ≠ OC / OD ⇔ a / b ≠ c / d ⇔ ad − bc ≠ 0 (nunca se esqueça de verificar
quando os denominadores são nulos; essa verificação às vezes faz você perceber
que tem que estudar alguns casos em separado).

A área do triângulo PAB é igual a |D|/2 em que

x y 1
D = a 0 1 = −ay − bx + ab
0 b 1

Da mesma forma, a área do triângulo PCD é igual a D ' / 2, em que

x y 1
D ' = c 0 1 = −cy − dx + cd
0 d 1

Assim, devemos ter

EUREKA! N°17, 2003

31
Sociedade Brasileira de Matemática

−ay − bx + ab = −cy − dx + cd

Seria muito bom nos livrarmos do módulo. O sinal de D depende da ordem em


que colocamos as coordenadas no determinante. Se os pontos correspondentes
estão dispostos no sentido anti-horário, D é positivo; se estão no sentido horário, é
negativo. Como P pertence ao interior de ABCD, PAB e PCD têm a mesma
orientação, de modo que realmente podemos nos livrar do módulo. Logo, tirando
o módulo e substituindo x e y, temos que as áreas de PAB e PCD são iguais se, e
somente se,

(a2 − b2 )c − (c2 − d 2 )a (a2 − b2 )d − (c2 − d 2 )b


(a − c) + (b − d ) = ab − cd (**)
2(ad − bc) 2(ad − bc)

Nada de abrir tudo com pressa! Queremos ac = bd, e isso significa que
provavelmente em algum momento fatoraremos a equação com ac – bd como um
dos fatores.

(**)
⇔ (a2 − b2 )(ac + bd − c2 − d 2 ) + (c2 − d 2 )(ac + bd − a2 − b2 ) = 2(ab − cd )(ad − bc)
⇔ (ac + bd )(a2 + c2 − b2 − d 2 ) − a2c 2 − a2 d 2 + b2c2 + b2 d 2 − a 2c2 − b2c2 + a2 d 2 + b2 d 2
= 2(a2bd − ab2c − acd 2 + bc2d )

⇔ ac(a2 + c2 ) − bd (b2 + d 2 ) − acb2 − acd 2 + bda2 + bdc2 − 2(a2c2 − b2d 2 )

= 2(a2bd − ab2c − acd 2 + bc2d )

⇔ ac(a2 + c2 ) − bd (b2 + d 2 ) − (−acb2 − acd 2 + bda2 + bdc2 ) − 2(a2c2 − b2 d 2 ) = 0

⇔ ac(a2 + c2 ) − bd (b2 + d 2 ) + ac(b2 + d 2 ) − bd (a2 + c2 ) − 2(ac − bd )(ac + bd ) = 0


⇔ (ac − bd )((a − c)2 + (b − d )2 ) = 0

⇔ ac = bd ou (a = c e b = d )

Não é possível termos a = c e b = d pois já vimos que ad ≠ bc . Logo as áreas de


PAB e PCD são iguais se, e somente se, ac = bd.

EUREKA! N°17, 2003

32
Sociedade Brasileira de Matemática

A geometria analítica tem uma pequena desvantagem: não passa de aplicações


extensivas do teorema de Pitágoras.

Apesar de Pitágoras resolver problemas como o que acabamos de ver, mesclar um


pouco as contas com trigonometria e números complexos pode vir a calhar.

Agora, alguns problemas para você pensar.

PROBLEMAS

01. Seja ABC um triângulo acutângulo, M o ponto médio do segmento BC, P o


ponto sobre o segmento AM tal PM = BM, H o pé da perpendicular de P a BC,
Q o ponto de interseção entre o segmento AB e a reta que passa através de H e
é perpendicular a PB e, finalmente, R o ponto de interseção entre o segmento
AC e a reta que passa através de H e é perpendicular a PC. Mostre que o
circuncírculo do triângulo QHR é tangente a BC no ponto H.

02. No triângulo ABC, AB = AC. D é um ponto sobre o lado BC tal que BD = 2


CD. Se P é o ponto de AD tal que ∠ABP = ∠PAC, prove que
2∠DPC = ∠BAC.

03. Um quadrilátero convexo está inscrito em uma circunferência de raio unitário.


Demonstre que a diferença entre seu perímetro e a soma das diagonais é maior
do que zero e menor do que 2.

04. (IMO) O prolongamento da bissetriz AL do triângulo acutângulo ABC


intercepta a circunferência circunscrita no ponto N. A partir do ponto L
traçam-se perpendiculares LK e LM aos lados AB e AC, respectivamente.
Prove que a área do triângulo ABC é igual a área do quadrilátero AKNM.

05. (Ibero) A circunferência inscrita no triângulo ABC é tangente aos lados BC,
CA e AB nos pontos D, E e F, respectivamente. AD corta a circunferência num
segundo ponto Q. Demonstrar que a reta EQ passa pelo ponto médio de AF se,
e somente se, AC = BC.

06. (IMO) Seja I o incentro do triângulo ABC. A circunferência inscrita no


triângulo ABC é tangente aos lados BC, CA e AB nos pontos K, L e M,
respectivamente. A reta que passa por B, paralela ao segmento MK, intercepta
as retas LM e LK nos pontos R e S, respectivamente. Prove que o ângulo ∠RIS
é agudo.

EUREKA! N°17, 2003

33
Sociedade Brasileira de Matemática

07. (Vietnã) Seja ABC um triângulo e A', B', C' pontos médios dos arcos BC, AC e
AB do circuncírculo de ABC, respectivamente. As retas A'B' e A'C'
interceptam o lado BC em M e N, respectivamente. Defina os pares de pontos
P, Q e R, S analogamente. Prove que MN = PQ = RS se, e somente se, ABC é
equilátero.

08. (IMO) Seja ABC um triângulo acutângulo com circuncentro O. Seja PA uma
altura do triângulo com P no lado BC.

Considere que ∠BCA ≥ ∠ABC + 30°.

Prove que ∠CAB + ∠COP < 90°.

09. (IMO) Num triângulo ABC, seja AP a bissetriz de ∠BAC com P no lado BC, e
seja BQ a bissetriz de ∠ABC com Q no lado CA.

Sabemos que ∠BAC = 60° e que AB + BP = AQ + QB.

Quais são os possíveis valores dos ângulos do triângulo ABC?

10. (Coréia) Sejam R e r o circunraio e o inraio, respectivamente, do triângulo


ABC, e R' e r' o circunraio e o inraio, respectivamente, do triângulo A'B'C'.
Prove que se ∠C = ∠C ' e Rr ' = R ' r então os triângulos são semelhantes.

11. (Turquia) Sejam AC e PC a área e o perímetro, respectivamente, do


quadrilátero cíclico C. Se a área e o perímetro do quadrilátero cujos lados são
tangentes ao circuncírculo de C são AT e PT , respectivamente, prove que
2
AC  PC 
≥ 
AT  PT 

12. (EUA) Seja ABCD um trapézio isósceles com AB // CD. O incírculo do


triângulo BCD toca CD em E. Seja F um ponto da bissetriz de ∠DAC tal que
EF ⊥ CD. O circuncírculo do triângulo ACF corta a reta CD em C e G.

Mostre que o triângulo AFG é isósceles.

EUREKA! N°17, 2003

34
Sociedade Brasileira de Matemática

13. (Balcânica, adaptado) Seja ABC um triângulo acutângulo e M, N e P as


projeções ortogonais do baricentro de ABC sobre seus lados. Prove que

2 [MNP] 1
< ≤
9 [ ABC ] 4

([XYZ] é a área do triângulo XYZ)

14. (Ibero) Dados dois círculos ω1 e ω2 , dizemos que ω1 bissecta ω2 quando se


intersectam e a corda comum é um diâmetro de ω2 . Se ω1 e ω2 são idênticas,
dizemos que ω1 e ω2 bissectam-se mutuamente. Considere dois círculos fixos
e não concêntricos ω1 e ω2 .

(a) Mostre que há infinitos círculos ω que bissectam tanto ω1 como ω2 .

(b) Encontre o lugar geométrico do centro de ω .

15. (Ibero) Seja ABC um triângulo acutângulo com circuncírculo ω centrado em


O. Seja AD, BE e CF as alturas de ABC. A reta EF corta ω em P e Q.

(a) Prove que AO ⊥ PQ.

(b) Se M é o ponto médio de BC, prove que AP2 = 2 AD ⋅ OM

16. (São Petersburgo) Seja AL uma bissetriz interna do triângulo ABC, com L
sobre BC. As retas paralelas l1 e l2 passam por B e C, respectivamente, e são
equidistantes de A. Os postos M e N pertencem a l1 e l2 , respectivamente, e
são tais que os pontos médios de LM e LN pertencem a AB e AC,
respectivamente. Prove que LM = LN.

17. (IMO) No plano, considere uma circunferência C, uma reta L tangente à


circunferência e M um ponto da reta L. Encontre o lugar geométrico dos pontos P
com a seguinte propriedade: existem dois pontos Q, R da reta L tais que M é o
ponto médio de QR e C é a circunferência inscrita no triângulo PQR.

EUREKA! N°17, 2003

35
Sociedade Brasileira de Matemática

 
A ENUMERABILIDADE DE × E O CHÃO TRIANGULAR
José Paulo Carneiro

♦ Nível Intermediário

Uma das maneiras mais conhecidas de mostrar que o conjunto × é ``


`` ` `
enumerável, isto é, que existe uma bijeção entre × e , (onde = {0; 1; 2;
``
…} é o conjunto dos números naturais) é exibir uma bijeção de × sobre , `
inspirada na figura:

(0; 0) (0; 1) (0; 2) (0; 3) …


(1; 0) (1; 1) (1; 2) (1; 3) …
(2; 0) (2; 1) (2; 2) (2; 3) …
(3; 0) (3; 1) (3; 2) (3; 3) …
… … … …

a qual sugere a enumeração:

(0; 0); (1; 0); (0; 1); (2; 0); (1; 1); (0; 2); (3; 0); (2; 1); (1; 2); (0; 3);… Ou seja,
colocamos, sucessivamente, os pares (a; b) tais que a soma a + b assuma os
valores 0; 1; 2; 3; …, e dentro da cada grupamento que tenha a + b constante
(correspondente, na figura, a uma das diagonais indicadas), ordenamos os pares
pela ordem natural de sua segunda componente.
Obtém-se então a bijeção:

f: `` `
× →
(0; 0) → 0
(1; 0) → 1
(0; 1) → 2
(2; 0) → 3
(1; 1) → 4
(0; 2) → 5
………….

Onde f(x; y) é o lugar que ocupa (x; y) nesta enumeração. (Como estamos
incluindo 0 em N, é preciso começar a contar a partir do 0-ésimo lugar).
Uma questão interessante é produzir uma fórmula explícita para esta função e
utilizar esta fórmula para provar que f é realmente uma bijeção.

EUREKA! N°17, 2003

36
Sociedade Brasileira de Matemática

``
Para isto, seja (x; y) ∈ × . Observando a figura, vê-se que se (x; y) for tal que
x + y = s > 0, então o par (x; y) é precedido, pelo menos, por todos os pares (u; v)
tais que u + v = 0; 1; 2;…;s – 1. Existe um par que tem soma 0, dois que têm soma
1, e assim por diante, até s pares que têm soma s – 1, de modo que esses pares são
s ( s + 1)
em número de 1 + ... + s = . Além disto, já na sua diagonal, o par (x; y) é
2
precedido por y pares.
( x + y )( x + y + 1) ( x + y )2 + x + 3 y
Portanto, f ( x; y ) = +y= . Finalmente,
2 2
constata-se diretamente que esta fórmula também é válida se (x; y) = (0; 0).
Podemos então concluir que f é dada pela fórmula:

f: `×` → `
( x + y) 2 + x + 3 y
f ( x; y ) =
2

Pode ser útil ao leitor testar esta fórmula para pares específicos.
Imaginemos agora que seja apresentado ao leitor, sem nenhuma menção a sua
origem, o seguinte problema: provar que a função definida por

f ( x; y ) =
( x + y) 2 + x + 3 y
2
é uma bijeção de ` × ` sobre ` .
Naturalmente, o leitor iniciará tentando provar que f é injetiva, mostrando que
f(a; b) = f(c; d) implica (a; b) = (c; d). Mas da equação
(a + b)2 + a + 3b (c + d )2 + c + 3d
= não é imediato concluir que (a; b) = (c; d), como
2 2
o leitor pode experimentar.

No entanto, se lembrarmos a maneira pela qual criamos a fórmula definidora de f,


podemos raciocinar do seguinte modo. Suponhamos primeiro que a + b = c + d
(ou seja, (a; b) e (c; d) estão na mesma diagonal). Neste caso, f(a; b) = f (c; d) ⇒
(a + b)2 + a + 3b = (c + d)2 + c + 3d ⇒ a + b + 2b = c + d + 2d ⇒ 2b = 2d ⇒ b = d
e, portanto, a = c.

Por outro lado; se a + b ≠ c + d, podemos supor, sem perda de generalidade, que


a + b > c + d. Mas então (a + b)2 > (c + d )2 e, como f (a; b) = f(c; d), obtém-se
(a + b)2 – (c + d)2 = 3d + c – (3b + a) > 0, mas (a + b)2 – (c + d )2 =

EUREKA! N°17, 2003

37
Sociedade Brasileira de Matemática

(a + b – c – d)(a + b + c + d) ≥ a + b + c + d, e logo 3d + c–(3b +a) ≥ a + b + c +d,


donde 2d ≥ 2a + 4b, e portanto c + d ≥ d ≥ a + 2b ≥ a + b, absurdo.
Logo, não pode ocorrer o caso a + b > c + d, e a conclusão final é que f é injetiva.

`
Provar a sobrejetividade de f, sem apelar para a forma pela qual criamos f, não
parece fácil. O que queremos provar é que, dado n ∈ , existe um par (x; y) tal
( x + y) + x + 3 y
2
que f ( x; y ) = = n. É aconselhável que o leitor primeiro tente
2
fazê-lo, para sentir a dificuldade de calcular x e y em função de n.

Para começar por um exemplo concreto, se n = 50, quem é (x; y) = f – 1(50)? Ou


seja, calcule x e y naturais (e já sabemos, pela injetividade, que serão únicos!) tais
( x + y )2 + x + 3 y
que = 50. Se apelarmos para o esquema das diagonais, vamos
2
iniciar perguntando em que diagonal está o 50o. par. Para isto, vamos descobrir
primeiro quais números da forma 1; 1 + 2 = 3; 1 + 2 + 3 = 6, etc., precedem 50.
Calculando "no braço", vemos que 50 é precedido por 1; 3; 6; 10; 15; 21; 28; 36;
45 = 1 + 2 + 3 + 4 + 5 + 6 + 7 + 8 + 9 (o próximo já seria 55). Isto significa que
f – 1(50) é precedido pelos 45 pares (u; v) que têm somas u + v = 0; 1; 2;…;8,
sendo o último deles igual a (0; 8) = f – 1(44) (não esqueça de contar a partir do 0).
A diagonal seguinte começa com o par (9; 0) = f – 1(45). Para chegar ao 50,
precisamos andar mais cinco pares sobre esta diagonal: (8; 1); (7; 2); (6; 3); (5; 4);
(4; 5), chegando até o (4; 5) = f – 1(50). Este par é o 50o. da seqüência (nunca
esquecendo de contar a partir do 0) e, portanto, é o único que satisfaz a f(x; y) =
50.
Para abordar o caso geral, vamos primeiro lembrar que os números T1 = 1; T2 = 1
k (k + 1)
+ 2 = 3, T3 = 1 + 2 + 3,…, Tk = 1 + ... + k = , são chamados números
2
triangulares (ver [1]). Dado n (vamos considerar somente n > 0, já que f – 1(0) =
(0; 0) é imediato), para determinar (x; y) = f – 1(n) pelo método usado acima, é
necessário primeiro determinar o maior número triangular Tk tal que Tk ≤ n , que
vamos chamar de chão triangular de n. Por exemplo, como vimos, o chão
triangular de 50 é 45.
A existência do chão triangular de n, no caso geral, decorre do seguinte: a
seqüência T1; …; Tm ;… é ilimitada superiormente; logo, algum Tm é maior do
que n; pelo Princípio da Boa Ordenação dos números naturais (ver [2]), seja Tj o
menor dos Tm que são maiores do que n; então Tj–1 é o chão triangular de n; de
fato, T j −1 ≤ n < T j .

EUREKA! N°17, 2003

38
Sociedade Brasileira de Matemática

Para mostrar a unicidade do chão triangular de n, suponha (por absurdo) que


Tp ≤ n < Tp +1 e Tq ≤ n < Tq +1 com p < q (sem perda de generalidade). Como a
seqüência (Tm) é estritamente crescente, n < Tp +1 ≤ Tq ≤ n, absurdo.

Uma vez então determinado Tk, o chão triangular de n, o par (x; y) = f – 1(n) estará
na diagonal de soma k, formada pelos pares (k; 0); (k – 1; 1);…;(0; k). Nesta
diagonal, f(k; 0) = Tk; f(k – 1; 1) = Tk + 1;…; f(0; k) = Tk + k = Tk + 1 – 1, ou seja, de
um modo geral: f(k – j; j) = Tk + j. Quando j = n – Tk, termos
f (k − n + Tk ; n − Tk ) = n . Logo: (x; y) = f −1 (n) = (k − n + Tk ; n − Tk ). Por exemplo:
−1
f (50) = (9 − 50 + 45;50 − 45) = (4;5).

Como conseqüência deste raciocínio, fica claro que f é injetiva e sobrejetiva e que
se pode determinar de modo único f – 1(n) para cada n pela fórmula:

 0; se n=0
f −1 (n) = 
(k − n + Tk ; n − Tk ); se n>0

onde Tk é o chão triangular de n.


Mas permanece um problema mais sutil; existe alguma fórmula fechada que
calcule k tal que Tk seja o chão triangular de um dado n?
x(x +1)
Para responder a isto, observe que a solução positiva de =n é
2
8n +1 −1 x(x +1)
x0 = e que a função f (x) = é crescente em [0; +∞[. Sendo, pois
2 2
k =  x0  (onde  x0  simboliza o chão inteiro de x0 , ou a parte inteira de x0 ),
k (k +1) x0 (x0 +1) (k +1)(k + 2)
temos k ≤ x0 < k +1 e, portanto: Tk = ≤ =n< = Tk +1.
2 2 2

EUREKA! N°17, 2003

39
Sociedade Brasileira de Matemática

Conclusão: uma expressão explícita para k tal que Tk seja o chão triangular de n
é:

 8n +1 −1
k = .
 2 
 401 −1
Por exemplo, para n = 50, temos: k =   = 9,51... = 9.
 2 

Referências:

[1] Carneiro, J.P.; Contar de duas maneiras, para generalizar; Eureka! No. 6; pp. 15-17.
[2] Carneiro, J.P; O Princípio da Descida Infinita de Fermat; Revista do Professor de Matemática,
No. 32; 3o. quadrimestre de 1996; pp. 39-44.

EUREKA! N°17, 2003

40
Sociedade Brasileira de Matemática

COMO É QUE FAZ?

PROBLEMA 1
PROPOSTO POR ASDRUBAL SANTOS (BOTUCATU – SP)
Encontre todas as funções f : \ →\
+ + tais que:
i) f ( x) ≥ 0, ∀x ∈ \ +

ii) f ( f (x) − x) = 2x, para todo x ∈ \. +

SOLUÇÃO DE GUILHERME RODRIGUES NOGUEIRA DE SOUZA (SÃO PAULO - SP)


Seja a função g : \ → \ tal que g(x) = f (x) − x, para todo x ∈ \
+ + +
Logo:
f (g(x)) = 2x
⇔ g(x) + g(g(x)) = 2x
⇔ g(g(x)) = −g(x) + 2x (I)
Seja: g 0 ( x) = x, g +1 ( x) = g ( g ( x)), para todo n ∈ .
n n
`
Logo, por (I):
g( g( g ( x))) = −g( g ( x)) + 2g ( x)
( n) ( n) ( n)

⇔ g (n+2) ( x) = −g (n+1) ( x) + 2g (n) ( x)


Seja an = g(n) (x), com x fixo.
Então chegamos na seguinte recursão:
an+2 = −an+1 + 2an
Resolvendo essa recursão chegamos em:
 g(x) + 2x  n  g(x) − x 
 ( −2)
n
an =   ⋅1 − 
 3   3 
g(x) + 2x  g(x) − x 
 ( −2)
n
⇔ g(n) (x) = −
3  3 
Para n suficientemente grande temos
g(x) + 2x n g(x) − x
<2 , desde que g( x) ≠ x.
3 3
Suponha, por absurdo que g(x) > x para algum x ∈ \ +

EUREKA! N°17, 2003

41
Sociedade Brasileira de Matemática

Tome n par. Temos que g (n) ( x) é negativo se n é suficientemente grande, o que é


\ \
absurdo, pois g é de + → + .
Suponha agora que g( x) < x. Também por absurdo, tomando n ímpar, concluímos
que g (n) ( x) é negativo, e como já vimos antes isso é um absurdo.
Portanto, para todo x positivo, g(x) > x e g(x) < x são falsos.
Logo g(x) = x, para todo x positivo. Então f(x) – x = x ⇔ f(x) = 2x
Vamos testar f(x) = 2x.
Temos: f ( f (x) − x) = 2x ⇔ f (2x − x) = 2x ⇔ f (x) = 2x ⇔ 2x = 2x,
então a única função que satisfaz o enunciado é f (x) = 2x.

PROBLEMA 2

\ → \ tais que f ( f (x)) = x − 2,∀x ∈\.


PROPOSTO POR ASDRUBAL SANTOS (BOTUCATU – SP)
2
Mostre que não existem funções f :

SOLUÇÃO DE TELMO LUIS CORREA JÚNIOR (SÃO PAULO – SP)


Vamos determinar os possíveis valores de x para que f ( f (x)) = x:
x = x2 − 2 ⇔ x = 2 ou x = – 1.
Agora, os valores tais que f ( f ( f ( f (x)))) = x :
⇔ f ( f ( x2 − 2)) = x
⇔ ( x2 − 2)2 − 2 = x
⇔ x4 − 4x2 − x + 2 = 0
Mas 2 e –1 devem ser raízes dessa equação, pois
f ( f (x)) = x ⇒ f ( f ( f ( f (x)))) = x .
Fatorando, obtemos:
( x − 2)( x +1)( x2 + x −1) = 0
 −1+ 5  −1− 5 
⇔ (x − 2)(x +1)  x −  x −  =0
 2 
 2 
−1+ 5 −1− 5
Logo os únicos valores tais que f ( f ( f ( f (x)))) = x são 2, –1, e .
2 2
Considere os valores x, f (x), f ( f (x)) e f ( f ( f (x))).

EUREKA! N°17, 2003

42
Sociedade Brasileira de Matemática

f(x) −1+ 5
f f Para x = , f ( f ( f ( f (x)))) = x.
2
Considere os possíveis valores de f(x): como
f ( f ( f ( f ( f (x))))) = f (x), f (x) pode ser
x
f(f (x)) −1− 5 −1+ 5
apenas 2, –1, , .
f 2 2
f f(f (f (x)) −1+ 5
De fato, se f (x) = , então f (x) = x ,
2
logo f ( f (x)) = f (x). Absurdo, pois temos
2
  −1+ 5    −1+ 5  −1+ 5
f  f 
   =   − 2 ≠ .
  2   2  2

Se f(x) = 2, como f (f (2)) = 2, então f( f ( f (x))) = 2,


2 −1+ 5
f f logo f(2) = , absurdo.
2
Se f(x) = –1, de modo análogo ao caso f(x) = 2
−1 + 5 chegamos a um absurdo.
2 f (2)
−1− 5
f f Se f(x) = , os possíveis valores para f( f(x))
2
2
−1+ 5 −1− 5
são , , 2 ou –1.
2 2

−1 − 5 −1 + 5
Se f (f (x)) = , então x = f (f (x)), absurdo.
2 2
f
f
−1 − 5  −1− 5  −1− 5
 2  = 2 ,
Se f (f (x)) = , então f 
−1 + 5 2  
2 f (f (x))
f f

f (f (f (x)))

EUREKA! N°17, 2003

43
Sociedade Brasileira de Matemática

−1 + 5  −1 − 5  −1 − 5
Logo = f(f(f  )) = , absurdo.
2  2  2
−1 + 5
Se f ( f(x)) = 2, f ( f (f ( f (x)))) = 2 ⇒ 2 = , absurdo.
2
−1 + 5
Se f ( f(x)) = –1, f ( f ( f ( f (x)))) = –1 ⇒ –1 = , absurdo.
2
Como em qualquer caso f(f(x)) = x2 – 2 é um absurdo, não existe função f nessas
condições.

PROBLEMA 3
PROPOSTO POR DAVI MÁXIMO ALEXANDRINO NOGUEIRA (FORTALEZA - CE)
Prove que é possível decompor o conjunto {1, 2, 3,…,2n} em dois subconjuntos A
e B não contendo nenhuma progressão aritmética de tamanho 2n.

SOLUÇÃO:
Vamos escolher o conjunto A aleatoriamente:
Cada elemento de U = {1, 2,…,2n} é colocado em A (ou em B = U \ A) com
probabilidade 1/2, de forma independente. Agora, se fixarmos uma PA com 2n
1 1
termos, a probabilidade de ela estar contida em A ou em B é 2 ⋅ 2n
= 2n−1 . Por
2 2
outro lado, podemos estimar o número de progressões aritméticas de tamanho 2n
2n
em U da seguinte forma: temos no máximo possibilidades para a razão e
2n − 1
n 22 n
no máximo 2 possibilidades para o termo inicial, e logo temos no máximo
2n − 1
tais PA's.
Como, para cada uma dessas PA's, a probabilidade de ela estar contida em A ou
1
em B é 2 n−1
, a probabilidade de alguma dessas PA's estar contida em A ou em B
2
22 n 1 2
é, no máximo, ⋅ 2n −1 = < 1 , para todo n ≥ 2, donde há exemplos
2n − 1 2 2n − 1
(de fato a grande maioria, se n é grande) de decomposições U = A ∪ B onde nem
A nem B contêm progressões aritméticas de tamanho 2n (para n = 1 podemos
tomar o exemplo A ={1} e B ={2}).

EUREKA! N°17, 2003

44
Sociedade Brasileira de Matemática


SOLUÇÕES DE PROBLEMAS PROPOSTOS
Publicamos aqui algumas das respostas enviadas por nossos leitores.

68. Seja ABC um triângulo de lados inteiros e área racional. Prove que existem
pontos X, Y, Z com coordenadas inteiras no plano 52 tais que o triângulo XYZ é
congruente ao triângulo ABC.

SOLUÇÃO DE ZOROASTRO AZAMBUJA NETO (RIO DE JANEIRO – RJ)


Sejam a, b, c as medidas dos lados do triângulo, respectivamente opostos aos
vértices A, B e C. Podemos supor que A = (0,0) , B = (c ,0) e C = ( m, h) . Temos
que c é inteiro e m2 + h2 = b2 ,( m − c)2 + h2 = a 2 , com a e b inteiros. Daí obtemos
b2 − a2 + c2 4b 2 c 2 − (b 2 − a 2 + c 2 ) 2 ch
m= e logo h2 = 2
. Como a área de ABC é
2c 4c 2
racional, segue que h é racional, e logo 4b 2 c 2 − (b 2 − a 2 + c 2 ) 2 é um quadrado
perfeito. Isso implica que b 2 − a 2 + c 2 é par (senão 4b 2 c 2 − (b 2 − a 2 + c 2 ) 2 seria
congruente a 3 módulo 4, absurdo).
r s
Assim, m = e h= com r, s inteiros, e portanto, r 2 + s 2 = b2 c 2 (pois
c c
m 2 + h 2 = b 2 ).
Vamos agora mostrar que existem x, y , u , v ∈ ] tais que x 2 + y 2 = c 2 , u 2 +v 2 = b 2 e
( x + yi)(u + vi) = r + si. Isso resolve o problema, pois o número complexo
x y
ϕ = − i tem módulo 1, e portanto a multiplicação por α é uma rotação em \ 2
c c
( x − yi) ( r + si)
que leva A, B, e C em X = (0,0), Y = ( x, − y ) e Z = ϕ (m + hi) = ⋅ =
c c
( x − yi)( x + yi )(u + vi )
= = u + vi = (u , v ). Assim, XYZ é congruente a ABC e
c2
seus vértices têm coordenadas inteiras.
Vamos mostrar a existência dos x, y, u, v: queremos mostrar que se
N (r + si) = r 2 + s 2 = b2 c 2 , com b, c, r, s ∈ = então existem x, y, u, v ∈ = com
( x + yi )(u + vi ) = r + si, N ( x + yi) = x 2 + y 2 = c 2 e N (u + vi ) = u 2 + v 2 = b 2 .
Vamos provar isso por indução em N (r + si) (se N (r + si ) = 1 o resultado é
óbvio), usando existência e unicidade de fatoração em ][i ] = {a + bi , a , b ∈ ]}
(vejam o artigo de Guilherme Fujiwara na Eureka! No. 14).

EUREKA! N°17, 2003

45
Sociedade Brasileira de Matemática

Se c = 1, tomamos x = 1, y = 0, u = r e v = s. Se c > 1, tome α um fator


irredutível de c. Temos que α b 2 c 2 = (r + si )(r − si ), e logo α r + si ou α r − si (e,
nesse caso α r + si ). Suponha então, sem perda de generalidade, que α r + si .
r + si  r + si  ( r + si ) ( r − si )
Temos então ∈ ][i] e N  = ⋅ . Se N (α ) é um
α  α  α α
quadrado perfeito (o que nesse caso equivale a α ser o produto de um elemento de
= por um elemento de {1, i}), existem, por hipótese de indução, x , y , u , v ∈ ]
r + si 2
com  )(u +vi), x2 + y 2 = c ,u2 + v2 = b2 , e basta tomar (x + yi) = α(x + yi )
= (x + yi
α N(α)
para concluir. Caso contrário, como α | c, α | c, e logo αα | c (pois α é irredutível e
α
∉ ][i]). Assim, α 2α 2 | c 2 | b 2 c 2 = (r + si )(r − si ), e portanto αα | r + si ou
α
 r + si  N ( r + si ) 2  c2 
α 2 | r + si. No primeiro caso, N   = = b ⋅  2 
, e podemos
 αα  N (αα )  N (α ) 
r + si 2
escrever  )(u + vi) com x 2 + y 2 = c 2 , u 2 + v2 = b2 , e basta
= ( x + yi
αα N (α )
tomar x + yi = αα ( x + yi
 ). O segundo caso é análogo:
 r + si  N ( r + si) 2  c2 
N 2 = = b ⋅  2 
, e podemos escrever
 α  N (α 2 )  N (α ) 
r + si c2
= ( 
x + 
yi )( u + vi ) com 
x 2
+ 
y 2
= , u 2 + v 2 = b 2 , e basta tomar
α2 N (α ) 2
x + yi = α 2 ( x + yi
 ).

69. Sejam a e b inteiros positivos tais que a n − 1 divide b n − 1 para todo inteiro
positivo n.
Prove que existe k ∈ 1 tal que b = a k .

SOLUÇÃO DE ZOROASTRO AZAMBUJA NETO (RIO DE JANEIRO – RJ)


Suponha por absurdo que b não seja uma potência de a.
Então existe k ∈ 1 tal que a k < b < a k +1 . Consideremos a seqüência
bn − 1 ∞
∈ , ∀ n ≥ 1.
1 1 1 1
xn = n
a −1
Como
a −1 a
n
= n
+
a 2n
+ ... = ∑
j =1 a
jn
, temos

EUREKA! N°17, 2003

46
Sociedade Brasileira de Matemática

∞ ∞ n n n
bn 1 b  b   b  bn 1
xn = ∑ − ∑ = +
   2 + ...  k + − n . Note que
j =1 a
jn
j =1 a
jn
a a  a  a a
kn
( n
− 1) a −1
( b a k +1 )
n
bn 1
como kn n = e tendem a 0 quando n cresce, se definimos
a (a − 1) 1 − a− n a −1
n

n n n n
b  b  b k
b bn 1
yn =   +  2  + ... +  k  = ∑  j  , temos que xn − yn = kn n − n
 a  a  a  j =1  a  a ( a − 1) a −1
tende a 0 quando n tende a infinito. Por outro lado, como yn é uma soma de k
progressões geométricas de razões b a j , 1 ≤ j ≤ k , yn satisfaz a equação de
recorrência C 0 y n + k + C 1 y n + k − 1 + ... + C k y n = 0, ∀ n ≥ 0 , onde
 b  b   b 
C0 x k + C1 x k −1 + ... + Ck −1 x + Ck = a k ( k +1) / 2  x −  x − 2  ...  x − k 
 a  a   a 
(vejam o artigo "Equações de recorrência" na Eureka! No. 9).
Note que todos os Ci são inteiros. Note também que
+ + + = − + − + + −
C0 xn + k C1 xn + k −1 ... Ck xn C0 ( xn + k yn + k ) C1 ( xn + k −1 yn + k −1 ) ... Ck ( xn yn )
tende a 0 quando n tende a infinito, pois xn + j − yn + j tende a 0 para todo j com
0 ≤ j ≤ k (e k está fixo). Como os Ci e os xn são todos inteiros, isso mostra que
C0 xn + k + C1 xn + k −1 + ... + Ck xn = 0 para todo n grande.
n
 b  bn 1
Agora, como x n = y n +  k +1  + ( k +1) n n − n , temos
a  a ( a − 1) a − 1
  b n+k − j
k 
C 0 xn + k + C1 x n + k −1 + ... + C k xn = ∑ C j   k +1  + z n + k − j  , onde
 
j=0  a  
bm 1
zm = (k +1)m m − m .
a ( a − 1) a − 1
n+k − j n
k
 b   b   b 
Note que ∑= 0 C k  a k + 1  = P  k + 1  ⋅  k + 1  , onde
a  a 
j

 b  b   b 
P( x) = C0 x k + C1 x k −1 + ... + Ck −1 x + Ck = a k ( k +1) / 2  x −  x − 2  ...  x − k  , donde
 a  a   a 
 b 
P  k +1  ≠ 0. Por outro lado, para todo j com
a 

EUREKA! N°17, 2003

47
Sociedade Brasileira de Matemática

( b a k +1 ) −
k− j
n
 b  1
0 ≤ j ≤ k , z n+ k − j  k +1  = , que tende a 0
( )( )
n+ k − j
a  a − 1 a k − j
− a − n
b a k n

 k   b 
n
quando n tende a infinito, donde wn =  ∑ C j xn + k − j   k +1  tende a
 j =0  a 
 b 
P  k +1  ≠ 0 , o que é um absurdo, pois, como vimos antes, wn é igual a 0 para
a 
todo n grande.

71. Considere três circunferências, tangentes duas a duas. Prove que há apenas
duas circunferências tangentes às três simultaneamente, e mostre como construí-
las.

SOLUÇÃO DE ANTONIO CAMINHA MUNIZ NETO (FORTALEZA – CE)

Prova Analisemos o caso em que as três circunferências, α, β, γ digamos, são


tangentes exteriormente duas a duas (a análise dos demais casos é completamente
análoga).

A
β

Seja A o ponto de tangência das circunferências β e γ. Aplicando à


configuração mostrada na figura acima a inversão I de pólo A e módulo igual à
potência de A em relação a α, a circunferência α permanece fixa, ao passo que as
circunferências β e γ são respectivamente transformadas em retas distintas r e s,
perpendiculares à reta que une seus centros. Como inversões preservam tangência
de curvas, segue que r e s são ainda tangentes a α, de modo que a situação é
exatamente a da figura abaixo (observe que r e s podem ser construídas com régua

EUREKA! N°17, 2003

48
Sociedade Brasileira de Matemática

e compasso: elas são as tangentes a α que são perpendiculares à reta dos centros
de β e γ):

r
s

Seja δ uma circunferência qualquer que tangência α, β e γ. É claro que δ não


contém A, donde concluímos que sua inversa δ’ ainda é uma circunferência,
ademais tangente às inversas de α, β e γ, quer dizer, tangente a α, r e s. Portanto,
segue imediatamente da figura acima que só há duas possibilidades para δ’,
digamos δ’1 e δ’2. Tais circunferências podem ser facilmente construídas com
régua e compasso, basta para tanto observar que, sendo R o raio e O o centro de α,
os centros de δ’1 e δ’2 são os pontos situados sobre a paralela a r por O, distando
2R de O. Portanto, há exatamente duas circunferências tangentes simultaneamente
α, β e γ e tais circunferências são as inversas δ1 e δ2 de δ’1 e δ’2 por I (as quais
também podem ser facilmente construídas).

73. Prove que, dado um inteiro positivo n, existe uma progressão aritmética
crescente formada por n inteiros positivos cujas somas dos dígitos também
formam uma progressão aritmética crescente, mas não existe uma progressão
aritmética infinita de inteiros positivos cujas somas dos dígitos formem uma
progressão aritmética crescente.

SOLUÇÃO DE JOSÉ DE ALMEIDA PANTERA (RIO DE JANEIRO – RJ)


Vamos mostrar, por indução em k, que para quaisquer inteiros positivos k ≥ 2 e N
e todo ε > 0 existe uma progressão aritmética crescente de inteiros positivos
( x j )− N ≤ j ≤k + N tal que se s ( x j ) é a soma dos dígitos de x j então ( s ( x j ))− N ≤ j ≤1 é
constante, ( s ( x j ))1≤ j ≤k é uma progressão aritmética crescente de razão r e

EUREKA! N°17, 2003

49
Sociedade Brasileira de Matemática

s ( xk +i ) − s ( xk +i −1 ) < ε r para 1 ≤ i ≤ N . De fato, para k = 2, podemos tomar


x1 = 102 m − 10m , x2 − x1 = 10m − 1, onde m é grande o suficiente para que m ≥ N (e
1
logo 10 m > N ) e < ε.
m
De fato, s ( xi ) = 9m , para 2 − 10 m ≤ i ≤ 1 [pois, para 1 ≤ a ≤ 10 m , a (10 m − 1) =
(a −1) ⋅10m +10m −1− (a −1), donde s (a ⋅ (10m − 1)) = s (a − 1) + 9m − s (a − 1) = 9m) ,
s (a ⋅ (10m − 1)) = s ( x2 ) = 18m e, para j > 2, digamos j = 2 + a,
x j = a ⋅10m + 102 m − 1 − a = 10 2 m + ( a − 1) ⋅10m + 10m − 1 − a , e, como
1 + a ≤ N < 10 m , s( x j +1 ) − s( x j ) = 1 + s(a) + 9m − s(a + 1) − (1 + s(a − 1) + 9m − s(a)) =
= 2s(a) − s(a − 1) − s(a + 1) < 9 m < ε ⋅ (9m).
Vamos agora conseguir uma tal progressão aritmética com k = 1 elementos. Para
 ε
isso, seja ( x j )− N −1≤ j ≤k + N +1 uma progressão como acima associada a  k , N + 1,  .
 3
1 ε
Podemos supor que N ≥ k e < . Seja A um inteiro tal que 10 ) > x k + N + 1 .
N 2
Considere a seqüência ( y j ) − N ≤ j ≤ k + N +1 , y j = x j + 10 ) ⋅ t j − k + 1 , onde
r − ( s ( xk +1 ) − s ( xk ))
t s = 10 2 m − 1 + s (10 m − 1) e m= ≥ (1 − ε ) N , onde
9
r = s( x2 ) − s( x1 ) (note que m é inteiro pois s ( x j ) ≡ x j (mod 9) para todo j).
Assim, s ( y j ) = s ( x j ) + s (t j − k −1 ). Temos então ( s ( y j )) − N ≤ j ≤1 constante e
s ( y j +1 ) − s ( y j ) = 9m , para 1 ≤ j ≤ k . De fato, para 1 ≤ j ≤ k − 1, isso segue do fato
análogo para ( x j )1≤ j ≤k , e de ( s (t j − k −1 ))1≤ j ≤k ser constante, e para j = k,
s (t j − k ) − s (t j − k −1 ) = 9m = r − ( xk +1 − xk ) , donde s ( yk +1 ) − s ( yk ) = r = s ( x2 ) − s ( x1 ). O
fato de termos s ( yk +1+i ) − s ( yk +i ) < ε r para 1 ≤ i ≤ N segue dos fatos análogos para
s ( xk +i +1 ) e s ( yi ).
Suponha agora que exista uma progressão aritmética infinita de inteiros positivos
( x j ) j ≥1 cujas somas dos dígitos ( s ( x j ) j ≥1 ) formam uma progressão aritmética
crescente. Tome um inteiro positivo k tal que 10 k > x2 > x1 , e note que
s ( x10k +1 ) = s ( x1 + 10k ( x2 − x1 )) = s ( x1 ) + s ( x2 − x1 ) = s ( x1 + 10 k +1 ( x2 − x1 )) = s ( x10k +1 +1 ) ,
absurdo.

EUREKA! N°17, 2003

50
Sociedade Brasileira de Matemática

74. Ache todas as funções f : # → # tais que:


f ( x + y ) + f ( x − y ) = 2 f ( x) ⋅ cos y, ∀x, y ∈ #.

SOLUÇÃO DE MARCÍLIO MIRANDA DE CARVALHO (TERESINA – PI)


π π
Faça y = e x = + t ⇒ f (π + t ) + f (t ) = 0 (I)
2 2
Faça x = 0 e y = t ⇒ f(t) + f(–t) = 2a cos t, onde a = f(0). (II)
π π π 
Faça x = e y = + t ⇒ f (π + t ) + f (−t ) = 2b cos  + t  = −2bsent , (III)
2 2 2 
π 
onde b = f   .
2
Somando (I) com (II) e subtraindo de (III) chegamos a f(t) = a cost + b sent,
∀t ∈ #. É fácil verificar, apenas substituindo, que todas as funções f(t) desta
forma funcionam.

75. Seja Tn um triângulo retângulo cujos lados medem (4 ⋅ n 2 , 4 ⋅ n 4 − 1, 4 ⋅ n 4 + 1) ,


onde n é um número inteiro positivo. Seja α n a medida do ângulo oposto ao lado
de medida 4 ⋅ n 2 . Mostre que, se n varia dentro dos inteiros positivos,
α 1 + α 2 + α 3 + ... = 90 0 .

SOLUÇÃO DE ANTONIO CAMINHA MUNIZ NETO (FORTALEZA – CE)


Seja Tn um triângulo retângulo cujos lados medem 4n2, 4n4 – 1 e 4n4 + 1, onde n é
um número inteiro positivo. Seja αn a medida, em graus, do ângulo oposto ao lado
de medida 4n2. Mostre que α1 + α2 + α3 + .... = 90°.

4n 2
Solução: Sabemos que tg αn = . Portanto, sendo αn = 2βn temos
4n 4 − 1
2tgβ n 4n 2
= .
1 − tg 2 β n 4n 4 − 1
1
Resolvendo a equação acima para tg βn obtemos tg βn = para todo n ≥ 1.
2n 2
Assim, basta provar que β1 + β2 + β3 + .... = 45°. Sendo bn = tg (β1 + β2 + ... + βn)
para n ≥ 1, temos então de provar que bn → 1 quando n → + ∞.

EUREKA! N°17, 2003

51
Sociedade Brasileira de Matemática

1
Temos b1 = tg β1 = e, pela fórmula para a tangente da soma,
2
bn + tg β n +1 bn + 2(n +1)2 2(n + 1) 2 bn + 1
1

bn +1 = = = .
1 − bn tg β n +1 1 − 2( nb+n 1)2 2( n + 1) 2 − bn
k
Provemos, por indução sobre k ≥ 1, que (*) bk = , o que terminará a
k +1
demonstração. A relação (*) é trivialmente verdadeira para k = 1. Suponha que já
provamos que ela é verdadeira para 1 ≤ k ≤ n, onde n ≥ 1 é um inteiro. Então
2(n + 1) 2 (nn+1 ) + 1 [2(n + 1)n + 1](n + 1)
bn+1 = =
2(n + 1) − (
2 n
n +1 ) 2(n + 1) 3 − n
(2n 2 + 2n + 1)(n + 1) n + 1
= = .
(2n 2 + 2n + 1)(n + 2) n + 2

76. Mostre que um polígono qualquer pode ser recortado e os recortes


reorganizados, sem superposição, de tal jeito que formem um quadrado.

SOLUÇÃO DE ANDERSON TORRES (SÃO PAULO – SP)


Este problema é tão legal que dá para generalizar! Vamos demonstrar que se dois
polígonos têm mesma área podemos fatiar um deles e reorganizar as fatias de
modo a produzir um polígono congruente ao segundo.
Lema 1: Um retângulo pode ser convertido em um novo retângulo do qual
conhecemos um dos lados (por exemplo, qualquer retângulo de área x pode ser
decomposto para formar um retângulo em que um dos lados mede 1 (e o outro
medirá x neste caso).
Prova: Basta exibir uma maneira de fazer isto. Veja este exemplo:
D C

1 K H N G
I
3
3'

2 2'
J
M
1'
A B E L F

EUREKA! N°17, 2003

52
Sociedade Brasileira de Matemática

1) Faça com que as bases dos retângulos estejam alinhadas. Prolongue o lado HG
até cortar o outro retângulo no ponto I.

2) Trace DJ//IB. Seja KI //AB e K ∈ DJ. Agora é só deslizar o triângulo DCJ até
DC se alinhar com IK. E depois encaixa DIK no espaço abaixo, em MLF.
E acabou!

Esta construção só vale se 2.HE ≥ DI. Caso não seja possível, basta fatiar da
maneira mais primitiva: fatia em partes iguais até cair no caso anterior:

Neste caso nem precisou!

Agora é só escrever. Dado um polígono qualquer, recortamos o desdito em


triângulos quaisquer. Cada um deles pode ser fatiado para virar um paralelogramo
(parte pela base média) e cada paralelogramo pode ser fatiado nun retângulo, é só
cortar em uma altura. Podemos transformar cada um destes retângulos em fatias
de lado 1, empilhá-los pelo lado comum e transformar isto tudo num quadrado
(oras, um quadrado é um retângulo equilátero) E fim!

77. Prove que as distâncias entre um ponto sobre uma circunferência e os quatro
vértices de um quadrado nesta inscrita não podem ser todos números racionais.

SOLUÇÃO DE CLAUDIO BUFFARA (SÃO PAULO – SP)


Seja o quadrado inscrito ABCD de lado L e suponhamos, sem perda de
generalidade, que o ponto P se encontra no arco AB.
Se P coincide com B, então m(PA) = m(PC) = L e m(PD) = L ⋅ 2 . Assim, se L é
racional, então m(PD) é irracional e se L é irracional, então m(PA) = m(PC) é
irracional. De forma análoga pode-se provar que se P coincide com A, então
m(PB), m(PD) ou m(PD) será irracional.
Suponhamos agora que P não coincide com A nem com B e que m(PA), m(PB),
m(PC) e m(PD) sejam todos racionais.
O quadrilátero PBCD é inscritível. Assim, pelo Teorema de Ptolomeu, teremos:
m(PD) ⋅ m(BC) + m(PB) ⋅ m(CD) = m(BD) ⋅ m(PC).

EUREKA! N°17, 2003

53
Sociedade Brasileira de Matemática

Mas m(BC) = m(CD) = L e m(BD) = L ⋅ 2 . Portanto:


m(PD) + m(PB) = m(PC) ⋅ 2 .
Mas:
m(PD) e m(PB) são racionais ⇒ m(PD) + m(PB) é racional e

m(PC) é racional ⇒ m(PC) ⋅ 2 é irracional

Ou seja, um número racional é igual a um número irracional.

Temos, portanto, uma contradição, a qual ocorreu em virtude da hipótese feita


inicialmente de serem m(PA), m(PB), m(PC), m(PD) todos racionais.

Conclusão: pelo menos um destes quatro segmentos tem de ter comprimento


irracional.

Observação: se tomarmos o quadrilátero inscritível PADB, o Teorema de


Ptolomeu nos dará:

m(PA)⋅ m(BD) + m(PB) ⋅ m(AD) = m(PD) ⋅ m(AB), ou seja:


m(PA) ⋅ 2 + m(PB) = m(PD), e mais uma vez cairemos na contradição de ter um
número racional igual a um número irracional.

78. Seja ABCD um quadrilátero convexo não trapézio, de diagonais AC e BD


iguais. Tomamos sobre os lados AB e CD, respectivamente, pontos P e Q tais que:

AP DQ AD
= =
PB QC BC

Mostre que os pontos P e Q são colinares com o ponto de interseção das


mediatrizes dos lados AD e BC.

SOLUÇÃO DE MARCOS FRANCISCO FERREIRA MARTINELLI (RIO DE JANEIRO – RJ)


Considere o quadrilátero ABCD AC = BD, onde O é o ponto de encontro das
diagonais e O' o ponto de encontro das mediatrizes de AD e BC.

EUREKA! N°17, 2003

54
Sociedade Brasileira de Matemática

P'
A

β O'β
D

Provarei que os quadriláteros AOO'D e BOO'C são inscritíveis (i).


Seja α = ∆ADO ˆ ' . Como DO' = O'A ⇒ ∆DAO ˆ ' =α
Observe que os ∆ACO ' e ∆BDO ' são congruentes (L.L.L), mais uma vez porque
O ' B = O ' C e, do enunciado, AC = BD.
ˆ = ODO
⇒ O ' AO ˆ ' e DBO ˆ ' ⇒ i ) está provado.
ˆ ' = ACO
Como o quadrilátero AOO'D é inscritível, DOO ˆ ' = DAO
ˆ ' =α
ˆ = 180° − O ' OA
E ainda, O ' OC ˆ = 180° − (180° − α ) = α
Como o quadrilátero BOO'C é inscritível , O ' OC ˆ = O ' BC ˆ (ii)
ˆ = α = O ' CB
De ii), observe que o ∆AO ' D  ∆O ' BC ⇒
AO ' BO ' AD AO '
= ⇒ = (iii)
AD BC BC BO '
DQ AD
Seja Q ∈ CD, = , bem como P' a interseção de QO' com AB. Se
QC BC
AP ' AD
provarmos que = , teremos P ' ≡ P.
P ' B BC
DQ AD DQ AO ' DO '
Como, por hipótese, = e de (iii), temos = = , e conclui-se
QC BC QC BO ' CO '
que O'Q é bissetriz do ∆DO ' C.

EUREKA! N°17, 2003

55
Sociedade Brasileira de Matemática

Seja DOˆ ' Q = Q ' Oˆ ' C = β ⇒ AO ' P ' = 180° − ( β + 180° − 2α ) = 2α − β e


P ' O ' B = 180° − ( β + 180° − 2α ) = 2α − β
AP ' AO ' AD
⇒ O ' P ' é bissetriz do ∆AO ' B ⇒ = = ⇒ está provado que P ' ≡ P.
P ' B O ' B BC
Portanto, P, O'e Q estão sobre uma mesma reta. (c.q.d.)

79. Temos uma fileira infinita de copos, cada um deles associado a um inteiro k, e
um número finito de pedras distribuídas de alguma maneira por esses copos. Se há
pelo menos duas pedras no copo k podemos pular uma pedra para o copo k – 1
e outra para o copo k + 1.
Prove que fazendo movimentos desse tipo um número suficientemente grande de
vezes, chega-se necessariamente a uma situação onde não é possível fazer nenhum
movimento desse tipo (i.e., onde há no máximo uma pedra em cada copo), e que a
configuração final não depende da escolha dos movimentos durante o processo.

SOLUÇÃO:
Sejam x1 ,..., xn ∈ ] as posições das n pedras. O número de pedras é sempre n, e em
um movimento trocamos duas pedras na posição k (digamos com xi = x j = k ) por
n
xi = k − 1 e x j = k + 1. Temos então que ∑x i permanece constante e
i =1
n

∑x i
2
aumenta a cada movimento, pois ( k + 1)2 + ( k − 1)2 = 2k 2 + 2 > 2k 2 . Seja
i =1
agora m o maior número de copos vazios entre dois copos ocupados. Então, se
m ≠ 0 , m não aumenta em nenhum movimento e, se m = 0, após um movimento m
passa a ser no máximo 1. Assim, a distância entre dois copos ocupados
1 n
consecutivos fica limitada, e como o centro de gravidade ∑ xi das pedras é
n i =1
n
constante, a "energia" ∑x i
2
também fica limitada, e como sempre aumenta, em
i =1
algum momento não será mais possível fazer nenhum movimento. O número de
n
movimentos é limitado por f ( x1 , ..., x n ) = A2 ∑ j 2 , onde A = m ax{2, r } e,
j =1

se x1 ≤ x 2 ≤ ... ≤ x n , r = max ( x i − x i −1 ) + 1. Para cada x 1 , . . . , x n , seja


2≤i ≤ n

g ( x1 ,..., xn ) ≤ f ( x1 ,..., xn ) o número máximo possível de movimentos a partir da


posição inicial x1 , x2 ,..., xn .

EUREKA! N°17, 2003

56
Sociedade Brasileira de Matemática

Suponha agora que exista uma posição inicial a partir da qual seja possível chegar
a duas posições finais distintas. Seja x1 ,..., xn uma tal posição com
g ( x1 ,..., xn ) mínimo. Esses dois jeitos de chegar em posições finais diferentes não
podem começar com o mesmo movimento, pois senão, após esse movimento, o
valor de g diminui, e a posição final passa a ser única pela minimalidade de
g ( x1 ,..., xn ) , absurdo.
Agora, se os movimentos iniciais das duas seqüências de movimentos que levam a
posições finais diferentes são feitos nas posições k e A , após cada um desses
movimentos o valor de g diminui e as posições finais ficam determinadas.
Por outro lado, se nos dois primeiros lances mexemos primeiro no copo k e depois
no copo A chegamos à mesma configuração que se primeiro mexermos no copo A
e depois no copo k (de fato, se inicialmente xi = x j = k e xr = xs = A , chegaremos
após esses dois lances, em qualquer ordem, em xi = k − 1, x j = k + 1, xr = A − 1 e
xs = A + 1 ) , donde as posições finais são iguais, absurdo.

1+ 5
80. Sejam α =
2
, A = { nα  , n ∈  *} e B = { nα 2
}
 , n ∈  * . Prove que

A ∩ B = ∅ e A∪ B = *.

Observação: x  é o inteiro tal que x  ≤ x < x  + 1.

SOLUÇÃO DE RODRIGO VILLARD MILET (RIO DE JANEIRO – RJ)


Temos que α² = α + 1.

1ªparte: A ∩ B = ∅
Suponha o contrário, ou seja, que existem m e n naturais tais que
[αm] = [α 2 n] = k . Daí temos que k < αm < k + 1 e k < α²n < k + 1
(a desigualdade é estrita, pois α é irracional), portanto :

m+n 1 1 m+n
< + 2 =1< ⇒ k < m + n < k +1
k +1 α α k

o que é uma contradição, já que k, m e n são naturais.

EUREKA! N°17, 2003

57
Sociedade Brasileira de Matemática

2ªparte: A ∪ B = 1*
Suponha que exista um natural h que não está nem em A nem em B. Então existem
naturais m e n tais que αm < h < h + 1 < α (m + 1) e α²n < h < h + 1< α²(n + 1).
Logo :
m+n 1 1 m+n+2
< + 2 =1< ⇒ m + n < h < h +1< m + n + 2
h α α h +1

o que é uma contradição, já que m + n + 1 é o único natural entre m + n e


m + n + 2.

Enviaram soluções de problemas anteriores os seguintes leitores da EUREKA!

Carlos Alberto da Silva Victor Nilópolis – RJ


Diêgo Veloso Uchôa Teresina – PI
Diogo Diniz Pereira da Silva e Silva Enviado via correio eletrônico
Helder Oliveira de Castro Mogi das Cruzes – SP
João Fernandes de Moura Niterói – RJ
Leno Silva Rocha Goiânia – GO
Murilo Rebouças Fernandes de Lima Goiânia – GO

EUREKA! N°17, 2003

58
Sociedade Brasileira de Matemática

PROBLEMAS PROPOSTOS

Convidamos o leitor a enviar soluções dos problemas propostos e sugestões de novos


problemas para os próximos números.

81) Num triângulo isósceles ABC com AB = BC, temos AC = BH, onde BH é a
altura relativa ao lado AC. Traçamos uma reta BD que corta o prolongamento
da reta AC em D de tal forma que os raios dos círculos inscritos nos triângulos
ABC e CBD são iguais. Determine o ângulo ABD ˆ .

82)
a) Demonstre a identidade
n
sen(2n+1α)
cos(α) ⋅ cos(2α) ⋅ cos(4α)...cos(2nα) = ∑cos(2 j α ) =
j =0 2n+1 ⋅ sen(α)

1 1 1 1 1 1 1 1 1  π  2
b) Prove que ⋅ + ⋅ + + ... = ∏cos  j+2  = .
2 2 2 2 2 2 2 2 2 j =0 2  π

83) Seja  = {0,1,2,3,...}.


Determine quantas funções f :  →  satisfazem f(2003) = 2003, f(n) ≤ 2003
para todo n ≤ 2003 e f(m + f(n)) = f(f(m)) + f(n), para todo m, n ∈ 1.

84) Prove que se A⊂ * = {1,2,3,...} é um conjunto não-vazio tal que


n ∈ A ⇒ 4n ∈ A e  n  ∈ A então A = * .
Obs.  x é o único inteiro tal que x −1 <  x ≤ x

85) Mostre que todo triângulo pode ser dividido em 9 pentágonos convexos de
áreas iguais.

86) Encontre todas as triplas de inteiros positivos (a, m, n) tais que am +1 divide
(a +1)n .

EUREKA! N°17, 2003

59
Sociedade Brasileira de Matemática

87) Seja a(1) = 1 e, para cada inteiro n ≥ 2, a(n) igual ao menor inteiro positivo
n
que não pertence a {a( j), j < n} tal que ∑a( j) seja múltiplo de n. Prove que
j =1

a(a(n)) = n para todo inteiro positivo n.

88) Prove que se r ∈ _ e cos(r ⋅π ) ∈_ então cos(r ⋅π ) ∈−1, − 12 ,0, 12 ,1.

9RFr VDELD«
2 JUDQGH /HRQDUG (XOHU DILUPRX TXH QmR Ki VROXo}HV LQWHLUDV
SRVLWLYDV SDUD D HTXDomR
;  \  ] Z 'XUDQWH  DQRV QLQJXpP FRQVHJXLX
GHPRQVWUDU LVWR 3DUHFLD VHU XPD DILUPDomR YHUGDGHLUD XPD
YH] TXH WDPEpP QLQJXpP SRGH SURYDU TXH HUD IDOVD
(QWUHWDQWR 1RDP (ONLHV GD 8QLYHUVLGDGH GH +DUYDUG
WUDEDOKDQGR FRP XP SRWHQWH FRPSXWDGRU HQFRQWURX
     
$ DILUPDomR GH (XOHU p IDOVD

Problema 81 proposto por Geraldo Perlino (Itapecerica da Serra – SP); Problema 82 proposto
por Clodoaldo Lessa (Mogi das Cruzes – SP); Problema 83 adaptado de um problema
proposto por Gibran M. de Souza (Natal – RN); Problema 84 proposto por Anderson Torres
(São Paulo – SP); Problema 85 proposto por Gibran M. de Souza (Natal – RN); Problema 88
proposto por C.G. Moreira e José Paulo Carneiro (Rio de Janeiro – RJ).

EUREKA! N°17, 2003

60
Sociedade Brasileira de Matemática

AGENDA OLÍMPICA
XXV OLIMPÍADA BRASILEIRA DE MATEMÁTICA

NÍVEIS 1, 2 e 3
Primeira Fase – Sábado, 7 de junho de 2003
Segunda Fase – Sábado, 13 de setembro de 2003
Terceira Fase – Sábado, 18 de outubro de 2003 (níveis 1, 2 e 3)
Domingo, 19 de outubro de 2003 (níveis 2 e 3 - segundo dia de prova).

NÍVEL UNIVERSITÁRIO
Primeira Fase – Sábado, 13 de setembro de 2003
Segunda Fase – Sábado, 18 e Domingo, 19 de outubro de 2003

IX OLIMPÍADA DE MAIO
10 de maio de 2003

XIV OLIMPÍADA DE MATEMÁTICA DO CONE SUL
23 a 30 de maio de 2003
Ica – Peru

XLIV OLIMPÍADA INTERNACIONAL DE MATEMÁTICA
07 a 19 de julho de 2003
Tóquio – Japão

X OLIMPÍADA INTERNACIONAL DE MATEMÁTICA UNIVERSITÁRIA
25 a 31 de julho de 2003
Universidade Babes-Bolyai, Cluj-Napoca, Romênia

XVIII OLIMPÍADA IBEROAMERICANA DE MATEMÁTICA
13 a 20 de setembro de 2003
Argentina

VI OLIMPÍADA IBEROAMERICANA DE MATEMÁTICA UNIVERSITÁRIA
8 de novembro de 2003

♦♦♦

EUREKA! N°17, 2003

61
Sociedade Brasileira de Matemática

COORDENADORES REGIONAIS
Alberto Hassen Raad (UFJF) Juiz de Fora – MG
Amarísio da Silva Araújo (UFV) Viçosa – MG
Ana Paula Bernardi da Silva (Universidade Católica de Brasília) Brasília – DF
Benedito Tadeu Vasconcelos Freire (UFRN) Natal – RN
Carlos Frederico Borges Palmeira (PUC-Rio) Rio de Janeiro – RJ
Claus Haetinger (UNIVATES) Lajeado – RS
Cleonor Crescêncio das Neves (UTAM) Manaus – AM
Élio Mega (Colégio Etapa) São Paulo – SP
Florêncio Ferreira Guimarães Filho (UFES) Vitória – ES
Gil Cunha Gomes Filho (Colégio ACAE) Volta Redonda – RJ
Ronaldo Alves Garcia (UFGO) Goiânia – GO
Reginaldo de Lima Pereira (Escola Técnica Federal de Roraima) Boa Vista – RR
Ivanilde Fernandes Saad (UC. Dom Bosco) Campo Grande– MS
Jacqueline Fabiola Rojas Arancibia (UFPB) João Pessoa – PB
Janice T. Reichert (UNOCHAPECÓ) Chapecó – SC
João Benício de Melo Neto (UFPI) Teresina – PI
João Francisco Melo Libonati (Grupo Educacional Ideal) Belém – PA
José Carlos dos Santos Rodrigues (Unespar) Campo Mourão – PR
José Cloves Saraiva (UFMA) São Luis – MA
José Gaspar Ruas Filho (ICMC-USP) São Carlos – SP
José Luiz Rosas Pinho (UFSC) Florianópolis – SC
José Vieira Alves (UFPB) Campina Grande – PB
Licio Hernandes Bezerra (UFSC) Florianópolis – SC
Luzinalva Miranda de Amorim (UFBA) Salvador – BA
Mário Rocha Retamoso (UFRG) Rio Grande – RS
Marcelo Rufino de Oliveira (Grupo Educacional Ideal) Belém – PA
Marcelo Mendes (Colégio Farias Brito, Pré-vestibular) Fortaleza – CE
Pablo Rodrigo Ganassim (Liceu Terras do Engenho) Piracicaba – SP
Ramón Mendoza (UFPE) Recife – PE
Raúl Cintra de Negreiros Ribeiro (Colégio Anglo) Atibaia – SP
Reinaldo Gen Ichiro Arakaki (INPE) SJ dos Campos – SP
Ricardo Amorim (Centro Educacional Logos) Nova Iguaçu – RJ
Sérgio Cláudio Ramos (IM-UFRGS) Porto Alegre – RS
Tadeu Ferreira Gomes (UEBA) Juazeiro – BA
Tomás Menéndez Rodrigues (U. Federal de Rondônia) Porto Velho – RO
Valdenberg Araújo da Silva (U. Federal de Sergipe) São Cristovão – SE
Valdeni Soliani Franco (U. Estadual de Maringá) Maringá – PR
Vânia Cristina Silva Rodrigues (U. Metodista de SP) S.B. do Campo – SP
Wagner Pereira Lopes (CEFET – GO) Jataí – GO

EUREKA! N°17, 2003

62
CONTEÚDO

AOS LEITORES 2

XIV OLIMPÍADA DE MATEMÁTICA DO CONE SUL 3


Enunciados e Soluções

XLIV OLIMPÍADA INTERNACIONAL DE MATEMÁTICA 13


Enunciados e Soluções

X OLIMPÍADA INTERNACIONAL DE MATEMÁTICA PARA ESTUDANTES UNIVERSITÁRIOS 23


Enunciados e Soluções

XVIII OLIMPÍADA IBERO-AMERICANA DE MATEMÁTICA 34


Enunciados e Soluções

ARTIGOS

A DESIGUALDADE DE ERDÖS - MORDELL 42


Anderson Torres

COMO É QUE FAZ? 53

SOLUÇÕES DE PROBLEMAS PROPOSTOS 54

PROBLEMAS PROPOSTOS 58

COORDENADORES REGIONAIS 62
Sociedade Brasileira de Matemática

AOS LEITORES

Chegamos à última edição de 2003 publicando soluções de diversas


olimpíadas internacionais: a Olimpíada de Matemática do Cone Sul, a Olimpíada
Internacional de Matemática, a Olimpíada Ibero-americana e a Olimpíada
Internacional de Matemática para Estudantes Universitários, da qual participamos
pela primeira vez. Publicamos também um artigo sobre a desigualdade de Erdös
Mordell, que ajuda a resolver um dos problemas mais difíceis que já caíram na
IMO.

Estamos propondo sete problemas bacanas na seção de problemas


propostos, que se somam aos 6 problemas da última edição dos quais ainda não
recebemos solução. Haverá portanto bastante diversão para o começo de 2004…

Em 2004 a Olimpíada Internacional de Matemática será realizada na


Grécia, assim como as Olimpíadas. Vamos torcer para o Brasil trazer belas
medalhas nos dois casos…

Abraços e feliz ano novo a todos,

Os editores

EUREKA! N°18, 2003

2
Sociedade Brasileira de Matemática

XIV OLIMPÍADA DE MATEMÁTICA DO CONE SUL


Enunciados e Soluções

PROBLEMA 1
Em um torneio de futebol entre quatro equipes, A, B, C e D, cada equipe joga com
cada uma das outras exatamente uma vez.

a) Decidir se é possível que, ao finalizar o torneio, as quantidades de gols


marcados e sofridos pelas equipes sejam:

A B C D
Gols marcados 1 3 6 7
Gols sofridos 4 4 4 5

Se a resposta é afirmativa, dê um exemplo com os resultados das seis partidas; em


caso contrário, justifique.

b) Decidir se é possível que, ao finalizar o torneio, as quantidades de gols


marcados e sofridos pelas equipes sejam:

A B C D
Gols marcados 1 3 6 13
Gols sofridos 4 4 4 11

Se a resposta é afirmativa, dê um exemplo com os resultados das seis partidas; em


caso contrário, justifique.

SOLUÇÃO DE FÁBIO DIAS MOREIRA (RIO DE JANEIRO – RJ)


a) A configuração apresentada é possível. Por exemplo,

A×B 1×1
A×C 0×0
A×D 0×3
B×C 0×3
B×D 2×0
C×D 3×4

EUREKA! N°18, 2003

3
Sociedade Brasileira de Matemática

b) Como D não joga contra si mesmo, todos os gols que marca são contra A, B e
C. Logo todo gol marcado por D aumenta em um a contagem de gols
marcados por D e aumenta em um a soma do número de gols sofridos por A, B
e C.
Mas como D marcou treze gols, A, B e C deveriam ter sofrido pelo menos
treze gols no total, mas sofreram apenas 12, absurdo! Logo a situação
apresentada é impossível.

PROBLEMA 2
Considere a seqüência {an} definida da seguinte maneira:
a1 = 1
a2 = 3
an+2 = 2an+1 an + 1, para todo inteiro n ≥ 1.

Provar que a máxima potência de 2 que divide a4006 − a4005 é 22003.

SOLUÇÃO DE RODRIGO AGUIAR PINHEIRO (FORTALEZA – CE)


Seja An = an − an −1 ; n ∈ e n≥2
Indução: ∀n ∈
* n
, temos 2 sendo a maior potência de 2 que divide A2 n e
2n +1 A2 n +1
Base de Indução: n = 1: a3 = 2a2 ⋅ a1 + 1 = 2 ⋅ 3 ⋅ 1 + 1 = 7 ⇒ A2⋅1 = a2 − a1 = 2 e
A2⋅1+1 = a3 − a2 = 4
1 2 1+1
Temos que 2 A2⋅1 mas 2 não divide A2⋅1 e 2 A2⋅1+1. OK!!
n = 2 : a4 = 2 ⋅ a3 ⋅ a2 + 1 = 2 ⋅ 7 ⋅ 3 + 1 = 43
a5 = 2 ⋅ a4 ⋅ a3 + 1 = 2 ⋅ 43 ⋅ 7 + 1 = 603
A4 = 43 − 7 = 36 e A5 = 603 − 43 = 560
36 = 22 ⋅ 9, logo 22 A2⋅2 , mas 23 não divide A2⋅2
560 = 24 ⋅ 7 ⋅ 5, logo 22 +1 A2⋅2 +1 .
Passo Indutivo: Supondo que para todos os naturais menores ou iguais a k, valha a
sentença da indução, provaremos que vale para k + 1.
A2 k + 2 = a2 k + 2 − a2 k +1 = (2 ⋅ a2 k +1 ⋅ a2 k + 1) − (2 ⋅ a2 k ⋅ a2 k −1 + 1)
= 2a2 k ( a2 k +1 − a2 k −1 )
= 2a2 k ( a2 k +1 − a2 k + a2 k − a2 k −1 )
⇔ A2 k + 2 = 2a2 k ( A2 k +1 + A2 k )

EUREKA! N°18, 2003

4
Sociedade Brasileira de Matemática

k +1
Seja A2 k = 2 ⋅ x2 k , x2 k ímpar (hipótese de indução) e A2 k +1 = 2 ⋅ x2 k +1 .
k

k +1
Temos: A2 k + 2 = 2 ⋅ a2 k (2 ⋅ x2 k +1 + 2 x2 k ) = 2 a2 k (2 ⋅ x2 k +1 + x2 k )
2k k

ímpar ímpar
k +1 k +2
Portanto 2 A2 k + 2 , mas 2 não divide A2 k + 2
A2 k + 3 = 2a2 k +1 ( A2 k + 2 + A2 k +1 ) = 2 ⋅ a2 k +1 (2k +1 ⋅ x2 k + 2 + 2k +1 ⋅ x2 k +1 )
= 2k + 2 a2 k +1 ( x2 k + 2 + x2 k +1 ) , portanto 2k +1+1 A2( k +1) +1
Para k = 2003, temos que 2
2003
A4006 = a4006 − a4005 , mas 22004 não divide A4006 .

PROBLEMA 3
Seja ABC um triângulo acutângulo tal que o ângulo B mede 60o . A circunferência
de diâmetro AC intersecta as bissetrizes internas de A e C nos pontos M e N
respectivamente (M ≠ A, N ≠ C). A bissetriz interna do ângulo B intersecta MN e
AC nos pontos R e S, respectivamente. Demonstrar que BR ≤ RS .

SOLUÇÃO DE THIAGO COSTA LEITE SANTOS (SÃO PAULO – SP)

30° 30°
M
K
N R
W α

α – 30° I
30°
30° β
α β
A T S C

EUREKA! N°18, 2003

5
Sociedade Brasileira de Matemática

Digamos que ∠BAC = 2α ≥∠BCA = 2β ,


Sem perda de generalidade 2α + 2β = 120° ⇒α + β = 60° .
N está dentro do triângulo, pois como AC é diâmetro
⇒∠ANC =∠AMC = 90° ⇒∠NAC = 90°− β ⇒
⇒∠NAM = 90 − β −α = 30° e ∠NAB =α − 30° .
De modo análogo M está fora do triângulo.
Assim, ∠MCA = 90°−α ⇒∠MCB = 90°−α − 2β = 30°− β
Como ∠KMI =∠NCA , pois ANMC é inscritível, ∠KMI = β =∠KCI ⇒ IKMC é
inscritível ⇒∠IKC = ∠IMC = 90°. Como ∠KMI = β e
∠KCM = 30° − β , ∠CKM = 60° = ∠NKB e ∠KWB = 60°. Como I é o incentro,
KI 1
IK = IT = r ⇒ sen30° =
= ⇒ BI = 2KI = 2r
BI 2
(r é o inraio). O triângulo KWB é equilátero, pois provamos que
∠BWK =∠BKW =∠WBK = 60°. Como BR é bissetriz de ∠WBK, BR ⊥WK .
No triângulo RKI:
RI 1 KI r
sen30° = = ⇒ RI = = .
KI 2 2 2
r 3r
⇒ BR = BI − RI = 2r − =
2 2
⇒ SI ≥ IT = r, pois ∠ITS = 90°
r r 3r
⇒ RS = RI + SI = + SI ≥ + r = = BR ⇒ RS ≥ BR .
2 2 2

PROBLEMA 4
No triângulo acutângulo ABC, os pontos H, G e M encontram-se sobre o lado BC,
de modo que AH, AG e AM são altura, bissetriz e mediana do triângulo,
respectivamente. Sabe-se que HG = GM, AB = 10 e AC = 14. Determinar a área do
triângulo ABC.

EUREKA! N°18, 2003

6
Sociedade Brasileira de Matemática

SOLUÇÃO DE RODRIGO AGUIAR PINHEIRO (FORTALEZA – CE)


A
α α
2 2
10 14
z

B x H k G k M y C

Seja BH = x, HG = GM = k, MG = y e AH = z.
Como AM é mediana, BM = MC ⇒ BH + HG + GM = MC
⇔ x + 2k = y (I)
Apliquemos o teorema da bissetriz interna, considerando a bissetriz AG:
AB BG 10 x + k
= ⇔ = ⇔5( y + k) = 7(x + k)
AC GC 14 y + k
⇔5y = 7x + 2k (II)
3
(I) em (II): 5y = 6x + x + 2k = 6x + y ⇔ y = x (III)
2
3 1
(III) em (I): x + 2k = y = x ⇔ x = 2k ⇔ x = 4k
2 2
3 3
(III): y = x = (4k) ⇔ y = 6k.
2 2
Apliquemos o teorema de Pitágoras no ∆AHC :
AC2 = AH2 + HC2 ⇔ z2 + (8k)2 =142
Teorema de Pitágoras no ∆AHB :
AB2 = AH2 + HB2 ⇔ z2 + (4k)2 =102
z2 + 64k2 =196
Temos: ⇒ z2 + 64k2 − (z2 +16k2 ) =196 −100
z2 +16k2 =100
⇔ 48k2 = 96 ⇔ k2 = 2 ⇔ k = 2
68 ⋅12⋅ 2
z2 +16k2 =100 ⇔ z2 +16⋅ 2 =100 ⇔ z2 = 68 ⇔ z = 68; Área = =12 34 .
2

EUREKA! N°18, 2003

7
Sociedade Brasileira de Matemática

PROBLEMA 5
Seja n = 3k + 1 , onde k é um inteiro, k ≥ 1 . Constrói-se um arranjo triangular
de lado n formado por círculos de mesmo raio como o mostrado na figura para
n = 7.

Determinar, para cada k, o maior número de círculos que podem ser coloridos de
vermelho de tal modo que não existam dois círculos vermelhos tangentes entre si.

ADAPTAÇÃO DA SOLUÇÃO DE THIAGO COSTA LEITE SANTOS (SÃO PAULO – SP)


Obviamente n = 6k0 + 1 ou n = 6k0 + 4 , quando k = 2k0 ou k = 2k0 + 1 ,
respectivamente. Observe que se tivermos seis círculos do seguinte formato:
nós só podemos pintar no máximo 2 bolas; caso pintemos 3,
haverá duas bolas pintadas tangentes, o que é absurdo!

Primeiro caso: k = 2k0 ⇒ n = 6k0 + 1 ⇒


Denote por ak o maior número de círculos que podemos pintar. No triângulo n × n
pegue as 3 últimas linhas:
Primeira linha 3k +2

Segunda linha
… 3k +3
… 3k + 4

3k +1
… 3k +2
… 3k +3
… 3(k+1)+1

EUREKA! N°18, 2003

8
Sociedade Brasileira de Matemática

Dividimos aqueles círculos em peças de 6 círculos e uma peça de 3 círculos da


seguinte forma:

3k +2
3k +3
… 3k + 4

1 2 3 4 3k + 2
2

3k + 2
Como cada peça tem no máximo 2 círculos pintados e nós temos
2
3k + 2
peças (note que ∈ pois k ≡ 0(mod 2)) e na peça podemos pintar
2
um círculo ⇒ ak +1 ≤ ak + 3(k + 1) nesse caso.

Segundo caso: k = 2k0 + 1 ⇒ n = 6k0 + 4


As três últimas linhas são assim:

3k +2
3k +3
… 3k + 4

 3k + 1 
ak +1 ≤ ak + 2   + r onde r é o número de círculos pintados da peça .
 2 
Temos que r ≤ 3 .(*)
Se r = 2 temos de novo ak +1 ≤ ak + 3(k + 1) e um jeito de montarmos é o seguinte:

EUREKA! N°18, 2003

9
Sociedade Brasileira de Matemática

Pegue a peça destacada e


1
coloque assim:
2
3
4
5
6
7
8
9

3k + 1= n A partir daí fazemos por


3k + 2 = n + 1 indução:
3k + 3 = n + 2
3k + 4 = n + 3

Nos dois casos (k par ou k ímpar), as três últimas linhas podem ser pintadas
assim: Suponha pintado até a linha n e pinte as linhas
n + 1, n + 2, n + 3 da seguinte forma.

Na linha n + i = 3k + 1 + i, 1 ≤ i ≤ 3 faça o seguinte: enumere de 1 a n + i.

Se i = 1: na linha n + 1, se o círculo tiver um número da forma 3k, pinte-o de


vermelho.

Se i = 2: na linha n + 2, se o círculo tiver um número da forma 3k + 2, pinte-o.

Se i = 3: na linha n + 3, se o círculo tiver um número da forma 3k + 1, pinte-o.


3k (k + 1)
Assim, teremos ak +1 ≥ ak + 3(k + 1), a1 = 4 ⇒ ak ≥ 1 + , ∀k ≥ 1.
2
 3k + 1 
Vamos ver o que acontece se em (*) tivermos r = 3 e 2 ⋅   + r círculos
 2 
pintados nas 3 últimas linhas.

A peça será pintada assim


As 3 últimas linhas serão pintadas assim:

EUREKA! N°18, 2003

10
Sociedade Brasileira de Matemática

… 3k – 1
… 3k
… 3k + 1
… 3k + 2
… 3k + 3
3k + 4

Na linha i coloque os números assim:

1 2 3 4 5 6 … i

Na linha 3k + 2 e 3k + 4 os números impares serão pintados e na linha 3k + 3

ninguém. Como cada peça da forma só tem no máximo 2 círculos pintados,


nas linhas 3k + 2 e 3k + 4 todos os números ímpares serão pintados, porque se não
ak +1 ≤ ak + 3(k + 1). Assim, nas 3 linhas anteriores (3k – 1, 3k e 3k + 1) temos no
máximo 2k círculos pintados (pois na linha 3k + 1 não podemos ter nenhum),
donde ak +1 ≤ ak −1 + 2k + 3k + 4 ≤ ak −1 + 3k + 3( k + 1) e, de qualquer jeito, temos
k
3k (k + 1)
ak ≤ 1 + ∑ 3 j = 1 + , por indução. A resposta, portanto, é
j =1 2
3k (k + 1)
ak = 1 + .
2

PROBLEMA 6
Demonstrar que existe uma seqüência de inteiros positivos x1 , x2 ,..., xn ,... que
satisfaz as duas condições seguintes:
i) contém exatamente uma vez cada um dos inteiros positivos,
para cada n = 1, 2,..., a soma parcial x1 + x2 + ... + xn é divisível por n .
n
ii)

SOLUÇÃO DE FÁBIO DIAS MOREIRA (RIO DE JANEIRO – RJ)


Definiremos indutivamente uma tal seqüência. Seja x1 = 1, x2 = 3, e suponha a
seqüência definida até x2n. Seja k o menor inteiro positivo que ainda não apareceu
entre x1 ,..., x2 n . Pelo Teorema Chinês dos Restos, existem infinitos x2 n +1 tais que

EUREKA! N°18, 2003

11
Sociedade Brasileira de Matemática

x2 n +1 ≡ − x1 − x2 ... − x2 n (mod(2n + 1) 2 n +1 ) , e
x2 n +1 ≡ − x1 − x2 ... − x2 n − k (mod(2n + 2) 2 n + 2 )

já que mdc(2n + 1, 2n + 2) = 1 ⇒ mdc((2n + 1)2 n +1 ,(2n + 2) 2 n + 2 ) = 1. Em


particular, existe um tal x2 n +1 ∉{x1 , x2 ,..., x2 n , k}, já que este último conjunto é
2 n +1
finito. Tomamos x2 n +1 = x2 n +1 e x2 n + 2 = k . Note que (2n + 1) x1 + ... + x2 n +1 e
(2n + 2) 2 n + 2 x1 + ... + x2 n + 2 , logo a extensão da seqüência respeita (ii). Como,
para 1 ≤ i ≤ 2n, x2 n +1 ≠ xi , x2 n +1 ≠ k por construção e x2 n + 2 ≠ xi pela definição de
k, a extensão respeita que cada inteiro aparece no máximo uma vez. Mas depois de
x extensões, todos os números 1, 2,…,x já apareceram, pois são os x menores
inteiros positivos. Logo todo inteiro positivo x aparece até o (2x + 2)-ésimo termo,
logo todo inteiro aparece pelo menos uma vez. Logo ( xi )i∈ respeita (i), e
portanto ( xi )i∈ satisfaz as condições do enunciado.

EUREKA! N°18, 2003

12
Sociedade Brasileira de Matemática

XLIV OLIMPÍADA INTERNACIONAL DE MATEMÁTICA


07 a 19 de julho, Tóquio - Japão

PROBLEMA 1
Seja A um subconjunto do conjunto S = {1,2, … ,1000000} com exatamente 101
elementos. Demonstre que existem números t1 , t 2 , … , t100 em S tais que os
conjuntos
A j = {x + t j | x ∈ A} , para j = 1, 2,… ,100,
são disjuntos dois a dois.

SOLUÇÃO DE FÁBIO DIAS MOREIRA (RIO DE JANEIRO – RJ)


101
Seja ∆A = {x − y | x, y ∈ A, x > y}. Certamente ∆A ⊂ S ,| ∆A |≤   = 5050, já que
 2 
 101
 pares de inteiros ( x, y ) ∈ A tais que x > y. Afirmamos que se
2
há 
 2 
T = {t1 ,..., t100 } é tal que t j − ti ∉ ∆A, ∀i, j ∈ {1,...,100}, então t1 ,..., t100 satisfazem o
enunciado. De fato, se houver x, y ∈ A, x > y, tais que x + ti = y + t j , então
x − y = t j − ti . Mas o lado esquerdo da igualdade está em ∆Aj , o direito não,
absurdo!
Então construímos T da seguinte forma:
i) 1∈ T
ii) Para cada elemento ti adicionado, proibimos todos os inteiros da forma
ti + x, x ∈ ∆A.
iii) Escolhemos o menor inteiro que não foi proibido, nem escolhido
anteriormente.
iv) Repetimos ii) e iii) 99 vezes.

Se existissem ti , t j , t j > ti em T, tais que t j − ti ∈ ∆A, então, como t j > ti ,


t j = ti + δ , δ ∈ *
+ , logo δ ∈ ∆A, absurdo, pois t j teria sido proibido, pois é da
forma ti + x, x ∈ ∆A.
Logo basta verificar que T ⊂ S . Mas como min T = 1 , basta verificar
max T ≤ 106. Mas se T = {1 = t1 < t2 < ... < t100 } , então t2 ≤ 5052 , pois há no
máximo 5050 inteiros proibidos entre ele e t1 , além de 1 já escolhido. Da mesma

EUREKA! N°18, 2003

13
Sociedade Brasileira de Matemática

maneira, t3 ≤ 10103,..., t100 ≤ 99 × 5050 + 100 = 500050 < 10 6 , concluindo a


demonstração.

PROBLEMA 2
Determine todos os pares de inteiros positivos (a, b) tais que
a2
2ab − b 3 + 1
2

é um inteiro positivo.

SOLUÇÃO DE ALEX CORRÊA ABREU (NITERÓI – RJ)


Primeiro caso: b = 1
a2
⇒ é inteiro ⇔ a é par ⇒ temos as soluções (2k, 1) k ∈ N*.
2a
Segundo caso: b > 1
como queremos inteiro positivo ⇒ 2ab 2 − b3 + 1 > 0 ⇒ b 2 (2a − b) + 1 > 0 e como
b > 1, devemos ter b ≤ 2a
agora:
2')se 2a ≥ b ≥ a, b 2 (2a − b) + 1 ≤ a 2 ⇒ a 2 (2a − b) + 1 ≤ a 2 ⇒ 1 ≤ a 2 (b − 2a + 1) ⇒
⇒ b = 2a . Temos então as soluções (a, 2a ) a ∈ N* .
2'') se b < a e (a, b) é uma solução, suponha que
a2
= k ⇒ a2 − 2kb2a + (b3 −1)k = 0 então a equação x 2 − 2kb 2 x + (b3 − 1) k = 0
2ab2 − b3 +1
tem uma solução a e a outra é 2kb 2 − a, que é positiva pois o produto das soluções
é (b3 − 1)k > 0.
Então vamos mostrar que
2a 2b2 ab3 − a
b ≥ 2kb2 − a ⇔ b ≥ − a ⇔ b ≥ ⇔ 2ab3 − b4 + b ≥ ab3 − a ⇔
2ab2 − b3 + 1 2ab2 − b3 + 1
⇔ ab3 + b − b4 + a ≥ 0 ⇔ b3 (a − b) + a + b ≥ 0 o que é verdade, pois a > b. Assim, pelo
4a2b2 2ab3 − 2a
caso 2'), b = 2(2kb2 − a) = − 2a = ⇒ 2ab3 − b4 + b = 2ab3 − 2a
2ab2 − b3 +1 2ab2 − b3 +1
b4 − b b2
⇔ 2a = b4 − b ⇔ a = , com b par, pois k = . Temos então as soluções
2 4
 b4 − b 
 , b  , b > 1 par.
 2 

EUREKA! N°18, 2003

14
Sociedade Brasileira de Matemática

PROBLEMA 3
Considere um hexágono convexo tal que para cada quaisquer dois lados opostos
verifica-se a seguinte propriedade: a distância entre os seus pontos médios é igual
a 3 2 vezes a soma dos seus comprimentos. Demonstre que todos os ângulos do
hexágono são iguais.

(Um hexágono convexo ABCDEF tem três pares de lados opostos: AB e DE ,


BC e EF , CD e FA ).

SOLUÇÃO OFICIAL
Primeira Solução:
Lema: Considere o triângulo PQR com ∠QPR ≥ 60°. Seja L o ponto médio de
QR.
Logo PL ≤ 3 QR 2 , com igualdade se e somente se o triângulo PQR é
equilátero.

Prova:

Q R
L

Seja S um ponto tal que o triângulo QRS é equilátero, e os pontos P e S


encontram-se no mesmo semiplano limitado pela linha QR. Logo o ponto P
pertence ao circuncírculo do triângulo QRS, que se encontra dentro do círculo de
centro L e raio 3 QR 2 . Isto completa a prova do lema.

EUREKA! N°18, 2003

15
Sociedade Brasileira de Matemática

B M A

F
C P

D N E
As diagonais principais do hexágono convexo formam um triângulo embora o
triângulo possa ser degenerado. Assim podemos escolher duas das três diagonais
que formam um ângulo maior ou igual a 60°. Sem perda de generalidade,
podemos assumir que as diagonais AD e BE do hexágono dado ABCDEF
satisfazem ∠APB ≥ 60° , onde P é a interseção dessas diagonais. Logo, usando o
lema, temos:
3
MN = ( AB + DE ) ≥ PM + PN ≥ MN ,
2

onde M e N são pontos médios de AB e DE, respectivamente. Assim pelo lema, os


triângulos ABP e DEP são equiláteros.

Consequentemente a diagonal CF forma um ângulo maior ou igual a 60° com uma


das diagonais AD e BE. Sem perda de generalidade, assumimos que
∠AQF ≥ 60°, onde Q é a interseção de AD e CF. Da mesma maneira acima,
deduzimos que os triângulos AQF e CQD são equiláteros. Isto implica que
∠BRC = 60°, onde R é a interseção de BE e CF. Usando o mesmo argumento
acima pela terceira vez, obtemos que os triângulos BCR e EFR são equiláteros.
Isto completa a solução.

EUREKA! N°18, 2003

16
Sociedade Brasileira de Matemática

Segunda Solução:
Seja ABCDEF o hexágono dado e seja a = AB, b = BC ,..., f = FA.

B a M A
b f
F
C

c e
d
D N E
Sejam M e N os pontos médios dos lados AB e DE, respectivamente. Temos
1 1 1 1
MN = a + b + c + d e MN = − a − f − e − d .
2 2 2 2
Assim temos
1
MN = (b + c − e − f ). (1)
2
Da propriedade dada temos

3 3
MN =
2
( a + d )≥
2
a−d . (2)

x = a – d, y = c – f, z = e – b. desde (1) e (2) obtemos:

y−z ≥ 3 x. (3)

Similarmente vemos que


z−x ≥ 3 y , (4)
x− y ≥ 3 z . (5)
Note que
2 2 2
(3) ⇔ y − 2 y ⋅ z + z ≥ 3 x ,

EUREKA! N°18, 2003

17
Sociedade Brasileira de Matemática

2 2 2
(4) ⇔ z − 2 z ⋅ x + x ≥ 3 y ,
2 2 2
(5) ⇔ x − 2 x ⋅ y + y ≥ 3 z .

Adicionando as últimas três inequações, obtemos

2 2 2
− x − y − z − 2 y ⋅ z − 2 z ⋅ x − 2 x ⋅ y ≥ 0,

2
ou − x + y + z ≥ 0. Assim x + y + z = 0 e valem as igualdades em todas as
inequações acima.
Daí, concluímos que:
x + y + z = 0,
y − z = 3 x , a // d // x,
z − x = 3 y , c // f // y,
x − y = 3 z , e // b // z.

Supondo que PQR é o triângulo tal que PQ = x, QR = y, RP = z. Podemos


assumir ∠QRP ≥ 60° , sem perda de generalidade. Seja L o ponto médio de QR,
logo PL = z − x 2 = 3 y 2 = 3Q R 2.
Segue do lema na solução 1 que o triângulo PQR é equilátero. Assim temos
∠ABC = ∠BCD = ... = ∠FAB = 120°.

Nota: Obtemos a caracterização completa dos hexágonos satisfazendo a


propriedade dada. São obtidos a partir do triângulo equilátero cortando seus cantos
na mesma altura.

PROBLEMA 4
Seja ABCD um quadrilátero convexo cujos vértices estão sobre uma
circunferência. Sejam P , Q e R os pés das perpendiculares às retas BC , CA e
AB , respectivamente, passando por D . Demonstre que PQ = QR se e só se as
bissetrizes dos ângulos ∠ABC e ∠ADC se intersectam sobre a reta AC .

EUREKA! N°18, 2003

18
Sociedade Brasileira de Matemática

SOLUÇÃO DE ALEX CORRÊA ABREU (NITERÓI – RJ)

β Q α C
A
R P

D
Seja CAB = β e BCA = α.
Primeiro temos que as bisetrizes se intersectam sobreAC⇔
AB AD sen α AD PQ
= ⇔ = .Mas = DC pois DQCP é inscritível
BC DC sen β CD sen QCP
de diâmetro DC⇒ PQ = DC ⋅ senα e, analogamente,
AD senα
QR = AD ⋅ senβ ⇒ PQ = QR ⇔ DC ⋅ senα = AD ⋅ senβ ⇔ =
DC senβ
como queríamos.

Observe que a nossa demonstração independe do fato de ABCD ser um


quadrilátero inscritível. De fato, o quadrilátero mostrado no desenho não o é: A
está no segmento BR e C está no segmento BP. Todos os argumentos utilizados
continuam válidos se modificarmos as posições de A ou C.

PROBLEMA 5
Sejam n um inteiro positivo e x1 , x2 , … , xn números reais tais que
x1 ≤ x2 ≤ ≤ xn .

(a) Demonstre que

2
 n n  2(n 2 − 1) n n
 ∑∑ xi − x j  ≤ ∑∑ ( xi − x j ) 2 .
 i =1 j =1  3 i =1 j =1

EUREKA! N°18, 2003

19
Sociedade Brasileira de Matemática

(b) Demonstre que a igualdade é válida se e só se x1 , x2 , … , xn formam uma


progressão aritmética.

SOLUÇÃO
Vamos provar (a) por indução em n. Para n = 1 os dois lados valem 0 e, para n =
2, valem 4( x2 − x1 ) . Façamos o passo da indução: Temos
2

n n
x1 ≤ x2 ≤ … ≤ xn ≤ xn +1 . Assim, sendo An = ∑∑ xi − x j e
i =1 j =1
n n n +1 n +1 n
Bn = ∑∑ ( xi − x j ) , temos An +1 = ∑∑ xi − x j = 2∑ ( xn +1 − x j ) + An
2

i =1 j =1 i =1 j =1 j =1
n

∑( x − x j ) + Bn . Queremos então provar que


2
e Bn +1 = 2 n +1
j =1

 n
A =  2n ⋅ xn +1 − 2∑ x j + An  ≤
2
(
 2 ( n + 1) − 1
2 2
)
⋅ Bn +1 =
n +1
 j =1  3
2n ( n + 2 )  n n 
= − n +1 ∑ j + ∑ x 2j + Bn  , ou equivalentemente, que
2
 2 nxn +1 4 x x 2
3  j =1 j =1 
4n2 ( n −1)
2
 2(n −1) n  2n(n + 2)  n
2 
n 
3
x − 4n
2
n+1 ∑ x j + A x
n  n+1 +  n ∑ j   n ∑xj  ≥ 0.
B + 2 x − A − 2
 3 j=1  3  j =1   j =1 
Olhando o lado esquerdo como uma função quadrática de xn +1 , concluímos que
n
2(n − 1)∑ x j + 3 An
j =1
ela atinge o seu mínimo para xn +1 = , quando temos
2n(n − 1)
2
 n

 n 
2  2( n − 1) ∑ x j + 3 An n 
An +1 =  2nxn +1 − 2∑ x j + An  =  − 2∑ x j + An  =
2 j =1

 j =1   n − 1 j =1 
 
 
2 n n
n+2 
⋅ An  , enquanto Bn +1 = 2nxn +1 − 4 xn +1 ∑ x j + 2∑ x j + Bn =
2 2

 n −1  j =1 j =1

EUREKA! N°18, 2003

20
Sociedade Brasileira de Matemática

 n  n
= 2 xn +1  nxn +1 − 2∑ x j  + 2∑ x 2j + Bn =
 j =1  j =1
n n
2( n − 1)∑ x j + 3 An 3 An − 2(n − 1)∑ x j n
+ 2∑ x 2j + Bn =
j =1 j =1
= ⋅
n( n − 1) 2( n − 1) j =1
2
n 
−2 ∑xj 
1   n 2  n   
2
  9An2 n
9An2
∑ ∑j ∑
j =1
= + + 2 x2
+ B = B + 2 n x −  x   + ,
n   j=1   2n(n −1)2
j n n j
n 2n(n −1)2  
j =1
  j =1

n n  n  n  
2
2n(n + 2)
mas Bn = ∑∑ ( x j − xi ) = 2  n∑ x j −  ∑ x j   , donde An2+1 ≤
2 2
Bn +1 é
 j =1  j =1   3
i =1 j =1

( n + 2)
2
2n( n + 2)   n + 1  9 An2 
equivalente a An2 ≤   Bn + 
( n − 1) 2n(n − 1) 2 
2
3  n 
⇔ 3(n + 2) An2 ≤ 2( n + 1)(n − 1) 2 Bn + 9 An2
2(n 2 − 1)
⇔ 3(n − 1) An2 ≤ 2( n + 1)(n − 1) 2 Bn ⇔ An2 ≤ Bn , que é a hipótese de
3
indução. Assim, o valor mínimo de nossa função quadrática é maior ou igual a 0, o
que implica o resultado.
Para provar b), note que, se valem as igualdades nas desigualdades acima,
2( n 2 − 1)
devemos ter em particular An2 = Bn donde, por hipótese de indução,
3
x j = x1 + ( j − 1)r para um certo r ≥ 0 , ∀j ≤ n; além disso, devemos ter
n n
2(n − 1)∑ x j + 3 An 2( n − 1)∑ x j + 6 ∑ ( j − i )r
j =1 j =1 i ≤i < j ≤ n
xn +1 = = =
2n(n − 1) 2n(n − 1)
n −1
1 n 3
= ∑ 1
n j =1
( x + ( j − 1) r ) + ∑ (n − k )kr =
n(n − 1) k =1
(n − 1) 3  n( n − 1)(n + 1) r 
x1 + r+   = x1 + nr , donde ( xi )1≤ i ≤ n +1 é uma
2 n( n − 1)  6 
progressão aritmética, como queríamos provar.

EUREKA! N°18, 2003

21
Sociedade Brasileira de Matemática

PROBLEMA 6
Seja p um número primo. Demonstre que existe um número primo q tal que,
p
para todo inteiro n, o número n − p não é divisível por q.

SOLUÇÃO DE SAMUEL BARBOSA FEITOSA (FORTALEZA - CE)


É evidente que p ≠ q pois se p = q basta tomarmos n múltiplo de q para termos um
absurdo.
q −1

Se encontrarmos um primo q tal que p q − 1 e p p


≠ 1(mod q) (1)
o problema terá acabado pois se n ≡ p (mod q ) , como p ≠ q então mdc (n, q)
p

q −1 q −1
p⋅
= 1, donde n p
≡p p
(mod q ) mas pelo pequeno teorema de Fermat temos
q −1
n ≡ 1(mod q ) . Isso produz um absurdo por (1), logo devemos ter
n ≠ p (mod q ) .
p

Para encontrarmos o nosso tal primo q que satisfaz (1) consideremos o número
X = p p −1 + p p − 2 + ... + p + 1 . É claro que X p p − 1 (2).
Seja q um divisor primo de X e k = ord q p . De (2) temos que
k p ⇒ k = 1 ou k = p . Se k = 1 temos p ≡ 1(mod q ) ⇒ p j ≡ 1(mod q ) ; daí
X ≡ 1 + 1... + 1 = p (mod q ) mas q X e q ≠ p , absurdo, logo k = p, mas
p q −1 ≡ 1(mod q ) ⇒ p = k q − 1 ⇒ q = py + 1 para algum y inteiro. Existe algum
fator primo q de X tal que y não seja divisível por p, pois se para todo
fator primo q de X tivermos q ≡ 1(mod p 2 ) , teríamos X ≡ 1(mod p 2 ) , mas
X ≡ p + 1 ≠ 1(mod p 2 ) logo existe um primo q divisor de X tal que p q − 1 e
q ≠ 1(mod p 2 ) . Mostremos que tal primo q satisfaz (1). Como k = ord q p = p e
q −1
q −1
q ≠ 1(mod p ) ⇒ k = p não divide
2
⇒ p p
≠ 1(mod q) , donde o primo
p
q satisfaz (1).

EUREKA! N°18, 2003

22
Sociedade Brasileira de Matemática

X OLIMPÍADA INTERNACIONAL DE MATEMÁTICA PARA


ESTUDANTES UNIVERSITÁRIOS
25 a 31 de Julho, Cluj - Napoca, Romênia

PROBLEMA 1
a) Seja a1 , a2 ,… , an ,... uma seqüência de números reais tais que a1 = 1 e
3
an +1 > an , ∀n.
2
an
Prove que a seqüência n −1
tem um limite finito ou tende a infinito.
3
 
2
b) Prove que para todo α > 1 existe uma seqüência a1 , a2 ,… , an ,... com as
an
mesmas propriedades, tal que lim n −1
= α.
n →∞
3
 
2
SOLUÇÃO
an
(a) Considere a seqüência bn = n −1
.
3
 
2
bn +1 a
Temos bn > 0, ∀n e = n +1 > 1 , logo (bn) é uma seqüência crescente.
bn  3an 
 2 
 
Mas toda seqüência monótona limitada é convergente. Logo, há duas opções
possíveis: ou (bn) é ilimitada e, como é crescente, tende a infinito ou então, se ela
for limitada, ela tem um limite finito (pois é crescente).

1
(b) Tome q = 1− e considere a seqüência bn = 1 + q + q 2 + ... + q n −1 .
α
α > 1 ⇒ 0 < q < 1 e portanto (bn) é uma seqüência convergente, com
1
lim bn = =α .
n →∞ 1− q

EUREKA! N°18, 2003

23
Sociedade Brasileira de Matemática

n −1
3
Portanto, a seqüência a n =   bn satisfaz as condições do enunciado.
2

Observação: A solução da letra (b) é bem mais natural do que parece. A letra (a)
induz você a pensar apenas na seqüência (bn). E na tentativa de encontrar uma
seqüência com determinadas condições, nada mais natural do que tentar uma
seqüência fácil como uma PG (e aí descobrir o valor de q necessário).

PROBLEMA 2
Sejam a1 , a2 ,… , a51 elementos não nulos de um corpo. Simultaneamente
trocamos cada elemento pela soma dos outros 50. Desta forma a nova seqüência
b1 , b2 ,… , b51 é uma permutação da anterior. Quais são os possíveis valores da
característica do corpo?

SOLUÇÃO
Seja p a característica do corpo.
51 51 51
S = ∑ bk = ∑ ( S − a k ) = 51S − ∑ a k = 51S − S , logo 49S = 0.
k =1 k =1 k =1
Se S ≠ 0 , temos que 49 = 0, logo p | 49 e como p é primo, p = 7. Um exemplo

( ) , com
*
de corpo que satisfaz essa propriedade é
7
a k = 1 , para k = 1,2,...,51.
Se S = 0, cada a k é igual a − bk = − a j , para algum j, donde a permutação σ tal
que a k = − aσ ( k ) possui um ponto fixo, pois os números 1,2,...,51 estão divididos
em pares (pela ação da σ ) e 51 é ímpar. Portanto existe um n tal que
a n = −aσ ( n ) = −a n , logo 2a n = 0 e como a n ≠ 0 , temos que 2 = 0 e o corpo
possui característica 2. Um exemplo de corpo que satisfaz essa propriedade é
GF(22), isto é, o corpo cujos elementos são polinômios, tomados módulo
x2 + x + 1, com coeficientes em 2 .
Basta tomar ( a1 , a 2 ,..., a 51 ) = (1, x,1 + x,1, x,1 + x,...,1, x,1 + x ) .

PROBLEMA 3
Seja A uma matriz quadrada n × n tal que 3A3 = A2 + A + I. Prove que (Ak )k ∈ N
converge a uma matriz idempotente B (i.e., a uma matriz B tal que B2 = B).

EUREKA! N°18, 2003

24
Sociedade Brasileira de Matemática

SOLUÇÃO
Seja m(x) o polinômio minimal de A. Como 3 A 3 − A 2 − A − I = 0 , m(x) deve
−1± 2
ser divisor de 3 x 3 − x 2 − x − 1 = ( x − 1)( x − λ1 )( x − λ 2 ) , com λ1, 2 = .
3
Isso implica que m(x) tem raízes distintas, e portanto A é diagonalizável, isto é,
existe uma matriz P inversível tal que A = P −1 ⋅ Diag (1,...,1, λ1,..., λ1, λ 2 ,..., λ 2 ) ⋅ P ,
onde Diag(a,b,c,...) representa a matriz diagonal de entradas a,b,c,...
Segue que Ak = P −1 ⋅ Diag (1,...,1, λk1 ,..., λk1 , λk2 ,..., λk2 ) ⋅ P .

Como λ1, 2 < 1 , segue que B = lim k →∞ Ak = P −1 ⋅ Diag (1,...,1,0,...,0) ⋅ P , que é


claramente idempotente.

SOLUÇÃO ALTERNATIVA
Seja Ak = Ak . Então 3 Ak +3 = Ak + 2 + Ak +1 + Ak . Resolvendo a recorrência (veja o artigo
"Equações de recorrência", na Eureka! 9), obtemos Ak = C0 + C1 ⋅ λ1k + C2 ⋅ λ2k para
todo k, sendo C0 , C1 , C2 matrizes quadradas de ordem n. Como | λ1,2 |≤ 1, lim Ak = C0 .
k →∞

Como lim Ak = lim A2 k ⇔ C02 = C0 , C0 é idempotente.


k →∞ k →∞

PROBLEMA 4
Determine o conjunto de todos os pares (a, b) de inteiros positivos para os quais o
conjunto dos inteiros positivos pode ser decomposto em dois conjuntos A e B tais
que a ⋅ A = b ⋅ B.

SOLUÇÃO
Note que o par (a, b) funciona se e somente se o par de coprimos
 a b 
 ,  funciona. Vamos então analisar os casos com mdc(a, b)=1.
 mdc (a, b) mdc (a, b) 
Suponha 1 ∈ A (o outro caso é análogo). Então, a ⋅ A = b ⋅ B ⇒ a ∈ b ⋅ B , e
portanto a é múltiplo de b (pois os elementos de B são todos inteiros), e
mdc(a, b) = 1 implica b = 1.
Reciprocamente, dado qualquer par da forma (a, 1), com a > 1, é possível
construir conjuntos A e B satisfazendo o enunciado. De fato, dado n ∈ N , seja kn a
maior potência de a que divide n. Tomando A = {n | kn é par}, B = {n | kn é ímpar}
temos A ∪ B = N , A ∩ B = ∅, a ⋅ A = 1 ⋅ B .

EUREKA! N°18, 2003

25
Sociedade Brasileira de Matemática

Portanto, os pares possíveis são os pares (a, b) com a ≠ b tais que a é múltiplo de
b ou b é múltiplo de a.

PROBLEMA 5

Sejam g :[0,1] → R uma função contínua e f n : (0,1] → R a seqüência de


1 x
x ∫0
funções definida por f 0 ( x ) = g ( x) e fn+1 ( x) = fn (t )dt, ∀x ∈ (0,1], n ≥ 0.

Determine lim f n ( x ) para todo x ∈ (0,1] .


n →∞

SOLUÇÃO

Veja inicialmente que se g é polinômio, o problema é fácil, pois


N N
ak
se g ( x) = ∑a
k =0
k
k x , temos claramente que f n ( x) = ∑
k = 0 ( k + 1)
n
x k , portanto

todos os coeficientes, com exceção do independente, tendem a zero quando


n → +∞ , e lim f n ( x) = a 0 = g (0) . A idéia é tentar mostrar que isso também
vale no caso de g não ser polinômio.
Para isso, vamos usar o teorema da aproximação de Stone-Weierstrass: Dado
ε
ε > 0 , existe um polinômio P tal que P( x) − g ( x) < , para todo x em [0,1]
3
~
(pois g é contínua e [0,1] é compacto). Agora olhe para a seqüência Pn tal que
x
~ 1 ~ ~
Pn +1 ( x) = ∫ Pn (t )dt e P0 = P .
x0
~
Como P é polinômio, lim Pn ( x) = P (0) , donde para n grande,
n → +∞

~ ε
Pn ( x) − P(0) < .
3
~ ε
Além disso, temos que se Pn ( x ) − f n ( x) < , então:
3
EUREKA! N°18, 2003

26
Sociedade Brasileira de Matemática

∫[ ] 1 ε ε
x x x
~ 1 ~ 1 ~
Pn +1 ( x) − f n +1 ( x) = Pn (t ) − f n (t ) dt ≤ ∫ Pn (t ) − f n (t ) dt < ∫ dt =
x 0
x0 x03 3
~ ε
Segue por indução que Pn ( x ) − f n ( x) < , para todo n natural e todo x em (0,1]
3
~ ε
já que P0 ( x ) − f 0 ( x) < . Agora fica fácil:
3
~ ~ ε ε ε
f n ( x) − g (0) ≤ f n ( x) − Pn ( x) + Pn ( x) − P(0) + P(0) − g (0) < + + = ε ,
3 3 3
para n grande, o que prova que lim f n ( x) = g (0) .
n → +∞

SOLUÇÃO ALTERNATIVA
Vamos provar que lim f n ( x ) = g (0), ∀x ∈ (0,1]. Para isso, vamos mostrar que,
n →∞

para todo x ∈ (0,1] e ε > 0, limsup f n ( x) − g (0) ≤ ε .


n →∞

Como g é contínua, existe δ > 0 tal que x < δ ⇒ g ( x) − g (0) < ε Se


0 < x < δ , para todo n ≥ 1,
1 x 1 x
f n ( x) − g (0) = ∫ ( f n −1 (t ) − g (0)) dt ≤ ∫ f n −1 (t ) − g (0) dt donde,
x 0 x 0
por indução, f n ( x) − g (0) < ε , ∀n ∈ . Se x ≥δ, temos
1 x 1 x
f n +1 ( x) − g (0) = ∫ ( f n (t ) − g (0) ) dt ≤ ∫ f n (t ) − g (0) dt =
x 0 x 0

=
1 δ
x ∫(
0
x
)
f n (t ) − g (0) dt + ∫ f n (t ) − g (0) dt , donde, se
δ

M n = max{ f n ( x) − g (0) , x ∈ (0,1]}, temos


1
M n +1 ≤ max{ε , max
δ < x ≤1 x
(δε + ( x − δ ) M n )} = max{ε , δε + (1 − δ ) M n }. Assim, se
L = lim sup M n (que existe, pois M n ≤ m ax{ε , M 0 }, ∀ n ∈ ) , temos
n→∞
L ≤ max{ε , δε + (1 − δ ) L}, donde L ≤ ε ou L ≤ δε + (1 − δ ) L ⇒ δ L ≤ δε ⇒ L ≤ ε ,
o que encerra a prova.

EUREKA! N°18, 2003

27
Sociedade Brasileira de Matemática

PROBLEMA 6
n −1
Seja f ( z ) = an z + an −1 z + ... + a1 z + a0 um polinômio com coeficientes
n

reais. Prove que se as raízes de f estão no semi-plano esquerdo


{z ∈ C | Re( z ) < 0} então ak ak +3 < ak +1ak + 2 para todo k = 0, 1,…, n – 3.

SOLUÇÃO
Podemos supor sem perda de generalidade que an = 1.
Nesse caso, f(z) pode ser fatorado como produto de monômios da forma z + a ou
z + a + bi com a > 0 (no segundo caso, se b ≠ 0, deve aparecer também o fator
z + a − bi ).
Como ( z + a + bi )( z + a − bi ) = z + 2az + a + b e
2 2 2

( z + a) + ( z + a) = z 2 + (a + a) z + aa, temos que f(z) pode ser obtido a partir


de um polinômio de grau 0 ou 1 com todos os coeficientes positivos por meio de
sucessivas multiplicações por polinômios da forma z + Az + B , com A, B > 0.
2

Em particular, todos os seus coeficientes são positivos.


Vamos agora proceder por indução: para n = 0 ou n = 1 o resultado vale por
vacuidade. Seja agora f(z) um polinômio de grau n + 2, da forma
f ( z ) = (an z n + ... + a0 )( z 2 + Az + B), com an = 1 e
f ( z ) = an z n + ... + a0 satisfazendo a hipótese de indução. Convencionando
ak = 0 se k < 0 ou k > n , e escrevendo f ( z ) = an + 2 z n + 2 + ... + a0 , temos,
para 0 ≤ k ≤ n + 2, ak = ak − 2 + Aak −1 + Bak . Queremos então provar que
(ak+1 + Aak+2 + Bak+3 )(ak−2 + Aak−1 + Bak ) < (ak + Aak+1 + Bak+2 )(ak−1 + Aak + Bak+1) ,
mas (ak + Aak+1 + Bak+2 )(ak−1 + Aak + Bak+1) − (ak+1 + Aak+2 + Bak+3)(ak−2 + Aak−1 + Bak )
= (ak ak−1 − ak+1ak−2 ) + B2 (ak+2ak+1 − ak+3ak ) + A2 (ak+1ak − ak+2ak−1) +
+B(ak+2ak−1 − ak+3ak−2 ) + A(ak2 − ak+2ak−2 ) + AB(ak2+1 − ak+3ak−1).
Temos, por hipótese de indução, ak ak−1 ≥ ak+1ak−2 , ak+2ak+1 ≥ ak+3ak , ak+1ak ≥ ak+2ak−1
e, além disso, ak+3ak ≤ ak+2ak+1 e ak+1ak−2 ≤ ak−1ak , donde
ak ak+1ak+3ak−2 ≤ ak ak+1ak+2ak−1, e logo ak+3ak−2 ≤ ak+2ak−1 (de fato temos ak ak+1 > 0 ,
a menos que k = n, quando ak +3ak−2 = ak+2ak−1 = 0) ; ak +2ak−1 ≤ ak+1ak e

EUREKA! N°18, 2003

28
Sociedade Brasileira de Matemática

ak+1ak−2 ≤ ak ak−1 , donde ak−1ak+1ak+2ak−2 ≤ ak−1ak+1ak2 , e logo ak+2ak−2 ≤ ak2 , e


analogamente ak+3ak−1 ≤ ak+1, sendo que pelo menos uma dessas duas últimas
2

desigualdades é estrita, o que conclui a prova.

SEGUNDO DIA

PROBLEMA 1
Sejam A e B matrizes reais n × n tais que AB + A + B = 0. Prove que AB = BA.

SOLUÇÃO
AB + A + B = 0 ⇒ AB + A + B + I = I ⇒ A( B + I ) + ( B + I ) = I ⇒ ( A + I )( B + I ) = I
Logo, A + I e B + I são inversas uma da outra, donde
( A + I )( B + I ) = ( B + I )( A + I ) = I .
Expandindo a última desigualdade vem BA + B + A + I = I e subtraindo esta da
igualdade dada no enunciado obtém-se AB = BA .

PROBLEMA 2
2x sen m t
Calcule o seguinte limite: lim+
x →0 ∫
x tn
dt (m, n naturais dados).

SOLUÇÃO
sen t
Como a função é decrescente em (0, π) , e tende a 1 quando t tende a 0+,
t
π sen(2 x) sen t
temos que, para x ∈ (0, ) e x < t < 2x: < < 1 e portanto:
2 2x t
m m 2 x sen m t 2x t m
 sen(2 x)  2 x t
  ⋅ ∫ n dt < ∫ n
dt < ∫ n dt .
 2x  x t x t x t
m
 sen(2 x) 
Como lim x →0   = 1 , a desigualdade acima mostra que o limite
 2x 
procurado é igual a

EUREKA! N°18, 2003

29
Sociedade Brasileira de Matemática

 t m − n +1 2 x
 x →0
→ 0, se m − n + 1 > 0
m − n +1 x
2x
 m − n +1 2 x
 t
lim x →0 ∫ t dt = 
m−n
x →0
→∞, se m − n + 1 < 0
x  m − n +1 x
 2x
 ln t x  x →0
→ ln 2, se m − n + 1 = 0


PROBLEMA 3
Seja A um subconjunto fechado de Rn e seja B o conjunto de todos os pontos b de
Rn tais que existe exatamente um ponto a0 em A tal que a0 − b = inf a − b .
a∈ A
Prove que B é denso em Rn.

SOLUÇÃO
Vamos mostrar que dado ε > 0 e x ∈ n , existe y ∈ B( x, ε) ∩ B .
Se x ∈ A , então basta tomar y = x.
Caso contrário, seja δ = inf a∈A | a − x | . Como A é fechado, existe um ponto
a ∈ A que realiza essa distância (basta observar por exemplo que B ( x, 2δ ) ∩ A é
compacto).
Se esse ponto a não for único, considere um ponto y = x + t ( a − x), t ∈ (0,1) do
segmento (x, a).
Seja a’ um outro ponto de A. Se a’ estiver no prolongamento desse segmento de
reta, então claramente | a '− y | > | a − y | .
Caso contrário, temos a desigualdade triangular estrita | a ' − x | < | a ' − y | + | y − x |
e portanto:
| a '− y | > | a'− x | − | x − y | ≥ | a − x | − | x − y |=| a − y | ,
a
onde a 2 desigualdade usa que a é um ponto de A tal que |a – x| é mínimo e a
igualdade final usa o fato que y está no segmento de reta (x, a).
Ou seja, todo ponto y escolhido dessa forma está em B. Escolhendo t
ε
suficientemente pequeno ( t = por exemplo) obtemos um ponto
2⋅ a − x
y ∈ B( x, ε) ∩ B como desejado.

PROBLEMA 4

EUREKA! N°18, 2003

30
Sociedade Brasileira de Matemática

Encontre todos os inteiros positivos n para os quais existe uma família F de


subconjuntos de três elementos de S ={1, 2, …, n} que satisfaz as seguintes
condições:
(i) para quaisquer elementos distintos a, b ∈ S existe exatamente um A ∈ F
tal que a, b ∈ A.
(ii) Se a, b, c, x, y, z são tais que {a, b, x}, {a, c, y}, {b, c, z} ∈ F então
{x, y, z} ∈ F.

SOLUÇÃO
Vamos mostrar que uma tal família F existe se e somente se n = 2 k − 1 para algum
inteiro positivo k.
De fato, se G = ( 2 ) = 2 × 2 × ... × 2 , onde está definida a adição
k

( x1 , x2 ,..., xn ) + ( y1 , y2 ,..., yn ) = ( x1 + y1 (mod 2), x2 + y2 (mod 2),..., xn + yn (mod 2)) ,


podemos definir S = G \ {(0,0,...,0)} e F = {{u , v, u + v}, u , v ∈ S , u ≠ v}.
A propriedade (i) segue com A = {a, b, a + b} e a propriedade ii) segue de
x = a + b , y = a + c ⇒ x + y = a + a + b + c = b + c = z.
Sejam agora S e F como no enunciado. Consideramos um conjunto G = S ∪ {0} ,
onde 0 é um elemento de G que não pertence a S (um "zero artificial") e uma
operação + definida em G por
 x se a, b ∈ S , a ≠ b e {a, b, x} ∈ F
0 se a = b

a+b=
 a se b = 0
b se a = 0
Não é difícil ver que com essa operação G é um grupo abeliano ( a + b = b + a
para quaisquer a, b em G, (a + b) + c = a + (b + c), para quaisquer a, b, c ∈ G e
para todo a ∈ G existe b em G com a+b=0. Segue que G é isomorfo a
( 2 ) para algum inteiro positivo k * , donde n = # S = # G − 1 = 2k − 1 , o que
k

conclui a solução.
* De fato, se a ∈ G , a ≠ 0, H = {0, a} é um subgrupo de G isomorfo a 2 , e, se
definimos em G a relação de equivalência x ~ y ⇔ y − x ∈ H , obtemos um
quociente, G/H, o conjunto das classes de equivalência x = { y ∈ G | y ~ x}, que é
um grupo com a operação x + y : = x + y , o qual têm as mesmas propriedades que
G. Além disso, G é naturalmente isomorfo a ( G H ) × H , e, por indução em #G ,
( ) ( )
r r +1
G H é isomorfo a 2 para algum r ∈ , donde G é isomorfo a 2 .

EUREKA! N°18, 2003

31
Sociedade Brasileira de Matemática

Obs.: O conjunto S é um espaço projetivo finito (a generalização k-dimensional do plano


projetivo) sobre o corpo 2 (ou seja, de ordem 2), cuja família das retas é F. De fato, a
propriedade (i) é equivalente a "por dois pontos passa uma única reta" e a propriedade (ii)
é equivalente ao axioma de Veblen-Young: "dado um triângulo ABC, se uma reta r corta
dois lados, então corta o terceiro lado também" (no caso, os pontos do triângulo são a,b,c e
a reta é r={x,y,z}). As propriedades (i) e (ii) são, então, equivalentes aos axiomas que
definem um espaço projetivo; veja o artigo "Aplicações de planos projetivos finitos em
Teoria dos Números e Combinatória", de Carlos Shine, na Eureka! 15.

PROBLEMA 5
a) Mostre que para toda função f : Q × Q → R existe uma função g : Q → R tal
que f ( x, y ) ≤ g ( x) + g ( y ), ∀x, y ∈ Q .
b) Encontre uma função f : R × R → R para a qual não existe g : R → R tal
que f(x, y) ≤ g(x) + g(y), ∀x, y ∈ R.

SOLUÇÃO
a) Q é enumerável, digamos Q = {r1 , r2 , r3 ,...}.
Assim, podemos definir g : Q → R por g ( rn ) = max{ f ( ri , rj ) ,1 ≤ i , j ≤ n}.
Assim, f ( ri , rj ) ≤ f ( ri , rj ) ≤ g ( rmax( i , j ) ) ≤ g ( ri ) + g ( rj ), ∀i, j.
0, se x = y

b) Podemos definir f ( x, y ) =  1
 x − y , se x ≠ y.

Se existisse g : → tal que f ( x, y ) ≤ g ( x ) + g ( y ), ∀x, y ∈ , se definirmos,

para cada inteiro positivo n, X n = { x ∈ | g ( x ) ≤ n 2}, teremos ∪X
n =1
n = ,

donde, como é não-enumerável, algum dos X n deve ser não enumerável, e


portanto tem pontos de acumulação, isto é, existe uma seqüência de termos
distintos ( y k ) k ∈ com yk ∈ X n para todo k tal que ( yk ) converge a um certo
1
x ∈ . Em particular, lim yk +1 − yk = x − x = 0 , e logo f ( yk , yk +1 ) =
k →∞ yk +1 − yk
n n
tende a +∞ , mas devemos ter f ( yk , yk +1 ) ≤ g ( yk ) + g ( yk +1 ) ≤ + = n para todo
2 2
k∈ , pois { yk , yk +1} ⊂ X n , donde ( f ( y k , yk +1 )) k ∈ é limitada, absurdo.

EUREKA! N°18, 2003

32
Sociedade Brasileira de Matemática

PROBLEMA 6
1 n ak
Seja a0 , a1 ,..., an ,... a seqüência definida por a0 = 1 , an+1 = ∑
n + 1 k =0 n − k + 2
.

ak
Calcule ∑2
k =0
k (se existir).

SOLUÇÃO
Os an são positivos e, por indução, tem-se an ≤ 1, ∀n (de fato, supondo válido até
n tem-se
1 n 1 1 n
an +1 ≤ ∑ ≤ ∑1 = 1 ).
n + 1 k =0 n − k + 2 n + 1 k =0

Considere então a função geratriz f ( x) = ∑ an x n (ele é convergente para 0 < x <
n =0
1 pela observação acima). Derivando e usando a expressão dada obtemos:
∞ ∞ ∞  n ak  n
f ' ( x) = ∑ n ⋅ an ⋅ x n −1 = ∑ (n + 1) ⋅ an +1 ⋅ x n = ∑  ∑  x
n =1 n =0 n =0  k =0 n − k + 2 
Trocando a ordem do somatório obtém-se:
∞  ∞ ak x n   ∞   ∞ x n − k  ∞ xm
f ' ( x) = ∑  ∑ =  ∑ ak x k  ⋅  ∑ = f ( x) ⋅ ∑
 
k =0  n = k n − k + 2   k =0   n = k n − k + 2  m=0 m + 2
Portanto,
x f ' ∞ x m +1 ∞  x m +1 x m +1 
ln( f ( x)) − ln( f (0)) = ∫ = ∑ = ∑  −

m = 0 ( m + 1)(m + 2) m = 0  ( m + 1) ( m + 2) 

0 f
Como f (0) = 1:
xm+1 1 ∞ xm+2
∞ ∞ xm+1 1 ∞ xm+1  1  1 1 
ln( f ( x)) = ∑ − ∑ = ∑ − ∑ =  ln  −  ln − x
m=0 m + 1 x m=0 m + 2 m=0 m + 1 x m=1 m + 1  1 − x  x  1 − x 

1
Colocando x = obtemos
2

1 an 1 e
ln f ( ) = ln 2 − 2 ln 2 + 1 = 1 − ln 2 , de modo que
2
∑2
n =0
n
= f ( ) = e ⋅ e − ln 2 = .
2 2

Estas soluções da International Mathematical Competition - 2003 foram redigidas por Márcio
Assad Cohen, Rodrigo Villard Milet e Carlos Gustavo Moreira do Rio de Janeiro – RJ.

EUREKA! N°18, 2003

33
Sociedade Brasileira de Matemática

XVIII OLIMPÍADA IBEROAMERICANA DE MATEMÁTICA


13 a 20 de setembro, Mar del Plata - Argentina

PRIMEIRO DIA

PROBLEMA 1
a) Têm-se duas sucessões, cada uma de 2003 inteiros consecutivos, e um
tabuleiro de 2 linhas e 2003 colunas

………
………

Decida se é sempre possível distribuir os números da primeira sucessão na


primeira linha e os da segunda sucessão na segunda linha, de modo que os
resultados obtidos ao somar os dois números de cada coluna formem uma nova
sucessão de 2003 números consecutivos.

b) E se trocássemos 2003 por 2004?


Tanto em a) como em b), se a resposta for afirmativa, explique como distribuiria
os números, e se for negativa, justifique o porquê.

SOLUÇÃO
Note que somar ou subtrair uma constante de uma sucessão de números
consecutivos a transforma em uma sucessão de números consecutivos.
Note também que somar ou subtrair uma constante de uma linha do tabuleiro
soma ou subtrai a mesma constante da sucessão formada pela soma das colunas,
logo esta operação não altera a "consecutividade" das linhas do tabuleiro.
Portanto, sem perda de generalidade, as sucessões escritas nas duas primeiras
linhas do tabuleiro são 1, 2, …,n onde n ∈ {2003, 2004}
a) Sim. Escreva na primeira linha 1, 2,…,2003 e na segunda linha 1002,
1003,…,2003, 1, 2, …, 1000, 1001, ou seja, o i-ésimo termo é i + 1001 se
i ≤ 1002. A seqüência final é 2i + 1001 se i ≤ 1002 e 2i − 1002 se
i ≥ 1003 , que é obviamente permutação de 1003, 1004, 1005,…,3003, 3004,
3005.

b) Não. Uma seqüência de 2004 números inteiros consecutivos tem forma


k + 1,..., k + 2004, k ∈ . Sua soma vale

EUREKA! N°18, 2003

34
Sociedade Brasileira de Matemática

2004 ⋅ 2005
2004k + = 2004k + 2004 ⋅ 1002 + 1002 ≡ 1002(mod 2004).
2
A soma dos números da primeira e da segunda linhas vale 1002 (mod 2004).
Como a terceira linha (formada pela soma das colunas) é formada pela soma das
duas primeiras linhas, a soma das números da terceira linha é
1002 + 1002 ≡ 0 (mod 2004). Mas se a terceira linha fosse composta por uma
sucessão de números consecutivos em alguma ordem, sua soma seria
1002 (mod 2004), absurdo! Logo a soma das colunas não pode formar uma
sucessão de números consecutivos.

PROBLEMA 2
Sejam C e D dois pontos da semicircunferência de diâmetro AB tais que B e C
estão em semiplanos distintos em relação à reta AD. Denotemos por M, N e P os
pontos médios de AC, DB e CD, respectivamente. Sejam OA e OB os circuncentros
dos triângulos ACP e BDP. Demonstre que as retas OAOB e MN são paralelas.

SOLUÇÃO

D
T π
P . +γ
S . 2
C .
π 2β
−β π .N
2 −γ
M. π 2
2γ −β .Q .
2 .
R. γ O2
β
γ β
A O B

Sejam 2α , 2 β e 2γ as medidas de CD, BD e AC , respectivamente. Seja O o


centro da semicircunferência ACDB. Sejam R e Q os pés das perpendiculares a
OP que passam por O2 e O1 , respectivamente.
Sejam ainda S e CP , T e PD pontos médios de CP e PD, respectivamente.

EUREKA! N°18, 2003

35
Sociedade Brasileira de Matemática

π
Note que SCM = β + pois é ângulo inscrito. Note ainda que MOA ⊥ AC e
2
SOA ⊥ CP , pois são mediatrizes de AC e CP . Logo MCSOA é inscritível e
π
MOA S = −β .
2
Como SOA ⊥ OAQ (pois SOA ⊥ CP ⊥ PO , logo PO // SOA ), QOAO = β .
OAQ SP CD CD
Logo = = = cos β ⇔ OAO = .
OAO OAO 4OAO 4cos β
CD
Analogamente, OB O = . Olhando para o triângulo OMA, MOA = γ
4cos γ
(pois M é ponto médio da corda AC , logo MOA = AC 2 ), logo
OM OM
= = cos γ ⇔ OM = R cos γ . Analogamente, ON = R cos β . Mas
OA R
OM R cos γ ⋅ 4cos β R cos β ON
= = = . Logo existe uma homotetia de
OA O CD CD OB O
4cos γ
centro O que leva M em OA e N em OB . Como homotetias preservam
paralelismo, MN // OAOB .

PROBLEMA 3
Pablo copia o seguinte problema:
Considere todas as sucessões de 2004 números reais ( x0 , x1 , x2 ,..., x2003 ),
tais que
x0 = 1,
0 ≤ x1 ≤ 2 x0 ,
0 ≤ x2 ≤ 2 x1 ,

0 ≤ x2003 ≤ 2 x2002 .
Entre todas estas sucessões, determine aquela para a qual a expressão seguinte
assume o seu maior valor: S = … .

EUREKA! N°18, 2003

36
Sociedade Brasileira de Matemática

Quando Pablo ia copiar a expressão S, apagaram o quadro. Só conseguia lembrar-


se de que S era da forma
S = ± x1 ± x2 ± ... ± x2002 + x2003 ,
onde o último termo, x2003 , tinha coeficiente +1, e os anteriores tinham coeficiente
+1 ou –1. Demonstre que Pablo, apesar de não ter o enunciado completo, pode
determinar com certeza a solução do problema.

SOLUÇÃO
Seja ci ∈ {−1,1} o coeficiente associado ao termo xi na expressão de S. Em
particular, c2003 = 1 . Dizemos que um termo xi é positivo se ci = 1 , ou que é
negativo se ci = −1 . Dizemos que xi está maximizado se xi = 2 xi −1 .
Lema: Se xi +1 ,..., x j estão maximizados, então:

ci xi + ci +1 xi +1 + ... + c j x j = xi (ci 20 + ci +1 21 + ... + c j 2 j −i )


k −i
Prova: É fácil ver que xk = 2 xi , i ≤ k ≤ j. A prova segue trivialmente.
Corolário: A soma acima tem o mesmo sinal de c j .
j −i
Prova: Como xi ≥ 0 , basta analisar o sinal de ci 2 + ... + c j 2
0
. Suponha c j
j j −1

positivo (i.e. = 1). Então ∑ ck 2 ≥ ∑ −2 + 2 = −2 + 1 + 2 = 1. O


k −i k −i j −i j −i j −i

k =i k =i

caso c j = −1 é análogo.
É obvio que maximizar os termos positivos aumenta a soma (já que
x ≤ y ⇒ [0, 2 x] ⊆ [0, 2 y ] , podemos aumentar um termo sem alterar nenhum
dos seguintes). Logo, sem perda de generalidade, na seqüência que maximiza S
todos os termos positivos são máximos.
Se houver algum termo negativo não maximizado, escolha o último deles,
digamos xi e maximize tanto ele quanto todos os termos que o seguem. Sejam a e
b os valores antigo e novo de xi e A e B os valores da soma
ci +1 xi +1 + ... + c2003 x2003 antes e depois da mudança na seqüência.
2003− i
Seja ainda C = 2 ci + 2 ci +1 + ... + 2
0 1
c2003 . Pelo lema, A = aC e B = bC.
EUREKA! N°18, 2003

37
Sociedade Brasileira de Matemática

Logo B – A = C(b – a) > 0, pois, pelo corolário, C > 0, e como b > a, b – a > 0.
Logo, na seqüência de S máximo, todos os termos são máximos ou poderíamos
aumentar S maximizando algum termo.
Logo x1 = 2 x0 = 2, x2 = 2 x1 = 2 2 ,..., xi = 2 xi −1 = 21 ,..., x2003 = 22003 , e
0 1 2003
portanto a seqüência que maximiza S é (2 , 2 ,..., 2 ).

SEGUNDO DIA

PROBLEMA 4
Seja M ={1, 2,…,49} o conjunto dos primeiros 49 inteiros positivos. Determine o
maior inteiro k tal que o conjunto M tenha um subconjunto de k elementos em que
não haja 6 números consecutivos. Para esse valor máximo de k, encontre a
quantidade de subconjuntos de m, de k elementos, que tenham a propriedade
mencionada.

SOLUÇÃO
Definição: Um conjunto A ⊆ M é feliz se não contém seis inteiros
consecutivos.
Seja N ⊆ M feliz, | N |≥ 42. Então se P = M − N ,| P |≤ 7. Logo a
interseção de P com algum dos conjuntos
A1 = {1, 2,3, 4,5,6} , A2 = {7,8,9,10,11,12} , A3 = {13,14,15,16,17,18}

A7 = {37,38,39, 40, 41, 42} , A8 = {43, 44, 45, 46, 47, 48, 49}
é vazia. Chame de i algum inteiro tal que Ai ∩ P = ∅ . Então Ai ⊂ N , absurdo,
pois então N teria uma seqüência de seis inteiros consecutivos. Por outro lado, é
óbvio que N = M − {6,12,18, 24,30,36, 42, 48} é feliz.
Assim, o maior k tal que existe N ⊆ M feliz é 41 (como se | N |≥ 42 então N
não é feliz, | N |≤ 41; e acabamos de exibir um exemplo para 41).
Seja N ⊆ M feliz, | N |= 41, P = M − N = {n1 < n2 < ... < n8 }.
Os oito elementos de P separam naturalmente N em nove conjuntos
N1 ,..., N9 , Ni ∩ N j = ∅, ∪N i = N , N i ≤ 5 e cada N i composto apenas
de números consecutivos. É fácil ver que o número de possíveis conjuntos N é o
número de soluções de | N1 | + | N 2 | +...+ | N 9 |= 41 (pois estamos escolhendo o

EUREKA! N°18, 2003

38
Sociedade Brasileira de Matemática

tamanho dos N i ' s ) onde | N i |≤ 5 e exigimos que os Ni estejam ordenados pelos


seus menores elementos. Seja ai =| N i | .
Seja bi = 5 − ai ⇔ 5 − bi = ai . Como 0 ≤ ai ≤ 5,0 ≤ bi ≤ 5. Substituindo na equação,
5 − b1 + 5 − bi + ... + 5 − b9 = 41 ⇔ 45 − (b1 + ... + b9 ) = 41 ⇔ b1 + ... + b9 = 4. Mas
então a restrição bi ≤ 5 é redundante, logo o número de N ⊆ M ,| N |= 41, N feliz, é
o número de soluções de b1 + ... + b9 = 4 nos inteiros não negativos, que é
 9 + 4 − 1 12  12 ⋅ 11 ⋅ 10 ⋅ 9
 = = = 495.
 4  4  24

PROBLEMA 5
No quadrado ABCD, sejam P e Q pontos pertencentes aos lados BC e CD
respectivamente, distintos dos extremos, tais que BP = CQ. Consideram-se pontos
X e Y, X ≠ Y, pertencentes aos segmentos AP e AQ respectivamente. Demonstre
que, quaisquer que sejam X e Y, existe um triângulo cujos lados têm os
comprimentos dos segmentos BX, XY e DY.

SOLUÇÃO

A 1 B
a
X
λ

1 b
y

1–λ

c
Y
D 1–λ Q λ C

Seja AB = BC = CD = DA = 1, BP = CQ = λ , BX = a, XY = b, YD = c.

EUREKA! N°18, 2003

39
Sociedade Brasileira de Matemática

Como a + b ≥ y = BY , basta provar que y > c para demonstrar que


a + b > c. Mas a mediatriz de BD é a reta AC , que divide o plano em dois
semiplanos β ∋ B e δ ∋ D . À exceção do ponto A, todo o segmento AQ está
contido em δ , logo Y ∈ δ ⇔ y > c (o caso Y ≡ A é trivial). Logo
a + b > c e, analogamente, c + b > a. Basta provar que a + c > b
Seja
A = (0,0); B = (1,0); C = (1,1); P = (1,α ); Q = ( β ,1); X = ( x,α x);
D = (0,1); y = ( y β , y ) , onde α + β = 1 (para que BP = CQ ).
Então a = ( x − 1) 2 + α 2 x 2 , c = β 2 y 2 + ( y − 1) 2 ,
b = ( x − y β ) 2 + (α x − y ) 2 = x 2 − 2 xy β + y 2 β 2 + α 2 x 2 − 2α xy + y 2 =
b = x 2α 2 + y 2 β 2 + x 2 + y 2 − 2 xy (α + β ) = x 2α 2 + y 2 β 2 + ( x − y ) 2 .
Mas b < a + c ⇔ b < a + 2ac + c
2 2 2
(já que a, b, c são positivos). Basta
demonstrar que a + c ≥ b , já que 2ac > 0. Logo basta provar que
2 2 2

( x − 1) 2 + α 2 x 2 + ( y − 1) 2 + β 2 y 2 ≥ x 2α 2 + y 2 β 2 + x 2 − 2 xy + y 2 ⇔
⇔ x 2 − 2 x + 1 + y 2 − 2 y + 1 − x 2 + 2 xy − y 2 ≥ 0 ⇔ (1 − x)(1 − y ) ≥ 0. Mas
x ∈ AP , logo X ∈ [0,1] ⇔ 1 − x ∈ [0,1]. Analogamente, 1 − y ∈ [0,1] e o
resultado segue trivialmente.

PROBLEMA 6
Definem-se as sucessões ( an ) n ≥ 0 ,(bn ) n ≥ 0 por:
a0 = 1, b0 = 4 e
an +1 = a 2001
n + bn , bn +1 = bn2001 + an para n ≥ 0.

Demonstre que 2003 não divide nenhum dos termos destas sucessões.
SOLUÇÃO
Observe que 2003 é primo; logo, se
x ≡/ 0 (mod 2003), x 2002 ≡ 1(mod 2003) ⇔ x 2001 ≡ x −1 (mod 2003).

EUREKA! N°18, 2003

40
Sociedade Brasileira de Matemática

Suponha que exista n tal que 2003 | an +1bn +1 (note que


a0b0 = 4 ≡/ 0(mod 2003) ); escolha o menor deles. Suponha que 2003 | an +1 (o
outro caso é análogo).
Temos 0 ≡ an +1 ≡ an
2001
+ bn ≡ an−1 + bn (mod 2003),
−1
logo an ≡ −bn (mod 2003). Como nem an nem bn são zero (mod 2003),
podemos inverter os dois lados, obtendo
an ≡ −bn−1 ⇔ bn−1 ≡ − an (mod 2003) (usamos o fato de que (−1) −1 ≡ −1). Mas
bn +1 ≡ bn−1 + an ≡ − an + an ≡ 0(mod 2003), logo 2003 | bn +1 ⇔ 2003 | an +1.
−1
É fácil ver que, nesse caso, an bn ≡ an (− an ) ≡ −1(mod 2003). Seja c = an −1 e
d = bn −1 (para n = 0, temos a1 = 12001 + 4 = 5 ≡/ 0(mod 2003) , logo podemos
−1 −1
supor n ≥ 1). Então an ≡ c + d e bn ≡ c + d (mod 2003), logo
an bn ≡ c −1c + c −1d −1 + dc + dd −1 ≡ 2 + cd + (cd ) −1 (mod 2003). (Como n é
−1
o menor possível, 2003 não divide ai bi para todo i ≤ n , logo existe (cd ) ).
Seja x = cd.
−1 ≡ 2 + x + x −1 ⇔ x 2 + 3x + 1 ≡ 0(mod 2003). Esta equação tem uma raiz
(que é cd), logo seu discriminante ∆ = 3 − 4 ⋅ 1 ⋅ 1 = 5 é resíduo quadrático (mod
2

2003) : de fato,
(2 x + 3)2 = 4( x 2 + 3x + 1) + 5 ≡ 5(mod 2003). Mas pela lei de Reciprocidade
2003−1 5−1
Quadrática, 
5  2003 ⋅
 = (−1) ⇔
2 2

 2003 5 
 5   3  5   5 
 ⋅   = (−1) ⇔  ⋅ (−1) =1⇔  =−1, logo 5 não é
2002
 resíduo
 2003  5   2003  2003
quadrático módulo 2003, absurdo! Assim, não é possível que
2003 | an +1bn +1 , logo {2003 x | x ∈ } ∩ ({ak }k∈ ∪ {bk }k∈ ) = ∅.

Estas soluções da Olimpíada Ibero-americana de Matemática - 2003 foram redigidas por


Fábio Dias Moreira de Rio de Janeiro – RJ.

EUREKA! N°18, 2003

41
Sociedade Brasileira de Matemática

A DESIGUALDADE DE ERDÖS-MORDELL
Anderson Torres, São Paulo - SP

♦ Nível Avançado

Neste artigo demonstraremos (várias vezes) a desigualdade de Erdös-Mordell


e mostraremos uma bela aplicação na resolução do problema 5 da IMO de 1996,
realizada em Mumbai, Índia.

1- Uma história do teorema


“Considere um triângulo ABC e um ponto P do mesmo plano. Sejam PA , PB , PC
as projeções ortogonais de P nos lados BC, CA, AB respectivamente. Vale então a
desigualdade:
2( PPA + PPB + PPC ) ≤ AP + BP + CP

com igualdade se e somente se P for o circuncentro de um triângulo ABC


eqüilátero”.

Este é o enunciado da famosa Desigualdade de Erdös-Mordell. Ela foi


inicialmente conjecturada pelo matemático húngaro Paul Erdös e demonstrada no
mesmo ano por Louis Mordell, na revista American Mathematical Monthly
(problema n° 3740). Logo após surgiram várias soluções e alguns artigos sobre a
desigualdade, cada uma usando variadas técnicas: trigonometria (Louis J. Mordell
e P.F. Barrow), desigualdades angulares e semelhanças (Leon Bankoff), teorema
de Ptolomeu (André Avez e Hojoo Lee), áreas de polígonos (V. Komornik).

Mostraremos algumas delas, acrescidas de um pequeno comentário:

Lema Importante: AP ⋅ BC ≥ AB ⋅ PPB + AC ⋅ PPC ,


com igualdade se, e somente se, PB PC // BC .

A esmagadora maioria das demonstrações difere apenas na demonstração desta


pequena desigualdade. Veja que esta desigualdade equivale a estas desigualdades
(as outras duas seguem por permutação cíclica das variáveis):

EUREKA! N°18, 2003

42
Sociedade Brasileira de Matemática

AB AC
AP ≥ ⋅ PPB + ⋅ PPC
CB BC
CB BA
BP ≥ ⋅ PPC + ⋅ PPA
AC CA
CA CB
CP ≥ ⋅ PPA + ⋅ PPB
BA AB

Ao somá-las, obtemos:
 CA BA   CB AB   AC BC 
AB + BP + CP ≥  +  ⋅ PPA +  +  ⋅ PPB +  +  ⋅ PPC
 BA CA   AB CB   BC AC 

e lembrando que a soma de um real positivo com seu inverso não pode ser menor
que 2, e esse valor só se iguala a dois se o número em questão for 1 (isto é
conseqüência da desigualdade das médias), a desigualdade segue, com igualdade
se e apenas se AB = BC = CA. Além disso, devemos ter PA PB // AB , PA PC // AC
e PB PC // BC , o que implica facilmente que P é o circuncentro do triângulo
ABC.

Vamos então demonstrar este lema!

Demonstração 1: (trigonometria)

O quadrilátero APB PPC é cíclico, pois os ângulos retos são opostos e somam
180°. Assim, pela Sagrada Lei dos Senos Generalizada,
PB PC PB PC
AP = = ⇔
sen ∠PB PPC sen ∠BAC
AP ⋅ BC = 2R ⋅ PB PC

em que R é o circunraio do triângulo ABC.

EUREKA! N°18, 2003

43
Sociedade Brasileira de Matemática

PB
PC

B PA C

Pela Sagrada Lei dos Cossenos,


PB PC ² = PPB ² + PPC ² − 2 ⋅ PPB ⋅ PPC ⋅ cos (∠PB PPC )
= PPB ² + PPC ² − 2 ⋅ PPB ⋅ PPC ⋅ cos (∠PA PPC + ∠PA PPB )
,
= ( PPB ⋅ sen ∠PB PPA + PPC ⋅ sen ∠PC PPA )²
+( PPB ⋅ cos∠PB PPA − PPC ⋅ cos ∠PC PPA )²
onde usamos o fato (bastante conhecido ☺): sen ² x + cos ² x = 1 para fatorar.
Assim, jogando um dos parênteses fora, obtemos:

PB PC ≥ PPB ⋅ sen ∠PB PPA + PPC ⋅ sen ∠PC PPA ⇔


PB PC ≥ PPB ⋅ sen ∠ACB + PPC ⋅ sen ∠ABC ⇔
AP ⋅ BC = 2 R ⋅ PB PC ≥ PPB ⋅ AB + PPC ⋅ AC

A igualdade ocorre se, e somente se,


PPB ⋅ cos ∠PB PPA − PPC ⋅ cos ∠PC PPA = 0 ⇔
2 ⋅ sen ∠PAB ⋅ cos ∠ABC = 2 ⋅ sen ∠PAC ⋅ cos ∠ACB ⇔
sen(∠PAB + ∠ABC ) + sen(∠PAB − ∠ABC ) = sen(∠PAC + ∠ACB ) + sen(∠PAC − ∠ACB );
mas ∠PAB + ∠ABC + ∠PAC + ∠ACB = 180° ,donde
sen(∠PAB + ∠ABC ) = sen(∠PAC + ∠ACB), e
logo sen(∠PAC − ∠ACB) = sen(∠PAB − ∠ABC )
∠PAB − ∠ABC = ∠PAC − ∠ACB ⇔
∠PAB − ∠PAC = ∠ABC − ∠ACB

EUREKA! N°18, 2003

44
Sociedade Brasileira de Matemática

e (fica como exercício mostrar que) isto equivale, de fato, a PB PC // BC .

Demonstração 2: (áreas de paralelogramos)


Escolha dois pontos B1 ∈ AC , C1 ∈ AB e construa os paralelogramos APC 'C1 e
APB ' B1 .
PB' , PC ' cortam BC em X,Y e B1C1 em X 1 ,Y1 respectivamente, caso P seja
interno ao triângulo (mas isto não afeta muito a demonstração). Veja que
B1 B ' C ' C1 é um paralelogramo.
A

PC PB

B C
Y X

Y1 X1 B1
C1

B'
C'

Por congruências, [ AB1C1 ] = [ PB ' C ' ] , em que [algo] significa área de algo.
Agora, veja:
[ AB1C1 ] − [PX1Y1 ] + [C ' C1Y ] + [B ' B1 X1 ] = [ APC ' C1 ] + [ APB ' B1 ] ⇔
[PB ' C '] − [ PX1Y1 ] + [C ' C1Y1 ] + [B ' B1 X1 ] = [ APC ' C1 ] + [ APB ' B1 ]
.
[C 'Y1 X1B '] + [C ' C1Y1 ] + [B ' B1 X1 ] = [ APC ' C1 ] + [ APB ' B1 ]
[ APC ' C1 ] + [ APB ' B1 ] = [B1B ' C ' C1 ]
Com isto vemos que
AC1 ⋅ PPC + AB1 ⋅ PPB ≤ B1C1 ⋅ C ' C1 = AP ⋅ B1C1 , com igualdade se, e apenas se,
C ' C1 ⊥ B1C1 , ou AP ⊥ B1C1 , ou seja, AP contém o circuncentro do triângulo

EUREKA! N°18, 2003

45
Sociedade Brasileira de Matemática

AB1C1 . Fazendo AC1 = AC, AB1 = AB , teremos por congruências (para variar...)
BC = C1 B1 , e (por paralelismo mesmo!☺) PB PC // BC , e pronto! Fim!

Observação: Veja que é possível modificar esta demonstração apenas usando uma
reflexão pela bissetriz do ângulo ∠ABC para obter os pontos
AC1 = AC, AB1 = AB , B1 ∈ AC, C1 ∈ AB . Esta observação será útil mais tarde.

Demonstração 3: (teorema de Ptolomeu)


Sejam B' , C ' pontos da reta PB PC tais que BB ' // CC ' ⊥ PB PC . Então é fácil
ver que
BC ≥ B ' C ' = B ' PC + PC PB + PB C ' ⇔
,
AP ⋅ BC ≥ AP ⋅ B ' PC + AP ⋅ PC PB + AP ⋅ C ' PB
com igualdade se, e somente se, PB PC // BC .

B'
PC
PB

C'
P

B PA C

Vamos calcular cada uma das parcelas em relação ao ponto P.


Veja que ∆APPC ~ ∆CPB C ' pois os ângulos correspondentes são iguais. De fato,
temos os ângulos retos, e ∠APPC = ∠APB PC = ∠CPB C ' (quadrilátero cíclico e
ângulos opostos pelo vértice).
Assim, obtemos as relações:

EUREKA! N°18, 2003

46
Sociedade Brasileira de Matemática

C ' PB PC P
= ⇔ AP ⋅ C ' PB = PC P ⋅ PB C .
PB C AP
Analogamente,
B ' PC PB P
= ⇔ AP ⋅ B ' PC = PB P ⋅ PC B
PC B PA

Pelo Teorema de Ptolomeu-Euler,

PB PC ⋅ AP = APB ⋅ PPC + APC ⋅ PPB

Adicionando as igualdades, obtemos:

AP ⋅ BC ≥ AP ⋅ ( B ' PC + PC PB + PB C ')
= PC P ⋅ PB C + PC P ⋅ APB + PB P ⋅ APC + PB P ⋅ PC B
= PPC ⋅ AC + PPB ⋅ AB

2- Problema 5, IMO 1996 (Mumbai, Índia)

“Seja ABCDEF um hexágono convexo tal que AB é paralelo a DE, BC é paralelo


a EF, e CD é paralelo a FA.
Sejam R A , RC , RE os circunraios dos triângulos FAB, BCD, DEF
respectivamente, e seja P o perímetro do hexágono.
Prove que: R A + RC + RE ≥ P / 2 .”
Este foi um dos problemas mais difíceis (e é considerado o mais difícil por muitos
problemistas) já propostos na história da IMO. Para se ter uma idéia, apenas seis
participantes (dois romenos e quatro armênios) fecharam este problema, enquanto
os seis estudantes da equipe chinesa zeraram-no!
Mostraremos neste artigo duas soluções. A primeira é um esboço de como foi
criado o problema, segundo a Banca Examinadora da IMO de 1996 (o problema
foi proposto pela Armênia), segundo a referência [Nairi M. Sedrakian, The
History of a Creation of a 1996 IMO Problem, Mathematics Competitions, n° 2
vol.9], e se assemelha muito com a solução oficial, presente na Eureka! N° 11. A
segunda (com algumas modificações), totalmente sintética, considerada a mais
bela das soluções, é de autoria de Ciprian Manolescu, da equipe da Romênia, o
único Perfect Score (também conhecido como Ouro-42) da IMO 1996.

EUREKA! N°18, 2003

47
Sociedade Brasileira de Matemática

Solução 1: usaremos o seguinte lema (demonstre-o!):


“Considere um triângulo de circunraio R, lados a e b, e o ângulo γ entre eles.
Então, para quaisquer α , β tais que α < π , β > 0,α + β + γ = 2π , é válida a
sen β senα
desigualdade: 2 R ≥ a ⋅ +b⋅ ”
senγ sen γ
 π  π
(Sugestão: note que sen β = cos  β −  , sen α = cos  α −  , e considere uma
 2  2
π
reta fazendo um ângulo α − com o lado b; calcule a medida da projeção
2
ortogonal do lado do triângulo oposto ao ângulo γ nessa reta).
Usando este lema, podemos estimar os raios. Para tal sejam
α = ∠FAB, β = ∠BCD, γ = ∠DEF ,
FA = a, AB = b, BC = c, CD = d , DE = e, EF = f .
Com isto,
senγ sen β senγ senα senα sen β
2 RA ≥ b ⋅ + a⋅ ; 2 RC ≥ c ⋅ +d⋅ ; 2 RE ≥ e ⋅ + f⋅
senα sen α sen β sen β senγ senγ
Vamos tentar obter outra estimativa para 2 RA , desta vez em relação aos
lados d e e.
Pela Sagrada Lei dos Senos, BF = 2 RA ⋅ senα . Podemos então escrever
BF ≥ b ⋅ senγ + a ⋅ sen β . Veja que BF não pode ser menor que a distância entre as
retas BC e EF. Olhando este fato, vamos projetar o ponto A nas retas BC, EF
obtendo os respectivos pontos ABC , AEF . Analogamente para o ponto D,
obtemos o retângulo ABC AEF DEF DBC .
A BC B c C D BC
β d

b D

A α
e
a
γ
A EF F f E D EF

EUREKA! N°18, 2003

48
Sociedade Brasileira de Matemática

Com isto, podemos escrever:

BF ≥ b ⋅ sen γ + a ⋅ sen β
= AABC + AAEF = ABC AEF = DBC DEF = DDBC + DDEF
= d ⋅ sen β + e ⋅ sen γ

Concluímos as seguintes desigualdades:

sen β senγ senα senγ sen β senα


2 RA ≥ d ⋅ + e⋅ ; 2 RC ≥ a ⋅ + f⋅ ;2 RE ≥ c ⋅ + b⋅
senα sen α sen β sen β senγ senγ

Somando tudo:

4 ⋅ ( RA + RC + RE )
 senα sen β 
≥ (a + d ) + 
 sen β sen α 
 senα senγ 
+ (b + e)  + 
 senγ senα 
 sen β senγ 
+ (c + f ) + 
 senγ sen β 

E o problema segue, aplicando a Desigualdade das Médias aos parênteses e


dividindo tudo por 4. E fim!

Solução 2: Este problema, por si só, já incita o uso de Erdös-Mordell ou de


alguma generalização conveniente (muito provavelmente até às ultimas
conseqüências (☺)). Para tal, devemos de algum modo produzir a configuração
deste teorema. Aproveitando o paralelismo, desenhe os paralelogramos MDEF,
NFAB, PBCD. Com isto já temos algo dentro do hexágono (mesmo que não seja
um ponto, como em Erdös-Mordell, mas já é alguma coisa... Às vezes é
necessário um pouco de coragem para não desistir de algumas idéias, mesmo
que pareçam não dar certo. Muitos problemas de IMO e vários problemas
difíceis em geral são, na verdade, aplicações de fatos simples até às últimas
conseqüências).

EUREKA! N°18, 2003

49
Sociedade Brasileira de Matemática

B
F
M
N C
P

X D Z

O problema agora é tentar achar um modo de identificar os raios. Lembrando


que raios e diâmetros têm tudo a ver com perpendicularidade, desenhe o
triângulo XYZ, com XFY ⊥ FN , YBZ ⊥ BP, ZDX ⊥ DM . Assim, o quadrilátero
FMDX é inscritível de diâmetro MX. Mas os triângulos FED e FMD são
congruentes, logo XM = 2 ⋅ RA . Com isso o problema é demonstrar a seguinte
desigualdade:
XM + YN + ZP ≥ BN + BP + DP + DM + FM + FN .

Vamos dividir em dois casos:

1- M = N = P. E este caso é a própria Desigualdade de Erdös-Mordell.


2- O triângulo MNP existe (não é degenerado). A partir daqui vamos adaptar a
demonstração de Erdös-Mordell.
Estimaremos XM primeiro. Sejam Y’ e Z’ as reflexões dos pontos Y e Z em
relação à bissetriz de ∠YXZ . Sejam G e H as projeções de M, X em Y’Z’
respectivamente. Como [XYZ] = [Y’XZ’] = [Z’MY’] + [XMZ’] + [Y’MX],
temos:
YZ ⋅ XH = YZ ⋅ MG + ZX ⋅ FM + XY ⋅ DM .
Mas, usando a desigualdade triangular no triângulo XMG e a desigualdade
cateto < hipotenusa no triângulo XHG (ou mesmo distância de X à reta Y’Z’),
obtemos:
XM + MG ≥ XG ≥ XH ⇒ XM ≥ XH − MG

Substituindo na igualdade recém-descoberta,


XY XZ
XM ≥ ⋅ DM + ⋅ FM
YZ YZ

EUREKA! N°18, 2003

50
Sociedade Brasileira de Matemática

Analogamente,
XY XZ
XM ≥ ⋅ DM + ⋅ FM
YZ YZ
YX YZ
YN ≥ ⋅ BN + ⋅ FN
XZ XZ
ZX ZY
ZP ≥ ⋅ BP + ⋅ DP
XY XY
Somando tudo:
XM + YN + ZP
XY XZ
≥ ⋅ DM + ⋅ FM
YZ YZ
YZ YX
+ ⋅ FN + ⋅ BN
XZ XZ
ZX ZY
+ ⋅ BP + ⋅ DP
XY XY

Agora falta pouco...Basta arranjar um modo de sumir com as frações. Agora


vamos usar a Desigualdade das Médias para concluir. Para tal, outra estimativa.
Primeiramente, veja que os triângulos XYZ e MNP são semelhantes, o que nos
FM − FN BN − BP DP − DM
permite definir k = = = . Com isto, podemos
XY YZ ZX
escrever:

ZX YX
⋅ BP + ⋅ BN
XY XZ
 XZ XY  BN + BP
= +  +
 XY XZ  2
 XZ XY  BN − BP
 − 
 XY XZ  2
 XZ ⋅ YZ XY ⋅ YZ 
≥ ( BP + BN ) − k ⋅  − 
 XY XZ 

Analogamente,

EUREKA! N°18, 2003

51
Sociedade Brasileira de Matemática

XZ XY  XZ ⋅ YZ XY ⋅ YZ 
⋅ BP + ⋅ BN ≥ ( BP + BN ) − k ⋅  − 
XY XZ  XY XZ 
YX ZY  XY ⋅ XZ XZ ⋅ YZ 
⋅ DM + ⋅ DP ≥ ( DM + DP ) − k ⋅  − 
ZY YX  YZ XY 
XZ YZ  YZ ⋅ XY XZ ⋅ XY 
⋅ FM + ⋅ FN ≥ ( FM + FN ) − k ⋅  − 
YZ XZ  XZ YZ 

Agora, basta somar estas desigualdades e acabamos o problema!!

REFERÊNCIAS :

[1]A demonstração de Hojoo Lee pode ser encontrada na famosa revista Forum Geometricorum, a
qual você pode ler no site http://forumgeom.fau.edu .Neste artigo você encontra as
referências de toda a história deste problema enquanto ele se passava na American Mathematical
Monthly.

[2]Na lista de discussão de problemas da OBM (obm-l@mat.puc-rio.br; ver também


www.obm.org.br/lista.htm) foi deixada, há algum tempo atrás, a demonstração de
Ciprian Manolescu.

[3]Na Internet tem uns livros do Kiran Kedlaya.Vá ao site abaixo e faça o download:
http://www.unl.edu/amc/a-activities/a4-for-students/problemtext/
Um deles trata sobre desigualdades, e outro sobre geometria euclidiana plana. Ainda tem uns dois
livros com provas de algumas olimpíadas de matemática de várias partes do mundo.

[4]Após uma longa caça achei este artigo,que trata de uma generalização interessante:
A weighted Erdös-Mordell Inequality for Polygons. Este livro pode ser encontrado no endereço:
www.math.technion.ac.il/~shafrir/pub_ps/m18.ps.gz

[5]Um site de divulgação cientifica: http.://mathworld.wolfram.com

[6]Rafael Tajra Fonteles - Trigonometria e desigualdades em problemas de olimpíadas,


Eureka! 11, p. 24-33.

[7]A segunda demonstração da desigualdade de Erdös-Mordell também foi objeto de uma


questão da fase final da OPM-2001. Confira no livro da OPM-2001 ou no site
http://www.opm.mat.br/

EUREKA! N°18, 2003

52
Sociedade Brasileira de Matemática

COMO É QUE FAZ?

PROBLEMA 3
PROPOSTO POR DAVI MÁXIMO ALEXANDRINO NOGUEIRA (FORTALEZA – CE)
É possível escolher 102 subconjuntos com 17 elementos cada do conjunto
{1,2,3...102} tais que a interseção de quaisquer 2 deles tem no máximo 3
elementos?

SOLUÇÃO
A ideia é olhar para o plano projetivo P sobre Z/17Z, que é o quociente de
(Z/17Z)3\{(0,0,0)} pela relação de equivalência x ~ y ⇔ y = a ⋅ x,
2
∃a ∈ 17 \ {0}. P tem 17 + 17 + 1 = 307 pontos e as retas em P têm 18 pontos
cada (e duas delas sempre se intersectam num ponto). Vamos fazer uma espécie de
quociente de P. Para isso, considere o isomorfismo linear T de (Z/17Z)3 dado por
T(x, y, z) = (y, z, x). Note que T3 = Id. As retas em P são dadas por vetores não
nulos w de (Z/17Z)3 (de fato por elementos de P): uma reta Rw é o conjunto dos v
tais que < v, w > = 0. Note que T tem um único ponto fixo em P: o elemento v0 =
[1:1:1], correspondente ao vetor (1,1,1). Jogamos v0 fora e dividimos os outros
306 pontos de P em 102 classes de equivalência de 3 elementos (as orbitas de T): a
classe de equivalência de u é {u, Tu, T(Tu)}. Temos 307 retas em P. Jogamos fora
a Rv0 . O quociente de cada uma das outras retas tem 17 elementos. De fato, se Rw
contém pontos da forma u e Tu, devemos ter <u, w> = 0 e < u, T*w> = <Tu, w> =
0. Como w não é v0 então T*w = T–1(w) não é múltiplo de w, donde há apenas um
elemento de P satisfazendo essas duas igualdades, isto é, apenas dois pontos em
Rw são identificados pela nossa equivalência, donde as retas (projetadas pelo
quociente por essa relação de equivalência) têm agora 17 pontos cada. Para cada
reta na projeção, existem exatamente 3 retas (em P) que se projetam sobre ela: Rw,
RTw e RT (Tw) . Dadas duas retas na projeção, elas se intersectam em (no máximo) 3
pontos: se elas são as projeções de Rw e Rv, suas interseções serão as projeções das
interseções de Rw com Rv, de RTw com Rv e de RT(Tw) com Rv. Assim, na projeção
(ou, se você preferir, no quociente), temos 102 pontos e 102 retas, cada uma com
17 elementos, sendo que duas delas se intersectam em (no máximo) 3 pontos.

EUREKA! N°18, 2003

53
Sociedade Brasileira de Matemática

SOLUÇÕES DE PROBLEMAS PROPOSTOS


Publicamos aqui algumas das respostas enviadas por nossos leitores.

81) Num triângulo isósceles ABC com AB = BC, temos AC = BH, onde BH é a
altura relativa ao lado AC. Traçamos uma reta BD que corta o prolongamento
da reta AC em D de tal forma que os raios dos círculos inscritos nos triângulos
ABC e CBD são iguais. Determine o ângulo ABD ˆ .

SOLUÇÃO DE FRANCISCO JARDEL ALMEIDA MOREIRA (FORTALEZA – CE)

X
O1 O2

A H C F E D

Sejam, O1 o incentro do ∆ABC , O2 o incentro do ∆BCD , X = BC ∩OO


1 2 ,

E = BO2 ∩ AD e finalmente, F o pé da perpendicular de O2 até AD .

1 2 FH é um retângulo, pois O1 H F = O2 FH = 90° e O1 H = O2 F ,


Note que OO
daí O1O2 // AD.
Agora observe que:
HCO1 + BCO1 + O2 CB + DCO2 = 180° ademais, HCO1 = BCO1 e O2 CB = DCO2
portanto, O1 CO2 = 90° . Do fato de O1 CO2 = 90° e O1O2 // AD, X é médio de
O1O2 (de fato, X O1C = H CO1 = X CO1 , donde XC = XO1 e logo CX é
mediana do triângulo retângulo O1CO2 )
é médio de HE. Logo, HB = AC ⇒ HE = AC = HB.

EUREKA! N°18, 2003

54
Sociedade Brasileira de Matemática

Como BH E = 45° e assim ABD = 2 ⋅ H BE = 90°.

82)
a) Demonstre a identidade
n
sen(2n+1α)
cos(α) ⋅ cos(2α) ⋅ cos(4α )...cos(2nα ) = ∑cos(2 j α) =
j =0 2n+1 ⋅ sen(α)

1 1 1 1 1 1 1 1 1 ∞
 π  2
b) Prove que ⋅ + ⋅ + + ... = ∏cos  j +2  = .
2 2 2 2 2 2 2 2 2 j =0 2  π

SOLUÇÃO DE WALLACE ALVES MARTINS (RIO DE JANEIRO – RJ)


a) Demonstraremos a identidade utilizando o princípio da indução.
sen(20+1α) sen2α 2senα cosα
Para n = 0 temos cosα e = = = cosα. . Assim que a
20+1senα 2senα 2senα
propriedade é válida para n = k, temos:
k
sen(2k +1α)
∏cos(2 jα) =
j =0 2k +1senα
. Multiplicando-se ambos os membros da identidade por

cos(2k +1α) temos:


sen(2 ⋅ 2k +1α )
k +1
sen(2k +1α )cos(2k +1α ) 2 sen(2( k +1)+1α )

j =0
cos(2 j
α ) =
2k +1 senα
=
2k +1 senα
=
2( k +1)+1 senα
.

Portanto a propriedade também é válida para n = k + 1. Logo, pelo princípio da


indução segue-se que a identidade é válida ∀n ∈ , n ≥ 0.

b) Demonstraremos primeiramente a identidade entre o primeiro membro e o


membro central da identidade acima; por indução.
 π  π  1
Para n = 0, temos que cos  0+2 
= cos   = . Assim a propriedade é
2   4 2
verdadeira para n = 0. Vamos mostrar que para todo k temos
1 1 1 1 1 1 1 1 1 k
 π 
⋅ + ⋅ ...⋅ + + ... + = ∏cos  j +2 . Para isso, vamos
2 2 2 2 2 2 2 2 2 j =0  2 
( k +1) Radicais

EUREKA! N°18, 2003

55
Sociedade Brasileira de Matemática

1 1 1 1 1  π 
mostrar que para todo k temos + + ... + = cos  k + 2  . Considere
2 2 2 2 2 2 
( k +1) Radicais

1 1 1 1 1 1 1  π   π 
x= + + ... + temos que x2 − = cos  k +2  ⇔ cos  k +2  = 2x2 −1.
2 2 2 2 2 2 2 2  2 
( k +2) Radicais

 π 
 k +2   π 
Como sabemos cos(2α ) = 2 cos α − 1. Logo
2
x = cos  2  = cos  k +3 
 2  2 
 
1 1 1 1 1  π 
satisfaz a equação. Logo x = + + ... + = cos  (k +1)+2 . Portanto a
2 2 2 2 2 2 
[( k +1)+1] Radicais

identidade também é válida para n = k + 1. Logo, pelo Princípio da Indução segue-


se que a identidade é válida ∀n ∈ , n ≥ 0. Em particular, quando n →∞ temos a
identidade conforme acima.
n
 π  π  π  π   π 
Agora, sabemos que ∏cos  2
j =0
j +2 

= cos   ⋅ cos   ⋅ cos   ⋅... ⋅ cos  n+2  =
 4 8  16  2 
 π   π  π  π  n π
= cos  n+2  ⋅ cos  n+1  ⋅ ...⋅ cos   ⋅ cos   = ∏cos(2 j α), onde α = n + 2 .
2  2  8  4  j =0 2
 π 
n
sen  2 n +1 , n + 2 
 2  1
Utilizando (a) temos: ∏ cos(2 j α ) = = .
n +1  π  n +1  π 
j =0
2 ⋅ sen  n + 2  2 sen  n + 2 
2  2 
 π  π senx
Sabemos que, quando n →∞ então 2n+1 ⋅ sen  n+2  → . (Pois lim = 1) .
2  2 x →0 x
 
 n
 π   1  2
Logo: lim ∏cos  j +2  = lim  = .
n→∞
 j =0  2 
n→∞
 2n+1 sen  π   π
  n+ 2  
 2 

EUREKA! N°18, 2003

56
Sociedade Brasileira de Matemática

83) Seja N = {0,1,2,3,...}.


Determine quantas funções f : N → N satisfazem f(2003) = 2003, f(n) ≤ 2003
para todo n ≤ 2003 e f(m + f(n)) = f(f(m)) + f(n), para todo m, n ∈ N.

84) Prove que se A ⊂ N* = {1,2,3,...} é um conjunto não-vazio tal que


n ∈ A ⇒ 4n ∈ A e  n  ∈ A então A = N* .
Obs.  x é o único inteiro tal que x −1 <  x ≤ x

85) Mostre que todo triângulo pode ser dividido em 9 pentágonos convexos de
áreas iguais.

86) Encontre todas as triplas de inteiros positivos (a, m, n) tais que am +1 divide
(a +1)n .

87) Seja a(1) = 1 e, para cada inteiro n ≥ 2, a(n) igual ao menor inteiro positivo
n
que não pertence a {a( j), j < n} tal que ∑a( j) seja múltiplo de n. Prove que
j =1

a(a(n)) = n para todo inteiro positivo n.


 1 1 
88) Prove que se r ∈ e cos(r ⋅π ) ∈ então cos(r ⋅π ) ∈ −1, − ,0, ,1.
 2 2 

Seguimos aguardando as soluções dos problemas: 83, 84, 85, 86, 87 e 88…

EUREKA! N°18, 2003

57
Sociedade Brasileira de Matemática

PROBLEMAS PROPOSTOS

Convidamos o leitor a enviar soluções dos problemas propostos e sugestões de novos


problemas para os próximos números.

89) Uma prova de múltipla escolha com n questões é feita por k alunos. Uma
resposta correta na i-ésima questão vale pi pontos, onde pi é um inteiro
positivo, para 1 ≤ i ≤ n. A nota de cada aluno é a soma dos pontos
correspondentes às questões que ele acertou. Após a realização a prova, foi
observado que, mudando os pesos pi, as notas dos alunos podem estar em
qualquer uma das k! possíveis ordens (em que não há duas notas iguais). Dado
n, qual é o maior valor possível de k?

90) Prove que, para todo inteiro positivo n e para todo inteiro não nulo a, o
polinômio xn + axn−1 + axn−2 + ... + ax −1 é irredutível, i.e., não pode ser escrito
como o produto de dois polinômios não constantes com coeficientes inteiros.

91) Um jardinero deve construir um canteiro com a forma de setor circular. Ele
dispõe de 100 metros de fio para cercá-lo.
Figura:

Qual deve ser o valor do raio do círculo para que o canteiro tenha área máxima?
Qual é a área máxima?

92) Seja (Fn)n∈N a seqüência de Fibonacci, definida por F1 = 1, F2 = 1 e


Fn+2 = Fn+1 + Fn , ∀n ∈ . Prove que mdc (Fm , Fn ) = Fmdc(m,n) para quaisquer
inteiros positivos m e n.

EUREKA! N°18, 2003

58
Sociedade Brasileira de Matemática

93) Um inteiro positivo n é dito perfeito se n é igual à soma dos divisores


positivos de n que são menores que n. Prove que um número par n é perfeito
se e somente se existe um número primo p ≥ 2 tal que 2 p −1 é primo e
n = 2p−1 (2 p −1) .

94) A ilha das amazonas é habitada por amazonas e homens.


As amazonas mandam em tudo, são inteligentíssimas, ciumentíssimas e muito
fofoqueiras. O que uma amazona mais gosta de fazer é trair outra amazona
com o marido desta. Consumada a traição, ela conta o seu feito a todas as
amazonas da ilha menos à amazona traída. As outras amazonas também não
contam nada à vítima da traição. Mas se uma amazona descobre que está
sendo traída ela mata o seu marido na próxima meia noite.
A rainha das amazonas, que é viúva, vê esta situação com desagrado. Ela vê
que há traição na ilha mas, como nunca ninguém descobre nada, nenhum
marido morre. No dia 1 de janeiro de 3333, então, contrariando a tradição, ela
chama todas as amazonas para a praça central e faz uma proclamação solene:
"Há traição nesta ilha."
Nenhuma amazona sonha em duvidar da palavra da rainha e todas as
amazonas sabem disso. Como já foi dito, todas são inteligentes e ciumentas:
estes e os outros fatos mencionados neste enunciado até aqui são
conhecimento comum entre as amazonas.
Supondo que haja 1000 amazonas na ilha e que 365 delas tenham sido traídas,
o que acontecerá?

95) "Resta-Um" é um jogo de tabuleiro na qual as peças ocupam um tabuleiro


formando parte de um reticulado retangular (na verdade, existem variações em
tabuleiros de reticulado triangular). O único movimento permitido consiste
em tomar duas peças em casas adjacentes vizinhas a uma casa vazia, e fazer a
peça mais distante da casa vazia pular sobre a outra peça, ocupando a casa
vazia. A peça pulada é retirada.

(esse movimento pode ser feito para a direita, para a esquerda, para cima ou
para baixo).
Agora imagine um tabuleiro que é um reticulado retangular infinito e uma reta
que contém uma linha do reticulado, dividindo-o em dois lados. Todas as casas
de um dos lados da linha estão vazias e cada casa do outro lado da linha
pode ou não ter uma peça.

EUREKA! N°18, 2003

59
Sociedade Brasileira de Matemática

Quantas peças, no mínimo, precisamos para chegar a uma casa do lado vazio
do tabuleiro, a uma distância n da linha ? Abaixo indicamos uma casa a
distância n, para n = 1,2,3,4,5.

5
4
3
2
1
… …
… …
… …
… … … … … … …

Você sabia…
Que 2
20996011
−1 é primo? Este é o maior primo conhecido, tem
6320430 dígitos e foi descoberto por Michael Shafer, um
participante do GIMPS (um projeto distribuído para procurar
primos de Mersenne. Veja: http://www.mersenne.org para mais
informações). Agora são conhecidos 40 expoentes p para os
p −1
quais 2 − 1 é primo (e portanto 2 (2 p −1) é perfeito - veja o
p

problema proposto 93): 2, 3, 5, 7, 13, 17, 19, 31, 61, 89, 107,
127, 521, 607, 1279, 2203, 2281, 3217, 4253, 4423, 9689,
9941, 11213, 19937, 21701, 23209, 44497, 86243, 110503,
132049, 216091, 756839, 859433, 1257787, 1398269,
2976221, 3021377, 6972593, 13466217 e 20996011.

Problema 89 proposto na 27 Olimpíada Russa de Matemática em 2001, problema 90 proposto


na Olimpíada Romena de Matemática de 1992, problema 91 proposto por Osvaldo Mello
Sponquiado (Ilha Solteira – SP).

EUREKA! N°18, 2003

60
Sociedade Brasileira de Matemática

Você sabia…
Que existem infinitos inteiros positivos ímpares k tais que
k ⋅ 2 + 1 é composto para todo n ∈ ? Tais inteiros k são
n

chamados números de Sierpinski. Em 1962, John Selfridge


provou que 78557 é um número de Sierpinski, e conjectura-se
que seja o menor deles. Atualmente há 11 números menores que
78557 sobre os quais não se sabe se são números de Sierpinski
ou não: 4847, 10223, 19249, 21181, 22699, 24737, 27653,
28433, 33661, 55459 e 67607. O número 5359 fazia parte
dessa lista até 6/12/2003, quando Randy Sundquist ( um
participante do Seventeen or Bust, um projeto distribuído
para atacar o problema de Sierpinski) encontrou o primo
5359 ⋅ 25054502 + 1, que tem 1521561 dígitos e é o quarto maior
primo conhecido, e maior primo conhecido que não é de
Merssenne.
Veja: http://www.seventeenorbust.com para mais informações.

Exercício: Prove que 78557 é um número de Sierpinski, e que


existem infinitos números de Sierpinski a partir das
congruências
78557 ⋅ 2 0 + 1 ≡ 0 (mod 3)
78557 ⋅ 21 + 1 ≡ 0 (mod 5)
78557 ⋅ 2 7 + 1 ≡ 0 (mod 7)
78557 ⋅ 211 + 1 ≡ 0 (mod 13)
78557 ⋅ 23 + 1 ≡ 78557 ⋅ 239 + 1 ≡ 0 (mod 73)
78557 ⋅ 215 + 1 ≡ 0 (mod 19)
78557 ⋅ 2 27 + 1 ≡ 0 (mod 37).

EUREKA! N°18, 2003

61
Sociedade Brasileira de Matemática

COORDENADORES REGIONAIS
Alberto Hassen Raad (UFJF) Juiz de Fora – MG
Amarísio da Silva Araújo (UFV) Viçosa – MG
Ana Paula Bernardi da Silva (Universidade Católica de Brasília) Brasília – DF
Benedito Tadeu Vasconcelos Freire (UFRN) Natal – RN
Carlos Frederico Borges Palmeira (PUC-Rio) Rio de Janeiro – RJ
Claus Haetinger (UNIVATES) Lajeado – RS
Cleonor Crescêncio das Neves (UTAM) Manaus – AM
Élio Mega (Colégio Etapa) São Paulo – SP
Florêncio Ferreira Guimarães Filho (UFES) Vitória – ES
Gil Cunha Gomes Filho (Colégio ACAE) Volta Redonda – RJ
Ronaldo Alves Garcia (UFGO) Goiânia – GO
Reginaldo de Lima Pereira (Escola Técnica Federal de Roraima) Boa Vista – RR
Ivanilde Fernandes Saad (UC. Dom Bosco) Campo Grande– MS
Jacqueline Fabiola Rojas Arancibia (UFPB) João Pessoa – PB
Janice T. Reichert (UNOCHAPECÓ) Chapecó – SC
João Benício de Melo Neto (UFPI) Teresina – PI
João Francisco Melo Libonati (Grupo Educacional Ideal) Belém – PA
José Carlos dos Santos Rodrigues (Unespar) Campo Mourão – PR
José Cloves Saraiva (UFMA) São Luis – MA
José Gaspar Ruas Filho (ICMC-USP) São Carlos – SP
José Luiz Rosas Pinho (UFSC) Florianópolis – SC
José Vieira Alves (UFPB) Campina Grande – PB
Licio Hernandes Bezerra (UFSC) Florianópolis – SC
Luzinalva Miranda de Amorim (UFBA) Salvador – BA
Mário Rocha Retamoso (UFRG) Rio Grande – RS
Marcelo Rufino de Oliveira (Grupo Educacional Ideal) Belém – PA
Marcelo Mendes (Colégio Farias Brito, Pré-vestibular) Fortaleza – CE
Pablo Rodrigo Ganassim (Liceu Terras do Engenho) Piracicaba – SP
Ramón Mendoza (UFPE) Recife – PE
Raúl Cintra de Negreiros Ribeiro (Colégio Anglo) Atibaia – SP
Reinaldo Gen Ichiro Arakaki (INPE) SJ dos Campos – SP
Ricardo Amorim (Centro Educacional Logos) Nova Iguaçu – RJ
Sérgio Cláudio Ramos (IM-UFRGS) Porto Alegre – RS
Tadeu Ferreira Gomes (UEBA) Juazeiro – BA
Tomás Menéndez Rodrigues (U. Federal de Rondônia) Porto Velho – RO
Valdenberg Araújo da Silva (U. Federal de Sergipe) São Cristovão – SE
Valdeni Soliani Franco (U. Estadual de Maringá) Maringá – PR
Vânia Cristina Silva Rodrigues (U. Metodista de SP) S.B. do Campo – SP
Wagner Pereira Lopes (CEFET – GO) Jataí – GO

EUREKA! N°18, 2003

62
CONTEÚDO

AOS LEITORES 2

XXV OLIMPÍADA BRASILEIRA DE MATEMÁTICA 4


Problemas e Soluções da Primeira Fase

XXV OLIMPÍADA BRASILEIRA DE MATEMÁTICA 17


Problemas e Soluções da Segunda Fase

XXV OLIMPÍADA BRASILEIRA DE MATEMÁTICA 32


Problemas e Soluções da Terceira Fase

XXV OLIMPÍADA BRASILEIRA DE MATEMÁTICA 49


Problemas e Soluções da Primeira Fase – Nível Universitário

XXV OLIMPÍADA BRASILEIRA DE MATEMÁTICA 54


Problemas e Soluções da Segunda Fase – Nível Universitário

XXV OLIMPÍADA BRASILEIRA DE MATEMÁTICA 63


Premiados

AGENDA OLÍMPICA 67

COORDENADORES REGIONAIS 68
Sociedade Brasileira de Matemática

AOS LEITORES
Neste número apresentamos os problemas e soluções da XXV Olimpíada
Brasileira de Matemática, realizada durante o ano passado. A seguir o discurso do
Prof. Jacob Palis Jr. na premiação da XXV Olimpíada Brasileira de Matemática,
realizada na VII Semana Olímpica, na cidade de Belo Horizonte - MG em janeiro
de 2004.
Os editores.

Antes de apresentar breves, e certamente apaixonadas palavras sobre as


Olimpíadas Brasileiras de Matemática, quero registrar a minha admiração por um
dos seus grandes e talvez o maior de seus precursores: o professor Shigeo
Watanabe. Embora físico é devido a ele uma pioneira e ampla atividade de
Olimpíadas de Matemática no Estado de São Paulo, com o apoio da Academia de
Ciências de São Paulo e da Secretaria Estadual de Educação de São Paulo.
Seu exemplo, mais do que qualquer outro, inspirou a todos nós.
Também congratulo-me com todos os alunos premiados, seus professores e
familiares.
A Olimpíada Brasileira de Matemática – OBM existe desde 1979. Segundo o
próprio relatório de atividades 1997 – 2003 do Programa Nacional de Olimpíadas
de Matemática, que gentilmente foi preparado a meu pedido por Nelly Carvajal e
Sonia de Souza Silva de Melo, a OBM até então, era relativamente limitada
devido à escassez de recursos e sua influência na melhoria do ensino resultava
consideravelmente menor que o almejado. Já desenvolvia, no entanto, um
extraordinário trabalho na busca de jovens talentos para a Matemática ou ciências
afins. Basta citar a impressionante lista, certamente incompleta, de excelentes
matemáticos que daí resultaram: Edson de Faria (USP), Nicolau Saldanha (PUC-
Rio), Pedro Paulo Schimer (USP), Eduardo Esteves (IMPA), Ralph Costa Teixeira
(FGV-Rio), Carlos Moreira, (Gugu) (IMPA), Eduardo Laber (PUC-Rio), Daniel
Tausk (USP), Artur Avila (CNRS, França).
A OBM caracterizou-se sempre pela extrema dedicação de seus dirigentes, aliada
à competência, bom gosto e fé inquebrantável quanto aos seus benefícios, não só
para a comunidade matemática, mas para a sociedade em geral.
Infelizmente, ao lado do idealismo dos olímpicos, nem sempre foi possível
participar da Olimpíada Internacional de Matemática, com sua equipe completa,
por falta de recursos. Houve ocasiões em que alguns de nós, matemáticos já
estabelecidos, cotizamos a passagem de um ou mais brasileiros, de excepcional
qualificação, para possibilitar a prticipação do Brasil na Olimpíada Internacional.
Em 1997, sonhei, já há anos totalmente convencido da importância das
Olimpíadas, ser possível modificar radicalmente a situação. Conversei bastante

EUREKA! N°19, 2004

2
Sociedade Brasileira de Matemática

com Gugu, Nicolau, Elon Lima, Eduardo Wagner, Augusto Morgado, Paulo Cézar
Pinto Carvalho, dentre outros. Daí, com minha convicção e paixão em níveis
elevados, parti para o convencimento da Diretoria do CNPq, sob a Presidência de
José Galizia Tundisi. A receptividade quanto à importância de um novo Programa
Nacional de Olimpíadas de Matemática foi excepcionalmente entusiástica. Nasceu
aí uma nova etapa da OBM, agora sim ampla e permanente de tão importante
atividade.
Os recursos multiplicaram-se consideravelmente, indo de muito pouco a cerca de
R$200.000 nesta transição e a R$400.000 agora. Com o entusiasmo renovado e até
ampliado de seus dirigentes, ouso dizer que o Programa Nacional de Olimpíadas
de Matemática tornou-se eternamente robusto. Não é mais possível pensar senão
em crescer, fortalecer-se técnica e administrativamente e contribuir decisivamente
para o formação de uma ampla e sólida competência nacional em matemática,
passando por uma almejada inclusão científica.
A OBM hoje é uma atividade da Sociedade Brasileira de Matemática,
compartilhada com o Instituto Nacional de Matemática Pura e Aplicada – IMPA e,
a partir de 2001, com o Instituto do Milênio Avanço Global e Integrado da
Matemática Brasileira, (IM-AGIMB).
Nesta nova etapa, a participação do Brasil em Olimpíadas Internacionais cresceu
exponencialmente incluindo além da Olimpíada Internacional, a Ibero-americana,
a Olimpíada de Maio, e a do Cone Sul, além de Olimpíadas Regionais. Foi criada
em 1998 a Eureka!, excelente publicação dedicada principalmente aos alunos e
professores da escola secundária e editada três vezes ao ano. Multiplicou-se o
incentivo à realização de Olimpíadas Regionais e ao fortalecimento das
coordenações regionais. O treinamento de alunos e professores em diversos níveis
passou a ser atividade permanente. A melhoria do ensino de matemática nas
escolas tornou-se um objetivo exequível e contínuo. Criou-se um Banco de
Questões e Biblioteca o um site interativo, assim como uma secretaria permanente
no IMPA. Estabeleceu-se a Semana Olímpica, como atividade anual, ocasião em
que há um intenso treinamento dos alunos premiados com medalhas de Ouro,
Prata, Bronze e Menções Honrosas.
Após cerca de sete anos, deixo a Presidência da Comissão de Olimpíadas da SBM,
muito feliz pelas conquistas que vocês obtiveram e com a certeza absoluta de que
muito mais será alcançado, de forma permanente. Lugares como Ribeirão Preto,
Uberaba e Uberlândia e tantos outros de Norte a Sul e de Leste a Oeste do país
devem fazer parte do mapa da OBM. Sonhem muito e partam para sua realização.

Estarei sempre com vocês


Jacob Palis Júnior

EUREKA! N°19, 2004

3
Sociedade Brasileira de Matemática

XXV OLIMPÍADA BRASILEIRA DE MATEMÁTICA


Problemas e Soluções da Primeira Fase

PROBLEMAS – NÍVEL 1

1. Onze cubinhos, todos de mesma aresta, foram colados conforme a figura a


seguir.

O menor número de cubinhos, iguais aos já utilizados, que devem ser


agregados ao sólido formado pelos onze cubinhos para obtermos um cubo
maciço é igual a:
A) 48 B) 49 C) 52 D) 53 E) 56

2. Na tabela a seguir vemos o consumo mensal de água de uma família durante


os 5 primeiros meses de 2003.

Meses Consumo (m3)


Janeiro 12,5
Fevereiro 13,8
Março 13,7
Abril 11,4
Maio 12,1

O consumo mensal médio dessa família durante os 5 meses foi:


A) 11,3 m3 B) 11,7 m3 C) 12,7 m3 D) 63,5 m3 E) 317,5 m3

3. Você possui muitos palitos com 6 cm e 7 cm de comprimento. Para fazer uma


fila de palitos com comprimento total de 2 metros, o número mínimo de
palitos que você precisa utilizar é:
A) 29 B) 30 C) 31 D) 32 E) 33

EUREKA! N°19, 2004

4
Sociedade Brasileira de Matemática

4. Em um quadrado mágico, a soma dos números de cada linha, coluna ou


diagonal é sempre a mesma. No quadrado mágico a seguir, o valor de x é:

1 14 x
26 13

A) 20 B) 22 C) 23 D) 25 E) 27

5. Considere um número inteiro x e faça com ele as seguintes operações


sucessivas: multiplique por 2, some 1, multiplique por 3 e subtraia 5. Se o
resultado for 220, o valor de x é:
A) um número primo.
B) um número par.
C) um número entre 40 e 50.
D) um número múltiplo de 3.
E) um número cuja soma dos algarismos é 9.

6. Escreva os números de 0 a 9 nos círculos ao lado,


de forma que eles cresçam no sentido anti-
horário. Em seguida, subtraia 1 dos números
ímpares e some 1 aos números pares. Escolhendo
três círculos consecutivos, qual é a maior soma
que se pode obter?
A) 19 B) 21 C) 23
D) 24 E) 25

7. O retângulo da figura a seguir está dividido em 7 quadrados. Se a área do


menor quadrado é igual a 1, a área do retângulo é igual a:

A) 42 B) 44 C) 45 D) 48 E) 49

EUREKA! N°19, 2004

5
Sociedade Brasileira de Matemática

8. Considere a seqüência oscilante: 1, 2, 3, 4, 5, 4, 3, 2, 1, 2, 3, 4, 5, 4, 3, 2, 1, 2,


3, 4, …
O 2003o termo desta seqüência é:
A) 1 B) 2 C) 3 D) 4 E) 5

9. João disse para Maria: “Se eu lhe der um quarto do que tenho, você ficará com
metade do que vai me sobrar”. Maria acrescentou: “E eu lhe daria 5 reais, se
lhe desse a metade do que tenho”. Juntos, os dois possuem:
A) 80 reais B) 90 reais C) 100 reais D) 120 reais E)130 reais

10. Uma escola precisa comprar mesas e cadeiras novas para seu refeitório, cada
mesa com 4 cadeiras, que serão distribuídas nos 3 setores do refeitório. Em
cada setor do refeitório cabem 8 fileiras de mesas e, em cada fileira, cabem 14
mesas. Quantas mesas e cadeiras deverão ser compradas?
A) 112 mesas e 448 cadeiras
B) 112 mesas e 1344 cadeiras
C) 336 mesas e 448 cadeiras
D) 336 mesas e 896 cadeiras
E) 336 mesas e 1344 cadeiras

11. As 4 colorações a seguir são consideradas iguais por coincidirem por rotação.

De quantos modos diferentes é possível colorir as casas de um tabuleiro 2 × 2


de branco ou preto de modo que não existam dois tabuleiros que coincidam
por rotação?
A) 4 B) 5 C) 6 D) 7 E) 8

12. Numa festa típica, cada prato de arroz foi servido para duas pessoas, cada
prato de maionese para três pessoas, cada prato de carne servia quatro pessoas
e cada prato de doces dava exatamente para cinco pessoas. Foram utilizados
77 pratos e todas as pessoas se serviram de todos os pratos oferecidos.
Quantas pessoas havia na festa?
A) 20 B) 30 C) 45 D) 60 E) 75

EUREKA! N°19, 2004

6
Sociedade Brasileira de Matemática

13. Na organização retangular de pontos da figura abaixo, a distância entre pontos


vizinhos em uma mesma linha ou coluna é igual a 1 cm.
C

E B

A área do pentágono ABCDE é, em cm2, é igual a:


19 21
A) 9 B) C) 10 D) E) 11
2 2

14. Um quadrado de área 1 foi cortado em cinco filas de 5


quadradinhos cada. Todos os quadradinhos são
congruentes.
Marcam-se os quadradinhos de uma linha qualquer, de
uma diagonal qualquer e de uma coluna qualquer, e, em
seguida, retiram-se os quadrados assinalados. A área
coberta pelos quadradinhos restantes vale, no mínimo,
2 11 12
A) B) C)
5 25 25
13 3
D) E)
25 5

15. Um troféu formado por cinco recipientes


cúbicos foi construído da seguinte maneira:
sob o cubo de lado 10 cm foi soldado o cubo
de lado 20 cm, sob este foi soldado o cubo de
lado 30 cm, e assim por diante. Toda a
superfície externa desse troféu deverá ser
coberta com um certo tipo de revestimento.
Quantos metros quadrados desse revestimento
serão necessários?
A) 1,5 B) 2,5 C) 2,7
D) 2,75 E) 3

EUREKA! N°19, 2004

7
Sociedade Brasileira de Matemática

16. Num certo aeroporto, Nelly caminhava calmamente à razão de um metro por
segundo; ao tomar uma esteira rolante de 210 metros, Nelly continuou
andando no mesmo passo e notou ter levado um minuto para chegar ao fim da
esteira. Se Gugu ficar parado nesta esteira, quanto tempo levará para ser
transportado?
A) 1min20s B) 1min24s C) 1min30s D) 1min40s E) 2min

17. Uma certa máquina tem um visor, onde aparece um número inteiro x, e duas
teclas A e B. Quando se aperta a tecla A o número do visor é substituído por
2x + 1. Quando se aperta a tecla B o número do visor é substituído por 3x – 1.
Se no visor está o número 5, apertando alguma seqüência das teclas A e B, o
maior número de dois algarismos que se pode obter é:
A) 85 B) 87 C) 92 D) 95 E) 96

18. A seqüência “22” descreve a si mesma, pois ela é formada por exatamente
dois 2. Analogamente, a seqüência “31 12 33 15” descreve a si mesma, pois é
formada por exatamente três 1, um 2, três 3 e um 5. Qual das seguintes
seqüências não descreve a si mesma?
A) 21 32 23 16 B) 31 12 33 18 C) 31 22 33 17 19
D) 21 32 33 24 15 E) 41 32 23 24 15 16 18

19. Camila e Lara estão disputando o seguinte jogo num tabuleiro 4 × 4: Camila
marca algumas casas do tabuleiro e informa à Lara o número de casas
marcadas na vizinhança de cada casa do tabuleiro. Neste jogo, duas casas
distintas são consideradas vizinhas se possuem um lado ou um canto (vértice)
em comum.
Lara deve descobrir quais casas foram marcadas por Camila. Após marcar
algumas casas, Camila passou para Lara o seguinte tabuleiro:
1 2 1 1

0 2 1 2

2 3 3 1

1 0 2 1

O número de casas marcadas foi:


A) 3 B) 4 C) 5 D) 6 E) 7

EUREKA! N°19, 2004

8
Sociedade Brasileira de Matemática

20. Imagine uma pilha com cem milhões de folhas de papel sulfite, cada uma com
0,1 milímetro de espessura. Assinale a alternativa mais próxima da altura da
pilha.
A) a sua altura.
B) o comprimento do maior animal do mundo, a baleia azul, que é cerca de
29 metros.
C) a altura do edifício mais alto do mundo, o Petronas Tower, que tem 88
andares.
D) a altura do pico mais alto do mundo, o Monte Everest, que é 8848 metros.
E) a distância do planeta Terra à Lua, que é muito maior que todas as
alternativas anteriores.

PROBLEMAS – NÍVEL 2

1. Veja o problema No. 7 do Nível 1.


2. Veja o problema No. 3 do Nível 1.

3. A maior raiz da equação (x – 37)2 – 169 = 0 é:


A) 39 B) 43 C) 47 D) 50 E) 53

4. Veja o problema No. 17 do Nível 1.


5. Veja o problema No. 4 do Nível 1.

6. Seja n = 9867. Se você calculasse n3 – n2 você encontraria um número cujo


algarismo das unidades é:
A) 0 B) 2 C) 4 D) 6 E) 8

7. Na figura, o número 8 foi obtido somando-se os dois números diretamente


abaixo de sua casinha. Os outros números nas três linhas superiores são
obtidos da mesma forma. Qual é o valor de x?

42

8
3 5 x 6

A) 7 B) 3 C) 5 D) 4 E) 6

EUREKA! N°19, 2004

9
Sociedade Brasileira de Matemática

8. Veja o problema No. 5 do Nível 1.

9. Os números a, b, e c são naturais consecutivos em ordem crescente. Então, o


2
valor de c − ab é igual a:
A) 0 B) 1 C) 2a + b D) 2a + c E) 2b + c

10. Veja o problema No. 8 do Nível 1.

11. Considere as seguintes definições:


• A média aritmética de dois números reais positivos é a metade da sua soma.
• A média harmônica de dois números reais positivos é o inverso da média
aritmética dos inversos desses números.
A diferença entre a média aritmética e a média harmônica dos números 4 e 6 é:
A) 0,1 B) 0,2 C) 0,3 D) 0,4 E) 0,5

12. Veja o problema No. 18 do Nível 1.

13. O dominó mais conhecido tem como maior peça o


duplo 6. Neste dominó são empregadas 28 peças
diferentes. Quantas peças tem o dominó cuja maior
peça é o duplo 8?
A) 34 B) 36 C) 42 D) 55 E) 45

14. Os quadrados dos números naturais maiores do que 2, subtraídos de seus


sucessores, formam a seqüência 5, 11, 19, ... . O primeiro elemento dessa
seqüência que não é um número primo é o:
A) quarto B) décimo C) sexto D) nono E) sétimo

15. Você está em um país estrangeiro, a LUCIÂNIA, e não conhece o


idioma, o LUCIANÊS, mas sabe que as palavras “BAK” e “KAB” significam
sim e não, porém não sabe qual é qual. Você encontra uma pessoa que entende
português e pergunta: "KAB significa sim?" A pessoa responde “KAB”. Pode-
se deduzir que:
A) KAB significa sim.
B) KAB significa não.
C) A pessoa que respondeu mentiu.
D) A pessoa que respondeu disse a verdade.
E) Não é possível determinar sem um dicionário LUCIANÊS-PORTUGUÊS.

EUREKA! N°19, 2004

10
Sociedade Brasileira de Matemática

16. Veja o problema No. 13 do Nível 1.


17. Veja o problema No. 11 do Nível 1.

2 2003 ⋅ 91001 2 2002 ⋅ 91001


18. O valor da soma + é:
41001 ⋅ 32003 41001 ⋅ 32003
1 2 4
A) B) C) 1 D) E) 2
3 3 3

19. Considere os números X = 2 700 , Y = 11200 e Z = 5 300 . Assinale a alternativa


correta:
A) X< Z< Y B) Y<X<Z C) Y<Z<X D) Z<X<Y E) Z<Y<X

20. Beatriz, Isabele e Nicole estão disputando um jogo fazendo lançamentos


sucessivos com uma moeda. Beatriz ganha se, em dois lançamentos
consecutivos, o primeiro resultar cara e o segundo coroa. Isabele ganha se
forem obtidas duas coroas em dois lançamentos consecutivos, e Nicole ganha
se forem obtidas duas caras em dois lançamentos consecutivos. Elas fazem os
lançamentos até que uma das jogadoras seja vencedora. Qual(is) jogadora(s)
possui(em) menos chances de ganhar o jogo?
A) Beatriz B) Isabele C) Nicole D) Beatriz e Nicole
E) As três têm a mesma chance.

21. Veja o problema No. 19 do Nível 1.

22. Divida os números 2, 3, 5, 7, 11, 13 e 17 em dois grupos x e y com produtos A


e B, respectivamente, de modo que A – B = 1.
A soma dos algarismos de A é:
A) 10 B) 11 C) 13 D) 14 E) 15

23. A figura a seguir mostra um quadrado ABCD e um triângulo eqüilátero BEF,


ambos com lado de medida 1cm . Os pontos A, B e E são colineares, assim
como os pontos A, G e F.

D C
F

A B E

EUREKA! N°19, 2004

11
Sociedade Brasileira de Matemática

A área do triângulo BFG é, em cm 2 :


1 1 3 3 3
A) B) C) D) E)
4 3 4 12 10

24. Carlinhos pensa num número ímpar positivo menor do que 100. Pedrinho
se dispõe a descobrir que número é esse fazendo a seguinte pergunta,
quantas vezes forem necessárias: “O número que você pensou é maior,
menor ou igual a x ? ”. Note que x é um número que Pedrinho escolhe.
Quantas perguntas desse tipo Pedrinho poderá ter que fazer até descobrir o
número pensado por Carlinhos?
A) 5 B) 7 C) 15 D) 25 E) 45

25. No triângulo ABC, AB = 20, AC = 21 e BC = 29. Os pontos D e E sobre o lado


BC são tais que BD = 8 e EC = 9. A medida do ângulo DÂE, em graus, é igual
a:
A) 30 B) 40 C) 45 D) 60 E) 75

PROBLEMAS – NÍVEL 3

1. O número 19AB, onde A e B são dígitos, é um quadrado perfeito. O valor de


AB da raiz quadrada do número cuja representação decimal é AB é:
A) 5 B) 6 C) 7 D) 8 E) 9

2. Veja o problema No. 8 do Nível 1.


3. Veja o problema No. 19 do Nível 1.

4. Cinco amigos, Arnaldo, Bernaldo, Cernaldo, Dernaldo e Ernaldo, devem


formar uma fila com outras 30 pessoas. De quantas maneiras podemos formar
esta fila de modo que Arnaldo fique na frente de seus 4 amigos?
(Obs.: Os amigos não precisam ficar em posições consecutivas.)
D)   5!
35! 35! 35 π 163
A) 35! B) C) E) e
5! 5  5

5. A Revolução Francesa, em 1789, trouxe muitas mudanças na


humanidade. Em 1791, após a Revolução Francesa, a Academia Francesa de
Ciências propôs um novo sistema de medidas. Esse sistema era baseado numa

EUREKA! N°19, 2004

12
Sociedade Brasileira de Matemática

medida “natural” de comprimento, chamada metro, que foi definida como um


décimo de milionésimo da distância do Pólo Norte ao Equador, medida em
torno da circunferência do meridiano que passa por Paris. Tal sistema foi
efetivamente adotado em 1795. A definição atual do metro é diferente mas o
valor é aproximadamente o mesmo.
Considerando os fatos acima, qual é a ordem de grandeza do volume do
planeta Terra, em metros cúbicos?
Obs.: Nesta questão você pode querer utilizar a fórmula do volume V da
4
esfera, V = π R3 , onde R é o raio da esfera.
3
A) 1016 B) 1021 C) 1026 D) 1031 E) 1036

6. Na seqüência de Fibonacci 1, 1, 2, 3, 5, 8, 13, 21, 34, 55, … cada termo, a


partir do terceiro, é igual à soma dos dois termos anteriores.
Quanto vale a soma infinita
1 1 2 3 5
+ + + + + +
8 13
+
21 34
+ +
55
2 4 8 16 32 64 128 256 512 1024
+ ,
onde o n-ésimo termo é o n-ésimo termo da seqüência de Fibonacci dividido
por 2n?
A) 3/2 B) 2 C) 5/2 D) 3 E) 1 + 5
2

7. O gráfico de y = x2 − 5 x + 9 é rodado 180o em torno da origem. Qual é a


equação da nova curva obtida?

A) y = x2 + 5 x + 9 B) y = x2 − 5 x − 9 C) y = − x2 + 5 x − 9
D) y = − x2 − 5 x + 9 E) y = − x2 − 5 x − 9

8. Um clube de tênis tem n jogadores canhotos e 2n jogadores destros e, ao


todo, há menos do que 20 jogadores. No último campeonato interno, no qual
cada jogador enfrentou cada um dos outros jogadores do clube exatamente
uma vez, a razão entre o número de jogos vencidos por jogadores canhotos e o
número de jogos vencidos por jogadores destros foi 3 : 4.
Qual é o valor de n?
A) 3 B) 4 C) 5 D) 6
E) São necessárias mais informações.

EUREKA! N°19, 2004

13
Sociedade Brasileira de Matemática

9. A figura abaixo mostra duas retas paralelas r e s. A reta r é tangente às


circunferências C1 e C3, a reta s é tangente às circunferências C2 e C3 e as
circunferências tocam-se como também mostra a figura.

s C1

C2

C3

As circunferências C1 e C2 têm raios a e b, respectivamente.


Qual é o raio da circunferência C3?
4ab
A) 2 a 2 + b 2 B) a + b C) 2 ab D)
a +b
E) 2b – a

10. Veja o problema No. 18 do Nível 1.

11. A função f é definida para todos os pares ordenados (x; y) de inteiros positivos
e tem as seguintes propriedades:
f(x; x) = x, f(x; y) = f(y; x), (x + y)f(x; y) = (2x + y)f(x; x + y).
Qual é o valor de f(21; 12)?
7 4 11 6 1
A) B) C) D) E)
4 7 6 11 2003

12. Veja o problema No. 14 do Nível 2.


13. Veja o problema No. 15 do Nível 2.
14. Veja o problema No. 20 do Nível 2.
15. Veja o problema No. 22 do Nível 2.
16. Veja o problema No. 23 do Nível 2.
17. Veja o problema No. 12 do Nível 1.
18. Veja o problema No. 24 do Nível 2.

EUREKA! N°19, 2004

14
Sociedade Brasileira de Matemática

19. Dois amigos, Augusto e Eduardo, atravessavam uma ponte onde passava uma
linha férrea.
Quando tinham percorrido dois quintos da ponte, ouviram o barulho de um
trem que se aproximava por trás deles. Apavorados, começaram a correr, cada
um para o seu lado. Tiveram sorte: Augusto, que tinha voltado, conseguiu sair
da ponte no exato instante em que o trem nela ia entrar. Por sua vez, Eduardo,
que continuou para a frente, conseguiu sair da ponte no instante em que o trem
também ia fazê-lo. Refeitos do susto, quando se encontraram, comentaram que
isto só foi possível porque correram a 15 km/h e o trem estava a x km/h. O
valor de x é:
A) 30 B) 45 C) 60 D) 75 E) 90

20. Seja N o menor inteiro positivo que pode ser escrito como a soma de 9, 10 e
11 inteiros positivos consecutivos. A soma dos algarismos de N é igual a:
A) 9 B) 18 C) 22 D) 27 E) 30

32003 + 22003
21. O maior inteiro que não supera é igual a:
32001 + 22001
A) 4 B) 6 C) 7 D) 8 E) 9

22. Seja T = (a, b, c) tal que existe um triângulo ABC cujas medidas dos lados
sejam BC = a, CA = b e AB = c satisfazendo c ≥ b ≥ a > 0 e a + b > c .
Definimos T 2 = (a 2 , b 2 , c 2 ) e T = ( a, b, c) como sendo,
respectivamente, o quadrado e a raiz quadrada do
"triângulo" T. Considere então as afirmativas:
1) O quadrado de um triângulo equilátero é equilátero.
2) O quadrado de um triângulo retângulo não é um triângulo.
3) T 2 é um triângulo se, e somente se, T é acutângulo.
4) T sempre é um triângulo para todo T.
5) Todos os ângulos de T são agudos.
O número de afirmativas verdadeiras é:
A) 1 B) 2 C) 3 D) 4 E) 5

23. Em um quadro negro escreve-se o número 1. As únicas alterações


permitidas são substituí-lo pelo seu dobro ou pelo seu quadrado. Qual é o
maior número que pode ser obtido após efetuarmos 2003 alterações?
4006 (2 )
2003
(2 )
2002

A) 22003 B) 42002 C) 2(2 )


D) 2 E) 2

EUREKA! N°19, 2004

15
Sociedade Brasileira de Matemática

24. Se f : \ → \ é uma função tal que, para todo x ∈ \, f ( x )( f ( x) − x ) = 0 ,


então
A) f é a função nula.
B) f é a função identidade, ou seja, f(x) = x para todo x real
C) f é a função nula ou a função identidade
D) Há 4 possíveis funções f
E) Há infinitas funções f

25. Veja o problema No. 25 do Nível 2.

GABARITO
NÍVEL 1 (5a. e 6a. séries)
1) D 6) C 11) C 16) B
2) C 7) C 12) D 17) D
3) A 8) C 13) B 18) D
4) E 9) B 14) C 19) B
5) A 10) E 15) C 20) D

NÍVEL 2 (7a. e 8a. séries)


1) C 6) C 11) B 16) B 21) B
2) A 7) E 12) D 17) C 22) C
3) D 8) A 13) E 18) C 23) D
4) D 9) E 14) C 19) C 24) A
5) E 10) C 15) D 20) B 25) C

NÍVEL 3 (Ensino Médio)


1) B 6) B 11) D 16) D 21) D
2) C 7) E 12) C 17) D 22) E
3) B 8) C 13) D 18) A 23) E
4) C 9) C 14) B 19) D 24) E
5) B 10) D 15) C 20) B 25) C

EUREKA! N°19, 2004

16
Sociedade Brasileira de Matemática

XXV OLIMPÍADA BRASILEIRA DE MATEMÁTICA


Problemas e Soluções da Segunda Fase

PROBLEMAS – NÍVEL 1 PARTE A


(Cada problema vale 3 pontos)

01. Quantas vezes aparece o algarismo 9 no resultado da operação 10100 – 2003?

02. Quantos números inteiros maiores do que 20032 e menores do que 20042 são
múltiplos de 100?

03. Quantos triângulos existem cujos lados


estão sobre alguns dos segmentos traçados
na figura ao lado?

04. Um estudante, com muito tempo livre e muita curiosidade, resolveu fazer o
seguinte: a cada minuto, ao mudar o horário em seu relógio digital, marcava em
seu caderno um X para cada algarismo  que aparecia no visor. Assim, se seu
relógio mostrava  ele marcava X e quando seu relógio mostrou  ele
marcou XX. Começou a fazer isso quando seu relógio mostrava  e parou
quase doze horas depois, quando o relógio mostrava .
Calcule a metade da quantidade de X que ele marcou em seu caderno.

05. A grande atração do OBM Parque é uma roda gigante (a figura mostra uma
roda gigante similar, porém com um número menor de cabines). As cabines são
numeradas com 1, 2, 3,…, no sentido horário. Quando a cabine 25 está na posição
mais baixa da roda-gigante, a de número 8 está na posição mais alta. Quantas
cabines tem a roda-gigante?

EUREKA! N°19, 2004

17
Sociedade Brasileira de Matemática

06. Anos bissextos são múltiplos de 4, exceto aqueles que são múltiplos de 100
mas não de 400. Quantos anos bissextos houve desde a Proclamação da República,
em 1889, até hoje?

07. Em um dado comum a soma dos pontos sobre faces opostas é sempre 7. Beatriz
construiu uma torre com 4 dados comuns iguais, colando as faces como mostrado
na figura. Qual é o menor número de pontos que Beatriz pode obter somando
todos os pontos das dezoito faces da superfície da torre?

08. Na multiplicação a seguir a, b, c e d são algarismos.

45
× a3

3bcd
Calcule b + c + d.

09. A média de cinco inteiros positivos diferentes é 11. Determine o maior valor
possível para o maior dos cinco inteiros.

EUREKA! N°19, 2004

18
Sociedade Brasileira de Matemática

10. Nove peças diferentes de dominó estão sobre uma mesa, parcialmente cobertos
por um pedaço de papel. Os dominós se tocam de modo que 1 ponto é vizinho a 1
ponto, 2 pontos são vizinhos a 2 pontos, etc. Qual o total de pontos escondidos
pelo papel?

PROBLEMAS – NÍVEL 1 PARTE B


(Cada problema vale 10 pontos)

PROBLEMA 1
Quais números inteiros positivos menores que 120 podem ser escritos como soma
de duas ou mais potências distintas de base 3 e exponente positivo? Por exemplo,
12 = 32 +31 é um número deste tipo mas 18 = 32 + 32 não é.

PROBLEMA 2
No desenho ao lado, o quadrado
ABCD tem área de 64 cm2 e o
quadrado FHIJ tem área de 36 cm2.
Os vértices A, D, E, H e I dos três
quadrados pertencem a uma mesma
reta. Calcule a área do quadrado
BEFG.

EUREKA! N°19, 2004

19
Sociedade Brasileira de Matemática

PROBLEMA 3
Considere o produto de todos os divisores positivos de um número inteiro
positivo, diferentes desse número. Dizemos que o número é poderoso se o produto
desses divisores for igual ao quadrado do número. Por exemplo, o número 12 é
poderoso, pois seus divisores positivos menores do que ele são 1, 2, 3, 4 e 6 e
1 ⋅ 2 ⋅ 3 ⋅ 4 ⋅ 6 = 144 = 122 . Apresente todos os números poderosos menores do que
100.

PROBLEMAS – NÍVEL 2

PROBLEMA 1
No desenho ao lado, o quadrado
ABCD tem área de 30 cm2 e o
quadrado FHIJ tem área de 20 cm2.
Os vértices A, D, E, H e I dos três
quadrados pertencem a uma mesma
reta. Calcule a área do quadrado
BEFG.

PROBLEMA 2
Dados os números inteiros de 1 a 26, escolha 13 dentre eles de forma que:
1) O número 4 está entre os números escolhidos.
2) Nenhum número escolhido é divisor de outro número escolhido.

PROBLEMA 3
Uma folha retangular ABCD de área 1000 cm2
foi dobrada ao meio e em seguida desdobrada
(segmento MN); foi dobrada e desdobrada
novamente (segmento MC) e finalmente,
dobrada e desdobrada segundo a diagonal BD.
Calcule a área do pedaço de papel limitado
pelos três vincos (região escura no desenho).

EUREKA! N°19, 2004

20
Sociedade Brasileira de Matemática

PROBLEMA 4
Veja o problema No. 3 do Nível 1 – Parte B.

PROBLEMA 5
Seja f : #*+ → #*+ , uma função tal que f ( x ) f ( y ) − f ( xy ) = x + y , quaisquer
y x
que sejam os reais não nulos x e y .
(a) Calcule f(1)
(b) Encontre uma fórmula para f(x)

PROBLEMA 6
Dizemos que um número N de quatro algarismos é biquadrado quando é igual à
soma dos quadrados de dois números: um é formado pelos dois primeiros
algarismos de N, na ordem em que aparecem em N e o outro, pelos dois últimos
algarismos de N, também na ordem em que aparecem em N.
Por exemplo, 1233 é biquadrado pois 1233 = 122 + 332. Encontre um outro
número biquadrado.

Observação: Lembre-se de que um número de quatro algarismos não pode


começar com zero.

PROBLEMAS – NÍVEL 3

PROBLEMA 1
No triângulo ABC, M é o ponto médio do lado AC, D é um ponto sobre o lado BC
tal que AD é bissetriz do ângulo BÂC e P é o ponto de interseção de AD e BM.
Sabendo que a área de ABC é 100, AB = 10 e AC = 30, calcule a área do triângulo
APB.

PROBLEMA 2
Veja o problema No. 6 do Nível 2

PROBLEMA 3
Entre 15 números reais distintos, o menor deles igual a 1, não há três que podem
ser lados de um triângulo. Quais valores o maior dos 15 números pode assumir?

PROBLEMA 4
O triângulo ABC é retângulo em A. Dentre os pontos P pertencentes ao perímetro
do triângulo, encontre aquele que minimiza a soma AP + BP + CP.

EUREKA! N°19, 2004

21
Sociedade Brasileira de Matemática

PROBLEMA 5
Um quadrado de lado 3 é dividido em 9 quadrados de lado unitário, formando um
quadriculado. Cada quadrado unitário é pintado de azul ou vermelho. Cada cor
1
tem probabilidade de ser escolhida e a cor de cada quadrado é escolhida
2
independentemente das demais. Qual a probabilidade de obtermos, após
colorirmos todos os quadrados unitários, um quadrado de lado 2 pintado
inteiramente de uma mesma cor?

PROBLEMA 6
Calcule a soma
2k +1
" + 32 + 1
n +1
n
21 22 23 24
∑3 2k
+1
= + + +
31 + 1 32 + 1 34 + 1 38 + 1
+ 2n
k =0

Soluções Nível 1 – Segunda Fase – Parte A

Problema 01 02 03 04 05 06 07 08 09 10
Resposta 98 40 17 66 34 27 58 15 45 22

Soluções Nível 1 – Segunda Fase – Parte B


SOLUÇÃO DO PROBLEMA 1:
Temos 31 = 3, 32 = 9, 33 = 27, 34 = 81 mas 35 = 243 (não serve) . Assim, os
números obtidos de acordo com as condições do problema são:
3 + 9 =12, 3 + 27 = 30, 3 + 81 = 84, 9 + 27 = 36, 9 + 81 = 90, 27 + 81 = 108,
3 + 9 + 27 = 39, 3 + 9 + 81 = 93, 3 + 27 + 81 = 111, 9 + 27 + 81 = 117.
Note que o número 3 + 9 + 27 + 81 = 120 não serve.

SOLUÇÃO DO PROBLEMA 2:
Os triângulos ABE e EHF são retângulos em A e H, respectivamente; a medida do
ângulo BÊF é de 90o; se a medida do ângulo HÊF é x, então a medida dos ângulos
ˆ e AÊB é 90o – x e, conseqüentemente, a medida do ângulo ABE
EFH ˆ é x; como
BE = EF (são lados do mesmo quadrado), então os triângulos mencionados são
congruentes (pelo caso ALA de congruência de triângulos). Utilizando o teorema
de Pitágoras, podemos escrever BE 2 = AB 2 + AE 2 , o que mostra que a área do
quadrado BEFG é a soma das áreas dos quadrados ABCD e FHIJ, ou seja, 64 + 36
= 100 cm2.

EUREKA! N°19, 2004

22
Sociedade Brasileira de Matemática


SOLUÇÃO DO PROBLEMA 3:
Os divisores positivos de um número inteiro N são d 1 , d 2 , d 3 , , d k , tais que
1 = d1 ≤ d 2 ≤ d 3 ≤ 
≤ dk = N e podemos observar que
1 ⋅ N = d 2 ⋅ d k −1 = d 3 ⋅ d k − 2 etc. Por exemplo, os divisores positivos de 12 são
1, 2, 3, 4, 6 e 12, de forma que 1×12 = 2×6 = 3×4. Note que ao excluir os divisores
1 e 12, restam 2, 3, 4 e 6, cujo produto é 2×3×4×6 = (2×6)×(3×4) = 12×12 = 122.

Assim, concluímos que o produto dos divisores positivos de um inteiro, excluindo


1 e o próprio número, é igual ao quadrado do número se, e somente se, o número
2
tem 6 divisores. Portanto, o número é da forma p5 ou p ⋅ q , onde p e q são
números primos positivos, distintos. Se o número é positivo menor do que 100,
temos as 16 seguintes possibilidades:

2 5 = 32
2 2 ⋅ 3 = 12
3 2 ⋅ 2 = 18
2 2 ⋅ 5 = 20
3 2 ⋅ 5 = 45 52 ⋅ 2 = 50
2 2 ⋅ 7 = 28 7 2 ⋅ 2 = 98
3 2 ⋅ 7 = 63 52 ⋅ 3 = 75
2 ⋅ 11 = 44
2

3 2 ⋅ 11 = 99
2 2 ⋅ 13 = 52
2 2 ⋅ 17 = 68
2 2 ⋅ 19 = 76
2 2 ⋅ 23 = 92

Soluções Nível 2 – Segunda Fase

SOLUÇÃO DO PROBLEMA 1:
Os triângulos ABE e EHF são retângulos G
em A e H, respectivamente; a medida do
ângulo BEFˆ é de 90°; se a medida do C B

ângulo HEF ˆ é x, então a medida dos F J


ˆ
ângulos EFH e AEB ˆ é 90° − x e,
conseqüentemente, a medida do ângulo D I
A E H
ˆ é x; como BE = EF (são lados do
ABE
mesmo quadrado), então os triângulos
mencionados são congruentes (pelo caso
ALA de congruência de triângulos).

EUREKA! N°19, 2004

23
Sociedade Brasileira de Matemática

Utilizando o teorema de Pitágoras, podemos escrever BE 2 = AB 2 + AE 2 , o que


mostra que a área do quadrado BEFG é a soma das áreas dos quadrados ABCD e
FHIJ, ou seja, 30 + 20 = 50cm2.

SOLUÇÃO DO PROBLEMA 2:
Todo número inteiro positivo n pode ser escrito na forma 2a ⋅ b, a ≥ 0, b > 0 e b
ímpar (chamamos b de parte ímpar de n). Considere dois números com a mesma
parte ímpar: n1 = 2a1 ⋅ b e n2 = 2a2 ⋅ b . Supondo, sem perda da generalidade, que se
a1 < a2 , então teremos que n1 é divisor de n2 .
Assim, como de 1 a 26 temos 13 partes ímpares possíveis, a saber: 1, 3, 5, 7, 9, 11,
13, 15, 17, 19, 21, 23 e 25, cada um dos números deve ter uma parte ímpar
diferente. Mais ainda, considerando que 1 divide todos os números inteiros, o
número com parte ímpar 1 é o que deve ter maior a.
Porém 4 = 22 ⋅ 1 e está entre os números escolhidos, logo para os demais números
escolhidos devemos ter a = 0 ou a = 1. E podemos determinar todas as escolhas
possíveis:
• 3 é divisor de 9; 15 e 21. Logo 2 ⋅ 3 = 6,9,15 e 21 devem estar na nossa
escolha.
• 5 é divisor de 15 e 25. Logo 2 ⋅ 5 = 10 e 25 devem estar na nossa escolha.
• 7 é divisor de 21. Logo 2 ⋅ 7 = 14 deve estar na nossa escolha.
• Com parte ímpar 11 podemos escolher 11 ou 22 e com parte ímpar 13, 13 ou
26. As demais escolhas são 17, 19 e 23.
Portanto as escolhas possíveis são (ordenadas segundo a parte ímpar):
4; 6; 10; 14; 9; 11 ou 22; 13 ou 26; 15; 17; 19; 21; 23; 25.

SOLUÇÃO DO PROBLEMA 3:
Vamos usar a notação [X] para denotar a área do polígono X.

A M B

E F
P
D N C
Sejam E e F os pontos de interseção como mostrados na figura. Sejam AB = 2a e
BC = 2b. Então AM = MB = DN = NC = a e ME = EN = b. Trace AN e seja P o

EUREKA! N°19, 2004

24
Sociedade Brasileira de Matemática

ponto de interseção dos segmentos AN e BD. Os segmentos AN e MC são


paralelos (pois AM = NC e AM || NC).
Como M é ponto médio de AB e MF || AP, temos que F é o ponto médio do
segmento PB. Analogamente P é o ponto médio do segmento DF. Segue então que
DP = PF = FB.
Por simetria verificamos que PE = EF e então EF/FB = 1/2. Portanto, podemos
escrever:
[ MEF ]
= 1/ 2.
[ MBF ]
1 1 125 2
Mas, por outro lado, [ MBE ] = [ ABD] = 125 , donde [ MEF ] = 125 = cm e
4 3 3
2 250 2
[ MBF ] = 125 = cm .
3 3

SOLUÇÃO DO PROBLEMA 4:
Veja a solução do problema No. 3 do Nível 1 – Parte B.

SOLUÇÃO DO PROBLEMA 5:
(a) Fazendo x = y = 1, obtemos [ f (1)]2 − f (1) = 2, donde, resolvendo a
equação, obtemos
f(1) = 2 ou f(1) = – 1. Este último valor não serve, pois o contra-domínio da
função é o conjunto dos números reais estritamente positivos. Portanto, f(1) = 2.

Fazendo y = 1 na identidade do problema obtemos


1
f ( x ) f (1) − f ( x) = x + . Substituindo o valor de f(1), obtemos a fórmula para
x
1
f(x): f ( x ) = x + .
x

SOLUÇÃO DO PROBLEMA 6:
Vamos separar o número de quatro dígitos em duas partes: os dois primeiros
dígitos, da esquerda para a direita, formam o número x e os dois restantes formam
o número y.
Então a propriedade significa que 100x + y = x 2 + y 2 . Esta igualdade pode ser
considerada uma equação do segundo grau em x: x2 −100x + y2 − y = 0. (3)
Resolvendo encontramos x = 50 ± 2500 − ( y 2 − y ). (4)

EUREKA! N°19, 2004

25
Sociedade Brasileira de Matemática

Com o exemplo do enunciado, y = 33 resulta em x = 12 com o sinal (–) na


expressão:
x = 50 − 1444 = 50 − 38 = 12.
Naturalmente outra solução aparece quando colocamos o sinal (+) na mesma
expressão:
x1 = 50 + 1444 = 50 + 38 = 88.
Então outro número com a mesma propriedade é 8833 = 882 + 332.

Soluções Nível 3 – Segunda Fase

SOLUÇÃO DO PROBLEMA 1:
A

αα

B D C

As alturas que passam por B dos triângulos ABC e ABM são iguais a distância d
de B à reta AC, logo
AM ⋅ d
área ABM 2 AM 1 1 1
= = = ⇒ área ABM = área ABC = ⋅ 100 = 50.
área ABC AC ⋅ d AC 2 2 2
2
área ABP BP
Analogamente, = . Pelo Teorema das Bissetrizes,
área ABM BM
BP AB 10 2 3
= = = ⇒ PM = BP
PM AM 15 3 2

Logo

área ABP BP BP BP BP 2 2 2
= = = = = ⇒ área ABP = área ABM = ⋅ 50 = 20.
área ABM BM BP + PM BP + 3 BP 5 BP 5 5 5
2 2

EUREKA! N°19, 2004

26
Sociedade Brasileira de Matemática

SOLUÇÃO DO PROBLEMA 2:
Veja a solução do problema No. 6 do Nível 2.

SOLUÇÃO DO PROBLEMA 3:
Sejam a, b, c reais positivos tais que a ≤ b ≤ c . Esses números são medidas dos
lados de um triângulo se, e somente se, c < a + b . Ou seja, não são se, e somente
se, c ≥ a + b.
Assim, sendo 1 = x1 < x2 < x3 < x4 < ... < x15 os números dados, devemos ter:

x3 ≥ x2 + x1
x4 ≥ x3 + x2
#
x15 ≥ x14 + x13

De fato, esse sistema de desigualdades equivale a não haver três que podem ser
lado de um triângulo. Observe que se, i < j < k , xk < x j + xi , então xk < xk −1 + xk − 2 .
Considere a seqüência de Fibonacci ( F0 = 0, F1 = 1 e Fn +2 = Fn +1 + Fn , n ≥ 0),
x3 ≥ x2 + x1 ;
x4 ≥ x3 + x2 ≥ x2 + x1 + x2 = 2 x2 + x1 ;
x5 ≥ x4 + x3 ≥ 2 x2 + x1 + x2 + x1 = 3 x2 + 2 x1 ;
x6 ≥ x5 + x4 ≥ 3x2 + 2 x1 + 2 x2 + x1 = 5 x2 + 3x1 ;
parece que xn ≥ Fn −1 x2 + Fn −2 x1 e, com efeito,
xk + 2 ≥ xk +1 + xk ≥ Fk ⋅ x2 + Fk −1 ⋅ x1 + Fk −1 ⋅ x2 + Fk −2 ⋅ x1 =
Fk +1 ⋅ x2 + Fk ⋅ x1
Portanto, sendo x2 = 1 + ξ , ξ > 0,
x15 ≥ F14 ⋅ x2 + F13 ⋅ x1 = 377 ⋅ (1 + ξ ) + 233 ⋅ 1 = 610 + 377ξ .

Como podemos tornar ξ tão pequeno quanto queiramos, o maior dos 15 números
pode assumir qualquer valor real maior do que 610.

EUREKA! N°19, 2004

27
Sociedade Brasileira de Matemática

SOLUÇÃO DO PROBLEMA 4:
Sejam a, b, c as medidas dos segmentos BC, AC e AB, C
respectivamente.
Consideraremos separadamente os casos em que P está
em AC, em AB e em BC.
Se P está em AC, então AP + CP = b. Então, minimizar a
AP + BP + CP reduz-se a minimizar BP. Isso ocorre
b
quando P coincide com A, pois a menor distância entre
um ponto e uma reta é determinada pelo pé da
perpendicular traçada a partir desse ponto.
Nesse caso o valor mínimo de AP + BP + CP é b + c.
O caso em que P está em AB é inteiramente análogo. A c B
Suponha, agora, que P está em BC. Etão BP + CP = a,
ou seja, minimizar AP + BP + CP reduz-se a
minimizar AP.

Isso ocorre quando AP é perpendicular a BC.


C
Essa medida está representada por d no diagrama ao lado.
Nesse caso, o mínimo de AP + BP + CP é a + d.
Assim, para completar a resolução da questão, basta
comparar a
a + d e b + c.
Temos, então, várias maneiras de concluir a resolução. b
Uma maneira:
b⋅c a⋅d d
Observe que = ⇔ bc = ad e a 2 = b2 + c 2 .
2 2 θ
Logo A c B

(a + d )2 = a 2 + 2ad + d 2 = b2 + c2 + 2bc + d 2 = (b + c)2 + d 2

e, como d 2 > 0,( a + d ) 2 > (b + c ) 2 ⇔ a + d > b + c.


Outra maneira: d = c ⋅ senθ ; b = a ⋅ senθ .

Logo (a + d ) − (b + c ) = a + c ⋅ senθ − a ⋅ senθ − c = (a − c )(1 − senθ ) > 0, isto é, a +


d > b + c.
Resposta: O ponto que minimiza AP + BP + CP é P = A (nesse caso AP + BP +
CP = b + c).

EUREKA! N°19, 2004

28
Sociedade Brasileira de Matemática

SOLUÇÃO DO PROBLEMA 5:
O quadrado de lado 2 pode ser

ou ou ou

A probabilidade de cada um desses quadrados de lado 2 ser inteiramente de uma


mesma cor é
4
1
2 ⋅   . Observe que todos os quatro quadrados unitários devem ser da mesma
2
cor azul ou vermelho. Os demais quadradinhos podem ser de qualquer cor.
Algumas configurações são consideradas pelo menos 2 vezes:

7
 1
Probabilidade 2 ⋅  
 2
ou

ou ou
6
1
Probabilidade 2 ⋅  
2

ou

Algumas configurações são consideradas pelo menos 3 vezes:

EUREKA! N°19, 2004

29
Sociedade Brasileira de Matemática

ou ou
8
1
Probabilidade 2 ⋅ 
2

ou

E as configurações com todos azuis ou todos vermelhos são contadas 4 vezes


9
1
(probabilidade:   ).
2
Pelo Princípio da Inclusão-Exclusão, a probabilidade pedida é:
4 7 6 8 9
1 1 1 1 1 95
4⋅ 2⋅  − 2⋅ 2⋅  − 4⋅ 2⋅  + 4⋅ 2⋅  − 2⋅  = .
2  2  2  2  2  256

SOLUÇÃO DO PROBLEMA 6:
Analisando casos pequenos:
21 2 2 21
= = 1 − = 1 −
31 + 1 4 4 31 + 1
1 2
2 2 36 4 22
+ = = 1 − = 1 −
31 + 1 32 + 1 40 40 (31 + 1) ⋅ (32 + 1)
21 22 23 3272 8 23
+ + = = 1 − = 1 −
31 + 1 32 + 1 34 + 1 3280 3280 (31 + 1) ⋅ (32 + 1) ⋅ (34 + 1)
(Observe que não compensaria simplificar as frações. Isso é comum quando
queremos descobrir um padrão.)
Parece então, que podemos conjecturar que
n
2k +1 2n+1
∑k =0 3
2k
+1
= 1 − n
(31 + 1) ⋅ (32 + 1)(34 + 1)...(32 + 1)
Simplificando um pouco essa expressão antes de tentar demonstrá-la.

EUREKA! N°19, 2004

30
Sociedade Brasileira de Matemática

(31 − 1)(31 + 1)(32 + 1)(34 + 1)...(32 + 1)


n

(31 + 1)(32 + 1)(34 + 1)...(32 + 1) = =


n

31 − 1
n+1
(32 − 1)(32 + 1)(34 + 1)...(32 + 1) (34 − 1)(34 + 1)...(32 + 1) 32 − 1
n n

= = ... = .
2 2 2
n
2k +1 2n+1 2 n+ 2
Ou seja, ∑ 2 k = 1 − 2n+1 = 1 − 2n+1
k =0 3 +1 3 −1 3 −1
2
Podemos agora demonstrar nossa conjuectura pelo uso direto do Princípio da
Indução Finita ou considerando que, se descobrirmos f(k) tal que
2k +1
k
= f (k + 1) − f (k ),
32 + 1
n
2k +1 n

∑k =0 3
2k
= ∑[ f (k + 1) − f (k )] =
+ 1 k =0

f (1) − f (0) + f (2) − f (1) + ... + f (n + 1) − f (n) = f (n + 1) − f (0)


2k +1
(f é a "integral discreta" de k
.)
32 + 1
Levando em conta novamente nossa conjectura, podemos inferir que
2k +1
f (k ) = − 2 k e, de fato,
3 −1
2k +2 2k +1 −2k +2 + 2k +1 (32 + 1) 2k +1 (32 + 1 − 2) 2k +1
k k

f (k + 1) − f (k ) = − k +1
+ = = =
32 − 1 32 − 1 (32 + 1)(32 − 1) (32 + 1)(32 − 1) 32 + 1
k k k k k k

Portanto
n
2k +1 −2n + 2  −21  n
2k +1 2n +2
∑3
k =0
2 k
+1
= f (n + 1) − f (0) =
32
n+1
−  20
−1  3 −1
 ⇔ ∑ 2k
k =0 3 +1
=1 − 2n+1
3 −1
.

EUREKA! N°19, 2004

31
Sociedade Brasileira de Matemática

XXV OLIMPÍADA BRASILEIRA DE MATEMÁTICA


Problemas e Soluções da Terceira Fase

PROBLEMAS – NÍVEL 1

PROBLEMA 1:
Quantos inteiros positivos menores que 1000 têm a soma de seus algarismos igual
a 7?

PROBLEMA 2:
Considere as seqüências de inteiros positivos tais que cada termo mais a soma dos
seus algarismos é igual ao termo seguinte. Por exemplo: 6, 12, 15, 21, 24, 30, 33,
39 é uma seqüência nessas condições.
Escreva a maior seqüência cujo último termo é 103 e que satisfaz tais condições.
Observação: maior seqüência é aquela com o maior número de termos.

PROBLEMA 3:
Os números 1, 2, 4, 8, 16, 32, 64,… são potências de 2.
Deseja-se dividir um quadrado de lado 2003 em outros quadrados cujos lados são
potências de 2. Mostre uma maneira de se fazer a divisão e obter 6364 quadrados
cujos lados são potências de 2.

PROBLEMA 4:
a) Dois quadrados estão posicionados de modo que o centro do primeiro é vértice
do segundo, como mostra a figura abaixo.

Se o lado do primeiro quadrado mede 12cm, quanto mede a área comum aos dois
quadrados?

b) Na figura a seguir, o paralelogramo tem lados de medida 12cm e 4cm e área


40cm2. Sejam P, Q, R e S os centros dos quadrados construídos externamente

EUREKA! N°19, 2004

32
Sociedade Brasileira de Matemática

sobre os quatro lados desse paralelogramo. Sabendo que o quadrilátero PQRS é


um quadrado, calcule a sua área.

PROBLEMA 5:
Queremos construir o perímetro de um retângulo utilizando 2003 varetas cujas
medidas são inteiros positivos. Para isso às vezes teremos de quebrar algumas
delas, mas todas as varetas e pedaços de varetas devem ser utilizados na
construção do retângulo.

a) Mostre que com uma única quebra nem sempre é possível construir o
retângulo.
b) Mostre que com duas quebras sempre é possível construir o retângulo.

PROBLEMAS – NÍVEL 2

PROBLEMA 1:
Num tabuleiro 2 × 2, como o mostrado a seguir, escreveremos números inteiros de
1 a 9 obedecendo à seguinte regra: A > B, C > D, A > C e B > D.

A B

C D

a) Quantos tabuleiros diferentes existem tais que B = C?


b) Quantos tabuleiros diferentes existem no total?

PROBLEMA 2:
Determine o menor número primo positivo que divide x2 + 5x + 23 para algum
inteiro x.

EUREKA! N°19, 2004

33
Sociedade Brasileira de Matemática

PROBLEMA 3:
O triângulo ABC está inscrito na circunferência S e AB < AC. A reta que contém A
e é perpendicular a BC encontra S em P (P ≠ A). O ponto X situa-se sobre o
segmento AC e a reta BX intersecta S em Q (Q ≠ B).
Mostre que BX = CX se, e somente se, PQ é um diâmetro de S.

PROBLEMA 4:
Mostre que x 2 + 4 y 2 − 4 xy + 2 x − 4 y + 2 > 0, quaisquer que sejam os reais x e y.

PROBLEMA 5:
São dados: uma circunferência K e um ponto A interior, fixo, distinto do centro.
Determine os pontos B, C e D sobre a circunferência de forma que a área do
quadrilátero ABCD seja a maior possível.

PROBLEMA 6:
Há N cidades na Tumbólia. Cada duas cidades desse país são ligadas por uma
rodovia ou uma ferrovia, não existindo nenhum par de cidades ligadas por ambos
os meios.
Um turista deseja viajar por toda a Tumbólia, visitando cada cidade exatamente
uma vez, e retornar a cidade onde ele começou sua jornada.
Prove que é possível escolher a ordem na qual as cidades serão visitadas de modo
que o turista mude o meio de transporte no máximo uma vez.

PROBLEMAS – NÍVEL 3

PROBLEMA 1: Veja o problema 2 do Nível 2.

PROBLEMA 2:
Seja S um conjunto de n elementos. Determine o menor inteiro positivo k com a
seguinte propriedade: dados quaisquer k subconjuntos distintos A1 , A2 ,..., Ak de S,
existe uma escolha adequada dos sinais + e – de modo que
±
* * *
± ± + −
S = A1 A2 ... Ak , onde Ai = Ai e Ai = S − Ai é o complementar de Ai em
relação a S.

PROBLEMA 3:
Seja ABCD um losango. Sejam E, F, G e H pontos sobre os lados AB, BC, CD e
DA, respectivamente, e tais que as retas EF e GH são tangentes à circunferência
inscrita no losango.
Prove que as retas EH e FG são paralelas.

EUREKA! N°19, 2004

34
Sociedade Brasileira de Matemática

PROBLEMA 4: Veja o problema 5 do Nível 2.

PROBLEMA 5:
Suponha que f : (0, +∞) → 5 satisfaz:

i) x < y ⇒ f ( x) < f ( y )
 2 xy  f ( x ) + f ( y )
ii) f ≥ , para todo x, y ∈ (0, +∞).
x+ y 2
Prove que existe x0 ∈ (0, +∞) tal que f ( x0 ) < 0.

PROBLEMA 6:
Um grafo cujo conjunto de vértices V tem n elementos é bacana se existir um
conjunto D ⊂ 1 e uma função injetiva f : V → 1, n 2 4  ∩ 1 tal que os vértices p
e q são ligados por uma aresta se e somente se f ( p ) − f ( q) ∈ D.
Mostre que existe n0 ∈ 1 tal que para todo n ≥ n0 existem grafos com n vértices
que não são bacanas.

Observação: Um grafo com conjunto de vértices V é um par (V, E) onde E é um


conjunto de subconjuntos de V, todos com exatamente dois elementos.
Um conjunto {p, q} é chamado de aresta se pertencer a E e neste caso dizemos
que esta aresta liga os vértices p e q.

SOLUÇÕES – NÍVEL 1

PROBLEMA 1: SOLUÇÃO DE VINÍCIUS H. CAMPOS SENRA (BELO HORIZONTE - MG):


700 é o último número possível até 1000 tal que a soma de seus algarismos seja
igual a 7: números menores que 700 têm soma dos algarismos maior que 7.
O primeiro número é 7 mesmo.
De 1 a 100, existem 8 números que a soma de seus algarismos é igual a 7: 07, 16,
25,…, 61, 70.
A medida que vai aumentando a ordem das centenas, diminui um número que é
possível fazer isto, ou seja:
De 101 a 200, existem 7 números: 106, 115,…, 160.
De 201 a 300, existem 6;
De 301 a 400, 5;
De 401 a 500, 4;
De 501 a 600, 3.

EUREKA! N°19, 2004

35
Sociedade Brasileira de Matemática

Apenas de 601 a 700 que não ocorre isso, pois fica incluído o 700 também, sendo
portanto 3 números. (601, 610 e 700).

Resposta: Somando todos esses resultados 36 números até 1000 têm a soma de
seus algarismos igual a 7.

PROBLEMA 2: SOLUÇÃO DE VITOR MORI (SÃO PAULO – SP)


Para a seqüência terminar em 103, devemos começar pelo fim.
Utilizando o diagrama da árvore, teremos:

92 82 68 61 53
103
91 77 70 62 49 38 28 23 16 8 4 2 1
101
100
86

Logo a maior seqüência é:


1, 2, 4, 8, 16, 23, 28, 38, 49, 62, 70, 77, 91, 101, 103.

PROBLEMA 3: SOLUÇÃO DO EDITOR


Tomamos um quadrado de lado 1, dobramos o seu lado e colocamos 5 quadrados
de lado 1 à esquerda e em cima para completar um quadrado de lado 3:
Dobramos de novo a figura e colocamos 13 quadrados de
lado 1 para completar um quadrado de lado 7. Dobramos
a figura e colocamos 29 quadrados de lado 1 para
completar um quadrado de lado 15. Dobramos a figura e
colocamos 61 quadrados de lado 1 para completar um
quadrado de lado 31. Quadruplicamos a figura e
colocamos 249 quadrados de lado 1 para completar um
quadrado de lado 125.
Octuplicamos a figura e colocamos 2001 quadrados de lado 1 para completar um
quadrado de lado 1001. Finalmente, dobramos a figura e colocamos mais 4005
quadrados de lado 1 para completar um quadrado de lado 2003. Gastamos assim,
no total, 4005 + 2001 + 249 + 61 + 29 + 13 + 5 + 1 = 6364 quadrados cujos lados
são potências de 2 para cobrir o quadrado de lado 2003.

PROBLEMA 4: SOLUÇÃO DE HENRIQUE PONDÉ DE OLIVEIRA PINTO (SALVADOR - BA)


a) Se prolongarmos os lados do segundo quadrado temos:

EUREKA! N°19, 2004

36
Sociedade Brasileira de Matemática

As retas m e s dividem o primeiro quadrado em quatro partes iguais, e então a


1
parte escura representa do primeiro quadrado (assim como as outras três
4
partes). Como a área do primeiro quadrado é 12. 12 = 144 então a área escura é
1
⋅ 144 = 36cm2.
4
Observação: Toda reta que passa pelo centro de um quadrado divide ele em duas
partes iguais. Como a reta r no exemplo a seguir:

A B

C
D
S
r

Pois se a reta S divide o quadrado em duas partes iguais, basta o triângulo ABO e o
triângulo CDO serem iguais.
Como os ângulos OCD l e OBA l são iguais, e AOBl e COD l são iguais, O lAB e
l também são iguais. Como OC = OB então os triângulos ABO e CDO, por
ODC
l = AOB
possuírem 3 ângulos iguais (COD l , OCD
l = OBA
l e ODC l = O lAB ) e o lado
igual (OC = OB ) são triângulos iguais.

EUREKA! N°19, 2004

37
Sociedade Brasileira de Matemática

1
b) As áreas rabiscadas são conhecidas (cada uma tem da área do quadrado na
4
qual está, diferente de PQRS)
N
T
X W
P A Q
G C
B
M H
Z I
O Q
S
J K P
DE L
R
U F Y

O triângulo ABC é igual ao DEF e o triângulo GHI é igual ao JKL. Se B l


AC = x e
 
GI H = 360° − 90° − 90° − (180° − x ) então GI H = 180 − (180 − x ) que é igual a x

então GIH = B ACl . Podemos dizer que AC = HI pois AC = 12cm − CJ ,
HI = 12cm – HM e CJ = MH (as retas W, N e Z dividem os dois grandes
quadrados de forma idêntica e em partes iguais). CJ = DI e DI = HM pois as
retas dividem o quadrado em 4 partes idênticas, logo CJ = MH . Pelo mesmo
raciocínio podemos deduzir que AB = IG . Sabendo que GI H = B AC l e que 
AB = IG e AC = HI então deduzimos que o triângulo ABC = GHI. Como o
triângulo ABC é igual ao DEF, GHI é igual ao JKL e o ABC é igual ao GHI, logo
os triângulos ABC, DEF, GHI e JKL são iguais. A área do quadrado PQRS é igual
a área riscada mais a área de BCJEDI mais a área de GHI mais a área de JKL.
Como a área de GHI mais a área de JKL é igual a área de ABC mais DEF então a
área total do quadrado é igual a: a área rabiscada mais a área de BCJEDI mais a
área de ABC mais a área de DEF. Isso tudo é igual a:

1 1 1 1
⋅ 4 cm 2 + ⋅ 12 cm 2 + ⋅ 4 cm 2 + ⋅ 12 cm 2 + 40 cm 2 =
4 4 4 4
= 4 cm + 36 cm + 4 cm + 36 cm + 40 cm 2 = 120 cm 2 .
2 2 2 2

EUREKA! N°19, 2004

38
Sociedade Brasileira de Matemática

PROBLEMA 5: SOLUÇÃO DA BANCA


a) Considere uma seqüência na qual cada termo é maior que a soma dos termos
anteriores, como por exemplo 1, 3, 3 2 , 33 ,... , 3 2003 . Obrigatoriamente temos que
quebrar a vareta de comprimento 3 2003 e colocar os pedaços em dois lados
opostos, pois 3 2003 é maior que o dobro da soma de todas as restantes. Agora, a
maior das varetas usadas nos dois lados restantes, 3n , é maior que a soma das
varetas 1, 3, 32 , 33 ,... , 3n+1 , o que torna impossível a construção do retângulo.

b) Quebrando inicialmente uma vareta qualquer ao meio, construímos dois lados


opostos. Em seguida, dividimos as varetas restantes em dois conjuntos A e B. Se
as somas dos comprimentos das varetas dos dois conjuntos forem iguais, não é
necessário fazer mais quebras. Caso contrário, passamos quantas varetas forem
necessárias de um para o outro até que a desigualdade das somas se inverta; agora
basta mais uma única quebra na última vareta que mudou de lado para que as
somas se igualem.

SOLUÇÕES – NÍVEL 2

PROBLEMA 1: SOLUÇÃO DE FELIPE GONÇALVES ASSIS (CAMPINA GRANDE - PB)


a) Temos que A > B e B > D, logo A > D.
Também sabemos que A > C e C > D, então podemos afirmar com certeza que
"D" é o menor número, pois B > D; C > D; A > D e "A" é o maior número pois A >
B; A > C; A > D.
Se "B" for igual a "C", teremos três números dispostos de tal forma que o menor
deles será "D", o maior será igual "A" e o outro será tanto "B" como "C". De
quantas formas então eu posso escolher 3 inteiros diferentes entre 1 e 9?
Vamos pensar da seguinte maneira: para escolher o primeiro número eu tenho 9
possibilidades: 1; 2; 3; 4; 5; 6; 7; 8; 9, já para cada escolha eu tenho outros 8
números para escolher, os de 1 a 9 exceto o primeiro escolhido. Finalmente, para
cada possibilidade eu tenho outros 7 inteiros para escolher, os 9 à exceção dos já
escolhidos, assim, eu tirei 9 × 8 × 7 possibilidades, ou seja: 504.
Todavia, a ordem dos números não importa, isto é, o menor deve ser "D", o maior
"A" e o do meio "B" e "C", e escolhendo da forma supracitada os mesmos números
são escolhidos 6 vezes (3!). Observe: "x"; "y" e "z" só podem ser usados juntos
uma vez mas nas 504 possibilidades aparecem 6 vezes: x – y – z ; x – z – y;
y – x – z; y – z – x; z – x – y; z – y – x
504
Logo, devemos dividir 504 por 6, assim = 84
6

EUREKA! N°19, 2004

39
Sociedade Brasileira de Matemática

9!
Finalmente concluímos que existem 84 = tabuleiros diferentes nos
(9 − 3)!⋅ 3!
quais A > B; C > D; A > C; B > D; B = C.

a) Como não podemos definir relação entre B e C, vamos analisar três casos:
1. B > C → A > B > C > D
2. B < C → A > C > B > D
3. B = C → A > B = C > D
Para o caso 1, temos que escolher 4 números distintos de 1 a 9 e pô-los em ordem
já descrita A é o maior, o segundo maior é B e D é o menor), seguindo o raciocínio
do quesito "a" temos 9 ⋅ 8 ⋅ 7 ⋅ 6 possibilidades de escolha sendo que os mesmos 4
números se repetem em 24 escolhas ou 4!.

Nós só utilizamos a; b; c; d uma vez, mas eles ocorrem 24 vezes, apenas alterando
a ordem, logo as possibilidades se reduzem:
9 ⋅ 8 ⋅ 7 ⋅ 6 3024 9!
= = 126 =
24 24 (9 − 4)!⋅ 4!
O caso 2 terá tantas possibilidades quanto o caso 1, apenas trocando B por C e o
caso 3 já foi estudado no quesito "a". Assim, o total de tabuleiros é:
126 + 126 + 84 = 336.

PROBLEMA 2: SOLUÇÃO DE GUILHERME R. NOGUEIRA DE SOUZA (SÃO PAULO – SP)


• Vamos analizar x 2 + 5 x + 23 módulo 2.
Temos: x 2 + x + 1 ≡ 0 para ser divisível por 2.
Se x ≡ 1 → 1 + 1 + 1 ≡ 1 ; se x ≡ 0 → 0 + 0 + 1 ≡ 1
Logo 2 não é o menor primo, que vamos chamar de p.
• Vamos analisar x 2 + 5 x + 23 módulo 3.
Temos: x 2 + 2 x + 2 ≡ 0 para ser divisível por 3.
Se x ≡ 0 → 0 + 0 + 2 ≡ 2 ; se x ≡ 1 → 1 + 2 + 2 ≡ 2 ; se x ≡ 2 → 1 + 1 + 2 ≡ 1 .
Logo p não é 3.
• Vamos analisar x 2 + 5 x + 23 módulo 5.
Temos: x 2 + 3 ≡ 0 para ser divisível por 5.
se x ≡ 1 → 1 + 3 ≡ 4 ; se x ≡ 2 → 4 + 3 ≡ 7 ; se x ≡ 3 → 4 + 3 ≡ 2 ;
se x ≡ 4 → 1 + 3 ≡ 4 ; se x ≡ 0 → 0 + 3 ≡ 3 .
Logo p não é 5.
• Vamos analisar x 2 + 5 x + 23 módulo 7.
Temos: x 2 − 2 x + 2 ≡ 0 para ser divisível por 7.

EUREKA! N°19, 2004

40
Sociedade Brasileira de Matemática

se x ≡ 1 → 1 − 2 + 2 ≡ 1 ; se x ≡ 2 → 4 − 4 + 2 ≡ 2 ;
se x ≡ 0 → 0 − 0 + 2 ≡ 2 ; se x ≡ 3 → 2 − 6 + 2 ≡ 5 ; se x ≡ 4 → 2 − 1 + 2 ≡ 3 ;
se x ≡ 5 → 4 − 3 + 2 ≡ 3 ; se x ≡ 6 → 1 − 5 + 2 ≡ 5 .
Logo p não é 7.
• Vamos analisar x 2 + 5 x + 23 módulo 11.
Temos: x 2 + 5 x + 1 ≡ 0 para ser divisível por 11.
se x ≡ 1 → 1 + 5 + 1 ≡ 7 ; se x ≡ 3 → 9 + 4 + 1 ≡ 3 ;
se x ≡ 5 → 3 + 3 + 1 ≡ 7 ; se x ≡ 2 → 4 + 10 + 1 ≡ 4 ; se x ≡ 4 → 5 + 9 + 1 ≡ 4 ;
se x ≡ 6 → 3 + 8 + 1 ≡ 11 ; se x ≡ 7 → 5 + 2 + 1 ≡ 8 ; se x ≡ 9 → 4 + 1 + 1 ≡ 6 ;
se x ≡ 0 → 0 + 0 + 1 ≡ 1 ; se x ≡ 8 → 9 + 7 + 1 ≡ 6 ; se x ≡ 10 → 1 + 6 + 1 ≡ 8 .
Logo p não é 11.
• Vamos analisar x 2 + 5 x + 23 módulo 13.
Temos: x 2 + 5 x − 3 ≡ 0 para ser divisível por 13.
se x ≡ 1 → 1 + 5 − 3 ≡ 3 ; se x ≡ 5 → 12 + 12 − 3 ≡ 8 ;
se x ≡ 9 → 3 + 6 − 3 ≡ 6 ; se x ≡ 2 → 4 + 10 − 3 ≡ 11 ; se x ≡ 6 → 10 + 4 − 3 ≡ 11 ;
se x ≡ 10 → 9 + 11 − 3 ≡ 4 ; se x ≡ 3 → 9 + 2 − 3 ≡ 8 ; se x ≡ 7 → 10 + 9 − 3 ≡ 3 ;
se x ≡ 11 → 4 + 3 − 3 ≡ 4 ; se x ≡ 4 → 3 + 7 − 3 ≡ 7 ; se x ≡ 8 → 12 + 1 − 3 ≡ 10 ;
se x ≡ 12 → 1 + 8 − 3 ≡ 6 , se x ≡ 0 → 0 + 0 − 3 ≡ 10
Logo p não é 13.
Chegamos até agora que p não é 2,3,5,7,11,13.
Então, p é no mínimo 17, e 17 divide x 2 + 5 x + 23 quando
x ≡ −2 : ( −2) 2 + 5( −2) + 23 = 17.
Logo o menor primo que divide x 2 + 5 x + 23 para algum x inteiro é 17.

PROBLEMA 3: SOLUÇÃO DE ANDRÉ LINHARES RODRIGUES (FORTALEZA – CE)


Vamos dividir o problema em duas partes:
a) BX = CX ⇒ PQ é um diâmetro de S.
l = α . Assim, temos que QBC
Seja ACB l = α (já que BX = CX) e
PlAC = 180° − 90° − α = 90° − α .
Observe que os ângulos PÂC e PBCl estão "olhando" para o mesmo arco. Assim,
l = P lAC = 90 − α ⇒ PBQ
vemos que PBC l = 90 − α + α = 90° ⇒ PQ é diâmetro.

EUREKA! N°19, 2004

41
Sociedade Brasileira de Matemática

A
Q
90 – α
X

B α
α
90 – α C

b) PQ é um diâmetro de S ⇒ BX = CX .
Se l = α , P lAC = PBC
ACB l = 90° − α . Mas PQ é diâmetro, donde
l l l
PBQ = 90° ⇒ 90° − α + QBC = 90° ⇒ QBC = α ⇒ ∆BXC é isósceles
⇒ BX = XC.
A figura poderia ser um pouco diferente:

A α
180° – 2α – β
P X

180° – α – β 180° – 2α
β
α α
B C

a) Chamando ABQ l de β e C BQ l = ACB l = α , teríamos


PlAB = −90° + α + β , B lAC = 180° − 2α − β e C l AQ = α , e o ângulo P l AQ seria
−90° + α + β + 180° − 2α − β + α = 90° .
b) Chamando QBC l de α, temos que C AQ l = α . Mas PQ é diâmetro
l = 180° − 90° − (90° − α ) =
⇒ P lAC + α = 90° ⇒ P lAC + α = 90° ⇒ P lAC = 90 − α ⇒ ACB

EUREKA! N°19, 2004

42
Sociedade Brasileira de Matemática

= α ⇒ ∆BCX é isósceles ⇒ BX = CX .
A figura pode ainda ficar da seguinte forma:

A α
180°– β –α – γ
X

β
α α=γ
T B
C

l = α e X BA
a) Seja X BC l = β . Seja T a interseção entre as retas BC e AP. Temos

que T lAB = α + β − 90° , B lAC = 180° − β − 2α e C lAQ = CBQ


l = α. Então,
P lAQ = 180° − α − (180° − β − 2α ) − (α + β − 90°) = 90° ⇒ PQ é diâmetro de S.
b) Seja l =α ,
QBC l =β
ABQ e l =γ.
ACB Então teremos que
B lAC = 180° − β − α − γ , QAC
l = α e B lAT = α + β − 90°. Como PQ é diâmetro,

P lAQ = 90° ⇒ QAT


l = 180° − 90° = 90° ⇒ α + β − 90° + 180° − β − α − γ + α = 90°
⇒ γ = α ⇒ ∆BXC é isósceles ⇒ BX = CX .

PROBLEMA 4: SOLUÇÃO DE MARICY MIKI HISAMOTO (SÃO PAULO – SP)


x +

2
4 y − 4xy + 2 x − 4 y + 2 > 0,

x2 −2⋅x⋅2 y +(2 y )2

( x − 2 y ) 2 + (2 x − 4 y ) + 1 + 1 = ( x − 2 y ) 2 + 2( x − 2 y ) + 2 = [( x − 2 y ) + 1]2 + 1

Como qualquer real elevado ao quadrado resulta em um número positivo,


[( x − 2 y ) + 1]2 é positivo para quaisquer x e y reais. Assim, [( x − 2 y ) + 1]2 + 1
também será positivo, logo x 2 + 4 y 2 − 4 xy + 2 x − 4 y + 2 é maior que 0.

PROBLEMA 5: VEJA A SOLUÇÃO DO PROBLEMA 4 DO NÍVEL 3

EUREKA! N°19, 2004

43
Sociedade Brasileira de Matemática

PROBLEMA 6: SOLUÇÃO DE EDUARDO FISCHER (LAJEADO – RS)


Vamos meter uma indução:
Se N = 1, 2, 3 é verdadeiro: qualquer caminho serve.
Se N > 3, separamos o ponto PN e consideramos (usando a hipótese de indução)
um caminho fechado passando pelas outras cidades, digamos P1 P2 ...PN −1 P1 ,
mudando o meio de transporte no máximo uma vez. Temos duas possibilidades:
a) O caminho usa só um meio de transporte. Nesse caso, o caminho fechado
P1 P2 ...PN P1 ou o PN P1...PN −1 muda de meio de transporte no máximo uma vez.
b) O caminho muda de meio de transporte exatamente uma vez, digamos de PN −1
para P1 (quando muda de rodovia para ferrovia, sem perda de generalidade).
Temos então os seguintes caminhos fechados, em cada caso abaixo:
b.1) PN −1 PN e PN P1 são ferrovias: P1 P2 ...PN −1 PN P1
b.2) PN −1 PN e PN P1 são rodovias: PN −1 PN P1 P2 ...PN −1
b.3) PN −1 PN é rodovia e PN P1 é ferrovia: P1 P2 ...PN −1 PN P1
b.4) PN −1 PN é ferrovia e PN P1 é rodovia: PN P1 P2 ...PN −1 PN
Pelo princípio indutivo concluímos que para qualquer número possível de cidades
em Tumbólia, nosso turista poderá visitá-las mudando o meio de transporte no
máximo uma vez.

SOLUÇÕES – NÍVEL 3

PROBLEMA 1: VEJA A SOLUÇÃO DO PROBLEMA 2 DO NÍVEL 2.

PROBLEMA 2:
ADAPTADA DA SOLUÇÃO DE MURILO VASCONCELOS DE ANDRADE (MACEIÓ - AL)
Vamos mostrar que kn = log 2 n  + 1 , ou seja, se 2k ≤ n < 2k +1 então kn = k + 1 ,
por indução em k. Primeiro vamos ver que kn ≤ k + 1. De fato, k1 = 1 e, dado n.
n n
Sejam dados conjuntos A1 , A2 ,..., Ak +1 ⊂ S . Como # Ak++1 ≥   ou # Ak−+1 ≥   ,
2 2
n n
pois Ak−+1 ∪ Ak++1 = S , segue que # ( S \ Ak++1 ) ≤   < 2k ou # ( S \ Ak−+1 ) ≤   < 2k .
2 2
Supondo sem perda de generalidade que # ( S \ Ak +1 ) < 2 , por hipótese de indução,
+ k

trocando Aj por iA j := A j ∩ ( S \ Ak++1 ) , para 1 ≤ j ≤ k , temos

EUREKA! N°19, 2004

44
Sociedade Brasileira de Matemática

S \ Ak +1 = i i 2± ∪ ... ∪ A
i k± , para alguma escolha dos sinais + e – , donde
±
A1 ∪ A
S = A1± ∪ A2± ∪ ... ∪ A2± ∪ Ak++1 , para a mesma escolha dos k primeiros sinais.
Por outro lado, se n ≥ 2k e, para 1 ≤ i ≤ k ,
 k −1 
Ai = ∑ σ j ⋅ 2 j σ j ∈ {0,1}, ∀j < k e σ i −1 = 1 ⊂ {0,1,..., n − 1} é o conjunto dos
 j =0 
k
naturais menores que 2 cujo (i – 1)-ésimo algarismo binário é igual a 1,
A1* ∩ A2* ∩ ... ∩ Ak* é não vazio para qualquer escolha dos sinais, donde
A1± ∪ A2± ∪ ... ∪ Ak± ≠ S = {0,1,..., n − 1} , para qualquer escolha dos sinais. Isso
mostra que kn > k , e portanto kn > k .

PROBLEMA 3: SOLUÇÃO DE RAFAEL MARINI SILVA (VILA VELHA – ES)


Não é difícil ver que o centro O da circunferência inscrita ao losango ABCD é
encontro das diagonais. Se B lAD = 2θ = BCDl , vamos mostrar que AH = AE , ou

\
CF CG
ainda, AH ⋅ CG = CF ⋅ AE = K ∈ constante; o que implica o resultado, pois os
triângulos AEH e CGF são semelhantes, e logo EH é paralelo a FG.
A Sendo I, J, K, L, P, P' os pontos de encontro
da circunferência inscrita com AD, DC, CB,
θ θ
I L AB, EF e GH respectivamente temos:
E H
l
I l = EOP
OE l = I OP (pois EI e EP são
O
D
P θ θ P' B 2
tangentes comuns à circunferência)
l
F G J l = FOP
OF l = J OP
J
θ θ
K 2
l l l l + F OP
I OJ = J OP + I OP = 2( POE l )=
C
l = D lAB = 2θ ⇒
l ) = 180° − I DF
2( EOF
l =θ,
⇒ EOF
l =θ.
analogamente temos: H OG
Destacando o triângulo ABC temos:

EUREKA! N°19, 2004

45
Sociedade Brasileira de Matemática

l = X , temos:
Fazendo BOG
A
l = 90° − X
AHO
θ l = 90° − θ + X
AOH
d OGC l
l = 90° − θ + X = AOH
2 90°– X
H
COG l
l = 90° − X = AHO
90°– θ + X l = O AH
OCG l
O θ–X
θ
X
B Logo ∆AOH ~ ∆CGO e
AH AO d2
⇒ AH ⋅ CG = AO ⋅ CO = ( AO ) =
2
90°– X = .
d CO CG 4
90°– θ + X
2 G Analogamente, temos
d2 AC
θ AE ⋅ CF = , constante pois d = é
4 2
C constante.

PROBLEMA 4: SOLUÇÃO DE FÁBIO DIAS MOREIRA (RIO DE JANEIRO - RJ)


Seja Γ a circunferência do problema. Suponha que B, C , D ∈ Γ são os pontos

F
que maximizam a área de ABCD (obviamente, ABCD é convexo). Então C é
ponto médio de BD : caso não fosse, considere M, ponto médio de BD . Então F
F HJJG HJJG
S ( ABMD ) = S ( ABD ) + S ( BMD ) > S ( ABD ) + S ( BCD ) = S ( ABCD ), pois, como M
é médio de BD , d ( M , BD) > d (C , BD) . Como ∆BMD e ∆BCD têm a mesma
HJJG
base, S ( BMD ) > S ( BCD ). Considere agora { A '} = (Γ ∩ AC )\{C}. Pelo mesmo

HJJG
argumento, B e D são pontos médios dos seus respectivos arcos A ' C.
HJJJGG HJJG
Em particular, isso implica que BD é a mediatriz de A ' C ⇒ O ∈ BD
F (em
HJJG C é médio de BD , e O é
particular, O é ponto médio de BD ). Além disso, como
médio de BD , CO ⊥ BD. Mas A ' C ⊥ BD , pois BD é mediatriz de A ' C . Em

JJJG
particular, AC ⊥ BD . Mas OC ⊥ BD ⇒ O, A, C são colineares.
Construção: Trace a semi-reta AO : ela intersecta Γ em C (pois o polígono deve
ser convexo com A, O, C colineares, logo esta é a única possível posição de C).
Como BD é ortogonal a AC em O, a perpendicular a AC por O determina B e D
- estes B e D são os que maximizam a área do quadrilátero #ABCD.

EUREKA! N°19, 2004

46
Sociedade Brasileira de Matemática

PROBLEMA 5: SOLUÇÃO DE ALEX CORRÊA ABREU (NITERÓI - RJ)

1 1
f  + f 
1 1  2   x  y  defina agora
Em (iii) troque x por e y por ⇒ f ≥
x y  x+ y 2
1
g ( x) = f   ⇒
 x
i) x < y ⇒ g ( x) > g ( y )

 x + y  g ( x) + g ( y)
ii) g ≥
 2  2
x+ y g (2 z − y ) + g ( y )
Fazendo z = ⇒ g ( z) ≥
2 2

Fazendo agora y = 1 ⇒ 2g ( z ) ≥ g (2z − 1) + g (1) ⇒ g (2z − 1) ≤ 2g ( z) − g (1) ∀z > 1


2( )
mas 2 z > 2 z − 1 ⇒ g (2 z − 1) ≥ g (2 z ) ⇒ g (2 z ) ≤ 2 g ( z ) − g (1). Vamos agora provar
por indução que g (2n z ) ≤ 2n g ( z) − (2 n − 1) g (1) = 2n ( g ( z ) − g (1)) + g (1) :

se g (2k z) ≤ 2 k g ( z ) − (2 k − 1) g (1) ⇒ g (2k +1 z ) ≤ 2 g (2k z) − g (1) ≤

2k +1 g ( z ) − (2k +1 − 2) g (1) − g (1) = 2k +1 g ( z ) − (2k +1 − 1) g (1) .

Se tomarmos z > 1, g ( z ) − g (1) < 0 ⇒ ∃ n tal que 2n ( g ( z ) − g (1)) + g (1) < 0 ⇒


1
∃x tal que g ( x) < 0 ⇒ ∃y = tal que f ( y ) < 0.
x
PROBLEMA 6: Solução adaptada das soluções de Davi Máximo Alexandrino Nogueira
e Rafael Daigo Hirama
A idéia é contar o número de pares (f; D), sendo f e D função e conjunto
correspondentes a um grafo bacana e comparar com o número de grafos com n
vértices.
Cada par (f; D) gera no máximo um grafo: de fato, dada f, ligamos os vértices p e
q se, e somente se, |f(p) – f(q)| ∈ D. Caso ocorra alguma contradição entre os pares
de vértices, o par não gera um grafo. Assim, basta provar que a quantidade de
grafos de n vértices é maior que a quantidade de pares (f; D), pois isso
demonstraria a existência de um grafo que não é gerado por nenhum par e,
portanto, não bacana.

EUREKA! N°19, 2004

47
Sociedade Brasileira de Matemática

No que se segue, x denota o maior inteiro menor que ou igual a x. Há


  n 2  !
 4  2 n
    ≤  n  funções injetoras de V, com n elementos, para [1, n2/4] ∩
  n 2  − n !  4 
 4  
  
N e, como podemos tomar D ⊂ [1, n2/4] ∩ 1, (senão, trocamos D por sua
 n2 
  n2
 4 
interseção com 1, n 4 ) temos no máximo 2
2
≤ 24 conjuntos D. Logo o
n 2
 n2  n
número de pares (f; D) é menor que ou igual a   ⋅2 4 .
 4 
 
n n 2 −n
 
2
Além disso, há 2 = 2 2
grafos com n vértices dados. Assim, basta mostrar
que para todo n suficientemente grande,
n2 −n n n2 n2 n n n−2
 n2  − 2 n log 2 n−2 n n
2 2 >   ⋅2

4 ⇔ 24 2 >2 2 ⇔ > log 2 ⇔ 2 8 > ,
 4  8 2 2
o que é verdade para n > 58 (sendo n > 58,
n−2 n−2
n−2
n−2
⋅ (n −8 2 − 1) n
2 8 = (1 + 1) 8 > 1+ + 8
> ). Na verdade, poderíamos
8 2 2
simplesmente notar que uma função exponencial cresce muito mais que qualquer
função polinomial, que por sua vez, cresce muito mais que qualquer função
logarítmica.
De fato, quando n fica muito grande quase nenhum grafo é bacana. Isto quer dizer
que, embora existam grafos bacanas com um número qualquer de vértices (como,
por exemplo, o grafo completo e o grafo vazio), a probabilidade de um grafo ser
bacana fica muito próxima de zero quando n é muito grande.

EUREKA! N°19, 2004

48
Sociedade Brasileira de Matemática

XXV OLIMPÍADA BRASILEIRA DE MATEMÁTICA


Problemas e Soluções da Primeira Fase – Nível Universitário

PROBLEMA 1
Seja X ⊆ 5 3 o poliedro convexo cujos vértices são todos os pontos ( x, y , z) ∈ =3
com x 2 + y 2 + z 2 = 2. Calcule o volume de X.

PROBLEMA 2
O tenista Berrando Gemigemi tem 30 dias para preparar-se para um torneio. Se ele
treina 3 dias seguidos ele tem fadiga muscular. Ele, então, decide que, durante
esses 30 dias, irá treinar 20 dias, sem nunca treinar 3 dias seguidos, e descansar
nos outros 10 dias. De quantas maneiras diferentes ele pode escolher os 10 dias de
descanso?

PROBLEMA 3
Sejam A e B matrizes reais n × n inversíveis. Mostre que se vale a condição
( AB) k = Ak B k para três valores inteiros consecutivos de k então AB = BA.

PROBLEMA 4
1 1 1 π2
Sabemos que ∑k
k >0
2
=1+ +
22 32
+ ... =
6
. Defina

1 1 1
f (n) = ∑ 2 = 1 + 2 + ... + 2 .
0< k ≤ n k 2 n
Prove que existe um número real a > 0 tal que existe o limite:
 π2 a 2
lim  f (n ) − + ⋅n .
n →∞
 6 n
Calcule a e este limite.

PROBLEMA 5
Sejam a e n inteiros, n > 1, mdc(a, n) = 1.
Prove que o polinômio (1 n ) (( X + a ) n − X n − a ) tem todos os coeficientes
inteiros se e somente se n é primo.

EUREKA! N°19, 2004

49
Sociedade Brasileira de Matemática

PROBLEMA 6
Defina a1 = 3, an +1 = an2 − 2.
log log an
Prove que lim = log 2 e calcule lim ( log log an − n log 2).
n →∞ n n →∞

(Observação: os logaritmos estão todos na base e).

SOLUÇÕES – NÍVEL UNIVERSITÁRIO


SOLUÇÃO DO PROBLEMA 1:
Os vértices de X são os doze pontos (±1, ±1,0),(±1,0, ±1),(0, ±1, ±1),
que são os pontos médios das arestas do cubo (±1, ±1, ±1) , donde X é obtido a
partir do cubo tirando fora uma pirâmide (ou tetraedro) em cada vértice.

O volume do cubo é 23 = 8.
Cada pirâmide tem base de área 1/2 e altura 1 logo tem volume igual a 1/6.
8 20
Assim o volume do sólido é igual a 8 − =
6 3

SOLUÇÃO DO PROBLEMA 2:
Podemos dividir os 30 dias em 10 blocos de três dias.
É claro que ele deverá descansar em exatamente um dia por bloco.
Se alguma vez ele descansa no dia central de um bloco depois disso ele não poderá
descansar no último dia de um bloco; analogamente, se alguma vez ele descansa
no primeiro dia de um bloco ele deverá descansar no primeiro dia de todos os
blocos que vierem depois. Assim, ele deve descansar no último dia nos primeiros

EUREKA! N°19, 2004

50
Sociedade Brasileira de Matemática

x blocos, depois descansar no dia central durante y blocos e finalmente descansar


no primeiro dia nos últimos z blocos, onde x + y + z = 10,
x, y, z ≥ 0, x, y, z ∈ =.
12 
O número de soluções é   = 66.
2 

SOLUÇÃO DO PROBLEMA 3:
Suponha que ( AB) k = Ak B k , ( AB) k +1 = Ak +1 B k +1 e ( AB) k +2 = Ak +2 B k +2 . Temos
então Ak +1 B k +1 = ( AB) k +1 = ( AB )( AB) k = AB Ak B k , e logo (multiplicando à
esquerda por A−1 e à direita por B − k ) obtemos Ak B = BAk .
Analogamente, usando a segunda e a terceira igualdades, obtemos Ak +1 B = BAk +1
Assim temos BAk +1 = Ak +1 B = A ⋅ Ak B = A ⋅ BAk , donde, multiplicando à direita por
A−k , obtemos BA = AB.

SOLUÇÃO DO PROBLEMA 4:
π2 1 1 1
Temos − f (n ) = + + + ...
6 (n + 1) 2
(n + 2) 2
(n + 3) 2
Podemos obter uma boa estimativa para esta soma estimando a área sob o gráfico
1
de y = 2 , x ≥ n , pela regra dos trapézios:
x
y

n n+1 n+2 n+3 x

EUREKA! N°19, 2004

51
Sociedade Brasileira de Matemática

1 ∞ 1
A área exata é
t∫
n2
dt = .
n
A área obtida pela aproximação, que é ligeiramente maior, é
1 1 1  1 1 1 
 2+ +  +  + ... =
2  n ( n + 1)  2  ( n + 1) (n + 2) 
2 2 2

1 1 1 1 1 1 π2
= + + + + ... = + − f (n )
2 n 2 ( n + 1)2 ( n + 2 )2 ( n + 3)2 2n 2 6
 π2 1 2 1
Donde  f ( n) − + n <
 6 n 2
Assim a = 1 e, se acreditarmos que esta aproximação é boa, teremos que o limite é
igual a 1/2.
Para demonstrarmos que o erro é realmente pequeno, devemos estimar a diferença
entre as áreas:
1  1 1  n +1 1 1  1 1  1 1 1
⋅ 2 + 2 
−∫ =  2+ 2 
− = 2 < 4
2 n (n + 1)  n t 2
2 n ( n + 1)  n(n + 1) 2n ( n + 1) 2
2n
Assim
π2 1 1 1 1 1 
0< − f (n ) − + 2 <  4 + + ... 
6 n 2n 2n ( n + 1) 4

1 ∞ 1 1 1
< ∫ 4 dt =
2 n −1 t 6 (n − 1)3
e com isso
1 1 n2  π2 1 2 1
− ⋅ <  f ( n ) − + n < ,
2 6 ( n − 1) 3  6 n 2
o que confirma que o limite é igual a 1/2.

SOLUÇÃO DO PROBLEMA 5:
n
n
Temos ( X + a)n = ∑  an−k X k . Assim
k =0  k 

 
n −1 n
( X + a)n − X n − a = ∑   a n− k X k + (a n − a).
k =1  k 

 n  n(n − 1)...(n − k + 1)
Se n é primo e 1 ≤ k ≤ n −1,   = é múltiplo de n, pois o
k k!
numerador é múltiplo de n mas o denominador não, e, pelo pequeno Teorema de

EUREKA! N°19, 2004

52
Sociedade Brasileira de Matemática

Fermat, a n − a é múltiplo de n. Assim, nesse caso, o polinômio


1
  (( X + a) − X − a ) tem todos os coeficientes inteiros.
n n

 n
Se n é composto, seja q um fator primo de n. Temos então que
 n  n(n − 1)...(n − q + 1)
 = não é múltiplo de n. De fato, se q k é a maior potência de
 
q q !
n
q que divide n, a maior potência de q que divide   é q k −1 , pois o único fator
q
múltiplo de q no numerador é n e o único fator múltiplo de q em q! = q(q – 1)… .
1
2 . 1 é q. Assim, nesse caso, o coeficiente de X q em   (( X + a)n − X n − a ) não é
 n
inteiro.

SOLUÇÃO DO PROBLEMA 6:
A recursão é satisfeita por an = α 2 + α −2 para qualquer número real α , de fato,
n n

( ) −2
2 n+1 n+1 n+1 n+1
por indução an +1 = an2 − 2 = α 2 + α −2 = α 2 + 2 + α −2 − 2 = α 2 + α −2 .
n n

1+ 5
Para a seqüência do problema, basta resolver α 2 + α −2 = 3 que tem raiz α = .
2
2n 2n
 1 + 5   −1 + 5 
Assim an =   +   . Assim,

 2   2 
2n
 1+ 5  1+ 5 
an =   + ε (n), onde lim ε ( n ) = 0 donde log an = 2 n ⋅ log   + ε 1 (n ),
n→ ∞
 2   2 
1+ 5 
lim ε 1 ( n ) = 0 e log log a n = n log 2 + log log 
 2  + ε 2 ( n ), n → ∞ ε 2
lim (n ) = 0
n→∞
 
loglog an 1+ 5 
Assim lim = log2 e lim (log log an − n log 2 ) = log log  .
n →∞ n →∞
 2 
n

EUREKA! N°19, 2004

53
Sociedade Brasileira de Matemática

XXV OLIMPÍADA BRASILEIRA DE MATEMÁTICA


Problemas e Soluções da Segunda Fase – Nível Universitário

PROBLEMA 1:
São dados uma parábola e um ponto A fora dela. Para cada ponto P da parábola,
seja t a tangente à parábola por P e r a reta paralela ao eixo da parábola por P. A
reta perpendicular a t por A corta r em Q. Prove que, ao variar P, o ponto Q
percorre uma hipérbole equilátera.

PROBLEMA 2:
a) Sejam p e q ∈ &[ x ] polinômios primos entre si com coeficientes complexos.
Suponha que existam 4 vetores (a, b) ∈ &2 , dois a dois linearmente
independentes (sobre & ), tais que ap + bq é o quadrado de um polinômio em
&[ x] . Prove que p e q são constantes.
b) Prove que não existem polinômios não constantes r, s, t, u ∈ &[ x ] tais que

f = , g = e f 2 = g ( g − 1)( g − a ), onde a ∈ & , a ≠ 0 , a ≠ 1.


r t
s u

PROBLEMA 3:
Seja p > 2 um número primo.
Seja Xp o conjunto de todas as matrizes quadradas A com coeficientes em = ( p) e

{
de ordem 4 para as quais A2 = I : X p = A ∈ ( = ( p) )
4× 4
A2 = I }
Calcule o número de elementos de Xp.

Observação: = ( p ) = {0,1, 2,..., p − 1} é o corpo finito com p elementos. A soma e


o produto são definidos módulo p; assim, por exemplo, em = (7) , 4 + 5 = 2 e 4 ⋅
5 = 6.

PROBLEMA 4:
Temos um dado de 6 faces, não necessariamente honesto. Jogamos o dado três
vezes e obtemos resultados a, b e c. Prove que P( a = c | a = b) ≥ P( a = c | a ≠ b) e
que vale a igualdade se e somente se o dado é honesto.

Observação: P( a = c | a = b) é a probabilidade condicional


P ( a = b = c)
P( a = c | a = b) = .
P( a = b)

EUREKA! N°19, 2004

54
Sociedade Brasileira de Matemática

1
Um dado é honesto se a probabilidade de cada face é .
6

PROBLEMA 5:
Uma função f : (−1,1) → 5 de classe C ∞ é bacana se existem um inteiro positivo
n e polinômios Pj ∈ 5[ t ], 0 ≤ j ≤ n, com Pn não identicamente nulo tais que
n

∑ P (t ) f
j =0
j
( j)
( t ) = 0, para todo t ∈ ( −1,1). Prove que se f e g são bacanas então

f + g e f ⋅ g também são bacanas.

Observação: Definimos f (0) = f e, para cada inteiro m ≥ 0, f ( m +1) = ( f ( m) ) '.

PROBLEMA 6:
Seja A = (aij )1≤i , j ≤n uma matriz tal que aij ∈{0,1}, para quaisquer i, j e
k
99 2 n
{(i, j ) | aij = 1} ≥ ⋅ n . Prove que tr ( Ak ) ≥   , para todo k ≥ 2.
100 2
n
Observação: Se B = (bij )1≤i, j ≤n é uma matriz quadrada então tr ( B) = ∑ bii denota
i =1
o traço de B.

SOLUÇÕES - NÍVEL UNIVERSITÁRIO


PROBLEMA 1: SOLUÇÃO DE LEONARDO AUGUSTO ZÃO (NILÓPOLIS – RJ)
Dada a parábola γ , fixamos os eixos coordenados de forma que tenhamos a
equação da parábola dada por γ : y = ax 2 . Supomos que A = (m, n) e P = ( x p , y p ).
Como p ∈ γ ⇒ y p = ax p2 ⇒ P = ( x p , ax p2 ).
Pelo enunciado, a reta t tem coeficiente angular 2ax p , e a reta perpendicular a t
−1
tem coeficiente angular .
2ax p
Assim, a reta perpendicular a t que passa por A é dada pela equação:
−1
(*) t1 = ( y − n) = ( x − m).
2ax p

EUREKA! N°19, 2004

55
Sociedade Brasileira de Matemática

A reta r é dada por r : x = x p


−1 m 1
Assim: Q = t1 ∩ r = ( x p , ( x p − m) + n) = ( x p , − + n)
2ax p 2ax p 2a
m 1 m 1
Em Q, variando x p , teremos: y = − + n. Se =q e n− = k , então:
2ax 2a 2a 2a
q q
y= + k ⇒ ( y − k) = ⇒ ( y − k) ⋅ x = q .
x x
Assim, se deslocarmos o eixo y para y – k, teremos a hipérbole equilátera:
m
yx = q, onde q = .
2a

PROBLEMA 2: SOLUÇÃO DE MÁRCIO AFONSO ASSAD COHEN (RIO DE JANEIRO – RJ)


a) Escrevendo a i ⋅ p + bi ⋅ q = v i2 , i = 1, 2,3,4 , temos que toda raiz a de vi é dupla
de ai p + bi q,:
 a i p(a ) + bi q (a) = 0  xp(a ) + yq(a ) = 0
 . Portanto o sistema  ,
a i p ' (a ) + bi q ' (a ) = 0  xp' (a ) + yq' (a ) = 0

é indeterminado (já que (ai, bi)    LVWR p p(a )q' (a) − q (a) p' (a) = 0 e as
soluções podem ser parametrizadas de modo que (a i , bi ) = t ⋅ (q(a),− p (a)) .
Como os (ai, bi) são 2 a 2 LI, os vi’s são 2 a 2 co-primos (porque se a fosse raiz de
vi e de vj, então (ai, bi) e (aj, bj) seriam ambos múltiplos de (q(a), – p(a)).
Além disso, toda raiz de vi é também raiz de h = pq'− qp' . Mais geralmente, se a
tem multiplicidade k como raiz de vi então a é raiz de h com multiplicidade
2k – 1 ≥ k. (pois, para cada n, a n-ésima derivada de n é uma soma de termos do
tipo p ( j ) q ( n − j ) − p ( n − j ) q ( j ) , que se anulam em a se n ≤ 2k − 1).
Vou mostrar agora que h é identicamente nulo, de modo que a derivada de p /q é
zero.
Seja m = grau (p), n = grau (q):

1o caso: m > n:
Cada vi tem grau m/2, exceto possivelmente um que pode ter grau n/2 (se existir
algum ai nulo) e portanto os vi’s contribuem com pelo menos 3m/2 + n/2 raízes de
h.

EUREKA! N°19, 2004

56
Sociedade Brasileira de Matemática

3m + n
Por outro lado, grau (h) = grau ( pq '− qp ' ) ≤ m + n − 1 < m + n < , onde a
2
última igualdade decorre de m > n. Logo, h ) 

2o caso: m = n:
Aqui pode ocorrer de algum polinômio aip + biq ter grau menor que n (como os
pares são 2 a 2 LI, isso ocorre no máximo uma vez).
Mas se ai p( x) + bi q ( x ) = Tl ( x ) , onde Tl é de grau l < n, então ai   H
1 1
h = pq '− p ' q = [( −bi q + Tl ) q '− ( −bi q '+ T 'l ) q ] = (Tl q '− T 'l q)
ai ai
tem grau ” l + n – 1.
l n
Por outro lado, os polinômios ai p + bi q estão contribuindo com + 3 raízes de
2 2
l + 3n
h. Como grau (h) ≤ l + n − 1 < l + n ≤ (já que l ≤ n), h é identicamente
2
nulo.
'
 p h p( x)
Portanto,   = 2 ≡ 0 → = cte . Como p e q são primos entre si, ambos
 
q q q ( x )
são constantes.

b) Substituindo f = r /s e g = t / u, veja que mdc(t, u) = 1 implica que os


polinômios t, u, t – u, t – au são 2 a 2 primos entre si (se dois deles tivessem uma
raiz comum α, é fácil ver que α seria raiz de t e de u).
Substituindo em f2 = g(g – 1)(g – a) temos: r 2 u 3 = s 2 t (t − u )(t − au ) . Como os
polinômios t, t –u, t – au são primos com u, eles devem ser divisores de r2. Como
eles são 2 a 2 primos entre si, cada um deve ser um quadrado perfeito (pois todas
as raízes de r2 e s2 tem multiplicidade par e s é primo com r).
Analogamente, como u3 é primo com t, t – u e t – au, ele deve ser divisor de s2.
Logo, u só tem raízes de multiplicidade par (se a fosse raiz com multiplicidade
ímpar de u, também o seria de u3 e portanto de s2) e portanto é quadrado de um
polinômio complexo.
Ou seja, escrevemos u, t, t – u e t – au como quadrados de polinômios, o que
contradiz a letra (a), pois os vetores (1, 0), (0, 1), (–1, 1), (– a, 1) são 2 a 2 LI (a 
0 e a   

EUREKA! N°19, 2004

57
Sociedade Brasileira de Matemática

PROBLEMA 3: SOLUÇÃO DE EDUARDO CASAGRANDE STABEL (PORTO ALEGRE – RS)


] p ]) um espaço vetorial.
Seja p um primo fixado ( p > 2) e V = ( 4

Proposição 1: Se A é uma matriz 4 × 4 com coeficientes em ] e A p


2
= I é uma
involução então V = E ⊕ F onde E = {u ∈ V : Au = u} e F = {u ∈V : Au = −u}

Demonstração: Suponha que A é involução. É claro que os espaços E e F do


enunciado só possuem o vetor nulo em comum, pois se Au = u e Au = – u
então u = – u e
u = (0, 0, 0, 0). Mostremos que E + F = V. Seja v ∈V qualquer. Então v = e + f
( Av + v ) (− Av + v )
. Temos Ae = ( A2 v + Av ) = (v + Av ) = e e
1 1
onde e = e f =
2 2 2 2
Af = (− A v + Av ) = ( −v + Av ) = − ( − Av + v ) = − f , logo e ∈ E e f ∈ F .
1 2 1 1
2 2 2
Suponha que V = E ⊕ F . Definiremos uma aplicação L : 4p → 4p tal que ] ]
L (v ) = e − f onde v = e + f é a decomposição única com e ∈ E e f ∈ F .
Afirmo que L é linear. Se v1 = e1 + f1 e o2 = e2 + f 2 então
L(o1 + o2 ) = L(e1 + e2 + f1 + f 2 ) = (e1 + e2 ) − ( f1 + f 2 ) = (e1 − f1 ) + (e2 − f 2 ) = L(v1 ) + L(v2 ).
]
Caso α ∈ p e v ∈V então L (α v ) = L (α e + α f ) = α e − α f = α L (v ). Mais ainda, L
é uma involução. De fato, se v = e + f ∈V então
L (v) = L ( Lv ) = L (e − f ) = e − (− f ) = e + f = v. Logo existe uma matriz A tal que
2

L (v ) = Av para todo v sendo A involução.


Deste modo, para cada matriz existe uma aplicação e vice-versa. Portanto o
número de matrizes A é igual ao número de decomposições V = E ⊕ F .
]
Proposição 2: Se E é subespaço de V = 4p e dim E = k então # E = p k .
Demonstração: Dois subespaços de mesma dimensão são isomorfos, basta contar
um subespaço específico de dimensão k. Se ei = (0,...,1,...,0) é o vetor canônico e
E = gerado (e1 ,..., ek ) é fácil ver que # E = p k .

Proposição 3: Seja M um espaço vetorial sobre ] p cuja dimensão é dim M = m.


m −1 m − n +1
Caso 1 ≤ n ≤ m, existem ( p − 1)( p m
− 1)...( p − 1) /( p n − 1)( p n −1 − 1)...( p − 1)
subespaços de M de dimensão n.

Demonstração: Faremos escolhas de n retas (subespaços de M de dimensão um)


ordenadas (r1 , r2 ,..., rn ) tal que cada ri +1 não esteja contida no subespaço gerado

EUREKA! N°19, 2004

58
Sociedade Brasileira de Matemática

por {r1 , r2 ,..., ri }. Para a primeira reta existem ( p n − 1) ( p − 1) escolhas possíveis,


pois duas retas quaisquer tem p elementos e só o vetor zero em comum. Ou seja,
retirando-se o zero (dá p n − 1 elementos), repartimos este conjunto em
( p n − 1) ( p − 1) cada um dos quais com os elementos de uma reta, sem o vetor
nulo. Com um argumento similar, chegamos a ( p n − p ) ( p − 1) possibilidades para
a segunda reta, ( p n − p 2 ) ( p − 1) para a terceira e assim por diante. São
( p m − 1)( p m − p)( p m − p 2 )...( p m − p n −1 )
possibilidades.
( p − 1) n
Cada escolha (r1 , r2 ,..., rn ) gera um subespaço N = gerado {r1 ,..., rn } de M de
dimensão n. Dado um N ⊂ M subespaço de dimensão n, por um argumento
( p n − 1)( p n − p )( p n − p 2 )...( p n − p n −1 )
similar, é gerado por escolhas
( p − 1) n
(r1 , r2 ,..., rn ) em N tal que ri +1 não está contido no espaço gerado {r1 , r2 ,..., ri }. Logo
existem
( pm −1)( pm − p)...( pm − pn−1) ( pn −1)( pn − p)...( pn − pn−1 ) ( pm −1)( pm−1 −1)...( pm−n+1 −1)
=
( p −1)n ( p −1)n ( pn −1)( pn−1 −1)...( p −1)
subespaços N ⊂ M de dimensão n.
A estratégia será contar o número de decomposições do espaço V em somas
diretas V = E ⊕ F .
Caso (dim E = 0 e dim F = 4) ou (dim E = 4 e dim F = 0). Contam-se as
seguintes possibilidades:
E = {0} e F = V ou E = V e F = {0}.
Assim, nesse caso, o número de possibilidades é 2 (o leitor pode verificar que
essas decomposições correspondem a A = I e A = – I ) (1)
Caso (dim E = 1 e dim F = 3) ou (dim E = 3 e dim F = 1) : Façamos só o caso
dim E = 3 e dim F = 1 , o outro é análogo. Pela proposição 3, existem
( p 4 − 1)( p3 − 1)( p 2 − 1) ( p 4 − 1)
= = 1 + p + p2 + p3
( p − 1)( p − 1)( p − 1)
3 2
p −1
( p 4 − p3 )
escolhas para o espaço E. Em V – E, restam ( p 4 − p 3 ) elementos e
( p − 1)
retas, portanto são ao todo ( p 4 − 1)( p 4 − p3 ) ( p − 1) 2 escolhas para E e F.
Contando também o caso dim E = 1 e dim F = 3 , temos:

EUREKA! N°19, 2004

59
Sociedade Brasileira de Matemática

( p 4 − 1)( p 4 − p 3 )
2 = 2(1 + p + p 2 + p 3 ) p 3 = 2 p 6 + 2 p 5 + 2 p 4 + 2 p3 (2)
( p − 1) 2
( p 4 − 1)( p3 − 1)
Caso (dim E = 2 = dim F ). Novamente pela proposição 3, existem
( p 2 − 1)( p − 1)
escolhas para E. Escolhido E, fazemos um procedimento análogo ao da
demonstração da proposição 3, escolhendo duas retas (r1 , r2 ) fora de E tal que r2
não está no subespaço gerado por r1 e E, para contar as possibilidades de F.
( p 4 − p 2 )( p 4 − p3 )
Resultam escolhas para (r1 , r2 ) de onde resultam para E e F:
( p − 1) 2
( p4 −1)( p3 −1) ( p4 − p2 )( p4 − p3 ) ( p2 −1)( p2 − p) ( p4 −1)( p3 −1) p2 ( p2 −1) p3 ( p −1)
⋅ = 2 ⋅
( p2 −1)( p − 1) ( p − 1)2 ( p −1)2 ( p −1)( p − 1) ( p2 − 1) p( p −1)
p4 ( p2 −1)( p2 +1)( p −1)( p2 + p +1) 4 4 3
= = p ( p + p + 2 p2 + p +1) = p8 + p7 + 2 p6 + p5 + p4 (3)
( p2 −1)( p −1)
Por fim, somando (1) + (2) + (3):
2 + (2 p6 + 2 p5 + 2 p4 + 2 p3 ) + ( p8 + p7 + 2 p6 + p5 + p4 ) = p8 + p7 + 4 p6 + 3p5 + 3p4 + 2 p3 + 2.

PROBLEMA 4: SOLUÇÃO DE YURI GOMES LIMA (FORTALEZA – CE)


Sejam x1 , x2 , x3 , x4 , x5 , x6 as probabilidades. Temos então que:
6
P(a = b) = x1 ⋅ x1 + x2 ⋅ x2 + ... + x6 ⋅ x6 = ∑ xi2
i =1
6
P(a ≠ b) = 1 − P(a = b) = 1 − ∑ xi2
i =1

P(a = b = c) = x1 ⋅ x1 ⋅ x1 + ... + x6 ⋅ x6 ⋅ x6 = ∑ xi3


P ( a = c e a ≠ b) = P ( a = c) − P ( a = b = c ) = P ( a = b) − P ( a = b = c) ⇒
⇒ P(a = c e a ≠ b) = ∑ xi2 − ∑ xi3
Logo queremos mostrar que:
P ( a = b = c) P ( a = c e a ≠ b)
P ( a = c | a = b) ≥ P ( a = c | a ≠ b) ⇔ ≥ ⇔
P ( a = b) P ( a ≠ b)
⇔ P ( a = b = c) ⋅ P ( a ≠ b) ≥ P ( a = c e a ≠ b) ⋅ P ( a = b) ⇔
⇔ ( ∑ xi3 )(1 − ∑ xi2 ) ≥ ( ∑ xi2 − ∑ xi3 )(∑ xi2 ) ⇔ ∑ xi3 ≥ (∑ xi2 ) .
2

EUREKA! N°19, 2004

60
Sociedade Brasileira de Matemática

Vamos mostrar então que se x1 ,..., xn são reais não-negativos tais que ∑x i = 1,
então
∑ x ≥ (∑ x ) .
3
i
2 2
i (*)
De fato, temos
∑ x = (∑ x ) ⋅ (∑ x ) = ( x
3
i
3
i i
3
1 + ... + xn3 )( x1 + ... + xn ) = ∑ xi4 + ∑ xi3 ⋅ x j ⇒
i≠ j

⇒ (*) ⇔ ∑ x + ∑ x x j ≥ ∑ x + 2∑ ( xi x j ) ⇔ ∑ x x j ≥ 2∑ ( xi x j )2
4
i
3
i
4
i
2 3
i
i< j i≠ j i< j
mas observe que por MA ≥ MG temos
xi3 xj + xi x3j ≥ 2 xi3 xj xi x3j = 2(xi xj )2 ⇒ ∑xi3 xj = ∑(xi3 xj + xi x3j ) ≥∑2(xi xj )2 = 2∑(xi x j )2 ,
i≠ j i< j i< j

como queríamos.
Para haver igualdade, devemos ter xi3 x j = xi x3j , ∀i ≠ j, ou seja,
xi2 = x 2j ⇒ xi = x j , ∀i, j, pois eles são positivos. Mas isso equivale a dizermos que
P( a = c | a = b) = P( a = c | a ≠ b) ⇔ o dado é honesto!

PROBLEMA 5:
SOLUÇÃO ADAPTADA DE CARLOS STEIN NAVES DE BRITO (S.J. DOS CAMPOS – SP)
n m
Suponha que ∑ P (t ) ⋅ f
j
( j)
(t ) = 0 e ∑ Q (t ) g j
( j)
(t ) = 0, para todo t ∈ ( −1,1).
j =0 j =0
n −1 m −1
Então f ( n ) (t ) = ∑ rj (t ) f ( j ) (t ), e g ( m ) (t ) = ∑ s j (t ) g ( j ) (t ), onde
j =0 j =0

rj (t ) = − Pj (t ) Pn (t ) e S j (t ) = −Q j (t ) Qm (t ) são funções racionais.


Como (a (t )b (t ))' = a '(t )b(t ) + a (t )b '(t ) e (a (t ) + b(t )) ' = a '(t ) + b '(t ), segue, por
`
indução, que, para todo k ∈ , ( f (t ) + g (t )) ( k ) pode ser escrito como combinação
linear dos f ( j ) (t ),0 ≤ j ≤ n − 1 e g ( j ) (t ), 0 ≤ j ≤ m − 1, com coeficientes funções
racionais de t gerado pelos f ( j ) (t ),0 ≤ j ≤ n − 1 e pelos g ( j ) (t ), 0 ≤ j ≤ m − 1.
m+n
Assim, deve haver uma combinação linear nula ∑ h (t ) ⋅ ( f + g )
j =0
j
( j)
(t ) das

m + n +1> m + n funções ( f + g ) ( j ) (t ),0 ≤ j ≤ m + n , com coeficientes


\
h j (t ) ∈ (t ). Multiplicando pelo m.d.c. dos denominadores dos h j (t ), concluimos
que f + g é bacana.

EUREKA! N°19, 2004

61
Sociedade Brasileira de Matemática

Provamos que f ⋅ g é bacana de modo análogo, observando que, para todo k ∈ `,


( f ⋅ g) (k )
pertence ao espaço vetorial de dimensão ≤ m ⋅ n sobre \(t ) gerado
pelas funções f (i )
(t ) ⋅ g (t ),0 ≤ i ≤ n − 1,0 ≤ j ≤ m − 1,
( j)
donde existe uma

\
m ⋅n
combinação linear nula ∑ l (t )( f ⋅ g )
j =0
j
(f)
(t ), com l j (t ) ∈ (t ), ∀j ≤ mn.

PROBLEMA 6: SOLUÇÃO DA BANCA


9n 9n
Existe X ⊂ {1, 2,..., n} com x ≥ tal que, para todo i ∈ x , { j ≤ n | aij = 1} ≥ .
10 10
n n
De fato, se há mais de linhas na matriz, cada uma delas com pelo menos
10 10
n2
entradas nulas, o número de entradas nulas da matriz será maior que ,
100
absurdo.
9n
Analogamente, existe y ⊂ {1,2,..., n} com y≥ tal que, para todo
10
9n
j ∈ y , {i ≤ n | aij = 1 > ≥ .
10
9n 9n 4n
Seja Z = X ∩ Y . Temos Z ≥ + −n= .
10 10 5
9n 9n 4n
Se i, j ∈ Z ,( A2 )ij ≥ + − n = . Vamos mostrar, por indução, que, se
10 10 5
k −2

i, j ∈ Z , ( Ak )ij ≥ 54 ⋅  53  ⋅ nk −1 , para todo k ≥ 2. De fato,


k −2 k −1

( A )ij = ∑ ( A )ir ⋅ arj ≥ ∑ ( A )ir ⋅ arj ≥  45n − 10n  ⋅ 54 ⋅  53  ⋅ nk −1 > 54 ⋅  53  ⋅ nk ,
n
k +1 k k

r =1 r∈Z

pois, de fato, {r ∈ Z | arj = 1} ≥ Z − ≥


n 4n n
− .
10 5 10
k −2 k k
4n 4  3   3 k  n 
Assim, para todo k ≥ 2, tr ( A ) ≥ ∑ ( A )ii ≥
k k k −1
⋅ ⋅  ⋅ n >   ⋅ n >   .
i∈Z 5 5  5  5  2

EUREKA! N°19, 2004

62
Sociedade Brasileira de Matemática

XXV OLIMPÍADA BRASILEIRA DE MATEMÁTICA


Resultado – Nível 1 (5a. e 6a. Séries)
NOME CIDADE – ESTADO PRÊMIO
Henrique Pondé de Oliveira Pinto Salvador - BA Ouro
Guilherme Philippe Figueiredo Fortaleza - CE Ouro
Rodrigo Clemente de Brito Pereira João Pessoa - PB Ouro
Henrique Hiroshi Motoyama Watanabe São Paulo - SP Ouro
Rafael Tupinambá Dutra Belo Horizonte - MG Ouro
Rafael Rabelo de Carvalho Brasília - DF Prata
Carolina Yumi Yida São Paulo - SP Prata
Alice Duarte Scarpa Goiânia - GO Prata
Bruna da Silveira Afonso Salvador - BA Prata
Daniella Alves Rebouças Vila Velha - ES Prata
Fábio Mallaco Moreira SJ dos Campos - SP Prata
Carlos Renato de Andrade Figueiredo Recife - PE Prata
Rodrigo Bartels Porto Alegre - RS Prata
Luiz Gustavo Antunes Magalhães Muriaé - MG Prata
Vitor Mori São Paulo - SP Prata
Illan Feiman Halpern Itatiaia - RJ Bronze
Vinicius Coêlho Machado Fortaleza - CE Bronze
Pollyanna Stéfani Borges Freitas Fortaleza - CE Bronze
Danilo Takeshi Abe Jaune São Paulo - SP Bronze
Gabriel Moreira Francisco Santo André - SP Bronze
Nicolas Mansur Beleski Curitiba - PR Bronze
Anna Clara Leite Pestana Recife - PE Bronze
Rafael Pacheco Gomes Fortaleza - CE Bronze
Luís Otávio Valente Barcellos Sta. Rita do Passa Quatro - SP Bronze
Hannah Drummond Davico de Barros Rio de Janeiro - RJ Bronze
James Jun Hong São Paulo - SP Bronze
Thiago da Silva Pinheiro São Paulo - SP Bronze
Bernardo Duque Guimarães Saraiva Rio de Janeiro - RJ Bronze
Guilherme Rodrigues Carvalho de Souza Rio de Janeiro - RJ Bronze
Tiago Paula e Silva de Holanda Cavalcanti Recife - PE Menção Honrosa
Mateus Sampaio de Mendonça Belo Horizonte - MG Menção Honrosa
Eduardo Barthel Monteiro SJ dos Campos - SP Menção Honrosa
Thales de Oliveira Gonçalves Vitória - ES Menção Honrosa
Lucas Matsumoto Tominaga Diadema - SP Menção Honrosa
Nathana Alcântara Lima Fortaleza - CE Menção Honrosa
Lucas Cordeiro Gonçalves de Carvalho Salvador - BA Menção Honrosa
Bruna Melo Coelho Loureiro Lauro de Freitas - BA Menção Honrosa
Rafael Issamu Isuyama São Paulo - SP Menção Honrosa
Bruna de Oliveira Neuenschwander Belo Horizonte - MG Menção Honrosa
Gabriel Somavilla Nunes Videira - SC Menção Honrosa
Vinícius Henrique Campos Senra Belo Horizonte - MG Menção Honrosa
Mariana Silva de Oliveira Salvador - BA Menção Honrosa
Hannah Menezes Lira Salvador - BA Menção Honrosa
Alessandro Wagner Palmeira Guarulhos - SP Menção Honrosa
Gustavo Henrique dos Santos Figueiredo Santo André - SP Menção Honrosa
André Bain São Paulo - SP Menção Honrosa
Luara Prado Louvison São Paulo - SP Menção Honrosa
Kamila Satomi Haida Lucas do Rio Verde - MT Menção Honrosa
Dan Zylberglejd Rio de Janeiro - RJ Menção Honrosa
Lucas Souza Carmo Carvalho Belo Horizonte - MG Menção Honrosa
Igor Dantas Rocha Campina Grande - PB Menção Honrosa
Felipe Ferreira Torres Fortaleza - CE Menção Honrosa

EUREKA! N°19, 2004

63
Sociedade Brasileira de Matemática

Resultado – Nível 2 (7a. e 8a. Séries)


NOME CIDADE – ESTADO PRÊMIO
Eduardo Fischer Encantado - RS Ouro
Guilherme Rodrigues Nogueira de Souza São Paulo - SP Ouro
Edson Augusto Bezerra Lopes Fortaleza - CE Ouro
Enzo Haruo Hiraoka Moriyama São Paulo - SP Ouro
André Linhares Rodrigues Fortaleza - CE Ouro
Leandro Farias Maia Fortaleza - CE Prata
Lucio Eiji Assaoka Hossaka Curitiba - PR Prata
José Marcos Andrade Ferraro São Paulo - SP Prata
Gustavo Sampaio Sousa Fortaleza - CE Prata
Maricy Miki Hisamoto São Paulo - SP Prata
Rafael Bandeira Lages Teresina - PI Prata
Paulo André Carvalho de Melo Rio de Janeiro - RJ Prata
Leonardo Esmeraldo de Aquino Fortaleza - CE Prata
Felipe Gonçalves Assis Campina Grande - PB Prata
Fernando Nascimento Coelho Fortaleza - CE Prata
Régis Prado Barbosa Fortaleza - CE Bronze
Cesar Ryudi Kawakami São Paulo - SP Bronze
Marlon Vieira de Lima Júnior Fortaleza - CE Bronze
Sérgio Ricardo Furtado Sampaio Filho Fortaleza - CE Bronze
Felipe Ferreira Villar Coelho Serra - ES Bronze
Iuri Lima Ribeiro Fortaleza - CE Bronze
Iris Chyun Mian Tseng São Paulo - SP Bronze
Filipe Alves Tomé Fortaleza - CE Bronze
Danilo Eiki Yokoyama São Paulo - SP Bronze
João José Ribeiro e Silva Salvador - BA Bronze
Caio Carvalho Torres Teresina - PI Bronze
Rafael Morioka Oda São Paulo - SP Bronze
Adriano Jorge Braun Vieira Neto Fortaleza - CE Bronze
Renato Rebouças de Medeiros Fortaleza - CE Bronze
Dante Mattos de Salles Soares Rio de Janeiro - RJ Menção Honrosa
Rafael da Silva Holanda Fortaleza - CE Menção Honrosa
Theodoro Ribeiro Gonçalves Neto Teresina - PI Menção Honrosa
Gabriel Caser Brito Rio de Janeiro - RJ Menção Honrosa
Regina Reis da Costa Alves Rio de Janeiro - RJ Menção Honrosa
Raphael Rodrigues Mata Salvador - BA Menção Honrosa
Felipe Diz Diz São Paulo - SP Menção Honrosa
Louise Rodrigues Martins Dandas Fortaleza - CE Menção Honrosa
Daniel de Almeida Piracicaba - SP Menção Honrosa
Renato Pinto Oliveira Rio de Janeiro - RJ Menção Honrosa
Pedro Paulo Albuquerque Goes Fortaleza - CE Menção Honrosa
Leonardo Simões Freire São Paulo - SP Menção Honrosa
Roberto Câmara Gentil Porto Fortaleza - CE Menção Honrosa
Pedro Paulo Gondim Cardoso Salvador - BA Menção Honrosa
David Mosiah Terceiro Batista Fortaleza - CE Menção Honrosa
Vinícius Marques Regitano Piracicaba - SP Menção Honrosa
Guilherme Hiroshigue Motomura Hashimoto São Paulo - SP Menção Honrosa
Kaique Knothe de Andrade Rio Claro - SP Menção Honrosa
Ramon Moreira Nunes Fortaleza - CE Menção Honrosa
Martin Alexander Barrios Gundelach Rio de Janeiro - RJ Menção Honrosa
Plicia Maciel Carvalho Redenção - PA Menção Honrosa
Tiago Pellegrini Travassos Vieira São Paulo - SP Menção Honrosa

EUREKA! N°19, 2004

64
Sociedade Brasileira de Matemática

Resultado – Nível 3 (Ensino Médio)


NOME CIDADE – ESTADO PRÊMIO
Fábio Dias Moreira Rio de Janeiro - RJ Ouro
Davi Máximo Alexandrino Nogueira Fortaleza - CE Ouro
Rafael Daigo Hirama Campinas - SP Ouro
Henry Wei Cheng Hsu São Paulo - SP Ouro
Alex Corrêa Abreu Niterói - RJ Ouro
Felipe Rodrigues Nogueira de Souza São Paulo - SP Prata
Thiago Costa Leite Santos São Paulo - SP Prata
Rafael Marini Silva Vila Velha - ES Prata
Larissa Cavalcante Queiroz de Lima Fortaleza - CE Prata
Rodrigo Aguiar Pinheiro Fortaleza - CE Prata
Telmo Luis Correa Junior Santo André - SP Prata
Samuel Barbosa Feitosa Fortaleza - CE Prata
Alex Cardoso Lopes São Paulo - SP Prata
Israel Franklim Dourado Carrah Fortaleza - CE Prata
Gabriel Tavares Bujokas São Paulo - SP Bronze
Renato Francisco Lopes Mello Jaboatão dos Guararapes - PE Bronze
Murilo Vasconcelos Andrade Maceió - AL Bronze
Francisco Bruno de Lima Holanda Fortaleza - CE Bronze
Davi Valle Ferreira Belo Horizonte - MG Bronze
Raphael Constant da Costa Rio de Janeiro - RJ Bronze
Ricardo Mizoguchi Gorgoll São Paulo - SP Bronze
Raul Celistrino Teixeira Adamantina - SP Bronze
Elton Gomes Coriolano Fortaleza - CE Bronze
Thomás Yoiti Sasaki Hoshina Rio de Janeiro - RJ Bronze
Rodrigo Kendy Yamashita Osasco - SP Bronze
Eduardo de Moraes Rodrigues Poço São Paulo - SP Bronze
Carlos Augusto David Ribeiro Fortaleza - CE Bronze
Germanna de Oliveira Queiroz Fortaleza - CE Bronze
Helder Oliveira de Castro Mogi das Cruzes - SP Bronze
Diogo dos Santos Suyama Belo Horizonte - MG Bronze
Guilherme Rodrigues Salerno Goiânia - GO Menção Honrosa
Larissa Rodrigues Ribeiro Fortaleza - CE Menção Honrosa
Domingos Afono de Moura Junior Rio de Janeiro - RJ Menção Honrosa
Lucas de Aragão Bittencourt Rio de Janeiro - RJ Menção Honrosa
Eduardo Martins Spina Jundiaí - SP Menção Honrosa
Hugo Francisco Lisboa Santos Rio de Janeiro - RJ Menção Honrosa
Rafael Assato Ando Campinas - SP Menção Honrosa
Andre Belem Ferreira da Silva Rio de Janeiro - RJ Menção Honrosa
Renato Mendes Coutinho Americana - SP Menção Honrosa
Ayran Ayres Barbosa Loriato Vitória - ES Menção Honrosa
Vitor Gabriel Kleine Mogi das Cruzes - SP Menção Honrosa
Victor de Andrade Lazarte São Paulo - SP Menção Honrosa
Daniel Ponciano dos Santos Barbosa Rio de Janeiro - RJ Menção Honrosa
Rodrigo Viana Soares Fortaleza - CE Menção Honrosa
Thiago Costas Casal Monteiro Silva Rio de Janeiro - RJ Menção Honrosa
Eduardo Vieira de Oliveira Aguiar Rio de Janeiro - RJ Menção Honrosa
Glauco Rocha Machado Rio de Janeiro - RJ Menção Honrosa
Juliana Gomes Varela Fortaleza - CE Menção Honrosa
Luty Rodrigues Ribeiro Fortaleza - CE Menção Honrosa

EUREKA! N°19, 2004

65
Sociedade Brasileira de Matemática

Resultado – Nível Universitário

NOME CIDADE - ESTADO PRÊMIO


Humberto Silva Naves São José dos Campos - SP Ouro
Márcio Afonso Assad Cohen Rio de Janeiro - RJ Ouro
Yuri Gomes Lima Fortaleza - CE Ouro
Carlos Stein Naves de Brito Goiânia - GO Ouro
Daniel Massaki Yamamoto São Paulo - SP Ouro
Rodrigo Villard Milet Rio de Janeiro - RJ Ouro
Eduardo Casagrande Stabel Porto Alegre - RS Prata
Rafael Tajra Fonteles Teresina - PI Prata
Diêgo Veloso Uchôa Teresina - PI Prata
Fabrício Siqueira Benevides Fortaleza - CE Prata
Thiago da Silva Sobral Fortaleza - CE Prata
Rodrigo Roque Dias São Paulo - SP Prata
Leonardo Augusto Zão Nilópolis - RJ Prata
Einstein do Nascimento Júnior Fortaleza - CE Prata
Eduardo Famini Silva Salvador - BA Bronze
Bernardo Freitas Paulo da Costa Rio de Janeiro - RJ Bronze
Daniel Nobuo Uno São Paulo - SP Bronze
Lucas de Melo Pontes e Silva Fortaleza - CE Bronze
Antonio Carlos Maldonado S. Alonso Munhoz Rio de Janeiro - RJ Bronze
Thiago Barros Rodrigues Costa Fortaleza - CE Bronze
João Alfredo Castellani Fajardo Freire Salvador - BA Bronze
Daniel Pinheiro Sobreira Fortaleza - CE Bronze
Eduardo Paiva Costa Teresina - PI Bronze
Giuliano Boava Florianópolis - SC Bronze
Rodrigo Angelo Muniz Cariacica - ES Bronze
Tertuliano Franco Santos Franco Salvador - BA Bronze
Diego Sadao Saito Tupã - SP Bronze
Márcio Rodrigo da Rocha Pinheiro Ananindeua - PA Bronze
Ítalo Gervásio Cavalcante Teresina – PI Menção Honrosa
Estillac Lins Maciel Borges Filho Belém - PA Menção Honrosa
Daniele Véras de Andrade Rio de Janeiro - RJ Menção Honrosa
Eduardo Bertoldi SJ dos Campos - SP Menção Honrosa
Domingos Dellamonica Júnior São Paulo - SP Menção Honrosa
André Luis Hirschfeld Danila São Paulo - SP Menção Honrosa
Jorge Peixoto de Morais Neto Goiânia - GO Menção Honrosa
Bruno Germano Borics Rio de Janeiro - RJ Menção Honrosa
Rodrigo José Gondim Neves Jaboatão dos Guararapes - PE Menção Honrosa
Gustavo Gomes Araújo Ribeirão Preto - SP Menção Honrosa
Eduardo Monteiro Nicodemos Rio de Janeiro - RJ Menção Honrosa

EUREKA! N°19, 2004

66
Sociedade Brasileira de Matemática

AGENDA OLÍMPICA
XXVI OLIMPÍADA BRASILEIRA DE MATEMÁTICA

NÍVEIS 1, 2 e 3
Primeira Fase – Sábado, 5 de junho de 2004
Segunda Fase – Sábado, 11 de setembro de 2004
Terceira Fase – Sábado, 16 de outubro de 2004 (níveis 1, 2 e 3)
Domingo, 17 de outubro de 2004 (níveis 2 e 3 - segundo dia de prova).

NÍVEL UNIVERSITÁRIO
Primeira Fase – Sábado, 11 de setembro de 2004
Segunda Fase – Sábado, 16 e Domingo, 17 de outubro de 2004

X OLIMPÍADA DE MAIO
8 de maio de 2004

XV OLIMPÍADA DE MATEMÁTICA DO CONE SUL
14 a 22 de maio de 2004
Assunção, Paraguai

XLV OLIMPÍADA INTERNACIONAL DE MATEMÁTICA
06 a 18 de julho de 2004
Atenas, Grécia

X OLIMPÍADA INTERNACIONAL DE MATEMÁTICA UNIVERSITÁRIA
23 a 29 de julho de 2004
Skopje, Macedônia

XIX OLIMPÍADA IBEROAMERICANA DE MATEMÁTICA
17 a 26 de setembro de 2004
Castellón, Espanha

VI OLIMPÍADA IBEROAMERICANA DE MATEMÁTICA UNIVERSITÁRIA
6 de novembro de 2004

♦♦♦

EUREKA! N°19, 2004

67
Sociedade Brasileira de Matemática

COORDENADORES REGIONAIS
Alberto Hassen Raad (UFJF) Juiz de Fora – MG
Amarísio da Silva Araújo (UFV) Viçosa – MG
Ana Paula Bernardi da Silva (Universidade Católica de Brasília) Brasília – DF
Benedito Tadeu Vasconcelos Freire (UFRN) Natal – RN
Carlos Frederico Borges Palmeira (PUC-Rio) Rio de Janeiro – RJ
Claus Haetinger (UNIVATES) Lajeado – RS
Cleonor Crescêncio das Neves (UTAM) Manaus – AM
Élio Mega (Colégio Etapa) São Paulo – SP
Florêncio Ferreira Guimarães Filho (UFES) Vitória – ES
Gil Cunha Gomes Filho (Colégio ACAE) Volta Redonda – RJ
Ronaldo Alves Garcia (UFGO) Goiânia – GO
Reginaldo de Lima Pereira (Escola Técnica Federal de Roraima) Boa Vista – RR
Ivanilde Fernandes Saad (UC. Dom Bosco) Campo Grande– MS
Jacqueline Fabiola Rojas Arancibia (UFPB) João Pessoa – PB
Janice T. Reichert (UNOCHAPECÓ) Chapecó – SC
João Benício de Melo Neto (UFPI) Teresina – PI
João Francisco Melo Libonati (Grupo Educacional Ideal) Belém – PA
José Carlos dos Santos Rodrigues (Unespar) Campo Mourão – PR
José Cloves Saraiva (UFMA) São Luis – MA
José Gaspar Ruas Filho (ICMC-USP) São Carlos – SP
José Luiz Rosas Pinho (UFSC) Florianópolis – SC
José Vieira Alves (UFPB) Campina Grande – PB
Licio Hernandes Bezerra (UFSC) Florianópolis – SC
Luzinalva Miranda de Amorim (UFBA) Salvador – BA
Mário Rocha Retamoso (UFRG) Rio Grande – RS
Marcelo Rufino de Oliveira (Grupo Educacional Ideal) Belém – PA
Marcelo Mendes (Colégio Farias Brito, Pré-vestibular) Fortaleza – CE
Pablo Rodrigo Ganassim (Liceu Terras do Engenho) Piracicaba – SP
Ramón Mendoza (UFPE) Recife – PE
Raúl Cintra de Negreiros Ribeiro (Colégio Anglo) Atibaia – SP
Reinaldo Gen Ichiro Arakaki (INPE) SJ dos Campos – SP
Ricardo Amorim (Centro Educacional Logos) Nova Iguaçu – RJ
Sérgio Cláudio Ramos (IM-UFRGS) Porto Alegre – RS
Tadeu Ferreira Gomes (UEBA) Juazeiro – BA
Tomás Menéndez Rodrigues (U. Federal de Rondônia) Porto Velho – RO
Valdenberg Araújo da Silva (U. Federal de Sergipe) São Cristovão – SE
Valdeni Soliani Franco (U. Estadual de Maringá) Maringá – PR
Vânia Cristina Silva Rodrigues (U. Metodista de SP) S.B. do Campo – SP
Wagner Pereira Lopes (CEFET – GO) Jataí – GO

EUREKA! N°19, 2004

68
CONTEÚDO

AOS LEITORES 2

X OLIMPÍADA DE MAIO 3
Enunciados e Resultado Brasileiro

XV OLIMPÍADA DE MATEMÁTICA DO CONE SUL 7


Enunciados e Resultado Brasileiro

XLV OLIMPÍADA INTERNACIONAL DE MATEMÁTICA 9


Enunciados e Resultado Brasileiro

XIX OLIMPÍADA IBERO-AMERICANA DE MATEMÁTICA 11


Enunciados e Resultado Brasileiro

XI OLIMPÍADA INTERNACIONAL DE MATEMÁTICA UNIVERSITÁRIA 13


Enunciados e Resultado Brasileiro

ARTIGOS

O TRIÂNGULO E SUAS PRINCIPAIS CIRCUNFERÊNCIAS 17


Eduardo Wagner

DOIS PROBLEMAS CHINESES SOBRE GEOMETRIA PROJETIVA 26


Helder Oliveira de Castro

RETA DE EULER E NÚMEROS COMPLEXOS 31


José Paulo Carneiro

COMO É QUE FAZ? 37

SOLUÇÕES DE PROBLEMAS PROPOSTOS 40

PROBLEMAS PROPOSTOS 60

COORDENADORES REGIONAIS 62
Sociedade Brasileira de Matemática

AOS LEITORES

Neste número apresentamos os resultados das equipes brasileiras e os


problemas propostos na X Olimpíada de maio, na XV Olimpíada do Cone Sul, na
XLV Olimpíada Internacional (IMO), na XI Olimpíada Internacional para
Estudantes Universitários (IMC) e na XIX Olimpíada Ibero-americana. Realmente
temos muito a comemorar: o primeiro colocado na Cone Sul, a maior nota do
Ocidente no IMC (lembrem-se de que boa parte da Europa e os Estados Unidos
ficam no Ocidente!), mais uma vez todos os integrantes de nossa equipe
conquistaram medalhas na IMO, colocando o Brasil à frente de diversos países de
grande tradição matemática, como a França e a Alemanha e fomos o primeiro país
a conquistar 4 medalhas de ouro na Ibero.

Você ainda poderá ler três excelentes artigos de Geometria, com os quais
certamente você aprenderá muito. Não se esqueça de que, caso não consiga
entender algum agora (ou mesmo todos, não há problema), vale a pena retornar a
eles depois.

Agradecemos as soluções de problemas propostos e os novos problemas


enviados pelos nossos leitores, que continuamos estimulando a colaborar com a
Eureka!. Agradecemos finalmente a Cícero Thiago Magalhães de Fortaleza – CE e
a Wilberson Ivo Della Nina de São José dos Campos – SP que colaboraram com a
revisão deste número.

Os editores

EUREKA! N°20, 2004

2
Sociedade Brasileira de Matemática

X OLIMPÍADA DE MAIO
Enunciados e Resultado Brasileiro

PRIMEIRO NÍVEL
Duração da Prova: 3 horas

PROBLEMA 1
Xavier multiplica quatro dígitos, não necessariamente distintos, e obtém um
número terminado em 7. Determine quanto pode valer a soma dos quatros dígitos
multiplicados por Xavier. Dê todas as possibilidades.

PROBLEMA 2
No interior de um quadrado 11 × 11, Pablo desenhou um
retângulo e prolongando seus lados dividiu o quadrado em
5 retângulos, como mostra a figura.
Sofia fez o mesmo, conseguindo, além disso, que os
comprimentos dos lados dos 5 retângulos fossem números
inteiros entre 1 e 10, todos distintos.
Mostre uma figura como a que Sofia fez.

PROBLEMA 3
Em cada casa de um tabuleiro 5 × 5 está escrito 1 ou 1 1 –1 1 1
– 1. Em cada passo troca-se o número de cada uma das
25 casas pelo resultado da multiplicação dos números 1 1 1 1 1
de todas as suas casas vizinhas. 1 1 1 1 1
Inicialmente se tem o tabuleiro da figura.
Mostre como fica o tabuleiro ao final de 2004 passos. 1 1 1 1 1

1 1 1 1 1
Observação: Duas casas são vizinhas se tiverem um
lado em comum.

PROBLEMA 4
Em um quadrado ABCD de diagonais AC e BD, chamamos de O o centro do
quadrado. Constrói-se um quadrado PQRS de lados paralelos aos de ABCD com P
no segmento AO, Q no segmento BO, R no segmento CO, S no segmento DO.
Se área (ABCD) = 2·área(PQRS) e M é o ponto médio do lado AB, calcule a
m
medida do ângulo AM P . (Não vale medir.)

EUREKA! N°20, 2004

3
Sociedade Brasileira de Matemática

PROBLEMA 5
Tem-se 90 cartões e em cada um estão escritos dois dígitos distintos: 01, 02, 03,
04, 05, 06, 07, 08, 09, 10, 12, e assim sucessivamente até 98.
Um conjunto de cartões é correto se não contém nenhum cartão que tenha o
primeiro dígito igual ao segundo dígito de outro cartão do conjunto.
Chamamos valor de um conjunto de cartões a soma dos números escritos em cada
cartão.
Por exemplo, os quatros cartões 04, 35, 78 e 98 formam um conjunto correto e seu
valor é 215, pois 04 + 35 + 78 + 98 = 215.
Encontre um conjunto correto que tenha o maior valor possível. Explique por que
é impossível obter um conjunto correto de maior valor.

SEGUNDO NÍVEL
Duração da Prova: 3 horas

PROBLEMA 1
Juliano escreveu cinco números inteiros positivos, não necessariamente distintos,
tais que seu produto seja igual à sua soma. Quais podem ser os números que
Juliano escreveu?

PROBLEMA 2
A mãe de Zezinho quer preparar n pacotes de 3 balas para dar de presente na festa
de aniversário, e para isto comprará balas sortidas de 3 sabores diferentes. Ela
pode comprar qualquer número de balas, mas não pode escolher quantas são de
cada sabor. Ela quer colocar em cada pacote uma bala de cada sabor, e se isto não
for possível usará somente balas de um sabor e todos os pacotes terão 3 balas
desse sabor. Determine o menor número de balas que ela deve comprar para
poder preparar os n pacotes. Explique por que se ela compra menos balas não terá
a certeza de poder preparar os pacotes como ela quer.

PROBLEMA 3
Temos uma mesa de bilhar de 8 metros de comprimento e 2 metros de largura,
com uma única bola no centro. Lançamos a bola em linha reta e, depois de
percorrer 29 metros, ela pára numa esquina da mesa. Quantas vezes a bola rebateu
nas bordas da mesa?
Nota: Quando a bola rebate na borda da mesa, os dois ângulos que formam sua
trajetória com a borda da mesa são iguais.

EUREKA! N°20, 2004

4
Sociedade Brasileira de Matemática

PROBLEMA 4
Ache todos os números naturais x, y, z que verificam simultaneamente
x ⋅ y ⋅ z = 4104 x + y + z = 77

PROBLEMA 5
Sobre um tabuleiro 9 × 9, dividido em casas 1 × 1, se colocam sem superposições
e sem sair do tabuleiro, peças da forma

Cada peça cobre exatamente 3 casas.


a) A partir do tabuleiro vazio, qual é a máxima quantidade de peças que se pode
colocar?
b) A partir do tabuleiro com 3 peças e colocadas como mostra o diagrama
seguinte,

qual é a máxima quantidade de peças que se pode colocar?

EUREKA! N°20, 2004

5
Sociedade Brasileira de Matemática

RESULTADOS

PRIMEIRO NÍVEL (Até 13 anos)


Gustavo Henrique dos Santos Figueiredo Medalha de Ouro Santo André - SP
Vinícius Henrique Campos Senra Medalha de Prata Belo Horizonte - MG
Rafael Pacheco Gomes Medalha de Prata Fortaleza - CE
Danilo Takeshi Abe Jaune Medalha de Bronze São Paulo - SP
Ilan Feiman Halpern Medalha de Bronze Itatiaia - RJ
Emanuelle Meneses Barros Medalha de Bronze Fortaleza - CE
Dayana Basilio Batista Medalha de Bronze Campo Grande - MS
Guilherme Albuquerque Pinto Rebello Menção Honrosa Rio de Janeiro - RJ
Bernardo Duque Guimarães Saraiva Menção Honrosa Rio de Janeiro - RJ
Amanda Maria Barradas M. de Santana Menção Honrosa Teresina - PI

SEGUNDO NÍVEL (Até 15 anos)


Eduardo Fischer Medalha de Ouro Encantado - RS
Lucio Eiji Assaoka Hossaka Medalha de Prata Curitiba - PR
Guilherme Nogueira de Souza Medalha de Prata São Paulo - SP
José Marcos Andrade Ferraro Medalha de Bronze São Paulo - SP
Paulo André Carvalho de Melo Medalha de Bronze Rio de Janeiro - RJ
Rodrigo Clemente de Brito Pereira Medalha de Bronze João Pessoa - PB
Henrique Pondé de Oliveira Pinto Medalha de Bronze Salvador - BA
Rafael Tupinambá Dutra Menção Honrosa Belo Horizonte - MG
Amanda Freitas Santos Menção Honrosa Rio de Janeiro - RJ
Edson Augusto Bezerra Lopes Menção Honrosa Fortaleza - CE

EUREKA! N°20, 2004

6
Sociedade Brasileira de Matemática

XV OLIMPÍADA DE MATEMÁTICA DO CONE SUL


Enunciados e Resultado Brasileiro

A XV Olimpíada de Matemática do Cone Sul foi realizada na cidade de


Caaguazú, Paraguai no período de 14 a 23 de Maio de 2004. A equipe brasileira
foi liderada pelos professores Pablo Rodrigo Ganassim (São Paulo – SP) e Márcio
Cohen (Rio de Janeiro – RJ).

RESULTADOS DA EQUIPE BRASILEIRA

BRA1 Gabriel Tavares Bujokas Medalha de Ouro


BRA2 Leandro Farias Maia Medalha de Prata
BRA3 André Linhares Rodrigues Medalha de Bronze
BRA4 Telmo Luis Correa Júnior Medalha de Bronze

PROBLEMA 1
Maxi escolheu 3 dígitos e, fazendo todas as permutações possíveis, obteve 6
números distintos, cada um com 3 dígitos. Se exatamente um dos números que
Maxi obteve é um quadrado perfeito e exatamente três são primos, encontrar os 3
dígitos que Maxi escolheu.
Dê todas as possibilidades para os 3 dígitos.

PROBLEMA 2
Dada uma circunferência C e um ponto P exterior a ela, traçam-se por P as duas
tangentes à circunferência, sendo A e B os pontos de tangência.
Toma-se um ponto Q sobre o menor arco AB de C. Seja M a interseção da reta AQ
com a perpendicular a AQ traçada por P, e seja N a interseção da reta BQ com a
perpendicular a BQ traçada por P.
Demonstre que, ao variar Q no arco AB, todas as retas MN passam por um mesmo
ponto.

PROBLEMA 3
Seja n um inteiro positivo. Chamamos Cn a quantidade de inteiros positivos x,
menores que 10n, tais que a soma dos dígitos de 2x é menor que a soma dos dígitos
de x.
Demonstre que Cn • (10n − 1) .
4
9

EUREKA! N°20, 2004

7
Sociedade Brasileira de Matemática

PROBLEMA 4
Arnaldo escolhe um inteiro a, a •  H %HUQDOGR HVFROKH XP LQWHLUR b, b • 
Ambos dizem, em segredo, o número que escolheram a Cernaldo, e este escreve
em um quadro os números 5, 8 e 15, sendo um desses a soma a + b.
Cernaldo toca uma campainha e Arnaldo e Bernaldo, individualmente, escrevem
em papéis distintos se sabem ou não qual dos números no quadro é a soma de a e
b, e entregam seus papéis para Cernaldo.
Se em ambos os papéis está escrito NÃO, Cernaldo toca novamente a campainha,
e o procedimento se repete.
Sabe-se que Arnaldo e Bernaldo são sinceros e inteligentes.
Qual é o número máximo de vezes que a campainha pode ser tocada até que um
deles escreva que sabe o valor da soma?

PROBLEMA 5
Utilizando triangulinhos eqüiláteros de papel, de lado 1, forma-se um triângulo
eqüilátero de lado 2 2004 . Desse triângulo retira-se o triangulinho de lado 1 cujo
centro coincide com o centro do triângulo maior.
Determine se é possível cobrir totalmente a superfície restante, sem superposições
nem buracos, dispondo-se somente de fichas em forma de trapézio isósceles, cada
uma formada por três triangulinhos eqüiláteros de lado 1.

PROBLEMA 6
Sejam m, n inteiros positivos. Em um tabuleiro m × n, quadriculado em
quadradinhos de lado 1, considere todos os caminhos que vão do vértice superior
direito ao inferior esquerdo, percorrendo as linhas do quadriculado exclusivamente
nas direções ← e ↓.
Define-se a área de um caminho como sendo a quantidade de quadradinhos do
tabuleiro que há abaixo desse caminho. Seja p um primo tal que rp(m) + rp(n) ≥ p,
onde rp(m) representa o resto da divisão de m por p e rp(n) representa o resto da
divisão de n por p.
Em quantos caminhos a área é um múltiplo de p?

EUREKA! N°20, 2004

8
Sociedade Brasileira de Matemática

XLV OLIMPÍADA INTERNACIONAL DE MATEMÁTICA


Enunciados e Resultado Brasileiro

A XLV Olimpíada Internacional de Matemática foi realizada na cidade de


Atenas, Grécia no período de 06 a 18 de julho de 2004. A equipe brasileira foi
liderada pelos professores Carlos Gustavo Moreira (Rio de Janeiro – RJ) e Carlos
Yuzo Shine (São Paulo – SP).

RESULTADOS DA EQUIPE BRASILEIRA

BRA1 Fábio Dias Moreira Medalha de Bronze


BRA2 Gabriel Tavares Bujokas Medalha de Prata
BRA3 Henry Wei Cheng Hsu Medalha de Bronze
BRA4 Rafael Daigo Hirama Medalha de Prata
BRA5 Rafael Marini Silva Medalha de Bronze
BRA6 Thiago Costa Leite Santos Medalha de Bronze

PRIMEIRO DIA

PROBLEMA 1

n
Seja ABC um triângulo acutângulo com AB ≠ AC. A circunferência de diâmetro

n
BC intersecta os lados AB e AC nos pontos M e N, respectivamente. Seja O o
ponto médio do lado BC. As bissetrizes dos ângulos BAC e MON intersectam-se
em R. Prove que as circunferências circunscritas aos triângulos BMR e CNR têm
um ponto em comum que pertence ao lado BC.

PROBLEMA 2
Determine todos os polinômios P(x) de coeficientes reais que satisfazem a
igualdade

P( a − b) + P(b − c) + P(c − a ) = 2 P( a + b + c )

para quaisquer números reais a, b, c, tais que ab + bc + ca = 0.

EUREKA! N°20, 2004

9
Sociedade Brasileira de Matemática

PROBLEMA 3
Um gancho é uma figura formada por seis quadrados unitários como no seguinte
diagrama

ou qualquer uma das figuras obtidas desta aplicando rotações ou reflexões.


Determine todos os retângulos m × n que podem ser cobertos com
ganchos de modo que:
i) O retângulo é coberto sem buracos e sem sobreposições;
ii) Nenhuma parte de nenhum gancho pode cobrir regiões fora do retângulo.

SEGUNDO DIA

PROBLEMA 4
Seja n ≥ 3 um inteiro. Sejam t1 , t2 ,..., tn números reais positivos tais que
1 1 1
n 2 + 1 > (t1 + t2 + ... + tn )  + + ... +  .
 t1 t2 tn 
Mostre que ti , t j e tk são as medidas dos lados de um triângulo para quaisquer i,
j, k com 1 ≤ i < j < k ≤ n.

PROBLEMA 5

n
Num quadrilátero convexo ABCD a diagonal BD não é bissetriz do ângulo ABC n
nem do ângulo CDA . Um ponto P no interior de ABCD satisfaz

∠PBC = ∠DBA e ∠PDC = ∠BDA.

Prove que os vértices do quadrilátero ABCD pertencem a uma mesma


circunferência se e só se AP = CP.

PROBLEMA 6
Um inteiro positivo é dito alternante se, na sua representação decimal, quaisquer
dois dígitos consecutivos têm paridade diferente.
Determine todos os inteiros positivos n tais que n tem um múltiplo que é
alternante.

EUREKA! N°20, 2004

10
Sociedade Brasileira de Matemática

XIX OLIMPÍADA IBERO-AMERICANA DE MATEMÁTICA


Enunciados e Resultado Brasileiro

A XIX Olimpíada Ibero-americana de Matemática foi realizada na cidade


de Castellón, Espanha no período de 17 a 26 de setembro de 2004.
A equipe brasileira foi liderada pelos professores Eduardo Wagner e Luciano
Guimarães Monteiro de Castro, ambos do Rio de Janeiro – RJ.

RESULTADOS DA EQUIPE BRASILEIRA

BRA1 Alex Corrêa Abreu Medalha de Ouro


BRA2 Fábio Dias Moreira Medalha de Ouro
BRA3 Gabriel Tavares Bujokas Medalha de Ouro
BRA4 Rafael Daigo Hirama Medalha de Ouro

PRIMEIRO DIA

PROBLEMA 1
Deve-se colorir as casas de um tabuleiro 1001 × 1001 de acordo com as seguintes
regras:
Se duas casas têm um lado comum, então pelo menos uma delas deve ser colorida.
De cada seis casas consecutivas de uma linha ou de uma coluna, devem colorir-se
sempre pelo menos duas delas que sejam adjacentes.
Determinar o número mínimo de casas que devem ser coloridas.

PROBLEMA 2
Considera-se no plano uma circunferência de centro O e raio r, e um ponto A
exterior a ela. Seja M um ponto da circunferência e N o ponto diametralmente
oposto a M. Determinar o lugar geométrico dos centros das circunferências que
passam por A, M e N quando M varia.

PROBLEMA 3
Sejam n e k números inteiros positivos tais que n é ímpar ou n e k são pares.
Provar que existem inteiros a e b tais que

mdc (a, n) = mdc(b, n) = 1 e k = a + b.

EUREKA! N°20, 2004

11
Sociedade Brasileira de Matemática

PROBLEMA 4
Determinar todos os pares (a, b), onde a e b são números inteiros positivos de dois
dígitos cada um, tais que 100a + b e 201a + b são quadrados perfeitos de quatro
dígitos.

PROBLEMA 5
Dado um triângulo escaleno ABC, designam-se por A', B', C' os pontos de
interseção das bissetrizes interiores dos ângulos A, B e C com os lados opostos,
respectivamente.

Sejam: A'' a interseção de BC com a mediatriz de AA',


B'' a interseção de AC com a mediatriz de BB' e
C'' a interseção de AB com a mediatriz de CC'.

Provar que A'', B'' e C'' são colineares.

PROBLEMA 6
Para um conjunto H de pontos no plano, diz-se que um ponto P do plano é um
ponto de corte de H, se existem quatro pontos distintos A, B, C e D em H tais que
as retas AB e CD são distintas e se cortam em P.
Dado um conjunto finito A0 de pontos no plano, constrói-se uma sucessão de
conjuntos A1 , A2 , A3 ,... da seguinte forma: para qualquer j ≥ 0, Aj +1 é a união de
Aj com o conjunto de todos os pontos de corte de Aj .
Demonstrar que se a união de todos os conjuntos da sucessão é um conjunto finito
então, para qualquer j ≥ 1, tem-se Aj = A1 .

EUREKA! N°20, 2004

12
Sociedade Brasileira de Matemática

XI OLIMPÍADA INTERNACIONAL DE MATEMÁTICA PARA


ESTUDANTES UNIVERSITÁRIOS
Enunciados e Resultado Brasileiro

A XI Olimpíada Internacional de Matemática para estudantes


universitários foi realizada na cidade de Skopje, Macedônia no período de 23 a 29
de julho de 2004.
A equipe brasileira foi liderada pelo professor Fernando Pimentel, da
cidade de Fortaleza – CE.

RESULTADOS DA EQUIPE BRASILEIRA

Yuri Gomes Lima UFC Medalha de Ouro


Humberto Silva Naves ITA Medalha de Prata
Carlos Stein Naves de Brito ITA Medalha de Prata
Alex Corrêa Abreu UFRJ Medalha de Prata
Eduardo Casagrande Stabel UFRGS Medalha de Bronze
Murilo Vasconcelos de Andrade IME Medalha de Bronze
Rafael Tajra Fonteles UFPI Medalha de Bronze
Thiago Barros Rodrigues Costa Unicamp Menção Honrosa
Diêgo Veloso Uchôa IME Menção Honrosa
Eduardo Famini Silva IME Menção Honrosa
Tertuliano Franco Santos Franco UFBA Menção Honrosa

PRIMEIRO DIA

PROBLEMA 1
Seja S um conjunto infinito de números reais tal que x1 + x2 + ... + xn ≤ 1 para todo
subconjunto finito {x1 , x2 ,..., xn } ⊂ S . Demonstre que S é enumerável.

PROBLEMA 2
Seja f1 ( x) = x 2 − 1, e para cada inteiro positivo n ≥ 2 defina f n ( x ) = f n −1 ( f1 ( x )).
Quantas raízes reais distintas tem o polinômio f 2004 ?

EUREKA! N°20, 2004

13
Sociedade Brasileira de Matemática

PROBLEMA 3
n
Seja An o conjunto de todas as somas ∑ arcsin x ,
k =1
k onde n ≥ 2, xk ∈ [0,1], e
n

∑x
k =1
k = 1.

i) Prove que An é um intervalo.


ii) Seja an o comprimento do intervalo An . Calcule nlim
→∞
an .

PROBLEMA 4
Suponha n ≥ 4 e seja S um conjunto finito de pontos no espaço \ , de maneira
3

que quaisquer quatro de seus pontos não sejam coplanares. Suponha que todos os
pontos de S podem ser coloridos de vermelho e azul de modo que qualquer esfera
que intersecte S em ao menos 4 pontos tenha a propriedade de que exatamente a
metade dos pontos na interseção de S com a esfera é azul. Prove que todos os
pontos de S encontram-se numa esfera.

PROBLEMA 5
 2n 
Seja S um conjunto de   + 1 números reais, onde n é um inteiro positivo.
n
Prove que onde existe uma seqüência monótona {ai }1≤i ≤ n + 2 ⊂ S tal que
xi +1 − x1 ≥ 2 xi − x1 ,
para todo i = 2, 3,…, n.

PROBLEMA 6
Para cada número complexo z diferente de 0 e 1 definimos a seguinte função:
1
f ( z) = ∑
log 4 z
onde a soma é sobre todos os ramos do logaritmo complexo.

P( z )
i) Prove que há dois polinômios P e Q tais que f ( z ) = para todo
Q( z )
z ∈ ^ − {0,1}.

EUREKA! N°20, 2004

14
Sociedade Brasileira de Matemática

ii) Prove que para todo z ∈ ^ − {0,1} temos


z3 + 4z 2 + z
f (z) = .
6( z − 1)4

SEGUNDO DIA

PROBLEMA 7
Seja A uma matriz real 4 × 2 e B uma matriz real 2 × 4 tal que

 1 0 −1 0 
 
0 1 0 −1
AB =  .
 −1 0 1 0 
 
 0 −1 0 1 
Encontre BA.

PROBLEMA 8
Sejam f , g :[a, b] → [0, ∞) duas funções continuas não decrescentes tais que
para cada x ∈ [a, b] temos

x x b b
∫ a
f (t ) dt ≤ ∫
a
g (t ) dt e ∫ a
f (t ) dt = ∫
a
g (t ) dt.

Prove que
b b
∫ a
1 + f (t ) dt ≥ ∫
a
1 + g (t ) dt.

PROBLEMA 9
Seja D um disco unitário fechado, e sejam z1 , z2 ,..., z n pontos fixados em D.
Prove que existe um ponto z em D tal que a soma das distancias desde z a cada um
dos n pontos é maior ou igual que n.

EUREKA! N°20, 2004

15
Sociedade Brasileira de Matemática

PROBLEMA 10
Para n ≥ 1 seja M uma matriz complexa n × n com autovalores λ1 , λ 2 ,..., λ k ,
distintos com respectivas multiplicidades m1 , m2 ,..., mk . Considere o operador
linear LM definido por LM X = MX + XM , para qualquer X matriz complexa
T

n × n . Encontre os autovalores de LM e suas multiplicidades.

PROBLEMA 11
Prove que
1 1 dx dy
∫∫ 0 0 1
≤ 1.
+ log y − 1
x
PROBLEMA 12
Para n ≥ 0 defina as matrizes An e Bn como segue: A0 = B0 = (1), e, para cada
n > 0,
 An − An −1   An −1 An −1 
An =  1  e Bn =  .
 An −1 Bn −1   An −1 0 

Denote por S(M) a soma de todos os elementos da matriz M. Prove que


S ( Ank −1 ) = S ( Akn −1 ) , para quaisquer n, k ≥ 2.

EUREKA! N°20, 2004

16
Sociedade Brasileira de Matemática

O TRIÂNGULO E SUAS PRINCIPAIS CIRCUNFERÊNCIAS


Eduardo Wagner, Rio de Janeiro - RJ

Nível Iniciante

Vamos tratar neste artigo das circunferências inscrita, circunscrita e exinscritas de


um triângulo. Mostraremos diversas propriedades, relações interessantes e alguns
problemas.
Em todo o artigo, o triângulo ABC possui lados AB = c, BC = a e CA = b. O seu
semiperímetro é p e sua área é S. Será necessário que o leitor conheça a fórmula de
Heron para área do triângulo em função de seus lados:
S = p( p − a)(p − b)( p − c) .

A circunferência inscrita
A circunferência inscrita tem centro I, incentro do triângulo, que é o ponto de
interseção das bissetrizes internas.

b
c
I
r

B a C

A área do triângulo ABC é a soma das áreas dos triângulos AIB, BIC e CIA, que
possuem altura igual a r, raio da circunferência inscrita. Portanto,

cr ar br a + b + c
S= + + = r = pr
2 2 2 2
A nossa primeira relação é:
S = pr

EUREKA! N°20, 2004

17
Sociedade Brasileira de Matemática

que permite calcular o raio da circunferência inscrita em um triângulo em função


de seus lados.

A circunferência circunscrita

Considere agora o triângulo ABC inscrito em uma circunferência de raio R. Seja


AH = h uma altura e seja AD um diâmetro dessa circunferência.

c b
h
2R

B H C

Os triângulos AHB e ACD são semelhantes uma vez que os ângulos AHB e ACD
são retos e os ângulos ABC e ADC são iguais pois subtendem o mesmo arco.
Logo,
AB AH
=
AD AC
c h
=
2R b

ou seja, bc = 2Rh. Multiplicando pelo comprimento do lado BC os dois lados,


temos
abc = 2Rah. Mas ah é o dobro da área do triângulo ABC e assim encontramos a
nossa segunda relação :
abc = 4RS

Ela permite calcular o raio da circunferência circunscrita a um triângulo em


função dos seus lados.

EUREKA! N°20, 2004

18
Sociedade Brasileira de Matemática

As circunferências exinscritas
A circunferência exinscrita relativa ao vértice A do triângulo ABC é tangente ao
lado BC e às retas AB e AC. Seu raio será designado por ra e seu centro por IA ,
chamado de exincentro (ou excentro) relativo ao vértice A do triângulo ABC. O
ponto IA é a interseção da bissetriz interna de A e das bissetrizes externas de B e
C. As outras duas circunferências exinscritas e os dois outros exincentros são
definidas de forma análoga.

IA
b

a ra

A c B

A área do triângulo ABC é igual a área de ABIA mais a área de ACIA menos a área
de BCIA. Assim,
cra bra ara b + c − a
S= + − = ra .
2 2 2 2
Observe que b + c – a = a + b + c – 2a = 2p – 2a = 2(p – a). Logo, a nossa nova
relação é:
S = ra ( p − a)
e, analogamente, temos
S = rb ( p − b)
S = rc (p − c)

que permitem calcular os raios das circunferências exinscritas em função dos lados
do triângulo ABC.

Para fixar o que apresentamos até aqui, resolva o problema a seguir.

EUREKA! N°20, 2004

19
Sociedade Brasileira de Matemática

Problema 1: Em um triângulo de lados 5, 7, e 8, calcule os raios das circunferências


inscrita, circunscrita e exinscritas.
7 3 10 3 5 3
Respostas: 3, , 2 3, , .
3 7 4

Duas relações
Primeira: S = r ⋅ ra ⋅ rb ⋅ rc

Esta é fácil de demonstrar. Multiplicando as relações da circunferência inscrita e


das exinscritas obtemos:
4 2
S = r ⋅ ra ⋅ rb ⋅rc ⋅ p( p − a)(p − b)( p − c) = r ⋅ ra ⋅rb ⋅ rc ⋅ S
o que demonstra a relação.

1 1 1 1
Segunda: = + +
r ra rb rc
Observe que
S S S S
+ + = p − a + p − b + p − c = 3p − (a + b + c) = p =
ra rb rc r
que demonstra a relação.

Problema: 2 Existe um triângulo cujas circunferências exinscritas tenham raios


1cm, 2cm e 6cm?
4 5 7 5 9 5
Resposta: sim. os lados medem , , centímetros.
5 5 5

Os pontos de tangência
Vamos agora localizar os pontos de tangência das circunferências inscrita e
exinscrita em relação da cada um dos lados. Consideremos inicialmente a
circunferência inscrita tangenciando os lados AB, BC e CA nos pontos L, M e N,
respectivamente.

EUREKA! N°20, 2004

20
Sociedade Brasileira de Matemática

N
L

B M C

Sejam AL = AN = x, BL = BM = y, CM = CN = z. Temos então o sistema:


x+y=c
y+z=a
z+x=b
que resolvido dá AL = AN = p – a, BL = BM = p – b, CM = CN = p – c.

Considerando uma das circunferências exinscritas como mostra a figura a seguir


temos:

B
C P

o perímetro do triângulo ABC é


2p = BA + AC + BC = BA + AQ + CQ + BC = BA + AR + CP + BC = BR + BP
= 2BP.
Logo, BP = p, o semiperímetro do triângulo.

Uma desigualdade interessante


a2
Em todo triângulo ABC, r ⋅ ra ≤ .
4
Esta desigualdade, além de interessante pelo seu aspecto, vai ser útil para a
resolução de outros problemas.
Observe a figura a seguir.

EUREKA! N°20, 2004

21
Sociedade Brasileira de Matemática

I F ra

r
A
D C E

a
Na figura acima, I é o incentro de ABC e J é o exincentro relativo ao vértice A.
Sabemos pelo ítem anterior que CD = p – c e que AE = p. Logo, CE = p – b e
portanto,
DE = p – c + p – c = 2p – (b + c) = a.

No triângulo retângulo IJF temos IJ ≥ r + ra , valendo a igualdade se, e somente


se AB = AC. Portanto,
(r + ra ) 2 ≤ IJ 2 = a 2 + (ra − r)2
2r ⋅ ra = a2 − 2r ⋅ ra
a2
r ⋅ ra ≤
4
como queríamos demonstrar. Repare que a igualdade ocorre se, e somente se, o
triângulo ABC é isósceles com vértice A.

A desigualdade entre os raios das circunferências inscrita e circunscrita

r 1
Em qualquer triângulo, ≤ .
R 2

Esta linda desigualdade é intrigante, pois afirma que o raio da circunferência


circunscrita não é menor que o dobro do raio da circunferência inscrita.
Há diversas demonstrações desta desigualdade; todas muito engenhosas. Mas,
seguindo o que estamos desenvolvendo neste artigo, vamos apresentar a
demonstração seguinte.
Considerando a desigualdade que acabamos de demonstrar, temos:

EUREKA! N°20, 2004

22
Sociedade Brasileira de Matemática

a2
r ⋅ ra ≤
4
b2
r ⋅ rb ≤
4
c2
r ⋅ rc ≤
4

Multiplicando estas três relações temos:

a2 b 2 c 2
r 2 ⋅ r ⋅ ra ⋅ rb ⋅ rc ≤
64
2
(4RS)
r 2S 2 ≤
64

r 1

R 2
A pergunta natural que devemos fazer é quando vale a igualdade. Repare que na
a2
demonstração da desigualdade r ⋅ ra ≤ , a igualdade vale se, e somente se, AB =
4
AC, quando as circunferências inscrita e exinscrita relativa ao vértice A são
tangentes no ponto médio do lado BC. Utilizando o mesmo argumento para as
r 1
outras desigualdades, concluímos que = ocorre se, e somente se o triângulo
R 2
ABC é equilátero.

Problema 3
A ( p − b)( p − c )
Sabendo que em um triângulo ABC, sin = (isto você poderá
2 bc
A B C 1
demonstrar mais tarde), mostre que sin ⋅ sin ⋅sin ≤ .
2 2 2 8
A relação dos cinco raios
Os raios das circunferências inscrita, circunscrita e exinscritas estão ligados pela
relação:

EUREKA! N°20, 2004

23
Sociedade Brasileira de Matemática

ra + rb + rc − r = 4R

Para demonstrar isto, necessitamos apenas de resultados anteriores e de alguma


manipulação algébrica.
S S aS
rb + rc = + =
p − b p − c ( p − b)( p − c)

S S
ra − r = −
p−a p

Somando,

 1 1 
ra + rb + rc − r = aS + 
 (p − b)(p − c) p(p − a) 

p(p − a) + (p − b)(p − c)
= aS
p(p − a)(p − b)( p − c)

2p 2 − p(a + b + c) + bc
= aS
S2

abc
= = 4R
S
O assunto não tem fim. Há muitíssimas outras relações entre os elementos de um
triângulo e suas principais circunferências; algumas legais e outras
desinteressantes. Mas, nosso objetivo foi fornecer um material básico para que os
alunos iniciantes possam se desenvolver e, por isso, paramos aqui.
Para fixar as idéias, você poderá curtir uns probleminhas bacanas na lista abaixo.

Problemas suplementares

Problema 4
Em um triângulo ABC com incentro I, a bissetriz interna do ângulo A encontra a
circunferência circunscrita em E. Prove que EB = EC = EI.

EUREKA! N°20, 2004

24
Sociedade Brasileira de Matemática

Problema 5
Dados um ângulo agudo XOY, um ponto P exterior e um número positivo k (como
sugerido na figura abaixo), mostre como se pode construir uma reta que passe por
P e que corte os lados do ângulo dado formando um triângulo de perímetro k.

O X

•P

Problema 6
Em um triângulo acutângulo, mostre que o simétrico do ortocentro em relação a
um lado pertence a circunferência circunscrita ao triângulo .

Problema 7 (de uma olimpíada internacional)


O triângulo acutângulo ABC está inscrito em uma circunferência. Sejam M, N e P
os pontos médios dos arcos AB, BC e CA, respectivamente. Prove que a área do
hexágono AMBNCP é maior ou igual que o dobro da área do triângulo ABC.

Problema 8
Em um quadrilátero convexo ABCD, as bissetrizes dos ângulos A e B cortam-se
em M, as bissetrizes dos ângulos C e D cortam-se em N e as retas AD e BC
cortam-se em P. Mostre que os pontos M, N e P são colineares.

Problema 9
Em um triângulo ABC com incentro I, e exincentros J, K, L, mostre que I é o
ortocentro do triângulo JKL.

Problema 10
Em um triângulo, mostre que a distância do ortocentro a um vértice é o dobro da
distância do circuncentro ao lado oposto. Mostre a seguir que o ortocentro, o
baricentro e o circuncentro são colineares.

Problema 11 (este é difícil)


Em um triângulo ABC, AX é uma bissetriz (X ∈ BC), N é o ponto médio de AX, e
M é o ponto médio de BC. Sendo I o incentro do triângulo, mostre que M, I e N
são colineares.

EUREKA! N°20, 2004

25
Sociedade Brasileira de Matemática

DOIS PROBLEMAS CHINESES SOBRE GEOMETRIA PROJETIVA


Helder Oliveira de Castro, São Paulo - SP

Nível Avançado

INTRODUÇÃO:
Para aqueles que nunca tiveram uma aula sobre esse assunto, depois de
consultarem [1] ou até alguns problemas em [2], podem ficar meio em dúvida
sobre como desenvolver essa poderosa ferramenta e apelarem para outros
métodos. É muito interessante quando, depois de horas e mais horas fazendo
centenas de cálculos intrincados de trigonometria e geometria analítica e gastando
algumas dúzias de folhas de almaço, desistimos de um problema de geometria sem
ter chegado a lugar algum. Motivado (na verdade irritado) por isso comecei a
estudar Geometria Projetiva, e é com esse intuito que gostaria de expor dois
problemas nos quais são exploradas técnicas de solução por polaridade,
fornecendo bases para que o leitor possa aplicá-las em outras situações. Mas para
começar é necessário retomar algumas definições de [1], que servem de alicerce
para a solução dos dois problemas que nos interessam.

PÓLO E RETA POLAR


Dados uma circunferência S, de centro O e raio R, e um ponto A, distinto de O,
definimos A’ tal que OA.OA’ = R2, e esta transformação é chamada de inversão. A
reta a que é perpendicular à OA’, passando por A’, é chamada de reta polar de A
em relação a S, e o ponto A é chamado de pólo de a em relação a S.

Teorema 1: Dados uma circunferência S no plano e pontos A e B, sejam a e b suas


respectivas polares em relação a S. Temos então que A ∈ b ⇒ B ∈ a.
a
b

O A A'
B'

EUREKA! N°20, 2004

26
Sociedade Brasileira de Matemática

Prova: Tome B ∈ a, e seja B’ ∈ OB tal que AB’⊥OB. Temos que ∆OAB’≈ ∆OBA’
pelo critério AA, e logo OA/OB = OB’/OA’ ⇒ OB.OB’ = OA.OA’ = R2 ⇔ B’ é o
inverso de B em relação a S, e como AB’⊥OB temos que A ∈ b.

Corolário 1: Para um ponto pertencente à própria circunferência, sua reta polar é


tangente à circunferência por ele.

Corolário 2: Se A é exterior à circunferência, sejam B e C os pontos de contato das


tangentes traçadas à circunferência por A. Então a reta polar de A passa por B e C.

Prova: Temos que A pertence às polares de B e C, e logo B e C devem pertencer à


polar de A.
Bem, finalmente vamos aos problemas chineses:

(CHINA-1997) O quadrilátero ABCD está inscrito num círculo S. Seja X o ponto de


intersecção entre os lados AB e CD e W o ponto de intersecção entre os lados AD e
BC. As tangentes traçadas por X intersectam S em Y e Z. Prove que W, Y e Z são
colineares.

Resolução: Antes de mais nada vamos ter que enunciar e provar um lema, que
também encontra-se em [1], mas que certamente no dia da prova você teria de
demonstrar. É assim:

LEMA: Se por um ponto M exterior a um círculo S traçarmos secantes que


intersectam-no nos pontos A, B, C e D (vide figura), e se tomarmos {P} = AB ∩
CD e {Q} = AC ∩ BD, então a polar de M em relação a S será a reta PQ.
P

S B

Q C

A D M

EUREKA! N°20, 2004

27
Sociedade Brasileira de Matemática

Há quem ache essa parte um pouco mais salgada, pois é justamente a parte mais
difícil do assunto o qual vamos tratar. Tome as retas polares de A, B, C e D como
a, b, c e d que, como vimos, são tangentes à S nos seus respectivos pólos. Defina
{R} = b ∩ c e {T} = a ∩ d. A reta polar de R será BC e a reta polar de T será AD,
pelo Corolário 2, e pelo Teorema 1 teremos que a polar de M será a reta RT. Basta
provar então que RT passa por P e Q, ou melhor, que R, P, Q e T são colineares.
Considere o hexágono ABB’CC’D, no qual B’ ≡ B e C’ ≡ C (vamos usar aqui a
estratégia proposta em [1]: fazer vértices de um hexágono coincidirem para
obtermos novas relações). Pelo Teorema de Pascal, P, R e Q são colineares (os 3
pontos de encontro dos 3 pares de lados opostos do hexágono devem ser
colineares). Analogamente no hexágono AA’BCDD’, com A’ ≡ A e D’ ≡ D,
teremos que P, Q e T são colineares. Segue que R, P, Q e T são colineares, como
queríamos demonstrar.
Bem, fim de Lema. O problema agora fica fácil: suponha que, no dia da prova,
você já conheça todas estas propriedades. Aí você as demonstra bem rápido na
folha de respostas, e para dar o Gran Finale, bem... vejamos:

C
B
Z
Y
W

S D

Note que podemos fazer uma certa analogia entre o problema e o Lema. O ponto
W corresponde ao ponto M do Lema, e o ponto X ao ponto P. Temos então que a
reta polar de W passa por X ⇒ a reta polar de X passa por W. Mas a reta polar de X

EUREKA! N°20, 2004

28
Sociedade Brasileira de Matemática

passa por Y e Z, pelo Corolário 2 ⇒ W, Y e Z são colineares, finalizando o


problema.

(CHINA-1996) Seja H o ortocentro do triângulo acutângulo ABC. As tangentes


traçadas por A ao círculo de diâmetro BC intersectam o círculo em P e Q. Prove
que P, Q e H são colineares.

Resolução:

H Q

B T O C
O

A idéia aqui é relativamente simples. Tome S ∈ TH tal que TS ⊥ AS. Sabemos que
∆HTO ≈ ∆HSA (AA) ⇒ HS/AH = HO/TH ⇒ HS = (AH.HO)/HT ⇒ HS.HT =
AH.HO. Como visto na figura, vamos usar Geometria Analítica:

COORDENADAS:

A (0, a)
B (– b, 0)
C (c, 0)
T((c – b)/2, 0) → T é ponto médio de BC.

EUREKA! N°20, 2004

29
Sociedade Brasileira de Matemática

Temos que BH ⊥ AC ⇔ m(BH).m(AC) = – 1 ⇔ (h/b).(a/(– c)) = – 1 ⇔ h = bc/a.

MEDIDAS:

r = ( b + c)/2 → raio do círculo por B e C


AH = a – h = (a2 – bc)/a
HO = h = bc/a
TH2 = h2 + ((c – b)/2 )2 = (bc/a)2 + (b – c)2/4.

Assim vem que TH.TS = TH.(TH + HS) = TH2 + TH.HS = TH2 + AH.HO = (bc/a)2
+ (b – c)2/4 + bc/a. (a2 – bc)/a = (bc/a)2 + (b – c)2/4 + bc – (bc/a)2 =
((b – c)2 + 4bc)/4 = ((b + c)/2)2 = r2 ⇒ de fato S ≡ H’, onde H’ é o inverso de H ⇒
A ∈ polar de H ⇒ H ∈ polar de A ⇒ H ∈ PQ.

Referências:
[1] Luciano G. M. Castro, Introdução à Geometria Projetiva, Eureka! N.º 8, pp. 16 – 27.
[2] http://www.kalva.demon.co.uk/. Site muito bom com um verdadeiro arsenal de problemas.

EUREKA! N°20, 2004

30
Sociedade Brasileira de Matemática

RETA DE EULER E NÚMEROS COMPLEXOS


José Paulo Carneiro, Rio de Janeiro - RJ

Nível Intermediário
INTRODUÇÃO:
O fato de os números complexos terem nascido no contexto da resolução de
equações algébricas fez com que muitas vezes sua utilidade em Geometria não
seja suficientemente explorada (uma notável exceção a esta tendência pode ser
encontrada em Eureka, Vol 6, no artigo Aplicações dos Números Complexos à
Geometria, do Prof. Edmilson Motta). Aqui, vamos usar a álgebra dos números
complexos para mostrar um belo resultado de Geometria, o fato de que, em
qualquer triângulo, o circuncentro K, o baricentro G e o ortocentro H são
colineares. A reta que contém estes três pontos é chamada Reta de Euler, já que
JJJJG JJJG
foi Euler o primeiro a chamar a atenção para este fato. Mais que isto, vamos
provar que, vetorialmente: KH = 3KG , o que, além de implicar que os três
pontos estão alinhados, acarreta que a distância KH é o triplo da distância KG e
que G e H estão na mesma semi-reta de origem K (ver Figura 1).
B

K
G
H
A
C
Figura 1

Para usar números complexos, de agora em diante estará fixado no plano um


sistema de coordenadas cartesianas ortogonais e as letras maiúsculas A, B,...,
designarão pontos do plano ou números complexos, de modo que cada ponto
JJJG
( x; y ) esteja identificado com o número complexo mais usualmente representado
por x + yi . Será fundamental a igualdade AB = B − A , a qual traduz que a
JJJG
translação definida pelo vetor AB é a mesma que leva a origem no complexo
B − A (Figura 2).

EUREKA! N°20, 2004

31
Sociedade Brasileira de Matemática

B–A

O
Figura 2

Deve ser observado que, mais usualmente, o símbolo AB designa o comprimento


do segmento AB . Porém aqui, como estamos identificando pontos do plano com
números complexos, o símbolo AB não será usado para o comprimento do
segmento AB , e sim para o produto dos complexos A e B.

Baricentro
JJJG JJJJG
É bem sabido que AG = 2GM , onde M é o ponto médio de BC e G é o
baricentro do triângulo ABC, isto é, o ponto de encontro das medianas do triângulo
(Figura 3)

B M C

Figura 3

 B+C 
Logo: G − A = 2(M − G ) , ou seja: G = A + 2  − G  = A + B + C − 2G ,
 2 
A+ B+C
de onde se conclui que G = .
3
EUREKA! N°20, 2004

32
Sociedade Brasileira de Matemática

Até aí, os complexos parecem não estar presentes. É que ainda não figura o
produto de complexos, que é a sua mais forte característica. Para efeito de soma e
de multiplicação por número real, os complexos funcionam apenas como vetores
do plano.

Um caso particular
Comecemos com um caso particular, a saber: vamos supor que os três vértices do
triângulo ABC estejam na circunferência unitária do plano, isto é, a circunferência
de centro na origem e raio 1, que é o conjunto dos complexos de módulo 1. Então,
o circuncentro de ABC coincide com a origem de coordenadas e
A = B = C = 1 . Mas para qualquer complexo z de módulo 1, temos:
zz =| z |2 = 1 (onde z é o conjugado de z). Conseqüentemente, A = 1/ A ,
B = 1/ B , C = 1/ C .
JJJJG JJJG
Para usar agora a condição AH ⊥ BC (onde H é o ortocentro de ABC), vamos
observar que o complexo v é perpendicular ao complexo w (considerados como
vetores não nulos) se e só se forem colineares com a origem os complexos v e
iw , ou seja, se e só se v iw for real (ver Figura 4).
y
w

v
x

iw

Figura 4

Por outro lado, um complexo é real se e só se for igual ao seu conjugado e,


v v v
portanto: v⊥w ⇔ = = ⇔ vw + vw = 0 (o leitor pode
iw iw −iw
verificar, colocando isto em coordenadas, que esta condição equivale à nulidade
do produto escalar dos dois vetores).
Temos, pois:

EUREKA! N°20, 2004

33
Sociedade Brasileira de Matemática

JJJJG JJJG
AH ⊥ BC ⇔ ( H − A)(C − B ) + ( H − A)(C − B) = 0
1 1  1
⇔ ( H − A)  −  +  H −  (C − B) = 0
C B  A
(B − C )  1
⇔ ( H − A) +  H −  (C − B ) = 0
BC  A
⇔ A( H − A) + BC (1 − AH ) = 0
⇔ AH = A2 + ABCH − BC
Atenção! lembre que AH não é o comprimento do segmento do segmento AH , e
sim o produto dos complexos A e H! O mesmo vale para BC, etc.
Analogamente:
JJJG JJJG
BH ⊥ CA ⇔ BH = B 2 + ABCH − CA

Subtraindo:
( A − B ) H = A2 − B 2 + C ( A − B )
( A − B) H = ( A − B)( A + B) + C ( A − B)
H = A+ B +C

Este resultado significa que, dados três complexos de módulo 1, sua soma é o
ortocentro do triângulo por eles formado.

Primeira generalização
Suponha agora que os três vértices do triângulo ABC estejam em uma
circunferência Ω , de centro na origem e raio r qualquer, ou seja, o circuncentro
de ABC coincide com a origem de coordenadas e A = B = C = r > 0 . Neste
A B
caso, como ilustra a Figura 5, os complexos (ou pontos) A ' = , B'= e
r r
C
C'= estarão na circunferência Γ , de centro na origem e raio 1. De fato:
r
A A
= = 1 , etc.
r r

EUREKA! N°20, 2004

34
Sociedade Brasileira de Matemática

B
A
B'
A'

O x

C'

C
Figura 5

Na verdade, a circunferência Γ e o triângulo A ' B ' C ' são os transformados da


circunferência Ω e do triângulo ABC, pela homotetia de centro O e razão 1 r , a
qual preserva ângulos, de modo que o ortocentro H ' do triângulo A ' B ' C ' é a
H
imagem do ortocentro H do triângulo ABC, isto é: H ' = . Mas então, pelo
r
resultado do caso particular estudado, temos:
H A B C
H '= = A '+ B '+ C ' = + + , donde se conclui que: H = A + B + C .
r r r r
Portanto: dados três complexos de mesmo módulo, sua soma é o ortocentro do
triângulo por eles formado (o que, em si mesmo, é um resultado curioso).

Caso geral
Passemos agora ao caso geral: dado um triângulo ABC qualquer, sejam K o seu
JJJG
circuncentro e r o raio do seu círculo circunscrito. Transladando os pontos A, B e
C pelo vetor KO (onde O é a origem do plano complexo), obtemos os complexos
(ou pontos) A − K , B − K e C − K , que pertencem a uma circunferência Ω de
centro na origem e raio r, como ilustra a Figura 6.

EUREKA! N°20, 2004

35
Sociedade Brasileira de Matemática

y
B

A
H
B–K K
A–K
H– K
C
O x

C–K
Figura 6

Sendo a translação uma isometria (que preserva distâncias e ângulos), o ponto


H − K será o ortocentro do triângulo de vértices A − K , B − K e C − K . Mas
então, pelo resultado anterior, temos:
H − K = A − K + B − K + C − K = A + B + C − 3K . Como A + B + C = 3G ,
onde G é o baricentro de ABC, temos: H − K = 3G − 3K = 3(G − K ) , ou
JJJJG JJJG
KH = 3KG , como se queria provar.

EUREKA! N°20, 2004

36
Sociedade Brasileira de Matemática

COMO É QUE FAZ?


PROBLEMA 4
PROPOSTO POR SAMUEL BARBOSA FEITOSA (FORTALEZA - CE)
a) Prove que, para todo inteiro positivo m, ∑µ(d) A
d |m
md
é divisível por m, para

todo inteiro A.
b) Defina a seqüência An por ∑A d |n
d = 2n. Prove que An é divisível por n, para

todo inteiro positivo n.

SOLUÇÃO:
a) Podemos supor A > 0, pois a afirmação só depende da classe de congruência
de A módulo m.

Seja g(m) = µ(d) Am d .
d |n

Temos ∑ g(k) = ∑∑µ(d) A


k| n k| n d | k
kd
= ∑ Ar ⋅ ∑ µ(d) = An .
r|n d|n / r

Dada uma seqüência ( xn )n≥1 .


Definimos seu período como o menor inteiro positivo t tal que xn+t = xn , ∀n ≥ 1 (se
existir). Veremos que g(n) é o número de seqüências de números inteiros ( xk )k≥1
de período n tais que 1 ≤ xk ≤ A, ∀k ≥ 1. De fato, o número de seqüências ( xk )k≥1
com xk +n = xk , ∀k ≥ 1 e xk ∈{1,2,..., A}, ∀k ≥ 1
é A n (uma tal seqüência é determinada pela escolha de x1, x2 ,..., xn ∈{1,2,..., A}).
Essas seqüências são exatamente as seqüências cujo período é um divisor d de n.
Assim, se f(d) é o número de tais seqüências de período d, f (d ) = An , ∀n ≥ 1,∑d |n

donde f (k) = g(k ), ∀k ≥ 1. Finalmente, o número g(n) de tais seqüências com


período n é múltiplo de n, pois, se declararmos duas seqüências ( xk )k≥1 e ( yk )k≥1
de período n equivalentes se existe t ∈ ` com xk +t = xk , ∀k ≥ 1, as classes de
equivalência de seqüências de período n têm exatamente n elementos: a classe de
equivalência de ( xk )k≥1 é {( xk +t )k≥1 ,0 ≤ t ≤ n −1} .

b) Temos, para A = 2, An = g(n) , na notação do item a), donde n | An , ∀n ≥ 1, pelo


item a).

EUREKA! N°20, 2004

37
Sociedade Brasileira de Matemática

PROBLEMA 5
PROPOSTO POR WILSON CARLOS DA SILVA RAMOS (BELÉM - PA) (de uma olimpíada chinesa)
Seja ABC um triângulo acutângulo de incentro I, ortocentro H e tal que AB ≠ AC .
B1 e C1 são os pontos médios de AC e AB. B2 é o ponto de interseção de IB1 com
AB. C2 é definido analogamente. Sejam também k a interseção de B2C2 com BC
e O o circuncentro de BHC. Se a área dos triângulos BKB2 e CKC2 é igual, mostre
que A, I e O são colineares.

SOLUÇÃO DE YURI GOMES LIMA (FORTALEZA - CE):


Lema: Seja ABC um triângulo de incentro I. Se M é o ponto médio de BC e X é o
ponto de tangência ao ex-incírculo relativo a BC com esse lado, então MI // AX .
Prova: Seja D como na figura e P a outra interseção de DI com o incírculo. A
homotetia de centro A que leva o incírculo no ex-incírculo leva P em X, e portanto
A, P, X são colineares. Sabemos que BD = XC = p – b.
A

D C
B
M X

Daí, M é médio de DX e assim MI é base média do triângulo PDX ⇒ MI // AX ❑


A
α
Z
B1
C1
I Y
B2 C
B K
C2

EUREKA! N°20, 2004

38
Sociedade Brasileira de Matemática

Note que
AB ⋅ ACsenα AB2 ⋅ AC2 ⋅ senα
[BKB2 ] = [CKC2 ] ⇒ [ ABC] = [ AB2C2 ] ⇒ = ⇒
2 2
AB AC2
⇒ AB ⋅ AC = AB2 ⋅ AC2 ⇒ = ⇒ BC2 // B2C. (1)
AB2 AC
Sejam Y, Z os pontos de tangência dos ex-incírculos relativos a AC e AB com esses
lados, respectivamente. Então, pelo lema temos B1I // BY (2) e C1I // CZ (3).
Daí, as relações (2), (1) e (3) implicam, nessa ordem, em:
AY AB AC2 AC1
= = = ⇒
AB1 AB2 AC AZ
AY AC1
⇒ = ⇒ AY ⋅ AZ = AB1 ⋅ AC1 ⇒
AB1 AZ
b c
⇒ ( p − c)( p − b) = ⋅ ⇒ (a + b − c)(a − b + c) = bc ⇒
2 2
⇒ a − ab + ac + ab − b2 + bc − ac + bc − c2 = bc ⇒ a2 = b2 + c2 − bc ⇒
2

1
⇒ cosα = ⇒α = 60° .
2
Isso garante que BHCl = BIC = 120°⇒ B, H, I ,C são concíclicos.
A

H
I

B C
O

IA

Para concluir, o centro I A do ex-incírculo relativo a BC está em AI e satisfaz


l = 90° ⇒ I I é diâmetro do circuncírculo de BHC, de modo que A, I e
l = ICI
I BIA A A
O são colineares.

EUREKA! N°20, 2004

39
Sociedade Brasileira de Matemática


SOLUÇÕES DE PROBLEMAS PROPOSTOS
Publicamos aqui algumas das respostas enviadas por nossos leitores.

83) Seja  = {0,1,2,3,...}.


Determine quantas funções f : ` → ` satisfazem f (2003) = 2003, f (n) ≤ 2003
para todo n ≤ 2003 e f (m + f (n)) = f ( f (m)) + f (n) , para todo m, n ∈` .

SOLUÇÃO DE ZOROASTRO AZAMBUJA NETO (RIO DE JANEIRO – RJ)


Fazendo m = n = 0, obtemos f ( f (0)) = f ( f (0)) + f (0), donde f (0) = 0, e logo
f ( f (0)) = f (0) = 0. Assim, fazendo m = 0, obtemos f ( f (n)) = f (n) para todo
n∈ ` , donde f ( y) = y, ∀y ∈ Im( f ). Seja t = min{ f (n); n ∈ `, f (n) > 0}. Note que
t ≤ 2003.
Temos t = f (n0 ) para um certo n0 ∈ ` , donde f (t ) = f ( f (n0 )) = f (n0 ) = t. Temos,
por indução, f (kt) = kt, ∀k ∈`.
De fato,
f ((k +1)t ) = f (kt + t) = f (kt + f (t)) = f ( f (kt )) + f (t ) = f (kt) + t = kt + t = (k + 1)t.
Além disso, para todo m∈ ` e k ∈ `,
f (m + kt) = f (m + f (kt)) = f ( f (m)) + f (kt ) = f (m) + kt.
Afirmamos que Im( f ) = { f (n), n ∈ `} = {kt, k ∈ `}. De fato, já vimos que
kt = f (kt)∈Im( f ), ∀k ∈`, e, se kt ≤ f (n) < (k +1)t, temos f ( f (n)) = f (n) = kt +( f (n) −kt),
donde f (n) = f (( f (n) − kt) + kt) = f (( f (n) − kt) + f (kt)) = f ( f ( f (n) − kt)) + f (kt) =
f ( f ( f (n) − kt)) + kt, donde f ( f ( f (n) − kt)) = f (n) − kt, e logo f (n) − kt ∈Im( f ),
mas 0 ≤ f (n) − kt < t, donde f (n) −kt = 0, para não contradizermos a definição de t.
No nosso caso, como f (2003) = 2003, temos t | 2003, donde, como 2003 é primo, t
= 1 ou t = 2003. Se t = 1, f (n) = n, ∀n∈`, o que claramente satisfaz as condições
do enunciado. Se t = 2003, 2003| f (n), ∀n∈`. Se 1 ≤ n ≤ 2002, devemos ter
f (n) ≤ 2003, donde f (n) ∈{0,2003}, e podemos escolher f(n) arbitrariamente em
{0, 2003} para 1 ≤ n ≤ 2002 (para o que temos, portanto, 22002 escolhas)
estendendo f a ` de modo que f (n + k ⋅ 2003) = f (n) + k ⋅ 2003, ∀n < 2003, k ∈`
(lembremos que f(0) = 0). De fato, para quaisquer m, n∈`, temos f (n) = 2003k,
para algum k ∈` , e, escrevendo
m = r + 2003 s, com 0 ≤ r ≤ 2002, s ∈`, temos

EUREKA! N°20, 2004

40
Sociedade Brasileira de Matemática

f (m + f (n)) = f (r + 2003(s + k)) = f (r) + 2003(s + k) =


= ( f (r) + 2003s) + 2003k = f (m) + f (n) = f ( f (m)) + f (n).
Assim, temos 1+ 22002 funções f que satisfazem as condições do enunciado.

84) Prove que se A⊂ * = {1,2,3,...} é um conjunto não-vazio tal que


n ∈ A ⇒ 4n ∈ A e  n  ∈ A então A = * .
Obs:  x é o único inteiro tal que x −1 <  x ≤ x .

SOLUÇÃO DE RODRIGO VILLARD MILET (RIO DE JANEIRO – RJ)


(i) n ∈ A ⇒ 4n ∈ A
 
(ii) n ∈ A ⇒ n ∈ A
Seja u um elemento de A (existe, pois A é não vazio).Veja que 1∈ A , pois, se u>1,
pela propriedade (ii), temos um elemento menor que u em A, logo, repetindo esse
argumento um número finito de vezes, temos que 1∈ A . Isso mostra que todas as
potências de 4 estão em A (por (i)).
Agora vou fazer o seguinte: dado m natural, mostrarei que existe alguma potência
de 4 no intervalo m 2 , (m + 1)2  , para algum k natural (daí, como a potência de
k k

 
4 está em A, usando a propriedade (ii) k vezes, temos que m está em A; note que
  x  =  x  , ∀x ≥ 1 ). Suponha então que para todo k tenhamos um t tal que
   
4 t < m 2 < (m + 1)2 < 4 t +1 . Daí, segue que ( m + 1) < 4t +1 = 4.4t < 4m 2 , logo
k k 2k k

2k
 1
1 +  < 4 , para todo k natural, o que é uma contradição, pois
 m
2k
 1 2k
> 4 , para k = m + 2, pois 2 = 2 + 2 = 4 ⋅ 2m > 4m. Logo, m ∈ A ,
k m
1 +  ≥ 1 +
 m  m
para todo m natural.

85) Mostre que todo triângulo pode ser dividido em 9 pentágonos convexos de
áreas iguais.

SOLUÇÃO DE JOSÉ DE ALMEIDA PANTERA (RIO DE JANEIRO – RJ)


Em primeiro lugar, aplicando uma transformação afim (que preserva relações
entre áreas e leva pentágonos convexos em pentágonos convexos) ao triângulo,

EUREKA! N°20, 2004

41
Sociedade Brasileira de Matemática

podemos supor, sem perda de generalidade, que o triângulo é equilátero de lado 1


1 3
(digamos com vértices (0, 0), (1, 0) e  ,  ). Fazemos então a seguinte
2 2 
construção, simétrica em relação à rotação de 120° em torno do centro O do
triângulo (e também simétrica em relação às bissetrizes internas do triângulo):
C

X3 Y3
Z3 W3

T3 T2
O
Y1 X2
W1 Z2
T1

Z1 W2
A X1 Y2 B
1
Temos AX 1 = AY1 = a = − ε, (onde ε > 0 é pequeno)
3
l 1 A = W1 Yl 1 A = 90°, X1 Z1 = YW
Z1 X 1 1 = b
(assim, por exemplo, X1 = (a, 0) e Z1 = (a, b)) ,
a2 3  b 3b
onde b é escolhido de modo que a área +  a −  do pentágono
4  2  2
3 1
AX 1 Z1W1Y1 seja igual a , que é da área do triângulo, ou seja, temos
36 9
1 − 3ε
 1 8ε 
b= −  − + ε 2 . Note que, se ε > 0 é pequeno então b > 0 é pequeno.
3 3  27 9 

Escolhemos T1 =  , b + h  de modo que a área (1 − 2a )  b +  do pentágono


a h
2   2
3
X 1Y2W2T1 Z1 seja também igual a . Ou seja, temos
36

EUREKA! N°20, 2004

42
Sociedade Brasileira de Matemática

3 3 3 3
h= − 2b = − 2b < (e h é próximo a se ε>0 é
18(1 − 2a) 1  6 36
18  + 2ε 
3 
pequeno).
Como os pentágonos AX 1 Z1W1Y1 , BX 2 Z 2W2Y2 e CX 3 Z 3W3Y3 são congruentes,
X 1Y2W2T1 Z1 , X 2Y3W3T2 Z 2 e X 3YW
1 1T3 Z 3
são congruentes, W1 Z1T1OT3 , W2 Z 2T2 OT1 e
1
W3 Z 3T3 OT2 são congruentes e os 6 primeiros têm da área do triângulo, todos têm
9
1
da área do triângulo.
9

86) Encontre todas as triplas de inteiros positivos (a, m, n) tais que am +1 divide
(a +1)n .

SOLUÇÃO DE ANDERSON TORRES (SÃO PAULO – SP)


É fácil ver que (1; m; n), (a; 1; n) são soluções.
Vamos então supor m ≥ 2, a ≥ 2.

Lema 1: m é ímpar.
Demonstração: vamos dividir em casos:
Caso 1: 4 divide ( a + 1).
m

Assim, a ≡ −1(mod 4). É um fato muito conhecido que (– 1) não é resíduo


m

quadrático módulo 4, pois todo quadrado perfeito ímpar é congruente a 1 módulo


4.
m
Logo a não é quadrado perfeito, e portanto m não pode ser par.
Caso 2: Existe um primo α ≠ 2 tal que α ( a + 1).
m

Então α ( a + 1) ( a + 1) , e, como α é primo, α a + 1.


m n

a ≡ −1(mod α) ⇒ a m + 1 ≡ ( −1) m + 1(mod α ), e já que α ( a + 1), temos


m

( −1) m ≡ ( −1)(mod α ) ⇒ m é ímpar.

Caso 3: O complementar dos casos 1 e 2.


Assim sendo, a m + 1 = 2 ⇒ a m = 1. Mas a ≥ 2 = 4 > 1, absurdo e fim do
m 2

lema.

EUREKA! N°20, 2004

43
Sociedade Brasileira de Matemática

Seja p m um primo (ímpar, como já sabemos).


ap +1
Então ( a + 1) ( a + 1) ( a + 1) ( a + 1) ⇒
p m n
( a + 1) n −1 .
a +1
xp +1
Seja f ( x) = = x p −1 − x p − 2 + x p − 3 − ... − x + 1.
x +1
Temos f ( −1) = p, logo f ( x) = ( x + 1) g ( x) + p para algum polinômio
g ∈ ][ x]. Em particular, f ( a ) = ( a + 1) g ( a ) + p.
Seja q um fator primo de f(a). Então q f ( a ) ( a + 1)
n −1
⇒ q ( a + 1).
Então 0 ≡ f ( a ) ≡ p(mod q ).
Como p e q são primos, p = q. Logo p ( a + 1) e f(a) é potência de p. Note que
f ( a) > 1. Vamos fazer mais.

Lema 2: f(a) = p.
Demonstração: Em dois casos:

2
Caso 1: p divide a + 1.
Então f ( a) = ( a + 1) g ( a ) + p ≡ p ≡ 0(mod p ) ⇒ p não divide f(a)
2 2

2
Caso 2: p não divide (a + 1).
a p + 1 ( hp − 1) p + 1
Então a + 1 = hp , onde p não divide h. Assim a = hp − 1, =
a +1 hp
 p  p  p  p p p
( hp − 1) =   ( hp) ( −1) +   hp (−1) −1 +   h 2 p 2 ( −1) − 2 + ... +   h p ,
p p p p p

0  1  2   p
 p
p ⋅ hp −   h2 p2 + ... + h p p p
(hp −1) +1
p
2   p  p
= = p −   hp +   h2 p 2 + ... + h p − 2 p p −1 ,
hp hp 2  3 
e assim,
(hp −1) p + 1  p  p −1 
≡ p −   hp ≡ p −   ⋅ p ⋅ hp ≡ p ≡ 0 (mod p ) .
2

hp 2   2 

EUREKA! N°20, 2004

44
Sociedade Brasileira de Matemática

2
Em todo caso, p não divide f ( a ) mas p f ( a ). Logo, como f ( a ) é potência
de p, f ( a ) = p.

ap +1
Assim, = p.
a +1

Lema 3: p = 3
Demonstração: vamos por absurdo. Suponha p ≥ 5.

ap +1
Então é crescente. De fato,
a +1
ap +1 a p −a a p− −1
1
=1+ = 1 + a ( a − 1) 2 ⇔
a +1 a +1 a −1
ap +1
= 1 + a (a − 1)( a p − 3 + a p − 5 + ... + 1), uma composição de funções
a +1
crescentes.
a p +1 2 p +1
Assim, p = ≥ (pois a ≥ 2 ) ⇒ 3 p ≥ 2 + 1 > 2
p p

a + 1 3
Mas é fácil ver que p ≥ 5 ⇒ 3 p < 2 (é uma indução simples).
p

E isto é absurdo! Logo p < 5 e p é primo ímpar, logo p = 3.


a +1
3

Assim, = 3 ⇔ a 3 − 3a − 2 = 0. É fácil ver que (– 1) é raiz disto.


a +1
a − 3a − 2
3

E = a 2 − a − 2 = 0 ⇔ a ∈ {2, −1}.
a +1
Como a > 0, temos a = 2.
Se m = pk, e 2m + 1 divide 3n, (2 ) + 1 divide (2 + 1) , pois k é ímpar, donde
k p k n

2 k + 1 é múltiplo de 3. Assim, (2 k ; p; n) é solução, e logo 2 k = 2, donde k = 1 e


m = p = 3. Assim, todas as outras soluções são da forma (2; 3; n), com n ≥ 2.

EUREKA! N°20, 2004

45
Sociedade Brasileira de Matemática

87) Seja a(1) = 1 e, para cada inteiro n ≥ 2, a(n) igual ao menor inteiro positivo
n
que não pertence a {a( j), j < n} tal que ∑a( j) seja múltiplo de n. Prove que
j =1

a(a(n)) = n para todo inteiro positivo n.

SOLUÇÃO DE SAMUEL BARBOSA FEITOSA (FORTALEZA – CE)


Sejam F2 = F1 = 1 e, para n ≥ 2, Fn = Fn −1 + Fn − 2 os números de Fibonacci: Pelo
Teorema de Zeckendorff sabemos que todo número natural pode ser escrito de
maneira única como soma de números de Fibonacci com índices maiores que 1 e
não consecutivos. (isso pode ser provado por indução: temos 1 = F2 e, se Fk é o
maior número de Fibonacci que é menor ou igual a n, devemos ter n − Fk < Fk −1
pois, caso contrário,n − Fk ≥ Fk −1 , e logo n ≥ Fk + Fk −1 = Fk +1 , absurdo;
escrevemos então n = Fk + ( n − Fk ) e aplicamos e resultado para n − Fk ) .
Vamos criar uma pequena variação desta representação; chamemos esta nova
representação de representação F.
Suponha que m = Fi1 + Fi2 + ... + Fin (na representação descrita no Teorema anterior)
Com i1 > i2 > ... > in > 1. Se in ≠ 2 a representação de m na representação – F
será a mesma. Se in = 2 ⇒ m será representando na F como:
Fi1 + Fi2 + ... + Fi"n−1 + F2 se in −1 é ímpar.
Fi1 + Fi2 + ... + Fin−1 + F1 se in −1 é par. (1 será F2 ).
Veja que todo número pode ser escrito na "F" de maneira única.
n
Seja S k = ∑ A . Afirmamos que:
i =1
i

Se K = Fi1 + Fi2 + ... + Fin com i1 > i2 > ... > in é a representação de K na F.
A) in par ⇒ Ak = Fi1 −1 + Fi2 −1 + ... + Fin −1 ∈ Sk = (Fi1 + Fi2 + ... + Fin )(Fi1 −1 + Fi2 −1 + ... + Fin −1)
B) in ímpar ⇒ Ak = Fi1−1 + Fi2 −1 + ... + Fin −1 ∈ Sk = Fi1+1 + Fi2 +1 + ... + Fin +1 ∈ Sk =
= ( Fi1 + Fi2 + ... + Fin )( Fi1 −1 + Fi2 −1 + ... + Fin −1 + 1)
Vamos provar a afirmação acima por indução:

Casos iniciais: A1 = 1, A2 = 3, A3 = 2, S1 = 1, S 2 = 4, S3 = 6 (verifica-se


facilmente que eles satisfazem as condições.

EUREKA! N°20, 2004

46
Sociedade Brasileira de Matemática

Supondo a afirmação acima verdadeira todo r ≤ k . Provemos que ela também é


verdadeira para k + 1. Suponhamos in > 3 (o caso em que in = 1, 2, 3 é
totalmente análogo ao que faremos agora, a única diferença consiste na utilização
das seguintes relações:
F1 + F3 + ... + F2 r +1 = F2 r + 2 , F2 + F4 + F6 + ... + F2 r = F2 r +1 − 1)
Vamos dividir agora em dois casos:
i) in é par ⇒ na "F" k + 1 = Fi1 + Fi2 + ... + Fin + F1. Sabemos que
Ak +1 + S k ≡ 0(mod k + 1) = (mod Fi1 + Fi2 + ... + Fin + F1 ). Mas
Sk = ( Fi1 + Fi2 + ... + Fin )( Fi1 −1 + Fi2 −1 + ... + Fin −1 ) ≡ −( Fi1 −1 + Fi2 −1 + ... + Fin −1 )(mod k + 1)
⇒ Ak +1 ≡ Fi1 −1 + Fi2 −1 + ... + Fin −1 (mod k +1). Veja que o número
Fi1 −1 + Fi2 −1 + ... + Fin −1 = Ak já está na seqüência. Logo
Ak +1 ≥ Fi1 −1 + Fi2 −1 + ... + Fin −1 + (k +1) = Fi1 +1 + Fi2 +1 + ... + Fin +1 + F2 (trocamos F1 por
F2 para termos uma representação –F)
mas pela nossa hipótese de indução o número Fi1 +1 + Fi2 +1 + ... + F2 só pode ter
aparecido na seqüência oriundo do número Fi1 + Fi2 + ... + F1 = k + 1 > k logo ele
ainda não está na seqüência ⇒ a(k + 1) = Fi1 +1 + Fi2 +1 + ... + F2 .
Vejamos que
S k +1 = a (k + 1) + S k = Fi1 +1 + ... + F2 + ( Fi1 + Fi2 + ... + Fn )( Fi1 −1 + ... + Fin −1 ) =
= Fi1 + Fi2 + ... + Fin + F2 + Fi1 −1 + Fi2 −1 + ... + Fin −1 + (Fi1 + Fi2 + ... + Fin )( Fi1 −1 + ... + Fin −1 ) =
= (Fi1 −1 + Fi2 −1 + ... + Fin −1 )(1+ Fi1 + Fi2 + ... + Fin ) + (Fi1 + Fi2 + ... + Fin + F2 ) =
= (1 + Fi1 + Fi2 + ... + Fin )( Fi1 −1 + Fi2 −1 + ... + Fin −1 + 1) =
= (Fi1 + Fi2 + ... + Fin + F1 )(Fi1 −1 + Fi2 −1 + ... + Fin −1 + 1) (que coincide exatamente com a
nossa afirmação para k + 1).

ii) in é ímpar. Este caso é análogo ao anterior.


Com a afirmação verdadeira é fácil concluir que a(a(n)) = n . Na F, a(a(n)) é
somar 1 a todos os índices e depois subtrair, ou o contrário, daí os índices ficam os
mesmos.

EUREKA! N°20, 2004

47
Sociedade Brasileira de Matemática

 1 
88) Prove que se r ∈_ e cos(r ⋅π ) ∈_ então cos(r ⋅π ) ∈−1, − ,0, ,1.
1
 2 2 
SOLUÇÃO DE ANDRÉS SÁNCHEZ PÉREZ (LA HABANA, CUBA)

r ∈_ ⇒ r =
f
onde p, q ∈ ] , com mdc ( p; q ) = 1 . cos(r ⋅ π )∈ Q ⇒ cos(r ⋅ π ) =
p
q g
donde f , g ∈ Z , também com mdc ( f ; g ) = 1 . Aplicando Moivre, se
z = cisα = cos α + isenα então
n
n
z n = cis ( n ⋅ α ) = cos ( n ⋅ α ) + isen ( n ⋅ α ) = (cos α + isenα ) = ∑   (cos α ) (isenα )
n n− j j

j =0  
j
n   n −1
   
j n   n 
 2  2 
= ∑ ( −1)   (cos α ) ( senα )2 j + i ∑ ( −1) j  ( cos α )n− 2 j−1 ( senα )2 j+1
n− 2 j

j =0 2 j j =0  2 j + 1

igualando parte real com parte real:


n
2
 
n
cos(n ⋅ α ) = ∑ (− 1)  2 j (cos α ) (senα )
n− 2 j
j 2j

j =0
n 
 
j n 
2  j
cos ( n ⋅ α ) = ∑ ( −1)   (cos α ) 1 − (cos α )2  =
n−2 j

j =0 2j  
 n
 
 n
 2   n  
n n−2 j  k  j 2k 
 2   n  k 
j  
 2 j  
= ∑ ( −1)   (cos α ) ∑ ( −1)   (cos α )  = ∑ ( −1) ∑     (cos α ) ,
j n−2 j

j =0 2 j  k =0 k  j =0  k = j    
2k j

 k  k 
lembrando que  = .
k − j   j 
  n 
n
2

j  n  k 
 
2
Logo temos que cos(n ⋅ α ) = (− 1)     (cos α )n−2 j . Agora, veja que
∑ ∑
j =0  k = j  2k  j 
 
sempre que cos α ∈ _,cos ( n ⋅ α ) ∈ _, para todo n∈] (pois também
cos β = cos(− β ) ). Se a ∈ ] e a é par então cos(a ⋅ π + α ) = cos α , e se a ∈ ] e
a é ímpar então cos ( a ⋅ π + α ) = − cos α. .

EUREKA! N°20, 2004

48
Sociedade Brasileira de Matemática

Logo, se cos α ∈ _,cos ( a ⋅ π + b ⋅ α ) ∈ _, ∀a, b ∈ ] .

 p   π ( aq + bp ) 
 ∈_ .
p
Fazendo α= ⋅ π ⇒ cos  a ⋅ π + b ⋅ ⋅ π  = cos  Como
q  q   q 
mdc ( p; q ) = 1, ∃ a, b inteiros tais que aq + bp = 1 . Por conseguinte, se
 π
cos   ∈ _ com q = 2 h ⋅ (2c + 1) , onde h ∈ `, c ∈ ] , supondo sem perda de
q
generalidade que 2c + 1 > 0 (pois cos α = cos (− α ) ), temos
 π   π  x
cos   ∈ _ ⇒ ∃x, y ∈ ] tais que cos 2c + 1  = y , com mdc ( x; y ) = 1 .
 2c + 1   

 2c +1
 2    2c2+1  2 c +1− 2 j
  2c + 1  k 
 π   
j

 π 
cos  ( 2c + 1)
 2c
=
+1
∑ ( )∑
−1
 2 k
   
 j  
cos
2c + 1


=
j =0  k = j 
 
2c +1− 2 j
c  c  2c + 1 k    π  c
j
c
 2c + 1 k   x 2c +1− 2 j
= ∑ ( −1)  ∑  = ∑ ( −1)  ∑ 
j
    cos     2 c +1−2 j ⇒
j =0  k = j  2k  j    2c + 1  j =0  k = j  2k  j   y
j  2c + 1 k   x 2 c +1−2 j
c c
⇒ −1 = ∑ (−1)  ∑     2 c +1−2 j ⇒
j =0  k = j  2k  j   y
j  2c + 1 k   2 c +1−2 j 2 j
c c
⇒ − y 2 c +1 = ∑ ( −1)  ∑     x ⋅y ⇒
j =0  k = j  2k  j  
 c j
c
 2c + 1 k   2 c −2 j 2 j 
⇒ − y 2 c +1 = x  ∑ ( −1)  ∑     x ⋅y 
 j =0 
  k = j  2k  j   
⇒ x − y 2 c +1 , mas como (x; y ) = 1 ⇒ x = −1, x = 0 ou x = 1

Por outro lado

c
 2c + 1 k  c
j 
c
 2c + 1 k   2 c +1−2 j 2 j −1
− y 2 c +1 = x ∑    + y ∑ ( −1) ∑     x ⋅y
k = 0  2 k  0  j =1  k = j  2k  j  
c
 2c + 1 k  c
 2c + 1
⇒ y ∑   ⇒ y ∑  
k = 0  2 k  0  k =0  2k 

EUREKA! N°20, 2004

49
Sociedade Brasileira de Matemática

2 c +1
2c + 1 c  2c + 1 c  2c + 1 
Do Binômio de Newton (1 + 1)2c +1 = ∑  =  ∑+  ∑  e
k =0  k  k =0  2k  k =0  2k + 1
2 c +1
2c + 1 c  2c + 1 c  2c + 1 
(1 − 1)2c+1 = ∑ (− 1)k   =  ∑  −  ∑
 ; somando e dividindo por
k =0  k  k =0  2k  k =0  2k + 1
 2c + 1
 = 2 , logo y = 2 ou y = −2 com t ∈ `, t ≤ 2c .
c
2, ∑
k =0 
2c t t
2 k 
 π  π π   2c − 1 
cos  = sen −  = sen  ⋅π  .
 2c + 1   2 2c + 1   2(2c + 1) 
Se e > 0 , e 2c − 1 e e 2 (2c + 1), e 2 (2c − 1) − 2 ( 2c + 1) ⇒ e −4 porém como e é
ímpar ( e 2c − 1 ) então e = 1 ⇒ mdc ( 2c − 1; 4c + 2 ) = 1 .

Volvendo a Moivre, e igualando parte imaginária a parte imaginária, com n


ímpar:
 n −1 
 
 2 
j n 
sen(n ⋅ α ) = ∑ (− 1)  2 j + 1(cos α ) (senα )2 j +1
n − 2 j −1

j =0
 n −1 
 
 2 
 n   n −1 
= ∑ (− 1) j 
j =0
(cos α )2  2 − j  (senα )2 j +1
 2 j + 1
 n −1 
 2 

[ ]
   n −1 
 n   − j
= ∑ (− 1) j 
j =0
 1 − (senα )2
 2 j + 1
 2  (senα )2 j +1
 n −1 
 2   n −1 − j  n −1  
 
(− 1) j 
n  2 2k 
= ∑  ∑ (− 1)k  2 − j ( ) (
 senα  senα
)2 j +1
j =0  2 j + 1  k = 0  k  
 
 n −1 
 2 
   n  n − 1 − k 
 j
= ∑ (− 1) j ∑ 
j =0

 k =0  2k + 1 2j − k 
(senα )2 j +1
  
Agora, veja que sempre que senα ∈ _ e n é ímpar, sen ( n ⋅ α ) ∈ _ . Se u ∈ ] e u
é par então sen(u ⋅ π + α ) = senα , e se u ∈] e u é ímpar então

EUREKA! N°20, 2004

50
Sociedade Brasileira de Matemática

sen(u ⋅ π + α ) = − senα , logo, se senα ∈ _ ⇒ sen (u ⋅ π + v ⋅ α ) ∈ _, ∀u , v ∈ ] com


2c − 1
v ímpar. Fazendo α = ⋅ π,
2 ( 2c + 1)
 2c − 1   (2u ( 2c + 1) + v ( 2c − 1)) 
sen  u ⋅ π + v ⋅ ⋅ π  = sen  ∈_ .
 2 (2c + 1)  
 2 ( 2 c + 1) 

Como mdc ( 2c − 1;4c + 2 ) = 1, ∃ u, v inteiros tal que u (4c + 2) + v(2c − 1) = 1 . do
 π 
Note que v é claramente ímpar. Por conseguinte, sen 
  ∈ _ .
 2 ( 2c + 1) 
2
 π    π 
= 1 − 2  sen 
 2 ( 2c + 1)  
cos  
 2c + 1  
  
 π   π  2
Se cos   = 0, sen   = ± que é uma contradição.
 2c + 1   2 ( 2c + 1)  2

 π  1  π  2t − 1
Se cos   = , sen   = ± . Como ambos (numerador e
 2c + 1  2  2 ( 2c + 1)  2t +1
t

denominador) são primos relativos (para t ≠ 0 ), então são quadrados perfeitos,


logo t é ímpar. Porém se t ≥ 3, 4 2t ⇒ 2t − 1 ≡ −1( mod 4 ) , o que é absurdo pois é
 π  π
um quadrado. Com t = 0, sen  = 0 , mas ≠ 2kπ . Então, nesse
 2 ( 2c + 1)  2c + 1
 
 π  1
caso, cos = .
 2c + 1  2
 π  1  π  2t + 1
Se cos   = − t , sen   = ± . Como ambos (numerador e
 2c + 1  2  2 ( 2c + 1)  2t +1
denominador) são primos relativos (para t ≠ 0 ), então são quadrados perfeitos, e
logo t é ímpar. Temos 2 t + 1 = d 2 ⇔ 2 t = (d + 1)(d − 1) ⇔ d − 1 = 2θ ,
d + 1 = 2θ + 2 = 2ω . Dividindo por 2 concluímos que θ = 1, ω = 2 e
 π  1  π 
t = 3 ⇒ cos  = − , porém, se c = 0, cos   = −1 , e para c ≥ 1 ⇒
 2c + 1  8  2c + 1 

EUREKA! N°20, 2004

51
Sociedade Brasileira de Matemática

π π  π   π 
0< < ⇒ cos  > 0 . Para t = 0, sen   = ±1 . Então, nesse
2c + 1 2  2c + 1   2 ( 2c + 1) 
 π 
caso, cos  = −1 .
 2c + 1 
 π  1  π π π 
Finalmente cos  ∈  , −1 ⇒ 2c + 1∈ {1,3}⇒ ∈  h ,  com
 c+  
2 1 2  q 2 3 ⋅ 2h 
π π   π  π
h ∈ ` . Se para ∈ _ , o que é
2
∈  h , h ≥ 2  , cos  h ⋅ 2h −2  = cos   =
q 2  2   4 2
π  π   π   π
∈ _ , o que
3
absurdo. Se ∈ , h ≥ 1 , , cos  ⋅ 2h −1  = cos   =
q 3⋅ 2 h
  3⋅2
h
 6 2
π  π π p  π π
também é absurdo. Assim, ∈ π , ,  , ⋅ π ∈  p ⋅ π , p ⋅ , p ⋅  e
q  2 3 q  2 3
p   1 1 
cos ⋅ π  ∈ − 1,− ,0, ,1 .
 q   2 2 

SEGUNDA SOLUÇÃO DE CARLOS GUSTAVO TAMM DE ARAUJO MOREIRA e JOSÉ PAULO


CARNEIRO (RIO DE JANEIRO - RJ)

p p q2 − p2
Note que se cos( x) = , então z := + i é uma raiz da unidade (isto é,
q q q
z n = 1 para algum inteiro positivo n) se e só se x é um múltiplo racional de π .
Assim, se x = r π, com r ∈ _ então z n = 1 para um certo inteiro positivo n (e logo
também temos z 2 n = 1 ). Vamos mostrar que q ≤ 2 , o que resolve o problema. Para
isso, vamos supor por absurdo que q > 2 (e p é primo com q).
Temos então dois casos:
a) q é ímpar. Nesse caso, para cada m ∈ ` , seja
xm = (( p + i q 2 − p 2 ) m − ( p − i q 2 − p 2 ) m ) / 2i q 2 − p2 . Temos x0 = 0, x1 = 1 e,
como p + i q 2 − p 2 e p − i q 2 − p 2 são raízes da equação x 2 − 2 px + q 2 = 0 ,
( xm ) satisfaz a recorrência xm +2 = 2 pxm +1 − q 2 xm , ∀m ∈ `.
Assim, xm + 2 ≡ 2 pxm +1 (mod q), ∀m ∈ `, donde, por indução, para todo
m −1
m ≥ 1, xm ≡ (2 p) (mod q). Em particular, como mdc(2 p, q) = 1, temos
mdc( xm , q) = 1, para todo m ≥ 1, e logo xm ≠ 0 , ∀m ≥ 1, mas

EUREKA! N°20, 2004

52
Sociedade Brasileira de Matemática

2n
 p + i q2 − p2 
( ) =(p−i )
n n
z 2n
=1⇒   = 1 ⇒ p + i q 2 − p2 q2 − p2 (pois
 q 
 
2
p + i q2 − p2  p + i p2 − q2 
=  ) , e logo xn = 0 , absurdo.
p − i q 2 − p 2  q 

b) q é par. Nesse caso, se


 m
  p − i q2 − p2 
m

p + i q2 − p2
ym =    −   i q 2 − p 2 , temos y0 = 0, y1 = 1
 2   2  
    
p + i q2 − p2 p − i q2 − p2 q2
e, como e são raízes da equação x 2 − px + = 0,
2 2 4
y , ∀m ∈ `.
q2
temos ym+ 2 = pym+1 −
4 m
Assim, ym +2 ≡ pym +1 (mod q 2) , donde, por indução, para todo
m ≥ 1, ym ≡ p m −1 (mod q 2), e logo mdc ( ym , q 2 ) = 1, para todo m ≥ 1, pois
mdc ( p, q 2 ) = 1. Em particular, ym ≠ 0, ∀m ≥ 1 (note que q > 2, logo q 2 ≥ 2,
2n
 p + i q2 − p2 
pois q é par). Por outro lado, z 2n
=1⇒   = 1, donde
 q 
 
n n
 p + i q2 − p2   p − i q2 − p 2 
  =  , como antes, e logo yn = 0, absurdo.
 2   2 
   

91) Um jardinero deve construir um canteiro com a forma de setor circular. Ele
dispõe de 100 metros de fio para cercá-lo.
Figura:

EUREKA! N°20, 2004

53
Sociedade Brasileira de Matemática

Qual deve ser o valor do raio do círculo para que o canteiro tenha área máxima?
Qual é a área máxima?

SOLUÇÃO DE GLAUBER MORENO BARBOSA (RIO DE JANEIRO - RJ)


Primeiramente nomeia-se os elementos do setor circular: (a)  o ângulo do setor
circular, (b) S, área máxima, (c) 2p, o perímetro do setor circular,
(d) k, comprimento do arco do setor circular.
Comprimento da circunferência = 2πr
Fazendo uma regra de três:
k θ 360k 180k
= ⇒θ = ⇒θ = (1)
2πr 360 2πr πr
Conforme o enunciado, o jardineiro dispõe de 100 metros de fio para cercar o
setor circular. Somando-se os segmentos referentes aos raios e ao arco do setor
circular, tem-se:
2 p = 2r + k = 100 ⇒ k = 100 − 2r (2)
Substituindo-se a relação (1) em (2) tem-se:
180k 180 (100 − 2r )
θ= ⇒θ = (3)
πr πr
Com (3) determina-se a área do setor circular com uma relação entre a área do
setor circular e a área da circunferência:
S θ θπr 2
= ⇒ S = (4)
πr 2 360 360
Substituindo-se o  em (3) na relação (4), tem-se uma nova relação para a área do
setor circular
180 (100 − 2 r ) 2
πr
θπr 2
πr 180 (100 − 2r ) r (100 − 2 r )r
S= = = = = −r 2 + 50r
360 360 360 2
S ( r ) = −r 2 + 50r (5)
Observando-se a função para a área S em (5), que é uma função do 2º grau, com
a < 0, e logo tem-se um máximo para a função. Assim S(r) é máximo em (5) para:
−b − 50
r = ⇒ = 25 (6)
2a −2
Para o valor de r em (6), tem a área máxima da função S(r) em (6):
− (25) 2 + 50(25) = −625 + 1250 ⇒ S = 625
Assim, se o raio para se ter área máxima é 25 metros e a área máxima
correspondente é 625 m2.

EUREKA! N°20, 2004

54
Sociedade Brasileira de Matemática

92) Seja (Fn)n∈ a seqüência de Fibonacci, definida por F1 = 1, F2 = 1 e


Fn+2 = Fn+1 + Fn , ∀n ∈ `. Prove que mdc (Fm , Fn ) = Fmdc(m, n) para quaisquer
inteiros positivos m e n.

SOLUÇÃO DE GIBRAN MEDEIROS DE SOUZA (NATAL - RN)


Para provarmos o que queremos temos que antes mostrar três premissas, que são:
I) Se m≥ 1 e n > 1, então Fn+m = Fn−1 ⋅ Fm + Fn ⋅ Fm+1

Prova: (por indução sobre m)


m = 1: Fn+1 = Fn−1 ⋅ F1 + Fn ⋅ F2 = Fn−1 + Fn (verdadeira)
m = 2: Fn+2 = Fn−1 ⋅ F2 + Fn ⋅ F3 = Fn−1 + 2Fn = ( Fn−1 + Fn ) + Fn = Fn + Fn+1 (verdadeira)
Seja r > 2 e suponhamos a propriedade verdadeira para todo k, 2 ≤ k < r, e para
todo n > 1.
Esta suposição, mais o fato de que a propriedade vale também para k = 1, nos
garante que:
Fn+(r−2) = Fn−1 ⋅ Fr −2 + Fn ⋅ Fr −1 e Fn+(r−1) = Fn−1 ⋅ Fr−1 + Fn ⋅ Fr

Somando membro a membro essas igualdades e levando em conta a fórmula


recursiva que define (Fn ):
Fn+r = Fn−1 ⋅ Fr + Fn ⋅ Fr+1

Ou seja, a fórmula vale também para r, sempre que n > 1 . O segundo princípio de
indução nos garante então que vale para todo m≥ 1 e qualquer n > 1.

II) Dois números de Fibonacci consecutivos ( Fn e Fn+1 ) são primos entre si.
Prova: Seja d = mdc(Fn Fn+1 ). Como Fn e Fn+1 são maiores que zero, o mesmo
ocorre com d. O fato de d ser divisor de Fn e Fn+1 implica que d | Fn−1 pois
Fn−1 = Fn+1 − Fn .
Dividindo Fn e Fn−1 , então d divide Fn−2 . Prosseguindo nesse raciocínio
chegaremos a conclusão que d | F2. Então d = 1, pois F2 = 1

III) Se m | n , então Fm | Fn .

EUREKA! N°20, 2004

55
Sociedade Brasileira de Matemática

Prova: Por hipótese n = mr, para algum r ∈` . Procedemos por indução sobre r.
Se r = 1, então m = n e é imediato que Fm | Fn .
Seja r ≥ 1 e admitamos que Fm | Fmr .
Então levado em conta a relação fornecida por I:
Fm(r+1) = Fmr+m = Fmr−1 ⋅ Fm + Fmr ⋅ Fm+1
Como Fm | Fmr −1 ⋅ Fm e Fm | Fmr ⋅ Fm+1 (pois, pela hipótese de indução, Fm divide
Fmr ), então Fm divide a soma desses dois produtos. Ou seja: Fm | Fm(r+1)
Com as três premissas em mão vamos à questão: Se d = mdc(m, n) , então prove
que mdc( Fm , Fn ) = Fd .

Prova: Mostraremos primeiro que se m = nq + r, então mdc( Fm , Fn ) = mdc( Fn , Fr ).


Observando as premissas feitas e levando em conta I:
mdc(Fm , Fn ) = mdc(Fnq+r , Fn ) = mdc(Fnq−1 ⋅ Fr + Fnq ⋅ Fr +1, Fn )
Considerando porém que mdc(a, b) = mdc(a + c, b), sempre que b|c, e ainda que
Fn | Fnq (premissa III), chegamos a:
mdc(Fm , Fn ) = mdc(Fnq−1 ⋅ Fr , Fn )
Mostremos que Fnq−1 e Fn são primos entre si. De fato, se d é um divisor comum a
esses dois números, então d | Fnq−1 e d | Fnq (devido a premissa III). Daí a premissa
II nos assegura que d = 1.
Ora, se mdc(Fnq−1, Fn ) = 1, então mdc(Fr , Fn ) = mdc(Fnq−1 ⋅ Fr , Fn ), donde:
mdc( Fm , Fn ) = mdc( Fn , Fr ).
Assim, supondo m > n, e aplicando o processo das divisões sucessivas para chegar
a d = mdc(m, n) :
m = nq1 + r1 (r1 < n)
n = rq1 2 + r2 (r2 < r1)

r1 = r2q3 + r3 (r3 < r2 )


..........................................
rn−2 = rn−1qn + rr (rn < rn−1)
rn−1 = rnqn+1 (onde rn = d )

EUREKA! N°20, 2004

56
Sociedade Brasileira de Matemática

o uso repetido do resultado anterior a cada uma das igualdades anteriores nos
levará a concluir que:
mdc( Fm , Fn ) = mdc( Frn−1 , Fd )
Como d | rn −1 , em virtude da premissa III, Fd | Frn−1 , logo: mdc( Fm , Fn ) = Fd , cqd.

93) Um inteiro positivo n é dito perfeito se n é igual à soma dos divisores


positivos de n que são menores que n. Prove que um número par n é perfeito
se e somente se existe um número primo p ≥ 2 tal que 2p −1 é primo e
n = 2 p−1 (2 p −1) .

SOLUÇÃO DE CARLOS ALBERTO DA SILVA VICTOR (NILÓPOLIS - RJ)


(1) Lema: Se 2k −1 é um número primo então k também é primo.
Prova: Suponha que k não seja primo, logo podemos escrever k = x1 ⋅ x2 onde x1 e
x2 são maiores que 1.
2k −1 = 2x1⋅x2 −1 = (2x1 −1)(2x1 (x2 −1) + 2x1 (x2 −2) + ... + 2x1 +1) , ou seja, 2k − 1 não é
primo. Logo k deverá ser necessariamente primo.

(2) Suponha n = 2 p −1 ⋅ (2 p − 1) com p primo ( p ≥ 2).


Seja S(n) a soma dos divisores de n menores que n:
S (n) = (1 + 2 + 22 + ... + 2 p −1 ) ⋅ (1 + 2 p − 1) − 2 p −1 ⋅ (2 p − 1)
S (n) = (2 p − 1) ⋅ 2 p − 2 p−1 ⋅ (2 p − 1) = 2 p−1 ⋅ (2 p+1 − 2 − 2 p + 1)
S (n) = 2 p −1 ⋅ (2 p − 1) = n.

(3) Suponha S (n) = n (n par). Vamos mostrar que existe um primo (2 p − 1) tal
que n = 2 p −1 ⋅ (2 p − 1).
Prova: Já que n é par podemos escrever na forma n = 2t ⋅ m , onde m é um número
inteiro positivo ímpar.
(
Já que n é perfeiro, temos que 2n = 1 + 2 + 22 + ... + 2t ⋅ ) ( ∑ (m ) ) , onde

∑ (m) é a soma dos divisores do ímpar m;


(2 − 1) ⋅ ∑ (m) = 2n = 2 ⋅ m . Como 2
t +1 t +1 t +1
− 1 é ímpar temos que (2t +1 − 1)m,
ou seja, m = (2t +1 − 1) ⋅ s onde s é um inteiro.

EUREKA! N°20, 2004

57
Sociedade Brasileira de Matemática

Suponha que s > 1, então 1, s e (2t +1 − 1) ⋅ s são divisores de m, e logo:

∑ (m) ≥ 1 + s + (2 − 1) ⋅ s > 2
t +1 t +1
⋅s e
(2 − 1) ⋅ ∑ (m) > (2 − 1) ⋅ 2
t +1 t +1 t +1
⋅ s = 2t +1 ⋅ m = 2n ,
o que é uma contradição, donde s = 1 e m = 2t +1 − 1 . Observe que
∑ (m ) = 2t +1 = 2

t +1

m
− 1 + 1 , ou seja, a soma dos divisores de m é a soma do

próprio m com a unidade, daí m = 2t +1 − 1 é primo, ou seja n = 2 p −1 ⋅ (2 p − 1)


para t + 1 = p e, pelo lema (1), concluímos que existe p primo como nas condições
do enunciado.

94) A ilha das amazonas é habitada por amazonas e homens.


As amazonas mandam em tudo, são inteligentíssimas, ciumentíssimas e muito
fofoqueiras. O que uma amazona mais gosta de fazer é trair outra amazona com o
marido desta. Consumada a traição, ela conta o seu feito a todas as amazonas da
ilha menos à amazona traída. As outras amazonas também não contam nada à
vítima da traição. Mas se uma amazona descobre que está sendo traída ela mata o
seu marido na próxima meia noite.
A rainha das amazonas, que é viúva, vê esta situação com desagrado. Ela vê que
há traição na ilha mas, como nunca ninguém descobre nada, nenhum marido
morre. No dia 1 de janeiro de 3333, então, contrariando a tradição, ela chama
todas as amazonas para a praça central e faz uma proclamação solene: "Há traição
nesta ilha."
Nenhuma amazona sonha em duvidar da palavra da rainha e todas as amazonas
sabem disso. Como já foi dito, todas são inteligentes e ciumentas: estes e os outros
fatos mencionados neste enunciado até aqui são conhecimento comum entre as
amazonas.
Supondo que haja 1000 amazonas na ilha e que 365 delas tenham sido traídas, o
que acontecerá?

SOLUÇÃO DE EDUARDO FISCHER (ENCANTADO - RS)


Se houvesse somente uma traição, a traída não saberia de nada, e como havia pelo
menos uma traição que ela não soubesse, mataria o seu marido na primeira noite.
Como na primeira noite ninguém morreu, uma mulher que soubesse de uma única
traição mataria seu marido na segunda noite, pois, como não houve morte na
primeira noite, havia algo que ela não sabia. Assim, se houver exatamente duas
traições, as traídas matarão seus maridos na seguinte noite.
...

EUREKA! N°20, 2004

58
Sociedade Brasileira de Matemática

Supondo que na (n –1)-ésima noite ninguém morreu, uma mulher que soubesse de
apenas (n –1) traições mataria seu marido na n-ésima noite, pois, como não houve
morte na (n –1)-ésima noite, havia algo que ela não sabia. Mostramos assim, por
indução em n, que, se houver exatamente n traições (i.e., n maridos traidores) as
traídas matarão seus maridos na n-ésima noite.
Lembrando que cada traída sabe de 364 traições, cada uma mataria o seu marido
depois de uma ano, no 365° dia (isto é, no reveillon de 3334).

Enviaram soluções de problemas os seguintes leitores da EUREKA!

Besaleel Ferreira de Assunção Júnior Teresina – PI


Carlos Augusto David Ribeiro Fortaleza – CE
Georges Cobiniano Sousa de Melo João Pessoa – PB
Glauber Moreno Barbosa Rio de Janeiro – RJ
Guilherme Marques dos Santos Silva Enviado via e-mail
José Renato Carneiro e Carneiro Ribeirão Preto – SP
Marcos Francisco Ferreira Martinelli Rio de Janeiro – RJ
Raphael Rodrigues Mata Salvador – BA
Wallace Alves Martins Rio de Janeiro – RJ
Wellington Klezewsky Pires Aquidauana – MS
Zózimo Pereira Campina Grande – PB

Continuamos aguardando soluções dos problemas 89, 90 e 95, propostos na Eureka! No. 18.

EUREKA! N°20, 2004

59
Sociedade Brasileira de Matemática

PROBLEMAS PROPOSTOS
Convidamos o leitor a enviar soluções dos problemas propostos e sugestões de novos
problemas para os próximos números.

96) No quadrilátero ABCD os ângulos A, C e D medem 100° e o ângulo ACB


mede 40°. Demonstre que BC ⋅ DA = ( BC + AB − DA)2 .

97) Seja p um primo ímpar. Encontre todas as funções f : ] → ] que satisfazem


as seguintes condições:

i) Se m ≡ n(mod p) então f (m) = f (n).


ii) f (mn) = f (m) ⋅ f (n) para quaisquer m, n∈ ]

98) Seja (an ) n∈< uma seqüência tal que a1 > 2 e an +1 = an2 − 2, ∀n ∈ `.

1 a1 − a12 − 4
Mostre que ∑
n =1 a1 ⋅ a2 ⋅ ... ⋅ an
=
2
.

99) Num triângulo, a razão entre os raios das circunferências circunscrita e inscrita
5
é . Os lados do triângulo estão em progressão aritmética e sua área é
2
numericamente igual ao seu perímetro. Determine os lados do triângulo.

100) a) Um conjunto X ⊂ ` é dito impressionante se existe m ∈ ` tal que,


para todo k ∈ ` , existem elementos de X, a1 < a2 < ... < ak , tais que
a j +1 − a j ≤ m, ∀j < k .
Determine se é possível particionar ` em um número finito de conjuntos,
nenhum deles impressionante.

b) Determine se é possível particionar ` em dois conjuntos A e B de modo


que nem A nem B contêm progressões aritméticas infinitas mas, para cada
q ∈ `, A e B contêm progressões aritméticas de q termos.

EUREKA! N°20, 2004

60
Sociedade Brasileira de Matemática

101) a) Sejam ai , bi , ci reais positivos, para 1 ≤ i ≤ 3.


Prove que (a13 + a23 + a33 )(b13 + b23 + b33 )(c13 + c23 + c33 ) ≥ (a1b1c1 + a2b2c2 + a3b3c3 )3 .
b) Sejam a, b, c, x, y, z reais positivos. Prove que
a 3 b 3 c 3 ( a + b + c )3
+ + ≥ .
x 2 y 2 z 2 ( x + y + z)2

Problema 96 proposto por Miguel Cruz (Holguin, Cuba), problema 97 (Coréia 2001) proposto
por Samuel Barbosa Feitosa (Fortaleza - CE), problema 98 proposto por Gleydson Chaves
Ricarte (Fortaleza - CE), problema 99 proposto por Geraldo Perlino (Itapecerica da Serra -
SP), problema 100 proposto por Anderson Torres (São Paulo - SP), problema 101 proposto
por Okakamo Matsubashi (São Paulo - SP).

EUREKA! N°20, 2004

61
Sociedade Brasileira de Matemática

COORDENADORES REGIONAIS

Alberto Hassen Raad (UFJF) Juiz de Fora – MG


Amarísio da Silva Araújo (UFV) Viçosa – MG
Ana Paula Bernardi da Silva (Universidade Católica de Brasília) Brasília – DF
Antonio Carlos Nogueira (UFU) Uberlândia – MG
Benedito Tadeu Vasconcelos Freire (UFRN) Natal – RN
Carlos Frederico Borges Palmeira (PUC-Rio) Rio de Janeiro – RJ
Claus Haetinger (UNIVATES) Lajeado – RS
Cleonor Crescêncio das Neves (UTAM) Manaus – AM
Élio Mega (Colégio Etapa) São Paulo – SP
Florêncio Ferreira Guimarães Filho (UFES) Vitória – ES
Ronaldo Alves Garcia (UFGO) Goiânia – GO
Reginaldo de Lima Pereira (Escola Técnica Federal de Roraima) Boa Vista – RR
Ivanilde Fernandes Saad (UC. Dom Bosco) Campo Grande– MS
Jacqueline Fabiola Rojas Arancibia (UFPB) João Pessoa – PB
Janice T. Reichert (UNOCHAPECÓ) Chapecó – SC
João Benício de Melo Neto (UFPI) Teresina – PI
João Francisco Melo Libonati (Grupo Educacional Ideal) Belém – PA
José Carlos dos Santos Rodrigues (Unespar) Campo Mourão – PR
José Cloves Saraiva (UFMA) São Luis – MA
José Gaspar Ruas Filho (ICMC-USP) São Carlos – SP
José Luiz Rosas Pinho (UFSC) Florianópolis – SC
José Vieira Alves (UFPB) Campina Grande – PB
Krerley Oliveira (UFAL) Maceió – AL
Licio Hernandes Bezerra (UFSC) Florianópolis – SC
Luzinalva Miranda de Amorim (UFBA) Salvador – BA
Mário Rocha Retamoso (UFRG) Rio Grande – RS
Marcelo Rufino de Oliveira (Grupo Educacional Ideal) Belém – PA
Marcelo Mendes (Colégio Farias Brito, Pré-vestibular) Fortaleza – CE
Pablo Rodrigo Ganassim (Liceu Terras do Engenho) Piracicaba – SP
Ramón Mendoza (UFPE) Recife – PE
Raúl Cintra de Negreiros Ribeiro (Colégio Anglo) Atibaia – SP
Reinaldo Gen Ichiro Arakaki (INPE) SJ dos Campos – SP
Ricardo Amorim (Centro Educacional Logos) Nova Iguaçu – RJ
Sérgio Cláudio Ramos (IM-UFRGS) Porto Alegre – RS
Tadeu Ferreira Gomes (UEBA) Juazeiro – BA
Tomás Menéndez Rodrigues (U. Federal de Rondônia) Porto Velho – RO
Valdenberg Araújo da Silva (U. Federal de Sergipe) São Cristovão – SE
Valdeni Soliani Franco (U. Estadual de Maringá) Maringá – PR
Vânia Cristina Silva Rodrigues (U. Metodista de SP) S.B. do Campo – SP
Wagner Pereira Lopes (CEFET – GO) Jataí – GO

EUREKA! N°20, 2004

62
CONTEÚDO

AOS LEITORES 2

XV OLIMPÍADA DE MATEMÁTICA DO CONE SUL 3


Enunciados e Soluções

XLV OLIMPÍADA INTERNACIONAL DE MATEMÁTICA 13


Enunciados e Soluções

ARTIGOS

CENTRO DE MASSA E APLICAÇÕES À GEOMETRIA 29


Emanuel Carneiro & Frederico Girão

SEQÜÊNCIA DE FIBONACCI 38
Cícero Thiago B. Magalhães

COMO É QUE FAZ? 43

SOLUÇÕES DE PROBLEMAS PROPOSTOS 46

PROBLEMAS PROPOSTOS 60

COORDENADORES REGIONAIS 62
Sociedade Brasileira de Matemática

AOS LEITORES

Chegamos a este número 21 com dois artigos: um de geometria e outro


sobre a seqüência de Fibonacci. São as primeiras publicações dos respectivos
autores na Eureka!. Esperamos que venham outras e que cada vez mais autores
contribuam com a revista. Apresentamos as soluções da Olimpíada do Cone Sul e da
Olimpíada Internacional de 2004, competições nas quais as equipes brasileiras
tiveram muito bom desempenho. Agradecemos mais uma vez as contribuições dos
leitores para a seção "Como é que faz?" e para a seção dos problemas propostos,
com soluções e novos problemas, que fazem da Eureka! cada vez mais uma obra de
criação coletiva.

Aproveitamos para registrar que foi criada em 2004 a Associação Olimpíada


Brasileira de Matemática – AOBM, uma pessoa jurídica destinada a ajudar as
Olimpíadas de Matemática no Brasil a crescerem e se consolidarem.

Esperamos que a AOBM sirva como instrumento para maior integração e


organização da comunidade olímpica. Estimulamos nossos leitores a se associarem à
AOBM, cujos sócios recebem gratuitamente a revista Eureka!. As informações
sobre como se associar à AOBM podem ser encontradas na nossa página na internet:
www.obm.org.br

Os editores

EUREKA! N°21, 2005

2
Sociedade Brasileira de Matemática

XV OLIMPÍADA DE MATEMÁTICA DO CONE SUL


Enunciados e Soluções
PROBLEMA 1
Maxi escolheu 3 dígitos e, fazendo todas as permutações possíveis, obteve 6
números distintos, cada um com 3 dígitos. Se exatamente um dos números que Maxi
obteve é um quadrado perfeito e exatamente três são primos, encontrar os 3 dígitos
que Maxi escolheu.
Dê todas as possibilidades para os 3 dígitos.

SOLUÇÃO DE LEANDRO FARIAS MAIA (FORTALEZA – CE)


Sejam x1 , x2 , x3 os dígitos escolhidos por Maxi. Note que x1 ≠ x2 , x1 ≠ x3 , x2 ≠ x3
pois as 6 reordenações: x1 x2 x3 ,..., x3 x2 x1 são distintas.
Agora faremos a lista de todos os números de 3 dígitos quadrados perfeitos:
10 2 = 100 202 = 400
302 = 900
11 = 121 21 = 441
2 2
312 = 961
12 = 144 22 = 484
2 2

132 = 169 232 = 529


14 2 = 196 242 = 576
152 = 225 252 = 625
16 2 = 256 262 = 676
17 2 = 289 27 2 = 729
182 = 324 282 = 784
19 2 = 361 292 = 841
Perceba que:
• Os números que têm algum zero não satisfazem o enunciado: 10 2 , 20 2 ,30 2.
• Reordenando 1, 6, 9 podemos obter 132 ,14 2 ,312. Assim, 132 ,14 2 ,312
não satisfazem o enunciado.
• O número deverá apresentar no mínimo 2 dígitos ímpares, pois se tiver no máximo
um, teremos (se tivermos) no máximo 2 números primos. Assim;
16 2 ,17 2 ,182 , 252 , 28 2 , 29 2 não satisfazem o enunciado.
• Os dígitos são distintos. Assim, 112 ,12 2 ,152 , 212 , 22 2 , 26 2
não satisfazem o enunciado.

EUREKA! N°21, 2005

3
Sociedade Brasileira de Matemática

Nos restam os números: 19 2 , 232 , 24 2 , 27 2. Reordenando:


19 2 = 361;136,163,316,361,613, 631.
232 = 529; 259, 295,529,592,925,952.
24 2 = 576;567,576, 657,675,756,765.
27 2 = 729; 279, 297,729, 792,927,972.
Perceba que os dígitos:
• 1, 3, 6 satisfazem o enunciado, pois, 163, 613, 631 (apenas) são primos e 361
(apenas) é quadrado perfeito.
• 2, 5, 9 não satisfazem o enunciado, pois, 295, 592, 925, 952 são compostos e 529
(apenas) é quadrado perfeito, assim teremos no máximo um primo.
• 5, 6, 7 não satisfazem o enunciado pelo mesmo raciocínio acima: 675, 765, 756 e
576 são compostos.
• 2, 7, 9 não satisfazem o enunciado pois todas as suas reordenações são múltiplos
de 9.
Portanto, os dígitos que Maxi escolheu foram 1, 3, 6.

PROBLEMA 2
Dada uma circunferência C e um ponto P exterior a ela, traçam-se por P as duas
tangentes à circunferência, sendo A e B os pontos de tangência.
Toma-se um ponto Q sobre o menor arco AB de C. Seja M a interseção da reta AQ
com a perpendicular a AQ traçada por P, e seja N a interseção da reta BQ com a
perpendicular a BQ traçada por P.
Demonstre que, ao variar Q no arco AB, todas as retas MN passam por um mesmo
ponto.

SOLUÇÃO DE LEANDRO FARIAS MAIA (FORTALEZA – CE)


Sejam α = (QBA, β = (QAB e "D" um ponto sobre AB de modo que PD ⊥ AB.
Como AP é tangente a "C", então: (QBA = (QAP = α. Analogamente:
(PBQ = β.
Agora veja que:
i) PNDB é inscritível, pois
(PNB = 90° = (PDB. Assim;
(NPD = (NBD = α e (NDP = (NBP = β.

EUREKA! N°21, 2005

4
Sociedade Brasileira de Matemática

αβ

N Q

α α
β β
A β α B
D

ii) PMDA é inscritível, pois (PMA = 90° = (PDA.


Assim; (PDM = (PAM = α e (MPD = (MAD = β. Portanto, de i) e ii) temos
que
• (NPD = (MDP = α ⇒ NP // MD.
• (NDP = (MPD = β ⇒ PM // ND.

F
Logo, PMDN é paralelogramo, e então NM e PD se cruzam no ponto médio (*),
assim para qualquer "Q" e AB (menor), MN passará por um ponto fixo que é o
ponto médio da altura PD.
Prova de (*):
A B
α θ
β

β
θ α
D C

AB // DC ⇒ (ACD = (CAB, (ABD = (BDC.


BC // AD ⇒ (ADB = (CBD.

EUREKA! N°21, 2005

5
Sociedade Brasileira de Matemática

Assim, ∆ACD ≡ ∆ACB( LAAO ) ⇒ DC = AB e AD = BC.

∆MDC ≡ ∆ABM ( ALA) ⇒ AM = MC e BM = MD.

PROBLEMA 3
Seja n um inteiro positivo. Chamamos Cn a quantidade de inteiros positivos x,
menores que 10n, tais que a soma dos dígitos de 2x é menor que a soma dos dígitos
de x.

Demonstre que Cn •
9
(10 − 1) .
4 n

SOLUÇÃO:
k k
Se m = ∑ a j ⋅ 10 , com a j ∈{0,1, 2,...,9}, ∀j ≤ k , temos 2m = ∑ (2a j ) ⋅10 . Note
j j

j=0 j =0

que, se a ∈ {0,1, 2,3, 4}, 2a ∈{0, 2, 4,6,8} tem apenas um dígito, e, se


a ∈ {5,6,7,8,9}, 2a ∈{10,12,14,16,18} tem dois dígitos, sendo o primeiro deles
k
igual a 1. Assim, na soma ∑ (2a ) ⋅10
j =0
j
j
, não há "vai um", pois, se (2ai ) ⋅10i e

(2a j ) ⋅10 têm dígitos não nulos na k-ésima, posição, com i < j, então k = j = i + 1,
j

sendo o dígito de (2ai ) ⋅10i igual a 1, nessa k-ésima posição, donde sua soma é
menor que 10 (pois o dígito corresponde de (2a j ) ⋅10 j é no máximo 8). Portanto, se
k k
s(m) denota a soma dos dígitos de m, s (m) = ∑ a j e s (2m) = ∑ s (2a j ). Agora, para
j =0 j =0

a ∈ {0,1, 2,3, 4}, s (2a ) = 2a, e, para a ∈ {5, 6, 7,8,9} , s (2a ) = 2a − 9. Portanto, para
0 ≤ a ≤ 9, s (2a ) + s (2 ⋅ (9 − a )) = 2a + 2(9 − a ) − 9 = 9 = a + (9 − a ). Assim, se
n −1
x = ∑ a j ⋅ 10 j , (com 0 ≤ a j ≤ 9, ∀j ) é um inteiro positivo menor que 10 n , temos
j=0
n −1
(10n − 1) − x = ∑ (9 − a j ) ⋅10 j , e logo, s (2 x ) + s (2(10n − 1 − x )) =
j =0

n −1 n −1 n −1 n −1
= ∑ s (2a j ) + ∑ s (2 ⋅ (9 − a j )) = ∑ ( s (2a j ) + s (2 ⋅ (9 − a j )) = ∑ (a j + (9 − a j )) =
j =0 j =0 j =0 j =0

EUREKA! N°21, 2005

6
Sociedade Brasileira de Matemática

n −1 n −1
= ∑ a j + ∑ (9 − a j ) = s ( x ) + s ((10n − 1) − x ). Assim, s (2 x ) < s ( x ) se e somente se
j =0 j =0

s (2(10 − 1 − x )) > s (10n − 1 − x ), e s (2 x ) = s ( x ) se e somente se


n

10n − 1 − A
s (2(10n − 1 − x )) = s (10n − 1 − x ). Portanto, Cn = , onde
2
A = #{1 ≤ x < 10n | s(2 x) = s( x)}. Note agora que s ( x ) ≡ x (mod 9) e
s (2 x ) ≡ 2 x (mod 9) , donde, se s (2 x ) = s ( x ), então 2 x ≡ x (mod 9) , e logo
10n − 1
x ≡ 0(mod 9). Assim, A ≤ #{1 ≤ x < 10 n | x ≡ 0(mod 9)} = , donde
9
4 n
Cn ≥ (10 − 1).
9

PROBLEMA 4
Arnaldo escolhe um inteiro a, a •  H %HUQDOGR HVFROKH XP LQWHLUR b, b •  $PERV
dizem, em segredo, o número que escolheram a Cernaldo, e este escreve em um
quadro os números 5, 8 e 15, sendo um desses a soma a + b.
Cernaldo toca uma campainha e Arnaldo e Bernaldo, individualmente, escrevem em
papéis distintos se sabem ou não qual dos números no quadro é a soma de a e b, e
entregam seus papéis para Cernaldo.
Se em ambos os papéis está escrito NÃO, Cernaldo toca novamente a campainha, e
o procedimento se repete.
Sabe-se que Arnaldo e Bernaldo são sinceros e inteligentes.
Qual é o número máximo de vezes que a campainha pode ser tocada até que um
deles escreva que sabe o valor da soma?

SOLUÇÃO DE TELMO LUIS CORREA JÚNIOR (SANTO ANDRÉ – SP)


Se a ≥ 9, a campainha toca apenas 1 vez: Arnaldo sabe que b ≥ 0, logo a única soma
possível é 15. O mesmo ocorre se b ≥ 9, com Bernaldo.
Caso contrário, a campainha toca 2 vezes: com a, b ≤ 8 não é possível decidir entre
as somas dadas na primeira vez.
Se a = 6, Arnaldo agora sabe que b ≤ 8, então a soma não pode ser 5 ou 15, apenas
8. O mesmo ocorre se b = 6. Se a ∈{0,1, 2,3,4,5,7,8}, ainda não é possível decidir
entre 5, 8 e 15. O mesmo ocorre para Bernaldo.
Caso a campainha toque pela terceira vez a, b ∈{0,1,2,3,4,5,7,8} .

EUREKA! N°21, 2005

7
Sociedade Brasileira de Matemática

Se a = 2, Arnaldo sabe que b ≠ 6 e b ≤ 8, logo a única soma possível é 5. O mesmo


ocorre se b = 2. Caso a ≠ 2 e b ≠ 2 , ninguém escreve o SIM. De fato:
Se a ∈{0,1,3, 4,5}, Arnaldo não pode decidir entre as somas 5 e 8.
Se a ∈{7,8}, Arnaldo não pode decidir entre as somas 8 e 15. O mesmo ocorre para
Bernaldo.
A campainha toca pela quarta vez, e ambos sabem que a, b ∈{0,1,3, 4,5,7,8}.
Se a = 3, Arnaldo sabe que b ≠ 2, logo a soma deve ser 8. Se a ∈{0,1, 4,5} não é
possível decidir entre 5 e 8, se a ∈{7,8} , não é possível decidir entre 8 e 15. O
mesmo ocorre com Bernaldo.
A campainha toca pela quinta vez: ambos sabem que a, b ∈{0,1,4,5,7,8} .
Se a = 5, Arnaldo sabe que b ≠ 3 , logo a soma deve ser 5. Se a ∈{0,1, 4} não é
possível decidir entre 5 e 8, se a ∈{7,8} não é possível decidir entre 8 e 15.
O mesmo para Bernaldo.
A campainha toca: sexta vez - ambos sabem que a, b ∈{0,1, 4,7,8}.
Se a = 0, Arnaldo sabe que b ≠ 5 , logo a soma deve ser 8. Se a ∈{1, 4}, não é
possível decidir entre 5 e 8, se a ∈{7,8}, entre 8 e 15.
O mesmo para Bernaldo.
Sétima vez: ambos sabem que a, b ∈{1,4,7,8}. Se a = 8, Arnaldo sabe que a soma
deve ser 15, se a ∈{1, 4}, não pode decidir entre 5 e 8, se a = 7, não pode decidir
entre 8 e 15. O mesmo com Bernaldo.
Oitava vez: ambos sabem que a, b ∈{1,4,7}. Se a = 7, Arnaldo sabe que a soma
deve ser 8, se a ∈{1, 4}, não pode decidir entre 5 e 8.
O mesmo para Bernaldo.
Nona vez: ambos sabem que a, b ∈{1,4}. Arnaldo sabe que única soma possível se
a = 1 é 5;
se a = 4, a soma pode ser 5 ou 8. O mesmo para Bernaldo.
A campainha toca pela décima vez: agora ambos sabem que a = b = 4, e determinam
a soma com segurança. Logo, a campainha pode ser tocada no máximo dez vezes.

PROBLEMA 5
Utilizando triangulinhos eqüiláteros de papel, de lado 1, forma-se um triângulo
eqüilátero de lado 2 2004 . Desse triângulo retira-se o triangulinho de lado 1 cujo
centro coincide com o centro do triângulo maior.
Determine se é possível cobrir totalmente a superfície restante, sem superposições
nem buracos, dispondo-se somente de fichas em forma de trapézio isósceles, cada
uma formada por três triangulinhos eqüiláteros de lado 1.

EUREKA! N°21, 2005

8
Sociedade Brasileira de Matemática

SOLUÇÃO DE GABRIEL TAVARES BUJOKAS (SÃO PAULO – SP)

Vamos mostrar por indução que qualquer n ≡ 1 (mod 9) é possível cobrir o triângulo
dividido em n partes cada lado. Como 22004 ≡ (+1) (mod 9) já que 26 n ≡ 1 e 6 | 2004,
26 n ≡ 1 (mod 9) e 6 | 2004 para n = 22004 é possível.
Vamos mostrar que se para n ≡ 1 (mod 9) é possível, então para n + 9 também é.
Temos que n + 9 ≡ 1 (mod 9). Cobriremos os 3 lados do triângulo maior assim:
Comece por um vértice A e cubra um lado a que este pertença com peças até
restarem 2 espaços (como n ≡ 1 (mod 3), isto é possível). Agora preencha da mesma
forma a segunda linha, até restarem 4 espaços.
Na terceira linha, deixe um espaço à esquerda e preencha até onde foi a segunda
linha (como indicado na figura acima). E, acima das primeiras duas peças da terceira
linha, ponha mais uma (como na figura).
Faça isso para todos os vértices. Observe que não há nem espaços nem
superposições. Assim criamos uma "barba" e o meio é um triângulo de lado
(n + 9) – 9 (muito fácil de ver, já que a borda tem "largura" 3). O centro do triângulo
grande coincide com o do de lado n. Assim, por hipótese de indução, este triângulo
central pode ser preenchido, logo o triângulo grande também!. A base da indução é
n =1, caso em que não há nada a fazer. Assim, nossa afirmação está provada.

EUREKA! N°21, 2005

9
Sociedade Brasileira de Matemática

PROBLEMA 6
Sejam m, n inteiros positivos. Em um tabuleiro m × n, quadriculado em
quadradinhos de lado 1, considere todos os caminhos que vão do vértice superior
direito ao inferior esquerdo, percorrendo as linhas do quadriculado exclusivamente
nas direções ← e ↓.
Define-se a área de um caminho como sendo a quantidade de quadradinhos do
tabuleiro que há abaixo desse caminho. Seja p um primo tal que rp(m) + rp(n) ≥ p,
onde rp(m) representa o resto da divisão de m por p e rp(n) representa o resto da
divisão de n por p.
Em quantos caminhos a área é um múltiplo de p?

SOLUÇÃO:
Para resolver este problema, usaremos técnicas de funções geratrizes descritas no
artigo "Séries formais", de Eduardo Tengan, da Eureka! No. 11, pp. 34 – 39.
Representaremos o tabuleiro por {1, 2,…,m} × {1, 2,…,n}.
Desta forma, os quadradinhos são representados por pares (i, j), com
1 ≤ i ≤ m,1 ≤ j ≤ n. Considere agora um caminho como no enunciado. Para
1 ≤ i ≤ m, seja bi o número de quadradinhos da forma (i, y) que estão abaixo do
caminho. Como o caminho só desce ou anda para a esquerda, temos
0 ≤ b1 ≤ b2 ≤ b3 ≤ ... ≤ bm (e, reciprocamente, tais bi determinam um caminho). A
m
área do caminho é ∑b .
i =1
i Definamos bm +1 = n e b0 = 0. Para 1 ≤ i ≤ m + 1, seja
m +1
yi = bi − bi −1 . Temos ∑y
i =1
i = bn +1 − b0 = n , e, em geral, para
k
k ≤ m + 1, ∑y
i =1
i = bk − b0 = bk . Portanto, a área do caminho é
m m m+1 m
A = ∑bi = ∑(m + 1− i) ⋅ yi = ∑(m + 1− i) ⋅ yi = ∑ j ⋅ ym+1− j .
i =1 i =1 i =1 j =0
m m
Assim, fazendo z j = ym +1− j , temos ∑ zj = n e
j =0
∑ j⋅z
j =0
j = A.

Assim, o problema é equivalente a achar o número de soluções de


m
z0 + z1 + ... + zm = n, com zi ≥ 0 para 0 ≤ i ≤ m + 1 e ∑ j⋅z
j =0
j
múltiplo de p. Para
m
1 1 1 1
isso, considere a função F ( x, y ) = ∏ = ⋅ ⋅ ... ⋅ . Como
k =0 1 − x y 1 − y 1 − xy 1 − xm y
k

EUREKA! N°21, 2005

10
Sociedade Brasileira de Matemática

1
= 1 + x k y + ( x k y ) 2 + ..., F ( x, y ) = ∑ ar , s x r y s , onde ar , s é o número de
1− x y
k
r , s∈<
m
soluções de z0 + z1 + ... + zm = s com zi ≥ 0, ∀i ≥ 0 e zi + 2 z2 + 3 z3 + ... = ∑ jz j = r.
j =0

Em particular, ar ,n é o número de caminhos como no enunciado com área r.


Queremos então calcular ∑a
p|r
r ,n , i.e., a soma dos ar ,n para os r múltiplos de p.

p −1
Para isso, consideremos a soma ∑ F (ξ
k =0
k
, y ), onde ξ = e 2π i p
é uma raiz p-ésima da
p −1
unidade (e logo ∑ξ
k =0
kr
= 0, sempre que r não é múltiplo de p). Temos
p −1 p −1
 p −1 
∑ F (ξ k
, y) = ∑ ∑< a r ,s ⋅ ξ kr ⋅ y s = ∑< a r ,s ⋅  ∑ ξ kr  ⋅ y s = ∑ p ⋅ ar ,s ⋅ y s (pois, se
k =0 k =0 r , s∈ r , s∈  k =0  r , s∈<
p|r

p −1 p −1
p | r, ∑ξ kr = p ) . Em particular, o coeficiente de y n em ∑ F (ξ k
, y ) é p vezes o
k =0 k =0
número de caminhos como no enunciado cuja área é um múltiplo de p.

Para k = 0, F (ξ k , y ) = F (1, y ) =
1
m +1
= (1 + y + y 2 + ...)m +1 , e o coeficiente de y n
(1 − y )
em F (1, y ) é o número de soluções de k1 + k2 + ... + km +1 = n , ki ≥ 0, ∀i , que é
m+n m
1 m
 . Por outro lado, para 1 ≤ k ≤ p − 1, F ( ξ k
, y ) = ∏
1
= m+1 ∏ −1
1
.
r =0 1 − ξ y y r =0 y − ξ kr
kr
 m
Escrevendo m = qp + rp ( m) = ( q + 1) p − ( p − rp (m)), com 0 ≤ rp (m) < p, temos
( q +1) p −1 ( q +1) p −1 ( q +1) p −1

∏ (1 − ξ y ) kr
∏ (1 − ξ kr y ) ∏ (1 − ξ y ) kr

F (ξ , y ) =
1 1
k
⋅ ( q +r =1)mp+−11 = r =m +1
= r =m +1
.
y (q +1) p (1 − y )
−p
y (q + 1) p − 1) q +1 +
p q 1
∏ (y −ξ−1 kr
) (y
r =0
( q +1) p −1
Como o produto ∏ (1 − ξ y ) kr
tem p − rp (m) − 1 termos, ao desenvolver esse
r = m+1

produto, obtemos termos da forma a ⋅ y s , com 0 ≤ s ≤ p − rp ( m) − 1. Por outro lado,

= (1 + y p + y 2 p + ...) têm expoente múltiplo


1 q +1
todas as potências de y em p q +1
(1 − y )

EUREKA! N°21, 2005

11
Sociedade Brasileira de Matemática

de p, donde os termos não-nulos em F (ξ k , y ) têm o expoente de y congruente a s


módulo p, para algum s com 0 ≤ s ≤ p − rp ( m) − 1. Entretanto, rp ( m) + rp ( n) ≥ p,
donde rp ( n) > p − rp ( m) − 1 , e, como n é congruente a rp ( n) módulo p, nenhum
termo não nulo em F (ξ k , y ) tem expoente n. Assim, o número procurado de
1 m + n
caminhos cuja área é um múltiplo de p é  .
p m 

9RFr VDELD«
4XH 225964951 − 1 p SULPR" (VWH Q~PHUR GH  GtJLWRV p R
PDLRU SULPR FRQKHFLGR H p XP SULPR GH 0HUVHQQH DVVLP FRPR R
VHJXQGR R WHUFHLUR H R TXDUWR PDLRUHV SULPRV FRQKHFLGRV TXH VmR
2 24036583 − 1, 220996011 − 1 H 213466917 − 1.
2 GHVFREULGRU GHVWH QRYR SULPR R   SULPR GH 0HUVHQQH
4

FRQKHFLGR  'U 0DUWLQ 1RZDN p XP RIWDOPRORJLVWD DOHPmR TXH


SDUWLFLSD GR *,036 XP SURMHWR FRRSHUDWLYR SDUD SURFXUDU SULPRV
GH 0HUVHQQH UHVSRQViYHO SHOD GHVFREHUWD GRV RLWR PDLRUHV SULPRV
GH 0HUVHQQH DWXDOPHQWH FRQKHFLGRV
9HMD KWWSZZZPHUVHQQHRUJ SDUD PDLRUHV LQIRUPDo}HV

EUREKA! N°21, 2005

12
Sociedade Brasileira de Matemática

XLV OLIMPÍADA INTERNACIONAL DE MATEMÁTICA


Enunciados e Soluções

Caros leitores: nossa equipe da IMO-2004 tinha seis alunos muito


diferentes, mas com uma característica em comum: garra. Todos eles adotaram a
tática de escrever tudo o que passava pelas suas cabeças, de modo a maximizar as
suas pontuações. Assim, não podemos transcrever as suas soluções ipsis literis. Ao
contrário, vamos colocar somente partes das soluções deles, com alguns comentários
sobre as suas idéias.
Carlos Yuzo Shine, vice-líder do Brasil na IMO-2004
PRIMEIRO DIA

PROBLEMA 1

n
Seja ABC um triângulo acutângulo com AB ≠ AC. A circunferência de diâmetro BC

n
intersecta os lados AB e AC nos pontos M e N, respectivamente. Seja O o ponto
médio do lado BC. As bissetrizes dos ângulos BAC e MON intersectam-se em R.
Prove que as circunferências circunscritas aos triângulos BMR e CNR têm um ponto
em comum que pertence ao lado BC.

SOLUÇÃO DE THIAGO COSTA LEITE SANTOS (SÃO PAULO - SP)


A

α/2 α/2
N ΓC
M
ϕ γ
ΓB ϕ R
β
π / 2– α
π / 2– α

β π – 2β π – 2γ
γ
B C
K O

EUREKA! N°21, 2005

13
Sociedade Brasileira de Matemática

Seja Γ a circunferência com diâmetro BC, ΓB o circuncírculo de BMR e ΓC o


circuncírculo de CNR. Suponhamos sem perda de generalidade que o circunraio de
ABC mede 1/2, de modo que BC = sen α, AC = sen β e AB = sen γ.

Aqui, Thiago nota (e prova) que o ponto comum às duas circunferências e o lado BC
só pode ser o pé da bissetriz:

Temos potΓ A = AM ⋅ AB = AN ⋅ AC. Mas potΓ B A = AM ⋅ AB e potΓC A = AN ⋅ AC,


logo A pertence ao eixo radical de ΓB e ΓC. Como R também pertence ao eixo
radical, a reta AR é o eixo radical de ΓB e ΓC. Logo o ponto comum a ΓB e ΓC e BC
só pode ser a interseção de AR e BC, ou seja, a interseção da bissetriz de ∠BAC e
BC.

Mais algumas considerações geométricas: como OM = ON (raios de Γ), ∠NOR =


∠MOR (OR é bissetriz de ∠MON) e OR é comum, os triângulos MOR e NOR são
congruentes (caso LAL), portanto ∠RNO = ∠RMO.
Nesse momento, Thiago percebe como poderia terminar o problema: seja K a
interseção da bissetriz de ∠BAC e BC. A idéia dele é provar que K pertence a BC,
ΓB e ΓC. Que pertence a BC é óbvio. Para provar que pertence a ΓB, basta provar que
B, M, R e K são concíclicos; e, de modo análogo, para provar que pertence a ΓC,
basta provar que C, N, R e K são concíclicos.

Aqui está a formalização dele: se provarmos que ∠RNO = α/2, teríamos ∠RNC = γ
+ α/2 e ∠RNC + ∠RKC = γ + α/2 + β + α/2 = π, portanto K pertence a ΓC.
Analogamente (aqui você deve verificar que Thiago usa o fato que ele provou antes
de que ∠RNO = ∠RMO), vamos ter K pertencente a ΓB, portanto ΓB e ΓC têm um
ponto comum em BC.

Deste modo, só falta provar que ∠RNO = α/2. Isso é um problema que costuma ser
resolvido de duas maneiras: (1) com “arrastão” (isto é, calculando todos os ângulos
na figura) ou (2) “arrastão” seguido de contas (isto é, com trigonometria, geometria
analítica ou complexos). Thiago resolveu adotar a opção (2), com trigonometria
(quando o coordenador viu as contas, ele comentou, “estava indo tão bem até aqui,
por que ele teve que fazer essas contas??”)

EUREKA! N°21, 2005

14
Sociedade Brasileira de Matemática

O triângulo NBO é isósceles, logo ∠ONC = ∠OCN = γ e ∠NOC = π – 2γ.


π − (π − 2β) − (π − 2γ )
Analogamente, ∠MOB = π – 2β e θ = ∠MOR = ∠NOR = =
2
π
– α.
2
Seja ∠RNO = ϕ. A idéia é calcular alguma função trigonométrica de ϕ (geralmente
acaba sendo a cotangente, por causa do truque da cotangente – veja o artigo
Geometria com Contas, Eureka! 17) e compará-la com a mesma função
trigonométrica de α/2 para provar que ϕ = α/2.

Mas por onde começar? Primeiro devemos procurar por triângulos que envolvam ϕ.
Um deles vem da própria definição de ϕ: RNO. Outro bem interessante é o ARN,
cujos ângulos envolvem tanto ϕ como α/2, além de ter RN em comum (um
segmento a menos para calcular, como você vai ver depois!). Então parece ser uma
boa estratégia utilizar esses dois triângulos.

Lei dos senos em RNO:


RN NO
= (1)
sen ( 2 − α ) sen ( 2 − α + ϕ )
π π

Lei dos senos em ARN:


RN AN
= (2)
sen 2 sen ( γ + ϕ − α2 )
α

Veja que se dividirmos (1) por (2), RN se cancela, então não precisamos calcular
BC senα
RN. Só precisamos de NO e AN. NO é fácil: é o raio = . Além disso,
2 2
AN = AC – NC = sen β – NC. Só falta NC!

Pela lei dos senos em NOC,


NC OC senαsen2 γ senα ⋅ 2senγ cos γ
= ⇔ NC = = = senα cos γ
sen2 γ sen γ 2 sen γ 2 senγ
logo AN = sen β – sen α cos γ = sen(α + γ) – sen α cos γ = sen α cos γ + sen γ cos α
– sen α cos γ = sen γ cos α.

Vamos usar repetidas vezes o fato de que sen(π/2 – x) = cos x e as fórmulas de


Prostaférese (que os gregos tiveram que deduzir na coordenação! procure-as na

EUREKA! N°21, 2005

15
Sociedade Brasileira de Matemática

Eureka! 17). Agora sim, dividindo (1) por (2) e substituindo NO e AN, obtemos a
nossa conta final:

2 sen ( γ + ϕ − 2 ) sen ( γ + ϕ − α2 ) senγ


senα α
sen α2
⇔ = ⇔ =
cos α senγ cos α ⋅ cos(α − ϕ) cos(α − ϕ) cos α2
⇔ senγ cos(α − ϕ) = sen ( γ + ϕ − α2 ) cos α2 (*)

Parece loucura, mas Thiago abriu os senos e co-senos. Mas isso tinha um motivo:
ele queria achar cotg ϕ (lembre-se: o segredo do sucesso nas contas é sempre ter
uma meta em mente!).

(*) ⇔ cos ϕ cos αsenγ + senϕsen γsenα = senϕ cos ( γ − α2 ) cos α2 + cos ϕsen ( γ − α2 ) cos α2

( ) (
⇔ cos ϕ cos αsenγ − sen ( γ − α2 ) cos α2 = senϕ cos ( γ − α2 ) cos α2 − senγsenα )
⇔ cos ϕ( sen(α + γ ) + sen( γ − α) − senγ − sen( γ − α)) =
senϕ(cos γ + cos( γ − α) − cos( γ − α) + cos( γ + α))
cos γ − cos β 2 sen ( 2 ) sen ( 2 ) sen ( π−α
2 )
β+γ β−γ
α
⇔ cotg ϕ = = = = cotg
senβ − senγ 2sen ( 2 ) cos ( 2 ) cos ( 2 )
β−γ β+γ π−α
2
α α α
Como cotg ϕ = cotg eϕ e pertencem ao intervalo ]0; π[, temos ϕ = ,o
2 2 2
que precisávamos.

PROBLEMA 2
Determine todos os polinômios P(x) de coeficientes reais que satisfazem a igualdade

P( a − b) + P(b − c) + P(c − a ) = 2 P( a + b + c )

para quaisquer números reais a, b, c, tais que ab + bc + ca = 0.

SOLUÇÃO DE FÁBIO DIAS MOREIRA (RIO DE JANEIRO - RJ)


Note inicialmente que
P(a – b) + P(b – c) + P(c – a) = 2P(a + b + c)
P(b – a) + P(a – c) + P(c – b) = 2P(b + a + c) (*)
P(b – a) + P(c – b) + P(a – c) = 2P(– a – b – c) (**)

EUREKA! N°21, 2005

16
Sociedade Brasileira de Matemática

Desde que ab + bc + ca = 0 (a primeira equação é uma aplicação direta; a segunda


segue se permutarmos a e b; a terceira se levarmos (a; b; c) em (– a; – b; – c), o que
é permitido pois (– a)(– b) + (– b)(– c) + (– c)(– a) = ab + bc + ca = 0).

Igualando (*) e (**), P(a + b + c) = P(– a – b – c) desde que ab + bc + ca = 0. Mas


dado S real, se tomarmos a = S, b = 0 e c = 0, então a + b + c = S e ab + bc + ca = 0.
Logo P(x) = P(–x) para todo x real, ou seja, P é par. Em particular, isto implica que
o grau de P, gr(P), é par.

Agora vem a parte da solução que usa análise. Ele provou um lema geral bem útil
para o problema.

Note inicialmente que se P(x) = k para todo x real então 3k = 2k ⇔ k = 0 que, de


fato, é uma solução. Suponha agora que P não é constante, de grau n. Seja Qx(λ) =
P(xλ) (aqui, Fábio usa uma idéia relacionada a homogeneidade). Fixado x, Qx é um
polinômio em λ de grau ≤ n (de fato, de grau n se x ≠ 0 e de grau 0 se x = 0).

Qx (λ)
Lema. lim = Cn x n , sendo Cn o coeficiente líder de P(x).
λ→+∞ λn

Demonstração:
Note que Qx(λ), Qx’(λ), …, Qx(n –1)(λ), λn, λn –1, …, λ são todos polinômios, logo são
deriváveis, e que para um desses polinômios qualquer, digamos R, temos
lim | R(λ) | = +∞ , pois todos eles têm grau maior ou igual a 1. Então, aplicando o
λ→∞

Qx (λ) Qx( n ) (λ)


teorema de L’Hôpital repetidas vezes, temos lim = lim . Mas se P(x)
λ→+∞ λ n λ→+∞ n!
= Cnxn + … + C1x + C0, Qx(λ) = Cnxnλn + … + C1xλ + C0, logo Qx(n)(λ) = n!Cnxn,
logo o limite desejado é mesmo Cnxn.

Agora, Fábio aproveita-se da homogeneidade da condição ab + bc + ca = 0: como


ab + bc + ca = 0 ⇔ (λa)(λb) + (λb)(λc) + (λc)(λa) = 0 para todo λ real, logo
P(λ(a – b)) + P(λ(b – c)) + P(λ(c – a)) = 2P(λ(a + b + c)) ⇔
Qa–b(λ) + Qb–c(λ) + Qc–a(λ) = 2Qa+b+c(λ) para todos λ, a, b, c reais, ab + bc + ca = 0.
Q (λ)
Como cada um dos limites lim x n , x ∈ {a – b; b – c; c – a}, n = gr(P) existe,
λ→+∞ λ

existe o limite da soma, que é a soma dos limites. Logo

EUREKA! N°21, 2005

17
Sociedade Brasileira de Matemática

Qa −b (λ) + Qb− c (λ ) + Qc − a (λ) Q (λ )


lim = 2 lim a +b +nc
λ→∞ λ n λ→∞ λ
⇔ Cn (a − b) + Cn (b − c ) + Cn (c − a ) = 2Cn (a + b + c )n
n n n

sendo a, b, c reais tais que ab + bc + ca = 0 e n = gr(P). Como Cn, coeficiente líder


de P, é não-nulo por hipótese, (a – b)n + (b – c)n + (c – a)n = 2(a + b + c)n (***)
Tome a = – 2, b = 3 e c = 6 (note que ab + bc + ca = 0). Então, substituindo em
(***), (–5)n + (–3)n + 8n = 2 ⋅ 7n
Como n é par, obtemos 5n + 3n + 8n = 2 ⋅ 7n. Se n ≥ 7,
n 7
8  1  1
8n =   7 n ≥ 7 n 1 +  > 7 n 1 + 7 ⋅  = 2 ⋅ 7 n , absurdo.
7  7  7
Logo n ≤ 6. Mas n = 6 implica 0 ≡ 2 ⋅ 76 ≡ 36 + 56 + 86 ≡ 1 + 1 + 1 ≡ 3 (mod. 7),
absurdo. Logo P(x) = Ax2 + B ou P(x) = Ax4 + Bx2 + C.

Fábio não precisava estudar o caso P(x) = Ax2 + B, pois isso é um caso particular do
caso P(x) = Ax4 + Bx2 + C, mas ele estudou. Na hora de estudar o caso P(x) = Ax4 +
Bx2 + C, ele viu que a conta seria grande. Mas nesse momento ele lembrou as
palavras que o prof. Luciano sempre diz: “tem que ter garra para ser campeão!” e
não fraquejou, fazendo as contas:

Substitua a = b = c = 0 na equação do enunciado. Segue imediatamente que C = 0.


Logo P(x) = Ax4 + Bx2. Substituindo (isso mesmo!) e tendo em mente que
ab + bc + ca = 0 e que “hay que hacer las cuentas, pero sin perder la simetría
jamás”,
(a – b)2[A(a – b)2 + B] + (b – c)2[A(b – c)2 + B] + (c – a)2[A(c – a)2 + B] =
= 2(a + b + c)2 [A(a + b + c)2 + B]
⇔ A[(a – b)4 + (b – c)4 + (c – a)4] + B[(a – b)2 + (b – c)2 + (c – a)2]
= 2A(a + b + c)4 + 2B(a + b + c)2
⇔ A[a4 – 4a3b + 6a2b2 – 4ab3 + b4 + b4 – 4b3c + 6b2c2 – 4bc3 + c4 + c4 – 4c3a + 6c2a2
– 4ca3 + a4] + B[a2 – 2ab + b2 + b2 – 2bc + c2 + c2 – 2ca + a2]
= 2A(a2 + b2 + c2 + 2ab + 2bc + 2ca)2 + 2B(a2 + b2 + c2 + 2ab + 2bc + 2ca)
⇔ 2A[a4 + b4 + c4 – 2ab(a2 + b2) – 2bc(b2 + c2) – 2ca(c2 + a2) + 3a2b2 + 3b2c2 +
3c2a2] + 2B[a2 + b2 + c2] = 2A[a4 + b4 + c4 + 2a2b2 + 2b2c2 + 2c2a2] + 2B[a2 + b2 +
c2] ⇔ A[a2b2 + b2c2 + c2a2 – 2ab(a2 + b2) – 2bc(b2 + c2) – 2ca(c2 + a2)] = 0
⇔ A[a2b2 + b2c2 + c2a2 – 2a2(ab + bc + ca) + 2a2bc – 2b2(ab + bc + ca) +
2ab2c – 2c2(ab + bc + ca) + 2abc2] = 0 ⇔ A[(ab + bc + ca)2 – 2abc(a + b + c) +
2abc(a + b + c)] = 0 ⇔ 0 = 0.

EUREKA! N°21, 2005

18
Sociedade Brasileira de Matemática

Como todas as passagens são equivalências, não há restrição sobre A e B. Portanto


os únicos polinômios que satisfazem as condições do enunciado são os da forma
P(x) = Ax4 + Bx2, A, B reais.

PROBLEMA 3
Um gancho é uma figura formada por seis quadrados unitários como no seguinte
diagrama

ou qualquer uma das figuras obtidas desta aplicando rotações ou reflexões.


Determine todos os retângulos m × n que podem ser cobertos com
ganchos de modo que:
i) O retângulo é coberto sem buracos e sem sobreposições;
ii) Nenhuma parte de nenhum gancho pode cobrir regiões fora do retângulo.

SOLUÇÃO OFICIAL
Considere um preenchimento do tabuleiro m × n. Para cada gancho A, existe um
único gancho B cobrindo o quadradinho “interno” de A com uma de suas
“extremidades”. Além disso, o quadradinho “interno” de B deve ser coberto por uma
das “extremidades” de A. Assim, num recobrimento, todos os ganchos formam
pares. Há apenas duas maneiras de formar tais pares. Em um caso, A e B formam um
retângulo 3 × 4; no outro, a sua união é um octógono, com lados 3, 2, 1, 2, 3, 2, 1, 2.
Então um tabuleiro m × n pode ser coberto por ganchos se e somente se pode ser
coberto com os pares de 12 quadradinhos acima. Suponha que tal recobrimento
exista. Então mn é divisível por 12. Provaremos que m ou n é divisível por 4.
Assuma por absurdo que isso não acontece; então m e n são ambos pares, pois mn é
divisível por 4. Imagine o tabuleiro dividido em quadradinhos unitários, com linhas
e colunas rotuladas 1, 2, …, m e 1, 2, …, n, respectivamente. Escreva 1 no
quadradinho (i, j) se exatamente um entre os números i e j é divisível por 4, e 2, se i
e j são ambos divisíveis por 4. Como o número de quadrados em cada linha e coluna
é par, a soma de todos os números escritos é par. Mas não é difícil verificar que um
retângulo 3 × 4 sempre cobre números com soma 3 ou 7; e o outro tipo de par
sempre cobre números com soma 5 ou 7. Conseqüentemente, o número de pares de
peças é par. Assim, mn é divisível por 24 e, portanto, por 8, absurdo, pois
supusemos que nem m nem n é múltiplo de 4.

EUREKA! N°21, 2005

19
Sociedade Brasileira de Matemática

Note também que nem m nem n pode ser 1, 2 ou 5 (qualquer tentativa de colocar as
peças nesses casos falha). Concluímos então que se um recobrimento é possível
então m ou n é divisível por 3, m ou n é divisível por 4 e m, n ∉ {1; 2; 5}.

Reciprocamente, se essas condições acima são verificadas, o recobrimento é


possível (utilizando somente retângulos 3 × 4). Isso é imediato quando 3 divide m e
4 divide n (ou vice-versa). Seja m divisível por 12 e n ∉ {1; 2; 5} (ou vice-versa).
Então n pode ser representado como a soma de vários 3’s e vários 4’s. Então o
tabuleiro pode ser particionado em retângulos m × 3 e m × 4, que são fáceis de
cobrir, utilizando novamente retângulos 3 × 4.

SEGUNDO DIA

PROBLEMA 4
Seja n ≥ 3 um inteiro. Sejam t1 , t2 ,..., tn números reais positivos tais que
1 1 1
n 2 + 1 > (t1 + t2 + ... + tn )  + + ... +  .
 t1 t2 tn 
Mostre que ti , t j e tk são as medidas dos lados de um triângulo para quaisquer i, j, k
com 1 ≤ i < j < k ≤ n.

SOLUÇÃO DE HENRY WEI CHENG HSU (SÃO PAULO - SP)


Suponha tn ≥ tn – 1 ≥ … ≥ t2 ≥ t1 e seja tn = t1 + t2 + k.

Temos
1 1 1  t1 + t2 + "+ tn−1 1 1 1 
n2 + 1 > (t1 + t2 + "+ tn−1 )  + + "+ + + tn  + + " +  +1
 t1 t2 tn−1  tn  t1 t2 tn−1 

Por Chebyshev (na verdade, poderíamos usar Cauchy ou MA-MH…),


1 1 1 
 + +" + 
(t1 + t2 + " + tn −1 )  t1 t2 tn −1  n − 1

n −1 n −1 n −1
1 1 1 
⇔ (t1 + t2 + " + tn −1 )  + + " +  ≥ (n − 1)
2

t
 1 2t t n −1 
Assim,

EUREKA! N°21, 2005

20
Sociedade Brasileira de Matemática

t1 + t2 + " + tn −1 1 1 1 
n2 + 1 > (n − 1)2 + 1 + + tn  + + " + 
tn  t1 t2 tn −1 
t +t t t t + " + tn −1 tn
+ +"+ n
t
⇔ 2n − 1 > 1 2 + n + n + 3
tn t1 t2 tn t3 tn −1
t t
Como k + n ≥ 2 , temos
tn tk
t +t t t t +t t t
2n − 1 > 2n − 6 + 1 2 + n + n ⇔ 5 > 1 2 + n + n
tn t1 t2 tn t1 t2
Sendo tn = t1 + t2 + k,
t +t t + t + k t1 + t2 + k k t k t k
5> 1 2 + 1 2 + ⇔ 5 > 1− +1+ 2 + + 1 +1+
+ +
t1 t2 k t1 t2 +
t1 t2 k + t1 t2 t2 t1
t1 t2 k k k
⇔2> + + + −
t2 t1 t2 t1 t1 + t2 + k
t t
Mas 1 + 2 ≥ 2 . Deste modo,
t2 t1
1 1 1  1 1 1
0> k + −  ⇔ 0 > k + − 
 t1 t2 t1 + t2 + k   t1 t2 tn 
1 1 1
Veja que + > pois t1 ≤ t2 ≤ tn. Logo k < 0 e tn < t1 + t2.
t1 t2 tn

O resto é mais fácil: sejam 1 ≤ i < j ≤ n. Temos ti + tj ≥ t1 + t2 > tn ≥ tk para


1 ≤ k ≤ n e, portanto, ti, tj e tk são sempre lados de um triângulo.

PROBLEMA 5

n
Num quadrilátero convexo ABCD a diagonal BD não é bissetriz do ângulo ABC n
nem do ângulo CDA . Um ponto P no interior de ABCD satisfaz

∠PBC = ∠DBA e ∠PDC = ∠BDA.

Prove que os vértices do quadrilátero ABCD pertencem a uma mesma circunferência


se e só se AP = CP.

EUREKA! N°21, 2005

21
Sociedade Brasileira de Matemática

SOLUÇÃO DE RAFAEL DAIGO HIRAMA (CAMPINAS - SP)

B
α1
α1 α 2

A C
P
β2
β1
β1

Primeiro ato: considerações gerais.


Vamos determinar a figura a partir de quatro ângulos: sejam ∠ABD = α1, ∠BDA =
β1, ∠CBD = α2, ∠CDB = β2. Então ∠DBP = α2 – α1 e ∠BDP = β2 – β1.

Veja que esses ângulos não dependem de P estar dentro do triângulo ABD (o que
implica ∠ABD > ∠DBC e ∠ADB > ∠BDC) ou P estar dentro do triângulo BCD (o
que implica ∠ABD < ∠DBC e ∠ADB < ∠BDC) pois se ∠ABD = ∠PBC então
∠ABD ± ∠DBP = ∠PBC ± ∠DBP (+ se P está dentro de BDC e – se P está dentro
de ABD) ⇒ ∠ABP = ∠DBC = α2. Para os ângulos com vértice D é análogo.

Usando lei dos senos nos triângulos ABP, BCP, ADP e DCP, temos
AP AB CP BC
= (1) = (2)
senα 2 sen∠APB senα1 sen∠BPC
AP AD CP DC
= (3) = (4)
senβ2 sen∠APD senβ1 sen∠DPC

De (1) e (4), temos


AP AB senα 2 sen∠DPC
= ⋅ ⋅ (I)
CP DC senβ1 sen∠APB

EUREKA! N°21, 2005

22
Sociedade Brasileira de Matemática

De (2) e (3), temos


AP AD senβ2 sen∠BPC
= ⋅ ⋅ (II)
CP BC senα1 sen∠APD

As equações (I) e (II) são fundamentais para a solução do problema, pois envolvem
AP e CP, lados do quadrilátero ABCD e, além disso, observando a figura e essas
equações, notamos algumas simetrias interessantes. Rafael começa a usá-las a partir
de agora.

Vamos lá: sen∠APD = sen(∠APD + ∠BPC – ∠BPC) = sen(∠APD + ∠BPC) ⋅


cos∠BPC – cos(∠APD + ∠BPC) ⋅ sen∠BPC.

Seja θ = ∠APD + ∠BPC = 360º – (∠APB + ∠CPD). Logo sen∠APD =


senθ ⋅ cos∠BPC – cosθ ⋅ sen∠BPC (III) e sen∠APB = sen(∠APB + ∠CPD –
∠CPD) = sen(∠APB + ∠CPD) ⋅ cos∠CPD – sen(∠APB + ∠CPD) ⋅ cos∠CPD =
–senθ ⋅ cos∠CPD – cosθ ⋅ sen∠CPD (IV) (lembre-se de que sen(360º – x) = –sen x
e cos(360º – x) = cos x).

Segundo ato: ABCD cíclico ⇒ AP = CP.


Supondo que ABCD é cíclico (esta é a primeira vez que vamos usar este fato),
AB CD AD BC
temos, pela lei dos senos estendida, que = e = .
senβ1 senα2 senα1 senβ 2
Substituindo essas duas últimas igualdades e (III) e (IV) em (I) e (II),
respectivamente, temos
PC sen∠APD senθ
(VI): = = − cos θ
AP sen∠BPC tg ∠BPC
PC sen∠APB senθ
(V): = =− − cos θ
AP sen∠CPD tg ∠CPD
Logo tg∠BPC = – tg ∠ CPD ou senθ = 0. A primeira possibilidade implica ∠BPC +
∠CPD = 180º, mas aí B, P e C seriam colineares e então BD bissectaria os ângulos
∠ABC e/ou ∠ADC.

Portanto senθ = 0 e então θ = 180º, pois θ = ∠APD + ∠BPC está entre 0 e 360º.

PC sen180,
Então = − cos180, = 1, ou seja, AP = PC.
AP tg ∠BPC

EUREKA! N°21, 2005

23
Sociedade Brasileira de Matemática

Terceiro ato: AP = CP ⇒ ABCD cíclico.


AD senβ2 sen∠APD CD senβ1 sen∠CPD
De (II) e (I), temos ⋅ = e ⋅ = .
BC senα1 sen∠BPC AB senα 2 sen∠APB
AD senβ2 CD senβ1 sen∠APD sen∠CPD
Multiplicando, temos ⋅ ⋅ ⋅ = ⋅ .
BC senα1 AB senα 2 sen∠BPC sen∠APB
AD AB CD BC
Aplicando a lei dos senos em ABD e BCD temos = e = .
senα1 senβ1 senα 2 senβ 2
sen∠APD sen∠APB
Assim, =
sen∠BPC sen∠CPD

Mas, fazendo as mesmas contas que fizemos em (V) e em (VI) (veja que as
igualdades à direita em (V) e (VI) não dependem de ABCD ser cíclico), vemos que
sen∠APD sen∠APB AD senβ2 AD BC
= = 1. E então × =1 ⇔ = . Sejam R1
sen∠BPC sen∠CPD BC senα1 senα1 senβ 2
e R2 os circunraios de ABD e BCD. Pela lei dos senos estendida,
AD BC
2 R1 = = = 2 R2 ⇒ R1 = R2.
senα1 senβ2

Logo os triângulos ABD e BCD têm o mesmo circunraio. Só temos duas


possibilidades para um circuncírculo de raio fixado em um segmento fixado: uma
das possibilidades é A, B, C e D serem concíclicos, que é o que queremos; a outra é
a circunferência que passa por A, B e D ser a simétrica da que passa por B, C e D em
relação a BD.

α1
α2

β1 β2 C

EUREKA! N°21, 2005

24
Sociedade Brasileira de Matemática

Neste último caso vamos ter ∠BAD = ∠BCD. Como α1 + β1 + ∠BAD = 180º e α2 +
β2 + ∠BCD = 180º, α1 + β1 = α2 + β2.

Mas veja que (α1 > α2 e β1 > β2) ou (α1 < α2 e β1 < β2) pois caso contrário, como P
está no interior do quadrilátero ABCD, P deveria estar no interior de ABD e BCD ao
mesmo tempo, o que é impossível. Logo não é possível que α1 + β1 = α2 + β2, e o
problema acabou.

Enfim, uma solução muito parecida com uma trilogia grega, porém sem nenhuma
tragédia.

PROBLEMA 6
Um inteiro positivo é dito alternante se, na sua representação decimal, quaisquer
dois dígitos consecutivos têm paridade diferente.
Determine todos os inteiros positivos n tais que n tem um múltiplo que é alternante.

SOLUÇÃO DE RAFAEL MARINI SILVA (RIO DE JANEIRO - RJ)


O principal diferencial da solução do Marini é que, em vez de procurar exemplos
particulares para múltiplos alternantes de n, como a maioria dos estudantes fez, ele
deu uma caracterização bastante interessante dos números alternantes. Vamos à
solução:

Seja T o conjunto dos números cuja representação decimal consiste em algarismos


todos menores do que 5. Veja que todo número alternante a pode ser escrito em uma
das duas seguintes formas:

102 n − 1

+ 2k =
se a é ímpar, a = 101010...101
2 n−1 dígitos
99
+ 2k, k ∈ T, k com no máximo 2n

dígitos; (I)
102 n − 1

+ 2k =
se a é par, a = 101010...1010
2 n dígitos
99
⋅ 10 + 2k, k ∈ T, k com no máximo

2n + 1 dígitos; (II)

Agora, Marini divide o problema em casos:

Caso 1: mdc(10; p) = 1.
Aqui, Marini toma sim um número particular. Não é muito difícil encontrar um
número alternante nesse caso, se você conhece o teorema de Euler-Fermat.

EUREKA! N°21, 2005

25
Sociedade Brasileira de Matemática

Tome k = 0 na forma (I) e n = ϕ (99p). Pelo teorema de Euler-Fermat, 10ϕ (99p) ≡ 1


102 ϕ(99 p ) − 1
(mod 99p) (veja que mdc(10; 99p) = 1) e portanto o número alternante é
99
múltiplo de p.

Caso 2: p é par, mas não é múltiplo de 5.


Assim, p = 2e+1u, sendo u ímpar e não múltiplo de 5. O número alternante que
102 n − 1
devemos encontrar é da forma (II). Assim, 2e+1u| ⋅ 10 + 2k, ⇔
99
102 n − 1
2eu| ⋅ 5 + k, k ∈ T, k com no máximo 2n + 1 algarismos.
99
102 n -1
A nova dificuldade nesse caso é a potência de 2, ou seja, que 2e| ⋅ 5 + k. Para
99
102 n − 1
n suficientemente grande (maior que e/2), temos ⋅ 5 + k ≡ 0 (mod 2e) ⇔
99
k ≡ 5 ⋅ 99–1 (mod 2e). Marini resolveu, então, provar um lema um pouco mais geral,
mas que faz muito sentido: o conjunto T contém uma boa parte dos números
inteiros, então…

Lema. Fixados a e m naturais e q ∈{2,5} , existe t ∈ T tal que t ≡ a (mod q ).


m

Demonstração. Indução sobre m.

Base de indução: m = 1: imediato, pois T > {0, 1, 2, 3, 4} > {0, 1}.

Passo de indução: suponha o fato válido para m = k. Vamos provar para m = k + 1.

Suponha que 0 ≤ t < 10k . Seja Xt = {t; t + 10k; …; t + (q – 1) ⋅ 10k}. Note que esse
conjunto está contido em T (pois q ≤ 5) e que Xt contém todos os possíveis restos
congruentes a t módulo qk+1 (qk+1 não divide 10k e q divide 10). Da hipótese de
indução, dado x natural existe t ∈ T com t ∈ T com t < 10k (se t ≥ 10k , trocamos t
pelo número formado pelos seus k últimos dígitos que é congruente a t módulo 10k,
e logo módulo qk) tal que x ≡ t (mod qk). Vendo módulo qk+1 temos que x é
congruente a um elemento de Xt. Considerando a união de todos os conjuntos Xt,
com t variando entre 0 e qk – 1, vemos que cobrimos todos os possíveis restos (de
fato, como Xt tem q elementos, a união tem qk+1 elementos) e portanto o lema está
demonstrado.

EUREKA! N°21, 2005

26
Sociedade Brasileira de Matemática

Assim, aplicando o lema à congruência k ≡ 5 ⋅ 99–1 (mod 2e), vemos que podemos
escolher k0 ∈ T que satisfaz essa congruência. Seja m o número de dígitos de k0.
Tome s cópias de k0 (com alguns zeros à esquerda para que fique com m + e + 1
s −1  (10m +e +1 )s − 1 
dígitos) e concatene-as, isto é, seja k = k0 ∑10 ( m + e +1) j
= k0   . Veja que
j =0
 10m +e +1 − 1 
 
k ainda é um elemento de T e que k ≡ 5 ⋅ 99–1 (mod 2e), pois 10m+e+1 ≡ 0 (mod 2e).
Vamos fazer com que k seja múltiplo de u. Para isso, basta tomar s = ϕ((10m+e+1 – 1)
⋅ u) que o resto segue de Euler-Fermat.

102 n − 1
Retomando: o número alternante que procuramos é a = ⋅ 10 + 2k. Já
99
encontramos k. Agora, basta escolher um n adequado par acertar o número de
algarismos de a. Tome n = ϕ(99u)v maior que a quantidade de dígitos de k. Pronto!
Temos
102 n − 1 102 n − 1
a= ⋅ 10 + 2k = 2( ⋅ 5 + k) ≡ 0 (mod 2e+1) (graças ao lema!)
99 99
102 n − 1
a= ⋅ 10 + 2k ≡ 0 + 0 ≡ 0 (mod u)
99
Logo a é múltiplo de p.

Caso 3: p é múltiplo de 5, mas é ímpar.


Seja p = 5eu, u ímpar. Agora vamos procurar um número alternante da forma (I), ou
102 n − 1
seja, da forma + 2k, k ∈ T. Para n suficientemente grande,
99
102 n − 1
+ 2k ≡ 0 (mod 5e) ⇔ k ≡ 198–1 (mod. 5e)
99

Novamente pelo lema, existe k0 ∈ T que satisfaz essa congruência. Assim como no
s −1  (10m +e +1 )s − 1 
caso anterior, tomemos k = k0 ∑10 
m + e +1
= k0  , com
j =0  10 m + e +1
− 1 
 
s = ϕ((10 m+e+1
– 1) ⋅ u) e n = ϕ(99u)v maior que a quantidade de dígitos de k. De
modo análogo ao caso anterior, conseguimos um alternante múltiplo de p.

Caso 4: p é par e múltiplo de 5, ou seja, p é múltiplo de 10.

EUREKA! N°21, 2005

27
Sociedade Brasileira de Matemática

102 n − 1
Neste caso, devemos tomar um alternante da forma (II), ou seja, a = ⋅ 10 +
99
102 n − 1
2k, k ∈ T. Seja p = 10u. Temos que 10u| ⋅ 10 + 2k ⇒ 5|k. Como k ∈ T, seu
99
último dígito só pode ser zero, de modo que 10|k. Seja k = 10w, w ∈ T. Temos então
102 n − 1 102 n − 1
que u| + 2w, w ∈ T. Como + 2w é ímpar, u deve ser ímpar, e
99 99
caímos no caso 1 ou 3. Logo existe um múltiplo alternante de u só quando u é ímpar,
ou seja, quando p não é múltiplo de 20.

Logo os inteiros positivos que não possuem múltiplo alternante são exatamente os
múltiplos de 20.

9RFr VDELD«
4XH H[LVWHP SURJUHVV}HV DULWPpWLFDV DUELWUDULDPHQWH
ORQJDV IRUPDGDV DSHQDV SRU Q~PHURV SULPRV" ,VWR IRL
SURYDGR UHFHQWHPHQWH SRU %HQ *UHHQ H 7HUHQFH 7DR
GRLV H[ROtPSLFRV  H HUD DWp HQWmR XPD FRQMHFWXUD
EDVWDQWH DQWLJD  R ~OWLPR SURJUHVVR LPSRUWDQWH HP VXD
GLUHomR KDYLD VLGR R UHVXOWDGR GH 9DQ GHU &RUSXW TXH
SURYRX HP  TXH H[LVWHP LQILQLWDV SURJUHVV}HV
DULWPpWLFDV GH WUrV WHUPRV IRUPDGDV SRU SULPRV
9HMD R DUWLJR XP SUHSULQW DLQGD QmR SXEOLFDGR GH *UHHQ
H 7DR HP KWWSDU[LYRUJDEVPDWK17 H R
DUWLJR H[SRVLWyULR GH %U\QD .UD VREUH R DVVXQWR HP
KWWSPDWKQRUWKZHVWHUQHGXaNUDSDSHUVSULPHVSGI

EUREKA! N°21, 2005

28
Sociedade Brasileira de Matemática

CENTRO DE MASSA E APLICAÇÕES À GEOMETRIA


Emanuel Carneiro & Frederico Girão – UFC

Nível Avançado

1. INTRODUÇÃO
Chamaremos de sistema de massas um conjunto de n pontos P1 , P2 ,..., Pn no plano,
sendo que ao ponto Pk = ( xk , yk ) está associada uma massa mk ∈ \, de modo que
m1 + m2 + ... + mn ≠ 0. Definiremos o centro de massa desse sistema como sendo o
ponto ( x, y) tal que:

m1 x1 + m2 x2 + ... + mn xn m y + m2 y2 + ... + mn yn
x= ; y= 1 1 ,
M M
Onde M = m1 + m2 + ... + mn é a massa associada a ele.

Notação:
Quando ao ponto (x, y) estiver associada uma massa m escreveremos (x, y)[m].

Observações:
(i) Podemos interpretar fisicamente o centro de massa de um sistema como sendo o
ponto onde ele concentra toda sua massa. Em termos práticos, isso nos ajuda a
simplificar, por exemplo, problemas de Dinâmica onde há aplicações de forças sobre
o sistema.

(ii) Podemos considerar os pontos em \n . Neste caso, o cálculo do centro de


massa de um sistema é análogo.

(iii) Claramente o centro de massa é único.

2. PROPRIEDADES BÁSICAS

Proposição 1.
Seja (x, y)[m] o centro de massa do sistema
S1 = {( x1 , y1 )[m1 ], ( x2 , y 2 )[m2 ],..., ( xk , y k )[m k ]}, e seja (a, b)[N] o centro de
massa do sistema S 2 = {( a1 , b1 )[ n1 ], ( a2 , b2 )[n2 ],..., ( al , bl )[ nl ]}. Então, se

EUREKA! N°21, 2005

29
Sociedade Brasileira de Matemática

M + N ≠ 0, o centro de massa do sistema S = S1 ∪ S 2 é o centro de massa do


sistema {( x , y )[ M ], (a , b )[ N ]}.

Demonstração:
Por definição o centro de massa do sistema S = S1 ∪ S 2 é o ponto
( X , Y )[ M + N ], onde:

X =
∑ k
i =1 mi xi + ∑ lj =1 n j a j
=
M x + Na
M +N M +N
que é justamente a primeira coordenada do centro de massa do sistema
{( x , y )[ M ], (a , b )[ N ]}. Para a segunda coordenada é análogo.

A proposição acima nos dá um algoritmo para calcular o centro de massa de um


sistema com n pontos. Para isso tomamos dois pontos ( x1 , y1 )[m1 ] e
( x2 , y 2 )[ m2 ] quaisquer desse sistema e os substituímos pelo seu centro de massa
com a massa m1 + m2 . Recaímos assim num sistema com n – 1 pontos e
continuamos o processo. Assim o cálculo de centros de massa resume-se apenas ao
caso n = 2, que estudamos a seguir:

Centro de massa de um sistema com duas massas


O centro de massa ( x , y )[ M ] de um sistema {( x1 , y1 )[m1 ], ( x 2 , y 2 )[m 2 ]} é
colinear com os pontos ( x1 , y1 ) e ( x2 , y 2 ) pois
x1 y1 1
x2 y2 1 = x1 y 2 + xy1 + x2 y − xy 2 − x2 y1 − x1 y
x y 1
 m x + m2 x2   m1 y1 + m2 y2 
= x1 y2 +  1 1  y1 +   x2
 m1 + m2   m1 + m2 
 m x + m2 x2   m1 y1 + m2 y2 
− 1 1  y 2 − x2 y1 −   x1
 m1 + m2   m1 + m2 
=0

E além disso se chamamos ( x1 , y1 ) = A, ( x2 , y 2 ) = B e ( x , y ) = G vale que:

EUREKA! N°21, 2005

30
Sociedade Brasileira de Matemática

JJJG JJJG
m1 AG + m2 BG = 0
tal fato é deixado como exercício para o leitor.

Observação:
Pela equação acima distinguimos alguns casos:

• As duas massas têm o mesmo sinal. Nesse caso o ponto G está entre A e B e vale
que m1 AG = m2 BG .

• As duas massas têm sinais contrários. Nesse caso G está fora do segmento AB e
vale que m1 AG = m2 BG .

3. APLICAÇÕES À GEOMETRIA
Exemplo 1: Vamos tomar um triângulo ABC qualquer e pôr massas iguais em seus
três vértices, ou seja consideraremos o sistema A[ p ], B[ p ], C [ p ]. Chamaremos de
G o centro de massa desse sistema. Como encontrar o ponto G?
(hummm…) denotaremos C.M. = centro de massa.

A[p]

P N
G

B[p] M[2p] C[p]

Vamos usar a proposição da seção anterior. O.C.M. de B[ p ] e C [ p ] é o seu ponto


médio M. Podemos então trocar B[ p ] e C [ p ] por M [2 p ]. Logo o ponto G será o
C.M. de A[ p ] e M [2 p ], que está sobre AM e divide AM na razão
AG 2
= .
GM 1
EUREKA! N°21, 2005

31
Sociedade Brasileira de Matemática

Sejam N e P os pontos médios de AC e AB. De modo análogo poderíamos ter


provado que G ∈ BN e que G ∈ CP. Esta é uma demonstração diferente que as
três medianas concorrem em G, que é portanto o baricentro do triângulo. Além
disso, segue do exposto acima que:
AG BG OG 2
= = =
GM GN GP 1
Exemplo 2: Denote por a, b, c, os lados do triângulo ABC da maneira usual.
Vamos pôr agora massas nos vértices do triângulo proporcionais aos lados opostos,
ou seja, considere o sistema A[ a ], B[b], C[ c ]. Seja I o C.M. desse sistema.
Você merece um prêmio se descobrir quem é I…
A[a]

c K
b
L
I

B[b] J[b+c] C[c]


A[a]
a

O raciocínio é igual ao do exemplo anterior. O C.M. de B[b ] e C [c ] é um ponto J


JB c
no lado BC tal que = , ou seja, J é o pé da bissetriz interna. Logo I será o
JC b
AI b + c
C.M. de A[ a ] e J [b + c ]. Tiramos daí que I ∈ AJ e que = .
IJ a
Sejam BL e CK bissetrizes internas. De modo análogo poderíamos ter provado que
I ∈ BL e que I ∈ CK , o que mostra que I é o incentro. As razões saem de graça:
BI a + c CI a + b
= ; =
IL b IK c
Exemplo 3: Seja p o semiperímetro do triângulo. Agora uma novidade: o sistema de
massas será A[ p − a ], B[ p − b], C[ p − c]. Seja N o C.M. desse sistema.
Você realmente merece um prêmio se descobrir quem é o N.

EUREKA! N°21, 2005

32
Sociedade Brasileira de Matemática

A[p – a]

p–b p–c

Z
Y
N
p–a p–a

B [p – b] p–c X[a] p–b C[p – c]

O C.M. de B[ p − b ] e C [ p − c ] é um ponto X sobre o lado BC tal que


BX p −c
= , donde concluímos que BX = p − c e que CX = p − b. Este
CX p−b
ponto X é onde o exincírculo relativo ao lado a toca este lado (como referência sobre
este fato podemos indicar [1]). Logo N será o C.M. de A[ p − a ] e
X [ p − c + p − b] = X [ a ].
AN a
Portanto N ∈ AX e = . Se considerarmos os pontos Y e Z onde os
NX p−a
exincírculos relativos aos lados b e c tocam estes lados, respectivamente, podemos
mostrar que N ∈ BY e N ∈ CZ . Conclusão: AX, BY e CZ sào concorrentes em N
que é chamado Ponto de Nagel do ∆ABC . Ora, ora, poderíamos saber disso usando
o teorema de Ceva (veja por exemplo [3]). Calma, o melhor ainda está por vir. As
razões aqui são cortesias para nós:

BN b CN c
= ; =
NY p − b NZ p −c

O próximo resultado foi o que nos motivou a escrever este artigo. Ele mostra toda a
beleza desta teoria, enquanto outros métodos são ineficazes. Para uma demonstração

EUREKA! N°21, 2005

33
Sociedade Brasileira de Matemática

completa (e bastante extensa) do próximo resultado usando a geometria plana


clássica, veja [2].

Teorema 3.1. No ∆ABC considere os pontos I, G e N como definidos acima.


Vale então que I, G e N são colineares e ainda:
NG 2
=
GI 1
Prova: Seja p o semiperímetro do triângulo. Considere um sistema de massas
A[ p ], B[ p ], C [ p ]. Já sabemos que o C.M. desse sistema é o baricentro G.
Fazendo uso da proposição 1, podemos dividir esse sistema em dois subsistemas
S1 = A[ a ], B[b ], C [c ] e S 2 = A[ p − a ], B[ p − b ], C [ p − c ]. O C.M. de S1 é o
incentro I com massa [a + b + c ] = [2 p ], enquanto o C.M. de S 2 é o ponto da
Nagel N com massa [ p − a + p − b + p − c ] = [ p ]. Logo G será o C.M. de
I [2 p ], N [ p ] o que implica I, N, G colineares (com G entre I e N) e ainda pela
equação do momento linear:
NG 2
=
GI 1

A[p] =[a + (p – a)]

I[p] x y O
2y G 2x

H N[2p]

B[p] =[b + (p – b)] C[p] =[c + (p – c)]

Corolário 3.1.1. Em um triângulo qualquer ABC, sejam I,G,N como acima, O o


circuncentro e I o ortocentro. Então os pontos I, O, N, H formam um trapézio.

EUREKA! N°21, 2005

34
Sociedade Brasileira de Matemática

Prova: Sabemos que H, G, O são colineares (reta de Euler) e que:


HG 2
=
GO 1
Segue então do teorema anterior que IO é paralelo a NH, logo I, O, N, H formam um
trapézio, cujo encontro das diagonais é G.

Podemos aplicar estes métodos do centro de massa em problemas que envolvem o


ortocentro, o baricentro e os exincentros, para saber que massas devem estar nos
vértices, veja o problema 1. Divirta-se resolvendo estes problemas. Vale usar tudo,
mas experimente a sua mais nova arma.

4. PROBLEMAS RELACIONADOS

Problema 1
 a   b   c 
(a) Verifique que o sistema A  ,B ,C
 cos A   cos B   cos C 
tem como C.M. o
ortocentro do triângulo.

(b) Verifique que o sistema A[ sen 2 A], B[ sen 2 B ], C [ sen 2C ] tem como C.M. o
circuncentro.

(c) Prove que o C.M. do sistema A[ − a ], B[b], C [c ] é o exincentro relativo ao lado


a. Verifique os análogos para os outros exincentros.

Problema 2
Sejam A, B, C, D pontos concíclicos. Sejam G A , GB , GC , G D os baricentros dos
triângulos BCD, ACD, ABD e ABC. Prove que G A , GB , GC , G D são concíclicos.

Problema 3
Sejam ABCD um quadrilátero no espaço de forma que AB, BC, CD e DA sejam
tangentes a uma esfera γ nos pontos X, Y, Z, W. Prove que estes pontos são
coplanares.

Problema 4
Sejam X, Y e Z os pontos onde o incírculo do triângulo ABC toca os lados BC, AC e
AB, respectivamente. Mostre que o incentro de ∆ABC está sobre a reta que passa
pelos pontos médios de BC e AX. (veja uma solução em [5]).

EUREKA! N°21, 2005

35
Sociedade Brasileira de Matemática

Problema 5
Considere 6 pontos em uma dada circunferência. Tomamos três destes pontos e
marcamos seu baricentro G1 . Em seguida, marcamos o ortocentro H 2 dos outros
três pontos e traçamos o segmento G1 H 2 . Mostre que todos os 6 possíveis
  = 20
3 
segmentos G1 H 2 passam por um ponto fixo.

Problema 6
Seja ABCD um quadrilátero convexo inscritível com os lados opostos AD e BC se
encontrando em P, e AB e CD em Q. Prove que o quadrilátero EFGH, determinado
l , é um losango.
l e CQB
em ABCD pelas bissetrizes de DPC

Problema 7
Seja PABC um tetraedro e sejam A1 , B1 , C1 os pontos médios das arestas BC, AC e
AB, respectivamente. Seja α um plano paralelo à face ABC que intercepta as arestas
PA, PB, PC nos pontos A2 , B2 , C 2 respectivamente.
(a) Prove que A1 A2 , B1 B2 , C1C 2 concorrem em um ponto D.
(b) Determine o lugar geométrico dos pontos D quando α varia.

Problema 8
(a) Considere 4 pontos que formam um sistema ortocêntrico (cada um é o ortocentro
do triângulo formado pelos outros três). Ponha massas iguais nesses 4 pontos. Prove
que o centro de massa é o centro do círculo dos nove pontos de cada um desses 4
triângulos (veja [1] e o problema proposto No. 107 na página 61).

(b) (Beltrami) Prove que o C.M. do sistema formado pelo incentro e pelos três
exincentros com massas iguais é o circuncentro.

Problema 9
Seja ABCD um quadrilátero convexo inscritível com os lados opostos AD e BC se
l e
encontrando em P, e AB e CD em Q. Prove que as bissetrizes dos ângulos DPC
l e a reta que une os pontos médios das diagonais do quadrilátero ABCD
CQB
(diagonal de Euler) concorrem.

EUREKA! N°21, 2005

36
Sociedade Brasileira de Matemática

Problema 10
(Banco IMO/97) No ∆ABC acutângulo, sejam AD, BE alturas e AP, BQ bissetrizes
internas. Sejam I e O o incentro e o circuncentro do triângulo ABC, respectivamente.
Prove que os pontos D, E e I são colineares se e somente se P, Q e O são colineares.

Agradecimentos: A nosso amigo Carlos Shine pela primeira versão digitada deste
material, na Semana Olímpica de 2001, em Salvador - BA.

REFERÊNCIAS:
[1] Coxeter, H.S.M.; Greitzer, S.L., Geometry Revisited, MAA, 1967.
[2] Johnson, R.A., Advanced Euclidean Geometry, Dover Publications, 1960.
[3] Castro, L.G.M., Introdução à Geometria Projetiva, Eureka!, vol 8, pp 16-27, 2000.
[4] Honsberger, R., Mathematical Morsels, MAA, 1978.
[5] Moreira, C.G.T., Wagner, E., 10 Olimpíadas Iberoamericanas de Matemática, OEI,
1996.

9RFr VDELD«
4XH 28433 ⋅ 2 + 1 p SULPR"
7830457

(VWH Q~PHUR GH  GtJLWRV p R TXLQWR PDLRU SULPR


FRQKHFLGR H IRL GHVFREHUWR HP 
,VWR PRVWUD TXH  QmR p XP Q~PHUR GH 6LHUSLQVNL
LVWR p XP QDWXUDO tPSDU N WDO TXH k ⋅ 2 + 1 p FRPSRVWR
n

SDUD WRGR n ∈ `  YHMD D (XUHND  SiJ   UHGX]LQGR


SDUD  R Q~PHUR GH QDWXUDLV PHQRUHV TXH  TXH p R
PHQRU Q~PHUR GH 6LHUSLQVNL FRQKHFLGR  VREUH RV TXDLV
QmR VH VDEH VH VmR Q~PHURV GH 6LHUSLQVNL RX QmR
FRQMHFWXUDVH TXH QHQKXP GHOHV VHMD   
       H

9HMD KWWSZZZVHYHQWHHQRUEXVWFRP SDUD PDLV
LQIRUPDo}HV LQFOXVLYH VREUH FRPR DMXGDU D SURYDU HVVD
FRQMHFWXUD 

EUREKA! N°21, 2005

37
Sociedade Brasileira de Matemática

SEQÜÊNCIA DE FIBONACCI
Cícero Thiago B. Magalhães

Nível Avançado

INTRODUÇÃO
O nome seqüência de Fibonacci, foi dado pelo matemático francês Edouard Lucas no
século XIX. Porém, a seqüência surgiu de um problema que estava proposto na obra
"Liber Abaci" de Leonardo de Pisa (1180 – 1250), conhecido como Fibonacci. O
problema era o seguinte: "Um homem põe um casal de coelhos dentro de um cercado.
Quantos pares de coelhos serão produzidos em um ano, se a natureza desses coelhos é
tal que a cada mês um casal gera um novo casal, que se torna fértil a partir do segundo
mês?" Depois de séculos de trabalho, é possível hoje citar uma quantidade enorme de
propriedades da seqüência do número de coelhos existentes após n meses. O objetivo
deste trabalho é apresentar algumas propriedades básicas desta seqüência.

Definição
A seqüência de Fibonacci é definida da seguinte maneira:
f1 = f 2 = 1 e f n = f n −1 + f n − 2 , ∀n > 2.
Por conveniência, algumas vezes usaremos f 0 = 0.

Propriedades básicas
(I) Para todo n ≥ 1 : f1 + f 2 + ... + f n = f n + 2 − 1
Prova: n = 1: f1 = f 3 − 1
Vamos supor q ≥ 1 e f1 + f 2 + ... + f q = f q + 2 − 1
n = q + 1 : f1 + f 2 + ... + f q + f q +1 = f q + 2 − 1 + f q +1 = f q + 3 − 1

(II) Se m ≥ 1 e n > 1, então f n + m = f n −1 f m + f n f m +1


Prova: Vamos fazer indução sobre m:
m = 1: f n +1 = f n −1 f1 + f n f 2 = f n −1 + f n (verdadeira)
m = 2 : f n + 2 = f n −1 f 2 + f n f3 = f n −1 + 2 f n = ( f n −1 + f n ) + f n = f n + f n +1
(verdadeira)
Seja q > 2 e suponhamos a propriedade válida para todo k , 2 ≤ k < q , e para todo
n > 1. Esta suposição, mais o fato de que a propriedade vale também para k = 1, nos
garante:

EUREKA! N°21, 2005

38
Sociedade Brasileira de Matemática

f n + ( q − 2) = f n −1 f q − 2 + f n f q −1
f n + ( q −1) = f n −1 f q −1 + f n f q
Somando membro a membro essas igualdades e levando em consideração a fórmula
recursiva que define ( f n ) :
f n + q = f n −1 f q + f n f q +1
Ou seja, a fórmula vale também para q, sempre que n > 1. O princípio da indução
nos garante então que vale para todo m ≥ 1 e qualquer n > 1.
(III) Dois números de Fibonacci consecutivos f n e f n +1 são primos entre si.
A prova fica como exercício.
(IV) Se m | n, então f m | f n .
Prova: Por hipótese n = mq, para algum q ∈ `. Usaremos indução sobre r.
Se q = 1, então m = n e é fácil ver que f m | f n . Seja q ≥ 1 e admitamos que
fm | f mq .
Então, usando a propriedade (II):
f m ( q +1) = f mq + m = f mq −1 f m + f mq f m +1
Como f m | f mq −1 f m e f m | f mq f m +1 (pois, pela hipótese de indução, fm divide
f mq ), então f m divide a soma desses dois produtos. Ou seja: f m | f m ( q +1) .
(V) Seja d = mdc(m, n), então mdc ( f m , f n ) = f d .
Prova: Indução em m + n. Se m = 1, mdc(m, n) = 1 e mdc(fm, fn) = mdc(1, fn) = 1 = f1.
Se m + n = 2 é trivial. Se m = n não há o que provar.
Se 2 ≤ m < n, f n = f m + ( n − m ) = f m − 1 f n − m + f m f n − m + 1 ⇒ m dc ( f m , f n ) =
( III )
mdc ( f m , f m −1 f n − m ) = m dc ( f m , f n − m ), que é igual, por hipótese de indução a
f mdc ( m , n − m ) = f mdc ( m , n ) .
Veja também a solução do problema proposto No. 92 na Eureka! 20, pp 55 – 57.
(VI) Seja x = x + 1, então, para n = 2, 3, … nós temos que
2

x n = f n x + f n −1 .
Prova: É trivial o caso n = 2. E se x = f n x + f n −1 para algum n ≥ 2, então
n

x n +1 = x n ⋅ x = ( f n x + f n −1 ) x
= f n x 2 + f n −1 x

EUREKA! N°21, 2005

39
Sociedade Brasileira de Matemática

= f n ( x + 1) + f n −1 x
= ( f n + f n −1 ) x + f n
= f n +1 x + f n ,
como desejado!
1+ 5 1− 5
Vejamos que as raízes de x = x + 1 são os números α = eβ=
2
.
2 2
Então, para todo n = 2, 3, … nós temos
α n = f n α + f n −1
e
β n = f n β + f n −1 .
Subtraindo as duas últimas equações temos que α − β = f n (α − β ), e notando
n n

que α − β = 5 , nós encontramos a fórmula de Binet


α n − βn
fn = .
α −β
Problema 1
1+ 5
Seja α = . Determine todos os n ∈ ` tais que α n − n 2 α seja inteiro.
2
Solução:
Note que α é raiz do polinômio p ( x ) = x − x − 1, com isso e usando a expressão
2

da propriedade (V) temos que:


α n − n 2 α = f n α + f n −1 − n 2 α = ( f n − n 2 )α + f n −1 .
Uma vez que α é irracional, segue da igualdade acima que α − n α só será
n 2

inteiro quando f n − n = 0, e nosso problema equivale a determinar todos os


2

n ∈ ` tais que f n = n . Para tanto, observe que:


2

f1 = 1, f 2 = 1, f 3 = 2, f 4 = 3, f 5 = 5, f 6 = 8, f 7 = 13,
f 8 = 21, f 9 = 34, f10 = 55, f11 = 89, f12 = 144, f13 = 233, f14 = 377.
Assim, f12 = 12 2 e f13 > 13 2 , f14 > 14 2. Por outro lado, se f n > n2 e
f n +1 > (n + 1) 2 , então
f n + 2 = f n +1 + f n > n 2 + (n + 1) 2 > (n + 2) 2 , desde que n > 3,

EUREKA! N°21, 2005

40
Sociedade Brasileira de Matemática

donde segue por indução que f n > n para n > 13. Assim, as únicas soluções são
2

n = 1 e n = 12.

Problema 2:
Sejam n e k dois inteiros positivos quaisquer. Então entre duas potências
k k +1
consecutivas n e n não podemos ter mais que n números de Finonacci.
Sugestão: use a propriedade (I)

Problema 3:
Seja f n a seqüência de Fibonacci ( f1 = 1, f 2 = 1, f n +1 = f n + f n −1 ). Calcule a série

fn

n=2 f n −1 ⋅ f n +1
.

Problema 4:
Ache a, se a e b são números inteiros tais que x − x − 1 é um fator de
2

ax17 + bx16 + 1.
Fortemente ligada à seqüência de Fibonacci, e tão interessante quanto, é a seqüência
de Lucas que é definida da seguinte maneira:
Ln = f n −1 + f n +1
L1 = 1, L2 = 3, L3 = 4,...
Obs: é fácil perceber que de acordo com a definição da seqüência de Lucas temos
que L0 = 2.
Usando a fórmula de Binet temos que:
α n −1 − β n −1 α n +1 − β n +1
Ln = f n −1 + f n +1 = +
α −β α −β
1  n1  n1 
= α  + α  − β  + β  .
α −β   α  β 
1+ 5 1− 5 1 1
Como α = e β= , então +α = 5 e + β = − 5. Portanto,
2 2 α β
Ln = α n + β n .

Problema 5:
Prove que f 2 n = f n Ln .

EUREKA! N°21, 2005

41
Sociedade Brasileira de Matemática

Problema 6:
Sejam L0 = 2, L1 = 1, e Ln + 2 = Ln +1 + Ln , para n ≥ 0, a seqüência de Lucas.
Prove que, para todo m ≥ 1,
m

∏L
k =1
2k
= f 2m +1 ,

onde f n é a seqüência de Fibonacci.

Problema 7:
Achar o termo geral pn se p0 = 1 e pn +1 = 5 pn − 3 pn , para n ≥ 0.
3

Problema 8:
Todo natural pode ser unicamente escrito como soma de números de Fibonacci
distintos, de índices não consecutivos e maiores que 1. (Teorema de Zeckendorff).

Problema 9:
93 19 93
Sejam a e b inteiros positivos tais que b é divisível por a e a é divisível por
b19 . Prove que ( a 4 + b 8 ) n+1 é divisível por ( ab ) n para todo n > 1.
f f

Problema 10:
Prove que nenhum número de Fibonacci é potência de 7.

Problema 11:
Sejam ( f n ) a seqüência de Fibonacci e, para todo inteiro positivo n,

Vn = f n2 + f n2+ 2 .
Prove que, para todo inteiro positivo n, Vn , Vn +1 , Vn + 2 são lados de um triângulo de
área 1 .
2

REFERÊNCIAS:
[1] Ross Honsberger, Mathematical Gems III, MAA, 1985.
[2] Hygino H. Domingues, Fundamentos da Aritmética, Atual, 1991.

EUREKA! N°21, 2005

42
Sociedade Brasileira de Matemática

COMO É QUE FAZ?


PROBLEMA 6
PROPOSTO POR RAFAEL ALVES DA SILVA (TERESINA - PI) da Olimpíada Ibero-americana de
Matemática de 1999.
Seja n um inteiro maior que 10 tal que cada um dos seus dígitos pertence ao
conjunto S = {1, 3, 7, 9}. Prove que n tem algum divisor primo maior ou igual a 11.

Solução: Como o último dígito de n é ímpar, n é ímpar.


Como o último dígito de n não é 0 nem 5, n não é múltiplo de 5. Assim, se n não
tivesse nenhum fator primo maior ou igual a 11, os únicos possíveis fatores primos
de n seriam 3 e 7. Mostraremos por indução que todo número natural cujos únicos
fatores primos são 3 e 7 é da forma n = 20k + r, onde k é natural e r ∈ {1, 3, 7, 9} , e
portanto o dígito das dezenas de n deve ser par, e logo não pode pertencer a
S = {1, 3, 7, 9} (e assim, se todos os dígitos de n pertencem a S, devemos ter
n < 10).
Para isso, note que 1 = 20 ⋅ 0 + 1, e ainda

n = 20k + 1 ⇒ 3n = 20 ⋅ (3k) + 3 e 7n = 20 ⋅ (7k) + 7


n = 20k + 3 ⇒ 3n = 20 ⋅ (3k) + 9 e 7n = 20 ⋅ (7k + 1) + 1
n = 20k + 7 ⇒ 3n = 20 ⋅ (3k + 1) + 1 e 7n = 20 ⋅ (7k + 2) + 9
n = 20k + 9 ⇒ 3n = 20 ⋅ (3k + 1) + 7 e 7n = 20 ⋅ (7k + 3) + 3,
e nossa afirmação está provada.

PROBLEMA 7

PROPOSTO POR RAFAEL ALVES DA SILVA (TERESINA - PI) da Olimpíada da Bielorrúsia de


2000.
Seja M = {1, 2, 3,..., 40}. Ache o menor inteiro n para o qual é possível particionar
M em M subconjuntos disjuntos tais que, sempre que a, b e c (não necessariamente
distintos) pertencem ao mesmo subconjunto, então a ≠ b + c.

Solução: Mostraremos que n = 4. Vamos ver como construir um exemplo de uma


partição de n em 4 conjuntos de acordo com o enunciado, a partir de exemplos
menores: temos a partição {1} = {1}, com n = 1, para M = {1}; a partição
{1, 2, 3, 4} = {1, 4} ∪ {2, 3}, com n = 2, para M = {1, 2, 3, 4}; a partição {1, 2, 3,
4, 5, 6, 7, 8, 9, 10, 11, 12, 13} = {1, 4, 10, 13} ∪ {2, 3, 11, 12} ∪ {5, 6, 7, 8, 9} com
n = 3, para M = {1, 2,…,13}; a idéia é, dada uma partição {1, 2,…, k} =

EUREKA! N°21, 2005

43
Sociedade Brasileira de Matemática

A1 ∪ A2 ∪... ∪ An , obter uma partição {1, 2,..., 3k + 1} = B1 ∪ B2 ∪ ... ∪ Bk ∪ Bk +1 ,


tomando Bk+1 = {k +1, k + 2,...,2k +1} e, para i ≤ k, Bi = Ai ∪(Ai + (2k +1)). Assim,
obtemos, para M = {1,2,...,40}, M = {1,4,10,13,28,31,37,40}∪{2,3,11,12,29,30,38,39}
∪{5,6,7,8,9,32,33,34,35,36}∪{14,15,16,17,18,19,20,21,22,23,24,25,26,27}, e é fácil
verificar que esta partição satisfaz a condição do enunciado.
Para mostrar que não é possível obter uma tal partição de M em apenas 3
subconjuntos, suponha que M = A ∪ B ∪ C. Como M tem 40 elementos, algum dos
conjuntos A, B ou C tem pelo menos 14 elementos. Digamos que A contenha os 14
elementos x1 < x2 < ... < x14 . Os 13 elementos x14 − x1 , x14 − x2 ,..., x14 − x13 devem
pertencer a B ∪C, pois, se x14 − xj = y ∈ A, então x14 = xj + y, com y , x j , x14 ∈ A,
contradição. Assim, B ou C contém pelo menos 7 desses 13 elementos, digamos
y1 < y2 < ... < y7 , onde yi = x14 − xji , onde j7 < j6 < ... < j1. Suponhamos que esses
elementos pertencem a B. Temos então y3 − y2 ∈ C e y3 − y1 ∈C, pois, se
y3 − y j = z ∈ B, para algum j ∈ {1, 2}, y3 = y j + z , com y3 , y j , z ∈ B , absurdo,
e, ( x14 − x j3 ) − ( x14 − x ji ) como y3 − yi = ( x14 − x j3 ) − ( x14 − x ji ) = x ji − x j3 , se
y3 − yi = z ∈ A, para algum i ∈ {1, 2}, teríamos x ji = x j3 + z , com xji , xj3 , z ∈ A ,
absurdo. Vamos agora considerar o elemento y2 − y1 > 0. Como y1 e y2 pertencem
a B, y2 − y1 não pode pertencem a B. Como y2 − y1 = ( y3 − y1 ) − ( y3 − y2 ), com
( y3 − y2 ), ( y3 − y1 ) ∈ C, y 2 − y1 não pode pertencer a C. Finalmente, como
y2 − y1 = (x14 − x j2 ) − (x14 − x j1 ) = x j1 − x j2 , com x j1 ∈ A e xj2 ∈ A, y2 − y1 não pode
pertencer a A, absurdo, pois y2 − y1 ∈ M = A ∪ B ∪ C.

Publicamos a seguir as soluções dos problemas 121 e 195 da seção "Olimpíadas ao redor do
mundo", por sugestão de Bruno, da Espanha.

121. (Rússia-2001) Os valores da função quadrática f (x ) = x 2 + ax + b para dois


inteiros consecutivos são os quadrados de dois inteiros também consecutivos.
Mostre que os valores da função quadrática são quadrados perfeitos para todos os
inteiros coincide com o conjunto dos valores de g para os inteiros.
Solução: Suponha f (m) = k 2 e f (m + 1) = (k + 1) 2 , com m e k inteiros. Seja
g ( x) = f ( x + m). O conjunto dos valores de f para os inteiros.

EUREKA! N°21, 2005

44
Sociedade Brasileira de Matemática

Temos g ( x) = x 2 + cx + d para certos valores de c e d. Temos d = g (0) = f ( m) = k 2


e 1 + c + d = g (1) = f (m + 1) = ( k + 1) 2 , donde d = k 2 e c = (k + 1) 2 − 1 − k 2 = 2k , ou
seja, g ( x) = x 2 + 2kx + k 2 = ( x + k ) 2 , e logo g(x) é um quadrado perfeito para todo x
inteiro.

195.(Eslovênia-2002) Sejam M o ponto médio da base AB do trapézio ABCD; E um


ponto interior ao segmento AC tal que BC e ME intersectam-se em F; G o ponto de
interseção de FD e AB; H o ponto de interseção de DE e AB. Mostre que M é o
ponto médio do segmento GH.

Solução: Podemos supor sem perda de generalidade, aplicando uma transformação


afim, que A = (0, 0), B = (1, 0), C = (1, 1), e logo D = (a, 1), para algum a < 1. Então
1 
M =  ,0  e E = (t, t), para algum t ∈ (0,1). Como F pertence a BC, F = (1, y) para
2 
1
algum y, e, supondo t ≠ , como F pertence a ME, y 1 = t t − 1
2
( ) (
2 2 )
, donde

t 1
y= (caso t = , BC e ME são paralelas e não se intersectam, logo não
2t − 1 2
existiria o ponto F). Como H pertence a AB, H = (x, 0) para um certo x. Como H
t 1 t (1 − a )
pertence a DE, = , donde (1 − t ) x = t (1 − a ), ou seja, x = .
t−x a−x 1− t
Finalmente, como G pertence a AB, G = (z, 0) para um certo z, e como G pertence a
1 t (2t − 1) t −1 (2t − 1) − at at − (2t − 1)
FD, = , donde ⋅z = , ou seja, z = .
a−z 1− z 2t − 1 2t − 1 1− t
x + z 1 1− t 1 G+H
Assim, = ⋅ = , e logo M = , ou seja, M é o ponto médio do
2 2 1− t 2 2
segmento GH.

EUREKA! N°21, 2005

45
Sociedade Brasileira de Matemática


SOLUÇÕES DE PROBLEMAS PROPOSTOS
Publicamos aqui algumas das respostas enviadas por nossos leitores.

90) Prove que, para todo inteiro positivo n e para todo inteiro não nulo a, o
polinômio xn + axn−1 + axn−2 + ... + ax −1 é irredutível, i.e., não pode ser escrito como
o produto de dois polinômios não constantes com coeficientes inteiros.

SOLUÇÃO (baseada em um argumento de Artur Avila):


1
Seja Pa (x) = xn + axn−1 + axn−2 + ... + ax −1. Como P−a (x) = −xn Pa   , Pa ( x) é
 x
irredutível se e somente se P−a ( x) é irredutível. Assim, podemos supor sem perda de
generalidade que a é um inteiro positivo. Nesse caso, Pa (0) = −1 e
Pa (1) = (n −1)a > 0, ∀n ≥ 2.
É claro que para n = 1 o polinômio Pa ( x) = x −1 é irredutível.
Para n ≥ 2 , deve existir α ∈( 0,1) com Pa (α ) = 0.
Seja Qa (x) = Pa (x) ⋅ (x −1) = (x −1)(xn −1) + ax(xn−1 −1) = xn+1 + (a −1)xn − (a +1)x +1.
Como α é raiz de Pa (α ) = 0.
Seja Qa (x) = Pa (x) ⋅ (x −1) = (x −1)(xn −1) + ax(xn−1 −1) = xn+1 + (a −1)xn − (a +1)x +1.
Como α é raiz de Pa ( x), α também é raiz de Qa ( x). Seja então
Q (x) 1
R(x) = a = xn + (a +α −1)xn−1 +α(a +α −1)xn−2 +α2 (a +α −1)xn−3 +... +αn−2 (a +α −1)x − .
x −α α
Temos R(1) = Qα (1) (1 −α ) = 0, e, se β ≠ α é raiz de Pa ( x), β é raiz de Qa ( x) e de
R(x). Mostraremos que β > 1 para todo β ∈^ , β ≠ α tal que β é raiz de Pa (x).
Como β é raiz de R(x), temos β n + ( a + α − 1) β n −1 + α ( a + α − 1) β n − 2 + ... +
1
+ α n −2 (a + α − 1) β =.
α
− − −
Se β ≤ 1, β n + ( a + α − 1) β n 1 + α ( a + α − 1) β n 2 + ... + α n 2 ( a + α − 1) β
n −1
+ ... + α n − 2 ( a + α − 1) β ≤ 1n + ( a + α − 1)1n −1 + ... +
n
≤ β + ( a + α − 1) β

EUREKA! N°21, 2005

46
Sociedade Brasileira de Matemática

1 1 1
+ α n −2 (a + α − 1) ⋅1 = R(1) + = = , valendo a igualdade se, e somente se,
α α α
β = β = 1, mas Pa (1) ≠ 0, donde β ≠ 1, absurdo (na verdade, mostramos que
todas as raizes de R ( x ) têm módulo maior ou igual a 1; em particular, α é raiz
simples de Pa ( x ), caso contrário α ∈ (0,1) seria raiz de R ( x)).
Suponha agora que Pa ( x ) = f ( x ) ⋅ g ( x), onde f e g são polinômios não constantes
com coeficientes inteiros. Como o coeficiente constante de Pa ( x ) é –1, os
coeficientes constantes de f(x) e de g(x) pertencem a {–1, 1} (e podemos supor sem
perda de generalidade que seus coeficientes líderes são iguais a 1). Assim, o produto
das raízes de g(x) (e de f(x)) pertencem a {–1, 1}. Por outro lado, podemos supor
sem perda de generalidade que α é raiz de f ( x ). Como todas as raízes de g(x) são
raízes de Pa ( x ) distintas de α , elas têm todas módulo maior que 1, e logo seu
produto não pode pertencer a {–1, 1}, absurdo.

95) "Resta-Um" é um jogo de tabuleiro na qual as peças ocupam um tabuleiro


formando parte de um reticulado retangular (na verdade, existem variações em
tabuleiros de reticulado triangular). O único movimento permitido consiste
em tomar duas peças em casas adjacentes vizinhas a uma casa vazia, e fazer a
peça mais distante da casa vazia pular sobre a outra peça, ocupando a casa
vazia. A peça pulada é retirada.

(esse movimento pode ser feito para a direita, para a esquerda, para cima ou
para baixo).
Agora imagine um tabuleiro que é um reticulado retangular infinito e uma reta
que contém uma linha do reticulado, dividindo-o em dois lados. Todas as casas
de um dos lados da linha estão vazias e cada casa do outro lado da linha pode
ou não ter uma peça.
Quantas peças, no mínimo, precisamos para chegar a uma casa do lado vazio
do tabuleiro, a uma distância n da linha ? Abaixo indicamos uma casa a
distância n, para n = 1,2,3,4,5.

EUREKA! N°21, 2005

47
Sociedade Brasileira de Matemática

5
4
3
2
1
… …
… …
… …
… … … … … … …

SOLUÇÃO DE BERNARDO FREITAS PAULO DA COSTA (RIO DE JANEIRO – RJ)


Vamos botar eixos no quadriculado, de acordo com a posição:
Y

"Fronteira"

Assim, a casa marcada com 9 é (0, 0), sua vizinha à esquerda é (1, 0) e acima é
(0, 1). Supomos, sem perda de generalidade, que a peça final termina na casa (0, n)
para n = 1, 2, 3, 4, 5. Usando isto, vemos que o movimento deveria "tender para o
centro e para cima".
Com esta idéia, vamos associar energias às peças nos quadrados do tabuleiro,
através da seguinte função:
ϕ 2 = ϕ + 1, ϕ > 1
E (a, b) = ϕ Φ onde  2
b a

Φ + Φ = 1, 0 < Φ < 1

EUREKA! N°21, 2005

48
Sociedade Brasileira de Matemática

Como as equações que os definem são simétricas, temos Φϕ = 1


1+ 5
>1
1± 5 2 1+ 5
Só para conferir: ϕ= ⇒ϕ =
2 1− 5 2
<0
2
5 −1
∈ (0,1)
−1 ± 5 2 5 −1
Φ= ⇒Φ=
2 −1 − 5 2
<0
2
Assim, se fizermos um movimento

a –2 a –1 a a –2 a
,a≤0
ou um movimento

a a +1 a +2 a a +2
,a≥0
a +2 a+
na direção do centro, estaremos trocando uma energia ϕ b Φ + ϕ b Φ 1 por
ϕ b Φ a , que são iguais por construção.
Já se fizermos

a a +1 a +2 a a +2
,a>0
ou o contrário (com a < 0)
trocaremos energia com perdas, pois a ≤ a + 2 (para a > 0; se
a < 0 é a ≤ a − 2 ).
Para movimentos verticais, vemos que

EUREKA! N°21, 2005

49
Sociedade Brasileira de Matemática

b+2

b+1

(
conserva energia ϕ b Φ + ϕ b +1Φ = ϕ b + 2 Φ
a a a
) e que
b+2

b+1

perde (pois ϕ b + 2 > ϕ b e ϕ b+1 > 0 ).


Assim, a energia das peças no tabuleiro não aumenta.
Vejamos a energia máxima do tabuleiro:
ET = E (coluna central) + E (quadrantes negativos) = E(coluna central) +
2E(1 quadrante negativo) =
0 ∞
= ∑ ϕ b Φ 0 + 2∑ E (coluna em a)
b =−∞ a =1

0
 0 a 
= ∑ϕ
b =−∞
+ 2∑  ∑ ϕ b Φ 
b

a =1  b =−∞ 
+∞ ∞ ∞
a  
= ∑ ϕ −b + 2∑ Φ  ∑ ϕ −b 
b =0 a =1  b =0 
 
1 ∞  1  1 1 1 1
= + 2∑ Φ   Mas ϕ 2 = ϕ + 1 ⇒ 1 = + 2 ⇒ 1 − = 2
a

1−
1 a =1 1−  1 ϕ ϕ ϕ ϕ
ϕ  
 ϕ

1 1
= + 2∑ Φ a
1 2 a =1
1 2
ϕ ϕ

   2Φ 
= ϕ 2  1 + 2∑ Φ a  = ϕ 2  1 + 
 a =1   1− Φ 

EUREKA! N°21, 2005

50
Sociedade Brasileira de Matemática

Como Φ 2 + Φ = 1, temos 1− Φ = Φ 2 , e assim


 2Φ 
( )
ET = ϕ 2  1 + 2  = ϕ 2 1 + 2 Φ = ϕ 2 (1 + 2ϕ ) = ϕ 2 (1 + ϕ + ϕ )
 Φ 
= ϕ (ϕ + ϕ ) = ϕ 2ϕ 3 = ϕ 5
2 2

Ora, como a energia total do tabuleiro é ϕ 5 , se tivermos um número finito de


movimentos (para poder chegar em (0, 5) de verdade) teremos usado um conjunto
finito de peças e portanto de energia estritamente menor do que ϕ 5 . Desta forma, é
impossível chegar a esta casa com movimentos "resta-um".
Decomponhamos ϕ n para n = 1, 2, 3, 4 para sabermos o número de peças

ϕ a + 2 = ϕ a +1 + ϕ a

necessárias. Vale lembrar as relações: Φ a = Φ a +1 + Φ a +2
 1
ϕ = 1 + Φ = 1 +
 ϕ
(ϕ = ϕ +1)
2

Φ= 1
ϕ
ϕ = 1 + Φ → 2 peças:
1

"Fronteira"

OK

Altura 2: ϕ 2 = ϕ + 1 = 1 + (1 + Φ ) só temos 1 peça com energia 1


= 1 + (Φ + Φ 2 + Φ ) só temos 3 peças com energia Φ
só temos 5 peças com energia Φ 2
4 peças: (em geral, para h ≥ 0, só temos 2h + 1 peças com energia Φ h )

"Fronteira"

EUREKA! N°21, 2005

51
Sociedade Brasileira de Matemática

"Fronteira"

"Fronteira"

Altura 3: ϕ 3 = ϕ 2 + ϕ = 2ϕ + 1 = 3 + 2Φ = 1 + 2Φ + (2)
= 1 + 2Φ + (2Φ + 2Φ 2 ) = 1 + 3Φ + 2Φ 2 + (Φ)
= 1 + 3Φ + 3Φ 2 + Φ 3 → 8 peças:
vai à (0, 2)

"Fronteira"

"Fronteira"

OK

"Fronteira"

EUREKA! N°21, 2005

52
Sociedade Brasileira de Matemática

Vamos ao último caso (altura 4):


ϕ 4 = ϕ 3 + ϕ 2 = 2ϕ 2 + ϕ = 3ϕ + 2 = 5 + 3Φ
= 1 + 3Φ + (4) = 1 + 3Φ + (4Φ + 4Φ 2 )
= 1 + 3Φ + 4Φ 2 + (4Φ 2 + 4Φ3 )
= 1 + 3Φ + 5Φ 2 + 4Φ 3 + (3Φ 2 ) = 1 + 3Φ + 5Φ 2 + 7 Φ 3 + 3Φ 4
Teoricamente, isto daria 1 + 3 + 5 +7 + 3 = 19 peças para chegar a altura 4.
Entretanto, isto é impossível. Vejamos:
Pintemos o tabuleiro com 3 cores em diagonal

B A C B A
C B A C B A
A C B A C B A
B A C B A C B A "Fronteira"
C B A C B A C B A
C B A C B A C B
C B A C B A C
C B A C B A
C

Toda vez que realizarmos um movimento, tiramos uma peça de duas cores
adjacentes e a terceira cor recebe uma peça a mais.
Assim, invertemos a paridade dos 3 simultaneamente. Portanto, a paridade relativa
se mantém.
As casas dentro da região indicada estarão todas ocupadas (suas energias somam
1 + 3Φ + 5Φ 2 + 7Φ 3 ).
A soma das cores é 5A + 7B + 4C
Se a soma final é 1A + 0B + 0C, temos que a paridade de B e C é a mesma, e oposta
à paridade de A, logo das 3Φ 4 que restam, temos uma quantidade par da cor A, uma
quantidade ímpar de cor B e uma quantidade para da cor C.
Assim, temos as seguintes possibilidades:
2A + 1B + 0C
0A + 3B + 0C
0A + 1B + 2C
Mas só há 1 casa B valendo Φ 4 logo a possibilidade OA + 3B + 0C está excluída.
Assim, concluímos que há 1 peça numa casa B, a única do nível Φ 4 .
Por simetria (ou seja, pintando o tabuleiro na outra direção : /// ),

EUREKA! N°21, 2005

53
Sociedade Brasileira de Matemática

temos que deve haver também uma peça na casa simétrica a essa casa B do nível
Φ 4 em relação à coluna central, a qual é uma casa C, e logo a solução é OA + 1B +
2C.
Assim, temos a seguinte configuração:

"Fronteira"
X X
? ?

X X
?

As casas marcadas com X são A ou simétricas a casas A, e logo estão proibidas.


As casas com • estão forçosamente ocupadas, e das casas com ?, uma, e apenas uma,
será ocupada (para fazer 19).
Isolemos a coluna central; sobra (pelo menos) um bloco consistindo apenas das
seguintes peças:

(ou do outro lado,


por simetria é
igual).

Ora, a peça : , para ser aproveitada, deve tomar uma peça que está a sua direita e ir
para a coluna – 1.
Entretanto, para terminarmos com peças apenas na coluna zero, as duas indicadas
pelas setas devem tomar as peças que estão à direita (pois senão não conseguiríamos
mandar todas as peças desse bloco para a coluna central usando apenas essas peças;
se usássemos outras peças para isso perderíamos energia). Nesse caso, as peças que
sobram do lado esquerdo ficam todas na altura 0, e a peça que estava originalmente
na coluna 0 e na coluna – 1 não conseguiría chegar na coluna central sem que haja
perda de energia. Assim, não conseguimos (agora por impedimento de movimento,
não de energia) chegar em (0, 4) com 19 peças.
Mas é possível com 20 (faço Φ 3 = Φ 4 + Φ 5 ):

EUREKA! N°21, 2005

54
Sociedade Brasileira de Matemática

"Fronteira" "Fronteira"

"Fronteira" "Fronteira"

"Fronteira"
"Fronteira"

"Fronteira"

OK!!

96) No quadrilátero ABCD os ângulos A, C e D medem 100° e o ângulo ACB mede


40°. Demonstre que BC ⋅ DA = ( BC + AB − DA)2 .

EUREKA! N°21, 2005

55
Sociedade Brasileira de Matemática

SOLUÇÃO DE HEYTOR BRUNO E MARLON JÚNIOR (FORTALEZA – CE)


A figura ilustra o problema:
A

20° 80°

60° B

D 100°

60°
40°

AC sen100°
∆ACD (Lei dos Senos): =
DA sen60°
BC sen80°
∆ACB (Lei dos Senos): =
AC sen 60°
AC BC
⇒ = ⇒ AC 2 = BC ⋅ DA
DA AC

Assim,

BC ⋅ DA = ( BC + AB − DA)2 ⇔ AC 2 = ( BC + AB − DA)2 . Basta provar então


que AC = BC + AB – DA.
AC sen60°
∆ACD (Lei dos Senos): DA =
sen100°

AC sen80° AC sen 40°


∆ACB (Lei dos Senos): BC = e AB =
sen60° sen 60°
Daí, AC + DA = BC + AB ⇔
AC sen60° AC sen80° AC sen 40°
⇔ AC + = + ⇔
sen100° sen60° sen60°

EUREKA! N°21, 2005

56
Sociedade Brasileira de Matemática

sen100° + sen60° sen80° + sen 40°


⇔ = .
sen100° sen60°

Usando o fato de que


 p+q  p−q
sen p + sen q = 2 sen   cos  ,
 2   2 

A equação acima é verdadeira se e somente se

2 sen80° cos 20° 2 sen60° cos 20°


= , o que é obviamente verdade.
sen100° sen 60°

99) Num triângulo, a razão entre os raios das circunferências circunscrita e inscrita é
5
. Os lados do triângulo estão em progressão aritmética e sua área é
2
numericamente igual ao seu perímetro. Determine os lados do triângulo.

SOLUÇÃO DE KELLEM CORRÊA SANTOS (RIO DE JANEIRO – RJ)


A
R 5
= ⇒ 2 R = 5r
r 2
R
a

a+q r
B

a–q

3a
Perimetro de ABC ⇒ 2 p = a + (a + q) + (a − q) = 3a ⇒ p =
2
a( a + q)(a − q)
pr = A( ABC ) =
4R

EUREKA! N°21, 2005

57
Sociedade Brasileira de Matemática

3a a (a 2 − q 2 ) a2 − q2 a2 − q2
r= ⇒ 3r = = ⇒ 15r 2 = a 2 − q 2 (1)
2 4R 2R 5r
Como a área é numericamente igual ao perímetro, temos:
pr = 2 p ⇒ r = 2
Por outro lado,
3a a  a  a 
A( ABC ) = p( p − a )( p − a − q)( p − a + q ) = ⋅ ⋅  − q  + q  =
2 2 2  2 
1  a2 
= a 3  − q2 
2  4 
De (1), temos:
15r 2 = a 2 − q 2 ⇒ 15 ⋅ 2 2 = a 2 − q 2 ⇒ 60 = a 2 − q 2 ⇒ q 2 = a 2 − 60
a  a2  a  3a 2 
Logo A( ABC ) = 3  − a 2 + 60  = 3  60 − 
2  4  2  4 
Também podemos escrever:
3a
A( ABC ) = pr = ⋅ 2 = 3a
2
Igualando:
a  3a 2 
3a = 3  60 − 
2  4 
 3a 2  36 3a 2 3a 2
62 = 3  60 −  ⇒ = 60 − ⇒ 12 = 60 − ⇒
 4  3 4 4
3a 2
⇒ 48 = ⇒ a 2 = 16 ⋅ 4 ⇒ a = 8, pois a > 0.
4
De (1):
15r 2 = a 2 − q 2 ⇒ 60 = 64 − q 2 ⇒ q = ±2
Logo, os lados do triângulo ABC são 6, 8 e 10.

101) a) Sejam ai , bi , ci reais positivos, para 1 ≤ i ≤ 3.


Prove que (a13 + a23 + a33 )(b13 + b23 + b33 )(c13 + c23 + c33 ) ≥ (a1b1c1 + a2b2c2 + a3b3c3 )3 .
b) Sejam a, b, c, x, y, z reais positivos. Prove que

EUREKA! N°21, 2005

58
Sociedade Brasileira de Matemática

a 3 b 3 c 3 ( a + b + c )3
+ + ≥ .
x 2 y 2 z 2 ( x + y + z)2

SOLUÇÃO DE MARCELO RIBEIRO DE SOUZA e WALLACE MARTINS (RIO DE JANEIRO – RJ)


a) Como esta é uma desigualdade homogênea, podemos supor
(a13 + a23 + a33 ) = 1
(b13 + b23 + b33 ) = 1
(c13 + c23 + c33 ) = 1
Com isto nossa desigualdade passa a ser:
1 ≥ (a1b1c1 + a2b2 c2 + a3b3c3 )3 . Isto é imediato de MA – MG:
(a13 + b13 + c13 )
≥ a1b1c1
3
(a23 + b23 + c23 )
≥ a2b2c2
3
(a33 + b33 + c33 )
≥ a3b3c3
3
Somando tudo e elevando ao cubo ficamos com:
1 ≥ ( a1b1c1 + a2b2 c2 + a3b3c3 ) cqd.
3

b) Este item decorre diretamente do anterior, tendo em vista que

 3  3 3
 
 a   b   c    3   3   3    3   3   3  
1 3 1 3 1 3 1 3 1 3 1 3

 2  +  2  +  2    x  +  y  +  z    x  +  y  +  z   ≥
 x 3   y 3   z 3                
 
3
 1 1 1 1 1 1 
 a ⋅ x3 ⋅ x3 + b ⋅ y3 ⋅ y3 + c ⋅ z3 ⋅ z 3 
 2 2 2 
x 
3 3
y z3
Portanto segue que
 a 3 b3 c 3  ( a + b + c )
3

 2 + 2 + 2 ≥ .
x y z  ( x + y + z )2

EUREKA! N°21, 2005

59
Sociedade Brasileira de Matemática

PROBLEMAS PROPOSTOS
Convidamos o leitor a enviar soluções dos problemas propostos e sugestões de novos
problemas para os próximos números.

102) Você recebe x metros de arame para cercar um terreno na forma de um


triângulo pitagórico (os lados são números inteiros), com a condição de que a
medida do cateto menor seja 24 metros. Qual deverá ser a medida do cateto
maior e o comprimento do arame, a fim de que a área seja:
a) máxima?
b) mínima?

103) Sejam A e B matrizes 2 × 2 com elementos inteiros.


Sabendo que A, A + B, A + 2B, A + 3B e A + 4B são invertíveis e que os
elementos das respectivas inversas também são todos inteiros, mostre que
A + 5B também é invertível e que os elementos da sua inversa também são
inteiros.

104) ABC é um triângulo. Mostre que existe um único ponto P de modo que:
( PA ) + ( PB ) + ( AB ) = ( PB ) + ( PC ) + ( BC ) = ( PC ) + ( PA ) + (CA)
2 2 2 2 2 2 2 2 2

105) O baricentro do triângulo ABC é G. Denotamos por g a , gb , gc as distâncias


desde G aos lados a, b e c respectivamente.
Seja r o rádio da circunferência inscrita. Prove que:
2r 2r 2r
a) g a ≥ , gb ≥ , g c ≥
3 3 3
g + gb + gc
b) a ≥3
r

106) Os polinômios P0 ( x, y, z ), P1 ( x, y, z ), P2 ( x, y, z ),... são definidos por


P0 ( x, y, z ) = 1 e Pm+1 ( x, y , z ) = ( x + z )( y + z ) Pm ( x, y , z + 1) − z 2 Pm ( x, y, z ),
∀m ≥ 0. Mostre que os polinômios Pm ( x, y , z ), m ∈ ` são simétricos em x, y,
z, i.e., Pm ( x, y, z) = Pm ( x, z, y) = Pm ( y, x, z) = Pm ( y, z, x) = Pm ( z, x, y) = Pm ( z, y, x),
para quaisquer x, y, z.

EUREKA! N°21, 2005

60
Sociedade Brasileira de Matemática

107) a) Dado um triângulo qualquer, prove que existe um círculo que passa pelos
pontos médios dos seus lados, pelos pés das suas alturas e pelos pontos médios
dos segmentos que unem o ortocentro aos vértices do triângulo (o chamado
"círculo dos nove pontos").

b) Prove que, se X é o centro do círculo dos nove pontos de um triângulo, H


o seu ortocentro, O seu circuncentro e G seu baricentro, então
JJJG 3 JJJG 1 JJJG
OX = ⋅ OG = ⋅ OH .
2 2

Problema 102 proposto por Sebastião Vieira do Nascimento (Campina Grande – PB); Problema
103 proposto por Carlos A. Gomes (Natal – RN); Problemas 104 e 106 propostos por Wílson
Carlos da Silva Ramos (Belém – PA); Problema 105 selecionado da XXXV Olimpíada
Matemática Espanhola, fase nacional, 1999 e enviado por Bruno Salgueiro Fanego (Espanha).

Agradecemos também o envio das soluções e a colaboração de:

Carlos Alberto da Silva Victor Nilópolis – RJ


Daniel Lopes Alves de Medeiros Fortaleza – CE
Gabriel Ponce Por e-mail
Georges Cobiniano Sousa de Melo João Pessoa – PB
Glauber Moreno Barbosa Rio de Janeiro – RJ
Jônatas de Souza Júnior Recife – PE
Rafael Silva Teresina – PI
Raphael Constant da Costa Rio de Janeiro – RJ

Seguimos aguardando o envio de soluções dos problemas propostos Nos. 89, 97, 98 e 100.

EUREKA! N°21, 2005

61
Sociedade Brasileira de Matemática

COORDENADORES REGIONAIS
Alberto Hassen Raad (UFJF) Juiz de Fora – MG
Américo López Gálvez (USP) Ribeirão Preto – SP
Amarísio da Silva Araújo (UFV) Viçosa – MG
Ana Paula Bernardi da Silva (Universidade Católica de Brasília) Brasília – DF
Antonio Carlos Nogueira (UFU) Uberlândia – MG
Ali Tahzibi (USP) São Carlos – SP
Benedito Tadeu Vasconcelos Freire (UFRN) Natal – RN
Carlos Frederico Borges Palmeira (PUC-Rio) Rio de Janeiro – RJ
Claus Haetinger (UNIVATES) Lajeado – RS
Cleonor Crescêncio das Neves (UTAM) Manaus – AM
Cláudio de Lima Vidal (UNESP) S.J. do Rio Preto – SP
Edson Roberto Abe (Colégio Objetivo de Campinas) Campinas – SP
Élio Mega (Colégio Etapa) São Paulo – SP
Éder Luiz Pereira de Andrade (UNESPAR/FECILCAM) Campo Mourão – PR
Eudes Antonio da Costa (Univ. do Tocantins) Arraias – TO
Florêncio Ferreira Guimarães Filho (UFES) Vitória – ES
Ivanilde Fernandes Saad (UC. Dom Bosco) Campo Grande– MS
Jacqueline Fabiola Rojas Arancibia (UFPB) João Pessoa – PB
Janice T. Reichert (UNOCHAPECÓ) Chapecó – SC
João Benício de Melo Neto (UFPI) Teresina – PI
João Francisco Melo Libonati (Grupo Educacional Ideal) Belém – PA
José Carlos dos Santos Rodrigues (Unespar) Campo Mourão – PR
José Cloves Saraiva (UFMA) São Luis – MA
José Gaspar Ruas Filho (ICMC-USP) São Carlos – SP
José Luiz Rosas Pinho (UFSC) Florianópolis – SC
José Vieira Alves (UFPB) Campina Grande – PB
Krerley Oliveira (UFAL) Maceió – AL
Licio Hernandes Bezerra (UFSC) Florianópolis – SC
Luzinalva Miranda de Amorim (UFBA) Salvador – BA
Mário Rocha Retamoso (UFRG) Rio Grande – RS
Marcelo Rufino de Oliveira (Grupo Educacional Ideal) Belém – PA
Marcelo Mendes (Colégio Farias Brito, Pré-vestibular) Fortaleza – CE
Marilane de Fraga Sant'Ana FACOS Osório – RS
Pablo Rodrigo Ganassim (Liceu Terras do Engenho) Piracicaba – SP
Ramón Mendoza (UFPE) Recife – PE
Raúl Cintra de Negreiros Ribeiro (Colégio Anglo) Atibaia – SP
Ronaldo Alves Garcia (UFGO) Goiânia – GO
Reginaldo de Lima Pereira (Escola Técnica Federal de Roraima) Boa Vista – RR
Reinaldo Gen Ichiro Arakaki (UNIVAP) SJ dos Campos – SP
Ricardo Amorim (Centro Educacional Logos) Nova Iguaçu – RJ
Sérgio Cláudio Ramos (IM-UFRGS) Porto Alegre – RS
Seme Guevara Neto (UFMG) Belo Horizonte – MG
Tadeu Ferreira Gomes (UEBA) Juazeiro – BA
Tomás Menéndez Rodrigues (U. Federal de Rondônia) Porto Velho – RO
Valdenberg Araújo da Silva (U. Federal de Sergipe) São Cristovão – SE
Valdeni Soliani Franco (U. Estadual de Maringá) Maringá – PR
Vânia Cristina Silva Rodrigues (U. Metodista de SP) S.B. do Campo – SP
Wagner Pereira Lopes (CEFET – GO) Jataí – GO

EUREKA! N°21, 2005

62
CONTEÚDO

AOS LEITORES 2

XXVI OLIMPÍADA BRASILEIRA DE MATEMÁTICA 3


Problemas e Soluções da Primeira Fase

XXVI OLIMPÍADA BRASILEIRA DE MATEMÁTICA 17


Problemas e Soluções da Segunda Fase

XXVI OLIMPÍADA BRASILEIRA DE MATEMÁTICA 29


Problemas e Soluções da Terceira Fase

XXVI OLIMPÍADA BRASILEIRA DE MATEMÁTICA 49


Problemas e Soluções da Primeira Fase - Nível Universitário

XXVI OLIMPÍADA BRASILEIRA DE MATEMÁTICA 55


Problemas e Soluções da Segunda Fase - Nível Universitário

XXVI OLIMPÍADA BRASILEIRA DE MATEMÁTICA 64


Premiados

AGENDA OLÍMPICA 68

COORDENADORES REGIONAIS 69
Sociedade Brasileira de Matemática

AOS LEITORES

O Programa Nacional de Olimpíadas de Matemática tem crescido muito nos últimos


anos, contando, atualmente, com a adesão ao Programa de mais de 5.000 escolas públicas
e privadas de todo o Brasil, o que implica em uma participação na Olimpíada Brasileira de
Matemática de cerca de 250.000 jovens estudantes e seus professores. Além disso, o
Programa Nacional de Olimpíadas de Matemática conta com a colaboração de professores
do ensino básico de todo o Brasil e de professores universitários de mais de 80 instituições
de ensino superior. Eles participam de todas as atividades da Olimpíada Brasileira de
Matemática, em atividades de coordenação, divulgação, treinamento de alunos,
aperfeiçoamento de professores e aplicação das distintas fases da Olimpíada Brasileira de
Matemática.

Em relação à promoção do ensino da Matemática em nível regional, temos


alcançado resultados extremamente positivos: através do apoio a 22 Olimpíadas Regionais
conseguimos atingir um universo de cerca de 150.000 estudantes e seus professores, os
quais são desafiados à resolução de problemas que estimulam o raciocínio e a criatividade.
No que se refere à participação em competições internacionais, o Programa Nacional de
Olimpíadas também tem muito a comemorar. Em 2005 os resultados são excepcionais:
Excelente resultado na Olimpíada de Matemática do Cone Sul (2 medalhas de Ouro, 2
medalhas de Prata); o primeiro estudante Latino-Americano premiado com Medalha de Ouro
Especial (Grand First Prize) e duas outras medalhas de Ouro na Olimpíada Internacional para
Estudantes Universitários (IMC), mais uma Medalha de Ouro na Olimpíada Internacional de
Matemática (IMO) e novamente quatro medalhas de ouro na Olimpíada Ibero-americana de
Matemática.

Todos estes resultados nacionais e internacionais demonstram que, além de


influenciar positivamente o ensino da Matemática em instituições de ensino fundamental,
médio e superior de todo o país, conseguimos detectar jovens muito talentosos que são
estimulados a seguir uma carreira científica, o que é fundamental para o crescimento da
Ciência e Tecnologia no Brasil.

Os editores

EUREKA! N°22, 2005

2
Sociedade Brasileira de Matemática

XXVI OLIMPÍADA BRASILEIRA DE MATEMÁTICA


Problemas e Soluções da Primeira Fase

PROBLEMAS – NÍVEL 1

1. Calcule o valor de 1997 + 2004 + 2996 + 4003.


A) 10000 B) 11000 C) 10900 D) 12000 E) 13000

2. Qual dos números a seguir é ímpar?


A) 7 × 8 B) 37 – 23 C) 9 × 36 D) 144 : 36 E) 17 × 61

3. Quanto é 26 + 26 + 26 + 26 – 44?
A) 0 B) 2 C) 4 D) 42 E) 44

4. O 20% de 40 é igual a
A) 5 B) 8 C) 10 D) 12 E) 20

2004 + 2004
5. Simplificando a fração , obtemos:
2004 + 2004 + 2004
113 1 2 2
A) 2004 B) C) D) E)
355 2004 3 7

6. Os alunos de uma escola participaram de uma excursão, para a qual dois ônibus
foram contratados. Quando os ônibus chegaram, 57 alunos entraram no primeiro
ônibus e apenas 31 no segundo. Quantos alunos devem passar do primeiro para o
segundo ônibus para que a mesma quantidade de alunos seja transportada nos
dois ônibus?
A) 8 B) 13 C) 16 D) 26 E) 31

7. Uma professora tem 237 balas para dar a seus 31 alunos. Qual é o número
mínimo de balas a mais que ela precisa conseguir para que todos os alunos
recebam a mesma quantidade de balas, sem sobrar nenhuma para ela?
A) 11 B) 20 C) 21 D) 31 E) 41

EUREKA! N°22, 2005

3
Sociedade Brasileira de Matemática

8. Dezoito quadrados iguais são construídos e sombreados como mostra a figura.


Qual fração da área total é sombreada?

7 4 1 5 1
A) B) C) D) E)
18 9 3 9 2

9. O preço de uma corrida de táxi é igual a R$2,50 ("bandeirada"), mais R$0,10 por
cada 100 metros rodados. Tenho apenas R$10,00 no bolso. Logo tenho dinheiro
para uma corrida de até:
A) 2,5 km B) 5,0 km C) 7,5 km D) 10,0 km E) 12,5 km

10. Um arquiteto apresenta ao seu cliente cinco plantas diferentes para o projeto de
ajardinamento de um terreno retangular, onde as linhas cheias representam a
cerca que deve ser construída para proteger as flores. As regiões claras são todas
retangulares e o tipo de cerca é o mesmo em todos os casos. Em qual dos
projetos o custo da construção da cerca será maior?

A) B) C) D) E)

11. 108 crianças da 5ª e 6ª séries vão fazer um passeio numa caverna. São formados
grupos iguais com mais de 5 porém menos de 20 alunos. Com relação ao número
de estudantes por grupo, de quantas formas diferentes eles podem ser feitos?
A) 2 B) 8 C) 5 D) 4 E) 3

12. desenho ao lado mostra o mapa de um país


(imaginário) constituído por cinco estados. Deseja-
se colorir esse mapa com as cores verde, azul e
amarela, de modo que dois estados vizinhos não
possuam a mesma cor. De quantas maneiras
diferentes o mapa pode ser pintado?
A) 12 B) 6 C) 10 D) 24 E) 120

EUREKA! N°22, 2005

4
Sociedade Brasileira de Matemática

13. Um artesão começa a trabalhar às 8h e produz 6 braceletes a cada vinte minutos;


seu auxiliar começa a trabalhar uma hora depois e produz 8 braceletes do mesmo
tipo a cada meia hora. O artesão pára de trabalhar às 12h mas avisa ao seu
auxiliar que este deverá continuar trabalhando até produzir o mesmo que ele. A
que horas o auxiliar irá parar?
A) 12h B) 12h30min C) 13h D) 13h30min
E) 14h30min

14. algarismo das unidades do número 1 × 3 × 5 × … × 97 × 99 é


A) 1 B) 3 C) 5 D) 7 E) 9

15. Dois quadrados, cada um com área 25cm2, são colocados lado a lado para formar
um retângulo. Qual é o perímetro do retângulo?
A) 30 cm B) 25 cm C) 50 cm D) 20 cm E) 15 cm

16. Se girarmos o pentágono regular, ao lado, de um ângulo de 252°,


em torno do seu centro, no sentido horário, qual figura será
obtida?

A) B) C) D) E)

17. Os resultados de uma pesquisa das cores de cabelo de 1200 pessoas são
mostrados no gráfico abaixo.

castanho 30% preto 24%

ruivo 16%
loiro

Quantas dessas pessoas possuem o cabelo loiro?


A) 60 B) 320 C) 360 D) 400 B E) 840

EUREKA! N°22, 2005

5
Sociedade Brasileira de Matemática

18. Um cubo pode ser construído, a partir dos dois pedaços de papelão apresentados
em uma das alternativas a seguir, bastando apenas dobrar nas linhas tracejadas e
unir nas linhas contínuas. Esses dois pedaços são:

A) B)

C) D)

E)

19. Ao somar cinco números consecutivos em sua calculadora, Esmeralda encontrou


um número de 4 algarismos: 2 0 0 *. O último algarismo não está nítido, pois o
visor da calculadora está arranhado, mas ela sabe que ele não é zero. Este
algarismo só pode ser:
A) 5 B) 4 C) 3 D) 2 E) 9

20. Sobre uma mesa estão três caixas e três objetos, cada um em uma caixa
diferente: uma moeda, um grampo e uma borracha. Sabe-se que
• A caixa verde está à esquerda da caixa azul;
• A moeda está à esquerda da borracha;
• A caixa vermelha está à direita do grampo;
• A borracha está à direita da caixa vermelha.

Em que caixa está a moeda?


A) Na caixa vermelha.
B) Na caixa verde.
C) Na caixa azul.
D) As informações fornecidas são insuficientes para se dar uma resposta.
E) As informações fornecidas são contraditórias.

EUREKA! N°22, 2005

6
Sociedade Brasileira de Matemática

21. Um feirante vende batatas e, para pesar, utiliza uma balança de dois pratos, um
peso de 1 kg, um peso de 3 kg e um peso de 10 kg. Considere a seguinte
afirmação: “Este feirante consegue pesar (com uma pesagem) n quilogramas de
batatas”. Quantos valores positivos de n tornam essa afirmação verdadeira,
supondo que ele pode colocar pesos nos dois pratos?
A) 7 B) 10 C) 12 D)13 E)14

22. O mapa ao lado mostra um conjunto 1 2 3 4


residencial onde as casas, numeradas, são
interligadas por 23 ruelas. O vendedor Zé 5

Ruela, que mora na casa 8, planeja passar por 6 7 8

todas as outras casas e retornar à sua,


9
percorrendo o menor número possível de
ruelas. Ele deixará de caminhar por quantas 10
11 12
ruelas?
A) 15 B) 10 C) 13 D)12
E) 11

23. O arranjo a seguir, composto por 32 hexágonos, foi montado com varetas, todas
com comprimento igual ao lado do hexágono. Quantas varetas, no mínimo, são
necessárias para montar o arranjo?



A) 113 B) 123 C) 122 D) 132 E) 152

24. Observe a figura:

Duas das figuras abaixo representam o objeto acima colocado em outras posições.
I) II)

EUREKA! N°22, 2005

7
Sociedade Brasileira de Matemática

III) IV)

Elas são:
A) I e II B) I e IV C) II e IV D) I e III E) II e III

25. Entre 1986 e 1989, época em que vocês ainda não tinham nascido, a moeda do
país era o cruzado (Cz$). Com a imensa inflação que tivemos, a moeda foi
mudada algumas vezes: tivemos o cruzado novo, o cruzeiro, o cruzeiro real e,
finalmente, o real. A conversão entre o cruzado e o real é: 1 real = 2.750.000.000
cruzados
Imagine que a moeda não tivesse mudado e que João, que ganha hoje 640 reais
por mês, tivesse que receber seu salário em notas novas de 1 cruzado. Se uma
pilha de 100 notas novas tem 1,5 cm de altura, o salário em cruzados de João
faria uma pilha de altura:
A) 26,4 km B) 264 km C) 26 400 km D) 264 000 km
E) 2 640 000 km

PROBLEMAS – NÍVEL 2

1. Veja o problema No. 3 do Nível 1

2. Se m e n são inteiros não negativos com m < n, definimos m ∇ n como a soma


dos inteiros entre m e n, incluindo m e n. Por exemplo, 5 ∇ 8 = 5 + 6 + 7 + 8 =
26.

22∇ 26
O valor numérico de é:
4∇6
A) 4 B) 6 C) 8 D) 10 E) 12

3. Entre 1986 e 1989, época em que vocês ainda não tinham nascido, a moeda do
país era o cruzado (Cz$). Com a imensa inflação que tivemos, a moeda foi
mudada algumas vezes: tivemos o cruzado novo, o cruzeiro, o cruzeiro real e,
finalmente, o real. A conversão entre o cruzado e o real é:

EUREKA! N°22, 2005

8
Sociedade Brasileira de Matemática

1 real = 2.750.000.000 cruzados

Imagine que a moeda não tivesse mudado e que João, que ganha hoje 640 reais
por mês, tivesse que receber seu salário em notas novas de 1 cruzado. Se uma
pilha de 100 notas novas tem 1,5 cm de altura, o salário em cruzados de João
faria uma pilha de altura:
A) 26,4km
B) 264km
C) 26400km
D) 264000km
E) 2640000km

4. Veja o problema No. 23 do Nível 1.


5. Veja o problema No. 14 do Nível 1.
6. Veja o problema No. 16 do Nível 1.

7. Há 1002 balas de banana e 1002 balas de maçã numa caixa. Lara tira, sem olhar
o sabor, duas balas da caixa. Seja p a probabilidade de as duas balas serem do
mesmo sabor e seja q a probabilidade de as duas balas serem de sabores
diferentes. Quanto vale a diferença entre p e q?
1 1 2 1
A) 0 B) C) D) E)
2004 2003 2003 1001
8. O perímetro de um retângulo é 100 e a diagonal mede x. Qual é a área do
retângulo?
x2 x2
A) 625 – x2 B) 625 – C) 1250 –
2 2
x2 x2
D) 250 – E) 2500 –
2 2
9. Veja o problema No. 19 do Nível 1.

2004
10. Para quantos inteiros positivos m o número é um inteiro positivo?
m2 − 2
A) um B) dois C) três D) quatro
E) mais do que quatro

11. Se x + y = 8 e xy = 15, qual é o valor de x2 + 6xy + y2?


A) 64 B) 109 C) 120 D) 124 E) 154

EUREKA! N°22, 2005

9
Sociedade Brasileira de Matemática

12. Dois espelhos formam um ângulo de 30o no ponto V. Um raio de luz, vindo de
uma fonte S, é emitido paralelamente a um dos espelhos e é refletido pelo outro
espelho no ponto A, como mostra a figura. Depois de uma certa quantidade de
reflexões, o raio retorna a S. Se AS e AV têm 1 metro de comprimento, a
distância percorrida pelo raio de luz, em metros, é

S A

30°
V

A) 2 B) 2 + 3 C) 1 + 2 + 3 D) (
2 1+ 3 )
E) 5 3

13. Na figura, quanto vale x?


A) 6° B) 12° C) 18° 5x
D) 20° E) 24°

3x 2x
6x

4x

14. Se 2(22x) = 4x + 64, então x é igual a:


A) – 2 B) – 1 C) 1 D) 2 E) 3

15. Qual é o maior valor da soma dos algarismos da soma dos algarismos de um
número de três algarismos?
A) 7 B) 8 C) 9 D) 10 E) 11

16. Veja o problema No. 10 do Nível 1.

17. Um ponto P pertence ao interior de um quadrado com 10 cm de lado. No


máximo, quantos pontos da borda do quadrado podem estar a uma distância de 6
cm do ponto P?
A) 1 B) 2 C) 4 D) 6 E) 8

EUREKA! N°22, 2005

10
Sociedade Brasileira de Matemática

18. Veja o problema No. 18 do Nível 1.

19. No triângulo PQR, a altura PF divide o lado QR em dois segmentos de medidas


QF = 9 e RF = 5. Se PR = 13, qual é a medida de PQ?
A) 5 B) 10 C) 15 D) 20 E) 25

20. Veja o problema No. 20 do Nível 1.

21. No desenho ao lado, o quadrilátero ABCD é um A


quadrado de lado 3 cm e os triângulos ABF e E
AED são ambos equiláteros. Qual é a área da
B
região destacada?
A) 2 cm2
B) 1,5 cm2 D
F
A) 3 cm2
D) 4,5 cm2 C
E) 2,5 cm2

22. Uma folha quadrada foi cortada em 42 quadrados menores, dos quais um tem
área maior do que 1 cm2 e os demais têm área de 1 cm2. Qual é a medida do
lado da folha?
A) 6 cm B) 12 cm C) 21 cm D) 19 cm E) 20 cm

23. Eu planejava fazer um curral quadrado, com uma certa área, usando uma certa
quantidade de cerca de arame farpado. Descobri, porém, que tenho 10% a
menos de cerca do que esperava. Por esta razão, a área cercada será:
A) 5% menor B) 10% menor C) 19% menor D) 20% menor
E) 25% menor

24. Veja o problema No. 13 do Nível 1.

20. Esmeralda, a digitadora, tentou digitar um número de seis algarismos, mas os


dois algarismos 1 não apareceram (a tecla devia estar com defeito). O que
apareceu foi 2004. Quantos são os números de seis algarismos que ela pode ter
tentado digitar?
A) 4 B) 8 C) 10 D) 15 E) 20

EUREKA! N°22, 2005

11
Sociedade Brasileira de Matemática

PROBLEMAS – NÍVEL 3

1. A função f é dada pela tabela a seguir.

1 2 3 4 5
f(x) 4 1 3 5 2

Por exemplo, f(2) = 1. Quanto vale f ( f (...( f ( f (4))...)) ?


2004 vezes
A) 1 B) 2 C) 3 D) 4 E) 5

2. Seja AB um segmento de comprimento 26, e sejam C e D pontos sobre o


segmento AB tais que AC = 1 e AD = 8. Sejam E e F pontos sobre uma
semicircunferência de diâmetro AB, sendo EC e FD perpendiculares a AB.
Quanto mede o segmento EF?
A) 5 B) 5 2 C) 7 D) 7 2 E) 12

3. As alturas de um triângulo medem 12, 15 e 20. O maior ângulo interno do


triângulo mede
A) 72o B) 75o C) 90o D) 108o E) 120o

4. Esmeralda, a digitadora, tentou digitar um número de seis algarismos, mas os


dois algarismos 1 não apareceram (a tecla devia estar com defeito). O que
apareceu foi 2004. Quantos são os números de seis algarismos que ela pode ter
tentado digitar?
A) 4 B) 8 C) 10 D) 15 E) 20

5. O produto dos números que aparecem nas alternativas incorretas dessa questão é
um cubo perfeito.
Assinale a alternativa correta.
A) 4 B) 8 C) 18 D) 54 E) 192

6. Qual é o menor inteiro positivo n para o qual qualquer subconjunto de n


elementos de {1,2,3,…,20} contém dois números cuja diferença é 8?
A) 2 B) 8 C) 12 D) 13 E) 15

EUREKA! N°22, 2005

12
Sociedade Brasileira de Matemática

7. Sejam
12 2 2 3 2
 + 1001
2
a= + + +
1 3 5 2001
e
12 2 2 3 2
 + 1001
2
b= + + + .
3 5 7 2003

Qual é o inteiro mais próximo de a – b?


A) 500 B) 501 C) 999 D) 1000 E) 1001

8. Uma ampulheta é formada por dois cones idênticos. Inicialmente, o cone


superior está cheio de areia e o cone inferior está vazio. A areia flui do cone
superior para o inferior com vazão constante. O cone superior se esvazia em
exatamente uma hora e meia. Quanto tempo demora até que a altura da areia no
cone inferior seja metade da altura da areia no cone superior?
A) 30min
B) 10h
C) 1h03min20s
D) 1h10min12s
E) 1h14min30s

9. A função real f, definida nos inteiros, satisfaz f(n) – (n + 1)f(2 – n) = (n + 3)2,


para todo n inteiro. Quanto vale f(0)?
A) –17 B) 0 C) 1 D) 2 E) 9

10. Com três algarismos distintos a, b e c, é possível formar 6 números de dois


algarismos distintos. Quantos conjuntos {a, b, c} são tais que a soma dos 6
números formados é 484?
A) Um B) Dois C) Três D) Quatro
E) Mais que quatro

11. Dois cubos têm faces pintadas de ocre ou magenta. O primeiro cubo tem cinco
faces ocres e uma face magenta. Quando os dois cubos são lançados, a
probabilidade de as faces viradas para cima dos dois cubos serem da mesma cor
(sim, ocre e magenta são cores!) é 1/2. Quantas faces ocres tem o segundo cubo?
A) 1 B) 2 C) 3 D) 4 E) 5

12. Veja o problema No. 10 do Nível 2.


13. Veja o problema No. 12 do Nível 2.

EUREKA! N°22, 2005

13
Sociedade Brasileira de Matemática

14. Para n inteiro positivo, definimos n! (lê-se “n fatorial”) o produto de todos os


inteiros positivos menores ou iguais a n. Por exemplo, 6! = 1 ⋅ 2 ⋅ 3 ⋅ 4 ⋅ 5 ⋅ 6.
Se n! = 215 ⋅ 36 ⋅ 53 ⋅ 72 ⋅ 11 ⋅ 13, então n é igual a
A) 13 B) 14 C) 15 D) 16 E) 17

15. Constrói-se o quadrado ABXY sobre o lado AB do heptágono regular ABCDEFG,


exteriormente ao heptágono. Determine a medida do ângulo BXC ˆ , em
radianos.
π 3π π 3π 3π
A) B) C) D) E)
7 7 14 14 28

16. O conjunto das raízes reais da equação x + 2 x − 1 + x − 2 x − 1 = 2 é


A) {1} B) {1, 2} C) [1, 2] D) ]1, 2[ E) {2}

17. No desenho ao lado, os segmentos AB e CD são D


perpendiculares ao segmento BC . Sabendo que o
ponto M pertence ao segmento AD e que o M
triângulo BMC é retângulo não isósceles, qual é a A 4

área do triângulo ABM ? 2


6 7 8 9
A) 1 B) C) D) E) B 6 C
5 5 5 5

18. Veja o problema No. 3 do Nível 2

19. O dono de uma loja empilhou vários blocos medindo 0,8m x 0,8m x 0,8m no
canto da loja e encostados numa parede de vidro que dá para a rua, conforme
mostra a figura abaixo.
Quantos blocos no máximo, uma pessoa de 1,80m de altura que está do lado de
fora da loja pode enxergar?

Obs. Consideramos que uma pessoa pode enxergar uma caixa se consegue ver
uma pequena região de área positiva de sua superfície.

EUREKA! N°22, 2005

14
Sociedade Brasileira de Matemática

A) 13 B) 14 C) 15 D) 16 E) 17

20. Veja o problema No. 4 do Nível 2.

21. Numa prova para uma sala com 30 alunos, a média aritmética das 10 piores
notas é 3 e a média aritmética das 10 melhores notas é 9. O menor valor possível
e o maior valor possível para a média da sala são, respectivamente:
A) 6 e 7 B) 5 e 7 C) 4 e 6 D) 3 e 9 E) 4 e 8

22. Veja o problema No. 20 do Nível 1.


23. Veja o Problema No. 25 do Nível 2.

24. Esmeralda escreveu (corretamente!) todos os números de 1 a 999, um atrás do


outro:
12345678910111213… 997998999.
Quantas vezes aparece o agrupamento “21”, nesta ordem?
A) 11 B) 21 C) 31 D) 41 E) 51

25. Um feirante vende batatas e, para pesar, utiliza uma balança de dois pratos, um
peso de 1 kg, um peso de 3 kg e um peso de 10 kg. Considere a seguinte
afirmação: “Este feirante consegue pesar (com uma pesagem) n quilogramas de
batatas”. Quantos valores positivos de n tornam essa afirmação verdadeira,
supondo que ele pode colocar pesos nos dois pratos?
A) 7 B) 10 C) 12 D)13 E)14

EUREKA! N°22, 2005

15
Sociedade Brasileira de Matemática

GABARITO
NÍVEL 1 (5a. e 6a. séries)

1) B 6) B 11) D 16) B 21) D


2) E 7) A 12) B 17) C 22) E
3) A 8) B 13) D 18) E 23) B
4) B 9) C 14) C 19) A 24) C
5) D 10) C 15) A 20) A 25) D

NÍVEL 2 (7a. e 8a. séries)

1) A 6) B 11) D 16) C 21) D


2) C 7) C 12) B 17) E 22) C
3) D 8) C 13) C 18) E 23) C
4) B 9) A 14) E 19) C 24) D
5) C 10) B 15) D 20) A 25) D

NÍVEL 3 (Ensino Médio)

1) D 6) D 11) C 16) C 21) B


2) D 7) B 12) B 17) B 22) A
3) C 8) C 13) B 18) E 23) D
4) D 9) A 14) D 19) B 24) C
5) D 10) B 15) E 20) B 25) D

EUREKA! N°22, 2005

16
Sociedade Brasileira de Matemática

XXVI OLIMPÍADA BRASILEIRA DE MATEMÁTICA


Problemas e Soluções da Segunda Fase

PROBLEMAS – NÍVEL 1 PARTE A


(Cada problema vale 3 pontos)

01. O número 1000…02 tem 20 zeros. Qual é a soma dos algarismos do número que
obtemos como quociente quando dividimos esse número por 3?

02. A soma de dois números primos a e b é 34 e a soma dos primos a e c é 33.


Quanto vale a + b + c?

03. No desenho, os quadriláteros ABCD, EFAG


A I F B
e IAJH são retângulos e H é ponto médio de
AE.
Calcule a razão entre a área do retângulo ABCD J H
e o triângulo AHI.
G
E

D C
04. Dizemos que um número natural é teimoso se, ao ser elevado a qualquer
expoente inteiro positivo, termina com o mesmo algarismo. Por exemplo, 10 é
teimoso, pois 102 ,103 ,104 ,..., são números que também terminam em zero. Quantos
números naturais teimosos de três algarismos existem?

05. Qual é o maior número natural menor que 100 cuja soma dos divisores positivos
é ímpar?

06. Na multiplicação a seguir, a, b e c são algarismos:

1 a b
b 3 ×
* * *
* * *
1c c 0 1
Calcule a + b + c.

EUREKA! N°22, 2005

17
Sociedade Brasileira de Matemática

07. Esmeralda, de olhos vendados, retira cartões de uma urna contendo inicialmente
100 cartões numerados de 1 a 100, cada um com um número diferente. Qual é o
número mínimo de cartões que Esmeralda deve retirar para ter certeza de que o
número do cartão seja um múltiplo de 4?

08. De quantos modos podemos sombrear quatro casas do tabuleiro 4 × 4 abaixo de


modo que em cada linha e em cada coluna exista uma única casa sombreada?

09. Juntando cubinhos de mesmo volume mas feitos de materiais diferentes - cada
cubo branco pesando 1 grama e cada cubo cinza pesando 2 gramas - formou-se um
bloco retangular, conforme mostrado na figura abaixo. Qual é a massa, em gramas,
desse bloco?

10. Na população de uma espécie rara de 1000 aves da floresta amazônica, 98%
tinham cauda de cor verde. Após uma misteriosa epidemia que matou parte das aves
com cauda verde, esta porcentagem caiu para 95%. Quantas aves foram eliminadas
com a epidemia?

EUREKA! N°22, 2005

18
Sociedade Brasileira de Matemática

PROBLEMAS – NÍVEL 1 PARTE B


(Cada problema vale 10 pontos)

PROBLEMA 1:
No desenho abaixo, o triângulo ABC é retângulo e os lados do polígono (região
escura) são paralelos ou coincidem com algum dos catetos do triângulo.
5 10
A

B C
Calcule x de modo que a área do polígono seja igual à do triângulo.

PROBLEMA 2:
Esmeralda, a digitadora, construiu uma tabela com 100 linhas e 100 colunas,
preenchendo uma casa com 1, se o número da linha da casa divide o número da
coluna e com 0, caso contrário. Assim, por exemplo, a casa da linha 2 e da coluna 4
foi preenchida com 1, porque 2 divide 4 e a casa na linha 3 e da coluna 7 foi
preenchida com 0.
1 2 3 4 5 6 … 99 100
1 1 1 1 1 1 1 1 1
2 0 1 0 1 0 1 … 0 1

3 0 0 1 0 0 1 … 1 0

100 0 0 0 1

a) Qual a soma dos números escritos na linha 5?


b) Qual a soma dos números da coluna 60?

PROBLEMA 3:
a) É possível dividir o conjunto {12, 22,…,72} em dois grupos A e B de modo que a
soma dos elementos de A seja igual à soma dos elementos de B? Justifique.

b) É possível dividir o conjunto {12, 22, 32,…,92} em dois grupos C e D de modo


que a soma dos elementos de C seja igual à soma dos elementos de D?
Justifique.

EUREKA! N°22, 2005

19
Sociedade Brasileira de Matemática

PROBLEMAS – NÍVEL 2 PARTE A


(Cada problema vale 3 pontos)

01. Veja o problema No. 6 do Nível 1.


02. Veja o problema No. 8 do Nível 1.

03. Qual é a soma dos algarismos do número 2004 × 2002 × 1998 × 1996 + 36 ?

04. Veja o problema No. 1 da Parte B do Nível 1.

05. Um polígono com 20 lados é chamado icoságono. Unindo-se três dos vértices de
um icoságono regular obtemos triângulos. Quantos são triângulos retângulos?

PROBLEMAS – NÍVEL 2 PARTE B


(Cada problema vale 10 pontos)

PROBLEMA 1:
(a) É possível dividir o conjunto {12, 22,…,72} em dois grupos A e B de modo que a
soma dos elementos de A seja igual à soma dos elementos de B? Justifique.
(b) É possível dividir o conjunto {12, 22, 32,…,92} em dois grupos C e D de modo
que a soma dos elementos de C seja igual à soma dos elementos de D?
Justifique.

PROBLEMA 2:
(a) Simplifique a expressão

(b) Certa calculadora tem duas teclas especiais: A e B. A tecla A transforma o


número x que está no visor em 1 . A tecla B transforma o número x que está no
x
visor em 1 − x .
Pedro tem um número no visor e aperta sucessivamente, de forma alternada, as duas
teclas:
A, B, A, B, ….
Após 1000 operações, o visor mostrava o número 2004. Que número Pedro tinha
inicialmente no visor?

EUREKA! N°22, 2005

20
Sociedade Brasileira de Matemática

PROBLEMA 3:
Uma folha de papel retangular ABCD foi
dobrada de modo que o vértice B foi levado no
ponto B’ sobre o lado AD. A dobra é EF, com E
sobre AB e F sobre CD.

Sabe-se que AE = 8, BE = 17 e C F = 3.

(a) Calcule a medida do segmento AB’.


(b) Calcule a medida do lado AD.

PROBLEMA 4:
Um número de 4 algarismos a b c d é chamado de legal quando a soma dos números
formados pelos dois primeiros e pelos dois últimos algarismos é igual ao número
formado pelos algarismos centrais (ou seja, ab + cd = bc). Por exemplo, 2307 é um
número legal pois 23 + 07 = 30.

(a) Qual é o menor número legal maior do que 2307?


(b) Quantos são os números legais de 4 algarismos?

PROBLEMAS – NÍVEL 3

PROBLEMA 1:
Cada um dos números x1 , x 2 , ..., x 20 04 pode ser igual a 2 − 1 ou a 2 +1.
Quantos valores inteiros distintos a soma
2004

∑k =1
x 2 k − 1 x 2 k = x1 x 2 + x 3 x 4 + x 5 x 6 + ... + x 2003 x 2 004 pode assumir?

PROBLEMA 2
Seja ABCD um trapézio retângulo de bases AB e CD, com ângulos retos em A e D.
Dado que a diagonal menor BD é perpendicular ao lado BC, determine o menor valor
possível para a razão C D .
AD

EUREKA! N°22, 2005

21
Sociedade Brasileira de Matemática

PROBLEMA 3:
Os doze alunos de uma turma de olimpíada saíam para jogar futebol todos os dias
após a aula de matemática, formando dois times de 6 jogadores cada e jogando entre
si. A cada dia eles formavam dois times diferentes dos times formados em dias
anteriores. Ao final do ano, eles verificaram que cada 5 alunos haviam jogado juntos
num mesmo time exatamente uma vez. Quantos times diferentes foram formados ao
longo do ano?

PROBLEMA 4:
Determine todas as soluções da equação n ⋅ 2 n −1 + 1 = m 2 , com n e m naturais.

PROBLEMA 5:
Dizemos que um número inteiro positivo é sinistro quando a soma de seus fatores
primos é igual à soma dos expoentes de sua decomposição em fatores primos.
Encontre todos os números sinistros de quatro algarismos.

PROBLEMA 6:
Sejam H, I e O o ortocentro, o incentro e o circuncentro do triângulo ABC,
respectivamente. A reta CI corta o circuncírculo de ABC no ponto L, distinto de C.
Sabe-se que AB = IL e AH = OH. Determine os ângulos do triângulo ABC.

EUREKA! N°22, 2005

22
Sociedade Brasileira de Matemática

SOLUÇÕES – SEGUNDA FASE – NÍVEL 1 – PARTE A

Problema 01 02 03 04 05 06 07 08 09 10
Resposta 064 036 032 360 098 010 076 024 262 600

SOLUÇÕES – SEGUNDA FASE – NÍVEL 1 – PARTE B


SOLUÇÃO DO PROBLEMA 1:
O polígono consiste na reunião de dois retângulos: um deles tem largura 10 e altura 2
e o outro tem largura 5 e altura x + 2 ; o triângulo tem catetos de medidas 15 e
x + 2 . Como a área do polígono é igual à área do triângulo, temos
15( x + 2)
10 ⋅ 2 + 5( x + 2) = ⇔ 40 + 10 x + 20 = 15 x + 30 ⇔ 5 x = 30 ⇔ x = 6
2

SOLUÇÃO DO PROBLEMA 2:
a) Cada linha apresenta 1 nas colunas cujos números são múltiplos do número da
linha. Assim, a linha 5 tem 1 nas colunas 5, 10, 15, etc. Até 100, existem 20
múltiplos de 5, logo a soma dos números na linha 5 é igual a 20.
b) Cada coluna apresenta 1 no cruzamento com as linhas cujos números são
divisores do número da coluna. Assim, a soma dos números da coluna 60 é igual ao
número de divisores de 60. Como 60 = 2 2 × 3 × 5 , concluímos que 60 tem 3.2.2 =
12 divisores. Logo, a soma dos números da coluna 60 é 12.

SOLUÇÃO DO PROBLEMA 3:
a) A soma total dos elementos é
12 + 22 + 32 + 42 + 52 + 62 + 72 = 1 + 4 + 9 + 16 + 25 + 36 + 49 = 140 .
Logo, cada um dos grupos deve conter elementos que somem 70. Examinando as
parcelas, vemos que 49 + 1 + 4 +16 = 70. Assim podemos escrever, por exemplo, A
= {12, 22, 42, 72} e B = {32, 52, 62}.
b) Como
12 + 22 + 32 + 42 + 52 + 62 + 72 + 82 + 92 = 140 + 64 + 81 = 285
é ímpar, é impossível dividir em dois grupos de mesma soma.

SOLUÇÕES – SEGUNDA FASE – NÍVEL 2 – PARTE A

Problema 01 02 03 04 05
Resposta 010 024 048 006 180

EUREKA! N°22, 2005

23
Sociedade Brasileira de Matemática

SOLUÇÕES – SEGUNDA FASE – NÍVEL 2 – PARTE B


SOLUÇÃO DO PROBLEMA 1:Ver o problema 3 – Parte B do Nível 1

SOLUÇÃO DO PROBLEMA 2:
1 1 1 1 x −1
a) 1 − = 1− = 1− = 1− = 1− =1+ x −1 = x
1 1 x x −1− x −1
1− 1− 1−
1 x −1 x −1 x −1
1−
x x
x −1 − x −1 2003
⇔ = 2004 ⇔ = 2004 ⇔ 2004 x − 2004 = −1 ⇔ x =
x −1 x −1 2004

SOLUÇÃO DO PROBLEMA 3:
a) A partir da dobra da folha podemos ver que B'E = BE = 17, e como AE = 8,
aplicando o teorema de Pitágoras temos AB´= B´E 2 − AE 2 = 172 − 82 = 15 .

SOLUÇÃO DO PROBLEMA 4:
Como ab + cd = bc ⇔ 10a + b + 10c + d = 10b + c ⇔ 10a + d = 9(b − c ) , ou seja,
10a + d é o número de dois algarismo a d , e é um múltiplo de 9.

a) Mantendo a = 2, temos d = 7. Além disso, 10 ⋅ 2 + 7 = 9(b − c) ⇔ b − c = 3 . O


menor valor de b que podemos escolher, após 3, é 4, e nesse caso, c = 1. O número
procurado é, então, 2417.

b) Uma vez que escolhemos b − c , a e d estão determinados: a é o algarismo das


dezenas de 9(b − c) , e d, o das unidades. Além disso, 9(b − c) ≥ 10 ⇔ b − c ≥ 2 .
Se b − c = 2 , (b, c ) ∈ {( 2,0 ); (3,1); ( 4, 2 );...; (9,0 )} , um total de 8 possibilidades. Da
mesma forma, vemos que se b − c = 3 , (b, c ) ∈ {(3,0 ); ( 4,1); (5, 2 );...; (9,6 )} , há um
total de 7 possibilidades. Para b − c = 4 , (b, c ) ∈ {( 4,0 ); (5,1); (6, 2 );...; (9,5 )} , 6
possibilidades, b − c = 5 ,
(b, c ) ∈ {(5,0 ); (6,1); (7, 2 );...; (9, 4 )} , 5 possibilidades, b − c = 6 ,
(b, c ) ∈ {(6,0 ); ( 7,1); (8,2 ); (9,3)} , 4 possibilidades, b − c = 7 ,
(b, c ) ∈ {(7,0 ); (8,1); (9,2 )} , 3 possibilidades, b − c = 8 , (b, c ) ∈ {(8,0 ); (9,1)} , 2
possibilidades e, finalmente, para b − c = 9 , (b, c ) = (9,0 ) , 1 possibilidade.
Há, portanto, um total de 8 + 7 + 6 + 5 + 4 + 3 + 2 + 1 = 36 números legais.

EUREKA! N°22, 2005

24
Sociedade Brasileira de Matemática

SOLUÇÕES – SEGUNDA FASE – NÍVEL 3

SOLUÇÃO DO PROBLEMA 1:
Os possíveis produtos x2 k −1 x2 k são ( 2 − 1)( 2 − 1) = 3 − 2 2,
( 2 + 1)( 2 + 1) = 3 + 2 2 e ( 2 − 1)( 2 + 1) = 1. Suponha que a produtos são
iguais a 3 − 2 2, b produtos são iguais a 3 + 2 2 e 1002 − a − b produtos são iguais
a 1.

A soma é igual a
a(3 − 2 2) + b(3 + 2 2) + 1002 − a − b = 1002 + 2a + 2b + 2(b − a) 2.

Assim, para que a soma seja inteira, devemos ter a = b. Logo a soma é igual a
1002 + 4a.

Como a varia de 0 a 501 (pois a + b não pode ser maior que 1002), a soma pode
assumir 502 valores inteiros.

SOLUÇÃO DO PROBLEMA 2:

l = BDC = α . Então DC = BD
Seja ABD e AD = BD sen α , donde
cos α
BD
= cos α =
DC 1 2
= ≥ 2.
AD BD sen α senα cos α sen2α

A igualdade ocorre quando sen 2α = 1, ou seja, quando α = 45°.

EUREKA! N°22, 2005

25
Sociedade Brasileira de Matemática

SEGUNDA SOLUÇÃO DO PROBLEMA 2:

Sejam H a projeção de B sobre DC, DH = m, HC = n e BH = h. ABDH é então um


retângulo, donde AD = BH = h.
Como o triângulo CBD é retângulo, temos h = mn. Logo,
2

DC DC m + n m + n
= = = .
AD BH h mn
Mas sabemos que ( m − n ) ≥ 0, donde m + n ≥ 2 mn . A igualdade ocorre
2

quando m = n.
Segue que
DC m + n
= ≥ 2.
AD mn
SOLUÇÃO DO PROBLEMA 3:
Para cada grupo de 5 alunos, existe um único time formado que os contém. Logo,
12  12 ⋅11 ⋅10 ⋅ 9 ⋅ 8
contamos   = = 792 times para cada 5 alunos escolhidos. Por
5 5!
 6
outro lado, em cada time de 6 jogadores, temos   = 6 modos de escolhermos
 5
cinco jogadores, ou seja, existem 6 grupos de 5 jogadores que geram o mesmo time
792
na nossa primeira contagem. Logo, o total de times formados é igual a = 132 .
6

SEGUNDA SOLUÇÃO DO PROBLEMA 3:


12 
Há   maneiras de escolher 6 dentre 12 alunos. Além disso, fixados 5 alunos, há 7
6
maneiras de montar um time com esses 5 alunos mais outro aluno. Assim,
considerando que cada 5 alunos jogaram juntos num mesmo time exatamente uma
vez, o total de maneiras de escolher 6 dentre 12 alunos é igual a 7 vezes o número de

EUREKA! N°22, 2005

26
Sociedade Brasileira de Matemática

maneiras de formar os times ao longo do ano. Logo o número de maneiras de formar


1 12  12 ⋅ 11 ⋅ 10 ⋅ 9 ⋅ 8 ⋅ 7
os times ao longo do ano é   = = 132 .
7  6  7 ⋅ 6 ⋅ 5 ⋅ 4 ⋅ 3 ⋅ 2 ⋅1

SOLUÇÃO DO PROBLEMA 4:
A equação é equivalente a n ⋅ 2n −1 = ( m − 1)( m + 1). Suponha n > 3. Temos m ímpar,
digamos m = 2k +1. A equação fica então n ⋅ 2n −3 = k ( k + 1). Portanto, 2n−3 divide k
ou k + 1, pois k ou k + 1 (o que for ímpar) divide n. Assim, k + 1 ≥ 2n −3 e
k ≤ n, donde n + 1 ≥ 2 n −3 .

Mostremos, por indução, que n + 1 < 2 n −3 para n > 5. Para n = 6 (base de indução),
temos 6 + 1 = 7 e 26 – 3 = 8. Supondo que a desigualdade é válida para n = k,
provemos que a mesma é válida para n = k + 1 (passo indutivo). De fato, temos k + 1
< 2k – 3 ⇔ 2(k + 1) < 2k – 2. Como k + 2 < 2(k + 1), temos k + 2 < 2k – 2, completando a
demonstração.

Assim, basta testar 0 ≤ n ≤ 5 . Portanto as soluções são (m; n) = (1;0) e (m; n) =


(9;5).

SOLUÇÃO DO PROBLEMA 5:
Seja n = p1α1 p2α2 ... pkαk , (com p1 < p2 < ... < pk primos) um número sinistro. Como
pi ≥ 2 para todo i, n ≥ 2α1 +α2 +...+αk .
Como n tem 4 algarismos, n < 10000, donde 2α1 +α2 +...+αk ≤ n < 10000 , e logo
α1 + α 2 + ... + α k ≤ 13.
Se um dos fatores primos fosse maior ou igual a 11, a soma dos fatores primos seria
≥ 11 , donde α1 + α 2 + ... + α k ≥ 11 e n ≥ 210 ⋅ 11 > 10000 , absurdo.
Assim, os únicos fatores primos possíveis são 2, 3, 5 e 7. Como 35 < 1000, se a soma
dos expoentes for ≤ 5, o número deve ser 55 = 3125 . A soma pode ser também igual
a 7, donde o número pode ser 24 ⋅ 53 = 2000, ou 23 ⋅ 54 = 5000 (note que
77 > 10000 ).
Não pode ser igual a 8 = 3 + 5 , pois 37 ⋅ 5 > 10000.
Pode ser igual a 9 = 7 + 2, podendo o número ser igual a 28 ⋅ 7 = 1792 ou
27 ⋅ 72 = 6272.

EUREKA! N°22, 2005

27
Sociedade Brasileira de Matemática

Pode ser igual a 10 = 2 + 3 + 5, podendo o número ser igual a


28 ⋅ 3 ⋅ 5 = 3840, 27 ⋅ 32 ⋅ 5 = 5760, 27 ⋅ 3 ⋅ 52 = 9600 ou 26 ⋅ 33 ⋅ 5 = 8640 (note que os
fatores primos não podem ser 3 e 7, pois 39 ⋅ 7 > 10000).
A soma não pode ser 11, nem 12 (pois 210 ⋅ 3 ⋅ 7 e 511 ⋅ 7 são maiores que 10000)
nem 13. Assim os números sinistros de quatro algarismos são 55 = 3125 ,
24 ⋅ 53 = 2000, 23 ⋅ 54 = 5000, 28 ⋅ 7 = 1792, 27 ⋅ 72 = 6272,
28 ⋅ 3 ⋅ 5 = 3840, 27 ⋅ 32 ⋅ 5 = 5760, 27 ⋅ 3 ⋅ 52 = 9600 e 26 ⋅ 33 ⋅ 5 = 8640.

SOLUÇÃO DO PROBLEMA 6:
Sendo α, β e γ as medidas dos ângulos internos nos vértices A, B e C,
α +β
respectivamente, temos m(∠IBL) = m(∠BIL) = . Logo BL = IL, e como BL
2
= ΑL e IL = AB, concluímos que o triângulo ABL é equilátero, logo o arco AB mede
60o e, portanto, m(∠ACB) = 120o.

O quadrilátero CXHY é inscritível, onde X e Y são os pés das alturas traçadas de A e


B. Logo ∠AHB mede 180o − 120o = 60o. Como m(∠AOB) = 120o, concluímos que o
quadrilátero OAHB é inscritível. (Isto também pode ser provado, por exemplo,
utilizando-se a propriedade de que o simétrico de H em relação a AB pertence ao
circuncírculo de ABC).
Isto implica que m(∠AHO) = m(∠ABO) = 30o, e como OH = AH, temos m(∠AOH) =
m(∠OAH) = 75o.
Finalmente, temos m(∠BAC) = m(∠OAH) − m(∠OAB) − m(∠XAH) = 75o − 30o −
30o = 15o; e m(∠ABC) = 180o − 120o − 15o = 45o.
L

A B

C
X

EUREKA! N°22, 2005

28
Sociedade Brasileira de Matemática

XXVI OLIMPÍADA BRASILEIRA DE MATEMÁTICA


Problemas e Soluções da Terceira Fase
PROBLEMAS – NÍVEL 1

PROBLEMA 1:
Encontre todos os números naturais n de três algarismos que possuem todas as
propriedades abaixo:
• n é ímpar;
• n é um quadrado perfeito;
• A soma dos quadrados dos algarismos de n é um quadrado perfeito.

PROBLEMA 2:
Com quatro triângulos eqüiláteros
de lado 1 é possível formar uma
peça, no formato de um triângulo
eqüilátero de lado 2, como mostra
a figura ao lado.

Imagine que você tenha muitos triângulos eqüiláteros de lado 1 de três tipos:
brancos, pretos e cinzas para formar peças como no exemplo acima. Duas peças
assim formadas são consideradas iguais quando podemos obter uma delas girando a
outra, conforme ilustrado abaixo, à esquerda.

Par de peças iguais Par de peças diferentes


Quantas peças diferentes podem ser formadas nas condições apresentadas?

PROBLEMA 3:
Dizemos que um número natural é composto quando pode ser escrito como produto
de dois números naturais maiores que 1. Assim, por exemplo, 91 é composto porque
podemos escrever 91 = 7 × 13.
Mostre que o número

EUREKA! N°22, 2005

29
Sociedade Brasileira de Matemática

 2 2004 + 2 
2  +1
é composto.

PROBLEMA 4:
Arnaldo e Bernaldo disputam um jogo num tabuleiro 2 × n:

As peças do jogo são dominós 2 × 1. Inicialmente Arnaldo coloca um dominó


cobrindo exatamente duas casas do tabuleiro, na horizontal ou na vertical. Os
jogadores se revezam colocando uma peça no tabuleiro, na horizontal ou na vertical,
sempre cobrindo exatamente duas casas do tabuleiro. Não é permitido colocar uma
peça sobre outra já colocada anteriormente.
Quem não conseguir colocar uma peça no tabuleiro perde.

Qual dos dois jogadores tem uma estratégia vencedora, ou seja, uma estratégia que o
leva à vitória quaisquer que sejam as jogadas de seu adversário, para:
(a) n = 2004?
(b) n = 2005?

PROBLEMA 5:
Considere o polígono P de 6 lados.

EUREKA! N°22, 2005

30
Sociedade Brasileira de Matemática

Com cópias de P, podemos cobrir todo o plano, sem sobreposições, como mostrado
a seguir.

Existe um polígono de 13 lados com o qual é possível cobrir todo o plano com suas
cópias, sem sobreposições? Caso seja possível, apresente um polígono. Caso não
seja, diga o porquê.

PROBLEMAS – NÍVEL 2

PROBLEMA 1:
Na figura, ABC e DAE são triângulos isósceles (AB = AC = AD = DE) e os ângulos
BAC e ADE medem 36°.

a) Utilizando propriedades geométricas, calcule a medida do ângulo EDˆ C .


b) Sabendo que BC = 2, calcule a medida do segmento DC.
c) Calcule a medida do segmento AC.

EUREKA! N°22, 2005

31
Sociedade Brasileira de Matemática

PROBLEMA 2:
A seqüência de algarismos
1, 2, 3, 4, 0, 9, 6, 9, 4, 8, 7, …

é construída da seguinte maneira: cada elemento, a partir do quinto, é igual ao último


algarismo da soma dos quatro anteriores.
a) Os algarismos 2, 0, 0, 4, juntos e nesta ordem, aparecem na seqüência?
b) Os algarismos iniciais 1, 2, 3, 4, juntos e nesta ordem, aparecem novamente na
seqüência?

PROBLEMA 3:
Esmeralda tem uma pilha com 100 pedras. Ela divide essa pilha em duas novas
pilhas e em seguida multiplica as quantidades de pedras nessas duas novas pilhas e
escreve o produto em um quadro. Ela então escolhe uma pilha com mais de uma
pedra e repete esse procedimento: a pilha é dividida em duas, as quantidades de
pedras nessas duas pilhas são multiplicadas e o produto escrito no quadro. Esta
operação é realizada até se obter apenas pilhas com 1 pedra cada.
Quais são os possíveis valores da soma de todos os produtos escritos no quadro?

PROBLEMA 4:
Em um jogo para dois participantes, Arnaldo e Bernaldo alternadamente escolhem
um número inteiro positivo. A cada jogada, deve-se escolher um número maior que o
último número escolhido e menor que o dobro do último número escolhido.
Nesse jogo, vence o jogador que conseguir escolher o número 2004. Arnaldo joga
primeiro e inicia com o número 2. Qual dos dois tem estratégia vencedora, ou seja,
consegue escolher o número 2004 independentemente das jogadas do adversário?

PROBLEMA 5:
Seja D o ponto médio da hipotenusa AB de um triângulo retângulo ABC. Sejam O1 e
O2 os circuncentros dos triângulos ADC e DBC, respectivamente.
ˆ
a) Mostre que O1 DO 2 é reto.

b) Mostre que AB é tangente ao círculo de diâmetro O1O2 .

PROBLEMA 6:
Considere todas as maneiras de colocarmos nas casas de um tabuleiro 10 × 10
exatamente dez vezes cada um dos algarismos 0, 1, 2, …, 9.
Encontre o maior inteiro n com a propriedade de que, em cada tabuleiro, alguma
linha ou alguma coluna contenha pelo menos n algarismos diferentes.

EUREKA! N°22, 2005

32
Sociedade Brasileira de Matemática

PROBLEMAS – NÍVEL 3

PROBLEMA 1:
Seja ABCD um quadrilátero convexo. Prove que os incírculos de ABC, BCD, CDA e
DAB têm um ponto em comum se, e somente se, ABCD é um losango.

PROBLEMA 2:
Determine todos os valores de n tais que é possível dividir um triângulo em n
triângulos de modo que não haja três vértices alinhados e em cada vértice incida o
mesmo número de segmentos.

Mostramos a seguir tal divisão para n = 7. Observe que em cada um dos seis vértices
incidem quatro segmentos.

!
PROBLEMA 3:
Seja x1 , x 2 , , x 2004 uma seqüência de números inteiros satisfazendo
x k +3 = x k + 2 + x k +1 x k , 1 ≤ k ≤ 2001 .

É possível que mais da metade de seus termos sejam negativos?

PROBLEMA 4:
Considere todas as maneiras de colocarmos nas casas de um tabuleiro 10 × 10
exatamente dez vezes cada um dos algarismos 0, 1, 2, …, 9.

Encontre o maior inteiro n com a propriedade de que, em cada tabuleiro, alguma


linha ou alguma coluna contenha pelo menos n algarismos diferentes.

EUREKA! N°22, 2005

33
Sociedade Brasileira de Matemática

PROBLEMA 5:
Considere a seqüência (a n ) n∈N com a0 = a1 = a2 = a3 = 1 e an a n −4 = a n −1a n−3 + an2− 2 .

Mostre que todos os termos dessa seqüência são números inteiros.

PROBLEMA 6:
Sejam a e b números reais. Considere a função f a ,b : R2 → R2 definida por
f a ,b ( x; y ) = (a − by − x 2 ; x) . Sendo P = ( x; y ) ∈ R 2 , definimos f a0,b ( P ) = P e
f ak,b+1 ( P ) = f a ,b ( f ak,b ( P)) , para k inteiro não negativo.

O conjunto per(a; b) dos pontos periódicos da função f a ,b é o conjuntos dos pontos


P de R2 para os quais existe um inteiro positivo n tal que f an,b ( P) = P .

Fixado o real b, prove que o conjunto Ab = {a ∈ R | per (a, b) ≠ 0/ } tem um menor


elemento. Calcule esse menor elemento.

SOLUÇÕES – NÍVEL 1

PROBLEMA 1: SOLUÇÃO DE DIANA VAISMAN (RIO DE JANEIRO - RJ)

Números ímpares que são quadrados perfeitos Soma dos quadrados dos algarismos

121 1+4+1=6
169 1 + 36 + 81 = 118
225 4 + 4 + 25 = 33
289 4 + 64 +81 = 149
361 9 + 36 + 1 = 46
441 16 + 16 + 1 = 33
529 25 + 4 + 81 = 110
625 36 + 4 + 25 = 65
729 49 + 4 + 81 = 134
841 64 + 16 + 1 = 81
961 81 + 36 + 1 = 118

Resposta: 841

EUREKA! N°22, 2005

34
Sociedade Brasileira de Matemática

PROBLEMA 2: SOLUÇÃO DE HUGO FONSECA ARAÚJO (JUIZ DE FORA - MG):


Posso pintar de 33 modos.
Com 1 cor o desenho é este:

z
z
z z

sendo z uma das três cores.


Com 2 cores tenho estes desenhos:

x x x
x z z
x z x z x x

Sendo x uma das três cores e z também.


Com 3 cores tenho estes desenhos

z z

x y
y y y x

Sendo x uma das três cores e y e z também.


Com 1 cor tenho 3 × 1 possibilidades.
Com 2 cores tenho 3 × 2 × 3 = 18 possibilidades.
Com 3 cores tenho 3 × 2 × 1 × 2 = 12 possibilidades.
No total tenho 12 + 18 + 3 = 33 possibilidades.

PROBLEMA 3: SOLUÇÃO DE ILLAN FEIMAN HALPERN (ITATIAIA - RJ):


O número 2(2 + 2) + 1 é equivalente a 4(2 +1) + 1.
2004 2003

Como toda potência de 2 é par então 22003 + 1 será ímpar.


Como 4 elevado a um número ímpar dá um número cujo último algarismo é 4, então
4(2 +1) + 1 terá 5 como último algarismo.
2003

Como todo número que termina com 5 é múltiplo de 5 então o número


2(2 + 2) + 1 será múltiplo de 5 e poderá ser escrito como 5x , com x maior do que 1, o
2004

que prova que 2(2 + 2) + 1 é composto.


2004

EUREKA! N°22, 2005

35
Sociedade Brasileira de Matemática

PROBLEMA 4: SOLUÇÃO DE JAMES JUN HONG (SÃO PAULO – SP):


Para n ímpar, Arnaldo tem a estratégia vencedora. Para n par, Bernaldo tem a
estratégia vencedora.
Quando o número n for ímpar, basta Arnaldo começar com um dominó na vertical.
Sejam quais forem as jogadas de Bernaldo, Arnaldo vencerá. Se Bernaldo puser um
dominó na vertical, Arnaldo deverá por na vertical também, em qualquer lugar do
tabuleiro. Se Bernaldo puser na horizontal, Arnaldo deverá por um exatamente acima
ou abaixo da peça de Bernaldo. Seguindo as regras, Arnaldo vencerá porque não
sobrará nenhum espaço e o número de jogadas será ímpar. Como Arnaldo começa,
ele também termina e Bernaldo não poderá colocar uma peça no tabuleiro cheio.
Se o número n for par, Bernaldo vence. As regras para preenchimento são as mesmas
já citadas: se Arnaldo puser um dominó na vertical, Bernaldo deverá fazer o mesmo.
Se Arnaldo puser na horizontal, Bernaldo deverá colocar na horizontal, acima ou
abaixo da peça de Arnaldo. Como o número de jogadas, seguindo as regras, será par,
Bernaldo terminará de preencher o tabuleiro porque Arnaldo começa.
Conseqüentemente, Arnaldo não poderá colocar nenhuma peça e perderá.
Lembramos que sendo o tabuleiro 2 × n, n é o número máximo de jogadas pois o
dominó ocupa 2 casas do tabuleiro.
a) sendo 2004 par, Bernaldo vence.
b) sendo 2005 ímpar, Arnaldo vence.

PROBLEMA 5:
SOLUÇÕES DE WALLACE J. INOCÊNCIO e CAROLINE RIGOLON VEIGA (RIO DE JANEIRO - RJ)

EUREKA! N°22, 2005

36
Sociedade Brasileira de Matemática

SOLUÇÕES – NÍVEL 2

PROBLEMA 1: SOLUÇÃO DE VALTER BARBALHO LIMA FILHO :


D

36° 2
36°
C
2
72°
72°
2 F 2
72°
36°
36° 72° 72°
A B
E

a) I) AD = DE ⇒ DEAl = DAEl = 180° − 36° = 72° ⇒ DAF = 36°


2
l = ACDl = 180° − 36°
II) AD = AC ⇒ ADC = 72° ⇒ EDC = 36°
2
b) I) ∆ADE ≡ ∆ACB( LAL) ⇒ AE = 2.
II) ∠AFE = 72° ⇒ AE = AF = 2
l = F lAD = 36° ⇒ AF = FD = 2
III) ADF
l = DCF
IV) DFC l = 72° ⇒ DC = FD = 2

c) I) Seja AC = AB = x.
AE FE 2 FE 4
Temos ∆AFE ~ ∆ACD = = ⇒ = ⇒ x ⋅ FE = 4 ⇒ FE =
AB CB x 2 x
4 x2 − 4
II) DE = x ⇒ DF = DE − FE ⇒ DF = x − ⇒ DF =
x x
x −4
2
III) DF = AF = 2 ⇒ = 2 ⇒ x2 − 2 x − 4 = 0 ⇒ x = 1 + 5
x
IV) AC = DE = x = 1 + 5.

PROBLEMA 2: SOLUÇÃO DE RÉGIS PRADO BARBOSA (FORTALEZA - CE):


a) note que se um natural n é par ou ímpar, n – 10k terá mesma paridade pois
n par =2t ⇒ 2t − 10k = 2(t − 5k ) → par

n ímpar =2t + 1 ⇒ 2t + 1 − 10k = 2(t − 5k ) + 1 → par

EUREKA! N°22, 2005

37
Sociedade Brasileira de Matemática

logo se a soma de quatro números da seqüência tiver certa paridade o quinto número
terá a mesma.
Seja agora: i = No. ímpar p = No. par
1, 2, 3, 4, 0, 9, 6, 9, 4, 8, 7, 8, 7, 0, 2…
i, p, i, p, p, i, p, i, p, p, i, p, i, p, p…

Note que as paridades se repetem de 5 em 5 números. Provemos que de fato ela é


sempre assim.

Por indução: 1, 2, 3, 4, 0, 9, 6, 9, 4, 8…



Caso inicial: i, p, i, p, p , i, p, i, p, p ...


a1 a2

Hipótese: suponha que dá certo até o ak − ésimo período.




Passo indutivo: ...i, p, i, p, p a, b, c, d, e…


ak

a ≡ p + i + p + p ≡ i(mod10), logo a é ímpar.


b ≡ i + p + p + a ≡ i + p + p + i ≡ p(mod10), logo b é par.
c ≡ p + p + a + b ≡ p + p + i + p ≡ i( mod10), logo c é ímpar.
d ≡ p + a + b + c ≡ p + i + p + i ≡ p(mod10), logo d é par.
e ≡ a + b + c + d ≡ i + p + i + p ≡ p(mod10), logo e é par.


Assim, a, b, c, d, e = i, p, i, p, p , e logo nunca poderemos chegar a 2, 0, 0, 4, pois não


ak +1

poderemos ter 4 p's seguidos.

b) Veja que, para esta seqüência, a seqüência dos grupos de 4 termos consecutivos
dela não poderá ter infinitos termos diferentes, pois não temos infinitas
possibilidades para a, b, c, d: serão no máximo
NNNN
a , b , c , d = 104 possibilidades (aqui são contadas possibilidades que, assim
10 10 10 10
como vimos no item a), não podem aparecer, mas o que queremos com isso não é
achar um número exato mas sim um máximo e mostrar que as possibilidades são
finitas). Assim, num certo ponto começarão a se repetir os números formando um
período, assim:

EUREKA! N°22, 2005

38
Sociedade Brasileira de Matemática

y
a, b, c, d
1, 2, 3, 4 …
x

Mas note que se: y é o último número antes de começar a repetição, x é o último
número do período e a, b, c, d são os 4 primeiros números do período, teremos y + a
+ b + c ≡ d (mod 10) e x + a + b + c ≡ d (mod 10)
logo y ≡ d – a – b – c ≡ x (mod 10), e x ≡ y (mod 10), 0 ≤ x ≤ 9 e 0 ≤ y ≤ 9 ⇒ x = y.
Se fizermos isso várias vezes veremos que na verdade o período é:

…1, 2, 3, 4 …

Logo ele aparecerá novamente.

PROBLEMA 3: SOLUÇÃO DE RÉGIS PRADO BARBOSA (FORTALEZA - CE):


Note que se temos uma pilha com a pedras e fazemos o processo dividindo-a em
duas pilhas com b e e pedras o número escrito será b × e, mas se a foi dividido em b
e, temos:
a2 − b2 − e2
b + e = a ⇒ (b + e)2 = a2 ⇒ b2 + 2be + e2 = a2 ⇒ 2be = a 2 − b2 − e2 ⇒ be = .
2
Assim note que a soma será:
b1 ⋅ b2 + b3 ⋅ b4 ... + bi ⋅ bi +1 = S ( bi = No. de pedras na pilha i)
1002 − b12 − b22 b12 − b32 − b42 b 2 − b2 − bi +12
+ ... + i −1 i =S
2 2 2
EUREKA! N°22, 2005

39
Sociedade Brasileira de Matemática

veja só que se bt > 1 ele ainda será dividido em mais pilhas, ou seja quando ele
bx2 − bt2 − by2 bt2 − bz2 − bw2
aparecer será: mas se é bt > 1 aparecerá também . Assim, os
2 2
bt2 terão soma 0, a não ser quando bt = 1 e, como a pilha inicial tem 100 pedras, no
fim são 100 pilhas com 1 pedra cada, e teremos:


1002 − 12 − 12 − ... − 12
10000 − 100 ⋅ 12 9900
S= 100 vezes
= = .
2 2 2
9900
S= ⇒ S = 4950 e esta é a única possibilidade.
2

PROBLEMA 4: SOLUÇÃO DE HENRIQUE PONDÉ DE OLIVEIRA PINTO (SALVADOR - BA):


Seja uma Rodada definida como a jogada de cada jogador assim na 1ª Rodada
Arnaldo escolheu o número 2 e na 2ª Rodada Bernaldo será obrigado a escolher um
número maior que 2 e menor que 2 × 2 = 4 logo na 2ª Rodada será escolhido o
número 3. Sejam os dois jogadores do enunciado os jogadores X e Y.
1. Suponha que o jogador X escolhe 2004 e ganha na Rodada n.
2. Para isso Y teria que ter escolhido qualquer número entre 1003 e 2003 na
Rodada n – 1 e isso é visto facilmente.
3. A jogada de X na Rodada n – 2 teria que ser 1002, pois se fosse maior ou
menor que 1002 Y não seria obrigado a escolher um número entre 1003 e
2003 na Rodada n – 1.
4. Para X ter escolhido 1002 na Rodada n – 2 Y teria que ter escolhido um
número entre 502 e 1001 na Rodada n – 3.
5. A jogada de X na Rodada n – 4 teria que ser 501, pois se fosse maior ou
menor que 501, Y não seria obrigado a escolher um número entre 502 e 1001
na Rodada n – 3.
6. Para X ter escolhido 501 na Rodada n – 4, Y teria que ter escolhido um
número entre 251 e 500 na Rodada n – 5.
7. A jogada de X na Rodada n – 6 teria que ser 250, pois se fosse maior ou
menor que 250, Y não seria obrigado a escolher um número entre 251 e 500
na Rodada n – 5.

Agora que o raciocínio da questão já foi monstrado podemos continuar sem escrever
tanto.

Rodada n – 6: (X): 250 (Como 7. mostrou)


Rodada n – 7: (Y): Entre 126 e 249
Rodada n – 8: (X): 125

EUREKA! N°22, 2005

40
Sociedade Brasileira de Matemática

Rodada n – 9: (Y): Entre 63 e 124


Rodada n – 10: (X): 62
Rodada n – 11: (Y): Entre 32 e 61
Rodada n – 12: (X): 31
Rodada n – 13: (Y): Entre 16 e 30
Rodada n – 14: (X): 15
Rodada n – 15: (Y): Entre 8 e 14
Rodada n – 16: (X): 7
Rodada n – 17: (Y): Entre 4 e 6
Rodada n – 18: (X): 3
Rodada n – 19: (Y): 2

Logo X = Bernaldo e Y = Arnaldo, e como X ganha, Bernaldo ganha.

Obs: Quando falo números entre r e s, r e s estão incluídos neste intervalo.

PS: Observe as jogadas de Bernaldo: 3, 7, 15, 31, 62, 125, 250, 501, 1002, 2004. Se
ele escolhe x na Rodada a escolhe 2x ou 2x + 1 na Rodada a + 2. Isso nos leva a
uma observação que se Bernaldo escolhe x na Rodada a, Arnaldo pode escolher
entre x + 1 e 2x – 1 na Rodada a + 1, o que implica que Bernaldo pode escolher 2x
ou 2x + 1 na Rodada a + 2, SEMPRE. Isso nos leva a pensar no sistema binário de
numeração. Ou seja, se na Rodada a o Jogador escolheu o número AnAn –1...A0,
na Rodada a + 2 pode escolher os números AnAn–1.....A00 ou AnAn–1...A01. Logo ele
pode ir “adicionando” algarismos à direita da representação binária do número. Veja
que os dois algarismos à esquerda de um número no sistema binário são 10 ou 11 e
que o 1º número do 1º jogador é 2 = (10)2 e o do 2º é 3 = (11)2. Logo se K é o
número a ser alcançado, se K começar com 10 no sistema binário o 1º ganha e se K
começar com 11, no sistema binário o 2º ganha. Como a representação binária de
2004 é 11111010100 que começa com 11 temos que o 2º ganha.

PROBLEMA 5: SOLUÇÃO DE LÚCIO EIJI ASSAOKA HOSSAKA (CURITIBA - PR):


a) Primeiro, é necessário dizer que ∆ABC pode ser inscrito em uma circunferência
l e reto e deve estar compreendendo um
F
de raio AD, onde AB é o diâmetro, pois ACB
arco AB = 180° , que é uma semi-circunferência. Logo, por D ser centro dessa
circunferência, AD = BD = CD, e os triângulos ACD e BCD são isósceles, tendo AC
e BC como base, respectivamente.
Logo, a mediatriz de AC contém O1 assim como D (pois D está à mesma distância
de A e C, assim como todos os pontos da mediatriz). Da mesma maneira, a mediatriz
EUREKA! N°22, 2005

41
Sociedade Brasileira de Matemática

de CB contém O2 e D (lembrando que o circuncentro é o encontro das mediatrizes


dos lados do triângulo).
A mediatriz de AC é perpendicular a AC, e a mediatriz de CB é perpendicular a CB.
Logo, ambas são perpendiculares entre si, pois AC e BC também o são. Como
ambas contém D e são retas, elas se interceptam em D, e além disso cada uma
l é reto.
contém um dos circuncentros, o que mostra que O1 DO2

b) Seja E o ponto médio de AD, e F o ponto médio de BD. A mediatriz de AD


l é um ângulo reto. Analogamente, DFO
contém E e O1 , e portanto O1 ED l é reto
2
também.
Como E, D e F são colineares (AB contém os três), então EO1 e FO2 são segmentos
paralelos entre si. Seja G o ponto médio de O1O2 . Como O1O2 D é um triângulo
retângulo G é o centro do círculo que deveremos provar que é tangente a AB.
Perceba que G é o ponto médio de O1O2 assim como D é o ponto médio de EF (é
ED O1G
óbvio que ED = DF, pois D é o ponto médio de AB), e que portanto = . DG
DF GO2
é, portanto, paralelo a EO1 e FO2 , pelo teorema de Tales. Assim, DG é
perpendicular a AB, e como DG é raio do círculo de diâmetro O1O2 , AB tangencia
esse círculo.

PROBLEMA 6: SOLUÇÃO DE LÚCIO EIJI ASSAOKA HOSSAKA (CURITIBA - PR):


Se são 10 linhas e 10 colunas, então há espaço para 20 "presenças" de alguns dos 10
algarismos. "Presença" significa que cada vez que um número aparece em alguma
linha ou coluna, significa uma "presença". No caso das linhas, por exemplo. Se um
algarismo estiver distribuído dez vezes em duas linhas, são duas "presenças".
Assim como se algarismo aparecer apenas 3 vezes em 3 linhas, são 3 "presenças". n
é maior que 3. Como já disse, cada algarismo ocupa pelo menos 3 linhas ou 3
colunas. Se um algarismo ocupar, por exemplo, 3 linhas, ocupará 4 colunas, pois se
fossem 3 linhas e 3 colunas, apareceria, no máximo, 9 vezes. E vice-versa. O melhor
jeito de distribuir os algarismos é fazer com que cada um tenha o mínimo de
presenças. A soma das presenças de cada algarismo é no mínimo 7 (3 linhas e 4
colunas, e vice-versa, e 2 linhas e 5 colunas, e vice-versa), sendo 7 × 10 = 70
presenças no total. Se são 60 disponíveis (10 linhas e 10 colunas, cada uma podendo
comportar 3 algarismos distintos), é impossível criar um tabuleiro com no máximo 3
algarismos diferentes em cada coluna e cada linha. portanto, n é maior que 3.
Assim, n é 4, pois é possível distribuir os números de forma e que cada linha e cada
coluna tenha até 4 algarismos diferentes:

EUREKA! N°22, 2005

42
Sociedade Brasileira de Matemática

Assim, existe um arranjo que limita o valor de n, e esse valor é 4.

9 0 9 0 9 0 9 0 9 0
9 0 9 0 9 0 9 0 9 0
6 6 5 5 5 7 7 7 8 8
6 6 5 5 5 7 7 7 8 8
6 6 5 5 6 8 7 7 8 8
6 6 5 5 6 8 7 7 8 8
1 2 2 2 2 3 3 3 3 4
1 1 2 2 2 3 3 3 4 4
1 1 1 2 2 3 3 4 4 4
1 1 1 1 2 3 4 4 4 4

SOLUÇÕES – NÍVEL 3

PROBLEMA 1: SOLUÇÃO DE EDSON AUGUSTO BEZERRA LOPES (FORTALEZA - CE):


Parte 1)
A B

D C
Se ABCD é um losango, então AB = BC = CD = DA, ou seja, os triângulos ABC,
BCD, CDA, DAB são todos isósceles. Veja também que AC ⊥ BD. Seja
AC ∩ BD = O . Como em um triângulo isósceles a altura relativa à base é também
bissetriz. AO, OB, OC, OD possam pelos incentros de DAB, ABC, BCD e CDA
respectivamente.
Como ao traçarmos uma perpendicular ao lado pelo incentro obtemos o ponto de
toque do incírculo a esse lado, O está contido nos 4 incirculos, já que é o ponto de
toque do incírculo na base dos 4 triângulos.

EUREKA! N°22, 2005

43
Sociedade Brasileira de Matemática

Parte 2)
A B

D C
Suponhamos agora que os 4 círculos têm um ponto em comum. Temos então que os
incírculos dos triângulos ABC e ACD tocam AC no mesmo ponto, pois do contrário
não teriam nenhum ponto em comum. Seja O esse ponto. Veja que se O estiver no
interior do triângulo ABC o incírculo do triângulo BDC não o conterá, e seguindo o
mesmo raciocínio vemos que O não está no interior do triângulo BDC. Logo O está
sobre BD ⇒ AC ∩ BD = O.
Sejam O1 , O2 , O3 e O4 Os incentros dos triângulos DAB, ABC, BCD e CDA,
respectivamente. Suponhamos que AC e BD não são perpendiculares. Suponhamos
l e DOC
agora, sem perda de generalidade, que AOB l são obtusos e BOC
l e AOD l
são agudos. Claramente O2 está no interior do triângulo AOB, pois já que
O2O ⊥ AC , temos O2 OC l > BOC l . Com o mesmo raciocínio encontramos que O e
1

O2 estão no interior do triângulo AOB e que O3 e O4 estão no interior do triângulo


COD. Seja G1 = AO1 ∩ BO2.
l l l l
Claramente G1 lAB =
DAB l = ABC ⇒ DAB + ABC = 180° − AG
e G1 BA l 1B
2 2 2
l + OBG
= 180° − (OAG l + AOB l ) < 180° − AOB
l ⇒ DAB
l + ABC l ) < 180°,
l < 2(180° − AOB
1 1
l > 90°. De modo análogo temos
pois AOB
BCD l < 180° ⇒ DAB
l + CDA l + ABCl + BCD l < 360° ⇒ 360° < 360° . Absurdo!
l + CDA
Assim, AC ⊥ BD, donde O1 ∈ AO, O2 ∈ BO, O3 ∈ CO, O4 ∈ DO ⇒ AO, BO, CO e DO
são, além de alturas, bissetrizes ⇒ ABC , BCD, CDA, DAB são isósceles
⇒ AB = BC = CD = DA
⇒ ABCD é um losango.

EUREKA! N°22, 2005

44
Sociedade Brasileira de Matemática

PROBLEMA 2: SOLUÇÃO DE LARISSA CAVALCANTE QUEIROZ DE LIMA (FORTALEZA - CE):


Seja N tal que o triângulo está dividido em N triângulos de modo que não há 3
vértices colineares e em cada vértice incida exatamente k segmentos.
Seja M o total de vértices (incluindo os vértices do triângulo dividido ABC).




180° ⋅ N = 360°(M − 3) + 180°


pois há N ∆'s soma dos ângulos de cada
vértice do interior de ABC
N
Contando a soma dos ângulos de todos os triângulos temos:

soma dos ângulos


lA+ B l
l +C

⇒ N = 2( M − 3) + 1 = 2M − 6 + 1 = 2M − 5
N +5
⇒M =
2

N
Contando o total de segmentos:
M ⋅k N ⋅ 3 + 3
=  há N triângulos e cada segmento é lado de dois triângulos, exceto os três segmentos: AB, BC, CA.
2 2 
há m vértices de
grau k cada

⇒ M ⋅ k = 3N + 3 ⇒ k =
3N + 3
M
=
3N + 3
=
6N + 6
( N + 5) / 2 N + 5
∈ ]
6( N + 5) 24 24
⇒k = − =6− .
N +5 N +5 N +5
como k é inteiro, N + 5 deve ser um divisor de 24.
Assim, N + 5 = 6,8,12 ou 24 (pois N é inteiro positivo)
⇒ N = 1,3,7 ou 19.

De fato, para cada um desses valores, há uma divisão do triângulo:

N = 1: N=3:

N = 7: N = 19:

EUREKA! N°22, 2005

45
Sociedade Brasileira de Matemática

Observação: as duas contagens feitas pela Larissa, se generalizadas, levam a uma


demonstração do Teorema de Euler: sendo V, A, e F o número de vértices, arestas e
faces, respectivamente, de um poliedro (ou se você quiser, um grafo plano), então
V – A + F = 2. No problema 2, F = n + 1, V = M e A é o número de segmentos.
Em contrapartida, também é possível resolver este problema usando o Teorema de
Euler.

PROBLEMA 3: SOLUÇÃO DE FÁBIO DIAS MOREIRA (RIO DE JANEIRO - RJ):


Sim. Se tomarmos a suficientemente grande e ( x1 , x2 , x3 ) = (a, −a, a), é trivial ver que
cada xi é um polinômio em a. Em particular, como a seqüência possui um número
finito de termos, podemos tomar a suficientemente grande de tal forma que cada
termo xi da seqüência tenha o sinal do coeficiente líder do polinômio.
Mas é fácil ver que a seqüência dos termos líderes é:
(a, −a, a, −a 2 , −2a 2 , −a3 ,2a 4 ,2a5 , −2a7 ,4a9 , −4a12 ,...) e é fácil provar, por indução que
dois termos líderes nunca se cancelam (basta notar que a partir de −4a12 , os
exponentes dos dois termos anteriores são sempre maiores que metade do expoente
c
do termo atual: isso é verdade para −4a12 , e se os expoentes são a, b e c, com a >
2
c a+b
e b > , a < b < c (já que a seqüência dos expoentes é crescente), então b > pois
2 2
a +b
b>ae c> pois c > a e c > b). Mas a seqüência dos sinais dos termos líderes é
2
claramente periódica de período 7:

(+, −, +, −, −, −, +, +, −, +, −, −, −, +, +, −, +,...)

Logo a seqüência definida com o a supracitado e x1 = a, x2 = −a, x3 = a tem pelo


4 
menos  ⋅ 2004  > 1002 termos negativos.
 7 

PROBLEMA 4:
Veja a solução do problema No. 6 do nível 2.

EUREKA! N°22, 2005

46
Sociedade Brasileira de Matemática

PROBLEMA 5: SOLUÇÃO DE FÁBIO DIAS MOREIRA (RIO DE JANEIRO - RJ):


Vamos demonstrar que an é sempre inteiro por indução: isto é evidentemente
verdadeiro para n = 0, 1, 2, 3, 4, 5, 6, 7 (pois a4 = 2, a5 = 3, a6 = 7 e a7 = 23) .
Suponha então que n ≥ 8 , e a0 , a1 , a2 ,..., an −1 são todos inteiros.
Lema: Se 0 < k < n, então (a k , ak −1 ) = 1.
Prova: O resultado é evidentemente verdadeiro para k =1.
Suponha-o verdadeiro para k – 1. Então
 a a + a2 
(ak , ak −1 ) =  k −1 k −3 k −2 , ak −1  ≤ (ak −1ak −3 + ak2−2 , ak −1 ) = (ak2−2 , ak −1 ) = 1, pois
 ak −4 
(ak −1 , ak − 2 ) = 1. Logo o resultado é válido para todo 0 < k < n.
Corolário: 1 < k < n ⇒ (ak , ak −2 ) = 1 e 2 < k < n ⇒ ( ak , ak −3 ) = 1.
Prova: Basta notar que
(ak , ak −2 ) ≤ (ak −1ak −3 + ak2−2 , ak −2 ) = (ak −1ak −3 , ak −2 ) ≤ (ak −1 , ak −2 ) ⋅ (ak −3 , a k −2 ) = 1⋅ 1 = 1
e (ak , ak −3 ) ≤ (ak −1ak −3 + ak2−2 , ak −3 ) = (ak2−2 , ak −3 ) ≤ (ak −2 , ak −3 ) 2 = 12 = 1.
Para provar que an é inteiro, basta provar que an −4 | an −1an −3 + an2−2 .
Em outras palavras, temos que demonstrar que
an −3an −1 + an2−2 ≡ 0 (mod an −4 ) ⇔
2
an −6 an −4 + an2−5 an −4 ⋅ an −2 + an2−3  a a + a2 
⋅ +  n −5 n −3 n −4  ≡ 0 (mod an −4 ) ⇔
an−7 an−5  an−6 
an −6 (an −6 an −4 + an −5 )(an −4 an −2 + an −3 ) + an −5 an −7 (an −5 an −3 + an2−4 )2
2 2 2
≡ 0 (mod an −4 )
an−5 an2−6 an−7
Pelo Lema, podemos multiplicar por an −5 an2−6 an −7 e manter a equivalência, logo basta
provar que
an2−6 (an−6 an−4 + an2−5 )(an−4 an−2 + an2−3 ) + an−5 an−7 (an−5 an−3 + an2−4 )2 ≡ 0(mod an−4 )
⇔ an2−5 an2−3 (an2−6 + an−7 an−5 ) ≡ 0(mod an−4 ) ⇔
⇔ an2−5 an2−3an−4an−8 ≡ 0(mod an−4 ) ⇔ 0 ≡ 0 (mod an −4 ).

PROBLEMA 6: SOLUÇÃO DE GABRIEL TAVARES BUJOKAS (SÃO PAULO - SP):


−(b + 1)2
Resposta: .
4
Seja k o tamanho do ciclo, P = ( x1 , x0 ), f ar,b ( P) = ( xr +1 , xr ), para todo r ∈ < .

EUREKA! N°22, 2005

47
Sociedade Brasileira de Matemática

x0 = a − bxk −2 − xk2−1
x1 = a − bxk −1 − x02
Logo x2 = a − bx0 − x12 ⇒ Sendo S1 = ∑ xi , S2 = ∑ xi2 , temos S1 (b + 1) + S 2 = ka
#
xk −1 = a − bxk −3 − xk2−2

S2 ∑ xi S1 S2
≥ ≥ ⇒ S2 ≥ 1
 
k k k
MQ ≥ MA
k

S12 x2 −(b +1)2


Logo: ka ≥ + S1(b +1) ⇒ + x(b +1) − ka tem raiz ⇒∆ = (b +1)2 + 4a ≥ 0 ⇒ a ≥ .
k k 4
−(b + 1)2  (b + 1) (b + 1)   (b + 1) (b + 1) 
Para a = : f a ,b  − ;−  =− ;−  ⇒ per (a,b) ≠∅.
4  2 2   2 2 

EUREKA! N°22, 2005

48
Sociedade Brasileira de Matemática

XXVI OLIMPÍADA BRASILEIRA DE MATEMÁTICA


Problemas e Soluções da Primeira Fase – Nível Universitário

PROBLEMA 1:
1 0 i 
 
Considere a matriz complexa A =  0 0 0  . Calcule A2004 .
 i 0 1
 

PROBLEMA 2:
x 2004
∫−1 1 + e x dx
1
Calcule a integral:

PROBLEMA 3:
Determine a equação da reta que tangencia a curva de equação y = 3x 4 − 4 x 3 em
dois pontos distintos.

PROBLEMA 4:
Quantas triplas ordenadas (A, B, C) de subconjuntos de {1, 2,..., n} existem para as
quais A ∩ B ∩ C = ∅; A ∩ B ≠ ∅; A ∩ C ≠ ∅ ?

PROBLEMA 5:
Considere a matriz A n × n definida por aij = n(i − 1) + j , para todos 1 ≤ i, j ≤ n.
As interseções de k linhas e k colunas quaisquer de A determinam uma submatriz de
ordem k de A. Seja ϕ ( n) a soma dos determinantes de todas as submatrizes de A.
ϕ (n )
a) Determine λ real de forma que lim λ exista e seja não nulo.
n →+∞ n

b) Determine o valor do limite acima para o valor de λ encontrado.

PROBLEMA 6:

1
Calcule ∑ (3k + 1)(3k + 2)(3k + 3) .
k =0

EUREKA! N°22, 2005

49
Sociedade Brasileira de Matemática

SOLUÇÕES – PRIMEIRA FASE – NÍVEL UNIVERSITÁRIO

SOLUÇÃO DO PROBLEMA 1:
−4 0 0  (−4)k 0 0 
   
Temos A 4= 0 0 0 , donde A 4k =  0 0 0  .
 0 0 −4  0 0 (−4)k 
   
−21002 0 0 
 
Em particular, A 2004=  0 0 0 .
 0 0 −21002 
 
Soluções cujo único erro se refere aos sinais:

SEGUNDA SOLUÇÃO DO PROBLEMA 1:


1 0 1  1 + i 0 0 
−1    
Diagonalizando A, temos A = XDX onde X =  0 1 0  , D =  0 0 0 
 1 0 −1   0 0 1− i
   
Assim,
 (1+ i)2004 0 0   −21002 0 0   −21002 0 0 
     
A2004 = X  0 0 0  X −1 = X  0 0 0  X −1 =  0 0 0 .
 0 (1− i)2004   0 −21002   0 −21002 
 0  0  0

SOLUÇÃO DO PROBLEMA 2:
x 2004 1 x
2004

∫−1 1 + e x ∫−1 1 + e− x dx .
1
Por substituição, dx =
Assim
1 x
2004
1  1 x 2004 1 x
2004
 1 1  1 1 
∫−1 1 + e x dx =  ∫−1
2  1+ e x
dx + ∫−1 1 + e − x
dx  = ∫ x 2004 
 2 −1
+
 1+ e 1+ e
x −x  dx

1 1 1 1
= ∫ x 2004 dx = ∫ x 2004 dx = .
2 − 1 0 2005

SOLUÇÃO DO PROBLEMA 3:
Queremos encontrar a e b tais que, P = 3 x 4 − 4 x3 − ax − b tenha duas raízes reais
duplas; em particular, P deve ser um quadrado perfeito.

EUREKA! N°22, 2005

50
Sociedade Brasileira de Matemática

3 x 4 − 4 x3 − ax − b = ( cx 2 + dx + e) 2
= c 2 x 4 + 2cdx3 + (2ce + d 2 ) x 2 + 2 dex + e 2
c c −8 −4
implica em c = ± 3, d = −2 , e = −2 donde a = , b = .
3 9 9 27
8 4
Assim a reta tem equação y = − x − .
9 27

SOLUÇÃO DO PROBLEMA 4:
Vamos contar inicialmente o número de escolhas tais que A ∩ B ∩ C = ∅ .

A B

S1 S1 S2

S5 S6

S3

Cada elemento de {1,2,…,n} pode ser colocado em um dos 7 subconjuntos indicados


acima
(S1, S2, …,S7). Logo há 7n tais escolhas.
Fazendo um raciocínio similar, temos, dentre essas escolhas, 6n com A ∩ B = ∅ ;
6n com A ∩ C = ∅ e 5n com A ∩ B = ∅ e A ∩ C = ∅.
Portanto, pelo princípio de Inclusão-Exclusão, há 7 n − 2 ⋅ 6n + 5n maneiras de
escolher A, B, C.

SOLUÇÃO ALTERNATIVA DO PROBLEMA 4:


Fixe o número k de elementos de A ∩ B (1 ” k ” n), e j de A ∩ C (1 ” j” n – k).

EUREKA! N°22, 2005

51
Sociedade Brasileira de Matemática

 n n − k 
Há   modos de escolher esses k elementos,   modos de escolher os j e 5
k   j 
regiões permitidas para cada um dos outros n – k – j elementos, de forma que a
resposta é:

n
 n  n − k  n − k − j
n−k n   n  n−k  n − k  n−k − j 
R= ∑ ∑   
k =1 j =1  k   j 
5 = ∑    ⋅ ∑ 

k =1   k  j =1 
5
j 



Utilizando o binômio de Newton, tem-se
n−k
 n − k  n−k − j
∑ 
j=0  j 
 5 = (1 + 5 ) n − k = 6 n − k .

Portanto,
n
 n  n−k n
 n  n−k
R= ∑ 
k 
k =1 
 6 − ∑   5 = 7 n − 6 n − (6 n − 5 n ) = 7 n − 2 ⋅ 6 n + 5 n .
 k =1  
k

SOLUÇÃO DO PROBLEMA 5:
Se k ≥ 3 , qualquer submatriz de ordem k de A determinada por k linhas e k colunas
tem determinante O. De fato, se j1 < j2 < j3 são índices de 3 das k colunas da
submatriz, denotando essas colunas por C j1 , C j2 e C j3 , temos que todas as entradas de
C j2 − C j1 são iguais a j2 − j1 e todas as entradas de C j3 − C j2 são iguais a j3 − j2 ,
j3 − j2 j −j j −j
donde C j3 − C j2 = ⋅ (C j2 − C j1 ), e logo C j3 = 3 1 ⋅ C j2 − 3 2 ⋅ C j1 .
j2 − j1 j2 − j1 j2 − j1

Assim, ϕ ( n) é a soma dos determinantes das submatrizes de A de ordem 1 ou 2. A


n2
n 2 (n 2 + 1)
soma dos determinantes das submatrizes de A de ordem 1 é ∑m = 2
.
m =1

As submatrizes de ordem 2 são obtidas da seguintes forma:


Dados 1 ≤ a < b ≤ n e 1 ≤ c < d ≤ n , associamos a seguinte submatriz de ordem 2:
 n(a − 1) + c n(a − 1) + d 
  , cujo determinante é
 n(b − 1) + c n(b − 1) + d 
n(( a − 1) d + (b − 1)c − (b − 1) d − ( a − 1) c) = n( a − b)( d − c). Assim, a soma dos
determinantes das submatrizes de ordem 2 é

EUREKA! N°22, 2005

52
Sociedade Brasileira de Matemática

2 2 2
      n j −1   n j ( j − 1) 
n  ∑ (a − b)  ∑ (d − c)  = −n  ∑ ( j − i)  = −n  ∑∑ r  = −n  ∑  =
 1≤a<b≤n  1≤c<d ≤n   1≤i< j ≤n   j =1 r =1   j =1 2 
2
n  n3  g (n)
= −  + g (n)  , onde lim 3 = 0 (de fato, g (n) é um polinômio de grau 2).
4 3 →∞
 n n
1  n (n + 1) 
2 2
Assim, como lim 7  =0 e
n→∞ n
 2 
1  n  n3    1  1 g(n) 2 
2
1
lim 7 ⋅  −  + g (n)   = lim  − ⋅  + 3   = − ,
n →∞ n  4  3 →∞ 
  n  4  3 n    36
ϕ (n ) 1
Temos λ = 7 e lim 7
=− .
n →∞ n 36

SOLUÇÃO DO PROBLEMA 6:

Decompondo em frações parciais, procuramos constantes A, B e C tais que:


1 A B C
= + + (1)
(3k + 1)(3k + 2)(3k + 3) 3k + 1 3k + 2 3k + 3
Comparando os numeradores, verifica-se que (1) é identidade para
1 1
A= , B = −1, C = .
2 2

1 2 1
Sendo S a soma procurada: 2S = ∑ 3k + 1 − 3k + 2 + 3k + 3 .
k =0

( )
1 ∞ 1
1
Como = ∫ x 3k dx , tem-se 2S = ∑ ∫ x 3k − 2 x 3k +1 + x 3k + 2 dx .
3k + 1 0 k =0 0

Trocando a integral com o somatório e somando a PG infinita:


 ∞ 3k 3k + 2   ∞
3k  (1 − x ) 2
1 1 1
2S = ∫  ∑ x − 2 x 3 k +1
+x dx = ∫  (1 − 2 x + x ) ⋅ ∑ x  dx = ∫
2
dx
0  k =0  0 k =0  0
1 − x3

EUREKA! N°22, 2005

53
Sociedade Brasileira de Matemática

1− x 1 −(2 x + 1) + 3
1 1
2S = ∫ dx = ∫ 2 dx
0 x + x +1
2
2 0 x + x +1
1 1 2x + 1 31 dx
2 ∫0 x 2 + x + 1 2 ∫0 
=− dx + 2
1  3
2

x+ 2 + 2 
   
1
1
x+
1 3
= − ln( x 2 + x + 1) + ⋅
2
⋅ arctan 2 = − 1 ln 3 + 3 ⋅ (π − π )
2 2 3 3 2 3 6
2 0
π 3 − 3ln 3
Logo, S =
12

EUREKA! N°22, 2005

54
Sociedade Brasileira de Matemática

XXVI OLIMPÍADA BRASILEIRA DE MATEMÁTICA


Problemas e Soluções da Segunda Fase – Nível Universitário

PROBLEMA 1:
A função derivável f : \ → \ tem as seguintes propriedades:
a) f (0) = 0 e f (2) = 2.
\
b) Para todo a ∈ \ {0}, a reta tangente ao gráfico de f no ponto P = (a, f (a ))
corta o eixo x em um ponto A e o eixo y em um ponto B de tal forma que A é o
ponto médio do segmento BP.

Calcule f (3).

PROBLEMA 2:
Prove que não existe um conjunto A ⊆ \ tal que:
2

• \ , { y ∈ \ (x, y) ∈ A} é finito.
Para todo x ∈
• Para todo y ∈ \ , {x ∈ \ ( x, y ) ∉ A} é enumerável.
Obs: Um conjunto A é dito enumerável se A = ∅ ou existe uma função sobrejetiva
f : ` → A.

PROBLEMA 3:
Seja A uma matriz real inversível de ordem n e At a sua transposta. Sejam
λ1 ≥ λ2 ≥ ... ≥ λn > 0 os autovalores de At A. Definimos a norma de A por A = λ1 e

o fator de dilatação de A por d ( A) = λ1 . Prove que, para quaisquer matrizes


λ2
AB
reais inversíveis A e B, d ( AB) ≥ ⋅ d ( B) .
A ⋅ B

PROBLEMA 4:
Seja ] n
]
= {( x1 , x2 ,..., xn ); x1 , x2 ,..., xn ∈ }.
Seja p um primo, k um inteiro positivo e P1 , P2 ,..., Pk , Q ∈ ] n
tais que para todo j,

1 ≤ j ≤ k,
Pj − Q
p
∉ n. ]
EUREKA! N°22, 2005

55
Sociedade Brasileira de Matemática

Prove que existe um polinômio f ( x1 ,..., xn ) com coeficientes inteiros com f ( Pj ) = 0

para todo j, 1 ≤ j ≤ k , e
f (Q)
p
∉ .]
PROBLEMA 5:
Seja m ≥ 2 um inteiro.
Arnaldo e Bernaldo disputam o seguinte jogo: cada jogador recebe, alternadamente,
um número N k e devolve para o outro jogador ou N k +1 = N k − 1 ou N k +1 =  k  .
N
 m 
Arnaldo começa recebendo um número inteiro positivo N0 . Quem devolver zero
vence o jogo.
Seja An (resp. Bn ) o conjunto dos valores de N0 , N0 < n , tais que Arnaldo (resp.
Bernaldo) tem estratégia vencedora.
A
Calcule lim n em função de m.
n →∞ B
n

PROBLEMA 6:
Seja γ :[0,2π ] → \ 2
uma função derivável, com derivada contínua, com
γ '(t ) = 1 para todo t e cuja imagem é uma curva simples fechada, isto é,

γ (t0 ) = γ (t1 ), t0 < t1 ⇔ t0 = 0, t1 = 2π .

Prove que existem 0 ≤ t0 < t1 < 2π tais que


2
γ (t0 ) − γ (t1 ) ≤ min {t1 − t0 ,2π + t0 − t1} .
π

EUREKA! N°22, 2005

56
Sociedade Brasileira de Matemática

PROBLEMA 1: SOLUÇÃO DE LEONARDO AUGUSTO ZÃO (NILÓPOLIS – RJ)


De acordo com o item (b) do enunciado, ao traçarmos a reta tangente ao gráfico no
ponto (a, f(a)), com a ≠ 0, obtemos:

f (x)
Reta tangente ao gráfico de f em (a, f(a))

P = (a, f(a))

α
A = (xA, 0) x

B = (0, yB)

B+P
Como A é ponto médio de BP, então: A =
2
a
Assim, x A = e yB = − f (a ) .
2
( f (a ) − yB ) 2 f (a )
Portanto, o coeficiente angular da reta é dado por tgα = =
(a − a / 2) a

Assim, f '( a) =
2 f (a )
a
\
, ∀a ∈ *.

Então chegamos à equação diferencial: f '( x) =


2 f ( x)
x
\
,x∈ *

= ⇒ [ln f ( x) ]' = [2ln x ]', x ∈ \


f '( x) 2 *
Resolvendo, temos: +
f ( x) x
Então ln f ( x) = 2ln x + k ⇒ ln f ( x ) = ln x 2 ⋅ e k ⇒ f ( x ) = e k ⋅ x 2 , ∀x ∈ \.
*
+

1
Para x = 2, f ( x) = 2. Então f (2) = e k ⋅ 22 ⇒ ek = .
2
1 2
Assim, a função f obedece f ( x ) = x , para x ∈ *+ .
2
\
1 2 9
Então, f (3) = ⋅ 3 , e finalmente, f (3) = .
2 2

EUREKA! N°22, 2005

57
Sociedade Brasileira de Matemática

PROBLEMA 2: SOLUÇÃO DE DAVI MÁXIMO ALEXANDRINO NOGUEIRA (FORTALEZA – CE)


Sejam
Ay = {x ∈ \ | ( x, y) ∈ A}
B = { y ∈ \ | ( x , y ) ∈ A}
Logo B é finito, ∀x ∈ \ e \ − A \.
x

x y é enumerável, ∀y ∈
Agora sejam y1 < y2 < y3 < ..., uma seqüência crescente infinita em \.
 A = \ − * (\ − A ) ≠ ∅, pois, do contrário, teríamos
∞ ∞
Logo, yi yi
i =1 i =1

\ = * (\ − A ), mas cada \ − A é enumerável ⇒ * (\ − A ) enumerável


∞ ∞

yi yi yi

⇒ \ enumerável, absurdo!
i =1 i =1

Agora, tome x ∈  Ay . Temos { y , y ,...,} ⊂ B , absurdo! (pois B é finito). Logo A


i 1 2 x x
i =1

não existe.

PROBLEMA 3: SOLUÇÃO DE ALEX CORRÊA ABREU (NITERÓI – RJ)


Sejam λ1 ≥ λ 2 ≥ ... ≥ λ n > 0; β 1 ≥ β 2 ≥ ... ≥ β n > 0; α 1 ≥ α 2 ≥ ... ≥ α n > 0 os
autovalores de ( AB)t AB, B t B e At A respectivamente. A desigualdade pedida é
claramente equivalente a λ 2 ≤ α 1 ⋅ β 2 .
Como X X é sempre simétrica e < X Xu, u >=< Xu, Xu > , temos pelo teorema do
T T

min-max (que provaremos no fim desta solução) que:


 < ABx, ABx >   < ABx, ABx > < Bx, Bx > 
λ 2 = min  max  = min  max ⋅
< x, x > 
.
dim S = n −1
 x∈S < x, x >  dim S = n −1  x∈S \{0} < Bx, Bx >
Por outro lado, como só há um subespaço de dimensão n (e ele inclui todos os
vetores), temos:
< Ax, Ax >  < Bx, Bx > 
α1 = max , e β 2 = min  max .
x∈\ n \{0} < x, x > dim S = n −1 x∈S \{0} < x, x >
 
< Bx, Bx >
Seja S o subespaço no qual β 2 é atingido, i.e, β 2 = max .
x∈S \{0} < x, x >

Temos então:
 < ABx, ABx > < Bx, Bx >  < ABx, ABx > < Bx, Bx >
λ 2 ≤ max  ⋅  ≤ max ⋅ max ,
x∈S \{0}
 < Bx, Bx > < x, x >  x∈S \{0} < Bx, Bx > x∈S \{0} < x, x >

EUREKA! N°22, 2005

58
Sociedade Brasileira de Matemática

(usamos respectivamente que o mínimo é ” TXH TXDOTXHU HOHPHQWR GR FRQMXQWR H


max(uv ) ≤ max u ⋅ max v )
O segundo termo do produto é exatamente β 2 . Fazendo x' = Bx e notando que
\
B( S ) ⊂ n , tem-se:
< ABx, ABx > < Ax ', Ax ' >
max ≤ max α1 , e juntando tudo obtemos λ 2 ≤ α 1 ⋅ β 2
x∈S \{0} < Bx, Bx > x '∈\ \{0} < x ', x ' >
n

Lema: (Teorema do Min Max ): λ1 = max < Ax, x >,


x =1

λ 2 = min max < Ax, x > , para toda A matriz simétrica, onde λ1 ≥ λ 2 ≥ ... ≥ λ n são
dim S = n −1 x∈S
x =1

seus autovalores.

Prova: Como A é simétrica existe base ortonormal x1 ,..., xn tal que Axi = λi xi logo,
se x = a1 x1 + ... + an xn , < Ax, x > = λ1a12 + λ 2 a22 + ... + λ n an2 ≤ λ1 (a12 + ... + an2 ) = λ1 ⇒
max < Ax, x > ≤ λ1 , mas, tomando x = x1 , < Ax1 , x1 > = λ1 . Para o λ 2 , seja L
subespaço gerado por x1 e x2 .
Assim, dim S = n − 1 ⇒ max < Ax, x > ≥ max < Ax, x > , mas se y = a1 x1 + a2 x2 ∈ L ∩ S ,
x∈S x∈L ∩S
x =1 x =1

a12 + a22 = 1 ⇒ max < Ax, x > ≥ < Ay, y > = λ1a12 + λ2 a22 ≥ λ2 (a12 + a22 ) = λ2 ⇒
x∈L∩S
x =1

min max < Ax, x > ≥ λ 2 . Agora, tomando S = ( x1 ) ⊥ temos que o mínimo é
dim S = n −1 x∈S
atingido.

] A
PROBLEMA 4: SOLUÇÃO DE YURI GOMES LIMA (FORTALEZA - CE)
Seja Q = (b1 ,..., bn ). Sejam também Pj = ( a j1 ,..., a jn ), a jA ∈ , 1 ≤ j ≤ k , 1 ≤ ≤ n.

Observe que
Pj − Q
∉ ] n  a j 1 − b1
⇔
a −b
,..., jn n

∉ ] n
⇔ existe i j ∈{1,2,..., n} tal
p  p p 
a ji j − bi j
que
p
∉ ] ⇔ p | (a ji j
− bi j )

Tome então
k
f ( x1 ,..., xn ) = ∏ ( xi j − a ji j )
j =1

Note que:

EUREKA! N°22, 2005

59
Sociedade Brasileira de Matemática

k
f ( x1 ,..., xn ) = ( xi j − a ji j )∏ ( xi − a ji ) = ( xi j − a ji ) R j ( x1 ,..., xn ) ⇒
)
A=1
) j

A≠ j
⇒ f ( Pj ) = f (a j ,..., a jn ) = (a ji j − a ji j ) R j (a j1 ,..., a jn ) = 0 ⇒ f ( Pj ) = 0
k
Ademais, f (Q) = f (b1 ,..., bn ) = ∏ (bij − a ji j ). Como p | (bij − a ji j ), ∀j, e p é primo
j =1

segue que p | f (Q) ⇒


f (Q)
p
∉ ] (c.q.d.)
`
PROBLEMA 5: SOLUÇÃO OFICIAL
Consideremos a seqüência a0 , a1 , a2 ,... com an ∈{0,1}, ∀n ∈ tal que an = 1 se
Arnaldo tem estratégia vencedora recebendo N0 = n e an = 0, caso contrário, i.e.,
caso Bernaldo tenha estratégia vencedora se Arnaldo recebe N0 = n . É fácil ver que,
 km + r  k
para k ≥ 1, se ak = 0, akm+ r = 1 para 0 ≤ r ≤ m − 1 (nesse caso   = ), e, se
 m 
1 − ( −1)r 0, se r é par
ak = 1 caso akm −1 = 1, temos akm +r = = para 0 ≤ r ≤ m − 1, e,
2 1, se r é ímpar
1 + ( −1) r 1, se r é par
caso akm −1 = 0, temos, para 0 ≤ r ≤ m − 1, akm+ r = = .
2 0, se r é ímpar
Suponhamos inicialmente que m é par. Assim, pela discussão acima, se
m m
ak = 1, para valores de r, com 0 ≤ r ≤ m − 1, temos akm +r = 0, e para os demais
2 2
valores de r nesse intervalo temos akm +r = 1. Assim, se Bn = {0 ≤ k < n ak = 0} e
m
An = {0 ≤ k < n ak = 1}, temos B n + A n = n , Bmn = ⋅ Vn + 1 e
2
m
Amn = m ⋅ Un + ⋅Vn −1, ∀n ≥ 1, donde, como An = n − Bn , temos
2
m Bmn 1  B  1
Bmn = ⋅ ( n − Bn ) + 1, e logo = ⋅ 1 − n  + . Como a solução de
2 mn 2  n  mn
1 1 Bn 1 1 1
t = (1 − t ) é t = , fazemos = + δ n , e obtemos δ mn = − δ n + . De
2 3 n 3 2 mn
Bmn Bmn+r Bmn + m −1
Bmn ≤ Bmn+r ≤ Bmn + (m −1) para 0 ≤ r ≤ m −1, obtemos ≤ ≤ ,
m(n + 1) mn + r mn

EUREKA! N°22, 2005

60
Sociedade Brasileira de Matemática

1 m
donde, para 0 ≤ r ≤ m − 1, δ mn + r − δ mn < < , desde que n ≥ m − 1. Assim,
n + 1 mn + r
m 2m 1
se N ≥ m(m − 1), δ n ≤ + δ m ⋅ N / m  ≤ + δ , de onde é fácil provar por
N N 2  N / m 
indução em k que, se k ≥ 2 e mk ≤ n < mk +1 , temos δ n ≤ k / 2k − 2 , donde lim δ n = 0.
n→∞

Bn 1 A n − Bn B 2
Portanto, se m é par lim = , donde lim n = lim = 1 − lim n = , e
n →∞ n 3 n →∞ n n →∞ n n →∞ n 3
An A n 23
lim = lim n = = 2.
n →∞ Bn n →∞ Bn n 1 3

Suponha agora m ímpar. Para cada seqüência finita α de 0's e 1's denotaremos por


α n a seqüência αα ...α obtida pela concatenação de n cópias da seqüências α .


n vezes

No que se segue, usaremos o termo bloco para denotar uma seqüência do tipo 1r 0 ,
com r ≥ 1. Veremos agora que (an )n ≥m +1 é obtida por uma concatenação de blocos
dos tipos 10, 1m 0 e 1m+10.
m ( m −1)
−1
A seqüência am +1 am +2 ...am( m +1) é (10) 2
⋅ 1m+10. Vamos mostrar por indução que,
 mk − 1 m k +1 − m m k +1 − 1 
para todo k ≥ 2, a seqüência β k =  a j , ≤ j≤ = − 1 é obtida
 m −1 m −1 m −1 
m
por uma concatenação de blocos dos tipos 10 e 1 0 , seguida por um único bloco de
m2 − 1 m3 − m
tipo 1m+10 (note que = m +1 e = m(m + 1), e logo já verificamos o
m −1 m −1
caso k = 2).
Se a j −1 = 0, a j +t = 1 para 0 ≤ t ≤ r − 1 e a j +r = 0, temos am ( j +r ) +s = am ( j +1)+s = 1 para
1 − (−1) s 0, se s é par
0 ≤ s ≤ m − 1, e, para 0 ≤ s ≤ mr − 1, amj + s = = . Assim, se
2 1, se s é ímpar
r = 1 ou r = m, (amj + s ) 0≤ s ≤mr −1 começa e termina por 0, e, se r = m + 1, (amj + s ) 0≤ s ≤mr −1
começa por 0 e termina por 1. Logo, blocos dos tipos 10 e 1m 0 geram apenas blocos
desses mesmos tipos, e blocos do tipo 1m+1 só podem ser gerados por blocos do tipo
1m+10 (eles aparecem no fim da seqüência gerada por 1m+10 ). Essas observações
provam a nossa afirmação.

EUREKA! N°22, 2005

61
Sociedade Brasileira de Matemática

Como os únicos blocos diferentes desses que aparecem são os do tipo 1m+10 de
m k +1 − m
tamanho total m + 2, os quais só aparecem entre as posições −m e
m −1
m k +1 − m m k +1 − m
, k ≥ 2, e como > m k + m, temos
m 1 − m 1 −

 log n  ( m + 2) log n
2un + (m + 1)vn ≥ n − ( m + 1) − (m + 2) ⋅  − 1 ≥ n − , donde
 log m  log m
2u + (m + 1)vn
lim n = 1. Além disso, pela discussão acima, seqüências dos tipos 10 e
n →∞ n

1m 0 geram seqüências desses mesmos tipos, donde


 m −1   m −1 
2
umn =  un +   vn + rn e vmn = un + vn + sn , para n ≥ m + 1, onde
 2   2 
m 2 log n log n
rn ≤ e sn ≤ os desvios rn e sn se devem a blocos gerados por
log m log m
log n
blocos incompletos e pelos blocos do tipo 1m+10, que são no máximo em
log m
( m + 2) log n
(ak ) k ≤n ). Como vimos acima, 2un + (m + 1)vn = n − wn , onde wn ≤ .
log m
u v
Sejam xn = n e yn = m n . Temos un + mvn ≤ An ≤ un + mvn + wn , donde
n n
A  log n 
xn + yn = n + O  , e, de 2un + (m + 1)vn = n − wn , segue que
n  n 
m +1  log n 
2 xn + ⋅ yn = 1 − O  .
m  n 

De Vmn = un + vn + sn e de vN − v N
≤ 1 segue que vN = u N  + v N  + O(log N ),
m   m m
 m    

1  log N  1  m + 1  1  log N 
donde y N = x N  + y N  + O   = 1 − ⋅ y  N   + y N  + O  =
m  m   N  2 m   N 
 m   m 
 m  m 

EUREKA! N°22, 2005

62
Sociedade Brasileira de Matemática

1 m −1  log N  m
= − y N  + O   . Escrevendo yn = + ε (note que a solução de
2 2m  m   N  3m − 1 n
1 m −1 m  m −1   log N 
t= − ⋅t é t= ) , temos ε N = −   ⋅εN  + O , donde
2 2m 3m − 1  2m   m   N 
m −1
0 ≤ limsup ε n ≤ 1 e limsup ε n ≤ ⋅ limsup ε n + 0, e logo limsup ε n = 0, ou
n→∞ n →∞ 2m n →∞ n→∞

m 1  m +1   log n 
seja, lim ε n = 0 , donde lim yn = e, de xn = 1 − ⋅ yn  + O  ,
n→∞ n →∞ 3m − 1 2 m   n 
1  m + 1 m  m −1 A  log n 
segue que lim xn = 1 − ⋅  = . De xn + yn = n + O  ,
n →∞ 2 m 3m − 1  3m − 1 n  n 
An 2m − 1 B A m
segue que lim = lim ( xn + yn ) = , e lim n = 1 − lim n = .
n →∞ n n →∞ −
3m 1 n →∞ n n →∞ n 3m − 1
A A n (2m − 1) (3m − 1) 2m − 1
Portanto, se m é ímpar, lim n = lim n = = .
n →∞ B
n
n →∞ B
n n m (3m − 1) m

PROBLEMA 6: SOLUÇÃO OFICIAL


(do Prof. Alexandre Fernandes, comunicada pelo Prof. Lev Birbrair, de Fortaleza, CE)
Suponha por absurdo que exista um contra-exemplo γ . Defina α :[0,2π ] → \ 2
por
α (t ) = γ (t ) − γ (t + π ), para 0 ≤ t ≤ π e α (t ) = γ (t ) − γ (t − π ) para π ≤ t ≤ 2π .
Pela hipótese de absurdo temos α (t ) > 2 para todo t. A função α é contínua (pois
γ (0) = γ (2π )) e derivável exceto talvez para t = 0, π .
Temos ainda α '(t ) ≤ γ '(t ) + γ '(t + π ) = 2 para todo t, donde o comprimento da
curva α é menor ou igual a 4π .
Mas a imagem de α é uma curva fechada simétrica em relação à origem (pois
α (t + π ) = −α (t ), ∀t ∈[0, π ]), que está no exterior da bola de raio 2, donde seu
comprimento é maior que 4π , absurdo.
α (0)

α(π) = – α (0)

EUREKA! N°22, 2005

63
Sociedade Brasileira de Matemática

XXVI OLIMPÍADA BRASILEIRA DE MATEMÁTICA


Resultado – Nível 1 (5a. e 6a. Séries)
NOME CIDADE – ESTADO PRÊMIO
Illan Feiman Halpern Itatiaia – RJ Ouro
Leonardo Pereira Stédile São Paulo – SP Ouro
James Jung Hong São Paulo – SP Ouro
Victor Reis de Abreu Cavalcanti Maceió – AL Ouro
Wagner Carlos Morêto Loyola Filho Vitória – ES Ouro
Rafael Alves da Silva Teresina – PI Ouro
Marcelo Tadeu de Sá Oliveira Sales Barreiras – BA Prata
Lereno Soares Netto Rio de Janeiro – RJ Prata
Dan Zylberglejd Rio de Janeiro – RJ Prata
Clarissa Maria Grosso Fabris São Paulo – SP Prata
Matheus Barros de Paula Taubaté – SP Prata
Renan Henrique Finder Joinville – SC Prata
Leonardo Shinizu Yojo São Paulo – SP Prata
Ana Luiza Ferron Zanella Pato Branco – PR Prata
Fernanda Daltro Costa Knoblauch Salvador – BA Prata
Thiago Ribeiro Ramos Varginha – MG Prata
Lucas Gouveia Omena Lopes Maceió – AL Prata
José Rodolfo Bezerra Mesquita Araújo João Pessoa – PB Prata
Débora Kiame Paias Atibaia – SP Bronze
Daniel Caueh Dunaiski Figueira Leal Curitiba – PR Bronze
Alessandro D'Alessio de Souza São Paulo – SP Bronze
Eduardo Kaiser Urubahy Nunes Fonseca Rezende – RJ Bronze
Caroline Rigolon Veiga Rio de Janeiro – RJ Bronze
Erick Magno Costa Alonso Uberaba – MG Bronze
Vinicius Tineli Paiva Vitória – ES Bronze
Marcelo Aires Moreira João Pessoa – PB Bronze
Rafael Fernandes Paixão Rio de Janeiro – RJ Bronze
Diana Vaisman Rio de Janeiro – RJ Bronze
Camila Miraglia Ribeiro Curitiba – PR Bronze
Stefano Tonnasini Itu – SP Bronze
Vitor Cruth Sturn São Paulo – SP Bronze
Hugo Fonseca Araújo Juiz de Fora – MG Bronze
Bárbara Seccato Ruis Chagas Vitória – ES Bronze
Rafael Sarto Müller Vitória – ES Menção Honrosa
Thiago Gonçales Piracicaba – SP Menção Honrosa
José Nilo Alves de Souza Neto Fortaleza – CE Menção Honrosa
Vitor Costa Fabris Criciúma – SC Menção Honrosa
Ana Luísa de Almeida Losnak São Paulo – SP Menção Honrosa
Rebeca Meirelles de Araújo Assis Salvador – BA Menção Honrosa
Gabriela Barini de Oliveira São José dos Campos – SP Menção Honrosa
Ítalo Leite de Camargo Assis – SP Menção Honrosa
Arthur Lerer Rio de Janeiro – RJ Menção Honrosa
André Saraiva Nobre dos Santos Fortaleza – CE Menção Honrosa
José Ailton Azevedo Araújo Filho Fortaleza – CE Menção Honrosa
César Ilharco Magalhães Juiz de Fora – MG Menção Honrosa
Mariana Hollos Fiorencio Rio de Janeiro – RJ Menção Honrosa
Fernando Fonseca Andrade Oliveira Belo Horizonte – MG Menção Honrosa
Beatriz Arruda Asfora Recife – PE Menção Honrosa
João Paulo Sousa Lucas Fortaleza – CE Menção Honrosa
Tayrini da Cruz Beligoli Juiz de Fora – MG Menção Honrosa
Mônica Monteiro Gondim Fortaleza – CE Menção Honrosa
Izabela Karennina Travizani Maffra Belo Horizonte – MG Menção Honrosa
Maíra Islena Tavares da Silva Belo Horizonte – MG Menção Honrosa
Caio Menezes Facó Fortaleza – CE Menção Honrosa
André Yoshio Oshiro Bastos São Paulo – SP Menção Honrosa
Natasha Scaranello Cartolano Rio Claro – SP Menção Honrosa
Laura Andrade Santiago Fortaleza – CE Menção Honrosa
Guilherme Alencar Sorensen Colares Fortaleza – CE Menção Honrosa
Eduardo de Almeida Grande Valinhos – SP Menção Honrosa
Breno Augusto Cardoso Barroso Fortaleza – CE Menção Honrosa

EUREKA! N°22, 2005

64
Sociedade Brasileira de Matemática

Resultado – Nível 2 (7a. e 8a. Séries)


NOME CIDADE – ESTADO PRÊMIO
Lucio Eiji Assaoka Hosaka Curitiba – PR Ouro
Regis Prado Barbosa Fortaleza – CE Ouro
Henrique Pondé de Oliveira Pinto Salvador – BA Ouro
Rafael Sampaio de Rezende Fortaleza – CE Ouro
Wilson Camara Marriel Rio de Janeiro – RJ Ouro
Rafael Tupynambá Dutra Belo Horizonte – MG Prata
Adenilson Arcanjo de Moura Júnior Fortaleza – CE Prata
José Airton Coêlho Lima Filho Fortaleza – CE Prata
Filipe Alves Tomé Fortaleza – CE Prata
Rafael Moura e Sucupira Fortaleza – CE Prata
Ramon Moreira Nunes Fortaleza – CE Prata
Rodrigo Clemente de Brito Pereira João Pessoa – PB Prata
Sérgio Ricardo Furtado Sampaio Filho Fortaleza – CE Prata
Giuliano Pezzolo Giacaglia Santo André – SP Prata
Daniel Ungaretti Borges Belo Horizonte – MG Bronze
Gabriel Marcos Pasmanik Eisencraft São Paulo – SP Bronze
Anna Kelly Krislane de Vasconcelos Fortaleza – CE Bronze
Paulo Vitor de Souza Albuquerque Rio de Janeiro – RJ Bronze
Artur de Almeida Losnak São Paulo – SP Bronze
Ricardo Turolla Bortolotti Rio Claro – SP Bronze
Henrique Hiroshi Motoyama Watanabe São Paulo – SP Bronze
Guilherme Philippe Figueiredo Fortaleza – CE Bronze
Alfredo Roque de Oliveira Freire Filho Salvador – BA Bronze
Lucas Zanotto Portela Curitiba – PR Bronze
Nathana Alcântara Lima Fortaleza – CE Bronze
Flávio Domingos de Azevedo Quadros Osasco – SP Bronze
Mateus Oliveira de Figueiredo Fortaleza – CE Bronze
Marilia Valeska Costa Medeiros Fortaleza – CE Bronze
Grazielly Muniz da Cunha Fortaleza – CE Menção Honrosa
Fábio Yoshiteru Nukui São Paulo – SP Menção Honrosa
Ana Luiza Viegas de Almeida Marília – SP Menção Honrosa
Vinicius Marques Regitano Piracicaba – SP Menção Honrosa
André Antonio Battagello Araçatuba – SP Menção Honrosa
Anderson Unlin Tsai São Paulo – SP Menção Honrosa
Mateos Kruchelski Tschá Curitiba – PR Menção Honrosa
Caio Gustavo Mesquita Angelo Taguatinga – DF Menção Honrosa
Guilherme Zatieff Topolski Chaves São Paulo – SP Menção Honrosa
Marlen Lincoln da Silva Fortaleza – CE Menção Honrosa
Janailson Rodrigues Lima Fortaleza – CE Menção Honrosa
Gilson Mauro Costa Fernandes Filho João Pessoa – PB Menção Honrosa
Diogo Bonfim Moraes Morant de Holanda Rio de Janeiro – RJ Menção Honrosa
Paulo Wayner Carvalho dos Santos Rio de Janeiro – RJ Menção Honrosa
Douglas Tirre Carnevale Oliveira Rio de Janeiro – RJ Menção Honrosa
Ivan Hitoshi Oyama Osasco – SP Menção Honrosa
Marlon Vieira de Lima Júnior Fortaleza – CE Menção Honrosa
Cássio Kendi Takamori São José dos Campos – SP Menção Honrosa
Jessica de Aguiar França Brasília – DF Menção Honrosa
Karin Naomi Harada de Oliveira São Paulo – SP Menção Honrosa
Diogo Soares Dórea da Silva Feira de Santana – BA Menção Honrosa
Mauricio Henrique Bezerra Cardoso Aracajú – SE Menção Honrosa
Érik Fernando de Amorim Araraquara – SP Menção Honrosa
Caio Prado Siqueira Campos Fortaleza – CE Menção Honrosa
Luan Leal Oliveira Tremembé – SP Menção Honrosa
Diego Eloi Misquita Gomes Fortaleza – CE Menção Honrosa
Cintia Mayumi Sakurai Kimura São Paulo – SP Menção Honrosa
Carolina Santos Andrade São Cristóvão – SE Menção Honrosa

EUREKA! N°22, 2005

65
Sociedade Brasileira de Matemática

Resultado – Nível 3 (Ensino Médio)


NOME CIDADE – ESTADO PRÊMIO
Fábio Dias Moreira Rio de Janeiro – RJ Ouro
Rafael Marini Silva Vila Velha – ES Ouro
Gabriel Tavares Bujokas São Paulo – SP Ouro
Rafael Daigo Hirama Campinas – SP Ouro
Guilherme Rodrigues Nogueira de Souza São Paulo – SP Ouro
Telmo Luis Correa Junior Santo André – SP Ouro
Thiago Costa Leite Santos São Paulo – SP Prata
Thomás Yoiti Sasaki Hoshina Rio de Janeiro – RJ Prata
Larissa Cavalcante Queiroz de Lima Fortaleza – CE Prata
Henry Wei Cheng Hsu São Paulo – SP Prata
Elton Gomes Coriolano Fortaleza – CE Prata
Larissa Rodrigues Ribeiro Fortaleza – CE Prata
Rodrigo Aguiar Pinheiro Fortaleza – CE Prata
Felipe Rodrigues Nogueira de Souza São Paulo – SP Prata
Ricardo Mizoguchi Gorgoll São Paulo – SP Prata
Guilherme Rodrigues Salerno Goiânia – GO Bronze
Edson Augusto Bezerra Lopes Fortaleza – CE Bronze
Luiz Adolfo Schiller Niterói – RJ Bronze
Marcus Edson Barreto Brito Fortaleza – CE Bronze
Lucas Pavan Barros Vila Velha – ES Bronze
Diogo dos Santos Suyama Belo Horizonte – MG Bronze
Felipe Ferreira Villar Coelho Serra – ES Bronze
José Marcos Andrade Ferraro São Paulo – SP Bronze
Francisco Bruno de Lima Holanda Fortaleza – CE Bronze
Luty Rodrigues Ribeiro Fortaleza – CE Bronze
Israel Franklin Dourado Carrah Fortaleza – CE Bronze
Douglas Bokliang Ang Cunha São José dos Campos – SP Bronze
Levi Máximo Viana Fortaleza – CE Bronze
Leandro Farias Maia Fortaleza – CE Bronze
Victor de Andrade Lazarte São Paulo – SP Bronze
Rafael Mendes de Oliveira Mesquita – RJ Menção Honrosa
Raphael Rodrigues Mata Salvador – BA Menção Honrosa
Thiago Jorge Marinho Vieira Fortaleza – CE Menção Honrosa
Ana Cláudia Onuchic São Paulo – SP Menção Honrosa
Paulo Victor Teixeira Eufrásio Fortaleza – CE Menção Honrosa
Carlos Augusto David Ribeiro Fortaleza – CE Menção Honrosa
Eduardo Fischer Encantado – RS Menção Honrosa
Edson Leandro Finotti Bittar São Paulo – SP Menção Honrosa
Leandro Farias Nogueira Recife – PE Menção Honrosa
Sheyne Cristina Leal Rio de Janeiro – RJ Menção Honrosa
Hugo Caetano da Silva Junior Teresina – PI Menção Honrosa
Erasto Villa Branco Neto Curitiba – PR Menção Honrosa
Antonia Taline de Souza Mendonça Fortaleza – CE Menção Honrosa
José Gerardo Arruda Junior Fortaleza – CE Menção Honrosa
José Mário da Silva Filho Fortaleza – CE Menção Honrosa
Bruno Cardoso Vieira Rio de Janeiro – RJ Menção Honrosa
Pedro Henrique Milet Pinheiro Pereira Rio de Janeiro – RJ Menção Honrosa
Mateus Gomes Filgueiras Fortaleza – CE Menção Honrosa
Vicente Matheus Moreira Zuffo São Paulo – SP Menção Honrosa

EUREKA! N°22, 2005

66
Sociedade Brasileira de Matemática

Resultado – Nível Universitário


NOME CIDADE - ESTADO PRÊMIO
Carlos Stein Naves de Brito São José dos Campos – SP Ouro
Alex Corrêa Abreu Niterói – RJ Ouro
Humberto Silva Naves São José dos Campos – SP Ouro
Bernardo Freitas Paulo da Costa Rio de Janeiro – RJ Ouro
Daniel Massaki Yamamoto São Paulo – SP Ouro
Murilo Vasconcelos Andrade Rio de Janeiro – RJ Ouro
Davi Máximo Alexandrino Nogueira Fortaleza – CE Prata
Carlo Pietro Sousa da Silva São Cristóvão – SE Prata
Yuri Gomes Lima Fortaleza – CE Prata
Tertuliano Franco Santos Franco Salvador – BA Prata
Rodrigo Roque Dias São Paulo – SP Prata
Diêgo Veloso Uchôa Teresina – PI Prata
Pietro Kreitlon Carolino Campinas – SP Prata
Thiago Barros Rodrigues Costa Fortaleza – CE Prata
Thiago da Silva Sobral Fortaleza – CE Prata
Leonardo Augusto Zão Nilópolis – RJ Bronze
Samuel Barbosa Feitosa Fortaleza – CE Bronze
Eduardo Famini Silva Rio de Janeiro – RJ Bronze
Einstein do Nascimento Júnior Fortaleza – CE Bronze
Eduardo Bertoldi Vila Velha – ES Bronze
Rafael Tajra Fonteles Teresina – PI Bronze
Diogo Diniz Pereira da Silva e Silva Campina Grande – PB Bronze
Kellem Corrêa Santos Rio de Janeiro – RJ Bronze
Domingos Dellamonica Junior São Paulo – SP Bronze
Luis Daniel Barbosa Coelho São José dos Campos – SP Bronze
Jorge Peixoto de Morais Neto São José dos Campos – SP Bronze
Anne Caroline Bronzi Ribeirão Preto – SP Bronze
Evandro Makiyama de Melo São Paulo – SP Menção Honrosa
Helder Oliveira de Castro Mogi das Cruzes – SP Menção Honrosa
Moyses Afonso Assad Cohen Rio de Janeiro – RJ Menção Honrosa
Rogério de Assis Medeiros Franco da Rocha – SP Menção Honrosa
Rodrigo Angelo Muniz Cariacica – ES Menção Honrosa
Fabrício Siqueira Benevides Fortaleza – CE Menção Honrosa
Giuliano Boava Florianópolis – SC Menção Honrosa
Filipe Rodrigues de Souza Moreira São José dos Campos – SP Menção Honrosa
Antonio Carlos Maldonado S. A. Munhoz Rio de Janeiro – RJ Menção Honrosa
Anderson Rodrigues Ferreira Rio de Janeiro – RJ Menção Honrosa
Eduardo Ferraz Castelo Branco Ferreira Rio de Janeiro – RJ Menção Honrosa
Arthur Omar de Andrade Lazarte São Paulo – SP Menção Honrosa
Vitor Gabriel Kleine Mogi das Cruzes – SP Menção Honrosa

EUREKA! N°22, 2005

67
Sociedade Brasileira de Matemática

AGENDA OLÍMPICA
XXVII OLIMPÍADA BRASILEIRA DE MATEMÁTICA

NÍVEIS 1, 2 e 3
Primeira Fase – Sábado, 11 de junho de 2005
Segunda Fase – Sábado, 03 de setembro de 2005
Terceira Fase – Sábado, 22 de outubro de 2005 (níveis 1, 2 e 3)
Domingo, 23 de outubro de 2005 (níveis 2 e 3 - segundo dia de prova).

NÍVEL UNIVERSITÁRIO
Primeira Fase – Sábado, 03 de setembro de 2005
Segunda Fase – Sábado, 22 e Domingo, 23 de outubro de 2005

XI OLIMPÍADA DE MAIO
14 de maio de 2005

XVI OLIMPÍADA DE MATEMÁTICA DO CONE SUL
23 a 28 de maio de 2005
Sucre, Bolívia

XLVI OLIMPÍADA INTERNACIONAL DE MATEMÁTICA
08 a 16 de julho de 2005
Yucatán, México

XII OLIMPÍADA INTERNACIONAL DE MATEMÁTICA UNIVERSITÁRIA
22 a 28 de julho de 2005
Blagoevgrad, Bulgária

XX OLIMPÍADA IBEROAMERICANA DE MATEMÁTICA
24 a 30 de setembro de 2005
Cartagena de Índias, Colômbia

VIII OLIMPÍADA IBEROAMERICANA DE MATEMÁTICA UNIVERSITÁRIA
19 de novembro de 2005

♦♦♦

EUREKA! N°22, 2005

68
Sociedade Brasileira de Matemática

COORDENADORES REGIONAIS
Alberto Hassen Raad (UFJF) Juiz de Fora – MG
Américo López Gálvez (USP) Ribeirão Preto – SP
Amarísio da Silva Araújo (UFV) Viçosa – MG
Ana Paula Bernardi da Silva (Universidade Católica de Brasília) Brasília – DF
Antonio Carlos Nogueira (UFU) Uberlândia – MG
Ali Tahzibi (USP) São Carlos – SP
Benedito Tadeu Vasconcelos Freire (UFRN) Natal – RN
Carlos Frederico Borges Palmeira (PUC-Rio) Rio de Janeiro – RJ
Claus Haetinger (UNIVATES) Lajeado – RS
Cleonor Crescêncio das Neves (UTAM) Manaus – AM
Cláudio de Lima Vidal (UNESP) S.J. do Rio Preto – SP
Edson Roberto Abe (Colégio Objetivo de Campinas) Campinas – SP
Élio Mega (Colégio Etapa) São Paulo – SP
Éder Luiz Pereira de Andrade (UNESPAR/FECILCAM) Campo Mourão – PR
Eudes Antonio da Costa (Univ. do Tocantins) Arraias – TO
Florêncio Ferreira Guimarães Filho (UFES) Vitória – ES
Ivanilde Fernandes Saad (UC. Dom Bosco) Campo Grande– MS
Jacqueline Fabiola Rojas Arancibia (UFPB) João Pessoa – PB
Janice T. Reichert (UNOCHAPECÓ) Chapecó – SC
João Benício de Melo Neto (UFPI) Teresina – PI
João Francisco Melo Libonati (Grupo Educacional Ideal) Belém – PA
José Carlos dos Santos Rodrigues (Unespar) Campo Mourão – PR
José Cloves Saraiva (UFMA) São Luis – MA
José Gaspar Ruas Filho (ICMC-USP) São Carlos – SP
José Luiz Rosas Pinho (UFSC) Florianópolis – SC
José Vieira Alves (UFPB) Campina Grande – PB
Krerley Oliveira (UFAL) Maceió – AL
Licio Hernandes Bezerra (UFSC) Florianópolis – SC
Luzinalva Miranda de Amorim (UFBA) Salvador – BA
Mário Rocha Retamoso (UFRG) Rio Grande – RS
Marcelo Rufino de Oliveira (Grupo Educacional Ideal) Belém – PA
Marcelo Mendes (Colégio Farias Brito, Pré-vestibular) Fortaleza – CE
Marilane de Fraga Sant'Ana FACOS Osório – RS
Pablo Rodrigo Ganassim (Liceu Terras do Engenho) Piracicaba – SP
Ramón Mendoza (UFPE) Recife – PE
Raúl Cintra de Negreiros Ribeiro (Colégio Anglo) Atibaia – SP
Ronaldo Alves Garcia (UFGO) Goiânia – GO
Reginaldo de Lima Pereira (Escola Técnica Federal de Roraima) Boa Vista – RR
Reinaldo Gen Ichiro Arakaki (UNIVAP) SJ dos Campos – SP
Ricardo Amorim (Centro Educacional Logos) Nova Iguaçu – RJ
Sérgio Cláudio Ramos (IM-UFRGS) Porto Alegre – RS
Seme Guevara Neto (UFMG) Belo Horizonte – MG
Tadeu Ferreira Gomes (UEBA) Juazeiro – BA
Tomás Menéndez Rodrigues (U. Federal de Rondônia) Porto Velho – RO
Valdenberg Araújo da Silva (U. Federal de Sergipe) São Cristovão – SE
Valdeni Soliani Franco (U. Estadual de Maringá) Maringá – PR
Vânia Cristina Silva Rodrigues (U. Metodista de SP) S.B. do Campo – SP
Wagner Pereira Lopes (CEFET – GO) Jataí – GO

EUREKA! N°22, 2005

69
CONTEÚDO

AOS LEITORES 2

XIX OLIMPÍADA IBEROAMERICANA DE MATEMÁTICA 3


Enunciados e Soluções

ARTIGOS
PESSOAS CONFIÁVEIS E NÃO-CONFIÁVEIS 10
Svetoslav Savchev

O TEOREMA DE STOLZ 15
Diêgo Veloso Uchôa & Renato Purita Paes Leme

CONTAS COM DESIGUALDADES 23


Márcio Afonso Assad Cohen & Rodrigo Villard Milet

O PROBLEMA IMPOSSÍVEL 32
Cássio Neri

COMO É QUE FAZ? 40

SOLUÇÕES DE PROBLEMAS PROPOSTOS 44

PROBLEMAS PROPOSTOS 59

AGENDA OLÍMPICA 61

COORDENADORES REGIONAIS 62
Sociedade Brasileira de Matemática

AOS LEITORES

Neste número publicamos um artigo do Prof. Svetoslav Savchev, da


Bulgária, um dos maiores especialistas internacionais em problemas de olimpíada,
além de três outros artigos de temas variados.
Nos tem alegrado bastante a boa qualidade de artigos e problemas enviados
por nossos colaboradores, que contribuem muito para a qualidade e a vitalidade de
nossa revista.

Publicamos também, além das seções de Problemas propostos e Como é que


faz, soluções da Olimpíada Ibero-americana de 2004, na qual todos os quatro
integrantes da equipe brasileira (Alex Corrêa Abreu, Fábio Dias Moreira, Gabriel
Tavares Bujokas e Rafael Daigo Hirama) conquistaram medalha de ouro. Como a
equipe de 2005 repetiu o feito, publicaremos no próximo número as suas soluções.

Aproveitamos também para registrar a realização da IX Semana Olímpica,


atividade que vem sendo realizada desde 1998. Nesta oportunidade o evento teve
lugar na cidade de Juiz de Fora – MG entre os dias 21 a 27 de janeiro de 2006.
Durante a Semana Olímpica 2006, reunimos alunos ganhadores da XXVII
Olimpíada Brasileira de Matemática nos seus quatro níveis de competição. Estes
alunos participaram de um treinamento intensivo com professores de diversas partes
do país como preparação para a futura formação das equipes que representarão o
Brasil em Olimpíadas Internacionais.

Os editores

EUREKA! N°23, 2006

2
Sociedade Brasileira de Matemática

XIX OLIMPÍADA IBEROAMERICANA DE MATEMÁTICA


Enunciados e Soluções
PRIMEIRO DIA
PROBLEMA 1
Deve-se colorir as casas de um tabuleiro 1001 × 1001 de acordo com as seguintes
regras:
Se duas casas têm um lado comum, então pelo menos uma delas deve ser colorida.
De cada seis casas consecutivas de uma linha ou de uma coluna, devem colorir-se
sempre pelo menos duas delas que sejam adjacentes.
Determinar o número mínimo de casas que devem ser coloridas.

SOLUÇÃO DE FÁBIO DIAS MOREIRA (RIO DE JANEIRO - RJ)


Lema: Todo retângulo 5 × 1 tem pelo menos três quadrados pintados.
Prova: Suponha que não, que o tabuleiro possui algum retângulo 5 × 1 (ou 1 × 5)
com dois ou menos quadrados pintados. Então, pela primeira condição, é
trivialmente impossível que só 1 ou 0 quadrados estejam pintados e, se dois
quadrados estiverem pintados, eles devem ser os da figura abaixo:
B

Mas então tomamos um retângulo 6 × 1 que contenha o 5 × 1 e que esteja contido no


tabuleiro (isto é sempre possível pois 5 < 1001).
Independente do quadrado agregado ser ou não colorido, chegamos a um absurdo
pois a segunda condição é violada.
Decompomos agora o tabuleiro em 200400 retângulos 5 × 1 e um quadrado 1 × 1
(200000 retângulos 5 × 1 na horizontal, contidos no quadrado 1000 × 1000 do canto
superior esquerdo, 200 na horizontal, no retângulo 1000 × 1 na borda inferior, e 200
na vertical, no retângulo 1 × 1000 na borda direita e o quadrado 1 × 1 no canto
inferior direito).
1000 1

1000 200000 200

1 200

EUREKA! N°23, 2006

3
Sociedade Brasileira de Matemática

Em cada um desses retângulos, pelo menos três quadrados estão coloridos, logo pelo
menos 601200 casas devem ser coloridos.
E, de fato, este mínimo pode ser atingido: numere as colunas e as linhas do tabuleiro
de 1 a 100. Pinte o quadrado (i, j) se somente se, i + j ≡ 1, 3 ou 4 (mód. 5). Cada um
dos retângulos 5 × 1 que criamos contém exatamente um quadrado de cada classe de
congruência, logo pintamos 601200 casas, pois o quadrado que não foi contado,
(1001, 1001), não é pintado (pois 1001 + 100 ≡ 2 (mód 5)).
Além disso é fácil ver que qualquer retângulo 6 × 1 contém duas casas adjacentes
pintadas.

PROBLEMA 2
Considera-se no plano uma circunferência de centro O e raio r, e um ponto A
exterior a ela. Seja M um ponto da circunferência e N o ponto diametralmente
oposto a M. Determinar o lugar geométrico dos centros das circunferências que
passam por A, M e N quando M varia.

SOLUÇÃO DE ALEX CORRÊA ABREU (NITERÓI - RJ)


A

P
O'
O

B
N

Seja Γ a circunferência de centro O e raio r, Γ ' a circunferência qua passa por A,


HJJG
M, N; O' seu centro, B = AO ∩ Γ ' e P ∈ AO tal que OPO l ' = 90°. Então
obviamente AP = PB. Agora como O está no eixo radical de Γ e
r2 r2
Γ ' ⇒ OA ⋅ OB = OM ⋅ ON = r 2 ⇒ OB = . Chamando OA = d ,temos OB = ,
OA d
2
r
d+
d = d −r
2 2

logo P é tal que OP = OA − AP = d − , e logo é fixo e portanto


2 2d
todos os centros O' estão na reta por P perpendicular a AO quando M varia.

EUREKA! N°23, 2006

4
Sociedade Brasileira de Matemática

Mas para todo O '' tq O '' PO l = 90° ⇒ se traçarmos Γ '' circunferência com centro
em O'' que passa por A ele também irá passar por B logo a potência de O será r 2 ⇒
se {M '', N ''} = Γ ∩ Γ '' então OM '' ⋅ ON '' = r 2 e logo M'' e N'' são diametralmente
opostos.
Assim, o lugar geométrico dos centros é uma reta perpendicular a AO que passa pelo
OA2 − r 2
ponto P tal que OP = na direção de A. (cqd).
2OA

PROBLEMA 3
Sejam n e k números inteiros positivos tais que n é ímpar ou n e k são pares. Provar
que existem inteiros a e b tais que

mdc (a, n) = mdc(b, n) = 1 e k = a + b.

SOLUÇÃO DE RAFAEL DAIGO HIRAMA (CAMPINAS - SP)


Devemos achar então, para n e k inteiros positivos, um a tal que mdc(a, n) =
= mdc (k – a, n) = 1 pois b = k – a.
mdc (k – a, n) = mdc(a – k, n), por definição.
Então devemos achar dois números que distam k, primos com n.
Deve ser útil o Teorema Chinês dos Restos.
Fatorando: n = p1α1 p2α2 ... ptαt , com pi ' s primos e α ,i s inteiros positivos e seja
k ≡ βi (mod pi ) com 0 ≤ βi < pi
Se trocarmos a – k por c queremos que:
c ≡ 0(mod pi ) e a = c + k ≡ 0 (mod pi) para todo i inteiro, 1 ≤ i ≤ t
Se cada um dos pares de congruência for obedecida, escolhendo as congruências
módulo pi e usando o Teorema Chinês dos Restos podemos achar tal c (pois cada
par de pi ' s são primos entre si).
Então precisamos provar que existe possibilidade para a congruência módulo pi
para o c. Queremos c ≡ 0(mod pi ) e c ≡ −k (mod pi ) .
Se pi for maior que 2, temos pelo menos 3 classes de congruências distintas, e
tirando essas duas: 0 e pi − β i , sobra pelo menos uma. Se pi é 2, então n é par e
pelo enunciado, k também é e então k ≡ 0(mod 2). Assim não são tirados duas
classes de congruência módulo 2, e sim apenas a classe do 0. Podemos fazer então
c ≡ 1(mod 2).
Então o método para construir a e b é:

EUREKA! N°23, 2006

5
Sociedade Brasileira de Matemática

• Fatora-se n e pega-se seus fatores primos p1 , p2 ,..., pt .


• Acha-se as classes de congruência para cada pi , de forma que c ≡ 0(mod pi ) e
c + k ≡ 0(mod pi ),
que sempre existe como já provamos. Pode haver várias, então encolhe-se qualquer
uma.
• Pelo Teorema Chinês dos Restos, existe c que obedece tais congruências, logo
a = −c e b = c + k , que obedecem a condição a + b = k e mdc(a, n) = mdc(b, n) = 1,
pois pi | c ⇒ pi | − c = a e pi | c + k = b e, assim, a e n não têm fatores comuns, nem
b e n os têm.

Obs: Realmente, se n for par e k for ímpar não dá pois ou a ou b é par e então ou
2 | mdc (a, n) ou 2 | mdc (b, n).

PROBLEMA 4
Determinar todos os pares (a, b), onde a e b são números inteiros positivos de dois
dígitos cada um, tais que 100a + b e 201a + b são quadrados perfeitos de quatro
dígitos.

SOLUÇÃO:
Devemos ter 100a + b = m 2 e 201a + b = n 2 , onde m e n são inteiros tais que
0 < m < n < 100. Daí segue que n 2 − m 2 = 101a.
Como n 2 − m 2 = (n − m )(n + m ) é múltiplo de 101, 0 < n − m < 100 e
n + m < 200, devemos ter n + m = 101, pois 101 é primo, e 101 não divide n – m. Daí
101a 101 − a
segue que n − m = = a, logo (101 − m ) − m = a, donde m = . De
n+m 2
 101 − a 
2

100a + b = m 2 , segue que   = 100a + b, e logo a − 602a + 10201 = 4b.


2

 2 
Temos 0 ≤ 4b < 400. Como f ( a ) = a 2
− 602a + 10201 é uma função quadrática cujo
602
mínimo é atingido para a = = 301, f (a ) é decrescente para 0 ≤ a ≤ 100. Como
2
f (16) = 825 > 400, f (17) = 256 e f (18) = −311 < 0, devemos ter a = 17, donde
101 − 17 256
m= = 42, n = 101 − m = 59 e b= = 64 . Com efeito,
2 4
m 2 = 1764 = 100 ⋅ 17 + 64, e n 2 = 3481 = 201 ⋅17 + 64.

EUREKA! N°23, 2006

6
Sociedade Brasileira de Matemática

Assim, o único par (a, b) que satisfaz as condições do enunciado é (a, b) = (17, 64).

PROBLEMA 5
Dado um triângulo escaleno ABC, designam-se por A', B', C' os pontos de interseção
das bissetrizes interiores dos ângulos A, B e C com os lados opostos,
respectivamente.
Sejam: A'' a interseção de BC com a mediatriz de AA',
B'' a interseção de AC com a mediatriz de BB' e
C'' a interseção de AB com a mediatriz de CC'.
Provar que A'', B'' e C'' são colineares.

SOLUÇÃO:
Suponhamos a > b > c , sem perda de generalidade. A perpendicular a AA' passando
por A é a bissetriz externa do ângulo A no triângulo ABC. Sua interseção A E com a
E
A '+ A
reta BC é tal que A '' = , pelo teorema de Tales. Por outro lado, se
2
BC = a, AC = b e AB = c são os lados do triângulo ABC, pelo teorema das
BA ' CA ' bB + cC
bissetrizes temos = , donde A ' = , e, pelo teorema das bissetrizes
c b b+c
B EA C EA bB − cC
externas, = , donde EA= .
c b b−c
1  bB + cC bB − cC  b 2 B − c 2 C
Assim, A '' =  + = 2 2 .
2 b+c b −c  b −c
A

EA A'' B A' C

a 2 A − c 2C a2 A − b2 B
Analogamente, B '' = e C '' = . Assim,
a2 − c2 a2 − b2
b2 − c2 a2 − b2
( b 2
− c 2
) (
A ''+ a 2
− b2
) (
C '' = a 2
− c 2
) B '', donde B '' =
a2 − c2
A''+
a2 − c2
C '' = tA''+ (1− t)C '',

EUREKA! N°23, 2006

7
Sociedade Brasileira de Matemática

b2 − c2
com t = , e logo B '' ∈ A '' C '', o que resolve o problema.
a2 − c2

PROBLEMA 6
Para um conjunto H de pontos no plano, diz-se que um ponto P do plano é um ponto
de corte de H, se existem quatro pontos distintos A, B, C e D em H tais que as retas
AB e CD são distintas e se cortam em P.
Dado um conjunto finito A0 de pontos no plano, constrói-se uma sucessão de
conjuntos A1 , A2 , A3 ,... da seguinte forma: para qualquer j ≥ 0, Aj +1 é a união de
Aj com o conjunto de todos os pontos de corte de Aj .
Demonstrar que se a união de todos os conjuntos da sucessão é um conjunto finito
então, para qualquer j ≥ 1, tem-se Aj = A1 .

SOLUÇÃO:
A

F
G
D

H
B E C

Observemos inicialmente que nenhum dos conjuntos Aj pode conter 4 pontos A, B,


C, D tais que D é interior ao triângulo ABC. De fato, se
AD ∩ BC = {E}, BD ∩ AC = {F }, CD ∩ AB = {G} e BD ∩ EG = {H }, temos que
ABE é um triângulo estritamente contido em ABC e H é interior a ABE.
Repetindo indefinidamente esta construção, obtemos infinitos pontos na união dos
conjuntos An , absurdo.
D

F
E

EUREKA! N°23, 2006

8
Sociedade Brasileira de Matemática

Suponha agora que algum dos conjuntos Aj contenha os vértices de um quadrilátero


convexo ABCD. Se ABCD não é um paralelogramo, podemos supor que AB não é
paralelo a CD (ver figura acima).
Prolongando AB e CD obtemos um ponto de interseção E. As diagonais AC e BD se
intersectam num ponto F interior ao triângulo ADE, absurdo pelo caso anterior.
Considere agora o fecho convexo C de A0 . Se C é um segmento de reta, temos
An = A1 = A0 , ∀n ∈ `.
Se C é um triângulo de vértices A, B e C, estes pontos pertencem a A0 , que, como
vimos, não pode conter nenhum ponto no interior de C. Se A0 contém pontos no
interior de dois lados de C, digamos D em AB e E em AC, BE e CD se intersectam
em F ∈ A1 , que é interior a C, absurdo.
A

D E

B C

Assim, A0 só pode conter os vértices A, B e C e, eventualmente, alguns pontos em


um dos lados do triângulo, digamos AB. É fácil ver que nesse caso ainda temos
An = A1 = A0 , ∀n ∈ `.
Caso C seja um quadrilátero, deve ser um paralelogramo ABCD.
Se A0 contém algum ponto no interior de algum lado do paralelogramo, digamos E
em AB, conterá os vértices do quadrilátero convexo EBCD, que não é um
paralelogramo, absurdo. Se algum Aj contém algum ponto do interior de C, este não
pode ser interior aos triângulos ABC e ACD, logo deve pertencer à diagonal AC, e,
analogamente, deve pertencer à diagonal BD, e portanto deve ser o centro O de
ABCD. Temos então dois casos:
i) Se A0 = { A, B, C , D} temos A1 = { A, B, C , D, O} e An = A1 , ∀n ≥ 1.
ii) Se A0 = { A, B, C , D, O}, temos An = A1 = A0 , ∀n ∈ `.
Finalmente, se C tem pelo menos 5 vértices consecutivos A, B, C, D, E, devemos ter
CD e DE paralelos a AB, pois ABCD e ABDE são quadriláteros convexos com
vértices em A0 , mas isso é um absurdo, e o problema está resolvido.

EUREKA! N°23, 2006

9
Sociedade Brasileira de Matemática

PESSOAS CONFIÁVEIS E NÃO-CONFIÁVEIS


Svetoslav Savchev, Bulgária
Tradução: Cesar Ryudi Kawakami e Rafael Morioka Oda – São Paulo – SP

♦ Nível Avançado

Em um grupo de n ≥ 3 pessoas, algumas são confiáveis e as outras não.


Uma pessoa que é confiável sempre diz a verdade, enquanto uma não-confiável às
vezes diz a verdade e às vezes mente. Sabemos que o número de pessoas não
n
confiáveis é no máximo k, onde k é um inteiro que satisfaz 0 < k < . Um
2
observador quer identificar quem é confiável e quem não é. Ele pode fazer
perguntar a qualquer pessoa do grupo sobre a confiabilidade de alguma outra.
Qual o número mínimo de perguntas necessárias para descobrir o que deseja?

Essa pergunta originou-se de uma questão da olimpíada de Moscou de 1978, que


perguntava sobre o número máximo e proposto pelo proeminente Sergei Konyagin.
Depois do concurso, muitos matemáticos, como Schlosman, Ruzsa e Galvin,
acharam a idéia tão atraente que publicaram outras variantes do problema. Um
possível número suficiente foi encontrado e o necessário, que era mais difícil, foi
sendo gradualmente melhorado. Finalmente, o problema foi completamente
estabelecido por Pavel Blecher, que já havia sido premiado na própria olimpíada de
Moscou. Nós seguimos sua idéia, numa versão um pouco diferente do problema.

O menor número de questões é n + k − 1. Nós começamos provando que esse


número é suficiente, usando indução em n ≥ 3.
3
Para n = 3 nós temos 0 < k < então k = 1, ou seja, no máximo uma pessoa
2
não é confiável em um grupo de 3. Numere as pessoas e pergunte para a pessoa 2 se
a 1 é confiável. Se a resposta for sim, então 1 é de fato confiável (caso contrário 1 e
2 não seriam confiáveis). Sabendo que 1 é confiável, basta perguntar a ele sobre 2 e
3. Portanto temos que 3 = 3 + 1 − 1 = n + k − 1 questões são suficientes.
Se a pessoa 2 responder não, então temos que ou 1 ou 2 é não-confiável
(senão a resposta teria sido sim). Portanto temos que 3 é confiável, e basta perguntar
a ele sobre 1 e 2. Novamente, três perguntas bastam.
Assuma que m + k − 1 questões são suficientes para todo grupo de tamanho
m ∈ ], m ≥ 3 e menor que n, e considere um grupo de n pessoas que satisfaz as

EUREKA! N°23, 2006

10
Sociedade Brasileira de Matemática

condições dadas. Comece perguntando sucessivamente às pessoas 2,3...,n sobre a


pessoa 1. Pare de perguntar assim que:

i) O número de respostas sim se iguale a k.


ii) O número de respostas não exceda o número de respostas sim.

Note que (i) ou (ii) irá necessariamente ocorrer devido à condição k < n / 2.

No caso (i), a pessoa 1 é confiável (ou então ele mais as k pessoas que
respoderam sim seriam não-confiáveis, absurdo). Suponha que m pessoas tenham
respondido não até aquele momento; todas elas são não confiáveis. Agora basta
perguntar a 1 sobre todas as pessoas no grupo, exceto ele mesmo e as m pessoas
mencionadas. Isto leva n – m – 1 questões, então (k + m) + (n – m + 1) = n + k – 1
questões são suficientes.
No caso (ii), existe um número m tal que as pessoas 2, 3,..., 2m são
perguntadas e temos m respostas não e m – 1 respostas sim. Observe que não
importa se 1 é confiável ou não, pelo menos m pessoas do grupo G1 ={1,2,...,2m}
são não confiáveis. Se 1 é confiável, então temos que as m que responderam não são
não-confiáveis. Se 1 não é confiável, então ele e as m – 1 pessoas que responderam
sim são não-confiáveis. Concluímos portanto que m ≤ k, já que existem apenas k
pessoas não-confiáveis, e assim temos que o grupo G 2 ao qual pertencem as n – 2m
pessoas restantes não é vazio, uma vez que k < n / 2. Além do mais, existem no
máximo k – m pessoas que não são confiáveis em G 2 , o que é menor que
(n – 2m)/2. Se m < k então 1 ≤ k – m < (n – 2m)/2, n – 2m ≥ n – 2(k – 1) ≥ 3, e
logo a hipótese de indução se aplica ao grupo G 2 . Então (n – 2m) + (k – m) – 1
questões são suficientes para saber quem é o que no grupo. A mesma conclusão é
trivial para m = k, caso em que G 2 consiste de n – 2k pessoas, todas confiáveis, e
portanto nenhuma pergunta é necessária.
Agora escolha uma pessoa confiável A em G 2 (existe sempre pelo menos
uma) e pergunte-a sobre 1. Dependendo da resposta, pergunte a A sobre que
disseram a verdade sobre 1. Para ser mais preciso, se 1 é confiável (não-confiável),
pergunte a aqueles que responderam sim (não). No máximo 1 + m questões são
necessárias nessa última etapa. Não há mais questões necessárias pois já sabemos
que aqueles que mentiram sobre 1 não são confiáveis. Um total de (2m – 1) +
(n –2m) + (k – m) – 1 + (1 + m) = n + k – 1 questões são suficientes novamente.
A cota superior está provada. A cota inferior é consideravelmente mais
difícil: mostrar que n + k – 1 perguntas são também necessárias. Considere uma

EUREKA! N°23, 2006

11
Sociedade Brasileira de Matemática

seqüência arbitrária de n + k – 2 questões ( p1 , q1 ),( p 2 , q 2 ),...,( p n +k − 2 , q n +k − 2 );


(1) onde a questão i pergunta à pessoa pi sobre a pessoa qi . “Arbitrária” em
particular quer dizer que cada questão de (1) pode depender das respostas às
perguntas anteriores. Definimos também uma seqüência r1 , r2 ,…, rn +k − 2 de
respostas, tal que, para cada i , a resposta ri depende apenas das questões ( p1 , q1 ),
( p 2 , q 2 ),...,( pi , qi ). Agora, serão apresentadas duas configurações diferentes de
pessoas confiáveis e não-confiáveis num grupo de n , e ambas consistentes não só
com as respostas r1 , r2 ,…, rn +k − 2 mas também com o limite de k pessoas não-
confiáveis. Então a seqüência de questões (1) falhará em diferenciar essas duas
configurações, e portanto não há n + k – 2 questões que podem resolver o problema
em geral.
Nós definimos a resposta ri em dois estágios: o primeiro, quando as
primeiras k – 1 questões são feitas, e a segunda, envolvendo as n – 1 questões
restantes.
Todas as respostas na primeira parte serão não. Depois que as k – 1
perguntas acabam, represente como pontos todas as pessoas p1 ,…, p k −1 ,
q1 ,…, q k −1 envolvidas nelas e ligue pi a qi por uma aresta, i = 1,..., k – 1. O grafo
obtido será usado a seguir. Chame de G1 ,..., Gm suas componentes conexas, com
conjuntos de vértices V1 ,..., Vm respectivamente. Então cada questão ( pi , qi ) na
primeira parte (ou seja, i = 1,..., k – 1) envolve duas diferentes pessoas do mesmo
grupo V j (j = 1,..., m). Se e1 , e2 ,…, em são os números de arestas em G1 ,..., Gm ,
temos que e1 + e2 +…+ em = k – 1 (2)
Finalmente, denote por W o conjunto de pessoas não envolvidas no primeiro
conjunto de perguntas, i.e. o complemento de V1 ∪ V2 ∪ ... ∪ Vm em relação ao
conjunto de todas as n pessoas.
As questões ( p k , q k ),...,( p n +k − 2 , q n +k − 2 ) são feitas na segunda etapa. De
acordo com elas, nós escolhemos um representante de cada grupo V j (j = 1,..., m)
mas podem haver dois diferentes tipos de representantes dependendo do grupo.
Se forem feitas perguntas sobre cada pessoa em V j na segunda parte,
dizemos que V j é um grupo com um verdadeiro representante escolhido da seguinte

EUREKA! N°23, 2006

12
Sociedade Brasileira de Matemática

maneira: ache o menor i tal que V j está contido na seqüência q k ,..., qi ; então qi é
o (verdadeiro) representante de V j .
Se há pessoas em V j sobre as quais não foram feitas perguntas na segunda
etapa, escolha uma qualquer dentre elas e chame-a de falsa representante.
No momento em que a questão ( pi , qi ) é feita, está claro se qi é ou não um
representante verdadeiro de algum V j . Conseqüentemente as próximas questões na
segunda fase são tais que a resposta ri depende somente das questões ( p1 , q1 ),
( p 2 , q 2 ),...,( pi , qi ):
Para cada i = k,..., n + k – 2, definimos a resposta ri como sim se qi pertence a W
ou se qi é um verdadeiro representante de um grupo V j ; caso contrário defina ri
como não.
Agora considere uma configuração S de n pessoas onde a pessoa p é:
• confiável se p pertence a W ou se p é um representante de algum grupo V j
(verdadeiro ou falso);
• não-confiável em qualquer outra hipótese.
Vamos mostrar que S tem as seguintes propriedades:
i) S é consistente com todas as respostas r1 , r2 ,…, rn +k − 2 , i.e. todas as respostas
dadas por pessoas confiáveis em S são verdadeiras;
ii) Não há mais que k – 1 pessoas não-confiáveis em S.
Para provar (i), observe que todas as respostas da segunda etapa rk ,…, rn +k − 2 são
verdadeiras, pela sua definição e pela definição de pessoas confiáveis e não-
confiáveis (usando o fato de que não perguntamos sobre nenhum representante falso
na segunda parte). Então nós precisamos apenas garantir que todas as pessoas
confiáveis deram respostas verdadeiras na primeira etapa. Escolha uma pessoa
confiável p. Se p pertence a W então não houve perguntas a p nem sobre p no
primeiro estágio. Suponha que p é representante de algum grupo V j . Então as
possíveis perguntas que lhe podem ter sido feitas na primeira etapa são sobre as
pessoas de seu próprio grupo que não ele próprio. Todas essas pessoas são não-
confiáveis, e todas as respostas da primeira parte foram não, portanto p disse a
verdade.
Para verificar (ii), lembre-se que, por construção, as pessoas não-confiáveis estão
todas nos grupos V1 ,..., Vm . Exatamente uma pessoa em cada grupo é confiável –
seu representante. Uma vez que o número de pessoas não-confiáveis em V j é

EUREKA! N°23, 2006

13
Sociedade Brasileira de Matemática

V j – 1, j = 1,..., m, e o número total u de pessoas não-confiáveis é igual. Agora nós


usamos o fato de que V j é o conjunto de vértices de um grafo conexo: em tal grafo,
o número de arestas é pelo menos o número de vértices menos um. Deste modo
e j ≥ V j – 1 para j = 1,..., m.

∑( V )
m
De (2) temos que u = j
− 1 ≤ e1 + e2 + ... + em = k − 1 , e logo (ii) é verdade.
j =1

Ter no máximo k – 1 pessoas não-confiáveis em S será usado para obter uma outra
configuração S’ consistente com as respostas r1 , r2 ,…, rn +k − 2 e tendo no máximo k
pessoas não-confiáveis. Descreveremos agora esta construção.
Se algum grupo V j tem algum representante falso q, ele é confiável em S. Defina q
como um não-confiável em S’ e deixe todas as outras pessoas como eram em S.
Como q é não-confiável na nova configuração, suas respostas são irrelevantes para a
consistência com r1 , r2 ,…, rn +k − 2 . Portanto S’ é consistente com as respostas se, e
somente se, cada pessoa confiável de S disse a verdade quando perguntada sobre q.
Mas não há pessoa confiável que foi perguntada sobre q. De fato, não perguntaram
sobre sua confiabilidade na segunda etapa pela definição de falso representante. E se
alguma pessoa foi perguntada sobre q na primeira fase, ela pertence ao mesmo
grupo V j que q, e portanto não é representante do grupo, o que significa que não é
confiável em ambas as configurações. Assim, S’ satisfaz as condições.
Se não há representantes falsos em S então foram feitas perguntas sobre todas as
pessoas em V1 ,..., Vm na segunda etapa. Porém, n – 1 questões foram feitas no total
durante ela, logo não houve perguntas sobre pelo menos uma pessoa q dentre as n.
Claramente, q pertence a W. Defina q como não-confiável em S’ e deixe todos os
restantes da mesma maneira que em S. Ninguém perguntou sobre q (nem na
primeira nem na segunda etapa), e portanto a nova configuração S’ satisfaz
novamente todas as condições, e a prova está completa.

EUREKA! N°23, 2006

14
Sociedade Brasileira de Matemática

O TEOREMA DE STOLZ
Diêgo Veloso Uchôa & Renato Purita Paes Leme

♦ Nível Avançado

INTRODUÇÃO
Apresentaremos um pequeno resultado sobre limites com uma série de conseqüências
interessantes e, em seguida, alguns problemas que podem ser resolvidos usando-o.
Usando a demontração do teorema como pretexto, vamos mostrar como a intuição
geométrica pode nos dar boas indicações sobre o caminho a seguir. Isso pode ser
resumido na velha máxima: “Pense geometricamente, prove algebricamente”.

Teorema (Stolz)
Seja {xn} uma seqüência estritamente crescente com lim x n = +∞ , e {yn} uma
y n +1 − y n y
seqüência arbitrária. Se lim = a então: lim n = a .
x n +1 − x n xn

Prova
Podemos pensar em ( x n , y n ) como uma seqüência de pontos do plano pertencentes
y n +1 − y n
a uma curva da forma: y = f (x ) , já que xn é crescente. Assim,
x n +1 − x n
representa a razão incremental entre dois pontos consecutivos. À medida que n
aumenta, essa razão vai se tornando arbitrariamente próxima do valor constante a,
ou seja, para x grande (xn com n grande) a curva f é praticamente uma reta. Isto é, se
y = f (x ) tiver uma assíntota, ela deve ser uma reta da forma y = ax + b para
algum b. Como para n grande y n ≈ ax n + b , quando n → ∞ ,

y n ax n + b b
≈ =a+ →a
xn xn xn

Naturalmente, isso não constitui uma prova (ainda, veremos que nem sempre os
pontos ( x n , y n ) se aproximam de uma reta quando n → ∞ ) – mas isso serve como
uma boa orientação para uma prova formal. Vejamos:

EUREKA! N°23, 2006

15
Sociedade Brasileira de Matemática

(x7,y7)
(x6,y6)
(x5,y5)
(x4,y4)
(x3,y3)
(x2,y2)
(x1,y1)

y n +1 − y n
Pela definição de limite, como lim = a:
x n +1 − x n
y n +1 − y n
∀ε > 0, ∃n0 tal que n > n0 ⇒ −a <ε
x n +1 − x n
y n +1 − y n
ou seja, a razão incremental está entre a − ε e a + ε . Geometricamente:
x n +1 − x n

y y n0 (a )( x xn )
0

y y n0 (a )( x x n0 )
(xn0,yn0)

A geometria acima nos diz que para n > n0 , todo ( x n , y n ) está entre essas retas, ou
seja:
y n0 + k − y n0
− a < ε , ∀k > 0
x n0 + k − x n0

EUREKA! N°23, 2006

16
Sociedade Brasileira de Matemática

Precisamos provar algebricamente o que a geometria nos mostra. Mas isso é simples:

(a − ε ) ⋅ ( x n0 +1 − x n0 ) < y n0 +1 − y n0 < (a + ε ) ⋅ ( x n0 +1 − x n0 )
(a − ε ) ⋅ ( x n0 + 2 − x n0 +1 ) < y n0 + 2 − y n0 +1 < (a + ε ) ⋅ ( x n0 + 2 − x n0 +1 )
...
(a − ε ) ⋅ ( x n0 + k − x n0 + k −1 ) < y n0 + k − y n0 + k −1 < (a + ε ) ⋅ ( x n0 + k − x n0 + k −1 )

Somando as expressões acima, temos:

(a − ε ) ⋅ ( x n0 + k − x n0 ) < y n0 + k − y n0 < (a + ε ) ⋅ ( x n0 + k − x n0 )

y n0 + k − y n0
Ou seja, −a <ε .
x n0 + k − x n0

Dividindo tudo por x n0 + k e fazendo k tender ao infinito, temos:

y n0 + k y n0

x n0 + k x n0 + k y n0 + k y n0  xn  x
−a <ε ⇒ − − a1 − 0  < ε 1 − n0
x n0 x n0 + k x n0 + k  xn +k  x n0 + k
1−  0 
x n0 + k
y n0 + k x n0 y n0 x n0
− a < ε 1− + + a⋅ < 2ε
x n0 + k x n0 + k x n0 + k x n0 + k

para k suficientemente grande. Logo: lim


yn
=a \
xn

A prova desse teorema é composta de algumas poucas linhas. Demoramos mais, pois
perdemos tempo discutindo a sua essência geométrica.
Comentamos que nem sempre yn se aproxima de uma reta axn + b, ou seja, que nem
sempre a seqüência bn = yn – axn converge. Por exemplo:

xn = n e yn = 2n + log(n)

EUREKA! N°23, 2006

17
Sociedade Brasileira de Matemática

No entanto, nas condições do teorema, bn é uma seqüência cujas razões incrementais


bn +1 − bn
em relação a xn tendem a zero quando n tende ao infinito, isto é: lim = 0,
x n +1 − x n
como é o caso de bn = log(n).

Corolário
Dada uma seqüência {an} tal que lim a n = a , o limite da média artimética dos n
primeiros termos da seqüência também é a. Ou seja:

a1 + ... + a n
lim = a.
n
Prova
n
Basta fazer y n = ∑a
i =1
n e x n = n . Como {xn} é estritamente crescente e tende ao

a1 + ... + a n a
infinito, podemos aplicar o Teorema de Stolz : lim = lim n +1 = a [
n 1
Problema Resolvido I
1 n ak
Mostre que se lim a n = a então lim ∑ = a.
ln n k =1 k

n
ak
Solução : Fazendo y n = ∑
k =1 k
e x n = ln n e aplicando o Teorema de Stolz

an +1
n +1
n
1 ak an +1
lim ∑ = lim
ln( n 1) − ln n
+
= lim

=a.
1  
n +1
ln n k =1 k
ln  1 +
  n + 1  
 

Problema Resolvido II
1 p + 2 p + ... + n p
Dado um número real p > 0, calcule lim .
n →∞ n p +1

Solução: Façamos: y n = 1 p + 2 p + ... + n p e x n = n p +1 que é estritamente

EUREKA! N°23, 2006

18
Sociedade Brasileira de Matemática

crescente e tende ao infinito. Logo podemos aplicar o Teorema de Stolz:


p
1  1
⋅ 1 + 
y n +1 − y n (n + 1) p n  n
= =
x n +1 − x n (n + 1) p +1 − n p +1  1  p +1
1 +  − 1
 n
(1 + x )
p +1
−1
Tomando x = 1 / n e, usando que lim = p + 1 (pois é a derivada de
x →0
x
(1 + x ) p 1 em x = 0 ) para calcular o limite quando x → 0 , temos:
+

x (1 + x ) p 1 1
lim = =
(1 + x )
x →0 (1 + x) p +1 − 1 p +1
−1 p +1
lim
x →0 x

y n +1 − y n 1 y 1
Logo: lim = , assim: lim n = [
x n +1 − x n 1 + p xn 1 + p

Problema Resolvido III


(Schweitzer Competition - alterada) Dada a seqüência definida recursivamente por:
0 < a1 < 1 e a n +1 = a n (1 − a n ) , para n > 1 ,
prove que:
(a) lim n ⋅ a n = 1
n ⋅ (1 − na n )
(b) lim =1
ln n
n
Solução: a) Queremos usar o Teorema de Stolz em lim . Para isso, devemos
1 / an
mostrar que 1 / a n é estritamente crescente e tende ao infinito, o que equivale a
mostrar que a n é estritamente decrescente e tende a 0+. Por indução:
0 < a 2 = a1 (1 − a1 ) < a1 < 1 pois 0 < a1 < 1

Assumindo como hipótese de indução que: 0 < ai +1 < ai < 1 , temos que:
0 < ai + 2 = ai +1 (1 − ai +1 ) < ai +1 < 1 , pois 0 < ai +1 < 1

EUREKA! N°23, 2006

19
Sociedade Brasileira de Matemática

Como a n é estritamente decrescente e limitada, ela converge. Logo ∃a = lim a n e:


a = a (1 − a ) ⇒ a = 0 . Aplicando o Teorema de Stolz:
( n + 1) − n a ⋅a a ⋅ [a (1 − an ) ] a 2 ⋅ (1 − an )
lim = lim n n +1 = lim n n = lim n =
1

1 an
− an +1 an
− [ an (1 − an ) ] a n
2

an +1 an
= lim(1 − an ) = 1.

b) Aplicando um truque algébrico:


 1  1
n ⋅  − 1na n −n
n ⋅ (1 − na n )  na n  n ⋅ (1 − na n ) an
= ⇒ lim = lim ⋅ lim na n ,
ln n ln n ln n ln n
1
−n
an
se lim existir. Mostremos que ele de fato existe e vale 1, por Stolz:
ln n
 1  1  1 1  1 1 
 − n − 1 −  − n  − −1 n − − 1
 an +1   an  = an +1 an =  n +1
a an  =
ln(n + 1) − ln n  n +1  1  
n +1
ln   ln  1 +
    
 n +1 
n

 a − an +1  nan
n n − 1
⋅ 1 − an
=  n +1 n n +1 =
a a
→ 1 quando n → +∞ [
 1    1  
n +1

ln  1 + ln  1 +
  n + 1     n + 1  
   

A seguir apresentamos alguns problemas (em ordem crescente de dificuldade) que


são resolvidos com a aplicação do Teorema de Stolz.
yn
1 - Seja {xn} uma seqüência de termos positivos tais que ∑ x n = ∞ e lim =a
xn
y1 + ... + yn
então lim = a.
x1 + ... + xn
2 – Seja {xn} uma seqüência de termos positivos tais que lim x n = a . Mostre que
lim n x1 x2 ...xn = a. Sugestão: Use o corolário após o Teorema de Stolz.

EUREKA! N°23, 2006

20
Sociedade Brasileira de Matemática

a n +1
3 – Para uma seqüência de termos positivos {a n } mostre que se lim = a então
an
lim n a n = a .
 nk 
4 – Seja k um inteiro fixo arbitrário maior que 1. Determine lim n   .
n
n
 j
5 – Calcule lim n ∏ 1 + .
j =1  n

6 – Prove que se {an} é uma seqüência que converge para a então :


na1 + (n − 1)a 2 + ... + 1a n a
lim = .
n2 2
a − a n −1
7 – Prove que se lim( a n − a n − 2 ) = 0 então lim n = 0.
n
1 k n 
8 – Calcule lim  k (1 + 2 + ... + n ) −
k k
, k natural fixo.
n k + 1

9 – (OBM – adaptado) Dada a seqüência {a n } definida recursivamente por a1 = 3


ln(ln a n )
e a n +1 = a n − 2 . Prove que lim = ln 2 .
2

10 – Para uma seqüência {a n } , considere a seqüência { An } , de médias arirméticas,


a1 + a 2 + ... + a n
ou seja, An = . Mostre que se lim An = A , então:
n
1 n ak
lim ∑ = A.
ln n k =1 k

Sugestão: Escreva a k em função de alguns Ai .

11 – Mostre que, se para a seqüência de termos positivos {a n } , o limite

EUREKA! N°23, 2006

21
Sociedade Brasileira de Matemática

 a 
lim n1 − n +1 
 an 
existe então o limite
−1
ln a n
lim
ln n

também existe e os dois são iguais. Ainda, se o primeiro limite é infinito, o segundo
também o é.
1 an
12 – Seja a1 = 1 e a n +1 = a n + para n ≥ 1 , prove que: lim =1
2 ⋅ ln n
n

∑a
k =1
k

Sugestão: Estude a n2 .

REFERÊNCIAS
[ 1 ] KACZOR, W.J e NOWAK, M.T – Problems in Mathematical Analysis I – Real
Numbers, Sequences and Series – American Mathematical Society
[ 2 ] LIMA, E. L. – Curso de Análise, Volume I – Projeto Euclides – IMPA
[ 3 ] APOSTOL, T – Calculus, Volume I – Addison-Weasley

Diêgo Veloso Uchôa e Renato Purita Paes Leme são alunos de Engenharia que gostam de
matemática. Cursam o 4º ano do INSTITUTO MILITAR DE ENGENHARIA e estudam no INSTITUTO
DE MATEMÁTICA PURA E APLICADA. Os dois agradecem ao professor Carlos Gustavo T. A.
Moreira pela revisão desse artigo.

EUREKA! N°23, 2006

22
Sociedade Brasileira de Matemática

CONTAS COM DESIGUALDADES


Márcio Afonso Assad Cohen & Rodrigo Villard Milet

♦ Nível Avançado

Vamos discutir aqui uma notação prática para lidar com funções simétricas e
estudar a utilização de duas poderosas ferramentas, as desigualdades de Muirhead,
mais conhecida como bunching e a de Schur.
Por simplicidade de notação, os resultados serão mostrados para três variáveis. O
leitor não terá dificuldade em generalizar esses resultados para n variáveis (quando
necessário, serão fornecidas dicas para essa generalização).

1. EXPRESSÕES SIMÉTRICAS

Uma função f ( x, y , z ) é simétrica quando se tem, para todo x, y, z:


f ( x, y , z ) = f ( x, z , y ) = f ( y , x, z ) = f ( y , z , x ) = f ( z , x, y ) = f ( z , y , x)
(isto é, trocar uma variável com outra não altera a expressão).
Dada uma função qualquer P ( x, y, z ) , definimos:
∑ P( x, y, z) = P( x, y, z) + P( x, z, y) + P( y, x, z) + P( y, z, x) + P(z, x, y) + P(z, y, x) .
sym

Exemplo 1. De acordo com a definição, verifica-se que:


∑x
sym
2
y = x2 y + x2 z + y2 x + y2 z + z 2 x + z 2 y ;

∑x
sym
3
= 2 ⋅ (x3 + y3 + z 3 ) ; ∑ xyz = 6 xyz
sym
(reflita um pouco para entender os coeficientes 2 e 6 dos exemplos anteriores!).

1.1. Propriedades
1. ∑ f é uma função simétrica, i.e, ∑ f ( x, y, z ) = ∑ f ( x, z, y) = ...
sym sym sym

2. Se σ( x, y , z ) é uma função simétrica, então ∑σ(x, y, z) ⋅ f (x, y, z) =σ(x, y, z) ⋅ ∑ f (x, y, z)


sym sym

(porque σ( x, y , z ) é uma constante para esse somatório).

Exemplo 2. (Quadrado e Cubo da soma) Sendo s = x + y + z , expresse 2s e 2s


2 3

como somas simétricas:

EUREKA! N°23, 2006

23
Sociedade Brasileira de Matemática

Solução:
2s = ∑ x ⇒ 2 s 2 = ∑ x ⋅ ( x + y + z ) = ∑ ( x 2 + xy + xz ) = ∑ ( x 2 + 2 xy )
sym sym sym sym

2s = 2s ⋅ s = ∑(x + 2xy) ⋅ (x + y + z) = ∑(x + x y + x z + 2x 2 y + 2xy2 + 2xyz) ∴


3 2 2 3 2 2

sym sym

2s = ∑(x + 6x y + 2xyz)
3 3 2

sym

Exemplo 3. Sendo x, y, z > 0, mostre que:


x y z 9
+ + ≤
( x + y )( x + z ) ( y + x)( y + z ) ( z + x)( z + y ) 4( x + y + z )
Solução: Usando a notação de simetria, podemos reescrever o problema como:
1 x 9
⋅∑ ≤ (Se você não entendeu o fator ½,
2 sym ( x + y )( x + z ) 4( x + y + z )
volte ao exemplo 1).
Multiplicando tudo pelo mmc ( x + y )( x + z )( y + z )( x + y + z ) :
2 ⋅ ∑ x( y + z )( x + y + z ) ≤ 9( x + y )( x + z )( y + z )
sym

Agora observe que; chamando o lado esquerdo de LE e o direito de LD:


 1 
(i) LD = 9 ⋅ (x + y)(x + z)(y + z) = 9 ⋅ (x2 y + x 2 z + ...+ xyz + xyz) = 9 ⋅ 
∑ x 2 y + ∑xyz
 sym 3 sym 
(para evitar erros, confira se a expressão está verdadeira para x = y = z = 1,
lembrando que ∑
1 = 6 nesse caso).
sym

(ii) LE= 2⋅ ∑(xy+ xz)(x + y + z) = 2⋅ ∑(x y + xy + xyz+ x z + xyz+ xz ) = ∑(8x y + 4xyz)


sym sym
2 2 2 2

sym
2

Portanto, LE ≤ LD ⇔ ∑ (8x
sym
2
y + 4 xyz) ≤ ∑ (9 x y + 3xyz) ⇔ ∑ x 2 y ≥ ∑ xyz .
sym
2

sym sym

A última inequação é consequência direta da desigualdade das médias:


x 2 y + x 2 z + y 2 x + y 2 z + z 2 x + z 2 y ≥ 6 xyz .

2. MUIRHEAD (vulgo “bunching”)


Dadas duas seqüências não-crescentes (a1 , a2 , a3 ) e (b1 , b2 , b3 ) , diz-se que a majora b
quando: a1 ≥ b1 ; a1 + a2 ≥ b1 + b2 ; e a1 + a2 + a3 = b1 + b2 + b3

EUREKA! N°23, 2006

24
Sociedade Brasileira de Matemática

Nesse caso, escreve-se [ a1 , a2 , a3 ] ≥ [b1 , b2 , b3 ] e se x, y, z são reais positivos temos:


∑x
sym
a1
y a2 z a3 ≥ ∑ xb1 y b2 z b3 .
sym

Demonstração:
Lema (smoothing): Se em ∑x
sym
u1
y u2 z u 3 substituirmos dois expoentes u i , u j (i < j)

por vi , v j com u i + u j = vi + v j e ui ≤ vi ≤ v j ≤ u j , o valor da expressão diminui


(ou não muda).
(isto é, manter a soma constante e diminuir a distância entre dois expoentes não
aumenta a expressão)

Demonstração do Lema:
S.p.g, suponha u1 = m + R , u 2 = m − R, v1 = m + r , v 2 = m − r :
∑ (x
sym
u1
y 2z 3 − x1y 2z 3)=
u u v v u

1
= ⋅ ∑ ( x m + R y m − R z u3 − x m + r y m − r z u 3 + x m − R y m + R z u 3 − x m − r y m + r z u 3 ) =
2 sym
1
= ⋅ ∑ z u3 x m y m ⋅ ( x R y − R + x − R y R − x r y − r − x − r y r )
2 sym
A última expressão é não-negativa, pois para a positivo fixo (no caso a = x/y), a
função f (t ) = a t + a − t é crescente:
1 1 1
f (R) − f (r) = a R − ar + R
− r = (aR − ar ) − R r (a R − ar ) = a−R−r (a R − ar )(aR ar −1)
a a a a

Se a > 1 e R > r > 0, essa expressão é obviamente positiva. Se 0 < a < 1, também
(troque a por 1/a).

Demonstração do teorema:
Como [a1 , a 2 , a 3 ] ≥ [b1 , b2 , b3 ] , se as seqüências não forem iguais, adotamos o
seguinte procedimento:
Seja i o maior índice tal que ai > bi , e j o menor índice maior do que i tal que
a j < b j . (o qual existe, pois ∑ a = ∑ b e ∑ a ≥ ∑ b , donde ∑ a > ∑ b
k k
k <i
k
k <i
k
k ≤i
k
k ≤i
k ).

Se i < k < j , ak = bk . Seja r = min{ai − bi , b j − a j } > 0. Podemos fazer o troca de ai


por ai − r e de a j por a j + r, de modo que o número de índices s tais que

EUREKA! N°23, 2006

25
Sociedade Brasileira de Matemática

as = bs aumenta de pelo menos uma unidade. A ordem dos ai é preservada, pois


ai − r ≥ bi ≥ b j ≥ a j + r e, se i < k < j, ai − r ≥ bi ≥ bk = ak . Como a soma simétrica
∑x
sym
a1
y a2 z a3 nunca aumenta (pelo lema), repetimos o procedimento até que

as = bs para todo s, o que mostra que, no início, tínhamos


∑x
sym
a1
y a2 z a3 ≥ ∑ xb1 y b2 z b3 .
sym

Obs. A demonstração do caso com n variáveis é análoga.


(A demonstração fornece uma idéia intuitiva para o conceito de majorar. Uma
seqüência na qual os expoentes estão próximos uns dos outros é sempre majorada
por uma com os termos mais espalhados!)

Exemplo 4. (Desigualdade das médias) Dados a1 , a 2 ,..., a n reais positivos, mostre que
a1 + a 2 + ... + a n n
≥ a1 a 2 ...a n
n
Solução: Vamos usar a versão de n variáveis de bunching:
1 1 1 1 
[1,0,0,...,0] >  , , ,...  ⇒
n n n n 
∑ a1 ≥ ∑ a11 n a21 n ... an1 n ⇒
sym sym

(n − 1)!⋅ (a1 + a2 + ... + an ) ≥ n ! ⋅ n a1a2 ...an ⇒ MA ≥ MG.

Exemplo 5. (Vingança olímpica) Dados a, b, c, x reais positivos, prove que

a x+2 + 1 b x+2 + 1 c x+2 + 1


+ + ≥3
a x bc + 1 b x ac + 1 c x ab + 1

Solução: Na notação simétrica, o problema pode ser reescrito como:


1 a x+ 2 + 1
⋅∑ x ≥3
2 sym a bc + 1
∑ (a
sym
x+2
+ 1)(b x ac + 1)(c x ab + 1) ≥ 6(a x bc + 1)(b x ac + 1)(c x ab + 1)

Desenvolvendo cada lado:

EUREKA! N°23, 2006

26
Sociedade Brasileira de Matemática

LE = ∑ (a x + 4 b x +1 c x +1 + 2a x + 3 b x c + a x + 2 + b x +1 c x +1 a 2 + 2a x bc + 1)
sym

LD = ∑ (a x + 2 b x + 2 c x + 2 + 3a x +1b x +1 c 2 + 3a x bc + 1)
sym

Temos LE ≥ LD já que, usando bunching três vezes, podemos escrever:


LE− LD= ∑(ax+4bx+1cx+1 − ax+2bx+2cx+2 ) + 2⋅ ∑(ax+3bxc − ax+1bx+1c2 ) + ∑(ax+2 − axbc) ≥ 0.
sym sym sym
Exemplo 6. (Torneio das cidades e Japão) Mostre que sendo a, b, c positivos de
produto 1 tem-se
1 1 1
+ + ≤1
a + b +1 b + c +1 a + c +1

Solução: Inicialmente, vamos nos livrar da restrição, homogeneizando a


inequação:
1 1 1 1
+ + ≤3
a + b + abc b + c + abc a + c + abc
3 3 3
abc
Para evitar as raízes, faça x = 3 a , y = 3 b , z = 3 c .
xyz

sym x + y + xyz
3 3
≤2⇔

∑ xyz( y
sym
3
+ z 3 + xyz)(x3 + z 3 + xyz) ≤ 2(x 3 + y 3 + xyz)( y 3 + z 3 + xyz)(x 3 + z 3 + xyz)

Desenvolvendo,
(x3 + z 3 + xyz)(y3 + z 3 + xyz) = z 6 + x3 z 3 + y 3 z 3 + x3 y3 + x 4 yz + 2z 4 yx + y 4 xz + x2 y 2 z 2
(*)
Logo, LE = ∑ (x
sym
7
yz + 4 x 5 y 2 z 2 + 3x 4 y 4 z + x 3 y 3 z 3 )

Multiplicando (*) por x 3 + y 3 + xyz e escrevendo na notação simétrica:


LD = ∑ ( x 7 yz + 2 x 6 y 3 + 2 x 5 y 2 z 2 + 3x 4 y 4 z + x 3 y 3 z 3 )
sym
(No início, é difícil olhar direto para os coeficientes. Nesse caso, não perca tempo.
Faça a conta com os 27 termos no LD e só depois escreva na notação de simetria!)

Cancelando os termos comuns, a desigualdade é portanto equivalente a



xym

x6 y3 ≥ x5 y 2 z 2 ,
sym

EUREKA! N°23, 2006

27
Sociedade Brasileira de Matemática

que segue por bunching já que [6,3,0] ≥ [5,2,2] .

3. SCHUR (para três variáveis)


Se x, y, z, r são positivos, tem-se:
∑ x ( x − y )( x − z ) ≥ 0 .
r

sym

Demonstração: Como a expressão é simétrica, podemos supor, sem perda de


generalidade, x ≥ y ≥ z , de forma que:

x r ( x − y )( x − z ) + y r ( y − x)( y − z ) = ( x − y )( x r ( x − z ) − y r ( y − z )) ≥ 0 ,
pois x − z ≥ y − z e x r ≥ y r .
Para completar a demonstração, basta somar essa desigualdade com
z ( z − x)( z − y ) ≥ 0 .
r

Quando r = 1, obtemos a principal variação da desigualdade de Schur:


∑ (x 3
+ xyz − 2 x 2 y ) ≥ 0.
sym

(Schur é muito útil quando o termo mais distribuído (ex: xyz) deve ser maior que
algum outro termo).

Exemplo 7. (Irã) Dados x, y, z reais positivos, mostre que

 1 1 1  9
( xy + yz + xz ) ⋅  + +  ≥ .
 ( x + y) ( y + z) ( x + z) 2
2 2
 4
Solução:
Tirando mmc queremos provar que:
2 ⋅ ∑ ( xy + yz + xz )( y + z ) 2 ( x + z ) 2 ≥ 9 ⋅ [( x + y )( x + z )( y + z )]
2

sym

Fazendo as contas,
( x + y )( x + z )( y + z ) = ( x + y )( z 2 + xy + xz + yz ) =
x 2 y + x 2 z + y 2 x + y 2 z + z 2 x + z 2 y + 2 xyz = s
  1 
LD = 9 ⋅ [( x + y )( x + z )( y + z )] = 9 s 2 = 9s ⋅  ∑ x 2 y  + 9 s ⋅  ⋅ ∑ xyz 
2

 sym   3 sym 
EUREKA! N°23, 2006

28
Sociedade Brasileira de Matemática

LD = 9 ⋅ ∑ ( x 4 y 2 + x 3 y 3 + x 4 yz + x 2 y 3 z + x 3 yz 2 + x 2 y 2 z 2 + 2 x 3 y 2 z ) +
sym

+3 ⋅ ∑ (6 x 3 y 2 z + 2 x 2 y 2 z 2 )
sym

LD = ∑ (9 x 4 y 2 + 9 x 4 yz + 9 x 3 y 3 + 54 x 3 y 2 z + 15 x 2 y 2 z 2 )
sym

No lado esquerdo, pondo σ = xy + xz + yz :


LE = 2 ⋅ σ ⋅ ∑[( y + z)(x + z)] = 2 ⋅σ ⋅ ∑(z 2 + σ )2 = 2 ⋅ (6σ 3 + 2σ 2 ∑x 2 + σ ∑ x 4 )
2

sym sym sym sym

Agora, usando o exemplo 2:


6σ 3 = 3 ⋅ ∑ ( x 3 y 3 + 6 x 3 y 2 z + 2 x 2 y 2 z 2 )
sym

2σ = ∑ ( x 2 y 2 + 2 x 2 yz );
2

sym

2σ 2
∑x = ∑(x y
sym
2

sym
2 2
+ 2x2 yz) ⋅ 2 ⋅ (x2 + y2 + z 2 ) = 2⋅ ∑(2x4 y2 + x2 y2 z 2 + 2x4 yz + 4x2 y3z)
sym

σ ∑ x 4 = ∑ x 4 ⋅ ( xy + xz + yz ) = ∑ (2 x 5 y + x 4 yz )
sym sym sym

Juntando tudo,
LE = 2 ⋅ ∑ (2 x 5 y + 4 x 4 y 2 +5 x 4 yz + 3 x 3 y 3 + 26 x 3 y 2 z + 8 x 2 y 2 z 2 )
sym

Usando duas vezes bunching e em seguida Schur temos:


LE − LD = ∑ (4 x 5 y − x 4 y 2 + x 4 yz − 3 x 3 y 3 − 2 x 3 y 2 z + x 2 y 2 z 2 ) =
sym

= ∑ ( x 5 y − x 4 y 2 ) + 3 ⋅ ∑ ( x 5 y − x 3 y 3 ) + xyz ∑ ( x 3 + xyz − 2 x 2 y ) ≥ 0
sym sym sym

Exemplo 8. (IMO) Sejam x, y, z reais positivos com xyz = 1. Mostre que:


x5 − x2 y5 − y 2 z5 − z 2
+ + ≥0.
x5 + y 2 + z 2 x2 + y5 + z 2 x2 + y 2 + z5
Solução:
Escrevendo na notação de simetria, o problema é equivalente a:
∑ (x
sym
5
− x 2 ) ⋅ (x 2 + y 5 + z 2 ) ⋅ (x 2 + y 2 + z 5 ) ≥ 0

EUREKA! N°23, 2006

29
Sociedade Brasileira de Matemática

Como (x2 + y 5 + z 2 ) ⋅ (x2 + y 2 + z 5 ) = x 4 + ( y 5 + z 2 + y 2 + z 5 )x2 + ( y5 + z 2 )(y 2 + z 5 ) ,


basta que:
∑ x5 ⋅ ( x 4 + 2 y 5 x 2 + 2 y 2 x2 + 2 y 7 + y 5 z 5 + z 2 y 2 ) ≥
sym

≥ ∑ x2 ⋅ (x4 + 2 y5 x2 + 2 y 2 x2 + 2 y7 + y5 z5 + z2 y 2 )
sym

∑ (x
sym
9
+ 2x 7 y 5 + 2x 7 y 2 + 2x 5 y 7 + x 5 y 5 z 5 + x 5 y 2 z 2 ) ≥

∑ (x
sym
6
+ 2 x 4 y 5 + 2x 4 y 2 + 2 x 2 y 7 + x 5 y 5 z 2 + x 2 y 2 z 2 )

Só bunching não vai resolver isso, pois o termo x5y5z5 fica sobrando do lado
esquerdo. Usando respectivamente a desigualdade das médias, bunching e xyz • 
porém temos:
∑ (x
sym
7
y 5 + x 7 y 5 + x 5 y 5 z 5 + x 5 y 2 z 2 ) ≥ ∑ 4 x 6 y 4, 25 z 1, 75 ≥
sym

∑ (2 x
sym
6
y 4 z 2 + x5 y5 z 2 + x4 y 4 z 4 ) ≥

∑ (2 x
sym
4
y 2 + x5 y 5 z 2 + x 2 y 2 z 2 )

O problema está resolvido somando-se essa desigualdade com:


∑ (x
sym
9
+ 2 x 7 y 5 ) ≥ ∑ ( x 7 yz + 2 x 6 y 5 z ) ≥ ∑ ( x 6 + 2 x 5 y 4 ) ,
sym sym
que vale por bunching e xyz •  UHVSHFWLYDPHQWH

4. EXERCÍCIOS PROPOSTOS:
(Assuma as variáveis reais positivas nos exercícios abaixo)
a b c 3
1. + + ≥ .
b+c a+c a+b 2
2. ( x + y − z ) ⋅ ( x − y) + ( y + z − x )( y − z ) + ( z + x − y )(z − x) ≥ 0 .
2 2 2 2 2 2 2 2 2 2 2 2

7
3. (IMO) Se x + y + z = 1, mostre que 0 ≤ xy + yz + zx − 2 xyz ≤ .
27
4. (Banco IMO) Se xyz = 1, mostre que
x3 y3 z3 3
+ + ≥ .
(1 + y )(1 + z ) (1 + z )(1 + x) (1 + x )(1 + y ) 4

EUREKA! N°23, 2006

30
Sociedade Brasileira de Matemática

1 1 1 3
5. (IMO) Sendo abc = 1, mostre que + 3 + 3 ≥ .
a (b + c ) b (a + c) c (a + b) 2
3

 1  1  1
6. (IMO) Se abc = 1, mostre que  a − 1 +  ⋅  b − 1 +  ⋅  c − 1 +  ≤ 1 .
 b  c  a
7. (Banco IMO)
(a + b − c) 2 ⋅ (a − b + c)2 ⋅ (−a + b + c)2 ≥ (a2 + b2 − c 2 ) ⋅ (a2 − b2 + c 2 ) ⋅ (−a2 + b2 + c 2 ) .
(b + c − a ) 2 (c + a − b ) 2 ( a + b − c) 2 3
8. (Japão) + + ≥ .
(b + c) + a2 2
(c + a ) + b
2 2
( a + b) + c
2 2
5
(2 x + y + z) 2 (2 y + x + z ) 2 ( 2 z + y + x) 2
9. (USA) + + ≤8.
2 x 2 + ( y + z ) 2 2 y 2 + ( x + z ) 2 2 z 2 + ( y + x) 2
10. (Bulgária) Se abc = 1, mostre que
1 1 1 1 1 1
+ + ≤ + + .
1+ a + b 1+ b + c 1+ a + c 2 + a 2 + b 2 + c

EUREKA! N°23, 2006

31
Sociedade Brasileira de Matemática

O PROBLEMA IMPOSSÍVEL
Cássio Neri
Instituto de Matemática, Universidade Federal do Rio de Janeiro

♦ Nível Avançado

INTRODUÇÃO:

O Problema Impossível é um lindo problema sobre números inteiros. Sua forma


original foi dada por Freudenthal [1] antes de ser popularizada por Martin Gardner1
[2]. As duas versões não são exatamente iguais. A versão de Gardner é a seguinte:

Dois números inteiros (não necessariamente diferentes) entre 2 e 20 são escolhidos.


Apenas a soma dos dois números é dada ao matemático Sérgio. Apenas o produto dos dois
números é dado ao matemático Paulo.
Por telefone, Sérgio diz a Paulo: "Não existem meios para que você determine
minha soma". Uma hora depois, Paulo telefona de volta para dizer: "Eu sei a sua soma".
Mais tarde, Sérgio telefona novamente para Paulo para anunciar: "Agora eu sei o seu
produto".
Quais são os números?

No problema original, Freudenthal fixou uma cota superior de 100 (não para os
números, mas para a soma; não obstante, vamos considerar apenas variações do
problema com cotas para os números, embora a versão de Freudenthal possa ser
analisada de modo análogo). Para simplificar o problema, Gardner preferiu usar a
cota superior igual a 20. Fazendo isto, "o Problema Impossível se tornou
literalmente impossível" como disse o próprio Gardner [3]. Neste texto vamos
explicar este ponto.
É surpreendente que o problema original seja bem posto já que, como pode-se
pensar, a pequena conversa telefônica não acrescenta nenhuma informação relevante
sobre os números. Mas, como veremos, esta conversa é rica de informações
matemáticas.

O Problema Impossível é um problema 3 em 1. Os problemas propostos a Paulo e a


Sérgio são diferentes do nosso e entre si. Cada um dos personagens tem uma
informação adicional (o produto para Paulo e a soma para Sérgio) que nós não
temos. Isto faz uma grande diferença.

1
O autor, além de traduzir o enunciado, tomou a liberdade de chamar os personagens de
Paulo e Sérgio em vez de P e S como na versão em inglês.

EUREKA! N°23, 2006

32
Sociedade Brasileira de Matemática

Nas duas primeiras seções consideraremos o problema como nos foi proposto.
Na última seção consideramos os problemas propostos a Paulo e a Sérgio.

SOLUÇÃO DO PROBLEMA

Para resolver o problema vamos usar um programa de computador já que existem


muitos casos a considerar. Um programa escrito em linguagem C está disponível em
[6]. Posteriormente veremos uma solução sem a necessidade de ajuda
computacional.

Vejamos a estratégia da solução. Aproximadamente falando, começaremos com dois


conjuntos grandes: o dos produtos admissíveis e o das somas admissíveis. De cada
frase do diálogo, extrairemos informação que nos permitirá reduzir o tamanho destes
conjuntos. Ao final, teremos apenas um produto e uma soma admissível.
Resolvendo uma equação de segundo grau, determinaremos os dois números.

Faremos a seguinte hipótese fundamental, caso contrário, o problema não tem


sentido matemático.

Hipótese fundamental: Paulo e Sérgio dizem a verdade.


Esta hipótese merece um comentário filosófico. Algum leitor pode julgar que tal
hipótese seja falsa já que Sérgio afirma que Paulo não pode determinar a soma e,
uma hora depois, Paulo anuncia que o fez. Um deles está mentindo? Não
necessariamente. A hipótese continua sendo plausível ao considerarmos o tempo.
Inicialmente Paulo não podia resolver o problema. Por alguma razão, que veremos a
seguir, Sérgio sabia disto e anunciou o fato. Paulo pensou e descobriu qual era esta
razão. Só depois de descobri-la (portanto, não antes de Sérgio se pronunciar) é que
Paulo foi capaz de determinar a soma.

Chamaremos de p ao produto e s à soma dos dois números. Seja N a cota superior


dada para os números. Nesta seção consideraremos N = 100. Sabemos, a priori, que
p não é primo já que ele é um produto de dois inteiros maiores que 1. Além disto, p
não pode ter nenhum divisor primo maior que N (por exemplo, p não pode ser 2 ·
101). Há outras restrições, por exemplo, p ≠ 11 ⋅11 ⋅11 e p ≠ 11 ⋅13 ⋅ 17. Veja que
sabemos muita coisa sobre p. Por outro lado, não temos nenhuma restrição sobre s,
exceto que 4 ≤ s ≤ 200.

Para simplificar a apresentação, faremos as seguintes definições.

EUREKA! N°23, 2006

33
Sociedade Brasileira de Matemática

Definição: Dado um inteiro m ≥ 4, dizemos que o par (i, j) é uma fatoração de m


se m = ij com i e j inteiros entre 2 e N.

Definição: Dado um inteiro m ≥ 4, dizemos que o par (i, j) é uma decomposição de


m se m = i + j com i e j inteiros entre 2 e N.
Repare que estas definições dependem de N. Assim, (2, 8) é uma fatoração de 16 se,
e somente se, N ≥ 8. Em se tratando de fatorações e decomposições, consideramos
(i, j) = (j, i).
As restrições sobre p, citadas anteriormente, são conseqüências do seguinte fato:
existe pelo menos uma fatoração para p. Os conjuntos dos produtos e das somas
admissíveis são dados, respectivamente, por
P0 = {m ∈ `; 4 ≤ m ≤ N 2 e m tem pelo menos uma fatoração},
S0 = {m ∈ `; 4 ≤ m ≤ 2 N }.
O programa informa que P0 tem 2880 elementos. Informa também que S0 tem 197,
entretanto isto é trivialmente constatado sem necessidade de computador.
Passemos à análise do diálogo. Inicialmente Sérgio diz que Paulo não pode
determinar a soma. Isto significa que p tem pelo menos duas fatorações distintas. De
fato, já sabemos que p tem fatoração. Se (i, j) fosse a única fatoração de p, então
Paulo saberia que s = i + j. Desta forma, o conjunto dos produtos admissíveis se
reduz a
P1 = {m ∈ P0 ; m tem pelo menos duas fatorações}.

Temos que p ∈ P1 e, segundo o programa, P1 tem 1087 elementos. Percebe-se


facilmente que se i e j são primos, com i, j ≤ N , então ij ∈ P0 \ P1. Porém, números
desta forma constituem apenas uma pequena parte de P0 \ P1 . Outros exemplos de
elementos de P0 \ P1 são N 2 e N ( N − 1).
Existe outra conseqüência da primeira frase de Sérgio: qualquer que seja a
decomposição (i, j) de s temos que ij ∈ P1 . com efeito, suponhamos que s tenha
decomposição (i, j) tal que ij ∉ P1 . Vamos mostrar que, neste caso, existe uma
circunstância na qual Paulo poderia determinar a soma, de modo que Sérgio não
poderia fazer sua primeira declaração. Isto ocorre quando p = ij pois teríamos p ∉ P1
e, portanto, (i, j) seria a única fatoração de p. Isto permitiria a Paulo concluir que
s = i + j. Em vista disso, o conjunto de somas admissíveis se reduz a

S1 = {m ∈ S 0 ; para toda decomposição (i, j) de m temos que ij ∈ P1}.

EUREKA! N°23, 2006

34
Sociedade Brasileira de Matemática

Temos que s ∈ S1 e, de acordo com o programa,

S1 = {11,17,23,27, 29,35,37,41,47,53}

Em seguida, Paulo anuncia saber a soma. Segue daí que existe uma única fatoração
(i, j) de p tal que i + j ∈ S1. De fato, s ∈ S1 , logo, p tem fatoração cuja soma está em
S1 . Se existisse mais de uma fatoração nessa condição, então Paulo não poderia
saber qual delas seria correta. Assim, o conjunto dos produtos admissíveis é
reduzido a

P2 = {m ∈ P1 ; existe uma única fatoração (i, j) de m tal que i + j ∈ S1}.

Temos que p ∈ P2 . O programa diz que P2 tem 86 elementos.

Finalmente, Sérgio diz que também sabe o produto. Graças a um argumento análogo
ao do parágrafo anterior, concluímos que existe uma única decomposição (i, j) de s
tal que ij ∈ P2 . Portanto, o conjunto das somas admissíveis se reduz a
S 2 = {m ∈ S1 ; existe uma única decomposição (i, j) de m tal que ij ∈ P2 }.
Temos que s ∈ S2 . O programa informa que S 2 = {17}. Logo, s = 17.
Neste momento, Sérgio já anunciou ter encontrado o produto. Como está dizendo a
verdade, então nós também podemos encontrá-lo. Com efeito, sabemos que a soma
vale 17 e, portanto, podemos nos colocar no lugar de Sérgio.
Outra maneira de proceder é reiterar o argumento que define P2 a partir de S1 e
aquele que define S 2 a partir de P2 para, assim, construir Pn a partir de S n −1 e S n a
partir de Pn . Repetimos isto até encontrar n ∈ ` tal que Pn e S n sejam unitários.
Mais precisamente, para n = 3,…, definimos
Pn = {m ∈ Pn−1 ; existe uma única fatoração (i, j) de m tal que i + j ∈ S n −1},
S n = {m ∈ S n −1 ; existe uma única decomposição (i, j) de m tal que ij ∈ Pn }.
Obtemos assim, P3 = {52} e S3 = {17}, logo p = 52 e s = 17, de onde segue que os
números são 4 e 13.

O ERRO DO GARDNER

Conhecendo a resposta, 4 e 13, a intuição diria que a mesma solução valeria se


trocássemos, como fez Gardner, a cota superior N de 100 para 20.

EUREKA! N°23, 2006

35
Sociedade Brasileira de Matemática

Surpreendentemente, isto está errado! Veremos que para N ≤ 61 os números 4 e 13


não resolvem o problema!
O que acontece se N ≤ 61 ? Neste caso 17 ∉ S2 . De fato, o programa mostra que
S 2 = ∅, o que significa que o problema não tem solução. Pode-se pensar que esta é
uma falha do método e que talvez, por outro argumento encontrássemos a solução,
mas concluímos o seguinte. Se s e p são os números fornecidos a Sérgio e Paulo,
respectivamente, e se ambos dizem a verdade, então obtemos, sucessivamente, os
seguintes fatos:

1. antes do diálogo: p ∈ P0 e s ∈ S0 ;
2. após a primeira frase de Sérgio: p ∈ P1 e s ∈ S1 ;
3. após a primeira frase de Paulo: p ∈ P2 ;
4. após a primeira frase de Sérgio: s ∈ S2 .

Logo, se os números são 4 e 13, então p = 52 ∈ P2 e s = 17 ∈ S2 . Suponhamos agora


que N ≤ 61. Vamos mostrar que 52 ∉ P2 ou 17 ∉ S2 , de onde seguirá que os
números não podem ser 4 e 13. Procedemos por absurdo supondo que
52 ∈ P2 e 17 ∈ S 2 .
Afirmaremos que 19,37 ∉ S1 . De fato, como 34 tem apenas a fatoração (2, 17), temos
que 34 ∉ P1 e, portanto, 2 + 17 = 19 ∉ S1 . Observamos que 186 pode ser escrito
como produto de dois números naturais apenas das seguintes formas:
1 ⋅ 186, 2 ⋅ 93, 3 ⋅ 62 e 6 ⋅ 31. Como N ≤ 61, a única fatoração de 186 é (6, 31), logo,
186 ∉ P1 . Segue daí que 6 + 31 = 37 ∉ S1 .
Agora, mostraremos que 70 ∈ P2 . Podemos escrever 70 como 1 ⋅ 70,2 ⋅ 35,5 ⋅ 14 e
7 ⋅ 10. Como já vimos, 37 = 2 + 35 e 19 = 5 + 14 não estão em S1 , porém,
17 ∈ S2 ⊂ S1 . Logo, 70 ∈ P2 .
Finalmente, entre todas as decomposições de 17 temos (4, 13) e (7, 10). Como
52,70 ∈ P2 temos que 17 ∉ S2 o que é uma contradição.
Salientamos os seguintes fatos:

Mostramos apenas que (4, 13) não é solução para N ≤ 61. Talvez, outra solução
apareça quando N ≤ 61. Entretanto, o programa em [6] mostra que S 2 = ∅
e, portanto, o problema não tem solução.

EUREKA! N°23, 2006

36
Sociedade Brasileira de Matemática

2. Mesmo os puristas, que não aceitam demonstrações assistidas por computador,


deverão aceitar que (4, 13) não é solução do problema quando N ≤ 61.

3. Quando dizemos que o problema não tem solução, significa que o nosso problema
não tem solução. Por outro lado, o problema proposto a Paulo tem solução desde
que a ele seja fornecido um número adequado. A situação de Sérgio não é muito
diferente da nossa. Abordaremos estas questões na próxima seção.

O PROBLEMA DE PAULO E O DE SÉRGIO

Nesta seção, consideraremos os outros problemas, especialmente, o dado a Paulo.


Em [6] está disponível um programa em linguagem C que poderia ser usado por
Paulo (ou por nós em seu lugar) e outro para Sérgio. Por hora, voltemos a considerar
N = 100.
Paulo sabe que p = 52 e que suas fatorações são (2, 26) e (4, 13). Assim, ele sabe
que s vale 28 ou 17. Após Sérgio falar pela primeira vez, Paulo procura as
decomposições de 28: (2, 26), (3, 25), (4, 24), (5, 23),… Ele pode parar por aí e
concluir que s ≠ 28 pois (5, 23) é a única fatoração de 115 (5 e 23 são primos) e,
portanto, 115 ∉ P1 e 28 ∉ S1 . Agora, ele pode procurar as decomposições de 17 (só
para ter certeza que s = 17): (2, 15), (3, 14), (4, 13), (5, 12), (6, 11), (7, 10) e (8, 9).
Estas decomposições são fatorações, respectivamente, de 30, 42, 52, 60, 66, 70 e 72.
Todos estes números estão em P1 já que cada um deles tem outra fatoração diferente
daquela aqui mostrada. Paulo conclui que s = 17.
Sérgio deve trabalhar mais que Paulo. Porém, ele também pode achar o produto após
alguns minutos2.
Consideremos, agora, outros valores para N. Para valores pequenos não podemos
resolver o problema. Da mesma forma, para N grande, por exemplo N = 866, não é
possível determinar os números pois Pn = {52,244} e S n = {17,65} para todo N ≥ 3.
Para Paulo, o efeito de trocar N é quase nulo: sabendo que p = 52 ele encontra s = 17
para todo N ≥ 13 (fato em acordo com a intuição). Mesmo a ambigüidade que
aparece para nós quando N = 866 não atrapalha Paulo. Neste caso, se ele recebesse
p = 244, então acharia s = 65.
Quando N ≤ 25,(2,26) não é mais fatoração de 52. Assim, Paulo não teria dúvida
de que s = 17. Portanto, Sérgio não diria que Paulo não podia determinar a soma.
Paulo acharia s mas Sérgio não acharia p (como veremos abaixo).

2
O autor se pôs no lugar de Sérgio e após, aproximadamente, 15 minutos achou o produto.
Nos seus cálculos ele considerou em P0 \ P1 apenas os números que são produtos de dois
primos.

EUREKA! N°23, 2006

37
Sociedade Brasileira de Matemática

Para N ≤ 61 , a situação de Sérgio é similar a nossa: ele não pode achar p.


A razão é a mesma de antes: (4, 13) e (7, 10) são duas decomposições de 17 com
produtos em P2 . Outra maneira de ver este fato é a seguinte. Tome N = 61 e
coloquemo-nos no lugar de Paulo, primeiro com p = 52 e, depois, com p = 70.
Em ambos os casos, após a primeira frase de Sérgio, encontraríamos s = 17.
Logo, Sérgio não poderia decidir se p = 52 ou p = 70. Na verdade, existe ainda uma
terceira possibilidade, p = 66, que aparece quando N = 61. Para N = 866, Sérgio
encontra p = 52, quando s = 17, ou p = 244, quando s = 65.

SOLUÇÃO SEM ASSISTÊNCIA COMPUTACIONAL

Vejamos agora uma solução do problema original, com N = 100, que não necessita
de assistência computacional.
Determinar P0 ou P1 manualmente seria muito trabalhoso (lembre-se que, segundo o
programa, esses conjuntos têm, respectivamente, 2880 e 1087 elementos). Vamos
procurar S1 sem computador mas com paciência.
Para concluir que S ∉ S1 basta existir uma decomposição (i, j) de S que seja a única
fatoração de ij. Vejamos alguns casos (cuja verificação deixamos para o leitor). Para
S = 200 temos a decomposição (100, 100). Para S = 199 e S = 198 temos (99, 100) e
(99, 99), respectivamente.
Se 99 ≤ S ≤ 197,(i, j ) = (97, S − 97) resolve pois 2 ≤ S − 97 ≤ 100 e ij tem fator
primo 97, logo, i, digamos, tem fator 97. Como 97 > 50, se i tivemos outro fator
primo, então teríamos i > 100, que é absurdo. Concluímos que i = 97 e j = S – 97.
Portanto, (S, S – 97) é a única fatoração de ij. Analogamente, se 55 ≤ S ≤ 153, então
(53, S − 53) é a única fatoração de 53(S – 53).
Resumindo, até aqui mostraremos que se S ≥ 55, então S ∉ S1 . Agora, se S ≤ 54 e S é
par, então pode-se verificar, caso a caso, que S tem decomposição (i, j) com i e j
primos (o caso geral, isto é, sem cota superior para S, é um problema em aberto
conhecido como Conjectura de Goldbach), logo, S ∉ S1 .
Analogamente, os ímpares 5, 7, 9, 13, 15, 19, 21, 25, 31, 33, 39, 43 e 45 são todos
da forma 2 + P com P primo e, portanto, nenhum deles está em S1 . Finalmente, (17,
34) é uma decomposição de 51 que é a única fatoração de 578 = 2 ⋅ 17 2 (pois
172 > 100), ou seja, 51∉ S1.
Concluímos que S1 ⊂ Si = {11,17,23,27, 29,35,37, 41, 47,53}. Quem quiser pode
mostrar que vale a inclusão contrária, mas isto não será necessário à nossa
demonstração.

EUREKA! N°23, 2006

38
Sociedade Brasileira de Matemática

Determinar P2 também exigiria muito esforço. Por isto, passaremos diretamente para
S 2 . Observaremos que se S ∈ S2 , então existe uma única decomposição (i, j) de S tal
que ij ∈ P e esta é a única fatoração de ij com soma em S ⊂ Si .
2 1

Assim, para concluir que S ∈ Si \ S 2 basta exibir duas decomposições distintas


(i1 , j1 ) e (i2 , j2 ) de S tais que i1 j1 e i2 j2 admitam, no máximo, estas fatorações com
soma em Si. Por exemplo, 11∉ S . De fato, considere as decomposições (2, 9) e (3,
2
8). Elas têm produtos 18 e 24, respectivamente. Além de (2, 9) e (3, 8), estes
números admitem as seguintes fatorações: (3, 6), (2, 12) e (4, 6) cujas somas 9, 14 e
10 estão fora de Si.
Analogamente, mostra-se que 23 ∉ S2 considerando as decomposições (4, 19) e
(10, 13). Para 27, considere (4, 23) e (8, 19); para 29 considere (6, 23) e (10, 19).
Finalmente, para S ∈{35,37,41,47,53} considere (31, S − 31) e (29, S − 29) (mostre
caso a caso). Finalmente, concluímos que S 2 = {17}, logo, a soma é 17.
As fatorações de 17 são (2, 15), (3, 14), (4, 13), (5, 12), (6, 11), (7, 10) e (8, 9).
Os produtos correspondentes são 30, 42, 52, 60, 66, 70 e 72. Ora, o produto
procurando está em P2 , logo, ele tem uma única fatoração cuja soma está em
S ⊂ Si . O único dos números anteriores com esta propriedade é 52. Por exemplo,
1

30 = 2 ⋅ 15 = 5 ⋅ 6 e 5 + 6 = 11 ∈ Si (além disto, é claro que 2 + 15 = 17 ∈ Si ). De modo


análogo consideraremos
42 = 3 ⋅14 = 2 ⋅ 21, 60 = 5 ⋅13 = 3 ⋅ 20,66 = 6 ⋅11 = 2 ⋅ 33,70 = 7 ⋅10 = 2 ⋅ 35 e 72 = 8 ⋅ 9 = 3 ⋅ 24.
Logo, o produto é 52 e os números são 4 e 13.

Referências

[1] HANS FREUDENTHAL, Nicuw Archief Voor Wiskunde, Ser 3, 17 (1969) 152
[2] MARTIN GARDNER, Mathematical Games, Scientific American, 241 (Dec. 1979) 22.
[3] MARTIN GARDNER, Mathematical Games, Scientific American, 242 (May 1980)
[4] LEE SALLOWS, The Impossible Problem, The Mathematical Intelligencer, 17:1 (1995)
27.
[5] ISAACS I. M. ISAACS, The Impossible Problem Revisited Again, Tha Mathematical
Intelligencer, 17:4 (1195) 4.
[6] URL: www.labma.ufrj.br/~cassio/f-impossivel.html

EUREKA! N°23, 2006

39
Sociedade Brasileira de Matemática

COMO É QUE FAZ?

Resolveremos a seguir dois problemas da IV Olimpíada Iberoamericana de


Matemática Universitária, por sugestão de Macelo da Silva Mendes, de Teresina –
PI.

1) Seja α > 0 um número real. Sejam 0 < x1 < x2 < x3 < ... as soluções reais da

1
equação x ⋅ sen( x α ) = ln x. Encontre os valores de α para os quais ∑ converge.
n =1 xn

SOLUÇÃO: Suponhamos inicialmente que α ≥ 1. Dado k ∈ `* , temos


(
(2k π)1 α ⋅ sen  ( 2k π ) )
1α α 
( )
 = 0 < ln ( 2k π ) , enquanto

 1α α 
1       1  
1α 1α 1α
 1    1 
 + π
   + π  = + π >  +    ,
π
             
2 k sen 2 k 2 k ln 2 k
 2  2     2   2   
   
pois x > ln x, para todo x > 0. Assim, para todo k ∈ `* , existe uma solução
 1 α  1  

yk ∈  ( 2 k π ) ,   2 k +  π   da equação x ⋅ sen ( xα ) = ln x . Assim,


   2    

∞ ∞ ∞
1 1 1 1 ∞ 1

n =1 xn
≥∑
k =1   1 

≥ ∑
k =1  1
≥ ∑
2π k =1 k + 1
= +∞, caso α ≥ 1.
 + π
  2k + 2  π  2 
2 k
   
Consideremos agora o caso 0 < α < 1. Seja f ( x ) = x ⋅ sen( x α ). Temos
α α α
f '( x ) = sen( x ) + α ⋅ x ⋅ cos( x ). Para cada k ∈ `, existe um único

(
wk ∈ ( 2k π ) , (( 2k + 1) π )
1α 1α
) com f '( wk ) = 0; para ( 2k π )

< x < wk temos

f '( x ) > 0, e, para wk < x < (( 2k + 1) π ) , temos



f '( x ) < 0. De fato, para

 1 
0 < x ≤   2k +  π  , temos que f '( x ) > 0, f ' é decrescente no intervalo
 2 
  1 

(((2k + 1) π) ) < 0.
1
1  α
  2k +  π  , ((2k + 1) π ) α  e f '
1
α
  2  
 

EUREKA! N°23, 2006

40
Sociedade Brasileira de Matemática

 1    

1 

 1  

Como f    2k +  π   =   2k +  π  > ln    2k +  π   , e
 2      2   2   
 
f (((2k + 1)π) ) = 0 < ln (((2k + 1) π) ),
1α 1α
existe uma solução yk de f ( x ) = ln x

 1 
< x < (( 2k + 1) π ) . Para ((2k + 1) π ) ≤ x ≤ (( 2k + 2 ) π ) ,
1α 1α 1α
com   2k +  π 
 2 
x ∈  wk , (( 2k + 1) π )  , temos

temos f ( x ) ≤ 0 < ln x. Para f '( x) ≤ 0 e
 
1
(ln x )' = > 0, donde a equação f ( x ) = x tem no máximo uma solução em
x
 w , (( 2k + 1) π )1 α  .
 k 
 5  

 5  

( )
Como f (( 2k + 2) π) = 0 < ln ((2k +1) π)
1α 1α
( )
e f  2k +  π  > ln  2k +  π ,

 2    
 2   
existe uma menor solução uk de f ( x ) = ln x com

 5 
( ( 2k + 2 ) π ) < uk <   2k +  π  . Como f (uk ) = uk sen (ukα ) = ln uk , donde

 2 

sen (ukα ) = uk sen (ukα ) = ln uk , temos sen (ukα ) =


ln uk
.
uk

 5 
Temos, para uk < y ≤  2k +  π , f '( y ) ≥ sen y α ≥ sen (ukα ) =
ln uk 1
> = ( ln ) ' (uk )
 2  uk uk
(pois uk > ( 2π )

> 2π > e), donde f ( y ) > ln y , e logo uk é a única solução de
 1 α  5  

 1 

f ( x ) ln x em (( 2k 2 ) ) ,   2k
>  + π + π 
   . Se   2k +   ≤ y ≤ wk ,
π
   2     2  
 
 1    
1α 1α




2    
1 
2 
(
f ( y ) ≥ f    2k +  π   =   2k +  π  > ln (( 2k + 1) π ) > ln wk ≥ ln y

)
  1  

( 1α 1 
α 
) 1  e
(de fato, ln (( 2k + 1) π) = ln   2k +  π < ln (( 2k + 1) π) = e ln    2k +  π  ≤
2  α 
  2   

EUREKA! N°23, 2006

41
Sociedade Brasileira de Matemática


 1   1  8  π  π
≤   2k +  π  ; ln (( 2k +1) π) < eln  2k + π segue de ln π < ln   < eln   .
 2   2  3 2 2
Assim, yk e uk são as únicas soluções de f ( x ) = ln x em
  1α
1    5  

  2k +  π  ,   2k +  π   , para todo k ∈` . Finalmente, se


  2    2   

π
x ∈ ( 0,1) , f ( x ) > 0 > ln x e, para 1≤ x ≤   , temos
2
π x x
f ( x) ≥ x ⋅ sen1 x ⋅ sen = > ≥ ln x, donde não há solução de f ( x) = ln x em
6 2 e
  π 1 α  ∞
1 ∞ 1 ∞
1
 0,    . Assim, ∑ <∑ 2 ⋅ < 2 ⋅∑ < +∞, no caso
k =0 ( k + 1)
1α 1α
  2   n=1 xn k =0  1 
  2k + 2  π 
  
0 < α < 1.
A resposta é, portanto, {α∈ \ 0 < α < 1}.

π 2π
3π nπ
2) Calcular
n=1
∑ cos n ⋅ cos n n
⋅ cos
...cos .
n
SOLUÇÃO: Observemos inicialmente que, para todo m∈ < , temos
sen(mx) = senx ⋅ Qm (cos x), onde Q0 = 0 e, para m ≥ 1, Qm é um polinômio de grau
m −1 e coeficiente líder 2m−1. Além disso, o coeficiente constante de Qm é 0 se m é
par e é igual a (−1)k , se m = 2k + 1 , para todo k ≥ 0 . Isto pode ser provado a partir
da identidade sen (( m + 1) x ) + sen (( m − 1) x ) = 2sen ( mx ) ⋅ cos x; temos então
Q0 ( y) = 0 , Q1 ( y) = 1 e Qm +1 ( y) = 2 yQm ( y) − Qm −1 ( y), ∀m ≥ 1.
 kπ 
Por outro lado, como sen(kπ) = 0 e sen   ≠ 0 para todo k inteiro com
m
 kπ 
1 ≤ k ≤ m − 1, temos que cos   é raiz de Qm ( y ), para 1 ≤ k ≤ m − 1. Como Qm tem
m
 kπ 
grau m – 1, tem coeficiente líder 2m−1 e os números cos   ,1 ≤ k ≤ m − 1, são todos
m
m−1
  kπ 
distintos, temos Qm ( y) = 2m−1 ⋅ ∏ y − cos   , ∀m ≥ 1.
k =1   m 

EUREKA! N°23, 2006

42
Sociedade Brasileira de Matemática

m −1
 kπ 
Em particular, o coeficiente constante de Qm ( y ) é ( −2 ) ⋅ ∏cos  .
m −1

k =1 m
0,se m é par
m−1
 kπ  
Assim, ∏ cos   =  ( −1)r .
k =1  m   2 r , se m = 2r + 1, r ∈ `
 2
 nπ 
Como cos   = cos π = −1, a série que queremos calcular vale
 n 

( −1)r
−1 4
−∑ 2r
=
=− .
r =0 2
1 5
1+
4
_______________________________

O leitor João Alexandre Júnior enviou uma solução do problema No. 2 do artigo "Os
problemas do Visitante Matemático", que pedia para calcular 44 . Como 44 = 4256
4 4

João Alexandre calculou manualmente, de forma sucessiva, 42 = 16, 44 = 162 = 256,


48 = 2562 = 65536, 416 = 655362 = 4294967296, 432 = ( 416 ) = 18446744073709551616,
2

( ) ( )
2 2
464 = 432 = 340282366920938463463374607431768211456, 4128 = 464 =
=115792089237316195423570985008687907853269984665640564039457584007
( )
2
913129639936 e, finalmente, 4256 = 4128 = 1340780792994259709957402499820
5846127479365820592393377723561443721764030073546976801874298166903
427690031858186486050853753882811946569946433649006084096.

EUREKA! N°23, 2006

43
Sociedade Brasileira de Matemática


SOLUÇÕES DE PROBLEMAS PROPOSTOS
Publicamos aqui algumas das respostas enviadas por nossos leitores.

89) Uma prova de múltipla escolha com n questões é feita por k alunos. Uma
resposta correta na i-ésima questão vale pi pontos, onde pi é um inteiro positivo, para
1 ≤ i ≤ n. A nota de cada aluno é a soma dos pontos correspondentes às questões que
ele acertou. Após a realização a prova, foi observado que, mudando os pesos pi, as
notas dos alunos podem estar em qualquer uma das k! possíveis ordens (em que não
há duas notas iguais). Dado n, qual é o maior valor possível de k?

SOLUÇÃO DE ZOROASTRO AZAMBUJA NETO (RIO DE JANEIRO – RJ)


Vamos mostrar que o maior valor possível de k é n. De fato, é fácil ver k pode ser
igual a n: dada uma bijeção σ = {1, 2,..., n} → {1, 2,..., n}, e supondo que, para
1 ≤ i ≤ n, o i-ésimo aluno acertou a i-ésima questão e errou todas as outras, se
atribuirmos peso pi = n + 1 − σ (i) à questão i para 1 ≤ i ≤ n, temos que o aluno de
número σ −1 (i ) obteve a i-ésima melhor nota, para 1 ≤ i ≤ n.
Suponha agora que k alunos tenham feito k provas como no enunciado. Para cada
i ≤ k , denotamos por v(i) ∈{0,1}n o resultado da i-ésima prova:
v(i) = (vi1 , vi 2 ,..., vin ), onde vij = 1 se o i-ésimo aluno acertou a j-ésima questão, e
vij = 0 caso contrário. Se k > n, os vetores v(i),1 ≤ i ≤ k , são linearmente
dependentes, ou seja existem constantes ci ,1 ≤ i ≤ k , não todas nulas, tais que
k

∑ c v(i) = 0. Assim, passando os termos com


i =1
i ci negativo para o lado direito da

igualdade, obtemos constantes positivas a1 , a2 ,..., ar , b1 , b2 ,..., bs com


r s
r , s ≥ 1, r + s ≤ k e ∑ ai v (α (i ) ) = ∑ b j v ( β ( j ) ) , onde
i =1 j =1

α (1),α (2),...,α (r ), β (1), β (2),..., β (s ) são os índices i tais que ci ≠ 0 os α ( j ) são


tais que cα ( j ) > 0 e os β ( j ) são tais que cβ ( j ) < 0 .
Agora, se o peso da i-ésima questão é pi > 0, a nota da i-ésima prova é
n
n(v(i)) = ∑ vij ⋅ p j .
j =1

EUREKA! N°23, 2006

44
Sociedade Brasileira de Matemática

s r
bi a
Sejam λ = ∑ bj ∑ a , b i i = s
e a i = r i .
j =1 i =1
∑b
j =1
j ∑a
i =1
i

Podemos supor sem perda de generalidade que λ ≥ 1 (senão trocamos os lados da


igualdade).
s r
Como ∑ b j ⋅ v ( β ( j) ) = ∑ ai ⋅ v (α (i) ) , temos
j =1 i =1
s r

∑ b ⋅ v ( β ( j ) ) = λ ⋅ ∑ a ⋅ v (α (i) ) e, como n(v) depende linearmente de v, temos


j =1
j
i =1
i

s
 s    r  
∑ b j n (v ( β j )) = n  ∑ b j ⋅ v ( β j )  = n  λ ∑ a i ⋅ v (α (i ) ) 
 ( ) ( )
j =1  j =1   i =1 
r r
= λ∑ a i ⋅ n (v (α (i )) ≥ ∑ a i ⋅ n (v (α (i) )) ,
i =1 i =1
s r s r
mas como ∑ b j = ∑ a i = 1, temos que
j =1 i =1
∑ b j ⋅ n (v ( β ( j) )) e
j =1
∑ a ⋅ n (v (α (i) ))
i =1
i

são respectivamente médias ponderadas dos n (v ( β ( j ) )) e dos n (v (α (i) )) , e a


desigualdade acima
claramente implica que não podemos ter
( ) ( )
n v (α (i) ) > n v ( β ( j ) ) para todo i ≤ r e j ≤ s , e portanto as notas não podem
ficar em qualquer ordem.
Nota: Se k > n então quaisquer vetores v1 , v2 ,..., vk ∈ \ n = {( x1 , x2 ,..., xn ); xi ∈ \ ,
∀i ≤ n} são linearmente dependentes, isto é, existem constantes ci ∈ \,1 ≤ i ≤ k
k
não todas nulas tais que ∑c v
i =1
i i = c1v1 + ... + ck vk = 0. Podemos provar isso por

indução em n. O resultado é claramente verdade se n = 1: se v1 , v2 ∈ \ não são


ambos nulos, c1 = − v2 e c2 = v1 são tais que c1v1 + c2 v2 = 0. Se m < n podemos
identificar \
m
com{( x1, x2 ,..., xm ,0,0,...,0) ∈ \n ; xi ∈ \, ∀i ≤ m}. Se vi = 0 para
algum i, podemos tomar ci = 1 e c j = 0, ∀j ≠ i. Se k > n + 1, e v1 , v2 ,..., vk são
vetores em \
n+1
com vi = ( x1(i ) ,..., xn(i+)1 ) para cada i ≤ k , temos duas
possibilidades:

EUREKA! N°23, 2006

45
Sociedade Brasileira de Matemática

i) xn(i+)1 = 0 para todo i ≤ k . Nesse caso vi ∈ \ n para todo i ≤ k e o resultado segue


imediatamente pela hipótese de indução.

ii) xn(i+)1 ≠ 0 para algum i. Podemos supor sem perda de generalidade que isso vale
para i = k.
x n( +j )1
Nesse caso temos v i = v j − ( k ) v k ∈ \ n para todo j com 1 ≤ j ≤ k − 1 e, como

x n +1
(n + 1)-ésima coordenada desses vetores se anula e k – 1 > n, por hipótese de
k −1
indução, existem c1 ,..., c k −1 ∈ \ não todos nulos tais que ∑ c
j =1
j ⋅ v j = 0, mas
k −1
v j = v j − a j ⋅ vk , onde a j = xn( +j )1 xn( k+)1 , e portanto ∑ c ( v
j =1
j j − a j vk ) = 0, donde
k k −1

∑ c j v j = 0, onde c j = c j para 1 ≤ j ≤ k − 1 e ck = −∑ a j c j , o que prova o


j =1 j =1
resultado.

97) Seja p um primo ímpar. Encontre todas as funções f : ] → ] que satisfazem as


seguintes condições:

i) Se m ≡ n(mod p) então f (m) = f (n).


ii) f (mn) = f (m) ⋅ f (n) para quaisquer m, n∈ ].

SOLUÇÃO DE EDEL PÉREZ CASTILLO (PINAR DEL RIO – CUBA)


Temos f (1) = f (1) ⋅ f (1) , o que implica f (1) ∈ {0,1}. Se f (1) = 0, então
f (m) = f (m ⋅1) = f (m) ⋅ f (1) = 0 , para todo m, o que claramente é uma solução.
Suponhamos então que f (1) = 1. De f (0) = f (0) ⋅ f (0), temos f (0) ∈{0,1}. Se
f (0) = 1, então f (0) = f (0 ⋅ m) = f (0) ⋅ f (m) implica f (m) = 1, para todo m, o
que também é uma solução.
Suponhamos agora que f (0) = 0. Se m é múltiplo de p, ou seja, m ≡ 0(mod p),
temos também f (m) = 0. Se m não é múltiplo de p então existe um inteiro x tal que
m ⋅ x ≡ 1(mod p) , o que implica 1 = f (1) = f ( m) ⋅ f ( x), e logo f (m) ∈ {−1,1},
pois f (m) e f ( x ) são inteiros.

EUREKA! N°23, 2006

46
Sociedade Brasileira de Matemática

Se p /| m e m é resíduo quadrático módulo p, i.e., se existe b inteiro com


b 2 ≡ m(mod p ) então f (m) = f (b) 2 = 1, pois f (b) ∈ {−1,1}.
Se m e n não são resíduos quadráticos módulo p então m ⋅ n é resíduo quadrático
módulo p (veja [MS]), e logo f (mn) = f (m) ⋅ f (n) = 1. Assim, f (m) = f (n), ou
seja, f toma o mesmo valor pertencente a {–1, 1} em todos os inteiros que não são
resíduos quadráticos módulo p.
Assim, as funções f que satisfazem as condições do enunciado são:
1) f (n) = 0, ∀n ∈ ]
2) f (n) = 1, ∀n ∈ ]
3) f (n) = 0 se p| n e f (n) = 1 se p /| n
4) f (n) = 0 se p| n, f (n) = 1 se p /| n e n é resíduo quadrático módulo p e
f (n) = −1 se n não é resíduo quadrático módulo p.

Nota: A função em 4) é usualmente denotada pelo símbolo de Legendre


n
f (n) =   . Veja a referência [MS]: Carlos G. Moreira e Nicolau Saldanha.
 p
Reciprocidade Quadrática. Eureka! No. 15, pp. 27 – 30.

98) Seja (an ) n∈< uma seqüência tal que a1 > 2 e an +1 = an2 − 2, ∀n ∈ `.

1 a1 − a12 − 4
Mostre que ∑ = .
n =1 a1 ⋅ a2 ⋅ ... ⋅ an 2

SOLUÇÃO DE RODRIGO VILLARD MILET e MÁRCIO ASSAD COHEN (RIO DE JANEIRO – RJ)
1
Faça a1 = x + ( isso é possível pois a1 > 2 ). Podemos considerar, sem perda de
x
1
generalidade, que x > 1 (se não for, troque x por ). Temos, por indução, que
x
2
2n −1 1  n −1 1  1
+ n −1 , pois  x 2 + n −1  − 2 = x 2 + n .
n
an = x
x2  x2  x2
Veja que é fácil calcular o produto dos “ai”, pois
 1  1  2 1  4 1   2 − 1  1
 x −  x +  x + 2  x + 4 ... x + n −1  = x 2 − n .
n 1 n
Então temos
 x  x  x  x   x 2
 x2

EUREKA! N°23, 2006

47
Sociedade Brasileira de Matemática

1
x−
1  ∞  x 2 
n+1
2n n

que ∑
∞ 1
=∑

x =  x − 1  ∞ x 
=  x − ∑ −
x2 .
∑ 2n+1 
n=1 a1 a 2 ...a n n =1
x2
n

1  x n=1 x −1  x  n =1 2n
 x −1 x
2n +1
− 1 
n
x2
O legal é que chegamos em uma soma telescópica:
N  x2 
n n +1 N +1
 x2  x2 x2
∑ − = − . Para ` → ∞ , a segunda parcela tende
n =1 x 2 − 1 2 n +1  x 2 − 1 x 2 N +1 − 1
n
 x − 1 
x2 1
a 1 (pois x > 1), logo este último somatório é igual a −1= 2 , portanto
x −1
2
x −1
∞ 1  1 1 1
∑ =x −  2 = . Isso finaliza o problema, pois
n =1 a1 a 2 ...a n  x  x −1 x
a ± a1 − 4
2
1
a1 = x + ⇒ x 2 − a1 x + 1 = 0 ⇒ x = 1 . Como x > 1, temos
x 2
a1 + a1 − 4 1 a1 − a1 − 4
2 2

x= . Isso dá = .
2 x 2

100) a) Um conjunto X ⊂ ` é dito impressionante se existe m ∈ ` tal que, para


todo k ∈ ` , existem elementos de X, a1 < a2 < ... < ak , tais que
a j +1 − a j ≤ m, ∀j < k .
Determine se é possível particionar ` em um número finito de conjuntos, nenhum
deles impressionante.
b) Determine se é possível particionar ` em dois conjuntos A e B de modo que nem
A nem B contêm progressões aritméticas infinitas mas, para cada q ∈ `, A e B
contêm progressões aritméticas de q termos.

SOLUÇÃO DE JOSÉ DE ALMEIDA PANTERA (RIO DE JANEIRO – RJ)


Não. Vamos provar, por r ∈ ` = {1, 2,3,...} que, se
indução em
` = A1 ∪ A2 ∪ ... ∪ An então existe j com 1 ≤ j ≤ r tal que A j é impressionante.

Para r = 1 isto é óbvio.


Suponha agora que ` = A1 ∪ A2 ∪ ... ∪ An ∪ An +1 , e que An +1 não é
impressionante. Então existem intervalos arbitrariamente grandes contidos em
` \ Ar +1 , isto é, existe uma seqüência infinita de inteiros positivos ( a j ) j∈< tal que

EUREKA! N°23, 2006

48
Sociedade Brasileira de Matemática

a j +1 > a j + j, ∀j ∈ `, e, para todo j ∈ ` e 1 ≤ r ≤ j , a j + s ∉ Ar +1 . Para cada


j ∈ `, seja f j :{1, 2,..., j} → {1, 2,..., r} tal que a j + s ∈ Af j ( s ) , para 1 ≤ s ≤ j.
Construiremos recursivamente g (1), g (2), g (3),... ∈ {1, 2,..., r} tais que, para todo
m ∈ `, existem infinitos j ≥ m tais que f j ( s ) = g ( s ), para 1 ≤ s ≤ m.
E ∈ `, se já escolhemos g(m) com a propriedade acima para todo
De fato, dado m
E , existem infinitos j ≥ m
m<m E tais que f j ( s) = g ( s), para todo s < m
E . Como só
( )
E para esses infinitos j, podemos extrair
há n possibilidades para o valor de f j m

( )
E tem sempre o mesmo valor, o
um subconjunto infinito desses j para os quais f j m

( )
E , por definição.
qual será g m

Considere agora a decomposição =E A1 ∪ EA2 ∪ ... ∪ E


` An , onde
n∈ E
A j ⇔ g ( n) = j. Por hipótese de indução, ∃q ≤ r tal que E
Aq é impressionante.
Afirmamos que o conjunto Aq também é impressionante. De fato, seja m tal que para
todo k ∈ `, existem elementos b < b < ... < b de E
1 2 Aq com b − b ≤ m, ∀i < k .
k i +1 i

Para esse mesmo m, dado k ∈ `, escolhemos j ≥ bk tal que f j ( s ) = g ( s ), para


1 ≤ s ≤ bk . Como bi é um elemento de EAq para 1 ≤ i ≤ k , g (bi ) = q, para 1 ≤ i ≤ k .
Assim, f j (bi ) = g (bi ) = q, para 1 ≤ i ≤ k , ou seja, a j + bi ∈ Aq , para
1 ≤ i ≤ k . Como ( a j + bi +1 ) − ( a j + bi ) = bi +1 − bi ≤ m, ∀i < k , concluímos que Aq é
impressionante.

b) Sim. Temos ` = A ∪ B, onde


A = {n ∈ ` 2 ≤ n < 22 k +1 , para algum inteiro k ≥ 0} e
2k

+ +
B = {n ∈ ` 22k 1 ≤ n < 22k 2 , para algum inteiro k ≥ 0} .
Temos que [0, +∞) \ A e [0, +∞) \ B contêm intervalos arbitrariamente grandes,
donde nem A nem B podem conter progressões aritméticas infinitas. Por outro lado,
para cada q ∈ `, se k ∈ ` é tal que 22 k ≥ q, A contém a progressão aritmética da
razão 1 e q termos {22 k , 2 2 k + 1,..., 2 2 k + q − 1}, e B contém a progressão aritmética
da razão 1 e q termos {22 k +1 , 22 k +1 + 1,..., 22 k +1 + q − 1}.

EUREKA! N°23, 2006

49
Sociedade Brasileira de Matemática

102) Você recebe x metros de arame para cercar um terreno na forma de um


triângulo pitagórico (os lados são números inteiros), com a condição de que a
medida do cateto menor seja 24 metros. Qual deverá ser a medida do cateto maior e
o comprimento do arame, a fim de que a área seja:
a) máxima?
b) mínima?

SOLUÇÃO DE JOSÉ FABRICIO LIMA (JOÃO PESSOA – PB)


Considere o triângulo abaixo:
Temos: a 2 = b 2 + 242
a 2 − b2 = 242
(a + b) ⋅ (a − b) = 26 ⋅ 32

a
b

24

Façamos 26 ⋅ 32 como sendo um produto de 2 fatores x ⋅ y tal que a + b = x e


x+ y x− y
a − b = y. Resolvendo o sistema temos que: a = e b= .
2 2
Como x + y e x – y são ambos divisíveis por 2, temos que x, y são números pares
pois a decomposição de 26 ⋅ 32 para x e y números ímpares é impossível.
x− y
Sendo b > 24 temos que > 24, logo x − y > 48.
2
Portanto as decomposições x ⋅ y tais que x − y > 48 são dadas por:
x = 25 ⋅ 32 e y = 2 ( x − y = 286 )
x = 2 4 ⋅ 32 e y = 22 ( x − y = 140 )
x = 23 ⋅ 32 e y = 23 ( x − y = 64 )
x = 25 ⋅ 3 e y = 2 ⋅ 3 ( x − y = 90 )

EUREKA! N°23, 2006

50
Sociedade Brasileira de Matemática

Note que a área do triângulo é 12 ⋅ b , sendo assim, para que a área seja:

a) Máxima: Basta que b seja o maior possível, ou seja, a diferença x – y seja a maior
x+ y
possível. Devemos ter então x = 25 ⋅ 32 = 288 e y = 2, donde a = = 145 e
2
x− y
b= = 143, e logo o comprimento da corda deve ser a + b + 24 = 312m.
2

b) Mínima: Basta escolher o menor valor para x – y, ou seja,


x − y 64 x + y 80
b= = = 32 e a = = = 40.
2 2 2 2
Logo, o comprimento de corda é:
x = a + b + 24 = 40 + 32 + 24 = 96m.

103) Sejam A e B matrizes 2 × 2 com elementos inteiros.


Sabendo que A, A + B, A + 2B, A + 3B e A + 4B são invertíveis e que os
elementos das respectivas inversas também são todos inteiros, mostre que
A + 5B também é invertível e que os elementos da sua inversa também são
inteiros.

SOLUÇÃO DE ASDRÚBAL PAFÚNCIO SANTOS (BOTUCATU - SP)


Para cada t ∈ \ , seja f (t ) o determinante da matriz A + tB. Temos que f(t) é uma
função polinomial de grau no máximo 2. Efetivamente, se
a b1   a2 b2   a1 + ta2 b1 + tb2 
A= 1  e B=  , f (t ) = det  + =
 c1 d1   c2 d2   c1 tc2 d1 + td2 
= ( a1 + ta2 )( d1 + td2 ) − (b1 + tb2 )(c1 + tc2 ).
Se uma matriz C com elementos inteiros é invertível e os elementos de sua inversa
são também inteiros, temos 1 = det I = det(C ⋅ C −1 ) = det C ⋅ det C −1 e, como
det C e det C −1 são inteiros, det C ∈ {−1,1}.

Assim, f (t ) = det( A + tB ) ∈ {−1,1}, para todo t ∈{0,1, 2,3, 4}, e logo um dos
valores 1 ou –1 é igual a f(t) para pelo menos três valores de t em {0, 1, 2, 3, 4}.
Como f(t) é um polinômio de grau no máximo 2, f(t) é necessariamente constante
igual a 1 ou –1, e logo, para todo t ∈ ] (e em particular para t = 5), A + tB tem

EUREKA! N°23, 2006

51
Sociedade Brasileira de Matemática

elementos inteiros e determinante pertencente a { –1, 1}, e logo tem uma inversa
com todos os elementos inteiros.

104) ABC é um triângulo. Mostre que existe um único ponto P de modo que:
( PA )2 + ( PB )2 + ( AB )2 = ( PB )2 + ( PC )2 + ( BC )2 = ( PC )2 + ( PA )2 + (CA)2
SOLUÇÃO DE FRANK DE CASTRO (SÃO PAULO – SP)
Considere o triângulo A ' B ' C ' cujos lados paralelos aos lados do triângulo ABC e
passam por A, B e C.
Considere também o ponto P, ortocentro do triângulo A'B'C' (figura abaixo)
A'

P
B
C

B' C'
A

Provaremos que:
( PA)2 + ( PB )2 + ( AB)2 = ( PB )2 + ( PC )2 + ( BC )2
P é ortocentro de A ' B ' C ' ⇔ 
= ( PC ) + ( PA) + (CA)
2 2 2

Nesse caso o ponto procurado será o ponto P e como o ortocentro do triângulo


A'B'C' é único, segue diretamente a unicidade do ponto P.
Utilizaremos vetores.
Temos:
JJJG JJJG
PA ' ⊥ CB ⇔ ( A '− P ) ⋅ ( B − C ) = 0 ⇔ A '⋅ B − A '⋅ C − P ⋅ B + P ⋅ C = 0 (I)
Como o quadrilátero A'BAC é um paralelogramo temos A '− C = B − A , ou
A ' = C + B − A (II).
JJJG JJJG
De (I) e (II) vem que PA ' ⊥ CB ⇔ (C + B − A ) ⋅ ( B − C ) − P ⋅ B + P ⋅ C = 0 ⇔
⇔ C ⋅ B − C 2 + B2 − B ⋅ C − A ⋅ B + A ⋅ C − P ⋅ B + P ⋅ C = 0 .

EUREKA! N°23, 2006

52
Sociedade Brasileira de Matemática

JJJG JJJG
Sendo C ⋅ B = B ⋅ C , Vem que PA ' ⊥ CB ⇔ B2 − P ⋅ B − A ⋅ B = C 2 − P ⋅ C − A ⋅ C.
Multiplicando a igualdade anterior por 2 e na seqüência somando P 2 + A2 a ambos
os membros, segue que:
JJJG JJJ
G
PA ' ⊥ CB ⇔ P2 + B2 − 2 P ⋅ B + A2 − 2 A ⋅ B + B2 = P 2 + C 2 − 2P ⋅ C + A2 − 2 A ⋅ C +
C2 ⇔ ( P − B) + ( A − B) = ( P − C ) + ( A − C ) .
2 2 2 2

Finalmente, somando ( P + A )2 a ambos os membros da última igualdade obtida


concluímos que:
JJJG JJJG
PA ' ⊥ CB ⇔ ( P − A) + ( P − B ) + ( A − B ) = ( P − C ) + ( P − A) + ( A − C ) ⇔
2 2 2 2 2 2

⇔ ( PA) + ( PB ) + ( AB ) = ( PC ) + ( PA) + (CA) .


2 2 2 2 2 2

racionando de forma análoga teremos:


JJJG JJJG
PB ' ⊥ CA ⇔ ( PA) + ( PB) + ( AB) = ( PB) + ( PC) + ( BC ) , onde B ' = A + C − B, e
2 2 2 2 2 2

JJJG JJJG
PC ' ⊥ BA ⇔ ( PB) + ( PC ) + ( BC) = ( PC) + ( PA) + (CA) , onde C ' = B + A − C.
2 2 2 2 2 2

Nessas condições, o ortocentro do triângulo A ' B ' C ' é o ponto P procurado.

Nota: Utilizamos:
JJJG JJJG
( X − Y ) ⋅ ( Z − W ) representando o produto escalar dos vetores YX e WZ .
JJJG JJJG
( X − Y ) ⋅ ( Z −W ) = 0 ⇔ YX ⊥ WZ.
JJJG JJJG
X ⋅ Y = ( X − O ) ⋅ (Y − O) = OX ⋅ OY onde o ponto O é a origem do nosso plano.
JJJG JJJG JJJG 2
X 2 = ( X − O) ⋅ ( X − O) = OX ⋅ OX = OX = (OX ) = ( XO ) .
2 2

JJJG JJJG JJJG 2 JJJG 2


( X − Y ) = ( X − Y ) ⋅ ( X − Y ) = YX ⋅ YX = YX = XY .
2

105) O baricentro do triângulo ABC é G. Denotamos por g a , gb , gc as distâncias


desde G aos lados a, b e c respectivamente.

Seja r o raio da circunferência inscrita. Prove que:


2r 2r 2r
a) g a ≥ , gb ≥ , g c ≥
3 3 3
g + gb + gc
b) a ≥3
r

EUREKA! N°23, 2006

53
Sociedade Brasileira de Matemática

SOLUÇÃO DE CARLOS ALBERTO DA SILVA VICTOR (NILÓPOLIS – RJ)


A

G
ga ha

B C

Observe que ha = 3 g a
a ⋅ ha 1  a+b+c
s = p⋅r = ⇒ ga = ⋅   ⋅ r.
2 3  a 
1  a+b+c 1  a +b+c
De forma análoga, temos: gb = ⋅   ⋅ r e gc = ⋅   ⋅ r.
3  b  3  c 
a) como a < b + c ∴ a + b + c > 2a e
1  a +b+c 1 2a 2r
ga = ⋅   ⋅ r ⇒ g a > ⋅ ⋅ r = ; de forma análoga, temos:
3  a  3 a 3
2r 2r
gb > e gc > .
3 3
2r
Note que a igualdade g a = não ocorrerá, pois caso contrário, teríamos:
3
1 a+b+c 2r
 ⋅r = ⇒ a + b + c = 2a ⇒ b + c = a e o triângulo não existiria.
3 a  3
r a +b + c a +b + c a +b + c 
g a + gb + gc =  + + 
3 a b c

NNN
 
r  b a a c b c
g a + gb + gc = 3 + + + + + +
3  a b c a c b
 ≥2 ≥2 ≥2  
r
logo g a + gb + gc ≥ (3 + 2 + 2 + 2 ) = 3r ou seja:
3
g a + gb + g c
≥ 3.
r
EUREKA! N°23, 2006

54
Sociedade Brasileira de Matemática

106) Os polinômios P0 ( x, y, z ), P1 ( x, y, z ), P2 ( x, y, z ),... são definidos por


P0 ( x, y, z ) = 1 e Pm+1 ( x, y , z ) = ( x + z )( y + z ) Pm ( x, y , z + 1) − z 2 Pm ( x, y, z ),
∀m ≥ 0. Mostre que os polinômios Pm ( x, y , z ), m ∈ ` são simétricos em x, y, z,
i.e., Pm ( x, y, z) = Pm ( x, z, y) = Pm ( y, x, z) = Pm ( y, z, x) = Pm ( z, x, y) = Pm ( z, y, x), para
quaisquer x, y, z.

SOLUÇÃO DE MARCOS FRANCISCO FERREIRA MARTINELLI (RIO DE JANEIRO – RJ)


Provarei, por indução finita, o seguinte:
I) Pn ( x, y , z ) = Pn ( x, z , y ) = Pn ( z , y , x ) = Pn ( y , x, z ) , ∀n ∈ `.
II) ( z − y ) Pn ( x, y , z ) + ( x + y ) Pn ( x, y + 1, z ) = ( x + z ) Pn ( x, y , z + 1) , ∀n ∈ `
i) P1 ( x, y, z ) = xy + xz + yz = P1 ( x, z , y ) = P1 ( z , y, x ) = P1 ( y, x, z ).
E ainda ( z − y ) P1 ( x, y , z ) + ( x + y ) P1 ( x, y + 1, z ) − ( x + z ) P1 ( x, y , z1 ) =
= ( z − y)( xy + xz + yz) +( x + y) x( y +1) + xz +( y +1) z −( x + z) xy + x( z +1) + y ( z +1) =
= ( z − y) + ( x + y) ( xy + xz + yz) + ( x + y)( x + z) − ( x + z)( xy + xz + yz) = ( x + z)( x + y) =
= ( z + x ) + ( xy + xz + yz ) − ( x + z )( xy + xz + yz ) = 0. Logo está provado que
( z − y) P1 ( x, y, z) +( x + y) P1 ( x, y +1, z) = ( x + z) ⋅ P1 ( x, y, z +1).

ii) Suponhamos que para n = m ( m∈`) temos:


( z − y) Pm ( x, y, z ) + ( x + y ) Pm ( x, y +1, z ) − ( x + z ) ⋅ Pm ( x, y, z +1) = 0.
Logo ( z − y ) Pm+1 ( x, y, z ) + ( x + y) Pm+1 ( x, y +1, z ) − ( x + z ) Pm+1 ( x, y, z +1) =
= ( z − y )  ( x + z )( y + z ) Pm ( x , y , z + 1) − z 2 Pm ( x , y , z ) +
+ ( x + y ) ( x + z )( y + z + 1) Pm ( x , y + 1, z + 1) − z 2 Pm ( x , y + 1, z ) −

− ( x + z ) ( x + z +1)( y + z +1) Pm ( x, y, z + 2) − ( z +1) Pm ( x, y, z +1) =


2

 
= − z (( z − y ) Pm ( x, y, z + 1) + ( x + y ) Pm ( x, y + 1, z ) − ( x + z ) Pm ( x, y, z + 1)) +
2

+( x + z)( y + z +1) (( z +1− y) Pm ( x, y, z +1) +( x + y) Pm ( x, y +1, z +1) −( x + z +1) Pm ( x, y, z +2)) = 0


(aqui usamos também a versão de (II) para n = m trocando z por z + 1).
Isto prova (II) para n = m + 1.

EUREKA! N°23, 2006

55
Sociedade Brasileira de Matemática

Supondo ainda para n = m ( m ∈ ` ) que


Pm ( x, y, z ) = Pm ( x, z, y ) = Pm ( z, y, x ) = Pm ( y, x, z ) , temos:
Pm+1 ( x, y, z ) − Pm+1 ( y, x, z ) = ( x + z )( y + z ) Pm ( x, y, z + 1) − z Pm ( x, y, z ) −
2

( y + z )( x + z ) Pm ( y, x, z + 1) − z 2 Pm ( y, x, z ) . Mas da hipótese de indução:


Pm+1 ( x, y, z ) − Pm+1 ( y, x, z ) = ( x + z )( y + z )  Pm ( x, y, z + 1) − Pm ( y, x, z + 1) −
− z 2  Pm ( x, y, z ) − Pm ( y, x, z ) = 0 ⇒ Pm+1 ( x, y, z ) = Pm+1 ( y, x, z ) ⋅
Pm+1 ( x, y, z ) − Pm+1 ( z, y, x ) = ( x + z )( y + z ) Pm ( x, y, z + 1) − z Pm ( x, y, z ) −
2

− ( x + z )( y + x ) Pm ( z, y, x + 1) − x2 Pm ( z, y, x ) =
= ( x + z ) ( y + z ) Pm ( x, y, z + 1) − ( y + x ) Pm ( z, y, x + 1) − ( z − x ) Pm ( z, y, x ) .
E, por hipótese de indução e (II) temos:
( y + z ) Pm ( x, y, z + 1) − ( y + x ) Pm ( x + 1, y, z ) − ( z − x ) Pm ( x, y, z ) =
( y + z ) Pm ( y, x, z + 1) − ( y + x ) Pm ( y, x + 1, z ) − ( z − x ) Pm ( y, x, z ) = 0 ⇒
Pm+1 ( x, y, z ) = Pm+1 ( z, y, x ).
A partir daí temos:
Pm+1 ( x, z, y) = Pm+1 ( z, x, y) = Pm+1 ( y, x, z) = Pm+1(x, y, z), c.q.d.

107) a) Dado um triângulo qualquer, prove que existe um círculo que passa pelos
pontos médios dos seus lados, pelos pés das suas alturas e pelos pontos médios dos
segmentos que unem o ortocentro aos vértices do triângulo (o chamado "círculo dos
nove pontos").

b) Prove que, se X é o centro do círculo dos nove pontos de um triângulo, H o seu


JJJG 3 JJJG 1 JJJG
ortocentro, O seu circuncentro e G seu baricentro, então OX = ⋅ OG = ⋅ OH .
2 2

EUREKA! N°23, 2006

56
Sociedade Brasileira de Matemática

SOLUÇÃO DE ANDERSON TORRES e CARLOS ALBERTO DA SILVA VITOR (SÃO PAULO – SP,
NILÓPOLIS – RJ)
A

HB
TA

HC
H O

B MA
C
HA

M'
H'

a) Seja H A o pé altura por A, M A o ponto médio de BC e TA o ponto médio de


AM.
Aplicando uma homotetia de centro H e razão 2, o ponto TA é levado no ponto A.
Assim sendo, o que queremos provar é que H ' e M ' (os pontos em que H A e M A
são levados na homotetia) estão no circuncírculo de ABC.
Temos que ∆BM A H ≡ ∆CM A M '
(de fato, LAL : BM A = CM A , ∠BM A H = CM AM ', M A H = M A M ' ). Analogamente
∆HM AC ≡ ∆M ' M A B. Com isto, ∆BHC ≡ ∆CM ' B pelo caso LLL (isto sai das
congruências).
Portanto ∠BHC = ∠BM ' C .
BC é mediatriz de HH ' (de fato HH A = H A H ' e ∠HH A B = ∠HH AC = 900 ).
Assim, ∆BHC ≡ ∆BH ' C e assim ∠BH ' C = ∠BHC = ∠BM ' C.
Para finalizar, basta demonstrar que ∠BHC + ∠BAC = 180°. E isto é simples:
∠BHC = ∠H B HH C , e o quadrilátero AH C HH B é cíclico
(∠AH C H + ∠AH B H = 90° + 90° = 180° ).
Assim ∠H B HH C + ∠BAC = 180° ⇒ ∠BHC + ∠BAC = 180°. c.q.d.

b) Se X é o centro deste círculo, a homotetia leva X em O.

EUREKA! N°23, 2006

57
Sociedade Brasileira de Matemática

JJJJG
JJJG JJJG JJJGOH
Assim HX = XO, e assim OX = . Pelo Lema abaixo,
2
JJJG JJJG
JJJG JJJG JJJG OH 3 ⋅ OG
OH = 3 ⋅ OG , e fim: OX = = , c.q.d.
2 2
Lema: O baricentro, o circuncentro e o ortocentro de qualquer triângulo estão
alinhados.

Prova: Considere a figura abaixo:

c H T
b

O S
R A
C
B M HA

AB = R (raio do círculo circunscrito).


c
= 2 R (da lei dos senos).
sen C
Seja O o circuncentro de ABC.
A reta OH intersecta AM em S. Observe que OM // AH A e que:
AT AT c
AT = c ⋅ cos A ⇒ AH = = = ⋅ cos A = 2 R cos A
cos(90 − C ) sen C sen C
AH AS
Do ∆OBM ⇒ OM = R cos A = , logo MS = e S é o baricentro G do
2 2
∆ABC.
JJJG JJJG
Assim, O, G e H estão alinhados, e OH = 3 ⋅ OG.

EUREKA! N°23, 2006

58
B
Sociedade Brasileira de Matemática
x C
D PROBLEMAS PROPOSTOS
Convidamos o leitor a enviar soluções dos problemas propostos e sugestões de novos problemas
para os próximos números.

108) Sejam A1 , A2 ,..., An conjuntos finitos. Para 1 ≤ k ≤ n, seja


Sk = ∑
1≤ i1 < i2 <...< ik ≤ n
Ai1 ∩ Ai2 ∩ ... ∩ Aik , a soma dos números de elementos das

interseções de k dos conjuntos Ai . Prove que:


a) O número de elementos que pertencem a exatamente r dos conjuntos Ai é
n
k
∑ ( − 1)
k−r
  S k , para 1 ≤ r ≤ n.
k =r r 
b) O número de elementos que pertencem a pelo menos r dos conjuntos Ai é
n
 k − 1
∑ ( − 1)
k −r
 −  S k , para 1 ≤ r ≤ n.
k =r r 1

109) Na figura abaixo, AB = AC , B l


AC = 100° e AD = BC . Mostre que
l é racional quando expresso em graus.
x = BCD
A

110) Um conjunto finito de inteiros positivos é chamado de Conjunto DS se cada


elemento divide a soma dos elementos do conjunto.
Prove que todo conjunto finito de inteiros positivos é subconjunto de algum
conjunto DS.

EUREKA! N°23, 2006

59
Sociedade Brasileira de Matemática

111) Prove que existem infinitos múltiplos de 7 na seqüência (an ) abaixo:


a 1 = 1 9 9 9 , a n = a n- 1 + p ( n ) , " n ³ 2 , onde p(n) é o menor primo que divide n.

112) a) Determine todos os inteiros positivos n tais que existe uma matriz n × n com
todas as entradas pertencentes a { –1, 0, 1} tal que os 2n números obtidos como
somas dos elementos de suas linhas e de suas colunas são todos distintos.

b) Para os inteiros positivos n determinados no item anterior, encontre o número de


matrizes n × n com a propriedade do enunciado.

Problema 109 proposto por José do Nascimento Pantoja Júnior de Ananindeua – PA,
problemas 110 e 111 propostos por Anderson Torres de São Paulo – SP.

Agradecemos também o envio das soluções e a colaboração de:

Diego Andrés de Barros Recife – PE


Dymitri Cardoso Leão Recife – PE
Geraldo Perlino Itapecerica da Serra – SP
Glauber Moreno Barbosa Rio de Janeiro – RJ
Kellem Corrêa Santos Rio de Janeiro – RJ
Macelo da Silva Mendes Teresina – PI
Marcos Francisco Ferreira Martinelli Rio de Janeiro – RJ
Michel Faleiros Martins Campinas – SP
Raphael Constant da Costa Rio de Janeiro – RJ
Rodrigo Cardaretti dos Nascimento Curitiba – PR
Rodrigo Peres Barcellos Rio de Janeiro – RJ

9RFr VDELD«
":0  ²  p SULPR" (VWH Q~PHUR GH  GtJLWRV p R
4
PDLRU SULPR FRQKHFLGR H p R   3ULPR GH 0HUVHQQH GHVFREHUWR
SHOR *,036 YHMD ZZZPHUVHQQHRUJ SDUD PDLV LQIRUPDo}HV
LQFOXVLYH VREUH FRPR DMXGDU D DFKDU RXWURV SULPRV GH 0HUVHQQH 
(VWD GHVFREHUWD IRL IHLWD HP 

EUREKA! N°23, 2006

60
Sociedade Brasileira de Matemática

AGENDA OLÍMPICA

XXVIII OLIMPÍADA BRASILEIRA DE MATEMÁTICA

NÍVEIS 1, 2 e 3
Primeira Fase – Sábado, 10 de junho de 2006
Segunda Fase – Sábado, 2 de setembro de 2006
Terceira Fase – Sábado, 28 de outubro de 2006 (níveis 1, 2 e 3)
Domingo, 29 de outubro de 2006 (níveis 2 e 3 - segundo dia de prova).

NÍVEL UNIVERSITÁRIO
Primeira Fase – Sábado, 2 de setembro de 2006
Segunda Fase – Sábado, 28 e Domingo, 29 de outubro de 2006

XII OLIMPÍADA DE MAIO
13 de maio de 2006

XVII OLIMPÍADA DE MATEMÁTICA DO CONE SUL
5 a 11 de maio de 2006
Escobar, Argentina

XLVII OLIMPÍADA INTERNACIONAL DE MATEMÁTICA
8 a 19 de julho de 2006
Ljubljana - Eslovênia.

XIII OLIMPÍADA INTERNACIONAL DE MATEMÁTICA UNIVERSITÁRIA
20 a 26 de julho de 2006
Odessa, Ucrânia

XXI OLIMPÍADA IBEROAMERICANA DE MATEMÁTICA
22 de setembro a 01 de outubro de 2006
Equador
♦♦♦

EUREKA! N°23, 2006

61
Sociedade Brasileira de Matemática

COORDENADORES REGIONAIS
Alberto Hassen Raad (UFJF) Juiz de Fora – MG
Américo López Gálvez (USP) Ribeirão Preto – SP
Amarísio da Silva Araújo (UFV) Viçosa – MG
Andreia Goldani FACOS Osório – RS
Antonio Carlos Nogueira (UFU) Uberlândia – MG
Ali Tahzibi (USP) São Carlos – SP
Benedito Tadeu Vasconcelos Freire (UFRN) Natal – RN
Carlos Alexandre Ribeiro Martins (Univ. Tec. Fed. De Paraná) pato Branco - PR
Carlos Frederico Borges Palmeira (PUC-Rio) Rio de Janeiro – RJ
Claus Haetinger (UNIVATES) Lajeado – RS
Cleonor Crescêncio das Neves (UTAM) Manaus – AM
Cláudio de Lima Vidal (UNESP) S.J. do Rio Preto – SP
Edson Roberto Abe (Colégio Objetivo de Campinas) Campinas – SP
Élio Mega (Colégio Etapa) São Paulo – SP
Éder Luiz Pereira de Andrade (UNESPAR/FECILCAM) Campo Mourão – PR
Eudes Antonio da Costa (Univ. do Tocantins) Arraias – TO
Florêncio Ferreira Guimarães Filho (UFES) Vitória – ES
Ivanilde Fernandes Saad (UC. Dom Bosco) Campo Grande– MS
Janice T. Reichert (UNOCHAPECÓ) Chapecó – SC
João Benício de Melo Neto (UFPI) Teresina – PI
João Francisco Melo Libonati (Grupo Educacional Ideal) Belém – PA
Jorge Costa Duarte Filho (UFPB) João Pessoa - PB
José Cloves Saraiva (UFMA) São Luis – MA
José Luiz Rosas Pinho (UFSC) Florianópolis – SC
José Vieira Alves (UFPB) Campina Grande – PB
José William Costa (Instituto Pueri Domus) Santo André – SP
Krerley Oliveira (UFAL) Maceió – AL
Licio Hernandes Bezerra (UFSC) Florianópolis – SC
Luzinalva Miranda de Amorim (UFBA) Salvador – BA
Mário Rocha Retamoso (UFRG) Rio Grande – RS
Marcelo Rufino de Oliveira (Grupo Educacional Ideal) Belém – PA
Marcelo Mendes (Colégio Farias Brito, Pré-vestibular) Fortaleza – CE
Newman Simões (Cursinho CLQ Objetivo) Piracicaba – SP
Raúl Cintra de Negreiros Ribeiro (Colégio Anglo) Atibaia – SP
Ronaldo Alves Garcia (UFGO) Goiânia – GO
Rogério da Silva Ignácio (Col. Aplic. da UFPE) Recife – PE
Reginaldo de Lima Pereira (Escola Técnica Federal de Roraima) Boa Vista – RR
Reinaldo Gen Ichiro Arakaki (LAC - Laboratório Associado de Computação) SJ dos Campos – SP
Ricardo Amorim (Centro Educacional Logos) Nova Iguaçu – RJ
Sérgio Cláudio Ramos (IM-UFRGS) Porto Alegre – RS
Seme Guevara Neto (UFMG) Belo Horizonte – MG
Tadeu Ferreira Gomes (UEBA) Juazeiro – BA
Tomás Menéndez Rodrigues (U. Federal de Rondônia) Porto Velho – RO
Turíbio José Gomes dos Santos (UFPB) João Pessoa – PB
Valdenberg Araújo da Silva (U. Federal de Sergipe) São Cristovão – SE
Valdeni Soliani Franco (U. Estadual de Maringá) Maringá – PR
Vânia Cristina Silva Rodrigues (U. Metodista de SP) S.B. do Campo – SP
Wagner Pereira Lopes (CEFET – GO) Jataí – GO

EUREKA! N°23, 2006

62
CONTEÚDO

XXVII OLIMPÍADA BRASILEIRA DE MATEMÁTICA 2


Problemas e Soluções da Primeira Fase

XXVII OLIMPÍADA BRASILEIRA DE MATEMÁTICA 14


Problemas e Soluções da Segunda Fase

XXVII OLIMPÍADA BRASILEIRA DE MATEMÁTICA 36


Problemas e Soluções da Terceira Fase

XXVII OLIMPÍADA BRASILEIRA DE MATEMÁTICA 59


Problemas e Soluções da Primeira Fase – Nível Universitário

XXVII OLIMPÍADA BRASILEIRA DE MATEMÁTICA 65


Problemas e Soluções da Segunda Fase – Nível Universitário

XXVII OLIMPÍADA BRASILEIRA DE MATEMÁTICA 73


Premiados

AGENDA OLÍMPICA 77

COORDENADORES REGIONAIS 78
Sociedade Brasileira de Matemática

XXVII OLIMPÍADA BRASILEIRA DE MATEMÁTICA


Problemas e Soluções da Primeira Fase

PROBLEMAS – NÍVEL 1

1. Sabendo-se que 9 174 532 ×13 = 119 268 916 , pode-se concluir que é
divisível por 13 o número:
A) 119 268 903 B) 119 268 907 C) 119 268 911
D) 119 268 913 E) 119 268 923

2. Numa caixa havia 3 meias vermelhas, 2 brancas e 1 preta. Professor Piraldo


retirou 3 meias da caixa. Sabendo-se que nenhuma delas era preta, podemos
afirmar sobre as 3 meias retiradas que:
A) são da mesma cor.
B) são vermelhas.
B) uma é vermelha e duas são brancas.
D) uma é branca e duas são vermelhas.
E) pelo menos uma é vermelha.

3. Diamantino colocou em um recipiente três litros de água e um litro de suco


composto de 20% de polpa e 80% de água. Depois de misturar tudo, que
porcentagem do volume final é polpa?
A) 5% B) 7% C) 8% D) 20% E) 60%

4. Perguntado, Arnaldo diz que 1 bilhão é o mesmo que um milhão de milhões.


Professor Piraldo o corrigiu e disse que 1 bilhão é o mesmo que mil milhões.
Qual é a diferença entre essas duas respostas?
A) 1 000 B) 999 000 C) 1 000 000 D) 999 000 000
E) 999 000 000 000

5. Numa seqüência, cada termo, a partir do terceiro, é a soma dos dois termos
anteriores mais próximos. O segundo termo é igual a 1 e o quinto termo vale
2005. Qual é o sexto termo?
A) 3 002 B) 3 008 C) 3 010 D) 4 002 E) 5 004

EUREKA! N°24, 2006

2
Sociedade Brasileira de Matemática

6. Um galão de mel fornece energia suficiente para uma abelha voar 7 milhões
de quilômetros. Quantas abelhas iguais a ela conseguiriam voar mil
quilômetros se houvesse 10 galões de mel para serem compartilhados entre
elas?
A) 7 000 B) 70 000 C) 700 000 D) 7 000 000
E) 70 000 000

7. Três anos atrás, a população de Pirajussaraí era igual à população que


Tucupira tem hoje. De lá para cá, a população de Pirajussaraí não mudou mas
a população de Tucupira cresceu 50%. Atualmente, as duas cidades somam
9000 habitantes. Há três anos, qual era a soma das duas populações?
A) 3 600 B) 4 500 C) 5 000 D) 6 000 E) 7 500

8. Um agricultor esperava receber cerca de 100 mil reais pela venda de sua safra.
Entretanto, a falta de chuva provocou uma perda da safra avaliada entre
1 1
e do total previsto. Qual dos valores a seguir pode representar a perda do
5 4
agricultor?
A) R$ 21.987,53 B) R$ 34.900,00 C) R$ 44.999,99
D) R$ 51.987,53 E) R$ 60.000,00

9. Devido a um defeito de impressão, um livro de 600 páginas apresenta em


branco todas as páginas cujos números são múltiplos de 3 ou de 4. Quantas
páginas estão impressas?
A) 100 B) 150 C) 250 D) 300 E) 430

10. Seis retângulos idênticos são reunidos para


formar um retângulo maior conforme
indicado na figura. Qual é a área deste 21 cm
retângulo maior?
A) 210 cm2 B) 280 cm2
2
C) 430 cm D) 504 cm2
2
E) 588 cm

11. O relógio do professor Piraldo, embora preciso, é diferente, pois seus


ponteiros se movem no sentido anti-horário. Se você olhar no espelho o
relógio quando ele estiver marcando 2h23min, qual das seguintes imagens
você verá?

EUREKA! N°24, 2006

3
Sociedade Brasileira de Matemática

E E E E E
A) B) C) D) E)

12. Uma placa decorativa consiste num quadrado


1m
de 4 metros de lado, pintada de forma simétrica
com algumas faixas, conforme indicações no 1m
desenho ao lado. Qual é a fração da área da 1m
placa que foi pintada?
1m
1m
1 1 3 6 7 1m
A) B) C) D) E)
2 3 8 13 11

13. Películas de insulfilm são utilizadas em janelas de edifícios e vidros de


veículos para reduzir a radiação solar. As películas são classificadas de
acordo com seu grau de transparência, ou seja, com o percentual da radiação
solar que ela deixa passar. Colocando-se uma película de 70% de
transparência sobre um vidro com 90% de transparência, obtém-se uma
redução de radiação solar igual a :
A) 3% B) 37% C) 40% D) 63% E) 160%

14. Na figura, os dois triângulos são eqüiláteros. Qual é o valor do ângulo x?

75° 65°

A) 30o B) 40o C) 50o D) 60o E) 70o

15. Um serralheiro solda varetas de metal para 10


produzir peças iguais que serão juntadas para 5
5
formar o painel abaixo. O desenho ao lado 10
apresenta as medidas, em centímetros, de uma 5 10
dessas peças. O serralheiro usa exatamente 20

EUREKA! N°24, 2006

4
Sociedade Brasileira de Matemática

metros de vareta para fazer o seu trabalho.

Qual dos desenhos abaixo representa o final do painel?

A) B) C)

D) E)

16. Dentre os números 1, 2, 3, 4, 5, 6, 7, 8, 9 e 10, escolha alguns e coloque-os


nos círculos brancos de tal forma que a soma dos números em dois círculos
vizinhos seja sempre um quadrado perfeito. Atenção: o 2 já foi colocado em um
dos círculos e não é permitido colocar números repetidos; além disso, círculos
separados pelo retângulo preto não são vizinhos.
2

A soma dos números colocados em todos os círculos brancos é:


A) 36 B) 46 C) 47 D) 49 E) 55

17. Figuras com mesma forma representam objetos de mesma massa. Quantos
quadrados são necessários para que a última balança fique em equilíbrio?

A) 7 B) 8 C) 9 D) 10 E) 12

EUREKA! N°24, 2006

5
Sociedade Brasileira de Matemática

18. As 10 cadeiras de uma mesa circular foram numeradas com números


consecutivos de dois algarismos, entre os quais há dois que são quadrados
perfeitos. Carlos sentou-se na cadeira com o maior número e Janaína, sua
namorada, sentou-se na cadeira com o menor número. Qual é a soma dos
números dessas duas cadeiras?
A) 29 B) 36 C) 37 D) 41 E) 64

19. Em um ano, no máximo quantos meses têm cinco domingos?


A) 3 B) 4 C) 5 D) 6 E) 7

20. As nove casas de um tabuleiro 3 × 3 devem ser pintadas de foram que cada
coluna, cada linha e cada uma das duas diagonais não tenham duas casas de
mesma cor. Qual é o menor número de cores necessárias para isso?
A) 3 B) 4 C) 5 D) 6 E) 7

PROBLEMAS – NÍVEL 2

1. Uma loja de sabonetes realiza uma promoção com o anúncio "Compre um e


leve outro pela metade do preço”. Outra promoção que a loja poderia fazer
oferecendo o mesmo desconto percentual é
A) "Leve dois e pague um” B) "Leve três e pague um”
C) "Leve três e pague dois” D) "Leve quatro e pague três”
E) "Leve cinco e pague quatro”

2. Veja o problema No. 13 do Nível 1.


3. Veja o problema No. 10 do Nível 1.
4. Veja o problema No. 4 do Nível 1.
5. Veja o problema No. 9 do Nível 1.

6. Platina é um metal muito raro, mais raro até do que ouro. Sua densidade é
21,45 g/cm3. Suponha que a produção mundial de platina foi de cerca de
110 toneladas em cada um dos últimos 50 anos e desprezível antes disso.
Assinale a alternativa com o objeto cujo volume é mais próximo do volume
de platina produzido no mundo em toda a história.
A) uma caixa de sapatos B) uma piscina
C) um edifício de dez andares D) o monte Pascoal E) a Lua

7. Veja o problema No. 5 do Nível 1.


8. Veja o problema No. 17 do Nível 1.

EUREKA! N°24, 2006

6
Sociedade Brasileira de Matemática

9. Entre treze reais não nulos há mais números positivos do que negativos.
13 × 12
Dentre os = 78 produtos de dois dos treze números, 22 são
2
negativos. Quantos números dentre os treze números dados são negativos?
A) 2 B) 7 C) 8 D) 9 E) 10

10. O desenho ao lado mostra um pedaço de papelão que será dobrado e colado
nas bordas para formar uma caixa retangular. Os ângulos nos cantos do
papelão são todos retos. Qual será o volume da caixa em cm3?
15 cm
40 cm

20 cm
A) 1 500 B) 3 000 C) 4 500 D) 6 000
E) 12 000

11. Sendo a, b e c números reais, pela propriedade distributiva da multiplicação


em relação à adição, é verdade que a × (b + c) = (a × b) + (a × c). A
distributiva da adição em relação à multiplicação a + (b × c) = (a + b) × (a +
c) não é sempre verdadeira, mas ocorre se, e somente se,
A) a = b = c = 1 ou a = 0 B) a = b = c
3
C) A igualdade nunca ocorre D) a + b + c = 1 ou a = 0
E) a = b = c = 0

12. Em certa cidade, acontece um fato interessante. Dez por cento dos
Baianos dizem que são Paulistas e dez por cento dos Paulistas dizem
que são Baianos. Todos os outros Paulistas e Baianos assumem a sua
verdadeira origem. Dentre os Paulistas e Baianos, 20%
dizem que são Paulistas. Que percentual os realmente Paulistas representam
dentre os Paulistas e Baianos?
A) 12,5% B) 18% C) 20% D) 22%
E) 22,5%

13. Veja o problema No. 14 do Nível 1.

EUREKA! N°24, 2006

7
Sociedade Brasileira de Matemática

14. As letras O, B e M representam números inteiros. Se O × B × M = 240, O


× B + M = 46 e O + B × M = 64, quanto vale O + B + M?
A) 19 B) 20 C) 21 D) 24 E) 36

15. Veja o problema No. 15 do Nível 1.


16. Veja o problema No. 19 do Nível 1.

17. Quantos números entre 10 e 13000, quando lidos da esquerda para a direita,
são formados por dígitos consecutivos e em ordem crescente?
Exemplificando, 456 é um desses números, mas 7890 não é:
A) 10 B) 13 C) 18 D) 22 E) 25

18. Um piloto percorreu três trechos de um rali, de extensões 240 km, 300 km e
400 km, respectivamente. As velocidades médias nos três trechos foram 40
km/h, 75 km/h e 80 km/h, mas não necessariamente nessa ordem. Podemos
garantir que o tempo total em horas gasto pelo piloto nos três trechos é:
A) menor ou igual a 13 horas
B) maior ou igual a 13 horas e menor ou igual a 16 horas
C) maior ou igual a 14 horas e menor ou igual a 17 horas
D) maior ou igual a 15 horas e menor ou igual a 18 horas
E) maior ou igual a 18 horas

19. Na figura, todas as circunferências menores têm o mesmo raio r e os centros


das circunferências que tocam a circunferência maior são vértices de um
a
quadrado. Sejam a e b as áreas cinzas indicadas na figura. Então a razão é
b
igual a:

b
a

A) 1 B) 2 C) 1 D) 3 E) 2
2 3 2

EUREKA! N°24, 2006

8
Sociedade Brasileira de Matemática

20. Um professor de Inglês dá aula particular para uma classe de 9 alunos, dos
quais pelo menos um é brasileiro. Se o professor escolher 4 alunos para fazer
uma apresentação, terá no grupo pelo menos dois alunos de mesma
nacionalidade; se escolher 5 alunos, terá no máximo três alunos de mesma
nacionalidade. Quantos brasileiros existem na classe?
A) 1 B) 2 C) 3 D) 4 E) 5

21. Um relógio, com ponteiros de horas, minutos e segundos, faz plim


toda vez que um ponteiro ultrapassa outro no mostrador. O número de
plins registrados em um certo dia, no período entre as 12 horas e 1 segundo
e as 23 horas, 59 minutos e 59 segundos é:
A) 732 B) 1438 C) 1440 D) 1446 E) 1452

22. Na figura, a reta PQ toca em N o círculo que passa por L, M e N. A reta LM


corta a reta PQ em R. Se LM = LN e a medida do ângulo PNL é α, α < 60o,
quanto mede o ângulo LRP?
L

α
P N R Q

A)3α – 180o B)180o – 2α C) 180o – α D) 90o – α /2 E) α

23. Os inteiros positivos x e y satisfazem a equação

x + 12 y − x − 12 y = 1 .
Qual das alternativas apresenta um possível valor de y?
A) 5 B) 6 C) 7 D) 8 E) 9

24. Veja o problema No. 16 do Nível 1.

EUREKA! N°24, 2006

9
Sociedade Brasileira de Matemática

25. Um bloco de dimensões 1 × 2 × 3 é colocado sobre um tabuleiro 8 × 8, como


mostra a figura, com a face X, de dimensões 1 × 2, virada para baixo. Giramos
o bloco em torno de uma de suas arestas de modo que a face Y fique virada
para baixo. Em seguida, giramos novamente o bloco, mas desta vez de modo
que a face Z fique virada para baixo. Giramos o bloco mais três vezes, fazendo
com que as faces X, Y e Z fiquem viradas para baixo, nessa ordem. Quantos
quadradinhos diferentes do tabuleiro estiveram em contato com o bloco?

A) 18 B) 19 C) 20 D) 21 E)22

PROBLEMAS – NÍVEL 3

1. Veja o problema No. 17 do Nível 2.

2. Os pontos L, M e N são pontos médios de arestas do cubo, como mostra a


figura. Quanto mede o ângulo LMN?

L
o o
A) 90 B) 105 C) 120o D) 135o E) 150o

3. Veja o problema No. 22 do Nível 2.


4. Veja o problema No. 14 do Nível 2.

EUREKA! N°24, 2006

10
Sociedade Brasileira de Matemática

5. Esmeralda digitou corretamente um múltiplo de 7 muito grande, com 4010


algarismos. Da esquerda para a direita, os seus algarismos são 2004
algarismos 1, um algarismo n e 2005 algarismos 2. Qual é o valor de n?
A) 3 B) 4 C) 5 D) 6 E) 7

6. Veja o problema No. 23 do Nível 2.


7. Veja o problema No. 25 do Nível 2.
8. Veja o problema No. 1 do Nível 2.
9. Veja o problema No. 6 do Nível 2.

10. A figura mostra um cubo de aresta 1 no qual todas as doze diagonais


de face foram desenhadas. Com isso, criou-se uma rede com 14 vértices (os 8
vértices do cubo e os 6 centros de faces) e 36 arestas (as 12 arestas do cubo e
mais 4 sobre cada uma das 6 faces). Qual é o comprimento do menor caminho
que é formado por arestas da rede e que passa por todos os 14 vértices?

A) 1 + 6 2 B) 4 + 2 2 C) 6 D) 8 + 6 2
E) 12 + 12 2

11. Uma das faces de um poliedro é um hexágono regular. Qual é a quantidade


mínima de arestas que esse poliedro pode ter?
A) 7 B) 9 C) 12 D) 15 E) 18

12. Veja o problema No. 19 do Nível 1.

13. O ponto D pertence ao lado BC do triângulo ABC. Sabendo que AB = AD = 2,


BD = 1 e os ângulos BAD e CAD são congruentes, então a medida do
segmento CD é:
3 4 5 6 7
A) B) C) D) E)
2 3 4 5 6

EUREKA! N°24, 2006

11
Sociedade Brasileira de Matemática

14. Esmeralda adora os números triangulares (ou seja, os números 1, 3, 6, 10,


15, 21, 28…), tanto que mudou de lugar os números 1, 2, 3, …, 11 do relógio
de parede do seu quarto de modo que a soma de cada par de números vizinhos
é um número triangular. Ela deixou o 12 no seu lugar original. Que número
ocupa o lugar que era do 6 no relógio original?
A) 1 B) 4 C) 5 D) 10 E) 11

15. Os termos an de uma seqüência de inteiros positivos satisfazem a relação


an+3 = an+2(an+1 + an) para n = 1, 2, 3…
Se a5 = 35, quanto é a4?
A) 1 B) 3 C) 5 D) 7 E) 9

16. Veja o problema No. 11 do Nível 2.


17. Veja o problema No. 19 do Nível 2.

18. Entre treze reaisnão nulos há mais números positivos do que


13 × 12
negativos. Dentre os = 78 produtos de dois dos treze números, 22
2
são negativos. Quantos números dentre os treze números dados são negativos?
A) 2 B) 7 C) 8 D) 9 E) 10

19. Traçando as quatro retas perpendiculares aos lados de um paralelogramo


não retângulo pelos seus pontos médios, obtém-se uma região do plano
limitada por essas quatro retas. Podemos afirmar que a área dessa região é
igual à área do paralelogramo se um dos ângulos do paralelogramo for igual a:
A) 30o B) 45o C) 60o D) 75o E) 90o

20. O número (2 + 2)3 (3 − 2) 4 + (2 − 2)3 (3 + 2) 4 é:


A) inteiro ímpar B) inteiro par
C) racional não inteiro D) irracional positivo E)
irracional negativo

21. Sejam A = 10(log10 2005) , B = 20053 e C = 2


2
2005
. Então:
A) A < B < C B) A < C < B
C) B < A < C D) B < C < A E) C < A < B

22. Veja o problema No. 18 do Nível 2.

EUREKA! N°24, 2006

12
Sociedade Brasileira de Matemática

23. Dois números inteiros são chamados de primanos quando pertencem a


uma progressão aritmética de números primos com pelo menos três termos.
Por exemplo, os números 41 e 59 são primanos pois pertencem à progressão
aritmética (41; 47; 53; 59) que contém somente números primos.
Assinale a alternativa com dois números que não são primanos.
A) 7 e 11 B) 13 e 53 C) 41 e 131 D) 31 e 43
E) 23 e 41

24. Um relógio, com ponteiros de horas, minutos e segundos, faz plim toda vez
que um ponteiro ultrapassa outro no mostrador. O número de plins registrados
em um certo dia no período entre as 12 horas e 1 segundo e as 23 horas, 59
minutos e 59 segundos é:
A) 732 B) 1438 C) 1440 D) 1446 E) 1452

25. Veja o problema No. 20 do Nível 2.

GABARITO
NÍVEL 1 (5a. e 6a. séries)
1) A 6) B 11) A 16) B
2) E 7) E 12) C 17) D
3) A 8) A 13) B 18) D
4) E 9) D 14) B 19) C
5) B 10) E 15) B 20) C

NÍVEL 2 (7a. e 8a. séries)


1) D 6) B 11) D 16) C 21) Anulada
2) B 7) B 12) A 17) D 22) Anulada
3) E 8) D 13) B 18) Anulada 23) C
4) E 9) A 14) B 19) C 24) B
5) D 10) B 15) B 20) C 25) B

NÍVEL 3 (Ensino Médio)


1) D 6) C 11) C 16) D 21) C
2) C 7) B 12) C 17) C 22) Anulada
3) Anulada 8) D 13) B 18) A 23) B
4) B 9) B 14) C 19) B 24) Anulada
5) B 10) A 15) D 20) B 25) C

EUREKA! N°24, 2006

13
Sociedade Brasileira de Matemática

XXVII OLIMPÍADA BRASILEIRA DE MATEMÁTICA


Problemas e Soluções da Segunda Fase

PROBLEMAS – Nível 1 PARTE A


(Cada problema vale 5 pontos)

01. O tanque do carro de Esmeralda, com capacidade de 60 litros, contém uma


mistura de 20% de álcool e 80% de gasolina ocupando metade de sua capacidade.
Esmeralda pediu para colocar álcool no tanque até que a mistura ficasse com
quantidades iguais de álcool e gasolina. Quantos litros de álcool devem ser
colocados?

02. Na seqüência de números 1, a, 2, b, c, d, ... dizemos que o primeiro termo é 1, o


segundo termo é a, o terceiro termo é 2, o quarto termo é b, e assim por diante.
Sabe-se que esta seqüência tem 2005 termos e que cada termo, a partir do terceiro,
é a média aritmética de todos os termos anteriores. Qual é o último termo dessa
seqüência?

03. Natasha é supersticiosa e, ao numerar as 200 páginas de seu diário, começou do


1 mas pulou todos os números nos quais os algarismos 1 e 3 aparecem juntos, em
qualquer ordem. Por exemplo, os números 31 e 137 não aparecem no diário,
porém 103 aparece.
Qual foi o número que Natasha escreveu na última página do seu diário?

04. Juliana foi escrevendo os números inteiros positivos em quadrados de papelão,


colados lado a lado por fitas adesivas representadas pelos retângulos escuros no
desenho abaixo. Note que cada fila de quadrados tem um quadrado a mais que a
fila de cima. Ela escreveu até o número 105 e parou. Quantos pedaços de fita
adesiva ela usou?

2 3

4 5 6

7 8 9 10

05. Lara tem cubos iguais e quer pintá-los de maneiras diferentes, utilizando as
cores laranja ou azul para colorir cada uma de suas faces.

EUREKA! N°24, 2006

14
Sociedade Brasileira de Matemática

Para que dois cubos não se confundam, não deve ser possível girar um deles de
forma que fique idêntico ao outro. Por exemplo, há uma única maneira de pintar o
cubo com uma face laranja e cinco azuis.
Quantos cubos pintados de modos diferentes ela consegue obter?

06. Um carpinteiro fabrica caixas de madeira abertas na parte de cima, pregando


duas placas retangulares de 600 cm2 cada uma, duas placas retangulares de 1200
cm2 cada uma e uma placa retangular de 800 cm2, conforme representado no
desenho.
Qual é o volume, em litros, da caixa? Note que l litro = 1000 cm3.

PROBLEMAS – Nível 1 PARTE B


(Cada problema vale 10 pontos)

PROBLEMA 1
Quatro peças iguais, em forma de triângulo retângulo, foram dispostas de dois
modos diferentes, como mostram as figuras.
H

I J M N
G
D
C
A
B
E
L K P O
F
Os quadrados ABCD e EFGH têm lados respectivamente iguais a 3 cm e 9 cm.
Calcule as áreas dos quadrados IJKL e MNOP.

EUREKA! N°24, 2006

15
Sociedade Brasileira de Matemática

PROBLEMA 2
Considere três números inteiros positivos consecutivos de três algarismos tais que
o menor é múltiplo de 7, o seguinte é múltiplo de 9 e o maior é múltiplo de 11.
Escreva todas as seqüências de números que satisfazem essas propriedades.

PROBLEMA 3
Cada peça de um jogo de dominó possui duas casas numeradas. Considere as 6
peças formadas apenas pelos números 1, 2 e 3.
(a) De quantos modos é possível colocar todas estas peças alinhadas em
seqüência, de modo que o número da casa da direita de cada peça seja igual ao
número da casa da esquerda da peça imediatamente à direita?
A seguir, mostramos dois exemplos:

(b) Explique por que não é possível fazer o mesmo com todas as 10 peças
formadas apenas pelos números 1, 2, 3 e 4.

PROBLEMAS – Nível 2 PARTE A


(Cada problema vale 4 pontos)

01. Veja o problema No. 3 do Nível 1 Parte A.

02. Quatro peças iguais, em forma de triângulo retângulo, foram dispostas de dois
modos diferentes, como mostram as figuras abaixo.
H

I J M N
G
D
C
A
B
E
L K P O
F

EUREKA! N°24, 2006

16
Sociedade Brasileira de Matemática

Os quadrados ABCD e EFGH têm lados respectivamente iguais a 3 cm e 9 cm.


Determine a medida do lado do quadrado IJKL.

03. Veja o problema No. 4 do Nível 1 parte A.

04. Um terreno quadrangular foi dividido em quatro lotes menores por duas cercas
retas unindo os pontos médios dos lados do terreno. As áreas de três dos lotes
estão indicadas em metros quadrados no mapa a seguir.

250

200 210

Qual é a área do quarto lote, representado pela região escura no mapa?

05. Seja a um número inteiro positivo tal que a é múltiplo de 5, a + 1 é múltiplo de


7, a + 2 é múltiplo de 9 e a + 3 é múltiplo de 11. Determine o menor valor que a
pode assumir.

PROBLEMAS – Nível 2 PARTE B


(Cada problema vale 10 pontos)

PROBLEMA 1
Gabriel resolveu uma prova de matemática com questões de álgebra, geometria e
lógica. Após checar o resultado da prova Gabriel observou que respondeu
corretamente 50% das questões de álgebra, 70% das questões de geometria e 80%
das questões de lógica. Gabriel observou, também, que respondeu corretamente
62% das questões de álgebra e lógica e 74% das questões de geometria e lógica.
Qual a porcentagem de questões corretas da prova de Gabriel?

PROBLEMA 2
O canto de um quadrado de cartolina foi cortado com uma tesoura. A soma dos
comprimentos dos catetos do triângulo recortado é igual ao comprimento do lado
do quadrado. Qual o valor da soma dos ângulos α e β marcados na figura abaixo?

EUREKA! N°24, 2006

17
Sociedade Brasileira de Matemática

27°
β

PROBLEMA 3
(a) Fatore a expressão x 2 − 9 xy + 8 y 2 .
(b) Determine todos os pares de inteiros (x; y) tais que 9 xy − x 2 − 8 y 2 = 2005 .

PROBLEMA 4
Veja o problema No. 3 do Nível 1 Parte B.

PROBLEMAS – Nível 3 PARTE A


(Cada problema vale 4 pontos)

01. Na figura, ABCDE é um pentágono regular e AEF é um triângulo eqüilátero.


Seja P um ponto sobre o segmento BF , no interior de ABCDE, e tal que o ângulo
PEˆ A mede 12º, como mostra a figura abaixo.
F

P
B E

C D

Calcule a medida, em graus, do ângulo PÂC.

EUREKA! N°24, 2006

18
Sociedade Brasileira de Matemática

02. Seja a um número inteiro positivo tal que a é múltiplo de 5, a + 1 é múltiplo de


7, a + 2 é múltiplo de 9 e a + 3 é múltiplo de 11. Determine o menor valor que a
pode assumir.

03. Veja o problema No. 4 do Nível 2 parte A.

04. A função f : \ → \ satisfaz f ( x + f ( y )) = x + f ( f ( y )) para todos os


números reais x e y. Sabendo que f (2) = 8 , calcule f(2005).

05. Você tem que determinar o polinômio p(x) de coeficientes inteiros positivos
fazendo perguntas da forma “Qual é o valor numérico de p(k)?”, sendo k um
inteiro positivo à sua escolha.

Qual é o menor número de perguntas suficiente para garantir que se descubra o


polinômio?

PROBLEMAS – Nível 3 PARTE B


(Cada problema vale 10 pontos)

PROBLEMA 1
Determine todos os pares de inteiros (x; y) tais que 9 xy − x 2 − 8 y 2 = 2005 .

PROBLEMA 2
Um prisma é reto e tem como base um triângulo equilátero. Um plano corta o
prisma mas não corta nenhuma de suas bases, determinando uma secção triangular
de lados a, b e c. Calcule o lado da base do prisma em função de a, b e c.

PROBLEMA 3
No campeonato tumboliano de futebol, cada vitória vale três pontos, cada empate
vale um ponto e cada derrota vale zero ponto. Um resultado é uma vitória, empate
ou derrota. Sabe-se que o Flameiras não sofreu nenhuma derrota e tem 20 pontos,
mas não se sabe quantas partidas esse time jogou. Quantas seqüências ordenadas
de resultados o Flameiras pode ter obtido? Representando vitória por V, empate
por E e derrota por D, duas possibilidades, por exemplo, são (V, E, E, V, E, V, V,
V, E, E) e (E, V, V, V, V, V, E, V).

PROBLEMA 4
Determine o menor valor possível do maior termo de uma progressão aritmética
com todos os seus sete termos a1, a2, a3, a4, a5, a6, a7 primos positivos distintos.

EUREKA! N°24, 2006

19
Sociedade Brasileira de Matemática

Curiosidade: No ano passado, os ex-olímpicos Terence Tao (Austrália, ouro na


IMO 1988) e Ben Green (Reino Unido, prata na IMO 1994) provaram que existem
progressões aritméticas arbitrariamente grandes com todos os termos primos
positivos. Tal questão remonta ao século XVIII, aparecendo nas pesquisas de
Lagrange e Waring.

Soluções Nível 1 – Segunda Fase – Parte A

Problema 01 02 03 04 05 06
Resposta 18 2 214 182 10 24

01. O tanque contém uma mistura de 30 litros, sendo 0, 2 × 30 = 6 litros de álcool


e 30 – 6 = 24 litros de gasolina. Portanto, para que as quantidades de gasolina
e álcool fiquem iguais, devem ser colocados no tanque 24 – 6 = 18 litros de
álcool.

1+ a
02. Como 2 é a média aritmética de 1 e a, podemos escrever = 2 , logo
2
1+ 2 + 3 1+ 3 + 2 + 2
1+ a = 4 ⇔ a = 3; portanto, b= = 2; c= = 2;
3 4
1+ 3 + 2 + 2 + 2
d= = 2 . Esses exemplos sugerem que todos os termos, a
5
partir do terceiro, são iguais a 2. De fato, quando introduzimos em uma
seqüência um termo igual à média de todos os termos da seqüência, a média
da nova seqüência é a mesma que a da seqüência anterior. Assim, o último
termo da seqüência dada é 2.

03. Natasha pulou os números 13, 31, 113, 130,131, 132, ..., 139, num total de 13
números. Portanto, na última página do seu diário escreveu o número 200 + 13
+1 = 214.

04. Olhando para o último número da fila n, vemos que ele é a soma de todos os
números de 1 a n: por exemplo, na fila 4, o último número da fila é 1 + 2 + 3 +
4 = 10. Note que para obter a quantidade de números até uma certa fila, basta
somar o número da fila ao total de números que havia antes dessa fila. Assim,
temos, fila 5 : 15, fila 6: 21, fila 7: 28, fila 8: 36, fila 9: 45, fila 10: 55, fila 11:
66, fila 12: 78, fila 13: 91, fila 14: 105

EUREKA! N°24, 2006

20
Sociedade Brasileira de Matemática

O número de fitas adesivas horizontais entre uma fila n – 1 e uma fila n é igual
a n – 1 e o número de fitas adesivas verticais numa fila n é igual n – 1.
Portanto, até a fila número 14, o número de fitas é

(1 + 2 +  + 13) + (1 + 2 +  + 13 ) = 2 ⋅
13 ⋅14
= 182.
2

05. Todas as faces azuis: uma maneira.


Cinco faces azuis e uma amarela: uma maneira.
Quatro faces azuis e duas amarelas: duas maneiras (duas faces amarelas
opostas ou duas faces amarelas adjacentes).
Três faces azuis e três faces amarelas: duas maneiras (três azuis com um
vértice comum – uma maneira ou três azuis com uma aresta comum duas a
duas – uma maneira)
Duas faces azuis e quatro amarelas: duas maneiras
Uma face azul e cinco amarelas: uma maneira.
Todas as faces amarelas: uma maneira.
Portanto, o número de maneiras diferentes de pintar o cubo é 10.

06. Sejam a, b e c as medidas da caixa, conforme indicado no desenho ao lado.


Segundo o enunciado, podemos escrever ab = 600, ac = 1200 e bc = 800.
Sabemos que o volume da caixa é abc. Utilizando as propriedades das
igualdades e de potências, podemos escrever
( ab ) ⋅ ( ac ) ⋅ (bc ) = 600 ⋅ 1200 ⋅ 800 ⇔ a 2 ⋅ b2 ⋅ c2 = 2 ⋅ 3 ⋅ 102 ⋅ 22 ⋅ 3 ⋅ 102 ⋅ 23 ⋅ 102 ⇔
( abc ) = 26 ⋅ 32 ⋅ 106 ⇔ abc = 26 ⋅ 32 ⋅ 106 ⇔ abc = 23 ⋅ 3 ⋅ 103 = 24 ⋅ 1000 cm3
2

Como 1 litro é igual a 1000 cm3, concluímos que o volume da caixa é de 24 litros.

Soluções Nível 1 – Segunda Fase – Parte B

SOLUÇÃO DO PROBLEMA 1:
1ª maneira: O quadrado IJKL e o quadrado MNOP têm como lados as hipotenusas
dos triângulos retângulos dados, logo têm a mesma área s. Fazendo os dois
quadrados coincidirem, concluímos que o dobro da soma t das áreas dos quatro
triângulos retângulos é a diferença entre as áreas dos quadrados IJKL e EFGH, ou
  ”  ” 
seja, 2t 92 32 2t 72 t 36 . Assim, s = 9 + 36 = 81 – 36 = 45 cm2.

2ª maneira: No quadrado IJKL, seja JC = x. Então IC = ID + DC = JC + DC = x +


3. Então, no quadrado EFGH, temos
  ”  ” 
HN NG x 3 x 9 2 x 6 x 3 . Portanto, a área do quadrado

EUREKA! N°24, 2006

21
Sociedade Brasileira de Matemática

IJKL, igual à soma das áreas dos quatro triângulos retângulos com a área do

quadrado ABCD, vale 4 ¸ 3¸ 32 3


3  36 9  45 e
2
a área do quadrado
MNOP, igual à diferença entre a área do quadrado EFGH e a soma das áreas dos

quatro triângulos retângulos, vale 92  4 ¸ 3¸ 3 3


 81 36  45 cm .2

2
SOLUÇÃO DO PROBLEMA 2:
Seja n = abc múltiplo de 11; então n – 1 deve ser múltiplo de 9 e n – 2 deve ser
múltiplo de 7.
Seja c ≠ 0 :
 
Como abc é múltiplo de 11, podemos ter a b c 0 ou a b c 11 .  
Como abc – 1 é múltiplo de 9, podemos ter
   
a b c 1 9 ou a b c 1 18 . No caso de a + b + c − 1 = 0 ,
teríamos n − 1 = 99 ⇔ n = 100 , que não é múltiplo de 11. Assim,
simultaneamente, somente podemos
a + b + c = 10 2b = 10 b=5
ter (i ) ⇔ ⇔ ou
a+c =b a+c=b a+c=5

a + b + c = 19 2b + 11 = 19 b=4
(ii ) ⇔ ⇔
a + c = b + 11 a + c = b + 11 a + c = 15
No caso (i) existem as seguintes possibilidades para n: 154, 253, 352, 451, que são
múltiplos de 11; para n – 1 temos os números 153, 252, 351, 450 e 549 são
múltiplos de 9. Para os números n – 2 temos 152, 251, 350, 449 e 548, dos quais
apenas 350 é múltiplo de 7.
No caso (ii) existem as seguintes possibilidades para n: 649, 748, 847 e 946, que
são múltiplos de 11; para n – 1 temos os números 648, 747, 846 e 945 são
múltiplos de 9. Para os números n – 2 temos 647, 746, 845 e 944, dos quais
nenhum é múltiplo de 7.
Seja c = 0:
Neste caso, n –1 tem os algarismos a, b –1 e 9. Assim,
a + b − 1 + 9 = 9 ou a + b − 1 + 9 = 18 ou seja, a + b = 1 ou a + b = 10 . Como
       
a b c a b 0 ou a b c a b 11 , concluímos que a = b.
Assim, a = b = 5, o que fornece os números n = 550, n –1 = 549 e n – 2 = 548, que
não é divisível por 7.
Portanto, a única seqüência de três números inteiros consecutivos nas condições
dadas é 350, 351 e 352.

EUREKA! N°24, 2006

22
Sociedade Brasileira de Matemática

SOLUÇÃO DO PROBLEMA 3:
1a maneira:
a) Podemos representar uma seqüência válida como uma seqüência de pares
ordenados. O primeiro exemplo é a seqüência [(1,1),(1,2),(2,2),(2,3),(3,3),(3,1)] e,
a partir dela, podemos criar outras seqüências válidas movendo o par da esquerda
para a direita (ou da direita para a esquerda). Assim, são válidas as seqüências
[(1,2),(2,2),(2,3),(3,3),(3,1),(1,1)], [(2,2),(2,3),(3,3),(3,1),(1,1), (1,2)],etc. num
total de 6 seqüências diferentes. Mudando a posição dos números dos pares
ordenados, podemos criar outras 6 seqüências: [(2,1), (1,1), (1,3),
(3,3),(3,2),(2,2)], [ (1,1), (1,3), (3,3),(3,2),(2,2), (2,1)], etc. Portanto, de acordo
com as regras dadas há 12 modos de colocar as peças em seqüência.

2a maneira:
a) As pontas devem ter o mesmo número, pois eles aparecem um número par de
vezes (se aparecer um número numa ponta e outro na outra, então há pelo
menos dois números que aparecem um número ímpar de vezes, o que não
ocorre). Alguma peça com dois números iguais deve aparecer em uma das
pontas, pois do contrário teríamos três das quatro peças centrais com duas
iguais, vizinhas, o que é impossível). Sendo assim, a seqüência pode ser
representada por XX-XY-YY-YZ-ZZ-ZX, onde para X temos três
possibilidades, para Y temos duas possibilidade e para Z, uma possibilidade,
num total de 3.2.1 = 6 possibilidades para a seqüência que começa com uma
dupla. Se a seqüência terminar com uma dupla, teremos novamente 6
possibilidades. Portanto, há 12 modos de colocar as seis peças em seqüência.

b) Para cada número, existem 4 peças.


Por exemplo, as peças com o número
1 estão desenhadas ao lado. O número
de vezes em que aparece o número 1 é
ímpar, logo a seqüência deveria
começar com 1 e terminar com outro
número ou começar com outro
número e terminar com 1. Neste caso,
os outros dois números deveriam
aparecer um número par de vezes,
pois não estariam na ponta, mas isso
não ocorre: todos os quatro números
aparecem um número ímpar de vezes.

EUREKA! N°24, 2006

23
Sociedade Brasileira de Matemática

Soluções Nível 2 – Segunda Fase – Parte A

Problema 01 02 03 04 05
Resposta 214 -------- 182 240 1735

01. Natasha pulou os números 13, 31, 113, 130,131, 132, ..., 139, num total de 13
números. Portanto, na última página do seu diário escreveu o número 200 + 13
+1 = 214.

02. Sejam x e y o maior e o menor catetos, respectivamente, do triângulo


retângulo. Como o lado do quadrado ABCD mede 3 cm, temos x – y = 3. Por
outro lado, como o lado de EFGH mede 9 cm, temos x + y = 9. Resolvendo o
sistema, encontramos x = 6 e y = 3. Logo, o lado do quadrado IJKL, que é a
hipotenusa do triângulo retângulo, mede 6 2 + 32 = 45 = 3 5 cm.

OUTRA SOLUÇÃO: O quadrado IJKL e o quadrado MNOP têm como lados as


hipotenusas dos triângulos retângulos dados, logo têm a mesma área s.
Fazendo os dois quadrados coincidirem, concluímos que o dobro da soma t
das áreas dos quatro triângulos retângulos é a diferença entre as áreas dos
quadrados IJKL e EFGH, ou seja, 2t = 92 – 32 , o que fornece t = 36.. Assim, s
= 9 + 36 = 81 – 36 = 45 cm2 e o lado do quadrado IJKL é 45 = 3 5 cm.

03. Olhando para o último número da fila n, vemos que ele é a soma de todos os
números de 1 a n: por exemplo, na fila 4, o último número da fila é 1 + 2 + 3 +
4 = 10. Note que para obter a quantidade de números até uma certa fila, basta
somar o número da fila ao total de números que havia antes dessa fila. Assim,
temos, fila 5 : 15, fila 6: 21, fila 7: 28, fila 8: 36, fila 9: 45, fila 10: 55, fila 11:
66, fila 12: 78, fila 13: 91, fila 14: 105
O número de fitas adesivas horizontais entre uma fila n – 1 e uma fila n é igual
a n – 1 e o número de fitas adesivas verticais numa fila n é igual n – 1.
Portanto, até a fila número 14, o número de fitas é

(1 + 2 +  + 13) + (1 + 2 +  + 13) = 2 ⋅
13 ⋅14
= 182.
2

04. Primeira Solução: Unindo os pontos médios de lados consecutivos do


quadrilátero, obtemos segmentos paralelos às suas diagonais e iguais à metade
delas. Portanto, o quadrilátero assim obtido é um paralelogramo. Os

EUREKA! N°24, 2006

24
Sociedade Brasileira de Matemática

segmentos traçados dividem cada um dos quatro lotes em duas partes. Todas
as partes internas têm a mesma área s, igual a 1/4 da área do paralelogramo.
Cada uma das partes externas tem área igual a 1/4 do triângulo determinado
pela diagonal correspondente. Assim, a + c é igual à metade da área do
quadrilátero, o mesmo ocorrendo com b + c. Daí, a + s + c + s = b + s + d + s.
Portanto, a área S desconhecida satisfaz S + 210 = 200 + 250, ou seja, S = 240.

b
a s
s
s
s
d
c

Segunda Solução: Ligando o ponto de interseção das retas que representam as


duas cercas aos vértices, obtemos:

M B
A

O N
Q

D P C

Observemos que, como AQ = QD e as alturas de OAQ e OQD que passam por O


são iguais, as áreas de OAQ e OQD são iguais.
Analogamente, as áreas de OAM e OMB; OBN e ONC; OCP e OPD são iguais.
Logo área OAQ + área OAM + área OCP + área ONC = área OQD + área OMB +
área OPD + área OBN ⇔ área AMOQ + área CNOP = área DPOQ + área BMON
⇔ área AMOQ = 200 + 250 – 210 = 240.

05. Como a + 3 é múltiplo de 11, a + 3 = 11b, b ∈ Z. Sendo a múltiplo de 5,


a − 10b = b − 3 também é, de modo que b – 3 =
5c ⇔ b = 5c + 3 ⇔ a = 11(5c + 3) − 3 = 55c + 30, c ∈ ]+2 O número a + 2 é múltiplo

EUREKA! N°24, 2006

25
Sociedade Brasileira de Matemática

de 9, assim como a + 2 – 54c – 36 = c – 4. Portanto


c − 4 = 9d ⇔ c = 9d + 4 ⇔ a = 55(9d + 4) + 30 = 495d + 250, d ∈ ]. Por fim,
sendo a + 1 múltiplo de 7, então a + 1 – 497d – 245 = a + 1 – 7 (71d + 35) = –
2d + 6 = –2(d – 3) também é, ou seja, d – 3 = 7k ⇔ d = 7 k + 3, k ∈ ] e
a = 495(7 k + 3) + 250 = 3465t + 1735 Logo o menor valor de a é 1735.

Soluções Nível 2 – Segunda Fase – Parte B

SOLUÇÃO DO PROBLEMA 1:

Vamos representar por A, G e L a quantidade de questões de Álgebra, Geometria e


Lógica da Prova e por a, g e l as questões respondidas acertadamente em cada uma
destas áreas. As condições do problema fornecem as seguintes equações:
a g l a+l g+l
= 0,5; = 0,7; = 0,8; = 0,62; = 0,74
A G L A+ L G+L

Substituindo as relações expressas pelas três primeiras equações nas outras duas,
obtemos:

0,5 A + 0,8L 3L
= 0,62 ⇒ 0,12 A = 0,18L ⇒A =
A+L 2
0,7G + 0,8L 3L
= 0,74 ⇒ 0,04G = 0,06 L ⇒ G =
G+L 2

A porcentagem de questões acertadas é:


3 3
0,5. L + 0,7. L + 0,8L
a + g + l 0,5 A + 0,7G + 0,8 L 2 2 2,6
= = = = 0,65 = 65%
A+G + L A+G + L 3 3
L+ L+L 4
2 2
SOLUÇÃO DO PROBLEMA 2:
Vamos denotar por A, B, C e D os vértices do quadrado e por MN o corte efetuado.
Como CM + CN = BC = CD, resulta que BM = CN e DN = MC. Em
conseqüência, os triângulos ADN e DCM são congruentes, o mesmo ocorrendo
com ABM e BCN (em cada caso, os triângulos são retângulos e possuem catetos
iguais). Logo, DÂN = CDM = α e BÂM = CBN = β. Assim, α + β + 27o = 90o e α
+ β = 63o.

EUREKA! N°24, 2006

26
Sociedade Brasileira de Matemática

A B

27o
β
M

α
D N C

SOLUÇÃO DO PROBLEMA 3:
a) x2 – 9xy + 8y2 = x2 – xy – 8xy + 8y2 = x(x – y) – 8y (x – y) = (x – 8y)(x – y).
Alternativamente, as raízes da equação do 2o grau x2 – 9xy + 8y2, de incógnita x,
são y e 8y. Logo, x2 – 9xy + 8y2 fatora em (x – 8y)(x – y).

b) A equação a ser resolvida é (x – y)(8y – x) = 2005 (*)


Observemos que a fatoração em primos de 2005 é 5 ⋅ 401.
Além disso, a soma dos fatores x – y e 8y – x é 7y, que é múltiplo de 7. A soma
dos fatores é ± 406, sendo que somente ± 406 é múltiplo de 7. Assim,

x − y = 5 e 8 y − x = 401 x = 63 e y = 58
ou ou
x − y = 401 e 8 y − x = 5 x = 459 e y = 58
(*) ou ⇔ ou
x − y = −5 e 8 y − x = −401 x = −63 e y = −58
ou ou
x − y = −401 e 8 y − x = −5 x = −459 e y = −58

As soluções são, portanto, (63; 58), (459;58), (–63; –58) e (–459; –58).
OUTRA SOLUÇÃO:
Observando a equação dada como uma equação do segundo grau em x, obtemos
x2 – 9yx + 8y2 + 2005 = 0 (*),
cujo discriminante é
∆ = (9y)2 – 4(8y2 + 2005) = 49y2 – 8020

EUREKA! N°24, 2006

27
Sociedade Brasileira de Matemática

Para que (*) admita soluções inteiras, seu discriminante deve ser um quadrado
perfeito; portanto
49y2 – 8020 = m2 ⇔ (7y – m)(7y + m) = 8020 = 22 ⋅ 5 ⋅ 401 (**)

Podemos supor, sem perda de generalidade, que m ≥ 0, pois se (m; y) é solução de


(**), então
(– m; y) também é. Observando também que 7y – m e 7y + m têm a mesma
paridade e
y – m ≤ 7y + m, então podemos dividir o problema em 4 casos:

• 7y – m = 2 e 7y + m = 4010 ⇔ m = 2004 e y = 2006/7, impossível;


• 7y – m = 10 e 7y + m = 802 ⇔ m = 396 e y = 58;
• 7y – m = – 802 e 7y + m = –10 ⇔ m = 396 e y = –58;
• 7y – m = – 4010 e 7y + m = – 2 ⇔ m = 2004 e y = – 2006/7, impossível.

9 y + m 9 ⋅ 58 + 396
Se y = 58, as soluções em x de (*) são = = 459 e
2 2
9 y − m 9 ⋅ 58 − 396
= = 63 .
2 2
9 y + m 9 ⋅ (−58) + 396
Se y = –58, as soluções em x de (*) são = = −63
2 2
9 y − m 9 ⋅ (−58) − 396
e = = −459 .
2 2

Logo as soluções são (63 ; 58), (459 ; 58), (– 63 ; – 58) e (– 459 ; – 58).

SOLUÇÃO DO PROBLEMA 4:
Veja a solução do problema No. 3 do Nível 1 parte B

Soluções Nível 3 – Segunda Fase – Parte A

Problema 01 02 03 04 05
Resposta 12 1735 240 2011 2

01 . Primeiro observamos que os ângulos internos de um pentágono regular


(5 − 2) ⋅ 180°
medem = 108° .
5

EUREKA! N°24, 2006

28
Sociedade Brasileira de Matemática

Como AF = AE = AB, o triângulo ABF é isósceles com


l l l
l ) = 180° − m( BAF ) = 180° − m(B AE ) − m(E AF ) = 180° − 108° − 60° = 6° .
l ) = m( AFB
m( ABF
2 2 2
l ) = m( AFE
No triângulo PEF, m( EFP l ) − m( AFB
l ) = 60° − 6° = 54° e
l ) = 180° − m( PEF
m( EPF l ) = 180° − 60° − 12° − 54° = 54° , ou seja, o
l ) − m( E FP
triângulo PEF é isósceles com PE = EF. Assim, como EF = AE, o triângulo PEA
l
também é isósceles com m( P lAE ) = m( E PAl ) = 180° − m( PEA) = 180° − 12° = 84°.
2 2
l ) 180° − 108°
180° − m( ABC
Além disso, m(C l AB) = = = 36° e
2 2
m(C l
AE ) = m( B l l ) = 108° − 36° = 72°.
AE ) − m(C AB
Logo, m( P lAC ) = m( P l
AE ) − m(C l
AE ) = 84° − 72° = 12°.

02. PRIMEIRA SOLUÇÃO:


Como a + 3 é múltiplo de 11, a + 3 = 11b, b ∈ ] . Sendo a múltiplo de 5,
a − 10b = b − 3 também é, de modo que b – 3 =
5c ⇔ b = 5c + 3 ⇔ a = 11(5c + 3) − 3 = 55c + 30, c ∈ ]
O número a + 2 é múltiplo de 9, assim como a + 2 – 54c – 36 = c – 4. Portanto
c − 4 = 9d ⇔ c = 9d + 4 ⇔ a = 55(9d + 4) + 30 = 495d + 250, d ∈ ].
Por fim, sendo a + 1 múltiplo de 7, então a + 1 – 497d – 245 = a + 1 – 7 (71d +
35) = –2d + 6 =
–2(d – 3) também é, ou seja, d – 3 = 7k ⇔ d = 7 k + 3, k ∈ ] e
a = 495(7 k + 3) + 250 = 3465t + 1735. Logo o menor valor de a é 1735.

SEGUNDA SOLUÇÃO:
As condições do problema equivalem a dizer que
2a − 5 = 2(a + 1) − 7 = 2(a + 2) − 9 = 2(a + 3) − 11 é múltiplo de 5, 7, 9 e 11, donde
é múltiplo de 5 ⋅ 7 ⋅ 9 ⋅ 11 = 3465. Assim, o menor valor de a é tal que
2a − 5 = 3465 , ou seja, a = 1735.

03. Ligando o ponto de interseção das retas que representam as duas cercas aos
vértices, obtemos:

EUREKA! N°24, 2006

29
Sociedade Brasileira de Matemática

M B
A

O N
Q

D P C

Observemos que, como AQ = QD e as alturas de OAQ e OQD que passam por O


são iguais, as áreas de OAQ e OQD são iguais.
Analogamente, as áreas de OAM e OMB; OBN e ONC; OCP e OPD são iguais.
Logo área OAQ + área OAM + área OCP + área ONC = área OQD + área OMB +
área OPD + área OBN ⇔ área AMOQ + área CNOP = área DPOQ + área BMON
⇔ área AMOQ = 200 + 250 – 210 = 240.

04. Substituindo y por 2 e x por a – f(2) = a – 8, obtemos f(a – f(2) + f(2)) = a – 8 +


f ( f (2)) ⇔ f(a) = a – 8 + f(8).
Substituindo a por 2 na última equação, obtemos f(2) = 2 – 8 + f(8) ⇔ 8 = 2 – 8 +
f(8) ⇔ f(8) = 14. Assim f(a) = a – 8 + 14 = a + 6 e f(2005) = 2005 + 6 = 2011.

05. A idéia da solução é perguntar o valor numérico de p(k) para k suficientemente


grande. Suponha que o polinômio seja: p(x) = an xn + an–1 xn – 1 + ... + a0, com an,
an – 1, ..., a0 inteiros positivos. Se k é um inteiro, tal que: k > M = máx {an, an-1, ...,
a0}, então p(k) é um inteiro, cujos dígitos na representação em base k são
exatamente os coeficientes do polinômio p(x). Podemos então tomar k igual a uma
potência de 10 suficientemente grande.
Logo para resolver o problema, basta perguntarmos o valor de p(1), assim
obtemos uma cota superior para M, e então perguntamos o valor de p(x) para x
igual a uma potência de 10 maior do que p(1). Portanto, o número mínimo de
perguntas que devemos fazer, para garantir que o polinômio p(x) seja determinado
sem sombra de dúvidas, é 2.
Por exemplo: Se p(1) = 29, perguntamos p(100), digamos que p(100) = 100613.
Então o nosso polinômio é p(x) = 10x2 + 6x + 13.

EUREKA! N°24, 2006

30
Sociedade Brasileira de Matemática

Soluções Nível 3 – Segunda Fase – Parte B

SOLUÇÃO DO PROBLEMA 1:
Temos 9 xy − x 2 − 8 y 2 = 2005 ⇔ xy − x 2 + 8 xy − 8 y 2 = 2005
⇔ x( y − x) + 8 y ( x − y ) = 2005 ⇔ ( x − y )(8 y − x) = 2005(*)
Observemos que a fatoração em primos de 2005 é 5 ⋅ 401.
Além disso, a soma dos fatores x – y e 8y – x é 7y, que é múltiplo de 7. Devemos
então escrever 2005 como produto de dois fatores, cuja soma é um múltiplo de 7.
Para isso, os fatores devem ser ± 5 e ± 401. A soma dos fatores é ± 406.

x − y = 5 e 8 y − x = 401 x = 63 e y = 58
ou ou
x − y = 401 e 8 y − x = 5 x = 459 e y = 58
(*) ou ⇔ ou
x − y = −5 e 8 y − x = −401 x = −63 e y = −58
ou ou
x − y = −401 e 8 y − x = −5 x = −459 e y = −58

As soluções são, portanto, (63; 58), (459;58), (–63; –58) e (–459; –58).

OUTRA SOLUÇÃO:
Observando a equação dada como uma equação do segundo grau em x, obtemos
x2 – 9yx + 8y2 + 2005 = 0 (*),
cujo discriminante é
∆ = (9y)2 – 4(8y2 + 2005) = 49y2 – 8020

Para que (*) admita soluções inteiras, seu discriminante deve ser um quadrado
perfeito; portanto
49y2 – 8020 = m2 ⇔ (7y – m)(7y + m) = 8020 = 22 ⋅ 5 ⋅ 401 (**)

Podemos supor, sem perda de generalidade, que m ≥ 0, pois se (m; y) é solução de


(**), então
(– m; y) também é. Observando também que 7y – m e 7y + m têm a mesma
paridade e
7y – m ≤ 7y + m, podemos dividir o problema em 4 casos:

• 7y – m = 2 e 7y + m = 4010 ⇔ m = 2004 e y = 2006/7, impossível;

EUREKA! N°24, 2006

31
Sociedade Brasileira de Matemática

• 7y – m = 10 e 7y + m = 802 ⇔ m = 396 e y = 58;


• 7y – m = – 802 e 7y + m = –10 ⇔ m = 396 e y = –58;
• 7y – m = – 4010 e 7y + m = – 2 ⇔ m = 2004 e y = – 2006/7, impossível.

9 y + m 9 ⋅ 58 + 396
Se y = 58, as soluções em x de (*) são = = 459 e
2 2
9 y − m 9 ⋅ 58 − 396
= = 63 .
2 2
9 y + m 9 ⋅ (−58) + 396
Se y = –58, as soluções em x de (*) são = = −63
2 2
9 y − m 9 ⋅ (−58) − 396
e = = −459 .
2 2

Logo as soluções são (63 ; 58), (459 ; 58), (– 63 ; – 58) e (– 459 ; – 58).

SOLUÇÃO DO PROBLEMA 2:

b 2 − )2

b a a2 − ) 2

) )

Podemos supor, sem perda de generalidade, a configuração acima e, portanto, pelo


teorema de Pitágoras:
A2 + ( b2 − A 2 − a 2 − A 2 ) =c (b − A )(a − A ) = a + b2 − c 2 − A 2 ⇔
2
2
⇔2 2 2 2 2 2

4(b2a2 − b2A2 − a2A2 + A4 ) = A4 + a4 + b4 + c4 − 2a2A2 − 2b2A2 + 2c2A2 + 2a2b2 − 2a2c2 − 2b2c2 ⇔


3A 4 − 2 ( a 2 + b2 + c 2 ) A 2 − ( a 4 + b 4 + c 4 − 2 a 2b 2 − 2 a 2 c 2 − 2b 2 c 2 ) = 0
O discriminante da equação do segundo grau acima, em A 2

∆ =  −2( a + b + c )  + 4 ⋅ 3 ⋅ ( a + b + c − 2a b − 2 a c − 2b2 c 2 ) =
2 2 2 2 4 4 4 2 2 2 2

EUREKA! N°24, 2006

32
Sociedade Brasileira de Matemática

16(a 4 + b4 + c 4 − a 2 b 2 − a 2 c 2 − b 2 c 2 ).
2
+ b2 + c 2 ) ± 16( a 4 + b4 + c 4 − a 2 b2 − a 2 c 2 − b2 c 2 )
Logo A 2 = 2(a ⇔
2 ⋅3
+ b 2 + c 2 ) ± 2 a 4 + b 4 + c 4 − a 2b 2 − a 2 c 2 − b 2 c 2
2
A 2 = (a
3
De fato, observando que A é menor ou igual a min {a, b, c}, temos
A 2 ≤ a + b + c . Portanto
2 2 2

3
( a 2 + b 2 + c 2 ) − 2 a 4 + b 4 + c 4 − a 2b 2 − a 2c 2 − b 2c 2
A= .
3

Observação: Outra maneira de obter as equações é trabalhar em R3, supondo, sem


A A 3 
perda de generalidade, que C = (0, 0, 0), A = (A,0, h) e B =  ,
 2 2 z 
, , com
 
h, z ≥ 0 . Obteríamos, então, as equações
A 2 + h 2 = a 2 , A 2 + z 2 = b 2 e A 2 + ( z − h )2 = c 2 , que nos leva à mesma equação da
solução acima.

Curiosidade: Para o triângulo 3, 4, 5 a medida do lado da projeção que é um


triângulo equilátero é aproximadamente e. O erro é de apenas 0,1%.

SOLUÇÃO DO PROBLEMA 3:
Primeira Solução:
Seja an o número de ordenadas de resultados (sem derrotas), cujo total de pontos
seja n. A pergunta do problema é: quanto vale a20?
Para responder a tal pergunta, iremos determinar uma relação recursiva entre os
termos dessa seqüência. Pensando no último resultado de uma ordenada de
resultados totalizando n pontos, ele pode ser E ou V. Se for E, então retirando o
último termo da ordenada, ela passa a totalizar n – 1 pontos. Se for V, então ao
retiramos o último resultado, a ordenada passa a totalizar n – 3 pontos. Disto,
concluímos que:
an = an – 1 + an – 3.
Calculando os valores da seqüência, temos: a1 = 1, a2 = 1, a3 = 2, a4 = 3, a5 = 4, a6
= 6, a7 = 9,
a8 = 13, a9 = 19, a10 = 28, a11 = 41, a12 = 60, a13 = 88, a14 = 129, a15 = 189, a16 =
277, a17 = 406, a18 = 595, a19 = 872 e a20 = 1278.

EUREKA! N°24, 2006

33
Sociedade Brasileira de Matemática

Logo existem 1278 possíveis seqüências ordenadas de resultados que o Flameiras


pode ter obtido.

Segunda Solução:
Sejam x e y o número de vitórias e empates do Flameiras, respectivamente. Temos
que: x ≥ 0,
y ≥ 0 x + y = 20. Dividindo em 7 possíveis casos:
1º caso: x = 0 e y = 20: Temos exatamente uma seqüência ordenada de resultados.
2º caso: x = 1 e y = 17: Uma seqüência ordenada deverá conter exatamente um
“V” e 17 “E”, portanto o número de seqüências ordenadas é exatamente o número
de anagramas da palavra:
“VEEEEEEEEEEEEEEEEE”, que é: (17 + 1)! / (17! ⋅ 1!) = 18.
3º caso: x = 2 e y = 14: Analogamente ao 2º caso, o número de seqüências
ordenadas é igual ao número de anagramas da palavra “VVEEEEEEEEEEEEEE”,
que é: (14 + 2)! / (14! ⋅ 2!) = 120.
4º caso: x = 3 e y = 11: (11 + 3)! / (11! ⋅ 3!) = 364 seqüências ordenadas.
5º caso: x = 4 e y = 8: (8 + 4)! / (8! ⋅ 4!) = 495 seqüências ordenadas.
6º caso: x = 5 e y = 5: (5 + 5)! / (5! ⋅ 5!) = 252 seqüências ordenadas.
7º caso: x = 6 e y = 2: (2 + 6)! / (2! ⋅ 6!) = 28 seqüências ordenadas.
Temos um total de 1 + 18 + 120 + 364 + 495 + 252 + 28 = 1278 seqüências
ordenadas de resultados possíveis.

SOLUÇÃO DO PROBLEMA 4:
Seja p, p + d, p + 2d, p + 3d, p + 4d, p + 5d, p + 6d a progressão aritmética, que
podemos supor crescente sem perda de generalidade. Então:
1) p ≠ 2.
De fato, se p = 2, p + 2d é par e maior do que 2 e, portanto, não é primo.

2) d é múltiplo de 2.
Caso contrário, como p é ímpar, p + d seria par e maior do que 2.

3) p ≠ 3
Senão, teríamos p + 3d múltiplo de 3, maior do que 3.

4) d é múltiplo de 3
Caso contrário, p + d ou p + 2d seria múltiplo de 3 e maior do que 3.

5) p ≠ 5
Senão teríamos p + 5d múltiplo de 5, maior do que 5.

EUREKA! N°24, 2006

34
Sociedade Brasileira de Matemática

6) d é múltiplo de 5.
Caso contrário, p + d , p + 2d, p + 3d ou p + 4d seria múltiplo de 5, maior do que
5.
De 1), 2), 3), 4), 5) e 6), p ≥ 7 e d é múltiplo de 30.
Se p = 7, observando que 187 = 11 ⋅ 17, então d ≥ 120.
Para d = 120, a seqüência é 7, 127, 247, 367, 487, 607, 727 a qual não serve, pois
247 = 13 ⋅ 19.
Para d = 150, a seqüência é 7, 157, 307, 457, 607, 757, 907 e satisfaz as condições
do problema.
Finalmente, se p ≠ 7, então d é múltiplo de 210 e o menor último termo possível
para tais seqüências é 11 + 6 ⋅ 210 = 1271.
Portanto a resposta é 907.

EUREKA! N°24, 2006

35
Sociedade Brasileira de Matemática

XXVII OLIMPÍADA BRASILEIRA DE MATEMÁTICA


Problemas e Soluções da Terceira Fase
PROBLEMAS – NÍVEL 1

PROBLEMA 1
Esmeraldinho tem alguns cubinhos de madeira de 2 cm de aresta. Ele quer
construir um grande cubo de aresta 10 cm, mas como não tem cubinhos
suficientes, ele cola os cubinhos de 2 cm de aresta de modo a formar apenas as
faces do cubo, que fica oco.
Qual é o número de cubinhos de que ele precisará?

PROBLEMA 2
Num tabuleiro quadrado 5 × 5 , serão colocados três botões idênticos, cada um no
centro de uma casa, determinando um triângulo.
De quantas maneiras podemos colocar os botões formando um triângulo retângulo
com catetos paralelos às bordas do tabuleiro?
Observação: Triângulo retângulo é todo triângulo que possui um ângulo de 90o.
Os lados que formam esse ângulo são chamados de catetos.

PROBLEMA 3
A partir da casa localizada na linha 1 e na coluna 50 de um tabuleiro 100 × 100 ,
serão escritos os números 1, 2, 3, 4, ..., n, como na figura a seguir, que apresenta
uma parte do tabuleiro e mostra como os números deverão ser colocados. O
número n ocupará a casa da linha 1, coluna 100.

Linha 100 ←  
   
Linha 10 ← 46
45 47
44 29 48
43 28 30 49
42 27 16 31 50
41 26 15 17 32 51
40 25 14 7 18 33 52
39 24 13 6 8 19 34 53
38 23 12 5 2 9 20 35 54
Linha 1 ← ... 37 22 11 4 1 3 10 21 36 55 ... n
↓ ↓ ↓
Coluna 1 Coluna 50 Coluna 100

EUREKA! N°24, 2006

36
Sociedade Brasileira de Matemática

a) Determine n.
b) Em qual linha e coluna aparecerá o número 2005?

PROBLEMA 4
No retângulo ABCD, com diagonais AC e BD, os lados AB e BC medem,
respectivamente, 13 cm e 14 cm. Sendo M a intersecção das diagonais,
considere o triângulo BME, tal que
ME = MB e BE = BA, sendo E ≠ A .
a) Calcule a área do triângulo BME.
b) Mostre que o segmento BD é paralelo ao segmento EC.

PROBLEMA 5
Um número inteiro positivo n tem a propriedade P se a soma de seus divisores
positivos é igual a 2n . Por exemplo: 6 tem a propriedade P, pois
1 + 2 + 3 + 6 = 2 ⋅ 6 , porém 10 não tem a propriedade P, pois
1 + 2 + 5 + 10 ≠ 2 ⋅ 10 .
Mostre que nenhum quadrado perfeito tem a propriedade P.
Observação: Um número inteiro positivo é um quadrado perfeito se é igual ao
quadrado de um inteiro. Por exemplo, 1 = 12 , 4 = 2 2 e 9 = 3 2 são quadrados
perfeitos.

PROBLEMAS – NÍVEL 2

PROBLEMA 1
Num tabuleiro quadrado 5 × 5 , serão colocados três botões idênticos, cada um no
centro de uma casa, determinando um triângulo.
De quantas maneiras podemos colocar os botões formando um triângulo retângulo
com catetos paralelos às bordas do tabuleiro?

PROBLEMA 2
No triângulo retângulo ABC, os catetos AB e BC medem, respectivamente, 3 cm e
4 cm. Seja M o ponto médio da hipotenusa AC e seja D um ponto, distinto de
A, tal que BM = MD e AB = BD.
a) Prove que BM é perpendicular a AD.
b) Calcule a área do quadrilátero ABDC.

EUREKA! N°24, 2006

37
Sociedade Brasileira de Matemática

PROBLEMA 3
(a − b)(b − c )(c − a) 1 a b c
Dado que = , qual é o valor de + + ?
(a + b)(b + c)(c + a ) 11 a+b b+c c+a

PROBLEMA 4
Em seu treino diário de natação, Esmeraldinho percorre várias vezes, com um
ritmo constante de braçadas, o trajeto entre dois pontos A e B situados na mesma
margem de um rio. O nado de A para B é a favor da corrente e o nado em sentido
contrário é contra a corrente. Um tronco arrastado pela corrente passa por A no
exato instante em que Esmeraldinho sai de A. Esmeraldinho chega a B e
imediatamente regressa a A. No trajeto de regresso, cruza com o tronco 6 minutos
depois de sair de A. A seguir, Esmeraldinho chega a A e imediatamente sai em
direção a B, alcançando o tronco 5 minutos depois da primeira vez que cruzou
com ele ao ir de B para A. Quantos minutos o tronco leva para ir de A até B?

PROBLEMA 5
Prove que o número 12005 + 22005 + 32005 + ... + 20052005 é múltiplo de 1 + 2 + 3 + ...
+ 2005.

PROBLEMA 6
A medida do ângulo B de um triângulo ABC é 120°. Sejam M um ponto sobre o
lado AC e K um ponto sobre o prolongamento do lado AB, tais que BM é a
bissetriz interna do ângulo ∠ABC e CK é a bissetriz externa correspondente ao
ângulo ∠ACB. O segmento MK intersecta BC no ponto P. Prove que ∠APM =
30°.

PROBLEMAS – NÍVEL 3

PROBLEMA 1:
Um número natural é palíndromo quando se obtém o mesmo número ao
escrevermos os seus dígitos na ordem inversa. Por exemplo, 481184, 131 e 2 são
palíndromos.

⋅111...1
Determine todos os pares de inteiros positivos (m, n) tais que 111...1 
é
m uns n uns
palíndromo.

PROBLEMA 2:
Determine o menor número real C para o qual a desigualdade

EUREKA! N°24, 2006

38
Sociedade Brasileira de Matemática

( ) (
C x12005 + x 22005 + x 32005 + x 42005 + x 52005 ≥ x1 x 2 x3 x 4 x5 x1125 + x 125
2 + x 125
3 + x 125
4 + x 125
5 )
16

é válida para todos os números reais positivos x1, x2, x3, x4, x5.

PROBLEMA 3:
Dizemos que um quadrado está contido em um cubo quando todos os seus pontos
estão nas faces ou no interior do cubo. Determine o maior A > 0 tal que existe um
quadrado de lado A contido num cubo de aresta 1.

PROBLEMA 4:
Temos quatro baterias carregadas, quatro baterias descarregadas e um rádio que
necessita de duas baterias carregadas para funcionar.
Supondo que não sabemos quais baterias estão carregadas e quais estão
descarregadas, determine o menor número de tentativas suficiente para
garantirmos que o rádio funcione. Uma tentativa consiste em colocar duas das
baterias no rádio e verificar se ele, então, funciona.

PROBLEMA 5:
Sejam ABC um triângulo acutângulo e F o seu ponto de Fermat, isto é, o ponto
interior ao triângulo ABC tal que os três ângulos AFˆ B , BFˆ C e CFˆ A medem 120
graus. Para cada um dos triângulos ABF, ACF e BCF trace a sua reta de Euler, ou
seja, a reta que liga o seu circuncentro e o seu baricentro.
Prove que essas três retas concorrem em um ponto.

PROBLEMA 6:
Dados a, c inteiros positivos e b inteiro, prove que existe x inteiro positivo tal que
a x + x ≡ b (mod c ) ,
ou seja, existe x inteiro positivo tal que c é um divisor de ax + x – b.

SOLUÇÕES – NÍVEL 1
PROBLEMA 1: SOLUÇÃO DE DANIEL LUCAS FILGUEIRA (FORTALEZA - CE)
Como cada cubinho tem 2 cm de aresta e o cubo tem 10 cm de aresta, então cabem
5 cubinhos no comprimento, na largura e na altura, então em todo o cubo cabem
125 cubinhos.
Se no lado do cubo coubessem n cubinhos, então o No. de cubinhos da parte de
dentro do cubo seria (n – 2) × (n – 2) × (n – 2). Como no lado do cubo cabem 5
cubinhos, então para sabermos o No. de cubinhos da parte de dentro, basta
substituir o n pelo 5, e ficaria o seguinte:
(5 – 2) × (5 – 2) × (5 – 2) = 3 × 3 × 3 = 27

EUREKA! N°24, 2006

39
Sociedade Brasileira de Matemática

Como em todo o cubo cabem 125 cubinhos, então para deixar o cubo oco, basta
tirar a parte de dentro, que tem 27 cubinhos.
Logo, Esmeraldinho precisaria de 125 – 27 = 98 cubinhos para formar o cubo oco.

PROBLEMA 2: SOLUÇÃO DE RAFAEL SUSSUMU YAMAGUTI MIADA (CAMPINAS - SP)


Se o botão correspondente ao ângulo reto estiver em (1, 1) teremos mais 4 casas
acima e 4 casas à direita, portanto 4 × 4 = 16 possibilidades.
Se ele estiver em (2, 1) teremos mais 4 casas acima, 3 casas à direita e 1 casa à
esquerda o que dá de novo 4 × 4 = 16 possibilidades. Do mesmo modo, vemos
que, para cada casa escolhida para o botão correspondente ao ângulo reto temos 16
possibilidades, e como no campo existem 25 casas, teremos portanto 25 × 16 =
400 possibilidades. Então teremos 400 possibilidades.

PROBLEMA 3: SOLUÇÃO DA BANCA


a) Quando for escrito o número n, todas as casas da diagonal que passa pela (linha
100; coluna 1) e (linha 1; coluna 100) e as que estão abaixo dela estarão
preenchidas e, nesse caso, 100 + 99 + 98 +...+ 3 + 2 + 1 = 5050 números terão
sido escritos no tabuleiro. Como começamos com o 1, o último, n, será 5050.
b) A quantidade de termos nas camadas (1, 2, 3), (4, 5, 6, 7, 8, 9, 10), (11, 12, 13,
14 , 15, 16, 17, 18, 19, 20, 21) aumenta de 4 em 4. Ao final da 31a camada, que
tem 3 + 30 × 4 = 123 números, terão sido escritos 3 + 7 + 11 +...+ 123 = 1953
números, ou seja, o último número dessa camada é 1953.
O termo que ocupa a linha mais alta em cada camada aumenta de 2 em 2 (veja que
a 1a camada sobe até alinha 2, a 2a camada até a linha 4, a 3a sobe até a linha 6, e
assim por diante). Assim, o termo da 31a camada que ocupa a linha mais alta
estará na linha 1 + (122 ÷ 2) = 62.
Por fim, a 32a camada iniciará na linha 1 e coluna 51 – 32 = 19, com o número
1954, e subirá até a linha 62 + 2 = 64. Como 2005 = 1954 + 51, o número 2005
aparecerá na linha 51 + 1 = 52 e coluna 19.

PROBLEMA 4: SOLUÇÃO MATHEUS BARROS DE PAULA (TAUBATÉ - SP)


a) Montando a figura, ela ficará assim:
E
13 α

B α C
α α
13
M
α
A 14 D

EUREKA! N°24, 2006

40
Sociedade Brasileira de Matemática

Os triângulos BEM e BAM são congruentes pelo critério LLL. Como a distância de
M ao lado AB é metade do lado AD, o triângulo BAM possui uma base de 13 cm e
13 × 7
uma altura de 7cm, e sua área é de = 45,5 cm2.
2
HJJG HJJG
b) O triângulo BEM é congruente ao triângulo CMD pelo critério LLL, logo a
distância de E à reta BD é idêntica à distância de C à reta BD , pois as alturas
HJJG HJJG
serão as mesmas.
Assim, EC // BD.

PROBLEMA 5:
BASEADA NA SOLUÇÃO DE GUSTAVO LISBÔA EMPINOTTI (FLORIANÓPOLIS - SC)
Um quadrado perfeito sempre tem um número ímpar de divisores, pois há pares de
números cujo produto é o quadrado perfeito dado e mais um número, a sua raiz.
Se o quadrado perfeito n for ímpar, então todos os seus divisores são ímpares, e
assim será sua soma. Logo a soma não pode ser 2n, pois 2n é par.
Se o quadrado perfeito n for par, então é igual a uma potência de 2 vezes o
quadrado de um ímpar. Os divisores ímpares de n são divisores desse quadrado e,
como já vimos, sua soma (de todos os divisores ímpares de n) é ímpar e logo a
soma de todos os divisores de n também é ímpar, não podendo ser igual a 2n, que
é par.
Portanto nenhum quadrado perfeito tem a propriedade P.

SOLUÇÕES – NÍVEL 2

PROBLEMA 1:
SOLUÇÃO DE HENRIQUE PONDÉ DE OLIVEIRA PINTO (SALVADOR – BA)
Ao invés de considerarmos um tabuleiro quadrado consideremos uma malha
pontilhada onde os pontos são centros de cada quadradinho.
Isto é:

EUREKA! N°24, 2006

41
Sociedade Brasileira de Matemática

Observe que para cada triângulo do enunciado existe um único conjunto dos 2
pontos extremos da hipotenusa. Ou seja, o conjunto de dois pontos extremos da
hipotenusa no triângulo abaixo é {8; 20}.
1 2 3 4 5

6 7 8 9 10

11 12 13 14 15

16 17 18 19 20

21 22 23 24 25

Ou seja os pontos 8 e 20 determinam a hipotenusa do triângulo abaixo. No entanto


para cada dois pontos que determinam a hipotenusa existem outros dois pontos
que podem ser o vértice oposto à hipotenusa. No exemplo acima os pontos 8 e 20
determinam a hipotenusa de dois triângulos retângulos; os triângulos 8; 18; 20 e 8;
10; 20. Basicamente, cada triângulo possui uma única hipotenusa e cada
hipotenusa é comum a dois triângulos retângulos do enunciado. Para provar que
cada hipotenusa pertence a dois triângulos retângulos distintos, vamos pegar uma
hipotenusa genérica de extremos (5K + a) e (5 N + b) (no quadriculado acima)
contando que ambos sejam menores que 25 e tanto a e b sejam maiores ou iguais a
1 e menores ou iguais a 5. Observe que K ≠ N e a ≠ b ou seja, ambos os pontos
estão em linhas e colunas diferentes pois se coincidirem em linhas e ou colunas
não há triângulos como definidos no enunciado com essa hipotenusa. Os dois
triângulos com essa hipotenusa são: (5K + a; 5K + b; 5 N + b) e
(5K + a; 5 N + a; 5N + b ) como para cada triângulo há uma única hipotenusa e
para cada hipotenusa dois triângulos, o número de triângulos é o dobro do de
hipotenusas. Vamos calcular o número de hipotenusas: O primeiro ponto pode
ficar em 25 lugares (todos os pontos) já o segundo pode ficar em 16 (todos que
não estão na mesma linha ou coluna do primeiro). Logo são 25 ⋅ 16 = 400 onde a
ordem das escolhas importa, mas a ordem não importa. Logo, como são duas
escolhas dividi-se por 2! = 2 e teremos 400/2 = 200 hipotenusas ⇒ 400 triângulos.
Logo a resposta é 400 triângulos.
Obs. Para um quadrado n × n a quantidade de triângulos é
n 2 ⋅ ( n − 1)
2

⋅ 2 = n ⋅ (n − 1) se generalizamos esse processo que foi utilizado.


2

EUREKA! N°24, 2006

42
Sociedade Brasileira de Matemática

PROBLEMA 2:
SOLUÇÃO DE RAFAEL TUPYNAMBÁ DUTRA (BELO HORIZONTE - MG)
B
90° – 2α
D

α
α
A M C

Chamemos BCAl = α.
AB 3
Temos senα = =
AC 5
BC 4
cos α = = .
AC 5

a) Como ∆ABC é retângulo em B, sabemos que B pertence à circunferência de


diâmetro AC. Desta forma, AM = MB = MD ⇒ AM = MD. Isso significa que M
pertence à mediatriz de AD . Como AB = BD, B também pertence à mediatriz de
AD . HJJJG
Assim, a mediatriz de AD é a reta BM e, assim, BM ⊥ AD.
c) D pertence à circunferência de centro M e raio BM , já que BM = MD. Assim,
D pertence ao circuncírculo do ∆ABC. Como AB = BD, temos AB = BD e, F F
l = BCA
assim, BCD l ⇒ BCD l = α. No triângulo ABC , B lAC = 90° − α. Logo, como
l = 180° − (90° − α ) = 90° + α. No triângulo BCD, temos
ABDC é inscritível, BDC
l = 180° − (90° + α ) − α = 90° − 2α.
C BD
BD ⋅ BC ⋅ sen (90° − 2α ) 3cm ⋅ 4cm ⋅ cos 2α
Área BCD = = = 6cm 2 ⋅ cos 2α e, como
2 2
3 4
sen α = e cos α = ,
5 5

EUREKA! N°24, 2006

43
Sociedade Brasileira de Matemática

  4 2  3  2   16 − 9  42 2
Área BCD = 6cm (cos α − sen α ) = 6cm    −    = 6cm2 
2 2 2 2
 = cm
 5   5    25  25
 
Usamos aqui o fato de que cos 2α = cos 2 α − sen2 α.
AB ⋅ BC 3cm ⋅ 4cm
Área ABC = = = 6cm 2 ⇒
2 2
 42  192 2
Área ABDC = Área ABC + Área BCD =  6 +  cm2 = cm .
 25  25

PROBLEMA 3:
SOLUÇÃO ADAPTADA DE MARCELO MATHEUS GAUY (SÃO JOSÉ DO RIO PRETO - SP)
Inicialmente, podemos observar que
a+b b+c c+a b c a  a b c 
+ + =3⇔ + + = 3− + + ,
a+b b+c c+a a+b b+c c+a  a +b b +c c +a 
a b c
ou seja, obter o valor de α = + + é equivalente a obter o valor de
a+b b+c c+a
b c a
β= + + .
a+b b+c c +a
Como já sabemos que β = 3 − α, basta agora conseguir outra relação entre α e
β aproveitando a igualdade fornecida no enunciado a qual envolve

γ=
( a − b ) ⋅ (b − c ) ⋅ ( c − a )
( a + b ) ⋅ (b + c ) ⋅ ( c + a )
Após alguns testes, substituindo valores em a, b e c, somos levados a supor que
a−b b−c c−a a−b b−c c−a
α − β = −γ , isto é, + + =− ⋅ ⋅ .
a+b b+c c+a a +b b +c c +a
(Temos acima uma incrível identidade, ela fornece infinitas triplas de reais cuja
soma é igual ao oposto do produto!).
Vamos demonstrar tal identidade:
a − b b − c c − a ( a − b)(b + c)(c + a) + (b − c)( a + b)(c + a) + (c − a)( a + b)(b + c)
+ + = (*)
a+b b+ c c+ a ( a + b)(b + c)(c + a)
Porém,
1) (a − b)(b + c)(c + a ) = (a − b )(b − c + 2c)( a − c + 2c ) =
(a − b)(b − c )(a − c ) + 2c ( a − b)(b − c + a − c + 2c) =
(a − b)(b − c )(a − c ) + 2c ( a − b)(a + b)

EUREKA! N°24, 2006

44
Sociedade Brasileira de Matemática

2) (b − c)( a + b)(c + a) + (c − a)( a + b)(b + c) =


(a + b)((b − c)(c + a ) + (c − a )(b + c)) =
(a + b)(bc + ba − c2 − ca + cb + c2 − ab − ac) =
(a + b)( 2bc − 2ac) = −2c (a − b)( a + b)
Logo, de 1 e 2,

(*) = −
( a − b)(b − c)(c − a) .
( a + b)(b + c)(c + a)
1 1 16
Assim, α −β = − ⇔ α − (3 − α ) = − ⇔ α = .
11 11 11

PROBLEMA 4:
SOLUÇÃO DE HENRIQUE WATANABE (SÃO PAULO - SP)
Vamos supor que a velocidade da corrente do rio é c e a velocidade de
Esmeraldinho é v (sem a corrente). Seja d o comprimento do rio.
Em 6 minutos os dois juntos percorreram 2d.
A velocidade no sentido A → B é (v + c ) e a velocidade no sentido B → A é
( v − c) .
O primeiro encontro foi à 6c do ponto A.
d
De A → B ele leva minutos.
v+c
O segundo encontro ocorreu à 11c do ponto A.
d d 2dv
Para ir e voltar Esmeraldinho leva: t = + =
v + c v − c v + c)(v − c)
(
11c 2dv + 11c (v − c)
e de A até 11c: . Logo = 11
v+c (v + c)(v − c)
⇔ 2dv + 11cv − 11c2 = 11v2 − 11c2 ⇔ v (11v − 11c − 2d ) = 0

De B até o primeiro encontro em 6c t =


d − 6c
. Logo 6 =
( d − 6c)(v + c) + d (v − c)
v −c (v + c)(v − c )
⇔ 6v2 − 6c2 = dv + dc − 6vc − 6c2 + dv − dc
⇔ 2v (3v + 3c − d ) = 0
11v − 11c − 2d = 0 11v − 11c = 2d
Como v ≠ 0: ⇔
3v + 3c − d = 0 6v + 6c = 2d

EUREKA! N°24, 2006

45
Sociedade Brasileira de Matemática

17c
∴11v − 11c = 6v + 6c ⇔ v =
5
17c 51c + 15c 66c
Então: d = 3 ⋅ + 3c = =
5 5 5
d 66c 1 66
O tronco leva: t = = ⋅ = minutos
c 5 c 5
66
O tronco vai de A para B em minutos, ou seja, 13 minutos e 12 segundos.
5

PROBLEMA 5: SOLUÇÃO DE HENRIQUE PONDÉ DE OLIVEIRA PINTO (SALVADOR - BA)


2005 ⋅ ( 2005 + 1)
Observe que 1 + 2 + 3 + ... + 2005 = = 2005 ⋅1003
2
Seja E = 12005 + 22005 + ... + 20052005
Vamos provar que 2005| E
Vendo E módulo 2005 temos:
E ≡12005 + 22006 +32005 +... +10012005 +10022005 + (−1002)2005 + (−1001)2005 +... +(−2)2005 +(−1)2005 + 02005
2005

como a2005 ≡ − ( −a ) ⇒ a2005 + (−a) 2005 ≡ 0 Temos que o n-ésimo termo da


2005

2005 2005

expressão acima irá se anular com o (2005 − n) ° e, portanto,


E ≡ 0(mod2005) ⇒ 2005| E .
Vamos provar agora que 1003 divide E. Vendo E módulo 1003 temos:
E ≡12005 + 22005 + ... +10012005 +10022005 + 02005 + (−1002)2005 + (−1001)2005 + ... + (−2)2005 + (−1)2005
1003

como a2005 ≡ − (−a)2005 ⇒a2005 + (−a)2005 ≡0 ⇒ cada n-ésimo termo irá se anular com o
1003 1003

(2006 − n)° termo e o 1003o. já é múltiplo de 1003 pois é igual a 10032005 temos
que E ≡ 0(mod1003) ⇒ 1003 | E
com o 1003 | E e 2005 | E e (1003,2005 ) = 1 ⇒ 1003 ⋅ 2005 | E ⇒
1 + 2 + ... + 2005 |12005 + 22005 + ... + 20052005 c.q.d.

EUREKA! N°24, 2006

46
Sociedade Brasileira de Matemática

PROBLEMA 6: SOLUÇÃO DE MARÍLIA VALESKA COSTA MEDEIROS (FORTALEZA - CE)


A

α α

60° Q M
60° 30°
B 30° θ C
30° P
30° β β
I
L

Observe a figura acima:


Vamos explicar como chegar até ela!
Sejam:
Q o ponto de intersecção de BM e AP .
l , que encontra o
I a intersecção de CK e a bissetriz externa de ABC
prolongamento de AC em L.
l ≡ KCL
BCK l =β (pelo enunciado)
Vamos provar que A, Q, P, I são colineares.
Usando Menelaus no ∆ABC, temos: M, P e K são colineares ⇒
AK BP CM
⋅ ⋅ = 1 (*)
BK CP AM
Só temos que:
Pelo Teorema da bissetriz interna:
(I) No ∆ABC
CM BC
=
AM AB
Pelo Teorema da bissetriz externa:

EUREKA! N°24, 2006

47
Sociedade Brasileira de Matemática

(II) No ∆ABC
AK AC
=
BK BC
Então, substituindo em (*):
AC BP BC AC BP AC CP
⋅ ⋅ =1⇒ ⋅ =1⇒ =
BC CP AB AB CP AB BP
Com este resultado, observe que AP é bissetriz do ângulo B l
AC .
Como Q é a intersecção de AP e BM , Q pertence a bissetriz do ângulo B l
AC , o
l .
que implica, que no ∆ABM , AQ é bissetriz de B AM
l .
Vamos provar que I pertence a bissetriz de BAM
Pelo teorema da bissetriz interna:
(III) No ∆BCK
CI BC
=
IK BK
Por (II), temos
AK BK IK IK CI
= = ⇒ =
AC BC CI AK AC
Observe que isto é nada mais nada menos do que a propriedade da bissetriz
l = BAM
interna. Logo AI é bissetriz do ângulo K AC l .
Assim, provamos que A, Q, P e I são colineares.
l = θ. No ∆ABC podemos observar que 2α + θ = 60°
Seja ACB
Só que:
l = 120° + 2α (teorema
BCL do ângulo externo). Sabemos, então, que
l = 2β ⇒ β = 60° + α
BCL
Observe que o ângulo AI K = α + θ + β = α + θ + 60° + α = 2α + θ + 60°
Logo, AI K = 120°
Olhando para o quadrilátero BPIK, observe que este é inscritível, pois:
l + PIK
PBK  = 60° + 120° = 180°
l ≡ I PK
Assim, I BK l = 30°
l ≡ I PK
Agora, observe que APM l (o.p.v)
l
Portanto, APM = 30°

EUREKA! N°24, 2006

48
Sociedade Brasileira de Matemática

SOLUÇÕES – NÍVEL 3

PROBLEMA 1:

N
SOLUÇÃO DE WILSON CAMARA MARRIEL (RIO DE JANEIRO - RJ)
10k − 1
Podemos escrever 11...1 como .
k 9
Então:
P  P P  
NN
10m (10n − 1) 10n − 1 n m n n −1 m − n n −1
m
10 − 1 10 − 1
n − 11...100...0 − 11...1 11...1099...988...89
11...1 ⋅ 11...1 = = 9 9 = = ,
m n 9 9 9 9 9
se supusermos m > n (o que não nos faz perder a generalidade!). Esse número tem
m + n – 1 algarismos.

2  
Então vejamos: (fazendo n > 9)

n −1 m− n n −1

11...1099...988...89 9
 !
x
2
k 1234567901
n −2

2  
n−1

onde x é o último termo do quociente antes de usar o 0 e k é o resto de 11...1 por 9.


n −1 m− n n −1

Isto é, acima temos parte da divisão euclidiana de 11...1099...988...89 por 9.

Dividimos nos casos para possíveis valores de k.

1o. caso (k = 0) ⇒ n = 9θ + 1. Continuando a divisão, obtemos

 
m−n n −1

0099...988...89 9
 !
x 011111
1234567901 

n −2
!10987
m− n

Contando da esquerda para a direita 8 é o (m + 1)o. algarismo; como


m + n −1
m +1> então está na metade direita do resultado. Logo, já que o 8 não
2
aparece no lado esquerdo e aparece no direito, temos uma assimetria, ABSURDO!

2o. caso (k = 1): n = 9θ +2

EUREKA! N°24, 2006

49
Sociedade Brasileira de Matemática

 
m −n n −1

1099...988...89 9
19
19
1234567901  !
22098
 !
x 122
%
n −2 m− n

18
088
mesmo caso anterior, ABSURDO!

3o. caso (k = 2): n = 9θ +3


 
m−n n −1

2099...988...89 9
29
29
1234567901 !
32098
 !
x 233
%
n −2 m− n

28
18
088
mesmo caso anterior, ABSURDO!

Ao tentarmos os 9 casos, vemos que um deles tem que ser ≤ 9. Basta conferir se é
verdade!
Vejamos:

n = 1 é verdade!
 11...11
 
n = 2 111...1  é verdade!
122...21 
 
 11...11
 
 111...1 
n=3 é verdade! Obs. É fácil ver que sempre dará certo de 1 à 9.
1111...1 
 
1233...321 

EUREKA! N°24, 2006

50
Sociedade Brasileira de Matemática

 11...111111111 

 $ 
 $
 11111111...111 
n = 9   é verdade! Pois basta ir somando 1 aos
 111111111...11 
1111111111...1 
 
1234567899...987654321 
posteriores.
Então nossos pares são:
(1, k ), ( 2, k ), (3, k ), ( 4, k ), (5, k ), (6, k ), (7, k ), (8, k ) , (9, k )
( m, n) =   tal que k ∈ < *.
(k ,1), (k ,2), (k ,3), (k ,4), (k ,5), (k ,6), (k ,7), (k ,8), (k ,9)

PROBLEMA 2: SOLUÇÃO DE ANDERSON HOSHIKO AIZIRO (SÃO PAULO – SP)


Para x1 = x2 = x3 = x4 = x5 = 1 temos C ⋅ 5 ≥ 516 ⇔ C ≥ 515.
Por bunching (ou desigualdade de Muirhead), temos que
∑x
sym
x x x x ≥∑ x1β1 x2β2 x3β3 x4β4 x5β5
α1 α2 α3 α4 α5
1 2 3 4 5
sym

no caso de termos uma desigualdade homogênea e simétrica (o que é o nosso


caso). Além disso, devemos ter
α1 ≥ β1 , α1 + α2 ≥ β1 + β2 , α1 + α2 + α3 ≥ β1 + β2 + β3 , α1 + α2 + α3 + α4 ≥ β1 + β2 + β3 + β4
e α1 + α2 + α3 + α4 + α5 = β1 + β2 + β3 + β4 +β5 .
A notação ∑xsym
α1 α2 α3 α4 α5
x x x x
1 2 3 4 5 significa que estamos somando todos os 5! = 120

termos da forma xiα1 1 xiα2 2 xiα3 3 xiα4 4 xiα5 5 , sendo (i1 , i2 , i3 , i4 , i5 ) uma permutação de (1, 2, 3,
4, 5). Aqui, impomos também α1 ≥ α2 ≥ α3 ≥ α4 ≥ α5 e o mesmo para os β ' s.
Observando que a desigualdade dada é simétrica, se abrirmos
(
x1 x2 x3 x4 x5 x 125
1 +x
125
2 +x 125
3 +x 125
4 +x 5 )
125 16
obtemos um somatório simétrico no qual o
maior expoente de algum dos termos é 125 ⋅ 16 + 1 < 2005. Podemos, então, aplicar
bunching.
A desigualdade é, então, equivalente a
1 516 α α α α α
∑ x1 ≥ ∑ x1 1 x 2 2 x 3 3 x 4 4 x5 5
2005
C
4! sym 5! sym

EUREKA! N°24, 2006

51
Sociedade Brasileira de Matemática

Explicando: ∑x
sym
2005
1 possui 5! termos no desenvolvimento no qual temos 4!

x12005 , 4!x22005 , 4! x32005 , 4! x42005 e 4!x52005 .

Além disso, temos 516 termos no desenvolvimento de


(x125
1 + x125
2 + x3 + x4 + x
125 125
5 )
125 16
e, para cada conjunto (α1 , α 2 , α 3 , α 4 , α 5 ) temos 5!
516
dos 516 termos e, portanto, há somatórios simétricos.
5!

1 516
Como por bunching ∑x
sym
2005
1 ≥ ∑ x1α1 x2α2 x3α3 x4α4 x5α5 , se C
sym
=
4! 5!
⇔ C = 515 a

desigualdade é válida.

Desse modo, concluímos que o menor número real C para o qual a desigualdade
( ) ( )
16
C x12005 + x22005 + x32005 + x42005 + x52005 ≥ x1 x2 x3 x4 x5 x1125 + x125
2 + x3 + x4 + x5
125 125 125
é válida
para todos os números reais positivos x1 , x2 , x3 , x4 , x5 é C = 515.

PROBLEMA 3: SOLUÇÃO DA BANCA

Primeiro, mostremos que podemos supor, sem perda de generalidade, que os


centros do cubo (que doravante chamaremos C) e do quadrado coincidem.
Suponha que os centros não coincidam. Considere os três planos distintos, cada
um deles paralelo a duas faces do cubo, que passam pelo centro do quadrado. Os
três planos determinam no cubo oito paralelepípedos; considere o de menores
dimensões (ou seja, algum que tem todas as dimensões menores ou iguais a 1/2).
Seja a a maior dimensão desse paralelepípedo. Então construa um cubo C0 de lado
2a com centro no centro do quadrado e faces paralelas às faces do cubo do
problema. O quadrado está contido nesse cubo, pois cada plano ou contém o
quadrado ou o corta em dois polígonos congruentes e simétricos em relação ao
centro do quadrado. Translade o cubo C0, incluindo o quadrado, que está em seu
interior, de modo que o centro de C0 coincida com o centro do cubo. Agora os
centros do quadrado e de C coincidem, e dado que 2a ≤ 1, C0 está contido em C, e
o quadrado ainda está contido no cubo C.

A figura a seguir mostra que A ≥ 3 42 .


EUREKA! N°24, 2006

52
Sociedade Brasileira de Matemática

2 2
1 1 3 2 3
Note que AB = CD =   + 12 +   = , AD = BC = ⋅ 2 e
4  4 4 4
2
1 3 3 2
AC = BD =   + 12 + 12 = = ⋅ 2.
 
2 2 4

1 3
4 A 4 3
4

D 1
4

1
B
4
3
4 C
3 1
4 4

Vamos provar que, na verdade, A = 3 42 . Suponha que exista um quadrado de lado


A > 3 42 . Podemos supor que o centro do quadrado coincide com o centro do
cubo. Seja S uma esfera com centro no centro O de C e que passa pelos quatro
vértices do quadrado, ou seja, de raio A
2 / 2 > 3/ 4. A figura a seguir mostra as
secções de S no cubo C. Numeramos as oito regiões contidas na superfície da
esfera e no interior do cubo com números romanos

EUREKA! N°24, 2006

53
Sociedade Brasileira de Matemática

I IV

III VIII

VI
V

VII II

Agora, vamos tentar localizar os vértices do quadrado de lado A > 3 42 em S.


Note que cada um dos quatro vértices deve pertencer a uma das regiões de I a
VIII. Suponhamos, sem perda de generalidade, que dois vértices opostos do
quadrado estão contidos nas regiões I e, conseqüentemente, II, já que vértices
opostos do quadrado são diametralmente opostos em S.
Considere o paralelepípedo de menores dimensões que contém as regiões I e,
digamos, III. Sejam x, x e 1 as suas dimensões. Vamos provar que dois pontos no
3 2
interior desse paralelepípedo estão a uma distância menor que .
4

I
x

III

EUREKA! N°24, 2006

54
Sociedade Brasileira de Matemática

Primeiro, considere uma face do cubo e sua interseção com a esfera. A partir da
figura a seguir, podemos concluir que o raio da esfera é
2 2
1 1 1
y2 +   +   = y2 + . Como o raio da esfera é maior que
 2  2 2
2
3 2 2 3 2 1 3 1 1 1
⋅ = , y + >   ⇔ y > . Conseqüentemente, x = − y < .
4 2 4 2 4 4 2 4
y y

1
r
2

1 3 2
A diagonal do paralelepípedo mede x 2 + x 2 + 12 < 2 ⋅ +1 = e, portanto,
16 4
dois vértices do quadrado não podem estar contidos em I e III. Como um dos
vértices pertence a I, não pode existir vértice do quadrado em III e, analogamente,
em IV e V. Da mesma forma, lembrando que um dos vértices do quadrado está em
II, não pode haver vértices do quadrado em VI, VII e VIII. Mas então não
sobraram regiões para os outros dois vértices do quadrado, absurdo.

Deste modo, o maior lado de um quadrado contido no cubo unitário é = A


3 2
4
.

PROBLEMA 4: SOLUÇÃO DE GABRIEL TAVARES BUJOKAS (SÃO PAULO – SP)


Separe as pilhas em 3 grupos, 2 de 3 pilhas e 1 de 2 pilhas, e faça todas as
 3   3   2
possibilidades dentro dos grupos. Faremos   +   +   = 7 tentativas.
 2  2  2
Como são 3 grupos e 4 baterias boas, um grupo terá 2 baterias boas, e em algum
momento serão testadas juntas.

EUREKA! N°24, 2006

55
Sociedade Brasileira de Matemática

Suponha que seja possível com apenas 6 tentativas. Considere o grafo com 8
vértices representando as 8 baterias e as arestas {i, j} representando que a bateria i
8 
e a j não foram testadas juntas. O grafo tem   − 6 = 22 arestas.
 2
Por Turán, ele tem um K4. De fato, o máximo de arestas sem K4 é
8   3   3  2
  −   −   −   = 21 .
 2  2   2  2 
Se as 4 baterias carregadas forem as respectivas aos vértices do K4, temos que
nunca duas delas foram testadas juntas, logo o algoritmo com 6 tentativas falha
nesta situação.
Resposta: Com 7 tentativas é possível.

Observação: o grafo completo de n vértices (também conhecido como n-clique),


notado por Kn, é um grafo em que todo par de vértices é ligado. O teorema de
Túran diz que, fixado o número de vértices n, o grafo que não contém um Kr com
a maior quantidade possível de arestas é o grafo (r – 1)-partido completo com
classes de vértices as mais distribuídas possíveis. Esse grafo é obtido da seguinte
forma: divida os vértices em r – 1 conjuntos, de modo que a diferença entre as
quantidades de vértices nos conjuntos seja no máximo 1; em seguida, ligue todo
par de vértices que não estão no mesmo conjunto.

Na aplicação do problema 4, Gabriel dividiu os 8 vértices em três grupos com 3, 3


 8  3  3  2
e 2 vértices. Daí a quantidade de arestas máxima ser   −   −   −   = 21 .
 2  2  2  2

Aliás, a resolução de Gabriel também resolve uma generalização do problema, no


qual há m baterias funcionando e n baterias descarregadas. Tente pensar nesse
problema!

EUREKA! N°24, 2006

56
Sociedade Brasileira de Matemática

PROBLEMA 5: SOLUÇÃO DE LEANDRO FARIAS MAIA (FORTALEZA – CE)


Construa um triângulo equilátero BXC, externo a ABC. O ponto O1 é o
circuncentro do ∆BFC e também de ∆BXC .
G é o baricentro do ∆ABC .
AG XO1
Temos: =2= ⇒ O1G // XF. Mas: O3 A = O3 F e O2 A = O2 F
GM O1M
⇒ AF ⊥ O2 O3 ⇒ O1G ⊥ O2 O3 .
Analogamente temos: O2G ⊥ O1O3 e O3G ⊥ O1O2 ⇒ G é o ortocentro do ∆O1O2O3 .
A

O3

O2
F G

G1

B M C

FG1 AG
Sendo G1 o baricentro do ∆FBC temos: =2= ⇒ G1G // AF
G1M GM
⇒ G1G ⊥ O2O3 ⇒ como G é o ortocentro de ∆O1O2O3 , então G1 está na altura
relativa a O2O3 . Portanto, O1G1 , O2 G 2 e O3G3 são concorrentes em G (seu
ortocentro).

PROBLEMA 6: SOLUÇÃO DE GABRIEL TAVARES BUJOKAS (SÃO PAULO – SP)


Lema: Seja p primo e a, b, r, α inteiros, a > 0, α > 0. Então existe x > 0 tal que
x ≡ r (mod p − 1)
a x + x ≡ b (mod p α )
Demonstração: Indução em α. Para α = 1 (Base):
x ≡ r (mod p − 1)
Se p | a , então obtemos , que tem solução pelo teorema chinês
x ≡ b (mod p )
dos restos.
Se p /| a; x = ( p − 1)(ar − b + r + l ⋅ c) + r; com l tal que x > 0.
De fato, x ≡ r (mod p − 1) e

EUREKA! N°24, 2006

57
Sociedade Brasileira de Matemática

(a −b+r +l ⋅c)
a x + x ≡ ar ( a p−1 ) + ( p −1)( ar − b + r + l ⋅ c ) + r ≡ ar − ar + b − r − l ⋅ c + r ≡b.
r

p p p

α
Passo: Da hipótese, existe x0 tal que a x0
+ x0 = b + t ⋅ p e x0 ≡ r (mod p − 1) .
Tomando x1 = x0 + ( p − 1) p ⋅ t : x1 ≡ x 0 ≡ r e α
p −1 p −1

a x1 + x1 ≡α +1 a x0 ⋅ a ( p −1) p
p
( α
) +x
t
0 + ( p − 1) p α ⋅ t ≡α +1 a x0 + x 0 − p α ⋅ t = b
p

Isso termina a demonstração do lema.

Seja c = p1α1 ⋅ p2α2 ⋅ ... ⋅ pnαn ; p1 < p2 < ... < pn a fatoração em primos de c.
Vamos mostrar por indução em n que ∃x tal que a x + x ≡ b (mod c n ) , onde
ci = p1α1 ⋅ p2α2 ⋅ ... ⋅ piαi .
(
Base: n = 1: a x + x ≡ b mod p1α1 . Caso especial do lema. )
Passo: Se ∃xi tal que a xi + xi ≡ b ( mod ci ) , pelo lema existe x tal que
x ≡ x i (mod pi +1 − 1)
α
a x + x ≡ b (mod p i +i1+1 )
Pelo teorema chinês dos restos ∃xi +1 tal que
xi +1 ≡ x i (mod mmc( pi +1 − 1; ci ; ϕ(c i )) )
α
xi +1 ≡ x (mod p i +i1+1 )
(
Observe que mdc p iα+i1+1 , mmc( p i +1 − 1; ci ; ϕ(c i )) = 1 pois pi + 1 é maior que todos )
os fatores primos de ci e, conseqüentemente, de ϕ(c i ) .
Logo xi +1 ≡ x i ≡ x (mod p i +1 − 1) e xi +1 ≡ x (mod p iα+i1+1 ) ⇒
α α
xi +1 ≡ x (mod ( p i +1 − 1) pi +i1+1 ) , ou seja, xi +1 = x + p i +i1+1 ( pi +1 − 1)l '
x+ p
αi +1
( pi+1 −1)⋅l '
Assim, a xi+1 + xi +1 ≡
α+
a i+1 +x ≡
α+
ax + x ≡
α+
be
pi+i1 1 pi+i1 1 pi+i1 1

a xi+1 + xi +1 ≡ a i ( i ) + xi + ci ⋅ k ≡ a xi + xi ≡ b onde ϕ(ci ) ⋅ l = k ⋅ ci = xi +1 − xi .


x +ϕ c ⋅l
ci ci ci

Portanto a xi+1 + xi +1 α≡+ b ⇔ a xi+1 + xi +1 ≡ b.


pi+i1 1 ⋅ci ci+1

Em especial, a xn + xn ≡ b ⇔ a xn + xn ≡ b .
cn c

EUREKA! N°24, 2006

58
Sociedade Brasileira de Matemática

XXVII OLIMPÍADA BRASILEIRA DE MATEMÁTICA


Primeira Fase – Nível Universitário

PROBLEMA 1
Seja f : →\ \ definida por f ( x) = x 3 + ax 2 + bx + c , sendo a, b e c inteiros.
Sabe-se que f(1) = f(–1) = 0.
As retas tangentes ao gráfico de f nos pontos A = (–1; 0) e B = (1; 0) cortam-se em
C. Calcule a área do triângulo ABC, sabendo-se que tal área é inteira.

PROBLEMA 2
π 4
Calcule a integral: ∫
0
ln(1 + tgx)dx

PROBLEMA 3
Determine o maior valor possível para o volume de um tetraedro inscrito no
x2 y2 z 2
elipsóide de equação + + =1.
9 16 25

PROBLEMA 4
Sejam A e B matrizes reais quadradas de mesma dimensão tais que, para todo
inteiro positivo k, ( A + B ) k = A k + B k . Prove que se A é invertível então B é a
matriz nula.

PROBLEMA 5
Determine todos os valores reais de α para os quais a matriz A = (a ij ) n×n definida
por aij = cos((i − 1) ⋅ jα), para 1 ≤ i, j ≤ n, tem determinante nulo.

PROBLEMA 6
Prove que existem pelo menos 2005 potências 27-ésimas distintas (isto é, números
da forma n27, com n inteiro positivo), todas com exatamente 2005 algarismos, tais
que qualquer uma pode ser obtida de qualquer outra a partir de uma permutação de
seus algarismos.

EUREKA! N°24, 2006

59
Sociedade Brasileira de Matemática

XXVII Olimpíada Brasileira de Matemática


GABARITO Primeira Fase

Soluções Nível Universitário

SOLUÇÃO DO PROBLEMA 1:
Pelo enunciado, temos
f(x) = (x – 1)(x + 1)(x – c) = x3 – cx2 – x + c, f'(x) = 3x2 – 2cx – 1, donde f '( –1) =
2(1 + c) e
f '(1) = 2(1 – c).
Assim, as equações das retas AC e BC são, respectivamente,
y = 2(1 + c)(x + 1) e y = 2(1– c)(x – 1).
Igualando para obter as coordenadas de C, temos
(1 + c)(x + 1) = (1– c)(x – 1)
x = –1/c
y = 2(c + 1)(c – 1)/c
Assim a área pedida é S = |2(c + 1)(c – 1)/c|, pois o triângulo ABC tem base AB =
2 e altura y = 2(c + 1)( c − 1) / c .
Como c e a área S são inteiros, temos c | 2(c + 1)(c – 1).
Mas (c + 1) e (c – 1) são primos com c, donde c | 2.
Assim c = ±1 ou c = ±2.
Os casos c = ±1 dão S = 0, um triângulo degenerado.
Os casos c = ±2 dão S = 3.
O valor da área é, portanto, igual a 3.

SOLUÇÃO DO PROBLEMA 2:
 senx   senx + cos x 
Temos ln(1 + tgx ) = ln 1 +  = ln  .
 cos x   cos x 
 π π π 2
Entretanto, sen  x +  = senx cos + sen cos x = (senx + cos x), e logo
 4 4 4 2
 π
senx + cos x = 2 sen  x + .
 4
  π
 2 sen  x + 4   ln 2 π
 = 
Assim, ln(1 + tgx ) = ln  + ln sen  x +  − ln cos x, donde
 cos x  2  4
 
 

EUREKA! N°24, 2006

60
Sociedade Brasileira de Matemática

π π ⋅ ln 2 π 4  π π


4
ln (1 + tgx ) dx = + ∫ ln sen  x +  dx −∫ 4 ln cos x dx.
0 8 0
 4 0

 π π  π  π 
Agora, sen  x +  = cos  −  x +   = cos  − x  , donde
 4   2  4   4 
π
 π π
π  π

∫0 + = ∫ − = ∫
4 4 4
ln sen  x  dx ln cos  x  dx ln cos y dy
 4 0
4  0

π π π ⋅ ln 2
(fazendo a substituição y = − x ), donde ∫ 4 ln (1 + tgx ) dx = .
4 0 8

SOLUÇÃO ALTERNATIVA DO PROBLEMA 2:


π
π
Seja I = ∫ 4 ln(1 + tgx )dx; faça u = − x, du = −dx.
0 4
π π
 π   1 − tgu   2 
Então I = ∫π ln  1 + tg  − u   (− du ) = ∫ 4 ln 1 + du = ∫ 4 ln 
0
  du
4  4  0
 1 + tgu  0
 1 + tgu 
π π
π ⋅ ln 2 π ⋅ ln 2 π ⋅ ln 2
= ∫ 4 ln 2du − ∫ 4 ln(1 + tgu ) du = − I ⇒ 2I = ⇒I = .
0 0 4 4 8

SOLUÇÃO DO PROBLEMA 3:
LEMA: O tetraedro de maior volume inscrito na esfera unitária x2 + y2 + z2 = 1 é o
tetraedro regular. Seus vértices podem ser tomados como (± c, ± c, ± c) com um
número par de sinais – onde c = 3 3 . Sua aresta é a = 2 6 3 e seu volume é V
= 8 3 27.
O elipsóide do problema é obtido a partir da esfera unitária aplicando a
transformação linear
 3 0 0
 
T = diag (3,4,5) =  0 4 0  . Tetraedros inscritos na esfera são levados em
0 0 5
 
tetraedros inscritos no elipsóide multiplicando o volume por |det (T)| = 60. Assim
um tetraedro de volume máximo é (±3c, ±4c, ± 5c), com um número par de sinais
–, de volume160 3 9.

Demonstração do LEMA:
A única parte não trivial é a de provar que um tetraedro de volume máximo deve
ser regular. Vamos provar que todas as faces de um tetraedro de volume máximo

EUREKA! N°24, 2006

61
Sociedade Brasileira de Matemática

são triângulos equiláteros. Para isso vamos fixar o vértice V0 e variar os vértices
V1, V2, V3 restritos ao círculo definido por estes pontos. Ora, com este tipo de
mudança a altura do tetraedro não muda, donde maximizamos o volume
maximizando a área do triângulo V1, V2, V3. É um fato sabido e de fácil
demonstração que o triângulo de área máxima inscrito em um círculo dado é o
equilátero.

SOLUÇÃO DO PROBLEMA 4:
Temos, de A2 + B 2 = ( A + B) 2 = ( A + B)( A + B) = A2 + AB + BA + B 2 que AB + BA
= 0.
Agora,
A3 + B3 = ( A + B)3 = ( A + B)( A + B) 2 = ( A + B)( A2 + B 2 ) = A3 + AB 2 + BA2 + B3 ,
donde
AB 2 + BA2 = 0. Como BA = − AB, 0 = AB 2 + BA2 = AB 2 − ABA = A( B 2 − BA) e,
como A é invertível, B 2 − BA = 0.
Temos, também
A3 + B 3 = ( A + B )3 = ( A + B ) 2 ( A + B) = ( A2 + B 2 )( A + B ) = A3 + A2 B + B 2 A + B 3 ,
donde A2 B + B 2 A = 0. Como B 2 = BA, segue que A2 B + BA2 = 0, e, como
BA = − AB, obtemos 0 = A2 B + BA2 = A2 B − ABA = A( AB − BA), donde AB = BA,
pois A é invertível.
Finalmente, de AB + BA = 0, segue que 2 AB = 0, donde, como A é invertível,
devemos ter B = 0.

SOLUÇÃO DO PROBLEMA 5:
Sabemos que para todo natural k existe um polinômio
Pk (t ) = ck ,k t + ... + ck ,1t + ck ,0
k

de grau k tal que cos(ka ) = Pk (cos a ) para todo a.


Por exemplo, P0 = 1, P1 = t , P2 = 2t 2 − 1
Temos portanto
aij = Pi −1 (cos( jα)) = ∑ ci −1, k (cos( jα ))k
0 ≤ k <i

Podemos agora, para i > 1, subtrair ci −1,0 vezes a primeira linha da i-ésima linha
sem alterar o determinante obtendo assim que, para i > 1,
a ij = ∑ ci −1, k ⋅ (cos( jα))k .
0< k < i

EUREKA! N°24, 2006

62
Sociedade Brasileira de Matemática

Para i > 2, subtraímos ci −1,1 vezes a segunda linha da i-ésima linha, ainda sem
alterar o determinante.
Repetindo o processo, vemos que det(A) = det(B) onde bij = ci −1,i −1 (cos( j α))i −1
Assim, a menos dos fatores ci −1,i −1 , B é uma matriz de Vandermonde, e seu
determinante é igual a
  ( j + j )α  
∏ ci −1,i −1 ⋅ ∏ (cos( j1 ⋅α) − (cos( j0 ⋅ α)) =(−2)n(n−1)/ 2 ⋅ ∏ci −1,i −1 ⋅ ∏ sen  1 0   ⋅
i≤n j0 < j1 i≤ n 1< j   2 
  ( j1 − j0 ) α  
 sen   .
  2 
Assim det(A) = 0 se e somente se existem 1 ≤ j0 < j1 ≤ n tais que
 ( j + j )α   ( j − j )α 
sen  1 0  = 0 ou sen  1 0  .
 2   2 
Mas isto ocorre se e somente se ( j1 ± j0 )α = 2k π , k inteiro.
Ou seja, det(A) = 0 se e somente se α = 2k π /( j1 ± j0 ) para alguma escolha de
1 ≤ j0 < j1 ≤ n
Falta verificar quais os valores possíveis de j1 ± j0 .
Para n ≤ 1 o problema é trivial (det(A) = 1), donde não há nenhum α com essa
propriedade.
Para n = 2, os únicos valores possíveis de j1 ± j0 são 1 e 3,
2kπ
donde α deve ser da forma , com k inteiro
3
Para n > 2, j1 ± j0 assume todos os valores inteiros positivos m até 2n − 1 , donde
α deve ser da forma 2k π m , com m ≤ 2n − 1 e k inteiro.

Observação:
Temos ainda ck ,k = 2k −1 para k > 1
donde ∏c i −1,i −1
= 2( n −1)( n − 2) / 2 e
i≤ n

det( A) = (−1) n ( n −1) / 2 ⋅ 2( n −1) .


2

  ( j1 + j0 ) α   ( j1 − j0 ) α  
∏  sen   ⋅ sen    ⋅
j1 < j0   2   2 

EUREKA! N°24, 2006

63
Sociedade Brasileira de Matemática

Demonstração da afirmação cos(ka) = Pk(cos(a)) [Não vale pontos extras]:


Temos cos((k + 1)a ) + cos((k − 1)a ) = 2cos(ka ) ⋅ cos a, donde, assumindo que o
resultado vale para k – 1 e para k, cos((k + 1)a ) = 2cos a ⋅ Pk (cos a) − Pk −1 (cos a ), o
que prova o resultado fazendo Pk +1 ( x) = 2 xPk ( x) − Pk −1 ( x), para k ≥ 1, com
P0 ( x) = 1 e P1 ( x ) = x. Note que, sabendo que o coeficiente líder ck ,k de Pk ( x ) é
2k −1 , segue imediatamente que o coeficiente líder ck +1, k +1 de Pk +1 ( x) é
2 ⋅ 2k −1 = 2k = 2( k +1)−1.

06. Vamos estimar inicialmente a quantidade de tipos de números de 2005


algarismos a menos de uma permutação de seus algarismos. Um tal tipo de
números está determinado pelas quantidades x0 , x1 ,..., x9 de algarismos iguais a 0,
1, …, 9, respectivamente; devemos ter x0 + x1 + ... + x9 = 2005.
Assim, a quantidade desses tipos de números é, no máximo, o número de soluções
de x0 + x1 + ... + x9 = 2005, com xi ≥ 0 para 0 ≤ i ≤ 9, que é
 2005 + 9   2014 
=  < 2014 < (10 ) = 10 .
9 4 9 36

 9   9 
Por outro lado n 27 tem 2005 algarismos se, e somente se,
2004 2005 2005 2004
102004 ≤ n 27 < 10 2005 ⇔ 10 ≤ n < 10 , donde há pelo menos 10
27 27 27
− 10 27

27
naturais n tais que n tem 2005 algarismos. Entretanto,
2005 2004 2004
 1  2004
 ln10  2004
ln10 ln10
10 27 −10 27 =10 27 1027 −1 =10 27 e 27 −1 >10 27 ⋅ >1074 ⋅ >1072 ≥ 2005⋅1036 ,
    27 27
donde, pelo princípio da casa dos pombos, há pelo menos 2005 naturais n tais que
n 27 tem 2005 algarismos e esses números n 27 são todos do mesmo tipo (seus
algarismos são os mesmos a menos de uma permutação).
1
Nota: É possível estimar 10 27 − 1 sem usar a desigualdade e x − 1 ≥ x. Por
exemplo:
1
1 1
 1 16 1 1 1 1
10 > 10 =  10 2  > 316 > (1,7 )8 > (1,3) 4 > (1,12) 2 > 1,05, donde
27 32

 
1
1
10 27 − 1 > 0,05 > (que foi o que usamos).
100

EUREKA! N°24, 2006

64
Sociedade Brasileira de Matemática

XXVII OLIMPÍADA BRASILEIRA DE MATEMÁTICA


SEGUNDA FASE – NÍVEL UNIVERSITÁRIO
PRIMEIRO DIA

PROBLEMA 1:
Determine, em função de n, o número de possíveis valores para o determinante de
A, dado que A é uma matriz real n × n tal que A3 − A2 − 3 A + 2I = 0 , onde I
representa a matriz identidade n × n, e 0 representa a matriz nula n × n.

PROBLEMA 2:
Sejam f e g funções contínuas distintas de [0, 1] em (0, + ∞) tais que
n +1
1 f ( x)
∫0 ∫0 ∫0 g ( x)n dx.
1 1
f ( x ) dx = g ( x ) dx. Para n ≥ 0 , seja y n =

Prove que ( yn )n ≥ 0 é uma seqüência crescente e divergente.

PROBLEMA 3:

\
n
Sejam v1 , v2 ,..., vn vetores em 2
tais que vi ≤ 1 para 1 ≤ i ≤ n e ∑v
i =1
i = 0. Prove

k
que existe uma permutação σ de {1, 2,…, n} tal que ∑v
j =1
σ( j ) ≤ 5 para qualquer

k com 1 ≤ k ≤ n.
Obs. Se v = ( x, y ) ∈ \ ,v =
2
x 2 + y 2 denota a norma euclidiana de v.

SEGUNDO DIA

PROBLEMA 4:
1
Considere a seqüência (an )n ≥1 dada por a1 = 1, an +1 = an + 2005
, ∀n ≥ 1.
a n

1
Prove que a série ∑ n⋅a
n =1
converge.
n

EUREKA! N°24, 2006

65
Sociedade Brasileira de Matemática

PROBLEMA 5:

1
∑n = ∫ x − x dx.
1
Prove que n 0
n =1

PROBLEMA 6:
Prove que para quaisquer naturais 0 ≤ i1 < i2 < ... < ik e 0 ≤ j1 < j2 < ... < jk , a matriz
 i + js  (ir + js )!
A = (ars )1≤ r ,s ≤ k dada por ars =  r = (1 ≤ r , s ≤ k ) é invertível.
 ir  ir ! js !

SOLUÇÕES

PROBLEMA 1:
SOLUÇÃO DE MOYSES AFONSO ASSAD COHEN (RIO DE JANEIRO – RJ)
Podemos escrever A3 − A2 − 3 A + 2I = 0 como ( A − 2 I ) ( A2 + A − I ) = 0.
Podemos concluir então que os possíveis autovalores de A são
−1 − 5 −1 + 5
λ1 = 2, λ 2 = e λ3 = . Seja di a multiplicidade do autovalor λ i .
2 2
Temos que o determinante de A é da forma: 2d1 ⋅ λ 2d2 ⋅ λ 3d3 , onde d1 + d2 + d3 = n.
Se provarmos que não existem duas combinações ( d1 , d2 , d3 ) e ( e1 , e2 , e3 ) tais que
2d1 ⋅ λ 2d2 ⋅ λ 3d3 = 2e1 ⋅ λ 2e2 ⋅ λ3e3 , então o número de possíveis valores para o
determinante, será o número de maneiras de escolher d1 , d 2 , d3 satisfazendo di > 0
e d1 + d2 + d3 = n. E o número de escolher esses di ' s

é
( n + 2 )! =
n! (n + 2)(n + 1) n 2 + 3n + 2
= .
n!2! n! 2 2
Vamos provar então que 2d1 ⋅ λ 2d2 ⋅ λ d33 = 2e1 ⋅ λ e22 ⋅ λ e33 se, e somente se, d1 = e1 ,
d 2 = e2 e d3 = e3 .
2d1 ⋅ λ 2d2 ⋅ λ 3d3 = 2e1 ⋅ λ 2e2 ⋅ λ e33 , d1 + d 2 + d3 = e1 + e2 + e3 = n
⇔ 2d1 ⋅ λ 2d2 ⋅ λ1n −d1 −d2 = 2e1 ⋅ λ 2e2 ⋅ λ 3n −e1 −e2 ,
Podemos usar também que λ 2 ⋅ λ3 = −1,
 −1 + 5   −1 − 5  +1 + 5 − 5 − 5 −4
(pois  ⋅ = = = −1.)
 2   2  4 4

EUREKA! N°24, 2006

66
Sociedade Brasileira de Matemática

d e
 1  2 n − d1 −d2  1  2 n −e1 −e2
⇔ 2 ⋅  −  ⋅ λ3
d1
= 2 ⋅  −  ⋅ λ3
e1

 λ3   λ3 
⇔ 2d1 ⋅ ( −1) 2 ⋅ λ3n− d1 − 2 d2 = 2e1 ⋅ ( −1) 2 ⋅ λ3n − e1 − 2 e2 ⇔ 2d1 − e1 = ( −1) e2 − d2 ⋅ λ(3d1 −e1 )+ 2(d 2 − e2 ) , e
d e

( )
I
como não podemos escrever −1 + 5 , onde I é um inteiro maior que 1, como
uma potência de 2, temos que a igualdade é verdadeira se, e somente se, os
expoentes são zero. Ou seja
d1 − e1 = 0 ⇒ d1 = e1
 e portanto d3 = e3 .
e2 − d 2 = 0 ⇒ d 2 = e2

PROBLEMA 2:
SOLUÇÃO DE HUMBERTO SILVA NAVES (S.J. DOS CAMPOS – SP)
Sejam:
1
f ( x ) n +1
un = yn +1 − yn = ∫ [ f ( x) − g ( x)] dx
0 g ( x) n +1
e
1
f ( x) n +1
n+ 2 (
vn = un+1 − un = ∫ f ( x) − g ( x ) ) dx
2

0
(
g x )

Sabemos que vn > 0, ∀n ∈ , pois


g ( x)
`
f ( x ) n +1
n+ 2 (
f ( x ) − g ( x ) ) > 0,
2

Para x ∈ f g
[0;1] e como e são contínuas, temos:
f ( x ) n +1
`
1

n +2 (
vn = ∫ f ( x ) − g ( x ) ) dx > 0, ∀n ∈ , logo (un )n ≥0 é crescente.
2

0
g ( x)
Vamos agora provar que u0 > 0 :
1 1 1

∫ [ f ( x) − g ( x )]dx =
f ( x)
u0 = u0 − ∫ ( f ( x ) − g ( x ) ) dx = ∫ ( f ( x ) − g ( x ) ) dx −
0 0
g ( x) 0

( f ( x) − g ( x)) `
1 2

=∫ dx > 0 ⇒ un ≥ uo > 0, ∀n ∈ , pois (un )u >0 é crescente.


0
g ( x)
Portanto claramente yn ≥ yo + n ⋅ uo , ∀n ≥ 0, e ( yn )n ≥0 é crescente e divergente.

EUREKA! N°24, 2006

67
Sociedade Brasileira de Matemática

PROBLEMA 3: SOLUÇÃO DA BANCA


Vamos usar a solução da versão em R do problema:
m
se α1 , α 2 ,..., α m são números reais com αi ≤ 1, ∀i ≤ m e ∑α
i=1
i = 0 então existe

uma permutação τ de {1, 2,…,m} tal que α τ( i ) tenha sinal contrário a ∑α


j <i
τ( j )

(i. e., com α τ (i ) ⋅ ∑ α τ ( j ) ≤ 0).


j <i

Seja I = {1, 2,…,n}. Escolhemos um conjunto X ⊂ I tal que ∑v


i∈X
i seja o maior

possível. Podemos supor (rodando os eixos coordenados, se necessário) que ∑v


i∈X
i

é um vetor da forma (0, y), com y > 0. Sejam π1 , π2 : 2 → \ \


as projeções na
primeira e na segunda coordenadas, respectivamente. Usando a versão em do \
problema para reordenar os elementos de X e de I\X, podemos supor que X = {1,
 r   k +1 
2,…,k} para um certo k > n, π1  ∑ vi  ≤ 1, ∀r ≤ k e π1  ∑ v j  ≤ 1, ∀s ≤ n − k .
 i =1   j = k +1 
Note agora que π2 (vi ) ≥ 0, ∀i ∈ X (e π2 ( vi ) ≤ 0, ∀i ∈ I \ X ) , pois, se j ∈ X e

π2 (v j ) < 0, teríamos ∑
i∈X \{ j}
vi > ∑v
i∈X
i , absurdo. Podemos então obter (como na

versão em \do problema) uma permutação σ de I que intercala os índices em X


e em I\X , preservando a ordem dos índices em X e em I\X, de modo que
 m 
π 2  ∑ vσ (i )  ≤ 1, ∀m ≤ n. Como os índices em X e em I\X aparecem em ordem,
 i =1 
 
 m   
teremos π1  ∑ vσ (i )  ≤ π1  ∑ vσ( i )  + π1  ∑ vσ (i )  ≤ 1 + 1 = 2, ∀m ≤ n, e logo
 i =1   i ≤i ≤ m   i ≤i ≤ m 
 σ (i )∈I \ X 
 
m

∑∀
i =1
σ (i ) ≤ 22 + 12 = 5, ∀m ≤ n.

EUREKA! N°24, 2006

68
Sociedade Brasileira de Matemática

PROBLEMA 4:
SOLUÇÃO DE DIÊGO VELOSO UCHÔA (RIO DE JANEIRO – RJ)
Primeiro recorde a expansão binomial de Newton:
n
n
( a + b ) = ∑   a n−k ⋅ b k .
n

k =0  k 

Agora usando isso verifique as seguintes contas:


2006
 1   2006  2005 1
( a + b ) =  an +  = ( an ) +  an ⋅ + ... > (an ) + 2006.
2006 2006 2006

( an )  ( an )
2005 2005
  1 
(pois an é sempre positivo)
Logo:
a22006 > a12006 + 2006
a32006 > a22006 + 2006
# #
( an +1 ) > ( an ) + 2006.
2006 2006

( an+1 ) > ( a1 ) + ( 2006 ) ⋅ n > (n + 1)


2006 2006

⇒ an+1 > ( n + 1) ⇒ ( n + 1) an+1 > ( n + 1) 2006 ⇒


1/ 2006 1+ 1

∞ ∞
1 1 1 1
<
( n + 1) an+1 ( n + 1)1+ 1
2006
⇒ ∑
n =1 n ⋅ an
< ∑
n =1 ( n )
1+ 1
2006

∞ ∞
1 1
Como ∑
n =1 n
α
com α > 1 converge segue que ∑ na
n =1
também converge.
n

PROBLEMA 5:
SOLUÇÃO DE LUÍS DANIEL BARBOSA COELHO (RIO DE JANEIRO - RJ)
( − x ) ⋅ ( ln x ) ( − x ) ⋅ ( ln x )
n n n n
∞ 1 ∞
=∑ → ∫ x ⋅ dx = ∫ ∑
1
−x ln( x − x ) ( − x ) ⋅ln x −x
x =e =e ⋅ dx
n=0 n! 0 0
n=0 n!

yn
devido ao tipo de convergência monótona da série de potências e y = ∑ ,
n =0 n!
podemos fazer troca da integral com o somatório, obtendo:
( −1)
n

∫ x − x ⋅ dx = ∑ ∫ x n ⋅ ( ln x ) ⋅ dx
1 1 n
0
n=0 n! 0

denotemos por I n( p ) a seguinte integral: I n( p ) = ∫ x p (ln x ) ⋅ dx, para todo p inteiro


1 n

não negativo.

EUREKA! N°24, 2006

69
Sociedade Brasileira de Matemática

n x p +1
Integrando por partes: u = (ln x ) → du = ⋅ ( ln x ) ⋅ dx; dv = x p ⋅ dx → v =
n n −1

x p +1
x p +1 n (1) p +1
⋅ (ln x ) ⋅ dx 10 − ∫ ⋅ x p ⋅ (ln x ) ⋅ dx = ⋅ (ln1) −
1 n −1
→ I n( p ) =
n n

p +1 0 ( p + 1) p +1
x p +1 n n
− lim+ ⋅ (ln x ) − ⋅ I n( −p1) = − ⋅ I n( −p1) , pois lim+ x p +1 ⋅ (ln x ) = 0.
n n

x→ 0 p +1 p +1 p +1 x → 0

 −n  ( p )
I n( p ) =   I n −1
 p +1
#
 −1  ( p )
I1( p ) =   I0
 p +1
(−1) n ⋅ n! ( p ) (−1)n ⋅ n! 1 p (−1) n ⋅ n!
I n( p ) = ⋅ I0 = ⋅ ∫ x ⋅ dx = , tomando p = n temos:
( p + 1) n
( p + 1) n 0 ( p + 1)n+1

(−1)n  (−1)n ⋅ n !  ∞ 1 ∞
1
∫ x − x ⋅ dx = ∑ ∑ ∑
1
⋅ n +1 
= = .
 ( n + 1)  n =0 ( n + 1)
n +1 n
0
n=0 n! n =1 n

PROBLEMA 6:
SOLUÇÃO DE HUMBERTO SILVA NAVES (S.J. DOS CAMPOS – SP)
Considere os seguintes pontos no reticulado: An = ( −in ;0) e Bn = ( 0; jn ) , onde
1≤ n ≤ k .
y

B4
B3
B2
B1

A4 A3 A2 A1 x

Um caminho ligando An com Bm é um caminho no reticulado partindo de An e


chegando em Bm que só pode ir para cima ou para a direita. Exemplos:

EUREKA! N°24, 2006

70
Sociedade Brasileira de Matemática

y y

Bm Bm

2 2

–4 –3 –2 –1 x
–4 –3 –2 –1 x
An
An
Não é caminho!
É caminho!

i + j 
Um fato interessante é que existem  n m  caminhos ligando An com Bm .
 in 
Uma rota é uma coleção de k caminhos (de cada An parte exatamente um caminho
e em cada Bm chega exatamente um caminho) e dizemos que uma rota é bem feita
se os caminhos não se cruzam em nenhum ponto do reticulado.
Vamos provar que o número de rotas bem feitas é igual a det(A).
Pela definição de determinante, temos:
k
 in + jσ ( n ) 
det( A) = ∑ (−1) I ( σ ) ⋅ ∏  .
σ∈S k n =1  in 
k
i + j 
Mas (−1) I ( σ ) ∏  n σ( n )  é exatamente o número de rotas ligando An com Bσ( n) ,
n =1  in 
para 1 ≤ n ≤ k , multiplicando pela paridade I(σ) da permutação σ.
Vamos provar que as rotas mal feitas se cancelam neste somatório:
y
Considere uma rota mal feita R e
4 B4 seja Y o ponto de intersecção com
X
3 B3 maior coordenada x (se existir mais
2 B2 de um, tome Y cuja coordenada y
1 B1
seja a menor possível).
Y

–5 –4 –3 –2 –1 x
A4 A3 A2 A1

σ = {(4 ; 4) ; (3 ; 3) ; (2 ; 1) ; (2 ; 2)}

EUREKA! N°24, 2006

71
Sociedade Brasileira de Matemática

Vamos trocar os respectivos caminhos que se cruzam em Y (se existir mais de 2


caminhos que se cruzam em Y, troque os caminhos que começam em An ' s de mais
maior coordenada x).
y Assim obtemos uma nova rota mal
feita, só que com a paridade de
4 B4 permutação correspondente trocada

X
3 B3 (−1) I ( σ) = − (−1) I ( σ)
Como a relação entre rotas mal
2 B2
feitas que acabamos de definir é
Y
1 B1 bijetora, então provamos que as
rotas mal feitas não contribuem para
–5 –4 –3 –2 –1 x o somatório, e como uma rota bem
A4 A3 A2 A1 feita possui a identidade como
permutação associada, provamos
 = {(4 ; 4) ; (3 ; 3) ; (2 ; 1) ; (2 ; 2)}
σ que: det (A) = número de rotas bem
feitas.
Como 0 ≤ i1 < i2 < ... < ik e 0 ≤ j1 < j2 < ... < jk , certamente o número de rotas bem
feitas é diferente de 0, pois
é uma rota bem feita.
y
B4
B3

B2
Y
B1

A4 A3 A2 A1 x

Errata: O item b) do problema No. 112 (Eureka! 23, p.60) foi proposto
equivocadamente: ao contrário do que pensávamos, parece não haver soluções
simples para ele.
Gostaríamos portanto de manter apenas o item a) do problema proposto No. 112.
Pedimos desculpas pelos inconvenientes causados.

EUREKA! N°24, 2006

72
Sociedade Brasileira de Matemática

XXVII OLIMPÍADA BRASILEIRA DE MATEMÁTICA


Resultado – Nível 1 (5a. e 6a. Séries)
NOME CIDADE - ESTADO PRÊMIO
Matheus Barros de Paula Taubaté – SP Ouro
Guilherme Vieira Melo Fortaleza – CE Ouro
Luis Musso Gualandi Vitória – ES Ouro
Rafael Dias da Fonsêca Arapiraca – AL Ouro
Rodrigo Rolim Mendes de Alencar Fortaleza – CE Ouro
Gustavo Lisbôa Empinotti Florianópolis – SC Prata
Iuri Rezende Souza Mineiros – GO Prata
Eduardo Cintra Simões Recife – PE Prata
João Mendes Vasconcelos Fortaleza – CE Prata
Gabriel Lima Guimarães Vitória – ES Prata
Jonas Rocha Lima Amaro Fortaleza – CE Prata
Bruno Cesar da Silva Guedes Recife – PE Prata
Kelve Torres Henrique Recife – PE Prata
Igor Rosiello Zenker São Paulo – SP Prata
Daniel Lucas Filgueira Fortaleza – CE Prata
Cleiton Vilela Figueiredo da Silva Recife – PE Prata
Andreza Lais da Silva Nascimento Recife – PE Prata
Ivan Seki Hellmeister São Paulo – SP Prata
Matheus Henrique Botelho Cordeiro Curitiba – PR Bronze
Breno Rocha Comin Leme – SP Bronze
Henrique Lopes de Mello Rio de Janeiro – RJ Bronze
Leonardo Henrique Caldeira Pires Ferrari Rio de Janeiro – RJ Bronze
Leonardo Gonçalves Fischer Fraiburgo – SC Bronze
Francisco Vagner Dantas Leite Filho Fortaleza – CE Bronze
Elder Massahiro Yoshida São Paulo – SP Bronze
Alex Lordello Magario Salvador – BA Bronze
Rafael Sussumu Yamaguti Miada Campinas – SP Bronze
Deborah Barbosa Alves São Paulo – SP Bronze
Diogo Silva Freitas Recife – PE Bronze
Matheus Barbosa Santos de Miranda João Pessoa – PB Bronze
Augusto dos Santos Morgan S. J. do Rio Pardo – SP Bronze
André Bandeira Pinheiro Fortaleza – CE Bronze
Fernando Fonseca Andrade Oliveira Belo Horizonte – MG Bronze
Eduardo F. Freire Neto Salvador – BA Menção Honrosa
Wellington Biing Jung Lee São Paulo – SP Menção Honrosa
Mac'simus Alec'sander de Castro Duarte Fortaleza – CE Menção Honrosa
Alessandro Macêdo de Araújo Fortaleza – CE Menção Honrosa
Camila Miraglia Ribeiro Curitiba – PR Menção Honrosa
Douglas Barbosa da Fonsêca Arapiraca – AL Menção Honrosa
Pedro Montebello Milani São Paulo – SP Menção Honrosa
Gabriel Ricardo Loecsh Siebiger Sobradinho – DF Menção Honrosa
Tiago Yparraguirre Viégas Niterói – RJ Menção Honrosa
Rafael de Melo Andrade Boituba – SP Menção Honrosa
Priscilla Lie Sato Yamaguti São Paulo – SP Menção Honrosa
João Lucas Camelo Sá Fortaleza – CE Menção Honrosa
Franciely Juliani Chutti Itajobi – SP Menção Honrosa
Frederico Nascimento Dutra Porto Alegre – RS Menção Honrosa
Isaac Jerônimo Moreira Fortaleza – CE Menção Honrosa
Anne Wang São Paulo – SP Menção Honrosa
Rafael Farias Marinheiro Recife – PE Menção Honrosa
Rafael Fernandes Paixão Rio de Janeiro – RJ Menção Honrosa
Filipe da Gama Martin Nanuque – MG Menção Honrosa
Humberto Lopes Tabatinga Neto Teresina – PI Menção Honrosa
Gregory Cosac Daher Rio de Janeiro – RJ Menção Honrosa
Renata Aimi Fukuda São Paulo – SP Menção Honrosa
Fabrício Catani de Freitas Sorocaba – SP Menção Honrosa
Bruno Giordano Leite Recife – PE Menção Honrosa
Victor Gonçalves Elias João Pessoa – PB Menção Honrosa
Letícia Duchein Ferreira Londrina – PR Menção Honrosa
Larissa Firakawa Tamashiro Jundiaí – SP Menção Honrosa
Douglas Souza Alves Junior Vassouras – RJ Menção Honrosa
Lara Guimarães Fernandes Peres Rio de Janeiro – RJ Menção Honrosa

EUREKA! N°24, 2006

73
Sociedade Brasileira de Matemática

Nível 2 (7a. e 8a. Séries)


NOME CIDADE – ESTADO PRÊMIO
Henrique Pondé de Oliveira Pinto Salvador – BA Ouro
Marcelo Matheus Gary S. J. do Rio Preto – SP Ouro
Rafael Tupynambá Dutra Belo Horizonte – MG Ouro
Pollyanna Stéfani Borges Freitas Fortaleza – CE Ouro
Iuri Souza Ramos Barbosa Brasília – DF Ouro
Guilherme Philippe Figueiredo Fortaleza – CE Ouro
Marcelo Tadeu de Oliveira Sá Barreiras – BA Prata
Marlen Lincoln da Silva Fortaleza – CE Prata
Henrique Watanabe São Paulo – SP Prata
Grazielly Muniz da Cunha Fortaleza – CE Prata
James Jun Hong São Paulo – SP Prata
Pedro Pinheiro de Negreiros Bessa Fortaleza – CE Prata
Marilia Valeska Costa Medeiros Fortaleza – CE Prata
Camilla Matias Morais Fortaleza – CE Prata
Márcio Rabello de Freitas Mesquita – RJ Prata
Alex Atsushi Takeda Londrina – PR Bronze
Renan Lima Novais Niterói – RJ Bronze
Rafael Horimoto de Freitas São Paulo – SP Bronze
Dielson de Britto Junior Rio de Janeiro – RJ Bronze
Hugo Fonseca Araújo Juiz de Fora – MG Bronze
Vitor Mori São Paulo – SP Bronze
Cindy Yuchi Tsai São Paulo – SP Bronze
Thiago Ribeiro Ramos Varginha – MG Bronze
Gabriel Moreira Francisco Santo André – SP Bronze
Tales Augusto Gonçalves Alphonse Paraguaçu Paulista – SP Bronze
Nathana Alcântara Lima Fortaleza – CE Bronze
Illan Feiman Halpern Itatiaia – RJ Bronze
Thiago da Silva Pinheiro São Paulo – SP Bronze
Júlio Cézar Batista de Souza Salvador – BA Bronze
Thiago Ide Sousa Suzano – SP Bronze
Danilo Marcolongo Afonso S. B. do Campo – SP Bronze
Caio José Fonseca Santos Rio de Janeiro – RJ Menção Honrosa
Caio Sérgio Parente Silva Rio de Janeiro – RJ Menção Honrosa
Isabella Amorim Gonçalez Fortaleza – CE Menção Honrosa
Ana Luísa de Almeida Losnak São Paulo – SP Menção Honrosa
Yuri Bastos Pereira Rio de Janeiro – RJ Menção Honrosa
Mateus Sampaio de Mendonça Belo Horizonte – MG Menção Honrosa
Alisson de Brito Ninomia São Paulo – SP Menção Honrosa
Alan Eduardo dos Santos Góes Fortaleza – CE Menção Honrosa
José Cabadas D. Neto Salvador – BA Menção Honrosa
Marcelo Rafael Silva Rempel Maringá – PR Menção Honrosa
Rafael Rabelo de Carvalho Brasília – DF Menção Honrosa
Davi Lopes Alves de Medeiros Fortaleza – CE Menção Honrosa
Gabriella Fonseca Ribeiro Betim – MG Menção Honrosa
Christian Eduardo de Umeki e Saiki São Paulo – SP Menção Honrosa
Marco Antonio Lopes Pedroso Santa Isabel – SP Menção Honrosa
Catarina Yu Na Kim São Paulo – SP Menção Honrosa
Rafael Alves da Silva Teresina – PI Menção Honrosa
Pedro Henrique Azevedo Damacena Fortaleza – CE Menção Honrosa
Renan Henrique Finder Joinville – SC Menção Honrosa
Ricardo Bioni Liberalquino Maceió – AL Menção Honrosa
Dalen Chen Kuang Osasco – SP Menção Honrosa
Izabela Karennina Travizani Maffra Belo Horizonte – MG Menção Honrosa
Jennifer Katherine Koshiba Yu São Paulo – SP Menção Honrosa
Felipe Onório da Silva Oliveira Botucatu – SP Menção Honrosa

EUREKA! N°24, 2006

74
Sociedade Brasileira de Matemática

Nível 3 (Ensino Médio)


NOME CIDADE – ESTADO PRÊMIO
Gabriel Tavares Bujokas São Paulo – SP Ouro
Guilherme Rodrigues Nogueira de Souza São Paulo – SP Ouro
Thomás Yoiti Sasaki Hoshina Rio de Janeiro – RJ Ouro
Regis Prado Barbosa Fortaleza – CE Ouro
Luty Rodrigues Ribeiro Fortaleza – CE Ouro
Rafael Mendes de Oliveira Rio de Janeiro – RJ Ouro
Cesar Ryudi Kawakami São Paulo – SP Prata
Jose Marcos Andrade Ferraro São Paulo – SP Prata
José Armando Barbosa Filho Fortaleza – CE Prata
Anderson Hoshiko Aiziro São Paulo – SP Prata
Leandro Farias Maia Fortaleza – CE Prata
André Linhares Rodrigues Fortaleza – CE Prata
Levi Máximo Viana Fortaleza – CE Prata
Leonardo Ribeiro de Castro Carvalho São Paulo – SP Prata
Wilson Camara Marriel Rio de Janeiro – RJ Prata
Fabiano Edson Carlos Fortaleza – CE Prata
Adenilson Arcanjo de Moura Junior Fortaleza – CE Bronze
Edson Augusto Bezerra Lopes Fortaleza – CE Bronze
Rodrigo Viana Soares Fortaleza – CE Bronze
Eduardo Fischer Encantado – RS Bronze
Rafael Sampaio de Rezende Fortaleza – CE Bronze
Rafael Montezuma Pinheiro Cabral Fortaleza – CE Bronze
Gustavo Sampaio Sousa Fortaleza – CE Bronze
Ramon Moreira Nunes Fortaleza – CE Bronze
Hector Kenzo Horiuti Kitahara São Paulo – SP Bronze
Francisco Tarcísio Guedes Lima Verde Neto Fortaleza – CE Bronze
Alexandre Hideki Deguchi Martani São Paulo – SP Bronze
Enzo Haruo Hiraoka Moriyama São Paulo – SP Bronze
Rafael Morioka Oda São Paulo – SP Bronze
André Lucas Ribeiro dos Santos Pindamonhangaba – SP Bronze
Michel Faleiros Martins Campinas – SP Bronze
Antônio Felipe Cavalcante Carvalho Fortaleza – CE Bronze
Rafael Moura e Sucupira Fortaleza – CE Menção Honrosa
Artur de Almeida Losnak São Paulo – SP Menção Honrosa
Tiago Porto Barbosa Fortaleza – CE Menção Honrosa
Willy George do Amaral Petrenko Rio de Janeiro – RJ Menção Honrosa
Douglas Bokliang Ang Cunha S. J. dos Campos – SP Menção Honrosa
Breno Vieira de Aguiar Fortaleza – CE Menção Honrosa
Beatriz Laiate Sorocaba – SP Menção Honrosa
Vinicius Gripp Barros Ramos Rio de Janeiro – RJ Menção Honrosa
Lucio Eiji Assaoka Hossaka Curitiba – PR Menção Honrosa
Mateus Oliveira de Figueiredo Fortaleza – CE Menção Honrosa
Marcus Edson Barreto Brito Fortaleza – CE Menção Honrosa
Flávio Henrique Moura Stakoviak Belém – PA Menção Honrosa
Ricardo Turolla Bortolotti Rio Claro – SP Menção Honrosa
Pedro Henrique Silva Belisário Rio de Janeiro – RJ Menção Honrosa
Filipe Alves Tomé Fortaleza – CE Menção Honrosa
Frederico de Souza Frydman Salvador – BA Menção Honrosa
Heytor Bruno Nobre Pitombeira das Virgens Fortaleza – CE Menção Honrosa
Daniel Lopes Alves de Medeiros Fortaleza – CE Menção Honrosa

EUREKA! N°24, 2006

75
Sociedade Brasileira de Matemática

Nível Universitário
NOME CIDADE – ESTADO PRÊMIO
Humberto Silva Naves S. J. dos Campos – SP Ouro
Bernardo Freitas Paulo da Costa Rio de Janeiro – RJ Ouro
Alex Corrêa Abreu Niterói – RJ Ouro
Rafael Daigo Hirama Campinas – SP Ouro
Diêgo Veloso Uchôa Teresina – PI Ouro
Fábio Dias Moreira Rio de Janeiro – RJ Ouro
Luís Daniel Barbosa Coelho Rio de Janeiro – RJ Prata
Carlos Stein Naves de Brito S.J. dos Campos – SP Prata
Yuri Gomes Lima Fortaleza – CE Prata
Rafael Marini Silva Vila Velha – ES Prata
Murilo Vasconcelos Andrade Maceió – AL Prata
Thiago Barros Rodrigues Costa Fortaleza – CE Prata
Felipe Rodrigues Nogueira de Souza São Paulo – SP Prata
Leonardo Augusto Zão Nilópolis – RJ Prata
Vitor Gabriel Kleine Mogi das Cruzes – SP Prata
Estillac Lins Maciel Borges Filho Belém – PA Bronze
Rodrigo Roque Dias São Paulo – SP Bronze
Eduardo de Moraes Rodrigues Poço São Paulo – SP Bronze
Gustavo Gomes de Araujo Ribeirão Preto – SP Bronze
Raphael Constant da Costa Rio de Janeiro – RJ Bronze
Davi Maximo Alexandrino Nogueira Fortaleza – CE Bronze
Jorge Peixoto de Morais Neto Goiânia – GO Bronze
Eduardo Ferraz Castelo Branco Ferreira Rio de Janeiro – RJ Bronze
Eduardo Famini Silva Rio de Janeiro – RJ Bronze
Moyses Afonso Assad Cohen Rio de Janeiro – RJ Bronze
Kellem Corrêa Santos Rio de Janeiro – RJ Menção Honrosa
Evandro Makiyama São Paulo – SP Menção Honrosa
Thiago da Silva Sobral S.J. dos Campos – SP Menção Honrosa
Pedro Paiva Zühlke Dioliveira Brasília – DF Menção Honrosa
Helder Oliveira de Castro Mogi das Cruzes – SP Menção Honrosa
Thiago Costa Leite Santos São Paulo – SP Menção Honrosa
Marcos Francisco Ferreira Martinelli Rio de Janeiro – RJ Menção Honrosa
Rogério de Assis Medeiros Franco da Rocha – SP Menção Honrosa
Samuel Barbosa Feitosa Fortaleza – CE Menção Honrosa
Elder Rodrigo Barbosa Campos Rio de Janeiro – RJ Menção Honrosa
Francisco Bruno de Lima Holanda Fortaleza – CE Menção Honrosa
Giovana Siracusa Gouveia Recife – PE Menção Honrosa
Henrique Roscoe de Oliveira Brasília – DF Menção Honrosa

EUREKA! N°24, 2006

76
Sociedade Brasileira de Matemática

AGENDA OLÍMPICA
XXVIII OLIMPÍADA BRASILEIRA DE MATEMÁTICA

NÍVEIS 1, 2 e 3
Primeira Fase – Sábado, 10 de junho de 2006
Segunda Fase – Sábado, 2 de setembro de 2006
Terceira Fase – Sábado, 28 de outubro de 2006 (níveis 1, 2 e 3)
Domingo, 29 de outubro de 2006 (níveis 2 e 3 - segundo dia de prova).

NÍVEL UNIVERSITÁRIO
Primeira Fase – Sábado, 2 de setembro de 2006
Segunda Fase – Sábado, 28 e Domingo, 29 de outubro de 2006

XII OLIMPÍADA DE MAIO
13 de maio de 2006

XVII OLIMPÍADA DE MATEMÁTICA DO CONE SUL
5 a 11 de maio de 2006
Escobar, Argentina

XLVII OLIMPÍADA INTERNACIONAL DE MATEMÁTICA
8 a 19 de julho de 2006
Ljubljana - Eslovênia.

XIII OLIMPÍADA INTERNACIONAL DE MATEMÁTICA UNIVERSITÁRIA
20 a 26 de julho de 2006
Odessa, Ucrânia

XXI OLIMPÍADA IBEROAMERICANA DE MATEMÁTICA
22 de setembro a 01 de outubro de 2006
Equador

IX OLIMPÍADA IBEROAMERICANA DE MATEMÁTICA UNIVERSITÁRIA
18 de novembro de 2006

♦♦♦

EUREKA! N°24, 2006

77
Sociedade Brasileira de Matemática

COORDENADORES REGIONAIS
Alberto Hassen Raad (UFJF) Juiz de Fora – MG
Américo López Gálvez (USP) Ribeirão Preto – SP
Amarísio da Silva Araújo (UFV) Viçosa – MG
Andreia Goldani FACOS Osório – RS
Antonio Carlos Nogueira (UFU) Uberlândia – MG
Ali Tahzibi (USP) São Carlos – SP
Benedito Tadeu Vasconcelos Freire (UFRN) Natal – RN
Carlos Alexandre Ribeiro Martins (Univ. Tec. Fed. De Paraná) pato Branco - PR
Carlos Frederico Borges Palmeira (PUC-Rio) Rio de Janeiro – RJ
Claus Haetinger (UNIVATES) Lajeado – RS
Cleonor Crescêncio das Neves (UTAM) Manaus – AM
Cláudio de Lima Vidal (UNESP) S.J. do Rio Preto – SP
Edson Roberto Abe (Colégio Objetivo de Campinas) Campinas – SP
Élio Mega (Colégio Etapa) São Paulo – SP
Éder Luiz Pereira de Andrade (UNESPAR/FECILCAM) Campo Mourão – PR
Eudes Antonio da Costa (Univ. do Tocantins) Arraias – TO
Florêncio Ferreira Guimarães Filho (UFES) Vitória – ES
Ivanilde Fernandes Saad (UC. Dom Bosco) Campo Grande– MS
Janice T. Reichert (UNOCHAPECÓ) Chapecó – SC
João Benício de Melo Neto (UFPI) Teresina – PI
João Francisco Melo Libonati (Grupo Educacional Ideal) Belém – PA
Jorge Costa Duarte Filho (UFPB) João Pessoa - PB
José Cloves Saraiva (UFMA) São Luis – MA
José Luiz Rosas Pinho (UFSC) Florianópolis – SC
José Vieira Alves (UFPB) Campina Grande – PB
José William Costa (Instituto Pueri Domus) Santo André – SP
Krerley Oliveira (UFAL) Maceió – AL
Licio Hernandes Bezerra (UFSC) Florianópolis – SC
Luzinalva Miranda de Amorim (UFBA) Salvador – BA
Mário Rocha Retamoso (UFRG) Rio Grande – RS
Marcelo Rufino de Oliveira (Grupo Educacional Ideal) Belém – PA
Marcelo Mendes (Colégio Farias Brito, Pré-vestibular) Fortaleza – CE
Newman Simões (Cursinho CLQ Objetivo) Piracicaba – SP
Raúl Cintra de Negreiros Ribeiro (Colégio Anglo) Atibaia – SP
Ronaldo Alves Garcia (UFGO) Goiânia – GO
Rogério da Silva Ignácio (Col. Aplic. da UFPE) Recife – PE
Reginaldo de Lima Pereira (Escola Técnica Federal de Roraima) Boa Vista – RR
Reinaldo Gen Ichiro Arakaki (LAC - Laboratório Associado de Computação) SJ dos Campos – SP
Ricardo Amorim (Centro Educacional Logos) Nova Iguaçu – RJ
Sérgio Cláudio Ramos (IM-UFRGS) Porto Alegre – RS
Seme Guevara Neto (UFMG) Belo Horizonte – MG
Tadeu Ferreira Gomes (UEBA) Juazeiro – BA
Tomás Menéndez Rodrigues (U. Federal de Rondônia) Porto Velho – RO
Turíbio José Gomes dos Santos (UFPB) João Pessoa – PB
Valdenberg Araújo da Silva (U. Federal de Sergipe) São Cristovão – SE
Valdeni Soliani Franco (U. Estadual de Maringá) Maringá – PR
Vânia Cristina Silva Rodrigues (U. Metodista de SP) S.B. do Campo – SP
Wagner Pereira Lopes (CEFET – GO) Jataí – GO

EUREKA! N°24, 2006

78
CONTEÚDO

AOS LEITORES 3

XI OLIMPÍADA DE MAIO 4
Enunciados e Resultado Brasileiro

XII OLIMPÍADA DE MAIO 7


Enunciados e Resultado Brasileiro

XVI OLIMPÍADA DE MATEMÁTICA DO CONE SUL 8


Enunciados e Resultado Brasileiro

XVII OLIMPÍADA DE MATEMÁTICA DO CONE SUL 12


Enunciados e Resultado Brasileiro

XLVI OLIMPÍADA INTERNACIONAL DE MATEMÁTICA 14


Enunciados e Resultado Brasileiro

XLVII OLIMPÍADA INTERNACIONAL DE MATEMÁTICA 15


Enunciados e Resultado Brasileiro

XX OLIMPÍADA IBEROAMERICANA DE MATEMÁTICA 18


Enunciados e Resultado Brasileiro

XXI OLIMPÍADA IBEROAMERICANA DE MATEMÁTICA 20


Enunciados e Resultado Brasileiro

ARTIGOS
A FÓRMULA DE HERÃO 22
Fabiano Alberton de Alencar Nogueira

ÁREAS PARA ACHAR RAZÕES DE SEGMENTOS 26


Cícero Thiago e Marcelo Mendes

PROBLEMAS SOBRE PONTOS 31


Davi Máximo e Samuel Feitosa

POLINÔMIOS SIMÉTRICOS 46
Carlos A. Gomes
Sociedade Brasileira de Matemática

OLIMPÍADAS AO REDOR DO MUNDO 53

SOLUÇÕES DE PROBLEMAS PROPOSTOS 57

PROBLEMAS PROPOSTOS 60

AGÊNDA OLÍMPICA 61

COORDENADORES REGIONAIS 62

EUREKA! N°25, 2007

2
Sociedade Brasileira de Matemática

AOS LEITORES

Chegamos ao número 25 da Eureka! apresentando as provas e os


excelentes resultados brasileiros dos dois últimos anos em diversas competições
internacionais de que o Brasil participa. Temos também quatro belos artigos e,
atendendo a muitos pedidos, a volta da seção Olimpíadas ao redor do Mundo,
agora com mais colaboradores. Agradecemos e continuamos estimulando a
participação da comunidade olímpica na elaboração da Eureka! com problemas
propostos, soluções e artigos, que têm feito da Eureka! um instrumento vivo de
difusão das olimpíadas de Matemática no Brasil, contribuindo para a preparação
em alto nível dos participantes da OBM em todo o país.

Os editores

EUREKA! N°25, 2007

3
Sociedade Brasileira de Matemática

XI OLIMPÍADA DE MAIO
PRIMEIRO NÍVEL
PROBLEMA 1
Num quadro negro havia seis figuras: um círculo, um triângulo, um quadrado, um
trapézio, um pentágono e um hexágono, pintados de seis cores: azul, branco,
vermelho, amarelo, verde e marrom. Cada figura tinha somente uma cor e todas as
figuras eram de cores diferentes.
No dia seguinte perguntou-se qual era a cor de cada figura.
Pablo respondeu: "O círculo era vermelho, o triângulo era azul, o quadrado era
branco, o trapézio era verde, o pentágono era marrom e o hexágono era amarelo."
Sofia respondeu: "O círculo era amarelo, o triângulo era verde, o quadrado era
vermelho, o trapézio era azul, o pentágono era marrom e o hexágono era branco."
Pablo errou três vezes e Sofia duas vezes, e sabe-se que o pentágono era marrom.
Determine se é possível saber com certeza qual era a cor de cada uma das figuras.
PROBLEMA 2
Um número inteiro chama-se autodivi se é divisível pelo número de dois
algarismos formado por seus dois últimos dígitos (dezenas e unidades). Por
exemplo, 78013 é autodivi pois é divisível por 13, 8517 é autodivi pois é divisível
por 17.
Encontre 6 números inteiros consecutivos que sejam autodivi e que tenham os
dígitos das unidades, das dezenas e das centenas distintos de 0.

PROBLEMA 3
Um segmento AB de largura 100 está dividido em 100 segmentos menores de
largura 1 mediante 99 pontos intermediários.
Ao extremo A designa-se o 0 e ao extremo B, o 1.
Gustavo designa a cada um dos 99 pontos intermediários um 0 ou um 1, a sua
escolha, e logo pinta cada segmento de largura 1 de azul ou de vermelho,
respeitando a seguinte regra:
São vermelhos todos os segmentos que têm o mesmo número em seus extremos e
são azuis os segmentos que têm números diferentes em seus extremos.
Determine se Gustavo pode designar os 0's e os 1's de modo a obter exatamente 30
segmentos azuis. E 35 segmentos azuis? (Em cada caso, se a resposta é sim,
mostre uma distribuição dos 0's e dos 1's, e se a resposta é não, explique o
porquê.)
PROBLEMA 4
Há duas figuras de papel: um triângulo equilátero e um retângulo. A altura do
retângulo é igual à altura do triângulo e a base do retângulo é igual à base do

EUREKA! N°25, 2007

4
Sociedade Brasileira de Matemática

triângulo. Divida o triângulo em três partes e o retângulo em duas, mediante cortes


retos, de modo que com os cinco pedaços possamos montar, sem buracos nem
superposições, um triângulo equilátero. Para montar a figura, cada parte pode ser
girada e/ou dar a volta. (Justifique que o triângulo montado é equilátero.)

PROBLEMA 5
a) Em cada casa de um tabuleiro 7 × 7 se escreve um dos números: 1, 2, 3, 4, 5, 6
ou 7 de forma que cada número esteja escrito em sete casas distintas. Será possível
que em nenhuma fila e em nenhuma coluna fiquem escritos números
consecutivos?
b) Em cada casa de um tabuleiro 5 × 5 se escreve um dos números: 1, 2, 3, 4 ou 5
de forma que cada número esteja escrito em cinco casas distintas. Será possível
que em nenhuma fila e nenhuma coluna fiquem escritos números consecutivos?

SEGUNDO NÍVEL
PROBLEMA 1
Determine o menor número de três dígitos que seja o produto de dois números de
dois dígitos, de forma que os sete dígitos destes três números sejam todos
diferentes.

PROBLEMA 2
Gonçalo escreve num quadro negro quatro números escolhidos entre 0, 1, 2, 3 ou
4. Pode repetir números. Nicolás realiza repetidas vezes a seguinte operação: troca
um dos números, a sua escolha, pelo resto da divisão por 5 do produto de outros
dois números do quadro negro, a sua escolha.
O objetivo de Nicolás é conseguir que os quatro números sejam iguais. Determine
se Gonçalo pode escolher os números iniciais de forma que seja impossível a
Nicolás alcançar seu objetivo.

PROBLEMA 3
No triângulo isósceles ABC, com AB = AC, seja M o ponto médio de BC. O Ponto
D no lado BC é tal que BADˆ = 1 BAC
ˆ . A reta perpendicular a AD por C corta a
6
AD em N de modo que DN = DM. Calcule os ângulos do triângulo ABC.

PROBLEMA 4
Num baile há 12 homens, numerados de 1 a 12 e 12 mulheres, numeradas de 1 a
12. A cada homem se designa um "amigo oculto" entre os outros 11. Todos
dançaram todas as músicas. Na primeira música cada homem dançou com a

EUREKA! N°25, 2007

5
Sociedade Brasileira de Matemática

mulher que tem seu mesmo número. A partir daí, cada homem dançou uma nova
música com uma mulher que havia dançado a música anterior com seu amigo
oculto.
Na terceira música os casais foram:
Homens 1 2 3 4 5 6 7 8 9 10 11 12
Mulheres 5 11 2 12 8 10 9 4 6 3 7 1
Encontre o número do amigo oculto de cada homem.

PROBLEMA 5
Sobre o tabuleiro 9 × 9 aterrissou a nave inimiga que cobre exatamente 5 casas do
tabuleiro, assim:

A nave é invisível.
Cada míssil defensivo cobre exatamente uma casa, e destrói a nave se bater numa
das 5 casas que esta ocupa.
Determine o número mínimo de mísseis que são necessários para destruir com
certeza a nave inimiga.
RESULTADOS BRASILEIROS PRIMEIRO NÍVEL
Leonardo Pereira Stedile São Paulo - SP Medalha de Ouro
James Jun Hong São Paulo - SP Medalha de Prata
Thiago Gonçales Piracicaba - SP Medalha de Prata
César Ilharco Magalhães Juiz de Fora - MG Medalha de Bronze
Fernando Fonseca Andrade Oliveira Belo Horizonte - MG Medalha de Bronze
Erick Magno Costa Alonso Uberaba - MG Medalha de Bronze
Maíra Islena T. da Silva Belo Horizonte - MG Medalha de Bronze
Matheus Barros de Paula Taubaté - SP Menção Honrosa
Wagner Carlos Morêto Loyola Filho Vitória - ES Menção Honrosa
André Y. O. Bastos São Paulo - SP Menção Honrosa

RESULTADOS BRASILEIROS SEGUNDO NÍVEL


Henrique Pondé de Oliveira Pinto Salvador - BA Medalha de Ouro
Rafael Tupinambá Dutra Belo Horizonte - MG Medalha de Prata
Thiago Ribeiro Ramos Varginha - MG Medalha de Prata
Victor Reis de Abreu Cavalcante Maceió - AL Medalha de Bronze
Lucas Zanotto Portela Curitiba - PR Medalha de Bronze
Lucio Eiji Assaoka Hossaka Curitiba - PR Medalha de Bronze
Tiago Madeira Itajaí - SC Medalha de Bronze
Hugo Musso Gualandi Vitória - ES Menção Honrosa
Giuliano Pezzolo Giacaglia Santo André - SP Menção Honrosa
Wilson Camara Marriel Rio de Janeiro - RJ Menção Honrosa
Illan Feiman Halpern Itatiaia - RJ Menção Honrosa

EUREKA! N°25, 2007

6
Sociedade Brasileira de Matemática

XII OLIMPÍADA DE MAIO


PRIMEIRO NÍVEL

PROBLEMA 1
Um calendário digital exibe a data: dia, mês e ano, com 2 dígitos para o dia, 2
dígitos para o mês e 2 dígitos para o ano. Por exemplo, 01-01-01 corresponde a
primeiro de janeiro de 2001 e 25-05-23 corresponde a 25 de maio de 2023. Em
frente ao calendário há um espelho. Os dígitos do calendário são como os da
figura abaixo:


Se 0, 1, 2, 5 e 8 se refletem, respectivamente, em 0, 1, 5, 2 e 8, e os outros dígitos
perdem sentido ao se refletirem, determine quantos dias do século, ao se refletirem
no espelho, correspondem também a uma data.

PROBLEMA 2
Um retângulo de papel de 3cm × 9cm é dobrado ao longo de uma reta,
fazendo coincidir dois vértices opostos. Deste modo se forma um
pentágono. Calcular sua área.

PROBLEMA 3
Há 20 pontos alinhados, separados por uma mesma distância:

• • • • • • • • • • • • • • • • • • • •
Miguel tem que pintar de vermelho três ou mais destes pontos, de maneira que os
pontos vermelhos estejam separados por uma mesma distância e seja impossível
pintar de vermelho exatamente um ponto a mais sem desobedecer a condição
anterior. Determinar de quantas maneiras Miguel poderá fazer a tarefa.

PROBLEMA 4
Com 150 cubinhos brancos de 1 × 1 × 1 arma-se um paralelepípedo de 6 × 5 × 5,
pintam-se as seis faces de azul e logo se desarma o paralelepípedo. Lucrecia deve
armar um novo paralelepípedo, sem buracos, usando exclusivamente cubinhos que
tenham ao menos uma face azul e de modo que as faces do paralelepípedo de
Lucrécia sejam todas completamente azuis.
Determinar as dimensões do paralelepípedo de maior volume que Lucrecia pode
armar.

EUREKA! N°25, 2007

7
Sociedade Brasileira de Matemática

PROBLEMA 5
Em algumas casas de um tabuleiro 10 × 10 coloca-se uma ficha de maneira que se
verifique a seguinte propriedade:
Para cada casa que tem uma ficha, a quantidade de fichas colocadas em sua
mesma linha deve ser maior ou igual que a quantidade de fichas colocadas em sua
mesma coluna.
Quantas fichas pode haver no tabuleiro?
Diga todas as possibilidades.

SEGUNDO NÍVEL

PROBLEMA 1
a +1 b +1
Determinar todos os pares de números naturais a e b tais que e são
b a
números naturais.

PROBLEMA 2
No quadro negro estão escritos vários números primos (alguns deles repetidos).
Mauro somou os números do quadro negro e Fernando multiplicou os números do
quadro negro. O resultado que obteve Fernando é igual a 40 vezes o resultado que
obteve Mauro. Determinar quais podem ser os números do quadro negro.
Diga todas as possibilidades.

PROBLEMA 3
Escrever um número inteiro positivo em cada casa de
modo que:
– Os seis números sejam distintos.
– A soma dos seis números seja 100.
Se cada número é multiplicado pelo seu vizinho
(no sentido dos ponteiros do relógio) e se somam
os seis resultados das seis multiplicações, obtém-
se o menor valor possível.
Explicar por que não é possível obter um valor
menor.

PROBLEMA 4
Seja ABCD um trapézio de bases AB e CD. Seja O o ponto de interseção de suas
diagonais AC e BD. Se a área do triângulo ABC é 150 e a área do triângulo ACD é
120, calcular a área do triângulo BCO.

EUREKA! N°25, 2007

8
Sociedade Brasileira de Matemática

PROBLEMA 5
Com 28 pontos forma-se uma "grade triangular" de lados iguais, como se mostra
na figura abaixo.
Uma operação consiste em escolher três pontos que sejam os vértices de um
triângulo equilátero e retirar estes três pontos da grade. Se após realizar várias
destas operações resta somente um ponto, em quais posições pode ficar esse
ponto?
Determinar todas as possibilidades e indicar em cada caso as operações realizadas.
Justificar por que o ponto que restou não pode estar numa outra posição.

RESULTADOS BRASILEIROS PRIMEIRO NÍVEL


Matheus Barros de Paula Taubaté - SP Medalha de Ouro
César Ilharco Magalhães Juiz de Fora - MG Medalha de Prata
Henrique L. de Mello Rio de Janeiro - RJ Medalha de Prata
Iuri Rezende Souza Mineiros - GO Medalha de Bronze
Elder Massahiro Yoshida São Paulo - SP Medalha de Bronze
Deborah Barbosa Alves São Paulo - SP Medalha de Bronze
Victor Gonçalves Elias João Pessoa - PB Medalha de Bronze
Leonardo Gonçalves Fischer Fraiburgo - SC Menção Honrosa
Wagner Carlos Morêto Loyola Filho Vitória - ES Menção Honrosa
Ivan Seiki Hellmeister São Paulo - SP Menção Honrosa

RESULTADOS BRASILEIROS SEGUNDO NÍVEL


Thiago Ribeiro Ramos Varginha - MG Medalha de Ouro
Marcelo Tadeu de Sá O. Sales Barreiras - BA Medalha de Prata
Rafael Horimoto de Freitas São Paulo - SP Medalha de Prata
Renan Henrique Finder Joinville - SC Medalha de Bronze
Illan Feiman Halpern Itatiaia - RJ Medalha de Bronze
Renan Lima Novais Niterói - RJ Medalha de Bronze
Rafael Rabelo de Carvalho Brasília - DF Medalha de Bronze
Rafael Pacheco Gomes Fortaleza - CE Menção Honrosa
Caio Sérgio Parente Silva Rio de Janeiro - RJ Menção Honrosa
Hugo Fonseca Araújo Juiz de Fora - MG Menção Honrosa

EUREKA! N°25, 2007

9
Sociedade Brasileira de Matemática

XVI OLIMPÍADA DE MATEMÁTICA DO CONE SUL


Enunciados e Resultado Brasileiro

A XVI Olimpíada de Matemática do Cone Sul foi realizada na cidade de


Sucre, Bolívia no período de 14 a 23 de Maio de 2005. A equipe brasileira foi
liderada pelos professores Emanuel Augusto de Souza Carneiro e Davi
Alexandrino Nogueira, ambos da cidade de Fortaleza – CE.

RESULTADOS DA EQUIPE BRASILEIRA

BRA1 Henrique Pondé de Oliveira Pinto Medalha de Ouro


BRA2 Guilherme R. Nogueira de Souza Medalha de Ouro
BRA3 Edson Augusto Bezerra Lopes Medalha de Prata
BRA4 Rafael Tupynambá Dutra Medalha de Prata

PRIMEIRO DIA

PROBLEMA 1
Considere a seguinte seqüência:
a1 = último dígito da soma dos dígitos do número 2005
a2 = último dígito da soma dos dígitos do número 20052005
a3 = último dígito da soma dos dígitos do número 200520052005 ...


an = último dígito da soma dos dígitos do número 20052005... 2005


n vezes 2005

Calcule: a1 + a2 + a3 + · · · + a2005

PROBLEMA 2
Seja ABC um triângulo acutângulo e sejam AN, BM e CP as alturas relativas aos
lados BC, CA e AB, respectivamente. Sejam R, S as projeções de N sobre os lados
AB, CA, respectivamente, e Q, W as projeções de N sobre as alturas BM e CP,
respectivamente.

Mostre que R, Q, W, S são colineares;


Mostre que MP = RS – QW.

EUREKA! N°25, 2007

10
Sociedade Brasileira de Matemática

PROBLEMA 3
A unidade monetária de um certo país se chama reo, e todas as moedas que
circulam são de números inteiros de reos. Em um grupo de três pessoas, cada uma
tem 60 reos em moedas (mas não se sabe que tipo de moedas cada uma tem). Cada
uma das três pessoas pode pagar a cada uma das outras qualquer valor inteiro entre
1 e 15 reos, inclusive, talvez com troco. Mostre que as três pessoas em conjunto
podem pagar exatamente (sem troco) qualquer valor inteiro entre 45 e 135 reos,
inclusive.

SEGUNDO DIA

PROBLEMA 4
Seja ABC um triângulo isósceles, com AB = AC. Uma reta r que passa pelo
incentro I de ABC intersecta os lados AB e AC nos pontos D e E, respectivamente.
F e G são pontos sobre o lado BC tais que BF = CE e CG = BD. Mostre que o
ângulo ∠FIG é constante ao variar r.

PROBLEMA 5
Diremos que um número de 20 dígitos é especial se é impossível representá-lo
como produto de um número de 10 dígitos por um número de 11 dígitos.
Determine qual é a máxima quantidade possível de números consecutivos que são
especiais.

PROBLEMA 6
No plano cartesiano traçamos circunferências de raio 1/20 com centros em cada
ponto de coordenadas inteiras. Mostre que qualquer circunferência de raio 100 que
se trace no plano intersecta pelo menos uma das circunferências pequenas.

EUREKA! N°25, 2007

11
Sociedade Brasileira de Matemática

XVII OLIMPÍADA DE MATEMÁTICA DO CONE SUL


Enunciados e Resultado Brasileiro

A XVII Olimpíada de Matemática do Cone Sul foi realizada na cidade de


Escobar, Argentina no período de 5 a 11 de Maio de 2006. A equipe brasileira foi
liderada pelos professores Carlos Yuzo Shine (São Paulo – SP) e Luzinalva
Miranda de Amorim (Salvador – BA).

RESULTADOS DA EQUIPE BRASILEIRA

BRA1 Henrique Pondé de Oliveira Pinto Medalha de Ouro


BRA2 Rafael Tupynambá Dutra Medalha de Prata
BRA3 Ramon Moreira Nunes Medalha de Prata
BRA4 Regis Prado Barbosa Medalha de Prata

PRIMEIRO DIA

PROBLEMA 1
No quadrilátero convexo ABCD, sejam E e F os pontos médios dos lados AD e
BC, respectivamente. Os segmentos CE e DF cortam-se em O. Demonstrar que se
as retas AO e BO dividem o lado CD em três partes iguais então ABCD é um
paralelogramo.

PROBLEMA 2
Duas pessoas, A e B, jogam o seguinte jogo: eles retiram moedas de uma pilha que
contém, inicialmente, 2006 moedas. Os jogadores jogam alternadamente retirando,
em cada jogada, 1 a 7 moedas; cada jogador guarda as moedas que retira. Se
quiser, um jogador pode passar (não retirar moedas em sua vez), mas para isso
deve pagar 7 moedas das que retirou da pilha em jogadas anteriores. Estas 7
moedas são colocadas em uma caixa separada e não interferem mais no jogo.
Ganha quem retira a última moeda, e A começa o jogo.
Determinar qual jogador pode assegurar a vitória, não importando como jogue o
outro. Mostrar uma estratégia vencedora e explicar por que é vencedora.

PROBLEMA 3
Seja n um número natural. A sucessão finita α de inteiros positivos tem, entre seus
termos, exatamente n números distintos (α pode ter números repetidos). Além
disso, se de um de seus termos qualquer subtraímos 1, obtemos uma sucessão que
tem, entre seus termos, pelo menos n números positivos distintos. Qual é o valor
mínimo que pode ter a soma de todos os termos da sucessão α?

EUREKA! N°25, 2007

12
Sociedade Brasileira de Matemática

SEGUNDO DIA

PROBLEMA 4
Daniel escreveu em uma lousa, de cima para baixo, uma lista de números inteiros
positivos menores ou iguais a 10. Ao lado de cada número da lista de Daniel,
Martín anotou a quantidade de vezes que esse número aparece na lista de Daniel e
obteve assim uma lista de mesmo tamanho.
Se lemos a lista de Martín de baixo para cima obtemos a mesma lista de números
que Daniel escreveu de cima para baixo. Encontre o maior tamanho que a lista de
Daniel pode ter.

PROBLEMA 5
Encontrar todos os inteiros positivos n tais que  n  − 2 divide n − 4 e  n  + 2
   
divide n + 4 .
( [r ] denota a parte inteira de r, ou seja, o maior inteiro que é menor ou igual a r.
Por exemplo: [2, 5] = 2 ;  3  = 1 ; [5] = 5 .)
 

PROBLEMA 6
Dividimos o plano em casinhas quadradas de lado 1, traçando retas paralelas aos
eixos coordenados. Cada casinha é pintada de branco ou preto. A cada segundo,
recolorimos simultaneamente todas as casinhas, de acordo com a seguinte regra:
cada casinha Q adota a cor que mais aparece na configuração de cinco casinhas
indicadas na figura

O processo de recoloração é repetido indefinidamente.

a) Determinar se existe uma coloração inicial com uma quantidade finita de


casinhas pretas tal que sempre há pelo menos uma casinha preta, não importando
quantos segundos se passaram desde o início do processo.

b) Determinar se existe uma coloração inicial com uma quantidade finita de


casinhas pretas tal que o número de casinhas pretas, após alguma quantidade de
segundos, seja pelo menos 1010 vezes maior que o número inicial de casinhas
pretas.

EUREKA! N°25, 2007

13
Sociedade Brasileira de Matemática

XLVI OLIMPÍADA INTERNACIONAL DE MATEMÁTICA


Enunciados e Resultado Brasileiro

A XLVI Olimpíada Internacional de Matemática foi realizada na cidade


de Mérida – México no período de 08 a 19 de julho de 2005. A equipe brasileira
foi liderada pelos professores Edmilson Luis Rodrigues Motta (São Paulo – SP) e
Onofre Campos da Silva Farias (Fortaleza – CE).

RESULTADOS DA EQUIPE BRASILEIRA

BRA1 Gabriel Tavares Bujokas Medalha de Ouro


BRA2 Thomás Yoiti Sasaki Hoshina Medalha de Bronze
BRA3 Leandro Farias Maia Menção Honrosa
BRA4 Guilherme Rodrigues Nogueira de Souza Menção Honrosa
BRA5 Levi Máximo Viana ****
BRA6 Edson Augusto Bezerra Lopes ****

PRIMEIRO DIA

PROBLEMA 1
São escolhidos seis pontos nos lados de um triângulo equilátero ABC: A1 e A2 em
BC, B1 e B2 em CA, C1 e C2 em AB. Estes pontos são os vértices de um
hexágono convexo A1 A2 B1B2C1C 2 cujos lados são todos iguais. Demonstre que as
retas A1 B2 , B1C 2 e C1 A2 são concorrentes.

PROBLEMA 2
Seja a1 , a2 ,... uma seqüência de inteiros que tem infinitos termos positivos e
infinitos térmos negativos. Suponhamos que para cada inteiro positivo n, os
números a1 , a2 ,..., an tem n restos distintos ao ser divididos entre n. Demonstre
que cada inteiro se encontra exatamente uma vez na sucessão.

PROBLEMA 3
Sejam x, y, z números reais positivos tais que xy z ≥ 1 .
Demonstre que
x5 − x 2 y5 − y 2 z5 − z 2
+ + ≥ 0.
x5 + y 2 + z 2 y 5 + z 2 + x 2 z 5 + x 2 + y 2

EUREKA! N°25, 2007

14
Sociedade Brasileira de Matemática

SEGUNDO DIA

PROBLEMA 4
Consideremos a seqüência infinita a 1 , a 2 , ... definida por
an = 2 n + 3n + 6 n − 1 ( n = 1, 2 , ...)
Determine todos os inteiros positivos que são relativamente primos relativos
(coprimos) com todos os termos da seqüência.

PROBLEMA 5
Seja ABCD um quadrilátero convexo que tem os lados BC e AD iguais e não
paralelos. Sejam E e F pontos nos lados BC e AD, respectivamente, que são
distintos dos vértices e satisfazem BE = DF.
As retas AC e BD se cortam em P e a reta EF corta AC e BD respectivamente em
Q e R. Consideremos todos os triângulos PQR que se formam quando E e F
variam. Demonstre que as circunferências circunscritas a esses triângulos têm em
comum outro ponto além de P.

PROBLEMA 6
Numa competição de matemática foram propostos 6 problemas aos estudantes.
Cada par de problemas foi resolvido por mais de 2 5 dos estudantes. Ninguém
resolveu os 6 problemas. Demonstre que há pelo menos 2 estudantes tais que cada
um tem exatamente 5 problemas resolvidos.

EUREKA! N°25, 2007

15
Sociedade Brasileira de Matemática

XLVII OLIMPÍADA INTERNACIONAL DE MATEMÁTICA


Enunciados e Resultado Brasileiro

A XLVII Olimpíada Internacional de Matemática foi realizada na cidade


de Ljubljana – Eslovênia no período de 08 a 19 de julho de 2006. A equipe
brasileira foi liderada pelos professores Luciano Guimarães Monteiro de Castro
(Rio de Janeiro – RJ) e Pablo Rodrigo Ganassim (São Paulo – SP).

RESULTADOS DA EQUIPE BRASILEIRA

BRA1 André Linhares Rodrigues Medalha de Bronze


BRA2 Guilherme Rodrigues Nogueira de Souza Medalha de Bronze
BRA3 Leandro Farias Maia Medalha de Bronze
BRA4 Leonardo Ribeiro de Castro Carvalho Medalha de Bronze
BRA5 Rafael Mendes de Oliveira Medalha de Bronze
BRA6 Régis Prado Barbosa Medalha de Bronze

PRIMEIRO DIA

PROBLEMA 1

nnnn
Seja ABC um triângulo com incentro I. Um ponto P no interior do triângulo
verifica
PBA + PCA = PBC + PCB.
Prove que AP ≥ AI , com igualdade se, e somente se, P = I.

PROBLEMA 2
Uma diagonal de um polígono regular P de 2006 lados é um segmento bom se
separa P em duas partes, cada uma tendo um número ímpar de lados de P. Os
lados de P também são segmentos bons.
Divide-se P em triângulos, traçando-se 2003 diagonais que, duas a duas, não se
cortam no interior de P. Determine o maior número de triângulos isósceles nos
quais dois lados são segmentos bons que podem aparecer numa divisão como essa.

PROBLEMA 3
Determine o menor número real M tal que a desigualdade
ab ( a 2 − b2 ) + bc (b2 − c 2 ) + ca ( c2 − a 2 ) ≤ M ( a 2 + b2 + c 2 )
2

é verdadeira para todos os números reais a, b, c.

EUREKA! N°25, 2007

16
Sociedade Brasileira de Matemática

SEGUNDO DIA

PROBLEMA 4
Determine todos os pares de inteiros (x, y) tais que
1 + 2 x + 22 x +1 = y 2 .

PROBLEMA 5
Seja P(x) um polinômio de grau n > 1 com coeficientes inteiros e seja k um inteiro
positivo. Considere o polinômio
Q ( x ) = P ( P(...P( P ( x))...)), onde P aparece k vezes. Prove que existem no
máximo n inteiros t tais que Q(t ) = t.

PROBLEMA 6
A cada lado b de um polígono convexo P associa-se a maior das áreas dos
triângulos contidos em P que têm b como um dos lados. Prove que a soma das
áreas associadas a todos os lados de P é pelo menos o dobro da área de P.

EUREKA! N°25, 2007

17
Sociedade Brasileira de Matemática

XX OLIMPÍADA IBEROAMERICANA DE MATEMÁTICA


Enunciados e Resultado Brasileiro

A XX Olimpíada Iberoamericana de Matemática foi realizada na cidade


de Cartagena de Índias – Colômbia no período de 22 de setembro a 1 de outubro
de 2005. A equipe brasileira foi liderada pelos professores Élio Mega (São Paulo –
SP) e Yuri Gomes Lima (Fortaleza – CE).

RESULTADOS DA EQUIPE BRASILEIRA

BRA1 Rafael Marini Silva Medalha de Ouro


BRA2 Thomás Yoiti Sasaki Hoshina Medalha de Ouro
BRA3 Gabriel Tavares Bujokas Medalha de Ouro
BRA4 Thiago Costa Leite Santos Medalha de Ouro

PRIMEIRO DIA

PROBLEMA 1
Determine todas as triplas de números reais (x, y, z) que satisfazem o seguinte
sistema de equações:
xyz = 8,
x 2 y + y 2 z + z 2 x = 73,
x( y − z ) 2 + y ( z − x) 2 + z ( x − y ) 2 = 98.

PROBLEMA 2
Uma pulga salta sobre os pontos inteiros de uma reta numérica. Em seu primeiro
movimento salta desde o ponto 0 e cai no ponto 1. A partir daí, se num movimento
a pulga salta desde o ponto a e cai no ponto b, no seguinte movimento salta desde
o ponto b e cai num dos pontos b + (b – a) – 1, b + (b – a), b + (b – a) + 1.
Demonstre que se a pulga caiu duas vezes sobre o ponto n, para n inteiro positivo,
então deve ter feito ao menos t movimentos, onde t é o menor inteiro maior ou
igual a 2 n .

PROBLEMA 3
Seja p > 3 um número primo. Se
1 1 1 1 n
+ p + p + ... + =
p
1 2 3 ( p − 1) p
m
onde o máximo divisor comum de n e m é 1, demonstre que p 3 divide n.

EUREKA! N°25, 2007

18
Sociedade Brasileira de Matemática

SEGUNDO DIA

PROBLEMA 4
Dados os inteiros positivos a e b, denota-se por ( a ∇ b ) o resto que é obtido ao
dividir a por b. Este resto é um dos números 0, 1,…, b – 1. Encontre todos os
pares de números (a, p) tais que p é primo e vale:

( a ∇ p ) + ( a ∇ 2p ) + ( a ∇ 3p ) + ( a ∇ 4p ) = a + p.
PROBLEMA 5
Seja O o circuncentro de um triângulo acutângulo ABC e A1 um ponto no arco
menor BC da circunferência circunscrita ao triângulo ABC. Sejam A2 e A3 pontos
nos lados AB e AC respectivamente, tais que ∠BA1 A2 = ∠OAC e
∠CA1 A3 = ∠OAB. Demonstre que a reta A2 A3 passa pelo ortocentro do
triângulo ABC.

PROBLEMA 6
Dado um inteiro positivo n, num plano consideram-se 2n pontos alinhados
A1 , A2 ,..., A2n . Cada ponto é pintado de azul ou vermelho mediante o seguinte
procedimento:
No plano dado são traçadas n circunferências com diâmetros de extremos Ai e
Aj , disjuntas duas a duas. Cada Ak ,1 ≤ k ≤ 2n, pertence exatamente a uma
circunferência. Pintam-se os pontos de modo que os dois pontos de uma mesma
circunferência levem a mesma cor.
Determine quantas colorações distintas dos 2n pontos podem-se obter ao variar as
n circunferências e a distribuição das duas cores.

EUREKA! N°25, 2007

19
Sociedade Brasileira de Matemática

XXI OLIMPÍADA IBEROAMERICANA DE MATEMÁTICA


Enunciados e Resultado Brasileiro

A XXI Olimpíada Iberoamericana de Matemática foi realizada na cidade


de Guayaquil – Equador no período de 22 de setembro a 1 de outubro de 2006. A
equipe brasileira foi liderada pelos professores Paulo Cézar Pinto Carvalho (Rio
de Janeiro – RJ) e Cícero Thiago Magalhães (Fortaleza – CE).

RESULTADOS DA EQUIPE BRASILEIRA

BRA1 André Linhares Rodrigues Medalha de Ouro


BRA2 Guilherme Rodrigues Nogueira de Souza Medalha de Ouro
BRA3 Leandro Farias Maia Medalha de Prata
BRA4 Leonardo Ribeiro de Castro Carvalho Medalha de Prata

PRIMEIRO DIA

PROBLEMA 1
No triângulo escaleno ABC, com ∠BAC = 90° , consideram-se as circunferências
inscrita e circunscrita. A reta tangente em A à circunferência circunscrita corta a
reta BC em M. Sejam S e R os pontos de tangência da circunferência inscrita com
os catetos AC e AB, respectivamente. A reta RS corta a reta BC em N. As retas AM
e SR cortam-se em U. Demonstre que o triângulo UMN é isósceles.

PROBLEMA 2
Consideram-se n números reais a1, a2,..., an não necessariamente distintos. Seja d a
diferença entre o maior e o menor deles e seja
s = ∑ (ai − a j )
i< j
Demonstre que
n2d
(n − 1)d ≤ s ≤
4
e determine as condições que devem satisfazer estes n números para que se
verifique cada uma das igualdades.

PROBLEMA 3
Colocam-se os números 1,2,3,...,n2 nas casas de um tabuleiro n × n, em alguma
ordem, um número por casa. Uma ficha encontra-se inicialmente na casa com o
número n2. Em cada passo, a ficha pode mover-se para qualquer das casas que têm

EUREKA! N°25, 2007

20
Sociedade Brasileira de Matemática

um lado em comum com a casa onde se encontra. Primeiro, a ficha desloca-se para
a casa com o número 1, e para isso toma um dos caminhos mais curtos (com
menos passos) entre o n2 e o 1. Da casa com o número 1 desloca-se para a casa
com o número 2, a partir daí para a casa com o número 3, e assim sucessivamente,
até regressar à casa inicial, tomando em cada um desses deslocamentos o caminho
mais curto. A ficha dá N passos no percurso completo. Determine o menor valor e
o maior valor possíveis de N.

SEGUNDO DIA

PROBLEMA 4
Determine todos os pares (a, b) de inteiros positivos tais que 2a + 1 e 2b − 1
sejam primos entre si e a + b divida 4ab + 1 .

PROBLEMA 5
Dada uma circunferência C , considere um quadrilátero ABCD com os seus quatro
lados tangentes a C, com AD tangente a C em P e CD tangente a C em Q. Sejam
X e Y os pontos em que BD corta C, e M o ponto médio de XY. Demonstre que
∠AMP = ∠CMQ .

PROBLEMA 6
Seja n > 1 um inteiro ímpar. Sejam P0 e P1 dois vértices consecutivos de um
polígono regular de n lados. Para cada k •  GHILQHVH Pk como o vértice do
polígono dado que se encontra na mediatriz de Pk-1 e Pk-2. Determine para que
valores de n a sucessão P0, P1, P2, … percorre todos os vértices do polígono

9RFr VDELD«
4XH 232582657-1 p SULPR " (OH WHP  GtJLWRV H p R
PDLRU SULPR FRQKHFLGR QR PRPHQWR )RL GHVFREHUWR HP  GH
VHWHPEUR GH  SRU &XUWLV &RRSHU H 6WHYHQ %RRQH GRLV
SDUWLFLSDQWHV GR *,036 TXH Mi WLQKDP GHVFREHUWR R SULPR
2 -1, R VHJXQGR PDLRU SULPR FRQKHFLGR. O *,036 p XP
30402457

SURMHWR FRRSHUDWLYR QD LQWHUQHW TXH Mi HQFRQWURX  SULPRV


GH 0HUVHQQH 9HMD www.mersenne.org SDUD PDLV LQIRUPDo}HV
LQFOXVLYH FRPR DMXGDU D DFKDU RXWURV SULPRV GH 0HUVHQQH

EUREKA! N°25, 2007

21
Sociedade Brasileira de Matemática

A FÓRMULA DE HERÃO
Fabiano Alberton de Alencar Nogueira

♦ Nível Intermediário

Uma fórmula que sempre exerceu sobre mim um grande fascínio é a


fórmula de Herão para o cálculo da área S de um triângulo qualquer de lados a, b e
c:
S= p ( p − a )( p − b )( p − c ) ,
a+b+c
onde p = é o semi-perímetro do triângulo. Sua dedução, no entanto,
2
apresenta na maioria dos livros uma certa dose de artificialidade. O objetivo deste
artigo é sugerir uma dedução que me ocorreu como sendo mais natural, além de
consideravelmente curta.
A inspiração veio quando estava revirando papéis velhos, alimentando minhas
saudades dos tempos em que competia nas Olimpíadas de Matemática. Numa
dessas sessões de nostalgia, deparei-me com uma questão da Olimpíada Estadual
de Matemática do Rio de Janeiro que propunha que se provasse uma desigualdade
envolvendo as medidas periféricas de um triângulo qualquer e o raio r do seu
círculo inscrito:
1 1 1 1
+ + ≥
( p − a) ( p − b) ( p − c)
2 2 2
r2
Refazendo sua solução, vislumbrei a possibilidade de “fazer as pazes” com a
fórmula de Herão, através do que passo a expor.
Primeiramente, listemos os pré-requisitos necessários à argumentação:
A área S de qualquer triângulo é metade do produto envolvendo um par de seus
lados e o seno do ângulo interno formado por eles.
1
S = ab sin Cˆ
2
b c


a

EUREKA! N°25, 2007

22
Sociedade Brasileira de Matemática

Os segmentos tangentes a um mesmo círculo, traçados pelo mesmo ponto, são


congruentes. A figura a seguir aplica este princípio aos três vértices de um
triângulo no qual foi construído o círculo inscrito, cujo raio é r.

z A
z

r
x r
y
r

C x y B

Na figura acima, as letras x, y e z denotam as medidas dos pares de segmentos que


são congruentes por serem tangentes ao círculo inscrito, traçados respectivamente
pelos vértices C, B e A.
Da mesma figura, retemos os fatos de que CB = x + y e CA = x + z . Também é
imediato que, sendo o perímetro
2 p = AB + CA + CB = ( y + z ) + ( x + y ) + ( z + y ) , temos que p = x + y + z .
A área S de qualquer triângulo é igual ao produto do semi-perímetro pelo raio do
círculo inscrito, ou seja, S = pr .

z A
z

x I
y
r

C x T y B

De fato, olhando para as bissetrizes do triângulo ABC, que concorrem no incentro


I, vemos que AI, BI e CI dividem o interior de ABC em três triângulos, sendo um
deles CBI. O raio r é a altura IT de CBI porque o círculo inscrito tangência o lado
CB no ponto T. Portanto, ao considerarmos que a área de CBI, de base
(x + y )r
CB = x + y e altura r, é dada por que um raciocínio análogo permite
2
EUREKA! N°25, 2007

23
Sociedade Brasileira de Matemática

( y + z)r (x + z )r
concluir que as áreas de ABI e ACI valem, respectivamente e ,
2 2
e que a área de ABC é a soma das áreas de CBI, ABI e ACI, temos:
( x + y ) r ( y + z ) r ( x + z ) r (2 x + 2 y + 2z ) r 2 pr
S= + + = = = pr
2 2 2 2 2

O Teorema de Pitágoras e um pouco de Trigonometria, em particular a fórmula de

duplicação de arcos sin 2α = 2sin α cos α , que será usada com 2α = Cˆ .


Estes são os ingredientes necessários à dedução da fórmula de Herão, numa versão
extremamente simples de se memorizar: S = pxyz !
E aqui vamos nós! Sem precisar dar novamente nome aos bois, temos

1 1
S = ab sin Cˆ = ( x + y )( x + z ) sin 2α ,
2 2

onde α = BCI
ˆ = . Ocorre que, no triângulo retângulo CTI, de catetos x e r,
2

podemos expressar o seno e o coseno de α = em função desses parâmetros,
2
lançando mão do Teorema de Pitágoras:

CI 2 = CT 2 + IT 2 = x 2 + r 2 ⇒ CI = x 2 + r 2 .

I
x2 + r2

r

2
C x T

EUREKA! N°25, 2007

24
Sociedade Brasileira de Matemática

Cˆ IT r Cˆ CT x
Logo sin = = e cos = = . Por outro lado,
2 CI x +r
2 2 2 CI x + r2
2

ˆ ˆ   
sabendo que sin Cˆ = 2sin C cos C , temos sin Cˆ = 2  x r
 2 2
2 xr
= 2 2 .
2 2  x + r  x + r
2 2
 x +r
S
2x
S p
Lembrando que S = pr ⇒ r = , substituímos sin Cˆ = 2
na fórmula
p S
x + 
2

 p
1
da área S = ( x + y )( x + z ) sin Cˆ . A manipulação abaixo encerra a dedução,
2
uma vez que se p = x + y + z , então x = p − c , y = p − b e z = p − a :
S
2 x
1 x ( x + z )( x + y )
S = ( x + z )( x + y )
p
⇒1= ⇒
2 S
2
 S 2

  +x
2 p 2 + x  2

 p p 

 S2   

p   (


⇒ p  2 + x 2  = x  x 2 + xy + xz + zy  
 x x + y + z = xp
)


×p
→ S 2 + p 2 x 2 = p 2 x 2 + pxyz ⇒

⇒S= pxyz = p ( p − a )( p − b )( p − c )

Observações:
É bem possível e provável que algum autor já tenha feito essa dedução em
essência, porém não a encontrei na minha (pobre) bibliografia.
A questão da OEM/RJ (creio ser do ano de 1988) que transcrevi pode ser resolvida
usando como lema uma desigualdade bem “manjada para os alunos olímpicos”:
a 2 + b2 + c 2 ≥ ab + ac + bc , válida para quaisquer reais a, b e c.
O fato S = pr pode e deve ser generalizado para todos os polígonos convexos
circunscritíveis. Curiosamente, o caso particular dos polígonos regulares, onde o
raio r é o apótema a p , é bem mais popular do que o caso geral. Mesmo que se
enfatize o caso particular S = pa p , vale a pena intuir a área do círculo a partir das
substituições a p = r e p = π r .

EUREKA! N°25, 2007

25
Sociedade Brasileira de Matemática

ÁREAS PARA ACHAR RAZÕES DE SEGMENTOS


Cícero Thiago e Marcelo Mendes - Grupo Teorema de Matemática

♦ Nível Avançado

Apresentaremos aqui uma simples, poderosa e útil ferramenta geométrica


para problemas envolvendo razões de segmentos. Como convenção, denotemos
por [Q] a área do polígono Q.

Seja ABC um triângulo e P, um ponto em seu interior. Sejam S = [ABC],


SA = [PBC], SB = [PAC] e SC = [PAB] (veja a figura abaixo, à esquerda). Temos S
= SA + SB + SC.

A A

SC SB C’
B’
P P
SA
B C B A’ C

Agora, prolongue AP até A’ sobre BC, e defina B’ e C’ analogamente (veja figura


acima, à direita). Como triângulos com mesma altura têm áreas proporcionais a
suas bases, temos:

AP [ PAB ] [ PAC ] [ PAB ] + [ PAC ] SC + S B


= = = = .
PA ' [ PBA '] [ PCA '] [ PBA '] + [ PCA '] SA

BP S A + SC CP S A + S B
Analogamente, = e = . Por outro lado, também temos
PB ' SB PC ' Sc
BA ' [ PBA '] [ ABA '] [ PAB ] SC
= = = = .
A ' C [ PCA '] [ ACA '] [ PAC ] S B
CB ' S A AC ' S B
Da mesma forma, = e = .
B ' A SC C ' B SA

EUREKA! N°25, 2007

26
Sociedade Brasileira de Matemática

Vejamos alguns exemplos.

Exemplo 1: Prove o teorema de Ceva: AX, BY, CZ são cevianas concorrentes de um


AZ BX CY
triângulo ABC ⇔ ⋅ ⋅ =1.
ZB XC YA

Solução: Primeiro, suponha que AX, BY, CZ sejam concorrentes. Pela teoria acima,
AZ S B BX SC CY S A AZ BX CY
temos = , = , = e, diretamente, ⋅ ⋅ =1.
ZB S A XC S B YA SC ZB XC YA
AZ BX CY
Reciprocamente, se ⋅ ⋅ = 1 e CZ não passasse pela interseção P de AX
ZB XC YA
e BY, então, sendo Z’ a interseção de CP e AB, teríamos pela primeira parte que
AZ ' BX CY AZ AZ '
⋅ ⋅ = 1 . Portanto = , um absurdo. Logo, CZ passa por P e
Z ' B XC YA ZB Z ' B
AX, BY e CZ são concorrentes.
A

Z
Z’
Y

P
B X C

Exemplo 2: (HUNGRIA/1936) S é um ponto no interior do ∆ABC tal que as áreas dos


triângulos ABS, BCS, CAS são todas iguais. Prove que S é o baricentro de ABC.

Solução: Seja T a área dos triângulos ABS, BCS, CAS. Daí, sendo M, N e P as
BM CN AP T
interseções de AS, BS e CS com os lados opostos, temos = = = =1 ,
MC NA PB T
isto é, M, N e P são os pontos médios dos lados BC, CA e AB e, portanto, S é o
baricentro de ABC.

EUREKA! N°25, 2007

27
Sociedade Brasileira de Matemática

P T N
S T
T
B M C

Exemplo 3: (BANCO IMO/1996) Seja ABC um triângulo acutângulo com circuncentro O


e raio R. Seja A1 ≠ O o ponto de interseção de AO com a circunferência
circunscrita ao triângulo BOC e defina analogamente B1 e C1. Mostre que OA1 ⋅
OB1 ⋅ OC1 ≥ 8R3. Quando ocorre a igualdade?

Solução:
Sejam D, E e F as interseções de AO, BO e CO com BC, CA e AB,
respectivamente. É fácil ver que AO = BO = CO = R. Usando as relações provadas
acima temos que:
AO [ AOB ] + [ AOC ] BO [ AOB ] + [ BOC ]
= , =
OD [ BOC ] OE [ AOC ]
CO [ AOC ] + [ BOC ]
= .
OF [ AOB]
l = C BO
Faça [ AOB ] = x,[ AOC ] = y ,[ BOC ] = z . É fácil perceber que DCO l = C lAO
l C e ∆DCO , logo ∆OA C ∆OCD. Com isso,
l é comum a ∆OA
e que COD 1 1

R OA R2
= 1 ⇒ OA1 = e analogamente,
OD R OD
R2 R2
OB1 = e OC1 = . Então,
OE OF
R6 R R R OA BO C O
O A1 .O B1 .O C1 = = ⋅ ⋅ ⋅ R3 = ⋅ ⋅ ⋅ R3 =
O D .O E .O F OD OE OF OD OE OF
( x + y )( x + z )( y + z ) 3
= ⋅R
xyz

EUREKA! N°25, 2007

28
Sociedade Brasileira de Matemática

2 xy 2 yz 2 zx 8 xyz 3
≥ R3 = R = 8R3 . A igualdade ocorre quando x = y = z .
xyz xyz
Pelo exemplo 2, O tem que ser baricentro para acontecer a igualdade.

B D C

A1

PROBLEMAS PROPOSTOS
1. (IME/1990;AIME/1985) Seja P um ponto no interior de um triângulo ABC,
dividindo-o em seis triângulos, quatro dos quais têm áreas 40, 30, 35 e 84, como
mostra a figura. Calcule a área do triângulo ABC.

84
P
35
40 30
B C

EUREKA! N°25, 2007

29
Sociedade Brasileira de Matemática

2. (IMO/1961) Considere triângulo P1P2P3 e um ponto P no interior do triângulo. As


retas P1P, P2P, P3P intersectam os lados opostos nos pontos Q1, Q2, Q3,
PP P P P P
respectivamente. Prove que dos números 1 , 2 , 3 , ao menos um é ≤ 2 e
PQ1 PQ2 PQ3
ao menos um é ≥ 2.

3. (AIME/1992) No triângulo ABC, A’, B’, C’ estão sobre os lados BC, AC e AB,
respectivamente. Dado que AA’, BB’, CC’ são concorrentes no ponto O e que
AO BO CO AO BO CO
+ + = 92 , encontre o valor de ⋅ ⋅ .
OA ' OB ' OC ' OA ' OB ' OC '

4. Seja P um ponto no interior do ∆ABC. Sejam D, E, F as interseções de AP, BP,


CP com BC, CA, AB, respectivamente. Prove que
PA PB PB PC PC PA
⋅ + ⋅ + ⋅ ≥ 12 .
PD PE PE PF PF PD

5. No triângulo ABC, os pontos L, M, N estão sobre BC, AC, AB, respectivamente,


e AL, BM, CN são concorrentes.
PL PM PN
Encontre o valor numérico de + + .
AL BM CN
AP BP CP
Encontre o valor numérico de + + .
AL BM CN

6. (IBERO/1985) Se AD, BE, CF são cevianas concorrentes no circuncentro O do


1 1 1 2
∆ABC, demonstre que + + = .
AD BE CF R
Sugestão: Usar problema 5

7. Em um ∆ABC, AD, BE, CF são concorrentes no ponto P tal que AP = PD = 6,


EP = 3, PB = 9 e CF = 20. Qual é a área do ∆ABC?

8. Em um triângulo ABC, seja S o ponto médio da mediana correspondente ao


vértice A e Q, o ponto de interseção de BS com o lado AC. Mostre que BS = 3QS.

9. Seja ABC um triângulo e P um ponto em seu interior tal que AP, BP e CP


intersectam os lados BC, CA e AB nos pontos D, E e F, respectivamente. Se AP =
a, BP = b, CP = c, PD = PE = PF = 3 e a + b + c = 43. Determine abc.

EUREKA! N°25, 2007

30
Sociedade Brasileira de Matemática

PROBLEMAS SOBRE PONTOS


Davi Máximo (UFC) e Samuel Feitosa (UFC)

♦ Nível Avançado

Distribuir pontos num plano ou num espaço é uma tarefa que pode ser
realizada de forma muito arbitrária. Por isso, problemas sobre pontos podem ser
de diversas naturezas. Nesse artigo, trataremos as principais técnicas para resolver
esses tipos de problemas,

1. Fecho Convexo
Pense no seguinte: dados n pontos num plano, podemos escolher alguns
deles formando o único polígono convexo que contém, junto com seu bordo e seu
interior, todos os n pontos. Tal afirmação pode ser provada por indução (que alias,
é uma ferramenta que sempre deve ser lembrada em problemas de matemática
discreta em geral). Tal polígono é chamado o fecho convexo desses n pontos.
Vamos ver que tão pouco já nos ajuda bastante em alguns problemas sobre pontos.

PROBLEMA 1
Seja S um conjunto finito de pontos, não havendo três colineares, tal que dados
quaisquer 4 pontos de S eles formam um quadrilátero convexo. Mostre que S é um
conjunto de vértices de um polígono convexo.

SOLUÇÃO:
Seja H o fecho convexo de S.
Suponha um ponto P de S no interior de
C H. Escolha uma triangulação de H
(assim como o fecho convexo, é
simples provar que todo polígono
convexo pode ser dividido por
triângulos tendo como lados diagonais
ou lados do polígono, tente indução).
. Assim, P fica no interior de algum
P triângulo ABC. Logo, o quadrilátero
ABCP não é convexo, absurdo!
H Portanto, S não pode ter pontos no
B
interior do seu fecho convexo, donde S
A é convexo, já que S não contém três
pontos colineares.
Os próximos problemas são resolvidos similarmente.

EUREKA! N°25, 2007

31
Sociedade Brasileira de Matemática

PROBLEMA 2
Mostre que dados 5 pontos, não três colineares, existe um quadrilátero convexo
com vértices nesses pontos.

PROBLEMA 3
Mostre que dado qualquer conjunto finito de pontos no plano existe uma reta por
dois destes pontos que divide o plano em dois semi-planos de modo que um desses
semi-planos não contém nenhum ponto do conjunto.

PROBLEMA 4
(Lista Cone Sul 2001) É possível que a reunião de um número finito de
quadriláteros não convexos seja um polígono convexo?
Podemos definir fecho convexo para um conjunto X qualquer do plano. Ele é o
“menor” conjunto convexo que contém X.

Definição: O fecho convexo H de X é a interseção de todos os conjuntos convexos


do plano que contém X.
Deixamos para o leitor a verificação dos seguintes fatos:

i-) H é convexo

ii-) No caso de um conjunto com um número finito de pontos esta definição


implica que H é um polígono convexo cujos vértices pertencem a este conjunto.
PROBLEMA 5
Dado um conjunto de N discos de raios unitários. Esses círculos podem se
intersectar (mas não coincidir). Mostre que existe um arco de comprimento maior

ou igual a pertencendo à circunferência de um desses discos que não é
N
coberto por nenhum outro disco.
(IDÉIA DA SOLUÇÃO) Consideremos o fecho
convexo H desse conjunto de discos. Um arco
que esteja na borda do fecho convexo não pode
ser coberto por outro disco. Mostre que a
“junção” de todos os arco no bordo de H é um
círculo de raio unitário. Como este círculo tem
perímetro 2π e no máximo juntamos N arcos,
pelo menos um dos arcos da junção é maior ou

igual a .
N

EUREKA! N°25, 2007

32
Sociedade Brasileira de Matemática

PROBLEMA 6
(OBM 96) Existe um conjunto A de n pontos ( n ≥ 3 ) em um plano tal que:
i) A não contém três pontos colineares;
ii) dados quaisquer três pontos pertencentes a A, o centro da circunferência que
contém estes pontos também pertence a A?
Os próximos dois problemas são de IMO e podem ser resolvidos usando só fecho
convexo (na realidade, muita raça também, que é algo imprescindível em qualquer
problema, principalmente de IMO).

PROBLEMA 7
(IMO 99/1) Determine todos os conjuntos finitos S de pontos do plano com pelo
menos três elementos que satisfazem a seguinte condição:
Para quaisquer dois pontos distintos A e B de S, a mediatriz do segmento AB é um
eixo de simetria de S.
(Veja a solução desse problema por Fabrício Siqueira Benevides na Eureka! N°.6)

PROBLEMA 8
(IMO 95/3) Determine todos os inteiros n > 3 para os quais existem n pontos
! !
A1 , A2 , , An no plano, e números reais r1 , r2 , rn satisfazendo as condições:
não há três pontos Ai , A j , Ak colineares;
para cada tripla i ,j, k (1 ≤ i < j < k ≤ n ) o triângulo Ai A j Ak tem área ri + rj + rk .

SOLUÇÃO:
Vamos fazer o caso n ≥ 5 . Considere, dentre os n pontos, cinco pontos
A1 , A2 , A3 , A4 , A5 e seu fecho convexo . Temos três casos:
1° Caso: O Fecho Convexo é um triângulo.
Podemos assumir que tal fecho é o triângulo A1 A2 A3 , A4 e A5 estão no interior
de A1 A2 A3 , com A5 fora de A1 A2 A4 e A4 fora de A1 A2 A4 (faça uma figura).
Podemos supor que os triângulos A1 A2 A4 e A2 A3 A5 têm interiores disjuntos.
Seguindo nossa notação para áreas, temos:
r1 + r2 + r3 = [ A1 A2 A3 ] > [ A1 A2 A4 ] + [ A2 A3 A5 ] = r1 + r2 + r4 + r2 + r3 + r5 , donde
0 > r4 + r2 + r5 = [ A2 A4 A5 ], absurdo!

2° Caso: O Fecho Convexo é um quadrilátero


Suponha A5 no interior do fecho convexo A1 A2 A3 A4 . Note que
[ A1 A2 A3 A4 ] = [ A1 A2 A3 ] + [ A1 A3 A4 ] = [ A1 A2 A4 ] + [ A2 A3 A4 ]

EUREKA! N°25, 2007

33
Sociedade Brasileira de Matemática

e portanto,
(r1 + r2 + r3 ) + (r3 + r4 + r1 ) = (r1 + r2 + r4 ) + (r2 + r4 + r3 ) ⇒ r1 + r3 = r2 + r4 .
Logo, 2[ A1 A2 A3 A4 ] = 3(r1 + r2 + r3 + r4 ) . Também,
[ A1 A2 A3 A4 ] = [ A1 A2 A5 ] + [ A2 A3 A5 ] + [ A3 A4 A5 ] + [ A4 A1 A5 ]
Logo, temos r5 = − (r1 + r2 + r3 + r4 ) 8 = − [ A1 A2 A3 A4 ] 12 < 0 . Agora, observe
que como A1 , A3 , A5 não são colineares, podemos supor um dos lados de
∠A1 A3 A5 < 180° . Então, um dos quadriláteros A1 A5 A3 A4 , A1 A5 A3 A2 é
convexo. Digamos, A1 A5 A3 A4 convexo. Então, temos r1 + r3 = r4 + r5 e
portanto, ficamos com r2 = r5 . Analogamente, usando que A2 , A4 , A5 não são
colineares, temos r5 = r1 ou r3 . Assim, três dos números r1 , r2 , r3, r4 , r5 são
negativos, obtendo uma área negativa. Absurdo!

3° Caso: O Fecho convexo é um pentágono


Suponha que r1 seja o menor deles. Traçando uma paralela l por A1 à reta A3 A4 .
Como [ A1 A3 A4 ] = r1 + r3 + r4 ≤ r2 + r3 + r4 = [ A2 A3 A4 ] , A2 pertence a l ou ao
semiplano definido por l oposto ao A3 A4 e, analogamente A5 . Como A1 , A2 , A5
não podem estar todos em l , temos ∠A2 A1 A5 > 180° , absurdo! .
Logo, n ≤ 4 . Um exemplo para n = 4 é um quadrado A1 A2 A3 A4 de lado 1 com
r1 = r2 = r3 = r4 = 1 6 .
Finalizamos essa parte com dois problemas bonitinhos.

PROBLEMA 9
(USAMO 2005) Seja n um inteiro positivo maior que 1. Suponha que são dados
2n pontos no plano, não havendo três colineares. Suponha que n dos 2n são
pintados de azul e os outros n de vermelho. Uma reta no plano é dita balanceada
se passa por um ponto azul e um ponto vermelho, e o número de pontos azuis em
cada um de seus lados é igual ao número de pontos vermelhos. Prove que existem
pelo menos duas retas balanceadas.
DICA: Prove que cada ponto do fecho convexo dos pontos está em pelo menos uma
reta balanceada.

PROBLEMA 10
(Kömal 2002) Dado um conjunto qualquer de pontos no plano, não contendo três
colineares, prove que é possível colorir os pontos com duas cores (azul e

EUREKA! N°25, 2007

34
Sociedade Brasileira de Matemática

vermelho) tal que todo semiplano contendo pelo menos três pontos do conjunto
contenha pelo menos um ponto de cada cor.

2. Princípio das Casas dos Pombos

O princípio da casas dos Pombos, PCP, é importante e deve ser lembrado sempre.
Ele é usado para provar existências (“...se n + 1 pombos estão em n casas, existe
pelo menos uma casa contendo pelo menos dois pombos...”). Nossos dois
primeiros problemas dessa sessão são apenas versões dificultadas daquele
exercício clássico do PCP. : dados cincos pontos num quadrado unitário, existem
dois cuja distância entre eles é menor que 2 .
2

PROBLEMA 11
(Japão 97) Prove que entre quaisquer dez pontos no interior de um círculo de
diâmetro 5, existem dois cuja distância entre eles é menor que 2.

PROBLEMA 12
(Coréia 97) Prove que entre quaisquer quatro pontos no interior de um círculo
unitário, existem dois deles cuja distância é menor que 2.

PROBLEMA 13
(Rioplatense 2002) Daniel escolhe um inteiro positivo n e diz a Ana. Com esta
informação, Ana escolhe um inteiro k e diz a Daniel. Daniel traça então n
circunferências em um papel e escolhe k pontos distintos com a condição de que
cada um deles pertença a alguma das circunferências que traçou. Em seguida,
apaga as circunferências que traçou, sobrando visíveis apenas os k pontos que
marcou. A partir desses pontos, Ana deve reconstruir pelo menos uma das
circunferências que Daniel traçou. Determinar qual o menor valor de k que
permite Ana alcançar seu objetivo independente de como Daniel escolha as n
circunferências e os k pontos.

SOLUÇÃO:
O valor mínimo é k = 2n 2 + 1 .
1° Passo: k = 2n 2 + 1 é suficiente.
Se são dados 2n 2 + 1 pontos marcados por Daniel, como estes pontos são
distribuídos em n circunferências, pelo Princípio das Casas dos Pombos, pelo
menos 2n+1 deles estão em uma mesma circunferência traçada por Daniel. Então,
se Ana traça todas as circunferências determinadas por estes 2n 2 + 1 , haverá uma

EUREKA! N°25, 2007

35
Sociedade Brasileira de Matemática

delas, digamos Γ , com pelo menos 2n+1 dos pontos. Como estes 2n+1 pontos
provém das n circunferências de Daniel, três deles estão numa mesma
circunferência traçada por ele, digamos ζ . Logo, ζ e Γ têm pelo menos três
pontos em comum, e portanto, são a mesma circunferência (abusando da
notação: ζ = Γ ). Assim, Ana consegue determinar umas das circunferências
traçadas por Daniel.
2° Passo: Se k < 2n 2 + 1 , Daniel pode traçar circunferências e escolher k pontos
de modo a tornar impossível para Ana determinar tais circunferências.
Basta considerar k = 2n 2 :

!
Traçamos n circunferências concêntricas
ζ 1 Γ1 , Γ2 , , Γn e outras n circunferências
,! , ζ
ζ 2
ζ 1 ,ζ 2 n duas a duas disjuntas, de
modo que ζ i corta Γ j em dois pontos
distintos, para i, j = 1,2,...,n. Há
Γn
Γ2 exatamente 2n 2 pontos de intersecção.
Se Daniel marca estes pontos e apagas
Γ1
suas circunferências, Ana não conseguirá
reconstruir com certeza nenhuma das
ζ

!
n circunferências, pois Daniel pode ter
traçado inicialmente tanto Γ1 , Γ2 , , Γn
quanto ζ 1 , ζ 2 , !, ζ n .
PROBLEMA 14
(Rioplatense 1999) Dois jogadores A e B disputam o seguinte jogo: A escolhe um
ponto de coordenadas inteiras do plano e o pinta de verde; em seguida B escolhe
10 pontos de coordenadas inteiras, ainda não coloridos e os pinta de amarelo. O
jogo continua assim com as mesmas regras: A e B escolhem um e dez pontos ainda
não coloridos e os pintam de verde e amarelo, respectivamente.
(a) O objetivo de A é obter 1112 pontos verdes que sejam as interseções de 111
retas horizontais e 111 retas verticais (i.e., paralelas aos eixos de coordenadas). O
objetivo de B é impedir-lhe. Determine qual dos jogadores tem uma estratégia
vencedora que lhe assegura seu objetivo.
(b) O objetivo de A é obter quatro pontos verdes que sejam vértices de um
quadrado de lados paralelos aos eixos coordenados. O objetivo de B é impedir-lhe.
Determine qual dos jogadores tem uma estratégia vencedora que lhe assegura seu
objetivo.

EUREKA! N°25, 2007

36
Sociedade Brasileira de Matemática

3. Idéias Extremais
Na matemática em geral, problemas de existência são muito comuns e
importantes. São aqueles problemas que nos pedem para provar que a existência
de alguma coisa. Na seção anterior, não explicitamente, nos deparamos com
problemas desse tipo. E não foi para vender o artigo que iniciamos ambas as
seções falando da importância dessas idéias, mas pelo o fato de que o PCP e o
Princípio Extremal juntos são as ferramentas mais indispensáveis para o ataque
desses problemas.
Mas afinal, que Princípio Extremal é esse? Digamos que temos um problema onde
nos é pedido para provar a existência de um elemento satisfazendo uma certa
propriedade P. Então, nós escolhemos um elemento que satisfaz maximalmente ou
minimalmente, ou seja, extremalmente (será que acabamos de inventar essas
palavras?) uma outra propriedade Q, não acidentalmente ligada com a desejada
propriedade P. O que será que isso nos dá? Vejamos alguns problemas.

PROBLEMA 15
(Austrália 91) São dados n ≥ 3 pontos no plano tais que a área de um triângulo
formado por quaisquer três deles é no máximo 1. Prove que os n pontos estão em
um triângulo de área no máximo 4.

!
SOLUÇÃO:
Sejam P1 , P2 , , Pn os n pontos. Dentre os triângulos considerados, seja ABC o
de maior área (o cara com a propriedade Q). Considere por A uma reta a paralela a
BC. Sendo assim, qualquer outro ponto Pi
deve estar no mesmo semiplano de B e C
definido por A, pois do contrário teríamos
Z
um absurdo [ PBC ] > [ ABC ] (aqui [X],
denota a área de X) . Analogamente,
considerando as retas b e c por B e C
A B paralelas a AC e AB, respectivamente,
concluímos que todos os pontos P devem
b
estar no triângulo XYZ (acompanhe a
figura ao lado). Como,
Y C c X [ XYZ ] = 4[ ABC ] < 4 ⋅ 1 = 4 , o resultado
segue. (isto é, XYZ satisfaz a propriedade
P).

EUREKA! N°25, 2007

37
Sociedade Brasileira de Matemática

PROBLEMA 16
(Putnam 1979) Sejam 2n pontos no plano escolhidos de modo que quaisquer 3
não são colineares, n deles são pintados de vermelho e n deles são pintados de
azul. Prove que é possível parear os pontos usando segmentos ligando cada ponto
vermelho a exatamente um ponto azul de modo que esses segmentos não se
cortem.

SOLUÇÃO:
Existem n 2 maneiras de parear esses pontos. É claro que alguns desses
pareamentos não cumprem a condição do enunciado. Olhemos em cada
pareamento a soma dos seus segmentos. Escolha o pareamento que tem soma
mínima. Suponha que nele existem dois segmentos AB e CD que se cortam
(com A e C vermelhos)
Pela desigualdade triangular temos:
A C
AO + OD > CD 
 ⇒ AB + CD > AD + CB
OB + OC > CB 
O
Logo se trocarmos AB e CD por AD e
D CB diminuiremos nossa soma. Assim
neste pareamento não temos dois
segmentos que se cortam.
B

PROBLEMA 17
(Teorema de Sylvester) Um conjunto S de pontos no plano tem a seguinte
propriedade: qualquer reta passando por 2 pontos passa também por um terceiro.
Mostre que todos os pontos estão sobre uma reta.

SOLUÇÃO:
Considere o conjunto L das retas que passam por dois pontos de S. Cada ponto de
S tem uma distância associada a cada reta de L. Como L e S são conjuntos finitos
então temos um número finito distâncias. Considere o par (l , s ) do ponto
s ∈ S e l ∈ L com a menor distância não nula associada. Como l passa por dois
pontos de S então deverá passar por um terceiro. Pelo menos dois pontos de S ,
digamos A e B, deverão estar em um “mesmo lado” de l determinado por P (pé
da perpendicular de s até l ).

EUREKA! N°25, 2007

38
Sociedade Brasileira de Matemática

Suponhamos que A esteja entre B e P. Seja


s m a reta que passa por B e s então:
distância(A,m) ≤ distância(P,m) <
< distância(s,l) Absurdo!

l B
P A
m
Assim todas as distâncias associadas têm que ser zero! Todos os pontos são
colineares!

A seguir, veja como usar o Teorema de Sylvester.

PROBLEMA 18
São dados ( N ≥ 3) pontos no plano, nem todos colineares. Mostre que são
necessários pelo menos n retas para unir todos os possíveis pares de pontos.

SOLUÇÃO:
Vamos tentar usar indução. Se N = 3 os três pontos formarão um triângulo. As
retas suportes dos três lados desse triângulo satisfazem nossa afirmação. Suponha
que a afirmação seja válida para N = k . Considere um conjunto T de N = k + 1
pontos. Como nem todos esses pontos estão sobre uma mesma reta decorre do
teorema de Sylvester que existe uma reta que passa por apenas dois pontos (A e B)
do conjunto. Pelo menos um dos conjuntos T \ { A} ou T \ {B} não poderá ter
todos os seus k pontos colineares. Então pela hipótese teremos pelo menos k
retas, mas a reta AB não foi contada, assim a afirmação também é verdadeira
para N = k + 1 .

PROBLEMA 19
Dado um conjunto finito S de pontos no plano onde não existem quatro sobre um
mesmo círculo e nem todos estão sobre uma mesma reta. Mostre que existe um
círculo que passa por três desses pontos e não contém nenhum ponto de S em seu
interior.

PROBLEMA 20
(Ibero 93) Prove que para qualquer polígono convexo de área 1, existe um
paralelogramo de área dois que o contém.

EUREKA! N°25, 2007

39
Sociedade Brasileira de Matemática

PROBLEMA 21
(OBM 94) Considere todos os círculos cujas circunferências passam por três
vértices consecutivos de um polígono convexo. Prove que um destes círculos
contém todo o polígono.

PROBLEMA 22
(Rioplatense 97) Agustina e Santiago jogam o seguinte jogo sobre uma folha
retangular:
Agustina diz um número n. Santiago, então marca n pontos sobre a folha . Em
seguida, Agustina escolhe alguns dos pontos marcados por Santiago. Santiago
ganha o jogo se consegue desenhar um retângulo com lados paralelos aos da folha,
que contenha todos os escolhidos por Agustina e nenhum dos restantes. Do
contrário, Agustina ganha.
Qual o menor número que deve escolher Agustina para assegurar-se da vitória,
independente como jogue Santiago?

PROBLEMA 23
(Rússia 2000) São dados 2n+1 segmentos em uma linha reta. Cada segmento
intersecta pelo menos n outros. Prove que um desses segmentos intersecta todos os
outros.

PROBLEMA 24
(Japão 2002) É dado um conjunto S de 2002 pontos no plano xy, não havendo dois
deles com a mesma abscissa x ou ordenada y. Para quaisquer dois desses pontos P
e Q , considere o retângulo cuja diagonal é PQ e cujos lados são paralelos aos
eixos. Denotemos por WPQ o número de pontos de S no interior desse retângulo,
sem contar com P e Q. Determine o maior valor N possível que satisfaz: não
importa como os pontos de S estão arranjados, existe pelo menos um par P e Q
deles com WPQ ≥ N .

PROBLEMA 25
Dados 2n + 2 pontos no plano, não havendo três colineares, prove que existem
dois deles que determinam uma reta que, dos 2n pontos restantes, separa n em um
semi-plano e os outros n no outro semi-plano.

PROBLEMA 26
(Banco IMO 93) Dados 2n+3 pontos num plano, não havendo três colineares nem
quatro concíclicos, prove que podemos escolher três deles de modo o círculo
passando por estes tem n dos pontos restantes no seu interior e n no exterior.

EUREKA! N°25, 2007

40
Sociedade Brasileira de Matemática

SOLUÇÃO: Basta considerar a figura abaixo. Deixamos os detalhes para o leitor.

A2 n +1

A2 n+ 2 = P
A2 n +3 = Q

A1
An +1

PROBLEMA 27
São dados n pontos num plano. Em cada ponto médio de um segmento ligando
dois desses pontos, colocamos um marcador. Prove que pelo menos 2n − 3
marcadores são utilizados.

4. Problemas de Coberturas
Nos problemas sobre pontos até agora, ficou claro que um pouco de geometria
(sintética, analítica, trigonométrica ou utilizando o plano complexo) pode ser útil.
Finalizaremos esse artigo com uma seção falando um pouco disso, em particular,
fazendo coberturas com círculos.

PROBLEMA 28
Seja C um círculo de raio 16 e A um anel tendo raio interno 2 e raio externo 3.
Agora suponha que um conjunto S de 650 pontos é selecionado dentro de C. Prove
que, não importa como os pontos de S são selecionados dentro de C, o anel A
pode ser colocado de modo a cobrir pelo menos 10 pontos de S.

SOLUÇÃO:
Queremos mostrar que existe um ponto X no plano que possui uma distância
maior que 2 e menor que 3 à pelo menos 10 pontos de S. Sobre cada ponto de S
coloque um anel A. Basta mostrarmos que existe um ponto X que está no interior
de pelo menos 10 desses anéis. As interseções desses anéis produzem pequenas
regiões (veja a figura).

EUREKA! N°25, 2007

41
Sociedade Brasileira de Matemática

Veja que existem três pequenas regiões


que estão em dois anéis e uma que está
em três. Somando as áreas dos três anéis
contaremos três regiões duas vezes e uma
três vezes. Somando a área de cada anel
temos
650.(9π − 4π ) = 3250π .Aumentando o
raio do círculo C para 19 poderemos
cobrir todos esses anéis. Se cada pequena
região foi contada no máximo 9 vezes
contaremos no máximo 9 vezes a área
desse novo círculo , ou seja,
9.19 2 π = 3249π < 3250π .
Assim existirá uma pequena região contida em pelo menos 10 anéis. Basta
escolhermos um ponto X dessa região.

PROBLEMA 29
(Teste de Seleção da Romênia para a IMO – 1978) M é um conjunto de 3n pontos
no plano tal que a maior distância entre quaisquer dois desses pontos é 1 unidade.
Prove que:

a. Para quaisquer 4 pontos de M, a distância entre algum par de pontos é pelo


1
menos
2
3
b. Algum círculo de raio ≤ cobre todo o conjunto M.
2
c. Existe algum par entre os 3n pontos de M cuja distância entre eles é no máximo
4
(3 n − 3 )

SOLUÇÃO:
a. Vamos tentar arranjar um triângulo não acutângulo em M. Considere o fecho
convexo de quatro pontos de M. Podemos ter um quadrilátero (degenerado quando
três pontos forem colineares) ou um triângulo com um ponto de M em seu interior.
No caso do quadrilátero como pelo menos um dos quatro ângulos internos é
≥ 90 , basta escolhermos o vértice com este ângulo e os adjacentes a ele. No caso
EUREKA! N°25, 2007

42
Sociedade Brasileira de Matemática

do triângulo vamos olhar para o ponto no interior. Esse ponto olha para os três
lados do triângulo com ângulos que somados resultam em 360 , . Pelo menos um
deles é ≥ 120 , . Seja XYZ um triângulo com ∠XYZ ≥ 90 , . Pela lei dos cossenos
1
temos: y 2 = x 2 + z 2 − 2 xz cos XYZ ≥ x 2 + z 2 . Como y ≤ 1 ⇒ x ou z ≤ .
2
b. Seja r = AB a maior distância entre dois pontos de M. Tracemos círculos de
raio r centrados em A e B. M deverá estar contido em cada um desse círculos.
Então M deverá estar contido na região de interseção entre eles. Tracemos um
3r 3
círculo de raio ≤ centrado no ponto médio C de AB. Veja que este novo
2 2
círculo cobre a região de interseção.
c. Vamos usar a mesma idéia do problema dos
Y anéis. Se dois ponto de M estão em um círculo
de raio r então a distância entre eles não pode
ser maior ou igual a 2r . Então nosso objetivo
é mostrar que existem dois pontos de M
A C B 2
dentro de um círculo de raio
(3 n − 3 )

M
X

3
Seja C um círculo de raio que cobre M. Centrado em cada ponto de M
2
tracemos um círculo de raio r . Suponha que Z é um ponto na interseção de dois
desses círculos. Então o círculo de centro Z e raio r cobre dois pontos de M(os
centros dos círculos que cobriam Z). Como mostrar pelo menos dois desses
2
círculos que traçamos irão se intersectar para r = ? Vamos aumentar
(3 n − 3 )
3
o raio de C e obter um círculo D de raio + r com mesmo centro. Todos esses
2
círculos estarão contidos em D. Se a área de D for menor que a soma das áreas de
cada círculo com certeza pelo menos dois deles terão interseção. Mas isso
2
 3 
acontece se 3n ⋅ π r 2 < π  + r  . Agora basta fazermos o estudo do sinal. Como
 2 
 

EUREKA! N°25, 2007

43
Sociedade Brasileira de Matemática

3+3 n
a maior raiz é e 3n − 1 > 0 se
2(3n − 1)
3 +3 n 3
r> OK! ⇔ r > . (Veja que podemos melhorar um pouco
2(3n − 1) 2(3 n − 3)
a cota do problema, pois 2 > 3 ).
2

PROBLEMA 30
(Ibero 97) Seja P = {P1 , P2 ,..., P1997 } um conjunto de 1997 pontos no interior de
um círculo de raio 1, com P1 sendo o centro do círculo. Para k = 1,2,...,1997 ,
seja x k a distância de Pk ao ponto de P mais próximo de Pk . Mostre que:
x1 + x 2 + ... + x1997 ≤ 9 .
2 2 2

SOLUÇÃO:
Note que x k ≤ 1 , para todo k.
Para cada ponto Pk , considere uma
xk
circunferência Γk de centro Pk e raio .
2
P1 Sendo assim, todos essas circunferências se
Pn tocam em no máximo 1 ponto e estão no
interior de uma circunferência de centro P1 e

raio 3/2. Logo, [Γ1 ] + [Γ2 ] + "+[Γ ] ≤ π (32)


k
2

donde:

" + π  x ( ) "+ x


2 2 2
π  x1  + π  x 2  + n  ≤ π 3 2 ⇒ x2 + x2 +

2
≤9.
 2  2 2 2 1 2 n

PROBLEMA 31
(IMO 89) São dados n e k inteiros positivos e um conjunto S de n pontos no plano
tais que
(i) não há três pontos em S colineares,
(ii) Para qualquer ponto P de S existem pelo menos k pontos de S eqüidistantes de
P.
1
Prove que k < + 2n .
2

EUREKA! N°25, 2007

44
Sociedade Brasileira de Matemática

PROBLEMA 32
(Ibero 98) Encontre o maior inteiro n para o qual existem pontos P1 , P2 ,..., Pn no
plano e números reais r1 , r2 ,..., rn tais que a distância entre Pi e Pj é ri + r j .

Referências Bibliográficas:

[1] Revista Eureka! N°6.


[2] Ross Honsberger, Mathematical Gems Vol.I, The Dolciani Mathematical Expositions,
MAA.
[3] Andreescu, Feng, Mathematical Olympiads: Olympiad problems from around the
world, 1999-2000, MAA 2000.
[4] Marcin Kuczma , International Mathematical Olympiads, 1986-1999, MAA 2003.
[5] www.mathlinks.ro

EUREKA! N°25, 2007

45
Sociedade Brasileira de Matemática

POLINÔMIOS SIMÉTRICOS
Carlos A. Gomes, UFRN, Natal – RN.

♦ Nível Avançado

Uma ferramenta bastante útil na resolução de problemas algébricos de


fatoração, na resolução de sistemas de equações não lineares, na resolução de
algumas equações irracionais são as funções polinomiais simétricas, que apesar de
seu grande poder algébrico são pouco divulgadas entre os nossos alunos. A
finalidade deste breve artigo é exibir de modo sucinto como estas ferramentas
podem ser úteis na resolução de alguns problemas olímpicos.

I. Polinômios Simétricos

Um polinômio f, a duas variáveis x, y, é dito simétrico quando f(x, y) = f (y, x) para


todos os valores x, y.

Exemplos:
a) σ1 = x + y e σ2 = x · y, são evidentemente polinômios simétricos (chamados
polinômios simétricos elementares).

b) Os polinômios da forma Sn = xn + yn, com n ∈  também são simétricos. Um


fato importante a ser observado é que um polinômio simétrico f(x, y) pode ser
representado como um polinômio em função de σ 1 e σ 2. Vejamos:

Se Sn = xn + yn, n ∈ , (n ≥ 2), então:


Sn = xn + yn = (x + y) (xn–1 + yn–1) – xy (xn – 2 + yn – 2) = σ 1 · Sn – 1 – σ 2 · Sn – 2 (n ≥ 2)
Mas,
S0 = x0 + y0 = 1 + 1 = 2
S1 = x1 + y1 = x + y = σ 1

Assim temos que:


S0 = 2
S1 = σ1
S2 = σ1 · S1 –σ2 · S0 = σ1 · σ1 –σ2 · 2 = σ12 – 2σ2
S3 = σ1 · S2 –σ2 · S1 = σ1 (σ12 – 2σ2) –σ2 · σ1 = σ13 – 3σ1 · σ2

E daí usando a lei de recorrência Sn = σ1 Sn – 1 – σ2 Sn – 2 (n ≥ 2) podemos


determinar Sn em função de σ1 e σ2 para qualquer número natural n.

EUREKA! N°25, 2007

46
Sociedade Brasileira de Matemática

Agora para garantirmos a afirmação anterior que todo polinômio simétrico f(x, y)
pode ser representado como um polinômio em σ1 e σ2 observemos o seguinte fato:

Num polinômio simétrico f(x, y) para os termos da forma a . xK . yK não temos


nenhum problema pois a · xK · yK = a(x · y)K = a · σ 2K. Agora com os termos da
forma b · xi · yK, com i < k devemos observar o seguinte fato: Como, por hipótese,
f(x, y) é simétrico se b · xi · yk, com i < k estiver presente em f(x, y) temos que b ·
xk · yi também deve estar presente em f(x, y), visto que deve ser satisfeita a
condição f(x, y) = f(y, x). Assim se agruparmos os termos b · xi · yk + b · xk · yi
(i < k) temos que:

b · xi · yk + b · xk · yi = b · xi · yi (xk - i + yk - i) = b · σ2i · Sk – i ,

mas como já mostramos anteriormente Sk – i pode ser escrito como um polinômio


em σ1 e σ2, pois k – i ∈ , visto que i < k.

II. Exemplos Resolvidos

01. (Funções simétricas elementares a 3 variáveis)

Definido:
σ1 = x + y + z
σ2 = xy + xz + yz
σ3 = x · y · z
Sn = xn + yn + zn, com n ∈  (n ≥ 2)

Mostre que:
a) Sn = σ1 · Sn – 1 –σ2 · Sn – 2 + σ3 · Sn – 3 (n ≥ 3 , com n∈ )
b) S3 = σ13 – 3σ1σ2 + 3σ3

Resolução:
Observe inicialmente que:
xn + yn + zn = (x + y + z) (xn – 1 + yn –1 + zn – 1) – (xy + xz + yz) (xn – 2 + yn – 2 + zn – 2) +
xyz (xn – 3 + yn – 3 + zn – 3)

e daí temos que:


Sn = σ1 · Sn – 1 –σ2 · Sn – 2 + σ3 · Sn – 3 (n ≥ 3, com n ∈ )
Agora temos que:

EUREKA! N°25, 2007

47
Sociedade Brasileira de Matemática

S0 = x0 + y0 + z0 = 1 + 1 + 1 = 3
S1 = x + y + z = σ1
S2 = x2 + y2 + z2 = (x + y + z)2 – 2 (xy + xz + yz) = σ12 – 2σ2
Agora fazendo n = 3 temos na lei de recorrência Sn = σ1 · Sn – 1 –σ2 · Sn – 2 + σ3 · Sn
– 3 temos que:
S3 = σ1 · S2 –σ2 · S1 + σ3 · S0 = σ1 (σ12 – 2σ2) –σ2 · σ1 + σ3 · 3
S3 = σ13 – 3σ1 · σ2 + 3σ3

02. a) Fatore x3 + y3 + z3 – 3xyz

Resolução:
Essa velha e manjada questão continua ainda hoje pegando alguns bons
professores e alunos. A sua solução pelos métodos tradicionais envolve uma boa
dose de atenção e de paciência para aplicar velhos “truques” de fatoração, por
outro lado ela é imediata usando os polinômios simétricos. Vejamos:

x3 + y3 + z3 – 3xyz = S3 – 3 · σ3

Mas de acordo com a questão anterior S3 = σ13 – 3σ1σ2 + 3σ3 e daí temos que
S3 – 3σ3 = σ13 – 3σ1σ2. Assim:

x3 + y3 +z3 – 3xyz = S3 – 3σ3 =


=σ13 – 3σ1σ2 =
=σ1 (σ12 – 3σ2) =
= [(x + y + z)2 – 3 (xy + xz + yz)]
= (x + y + z) (x2 + y2 + z2 – xy – xz – yz)

Obs. (para os mais curiosos): Na RPM 41, pág.38 existe uma bela resolução desse
problema usando um determinante.

b) Usando a fatoração obtida em (a), verifique a famosa desigualdade das médias


a+b+c
aritmética e geométrica. Se a, b, c ∈ #+ então 3 abc ≤ e a igualdade
3
ocorre, se, e somente se, a = b = c.

De fato, em (a) verificamos que


x3 + y3 + z3 – 3xyz = (x + y + z) (x2 + y2 + z2 – xy – xz – yz).
Vamos mostrar inicialmente que se x, y, z são números reais positivos então:

EUREKA! N°25, 2007

48
Sociedade Brasileira de Matemática

(x + y + z) · (x2 + y2 + z2 – xy – xy – yz) ≥ 0

De fato,
1
x2 + y2 + z2 – xy – xz + yz = (2x2 + 2y2 + 2z2 – 2xy – 2xz – 2zy)
2
1
= (x2 – 2xy + y2 + x2 – 2xz + z2 + y2 – 2yz + z2)
2
1
= [(x – y)2 + (x – z)2 + (y – z)2] ≥ 0 (Soma de
2
quadrados)

Ora, como estamos supondo x, y, z reais positivos temos que x + y + z ≥ 0 e daí


(x + y + z) (x2 + y2 + z2 – xy – xz – yz) ≥ 0 (pois é o produto de fatores ≥ 0).

Assim temos que:

x3 + y3 + z3 – 3xyz = (x + y + z) (x2 + y2 + z2 – xy – xz – yz) ≥ 0


e daí
x3 + y 3 + z 3
3xyz ≤ x3 + y3 + z3 ⇒ xyz ≤
3
fazendo x3 = a, y3 = b e z3 = c temos que:
a+b+c
3
a .3 b .3 c ≤
3
e daí
a+b+c
3
abc ≤
3

Com a igualdade ocorrendo se e somente se a = b = c, pois em (x + y + z) · (x2 + y2


+ z2 – xy – xz – yz) ≥ 0 a igualdade ocorre apenas quando x = y = z, visto que x + y
+ z > 0, uma vez que x, y, z são números reais positivos e além disso,
1
(x2 + y2 + z2 – xy – xz – yz) = [(x – y)2 + (x – z)2 + (y – z)2] = 0 ⇔ x = y = z.
2

03. Fatore (x + y + z)3 – (x3 + y3 + z3)

Resolução:
(x + y + z)3 – (x3 + y3 + z3) = σ13 – S3

EUREKA! N°25, 2007

49
Sociedade Brasileira de Matemática

Mas, no exemplo anterior vimos que S3 = σ13 – 3σ1σ2 + 3σ3 e daí

(x + y + z)3 – (x3 + y3 + z3) = σ13 – (σ13 – 3σ1σ2 + 3σ3)


= 3 · (σ1σ2 – σ3)
= 3 · [(x + y + z) (xy + xz + yz) – xyz]
= 3 · (x2y + x2z + xyz + xy2 + xyz + y2z + xyz + xz2 + yz2
– xyz)
= 3 · [xy(x + y) + xz(x + y) + yz(y + z) + xz(y + z)]
= 3 · [(x + y)(xy + xz) + (y + z)(yz + xz)]
= 3 · [(x + y) . x(y + z) + (y + z) . z(x + y)]
= 3 · (x + y)(y + z)(x + z)

04. Se x1 e x2 são as raízes da equação x2 – 6x + 1 = 0 determine o valor de x15 +


x25.

Resolução:
Fazendo Sn = x1n + x2n, n ∈  , queremos determinar S5 = x15 + x25
Temos que:
σ1 = x1 + x2 = 6
σ2 = x1 · x2 = 1
S0 = x10 + x20 = 1 + 1 = 2
S1 = x1 + x2 = 6
Sn = σ1 · Sn – 1 –σ2 · Sn – 2 = 6 Sn – 1 – Sn – 2

e daí

S2 = 6 · S1 – S0 = 6 · 6 – 2 = 34
S3 = 6 · S2 – S1 = 6 · 34 – 6 = 198
S4 = 6 · S3 – S2 = 6 · 198 – 34 = 1154
S5 = 6 · S4 – S3 = 6 · 1154 – 198 = 6726
Assim x15 + x25 = 6726

05. Determine todas as soluções reais do sistema

 x + y + z =1
 3
 x + y + z + xyz = x + y + z + 1
3 3 4 4 4

De acordo com o sistema acima temos que:

EUREKA! N°25, 2007

50
Sociedade Brasileira de Matemática

 σ1 = 1
 , onde S n = x n + y n + z n , n ∈ <
 3
S + σ 3 = S 4 + 1

Mas, S3 = σ13 – 3σ2σ1 + 3σ3 e S4 = σ14 – 4σ12σ2 + 2σ22 + 4σ1. σ3 (verifique isto!) e
daí
S3 + σ3 = S4 + 1 ⇒ σ13 – 3σ1σ2 + 3σ3 + σ3 = σ14 – 4σ12 σ2 + 2σ22 + 4σ1σ3 + 1
Como σ1 = 1 temos que:
1 – 3 σ2 + 4σ3 = 1 – 4σ2 + 2σ22 + 4σ3 + 1 ⇒ 2σ22 –σ2 + 1 = 0
Como, ∆ = (–1)2 – 4 · 2 · 1 = –7 < 0, concluímos que existem raízes reais.
Uma outra aplicação interessante dos polinômios simétricos pode ser encontrada
na resolução de algumas equações irracionais. Vejamos:

06. Determine todas as raízes reais da equação abaixo:

4
272 − x + 4 x = 6
Resolução:

Fazendo 4
x=y e 4
272 − x = z temos que
 y+z=6
x = y4 e 272 – x = z4 ⇒  4
 y + z = 272
4

e agora lembrando que: σ1 = y + z e σ2 = y · z e Sn = yn + zn, com n ∈ 


 σ1 = 6  σ1 = 6
 ⇒ 4
S 4 = 272 σ1 − 4σ1 .σ2 + 2σ 2 = 272
2 2

Logo, 64 – 4 · 62 · σ2 + 2σ22 = 272 ⇒ σ22 – 72σ2 + 512 = 0 ⇒ σ2 = 64 ou σ2 = 8


y + z = 6
Assim, se σ 2 = 64 ⇒  ⇒ Não existem soluções reais.
 y ⋅ z = 64

y + z = 6
Por outro lado, se σ2 = 8 ⇒  ⇒ y = 2 e z = 4 ou z = 2 e y = 4
 y⋅ z = 8
Assim concluímos que:
y = 2 ⇒ x = 16
y = 4 ⇒ x = 256

Logo as raízes reais da equação são 16 e 256.

EUREKA! N°25, 2007

51
Sociedade Brasileira de Matemática

III. Problemas:

01. Se α , β e γ são as raízes da equação x3 + 3x2 – 7x + 1 = 0. Determine o valor


de α 3 + β 3 + γ 3 + α 4 + β 4 + γ 4
 x+ y =a

02. Mostre que se o sistema  x 2 + y 2 = b tem solução, então a3 – 3ab + 2c = 0
 x3 + y 3 = c

03. x, y, z são números reais tais que x + y + z = 5 e yz + zx + xy = 3. Verifique que
13
−1 ≤ z ≤ .
3

04. Se x + y + z = 0, verifique que, para n = 0, 1, 2, ... vale a relação:


1 2
xn + 3 + yn + 3 + zn + 3 = xyz(xn + yn + zn) + (x + y2 + z2)(xn + 1 + yn + 1 + zn + 1)
2

05. Determine as raízes reais da equação 5


33 − x + 5 x = 3 .

06. Verifique que:


(x + y + z)3 – (y + z –x)3 – (x + z – y)3 – (x + y – z)3 = 24xyz.

07. Dados a, b e c números reais positivos tais que


log a b + log b c + log c a = 0 ,determine o valor de (log a b ) + (log b c ) + (log c a ) .
3 3 3

08. Se α, β e γ são números complexos tais que α + β + γ = 1, α2 + β2 + γ 2 = 3 e


α3 + β3 + γ 3 = 7 , determine o valor de α21 + β21 + γ21 .

Referências:

[1] Barbeau, E. J., Polynomials – Problems books in Mathematics – Springer Verlag.


[2] Engel, Arthur , Problem-Solving Strategies – Springer Verlag.
[3] www.obm.org.br
[4] Mathematical Excalibur.

EUREKA! N°25, 2007

52
Sociedade Brasileira de Matemática

OLIMPÍADAS AO REDOR DO MUNDO

Þ A partir desse número a EUREKA! volta a apresentar problemas de


olimpíadas de vários países do mundo. Como antes, esperamos contar com a
colaboração dos leitores para a apresentação das soluções dos problemas
propostos. Aos leitores que se interessarem pela solução de algum problema
particular, pedimos contatar à OBM, através de carta ou e-mail. Repassada a nós a
mensagem, teremos o maior prazer de apresentar as soluções solicitadas no
número subsequente da EUREKA! Bom divertimento!

Antonio Caminha
Antonio Luiz Santos
Bruno Holanda

ÞÞÞ
Samuel Barbosa

211. (Baltic Way – 2004) Uma seqüência a1, a2, ... de números reais não-negativos
satisfaz, para n = 1, 2, ..., as seguintes condições:

(a) an + a2n ≥ 3n.


(b) a n +1 + n ≤ 2 a n (n + 1) .
(i) Prove que an ≥ n para todo n = 1, 2, ...
(ii) Dê exemplo de uma tal seqüência.

212. (Baltic Way – 2004) Seja P um polinômio com coeficientes não-negativos. Prove
que se P(1/x)P(x) ≥ 1 para x = 1, então tal desigualdade se verifica para todo real
positivo x.

213. (Baltic Way – 2004) Ache todos os conjuntos X, consistindo de ao menos dois
inteiros positivos, tais que para todos m, n ∈ X, com n > m, exista um elemento k
de X tal que n = mk2.

214. (Rússia – 2004) São dados um natural n > 3 e reais positivos x1, x2, ..., xn cujo
produto é 1. Prove a desigualdade
1 1 1
+ + ... + > 1.
1 + x1 + x1 x 2 1 + x 2 + x 2 x3 1 + x n + x n x1

EUREKA! N°25, 2007

53
Sociedade Brasileira de Matemática

215. (Rússia – 2004) Uma seqüência a1, a2, ... de números racionais não-negativos é
tal que am + an = amn para todos m, n naturais. Prove que os termos da seqüência
não podem ser todos distintos.

216. (Rússia – 2004) Sejam IA e IB os centros das circunferências ex-inscritas a um


triângulo ABC que tangenciam os lados BC e AC, respectivamente. Seja ainda P
um ponto sobre a circunferência circunscrita de ABC. Prove que o ponto médio do
segmento que une os circuncentros dos triângulos IACP e IBCP coincide com o
circuncentro de ABC.

1 ln( x + 1)
217. (Putnam – 2005) Calcule o valor da integral ∫0 x2 +1
dx .

218. (Putnam – 2005) Encontre todas as funções diferenciáveis f: (0, +∞) → (0, +∞)
a x
para as quais exista um real positivo a tal que f '   = para todo real
 x f ( x)
positivo x.

219. (Putnam – 2005) Seja Sn o conjunto de todas as permutações de 1, 2, ..., n. Para π


∈ Sn, seja σ(π) = 1 se π for uma permutação par e σ(π) = –1 se π for uma
permutação ímpar. Denote ainda por v(π) o número de pontos fixos de π. Prove
que

σ (π ) n
∑ v (π ) + 1
= (–1)n +1 n + 1 .
π ∈S n

220. (Moldávia – 2006) Seja a e b os catetos de um triângulo retângulo, c sua


hipotenusa e h a altura relativa à mesma. Encontre o maior valor possível de
c+h
.
a+b

221. (Moldávia – 2006) Seja n > 1 um inteiro positivo e M = {0, 1, 2, ..., n – 1}. Para a
inteiro não-nulo, nós definimos a função fa: M → M tal que fa(x) é o resto da
divisão de ax por n. Encontre uma condição necessária e suficiente para que fa seja
uma bijeção. Se fa for uma bijeção e n for um número primo, prove que an(n – 1) – 1
é divisível por n2.

EUREKA! N°25, 2007

54
Sociedade Brasileira de Matemática

222. (Moldávia – 2006) O quadrilátero convexo ABCD é inscritível. As tangentes a


sua circunferência circunscrita em A e C se intersectam em P, tal que P não está
sobre a reta BD e PA2 = PB×PD. Prove que a reta BD passa pelo ponto médio do
segmento AC.

A A
223. (Bielorússia – 2005) Seja H o ponto de interseção das alturas BB1 e CC1 do
uma reta passando por A, tal que ⊥ AC.
A
triângulo acutângulo ABC. Seja
Prove que as retas BC, B1C1 e possuem um ponto em comum se e somente se H
for o ponto médio de BB1.

224. (Bielorússia – 2005) Ache todas as funções f :  →  satisfazendo

f (m − n + f (n)) = f (m) + f (n) ,


para todos m, n ∈ .

225. (Bielorússia – 2005) Prove que

 2 3  3  1  1
 a + b + . b 2 + a +  ≥  2a + . 2b +  ,
 4  4  2  2

Para quaisquer reais positivos a e b.

226. (Bulgária – 2005) Ache todos os naturais de quatro algarismos m, menores que
2005 e para os quais existe um natural n < m tal que m – n possui no máximo 3
divisores e mn seja um quadrado perfeito.

227. (Bulgária – 2005) Ivo escreve todos os inteiros de 1 a 100 (inclusive) em cartas e
dá algumas delas para Iana. Sabe-se que para quaisquer duas destas cartas, uma de
Ivo e outra de Iana, a soma dos números não está com Ivo e o produto não está
com Iana. Determine o número de cartas de Iana sabendo que a carta 13 está com
Ivo.

228. (Bulgária – 2005) Ache todas as triplas de inteiros positivos (x, y, z) tais que

2005 2005 2005


+ + ,
x+ y y+z z+ x
também seja um inteiro.

EUREKA! N°25, 2007

55
Sociedade Brasileira de Matemática

229. (Eslovênia – 2005) Encontre todos os números primos p para os quais o número
p2 + 11 tem menos que 11 divisores.

230. (Eslovênia – 2005) Denote por I o incentro do triângulo ABC. Sabe-se que AC +
AI = BC. Encontre a razão entre as medidas do ângulos ∠BAC e ∠CBA.

ÞÞÞ

9RFr VDELD«
4XH ⋅   ⋅  H ⋅ 
VmR SULPRV" (VVHV IRUDP R RLWDYR R QRQR H R GpFLPR SULPRV
GHVFREHUWRV SHOR SURMHWR VHYHQWHHQ RU EXVW VHQGR VHXV
GHVFREULGRUHV 5LFKDUG +DVVOHU 'HUHN *RUGRQ H .RQVWDQWLQ
$JDIRQRY UHVSHFWLYDPHQWH ,VVR PRVWUD TXH   H
 QmR VmR Q~PHURV GH 6LHUSLQVNL LH tPSDUHV N WDLV
TXH N ⋅    p FRPSRVWR SDUD WRGR Q ∈ 
3
 YHMD D (XUHND 

SiJ   UHGX]LQGR SDUD  R Q~PHUR GH QDWXUDLV PHQRUHV

TXH  TXH p R PHQRU Q~PHUR GH 6LHUSLQVNL FRQKHFLGR 

VREUH RV TXDLV QmR VH VDEH VH VmR RX QmR Q~PHURV GH

6LHUSLQVNL       H

 9HMD www.seventeenorbust.com SDUD PDLV

LQIRUPDo}HV LQFOXVLYH VREUH FRPR SDUWLFLSDU GR SURMHWR 

EUREKA! N°25, 2007

56
Sociedade Brasileira de Matemática

 SOLUÇÕES DE PROBLEMAS PROPOSTOS


Publicamos aqui algumas das respostas enviadas por nossos leitores.

108) Sejam A1 , A2 ,..., An conjuntos finitos. Para 1 ≤ k ≤ n, seja


Sk = ∑
1≤ i1 < i2 <...< ik ≤ n
Ai1 ∩ Ai2 ∩ ... ∩ Aik , a soma dos números de elementos das

interseções de k dos conjuntos Ai . Prove que:

a) O número de elementos que pertencem a exatamente r dos conjuntos Ai é


n
k
∑ ( − 1)
k−r
  S k , para 1 ≤ r ≤ n.
k =r r 
b) O número de elementos que pertencem a pelo menos r dos conjuntos Ai é
n
 k − 1
∑ ( − 1)
k −r
  S k , para 1 ≤ r ≤ n.
k =r  r −1 

SOLUÇÃO ADAPTADA DA SOLUÇÃO DE ANDERSON TORRES (SÃO PAULO - SP)


a) Sejam I = {1, 2,..., n} e P ⊂ I um conjunto fixo com r elementos. O total de
elementos que pertencem a todos os Ai com com i ∈ P e que não pertencem a
nenhum outro Aj com j ∈ I \ P é exatamente o total de elementos do conjunto

( ∩ A ) que não pertencem a ( ∪ ( A ∩ ( ∩ A ))).


i∈ P
i
j∈ I \ P
j
i∈ P
i

Podemos usar Inclusão-Exclusão:

∩ Ai − ∪
i∈ P j∈ I − P ( A ∩ ( ∩ A )) =
j
i∈ P
i

∩ Ai −
i∈ P ∑
K⊃P
∩ Ai +
i∈ K ∑
K⊃P
∩ Ai ... =
i∈ k ∑
K⊃P
( − 1) K − P ∩ Ai
i∈ K
K = r +1 K =r+ 2

Agora, variando o conjunto P:

∑ ∑ ( − 1) ∩ Ai = ∑ ∑ ( − 1)
K −P K −P
∩ Ai
i∈ K i∈ K
P =r K ⊃P K ⊂ I P⊂ K
K ≥r P =r

EUREKA! N°25, 2007

57
Sociedade Brasileira de Matemática

O conjunto de índices P ⊂ K com P = r (onde K = k ) pode ser escolhido de


k 
  maneiras para cada K, e já que i∩
∈K
Ai não depende de P, a soma acima
r 
vale
 
k  
n
k 
∑ ( − 1) k −r
   ∑ i∩ Ai  = ∑ ( − 1) k −r
  Sk .
r≤k ≤n  r   K ⊂ I ∈K  k =r r 
 K =k 
n
k 
b) Somando a expressão ∑ ( − 1) k −m
  S k de m = r até n, e usando a
k =m m
identidade
k
k  k −r
k   k −1  k − r
∑ ( − 1) k −m
 = ∑ ( − 1) j
  = ( − 1)
k −r
 −  = ( − 1)
k −r
 − ,
m =r m j=0  j k r r 1 
que pode ser facilmente provada por indução (ver nota), o resultado segue.
Nota: Para provar por indução a identidade
r
k r k
− 1
∑ ( − 1)   = ( − 1) 
j
 , 0 ≤ r ≤ k , basta ver que isto vale para r = 0 e,
j=0  j  r 
r +1
k  k  r
k
se r < k, ∑ ( − 1) j   = ( − 1) r +1   + ∑ ( − 1) j   =
j=0  j  r + 1 j=0  j
 k  r  k − 1 r +1   k   k − 1   r +1  k − 1 
= (−1) r +1   + (−1)   = (−1)    −   = (−1)  .
 r + 1  r    r + 1  r    r +1

109) Na figura abaixo, AB = AC , B l


AC = 100° e AD = BC . Mostre que
l é racional quando expresso em graus.
x = BCD
A

B
x C
D

EUREKA! N°25, 2007

58
Sociedade Brasileira de Matemática

SOLUÇÃO DE GIBRAN MEDEIROS

A 40° + x
60°
100°

B 40° 60°
40°
x C
D

Construimos o triângulo eqüilátero ACE. Os triângulos BCE e ACD são


congruentes, pois CE = AC , ∠BCE = ∠DAC = 100° e BC = AD. Logo
∠BCE = 40° + x. Como o triângulo ABE é isósceles, temos
∠AEB = ∠ABE = 10° e, portanto 60° − ( 40° + x ) = 10°, donde x = 10°.
Continuamos aguardando as soluções dos seguintes problemas propostos:

110) Um conjunto finito de inteiros positivos é chamado de Conjunto DS se cada


elemento divide a soma dos elementos do conjunto.
Prove que todo conjunto finito de inteiros positivos é subconjunto de algum
conjunto DS.

111) Prove que existem infinitos múltiplos de 7 na seqüência (an ) abaixo:


a1 = 1999, an = an −1 + p (n), ∀n ≥ 2 , onde p(n) é o menor primo que divide n.

112) a) Determine todos os inteiros positivos n tais que existe uma matriz n × n
com todas as entradas pertencentes a { –1, 0, 1} tal que os 2n números obtidos
como somas dos elementos de suas linhas e de suas colunas são todos distintos.
Nota: Como explicado na página 72 da Eureka! No. 24, o item b) do problema 112
foi anulado.

Agradecemos também o envio das soluções e a colaboração de:


André Araújo Fortaleza – CE
Carlos Alberto da Silva Victor Nilópolis – RJ
Diego Andrés de Barros Recife – PE
Dymitri Cardoso Leão Recife – PE
Fábio Soares Piauí Recife – PE
Janderson Alencar Recife – PE
Mardônio Luz do Amaral Recife – PE
Michel Angelucci Ibitinga – SP
Renan e Gabriel Lima Novais Niterói – RJ
Samuel Liló Abdalla Sorocaba – SP

EUREKA! N°25, 2007

59
Sociedade Brasileira de Matemática

PROBLEMAS PROPOSTOS
Convidamos o leitor a enviar soluções dos problemas propostos e sugestões de novos
problemas para próximos números.

113) a1 , a2 , a3 ,... formam uma seqüência de inteiros positivos menores que 2007
am + an
tais que é inteiro, para quaisquer inteiros positivos m, n.
am + n
Prove que a seqüência (an) é periódica a partir de um certo ponto.

114) Sabendo que sen x + sen y + sen z + sen w = 0 e


cos x + cos y + cos z + cos w = 0, mostre que
sen 2003 x + sen 2003 y + sen 2003 z + sen 2003w = 0.

115) Suponha que ABC é um triângulo com lados inteiros a, b e c com


l = 60° e mdc(a, b) = mdc( a, c) = mdc(b, c) = 1. Prove que c ≡ 1(mod 6) .
BCA

116) Seja ABC um triângulo e sejam X, Y e Z as reflexões de A, B e C em


relação às retas BC, CA e AB, respectivamente. Prove que x, y e z são colineares se
e somente se cos A ⋅ cos B ⋅ cos C = − 3 8.

117) Sejam r e s duas retas reversas (i.e., não contidas num mesmo plano) e A, B,
C, D, E E,C
A, B E
E, D E, B
pontos tais que A, B , A E ∈ r , C , D, C E ∈ s, AB = AB
E, D EE e
ED
CD = C E. Prove que os tetraedros ABCD e EABC E têm o mesmo volume.
E ED

2 + 9an
118) Considere a seqüência (an )n ≥1 dada por a1 = 1 e an +1 = , ∀n ≥ 1.
3 + 9an
Prove que (an ) converge e calcule a seu limite.

Problema 113 proposto por Anderson Torres (São Carlos – SP); Problema 114 proposto por
Carlos A. Gomes (Natal – RN); Problema 115 proposto por Gabriel Ponce (enviado por e-
mail); Problema 116 e 117 propostos por Wilson Carlos da Silva Ramos (Belém – PA);
Problema 118 proposto por Sidnei Belcides Avelar (Macapá – AP)

EUREKA! N°25, 2007

60
Sociedade Brasileira de Matemática

AGENDA OLÍMPICA
XXIX OLIMPÍADA BRASILEIRA DE MATEMÁTICA

NÍVEIS 1, 2 e 3
Primeira Fase – Sábado, 16 de junho de 2007
Segunda Fase – Sábado, 15 de setembro de 2007
Terceira Fase – Sábado, 27 de outubro de 2007 (níveis 1, 2 e 3)
Domingo, 28 de outubro de 2007 (níveis 2 e 3 - segundo dia de prova).

NÍVEL UNIVERSITÁRIO
Primeira Fase – Sábado, 15 de setembro de 2007
Segunda Fase – Sábado, 27 e Domingo, 28 de outubro de 2007

XIII OLIMPÍADA DE MAIO
12 de maio de 2007

XVIII OLIMPÍADA DE MATEMÁTICA DO CONE SUL
Uruguai
12 a 17 de junho de 2007

XLVIII OLIMPÍADA INTERNACIONAL DE MATEMÁTICA
19 a 31 de julho de 2007
Vietnã

XIV OLIMPÍADA INTERNACIONAL DE MATEMÁTICA UNIVERSITÁRIA
3 a 9 de agosto de 2007
Blagoevgrad, Bulgária

XXII OLIMPÍADA IBEROAMERICANA DE MATEMÁTICA
6 a 16 de setembro de 2007
Portugal

X OLIMPÍADA IBEROAMERICANA DE MATEMÁTICA UNIVERSITÁRIA
♦♦♦

EUREKA! N°25, 2007

61
Sociedade Brasileira de Matemática

COORDENADORES REGIONAIS
Alberto Hassen Raad (UFJF) Juiz de Fora – MG
Américo López Gálvez (USP) Ribeirão Preto – SP
Amarísio da Silva Araújo (UFV) Viçosa – MG
Andreia Goldani FACOS Osório – RS
Antonio Carlos Nogueira (UFU) Uberlândia – MG
Ali Tahzibi (USP) São Carlos – SP
Benedito Tadeu Vasconcelos Freire (UFRN) Natal – RN
Carlos Alexandre Ribeiro Martins (Univ. Tec. Fed. de Paraná) Pato Branco - PR
Carmen Vieira Mathias (UNIFRA) Santa María – RS
Claus Haetinger (UNIVATES) Lajeado – RS
Cleonor Crescêncio das Neves (UTAM) Manaus – AM
Cláudio de Lima Vidal (UNESP) S.J. do Rio Preto – SP
Edson Roberto Abe (Colégio Objetivo de Campinas) Campinas – SP
Élio Mega (Colégio Etapa) São Paulo – SP
Eudes Antonio da Costa (Univ. Federal do Tocantins) Arraias – TO
Fábio Brochero Martínez (UFMG) Belo Horizonte – MG
Florêncio Ferreira Guimarães Filho (UFES) Vitória – ES
Genildo Alves Marinho (Centro Educacional Leonardo Da Vinci) Taguatingua – DF
Ivanilde Fernandes Saad (UC. Dom Bosco) Campo Grande– MS
Jacqueline Rojas Arancibia (UFPB)) João Pessoa – PB
Janice T. Reichert (UNOCHAPECÓ) Chapecó – SC
João Benício de Melo Neto (UFPI) Teresina – PI
João Francisco Melo Libonati (Grupo Educacional Ideal) Belém – PA
José Cloves Saraiva (UFMA) São Luis – MA
José Luiz Rosas Pinho (UFSC) Florianópolis – SC
José Vieira Alves (UFPB) Campina Grande – PB
José William Costa (Instituto Pueri Domus) Santo André – SP
Krerley Oliveira (UFAL) Maceió – AL
Licio Hernandes Bezerra (UFSC) Florianópolis – SC
Luzinalva Miranda de Amorim (UFBA) Salvador – BA
Mário Rocha Retamoso (UFRG) Rio Grande – RS
Marcelo Rufino de Oliveira (Grupo Educacional Ideal) Belém – PA
Marcelo Mendes (Colégio Farias Brito, Pré-vestibular) Fortaleza – CE
Newman Simões (Cursinho CLQ Objetivo) Piracicaba – SP
Nivaldo Costa Muniz (UFMA) São Luis – MA
Raúl Cintra de Negreiros Ribeiro (Colégio Anglo) Atibaia – SP
Ronaldo Alves Garcia (UFGO) Goiânia – GO
Rogério da Silva Ignácio (Col. Aplic. da UFPE) Recife – PE
Reginaldo de Lima Pereira (Escola Técnica Federal de Roraima) Boa Vista – RR
Reinaldo Gen Ichiro Arakaki (UNIFESP) SJ dos Campos – SP
Ricardo Amorim (Centro Educacional Logos) Nova Iguaçu – RJ
Sérgio Cláudio Ramos (IM-UFRGS) Porto Alegre – RS
Seme Gebara Neto (UFMG) Belo Horizonte – MG
Tadeu Ferreira Gomes (UEBA) Juazeiro – BA
Tomás Menéndez Rodrigues (U. Federal de Rondônia) Porto Velho – RO
Valdenberg Araújo da Silva (U. Federal de Sergipe) São Cristovão – SE
Valdeni Soliani Franco (U. Estadual de Maringá) Maringá – PR
Vânia Cristina Silva Rodrigues (U. Metodista de SP) S.B. do Campo – SP
Wagner Pereira Lopes (CEFET – GO) Jataí – GO
William Beline (UNESPAR/FECILCAM) Campo Mourão – PR

EUREKA! N°25, 2007

62
CONTEÚDO

XXVIII OLIMPÍADA BRASILEIRA DE MATEMÁTICA 2


Problemas e Soluções da Primeira Fase

XXVIII OLIMPÍADA BRASILEIRA DE MATEMÁTICA 15


Problemas e Soluções da Segunda Fase

XXVIII OLIMPÍADA BRASILEIRA DE MATEMÁTICA 32


Problemas e Soluções da Terceira Fase

XXVIII OLIMPÍADA BRASILEIRA DE MATEMÁTICA 55


Problemas e Soluções da Primeira Fase – Nível Universitário

XXVIII OLIMPÍADA BRASILEIRA DE MATEMÁTICA 59


Problemas e Soluções da Segunda Fase – Nível Universitário

XXVIII OLIMPÍADA BRASILEIRA DE MATEMÁTICA 70


Premiados

AGENDA OLÍMPICA 74

COORDENADORES REGIONAIS 75
Sociedade Brasileira de Matemática

XXVIII OLIMPÍADA BRASILEIRA DE MATEMÁTICA


Problemas e soluções da Primeira Fase

PROBLEMAS – NÍVEL 1
1. Em um tanque há 4000 bolinhas de pingue-pongue. Um menino começou a
retirar as bolinhas, uma por uma, com velocidade constante, quando eram 10h.
Após 6 horas, havia no tanque 3520 bolinhas. Se o menino continuasse no mesmo
ritmo, quando o tanque ficaria com 2000 bolinhas?
A) às 11h do dia seguinte
B) às 23h do mesmo dia
C) às 4h do dia seguinte
D) às 7h do dia seguinte
E) às 9h do dia seguinte

2. O gráfico a seguir apresenta informações sobre o impacto causado por 4 tipos


de monocultura ao solo. Para cada tipo de monocultura, o gráfico mostra a
quantidade de água, em litros, e a de nutrientes (nitrogênio, fósforo e potássio),
em quilogramas, consumidos por hectare para a produção de 1kg de grãos de soja
ou 1kg de milho ou 1kg de açúcar ou 1kg de madeira de eucalipto. Sobre essas
monoculturas, pode-se afirmar que:

2000

1500

1000

500

0
cana-de- soja milho eucalipto
açucar

água nutrientes

A) O eucalipto precisa de cerca de 1/3 da massa de nutrientes necessários de que


a cana-de-açúcar precisa para se desenvolver.
B) O eucalipto é a que mais seca e empobrece o solo, causando desequilíbrio
ambiental.
C) A soja é cultura que mais precisa de nutrientes.
D) O milho precisa do dobro do volume de água de que precisa a soja.
E) A cana-de-açúcar é a que necessita do ambiente mais úmido para crescer.

EUREKA! N°26, 2007

2
Sociedade Brasileira de Matemática

3. Um time de futebol ganhou 8 jogos mais do que perdeu e empatou 3 jogos


menos do que ganhou, em 31 partidas jogadas. Quantas partidas o time venceu?
A) 11 B) 14 C) 15 D) 17 E) 23

4. Efetuando as operações indicadas na expressão


 22007 + 22005 
 2006 2004 
× 2006
2 +2 
obtemos um número de quatro algarismos. Qual é a soma dos algarismos desse
número?
A) 4 B) 5 C) 6 D) 7 E) 8

5. Quantos números de três algarismos ímpares distintos são divisíveis por 3?


A) 18 B) 24 C) 28 D) 36 E) 48

6. Uma empresa de telefonia celular oferece planos mensais de 60 minutos a um


custo mensal de R$ 52,00, ou seja, você pode falar durante 60 minutos no seu
telefone celular e paga por isso exatamente R$ 52,00. Para o excedente, é cobrada
uma tarifa de R$ 1,20 cada minuto. A mesma tarifa por minuto excedente é
cobrada no plano de 100 minutos, oferecido a um custo mensal de R$ 87,00. Um
usuário optou pelo plano de 60 minutos e no primeiro mês ele falou durante 140
minutos. Se ele tivesse optado pelo plano de 100 minutos, quantos reais ele teria
economizado?
A) 10 B) 11 C) 12 D) 13 E) 14

7. Quantos triângulos isósceles têm como vértices os


vértices do pentágono regular desenhado ao lado?
A) 5 B) 10 C) 15 D) 20
E) 25

8. Dos números a seguir, qual é o único que pode ser escrito como produto de
quatro naturais consecutivos?
A) 712 B) 548 C) 1026 D) 1456 E) 1680

9. Ao redor de um grande lago existe uma ciclovia de 45 quilômetros de


comprimento, na qual sempre se retorna ao ponto de partida se for percorrida
num único sentido. Dois amigos partem de um mesmo ponto com velocidades
constantes de 20 km por hora e 25 km por hora, respectivamente, em sentidos
opostos. Quando se encontram pela primeira vez, o que estava correndo a 20 km

EUREKA! N°26, 2007

3
Sociedade Brasileira de Matemática

por hora aumenta para 25 km por hora e o que estava a 25 km por hora diminui
para 20 km por hora. Quanto tempo o amigo que chegar primeiro ao ponto de
partida deverá esperar pelo outro?
A) nada B) 10 min C) 12 min D) 15 min E) 18 min

10. Num relógio digital, as horas são exibidas por meio de quatro algarismos. Por
exemplo, ao mostrar 00:00 sabemos que é meia-noite e ao mostrar 23:59 sabemos
que falta um minuto para meia-noite. Quantas vezes por dia os quatro algarismos
mostrados são todos pares?
A) 60 B) 90 C) 105 D) 180 E) 240

11. São dadas duas tiras retangulares de papel com 20


cm de comprimento, uma com 5 cm de largura e outra
com 11 cm de largura. Uma delas foi colada sobre a
outra, perpendicularmente, de modo a formar a figura
ilustrada ao lado. Qual é o perímetro dessa figura, em
centímetros? 90°

A) 50 B) 60 C) 80 D) 1
E) 120
12. Seis amigos planejam viajar e decidem fazê-lo em duplas, cada uma
utilizando um meio de transporte diferente, dentre os seguintes: avião, trem e
carro. Alexandre acompanha Bento. André viaja de avião. Carlos não acompanha
Dário nem faz uso do avião. Tomás não anda de trem. Qual das afirmações a
seguir é correta?
A) Bento vai de carro e Carlos vai de avião.
B) Dário vai de trem e André vai de carro.
C) Tomás vai de trem e Bento vai de avião.
D) Alexandre vai de trem e Tomás vai de carro.
E) André vai de trem e Alexandre vai de carro.

13. Usando pastilhas de cerâmica preta na forma de quadradinhos foi composta


uma decoração numa parede, mostrada parcialmente abaixo:

EUREKA! N°26, 2007

4
Sociedade Brasileira de Matemática

Quantas pastilhas foram empregadas em toda a decoração considerando-se que na


última peça montada foram utilizadas 40 pastilhas?
A) 60 B) 68 C) 81 D) 100 E) 121

14. Sara foi escrevendo nas casas de um tabuleiro 95 por 95 os múltiplos


positivos de 4, em ordem crescente, conforme a figura a seguir.

4 8 12 16 20 … 376 380
760 756 752 748 744 … 388 384
764 → → → → … → →
← ← ← ← ← … ← ←

U
O número que Sara escreveu onde se encontra a letra U é:
A) 35192 B) 35196 C) 36100 D) 36104 E) 36108

15. O desenho à direita representa dois quadrados


menores congruentes de lado 20 e um quadrado
maior. O vértice O é o único ponto comum aos
dois quadrados menores e é o centro do quadrado O
A B
maior. Os vértices A, O e B estão alinhados e a
área da região do quadrado maior não pintada é
igual a 36% da área de toda a região pintada. Qual
é a área do quadrado maior?
A) 420 B) 496 C) 576 D) 640
E) 900
16. Um certo número inteiro positivo, quando dividido por 15 dá resto 7. Qual é a
soma dos restos das divisões desse número por 3 e por 5?
A) 2 B) 3 C) 4 D) 5 E) 6

17. No fim de 1994, Neto tinha a metade da idade de sua avó. A soma dos anos
de nascimento dos dois é 3844. Quantos anos Neto completa em 2006?
A) 55 B) 56 C) 60 D) 62 E) 108

18. A figura a seguir representa um Tangram, quebra-cabeças chinês formado


por 5 triângulos, 1 paralelogramo e 1 quadrado. Sabendo que a área do Tangram a
seguir é 64 cm2, qual é a área, em cm2, da região sombreada?

EUREKA! N°26, 2007

5
Sociedade Brasileira de Matemática

A) 7,6 B) 8 C) 10,6 D) 12 E) 21,3

19. As permutações da palavra BRASIL foram listadas em ordem alfabética,


como se fossem palavras de seis letras em um dicionário. A 361ª palavra nessa
lista é:
A) BRISAL B) SIRBAL C) RASBIL D) SABRIL E) LABIRS

20. No planeta POT o número de horas por dia é igual a número de dias por
semana, que é igual ao número de semanas por mês, que é igual ao número de
meses por ano. Sabendo que em POT há 4096 horas por ano, quantas semanas há
num mês?
A) 8 B) 12 C) 64 D) 128 E) 256

PROBLEMAS – NÍVEL 2
1. Veja o problema No. 4 do nível 1.
2. Veja o problema No. 11 do nível 1.

3. Se um número de dois dígitos é 5 vezes a soma de seus dígitos, então o número


formado pela troca dos dígitos é a soma dos dígitos multiplicada por:
A) 3 B) 5 C) 6 D) 4 E) 7

4. Veja o problema No. 9 do nível 1.


5. Na figura, AB = AC, AE = AD e o ângulo BAD mede 30o. Então o ângulo x
mede:
A

30°

x
B C
D

A)10o B) 20o C) 15o D) 30o E) 5o

EUREKA! N°26, 2007

6
Sociedade Brasileira de Matemática

6. A soma de três números naturais consecutivos é igual ao produto desses três


números. A soma dos quadrados desses números é:
A) 14 B) 15 C) 18 D) 24 E) 36

7. Veja o problema No. 17 do nível 1.

8. Três quadrados são colados pelos seus vértices entre si e a dois bastões
verticais, como mostra a figura.

x
75°

30° 126°

A medida do ângulo x é:
A) 39º B) 41º C) 43º D) 44º E) 46º

9. Sejam a, b e c inteiros e positivos. Entre as opções abaixo, a expressão que


não pode representar o número 24 é:
A) ab3 B) a2b3 C) a c b c D) ab2c3 E) a b bc c a

10. O número de quadrados que podem ser construídos com vértices nos pontos
da figura abaixo é:

A) 18 B) 14 C) 9 D) 20 E) 10

11. Veja o problema No. 12 do nível 1.

13. O máximo divisor comum de todos os termos da seqüência a n = n 3 − n ,


n = 1, 2, 3, ... é:
A) 2 B) 3 C) 4 D) 5 E) 6

EUREKA! N°26, 2007

7
Sociedade Brasileira de Matemática

14. Samuel possui três irmãos a mais do que irmãs. O número de irmãos de
Samila, irmã de Samuel, é igual ao dobro do número de suas irmãs. O número de
filhos (homens e mulheres) que possui o pai de Samuel e Samila é:
A) 10 B) 13 C) 16 D) 17 E) 20

15. Veja o problema No. 18 do nível 1.

16. João escreveu todos os números com menos de 4 dígitos usando apenas os
algarismos 1 e 2 numa folha de papel e depois somou todos eles. O valor obtido
foi:
A) 2314 B) 3000 C) 1401 D) 2316 E) 1716

17. Sejam a, b e c números reais positivos cuja soma é 1. Se a, b e c são as


medidas dos lados de um triângulo, podemos concluir que
1 1 1
A) 0 < a − b < e 0 < b − c < e 0 < c − a <
2 2 2
1 1 1
B) a < e b < e c <
2 2 2
1 1 1
C) a + b < e b + c < e c + a <
2 2 2
1 1 1
D) a ≤ e b ≤ e c ≤
3 3 3
1 1 1
E) a ≥ e b ≥ e c ≥
3 3 3
18. O número de soluções inteiras e positivas do sistema abaixo é:
a + b = c 2

a + b + c = 30
A) 45 B) 23 C) 24 D) 25 E) 72

19. Um número com dois dígitos distintos e não nulos é chamado de bonito se o
dígito das dezenas é maior do que o dígito das unidades. A quantidade de
números bonitos é:
A) 72 B) 36 C) 35 D) 64 E) 56

20. O professor Piraldo aplicou uma prova para seus cinco alunos e, após corrigi-
las, digitou as notas em uma planilha eletrônica que calcula automaticamente a
média das notas à medida que elas são digitadas. Piraldo notou que após digitar
cada nota a média calculada pela planilha era um número inteiro. Se as notas dos

EUREKA! N°26, 2007

8
Sociedade Brasileira de Matemática

cinco estudantes são, em ordem crescente, 71, 76, 80, 82 e 91, a última nota que
Piraldo digitou foi:
A) 71 B) 76 C) 80 D) 82 E) 91

21. Simplificando a expressão:

2+ 3 . 2+ 2+ 3 . 2+ 2+ 2+ 3 . 2− 2+ 2+ 3
obtemos:
A) 2 B) 3 C) 1 D) 2 + 2 E) 2 + 3

22. Ludmilson percebeu que para numerar as páginas de um livro,


consecutivamente, a partir da página 2, foram usados 2006 algarismos. O número
de páginas do livro de Ludmilson é:
A)701 B) 702 C) 703 D) 704 E) 705

23. Sejam x, y, z números reais não nulos tais que x + y + z = 0 . O valor de


 1 1 1 
(x 2
y 2 z 2 )  3 3 + 3 3 + 3 3  é:
x y x z y z 
A) 0 B) 1 C) 2 D) 3 E) 4

24. Veja o problema No. 10 do nível 1.


25. Na figura a seguir, ABC é um triângulo qualquer e ACD e AEB são triângulos
eqüiláteros. Se F e G são os pontos médios de EA e AC, respectivamente, a razão
BD
é:
FG
D

A 1
F
E A) B) 1
2
G
3
C) D) 2
2
B C
E) Depende das medidas dos lados de ABC.

EUREKA! N°26, 2007

9
Sociedade Brasileira de Matemática

PROBLEMAS – NÍVEL 3
1. Veja o problema No. 17 do nível 1.

2. Quantos resultados diferentes podemos obter somando pares de números


distintos do conjunto {1, 2,…, 2006} ?
A) 2006 B) 2007 C) 4009 D) 4011 E) 4012

3. Uma colônia de amebas tem inicialmente uma ameba amarela e uma ameba
vermelha. Todo dia, uma única ameba se divide em duas amebas idênticas. Cada
ameba na colônia tem a mesma probabilidade de se dividir, não importando sua
idade ou cor. Qual é a probabilidade de que, após 2006 dias, a colônia tenha
exatamente uma ameba amarela?
1 1 1 1 2006
A) 2006 B) C) D) E)
2 2006 2007 2006⋅ 2007 2007

4. Veja o problema No. 8 do nível 2.

5. Os dois números reais a e b são não nulos e satisfazem ab = a – b. Assinale a


a b
alternativa que exibe um dos possíveis valores de + − ab .
b a
1 1 1
A) –2 B) − C) D) E) 2
2 3 2

6. De quantas maneiras podemos colocar, em cada espaço abaixo, um entre os


algarismos 4, 5, 6, 7, 8, 9, de modo que todos os seis algarismos apareçam e
formem, em cada membro, números de dois algarismos que satisfazem a dupla
desigualdade?
__>__>__

A) 100 B) 120 C) 240 D) 480 E) 720

7. Que expressão não pode representar o número 24 para valores inteiros


positivos convenientes de a, b e c?
A) ab3 B) a2b3 C) acbc D) ab2c3 E) abbcca

8. Qual dos valores abaixo de x é tal que 2 x 2 + 2 x + 19 não é um número primo?


A) 50 B) 37 C) 9 D) 5 E) 1

9. Veja o problema No. 17 do nível 2.

EUREKA! N°26, 2007

10
Sociedade Brasileira de Matemática

10. Uma seqüência tem 9 números reais, sendo o primeiro 20 e o último 6. Cada
termo da seqüência, a partir do terceiro, é a média aritmética de todos os
anteriores. Qual é o segundo termo da seqüência?
A) –8 B) 0 C) 4 D) 14 E) 2006

11. Quantos ternos de números reais x, y, z satisfazem o sistema abaixo?


x( x + y + z ) = 2005
y ( x + y + z ) = 2006
z ( x + y + z ) = 2007
A) Nenhum B) 1 C) 2 D) 3 E) 2006

12. Arnaldo tem vários quadrados azuis 1 × 1 , vários quadrados amarelos 2 × 2 e


vários quadrados verdes 3 × 3 e quer montar um quadrado maior no qual
apareçam as três cores. Qual é a menor quantidade de quadrados que ele poderá
utilizar ao todo?
A) 3 B) 6 C) 7 D) 8 E) 9
x − 5 2006
13. Sejam x e y números racionais. Sabendo que também é um
4 − y 2006
número racional, quanto vale o produto xy?
A) 20
B) Pode ser igual a 20, mas também pode assumir outros valores.
C) 1
D) 6
E) Não se pode determinar.

14. Veja o problema No. 20 do nível 2.

15. Veja o problema No. 25 do nível 2.

16. O inteiro positivo x é múltiplo de 2006 e x está entre 2005 e 2007. Qual é o
número de possíveis valores de x?
A) 1 B) 2 C) 3 D) 4 E) 5

17. Na figura temos dois semicírculos de diâmetros PS, de medida 4, e QR,


paralelo a PS. Além disso, o semicírculo menor é tangente a PS em O. Qual é a
área destacada?

EUREKA! N°26, 2007

11
Sociedade Brasileira de Matemática

Q R
A) 2π – 2
B) 3π
C) π
D) 4
P S
E) 2π – 4

18. Iniciando com o par (2048, 1024), podemos aplicar quantas vezes quisermos a
 3a + b a + 3b 
operação que transforma o par (a, b) no par  ,  , então, dentre os
 4 4 
seguintes pares:
1) (1664, 1408)
2) (1540, 1532)
3) (1792, 1282)
4) (1537, 1535)
5) (1546, 1526)

A) Todos podem ser obtidos.


B) Apenas o par 4 não pode ser obtido.
C) Apenas o par 3 não pode ser obtido.
D) Existem exatamente dois pares que não podem ser obtidos.
E) Existem mais de dois pares que não podem ser obtidos.

19. Num tabuleiro retangular de 13 linhas e 17 colunas colocamos números em


cada casinha da seguinte maneira: primeiro, numeramos as casinhas da primeira
linha, da esquerda para a direita, com os números 1, 2, 3, …, 17, nessa ordem;
depois numeramos a segunda linha, também da esquerda para a direita, com os
números de 18 a 34, e assim por diante. Após preenchermos todo o tabuleiro,
colocamos em cada casinha um segundo número, numerando as casinhas da
primeira coluna, de cima para baixo, com os números 1, 2, 3, …, 13, nessa
ordem, depois numeramos a segunda coluna, também de cima da baixo, com os
números de 14 a 26, e assim por diante. Deste modo, cada casinha tem dois
números. Quantas casinhas têm dois números iguais?
A) 2 B) 3 C) 4 D) 5 E) 6

20. Altino está encostado num muro bem alto, durante a noite. A rua onde Altino
está é iluminada por uma lâmpada no topo de um poste de 4 metros de altura, a
10 metros de distância do muro. Altino, um rapaz de 2 metros de altura, anda em
direção ao muro. Seja f(x) a altura, em metros, da sombra de Altino produzida

EUREKA! N°26, 2007

12
Sociedade Brasileira de Matemática

pela lâmpada no muro quando Altino está a uma distância de x metros do muro.
Qual alternativa representa melhor o gráfico de f(x)?
f(x) f(x) f(x)

x x
A) B) x
C)

f(x) f(x)

D) x x
E)
21. O piso de um quarto tem forma de um quadrado de lado 4 m. De quantas
maneiras podemos cobrir totalmente o quarto com oito tapetes iguais de
dimensões 1 m e 2 m? Mostramos abaixo três maneiras de fazê-lo:

A) 27 B) 30 C) 34 D) 36 E) 52

22. Dois pontos A e B de um plano α estão a 8 unidades de distância. Quantas


retas do plano α estão a 2 unidades de A e 3 unidades de B?
A) 1 B) 2 C) 3 D) 4 E) 5

23. Considere os 2161 produtos 0 ⋅ 2160 , 1 ⋅ 2159 , 2 ⋅ 2158 , ..., 2160 ⋅ 0 . Quantos
deles são múltiplos de 2160?
A) 2 B) 3 C) 12 D) 13 E) 2161

24. Qual é o menor valor que a expressão


x2 + 1 + ( y − x)2 + 4 + ( z − y)2 + 1 + (10 − z )2 + 9 pode assumir, sendo x, y e z
reais?
A) 7 B) 13 C) 4 + 109 D) 3 + 2 + 90 E) 149

EUREKA! N°26, 2007

13
Sociedade Brasileira de Matemática

25. Um cubo de aresta 1 é cortado em quatro regiões por dois planos: um deles
contém as arestas AB e CD e o outro contém as arestas AE e DF. Qual é o volume
da(s) maior(es) das quatro regiões?
C D 1 1 2
A) B) C)
E 4 3 4

3 1
D) E)
F 8 2
A B

GABARITO
NÍVEL 1 (5ª. e 6ª. Séries)
1) A 6) D 11) C 16) B
2) A 7) B 12) D 17) C
3) B 8) E 13) E 18) D
4) D 9) A 14) C 19) E
5) B 10) C 15) C 20) A

NÍVEL 2 (7ª. e 8ª. Séries)


1) D 6) A 11) D 16) C 21) C
2) C 7) C 12) C 17) B 22) E
3) C 8) A 13) E 18) C 23) D
4) A 9) B 14) C 19) B 24) C
5) C 10) D 15) D 20) C 25) D

NÍVEL 3 (Ensino Médio)


1) C 6) B 11) C 16) D 21) D
2) C 7) B 12) D 17) A 22) D
3) C 8) B 13) A 18) D 23) D
4) A 9) B 14) C 19) D 24) E
5) E 10) A 15) D 20) A 25) B

EUREKA! N°26, 2007

14
Sociedade Brasileira de Matemática

XXVIII OLIMPÍADA BRASILEIRA DE MATEMÁTICA


Problemas e Soluções da Segunda Fase

PROBLEMAS – Nível 1 PARTE A


(Cada problema vale 5 pontos)
2 2 23 2 4 2 2005 2 2006
01. Qual é a soma dos algarismos do número + + + + + ?
2 2 2 23 2 2004 2 2005

02. A massa de gordura de uma certa pessoa corresponde a 20% de sua massa
total. Essa pessoa, pesando 100 kg, fez um regime e perdeu 40% de sua gordura,
mantendo os demais índices. Quantos quilogramas ela pesava ao final do regime?

03. Quantos os números de dois algarismos têm a soma desses algarismos igual a
um quadrado perfeito? Lembre-se que, por exemplo, 09 é um número de um
algarismo.

04. Os números de 1 a 99 são escritos lado a lado: 123456789101112...9899.


Então aplicamos a seguinte operação: apagamos os algarismos que aparecem nas
posições pares, obtendo 13579012...89. Repetindo essa operação mais 4 vezes,
quantos algarismos irão sobrar?

05. Com a parte destacada da folha retangular ao


lado, pode-se montar um cubo. Se a área da folha
é 300cm2, qual é o volume desse cubo, em cm3?

06. Na tabela a seguir, escreva os números de 1 a 9 em cada coluna, de modo que


a soma dos números escritos nas 9 linhas seja a mesma, igual a Y. Seja X a soma
dos números de cada coluna. Calcule X + Y.
Y
Y
Y
Y
Y
Y
Y
Y
Y
X X X

EUREKA! N°26, 2007

15
Sociedade Brasileira de Matemática

PROBLEMAS – Nível 1 PARTE B


(Cada problema vale 10 pontos)

PROBLEMA 1
Jade escreveu todos os números de 3 algarismos em cartões amarelos, um por
cartão e escreveu todos os números de 4 algarismos em cartões azuis, um por
cartão. Os cartões são todos do mesmo tamanho.
a) Ao todo, quantos cartões foram utilizados? Lembre-se que, por exemplo, 037 é
um número de dois algarismos, bem como 0853 é um número de três algarismos.
b) Todos os cartões são então colocados numa mesma urna e embaralhados.
Depois Jade retira os cartões, um a um, sem olhar o que está pegando. Quantos
cartões Jade deverá retirar para ter certeza de que há dois cartões azuis entre os
retirados?

PROBLEMA 2
No quadriculado a seguir, cada quadradinho tem 1 cm2 de área.

a) Qual é a área e o perímetro da figura formada pelos quadradinhos pintados de


cinza?
b) Pintando outros quadradinhos, podemos aumentar a área dessa figura, sem
mudar o seu perímetro. Qual é o valor máximo da área que podemos obter dessa
maneira?

PROBLEMA 3
Esmeralda inventou uma brincadeira. Digitou alguns algarismos na primeira linha
de uma folha. Depois, na segunda linha, fez a descrição dos algarismos digitados
da seguinte maneira: ela apresentou as quantidades de cada um dos que
apareceram, em ordem crescente de algarismo. Por exemplo, após digitar
21035662112, ela digitou 103132131526, pois em 21035662112 existe um
algarismo 0, três algarismos 1, três algarismos 2, um algarismo 3, um algarismo 5
e dois algarismos 6.
a) Ela começou uma nova folha com 1. Fez, então, sua descrição, ou seja, digitou
11 na segunda linha. Depois, descreveu 11, ou seja, digitou 21 na terceira linha, e
assim continuou. O que ela digitou na 10a linha da folha?

EUREKA! N°26, 2007

16
Sociedade Brasileira de Matemática

b) Esmeralda gostou tanto de fazer isso que decidiu preencher várias folhas com
essa brincadeira, começando com 01 na primeira linha da primeira folha. Quais
são os dois primeiros algarismos da esquerda do que ela digitou na 2006a linha?

PROBLEMAS – Nível 2 PARTE A


(Cada problema vale 4 pontos)

01. Esmeralda posicionou todos os números naturais de 1 a 2006 no seguinte


arranjo em forma de pirâmide:
21
20 13 22
19 12 7 14 23
18 11 6 3 8 15 24
17 10 5 2 1 4 9 16 25

Em qual andar se encontrará o número 2006? (Por exemplo: o número 1 está no


primeiro andar, o 6 no segundo andar e o 23 no terceiro).
02. A soma dos quadrados de três inteiros consecutivos é igual a 302. Qual é a
soma desses números?
03. Seja ABC um triângulo retângulo em A. Considere M e N pontos sobre a
hipotenusa BC tais que CN = NM = MB. Os pontos X e Y são tais que XA = AM e
YA = AN. Determine a área do quadrilátero XYBC, sabendo que o triângulo ABC
tem área 12 cm2.
C

A B

04. Um tabuleiro de xadrez 8 × 8 será decomposto em retângulos que satisfazem


simultaneamente as seguintes propriedades:
(i) cada retângulo possui um número inteiro de casas;
(ii) os diversos retângulos possuem números de casas distintos entre si;
(iii) cada retângulo possui a mesma quantidade de casas brancas e pretas.

EUREKA! N°26, 2007

17
Sociedade Brasileira de Matemática

Qual é o maior número de retângulos que pode ter a decomposição do tabuleiro?

05. A partir de uma terna ordenada (a, b, c), obtemos uma seqüência de ternas
através de sucessivas transformações do tipo:
(a, b, c) → (a2 ⋅ b, a – b + c, b – c).
Por exemplo, a partir da terna (1, 2, 3), obtemos a seguinte seqüência:
(1, 2, 3) → (2, 2, –1) → (8, –1, 3) → (–64, 12, –4) ...
Se começarmos com (1, 1, 1) como a primeira terna ordenada de uma seqüência,
qual será a soma dos três termos da terna que ocupará a 2006a posição nesta
seqüência?

PROBLEMAS – Nível 2 PARTE B


(Cada problema vale 10 pontos)
PROBLEMA 1
Na Rua do Gengibre, existem n casas numeradas de 1 a n ( n ∈ ). As casas de
numeração par ficam todas de um mesmo lado da rua, com as casas de numeração
ímpar do lado oposto. O prefeito Ludmilson Amottarim resolveu derrubar
alguma(s) casa(s) a fim de que as somas dos números das casas fossem iguais dos
dois lados da rua. Para atingir o seu objetivo, qual é o número mínimo de casas
que o prefeito deve derrubar se:
a) a rua tem n = 15 casas?
b) a rua tem n = 16 casas?
c) a rua tem n = 2006 casas?

PROBLEMA 2 No triângulo ABC isósceles abaixo, I é o encontro das bissetrizes e


H é o encontro das alturas. Sabe-se que ∠HAI = ∠HBC = α. Determine o
ângulo α.
B

A C

EUREKA! N°26, 2007

18
Sociedade Brasileira de Matemática

PROBLEMA 3
Sejam a e b números reais distintos tais que a2 = 6b + 5ab e b2 = 6a + 5ab.
a) Determine o valor de a + b.
b) Determine o valor de ab.

PROBLEMA 4
Todos os inteiros de 1 a 2006 são escritos num quadro. Então, cada um destes
números é substituído pela soma de seus algarismos. Estas substituições são
realizadas repetidas vezes até que tenhamos 2006 números com 1 algarismo cada.
Dos números que restaram no quadro, qual aparece mais vezes: o 1 ou o 2?

PROBLEMAS – Nível 3 PARTE A


(Cada problema vale 4 pontos)
01. O par ordenado (83; 89) é chamado de par centenário porque 83 + 8 + 9 = 89
+ 8 + 3 = 100, isto é, a soma de cada número com os dígitos do outro número é
100. Quantos são os pares centenários?
02. Na figura a seguir, o pentágono regular ABCDE e o triângulo EFG estão
inscritos na circunferência Co, e M é ponto médio de BC. Para qual valor de α ,
em graus, os triângulos EFG e HIG são semelhantes?

A
G
α I
H
B E

M
F Co

C D

03. Esmeralda e Jade correm em sentidos opostos em uma pista circular,


começando em pontos diametralmente opostos. O primeiro cruzamento entre elas
ocorre depois de Esmeralda ter percorrido 200 metros. O segundo cruzamento
ocorre após Jade ter percorrido 350 metros entre o primeiro e o segundo ponto de
encontro. As velocidades das moças são constantes. Qual é o tamanho da pista,
em metros?
04. Qual a maior quantidade de lados que pode ter uma secção determinada por
um plano em um octaedro regular?

EUREKA! N°26, 2007

19
Sociedade Brasileira de Matemática

05. Ao jogarmos uma certa quantidade de dados cúbicos com faces numeradas de
1 a 6, a probabilidade de obtermos soma dos pontos 2006 é igual à probabilidade
de obtermos soma dos pontos S. Qual é o menor valor possível de S?

PROBLEMAS – Nível 3 PARTE B


(Cada problema vale 10 pontos)

PROBLEMA 1
Seja n inteiro positivo. De quantas maneiras podemos distribuir n + 1 brinquedos
distintos para n crianças de modo que toda criança receba pelo menos um
brinquedo?

PROBLEMA 2
Encontre todos os pares de inteiros positivos (a; b) tais que (a + 1)(b + 1) é
múltiplo de ab + 1.

PROBLEMA 3
No triângulo ABC tem-se AB = 4, AC = 3 e o ângulo BÂC mede 60o. Seja D o
ponto de intersecção entre a reta perpendicular a AB passando por B e a reta
perpendicular a AC passando por C. Determine a distância entre os ortocentros
dos triângulos ABC e BCD.

PROBLEMA 4
A seqüência Fn é definida por F1 = F2 = 1 e Fn = Fn – 1 + Fn – 2 para n ≥ 3. Encontre
todos os pares de inteiros positivos (m, n) tais que Fm ⋅ Fn = mn.

Soluções Nível 1 – Segunda Fase – Parte A

Problema 01 02 03 04 05 06
Resposta 5 92 17 6 125 60
2 2 23 2 4 22005 22006
01. + + + + + = 2 + 2 + 2 + + 2 + 2 = 2005 ⋅ 2 = 4010 .
2 2 2 23 22004 22005 2005 parcelas iguais
A soma dos algarismos desse número é 4 + 0 + 1 + 0 = 5 .

02. Como 20% da massa total dessa pessoa correspondem à massa de gordura, ela
tem 20% ⋅ 100 = 20 kg de gordura. Ela perdeu 40% da sua gordura, ou seja,
perdeu 40% ⋅ 20 = 8 kg de gordura, e como manteve os demais índices, ela pesava
ao final do regime 100 − 8 = 92 kg.

EUREKA! N°26, 2007

20
Sociedade Brasileira de Matemática

03. A soma dos algarismos dos números de dois algarismos varia de 1 a 18.
Dessas somas, as que são quadrados perfeitos são 1, 4, 9 e 16. Temos então
• Soma 1: número 10
• Soma 4: números 13, 22, 31 e 40
• Soma 9: números 18, 27, 36, 45, 54, 63, 72, 81 e 90
• Soma 16: números 79, 88 e 97
Portanto, nas condições propostas, há 17 números.

04. A quantidade inicial de algarismos é 9 + 2 × 90 = 189 , dos quais 94 aparecem


nas posições pares e 95 nas posições ímpares. Apagados os algarismos que
aparecem nas posições pares, sobram 95 algarismos; desses, 47 estão nas
posições pares e 48 nas posições ímpares. Repetindo a operação, restam 48
algarismos, sendo 24 algarismos em posições pares e 24 em posições ímpares. Na
terceira aplicação da operação restam 12 algarismos e, na quarta, sobram 6
algarismos.

05. Como a área da folha é 300cm2, cada quadrado destacado tem área
300
= 25 cm2 e, portanto, lado medindo 5cm. Logo o volume desse cubo é
12
5 3 = 125 cm3.

06. A soma dos 27 números escritos na tabela é igual a


3 vezes X e a 9 vezes o Y. Como X é a soma dos 1 5 9 Y
números de cada coluna, temos 2 6 7 Y
X = 1 + 2 + 3 + + 9 = 45 . Portanto 3 4 8 Y
3 ⋅ (1 + 2 + 3 + + 9) = 9 ⋅ Y ⇔ 3 ⋅ 45 = 9 ⋅ Y ⇔ Y = 15 4 9 2 Y
5 7 3 Y
Logo X + Y = 45 + 15 = 60 . O desenho ao lado 6 8 1 Y
mostra uma forma de escrever os números na tabela. 7 2 6 Y
8 3 4 Y
9 1 5 Y
X X X

Soluções Nível 1 – Segunda Fase – Parte B

SOLUÇÃO DO PROBLEMA 1:
a) Há 999 − 100 + 1 = 900 números de três algarismos, escritos em cartões
amarelos, e 9999 − 1000 + 1 = 9000 números de quatro algarismos, escritos em
cartões azuis. Ao todo, foram utilizados 900 + 9000 = 9900 cartões.
b) Como existe a possibilidade de serem retirados todos os cartões amarelos antes
de aparecer algum azul, para Jade ter certeza de que há dois cartões azuis entre os

EUREKA! N°26, 2007

21
Sociedade Brasileira de Matemática

retirados ela deverá retirar 900 + 2 = 902 cartões.

SOLUÇÃO DO PROBLEMA 2:
Como cada quadradinho tem 1 cm2 de área, o lado de cada um mede 1 cm.

a) Há 20 quadradinhos pintados de cinza. Logo a área da figura formada é


20 ⋅ 1 cm 2 = 20 cm 2 e como há 8 segmentos verticais à esquerda e 8 à direita
além de 9 segmentos horizontais pela parte de cima e 9 pela debaixo, o perímetro,
que é a soma das medidas de todos os lados, é 2 ⋅ 8 + 2 ⋅ 9 = 16 + 18 = 34 cm .
b) O quadriculado inteiro é um retângulo de lados 8 cm e 9 cm, e portanto de
perímetro 2 ⋅ 8 + 2 ⋅ 9 = 16 + 18 = 34 cm . Deste modo, o valor máximo da área que
podemos obter é quando a figura for igual a todo o quadriculado e, assim, a área
será 8 ⋅ 9 = 72 cm 2 .

SOLUÇÃO DO PROBLEMA 3:
a) Ela escreveu em cada uma das 9 primeiras linhas, na seguinte ordem, 1, 11, 21,
1112, 3112, 211213, 312213, 212223 e 114213. Logo na 10ª. linha ela escreveu
31121314.
b) Esmeralda escreveu em cada uma das primeiras linhas, na seguinte ordem, 01,
1011, 1031, 102113, 10311213, 10411223, 1031221314, 1041222314,
1031321324, 1031223314, 1031223314,..., e percebeu que, a partir da 10ª. linha,
o número 1031223314 começa a repetir.
Portanto os dois primeiros algarismos da esquerda do número que ela digitou na
2006a. linha serão 1e 0.

Soluções Nível 2 – Segunda Fase – Parte A

Problema 01 02 03 04 05
Resposta 20 30 ou – 30 ou ± 30 32 07 00

EUREKA! N°26, 2007

22
Sociedade Brasileira de Matemática

01. Os números da coluna do meio podem ser dados por: 1 + 2 + 4 + 6 + 8 +...+


2n = n2 + n + 1. Dessa forma o número do topo é: 442 + 44 + 1 = 1981. Como
1981 está no 45° andar, e 2006 –1981 = 25, 2006 deve estar no 20° andar.

02. Podemos representar os três inteiros consecutivos por n − 1, n e n + 1 .


Temos
( n −1) + n2 + ( n + 1) = 302 ⇔ n2 − 2n + 1 + n2 + n2 + 2n + 1 = 302 ⇔ 3n2 + 2 = 302
2 2

⇔ 3n2 = 300 ⇔ n2 = 100 ⇔ n = −10 ou n = 10

Portanto, os três inteiros consecutivos são −11, −10 e − 9 ou 9,10 e 11 .


Se admitirmos que estamos falando de inteiros positivos, a resposta é
9 + 10 + 11 = 30 .
Rigorosamente falando a resposta deveria ser: se os inteiros são positivos, então a
sua soma é 30 e se os inteiros são negativos, então sua soma é –30.

Pontuação: 4 pontos para 30 ou para – 30 ou para ± 30.

03.

Observe que os triângulos AXY e ANM são congruentes, e <YXA = <AMN. Assim,
XY || MN e como XY = MN = MC = NB, segue que os quadriláteros XYCM e
XYNB são paralelogramos, como A é ponto médio de XM e NY temos que [AYC]
= [BAX] = (2/3).12 = 8. Logo, [XYCB] = (8/3).12 = 32.

04. Cada retângulo da decomposição possui um número par de casas, pois possui
a mesma quantidade de casas brancas e pretas. Veja que a maior quantidade de
números pares distintos tais que a soma não supera 64 é 2 + 4 + 6 + 8 + 10 + 12 +

EUREKA! N°26, 2007

23
Sociedade Brasileira de Matemática

14 = 56, pois 2 + 4 + 6 + 8 + 10 + 12 + 14 + 16 = 72, ou seja, a soma de 8


números pares distintos é sempre maior que 64. Portanto, a decomposição pode
ter no máximo 7 retângulos. Abaixo uma decomposição com 7 retângulos.

05. Fazendo as primeiras transformações, obtemos a seguinte seqüência:


(1, 1, 1) → (1, 1, 0) → (1, 0, 1) → (0, 2, –1) → (0, –3, 3) → (0, 6, –6) → ...
Primeiramente, vemos que a partir da quarta terna, o primeiro vai ser sempre
igual a 0 (zero). Então, a partir desta terna, as transformações são do tipo: (0, b, c)
→ (0, – b + c, b – c). Logo, a partir da quarta terna ordenada da seqüência, a
soma dos termos de todas as ternas será igual a 0 – b + c + b – c = 0.
Logo, a soma dos três termos da terna que ocupará a 2006ª posição nesta
seqüência é igual a 0 (zero).

Soluções Nível 2 – Segunda Fase – Parte B

SOLUÇÃO DO PROBLEMA 1:
Vamos usar a notação:
S_par = soma de todas as casas de numeração par;
S_ímpar = soma de todas as casas de numeração ímpar.

a) Para este caso, temos: S_par = 2 + 4 + 6 + 8 + 10 + 12 + 14 = 56 e S_ímpar =


1 + 3 + 5 + 7 + 9 + 11 + 13 + 15 = 64. Como a diferença entre as somas é par e
S_ímpar > S_par, há a necessidade de retirar pelo menos duas casas do lado
ímpar como, por exemplo, as casas de numeração 7 e 1. Aí, teremos S_par =
S_ímpar = 56. Assim, o prefeito deve derrubar pelo menos 2 casas.

b) Para este caso, temos: S_par = 2 + 4 + 6 + 8 + 10 + 12 + 14 +16 = 72 e


S_ímpar = 1 + 3 + 5 + 7 + 9 + 11 + 13 +15 = 64. Como a diferença entre as
somas é par e S_par > S_ímpar, pode-se retirar apenas uma casa do lado par: a
casa de numeração 8.
Aí, teremos S_par = S_ímpar = 64. Assim, o prefeito deve derrubar 1 casa.

c) Para este caso, temos: S_par = 2 + 4 + 6 + 8 + 10 + ... + 2006 e S_ímpar = 1 +


3 + 5 + ... + 2005. Assim, temos S_par – S_ímpar = (2 – 1) + (4 – 3) + ... + (2006

EUREKA! N°26, 2007

24
Sociedade Brasileira de Matemática

– 2005) = 1003. Como 1003 é ímpar, uma única casa não é suficiente, mas retirar
as casas de numeração 1006 e 3 basta para que S_par = S_ímpar. Assim, o
número mínimo de casas que o prefeito deve derrubar é 2 casas.

SOLUÇÃO DO PROBLEMA 2:
B

I
Q
H
A C

Como o triângulo é isósceles concluímos que, ∠CBM = ∠ABM e ∠ACB = 90o –


α, com isso, ∠CAQ = α, pois AQ é uma altura. Como AI é bissetriz, então ∠CAI
= ∠IAB = 2α. Finalmente no ∆ AMB: α + α + 2α + α = 90o ⇒ α = 18o.

SOLUÇÃO DO PROBLEMA 3:
a) Subtraindo as duas equações dadas temos a 2 − b 2 = 6(b − a ) ou seja
(a − b)(a + b + 6) = 0 . Como a ≠ b , temos a + b = −6 .
b) Da parte a), elevando ao quadrado, a 2 + b 2 + 2ab = 36 . Mas, somando as
equações dadas, temos a 2 + b 2 = 6(a + b) + 10ab = −36 + 10ab . Portanto,
−36 + 2ab + 10 ab = 36 o que dá ab = 6.

SOLUÇÃO DO PROBLEMA 4:
Quando trocamos um inteiro positivo pela soma de seus algarismos, não
alteramos o resto da divisão por 9. Isto é explicado pela decomposição do inteiro
na forma:
abcd = 1000a + 100b + 10c + d = 999a + 99b + 9c + a + b + c + d
Daí, temos que:
abcd – ( a + b + c + d ) = 999a + 99b + 9c = 9(111a + 11b + c)
Logo, abcd e a + b + c + d deixam o mesmo resto na divisão por 9.
Como todos os números que restaram no quadro estão entre 0 e 9, inclusive,
todos os números 1 restantes no quadro são originados a partir de números que
deixam resto 1 na divisão por 9 (1, 10, 19, 28, 37, ..., 1999). Da mesma forma,

EUREKA! N°26, 2007

25
Sociedade Brasileira de Matemática

todos os números 2 restantes no quadro são originados a partir de números que


deixam resto 2 na divisão por 9 (2, 11, 20, 29, 38, ..., 2000). Comparando, vemos
que cada um dos números 1 e 2 aparece 223 vezes no quadro. Portanto, ambos os
números 1 e 2 aparecem o mesmo número de vezes.

Soluções Nível 3 – Segunda Fase – Parte A

Problema 01 02 03 04 05
Resposta 9 36 750 6 339

01. Sejam a, b, c e d algarismos tais que o par (ab, cd) é centenário. Então,
(10a + b) + (c + d ) = (10c + d ) + ( a + b) = 100
como b + c + d ≤ 27 , 10a ≥ 73 , e assim a ≥ 8 e, de modo análogo, c ≥ 8 . Ainda
mais,
(10a + b) + (c + d ) = (10c + d ) + (a + b) ⇔ 9a = 9c ⇔ a = c .
Temos então 2 casos:
I) a = c = 8 ⇒ 80 + b + 8 + d = 100 ⇔ b + d = 12 , sendo esta uma condição
necessária e suficiente para o par em questão ser centenário. Obtemos assim os 7
seguintes pares:
(83;89), (84;88), (85;87), (86;86), (87;85), (88;84) e (89;83).
II) a = c = 9 ⇒ 90 + b + 9 + d = 100 ⇔ b + d = 1 , obtendo outros 2 pares
centenários:
(90;91) e (91;90).
Há, assim, 9 pares centenários.
02. Seja J a interseção dos segmentos BC e FG. Como M é ponto médio do
segmento BC, oposto ao vértice E, conclui-se que EF é diâmetro, e
∠FGE = ∠BMF = 90° . Sendo ABCDE um pentágono regular, ∠ABC = 108° .
No ∆GHI : ∠GHI = α ⇒ ∠GIH = 90° − α .
No ∆BJH : ∠BHJ = α ⇒ ∠BJH = 72° − α .
No ∆FJM : ∠FJM = 72° − α ⇒ ∠JFM = 18° + α .
Para que os triângulos EFG e HIG sejam semelhantes, como α ≠ 18° + α , a única
possibilidade é termos 90° − α = 18° + α ⇔ α = 36° .

03. No momento do primeiro cruzamento, Esmeralda e Jade percorreram a


distância total igual à metade da extensão da pista. Entre o primeiro e o segundo
cruzamento, as moças percorreram uma distância total igual à extensão da pista.
Portanto Esmeralda correu o dobro da distância que correu até o primeiro
cruzamento, ou seja, 2 ⋅ 200 = 400 metros e, deste modo, a extensão da pista é
400 + 350 = 750 metros.

EUREKA! N°26, 2007

26
Sociedade Brasileira de Matemática

Até o primeiro encontro Entre o primeiro e o segundo encontro

04. Considere os três planos que passam pelo centro do octaedro e contêm 4 das
12 arestas do octaedro, formando três quadrados. A secção corta no máximo dois
lados de cada quadrado. Portanto corta no máximo 6 arestas do octaedro. Assim,
a maior quantidade de lados que uma secção pode determinar no octaedro regular
é 6. Um exemplo de secção hexagonal é um plano paralelo a duas faces opostas.

05. Seja n > 1 a quantidade de dados. Podemos representar um lançamento dos n


dados com a n-upla (a1, a2, …, an), sendo ai o resultado do dado i. Como ai é um
inteiro entre 1 e 6, existe uma bijeção entre os pares (a1, a2, …, an) e (7 – a1, 7 –
a2, …, 7 – an), de modo que a probabilidade de obter soma S = a1 + a2 + … + an é
a mesma de obter soma (7 – a1) + (7 – a2) + … + (7 – an) = 7n – S . Além disso,
para n + 1 ≤ S ≤ 7 n / 2 , a probabilidade de obter soma S – 1 é menor do que a
probabilidade de obter soma S. Portanto as somas distintas S e T têm a mesma
probabilidade de ocorrer se, e somente se, T = 7n – S. Em particular, a única soma
com a mesma probabilidade de ocorrer que a soma 2006 é 7n – 2006.
Como 2006 = 334 ⋅ 6 + 2 , precisamos jogar, no mínimo, 335 dados, ou seja,
n ≥ 335 . Pelo fato acima, o valor procurado é 7 ⋅ 335 − 2006 = 339 .

Soluções Nível 3 – Segunda Fase – Parte B

SOLUÇÃO DO PROBLEMA 1:
Uma solução:
Observe que teremos 1 criança com 2 brinquedos, enquanto cada uma das n – 1
crianças restantes terá apenas 1 brinquedo. Assim, temos n possibilidades para a

EUREKA! N°26, 2007

27
Sociedade Brasileira de Matemática

 n + 1
escolha da felizarda criança, e   possibilidades para escolher os 2
 2 
brinquedos desta criança. Restando n – 1 brinquedos e n – 1 crianças, temos (n –
1)! modos de distribuir estes brinquedos entre estas crianças. Assim, temos um
 n + 1  n + 1
total de n ( n − 1)! =  n! modos de distribuir os n + 1 brinquedos entre
 2   2 
as n crianças.

Outra solução:
Observe que teremos 1 criança com 2 brinquedos, enquanto cada uma das n – 1
crianças restantes terá apenas 1 brinquedo. Temos n escolhas para a criança que
terá dois brinquedos. Escolhida tal criança, o número de maneiras de distribuir os
n + 1 brinquedos é igual ao número de anagramas da palavra A1A1A2A3…An, que é
(n + 1)! (n + 1)!
= . Assim, o total de maneiras de distribuir os n + 1 brinquedos
2! 2
(n + 1)!
entre as n crianças é n ⋅ .
2

SOLUÇÃO DO PROBLEMA 2:
Uma solução:
ab + 1 (a + 1)(b + 1) ⇒ ab + 1 ab + a + b + 1 ⇒ ab + 1 (ab + a + b + 1) − (ab + 1) ⇒
⇒ ab + 1 a + b ⇒ ab + 1 ≤ a + b ⇒ a (b − 1) ≤ b − 1 . Desta última desigualdade,
observamos que, se b > 1 , então a ≤ 1 ⇒ a = 1 , ou seja, um dentre os inteiros a e
b vale 1. Suponha, então, sem perda de generalidade, que a = 1. Substituindo,
obtemos a = 1 ⇒ b + 1 2(b + 1) , o que é válido para todo inteiro positivo b. As
soluções são, então, (1, b) e (a, 1).

Outra solução:
Como (a + 1)(b + 1) é múltiplo de ab + 1, existe um inteiro positivo k tal que
(a + 1)(b + 1) = k (ab + 1) ⇔ a( kb − b − 1) = b − k + 1 . Se kb – b – 1 = 0, então
1
k = 1 + , que é inteiro se, e somente se, b = 1. Se kb − b − 1 ≠ 0 então
b
b − k +1
a= ≥ 1 ⇒ b − k + 1 ≥ kb − b − 1 ⇔ k ≤ 2 . Se k = 1, obtemos a = – (b + 1)
kb − b − 1
< 0. Logo k = 2 e a = 1. Verifica-se que (1, b) e (a, 1) são realmente as soluções.

EUREKA! N°26, 2007

28
Sociedade Brasileira de Matemática

SOLUÇÃO DO PROBLEMA 3:
Uma solução:
Sejam A’ o ortocentro do triângulo BCD e D’ o ortocentro do triângulo ABC.
C A’

D’
D

60o
A B

Como as retas CD’ e BD são ambas perpendiculares a AB, são paralelas.


Analogamente, as retas BD’ e CD são paralelas. Logo o quadrilátero BDCD’ é
um paralelogramo e, portanto, os triângulos BCD e BD’C são congruentes.
Da mesma maneira, as retas AB e CA’ são paralelas, pois são perpendiculares a
BD. Analogamente, as retas AC e BA’ são paralelas. Logo o quadrilátero CABA’ é
um paralelogramo e, assim, os triângulos ABC e A’CB são congruentes.
Conseqüentemente, os quadriláteros ABDC e A’CD’B são congruentes, de modo
que a distância entre os ortocentros A’D’ é igual a AD.
Devemos, então, calcular AD. Como os ângulos ABˆ D e ACˆ D são ambos retos,
somam 180o e, portanto, o quadrilátero ABCD é inscritível, sendo AD diâmetro de
seu circuncírculo.
C

3 D

60o
A 4 B

Pela lei dos co-senos,


1
BC 2 = AB 2 + AC 2 − 2 ⋅ AB ⋅ AC ⋅ cos 60 ⇔ BC 2 = 4 2 + 32 − 2 ⋅ 4 ⋅ 3 ⋅ ⇔ BC = 13
2
Enfim, pela lei dos senos,
BC 13 2 39
AD = 2 R = = =
sen 60 3 3
2

EUREKA! N°26, 2007

29
Sociedade Brasileira de Matemática

2 39
e, portanto, a distância entre os ortocentros é .
3
Outra solução:
Sejam A’ o ortocentro do triângulo BCD e D’ o ortocentro do triângulo ABC.

y C A’

D’
D

60o
x
A B

Sejam A = (0;0) e B = (4;0). Sendo AC = 3 e m(BÂC) = 60o, podemos supor que C


3 3 3
= (3 cos 60 ;3 sen 60 ) =  ;  . Como a reta CD’ é perpendicular ao eixo x,
2 2 
 
3
admite equação x = . Além disso, sendo a reta BD’ perpendicular à reta AC, de
2
−1
coeficiente angular tg 60 = 3 , seu coeficiente angular é . Logo, sendo
3
 3  a−0 1 5 3
D' =  ; a  , 3 =− ⇔a= .
2  2 −4 3 6
Calculemos agora A’. Como A’ pertence à perpendicular a BD por C, então
 3 3
A' =  b;  . A reta CD é perpendicular a AC e, portanto, tem coeficiente

 2 
−1
angular . Enfim, sendo A’B perpendicular a CD, tem coeficiente angular
3
−1
3 3
−0 11
= 3 . Deste modo, 2
= 3⇔b= .
−1
3
b−4 2
Logo a distância entre os ortocentros A’ e D’ é
2
 11 3   3 3 5 3 
2
2 39
 −  + − = .
 2 2   2 6  3

EUREKA! N°26, 2007

30
Sociedade Brasileira de Matemática

SOLUÇÃO DO PROBLEMA 4:
Os primeiros valores da seqüência são:
F1 = 1, F2 = 1, F3 = 2, F 4 = 3, F 5 = 5, F 6 = 8, F 7 = 13, F 8 = 21, F 9 = 34
Nota-se que, para n > 7, Fn > 2n. De fato, indutivamente, se Fn > 2n e Fn + 1 > 2(n
+ 1) então Fn + 2 = Fn + 1 + Fn > 2(n + 1) + 2n > 2(n + 2).
3n 4n
Portanto Fn > 2n > > > n para n > 7, de modo que para resolver as
2 3
3n 4n
equações Fn = n, Fn = , Fn = , basta testar os valores de n menores ou
2 3
iguais a 7.
Se n > 5, de Fm ⋅ Fn = mn devemos ter Fm < m, donde m < 5. Logo pelo menos um
dos números m e n deve ser no máximo 5. Suponha, sem perda de generalidade, n
≤ 5. Observando os possíveis valores de n:
• n = 1 ⇒ Fm = m, cujas soluções são m = 1 e m = 5.
• n = 2 ⇒ Fm = 2m, que não possui solução.
• n = 3 ⇒ 2Fm = 3m, que não possui solução.
• n = 4 ⇒ 3Fm = 4m, que possui a única solução m = 6.
• n = 5 ⇒ Fm = m, cujas soluções são m = 1 e m = 5.

Os pares (m, n) que satisfazem a relação pedida são:


(1, 1), (1, 5), (4, 6), (5, 1), (5, 5) e (6, 4).

EUREKA! N°26, 2007

31
Sociedade Brasileira de Matemática

XXVIII OLIMPÍADA BRASILEIRA DE MATEMÁTICA


Problemas e Soluções da Terceira Fase

PROBLEMAS – NÍVEL 1
PROBLEMA 1
Considere as seguintes seqüências:

S1: 12345678, 81234567, 78123456, ..., na qual o último algarismo do termo


anterior (algarismo das unidades) torna-se o primeiro algarismo à esquerda do
próximo termo.

S2: 1234567898765, 5612345678987, 7856123456789, ..., na qual o algarismo


das unidades torna-se o primeiro algarismo à esquerda do próximo termo, e o das
dezenas torna-se o segundo algarismo à esquerda.
a) Apresente o quinto termo da seqüência S1 e o quarto termo da seqüência S2.
b) A seqüência S1 tem 2006 termos. Qual é o seu último termo?
c) A seqüência S2 termina quando o primeiro termo se repete. Quantos termos
tem essa seqüência?

PROBLEMA 2
Na adição abaixo, cada símbolo representa um único algarismo e símbolos
diferentes representam algarismos diferentes.

Determine o valor de cada símbolo, ou seja, descubra tais valores e mostre que
não existem outras possibilidades.

PROBLEMA 3
Um atirador lança flechas no alvo representado
ao lado. Os números indicam a pontuação
obtida em cada região atingida pela flecha (se a
flecha acertar exatamente uma linha, a
pontuação é a menor das duas regiões). Note
que a região fora do retângulo não rende
pontos.

EUREKA! N°26, 2007

32
Sociedade Brasileira de Matemática

a) Se numa competição, cada participante atira 2 flechas, quantas pontuações


diferentes podem ser obtidas?
b) Numa outra competição, cada participante atirou 3 flechas.
Curiosamente, não houve empates e todas as pontuações possíveis foram
atingidas. Quantos participantes havia nesta competição?

PROBLEMA 4
Dentre os polígonos de 5 lados, o maior número possível de vértices
alinhados, isto é, pertencentes a uma única reta, é três, como mostrado a seguir.

Qual é a maior quantidade de vértices alinhados que um polígono de 12 lados


pode ter?

Atenção: além de desenhar um polígono de 12 lados com o número


máximo de vértices alinhados, lembre-se de mostrar que não existe um outro
polígono de 12 lados com mais vértices alinhados do que este.

PROBLEMA 5
A partir do tabuleiro mostrado nas figuras abaixo e quatro peças, duas circulares
cinzas e duas quadradas pretas, Esmeraldinho inventou o seguinte jogo:
• Inicialmente, as peças são colocadas no tabuleiro como mostra a figura 1.

Figura 1

EUREKA! N°26, 2007

33
Sociedade Brasileira de Matemática

• A meta do jogo é, após um certo número de movimentos, trocar as peças


de posição, chegando na situação mostrada na figura 2.

Figura 2

• Cada movimento consiste em mover uma das


quatro peças uma ou mais casas acima, abaixo, à
esquerda ou à direita; todavia, tal peça não pode A
“pular” nenhuma peça que, eventualmente, esteja
no caminho, ou ocupar uma casa onde já existe
uma peça. Por exemplo, a peça marcada com A só
pode se mover para alguma das casas destacadas
em cinza.

• Os movimentos dos círculos e dos quadrados são alternados. O jogo


começa com um movimento de um dos quadrados.

Determine a menor quantidade total de movimentos necessários para terminar o


jogo. Mostre, passo-a-passo, através de desenhos, como movimentar as peças
com esta quantidade de movimentos e prove que não é possível terminar o jogo
com menos movimentos.

PROBLEMAS – NÍVEL 2

PROBLEMA 1
Escrevemos, em fila, os números 1, 2, 3, …, n. A cada passo, tomamos os dois
últimos números da fila anterior, escrevemos primeiramente o último, depois
o penúltimo e, enfim, os outros
n – 2, na ordem em que aparecem. Por exemplo, para n = 12 obtemos
1, 2, 3, 4, 5, 6, 7, 8, 9, 10, 11, 12 → 12, 11, 1, 2, 3, 4, 5, 6, 7, 8, 9, 10
→ 10, 9, 12, 11, 1, 2, 3, 4, 5, 6, 7, 8 → ...
Qual a menor quantidade de passos necessários para escrevermos
novamente os números 1, 2, 3, …, n, nessa ordem, quando
(a) n = 2006?
(b) n = 2005?

EUREKA! N°26, 2007

34
Sociedade Brasileira de Matemática

PROBLEMA 2
Veja o problema No. 4 do Nível 1

PROBLEMA 3
Encontre todos os pares ordenados (x; y) de inteiros tais que x3 – y3 = 3(x2 – y2).

PROBLEMA 4
Quantos subconjuntos {a, b, c} de três elementos distintos de {1, 2, 3, …, 100}
são tais que b é a média aritmética de a e c (a < b < c)?

PROBLEMA 5
Seja ABC um triângulo acutângulo e H o seu ortocentro. Sejam M, N e R os
pontos médios de AB, BC e AH, respectivamente. Determine a medida do ângulo
MNˆ R se o ângulo ABˆ C mede 70o.

PROBLEMA 6
Em um torneio de tênis de mesa (no qual nenhum jogo termina empatado), cada
um dos n participantes jogou uma única vez contra cada um dos outros. Sabe-se
que, para todo k > 2, não existem k jogadores J1, J2, …, Jk tais que J1 ganhou de
J2, J2 ganhou de J3, J3 ganhou de J4, …, Jk – 1 ganhou de Jk, Jk ganhou de J1.

Prove que existe um jogador que ganhou de todos os outros e existe um jogador
que perdeu de todos os outros.

PROBLEMAS – NÍVEL 3
PROBLEMA 1
Seja ABC um triângulo, P o pé da bissetriz interna relativa ao lado AC e I
seu incentro. Se AP + AB = CB, prove que API é um triângulo isósceles.

PROBLEMA 2
Seja n um inteiro, n ≥ 3 . Definimos f(n) como a maior quantidade possível de
triângulos isósceles cujos vértices pertencem a algum conjunto de n pontos do
plano sem três pontos colineares. Prove que existem constantes positivas a e b
tais que an2 < f(n) < bn2, para todo n inteiro, n ≥ 3 .

PROBLEMA 3
Determine todas as funções f : R → R tais que
f ( xf ( y ) + f ( x) ) = 2 f ( x) + xy
para todos x, y reais.

EUREKA! N°26, 2007

35
Sociedade Brasileira de Matemática

PROBLEMA 4
Um número inteiro positivo é arrojado quando tem 8 divisores positivos cuja
soma é 3240. Por exemplo, o número 2006 é arrojado porque seus 8 divisores
positivos, 1, 2, 17, 34, 59, 118, 1003 e 2006, somam 3240. Encontre o menor
número inteiro positivo arrojado.

PROBLEMA 5
Seja P um polígono convexo de 2006 lados. As 1003 diagonais ligando vértices
opostos e os 1003 segmentos que ligam os pontos médios dos lados opostos são
concorrentes, ou seja, todos os 2006 segmentos possuem um ponto em comum.
Prove que os lados opostos de P são paralelos e congruentes.

PROBLEMA 6
O professor Piraldo participa de jogos de futebol em que saem muitos gols e tem
uma maneira peculiar de julgar um jogo. Um jogo com placar de m gols a n gols,
m ≥ n , é dito equilibrado quando m ≤ f (n) , sendo f(n) definido por f(0) = 0 e,
para n ≥ 1 , f (n) = 2n − f ( r ) + r , onde r é o maior inteiro tal que r < n e
f (r ) ≤ n .
1+ 5
Sendo φ = , prove que um jogo com placar de m gols a n, m ≥ n , está
2
equilibrado se m ≤ φn e não está equilibrado se m ≥ φn + 1 .

SOLUÇÕES – NÍVEL 1

SOLUÇÃO DO PROBLEMA 1: BRUNO SILVA MUCCIACCIA (VITÓRIA – ES)


S1: 12345678, 81234567, 78123456, 67812345, 56781234

a) 5º termo da seqüência S1: 56781234


4º termo da seqüência S2: 9878561234567

S2: 1234567898765, 5612345678987, 7856123456789, 9878561234567.

b) O último termo da seqüência S1 é: 45678123


Pois quando se aumenta de 8 em 8, a seqüência se repete, então o 2001º número é
igual ao 1º. E o 2006º número é igual ao 6º.

c) Essa seqüência tem 43 termos, pois se percebe que quando as posições


aumentam de 7 em 7, os números ímpares permanecem no lugar e os pares
andam 2 casas para a direita, só quando o número par estiver na penúltima

EUREKA! N°26, 2007

36
Sociedade Brasileira de Matemática

posição que ele anda três casas; assim, como há 13 algarismos andando 5
vezes duas casas, os algarismos pares andam 10 casas para a direita, mas, no
final da seqüência, andam três casas, então andando 6 × 7 vezes, eu repito a
sequência. Como começa no No.1, na seqüência há 6 × 7 + 1 termos igual a
43. (Pois a seqüência acaba quando o número se repete).

SOLUÇÃO DO PROBLEMA 2: SOLUÇÃO DA BANCA

ab
bc
+ ca
abc
Como abc < 3 × 99 = 297 ⇒ a = 0, a = 1 ou a = 2.
Como a + b + c ≡ c (mod 10) ⇒ a + b ≡ 0 (mod 10) ⇒ b + 1 ≡ 0 (mod 10) ou
b + 2 ≡ 0 (mod 10), temos b = 8 ou b = 9 (não podemos ter a = b = 0).
Como a + b + c + 1 = 10 a + b, temos c + 1 = 9a.
Se a = 1, então c = 8
Se a = 2, c = 17 (não é possível!).
Logo, a única solução é a = 1, b = 9 e c = 8.

SOLUÇÃO DO PROBLEMA 3: COLABORAÇÃO DE RÉGIS P. BARBOSA (FORTALEZA – CE)


a) Vejamos se é possível obter uma mesma pontuação de dois jeitos diferentes:
(a, b) e (c, d), com a ≥ b, c ≥ d e a + b = c + d . Se tivermos
a = c, a + b = a + d ⇒ b = d , e logo ( a , b ) ⇒ (c , d ). Agora suponhamos sem
perda de generalidade a > c. Teremos a ≤ a + b = c + d ≤ 2c, mas, se olharmos
as pontuações nas regiões, podemos observar que a menor pontuação possível
maior que c é 3c ou 3c + 1. No caso c = 0, 1 ≤ a ≤ 2c = 0. Absurdo! E se
c > 0, 3c ≤ a ≤ 2c ⇒ c ≤ 0, absurdo!
Assim só tem um jeito para cada pontuação possível (a, b) com a ≥ b, dada a
soma S = a + b. Agora vamos contá-las.
Se a = b, pode ser: ( 0, 0 ) ; (1,1) ; ( 3, 3 ) ; ( 9, 9 ) ; ( 27, 27 ) ; ( 81,81) . Se a > b, basta
escolhermos um par de pontuações distintas pois a ordem está definida. Temos 6
possibilidades para o primeiro número e 5 para o segundo (que não pode ser igual
ao primeiro) e dividimos por 2 já que os pares são contados duas vezes (aparecem

EUREKA! N°26, 2007

37
Sociedade Brasileira de Matemática

6⋅5
tanto {0, 1} quanto {1, 0}), obtendo: = 15 possibilidades. Assim são 6 +
2
15 = 21 pontuações possíveis.
b) Usaremos um raciocínio parecido com o do item (a). Veja quando a mesma
pontuação pode ser obtida de dois modos: ( a , b, c ) e ( d , e, f ), com
a ≥ b ≥ c, d ≥ e ≥ f e a + b + c = d + e + f
Se a = d recai no item (a): (b ; c); (e, f) com b ≥ c, e ≥ f e b + c = e + f já
vimos que nesse caso b = e e c = f.
Assim suponhamos a > d. Temos: a ≤ a + b + c = d + e + f ≤ 3d .
Se d = 0,1 ≤ a ≤ 0. Absurdo!
Assim tomemos d > 0. Se a > d já vimos que a ≥ 3d ; como
a ≤ a + b + c = d + e + f ≤ 3d ≤ a ocorrerão todas as igualdades: b = c = 0 e e =
f = d, e concluímos que as únicas pontuações obtidas de dois modos são:
( 3 x, 0, 0 ) e ( x, x, x ) com x > 0 e 3 x ≤ 81 ⇒ x ≤ 27 já que a é uma pontuação
de uma flecha. Temos agora três casos:
(i) a > b > c : Temos que tomar 3 números distintos, seguindo o raciocínio do
item (a), teremos: 6 ⋅ 5 ⋅ 4 mas dessa vez cada trio aparecerá 6 vezes:
{ x, y , z};{ x, z , y};{ y , x, z};{ y , z , x};{ z , x, y};{ z , y , x} que são tudo a mesma
6 ⋅5⋅4
coisa, assim temos = 20 possibilidades.
6
(ii) a > b = c ou a = b > c : Basta contar os pares e multiplicar por dois pois o
par (a, b) gera (a, a, b) e (a, b, b). Já vimos no item (a) que são 15 pares, assim
temos aqui 30 possibilidades.
(iii) a = b = c: Como deduzimos as somas obtidas por (x, x, x) com 0 < x ≤ 27 ,
que já foram contadas então devemos contar apenas: x = 0 e x = 81, isto é,
somente mais 2 possíveis somas.
Assim, como o número de participantes é igual ao número de somas possíveis
distintas, são: 20 + 30 + 2 = 52 participantes.

SOLUÇÃO DO PROBLEMA 4
Ver solução do problema 2 do nível 2.

EUREKA! N°26, 2007

38
Sociedade Brasileira de Matemática

SOLUÇÃO DO PROBLEMA 5: LEONARDO BURATO FOUREAUX (LINHARES - ES)


Com 8 movimentos.

1 2 3 4

5 6 7 8

Resultado final

Não existe outra maneira de mover as peças com menos


movimentos, pois o mínimo de cada peça para chegar ao
lugar da outra é de 2 movimentos e sendo 4 peças, são no
mínimo 4 ⋅ 2 = 8 movimentos.

SOLUÇÕES – NÍVEL 2

SOLUÇÃO DO PROBLEMA 1: THIAGO S. WARWAR TEIXEIRA (RIO DE JANEIRO – RJ)


a) n = 2006
n
Com n sendo par, a cada passos, partindo do início, nota-se que todos os
2
números ímpares trocarão de posição com o número par a sua frente, depois todos
os pares trocarão de posição com o número ímpar que estiver a sua frente, e assim
sucessivamente.
Logo, com n = 2006, teremos, após 1003 passos:
2, 1, 4, 3, 6, 5..., 2004, 2003, 2006, 2005

EUREKA! N°26, 2007

39
Sociedade Brasileira de Matemática

E, depois de mais 1003 passos, teremos:


1, 2, 3, 4, 5, 6, ..., 2003, 2004, 2005, 2006 (ordem inicial)
Ou seja, para n = 2006, a menor quantidade de passos necessários para
reescrever: 1, 2, 3,..., 2005, 2006, é de 2006 passos.

b) n = 2005
Para resolver esta questão, recorreremos a exemplos menores:
1, 2, 3 → 3, 2, 1 → 1, 2, 3 → 2 passos
1, 2, 3, 4, 5 → 5, 4, 1, 2, 3 → 3, 2, 5, 4, 1 → 1, 4, 3, 2, 5 → 5, 2, 1, 4, 3 → 3, 4, 5,
2, 1 → 1, 2, 3, 4, 5 → 6 passos
Com base nos exemplos, percebe-se que os números ímpares: 1, 3, 5... estarão
sempre nas posições ímpares: 1ª., 3ª., 5ª., ..., não necessariamente nesta ordem,
conseqüentemente, os pares estarão nas posições pares.
Nota-se também que para os x números ímpares reestabelecerem sua ordem
original, deverão ser feitos x passos, e para os (x – 1) números pares, serão
tomados (x – 1) passos.
Logo para achar o número mínimo de passos necessários, devemos calcular o
mínimo múltiplo comum entre x e (x – 1), que independente do valor de x, será
sempre x ⋅ ( x − 1).
Então, para calcular o valor mínimo de passos para n= 2005, devemos multiplicar
1003 (quantidade de ímpares) por 1002 (quantidade de pares), o que resulta em
1005006.

SOLUÇÃO DO PROBLEMA 2: ILLAN FEIMAN HALPERN (ITATIAIA – RJ)


A maior quantidade de vértices alinhados que um polígono de 12 lados pode ter é
8. Um exemplo de polígono assim é:

Mostrarei agora que não existe polígono de 12 lados com 9 vértices colineares.
Em um polígono 3 vértices consecutivos não podem ser colineares, escolhendo 9
pontos em 12, pelo menos haverá um trio de pontos consecutivos. Demostração:
Nomeie os vértices em ordem de V1 a V12 , sendo que V1 é consecutivo de V12 e
V2 ; V2 é consecutivo de V1 e V3 e assim por diante. Sejam:
G1 = {V1 ;V2 ;V3 }

EUREKA! N°26, 2007

40
Sociedade Brasileira de Matemática

G2 = {V4 ;V5 ; V6 }
G3 = {V7 ; V8 ;V9 }
G4 = {V10 ;V11 ;V12 }
Pelo princípio da casa dos pombos, escolhendo-se 9 vértices, haverá pelo menos
um conjunto com 3 vértices escolhidos.

SOLUÇÃO DO PROBLEMA 3: COLABORAÇÃO DE RÉGIS P. BARBOSA (FORTALEZA – CE)


Temos: x − y = 3( x − y ) ⇒ ( x − y )( x + xy + y ) = 3( x − y )( x + y ).
3 3 2 2 2 2

x − y = 0 ⇒ x 3 − y 3 = 0 e 3( x 2 − y 2 ) = 0. Temos assim as soluções


( x, y ) = ( a , a ), ∀a ∈ Z.
Agora suponhamos x − y ≠ 0; assim podemos cortar logo:
x 2 + xy + y 2 = 3 x + 3 y ⇒ x 2 + ( y − 3) x + ( y 2 − 3 y ) = 0, e logo
∆ = ( y − 3) 2 − 4( y 2 − 3 y ) ⇒ ∆ = y 2 − 6 y + 9 − 4 y 2 + 12 y = −3 y 2 + 6 y + 9 =
= −3( y + 1)( y − 3)
Note que x ∈ Z ⇒ ∆ ≥ 0 ⇒ −3( y + 1)( y − 3) ≥ 0 ⇒ ( y + 1)( y − 3) ≤ 0.
Analisando a inequação, se y < −1 ⇒ ( y + 1) < 0, ( y − 3) < 0 ⇒ ( y +1)( y − 3) > 0 e
y > 3 ⇒ ( y − 3) > 0 e ( y + 1) > 0 ⇒ ( y + 1)( y − 3) > 0.
Assim os y´s que buscamos satisfazem: −1 ≤ y ≤ 3, y ∈ Z ⇒ y = −1, 0,1, 2, 3.
Vamos verificar para cada um os possíveis valores de x.
(I) y = −1 ⇒ ∆ = 0 e a equação é: x − 4 x + 4 = 0 ⇒ ( x − 2) = 0, donde
2 2

x = 2 ⇒ ( x, y ) = (2, −1).
(II) y = 0 ⇒ ∆ = 9 e a equação é: x − 3 x = 0 ⇒ x ( x − 3) = 0 , donde x = 0 ou
2

x = 3 ⇒ ( x, y ) = (0, 0), (3, 0).


(III) y = 1 ⇒ ∆ = 12 e a equação é:
−b ± ∆ 2 ± 12
x2 − 2x − 2 = 0 ⇒ x = = ⇒ x = 1 ± 3. Assim, para y = 1,
2a 2
não existe x ∈ Z que satisfaz a equação do segundo grau.
(IV) y = 2 ⇒ ∆ = 9 e a equação é:
−b ± ∆ 1± 3
x2 − x − 2 = 0 ⇒ x = = ⇒ x = 2 ou x = − 1 ⇒ ( x , y ) = (2, 2), ( − 1, 2).
2a 2
(V) y = 3 ⇒ ∆ = 0 e a equação é: x = 0 ⇒ x = 0 ⇒ ( x, y ) = (0, 3).
2

EUREKA! N°26, 2007

41
Sociedade Brasileira de Matemática

Assim os pares ordenados são: ( x , y ) = (2, − 1), ( − 1, 2), (3, 0), (0, 3) e ( a , a ),
∀ a ∈ Z.

SOLUÇÃO DO PROBLEMA 4: RAFAEL HORIMOTO DE FREITAS (SÃO PAULO – SP)


A+C
A soma de A e C deve ser par pois a média entre A e C , , resulta em um
2
inteiro B, para isso A e C devem ser números pares ao mesmo tempo ou devem
ser números ímpares ao mesmo tempo.
No começo podemos escolher 100 A´s diferentes; para cada A ímpar, restam 49
ímpares, e, se A for par, restam 49 pares para escolher no lugar de C, e por último
só há um número B para escolher pois só há uma média aritmética entre A e C.
Em metade dos casos ocorrerá A > C, pois para cada casa em que A < C podemos
trocar os valores de A e C, metade dos casos são inválidos.
100 ⋅ 49 ⋅ 1
No final temos = 2450 subconjuntos.
2
SOLUÇÃO DO PROBLEMA 5: RAFAEL GRIBEL DE PAULA NEVES (RIO DE JANEIRO – RJ)
A

20° θ

R I
M α
20° + θ

H
K
α 20° β
70°– α 70° w w C
B J N

MR é base média do ∆ABH → AM R ≅ ABH


MJ é mediana relativa a AB (e AJB é retângulo) → B AJ ≅ AJ M
MN é base média do ∆ABC → BM N ≅ B AC
Seja K a interseção de BH e MN . No ∆BKM , α + 20° + θ = 90° → NMR = 90°

EUREKA! N°26, 2007

42
Sociedade Brasileira de Matemática

Os triângulos JNR e MNR são retângulos e dividem a mesma base NR → o


quadrilátero MJNR é inscritível → M J R ≅ M N R , donde M N R = 20° .

SOLUÇÃO DO PROBLEMA 6 PARTE A: RENAN HENRIQUE FINDER (JOINVILLE – SC)


Inicialmente, vamos provar o teorema para n = 3. Chamemos de vitorioso o
jogador que venceu os demais. Usemos a notação (X♦Y) para (X venceu Y).
Sejam A, B e C os três jogadores. Suponhamos (sem perda de generalidade)
(A♦B). Se (B♦C), então, como não podemos ter (C♦A), (teríamos
J1 = A, J 2 = B e J 3 = C violando o enunciado), temos A vitorioso. Senão,
temos que (C♦B), ou seja, o vencedor de A contra C é vitorioso.
Suponhamos agora que o enunciado valha para n jogadores. Em um torneio com
os jogadores A1 , A2 , A3 ... e An +1 , haverá um “subtorneio” entre os jogadores
A1 , A2 , A3 ,... e An . Suponhamos sem perda de generalidade que seja A1 é o
vitorioso do subtorneio. Se A1 ♦ An +1 , A1 é o vitorioso do torneio.
Se An +1 ♦ A1 , podemos analisar ternas de jogadores em “subtorneios” para
chegarmos a uma conclusão.

Terna Conclusão
A1 , A2 , An +1 ( An +1 ♦ A1 ∧ A1 ♦ A2 ) ⇒ An +1 ♦ A2
A1 , A3 , An +1 ( An +1 ♦ A1 ∧ A1 ♦ A3 ) ⇒ An +1 ♦ A3

A1 , An , An +1 ( An +1 ♦ A1 ∧ A1 ♦ An ) ⇒ An +1 ♦ An

Concluímos que An +1 é vitorioso. De qualquer modo, há um vitorioso. Assim,


indutivamente, confirma-se o enunciado. Analogamente, conclui-se que há um
jogador que tenha perdido todas as partidas.
Obs. O teorema provado é mais geral. Poderia ser enunciado como “se um
jogador A vencer B e B vencer C, é impossível C vencer A. Então, há um jogador
que vença todos os demais”. Ele se torna até intuitivo se o enunciarmos assim:
“quando um jogador vence outro, que vence um terceiro, o primeiro vencerá o
terceiro”.

PARTE B: EDSON RYOKEI ONAGA (SÃO PAULO – SP)


Para indicar o vencedor de uma disputa, vamos utilizar uma seta: J x → J y (Isso
significa que J x perdeu para J y ). A seta aponta para o vencedor.

EUREKA! N°26, 2007

43
Sociedade Brasileira de Matemática

Vamos supor que nenhum jogador perdeu todas as partidas. Assim, J1 ganhou,
pelo menos 1 partida.
O esquema dele será assim:
J1 ← J 2
Como J 2 também não perdeu todas, o esquema ficará assim:
J1 ← J 2 ← J 3 ← ...
Observe que nenhum jogador pode aparecer 2 vezes nessa seqüência.
Vejamos o porquê:
Supondo que J 2 apareça de novo no esquema:
J1 ← J 2 ← J 3 ← J 4 ← J 2
Observe que ocorre a seguinte situação:
J 2 ganhou de J 3 , J 3 ganhou de J 4 e J 4 ganhou de J 2 .
Como o enunciado da questão não permite essa situação, não podemos repetir
nenhum jogador na seqüência.
Como n não é infinito, essa seqüência é finita.
O único modo de terminar o esquema é se algum jogador não perder nenhuma
partida, pois, após um invicto, não poderemos colocar nenhum outro jogador.
Logo, há um jogador que ganhou de todos os outros.
Da mesma forma que a seqüência tem um fim à direita, ela deve ter um fim à
esquerda.
Pelas mesmas condições citadas acima, o último à esquerda do esquema será o
jogador que perder de todos os outros.
Há um jogador que perdeu de todos os outros.

SOLUÇÕES – NÍVEL 3
SOLUÇÃO DO PROBLEMA 1: RENAN LIMA NOVAIS (RIO DE JANEIRO - RJ)
I) Desenhando a figura da questão, temos:

A C
P
II) Por ser o ponto I incentro, sabemos que este ponto eqüidista dos três lados do
∆ABC , podendo-se inscrever um círculo no ∆ABC . Além disso, por ser o
EUREKA! N°26, 2007

44
Sociedade Brasileira de Matemática

ponto I o incentro do triângulo ∆ABC , temos que AI é bissetriz do ângulo


B AC .
III) Observemos agora as alterações da figura:

A C
P
IV) Aplicando o teorema das bissetrizes internas no triângulo ∆ABP , de
bissetriz AI , temos:
BI PI AP AB AB AP + AB
= ⇒ = ⇒ =
AB AP PI BI BI PI + BI
Mas como é dito no enunciado da questão que AP + AB = BC , logo temos:
AB BC AB BI
= ⇒ =
BI PB BC PB
V) Assim, podemos notar que os triângulos ABI e CBP são semelhantes pelo
AB BI
caso LAL de semelhança (pois = e ABP ≡ P BC ).
BC PB
Logo BCP ≡ B AI .
VI) Observemos agora a figura novamente alterada:

A C
P

EUREKA! N°26, 2007

45
Sociedade Brasileira de Matemática

VII) Como B I A ≡ B PC , AI P é suplemento de B I A e, conseqüentemente,


suplemento de B PC e B PA é suplemento de B PC , logo AI P ≡ B PA. Assim,
o triângulo AIP é isósceles.

SOLUÇÃO DO PROBLEMA 2: LÚCIO ASSAOKA HOSSAKA (CURITIBA – PR)


Primero vamos provar que existe b. Podemos escolher dois pontos de
n ( n − 1)
formas, e traçar um segmento entre eles. Suponha agora que sobre cada
2
um deles haja dois triângulos isósceles com base no segmento em questão. Mais
de dois triângulos com base no mesmo segmento implicaria em 3 vértices
colineares, contidos na mediatriz do segmento. (Observe que a intenção não é a
de obter um número exato, e sim uma cota razoável). Assim, certamente
n ( n − 1)
f (n) ≤ ⋅ 2 = n ( n − 1) = n 2 − n, donde se ve que b = 1 é suficiente, ou
2
seja, existe.
No caso de a, vamos dividir em dois casos: n par e n ímpar.

n ímpar: Se arranjarmos os n pontos como vértices de um polígono regular de n


vértices, podemos criar um valor mínimo que sabemos que não é necessariamente
( n − 1) 1
f(n), mas que é menor ou igual a ele. Veremos que esse valor é ⋅n⋅ .
2 3
n −1
Veja por que: para cada vértice há pares de outros vértices equidistantes,
2
que formarão a base de um triângulo isósceles. São n vértices, e multiplicamos
1
por para evitar a possível contagem de triângulos mais de uma vez (como no
3
caso do eneágono regular, por ex.).
n2 − n
Resolvendo > an 2 , queremos que a seja tal que a inequação seja
6
1 1
verdadeira para n ≥ 3. Isso equivale a  1 −  > a. Como
6 n
1 1 1 2 1 1
n ≥ 3,  1 −  ≥ ⋅ = , e logo qualquer a < serve. Ou seja a existe para
6 n 6 3 9 9
n ímpar, pelo menos.

EUREKA! N°26, 2007

46
Sociedade Brasileira de Matemática

n par: análogo ao caso anterior, com a diferença de que colocamos um ponto no


centro do polígono regular, que agora tem n – 1 vértices. O número de possíveis
( n − 1)( n − 2) ( n − 1)( n − 2)
triângulos isósceles é de + > an 2 (a última parcela
6 2
corresponde aos que tem vértice no ponto central). Isso equivale a
2 2  1  2 
( n − 1)( n − 2) > an 2 ⇔  1 −   1 −  > a.
3 3  n  n 
2  1  2  2 3 1 1 1
Como n ≥ 4,  1 −   1 −  ≥ ⋅ ⋅ = , e logo qualquer a < serve.
3  n  n  3 4 2 4 4
Existe a constante nesse caso também, finalizando a demonstração.

SOLUÇÃO DO PROBLEMA 3: RAFAEL MENDES DE OLIVEIRA (RIO DE JANEIRO – RJ)


Fazendo x = 1 na equação original, temos que:
f ( f ( y ) + f (1)) = 2 f (1) + y ∴ f é sobrejetora, pois repare que, fazendo y = a –
2f(1), temos que f ( f ( a − 2 f (1)) + f (1)) = a , ∀a ∈ .
Como f é sobrejetora, ∃α ∈ R tal que f (α ) = 0.
Se α = 0, temos que, fazendo y = 0 na equação original, obteremos
f ( f ( x )) = 2 f ( x ). Como f é sobrejetora, para todo a ∈ R ∃ xa tal que
f ( xa ) = a. De f ( f ( x )) = 2 f ( x ), obtemos:
f ( f ( xa )) = 2 f ( xa ) ⇒ f ( a ) = 2 a , ∀a ∈ R.
Testando f ( x ) = 2 x na equação original, vemos que esta função obviamente não
é solução. Logo, podemos concluir que α ≠ 0.
Como α ≠ 0, fazendo x = α na equação original, obtemos:
f (α f ( y )) = α y. Logo, concluímos que f também é injetora, pois
f ( x) = f ( y) ⇒ f (α f ( x)) = f (α f ( y)) ⇒ α y = α x e como
α ≠ 0, α y = α x ⇒ x = y (logo f é injetora).
Fazendo x = α na equação original, obtemos:
f (α f ( y )) = α y. Fazendo y = 0 em f ( x f ( y ) ) = α y , temos que:
f (α f (0)) = 0. Como f (α ) = 0, temos que α = α f (0), pois f é injetora. Logo,
como α ≠ 0, temos que f (0) = 1.
Fazendo x = y = 0 na equação original, obtemos:
f(1) = 2. Fazendo x = y = –1 na equação original, temos que f(–1) = 0.
Fazendo x = – 1 na equação original, temos que:

EUREKA! N°26, 2007

47
Sociedade Brasileira de Matemática

f ( − f ( y )) = − y. Fazendo y = 0 na equação original temos:


f ( x + f ( x )) = 2 f ( x ).
Fazendo x := − f (u ) e y := − f (v ) na equação original, temos que:
f ( f (u )v − u ) = −2u + f (u ) f (v ). Fazendo u = 1 nesta última, temos que:
f (2 y − 1) = 2 f ( y ) − 2 (*).
Fazendo x = 1 na equação original, temos que f ( f ( y ) + 2) = y + 4.
De (*) temos: f (2 y − 1) + 2 = 2 f ( y ). Aplicando f dos dois lados desta última
igualdade, temos que f ( f (2 y − 1) + 2) = f (2 f ( y )).
Como f ( f ( x ) + 2) = x + 4 , temos que, fazendo x = 2y – 1 nesta última, temos:
f ( f (2 y − 1) + 2) = 2 y + 3.
Fazendo y = 0 na equação original, temos f ( f ( x ) + x ) = 2 f ( x ) (**).
Logo, fazendo x = y nesta última e aplicando f dos dois lados, temos que
f ( f ( f ( y ) + y )) = f (2 f ( y )).
Fazendo y = –1 na equação original, temos que f ( f ( x )) = 2 f ( x ) − x ∴ fazendo
x = f ( y ) + y nesta última, temos:
f ( f ( f ( y ) + y )) = 2 f ( f ( y ) + y ) − ( f ( y ) + y ). Como, por (**),
f ( y + f ( y )) = 2 f ( y ), temos que a última igualdade fica:
f ( f ( f ( y ) + y )) = 4 f ( y ) − f ( y ) − y = 3 f ( y ) − y.
Como 2 y + 3 = f ( f (2 y − 1) + 2) = f (2 f ( y )) = f ( f ( f ( y ) + y )) = 3 f ( y ) − y ,
Temos que 3 f ( y ) − y = 2 y + 3 ⇒ f ( y ) = y + 1, ∀y ∈ R.
Testando na equação original, vemos que os dois lados ficam xy + 2x + 2 ∴temos
que f(x) = x + 1 é a única função que satisfaz o problema.
Resposta: f(x) = x + 1.

SOLUÇÃO DO PROBLEMA 4: BRENO VIEIRA DE AGUIAR (FORTALEZA – CE)


(interpretando que um número arrojado deve ter exatamente 8 divisores)
i) A quantidade de divisores positivos de um número, é calculada pelo produto de
cada expoente dos seus fatores primos mais um. Daí, como o número tem
exatamente 8 divisores, ele pode ser das formas: Seja N inteiro positivo arrojado
que procuramos:
I) N = p7; p primo
II) N = p ⋅ q ; p , q primos distintos
3

III) N = p ⋅ q ⋅ t ; p , q , t primos distintos

EUREKA! N°26, 2007

48
Sociedade Brasileira de Matemática

ii) Analisemos cada caso:


I) Se N = p , então:
7

D( N ) = 1, p, p 2 , p 3 , p 4 , p 5 , p 6 , p 7 ⇒ 1 + p + p 2 + p 3 + p 4 + p 5 + p 6 + p 7 = 3240.
p8 − 1
Note que: 1 + p + p 2 + p3 + p 4 + p5 + p6 + p7 = e que para
p −1
p8 − 1
p ≥3⇒ ≥ 3280 , já que 1 + p + p 2 + p 3 + p 4 + p 5 + p 6 + p 7 é
p −1
crescente. Daí, p ≥ 3 não serve. Para p = 2, temos:
1 + p + p 2 + p 3 + p 4 + p 5 + p 6 + p 7 = 255 ⇒ p = 2 não serve!
Logo N = p não serve.
7

II)
N = p ⋅ q 3 ⇒ D(n) = 1, p, q, pq, q 2 , p ⋅ q 2 , q 3 , p ⋅ q 3 ⇒ 1 + p + q + p ⋅ q + q 2 + pq 2 +
+ q 3 + pq 3 = 3240 ⇒ ( p + 1)(1 + q + q 2 + q 3 ) = 3240. Perceba que estamos atrás
do menor inteiro arrojado > 0 e já temos que 2006 é um inteiro arrojado. Daí,
N ≤ 2006 .
p≥2
⇒ p ⋅ q 3 ≤ 2006 ⇒ q 3 ≤ 1003 ⇒ q ≤ 3 1003 < 11 ⇒ q = 2 ou 3 ou 5 ou 7.
Para
q = 2 ⇒ ( p +1) ⋅ (1+ 2 + 22 + 23 ) = 3240 ⇒ ( p +1) ⋅15 = 3240 ⇒ p + 1 = 816 ⇒ p = 215 ⇒
não serve.
Para
q = 3 ⇒ ( p +1) ⋅ (1+ 3 + 32 + 33 ) = 3240 ⇒ ( p +1) ⋅ 40 = 3240 ⇒ p +1 = 81 ⇒ p = 80 ⇒
não serve.
Para q = 5 ⇒ ( p +1) ⋅ (1+ 5 + 5 + 5 ) = 3240 ⇒ ( p + 1) ⋅156 = 3240 ⇒ p ∉ Z ⇒
2 3

não serve.
Para q = 7 ⇒ ( p +1) ⋅ (1+ 7 + 7 + 7 ) = 3240 ⇒ ( p +1) ⋅ 400 = 3240 ⇒ p ∉ Z ⇒
2 3

não serve.
Logo Para N = p ⋅ q não serve.
3

III)
N = p ⋅ q ⋅ t ⇒ D( N ) = 1, p, q, t, pq, qt, tp, pqt ⇒1+ p + q + t + pq + qt + tp + pqt = 3240 ⇒
⇒ ( p +1)(q +1)(t +1) = 3240. Então temos que achar três números posteriores a três

EUREKA! N°26, 2007

49
Sociedade Brasileira de Matemática

primos, tal que o produto desses números é 3240. Então esses primos são
divisores positivos de 3240 menos um. Então vamos ver quais são os D(3240)–1
que são primos:
1
3240 2 2
1620 2 4
810 2 8
405 3 3,6,12,24
135 3 9,18,36,72
45 3 27,54,108,216
15 3 81,162,324,648
5 5 5,10,20,40,15,30,60,120, 45, 90, 180, 360, 135, 270, 540, 1080, 405, 810, 1620, 3240.
1
D(3240) −1 = 0,1,2,3,4,5,7,8,9,11,14,17,19,23,26,29,35,39,44,53,59,71,80,89,107,
119,134,161,179,215,269,... Note que no mínimo (p + 1)(q + 1) é 12 (quando p =
2 e q = 3), então no máximo t + 1 é 270 ⇒ tmáx = 269.
Os primos possíveis são: 2, 3, 5, 7, 11, 17, 19, 23, 29, 53, 59, 71, 89, 107, 179,
269.
Para t = 269 ⇒ (p + 1)(q +1) = 12 ⇒ p = 2 e q = 3 ⇒ N = 1614
Para t = 179 ⇒ (p + 1)(q +1) = 18 ⇒ p = 2 e q = 5 ⇒ N = 1790
Para t = 107 ⇒ (p + 1)(q +1) = 30 ⇒ ∃ p, q primos
Para t = 89 ⇒ (p + 1)(q +1) = 36 ⇒ p = 2 e q = 11 ⇒ N = 1958
Para t = 71 ⇒ (p + 1)(q +1) = 45 ⇒ ∃ p, q primos
Para t = 59 ⇒ (p + 1)(q +1) = 54 ⇒ p = 2 e q = 17 ⇒ N = 2006
Para t = 53 ⇒ (p + 1)(q +1) = 60 ⇒ p = 2 e q = 19 ⇒ N = 2014
Para t = 29 ⇒ (p + 1)(q +1) = 108 ⇒ p = 5 e q = 17 ⇒ N = 2465
Para t = 23 ⇒ (p + 1)(q +1) = 135 ⇒ ∃ p, q primos
Para t = 19,17, 11, 7, 5, 3 ou 2 é análogo aos anteriores.
Daí, o menor N é 1614.
iii) Então o menor inteiro positivo arrojado é 1614.

SOLUÇÃO DO PROBLEMA 4: PEDRO PAULO GONDIM CARDOSO (SALVADOR – BA)


(Interpretação que um número arrojado pode ter mais que 8 divisores)
Inicialmente observa-se que 1260 é um número arrojado, pois 1260 + 630 + 420
+ 315 + 252 + 210 + 90 + 63 = 3240. Agora deve-se provar que não há nenhum
número arrojado menor que 1260.
Um número arrojado tem como divisores a, b, c, d, e, f, g, h tais que
1 1 1 1 1 1 1 1
 + + + + + + +  K = 3240, onde K é o valor do número
a b c d e f g h
EUREKA! N°26, 2007

50
Sociedade Brasileira de Matemática

arrojado. Se existir um número arrojado menor que 1260, devem existir naturais
não nulas a, b, c, d, e, f, g, h tais que
1 1 1 1 1 1 1 1 3240 18
+ + + + + + + > = = 2, 571428.
a b c d e f g h 1260 7
Se existir um número arrojado menor que K, ele não poderia ser ímpar, senão os
menores valores para a, b, c, d, e, f, g, h seriam 1, 3, 5, 7, 11, 13, 15 e 17 e
1 1 1 1 1 1 1 3240
1 + + + + + + + < 2,3 < . Se existisse um número arrojado
3 5 7 9 11 13 15 1260
menor que K, ele teria que ser múltiplo de 3, senão as menores valores para a, b,
c, d, e, f, g, h seriam 1, 2, 4, 5, 7, 8, 10, 11 e
1 1 1 1 1 1 1 3240
1 + + + + + + + < 2, 5 < . Então, se existisse um número
2 4 5 7 8 10 11 1260
arrojado menor que 1260, ele teria que ser múltiplo de 6.
O menor valor possível para K corresponde aos menores valores possíveis para a,
b, c, d, e, f, g, h, que são 1, 2, 3, 4, 5, 6, 7, 8. Então:
1 1 1 1 1 1 1 1
 + + + + + + +  K = 3240 ⇒
a b c d e f g h
1 1 1 1 1 1 1 1  2283
⇒  + + + + + + +  K = 3240 ⇒ K = 3240 ⇒
1 2 3 4 5 6 7 8  840
3240 ⋅ 840
⇒ K = ⇒ K ≅ 1191,1.
2283
Se houver um número arrojado menor que 1260, ele deve ser maior que 1191, 1 e
múltiplo de 6. Então as únicas possibilidades para K são 1194, 1200, 1206, 1212,
1218, 1224, 1230, 1236, 1242, 1248 e 1254;
• Se K = 1194.
A soma dos oito maiores (e únicos) divisores de 1194 é 1194 + 597 + 398 + 199
+ 6 + 3 + 2 + 1 = 2400 < 3240. Portanto 1194 não é arrojado.
• Se K = 1200.
A soma dos oito maiores divisores de 1200 é 1200 + 600 + 400 + 300 + 240 +
200 + 150 + 120 = 3210 < 3240. Portanto 1200 não é arrojado.
• Se K = 1206.
A soma dos oito maiores divisores de 1206 é 1206 + 603 + 402 + 201 + 134 + 67
+ 18 + 9 = 2640 < 3240. Portanto 1206 não é arrojado.
• Se K = 1212.
A soma dos oito maiores divisores de 1212 é 1212 + 606 + 404 + 303 + 202 +
101 + 12 + 6 = 2846 < 3240. Portanto 1212 não é arrojado.
• Se K = 1218.

EUREKA! N°26, 2007

51
Sociedade Brasileira de Matemática

A soma dos oito maiores divisores de 1218 é 1218 + 609 +406 + 203 + 174 + 87
+ 58 + 42 = 2797 < 3240. Portanto 1218 não é arrojado.
• Se K = 1224.
A soma dos oito maiores divisores de 1224 é 1224 + 612 + 408 + 306 + 204 +
153 + 136 + 102 = 3145 < 3240. Portanto 1224 não é arrojado.
• Se K = 1230.
A soma dos oito maiores divisores de 1230 é 1230 + 615 + 410 + 246 + 205 +
123 + 82 + 41 = 2952 < 3240. Portanto 1230 não é arrojado.
• Se K = 1236.
A soma dos oito maiores divisores de 1236 é 1236 + 618 + 412 + 309 + 206 +
103 + 12 + 6 = 2902 < 3240. Portanto 1236 não é arrojado.
• Se K = 1242.
A soma dos oito maiores divisores de 1242 é 1242 + 621 + 414 + 207 + 138 + 69
+ 54 + 46 = 2791 < 3240. Portanto 1242 não é arrojado.
• Se K = 1248.
A soma dos oito maiores divisores de 1248 é 1248 + 624 + 416 + 312 + 208 +
156 + 104 + 96 = 3164 < 3240. Portanto 1248 não é arrojado.
• Se K = 1254.
A soma dos oito maiores divisores de 1254 é 1254 + 627 + 418 + 209 + 114 + 66
+ 57 + 38 = 2783 < 3240. Portanto 1254 não é arrojado.
Como a soma dos oito maiores divisores é menor que 3240, em todas as
possibilidades, é evidente que a soma de quaisquer outros oito divisores também
será menor que 3240.
Portanto não há nenhum inteiro positivo menor que 1260 que seja arrojado. Logo
o menor número inteiro positivo arrojado é 1260.

SOLUÇÃO DO PROBLEMA 5: LEANDRO FARIAS MAIA (FORTALEZA – CE)


Vamos dividir em duas partes:
Parte 1: A
M B
Os lados opostos de de P são paralelos.
Seja AB um lado de P e A´B´seu lado oposto.
Suponha que AB não seja paralelo a B´A´. Por B´,
T
trace uma paralela ao lado AB, até trocar AA´ em
C. Sendo M e N pontos médios dos lados AB e
A´B´, respectivamente, temos:
X C
B´ X XT XC
AB // B´C ⇒ = = ⇒ B´ X = XC B´
N A´
BM TM AM

EUREKA! N°26, 2007

52
Sociedade Brasileira de Matemática

 B´ X = XC
Mas veja que:  ⇒ NX // CA´, absurdo ( NX ∩ CA´= T ).
 B´N = NA´
Portanto, devemos ter: AB // A´B´.
Parte 2: A1 A2
A2006
AB = A´B´. A3
Numerando os vértices, temos que:
A1003 + i é oposto de Ai ,1 ≤ i ≤ 1003
Temos: T
A1 A2 // A1004 A1005 , A2 A3 // A1005 A1006 ,..., A1003 A1004 //
// A2006 A1 .
A1006 A1003
A1005 A1004
Logo:

A1T AT AT A T A T AT A T
= 2 = 3 = ... = 1003 = 1004 ⇒ 1 = 1004 ⇒ A1T = TA1004 .
TA1004 TA1005 TA1006 TA2006 TA1 TA1004 TA1
Portanto teremos:
AT AT AT A T
1 = 1 = 2 = 3 = ... = 1003
TA1004 TA1005 TA1006 TA2005
Ai T Ai Ai +1
Mas: = = 1 ⇒ Ai Ai +1 = A1003 + i A1004 + i , o que acaba, pois
TA1003 + i A1003 + i A1004 + i
Ai Ai +1 e A1004 + i são lados opostos.

SOLUÇÃO DO PROBLEMA 6: JOSÉ MARCOS ANDRADE FERRARO (SÃO PAULO - SP)


Vamos listar os primeiros termos para estabelecer uma base de indução.

n 0 1 2 3 4 5 6 7 8 9 10 11
f(n) 0 2 3 5 7 8 10 11 13 15 16 18

Lema: n = f (r ) ou n = f (r ) + 1.
Prova: é equivalente a f (r + 1) = ( f (r ) + 1 ou f (r ) + 2).
Suponha que isto aconteça para r < n – 1.
Então f ( n) = 2n − f ( r´) + r´, f (n − 1) = 2n − 2 − f (r ) + r ⇒
f (n) − f (n − 1) = 2 − f (r´) + r´+ f (r ) − r.

EUREKA! N°26, 2007

53
Sociedade Brasileira de Matemática

Temos r´= r ou r´= r + 1 , n = f (r ) ou n = f (r ) + 1 e ( f (r´) = f (r ) + 1 ou


f (r´) + 2). Se r´= r , f (n) − f (n − 1) = 2.
Se r´= r + 1, f (n) − f (n − 1) = 3 + f (r ) − f (r´) = (2 ou 1), o que termina a prova
do Lema.
Vamos agora provar por indução que k φ − 1 < f (k ) < k φ + 1, para todo k ∈ .
Suponha que isso vale para 0 ≤ k ≤ n − 1.
n +1 n −1
Se n = f (r ), rφ − 1 < n < rφ + 1 ⇒ >r>
φ φ
f ( n) = 2n − f ( r ) + r = 2n − n + r = n + r
n +1 n −1
n+ ≥n+r ≥n+ ⇒
φ φ
1 1 1
nφ + > f (n) ≥ nφ − ⇒ como 1 > , nφ + 1 > f (n) > nφ − 1.
φ φ φ
n n−2
Se n = f (r ) + 1, rφ − 1 < n − 1 < rφ + 1 ⇒ > r >
φ φ
f ( n ) = 2 n − n + 1 + r ⇒ f ( n) = n + 1 + r
n n−2
n +1+ > n +1+ r > n +1+ ⇒
φ φ
φ−2
nφ + 1 > f (n) ≥ nφ + > nφ − 1, cqd.
φ
Assim, φn + 1 > f (n) > φn − 1, ∀n ≥ 0.
Como f (n) ∈ , f (n) > φn − 1 e φn − 1∈ − , temos f (n) ≥ φn − 1 + 1 = φn  .
Se m ≤ φn, m < φn pois m∈ e φn ∈ − . Assim m ≤ φn  daí
f (n) ≥ φn  ≥ m. Logo se m ≤ φn, f (n) ≥ m, e portanto o jogo é equilibrado.
Por outro lado, se m > φn + 1 > f (n), o jogo é desequilibrado, cqd.

EUREKA! N°26, 2007

54
Sociedade Brasileira de Matemática

XXVIII OLIMPÍADA BRASILEIRA DE MATEMÁTICA


Problemas e Soluções da Primeira Fase – Nível Universitário

PROBLEMA 1
1
ex − 1 − x
Calcule ∫ (e x − 1) ⋅ x dx
−1

PROBLEMA 2
Seja N um inteiro positivo. Calcule, em função de N, o volume do sólido definido
por:
 x, y, z ∈ [0, +∞)

  x  +  y  +  z  ≤ N
PROBLEMA 3
Dada f: → [ 0, +∞ ) duas vezes diferenciável com f (0) = 0 , f '(0) = 1 ,
1 3
1 + f ( x) = , ∀x ∈ [0,1] , mostre que f (1) < .
f "( x) 2

PROBLEMA 4
Dada uma hipérbole e uma reta não paralela às assíntotas, determine o lugar
geométrico dos pontos médios das cordas da hipérbole paralelas à reta dada.

Obs: Uma corda de uma hipérbole é um segmento cujos extremos pertencem à


hipérbole.

PROBLEMA 5
2 2 2
As funções y1 (t ) = (1 + t 2 ) ⋅ et , y2 (t ) = (t + t 2 ) ⋅ et e y3 (t ) = (−1 − t + t 2 ) ⋅ et são
soluções da equação diferencial y"(t ) + a(t ) ⋅ y '(t ) + b(t ) ⋅ y (t ) = c(t ) , onde
a (t ), b(t ), c(t ) são funções duas vezes diferenciáveis.
Determine uma função duas vezes diferenciável y (t ) tal que
y"(t ) + a(t ) ⋅ y '(t ) + b(t ) ⋅ y (t ) = c(t ) , y (0) = 0, y '(0) = 0 .

PROBLEMA 6
Escolha três pontos x1, x2, x3 aleatoriamente, independentemente e com
distribuição uniforme no intervalo [0, 1]. Determine, em função do número
positivo m, a probabilidade de que
min{| x1 − x2 |,| x1 − x3 |,| x2 − x3 |} > m .

EUREKA! N°26, 2007

55
Sociedade Brasileira de Matemática

SOLUÇÃO DO PROBLEMA 1:
ex − 1 − x e− x − 1 + x xe x − e x + 1
Seja f ( x) = . Temos f (− x) = = , logo
(e x − 1) ⋅ x (e − x − 1) ⋅ (− x) (e x − 1) ⋅ x
xe x − x
f ( x) + f (− x) = = 1.
(e x − 1) ⋅ x
1 1 1
Assim, ∫
−1
f ( x)dx = ∫ ( f ( x) + f (− x))dx = ∫ 1dx = 1 .
0 0

SOLUÇÃO DO PROBLEMA 2:
O sólido é a união dos cubos unitários [i, i +1) × [j, j + 1) × [k, k + 1) para i, j, k
inteiros não negativos, i + j + k ≤ N. Como cada cubinho tem volume 1, o volume
do sólido é igual ao número de triplas (i, j, k) como acima.
 N + 3
O número de triplas (e portanto o volume) é igual a   = (N + 1)(N + 2)(N +
 3 
3)/6.
Isto pode ser demonstrado de várias formas. Por exemplo, podemos pensar que
temos uma fileira de N + 3 quadrados e vamos escolher 3 posições e botar um
marcador em cada uma delas: i será o número de quadrados antes do primeiro
marcador, j o número de quadrados entre o primeiro e o segundo marcadores e k
o número de quadrados entre o segundo e o terceiro marcadores. Claramente, a
cada configuração corresponde uma tripla e vice-versa.

SOLUÇÃO DO PROBLEMA 3:
1
Como f(x) ≥ 0 para todo x ∈ R, temos 0 < = f ´´(x) , logo f´ é crescente em
1+ f (x)
[0,1], o que implica que f´(x) > 1 para x > 0, ou seja f também é crescente em
[0,1].
1 1
Assim, para x > 0, temos 0 < = f ´´( x) = < 1 , logo
1 + f ( x) 1 + f ( x)
x x x

∫ 0dt < ∫ f ´´(t )dt < ∫ 1dt


0 0 0
⇔ 0 < f ´( x) − f ´(0) < x ⇔ 1 < f ´( x) < x + 1 ⇒
x x x
x2 3
∫ 1dt < ∫ f ´(t )dt < ∫ (t + 1)dt ⇔ x < f ( x) <
0 0 0
2
+ x . Em particular, f (1) < .
2

EUREKA! N°26, 2007

56
Sociedade Brasileira de Matemática

PRIMEIRA SOLUÇÃO DO PROBLEMA 4:


Seja r a reta dada, não paralela às assíntotas. Considere o plano euclidiano como
subconjunto do plano projetivo, da maneira usual. Seja P o ponto de r sobre a reta
do infinito. O feixe de retas paralelas a r corresponde ao feixe de retas do plano
projetivo que passam por P. Para cada uma das retas do feixe, que corta a
hipérbole em dois pontos A e B, o ponto médio M do segmento AB corresponde
ao conjugado harmônico de P em relação a A e B. Logo M pertence à reta polar
de P em relação à hipérbole. Seja p essa reta polar. Como o pólo da reta do
infinito é o centro O da hipérbole, concluímos que p passa por O. Há, portanto,
dois casos a considerar:
1) Se existe uma tangente à hipérbole paralela à reta r, com ponto de
tangência T (e portanto existirá uma outra tangente paralela a r no ponto
T´, simétrico de T em relação a O), o lugar geométrico é a reta OT menos
o segmento TT ´.
2) Caso contrário, o lugar geométrico é uma reta completa passando por O,
que pode ser obtida traçando-se uma corda arbitrária paralela a r (neste
caso toda reta paralela corta a hipérbole em dois pontos distintos, um em
cada ramo da hipérbole) e unindo seu ponto médio a O.
SEGUNDA SOLUÇÃO:
Após uma mudança de coordenadas afins, podemos considerar que a hipérbole
tem equação
xy = 1. Sendo m o coeficiente angular da reta r, queremos determinar o lugar
geométrico dos pontos médios das intersecções das retas de equações y = mx + t (
t ∈ R) com a hipérbole. Sejam (x1, y1) e (x2, y2) esses pontos de intersecção. Então
x1 e x2 são as raízes da equação
x(mx + t) = 1 ⇔ mx2 + tx – 1 = 0 (1). Logo a abscissa do ponto médio é igual a
1 1 t
+ −
t y1 + y2 x1 x2 x1 + x2 t
− , e sua ordenada vale = = = 2m = . Logo o ponto
2m 2 2 2 x1 x2 2 4

m
m
médio pertence à reta de equação y = − x . Reciprocamente, um ponto dessa
2
reta pertence ao lugar geométrico desde que a equação (1) tenha duas raízes reais,
ou seja, quando t 2 + 4m > 0 . Assim, se m > 0, o lugar geométrico é toda a reta de
m
equação y = − x . Quando m < 0, desta reta devem ser retirados os pontos para
2
−m −m
os quais −2 − m ≤ t ≤ 2 − m , ou seja, para os quais ≤x≤ .
m m

EUREKA! N°26, 2007

57
Sociedade Brasileira de Matemática

SOLUÇÃO DO PROBLEMA 5:
Podemos tomar y1 como solução particular e y2 – y1 e y3 – y1 como soluções
linearmente independentes da equação homogênea associada
y ''(t ) + a (t ) ⋅ y '(t ) + b(t ) ⋅ y (t ) = 0 . Assim a solução geral da equação é
y = c1 y1 + c2 y2 + c3 y3, c1 + c2 + c3 = 1 ou, equivalentemente,
t2
y(t) = (c4 + c5 t + t2) ⋅ e .
Temos y(0) = c4 e y’(0) = c5, donde as condições do enunciado nos dão
2
y = t 2 ⋅ et .

SOLUÇÃO DO PROBLEMA 6:
Seja X=[0, 1]3. Temos X = A1 ∪ A2 ∪ A3 ∪ A4 ∪ A5 ∪ A6 , onde
A1 = {( x, y , z ) ∈ X | x ≤ y ≤ z} , A2 = {( x, y, z ) ∈ X | x ≤ z ≤ y} ,
A3 = {( x, y, z ) ∈ X | y ≤ x ≤ z} , A4 = {( x, y, z ) ∈ X | y ≤ z ≤ x} ,
A5 = {( x, y, z ) ∈ X | z ≤ x ≤ y} e A6 = {( x, y, z ) ∈ X | z ≤ y ≤ x} .
Os conjuntos Ak ,1 ≤ k ≤ 6 têm todos volume 1/6.
Seja agora Y = {( x1 , x 2 , x3 ) ∈ X | min{| x1 − x 2 |, | x1 − x3 |, | x 2 − x3 |} > m} .
Temos
Y = B1 ∪ B2 ∪ B3 ∪ B4 ∪ B5 ∪ B6 , onde Bk = Y ∩ Ak ,1 ≤ k ≤ 6 .
Todos os conjuntos Bk ,1 ≤ k ≤ 6, têm o mesmo volume. Vamos então calcular o
volume do conjunto B1 . Como X tem volume 1, a probabilidade desejada será
6 ⋅ vol ( B1 ) .
Temos B1 = {( x, y , z ) ∈ X | y > x + m, z > y + m} . Claramente, se m ≥ 1 / 2 ,
B1 é vazio, e portanto a probabilidade desejada é 0 para todo m ≥ 1 / 2 .
Suponha agora 0 ≤ m ≤ 1 / 2 . Considere a translação f : B1 → X dada por
f ( x, y, z ) = ( x, y − m, z − 2m) .
Temos f ( B1 ) = {( x, y, z ) ∈ [0,1 − 2m]3 | x < y < z} , e portanto f ( B1 ) tem o
mesmo volume de {( x, y, z ) ∈ [0,1 − 2m]3 | x ≤ y ≤ z} = g ( A1 ) , onde g é a
homotetia dada por g ( p) = (1 − 2m) p . Assim, temos
vol ( B1 ) = vol ( f ( B 1 )) = vol ( g ( A1 )) = (1 − 2m) 3 ⋅ vol ( A1 ) = (1 − 2m) 3 / 6 , e
logo, para 0 ≤ m ≤ 1 / 2 , a probabilidade desejada é igual a
6 ⋅ vol ( B1 ) = (1 − 2m) 3 .

EUREKA! N°26, 2007

58
Sociedade Brasileira de Matemática

XXVIII OLIMPÍADA BRASILEIRA DE MATEMÁTICA


Problemas e Soluções da Segunda Fase – Nível Universitário

PROBLEMA 1:
Seja f : → uma função integrável e crescente. Prove que
1 1
1
∫0 xf ( x)dx ≥ 2 ∫0 f ( x)dx
PROBLEMA 2:
Prove que, para todo inteiro n ≥ 2 , o número de matrizes quadradas 2 × 2 com
entradas inteiras e pertencentes ao conjunto {0, 1, 2, …, n – 1} que têm
1
determinante da forma kn + 1 para algum k inteiro é dado por n3 ⋅ ∏ (1 − 2 ) .
p primo p
p|n

PROBLEMA 3:
Uma mesa de bilhar tem o formato de elipse e não tem caçapas. Quando uma bola
bate em um ponto P na borda da mesa, ela segue uma direção simétrica em
relação à reta normal à elipse em P. Prove que se uma bola parte de um ponto A
da elipse e, após bater na mesa nos pontos B e C, retorna a A, então ela baterá
novamente em B.

PROBLEMA 4:
Seja p um polinômio irredutível em Q[x] de coeficientes racionais e grau maior
do que 1. Prove que se p admite duas raízes r e s cujo produto é 1 então o grau de
p é par.

PROBLEMA 5:
Seja f :[0, +∞) → [0, +∞) uma função crescente e bijetora. Prove que a série
∞ ∞ −1
1 f ( n)

n =1 f ( n)
converge se, e somente se, a série ∑
n =1 n 2
converge, sendo f −1
a

função inversa de f.

PROBLEMA 6:
Considere as matrizes
1 2 1 0 
A=  e B= 
0 1  2 1
EUREKA! N°26, 2007

59
Sociedade Brasileira de Matemática

Prove que, para n > 1, não existem inteiros a1 , a2 , a3 ,…, an e b1 , b2 ,…, bn −1 , bn com
a2 , a3 ,..., an e b1 , b2 ,..., bn −1 não nulos tais que
A a1 ⋅ B b1 ⋅ A a2 ⋅ B b2 ⋅ … ⋅ A an ⋅ B bn = I ,
onde I é a matriz identidade de ordem 2.

SOLUÇÃO DO PROBLEMA 1: LEVI MÁXIMO VIANA (FORTALEZA – CE)


x x
Chame F ( x) = ∫ tf (t )dt e G ( x) = ∫ f (t )dt , x ∈ [0,1]. Veja então que
0 0

F´( x) = xf ( x) = xG´( x), mas como f ( x) = G´( x) é crescente, temos f ( x) ≥ f ( x 2 ),


G( x2 ) '
já que 0 ≤ x ≤ 1, logo F´( x) ≥ xG´( x ) =
2
. Integrando de 0 a 1 temos:
2
1
2
1 G( x ) ' G (12 ) − G (02 ) G (1)
∫0 F ´( x ) dx ≥ ∫0 2 dx ⇒ F (1) − F (0) ≥
2
⇒ F (1) ≥
2
1 1 1
⇒ ∫ xf ( x)dx ≥ ∫ f ( x)dx . cqd.
0 2 0

SOLUÇÃO DO PROBLEMA 2: MURILO VASCONCELOS ANDRADE (MACEIÓ – AL)


Primeiramente provaremos que o enunciado vale para n primo:
a b 
Seja n primo e   uma matriz cujo determinante é da forma k ⋅ n + 1, temos
c d
então que a ⋅ d − b ⋅ c ≡ 1(mod n).
Seja h(i) o número de pares ordenados de inteiros no conjunto
{0,1, 2,..., n − 1} cujo
produto é igual a i(mod n). Desta maneira, temos que o
número de matrizes satisfazendo as condições do enunciado é:
n −1

∑ h(i) ⋅ h(i + 1)
i =0

(Onde cada h(i) representa o número de escolhas possíveis para b e c, tais que
bc ≡ i (mod n) e h(i + 1) representa o número de escolhas para a e d, tais
que ad ≡ i + 1(mod n).

Naturalmente h(0) = 2n − 1 (os possíveis pares são (0,0), (0, 1), ...,(0, n –1),

(1, 0), (2, 0),..., (n – 1, 0).

EUREKA! N°26, 2007

60
Sociedade Brasileira de Matemática

Alem disso, h(i ) = ϕ (n), para 1 ≤ i ≤ n − 1 (aqui ϕ representa a função ϕ de


Euler, que associa a cada inteiro positivo no número de inteiros menores que n e
que são primos com n. No caso de n primo, ϕ (n) = n − 1). Vamos provar isto:

Seja k primo com n. Vamos provar que existe k´ tal que k ⋅ k´≡ i (mod n) : a
seqüência (k r
mod n ) assume um número finito de valores (entre 0 e n –1).
Existem então dois números iguais na seqüência digamos k 1 = k 2 com
n n

n2 > n1 ) . Assim, k ⋅ ( ik n2 − n1 −1 ) = i e portanto h ( i ) ≥ ϕ (n). Como


( 2n − 1) + ϕ (n) ⋅ (n − 1) = n2 , segue que não podemos ter h(i ) > ϕ (n) para algum
n −1 n−2

i. Assim, ∑ h(i) ⋅ h(i + 1) = (2n −1)(n −1) + ∑ (n −1) ⋅ (n −1) + (n − 1)(2n −1) =
r =0 i =1

 1 
= (n − 1) [ (n − 1)(n − 2) + 4n − 2] = n(n − 1)(n + 1) = n3 ∏ 1- 2 
p primo  p 
pn

O resultado então fica provado para n primo.

Vamos agora mostrar por indução que o resultado vale para n = p potência de
k

k −1
primo. Para k = 1 já foi provado. Suponha que k ≥ 2 e que vale para p .

 a b   pk −1 ⋅ n1 + a´ pk −1 ⋅ n2 + b´ 
Seja n = p e   =  k −1  , com ad − bc ≡ 1(mod p k ).
k
k −1
 c d   p ⋅ n3 + c´ p ⋅ n4 + d´

⇒ a´d´−b´c´≡ 1(mod pk −1 ).

2k −2
Assim, como p ≡ 0(mod pk ),

ad − bc ≡ pk −1 (n1 ⋅ d´+n4 ⋅ a´−b´⋅n3 − n2 ⋅ c´) + a´d´−b´c ≡ pk −1 (n1d´+n4 a´−b´n3 − n2c´) +


+ ⋅ pk −1 +1 ≡ 1(mod pk ) ⇒ n1d´+n4 a´−b´n3 − n2c´+ ≡ 0(mod p)

EUREKA! N°26, 2007

61
Sociedade Brasileira de Matemática

k −1
(aqui é tal que ⋅ p +1 = a´d´−b´c´ ).

3(k −1)  1
1−
k −1
Como a´⋅d´−b´⋅c ≡ 1(mod p ) existem (por hipótese de indução), p 
 p2 
maneiras de escolhermos a´, d´, b´ e c´. Fixando-se estes valores (e portanto
também) temos que um deles é primo com n = p , pois, caso não fosse assim,
k

a´d´−b´c´ ≡ 1(mod p), absurdo! Seja (a, n) = 1, por exemplo. Existem então p
valores possíveis (módulo p) para cada um dos n1, n2 , n3 e para cada combinação
destes, apenas um valor para n4 tal que n1d´+n4a´−b´n3 − n2c´+ ≡ 0(mod p) .

 1  1
Existem então ao todo p3(k −1) ⋅ 1− 2  ⋅ p ⋅ p ⋅ p = p3k 1− 2  maneiras de
 p   p 
escolhermos a, b, c, d. Isto termina nossa prova por indução.

Para n = p11 p22 ... pk k é fácil ver que ad − bc ≡ 1(mod n) ⇔ ad − bc ≡ 1(mod pi i ),


a a a a

∀i ∈{1,2,..., k} e portanto o número de matrizes que satisfazem as condições do


k
 1 3  1
enunciado é igual a ∏p 3ai
i 1− 2  = n ⋅ ∏ 1− 2 , cqd.
i= j  pi  p primo  p 
pn

SOLUÇÃO DO PROBLEMA 3: EDUARDO POÇO (SÃO PAULO – SP)


AB e BC têm o mesmo ângulo com a normal em B ⇔ têm o mesmo ângulo
com a tangente à elipse em B ⇔
B− A C−B  B− A C−B 
× VT = VT × ⇔ +  × VT = 0 , sendo VT o vetor
B− A C−B  B − A C − B 
tangente à elipse em B.

Parametrizando a elipse: P(t ) = ( k cos t ,sen t ) , 0 ≤ t ≤ 2π , sem perda de


d
generalidade. Vetor tangente no ponto P (t ) : V (t ) = P (t ) = ( − k sen t , cos t ) .
dt

EUREKA! N°26, 2007

62
Sociedade Brasileira de Matemática

Sejam P (a) = A , P(b) = B , P(c) = C . As hipóteses sobre refletir de AB para


BC e sobre refletir de BC para CA se tornam:
 B− A C−B   C − B A−C 
 +  × V (b) = 0  +  × V (c ) = 0
 B − A C − B   C − B A − C 
Temos que provar o seguinte, que CA reflete em AB:
 A−C B− A 
 +  × V (a) = 0
 A − C B − A 
Distribuindo o produto vetorial (vamos denotar como um escalar, pois todos os
vetores em jogo estão no mesmo plano e os produtos vetoriais terão a mesma
direção):
P ( x) × V ( y ) = (k cos x, sen x ) × (− k sen y, cos y ) = k cos( x − y )
Após dividir por k, as hipóteses se tornam:
1 − cos(a − b) cos(c − b) − 1
+ =0
B−A C−B
1 − cos(b − c) cos(a − c) − 1
+ =0
C−B A−C
E queremos provar:
1 − cos(c − a) cos(b − a) − 1
+ =0
A−C B− A

Para provar isso, basta somar as hipóteses.

SOLUÇÃO DO PROBLEMA 4: THIAGO BARROS RODRIGUES COSTA (FORTALEZA – CE)


Seja p um polinômio irredutível em [ x] admitindo duas raízes r e s cujo produto

(
é 1 r = 1s . )
Temos p ( x ) = x n + an −1 x n −1 + ... + a1 x + a0 (podemos supor p mônico sem perda de
generalidade).
Como p é irredutível em [ x] , p deve ser igual ao polinômio minimal de r e s
sobre ⇒ se g ( x) ∈ [ x] é tal que g ( s ) = 0 ou g ( r ) = 0, então p( x) g ( x).

Seja g ( x) =
1 n 1
a0
⋅x p ( )
x
.

EUREKA! N°26, 2007

63
Sociedade Brasileira de Matemática

É fácil ver que todas as potências de x serão ≥ 0 ⇒ g ( x) é um polinômio sobre


[ x] . Mas

g (r ) =
1 n
a0
r p 1
r ( ( ))
1
= r n ⋅ p( s ) = 0 ⇒ g (r ) = 0 ⇒ p ( x) g ( x).
a0
Mas g é mônico e tem o mesmo grau de p ⇒
1
p( x) = g ( x) ⇒ p ( x) = x n ⋅ p 1
a0 x ( )
Se α é raiz de p( x) (obviamente α ≠ 0 pois p é irredutível e tem grau > 1),
então,

( α ) = a1 ⋅ α1 ⋅ p(α ) = 0.
p 1
n
n

Logo as raízes de p sempre aparecem aos pares α , 1α . Além disso,


k k
1  k  1  1 n 1
( x −α ) p( x) ⇒ x  − α  = ( −α )  x − 
k k
x p   = p( x).
x   α  a0  x
Veja que α = 1α ⇔ α = 1 ou –1. Mas nenhum desses valores pode ser raiz de p
pois o polinômio minimal deles sobre tem grau 1, e p tem grau > 1.
⇒ as raízes aparecem aos pares α , 1α ( )
⇒ p tem um número par de raízes ⇒ p tem grau par.

SOLUÇÃO DO PROBLEMA 5: EDUARDO POÇO (SÃO PAULO – SP)


Note que f (0) = 0 , e toda função crescente e bijetora é contínua, e logo
integrável em qualquer intervalo finito.

1
Como 1 f ( x) é decrescente, pelo critério da integral ∑ f ( n)
n =1
converge se e

∞ 1
somente se ∫1 f ( x)
dx converge. Por outro lado,

∞ f −1 ( x) ∞ n +1 f
−1
( x)
∫ 2
dx = ∑ ∫ 2
dx, e
1 x n =1
n x
n +1
f −1 (n) 4n 2 ≤ f −1 (n) (n + 1) 2 ≤ ∫ f −1 ( x) x 2 dx ≤ f −1 (n + 1) n 2 ≤ 4 f −1 (n + 1) (n + 1) 2 ,
n
donde

EUREKA! N°26, 2007

64
Sociedade Brasileira de Matemática

1 ∞ f −1 ( n ) ∞ f −1 ( x ) ∞ ∞


4 n =1 n 2
≤ ∫ 1 x 2
dx ≤ 4 ∑ f
n =1
−1
( n + 1) ( n + 1) 2 = 4 ∑ f
n=2
−1
(m ) m 2 ≤
∞ −1
f ( x)
≤ 4∑ f −1 ∞

m =1
( m ) m 2 , e, em particular, ∫1 x 2
dx converge

⇔ ∑f
n =1
−1
( n ) ( n ) 2 converge. Basta agora relacionar a convergência de

∞ ∞
1 f −1 ( x)

1
f ( x)
dx e de
∫ 1
x2
dx , que são integráveis em intervalos finitos.

Um resultado conhecido (que pode ser facilmente verificado derivando) é a


fórmula da primitiva da inversa:

∫g
−1
( )
( x)dx = xg −1 ( x) − G g −1 ( x) , sendo G (x) uma primitiva de g (x)

1 1
Utilizando g −1 ( x) = , temos então g ( x) = f −1   , e assim:
f ( x)  x

b
1 1 1  1   1 
∫ f ( x) dx = b f (b) − 1 f (1) − G f (b)  + G f (1) 
1
1
b f (b )
1 b 1 1
∫ f ( x) dx = f (b) − f (1) − ∫
1 1
f −1  dx
 x
f (1)

Essa relação também pode ser percebida diretamente do gráfico de 1 / f ( x) , que


é uma função decrescente. Com a transformação x = 1 / y :

b f (b )
1 b 1 f −1 ( y )

1
f ( x)
dx = −
f (b) f (1)
+

f (1)
y2
dy (I).

EUREKA! N°26, 2007

65
Sociedade Brasileira de Matemática


1
Suponha agora que
∫ f ( x) dx converge. Logo:
1
x x ∞
x 1 1 1
lim
x → ∞ f ( x)
= 2 lim
x →∞ ∫
x/2
f ( x)
dt ≤ 2 lim
x →∞ ∫
x/2
f (t )
dt ≤ 2 lim
x →∞ ∫
x/2
f (t )
dt = 0

x
Então lim = 0 . Fazendo b → ∞ em (I), temos a integral à esquerda
x → ∞ f ( x)

b
convergindo e a parcela indo para zero, o que resulta num valor finito para
f (b)

f −1 ( x)

1
x2
dx , donde concluímos que essa integral converge.


f −1 ( x) f −1 ( x)
Da mesma forma, se

1
x2
dx converge, prova-se que lim
x →∞ x
=0,e

como f é crescente e bijetora, podemos ir para o infinito com x = f ( y ) , de onde


f −1 ( f ( y )) y
tiramos lim = 0 ⇔ lim = 0 . Novamente aplicando o limite
y →∞ f ( y) y → ∞ f ( y)
b
b → ∞ em (I), temos que a integral à direita e convergem para valores
f (b)

1
finitos, donde
∫ 1
f ( x)
dx converge.

Retornando às séries, pelo critério da integral, a equivalência na convergência das


integrais, já provada, transmite equivalência na convergência das séries.

SOLUÇÃO DO PROBLEMA 6: COLABORAÇÃO DE DARIO BERALDO (PISA – ITÁLIA)


Indutivamente, para cada inteiro k,
 1 2k  1 0
Ak =  e B =
k
.
0 1   2k 1

EUREKA! N°26, 2007

66
Sociedade Brasileira de Matemática

Sejam A, B, I transformações na reta projetiva (uma reta do plano projetivo), de


modo que, em coordenadas homogêneas,
 1
1 1 + 2kt   2k + 
Ak   =   = t
t   t   1 
 
e
 1 
h s  s  
B  = = 1  ,
1
   1 + 2 hs   2 h +
 s 
para cada s, t ∈ R .
*

Assim, para s irracional,


 1 
 2k +
 s  1
Ak B h   =  2h +  .
1   s
 1 
Iterando, eventualmente obtemos a seguinte formula:
 1 
 2a1 + 1 
 2b1 + 
s  
Aa1 B b1 ... Aan Bbn   =  1.
1   2an + 
  1
2bn +
  s
 1 
 
Suponhamos, por contradição, que existem inteiros a1 , b1 ,..., an , bn tais que
M := Aa1 Bb1 ... Aan B bn = I .
Dado um número transcendente s, a hipótese M = I implica em particular que as
linhas determinadas por
 1 
 2a1 + 1 
 2b1 + 
s  
1  e  1 
   2an + 
 1
2bn +
 s
 1 
 

EUREKA! N°26, 2007

67
Sociedade Brasileira de Matemática

devem ser as mesmas, i.e.


1
s = 2a1 +
1
2b1 +
1
2an +
1
2bn +
s
Agora, basta certificar-se que existe uma relação algébrica não trivial (certamente
quadrática) para s, o que é um absurdo.

Usaremos a seguinte notação:


1
{c0 , c1 ,..., cn } = c0 + 1
.
c1 +

1
cn −1 +
cn
Então, nossa relação é s = {2a1 , 2b1 ,..., 2an , 2bn , s} = {m0 , m1 , m2 ,...mN , s} ,
em que os m j são inteiros pares. Precisamos uma regra para escrever frações
contínuas como uma simples fração, i.e.
Lema: Dado {c0 , c1 ,..., ck } , para cada inteiro 0 ≤ j ≤ k ,
pj
{c ,..., c } = q
0 j
j

onde
p j = c j p j −1 + p j − 2 , q j = c j q j −1 + q j − 2 (com p0 = c0 , p1 = c0 c1 + 1, q0 = 1, q1 = c1 ).
Esta é uma indução fácil: temos {c0 ,..., c j , c j +1} = {c0 ,..., c j −1 , c j + 1 c j +1} =
 1 
 c j +  p j −1 + p j − 2
 c j +1  c j +1 (c j p j −1 + p j − 2 ) + p j −1 c j +1 p j + p j −1 p j +1
= = = = , cqd.
 1  c j +1 (c j q j −1 + q j − 2 ) + q j −1 c j +1q j + q j −1 q j +1
 c j +  q j −1 + q j − 2
 c j +1 
Em particular, aplicando o lema para {m0 , m1 , m2 ,...mN , s} , obtemos, no caso em
que bn ≠ 0,
pN s + pN −1
s= (*)
qN s + qN −1

EUREKA! N°26, 2007

68
Sociedade Brasileira de Matemática

onde pN , qN , pN −1 , qN −1 são inteiros (pelo lema e o fato que p0 , p1 , q0 , q1 e os m´s


são inteiros). Note que qN ≠ 0. De fato, q0 = 1, q1 = m1 = 2b1 ≠ 0 e, para
k ≥ 1, qk +1 = mk +1qk + qk −1 , o que, como mk +1 é um inteiro par não nulo (pelo
menos para 1 ≤ k < N − 1) implica, por indução, que qk +1 > qk . De fato, teremos
qk +1 ≥ mk +1qk − qk −1 ≥ 2 qk − qk −1 > qk . No caso em que mN = 2bn = 0, teremos

{
s = {2a1 , 2b1 ,..., 2an ,1/ s} = m0 , m1 ,..., mN −1 , 1
s } = pq N −1 / s + pN − 2 pN −1 + pN − 2 s
N −1 / s + qN − 2
=
qN −1 + qN − 2 s
,

e, como N = 2n − 1 ≥ 3, N − 2 ≥ 1 e logo qN − 2 ≠ 0. Isto implica que s é uma raiz


de uma equação de segundo grau com coeficientes inteiros, e segue a conclusão.

Obs. Em vez de tomar s transcendente, poderíamos tomar, por exemplo, s = 3 2.


p
Temos 3 2 irracional, pois 1 < 3 2 < 2 e, se 2 = q com p e q inteiros, q > 1 e
3

p3 , mas q 3 > 1 e mdc ( p3 , q 3 ) = 1, donde


mdc( p, q) = 1, teríamos 2 =
q3
p3 ∉ , absurdo. Se 3
2 fosse raiz de uma equação do segundo grau
q3
ax 2 + bx + c = 0 com a, b, c ∈ e a ≠ 0, teríamos a 3 4 + b 3 2 + c = 0, donde
b c
3
4 = − 3 2 − = u + v 3 2, com u = −c a e v = −b a racionais. Multiplicando
a a
por 3
( )
2, teríamos 2 = u 3 2 + v 3 4 = u 3 2 + v u + v 3 2 = uv + ( u + v ) 3 2. Se
2

2 − uv
u + v 2 ≠ 0, teríamos
3
2= ∈ , absurdo. Assim, u + v2 = 0 e
u + v2
uv = 2 ⇒ −v3 = 2 ⇒ 3 2 = −v ∈ , absurdo.

EUREKA! N°26, 2007

69
Sociedade Brasileira de Matemática

XXVIII OLIMPÍADA BRASILEIRA DE MATEMÁTICA


Resultado – Nível 1 (5a. e 6a. Séries)

NOME CIDADE - ESTADO PRÊMIO


Otávio Augusto de Oliveira Mendes Pilar do Sul - SP Ouro
Guilherme Cherman Perdigãao de Oliveira Rio de Janeiro - RJ Ouro
Bruno Silva Mucciaccia Vitória - ES Ouro
João Lucas Camelo Sá Fortaleza - CE Ouro
Douglas Souza Alves Junior Vassouras - RJ Ouro
Kayo de França Gurgel Fortaleza - CE Prata
Rafael Ferreira Antonioli S. B. do Campo - SP Prata
Nikolas Leonel Carvalho Salvador - BA Prata
Gabriela de Paula Gonzalvez Jundiaí - SP Prata
Rodrigo Nagamine Santo André - SP Prata
Ana Thais Castro de Santana Rio de Janeiro - RJ Prata
Ycaro César Campello Izaias Fortaleza - CE Prata
Débora Jun Portugheis Campinas - SP Prata
Gustavo Lopes Perosini Tabapuã - SP Prata
Felipe Figueiredo Souza e Silva Nova Lima - MG Prata
Marla Rochana Braga Monteiro Fortaleza - CE Prata
Alexandre Crepory Abbott de Oliveira Brasília - DF Bronze
Rafael Kazuhiro Miyazaki São Paulo - SP Bronze
Tiago Leandro Estevam Dias Rio de Janeiro - RJ Bronze
Daniel Silva Luiz Crispin Fortaleza - CE Bronze
Ana Lívia Ruegger Saldanha Araras - SP Bronze
Wladimir José Lopes Martins Recife - PE Bronze
Daniel dos Santos Bossle Porto Alegre - RS Bronze
Erica Saldanha Freire Simões Fortaleza - CE Bronze
Lucas Almeida Rocha Taubaté - SP Bronze
Hugo Rodrigues Martins Dantas Fortaleza - CE Bronze
Daniel Cardoso de Sousa Teresina - PI Bronze
Nicolas Iso Magosso Grigolli Gibin S. B. do Campo - SP Bronze
Marcos Massayuki Kawakami São Paulo - SP Bronze
Paula Dias Garcia Brasília - DF Bronze
Renner Tetzner Ramos Vitória - ES Bronze
Bernardo de Andrade Macêdo Rio de Janeiro - RJ Bronze
Marina Pessoa Mota Fortaleza - CE Mençâo Honrosa
Luis Henrique Kobaya Shi Higa Campo Grande - MS Menção Honrosa
Gabriel Militão Vinhas Lopes Fortaleza - CE Menção Honrosa
Filipe José Oliveira Sabóia Fortaleza – CE Menção Honrosa
Vitor Silveira da Costa Curitiba - PR Menção Honrosa
Tiago da Ávila Palhares Ponte Nova - MG Menção Honrosa
Gustavo Pereira de Castro Rio de Janeiro - RJ Menção Honrosa
Douglas Michael da Costa Cezar Santa Maria - RS Menção Honrosa
Bruna Rufino Leão Teresina - PI Menção Honrosa
Thomas Rincon Reis Belo Horizonte - MG Menção Honrosa
Ramon Silva de Lima São Paulo - SP Menção Honrosa
Eric Tada de Souza São Paulo – SP Menção Honrosa
Rodrigo Gabriel Caetano Piracicaba - SP Menção Honrosa
Renato Soares Nunes Rio de Janeiro - RJ Menção Honrosa
Rafael Alves Pinheiro Parnamirim - RN Menção Honrosa
Leticia da Silva Inácio S.J. da Boa Vista - SP Menção Honrosa
José Elton Albuquerque Filho Fortaleza - CE Menção Honrosa
Luiz Fernando Cirigliano Villela Uberaba - MG Menção Honrosa
Gabriel Ilharco Magalhães Juiz de Fora Menção Honrosa
Fernanda Bahia de Carvalho Coutinho Belo Horizonte - MG Menção Honrosa
Victor Venturi Campinas - SP Menção Honrosa
Lara Timbó Araújo Fortaleza - CE Menção Honrosa
Julia Langraf Scatolin Pirassununga - SP Menção Honrosa
Francisco Carvalho Osório de Souza Campinas - SP Menção Honrosa
Luiza Christófaro Bragança de Matos Belo Horizonte - MG Menção Honrosa
Francisco Jairo Rodrigues Lima Fortaleza - CE Menção Honrosa
Otavia Ruanna Cordeiro de Oliveira Salgueiro - PE Menção Honrosa
Carolina Yumi Vezato Araraquara - SP Menção Honrosa

EUREKA! N°26, 2007

70
Sociedade Brasileira de Matemática

Nível 2 (7a. e 8a. Séries)


NOME CIDADE – ESTADO PRÊMIO
Renan Henrique Finder Joinville - SC Ouro
Hugo Fonseca Araújo Juiz de Fora - MG Ouro
Thiago Ribeiro Ramos Varginha - MG Ouro
Matheus Barros de Paula Taubaté - SP Ouro
Rafael Alves da Silva Teresina - PI Ouro
Robério Soares Nunes Ribeirão Preto - SP Prata
Leonardo Pereira Stedile São Paulo - SP Prata
Gustavo Lisbôa Empinotti Florianópolis - SC Prata
Victor Reis de Abreu Cavalcanti Maceió - AL Prata
Thiago Augusto da Silva Baleixo Rio de Janeiro - RJ Prata
Heverton Carlos Bezerra de Azevedo Rio de Janeiro - RJ Prata
James Jun Hong São Paulo - SP Prata
Leonardo Caruso de Oliveira Rio de Janeiro - RJ Prata
Pedro Caetano Cardoso Rio de Janeiro - RJ Prata
Illan Feiman Halpern Itatiaia - RJ Prata
Davi de Melo Pontes Mendes Fortaleza - CE Bronze
Matheus Araújo Marins São Gonçalo - RJ Bronze
Marcelo Tadeu de Sá Oliveira Sales Barreiras - BA Bronze
Dan Zylberglejd Rio de Janeiro – RJ Bronze
João Mendes Vasconcelos Fortaleza - CE Bronze
José Ailton Azevedo Araújo Filho Fortaleza - CE Bronze
Leonardo Shimizu Yojo São Paulo – SP Bronze
Gelly Whesley Silva Neves Fortaleza - CE Bronze
Frederico Gaia Costa da Silva Teresina - PI Bronze
Ana Luísa de Almeida Losnak São Paulo - SP Bronze
Saulo Moraes de Faria Niterói - RJ Bronze
Guilherme Salvador Vieira Rio Claro - SP Bronze
Fernando Fonseca Andrade Oliveira Belo Horizonte - MG Bronze
Rafael Farias Cação Campo Grande - MS Bronze
Edson Ryokei Onoga São Paulo - SP Bronze
Rafael Gribel de Paula Neves Rio de Janeiro - RJ Bronze
Luiz Castelo Branco Cavalcante Teresina - PI Bronze
Filipe Gabriel Soares Rodrigues Teresina - PI Menção Honrosa
Rafael Horimoto de Freitas São Paulo - SP Menção Honrosa
Germano Luis Lopes de Mello Rio de Janeiro - RJ Menção Honrosa
Silvio Tacla Alves Barbosa São Paulo – SP Menção Honrosa
Pedro Pacheco Louzada Rio de Janeiro – RJ Menção Honrosa
André Saraiva Nobre dos Santos Fortaleza - CE Menção Honrosa
Mateus Bezerra Alves da Costa Fortaleza - CE Menção Honrosa
Leandro Lyra Braga Dognini Barcarena - PA Menção Honrosa
Gabriel de Andrade Issisaki Guaíra - SP Menção Honrosa
Isabella Amorim Gonzalez Maceió - AL Menção Honrosa
André Bina Possatto São Caetano - SP Menção Honrosa
Obed Leite Vieira Fortaleza – CE Menção Honrosa
Eduardo Kaiser Ururahy Nunes Itatiaia - RJ Menção Honrosa
Paulo Ricardo de Souza Costa Rio de Janeiro - RJ Menção Honrosa
Guilherme Vieira Melo Fortaleza - CE Menção Honrosa
Stephane Hilda Barbosa Lima Fortaleza - CE Menção Honrosa
Camila Miraglia Ribeiro Curitiba - PR Menção Honrosa
Patrícia Fernanda Hongo Bragança Paulista - SP Menção Honrosa
Marcel Ichiro Bastos Kamiyama São Paulo - SP Menção Honrosa
Camilla Kikuchi São Paulo - SP Menção Honrosa
Yuri Santana do Carmo Belém - PA Menção Honrosa
Juliana Rangel Cenzi Jacareí - SP Menção Honrosa
Rafael Eiki Takemura São Paulo – SP Menção Honrosa

EUREKA! N°26, 2007

71
Sociedade Brasileira de Matemática

Nível 3 (Ensino Médio)


NOME CIDADE – ESTADO PRÊMIO
Jose Marcos Andrade Ferraro São Paulo – SP Ouro
Henrique Pondé de Oliveira Pinto Salvador - BA Ouro
Guilherme Rodrigues Nogueira de Souza São Paulo - SP Ouro
Leandro Farias Maia Fortaleza – CE Ouro
Ramon Moreira Nunes Fortaleza – CE Ouro
Rafael Mendes de Oliveira Rio de Janeiro – RJ Ouro
Regis Prado Barbosa Fortaleza - CE Ouro
Edson Augusto Bezerra Lopes Fortaleza - CE Prata
André Linhares Rodrigues Fortaleza – CE Prata
Willy George do Amaral Petrenko Rio de Janeiro - RJ Prata
Leonardo Ribeiro de Castro Carvalho São Paulo - SP Prata
Rodrigo Viana Soares Fortaleza – CE Prata
Artur de Almeida Losnak São Paulo - SP Prata
Paulo André Carvalho de Melo Rio de Janeiro - RJ Prata
Renato Rebouças de Medeiros Fortaleza - CE Prata
Rafael Sampaio de Rezende Fortaleza - CE Prata
Rafael Tupynambá Dutra Belo Horizonte - MG Prata
Alfredo Roque de Oliveira Freire Filho Salvador - BA Prata
Rafael Morioka Oda São Paulo - SP Bronze
Adenilson Arcanjo de Moura Júnior Fortaleza - CE Bronze
Giuliano Pezzolo Giacaglia São Paulo - SP Bronze
Wilson Camara Marriel Rio de Janeiro - RJ Bronze
Rafael Sabino Lima Rio de Janeiro - RJ Bronze
Marlen Lincoln da Silva Fortaleza – CE Bronze
César Ryudi Kawakami São Paulo – SP Bronze
Hugo Musso Gualandi Vitória - ES Bronze
Raphael Rodrigues Mata Salvador - BA Bronze
Alexandre Hideki Deguchi Martani São Paulo – SP Bronze
Ricardo Turolla Bortolotti Rio Claro – SP Bronze
Felipe Gonçalves Assis Campina Grande - PB Bronze
Max Douglas Peixoto da Silva Fortaleza - CE Bronze
Êurope Moraes Gorito Rio de Janeiro - RJ Menção Honrosa
Fernando Nascimento Coelho Fortaleza – CE Menção Honrosa
Guilherme Philippe Figueiredo Fortaleza - CE Menção Honrosa
Marcelo Matheus Gauy S.J. do Rio Preto - SP Menção Honrosa
Luiz Carlos da Silva Sobral Aracaju - SE Menção Honrosa
Iuri Lima Ribeiro Fortaleza - CE Menção Honrosa
Alex Atsushi Takeda Londrina - PR Menção Honrosa
Gabriel Caser Brito Rio de Janeiro - RJ Menção Honrosa
Jose Armando Barbosa Filho Fortaleza - CE Menção Honrosa
Henrique Hiroshi Motoyama Watanabe São Paulo - SP Menção Honrosa
João Luiz de Oliveira Madeira São Paulo - SP Menção Honrosa
Lúcio Eiji Assaoka Hossaka Curitiba - PR Menção Honrosa
Pedro Pinheiro de Negreiros Bessa Fortaleza - CE Menção Honrosa
Davi Lopes Alves de Medeiros Fortaleza - CE Menção Honrosa
Paulo Sérgio de Castro Moreira Fortaleza - CE Menção Honrosa
Roberto Akiba de Oliveira Sorocaba - SP Menção Honrosa
Enzo Haruo Hiraoka Moriyama São Paulo - SP Menção Honrosa
Alexandre Azevedo Cezar Fortaleza – CE Menção Honrosa
José Airton Coelho Lima Filho Fortaleza – CE Menção Honrosa
Pedro Paulo Gondim Cardoso Salvador - BA Menção Honrosa
Thiago da Silva Pinheiro São Paulo - SP Menção Honrosa

EUREKA! N°26, 2007

72
Sociedade Brasileira de Matemática

Nível Universitário

NOME CIDADE – ESTADO PRÊMIO


Fábio Dias Moreira Rio de Janeiro - RJ Ouro
Alex Corrê Abreu Niterói - RJ Ouro
Humberto Silva Naves S.J. dos Campos - SP Ouro
Samuel Barbosa Feitosa Fortaleza - CE Ouro
Levi Máximo Viana Rio Janeiro - RJ Ouro
Rafael Daigo Hirama S.J. dos Campos - SP Prata
Rafael Marini Silva Vila Velha - ES Prata
Thiago Barros Rodrigues Costa Fortaleza - CE Prata
Henry Wei Cheng Hsu São Paulo - SP Prata
Murilo Vasconcelos Andrade Rio de Janeiro - RJ Prata
Luiz Felipe Marini Silva S.J. dos Campos - SP Prata
Thiago da Silva Sobral S.J. dos Campos - SP Prata
Felipe Rodrigues Nogueira de Souza São Paulo - SP Prata
Thiago Costa Leite Santos São Paula - SP Prata
Raphael Constant da Costa Rio de Janeiro - RJ Prata
Eduardo de Moraes Rodrigues Poço São Paulo - SP Bronze
Elton Gomes Coriolano Fortaleza - CE Bronze
Elder Rodrigo Barbosa Coelho Rio de Janeiro - RJ Bronze
Ronaldo Rodrigues Pelá S.J. dos Campos - SP Bronze
Estillac Lins Maciel Borges Filho Belém - PA Bronze
Thomás Yoiti Sasaki Hoshina Rio de Janeiro - RJ Bronze
Luís Daniel Barbosa Coelho Rio de Janeiro - RJ Bronze
Renato Francisco Lopes Mello J. dos Guararapes - PE Bronze
Pedro Henrique Milet Pinheiro Pereira Rio de Janeiro – RJ Bronze
Pedro Henrique Silva Belisário Rio de Janeiro – RJ Bronze
Luty Rodrigues Ribeiro S.J. dos Campos - SP Bronze
José Mário da Silva Filho S.J. dos Campos – SP Bronze
Kellem Corrêa Santos Rio de Janeiro – RJ Bronze
Marcos Francisco Ferreira Martinelli Rio de Janeiro - RJ Bronze
Evandro Makiyama São Paulo – SP Menção Honrosa
Nilson Maciel de Paiva Júnior Rio de Janeiro - RJ Menção Honrosa
Igor de Castro Lima Rio de Janeiro – RJ Menção Honrosa
Eric Campos Bastos Guedes Niterói - RJ Menção Honrosa
Marcelo de Araújo Barbosa S.J. dos Campos - SP Menção Honrosa
Pedro Meira de Vasconcelos Bezerra Recife - PE Menção Honrosa
Moyses Afonso Assad Cohen Rio de Janeiro - RJ Menção Honrosa
Davi de Melo Jorge Barbosa Fortaleza -CE Menção Honrosa
Rodrigo Pereira Maranhão Rio de Janeiro - RJ Menção Honrosa

EUREKA! N°26, 2007

73
Sociedade Brasileira de Matemática

AGENDA OLÍMPICA
XXIX OLIMPÍADA BRASILEIRA DE MATEMÁTICA

NÍVEIS 1, 2 e 3
Primeira Fase – Sábado, 16 de junho de 2007
Segunda Fase – Sábado, 15 de setembro de 2007
Terceira Fase – Sábado, 27 de outubro de 2007 (níveis 1, 2 e 3)
Domingo, 28 de outubro de 2007 (níveis 2 e 3 - segundo dia de prova).

NÍVEL UNIVERSITÁRIO
Primeira Fase – Sábado, 15 de setembro de 2007
Segunda Fase – Sábado, 27 e Domingo, 28 de outubro de 2007

XIII OLIMPÍADA DE MAIO


12 de maio de 2007

XVIII OLIMPÍADA DE MATEMÁTICA DO CONE SUL
Uruguai
12 a 17 de junho de 2007

XLVIII OLIMPÍADA INTERNACIONAL DE MATEMÁTICA
19 a 31 de julho de 2007
Vietnã

XIV OLIMPÍADA INTERNACIONAL DE MATEMÁTICA UNIVERSITÁRIA
3 a 9 de agosto de 2007
Blagoevgrad, Bulgária

XXII OLIMPÍADA IBEROAMERICANA DE MATEMÁTICA
6 a 16 de setembro de 2007
Coimbra, Portugal

X OLIMPÍADA IBEROAMERICANA DE MATEMÁTICA UNIVERSITÁRIA
5 de novembro de 2007

EUREKA! N°26, 2007

74
Sociedade Brasileira de Matemática

COORDENADORES REGIONAIS
Alberto Hassen Raad (UFJF) Juiz de Fora – MG
Américo López Gálvez (USP) Ribeirão Preto – SP
Amarísio da Silva Araújo (UFV) Viçosa – MG
Andreia Goldani FACOS Osório – RS
Antonio Carlos Nogueira (UFU) Uberlândia – MG
Ali Tahzibi (USP) São Carlos – SP
Benedito Tadeu Vasconcelos Freire (UFRN) Natal – RN
Carlos Alexandre Ribeiro Martins (Univ. Tec. Fed. de Paraná) Pato Branco - PR
Carmen Vieira Mathias (UNIFRA) Santa María – RS
Claus Haetinger (UNIVATES) Lajeado – RS
Cleonor Crescêncio das Neves (UTAM) Manaus – AM
Cláudio de Lima Vidal (UNESP) S.J. do Rio Preto – SP
Denice Fontana Nisxota Menegais (UNIPAMPA) Bagé – RS
Edson Roberto Abe (Colégio Objetivo de Campinas) Campinas – SP
Élio Mega (Faculdade Etapa) São Paulo – SP
Eudes Antonio da Costa (Univ. Federal do Tocantins) Arraias – TO
Fábio Brochero Martínez (UFMG) Belo Horizonte – MG
Florêncio Ferreira Guimarães Filho (UFES) Vitória – ES
Francinildo Nobre Ferreira (UFSJ) São João del Rei – MG
Genildo Alves Marinho (Centro Educacional Leonardo Da Vinci) Taguatingua – DF
Ivanilde Fernandes Saad (UC. Dom Bosco) Campo Grande– MS
Jacqueline Rojas Arancibia (UFPB)) João Pessoa – PB
Janice T. Reichert (UNOCHAPECÓ) Chapecó – SC
João Benício de Melo Neto (UFPI) Teresina – PI
João Francisco Melo Libonati (Grupo Educacional Ideal) Belém – PA
José Luiz Rosas Pinho (UFSC) Florianópolis – SC
José Vieira Alves (UFPB) Campina Grande – PB
José William Costa (Instituto Pueri Domus) Santo André – SP
Krerley Oliveira (UFAL) Maceió – AL
Licio Hernandes Bezerra (UFSC) Florianópolis – SC
Luciano G. Monteiro de Castro (Sistema Elite de Ensino) Rio de Janeiro – RJ
Luzinalva Miranda de Amorim (UFBA) Salvador – BA
Mário Rocha Retamoso (UFRG) Rio Grande – RS
Marcelo Rufino de Oliveira (Grupo Educacional Ideal) Belém – PA
Marcelo Mendes (Colégio Farias Brito, Pré-vestibular) Fortaleza – CE
Newman Simões (Cursinho CLQ Objetivo) Piracicaba – SP
Nivaldo Costa Muniz (UFMA) São Luis – MA
Osvaldo Germano do Rocio (U. Estadual de Maringá) Maringá – PR
Raúl Cintra de Negreiros Ribeiro (Colégio Anglo) Atibaia – SP
Ronaldo Alves Garcia (UFGO) Goiânia – GO
Rogério da Silva Ignácio (Col. Aplic. da UFPE) Recife – PE
Reginaldo de Lima Pereira (Escola Técnica Federal de Roraima) Boa Vista – RR
Reinaldo Gen Ichiro Arakaki (UNIFESP) SJ dos Campos – SP
Ricardo Amorim (Centro Educacional Logos) Nova Iguaçu – RJ
Sérgio Cláudio Ramos (IM-UFRGS) Porto Alegre – RS
Seme Gebara Neto (UFMG) Belo Horizonte – MG
Tadeu Ferreira Gomes (UEBA) Juazeiro – BA
Tomás Menéndez Rodrigues (U. Federal de Rondônia) Porto Velho – RO
Valdenberg Araújo da Silva (U. Federal de Sergipe) São Cristovão – SE
Vânia Cristina Silva Rodrigues (U. Metodista de SP) S.B. do Campo – SP
Wagner Pereira Lopes (CEFET – GO) Jataí – GO

EUREKA! N°26, 2007

75
CONTEÚDO

AOS LEITORES 2

XVIII OLIMPÍADA DE MATEMÁTICA DO CONE SUL 3


Enunciados e resultado brasileiro

XIX OLIMPÍADA DE MATEMÁTICA DO CONE SUL 11


Enunciados e resultado brasileiro

ARTIGOS
JOGOS E FEIJOADA NO SÃO PAULO´S 13
Emanuel Carneiro

SUBSTITUIÇÕES ENVOLVENDO NÚMEROS COMPLEXOS 17


Diego Veloso Uchôa

INTEGRAIS DISCRETAS 25
Eduardo Poço

PRODUTOS NOTÁVEIS 32
Onofre Campos

OLIMPÍADAS AO REDOR DO MUNDO 38

COMO É QUE FAZ 48

SOLUÇÕES DE PROBLEMAS PROPOSTOS 50

PROBLEMAS PROPOSTOS 58

AGENDA OLÍMPICA 61

COORDENADORES REGIONAIS 62
Sociedade Brasileira de Matemática

AOS LEITORES

É com grande alegria que comemoramos em 2008 os 10 anos da Revista


EUREKA! e transmitimos aos leitores a nossa satisfação pela acolhida recebida
neste período. Durante estes 10 anos de existência temos procurado atender ao
leitor mais exigente, apresentando uma publicação específica que além de
fornecer material atualizado e de alto nível acadêmico, tem tornado o estudo da
matemática olímpica muito mais interessante e acessível a professores e jovens
olímpicos de todo o Brasil.

Neste número especial da revista apresentamos quatro artigos, cujos


autores são todos ex-olímpicos de grande destaque, além de um bom número de
novos problemas propostos por nossos leitores, que estão cada vez mais
inspirados. Agradecemos também a valiosa ajuda dos alunos que trabalharam na
revisão deste número da Eureka!: Álvaro Lopes Pedroso, Ana Luísa de Almeida
Losnak, Custódio Moreira Brasileiro Silva, Elder Massahiro Yoshida, Guilherme
Phillippe Figueiredo Hanon Guy Lima Rossi, Henrique Pondé de Oliveira Pinto,
Illan Feiman Halpern, Marco Antonio Lopes Pedroso, Rafael Horimoto de
Freitas, Renan Henrique Finder, Talita Alessandra da Silva, Thiago Saksanian
Hallak e Thiago da Silva Pinheiro, e particularmente ao Prof. Carlos Yuzo Shine,
que coordenou a revisão e que foi responsável pela seção “Como é que faz” deste
número.

Continuaremos contando com o entusiasmo e a colaboração dos nossos


leitores para que a EUREKA! continue sendo um instrumento útil à formação
matemática e à preparação olímpica do nosso público. Esperamos que gostem
deste número. Divirtam-se!

Os editores

EUREKA! N°27, 2008

2
Sociedade Brasileira de Matemática

XVIII OLIMPÍADA DE MATEMÁTICA DO CONE SUL


Enunciados e Resultado Brasileiro

A XVIII Olimpíada de Matemática do Cone Sul foi realizada na cidade


de Atlântida, Uruguai no mês de junho de 2007. A equipe brasileira foi liderada
pelos professores Yuri Gomes Lima e Samuel Barbosa Feitosa, ambos da cidade
de Fortaleza – CE.

RESULTADOS DA EQUIPE BRASILEIRA

BRA1 Renan Henrique Finder Medalha de Ouro


BRA2 Marcelo Tadeu de Sá Oliveira Sales Medalha de Prata
BRA3 Grazielly Muniz da Cunha Medalha de Prata
BRA4 Thiago Ribeiro Ramos Medalha de Prata

PRIMEIRO DIA

PROBLEMA 1
Achar todos os pares de inteiros (x, y) que satisfazem

x 3 y + x + y = xy + 2 xy 2 .

SOLUÇÃO DE MARCELO TADEU DE SÁ OLIVEIRA SALES (SALVADOR – BA)


De x 3 y + x + y = xy + 2 xy 2 temos:
x 3 y + x − xy − 2 xy 2 = − y ⇒ x( x 2 y + 1 − y − 2 y 2 ) = − y ⇒ x | y
xy + 2 xy 2 − y − x 3 y = x ⇒ y ( x + 2 xy − 1 − x3 ) = x ⇒ y | x
Então x | y e y | x com exceção de x = 0 ou y = 0. Nesses dois casos temos que
ambos têm que ser 0. Assim (x, y) = (0, 0) é a nossa primeira solução. Se x | y
então x ≤ y (eu já desconsiderei x = 0 e y = 0) e se y | x então y ≤ x , daí
x = y . Assim temos dois casos:
Primeiro caso: x = y
Substituindo temos x 4 + 2 x = x 2 + 2 x 3 . Como x ≠ 0 e y ≠ 0 então podemos
simplificar. Assim, x 3 + 2 = x + 2 x 2 e daí x | 2 , então x ∈{−2, −1,1, 2}. Desses
valores, o único que não admite solução é x = – 2 então para esse caso
( x, y ) = (−1, −1);(1,1);(2, 2).

EUREKA! N°27, 2008

3
Sociedade Brasileira de Matemática

Segundo caso: x = – y.
Substituindo temos − x 4 = − x 2 + 2 x 3 .
−2 ± 8
Como x ≠ 0 temos − x 4 = − x 2 + 2 x 3 ⇔ − x 2 = −1 + 2 x e daí x = que não
2
é inteiro, então não há solução para esse caso.
Assim as soluções são ( x, y ) = (−1, −1);(0,0);(1,1) e (2, 2).

PROBLEMA 2
Considere 100 inteiros positivos tais que sua soma é igual ao seu produto.
Determinar a quantidade mínima de números 1 que podem existir entre os 100
inteiros.

SOLUÇÃO DE RENAN HENRIQUE FINDER (JOINVILLE – SC)


Seja K o número de 1´s que aparecem. Sejam a1 , a2 ...a100 os números, com
a1 = a2 = ... = aK = 1.
a1 + a2 + ... + a100 = a1a2 ...a100
K + aK +1 + ... + a100 = aK +1aK + 2 ...a100
Vamos minimizar aK +1aK + 2 ...a100 − aK +1 − aK + 2 − ... − a100 . Para isso, suponha
a j ≥ 2.
Note que
aK +1 ...a j ...a100 − aK +1 − ... − a j − ... − a100 ≥ 2aK ...a j −1a j +1 ...a100 − 2 − aK +1 − ...
−a j −1 − a j +1 − ... − a100 ⇔ aK +1 ...a j ...a100 − a j ≥ 2aK +1 ...a j −1a j +1 ...a100 − 2 ⇔
aK +1...a j −1a j +1...a100 (a j − 2) ≥ a j − 2, o que ocorre de fato. Então, a diferença é
mínima quando aK +1 = aK + 2 = ... = a100 = 2. Logo,
K = aK +1aK + 2 ...a100 − aK +1 − ... − a100 ≥ 2100− K − 2 ⋅ (100 − K ) ⇒ 0 ≥ 2100− K − 200 + K ⇒
2100 − K ≤ 200 − K . Se K ≤ 93 200 − K ≤ 107 e 2100 − K ≥ 27 = 128, o que obriga
K ≥ 94. Note que há um exemplo para K = 95:
a1 = a2 = ... = a45 = 1
a96 = a95 = 2
a98 = a99 = a100 = 3
A soma é 195 + 2 ⋅ 2 + 3 ⋅ 3 = 95 + 4 + 9 = 108 e o produto é 22 ⋅ 33 = 108. Resta o
caso K = 94, isto é, o caso a + b + c + d + e + f + 94 = abcdef . Supondo a, b ≥ 3,
minimizemos abcdef − a − b − c − d − e − f . Temos

EUREKA! N°27, 2008

4
Sociedade Brasileira de Matemática

abcdef − a − b − c − d − e − f ≥ 3bcdef − 3 − b − c − d − e − f ⇔ ( a − 3)bcdef ≥ a − 3,


o que ocorre. Além disso,
abcdef − a − b − c − d − e − f ≥ 2abcde − a − b − c − d − e − 2 ⇔ abcde( f − 2) ≥ f − 2,
o que ocorre, pois f ≠ 1, logo f ≥ 2.
Concluímos que a expressão é mínima se a = 3 e f = 2. Analogamente, ela é
mínima quando b = 3 e c = d = e = 2. Então,
abcdef − a − b − c − d − e − f ≥ 32 24 − 4 ⋅ 2 − 3 ⋅ 2 = 144 − 14 = 130 > 94.
Então, é impossível abcdef − a − b − c − d − e − f = 94 se duas das variáveis
forem ≥ 3 . Já se só uma for ≥ 3 (digamos que a, teremos b = c = d = e = f = 2,
logo
25 a − 10 − a = 94 ⇒ 31a = 104, absurdo, pois 31 /| 104. Se todas as variáveis
forem iguais a 2, obtemos também 31a = 104 ⇒ 31 ⋅ 2 = 104, absurdo. Então
K = 95 é o máximo que podemos obter.

PROBLEMA 3
Seja ABC um triângulo com todos os seus ângulos agudos, de alturas AD, BE e
CF (com D em BC, E em AC e F em AB). Seja M o ponto médio do segmento
BC. A circunferência circunscrita ao triângulo AEF corta a reta AM em A e X. A
reta AM corta a reta CF em Y. Seja Z o ponto de encontro entre as retas AD e BX.
Demonstrar que as retas YZ e BC são paralelas.

SOLUÇÃO DA BANCA
Observemos que AFHE é inscritível, pois ∠AEH = ∠AFH = 90°.
Daí que ∠AXH = 90°.
Seja A´ um ponto sobre a semireta AM tal que AM = MA´. O quadrilátero ABA´C
é um paralelogramo, onde

∠A´BH = ∠A´BC + ∠CBH


= ∠ACB + (90° − ∠ACB)
= 90°
= ∠A´ XH ,
Ou seja que o quadrilátero BHXA´ é inscritível. Alem disso BHCA´ também é
inscritível já que

∠BHC + ∠BA´C = (180° − ∠BAC ) + ∠BAC = 180°.

Desta forma os pontos B, H, X, C são concíclicos, onde

EUREKA! N°27, 2008

5
Sociedade Brasileira de Matemática

∠XBM = ∠XBC = ∠XA´C = ∠BAM (1)

Seja T = AB ∩ XH . Notemos que H também é o ortocentro do triângulo ATY,


uma vez que TY ⊥ AY e YF ⊥ AT . Daí que AH ⊥ TY , então TY // BC.
Se provamos que Z ∈ TY , o problema estará terminado. Seja então Z ´= AD ∩ TY .
Vamos mostrar que Z = Z ´. Agora, ∠HZ ´Y = ∠HXY = 90° então HXYZ´ é
inscritível, e portanto,

∠XZ ´Y = ∠XHY = ∠FAX − ∠BAM (2)

Das relações (1) e (2) segue-se que ∠XZ ´Y = ∠XBM , ou seja, que os pontos B, Z´
e X são colineares. Porém então Z ´∈ AD ∩ BX = Z e assim Z = Z ´ , como
queríamos.

F X
H
Y
Z

B D M C

PROBLEMA 4
Considere um tabuleiro 2007 × 2007. São pintadas algumas casas do tabuleiro.
Dizemos que o tabuleiro é charrua se nenhuma linha está totalmente pintada e
nenhuma coluna está totalmente pintada.

a) Qual é o número máximo k de casas pintadas que um tabuleiro charrua pode


ter?
b) Para tal número k, calcular o número de tabuleiros charruas distintos que
existem.

EUREKA! N°27, 2008

6
Sociedade Brasileira de Matemática

SOLUÇÃO DE GRAZIELLY MUNIZ DA CUNHA (FORTALEZA – CE)


a) Note que todas as colunas têm que ter no máximo 2006 casas pintadas. Como
são 2007 colunas então o número de casas pintadas é no máximo
2007 × 2006, número que é atingido pintando todas as casas, exceto uma
diagonal, como na figura abaixo

2007

2007
b) como para k no máximo iremos pintar 2006 casas em cada coluna, então temos
que escolher qual casa ficará sem ser pintada. E note que não podemos ter duas
casas sem serem pintadas em uma mesma linha, pois se não terá uma linha que
ficará toda preenchida. Logo para a primeira coluna poderemos escolher qualquer
uma das 2007 casas para não ser pintada, na segunda coluna podemos escolher
qualquer uma de 2006 casas, pois não podemos escolher uma casa que esteja na
mesma linha qua a que foi escolhida na primeira coluna, na terceira coluna temos
2005 escolhas, na quarta 2004 escolhas e assim sucessivamente, logo são
2007 × 2006 × 2005... = 2007! maneiras de escolher, logo são 2007! tabuleiros
charruas distintos, com o número k.

PROBLEMA 5
Seja ABCDE um pentágono convexo que satisfaz as seguintes condições:
• Existe uma circunferência Γ tangente a cada um de seus lados.
• As medidas de todos os seus lados são números inteiros.
• Ao menos um dos lados do pentágono mede 1.
• O lado AB mede 2.
Seja P o ponto de tangência de Γ com o lado AB.

a) Determinar as medidas dos segmentos AP e BP.

EUREKA! N°27, 2008

7
Sociedade Brasileira de Matemática

b) Dar um exemplo de um pentágono que satisfaz as condições estabelecidas.

SOLUÇÃO DE RENAN HENRIQUE FINDER (JOINVILLE – SC)


a) Sejam Q, R, S e T os pontos de tangência de Γ em BC, CD, DE e EA,
respectivamente, como a seguir:

E
S T

D
A

R P

C Q B

Seja AP = x. Temos:
AB = 2 ⇒ BP = 2 – x
QB = BP ⇒ QB = 2 – x
QC = BC – QB = BC – 2 + x
CR = QC ⇒ CR = BC – 2 + x
DR = CD – CR = CD – BC + 2 – x
DS = DR ⇒ DS = CD – BC + 2 – x
ES = DE – DS = DE – C D + BC – 2 + x
ET = ES ⇒ ET = DE – CD + BC – 2 + x
AE = AT + TE = x + DE – CD + BC – 2 + x ⇒ 2x = AE – DE + CD – BC +2 ∈ *+
Como x < AB = 2 ⇒ 2 x < 4, pode-se ter 2 x = 3, 2 x = 2 ou 2 x = 1. O item b)
3 1
mostra uma configuração para x = e o caso x = é obviamente análogo
2 2
(troque A por B e C por E). Resta ver o que acontece se x = 1.
Temos AT = 1 ⇒ AE > 1. Então AE ≥ 2, porque AE ∈ . Como ET = AE − 1,
vale que ET ∈ e ET ≥ 1.
Assim, SE ≥ 1, pois SE = TE. Desse modo, DE > SE ⇒ DE ≥ 2. Como
DS = DE − SE tem-se DS ≥ 1, mas então DR ≥ 1 ⇒ DC > 1. Logo DC ≥ 2, uma
vez que DC ∈ . Mas então CR ≥ 1, já que CR = DC − DR = DC − DS ∈ . E
também CQ ≥ 1 e CQ ∈ , já que RC = QC. Deste modo, BC > CQ ⇒ BC > 1. E
AB > AP = 1. Então todos os lados são maiores que 1: absurdo.

EUREKA! N°27, 2008

8
Sociedade Brasileira de Matemática

3 w 1 3
Obs. No desenho do item b), de fato AP = , pois AP = x + = 1 + = (o
2 2 2 2
ponto P bissecta o segmento de medida w pois esse segmento é o lado de um
hexágono regular). Por outro lado, na verdade já provamos que só podíamos ter
3 1 3 1
AP = ou AP = . Isso mostra que AP = e BP = é uma possibilidade (e
2 2 2 2
3 1
que, analogamente, BP = e AP = também é).
2 2

b) Tome um hexágono regular de lado 1, seu incírculo Γ , dois de seus lados não
opostos e não adjacentes e os prolongue. Seja A a intersecção obtida. O
pentágono ABCDE é o fecho convexo da união dos pontos do hexágono e do
ponto A, como a seguir:
C D

B E

P
w 120° 120° z

x y

Note que o triângulo de lados x e y, com vértice em A, tem dois ângulos de


180° − 120° = 60°, logo é eqüilátero, e de lado 1 (seu terceiro lado é lado do
hexágono L )! Como w e z são lados do hexágono, w = x = y = z = 1. Então:
• ABCDE é circunscrível
• BC = CD = DE = 1
• AB = EA = w + x = y + z = 1 + 1 = 2
Todos os lados são inteiros, como queríamos.

PROBLEMA 6
Demonstrar que, para cada inteiro positivo n, existe um inteiro positivo k tal que a
representação decimal de cada um dos números k, 2k,..., nk contém todos os
dígitos 0, 1, 2, 3, 4, 5, 6, 7, 8, 9.

EUREKA! N°27, 2008

9
Sociedade Brasileira de Matemática

SOLUÇÃO DE MARCELO TADEU DE SÁ OLIVEIRA SALES (SALVADOR – BA)


Lema: Para todo n, existe k ∈ *
+ tal que nk contém todos os dígitos 0, 1, 2, ..., 9,
onde n ∈ *
+ .
Demonstração: Ao fatorarmos n temos que n é da forma 2a ⋅ 5b ⋅ q onde q é o
produto dos outros fatores primos de n. Seja c = máx (a, b).
Vou mostrar que para n = 10c ⋅ q o lema é válido. Temos que 2a ⋅ 5b ⋅ q é divisor
de 10c ⋅ q, e por Bézout existe um x tal que 10c ⋅ qx ≡ 10c (mod⋅ 10c +1 ) , pois
mdc(q,10) = 1
Assim, se multiplicarmos 10c ⋅ qx por 2, 3, ..., 9 ele dará restos 2 ⋅ 10c ,...,9 ⋅ 10c ,
ou seja, aparecerem os dígitos que nós quisermos na base decimal. Considere
10c (qx + 2 ⋅ 10 p qx + 3qx ⋅ 102 p + ... + 9qx ⋅ 108 p ) onde x é o número tal que
10c ⋅ qx ≡ 10c (mod⋅ 10c +1 ) e p é um inteiro tal que 10 p > 9 ⋅ 10c ⋅ qx.
Então k = 10( x + 2 ⋅ 10 p ⋅ x + 3 ⋅ 102 p ⋅ x + ... + 9 ⋅ 108 p ⋅ x) satisfaz as nossas condições
pois ao multiplicarmos k por 10c ⋅ q temos o seguinte:
Em 10c ⋅ q ⋅ x vai aparecer um dígito 1 porque 10c ⋅ q ⋅ x ≡ 10c (mod10c +1 ). Em
10c + p ⋅ q ⋅ 2 x vai aparecer um dígito 2.
Em 10c +8 p ⋅ q ⋅ 9 x vai aparecer um dígito 9 e eu multipliquei tudo por 10 para
aparecer o 0. Assim k é o que queríamos.
Agora vou terminar o problema por indução
Casos iniciais → n = 1 e k = 1234567890
k = 1234567890617283945
n=2e
ou 6172839450
Passo indutivo → suponha que até n é verdadeira então existe um k tal que k, 2k,
3k,...,nk têm todos os dígitos 0,1,...,9. Observe que se multiplicarmos k por
10 para ∈ +* continua sendo verdade para k , 2k , caso, nk .
Pelo lema, temos que existe um r tal que (n + 1)r tem todos os dígitos 0,1,...,9.
Assim, fazendo s = 10 k + r , onde é um inteiro tal que 10 > (n + 1)r , esse s
satisfaz as condições do enunciado para 1 até (n + 1) pois de 1 até n teremos que
s, 2 s,..., ns terão todos os dígitos porque 10 k ,...,10 nk têm (e porque é grande o
suficiente para que nr < 10 ).
E para n + 1 temos que (n + 1)r < 10 terá todos os dígitos. Assim s satisfaz as
condições. Portanto para todo n inteiro positivo existe tal inteiro k.

EUREKA! N°27, 2008

10
Sociedade Brasileira de Matemática

XIX OLIMPÍADA DE MATEMÁTICA DO CONE SUL


Enunciados e Resultado Brasileiro

A XIX Olimpíada de Matemática do Cone Sul foi realizada na cidade de


Temuco, Chile no mês de maio de 2008. A equipe brasileira foi liderada pelos
professores Cícero Thiago Magalhães e Bruno Holanda, ambos da cidade de
Fortaleza – CE.

RESULTADOS DA EQUIPE BRASILEIRA

BRA1 Gustavo Lisbôa Empinotti Medalha de Bronze


BRA2 Marcelo Tadeu de Sá Oliveira Sales Medalha de Prata
BRA3 Matheus Araújo Marins Medalha de Bronze
BRA4 Matheus Secco Torres da Silva Medalha de Prata

PRIMEIRO DIA

PROBLEMA 1
Defina I(n) como o resultado de inverter os números de um algarismo. Por
exemplo, I (123) = 321, etc. Calcule todos os inteiros 1 ≤ n ≤ 10000 tais que
n
I (n) =   .
2

PROBLEMA 2
Seja ABC um triângulo, P um ponto em seu interior e X, Y e Z pontos em BC, AC
e AB respectivamente tais que ∠PZB = ∠PXC = ∠PYA. Considere os pontos U,
W e V sobre BC, AC e AB (ou seus prolongamentos, se necessário) tais que PV =
2PY; PU = 2PX e PW = 2PZ. Sabendo que a área de XYZ é 1, calcule a área de
UVW.

PROBLEMA 3
Dois amigos A e B devem resolver a seguinte adivinha: cada um deles recebe um
número do conjunto {1, 2, ..., 250} mas não vê o número que o outro recebeu. O
objetivo é que cada amigo descubra o número do outro. O procedimento que
devem seguir é anunciar, por turnos, números inteiros positivos não
necessariamente distintos: primeiro A diz um número, em seguida B diz um
número, depois novamente A, etc., de modo que a soma de todos os números

EUREKA! N°27, 2008

11
Sociedade Brasileira de Matemática

anunciados seja 20. Demonstrar que existe uma estratégia de modo que, através
de um acordo prévio A e B possam atingir o objetivo, sem importar quais
números cada um receba no começo da adivinha.

SEGUNDO DIA

PROBLEMA 4
Qual é o maior número de casas que se pode colorir num tabuleiro 7 × 7 de
maneira que todo subtabuleiro 2 × 2 tenha no máximo 2 casas coloridas?

PROBLEMA 5
Seja ABC um triângulo isósceles de base AB. Uma semicircunferência C com
centro no segmento AB e tangente aos lados iguais AC e BC. Considera-se uma
reta tangente a C que corta os segmentos AC e BC em D e E, respectivamente.
Suponha que as retas perpendiculares a AC e BC, traçadas respectivamente por D
e E, se cortam em P interior ao triângulo ABC. Seja Q o pé da perpendicular à
PQ 1 AB
reta AB que passa por P. Demonstrar que = ⋅ .
CP 2 AC

PROBLEMA 6
Dizemos que um número é capicua se ao inverter a ordem de seus algarismos
obtivermos o mesmo número. Achar todos os números que tem pelo menos um
múltiplo não-nulo que seja capicua.

EUREKA! N°27, 2008

12
Sociedade Brasileira de Matemática

JOGOS E FEIJOADA NO SÃO PAULO´S


Emanuel A. S. Carneiro

♦ Nível Iniciante

Bem próximo ao Robert Lee Moore hall, sede do Departamento de


Matemática da Universidade do Texas em Austin, fica o celebrado restaurante
brasileiro São Paulo´s. Comida muito boa (definitivamente a melhor feijoada da
cidade), além do velho e bom guaraná Antártica são apenas alguns dos fatores
que nos levam (a comunidade brasileira aqui em Austin) a almoçar regularmente
no São Paulo´s.
Certo dia eu estava a almoçar com dois professores do departamento e
algo me fez lembrar dos meus tempos de olimpíada de Matemática. Fernando
Rodriguez-Villegas, argentino, professor na área de teoria dos números e Tamás
Hausel, húngaro, que trabalha nas áreas de geometria algébrica e topologia.
Enquanto saboreávamos as nossas feijoadas (eu e o Tamás, o Fernando no bobó
de camarão), conversávamos sobre jogos matemáticos. Dr. Rodriguez-Villegas,
um matemático extraordinário e super simpático, que além de fazer pesquisa do
mais alto nível em teoria dos números é bastante interessado em jogos e puzzles
matemáticos, nos explicava tópicos do curso que estava a ensinar nesse semestre
(Math, Puzzles and Computers) além de outras idéias de jogos que ele próprio
havia inventado.
Entre as torres de Hanói, Nim, Resta um, e coisas do tipo escritas em
guardanapos do São Paulo´s, Tamás lembrou-se de algo e me perguntou:

– Emanuel, você conhece o jogo do “15 out of 3” (15 de 3)?


Respondi que não. Ele então me deu a formulação do jogo:

– (O jogo 15 out of 3) os números de 1 a 9 estão sobre a mesa. Dois jogadores


alternadamente escolhem números para si (sem repetição) e ganha quem primeiro
completar 15 somando três de seus números.
(sugiro agora que os leitores joguem um pouquinho antes de prosseguir e
“desvendar” o mistério).
Pensei comigo mesmo: “Hummm... isso não me parece estranho...”. Eu disse:
– Bem, minha intuição me leva a crer que o primeiro jogador está em melhor
situação para negociar do que o segundo, pois vai receber mais números ao
final...

Tomás foi adiante e disse:

EUREKA! N°27, 2008

13
Sociedade Brasileira de Matemática

– Sim, você está correto. Vamos jogar! Você começa, escolha o seu primeiro
número.

– Está bem. Eu escolho o 5.


Nesse momento ele parou e me olhou curioso. – “Por que você escolheu o 5?”
perguntou. Eu disse:

– Não sei exatamente o que é, mas algo me faz lembrar um quadrado mágico, e
como o 5 sempre está no meio, achei que tinha mais chances de ganhar...

– Sua intuição mais uma vez está correta. Muito bem. Naturalmente, você deve
saber qua a menos de rotações e reflexões a configuração do quadrado mágico é
única. Por que a gente não desenha um quadrado mágico 3 × 3 aqui e tenta jogar
olhando para ele?

Após um minuto tentando lembrar como se faz um quadrado mágico,


desenhamos no guardanapo:

6 7 2

1 5 9

8 3 4

Quando vi os números na mesa matei a charada. Um quadrado mágico de 3 × 3


como visto acima é uma disposição dos números 1, 2, 3, ..., 9, sem repetição, de
modo que cada linha, coluna ou diagonal some a mesma quantidade. Nesse caso a
soma comum será 15 e o que vemos acima são todas as maneiras possíveis de se
escrever 15 como soma de 3 números:

15 = 9 + 1 + 5 = 9 + 2 + 4 = 8 + 1 + 6 = 8 + 2 + 5 =
= 8 + 3 + 4 = 7 + 2 + 6 = 7 + 3 + 5 = 6 + 5 + 4.

Se pensarmos então que o primeiro jogador marca X sobre os números do


quadrado mágico e o segundo marca O, o objetivo do jogo passa a ser completar
uma linha, coluna ou diagonal com seus símbolos. O jogo 15 out of 3 que ele me
propôs nada mais é do que uma formulação equivalente, belíssima e engenhosa,

EUREKA! N°27, 2008

14
Sociedade Brasileira de Matemática

do milenar jogo da velha (em inglês “tic-tac-toe”). Fiquei pasmo, havia ganho o
meu dia. Por alguns momentos não consegui parar de pensar na beleza e no poder
da matemática, presente até nos mínimos detalhes da nossa vida. Senti-me
orgulhoso de poder ser um pesquisador que tenta compreender essa ciência e
pequenos fatos como esse me fazem, a cada dia, ter mais consciência de que ela é
muito maior do que nós.

Observamos: A história acima se passou no dia 02 de maio de 2007. Fiquei com o


guardanapo como recordação. O jogo 15 out of 3 é um belo exemplo para se
mostrar como uma pessoa que sabe matemática realmente pode levar vantagem
sobre uma pessoa menos interessada pelo assunto. Todos sabemos que o jogo da
velha não admite estratégia vencedora, mas mesmo assim o professor Tamás
Hausel jogava o 15 out of 3 com seus alunos e ganhava na maioria das vezes.
Naturalmente, ele sempre tinha seu quadrado mágico para consultas. Até que um
dia ele esqueceu-se do quadrado mágico em casa e foi derrotado por uma aluna.

Guardanapo da discussão no São Paulo´s

No verso do guardanapo acima, há outras discussões também belíssimas sobre as


torres de Hanói e versões relacionadas (de formas mais engenhosas do que a

EUREKA! N°27, 2008

15
Sociedade Brasileira de Matemática

analogia acima) inventadas pelo Dr. Rodríguez-Villegas. Ele ainda está buscando
a melhor formulação para seu jogo para poder patenteá-lo e disponibilizá-lo ao
público em geral. Isso então vai ficar para uma outra história.

PROBLEMA 1: Prove que em um quadrado mágico 3 × 3, como foi descrito acima:


(a) a soma comum deve ser 15.
(b) o número do centro deve ser 5.

PROBLEMA 2: Usando o problema anterior, prove que só existe um quadrado


mágico 3 × 3 (a menos de rotações e reflexões). Verifique também que não há
estratégia vencedora para o jogo da velha, em outras palavras, se os dois
jogadores jogam certo, sempre dá empate.

REFERÊNCIAS:

[1] Para ver outras discussões sobre jogos e invariantes, há outras listas em minha página
pessoal: http://www.math.utexas.edu/users/ecarneiro na seção math olympiads.
[2] Para mais informações sobre os trabalhos e o curso (Math, Puzzles and Computers) do
Dr. Rodriguez-Villegas sua página pessoal é www.math.utexas.edu/users/villegas/S07.
Um dos jogos que ele criou está descrito no paper: Rodriguez Villegas, F.; Sadun, L.:
Voloch, J.F. Blet: a mathematical puzzle. Amer. Math. Monthly 109 (2002), no. 8, 729-
740.

[3] URL: página pessoal do Dr. Hausel é http://www.math.utexas.edu/users/hausel

EUREKA! N°27, 2008

16
Sociedade Brasileira de Matemática

SUBSTITUIÇÕES ENVOLVENDO NÚMEROS COMPLEXOS


Diego Veloso Uchôa

♦ Nível Avançado

É bastante útil em problemas de olimpíada onde temos igualdades ou


queremos encontrar um valor de um somatório fazermos substituições por
números complexos aliada a outras ferramentas. Para alguns problemas que
possuam equações com funções seno e co-seno é importante saber a fórmula de
Euler que escreve um número complexo na forma polar o que simplifica quando
fazemos multiplicações ou somatórios.
Um número complexo pode se escrever na sua forma trigonométrica
ρ (cosθ + isenθ ) ou na sua forma polar ρ ⋅ e iθ de onde temos que
cos θ + isenθ = eiθ (Fórmula de Euler).
Segundo essa equação podemos fazer θ = α ou θ = −α , de onde
temos:
eiα = cos α + isenα (I)
e −iα = cos α − isenα (II)
Somando I com II, temos:
e iα + e − iα
cos α =
2
Subtraindo I de II, temos:
eiα − e −iα
senα =
2i
Segundo a fórmula de Euler podemos verificar imediatamente a fórmula de De
Moivre:
Para todo n natural temos que ( cos α + isenα ) = cos(nα ) + isen(nα ) .
n

No seguinte problema da OIMU, quais idéias imediatas poderíamos ter sem


conhecer a fórmula de Euler?

PROBLEMA 1: (OIMU – 2001)


Calcule:

 π   2π   (n − 1)π   nπ  
∑ cos n  cos n
... cos
  n
 cos 
  n 
n =1 

EUREKA! N°27, 2008

17
Sociedade Brasileira de Matemática

  π   2π   (n − 1)π   nπ 
Pn = cos  cos ... cos  cos  , observe
n   n 
SOLUÇÃO: Seja
 n  n  
 k 
de imediato que se n = 2k então P2 k = 0 pois cos π  = 0 . Portanto
 2k 
considere n = 2k + 1 então

  π   2π   2kπ   (2k + 1)π 


P2 k +1 = cos  cos ... cos  cos 
  2k + 1   2k + 1   2 k + 1   2k + 1 
 i   2k + 1 − i 
Observe que cos  π  = − cos  π ⇒
 2k + 1   2k + 1 

2
  π   2π   kπ  k +1
P2 k +1 = cos  cos ... cos  ⋅ (−1) ,
  2k + 1   2k + 1   2k + 1 

Considere

~   π   2π   (k − 1)π   kπ 
P 2 k +1 = cos  cos ... cos  cos 
  2 k + 1   2k + 1   2 k + 1   2k + 1  
−j
 w +w w2 j + 1
π j
i  j
Fazendo w = e 2 k +1 então cos π= = portanto
 2k + 1  2 2w j
~  w2 + 1  w4 + 1   w2 k + 1  
P 2 k +1 =   2  
... k 
. Faça então a seguinte multiplicação no
 2 w  2 w   2 w  
numerador e no denominador

~  w2 + 1  w4 + 1   w2 k + 1  ( w + 1)( w3 + 1)...( w2 k −1 + 1) 
P 2 k +1 =   2  
... k  3 2 k −1 ,
 2 w  2 w   2w  ( w + 1)( w + 1)...( w + 1) 
agrupando no numerador os termos tais que

( w 2 j + 1)( w 2 k +1−2 j + 1) = ( w 2 k +1 + w 2 k +1− 2 j + w 2 j + 1) = w 2 j + w 2 k +1− 2 j , já que


w 2 k +1 = −1 , com j variando de 1 até k. Agrupando agora os termos do
denominador podemos ver que

EUREKA! N°27, 2008

18
Sociedade Brasileira de Matemática

~ 1 ( w2 + w2 k −1 )( w4 + w2 k −3 )...( w2 k + w)
P 2 k +1 = , e usando que w 2 k +1 = −1
2k ( w2 k + w)( w2 + w2 k −1 )...( wk +1 + wk )
podemos simplificar a expressão para
~ 1 ( w2 − w−2 )( w4 − w−4 )...( w2 k − w−2 k )
P 2 k +1 = . Agora, olhando para o
2k (− w−1 + w)( w2 − w−2 )...(− w− k + wk )
−2 j
numerador, podemos escolher os termos ( w − w ) tais que 2 j > k que são
2j

m termos (para algum m) e substituí-los por (−1)( w2 k +1+ 2 j − w− (2 k +1− 2 j ) ) de forma


que o numerador e o denominador serão iguais a menos de um sinal (e do fator
2 k ), i.e,
~ 1 (− w −1 + w)( w 2 − w −2 )...(− w − k + w k ) ~
m 1
P 2 k +1 = k −1 −2 −k
( − 1) m
⇒ P 2 k +1 = ( −1)
2 (− w + w)( w − w )...(− w + w )
2 k
2k
2
k +1   k +1 1 1
~
Portanto temos que P2 k +1 = (−1)  P 2 k +1  = (−1) 2k
= (−1) k +1 k , e então
  2 4
como Pn = 0 para todo n par,
∞ ∞ ∞ k
 1 1 1

n =1
Pn = ∑
k =0
P2 k +1 = ∑ −  −  = −1 + − + ... (soma de P.G infinita), e assim
k =1  4  4 16

−1 4
∑P n =
1
=− .
5
n =1 1+
4
Observação: Esse problema pede para demonstrarmos um resultado relacionado
aos polinômios de Chebyshev do segundo tipo.

PROBLEMA 2: (OBM – U 2001)

−x
Seja f ( x) = e ⋅ senx . Calcule f (2001) (0) . (Denotamos por f ( n ) ( x) a
derivada de ordem n no ponto x; assim, f (2) ( x) = f '' ( x) ).

e ix − e − ix e (i −1) x e − ( i +1) x
SOLUÇÃO: f ( x) = e − x ⋅ senx = e − x ⋅ = − ⇒ após n
2i 2i 2i
derivações teremos

EUREKA! N°27, 2008

19
Sociedade Brasileira de Matemática

1
f ( n ) ( x) =  (i − 1) n ⋅ e x (i −1) − ( −1 − i ) n ⋅ e − ( i +1) x  ; para n = 2001 e x = 0, temos
2i  
1
que f (2001) (0) =  (i − 1) 2001 − (−1 − i ) 2001  sendo que (i − 1) 4 = (−(i + 1)) 4 = −4.
2i

Assim
4500
1
( ) ( ) ⋅ ( −1 − i )  =
500 500
f (2001) (0) = (i − 1) 4 ⋅ (i − 1) − (−(i + 1)) 4 (2i ) = 21000. Ou
2i   2i
tro caminho possível para a solução desse problema seria: após 4 derivações de f
perceber um ciclo e assim calcular f (2001) (0) , método esse mais trabalhoso do que
o apresentado.

PROBLEMA 3: (IMO – 1963)

π   2π   3π  1
Prove que cos  − cos  + cos  =
7  7   7  2

π
i
SOLUÇÃO: Fazendo w = e 7 o problema se torna equivalente a demonstrar que:

w + w −1 w 2 + w −2 w 3 + w −3 1 w2 + 1 w4 + 1 w6 + 1
− + = ⇔ − + =1⇔
2 2 2 2 w w2 w3

( w 4 + w 2 ) − ( w 5 + w) + ( w 6 + 1) = w 3 ⇔ w 6 − w 5 + w 4 − w 3 + w 2 − w + 1 = 0
.
Veja que isso é a soma dos termos de uma P.G cujo primeiro termo é 1 e a razão
é − w.
Somando a P.G:
π
(− w) 7 − 1 1−1 i
S PG = = = 0 . Lembre que w = e 7 ⇒ w7 = −1
− w −1 − w −1
e portanto a igualdade é realmente verdadeira.

O seguinte resultado (muito conhecido) tem por objetivo mostrar a


importância dos números complexos em problemas de alto grau de dificuldade e
que aparentemente não têm nenhuma conexão com números complexos.

EUREKA! N°27, 2008

20
Sociedade Brasileira de Matemática

1 π2∞
PROBLEMA 4: Prove que ∑ 2 =
n =1 n 6

π
SOLUÇÃO: Sabemos que para 0 < x < a desigualdade senx < x < tan x é
2
1
verdadeira. De onde segue que cot x < < 1 + cot 2 x . Agora fazendo
2

x2

x= com k = 1,2,..., m e somando de k = 1 até k = m nós obtemos
2m + 1

m
kπ (2m + 1) 2 m
1 m

(i) ∑ cot ≤ ∑ ≤ + ∑
2
m cot 2
k =1 2m + 1 π2 k =1 k
2
k =1 2m + 1

Observe que essa inequação está próxima da desejada, a idéia agora é tentar
mostrar que quando m → ∞ o termo central fica “imprensado” entre dois limites
que convergem para um mesmo valor.
Para isso vamos usar um truque que usa números complexos. Pela lei de De
Moivre e usando binômio de Newton temos :
n
 n
cos(nt ) + isen(nt ) = (cos t + sent )n = sennt (cot(t ) + i)n = sennt ∑ k  ⋅ i
k =0
k
⋅ cot n−k (t )

Fazendo n = 2m + 1 e igualando as partes imaginárias, temos:

sen((2m + 1)t )  2m + 1 2 m  2m + 1 2 m −1 m
2 m +1
=  (cot t ) −   (cot t ) + ... + (−1) . (*)
sen t  1   3 
Agora podemos tratar essa igualdade por meio de um polinômio
 2m + 1 m  2m + 1 m −1 m
Pm ( x) =  x −  x + ... + (−1)
 1   3 
kπ kπ
Substituindo t = )=0,
2
em (*) para 1 ≤ k ≤ m nos dá Pm (cot
2m + 1 2m + 1
 kπ   kπ 
pois sen  (2m + 1) =0 e sen   ≠ 0. Então,
 2m + 1   2m + 1 

EUREKA! N°27, 2008

21
Sociedade Brasileira de Matemática


x k = cot 2 , k = 1,..., m são as “m” raízes de Pm cuja soma é
2m + 1
 2m + 1
 
m
kπ  3  m(2m − 1)

k =1
cot 2
=
2m + 1  2m + 1
=
3
(ii)
 
 1 
De (i) e (ii) segue
m(2m − 1) (2m + 1) 2 m
1 m(2m − 1)
3

π2
∑k
k =1
2
≤ m+
3
π2
Multiplicando essas desigualdades por e fazendo m → ∞ chegamos
(2m + 1) 2
ao resultado desejado.

Exercícios para treinamento:

PROBLEMA 5: (IME 1990/1991)


 (2n + 1)θ 
sen  
1  2 
Prove que + cos(θ ) + cos(2θ ) + ... + cos(nθ ) =
2 θ 
2sen  
2
PROBLEMA 6: (IME-2000/2001) Dois números complexos são ortogonais se suas
representações gráficas forem perpendiculares entre si. Prove que dois números
complexos Z1 e Z 2 são ortogonais se e somente se:
Z1 ⋅ Z 2 + Z 1 ⋅ Z 2 = 0

 kπ  π 
n
PROBLEMA 7: Prove que ∑ sen 
k =1
 = cot  
n   2n 

PROBLEMA 8: Prove a identidade trigonométrica:


1 n
n
cos n (θ ) =
2n
∑  k  cos ( (n − 2k )θ ).
k =1
PROBLEMA 9: (IME – 2005/2006)
Sejam as somas S 0 e S1 definidas por

EUREKA! N°27, 2008

22
Sociedade Brasileira de Matemática

S 0 = C n0 + C n3 + C n6 + C n9 + ... + C n3[ n / 3]
S1 = C n1 + C n4 + C n7 + C n10 + ... + C n3[( n −1) / 3]+1
Calcule os valores de S 0 e S1 em função de n, sabendo que [r] representa o maior
inteiro menor ou igual ao número r.

PROBLEMA 10: (Putnam 1970)


Prove que a série de potências de e ⋅ cos(bx) (com a e b positivos) ou não tem
ax

nenhum coeficiente zero ou possui infinitos zeros.

PROBLEMA 11: Ache uma fórmula geral para:


n −1
 2 kπ 
∑ (k + 1) cos 
k =0 n
.

PROBLEMA 12: (OBM – Nível U 2004)


1
Calcule o valor de ∑ (3k + 1)(3k + 2)(3k + 3) .
k =0
n
 2n + 1 
PROBLEMA 13: (IMO 1974) Prove que o número ∑  2k + 1 ⋅ 2
K =1
3k
não é divisível

por 5 para qualquer inteiro n ≥ 0 .


n
PROBLEMA 14: Calcule o valor de ∑  .
k ≡ 2(mod3)  k 

PROBLEMA 15: (IMC 99) Atiramos um dado (com faces de número 1, 2,..., 6) n
vezes. Qual é a probabilidades de que a soma dos valores obtidos seja múltiplo de
5? Admita que as faces sejam igualmente prováveis.
Dica: Use a função
n
 x + x 2 + x3 + x 4 + x5 + x 6 
f ( x) =   .
 6 
PROBLEMA 16: Mostre que dados n pontos no círculo unitário sempre existe um
outro ponto no círculo unitário tal que o produto de suas distâncias aos n pontos
dados é maior ou igual a 2.

PROBLEMA 17: (OBM – Nível U 2007)

EUREKA! N°27, 2008

23
Sociedade Brasileira de Matemática

Dados números reais a1 , a2 ,..., an não todos nulos, encontre o (menor) período da
função
n
f ( x) = ∑ ak cos(kx).
k =1

PROBLEMA 18: (Miklós Schweitzer-1956)


{
Ache o mínimo de máx 1 + z , 1 + z 2 } se z percorre todos os números
complexos.

PROBLEMA 19: (IMO – 1995)


Seja p um primo ímpar. Ache o número de subconjuntos A de {1,2,...,2 p} tais
que
a) A tem exatamente p elementos
b) A soma de todos elementos de A é divisível por p
Dica: Use o polinômio f ( x, y ) = (1 + xy )(1 + x 2 y )...(1 + x 2 p y ).

BIBLIOGRAFIA

[1] E. Lozansky. C. Rousseau, Wining Solutions, Springer Velrlag, New York, 1996.
[2] Contests in Higher Mathematics, Hungary 1949–1961: in memoriam Miklós
Schweitzer, eds.: G. Szász, L. Gehér, I. Kovács and L. Pintér, Akadémiai Kiadó,
Budapest, 1968.

[3] URL: http://www.ime.eb.br (Site do Instituto Militar de Engenharia)

[4] URL : http://www.obm.org.br (Site da Olimpíada Brasileira de Matemática)

EUREKA! N°27, 2008

24
Sociedade Brasileira de Matemática

INTEGRAIS DISCRETAS
Eduardo Poço
♦ Nível Avançado

Integral discreta: dizemos que F (n) é integral discreta de f (n) se e somente se:

F ( n + 1) − F ( n) = f (n) , para n inteiro (a princípio).

Da mesma forma, dizemos que f (n) é a derivada discreta de F (n) .


n
Notação: ∑ f ( n) = F ( n)

Utilidade: conhecida a integral discreta F (n) da função f (n) , temos condições de


fazer o somatório:
b

∑ f (k ) = F (b + 1) − F (a) , a e b inteiros
k =a

A integral discreta transforma uma soma em soma telescópica.


Sabendo de algumas propriedades, é possível trabalhar dinamicamente com
integrais discretas para obter fórmulas novas a partir de outras conhecidas. Aqui,
não queremos provar que uma função dada é integral discreta de outra, pois essa
verificação é simples. Queremos obter ferramentas que nos possibilitem ACHAR
integrais discretas de forma rápida, para no final poder calcular o valor de um
somatório que tenha surgido de algum problema. Em alguns casos, é suficiente
saber a “cara” da integral discreta (ou seja, se é um polinômio, exponencial etc).

Algumas integrais discretas (o exercício de verificação é simples):

n n

∑ c = cn ∑ n.n! = n!
n
qn n

∑q n
=
q −1
∑ log a n = log a ( n − 1)!

EUREKA! N°27, 2008

25
Sociedade Brasileira de Matemática

 k  k
n
− cos  kn −  n
sen  kn − 
 2
∑ cos kn =   k  
2
∑ sen kn =  
k
2sen   2sen  
2 2
n
n sen ( 2n − 1)
∑ sen 2 n = 2 − 4sen1
n
n sen ( 2n − 1)
∑ cos2 n = 2 + 4sen1
n
n  n 
∑  k  =  k + 1
   
n
 n + k   +k
n
∑ n  =  n −1 
   

Propriedades

1) Assim como integrais contínuas (as primitivas), existem várias integrais


discretas para uma dada função, e todas elas diferem por uma constante.

Exemplo: 2n e 2n + 1 são integrais discretas de f (n) = 2n . Verifique pela


definição!

2) Integração discreta é uma transformação linear:

n n n

∑ [ a. f (n) + b.g (n)] = a∑ f (n) + b∑ g (n) , para constantes a e b.

A igualdade nos fornece uma integral discreta para a função do lado esquerdo,
lembre-se que podemos somar constantes do lado direito e continuar com uma
integral discreta.

n
3) Integral discreta do produto (por partes): sendo ∑ f ( n) = F ( n) e
n

∑ g (n) = G(n) , então:

EUREKA! N°27, 2008

26
Sociedade Brasileira de Matemática

n n

∑ F (n) g (n) = F (n)G(n) − ∑ f (n)G(n + 1)


n n
Exemplo: Calcule ∑ n sen n e ∑ n 2
sen n , comparando com o cálculo de

∫ x sen xdx e ∫ x
2
sen xdx

4) Sendo f ( x, n) uma função das variáveis x e n, derivável na variável x, então:

n
∂ ∂ n
∑ ∂x f ( x , n ) =
∂x
∑ f ( x, n )

Podemos usar a própria variável n, se a função tiver derivada nessa variável:

n n
d d
∑ dn f (n) = dn ∑ f (n)
n
Exemplo: Calcule ∑ nx n
, com x uma constante em relação a n.

5) Seguindo um caminho análogo, temos que:

∑ ( ∫ f ( x, n)dx ) = ∫  ∑ f ( x, n) dx + Cn
n
 n

Para alguma constante C. Essa constante é encontrada através de valores iniciais


conhecidos das funções.

(−1) k +1
0
n
xn
Exemplo: Prove que ∑
k =1 k
= ln 2 + ∫ x − 1 dx
−1

Aplicação: Soma de potências consecutivas.

Seja a seguinte função:

EUREKA! N°27, 2008

27
Sociedade Brasileira de Matemática

n
S m (n) = ∑ k m = 1m + 2m + ... + n m
k =1

Há uma fórmula recursiva em que podemos calcular S m (n) a partir de valores


anteriores (tente prová-la como exercício):

m −1 m + 1
 
(m + 1) S m (n) = (n + 1) m +1 − 1 − ∑   S k ( n)
k =0  k 

O problema dessa fórmula é a praticidade: precisamos de todas as funções


anteriores, e ainda assim faremos um trabalho algébrico grande. Com integrais
discretas, conseguimos obter S m (n) a partir de S m −1 ( n) apenas com um trabalho
aritmético.
n
Inicialmente, se queremos S m ( n) , queremos sua integral discreta ∑n m
. Usando

a propriedade que nos permite trocar a integral discreta com a contínua


(escolhendo a própria variável n como variável de integração contínua):

∑(∫ n )
n
 n 
m −1
dn = ∫  ∑ n m −1 dn + Cn
 

A integral contínua pode ser realizada sem problemas:

nmn
 n 
∑ m = ∫  ∑ nm−1 dn + Cn

Renomeando a constante a ser encontrada:

n
 n 
∑n m
= m ∫  ∑ n m −1 dn + Cn
 

Essa constante pode ser encontrada pela diferença entre integrais discretas
quando n = 0 , fornecendo o oposto da soma dos outros coeficientes já obtidos
pela integração contínua. Resumindo:

EUREKA! N°27, 2008

28
Sociedade Brasileira de Matemática

n
Se ∑n m −1
= am n m + am −1n m −1 + ... + a2 n 2 + a1n , então:

∑n m
= bm +1n m +1 + bm n m + ... + b2 n 2 + b1n
m +1
m
Com bk = ak −1 , para k = 1, 2,..., m + 1 , e b0 = −∑ bk .
k k =1

Alguns valores:

∑1 = n
n
n2 n
∑n = 2 2

n
n3 n 2 n
∑ n 2
=
3
− +
2 6
4
n
n n3 n 2
∑ n 3
=
4

2
+
4
5 4
n
n n n3 n
∑ n4 = 5 − 2 + 3 − 30

Aplicação: Soma de potências multiplicadas por progressão geométrica


n
Agora procuraremos ∑n m
x n , com x uma constante em relação a n. Observe:

d n m n n n n

dn
∑ n x = ∑ ( mn m −1 n
x + n m n
x ln x ) = m ∑ n m −1 n
x + ln x ∑ nm xn
n
Das formas iniciais de ∑n m
x n , encontramos uma função da forma:
n

∑n x = x n ( am −1n m −1 + am − 2 n m − 2 + ... + a2 n 2 + a1n )


m −1 n

com as constantes ak sendo funções de x, mas não dependendo de n. É natural


procurar uma integral discreta com a seguinte forma:

EUREKA! N°27, 2008

29
Sociedade Brasileira de Matemática

∑n m
x n = x n ( bm n m + bm −1n m −1 + ... + b2 n 2 + b1n )

m
Essa forma pode ser encontrada, e os coeficientes satisfazem bk = ak −1 ,
k
x m
i = 1, 2,..., m e b0 = ∑ bk .
1 − x k =1
Alguns valores:
n
( n − 1) x n +1 − nx n xn
∑ nx =
n
= 2 (
 x − 1) n − x  , x ≠ 1
( x − 1) ( x − 1) 
2

∑ n2 n
= 2n ( n − 2 )
n

∑n 2
2 n = 2 n ( n 2 − 4n + 6 )
n

∑n 2 3 n
= 2n ( n3 − 6n 2 + 18n − 26 )
n

∑n 4
2n = 2n ( n 4 − 8n3 + 36n 2 − 104n + 150 )

Problemas

1- Calcule as seguintes integrais discretas:

n
n n
2n (n − 1)
a) ∑n 2
  e) ∑ (n + 1)!
 3

n
n n
1
b) ∑3 n
  f) ∑n 2
+n
 2

n n
1 1
c) ∑ (n + 2)n! g) ∑ n +1 + n

2
n
n n
1
d) ∑   h) ∑ cos n cos(n + 1)
 2

EUREKA! N°27, 2008

30
Sociedade Brasileira de Matemática

n
sen 2 k
2- Calcule: lim ∑
n →∞
k =1 n

k =1
∑k 4
.2k
3- Calcule lim
n →∞ n
∑ k .2
k =1
3 n

n
n m +1
∑k
k =1
m

m +1
4- Calcule lim , com m inteiro positivo.
n →∞ nm

2n + 1 ∞
1 n
1
5- Prove que ∑ hn = ∑ , sendo hn = ∑ .
(n + n)
2 3
n =1 n k =1 k
2
n =1

6- Ache a derivada (contínua) da função gama Γ( n) para n inteiro positivo,


sabendo que Γ '(1) = −γ , a constante de Euler (um valor conhecido) e Γ(1) = 1 . A
função gama satisfaz Γ( x + 1) = x Γ( x) , para todo x real, assim Γ(n) = ( n − 1)!
para n inteiro positivo.

7- (OBM2002) O diâmetro de um conjunto S ⊂ R é definido como sendo


D( S ) = max( S ) − min( S ) . O conjunto vazio, por definição, tem diâmetro igual a
zero. Calcule a soma dos diâmetros de todos os subconjuntos de A = {1, 2,3,..., n} ,
em função de n.

REFERÊNCIAS:

[1] Uma referência sobre somatórios e algumas considerações históricas sobre o


raciocínio humano e implementação de algoritmos em computadores: “A = B”,
Marko Petkovsek, Herbert S. Wilf, Doron Zeilberger.

EUREKA! N°27, 2008

31
Sociedade Brasileira de Matemática

PRODUTOS NOTÁVEIS
Uma lista de problemas
Onofre Campos
♦ Nível Iniciante

1 1
1. Se x é um número real tal que tal que x +
= 5, determine o valor de x 2 + 2 .
x x
1
Solução: Elevando ambos os membros da equação x + = 5 ao quadrado,
x
obtemos:
1 1
x 2 + 2 x ⋅ + 2 = 25,
x x
1
e daí, x 2 + 2 = 23.
x

2. Fatore a expressão E = x3 − 5 x 2 − x + 5.

Solução: Temos
E = x3 − 5 x 2 − x + 5
= x 2 ( x − 5) − ( x − 5)
= ( x − 5)( x 2 − 1)
= ( x − 5)( x − 1)( x + 1).
3. Simplifique a expressão
x2 y2 z2
A= + + .
( x − y )( x − z ) ( y − z )( y − x) ( z − x)( z − y )

Solução: Note que podemos escrever a expressão acima da seguinte forma:


x2 y2 z2
A= − + .
( x − y )( x − z ) ( x − y )( y − z ) ( x − z )( y − z )
Assim, reduzindo a expressão ao mesmo denominador comum vem:
x2 ( y − z) − y 2 ( x − z) + z 2 ( x − y)
A= .
( x − y )( y − z )( x − z )
Por outro lado, desenvolvendo o denominador, obtemos:
( x − y )( y − z )( x − z ) = ( xy − xz − y 2 + yz )( x − z )
= x 2 y − xyz − x 2 z + xz 2 − xy 2 + y 2 z + xyz − yz 2 = x 2 ( y − z ) − y 2 ( x − z ) + z 2 ( x − y ).

EUREKA! N°27, 2008

32
Sociedade Brasileira de Matemática

Portanto:
x2 ( y − z) − y 2 ( x − z) + z 2 ( x − y)
A= 2 = 1.
x ( y − z) − y 2 ( x − z) + z 2 ( x − y)

4. Se x + y + z = 0, mostre que x 3 + y 3 + z 3 = 3 xyz.

Solução: Observe que


0 = ( x + y + z )3 = x3 + y 3 + z 3 + 3( x + y )( y + z )( x + z ).
Como x + y = − z , y + z = − x e x + z = − y, então:
x 3 + y 3 + z 3 + 3(− y )(− x)(− y ) = 0 ⇒ x 3 + y 3 + z 3 = 3 xyz.

5. Calcule o valor da expressão


 (2004)3 − (1003)3 − (1001)3 
S = .
 2004 ⋅ 1003 ⋅1001 

Solução: Vamos tomar x = 1003 e y = 1001. Dessa forma, a expressão S se reduz


a:
( x + y )3 − x 3 − y 3
S= .
xy ( x + y )
Mas, como sabemos, ( x + y )3 = x 3 + 3 x 2 y + 3xy 2 + y 3 .
Dessa forma, obtemos:
3 x 2 y + 3 xy 2 3xy ( x + y )
S= = = 3.
xy ( x + y ) xy ( x + y )

6. Sabendo que x, y e z são reais satisfazendo xyz = 1, calcule o valor da


expressão:
1 1 1
A= + + .
1 + x + xy 1 + y + yz 1 + z + xz
Solução: Como xyz = 1, então x ≠ 0, y ≠ 0 e z ≠ 0.
Assim,
z x 1
A= + +
z (1 + x + xy ) x(1 + y + yz ) 1 + z + xz
z x 1
= + +
z + xz + xyz x + xy + xyz 1 + z + xz

EUREKA! N°27, 2008

33
Sociedade Brasileira de Matemática

z x 1 z xz 1
= + + = + +
1 + z + xz 1 + x + xy 1 + z + xz 1 + z + xz 1 + z + xz 1 + z + xz
1 + z + xz
= = 1.
1 + z + xz
a2 b2
7. Se ab = 1 e a 2 + b 2 = 3, determine 2 + 2 + 2.
b a
Solução: Temos:
a2 b2 a 4 + 2a 2 b 2 + b 4 (a 2 + b 2 ) 2
+ + 2 = = = 9.
b2 a 2 a 2b2 ( ab) 2

x y z a b c x2 y 2 z 2
8. Prove que se + + =1 e + + = 0, então 2 + 2 + 2 = 1.
a b c x y z a b c
x y z
Solução: Elevando a equação + + = 1 ao quadrado, obtemos:
a b c
2 2
x y z2 x y y z x z
2
+ 2 + 2 + 2 + +  = 1,
a b c a b b c a c
ou seja,
x2 y 2 z 2  xyc + xzb + yza 
+ + + 2  = 1.
a2 b2 c2  abc 
a b c
Por outro lado, da equação + + = 0, temos xyc + xzb + yza = 0. Logo,
x y z
x2 y 2 z 2
+ + = 1.
a 2 b2 c2
9. Se a, b e c são três números distintos e satisfazem as equações:
a 3 + pa + q = 0
 3
b + pb + q = 0
 3
c + pc + q = 0,
calcule a + b + c.

Solução: Multiplicando a segunda equação por – 1 e somando com a primeira,


obtemos:
a 3 − b3 + p (a − b) = 0,
ou ainda,

EUREKA! N°27, 2008

34
Sociedade Brasileira de Matemática

(a − b)(a 2 + ab + b 2 ) + p (a − b) = 0,
(a − b)(a 2 + ab + b 2 + p) = 0.
Como a − b ≠ 0, pois os números são distintos, obtemos:
a 2 + ab + b 2 + p = 0. (*)
Analogamente, multiplicando a terceira equação por – 1 e somando com a
primeira equação, obtemos:
a 2 + ac + c 2 + p = 0. (**)
Agora, multiplicando (**) por –1 e somando com (*), obtemos:
ab − ac + b 2 − c 2 = 0,
a (b − c) + (b − c)(b + c) = 0,
(b − c)(a + b + c) = 0.
Daí, como b − c ≠ 0, segue que a + b + c = 0.

10. Sejam a, b e c números reais distintos e não nulos. Se a + b + c = 0, mostre


que
 a − b b − c c − a  c a b 
 + +  + +  = 9.
 c a b  a − b b − c c − a 
a −b b−c c−a
Solução: Façamos x = , y= e z= .
c a b
Assim, devemos provar que
1 1 1
( x + y + z )  + +  = 9,
x y z
ou seja,
x+ y+z x+ y+z x+ y+ z
+ + = 9,
x y z
ou ainda,
y+z x+z x+ y y+z x+z x+ y
1+ +1+ +1+ =9 ⇒ + + = 6.
x y z x y z
Mas,
x + y  a − b b − c  b  a 2 − ab + bc − c 2 b
= +  = ⋅
z  c a  c − a  ac c−a
( a 2 − c 2 ) − b( a − c ) b (a − c)(a + c) − b( a − c) b
= ⋅ = ⋅
ac c−a ac c−a

EUREKA! N°27, 2008

35
Sociedade Brasileira de Matemática

(a − c)(a + c − b) b (−b − b)b 2b 2


= ⋅ =− = .
ac c−a ac ac
y + z 2c 2 x + z 2a 2
Analogamente, concluímos que = e = . Logo, pelo exercício
x ab y bc
4, segue que
y + z x + z x + y 2a 2 2b 2 2c 2  a 3 + b3 + c 3   3abc 
+ + = + + = 2  = 2⋅  = 6,
x y z bc ac ab  abc   abc 
como queríamos provar.

Exercícios Propostos

1. Fatore a expressão S = x 4 + x 2 + 1.
2. Determine a expressão que deve ser multiplicada por x 3 2 + 2 3 x para
obtermos 2 x( x 2 + 4).
3. Calcule o valor da expressão
2 2
 ( x + 1) 2 ( x 2 − x + 1) 2   ( x + 1) 2 ( x 2 + x + 1) 2 
S =  ⋅  .
 ( x 3 − 1) 2   ( x 3 + 1) 2 
 x  y
4. Se x 2 + y 2 = 3 xy, calcule  1 +  1 +  .
 y  x 
5. Simplifique
(x + y 2 + z 2 + xy + yz + xz ) − ( x + y + z ) ( x 2 + y 2 + z 2 ) .
2 2 2

6. Fatore as seguintes expressões:


(a) x 3 + 5 x 2 + 3 x − 9;
(b) ( x − y ) z 3 − ( x − z ) y 3 + ( y − z ) x 3 ;
(c) ( x 2 + x + 3)( x 2 + x + 4) − 12;
(d) x 4 + 4 y 4 ;
(e) ( x − y )3 + ( y − z )3 + ( z − x)3 ;
(f) ( x + y + z )3 − x3 − y 3 − z 3 ;
(g) (a + 2b − 3c)3 + (b + 2c − 3a )3 + (c + 2a − 3b)3 .

7. Simplifique as expressões:
1 1 1 1 1
(a) − − − − ;
1 − x 1 + x 1 − x 1 − x 1 − x8
2 4

EUREKA! N°27, 2008

36
Sociedade Brasileira de Matemática

1 1 1
(b) + + ;
( x − y )( x − z ) ( y − x)( y − z ) ( z − x)( z − y )
( x 2 − y 2 )3 + ( y 2 − z 2 )3 + ( z 2 − x 2 ) 3
(c) .
( x − y )3 + ( y − z )3 + ( z − x )3
x y z
8. Prove que se + + = 0, então
y−z z−x x− y
x y z
+ + = 0.
( y − z ) 2 ( z − x)2 ( x − y )2
9. Para que os valores de a ∈ a expressão a 4 + 4 é um número primo?
10. Prove que se a + b + c = 0 então
a 5 + b5 + c 5 a 3 + b3 + c3 a 2 + b 2 + c 2
= ⋅ .
5 3 2
11. Mostre que (a + b)7 − a 7 − b7 = 7 ab( a + b)( a 2 + ab + b 2 ) 2 .
12. Prove que se a + b + c = 0, então
a 7 + b 7 + c 7 a 5 + b5 + c 5 a 2 + b 2 + c 2
= ⋅ .
7 5 2
13. Se a, b e c são reais não nulos que satisfazem a + b + c = 0, calcule
(a 3 + b3 + c3 ) 2 (a 4 + b 4 + c 4 )
.
(a 5 + b5 + c5 ) 2
14. Prove que se x, y e z são racionais distintos então a expressão
1 1 1
+ +
( y − z) 2
( z − x) 2
( x − y )2
é um quadrado perfeito.
15. Fatore 8( x + y + z )3 − ( x + y )3 − ( y + z )3 − ( x + z )3 .

EUREKA! N°27, 2008

37
Sociedade Brasileira de Matemática

OLIMPÍADAS AO REDOR DO MUNDO


Þ Apresentamos, como sempre, questões que não são encontradas
facilmente na Internet. Divirtam-se e enviem as suas soluções.
Continuamos à disposição na OBM para aqueles que estiverem
interessados na solução de algum problema particular. Para tanto, basta contactar
a OBM, através de carta ou e-mail.
Bruno Holanda
Carlos Augusto David Ribeiro

ÞÞÞ

Primeiramente vamos aos problemas propostos deste número

224.(Balcânica Junior - 2007) Seja a um real positivo tal que a = 6( a + 1). Prove
3

que a equação x 2 + ax + a 2 − 6 = 0 não possui solução real.

225.(Bulgária - 2007) Ache todos os inteiros positivos x, y tais que o número


( x 2 + y )( y 2 + x) é a quinta potência de um primo.

226.(Inglaterra - 2007) Seja ABC um triângulo acutãngulo com AB > AC e


∠BAC = 60°. Seja O o circuncentro e H o ortocentro. A reta OH encontra AB
em P e AC em Q. Prove que PO = HQ.

227.(Austria - 2007) Sejam 0 < x0 , x1 ,..., x669 < 1 reais distintos. Mostre que
existem i, j ∈ {0,1,..., 669} para os quais
1
0 < xi x j ( x j − xi ) < .
2007
228.(Bulgária - 2008) Para cada inteiro positivo n, seja τ (n) a quantidade de
divisores de n maiores que 2008. Defina an = 0 se τ (n) é par e an = 1 caso
contrário. O número α = 0, a1a2 ...an ... é racional?

229.(Bielorrússia - 2001) No losango ABCD, ∠A = 60°.


Os pontos F, H, e G estão sobre os segmentos AD, DC e AC de modo que DFGH
é um paralelogramo. Prove que FBH é um triângulo eqüilátero.

EUREKA! N°27, 2008

38
Sociedade Brasileira de Matemática

230.(Rússia - 2007) Sejam a, b, c números reais. Prove que pelo menos uma das três
equações
x 2 + (a − b) x + (b − c) = 0,
x 2 + (b − c) x + (c − a) = 0,
x 2 + (c − a) x + (a − b) = 0,
possui solução real.

231.(Bulgária - 2007) No triângulo∆ABC , com ∠ACB = 60°, sejam AA1 e BB1


( A1 ∈ BC , B1 ∈ AC ) as bissetrizes de ∠BAC e ∠ABC. A reta A1 B1 encontra o
circuncírculo do triângulo ∆ABC nos pontos A2 e B2 .
(a) Sejam O e I o circuncentro e o incentro do ∆ABC , respectivamente,
prove que OI é paralelo a A1 B1 .
(b) Se R é o ponto médio do arco AB, não contendo o ponto C, P e Q são
os pontos médios de A1 B1 e A2 B2 , respectivamente, prove que RP = RQ.

232.(Romênia - 2007) Encontre todos os conjuntos A de pelo menos dois inteiros


positivos, tais que para quaisquer dois elementos distintos x, y ∈ A tenhamos
x+ y
∈ A. Aqui mdc(x, y) denota o máximo divisor comum de x e y.
mdc( x, y )

233.(Romênia - 2007) Determine todas as progressões aritméticas infinitas de


inteiros positivos, com a seguinte propriedade: existe N ∈ , tal que para
qualquer primo p, p > N, o p-ésimo termo da seqüência também é primo.

234.(Bulgária - 2007) O incírculo do triângulo acutângulo ∆ABC toca os lados AB,


BC e CA nos pontos P, Q e R respectivamente. O ortocentro H do triângulo
∆ABC está sobre o segmento QR.
(a) Prove que PH ⊥ QR.
(b) Sejam I e O o incentro e o circuncentro do ∆ABC , respectivamente,
e N o ponto comum entre o lado AB e ex-incírculo relativo a este lado.
Prove que os pontos I, O e N são colineares.

235.(Olimpíada Checa e Eslovaca – 2007) Se x, y, z são números reais no intervalo (–1,


1) satisfazendo xy + yz + zx = 1, mostrre que:
6 3 (1 − x 2 )(1 − y 2 )(1 − z 2 ) ≤ 1 + ( x + y + z ) 2 .

EUREKA! N°27, 2008

39
Sociedade Brasileira de Matemática

236.(Romênia – 2007) Um conjunto de pontos no plano é livre se não existe


triângulo eqüilátero cujos vértices estão entre os pontos do conjunto. Mostre que
qualquer conjunto de n pontos no plano contém um subconjunto livre com pelo
menos n pontos.

SOLUÇÃO DE PROBLEMAS PROPOSTOS NOS NÚMEROS ANTERIORES:

211. (Baltic Way – 2004) Uma seqüência a1, a2, ... de números reais não-negativos
satisfaz, para n = 1, 2, ..., as seguintes condições:

(a) an + a2n ≥ 3n.


(b) a n +1 + n ≤ 2 a n (n + 1) .
(i) Prove que an ≥ n para todo n = 1, 2, ...
(ii) Dê exemplo de uma tal seqüência.

SOLUÇÃO DE ESTILLAC LINS MACIEL BORGES FILHO (BELÉM – PA)


(i) Utilizando a desigualdade das médias, temos:
an + n + 1
a n +1 + n ≤ 2 a n (n + 1) ≤ 2 ⇒ a n +1 + n ≤ a n + n + 1 ⇒ a n +1 ≤ a n + 1
2
Supondo válido que a n + k ≤ a n + k , temos que:
a n + k +1 ≤ a n + k + 1 ≤ a n + k + 1
E como a desigualdade vale para k = 1 , fica provado por indução que
a n + k ≤ a n + k , ∀k ∈ .
Em particular, para k = n , temos:
a 2 n ≤ a n + n ⇒ a n + a 2 n ≤ 2a n + n
Finalmente, como a n + a 2 n ≥ 3n , temos:
3n ≤ a n + a 2 n ≤ 2a n + n ⇒ 2a n ≥ 2n ⇒ a n ≥ n
(ii) Uma seqüência que satisfaz as condições é a n = n + 1 . De fato, temos:
(1) a n + a 2 n = n + 1 + 2n + 1 = 3n + 2 ≥ 3n
(2) an+1 + n = 2n + 2 = 2(n + 1) = 2 (n + 1)(n + 1) = 2 an (n + 1) ≤ 2 an (n + 1)

212. (Baltic Way – 2004) Seja P um polinômio com coeficientes não-negativos. Prove
que se P(1/x)P(x) ≥ 1 para x = 1, então tal desigualdade se verifica para todo real
positivo x.

EUREKA! N°27, 2008

40
Sociedade Brasileira de Matemática

SOLUÇÃO DE GELLY WHESLEY (FORTALEZA – CE)


Para x > 0 temos P ( x) > 0.
Da condição dada, temos ( P (1)) 2 ≥ 1.
Agora, denote P ( x) = a0 x n + a1 x n −1 + ... + an .
Então:
1  1 n
1
n −1

P ( x) P   = ( a0 x n + a1 x n −1 + ... + an ) ⋅  a0   + a1   + ... + an 
 x  x  x 
 
≥ (Por Cauchy-Schwarz)
2
 a0 a1 
 ⋅ a0 x n + ⋅ a1 x n −1 + ... + an ⋅ an  =
 xn xn −1 
(a0 + a1 + ... + an ) = ( P(1)) ≥ 1.
2 2

213. (Baltic Way – 2004) Ache todos os conjuntos X, consistindo de ao menos dois
inteiros positivos, tais que para todos m, n ∈ X, com n > m, exista um elemento k
de X tal que n = mk2.

SOLUÇÃO DE ESTILLAC LINS MACIEL BORGES FILHO (BELÉM – PA)


Suponha que o número 1∈ X . Logo, seja p ∈ X , com p > 1 , temos que deve
existir k ∈ X , tal que p = k 2 , isto é, p é quadrado perfeito. Obviamente,
k > 1 e com raciocínio análogo, concluímos que k também deve ser quadrado
perfeito e sua raiz quadrada deve pertencer a X . Ou seja, estendendo o
1
raciocínio, todas as potências de p da forma p 2n
, com n ∈ também devem
1
pertencer a X , o que é impossível, dado que p 2n ∉ para algum valor de n .
Logo, 1 ∉ X .
Vamos então tentar montar o conjunto X . Para tal, vamos supor, inicialmente
que X possua somente dois elementos p 1 e p 2 , com p 2 > p1 > 1 . Desta
forma, temos que deve existir k ∈ X , tal que p 2 = p1 k 2 . Obviamente, k < p 2
3
e, portanto, a única alternativa é k = p1 e, portanto, p 2 = p1 . De fato, todo
{ }
conjunto X = n, n 3 , com n ∈ N e n > 1 , satisfaz as condições do problema e
somente tais conjuntos de dois elementos satisfazem, conforme verificado.
Vamos então supor que o conjunto X tenha mais do que dois elementos, isto é,
X = {p1 , p 2 , p3 , p 4 ,..., p r } , com p1 ,..., p r ∈ N . Suponha, sem perda de

EUREKA! N°27, 2008

41
Sociedade Brasileira de Matemática

generalidade, que p r > p r −1 > ... > p 3 > p 2 > p1 > 1 . Com um raciocínio
3
análogo ao parágrafo anterior, concluímos que p 2 = p1 . Seguindo o raciocínio,
temos que:
2
(1) p3 = p1 k1
3 2
(2) p3 = p1 k 2
onde k1 , k 2 ∈ X .
Obviamente, temos que k1 < p 3 , k 2 < p 3 e k1 ≠ k 2 . Portanto, como p3 é o
k1 = p1
terceiro menor elemento do conjunto, só restam as possibilidades  3
ou
k 2 = p1
k1 = p13 3
 . Porém, no primeiro caso, temos que p3 = p1 = p 2 : absurdo! Já no
k 2 = p1
segundo caso, temos que:
3 7
(1) p3 = p1 ( p1 ) 2 = p1
3 2 5
(2) p3 = p1 p1 = p1
5 7
Portanto, temos que p1 = p1 ⇒ p1 = 0 ou p1 = 1 . Absurdo!
Logo, não é possível que o conjunto X possua mais de 2 elementos. E assim,
todos os conjuntos que satisfazem o enunciado são:
{ }
X = n, n 3 , com n ∈ e n > 1 .

223. (Bielorússia – 2005) Seja H o ponto de interseção das alturas BB1 e CC1 do
triângulo acutângulo ABC. Seja uma reta passando por A, tal que ⊥ AC.
Prove que as retas BC, B1C1 e possuem um ponto em comum se e somente se
H for o ponto médio de BB1.

EUREKA! N°27, 2008

42
Sociedade Brasileira de Matemática

SOLUÇÃO DE DAVI LOPES ALVES DE MEDEIROS (FORTALEZA – CE)


A

B1
C1
M

B
P

Seja P o ponto de interseção de BC e . É suficiente mostrarmos que B1 , C1 , P


são colineares ⇔ BM = MB1.

CB1 AC1 BM
i) C , M , C1 colineares ⇒ por Menelaus no ∆BAB1 : ⋅ ⋅ = 1, (I)
AC C1 B MB1
BB1 ⊥ AC e
ii) ⊥ AC ⇒ // BB1 e daí ∆ACP ∼ ∆BCB1 , donde
PB AB1 PC CB
= (II) e = (III)
BC CB1 AC CB1
PC AB1 C1 B
iii) P, B1 e C1 são colineares ⇔ ⋅ ⋅ = 1 (IV)
PB CB1 AC1
Multiplicando (I) e (IV) membro a membro, temos que P, B1 e C1 são colineares
CB1 AC1 BM PC AB1 C1 B BM  PC AB1 
⇔ ⋅ ⋅ ⋅ ⋅ ⋅ =1⇔  ⋅  = 1 (V)
AC C1 B MB1 PB CB1 AC1 MB1  AC PB 
AB1 CB1
Mas de (II) : = e multiplicando este resultado por (III)
PB BC
PC AB1 CB CB1
⋅ = ⋅ = 1 (*)
AC PB CB1 CB

EUREKA! N°27, 2008

43
Sociedade Brasileira de Matemática

Substituindo (*) em (V), temos que P, B1 , C1 são colineares


BM
⇔ = 1 ⇔ BM = MB1. c.q.d.
MB1

224. (Bielorússia – 2005) Ache todas as funções f : N → N satisfazendo

f (m − n + f (n)) = f (m) + f (n) ,


para todos m, n ∈ N.

SOLUÇÃO DE ESTILLAC LINS MACIEL BORGES FILHO (BELÉM – PA)


Primeiramente, seja k ∈ . Fazendo m = n = k , temos:
f (k − k + f (k )) = f (k ) + f (k ) ⇒ f ( f (k )) = 2 f (k ) , ∀k ∈
Em seguida, façamos m = 0 e n = f (k ) , temos:
f (− f (k ) + f ( f (k ))) = f (0) + f ( f (k )) ⇒ f (− f (k ) + 2 f (k )) = f (0) + 2 f (k )
⇒ f ( f (k )) = f (0) + 2 f (k ) ⇒ f (0) = 0
Agora, seja x ∈ , tal que x = f (1) . Tomando m = k + 1 e n = 1 , temos:
f (k + x) = x + f (k + 1) , ∀k ∈
Seja t ∈ , com t ≥ 1 . Substituindo sucessivamente na equação anterior k por
(t − j ) x + ( j − 1) , com 1 ≤ j ≤ t , temos:
f (tx) = x + f ((t − 1) x + 1)
f ((t − 1) x + 1) = x + f ((t − 2) x + 2)
f ((t − 2) x + 2) = x + f ((t − 3) x + 3)

f ((t − j + 1) x + j − 1) = x + f ((t − j ) x + j )

f (2 x + t − 2) = x + f ( x + t − 1)
f ( x + t − 1) = x + f (t )

Somando as equações anteriores, temos:


f (tx) = tx + f (t ) , ∀t ∈
Com este resultado em mãos, vamos provar, por indução, que f (k ) = kx ,
∀k ∈ . De fato, temos:
(1) Se k = 0 , temos f (k ) = f (0) = 0 = kx
(2) Se k = 1 , temos f (k ) = f (1) = x = kx

EUREKA! N°27, 2008

44
Sociedade Brasileira de Matemática

(3) Se f (t ) = tx , para t ∈ , temos:


f (t − 1 + x) = x + f (t ) = x + tx = (t + 1) x ⇒ f ( f (t − i + x)) = f ((t + i) x) = (t + 1) x + f (t + 1)
⇒ 2 f (t − 1 + x) = (t + 1) x + f (t + 1)
⇒ (t + 1) x + f (t + 1) = 2(t + 1) x ⇒ f (t + 1) = (t + 1) x
Portanto, nos resta descobrir quais os valores possíveis para x = f (1) . Para isso,
vamos utilizar o último resultado encontrado na relação inicial proposta no
problema, para n ≠ 0 :
f (m − n + f (n)) = (m − n + nx) x = mx − nx + nx 2
f (m) + f (n) = mx + nx
x = 0
2 2  2
⇒ mx − nx + nx = mx + nx ⇒ nx = 2nx ⇒ x = 2 x ⇒ ou
x = 2

Logo, as funções possíveis são: f ( y ) = 0 e f ( y ) = 2 y . É fácil ver que ambas
satisfazem as condições e, portanto, são todas as funções procuradas.
227. (Bulgária – 2005) Ivo escreve todos os inteiros de 1 a 100 (inclusive) em cartas
e dá algumas delas para Iana. Sabe-se que para quaisquer duas destas cartas, uma
de Ivo e outra de Iana, a soma dos números não está com Ivo e o produto não está
com Iana. Determine o número de cartas de Iana sabendo que a carta 13 está com
Ivo.

SOLUÇÃO DE ESTILLAC LINS MACIEL BORGES FILHO (BELÉM – PA)


Primeiramente, notamos que a carta de número 1 deve estar com Iana. De fato, se
a carta 1 estivesse com Ivo, para qualquer carta y de Iana, o produto dos
números também estaria com Iana, contradizendo a hipótese. Isso implica que ou
a carta 1 pertence a Iana ou Iana não possui cartas, o que não é verdade por
hipótese. Sendo assim, Iana possui a carta 1 e, dada qualquer carta x de Ivo,
temos que Iana deverá possuir a carta x + 1 .
Continuando o raciocínio, temos que Iana também deverá possui as cartas
2 x + 1 , 3 x + 1 , ..., kx + 1 , com kx + 1 ≤ 100 e k ∈ .
Além disso, Iana também deve possuir a carta x − 1 , pois caso contrário, Iana
deveria possui a carta x − 1 + 1 = x , o que não é verdade. E logo, Iana também
deve possuir as cartas 2 x − 1 , ..., kx − 1 , com kx − 1 ≤ 100 e k ∈ .
Assim, sabendo que Ivo possui a carta de número 13 , já sabemos que Iana possui
as cartas:
1, 12, 14, 25, 27, 38, 40, 51, 53, 64, 66, 77, 79, 90, 92

EUREKA! N°27, 2008

45
Sociedade Brasileira de Matemática

Notemos agora que a carta número 2 deve pertencer à Iana. De fato, se a carta
pertencesse a Ivo, teríamos pelo raciocínio anterior que Iana deveria possuir todas
as cartas ímpares, o que não é possível, já que Ivo possui a carta número 13 .
Como a carta 2 pertence a Iana e a carta 13 pertence a Ivo, temos que a carta
26 deve pertencer a Ivo e, conseqüentemente, a carta 52 também. Portanto, até
o momento, temos:
Ivo: 13, 26, 52
Iana: 1, 2, 12, 14, 25, 27, 38, 40, 51, 53, 64, 66, 77, 79, 90, 92
Novamente, temos que a carta 3 deve pertencer a Iana, pois caso pertencesse a
Ivo, a carta 6 = 3 × 2 também pertenceria a Ivo, assim como a carta 12 = 6 × 2 , o
que não é verdade. Analogamente, a carta 4 também pertence a Iana, pois se
pertencesse a Ivo, também pertenceria a Ivo a carta 12 = 4 × 3 . Isso implica que as
cartas 39 e 78 pertencem a Ivo. Portanto, até o momento:
Ivo: 13, 26, 39, 52, 78
Iana: 1, 2, 3, 4, 12, 14, 25, 27, 38, 40, 51, 53, 64, 66, 77, 79, 90, 92
Se Ivo possuísse o número 5 , também deveria possuir o número 10 = 5 × 2 e
assim, deveria possuir o número 40 = 10 × 4 , o que não é verdade. Logo Iana
possui o número 5.
Já os números 6 e 7 também não podem estar com Ivo, pois neste caso, os
números 12 = 6 × 2 e 14 = 7 × 2 não poderiam estar com Iana, o que não acontece.
Logo, Iana também possui os números 6 e 7 .
Assim, até o momento:
Ivo: 13, 26, 39, 52, 78
Iana: 1, 2, 3, 4, 5, 6, 7, 12, 14, 25, 27, 38, 40, 51, 53, 64, 66, 77, 79, 90, 92
Analogamente, temos que se 8, 9, 10 e 11 pertencessem a Ivo, 40 = 8 × 5 ,
27 = 9 × 3 , 40 = 10 × 4 e 77 = 11 × 7 também deveriam pertencer a Ivo, o que não
é verdade. Logo, 8, 9, 10 e 11 também pertencem a Iana.
Neste momento, temos que:
Ivo: 13, 26, 39, 52, 78
Iana: 1, 2, 3, 4, 5, 6, 7, 8, 9, 10, 11, 12, 14, 25, 27, 38, 40, 51, 53, 64, 66, 77, 79, 90, 92
Agora notamos que Ivo possui todos os múltiplos de 13 menores que 100 .
Todos os demais números menores que 100 devem então ter a forma 13k + t ,
com k , t ∈ e 0 < t < 13 . Como Iana possui todos os números t ∈ com
0 < t < 13 , temos que todos os número menores que 100 que não são múltiplos
de 13 devem pertencer a Iana, pois 13k pertence a Ivo e a soma não pode
pertencer a Ivo.

EUREKA! N°27, 2008

46
Sociedade Brasileira de Matemática

Logo, Ivo possui apenas 7 números e Iana possui os 93 restantes.

230. (Eslovênia – 2005) Denote por I o incentro do triângulo ABC. Sabe-se que AC +
AI = BC. Encontre a razão entre as medidas do ângulos ∠BAC e ∠CBA.

SOLUÇÃO DE RAFAEL ALVES DA PONTE (FORTALEZA – CE)


Denote por I o incentro do triângulo ABC. Sabe-se que AC + AI = BC. Encontre a
razão entre as medidas dos ângulos ∠BAC e ∠CBA.

A
α α

I
β θ
θ
B C

Construa AI ´ em AC de modo que AI = AI ´ , conforme a figura acima.


Sejam ∠BAI ´= ∠IAC = α e ∠IBA = ∠IBC = θ . Note que ∆BI ´C é isósceles,
e sendo ∠I ´BA = β , ∠BI ´ A mede β + 2θ e, pelo teorema do ângulo externo,
∠BAC = 2β + 2θ , donde vem α = β + θ [*]. Veja que ∠I ´ AI = 180° − α , e
visto que ∠I ´BI = α (por [*]), I ´BIA é inscritível. Como as cordas AI e AI ´
são congruentes, β = θ , daí
∠BAC
α = 2θ ⇔ 2α = 2 ⋅ 2θ ⇔ ∠BAC = 2∠CBA ⇔ =2
∠CBA

EUREKA! N°27, 2008

47
Sociedade Brasileira de Matemática

COMO É QUE FAZ?

PROBLEMA PROPOSTO POR WILSON CARLOS DA SILVA RAMOS (BELÉM – PA)


Dado um triângulo ABC com incentro I, considere uma reta variável l passando
por I que intersecta o lado AB em P, o lado AC em N e a reta suporte do lado BC
AB AC BC
em M. Prove que o valor de + − independe da escolha de l.
PA⋅ PB NA⋅ NC MB ⋅ MC
SOLUÇÃO:
Suponha, sem perda de generalidade, que B está entre M e C.
A

α/2 α/2

θ N

ε
I
φ
P
β/2 γ/2
β/2 γ/2
M
B C
AB PA + PB 1 1
Primeiro, note que = = + ,
PA ⋅ PB PA ⋅ PB PA PB
AC NA + NC 1 1 BC MC − MB 1 1
= = + e = = − , de
NA ⋅ NC NA ⋅ NC NA NC MB ⋅ MC MB ⋅ MC MB MC
modo que queremos provar que
1 1 1 1 1 1
+ + + − +
PA PB NA NC MB MC
não depende da reta l.
Isso é um trabalho para a lei dos senos! De fato, nos triângulos AIP e AIN, e
r
lembrando que, sendo r o inraio de ABC, AI = ,
sen α2

EUREKA! N°27, 2008

48
Sociedade Brasileira de Matemática

AP AI 1 sen α2 sen (θ + α2 )
= ⇔ = e
sen θ sen (θ + α2 ) PA r sen θ
NA AI 1 sen α2 sen (θ − α2 )
= ⇔ =
sen θ sen (θ − α2 ) NA r sen θ
1 1
Somando e , obtemos
PA NA
1 1 sen α2 (sen (θ + α2 ) + sen (θ − α2 ))
+ =
PA NA r sen θ
Utilizando a fórmula de Prostaférese sen x + sen y = 2 sen x +2 y cos ( ) ( ),
x− y
2

  (θ + α2 ) + (θ − α2 )   (θ + α2 ) − (θ − α2 ) 
senα2  2sen  cos
 

 senα (2senθ cosα )
 2 2
1 1      senα
+ = = 2 2
=
PA NA r senθ r senθ r
Sorte grande! Esse valor não depende da escolha de l, já que r e α só dependem
do triângulo ABC. Podemos concluir, analogamente, que
1 1 sen γ
+ =
NC MC r
também não depende de l.
1 1
Já o caso de − , como era de se esperar, é um pouquinho diferente, mas
PB MB
só um pouquinho: pela lei dos senos nos triângulos MIB e PIB,
MB
=
BI

1
=
(
sen β2 sen β2 − φ )
(
sen φ sen β2 − φ ) MB r sen φ
e

PB
=
BI

1
=
sen β2 sen β2 + φ ( )
sen φ sen β2 + φ ( PB )r sen φ
α β
Note a mudança de sinal de θ − para − φ : “destrocando” o sinal, obtemos
2 2


1
=
(
sen β2 sen φ − β
2
) e aí podemos trabalhar como nos demais casos, obtendo
MB r sen φ
1 1 sen β
+ − =
MB PB r
sen α + sen β + sen γ
A soma pedida é, então, igual a , que não depende de l.
r

EUREKA! N°27, 2008

49
Sociedade Brasileira de Matemática

SOLUÇÕES DE PROBLEMAS PROPOSTOS


Publicamos aqui algumas das respostas enviadas por nossos leitores.

110) Um conjunto finito de inteiros positivos é chamado de Conjunto DS se cada


elemento divide a soma dos elementos do conjunto.
Prove que todo conjunto finito de inteiros positivos é subconjunto de algum
conjunto DS.

SOLUÇÃO DE ZOROASTRO AZAMBUJA NETO (RIO DE JANEIRO – RJ)


Basta provar que, para todo n inteiro positivo, existe um conjunto DS que contém
{1,2,...,n}. Para n ≤ 3 , isso segue do fato de {1,2,3} ser um conjunto DS.
Vamos mostrar, por indução em n, que, para todo n ≥ 3 , existe um conjunto DS,
X n = {a1 , a2 ,..., ak ( n ) }, com a j = j para 1 ≤ j ≤ n. Note que a soma dos seus
k (n)
elementos S n = ∑a
j =1
j é par (pois a2 = 2 ∈ X n ⇒ 2 | S n ).

Dado n≥3 e um conjunto Xn como acima, podemos tomar


 (n + 1)(n + 2) (n + 1)(n + 2) (n + 1)(n + 2) 
X n +1 = 1, 2,3,..., n, n + 1, a2 , a3 ,..., ak ( n ) 
 2 2 2 
A soma de seus elementos é
(n +1)(n + 2) k (n) (n +1)(n + 2) (n +1)(n + 2)
Sn+1 = 1+ 2 + ... + n + (n +1) +
2
∑j =2
aj =
2
+
2
(Sn − a1) =

(n +1)(n + 2)
= ⋅ Sn , pois a1 = 1.
2
Como S n é par, S n +1 é múltiplo de n + 1. Como S n é múltiplo de a j para todo j,
(n + 1)(n + 2) (n + 1)(n + 2)
S n +1 = ⋅ S n é múltiplo de ⋅ a j para todo j, e logo
2 2
X n +1 é um conjunto DS.

111) Prove que existem infinitos múltiplos de 7 na seqüência (an ) abaixo:


a1 = 1999, an = an −1 + p(n), ∀n ≥ 2 , onde p(n) é o menor primo que divide n.

EUREKA! N°27, 2008

50
Sociedade Brasileira de Matemática

SOLUÇÃO DE JOSÉ DE ALMEIDA PANTERA (RIO DE JANEIRO – RJ)


Seja N um número da forma 510510r = 2 ⋅ 3 ⋅ 5 ⋅ 7 ⋅11 ⋅13 ⋅17 ⋅ r , onde r é um
inteiro positivo. Temos então p ( N + 2) = 2, p ( N + 3) = 3, p ( N + 4) = 2,
p( N + 5) = 5, p( N + 6) = 2, p( N + 7) = 7, p( N + 8) = 2, p( N + 9) = 3,
p( N + 10) = 2, p( N + 11) = 11, p( N + 12) = 2, p( N + 13) = 13,
p( N + 14) = 2, p( N + 15) = 3, p( N + 16) = 2, e p( N + 17) = 17. Assim,
aN + 2 ≡ aN +1 + 2(mod 7), aN +3 ≡ aN +1 + 5(mod 7), aN + 4 ≡ aN +1 (mod 7),
aN +5 ≡ aN +1 + 5(mod 7), aN + 6 ≡ aN +1 (mod 7), aN + 7 ≡ aN +1 (mod 7),
aN +8 ≡ aN +1 + 2(mod 7), aN +9 ≡ aN +1 + 5(mod 7), aN +10 ≡ aN +1 (mod 7),
aN +11 ≡ aN +1 + 4(mod 7), aN +12 ≡ aN +1 + 6(mod 7), aN +13 ≡ aN +1 + 5(mod 7),
aN +14 ≡ aN +1 (mod 7), aN +15 ≡ aN +1 + 3(mod 7), aN +16 ≡ aN +1 + 5(mod 7) e
aN +17 ≡ aN +1 + 1(mod 7), e portanto sempre há um múltiplo de 7 em
{aN +1 , aN + 2 , aN +3 , aN +11 , aN +12 , aN +15 , aN +17 }, pois aN +1 , aN + 2 , aN +3 , aN +11 , aN +12 ,
aN +15 , aN +17 percorrem todas as classes de congruência módulo 7.
112) Determine todos os inteiros positivos n tais que existe uma matriz n × n com
todas as entradas pertencentes a { –1, 0, 1} tal que os 2n números obtidos como
somas dos elementos de suas linhas e de suas colunas são todos distintos.

SOLUÇÃO DE ASDRÚBAL PAFÚNCIO SANTOS (BOTUCATU – SP)


Vamos mostrar que existe uma matriz como no enunciado se e somente se n é
par. Se n é par, digamos n = 2k, podemos construir uma matriz A = ( aij )1≤i , j ≤ 2 k
com aij = 1 se 1 ≤ i, j ≤ k , aij = −1 se k + 1 ≤ i, j ≤ 2k , aij = 1 se i ≤ k e
j ≥ k + i, aij = 0 se i ≤ k e k + 1 ≤ j < k + i, aij = −1 se i ≥ k + 1, j ≤ k e
j + k > i, e aij = 0 se i ≥ k + 1 e j + k ≤ i. É fácil ver que os 2n números obtidos
como somas dos elementos das linhas e das colunas de A é
{n, n − 1, n − 2,...,1, 0, −1,..., −(n − 2), −(n − 1)}.
Suponha agora que exista uma matriz A = (aij )1≤i , j ≤ n como no enunciado, com
aij ∈ {−1, 0,1}, ∀i, j ≤ n. Claramente permutar linhas ou colunas não altera as
propriedades do enunciado. Como há 2n + 1 elementos em
{− n, −(n − 1),..., −1, 0,1,..., n − 1, n}, um desses elementos, digamos c, não é
valor da soma dos elementos de nenhuma linha ou coluna de A. Podemos supor
(trocando o sinal de todos os elementos de A, se necessário) que esse elemento c

EUREKA! N°27, 2008

51
Sociedade Brasileira de Matemática

que falta é menor ou igual a 0. Podemos supor (permutando linhas e colunas, se


necessário) que os valores 1, 2,..., n – 1, n são obtidos como somas dos elementos
das primeiras k linhas e das primeiras n – k colunas de A, para certo k ≤ n. Sejam
x= ∑ aij , y =
1≤i ≤ k

aij ,
1≤i ≤ k
z= ∑
aij e w =
k <i ≤ n
aij . Temos então ∑
k <i ≤ n
1≤ j ≤ n − k n−k < j ≤n 1≤ j ≤ n − k n−k < j ≤n

n(n + 1) n(n + 1)
= 1 + 2 + ... + n = ( x + y ) + ( x + z ) = 2 x + y + z , e − −c =
2 2
= ( z + w) + ( y + w) = 2w + y + z. Portanto, 2 x − 2 w = n(n + 1) + c ≥ n 2 , pois
c ≥ − n, e, como claramente temos x ≤ k (n − k ) e w ≥ −k (n − k ), 4k (n − k ) ≥
≥ 2 x − 2 w ≥ n 2 ⇒ 0 ≥ n 2 − 4kn + 4k 2 = (n − 2k ) 2 ,
donde n − 2k = 0, e
portanto n é par. Note que, nesse caso, o elemento c que falta deve ser
necessariamente igual a –n (ou n, se for positivo).

114) Sabendo que sen x + sen y + sen z + sen w = 0 e


cos x + cos y + cos z + cos w = 0, mostre que
sen 2003 x + sen 2003 y + sen 2003 z + sen 2003 w = 0.

SOLUÇÃO BASEADA NAS SOLUÇÕES ENVIADAS POR SAMUEL LILÓ ABDALLA E


DOUGLAS RIBEIRO SILVA
De cos x + cos y + cos z + cos w = 0 e sen x + sen y + sen z + sen w = 0,
obtemos
eix + eiy + eiz + eiw = (cos x + isenx) + (cos y + iseny) + (cos z + isenz) + (cos w+ isenw) = 0.
Vamos supor sem perda de generalidade 0 ≤ x ≤ y ≤ z ≤ w < 2π . Temos
y − x ≤ π e w − z ≤ π , senão os quatro números complexos eix , eiy , eiz e eiw
pertenceriam a um mesmo semicírculo do círculo unitário, e sua soma não
poderia ser 0. Temos eix + eiy = (cos x + cos y ) + i (senx + seny ) =
 x+ y 
 y − x   x+ y  x + y   y − x  i 
= 2 cos    sen   + i cos    = 2 cos  e
2 
.
 2   2   2   2 
 w+ z 
 w − z  i 
Analogamente, e + e = 2 cos 
iz iw
e
2 
.
 2 
Como eix + eiy = − ( eiz + eiw ) , obtemos

EUREKA! N°27, 2008

52
Sociedade Brasileira de Matemática

 x+ y   w+ z 
 y − x  i 2   w − z  i 2  y−x π
2 cos  e = −2 cos  e . Temos ainda 0 ≤ ≤ e
 2   2  2 2
w− z π  y−x  w− z 
0≤ ≤ , donde cos   ≥ 0 e cos   ≥ 0. Temos agora dois
2 2  2   2 
casos:
 y−x  w− z 
i) cos   = 0. Nesse caso, devemos ter também cos   = 0, e,
 2   2 
portanto y − x = w − z = π , seny = −senx e senw = −senz , donde segue
imediatamente o resultado.
 y−x  w− z   y−x
ii) cos   > 0. Nesse caso, devemos ter cos   = cos  >0e
 2   2   2 
 x+ y   w+ z 
i   w+ z x+ y
 w− z y − x
e  2 
= −e  2 
=
, donde +π e = . Somando,
2 2 2 2
obtemos w = y + π e, substraindo, obtemos z = x + π . Assim, senw = − seny e
senz = − senx , donde segue o resultado, como antes.

116) Seja ABC um triângulo e sejam X, Y e Z as reflexões de A, B e C em


relação às retas BC, CA e AB, respectivamente. Prove que x, y e z são colineares
se e somente se cos A ⋅ cos B ⋅ cos C = − 3 8.

SOLUÇÃO ENVIADA POR DOUGLAS RIBEIRO SILVA COM CONTRIBUIÇÕES DE CARLOS


EDDY ESAGUY NEHAB E MARCIO COHEN
Construam o triângulo ABC e as reflexões X, Y e Z de seus vértices A, B e C.
Se, dados U, V, W no plano, (UVW) denota a área (orientada) do triângulo UVW,
temos S(XYZ) = [S(ABC) + S(CBX) + S(ACY) + S(BAZ)] – S(AZY) – S(BXZ) –
S(CYX). (*) Ver nota abaixo.
Temos S(ABC) = S(CBX) = S(ACY) = S(BAZ), por construção.
As áreas de AZY, BXZ e CYX podem ser somadas ou subtraídas, dependendo de
os ângulos YAZˆ = 3 A, ZBX ˆ = 3B e XCY ˆ = 3C (onde A, B e C denotam
os ângulos internos do triângulo ABC) serem maiores ou menores que 180 graus.
Valerá a igualdade se usarmos as expressões S(AZY) = bc · sen(3A)/2, S(BXZ) =
ac · sen(3B)/3 e S(CYX) = ab · sen(3C)/2.
Então a relação passa a ser S(XYZ) = 4S(ABC) – bc · sen(3A)/2 – ac · sen(3B)/3 –
ab · sen(3C)/2.

EUREKA! N°27, 2008

53
Sociedade Brasileira de Matemática

Agora substituímos sen(3θ ) = – 4·(sen(θ )) 3 + 3 · sen(θ ) para θ = A, B, C e


substituímos também bc/2 ac/2 e ab/2 respectivamente por S(ABC)/senA,
S(ABC)/senB e S(ABC)/senC, devido à fórmula de área em função dos lados e do
ângulo para o triângulo original.
Fazendo as devidas substituições acima, simplificamos os senos e basta trocar
(sen(θ ))2 por 1 – (cos(θ ))2 para θ = A, B, C para chegar em S(XYZ) = S(ABC) ·
[7 – 4((cosA)2 + (cosB)2 + (cosC)2)].
Para que os três pontos estejam alinhados, a área do triângulo XYZ deve ser igual
a zero, donde 7 – 4((cosA)2 + (cosB)2 + (cosC)2) = 0
Façamos z = (cosA)2 + (cosB)2 + (cosC)2 Dai, como cos2x = 2(cosx)2 – 1 temos
z = (1 + cos2A)/2 + (1+cos2B)/2 + (cosC)2 = 1 + (cos(2A) +cos(2B))/2 +
(cosC)2.
Mas cos(2A) + cos(2B) = 2cos(A + B)cos(A – B) = –2cosC cos(A – B).
Substituindo em z:
z = 1 – cosC [cos(A – B) – cosC] = 1 – cosC [cos(A – B) + cos(A + B)]. Daí
temos:
z = 1 – cosC. [2cosA.cosB] = 1 – 2cosA.cosB.cosC.
Substituindo este z na expressão anterior, chegamos na desejada expressão do
enunciado:
0 = 7 – 4 ((cosA)2 + (cosB)2 + (cosC)2) = 7 – 4 (1 – 2cosA.cosB.cosC) =
3+8cosA.cosB.cosC.
Logo, chegamos na esperada relação cosA·cosB·cosC = –3/8.
Nota: A igualdade(*) pode ser mostrada por meio de algumas figuras,
considerando alguns casos, mas daremos a seguir uma prova algébrica dela. A
2
área (orientada) de um triângulo UVW no plano é igual à metade do
determinante det (V – U , W– U) da matriz cujas linhas coincidem com os vetores
V – U e W – U. Se identificarmos cada vetor ( x, y ) ∈ 2 com ( x, y, 0) ∈ 3 , o
produto vetorial (V – U) × (W– U) é igual ao vetor (0,0, det (V – U, W – U)) =
(0,0, 2 S (UVW)).
Basta provar então que (Y – X) × (Z – X) = (B – A) × (C – A) + (B – C) × (X – C) +
(C – A) × (Y – A) + (A – B) × (Z – B) – (Z – A) × (Y – A) – (X – B) × (Z – B) –
(Y – C) × (X – C), mas o lado direito é igual a
(B – A) × (C – A) + (B – Y) × (X – C) + × (C – Z) × (Y – A) + (A – X) × (Z – B),
que, desenvolvendo (e usando a desigualdade U × V = – V × U, para quaisquer U,
V), é igual a B × C – B × A – A × C + B × X – B × C – Y × X + Y × C + C × Y –
C×A–Z×Y+Z×A+A×Z–A×B–X×Z+X×B=–Y×X–Z×Y– X×Z
= Y × Z – Y × X – X × Z = (Y – X) × (Z – X).

EUREKA! N°27, 2008

54
Sociedade Brasileira de Matemática

117) Sejam r e s duas retas reversas (i.e., não contidas num mesmo plano) e A,
B, C, D, A, B, C , D pontos tais que A, B, A, B ∈ r , C , D, C , D ∈ s, AB = AB e
CD = C D. Prove que os tetraedros ABCD e ABC D têm o mesmo volume.

SOLUÇÃO DE DOUGLAS RIBEIRO SILVA (RECIFE - PE)


Note que o que o problema pede é equivalente demonstrar que o volume do
tetraedro só depende da medida de duas arestas reversas e da distância entre
as retas-suporte dessas duas arestas. Na figura que segue, as arestas
reversas são AB e CD. A distância entre as retas suporte é EF.

O volume do tetraedro será calculado a partir da área da base ABC e a altura


relativa a D

V = (Área (ABC) · HD)/3

Note que a área de ABC pode ser definida como AB · EC/2, pois como EC está no
mesmo plano de EF e EF é perpendicular a AB, EC também é (Teorema das 3
perpendiculares).

V = ((AB · EC/2) · HD)/3

Conservando a área do triangulo retangulo EFC temos: EF · FC/2 = EC · FG/2.


Logo
FG = EF · FC/EC
Pela semelhança dos triangulos FGC e DHC tiramos o valor de HD:

HD/CD = FG/FC Logo HD = FG · CD/FC = (EF · FC/EC) · (CD/FC)

Logo HD = EF · CD/EC

Finalizando, temos que V = (AB · EC/2) · (EF · CD/EC)/3


Logo V = AB · CD · EF/6
Assim, provamos que o volume de um tetraedro não depende da posição dos
segmentos AB e CD nas suas retas-suporte, mas sim, unicamente dos tamanhos
dos segmentos e da distância entre as retas-suporte dos mesmos.

EUREKA! N°27, 2008

55
Sociedade Brasileira de Matemática

C
A
G
H
E

2 + 9an
118) Considere a seqüência (an ) n ≥1 dada por a1 = 1 e an +1 = , ∀n ≥ 1.
3 + 9an
Prove que (an ) converge e calcule o seu limite.

SOLUÇÃO DE ESTILLAC LINS MACIEL BORGES FILHO (BELÉM – PA)


1+ 3
Primeiramente, iremos provar que a n > , para todo n ∈ , utilizando
3
indução. De fato:
1+ 3
(1) a1 = 1 >
3
1+ 3
(2) Supondo a n > , temos 1 + 3a n > 2 + 3 . Portanto:
3
2 + 9a n 1 1 1 1 1+ 3
a n +1 = = 1− ( ) > 1− ( )=
3 + 9a n 3 1 + 3a n 3 2+ 3 3
1+ 3
Em seguida, provaremos que, se a n > , temos que a n +1 < a n . De fato:
3
1+ 3
an > ⇒ 3a n − 1 > 3 ⇒ (3a n − 1) 2 > 3
3
EUREKA! N°27, 2008

56
Sociedade Brasileira de Matemática

Portanto,
2 + 9a n 3 − (3a n − 1) 2
a n +1 − a n = − an = <0
3 + 9a n 3 + 9a n
Logo, (a n ) é uma seqüência estritamente decrescente limitada inferiormente por
1+ 3
, o que garante sua convergência. Vamos mostrar agora que
3
1+ 3
lim a n = . Seja x = lim an = lim an +1. Temos então
3
2 + 9an 2 + 9 x
x = lim an +1 = lim = , donde 9 x 2 − 6 x − 2 = 0. Como x ≥ 0, devemos ter
3 + 9an 3 + 9 x
3 + 27 1 + 3
x= = .
9 3

Obs: a solução acima é bastante artificial. Ela é construída já se sabendo de


antemão qual o provável limite da seqüência. Este provável limite é obtido
facilmente fazendo a n +1 = a n = x e resolvendo a equação
2 + 9x 1+ 3
x= ⇒x= , pois x ≥ 0.
3 + 9x 3

Continuamos aguardando as soluções dos problemas a seguir:

113) a1 , a2 , a3 ,... formam uma seqüência de inteiros positivos menores que 2007
am + an
tais que é inteiro, para quaisquer inteiros positivos m, n.
am + n
Prove que a seqüência (an) é periódica a partir de um certo ponto.

115) Suponha que ABC é um triângulo com lados inteiros a, b e c com


BCA = 60° e mdc(a, b) = mdc(a, c) = mdc(b, c) = 1. Prove que
c ≡ 1(mod 6) .

EUREKA! N°27, 2008

57
Sociedade Brasileira de Matemática

PROBLEMAS PROPOSTOS
Convidamos o leitor a enviar soluções dos problemas propostos e sugestões de novos
problemas para próximos números.

119. Mostre que não existem inteiros positivos a e b tais que tais que
(36a + b)(36b + a) seja uma potência de 2.

120. Sejam a, b, c números reais e soma Sn definida como S n = a n + b n + c n , para


qualquer n inteiro não negativo, Sabe-se que S1 = 2, S2 = 6 e S3 = 14, mostre que
Sn2 − S n −1 ⋅ S n +1 = 8 para todo inteiro n > 1.

121. Na figura abaixo o lado do quadrado vale 4, obter o valor da altura h para que
a área da região 1 seja igual a área da região 2.

2
4 4
h

122. Dado um triângulo ABC tal que AB = AC = a + b e BC = a , traça-se uma


ceviana partindo de B determinando em AC um ponto D tal que DA = a e
DC = b . Sabendo que ABD = 10° , determine os ângulos internos desse triângulo.

123. Determine todas as funções f : *→ * tais que


2 2 3 2 2
2 f (m + n ) = f (m) f (n) + f (m) f (n) , para quaisquer m, n ∈ * distintos.
Obs. * = {1, 2,3,...} é o conjunto dos inteiros positivos.

EUREKA! N°27, 2008

58
Sociedade Brasileira de Matemática

124. Considere a seqüência ( an ) n ≥1 definida por a1 = a2 = a3 = a4 = 1 e


an −1an −3 + an2− 2
an = , ∀n ≥ 5.
an − 4
Prove que an é um inteiro positivo, para todo inteiro positivo n.

125. Considere dois naturais m≥2 e n ≥ 2, e as seqüências


( a0 , a1 , a2 ,..., amn ), ai ∈ {0,1}.
As seqüências de tipo m satisfazem as condições:
• ak ak + m = 0, para todo k;
• Se ak ak +1 = 1 então m divide k
As seqüências de tipo n são definidas analogamente. Prove que existem tantas
seqüências do tipo m quanto do tipo n.

126. As circunferências Γi , 0 ≤ i ≤ 5, são tangentes a uma circunferência Γ nos


pontos Ai . Além disso, Γi é tangente a Γi +1 para 0 ≤ i ≤ 5 e Γ5 é tangente a Γ 0 .
Prove que A0 A3 , A1 A4 , A2 A5 são concorrentes.

127. Determine todos os inteiros positivos k tais que existem inteiros positivos x,
x2 + y2 + z 2
y, z com = k.
xyz
128. Barango Joe era um sapo de mútiplos talentos que habitava a Terra das
Chances Diminutas, localizada no alto de uma montanha.
Após sua maioridade, Barango Joe decidiu tentar a vida no Reino das Grandes
Oportunidades, localizado no cume da montanha vizinha.
Para isso, ele atravessaria a extensa ponte de madeira por cima do Desfiladeiro da
Morte. Entretanto, a ponte era guardada pela Esfinge Vegas, exímia jogadora que
sempre desafiava os viajantes para algum jogo. O viajante vitorioso tinha a
passagem franqueada; e o perdedor era lançado ao abismo.
Assim chegando à cabeceira da ponte, Barango Joe foi desafiado a uma partida de
“Pachang” jogo que lembra o “Black Jack” ou “Vinte e um”, mas é jogado por 2
oponentes da seguinte maneira:
Os jogadores, designados por “banca” e “apostador”, utilizam um dado gerador
de números aleatórios reais uniformemente distribuídos no intervalo [0,1]
Inicialmente, a banca sorteia um número X. Se não estiver satisfeita com o
número obtido, pode descartá-lo e então sortear um novo número. Este
procedimento pode ser executado 2 vezes, Isto é, pode haver até 3 sorteios na
definição do número X da banca.

EUREKA! N°27, 2008

59
Sociedade Brasileira de Matemática

Então, o apostador sorteia quantos números forem necessários até que a soma de
seus números ultrapasse o número X da banca. Neste momento, se esta soma for
inferior a 1, o apostador ganha; caso contrário, perde.

Ou seja, para ganhar, o apostador precisa “chegar mais próximo” de 1 que a


banca, sem no entanto “estourar o limite” de 1.

Após explicar as regras do Pachang, a Esfinge Vegas deu uma opção ao sapo:
- Você prefere ser a banca ou o apostador?
O que o Barango Joe deveria responder?

Obs. Utilize lápis, papel, e uma calculadora científica simples.

129. Um coelho está numa rua infinita dividida em quadrados numerados pelos
inteiros, e começa no quadrado 0. Se num dado momento ele está no quadrado k,
1
ele escolhe, com probabilidade , pular para o quadrado k + 2 ou, também com
2
1
probabilidade , pular para o quadrado k – 1. Ele continua esse processo
2
indefinidamante. Dado m ∈ , determine a probabilidade de, em algum momento,
o coelho pisar no quadrado m.

Problema 119 proposto por Adriano Carneiro, problemas 120 e 121 proposto por Samuel
Liló Abdalla, de Sorocaba – SP, problema 122 proposto por Renan Lima Novais, do Rio de
Janeiro – RJ, problema 123 proposto por Wilson Carlos da Silva Ramos, de Belém – PA,
problemas 124, 125, 126 e 127 propostos por Anderson Torres, de São Paulo – SP, problema
128 proposto por Rogério Ponce da Silva, do Rio de Janeiro – RJ, problema 129 proposto
por Nicolau Corção Saldanha e Zoroastro Azambuja Neto, do Rio de Janeiro – RJ.

Agradecemos também o envio das soluções e a colaboração de:


Gelly Whesley Fortaleza – CE
Evandro A. dos Santos Campinas – SP
Davi Lopes Alves de Medeiros Fortaleza – CE
Rafael Alves da Ponte Fortaleza – CE
André Felipe M da Silva Rio de Janeiro – RJ
Carlos Alberto da Silva Victor Nilópolis – RJ

EUREKA! N°27, 2008

60
Sociedade Brasileira de Matemática

AGENDA OLÍMPICA
XXX OLIMPÍADA BRASILEIRA DE MATEMÁTICA

NÍVEIS 1, 2 e 3
Primeira Fase – Sábado, 14 de junho de 2008
Segunda Fase – Sábado, 13 de setembro de 2008
Terceira Fase – Sábado, 25 de outubro de 2007 (níveis 1, 2 e 3)
Domingo, 26 de outubro de 2008 (níveis 2 e 3 - segundo dia de prova).

NÍVEL UNIVERSITÁRIO
Primeira Fase – Sábado, 13 de setembro de 2008
Segunda Fase – Sábado, 25 e Domingo, 26 de outubro de 2008

XIV OLIMPÍADA DE MAIO


10 de maio de 2008

XIX OLIMPÍADA DE MATEMÁTICA DO CONE SUL
Temuco – Chile
18 a 23 de junho de 2008

XLIX OLIMPÍADA INTERNACIONAL DE MATEMÁTICA
10 a 22 de julho de 2008
Madri – Espanha

XIV OLIMPÍADA INTERNACIONAL DE MATEMÁTICA UNIVERSITÁRIA
25 a 31 de julho de 2008
Blagoevgrad, Bulgária

XXIII OLIMPÍADA IBEROAMERICANA DE MATEMÁTICA
18 a 28 de setembro de 2008
Salvador, Bahia – Brasil

XI OLIMPÍADA IBEROAMERICANA DE MATEMÁTICA UNIVERSITÁRIA

EUREKA! N°27, 2008

61
Sociedade Brasileira de Matemática

COORDENADORES REGIONAIS
Alberto Hassen Raad (UFJF) Juiz de Fora – MG
Américo López Gálvez (USP) Ribeirão Preto – SP
Amarísio da Silva Araújo (UFV) Viçosa – MG
Andreia Goldani FACOS Osório – RS
Antonio Carlos Nogueira (UFU) Uberlândia – MG
Ali Tahzibi (USP) São Carlos – SP
Benedito Tadeu Vasconcelos Freire (UFRN) Natal – RN
Carlos Alexandre Ribeiro Martins (Univ. Tec. Fed. de Paraná) Pato Branco - PR
Carmen Vieira Mathias (UNIFRA) Santa María – RS
Claus Haetinger (UNIVATES) Lajeado – RS
Cleonor Crescêncio das Neves (Inst. de Tec. e Educ. Galileo da Amazônia) Manaus – AM
Cláudio de Lima Vidal (UNESP) S.J. do Rio Preto – SP
Denice Fontana Nisxota Menegais (UNIPAMPA) Bagé – RS
Edson Roberto Abe (Colégio Objetivo de Campinas) Campinas – SP
Élio Mega (Faculdade Etapa) São Paulo – SP
Eudes Antonio da Costa (Univ. Federal do Tocantins) Arraias – TO
Fábio Brochero Martínez (UFMG) Belo Horizonte – MG
Florêncio Ferreira Guimarães Filho (UFES) Vitória – ES
Francinildo Nobre Ferreira (UFSJ) São João del Rei – MG
Genildo Alves Marinho (Centro Educacional Leonardo Da Vinci) Taguatingua – DF
Ivanilde Fernandes Saad (UC. Dom Bosco) Campo Grande– MS
Jacqueline Rojas Arancibia (UFPB)) João Pessoa – PB
Janice T. Reichert (UNOCHAPECÓ) Chapecó – SC
João Benício de Melo Neto (UFPI) Teresina – PI
João Francisco Melo Libonati (Grupo Educacional Ideal) Belém – PA
José Luiz Rosas Pinho (UFSC) Florianópolis – SC
José Vieira Alves (UFPB) Campina Grande – PB
José William Costa (Instituto Pueri Domus) Santo André – SP
Krerley Oliveira (UFAL) Maceió – AL
Licio Hernandes Bezerra (UFSC) Florianópolis – SC
Luciano G. Monteiro de Castro (Sistema Elite de Ensino) Rio de Janeiro – RJ
Luzinalva Miranda de Amorim (UFBA) Salvador – BA
Mário Rocha Retamoso (UFRG) Rio Grande – RS
Marcelo Rufino de Oliveira (Grupo Educacional Ideal) Belém – PA
Marcelo Mendes (Colégio Farias Brito, Pré-vestibular) Fortaleza – CE
Newman Simões (Cursinho CLQ Objetivo) Piracicaba – SP
Nivaldo Costa Muniz (UFMA) São Luis – MA
Osvaldo Germano do Rocio (U. Estadual de Maringá) Maringá – PR
Raul Cintra de Negreiros Ribeiro (Colégio Anglo) Atibaia – SP
Ronaldo Alves Garcia (UFGO) Goiânia – GO
Rogério da Silva Ignácio (Col. Aplic. da UFPE) Recife – PE
Reginaldo de Lima Pereira (Escola Técnica Federal de Roraima) Boa Vista – RR
Reinaldo Gen Ichiro Arakaki (UNIFESP) SJ dos Campos – SP
Ricardo Amorim (Centro Educacional Logos) Nova Iguaçu – RJ
Sérgio Cláudio Ramos (IM-UFRGS) Porto Alegre – RS
Seme Gebara Neto (UFMG) Belo Horizonte – MG
Tadeu Ferreira Gomes (UEBA) Juazeiro – BA
Tomás Menéndez Rodrigues (U. Federal de Rondônia) Porto Velho – RO
Valdenberg Araújo da Silva (U. Federal de Sergipe) São Cristovão – SE
Vânia Cristina Silva Rodrigues (U. Metodista de SP) S.B. do Campo – SP
Wagner Pereira Lopes (CEFET – GO) Jataí – GO

EUREKA! N°27, 2008

62
CONTEÚDO

XXIX OLIMPÍADA BRASILEIRA DE MATEMÁTICA 2


Problemas e Soluções da Primeira Fase

XXIX OLIMPÍADA BRASILEIRA DE MATEMÁTICA 15


Problemas e Soluções da Segunda Fase

XXIX OLIMPÍADA BRASILEIRA DE MATEMÁTICA 35


Problemas e Soluções da Terceira Fase

XXIX OLIMPÍADA BRASILEIRA DE MATEMÁTICA 57


Problemas e Soluções da Primeira Fase – Nível Universitário

XXIX OLIMPÍADA BRASILEIRA DE MATEMÁTICA 62


Problemas e Soluções da Segunda Fase – Nível Universitário

XXIX OLIMPÍADA BRASILEIRA DE MATEMÁTICA 73


Premiados

AGENDA OLÍMPICA 77

COORDENADORES REGIONAIS 78
Sociedade Brasileira de Matemática

XXIX OLIMPÍADA BRASILEIRA DE MATEMÁTICA


Problemas e soluções da Primeira Fase

PROBLEMAS – NÍVEL 1

01) Observe as multiplicações a seguir:


101× 11 = 1111
101× 111 = 11211
101× 1111 = 112211
101× 11111 = 1122211
K
Qual é a soma dos algarismos do número obtido quando multiplicamos 101 pelo
142K
número 11111 43
11 ?
2007 algarismos 1

A) 1001 B) 2007 C) 2009 D) 4008 E) 4014

02) Quantos números inteiros positivos de três algarismos têm a soma de seus
algarismos igual a 4?
Observação: lembre-se de que zeros à esquerda não devem ser contados como
algarismos; por exemplo, o número 031 tem dois algarismos.
A) 4 B) 6 C) 7 D) 10 E) 12

03) Juntando dois retângulos iguais lado a lado, sem sobreposição, podemos
formar dois tipos de figura: um quadrado de área igual a 144 cm2 ou um retângulo
de largura diferente do comprimento. Qual é o perímetro deste último retângulo,
em cm?
A) 12 B) 24 C) 48 D) 60 E) 72

04) A figura ao lado é formada por dois quadrados de


área 100 cm2 cada um, parcialmente sobrepostos, de
modo que o perímetro da figura (linha mais grossa) é
igual 50 cm. Qual é a área da região comum aos dois
quadrados, em cm2 ?
A) 20 B) 25 C) 30 D) 40
E) 50

EUREKA! N°28, 2008

2
Sociedade Brasileira de Matemática

05) A soma de todos os números positivos ímpares até 2007 menos a soma de
todos os números positivos pares até 2007 é igual a:
A) 1003 B) 1004 C) 2005 D) 2006 E) 2007

06) Sílvia pensou que seu relógio estava atrasado 10 min e o acertou, mas na
verdade o relógio estava adiantado 5 min. Cristina pensou que seu relógio estava
adiantado 10 min e o acertou, mas na verdade o relógio estava atrasado 5 min.
Logo depois, as duas se encontraram, quando o relógio de Sílvia marcava 10
horas. Neste momento, que horas o relógio de Cristina indicava?
A) 9h 30min B) 9h 50min C) 10h D) 10h 5min E) 10h 15min

a
07) A fração , onde a e b são inteiros 0 1
b
positivos, representa um número entre 0
e 1, na posição indicada no desenho ao a
lado. Qual é um possível valor para a b
soma a + b ?
A) 1 B) 2 C)3
D) 4 E) 5

08) Em uma prova de olimpíada, 15% dos estudantes não resolveram nenhum
problema, 25% resolveram pelo menos um problema, mas cometeram algum erro,
e os restantes, 156 estudantes, resolveram todos os problemas corretamente. O
número de estudantes que participaram da olimpíada foi:
A) 200 B) 260 C) 93 D) 223 E) 300

09) Em uma certa cidade, a razão entre o número de homens e mulheres é 2 : 3 e


entre o número de mulheres e crianças é 8 : 1. A razão entre o número de adultos
e crianças é:
A) 5 : 1 B) 16 : 1 C) 12 : 1 D) 40 : 3 E) 13 : 1

10) Na figura, o lado AB do triângulo G


eqüilátero ABC é paralelo ao lado DG do A
quadrado DEFG. Qual é o valor do ângulo x? F
A) 80o B) 90o C) 100o x
D) 110o E) 120o D

B C E

EUREKA! N°28, 2008

3
Sociedade Brasileira de Matemática

11) Uma loja de CD`s realizará uma liquidação e, para isso, o gerente pediu para
Anderlaine multiplicar todos os preços dos CD`s por 0,68. Nessa liquidação, a
loja está oferecendo um desconto de:
A) 68% B) 6,8% C) 0,68% D) 3,2% E) 32%

12) Esmeralda e Pérola estão numa fila. Faltam 7 pessoas para serem atendidas
antes de Pérola e há 6 pessoas
depois de Esmeralda. Duas outras pessoas estão entre Esmeralda e Pérola. Dos
números abaixo, qual pode ser o número de pessoas na fila?
A) 9 B) 11 C) 13 D) 14 E) 15

13) Preenchemos as casas vazias da tabela ao lado x 1 2 3 5 7 11 13


com o produto dos números que estão sombreados
na mesma linha e na mesma coluna da casa vazia 1
a ser preenchida. Quantas dessas casas conterão 2
números primos? 3
A) 6 B) 7 C) 12 5
D) 14 E) 26 7
11
13

14) O conteúdo de uma garrafa de refrigerantes enche três copos grandes iguais e
mais meio copo pequeno ou 5 desses copos pequenos iguais mais a metade de um
daqueles grandes. Qual é a razão entre o volume de um copo pequeno e o de um
grande?
A) 2 B) 3 C) 7 D) 5 E) 3
5 7 10 9 5

15) Um código de barras é formado por barras


verticais pretas de três larguras diferentes. Duas
barras pretas sempre são separadas por uma
barra branca, também com três larguras
diferentes. O código começa e termina com uma
barra preta, como no exemplo ao lado.
Considere um código S, formado por uma barra
preta fina, duas médias e uma grossa, separadas
por barras brancas finas. Quantos códigos S
diferentes podem ser assim formados?
A) 4 B) 6 C) 12
D) 24 E) 36

EUREKA! N°28, 2008

4
Sociedade Brasileira de Matemática

16) No quadriculado ao lado, cada


quadradinho tem 1 cm2. Os segmentos
inclinados ligam pontos médios dos lados
dos quadradinhos ou um vértice ao centro
de um quadradinho. Qual é a área ocupada
pela sigla OBM, em cm2?
A) 28 B) 32 C) 33
D) 34 E) 35

17) Lina e Lana brincam da seguinte maneira: a primeira a jogar pensa em um


número de 10 a 99 e diz apenas a soma dos algarismos do número; a segunda tem
então que adivinhar esse número. Qual é o maior número de tentativas erradas
que a segunda pessoa pode fazer?
A) 7 B) 8 C) 9 D) 10 E) 11

18) Anita imaginou que levaria 12 minutos para terminar a sua viagem, enquanto
dirigia à velocidade constante de 80 km/h, numa certa rodovia. Para sua surpresa,
levou 15 minutos. Com qual velocidade constante essa previsão teria se
realizado?
A) 90 km/h B) 95 km/h C) 100 km/h D) 110 km/h E) 120 km/h

19) O gráfico ao lado mostra o


percentual de acertos numa prova de 60 70%
testes de seis candidatos finalistas de um 60%
concurso. Qual foi o número médio de 50%
questões erradas por esses candidatos 40%
nessa prova? 30%
A) 14 B) 24 C) 30 20%
D) 32 E) 40 10%

A B C D E F

20) Ao efetuar a soma 131 + 132 + 133 + L + 132006 + 132007 obtemos um número
inteiro. Qual é o algarismo das unidades desse número?
A) 1 B) 3 C) 5 D) 7 E) 9

EUREKA! N°28, 2008

5
Sociedade Brasileira de Matemática

PROBLEMAS – NÍVEL 2

01) Veja o problema No. 1 do Nível 1.


02) Veja o problema No. 7 do Nível 1.
03) Veja o problema No. 10 do Nível 1.

04) Em uma certa cidade, a razão entre o número de homens e mulheres é 2 : 3 e


entre o número de mulheres e crianças é 8 : 1. A razão entre o número de adultos
e crianças é:
A) 5 : 1 B) 16 : 1 C) 12 : 1 D) 40 : 3 E) 13 : 1

05) Veja o problema No. 8 do Nível 1.

06) Se N é o quadrado do quadrado de um número inteiro e tem 12 como fator,


N
o menor valor para é:
12
A) 3 B) 12 C) 36 D) 54 E) 108

07) O jardim da casa de Maria é formado por cinco quadrados de igual área e tem
a forma da figura abaixo. Se AB = 10 m, então a área do jardim em metros
quadrados é:

500 100
A) 200 B) 10 5 C) 100 D) E)
3 3

EUREKA! N°28, 2008

6
Sociedade Brasileira de Matemática

08) Sejam a, b, c e k números reais diferentes de zero satisfazendo as relações


a b c
k= = = . Qual é o número de possíveis valores que k pode
b+c c+a a+b
assumir?
A) 0 B) 1 C) 2 D) 3 E) 4

09) Doze pontos estão sobre um círculo. Quantos polígonos convexos podemos
formar com vértices nesses 12 pontos?
A) 4017 B) 220 C) 4095 D) 66 E) 3572

10) De quantas maneiras diferentes podemos escrever o número 2007 como soma
de dois ou mais números inteiros positivos e consecutivos?
A) 1 B) 2 C) 3 D) 4 E) 5

11) As equações do 2o grau 2007 x 2 + 2008 x + 1 = 0 e x 2 + 2008 x + 2007 = 0


têm uma raiz comum. Qual é o valor do produto das duas raízes que não são
comuns?
A) 0 B) 1 C) 2007 D) 2008 E) 2007

12) Qual é o máximo valor que o número a (b + c) − b( a + c) pode assumir se


a, b e c , são inteiros satisfazendo 1 ≤ a ≤ 10 , 1 ≤ b ≤ 10 e 1 ≤ c ≤ 10 ?
A) 80 B) 81 C) 84 D) 90 E) 100
13) A quantidade de inteiros x com três dígitos tais que 6x e 7x possuem a mesma
quantidade de
dígitos é:
A) 767 B) 875 C) 876 D) 974 E) 975

14) A figura abaixo é formada por três quadrados de lado 1 e um retângulo que os
contorna.

A área do retângulo é:
A) 3 2 B) 4 2 C) 6 D) 6 2 E) 8

EUREKA! N°28, 2008

7
Sociedade Brasileira de Matemática

15) Se x é real positivo e 1 + (x2 + x)(x2 + 5x + 6) = 1812, então o valor de x(x + 3)


é:
A) 180 B) 150 C) 120 D) 182 E) 75

16) A figura abaixo mostra um retângulo, um pentágono, um triângulo e um


círculo, com áreas respectivamente 121, 81, 49 e 25 centímetros quadrados. A
diferença entre a área preta e a área cinza, em centímetros quadrados, é:

A) 25 B) 36 C) 49 D) 64 E) 81

17) As seguradoras de automóveis A e B cobram um valor anual (prêmio) mais


um valor que o usuário deve pagar em caso de acidente (franquia). Jean quer
fazer um seguro para seu automóvel e recebeu as seguintes propostas das
seguradoras:
Seguradora A: Prêmio anual de R$ 1500,00 e franquia de R$ 1400,00
Seguradora B: Prêmio anual de R$ 1700,00 e franquia de R$ 700,00
Para valer a pena Jean contratar a Seguradora A, ele não deve se acidentar com o
carro por pelo menos N anos. O valor de N é:
A) 2 B) 3 C) 4 D) 5 E) 6

18) O desenho abaixo mostra um dado comum cujas somas das pontuações em
faces opostas é
sempre igual a 7. Ele é colocado em uma mesa horizontal com a face “1” voltada
para Leste. O dado é, então, movido quatro vezes.

Norte

Leste

EUREKA! N°28, 2008

8
Sociedade Brasileira de Matemática

Um movimento consiste em uma rotação de 90 ° em relação a uma aresta. Depois


do primeiro movimento a face em contato com a mesa passa a ser a “1”, depois a
“2”, então a “3” e, finalmente, a face “5”. Para que sentido está voltada a face “1”
após esta seqüência de movimentos?
A) Oeste B) Leste C) Norte D) Sul E) Cima

19) Uma avenida possui 100 prédios numerados de 1 a 100, onde prédios com
numeração par se situam do lado direito da rua e prédios com numeração ímpar
se situam no lado esquerdo. A quantidade de andares de cada prédio é igual à
soma dos algarismos do número correspondente ao prédio. Assim, podemos
afirmar que:
A) A quantidade de prédios com mais de 10 andares é maior do lado direito da
rua.
B) A quantidade de prédios com menos de 5 andares é maior do lado direito da
rua.
C) Pelo menos metade dos prédios possui 10 ou mais andares.
D) Em ambos os lados da rua há a mesma quantidade de prédios com exatos 8
andares.
E) Pelo menos 25% dos prédios possui menos de 5 andares.

20) Qual o menor perímetro inteiro possível de um triângulo que possui um dos
5 3
lados com medida igual a ?
2
A) 8 B) 9 C) 10 D) 11 E)12

21) Determine em qual dos horários abaixo o ângulo determinado pelos ponteiros
de um relógio é o menor.
A) 02h30 B) 06h20 C) 05h40 D) 08h50 E) 09h55

22) O máximo divisor comum entre os números 1221, 2332, 3443, 4554,........,
8998 é:
A) 3 B) 33 C) 37 D) 11 E) 101

23) Uma mesa de bilhar tem dimensões de 3 metros por 6 metros e tem caçapas
nos seus quatro cantos P, Q, R e S. Quando uma bola bate na borda da mesa, sua
trajetória forma um ângulo igual ao que a trajetória anterior formava.

EUREKA! N°28, 2008

9
Sociedade Brasileira de Matemática

R Q

S P
Uma bola, inicialmente a 1 metro da caçapa P, é batida do lado SP em direção ao
lado PQ, como mostra a figura. A quantos metros de P a bola acerta o lado PQ se
a bola cai na caçapa S após duas batidas na borda da mesa?
6 3 2 3
A) 1 B) C) D) E)
7 4 3 5

24) Considere todos os números abc de três algarismos onde b = a2 + c2 e a ≠


0 . A diferença entre o maior e o menor destes números é um número:
A) Múltiplo de 3 B) Primo
C) Com último algarismo igual a 7 D) Cuja soma dos algarismos é 10
E) Múltiplo de 7

25) Seja {an} uma seqüência na qual cada termo é definido como o dobro da
soma dos algarismos do termo anterior, mais uma unidade. Por exemplo, se an =
234, então an +1 = 2(2 + 3 + 4) +1.
Se, a1 = 1 o valor de a31 + a32 + a33 + a34 + a35 é igual a:
A) 44 B) 54 C) 64 D) 77 E) 84

PROBLEMAS – NÍVEL 3

01) A figura mostra dois quadrados sobrepostos. Qual é o valor de x + y, em


graus?

y
A) 270 B) 300 C) 330 D) 360 E) 390

EUREKA! N°28, 2008

10
Sociedade Brasileira de Matemática

02) Um número de quatro dígitos é dito peroba se possui pelo menos dois dígitos
vizinhos com a mesma paridade. Quantos números perobas existem?
A) 8999 B) 8874 C) 7875 D) 8000 E) 7750

03) Veja o problema No. 15 do Nível 2.


04) Veja o problema No. 18 do Nível 2.

05) Os números 72, 8, 24, 10, 5, 45, 36, 15 são agrupados em duplas de modo que
o produto de cada dupla é o mesmo. Qual número fica com o 10?
A) 36 B) 45 C) 24 D) 15 E) 72

06) Tintas pretas opacas absorvem 97% da luz, refletindo o restante. Cientistas
desenvolveram uma nova cobertura superpreta que é “dez vezes mais preta” que
tintas pretas opacas, querendo dizer que ela reflete 1/10 da luz refletida pelas
tintas pretas opacas. Que porcentagem de luz a nova cobertura absorve?
A) 9,7 B) 90,3 C) 99,7 D) 99,9 E) 970

07) Considere a seguinte seqüência:


27 = 3 × 3 × 3 , 207 = 3 × 3 × 23 , 2007 = 3 × 3 × 223 , 20007 = 3 × 3 × 2223 , ...
Qual dos seguintes inteiros é um múltiplo de 81?
A) 200.007 B) 20.000.007
C) 2.000.000.007 D) 200.000.000.007
E) 20.000.000.000.007

08) Qual dos inteiros positivos abaixo satisfaz a seguinte equação:


4 5 6 n4 − 6 n4 − 5 n4 − 4
+ + + L + + + = 309 ?
n4 n4 n4 n4 n4 n4
A) 2007 B) 309 C) 155 D) 25 E) 5

09) O desenho abaixo mostra um semicírculo e um triângulo isósceles de mesma


área. Qual é o valor de tg x ° ?

xo

EUREKA! N°28, 2008

11
Sociedade Brasileira de Matemática

3 π 2 π
A) 1 B) C) D) E)
2 3 π 2

10) Um episódio muito conhecido na Matemática foi quando ao visitar o grande


matemático Ramanujam no hospital, o outro grande matemático Hardy disse que
o número do táxi que o trouxe, 1729, era um número sem graça; Ramanujam
respondeu prontamente: “Não diga isso, Hardy! 1729 é o menor número inteiro
positivo que pode ser escrito como soma de dois cubos perfeitos positivos de duas
maneiras diferentes!” De fato, 1729 = 103 + 93 = 123 + 13.
Um outro episódio não muito conhecido na Matemática foi quando o pequeno
matemático Muralijam foi visitado pelo outro pequeno matemático Softy, que
disse que o número do lotação que o trouxe era um número sem graça. Muralijam
responde imediatamente: “Não, Softy, ele é o menor inteiro positivo que pode ser
escrito como soma de dois quadrados perfeitos positivos de duas maneiras
diferentes!”
A que número Muralijam e Softy se referem?
A) 18 B) 41 C) 45 D) 50 E) 65

11) Dizemos que uma palavra Q é quase-anagrama de outra palavra P quando Q


pode ser obtida retirando-se uma letra de P e trocando a ordem das letras
restantes, resultando em uma palavra com uma letra a menos do que P. Um
quase-anagrama pode ter sentido em algum idioma ou não. Por exemplo, RARO,
RACR e ARCO são quase-anagramas de CARRO.
Quantos são os quase-anagramas da palavra BACANA que começam com A?
A) 48 B) 60 C) 72 D) 96 E) 120

12) As cidades Aópolis, Beópolis e Ceópolis são ligadas por estradas retas. Sabe-
se a estrada que liga Aópolis e Beópolis é perpendicular à estrada que liga
Aópolis e Ceópolis. Rubens mora em Beópolis e tem um compromisso em
Ceópolis. Todavia, a estrada que liga Beópolis a Ceópolis está interditada, de
modo que Rubens é obrigado a fazer o trajeto Beópolis-Aópolis-Ceópolis. Para
chegar ao compromisso na hora certa, Rubens trafega com uma velocidade 24%
maior do que trafegaria se utilizasse a estrada interditada.

Se α é o menor ângulo do triângulo determinado pelas três estradas, então


1 1 1
A) 0 < tgα < B) < tgα <
6 6 5
1 1 1 1 1
C) < tgα < D) < tgα < E) < tgα < 1
5 4 4 3 3
EUREKA! N°28, 2008

12
Sociedade Brasileira de Matemática

13) Todo número real a pode ser escrito de forma única como a = a  + {a} , em
que a  é inteiro e 0 ≤ {a} < 1 . Chamamos a  parte inteira de a e {a} parte
fracionária de a.
Se x +  y  + {z} = 4,2 , y + z  + {x} = 3,6 e z + x  + {y} = 2 , quanto vale x – y +
z?
A) –1 B) –0,5 C) 0 D) 0,5 E) 1

14) Dizemos que um natural X é um repunit quando os seus algarismos são todos
iguais a 1, ou seja, quando X é da forma 11…1.
Sejam p, q e r inteiros, p > 0 , tais que pX 2 + qX + r é um repunit sempre que X
é um repunit. Qual dos valores a seguir é um possível valor de q?
A) –2 B) –1 C) 0 D) 1 E) 2

15) O conjunto dos valores de c para os quais a equação x= x + c possui


solução real está contido em:
A) [−1; ∞[ B) ] − ∞;1] C) [−3;2] D) [−2;3] E) Z

16) No triângulo ABC, AD é a altura relativa ao lado BC. Se AB = DC = 1,


assinale a alternativa que corresponde à área máxima do triângulo ABC.
3 3 3 2 2 1
A) B) C) D) E)
8 2 3 2 2

17) O número de pares (x, y) de inteiros positivos que satisfazem a equação


x8 + 3y 4 = 4x 2 y 3 ,
com 1≤ y ≤ 2007, é igual a:
A) 40 B) 41 C) 42 D) 43 E) 44

18) Sejam a, b e c números tais que


a2 – ab = 1
b2 – bc = 1
c2 – ac = 1
O valor de abc⋅(a + b + c) é igual a:
A) 0 B) 1 C) 2 D) −1 E) −3

19) Veja o problema No. 19 do Nível 2.


20) Veja o problema No. 20 do Nível 2.
21) Veja o problema No. 21 do Nível 2.

EUREKA! N°28, 2008

13
Sociedade Brasileira de Matemática

22) O máximo divisor comum entre os números 1221, 2332, 3443, 4554,........,
8998 é:
A) 3 B) 33 C) 37 D) 11 E) 101

23) Veja o problema No. 23 do Nível 2.


24) Veja o problema No. 24 do Nível 2.
25) Veja o problema No. 25 do Nível 2.

GABARITO
NÍVEL 1 (5ª. e 6ª. Séries)
1) E 6) A 11) E 16) D
2) D 7) E 12) B 17) B
3) D 8) B 13) C 18) C
4) E 9) D 14) D 19) D
5) B 10) E 15) C 20) E

NÍVEL 2 (7ª. e 8ª. Séries)


1) E 6) E 11) B 16) D 21) E
2) E 7) C 12) D 17) B 22) D
3) E 8) C 13) C 18) A 23) B
4) D 9) A 14) C 19) B 24) Anulada
5) B 10) E 15) A 20) B 25) Anulada

NÍVEL 3 (Ensino Médio)


1) A 6) C 11) B 16) A 21) E
2) C 7) E 12) D 17) E 22) D
3) A 8) E 13) B 18) D 23) B
4) A 9) E 14) E 19) B 24) Anulada
5) A 10) D 15) A 20) B 25) D

EUREKA! N°28, 2008

14
Sociedade Brasileira de Matemática

XXIX OLIMPÍADA BRASILEIRA DE MATEMÁTICA


Problemas e Soluções da Segunda Fase

PROBLEMAS – Nível 1 PARTE A


(Cada problema vale 5 pontos)

01. O número N = 1010010100101... contém somente os algarismos 0 e 1, de


modo que o número de algarismos 0 entre dois algarismos 1 é um ou dois,
alternadamente. O número N tem exatamente 101 algarismos. Qual é a soma de
todos os algarismos do número N?

02. Uma folha de papel tem 20 cm de


comprimento por 15 cm de largura.
Dobramos essa folha ao meio,
paralelamente à sua largura. Em
seguida, dobramos a folha retangular
dupla, de modo que dois vértices
opostos coincidam. Ao desdobrar a
folha, as marcas da segunda dobra
dividem a folha em duas partes,
conforme mostrado na figura ao lado.
Qual é a área da parte escura, em cm2?

03. Observe as igualdades a seguir:


1+ 2 +1 = 4
1+ 2 + 3 + 2 +1 = 9
1 + 2 + 3 + 4 + 3 + 2 + 1 = 16
M
1 + 2 + 3 + L + 2006 + 2007 + 2006 + L 3 + 2 + 1 = A
Qual é o valor de A ?
2232

04. Uma folha retangular de cartolina foi cortada ao longo de sua diagonal. Num
dos pedaços restantes, na forma de um triângulo retângulo, foram feitos dois
cortes, paralelos aos lados menores, pelos meios desses lados. Ao final sobrou um
retângulo de perímetro 129 cm. O desenho abaixo indica a seqüência de cortes.

EUREKA! N°28, 2008

15
Sociedade Brasileira de Matemática

Em centímetros, qual era o perímetro da folha antes do corte?

05. Um reservatório cúbico internamente tem 2 metros de lado e contém água até
a sua metade. Foram colocados no reservatório 25 blocos retangulares de
madeira, que não absorvem água, de dimensões 20 × 30 ×160 centímetros.
Sabendo que 80% do volume de cada bloco permanece submerso na água,
calcule, em centímetros, a altura atingida pela água, no reservatório.

06. A adição ao lado está incorreta. Entretanto, se


substituirmos somente um certo algarismo a, toda
vez que ele aparece, por um certo algarismo b, a
conta fica correta. Qual é o valor de a b ?

PARTE B
(Cada problema vale 10 pontos)

PROBLEMA 1
A área do quadrado ABCD é 300 cm2. Na figura, M é ponto
médio de CD e o ponto F pertence à reta BC.

a) Qual é a área do triângulo ABF ?


M
b) Qual é a área do triângulo ADF ?

PROBLEMA 2
Esmeralda comprou seis discos de ferro para usar num aparelho de ginástica.
Esses discos têm massas 1, 2, 3, 4, 5 e 6 quilogramas, respectivamente.
Esmeralda pode combiná-los e obter outras massas, como por exemplo:
1 disco de 2 kg + 1 disco de 6 kg = 8 kg.
Qual a maior quantidade de massas diferentes que ela pode obter?

EUREKA! N°28, 2008

16
Sociedade Brasileira de Matemática

PROBLEMA 3
Observe como o quadriculado ao lado
é preenchido.
3
a) Qual é a soma dos elementos da 3
diagonal 9? 3

3
b) Qual é o resto da divisão por 100 da
soma dos elementos da diagonal
2007?

PROBLEMAS – Nível 2 PARTE A


(Cada problema vale 4 pontos)

01. Ludmilson descobriu que o produto da idade que tinha há 55 anos atrás pela
idade que terá daqui a 55 anos é igual ao cubo de um número primo. Qual é a
idade atual de Ludmilson?
f (10 −8 ) − f (103 )
02. Sendo f(x) = 100x + 3, calcule o valor de − f (−1) .
10 −8 − 103

03. Na figura abaixo temos um pentágono regular, um quadrado e um triângulo


eqüilátero, todos com a mesma medida de lado.
Q
C
P E
R

S T A D
Determine a medida, em graus, do ângulo ∠QCE.

04. Um inteiro positivo K tem n algarismos e é igual a 2608.n. Determine a soma


dos algarismos de K

05. Em 1949 o matemático indiano D. R. Kaprekar, inventou um processo


conhecido como Operação de Kaprekar. Primeiramente escolha um número de

EUREKA! N°28, 2008

17
Sociedade Brasileira de Matemática

quatro dígitos (não todos iguais), em seguida escreva a diferença entre o maior e
o menor número que podem ser formados a partir de uma permutação dos dígitos
do número inicial. Repetindo o processo com cada número assim obtido,
obtemos uma seqüência. Por exemplo, se o primeiro número for 2007, o segundo
será 7200 – 0027 = 7173. O terceiro será 7731 – 1377 = 6354.
Começando com o número 1998, qual será o 2007-ésimo termo da seqüência?

PROBLEMAS – Nível 2 PARTE B


(Cada problema vale 10 pontos)

PROBLEMA 1
O triângulo ABC é retângulo em B. Sejam I o centro da circunferência inscrita em
ABC e O o ponto médio do lado AC. Se ∠AOI = 45°, quanto mede, em graus, o
ângulo ∠ACB?

PROBLEMA 2
Sejam α e β as raízes da equação quadrática (x – 2)(x – 3) + (x – 3)(x + 1) + (x +
1)(x – 2) = 0.

1 1 1
Determine o valor de + + .
(α + 1)( β + 1) (α − 2)( β − 2) (α − 3)( β − 3)

PROBLEMA 3
a) Determine a quantidade de divisores do número N = 235 – 23.
b) Mostre que para todo número natural n , n5 – n é múltiplo de 30.

PROBLEMA 4
Um quadrado 4 × 4 é dividido em 16 quadrados unitários. Cada um dos 25
vértices desses quadrados deve ser colorido de vermelho ou azul. Ache o número
de colorações diferentes tais que cada quadrado unitário possua exatamente dois
vértices vermelhos.

PROBLEMAS – Nível 3 PARTE A


(Cada problema vale 4 pontos)

01. Quantos divisores positivos do número 123456 são menores que 2007?

EUREKA! N°28, 2008

18
Sociedade Brasileira de Matemática

02. Considere o conjunto A dos pares ordenados (x;y) de reais não negativos tais
que x + y = 2. Se a probabilidade de um elemento de A escolhido aleatoriamente
estar a uma distância da origem menor ou igual a 5 3 é p, quanto vale 2535p2?

03. Qual é a soma dos algarismos do inteiro mais próximo de 1


424 3
111...1?
1000 uns

04. Veja o problema 1 da parte B do nível 2.


05. Veja o problema 4 da parte B do nível 2.

PROBLEMAS – Nível 3 PARTE B


(Cada problema vale 10 pontos)
PROBLEMA 1
Ache todos os pares (x, y) de inteiros positivos tais que
2(x + y) + xy = x2 + y2.

PROBLEMA 2
Encontre todos os números n de seis algarismos da forma AAABBB, em que A e B
são algarismos diferentes e não nulos e n + 1 é um quadrado perfeito.

PROBLEMA 3
No quadrilátero convexo ABCD, ∠A + ∠B = 120°, AD = BC = 5 e AB = 8.
Externamente ao lado CD, construímos o triângulo eqüilátero CDE. Calcule a
área do triângulo ABE.

PROBLEMA 4
Em um certo país há 21 cidades e o governo pretende construir n estradas (todas
de mão dupla), sendo que cada estrada liga exatamente duas das cidades do país.
Qual o menor valor de n para que, independente de como as estradas sejam
construídas, seja possível viajar entre quaisquer duas cidades (passando,
possivelmente, por cidades intermediárias)?

EUREKA! N°28, 2008

19
Sociedade Brasileira de Matemática

Soluções Nível 1 – Segunda Fase – Parte A


Problema 01 02 03 04 05 06
Resposta 41 150 81 258 148 64

1. [41] O número é formado por blocos iguais, de 5 algarismos na forma


“10100”. Como o número tem 101 algarismos, concluímos que é formado por
20 desses blocos inteiros mais o primeiro algarismo de um bloco, que é 1. A
soma dos algarismos de cada bloco é 1 + 0 +1 + 0 + 0 = 2, portanto a soma
dos algarismos de N é 20 × 2 + 1 = 41 .

2. [150] O desenho abaixo à esquerda mostra como fica a folha após a primeira
dobra. À direita, mostra como fica a folha após as duas dobras.

Observamos que CE = EA e que CF = FA. Por uma propriedade da dobra,


sabemos que o segmento FE é perpendicular ao segmento AC e esses segmentos
se cruzam em seus pontos médios. Portanto, os quatro triângulos que compõem o
quadrilátero AECF são congruentes; são congruentes também os triângulos EBC
e FDA. Portanto, a dobra FE divide o retângulo ABCD em dois trapézios, EBCF e
AEFD, de mesma área. Desdobrando inteiramente a folha, obtemos duas metades
iguais. Portanto, a área do pentágono convexo BEFE’B’ é igual à área do
pentágono não convexo AA’E’FE, ou seja, a área da parte escura é metade da área
15 × 20
da folha, portanto igual a = 150 cm 2 .
2

EUREKA! N°28, 2008

20
Sociedade Brasileira de Matemática

3. [81] Pelo padrão observado, as somas são iguais ao quadrado da parcela


central (aquela cujo número de parcelas à esquerda é igual ao número de
parcelas à direita).
2
A 2007 2  2007 
Portanto, A = 2007 2 e, assim, = =  = 9 = 81 .
2

 223 
2 2
223 223

4. [258] O retângulo que sobra após os cortes tem lados iguais às metades dos
lados da cartolina original, cujo perímetro, então, é o dobro do perímetro
desse retângulo. Logo, o perímetro da cartolina antes do corte é
129 × 2 = 258 cm.

5. [148] O volume de cada bloco de madeira é 0, 2 × 0,3 × 1, 60 = 0, 096 m 3 ; o


volume de cada bloco que fica submerso no líquido é 0,80 × 0, 096 m 3 . O
volume de líquido deslocado pelos 25 blocos é igual a
25 × 0,80 × 0, 096 =1,92 m3 . Como o reservatório é um cubo de 2 m de
lado, sua base é um quadrado de área 4 m2. Podemos pensar no líquido
deslocado como se fosse um bloco cuja base é igual à base do reservatório, de
altura h e volume acima.

1,92
Portanto 4h = 1,92 ⇔ h = = 0, 48 m = 48 cm . Como a altura inicial
4
do líquido era 100 cm, a nova altura será 148 cm.

EUREKA! N°28, 2008

21
Sociedade Brasileira de Matemática

6. [64] À primeira inspeção, podemos admitir que os três algarismos à direita de


todos os números estão corretos, isto é, estão corretamente escritos os
algarismos 0, 1, 3, 4, 5, 6 e 8. Portanto, dentre os algarismos 2, 7 e 9, um
deles está escrito incorretamente. O 9 está escrito corretamente, pois se o
mudarmos, a soma com 2 não estará certa. Logo ou 2 ou 7 está errado. Se o 7
estiver errado, então 2 estará correto, mas isso não é possível pois a soma de
2 com 4 mais 1 não estaria certa. Logo, o 2 é que deve ser substituído;
olhando novamente a soma de 2 com 4 mais 1 resultando 1 vemos que o
resultado só dará certo se no lugar de 2 colocarmos 6. Fazendo a substituição,
verificamos que o resto se encaixa. Teremos, então, a b = 26 = 64 .

Soluções Nível 1 – Segunda Fase – Parte B

1. Temos m( FMCˆ ) = m( AMD


ˆ ) (ângulos opostos pelo vértice),
ˆ ) = m( FCM
m( ADM ˆ ) (pois ABCD é quadrado, logo esses ângulos são
retos) e MC = MD (pois M é ponto médio de CD). Logo, os triângulos AMD e
FMC são congruentes.

EUREKA! N°28, 2008

22
Sociedade Brasileira de Matemática

a) Vemos que a área ∆ABF = área ∆FMC + área ABCM .


Como área ∆FMC = área ∆AMD , temos:
área ∆ABF = área ∆AMD + área ABCM = área do quadrado ABCD = 300
cm2.
b) área ∆ADF = área ∆AMD + área ∆DMF =
= área ∆FMC + área ∆DMF = área ∆FCD

Como AD = FC, CD é lado comum e os ângulos Cˆ e Dˆ são retos,


concluímos que os triângulos FCD e ADC são congruentes,
área ABCD
logo área ∆FCD = área ∆ADC = . Portanto, a área do triângulo
2
300
ADF é igual a = 150 cm 2 .
2
2. Dadas as massas de 1 a 6, podemos adicionar 1 a 6, 2 a 6, etc, até obter todos
os pesos de 7 a 11; podemos adicionar 1 + 5 a 6, 2 + 5 a 6, etc, até obter todos
os pesos de 12 a 15; podemos adicionar 1 + 4 + 5 a 6, etc, obtendo os pesos
de 16 a 18; somando 1 + 3 + 4 + 5 a 6 obtemos 19; 2 + 3 + 4 + 5 a 6 obtemos
20 e, finalmente, somando 1 + 2 + 3 + 4 + 5 a 6 obtemos 21. Portanto, a
quantidade de massas diferentes que Esmeralda pode obter é 21.

3. Pode-se concluir, examinando a tabela, que a soma dos elementos da


diagonal n é igual a 2n + (n – 1)k, onde k é o algarismo das unidades do
número n. Por exemplo, na diagonal de número 4 a soma dos números é

EUREKA! N°28, 2008

23
Sociedade Brasileira de Matemática

2 ⋅ 4 + ( 4 − 1) ⋅ 4 = 20 , na diagonal de número 10 a soma dos números é


2 ⋅10 + (10 − 1) ⋅ 0 = 20 , etc.
a) Na diagonal de número 9, a soma dos elementos é 2 ⋅ 9 + ( 9 − 1) ⋅ 9 = 90 . De
outra forma, na diagonal 9 há 10 números 9; portanto a soma é 10 ⋅ 9 = 90 .
b) Na diagonal 2007 a soma será
2 ⋅ 2007 + ( 2007 − 1) ⋅ 7 = 4014 + 14042 = 18056 .
O resto da divisão desse número por 100 é 56.

Soluções Nível 2 – Segunda Fase – Parte A

Problema 01 02 03 04 05
Resposta 66 197 174 8 6174

01. Seja x a idade de Ludmilson. Logo, ( x − 55)( x + 55) = p 3 , onde p é


primo. Temos então, duas possibilidades:
i)
 x − 55 = 1

 x + 55 = p
3

Nesse caso teríamos x = 56 e p = 111 , absurdo, pois 111 não é primo.


ii)
 x − 55 = p

 x + 55 = p
2

Com isso, 110 = p 2 − p = p ( p − 1) = 11.10 . E assim teremos p = 11 e x = 66 .


Logo, a idade de Ludmilson é 66 anos.

02. (100.10-8 + 3 – 100.10 3 – 3) / (10– 8 – 10 3) – 100.(–1) – 3 = 100(10–8 – 10 3)


/ (10–8 – 103) + 97 = 100 + 97 = 197.

03. Note que os triângulos PTA, ABD, BCE, e PQC são todos isósceles. Como
∠ STP = 108°, ∠ PTA = ∠ PAT = 72°. Assim, temos que ∠ TPA = 36° e
∠ BAD = ∠ BDA = 18°. Além disso, ∠ ABD = 144° e ∠ CBE = 66°. Como
∠ QPC = 126°, temos que ∠ QCP = 27° e ∠ ECB = 57°. Logo, ∠ QCE = 174°.

EUREKA! N°28, 2008

24
Sociedade Brasileira de Matemática

04. Tente 1, 2, 3 ... e perceba que, somente com n = 5, K terá 5 algarismos.


Assim, K = 2608 . 5 = 13040. Com isso, a soma dos algarismos de K é 8.

05. A partir do sétimo termo, todos serão iguais a 6174.

Soluções Nível 2 – Segunda Fase – Parte B

SOLUÇÃO DO PROBLEMA 1:

Como ABC é um triângulo retângulo, então AO = BO = CO. Se


∠ABI = ∠AOI = 45 o e ∠BAI = ∠OAI , então ? ABI = ? AOI (ALA). Com
isso, AB = AO = BO, e portanto, triângulo ABO é eqüilátero. Assim,
∠ACB = 30 o .

SOLUÇÃO DO PROBLEMA 2:
É fácil ver que ( x − 2)( x − 3) + ( x − 3)( x + 1) + ( x + 1)( x − 2) = 3( x − α )( x − β ) .
Fazendo x = –1, 2 e 3, nesta igualdade, temos que,
4
(α + 1)( β + 1) = 4 , (α − 2)( β − 2) = −1 , (α − 3)( β − 3) = .
3
1 1 1 1 3
Com isso, + + = − 1 + = 0.
(α + 1)( β + 1) (α − 2)( β − 2) (α − 3)( β − 3) 4 4

SOLUÇÃO DO PROBLEMA 3:
a) N = 23 ⋅ (234 − 1) = 23 ⋅ (232 + 1)(232 − 1) = 23 ⋅ (232 + 1)(23 + 1)(23 − 1) =

EUREKA! N°28, 2008

25
Sociedade Brasileira de Matemática

23 ⋅ 530 ⋅ 24 ⋅ 22 = 25 ⋅ 3 ⋅ 5 ⋅11 ⋅ 23 ⋅ 53 O número de divisores (positivos) de N é


6 × 2 × 2 × 2 × 2 × 2 = 192 .
b) N = n5 − n = n(n 2 + 1)(n + 1)(n − 1).
Necessariamente, n ou n + 1 é par. Logo, 2 divide N. Do mesmo modo, um dos
números n– 1, n ou n + 1 é múltiplo de 3. Logo 3 também divide N. Finalmente,
se nenhum dos 3 números n –1, n ou n + 1 é múltiplo de 5, então n é da forma 5k
+ 2 ou 5k + 3. No primeiro caso, temos n 2 + 1 = 25k 2 + 10k + 5 e, no segundo,
n 2 + 1 = 25k 2 + 15k + 10 , ambos múltiplos de 5. Portanto, um dos números
n, n − 1, n + 1 ou n 2 + 1 é múltiplo de 5.
Assim N é, simultaneamente, múltiplo dos números primos entre si 2, 3 e 5, o que
prova que N é múltiplo de 30.

SOLUÇÃO DO PROBLEMA 4:
Vamos começar colorindo a primeira linha de vértices. Cada coloração dessa
linha é uma seqüência de letras “A” e “V”, por exemplo, A V V A V. Observe
que, uma vez colorida a primeira linha, se aparecerem duas letras consecutivas
iguais, o restante dos vértices do tabuleiro já estão determinados. De fato, ao
aparecer dois V’s consecutivos, os dois vértices imediatamente abaixo deles
deverão ser coloridos com dois A’s, os que estão mais abaixo deverão ter dois
V’s, e assim por diante. Isto completa a coloração dessas duas colunas. Dessa
forma, cada coluna vizinha também estará determinada, pois em cada retângulo
teremos três vértices previamente coloridos, o que obriga o quarto vértice a ter
sua cor determinada. Então, para cada seqüência de A’s e V’s na primeira linha
que contém pelo menos duas letras iguais consecutivas, há exatamente uma
maneira de colorir o tabuleiro. Como há 25 – 2 = 30 de tais seqüências, contamos
30 colorações possíveis.

A V V A V
A A
V V
A A
V V

Falta-nos analisar um segundo caso, em que não há duas letras consecutivas


iguais na primeira linha. Há duas possibilidades de seqüências: começando com
A ou começando com V.

EUREKA! N°28, 2008

26
Sociedade Brasileira de Matemática

A V A V A
V

Para cada uma dessas seqüências, há duas maneiras de escolhermos a primeira


letra da segunda linha. Uma vez escolhida esta letra, a segunda linha inteira
também estará determinada. Para a primeira letra da terceira linha também há 2
possibilidades. Com este raciocínio, cada vez que escolhemos a primeira letra de
uma linha, determinamos a coloração desta linha. Logo, como há duas maneiras
de escolhermos a primeira letra de cada linha, há 25 = 32 maneiras de colorirmos
o tabuleiro, neste segundo caso. Logo, o total de colorações é igual a 30 + 32 =
62.

Observação: Veja que, no caso geral, para um quadrado n × n, o raciocínio é


análogo. No primeiro caso, teremos 2n + 1 – 2 colorações; no segundo caso, mais
2n + 1. Logo, teremos 2⋅2n+1 – 2 = 2n + 2 – 2 colorações.

Soluções Nível 3 – Segunda Fase – Parte A

Problema 01 02 03 04 05
Resposta 17 3024 1500 30 62

01. Seja a fatoração de 123456 = 2 6 ⋅ 3 ⋅ 643 e seja d um de seus divisores


menores do que 2007. Podemos analisar dois casos:

- d não é múltiplo de 643: então d é um divisor de 2 6 ⋅ 3 = 192 < 2007 . Portanto


podemos contar todos os divisores de 192, que são (6 + 1)(1 + 1) = 14 divisores.

- d é múltiplo de 643: 1 ⋅ 643 = 643 , 2 ⋅ 643 = 1286 e 3 ⋅ 643 = 1929 são menores
que 2007, mas a partir de 4 ⋅ 643 = 2572 , eles são maiores que 2007. Portanto há
3 divisores neste caso.

Portanto o total de divisores d de 123456 menores do que 2007 é 14 + 3 = 17.

02. Seja B o conjunto dos pontos de A cuja distância à origem é menor do que 5
3
e seja P = ( x; y ) um ponto de B. Sabe-se que P está sobre o segmento

EUREKA! N°28, 2008

27
Sociedade Brasileira de Matemática

x + y = 2; x, y ≥ 0 e que a distância x 2 + y 2 de P à origem é menor ou igual a


5 . Portanto:
3
x+ y =2 y = 2−x y = 2− x

x2 + y2 ≤ 5( 3) 2 ⇔
x2 + 4 − 4x + x2 ≤
25 ⇔ 2
9
11
2x − 4x + ≤ 0
9
11
4 ± 16 − 8
11 9 = 1 ± 14 , que nos dá os
As raízes de 2 x 2 − 4 x + = 0 são x0 =
9 4 6
 14 14   14 14 
pontos extremos P1 = 1 − ;1 +  e P2 = 1 + ;1 − de B. Pela
 6 6   6 6 
inequação, temos que os pontos de B estão na reta x + y = 2 , delimitados pelos
pontos P1 e P2 , logo B é o segmento de reta P1 P2 .
Queremos a probabilidade p de escolher um ponto do conjunto A estar contido no
segmento P1 P2 , que é a razão entre P1 P2 e o comprimento de A. Como A está
delimitado pelos pontos (0;2) e (2;0) , seu comprimento vale
(0 − 2) 2 + (2 − 0) 2 = 2 2 . O comprimento de B vale
2 2 2
     
1 + 14 − 1 + 14  + 1 − 14 − 1 − 14  = 2 14  = 2 7 , portanto
 6 6   6 6   3  3
  
2 7
14 14
p= 3 = e 2 5 35 p 2 = 2 5 ⋅ 35 ⋅ = 2 4 ⋅ 33 ⋅ 7 = 3024 .
2 2 6 36
67
1000 8
noves

99 K 9 101000 − 1
03. Inicialmente, temos 12
11 K31 = 9 = . Portanto
1000 uns 9
101000 − 1
1
4 2K
11 31 =
4 9
.
1000 uns
Com isso, observando que
101000 − 1 (10 500 − 1)(10 500 + 1) (10 500 − 1)(10 500 − 1) 10 500 − 1
= > = e
9 9 9 3

EUREKA! N°28, 2008

28
Sociedade Brasileira de Matemática

101000 − 1 101000 10500 10 500 − 1 10 500


< = , temos <1
424
11 K 3 1 < .
9 9 3 3 1000 uns
3
10500 − 1 10 500 + 2 10 500
Como é inteiro e seu consecutivo, , é maior do que ,o
3 3 3
6 78
500 noves

10 − 1 99K 9
500
inteiro mais próximo de 1 4 2K
11 431 é 3
=
3
= 33
{ K 3 , cuja soma dos
1000 uns 500 três

dígitos é 3 ⋅ 500 = 1500 .

04. Veja a solução do problema 1 da parte B do nível 2.


05. Veja a solução do problema 4 da parte B do nível 2.

Soluções Nível 3 – Segunda Fase – Parte B


SOLUÇÃO DO PROBLEMA 1:
Uma solução:
Multiplicando a equação dada por 2, obtemos 2x2 + 2y2 – 2xy – 4x – 4y = 0, ou
ainda,
(x2 – 4x + 4) + (y2 – 4y + 4) + (x2 – 2xy + y2) = 8.

Daí, (x – 2)2 + (y – 2)2 + (x – y)2 = 8. A única maneira de escrevermos 8 como a


soma de três quadrados é 8 = 0 + 4 + 4, em alguma ordem. Logo (x – 2, y – 2) =
(0, 2), (2, 0) ou (2, 2), de onde concluímos que as soluções são (x, y) = (2, 4), (4,
2) ou (4, 4).

Outra solução:
Escrevendo a equação dada como uma equação do segundo grau em x, temos:
x2 – (y + 2)x + (y2 – 2y) = 0.

O discriminante desta equação é ∆ = (y + 2)2 – 4(y2 – 2y) = −3y2 + 12y + 4.


4 3 4 3
Resolvendo a inequação ∆ ≥ 0, ainda obtemos 2 − ≤ y≤2+ .
3 3

Como y é inteiro positivo, as únicas possibilidades são y = 1, 2, 3 ou 4.

• Se y = 1, ficamos com ∆ = 13, que não é quadrado perfeito. Logo, este caso não
tem solução.

EUREKA! N°28, 2008

29
Sociedade Brasileira de Matemática

4±4
• Se y = 2, obtemos ∆ = 16 e x = = 0 ou 4. Como x é inteiro positivo, a
2
única solução neste caso é (x, y) = (4, 2).
• Se y = 3, ficamos com ∆ = 13, absurdo!
6±2
• Se y = 4, obtemos ∆ = 4. Neste caso, x = = 2 ou 4. Logo, (x, y) = (2, 4) ou
2
(4, 4).
Portanto, o conjunto solução é {(2, 4), (4, 2), (4, 4)}.

Mais uma solução:


Observe que 8(x + y) = 4x2 – 4xy + 4y2 = (x + y)2 + 3(x – y)2 ≥ (x + y)2, de modo
que
8(x + y) ≥ (x + y)2, ou seja, x + y ≤ 8.

Além disso, note que x2 – xy + y2 = 2(x + y) é par, e portanto ao menos uma das
parcelas do primeiro membro é par (se todos forem ímpares, x2 – xy + y2 é ímpar),
o que implica que x ou y é par. Suponha, sem perda de generalidade, que x é par.
Então y2 = 2(x + y) + xy – x2 é par e, assim, y também é par.

Logo, dos dois fatos acima, conclui-se que as únicas possibilidades para os pares
(x, y) são (2, 2), (2, 4), (2, 6), (4, 2), (4, 4) e (6, 2). Substituindo os pares, vemos
que as únicas soluções são (2, 4), (4, 2) e (4, 4).

SOLUÇÃO DO PROBLEMA 2:
Seja k inteiro positivo tal que k2 = n + 1.

Primeiro, notemos que o algarismo das unidades dos quadrados perfeitos são 0, 1,
4, 5, 6 e 9, de modo que B é igual a 9, 3, 4, 5 ou 8.

Porém, podemos eliminar alguns casos:


• Se B = 9, pois nesse caso k2 = AAABBB + 1 terminaria com exatamente três
zeros (note que A não pode ser igual a 9, pois é diferente de B);
• Se B = 3, k2 terminaria com 34, e seria par e não múltiplo de 4, já que os dois
últimos algarismos de todo múltiplo de 4 formam outro múltiplo de 4, um
absurdo.
• Se B = 4, k2 terminaria com 45, e seria múltiplo de 5 mas não de 25, já que os
dois últimos algarismos de um múltiplo de 25 são 25, 50, 75 ou 00. Outro
absurdo.

EUREKA! N°28, 2008

30
Sociedade Brasileira de Matemática

Sobram somente os casos B = 5 e B = 8.

Observe que n = k2 – 1 = (k – 1)(k + 1) = AAABBB = 111(1000A + B) é múltiplo


de 111 = 3 ⋅ 37 e, portanto, os primos 3 e 37 dividem k + 1 ou k – 1, de modo que
k é da forma 111x ± 1 ou 111x ± 38 . Além disso,
111556 ≤ k < 1000000 ⇒ 300 < k < 1000 , de modo que 3 ≤ x ≤ 9 .
2

• k = 111x ± 1 :
Temos AAABBB = k 2 − 1 = 1112 x 2 ± 222 x ⇔ 1000 A + B = 111x 2 ± 2 x . O dígito
das unidades de 1000A + B é B. Note que 111x 2 ± 2 x = 2(55 x 2 ± x) + x 2 tem a
mesma paridade que x. Assim, se B = 5, x é ímpar, ou seja, é 3, 5, 7 ou 9. Se x =
3, 5, 7, 9, o algarismo das unidades de 111x 2 + 2 x é 5, 5, 3, 9, respectivamente,
de modo que x = 3 ou x = 5, para o qual 1000A + B iguala 111 ⋅ 9 + 6 = 1005 e
111 ⋅ 25 + 10 = 2785 , o que gera a solução x = 3, A = 1 e n = 111555. Além disso,
se x = 3, 5, 7, 9, o algarismo das unidades de 111x 2 − 2 x é 3, 5, 5, 3,
respectivamente, de modo que as únicas possibilidades são x = 5 ou x = 7, para os
quais 1000 A + B iguala 2765 e 111 ⋅ 49 − 14 = 5425 respectivamente, o que
também não é possível.

Se B = 8, x é par, ou seja, é 4, 6 ou 8. Se x = 4, 6, 8, o algarismo das unidades de


111x 2 + 2 x é
4, 8, 0, respectivamente, de modo que obtemos x = 6 e 1000A + B =
111 ⋅ 36 + 12 = 4008 , ou seja, A = 4. Obtemos assim a solução n = 444888. Além
disso, se x = 4, 6, 8, o algarismo das unidades de 111x 2 − 2 x é 8, 4, 8
respectivamente, de modo que obtemos x = 4 ou x = 8, para os quais
1000A + B igual a 111 ⋅ 16 − 8 = 1768 e 111 ⋅ 64 − 16 = 7088 , respectivamente, o
que não é possível.

• k = 111x ± 38 : Temos AAABBB = k 2 − 1 = 1112 x 2 ± 2 ⋅ 111 ⋅ 38 x + 38 2 − 1


= 1112 x 2 ± 111 ⋅ 76 x
+ 37 ⋅ 39 = 111(111x 2 ± 76 x + 13) ⇔ 1000 A + B = 111x 2 ± 76 x + 13 . Estudemos,
como no caso anterior, o dígito das unidades de 111x 2 ± 76 x + 13 . Se B = 5, x é
par, ou seja, é igual a 4, 6 ou 8. Se x = 4, 6, 8, o algarismo das unidades de
111x 2 + 76 x + 13 é 3, 5, 5, respectivamente, de modo que x = 6 ou 8, para os
quais 1000A + B iguala respectivamente 111 ⋅ 36 + 76 ⋅ 6 + 13 = 4465 e
111 ⋅ 64 + 76 ⋅ 8 + 13 = 7725 , nenhum dos dois gerando solução. Além disso, se

EUREKA! N°28, 2008

31
Sociedade Brasileira de Matemática

x = 4, 6, 8, o algarismo das unidades de 111 x 2 − 76 x + 13 é 5, 3, 9,


respectivamente, de modo que x = 2 e 1000A + B igual a
111 ⋅ 16 − 76 ⋅ 4 + 13 = 1485 , o que não é possível.

Se B = 8, x é ímpar, ou seja, é igual a 3, 5, 7 ou 9. Se x = 3, 5, 7, 9 o algarismo


das unidades de 111x 2 + 76 x + 13 é 0, 8, 4, 8, respectivamente, de modo que x =
5 ou x = 9, para os quais
1000A + B = 111 ⋅ 25 + 76 ⋅ 5 + 13 = 3168 e k = 111 ⋅ 9 + 38 > 1000 , o que não é
possível. Além disso, se x = 3, 5, 7, 9 o algarismo das unidades de
111x 2 − 76 x + 13 é 4, 8, 0, 0, respectivamente, de modo que x = 5, para o qual
1000A + B = 111 ⋅ 25 − 76 ⋅ 5 + 13 = 2408 , o que não é possível.

Portanto os únicos números n que satisfazem o enunciado são 111555 e 444888.

SOLUÇÃO DO PROBLEMA 3:
Uma solução:
E
F

D C

A
Prolongue AD e BC até se encontrarem no ponto F. Veja que ∠AFB = 60° =
∠DEC. Com isso, o quadrilátero FECD é inscritível. Temos:

(i) ∠FDE = ∠FCE = α ⇒ ∠ADE = ∠BCE = 180° − α.


(ii) AD = BC e ED = EC.

De (i) e (ii), concluímos que ∆ADE ≡ ∆BCE. Portanto, EA = EB.

EUREKA! N°28, 2008

32
Sociedade Brasileira de Matemática

Além disso, ∠DEA = ∠CEB, de onde concluímos que ∠AEB = ∠DEC = 60°.
Dessa forma, o triângulo ABE é eqüilátero de lado 8 e sua área é igual a
82 3
= 16 3 cm2.
4

Outra solução:
Considere os pontos no plano complexo. Representaremos o número complexo
correspondente ao ponto X com a letra correspondente minúscula x. Fixemos o
ponto médio de AB como origem e sejam a = –4 e b = 4. Assim, sendo
α = ∠BAD e β = ∠ABC , ambos no sentido anti-horário, podemos encontrar as
coordenadas de C e D:
5
c − b = (a − b) cis(− β ) ⇔ c = 4 − 5 cis(− β )
8
5
d − a = (b − a) cis α ⇔ d = −4 + 5 cis α
8
π
Sendo ω = cis a raiz sexta da unidade e raiz da equação x 2 − x + 1 = 0 ,
3
e − d = (c − d )ω ⇔ e = (1 − ω )d + cω = ωc − ω 2 d = 4ω − 5ω cis(− β ) + 4ω 2 − 5ω 2 cis α
 π   2π 
⇔ e = 4(ω + ω − 1) − 5 cis − β  + cis + α  
 3   3 
 1+ i 3    π 2π   2π 
⇔ e = 4 2 ⋅ − 1 − 5 cis − + α  + cis + α  
 
 2   3 3   3 
  2π   2π 
⇔ e = 4 3i − 5 cis + α + π  + cis + α  
  3   3 
  2π   2π 
⇔ e = 4 3i − 5 − cis + α  + cis + α   = 4 3i
  3   3 

Assim, o triângulo ABE, com pontos de coordenadas A = (–4, 0), B = (4, 0) e


8⋅4 3
E = (0,4 3 ) , é eqüilátero e tem área = 16 3 cm2.
2

EUREKA! N°28, 2008

33
Sociedade Brasileira de Matemática

SOLUÇÃO DO PROBLEMA 4:
Escolha 20 das cidades do país. Ligando duas quaisquer delas por uma estrada,
 20  20.19
utilizaremos   = = 190 estradas, e a cidade restante não poderá ser
2 2
alcançada de automóvel. Logo se deve construir pelo menos 191 estradas. Vamos
mostrar que com essa quantidade é possível atingir nosso objetivo.

Suponha que n = 191, mas que seja possível dividir as cidades do país em dois
grupos A e B, digamos com a e b cidades, respectivamente, de tal sorte que
nenhuma cidade de A possa ser alcançada de automóvel a partir de qualquer
a b
cidade de B. Então o número de estradas no país é no máximo   +   , de
 2  2
a b
modo que   +   ≥ 191, ou ainda, (a2 + b2) – (a + b) ≥ 2⋅191 = 382.
 2  2

Como a + b = 21, segue da inequação acima que a2 + b2 ≥ 282 + 21 = 403. Logo


(a + b) 2 − (a 2 + b 2 ) 441 − 403
ab = ≤ = 19.
2 2

Mas, como a + b = 21 e a e b são naturais, temos ab ≥ 1⋅20 = 20, uma


contradição.

Logo, se n = 191, sempre é possível viajar entre quaisquer duas cidades.

EUREKA! N°28, 2008

34
Sociedade Brasileira de Matemática

XXIX OLIMPÍADA BRASILEIRA DE MATEMÁTICA


Problemas e Soluções da Terceira Fase

PROBLEMAS – NÍVEL 1
PROBLEMA 1
Parte das casas de um quadriculado com o
mesmo número de linhas (fileiras
horizontais) e colunas (fileiras verticais) é
pintada de preto, obedecendo ao padrão
apresentado pelo desenho ao lado.
a) Quantas casas serão pintadas num
quadriculado com 14 linhas e 14 colunas, de
acordo com esse padrão?
b) Quantas linhas tem um quadriculado com
199 casas pintadas?

PROBLEMA 2
Uma sala quadrada com 81 m2 de área tem o seu piso inteiramente coberto por
dois tapetes retangulares A e B, que não se superpõem, conforme mostrado na
figura (1) abaixo. Em certo momento, o tapete B é deslocado, o tapete A é girado
de 90o e colocado sobre o tapete B, conforme indicado na figura (2).

Sabendo que a área do tapete B é o dobro da área do tapete A, calcule a área da


parte do piso que ficou descoberta.

PROBLEMA 3
Em uma face de cada um de três cartões foi escrito um número inteiro positivo.
Em seguida, os cartões foram colocados lado a lado sobre uma mesa, com a face
numerada para baixo.

EUREKA! N°28, 2008

35
Sociedade Brasileira de Matemática

Arnaldo, Bernaldo e Cernaldo sabem que:

I. Os números escritos nos cartões são todos diferentes.


II. A soma dos três números é 13.
III. Os números crescem da esquerda para a direita.

a) Considerando as condições I, II e III, escreva todas as possibilidades de


numeração dos cartões.
b) Agora é hora de descobrir os números que foram escritos nos cartões.
Primeiramente, Arnaldo olha o número do primeiro cartão à esquerda e diz que
não tem informações suficientes para descobrir os outros dois números sem
levantar os outros cartões. Depois, Bernaldo levanta o último cartão à direita, olha
o número e diz também que não consegue descobrir os dois números à esquerda,
sem levantar todos os cartões. E o mesmo acontece com Cernaldo, que levanta o
cartão do meio, olha seu número e afirma que não consegue descobrir os números
nos outros dois cartões.
Sabendo que todos ouvem o que os demais dizem, mas não vêem o cartão que o
outro olhou, qual número está escrito no cartão do meio?

PROBLEMA 4
Considere a tabela a seguir com quatro linhas (fileiras horizontais) e quatro
colunas (fileiras verticais) a qual está preenchida com números naturais,
ocorrendo repetições de números:

1 0 0 3
5 1 2 4
1 1 2 3
6 1 4 0

Ao somarmos os números de cada uma de suas linhas (L1, L2, L3 e L4) e colunas
(C1, C2, C3 e C4) obtemos 8 números distintos: 3, 4, 7, 8, 10, 11, 12, 13. Veja:

Soma da
C1 C2 C3 C4
linha
L1 1 0 0 3 4
L2 5 1 2 4 12
L3 1 1 2 3 7
L4 6 1 4 0 11
Soma da
13 3 8 10
coluna

EUREKA! N°28, 2008

36
Sociedade Brasileira de Matemática

Apresente, se for possível:


a) uma tabela com 4 linhas e 4 colunas, formada por números naturais, podendo
ocorrer repetições de números, na qual apareçam como somas de linhas ou
colunas os números de 1 a 8.
b) uma tabela com 8 linhas e 8 colunas, formada por números naturais, podendo
ocorrer repetições de números, na qual apareçam como somas de linhas ou
colunas os números de 1 a 16.
c) uma tabela com 9 linhas e 9 colunas, formada por números naturais, podendo
ocorrer repetições de números, na qual apareçam como somas de linhas ou
colunas os números de 1 a 18.

Atenção: caso seja impossível montar alguma tabela, você deve explicar porque.

PROBLEMA 5
Sendo A = 555555
1424 L35 × 222222
1424 L
32 , calcule a soma dos algarismos de
2007 cincos 2007 dois

9 × A . Não se esqueça de justificar a sua resposta.

PROBLEMAS – NÍVEL 2
PROBLEMA 1
Seja ABC um triângulo e O seu circuncentro. Seja ainda P a intersecção das
retas BO e AC e S a circunferência circunscrita a AOP. Suponha que BO = AP e
que a medida do arco OP em S que não contém A é 40°. Determine a medida do
ângulo ∠OBC.
Obs: A circunferência circunscrita de um triângulo é a circunferência que
passa pelos seus vértices e seu centro é chamado de circuncentro.

EUREKA! N°28, 2008

37
Sociedade Brasileira de Matemática

PROBLEMA 2
Considere a tabela a seguir com quatro linhas (fileiras horizontais) e quatro
colunas (fileiras verticais) a qual está preenchida com números naturais,
ocorrendo repetições de números:

1 0 0 3
5 1 2 4
1 1 2 3
6 1 4 0

Ao somarmos cada uma de suas linhas (L1, L2, L3 e L4) e colunas (C1, C2, C3 e
C4) obtemos 8 números distintos: 3, 4, 7, 8, 10, 11, 12, 13. Veja:

C1 C2 C3 C4 Soma da
Linha
L1 1 0 0 3 4
L2 5 1 2 4 12
L3 1 1 2 3 7
L4 6 1 4 0 11
Soma da 13 3 8 10
Coluna

Apresente, se for possível:


a) uma tabela com 4 linhas e 4 colunas, formada por números naturais, podendo
ocorrer repetições de números, na qual apareçam como somas de linhas ou
colunas os números de 1 a 8.
b) uma tabela com 8 linhas e 8 colunas, formada por números naturais, podendo
ocorrer repetições de números, na qual apareçam como somas de linhas ou
colunas os números de 1 a 16.
c) uma tabela com 9 linhas e 9 colunas, formada por números naturais, podendo
ocorrer repetições de números, na qual apareçam como somas de linhas ou
colunas os números de 1 a 18.
Atenção: caso seja impossível montar alguma tabela, você deve explicar porque.

PROBLEMA 3
a 29 − 1
Mostre que existe um inteiro positivo a tal que tem pelo menos 2007
a −1
fatores primos distintos.

EUREKA! N°28, 2008

38
Sociedade Brasileira de Matemática

SEGUNDO DIA

PROBLEMA 4
Prove que não existem soluções inteiras e positivas para a equação
3m + 3n + 1 = t 2 .
PROBLEMA 5
Seja ABC um triângulo retângulo isósceles. K e M são pontos sobre hipotenusa
AB, com K entre A e M, e o ângulo ∠ KCM = 45 o. Prove que AK2 + MB2 = KM2.

PROBLEMA 6
Quadradinhos iguais estão arrumados formando um tabuleiro n × n. Ludmilson e
Ednalva jogam o seguinte estranho jogo. Cada jogada de Ludmilson consiste em
retirar 4 quadradinhos que formem um quadrado 2 × 2. Cada jogada de Ednalva
consiste em retirar apenas 1 quadradinho. Ludmilson e Ednalva jogam
alternadamente, sendo Ludmilson o primeiro a jogar. Quando Ludmilson não
puder fazer sua jogada, então Ednalva fica com todas as peças restantes do
tabuleiro. Ganha o jogo aquele que possuir mais quadradinhos no final. Diga se é
possível que Ednalva ganhe o jogo, não importando como Ludmilson jogue, em
cada um dos seguintes casos:
a) n = 10.
b) Caso geral (n qualquer).

TERCEIRA FASE – NÍVEL 3 (Ensino Médio)


PRIMEIRO DIA

PROBLEMA 1
Seja f(x) = x2 + 2007x + 1. Prove que, para todo n inteiro positivo, a equação
f ( f (K ( f ( x)) K)) = 0 tem pelo menos uma solução real
14243
n vezes

PROBLEMA 2
Para quantos números inteiros c, − 2007 ≤ c ≤ 2007 , existe um inteiro x tal que x2
+ c é múltiplo de 22007

EUREKA! N°28, 2008

39
Sociedade Brasileira de Matemática

PROBLEMA 3
São dados n pontos no plano, os quais são os vértices de um polígono convexo.
Prove que o conjunto das medidas dos lados e das diagonais do polígono tem pelo
menos n / 2 elementos distintos.

Observação: x  denota o maior número inteiro que não excede x. Por exemplo,
2,5 = 2 , 3 = 3 e − 1,2 = −2 .
SEGUNDO DIA

PROBLEMA 4
Arrumam-se 20072 quadradinhos iguais, formando um tabuleiro 2007 × 2007 .
Arnaldo e Bernaldo disputam o seguinte jogo: cada jogada de Arnaldo consiste
em retirar 4 quadradinhos que formem um quadrado 2 × 2 . Cada jogada de
Bernaldo consiste em retirar apenas 1 quadradinho. Os jogadores jogam
alternadamente, sendo Arnaldo o primeiro a jogar. Quando Arnaldo não puder
fazer sua jogada, Bernaldo fica com todas as peças restantes do tabuleiro. Ganha
o jogo aquele que possuir mais quadradinhos no final.
É possível que Bernaldo ganhe o jogo, não importando como Arnaldo jogue?

PROBLEMA 5
Seja ABCD um quadrilátero convexo, P a interseção das retas AB e CD, Q a
interseção das retas AD e BC e O a interseção das diagonais AC e BD. Prove que
se ∠POQ é um ângulo reto então PO é bissetriz de ∠AOD e QO é bissetriz de
∠AOB.

PROBLEMA 6
Dados números reais x1 < x2 < … < xn, suponha que todo número real ocorre no
máximo duas vezes entre as diferenças xj – xi, com 1 ≤ i < j ≤ n . Prove que há
pelo menos n / 2 números reais que ocorrem exatamente uma vez entre tais
diferenças.

Observação: caso você tenha se esquecido da prova de ontem, x  denota o


maior número inteiro que não excede x. Por exemplo, 2,5 = 2 , 3 = 3 e
− 1,2 = −2 .

EUREKA! N°28, 2008

40
Sociedade Brasileira de Matemática

SOLUÇÕES – NÍVEL 1
SOLUÇÃO DO PROBLEMA 1: LIARA GUINSBERG (SÃO PAULO – SP)
Considerando a figura, conseguimos ver um padrão (de cima para abaixo e da
esquerda para a direita).
Número de quadrados pintados:
2×2:2
3×3:7
4×4:8
5 × 5 : 17
6 × 6 : 18
7 × 7 : 31
8 × 8 : 32
Podemos perceber que, do 3 × 3 (7 pintados) para o 4 × 4 (8 pintados) que o
número aumentou 1 unidade pintada.
O fato se deve à seqüência de quadrados pintados, do 2 × 2 para o 3 × 3, o
número de quadrados pretos cresceu em 5 unidades enquanto o branco
permaneceu igual, mas do 3 × 3 para o 4 × 4, o número de brancos aumentou 6,
enquanto o preto somente 1. Em geral, se n é par, do n × n para o (n + 1) × (n + 1)
o número de quadrados pretos cresce em 2n + 1 unidades, mas se n é ímpar cresce
em apenas 1 unidade.
Para o caso do quadrado n × n, com n par, como a quantidade de casas pretas é
n2
igual à quantidade de casas brancas, a quantidade de casas pretas será . Para o
2
caso do quadrado n × n, com n ímpar, percebemos que, a quantidade de casas
(n + 1)2
pretas será − 1 (devido às descobertas anteriores). Com efeito, para n par,
2
( n + 2 ) − 1, e , para n ímpar, (n + 1)2 − 1 + 1 =  n + 1 2 .
2
n2
+ 2n + 1 =  
2 2 2  2 

Usando estes fatos:


142
a) Num quadriculado de 14 × 14, usamos o padrão para pares: = número de
2
196
casas pretas = = 98 casas pretas.
2
b) Para descobrirmos quando o quadrado tem 199 casas pintadas, vamos testar os
casos:

EUREKA! N°28, 2008

41
Sociedade Brasileira de Matemática

n2
Usando o padrão para n par, temos: = 199 ⇔ n 2 = 398, mas e equação não tem
2
solução inteira.
Usando o padrão para n ímpar, vemos que:
(n + 1)2 (n + 1)2
− 1 = 199 ⇔ = 200, achamos (n + 1) = 20, donde n = 19, portanto
2 2
o número de linhas será igual a 19.

SOLUÇÃO DO PROBLEMA 2: CAROLINA RODRIGUES SILVA (FORTALEZA – CE)

(2) (1)

A B
A

Na figura 1 chamamos a área de A de x e a de B de 2x. Teremos então 3x = 81 m2


e x = 27 m2, então a área de A = 27 m2 e seus lados são: 3 e 9; área de B = 54m2 e
seus lados 6 e 9.
Na figura 2, vemos que se juntarmos as áreas descobertas teremos como largura 3
e altura 9 – 3 = 6. Obtemos assim como área do piso que ficou descoberta o
seguinte valor: 6 × 3 = 18m2.

SOLUÇÃO DO PROBLEMA 3: FELIPE BELLIO DA NÓBREGA (RIO DE JANEIRO – RJ)


x, y , z : números nos cartões
vamos supor x< y<z
x + y + z = 13
a) 1 + 2 + 10 2+3+8 3+4+6
1+3+9 2+4+7
1+4+8 2+5+6
1+5+7
b) Quando Arnaldo olha, pode-se eliminar o 3 + 4 + 6, pois ele saberia, já que é o
único que começa com 3.

EUREKA! N°28, 2008

42
Sociedade Brasileira de Matemática

Quando Bernaldo olha, pode-se eliminar o 1 + 2 + 10, o 1 + 3 + 9 e o 2 + 5 + 6. O


primeiro porque é o único que acaba com 10. O segundo com 9. E o último, já
que não pode ser o 3 + 4 + 6 graças a Arnaldo é o único que acaba com 6.
Quando Cernaldo olha, pode-se eliminar o 1 + 5 + 7 e o 2 + 3 + 8. Já que o 2 + 5
+ 6 foi eliminado por Bernaldo, o 1 + 5 + 7 é o único com 5 no meio. E já que
Bernaldo também eliminou o 1 + 3 + 9, o 2 + 3 + 8 é o único com 3 no meio.
Resposta: Assim sobraram apenas o 1 + 4 + 8 e o 2 + 4 + 7. Então o 4 está no
cartão do meio.

SOLUÇÃO DO PROBLEMA 4: RAFAEL KAZUHIRO MIYAZAKI (SÃO PAULO – SP)

a)
Soma da
C1 C2 C3 C4 linha
L1 0 0 0 1 1
L2 5 0 0 0 5
L3 2 4 1 1 8
L4 0 2 2 0 4
Soma da
coluna 7 6 3 2

b)
Soma da
C1 C2 C3 C4 C5 C6 C7 C8 linha
L1 0 0 0 1 0 0 0 0 1
L2 0 0 0 0 4 0 1 0 5
L3 0 3 0 1 0 0 0 0 4
L4 3 0 2 0 0 0 3 0 8
L5 3 0 3 0 2 1 0 0 9
L6 0 0 3 0 1 0 2 7 13
L7 1 0 2 0 3 3 0 7 16
L8 0 0 0 0 1 11 0 0 12
Soma da
coluna 7 3 10 2 11 15 6 14

c) Não é possível. Para que seja possível montar uma tabela, a soma das somas
das colunas e das somas das linhas deve ser igual ao dobro da soma dos números
”internos”(números preenchendo a tabela, exceto os de soma).
1 + 2 + 3 +...+ 16 + 17 + 18 = 171

EUREKA! N°28, 2008

43
Sociedade Brasileira de Matemática

171
2n = 171 ⇒ n = , onde n é a soma dos números “internos” e estes devem ser
2
171
naturais, mas não é natural. Portanto não podemos montar a tabela pedida.
2
SOLUÇÃO DO PROBLEMA 5: SOLUÇÃO DA BANCA
Observamos inicialmente que
9 × 5 × 2 = 9 × 10 = 90
9 × 55 × 22 = 9 × 1210 = 10890
9 × 555 × 222 = 9 × 123210 = 1108890
9 × 5555 × 2222 = 9 × 12343210 = 111088890
9 × 55555 × 22222 = 9 × 1234543210 = 11110888890

Isso nos leva a conjecturar que


9 × A = 9 × 555...555
1424 3 × 222...22
1424 3 = 111...1110888...8890
1424 3 1 424 3
2007 cincos 2007 dois 2006 uns 2006 oitos

Para mostrar que nossa conjectura é verdadeira, devemos garantir que, ao


continuar as multiplicações acima, o padrão se repete. Digamos que você já tenha
feito n multiplicações e tenha obtido
9 × 555...555
1424 3 × 222...22
1424 3 = 111...1110888...8890.
1424 3 1 4243 Então
n cincos n dois n -1 uns n -1 oitos

   
9 × 555...555
1424 3 × 222...22
1424 3 = 9 ×  555...55550
14243 + 5  ×  222...220
1
424 3 + 2 =
n +1 cincos n +1 dois  n cincos   n dois 
9 × 555...55550
14243 1 × 424 3
222...220 + 9 × 14243
555...55550 × 2 + 9 × 5 × 1424 3 +9 ×5 ×2 =
222...220
n cincos n dois n cincos n dois

9 × 555...5555
14243 × 222...2200
1
4243 + 9 × 111...111100
14243 + 9 × 111...111100
14243 + 90 =
n cincos n dois n uns n uns

1424 3 1 424 3
111...1110888...889000 + 2 × 999...9900
1424 3 + 90 =
n −1 uns n −1 oitos n noves

1424 3 1 424 3
111...1110888...889000 + 1999...9800
123 + 90 =
n −1 uns n −1 oitos n −1 noves

1424 3 1 424 3
111...1110888...8890
n uns n oitos

Portanto, nossa conjectura é verdadeira. Logo, a soma dos algarismos de 9 × A é


igual a 2006 × 1 + 2006 × 8 + 9 = 2006 × 9 + 9 = 2007 × 9 = 18063.

EUREKA! N°28, 2008

44
Sociedade Brasileira de Matemática

SOLUÇÕES – NÍVEL 2
PROBLEMA 1: SOLUÇÃO DE HERMANO HENRIQUE DA SILVA (FORTALEZA – CE)

30°

O
120°
80° 60°
30°
20° 80° 100° 20° C
A P
Propriedade do circuncentro:
Está a igual distância dos vértices!
Como O é o circuncentro, AO = BO = AP, logo ∆APO é isósceles e como o
» = 40° ⇒ ∠OAP = 20°, ∠AOP = ∠APO = 80°.
OP
Daí, ∠OPC = 100°, ∠OCP = 20°, ∠POC = 60°. Logo ∠BOC = 120°, mas
∆BOC é isosceles, daí ∠OBC = ∠OCB = 30°.

PROBLEMA 2: Veja a solução do problema No. 4 do Nível 1.

PROBLEMA 3: SOLUÇÃO DA BANCA


Observe a seguinte fatoração
(a 2 )29 − 1 a 29 + 1 a 29 − 1
= ⋅ .
a2 − 1 a +1 a −1
Sabemos que a 29 + 1 = (a + 1)(a 28 − a 27 + a 26 − ... − a + 1) e
a 29 − 1 = ( a − 1)(a 28 + a 27 + ... + a + 1). Dessa forma cada uma das frações
a +1 a −1
29 29
e é inteira.
a +1 a −1
Além disso, se a for par, pelo lema de Euclides:
mdc(a 29 + 1, a 29 − 1) = mdc(a 29 + 1, 2) = 1.

EUREKA! N°28, 2008

45
Sociedade Brasileira de Matemática

 a 29 + 1 a 29 − 1 
Assim, mdc  ,  = 1. Com isso, podemos concluir que, se a for
 a + 1 a −1 
(a 2 )29 − 1
maior que 1, possui pelo menos um divisor primo a mais do que
a2 − 1
a 29 − 1
. Portanto, o número a = 32 satisfaz às condições do problema.
2007

a −1

PROBLEMA 4: SOLUÇÃO DE MATHEUS SECCO TORRES DA SILVA (RIO DE JANEIRO – RJ)


3m + 3n + 1 = t 2 .
Sabe-se que todos os números da forma 3k são ímpares. Assim, 3m + 3n é um
número par obrigatoriamente. Logo, 3m + 3n + 1 é um ímpar. Sendo t 2 um
número ímpar, t também deve ser ímpar, então podemos escrever t = 2k + 1, onde
k é inteiro positivo.
Voltando à equação original, obtemos: 3m + 3n + 1 = 4k 2 + 4k + 1; 3m + 3n = 4k (k + 1).
Pelo princípio da Casa dos Pombos, k(k + 1) é um número par necessariamente,
fazendo com que 4k (k + 1) seja múltiplo de 8.
Devemos ter então 3m + 3n ≡ 0 (mod8). Porém, 3m ≡ 1 ou 3 (mod8) e
3n ≡ 1 ou 3 (mod8). Assim, 3m + 3n ≡ 0 (mod8) é um absurdo!
Por isso, a equação 3m + 3n + 1 = t 2 não tem soluções nos inteiros positivos (c.q.d.)

PROBLEMA 5: SOLUÇÃO DE DEBORAH BARBOSA ALVES (SÃO PAULO – SP)


A

45°
K

a
45°
ß 45°
C a B ≡ A´

EUREKA! N°28, 2008

46
Sociedade Brasileira de Matemática

Girando o ∆AKC em torno de C, até A´≡ B, temos o ∆CK ´ A´ (ou ∆CK ´B ) em


que ∠CA´K ´= ∠CAK = 45°, então ∠MBK ´ é reto.
Sendo ∠ACK = α e ∠MCB = β , ∠BCK ´= ∠ACK = α .
Como ∠ABC é retângulo, e a hipotenusa é AB, ∠ACB é reto. Então
α + β + 45° = 90° ⇔ α + β = 45°.
Como KC = K ´C ; ∠MCK ´= α + β = 45° = ∠KCM e como ∆MCK e
∆MCK´ são congruentes (caso LAL) Então, todos os seus ladas e ângulos são
iguais. Assim, KM = K ´M .
∆ACK é congruente com ∆A´CK ´, por construção.
Então AK = A´K ´= BK ´.
∆MK ´B (ou ∆MK ´ A´) é retângulo.
Então, pelo teorema de Pitágoras, temos:
MB 2 + BK ´2 = MK ´2
e como BK ´= A´K ´= AK e MK ´= KM , AK 2 + MB 2 = KM 2 .

PROBLEMA 6:
BASEADA NA SOLUÇÃO DE JOÃO MENDES VASCONCELOS (FORTALEZA – CE)
n2
a) Se n é par, dividimos o tabuleiro em quadrados 2 × 2. Em cada jogada,
4
Ludmilson retira um quadrado 2 × 2 desses em que dividimos o tabuleiro. Nas
 n2 
primeiras   jogadas, Ednalva retirou quadrados pertencentes a, no máximo,
8
 n2  n2
  desses quadrados 2 × 2. Assim, se k − 1 < , no momento de Ludmilson
8 8
 n2  n 2 n 2 n 2
fazer a k-ésima jogada, foram tocados no máximo k − 1 +   < + =
8 8 8 4
desses quadrados 2 × 2, e portanto sobra algum desses quadrados para Ludmilson
 n2 
retirar. Assim, Ludmilson consegue retirar pelo menos   desses quadrados,
8
 n2  n 2
que contêm 4   ≥ quadrados 1 × 1, ficando com pelo menos a metade dos
8 2
quadradinhos do tabuleiro.

EUREKA! N°28, 2008

47
Sociedade Brasileira de Matemática

 n2  102
Se n = 10, 4   = 4 ⋅ 13 = 52 > , e Ludmilson de fato ganha o jogo.
8 2
Obs.:  x  denota o menor inteiro que é maior ou igual a x

b) Para fazermos o caso geral, dividiremos em casos:

Primeiro caso: n é par:


Como vimos acima, Ludmilson consegue retirar pelo menos metade dos
quadradinhos do tabuleiro, e logo Ednalva não consegue ganhar o jogo. Na
verdade Ludmilson ganha se n for da forma 4k + 2 e o jogo empata se n for da
forma 4k.

Segundo caso: n é ímpar.


Nós faremos uma pintura como segue:
A cada duas linhas, uma ficará em branco e outra será pintada em um
quadradinho sim e um não.
Veja a figura para melhor compreensão:

...

...
n
M M M
...

n
Como n é ímpar, as linhas pintadas terão um quadradinho pintado a menos que os
não pintados. Pelo mesmo motivo, o número de linhas pintadas será uma unidade
menor que o de não pintadas. Isso garante que o número de casas pintadas seja
mínimo e nós possamos ter ao mesmo tempo todos os quadrados 2 × 2 com uma
casa pintada. Agora vamos contar o número de quadrados pintados:
n −1
Em cada linha pintada, nós temos quadrados pintados.
2
n −1 (n − 1)2
Como são linhas pintadas, o total de quadradinhos pintados será .
2 4
EUREKA! N°28, 2008

48
Sociedade Brasileira de Matemática

A estratégia de Ednalva se resume a retirar, a cada jogada, um quadradinho preto


até que não reste mais nenhum. Percebemos também que a cada jogada de
Ludmilson ele também retira um quadradinho preto obrigatoriamente, já que
todos os quadrados 2 × 2 do tabuleiro estão pintados em uma casa.
 (n − 1) 2   (n − 1) 2 
Desse modo, após   jogadas de Ludmilson, e   jogadas de
 8   8 
 (n − 1)2   (n − 1) 2  (n − 1)2
Ednalva, são retiradas  + = casas pintadas, ou seja,
 8   8  4
todas as casas pintadas, e Ludmilson não consegue mais jogar. Como, ao final,
 ( n − 1) 2   ( n − 1)2 1  (n − 1) 2 n2
Ludmilson tem 4   ≤ 4  + = +2< quadradinhos
 8   8 2  2 2
(pois n ≥ 3 nesse caso), Ednalva vence sempre nesse caso.

SOLUÇÕES – NÍVEL 3
PROBLEMA 1: BASEADA NA SOLUÇÃO DE LEANDRO FARIAS MAIA (FORTALEZA – CE)
Sejam f 1 ( x) = f ( x) e para cada n ≥ 1, f n +1 ( x) = f ( f n ( x)).
−2007 + ∆1
Sejam ∆1 = 2007 2 − 4, x1 = .
2
Temos f ( x1 ) = 0. Vamos mostrar por indução que existe uma seqüência de reais
−2007 + ∆n
positivos ( ∆ n ) tal que, definindo xn = , temos f ( xn +1 ) = xn , para
2
todo n , donde f n +1 ( xn +1 ) = f n ( xn ) = 0.
−2007 + ∆ n+1
Para isso, note que a maior raiz de x 2 + 2007 x + 1 = xn é , onde
2
∆ n +1 = 2007 2 − 4 + 4 xn = 2007 2 − 4018 + 2 ∆ n > 0, c.q.d.

PROBLEMA 2: SOLUÇÃO DE RAMON MOREIRA NUNES (FORTALEZA – CE)


Vamos provar que todo número da forma 8q + 1 é resíduo quadrático módulo 2n
(usaremos no que segue a palavra resíduo significando resíduo quadrático) e que
são os únicos resíduos ímpares para n maior ou igual a 3.
Temos que 1 é o único resíduo ímpar módulo 8. De fato,
(2k + 1) 2 = 4k (k + 1) + 1 ≡ 1(mod8), ∀k ∈ ¢. Assim, se n ≥ 3 então todo número
ímpar que é resíduo módulo 2n é congruente a 1 módulo 8.

EUREKA! N°28, 2008

49
Sociedade Brasileira de Matemática

Mostraremos, por indução que todo número da forma 8q + 1 é resíduo mod 2k,
para todo k ≥ 3.
Caso inicial k = 3: 8q +1 é resíduo mod 8 porque 1 é resíduo mod 8.
Passo: Todo número da forma 8q + 1 é resíduo mod 2k; tome x dessa forma.
Então, existe y ∈ ¢ com y 2 ≡ x(mod 2k ). Se y 2 ≡ x(mod 2k +1 ), acabou. Senão,
y2 ≡ x + 2k (mod 2k +1 ), e ( y + 2 k −1 ) = y 2 + 2 k y + 2 2 k − 2 ≡ y 2 + 2 k ≡ x (mod 2 k +1 ),
2

donde x é resíduo módulo 2k +1 , e concluímos a demonstração.


Aprendemos a contar os números ímpares resíduos quadráticos. Como
x2 + c ≡ 0(mod22007 ) para algum x é o mesmo que −c ≡ x2 (mod22007 ) para algum x,
queremos saber o número de c´s tais que – c é resíduo quadrático; bem, entre os
ímpares temos: −2001,..., +2007. Quantos números temos entre eles?
Como −2001 = −8(250) −1 e 2007 = 8 ⋅ 251 – 1, temos 502 ímpares.
Agora para os pares: é claro que c tem que ser múltiplo de 4, pois
x2 + c ≡ 0(mod22007 ) ⇒ x2 + c ≡ 0(mod4) , que só acontece para c múltiplo de 4. Bem,
claro também que x deve ser par, ou seja, x = 2y; queremos
4y2 + 4d ≡ 0(mod22007 ) ⇔ y2 + d ≡ 0(mod22005 ); novamente, sabemos contar os y
ímpares.
Tínhamos os seguintes múltiplos de 4: – 2004, .., 2004; dividindo por 4, ficamos
com: – 501, – 500, ..., 500, 501. Os ímpares da forma –8q – 1 são –497,..., 495;
como –497 = – 8 (62) – 1 e 495 = –8 (–62) – 1, temos 125 ímpares aqui.
Agora, seguindo o algoritmo, pegamos os múltiplos de 4:
– 500,...,500 e vemos quais deles são simétricos de resíduos mod22005.
Dividindo por 4, vemos que eles correspondem aos elementos de – 125,...,125
que são simétricos de resíduos mod 22003.
Encontramos agora os números –8q – 1 entre esses:
Veja que – 121 = –8 (15) – 1 e 119 = –8(–15) – 1; são 31 números aqui.
Múltiplos de 4: – 124, ..., 124, dividindo por 4: – 31,..., 31 (mod 22001 agora)
– 25 = –8 (3) – 1 e 31 = –8(–4) –1; temos 8 números aqui.
Múltiplos de 4: – 28, ..., 28, dividindo por 4: –7,..., 7 (módulo 21999 agora).
–1 = –8(0) – 1 e 7 = –8(–1) –1; temos 2 números aqui.
Múltiplos de 4: –4, 0, 4, dividindo: –1, 0, 1 (mod 21997 agora); desses números
somente –(–1) e –(0) (1 e 0) são resíduos, –(1) não é, logo temos 2 números aqui.
Total: 502 + 125 + 31 + 8 + 2 + 2 = 670 números.

EUREKA! N°28, 2008

50
Sociedade Brasileira de Matemática

PROBLEMA 3: SOLUÇÃO DA BANCA


Primeiro considere dois pontos P e Q do polígono cuja distância é máxima. Tome
Q de modo que PQ separe o polígono em dois polígonos, um deles com PQ como
única distância máxima.
Em cada um desses dois polígonos vamos aplicar o seguinte
Lema: Seja A1 A2 ...Ak um polígono convexo tal que a maior distância entre dois de
seus vértices, incluindo diagonais, é A1 Ak . Então esse polígono tem k – 2
distâncias diferentes; caso A1 Ak seja a única distância máxima, então há k – 1
distâncias diferentes.
Demonstração: Sejam Ap e Aq , 1 < p < q < k dois vértices do polígono. Vamos
provar que, para quaisquer m e n com p < m ≤ n < q um dos segmentos Ap An , Aq Am
é menor do que Ap Aq . Em seguida, conseguiremos uma seqüência de k – 2
distâncias diferentes.
Como conseguir distâncias menores? Ou, de modo mais geral, como compara
segmentos? Muitas vezes é melhor transferir tudo para ângulos, para que
possamos fazer...isso mesmo, um arrastão!
Sejam α = ∠Am A1 Ak ,α1 = ∠A1 Am Ak ,α2 = ∠Ap Am Aq ,α3 = ∠Aq Ap Am , A a interseção de
Ap Aq e A1 Am (note que, como o polígono é convexo, A está no interior do
segmento Ap Aq ) e α4 = ∠Am AAq .

An
Am
ß4
α4 Aq
Ap B
A

α
A1 Ak
Suponha que Ap Aq ≤ Am Aq . Então, no triângulo Am Ap Aq , α2 ≤ α3. Além disso, pelo
teorema do ângulo externo no triângulo AAp Am ,α3 < α4 . Ademais, α1 < α2 e, sendo

EUREKA! N°28, 2008

51
Sociedade Brasileira de Matemática

A1 Ak a maior distância de todas (e esse é o passo decisivo da demonstração e


mostra o poder do princípio do extremo), no triângulo A1 Am Ak ,α < α1. Logo
α < α1 < α2 ≤ α3 < α4 ⇒α < α4 .
Definindo os β´s analogamente e supondo que Ap Aq ≤ An Ap , obtemos β < β4 .
Porém, observando os quadriláteros A1 Ak An Am e ABAn Am , temos que
α + β +∠A1 Am An +∠Ak An Am = α4 + β4 +∠AAm An +∠BAn Am = 360°⇒α + β = α4 + β4 .
Mas
α < α4
⇒α + β < α4 + β4 ,
β < β4
contradição.

O caso em que m = n fica a cargo do leitor.

Para terminar, basta fazer uma espécie de “zigue-zague”. Comece com A2 Ak −1,
que é menor do que A1 Ak (por quê?). Pelo que acabamos de provar, A2 Ak −2 ou
A3 Ak −1 é menor do que A2 Ak −1. Suponha, por exemplo, que A3 Ak −1 seja menor.
Então, aplicando o nosso fato de novo, A4 Ak −1 ou A3 Ak −2 é menor do que A3 Ak −1.
Continuamos assim, até acabar o polígono, e assim conseguimos k – 2 distâncias
diferentes.
No caso em que A1 Ak é a única distância máxima, fica para você provar (use o
poder do arrastrão novamente!) que, no quadrilátero A1 A2 Ak −1 Ak , uma das diagonais
(na verdade as duas) é menor do que A1 Ak (bem, isso é imediato) e maior do que
A2 Ak −1 , de modo que ganhamos mais uma distância, totalizando k– 1.
Agora, vamos terminar o problema. Lembre que cortamos o polígono original do
problema em dois por uma diagonal PQ com medida máxima. Suponha que os
polígonos obtidos tenham k + 1 e n – k + 1 lados, sendo que o de k + 1 lados tem
a distância máxima única. Nele, obtemos (k + 1) – 1 = k distâncias diferentes, e
no outro, (n – k + 1) – 2 = n – k – 1. Então conseguimos d = máx{k, n − k −1}
k + (n − k − 1) n − 1 n
distâncias. Mas d ≥ = ⇒ d ≥  .
2 2 2

PROBLEMA 4: SOLUÇÃO DE HENRIQUE PONDÉ DE OLIVEIRA PINTO (SALVADOR – BA)


Numeremos as casas do tabuleiro de acordo com o seguinte padrão:

EUREKA! N°28, 2008

52
Sociedade Brasileira de Matemática

A B A B A B

C D C D C D

A B A B A B

C D C D C D

A B A B A B

C D C D C D

É fácil ver que cada quadrado 2 × 2 de Arnaldo ocupa exatamente uma casa de
cada tipo (A; B; C e D).
Agora uma contagem simples nos mostra a quantidade de casas de cada tipo.
Tipo Quantidade

A 10042

B 1004 ⋅ 1003

C 1003 ⋅ 1004

D 10032

Veja que a soma total é de fato 20072.


Veja que as casas tipo D são as menos numerosas. Agora suponha que Bernaldo
só jogue em casas tipo D. Teremos que a cada jogada de cada um dos jogadores
exatamente uma casa tipo D é ocupada. Assim após 10032 jogadas Arnaldo não
10032 + 1
poderá mais jogar. Como Arnaldo começa, quando ele fizer sua -ésima
2
jogada acabarão as casas D¸ então Bernaldo pode escolher qualquer casa que
(10032 + 1)
Arnaldo não poderá jogar novamente. Então assim Arnaldo terá 4 ⋅
2
2
2007
casas. Ele só ganha se pegar mais que ou seja
2
4(10032 + 1) 2007 2
> ⇒ 20062 + 4 > 2007 2. Absurdo.
2 2
Logo jogando assim Bernaldo ganha independentemente de como Arnaldo jogar.

PROBLEMA 5: SOLUÇÃO DE RAFAEL TUPYNAMBÁ DUTRA (BELO HORIZONTE - MG)


Sejam E a interseção de OC e PQ e F a interseção de BD e PQ.

EUREKA! N°28, 2008

53
Sociedade Brasileira de Matemática

D
O
B

F P
Q E

PE CD QB
Pelo teorema de Ceva aplicado ao triângulo CPQ, ⋅ ⋅ =1.
QE PD CB
PF CD QB
Pelo teorema de Menelaus aplicado ao triângulo CPQ, ⋅ ⋅ = 1.
QF PD CB
PE PF
Assim, temos = e, portanto, P, Q, E, F formam uma quádrupla
QE QF
harmônica. Assim sendo, OP, OQ, OE, OF formam um feixe harmônico.
Portanto qualquer reta intersecta esse feixe em uma quádrupla harmônica.
Vamos criar uma reta r que passa por Q e é perpendicular a OQ.
O
P

P∞'
E
F’
Q E’ r
F
o
Supondo PÔQ = 90 , provaremos que OQ é bissetriz de FÔE, o que mostra que
OQ é bissetriz de AÔB. Analogamente, teremos OP bissetriz de AÔD (as duas
bissetrizes das retas AC e BD são perpendiculares).
Como PÔQ = 90o, temos OP // r. Assim, r intersecta o feixe harmônico na
quádrupla harmônica P∞' , Q, F’, E’, sendo P∞' o ponto do infinito
correspondente ao feixe de retas paralelas a r. Dessa forma, precisamos ter QE’ =
QF’, ou seja, Q é o ponto médio de E’F’. Assim, pelo teorema de Pitágoras,
temos OF’ = OE’ = OQ 2 + QE ' 2 e, como o triângulo OE’F’ é isósceles, a
altura OQ também é bissetriz de E’ÔF’, de EÔF e de AÔB, como queríamos
demonstrar.

EUREKA! N°28, 2008

54
Sociedade Brasileira de Matemática

Observação: a maioria das soluções utilizou trigonometria ou geometria analítica,


eventualmente com algumas aplicações dos teoremas de Menelaus e de Ceva. A
demonstração de Rafael é bastante interessante por explorar o potencial da
Geometria Projetiva, evitando cálculos. Veja a edição 8 da Eureka! para ver a
teoria utilizada nesse problema.

PROBLEMA 6: SOLUÇÃO DE RÉGIS PRADO BARBOSA (FORTALEZA - CE)


Seja Ai = {x j − xi , i < j ≤ n} . Note que, se a ≠ b, x a − xi ≠ xb − xi pois
x a ≠ xb ; assim, Ai = n − i .
Considere agora Ak ∩ Am com k > m. Se Ak ∩ Am ≥ 2 , então existem a, b, c, d
distintos tais que x a − x k = x b − x m ⇔ x a − xb = x k − x m e
xc − x k = x d − x m ⇔ x c − x d = x k − x m . Assim, x a − xb = x c − x d = x k − x m ,
ou seja, um real aparece três vezes como diferença, um absurdo. Logo
Ak ∩ Am ≤ 1 .
Vamos contar os reais que aparecem duas vezes do seguinte modo: se ele
pertence a Ak e Am , k > m , a contagem é registrada na linha de Ak (ou seja, no
conjunto de maior índice). Façamos então tal contagem, começando de An e indo
até A1 . Isto quer dizer que se o número aparece outra vez em outro conjunto, ele
o faz em um conjunto de índice menor.
Para n par:
Índice Quantidade de elementos Reais que aparecem duas vezes
n An = 0 0
n −1 An−1 = 1 ≤1
M M M
n n n
+1 An / 2+1 = − 1 ≤ −1
2 2 2
n n n
An / 2 = ≤ −1
2 2 2
n n n
−1 An / 2−1 = + 1 ≤ −2
2 2 2
M M M
2 A2 = n − 2 ≤1
1 A1 = n − 1 0

EUREKA! N°28, 2008

55
Sociedade Brasileira de Matemática

Justificando a contagem acima: note que há k −1 ≤ Ak conjuntos com índice


menor do que k. Como Ak ∩ Am ≤ 1 para m < k, há no máximo k – 1 números
que podem se repetidos nos conjuntos de índice menor; ou seja, a quantidade de
novos números de Ak que aparecem duas vezes é menor ou igual a k – 1; os
outros podem aparecer duas vezes, mas eles já foram contados nos conjuntos de
índice maior. Além disso, a quantidade de números de Ak que aparecem duas
vezes é menor ou igual à quantidade total de elementos de Ak . Logo a
quantidade de novos números que aparecem duas vezes é no máximo
{
min Ak , k − 1 . }
Com isso, a quantidade de números que aparecem duas vezes é menor ou igual a
 n 
21 + 2 + 3 + L + − 1 = 2 ⋅
( n
2
− 1) ⋅ n2 n 2 − 2n
= .
 2  2 4
A quantidade dos números que aparecem uma vez pode ser obtida tomando o
n(n − 1)
n
total de elementos ∑ Ai
2
=e subtraindo dele duas vezes a quantidade
i =1
de números que aparecem duas vezes. Sendo d1 a quantidade de números que
aparecem uma vez e d 2 a quantidade de números que aparecem duas vezes,
n(n − 1) n(n − 1) n 2 − 2n n  n 
então d1 = − 2d 2 ≥ − 2⋅ = =  .
2 2 4 2 2
n −1 n 
Analogamente, para n ímpar, prova-se que d1 ≥ =  .
2 2

Nota dos editores: Régis fez o estudo completo do caso n ímpar; porém, o
procedimento é totalmente análogo e foi decidido não colocá-lo aqui.

EUREKA! N°28, 2008

56
Sociedade Brasileira de Matemática

XXIX OLIMPÍADA BRASILEIRA DE MATEMÁTICA


Problemas e Soluções da Primeira Fase – Nível Universitário

PROBLEMA 1:
Joãozinho joga repetidamente uma moeda comum e honesta. Quando a moeda dá
cara ele ganha 1 ponto, quando dá coroa ele ganha 2 pontos.
Encontre a probabilidade (em função de n) de que Joãozinho em algum momento
tenha exatamente n pontos.

PROBLEMA 2:
Dados números reais a1 , a2 ,..., an não todos nulos, encontre o (menor) período
da função
n
f ( x) = ∑ ak cos(kx).
k =1
PROBLEMA 3:
Calcule o volume do sólido definido pelas desigualdades abaixo:
z ≥ 3x2 + 2 y 2 , 3x 2 + 2y 2 + 5 z 2 ≤ 1

PROBLEMA 4:
Seja a um inteiro não nulo.
Prove que se a é uma n-ésima potência modulo 4a2, ou seja, existe um inteiro b
tal que a − bn
é múltiplo de 4a2, então a é uma n-ésima potência.

PROBLEMA 5:
Calcule os autovalores da matriz (n + 1) × (n + 1) abaixo:
0 n 
 
 1 0 n −1 
M = 2 0 O 
 
 O 0 1
 n 0 

Em outras palavras, M i ,i +1 = n + 1 − i, M i +1,i = i, M ij = 0 se i − j ≠ 1.
Obs: Os autovalores de M são as raízes da seguinte equação em x: det(M − xI) =
0.

EUREKA! N°28, 2008

57
Sociedade Brasileira de Matemática

PROBLEMA 6:
Seja y(t) uma função real de variável real tal que
y´´(t ) + et y´(t ) + 3ty (t ) = 2sen(t ) + tg(t ), y (0) = 1, y´(0) = 0.
2

Calcule o limite:
ty´(t )
lim .
t →0 y (t ) − 1

Soluções Nível Universitário


SOLUÇÃO DO PROBLEMA 1:
1
Seja pn a probabilidade pedida. Claramente p0 = 1, p1 = .
2
A probabilidade de que ele nunca tenha n pontos é 1 − pn . Por outro lado, a
única forma de nunca ter n pontos é completar n – 1 pontos e depois tirar coroa.
Assim:
pn −1
1 − pn =
2
2 p 1 1 2
donde − pn = n −1 − =  pn −1 −  e portanto
3 2 3 2 3
n
2  2  1
pn − =  p0 −  ⋅  − 
3  3  2
n
2 1 1
pn = + −  .
3 3 2

SOLUÇÃO DO PROBLEMA 2:
{
Seja m = mdc k ak ≠ 0 . }

Claramente é um período de f: afirmamos que este é o menor período.
m
n
 ak k ak − k 
Escreva f ( x ) = ∑  2 z + 2 z , z = e
k =1  
ix

EUREKA! N°28, 2008

58
Sociedade Brasileira de Matemática

n
 a w k k ak w − k − k 
f ( x + p) = ∑  k z + z  , w = eip
k =1  2 2 
Duas funções racionais só são iguais (ou iguais para números complexos de
módulo 1) se seus coeficientes forem iguais. Assim, se p é um período, temos
kp
ak wk = ak para k = 1,..., n. Em outras palavras ou ak = 0 ou ∈ ¢. .


Equivalentemente, p deve ser um múltiplo inteiro de .
m

SOLUÇÃO DO PROBLEMA 3:
Seja A( a ) a área da elipse 3x + 2 y ≤ a.
2 2

a a πa
Os semieixos da elipse são e donde A( a ) = .
2 3 6
O sólido do problema pode ser descrito como a união disjunta de
21 − 1
3 x 2 + 2 y 2 ≤ z , 0 ≤ z < b, b =
10
1
3z 2 + 2 y 2 ≤ 1 − 5 z 2 , b ≤ z ≤
5
donde
πz 1
π
(1 − 5z ) dz
1
A(1 − 5 z )dz = ∫0
b
v = ∫ A( z )dz + ∫ dz + ∫
b 2
5 2 5
b
0 b 6 6
π b 1 2
1 5  π  31 2 5 7 21 
=  + −b− + b3  =  + − 
6 2 5 3 5 3  6  300 15 100 

SOLUÇÃO DO PROBLEMA 4:
2
Suponha a uma n-ésima potência mod 4a
Escreva a = 2 2 ⋅ 3 3 ... ⋅ p ⋅ ...
e e
p e

Vamos provar que o expoente e p é múltiplo de n.


Segue da hipótese que a = b ⋅ p
ep 2ep
é n-ésima potência módulo p onde mdc(b,
p) = 1.

EUREKA! N°28, 2008

59
Sociedade Brasileira de Matemática

Assim existem c, d com mdc(c, p) = 1,


c n p nd ≡ b ⋅ p p (mod p ) donde nd = e p . Assim
e 2e p
a é uma n-ésima
potência.

Falta provar que se n é par então a > 0.


Suponha por absurdo o contrário: n par, a < 0.
Escreva a = −2 b% , b% ímpar, b% > 0.
nd

nd + 2
Assim a e – a são ambos n-ésimas potências módulo 2 :
c n ⋅ 2nd ≡ −b% ⋅ 2nd (mod 22 nd + 2 ) c n ≡ −b% (mod 2nd + 2 )
b% e −b% são
%c n ⋅ 2nd ≡ b% ⋅ 2nd (mod 22 nd + 2 ) ⇒ c% n ≡ b% (mod 2nd + 2 ) ⇒
quadrados módulo 4 ⇒ –1 é quadrado módulo 4 ⇒ Absurdo!

SOLUÇÃO DO PROBLEMA 5:
Os autovalores são n, n − 2, n − 4,..., − n + 2, − n, ou seja, 2k − n para
k = 0,1,..., n.
t
Vamos exibir os autovetores de M .
Interprete o vetor ( a0 , a1 ,..., an ) ∈ ¡ n +1 como o polinômio
P = a0 x n + a1 x n −1 y + ... + an y n .
∂p ∂p
t
O polinômio correspondente a M ( a0 ,..., an ) é y + x . Se expandirmos
∂x ∂y
os polinômios em u = x+ y e v = x − y este operador passa a ser
∂p ∂p
u −v .
∂u ∂v
 ∂ ∂ 
Mas  u − v  ( u k v n − k ) = ( 2k − n ) u k v n − k .
 ∂u ∂v 
Assim este é o autovetor associado ao autovalor (2k – n).

SOLUÇÃO DO PROBLEMA 6:
Expanda as funções y, e e 2 sen(t ) + tg (t ) em series de potências:
2
t

y (t ) = a0 + a1t + a2 t 2 + ...

EUREKA! N°28, 2008

60
Sociedade Brasileira de Matemática

et = b0 + b1t + b2 t 2 + ...
2

2 sent + tgt = c0 + c1t + c2 t 2 + ...


ty´ a t + 2a2 t 2 + ... + nan t n + ...
Temos lim = lim 1
t →0 y − 1 t → 0 a1t + a2 t 2 + ... + an t n + ...
donde este limite é igual a N se aN ≠ 0 e am = 0 para 0 < m < N .

Substituindo as séries de potências na EDO:


2a2 +6a3t +... +(n+1)(n+2)an+2tn +...
+b0a1 +(ba
1 1 + 2b0a2 )t +... +(bna1 + 2bn−1a2 +... + kb0+1−k ak +... + (n +1)b0an+1)t +3a0t +3at
n
1 +... +
2

+3an−1tn +... = c0 +ct


1 + c2t +... + cnt +... donde
2 n

1
an + 2 = ( cn − bn a1 − 2bn −1a2 − ... − (n + 1)b0 an +1 − 3an −1 )
(n + 1)(n + 2)
Segue facilmente que a1 = a2 = a3 = 0.
Se a1 = a2 = ... = an +1 = 0 e cn = 0 temos an + 2 = 0 e se
a1 = a2 = ... = an +1 = 0 e cn ≠ 0 temos an + 2 ≠ 0. Devemos portanto procurar
° tal que c ° ≠ 0 e c = 0 para 1 < m < N
N °.
N m
3
t t5
Temos sent = t − + + ...
6 120
t 3 2t 5
tan t = t + + + ...
3 15
3t 5
2 sent + tan t = 3t + + ...
20
ty´
Assim N° = 5 donde lim =7.
t →0 y − 1

EUREKA! N°28, 2008

61
Sociedade Brasileira de Matemática

XXIX OLIMPÍADA BRASILEIRA DE MATEMÁTICA


Problemas e Soluções da Segunda Fase – Nível Universitário

PRIMEIRO DIA

PROBLEMA 1:
Considere a função de R em R dada por f(x) = ax2 + bx + c, com a, b, c ∈ ¡ e ac
< 0. Prove que, para todo n inteiro positivo, a equação f ( f (K ( f ( x)) K)) = 0 tem
14243
n vezes
pelo menos uma solução real.

PROBLEMA 2:
Dado um inteiro positivo n, mostre que existe um inteiro positivo N com a
seguinte propriedade: se A é um subconjunto de {1,2, K , N } com pelo menos N/2
elementos, então existe um inteiro positivo m ≤ N − n tal que
k
A ∩ {m + 1, m + 2, K , m + k} ≥
2
para todo k = 1, 2, …, n.

PROBLEMA 3:
Considere o conjunto Pn dos polinômios mônicos de grau n > 0 e coeficientes
complexos p( x) = x n + a n−1 x n−1 + L + a 0 satisfazendo
+ L + a n −1
2 2 2
a0 + a1 =1.

Para p( x) ∈ Pn , seja r ( p (x) ) o máximo entre os módulos das raízes de p(x) e


s (n) = sup r ( p ( x) ) .
p ( x )∈Pn

Determine lim s (n) .


n →∞

SEGUNDO DIA

PROBLEMA 4:
Seja f : ¡ → ¡ uma função contínua tal que f ( f ( x)) = e x para todo x ∈ ¡ .
Prove que, para todo n inteiro positivo,

EUREKA! N°28, 2008

62
Sociedade Brasileira de Matemática

f ( x)
lim = +∞ .
x →+∞ xn

PROBLEMA 5:
Seja A uma matriz real quadrada simétrica de ordem n, e λ1 ≤ λ2 ≤ ⋅⋅⋅ ≤ λn seus
autovalores (contados com multiplicidade). Determine, em função de
λ1 , λ2 , ⋅⋅⋅, λn :
a) O número de matrizes reais B simétricas de ordem n tais que B 2 = A .
b) O número de matrizes reais B de ordem n tais que B 2 = A .

PROBLEMA 6:
Para a, b ∈ ¤ , definimos o conjunto
S (a , b) = {ax 2 + by 2 | x, y ∈ ¤}
dos números racionais que podem ser escritos na forma ax 2 + by 2 com
x, y ∈ ¤ . Dados a, b, c, d racionais não nulos, mostre que S(a,b) = S(c,d) se, e
ab
somente se, é o quadrado de um racional e existe um racional não nulo
cd
q ∈ S ( a , b ) ∩ S (c , d ) ,

SOLUÇÕES

PROBLEMA 1: SOLUÇÃO DE MAURÍCIO RODRIGUES COLLARES NETO (ARACAJU – SE)


1 ) Provemos a afirmação por indução.
Para n = 1 o discriminante da equação do segundo grau b 2 − 4ac é positivo (pois
ac < 0) e, portanto, a equação possui raiz real.
Suponhamos agora a afirmação válida para n. Para provar a afirmação para n + 1,
vejamos que f ( f (...( f ( x))...)) é um polinômio cujo coeficiente do termo líder
1424
3
n +1 vezes
é uma potência ímpar de a (para n = 1 isto é verdade; se isto é válido para n,
temos que f (...( f ( x))) = f (a 2r +1 x k + ...) = a(a 2 r +1 x k + ...)2 + b(a 2 r +1 x k + ...) + c
123
n +1 vezes

que possui termo líder igual a x 2k com coeficiente a 4 r +3 ). Assim, se


a < 0(resp. a > 0), lim f ( f (...( f ( x)) = −∞ (resp. + ∞) (basta colocar o termo
x →+∞ 1424 3
n +1vezes
líder em evidência e verificar que os outros termos vão a zero). Mas

EUREKA! N°28, 2008

63
Sociedade Brasileira de Matemática

f ( f (...( f ( x))...))
1424 3
tem uma raiz r, por hipótese, e
n vezes

f ( f (...( f (r ))...)) = f (0) = c > 0 (resp. < 0). (Pois ac < 0). Assim, pelo
1424 3
n +1 vezes
Teorema do Valor Intermediário, como a função passa de um valor positivo (resp.
negativo) para um valor negativo (resp. positivo) e é contínua, ela tem raiz real:

PROBLEMA 2:
SOLUÇÃO ADAPTADA DA SOLUÇÃO DE RAFAEL DAIGO HIRAMA (S.J. DOS CAMPOS – SP)
Fixe N. Para cada 1 ≤ i ≤ N , defina ai = A ∩ {1,..., i} .
Temos
k  m+k   m
A ∩ {m + 1,..., m + k} = am + k − am ≥ ⇔  am + k −  −  am −  ≥ 0.
2  2   2
i
Isso nos induz a definir bi = ai − , e portanto a desigualdade anterior equivale a
2
bm + k ≥ bm . Assim, queremos mostrar que, se N é suficientemente grande, então
existe m ≤ N − n tal que
bm ≤ bm +1 ,..., bm + n . (I)
É claro que
1 i +1 i +1 i +1 1
bi − = ai − ≤ ai +1 − ≤ (ai + 1) − = bi + , ou seja:
2 2 2 2 2
 1 1
bi +1 ∈ bi − , bi , bi +  .
 2 2
Tome N > n(n + 2) e suponha que (I) não ocorra para cada m ≤ N − n. Em
1 1
particular, para m = 1, existe i1 ∈ {2,..., n + 1} tal que bi1 < b1 ⇒ bi1 ≤ b1 − < .
2 2
Por indução, construímos uma seqüência i1 ,..., in +1 de índices tal que
i j +1 ∈ {i j + 1,..., i j + n} e bi j +1 < bi j . Podemos fazer isso pois N − n > n(n + 1).
1
Daí, bin+1 < bin < ... < bi1 , e, como bi j +1 < bi j ⇒ bi j+1 ≤ bij − , para todo j ≤ n,
2
1 1 1 n
temos bin+1 ≤ bi1 − (n + 1) ≤ − (n + 1) ⋅ = − .
2 2 2 2

EUREKA! N°28, 2008

64
Sociedade Brasileira de Matemática

n
Assim, bin+1 ≤ − .
2
N N
Como A ∩ {1,..., N } = A, aN = A ≥ , donde bN = aN − ≥ 0, e a
2 2
desigualdade anterior garante que in +1 ≤ N − n. Assim, se bm = min{b1 ,..., bN },
n n 1
então bm ≤ − . (pois bN − bm ≥ e b j +1 − b j ≤ para todo j), o que garante
2 2 2
que m ≤ N − n e bm ≤ bm +1 ,..., bm + n .

PROBLEMA 3: SOLUÇÃO DE FÁBIO DIAS MOREIRA (RIO DE JANEIRO – RJ)


Por Cauchy- Schwarz, temos que
 n −1   n −1 2i   n −1
2
i 
2

 ∑ ai  ⋅ ∑ p  ≥  ∑ ai p  .
 i =0   i =0   i =0 
2
α 2n − 1 n −1
Chame α = p ; então 2
α −1
≥ ∑a p
i =0
i
i
. Se p é raiz de p(x) e p > 1, então,
n−1
α 2n −1 2n
como ∑ai pi =−pn , temos
i=0 α −1
2
≥ α ⇔ α 2n −1 ≥ α 2n+2 −α 2n ⇔α 2n (2 −α 2 ) ≥ 1,

logo, como α ≥ 0, α < 2.


Por outro lado, se α 2 = 1 + ε (0 < ε < 1), a desigualdade de Bernoulli diz que
1
α 2 n ≥ 1 + nε , e portanto, para que α 2 n (2 − α 2 ) ≥ 1 ⇔ α 2 n ≥ , é suficiente
1− ε
n −1
que (1 + nε )(1 − ε ) ≥ 1 ⇔ 1 + ( n − 1) ε − nε 2 ≥ 1 ⇔ ε ≤ .
n
n −1 λ 2 −1
Chame λ = 1 + e k= ; defina p(x) tal que ai = − k ⋅ λ i , para
n λ −1
2n

0 ≤ i ≤ n − 1.

2
É fácil verificar que ai = 1. Ademais,
n −1
λ 2n − 1 1
p (λ ) = λ n − ∑ k ⋅ λ 2 i = λ n − k ⋅ = λ n − ≤ 0, já que
i=0 λ −1
2
k

EUREKA! N°28, 2008

65
Sociedade Brasileira de Matemática

1 λ 2n − 1
λ − ≤0⇔λ ≤ 2
n 2n
⇔ λ 2 n (2 − λ 2 ) ≥ 1, o que é verdadeiro pela
k λ −1
n −1
definição de λ (temos λ 2 = 1 + ε , com ε = ). Logo p ( x ) possui uma raiz
n
maior ou igual a λ (já que lim p ( x ) = +∞).
x →+∞

n −1 n −1
Assim, s (n) ≥ 1 + ⇒ lim inf s (n) ≥ lim 1 + = 2. Mas α < 2
n n →∞ n →∞ n
implica s (n) ≤ 2 ⇒ lim sup s( n) ≤ 2 e, finalmente, lim s (n) = 2.
n →∞ n →+∞

PROBLEMA 4: SOLUÇÃO DE FÁBIO DIAS MOREIRA (RIO DE JANEIRO – RJ)


Note inicialmente que f ( f ( f ( x)) = e f ( x ) = f (e x ). Fazendo as substituições
f ( x) e f ( y) lim f ( y ) − ny
x = e , y = e , z = e , temos lim n = lim ny = e y→∞
y z w
.
x →∞ x y →∞ e

y →∞ z →∞ z →∞
(
lim ( f ( y ) − ny ) = lim ( e f ( z ) − ne z ) = lim e z e f ( z )− z − n  . )
Logo basta provar que lim ( f ( z ) − z ) = +∞.
z →∞

Mas
z →∞ w→∞
(
lim ( f ( z ) − z ) = lim e w e f ( w )− w − 1 , ou )
seja, basta que
f ( w) ≥ w + c para c > 0 fixo e todo w suficientemente grande.
Como f ( x) = f ( y ) ⇒ f ( f ( x)) = f ( f ( y )) ⇒ e x = e y ⇒ x = y, f é injetora e,
em virtude de ser contínua, é monótona.
Se f fosse decrescente, como ]0, +∞[ ⊆ Im( f ), teríamos lim f ( x) = +∞ mas,
x →−∞

por outro lado, lim e = lim f ( f ( x)) = 0, logo lim f ( x) = 0. Isso implica
x
x →−∞ x →−∞ x →+∞

Im( f ) = ]0, +∞[ , mas então f ( f (¡)) = ]0, +∞[ = f ( ]0, +∞[ ) = ]0, f (0)[ ,
absurdo!
Logo f é crescente.
Se lim f ( x) = −∞, então lim f ( f ( x)) = −∞ mas lim e x = 0, logo
x →−∞ x →−∞ x →−∞

lim f ( x) = A, onde A < 0 é uma constante (veja que se A ≥ 0 então


x →−∞

f ( f (¡)) = f (]A; +∞[) =] f ( A); +∞[⇒ f ( A) = 0 ⇒ f ( A) = 0 < eA = f ( f ( A)) = f (0),


absurdo).

EUREKA! N°28, 2008

66
Sociedade Brasileira de Matemática

Defina I 0 = ]−∞, A] e I n +1 = f ( I n ); assim I1 = ] A, 0] ,


I 2 =  0, e A  , I 3 =  e A ,1 ... . É fácil ver que os I i ´s formam uma partição de ¡ ,
e logo f ( x) > x, ∀x ∈ ¡.
Considere g ( x) = f ( x) − x. No intervalo ]−∞, 2 A[ , g ( x) > A − 2 A = A . No
intervalo compacto ]2 A,1[ , a função g assume um mínimo positivo. Logo
g : ]−∞,1] → ¡ assume um mínimo positivo, digamos k.
Com isso provamos que f ( x ) ≥ x + k para x ∈ I 0 ∪ I 1 ∪ I 2 ∪ I 3 ; vamos
provar por indução que isso vale para x ∈ I 2 k ∪ I 2 k + 1 para todo k ≥ 2 :
de fato, x = f ( f ( y )) = e y para y ∈ I 2 k − 2 ∪ I 2 k −1 e portanto
f ( x ) = f ( f ( f ( y ))) = e f ( y ) ≥ e y + k = e y ⋅ ek > e y ⋅ (1 + k ) > e y + k , já
que y > 0 ⇒ e y > 1. Mas e y = f ( f ( y )) = x e portanto
f ( x) > x + k , concluindo a demonstração.

PROBLEMA 5: SOLUÇÃO DA BANCA


Primeiro afirmamos que o autoespaço V de A associado a um autovalor λ
V = {v | Av = λ v}
é invariante por B. Suponha v ∈ V . Afirmamos que Bv ∈ V .
A( Bv) = B3v = B( Av) = B(λ v) = λ Bv.
Assim devemos em cada tal autoespaço definir B. Note que a definição de B em
cada autoespaço é independente.
(a) Se existir algum autovalor λ negativo então não existe B pois seus
autovalores µ deveriam satisfazer µ 2 = λ < 0 ⇒ µ ∉¡, o que contradiz a
hipótese de B ser simétrica.
Se existir autovalor zero (λ = 0) com multiplicidade k então B restrita a V deve
ser igual a 0 pois B é diagonalizável e todos os seus autovalores são iguais a 0.
Se existir autovalor positivo (λ > 0) com multiplicidade 1 então há duas
possibilidades correspondentes a µ = λ e µ = − λ .
Se existir autovalor positivo (λ > 0) com multiplicidade maior que 1 então há
infinitas possibilidades pois podemos escolher de infinitas maneiras subespaços
complementares para corresponderem a λ e − λ.
Assim, o número pedido é:

EUREKA! N°28, 2008

67
Sociedade Brasileira de Matemática

0, se algum λ for negativo.


 k
 2 , se nenhum λ for negativo, se todo λ positivo for simples e se houver k

 autovalores positivos.
 ∞, se nenhum λ for negativo e houver pelo menos um autovalor positivo com

 multiplicidade maior que 1.

(b) Os casos λ > 0 são como no item (a).


Se existir autovalor zero com multiplicidade 1 então ainda há apenas uma opção.
Por outro lado, se existir autovalor zero com multiplicidade maior do que 1 então
há infinitas possibilidades para B (pois há infinitas matrizes B com B 2 = 0) .
Se existir autovalor negativo com multiplicidade par (2k) então há infinitas
soluções: basta tomar B = λ ⋅ J , J 2 = − I . Há infinitas tais matrizes J pois
basta definir J em uma base w1 , w2 ,..., wk , wk +1 ,..., w2 k por
Jw j = wk + j , Jwk + j = − w j , j = 1,..., k .

Note que neste caso B tem autovalores λ i e − λ i com multiplicidades k e k.


Se existir autovalor negativo com multiplicidade ímpar então é impossível pois
não há como os autovalores de B virem aos pares conjugados.

Assim, o número pedido é:


0, se existe autovalor negativo com multiplicidade ímpar,
 k
2 , se não existe autovalor negativo, todos os autovalores são simples,

 e há k autovalores positivos.
 ∞, se todos os autovalores negativos têm multiplicidade par e

 existe pelo menos um autovalor com multiplicidade maior do que 1.

PROBLEMA 6: SOLUÇÃO DA BANCA

(
ax 2 + by 2 pode ser visto como x ⋅ a + y ⋅ −b )( x ⋅ )
a − y ⋅ −b , o que
funciona como uma espécie de "norma" de x ⋅ a + y ⋅ −b . Vamos usar o fato
de que o produto ou a razão de dois números dessa forma são da forma
z + w ⋅ − ab , e assim a raiz quadrada que aparece só depende do produto ab.

EUREKA! N°28, 2008

68
Sociedade Brasileira de Matemática

2
z
= r 2 , com r ∈ ¤* , cz 2 + dw2 = c%   + dw2 , onde c% = cr 2 , donde
ab
Se
cd r
% = r 2 cd = ab e S (c% , d ) = S (c , d ). Assim, se ab é o quadrado de um
cd
cd
racional, podemos supor sem perda de generalidade que ab = cd.
 ab  2 ab
Se ax 2 + by 2 = tz 2 +   w é não nulo (se é quadrado, podemos
 t  cd
juntar esse quadrado com uma variável e supor ab = cd), e queremos provar que
 ab  2
qualquer número da forma tu 2 +  v também é da forma ar 2 + bs 2 ,
 t 
escrevemos
 2  ab  2   2  ab  2    ab  
 tz +   w  ⋅  tu +   v  ( ax 2 + by 2 ) ⋅  tu 2 +   v 2 
 ab   t     t    t  
tu 2 +   v 2 =  =  .
 t   2  ab  2   2  ab  2 
 tz +  t  w   tz +  t  w 
       
Abusando de notação, isso é

Acontece que
 −ab 
( ax 2   ab  
( )
+ by2 ) ⋅  tu2 +   v2  N x ⋅ a + y ⋅ −b ⋅ N  u ⋅ t + v ⋅
 t   t 

 =  .
 2  ab  2   −ab 
 tz +  t  w  N  z ⋅ t + w⋅ 
     t 
 − ab 
u ⋅ t + v⋅  uzt + vwab + (vz − uw) −ab
 t = t , que, multiplicado por
 −ab   ab  2
 z ⋅ t + w⋅  tz +   w
2

 t   t 
x ⋅ a + y ⋅ −b , dá um número da forma k ⋅ a + l ⋅ −b , a saber

EUREKA! N°28, 2008

69
Sociedade Brasileira de Matemática

  vwab     vwab  
 x  uzt + t  − by ( vz − uw )  a +  y  uzt + t  + ax ( vz − uw )  −b
        ,
 2  ab  2 
 tz +  t  w 
   
cuja "norma" ak + bl , ou seja,
2 2

2 2
  vwab     vwab  
 x  uzt + t  − by ( vz − uw )   y  uzt + t  + ax ( vz − uw ) 
a     +b    
2 2
 2  ab  2   2  ab  2 
 tz +    w  tz +    w
  t     t  
 ab  2
é igual a tu 2 +   v , como queríamos (isso pode ser verificado diretamente,
 t 
mas chutar essa última expressão seria um pouco de sorte...).
(Note que a condição de que há um valor comum não nulo das formas ax 2 + by 2
e cu 2 + dv 2 é importante. Não é o caso, por exemplo, se a = 2, b = 3, c = 6 e
d = 1).
cd
Vamos agora mostrar a outra implicação. Queremos provar que se não é o
ab
quadrado de um racional então as imagens de ax 2 + by 2 e de cx 2 + dy 2
(quando x e y percorrem os racionais) são diferentes. A imagem de
ax 2 + by 2 com x e y racionais não muda se multiplicarmos a ou b pelo
quadrado de um racional não nulo. Assim, podemos supor que em ax 2 + by 2 e
cx 2 + dy 2 temos a, b, c, d inteiros livres de quadrados. É claro que, se os sinais
de ab e cd forem diferentes, as imagens não podem ser iguais. Assim, nos casos
ab
interessantes, > 0. Basta ver então que cada primo p divide um número par
cd
ab
de números dentre a, b, c, d para concluir que é o quadrado de um racional.
cd
Se p divide exatamente 3 deles, digamos a, b e c, ou seja, a = pk, b = pl, c = pm,
 2  y 
2

temos que ax + by = p(kx + ly ) e cx + dy = pmx + dy = p mx + pd   


2 2 2 2 2 2 2 2
  p  

EUREKA! N°28, 2008

70
Sociedade Brasileira de Matemática

têm a mesma imagem se e só se kx 2 + ly 2 e mz 2 + pdw2 têm a mesma


imagem, e agora p divide exatamente um número dentre k, l, m e pd. Assim,
reduzimos o problema a provar que, se um primo p divide exatamente um dentre
os números a, b, c, d, digamos a, então ax 2 + by 2 e cx 2 + dy 2 não têm a
mesma imagem. Suponhamos por absurdo que tenham. Notemos primeiro que
para quaisquer u e v inteiros, existiriam x e y racionais com
ax 2 + by 2 = cu 2 + dv 2 . Como p | a e a é livre de quadrados, a maior
potência de p que divide ax 2 é ímpar e a maior potência de p que divide by 2 é
par, donde x e y não podem ter p no denominador, senão a maior potência de p
que dividiria ax 2 + by 2 seria negativa e logo ax 2 + by 2 = cu 2 + dv 2 não
poderia ser inteiro. Assim, x e y podem ser vistos como inteiros módulo p, e
cu 2 + dv 2 ≡ by 2 (mod p), donde bcu 2 + bdv 2 ≡ (by ) 2 (mod p) é
quadrado mod p para quaisquer u, v inteiros. Fazendo u = 1, v = 0 temos que bc é
quadrado mod p. Fazendo v = 1, temos que bcu 2 + bd é quadrado mod p para
todo u inteiro, donde, como bc é quadrado mod p, u2 + d/c é quadrado mod p para
todo u inteiro (note que b, c e d são invertíveis mod p). Ou seja, se r é quadrado
d kd
mod p então r + também é, mas isso implica por indução que r + é
c c
quadrado mod p para todo k natural, donde todo inteiro é quadrado mod p. Isso só
é possível se p = 2.
Se p = 2 dividir um número par de números dentre a, b, c, d teremos que todo
primo p divide um número par de números dentre a, b, c, d, como queríamos.
Caso contrário, teremos ainda algum trabalho extra, que realizaremos a seguir.
Podemos supor como antes que 2 divide a mas não divide bcd. Sejam
r = mdc(c, d ) e K o produto dos primos ímpares que dividem ab mas não
c d
dividem cd. Sejam m = e n = . Se n (que, como d, é ímpar) for
r r
congruente a 3 ou –3 módulo 8, tomaremos R = (4rK ) 2 ⋅ c + d , e se n for
congruente a 1 ou –1 módulo 8, tomaremos R = (2rK ) 2 ⋅ c + d . Temos em
R
qualquer caso que R pertence à imagem de cx 2 + dy 2 . Além disso, é
r
congruente a 3 ou –3 módulo 8 e é primo com m, n, r e K, e portanto é primo com
R
a, b, c e d. tem que ter algum fator primo q congruente a 3 ou –3 módulo 8
r
que aparece com expoente ímpar em sua fatoração (pois um produto de números

EUREKA! N°28, 2008

71
Sociedade Brasileira de Matemática

que são 1 ou –1 módulo 8 ainda é dessa forma). Temos que R é um número da


forma c ⋅ x 2 + d com x inteiro, e portanto c ⋅ x 2 + d = R = 0(mod q ),
donde −cd = (cx) 2 (mod q ). Por outro lado, se as imagens são iguais, existem
U V
u e v racionais com au 2 + bv 2 = R. Podemos escrever u = e v =
D D
onde D é o menor denominador comum de u e v. Temos então
aU 2 + bV 2 = R ⋅ D 2 = 0(mod q). Se q j é a maior potência de q que divide
U e V simultaneamente, escrevemos U = q j ⋅ T e V = q j ⋅ S obtendo então
R
aT 2 + bS 2 = R ⋅ q −2 j ⋅ D 2 , que ainda é múltiplo de q. Como q , q é primo
r
com a e b, e logo q não pode dividir T, caso contrário q dividiria bS2, donde q
dividiria também S, contradizendo a escolha de j. Assim,
− ab ⋅ T = (bS ) (mod q ) implica que –ab é um quadrado módulo q.
2 2

ab (− ab)
Portanto, = também é um quadrado módulo q, mas, pelas
cd (−cd )
ab
considerações anteriores, = 2 ⋅ w2 , para algum racional w, e daí seguiria
cd
que 2 é quadrado módulo q, o que é um absurdo, pois q é congruente a 3 ou –3
módulo 8 (veja o artigo “ Reciprocidade quadrática”, de Carlos Gustavo Moreira
e Nicolau Saldanha, na Eureka! No. 15).

Errata: Na Eureka! No. 27, no artigo “Substituições envolvendo números


complexos”, de Diego Veloso Uchoa, na página 21, o trecho entre as linhas 12 e
15 deveria ser:

Fazendo n = 2m + 1 e igualando as partes imaginárias, temos:


sen((2m + 1)t )  2m + 1  2m + 1  2 m −1
2 m +1
=  (cos t ) − 
2 m
 (cos t ) + ... + ( −1) . (*)
m

sen t  1   3 
Agora podemos tratar essa igualdade por meio do polinômio
 2m + 1 m  2m + 1 m −1
Pm ( x) =  x −  x + ... + (−1) .
m

 1   3 

EUREKA! N°28, 2008

72
Sociedade Brasileira de Matemática

XXIX Olimpíada Brasileira de Matemática


Nível 1 (5ª. e 6ª. Séries)
Nome Cidade – Estado Prêmio
Rafael Kazuhiro Miyazaki São Paulo – SP Ouro
Guilherme Renato Martins Unzer São Paulo – SP Ouro
Marina Pessoa Mota Fortaleza – CE Ouro
Ivan Tadeu Ferreira Antunes Filho Lins – SP Ouro
Danilo Hikari Motoyama Watanabe São Paulo – SP Ouro
Arthur Oenning Fagundes Palmas – TO Prata
Breno Levi Correa Campo Belo – MG Prata
Thomás Rincon Reis Belo Horizonte – MG Prata
Lucas Finger Roman Florianópolis – SC Prata
Lucas Nishida Pedreira – SP Prata
Ana Cristina Barreto Sabino de Araújo Itapissuma – PE Prata
Gabriel Santa Rosa Cavaresi Birigüi – SP Prata
Victor Kioshi Higa São Paulo – SP Prata
Ana Beatrice Bonganha Zanon Santo André – SP Prata
Maria Paula Silva Serrasqueiro Brasília – DF Prata
Débora Barreto Ornellas Salvador – BA Bronze
Igor Araújo Rio de Janeiro – RJ Bronze
Pedro Ivo Coelho de Araújo Caucaia – CE Bronze
Ramon Silva de Lima São Paulo – SP Bronze
Dênnis Dantas de Souza Campina Grande – PB Bronze
Nathália Roscoe e Firace Belo Horizonte – MG Bronze
Renan Fernandes Moreira Taubaté – SP Bronze
Nicolas Seoane Miquelin Mauá – SP Bronze
Nicolas Fernandez Leitão Florianópolis – SC Bronze
Tiago Sueda Limone Jundiaí – SP Bronze
Gabriel Pacianotto Gouveia São Paulo – SP Bronze
Jonathan Henrique de Oliveira Cordeirópolis – SP Bronze
Murilo Dória Guimarães São Paulo – SP Bronze
Julio Barros de Paula Taubaté – SP Bronze
Cesar Nobuo Moniwa Ishiuchi Campinas – SP Bronze
Danilo Kenji Shido São Paulo – SP Menção Honrosa
Francisco Markan Nobre de Souza Filho Fortaleza – CE Menção Honrosa
Natália Rodrigues Parrode Goiânia – GO Menção Honrosa
João Felipe Ribeiro Soares Brasília – DF Menção Honrosa
Sofia Sayuri Yamamura Araçatuba – SP Menção Honrosa
Paula Dias Garcia Brasília – DF Menção Honrosa
Lara Timbó Araújo Fortaleza – CE Menção Honrosa
Nathalia Novello Fernandes Ribeiro Rio de Janeiro – RJ Menção Honrosa
Eric Luiz Rodrigues de França Recife – PE Menção Honrosa
Pedro Ducci Serafim Campinas – SP Menção Honrosa
Lucas Bitran Giestas Vitória – ES Menção Honrosa
Wederson Santos Silva Massaranduba – PB Menção Honrosa
Ayrton Barros de Lira Recife – PE Menção Honrosa
Leonardo Kazunori Tsuji São Paulo – SP Menção Honrosa
Lucas Guedes de Almeida Rocha Maceió – AL Menção Honrosa
Liang Wei Dong Salvador – BA Menção Honrosa
Rafael Wingester Ribeiro de Oliveira Belo Horizonte – MG Menção Honrosa
Hugo Diehl de Souza Criciúma – SC Menção Honrosa
Matheus de Oliveira Leão Teresina – PI Menção Honrosa
Rodolfo Vieira Fontenele Cocal dos Alves – PI Menção Honrosa
Henrique Gasparini Fiúza do Nascimento Brasília – DF Menção Honrosa
Victor Venturi Campinas – SP Menção Honrosa
Gabrielle Macanhan Guimarães Anápolis – GO Menção Honrosa
Reinaldo Abad Junior Guarulhos – SP Menção Honrosa
Henrique Vieira G. Vaz São Paulo – SP Menção Honrosa
Gabriela Loiola Vilar Fortaleza – CE Menção Honrosa
Igor Tetsuo Boninsenha Kunizaki Taubaté – SP Menção Honrosa
Marcelo Cargnelutti Rossato Santa Maria – RS Menção Honrosa
Arthur Ferreira do Nascimento São Paulo – SP Menção Honrosa
Liara Guinsberg São Paulo – SP Menção Honrosa
Filipe Bellio da Nóbrega Rio de Janeiro – RJ Menção Honrosa
Israel Rodrigues Soares Goiânia – GO Menção Honrosa
Matheus Carneiro Campagnani Niterói – RJ Menção Honrosa

EUREKA! N°28, 2008

73
Sociedade Brasileira de Matemática

Nível 2 (7a. e 8a. Séries)


Nome Cidade – Estado Prêmio
João Mendes Vasconcelos Fortaleza – CE Ouro
Matheus Barros de Paula Taubaté – SP Ouro
Gabriel Militão Vinhas Lopes Fortaleza – CE Ouro
Thiago Saksanian Hallak São Paulo – SP Ouro
Paulo Henrique Dias Vieira Rio de Janeiro – RJ Ouro
João Lucas Camelo Sá Fortaleza – CE Prata
Ana Beatriz Prudêncio de Almeida Rebouças Fortaleza – CE Prata
Hanon Guy Lima Rossi São Paulo – SP Prata
Danilo Silva de Albuquerque Fortaleza – CE Prata
Felipe Vieira de Paula Fortaleza – CE Prata
Leonardo Ferreira Patrício Rio de Janeiro – RJ Prata
Deborah Barbosa Alves São Paulo – SP Prata
Vinicius Cipriano Klein Venda Nova do Imigrante – ES Prata
Fernando Fonseca Andrade Oliveira Belo Horizonte – MG Prata
Maria Clara Mendes Silva Pirajuba – MG Prata
Ruan Alves Pires Rio de Janeiro – RJ Bronze
Natan Lima Viana Fortaleza – CE Bronze
Gleycianne Arruda de Freitas Silva Fortaleza – CE Bronze
Matheus Secco Torres da Silva Rio de Janeiro – RJ Bronze
Felipe Mostavenco Carmo Rio de Janeiro – RJ Bronze
Jonas Rocha Lima Amaro Fortaleza – CE Bronze
Gustavo Lisbôa Empinotti Florianópolis – SC Bronze
Guilherme da Rocha Dahrug Santo André – SP Bronze
Victorio Takahashi Chu São Paulo – SP Bronze
Itamar Sales de Oliveira Filho Cedro – CE Bronze
Francisco Vagner Dantas Leite Filho Fortaleza – CE Bronze
Kayo de França Gurgel Fortaleza – CE Bronze
Rodrigo Rolim Mendes de Alencar Fortaleza – CE Bronze
Igor Rosiello Zenker São Paulo – SP Bronze
Daniel Lucas Filgueira Fortaleza – CE Bronze
Mario Valney Pereira de Andrades Fortaleza – CE Bronze
Rafael Dias da Fonseca Maceió – AL Bronze
Matheus Cordeiro Wilhelm da Costa Rio de Janeiro – RJ Menção Honrosa
Elder Massahiro Yoshida São Paulo – SP Menção Honrosa
Lucas de Freitas Smaira Guaxupé – MG Menção Honrosa
Léo Nunes Benevides Fortaleza – CE Menção Honrosa
Felipe Bento Vargas de Moraes Rio de Janeiro – RJ Menção Honrosa
Rubens Cainan Sabóia Monteiro Fortaleza – CE Menção Honrosa
Alessandro Macêdo de Araújo Fortaleza – CE Menção Honrosa
Sandoel de Brito Vieira Cocal dos Alves – PI Menção Honrosa
Rafael Ferreira Antonioli S. B. do Campo – SP Menção Honrosa
Leonardo Victor Maciel Pontes Fortaleza – CE Menção Honrosa
Bryan Levy Salinas Carrillo São Paulo – SP Menção Honrosa
Filipe José Oliveira Sabóia Fortaleza – CE Menção Honrosa
Débora Jun Portugheis Campinas – SP Menção Honrosa
Kelve Torres Henrique Recife – PE Menção Honrosa
Nicolás Francisco E. C. Hespanhol Santos Bauru – SP Menção Honrosa
André Austregesilo Scussel Fortaleza – CE Menção Honrosa
Álvaro Lopes Pedroso Santa Isabel – SP Menção Honrosa
Wellington Biing Jung Lee São Paulo – SP Menção Honrosa
Bruno César da Silva Guedes Recife – PE Menção Honrosa
Luiz Filipe Martins Ramos Niterói – RJ Menção Honrosa
Jéssica Kazumi Okuma São Paulo – SP Menção Honrosa
Pedro Vieira Rodrigues Serradas Rio de Janeiro – RJ Menção Honrosa
Leonardo Henrique Caldeira Pires Ferrari Rio de Janeiro – RJ Menção Honrosa
Carlos Henrique de Andrade Silva Fortaleza – CE Menção Honrosa
Gregory Cosac Daher Rio de Janeiro – RJ Menção Honrosa

EUREKA! N°28, 2008

74
Sociedade Brasileira de Matemática

Nível 3 (Ensino Médio)


Nome Cidade – Estado Prêmio
Rafael Tupynambá Dutra Belo Horizonte – MG Ouro
Régis Prado Barbosa Fortaleza – CE Ouro
Ramon Moreira Nunes Fortaleza – CE Ouro
Henrique Pondé de Oliveira Pinto Salvador – BA Ouro
Henrique Hiroshi Motoyama Watanabe São Paulo – SP Ouro
Adenilson Arcanjo de Moura Junior Fortaleza – CE Prata
Renan Henrique Finder São Paulo – SP Prata
Guilherme Philippe Figueiredo São Paulo – SP Prata
Marco Antonio Lopes Pedroso Santa Isabel – SP Prata
Rafael Sampaio de Rezende Fortaleza – CE Prata
Giuliano Pezzolo Giacaglia Santo André – SP Prata
Jorge Henrique Craveiro de Andrade Rio de Janeiro – RJ Prata
Marcelo Matheus Gauy S.J. do Rio Preto – SP Prata
Mateus Oliveira de Figueiredo Fortaleza – CE Prata
Paulo Sérgio de Castro Moreira Fortaleza – CE Prata
Robério Soares Nunes Ribeirão Preto – SP Prata
Marlen Lincoln da Silva Fortaleza – CE Bronze
Esdras Muniz Mota Fortaleza – CE Bronze
Grazielly Muniz da Cunha Fortaleza – CE Bronze
Davi Lopes Alves de Medeiros Fortaleza – CE Bronze
Gabriel Luís Mello Dalalio S. J. dos Campos – SP Bronze
José Airton Coêlho Lima Filho Fortaleza – CE Bronze
Leandro Farias Maia Fortaleza – CE Bronze
Marcos Victor Pereira Vieira Fortaleza – CE Bronze
Alfredo Roque de Oliveira Freire Filho S. J. dos Campos – SP Bronze
Francisco Osman Pontes Neto Fortaleza – CE Bronze
Leonel Lopes Lima Neto Maceió – AL Bronze
Renan Braz Parente Fortaleza – CE Bronze
Alex Atsushi Takeda Londrina – PR Bronze
Marcelo Tadeu de Sá Oliveira Sales Salvador – BA Bronze
Thiago Ribeiro Ramos Varginha – MG Bronze
Luiz Paulo Freire Moreira Fortaleza – CE Menção Honrosa
Antônio Felipe Cavalcante Carvalho Fortaleza – CE Menção Honrosa
Luca Mattos Möller Niterói – RJ Menção Honrosa
Hugo Fonseca Araújo Juiz de Fora – MG Menção Honrosa
Fernando Nascimento Coelho Fortaleza – CE Menção Honrosa
Filipe de Almeida Araujo Vital Rio de Janeiro – RJ Menção Honrosa
Illan Feiman Halpern Itatiaia – RJ Menção Honrosa
Alexandre Nobuo Kunieda São Paulo – SP Menção Honrosa
Alysson Espíndola de Sá Silveira Fortaleza – CE Menção Honrosa
Thiago S. Pinheiro São Paulo – SP Menção Honrosa
Orlando Alencar Lustosa Neto Fortaleza – CE Menção Honrosa
Ricardo Turolla Bortolotti Rio Claro – SP Menção Honrosa
Gustavo Pacianotto Gouveia São Paulo – SP Menção Honrosa
Felipe Holanda Moreira Fortaleza – CE Menção Honrosa
Artur de Almeida Losnak São Paulo – SP Menção Honrosa
Rafael Parpinel Cavina São Paulo – SP Menção Honrosa
Filipe Alves Tomé Fortaleza – CE Menção Honrosa
Custodio Moreira Brasileiro Silva Caém – BA Menção Honrosa
Marília Valeska Costa Medeiros Fortaleza – CE Menção Honrosa
Pollyanna Stéfani Borges Freitas Fortaleza – CE Menção Honrosa
Gustavo Sampaio Sousa Fortaleza – CE Menção Honrosa
Joas Elias dos Santos Rocha Muribeca – SE Menção Honrosa
Raphael Luiz França Greco Rio de Janeiro – RJ Menção Honrosa
Rafael Morioka Oda São Paulo – SP Menção Honrosa

EUREKA! N°28, 2008

75
Sociedade Brasileira de Matemática

Nível Universitário
Nome Cidade – Estado Prêmio
Fábio Dias Moreira Rio de Janeiro – RJ Ouro
Rafael Marini Silva Vila Velha – ES Ouro
Guilherme Rodrigues Nogueira de Souza São Paulo – SP Ouro
José Marcos Andrade Ferraro São Paulo – SP Ouro
Rafael Daigo Hirama Campinas – SP Ouro
Eduardo de Moraes Rodrigues Poço São Paulo – SP Prata
Felipe Rodrigues Nogueira de Souza São Paulo – SP Prata
Murilo Vasconcelos Andrade Maceió – AL Prata
Leonardo Ribeiro de Castro Carvalho São Paulo – SP Prata
Luty Rodrigues Ribeiro S.J. dos Campos – SP Prata
André Linhares Rodrigues Campinas – SP Prata
Maurício de Lemos Rodrigues Collares Neto Aracaju – SE Prata
Henry Wei Cheng Hsu São Paulo – SP Prata
Kellem Correa Santos Rio de Janeiro – RJ Prata
Levi Maximo Viana Rio de Janeiro – RJ Bronze
Ronaldo Rodrigues Pelá São Carlos – SP Bronze
Luís Daniel Barbosa Coelho Rio de Janeiro – RJ Bronze
Thiago Costa Leite Santos São Paulo – SP Bronze
Helder Toshiro Suzuki São Paulo – SP Bronze
Raphael Constant da Costa Rio de Janeiro – RJ Bronze
Rafael Sabino Lima Rio de Janeiro – RJ Bronze
Erick Costa e Silva Talarico Rio de Janeiro – RJ Bronze
Rodrigo Aguiar Pinheiro S.J. dos Campos – SP Bronze
Renato Rebouças de Medeiros S.J. dos Campos – SP Bronze
José Armando Barbosa Filho S.J. dos Campos – SP Bronze
Evandro Makiyama de Melo São Paulo – SP Bronze
Tiago Barbin Batalhão São Carlos – SP Bronze
Gabriel Ponce São Carlos – SP Bronze
Vitor Gabriel Kleine S.J. dos Campos – SP Bronze
Alexandre Hideki Deguchi Martani São Paulo – SP Bronze
Vitor Humia Fontoura Rio de Janeiro – RJ Bronze
Ana Maria Menezes de Jesus Itabaiana – SE Bronze
Eduardo Fischer Encantado – RS Bronze
Anderson Hoshiko Aiziro São Paulo – SP Bronze
Daniel Lopes Alves de Medeiros S.J. dos Campos – SP M. Honrosa
Paulo André Carvalho de Melo Rio de Janeiro – RJ M. Honrosa
Pedro Meira de Vasconcellos Bezerra Recife – PE M. Honrosa
Willy George do Amaral Petrenko Rio de Janeiro – RJ M. Honrosa
Gustavo Antônio da Silva Amaro São Carlos – SP M. Honrosa
Ricardo Monteiro da Silva Lanna Belo Horizonte – MG M. Honrosa
Felipe Gonçalves Assis Campina Grande – PB M. Honrosa
Elder Rodrigo Barbosa Campos Rio de Janeiro – RJ M. Honrosa
Matheus Pimentel Rodrigues Rio de Janeiro – RJ M. Honrosa
Rafael Montezuma Pinheiro Cabral Fortaleza – CE M. Honrosa
Nivan Roberto Ferreira Júnior Olinda – PE M. Honrosa
Elton Gomes Coriolano Campinas – SP M. Honrosa
Thomás Yoiti Sasaki Hoshina Rio de Janeiro – RJ M. Honrosa
Samir Rodrigues Vieira Fortaleza – CE M. Honrosa
Frederico de Souza Frydman S.J. dos Campos – SP M. Honrosa
Jordan Freitas Piva Rio de Janeiro – RJ M. Honrosa
Rodrigo Viana Soares Fortaleza – CE M. Honrosa
Bruno Euzébio dos Santos Malhados – SE M. Honrosa
Antonia Taline de Souza Mendonça Rio de Janeiro – RJ M. Honrosa

EUREKA! N°28, 2008

76
Sociedade Brasileira de Matemática

AGENDA OLÍMPICA
XXX OLIMPÍADA BRASILEIRA DE MATEMÁTICA

NÍVEIS 1, 2 e 3
Primeira Fase – Sábado, 14 de junho de 2008
Segunda Fase – Sábado, 13 de setembro de 2008
Terceira Fase – Sábado, 25 de outubro de 2007 (níveis 1, 2 e 3)
Domingo, 26 de outubro de 2008 (níveis 2 e 3 - segundo dia de prova).

NÍVEL UNIVERSITÁRIO
Primeira Fase – Sábado, 13 de setembro de 2008
Segunda Fase – Sábado, 25 e Domingo, 26 de outubro de 2008

XIV OLIMPÍADA DE MAIO


10 de maio de 2008

XIX OLIMPÍADA DE MATEMÁTICA DO CONE SUL
Temuco – Chile
18 a 23 de junho de 2008

XLIX OLIMPÍADA INTERNACIONAL DE MATEMÁTICA
10 a 22 de julho de 2008
Madri – Espanha

XIV OLIMPÍADA INTERNACIONAL DE MATEMÁTICA UNIVERSITÁRIA
25 a 31 de julho de 2008
Blagoevgrad, Bulgária

XXIII OLIMPÍADA IBEROAMERICANA DE MATEMÁTICA
18 a 28 de setembro de 2008
Salvador, Bahia – Brasil

XI OLIMPÍADA IBEROAMERICANA DE MATEMÁTICA UNIVERSITÁRIA

EUREKA! N°28, 2008

77
Sociedade Brasileira de Matemática

COORDENADORES REGIONAIS
Alberto Hassen Raad (UFJF) Juiz de Fora – MG
Américo López Gálvez (USP) Ribeirão Preto – SP
Amarísio da Silva Araújo (UFV) Viçosa – MG
Andreia Goldani FACOS Osório – RS
Antonio Carlos Nogueira (UFU) Uberlândia – MG
Ali Tahzibi (USP) São Carlos – SP
Benedito Tadeu Vasconcelos Freire (UFRN) Natal – RN
Carlos Alexandre Ribeiro Martins (Univ. Tec. Fed. de Paraná) Pato Branco – PR
Carmen Vieira Mathias (UNIFRA) Santa María – RS
Claus Haetinger (UNIVATES) Lajeado – RS
Cleonor Crescêncio das Neves (Inst. de Tec. e Educ. Galileo da Amazônia) Manaus – AM
Cláudio de Lima Vidal (UNESP) S.J. do Rio Preto – SP
Denice Fontana Nisxota Menegais (UNIPAMPA) Bagé – RS
Edson Roberto Abe (Colégio Objetivo de Campinas) Campinas – SP
Élio Mega (Faculdade Etapa) São Paulo – SP
Eudes Antonio da Costa (Univ. Federal do Tocantins) Arraias – TO
Fábio Brochero Martínez (UFMG) Belo Horizonte – MG
Florêncio Ferreira Guimarães Filho (UFES) Vitória – ES
Francinildo Nobre Ferreira (UFSJ) São João del Rei – MG
Genildo Alves Marinho (Centro Educacional Leonardo Da Vinci) Taguatingua – DF
Ivanilde Fernandes Saad (UC. Dom Bosco) Campo Grande – MS
Jacqueline Rojas Arancibia (UFPB)) João Pessoa – PB
Janice T. Reichert (UNOCHAPECÓ) Chapecó – SC
João Benício de Melo Neto (UFPI) Teresina – PI
João Francisco Melo Libonati (Grupo Educacional Ideal) Belém – PA
Jose de Arimatéia Fernandes (UFPB) Campina Grande – PB
José Luiz Rosas Pinho (UFSC) Florianópolis – SC
José Vieira Alves (UFPB) Campina Grande – PB
José William Costa (Instituto Pueri Domus) Santo André – SP
Krerley Oliveira (UFAL) Maceió – AL
Licio Hernandes Bezerra (UFSC) Florianópolis – SC
Luciano G. Monteiro de Castro (Sistema Elite de Ensino) Rio de Janeiro – RJ
Luzinalva Miranda de Amorim (UFBA) Salvador – BA
Mário Rocha Retamoso (UFRG) Rio Grande – RS
Marcelo Rufino de Oliveira (Grupo Educacional Ideal) Belém – PA
Marcelo Mendes (Colégio Farias Brito, Pré-vestibular) Fortaleza – CE
Newman Simões (Cursinho CLQ Objetivo) Piracicaba – SP
Nivaldo Costa Muniz (UFMA) São Luis – MA
Osnel Broche Cristo (UFLA) Lavras – MG
Osvaldo Germano do Rocio (U. Estadual de Maringá) Maringá – PR
Raul Cintra de Negreiros Ribeiro (Colégio Anglo) Atibaia – SP
Ronaldo Alves Garcia (UFGO) Goiânia – GO
Rogério da Silva Ignácio (Col. Aplic. da UFPE) Recife – PE
Reginaldo de Lima Pereira (Escola Técnica Federal de Roraima) Boa Vista – RR
Reinaldo Gen Ichiro Arakaki (UNIFESP) SJ dos Campos – SP
Ricardo Amorim (Centro Educacional Logos) Nova Iguaçu – RJ
Sérgio Cláudio Ramos (IM-UFRGS) Porto Alegre – RS
Seme Gebara Neto (UFMG) Belo Horizonte – MG
Tadeu Ferreira Gomes (UEBA) Juazeiro – BA
Tomás Menéndez Rodrigues (U. Federal de Rondônia) Porto Velho – RO
Valdenberg Araújo da Silva (U. Federal de Sergipe) São Cristovão – SE
Vânia Cristina Silva Rodrigues (U. Metodista de SP) S.B. do Campo – SP
Wagner Pereira Lopes (CEFET – GO) Jataí – GO

EUREKA! N°28, 2008

78
CONTEÚDO
XIV OLIMPÍADA DE MAIO 2

XLVIII OLIMPÍADA INTERNACIONAL DE MATEMÁTICA 5


Enunciados e Resultado Brasileiro

XLIX OLIMPÍADA INTERNACIONAL DE MATEMÁTICA 7


Enunciados e Resultado Brasileiro

XXII OLIMPÍADA IBEROAMERICANA DE MATEMÁTICA 9


Enunciados e Resultado Brasileiro

XXIII OLIMPÍADA IBEROAMERICANA DE MATEMÁTICA 12


Enunciados e Resultado Brasileiro

ARTIGOS
0,999... OU “COMO COLOCAR UM BLOCO QUADRADO EM UM BURACO REDONDO” 14
Pablo Emanuel

ALGORITMO DE GOSPER E APLICAÇÕES 21


Humberto Silva Naves

DOMINGO REGADO A REPUNITS 27


Valberto Rômulo Feitosa Pereira

HOMOTETIAS, COMPOSIÇÃO DE HOMOTETIAS EO PROBLEMA 6 DA IMO 2008 32


Carlos Yuzo Shine

COMO É QUE FAZ? 44

OLIMPÍADAS AO REDOR DO MUNDO 48

SOLUÇÕES DE PROBLEMAS PROPOSTOS 52

PROBLEMAS PROPOSTOS 62

AGENDA OLÍMPICA 63

COORDENADORES REGIONAIS 64
Sociedade Brasileira de Matemática

XIV OLIMPÍADA DE MAIO


PRIMEIRO NÍVEL
PROBLEMA 1
Quantos números distintos de 6 algarismos e múltiplos de 45 podem ser escritos
colocando um dígito à esquerda e outro à direita de 2008?

PROBLEMA 2
No colégio Olímpico as provas são avaliadas com números inteiros, a menor nota
possível é 0, e a maior é 10. Na aula de Matemática o professor aplica as provas.
Este ano a turma tem 15 alunos. Quando um dos alunos tira na primeira prova uma
nota menor que 3 e na segunda prova uma nota maior que 7, o aluno é chamado de
aluno superado. O professor, ao terminar de corrigir as provas, fez uma média com
as 30 notas e obteve 8. Qual é a maior quantidade de alunos superados que pode ter
tido a turma?

PROBLEMA 3
Num quadro negro estão escritos os números inteiros de 1 a 2008 inclusive.
Apagam-se dois números e escreve-se a diferença entre eles. Por exemplo, se
apagamos o número 5 e 241, escrevemos 236. Assim continuamos, apagando os
números e escrevendo a diferença, até que fica somente um número. Determine se
o número que ficou por último pode ser 2008. E 2007?
Em cada caso, se a resposta é afirmativa indique uma seqüência com esse número
final, e se é negativa, explique o porquê.

PROBLEMA 4
Sobre o lado AB de um quadrado ABCD é desenhado exteriormente o triângulo
retângulo ABF, de hipotenusa AB. Sabe-se que AF = 6, e que BF = 8. Chamamos
de E o centro do quadrado. Calcule o comprimento de EF.

PROBLEMA 5
Num tabuleiro de 16 × 16 colocamos 25 moedas, como na figura abaixo. É
permitido selecionar 8 linhas e 8 colunas e retirar do tabuleiro todas as moedas que
se encontram nessas linhas e colunas. Determine se é possível retirar todas as
moedas do tabuleiro.
Se a resposta é afirmativa, indique as 8 linhas e as 8 colunas selecionadas, e se é
negativa, explique o porquê.

EUREKA! N°29, 2009

2
Sociedade Brasileira de Matemática

SEGUNDO NÍVEL
PROBLEMA 1
Num quadro negro está escrita a seguinte expressão:
1 − 2 − 22 − 23 − 24 − 25 − 26 − 27 − 28 − 29 − 210.
Juan distribui parêntesis de distintas maneiras e efetua o cálculo que fica. Por
exemplo:
1 − 2 − (22 − 23 ) − 24 − (25 − 26 − 27 ) − 28 − (29 − 210 ) = 403 ou
1 − (2 − 22 (−23 − 24 ) − (25 − 26 − 27 )) − (28 − 29 ) − 210 = −933.
Quantos resultados diferentes pode obter Juan?

PROBLEMA 2
No retângulo ABCD de lados AB, BC, CD e DA, seja P um ponto do lado AD tal
que BPCµ = 90° . A perpendicular a BP traçada por A corta BP em M e a
perpendicular a CP traçada por D corta CP em N.
Demonstre que o centro do retângulo está no segmento MN.

PROBLEMA 3
Nos números 1010...101 estão alternados uns e zeros; se há n uns, há n – 1 zeros
(n ≥ 2).
Determine os valores de n para os quais o número 1010...101, que tem n uns, é
primo.

PROBLEMA 4
No plano há 16 retas tais que não há duas paralelas nem três concorrentes.
Sebastián tem que pintar os 120 pontos que são interseção de duas retas de modo
que em cada reta todos os pontos pintados sejam de cor diferente.
Determine o número mínimo de cores que Sebastián precisa para concluir a tarefa.
E se as retas são 15 (neste caso, os pontos são 105)?

EUREKA! N°29, 2009

3
Sociedade Brasileira de Matemática

PROBLEMA 5
Matias cobriu um tabuleiro quadrado de 7 × 7, dividido em casas de 1 × 1, com
peças dos três tipos a seguir

Tipo 1 Tipo 2 Tipo 3

sem buracos nem superposições, e sem sair do tabuleiro.


Cada peça do tipo 1 cobre exatamente 3 casas e cada peça do tipo 2 ou do tipo 3
cobre exatamente 4 casas.
Determine a quantidade de peças do tipo 1 que Matias pode ter utilizado.
(As peças podem girar e ser viradas).

RESULTADO BRASILEIRO

2008: Nível 1 (até 13 anos)


Nome Cidade - Estado Pontos Prêmio
Rafael Kazuhiro Miyazaki São Paulo – SP 45 Medalha de Ouro
Débora Ornellas Salvador – BA 36 Medalha de Prata
Nicolas Seoane Miquelin Mauá – SP 34 Medalha de Prata
Guilherme Renato Martins Unze São Paulo – SP 33 Medalha de Bronze
Ana Beatrice Bonganha Zanon Santo André – SP 26 Medalha de Bronze
Paula Dias Garcia Brasília – DF 24 Medalha de Bronze
Lara Timbó Araújo Fortaleza – CE 24 Medalha de Bronze
Francisco Markan Nobre de Souza Filho Fortaleza – CE 24 Menção Honrosa
Henrique Gasparini Fiúza do Nascimento Brasília – DF 23 Menção Honrosa
Henrique Vieira G. Vaz São Paulo – SP 23 Menção Honrosa

2008: Nível 2 (até 15 anos)


Nome Cidade - Estado Pontos Prêmio
João Lucas Camelo Sá Fortaleza – CE 37 Medalha de Ouro
Guilherme da Rocha Dahrug Santo André – SP 30 Medalha de Prata
Matheus Barros de Paula Taubaé – SP 29 Medalha de Prata
Rafael Ferreira Antonioli S.B. do Campo – SP 23 Medalha de Bronze
Nara Gabriela de Mesquita Peixoto Fortaleza – CE 22 Medalha de Bronze
Rodrigo Nagamine Santo André – SP 22 Medalha de Bronze
Henrique Lopes de Mello Rio de Janeiro – RJ 21 Medalha de Bronze
Victorio Takahashi Chu São Paulo – SP 21 Menção Honrosa
Jonas Rocha de Lima Amaro Fortaleza – CE 17 Menção Honrosa
Deborah Barbosa Alves São Paulo – SP 16 Menção Honrosa

EUREKA! N°29, 2009

4
Sociedade Brasileira de Matemática

XLVIII OLIMPÍADA INTERNACIONAL DE MATEMÁTICA


Enunciados e Resultado Brasileiro

A XLVIII Olimpíada Internacional de Matemática foi realizada na cidade


de Hanói, Vietnã no período de 19 a 31 de julho de 2007. A equipe brasileira foi
liderada pelos professores Carlos Gustavo Moreira do Rio de Janeiro – RJ e Onofre
Campos da Silva Farias de Fortaleza – CE.

RESULTADOS DA EQUIPE BRASILEIRA

BRA1 Régis Prado Barbosa Medalha de Prata


BRA2 Henrique Pondé de Oliveira Pinto Medalha de Prata
BRA3 Ramón Moreira Nunes Medalha de Bronze
BRA4 Rafael Sampaio de Rezende Medalha de Bronze
BRA5 Rafael Tupynambá Dutra Medalha de Bronze
BRA6 Guilherme Phillipe Figueiredo Menção Honrosa

PRIMEIRO DIA

PROBLEMA 1:
Sejam a1 , a2 ,..., an números reais. Para cada i(1 ≤ i ≤ n) definimos
d i = max{a j :1 ≤ j ≤ i} − min{a j : i ≤ j ≤ n}
e
d = max{d i :1 ≤ i ≤ n}.

(a) Prove que para quaisquer números reais x1 ≤ x2 ≤ ... ≤ xn ,


d
max{ xi − ai :1 ≤ i ≤ n} ≥. (*)
2
(b) Prove que existem números reais x1 ≤ x2 ≤ ... ≤ xn para os quais vale a igualdade
em (*).

PROBLEMA 2:
São dados cinco pontos A, B, C D e E tais que ABCD é um paralelogramo e BCED
é um quadrilátero cíclico (e convexo). Seja l uma recta que passa por A.

EUREKA! N°29, 2009

5
Sociedade Brasileira de Matemática

Suponhamos que l intersecta o segmento DC num ponto interior F e a recta BC


em G.
Suponhamos também que EF = EG = EC. Prove que l é a bissectriz do ângulo
DAB.

PROBLEMA 3:
Numa competição de Matemática alguns participantes são amigos. A amizade é
sempre recíproca. Dizemos que um grupo de participantes é um clique se dois
quaisquer deles são amigos (em particular, qualquer grupo com menos de dois
participantes é um clique). O tamanho de um clique é o número de seus elementos.
Sabe-se que nesta competição o tamanho máximo dos cliques é par.
Prove que os participantes podem ser distribuídos em duas salas, de modo que o
tamanho máximo dos cliques contidos numa sala é igual ao tamanho máximo dos
cliques contidos na outra sala.

SEGUNDO DIA

PROBLEMA 4:
No triângulo ABC a bissectriz do ângulo BCA intersecta a circunferência
circunscrita em R ( R ≠ C ), a mediatriz de BC em P e a mediatriz de AC em Q.
O ponto médio de BC é K e o ponto médio de AC é L. Mostre que os triângulos
RPK e RQL têm áreas iguais.

PROBLEMA 5:
Sejam a e b inteiros positivos tais que 4ab – 1 divide (4a 2 − 1) 2 . Prove que a = b.

PROBLEMA 6:
Seja n um número inteiro positivo. Considere o conjunto
S = {( x, y , z ) : x, y , z ∈{0,1,..., n}, x + y + z > 0}
de (n + 1)3 − 1 pontos no espaço tridimencional. Determine o menor número
possível de planos cuja união contém todos os pontos de S mas não contém (0, 0,
0).

EUREKA! N°29, 2009

6
Sociedade Brasileira de Matemática

XLIX OLIMPÍADA INTERNACIONAL DE MATEMÁTICA


Enunciados e Resultado Brasileiro

A XLIX Olimpíada Internacional de Matemática foi realizada na cidade de


Madri, Espanha no período de 10 a 22 de julho de 2008. A equipe brasileira foi
liderada pelos professores Luciano Monteiro de Castro do Rio de Janeiro – RJ e
Carlos Yuzo Shine de São Paulo – SP.

RESULTADOS DA EQUIPE BRASILEIRA

BRA1 Davi Lopes Alves de Medeiros Medalha de Prata


BRA2 Henrique Pondé de Oliveira Pinto Medalha de Prata
BRA3 Marcelo Matheus Gauy Medalha de Bronze
BRA4 Rafael Tupynambá Dutra Medalha de Prata
BRA5 Régis Prado Barbosa Medalha de Prata
BRA6 Renan Henrique Finder Medalha de Prata

PRIMEIRO DIA

PROBLEMA 1:
Seja ABC um triângulo acutângulo e seja H o seu ortocentro. A circunferência de
centro no ponto médio de BC e que passa por H intersecta a reta BC nos pontos
A1 e A2 .
Analogamente, a circunferência de centro no ponto médio de CA e que passa por H
intersecta a reta CA nos pontos B1 e B2 , e a circunferência de centro no ponto
médio de AB e que passa por H intersecta a reta AB nos pontos C1 e C2 . Mostre
que A1 , A2 , B1, B 2 ,C 1, C 2 estão sobre uma mesma circunferência.

PROBLEMA 2:
(a) Prove que
x2 y2 z2
+ + ≥1 (*)
( x − 1) 2
( y − 1) 2
( z − 1)2
para todos os números reais x, y, z, diferentes de 1, com xyz = 1.

(b) Prove que existe uma infinidade de ternos de números racionais x, y, z,


diferentes de 1, com xyz = 1, para os quais ocorre a igualdade em (*).

EUREKA! N°29, 2009

7
Sociedade Brasileira de Matemática

PROBLEMA 3:
Prove que existe um número infinito de inteiros positivos n tais que n 2 + 1 tem um
divisor primo maior que 2n + 2n .

PROBLEMA 4:
Determine todas as funções f ]0, ∞[ → ]0, ∞[ (ou seja, f é uma função dos reais
positivos para os reais positivos) tais que

( f ( w) ) + ( f ( x) )
2 2
w2 + x 2
=
f ( y2 ) + f (z2 ) y2 + z2

para todos os números reais positivos w, x, y com wx = yz.

PROBLEMA 5:
Sejam n e k números inteiros positivos tais que k ≥ n e k − n é um número par.
São dadas 2n lâmpadas numeradas de 1 a 2n, cada uma das quais pode estar acesa
ou apagada.
Inicialmente todas as lâmpadas estão apagadas. Uma operação consiste em alterar
o estado de exatamente uma das lâmpadas (de acessa para apagada ou de apagada
para acesa). Consideremos seqüências de operações.
Seja n o número de seqüências com k operações após as quais as lâmpadas de 1 a n
estão todas acessas e as lâmpadas de n + 1 a 2n estão todas apagadas.
Seja M o número de seqüências com k operações após as quais as lâmpadas de 1 a
n estão todas acesas e as lâmpadas de n + 1 e 2n estão todas apagadas, e durante as
quais todas as lâmpadas de n + 1 a 2n permanecem sempre apagadas.
N
Determine a razão .
M

PROBLEMA 6:
Seja ABCD um quadrilátero convexo cujos lados BA e BC tam comprimentos
diferentes. Sejam w1 e w2 as circunferências inscritas nos triângulos ABC e ADC,
respectivamente.
Suponhamos que existe um circunferência w tangente à reta BA de forma que A
está entre B e o ponto de tangência, tangente à reta BC de forma que C está entre B
e o ponto de tangência, e que também seja tangente às retas AD e CD. Prove que as
tangentes comuns exteriores a w1 e w2 se intersectam sobre w.

EUREKA! N°29, 2009

8
Sociedade Brasileira de Matemática

XXII OLIMPÍADA IBEROAMERICANA DE MATEMÁTICA


Enunciados e Resultado Brasileiro

A XXII Olimpíada Iberoamericana de Matemática foi realizada na cidade


de Coimbra, Portugal no período de 6 a 16 de setembro de 2007. A equipe
brasileira foi liderada pelos professores Eduardo Wagner e Edmilson Motta da
cidade do Rio de Janeiro – RJ e São Paulo – SP respectivamente.

RESULTADOS DA EQUIPE BRASILEIRA

BRA1 Guilherme Rodrigues Nogueira de Souza Medalha de Ouro


BRA2 Henrique Pondé de Oliveira Pinto Medalha de Prata
BRA3 Ramon Moreira Nunes Medalha de Ouro
BRA4 Régis Prado Barbosa Medalha de Ouro

PRIMEIRO DIA

PROBLEMA 1:
Dado um inteiro positivo m, define-se a sucessão {an } da seguinte maneira:
m
a1 = , an +1 = an  an  , si n ≥ 1.
2
Determinar todos os valores de m para os quais a2007 é o primeiro inteiro que
aparece na sucessão.
Nota: Para um número real x se define  x  como o menor inteiro que é maior ou
igual a x.
Por exemplo, π  = 4,  2007  = 2007.

PROBLEMA 2:
Sejam ABC um triângulo com incentro I e Γ uma circunferência de centro I, de
raio maior que o da circunferência inscrita e que não passa por nenhum dos
vértices. Sejam X 1 o ponto de intersecção de Γ com a reta AB mais perto de
B, X 2 e X 3 os pontos de intersecção de Γ com a recta BC sendo X 2 o mais perto
de B e X 4 o ponto de intersecção de Γ com a recta CA mais perto de C. Seja K o
ponto de intersecção das rectas X 1 X 2 e X 3 X 4 . Demonstre que AK corta o
segmento X 2 X 3 no seu ponto médio.

EUREKA! N°29, 2009

9
Sociedade Brasileira de Matemática

PROBLEMA 3:
Duas equipes, A e B disputam o território limitado por uma circunferência.
A tem n bandeiras azuis e B tem n bandeiras brancas ( n ≥ 2, fixo). Jogam
alternadamente e A começa o jogo. Cada equipe, na sua vez, coloca uma das suas
bandeiras num ponto da circunferência que não se tenha usado numa jogada
anterior. Cada bandeira, uma vez colocada, não se pode mudar de lugar.
Uma vez colocadas as 2n bandeiras reparte-se o território entre as duas equipes.
Um ponto do território é da equipe A se a bandeira mais próxima dele é azul, e é da
equipe B se a bandeira mais próxima dele é branca. Se a bandeira azul mais
próxima de um ponto está à mesma distância que a bandeiras branca mais próxima
deste ponto, então o ponto é neutro (não é de A nem de B). Uma equipe ganha o
jogo se seus pontos cobrem uma área maior que a área coberta pelos pontos da
outra equipe. Há empate se ambos cobrem áreas iguais.
Demonstre que, para todo n a equipe B tem estratégia para ganhar o jogo.

SEGUNDO DIA

PROBLEMA 4:
Em um tabuleiro quadriculado de tamanho 19 × 19, uma ficha chamada dragão dá
saltos da seguinte maneira: desloca-se 4 casas numa direcção paralela a um dos
lados do tabuleiro e 1 casa em direção perpendicular à anterior.

X X

X X
D

A partir de D, o dragão pode saltar para uma das quatro posições X.

Sabe-se que, com este tipo de saltos, o dragão pode mover-se de qualquer casa a
qualquer outra.
A distancia dragoniana entre duas casas é o menor número de saltos que o dragão
deve dar para mover-se de uma casa a outra.
Seja C uma casa situada num canto do tabuleiro e seja V a casa vizinha a C que a
toca num único ponto.

EUREKA! N°29, 2009

10
Sociedade Brasileira de Matemática

Demonstre que existe alguma casa X do tabuleiro tal que a distância dragoniaana
de C a X é maior que a distãncia dragoniana de C a V.

PROBLEMA 5:
Um número natural n é atrevido se o conjunto dos seus divisores, incluindo 1 e n,
pode ser dividido em três subconjuntos tais que a soma dos elementos de cada
subconjunto é a mesma nos três. Qual é a menor quantidade de divisores que pode
ter um número atrevido?

PROBLEMA 6:
Seja F a família de todos os hexágonos convexos H que satisfazem as seguintes
condições:

a) os lados opostos de H são paralelos;


b) quaisquer três vértices de H podem ser cobertos por uma faixa de largura 1.

Determine o menor número real l tal que cada um dos hexágonos da família F
pode ser coberto com uma faixa de largura l .

Nota: Uma faixa de largura l é a região do plano compreendida entre duas rectas
paralelas que estão à distancia l (incluídas ambas as rectas paralelas).

EUREKA! N°29, 2009

11
Sociedade Brasileira de Matemática

XXIII OLIMPÍADA IBEROAMERICANA DE MATEMÁTICA


Enunciados e Resultado Brasileiro

A XXIII Olimpíada Iberoamericana de Matemática foi realizada na cidade


de Salvador, Bahia no período de 20 a 28 de setembro de 2008. A equipe brasileira
foi liderada pelos professores Eduardo Wagner e Fábio Dias Moreira, ambos da
cidade de Rio de Janeiro – RJ. Com este resultado a equipe brasileira obteve
também a maior pontuação total da competição ficando em primeiro lugar com 155
pontos.

RESULTADOS DA EQUIPE BRASILEIRA

BRA1 Henrique Ponde de Oliveira Pinto Medalha de Ouro


BRA2 Renan Henrique Finder Medalha de Prata
BRA3 Ramon Moreira Nunes Medalha de Ouro
BRA4 Régis Prado Barbosa Medalha de Prata

PRIMEIRO DIA

PROBLEMA 1:
Os números 1, 2, 3,..., 20082 são distribuídos num tabuleiro 2008 × 2008, de modo
que em cada casa haja um número distinto. Para cada linha e cada coluna do
tabuleiro calcula-se a diferença entre o maior e o menor dos seus elementos. Seja S
a soma dos 4016 números obtidos.
Determine o maior valor possível para S.

PROBLEMA 2:
Sejam ABC um triângulo escaleno e r a bissectriz externa do ângulo ∠ABC. Sejam
P e Q os pés das perpendiculares à recta r que passam por A e C, respectivamente.
As rectas CP e AB intersectam-se em M e as rectas AQ e BC intersectam-se em N.
Demonstre que as rectas AC, MN e r têm um ponto em comum.

PROBLEMA 3:
Sejam m e n inteiros tais que o polinômio P ( x) = x 3 + mx + n tem a seguinte
propriedade: se x e y são inteiros e 107 divide P ( x) − P ( y ), então 107 divide x – y.
Demosntre que 107 divide m.

EUREKA! N°29, 2009

12
Sociedade Brasileira de Matemática

SEGUNDO DIA

PROBLEMA 4:
Demonstre que não existem inteiros positivos x e y tais que
x 2008 + 2008! = 21y.

PROBLEMA 5:
Seja ABC um triângulo e X, Y, Z pontos interiores dos lados BC, AC, AB
respectivamente. Sejam A´, B´, C´ os circuncentros dos triângulos AZY, BXZ, CYX,
respectivamente.
Demonstre que
( ABC )
( A´B´C´) ≥
4
e que a igualdade ocorre se, o somente se, as rectas AA´, BB´, CC´ têm um ponto
em comum.

Observação: Para um triângulo qualquer RST, denotamos a sua área por (RST).

PROBLEMA 6:
Numa partida de biribol enfrentam-se duas equipes de quatro jogadores cada uma.
Organiza-se um torneio de biribol em que participam n pessoas, que formam
equipes para cada partida (as equipes não são fixas). No final do torneio observou-
se que cada duas pessoas disputaram exactamente uma partida em equipes rivais.
Para que valores de n é possível organizar um torneio com tais características?

EUREKA! N°29, 2009

13
Sociedade Brasileira de Matemática

0,999... OU “COMO COLOCAR UM BLOCO QUADRADO


EM UM BURACO REDONDO”
Pablo Emanuel

• Nível Intermediário

Quando um jovem estudante de matemática começa a estudar os números


reais, é difícil não sentir certo desconforto e estranhamento. De repente, algumas
coisas que faziam sentido param de funcionar tão bem assim. Com certeza, uma
das mais estranhas pode ser resumida na igualdade

0,9999..... = 1,0000...

Como assim? No mundo dos números inteiros (com a exceção muito razoável de 0
= –0), qualquer número tem uma única representação decimal (os zeros à esquerda
não são um problema sério, ou definimos que nunca podemos começar com 0 – o
que deixa o próprio zero em uma situação meio desconfortável de ter uma
representação decimal vazia – ou definimos que sempre vamos completar com
infinitos zeros à esquerda), e esta se comporta bem – i.e. qualquer número da forma
8xxxxxxx é menor que qualquer número da forma 9yyyyyyy com o mesmo número
de dígitos. Como pode haver um número começado por 0,9 que não seja menor que
um número começado por 1,0 e, pior, que seja igual!? A resposta explica, mas não
convence: se definirmos as seqüências:

an = 0.999999...9 (n dígitos 9) e
bn = 1.000000...0 (n dígitos 0)

temos que an < 0.999... ⇐ 1.000... < bn. No entanto, como bn – an = 1/10n, que
converge para 0, a diferença entre 1.000... e 0.999..., que é menor que qualquer das
diferenças bn – an só pode ser 0, logo os números são iguais.

OK, entendido, mas ainda tem caroço neste angu. Por que a representação decimal,
que se comporta tão bem para números inteiros tem este tipo de esquisitice para
números reais? A verdade é que usar a representação decimal para números reais é
enfiar um bloco quadrado em um buraco redondo.

Em primeiro lugar, até agora estamos usando a representação decimal (base 10)
apenas porque é a representação com que estamos mais acostumados. Será que o

EUREKA! N°29, 2009

14
Sociedade Brasileira de Matemática

problema está com o número 10? Infelizmente não. Mesmo que usemos outras
bases, o problema continua:

0,1111.... = 1,000.... em base 2


0,2222... = 1,000... em base 3

e o mesmo problema acontece em qualquer base que escolhamos (e a demonstração


é exatamente a mesma, mutatis mutandis, que fizemos lá em cima).

Antes de entrarmos a fundo em por que a representação decimal (ou em qualquer


base N) tem estes problemas, é bom ver que outras opções nós temos para
representar os números reais. Em primeiro lugar, nossas notações foram feitas para
trabalhar com números inteiros, que é o que nos é familiar (a ponto de Pitágoras
afirmar que os números inteiros são a fundação do universo). Dos números inteiros
conseguimos de forma natural derivar os números racionais (os números da forma
a/b, onde a e b são inteiros). O pulo para os números reais irracionais, no entanto, é
menos natural, e as únicas maneiras que temos para tentar nomear os números
irracionais são através das aproximações por números racionais. A própria
representação decimal é a aproximação por números racionais da forma A/10n (ou
A/Nn, em base N). Duas outras formas de representar os números reais são as
frações contínuas e os cortes de Dedekind.

A representação por frações contínuas tem a forma

N1 + 1
_________________
N2 + 1
___________
N3 + ...

O algoritmo para construir a representação por frações contínuas de um número x é


bastante simples. Em primeiro lugar, N1 é a parte inteira de x (i.e. o maior número
inteiro que é menor ou igual a x). Se x – N1 = 0, acabou, senão, x – N1 é maior que
0 e menor que 1, portanto y = 1/(x – N1) é um número maior que 1. Seja N2 então a
parte inteira deste número, o que equivale a dizer que 1/(N2 + 1) < x – N1 ≤ 1/N2.
Agora repita o processo com y no lugar de x (i.e. N3 é o número tal que 1/(N3 + 1) <
y – N2 ⇐ 1/N3), e assim por diante até chegar a um número inteiro ou continuando
para sempre.

EUREKA! N°29, 2009

15
Sociedade Brasileira de Matemática

A representação por frações contínuas não tem o mesmo problema da


representação decimal (cada número real tem uma e somente uma representação
por frações contínuas), mas tem outros inconvenientes. O primeiro, que não é tão
sério assim, é que os números Ni, que seriam equivalentes aos dígitos, podem ser
arbitrariamente grandes, ao contrário dos dígitos decimais que só podem ser de 0 a
9. O segundo, muito mais sério, é que é terrivelmente difícil fazer contas com
frações contínuas. Este foi o principal motivo de a representação decimal ter
substituído a numeração romana na Europa no século XV – era imensamente mais
simples fazer contas com a representação decimal do que com a representação
romana. (E este também é o principal motivo de a representação binária ter tomado
espaço da representação decimal no século XX, é ainda mais fácil fazer contas em
binário).

Outra representação dos números reais é através dos cortes de Dedekind. A


diferença do corte de Dedekind para a representação decimal ou de frações
contínuas é que, enquanto as últimas usam uma seqüência convergente de números
racionais, os cortes usam conjuntos sem uma seqüência definida.

Um corte de Dedekind é um conjunto de números racionais limitado, que não


possui um maior elemento e fechado inferiormente. Ou seja, um conjunto de
racionais A tal que:

1) existe um racional X tal que X > a para todo a pertencente a A


2) para todo a pertencente a A, existe um b pertencente a A tal que b > a.
3) Se a pertence a A, e um racional c < a, então c pertence a A.

Em outras palavras, cada um destes conjuntos é o conjunto de todos os números


racionais menores que um número real determinado.
Esta representação é útil teoricamente para provar algumas propriedades dos
números reais, mas é praticamente impossível de ser utilizada na prática para as
tarefas corriqueiras do dia-a-dia.

A conclusão é que, apesar dos seus defeitos, a representação de base N é a


representação mais prática que temos para os números reais. Então vamos entendê-
la mais a fundo.

Em primeiro lugar, a parte antes da vírgula (ou ponto decimal, dependendo de em


qual lugar do mundo você vive) é apenas a parte inteira do número, com a nossa
velha e bem comportada representação decimal de números inteiros. Vamos nos
concentrar então na parte à direita da vírgula, os números entre 0 e 1.

EUREKA! N°29, 2009

16
Sociedade Brasileira de Matemática

0,0 0,1 0,2 0,3 0,4 0,5 0,6 0,7 0,8 0,9

O primeiro dígito decimal diz em qual dos dez intervalos acima o número está.
Para descobrir o segundo dígito, basta subdividir cada intervalo novamente em 10
pedaços, ou, equivalentemente, multiplicar o número por 10 e ver em qual dos
intervalos a parte fracionária deste novo número (10x) cai, de acordo com o gráfico
abaixo:

0,9

0,8

0,7

0,6

0,5

0,4

0,3

0,2

0,1

0,0
0,0 0,1 0,2 0,3 0,4 0,5 0,6 0,7 0,8 0,9

Os pontos onde nasce o nosso problema são justamente estes pontos marcados na
nossa linha (0; 0,1; 0,2; ...; 1). Repare que o gráfico acima é descontínuo nestes
pontos – por exemplo, as imagens dos pontos se aproximando de 0,2 pela esquerda
tendem a 1, enquanto pela direita tendem a 0. Dito de outra forma, os números se
aproximando pela esquerda de 0,2 vão ter uma seqüência longa de 9’s depois do
0,1 e pela direita uma seqüência longa de 0’s depois do 0,2.

Antes de prosseguir com a análise, já vimos que não existe nada de especial no
número 10, portanto, por pura preguiça, a partir de agora vou mudar para base 2,
mas deixo ao leitor desconfiado a tarefa de reescrever tudo o que se segue em base
10.

Apenas para começar do ponto onde paramos, o gráfico acima para base 2 fica
assim:

EUREKA! N°29, 2009

17
Sociedade Brasileira de Matemática

0,1

0,0
0,0 0,1

Relembrando, o problema está no ponto da descontinuidade, neste caso o 0,1


(também conhecido como ½). A origem do problema é o fato de haver números
começados por 0,0 arbitrariamente próximos de 0,1, quando, naturalmente,
números começados por 0,0 deveriam ser distintamente menores do que 0,1.
Vamos tentar corrigir este problema dando um pequeno espaço entre os intervalos
no nosso desenho.

0,1

0,0
0,0 0,1

Vamos passar agora para o segundo dígito.

0,1

0,0
0,00 0,01 0,10 0,11

EUREKA! N°29, 2009

18
Sociedade Brasileira de Matemática

O espaço que demos entre os intervalos do primeiro dígito naturalmente se propaga


para o segundo dígito (senão teríamos o mesmo problema nos números 0,01 – ¼ - e
0,11 – ¾). [Em base 10 seriam 9 buracos na primeira rodada e 90 na segunda,
entenderam a minha preguiça?]

Neste ponto, já não é difícil imaginar o processo para o terceiro dígito – criar mais
um buraco no meio de cada um dos 4 intervalos restantes, e assim por diante.

O conjunto dos números que sobram depois de fazermos este processo infinitas
vezes é um conjunto em que podemos usar a nossa representação de base 2 (ou
base 10, se fizermos o processo no desenho original lá de cima) sem medo – cada
número tem apenas uma representação, e elas respeitam a ordem e as distâncias
que esperaríamos: por exemplo, se um número começa por 0,11010 ele é
estritamente menor que qualquer número que comece por 0,11011. O único
problema é que o conjunto que temos no final deste processo é tão esfarelado que
ele não contém nem um intervalinho que seja. Se escolhêssemos um número entre
0 e 1 aleatoriamente, ele teria 0% de chance de estar no nosso conjunto – repare
que eu disse 0% não uma em um quaquilhão elevado a um googleplex, não a
chance de ganhar na megasena todas as semanas pelo resto da vida, mas zero!

Este conjunto é tão importante que tem um nome – conjunto de Cantor, em


homenagem ao matemático Georg Cantor que, entre outras coisas criou a teoria dos
conjuntos (todos os conjuntos então são um pouquinho conjuntos de Cantor) e
provou que existiam mais números irracionais do que números racionais. Dado que
acabamos de ver que o próprio sistema de representação decimal que usamos todos
os dias na verdade é um conjunto de Cantor, é de se admirar que apenas no final do
século XIX tenhamos passado a conhecê-lo.

Pelo que acabamos de ver, o nosso sistema de numeração é um mapeamento entre


um conjunto de Cantor e o intervalo [0,1]. O cerne da nossa questão é que esta
função não é injetiva, ou seja, existem elementos distintos a e b no conjunto de
Cantor que são mapeados para o mesmo número real (a diferente de b, f(a) = f(b)),
por exemplo, 0,19999... e 0,2. [Parênteses: outra forma de ver isto é afirmar que o
intervalo [0,1] é o espaço quociente do conjunto de Cantor pela relação de
equivalência entre os números terminados por 999... e os seus correspondentes
terminados por 000..., ver o artigo Egalité] no endereço:
http://www.impa.br/~gugu/pablo-egalite.doc

Vamos tentar desenhar o gráfico desta função:

EUREKA! N°29, 2009

19
Sociedade Brasileira de Matemática

Os dois extremos do buraco do meio são os pontos 0,01111.... e 0,10000... que


representam o mesmo número ½, portanto a função é constante igual a ½ neste
intervalo. Da mesma forma cada um dos buracos de “gerações” maiores
corresponde a diferentes platôs no gráfico. O mais surpreendente é que, depois de
criarmos os infinitos platôs, o gráfico final é um gráfico contínuo, que é chamado
de função de Cantor, ou de “escada do diabo” (porque sempre existem infinitos
degraus entre quaisquer dois degraus da escada, você nunca conseguiria sair do
lugar – assim como Aquiles correndo atrás da tartaruga). Infelizmente, é o que se
tem quando se tenta botar um bloco quadrado em um buraco redondo, ou neste
caso, botar um intervalo dentro de um conjunto de Cantor.

EUREKA! N°29, 2009

20
Sociedade Brasileira de Matemática

ALGORITMO DE GOSPER E APLICAÇÕES


Humberto Silva Naves

• Nível Avançado

Continuando com as idéias do artigo “Integrais discretas” (de Eduardo


Poço na Eureka número 27), vamos tentar descobrir “fórmulas fechadas” para
alguns somatórios da forma:
n −1
(*)∑ zk
k =0

Algumas considerações devem ser feitas antes de continuarmos:


1- Vamos assumir que zk é uma seqüência hipergeométrica, isto é, a razão
r (k ) ≡ zk +1 / zk é uma função racional de k.
2- Por “fórmula fechada”, entendemos que existe uma seqüência
hipergeométrica sn tal que sn = ∑ kn −=10 zk (essa definição não é tão
restritiva assim, pois veremos que a classe das seqüências hipergeométricas
é bastante ampla).
Por exemplo, vamos tentar achar uma fórmula fechada para:
2
 2k 
n −1  
sn = ∑ zk , onde zk =   2 k
k
k =0 (k + 1)4

(2k + 1) 2
Claramente zk é hipergeométrica, uma vez que r (k ) = zk +1 / zk = é
4(k + 1)(k + 2)
uma função racional de k. Por (*), vale:

zn sn +1
zn = sn +1 − sn ⇔ = −1
sn sn
−1
s 
A nossa fórmula fechada sn é hipergeométrica, logo  n +1 − 1 é uma função
 sn 
racional y(n), portanto:
sn = y (n) ⋅ zn ⇔ y (n + 1) zn +1 − y ( n) zn = zn ⇔

EUREKA! N°29, 2009

21
Sociedade Brasileira de Matemática

⇔ r ( n ) y (n + 1) − y ( n) = 1

(2n + 1)2
onde: r (n) =
4(n + 1)(n + 2)
Como y(n) é racional, existem polinômios P (n), Q( n) com mdc{P (n), Q (n)} = 1
P (n)
tais que y (n) = e substituindo na equação anterior, vale:
Q ( n)

(2n + 1)2 P (n + 1)Q(n) − 4(n + 1)(n + 2)Q( n + 1) P (n) =


= 4(n + 1)( n + 2)Q (n + 1)Q( n), ∀n ≥ 0
Disto concluí-se que:
1- Q(n) | 4(n + 1)( n + 2)Q (n + 1)
2- Q(n + 1) (2n + 1) 2 Q (n) ⇔ Q (n) (2n − 1)2 Q (n − 1)

(Observação: todas as relações de divisibilidade e os mdc´s referencem-se aos


polinômios em si e não aos seus valores em um dado ponto)
Por (1), temos que se α é uma raiz de Q então: α = –1 ou α = –2 ou α + 1 é raiz de
Q. E por (2) se α é raiz de Q então: α = 1 ou α – 1 também é raiz de Q. Portanto,
2
como Q não pode ter infinitas raízes, Q não possui raiz alguma, isto é, Q(n) é
constante (sem perda de generalidade, Q(n) = 1).
Disto concluí-se que:
(2n + 1)2 P (n + 1) − 4(n + 1)(n + 2) P(n) = 4(n + 1)(n + 2)

Logo: (n + 1)(n + 2) P(n + 1) ⇔ n(n + 1) P (n) , daí P (n) = n( n + 1) P (n) , então vale:

(2n + 1)2 P (n + 1) − 4n(n + 1) P (n) = 4, onde P (n) é um polinômio

Seja d = deg P, P (n) = ad n d + ... + a0 ; substituindo na equação anterior, vale (para o


caso d > 0):
(4n 2 + 4 n + 1)(ad (n + 1) d + ... + a0 ) − (4n 2 + 4n)(ad n d + ... + a0 ) = 4 ⇔

(4n 2 + 4 n + 1)(ad n d + (ad −1 + dad )n d −1 + ...)

−(4n 2 + 4n)(ad n d + ad −1n d −1 + ...) = 4 ⇔

EUREKA! N°29, 2009

22
Sociedade Brasileira de Matemática

4dad n d +1 + ... = 4

Mas, se d > 0, 4dad n d +1 + ... = 4 é um polinômio de grau d + 1. Portanto


d = 0 ⇒ P(n) = 4 (que de fato é solução da equação anterior), donde concluímos
que: y (n) = 4n(n + 1), logo:
2 2 2
 2k   2n   2n 
n −1     n 
 k  = 4n(n + 1)  n  =  n 

k = 0 ( k + 1)4
2k
(n + 1)42 n 42 n −1
(é fácil ver que a fórmula vale para n = 0 e n = 1, e, pela identidade que provamos,
vale sempre).
Vamos agora generalizar essa idéia para qualquer seqüência hipergeométrica zk . O
leitor atento deve ter notado que na conclusão de que Q(n) = 1, implicitamente
usamos o fato que: mdc{A(n), B(n + h)} = 1, para qualquer inteiro h ≥ 0 , onde
A(n) = (2n + 1) 2 , e B (n) = 4(n + 1)(n + 2) , são o numerador e o denominador de
r(n) respectivamente.
zn +1
Mas nem sempre é possível escrever r (n) = como a razão de dois
zn
polinômios satisfazendo as condições:
mdc{ A(n), B (n + h)} = 1, ∀h ∈ ¢ + .
O leitor pergunta: “Então essa técnica não se aplica para todos os casos?!?!?”.
O autor responde: “Não se desesperem!”, uma vez que é possível escrever r(n) da
seguinte forma:
A(n) C (n + 1)
(**)r (n ) =
B ( n) C ( n)

onde A(n), B(n), C(n) são polinômios tais que: mdc{ A(n), B (n + h)} = 1, ∀h ∈ ¢ + .
Não vamos demonstrar esse fato aqui (pois é um dos exercícios deste artigo), mas
(n + 4)(n + 3)
vamos exibir um exemplo bem ilustrativo! Como escrever r (n) = da
n 2 (n + 1)
forma (**)?
Note que:

EUREKA! N°29, 2009

23
Sociedade Brasileira de Matemática

1 n + 4 n + 3 1 n + 4 n + 3 n + 2 n + 1 n + 3 n + 2 A(n) C (n + 1)
r (n) = ⋅ ⋅ = ⋅ ⋅ ⋅ ⋅ ⋅ ⋅ = ,
n n n + 1 n n + 3 n + 2 n + 1 n n + 2 n + 1 B ( n) C ( n)

onde A(n) = 1, B(n) = n e C (n) = n( n + 1) 2 (n + 2) 2 (n + 3).


De forma geral se α é uma raiz do denominador de r(n) e α – h é uma raiz do
numerador de r(n), onde h ∈ ¢ + , então vale:

n − (α − h) n − α + h n −α + h −1 n − α + 1 T (n + 1)
= ⋅ ⋅K ⋅ =
n −α n −α + h −1 n − α + h − 2 n −α T ( n)
Onde T (n) = (n − α + h − 1)(n − α + h − 2)L (n − α )
Vamos tentar usar a fórmula (**) em
r(n) y(n + 1) – y(n) = 1 ⇔
A(n) C ( n + 1)
⇔ y ( n + 1) ⋅ − y (n) = 1
B (n) C (n)

B (n − 1)
Como y(n) é racional em n, podemos fazer a substituição y (n) = y (n),
C ( n)
portanto:
A(n ) y (n + 1) − B (n − 1) y( n) = C (n) (***)

Agora o milagre acontece! Se y (n) é uma função racional que satisfaz (***), então
P ( n)
y (n) é um polinômio! Se y (n) = , com mdc{P (n), Q (n)} = 1, então:
Q( n)

A( n) P(n + 1)Q(n) − B (n − 1) P (n )Q(n + 1) = C (n )Q (n )Q( n + 1)


Logo:

1- Q(n) B (n − 1)Q (n + 1)

2- Q(n + 1) A( n)Q( n) ⇔ Q (n) A(n − 1)Q( n + 1)

Logo se α é raiz de Q , então:

1- α é raiz de B(n – 1) ou α é raiz de Q(n + 1)

EUREKA! N°29, 2009

24
Sociedade Brasileira de Matemática

2- α é raiz de A(n – 1) ou α é raiz de Q(n − 1)

Como mdc{ A(n), B (n + h)} = 1, ∀h ∈ ¢ + , Q (n) = 1.


Vamos agora resolver a seguinte equação polinomial:
A(n ) y (n + 1) − B( n − 1) y( n) = C (n), onde y (n) = ad n d + ... + a0
Temos casos a considerar
1- deg A ≠ deg B
2- deg A = deg B e δ A ≠ δ B ( δ A é o coeficiente líder de A):
Nesses casos, pela equação (***), vale:
deg C = deg y + max{deg a,deg b} ⇔
d = deg C − max{deg A,deg B}.
3- deg A = deg B = m e δ A = δ B = k
Se d > 0, então:
(kn m + an m −1 + ...)(ad (n + 1) d + K + a0 )
−(kn m + bn m −1 + ...)(ad n d + K + a0 ) = C (n) ⇔
(kn m + an m −1 + ...)( ad n d + (ad −1 + dad )n d −1 +K)
−(kn m + bn m −1 + ...)(ad n d + ad −1n d −1 + K) = C (n) ⇔
[(a ⋅ ad + k (ad −1 + dad )) − (b ⋅ ad + kad −1 )]n m + d −1 + K = C (n)
b−a
Se (a − b ) ⋅ ad + kdad = 0 ⇒ d = , mas se
k
(a − b ) ⋅ ad + kdad ≠ 0 ⇒ deg C = m + d − 1 ⇒ d = deg C − deg A + 1
Em todos os casos, é possível calcular o valor de d e uma vez calculado o valor de
d, a equação polinomial se transforma em um sistema linear com d variáveis que
pode ser resolvido (quando possível) usando técnicas básicas de álgebra linear.
Exercícios:
1) Calcule os seguintes somatórios
1
a) ∑ nk −=10 2
k + 5k − 1
k 4 4k
b) ∑ nk −=10
 2k 
 
 k 

EUREKA! N°29, 2009

25
Sociedade Brasileira de Matemática

c) ∑ n −1
k =0 k 2ak

2) Prove que qualquer função racional r(n), pode ser escrita como:
A(n)C (n + 1)
r ( n) = com mdc{ A(n), B (n + h)} = 1, ∀h ∈ ¢ + e
B (n )C (n)
mdc{ A(n), C (n)} = mdc{B (n), C (n + 1)} = 1 (o leitor atento novamente notará que
adicionamos duas novas restrições).
3) Prove que a menos de multiplicações por constantes, A(n), B(n) e C(n) são
únicos no exercício anterior (a forma acima chama-se “forma canônica”).
4) Dizemos que duas seqüências hipergeométricas são similares, quando a razão
dos duas é uma função racional e neste caso, escrevemos sn : tn . Prove:
a) Se sn é não constante, então ∆sn : sn , onde ∆sn = sn +1 − sn .
b) Se sn e tn são hipergeométricas, e sn + tn ≠ 0 então sn + tn é hipergeométrica se
e somente se sn : tn . (Esse resultado pode levar o leitor a indagar sobre a nossa
suposição inicial do que seria uma “fórmula fechada” ser algo bem restritivo).
c) Se tn(1) , tn(2) ,..., tn( m ) são hipergeométricas e vale ∑ m (i )
t
i =1 n = 0, então tn( i ) : tn( j ) para
algum i, j com 1 ≤ i < j ≤ m.

d) Se vale ∑ n −1
k =0 zk = tn(1) +tn(2) + ... + tn( m ) , onde tn(1) , tn(2) ,..., tn( m ) e zn são
hipergeométricas, então ∑ n −1
k =0 zk é hipergeométrica! (Ufa! De fato nossa definição
de fórmula fechada não é tão restritiva assim).

EUREKA! N°29, 2009

26
Sociedade Brasileira de Matemática

DOMINGO REGADO A REPUNITS


Valberto Rômulo Feitosa Pereira
Cefetce – Uned Cedro
• Nível Iniciante

No final do ano de 2007 fui convidade pelo professor e amigo Onofre


Campos, por quem tenho admiração, para ministrar aulas para um jovem que não
podia se locomover, pelo valor que eu iria receber pelas aulas, pensei que o rapaz
pertencia a uma família muito rica; grande foi a minha surpresa ao perceber
justamente o contrário: em segunda conversa com o supra-citado professor, soube
da frágil situação financeira do jovem aluno, mas também, por outro lado, do seu
incrível potencial e de sua força de vontade, fatos que me entusiasmaram em
conhecê-lo. Este aluno era Ricardo Oliveira, o qual havia conquistado duas
medalhas de Ouro na OBMEP.
No último encontro que tive com Ricardo, em sua residência, ainda
promovido pelo projeto de iniciação científica, deparamo-nos com o seguinte
problema:
“O inteiro A é formado por 666 algarismos iguais a 3, e o número B por
666 algarismos iguais a 6. Que algarismos apareceram no produto AB?”
Enquanto Ricardo fazia uma atividade, eu folhava uma apostila que
continha as colunas semanais – Olimpíada de Matemática – do jornal O Povo em
parceria com o Departamento de Matemática da UFC. Neste momento, vi o
problema acima e falei:
- Olha Ricardo que belo problema!
Nesse instante Ricardo para sua atividade, lê o problema e passa a resolvê-
lo. Eu também caio na tentativa de resolvê-lo, lembrei que:
10n − 1
14243 =
111111...11
9
.
n algarismos 1

Minha solução com o uso desta informação saiu; Ricardo sem esta
informação errou por um algarismo. Expliquei a Ricardo minha solução,
percebemos que a informação que eu havia usado era importante. A aula
continuou, mas ainda fiquei pensando como esta igualdade daria para resolver
belos problemas.
No dia seguinte tive uma conversa com meu amigo Secco, olímpico do Rio
de Janeiro. Perguntei-lhe se conhecia problemas que em sua solução usava esta

EUREKA! N°29, 2009

27
Sociedade Brasileira de Matemática

igualdade; Secco falou que conhecia e mais ainda: estes números eram chamados
de Repunits e indicou [4]. Com a dica de Secco e o entusiasmo de Ricardo,
cataloguei cinco problemas da antiga coluna, os quais passaremos a resolver.
Também apresentei aos meus alunos do projeto OBMEP 2008, realizado no Cefet,
Uned de Cedro-Ce.

1. REPUNITS
Os Repunits são números que só têm algarismos 1, por exemplo:
11, 111, 1111, 11111, ...
Estes números podem ser escritos de outra forma, vejamos:
678
k
678
k

999...9 1000...0 − 1 10k − 1


123 = 9 =
111...1
9
=
9
.
k

A beleza destas informações é poder resolver problemas interessantes sem usar


técnicas sofisticadas.

2. EXEMPLOS
Exemplo 1: O inteiro positivo n é formado de k algarismos 9. Mostre que a soma de
todos os algarismos de n2 é igual a 9k.

Demonstração: Pelas hipóteses temos


10k − 1
N = 999...9
123 = 9(111...1) = 9 = 10k − 1.
k 9
Calculemos N2 da seguinte forma:
N 2 = N .N
N 2 = (999...9).(10k − 1)
N 2 = 999...9000...0
123 123 − 999...9
123
k k k

N = 999...98000...01
123 123
2

k −1 k −1

A soma dos algarismos é: 9(k − 1) + 8 + 1 = 9k .

EUREKA! N°29, 2009

28
Sociedade Brasileira de Matemática

Exemplo 2: Mostre que os números 49, 4489, 444889, ..., obtidos colocando o
número 48 no meio do número anterior, são quadrados de números inteiros.
Demonstração: Vejamos as igualdades:
49 = 4.1.101 + 8.1 + 1
4489 = 4.11.102 + 8.11 + 1
444889 = 4.111.103 + 8.111 + 1
De modo geral temos: N = 444...488...89
123 { = 4.111...1.10
123
n
+ 8.111...1
123 + 1.
n n −1 n n

10 − 1 n

123 = 9
Substituindo 11...11 na expressão acima ficamos:
n

4 8
N = (10n − 1).10n + (10n − 1) + 1
9 9
4 4 8 8
N = 102 n − 10n + 10n − + 1
9 9 9 9
2
 2.10n + 1 
N = 
 3 
O número 2.10 + 1 é múltiplo de 3, portanto N é um quadrado perfeito.
n

Exemplo 3: Para cada inteiro positivo n, sejam A(n) e B(n) dois números inteiros
formados por 2n algarismos iguais a 1 e n algarismos iguais a 2 respectivamente.
Mostre que A(n) – B(n) é um quadrado perfeito.
Demonstração. Pelas hipóteses temos:
A(n) − B (n) = 111...1
123 − 222...2
123
2n n

10n − 1 102 n − 1
123
Como 222...2 = 2 e 123
111...1 = , substituindo teremos:
n 9 2n 9
102 n − 1 10n − 1
A(n) − B (n) = −2
9 9
(10n − 1)(10n + 1) 10n − 1
A(n) − B (n) = −2
9 9
(10 − 1)[(10 + 1) − 2]
n n
A(n) − B (n) =
9
(10n − 1) 2
A(n) − B (n) =
9

EUREKA! N°29, 2009

29
Sociedade Brasileira de Matemática

(10n − 1)2
A(n) − B (n) = .
32
Assim A(n) − B (n) é quadrado perfeito.

Exemplo 4: Sem efetuar a multiplicação, calcule o valor de (999.999.999) 2 .


Solução: Vamos escrever a expressão (999.999.999) 2 da seguinte maneira:
2
 109 − 1 
(999.999.999) =  9.
2
 = 10 − 2.10 + 1 = 1000...0
18 9
123 − 2 000...0
123 + 1
 9  18 − zeros 9 − zeros

fazendo as contas ficamos:


(999.999.999) 2 = 999...98000...01
123 123
8 − noves 8 − zeros
Finalmente o problema motivdor do nosso trabalho.

Exemplo 5: O inteiro A é formado por 666 algarismos iguais a 3, e o número B por


666 algarismos iguais a 6. Que algarismos apareceram no produto AB?
10666 −1
Solução: Como A = 666...6
123 = 123
6.111...1 e B = 123 123
333...3 = 3.111...1 = 3. , vamos
666 666 666 666 9
calcular AB mas usando alguns artifícios, como segue abaixo:
10666 − 1
AB = 3.6.(111...1).
9
AB = 2.(111...1)(10 − 1)
666

AB = (222...2)(10666 − 1)
AB = (222...2).10666 − 222...2
AB = 222...2 123 − 222...2
123 000...0 123
666 666 666

AB = 222...21777...78
123 123
665 665

Logo apareceram no produto AB:


- Um algarismo 1;
- Um algarismo 8;
- 665 algarismos 2;
- 665 algarismos 7.

EUREKA! N°29, 2009

30
Sociedade Brasileira de Matemática

3. PROBLEMAS PROPOSTOS

1. Achar a soma: 2 + 22 + ... + 222...2


se a última parcela tem n algarismos iguais a 2.

2. 123 = 222...2
123 + (333...3)
2
Prove que: 111...1 123 .
2n n n

3. 123 divisível por 41 se e somente se n é divisível por 5.


Prove que se 111...1
n
4. Mostre que nenhum inteiro da seqüência: 11,111,1111,11111,... é um
quadrado perfeito.

5. Mostrar que os inteiros: 1111,111111,..., cada um dos quais é formado por


um número par de algarismos 1, são compostos.

Nota dos editores: Não é difícil mostrar que se 111...1


123 é primo então n é primo
n
(exercício!) . Os únicos valores de n para os quais se sabe provar atualmente que
123 é primo são 2, 19, 23, 317 e 1031. Recentemente (entre 1999 e 2007) foram
111...1
n

123 é provavelmente primo (i.e.,


descobertos os seguintes valores de n tais que 111...1
n
passa por diversos testes probabilísticos de primalidade): 49081, 86453, 109297 e
270343. De acordo com os testes já realizados, qualquer outro repunit primo deve
ter mais de 400.000 algarismos.

REFERÊNCIAS

[1] Emanuel Carneiro, Francisco Antonio M. de Paiva, Onofre Campos, Olimpíadas


Cearenses de Matemática do Ensino Fundamental, Edições Realce Editora e Indústria
Gráfica, Fortaleza, 2006.
[2] Alencar Filho, Edgar de, Teoria Elementar dos Números, Nobel, São Paulo, 1988.
[3] Coluna Semanal Olimpíadas de Matemática, Jornal O Povo em parceria com o
Departamento de Matemática da UFC, No. 01, ao No. 200.
[4] Titu Andreescu, Razvan Gelca, Mathematical Olympiad Challenges, 2000.

EUREKA! N°29, 2009

31
Sociedade Brasileira de Matemática

HOMOTETIAS, COMPOSIÇÃO DE HOMOTETIAS


E O PROBLEMA 6 DA IMO 2008
Carlos Yuzo Shine
• Nível Avançado

Antes de começar a discussão, vamos enunciar o problema 6 da IMO 2008,


que é a motivação principal desse artigo.

Problema 6, IMO 2008. Seja ABCD um quadrilátero convexo cujos lados


BA e BC têm comprimentos diferentes. Sejam w1 e w2 as circunferências inscritas
nos triângulos ABC e ADC, respectivamente. Suponhamos que existe uma
circunferência w tangente à reta BA de forma que A está entre B e o ponto de
tangência, tangente à reta BC de forma que C está entre B e o ponto de tangência, e
que também seja tangente às retas AD e CD. Prove que as tangentes comuns
exteriores a w1 e w2 se intersectam sobre w.
É claro que um problema de geometria não pode ficar sem um bom
desenho. É razoavelmente difícil desenhar a figura do problema e sugerimos que o
leitor tente fazê-lo por conta própria (dica: comece com o círculo w ). Não se perca:
queremos provar que o ponto Z está sobre a circunferência w.

C
w
K B
D w O2 O1
Z 2
w1
O L

Quem já estudou homotetia já deve ter enxergado diversas homotetias entre as


circunferências, mas muitos dos mais poderosos olímpicos do mundo foram
derrotados por esse problema. De fato, dos 535 estudantes que participaram da
IMO 2008, somente 13 resolveram (um deles fez 6 pontos) e 53 conseguiram pelo
menos um ponto. Isto quer dizer que mais de 90% dos estudantes zeraram o
problema!

EUREKA! N°29, 2009

32
Sociedade Brasileira de Matemática

Isso é sinal de que esse problema deve ter algo novo para ser explorado. De fato,
uma transformação geométrica que esteve em voga nos anos 80 e desapareceu nos
anos 90 foi a homotetia. E ela voltou, discretamente em 2007 e com tudo em 2008!
Vamos definir homotetia, ver algumas de suas propriedades e expandir as adéias
envolvidas nessa transformação.

1. Homotetia: definição
Você vai ver que homotetia nada mais é do que “fazer sombrinha”. Aparecem
muitos paralelismos, mas o mais interessante são as colinearidades que irão
aparecer. No início parece mágica; mas um bom matemático sempre revela seus
truques!
Vamos começar com a definição de homotetia com razão positiva ou homotetia
direta:

Definição 1.1. Homotetia de uma figura F com centro O e razão k, um número real
positivo, é uma transformação geométrica que associa a cada ponto P de F o ponto
P´ sobre a semi-reta OP, de origem O, tal que OP´= k ⋅ OP.

F F´
O
B

Talvez com vetores seja mais interessante: sendo O o centro da homotetia, o ponto
uuur uuur
P é transformado no ponto P´ de modo que OP´ = k ⋅ OP. Note que a homotetia é
uma função σ que leva pontos do plano (ou do espaço, se você estiver trabalhando
em dimensões maiores) a pontos do plano (espaço). De fato, podemos fazer
P´= σ ( P ), tal que σ ( P) − O = k ⋅ ( P − O) ⇔ σ ( P) = O + k ⋅ ( P − O).
Com isso, podemos definir homotetias para k negativo também, obtendo as
chamadas homotetias de razão negativa ou homotetias inversas:

EUREKA! N°29, 2009

33
Sociedade Brasileira de Matemática

Definição 1.2. Homotetia de uma figura F com centro O e razão k, sendo k um


número real negativo, é uma transformação geométrica que associa a cada ponto
uuur uuur
P de F o ponto P´sobre a reta OP, de origem O, tal que OP´ = k ⋅ OP.

A
C

O


2. Propriedades da homotetia
As principais propriedades de homotetias têm a ver com colinearidade e
concorrência. Algumas têm a ver com paralelismo.

2.1. Colinearidade
• O centro de homotetia, o ponto e seu transformado são colineares. Em
outras palavras, O, P e P´= σ ( P ) são colineares. Isso decorre diretamente
da definição, mas homotetias não vêm de graça!
Normalmente as encontramos nos problemas e, com essa propriedade,
obtemos pontos colineares.

2.2. Concorrência
• O centro de homotetia pertence a todas as retas que ligam pontos a seus
transformados. Em outras palavras, O pertence a toda reta do tipo
PP´= Pσ ( P). Novamente, uma propriedade que decorre diretamente da
definição (na verdade, é a mesma da colinearidade!), mas que aparece
quando descobrimos alguma homotetia.

2.3. Paralelismo
• A reta que liga dois pontos é paralela à reta que liga os seus tranformados.
Em outras palavras, PQ e P´Q´= σ ( P )σ (Q ) são paralelas. A
demonstração desse fato vem da semelhança entre OPQ e OP´Q´ (pelo
caso LAL´) .

EUREKA! N°29, 2009

34
Sociedade Brasileira de Matemática

• Dois triângulos com lados respectivamente paralelos são homotéticos.


Para provar isso, sendo ABC e DEF os triãgulos com AB, DE, AC, DF e
BC, EF respectivamente paralelos, use o teorema de Desargues para
provar que esses triângulos são perspectivos.

Em particular, algumas figuras são sempre semelhantes: os círculos! Com isso,


temos a seguinte propriedade:

2.4. Círculos

• Dois círculos são sempre homotéticos. Na maioria dos casos, eles admitem
duas homotetias, uma direta e uma inversa. No caso de círculos disjuntos,
os centros de homotetias são fáceis de encontrar: são as interseções das
tangentes comuns internas (inversa) e das tangentes comuns externas
(direta).

A O–
B O+

Com isso, podemos resolver alguns problemas. Homotetia esteve bastante na moda
na IMO durante o início dos anos 80, como você vai ver nos exemplos e nos
exercícios.

Exemplo 2.1.

Problema 5, IMO 1981. Três círculos congruentes têm um ponto comum O e estão
no interior de um triângulo. Cada círculo é tangente a dois lados do triângulo.
Prove que o incentro e o circuncentro do triângulo e o ponto O são colineares.

Resolução:
O nome do ponto dado não é O por acaso: sejam A, B e C os centros dos três
círculos congruentes e A´B´C´ o triângulo cujos lados tangenciam esses três

EUREKA! N°29, 2009

35
Sociedade Brasileira de Matemática

círculos. Note que os raios dos círculos congruentes são OA = OB = OC , isto é, O é


circuncentro de ABC. Além disso, das tangências dos círculos com os lados temos
que AA´, BB´ e CC´são as bissetrizes do triângulo A´B´C´ e se interceptam no
incentro I do triângulo.

I O´
A O
B

A´ B´

As distâncias de A e B a A´B´ são iguais aos raios dos círculos congruentes a são,
portanto, iguais.
Então AB e A´B´ são paralelos. Analogamente, AC é paralelo a A´C´ e BC é
paralelo a B´C´, de modo que os triângulos ABC e A´B´C´ são homotéticos. O
centro de homotetia é I. Essa homotetia leva O ao circuncentro O´ de A´B´C´.
Assim, I, O e O´ são colineares.
Note que a dificuldade foi achar a homotetia; depois bastou aplicar a propriedade
de colinearidade.

Exercícios:
01. (Problema 2, IMO 1982) Seja A1 A2 A3 um triângulo escaleno com lados a1 , a2 e a3
( ai é o lado oposto a Ai ). Seja M i o ponto médio do lado ai e Ti o ponto onde o
incírculo do triângulo toca o lado ai , para i = 1, 2, 3. Seja Si o simétrico de Ti em
relação à bissetriz interna do ângulo Ai . Prove que as retas M 1S1 , M 2 S2 e M 3 S3 são
concorrentes.

EUREKA! N°29, 2009

36
Sociedade Brasileira de Matemática

02. (Problema 2, IMO 1983) Seja A um dos dois pontos de interseção dos círculos C1
e C2 , de centros O1 e O2 , respectivamente. Uma das tangentes comuns aos
círculos toca C1 em P1 e C2 em P2 , e a outra toca C1 em Q1 e C2 em Q2 . Seja
M 1 o ponto médio de PQ 1 1 e M2 o ponto médio de P2Q2 . Prove que
∠O1 AO2 = ∠M 1 AM 2 .

03. (Prova de seleção 2008, Banco da IMO 2007) As diagonais do trapézio ABCD
cortam-se no ponto P. O ponto Q está na região determinada pelas retas paralelas
BC e AD tal que ∠AQD = ∠CQB e a reta CD corta o segmento PQ. Prove que
∠BQP = ∠DAQ.

3. O Fenômeno Homotético Circular


Algumas aplicações de certos teoremas são tão conhecidos quanto os próprios. Para
homotetias, é o caso com o fenômeno homotético circular, que mostra uma
colinearidade bastante interessante envolvendo incírculo e ex-incírculo.
Fenômeno Homotético Circular. Seja ABC um triângulo e sejam K e L os pontos
de tangência do incírculo e ex-incírculo relativo a A em BC. Então A, L e o ponto
K´ diametralmente oposto a K no incírculo são colineares.
Demonstração:
A

B´ C´

I

B K L C

IA

Basta traçar a reta B´C´ paralela a BC que tangencia o incírculo de ABC em K´.
ABC e AB´C´ são homotéticos com centro em A. Para terminar, o incículo de ABC
é ex-incírculo de AB´C´, de modo que os pontos K´e L são correspondentes na
homotetia e estão, portanto, alinhados com A.

EUREKA! N°29, 2009

37
Sociedade Brasileira de Matemática

Vale a pena lembrar também que, na figura acima, BK = LC.

Exercícios:
04 (Problema 4, IMO 1992) No plano, considere uma circunferência C, uma reta
L tangente à circunferência e M um ponto da reta L. Encontre o lugar geométrico
dos pontos P com a seguinte propriedade: existem dois pontos Q, R da reta L tais
que M é o ponto médio de QR e C é a circunferência inscrita no triângulo PQR.

4. Composição de Homotetias
A principal inovação na IMO 2008 no problema 6 foi explorar o seguinte fato:

Composição de Homotetias. Se σ 1 é uma homotetia de centro O1 e σ 2 é uma


homotetia de centro O2 então a composição de homotetias σ = σ 2 o σ1 é uma
homotetia de centro O, e O1 , O2 e O estão alinhados.
A única exceção é quando a composição σ é uma translação.

Demonstração
Utilizaremos vetores para provar esse fato
Seja P um ponto qualquer e sejam k1 e k2 as razões de homotetia de σ 1 e σ 2 ,
respectivamente. Então σ 1 ( P ) = O1 + k1 ⋅ ( P − O1 ) e, portanto,
σ ( P ) = σ 2 o σ 1 ( P ) = σ 2 (σ 1 ( P )) = O2 + k 2 ⋅ (σ 1 ( P) − O2 )
= O2 + k 2 ⋅ (O1 + k1 ⋅ ( P − O1 ) − O2 )
= k2 (1 − k1 ) ⋅ O1 + (1 − k2 ) ⋅ O2 + k1k2 ⋅ P (*)
Primeiro, se σ é uma homotetia, então sua razão é k1k2 (as figuras são
“multiplicadas” por k1 e depois por k2 ; ou seja, são “multiplicadas” por k1k2 ).
Assim, para provarmos que σ é uma homotetia, temos que provar que existe um
ponto O tal que
σ ( P ) = O + k1k2 ⋅ ( P − O ) = (1 − k1k2 ) ⋅ O + k1k2 ⋅ P (**)

Comparando os coeficientes em (*) e (**) concluímos que


(1 − k1k2 )O = k2 (1 − k1 ) ⋅ O1 + (1 − k2 ) ⋅ O2 . Se k1k2 = 1, σ é uma translação
k (1 − k1 ) ⋅ O1 + (1 − k2 ) ⋅ O2
(verifique!). Caso contrário, O= 2 e, como
1 − k1k2
k2 (1 − k1 ) + (1 − k2 ) = k2 − k1k2 + 1 − k2 = 1 − k1k2 , O é uma média ponderada de O1 e
O2 . Em outras palavras, O pertence à reta O1O2 .

EUREKA! N°29, 2009

38
Sociedade Brasileira de Matemática

Os partidários da geometria sintética devem estar sentindo falta de uma


demonstração sintética. Vamos provar a parte da colinearidade sinteticamente.

Demonstração sintética da colinearidade


Considere os pontos P e Q e seus transformados
P1 = σ 1 ( P), Q1 = σ 1 (Q), P2 = σ 2 (P1 ) = σ (P ) e Q2 = σ 2 (Q1 ) = σ (Q ).

O2 P1 P2

P
O
O1 Q Q1

Q2

Note que, das homotetias, PQ PQ 1 1 e P2 Q2 são paralelos. Em termos projetivos,


eles são concorrentes em um ponto do infinito. Isto quer dizer que os triângulos
1 2 e QQ1Q2 são perspectivos e podemos aplicar o teorema de Desargues: as
PPP
interseções entre lados correspondentes, PP1 ∩ QQ1 = {O1}, P1 P2 ∩ Q1Q2 = {O2 } e
PP2 ∩ QQ2 = {O} são colineares.

4.1 Detalhe técnico


Geralmente, trabalhamos com homotetias sinteticamente, e aparecem homotetias
diretas e inversas. Homotetias inversas “multiplicam” figuras por fatores negativos,
de modo que a composição de duas homotetias do mesmo tipo é direta e a
composição de duas homotetias de tipos diferentes é inversa. Para facilitar, a
homotetia inversa faz o papel do sinal de menos e a homotetia direta, do sinal de
mais. Na composição de homotetias, seguimos a regra dos sinais da multiplicação.

Agora estamos prontos para resolver o problema 6 da IMO 2008. Vamos


reenunciar o problema e resolvê-lo.

Exemplo 4.1
Problema 6, IMO 2008. Seja ABCD um quadrilátero convexo cujos lados BA e
BC têm comprimentos diferentes. Sejam w1 e w2 as circunferências inscritas nos
triângulos ABC e ADC, respectivamente. Suponhamos que existe um

EUREKA! N°29, 2009

39
Sociedade Brasileira de Matemática

circunferância w tangente à reta BA de forma que A está entre B e o ponto de


tangência, tangente à reta BC de forma que C está entre B e o ponto de tangência, e
que também seja tangente às retas AD e CD. Prove que as tangentes comuns
exteriores a w1 e w2 se intersectam sobre w.

Resolução
Vamos começar trabalhando com segmentos tangentes.

E C
w G
Z O2 K B
D O1
w2
O L w1

A
H
F

Temos BE = BF, AF = AG, CE = CH e DG = DH. Então AB = BF – AF = BE – AG


= BC + CE – (AD + DG) = BC – AD + (CH – DH) = BC – AD + CD ⇒ AB + AD =
BC + CD. Note que esse fato depende somente de w ser tangente aos
prolongamentos dos lados do quadrilátero ABCD (guarde esse fato, ele pode ser
útil em outros problemas!). Isso implica

AC + CD − AD AC + AB − BC
= ⇔ CK = AL.
2 2

Essa igualdade é simples, mas abre muitas portas para nós! De fato, ela quer dizer
que os ex-incírculos w3 e w4 relativos a AC dos triângulos ABC e ADC tocam AC
em K e L, respectivamente. Isso nos dá muitas, mas muitas homotetias, e pelo
menos duas oportunidades de utilizar o fenômeno hometético circular!
Desenhemos as circunferências:

EUREKA! N°29, 2009

40
Sociedade Brasileira de Matemática

E C w3
w G
w4
Z B
D O2 K O1
w2 L
O w1

A
H
F

Vamos compor homotetias para descobrir colinearidades, utilizando w3 e w4 como


“intermediários”!
σ 24 σ 41 σ 21
• w2 → w4 → w1 e w2 → w1. O centro K da homotetia (direta)
σ 24 , o centro B da homotetia (direta) σ 41 e o centro Z da homotetia (direta)
σ 21 estão alinhados. Isso quer dizer que Z pertence à reta BK.
σ 23 σ 31 σ 21
• w2 → w3 → w1 e w2 → σ 1. O centro D da homotetia (direta)
σ 23 , o centro L da homotetia (direta) σ 31 e o centro Z da homotetia
(direta) σ 21 estão alinhados. Isso quer dizer que Z pertence à reta DL.
Com isso, concluímos que Z é a interseção de BK e DL.
Note que até agora não envolvemos o círculo w nas homotetias. Agora é hora, mas
vamos provar colinearidades de outra forma. Seja W a interseção de BK e w.
Provaremos que W = Z, resolvendo o problema.

r s
E C w3
w G
w4
Z K B
T O O1
O D w2 2 L w1

A
H
F

EUREKA! N°29, 2009

41
Sociedade Brasileira de Matemática

Primeiro, note que a homotetia direta σ 4 que leva w4 a w tem centro B e, portanto,
leva K a W. Mais ainda: como AC é tangente a w4 em K, a reta r paralela a AC que
passa por W é tangente a w, pois a reta AC é levada a r por σ 4 .
Agora, considere a homotetia inversa σ 2 que leva w a w2 . Essa homotetia tem
centro em D, leva W a T e r a s, que é paralela a r e AC e é tangente a w2 . Assim,
D, W e T estão alinhados, ou seja, W pertence à reta DT.
Falta ainda identificar melhor o ponto T. Na verdade, ele é bem conhecido: como s
e AC são paralelos, T e K são diametralmente opostos. Podemos, assim, aplicar o
fenômeno homotético circular: D, T e L são colineares e L também pertence à reta
DT.
Portanto D, L e W são colineares, de modo que W pertence a DL. Como W
pertence, por definição, à reta BK, W é a interseção de BK e DL, e só pode ser igual
a Z.

Observação: Note que a condição AB ≠ AC é importante para que as retas BK e


DL não coincidam.

Exercícios

05. (Banco da IMO 2007) O ponto P pertence ao lado AB do quadrilátero convexo


ABCD. Seja w o incírculo do triângulo DPD e I o seu incentro. Suponha que w é
tangente aos incírculos dos triângulos APD e BPC em K e L, respectivamente. As
retas AC e BD se encontram em E e as retas AK e BL se encontram em F. Prove
que os pontos E, I e F são colineares.

06. (Romênia) Seja ABC um triâgulo e wa , wb , wc círculos dentro de ABC tangentes


exteriormente dois a dois, tais que wa é tangente a AB e AC, wb é tangente a AB e
BC e wc é tangente a AC e BC. Sejam D o ponto de tangência entre wb e wc , E o
ponto de tangência entre wa e wc e F o ponto de tangência entre wa e wb . Prove
que as retas AD, BE e CF têm um ponto em comum.

07. (Irã) Sejam w e Ω o incírculo e o circuncírculo do triângulo ABC. w toca BC, CA


e AB em D, E e F respectivamente. Os três círculos wa , wb e wc tangenciam w em
D, E e F, respectivamente, e Ω em K, L e M, respectivamente.

(a) Prove que DK, EL e FM têm um ponto P em comum.


(b) Prove que o ortocentro do triângulo DEF pertence à reta OP.

EUREKA! N°29, 2009

42
Sociedade Brasileira de Matemática

08. Seja Γ uma circunferência e A, B e C pontos em seu interior. Construa as


seguintes três circunferências: Γ1 tangente a Γ, AB e AC; Γ 2 tangente a Γ, AB e
BC; Γ3 tangente a Γ, AC e BC. Sendo C1 , C2 e C3 os respectivos pontos de
tangência de Γ1 , Γ2 , Γ3 com Γ , prove que AC1 , BC2 e CC3 passam por um mesmo
ponto.

Você sabia…
Que 33661⋅27031232 + 1 é primo? Esse foi o décimo primeiro
primo descoberto pelo projeto "seventeen or bust" e foi
encontrado por Sturle Sunde em 17 de outubro de 2007.
Isso mostra que 33661 não é um número de Sierpinski
(números de Sierpinski são naturais ímpares k tais que
k ⋅ 2n + 1 é composto para todo n ∈ N; veja a Eureka! 18, pág.
61 e a Eureka! 25 página 56), reduzindo para 6 o número de
naturais menores que 78557 (que é o menor número de
Sierpinski conhecido), sobre os quais não se sabe se são ou
não números de Sierpinski: 10223, 21181, 22699, 24737,
55459 e 67607. Veja www.seventeenorbust.com para
mais informações (inclusive sobre como participar do
projeto).

EUREKA! N°29, 2009

43
Sociedade Brasileira de Matemática

COMO É QUE FAZ?

PROBLEMA PROPOSTO POR MARCEL MENEZES DE ANDRADE PRADO


Seja ABC um triângulo e P um ponto em seu interior tal que AP, BP e CP
intersectam os lados BC, CA e AB nos pontos D, E e F, respectivamente. Se
AP = a, BP = b, CP = c, PD = PE = PF = 3 e a + b + c = 43, determine abc.

SOLUÇÃO:
Escrevemos P em coordenadas baricêntricas:

t1 A + t2 B
F=
t1 + t2 t1 A + t3C
E=
t1 + t3
P

B C
t B + t3C
D= 2
t 2 + t3

P = t , A + t2 B + t3C , com t1 , t 2 , t3 ∈ (0,1) tais que t1 + t 2 + t3 = 1. Devemos ter então


t 2 B + t3 C  t B + t3C  t1 A + t3C
D= (note que P = t1 A + (1 − t1 )  2  = t1 A + (1 − t1 ) D ) , E =
t2 + t3  1 − t1  t1 + t3
t1 A + t2C
e F= .
t1 + t2
Temos
 1  t B + t3C
3 = PD = D − P =  − 1 ( t2 B + t3C ) − t1 A = t1 2 − A = t1 D − A =
 t2 + t3  t 2 + t3

EUREKA! N°29, 2009

44
Sociedade Brasileira de Matemática

( )
= t1 AD = t1 AP + PD = t1 (a + 3), donde t1 =
a+3
3
. Analogamente, t2 =
3
b+3
e

3 3 3 3
t3 = . Como t1 + t 2 + t3 = 1, temos + + = 1, donde
c+3 a+3 b+3 c+3
3 ( ( b + 3 )( c + 3) + ( a + 3)( c + 3) + ( a + 3)( b + 3) ) = ( a + 3)( b + 3 )( c + 3) , e logo
3 ( 6 a + 6 b + 6 c + 27 ) = abc + 9 a + 9 b + 9 c + 27, e finalmente
abc = 9 ( a + b + c ) + 54 = 9 ⋅ 43 + 54 = 441.

PROBLEMA PROPOSTO POR WILSON CARLOS DA SILVA RAMOS


Determine todas as funções F : ( 0, +∞ ) → ¡ que sejam deriváveis em x = 1 e que
satisfaçam:
F ( xy ) = xF ( y ) + yF ( x), ∀x, y ∈ (0, +∞).

SOLUÇÃO:
Fazendo y = x, obtemos F ( x 2 ) = 2 xF ( x), ∀x + (0, +∞).
Com x = 1 temos F (1) = 2 F (1), donde F (1) = 0. Vamos mostrar por indução que
2k −1
F ( x ) = 2 x F ( x), para todo inteiro positivo k e todo x ∈ (0, +∞). Para k = 1
2k k 2

isso é a afirmação anterior e, supondo que isso vale para um certo k,


2k −1 2k +1 −1
k +1
F ( x 2 ) = F (( x 2 )2 ) = 2 x 2 F ( x 2 ) = 2 ⋅ x 2 ⋅ 2k ⋅ x ⋅ F ( x) = 2k +1 ⋅ x ⋅ F ( x),
k k k k
2 2 2

como queríamos mostrar.


2k −1
h⋅
Fazendo x = e h 2k
, obtemos F (e ) = 2 e F (eh 2 ), ∀k ≥ 1.
k
h k 2k +1

2k −1
F(eh 2 ) − F(1)
h k
⋅h
Como lim 2 e k 2k +1 h 2k
F(e ) = e lim2 F (e
h2 k h 2k
) = e lim2 (e −1)lim
h2 k 2k
=
eh 2 −1
k
k →+∞ k →∞ k →∞ k →∞

= he h 2 F ´(1), temos F (e h ) = che h 2 , onde c = F ´(1), para todo h > 0.


Fazendo x = e h , obtemos F ( x) = c(log x) ⋅ x , ∀x ∈ (0, +∞).
Todas as funções desse tipo satisfazem a equação funcional.
De fato, F ( xy ) = c ( log x + log y ) xy = xF ( y ) + yF ( x), ∀x, y ∈ (0, +∞).

Resolvemos a seguir, a pedido de Mauro Felix de Sousa, três problemas da seção


“Olimpíadas ao redor do mundo”, propostos nas Eureka! No. 9 e 10.

EUREKA! N°29, 2009

45
Sociedade Brasileira de Matemática

47 (Irã – 1999)
Determine todas as funções f : ¡ → ¡ que satisfazem
f ( f ( x) + y ) = f ( x 2 − y ) + 4 f ( x) y para todos os números reais x e y.

SOLUÇÃO:
Fazendo x = y = 0, obtemos f ( f (0)) = f (0).
Fazendo x = 0, y = − f (0), obtemos f (0) = f ( f (0)) − 4 f (0)2 , donde −4 f (0) 2 = 0, e
logo f (0) = 0. Fazendo agora y = 0, obtemos f ( f ( x )) = f ( x 2 ), ∀x ∈ ¡. Se
f ( a ) = f (b ) com a ≠ b, teremos
f (b 2 − y ) = f ( f (b ) + y ) − 4 f (b ) y = f ( f ( a ) + y ) − 4 f ( a ) y = f ( a 2 − y ), ∀y ∈ ¡,
donde, se t = b 2 − a 2 , f ( z + t ) = f ( z ), ∀z ∈ ¡.
Note agora que, se t ≠ 0, fazendo y = t obtemos f ( f ( x )) = f ( f ( x ) + t ) =
= f ( x 2 − t ) + 4 f ( x )t = f ( x 2 ) + 4 f ( x )t = f ( f ( x )) + 4 f ( x )t , ∀x ∈ ¡ ,
donde
4 f ( x)t = 0, ∀x ∈ ¡, e logo f ( x) = 0, ∀x ∈ ¡, o que é uma solução (claramente).
Por outro lado, fazendo x = 0 na equação funcional, obtemos
f ( y ) = f ( − y ), ∀ y ∈ ¡ . Assim, se f não é identicamente nula,
f (a ) = f (b) ⇔ a = b ou a = −b. Em particular, como f ( f ( x)) = f ( x 2 ), ∀x ∈ ¡,
para todo x∈¡ devemos ter f ( x) = x 2 ou f (x) =−x2. Se para algum
x ≠ 0, f ( x) = − x2 , temos f (x2 − y) = f ( y − x2 ) = f ( f (x) + y) = f (x2 − y) + 4 f (x) y,∀y ∈¡,
donde −4x2 y = 4 f (x) y = 0,∀y ∈¡, absurdo. Assim, devemos ter f ( x ) = x2 , ∀ x ∈ ¡.
Isso é outra solução, pois, de fato, (x2 + y)2 = (x2 − y)2 + 4x2 y,∀x, y ∈¡.

71 (Belarus – 2000)
Determine todos os pares de inteiros positivos (m, n) que satisfazem a equação
(m − n)2 (n 2 − m) = 4m 2 n.

SOLUÇÃO:
Seja d = mdc(m, n). Temos m = da, n = db, com mdc(a , b) = 1, e a equação
equivale a (a − b) 2 (db 2 − a ) = 4a 2b. Como mdc(b, a − b) = 1, segue que b | db 2 − a,
donde b | a, e logo b = 1. Temos então (a − 1) 2 (d − a) = 4 a 2 . Como mdc(a, a −1) = 1
e (a − 1) 2 | 4a 2 , segue que (a − 1) 2 | 4, donde a – 1 = 1 ou a – 1 = 2. No primeiro
caso, temos a = 2, d – a = 16, donde d = 18, e no segundo a = 3, d – a = 9,
donde d = 12.
Assim, as duas soluções são (m, n) = (36, 18) e (m, n) = (36, 12).

EUREKA! N°29, 2009

46
Sociedade Brasileira de Matemática

89 (Balcânica – 2000)
Determinar todas as funções f :¡ → ¡ que possuem a propriedade:
f ( x f ( x) + f ( y )) = f ( x) + y, para todos os números reais x e y.
2

SOLUÇÃO:
Fazendo x = 0, obtemos f ( f ( y )) = f (0) 2 + y, ∀y ∈ ¡, donde f é uma bijeção, e logo
existe a tal que f(a) = 0. Fazendo x = a, obtemos f ( f ( y )) = y, ∀y ∈ ¡, e
logo f (0) = 0. Fazendo y = 0 obtemos então f ( xf ( x)) = f ( x)2 , ∀x ∈ ¡, donde
xf ( x) = f ( f ( x) 2 ), ou seja, f ( z 2 ) = zf ( z ), ∀z ∈ ¡ (basta fazer x = f(z), donde
f ( x) = z ). Aplicando f dos dois lados da equação obtemos
f ( f ( x) 2 + y ) = f ( f ( xf ( x) + f ( y ))) = xf ( x) + f ( y ) = f ( f ( x)2 ) + f ( y ), donde,
fazendo f ( x) = t ≥ 0, obtemos
2
f ( y + t ) = f ( y ) + f (t ), ∀y ∈ ¡, t ≥ 0 (lembramos
que f é sobrejetiva, donde t = f ( x) pode assumir qualquer valor não–negativo).
2

Assim, 2 f ( x) + f (1) = f (2 x + 1) = f (( x + 1)2 − x2 ) = f (( x + 1)2 ) − f ( x2 ) =


= ( x + 1) f ( x + 1) − xf ( x) = xf (1) + f ( x) + f (1), ∀x ∈ ¡, donde f ( x) = xf (1), ∀x ∈ ¡. .
Como f (1)2 = f ( f (1)) = 1, isso implica que as únicas soluções são
f ( x) = x, ∀x ∈ ¡ e f ( x) = − x, ∀x ∈ ¡ (é fácil ver que essas funções satisfazem o
enunciado).

Você sabia…
Que 243112609-1 e 237156667-1 são primos? Eles têm 12978189 e
11185272 dígitos respectivamente e são os dois maiores primos
conhecidos no momento. Foram descobertos em 23 de agosto de
2008 e 6 de setembro de 2008 por Edson Smith e Hans –
Michael Elvenich respectivamente, dois participantes do GIMPS.
O GIMPS é um projeto cooperativo na internet que já encontrou
12 primos de Mersenne. Veja www.mersenne.org para mais
informações, inclusive como ajudar a achar outros primos de
Mersenne.

EUREKA! N°29, 2009

47
Sociedade Brasileira de Matemática

OLIMPÍADAS AO REDOR DO MUNDO


ü Neste número apresentamos algumas soluções enviadas pelos leitores
da nossa seção.

Bruno Holanda
Carlos Augusto David Ribeiro

Obs. Nas Eurekas 25 e 27 apareceram, por descuido na edição, problemas na seção


Olimpíadas ao redor do mundo com numeração repetida. Nesses casos,
procuraremos mencionar o exemplar em que o problema foi publicado, ao nos
referirmos a um desses problemas.

üüü

224. (Balcânica Júnior – 2007) Eureka! No. 27 Seja a um real positivo tal que
a 3 = 6 ( a + 1) . Prove que a equação x 2 + ax + a 2 − 6 = 0 não possui solução real.

SOLUÇÃO DE JOSÉ DO NASCIMENTO PANTOJA JÚNIOR (FORTALEZA – CE)


6
a 3 = 6a + 6 ⇒ a( a 2 − 6) = 6 ∴ a 2 = + 6, e como a > 0 concluímos que a 2 > 6, daí:
a
a ( a 2 − 6 ) = 6 < a 2 ⇒ a 2 − a − 6 < 0, ou seja, o valor de a é tal que:

1 − 12 − 4 ⋅ 1 ⋅ ( −6 ) 1 + 12 − 4 ⋅ 1 ⋅ ( −6 )
−2 = <a< = 3, e como já sabíamos que
2 ⋅1 2 ⋅1
a > 6 > −2, fica: 6 < a < 3.
Substituindo na equação em x, (a 2
− 6 ) por (6 / a ), teríamos como raízes
6
−a ± a 2 − 4 ⋅1 ⋅
a 24
x= , mas estas raízes não são reais, pois a 2 − < 0, devido ao
2 a
fato de a 3 = 6 ( a + 1) < 6 ( 3 + 1) = 24.

226. (Inglaterra – 2007) Eureka! No. 27 Seja ABC um triângulo acutângulo com
AB > AC e ∠BAC = 60°. Seja O o circuncentro e H o ortocentro. A reta OH
encontra AB em P e AC em Q. Prove que PO = HQ.

EUREKA! N°29, 2009

48
Sociedade Brasileira de Matemática

SOLUÇÃO DE EMERSON RAMOS BARROSO (FORTALEZA – CE)


A

H O P
Q

C M B

Seja M o ponto médio de BC.


Sabemos inicialmente que
AH = 2OM (1)
e que

∠QAH = ∠OAP (2)


Temos que
∠BOC = 2∠BAC = 120° ⇒ ∠BOM = ∠MOC = 60° ⇒ OM = OB cos 60° = AO cos 60° ⇒
2OM = AO, e por (1):
AO = AH (3)
⇒ ∠AHO = ∠AOH
e como ∠AHO e ∠AOH são ângulos externos dos triângulos AQH e AOP, temos
∠HQA + ∠QAH = ∠OPA + ∠PAO e por (2):
∠AQH = ∠APO
Logo o triâgulo AQP é isósceles, onde AP = AQ. Também por (2) e (3), temos que
os triângulos APO e AQH são congruentes (A.L.A.) e assim:
PO = HQ,
como queríamos demonstrar.

229. (Bielorússia – 2001) Eureka! No. 27 No losango ABCD, ∠A = 60° . Os pontos F,


H e G estão sobre os segmentos AD, CD e AC de modo que DFGH é um
paralelogramo. Prove que FBH é um triângulo equilátero.

EUREKA! N°29, 2009

49
Sociedade Brasileira de Matemática

SOLUÇÃO DE ANTONIO MARCOS S. ALMEIDA (MANAUS – AM)


A

30º
l
l
F
b
? l 60º
B D

? G a
b
l
l
H
30º 30º
b
C
µ ≡ H CG
Sendo AD // GH , pois DFGH é paralelogramo, temos CGH µ e o triângulo
HCG é isósceles, logo HC = GH = b. Mas FD = b, logo os triângulos BCH e
µ = 60°, já que ABCD é losango.
BDF são congruentes, pois BC = BD = l e F DB
Portanto BH ≡ BF e CBH µ ≡ DBFµ .
Observe que CBD µ ≡ CBH µ + H BD
µ ≡ H BD µ + DBFµ ≡ H BF µ = 60°. Desta forma
conclui-se que o triângulo BFH é equilátero, pois é isósceles e tem o ângulo do
vértice de 60°.

230. (Rússia – 2007) Eureka! No. 27


Sejam a, b, c números reais. Prove que pelo menos uma das três equações
x 2 + (a − b) x + (b − c) = 0,
x 2 + (b − c ) x + ( c − a ) = 0
x 2 + (c − a ) x + ( a − b ) = 0
possui solução real.

SOLUÇÃO DE JEAN PIERRE YOUYOUTE (RIO DE JANEIRO - RJ)


Analisaremos o caso em que as duas primeiras equações não tem solução real e
provaremos que a terceira possui solução real. Os demais casos são análogos.
∆1 = (a − b) 2 − 4(b − c) < 0
⇒ (a − b) 2 + (b − c) 2 < 4(b − a ).
∆ 2 = (b − c) − 4(c − a ) < 0
2

EUREKA! N°29, 2009

50
Sociedade Brasileira de Matemática

Como a, b, c são reais, então:


0 < (a − b)2 + (b − c)2 < 4(b − a ) ⇒ 4(b − a) > 0 ⇔ 4(a − b) < 0
∆ 3 = (c − a) 2 − 4(a − b)
∆ 3 > 0 ⇔ (c − a )2 > 4(a − b). Mas (c − a )2 ≥ 0 > 4(a − b) ⇒ (c − a ) 2 > 4(a − b).
Portanto, a terceira equação possui solução real.

235. (Olimpíada Checa e Eslovaca – 2007) Eureka! No. 27


Se x, y, z são números reais no intervalo (–1, 1) satisfazendo xy + yz + zx = 1,
mostre que
6 3 (1 − x 2 )(1 − y 2 )(1 − z 2 ) ≤ 1 + ( x + y + z ) 2

SOLUÇÃO DE JEAN PIERRE YOUYOUTE (RIO DE JANEIRO - RJ)


1 + ( x + y + z ) 2 ≥ 6 3 (1 − x 2 )(1 − y 2 )(1 − z 2 ) ⇔
1 + x 2 + y 2 + z 2 + 2( xy + yz + zx) ≥ 6 3 (1 − x 2 )(1 − y 2 )(1 − z 2 ) ⇔
3 + x 2 + y 2 + z 2 ≥ 6 3 (1 − x 2 )(1 − y 2 )(1 − z 2 ).
Aplicando a desigualdade das médias obtemos:
(1 + x)(1 − y ) + (1 + y )(1 − z ) + (1 + z )(1 − x) ≥ 3 3 (1 − x 2 )(1 − y 2 )(1 − z 2 ) ⇔
1 + x − x − xy + 1 + y − z − yz + 1 + z − x − zx ≥ 3 3 (1 − x 2 )(1 − y 2 )(1 − z 2 ) ⇔
3 − ( xy + yz + zx) ≥ 3 3 (1 − x 2 )(1 − y 2 )(1 − z 2 ) ⇔
3 + 1 ≥ 6 3 (1 − x 2 )(1 − y 2 )(1 − z 2 ).
Mas x 2 + y 2 + z 2 ≥ xy + yz + zx ⇔ x 2 + y 2 + z 2 ≥ 1.
Logo,
3 + x 2 + y 2 + z 2 ≥ 3 + 1 ≥ 6 3 (1 − x 2 )(1 − y 2 )(1 − z 2 ) ⇒
3 + x 2 + y 2 + z 2 ≥ 6 3 (1 − x 2 )(1 − y 2 )(1 − z 2 ).

üüü

EUREKA! N°29, 2009

51
Sociedade Brasileira de Matemática

SOLUÇÕES DE PROBLEMAS PROPOSTOS


! Publicamos aqui algumas das respostas enviadas por nossos leitores.

113. a1 , a2 , a3 ,... formam uma seqüência de inteiros positivos menores que 2007
a + an
tais que m é inteiro, para quaisquer inteiros positivos m, n.
am + n
Prove que a seqüência (an) é periódica a partir de um certo ponto.

SOLUÇÃO DE ZOROASTRO AZAMBUJA NETO (RIO DE JANEIRO – RJ)


Seja c < 2007 o maior inteiro positivo tal que a seqüência (an )r∈¥ tem infinitos
termos iguais a c.
Existe n0 ∈ ¥ tal que an ≤ c, ∀n > n0 . Note que, se N , n ∈ ¥ são tais que
n0 < N − n < N e aN = an = c, então, como c = aN = a( N −n)+n divide aN −n + an = aN −n + c,
segue que c | aN − n e, como aN − n ≤ c, devemos ter aN − n = c.
Precisaremos agora do seguinte
Lema: Para quaisquer inteiros positivos b1 , b2 ,..., bk , existe m0 ∈ ¥ tal que qualquer
múltiplo de d = mdc(b1 , b2 ,..., bk ) que seja maior que m0 pode ser escrito como
r1b1 + r2b2 + ... + rk bk com r1 , r2 ,..., rk ∈ ¥.
Prova: Vamos usar indução em k. Para k = 1 o resultado é óbvio.
Suponhamos que exista m ° 0 tal que, se d° = mdc(b , b ,...b ), então todo múltiplo de
1 2 k
° °
d maior que m 0 se escreva como r ⋅ b + ... + r b , com r ∈ ¥, ∀i ≤ k . Sejam agora
1 1 k k i

d = mdc(b1 , b2 ,..., bk , bk +1 ) = mdc( d° , bk +1 ) e m0 = m


° 0 + b d° . Sabemos que d pode ser
k +1
° °
escrito como dx + b y, com x, y ∈ ¢. Se t ⋅ d = d ⋅ (tx) + b (ty ) > m , tomamos
k +1 k +1 0

q, s ∈ ¢ com ty = q ⋅ d° + s e 0 ≤ s < d° .
Temos então t ⋅ d = d° ⋅ (tx + q ⋅ bk +1 ) + bk +1 (ty − q ⋅ d° ) = d° ⋅ (tx + q ⋅ bk +1 ) + bk +1 ⋅ s.
Temos s ∈ ¥, e d° ⋅ (tx + q ⋅ b ) = td − b ⋅ s > m − b ⋅ s > m − b ⋅ d° = m
k +1 k +1 0 k +1 0
°0,
k +1

donde, pela hipótese de indução, existem r1 , r2 ,..., rk ∈ ¥ com


d° ⋅ (tx + q ⋅ bk +1 ) = r1b1 + r2b2 + ... + rk bk , e, definindo rk +1 = s ∈ ¥, teremos
t ⋅ d = r1b1 + r2b2 + ... + rk bk + rk +1bk +1 , o que termina a prova do Lema.
Sejam agora X = {n ∈ ¥ | an = c}, e d o máximo divisor comum dos elementos de
X. Existem b1 , b2 ,..., bk ∈ X com mdc(b1 , b2 ,..., bk ) = d , e existe m0 ∈ ¥ tal que todo
múltiplo de d maior que m0 se escreve como r1b1 + r2b2 + ... + rk bk com

EUREKA! N°29, 2009

52
Sociedade Brasileira de Matemática

r1 , r2 ,..., rk ∈ ¥, pelo Lema. Afirmamos que, se se n > n0 é múltiplo de d então


an = c. De fato, como vimos anteriormente, se N > N − bi > n0 e aN = c então
aN − bi = c, para todo i ≤ k .
° > n + m tal que a ° = c, temos N
Tomando N ° −n >m
° múltiplo de d, donde N
0 N 0

também é múltiplo de d, e logo se escreve como r1b1 + r2 b2 + ... + rk bk , com


r1 , r2 ,..., rk ∈ ¥. Aplicando a observação acima várias vezes, concluímos que
an = aN − r1b1 − r2b2 ,...,rk bk é igual a c, o que mostra nossa afirmação.
Seja agora n > m0 .
Seja t inteiro positivo tal que n + t é múltiplo de d.
Temos então que c = an + t divide an + at , e, analogamente, c = an + d + t divide
an + d + at , donde c divide ( an + d + at ) − ( an + at ) = an + d − an . Como an e an + d são
inteiros positivos menores ou iguais a c, devemos ter an + d = an . Assim, an + d = an
para todo n > n0 , o que prova o resultado.
115. Suponha que ABC é um triângulo com lados inteiros a, b e c com µ = 60°
BCA
e mdc(a, b) = mdc(a , c) = mdc(b, c) = 1.
Prove que c ≡ 1(mod 6) .

SOLUÇÃO DE EDEL PÉREZ CASTILLO (PINAR DEL RIO – CUBA)


c 2 = a 2 + b 2 − 2ab cos(60°) = a 2 + b 2 − ab
Demonstremos que c é ímpar.
Como mdc(a , b) = 1 então a é ímpar ou b é ímpar. Suponhamos que b é ímpar.
Se a é par ou ímpar então c é ímpar.
Demonstremos que c não é divisível por 3.
Se 3 | c ⇒ 9 | c 2 então c 2 = a 2 + b 2 − ab = (a + b) 2 − 3ab
3 | ( a + b ) ⇒ 3 | ( a + b ) ⇒ 9 | ( a + b ) ⇒ 9 | 3ab ⇒ 3 | ab
2 2

Então 3 | a ou 3 | b, o que em qualquer caso contradiz que


mdc(a , b) = mdc(a , c) = mdc(b, c) = 1.
Logo c não é divisível por 3.
Demonstremos que c ≡ 1(mod 3).
c2 = a 2 + b2 − ab ⇒ 4c2 = 4a2 − 4ab + 4b2 = ( 2a − b) + 3b2 ⇒ ( 2c − 2a + b)( 2c + 2a − b) = 3b2 .
2

Os números ( 2c − 2a + b ) e ( 2c + 2a − b ) são ímpares porque b é ímpar.

EUREKA! N°29, 2009

53
Sociedade Brasileira de Matemática

Seja d um divisor comum de ( 2c − 2a + b ) e ( 2c + 2a − b ) . Então d | 4c e


d | 2(a − b) porém como d é ímpar temos que d | c e d | (2a − b). Já demonstramos
que mcd ( c,3) = 1 então mcd (d ,3) = 1.

Da equação 4c 2 = ( 2a − b ) + 3b 2 deduzimos que d 2 | 3b 2 ⇒ d 2 | b 2 ⇒ d | b ⇒ d = 1


2

(porque d | c e mcd ( b, c ) = 1 ).
Como mcd ( 2c − 2a + b , 2c + 2a − b ) = 1 e seu produto é 3b 2 temos que:
2c − 2 a + b = 3 x 2
2c + 2 a − b = y 2
Ou também poderia ser:
2c − 2 a + b = y 2
2c + 2 a − b = 3 x 2
Os valores de x, y são ímpares porque b é ímpar.
Em qualquer caso temos que c = ( 3 x 2 + y 2 ) 4

3( x2 − y2 )
c− y = 2
⇒ c − y 2 ≡ 0(mod 3) ⇒ c ≡ y 2 (mod 3)
4
Logo c ≡ 1(mod3), porque c não é divisível por 3.
Como c ≡ 1(mod3) então c ≡ 1(mod 6) ou c ≡ 4(mod 6), porém c é ímpar, logo
c ≡ 1(mod 6).

119. Mostre que não existem inteiros positivos a e b tais que


( 36 a + b )( 36b + a ) seja uma potência de 2.

SOLUÇÃO DE MARCILIO MIRANDA DE CARVALHO (TERESINA – PI)


Se ( 36a + b )( 36b + a ) ( 36a + b ) e ( 36b + a ) são
é uma potência de 2, então
potências de 2 maiores que 36. Daí ( 36a + b ) e são ( 36b + a ) são pares, logo a e b
são pares. Agora seja S o conjunto de pares (a, b) tais que ( 36a + b )( 36b + a ) seja
uma potência de 2. Se S não é vazio então S possui um par (a, b) tal que a é
mínimo. Como a e b são pares então a = 2c e b = 2d, logo 4 ( 36c + d )( 36d + c ) é
uma potência de 2, daí ( 36c + d )( 36d + c ) é uma potência de 2, onde c < a,
absurdo!

EUREKA! N°29, 2009

54
Sociedade Brasileira de Matemática

120. Sejam a, b, c números reais e soma Sn definida como Sn = a n + bn + cn , para


qualquer n inteiro não negativo. Sabe-se que S1 = 2, S2 = 6 e S3 = 14. Mostre que
Sn2 − Sn −1 ⋅ Sn +1 = 8 para todo inteiro n > 1.

SOLUÇÃO DE MARIA CLARA MENDES SILVA (PIRAJUBA – MG)


Vamos definir como:
Ω1 = a + b + c
Ω 2 = ab + ac + bc
Ω3 = abc
Veja que:
an + bn + c n = ( a + b + c ) ( an−1 + bn−1 + c n−1 ) − ( ab + bc + ac ) ( an −2 + bn −2 + cn−2 )
+ abc ( a n −3 + bn −3 + c n −3 ) .
Isso pode ser verificado diretamente abrindo os produtos.
Substituindo com a nossa notação:
Sn = Ω1 Sn −1 − Ω 2 S n − 2 + Ω3 Sn − 3 .
Assim podemos definir Sn por recorrência.
Falta descobrir Ω1 , Ω 2 e Ω3 .
Inicialmente note que S1 = a + b + c = 2.
Assim Ω1 = 2.
Elevando a + b + c ao quadrado:
4 = ( a + b + c ) = a 2 + b2 + c 2 + 2 ( ab + bc + ac ) = S 2 + 2Ω 2 = 6 + 2Ω2 .
2

Resolvendo achamos Ω 2 = −1.


Finalmente:
14 = a 3 + b3 + c 3 = ( a + b + c ) ( a 2 + b2 + c 2 − ab − ac − bc ) + 3abc =
= 2 ⋅ (6 + 1) + 3Ω3 = 14 + 3Ω 3 ⇒ Ω3 = 0.
Na equação de recorrência que tínhamos obtido anteriormente:
Sn = 2 S n −1 + S n − 2 .
Agora usaremos indução em n:
Para n = 2, o que queremos demonstrar é claramente verdadeiro, uma vez que
S2 2 − S1S3 = 62 − 28 = 8.
Suponhamos que essa propriedade seja verdadeira para todo natural menor que um
certo n, queremos provar que também vale para n.
Sn2 − S n −1 ⋅ S n +1 = Sn2 − S n −1 ⋅ ( 2 Sn + Sn −1 ) = Sn2 − 2S n −1 Sn − S n2−1 =

EUREKA! N°29, 2009

55
Sociedade Brasileira de Matemática

= − Sn2−1 − S n ( 2S n −1 − S n ) .
Como 2 S n −1 + S n − 2 = S n ,
2 S n −1 − Sn = − Sn − 2 .
Substituindo:
Sn2 − S n −1 ⋅ S n +1 = − S n2−1 + S n ⋅ S n − 2 .
Essa expressão tem o mesmo módulo do que a obtida se trocarmos todos seus
sinais, ou seja Sn2−1 − S n − 2 ⋅ S n . Mas pela hipótese de indução o módulo desta é 8.
Assim, o módulo de Sn2 − S n −1S n +1 é 8. Por indução provamos que essa sentença é
válida para todo n inteiro positivo.

121. Na figura abaixo o lado do quadrado vale 4, obter o valor da altura h para que a
área da região 1 seja igual a área da região 2.

2
4 4
3 h

SOLUÇÃO DE BRUNO SALGUEIRO FANEGO (VIVEIRO – ESPANHA)


A união das regiões 1 e 3 é um quadrante de um semicírculo de raio 4 (de área
π ⋅ 42
) e a união das regiões 2 e 3, um retângulo de lados 4 e h (de área 4 ⋅ h),
4
donde, sendo Ai a área da região i, 1 ≤ i ≤ 3, teremos
π ⋅ 42
A1 = A2 ⇔ A1 + A3 = A2 + A3 ⇔ = 4⋅h ⇔ h = π.
4

122. Dado um triângulo ABC tal que AB = AC = a + b e BC = a , traça-se uma


ceviana partindo de B determinando em AC um ponto D tal que DA = a e DC = b .
Sabendo que ABDµ = 10° , determine os ângulos internos desse triângulo.

EUREKA! N°29, 2009

56
Sociedade Brasileira de Matemática

SOLUÇÃO DE CARLOS ALBERTO DA SILVA VICTOR (NILÓPOLIS – RJ)


A
 AD = BC = a
? 
 AB = AC
D

10°
B C

Tomando ∠BAC = θ e, usando que AD = BC e a lei dos senos nos triângulos ABD
θ 
e BDC, encontramos: sen10° = 2 sen   ⋅ sen (θ + 10° ) .
2  
Observe que θ = 20° é solução. Vamos mostrar que ela é única para o triângulo em
questão.
Não é difícil de verificar que θ < 80°, pois não poderíamos ter AD = BC para
θ 
θ ≥ 80° (BC será o maior lado); conseqüentemente, θ + 10 < 90°, e sen   e
2
sen(θ + 10°) serão crescentes no primeiro quadrante.
θ 
I) Suponha que θ > 20° ⇒ sen10° = 2sen   ⋅ sen (θ + 10° ) > 2sen10° ⋅ sen30° = sen10°
2  
(absurdo).
θ 
II) Suponha que θ < 20° ⇒ sen10° = 2 sen   ⋅ sen (θ + 10° ) < 2 sen10° ⋅ sen30° = sen10°
2  
(absurdo).

Conclusão: θ = 20° é a única solução.

127. Determine todos os inteiros positivos k tais que existem inteiros positivos x, y,
x2 + y 2 + z 2
z com = k.
xyz

SOLUÇÃO DE EDEL PÉREZ CASTILLO (PINAR DEL RIO – CUBA)


Se x = y = z = 1 obtemos k = 3; se x = y = z = 3 obtemos k = 1. Falta provar que
estes são os únicos valores que pode tomar k.
Provemos que k deve ser ímpar.

EUREKA! N°29, 2009

57
Sociedade Brasileira de Matemática

x2 + y 2 + z 2
Seja S = {( x, y , z ) ∈ (¥*)3 : = k onde k é par}
xyz
Provemos que este conjunto é vazio.
Se não é vazio existem x0 , y0 , z0 ∈ S tais que x0 + y0 + z0 é mínimo.
Temos x02 + y02 + z02 = kx0 y0 z0 . Como o membro direito é par , há dois casos:
(1) Um dos números é par e os outros dois números são ímpares.
(2) Os três são pares.
No caso (1), se x0 é par, y0 , z0 ímpares, então x02 + y02 + z02 ≡ 2(mod 4) e no outro
membro temos que kx0 y0 z0 ≡ 0(mod 4) porque k e x0 são pares. Contradição.
No caso (2) x0 = 2 x1 , y0 = 2 y1 , z0 = 2 z1 facilmente comprovamos que:
x12 + y12 + z12
= 2k , logo ( x1 , y1 , z1 ) ∈ S .
x1 y1 z1
x1 + y1 + z1 = ( x0 + y0 + z0 ) 2 < x0 + y0 + z0 .
Isto é absurdo porque x0 + y0 + z0 é mínimo.
Logo o conjunto S é vazio.
Provemos que k ≤ 3, o que resolve o problema.
x2 + y 2 + z 2
Seja C = {( x, y , z ) ∈ (¥*)3 : = k ≥ 4}
xyz
Provemos que este conjunto em C é vazio.
Se não é vazio existem x0 , y0 , z0 tais que x0 + y0 + z0 é mínimo.
Podemos supor que x0 ≤ y0 ≤ z0 .
Então:
x0 y z
+ 0 + 0 ≥4
y0 z0 x0 z0 x0 y0
x0 y0 z
Mas ≤1 e ≤ 1, donde temos que 1 + 1 + 0 ≥ 4, o que implica que
y0 z0 x0 z0 x0 y0
z0 ≥ 2 x0 y0 .
Consideremos a equação t 2 − kx0 y0 t + x02 + y02 = 0 que tem a z0 como uma das suas
soluções. Seja z1 a outra solução da equação; então temos:
(1) z0 + z1 = kx0 y0
(2) z0 z1 = x02 + y02
De (1) deduzimos que z1 é inteiro, de (2) deduzimos que z1 é positivo. Logo
( x0 , y0 , z1 ) ∈ C mas x0 + y0 + z0 ≤ x0 + y0 + z1 porque x0 + y0 + z0 é mínimo.

EUREKA! N°29, 2009

58
Sociedade Brasileira de Matemática

Então 2 x0 y0 ≤ z0 ≤ z1 , e substituindo em (2) obtemos 4 z02 y02 ≤ x02 + y02 ≤ 2 y02 donde se
obtém que 4 x02 ≤ 2 , o que é absurdo. Portanto o conjunto C é vazio.

128. Barango Joe era um sapo de mútiplos talentos que habitava a Terra das
Chances Diminutas, localizada no alto de uma montanha.
Após sua maioridade, Barango Joe decidiu tentar a vida no Reino das Grandes
Oportunidades, localizado no cume da montanha vizinha.
Para isso, ele atravessaria a extensa ponte de madeira por cima do Desfiladeiro da
Morte. Entretanto, a ponte era guardada pela Esfinge Vegas, exímia jogadora que
sempre desafiava os viajantes para algum jogo. O viajante vitorioso tinha a
passagem franqueada; e o perdedor era lançado ao abismo.
Assim chegando à cabeceira da ponte, Barango Joe foi desafiado a uma partida de
“Pachang” jogo que lembra o “Black Jack” ou “Vinte e um”, mas é jogado por 2
oponentes da seguinte maneira:
Os jogadores, designados por “banca” e “apostador”, utilizam um dado gerador de
números aleatórios reais uniformemente distribuídos no intervalo [0,1].
Inicialmente, a banca sorteia um número X. Se não estiver satisfeita com o número
obtido, pode descartá-lo e então sortear um novo número. Este procedimento pode
ser executado 2 vezes, Isto é, pode haver até 3 sorteios na definição do número X
da banca.
Então, o apostador sorteia quantos números forem necessários até que a soma de
seus números ultrapasse o número X da banca. Neste momento, se esta soma for
inferior a 1, o apostador ganha; caso contrário, perde.
Ou seja, para ganhar, o apostador precisa “chegar mais próximo” de 1 que a banca,
sem no entanto “estourar o limite” de 1.
Após explicar as regras do Pachang, a Esfinge Vegas deu uma opção ao sapo:
- Você prefere ser a banca ou o apostador?
O que o Barango Joe deveria responder?
Obs. Utilize lápis, papel, e uma calculadora científica simples.

SOLUÇÃO DE RAFAEL TUPYNAMBÁ DUTRA (BELO HORIZONTE – MG)


Primeiramente, supondo que X já tenha sido escolhido, vamos determinar a função
f ( X ) que dá a probabilidade de o apostador ganhar. Seja pn (x) a função
densidade de probabilidade para a soma dos números do apostador (ou seja, após
n números sorteados pelo apostador, a probabilidade de a soma dos n – 1 ser
menor ou igual a X e a soma desses números estar entre a e b é dada por
b

∫ p ( x)dx ). Definimos
a
n qn (x) igual a pn (x) para x ∈ [0, X ] e qn (x) igual a 0

EUREKA! N°29, 2009

59
Sociedade Brasileira de Matemática

1
caso contrário. Seja também g n = ∫ p ( x)dx
X
n a probabilidade de o apostador

ganhar após n números sorteados. Temos p1 ( x) = 1, ∀x ∈ [0,1] .


x n −1 X n −1
Lema: qn ( x) = pn ( x) = para x ∈ [0, X ] e pn ( x) = para
(n − 1)! (n − 1)!
x ∈ ( X ,1] .
Prova por indução: o caso inicial n = 1 é trivial. Hipótese de indução:
x n −1
qn ( x) = para x ∈ [0, X ] . Caso o jogo continue, a densidade de
(n − 1)!
probabilidade da soma dos n primeiros números é dada por qn (x) (afinal, para o
jogo continuar, essa soma deve estar no intervalo [0, X ] ). E a densidade de
probabilidade do (n + 1) -ésimo número é igual a p1 ( x) = 1∀x ∈ [0,1] . Assim,
x
somando, obtemos pn +1 ( x) = ∫ q (t )dt .
x −1
n Dessa forma, para x ∈ [0, X ] , temos

t n −1
x x
xn
pn +1 ( x) = ∫ qn (t )dt = ∫
x −1 0
( n − 1 )!
dt =
n!
. E para x ∈ ( X ,1] ,

t n −1
x X
Xn
pn +1 ( x) = ∫ qn (t )dt = ∫
x −1 0
(n − 1)!
dt =
n!
, c.q.d.

X n −1
1
Pelo lema, temos g n = ∫ pn ( x)dx = (1 − X )
X
(n − 1)!
e a probabilidade de o

∞ ∞
X n −1
apostador ganhar é f ( X ) = ∑ g n = (1 − X )∑
n =1 n =1 ( n − 1)!
= (1 − X )e X .

Agora imagine que a banca tenha k sorteios disponíveis para determinar X .


Seja Pk a probabilidade de o apostador ganhar nesse caso. Queremos calcular P3 .
Vamos calcular Pk recursivamente.
Primeira situação: imagine que a banca tem apenas 1 sorteio disponível para
determinar X . Sabendo que, dado X , a probabilidade de o apostador ganhar é
1
f ( X ) = (1 − X )e X , descobrimos que P1 = ∫ f ( X )dX = e − 2 ≅ 0,71828 .
0

EUREKA! N°29, 2009

60
Sociedade Brasileira de Matemática

Segunda situação: imagine agora que a banca tem até 2 sorteios para determinar
X . Se, no primeiro sorteio, ela obteve um número x , ela pode ficar com esse
número ou descartá-lo. A probabilidade de o apostador ganhar será
f ( x) = (1 − x)e x no 1º caso e será P1 ≅ 0,71828 no 2º caso. Assim, a banca deve
descartar x somente se (1 − x)e x > P1 . Seja a a raiz positiva da equação
transcendente (1 − a )e a = P1 . Temos a ≅ 0,60954 . Dessa forma, a banca deve
descartar o número x se x < a e mantê-lo se x > a . Se a banca usar essa
estratégia, a probabilidade de o apostador vencer será
1
P2 = P1a + ∫ f ( X )dX =P1a + e − (2 − a)e a ≅ 0,59823 .
a
Terceira situação: agora considere o problema original (a banca tem até 3 sorteios
para determinar X ). Se, no primeiro sorteio, ela obteve o número x , ela pode
mantê-lo ou descartá-lo. A probabilidade de o apostador ganhar será
f ( x) = (1 − x)e x no 1º caso e será P2 ≅ 0,59823 no 2º caso. Assim, a banca deve
descartar x somente se (1 − x)e x > P2 . Seja b a raiz positiva da equação
transcendente (1 − b)eb = P2 . Temos b ≅ 0,70416 . Dessa forma, a banca deve
descartar o número x se x < b e mantê-lo se x > b . Se a banca usar essa
estratégia, a probabilidade de o apostador vencer será
1
1
P3 = P2b + ∫ f ( X )dX =P2b + e − (2 − b)eb ≅ 0,51915 > . Assim, o apostador tem
b
2
probabilidade maior que 1 de ganhar, mesmo quando a banca usa a melhor
2
estratégia possível. Barango Joe deve responder que prefere ser o apostador.
Agradecemos o envio de soluções e a colaboração de:
Alexandre Salim Saud de Oliveira Niterói – RJ
Alixanzito R. S. Costa Fortaleza – CE
André Felipe M. da Silva Rio de Janeiro – RJ
Doraci Gabriel da Rosa Fartura – SP
Evandro Makiyama de Melo São Paulo – SP
Flávio Antonio Alves Amparo – SP
Glauber Moreno Barbosa Rio de Janeiro – RJ
Kellem Corrêa Santos Rio de Janeiro – RJ
Larissa Brito Sousa Fortaleza – CE
Marcel Menzes de Andrade Prado Brasília – DF
Oswaldo Mello Sponquiado São Paulo – SP
Renato Carneiro de Souza Belo Horizonte – MG
Samuel Liló Abdalla Sorocaba – SP
Wallace Alves Martins Rio de Janeiro – RJ
Continuamos aguardando soluções para os problemas 123, 124, 125, 126 e 129.

EUREKA! N°29, 2009

61
Sociedade Brasileira de Matemática

PROBLEMAS PROPOSTOS
* Convidamos o leitor a enviar soluções dos problemas propostos e sugestões de novos
problemas para próximos números.

130) Suponha que a, b, c ∈ ¡ e a equação x 2 − ( a + b + c ) x + ( ab + ac + bc) = 0 não


tem raízes reais. Prove que a, b e c têm todos o mesmo sinal e existe um triângulo
de lados a, b e c.

131) a) Considere o seguinte jogo: no início um jogador A entrega um número


k ≥ 2 ao jogador B . Quando A entrega um número m ≥ 2 a B, B pode devolver
m – 1 ou m + 1 a A. Quando A recebe um número n ≥ 2 deve, se n for ímpar
n
devolver 3n a B; se n for par mas não múltiplo de 4, pode devolver ou 3n a B, e,
2
n n
se n for múltiplo de 4, pode devolver , ou 3n a B. Qualquer jogador ganha o
4 2
jogo se devolver 1 ao adversário. Caso algum jogador devolva ao adversário um
número maior que 1000k, o jogo empata. Determine, para cada valor de k ≥ 2 , se
algum dos jogadores tem estratégia vencedora, e, nesses casos, qual deles.

b) Resolva o item anterior supondo que A, ao receber um número n ≥ 2, deve


n
devolver 3n a B se n for ímpar, deve devolver a B se n for par mas não múltiplo
2
n
de 4 e deve devolver a B se n for múltiplo de 4.
4

132) a) Considere uma família ℑ de 2000 círculos de raio 1 no plano tal que dois
círculos de ℑ nunca são tangentes e cada círculo de ℑ intersecta pelo menos dois
outros círculos de ℑ . Determine o número mínimo possível de pontos do plano
que pertencem a pelo menos dois círculos de ℑ .

Problema 130 proposto por Wílson Carlos da Silva Ramos (Belém – PA), problema 131
proposto por Benedito Tadeu Vasconcelos Freire (Natal – RN), problema 132 proposto por
Juan Manuel Conde Calero (Alicante – Espanha).

EUREKA! N°29, 2009

62
Sociedade Brasileira de Matemática

AGENDA OLÍMPICA
XXXI OLIMPÍADA BRASILEIRA DE MATEMÁTICA

NÍVEIS 1, 2 e 3
Primeira Fase – Sábado, 06 de junho de 2009
Segunda Fase – Sábado, 12 de setembro de 2009
Terceira Fase – Sábado, 17 de outubro de 2009 (níveis 1, 2 e 3)
Domingo, 18 de outubro de 2009 (níveis 2 e 3 - segundo dia de prova).

NÍVEL UNIVERSITÁRIO
Primeira Fase – Sábado, 12 de setembro de 2009
Segunda Fase – Sábado, 17 e Domingo, 18 de outubro de 2008

XV OLIMPÍADA DE MAIO
09 de maio de 2009

XX OLIMPÍADA DE MATEMÁTICA DO CONE SUL
14 a 20 de abril de 2009
Mar del Plata – Argentina

L OLIMPÍADA INTERNACIONAL DE MATEMÁTICA
10 a 22 de julho de 2009
Bremen – Alemanha

XVI OLIMPÍADA INTERNACIONAL DE MATEMÁTICA UNIVERSITÁRIA
25 a 30 de julho de 2009
Budapeste, Hungria

XXIV OLIMPÍADA IBEROAMERICANA DE MATEMÁTICA
Setembro de 2009
Mérida – México

XII OLIMPÍADA IBEROAMERICANA DE MATEMÁTICA UNIVERSITÁRIA

EUREKA! N°29, 2009

63
Sociedade Brasileira de Matemática

COORDENADORES REGIONAIS
Alberto Hassen Raad (UFJF) Juiz de Fora – MG
Américo López Gálvez (USP) Ribeirão Preto – SP
Amarísio da Silva Araújo (UFV) Viçosa – MG
Andreia Goldani FACOS Osório – RS
Antonio Carlos Nogueira (UFU) Uberlândia – MG
Benedito Tadeu Vasconcelos Freire (UFRN) Natal – RN
Carlos Alexandre Ribeiro Martins (Univ. Tec. Fed. de Paraná) Pato Branco – PR
Carmen Vieira Mathias (UNIFRA) Santa María – RS
Claus Haetinger (UNIVATES) Lajeado – RS
Cleonor Crescêncio das Neves (EDETEC) Manaus – AM
Cláudio de Lima Vidal (UNESP) S.J. do Rio Preto – SP
Denice Fontana Nisxota Menegais (UNIPAMPA) Bagé – RS
Edson Roberto Abe (Colégio Objetivo de Campinas) Campinas – SP
Eduardo Tengan (USP) São Carlos – SP
Élio Mega (Faculdade Etapa) São Paulo – SP
Eudes Antonio da Costa (Univ. Federal do Tocantins) Arraias – TO
Fábio Brochero Martínez (UFMG) Belo Horizonte – MG
Florêncio Ferreira Guimarães Filho (UFES) Vitória – ES
Francinildo Nobre Ferreira (UFSJ) São João del Rei – MG
Genildo Alves Marinho (Centro Educacional Leonardo Da Vinci) Taguatingua – DF
Ivanilde Fernandes Saad (UC. Dom Bosco) Campo Grande – MS
Jacqueline Rojas Arancibia (UFPB)) João Pessoa – PB
Janice T. Reichert (UNOCHAPECÓ) Chapecó – SC
João Benício de Melo Neto (UFPI) Teresina – PI
João Francisco Melo Libonati (Grupo Educacional Ideal) Belém – PA
Jose de Arimatéia Fernandes (UFPB) Campina Grande – PB
José Luiz Rosas Pinho (UFSC) Florianópolis – SC
José Vieira Alves (UFPB) Campina Grande – PB
José William Costa (Instituto Pueri Domus) Santo André – SP
Krerley Oliveira (UFAL) Maceió – AL
Licio Hernandes Bezerra (UFSC) Florianópolis – SC
Luciano G. Monteiro de Castro (Sistema Elite de Ensino) Rio de Janeiro – RJ
Luzinalva Miranda de Amorim (UFBA) Salvador – BA
Mário Rocha Retamoso (UFRG) Rio Grande – RS
Marcelo Rufino de Oliveira (Grupo Educacional Ideal) Belém – PA
Marcelo Mendes (Colégio Farias Brito, Pré-vestibular) Fortaleza – CE
Newman Simões (Cursinho CLQ Objetivo) Piracicaba – SP
Nivaldo Costa Muniz (UFMA) São Luis – MA
Osnel Broche Cristo (UFLA) Lavras – MG
Osvaldo Germano do Rocio (U. Estadual de Maringá) Maringá – PR
Raul Cintra de Negreiros Ribeiro (Colégio Anglo) Atibaia – SP
Ronaldo Alves Garcia (UFGO) Goiânia – GO
Rogério da Silva Ignácio (Col. Aplic. da UFPE) Recife – PE
Reginaldo de Lima Pereira (Escola Técnica Federal de Roraima) Boa Vista – RR
Reinaldo Gen Ichiro Arakaki (UNIFESP) SJ dos Campos – SP
Ricardo Amorim (Centro Educacional Logos) Nova Iguaçu – RJ
Sérgio Cláudio Ramos (IM-UFRGS) Porto Alegre – RS
Seme Gebara Neto (UFMG) Belo Horizonte – MG
Tadeu Ferreira Gomes (UEBA) Juazeiro – BA
Tomás Menéndez Rodrigues (U. Federal de Rondônia) Porto Velho – RO
Valdenberg Araújo da Silva (U. Federal de Sergipe) São Cristovão – SE
Vânia Cristina Silva Rodrigues (U. Metodista de SP) S.B. do Campo – SP
Wagner Pereira Lopes (CEFET – GO) Jataí – GO

EUREKA! N°29, 2009

64
CONTEÚDO

XXX OLIMPÍADA BRASILEIRA DE MATEMÁTICA


Problemas e Soluções da Primeira Fase 2

XXX OLIMPÍADA BRASILEIRA DE MATEMÁTICA


Problemas e Soluções da Segunda Fase 16

XXX OLIMPÍADA BRASILEIRA DE MATEMÁTICA


Problemas e Soluções da Terceira Fase 33

XXX OLIMPÍADA BRASILEIRA DE MATEMÁTICA


Problemas e Soluções da Primeira Fase Nível Universitário 51

XXX OLIMPÍADA BRASILEIRA DE MATEMÁTICA


Problemas e Soluções da Segunda Fase Nível Universitário 57

XXX OLIMPÍADA BRASILEIRA DE MATEMÁTICA


Premiados 66

AGENDA OLÍMPICA 70

COORDENADORES REGIONAIS 71
Sociedade Brasileira de Matemática

XXX OLIMPÍADA BRASILEIRA DE MATEMÁTICA


Problemas e Soluções da Primeira Fase

PROBLEMAS – NÍVEL 1

01) Com segmentos de 1 cm de comprimento podemos formar triângulos. Por


exemplo, com nove desses segmentos podemos formar um triângulo eqüilátero de
lado 3 cm. Com qual número de segmentos a seguir é impossível formar um
triângulo?
A) 4 B) 5 C) 6 D) 7 E) 8

02) Esmeralda compra cinco latas de azeite a quatro reais e setenta centavos a lata,
cinco latas de leite em pó a três reais e doze centavos cada e três caixas de iogurte
com seis iogurtes cada caixa ao preço de oitenta centavos por iogurte. Paga com
uma nota de cinqüenta reais e quer saber quanto irá receber de troco. Qual das
expressões aritméticas a seguir representa a solução para este problema?
A) 50 − 5 × (4,70 + 3,12) + 18 × 0,80
B) 5 × 4,70 + 5 × 3,12 + 3 × 6 × 0,80 − 50
C) − [5 × (4,70 + 3,12) + 3 × 6 × 0,80] + 50
D) 50 − [5 × (4,70 + 3,12) + 3 × 6 + 0,80]
E) 50 − [5 × (4,70 + 3,12) + 6 × 0,80]

03) Uma pesquisa foi feita entre pessoas de ambos os sexos, em igual número, com
a seguinte pergunta: Entre as cores azul, vermelho e amarelo, qual é a cor que
você prefere?
Cada pessoa apresentou a sua preferência por uma, e só uma, dessas cores. E o
resultado da pesquisa aparece nos gráficos abaixo:

EUREKA! N°30, 2009

2
Sociedade Brasileira de Matemática

Podemos concluir que, em relação ao total de pessoas pesquisadas, a ordem de


preferência das cores é:
A) I, II, III B) I, III, II C) II, I, III D) II, III, I E) III, II, I

04) O quociente e o resto na divisão de 26097 por 25 são, respectivamente:


A) 1043 e 22 B) 1044 e 3 C) 143 e 22 D) 1044 e 22 E) 144 e 3

05) Numa reunião da comunidade do bairro, cada uma das 125 pessoas presentes
recebeu um número diferente, a partir do número 1 até o 125. Em dado momento,
foi feita uma lista das pessoas com número par e das pessoas com número múltiplo
de 3, que deveriam participar de um projeto. Algumas pessoas reclamaram,
dizendo que o seu nome aparecia duas vezes na lista. Quantas pessoas apareceram
duas vezes na lista?
A) 2 B) 6 C) 20 D) 41 E) 62

06) Sobre uma mesa retangular de uma


sala foram colocados quatro sólidos,
mostrados no desenho. Uma câmera no
teto da sala, bem acima da mesa,
fotografou o conjunto. Qual dos esboços a
seguir representa melhor essa fotografia?

EUREKA! N°30, 2009

3
Sociedade Brasileira de Matemática

07) Uma classe tem 22 alunos e 18 alunas. Durante as férias, 60% de todos os
alunos dessa classe foram prestar trabalho comunitário. No mínimo, quantas alunas
participaram desse trabalho?
A) 1 B) 2 C) 4 D) 6 E) 8

08) Uma urna contém 2008 cartões. Cada cartão recebeu um número diferente, a
partir do número 1 até o 2008. Retiram-se dois cartões ao acaso e somam-se os
números dos cartões. Quantos números ímpares diferentes podem ser obtidos dessa
maneira?
A) 1004 B) 1005 C) 2007 D) 2008 E) 4016

09) Juntando quatro trapézios iguais de bases 30


30cm
cm e 50 cm, como o da figura ao lado, podemos
formar um quadrado de área 2500 cm2, com um
“buraco” quadrado no meio. Qual é a área de 45o 45o
50 cm
cada trapézio, em cm2?

A) 200 B) 250 C) 300 D) 350 E) 400

10) Quantos números pares de três algarismos têm dois algarismos ímpares?
A) 20 B) 48 C) 100 D) 125 E) 225
2 5
11) Sabe-se que do conteúdo de uma garrafa enchem de um copo. Para
9 6
encher 15 copos iguais a esse, quantas garrafas deverão ser usadas?
A) 2 B) 3 C) 4 D) 5 E) 6

12) Quantos quadrados têm como vértices os pontos do


reticulado ao lado?

A) 6 B) 7 C) 8 D) 9

E) 10

EUREKA! N°30, 2009

4
Sociedade Brasileira de Matemática

13) A primeira fase da OBM se realiza no dia 14 de junho, um sábado do ano


bissexto 2008. Daqui a quantos anos o dia 14 de junho será novamente no sábado?
A) 4 B) 5 C) 6 D) 7 E) 8

14) No desenho temos AE = BE = CE = CD.


Além disso, α e β são medidas de ângulos.
α
Qual é o valor da razão ?
β

3 4
A) B) C) 1
5 5

5 5
D) E)
4 3

15) Na multiplicação ao lado, alguns algarismos, não


necessariamente iguais, foram substituídos pelo sinal *.
Qual é a soma dos valores desses algarismos?

A) 17 B) 27 C) 37 D) 47

E) 57

16) Três amigos moram na mesma rua: um médico, um engenheiro e um professor.


Seus nomes são: Arnaldo (A), Bernaldo (B) e Cernaldo (C). O médico é filho único
e o mais novo dos três amigos. Cernaldo é mais velho que o engenheiro e é casado
com a irmã de Arnaldo. Os nomes do médico, do engenheiro e do professor, nessa
ordem, são:
A) A, B, C B) C, A, B C) B, A, C D) B, C, A E) A, C, B

EUREKA! N°30, 2009

5
Sociedade Brasileira de Matemática

17) Dois cartões iguais têm a forma de


um triângulo retângulo de lados 5 cm, 12
cm e 13 cm. Esmeralda juntou os dois
cartões sobre uma folha de papel e,
contornando as beiradas com um lápis,
obteve uma figura como a ao lado, que
está fora de escala. Qual é o perímetro
dessa figura?

A) 28 cm B) 35 cm C) 42 cm D) 43 cm E) 60 cm

18) Qual é o maior número de algarismos que devem ser apagados do número de
1000 algarismos 20082008…2008, de modo que a soma dos algarismos restantes
seja 2008?
A) 130 B) 260 C) 510 D) 746 E) 1020

19) Soninha tem muitos cartões, todos com o mesmo desenho em uma das faces.
Ela vai usar cinco cores diferentes (verde, amarelo, azul, vermelho e laranja) para
pintar cada uma das cinco partes do desenho, cada parte com uma cor diferente, de
modo que não haja dois cartões pintados da mesma forma. Na figura abaixo, por
exemplo, os cartões são iguais, pois um deles pode ser girado para se obter o outro.
Quantos cartões diferentes Soninha conseguirá produzir?

A) 16 B) 25 C) 30 D) 60 E) 120

EUREKA! N°30, 2009

6
Sociedade Brasileira de Matemática

20) Três carros com velocidades constantes cada um, na mesma estrada, passam no
mesmo momento por Brasilópolis. Ao viajar 100 quilômetros, o carro A passa por
Americanópolis, 20 quilômetros à frente do carro B e 50 quilômetros à frente do
carro C. Quando o carro B passar por Americanópolis, quantos quilômetros estará à
frente do carro C?
A) 20 B) 25,5 C) 30 D) 35 E) 37,5

PROBLEMAS – NÍVEL 2

01) Veja o problema No. 14 do Nível 1.

02) Quantos dos números abaixo são maiores que 10?


3 11 , 4 7 , 5 5 , 6 3 , 7 2
A) 1 B) 2 C) 3 D) 4 E) 5

03) 1212 é igual a:


12
A) 6 6 B) 12 2 3
C) 212.3 6 D) 612 E) 12

04) Uma grande empresa possui 84 funcionários e sabe-se que cada funcionário
fala pelo menos uma das línguas entre Português e Inglês. Além disso, 20% dos
que falam Português também falam Inglês e 80% dos que falam Inglês também
falam Português. Quantos funcionários falam as duas línguas?
A) 12 B) 14 C) 15 D) 16 E) 18

05) Edmilson, Carlos e Eduardo ganharam um total de R$150,00 lavando carros.


Eles ganharam quantidades diferentes de dinheiro. Como eles são muito amigos
decidiram dividir o dinheiro ganho em partes iguais. Para isto, Edmilson deu
metade do que ganhou para dividir em partes iguais entre Carlos e Eduardo, porém,
Carlos tinha muito dinheiro e, portanto, deu R$ 10,00 a cada um dos outros dois.
Finalmente, para que cada um tivesse a mesma quantidade de dinheiro, Eduardo
deu R$ 2,00 a Edmilson. Quanto Eduardo ganhou antes da divisão?
A) R$ 76,00 B) R$ 51,00 C) R$ 23,00 D) R$ 50,00 E) R$ 100,00

06) Nove números são escritos em ordem crescente. O número do meio é a média
aritmética dos nove números. A média aritmética dos 5 maiores é 68 e a média
aritmética dos 5 menores é 44. A soma de todos os números é:
A) 560 B) 504 C) 112 D) 56 E) 70

EUREKA! N°30, 2009

7
Sociedade Brasileira de Matemática

07) Veja o problema No. 12 do Nível 1.

08) Veja o problema No. 13 do Nível 1.

09) Os algarismos a , b e c são tais que os números de dois algarismos aa , bc e


2
cb são números primos e aa + bc + cb = aa . Se b < c , então bc é igual a:
A) 19 B) 17 C) 37 D) 29 E) 59

10) Cinco inteiros positivos a, b, c, d , e maiores que um satisfazem as seguintes


condições:
a (b + c + d + e) = 128
b(a + c + d + e) = 155
c(a + b + d + e) = 203
d (a + b + c + e) = 243
e(a + b + c + d ) = 275
Quanto vale a soma a + b + c + d + e ?
A) 9 B) 16 C) 25 D) 36 E) 49

11) Em um triângulo ABC foi traçada a altura AH. Sejam M e N pontos sobre os
lados AB e AC, respectivamente, tais que HM é perpendicular a AB e HN é
perpendicular a AC. Achar MN, sabendo que o perímetro do triângulo órtico do
triângulo ABC é igual a 10.
Observação: o triângulo órtico de um triângulo é aquele cujos vértices são as
interseções das alturas do triângulo com os respectivos lados. Pode-se demonstrar
que o incentro (encontro das bissetrizes) do triângulo órtico é sempre igual ao
ortocentro (encontro das alturas) do triângulo original.
A) 5 B) 6 C) 7 D) 8 E) 9

12) Quantos números inteiros positivos menores que 500 têm exatamente 15
divisores inteiros positivos?
A) 0 B) 1 C) 2 D) 3 E) 4

13) Seja P (n) a soma dos algarismos pares do número n . Por exemplo,
P (1234) = 2 + 4 = 6. Qual o valor de P (1) + P (2) + P (3) + ... + P(100) ?
A) 200 B) 360 C) 400 D) 900 E) 2250

EUREKA! N°30, 2009

8
Sociedade Brasileira de Matemática

14) De quantas maneiras podemos dividir R$ 10,00 em moedas de 10 centavos e de


25 centavos, se pelo menos uma moeda de cada valor tem que ser usada?
A) 15 B) 16 C) 17 D) 18 E) 19

15) Sejam a, b, c, d números inteiros tais que a < 2b , b < 3c , c < 4d . Se


d < 40 , o maior valor possível de a será:
A) 960 B) 959 C) 951 D) 934 E) 927

16) A figura abaixo é um exemplo de um quadrado mágico de ordem 4. A soma


dos 4 números em cada linha, coluna e diagonal é 34. Então dizemos que a soma
mágica deste quadrado mágico é 34. Suponha que exista um quadrado mágico de
ordem 7, formado pelos números inteiros de 1 a 49. Determine sua soma mágica.

16 3 2 13

5 10 11 8

9 6 7 12

4 15 14 1

A) 175 B) 2450 C) 1225 D) 190 E) 100

17) Observe que:


32 + 4 2 = 5 2 ,
3 2 + 4 2 + 12 2 = 13 2 ,
3 2 + 4 2 + 12 2 + 84 2 = 85 2.
Qual o menor valor possível da soma x + y com x, y inteiros positivos tais que
3 2 + 4 2 + 12 2 + 84 2 + x 2 = y 2 ?
A) 289 B) 250 C) 425 D) 795 E) 103

18) Um número de três algarismos é 629 vezes menor que a soma de todos os
outros números de três algarismos. Este número é:
A) 450 B) 785 C) 630 D) 471 E) 525

19) Veja o problema No. 19 do Nível 1.

EUREKA! N°30, 2009

9
Sociedade Brasileira de Matemática

20) Em um triângulo ABC, ∠ A = 20o e ∠ B = 110o. Se I é o incentro (centro da


circunferência inscrita) e O o circuncentro (centro da circunferência circunscrita)
do triângulo ABC, qual a medida do ângulo ∠IAO ?
A) 20o B) 25o C) 30o D) 40o E) 35o

21) Veja o problema No. 7 do Nível 1.

22) Na figura abaixo os pontos A, B, C são colineares, assim como os pontos D, E,


F. As duas retas ABC e DEF são paralelas.
A B C

A1 A A3
2

D E F
Sendo A1, A2 e A3 as áreas das regiões destacadas na figura, podemos afirmar que:
A) A2 = 2A1 = 2A3 B) A2 = A1 + A3 C) A2 > A1 + A3
D) A2 < A1 + A3 E) A22 = A1.A3

23) O grupo A da última Copa do Mundo de futebol terminou com os seguintes


resultados:
Equipe Número de Pontos
Áustria 7
Brasil 5
Camarões 4
Dinamarca 0

Sabe-se que Áustria e Camarões levaram apenas 1 gol, cada um. Além disso, Brasil
e Dinamarca marcaram apenas 1 gol, cada um, enquanto que Áustria marcou 3
gols. Qual o resultado da partida Áustria × Dinamarca?
Observação: no grupo, cada seleção joga com as demais exatamente uma vez e, em
cada partida, o time vencedor ganha 3 pontos, o perdedor não ganha nem perde
pontos e, em caso de empate, cada time ganha 1 ponto.
A) 1 × 0 B) 2 × 1 C) 2 × 0 D) 0 × 0
E) Nada se pode afirmar.

EUREKA! N°30, 2009

10
Sociedade Brasileira de Matemática

24) Abaixo temos um quadrado mágico multiplicativo, onde o produto dos


números em cada linha, coluna e diagonal é o mesmo e igual ao número de quatro
dígitos ABCD, onde cada letra representa um dígito e cada casa contém um número
inteiro. Se AC representa o número de dois dígitos no centro do quadrado, a soma
A + B + C + D vale:
4

AC

C 24

A) 17 B) 18 C) 19 D) 20 E) 21

25) Tenho um cubo de madeira, com três faces vermelhas e três faces azuis. O
cubo é cortado em 3×3×3 = 27 cubos menores. Quantos destes cubos menores têm,
pelo menos, uma face vermelha e outra azul?
A) 6 B) 12 C) 14 D) 16
E) depende de quais faces do cubo são vermelhas e quais são azuis.

PROBLEMAS – NÍVEL 3

01) Veja o problema No. 14 do Nível 1.

02) Sendo x = 10–2008, assinale a alternativa que apresenta o maior valor.


1 1 1 x
A) B) C) D) x E)
x x( x + 1) 1 1
1+ x+
1 x
1+
x
1
03) O número inteiro positivo a e o número localizam-se na reta da seguinte
a
maneira:

Qual é a soma desses dois números?


9 9 81 82
A) B) C) D) E) 9
81 80 9 9

EUREKA! N°30, 2009

11
Sociedade Brasileira de Matemática

04) Veja o problema No. 4 do Nível 2

05) Rafael tem 10 cartões. Cada um tem escrito um dos números 3, 8, 13, 18, 23,
28, 33, 48, 53, 68, e todos os dez números aparecem. Qual o menor número de
cartões que Rafael pode escolher de modo que a soma dos números nos cartões
escolhidos seja exatamente 100?
A) 2 B) 3 C) 4 D) 5
E) não é possível obter soma 100 com esses cartões.

06) Em uma pista de corrida, cujo formato é de um polígono regular de n vértices,


numerados de 1 até n no sentido anti-horário, existem três pessoas: Nelly, Sônia e
Penha, estando inicialmente todas em um mesmo vértice. Em um dado momento
elas começam a caminhar pelos lados do polígono. Nelly caminha no sentido anti-
horário, enquanto que Sônia e Penha caminham no sentido contrário. Nelly cruza
com Sônia pela primeira vez em um vértice e com Penha dois vértices à frente. A
velocidade de Nelly é o dobro da velocidade de Sônia e a velocidade de Sônia é o
dobro da velocidade de Penha. Quantos vértices tem o polígono?
A) 30 B) 60 C) 15 D) 10 E) 6

07) Veja o problema No. 6 do Nível 2.

08) A primeira fase da OBM se realiza no dia 14 de junho, um sábado do ano


bissexto 2008. Daqui a quantos anos o dia 14 de junho será novamente no sábado?
A) 4 B) 5 C) 6 D) 7 E) 8

09) Veja o problema No. 14 do Nível 2.

10) O inteiro n é tal que n⋅2n possui 2008 divisores a mais que n. A soma dos
algarismos de n é igual a:
A) 5 B) 7 C) 9 D) 11 E) 12

11) Quantos dos números 2, 3, 5, 7, 11 são divisores de 3714 – 414?


A) um B) dois C) três D) quatro E) cinco

12) Veja o Problema No. 25 do Nível 2.

EUREKA! N°30, 2009

12
Sociedade Brasileira de Matemática

13) O número de soluções reais do sistema


a 2 = b + 2
 2
b = c + 2
c 2 = a + 2

é igual a:
A) 0 B) 1 C) 2 D) 4 E) 8

14) Arnaldo, Bernaldo, Cernaldo e Dernaldo baralharam as 52 cartas de um baralho


e distribuíram 13 cartas para cada um. Arnaldo ficou surpreso: “Que estranho, não
tenho nenhuma carta de espadas.” Qual a probabilidade de Bernardo também não
ter cartas de espadas?
39! 26! 39!39! 26!26! 39!13!
A) B) C) D) E)
26!52! 13!39! 26!52! 13!39! 52!

15) Veja o problema No. 19 do Nível 2.

16) Dado o quadrilátero ABCD tal que ∠CAD = 25°, ∠ACD = 45° e ∠BAC =
∠BCA = 20°, qual o valor do ângulo ∠DBC?
A) 40° B) 45° C) 50° D) 55° E) 60°

17) No triângulo PQR isósceles, com PQ = PR = 3 e QR = 2, a tangente à sua


circunferência circunscrita no ponto Q encontra o prolongamento do lado PR em X.
O valor de RX é:
16 12 8 9 9
A) B) C) D) E)
5 5 3 2 4

18) Dado um triângulo ABC de lados AB = 3, BC = 4 e AC = 5. Sejam R1 e R2,


respectivamente, os raios da circunferência inscrita e da circunferência com centro
R1
sobre o lado BC que passa por B e é tangente ao lado AC. A razão vale:
R2
3 2 3 8 4
A) B) C) D) E)
4 3 2 9 5

19) Qual o número de soluções reais do sistema


x⋅ | x | + y⋅ | y |= 1 e x  +  y  = 1 ,
EUREKA! N°30, 2009

13
Sociedade Brasileira de Matemática

onde x  representa a parte inteira de x?


A) 0 B) 1 C) 2 D) 4 E) infinitas

20) Um número de quatro dígitos é dito paladino se é múltiplo de 9 e nenhum de


seus dígitos é nulo. Quantos números paladinos existem?
A) 1284 B) 1024 C) 849 D) 1109 E) 729

21) Considere a função f, definida no conjunto dos números reais e satisfazendo


cx
f ( x) = , para todo x ≠ −3/2. Determine o número de tais funções f para as
2x + 3
quais
f (f (x)) = x, para todo x tal que f (f (x)) está bem definida.
A) 0 B) 1 C) 2 D) 4 E) infinitas.

22) O brinquedo favorito de Cícero é um cone reto de vidro com 5 cm de altura.


Cícero encheu o cone com areia até a altura de 3 cm, como mostrado na figura 1.
Em seguida, Cícero fechou a base do cone e virou-o de cabeça para baixo, como
indicado na figura 2. A que altura da base do cone, em cm, ficou a marca de areia?

3 cm
?

Figura 1 Figura 2

A) 1 B) 2 C) 5 − 3 98 D) 3
98
3
98
E) 1 −
5

23) Veja o problema No. 24 do Nível 2.

24) Considere 10 pessoas, todas de alturas diferentes, as quais devem ficar em fila
de tal modo que, a partir da pessoa mais alta, as alturas devem decrescer para
ambos os lados da fila (se a pessoa mais alta for a primeira ou a última da fila,

EUREKA! N°30, 2009

14
Sociedade Brasileira de Matemática

todas as pessoas a partir dela devem estar em ordem decrescente de altura).


Obedecendo essas condições, de quantos modos essas pessoas podem ficar em fila?
A) 256 B) 768 C) 1260 D) 512 E) 2560

25. Veja o problema No. 10 no Nível 2.

GABARITO

NÍVEL 1 (6º. ou 7º. Anos)


1) A 6) E 11) C 16) C
2) C 7) B 12) E 17) C
3) B ou D 8) C 13) C 18) D
4) A 9) E 14) D 19) C ou D
5) C 10) D 15) C 20) E

NÍVEL 2 (8º. ou 9º. Anos)


1) D 6) B 11) A 16) A 21) B
2) C 7) E 12) D 17) A 22) B
3) C 8) C 13) C 18) B 23) B
4) D 9) C 14) E 19) C ou D 24) B
5) C 10) D 15) E 20) C 25) E

NÍVEL 3 (Ensino Médio)


1) D 6) C 11) D 16) C 21) B
2) A 7) B 12) B 17) B 22) C
3) D 8) C 13) E 18) B 23) B
4) D 9) E 14) D 19) C 24) D
5) D 10) A ou B 15) C ou D 20) E 25) D

EUREKA! N°30, 2009

15
Sociedade Brasileira de Matemática

XXX OLIMPÍADA BRASILEIRA DE MATEMÁTICA


Problemas e Soluções da Segunda Fase

PROBLEMAS – NÍVEL 1 – PARTE A


(Cada problema vale 5 pontos)

01. Nicanor quer completar o Sudoku ao lado,


de modo que em cada linha (fileira horizontal) 2 1
e cada coluna (fileira vertical) apareçam todos 5
os números de 1 a 6. Qual é a soma de todos
4 2
os números que faltam para completar o
Sudoku? 6 4
6 3 2

02. A partir das igualdades


32 − 12 = 8 = 8 ⋅ 1,
5 2 − 32 = 16 = 8 ⋅ 2,
7 2 − 5 2 = 24 = 8 ⋅ 3,

e 2009 2 − 2007 2 = 8 ⋅ N,

podemos escrever 2009 2 − 1 = 4 ⋅ N ⋅ ( N + 1) . Qual é o valor de N?

03. Certo banco brasileiro obteve um lucro de R$ 4,1082 bilhões ao final do


primeiro semestre de 2008. Esse valor representa um aumento de 2,5% em relação
ao resultado obtido no mesmo período do ano passado. Qual é a soma dos dígitos
do número inteiro que representa, em reais, o lucro desse banco no primeiro
semestre de 2007?

04. A piscina do clube que Esmeralda freqüenta tem a forma


de um hexágono (polígono com seis lados), com um ângulo
interno de 270º, os demais ângulos de 90º e os quatro lados
menores com 12 metros cada. Esmeralda costuma nadar
pelo meio da piscina, a partir do ponto A, descrevendo o
trajeto representado, na figura, pelo ângulo reto ABC, em
que AB = BC.

EUREKA! N°30, 2009

16
Sociedade Brasileira de Matemática

Certo dia, ela nadou por esse trajeto 4 vezes, isto é, foi e voltou 2 vezes. Quantos
metros ela percorreu?

05. Com o dinheiro que Carlinhos tinha, poderia ter comprado 600 gramas de queijo
ou 400 gramas de presunto. Usando esse dinheiro, ele resolveu comprar
quantidades iguais de presunto e queijo. Quantos gramas de cada item ele
comprou?

06. Quantos números inteiros maiores que zero e menores que 100 possuem algum
divisor cuja soma dos dígitos seja 5?

PROBLEMAS – NÍVEL 1 – PARTE B


(Cada problema vale 10 pontos)

PROBLEMA 1
Zezinho tem 37 cartões quadrados de lado 6 cm e 21 cartões quadrados de lado 9
cm. Ele quer colar esses cartões lado a lado, sem sobrepô-los nem deixar buracos,
formando quadrados maiores.
a) Apresente, através de desenhos, duas maneiras diferentes de Zezinho construir
um quadrado de lado 27 cm.
b) Quantos cartões são necessários para construir o quadrado com a maior área
possível?

PROBLEMA 2
Para construir o arranjo triangular de letras ao lado, que tem
2008 linhas, obedeceu-se a uma certa regra.
a) Quantas vezes a palavra OBM aparece completamente na
maior coluna desse arranjo?
b) Quantas vezes a letra O aparece no arranjo?

PROBLEMA 3
Em Ferius, os pontos do dominó vão de 0 a 7, ao contrário de um dominó comum,
em que os pontos vão de 0 a 6. Uma peça do dominó de Ferius é chamada
importante se a soma de seus pontos é par. Por exemplo, os seguintes dominós são
importantes:

EUREKA! N°30, 2009

17
Sociedade Brasileira de Matemática

a) Quantas peças diferentes possui o dominó jogado em Ferius?


b) Quantas dessas peças são importantes?
c) Qual é a soma dos pontos de todas as peças importantes?

PROBLEMAS – NÍVEL 2 – PARTE A


(Cada problema vale 5 pontos)

01. Sejam x e y números reais positivos satisfazendo as equações x 2 + y 2 = 1 e


17 1
x4 + y 4 = . Calcule o valor de .
18 xy

02. Um viajante, que se encontrava perdido na floresta, andou 1 metro para o Leste,
2 metros para o Norte, 3 para o Oeste, 4 para o Sul, 5 para o Leste, 6 para o
Norte,..., 2006 metros para o Norte, 2007 para o Oeste e 2008 para o Sul. Calcule,
em metros, o valor inteiro mais próximo da distância entre as posições inicial e
final do viajante.

03. Os números α e β são as raízes da equação x 2 − x − 1 = 0. Calcule


13 ⋅ α 5 + 5 ⋅ β 7 .

04. Em um triângulo ABC, seja D um ponto sobre o lado BC tal que DB = 14, DA =
13 e DC = 4. Sabendo que o círculo circunscrito ao triângulo ADB tem raio igual
ao do círculo circunscrito ao triângulo ADC, calcule a área do triângulo ABC.

05. Dado um número natural N, multiplicamos todos os seus algarismos. Repetimos


o processo com o número obtido até obtermos um número com um algarismo. Este
número será chamado de primitivo de N. Por exemplo, como 3 ⋅ 2 ⋅ 7 = 42 e
4 ⋅ 2 = 8, concluímos que o primitivo de 327 é 8. Calcule a soma dos algarismos do
maior número natural com todos os algarismos diferentes cujo primitivo é ímpar.

EUREKA! N°30, 2009

18
Sociedade Brasileira de Matemática

PROBLEMAS – NÍVEL 2 – PARTE B


(Cada problema vale 10 pontos)

PROBLEMA 1
Encontre todos os triângulos retângulos, de lados com medidas inteiras, nos quais a
área tem valor numérico igual ao do perímetro.

PROBLEMA 2
No quadro negro são escritos os números 12 , 22 ,32 , 42 ,..., 20082. Pedro e Igor jogam
um jogo onde eles apagam alternadamente um número por vez até sobrarem apenas
dois números. Se a diferença entre estes dois números for múltiplo de 2009, Igor
vence. Caso contrário, quem vence é Pedro. Sabendo que Pedro é o primeiro a
jogar, diga quem possui a estratégia vencedora. Justifique sua resposta.

PROBLEMA 3
Seja ABC um triângulo acutângulo com BC = 5. Seja E o pé da altura relativa ao
lado AC e F o ponto médio do lado AB. Se BE = CF = 4, calcule a área do triângulo
ABC.

PROBLEMA 4
Um país tem 8 cidades, A1, A2, ..., A6, B, C, ligadas por rodovias de mão dupla
satisfazendo as seguintes condições: B e C são ambas ligadas às cidades A1, A2, ...,
A6, mas não são ligadas uma à outra; A1, A2, ..., A6 são ligadas duas a duas. Calcule
o número de maneiras distintas de viajar de carro de B a C, sem passar duas vezes
por uma mesma cidade.

PROBLEMAS – NÍVEL 3 – PARTE A


(Cada problema vale 5 pontos)

01. Um trapézio isósceles ABCD, com lados paralelos AB e CD, é tal que a diagonal
BD mede 100 m e o ângulo BDC mede 30°. Seja S a área do trapézio em m2.
Determine S ⋅ 3.

02. Se x é um número real, denotamos por  x  o maior inteiro que é menor ou


igual a x. Por exemplo,  2  = 2, π  = 3 e  −2,1 = −3. Calcule o valor da soma
 4 1  +  4 2  +  4 3  +  4 4  + ... +  4 2008  .
         
03. Um inteiro positivo n é chamado de auto-replicante se os últimos dígitos de n 2
formam o número n. Por exemplo, 25 é auto-replicante pois 252 = 625. Determine

EUREKA! N°30, 2009

19
Sociedade Brasileira de Matemática

a soma de todos os números auto-replicantes com exatamente 4 dígitos (isto é,


números auto-replicantes n com 1000 ≤ n ≤ 9999 ).

04. Quantas permutações de 1, 2, 3, ..., 9 há com a propriedade de que, para todo


1 ≤ i < 9, os números que aparecem entre i e i + 1 (onde i pode aparecer tanto antes
como depois de i + 1) são todos menores do que i? Por exemplo, 976412358 é uma
permutação com esta propriedade.

05. Suponha que α ∈ é raiz de algum polinômio não-nulo com coeficientes


racionais. O polinômio minimal de α é o polinômio de menor grau m(x) tal que:
• m(α ) = 0;
• m( x) é Mônico (isto é, o seu coeficiente líder é 1) e todos os seus
coeficientes são racionais.

Por exemplo, o polinômio minimal de 2 é x 2 − 2. Determine o produto dos


coeficientes não nulos do polinômio minimal de 3
−27 + 5 33 − 3 27 + 5 33.

PROBLEMAS – NÍVEL 3 – PARTE B


(Cada problema vale 10 pontos)

PROBLEMA 1
Determine todos os inteiros positivos m e n tais que
m 2 + 161 = 3n

PROBLEMA 2
Determine a quantidade de funções f :{1, 2,3, 4,5} → {1, 2,3, 4,5} tais que
f ( f ( x)) = f ( x) para todo x ∈ {1, 2,3, 4,5}.

PROBLEMA 3
Um trapézio ABCD, com lados paralelos AB e CD, está inscrito em uma
circunferência de raio 25. Sabe-se que CD é um diâmetro e a altura desse trapézio é
24. Seja E um ponto no arco menor determinado por A e B e sejam F e G os pontos
AF ⋅ BG
de interseção de ED e EC com AB, respectivamente. Calcule .
FG

PROBLEMA 4
Em uma matriz 2008 × 2008 o elemento na linha i e coluna j é o número i + j (as

EUREKA! N°30, 2009

20
Sociedade Brasileira de Matemática

linhas e colunas são numeradas de 1 a 2008). Escolhem-se 2008 elementos desta


matriz de modo que não haja dois elementos escolhidos numa mesma linha ou
coluna. Os elementos são multiplicados. Qual o menor produto que se pode obter
desta forma?

Soluções Nível 1 – Segunda Fase – Parte A


Problema 01 02 03 04 05 06
Resposta 91 1004 12 144 240 34

01.[91] A soma de todos os números do Sudoku completo é igual a 6 vezes a soma


dos números em cada linha, ou seja, 6 × (1 + 2 + … + 6 ) = 6 × 21 = 126 . A soma dos
números que já estão escritos no Sudoku é 35. Logo a soma dos números que
faltam para completar o Sudoku é126 – 35 = 91.

02. [1004] Temos:


20092 −12 = 4 ⋅ N ⋅ ( N +1) ⇔ ( 2009 −1)( 2009 +1) = 4N( N +1) ⇔ 2008⋅ 2010 = 4N( N +1) ⇔
2008 2010 4N( N +1)
⇔ ⋅ = ⇔1004 ⋅1005 = N( N +1) ⇔ N = 1004
2 2 2⋅ 2

Soluções alternativas:

1a solução
Cada linha pode ser associada a um número ímpar e a um múltiplo de 8 da seguinte
forma: na linha 1 temos o quadrado de 1 = 2 ⋅ 1 − 1 (no lado esquerdo da igualdade)
e 8 vezes 1 (no lado direito da igualdade), na linha 2 temos o quadrado de
3 = 2 ⋅ 2 − 1 e 8 vezes 2, na linha 3 temos o quadrado de 5 = 2 ⋅ 3 − 1 e 8 vezes 3 e
assim sucessivamente, até chegarmos à linha N onde temos o quadrado de
2007 = 2N − 1 e 8 vezes N.
Assim, 2N − 1 = 2007 ⇔ 2N = 2008 ⇔ N = 1004 .

2a solução
Cada linha pode ser associada um múltiplo de 8 da seguinte forma: na linha 1
temos 8 vezes 1 (no lado direito da igualdade), na linha 2 temos 8 vezes 2, na linha
3 temos 8 vezes 3 e assim sucessivamente, até chegarmos a última linha, onde
2009 − 1
temos 2009 2 − 2007 2 = 8 ⋅ N , que é a linha = 1004 , ou seja, N = 1004.
2

EUREKA! N°30, 2009

21
Sociedade Brasileira de Matemática

3a solução
Temos:
20092 − 20072 = 8 ⋅ N ⇔ (2009− 2007)(2009+ 2007) = 8 ⋅ N ⇔ 2 ⋅ 4016= 8 ⋅ N ⇔ N = 1004

03. [12] Seja x o lucro desse banco no primeiro semestre de 2007, em bilhões de
reais.
Logo x + 2 ,5% ⋅ x = 4 ,1082 ⇔ x + 0 ,025 x = 4 ,1082 ⇔ 1,025 x = 4 ,1082 ⇔ x = 4 ,008
bilhões de reais, ou seja, o lucro foi de R$ 4008000000,00, cuja soma dos dígitos é
12.

04. [144] A partir das informações dadas, concluímos


D
que na figura ID = DE = EF = FG = 12 metros e que A é I

o ponto médio de ID , ou seja, AD = 6 metros e, da


mesma forma, FC = 6 metros. E F
Logo AB = BC = 12 + 6 = 18 metros e, portanto,
Esmeralda nadou 4 ⋅ (18 + 18) = 4 ⋅ 36 = 144 metros.

Q
05. [240] Supondo que Carlinhos tem Q reais, o preço do grama de queijo é e
600
Q
o preço do grama de presunto é . Seja m a quantidade, em gramas, de queijo e
400
de presunto que Carlinhos comprou. Dessa forma:
Q Q  1 1  1 400 × 600 240000
m⋅ + m⋅ = Q ⇔ m +  =1⇔ m = 1 = = = 240
600 400  600 400  1 400 + 600 1000
+
600 400
Portanto ele comprou 240 gramas de cada item.

06. [34] São os múltiplos de 5, que nesse intervalo são 19; os múltiplos de 14, que
são 6 (pois o 70 já foi contado); os múltiplos de 23, que são 4; os múltiplos de 32,
que são 3 e, finalmente, os múltiplos de 41, que são 2. Note que o único múltiplo
de 50 no intervalo, que é o próprio 50, já foi contato nos múltiplos de 5. Portanto
ao todo são 19 + 6 + 4 + 3 + 2 = 34 números.

EUREKA! N°30, 2009

22
Sociedade Brasileira de Matemática

Soluções Nível 1 – Segunda Fase – Parte B


PROBLEMA 1
a) Os desenhos mostram as duas
formas de construção dos quadrados.
Elas são as únicas possíveis.
De fato, sendo x o número de
quadrados de lado 6 cm e y o número
de quadrados de lado 9 cm usados para
construir um lado de 27 cm, temos:
9 − 2x
6x + 9y = 27 ⇔ 2x + 3y = 9 ⇔ y = Como x e y são inteiros não negativos,
3
podemos substituir x apenas por 0, 1, 2, 3 ou 4. As únicas soluções para essa
situação são x = 0 e y = 3 ou x = 3 e y = 1, representadas nos desenhos.

b) Repetindo mais 3 vezes a segunda construção acima, obtém-se um quadrado de


lado 54 cm, com a utilização de 36 cartões de lado 6 cm e 20 cartões de lado 9 cm,
sobrando apenas 1 cartão de lado 6 cm e 1 cartão de lado 9 cm. Esse quadrado é o
maior que se pode construir, usando-se o maior número de cartões, 56 cartões.

De fato, como os quadrados construídos com os cartões devem ter lados com
medidas inteiras, concluímos que o quadrado maior do que o construído deveria ter
lado de 60 cm, pelo menos, já que o cartão menor tem lado 6 cm. Como
602 − 542 = 684 cm 2 é maior do que 6 2 + 9 2 = 117 cm 2 , que é a soma das áreas
dos quadrados que sobraram, concluímos que realmente o quadrado de lado 54 cm
é o maior que se pode construir usando o maior número de cartões.

PROBLEMA 2
a) A maior coluna tem 2008 letras e OBM é um bloco de 3 letras. Como
2008 = 669 ⋅ 3 + 1 , o número de vezes em que a palavra OBM aparece
completamente na maior coluna é 669.

b) Da esquerda para a direita, fazendo a contagem ao longo das flechas, a primeira


passa por
2008 letras O. Como a segunda inicia 3 linhas abaixo, ela passa por
2008 − 3 = 2005 letras O. Nesse padrão, a próxima passará
por 2002 letras O, a seguinte, por 1999, e assim até a última
flecha, que passará por 1.
Portanto o número de vezes que a letra O aparece no arranjo é

EUREKA! N°30, 2009

23
Sociedade Brasileira de Matemática

(2008 + 1) ⋅ 670
2008 + 2005 + 2002 + 1999 + +1 = = 673015 .
2

PROBLEMA 3
8⋅7
a) Há = 28 peças com quantidades diferentes de pontos em cada lado e 8 com
2
quantidades iguais, ou seja, o dominó de Ferius tem 28 + 8 = 36 peças diferentes.

Outra solução:
O dominó comum possui 28 peças. Como existem mais 8 novas peças que possuem
alguma casa marcando 7 pontos, o dominó de Ferius tem 28 + 8 = 36 peças
diferentes.
b) Como a soma de um par e um ímpar é ímpar e há 4 quantidades ímpares de
pontos (1, 3, 5, 7) e 4 quantidades pares de pontos (0, 2, 4, 6), há 4 ⋅ 4 = 16 peças
que não são importantes. Logo existem 36 − 16 = 20 peças importantes.

c) Cada quantidade de pontos aparece exatamente 9 vezes. Assim a soma dos


pontos de todas as peças é 9 ⋅ (1 + 2 + 3 + + 7) = 252 . A soma dos pontos de todas
as peças que não são importantes é 4 ⋅ (1 + 2 + 3 + + 7) = 112 , pois cada
quantidade de pontos aparece exatamente 4 vezes em peças que não são
importantes. Assim, a soma pedida é 252 − 112 = 140 .

Soluções Nível 2 – Segunda Fase – Parte A


Problema 01 02 03 04 05
Resposta 6 1420 144 108 22

01. De
17
= x 4 + y 4 = ( x 2 + y 2 ) − 2( xy ) 2 = 1 − 2( xy ) 2 ,
2

18
1 1
obtemos ( xy ) = , e daí
2
= 6.
36 xy

02. O deslocamento líquido do viajante na direção Leste-Oeste foi de


(1 − 3) + (5 − 7) + ... + (2005 − 2007) = (−2) + (−2) + ... + (−2) = −1004.
502 vezes

EUREKA! N°30, 2009

24
Sociedade Brasileira de Matemática

Analogamente, o deslocamento líquido na direção Norte-Sul foi de –1004.


Portanto, pelo teorema de Pitágoras a distância entre as posições inicial e final do
viajante é 1004 2. Observe agora que, como 2 ≅ 1, 414, temos
1004 2 ≅ 1419,656. Para ter certeza se estamos usando uma aproximação boa o
suficiente, basta checar se 1419,5 < 1004 2 < 1420, quer dizer, se
(1419,5) 2 < 10042 ⋅ 2 < 14202. Mas é fácil efetuar os cálculos e verificar que essas
desigualdades realmente se verificam. Logo, a melhor aproximação pedida é 1420
metros.

03. Veja que α + β = 1 e


α 3 = α ⋅ α 2 = α (α + 1) = α 2 + α = 2α + 1,
α 4 = α ⋅ α 3 = α (2α + 1) = 2α 2 + α = 3α + 2,
α 5 = α ⋅ α 4 = α (3α + 2) = 3α 2 + 2α = 5α + 3.
Analogamente,
β 7 = β 4 ⋅ β 3 = (5β + 3)( β + 1) = 5β 2 + 8β + 3 = 13β + 8.
Portanto, 13α 5 + 5β 7 = 13(5α + 3) + 5(13β + 8) = 65(α + β ) + 79 = 65 + 79 = 144.

04. Como os dois círculos circunscritos são iguais, segue do teorema do ângulo
inscrito que ∠ACB = ∠ABC e, com isso, AB = AC.

M
D
C

Seja AM a altura relativa ao lado BC. Como ABC é isósceles de base BC, segue que
AM também é mediana, e daí MC = 9. Portanto, MD = 5 e, pelo teorema de

EUREKA! N°30, 2009

25
Sociedade Brasileira de Matemática

Pitágoras, AM = 12. Finalmente, a área do triângulo ABC é


1 1
( AM )( BC ) = (12 )(18) = 108.
2 2

05. Para que o primitivo de um número seja ímpar, todos os seus algarismos
precisam ser ímpares, pois o produto de um número par por um número qualquer é
sempre um número par. Assim, só nos restam os algarismos 1, 3, 5, 7 e 9 para
construir o número pretendido. Por outro lado, como os algarismos precisam ser
todos diferentes, o número terá, no máximo, 5 algarismos. Contudo, qualquer
número com 5 algarismos ímpares e todos distintos tem primitivo 0. De fato, o
produto dos números 1, 3, 5, 7 e 9 é 945 e seu primitivo é 0. O maior número com
4 algarismos ímpares e todos diferentes é 9753, mas esse número tem primitivo 0.
O número que o antecede e tem seus 4 algarismos ímpares e distintos é 9751, e seu
primitivo é 5. Portanto, a soma de seus algarismos é 9 + 7 + 5 + 1 = 22.

Soluções Nível 2 – Segunda Fase – Parte B


SOLUÇÃO DO PROBLEMA 1:
Os catetos do triângulo medem a e b, e a hipotenusa mede c. Como a área e o
1 1
perímetro são iguais, temos ab = a + b + c, e daí c = ab − a − b. Usando o
2 2
teorema de Pitágoras, segue que
1 1
a 2 + b 2 = ( ab − a − b) 2 = a 2 + b 2 + 2ab − a 2 b − b 2 a + a 2b 2 ,
2 4
ou ainda 8ab − 4a 2 b − 4b 2 a + a 2 b 2 = 0. . Dividindo por ab, obtemos
( a − 4 )( b − 4 ) = 8, de maneira que a – 4 divide 8. Portanto, os possíveis valores de
a são 2, 3, 5, 6, 8 e 12. Determinando os valores de b e c, encontramos os
triângulos de lados 5, 12, 13 ou 6, 8, 10.

SOLUÇÃO DO PROBLEMA 2:
Note que ( 2009 − x ) − x 2 = 2009 ( 2009 − 2 x ) , um múltiplo de 2009. Assim,
2

sempre que Pedro apagar um número, x2 digamos, basta Igor apagar o número
(2009 – x)2. Desse modo, no final restarão dois números cuja diferença é um
múltiplo de 2009.

EUREKA! N°30, 2009

26
Sociedade Brasileira de Matemática

SOLUÇÃO DO PROBLEMA 3:
A

F E

B C

Seja D o pé da perpendicular baixada de F a AC. Pelo teorema de Pitágoras, segue


que EC = BC 2 − BE 2 = 52 − 42 = 3. Por outro lado, por semelhança de
1
triângulos temos FD = BE = 2 e AE = 2 DE. Portanto,
2
DC = CF 2 − FD 2 = 42 − 22 = 2 3,
e daí DE = 2 3 − 3, de maneira que AE = 4 3 − 6. Finalmente,
1
2
1
( )
[ ABC ] = ( AE + EC ) BE = 4 3 − 6 + 3 ⋅ 4 = 8 3 − 6.
2

SOLUÇÃO DO PROBLEMA 4:
Há duas escolhas envolvidas e que determinam a maneira de viajar de B a C: por
quais dentre as cidades A1 ,..., A6 devemos passar, e em que ordem. Digamos que
escolhamos passar por exatamente k dentre as cidades A1 ,..., A6 , com 1 ≤ k ≤ 6; o
6
número de modos de escolher as k cidades é   . Por outro lado, após
k 
escolhermos as k cidades, devemos escolher em que ordem vamos visitá-las, o que
corresponde a k! possibilidades. Logo, o número de modos de viajar de B a C é
6
6 6
6! 6! 6! 6!
∑  
k =1  k 
k ! = ∑
k =1 ( 6 − k ) !
= + + ... + = 1956.
5! 4! 0!

Soluções Nível 3 – Segunda Fase – Parte A


Problema 01 02 03 04 05
Resposta 7500 9779 9376 256 18

EUREKA! N°30, 2009

27
Sociedade Brasileira de Matemática

01. Seja P a projeção ortogonal de B sobre CD.


A B

100

30°
D P C

CD − AB AB + CD
Temos que CP = logo PD = CP + AB = . Assim, a área do
2 2
trapézio é:
AB + CD
S = BP ⋅ = BP ⋅ PD = (100 sen30°) ⋅ (100 cos30°) = 2500 3 m 2 e portanto
2
S 3 = 7500.

02. Observe que para i ≥ 1 temos


 4 n  = i ⇔ i ≤ 4 n < i + 1 ⇔ i 4 ≤ n < (i + 1) 4 e assim há ( i + 1)4 − i 4 números n tais
 
que  4 n  = i.
Portanto a soma pedida é:
1⋅ ( 24 − 14 ) + 2 ⋅ ( 34 − 24 ) + 3 ⋅ ( 44 − 34 ) + 4 ⋅ ( 54 − 44 ) + 5 ⋅ ( 64 − 54 ) + 6 ⋅ ( 2008 − 64 + 1) = 9779.

03. Seja n um inteiro de 4 dígitos. Temos que n é auto-replicante se e somente se


n 2 − n é divisível por 10000, isto é, 24 | n( n − 1) e 54 | n(n − 1) . Como n e n – 1 são
primos entre si, temos 4 possibilidades:

• 2 4 | n e 54 | n
• 24 | ( n − 1) e 54 | ( n − 1)
• 24 | n e 54 | ( n − 1)
• 24 | ( n − 1) e 54 | n.

A primeira possibilidade implica que 104 | n, o que é impossível pois


1000 ≤ n ≤ 9999. Da mesma forma, a segunda não ocorre.

EUREKA! N°30, 2009

28
Sociedade Brasileira de Matemática

Na terceira possibilidade, de 54 | ( n − 1) temos que n = 625k + 1 para algum k


inteiro e que 625k + 1 ≡ 0(mod16) ⇔ k + 1 ≡ 0(mod16) ⇔ k ≡ 15(mod16)
Assim, k = 15 + 16 para algum inteiro e n = 625 (15 + 16 ) + 1 = 9376 + 10000
E como 1000 ≤ n ≤ 9999, a única possibilidade é n = 9376.
Finalmente, para a quarta possibilidade, temos que n = 625k, k inteiro, e que
n − 1 ≡ 0(mod16) ⇔ k ≡ 1(mod16) .
Assim, k = 1 + 16 , inteiro, e n = 625(1 + 16 ) = 625 + 10000 . Como
1000 ≤ n ≤ 9999, não há soluções neste caso.
Logo o único número auto-replicante de 4 dígitos é 9376.

04. Da propriedade, decorre que 9 só pode aparecer ou como primeiro ou como


último elemento da permutação e que os elementos de 1 a 8 formam uma
permutação com a mesma propriedade. Assim, o número pedido é o dobro do
número de permutações de 1, 2,...,8 com a mesma propriedade. Da mesma forma, o
número de permutações de 1, 2,.., 8 com a propriedade é o dobro do número de
permutações de 1, 2,.., 7 com a propriedade. Repetindo o raciocínio, concluímos
que o número pedido é portanto 28 = 256.

05. Seja α = 3 −27 + 5 33 − 3 27 + 5 33. Temos

( )
α 3 = −27 + 5 33 − 27 + 5 33 − 3 3 −27 + 5 33 ⋅ 3 27 + 5 33 ⋅ ( 3
−27 + 5 33 − 3 27 + 5 33 )
⇒ α 3 = −54 − 3 3 96 ⋅ α ⇒ (α 3 + 54 ) = −25 ⋅ 34 α 3
3

Agora faça 18 y = α 3 . Temos


(18 y + 54 ) = −26 ⋅ 36 y
3

⇔ ( y + 3) = −23 y ⇔ y 3 + 9 y 2 + 35 y + 27 = 0 ⇔ ( y + 1)( y 2 + 8 y + 27) = 0


3

Como α , e portanto y, são reais e y 2 + 8 y + 27 = 0 não tem raízes reais,


concluímos que y = –1 e portanto α = − 3 18 (pasmem!). Assim, α é raiz do
polinômio x 3 + 18 = 0, que é o polinômio minimal de α já que x 3 + 18 = 0 não
possui raízes racionais.

EUREKA! N°30, 2009

29
Sociedade Brasileira de Matemática

Soluções Nível 3 – Segunda Fase – Parte B


SOLUÇÃO DO PROBLEMA 1:
Olhando a equação módulo 7, temos: m 2 ≡ 3n , porém m 2 só poderá ser congruente
a 0,1,2,4 enquanto que se n for ímpar 3n só poderá ser congruente a 3, 5, 6, então n
deverá ser par. Logo existe n0 ∈ tal que n = 2n0 . Voltando à equação original
temos:
( )( )
m 2 + 161 = 32 n0 ⇔ 32 n0 − m 2 = 161 ⇔ 3n0 − m 3n0 + m = 161. Como m e n são
inteiro positivos, logo o módulo de (3
n0
)
− m é menor que (3 n0
)
+ m , e como

(3 n0
)
− m é positivo e 161 = 7 ⋅ 23, então temos as opções:

3n0 − m = 1 e 3n0 + m = 161 ⇔ 3n0 = 81 e m = 80 ⇔ n0 = 4 e
m = 80 ⇔ n = 8 e m = 80
• 3n0 − m = 7 e 3n0 + m = 23 ⇔ 3n0 = 15 e m = 8. Não há solução inteira.
Logo m = 80 e n= 8 é a única solução.

SOLUÇÃO DO PROBLEMA 2:
Para que f ( f ( x ) ) = f ( x ) então a imagem de f deverá só conter pontos fixos.
Utilizando esse fato temos:

• Com 5 pontos fixos na imagem teremos 1 função possível.


 5
• Com 4 pontos fixos na imagem teremos   4 = 20 funções
1 
5
• Com 3 pontos fixos na imagem teremos   ⋅ 32 = 90 funções
 2
 5
• Com 2 pontos fixos na imagem teremos   ⋅ 23 = 80 funções
 3
5
Com 1 ponto fixo na imagem teremos   ⋅ 14 = 5 funções

 4
logo o total de funções f satisfazendo f ( f ( x)) = f ( x) igual a 196.

EUREKA! N°30, 2009

30
Sociedade Brasileira de Matemática

SOLUÇÃO DO PROBLEMA 3:

E
A B
F G

D C
O

Como ABE ≅ ADE (ambos enxergam o arco AE ) temos que ∆FBE ∼ ∆FDA e
portanto
FB BE
= (1)
FD DA
Analogamente, das semelhanças ∆EBG ∼ ∆ACG , ∆AEG ∼ ∆CBG e
∆AEF ∼ ∆DBF obtemos respectivamente
BG EB
= (2)
CG AC

AE AG
= (3)
CB CG

AE AF
= (4)
DB DF
Assim, utilizando o fato que ABCD é isósceles (de modo que AD = BC e BD = AC)
temos
AF ⋅ BG (2) e (4) 1 AE ⋅ DF CG ⋅ EB
= ⋅ ⋅
FG FG DB AC
1 ( AE ⋅ CG )( DF ⋅ EB ) (1) e (3) AD 2 AG ⋅ BF
= = ⋅
AC 2 FG AC 2 FG
2
 AD  ( AF + FG )( BG + FG )
= 
 AC  FG

EUREKA! N°30, 2009

31
Sociedade Brasileira de Matemática

2
 AD  FG ( AF + FG + BG ) + AF ⋅ BG
= 
 AC  FG
2
 AD   AF ⋅ BG 
=  ⋅  AB + 
 AC   FG 
Em suma, temos
2
AF ⋅ BG  AD   AF ⋅ BG 
=  ⋅  AB + 
FG  AC   FG 
AF ⋅ BG AD 2 ⋅ AB
⇔ =
FG AC 2 − AD 2

Utilizando o fato de que ABCD é isósceles com base CD = 50 e altura 24,


aplicando Pitágoras várias vezes é fácil calcular AB = 14, AD = 30, AC = 40.
AF ⋅ BG
Assim, = 18 .
FG

SOLUÇÃO DO PROBLEMA 4:
Vamos mostrar que o menor produto é obtido quando tomamos os elementos da
diagonal principal. Neste caso, o produto é dado por
(1 + 1)(2 + 2)(3 + 3)...(2008 + 2008) = 2 ⋅ 2008!
2008

Suponha que todos os elementos (1, 1), (2, 2),..., (i – 1, i – 1) tenham sido
escolhidos mas que os elementos nas i– ésimas linha e colunas sejam (i, j) e (k, i)
com j e k maiores ou iguais a i + 1. Vamos mostrar que trocando estes dois
elementos por (i, i) e (k, j) obtemos um produto menor. De fato, para isto devemos
mostrar que

(i + i )( j + k ) < (i + j )(i + k )
⇔ 2i ( j + k ) < i 2 + ( j + k )i + jk
⇔ i 2 − ( j + k )i + jk > 0
⇔ (i − j )(i − k ) > 0
O que é verdade, já que i − j < 0 e i − k < 0.

EUREKA! N°30, 2009

32
Sociedade Brasileira de Matemática

XXX OLIMPÍADA BRASILEIRA DE MATEMÁTICA


Problemas e Soluções da Terceira Fase

NÍVEL 1 (6o. e 7o. Anos)

PROBLEMA 1
Um quadrado de lado 12 foi dividido em sete regiões retangulares que não se
sobrepõem, conforme a figura. Uma delas é um quadrado de vértice C, cuja área é
metade da área de cada um dos dois retângulos vizinhos; outra é um quadrado de
vértice A, cuja área é metade da área de cada um dos dois retângulos vizinhos.
A B

D C
a) Mostre que o quadrilátero destacado é um quadrado.
b) Calcule a área do quadrado destacado.

PROBLEMA 2
Esmeralda escolhe um número inteiro positivo qualquer e realiza a seguinte
operação com ele: cada um de seus algarismos é trocado pelo seu sucessor, com
exceção do 9, que é trocado por 0. Em seguida, os eventuais zeros que aparecem à
esquerda são eliminados. Por exemplo, ao se realizar a operação no número
990003953 obtém-se 1114064 (note que os dois zeros à esquerda gerados pelos
dois primeiros algarismos 9 foram eliminados).

A operação é repetida até que se obtenha 0. Por exemplo, começando com 889,
obtemos a seqüência de números
889, 990, 1, 2, 3, 4, 5, 6, 7, 8, 9, 0

a) Apresente a sequência de números quando o primeiro número é 2008.

EUREKA! N°30, 2009

33
Sociedade Brasileira de Matemática

b) Mostre que, independente do número inicial, após uma quantidade finita de


operações Esmeralda obtém 0.

PROBLEMA 3
Jade tem n peças iguais 3× 1 e quer utilizá-las para cobrir um tabuleiro 3 × n ,
sendo n um inteiro positivo. Por exemplo, para n = 4 ela pode cobrir o tabuleiro da
seguinte maneira:

a) Determine de quantas maneiras Jade pode fazer a cobertura para n = 1, 2, 3, 4, 5,


6, 7.
b) De quantas maneiras Jade pode cobrir o tabuleiro para n = 15?

PROBLEMA 4
Considere o seguinte hexágono:

Com cópias desse polígono podemos cobrir todo o plano, sem sobreposições, como
mostra a figura a seguir.

a) É possível cobrir o plano com cópias de um pentágono regular?

EUREKA! N°30, 2009

34
Sociedade Brasileira de Matemática

Observação: um polígono é regular quando todos os seus lados são de mesma


medida e todos os seus ângulos internos são iguais.
b) Seja ABCDE um pentágono com todos os lados iguais e tal que a medida do
ângulo interno nos vértices A e B são m( Aˆ ) = 100 e m( Bˆ ) = 80 . Mostre como é
possível cobrir todo o plano com cópias desse pentágono, sem sobreposições.

PROBLEMA 5
Vamos chamar de garboso o número que possui um múltiplo cujas quatro
primeiras casas de sua representação decimal são 2008. Por exemplo, 7 é garboso
pois 200858 é múltiplo de 7 e começa com 2008. Observe que
200858 = 28694 × 7 .
a) Mostre que 17 é garboso.
b) Mostre que todos os inteiros positivos são garbosos.

TERCEIRA FASE – NÍVEL 2 (8o. e 9o. Anos)

PRIMEIRO DIA
PROBLEMA 1
Em cada casa de um tabuleiro n × n , colocamos um dos números 1,2,3,4, de modo
que cada casa tem exatamente uma casa vizinha com o mesmo número. É possível
fazer isso quando

a) n = 2007 ?
b) n = 2008 ?

Observação. Duas casas são vizinhas se possuem um lado em comum.

PROBLEMA 2
Seja P um pentágono convexo com todos os lados iguais. Prove que se dois dos
ângulos de P somam 180 graus, então é possível cobrir o plano com P, sem
sobreposições.

PROBLEMA 3
Prove que existem infinitos inteiros positivos n tais que
5 n−2 − 1
n
é um inteiro.

EUREKA! N°30, 2009

35
Sociedade Brasileira de Matemática

TERCEIRA FASE – NÍVEL 2 (8o. e 9o. Anos)

SEGUNDO DIA
PROBLEMA 4
p2 + q2
Mostre que se p,q são inteiros positivos primos tais que r = é inteiro,
p+q
então r é primo.

PROBLEMA 5
Seja ABC um triângulo acutângulo e O, H seu circuncentro e ortocentro,
respectivamente. Sabendo que
AB
= BH = OB,
2
calcule os ângulos do triângulo ABC.

PROBLEMA 6
Sendo A um conjunto de números inteiros, definimos S(A) como o conjunto
formado pelas somas de dois elementos, não necessariamente distintos e D(A)
como o conjunto formado pelas diferenças de dois elementos, não necessariamente
distintos. Por exemplo, se A = {1, 2, 3, 10} então S(A) = {2, 3, 4, 5, 6, 11, 12, 13,
20} e D(A) = {–9, –8, –7, –2, –1, 0, 1, 2, 7, 8, 9}.

Mostre que existe um conjunto finito A tal que S(A) tem no máximo 1097 elementos
e D(A) tem no mínimo 10100 elementos.

TERCEIRA FASE – NÍVEL 3 (Ensino Médio)

PRIMEIRO DIA
PROBLEMA 1
Vamos chamar de garboso o número que possui um múltiplo cujas quatro
primeiras casas de sua representação decimal são 2008. Por exemplo, 7 é garboso
pois 200858 é múltiplo de 7 e começa com 2008. Observe que
200858 = 28694 × 7 .
Mostre que todos os inteiros positivos são garbosos.

PROBLEMA 2
Sobre uma reta há um conjunto S de 6n pontos. Destes, 4n são escolhidos ao acaso
e pintados de azul; os 2n demais são pintados de verde. Prove que existe um
segmento que contém exatamente 3n pontos de S, sendo 2n pintados de azul e n
pintados de verde.

EUREKA! N°30, 2009

36
Sociedade Brasileira de Matemática

PROBLEMA 3
Sejam x, y, z reais quaisquer tais que x + y + z = xy + yz + zx. Encontre o valor
mínimo de
x y z
2
+ 2 + 2
x +1 y +1 z +1

TERCEIRA FASE – NÍVEL 3 (Ensino Médio)

SEGUNDO DIA
PROBLEMA 4
Seja ABCD um quadrilátero cíclico e r e s as retas simétricas à reta AB em relação
às bissetrizes internas dos ângulos ∠CAD e ∠CBD , respectivamente. Sendo P a
interseção de r e s e O o centro do círculo circunscrito a ABCD, prove que OP é
perpendicular a CD.

PROBLEMA 5
Prove que para quaisquer inteiros a > 1 e b > 1 existe uma função f dos inteiros
positivos nos inteiros positivos tal que f (a ⋅ f ( n)) = b ⋅ n para todo n inteiro
positivo.

PROBLEMA 6
O profeta venusiano Zabruberson enviou a seus discípulos uma palavra de 10000
letras, sendo cada uma delas A ou E: a Palavra Zabrúbica. Seus seguidores
passaram a considerar, para 1 ≤ k ≤ 10000 , cada palavra formada por k letras
consecutivas da Palavra Zabrúbica uma palavra profética de tamanho k. Sabe-se
que há no máximo 7 palavras proféticas de tamanho 3. Determine o número
máximo de palavras proféticas de tamanho 10.

SOLUÇÕES – TERCEIRA FASE – NÍVEL 1 (6o. e 7o. Anos)

PROBLEMA 1
SOLUÇÃO DE LUCAS CAWAI JULIÃO (CAUCAIA – CE)
a) Vamos chamar o lado do quadrado de vértice C de x, e o lado do quadrado de
vértice A de y.
Como os retângulos que estão vizinhos a esses quadrados têm o dobro da área
deles, então eles irão ter a largura com a mesma medida dos quadrados e
comprimento, igual ao dobro do lado do quadrado. Veja a figura:

EUREKA! N°30, 2009

37
Sociedade Brasileira de Matemática

A y 2y x B
y

2x
2y 2x – y

2x y
x x

D 2x x C

Podemos ver que um lado do quadrado maior mede 3x. Para calcularmos o lado do
quadrilátero central, basta retirarmos o que não pertence a ele. Logo, retiraremos
x + y . Mas isso ocorrerá dos dois lados, então os dois lados do quadrilátero
destacado são iguais a 2x – y. Assim temos que ele é um quadrado.

b) Como um lado do quadrado maior é 12, e já havíamos falado que também é


igual a 3x. Logo x = 4. Mas também podemos perceber que a medida 2x é
8
equivalente a 3y. Como x = 4, então y = .
3
Agora, como o lado do quadrado destacado é 2x – y, então sua área é
( 2x − y )
2
. Substituindo x e y, e resolvendo temos que a área do quadrado destacado
256
é .
9

PROBLEMA 2
SOLUÇÃO DE LARA VIANA DE PAULA CABRAL e RAFAEL RODRIGUES ROCHA DE MELO
(FORTALEZA – CE)
a) A sequencia é 2008, 3119, 4220, 5331, 6442, 7553, 8664, 9775, 886, 997, 8, 9, 0
b) Independente do dígito que ocupa a 1ª posição do número, após uma certa
quantidade de operações, ele chegará a 9 e, basta mais uma operação para ele
chegar a 0, que “desaparecerá”, e o número ficará assim com um dígito a menos.
Em seguida, independente do dígito que agora ocupa a 2ª posição, após uma certa
quantidade de operações ele também chegará a 9 e, logo depois, a 0, que também
“desaparecerá”, e o número terá assim outro dígito a menos.
Continuando esse processo até o número ter um único dígito, esse dígito também
chegará a 9 e, depois, a 0, encerrando o processo.

EUREKA! N°30, 2009

38
Sociedade Brasileira de Matemática

PROBLEMA 3
SOLUÇÃO OFICIAL DA BANCA
Seja fn o número de maneiras possíveis de cobrir o tabuleiro 3 × n . Se a primeira
coluna é coberta por uma peça vertical, falta cobrir um tabuleiro 3 × ( n − 1) . Senão,
começamos com três peças na horizontal, e falta cobrir um tabuleiro 3 × ( n − 3) .
Assim, temos f n = f n-1 + f n- 3 , para todo n ≥ 4. Como claramente temos f1 = 1,
f 2 = 1 e f 3 = 2 , temos f4 = f3 + f1 = 3, f5 = f4 + f2 = 4, f6 = f5 + f3 = 6, f7 = f6 + f4 = 9,
f8 = f7 + f5 = 13, f9 = f8 + f6 = 19, f10 = f9 + f7 = 28, f11 = f10 + f8 = 41, f12 = f11 + f9 = 60,
f13 = f12 + f10 = 88, f14 = f13 + f11 = 129 e, finalmente, f15 = f14 + f12 = 189.
Assim, as respostas são:
a) 1, 1, 2, 3, 4, 6, 9, respectivamente.
b) De 189 maneiras.

PROBLEMA 4
SOLUÇÃO OFICIAL DA BANCA
a) Não é possível. Para que seja possível cobrir o plano com uma figura, em cada
vértice determinado pelas figuras que a cobrem a soma dos ângulos internos deve
ser 180º ou 360º:

Soma 180º Soma 360º


Todo pentágono pode ser cortado em três triângulos, de modo que a soma de seus
ângulos internos é 3 ⋅180 = 540 . Assim, cada ângulo interno de um pentágono
540
regular é = 108 . Como 108 < 180 < 2 ⋅ 108 e 3 ⋅108 < 360 < 4 ⋅108 ,
5
não é possível cobrir o plano com cópias de um pentágono regular.

b) Note que, como m( Aˆ ) + m( Bˆ ) = 180 , EA e BC são paralelos, de modo que


EABC é um losango. Assim CE = DE = CD e CDE é um triângulo equilátero.

EUREKA! N°30, 2009

39
Sociedade Brasileira de Matemática

Assim é possível cobrir o plano com o pentágono ABCDE, como mostra a figura a
seguir:
A B

E C
… …

PROBLEMA 5
SOLUÇÃO ADAPTADA DA SOLUÇÃO DE GABRIEL YASHIMI BARRÓN TOYAMA
(SÃO PAULO – SP)

a) Observe que 200800 dividido por 17 tem resto 13. Assim, 200804 é múltiplo de
17 e, portanto, 17 é garboso. Na verdade, 17 tem infinitos múltiplos começados por
2008.

b) Seja x a quantidade de algarismos de um número inteiro positivo y qualquer.


Considere o resto m da divisão de 2008 ⋅ 10 x por y. Temos 0 ≤ m ≤ y − 1, e portanto
1 ≤ y − m ≤ y. Como y tem x algarismos, y < 10 x , e logo 1 ≤ y − m ≤ y < 10 x . Assim,
y – m tem no máximo x algarismos, e portanto 2008 ⋅ 10 x + ( y − m ) começa sua
representação decimal por 2008. Como 2008 ⋅ 10 x = y ⋅ z + m, para algum inteiro z,
2008 ⋅ 10 x + ( y − m ) = y ⋅ ( z + 1) é múltiplo de y, e portanto y é garboso.

SOLUÇÕES – TERCEIRA FASE – NÍVEL 2 (8o. e 9o. Anos)

PROBLEMA 1
SOLUÇÃO DE DANIEL DOS SANTOS BOSSLE (PORTO ALEGRE – RS)
Perceba que a distribuição dos números no tabuleiro forma dominós 2 × 1, pois a
cada casa está associada exatamente uma casa vizinha com o mesmo número.
Logo, para que todos os dominós se encaixem, deve haver um número par de casas
no tabuleiro.
Assim, é impossível cobrir um tabuleiro 2007 × 2007.

EUREKA! N°30, 2009

40
Sociedade Brasileira de Matemática

Por outro lado, é possível cobrir um 2008 × 2008. Uma solução é a seguinte,
bastando repetir o padrão até o fim:

1 1 2 2 3 3 4 4
2 2 3 3 4 4 1 1
3 3 4 4 1 1 2 2
4 4 1 1 2 2 3 3

Assim, as respostas são:


a) Não
b) Sim

PROBLEMA 2
SOLUÇÃO DE JOÃO LUCAS CAMELO SÁ (FORTALEZA – CE)
Suponha que os ângulos suplementares sejam adjacentes.
Vamos chamá-los de A e B e os outros de C , D e E .
Observe a montagem a seguir:
r
A B A B A

C E C E C
B D D
D D
E C E C E C

A B A B A B
s

Como A + B + C + D + E = 180°(5 − 2) = 540° e A + B = 180°, temos que


C + D + E = 360°. Logo, é possível encaixar os pentágonos desta maneira, em
“faixas”. Ao encaixarmos faixa sobre a outra pelas retas r e s da figura, poderemos
cobrir o plano inteiro.
Temos agora que analisar o caso quando os suplementares (dessa vez A e C ) não
são adjacentes. Sendo B o ângulo do vértice entre A e C , e D e E os outros
ângulos, temos a seguinte configuração:
(Lembrando que B + D + E = 360° )

EUREKA! N°30, 2009

41
Sociedade Brasileira de Matemática

B C A E B E

A
C
D B
E D D
D = D
B E
D
C
A

E A C B E B

Hexágono α

Vamos mostrar que podemos agrupar vários “hexágonos” α de modo a cobrir o


plano. Basta seguir as faixas abaixo:

Faixas:

B E
D D

E B D D

B E D D
D D

E B D D

B E D D
D D

E B

Como os ângulos de fora valem 360° − B − E = D e os da ponta também, é


possível encaixar, cobrindo todo o plano.

EUREKA! N°30, 2009

42
Sociedade Brasileira de Matemática

PROBLEMA 3
SOLUÇÃO DE JOÃO LUCAS CAMELO SÁ (FORTALEZA – CE)

Seja p um primo ≥3 e diferente de 5. Temos


52 p − 2 − 1 52( p −1) − 1 (5( p −1) − 1) (5( p −1) + 1)
= = . Analisando módulo p, pelo pequeno
2p 2p p 2
Teorema de Fermat, 5 p −1 ≡ 1(mod p ) ⇔ 5 p −1 − 1 ≡ 0(mod p ) e
p −1 p −1
5 ≡ 1(mod 2) ⇒ 5 + 1 ≡ 0(mod 2).
p −1
5 −15 p −1 + 1 5n − 2 − 1
Assim, é inteiro ⇒
é inteiro e é inteiro quando n = 2p.
p 2 n
Como existem infinitos primos p, existem infinitos n que satisfazem a condição do
enunciado.

PROBLEMA 4
SOLUÇÃO DE JOÃO LUCAS CAMELO SÁ (FORTALEZA – CE)
p 2 + q 2 2q 2
Suponha p = q ⇒ = = q ⇒ r = q ⇒ r é primo.
p+q 2q
p 2 + q 2 p 2 − q 2 + 2q 2 2q 2
Caso contrário, = = p−q+ ∈ ⇒ p + q | 2q 2 .
p+q p+q p+q
Analogamente, p + q | 2 p 2 . Como p ≠ q,( p, q) = 1 ⇒ (2 p 2 , 2q 2 ) = 2( p 2 , q 2 ) = 2.
Logo p + q | (2 p 2 , 2q 2 ) ⇒ p + q | 2 ⇒ p + q ≤ 2. Mas p, q ≥ 2, absurdo. Logo,
p = q , e portanto r é primo.
Obs.: João Lucas utilizou a notação (a, b) = mdc(a, b).

PROBLEMA 5
SOLUÇÃO DE MARIA CLARA MENDES SILVA (PIRAJUBA – MG)
A
30°
45°

M H

120°
O
150°
30° 45°
α = 15° 15° = α
C B

EUREKA! N°30, 2009

43
Sociedade Brasileira de Matemática

O circuncentro é equidistante dos 3 vértices.


AB
BH = OC = OA = OB = .
2
AB = 2OB.
Aí AB 2 = 2OB 2 = OB 2 + OA2 ⇔ ∆OAB é retângulo em O pela recíproca do
Teorema de Pitágoras. Como OA = OB, ele também é isósceles e
O AB = OBA = 45° . Seja M o ponto médio de AC. OM é perpendicular a AC, e
temos que OM =
BH OA
2
=
2
. Aí AO é o dobro de OM, logo sen M AO = 0,5 e ( )
 π
como M AO ∈ 0,  , M AO = 30°. Logo OCM = 30°, já que ∆COA é isósceles.
 2
Assim COA = 180° − 60° = 120° e COB = 360° − 90° − 120° = 150°.
180° − 150°
Finalmente α = = 15° . Os ângulos são:
2
30° + 45° = 75°,30° + 15° = 45° e 45° + 15° = 60°.

PROBLEMA 6
SOLUÇÃO OFICIAL DA BANCA
Considere o conjunto C = {0,1,3}. Temos S (C ) = {0,1, 2,3, 4,6} e
D(C ) = {−3, −2, −1,0,1, 2,3}. Assim, S(C) tem 6 elementos, enquanto D(C) tem 7.
Vamos agora, para cada inteiro positivo n, considerar o conjunto An dos naturais
com no máximo n algarismos na base 7, todos pertencentes a C, isto é,
 n −1  n −1 n −1
A( n ) = ∑ a j ⋅ 7 j ; a j ∈ C ,0 ≤ j ≤ n − 1 . Dados a = ∑ a j ⋅ 7 j e b = ∑ b j ⋅ 7 j em
 j =0  j =0 j =0
n −1
An , com a j , bj ∈ C, para 0 ≤ j ≤ n − 1, temos a + b = ∑ ( a j + bj ) ⋅ 7 j e
j =0

n −1  n −1 
a − b = ∑ ( a j − b j ) ⋅ 7 j. Assim, temos S ( An ) = ∑ u j ⋅ 7 j , u j ∈ S (C ),0 ≤ j ≤ n − 1
j =0  j =0 
 n −1 
e D ( An ) = ∑ v j ⋅ 7 j , v j ∈ D (C ),0 ≤ j ≤ n − 1 .
 j =0 
Como S(C) tem 6 elementos entre 0 e 6, e a representação em base 7 é única, S( An )
7 n − 1 n −1
tem exatamente 6n elementos. Por outro lado, como = ∑ 3 ⋅ 7 j , temos
2 j =0

EUREKA! N°30, 2009

44
Sociedade Brasileira de Matemática

7n − 1  7n − 1 
+ D ( An ) : =  + m, m ∈ D ( An )  =
2  2 
 n−1   n−1 
= ∑( 3 + v j ) ⋅ 7 j , v j ∈ D(C),0 ≤ j ≤ n − 1 = ∑ rj ⋅ 7 j , rj ∈{0,1,2,3,4,5,6},0 ≤ j ≤ n − 1 =
 j =0   j =0 
= {0,1, 2,...,7 − 1} ,
n
pois todo inteiro entre 0 e 7 −1
n
pode ser representado na base 7,
usando os algarismos 0, 1, 2, 3, 4, 5 e 6.

Assim, D( An ) tem 7n elementos.


105 10100
Como 66 < < 105 < 76 ,6120 < 20 < 1097 < 10100 < 7120 , e portanto o conjunto
2 2
A = A120 , que tem 3120 elementos, certamente satisfaz as condições do enunciado.

SOLUÇÕES – TERCEIRA FASE – NÍVEL 3 (ENSINO MÉDIO)

PROBLEMA 1
SOLUÇÃO DE CUSTÓDIO M. B. SILVA
Seja n um inteiro positivo. Como n é inteiro finito, n < 10k , para algum k.
Seja p = 10k ⋅ 2008 + n − q , onde q < n é o resto da divisão de 10k ⋅ 2008 por n.
Assim, n − q < 10k e portanto p começa com 2008 e é múltiplo de n.

PROBLEMA 2
SOLUÇÃO DE RAFAEL SUSSUMU YAMAGUTI MIADA (SÃO PAULO – SP)
Considere que os pontos são numerados de 1 a 6n. Sabe-se que, para 1 ≤ b ≤ 3n + 1,
um segmento de b até 3n + b − 1 contém exatamente 3n pontos e será representado
como b → 3n + b − 1. Como os pontos devem ser consecutivos, pode-se formar 3n
+ 1 segmentos (1 → 3n;2 → 3n + 1;3 → 3n + 2,...,3n + 1 → 6n) .
Vamos analisar a variação do número de pontos verdes de b → 3n + b − 1 até
b + 1 → 3n + b. Considere que em b → 3n + b − 1 há z pontos verdes. Pode
acontecer:

z pontos → z – 1 pontos: Sai um ponto verde e não entra outro ponto verde no
segmento.

z pontos → z pontos: Sai um ponto verde e entra outro ponto verde no


segmento.

z pontos → z pontos: Não sai um ponto verde e não entra outro ponto

EUREKA! N°30, 2009

45
Sociedade Brasileira de Matemática

verde no segmento.

z pontos → z + 1 pontos: Não sai um ponto verde e entra outro ponto verde
no segmento.

(quantidade de pontos verdes) (em relação ao segmento anterior)

Maior variação: 1 ponto para mais ou para menos.


Considere então os pontos de 1 → 3n e 3n + 1→ 6n (usando o fato de que há 2n
pontos verdes e 4n pontos azuis).
Se em 1 → 3n há n + k pontos verdes, em 3n + 1 > 6n haverá n – k pontos verdes.
Além disso, em 1 → 3n haverá 2n – k pontos azuis e em 3n – 1 → 6n haverá 2n+ k
pontos azuis. Temos os seguintes casos:
a) para k = 0: é verdadeiro na primeira e última sequência (1→ 3n e 3n + 1 → 6n)
(verdadeiro!).

b) para k < 0: deve aumentar o número de pontos verdes de 1→ 3n a 3n + 1 → 6n,


porém com a máxima variação entre cada sequência é 1 ponto e
n + k < n < n – k , conclui-se que existe a → a + 3n – 1 talque o número de pontos
verdes é igual a n (verdadeiro!). O caso k > 0 é análogo.
Como há 3n pontos na sequência a → a + 3n – 1 e os pontos são verdes ou azuis,
pode-se qualificar o fato de que existe uma sequência a → a + 3n – 1 tal que há n
pontos verdes e 2n pontos azuis como verdadeiro.

PROBLEMA 3
SOLUÇÃO DE RÉGIS PRADO BARBOSA (FORTALEZA – CE)
Para (x, y, z) = (–1, –1, 1), temos
x + y + z = –1 – 1 + 1 = –1
xy + yz + zx = (–1)(–1) + (–1)1 + (–1)1 = –1
e
x y z 1 1 1 1
+ 2 + 2 =− − + =− .
x +1 y +1 z +1
2
2 2 2 2
1
Provaremos que − é o mínimo, ou seja, sendo x + y + z = xy + yz + zx
2
x y z 1
mostraremos que 2 + 2 + 2 ≥− .
x +1 y +1 z +1 2

A desigualdade é equivalente a

EUREKA! N°30, 2009

46
Sociedade Brasileira de Matemática

2∑ x( y 2 + 1 )( z 2 + 1 ) ≥ −( x2 + 1 )( y 2 + 1 )( z 2 + 1 )
cic

⇔ 2∑( xy 2 z 2 + xy 2 + xz 2 + x ) ≥ −( x2 y 2 z 2 + x2 y 2 + y 2 z 2 + z 2 x2 + x2 + y 2 + z 2 + 1 )
cic

⇔ ∑( x2 y 2 + x2 + 2 xy 2 z 2 + 2 x ) + 2∑ x2 y + x2 y 2 z 2 + 1 ≥ 0
cic sim

(usamos as anotações ∑
cic
e ∑
sim
para denotar soma cíclica e soma simétrica

respectivamente).
Mas
∑ xy 2 z 2 = xyz( xy + yz + zx ) = xyz( x + y + z ) = ∑ x 2 yz e
cic cic

∑x
sim
2
y = x y + xy + x z + x z + y z + yz + 3 xyz − 3 xyz
2 2 2 2 2 2

= xy ( x + y + z ) + yz ( x + y + z ) + zx ( x + y + z ) − 3 xyz
= ( xy + yz + zx )( x + y + z ) − 3xyz = ( x + y + z ) − 3 xyz
2

Assim, a desigualdade é equivalente a


∑( x y 2 + x 2 + 2 x 2 yz ) + 2 x + 2 ( x + y + z ) − 6 xyz + x 2 y 2 z 2 + 1 ≥ 0
2 2

cic

Agora montemos quadrados:


x 2 − 2 xyz + y 2 z 2 = ( x − yz ) 2
y 2 − 2 xyz + x 2 z 2 = ( y − zx) 2
z 2 − 2 xyz + x 2 y 2 = ( z − xy ) 2
( x + y + z ) 2 + 2( x + y + z ) + 1 = ( x + y + z + 1) 2
( x + y + z ) 2 + 2 xyz ( x + y + z ) + x 2 y 2 z 2 = ( x + y + z + xyz ) 2
Observando que ∑x
cic
2
yz = xyz ( x + y + z ) , a desigualdade é equivalente a

( x − yz ) 2 + ( y − zx) 2 + ( z − xy ) 2 + ( x + y + z + 1) 2 + ( x + y + z + xyz ) 2 ≥ 0 ,
que é verdadeira pois A 2 ≥ 0 para todo A real.
1
Logo o mínimo da soma dada é − .
2
PROBLEMA 4
SOLUÇÃO DE MARCO ANTONIO LOPES PEDROSO (SANTA ISABEL – SP)
Para termos as bissetriz de C AD e C BD determinados não precisa saber as
posições dos pontos C e D, basta a posição do ponto médio do arco DC , que

EUREKA! N°30, 2009

47
Sociedade Brasileira de Matemática

vamos chamar de L. Perceba também que a mediatriz de CD é perpendicular a CD


e passa por O; desse modo o nosso problema passa a ser provar que P também está
na mediatriz de CD. Mas já sabemos que L está na mediatriz de CD, então na
realidade queremos provar que O, P, L são colineares (agora podemos esquecer o C
e o D e pensar só no L).
Então nosso problema passa a ser:
L

O
α β
α β
A B Provar que O,P,L
são colineares.

Perceba que LA é a bissetriz externa do ∆PAB relativa ao vértice A; e LB é a


bissetriz externa do ∆PAB relativa ao vértice B.
Logo L é o ex-incentro do ∆PAB relativo a P; desse modo PL é bissetriz do ângulo
APB.
É natural pensar no incentro I do ∆PAB; como PL é bissetriz de APB então P, I, L
são colineares.
L

O
α β
α β
A 90° – α 90° – β B
90° – α I 90° – β

EUREKA! N°30, 2009

48
Sociedade Brasileira de Matemática

Como bissetriz interna e externa de um ângulo são perpendiculares então IA ⊥ AL;


assim como LB ⊥ BI , desse modo temos L AI = 90° = LBI ⇒ o quadrilátero LAIB
é inscritível, e seu centro está no centro da hipotenusa do ∆LAI . Logo O está no
ponto médio de LI (pois é o centro da circunferência que passa por L, A, B).
Então L, O, I são colineares, e como já provamos que P, I, L são colineares então
P, O, L são colineares, como queríamos demonstrar.

PROBLEMA 5
SOLUÇÃO DE GABRIEL LUIS MELLO DALALIO (S.J. DOS CAMPOS – SP)
Seja S n a sequência crescente dos inteiros positivos não múltiplos de a e Rn a
sequência crescente dos inteiros positivos não múltiplos de b.
Definindo f : * → *:
se a |n, f (n)=Rk , onde k é tal que S k = n

f (n)= se n = ak, com k ∈ *, b|k, f (n) = b ⋅ Si onde i é tal que Ri = k,
se n = abj, com j ∈ *, f ( n) = ab ⋅ f ( j )

Devemos provar que a recursão acaba, mas de fato, como ao passar pelo terceiro
caso precisamos do valor da função em um número j < n , já que n = abj, e
portanto alguma hora a recursão cai em algum dos dois primeiros casos.
Vamos provar que f (af (n)) = bn para todo n inteiro positivo.
Temos os seguintes casos:
1) a | n ⇒ f ( af (n)) = f ( a Rk ), onde S k = n, e temos
f (a Rk ) = b ⋅ S k =bn , donde f (a f (n)) = bn quando a | n .
2) a |n . Temos dois subcasos:
2.i) n = ak, b | k ⇒ f (a f (n)) = f ( af (ak)) = f (ab . Si), onde Ri = k, e temos
f (ab Si) = abf(Si) = abRi = bak = bn ⇒ f(af(n)) = bn quando n = ak e b | k .
2.ii) n = abj, j ∈ * ⇒ f (a f (n)) = f (af (abj)) = f (ab· a f( j)) = ab· f(af (j)).
Se ab | j, f (af ( j )) = bj ⇒ f (af (n)) = ab ⋅ bj = bn.
Se ab | j, j=abi,i ∈ *, i < j. Podemos supor, por indução, que f(a f(j)) = bj,
donde f ( af (n)) = ab ⋅ f (af ( j )) = ab ⋅ bj = bn, c.q.d.

PROBLEMA 6
SOLUÇÃO OFICIAL DA BANCA
Seja f(n) o número de palavras proféticas de tamanho n.

EUREKA! N°30, 2009

49
Sociedade Brasileira de Matemática

Temos f (1) ≤ 2, f (2) ≤ 4 e f (3) ≤ 7. Assim, há uma palavra XYZ de três


letras X,Y,Z pertencentes a {A,E} que não é profética. Para n ≥ 1, uma
palavra proféticas de tamanho n + 3 pode ser de três tipos (no máximo):
- uma palavra profética de tamanho n + 2 seguida da letra U ∈{A, E} distinta de Z,
- uma palavra profética de tamanho n + 1 seguida de TZ, onde T ∈{ A,E} é a letra
distinta de Y ou uma palavra profética de tamanho n seguida de SYZ,
onde S ∈ { A,E} é a letra distinta de X.
Assim, f ( n + 3) ≤ f ( n + 2) + f ( n + 1) + f ( n ) para todo n ≥ 1; logo,
f (4) ≤ 13, f (5) ≤ 24, f (6) ≤ 44, f (7) ≤ 81, f (8) ≤ 149, f (9) ≤ 274 e f (10) ≤ 504.
Vamos agora ver que é possível que haja 504 palavras proféticas
de tamanho 10. Para isso observamos inicialmente que há 504 palavras
de tamanho 10 que não têm três letras E consecutivas. Para n ≥ 1, uma
palavra de tamanho n + 3 sem 3 E´s seguidos pode ser de três tipos,
todos distintos: uma palavra sem 3 E´s seguidos de tamanho n + 2 seguida
da letra A, uma palavra sem 3 E´s seguidos de tamanho n + 1 seguida de
AE ou uma palavra sem 3 E´s seguidos de tamanho n seguida de AEE. Isso
mostra que, se g(n) é o número de palavras de n letras, todas A ou E,
sem 3 E´s consecutivos, então g(n + 3) = g(n + 2) + g(n + 1) + g(n) para todo
n ≥ 1. Como g(1) = 2, g(2) = 4 e g(3) = 7, segue que g(10) = 504. Agora; como
11 ⋅ 504 < 10000, basta listar todas essas palavras, colocar uma letra A no final de
cada uma delas e concatená-las, completando com letras A até obtermos uma
palavra de10.000 letras para concluir.

EUREKA! N°30, 2009

50
Sociedade Brasileira de Matemática

XXX OLIMPÍADA BRASILEIRA DE MATEMÁTICA


Problemas e Soluções da Primeira Fase Nível Universitário

PROBLEMA 1
Determine todos os valores inteiros de n para os quais a equação x 3 − 13 x + n = 0
possua três raízes inteiras.

PROBLEMA 2
Considere as retas de equações paramétricas

( x, y, z ) = (0,0,1) ⋅ t
( x, y, z ) = (1, 2,0) + (1,0,0) ⋅ t
( x, y, z ) = (1,1,1) + (0,1,0) ⋅ t
( x, y, z ) = (1,0,0) + (1,1,1) ⋅ t
Quantas retas intersectam simultaneamente as 4 retas acima?

PROBLEMA 3
Esmeralda passeia pelos pontos de coordenadas inteiras do plano. Se, num dado
momento, ela está no ponto (a, b), com um passo ela pode ir para um dos seguintes
pontos: (a +1, b), (a –1, b), (a,b + 1) ou (a, b – 1). De quantas maneiras Esmeralda
pode sair do (0, 0) e andar 2008 passos terminando no (0,0)?

PROBLEMA 4
Suponha que existem duas matrizes inversíveis n × n , A e B, diferentes da matriz
identidade I e satisfazendo as relações
 A7 = I
 −1
 ABA = B
2

Mostre que existe um inteiro k > 0 tal que B k = I e determine o menor k com esta
propriedade.

PROBLEMA 5
Dizemos que uma hipérbole cobre um ponto se este pertence a uma das duas
regiões infinitas por ela determinada que contêm os focos.
Qual o menor número de hipérboles necessárias para cobrir todos os pontos do
plano?

EUREKA! N°30, 2009

51
Sociedade Brasileira de Matemática

PROBLEMA 6
πk 
Seja Pn = ∑ sen
0≤ k ≤ n
n
 .
 n 
Pn Pn +1
Calcule lim .
n →∞ n

SOLUÇÕES PRIMEIRA FASE – NÍVEL UNIVERSITÁRIO

PROBLEMA 1
Sejam α , β , γ as três raízes do polinômio. As relações de Girard implicam que
s = α + β + γ = 0 e p = αβ + βγ + γα = −13, logo α 2 + β 2 + γ 2 = s 2 − 2 p = 26.
As únicas possibilidades para {α , β , γ }:{+4, −3, −1} ou {−4, +3, +1}.
Logo n = −αβγ = ±12.

PROBLEMA 2
PRIMEIRA SOLUÇÃO
Sejam A = (0,0, a), B = (1 + b,2,0), C = (1,1 + c,1) e D = (1 + d , d , d ) pontos genéricos,
um sobre cada uma das 4 retas dadas. Esses pontos são colineares se, e somente se,
a matriz

 0 0 a 1
1 + b 2 0 1
M =
 1 1+ c 1 1
 
1 + d d d 1
tem posto 2. Subtraindo a primeira linha de M das demais obtemos a matriz
equivalente

 0 0 a 1
1 + b 2 − a 0 
M =
 1 1 + c 1 − a 0
 
1 + d d d − a 0
que tem posto 2 se, e somente se
2 −a d d −a
1+ b = = e 1+ d = = .
1+ c 1− a 1+ c 1− a

Três dessas quatro igualdades nos permitem expressar b, c e d em função de a:

EUREKA! N°30, 2009

52
Sociedade Brasileira de Matemática

1 a−2 1
b= ,c = , d = ; a quarta, então, equivale a 2a 2 − a − 2 = 0, equação que
a −1 a a
possui duas soluções reais. Logo há duas retas que intersectam simultaneamente as
4 retas dadas.

SEGUNDA SOLUÇÃO
As coordenadas de Plücker das quatro retas são:

r1 : 0,0,1 0,0,0
r2 : 1,0,0 0,0, −2
r3 : 0,1,0 −1,0,1
r4 : 1,1,1 0, −1,1
Qualquer solução r : d x , d y , d z px , p y , pz tem que ser ortogonal às quatro retas.
Resolvendo o sistema linear, temos que r : 2α , β − α ,0 β , 2α + β ,α ; finalmente,
como rd ⋅ rp = 0, temos que ter
β −3 ± 17
β 2 + 3αβ − 2α 2 = 0 ⇔ =
α 2
logo existem duas retas que intersectam as quatro retas dadas.

PROBLEMA 3
Cada movimento de subida (↑) deva ser compensado por um movimento de
descida ( ↓ ) , e cada movimento para a esquerda ( ← ) deve ser compensado por um
movimento para a direita ( → ) . Assim, se fizermos k movimentos ↑ , temos que
fazer também k movimentos ↓, 1004 − k movimentos ← e 1004 − k movimentos
→.
Para cada k, o número de caminhos é, portanto, igual ao número de anagramas
com 4 letras distintas, duas aparecendo k vezes e as outras duas, 1004 − k vezes
cada. Logo a resposta é

1004
2008!
R=∑
k =0 k ! k !(1004 − k )!(1004 − k )!

EUREKA! N°30, 2009

53
Sociedade Brasileira de Matemática

1004
2008! 1004!1004!
=∑ ⋅
k = 0 1004!1004! k !k !(1004 − k )!(1004 − k )!
2
 2008  1004 1004 
= ∑  .
 1004  k = 0  k 

Considere agora um conjunto de n meninos e n meninas. De quantas maneiras


 2n 
podemos escolher um grupo de n crianças? Por um lado, a resposta é   .
 n
2
 n  n   n 
Por outro lado, se escolhermos k meninos    =   maneiras de formar
 k  n − k   k 
um grupo. Logo
2
n
 n   2n 
∑   = 
k =0  k   n
e portanto
2
 2008 
R=  .
1004 
SEGUNDA SOLUÇÃO
2
 2008 
Esmeralda tem   maneiras de escolher dois conjuntos de 1004 passos dentre
 1004 
os 2008 passos que andará: o conjunto X dos passos para cima ou para a direita
( ↑ ou → ) e o conjunto Y dos passos para baixo ou para a direita ( ↓ ou →) .
Essas escolhas determinam unicamente todos os passos: O conjunto dos passos
para a direita será X ∩ Y , para a esquerda será X c ∩ Y c , para cima X ∩ Y c e para
baixo X c ∩ Y (onde X c e Y c denotam os complementares de X e Y,
respectivamente). Se X ∩ Y = k , teremos X c ∩ Y = 1004 − k , X ∩ Y c = 1004 − k
2
 2008 
e X c ∩ Y c = k . Assim, a resposta é   .
 1004 

PROBLEMA 4
Note que B 4 = ( B 2 ) = ( ABA−1 ) = AB 2 A−1 = A ( ABA−1 ) A−1 = A2 BA−2 .
2 2

De forma análoga,

EUREKA! N°30, 2009

54
Sociedade Brasileira de Matemática

B 8 = A3 BA−3 , B16 = A4 BA−4 , B 32 = A5 BA−5 , B 64 = A6 BA−6 , B128 = A7 BA−7 = B, logo


B = I.
127

Suponha agora que existe 0 < k < 127 tal que B k = I ; como 127 é primo, o m.d.c.
entre 127 e k vale 1. Pelo Teorema de Bézout, existem a, b inteiros tais que
127 a + kb = 1; então
B = B1 = B127 a + kb = ( B127 ) ⋅ ( B k ) = I .
a b

Isso é uma contradição, pois B ≠ I . Logo o menor valor de k é 127.


Nota: Não é necessário exibir exemplos de tais matrizes A e B, mas tais exemplos
existem. Podemos fazer n = 127, enumerar uma base de 127 como {e0 , e1 ,..., e126 } e
definir A e B por Ae j = e2 j (mod127) e Be j = e2 j +1(mod127) ,0 ≤ j ≤ 126.

PROBLEMA 5
Como toda hipérbole tem duas assíntotas não paralelas, dadas duas hipérboles,
sempre existe pelo menos um ponto comum a uma assíntota de cada uma delas.
Esse ponto não é coberto por qualquer uma das duas hipérboles, logo é impossível
cobrir todo o plano com apenas duas hipérboles.
As seguintes três hipérboles cobrem todo o plano:
x2 − y 2 = 1
( y − 2) 2 − x 2 = 1
( y + 2) 2 − x 2 = 1
De fato, para qualquer ( x, y ) ∈ 2
, vale pelo menos das seguintes desigualdades:
x > y + 1,
2 2
x < ( y − 1) − 1
2 2
ou x 2 < ( y + 2) 2 − 1. Com efeito,
max {( y − 2) 2 2
}
− 1, ( y + 2 ) − 1 = ( y + 2 ) − 1 = y 2 + 4 y + 3 > y 2 + 1.
2

Assim o número mínimo de hipérboles necessárias para cobrir todos os pontos do


plano é 3.

PROBLEMA 6
Observe inicialmente que
π π π
( Pn − 1) < ∫0 senn xdx < ( Pn + 1) .
n n
π
Defina I n = ∫ sen n xdx. Integrando por partes, temos que, para n > 2,
0
π
I n = ∫ sen n −1 xsenxdx =
0

EUREKA! N°30, 2009

55
Sociedade Brasileira de Matemática

π π
=  − sen n −1 x cos x  − ∫ (n − 1) sen n − 2 x cos x(− cos x)dx =
0 0
π
= ( n − 1) ∫ sen n−2
x(1 − sen 2 x) dx =
0

= ( n − 1) I n − 2 − (n − 1) I n
n −1 I
e portanto I n = I n − 2 ; daí segue que lim n = 1. Como, para todo n,
n n →∞ I
n−2

In
I n−2 ≥ I n −1 ≥ I n , temos lim = 1.
n →∞ I
n −1

π
Como I1 = 2 e I 2 = , temos que para todo k ≥ 0,
2
(2k )!! (2k + 1)!!
I 2 k +1 = ⋅ 2, I 2 k + 2 = ⋅π ,
(2k + 1)!! (2k + 2)!!
2π (2k + 1)
onde n !! = ∏ k > 0 ( n − 2k ). Assim, lim(2k + 1) I 2 k +1 I 2 k + 2 = lim = 2π , e
k →∞ k →∞ 2k + 2
2π (2k + 2)
lim(2k + 2) I 2 k + 2 I 2 k + 3 = lim = 2π , ou seja, lim k →∞ nI n I n +1 = 2π , donde
k →∞ k →∞ 2k + 3
PP nI I 2
lim n n +1 = lim n 2n +1 = .
k →∞ n k →∞ π π

Obs.: Alternativamente, pela aproximação de Stirling,


(2k − 1)!! (2k )!
= =
(2k )!! (2k )!!2
(2k )!
= 2
~
 2k k !
(2k ) 2 k e −2 k 4π k (1 + O(k −1 ))
~ k 2 k −2 k ~
4 k e 2π k (1 + O( k −1 ))
1  −3 
~ + Ok 2 
πk  
2π  −3  2n
e portanto I n ~ + O  n 2  . Mas isso implica ~ + O(1) e portanto
n   π
Pn Pn +1 2
lim n →∞ = .
n π

EUREKA! N°30, 2009

56
Sociedade Brasileira de Matemática

XXX OLIMPÍADA BRASILEIRA DE MATEMÁTICA


Problemas e Soluções da Segunda Fase Nível Universitário

PRIMEIRO DIA

PROBLEMA 1
x 2008
Seja f n : → dada por f ( x) =
+ x 2 − nx, para cada n ∈ , e seja mn o
2008
m
valor mínimo assumido por f n . Determine α ∈ tal que o limite lim αn existe e
n →∞ n

é não-nulo, e calcule esse limite (para esse valor de α ).

PROBLEMA 2
No 3
, considere a elipse ε1 definida pelas equações x=0 e
41 y + 41z − 80 yz + 36 y + 36 z − 81 = 0, e a elipse
2 2
ε2 definida pelas equações
y = 0 e 71x + 41z − 40 xz + 18 x + 36 z − 81 = 0. Prove que existe uma única
2 2

superfície cônica de revolução no 3


que intersecta o plano x = 0 em ε1 e o plano
y = 0 em ε2, e determine a interseção dessa superfície com o plano z = 0.

PROBLEMA 3

Mostre que existem a1 , a2 ,...,∈ tais que a série ∑a x
n =1
n
n
converge para todo

x∈ e, definindo f ( x) = ∑ an x n , temos:
n =1

i) f é uma bijeção de em que satisfaz f ´( x) > 0, ∀x ∈ .


ii) f ( ) = A , onde A = {α ∈ | ∃p( x) polinômio com coeficientes inteiros
tal que p(α ) = 0} é o conjunto dos algébricos reais.

SEGUNDO DIA

PROBLEMA 4
Seja Q = [0,1] × [0,1] ⊂ 2 um quadrado de lado 1 e f : Q → uma função
contínua e positiva. Prove que é possível dividir Q em duas regiões R1 e R2 de

EUREKA! N°30, 2009

57
Sociedade Brasileira de Matemática

mesma área, separadas por um segmento de reta, tais que


∫ R1 f ( x, y)dxdy = ∫ R2 f ( x, y)dxdy.
PROBLEMA 5
Prove que não existe uma matriz 7 × 7, A = (aij )1≤i , j ≤ 7 , com aij ≥ 0,1 ≤ i, j ≤ 7 cujos
autovalores (contados com multiplicidade) são: 6, –5, –5, 1, 1, 1, 1.

PROBLEMA 6

λ 1 ∞ 1
Prove que ∑ (λ + n
n =1
2 2
)
< ∑
2 n =1 λ + n 2
, ∀λ ≥ 0.

SOLUÇÕES SEGUNDA FASE – NÍVEL UNIVERSITÁRIO

PRIMEIRO DIA

PROBLEMA 1
SOLUÇÃO DE EDUARDO POÇO (SÃO PAULO – SP)
Seja xn = 2007 n. Como mn é o mínimo de f n , então:
 1 
mn ≤ f ( xn ) =  − 1 n 2007 n + 2007 n 2 , ∀n ∈ .
 2008 
2008
x
Seja agora g n ( x) = − nx.
2008
Temos f n ( x ) ≥ g n ( x ) ,∀x ∈ , e sendo kn o valor mínimo de g n :
f n ( x ) ≥ g n ( x ) ≥ kn ,∀x ∈ ⇒ mn ≥ kn ,∀n ∈
Calculando kn : g n ( x ) mínimo ⇒ g´n ( x ) = 0 ⇒
 1 
⇒ x 2007 − n = 0 ⇒ x = 2007 n ⇒ kn =  − 1 n 2007 n
 2008 
Assim:
1 1 2
 1  1+ 2007  1  1+ 2007
 − 1  n ≤ m n ≤  − 1  n + n 2007

 2008   2008 
2 1 1 2008
Como <1+ , devemos ter α = 1 + = , e o limite é
2007 2007 2007 2007
EUREKA! N°30, 2009

58
Sociedade Brasileira de Matemática

1 2007
−1 = − .
2008 2008

PROBLEMA 2
SOLUÇÃO OFICIAL DA BANCA
Sejam ( x0 , y0 ,z0 ) o vértice do cone e ( a,b,c ) um vetor não nulo na direção do eixo
do cone. Se ( x, y,z ) é um ponto do cone, existe um ângulo θ tal que o ângulo
entre os vetores ( x − x0 , y − y0 ,z − z0 ) e ( a,b,c ) é sempre θ ou π − θ , e em
particular o módulo de seu cosseno é igual a cos θ . Assim,
a( x − x0 ) + b( y − y0 ) + c( z − z0 ) ( a,b,c ) ⋅ ( x − x0 , y − y0 ,z − z0 ) = cosθ ,
=
a +b +c ⋅
2 2 2
( x − x0 ) + ( y − y0 ) + ( z − z0 )
2 2 2
( a,b,c ) ⋅ ( x − x0 , y − y0 ,z − z0 )
e podemos escrever a equação do cone como
( a ( x − x ) + b( y − y ) + c ( z − z ) )
0 0 0
2
−d 2
(( x − x ) + ( y − y ) + ( z − z ) ) = 0,
0
2
0
2
0
2
onde

d = cosθ . a2 + b2 + c2 , a qual pode ser escrita na forma


Ax + By + Cz + Dxy + Exz + Fyz + Gx + Hy + Iz + J = 0. Como, fazendo x = 0, obtemos
2 2 2

uma equação da elipse ε1, podemos supor (ajustando o módulo de (a, b, c), e
trocando os sinais, se necessário) que B = 41, C = 41, F = –80, H = 36, I = 36 e
J = –81, e como, fazendo y = 0, obtemos uma equação da elipse ε2, teremos A =
71, E = –40 e G = 18 (note que os coeficientes de z2, z e o coeficiente constante são
os mesmos nas equações dadas de ε1 e ε2).
Assim, basta determinar D.
Comparando as equações, devemos ter a2 − d 2 = 71, b2 − d 2 = c2 − d 2 = 41,2ac = −40 e
2bc = −80 ou a2 − d 2 = −71, b2 − d 2 = c2 − d 2 = −41,2ac = 40 e 2bc = 80. Nos dois casos,
b = 2a, donde b2 − d 2 ≥ a2 − d 2 , o que não acontece no primeiro caso. Assim, ocorre
o segundo caso, e portanto D = −2ab = −b2 . Como b2 − d 2 = c2 − d 2 , b = c , e portanto
2 80 2bc
D = −b2 = − b = − bc = − =−
= −40. Em particular, a interseção do cone com o
2 2
plano z = 0 é dada pela equação 71x2 + 41y2 − 40xy +18x + 36y − 81 = 0, e logo é uma
elipse.
(
Obs.: Fazendo ( x0 ,y0 ,z0 ) = (1,2,2) ,( a,b,c) = 10, 2 10, 2 10 e d = 9 obtemos a )
equação desejada (após trocar os sinais).

EUREKA! N°30, 2009

59
Sociedade Brasileira de Matemática

PROBLEMA 3
SOLUÇÃO OFICIAL DA BANCA
Construiremos uma função que satisfaz as condições do enunciado da forma

f (x) = x + ∑ cn ⋅ g n ( x ) , onde as funções g n têm derivada limitada na reta real e as
n =1

constantes cn
convergem a 0 muito rápido, de tal forma que
1 1 3
{ 2
}
g´n ≤ sup g´n ( x ) ,x ∈ < n+1 ,∀n ≥ 1, o que garantirá que < f´ ( x ) < ,∀x ∈ ,
2 2
e portanto f será uma bijeção crescente de em .
Os conjuntos e A são subconjuntos enumeráveis densos de . Podemos
enumerá-los como = { xn ,n ∈ } e A = { yn ,n ∈ } . Para cada n ≥ 1 construiremos
um conjunto Bn ⊂ com n elementos, com Bn ⊂ Bn +1 ,∀n ≥ 1 e tomaremos
g n ( x ) = ∏ sen ( x − b ) , que é limitada e tem derivada limitada em . Tomamos
b∈Bn

B1 = {0} e c1 = 0.
1 1 1 1
Como sen u ⋅ cos v = sen( u + v) + sen( u − v) , sen u ⋅ sen v = cos ( u − v) − cos ( u + v) ,
2 2 2 2
sen ( x + a ) = cos a ⋅ sen x + sen a ⋅ cos x, cos ( x + a ) = cos a cos x − sen a sen x,
sen ( −u ) = − sen u e cos ( −u ) = cos u, podemos escrever gn ( x ) como
( −1) x
k 2 k +1

∑ ( r ( ) sen ( kx ) + s( ) cos ( kx ) ) .
n ∞
Como, para todo x ∈ , sen x = ∑
n n
e
k = 0 ( 2k + 1)!
k k
k =0

( −1) x2k
k
∞ ∞
cos x = ∑ , temos que g n ( x ) = ∑ am( n) x m , onde os am( n) são tais que existe
k =0 ( 2k ) ! m=0

( n)
uma constante kn > 0 tal que a m ≤ kn nm m!,∀m ≥ 0.
2m
m 3
Como, para todo m ≥ 1, m ! ≥ ∏ k >   , temos que
k>
m  3 
3
2m 3 m
(n) m  n ⋅ 32 3 
am ⋅ m m 2
≤ kn ⋅ n ⋅ mm m 2
m! < k n ⋅ n ⋅ m
m m 2
  = kn ⋅  1 6  , donde
3  m 
lim am( n) ⋅ mm 2 = 0. Em particular, existe jn ≥ 1 tal que am( n) < 1 mm 2 ,∀m ≥ jn .
m →∞

EUREKA! N°30, 2009

60
Sociedade Brasileira de Matemática

 1 1 
Definimos tn = max am( ) + 1 e wn = min  n +1
n
, n .
0≤ m< jn
 2 ⋅ sup g´n ( x ) ,x ∈ 2 ⋅ tn ⋅ jnjn 2 
 { }
1
Temos então wn ⋅ am( ) < n m 2 ,∀m ≥ 1,n ≥ 0. Escolheremos as constantes cn
n

2 ⋅m
satisfazendo sempre cn < wn (sempre escolhemos cn depois de já ter escolhido o
conjunto Bn , e logo já tendo determinado a função gn ). Teremos então
∞ ∞ ∞
f ( x ) = x + ∑ cn ⋅ gn ( x ) = ∑ am ⋅ x m , onde a1 = 1 + ∑ cn a1( n) satisfaz
n =1 m =1 n =1
∞ ∞
1
a1 < 1 + ∑ wn ⋅ a1( n ) < 1 + ∑ n
=2 e
n =1 n =1 2
∞ ∞ ∞ ∞
1 1
am ≤ ∑ cn ⋅ am( n) < ∑ wn ⋅ am( n) < ∑ = m 2 ,∀m ≥ 2. Em particular ∑a m ⋅ xm
n=1 2 ⋅ m
n m2
n=1 n =1 m m =1
convergirá para todo x ∈ .
Escolheremos agora os conjuntos Bn e as constantes cn −1 ,n ≥ 2, recursivamente.
Para n par, tomamos o menor k tal que xk ∉ Bn−1 , e definimos Bn = Bn−1 ∪ { xk } .
Como π é irracional, g n−1 ( xk ) ≠ 0. Assim, como A é denso em , podemos
escolher cn −1 ∈ ( −wn −1 ,wn −1 ) tal que xk + ∑
1≤ m< n −1
cm ⋅ g m ( xk ) + cn −1 ⋅ g n−1 ( xk ) ∈ A, e tal

que, se y = a + 2kπ , com a ∈ , k ∈ \ {0} , então y + ∑


1≤ m≤ n −1
cm ⋅ g m ( y ) ∉ A (de fato

o conjunto dos cn −1 ∈ tais que a última condição falha é enumerável – usamos


aqui o fato de π ser transcendente).
n−2
Seja agora n≥3 ímpar. Seja f n−2 ( x ) = x + ∑ cm ⋅ g m ( x ) . Temos que
m =1

1 2 < f´n−1 ( x ) < 3 2 ,∀x ∈ , e logo f n − 2 é uma bijeção crescente de em .


Considere agora o menor r ∈ tal que yr ∉{ f n− 2 ( b ) ,b ∈ Bn −1} . Temos, por
construção, que f −1
n−2 ( yr ) não é da forma a + 2kπ , com a ∈ Bn −1 ,k ∈ . Assim,
gn −1 ( f n−−12 ( yr ) ) ≠ 0, e portanto podemos escolher cn −1 ∈ ( − wn−1 ,wn −1 ) tal que, se
n −1
f n−1 ( x ) = x + ∑ cm ⋅ gm ( x ) , f n−−11 ( yr ) ∈ , e tal que, se y = a + 2k π , com
m =1

a ∈ ,k ∈ \ {0} , então f n−1 ( y ) = y + ∑


1≤ m≤ n −1
cm ⋅ gm ( y ) ∉ A. Tomamos então

EUREKA! N°30, 2009

61
Sociedade Brasileira de Matemática

Bn = Bn −1 ∪ { f n−−11 ( yr )} . Ao final dessa construção é claro que f ( x ) ∈ A para todo


x ∈ e f −1 ( y ) ∈ para todo y ∈ A.
Obs.: Se não quisermos usar o fato de π ser transcendente podemos trocar as
funções sen ( x − c ) por sen (πα ( x − c ) ) , com α transcendente.

SEGUNDO DIA

PROBLEMA 4
SOLUÇÃO DE RENATO REBOUÇAS DE MEDEIROS (S.J. DOS CAMPOS – SP)
Lema: Se um segmento de reta divide o retângulo em duas regiões R1 e R2 da
mesma área, então o segmento passa pelo centro do retângulo, que é (0,5, 0,5), no
caso.
Prova: De fato, se o segmento dividisse igualmente a área do retângulo e não
passasse pelo seu centro, aconteceria:

A
M (0,5, 0,5) Segmento que
passa pelo centro ( AB )

B
Segmento que não
passa pelo centro ( AB )

1
Nessa situação cada metade limitada por AB teria área , enquanto cada metade
2
limitada por A´ B também teria essa mesma área (por semelhança de triângulos,
A´ B divide igualmente) e o triângulo AA´B teria área nula, absurdo. Assim, tais
segmentos passam pelo centro do retângulo.
y
1

M Ө

0 x
1

EUREKA! N°30, 2009

62
Sociedade Brasileira de Matemática

Girando um segmento (de comprimento variável) em torno do centro do retângulo


(M), graças ao lema, obtêm-se todas as possibilidades de dividir a área em duas
regiões de mesma área.
Orientando esse segmento, é possível denominar por A1 a integral sobre a região à
direita da seta ( R1 ) e por A2 a integral sobre a região à esquerda da seta, que será
R2 . Além disso, A1 = A1 (θ ) e A2 = A2 (θ ) , sendo θ o ângulo no sentido anti-
horário entre o segmento e um eixo horizontal por M.
Resumindo, A1 (θ ) = ∫ f ( x, y ) dxdy e A1 (θ ) = ∫ f ( x, y ) dxdy.
R1 R2

Outra constatação é que A1 (π ) = A2 ( 0 ) e A2 (π ) = A1 ( 0 ) , pois as regiões R1 e


R2 de θ =0 são trocadas para θ =π. Ainda, tem-se que
A1 (θ ) + A2 (θ ) = ∫ f ( x, y ) dxdy.
Q

Como f :Q → *
+ é contínua, há indícios de que A1 (θ ) e, conseqüentemente,
A2 (θ ) são funções contínuas em θ em todo o intervalo [ 0,π ] em que as funções
A1 e A2 merecem análise. Provaremos este fato no final da solução.
Como as funções A1 (θ ) e A2 (θ ) têm soma constante e trocam de valor entre
θ = 0 e θ = π , além de serem contínuas, então as duas assumem o mesmo valor
para algum θ , com 0 ≤ θ < π .
De fato, basta ver que, se A1 ( 0 ) ≠ A2 ( 0 ) , então a função G (θ ) = A1 (θ ) − A2 (θ ) no
mesmo domínio de A1 e A2 tem um sinal em θ = 0 e o sinal contrário em θ = π ,
pois G ( 0 ) = A1 ( 0 ) − A2 ( 0 ) e G (π ) = A1 (π ) − A2 (π ) = −  A1 ( 0 ) − A2 ( 0 )  = −G ( 0 ) .
Então G tem uma raiz entre 0 e π , por ser contínua e pelo Teorema do Valor
Intermediário. Nessa raiz, digamos θ´, têm-se as condições desejadas. Se
A1 ( 0 ) = A2 ( 0 ) , as condições já são satisfeitas para o segmento inicial.
Resta provar que A1 (θ ) é contínuo em θ .
Para isso, basta ver que, por f ser contínua em [ 0,1] × [ 0 ,1] , e ainda positiva, assume
um valor máximo M e um valor mínimo m. Sendo assim,
2
 2 2
A1 (θ + ∆θ ) − A1 (θ ) ≤ ( M − m ) ⋅   ⋅ θ (pois e na figura são ≤ ).
 2  2

EUREKA! N°30, 2009

63
Sociedade Brasileira de Matemática

∆Ө

2 ⋅ε
Assim, ∀ε > 0, para δ = > 0. Temos θ − θ 0 < δ ⇒ A1 (θ ) − A1 (θ 0 ) < ε
M − m +1
e assim A1 é contínua, permitindo aplicar o Teorema do Valor Intermediário para
G como feito acima.

PROBLEMA 5
SOLUÇÃO DE FABIO DIAS MOREIRA (RIO DE JANEIRO – RJ)
Se os autovalores de A são (6, – 5, –5, 1, 1, 1, 1), os autovalores de B = A3 são
(216, –125, – 125, 1, 1, 1, 1). Por outro lado, se X = ( xij ) e Y = ( yij ) são matrizes
com xij ≥ 0, yij ≥ 0, então a mesma propriedade vale para Z = XY : de fato,
7
zij = ∑ xij ykj ≥ 0. Logo bij ≥ 0 ∀ 1 ≤ i, j ≤ 7 e daí Tr B = b11 + ... + b77 ≥ 0.
k =1
Mas Tr B = 216 − 125 − 125 + 1 + 1 + 1 + 1 = −30, contradição.
Portanto não existe nenhuma matriz A com a propriedade pedida.

PROBLEMA 6
SOLUÇÃO OFICIAL DA BANCA
Começamos com a expressão de senx como produto infinito:

 x 
2
senx = x∏ 1 − 2 2 .
n =1  n π 
Sabendo que essa igualdade vale para todo x ∈ , como os dois lados definem
eix − e − ix
funções analíticas em (lembramos que senx = ), a igualdade vale para
2i
todo x ∈ .
Fazendo x = −π iy obtemos

EUREKA! N°30, 2009

64
Sociedade Brasileira de Matemática

senh (π y ) eπ y − e −π y ∞
 x2  ∞
 y
= = senx = x ∏  1 − 2 2  = π iy ∏  1 +  donde
i 2i n =1  n π  n =1  n 

 y2 
senh (π y ) = π y∏  1 + 2  , para todo y ∈ , e, em particular, para todo y > 0.
n =1  n 

 y2 
Aplicando logaritmos, obtemos log senh (π y ) = logπ + log y + ∑log 1 + 2  , ∀y > 0.
n=1  n 
Derivando, temos:
cos h (π y ) 1 ∞
2y
(*) π = +∑ 2 , ∀ y > 0.
senh (π y ) y n =1 y + n 2
Derivando novamente, obtemos, para todo y > 0,
π2 (
π 2 ( ( senh(π y) ) ) − ( cos h(π y) )
2 2
) =− 1 + ∞ 
2 4 y2 

− = ∑ y −
+ n 2 ( y 2 + n2 ) 2 
( senh (π y ) ) ( senh (π y ) )
2 2
y2 n =1
2

 
 ∞ 
1 ∞
y2 = 1 − π2
Portanto, 2  ∑ 2 − 2 ∑ , ∀y > 0.
 n =1 y + n 2 n =1 ( y + n )  y 2 ( senh (π y ) )2
2 2 2
 
O lado direito dessa igualdade é positivo para todo y > 0, de fato, isso equivale a
senh (π y ) > π y , ∀ y > 0; os dois lados dessa última desigualdade coincidem para
 eπ y + e − π y 
y = 0, e a derivada do lado esquerdo, que é π cosh (π y ) = π   é maior
 2 
que π , que é a derivada do lado direito, para todo y > 0.

1 ∞
y2
Portanto temos, para todo y > 0, ∑ 2 > 2 ∑ .
n =1 y + n n =1 ( y + n )
2 2 2 2

Essa desigualdade obviamente também vale para y = 0. Finalmente, tomando


y = λ , concluímos que a desigualdade do enunciado vale para todo λ ≥ 0.

EUREKA! N°30, 2009

65
Sociedade Brasileira de Matemática

XXX OLIMPÍADA BRASILEIRA DE MATEMÁTICA


Resultado – Nível 1 (6o. e 7o. Anos)
NOME CIDADE – ESTADO PRÊMIO
Guilherme Renato Martins Unzer São Paulo – SP Ouro
Francisco Markan Nobre de Souza Filho Fortaleza – CE Ouro
Rafael Rodrigues Rocha de Melo Fortaleza – CE Ouro
Liara Guinsberg São Paulo – SP Ouro
Pedro Augusto de Paula Barbosa Belo Horizonte – MG Ouro
Matheus Silva Lima Bragança Paulista – SP Ouro
Vinícius Canto Costa Salvador – BA Prata
Michel Rozenberg Zelazny São Paulo – SP Prata
Vinicius Luiz Ferreira Belo Horizonte – MG Prata
Lucas Cawai Julião Pereira Caucaia – CE Prata
Mário de Mello Figueiredo Neto Petrópolis – RJ Prata
Henrique Vieira G. Vaz São Paulo – SP Prata
Elias Brito Oliveira Brasília – DF Prata
Igor Albuquerque Araujo Belo Horizonte – MG Prata
Luis Fernando Veronese Trivelatto Cascavel – PR Prata
Daniel Lima Santanelli Rio de Janeiro – RJ Prata
Glauber Lima da Cunha Júnior Fortaleza – CE Bronze
Tiago Sueda Limone Jundiaí – SP Bronze
Leyberson Pereira Assunção Fortaleza – CE Bronze
João Marcos Carnieleto Nicolodi Florianópolis – SC Bronze
Fellipe Sebastiam da Silva P. Pereira Recife – PE Bronze
Lucas Cardoso Zuccolo São Paulo – SP Bronze
Maria Clara Cardoso São Paulo – SP Bronze
Daniel Vincent Cacsire Garibay São Carlos – SP Bronze
Rafael Tedeschi Eugênio Pontes Barone Araçatuba – SP Bronze
Gabriel Nogueira Coelho de Togni de Souza Rio de Janeiro – RJ Bronze
Thomas Akio Ikeda Valvassori Mogi das Cruzes – SP Bronze
William Cechin Guarienti Porto Alegre – RS Bronze
Gabriel Yoshimi Barrón Toyama Brasília – DF Bronze
Daniel Behrens Cardoso Salvador – BA Bronze
Ricardo Vidal Mota Peixoto Vassouras – RJ Menção Honrosa
Jardel da Silva Pires Santos Dumont – MG Menção Honrosa
Felipe Mendes de Holanda Lins Recife – PE Menção Honrosa
Daniel Shinji Hoshi São Paulo – SP Menção Honrosa
Júlio César de Barros Santo André – SP Menção Honrosa
Gabriel Queiroz Moura Teresina – PI Menção Honrosa
Lucas Carvalho Daher Anápolis – GO Menção Honrosa
Daiana Luna Rio de Janeiro – RJ Menção Honrosa
Filipe Santana do Vale Salvador – BA Menção Honrosa
Rosane Thiemi Toma Gundim Campo Grande – MS Menção Honrosa
Guilherme de Oliveira Rodrigues Fortaleza – CE Menção Honrosa
Breno Soares da Costa Vieira Jaboatão dos Guararapes – PE Menção Honrosa
Lara Viana de Paula Cabral Fortaleza – CE Menção Honrosa
Nicolas Chiu Ogassavara São Paulo – SP Menção Honrosa
Juliana Amoedo Amoedo Plácido Salvador – BA Menção Honrosa
Bruno Cordeiro de Macedo São Paulo – SP Menção Honrosa
Arthur Schott Lopes Curitiba – PR Menção Honrosa
Bruno Costa Silva Rio de Janeiro – RJ Menção Honrosa
Alessandro Augusto Pinto de Oliveira Pacanowski Rio de Janeiro – RJ Menção Honrosa
Vitória Carolina Rondon Pereira Jacareí – SP Menção Honrosa
Jorge Luiz Soares Pereira Rio de Janeiro – RJ Menção Honrosa
Pedro Carvalho da Fonseca Guimarães Cachoeira Paulista – SP Menção Honrosa
Guilherme Ryu Odaguiri Kobori São Paulo – SP Menção Honrosa
Lorena Marroni Carvalho Amparo – SP Menção Honrosa
Luiz Akyhito Miyazaki Serra Negra – SP Menção Honrosa
Marcos Felipe Nunes Lino Ribeiro São Paulo – SP Menção Honrosa
Gustavo Souto Henriques Campelo João Pessoa – PB Menção Honrosa
João Vitor Fernandes Paiva Rio de Janeiro – RJ Menção Honrosa
Fábio Kenji Arai São Paulo – SP Menção Honrosa
Roberto Tadeu Abrantes de Araújo Rio de Janeiro – RJ Menção Honrosa
Lucas Butschkau Vida Pinhais – PR Menção Honrosa
João Ribeiro Pacheco Salvador – BA Menção Honrosa

EUREKA! N°30, 2009

66
Sociedade Brasileira de Matemática

Nível 2 (8o. e 9o. Anos)


Nome Cidade - Estado Prêmio
João Lucas Camelo Sá Fortaleza – CE Ouro
Gustavo Haddad F. e Sampaio Braga S. J. dos Campos – SP Ouro
Gabriel Militão Vinhas Lopes Fortaleza – CE Ouro
Rubens Cainan Sabóia Monteiro Fortaleza – CE Ouro
Maria Clara Mendes Silva Pirajuba – MG Ouro
Marla Rochana Braga Monteiro Fortaleza – CE Ouro
Otávio Araújo de Aguiar Fortaleza – CE Prata
Caíque Porto Lira Fortaleza – CE Prata
Tuane Viana Pinheiro Rio de Janeiro – RJ Prata
Carlos Henrique de Andrade Silva Fortaleza – CE Prata
Otávio Augusto de Oliveira Mendes Pilar do Sul – SP Prata
Rafael Kazuhiro Miyazaki São Paulo – SP Prata
Vinicius Cipriano Klein Venda do Imigrante – ES Prata
Daniel dos Santos Bossle Porto Alegre – RS Prata
Mateus Braga de Carvalho Teresina – PI Prata
Luiz Henrique Vieira Leão Rio de Janeiro – RJ Prata
Kayo de França Gurgel Fortaleza – CE Prata
Davi Coelho Amorim Fortaleza – CE Bronze
Rodolfo Rodrigues da Costa Fortaleza – CE Bronze
Rafael Henrique dos Santos Santa Cruz do Rio Pardo – SP Bronze
Ivan Tadeu Ferreira Antunes Filho Lins – SP Bronze
Lucas Cordeiro Herculano Fortaleza – CE Bronze
Breno Leví Corrêa Campo Belo – MG Bronze
Tábata Cláudia Amaral de Pontes São Paulo – SP Bronze
Marina Pessoa Mota Fortaleza – CE Bronze
Marcos Massayuki Kawakami São Paulo – SP Bronze
Bruno Ferri de Moraes São Paulo – SP Bronze
Tiago Leandro Estevam Dias Rio de Janeiro – RJ Bronze
Gabriel Pacianotto Gouveia São Paulo – SP Bronze
Filipe José Oliveira Sabóia Fortaleza – CE Bronze
Daniel Prince Carneiro São Lourenço – MG Bronze
Bruno Moraes Moreno Porto Nacional – TO Bronze
Renan Fernandes Moreira Taubaté – SP Menção Honrosa
Murilo Dória Guimarães São Paulo – SP Menção Honrosa
Lucas Nishida Pedreira – SP Menção Honrosa
Leonardo Ferreira Patrício Rio de Janeiro – RJ Menção Honrosa
Victor Kioshi Higa São Paulo – SP Menção Honrosa
Marina de Moura Faleão Recife – PE Menção Honrosa
Lucas Almeida Pereira de Lima Recife – PE Menção Honrosa
Arthur Ribeiro Notaro Recife – PE Menção Honrosa
Pedro Mendonça de Lima Goiânia – GO Menção Honrosa
Rafael Ferreira Antonioli S.B. do Campo – SP Menção Honrosa
Lucas Okumura Ono São Paulo – SP Menção Honrosa
Vinicius Affonso de Carvalho São Paulo – SP Menção Honrosa
Julio Barros de Paula Taubaté – SP Menção Honrosa
João Francisco Goes Braga Takayanagi São Paulo – SP Menção Honrosa
Débora Barreto Ornellas Salvador – BA Menção Honrosa
Luis Henrique Kobayashi Higa Campo Grande – MS Menção Honrosa
Vítor Gabriel Barra Souza Juiz de Fora – MG Menção Honrosa
Pedro Ivo Coêlho de Araújo Caucaia – CE Menção Honrosa
Guilherme Cherman Perdigão de Oliveira Rio de Janeiro – RJ Menção Honrosa
Cesar Nobuo Moniwa Ishiuchi Campinas – SP Menção Honrosa
Letícia Dias Mattos Contagem – MG Menção Honrosa
Raul Aragão Rocha Recife – PE Menção Honrosa
Tiago de Ávila Palhares Brasília – DF Menção Honrosa
Gabriel Leal Teixeira de Souza Rio de Janeiro – RJ Menção Honrosa

EUREKA! N°30, 2009

67
Sociedade Brasileira de Matemática

Nível 3 (Ensino Médio)


Nome Cidade - Estado Prêmio
Henrique Pondé de Oliveira Pinto São Paulo – SP Ouro
Régis Prado Barbosa Fortaleza – CE Ouro
Marcelo Tadeu de Sá Oliveira Sales Salvador – BA Ouro
Guilherme Philippe Figueiredo São Paulo – SP Ouro
Marcelo Matheus Gauy S.J. do Rio Preto – SP Ouro
Gabriel Luís Mello Dalalio S.J. dos Campos – SP Ouro
Renan Henrique Finder São Paulo – SP Prata
Alfredo Roque de Oliveira Freire Filho Salvador – BA Prata
Rafael Tupynambá Dutra Belo Horizonte – MG Prata
Marco Antonio Lopes Pedroso Santa Isabel – SP Prata
Thiago da Silva Pinheiro São Paulo – SP Prata
Eduardo Queiroz Peres Jundiaí – SP Prata
Davi Lopes Alves de Medeiros Fortaleza – CE Prata
Ricardo Turolla Bortolotti Rio de Janeiro – RJ Prata
Alex Atsushi Takeda Londrina – PR Prata
Deborah Barbosa Alves São Paulo – SP Prata
Marlen Lincoln da Silva Fortaleza – CE Bronze
Hugo Fonseca Araújo Rio de Janeiro – RJ Bronze
Rafael Parpinel Cavina São Paulo – SP Bronze
Thiago Ribeiro Ramos Varginha – MG Bronze
Rafael Horimoto de Freitas São Paulo – SP Bronze
Rafael Alves da Ponte Fortaleza – CE Bronze
Robério Soares Nunes Ribeirão Preto – SP Bronze
Gustavo Lisbôa Empinotti Florianópolis – SC Bronze
Henrique Hiroshi Motoyama Watanabe São Paulo – SP Bronze
Ivan Guilwon Mitoso Rocha Fortaleza – CE Bronze
José Airton Coêlho Lima Filho Fortaleza – CE Bronze
Hudson do Nascimento Lima Fortaleza – CE Bronze
Hanon Guy Lima Rossi São Paulo – SP Bronze
Ricardo Bioni Liberalquino Maceió – AL Bronze
Luiz Filipe Martins Ramos Rio de Janeiro – RJ Bronze
Illan Feiman Halpern São Paulo – SP Bronze
Matheus Secco Torres da Silva Rio de Janeiro – RJ Bronze
Matheus Araújo Marins Rio de Janeiro – RJ Menção Honrosa
Paulo Cesar Neves da Costa Brasília – DF Menção Honrosa
Leonardo Pereira Stedile São Paulo – SP Menção Honrosa
Jonas Rocha Lima Amaro Fortaleza – CE Menção Honrosa
Matheus Barros de Paula Taubaté – SP Menção Honrosa
Rafael Sussumu Yamaguti Miada Valinhos – SP Menção Honrosa
Rafael Alves da Silva Teresina – PI Menção Honrosa
James Jun Hong São Paulo – SP Menção Honrosa
Joas Elias dos Santos Rocha Muribeca – SE Menção Honrosa
Gelly Whesley Silva Neves Fortaleza – CE Menção Honrosa
João Mendes Vasconcelos Fortaleza – CE Menção Honrosa
Marilia Valeska Costa Medeiros Fortaleza – CE Menção Honrosa
Júlio Cézar Batista de Souza Salvador – BA Menção Honrosa
José Cabadas Duran Neto Salvador – BA Menção Honrosa
Thiago Saksanian Hallak São Paulo – SP Menção Honrosa
Luiz Eduardo Schiller Rio de Janeiro – RJ Menção Honrosa
Esdras Muniz Mota Fortaleza – CE Menção Honrosa
Fábio Luís de Mello São Paulo – SP Menção Honrosa
Ana Beatriz Prudêncio de Almeida Rebouças Fortaleza – CE Menção Honrosa
Victorio Takahashi Chu São Paulo – SP Menção Honrosa
Isabella Amorim Gonçalez Maceió – AL Menção Honrosa
Grazielly Muniz da Cunha Fortaleza – CE Menção Honrosa
José Leandro Pinheiro Deputado Irapuan Pinheiro – CE Menção Honrosa
Antônio Deromir Neves Silva Júnior Fortaleza – CE Menção Honrosa

EUREKA! N°30, 2009

68
Sociedade Brasileira de Matemática

Nível Universitário
Nome Cidade - Estado Prêmio
Fábio Dias Moreira Rio de Janeiro – RJ Ouro
Rafael Daigo Hirama S.J. dos Campos – SP Ouro
Guilherme Rodrigues Nogueira de Souza São Paulo – SP Ouro
Eduardo Poço São Paulo – SP Ouro
Ramón Moreira Nunes Fortaleza – CE Ouro
Renato Rebouças de Medeiros S.J. dos Campos – SP Prata
Thiago Costa Leite Santos São Paulo – SP Prata
Raphael Constant da Costa Rio de Janeiro – RJ Prata
André Linhares Rodrigues Campinas – SP Prata
Edson Augusto Bezerra Lopes Fortaleza – CE Prata
Marcelo de Araújo Barbosa S.J. dos Campos – SP Prata
Levi Máximo Viana Rio de Janeiro – RJ Prata
Thomás Yoiti Sasaki Hoshina Rio de Janeiro – RJ Prata
Felipe Gonçalves Assis Campina Grande – PB Prata
Fernando Nascimetno Coelho S.J. dos Campos – SP Prata
Marcos Victor Pereira Vieira S.J. dos Campos – SP Prata
Reinan Ribeiro Souza Santos Aracajú – SE Bronze
Mateus Oliveira de Figueiredo S.J. dos Campos – SP Bronze
Caio Ishizaka Costa S.J. dos Campos – SP Bronze
Paulo Sérgio de Castro Moreira S.J. dos Campos – SP Bronze
Pedro henrique Milet Pinheiro Pereira Rio de Janeiro – RJ Bronze
Willy George do Amaral Petrenko Rio de Janeiro – RJ Bronze
Jorge Henrique Craveiro de Andrade Rio de Janeiro – RJ Bronze
Leandro Farias Maia Fortaleza – CE Bronze
Alysson Espíndola de Sá Silveira S.J. dos Campos – SP Bronze
Leonardo Ribeiro de Castro Carvalho São Paulo – SP Bronze
Rafael Montezuma Pinheiro Cabral Fortaleza – CE Bronze
Rafael Sampaio de Rezende Fortaleza – CE Bronze
Luty Rodrigues Ribeiro S.J. dos Campos – SP Bronze
Leandro Augusto Lichtenfelz Florianópolis – SC Bronze
Sidney Cerqueira Bispo dos Santos Filho S.J. dos Campos – SP Bronze
André Jorge Carvalho São Paulo – SP Menção Honrosa
Jordan Freitas Piva Rio de Janeiro – RJ Menção Honrosa
Juan Raphael Diaz Simões São Paulo – SP Menção Honrosa
Eduardo Fischer Encantado – RS Menção Honrosa
Adenilson Arcanjo de Moura Júnior Fortaleza – CE Menção Honrosa
Kellem Correa Santos Rio de Janeiro – RJ Menção Honrosa
Roberto Akiba de Oliveira São Paulo – SP Menção Honrosa
José Armando Barbosa Filho S.J. dos Campos – SP Menção Honrosa
Daniel Lopes Alves de Medeiros S.J. dos Campos – SP Menção Honrosa
Vitor Humia Fontoura Rio de Janeiro – RJ Menção Honrosa
Bruno da Silva Santos Belford Roxo – RJ Menção Honrosa
Luca Mattos Möller S.J. dos Campos – SP Menção Honrosa
Marcelo Salhab Brogliato Rio de Janeiro – RJ Menção Honrosa
José Marcos Andrade Ferraro São Paulo – SP Menção Honrosa
Evandro Makiyama São Paulo – SP Menção Honrosa
Antonia Taline de Souza Mendonça Rio de Janeiro – RJ Menção Honrosa
Diego Andrés de Barros Lima Barbosa Rio de Janeiro – RJ Menção Honrosa
Gabriel Ponce São Carlos – SP Menção Honrosa
Luiz Paulo Freire Moreira Fortaleza – CE Menção Honrosa
Felipe Rodrigues Nogueira de Souza São Paulo – SP Menção Honrosa
Igor Magalhães Oliveira Maceió – AL Menção Honrosa
Rafael Ghussn Cano Campinas – SP Menção Honrosa

EUREKA! N°30, 2009

69
Sociedade Brasileira de Matemática

AGENDA OLÍMPICA
XXXI OLIMPÍADA BRASILEIRA DE MATEMÁTICA

NÍVEIS 1, 2 e 3
Primeira Fase – Sábado, 06 de junho de 2009
Segunda Fase – Sábado, 12 de setembro de 2009
Terceira Fase – Sábado, 17 de outubro de 2009 (níveis 1, 2 e 3)
Domingo, 18 de outubro de 2009 (níveis 2 e 3 - segundo dia de prova).

NÍVEL UNIVERSITÁRIO
Primeira Fase – Sábado, 12 de setembro de 2009
Segunda Fase – Sábado, 17 e Domingo, 18 de outubro de 2008

XV OLIMPÍADA DE MAIO
09 de maio de 2009

XX OLIMPÍADA DE MATEMÁTICA DO CONE SUL
14 a 20 de abril de 2009
Mar del Plata – Argentina

L OLIMPÍADA INTERNACIONAL DE MATEMÁTICA
10 a 22 de julho de 2009
Bremen – Alemanha

XVI OLIMPÍADA INTERNACIONAL DE MATEMÁTICA UNIVERSITÁRIA
25 a 30 de julho de 2009
Budapeste, Hungria

XXIV OLIMPÍADA IBEROAMERICANA DE MATEMÁTICA
17 a 27 de setembro de 2009
Querétaro, México

XII OLIMPÍADA IBEROAMERICANA DE MATEMÁTICA UNIVERSITÁRIA

EUREKA! N°30, 2009

70
Sociedade Brasileira de Matemática

COORDENADORES REGIONAIS
Alberto Hassen Raad (UFJF) Juiz de Fora – MG
Américo López Gálvez (USP) Ribeirão Preto – SP
Amarísio da Silva Araújo (UFV) Viçosa – MG
Andreia Goldani FACOS Osório – RS
Antonio Carlos Nogueira (UFU) Uberlândia – MG
Benedito Tadeu Vasconcelos Freire (UFRN) Natal – RN
Carlos Alexandre Ribeiro Martins (Univ. Tec. Fed. de Paraná) Pato Branco – PR
Carmen Vieira Mathias (UNIFRA) Santa María – RS
Claus Haetinger (UNIVATES) Lajeado – RS
Cleonor Crescêncio das Neves (EDETEC) Manaus – AM
Cláudio de Lima Vidal (UNESP) S.J. do Rio Preto – SP
Denice Fontana Nisxota Menegais (UNIPAMPA) Bagé – RS
Edson Roberto Abe (Colégio Objetivo de Campinas) Campinas – SP
Eduardo Tengan (USP) São Carlos – SP
Élio Mega (Grupo Educacional Etapa) São Paulo – SP
Eudes Antonio da Costa (Univ. Federal do Tocantins) Arraias – TO
Fábio Brochero Martínez (UFMG) Belo Horizonte – MG
Florêncio Ferreira Guimarães Filho (UFES) Vitória – ES
Francinildo Nobre Ferreira (UFSJ) São João del Rei – MG
Genildo Alves Marinho (Centro Educacional Leonardo Da Vinci) Taguatingua – DF
Ivanilde Fernandes Saad (UC. Dom Bosco) Campo Grande – MS
Jacqueline Rojas Arancibia (UFPB)) João Pessoa – PB
Janice T. Reichert (UNOCHAPECÓ) Chapecó – SC
João Benício de Melo Neto (UFPI) Teresina – PI
João Francisco Melo Libonati (Grupo Educacional Ideal) Belém – PA
Jose de Arimatéia Fernandes (UFPB) Campina Grande – PB
José Luiz Rosas Pinho (UFSC) Florianópolis – SC
José Vieira Alves (UFPB) Campina Grande – PB
José William Costa (Instituto Pueri Domus) Santo André – SP
Krerley Oliveira (UFAL) Maceió – AL
Licio Hernandes Bezerra (UFSC) Florianópolis – SC
Luciano G. Monteiro de Castro (Sistema Elite de Ensino) Rio de Janeiro – RJ
Luzinalva Miranda de Amorim (UFBA) Salvador – BA
Mário Rocha Retamoso (UFRG) Rio Grande – RS
Marcelo Rufino de Oliveira (Grupo Educacional Ideal) Belém – PA
Marcelo Mendes (Colégio Farias Brito, Pré-vestibular) Fortaleza – CE
Newman Simões (Cursinho CLQ Objetivo) Piracicaba – SP
Nivaldo Costa Muniz (UFMA) São Luis – MA
Osnel Broche Cristo (UFLA) Lavras – MG
Osvaldo Germano do Rocio (U. Estadual de Maringá) Maringá – PR
Raul Cintra de Negreiros Ribeiro (Colégio Anglo) Atibaia – SP
Ronaldo Alves Garcia (UFGO) Goiânia – GO
Rogério da Silva Ignácio (Col. Aplic. da UFPE) Recife – PE
Reginaldo de Lima Pereira (Escola Técnica Federal de Roraima) Boa Vista – RR
Reinaldo Gen Ichiro Arakaki (UNIFESP) SJ dos Campos – SP
Ricardo Amorim (Centro Educacional Logos) Nova Iguaçu – RJ
Sérgio Cláudio Ramos (IM-UFRGS) Porto Alegre – RS
Seme Gebara Neto (UFMG) Belo Horizonte – MG
Tadeu Ferreira Gomes (UEBA) Juazeiro – BA
Tomás Menéndez Rodrigues (U. Federal de Rondônia) Porto Velho – RO
Valdenberg Araújo da Silva (U. Federal de Sergipe) São Cristovão – SE
Vânia Cristina Silva Rodrigues (U. Metodista de SP) S.B. do Campo – SP
Wagner Pereira Lopes (CEFET – GO) Jataí – GO

EUREKA! N°30, 2009

71
CONTEÚDO

AOS LEITORES 2

XV OLIMPÍADA DE MATEMÁTICA DE MAIO


Problemas e Resultado Brasileiro 3

XX OLIMPÍADA DE MATEMÁTICA DO CONE SUL


Problemas e Resultado Brasileiro 7

L OLIMPÍADA INTERNACIONAL DE MATEMÁTICA (IMO)


Problemas e Resultado Brasileiro 9

XXIV OLIMPÍADA IBEROAMERICANA DE MATEMÁTICA


Problemas e Resultado Brasileiro 11

ARTIGOS
PAR OU ÍMPAR? EIS A QUESTÃO
Samuel Barbosa Feitosa e Einstein do Nascimento Júnior 13

GEOMETRIA DO TRIÂNGULO: FATOS E PROBLEMAS


Carlos Yuzo Shine 28

SÉRIE HARMÔNICA DE NÚMEROS PRIMOS


Lenimar Nunes de Andrade 45

COMO É QUE FAZ? 50

SOLUÇÕES DE PROBLEMAS PROPOSTOS 52

PROBLEMAS PROPOSTOS 59

AGENDA OLÍMPICA 61

COORDENADORES REGIONAIS 62
Sociedade Brasileira de Matemática

AOS LEITORES

Por mais um ano consecutivo estamos iniciando a realização da Olimpíada


Brasileira de Matemática.
A Olimpíada Brasileira de Matemática – OBM tem crescido substancialmente nos últimos
anos, contando, em 2009, com a adesão ao evento de mais de 3.700 escolas, sendo 2.180
da rede pública e 1.608 da rede privada de ensino, o que implicou em uma participação
efetiva de cerca de 180.000 jovens estudantes e seus professores. Além disso, a iniciativa
contou com a colaboração de professores universitários em 155 instituições de ensino
superior: eles participaram de todas as atividades, inclusive aquelas referentes à OBM
Nível Universitário em atividades de coordenação, divulgação, treinamento de alunos,
aperfeiçoamento de professores e aplicação das distintas fases da Olimpíada Brasileira de
Matemática. Paralelamente, o projeto apóiou a realização de Olimpíadas Regionais de
Matemática, contando com a participação de 165.148 estudantes das escolas públicas e
privadas em todo o Brasil nas competições estaduais.
No que se refere à participação em competições internacionais, os resultados foram
excelentes: Em particular, Henrique Ponde conquistou a oitava medalha de Ouro do Brasil
na IMO. Além disso, nesta IMO, todos os alunos da equipe brasileria ganharam medalhas.

Durante 2009 a CAPES e o CNPq lançaram o Programa de Iniciação Científica – Mestrado


(PICME) para medalhistas da OBMEP e OBM, beneficiando 19 estudantes premiados na
Olimpíada Brasileira de Matemática – OBM, com o objetivo de aumentar o número de
matemáticos no país, e oferecer uma formação matemática mais sólida a jovens
profissionais de outras áreas científicas e tecnológicas.

Todos estes resultados nacionais e internacionais demonstram que, além de influenciar


positivamente o ensino da Matemática nas instituições de ensino fundamental, médio e
superior, conseguimos detectar jovens muito talentosos que são estimulados a seguir uma
carreira científica, o que é fundamental para o crescimento da Ciência e Tecnologia no
país. A Olimpíada Brasileira de Matemática é um projeto conjunto da Sociedade Brasileira
de Matemática, do Instituto Nacional de Matemática Pura e Aplicada (IMPA) e conta com o
apoio do Conselho Nacional de Desenvolvimento Científico e Tecnológico (CNPq) e do
Instituto Nacional de Ciência e Tecnologia de Matemática (INCTMat).

Os editores

EUREKA! N°31, 2010

2
Sociedade Brasileira de Matemática

XV OLIMPÍADA DE MAIO
PRIMEIRO NÍVEL

PROBLEMA 1
A cada número natural de dois algarismos associamos um dígito da seguinte forma:
Multiplicam-se seus algarismos. Se o resultado é um dígito, este é o dígito
associado. Se o resultado é um número de dois dígitos, multiplicam-se estes dois
algarismos, e se o resultado é um dígito, este é o dígito associado. Caso contrario,
repetimos a operação. Por exemplo, o dígito associado a 32 é o 6 pois 3 ⋅ 2 = 6; o
dígito associado a 93 é o 4 pois 9 ⋅ 3 = 27, 2 ⋅ 7 = 14, 1 ⋅ 4 = 4.
Encontre todos os números de dois algarismos aos que se associa o dígito 8.

PROBLEMA 2
Encontre números primos p, q, r para os quais p + q 2 + r 3 = 200 . Diga todas as
possibilidades.
Obs: Lembre-se que o número 1 não é primo.

PROBLEMA 3
Temos 26 cartões e cada um tem escrito um número. Há dois com o número 1, dois
com o número 2, dois com o 3, e assim por diante até dois com o 12 e dois com o
13. Deve-se distribuir os 26 cartões em pilhas de maneira que sejam cumpridas as
duas condições a seguir:
• Se dois cartões têm o mesmo número estão na mesma pilha.
• Nenhuma pilha contém um cartão cujo número é igual à soma dos números
de dois cartões dessa mesma pilha.
Determine qual é o número mínimo de pilhas que temos que formar. Dê um
exemplo com a distribuição dos cartões para esse número de pilhas e justifique por
quê é impossível ter menos pilhas.

PROBLEMA 4

Três circunferências são tangentes entre si, tal


como mostramos na figura.
A região do círculo exterior que não está coberta
pelos dois círculos interiores tem área igual a 2π.
P Q
Determine o comprimento do segmento PQ.

EUREKA! N°31, 2010

3
Sociedade Brasileira de Matemática

PROBLEMA 5
Pelas linhas de um tabuleiro quadriculado formado por 55 linhas horizontais e 45
linhas verticais caminha uma formiga. Queremos pintar alguns trechos de linhas
para que a formiga possa ir de qualquer cruzamento até outro cruzamento qualquer,
caminhando exclusivamente pelos trechos pintados. Se a distância entre linhas
consecutivas é de 10 cm, qual é a menor quantidade possível de centímetros que
deverão ser pintados?

SEGUNDO NÍVEL

PROBLEMA 1
Inicialmente no quadro está escrito o número 1. Em cada passo, apaga-se o número
do quadro e se escreve outro, que é obtido aplicando alguma das seguintes
operações:
1
• Operação A: Multiplicar o número escrito no quadro por .
2
• Operação B: Trocar o número escrito no quadrado pela diferença entre 1 e
ele.
3
Por exemplo, se no quadro está escrito o número podemos substituí-lo por
8
1 3 3 3 5
⋅ = ou por 1 − = .
2 8 16 8 8
Encontre uma sequência de passos ao fim dos quais o número do quadro seja
2009
.
2 2009

PROBLEMA 2
Seja ABCD um quadrilátero convexo tal que o triângulo ABD é equilátero e o
µ = 90o . Se E é o ponto médio do lado AD,
triângulo BCD é isósceles, com C
µ .
determine a medida do ângulo CED

PROBLEMA 3
Na seguinte soma: 1 + 2 + 3 + 4 + 5 + 6, se suprimirmos os dois primeiros sinais de
“+” obtemos a nova soma 123 + 4 + 5 + 6 = 138. Suprimindo três sinais de “+”
podemos obter 1 + 23 + 456 = 480.
Consideremos agora a soma 1 + 2 + 3 + 4 + 5 + 6 + 7 + 8 + 9 + 10 + 11 + 12 + 13,
na qual serão suprimidos alguns sinais de “+”. Quais são os três menores múltiplos
de 100 que podemos obter desta forma?

EUREKA! N°31, 2010

4
Sociedade Brasileira de Matemática

PROBLEMA 4
Cada casa de um tabuleiro 5 × 5 é pintada de vermelho ou de azul, de tal forma que
seja cumprida a seguinte condição: “Para quaisquer duas filas e duas colunas, das 4
casas que estão em suas interseções, há 4, 2 ou 0 pintadas de vermelho.” De
quantas formas podemos pintar o tabuleiro?

PROBLEMA 5
Um jogo de paciência se inicia com 25 cartas em fila. Algumas estão viradas para
cima, e outras viradas para baixo.
Em cada movimento devemos escolher uma carta que esteja virada para cima,
retirá-la e virar as cartas vizinhas à que foi retirada (se houver).
Ganha-se o jogo de paciência quando conseguimos, repetindo este movimento,
retirar as 25 cartas da mesa.
Se inicialmente há n cartas viradas para cima, encontre todos os valores de n para
os quais se pode ganhar o jogo. Explique a estratégia vencedora,
independentemente da localização inicial das cartas viradas para cima, e justifique
por quê é impossível ganhar para os outros valores de n.
Duas cartas são vizinhas quando uma está imediatamente ao lado de outra, à direita
ou à esquerda.
Por exemplo: a carta marcada com A tem duas
cartas vizinhas e a marcada com B apenas uma.
Depois de retirar uma carta fica um espaço, de B A C D
modo que a marcada com C tem unicamente
uma carta vizinha, e a marcada com D não tem
nenhuma.

EUREKA! N°31, 2010

5
Sociedade Brasileira de Matemática

RESULTADO BRASILEIRO

2009: Nível 1 (até 13 anos)


Nome Cidade - Estado Pontos Prêmio
Luis Fernando Veronese Trivelatto Cascavel - PR 30 Medalha de Ouro
Lucas Carvalho Daher Anápolis - GO 29 Medalha de Prata
Guilherme Renato Martins Unzer São Paulo - SP 27 Medalha de Prata
Elias Brito Oliveira Brasília - DF 26 Medalha de Bronze
Lucas Cardoso Zuccolo São Paulo - SP 25 Medalha de Bronze
Gustavo Lima Lopes Barra de São Fco. - ES 24 Medalha de Bronze
Rafael Rodrigues Rocha de Melo Fortaleza - CE 24 Medalha de Bronze
Igor Albuquerque Araújo Rio de Janeiro - RJ 23 Menção Honrosa
Liara Guinsberg São Paulo - SP 23 Certificado
Fellipe Sebastiam da Silva Paranhos Pereira Rio de Janeiro - RJ 23 Certificado

2009: Nível 2 (até 15 anos)


Nome Cidade - Estado Pontos Prêmio
João Lucas Camelo Sá Fortaleza - CE 50 Medalha de Ouro
César Ilharco Magalhães Barbacena - MG 42 Medalha de Prata
Bruno Silva Mucciaccia Vitória - ES 40 Medalha de Prata
Daniel dos Santos Bossle Porto Alegre - RS 40 Medalha de Bronze
Gustavo Haddad Francisco e Sampaio Braga S.J. dos Campos - SP 38 Medalha de Bronze
Otávio Araújo de Aguiar Fortaleza - CE 38 Medalha de Bronze
Gabriel Militão Vinhas Lopes Fortaleza - CE 35 Medalha de Bronze
Lara Timbó Araújo Fortaleza - CE 33 Menção Honrosa
Artur A. Scussel Fortaleza - CE 32 Menção Honrosa
Bruno Ferri de Moraes São Paulo - SP 31 Menção Honrosa

EUREKA! N°31, 2010

6
Sociedade Brasileira de Matemática

XX OLIMPÍADA DE MATEMÁTICA DO CONE SUL


Enunciados e resultado brasileiro

A XX Olimpíada de Matemática do Cone Sul foi realizada na cidade de


Mar del Plata, Argentina entre os dias 16 e 17 de abril de 2009. A equipe foi
liderada pelos professores Pablo Rodrigo Ganassim, de São Paulo – SP e Alex
Correa Abreu, de Niterói – RJ.

RESULTADOS DA EQUIPE BRASILEIRA

BRA1 Deborah Barbosa Alves Medalha de Prata


BRA2 Gabriel Militão Vinhas Lopes Medalha de Bronze
BRA3 Matheus Barros de Paula Medalha de Prata
BRA4 Matheus Secco Torres da Silva Medalha de Bronze

PROBLEMA 1
Os quatro círculos da figura determinam 10 regiões
limitadas. Nessas regiões são escritos 10 números
inteiros positivos distintos cuja soma é 100, um
número em cada região. A soma dos números
contidos em cada círculo é igual a S (a mesma para
os quatro círculos). Determine o maior e o menor
valor possível de S.

PROBLEMA 2
Um corchete é composto por três segmentos de comprimento 1, que formam dois
ângulos retos como mostra a figura.

É dado um quadrado de lado n dividido em n2 quadradinhos de lado 1 por meio de


retas paralelas aos seus lados. Corchetes são colocados sobre esse quadrado de
modo que cada segmento de um corchete cubra um lado de algum quadradinho.
Dois segmentos de corchete não podem ficar sobrepostos.
Determine todos os valores de n para os quais é possível cobrir os lados dos n2
quadradinhos.

EUREKA! N°31, 2010

7
Sociedade Brasileira de Matemática

PROBLEMA 3
Sejam A, B e C três pontos tais que B é ponto médio do segmento AC e seja P um
ponto tal que ∠PBC = 60º. São construídos o triângulo equilátero PCQ tal que B e
Q estão em semiplanos diferentes em relação a PC, e o triângulo equilátero APR tal
que B e R estão no mesmo semiplano em relação a AP. Seja X o ponto de
interseção das retas BQ e PC; seja Y o ponto de interseção das retas BR e AP.
Demonstre que XY e AC são paralelos.

PROBLEMA 4
Ana e Beto jogam em um tabuleiro de 11 linhas e 9 colunas. Primeiro Ana divide o
tabuleiro em 33 zonas. Cada zona é formada por 3 casas adjacentes alinhadas
vertical ou horizontalmente, como mostra a figura.

Depois, Beto escreve em cada casa um dos números 0, 1, 2, 3, 4, 5, de modo que a


soma dos números de cada zona seja igual a 5. Beto ganha se a soma dos números
escritos em cada uma das 9 colunas do tabuleiro é um número primo; caso
contrário, Ana ganha. Demonstre que Beto tem uma estratégia vencedora.

PROBLEMA 5
Dada uma sequência S de 1001 números reais positivos não necessariamente
distintos, e dado um conjunto A de números inteiros positivos distintos, a operação
permitida é: eleger um k ∈ A (k = 1001), selecionar k números de S, calcular a
média dos k números (média aritmética) e substituir cada um dos k números
selecionados por essa média.
Se A é um conjunto tal que para cada S pode-se conseguir, mediante uma sucessão
de operações permitidas, que os números sejam todos iguais, determine o menor
valor possível do maior elemento de A.

PROBLEMA 6
Pablo tem uma certa quantidade de retângulos cujas áreas somam 3 e cujos lados
são todos menores ou iguais a 1. Demonstre que com esses retângulos é possível
cobrir um quadrado de lado 1 de modo que os lados dos retângulos sejam paralelos
aos lados do quadrado.
Nota: Os retângulos podem estar sobrepostos e podem sair parcialmente do
quadrado.

EUREKA! N°31, 2010

8
Sociedade Brasileira de Matemática

L OLIMPÍADA INTERNACIONAL DE MATEMÁTICA (IMO)


Enunciados e resultado Brasileiro

A L Olimpíada Internacional de Matemática (IMO) foi realizada na cidade


de Bremen, Alemanha entre os dias 14 e 21 de julho de 2009. A equipe foi liderada
pelos professores Carlos Yuzo Shine, de São Paulo – SP e Ralph Costa Teixeira, de
Niterói – RJ.

RESULTADOS DA EQUIPE BRASILEIRA

BRA1 Henrique Ponde de Oliveira Pinto Medalha de Ouro


BRA2 Renan Henrique Finder Medalha de Prata
BRA3 Marcelo Tadeu de Sá Oliveira Sales Medalha de Prata
BRA4 Matheus Secco Torres da Silva Medalha de Prata
BRA5 Marco Antonio Lopes Pedroso Medalha de Bronze
BRA6 Davi Lopes Alves de Medeiros Medalha de Bronze

PRIMEIRO DIA

PROBLEMA 1
Seja n um inteiro positivo e sejam a1 ,...,ak ( k ≥ 2 ) inteiros distintos do conjunto
{1,...,n} tais que n divide ai ( ai +1 − 1) , para i = 1,...,k − 1. Demonstre que n não
divide ak ( a1 − 1) .

PROBLEMA 2
Seja ABC um triângulo cujo circuncentro é O. Sejam P e Q pontos interiores dos
lados CA e AB, respectivamente. Sejam K, L, e M os pontos médios dos segmentos
BP, CQ e PQ, respectivamente, e Γ a circunferência que passa por K, L, e M.
Suponha que a recta PQ é tangente à circunferência Γ . Demonstre que OP = OQ.

PROBLEMA 3
Seja s1 ,s2 ,s3 ,... uma sucessão estritamente crescente de inteiros positivos tal que as
subsucessões
ss1 ,ss2 ,ss3 ,... e ss1 +1 ,ss2 +1 ,ss3 +1 ,...

são ambas progressões aritméticas. Demonstre que a sucessão s1 ,s2 ,s3 ,... também é
uma progressão aritmética.

EUREKA! N°31, 2010

9
Sociedade Brasileira de Matemática

SEGUNDO DIA

PROBLEMA 4
Seja ABC um triângulo com AB = AC. As bissectrizes dos ângulos ∠CAB e
∠ABC intersectam os lados BC e CA em D e E, respectivamente. Seja K o incentro
do triângulo ADC.
Suponha que BEKµ = 45°. Determine todos os possíveis valores de C µ
AB.

PROBLEMA 5
Determine todas as funções f do conjunto dos inteiros positivos no conjunto dos
inteiros positivos tais que, para todos os inteiros positivos a e b, existe um triângulo
não degenerado cujos lados medem,

a, f (b) e f (b + f(a) – 1).

(Um triângulo é não degenerado se os seus vértices não são colineares).

PROBLEMA 6
Sejam a1 ,a2 ,...,an inteiros positivos distintos e M um conjunto de n – 1 inteiros
positivos que não contém o número s = a1 + a2 + ... + an . Um gafanhoto pretende
saltar ao longo da recta real. Ele começa no ponto 0 e dá n saltos para a direita de
comprimentos a1 ,a2 ,...,an , em alguma ordem.
Prove que essa ordem pode ser escolhida de modo que o gafanhoto nunca caia num
ponto de M.

EUREKA! N°31, 2010

10
Sociedade Brasileira de Matemática

XXIV OLIMPÍADA IBEROAMERICANA DE MATEMÁTICA


Enunciados e resultado Brasileiro

A XXIV Olimpíada Iberoamericana de Matemática foi realizada na cidade


de Santiago de Queretaro, México no período de 17 a 27 de setembro de 2009. A
equipe brasileria foi liderada pelos professores Onofre Campos, de Fortaleza – CE
e Luzinalva Miranda de Amorim, de Salvador – BA.

RESULTADOS DA EQUIPE BRASILEIRA


BRA1 Renan Henrique Finder Medalha de Ouro
BRA2 Matheus Secco Torres da Silva Medalha de Ouro
BRA3 Marco Antonio Lopes Pedroso Medalha de Prata
BRA4 Marcelo Tadeu de Sá Oliveira Sales Medalha de Prata

PRIMEIRO DIA

PROBLEMA 1
Seja n um natural maior que 2. Suponhamos que n ilhas estejam localizadas ao
redor de um círculo e que entre cada duas ilhas vizinhas haja duas pontes, como na
figura:

x1 x2

xn x3

xn – 1 xj

Partindo da ilha x1 , de quantas maneiras se podem percorrer as 2n pontes passando


por cada ponte exatamente uma vez?

EUREKA! N°31, 2010

11
Sociedade Brasileira de Matemática

PROBLEMA 2
Para cada inteiro positivo n definimos an = n + m, onde m é o maior inteiro tal que
22 ≤ n 2n . Determinar quais inteiros positivos não aparecem na sequência an .
m

PROBLEMA 3
Sejam C1 e C2 duas circunferências de centros O1 e O2 , com o mesmo raio, que se
intersectam em A e B. Seja P um ponto sobre o arco AB de C2 que está dentro de
C1 . A reta AP intersecta C1 em C, a reta CB intersecta C2 em D e a bissetriz de
∠CAD intersecta C1 em E e C2 em L. Seja F o simétrico do ponto D em relação
ao ponto médio de PE. Demonstrar que existe um ponto X que satisfaz
∠XFL = ∠XDC = 30° e CX = O1O2 .

SEGUNDO DIA

PROBLEMA 4
Seja ABC um triângulo com AB ≠ AC. Sejam I o incentro de ABC e P o outro
ponto de interseção da bissetriz externa do ângulo A com o circuncírculo de ABC.
A reta PI intersecta o circuncírculo de ABC no ponto J. Demonstrar que os
circuncírculos dos triângulos JIB e JIC são tangentes às retas IC e IB,
respectivamente.

PROBLEMA 5
A sequência an está definida por
1
a1 = 1, a2 k = 1 + ak e a2 k +1 =
, para todo inteiro k ≥ 1.
a2 k
Demonstrar que todo número racional positivo aparece exatamente uma vez nesta
sequência.

PROBLEMA 6
Ao redor de uma circunferência marcam-se 6000 pontos, cada um dos quais se
pinta com uma de 10 cores dadas, de modo que entre quaisquer 100 pontos
consecutivos sempre figuram as 10 cores. Achar o menor inteiro k com a seguinte
propriedade: para toda coloração deste tipo existem k pontos consecutivos nos
quais se encontram as 10 cores.

EUREKA! N°31, 2010

12
Sociedade Brasileira de Matemática

PAR OU ÍMPAR? EIS A QUESTÃO


Einstien do Nascimento Jr e Samuel Barbosa Feitosa

Paridade

Quando duas pessoas estão indecisas sobre uma escolha, muitas vezes elas
utilizam uma brincadeira chamada Par ou ímpar para se decidirem. Por trás desse
simples critério, podem se resolver problemas que parecem ser bastante
complicados. Dizemos que um númreo tem paridade par se ele for par, e paridade
ímpar, se ele for ímpar. Observar a paridade de um número é algo bem simples mas
com aplicações fantásticas em problemas de olimpíadas. Vejamos um exemplo:

Paridade como invariante

Vamos começar com um problema bastante famoso que já foi utilizado até em
entrevistas para grandes empresas de computação.

Problema 1. 100 pessoas são postas em uma fila e cada uma delas recebe um
chapéu, que pode ser preto ou branco. Cada pessoa só consegue ver os chapéus das
pessoas que estão a sua frente. É pedido que cada uma delas tente adivinhar a cor
do seu chapéu. Qual o máximo número de acertos que se pode garantir, dado que
as pessoas podem combinar uma estratégia antes de recebê-los.

Solução: Facilmente consegue-se 50 acertos. Podemos dividir as pessoas em pares:


(100,99), (98, 97),...(2, 1) e assim o maior número de cada par falar a cor da pessoa
da frente. Que apenas precisa repeti-lo, para garantir 1 acerto por par. De uma
forma um pouco mais elaborada, se garante 66 acertos. Separando em trios: (100,
99, 98),...(4, 3, 2). O maior número de cada trio pode falar BRANCO caso os dois
da sua frente tenham a mesma cor e PRETO, caso as cores sejam distintas.
Assim, após o maior número falar, o número do meio pode acertar sua cor e em
seguida, o primeiro do trio pode acertar a dele. Curiosamente esse número pode
chegar a 99 acertos utilizando esse poderoso argumento que é a paridade. Notemos
que ninguém sabe a cor do último da fila. Então não importa a estratégia de ordem
das pessoas, nenhuma informação pode ser obtida para esse chapéu. O que não
ocorre com os 99 chapéus restantes. Note ainda que a diferença de conhecimentos
entre a pessoa e a pessoa que encontra atrás dela é apenas o seu chapéu. Então,
basta seguir a estratégia: As cores serão faladas das pessoas de trás para as da
frente. E a última pessoa vai falar BRANCO caso a quantidade de chapéus brancos
a sua frente seja par e PRETO, caso contrário. Como a 99ª. pessoa sabe a paridade
da quantidade de chapéus brancos estritamente à sua frente, e a paridade da

EUREKA! N°31, 2010

13
Sociedade Brasileira de Matemática

quantidade de chapéus brancos à sua frente, incluindo ela mesma, que foi
informada pela 100ª. pessoa, ela acertará o seu chapéu. A 98ª., computando ambas
as informações pode acertar o dela, e assim sucessivamente.

Problema 2. Em cada casa de um tabuleiro de 5 x 5 está escrito 1 ou –1. Em cada


passo troca-se o número de cada uma das 25 casas pelo resultado da multiplicação
dos números de todas as suas casas vizinhas. Inicialmente se tem o tabuleiro da
figura. Mostre como fica o tabuleiro ao final de 2004 passos.

Observação: Duas casas são vizinhas se tiverem um lado em comum.

1 1 –1 1 1
1 1 1 1 1
1 1 1 1 1
1 1 1 1 1
1 1 1 1 1

Dica: Muitas vezes, quando não se tem ideia de como será a solução de uma
questão, pode-se obter várias pistas fazendo alguns casos iniciais do enunciado,
esperando observar algum padrão. Meus números da sorte são 5 e 9. Ao achar um
padrão repetitivo, basta analisar em que caso cairá o número 2004.

Problema 3. Em cada um dos 10 degraus de uma escada existe uma rã. Cada rã
pode, de um pulo, colocar-se em outro degrau, mas quando uma rã faz isso, ao
mesmo tempo, uma outra rã pulará a mesma quantidade de degraus em sentido
contrário: uma sobe e outra desce. Conseguirão as rãs colocar-se todas juntas num
mesmo degrau?

Solução: Uma maneira muito utilizada para atacar problemas onde é dada uma
condição inicial e um conjunto de operações para manipulá-la é tentar procurar o
que não muda, independentemente dos movimentos que utilizamos. Note que se
uma rã vai de um degrau par para um ímpar (muda de paridade), a outra rã que se
movimenta com ela também pulará um número ímpar de degraus, mudando
também a paridade. Caso a primeira não mude, a sua parceira de movimento
também permanecerá num degrau de mesma paridade. UM INVARIANTE:
Paridade da quantidade de rãs em degraus de número par (comprove testando os
movimentos possíveis). Como na posição inicial há 5 rãs nos degraus de posição
par e na posição final há ou dez ou zero rãs nos degraus de posição par, a posição
final NÃO pode ser obtida da posição inicial apenas fazendo essas operações
permitidas.

EUREKA! N°31, 2010

14
Sociedade Brasileira de Matemática

Essa estratégia de invariantes é utilizada principalmente para provar a


impossibilidade de ocorrer algum evento.

Definiremos uma peça príncipe (que não existe no jogo de xadrez) como uma que
só pode andar na horizontal e vertical, uma casa por vez. Um jeito comum de fazer
notações em um tabuleiro de xadrez é nomear as colunas da esquerda para a direita
de a a h e as linhas de baixo para cima de 1 a 8 tomando o referencial da pessoa
que joga com as casas brancas.

Problema 4. Sobre um tabuleiro de xadrez, um príncipe começa do quadrado a1 e


retorna após fazer alguns movimentos. Mostre que o príncipe fez um número par
de movimentos.

Solução: veja que em cada movimento, o príncipe muda para uma casa de cor
oposta. Como a casa a1 é preta, após um número ímpar de movimentos o príncipe
estará numa casa da cor branca. Para ele ter retornado até a casa preta do início, ele
deverá ter feito um número par de movimentos.

Problema 5. Pode um príncipe começar do quadrado a1 de um tabuleiro de xadrez,


ir até o quadrado h8, visitando cada um dos quadrados restantes exatamente uma
vez?

Solução: A resposta é não. Em cada movimento, o príncipe pula para um quadrado


da cor oposta. Como o príncipe tem que fazer 63 movimentos, o último movimento
irá deixá-lo em uma casa da cor oposta a cor de a1.
Entretanto, a1 e h8 têm a mesma cor. Isto é um absurdo.

O último problema nos conduz a um tipo muito importante de demonstração:


prova por absurdo. Suponha que lhe perguntaram se é possível somar cinco
números ímpares e obter o número 100. Após algumas tentativas você começa a
desconfiar que isto não é possúvel. Mas como provar que não é possível? Se
realmente fosse possível somar 5 números ímpares e obter 100 o que aconteceria?
Como a soma de cinco números ímpares é sempre ímpar obteríamos que 100 é um
número ímpar. Mas 100 não é ímpar! Logo não é possível existirem tais 5
números. Para provar que algo não é possível, basta supormos que é possível e
chegarmos a um absurdo.

Problema 6. Uma linha poligonal fechada é composta por 11 segmentos. Pode uma
reta (não contendo um vértice da linha poligonal) intersectar cada um desses
segmentos?

EUREKA! N°31, 2010

15
Sociedade Brasileira de Matemática

Problema 7. Três bolas de gude, A, B, e C estão no chão. Um movimento permitido


é passar uma bola entre as outras duas. É possível, após 25 movimentos, que todas
as bolas estejam nas suas posições originais?
Dica: Que horas são? (Sentidos horário e anti-horário...)

Problema 8. Kátia e seus amigos estão em um círculo. Sabemos qua ambos os


vizinhos de cada criança são do mesmo sexo. Determine o número de garotas
sabendo que existem 5 garotos no círculo.
Dica: Comece a analisar por um vizinho da Kátia.

Problema 9. (Rússia 1970) O rei Luis estava desconfiado de alguns de seus


cortesãos. Ele fez uma lista completa de cada um dos seus cortesãos e disse a cada
um deles para espionar um outro cortesão. O primeiro da lista foi espionar o
cortesão que estava espionando o segundo da lista, o segundo da lista foi espionar o
cortesão que estava espionando o terceiro da lista, e assim sucessivamente; o
penúltimo foi espionar o cortesão que estava espionando o último e o último foi
espionar o cortesão que estava espionando o primeiro. Prove que o rei Luis tinha
um número ímpar de cortesãos.

Solução. Seja n o número de cortesão da lista e suponha que n é par. Coloque-os


sentados ao redor de uma mesa circular de modo que cada um esteja espionando o
seu vizinho da direita.

1
Y X
n 2
2

O cortesão 1 espia o cortesão X que espia o cortesão 2, o cortesão 2 espia o


n
cortesão Z que espia o cortesão 3, e assim sucessivamente até que o cortesão
2
n
espia o cortesão Y que espia o cortesão + 1. Como os números 1, 2, 3,...,n devem
2
EUREKA! N°31, 2010

16
Sociedade Brasileira de Matemática

n
se alternar sobre o círculo, concluímos que o cortesão + 1 é igual ao cortesão 1,
2
ou seja, n = 0. Esse absurdo mostra que n é ímpar.

Problema 10. Um cubo 1 × 1 × 1 está posicionado em um plano quadriculado de modo


que uma de suas faces coincide com um dos quadradinhos do plano. Em cada
movimento podemos “tombar” o cubo por uma de suas arestas, fazendo coincidir
uma face, que tinha essa aresta, com um dos quadradinhos do plano. È possível
fazer o cubo voltar a sua posição inicial após 2005 movimentos?
Dica: Alguém aí joga xadrez?

Paridade e Contagens

Nesta seção, abordaremos duas ideias muito simples:

1. Se contamos os elementos de um conjunto de duas maneiras diferentes, os


valores obtidos devem ter a mesma paridade (Porque são iguais!)

2. Se os elementos de um conjunto podem ser pareados então o conjunto tem uma


quantidade par de elementos.

Problema 11. Em Brasilândia existem apenas 9 casas muito distantes entre si. È
possível que cada casa esteja ligada a exatamente 7 outras casas através de
estradas?
Solução: Não é possível. Some a quantidade de estradas que saem de cada casa.
Bem, facilmente obtemos 9 × 7 estradas. Como cada estrada liga duas cidades, a
contagem que fizemos contou cada estrada duas vezes. Logo o número obtido teria
que ser par.
Você deve ter ficado com uma pulga atrás da orelha. Será que cada casa ligada a
exatamente 7 outras foi realmente crucial? É possível revolvermos o problema
anterior com um eneunciado mais geral:

Problema 12. Prove que numa festa com n pessoas, o número de pessoas que
conhecem um número ímpar de outras pessoas na festa á par.

Solução: Numere as pessoas de 1 até n e denote por d i o número de amigos da


pessoa i. Imagine que existe um fio entre duas pessoas que se conhecem. Se E
denota a quantidade de fios, temos
d1 + d2 + ... + d n = 2 E,

EUREKA! N°31, 2010

17
Sociedade Brasileira de Matemática

pois cada fio é contado duas vezes, um para cada ponta. Como o lado direito é par,
no lado esquerdo devemos ter uma quantidade par de números ímpares.

Problema 13. (Olimpíada de Maio 2000) O conjunto {1, 2, 3, 4} pode ser dividido
em dois subconjuntos A = {1,4} e B = {3, 2} sem elementos comuns e tais que a
soma dos elementos de A seja igual à soma dos elementos de B. Essa divisão é
impossível para o conjunto {1, 2,3, 4,5} e também para o conjunto {1, 2, 3, 4, 5, 6} .
Determine todos os valores de n para os quais o conjunto dos primeiros n números
naturais pode ser dividido em dois subconjuntos sem elementos comuns tais que a
soma dos elementos de cada subconjunto seja a mesma.

Solução. Como a soma dos elementos de A deve ser igual à soma dos elementos de
B, a soma dos números do conjunto {1, 2,3,...,n} deve ser o dobro da soma dos
elementos de A, ou seja, deve ser um número par. Você já deve saber que

n ( n + 1)
1 + 2 + 3 + ... + n = .
2
Você não sabia disso? Não fique aí parado! Tente descobrir porque isso é verdade!
n ( n + 1)
Veja que é par se n ( n + 1) é múltiplo de 4. Como estamos interessados no
2
resto na divisão por 4 de algum número, talvez seja interessante procurar quais os
possíveis restos de n na divisão por 4. Podemos escrever n na forma n = 4q + r
onde r = 0,1, 2 ou 3. Mãos à obra!
n ( n + 1)
1. Se n = 4q então = 2q ( 4q + 1) é par.
2
n ( n + 1)
2. Se n = 4q + 1 então = ( 2q + 1)( 4q + 1) é ímpar.
2
n ( n + 1)
3. Se n = 4q + 2 então = ( 2q + 1)( 4q + 1) é ímpar.
2
n ( n + 1)
4. Se n = 4q + 3 então = ( 2q + 2 )( 4q + 3) é par.
2
Podemos concluir que n deve ser da forma 4q ou 4q + 3. Acabou? Não! Precisamos
construir EXEMPLOS para cada uma dessas possibilidades mostrando que
realmente esses valores satisfazem as condições do problema.
Para n = 4q, considere os conjuntos

EUREKA! N°31, 2010

18
Sociedade Brasileira de Matemática

A = {(1, 4 ) ,( 5,8 ) ,( 9 ,12 ) ,...,( 4q − 3, 4q )} .


B = {( 2,3) ,( 6,7 ) ,(10 ,11) ,...,( 4q − 2 ,4q − 1)} .
Para n = 4q + 3, considere os conjuntos
A = {( 4,7 ) ,( 8,11) ,(12,15 ) ,...,( 4q, 4q + 3)} ∪ {(1, 2 )} .
B = {( 5,6 ) ,( 9 ,10 ) ,(13,14 ) ,...,( 4q + 1, 4q + 2 )} ∪ {( 3)} .
Note que os conjuntos foram divididos em parêntesis. Cada parêntese de A possui
correspondente em B com a mesma soma, facilitando a construção de um exemplo
generalizado.

Problema 14. Podemos desenhar uma linha poligonal fechada feita por 9 segmentos
de reta, cada um deles intersectando exatamente outro segmento?

Solução. Se tal construção é possível, então todos os segmentos podem ser


agrupados em pares de segmentos intersectantes.
Mas o número de segmentos é ímpar! Absurdo!

Os próximos dois problemas tratam de dominós. Um dominó consiste de um


tabuleiro 1 x 2 com pontos em cada casinha.
A quantidade de pontos varia de 0 até 6. Então, o número total de dominós
distintos é 28.

Problema 15. Todos os dominós são arranjados em uma cadeia de duas pontas (a
quantidade de pontos na extremidade de dois dominós consecutivos é a mesma). Se
em uma ponta existe o número 5, qual é o número de outra ponta?

Problema 16. Em um conjunto de dominós, descartamos todos aqueles que possuem


pelo menos uma casinha vazia. É possível arranjarmos todos os restantes em uma
cadeia?

Problema 17. (Eslovênia 1992) Prove que para quaisquer inteiros positivos
a1 ,a2 ,...,an o número:
a1 − a2 + a2 − a3 + ... + an − a1
é par.
Observação: x − y é chamado de valor absoluto da diferença entre x e y e denota o
máximo entre x – y e y – x. Na reta real, ele representa a distância entre os números
x e y.

EUREKA! N°31, 2010

19
Sociedade Brasileira de Matemática

Solução: Perceba que x − y = ± x ± y para alguma escolha de sinais. Então a soma


total é
± a1 ± a2 ± a2 ± a3 ± ... ± an ± a1 .
Como cada número ai aparece duas vezes, basta mostrarmos que cada uma das
expressões ± a1 ± ai é par para qualquer escolha de sinais. Vejamos os casos:

1. ± ai ± ai = + ai + ai = 2ai é par.
2. ± ai ± ai = − ai + ai = 0 é par.
3. ± ai ± ai = + ai − ai = 0 é par.
4. ± ai ± ai = − ai − ai = −2ai é par.

1.3 Miscelânia

Problema 18. Podemos trocar uma nota de 25 reais usando dez notas que podem
assumir os valores 1, 3, 5?

Solução. Não. Como a soma de um número par de números ímpares é par, a soma
dos valores dessas 10 notas só pode ser um número par. Mas 25 é ímpar.

Problema 19. Peter comprou um caderno com 96 folhas, e numerou com os


números de 1 até 192. Victor rasgou 25 folhas consecutivas do caderno, e
adicionou os 50 números. Victor pode ter obtido o número 1990 como resultado da
soma?

1 1 1 1 1 1
Problema 20. Prove que a igualdade + + + + + = 1 não admite soluções
a b c d e f
com todos os números sendo ímpares.
Dica: Faça o produto dos denominadores.

Problema 21. O produto de 21 inteiros é igual a 1. Mostre que sua soma não pode
ser zero.
Dica: compare as quantidades de números positivos e negativos.

Problema 22. Três gafanhotos estão brincando ao longo da uma linha. Na sua vez,
cada gafanhoto pode pular sobre um outro gafanhoto, mas não sobre os outros dois.
Eles podem retornar para suas posições iniciais após 1991 movimentos?

EUREKA! N°31, 2010

20
Sociedade Brasileira de Matemática

Solução. Sejam A, B, C os três gafanhotos. Estaremos interessados apenas na


ordem em que os gafanhotos se dispõem ao longo da reta, digamos que
inicialmente eles estão na ordem (A, B, C). Podemos fazer os seguintes
movimentos:
1 2 3 4
( A,B,C ) → ( B,A,C ) → ( B,C,A) → ( C,B,A) → ...

Em cada passo, disponha as letras A, B e C em um círculo (como mostra a figura) e


leia a palavra ABC. Percebeu alguma coisa? Antes de efetuarmos nosso primeiro
movimetno, a leitura estava no sentido “horário” e logo em seguida passou para o
sentido “anti-horário”. Como cada movimento alternar os sentidos, após 1991
movimentos estaremos em um sentido diferente do original. Logo, não é possível
retornarmos para a posição original.

A B

C B C A

Observação: Compare com o problema 7.

Problema 23. Os números de 1 até 10 são escritos em uma linha. Podemos colocar
os sinais + e – entre eles de modo que o resultado da expressão resultante seja 0?

Solução: Não é possível. Perceba que quando escolhemos um número para


trocarmos de sinal, por exemplo, de + para –, a soma total varia o dobro do número
escolhido, ou seja, a paridade da soma não muda. Basta ver agora que
1 + 2 + ... + 10 = 55 não tem a mesma paridade que 0. Um INVARIANTE é a
paridade da soma.

Problema 24. Um gafanhoto pula ao longo de uma linha. No seu primeiro pulo, ele
anda 1cm, no segundo 2cm, e assim sucessivamente. Ele pode pular para a
esquerda ou para a direita. Mostre que após 1985 pulos, o gafanhoto não pode
retornar ao ponto em que começou.
Dica: Perceba que você pode associar aos pulos do gafanhoto um número com

EUREKA! N°31, 2010

21
Sociedade Brasileira de Matemática

sinal (+ se o pulo é para a esquerda e – se é para a direita). Agora use o problema


anterior.

Problema 25. Os números 1, 2,...,1984, 1985 são escritos em um tabuleiro. A


operação permitida é apagar dois números e colocar sua diferença positiva. Após
algumas operações, resta apenas um único número no tabuleiro. Pode este número
ser 0?

Problema 26. Pode um tabuleiro 8 × 8 ser coberto com dominós 1 × 2 de modo que
somente os quadrados a1 e h8 não sejam cobertos?

Solução. Não é possível. Pinte o tabuleiro de preto e branco da maneira usual. Cada
dominó cobre exatamente um quadrado preto e outro branco (Invariante), portanto,
a quantidade de quadrados pretos cobertos é igual à quantidade de quadrados
brancos cobertos. Como a1 e h8 têm a mesma cor, sobrariam 30 quadrados de uma
cor e 32 de outra para serem cobertos. Absurdo!

Problema 27. 45 pontos são escolhidos sobre a reta AB, todos fora do segmento de
reta AB. Prove que a soma das distâncias desses pontos ao ponto A não pode ser
igual à soma das distâncias ao ponto B.

Solução. Sejam A e B dispostos, sem perda de generalidade como na figura abaixo.


Tomemos um ponto X.

A B X

X pode estar à direita de B ou à esquerda de A. Ou ocorre: AX + AB = BX ou


BX + AB = AX . Assim, se estivéssemos somando em x as distâncias dos 45 pontos
para A e em y para B, estaríamos na verdade, só somando uma diferença de AB em
x ou em y. Como 45 é ímpar, não podemos “distribuir” uma igual quantidade de
AB´ s para o grupo de A e o de B. Assim, segue que não é possível.

Problema 28. Um número de 17 dígitos é somado com o seu reverso (um número
com os mesmo dígitos mas escritos na ordem inversa). Mostre que sua soma
contém pelo menos um dígito par.

Problema 29. Existem 100 soldados em um quartel. Toda noite, três deles ficam de
guarda. Após um certo período de tempo, é possível que cada soldado tenha ficado
de guarda exatamente uma vez com cada outro soldado?

EUREKA! N°31, 2010

22
Sociedade Brasileira de Matemática

Solução: Suponha, por absurdo, que seja possível. Tomemos o Soldado Ryan, ele
possui 99 companheiros. Suponha que ele em particular tenha conseguido ficar
exatamente uma vez de pernoite com cada um dos outros. A cada dia, Ryan
formava 2 duplas diferentes, que não poderiam se repetir nos dias posteriores. Caso
Ryan tivesse pernoitado x vezes, a quantidade de duplas que ele teria formado seria
2x, que por hipótese, deve ser igual a 99. Chegando à conclusão que 99 é par.
Absurdo!

Problema 30. 25 garotos e 25 garotas estão sentados ao redor de uma mesa. Prove
que é sempre possível encontrar uma pessoa tal que ambos os seus vizinhos são
garotas.

Solução: Suponha, por absurdo, que não necessariamente haja uma pessoa que
possua duas garotas como vizinhas. Denotemos h para garoto e m para garota.
Cada pessoa ou possui como vizinho 2h ou h+m. Somando todas as 50
possibilidades, devemos estar contando cada pessoa duas vezes (já que essa é
vizinho de duas pessoas). Assim: x ( 2h ) + y ( h + m ) = 50h + 50m onde x é o
número de pessoas que têm 2 garotos como vizinhos e y é o número de pessoas que
têm um garoto e uma garota. Notemos ainda que x + y = 50. Obtemos
xh = ( 50 − y ) m assim xh = xm. Mas x garotos só serão iguais a x garotas, se x for
nulo. Assim, todas as pessoas têm um garoto e uma garota como vizinhos.
Pintemos as 50 posições do círculo apenas de branco e preto. E analisemos apenas
as pretas. Todas as pretas terão que ter vizinhos sendo um garoto e uma garota.
Logo, as casas brancas serão alternadas: garoto, garota, garoto... Absurdo. Pois
com 25 casas brancas, na última e na primeira brancas haverá 2 garotos. Absurdo!
Segue o resultado.

Problema 31. (Ucrânia 1997) Um tabuleiro é colorido de branco e preto da maneira


usual, e cada casa contém um inteiro. Sabemos que a soma dos números em cada
coluna e a soma dos números em cada linha é par. Mostre que a soma dos números
nas casas pretas é par.

Solução. Suponha sem perda de generalidade que o quadrado do canto esquerdo


superior é preto. A partir desse quadrado, numere as colunas da esquerda para a
direita e as linhas de cima para baixo. Some os números das colunas em posições
ímpares e os números das linhas em posições pares. Perceba que cada quadrado
preto do tabuleiro é contado apenas uma vez nessa soma enquanto que os
quadrados brancos das linhas e colunas mencionadas são contados duas vezes.
Logo, esse soma tem a mesma paridade que a soma de todos os números escritos

EUREKA! N°31, 2010

23
Sociedade Brasileira de Matemática

nos quadrados pretos. Como a soma de quaisquer linhas e colunas é par, a soma
dos números nos quadrados pretos é par.

Problema 32. Considere um tabuleiro 1998 × 2002 pintado alternadamente de preto


e branco da maneira usual. Em cada casa do tabuleiro, escrevemos 0 ou 1, de modo
que a quantidade de 1´s em cada linha e em cada coluna do tabuleiro é ímpar.
Prove que a quantidade de 1´s escritos nas casa brancas é par.
Dica: Tente imitar a solução anterior.

Problema 33. (Austrália 2007) Em cada casa de um tabuleiro 2007 × 2007


escrevemos um número inteiro ímpar. Sejam Z i a soma dos números na i-ésima
linha e S j a soma dos números na j-ésima coluna, para 1 ≤ i, j ≤ 2007. Além disso,
sejam A = Z i ⋅ Z 2 ...Z 2007 e B = S1 ⋅ S 2 ...S 2007 . Mostre que A + B não pode ser igual a
zero.

Problema 34. (China 1986) É possível arranjar os números 1, 1, 2, 2, 3, 3,...,1986,


1986 em fila de modo que entre quaisquer dois i´s hajam (i – 1) números?

Solução: Vamos tentar fazer alguns casos pequenos. É fácil ver que não
conseguimos fazer o que o enunciado pede com os números 1, 1, 2, 2 mas com os
números 1, 1, 2, 2, 3, 3, 4, 4 temos um exemplo:

1º. 2º. 3º. 4º. 5º. 6º. 7º. 8º.


a3 a4 a2 b3 b2 b4 a1 b1
3 4 2 3 2 4 1 1

Contados da squerda para a direita, denotemos por ai e bi as posições do primeiro


e segundo número i, respectivamente. No nosso exemplo, a2 = 3 e b2 = 5.
Como existem i – 1 números entre dois números i´s, devemos ter bi − ai = i. Se é
possível escrever os números 1, 1, 2, 2, ..., n, n em linha como no enunciado,

EUREKA! N°31, 2010

24
Sociedade Brasileira de Matemática

obtemos:
( a1 + a2 + ...an ) + ( b1 + b2 + ...bn ) = 1 + 2 + ... + 2n = n ( 2n + 1)
n ( n + 1)
( b1 − a1 ) + ( b2 − a2 ) + ...( bn − an ) = 1 + 2 + ...+ n = .
2
Somando as duas linhas,
n ( 5n + 3 )
2 ( b1 + b2 + ...bn ) =
2
n ( 5n + 3)
Como o lado esquerdo é sempre par, a fração deve ser um inteiro par.
2
Isso já restringe os possíveis valores de n.
Para n = 1986,
n ( 5n + 3 )
= 9863469
2
é ímpar e conseqüentemente não é possível dispormos esses números em linha.
Uma pergunta natural que você deve tentar responder é: para quais n tal
distribuição é possível?

Problema 35. É possível arranjar os números de 1 até 9 em uma sequência, de modo


que exista uma quantidade ímpar de números entre 1 e 2, entre 2 e 3,..., e entre 8 e
9?

Problema 36. (Rússia 1984) O número de todos os inteiros positivos de 64 dígitos


sem zeros em sua representação e que são divisíveis por 101 é par ou ímpar?

Solução: Precisamos bolar alguma maneira de agrupar os números em pares. Seja


A = 11
12...3
110 repetições do número 1.
64 vezes

Como 1111 é múltiplo de 101 é fácil ver que A é múltiplo de 101. Para todo
número de 64 dígitos a = a1a2 ...a63 a64 , sem zeros em sua representação decimal,
considere o seu conjugado b = b1b2 ...b63b64 = (10 − a1 )(10 − a2 ) ...(10 − a64 ). Nenhum
dígito de a é igual a zero, portanto, cada número 10 − ai pertence ao conjunto
{1,2,..,9} . Da equação a + b = A obtemos que a é divisível por 101 se e somente se
b é divisível por 101 (lembre-se que A é múltiplo de 101). Como o único número
que é igual ao seu conjugado é o número 55 123...55 (que é múltiplo de 101) e os
64 vezes
demais números que satisfazem o enunciado podem ser pareados, concluímos que a

EUREKA! N°31, 2010

25
Sociedade Brasileira de Matemática

quantidade procurada é ímpar.

Problema 37. (Putnam 1997) Seja Bn a quantidade de n – uplas ordenadas de


inteiros positivos ( a1 ,a2 ,...,an ) tais que
1 1 1
+ + ... + = 1
a1 a2 an
B10 é par ou ímpar?

Solução: Uma ideia natural é tentar agrupar as soluções em pares. Qualquer


solução com a1 ≠ a2 pode ser pareada com a outra solução obtida pela troca de
posição entre a1 e a2 . Logo, B10 tem a mesma paridade que o número de soluções
com a1 = a2 . Das soluções com a1 = a2 , podemos parear aquelas que tem a3 ≠ a4
da mesma maneira. Repetindo esse argumento com ( a5 ,a6 ) ,( a7 ,a8 ) e ( a9 ,a10 ) ,
concluímos que a paridade de B10 é a mesma do número de soluções com
a5 = a6 ,a7 = a8 e a9 = a10 , ou seja, das soluções de:
2 2 2 2 2
+ + + + = 1.
a1 a3 a5 a7 a9
Como anteriormente, podemos nos restrigir à quantidade de soluções com a1 = a3 e
a5 = a7 , que é igual ao número de soluções da equação:
4 4 2
+ + = 1.
a1 a5 a9
Mais uma vez, podemos nos restringir à quantidade de soluções com a1 = a5 , que é
igual ao número de soluções da equação:
8 2
+ = 1.
a1 a9
Agora ficou fácil! Basta contar explicitamente o número de soluções da equação
anterior. Como fazer isso? Bem, ela pode ser fatorada como:
( a1 − 8 )( a9 − 2 ) = 16
que admite 5 soluções correspondendo às fatorações de 16 como 2i × 24−i para
i = 0,1,2 ,3,4. Então B10 é ímpar.

Problema 38: Prove que numa festa com 2n pessoas existem duas com um número
par de amigos em comum.

EUREKA! N°31, 2010

26
Sociedade Brasileira de Matemática

Solução: Suponha que quaisquer duas pessoas tenham um número ímpar de amigos
em comum e seja A um dos participantes da festa. Seja M = { F1 ,F2 ,...,Fk } o
conjunto dos amigos de A. Considere uma nova festa restrita apenas ao conjunto M.
Como cada Fi tem um número ímpar de amigos em comum com A, na nova festa,
cada Fi possui um número ímpar de amigos. Pelo problema 12, k deve ser par. O
mesmo argumento vale para qualquer pessoa na festa e conseqüentemente todos
têm um número par de amigos. Peça para cada um dos amigos de A fazerem uma
lista de seus amigos diferentes de A. A soma da quantidade de nomes listados é par,
pois é uma soma de uma quantidade par (igual a k) de números ímpares (cada Fi
possui um número ímpar de amigos diferentes de A). Agora comparemos o número
de aparições de cada uma das 2n − 1 pessoas diferentes de A nessas listas. Se cada
uma delas aparecer em um número ímpar de listas, a soma total de todos os nomes
em todas as listas seria ímpar. (Lembre-se que a soma de uma quantidade ímpar de
números ímpares é ímpar!). Mas isso é uma contradição. Logo, existe uma pessoa
diferente de A que aparece em um número par de listas, e portanto tem um número
par de amigos em comum com A.

Problema 39. Alex desenhou uma coleção de K retas no plano em posição geral
(quaisquer duas retas se intersectam em um ponto e quaisquer três definem um
triângulo não degenerado). Para quais valores de K é sempre possível (não importa
como as retas são desenhadas) colocar um elemento do conjunto {1, 2 ,...,K − 1} em
cada ponto de interseção das retas de modo que em toda reta não existam números
iguais.

Problema 40. (Rússia) Em cada planeta de um sistema solar existe um astrônomo


observando o planeta mais próximo. As distâncias entre os planetas são distintas
duas a duas. Demonstre que se a quantidade de planetas é ímpar, então existe pelo
menos um planeta que não é observado.
Dica: Procure as cadeias de planetas que um olha para o outro que olha para o
outro com mais de 2 planetas.

REFERÊNCIAS
[1] D. Fomin, S. Genkin e I. Itenberg, Mathematical Circles, MAS (1996).
[2] C. Augusto, S. Feitosa, B. Holanda e Y. Lima, treinamento Cone Sul 2007, Fortaleza, Realce
(2007).
[3] P. J. Taylor, Tournament of the Towns 1980 to 1984, Australian Mathematical Trust (1993).
[4] D. Fomin e A. Kirichenko, Leningrand Mathematical Olympiadas 1987-1991, MathPro Press
(1994).
[5] E. Wagner, Paridade, Eureka! No. 2, pp. 32-38, (1998).

EUREKA! N°31, 2010

27
Sociedade Brasileira de Matemática

GEOMETRIA DO TRIÂNGULO: FATOS E PROBLEMAS


Carlos Yuzo Shine

1. O Teorema de Miquel

Começamos com o teorema em si, que é um dos vários pequenos milagres dos
chamados quadriláteros completos (veja um pouco mais desses “milagres” nos
exercícios!), que são os quadriláteros conhecidos unidos com as retas que contêm
os lados. Isto é, um quadrilátero completo é a união de quatro retas em vez de
quatro segmentos.

Teorema de Miquel. Sejam a, b, c, d quatro retas coplanares, de modo que não há


duas paralelas nem três concorrentes. Os circuncírculos dos quatro triângulos
determinados pelas quatro retas passam por um mesmo ponto, denominado ponto
de Miquel das quatro retas.

Demonstração:
Seja M a intersecção dos circuncírculos de CEF e BDF na figura acima. Então
∠MEA = ∠MEC = 180° − ∠MFC = ∠BFM = ∠BDM = 180° − ∠ADM , de modo
que ∠MEA + ∠ADM = 180° e, portanto, MDAE é inscritível. Isso quer dizer que M
pertence ao circuncírculo de ADE. Analogamente, prova-se que M pertence ao

EUREKA! N°31, 2010

28
Sociedade Brasileira de Matemática

circuncírculo de ABC.
Vamos resolver, a título de exemplo, o problema 6 da olimpíada norteamericana de
2006.

Exemplo 1.1.
(USAMO 2006, Problema 6) Seja ABCD um quadrilátero e E e F os pontos sobre
os lados AD e BC, respectivamente, tal que AE / ED = BF / FC. A semirreta FE
corta as semirretas BA e CD em S e T, respectivamente. Prove que os
circuncírculos dos triângulos SAE, SBF, TCF e TDE passam por um mesmo ponto.

Resolução:
Ao fazer a figura, você provavelmente vai notar uma certa semelhança com a
figura anterior.

Queremos provar que os pontos de Miquel de ADTS e BCTS coincidem! Isso não é
difícil, na verdade: seja M a intersecção dos circuncírculos de SAE e SBF.
Mostraremos que os circuncírculos de TED e TFC também passam por M.
Um arrastão e uma semelhança dão conta do recado: primeiro, note que
∠AME = ∠ASE = ∠BSF = ∠BMF e ∠MEA = ∠MSA = ∠MSB = ∠MFB. Então os
triângulos AME e BMF são semelhantes, e da igualdade AE / ED = BF / FC os
triângulos MAD e MBC são semelhantes também. Uma rápida verificação mostra
que MAB e MDC também são semelhantes: de fato (pois como MAD e MBC são

EUREKA! N°31, 2010

29
Sociedade Brasileira de Matemática

AM DM
semelhantes então ∠DMA = ∠CMB ) e = (novamente da semelhança).
BM CM
Você pode imaginar que o triângulo MAD “gira” em torno de M e, após um “acerto
de escala”, é transformado no triângulo MBC. Isso é uma transformação
geométrica conhecida como roto-homotetia de centro M. Assim, A é levado em B e
D é elevado em C. Note que a semelhança obtida anteriormente envolve o centro
de roto-homotetia M, os pontos e suas imagens na transformação. Isso na verdade
sempre acontece (é uma das semelhanças automáticas).
Agora podemos terminar o problema: da semelhança entre MAB e MDC, os
ângulos externos ∠MDT e ∠MAS são congruentes. Como M pertence ao
circuncírculo de SAE, ∠MAS = ∠MES = ∠MET , ou seja, ∠MDT = ∠MET , o que
significa que MEDT é cíclico e, portanto, M pertence ao circuncírculo de TED.
Utilizando outra semelhança automática, entre MEF e MDC (pois E é levado em
F!), prova-se que M pertence também ao circuncírculo de TFC.
Note que se U é a interseção de AB e CD, então pelo teorema de Miquel M também
pertence ao circuncírculo de STU. Então na vaerdade cinco círculos passam pelo
ponto M!
A seguinte versão do teorema de Miquel também é útil:

Teorema de Miquel para triângulos. Seja ABC em triângulo e D, E, F pontos sobre


as retas BC, CA, AB, respectivamente. Então os circuncírculos de AEF, BFD e
CDE têm um ponto em comum. Esse ponto também é chamado de ponto de
Miquel.

Demonstração:
Seja M a segunda interseção dos circuncírculos de AEF e BFD. Então
∠CDM = ∠BFM = ∠AEM = 180° − ∠CEM .

EUREKA! N°31, 2010

30
Sociedade Brasileira de Matemática

Exercícios:
01. Demonstre o teorema de Miquel para quadriláteros utilizando o teorema de
Miquel para triângulos.

02. Seja ABCDE um pentágono convexo e F, G, H, I, J as interseções dos


prolongamentos de EA, AB, AB, BC, CD, DE e DE, EA respectivamente. Prove que
as segundas interseções dos circuncírculos de ABF, BCG, BCG, CDH, DEI, DEI,
EAJ, e EAJ, ABF pertencem a uma mesma circunferência.

03. Considere um quadrilátero completo. Seja M o seu ponto de Miquel. Prove que:
(a) os circuncentros dos quatro triângulos determinados pelo quadrilátero e M estão
sobre uma mesma circunferência.
(b) as projeções ortogonais de M sobre as quatro retas do quadrilátero pertencem a
uma mesma reta r; além disso, M é o único ponto do plano com essa propriedade.
(c) os ortocentros dos quatro triângulos pertencem a uma mesma reta s.
(d) as retas r e s são paralelas, e a distância de M e r é metade da distância de M a s.

04. Seja ABCD um quadrilátero convexo e X e Y as interseções dos lados opostos


AD e BC e AB e CD, respectivamente. Prove que os pontos médios de AC, BD e XY
são colineares.

Observação: a reta que passa pelos três pontos é a reta de Gauss do quadrilátero
completo.

2. Conjugados isogonais

A ideia de conjugado é fazer uma associação entre objetos. Objetos conjugados


supostamente têm propriedades semelhantes. Isso é bastante comum em equações:
se um número é raiz, então o conjugado também é raiz. Em geometria, também
existe a ideia de conjugado. De fato, dado um triângulo, cada ponto tem um
conjugado isogonal e um conjugado isotômico. Aqui, trataremos somente de
conjugados isogonais.

Definição 2.1. Dado um triângulo ABC, o conjugado isogonal em relação a ABC de


um ponto T do plano de ABC é obtido refletindo as retas TA, TB e TC em relação às
bissetrizes internas de ABC que passam por A, B, e C, respectivamente. As retas
resultados são concorrentes no isogonal T −1 de T.
A seguir, as linhas pontilhadas são as bissetrizes, e as cevianas cinzas são as
reflexões das cevianas pretas.

EUREKA! N°31, 2010

31
Sociedade Brasileira de Matemática

O fato de que as retas isogonais são concorrentes é extremamente importante, tanto


que será enunciado novamente.

Teorema fundamental dos conjugados isogonais. Dados um triângulo e três retas


que passam pelos respectivos vértices e concorrem em um ponto P, as retas
isogonais a elas, obtidas através da reflexão em relação à bissetriz interna
correspondente, são concorrentes no conjugado isogonal P −1 de P.

Demonstração
Por que as cevianas cinzas são concorrentes? Isso decorre de duas aplicações do
teorema de Ceva trigonométrico: primeiro com as cevianas concorrentes em T e
depois, com as cevianas concorrentes em T −1 , que formam os mesmos ângulos que
as outras cevianas, porém no sentido contrário.
Na verdade, pode ocorrer de as três cevianas serem paralelas. Isso ocorre se, e
somente se, T está sobre o circuncírculo de ABC; nesse caso, pensamos
projetivamente, ou seja, o conjugado isogonal é um ponto do infinito.

2.1 para que servem isogonais?

O que é mais útil em conjugados isogonais é simplesmente que as cevianas são


reflexões umas das outras em relação às bissetrizes, e isso costuma levar a algumas
igualdades entre ângulos um pouco mais difíceis de obter ou mesmo de se imaginar
com contas.

EUREKA! N°31, 2010

32
Sociedade Brasileira de Matemática

Exemplos 2.1
No triângulo ABC, P e Q são pontos no interior de ABC tais que
∠CBP = ∠PBQ = ∠QBA = ∠ABC / 3 e ∠BCP = ∠PCQ = ∠QCA = ∠ACB / 3.
Sejam D e E as projeções ortogonais de P sobre AB e AC, respectivamente. Prove
que AQ é perpendicular a DE.

Resolução
Seja θ = ∠PAD. Então ∠APD = 90° − θ e, como ∠ADP e ∠AEP são retos, o
quadrilátero ADPE é inscritível. Logo ∠AED = ∠APD = 90° − θ.

Olhando a figura, note que basta provarmos que ∠QAC = θ. Aí é que entram os
conjugados isogonais.
Como ∠PBC = ∠QBA e ∠BCP = ∠QCA, os pares de retas BP;BQ e CP;CQ
são simétricos entre si em relação às bissetrizes de ∠ABC e ∠ACB,
respectivamente. Ou seja, P e Q são conjugados isogonais e, portanto, ∠PAB e
∠QAC também são iguais. Logo ∠QAC = θ e o ângulo entre as retas AQ e DE é
180° − θ − ( 90 − θ ) = 90°.
Note que para provar o resultado na conta, bastaria repetir a demonstração do
teorema fundamental dos conjugados isogonais. Mas o mais interessante é que,
sabendo da existência dos conjugados isogonais, é natural pensar nessa solução.
Em contraste, fazer a conta sem pensar em conjugados isogonais não parece ser tão
natural. Então dá para pensar que os conjugados isogonais nos economizaram não
só fazer a conta, mas mostraram onde fazer as contas relevantes.

EUREKA! N°31, 2010

33
Sociedade Brasileira de Matemática

2.2. Conjugados isogonais dos pontos notáveis

Você já deve estar familiarizado com os pontos notáveis do triângulo: o baricentro


(encontro das medianas), o incentro (encontro das bissetrizes internas), o ortocentro
(encontro das alturas) e o circuncentro (encontro das mediatrizes). Quais são os
conjugados isogonais desses pontos? Vamos aproveitar e conhecer mais um ponto
notável (mas não tão conhecido).
Vamos fazer isso em ordem de dificuldade.

Incentro
As reflexões coincidem com as próprias bissetrizes. Logo o conjugado isogonal do
incentro, que é o encontro das bissetrizes internas, é ele mesmo.

O mesmo vale para os ex-incentros (encontros de duas bissetrizes externas e uma


bissetriz interna e centros dos ex-incírculos, que são tangentes externamente aos
lados ou seus prolongamentos). Pense sobre o assunto!

Ortocentro e circuncentro
A figura a seguir deve convencê-lo de que o ortocentro e o circuncentro são
conjugados isogonais.

Baricentro
Os isogonais das medianas são as simedianas (SImétrico + MEDIANA). O ponto
de encontro das simedianas é o ponto de Lemoine, também conhecido como ponto
simediano. O ponto de Lemoine é costumeiramente denotado por K.
Primeiro, vamos aprender a traçá-las de modo mais prático.

EUREKA! N°31, 2010

34
Sociedade Brasileira de Matemática

Lema: Seja D a interseção das retas tangentes ao circuncírculo do triângulo ABC


por B e C. Então a reta AD contém a simediana que passa por A.

Demonstração

Construa o paralelogramo ABEC. Então AD contém a mediana AM. Afirmamos


que D e E são conjugados isogonais. De fato, ∠BCE = ∠B e o ângulo entre AC e
CD, pela tangência, é igual a ∠B.
Assim, as retas CD e CE são conjugadas isogonais. Analogamente, BD e BE
também são, e o resultado segue do teorema fundamental dos conjugados
isogonais.

Exercícios

05. Sejam P e Q pontos no interior do ângulo ∠BAC tais que BP = CP,BQ = CQ


e ∠ABP + ∠AQC = 180°. Prove que ∠BAP = ∠CAQ.

06. As retas obtidas através das reflexões da diagonal BD do quadrilátero ABCD


em relação às bissetrizes de ∠B e ∠D passam pelo ponto médio de AC. Prove que
as reflexões da diagonal AC do quadrilátero ABCD em relação às bissetrizes de

EUREKA! N°31, 2010

35
Sociedade Brasileira de Matemática

∠A e ∠C passam pelo ponto médio de BD.

07. (Prova de Seleção EUA, 2008) Seja ABC um triângulo e G o seu baricentro. O
ponto P varia sobre o segmento BC. Os pontos Q e R pertencem aos lados AC e AB
respectivamente, e são tais que PQ é paralelo a AB e PR é paralelo a AC. Prove
que, ao variar P sobre BC, o circuncírculo de AQR passa por um ponto fixado X tal
que ∠BAG = ∠CAX .

08. (IMO 2004, Problema 5) Num quadrilátero convexo ABCD a diagonal BD não
é bissetriz do ângulo ∠ABC nem do ângulo ∠CDA. Um ponto P no interior de
ABCD satisfaz
∠PBC = ∠DBA e ∠PDC = ∠BDA.
Prove que os vértices do quadrilátero ABCD pertencem a uma mesma
circunferência se, e somente se, AP = CP.

3. Triângulo Pedal

Definição 3.1. Seja P um ponto no plano do triângulo ABC e D, E e F as projeções


de P sobre as retas BC, CA e AB. O triângulo DEF é o triângulo pedal de P em
relação ao triângulo ABC.

O que triângulos pedais têm de especial? Primeiro, aparecem muitos ângulos retos,
o que propicia o aparecimento de quadriláteros inscritíveis. Segundo, eles
normalmente minimizam áreas.

Teorema do mínimo. Dados dois triângulos T e ABC, considere todos os triângulos


DEF semelhantes a T, todos na mesma ordem, com D sobre o lado BC, E sobre o
lado CA e F sobre o lado AB. Dentre todos esses triângulos, o de menor área é o
triângulo pedal de algum ponto P.

Demonstração
Não provaremos aqui a existência de um triângulo de área mínima (caso você
esteja curioso, estude topologia e depois volte!).
Seja DEF o triângulo de área mínima. Seja M o ponto de Miquel de ABC e DEF, e
sejam P, Q e R as projeções de M sobre os lados.

EUREKA! N°31, 2010

36
Sociedade Brasileira de Matemática

Note que o quadrilátero CPMQ é inscritível (pois ∠MPC e ∠MQC são retos), de
modo que ∠DME = ∠PMQ = 180° − ∠C. Portanto, ∠PMD = ∠QME : imagine o
ângulo ∠DME girando em torno de M para coincidir com ∠PMQ;MD vira MP e
ME vira MQ. Analogamente, ∠RMF = ∠QME.
Portanto os triângulos PMD, QME e RMF são semelhantes e induzem uma roto-
homotetia (você se lembra o que é isso?) que leva DEF a PQR. A razão de
MP
homotetia é ≤ 1, de modo que a área de PQR é menor ou igual à área de DEF.
MD
Como DEF tem área mínima, os triângulos devem ser congruentes e deste modo
MP = MD, ou seja, P = D. Analogamente, Q = E e R = F, de modo que DEF é o
triângulo pedal de P.

Exemplo 3.1.
(Prova de seleção EUA, 2008) Sejam P, Q, R pontos sobre os lados BC, CA, AB de
um triângulo acutângulo ABC tais que PQR é equilátero e tem área mínima entre
todos tais triângulos equiláteros. Prove que a reta perpendicular a QR que passa por
A, a reta perpendicular a RP que passa por B e a reta perpendicular a PQ que passa
por C têm um ponto comum.

Resolução
Pelo teorema do mínimo, PQR é triângulo pedal de algum ponto T.

EUREKA! N°31, 2010

37
Sociedade Brasileira de Matemática

Como os ângulos ∠TQA e ∠TRA são ambos retos, o quadrilátero AQTR é


inscritível, e o seu circuncentro é o ponto médio A´ de AT . Assim, a reta
perpendicular a QR e que passa por A, que contém a altura relativa a QR, é isogonal
a AT, que contém o circuncentro, em relação ao triângulo AQR. Como os ângulos
∠BAC e ∠QAR são iguais, a perpendicular e AT são isogonais em relação ao
triângulo ABC também. O análogo para as perpendiculares a PR por B e a PQ por
C. Como AT, BT e CT são concorrentes em T, seus isogonais são concorrentes no
conjugado isogonal de T.
A título de curiosidade, o ponto T é o primeiro ponto isodinâmico. Os dois pontos
isodinâmicos (adivinhe o nome do outro ponto!) são os pontos de interseção dos
círculos de Apolônio de A, B e C (que passam pelos vértices, o pé da bissetriz
interna e têm centro sobre o lado oposto). Os seus conjugados isogonais são os
pontos de Fermat. O primeiro ponto de Fermat é o ponto cuja soma das distâncias
aos vértices é mínima (supondo que os ângulos internos do triângulo são todos
menores do que 120° ). Veja [5] para aprender isso e muito, muito mais.

3.1. Voltando às simedianas

Uma aplicação interessante da ideia de triângulo pedal está relacionada às


simedianas. Uma outra maneira de construir as simedianas é a seguinte:

Lema. Construa quadrados ABBc Ac , BCCa Ba e CAAb Cb externamente sobre os


lados do triângulo ABC. Prolongue Ac Bc , Ba Ca e Cb Ab para obter o triângulo
A´ B´C´. Então as retas AA´,BB´ e CC´ concorrem no ponto simediano K de ABC.

EUREKA! N°31, 2010

38
Sociedade Brasileira de Matemática

Demonstração
Por simplicidade, sejam BC = a, CA = b e AB = c e L o encontro de AA´,BB´,CC´.
Queremos provar que L = K.
Primeiro, como os pares de retas AB; A´ B´,BC;B´C´ e CA;C´ A´ são paralelos, os
triângulos ABC e A´ B´C´ são semelhantes. Seja k a razão de semelhança. Sejam
ka ,kb e kc as distâncias de L a BC, CA e AB, respectivamente. Das semelhanças
entre LAB; LA’B, LBC; LB’C’ e LCA; LC’A’, todas de razão k,

ka k k k k k k
= b = c =k ⇔ a = b = c =
ka + a kb + b kc + c a b c 1− k

Isto quer dizer que as distâncias de L a cada um dos lados é proporcional aos seus
comprimentos. Além disso, considerando uma semelhança prova-se que um ponto
X pertence a, digamos, AL se, e somente, as distâncias de X aos lados AB e AC são
proporcionais a seus comprimentos. Basta provar que a simediana por A tem a
mesma propriedade. Para isso, considere a construção anterior, sendo D o mesmo
ponto definido anteriormente.

EUREKA! N°31, 2010

39
Sociedade Brasileira de Matemática

Sendo x e y as distâncias de D a AB e AC, respectivamente, considerando que o


ângulo entre AB e BD é ∠FBD = ∠ACB = ∠C e o ângulo entre AC e CD é
∠DCE = ∠ABC = ∠B (não se preocupe com triângulos obtusângulos; nesse caso,
troque o ângulo obtuso por seu suplementar), nos triângulos retângulos BDF e
CDE,x = BDsen∠C e y = DCsen∠B. Observando ainda que, sendo DB e DC
x sen∠C AB
tangentes, DB = DC, temos = = . Logo D pertence a AL e,
y sen∠B AC
consequentemente, K também. Da mesma forma provamos que K pertence a BL e
CL, de modo que L = K.
Assim como no teorema das bissetrizes, as simedianas dividem os lados opostos
em razões interessantes.
2
BN  AB 
Lema. Seja ABC um triângulo e NA uma simediana. Então =  .
CN  AC 
Demonstração

EUREKA! N°31, 2010

40
Sociedade Brasileira de Matemática

Já provamos anteriormente que as distâncias do ponto simediano K aos lados são


proporcionais a seus comprimentos. Então existe t real tal que ka = ta, kb = tb e kc =
tc ⋅ c tc 2 tb ⋅ b tb 2
tc. Assim, as áreas de KAB, KAC e KBC são = , = e
2 2 2 2
ta ⋅ a ta 2
= , respectivamente. Logo
2 2
BN área ABN área KBN área ABN − área KBN área KAB c 2  AB 
2

= = = = = 2 = 
CN área ACN área KCN área ACN − área KCN área KAC b  AC 

Lema. Sejam d a ,d b e d c as distâncias de um ponto P aos lados BC, CA, e AB do


BN c ⋅ d c
triângulo ABC. Se AP corta BC em N , então = .
CN b ⋅ db
Demonstração
Fica a cargo do leitor.

3.2 A desigualdade de Erdös-Mordell


Um dos principais teoremas sobre triângulos pedais é a desigualdade de Erdös-
Mordell:

Desigualdade de Erdös-Mordell. Seja P um ponto no plano do triângulo ABC e


d a ,d b ,d c as distâncias de P às retas BC, CA, AB respectivamente. Então
PA + PB + PC ≥ 2 ( d a + d b + d c )
Demonstração
Seja PA = d. “Multiplique” a figura original por d e construa triângulos
semelhantes aos triângulos obtidos por PA e as projeções de P sobre AB e AC:

EUREKA! N°31, 2010

41
Sociedade Brasileira de Matemática

Note que ∠GDE = 90° − β e ∠HDF = 90° − α , de modo que G, D e H são


colineares. Além disso, ∠EGD e ∠FHD são ambos retos, de modo que as retas
EG e FH são paralelas. A distância entre essas duas retas é GH = AB ⋅ d b + AC ⋅ d c ,
que é menor ou igual a EF = d ⋅ BC. Lembrando que d = PA, temos
AB AC
AB ⋅ db + AC ⋅ d c ≤ PA ⋅ BC ⇔ PA ≥ ⋅ db + ⋅ d c . Analogamente,
BC BC
AB BC
PB ≥ ⋅ da + ⋅ dc
AC AC
AC BC
PC ≥ ⋅ da + ⋅ db
AB AB
1
Somando as três desigualdades e lembrando que t + ≥ 2 para todo t real positivo,
t
 AB AC   AB BC   AC BC 
PA + PB + PC ≥  +  da +  +  db +  +  dc ≥ 2 ( d a + d b + d c )
 AC AB   BC AB   BC AC 

Exemplo 3.2.
(IMO 1991, Problema 4) Sejam ABC um triângulo e M um ponto interior. Mostre
que pelo menos um dos ângulos ∠MAB,∠MBC e ∠MCA é menor ou igual a 30°.

Resolução:
Sejam P, Q e R as projeções de M sobre BC, CA, e AB, respectivamente.

Pela desigualdade de Erdös-Mordell, MA + MB + MC ≥ 2 ( MP + MQ + MR ) . Se

EUREKA! N°31, 2010

42
Sociedade Brasileira de Matemática

MR MP MQ 1
todas as razões, , são maiores do que , então
MA MB MC 2
MA < 2 MR,MB < 2 MP e MC < 2 MQ, e MA + MB + MC < 2 ( MP + MQ + MR ) ,
MR 1
contradição. Então uma das razões, digamos, , é menor ou igual a . Todavia,
MA 2
MR
= sen∠MAB, de modo que ∠MAB ≤ 30°.
MA

Exercícios
09. Dado um triângulo com perímetro L, seja P o perímetro de um triângulo pedal.
Prove que L ≥ 2 P.
Quando ocorre a igualdade?

10. Seja P um ponto interior ao triângulo ABC e T o seu triângulo pedal. Prove que
R 2 − OP 2
a área de T é igual a vezes a área de ABC.
4R2

11. Seja G o baricentro do triângulo ABC e D, E, F as projeções ortogonais de G


sobre os lados BC, CA e AB, respectivamente. Prove que
4 área DEF 1
< ≤
27 área ABC 4

12. Seja P um ponto qualquer no plano do triângulo ABC. As projeções de P sobre


BC, CA, AB são D, E e F respectivamente.
(a) Prove que as perpendiculares a EF, FD, DE por A, B, C respectivamente têm
um ponto P´em comum.
(b) Sejam Q e Q´ as segundas interseções de AP e AP´ com o circuncírculo de
ABC, respectivamente.
Prove que as retas QQ´ e BC são paralelas.

13. Os pontos X, Y e Z estão sobre BC, CA, e AB, respectivamente, e são tais que
XYZ e ABC são semelhantes, nessa ordem. Prove que o circuncentro de XYZ é
equidistante aos ortocentros de ABC e XYZ.

14. (Ibero 2008, Problema 5) Seja ABC um triângulo e X, Y, Z pontos interiores dos
lados BC, AC, AB, respectivamente. Sejam A´, B´, C´ os circuncentros dos
triângulos AZY, BXZ, CYX, respectivamente.
Demonstre que

EUREKA! N°31, 2010

43
Sociedade Brasileira de Matemática

( ABC )
( A´ B´C´ ) ≥
4
e que a igualdade ocorre se, e somente se, as retas AA´,BB´,CC´ têm um ponto em
comum.
Observação: Para um triângulo qualquer RST, denotamos a sua área por (RST).

15. (OPM, 2001)

uuur uuur
(a) Na figura acima, considere pontos B1 e C1 sobre as semirretas AB e AC,
respectivamente.

(i) Mostre que a soma das áreas dos paralelogramos com lados AB1 e AM e com
lados AC1 e AM é igual à área do paralelogramo tal que um de seus lados é B1C1 e
o outro é paralelo e igual a AM.

(ii) Tomando AB1 = AC e AC1 = AB, conclua que AB ⋅ v + AC ⋅ w ≤ BC ⋅ x


(b) Prove a Desigualdade de Erdös-Mordell: 2 ( u + v + w ) ≤ x + y + z

Referências Bibliográficas

[1] Uma ótima fonte de problemas é o Mathlinks: http://www.mathlinks.ro/ (em inglês).


[2] Para quem gosta de Geometria, o Forum Geometricorum é um prato cheio! Tudo sobre
quadriláteros completos foi retirado do artigo Steiner´s Theorems on the Complete
Quadrilateral, de Jean – Pierre Ehrmann, Volume 4 (2004), pp 35-52.
[3] Para quem quer saber mais sobre o teorema de Erdös-Mordell, na Eureka! 18.
[4] O livro Modern Geometry of the Triangle, de William Gallatly, contém muita
informação interessante, incluindo a maior parte dos fatos sobre simedianas e o ponto
simediano.
[5] Mais conjugados isogonais? Isso e muito mais no livro Geometry of Conics (o “livro do
bode” – Veja a Capa!), de A. V. Akoplyan e A. A. Zaslavsky.

EUREKA! N°31, 2010

44
Sociedade Brasileira de Matemática

SÉRIE HARMÔNICA DE NÚMEROS PRIMOS


Lenimar Nunes de Andrade
UFPB – João Pessoa, PB

1. Série harmônica
Há séculos que se sabe que a soma dos recíprocos dos números inteiros
positivos
1 1 1 1
H n = 1 + + + + ... +
2 3 4 n
pode ultrapassar o valor de qualquer constante positiva pré-estabelecida, bastando,
para isso, somar determinada quantidade de parcelas, considerando o valor de n
suficientemente grande. Quando consideramos uma infinidade de parcelas desse
tipo, temos uma série infinita conhecida pelo nome de série harmônica, e, como a
soma H n vai aumentando à medida que n aumenta e ultrapassa qualquer valor pré-
estabelecido, temos que se trata de uma série divergente. Existem pelo menos 20
demonstrações diferentes desse fato (veja, por exemplo, a referência [3]) e algumas
demonstrações simples podem ser encontradas em [1] ou [2].*
Sabe-se que o crescimento das somas parciais H n da série harmônica é bastante
lento. Se somarmos 1000 termos da série, obtemos 7, 4855 como resultado. Se
somarmos 1000000 de termos, obtemos 14,3927. Para a soma ultrapassar 100,
estima-se que seja necessário somar-se aproximadamente 1,5 × 1043 parcelas – um
número de parcelas tão grande que nem os computadores mais modernos de hoje
em dia, trabalhando ininterruptamente ao longo de vários milênios, conseguiriam
efetuar todos os cálculos.
Se for escolhido um determinado algarismo, e retirados da série harmônica todos
os termos que contenham esse algarismo, então surpreendentemente, obtem-se uma
série infinita na qual a soma dos n primeiros termos é sempre inferior a 80, não

*
Nota do editor: Uma demonstração particularmente simples deste fato é a seguinte:
1 1 1 1 1 1 1  1 1 
H 2k = 1 ++  +  +  + + +  + ... +  k −1 + ... + k  ≥
2 3 4 5 6 7 8  2 +1 2 
1 1 1 1 1 1 1  1 1 1  1 1 1 1
≥ 1 + +  +  +  + + +  + ... +  k + k + ... + k  = 1 + + + + ... +
2  4 4 8 8 8 8 2 2 2  2 2 2 2

k
=1+ , que pode ultrapassar o valor de qualquer constante positiva pré-estabelecida.
2
EUREKA! N°31, 2010

45
Sociedade Brasileira de Matemática

importando qual seja o valor de n**. Obtemos, portanto, o que chamamos de série
convergente.

2. Série dos recíprocos de números primos


A sequência de números primos 2, 3, 5, 7, 11, 13,... é infinita. Apesar de estarem
bem próximos uns dos outros, para valores pequenos, à medida que aumentamos o
valor de n, torna-se difícil encontrar números primos maiores do que n, e eles vão
ficando cada vez mais distantes uns dos outros em média.
Se retirarmos da série hermônica todos os termos cujos denominadores não sejam
primos, obtemos ainda assim uma série infinita:
1 1 1 1 1 1 1
+ + + + + + + ...
2 3 5 7 11 13 17
vamos denominar a série assim obtida de série harmônica de números primos.
O principal objetivo deste artigo é mostrar que essa série harmônica de números
primos é divergente, ou seja, a soma dos seus n primeiros termos pode ultrapassar
qualquer valor pré-estabelecido. Esse fato foi observado pela primeira vez por
Leonhard Euler (1707–1783).
O crescimento do valor das somas dos n primeiros termos da série harmônica de
números primos é exatamente lento. Muito mais lento do que o da série harmônica.
Com os recursos computacionais atuais, é impossível realizar a tarefa de somar
uma certa quantidade de termos dessa série e obtermos o resultado igual ou
superior a 5,0.

3. Demonstração da divergência
Inicialmente, vamos mostrar que dado um n inteiro positivo temos que
k
 1 k k2
1 +  < 1 + + 2
 n n n
para todo inteiro positivo k tal que k ≤ n. Em particular, fazendo k = n, obtemos
que
n
 1 n n2
 1 +  < 1 + + 2 = 3. (1)
 n n n
Para isso, vamos usar o método da indução matemática. Para k = 1, a desigualdade
1 1 1
reduz-se a 1 + < 1 + + 2 o que é verdadeiro. Suponho a desigualdade válida
n n n
para k, vamos verificar que vale para k + 1 também:

**
Veja o problema proposto no. 141, na página 60.

EUREKA! N°31, 2010

46
Sociedade Brasileira de Matemática

k +1
 1   1   k k  1 
k
 1
2
k k2 1
1 +  =  1 +   1 +  <  1 + + 2  1 +  = 1 + + 2 +
 n  n   n   n n  n  n n n

k + 1 ( k + 1) k 2 − n ( k + 1) k + 1 ( k + 1)
2 2
k k2
+ 2 + 3 =1+ + 2
+ 3
<1+ + ,
n n n n 14 4244
n 3 n n2
<0

onde utilizamos que k < n ( k + 1) porque k ≤ n. Desse modo, a desigualdade fica


2

demonstrada.
Escolhido n, um inteiro positivo qualquer, consideremos r o inteiro tal que
2r ≤ n < 2r +1 , ou seja, r é o expoente da maior potência de 2 que não ultrapassa n.
Sejam p1 = 2 , p2 = 3 , p3 = 5 ,..., ps os primos positivos menores ou iguais a n. Se m
for um inteiro tal que 1 ≤ m ≤ n, então o Teorema Fundamental da Aritmética nos
garante que m pode ser escrito de modo único como um produto de potências dos
primos pk com expoentes inteiros não negativos t1 ,t2 ,...,ts :
m = p1t1 p2t2 p3t3 ...psts
Note que nenhum dos expoentes tk pode ser maior do que r , pois, se assim fosse,
teríamos m ≥ pktk ≥ 2tk ≥ 2r +1 > n, o que seria um absurdo. Assim, para todo k,
temos 0 ≤ tk ≤ r.
Temos, então, a seguinte desigualdade:
1 1 1 1  1 1 1   1 1 1 
1+ + + + ... + <  1 + + + ... + r  × 1 + + + ... + r ×
2 3 4 n  2 4 2   3 9 3 
 1 1 1  1 1 1 
1 + + + ... + r  × ... ×  1 + + 2 + ... + r  (2)
 5 25 5   ps ps ps 
Sua demonstração consiste na observação de que cada parcela 1 m que aparece do
lado esquerdo da desigualdade pode ser escrita de modo único na forma
1 1 1 1
= t1 × t2 × ... × ts
m p1 p2 ps
1
e que cada fração da forma ocorre uma única vez como uma das parcelas do
pktk
 1 1 1 
fator 1 + + 2 + ... + r  que aparece no segundo membro da desigualdade.
 pk pk pk 

EUREKA! N°31, 2010

47
Sociedade Brasileira de Matemática

1 2
Supondo q ≥ 2 , temos 2q − q ≥ 2 , ou seja, 2 ( q − 1) ≥ q que equivale a ≤ e
q −1 q
calculando a seguinte soma de uma progressão geométrica de razão 1 q , obtemos:
1
1 − r +1
1 1 1
1 + + 2 + ... + r =
q
<
1
=
q
=
( q − 1) + 1 = 1 + 1
q q q 1−
1
1−
1 q −1 q −1 q −1
q q
Usando agora a desigualdade (1) obtida no início desta seção, obtemos
q −1
 1 
1
1
1 +  < 3 que é equivalente a 1 + < 3 q −1 .
 q − 1  q − 1
Aplicando-se logaritmos, obtemos:
 1   q1−1 
log 1 +  < log 3 ,
 q −1  
 
ou seja,
 1  log 3 2 log 3
log 1 + < ≤
 q −1  q −1 q
de onde finalmente obtemos
 1 1 1   1  2log3
log 1 + + 2 + ... + r  < log  1 + < (3)
 q q q   q −1  q

Aplicando-se logaritmos aos dois membros da desigualdade (2), e usando-se


propriedade log ( ab ) = log a + log b, obtemos:
 1 1 1 1  1 1 1
log 1 + + + + ... +  < log  1 + + + ... + r  +
 2 3 4 n  2 4 2 
 1 1 1  1 1 1 
log 1 + + + ... + r  + ... + log  1 + + 2 + ... + r  (4)
 3 9 3   ps ps ps 
Usando-se várias vezes o resultado (3) na desigualdade (4), obtemos:
 1 1 1 1  2 log 3 2 log 3 2 log 3 2 log 3
log 1 + + + + ... +  < + + + ... +
 2 3 4 n 2 3 5 ps
1 1 1 1 
= 2 log 3 + + + ... +  .
2 3 5 ps 

EUREKA! N°31, 2010

48
Sociedade Brasileira de Matemática

1 1 1
Se existisse uma constante L tal que + + ... + < L para todo inteiro positivo
2 3 ps
 1 1 1 1
s, então teríamos log 1 + + + + ... +  < 2 log 3 × L = log ( 32 L ) (5)
 2 3 4 n
1 1 1 1
o que implicaria 1 + + + + ... + < 32 L , para todo n natural, (6)
2 3 4 n
o que seria um absurdo, pois a série harmônica não é limitada e, para algum n, a
 1 1 1
soma 1 + + + ... +  ultrapassaria a constante 32 L .
 2 3 n

4. Algumas somas parciais


Sabe-se que quando maior o valor de n, mais próximo de ln ( ln ( n ) ) + B1 será a
soma de todos os recíprocos de primo inferiores a n. A constante B1 é conhecida
como constante de Mertens e tem valor igual a 0,2614972128....
Com a ajuda de um computador, se somarmos os recíprocos dos números primos
inferiores a 1000, obtemos 2,1990 como resultado. Observe que esse valor é
próximo de ln ( ln (1000 ) ) + B1 ≈ 2 ,1941.
Somando-se todos os recíprocos de númreos primos inferiores a 107 (dez milhões),
obtemos uma soma total igual a 3,041449. Calculando-se
( )
ln ln (107 ) + B1 , obtemos 3,041440 que é muito próximo da soma obtida.
A aproximação ln ( ln ( n ) ) + B1 ≈ S para a soma dos recíprocos de primos inferiores
a n pode ser escrita na forma
S − B1
n ≈ ee .
5− B1
Por exemplo, para chegar a 5,0, a soma necessitaria de n = ee = ee ≈ 4, 2 × 1049
4 ,7385

parcelas, o que é um número realmente assustador: nem o computador mais rápido


de hoje em dia, trabalhando incessantemente por milênios a fio, conseguiria somar
tal quantidade de termos.

Referências Bibliográficas
[1] G. Ávila, “As séries infinitas”, RPM 30, 1996.
[2] G. Garbi, “A surprendente série harmônica”, RPM 42, 2000.
[3] S.J. Kifowit, T. A. Stamps, “The harmonic series diverges again and again”, The
AMATYC Review, Vol. 27, No. 2, 2006.
[4] D. O. Shkiyarsky, N. N. Chentsov, I. M. Yaglom, “Selected problemas and theorems in
elementary Mathematics – Arithmetic and Algebra”, Mir Publishers, Moscow, 1979.

EUREKA! N°31, 2010

49
Sociedade Brasileira de Matemática

COMO É QUE FAZ?


PROBLEMA PROPOSTO POR WILSON CARLOS DA SILVA RAMOS (BELÉM – PA)


 4 xy + 4 ( x + y ) +
3 85
2 2
=
( x + y) 3
2

1) Resolva o sistema 
 2 x + 1 = 13
 x+ y 3
1
Solução: Da segunda equação, obtemos y = − x. Substituindo esse valor de
13
− 2x
3
2
4x  13  85
y na primeira equação, obtemos + 4 y 2 + 3  − 2 x  = . Fazendo
13
− 2x 3  3
3
13 1  13  1 1 u 13
u = − 2 x, temos x =  − u  , e y = − x = + − ,
3 2 3  u u 2 6
2
26 2 13  85 26 68 26 4
donde − 2 +  + u −  + 3u 2 = , ou seja, 4u 2 − u − − + = 0.
3u u 3 3 3 9 3u u 2
1 1 26 140
Fazendo w = u + , temos u 2 + 2 = w2 − 2, donde 4 w2 − w − = 0, e logo
u u 3 9
10 7 1 10 1 −7
w= ou w = − . Assim, u + = ou u + = . A primeira dessas
3 6 u 3 u 6
1
equações tem soluções u = 3 e u = , que nos dão as duas soluções
3
 1  13  2 1 1  1  13  1 
reais  x =  − u  = , y = − x = −  e  x =  − u  = 2, y = − x = 1 . A
 2 3  3 u 3  2 3  u 
segunda não tem soluções reais, mas tem as soluções complexas conjugadas
7 i 95
u=− ± , que nos dão as soluções complexas correspondentes
12 12
 59 + i 95 −73 − i 95   59 − i 95 −73 + i 95 
( x, y ) =  ,  e ( x, y ) =  ,  .
 24 24   24 24 

EUREKA! N°31, 2010

50
Sociedade Brasileira de Matemática

PROBLEMA PROPOSTO POR MARCÍLIO MIRANDA DE CARVALHO (TERESINA – PI)


(teste de seleção da Romênia para IMO de 1978)

2) Para cada n natural, resolva a equação:


sen x ⋅ sen 2 x...sen nx + cos x ⋅ cos 2 x...cos nx = 1

SOLUÇÃO DE RENAN HENRIQUE FINDER (JOINVILLE – SC)

Se n ≥ 2, usando a desigualdade triangular e o fato de que max { sen α , cos α } ≤ 1 ,


temos
1 = sen x sen 2x...sen nx + cos x cos2 x...cos nx ≤ sen x sen 2x...sen nx + cos x cos2 x...cos nx
≤ sen x ⋅ sen 2 x + cos x ⋅ cos 2 x
Pela desigualdade de Cauchy-Schwarz, 1 ≤ sen 2 x + cos 2 x sen 2 2 x + cos 2 2 x = 1.
Para que ocorra a igualdade, devemos ter sen x cos 2 x = cos x sen 2 x, logo
sen x = sen ( 2 x − x ) = 0, e portanto x = mπ , m ∈ ¢ ⇒ senx = 0.
Então cos x cos 2 x...cos nx = 1. Se m é par, isso sempre ocorre. Se m é ímpar,
n
cos x = −1, cos 2 x = 1, cos3 x = −1,..., logo cos x...cos nx = ( −1)  2  . Portanto,
n
• Se n ≥ 2 e   é par (ou seja, se n é da forma 4j – 1 ou 4j), as soluções são
2
x = mπ , m ∈ ¢.
n
• Se n ≥ 2 e   é ímpar (ou seja, se n é da forma 4j + 1 ou 4j + 2), as
2
soluções são x = 2mπ , m ∈ ¢.
2 2 2 π  π
Se n = 1 o problema equivale a = sen x + cos x ⇔ sen = sen  x + 
2 2 2 4  4
π
⇔ x = 2mπ ou x = 2mπ + , m ∈ ¢.
2

EUREKA! N°31, 2010

51
Sociedade Brasileira de Matemática

SOLUÇÕES DE PROBLEMAS PROPOSTOS


! Publicamos aqui algumas das respostas enviadas por nossos leitores.

123. Determine todas as funções f : ¥* →¥* tais que


2 f (m + n ) = f (m) f (n) + f (m) f (n) , para quaisquer m, n ∈ ¥ * distintos.
2 2 3 2 2

Obs: ¥* = {1, 2,3,...} é o conjunto dos inteiros positivos.

SOLUÇÃO DE ÍTALO DOWELL LIRA MELO (TERESINA – PI)


Primeiro note que se f é uma função constante então f é solução. Agora
suponhamos que exista uma função não constante que seja solução. Assim existem
naturais a e b com f ( a ) < f ( b ) .
Daí temos que 2 f ( a ) = f ( a ) + f ( a ) < f ( a ) f ( b ) + f ( a ) f ( b ) < 2 f ( b ) .
3 3 3 2 2 3

Como 2f ( a2 + b2 ) = f ( a) f ( b) + f ( a) f ( b) , segue que 2 f ( a) < 2 f ( a2 + b2 ) < 2 f ( b) .


3 2 2 3 3 3

Se dividirmos por 2 encontramos que f ( a ) < f ( a 2 + b2 ) < f ( b ) ⇒


3 3 3

⇒ f ( a ) < f (a 2 + b 2 ) < f ( b ) .
Isto nos diz que entre quaisquer dois valores distintos de f podemos encontrar um
outro valor de f mas isto não pode ocorer sempre uma vez que f assume valores em
¥*. Esta contradição mostra que tal função não existe. Assim as funções
constantes são as únicas soluções.

124. Considere a seqüência (an )n ≥1 definida por a1 = a2 = a3 = a4 = 1 e


an −1an −3 + an2− 2
an = , ∀n ≥ 5.
an − 4
Prove que an é um inteiro positivo, para todo inteiro positivo n.

SOLUÇÃO DE ZOROASTRO AZAMBUJA NETO (RIO DE JANEIRO – RJ)


Vamos provar por indução que, para todo n ≥ 5, valem as seguintes afirmações:
• ak − 4 ak −1ak −3 + ak2− 2 , ∀k ∈ ¥,5 ≤ k ≤ n (e logo ak ∈ ¥, ∀k ≤ n ).
• ak −3 ak3− 2 + ak2−1ak − 4 , ∀k ∈ ¥,5 ≤ k ≤ n .
• ak − 2 ak −1ak2− 4 + ak3− 3 , ∀k ∈ ¥,5 ≤ k ≤ n .
• mdc ( ak −1 , ak ) = 1, ∀k ∈ ¥,1 ≤ k − 1 < k ≤ n.
• mdc ( ak − 2 , ak ) = 1, ∀k ∈ ¥,1 ≤ k − 2 < k ≤ n.

EUREKA! N°31, 2010

52
Sociedade Brasileira de Matemática

• mdc ( ak −3 , ak ) = 1, ∀k ∈ ¥,1 ≤ k − 3 < k ≤ n.


Note que, como a1 = a2 = a3 = a4 = 1, temos a5 = 2 e a6 = 3 , e portanto os itens
acima se verificam para todo n ≤ 6 .
Vamos agora verificá-los para k= n + 1: queremos mostrar que
 a a + an2− 2 
an −3 an an − 2 + an2−1 =  n −1 n − 3 + 2
=
( an−1an−3 + an2−2 ) an−2 + an2−1an−4 .
 n−2
a a n −1
 an − 4  an − 4
Como o lado direito é inteiro e mdc ( an −3 , an − 4 ) = 1, isso equivale a mostrar que
an −3 ( an −1an − 3 + an2− 2 ) an − 2 + an2−1an − 4 , o que segue de an −3 an3− 2 + an2−1an − 4 .
Também queremos mostrar que
an3−1an2− 4 + ( an −1an − 3 + an2−2 ) an −3
2 2
 a a + an2− 2 
an − 2 a 3
n −1 +a a
2
n n −3 =a 3
n −1 +  n −1 n −3  n −3
a = .
 an − 4  an2− 4
Como o lado direito é inteiro e mdc ( an − 2 , an − 4 ) = 1, isso equivale a mostrar que
an − 2 an3−1an2− 4 + ( an −1an −3 + an2−2 ) an −3 , o que equivale a
2

an − 2 an3−1an2− 4 + an2−1an3−3 = an2−1 ( an −1an2− 4 + an3−3 ) , e isso segue de a n − 2 a n −1 a n2− 4 + a n3− 3 .


E também queremos mostrar que

an −1 a a + a = 
 an −1an − 3 + an2− 2  2 ( an −1an −3 + an2− 2 ) an2−3 + an3− 2 an − 4
 an −3 + an − 2 =
2 3 3
n n −3 n−2 .
 an − 4  an − 4
Como o lado direito é inteiro e mdc ( an −1 , an − 4 ) = 1, isso equivale a mostrar que
an −1 ( an −1an − 3 + an2− 2 ) an2−3 + an3− 2 an − 4 , o que equivale a

an −1 an2− 2 an2− 3 + an3− 2 an − 4 = an2− 2 ( an2−3 + an − 2 an − 4 ) , que segue de an −1 an2−3 + an − 2 an − 4 ,


que por sua vez segue da igualdade an −1an −5 = an2−3 + an − 2 an − 4 , que vem da definição
de an −1.
Finalmente, de an +1an −3 = an an − 2 + an2−1 , segue que mdc ( an +1 , an ) mdc ( an , an2−1 ) = 1,

mdc ( an +1 , an −1 ) mdc ( an −1 , an an − 2 ) = 1 e mdc ( an +1 , an − 2 ) mdc ( an − 2 , an2−1 ) = 1.

125. Considere dois naturais m≥2 e n ≥ 2, e as seqüências


(a0 , a1 , a2 ,..., amn ), ai ∈ {0,1}.
As seqüências de tipo m satisfazem as condições:

EUREKA! N°31, 2010

53
Sociedade Brasileira de Matemática

• ak ak + m = 0, para todo k;
• Se ak ak +1 = 1 então m divide k
As seqüências de tipo n são definidas analogamente. Prove que existem tantas
seqüências do tipo m quanto do tipo n.

SOLUÇÃO DE JOSÉ DE ALMEIDA PANTERA (RIO DE JANEIRO – RJ)


Considere a matriz m × n, B = ( bij )1≤i ≤ m dada por bij = am (i −1) + j ,1 ≤ i ≤ m,1 ≤ j ≤ n.
1≤ j ≤ n

Temos que ( a0 ,..., amn ) é uma sequência de tipo m se e somente se B é uma matriz
m × n cujas entradas pertencem a {0, 1} sem dois termos vizinhos iguais a 1 numa
mesma linha ou numa mesma coluna, e tal que b1m = 1 ⇒ a0 = 0.
Considere agora a função f que leva uma matriz B m × n na matriz f ( B ) n × m
dada por ( f ( B )) i, j
= B( m +1− j ),( n +1−i ) . Temos que f é uma bijeção entre as matrizes
m × n cujas entradas pertencem a {0, 1} sem dois termos vizinhos iguais a 1 numa
mesma linha ou numa mesma coluna e as matrizes n × m cujas entradas pertencem
a {0, 1} sem dois termos vizinhos iguais a 1 numa mesma linha ou numa mesma
coluna, tal que ( f ( B ) )1,m = B1,n . Isso mostra que o número de seqüências do tipo
m é igual ao número de seqüências do tipo n.

126. As circunferências Γi , 0 ≤ i ≤ 5, são tangentes a uma circunferência Γ nos


pontos Ai . Além disso, Γi é tangente a Γi+1 para 0 ≤ i ≤ 5 e Γ5 é tangente a Γ0 .
Prove que A0 A3 , A1 A4 , A2 A5 são concorrentes.

SOLUÇÃO DE MATHEUS SECCO TORRES DA SILVA (RIO DE JANEIRO – RJ)

Sejam Oi os centros de Γi e ri os raios de Γi . Além disso, seja r o raio de Γ , que


tem centro O. Vamos supor inicialmente que as circunferências Γi são exteriores a
Γ . Usando Ceva trigonométrico no ∆A2 A4 A0 , devemos provar que
sen ( < A1 A4 A2 ) sen ( < A3 A0 A4 ) sen ( < A5 A2 A0 )
⋅ ⋅ =1⇔
sen ( < A1 A4 A0 ) sen ( < A3 A0 A2 ) sen ( < A5 A2 A4 )
 AOA   A OA   A OA 
sen  < 1 2  sen  < 3 4  sen  < 5 0 
 2 
⋅  2 
⋅  2 
=1
 A OA   A OA   A OA 
sen  < 2 3  sen  < 4 5  sen  < 0 1 
 2   2   2 

EUREKA! N°31, 2010

54
Sociedade Brasileira de Matemática

A0
A5 A1

A4 A2

A3

Denotamos < AOA


i i +1 = α i ,0 ≤ i ≤ 5, índices módulo 6.

Mas no ∆OOi Oi +1 , temos:


( ri + ri +1 ) = ( r + ri ) + ( r + ri +1 ) − 2 ( r + ri )( r + ri +1 ) ⋅ cos α i ⇒
2 2 2

2ri ri +1
cos α i = 1 − ⇒
( r + ri )( r + ri +1 )
αi 2ri ri +1
1 − 2 sen 2 =1− ⇒
2 ( r + ri )( r + ri +1 )
αi ri ri +1
sen 2 = , donde
2 ( r + ri )( r + ri +1 )
α  α  α 
sen 2  1  sen 2  3  sen 2  5 
 2 ⋅  2 ⋅  2  =1⇒
 α   α   α 
sen 2  2  sen 2  4  sen 2  6 
 2   2   2 
α  α  α 
sen  1  sen  3  sen  5 
 2 ⋅  2 ⋅  2  = 1, pois sen  α i  > 0, ∀i ≤ 5,
 
α  α  α   2
sen  2  sen  4  sen  6 
 2   2   2 
donde obtemos o desejado.
Isso conclui a prova!.
Obs.: Supusemos que as circunferências Γi estavam no exterior de Γ1 , mas se

EUREKA! N°31, 2010

55
Sociedade Brasileira de Matemática

αi ri ri +1
fossem interiores, obteríamos sen2 = , o que também nos daria o
2 ( r − ri )( r − ri +1 )
desejado, com um argumento análogo.

129. Um coelho está numa rua infinita dividida em quadrados numerados pelos
inteiros, e começa no quadrado 0. Se num dado momento ele está no quadrado k,
1
ele escolhe, com probabilidade , pular para o quadrado k + 2 ou, também com
2
1
probabilidade , pular para o quadrado k – 1. Ele continua esse processo
2
indefinidamante. Dado m ∈ ¢ , determine a probabilidade de, em algum momento,
o coelho pisar no quadrado m.

SOLUÇÃO DE ASDRUBAL PAFÚNCIO SANTOS (BOTUCATU – SP)


Denotemos por am a probabilidade de, em algum momento, o coelho pisar no
quadrado m.
Seja n ≠ 0 um inteiro. Após o primeiro passo do coelho, ele pode estar no quadrado
1 1
–1, com probabilidade , ou no quadrado 2, com probabilidade , ficando a
2 2
distâncias respectivamente n + 1 e n – 2 do quadrado n. Assim, para todo
1 1
n ≠ 0, an = an − 2 + an +1 ( *) .
2 2
1 2 n 5  n 
Como ∑   tende a 0 exponencialmente rápido, com probabilidade total
2n k =0  k 
temos que, para um certo n0 ∈ ¥, e para todo n ≥ n0 , pelo menos 40% dos n
primeiros passos do coelho são para frente, o que faz com que, após n passos, ele
2n 3n n
esteja num quadrado de número maior ou igual a 2 ⋅ − = , para todo
5 5 5
n ≥ n0 . Em particular, a probabilidade de o coelho pisar no quadrado m tende a 0
quando m tende a −∞ .

De (*), temos am + 3 = 2am + 2 − am , ∀m ≥ 0, donde existem constantes A, B, C com


m m
1+ 5  1− 5 
am = A + B   + C   , ∀m ≥ 0, pois o polinômio característico da
 2   2 

EUREKA! N°31, 2010

56
Sociedade Brasileira de Matemática

  1 + 5   1− 5 
recorrência acima é x3 − 2 x 2 + 1 = ( x − 1)  x −    x −    .
 
  2    2 
Como ( am ) é limitada, devemos ter B = 0. Como a0 = 1, devemos ter A + C = 1.
De (*), também temos a− m −3 = 2a− m − a− m +1 , ∀m ≥ 1, ou seja, fazendo bk = a− k ,
temos bk + 3 = 2bk +1 − bk , ∀k ≥ 0, donde, como
  −1 + 5    −1 − 5  
x3 − 2 x + 1 = ( x − 1)  x − 
  

x −    , existem ° °,C
A, B ° com
  2    2 
k k

bk = ° °  −1 + 5  + C
A+ B °  −1 − 5  , ∀k ≥ 0. Como b não só é limitado como
   2 
k
 2  
tende a 0 quando k tende a +∞ , devemos ter ° A=C° = 0, e como b = a = 1,
0 0
° = 1.
devemos ter B
m −m m
 1− 5  −1+ 5   1+ 5 
Assim, am = 1 − C + C   , ∀m ≥ 0 e am = b−m =   =   , ∀m ≤ 0.
 2   2   2 
1 1
Fazendo n = 1 em (*), obtemos a1 = a−1 + a2 , donde
2 2
 1 − 5  1  −1 + 5  1   3 − 5 
1 − C + C   =   + 1 − C + C    , e portanto
 2  2 2  2  2 
7−3 5
C= .
2
 3 5 − 5  7 − 3 5  1 − 5  m
 +    , ∀m ≥ 0
 2  2   2 
Assim, temos am =  m
.
 1 + 5 
  , ∀m ≤ 0
 2 

130. Suponha que a, b, c ∈ ¡ e a equação x 2 − ( a + b + c ) x + ( ab + ac + bc) = 0 não


tem raízes reais. Prove que a, b e c têm todos o mesmo sinal e existe um triângulo
de lados a, b e c.

EUREKA! N°31, 2010

57
Sociedade Brasileira de Matemática

SOLUÇÃO DE DANIEL EITI NISHIDA KAWAI (TAUBATÉ – SP)


Se a equação x 2 − ( a + b + c ) ⋅ x + ( ab + ac + bc ) = 0 não tem raízes reais, temos
∆ < 0 ⇔ ( a + b + c ) − 4 ⋅ 1 ⋅ ( ab + ac + bc ) < 0 ⇔
2

⇔ a 2 + b2 + c 2 + 2ab + 2 ac + 2bc − 4ab − 4ac − 4bc < 0 ⇔


⇔ a 2 + b2 + c 2 − 2ab − 2 ac − 2bc < 0 ⇔
⇔ a 2 + b2 + c 2 + 2ab − 2 ac − 2bc − 4ab < 0 ⇔ ( a + b − c ) − 4ab < 0 ⇔
2

⇔ ( a + b − c ) < 4ab ⇔ 0 ≤ ( a + b − c ) < 4ab ⇒ 4ab > 0 ⇒ ab > 0 ⇒ a e b têm o


2 2

mesmo sinal. De maneira análoga, b e c têm o mesmo sinal. Assim, a, b e c têm


todos o mesmo sinal, e logo ab = a b , ac = a c e bc = b c .
Assim, a 2 + b2 + c 2 − 2 ab − 2 ac − 2 bc < 0, donde
⇒ a 2 + b2 + c 2 + 2 ab − 2 ac − 2 bc < 4 ab ⇒
⇒ a + b + c + 2 a b −2 a c −2 b c <4 a b ⇒
2 2 2

⇒( a + b − c) < 2 a ⋅ b ( ) ⇒ −2 a ⋅ b < a + b − c ⇒
2 2

b + b ⇒( c ) <( a ) + 2 a ⋅ b +( b )
2 2 2
⇒ c < a +2 a ⋅ ⇒

( c ) <( b ) ⇒ c < a + b.
2 2
⇒ a +
De maneira análoga,
b < a + c e a < b + c . Assim, existe um triângulo de lados
a, b e c.

Agradecemos o envio de soluções e a colaboração de:

Carlos Alberto da Silva Victor (Nilópolis – RJ) Prob. 123, 130


Rodrigo dos Anjos Azevedo (Três Rios – RJ) Prob. 130
Vinicius dos Nascimento S. Mano (Petrópolis – RJ) Prob. 130
Marcelo Robeiro de Souza (Rio de Janeiro – RJ) Prob. 130
Jheimyson Rego Barnabé (Imperatriz – MA) Prob. 130
Flávio Antonio Alves (Amparo – SP) Prob. 130
Ítalo Dowell Lira Melo (Teresina – PI) Prob. 130
Curro Fernández López (Lugo, Espanha) Prob. 42
Miguel Amengual Covas (Mallorca, Espanha) Prob. 110
Bruno Salgueiro Fanego (Galicia, Espanha) Prob. 116, 117, 118

Continuamos aguardando soluções para os problemas 131 e 132.

EUREKA! N°31, 2010

58
Sociedade Brasileira de Matemática

PROBLEMAS PROPOSTOS
* Convidamos o leitor a enviar soluções dos problemas propostos e sugestões de novos
problemas para próximos números.

133) Considere um n–ágono regular inscrito em um círculo unitário, fixe um


vértice i e denote por dj a distância entre este vértice i e o vértice j. Prove que
n −1

∏ (5 − d ) = F
j ≠i
2
j n
2
onde F1 = 0, F1 = 1 e Fn = Fn −1 , Fn − 2 se n ≥ 2.
j =0

134) Considere a operação ⋅ entre dois vetores do ¡3 definida por:


( x, y , z ) ⋅ ( u, v, w ) = ( xu + yw + zv, xw + zu + yv, xv + yu + zw )
Prove que, para todo k ≥ 1, se ( x, y , z ) = ( 0,0,0 ) então x = y = z = 0.
k

( x, y , z ) ∈ ¡ 3 , ( x, y , z ) = ( x, y , z ) k > 1,
1
Obs.: Para qualquer e, para todo
( x, y , z ) = ( x , y , z ) ⋅ ( x, y , z ) .
k k −1

135) Considere um hemisfério cuja base é um círculo ( C1 ) . Um círculo ( C2 ) do


hemisfério é paralelo a ( C1 ) , de forma que existem n círculos do hemisfério,
congruentes, tangentes entre si, a ( C1 ) e a ( C2 ) . Mostre que a razão K(n) entre os
cos 2 π n
raios de ( C2 ) e ( C1 ) é igual a: K ( n ) = .
1 + sen 2 π n
136) Sejam R, r1 , r2 e r3 os raios dos círculos de centro O, O1 , O2 e O3 ,
respectivamente, conforme a figura abaixo. Prove que: R = r1r2 + r1r3 + r2 r3 .

O1

R
O
R2
O2
O3

137) Sendo A um conjunto de quinze pontos de ¡ 2 tal que a distância de cada


ponto à origem é positiva e menor do que 1 e que quaisquer dois deles nunca sejam

EUREKA! N°31, 2010

59
Sociedade Brasileira de Matemática

colineares com a origem. Mostre que existe um triângulo com dois vértices em A e
1
um na origem cuja área é menor que .
4

138) Calcule o máximo divisor comum entre todos os números da forma x ⋅ y ⋅ z ,


onde ( x, y , z ) percorre todas as soluções inteiras da equação x 2 + y2 = z 2 com
x ⋅ y ⋅ z ≠ 0.

139) Determine todos os inteiros positivos x, y, z satisfazendo x3 − y3 = z 2 , onde y


é primo, z não é divisível por 3 e z não é divisível por y.

140) Mostre que 2903n − 803n − 464n + 261n é divisível por 1897, para todo n ∈ ¥.

141) Dado a ∈ {0,1, 2,3, 4,5,6,7,8,9} , seja X ≠ ∅ um conjunto finito de inteiros


positivos, tal que nenhum dos seus elementos possui o algarismo a em sua
1
representação decimal. Prove que ∑ < 80.
n∈ X n

Problema 133 e 134 proposto por Evandro Makiyama de Melo (São Paulo – SP) (foram
propostos originalmente na IX e na II Olimpíada Iberoamericana de Matemática Universitária,
respectivamente); 135 e 136 propostos por Ramilson Medeiros Pitombeira (Rio de Janeiro –
RJ); 137 proposto por Ítalo Dowell Lira Melo (Teresina – PI); 138 proposto por Luiz Felipe
Silva; 139 proposto por Adriano Carneiro (Caucaia – CE); 140 proposto por Wilson Carlos da
Silva Ramos (Belém – PA).

EUREKA! N°31, 2010

60
Sociedade Brasileira de Matemática

AGENDA OLÍMPICA
XXXII OLIMPÍADA BRASILEIRA DE MATEMÁTICA

NÍVEIS 1, 2 e 3
Primeira Fase – Sábado, 12 de junho de 2010
Segunda Fase – Sábado, 18 de setembro de 2010
Terceira Fase – Sábado, 16 de outubro de 2010 (níveis 1, 2 e 3)
Domingo, 17 de outubro de 2010 (níveis 2 e 3 - segundo dia de prova).

NÍVEL UNIVERSITÁRIO
Primeira Fase – Sábado, 18 de setembro de 2010
Segunda Fase – Sábado, 16 e Domingo, 17 de outubro de 2010

ASIAN PACIFIC MATH OLYMPIAD (APMO)


06 de março de 2010

XVI OLIMPÍADA DE MAIO


08 de maio de 2010

XXI OLIMPÍADA DE MATEMÁTICA DO CONE SUL


13 a 19 de junho de 2010
Águas de São Pedro, SP – Brasil

LI OLIMPÍADA INTERNACIONAL DE MATEMÁTICA


02 a 14 de julho de 2010
Astana, Cazaquistão

XVII OLIMPÍADA INTERNACIONAL DE MATEMÁTICA UNIVERSITÁRIA


24 a 30 de julho de 2010
Blagoevgrad, Bulgária

XXIV OLIMPÍADA IBEROAMERICANA DE MATEMÁTICA


17 a 27 de setembro de 2010
Paraguai

II COMPETIÇÃO IBEROAMERICANA INTERUNIVERSITÁRIA DE MATEMÁTICA


3 a 9 de outubro de 2010
Rio de Janeiro, Brasil

XIII OLIMPÍADA IBEROAMERICANA DE MATEMÁTICA UNIVERSITÁRIA

EUREKA! N°31, 2010

61
Sociedade Brasileira de Matemática

COORDENADORES REGIONAIS
Alberto Hassen Raad (UFJF) Juiz de Fora – MG
Américo López Gálvez (USP) Ribeirão Preto – SP
Andreia Goldani FACOS Osório – RS
Antonio Carlos Nogueira (UFU) Uberlândia – MG
Benedito Tadeu Vasconcelos Freire (UFRN) Natal – RN
Carmen Vieira Mathias (UNIFRA) Santa María – RS
Claus Haetinger (UNIVATES) Lajeado – RS
Cláudio de Lima Vidal (UNESP) S.J. do Rio Preto – SP
Denice Fontana Nisxota Menegais (UNIPAMPA) Bagé – RS
Disney Douglas Lima de Oliveira (UFAM) Manaus – AM
Edson Roberto Abe (Colégio Objetivo de Campinas) Campinas – SP
Edney Aparecido Santulo Jr. (UEM) Maringá – PR
Élio Mega (Grupo Educacional Etapa) São Paulo – SP
Eudes Antonio da Costa (Univ. Federal do Tocantins) Arraias – TO
Fábio Brochero Martínez (UFMG) Belo Horizonte – MG
Florêncio Ferreira Guimarães Filho (UFES) Vitória – ES
Francinildo Nobre Ferreira (UFSJ) São João del Rei – MG
Genildo Alves Marinho (Centro Educacional Leonardo Da Vinci) Taguatingua – DF
Graziela de Souza Sombrio (UNOCHAPECÓ) Chapecó – SC
Gilson Tumelero (UTFPR) Pato Branco – PR
Ivanilde Fernandes Saad (UC. Dom Bosco) Campo Grande – MS
João Benício de Melo Neto (UFPI) Teresina – PI
João Francisco Melo Libonati (Grupo Educacional Ideal) Belém – PA
Jose de Arimatéia Fernandes (UFPB) Campina Grande – PB
José Luiz Rosas Pinho (UFSC) Florianópolis – SC
José Vieira Alves (UFPB) Campina Grande – PB
José William Costa (Instituto Pueri Domus) Santo André – SP
Krerley Oliveira (UFAL) Maceió – AL
Licio Hernandes Bezerra (UFSC) Florianópolis – SC
Luciano G. Monteiro de Castro (Sistema Elite de Ensino) Rio de Janeiro – RJ
Luzinalva Miranda de Amorim (UFBA) Salvador – BA
Marcelo Rufino de Oliveira (Grupo Educacional Ideal) Belém – PA
Marcelo Mendes (Colégio Farias Brito, Pré-vestibular) Fortaleza – CE
Newman Simões (Cursinho CLQ Objetivo) Piracicaba – SP
Nivaldo Costa Muniz (UFMA) São Luis – MA
Nivaldo de Góes Grulha Jr. (USP – São Carlos) São Carlos – SP
Osnel Broche Cristo (UFLA) Lavras – MG
Uberlândio Batista Severo (UFPB)) João Pessoa – PB
Raul Cintra de Negreiros Ribeiro (Colégio Anglo) Atibaia – SP
Ronaldo Alves Garcia (UFGO) Goiânia – GO
Rogério da Silva Ignácio (Col. Aplic. da UFPE) Recife – PE
Reginaldo de Lima Pereira (Escola Técnica Federal de Roraima) Boa Vista – RR
Reinaldo Gen Ichiro Arakaki (UNIFESP) SJ dos Campos – SP
Ricardo Amorim (Centro Educacional Logos) Nova Iguaçu – RJ
Sérgio Cláudio Ramos (IM-UFRGS) Porto Alegre – RS
Seme Gebara Neto (UFMG) Belo Horizonte – MG
Tadeu Ferreira Gomes (UEBA) Juazeiro – BA
Tomás Menéndez Rodrigues (U. Federal de Rondônia) Porto Velho – RO
Valdenberg Araújo da Silva (U. Federal de Sergipe) São Cristovão – SE
Vânia Cristina Silva Rodrigues (U. Metodista de SP) S.B. do Campo – SP
Wagner Pereira Lopes (CEFET – GO) Jataí – GO

EUREKA! N°31, 2010

62
CONTEÚDO

XXXI OLIMPÍADA BRASILEIRA DE MATEMÁTICA


Problemas e soluções da Primeira Fase 2

XXXI OLIMPÍADA BRASILEIRA DE MATEMÁTICA


Problemas e soluções da Segunda Fase 14

XXXI OLIMPÍADA BRASILEIRA DE MATEMÁTICA


Problemas e soluções da Terceira Fase 34

XXXI OLIMPÍADA BRASILEIRA DE MATEMÁTICA


Problemas e soluções da Primeira Fase Nível Universitário 59

XXXI OLIMPÍADA BRASILEIRA DE MATEMÁTICA


Problemas e soluções da Segunda Fase Nível Universitário 65

XXXI OLIMPÍADA BRASILEIRA DE MATEMÁTICA


Premiados 75

AGENDA OLÍMPICA 81

COORDENADORES REGIONAIS 82
Sociedade Brasileira de Matemática

XXXI OLIMPÍADA BRASILEIRA DE MATEMÁTICA


Problemas e soluções da Primeira Fase

PROBLEMAS – NÍVEL 1

1 1 5
1. Se de um número é , quanto vale desse número?
8 5 8
1 1 8
A) B) C) 1 D) E) 2
8 5 5

2. Na figura, C é um ponto do segmento BD tal que


A
ACDE é um retângulo e ABCE é um paralelogramo
de área 22 cm2. Qual é a área de ABDE, em cm2?
E B
A) 28 B) 33 C) 36
D) 42 E) 44
C

D
3. Numa festa, o número de pessoas que dançam é igual a 25% do número de
pessoas que não dançam. Qual é a porcentagem do total de pessoas na festa que
não dançam?
A) 50% B) 60% C) 75% D) 80% E) 84%

4. De quantas maneiras dois casais podem sentar-se em quatro cadeiras em fila se


marido e mulher devem sentar-se em cadeiras vizinhas?
A) 2 B) 4 C) 8 D) 12 E) 24

5. Eliana tem 27 cubos iguais em tamanho, mas 4 são brancos e os demais, pretos.
Com esses 27 cubos, ela monta um cubo maior. No máximo, quantas faces
inteiramente pretas ela poderá obter?
A) 1 B) 2 C) 3 D) 4 E) 5

6. A figura abaixo é o mapa de um bairro: os pontos A, B, C e D são as casas e os


segmentos são as ruas. De quantas casas é possível fazer um caminho que passa
exatamente uma vez por cada uma das ruas? É permitido passar mais de uma vez
por uma mesma casa.

EUREKA! N°32, 2010

2
Sociedade Brasileira de Matemática

D B

C
A) 0 B) 1 C) 2 D) 3 E) 4

7. Se a = 240, b = 320 e c = 710, então:


A) c < b < a B) a < c < b C) b < a < c D) b < c < a E) c < a < b

8. Esmeralda lançou um dado dez vezes e obteve 57 como soma de todos os pontos
obtidos nesses lançamentos. No mínimo, quantas vezes saíram 6 pontos?
A) 5 B) 6 C) 7 D) 8 E) 9

9. Usando palitos de fósforos, podemos construir um


hexágono regular, formado por seis triângulos
equiláteros unitários, como mostra a figura. Juntando
mais palitos a esse hexágono, queremos obter outro
hexágono regular com o quádruplo da área, também
formado por triângulos equiláteros unitários.
Quantos palitos deverão ser acrescentados?
A) 12 B) 24 C) 30
D) 36 E) 48

10. Cinco cartas iguais têm um lado


branco e um lado preto. Elas se
encontram em fila com a face branca
para cima. Um movimento consiste em
escolher um único par de cartas vizinhas
e virá-las. No mínimo, quantos
movimentos são necessários para que as
cartas fiquem como na figura ao lado?
A) 2 B) 3 C) 4 D) 5 E) Não é possível
obter a configuração acima.

EUREKA! N°32, 2010

3
Sociedade Brasileira de Matemática

11. Uma barra de chocolate é dividida entre Nelly, Penha e Sônia. Sabendo que
2 1
Nelly ganha da barra, Penha ganha e Sônia ganha 70 gramas, o peso da
5 4
barra, em gramas, é:
A) 160 B) 200 C) 240 D) 280 E) 400

12. Numa fila para compra de ingressos para um jogo da seleção brasileira, havia
49 pessoas: 25 corintianos, 14 flamenguistas e 10 gremistas. Sabendo que cada
pessoa da fila torce para um único time, dois torcedores do mesmo time não estão
em posições consecutivas, podemos concluir que:
A) tal fila não existe.
B) algum dos torcedores das extremidades da fila é gremista.
C) algum dos torcedores das extremidades da fila é flamenguista.
D) algum flamenguista é vizinho de um gremista.
E) algum gremista é vizinho de dois corintianos.

13. Na figura, P é um ponto da reta CD. A região A B


cinza é comum ao retângulo ABCD e ao triângulo
ADP.
Se AB = 5 cm, AD = 8 cm e a área da região cinza Q
3
é da área do retângulo, quanto vale a distância
4
PC?
A) 1 cm B) 2 cm C) 3 cm D C P
D) 4 cm E) 5 cm

14. Numa pesquisa sobre o grau de escolaridade, obtiveram-se os resultados


expressos no gráfico abaixo:
Que fração do total de entrevistados representa o total de pessoas que terminaram
pelo menos o Ensino Fundamental?

EUREKA! N°32, 2010

4
Sociedade Brasileira de Matemática

1 3 5 11 16
A) B) C) D) E)
17 13 16 13 17

15. Um número natural A de três algarismos detona um número natural B de três


algarismos se cada algarismo de A é maior do que o algarismo correspondente de
B. Por exemplo, 876 detona 345; porém, 651 não detona 542 pois 1 < 2. Quantos
números de três algarismos detonam 314?
A) 120 B) 240 C) 360 D) 480 E) 600

16. O relógio de parede indica inicialmente meio-dia. 12


Os ponteiros das horas e dos minutos irão formar um
ângulo de 90 graus pela primeira vez:
A) entre 12h e 12h10min. 9 3
B) entre 12h10min e 12h15min.
C) entre 12h15min e 12h20min.
D) entre 12h20min e 12h25min. 6
E) após as 12h25min.

17. Eduardo escreveu todos os números de 1 a 2009 numa folha de papel. Com os
amigos, combinou o seguinte: cada um deles poderia apagar quantos números
quisesse e escrever, no fim da lista, o algarismo das unidades da soma dos números
apagados. Por exemplo, se alguém apagasse os números 28, 3, 6, deveria escrever
no fim da lista o número 7, pois 28 + 3 + 6 = 37. Após algum tempo, sobraram
somente dois números. Se um deles era 2000, qual dos números a seguir poderia
ser o outro?
A) 0 B) 1 C) 3 D) 5 E) 6

18. Uma folha de caderno de Carlos é um retângulo com dois lados (bordas)
amarelos de 24 cm e dois lados (bordas) vermelhos de 36 cm. Carlos pinta cada
ponto do retângulo na mesma cor do lado mais próximo desse ponto. Qual é a área
da região pintada de amarelo?
A) 144 cm2 B) 288 cm2 C) 364 cm2 D) 442 cm2 E) 524 cm2

19. O professor Piraldo aplicou uma prova de 6 questões para 18 estudantes. Cada
questão vale 0 ou 1 ponto; não há pontuações parciais. Após a prova, Piraldo
elaborou uma tabela como a seguinte para organizar as notas, em que cada linha
representa um estudante e cada coluna representa uma questão.

EUREKA! N°32, 2010

5
Sociedade Brasileira de Matemática

Questões→ 1 2 3 4 5 6
Estudantes

Arnaldo 0 1 1 1 1 0
Bernaldo 1 1 1 0 0 1
Cernaldo 0 1 1 1 1 0

Piraldo constatou que cada estudante acertou exatamente 4 questões e que cada
questão teve a mesma quantidade m de acertos. Qual é o valor de m?
A) 8 B) 9 C) 10 D) 12 E) 14

20. Alguns cubos foram empilhados


formando um bloco. As figuras ao lado
representam a vista da esquerda e da frente
desse bloco. Olhando o bloco de cima,
qual das figuras a seguir não pode ser
vista? vista da esquerda vista da frente
esquerda

esquerda

esquerda

esquerda

esquerda
A) B) C) D) E)

frente frente frente frente frente

PROBLEMAS – NÍVEL 2

1. Veja o Problema No. 1 do Nível 1.


2. Veja o Problema No. 9 do Nível 1.
3. Veja o problema No. 4 do Nível 1.
1 1
4. Se = 4, o valor de é:
x+5 x+6
1 1 2 4
A) B) C) D) E) 1
5 4 3 5
5. Veja o Problema No. 6 do Nível 1.

6. Os inteiros positivos m e n satisfazem 15m = 20n. Então é possível afirmar, com


certeza, que mn é múltiplo de:

EUREKA! N°32, 2010

6
Sociedade Brasileira de Matemática

A) 5 B) 10 C) 12 D) 15 E) 20
7. Veja o problema No. 15 do Nível 1.
8. Veja o Problema No. 11 do Nível 1.
9. Veja o Problema No. 8 do Nível 1.

10. Na figura abaixo, α = 18 e AB = AC = AD = AE. O valor do ângulo β é:

ααα

B
β
C E
D

A) 18o B) 36o C) 15o D) 20o E) 30o

11. Veja o Problema No. 10 do Nível 1.

12. Na figura abaixo, ABCDE é um pentágono regular, CDFG é um quadrado e


DFH é um triângulo equilátero. O valor do ângulo β é:

H F
β
G
D

E
C

A B

A) 30o B) 36o C) 39o D) 45o E) 60o

EUREKA! N°32, 2010

7
Sociedade Brasileira de Matemática

13. Veja o problema No. 12 do Nível 1.


14. Veja o Problema No. 13 do Nível 1.

15. A famosa Conjectura de Goldbach diz que todo número inteiro par maior que 2
pode ser escrito como a soma de dois números primos. Por exemplo, 18 pode ser
representado por 5 + 13 ou, ainda, por 7 + 11. Considerando todas as possíveis
representações de 126, qual a maior diferença entre os dois primos que a formam?
A) 112 B) 100 C) 92 D) 88 E) 80

16. Na figura ao lado, E é o ponto médio de


AB, F é o ponto médio de AC e BR = RS = SC. A
Se a área do triângulo ABC é 252, qual é a área
do pentágono AERSF?
A) 168 E F
B) 189
C) 200
D) 210
E) 220 B R S C

17. Quantos pares ordenados (x, y) de números reais satisfazem a equação


( x − y ) + ( x − y − 2)
2 2 2
= 0?
A) 0 B) 1 C) 2 D) 3 E) infinitos

18. Veja o Problema No. 19 do Nível 1.

19. Entre os inteiros positivos n + 4018, n = 1, 2,..., 20092 , quantos são quadrados
perfeitos?
A) 1945 B) 1946 C) 1947 D) 1948 E) 1949

20. Para cada número natural n, seja S n a soma dos dez primeiros múltiplos
positivos de n. Por exemplo, S 2 = 2 + 4 + 6 + 8 + 10 + 12 + 14 + 16 + 18 + 20.
Quanto é S1 + S 2 + S 3 + + S10 ?
A) 2925 B) 3025 C) 3125 D) 3225 E) 3325

EUREKA! N°32, 2010

8
Sociedade Brasileira de Matemática

21. Em uma folha quadriculada em que


cada quadrado tem lado 2cm, são
desenhados dois círculos como na
figura ao lado. A distância mínima entre
os dois círculos mede:
A) 3cm
B) 10 cm
C) ( )
10 + 3 cm

D) ( 10 − 2 ) cm

E) ( 10 − 3) cm

22. Quantos números naturais de 1 a 100, inclusive, podem ser escritos na forma de
potência a b , com a, b ∈ e a, b > 1?
A) 10 B) 12 C) 14 D) 16 E) 18

23. Veja o Problema No. 18 do Nível 1.

24. Os inteiros 0 < x < y < z < w < t são tais que w = z(x + y) e t = w(y + z). Sendo w
= 9, então t é igual a
A) 45 B) 54 C) 63 D) 72 E) 81

25. Veja o Problema No. 20 do Nível 1.

PROBLEMAS – NÍVEL 3

1. Veja o problema No. 15 do Nível 1.


2. Veja o problema No. 6 do Nível 2.

3. Se x2 = x + 3 então x3 é igual a:
A) x2 + 3 B) x + 4 C) 2x + 2 D) 4x + 3 E) x2 – 2

4. Na figura, o quadrado A’B’C’D’ foi obtido a partir de uma rotação no sentido


horário do quadrado ABCD de 25 graus em torno do ponto médio de AB. Qual é o
ângulo agudo, em graus, entre as retas AC e B’D’?

EUREKA! N°32, 2010

9
Sociedade Brasileira de Matemática

D’
D C

C’

A’
B
A
B’
A) 5 B) 25 C) 45 D) 65 E) 85

5. Um dos cinco números a seguir é divisor da soma dos outros quatro. Qual é esse
número?
A) 20 B) 24 C) 28 D) 38 E) 42

6. Sempre que Agilulfo volta para casa depois da escola com uma advertência, se
sua mãe está em casa, ela o coloca de castigo. Sabendo-se que ontem à tarde
Agilulfo não foi colocado de castigo, qual das seguintes afirmações é certamente
verdadeira?
A) Agilulfo recebeu advertência ontem.
B) Agilulfo não recebeu advertência ontem.
C) Ontem à tarde a sua mãe estava em casa.
D) Ontem à tarde a sua mãe não estava em casa.
E) Nenhuma das afirmações acima é certamente verdadeira.

7. Qual é o menor valor de n > 1 para o qual é possível colocar n peças sobre um
tabuleiro n × n de modo que não haja duas peças sobre a mesma linha, mesma
coluna ou mesma diagonal? As figuras a seguir mostram pares de peças na mesma
linha, na mesma coluna e na mesma diagonal em diversos tabuleiros.

• •
• •

A) 3 B) 4 C) 5 D) 6 E) 7

EUREKA! N°32, 2010

10
Sociedade Brasileira de Matemática

8. Na figura a seguir, ABCD é um quadrado de lado L


4, K pertence ao lado AD, L pertence ao lado AB, M A B
pertence ao lado BC e KLM é um triângulo
retângulo isósceles, sendo L o ângulo reto. Então a K
área do quadrilátero CDKM é igual a
A) 6
B) 8 M
C) 10
D) 12 D C
E) 14

9. Veja o Problema No. 6 do Nível 1.


10. Veja o Problema No. 16 do Nível 1.

11. Considere o número inteiro positivo n tal que o número de divisores positivos
do dobro de n é igual ao dobro do número de divisores positivos de n. Podemos
concluir que n é
A) um número primo B) um número par C) um número ímpar
D) um quadrado perfeito E) potência inteira de 2

12. Esmeralda tem cinco livros sobre heráldica em uma estante. No final de
semana, ela limpou a estante e, ao recolocar os livros, colocou dois deles no lugar
onde estavam antes e os demais em lugares diferentes de onde estavam. De quantas
maneiras ela pode ter feito isso?
A) 20 B) 25 C) 30 D) 34 E) 45

13. Veja o Problema No. 19 do Nível 1.

14. Seja f : → uma função tal que f(0) = 0, f(1) = 1, f(2) = 2 e f(x + 12) = f(x
+ 21) = f(x) para todo x ∈ . Então f(2009) é:
A) 0 B) 1 C) 2 D) 3 E) 2009

EUREKA! N°32, 2010

11
Sociedade Brasileira de Matemática

15. Na figura, CD = BC, ∠BAD = 72 , AB é o D


diâmetro e O o centro do semicírculo.
Determine a medida do ângulo ∠DEC.
A) 36o
B) 42o C
C) 54o E
D) 63o
E) 18o
O
A B

16. Sabe-se que 2x2 – 12xy + ky2 ≥ 0 para todos x, y reais. O menor valor real de k
é
A) 9 B) 16 C) 18 D) 27 E) 36

17. Veja o problema No. 15 do Nível 2.

18. Um subconjunto de {1,2,3,…,20} é superpar quando quaisquer dois de seus


elementos têm produto par. A maior quantidade de elementos de um subconjunto
superpar é:
A) 3 B) 4 C) 6 D) 7 E) 11

19. Veja o problema No. 20 do Nível 2.

20. Os círculos C1 e C2, de raios 3 e R


4, respectivamente, são tangentes N
externamente em T. As tangentes P
externas comuns tocam C1 em P e
Q e C2 em R e S. A tangente interna C1 T
comum em T corta as tangentes
externas nos pontos M e N, como
mostra a figura. A razão entre as Q C2
áreas dos quadriláteros MNPQ e M
S
MNRS é:
1 9 3 3 13
A) B) C) D) E)
7 16 4 2 15

EUREKA! N°32, 2010

12
Sociedade Brasileira de Matemática

21. Dois carros deixam simultaneamente as cidades A e B indo de uma cidade em


direção à outra, com velocidades constantes, e em sentidos opostos. As duas
cidades são ligadas por uma estrada reta. Quando o carro mais rápido chega ao
ponto médio M de AB, a distância entre os dois carros é de 96 km. Quando o carro
mais lento chega ao ponto M, os carros estão a 160 km um do outro. Qual a
distância, em km, entre as duas cidades?
A) 320 B) 420 C) 480 D) 520 E) 560

22. Seja N = 8 8 , em que aparecem 2009 números 8. Agilulfo ficou de castigo:


ele deve escrever a soma dos dígitos de N, obtendo um número M; em seguida,
deve calcular a soma dos dígitos de M; e deve repetir o procedimento até obter um
número de um único dígito. Vamos ajudar Agilulfo: esse dígito é
A) 1 B) 2 C) 3 D) 7 E) 8

23. Veja o Problema No. 20 do Nível 1.


24. Veja o Problema No. 18 do Nível 1.

25. Os lados de um triângulo formam uma progressão aritmética de razão t. Então a


distância entre o incentro e o baricentro deste triângulo é:
t t 2t
A) t B) C) D) E) faltam dados
2 3 3
GABARITO
NÍVEL 1 – (6º. ou 7º. Anos)
1) C 6) C 11) B 16) C
2) B 7) A 12) E 17) D
3) D 8) C 13) E 18) B
4) C 9) C 14) E 19) D
5) D 10) B 15) B 20) C

NÍVEL 2 – (8º. ou 9º. Anos)


1) C 6) C 11) B 16) A 21) E
2) C 7) B 12) C 17) C 22) B
3) C 8) B 13) E 18) D 23) B
4) D 9) C 14) E 19) B 24) A
5) C 10) A 15) B 20) B 25) C

NÍVEL 3 – (Ensino Médio)


1) B 6) E 11) C 16) C 21) C
2) C 7) B 12) A 17) B 22) A
3) D 8) B 13) D 18) E 23) C
4) D 9) C 14) C 19) B 24) B
5) D 10) E 15) C 20) E 25) C

EUREKA! N°32, 2010

13
Sociedade Brasileira de Matemática

XXXI OLIMPÍADA BRASILEIRA DE MATEMÁTICA


Problemas e soluções da Segunda Fase

PROBLEMAS – NÍVEL 1 – PARTE A


(Cada problema vale 5 pontos)

01. A figura ao lado mostra


castelos de cartas de 1, 2 e 3
andares. Para montar esses
castelos, foram usadas 2, 7 e 15
cartas, respectivamente.
Quantas cartas serão necessárias
para montar um castelo de 5
andares?

02. Numa classe do 6º ano, de cada 11 estudantes, 4 são meninas. Se há 15 meninos


a mais que meninas, quantos alunos há na classe?

03. Num curso com duração de cinco dias, a frequência dos alunos foi registrada na
tabela abaixo:
Dia de aula
1º dia 2º dia 3º dia 4º dia 5º dia
Quantidade de alunos presentes
271 296 325 380 168

Cada aluno faltou exatamente dois dias. No dia de menor frequência, de quantos
por cento foi o total de faltas?

04. Mariazinha deseja cobrir o tampo de uma


mesa retangular de 88 cm por 95 cm colando
quadrados de cartolina de lado 10 cm, a partir de
um canto, como mostrado na figura. Ela cola os
quadrados sem buracos nem superposições, até
chegar às bordas opostas. Aí, em vez de cortar as
folhas para não ultrapassar as bordas, ela as
sobrepõe, formando regiões retangulares com
duas folhas de espessura (região cinza) e uma
pequena região retangular com quatro folhas de espessura (região preta). Qual é
a área da região coberta por quatro folhas?

EUREKA! N°32, 2010

14
Sociedade Brasileira de Matemática

05. O número 200920092009... 2009 tem 2008 algarismos. Qual é a menor


quantidade de algarismos que devem ser apagados, de modo que a soma dos
algarismos que restarem seja 2008?

06. Dizemos que dois ou mais números, com a mesma quantidade de algarismos,
são membros da mesma família, quando todos possuem pelo menos um algarismo
comum. Por exemplo, os números 72, 32, 25 e 22 pertencem à mesma família, pois
todos possuem o algarismo 2, enquanto que os números 123, 245 e 568 não
pertencem à mesma família, pois não há um algarismo que apareça nesses três
números. Qual é a maior quantidade de membros de uma família, cujos elementos
têm três algarismos?

PROBLEMAS – NÍVEL 1 – PARTE B


(Cada problema vale 10 pontos)

PROBLEMA 1
Carlinhos tem folhas iguais na
forma de triângulos retângulos de
lados 6 cm, 8 cm e 10 cm. Em
cada triângulo, o ângulo
assinalado opõe-se ao menor lado.
Fazendo coincidir lados iguais
desses triângulos sobre uma mesa,
sem superpor as folhas, ele
desenha o contorno de cada figura
obtida (linha grossa), como nos
exemplos ao lado. O perímetro de
uma figura é o comprimento do
seu contorno.

a) Qual é a diferença entre os perímetros das figuras 1 e 2 do exemplo?


b) Com figuras de três triângulos, qual é o maior perímetro que pode ser obtido?

PROBLEMA 2
Esmeralda ia multiplicar um número A de três algarismos por outro número B de
dois algarismos, mas na hora de multiplicar inverteu a ordem dos dígitos de B e
obteve um resultado 2034 unidades maior.
a) Qual era o número A, se os dígitos de B eram consecutivos?
b) Qual seria o número A, se os dígitos de B não fossem consecutivos?

EUREKA! N°32, 2010

15
Sociedade Brasileira de Matemática

PROBLEMA 3
Um campeonato de xadrez de 7 rodadas, com 4 jogos por rodada, tem 8
participantes, cujas pontuações por jogo são as usuais: um ponto por vitória, meio
ponto por empate e nenhum ponto por derrota. Cada par de jogadores se enfrenta
exatamente uma vez.

a) Ao término da terceira rodada, é possível que um grupo de jogadores esteja em


primeiro lugar e o restante dos jogadores esteja em segundo lugar? Explique por
meio de um exemplo.

b) Ao término da terceira rodada, é possível que todos os jogadores tenham


pontuações diferentes? Explique.

PROBLEMAS – NÍVEL 2 – PARTE A


(Cada problema vale 5 pontos)

01. Esmeralda tem uma garrafa com 9 litros de uma mistura que tem 50% de álcool
e 50% de água. Ela quer colocar água na garrafa de tal forma que apenas 30% da
mistura seja de álcool. Quantos litros de água ela irá colocar?

02. Se a, b, c e d são, em alguma ordem, 1, 2, 3 e 4. Qual é o maior valor possível


de
ab + bc + cd + da?

03. Dizemos que dois ou mais números, com a mesma quantidade de algarismos,
são membros da mesma família, quando todos possuem pelo menos um algarismo
em comum. Por exemplo, os números 32, 25 e 22 pertencem à mesma família,
enquanto que 123, 245 e 568 não pertencem à mesma família, pois 123 e 568 não
pertencem à mesma família. Qual é a maior quantidade de membros de uma
família, cujos elementos têm três algarismos?

04. Determine a quantidade de inteiros de dois algarismos que são divisíveis pelos
seus algarismos.

05. Na figura abaixo, ABCD e EFGH são quadrados de lado 48 cm. Sabendo que A
é o ponto médio de EF e G é o ponto médio de DC, determine a área destacada em
cm2.

EUREKA! N°32, 2010

16
Sociedade Brasileira de Matemática

A L B

H
F
K

D G C

PROBLEMAS – NÍVEL 2 – PARTE B


(Cada problema vale 10 pontos)

PROBLEMA 1
Sejam m e n dois inteiros positivos primos entre si. O Teorema Chinês dos Restos
afirma que, dados inteiros i e j com 0 ≤ i < m e 0 ≤ j < n, existe exatamente um
inteiro a, com 0 ≤ a < m⋅n, tal que o resto da divisão de a por m é igual a i e o resto
da divisão de a por n é igual a j. Por exemplo, para m = 3 e n = 7, temos que 19 é o
único número que deixa restos 1 e 5 quando dividido por 3 e 7, respectivamente.
Assim, na tabela a seguir, cada número de 0 a 20 aparecerá exatamente uma vez.

Restos
por 7
0 1 2 3 4 5 6
Restos
por 3
0
1 19
2

Qual a soma dos números das casas destacadas?

PROBLEMA 2
Observe:
(x – r)(x – s) = x2 – (r + s)x + rs

Assim, substituindo x por r e por s, obtemos


r 2 − (r + s ) r + rs = 0 a ( r n + 2 − (r + s ) r n +1 + rs ⋅ r n ) = 0

s 2 − (r + s ) s + rs = 0 b( s n + 2 − ( r + s ) s n +1 + rs ⋅ s n ) = 0

EUREKA! N°32, 2010

17
Sociedade Brasileira de Matemática

Somando as duas equações e sendo S n = a ⋅ r n + b ⋅ s n , verifica-se que


S n + 2 = (r + s ) S n +1 − rsS n
Dados S1 = ar + bs = 1 , S 2 = ar 2 + bs 2 = 2 , S 3 = ar 3 + bs 3 = 5 e
4 4 5 5
S 4 = ar + bs = 6 , determine S 5 = ar + bs .

PROBLEMA 3
Seja N é o ponto do lado AC do triângulo ABC tal que AN = 2 NC e M o ponto
do lado AB tal que MN é perpendicular a AB . Sabendo que AC = 12 cm e que o
baricentro G do triângulo ABC pertence ao segmento MN, determine o
comprimento do segmento BG.
OBS: Baricentro é o ponto de interseção das medianas do triângulo.

PROBLEMA 4
Um campeonato de xadrez de 7 rodadas, com 4 jogos por rodada, tem 8
participantes, cujas pontuações por jogo são as usuais: um ponto por vitória, meio
ponto por empate e nenhum ponto por derrota. Cada par de jogadores se enfrenta
exatamente uma vez.

a) Ao término da terceira rodada, é possível que todos os jogadores tenham


pontuações distintas?

b) Se no final do campeonato todos os jogadores têm pontuações distintas qual o


menor número possível de pontos obtidos pelo primeiro colocado?

PROBLEMAS – NÍVEL 3 – PARTE A


(Cada problema vale 5 pontos)

01. Veja o problema No. 1 do Nível 2.

02. No triângulo retângulo ABC, ∠A = 90º, AB = 5cm e BC = 9cm. Se I é o incentro


de ABC, determine o comprimento do segmento CI.

03. Seja c a maior constante real para a qual

x2 + 3y2 ≥ c⋅(x2 + xy + 4y2).


para todos x, y reais.

Determine o inteiro mais próximo de 2009⋅c.

EUREKA! N°32, 2010

18
Sociedade Brasileira de Matemática

04. No programa de auditório Toto Bola, o apresentador Ciço Magallanes dispõe de


duas caixas idênticas. Um voluntário da platéia é chamado a participar da seguinte
brincadeira: ele recebe dez bolas verdes e dez bolas vermelhas e as distribui nas
duas caixas, sem que o apresentador veja, e de modo que em cada caixa haja pelo
menos uma bola. Em seguida, o apresentador escolhe uma das caixas e retira uma
bola. Se a bola for VERDE, o voluntário ganha um carro. Se for VERMELHA, ele
ganha uma banana. A máxima probabilidade que o voluntário tem de ganhar um
m
carro é igual a , em que m e n são inteiros positivos primos entre si. Determine o
n
valor de m + n.

05. Determine o maior inteiro n menor que 10000 tal que 2n + n seja divisível por 5.

PROBLEMAS – NÍVEL 3 – PARTE B


(Cada problema vale 10 pontos)

PROBLEMA 1
Determine a quantidade de números n = a1a2a3a4a5a6, de seis algarismos distintos,
que podemos formar utilizando os algarismos 1, 2, 3, 4, 5, 6, 7, 8, 9 de modo que as
seguintes condições sejam satisfeitas simultaneamente:
i) a1 + a6 = a2 + a5 = a3 + a4;
ii) n é divisível por 9.

PROBLEMA 2
Encontre todos os inteiros a > 0 e b > 0 tais que
4 ⋅ 3 a = 11 + 5 b
PROBLEMA 3
Para cada inteiro positivo n, seja An = {x ∈ R+ ; x ⋅ x  = n} , em que R+ é o conjunto
dos reais positivos e x  é o maior inteiro menor ou igual a x.
Determine a quantidade de elementos do conjunto
A1 ∪ A2 ∪ A3 ∪ ... ∪ A2009.

PROBLEMA 4
No triângulo ABC, temos ∠A = 120° e BC = 12 cm. A circunferência inscrita em
ABC tangencia os lados AB e AC, respectivamente, nos pontos D e E. Sejam K e L
os pontos onde a reta DE intersecta a circunferência de diâmetro BC. Determine a
distância entre os pontos médios dos segmentos BC e KL.

EUREKA! N°32, 2010

19
Sociedade Brasileira de Matemática

SOLUÇÕES NÍVEL 1 – SEGUNDA FASE – PARTE A

Problema 01 02 03 04 05 06
Resposta 40 55 65 10 392 252

01. Para fazer um novo andar num castelo já construído, precisamos de três cartas
para cada andar anterior mais duas para o topo. Assim, a partir do castelo de 3
andares, para fazer o de 4 andares, precisamos de mais 3 × 3 + 2 = 11 cartas, num
total de 15 + 11 = 26 cartas. Portanto, para fazer o castelo de 5 andares, precisamos
de 26 + 4 × 3 + 2 = 40 cartas.

Solução alternativa:
Para acrescentarmos um quarto andar a um castelo de 3 andares, precisamos de 3
cartas para separar a base dos demais andares e 4 pares de cartas para a base,
totalizando 3 + 2.4 = 11 cartas a mais. Veja a figura a seguir:

Analogamente, para acrescentarmos um quinto andar a um castelo de 4 andares,


precisamos de 4 cartas para separar a base dos demais andares e 5 pares de cartas
para a base, totalizando 4 + 2.5 = 14 cartas a mais. Assim, para montar um castelo
de 5 andares, precisamos de 15 + 11 + 14 = 40 cartas.
Observação: De fato, o acréscimo de um n-ésimo andar necessita de n − 1 cartas
para apoiar a base anterior, e n pares de cartas para a nova base. Portanto, são
acrescentadas n − 1 + 2 ⋅ n = 3n − 1 cartas por andar.

02. Seja x a quantidade de meninas. Assim, a quantidade de meninos é x + 15 e a


quantidade total de alunos será 2 x + 15 . Fazendo a proporção, temos:
x 4
=
2 x + 15 11
Resolvendo a equação, obtemos x = 20 .

EUREKA! N°32, 2010

20
Sociedade Brasileira de Matemática

03. Se cada aluno compareceu exatamente três dias, o número total de alunos do
271 + 296 + 325 + 380 + 168 1440
curso é = = 480 . A menor frequência foi de
3 3
168 alunos, num total de 480 – 168 = 312 faltas. Portanto, o percentual de faltas
312
nesse dia foi = 0, 65 = 65% .
480

04. Na direção da medida 88 cm, Mariazinha irá usar 9 folhas e na direção da


medida 95 cm, irá usar 10 folhas. Mariazinha começa colando as folhas sem
sobreposição da esquerda para a direita e de cima para baixo (como na figura) e ao
chegar às bordas direita e inferior, desloca, respectivamente, 2 cm à esquerda e 5
cm para cima (as regiões em cinza representam as sobreposições de 2 folhas). A
região retangular preta é a intersecção dessas duas faixas de sobreposição, logo é
coberta por 4 folhas. Sua área é de 10 cm2.

05. No número existem 502 algarismos 2 e 502 algarismos 9. Para retirar a menor
quantidade possível de algarismos, devemos tentar deixar a maior quantidade
possível de algarismos 2. Porém, a soma de todos os algarismos 2 é 1004. Ainda
falta 1004 para completar a soma 2008. Como 1004 = 9 × 111 + 5 devemos deixar
pelo menos 111 algarismos 9. Porém, é impossível deixar exatamente 111
algarismos 9. Se deixarmos 112 algarismos 9, devemos deixar 500 algarismos 2.
Portanto, deve-se retirar no mínimo 2 + 390 = 392 algarismos.

06. Como todos os membros de uma família devem possuir pelo menos um
algarismo comum, a maior quantidade de membros de uma família cujos elementos
têm três algarismos é igual ao número de elementos de qualquer conjunto formado

EUREKA! N°32, 2010

21
Sociedade Brasileira de Matemática

por todos os números de três algarismos que possuem um determinado algarismo


em sua representação decimal. O algarismo das centenas não pode ser zero. Vamos
contar então todos os números que têm um determinado algarismo a, não nulo, pois
há mais deles. Há 9 × 9 = 81 números em que a aparece uma única vez, como
algarismo das centenas. Há 8 × 9 = 72 números em que a aparece uma única vez,
como algarismo das dezenas (lembre-se que o das centenas não pode ser 0) e há 72
números em que o a aparece uma única vez, como algarismo das unidades.
Há 9 números com a na centena e na dezena, menos na unidade, 9 números com a
na centena e na unidade, menos na dezena e 8 números com a na dezena e na
unidade, menos na centena e um único número formado inteiramente de a. A
quantidade total de números em que figura o algarismo não nulo a é 81 + 72 + 72 +
9 + 9 + 8 + 1 = 252.

Solução alternativa:
Para simplificar o raciocínio, vamos contar quantos números de três algarismos não
contêm um algarismo a, não nulo, fixado. Assim, nessa situação, existem 8
escolhas para o algarismo das centenas (não pode ser 0 ou a), 9 escolhas para o
algarismo das dezenas (não pode ser a), e 9 escolhas para os algarismos das
unidades (não pode ser a). Logo, pelo Princípio Fundamental da Contagem, há
8.9.9 = 648 números que não possuem o algarismo a. Assim, como existem 900
números de 3 algarismos, há 900 – 648 = 252 números que possuem o algarismo a
( a ≠ 0 ). Essa é a maior quantidade de membros que uma família pode ter.

Observação:
Podemos verificar que a família formada por todos os números de três algarismos
que possuem o zero tem 900 − 9 ⋅ 9 ⋅ 9 = 171 membros.

SOLUÇÕES NÍVEL 1 – SEGUNDA FASE – PARTE B


PROBLEMA 1

EUREKA! N°32, 2010

22
Sociedade Brasileira de Matemática

a) O perímetro da primeira figura é 8 + 6 + 6 + 10 + 6 = 36 e da segunda figura é


10 + 8 + 6 + 8 + 8 = 40 . Portanto a diferença é 40 − 36 = 4 .

b) A figura de maior perímetro é obtida quando fazemos coincidir os dois menores


lados de cada um dos triângulos. Isso é mostrado na figura ao lado cujo perímetro é
10 + 10 + 10 + 8 + 6 = 44 (há outras com o mesmo perímetro).

PROBLEMA 2
Seja A o número de três dígitos e B = 10 x + y o número de dois dígitos. Portanto,
ao trocar a ordem dos dígitos de B , obtemos o número 10 y + x . Montando a
equação segundo as condições do problema, temos:
A(10 x + y ) − A(10 y + x) = 9 A( x − y ) = 2034
Com isso,
A( x − y ) = 226 = 2 ⋅113
Daí, se x, y são consecutivos, A = 226 , caso contrário A = 113 .

PROBLEMA 3
a) Sim, é possível. Por exemplo (há outros), podem existir quatro jogadores com
pontuação 2 e outros quatro com pontuação 1. Fazendo A, B, C, D o primeiro
grupo e E, F, G, H o segundo grupo, temos:

1ª Rodada
A vence E
B vence F
C vence G
D vence H

2ª Rodada
A empata com B
E empata com F
C empata com D
G empata com H

3ª Rodada
A empata com F
B empata com E
C empata com H
D empata com G

EUREKA! N°32, 2010

23
Sociedade Brasileira de Matemática

b) Após três rodadas, um jogador pode acumular no máximo 3 pontos. Como as


1
pontuações são múltiplos inteiros de , os possíveis valores de pontuação após a
2
1 3 5
terceira rodada são: 0, ,1, , 2, ,3 (7 resultados possíveis)
2 2 2

Como existem 8 jogadores e apenas 7 possibilidades, dois jogadores terão


pontuações iguais.

SOLUÇÕES NÍVEL 2 – SEGUNDA FASE – PARTE A

Problema 01 02 03 04 05
Resposta 06 25 252 14 1704

01. Inicialmente temos 4,5 litros de água e 4,5 litros de álcool. Colocados x litros de
30
água, para termos 30% de álcool na mistura, basta que (9 + x) = 4,5 , então
100
x = 6.

02. É fácil ver que ab + bc + cd + da = b ( a + c ) + b ( c + a ) = ( a + c )( b + d ) .


Suponha sem perda de generalidade que a = 1. . Com isso, {a,c} = {1, 2} ,{1,3} ou
{1,4} e conseqüentemente {b,d } = {3,4} ,{2,4} ou {2,3} , respectivamente. Assim
os possíveis valores do produto são 21, 24 e 25 e o máximo é 25.

03. O algarismo das centenas não pode ser zero. Vamos contar então todos os
números que têm um determinado algarismo x, não nulo, pois há mais deles. Há
9 × 9 = 81 números em que x aparece uma única vez, como algarismo das
centenas. Há 8 × 9 = 72 números em que x aparece uma única vez, como algarismo
das dezenas (lembre-se que o das centenas não pode ser 0) e há 72 números em que
o x aparece uma única vez, como algarismo das unidades. Há 9 números com x na
centena e na dezena, menos na unidade, 9 números com x na centena e na unidade,
menos na dezena e 8 números com x na dezena e na unidade, menos na centena e
um único número formado inteiramente de x. A quantidade total de números em
que figura o algarismo não nulo x é 81 + 72 + 72 + 9 + 9 + 8 + 1 = 252

EUREKA! N°32, 2010

24
Sociedade Brasileira de Matemática

04. Seja n = 10 A + B o número de dois dígitos. Se A divide n , então A divide


B . Se A > 5 , então B = A , pois B não pode ser 0 e B < 10 < 2 A .

Listemos as possibilidades:

Se A = 1 então AB pode ser 11, 12, 15.


Se A = 2 , então AB pode ser 22, 24.
Se A = 3 , então AB pode ser 33, 36.
Se A = 4 , então AB pode ser 44, 48.
Se A = 5 , então AB pode ser 55.
Se A = 6 , então AB pode ser 66.
Se A = 7 , então AB pode ser 77.
Se A = 8 , então AB pode ser 88.
Se A = 9 , então AB pode ser 99.

Logo, o total de números é 3 + 2 + 2 + 2 + 5 = 14.

05. Sejam K a interseção dos lados AD e FG , e L a interseção dos lados AB e


EH . Por simetria, veja que KD = KF e AK = KG . Considere FK = x . Dessa
forma, AK = 48 − x . Usando teorema de Pitágoras no triângulo AFK , temos:
242 + x 2 = ( 48 − x ) .
2

Que nos dá x = 18 .
Agora, veja que os triângulos AFK e ALE são semelhantes. Portanto,
AE EL
= .
FK AF
Assim, EL = 32 .
Para achar a área procurada, basta subtrair a área do quadrado EFGH das áreas
dos triângulos AFK e AEL . Portanto a área será 1704.
E

A L B

H
F
K

D G C

EUREKA! N°32, 2010

25
Sociedade Brasileira de Matemática

SOLUÇÕES NÍVEL 2 – SEGUNDA FASE – PARTE B

PROBLEMA 1:

0 1 2 3 4 5 6
0 0 15 9 3 18 12 6
1 7 1 16 10 4 19 13
2 14 8 2 17 11 5 20

A resposta é 15 + 8 + 10 + 11 + 12 + 13 = 69.

PROBLEMA 2:
S 4 = (r + s ) S 3 − rsS 2 = (r + s ).5 − rs.2 = 5r + 5s − 2rs = 6
S 3 = (r + s ) S 2 − rsS1 = (r + s ).2 − rs.1 = 2r + 2 s − rs = 5
Com isso, encontramos que r + s = −4 e rs = −13 .
Daí, S5 = (r + s)S 4 − rsS3 = −24 + 65 = 41.

PROBLEMA 3:

Se BP é uma mediana do triângulo então AP = CP = 6 e PN = 2. Como G é o


PG 1 PN 1
baricentro do triângulo então = e = , assim, pela recíproca do
GB 2 NC 2
teorema de Tales, GN é paralelo a BC e ∠B = 90o . Como o triângulo ABC é
retângulo então AP = CP = BP = 6. Com isso, BG = 4 e GP = 2.

EUREKA! N°32, 2010

26
Sociedade Brasileira de Matemática

PROBLEMA 4:
a) Após três rodadas, um jogador pode acumular no máximo 3 pontos. Como as
pontuações são múltiplos inteiros de ½ , os possíveis valores de pontuação após a
terceira rodada são:
0,1/2, 1, 3/2, 2, 5/2, 3
Como existem 8 jogadores e apenas 7 possibilidades, dois jogadores terão
pontuações iguais.

b) Se k é a pontuação do primeiro colocado e todas as pontuações são distintas, a


soma das pontuações dos oito jogadores será no máximo:

 1  3  5  7
k +  k −  + ( k − 1) +  k −  + ( k − 2 ) +  k −  + ( k − 3) +  k −  = 8k − 14
 2  2  2  2
Como foram disputados exatamente 4 × 7 = 28 pontos, temos

8k − 14 ≥ 28
1 1
Logo, k ≥ 5 + pois as pontuações são múltiplos inteiros de . Basta mostrarmos
2 2
um exemplo onde este valor é atingido.

Na tabela abaixo, marcamos na interseção da linha Ai com a coluna A j o número


de pontos que Ai ganhou na partida disputada contra Aj.
A1 A2 A3 A4 A5 A6 A7 A8 Total
A1 X 1 1 1 1 1 ½ 0 5+½

A2 0 x 1 1 1 1 1 0 5

A3 0 0 x 1 1 1 1 ½ 4+½

A4 0 0 0 X 1 1 1 1 4

A5 0 0 0 0 X 0 0 0 0

A6 0 0 0 0 1 X ½ 1 2+½

A7 ½ 0 0 0 1 ½ x 1 3

A8 1 1 ½ 0 1 0 0 x 3+½

EUREKA! N°32, 2010

27
Sociedade Brasileira de Matemática

SOLUÇÕES NÍVEL 3 – SEGUNDA FASE – PARTE A

Problema 01 02 03 04 05
Resposta 0069 0006 1339 0033 9993

01. [RESPOSTA: 0069]

SOLUÇÃO:
0 1 2 3 4 5 6
0 0 15 9 3 18 12 6
1 7 1 16 10 4 19 13
2 14 8 2 17 11 5 20

A resposta é 15 + 8 + 10 + 11 + 12 + 13 = 69.

02. [RESPOSTA: 0006]

SOLUÇÃO: Pelo teorema de Pitágoras, é imediato que

AC 2 = 9 2 − 5 2 = 56 ∴ AC = 2 14 .

Seja r o raio do círculo inscrito, como mostrado na figura abaixo.


B
5–r
9
5–r

5 I 2 14 − r

r r

A r C
2 14 − r
Como os comprimentos das tangentes ao círculo inscrito partindo de cada vértice
são iguais, ficamos com a equação
(5 – r) + (2 14 − r ) = 9,
de onde obtemos r = 14 − 2 . Novamente pelo teorema de Pitágoras, obtemos:

EUREKA! N°32, 2010

28
Sociedade Brasileira de Matemática

CI 2 = r 2 + ( 2 14 − r ) 2 = ( 14 − 2) 2 + ( 14 + 2) 2 = 36 ∴ CI = 6 .

03. [RESPOSTA: 1339]

SOLUÇÃO: Fazendo x = t⋅y, a equação inicial reduz-se a

t2 + 3 ≥ c⋅(t2 + t + 4).

Logo, devemos ter (c – 1)t2 + ct + (4c – 3) ≤ 0, para todo t real. Para isto, devemos
ter c – 1 < 0 e o discriminante ∆ = c2 − 4⋅(c – 1)⋅(4c – 3) ≤ 0.

2
Da última inequação, obtemos −15c2 + 28c – 12 ≤ 0, cuja solução é c ≤ ou
3
6
c≥ . Como c < 1, o maior valor possível de c é 2/3. Daí, 2009⋅c = 1339,333... .
5

04. [RESPOSTA: 0033]

SOLUÇÃO: Seja P(a, b) a probabilidade de o voluntário ganhar o carro no caso em


que ele tenha colocado a bolas VERDES e b bolas VERMELHAS na caixa 1.
Então, necessariamente haverá (10 – a) bolas VERDES e (10 – b) bolas
VERMELHAS na caixa 2. Segue que

1 a 1 10 − a
P ( a, b ) = ⋅ + ⋅ .
2 a + b 2 20 − a − b

Podemos supor, sem perda de generalidade, que a + b ≤ 10, já que as caixas são
idênticas. Suponha, ainda, que haja alguma bola VERMELHA na caixa 1. Vejamos
o que acontece com essa probabilidade se transferirmos uma bola VERDE da caixa
2 para a caixa 1 e uma bola VERMELHA da caixa 1 para a caixa 2. Ficamos com

1 a +1 1 9−a
P( a + 1, b − 1) = ⋅ + ⋅ .
2 a + b 2 20 − a − b
Dessa forma,
1  1 1 
P( a + 1, b − 1) − P( a, b) = ⋅ − ≥0,
2  a + b 20 − a − b 
pois a + b ≤ 10.

EUREKA! N°32, 2010

29
Sociedade Brasileira de Matemática

Assim, o voluntário sabe que, enquanto houver bola VERMELHA na caixa que
contém menos bolas, a probabilidade pode ser aumentada, bastando, para isto, que
ele troque uma das bolas VERMELHAS desta caixa com uma VERDE da outra.
Por isso, para maximizarmos a probabilidade, basta considerarmos o caso em que a
caixa 1 contém apenas bolas VERDES e a caixa 2 contém o restante das bolas.
Teremos
1 1 10 − a
P( a,0) = + ⋅
2 2 20 − a
.
1  10 − a  1  10  5
= 1 +  = 2 −  =1− ⋅
2 20 − a  2  20 − a  20 − a

Logo, a probabilidade será máxima quando a for mínimo. Como em cada caixa
deve haver pelo menos uma bola, devemos ter a = 1. Neste caso, a probabilidade é:
5 14
P(1,0) = 1 − = .
19 19
Segue que m = 14, n = 19 e m + n = 33.

05. [RESPOSTA: 9993]

SOLUÇÃO: Vamos analisar os restos das divisões de 2n e n por 5.

1 2 3 4 5 6 7 8 9 10 11 12 13 14 15 16 17 18 19 20
n 1 2 3 4 0 1 2 3 4 0 1 2 3 4 0 1 2 3 4 0
2n 2 4 3 1 2 4 3 1 2 4 3 1 2 4 3 1 2 4 3 1
2n
3 1 1 0 2 0 0 4 1 4 4 3 0 3 3 2 4 2 2 1
+n

Veja que os restos das divisões de 2n por 5 formam uma seqüência de período 4,
enquanto que os restos das divisões de n por 5 formam uma seqüência de período
5. Logo, os restos das divisões de 2n + n formam uma seqüência de período 20,
dada pela última linha da tabela acima. Dessa forma, tomando os números de 1 a
10000 em intervalos de tamanho 20, o maior n tal que 2n + n deixa resto zero na
divisão por 5 é o 13o termo do ultimo intervalo, ou seja, o número 9980 + 13 =
9993.

SOLUÇÕES NÍVEL 3 – SEGUNDA FASE – PARTE B


PROBLEMA 1:
Seja k = a1 + a6 = a2 + a5 = a3 + a4. Temos 3k = a1 + a2 + ... + a6 é múltiplo de 9,
uma vez que n é múltiplo de 9. Daí, segue que k é múltiplo de 3. Mas, como os
algarismos são distintos, perceba que

EUREKA! N°32, 2010

30
Sociedade Brasileira de Matemática

1 + 2 + ... + 6 ≤ a1 + a2 + ... + a6 ≤ 4 + 5 + ... + 9 ⇔ 21 ≤ 3k ≤ 39 ⇔ 7 ≤ k ≤ 13.

Como k é múltiplo de 3, temos dois casos: k = 9 e k = 12.

1o caso: k = 9. Veja que é suficiente escolhermos a1, a2 e a3, pois a4 = 9 – a3, a5 = 9


– a2 e a6 = 9 – a1. Como os dígitos devem ser distintos, devemos escolher a1, a2 e
a3 de modo que haja no máximo um dígito em cada um dos conjuntos {1, 8}, {2,
7}, {3, 6} e {4, 5}. Esta escolha pode ser feita da seguinte forma:

• Escolhemos três dos quatro conjuntos: 4 maneiras;


• Em cada um dos três conjuntos acima, escolhemos um dos dois dígitos: 23 = 8
maneiras;
• Permutamos os dígitos escolhidos: 3! = 6 maneiras.
Logo, o total de números, neste caso, é igual a 4×8×6 = 192.

2o caso: k = 12. Neste caso, os dígitos a1, a2 e a3 devem ser escolhidos do conjunto
{3, 4, 5, 7, 8, 9} de modo que haja no máximo um dígito em cada um dos
conjuntos
{3, 9}, {4, 8} e {5, 7}. Esta escolha pode ser feita da seguinte maneira:

• Em cada um dos três conjuntos acima, escolhemos um dos dois dígitos: 23 = 8


maneiras;
• Permutamos os dígitos escolhidos: 3! = 6 maneiras.

Logo, o total de números, neste caso, é igual a 8 × 6 = 48.

O total de números é, portanto, 192 + 48 = 240.

PROBLEMA 2:
Analisando a equação módulo 5, obtemos 4 ⋅ 3a ≡ 1( mod 5 ) ⇔ 3a ≡ 4 ( mod 5 ) .
Mas os valores de 3a mod 5 são periódicos de período 4:

a 0 1 2 3 4 5 6 7
3ª mod 5 1 3 4 2 1 3 4 2

Assim, concluímos que 3a ≡ 4 ( mod 5 ) ⇔ a = 2 + 4t para t ∈ .


Agora, analisando a equação módulo 3, obtemos
11 + 5 ≡ 0 ( mod 3) ⇔ ( −1) ≡ 1( mod 3) o que ocorre se, e só se, b é par. Portanto a
b b

EUREKA! N°32, 2010

31
Sociedade Brasileira de Matemática

e b são ambos pares, digamos a= 2c e b = 2d para dois inteiros positivos c, d.


Assim,
4 ⋅ 3a = 11 + 5b ⇔ ( 2 ⋅ 3c ) − 52 d = 11
2

⇔ ( 2 ⋅ 3c − 5d )( 2 ⋅ 3c + 5d ) = 11
2 ⋅ 3c − 5d = 1

2 ⋅ 3c + 5d = 11
3c = 3 a = 2⋅c = 2
⇔ ⇔
5d = 5 b = 2⋅d = 2
Assim, a única solução é: ( A,B ) = ( 2 , 2 )

PROBLEMA 3:
Vamos fazer o gráfico da função
f ( x) = x ⋅ x  . Para cada k natural, se
k ≤ x ≤ k + 1 , temos x  = k . Logo, o gráfico
de f é formado por segmentos de reta y = k⋅x,
como mostra a figura ao lado: 6
Assim, para um n fixo, a equação f(x) = n tem
no máximo uma solução. Portanto, a quantidade
de elementos de 4
A1 ∪ A2 ∪ A3 ∪ ... ∪ A2009
é igual à quantidade de inteiros n, tais que 1 ≤ n 2
≤ 2009, para os quais f (x) = n admite solução, 1
isto é, os n tais que
f (k) = k2 ≤ n < k(k + 1) = k2 + k, 1 2 3
para algum k ∈ N.

PROBLEMA 4
Vamos mostrar inicialmente que BL e CK são as bissetrizes dos ângulos B e C do
∆ABC. Para isto, sejam K´ e L´ as intersecções das bissetrizes de C e B com a
circunferência de diâmetro BC , como na figura. Seja ainda I o incentro de ∆ABC
e β e γ as medidas de B e C , respectivamente, de modo que β + γ = 60°.

EUREKA! N°32, 2010

32
Sociedade Brasileira de Matemática


A γ /2
120º E´
D´ G

F
I
β /2
γ /2
β /2 γ /2
B C

Sejam D´ e E´ as intersecções de K´ L´ com os lados AB e AC do triângulo.


Para mostrar que K´ L´ = KL, basta mostrar que E´ e D´ são as projeções
ortogonais de I aos lados AC e AB . Como BC é diâmetro, temos que BL´C é
reto, assim se mostrarmos que o quadrilátero IE´ L´C é cíclico, provaremos que
IE´C é reto, e analogamente para D´.
Denote por F e G os encontros das bissetrizes de C e B com os lados opostos.

( ) ( ) ( ) ( )
Temos m GIC = m F IB = m AFC − m F BI = β + − =
γ β β+γ
2 2 2
= 30°. da

( ) ( ) β
mesma forma, temos m GE´L´ = m BGA − m ( GL´ E´ ) = + γ − =
2
γ β+γ
2 2
= 30°

pois m ( GL´ E´ ) = m ( BL´K´ ) = m ( BCK´ ) já que ambos os ângulos subtendem o

mesmo arco BK´. Assim, m ( GE´L´ ) = m ( GIC ) , provando que IE´L´C é cíclico.
Sendo O o ponto médio de BC, temos
( ) ( ) ) (
m K OL = 180° − m LOC − m K OB = 180° − β − γ = 120°

m ( LOK ) BC
Assim a distância pedida é LO ⋅ cos = ⋅ cos 60° = 3cm.
2 2

EUREKA! N°32, 2010

33
Sociedade Brasileira de Matemática

XXXI OLIMPÍADA BRASILEIRA DE MATEMÁTICA


Problemas e soluções da Terceira Fase

TERCEIRA FASE – NÍVEL 1

PROBLEMA 1
A sequência 121, 1221, 12221, ... contém todos os números da forma 122 … 21 . A
n dígitos 2

quantidade de dígitos 2 indica a posição do número na sequência. Por exemplo, o


número 122222221 é o sétimo termo da sequência.

a) Dentre os 2009 primeiros termos da sequência, quantos são divisíveis por 3?


b) Qual é o menor número múltiplo de 1001 da sequência?

PROBLEMA 2
O hexágono regular ABCDEF tem área de 12 cm2.
a) Traçando segmentos a partir de um vértice, o hexágono ABCDEF foi repartido
em 4 triângulos, conforme figura. Calcule as áreas desses triângulos.

b) Usando os quatro triângulos em que foi dividido o hexágono, podemos montar o


retângulo PQRS, na figura. Qual é a área desse retângulo?

EUREKA! N°32, 2010

34
Sociedade Brasileira de Matemática

PROBLEMA 3
As casas de um tabuleiro 4 × 4 devem ser numeradas
de 1 a 16, como mostrado parcialmente no desenho,
formando um Quadrado Mágico, ou seja, as somas
dos números de cada linha, de cada coluna e de cada
uma das duas diagonais são iguais.

a) Que números devem ser escritos no lugar de X e de Y?


b) Apresente o Quadrado Mágico completo na sua folha de respostas.

PROBLEMA 4
Carlinhos tem várias peças formadas por quatro quadradinhos de lado unitário, na
forma de L:

Ele forma figuras maiores com essas peças, fazendo coincidir um ou mais lados
dos quadradinhos, como no exemplo, em que foram usadas duas dessas peças,
fazendo coincidir um lado unitário. Não é permitido formar buracos nas figuras.

Permitido Não permitido


a) Desenhe uma figura cujo perímetro é 14.
b) Descreva como formar uma figura de perímetro 2010.
c) É possível formar uma figura de perímetro ímpar? Justifique sua resposta.

PROBLEMA 5
Um dominó é formado por 28 peças diferentes. Cada peça tem duas metades, sendo
que cada metade tem de zero a seis pontos:

EUREKA! N°32, 2010

35
Sociedade Brasileira de Matemática

Esmeralda coloca 4 peças de dominó dentro de um estojo, respeitando as regras do


jogo, isto é, peças vizinhas se tocam em metades com as mesmas quantidades de
pontos. Caso seja possível guardar as quatro peças no estojo, dizemos que o
conjunto de quatro peças é precioso.

Por exemplo, a figura acima mostra as maneiras de guardar o conjunto precioso

formado pelas peças , , , .


a) Mostre que um conjunto precioso não pode conter duas peças duplas.
A figura abaixo mostra as peças duplas.

b) Quantos conjuntos preciosos contêm uma peça dupla?


c) Determine a quantidade total de conjuntos preciosos.

EUREKA! N°32, 2010

36
Sociedade Brasileira de Matemática

TERCEIRA FASE – NÍVEL 2

PRIMEIRO DIA

PROBLEMA 1
Veja o problema No. 5 do Nível 1.

PROBLEMA 2
Seja A um dos pontos de interseção de dois círculos com centros X e Y. As
tangentes aos círculos em A intersectam novamente os círculos em B e C. Seja P o
ponto de plano tal que PXAY é um paralelogramo. Prove que P é o circuncentro do
triângulo ABC.

PROBLEMA 3
Prove que não existem inteiros positivos x e y tais que x3 + y3 = 22009.

SEGUNDO DIA

PROBLEMA 4
Resolva, em números reais, o sistema
1 1 1
x+ =y+ =z+
y z x
xyz = 1.

PROBLEMA 5
Uma formiga caminha no plano da seguinte maneira: inicialmente, ela anda 1cm
em qualquer direção. Após, em cada passo, ela muda a direção da trajetória em 60o
para a esquerda ou direita e anda 1cm nessa direção. É possível que ela retorne ao
ponto de onde partiu em
(a) 2008 passos?
(b) 2009 passos?

1 cm

1 cm
60°
60°
1 cm

EUREKA! N°32, 2010

37
Sociedade Brasileira de Matemática

PROBLEMA 6
Seja ABC um triângulo e O seu circuncentro. As retas AB e AC cortam o
circuncírculo de OBC novamente em B1 ≠ B e C1 ≠ C , respectivamente, as retas
BA e BC cortam o circuncírculo de OAC em A2 ≠ A e C 2 ≠ C , respectivamente, e
as retas CA e CB cortam o circuncírculo de OAB em A3 ≠ A e B3 ≠ B ,
respectivamente. Prove que as retas A2A3, B1B3 e C1C2 passam por um mesmo
ponto.

TERCEIRA FASE – NÍVEL 3

PRIMEIRO DIA

PROBLEMA 1
Esmeralda escreve 20092 números inteiros em uma tabela com 2009 linhas e 2009
colunas, colocando um número em cada casa da tabela. Ela soma corretamente os
números em cada linha e em cada coluna, obtendo 4018 resultados. Ela percebeu
que os resultados são todos distintos. É possível que esses resultados sejam todos
quadrados perfeitos?

PROBLEMA 2
Considere um primo q da forma 2p + 1, sendo p > 0 um primo. Prove que existe
um múltiplo de q cuja soma dos algarismos na base decimal é menor ou igual a 3.

PROBLEMA 3
São colocadas 2009 pedras em alguns pontos (x, y) de coordenadas inteiras do
plano cartesiano. Uma operação consiste em escolher um ponto (a, b) que tenha
quatro ou mais pedras, retirar quatro pedras de (a, b) e colocar uma pedra em cada
um dos pontos

(a, b – 1), (a, b + 1), (a – 1, b), (a + 1, b).

Mostre que, após um número finito de operações, cada ponto terá no máximo três
pedras. Além disso, prove que a configuração final não depende da ordem das
operações.

EUREKA! N°32, 2010

38
Sociedade Brasileira de Matemática

SEGUNDO DIA

PROBLEMA 4
Mostre que existe um inteiro positivo n0 com a seguinte propriedade: para qualquer
inteiro n ≥ n0 é possível particionar um cubo em n cubos menores.

PROBLEMA 5
Seja ABC um triângulo e O seu circuncentro. As retas AB e AC cortam o
circuncírculo de OBC novamente em B1 ≠ B e C1 ≠ C , respectivamente, as retas
BA e BC cortam o circuncírculo de OAC em A2 ≠ A e C 2 ≠ C , respectivamente, e
as retas CA e CB cortam o circuncírculo de OAB em A3 ≠ A e B3 ≠ B ,
respectivamente. Prove que as retas A2A3, B1B3 e C1C2 passam por um mesmo
ponto.

PROBLEMA 6
Seja n > 3 um inteiro fixado e x1, x2, …, xn reais positivos. Encontre, em função de
n, todos os possíveis valores reais de

x1 x2 x3 x n −1 xn
+ + + + +
x n + x1 + x 2 x1 + x 2 + x3 x 2 + x3 + x 4 xn−2 + x n −1 + x n x n −1 + x n + x1

SOLUÇÕES DA TERCEIRA FASE – NÍVEL 1

PROBLEMA 1: SOLUÇÃO DE PEDRO HENRIQUE ALENCAR COSTA (FORTALEZA – CE)


(a) Um número divisível por 3 tem a soma de seus algarismos como múltiplo de 3.
Assim, o primeiro termo múltiplo de 3 é 1221, pois 1 + 2 + 2 + 1 = 6, que é
múltiplo de 3.
O próximo é o mesmo com 3 algarismos 2 a mais. Então, para saber quantos
n−2
múltiplos de 3 escritos dessa forma existem até n, fazemos: + 1. Sendo n=
3
2009 − 2 2007
2009, fica: +1 = + 1 = 669 + 1 = 670.
3 3
(b) Vejamos inicialmente um exemplo de como multiplicar por 1001. Temos 1001
vezes 80 = 80080, pois:

EUREKA! N°32, 2010

39
Sociedade Brasileira de Matemática

0080
0080
+
0080080
7 a lg arismos

O primeiro termo da sequencia que é múltiplo de 1001 possui 7 algarismos, sendo


ele desta forma 1222221, que é igual a 1221 × 1001, pois:
1221
1221
+
1222221

É fácil verificar que os termos anteriores não são múltiplos de 1001.

PROBLEMA 2: SOLUÇÃO DE ANA BEATRIZ MOTTA ARAGÃO CORTEZ (CAMPINAS – SP)


a)

A B

F G C

E D

Seja G o centro do hexágono. A área GDE e GCD é igual a de AFE.


Tomando a figura como desenho representativo, podemos dividir o hexágono em
seis figuras de áreas iguais: AFE; AGE; GDE; GCD; AGC; ABC. Sabendo que sua
área é de 12 cm2, dividimos-na por 6 (número de partes em que o hexágono foi
fracionado; assim, cada fração tem 2 cm2 de área (12 cm2 : 6). Para calcularmos a
área dos triângulos pedidos, é só fazer:

AFE → 2cm 2
AED → AGE+ EGD → 2cm 2 + 2cm 2 → 4cm 2
ADC → AGC+ GCD → 2cm 2 + 2cm 2 → 4cm 2
ABC → 2cm 2

EUREKA! N°32, 2010

40
Sociedade Brasileira de Matemática

Temos então a área dos dois triângulos iguais AFE e ABC como 2cm 2 (cada um) e
a área dos outros dois triângulos iguais AED e ADC como 4cm 2 (cada um),
totalizando 12cm 2 .

Obs. Há outras formas de resolver o problema com este mesmo raciocínio.


Poderíamos dividi-lo em 3 losangos, ou 12 pequenos triângulos por exemplo.

b) Dividimos a figura, com um raciocínio parecido com o da letra a).


P

W
U
Q
S V
T

Cada triângulo acima possui a mesma área. Utilizando a informação de que o


triângulo em questão (SVR ou PQU) possui área de 2cm2, calculamos a área do
quadrilátero multiplicando 2cm 2 pelo número em que foi fracionada a figura, o que
dá 2cm 2 ⋅ 8 = 16cm 2 , que é a área do retângulo PQRS.

PROBLEMA 3: SOLUÇÃO DE DIMAS MACEDO DE ALBUQUERQUE (FORTALEZA – CE)


a) Veja os quadrados mágicos:

a1 a2 a3 a4 14 11 5 X

a5 a6 a7 a8 8
=
a9 a10 a11 a12 12 3

a13 a14 a15 a16 Y

Vendo-os, posso afirmar que a soma total do quadrado é a1 + a2 + ⋅⋅ ⋅ + a16 o que


equivale a 1 + 2 + ⋅⋅⋅ + 16 que é igual a (16 ⋅ 17 ) ÷ 2 = 136. Sabendo que em cada

EUREKA! N°32, 2010

41
Sociedade Brasileira de Matemática

linha a soma é a mesma, a soma de cada uma delas será 136 ÷ 4 = 34 . Como em
cada linha, coluna e diagonal a soma será 34 os valores de X e Y serão:
X = 34 − (14 + 11 + 5 ) = 34 − 30 = 4
Y = 34 − (14 + 8 + 3) = 34 − 25 = 9.

b) Vamos denominar os espaços vazios do quadrado de: b1 ,b2 , b3 ,b4 ,b5 ,b6 , b7 e b8
como mostra a figura:

D1 C1 C2 C3 C4
L1 14 11 5 4

L2 b1 8 b2 b3

L3 12 b4 3 b5

L4 b6 b7 b8 9
D2

Sabendo que em cada linha, coluna ou diagonal a soma é 34, temos as seguintes
equações:
b3 +b5 = 22 (as raízes só podem ser 15 e 6, pois alguns dos números dos outros
pares já aparecem).
b2 +b8 = 26 (as raízes só podem ser 16 e 10, pois alguns dos números dos outros
pares já aparecem).
b4 +b7 = 15 (as raízes só podem ser 13 e 2, pois alguns dos números dos outros
pares já aparecem).
b1+b6 = 8 (as raízes só podem ser 7 e 1, pois se fossem 6 e 2 não daria certo, pois o
2 já aparece em b4 ou b7 ).
Sendo assim, na linha 3 a única combinação qua dá certo é b4 = 13 e b5 = 6, caso
fossem valores diferentes a soma da linha não daria 34. Tendo descoberto esses
dois valores eu posso descobrir os outros:
Se b3 não é 6, só pode ser 15.
Se b7 não é 13, só pode ser 2.
Na linha 2 a única combinação que dá certo é b1 = 1 e b2 = 10, pois caso fossem
outros valores a soma não daria 34.

EUREKA! N°32, 2010

42
Sociedade Brasileira de Matemática

Tendo descoberto esses outros dois valores posso descobrir mais outros: Se b1 = 1 ,
b6 só pode ser 7. Logo se b6 é 7 e b7 é 2, b8 só pode ser 16.
Sabendo todos os valores desconhecidos, o quadrado mágico completo é assim:

14 11 5 4

1 8 10 15

12 13 3 6

7 2 16 9

PROBLEMA 4: SOLUÇÃO DE ISABELLA AYRES PINHEIRO DE LIMA (GOIÂNIA – GO)


a)

P =14

b) Primeiro, vamos utilizar figuras de perímetro 12, nas ‘pontas’ da figura:

Esses dois lados estarão no meio da figura, e por isso, não serão contados, ou seja,
o perímetro que essa figura vai ocupar na “grande” figura será de apenas 10. Como
são duas desses figuras (nas ‘pontas”), já conseguimos 20 de perímetro dos 2010
que precisamos.
Agora colocamos figuras de perímetro 14 no “meio”

⋅⋅⋅

Como 4 lados de cada figura estarão no meio da grande figura, cada uma delas
ocupará 10, no perímetro 2010.
Teremos que usar 199 destas figuras de perímetro 14, no meio; e 2 figuras de
perímetro 12, nas pontas. Ao todo: 2 × 10 + 199 × 10 = 2010 .

EUREKA! N°32, 2010

43
Sociedade Brasileira de Matemática

c) Não é possível formar uma figura de perímetro ímpar, porque uma simples peça

tem perímetro par e, toda vez que adicionamos outra peça, o perímetro
aumentou em 10 – 2. (número de lados usados na colagem), que é sempre par.
PROBLEMA 5
Veja a solução do problema 1 do nível 2.

SOLUÇÕES DA TERCEIRA FASE – NÍVEL 2

PROBLEMA 1: SOLUÇÃO DE VINÍCIUS CANTO COSTA (SALVADOR – BA)

a) Supondo o contrário, isto é, que seja possível um conjunto precioso com 2 peças
duplas, elas estariam intercaladas por uma peça, pois caso contrário, elas se
encaixariam e isto não é possível pois não tem números em comum e isto não seria
de acordo com a regra. Assim, as peças estariam arrumadas dessa forma:

X ? ?
X Y
? ? Y

e as outras duas como iriam se encaixar com as peças duplas de X e Y, seriam da


forma

X
Y

Mas isto é um absurdo, pois não existem peças iguais no jogo (c.q.d).

b) Se formarmos um conjunto precioso com uma peça dupla, ele seria organizado
dessa forma, seguindo as regras do jogo:

EUREKA! N°32, 2010

44
Sociedade Brasileira de Matemática

X X Y
X Y
X Z Z

Logo, se nós escolhermos as peças duplas e as que não têm contato com ela, nós
formamos o conjunto: Apenas pegamos a peça com o número da peça dupla e um
dos números da que não é dupla e a outra com o número da peça dupla e o outro
número da que não é dupla e organizamos da maneira certa, que é única, como
6
podemos observar. Logo, a quantidade será 7 ⋅   = 105, pois são 7 peças duplas
 2
e a outra peça deve ter números diferentes entre si e da peça dupla também, logo,
são 2 números para escolher em 6, já que uma não pode ser usada.

c) A quantidade total de conjuntos preciosos será a quantidade que inclui uma peça
dupla mais a que não tem esse tipo de peça. Já temos pelo item b) que com peça
dupla é 105. Basta contar os conjuntos sem peça dupla.
Esses conjuntos serão da forma

X X Y
W Y
W Z Z

com todos os números diferentes dois a dois. Repare que para cada conjunto de 4
números de 0 a 6 temos 3 conjuntos preciosos que seriam:

X X Y X X Z X X Z
W 1º. Y , W 2º. Z , Y 3º. Z

W Z Z W Y Y Y W W

EUREKA! N°32, 2010

45
Sociedade Brasileira de Matemática

7
Logo os conjuntos preciosos sem peça dupla totalizam   ⋅ 3, que são as maneiras
 4
de escolher 4 números dentre 7 vezes 3. Assim,
7 7 ⋅ 6 ⋅ 5 ⋅ 4 ⋅ 3 ⋅ 2 ⋅1
105 +   ⋅ 3 = 105 + ⋅ 3 = 105 + 7 ⋅ 5 ⋅ 3 = 105 + 105 = 210 é a
 4 4 ⋅ 3 ⋅ 2 ⋅1 ⋅ 3 ⋅ 2 ⋅1
quantidade total de conjuntos preciosos.

PROBLEMA 2: SOLUÇÃO DE FRANCISCO MARKAN NOBRE DE SOUZA FILHO (FORTALEZA –


CE)

X X1

α β
X 90° – β Y
180° – 2α

α P β

C
B
Se P é circuncentro de ABC , então ele deve ser a interseção entre as mediatrizes
dos segmentos AB e AC. Como AB e AC são tangentes às circunferências,
B AY = C AX = 90°. Esses dois ângulos têm B AC em comum e portanto
B AX = C AY , que chamarei de α :
B AX = C AY = α
B AC = 90° − α
X AY = 90° + α

EUREKA! N°32, 2010

46
Sociedade Brasileira de Matemática

Da última igualdade, como PXAY é paralelogramo, temos P X A = 90° − α . Por


outro lado, como o triângulo BXA é isósceles ( XA = XB ) , temos
AX B = 180° − 2α , ou seja, PX é bissetriz do ângulo AX B. Usando mais uma vez
que BXA é isósceles, PX também é a mediana e altura relativa ao lado AB. Assim,
PX é a mediatriz do segmento AB.
Pela mesma rzão, PY é a mediatriz do segmento AC, o que conclui a prova.

PROBLEMA 3: SOLUÇÃO DE LUCAS CAWAI JULIÃO PEREIRA (CAUCAIA – CE)


Para provarmos o que o enunciado quer, basta analisar a equação módulo 7.
Queremos descobrir, então, quais os restos que um cubo qualquer i3 deixa na
divisão por 7. Conseguimos isso elevando ao cubo os possíveis restos que um
número qualquer deixa por 7, que são 0, 1, 2, 3, 4, 5, e 6. Concluímos que os
possíveis restos que um cubo pode deixar são 0, 1 e 6.
Agora analisemos as potências de 2 módulo 7.

21 ≡ 2 ( mod 7 )
22 ≡ 4 ( mod 7 )
23 ≡ 1( mod 7 )
24 ≡ 2 ( mod 7 )

Encontramos o período, então dividimos 2009 por 3. Como o resto dessa divisão é
2, logo 22009 ≡ 4 ( mod 7 ) .
Daí encontramos um absurdo já que qualquer soma dos possíveis restos de dois
cubos jamais será 4. Logo x 3 + y 3 = 22009 não possui solução nos inteiros.

PROBLEMA 4: SOLUÇÃO DE ANDRE MACIEIRA BRAGA COSTA (BELO HORIZONTE – MG)


Olhemos para a primeira equação:
1 1 1
x + = y + → Vamos substituir o termo em termos das variáveis x e y.
y z z
1
Da segunda equação, temos: xyz = 1 → xy = (substituímos na primeira equação)
z
1
x+ = y + xy (multiplicamos tudo por y)
y
xy + 1 = y 2 + xy 2 (reordenando)
y 2 ( x + 1) − xy − 1 = 0 (resolvemos pela forma de Bháskara)

EUREKA! N°32, 2010

47
Sociedade Brasileira de Matemática

1
y1 = 1 e y2 = − .
x +1
Separemos em dois casos:
1) Caso y = 1.

Substituímos na segunda igualdade:


1 1 1 1
y + = z + →1+ = z +
z x z x
 xyz = 1
 x ⋅1 ⋅ z = 1
 1
 x ⋅ z = 1 Como xyz = 1, temos z = e daí
 x
z = 1
 x
1
1 + = z + z (multiplicando por z. z ≠ 0 )
z
2 z 2 − z − 1 = 0 (Resolvendo pela fórmula de Bháskara)
1
z1 = 1 e z2 = − .
2
1
Da equação z = ,x1 = 1 e x2 = −2.
x
 1
Nesse caso, temos as soluções (1, 1, 1) e  −2 ,1,  .
 2
1
2) Caso y = − ( x ≠ −1)
x +1
Substituímos na segunda equação:
 1 
x −  z =1
 x +1
x 1
− =
x +1 z
x +1 1 1 x x +1 1 1
z=− ; note que, nesse caso, y = + = − − = −1 = − + =z+ , e
x z x +1 x +1 x x x
a segunda igualdade também é satisfeita.

EUREKA! N°32, 2010

48
Sociedade Brasileira de Matemática

Resposta:
a +1
(1,1,1) ;  −2,1,−  e todas as triplas da forma 
1 1
 a, − ,−  , com
 2  a +1 a 
a ∈ − {0, −1} .
 1  1 a +1
Obs.  −2 ,1,−  → esta solução é da forma  a, − ,− .
 2  a + 1 a 

PROBLEMA 5: SOLUÇÃO DE DANIEL EITI NISHIDA KAWAI (SÃO PAULO – SP)

120° 120°
°
60 120°
60°
120° 120°

Temos o diagrama infinito de possíveis posições em que a formiga pode chegar.

a) Resposta: Sim.
Para voltar à posição inicial em 2008 passos, basta seguir as instruções abaixo:

EUREKA! N°32, 2010

49
Sociedade Brasileira de Matemática

Posição inicial

Posição
inicial

Dê 333 voltas no hexágono (isso dará 1998 passos e depois siga o trajeto abaixo,
em que são usados 10 passos e volta-se à posição inicial. No total, s formiga dará
2008 passos e voltará à posição inicial

b) Resposta: Não.
Pinte as posições da figura inicial de preto e branco alternadamente. A formiga
começa em uma bolinha preta e toda bolinha preta está cercada de bolinhas brancas
e toda bolinha branca está cercada de bolinhas pretas. Assim, quando a formiga
anda um númro par de passos, ela sempre termina em uma bolinha preta e quando
anda um número ímpar de passos, ela sempre terminará em uma bolinha branca.
Como 2009 é ímpar, a formiga, se começar em uma bolinha preta, sempre
terminará em uma bolinha branca; logo, será impossível voltar à posição inicial
depois de 2009 passos.

PROBLEMA 6: SOLUÇÃO DE DANIEL EITI NISHIDA KAWAI (SÃO PAULO – SP)

O1

O X
M
A C1 C

EUREKA! N°32, 2010

50
Sociedade Brasileira de Matemática

Temos AO ≅ BO CO e BO1 ≅ CO1 C1O1 ≅ OO1 . Além disso, C1 BO e C1 CO


são ângulos inscritos do mesmo arco de circunferência
C1 MO
C1 MO ⇒ C1 BO = C1 CO = . Como ∆ACO é isósceles (já que AO ≅ CO ),
2
C1 AO = C1 CO ⇒ C1 AO = C1 BO. Como ∆ABO é isósceles,
B AO = ABO ⇒ B AO + C1 AO = ABO + C1 BO ⇒ B AC1 = ABC1 ⇒ ∆ABC1 é isósceles
→ C1 ∈ mAB . De maneira análoga, A2 ∈ mBC , A3 ∈ mBC , B1 ∈ mAC , B3 ∈ mAC e
C2 ∈ mAB ⇒ A2 A3 ⊂ mBC , B1 B3 ⊂ mAC e C1C2 ⊂ mAB . Como mAB ,mAC e mBC se
encontram em 0, as retas A2 A3 ,B1 B3 e C1C2 passam por um mesmo ponto.
Obs. mXY é mediatriz do segmento XY .

SOLUÇÕES DA TERCEIRA FASE – NÍVEL 3

PROBLEMA 1: SOLUÇÃO DE HUGO FONSECA ARAÚJO (RIO DE JANEIRO – RJ)


Sim, é possível. Considerando a tabela como uma matriz aij tome aij = 0 , para
1 ≤ i, j ≤ 2008, e ai ,2009 = ( 2i − 1) ,a2009 ,i = ( 2i ) , para 1 ≤ i ≤ 2008.
2 2

12
32

22 42 ... d

Então já temos 4016 fileiras cujas somas são quadrados perfeitos e distintos. As
duas que faltam são a última linha e última coluna. Seja a2009 ,2009 = d .
Queremos que
12 + 32 + ... + 40152 + d = b 2
,
22 + 42 + ... + 40162 + d = c 2
onde b, c são distintos e maiores que 4016. Subtraindo as equações, temos:

EUREKA! N°32, 2010

51
Sociedade Brasileira de Matemática

c 2 − b 2 = 1 + 2 + ... + 4016 = 2008 ⋅ 4017


⇒ ( c − b )( c + b ) = 2008 ⋅ 4017.
Tomando c = 502 ⋅ 4017 + 2 e b = 502 ⋅ 4017 − 2 , a igualdade acima é satisfeita.
Para concluir, tome
d = b 2 − (12 + 32 + ... + 40152 ) = ( 502 ⋅ 4017 − 2 ) − (12 + 32 + ... + 40152 ) .
2

Desse modo, 12 + 32 + ... + 40152 + d = ( 502 ⋅ 4017 − 2 )


2

22 + 42 + ... + 40162 + d = ( 502 ⋅ 4017 + 2 ) .


2

PROBLEMA 2: SOLUÇÃO DE MATHEUS SECCO TORRES DA SILVA (RIO DE JANEIRO – RJ)


Vamos organizar as idéias.
Se p = 2,q = 5 e 10 satisfaz.
Se p > 2, o múltiplo de q só poderá ter soma 2 ou 3, pois se tivesse soma 1, seria
+
uma potência de 10, e como q é primo > 5, q não divide 10n ,n ∈ .
Então, devemos conseguir um múltiplo com soma 2 ou 3.
• Múltiplos com soma 2: 10a + 1
• Múltiplos com soma 3: 10a + 10b + 1 ( a ≥ b ) .
q −1 q −1
Pelo Pequeno Teorema de Fermat, 10q −1 ≡ 1( q ) ⇒ 10 2
≡ 1( q ) ou 10 2
≡ −1( q )
q −1 q −1
Se 10 ≡ −1( q ) ,10
2 2
+ 1 satisfaz as condições do problema (tem soma dos
dígitos 2).
q −1
Suponha então que 10 2
≡ 1( q ) ⇒ 10 p ≡ 1( q ) ⇒ ord q 10 = 1 ou p.
Se ord q 10 = 1,q ≠ 9 ⇒ q = 3 ⇒ p = 1, absurdo. Logo, ord q 10 = p.
Nesse caso, vamos tentar um múltiplo com soma 3, isto é, vamos procurar inteiros
positivos a e b tais que 10a + 10b + 1 ≡ 0 (q).
101 ,102 ,...,10 p são p resíduos distintos módulo q = 2 p + 1. De fato, se
10 x ≡ 10 y ( q ) com y < x ≤ p ,10 ( x − y ) ≡ 1 ( q ) , 0 < x − y < p , contradição, pois
ord q 10 = p .
Se ∃ x tal que 10 x ≡ p ( q ) , tomemos a = b = x ⇒ 2 ⋅ 10 x + 1 ≡ 0 ( q ) e o problema
acaba. Suponha então que ∃ x tal que 10 x ≡ p ( q ) .
Temos então p resíduos para 101 ,102 ,...,10 p dentre 0 ,1, 2 ,3,..., p − 1, p + 1,..., 2 p.
Vamos considerar a lista formada por esses p resíduos.

EUREKA! N°32, 2010

52
Sociedade Brasileira de Matemática

0 não está na lista, pois q = 2 p + 1 > 5.


Se ∃ y tal que 10 y ≡ −1( q ) , teríamos 102 y ≡ 1( q ) e
ord q 10 = p ⇒ p 2 y ⇒ 2 y = kp ⇒ 2 kp ⇒ 2 k ⇒ k = 2k0 ⇒ y = k0 p ⇒

10 y = 10k0 p ≡ (10 p ) ≡ 1( q ) , absurdo, pois estamos supondo 10 y ≡ −1( q ) .


k0

Logo, 0 e 2p não entram na lista!


Considere os pares (1, 2 p − 1) ; ( 2 , 2 p − 2 ) ; ( 3, 2 p − 3) ;...( p − 1, p + 1) .
Eles incluem todos os resíduos que 101 ,102 ,...,10 p podem assumir.
Temos p – 1 pares e p resíduos a escolher. Pelo Princípio da Casa dos Pombos,
escolheremos dois números do mesmo par. Mas a soma de dois números do mesmo
par é 2p(mod q).
Logo, ∃x, y com 10 x + 10 y ≡ 2 p ( q ) ⇒
A = 10 x + 10 y + 1 ≡ 0 ( q ) ⇒ A é múltiplo de q e tem soma dos dígitos 3.

PROBLEMA 3: SOLUÇÃO DE RENAN HENRIQUE FINDER (JOINVILLE – SC)


Vamos chamar “operação” o ato de tirar 4 pedras de (a, b) e colocar uma pedra em
cada um dos pontos ( a + 1,b ) ,( a − 1,b ) ,( a,b − 1) e ( a,b + 1) . Não faremos distinção
de operações no mesmo ponto que usam pedras diferentes; assim, atentaremos para
quantas pedras há em cada ponto, e não quais.
Provemos por indução o seguinte resultado, trivial para n ≤ 4.
Para qualquer n, existe A ( n ) tal que, para quaisquer pedras p1 , p2 ,..., pn , não é
possível realizar mais de A ( n ) operações.
Suponhamos que isso valha para todo k < n para fazer o caso em que temos n
pedras. É importante observar que A ( n ) depende apenas de n e não da distribuição
das pedras.
Claramente o centro de massa das pedras é invariante. Logo, podemos fixá-lo como
origem (desconsiderando a hipótese de as pedras terem coordenadas inteiras).
Dada uma sequência de operações, chamaremos pi ( t ) a posição de pi após t
operações , de modo que pi ( 0 ) é a posição inicial de pi .
Note que ∀x ∈ ,( x − 1) + ( x + 1) = 2 x 2 + 2
2 2

⇒∀a,b ∈ ,( a + 1) + b2  + ( a −1) + b2  + a2 + ( b −1)  + a2 + ( b + 1)  = 4a2 + 4b2 + 4


2 2 2 2
       
Assim, se uma operação em (a, b) move para p1 , p2 , p3 , p4 , com
p1 ( t + 1) = ( a + 1,b) , p2 ( t + 1) = ( a − 1,b) , p3 ( t + 1) = ( a,b + 1) e p4 ( t + 1) = ( a,b − 1) ,

EUREKA! N°32, 2010

53
Sociedade Brasileira de Matemática

n n
⇒ ∑ pi ( t ) = ∑ pi ( t −1) + 4.
2 2

i =1 i =1
n
4t t
Por indução, ∀t, ∑ pi ( t ) ≥ 4t ⇒∃i : pi ( t ) ≥ ⇒ pi ( t ) ≥ 2
2 2

i =1 n n
Vamos escolher t grande (veremos que t ≥ 9n3 A ( n − 1) basta).
2

 t 
Definimos p = pi ( t ) . Da invariância do centro de massa, ∃j , p j ( t ) ∉ C  p, ,
 n 

onde C ( p,r ) = { x ∈ 2
: x − p ≤ r} é o círculo de centro p e raio r. Isso ocorre
 t 
porque, se todas as pedras estivessem em C  p,  , que é convexo, seu centro de
 n 

 t  t
massa também estaria, o que significa ( 0 ,0 ) ∈ C  p,  ⇔ p ≤ , absurdo.
 n n
Agora vemos as regiões
R1 = C ( p,3 A ( n − 1) )
R2 = C ( p,6 A ( n − 1) )
R3 = C ( p,9 A ( n − 1) )

Rn = C ( p,3nA ( n − 1) ) .
2
Uma das n + 1 regiões R1 ,R2 \ R1 ,R3 \ R2 ,...Rn \ Rn −1 e \ Rn não contém nenhuma
t  t 
pedra. Como t ≥ 9n3 A ( n − 1) ⇔ ≥ 3nA ( n − 1) , teremos p j ∉ C  p,
2
 e
n  n 
 t 
Rn ⊂ C  p,  ⇒ p j ∉ Rn ⇒ \ Rn não está vazia. R1 não está vazia porque
2

 n 
p ∈ R1 . Logo, ∃k ,1 ≤ k ≤ n tal que Rk \ Rk −1 não tem pedras.
Assim temos até n – 1 pedras em Rk −1 e até n – 1 pedras em 2
− Rk .
A distância entre uma pedra de Rk −1 e uma fora de Rk é sempre pelo menos
3 A ( n − 1) (vide definição). As pedras em Rk −1 e as fora de Rk se moverão
independentemente até que duas delas ocupem a mesma posição. Para que isso

EUREKA! N°32, 2010

54
Sociedade Brasileira de Matemática

ocorra, pela hipótese de indução, as pedras fora de Rk não realizarão mais de


A ( n − 1) movimentos, bem como as de dentro de Rk −1 . Portanto, depois de
2 A ( n − 1) rodadas, cada pedra se deslocará no máximo A ( n − 1) unidades, logo
uma pedra fora de Rk não poderá ficar no mesmo ponto que uma pedra que estava
dentro de Rk −1 , o que torna os dois conjuntos necessariamente independentes.
Assim, basta tomar A ( n ) = 9n3 A ( n − 1) + 2 A ( n − 1) .
2

Isso resolve a primeira parte.

Para a segunda parte, comecemos lembrando que, se chegarmos à mesma


configuração de duas maneiras diferentes, a igualdade
n n

∑ p (t ) = 4t + ∑ pi ( 0)
2 2
i
i =1 i =1

diz que o número de operações, t, é igual nas duas maneiras.


Para a prova, suponhamos que na configuração inicial, os pontos com 4 pedras ou
mais sejam X1 ,X 2 ,... e X e . Considere também uma sequência de operações que leva
o plano a um estado em que não é possível fazer mais operações. Certamente,
ocorreram operações com centro em X1 ,X 2 ,... e X e .
Considerando duas sequências de operações O1 ,O2 ,...,Oα e O1´ ,O´2 ,...,O´β que
terminam em uma configuração na qual não é possível fazer mais operações,
provaremos que uma é permutação da outra via indução em min{α,β} , o que
resolve o problema.
Seja X um ponto em que há mais de quatro pedras no princípio. Seja Oγ a primeira
operação em {O1 ,O2 ,...,Oα } com centro em X.
Vamos provar que a sequência de operações Oγ ,O1 ,O2 ,...,Oγ−1 ,Oγ+1 ,...Oα leva ao
mesmo resultado que O1 ,O2 ,...,Oα .
Basta provar que Oγ ,O1 ,...,Oγ−1 ,Oγ+1 ,...Oα é uma sequência de operações válidas, já
que cada operação tira o mesmo número de pedras de cada ponto e coloca o mesmo
número em cada ponto, independentemente de quando foi realizada, de forma que
as operações são comutativas.
Pelo mesmo argumento, basta ver que Oγ ,O1 ,...,Oγ−1 é uma sequência possível. Mas
começar com Oγ é claramente possível e, da minimalidade de γ , as operações
O1 ,... e Oγ−1 têm centro em um ponto diferente de X. Assim, Oγ só pode ter

EUREKA! N°32, 2010

55
Sociedade Brasileira de Matemática

aumentado o número de pedras nos centros de O1 ,... e Oγ−1 , e não diminuído, o que
faz com que toda essa sequência seja possível.
Em outras palavras, sem perda de generalidade, O1 = Oγ ⇒ O1 tem centro em X.
Analogamente, podemos supor que, O1' tem centro em X. Agora, após a realização
da operação com centro em X, min{α,β} diminui, e vemos que as seqüências
´
O2 ,...,Oα e O2´,...,O são iguais pela hipótese de indução.
β

PROBLEMA 4: SOLUÇÃO DE MARLEN LINCOLN DA SILVA (FORTALEZA – CE)


Seja S = {n ∈ : é possível particionar um cubo em n cubos menores}.
Lema: Se x, y ∈ S , então x + y − 1∈ S.
Prova: Particione o cubo inicial em x cubos menores. Escolha um desses cubos e o
particione em y cubos. Daí, o cubo inicial estará particionando em x + y – 1 cubos
menores.
Claramente, n3 ∈ S , para n ≥ 2 inteiro.
Assim 23 ∈ S; portanto, ∀x ∈ S ,x + 8 − 1 = x + 7 ∈ S. Para termos o resultado
desejado, basta provarmos que existem a1 ,a2 ,...,a7 ∈ S , tais que ai ≡ a j ( mod 7 ) ,
para 1 ≤ i < j ≤ 7 (basta escolhermos n0 = max {ai ,1 ≤ i ≤ 7} ).
De fato, se x ≥ n0 = max {ai ,1 ≤ i ≤ 7} e x ≡ a j ( mod 7 ) ,x = a j + 7 k ∈ S.
Seja S´ = { x ( mod 7 ) ,x ∈ S } . De forma análoga, se x, y ∈ S´ então
( x + y − 1) ( mod 7 ) ∈ S.
Claramente 1,6 ,0 ∈ S , já que 23 ,33 e 73 ∈ S. Logo ( 6 + 6 − 1) mod 7 = 4 ∈ S´ e
( 0 + 6 − 1) mod 7 = 5 ∈ S´. Então ( 4 + 0 − 1) mod 7 = 3 ∈ S' e
( 3 + 0 − 1) mod 7 = 2 ∈ S' ⇒ S´ = {0,1,2,3,4,5,6} e o problema está acabado.

PROBLEMA 5: SOLUÇÃO DE GUSTAVO LISBOA EMPINOTTI (FLORIANÓPOLIS – SC)


Considere uma inversão com respeito ao circuncírculo do ∆ABC. Como o
circuncírculo do ∆AOC passa pelo centro de inversão (O), seu inverso é uma reta
pelos inversos de A e C. Mas A e C pertencem ao círculo de inversão, de modo que
são seus próprios inversos. Ou seja, a reta AC é o inverso do circuncínculo de
∆AOC . O inverso do ponto A2 é a interseção do inverso do circuncírculo de
∆AOC –que é AC – com o inverso de AB –que é o circuncírculo do ∆AOB – isto

EUREKA! N°32, 2010

56
Sociedade Brasileira de Matemática

é, é o ponto A3 . Então A2 e A3 são inversos, logo A2 A3 passa pelo centro de


inversão, O. Analogamente, B1 B3 e C1C2 passam por O, como queríamos.

PROBLEMA 6: SOLUÇÃO DE RENAN HENRIQUE FINDER (JOINVILLE – SC)


Sendo rn ( a ) o resto da divisão do inteiro a por n (i.e., o único número r em ]0,n ]
tal que a ≡ r mod ( n ) , definamos xa = xrn ( a ) . Claramente, a função S : ( * n
+ ) →
dada por:
n
xi n
xi
S ( x1 ,...xn ) = ∑ satisfaz S ( x1 ,...xn ) > ∑ =1
i =1 xi −1 + xi + xi +1 i =1 x1 + x2 + ... + xn

Além disso, f ( t ) = S (1,t,t 2 ,t 3 ,...,t n −1 ) =


1
+
( n − 2) t + t n −1
1 + t + t n −1 t 2 + t + 1 1 + t n − 2 + t n −1
n
É contínua e tal que f (1) = e lim f ( t ) = 0 + ( n − 2 ) 0 + 1 = 1, de modo que
3 t →∞

 n
S ( x1 ,...xn ) pode assumir qualquer valor no intervalo 1,  . Por outro lado, se n é
 3
n 1 t  n
par, g (1,t,1,t,...,1,t ) =  +  é contínua e tal que g (1) = , enquanto
2  2t + 1 t + 2  3
n n n
lim g ( t ) = . Portanto, S ( x1 ,...,xn ) assume todos os valores do intervalo  ,  .
t →∞ 2 3 2
Se n for ímpar, definamos a função
2 n −1 t n−3 1 n
h ( t ) = S (1,t,1,t,...,1,t,1) = + ⋅ + ⋅ . Temos h (1) = e
t+2 2 t+2 2 2t + 1 3
n −1
h (t ) → quando t → ∞. Segue disso que n pode tomar qualquer valor em
2
 n n − 1
 3 , 2  . No caso em que n é par, não é difícil resolver o problema se notarmos
 
x x
que < ∀x, y,z ∈ *+ .
x+ y+z x+ y
De fato:
2k
xi k
x2i −1 x2i
∑i =1 xi −1 + xi + xi +1
= ∑
i =1 x2 i − 2 + x2 i −1 + x2 i
+
x2i −1 + x2i + x2i +1
k
x2i −1 x2i
<∑ + = k.
i =1 x2 i −1 + x2 i x2i −1 + x2i

EUREKA! N°32, 2010

57
Sociedade Brasileira de Matemática

Assim, sob a hipótese de n ser par, os valores possíveis de S ( x1 ,...xn ) são os


 n
elementos de 1,  .
 2
 n − 1
Para o caso n ímpar, queremos mostrar que a imagem de S é 1, .
 2 
Adaptaremos a idéia usada anteriormente.
Veja que essa idéia prova que
x4 x5 xn −1 xn n−3
+ + ... + + <
x3 + x4 + x5 x4 + x5 + x6 xn − 2 + xn −1 + xn xn −1 + xn + xn +1 2
n −1
Logo, para que se garanta que S ( x1 ,x2 ,...xn ) < , é suficiente termos
2
x1 x2 x3 x + x + x3
+ + ≤ 1 2
x0 + x1 + x2 x1 + x2 + x3 x2 + x3 + x4 x1 + x2 + x3
as condições x0 ≥ x3 e x4 ≥ x1 implicam essa desigualdade. Se supusermos por
n −1
absurdo S ( x1 ,...xn ) ≥ , concluímos que x0 ≥ x3 ⇒ x1 > x4 e x1 ≤ x4 ⇒ x0 < x3 .
2
Analogamente, supondo x0 ≤ x3 ,
x1 > x4 ⇒ x2 > x5
x2 > x5 ⇒ x3 > x6

x3 n − 4 > x3 n −1 ⇒ x3 n −3 > x3n

O absurdo é que x0 ≥ x3 > x6 > x9 > ... > x3n = x0 . A suposição x1 ≤ x4 pode ser
tratada similarmente.

Obs: Outra maneira de se fazer o caso x0 ≤ x3 é definir yn = x3− n , o que dá y3 ≤ y0 .


Alem disso,
n −1 n −1
S ( x1 ,...,xn ) < ⇔ S ( y1 ,..., yn ) < , o que já sabemos provar.
2 2

EUREKA! N°32, 2010

58
Sociedade Brasileira de Matemática

XXXI OLIMPÍADA BRASILEIRA DE MATEMÁTICA


Problemas e soluções da Primeira Fase – Nível Universitário

PROBLEMA 1
(a) Encontre o valor mínimo da função f : → dada por f ( x ) = e( x / e ) − x
(aqui e = 2,71828... é a base do logaritmo natural).
(b) Qual destes números é maior: e π ou πe ?

PROBLEMA 2
Seja ζ ∈ uma raiz de x 7 − 1, com ζ ≠ 1. Existe um polinômio Mônico p de grau 2
com coeficientes inteiros cujas raízes são os números z1 = ζ + ζ 2 + ζ 4 e
z2 = ζ + ζ + ζ . Calcule p ( 3) .
3 5 6

PROBLEMA 3
A rã Dõ descansa no vértice A de um triângulo equilátero ABC. A cada minuto a rã
salta do vértice em que está para um vértice adjacente, com probabilidade p de o
salto ser no sentido horário e 1 – p de ser no sentido anti-horário, onde p ∈ ( 0,1) é
uma constante. Seja Pn a probabilidade de, após n saltos, Dõ estar novamente no
vértice A.
(a) Prova que, qualquer que seja p ∈ ( 0 ,1) ,lim Pn = 1 / 3.
n →∞

(b) Prove que existe p ∈ ( 0 ,1 / 100 ) tal que, para algum n ∈ ,Pn = 1 / π.

PROBLEMA 4
Determine a quantidade de números inteiros positivos n menores ou iguais a 31!
Tais que 3n + n é divisível por 31.

PROBLEMA 5
Dados os números reais a, b, c, d, considere a matriz

a b c d
 
d a b c
A= .
c d a d
 
b c d a
Se f ( x ) = a + bx + cx 2 + dx3 , prove que

EUREKA! N°32, 2010

59
Sociedade Brasileira de Matemática

det A = f (1) f ( i ) f ( −1) f ( −i ) .


(Aqui i representa a unidade imaginária.)

PROBLEMA 6
Considere a sequência a0 ,a1 ,a2 ,... definida por a0 = 0 ,a1 = π / 3 e, para n ≥ 1,
π ( a0 an + a1an −1 + a2 an − 2 + ... + an a0 )
an +1 = .
3 ( n + 1)
Calcule

ak a1 a2 a3
∑2
k =0
k
= a0 + + + + ... .
2 4 8

SOLUÇÕES NÍVEL UNIVERSITÁRIO – PRIMEIRA FASE

PROBLEMA 1
1
a) A derivada da função f é f´ ( x ) = ⋅ e( x e) − 1, que se anula apenas para x = e,
e
sendo negativa para x < e e positiva para x > e. Assim, o valor mínimo de f é f (e) =
0.
b) Pelo resultado do item anterior, como π ≠ e temos que f ( π ) > 0 , logo e( π e) > π ,
ou seja, e( ) > πe .
π

PROBLEMA 2
Um polinômio que satisfaz as condições do enunciado é
p ( x ) = x − ( z1 + z2 ) x + z1 z2 .
2

ζ7 − 1
z1 + z2 = ζ + ζ 2 + ζ 3 + ζ 4 + ζ 5 + ζ 6 = −1 + = −1.
ζ −1
z1 z2 = ζ 4 + ζ 6 + ζ 7 + ζ 5 + ζ 7 + ζ 8 + ζ 7 + ζ 9 + ζ10 = 3 + ζ + ζ 2 + ζ 3 + ζ 4 + ζ 5 + ζ 6 = 2.
Logo p ( x ) = x 2 + x + 2 e p ( 3) = 14.

PROBLEMA 3
a) Sejam A, B, e C os vértices do triângulo no sentido anti-horário. Seja Qn (resp.
Rn ) a probabilidade de, após n saltos, Dõ estar no vértice B (resp. C).
Temos P0 = 1,Q0 = R0 = 0 e, para todo n ≥ 0 ,

EUREKA! N°32, 2010

60
Sociedade Brasileira de Matemática

 Pn +1 = (1 − p ) Rn + pQn

(*) Qn +1 = (1 − p ) Pn + pRn

 Rn +1 = (1 − p ) Qn + pPn
Dado n natural, seja Dn = max { Pn − Qn , Qn − Rn , Rn − Pn } . Vamos provar que,
para todo n, Dn +1 ≤ max { p,1 − p} . Dn . Dado n, há 6 possibilidades para a ordem
dos números Pn ,Qn ,Rn . Vamos analisar o caso Pn ≤ Qn ≤ Rn (os outros 5 casos são
análogos). Nesse caso, a maior distância Dn entre dois dos números Pn ,Qn e Rn é
Rn − Pn . De (*), obtemos:
Pn +1 − Qn +1 = (1 − p )( Rn − Pn ) + p ( Qn − Rn ) ≤ max {(1 − p )( Rn − Pn ) , p ( Rn − Qn )} ,
≤ max {1 − p, p} . Dn , pois Rn − Pn e Qn − Rn têm sinais contrários.
Qn +1 − Rn +1 = (1 − p )( Pn − Qn ) + p ( Rn − Pn ) ≤ max {(1 − p )( Qn − Pn ) , p ( Rn − Pn )}
≤ max { p,1 − p} ⋅ Dn , pois Pn − Qn e Rn − Pn têm sinais contrários.
Rn +1 − Pn +1 = (1 − p )( Qn − Rn ) + p ( Pn − Qn ) = (1 − p )( Rn − Qn ) + p ( Qn − Pn ) ≤
≤ max { p,1 − p} ⋅ ( Rn − Qn + Qn − Pn ) = max { p,1 − p} ⋅ ( Rn − Pn ) = max { p,1 − p} ⋅ Dn .
Assim, Dn +1 = max { Pn +1 − Qn +1 , Qn +1 − Rn +1 , Rn +1 − Pn +1 } ≤ max { p,1 − p} ⋅ Dn , para

todo n ≥ 0 , donde Dn ≤ ( max { p,1 − p} ) ,∀n ≥ 0.


n

Como Pn + Qn + Rn = 1,∀n ∈ ,
1 P + Qn + Rn ( Pn − Qn ) + ( Pn − Rn ) 2Dn 2
≤ ⋅ ( max { p,1 − p} ) ,∀n ≥ 0.
n
Pn − = Pn − n = ≤
3 3 3 3 3
1
Como 0 < max { p,1 − p} < 1, segue imediatamente que lim Pn − = 0 , e que
n →∞ 3
1
lim Pn = .
n →∞ 3

b) Para p = 0 teríamos Pn = 1 quando n é múltiplo de 3 e Pn = 0 caso contrário.


Por outro lado, tomando p = 1 100 , temos lim P3 k +1 = 1 3 . Em particular, existe
k →∞

1 1 1
r∈ tal que P3r +1 > , pois < .
π π 3
EUREKA! N°32, 2010

61
Sociedade Brasileira de Matemática

Considerando P3r +1 = P3r +1 ( p ) como função de p, temos que P3r +1 ( p ) é um


polinômio (de grau no máximo 3r + 1) em p, e portanto depende continuamente de
 1  1
p. Como P3r +1 ( 0 ) = 0 e P3r +1   > , existe, pelo teorema do valor
 100  π
1 1
intermediário, p com 0 < p < tal que P3r +1 ( p ) = .
100 π

Solução alternativa para o item a):

Podemos (Como no início da solução anterior), escrever


 Pn +1   0 p 1 − p   Pn 
     
 Qn +1  = 1 − p 0 p  ⋅  Qn  ,∀n ≥ 0.
 R   p 1− p 0   Rn 
 n +1  
1  2 p −1
Os autovalores dessa matriz 3 × 3 são 1 e − ± i 3 ⋅  .
2  2 
As normas dos autovalores distintos de 1 são iguais a 1 − 3 p (1 − p ) < 1, donde
Pn ,Qn e Rn convergem a certos números, que denotaremos por x, y, z,
respectivamente.
Devemos então ter:
x = (1 − p ) z + py, y = (1 − p ) x + pz,z = (1 − p ) y + px, donde
x = (1 − p ) z + p ( (1 − p ) x + pz ) ⇒ (1 − p + p 2 ) x = (1 − p + p 2 ) z ⇒ x = z ⇒
⇒ y = (1 − p ) x + px = x, e logo x = y = z = 1 3 (pois x + y + z = 1).

PROBLEMA 4
Pelo pequeno teorema de Fermat, 330 ≡ 1( mod 31) , e logo ( 3 ( mod 31) )
n
é
periódico com período divisor de 30.
Por outro lado, obviamente ( n ( mod 31) ) é periódico com período 31.
Portanto, ( 3n + n ( mod 31) ) é periódico com período divisor de 31 ⋅ 30 = 930.
Pelo teorema chinês dos restos, para cada a com 0 ≤ a ≤ 29 e b com 0 ≤ b ≤ 30 ,
existe um único c ( a,b ) com 0 ≤ c ( a,b ) ≤ 929 tal que c ( a,b ) ≡ a ( mod 30 ) e
c ( a,b ) ≡ b ( mod 31) .
Temos 3c ( a ,b ) + c ( a,b ) ≡ 3a + b ( mod 31) .

EUREKA! N°32, 2010

62
Sociedade Brasileira de Matemática

Fixando a e fazendo b variar, 3a + b percorre todas as 31 classes (mod 31). Assim,


3m + m , 0 ≤ m ≤ 929 passa 30 vezes por cada classe (mod 31). Como
930 = 31 ⋅ 30 31 !, 3n + n ≡ 0 ( mod 31) para 31!/31 = 30! inteiros positivos menores
ou iguais a 31!.

PROBLEMA 5
1ª. Solução
Se x é uma raiz quarta da unidade, temos
xf ( x ) = d + ax + bx 2 + cx 3 ,x 2 f ( x ) = c + dx + ax 2 + bx3 e
x 3 f ( x ) = b + cx + dx 2 + ax 3 , de modo que
 1   f ( x)  1
   xf x   
x ( ) x
A 2  =  2 = f ( x) 2  .
 x   x f ( x)  x 
 3   3   3
 x   x f ( x)  x 
Assim, o vetor (1,x,x 2 ,x3 ) é autovetor de A, com autovalor f(x), para x = 1, i, –1, –
i. deduzimos que A possui 4 autovetores independentes e, portanto, det A é o
produto dos respectivos autovalores, ou seja, det A = f (1) f ( i ) f ( −1) f ( −i ) .

2ª. Solução
Observamos que det A é um polinômio do quarto grau nas variáveis a, b, c, d,
enquanto f(1), f(i), f ( –1), f (– i) são polinômios irredutíveis distintos do primeiro
grau nessas mesmas variáveis. Podemos realizar operações lineares nas linhas de A
para provar que o polinômio det A é divisível por f(1), f(i), f (–1), f (–i). Isto fica
mais rápido utilizando a mesma ideia da primeira Solução: se x é raiz quarta da
unidade, multiplicando a segunda coluna por x, a terceira por x2 e a quarta por x3 e
somando tudo isso à primeira coluna, obtemos ( f ( x ) ,xf ( x ) ,x 2 f ( x ) ,x3 f ( x ) ) .
Assim, temos det A = kf (1) f ( i ) f ( −1) f ( −i ) , onde k é uma constante a ser
determinada. Fazendo a = 1 e b = c = d = 0, obtemos det A = 1 e
f (1) f ( i ) f ( −1) f ( −i ) = 1, logo k = 1, como queríamos demonstrar.

PROBLEMA 6
Defina

f ( x ) = ∑ an x n = a0 + a1 x + a2 x 2 + ... + an x n + ...
n =0
Então

EUREKA! N°32, 2010

63
Sociedade Brasileira de Matemática

( f ( x ) ) = ∑  ∑ a a  n
∞ n
2
l x
n −l

n =0 l =0 
= ( a0 a0 ) + ( a0 a1 + a1a0 ) x + ...
+ ( a0 an + a1an −1 + a2 an − 2 + ... + an a0 ) x n + ...
e

f´ ( x ) = ∑ ( n + 1) an +1 x n
n=0

= a1 + 2a2 x + ... + ( n + 1) an +1 x n + ...


π
Pela condição do enunciado, os coeficientes de f´ ( x ) e de ( f ( x ) ) coincidem,
2

3
π π
exceto pelo coeficiente constante. Temos portanto f´ ( x ) − ( f ( x ) ) = . Logo
2

3 3
temos

df π 2 df π
= ( f + 1) ⇔ ∫ 2 = ∫ dx ⇔
dx 3 f +1 3
π π
⇔ arctg f = ( x + C ) ⇒ f ( x ) = tg  ( x + C ) 
3 3 
Como f ( 0 ) = a0 = 0 , concluímos que C = 0, e portanto

ak 1 π 1
∑2
k =0
k
= f   = tg   =
2 6 3
.

EUREKA! N°32, 2010

64
Sociedade Brasileira de Matemática

XXXI OLIMPÍADA BRASILEIRA DE MATEMÁTICA


Problemas e soluções da Segunda Fase – Nível Universitário

PRIMEIRO DIA

PROBLEMA 1
Seja f :[0,1] → [0,1] crescente, derivável e inversível.
1 1
Se ∫
0
f ( x)dx = ∫ f −1 ( x)dx , prove que existem dois reais diferentes a e b,
0

0 ≤ a < b ≤ 1 , tais que f '(a ) = f '(b) = 1 .

Obs.: f −1 denota a inversa da função f .

PROBLEMA 2
Seja N = {0,1,2,3,...}. Dados conjuntos A, B ⊂ N , para cada inteiro positivo n
denote por r(A, B, n) o número de soluções da equação a + b = n, a ∈ A, b ∈ B .
Prove que existe n0 ∈ tal que r(A, B, n + 1)> r(A, B, n) para todo n > n0 se e
somente se N \ A e N \ B são finitos.

PROBLEMA 3
Dados n, a1 , a2 ,..., an inteiros positivos, definimos q0 = 1, q1 = a1 e
qk +1 = ak +1qk + qk −1 , para 1 ≤ k ≤ n − 1 .
Prove que, dado c > 1, existe K > 0 tal que, para todo M > K, existem n inteiro
positivo e a1 , a2 ,..., an pertencentes a {1,2} tais que M ≤ qn < c ⋅ M .

SEGUNDO DIA

PROBLEMA 4
Seja H o hiperboloide de equação 3x 2 + 3y 2 − z 2 − 1 = 0 .
i) Prove que todo ponto ( x, y, z ) ∈ H pertence a exatamente duas retas contidas
em H.
ii) Prove que todas as retas contidas em H formam o mesmo ângulo com o plano de
equação z = 0, e determine esse ângulo.

EUREKA! N°32, 2010

65
Sociedade Brasileira de Matemática

PROBLEMA 5
Ache todas as funções f : Z → Z que satisfazem:
i) f (f (n)) = f (n + 1), para todo n ∈ Z .
ii) f(2009n + 2008) = 2009.f(n) para todo n ∈ Z .

PROBLEMA 6
Para n inteiro positivo seja f(n) o número de produtos de inteiros maiores que 1
cujo resultado é no máximo n, isto é, f(n) é o número de k-uplas ( a1 , a2 ,..., ak )
onde k é algum natural, ai ≥ 2 é inteiro para todo i e a1 ⋅ a2 ⋅ ... ⋅ ak ≤ n (contando a
0-upla vazia ( ), cujo produto dos termos é 1).
Assim, por exemplo, f(1) = 1, por causa da 0-upla ( ) e f(6) = 9, por causa da 0-upla
( ), das 1-uplas (2), (3), (4), (5) e (6) e das 2-uplas (2, 2), (2, 3) e (3, 2).

1
Seja α > 1 tal que ∑ mα
m =1
= 2.

a) Prove que existe uma constante K > 0 tal que f ( n) ≤ K ⋅ nα para todo inteiro
positivo n.
b) Prove que existe uma constante c > 0 tal que f ( n) ≥ c ⋅ nα para todo inteiro
positivo n.

PROBLEMA 1: SOLUÇÃO DE MARLON DE OLIVEIRA GOMES (FORTALEZA – CE)


(I) Notemos primeiramente que f (0) = 0 e f (1) = 1. De fato, f é inversível e
portanto sobrejetora, logo, existem a e b ∈ [ 0,1] tais que f ( a ) = 0 e f ( b ) = 1.
Se a ≠ 0, b ≠ 1,
f ( t ) < 0, ∀t ∈ [ 0, a )
f ( t ) > 1, ∀t ∈ ( b,1]
Um absurdo.
(II) Afirmação: f possui um ponto fixo em (0, 1). Isto é, ∃c ∈ ( 0,1) tal que
f ( c ) = c.
Prova: Suponha que seja f ( x ) > x, ∀x ∈ ( 0,1) .

( )
Então, f −1 f ( x ) = x > f −1 ( x ) ∀x, pois f crescente ⇔ f −1 crescente.
Logo, seria f ( x ) > x > f −1 ( x ) , ∀x ∈ ( 0,1) ⇒

EUREKA! N°32, 2010

66
Sociedade Brasileira de Matemática

1 1 1
∫ f ( x ) dx > ∫ xdx > ∫ f −1 ( x ) dx, um absurdo, pois
0 0 0
1 1
∫ f ( x ) dx = ∫
0 0
f −1 ( x ) dx.
Se supusermos f ( x ) < x, ∀x ∈ ( 0,1) temos um resultado análogo.
Suponha agora que existam x1 e x2 ∈ ( 0,1) tais que f ( x1 ) > x1 e f ( x2 ) < x2 .
Sendo f diferenciável, é também contínua, donde g : [ 0,1] → , g (t ) = f (t ) − t é
contínua.
Note que g ( x1 ) > 0 e g ( x2 ) < 0, logo, pelo teorema do valor intermediário,
existe c ∈ ( x1 , x2 ) tal que g ( x ) = 0 ⇒ f ( c ) = c, o que prova o resultado.
(III) Pelo teorema do valor médio, existem a ∈ ( 0, c ) e b ∈ ( c,1)
 f (c)
 f (c) − f ( 0 ) = f ´( a ) ⋅ ( c − 0 ) ⇒ f ´( a ) = =1
 c
tais que  .
 f (1) − f c = f ´ b ⋅ 1 − c ⇒ f ´ b = 1 − f ( c ) = 1
 ( ) ( )( ) ( )
1− c

PROBLEMA 2: SOLUÇÃO DE RAMON MOREIRA NUNES (FORTALEZA – CE)


Para cada n ∈ , seja q ( A, B, n ) o número de pares ( x, y ) tais que x + y = n e,
além disso, x ∉ A ou y ∉ B . Assim, q ( A, B, n ) é o número de pares (x, y) com
x + y = n que r ( A, B, n ) não conta. Portanto, q ( A, B, n ) = n + 1 − r ( A, B, n ) .
Veja que: r ( A, B, n + 1) > r ( A, B, n ) ⇔
⇔ r ( A, B, n + 1) ≥ r ( A, B, n ) + 1 ⇔
⇔ n + 1 − q ( A, B, n + 1) ≥ n − q ( A, B, n ) + 1 ⇔
⇔ q ( A, B, n + 1) ≤ q ( A, B, n )
∀n ≥ n0 ; daí, q ( A, B, n ) ≤ q ( A, B, n0 ) , ∀n ≥ n0 .
Como só existem finitos n menores que n0 , isso nos diz que q ( A, B, n ) é limitada
como função de n.
Agora, se (x, y) é tal que x + y = n e x ∉ A, então ( x, y ) é contado por
q ( A, B, n ) . Dessa forma, q ( A, B, n ) ≥ # ({1, 2,..., n} \ A) .

EUREKA! N°32, 2010

67
Sociedade Brasileira de Matemática

Se \ A fosse infinito, poderíamos tomar # ({1, 2,..., n} \ A ) tão grande quanto


quiséssemos, mas como q ( A, B, n ) é limitada isso não pode ocorrer, logo, \A
é finito. Análogo \ B também é finito.
Agora, suponha \ A e \ B finitos.
 \ A = {a1 , a2 ..., ak } ; a1 < ... < ak

 \ B = {b1 , b2 ..., bm } ; b1 < ... < bm
tome n0 = ak + bm . Se n > n0 , tem-se que (x, y) com x + y = n então x > ak ou
y > bm pois x ≤ ak e y ≤ bm ⇒ n = x + y ≤ ak + bm = n0 . Absurdo.
Logo (x, y) não é contado por r ( A, B, n ) apenas quando x ∈ \ A ou y ∈ \B
(ambos não ocorrem simultaneamente); o primeiro caso ocorre n vezes e o segundo
m vezes. Assim, r ( A, B, n ) = n + 1 − k − m , e logo r satisfaz a condição do
enunciado.

PROBLEMA 3: SOLUÇÃO DA BANCA


Vamos escolher dois inteiros positivos grandes r, s e tomar m = r + s, a j = 1 para
1 ≤ j ≤ r e a j = 2 para r + 1 ≤ j ≤ r + s = m. Seja qk = qk ( a1 , a2 ,..., ak ) , para
1 ≤ k ≤ m. Temos qk +1 = qk + qk −1 , para 1 ≤ k ≤ r − 1, e portanto q j = Fj +1 , para
0 ≤ j ≤ r , onde, para j ≥ 1
1   1 + 5   1 − 5   1 + o (1)  1 + 5 
j j j

Fj =   −  =  
5   2   2   5  2 
 
é o j-ésimo termo da sequência de Fibonacci. Assim,
j
 1+ 5  1+ 5
q j = (1 + o (1) ) c   para j grande, onde c = . Por outro lado, temos
 2  2 5
qk +1 = 2qk + qk −1 para r ≤ k ≤ m − 1, e logo qr + j = u j +1qr + u j qr −1 , onde ( u j )
j ≥0

a sequência dada por u0 = 0, u1 = 1 e uk + 2 = 2uk +1 + uk , para k ≥ 0. Como


 = 1 + o (1) 1 + 2
1 
( ) − (1 − 2 ) ( )
k k k
uk =  1+ 2  , para k ≥ 0,
2 2  2 2
obtemos

EUREKA! N°32, 2010

68
Sociedade Brasileira de Matemática

1 + o (1)
((1 + 2 ) q + q ) (1 + 2 )
j
qr + j = r r −1
2 2
1 + o (1) 
r
5 −1  j  1+ 5 
= 1 + 2 +
2 
 c 1 + 2   ( )
2 2   2 
r
4 + 10 + 2 j  1+ 5 
= (1 + o (1) )
8
c 1 + 2   , ( )
 2 
para j e r grandes.
 1+ 5 
Como log 1 + 2 ( ) log   é irracional (pois não é possível termos
 2 
m
 1+ 5 
( )
n
1 + 2 =   , com m e n inteiros positivos), a conclusão do problema
 2 
segue (tirando logaritmos) do seguinte fato, que é provado a seguir:
Dados α , β > 0 tais que α β é irracional, ε > 0 e r > 0, existe x0 > 0 tal que,
para qualquer x ∈ , x ≥ x0 , existem inteiros m, n ≥ r tais que
mα + nβ − x < ε .

Para provar este fato, notemos que, se (p n qn ) n≥0


é a sequência de reduzidas da

fração contínua de α β , podemos escolher k natural com q 2 k +1 > β ε . Teremos


então (ver o artigo sobre Frações Contínuas na Revista Eureka! No. 3)
−ε β < −1 q 2 k + 2 < q 2 k +1 α β − p 2 k +1 < 0 < q 2 k α β − p 2 k < 1 q 2 k +1 < ε β
e portanto −ε < q 2 k +1α − p 2 k +1β < 0 < q 2 kα − p 2 k β < ε . Seja X o conjunto de
todos os números da forma aα + bβ com a, b ≥ 0 inteiros. Se
y = aα + bβ ≥ q 2 k +1α + p 2 k β pertence a X, temos a ≥ q 2 k +1 ou b ≥ p 2 k , e
portanto, tomando δ1 = p 2 k +1β − q 2 k +1α e δ 2 = q 2 kα − p 2 k β , temos
0 < δ1 , δ 2 < ε e teremos y + δ1 ∈ X ou y + δ 2 ∈ X . Usando este fato
repetidamente, segue que, para todo z ≥ y, [ z , z + ε ) ∩ X ≠ ∅. Como, para
 q α + p2k β 
k0 :=  2 k +1  , temos k0α ∈ X e k0α ≥ q 2 k +1α + p 2 k β , portanto, para
 α 

EUREKA! N°32, 2010

69
Sociedade Brasileira de Matemática

todo z ≥ k0α , [ z , z + ε ) ∩ X ≠ ∅. Assim, tomando x0 := k0α + r (α + β ) , temos


que, dado x ≥ x0 , temos x − r (α + β ) ≥ k0α , e portanto
 x − r (α + β ) , x − r (α + β ) + ε ) ∩ X ≠ ∅. Assim, existem a, b ≥ 0 inteiros
com aα + bβ ∈  x − r (α + β ) , x − r (α + β ) + ε ) , e portanto
x ≤ ( r + a ) α + ( r + b ) β < x + ε , o que prova o fato acima.

PROBLEMA 4: SOLUÇÃO DE GABRIEL LUIS MELLO DALALIO (S.J. DOS CAMPOS – SP)
Tomando um ponto ( x, y , z ) ∈ H , podemos representar uma reta que passa por
esse ponto como:
r : {( x + ka, y + kb, z + kc ) , ∀k ∈ }. Para ( a, b, c ) que façam r estar contida em
H teremos:
3 ( x + ka ) + 3 ( y + kb ) − ( z + kc ) − 1 = 0, ∀k ∈
2 2 2

⇒ 3 x 2 + 6 xka + 3k 2 a 2 + 3 y 2 + 6 ykb + 3k 2b 2 − z 2 − 2 zkc − k 2 c 2 − 1 = 0, ∀k ∈
Como 3 x 2 + 3 y 2 − z 2 − 1 = 0, pois ( x, y, z ) ∈ H tem-se:
3 xa + 3 yb − zc = 0 (I)
 2
3a + 3b − c = 0 (II)
2 2

(I) (II)

(3x, 3y, –z) 30o Ө


vetor normal
60o

Cone

Plano

Como o menor ângulo que a direção (a, b, c) forma com o plano z = 0 é igual a
 c 
arctg   , por (II) tem-se:
 a +b 
2 2

EUREKA! N°32, 2010

70
Sociedade Brasileira de Matemática

 
   c 
( )
c
θ = arctg   = arctg  2  = arctg 3 = 60°.
 a +b
2 2
  c 
 
 3 
Assim está provado ii), qualquer reta contida em H terá ângulo igual a 60° com o
plano z = 0.
A solução do sistema é a interseção do plano com o cone, que pode ser apenas a
origem, o que indicaria que não haveria r possível, ou pode ser igual a duas retas,
indicando duas direções possíveis para (a, b, c), ou seja, duas retas r possíveis, ou
ainda uma reta apenas de interseção, que é o caso de ( 3 x,3 y , − z ) ter ângulo
menor de 30° com o plano z = 0.
Para que haja interseção de duas retas, o vetor normal do plano ( 3 x,3 y, − z ) deve
ter um ângulo formado com o plano z = 0 menor que o ângulo do cone, que é de
30° . Esse ângulo é dado por
 z   z   1 
arctg   < arctg   = arctg   = 30°
 9x + 9 y 
2 2 2
 3z   3
 
( )
(pois 9 x 2 + 9 y 2 = 3 z 2 + 1 > 3 z 2 ).
 z 
Então, como arctg   < 30°, fica provado o item i).
 9x2 + 9 y2 
 
PROBLEMA 5: SOLUÇÃO DE JOSIAS ELIAS DOS SANTOS ROCHA (MURIBECA - SE)
Se f é injetiva, teremos f (n) = n + 1, pois f ( f (n) ) = f ( n + 1) e além disso a
função f (n) = n + 1 satisfaz (ii) f (2009n + 2008) = 2009 f ( n ) . Suponhamos
então que f (n1 ) = f ( n2 ) com n1 < n2 ⇒ f ( n1 ) = f ( n2 ) ⇒ f ( n1 +1) = f ( n2 +1) ;
2 2

indutivamente teremos f ( n1 + k ) = f ( n2 + k ) para todo k ≥ 0 , e assim f será


periódica a partir de n1 com periodo n2 − n1.
Daí f ( ∩ [ n1 , +∞ ) ) é finito, f ( ∩ [ n1 , +∞ ) ) = {a1 ,..., ak } .
Suponhamos sem perda de generalidade que a1 ≥ max ai : i = 1,...k { } e que
f ( m ) = a1 com m > 0 e m > n1 (f é periódica) ⇒ f ( 2009m + 2008 ) = 2009a1 ,
mas 2009m + 2008 ∈ ∩ [ n1 + ∞ ) ⇒ f ( 2009m + 2008 ) ∈ f ( ∩ [ n1 , +∞ ) )

EUREKA! N°32, 2010

71
Sociedade Brasileira de Matemática

⇒ 2009 a1 ≤ a1 ⇒ a1 = 0 ⇒ a1 = 0 ⇒ f ( ∩ [ n1 , +∞ ) ) = {0}.

Note-se ainda que f ( −1) = 0 pois basta fazer n = −1 em (ii)


⇒ 2009 f ( −1) = f ( −1) ⇒ f ( −1) = 0. Assim f ( −1) = f ( n1 ) = 0 ⇒ f é
periódica a partir de – 1 e f ( n ) = 0, ∀n ≥ −1. Seja n < −1, com
f ( n ) = r ⇒ f ( 2009n + 2008) = 2009r ⇒ f ( 2009 ( n + 1) ) = f ( 2009r ) . Assim,
ou r = n +1 ou f é periódica a partir de
2009 ( n + 1) ⇔ f ( )
∩  2009 ( n + 1) , +∞ ) = {0}. Fazendo m1 = 2009 ( n + 1) e
mk = 2009mk −1 + 2008, teremos −1 > m1 > m2 > ..., e além disso temos
f ( mk ) = 0,∀k pois f ( m1 ) = 0 e f ( mk +1 ) = f ( 2009mk + 2008) = 2009 f ( mk ) = 0

se f ( mk ) = 0. Mas para todo n < −1 podemos achar mi e mi +1 tais que


mi +1 ≤ n < mi ⇒ f ( n ) = 0, logo f ( n ) = 0, ∀n. Assim temos apenas as seguintes
soluções:

(1) f ( n ) = n + 1, ∀n ∈ . (2) f ( n ) = 0, ∀n ∈ .

 n + 1, se n ≤ −1
(3) f ( n ) =  .
0, se n ≥ −1
PROBLEMA 6: SOLUÇÃO DA BANCA

Vamos inicialmente estender a função f para [1,+∞) definindo


f ( x) = f ( x ) , ∀x ∈ [1,+∞) . Podemos agora mostrar que a função f
satisfaz a recorrência seguinte:
 
x
x
f ( x) = 1 + ∑ f ( ), ∀x ∈ [1, +∞) . De fato, temos o vetor ( ) (correspondente a
m=2 m
k = 0 ), e, se k ≥ 1 e (a1 , a2 ,..., ak ) é tal que a j é inteiro, a j ≥ 2, ∀j e
x x
a1a2 ...ak ≤ x , então 2 ≤ a1 ≤  x  e, se a1 = m , a2 ...ak ≤ e há f  
m m
possíveis escolhas para (a2 ,..., ak ) .

EUREKA! N°32, 2010

72
Sociedade Brasileira de Matemática


1
Note também que α < 2 , pois a função ζ (α ) = ∑ mα
m =1
é decrescente e

1 1 1 1 1 1 1 1 1 1 1 1 1
∑m
m =1
2
= 1+ + + + + +
4 9 16 25 36 49
+...<1+ + + + + + +...
4 4 16 16 16 16
1 1
=1+ + +...= 2.
2 4
Vamos resolver o item a): Mostraremos que f ( x ) ≤ xα , ∀x ≥ 1, por indução em
x
 x  . Temos f ( x ) = 1 ≤ x , ∀x ∈ (1, 2] . Para x ≥ 2 temos   <  x  , ∀m ≥ 2,
α

m
e portanto, por hipótese de indução,
α
 x 
x
 x 
x  1 
f ( x ) = 1 + ∑ f   ≤ 1 + ∑   = 1 + xα  1 − ∑ α 
m m=2  m 

m=2  m > x  m 

1  ∞ 1 
(pois ∑ mα
m=2
= ∑ α
 m =1 m
 − 1 = 1).

α α α α α
1  x   x   x   x   x 
Como x ∑ α > 
α
 +  +  +  +  ≥
m >  x  m  x +1   x + 2   x + 3   x + 4   x + 5 
α α α α α 2 2 2 2 2
 2  2  2  2  2  2  2  2  2  2
≥   +   +   +   +   >   +   +   +   +   > 1, para
 3  4  5  6  7  3  4  5  6  7
todo x ≥ 2, segue que f ( x ) < xα , o que prova o resultado.
Vamos agora resolver o item b). Mostraremos inicialmente que 2α > 3. De fato,
1 1 1 1 1 1 1 2 4 1 1 1 1 
1= α
+ α + α + ... > α + α + α + ... = α + α + α + ... =  α −1 + α −1 + α −1 + ...
2 3 4 2 4 4 2 4 8 2 2 4 8 
1 1
⋅ α −1
= 2 2 = 1 , donde 2α − 2 > 1 e logo 2α > 3. Vamos mostrar que, se
α
1
1 − α −1 2 − 2
2
3k 1
k ≥ 6 e x < 2k , então f ( x ) ≥ cxα ⋅ k ≥ cxα , onde c = o que
3 −9⋅2 k
20480
claramente implica o resultado. Note que essa desigualdade vale para k = 6 e
x < 26 , pois nesse caso

EUREKA! N°32, 2010

73
Sociedade Brasileira de Matemática

3k α 81 xα x2 212
cx kα
= cx α
< 5⋅c ⋅ x = < < = 1 ≤ f ( x).
3 − 9 ⋅ 2k 17 4096 4096 4096
Vamos agora mostrar essa desigualdade por indução em k.
Suponhamos que ela vale para um certo k ≥ 6.
3k +1
Mostraremos que f ( x ) ≥ cxα ⋅ para todo x com 2k ≤ x < 2k +1. Para
3k +1 − 9 ⋅ 2k +1
x
um tal valor de x, temos 1 ≤ < 2k para 2 ≤ m ≤  x  , e logo
m
α
x
 x 2k
 x 3 k 2k
 x 3k  1 
f ( x) =1+ ∑ f   > ∑ f   ≥ c ⋅ k k ∑
⋅  = c ⋅ ⋅xα ⋅ 1− ∑ α .
m=2  m  m=2  m  3 − 9⋅ 2 m=2  m  3 − 9⋅ 2
k k
 m>2k m 
1 1 1 1 1 1 1
Temos ∑ α < ∑ α = + +... + + ( k+1)α +... + +... <
( 2 ) ( 2 +1) ( 2 −1) 2 ( 2 −1)
α α α α
m>2k m m≥2k m
k k k +1 k +2

k k +1
2k 2 k +1 1 1 2 2
< kα
+ ( k +1)α
+ ... = k (α −1) + ( k +1)(α −1) + ... <   +   + ...
2 2 2 2 3 3
k k +1 k
 α −1 2α 3 2 2 2
 pois 2 = >  , e, como   +   + ... = 3 ⋅   , temos
 2 2 3 3 3
3k  2 
k
3 k +1
f (x) > c ⋅ k ⋅ x α
⋅  1 − 3     > c ⋅ ⋅ xα , pois
3 − 9 ⋅ 2k   3  3 k +1
− 9 ⋅ 2 k +1
 
3k   2   3 − 3⋅ 2
k k k
3k +1
⋅  1 − 3 ⋅     = > (de fato, essa última
3k − 9 ⋅ 2k   3   3 −9⋅2
k k
3k +1 − 9 ⋅ 2k +1
6 ⋅ 2k 9 ⋅ 2 k +1
desigualdade pode ser escrita como 1 + k > 1 + k +1 , que
3 − 9 ⋅ 2k 3 − 9 ⋅ 2 k +1
( ) ( )
equivale a 6 3k +1 − 9 ⋅ 2k +1 > 18 3k − 9 ⋅ 2k , que por sua vez equivale a
162 ⋅ 2k > 108 ⋅ 2k , que obviamente vale para todo k).
Obs: α é aproximadamente igual a 1,7286472389....
f ( x) −1
Kálmar provou que lim = = 0,3181736521...
x→∞ xα
αζ ´(α )

EUREKA! N°32, 2010

74
Sociedade Brasileira de Matemática

XXXI OLIMPÍADA BRASILEIRA DE MATEMÁTICA


PREMIADOS
Nível 1 (6º. e 7º. Anos)
NOME CIDADE – ESTADO PRÊMIO
Alexandre Mendonça Cardoso Salvador – BA Ouro
Daniel de Almeida Souza Brasília – DF Ouro
Pedro Henrique Alencar Costa Fortaleza – CE Ouro
Ana Beatriz Motta Aragão Cortez Campinas – SP Ouro
Cristhian Mafalda Leme – SP Ouro
Érika Rizzo Aquino Goiânia – GO Ouro
Bianca Lima Barretto Salvador – BA Prata
Adriana de Sousa Figueiredo Porto Alegre – RS Prata
Ricardo Ken Wang Tsuzuki São Paulo – SP Prata
João Pedro Sedeu Godoi Rio de Janeiro – RJ Prata
Leonardo Gomes Gonçalves Brasília – DF Prata
Leonardo Gushiken Yoshitake São Paulo – SP Prata
Paulo Henrique Omena de Freitas São Paulo – SP Prata
Edgar Kenji Ishida São Paulo – SP Prata
Rodrigo Pommot Berto Brasília – DF Prata
Kiane Sassaki Menezes Rio de Janeiro – RJ Prata
Dimas Macedo de Albuquerque Fortaleza – CE Prata
Mauricio Najjar da Silveira São Paulo – SP Bronze
Murilo Corato Zanarella Amparo – SP Bronze
Victor Almeida Costa Fortaleza – CE Bronze
Elcio Koodiro Yoshida São Paulo – SP Bronze
Carolina Lima Guimarães Vitória – ES Bronze
Bruno da Silveira Dias Florianópolis – SC Bronze
Emilly Guaris Costa Maceió – AL Bronze
Bruno Almeida Costa Fortaleza – CE Bronze
Gabriel Averbug Zukin Rio de Janeiro – RJ Bronze
Marcelo Ericsson de Carvalho São Paulo – SP Bronze
Sarah Barreto Ornellas Salvador – BA Bronze
Isabella Ayres Pinheiro de Lima Goiânia – GO Bronze
Shadi Bavar Blumenau – SC Bronze
Viviane Silva Souza Freitas Salvador – BA Bronze
Matheus José Araújo Oliveira Recife – PE Bronze
Beatriz Miranda Macedo Niterói – RJ Bronze
Matheus Uchôa Constante Goiânia – GO Bronze
Julio S. Akiyoshi São Paulo – SP Menção Honrosa
Antonio Wesley de Brito Vieira Cocal de Alves – PI Menção Honrosa
Vinicius Jóras Padrão Rio de Janeiro – RJ Menção Honrosa
Mateus Guimarães Lima de Freitas Fortaleza – CE Menção Honrosa
Vitor Dias Gomes Barrios Marin Presidente Prudente – SP Menção Honrosa
Mariana Teatini Ribeiro Belo Horizonte – MG Menção Honrosa
Rodrigo Silva Ferreira Salvador – BA Menção Honrosa
Artur Souto Martins Fortaleza – CE Menção Honrosa
Tiago Martins Nápoli Itú – SP Menção Honrosa

EUREKA! N°32, 2010

75
Sociedade Brasileira de Matemática

Laís Monteiro Pinto Rio de Janeiro – RJ Menção Honrosa


Guilherme Anitele Silva Presidente Prudente – SP Menção Honrosa
Pedro Papa Paniago Belo Horizonte – MG Menção Honrosa
Gabriel Yudi Hirata São Paulo – SP Menção Honrosa
Iago Carvalho de Moraes Recife – PE Menção Honrosa
Adam Yuuki Oyama Curitiba – PR Menção Honrosa
Luíze Mello D´urso Vianna Rio de Janeiro – RJ Menção Honrosa
Enrico Pascucci Loffel S.B.do Campo – SP Menção Honrosa
Daniel Charles M. Gomes Mogi das Cruzes – SP Menção Honrosa
Ellen Tamie Ikefuti Morishigue Bastos – SP Menção Honrosa
Ana Emília Hernandes Dib S.J. do Rio Preto – SP Menção Honrosa
Marcelo Liu Guo São Paulo – SP Menção Honrosa
Gabriel Queiroz Moura Teresina – PI Menção Honrosa
Gabriel Branco Frizzo Curitiba – PR Menção Honrosa
Ana Jéssyca Mendes Belarmino Fortaleza – CE Menção Honrosa
Mariana Bonfim Moraes Morant de Holanda Rio de Janeiro – RJ Menção Honrosa
Ricardo Vidal Mota Peixoto Vassouras – RJ Menção Honrosa
Bruno de Marchi Andrade Valinhos – SP Menção Honrosa
Juliana Amoedo Amoedo Plácido Salvador – BA Menção Honrosa

Nível 2 (8º. e 9º. Anos)


NOME CIDADE – ESTADO PRÊMIO
André Macieira Braga Costa Belo Horizonte – MG Ouro
Gabriel Ilharco Magalhães Juiz de Fora – MG Ouro
Daniel Eiti Nishida Kawai Atibaia – SP Ouro
Henrique Gasparini Fiuza do Nascimento Brasília – DF Ouro
Marina Pessoa Mota Fortaleza – CE Ouro
Fellipe Sebastiam S. P. Pereira Rio de Janeiro – RJ Ouro
Liara Guinsberg São Paulo – SP Prata
Lara Timbó Araújo Fortaleza – CE Prata
Victor Kioshi Higa São Paulo – SP Prata
Fernando Lima Saraiva Filho Eusébio – CE Prata
Lucas Cawai Julião Pereira Caucaia – CE Prata
Mateus Henrique Ramos de Souza Pirapora – MG Prata
Henrique Vieira Gonçalves Vaz São Paulo – SP Prata
Lucas Nishida Pedreira – SP Prata
Pedro Víctor Falci de Rezende Juiz de Fora – MG Prata
Rafael Kazuhiro Miyazaki São Paulo – SP Prata
Francisco Markan Nobre de Souza Filho Fortaleza – CE Bronze
Vincent Cherng Hsi Lee São Paulo – SP Bronze
Vinícius Canto Costa Salvador – BA Bronze
Thiago Poeiras Silva Belo Horizonte – MG Bronze
Victor de Oliveira Bitaraes Betim – MG Bronze
Victor Hugo Corrêa Rodrigues Rio de Janeiro – RJ Bronze
Luciano Drozda Dantas Martins Fortaleza – CE Bronze
Breno Leví Corrêa Campo Belo – MG Bronze

EUREKA! N°32, 2010

76
Sociedade Brasileira de Matemática

Wilson Aparecido Sedano Filho Paulínia – SP Bronze


Victor Venturi Campinas – SP Bronze
Daniel Lima Santanelli Rio de Janeiro – RJ Bronze
Felipe Penha Alves São Luís – MA Bronze
Lucas Grimauth Evangelista São Paulo – SP Bronze
Gabriel Nogueira Coelho de Togni de Souza Rio de Janeiro – RJ Bronze
Ana Thais Castro de Santana Rio de Janeiro – RJ Bronze
Caio Cesar do Prado Dorea Reis Nova Iguaçu – RJ Bronze
Rafael Rodrigues Rocha de Melo Caucaia – CE Menção Honrosa
Murilo Freitas Yonashiro Coelho São Paulo – SP Menção Honrosa
Gabriel José Moreira da Costa Silva Maceió – AL Menção Honrosa
Nathalia Novello Fernandes Ribeiro Rio de Janeiro – RJ Menção Honrosa
Gabriel Pacianotto Gouveia São Paulo – SP Menção Honrosa
Pedro Ivo Coêlho de Araújo Fortaleza – CE Menção Honrosa
Elias Brito Oliveira Brasília – DF Menção Honrosa
Fernando Tomimura Miyashiro São Paulo – SP Menção Honrosa
Igor Augusto Marques do Carmo Juiz de Fora – MG Menção Honrosa
Juliane Trianon Fraga São Paulo – SP Menção Honrosa
Aimê Parente de Sousa Fortaleza – CE Menção Honrosa
Tadeu Pires de Matos Belfort Neto Fortaleza – CE Menção Honrosa
Yuri Zeniti Sinzato Brasília – DF Menção Honrosa
Gabriel Sena Galvão Brasília – DF Menção Honrosa
Filipe Mourão Leite Teresina – PI Menção Honrosa
Gabriela Loiola Vilar Fortaleza – CE Menção Honrosa
Marcelo Cargnelutti Rossato Santa Maria – RS Menção Honrosa
Pedro Henrique Botolozo Maria Curitiba – PR Menção Honrosa
Jair Gomes Soares Júnior Montes Claros – MG Menção Honrosa
Maria Clara Cardoso São Paulo – SP Menção Honrosa
Júlio César Prado Soares Brasília – DF Menção Honrosa
Fábio Kenji Arai São Paulo – SP Menção Honrosa
Julio Barros de Paula Taubaté – SP Menção Honrosa
Francisco Matheus Gonçalves de Souza João Pessoa – PB Menção Honrosa
Daniel Kantorowitz Bragança Paulista – SP Menção Honrosa
Ivan Tadeu Ferreira Antunes Filho Lins – SP Menção Honrosa
Vitor Ramos de Paula Belo Horizonte – MG Menção Honrosa
Victor Santos de Andrade Teresina – PI Menção Honrosa

Nível 3 (Ensino Médio)


NOME CIDADE – ESTADO PRÊMIO
Renan Henrique Finder São Paulo – SP Ouro
Marcelo Tadeu de Sá Oliveira Sales Salvador – BA Ouro
Davi Lopes Alves de Medeiros Fortaleza – CE Ouro
Matheus Secco Torres da Silva Rio de Janeiro – RJ Ouro
Hugo Fonseca Araújo Rio de Janeiro – RJ Ouro
Marco Antonio Lopes Pedroso Santa Isabel – SP Ouro
Gustavo Lisbôa Empinotti Florianópolis – SC Prata

EUREKA! N°32, 2010

77
Sociedade Brasileira de Matemática

Marlen Lincoln da Silva Fortaleza – CE Prata


Deborah Barbosa Alves São Paulo – SP Prata
Illan Feiman Halpern São Paulo – SP Prata
Thiago Ribeiro Ramos Varginha – MG Prata
Hanon Guy Lima Rossi São Paulo – SP Prata
João Lucas Camelo Sá Fortaleza – CE Prata
Carlos Henrique de Andrade Silva Fortaleza – CE Prata
Custódio Moreira Brasileiro Silva Embu – SP Prata
Rafael Alves da Ponte Fortaleza – CE Prata
Matheus Barros de Paula Taubaté – SP Bronze
Bruno Silva Mucciaccia Vitória – ES Bronze
Matheus Araujo Marins São Gonçalo – RJ Bronze
Guilherme da Rocha Dahrug Santo André – SP Bronze
Victorio Takahashi Chu São Paulo – SP Bronze
Voltaire Laplace dos Reis Manhuaçu – MG Bronze
Jardiel Freitas Cunha Recife – PE Bronze
Rafael Horimoto de Freitas São Paulo – SP Bronze
Lucas de Freitas Smaira Guaxupé – MG Bronze
Robério Soares Nunes Riberio Preto – SP Bronze
Gabriel Militão Vinhas Lopes Fortaleza – CE Bronze
Alvaro Lopes Pedroso Santa Isabel – SP Bronze
Alan Anderson da Silva Pereira União dos Palmares – AL Bronze
Maria Clara Mendes Silva Pirajuba – MG Bronze
Nara Gabriela de Mesquita Peixoto Fortaleza – CE Bronze
Rodrigo de Sousa Serafim da Silva Itatiba – SP Bronze
Rodrigo Rolim Mendes de Alencar Fortaleza – CE Menção Honrosa
João Mendes Vasconcelos Fortaleza – CE Menção Honrosa
Otávio Augusto de Oliveira Mendes Pilar do Sul – SP Menção Honrosa
Renan Roveri do Amaral Gurgel Jundiaí – SP Menção Honrosa
Fernando Fonseca Andrade Oliveira Belo Horizonte – MG Menção Honrosa
Caíque Porto Lira Fortaleza – CE Menção Honrosa
Gustavo Haddad Francisco e S. Braga S.J.dos Campos – SP Menção Honrosa
Gustavo Cellet Marques São Paulo – SP Menção Honrosa
Wagner Rosales Chaves Jundiaí – SP Menção Honrosa
Wagner Carlos Morêto Loyola Filho Vitória – ES Menção Honrosa
James Jun Hong São Paulo – SP Menção Honrosa
Kayo de França Gurgel Fortaleza – CE Menção Honrosa
Nathana Alcântara Lima Fortaleza – CE Menção Honrosa
Gabriel Lima Guimarães Vitória – ES Menção Honrosa
Ivan Guilhon Mitoso Rocha Fortaleza – CE Menção Honrosa
Elder Massahiro Yoshida São Paulo – SP Menção Honrosa
Ruan Alves Pires Rio de Janeiro – RJ Menção Honrosa
André Saraiva Nobre dos Santos Fortaleza – CE Menção Honrosa
Luiz Filipe Martins Ramos Niterói – RJ Menção Honrosa
Felipe Abella C. Mendonça de Souza João Pessoa – PB Menção Honrosa
Eduardo Machado Capaverde Florianópolis – SC Menção Honrosa
Thales Sinelli Lima São Paulo – SP Menção Honrosa
Tuane Viana Pinheiro Rio de Janeiro – RJ Menção Honrosa
Vinícius Cipriano Klein Viçosa – MG Menção Honrosa

EUREKA! N°32, 2010

78
Sociedade Brasileira de Matemática

André Austregésilo Scussel Fortaleza – CE Menção Honrosa


Thiago Augusto da Silva Baleixo Rio de Janeiro – RJ Menção Honrosa
Marcos Massayuki Kawakami São Paulo – SP Menção Honrosa
Ana Luísa de Almeida Losnak São Paulo – SP Menção Honrosa

Nível Universitário
NOME CIDADE – ESTADO PRÊMIO
Leonardo Ribeiro de Castro Carvalho São Paulo – SP Ouro
Régis Prado Barbosa Fortaleza – CE Ouro
Rafael Assato Ando Campinas – SP Ouro
Rafael Daigo Hirama Campinas – SP Ouro
Guilherme Rodrigues Nogueira de Souza São Paulo – SP Ouro
Adenilson Arcanjo de Moura Junior Fortaleza – CE Ouro
Felipe Rodrigues Nogueira de Souza São Paulo – SP Ouro
Ramon Moreira Nunes Fortaleza – CE Ouro
Thomás Yoiti Sasaki Hoshina Rio de Janeiro – RJ Prata
Rafael Tupynambá Dutra Belo Horizonte – MG Prata
Luís Fernando Schultz Xavier da Silveira Florianópolis – SC Prata
Thiago Costa Leite Santos São Paulo – SP Prata
Gabriel Ponce São Carlos – SP Prata
Carlos Henrique Melo Souza S.J. dos Campos – SP Prata
Marcelo Matheus Gauy São Paulo – SP Prata
Antônio Felipe Cavalcante Carvalho Fortaleza – CE Prata
Kellem Corrêa Santos Rio de Janeiro – RJ Prata
Rafael Montezuma Pinheiro Cabral Fortaleza – CE Prata
Ricardo Turolla Bortolotti Rio de Janeiro – RJ Bronze
Alexandre Hideki Deguchi Martani São Paulo – SP Bronze
Rafael Sampaio de Rezende Fortaleza – CE Bronze
Maurício de Lemos Rodrigues Collares Neto Aracajú – SE Bronze
Joas Elias dos Santos Rocha Muribeca – SE Bronze
Gabriel Luís Mello Dalalio S.J. dos Campos – SP Bronze
Carlos Coelho Lechner Rio de Janeiro – RJ Bronze
Enzo Haruo Hiraoka Moriyama São Paulo – SP Bronze
Paulo Sérgio de Castro Moreira Fortaleza – CE Bronze
Helder Toshiro Susuki São Paulo – SP Bronze
Mateus Oliveira de Figueiredo Fortaleza – CE Bronze
Gabriel Caser Brito Rio de Janeiro – RJ Bronze
Paulo André Carvalho de Melo S.J. dos Campos – SP Bronze
Caio Ishizara Costa S.J. dos Campos – SP Bronze
Willy George do Amaral Petrenko Rio de Janeiro – RJ Bronze
Sidney Cerqueira Bispo dos Santos Filho S.J. dos Campos – SP Bronze
Rafael Endlich Pimentel Vitória – ES Bronze
Guilherme Philippe Figueiredo São Paulo – SP Bronze
Bruno da Silva Santos Belford Roxo – RJ Bronze
Francisco Osman Pontes Neto Fortaleza – CE Bronze
Luty Rodrigues Ribeiro S.J. dos Campos – SP Bronze

EUREKA! N°32, 2010

79
Sociedade Brasileira de Matemática

Renato Rebouças de Medeiros Fortaleza – CE Menção Honrosa


José Olegário de Oliveira Neto S.J. dos Campos – SP Menção Honrosa
Eduardo Fischer Encantado – RS Menção Honrosa
Guilherme Lourenço Mejia S.J. dos Campos – SP Menção Honrosa
Jorge Henrique Craveiro de Andrade Rio de Janeiro – RJ Menção Honrosa
Edson Augusto Bezerra Lopes Fortaleza – CE Menção Honrosa
Eric Campos Bastos Guedes Niterói – RJ Menção Honrosa
Thiago da Silva Pinheiro São Paulo – SP Menção Honrosa
Leandro Farias Maia Fortaleza – CE Menção Honrosa
Pedro Paulo Albuquerque Goes Fortaleza – CE Menção Honrosa
Álvaro Krüger Ramos Porto Alegre – RS Menção Honrosa
Alysson Espíndola de Sá Silveira Fortaleza – CE Menção Honrosa
Alfredo Roque de Oliveira Freire Filho S.J. dos Campos – SP Menção Honrosa
Rafael Ghussn Cano Campinas – SP Menção Honrosa
Antônio Deromir Neves da Silva Júnior Fortaleza – CE Menção Honrosa
Rafael Sabino Lima Rio de Janeiro – RJ Menção Honrosa
Hudson do Nascimetno Lima Fortaleza – CE Menção Honrosa
Diego Andrés de Barros Lima Barbosa Rio de Janeiro – RJ Menção Honrosa
Reinan Ribeiro Souza Santos Aracaju – SE Menção Honrosa
Marcos Victor Pereira Vieira Fortaleza – CE Menção Honrosa
Daniel Ungaretti Borges Belo Horizonte – MG Menção Honrosa

EUREKA! N°32, 2010

80
Sociedade Brasileira de Matemática

AGENDA OLÍMPICA
XXXII OLIMPÍADA BRASILEIRA DE MATEMÁTICA

NÍVEIS 1, 2 e 3
Primeira Fase – Sábado, 12 de junho de 2010
Segunda Fase – Sábado, 18 de setembro de 2010
Terceira Fase – Sábado, 16 de outubro de 2010 (níveis 1, 2 e 3)
Domingo, 17 de outubro de 2010 (níveis 2 e 3 - segundo dia de prova).

NÍVEL UNIVERSITÁRIO
Primeira Fase – Sábado, 18 de setembro de 2010
Segunda Fase – Sábado, 16 e Domingo, 17 de outubro de 2010

ASIAN PACIFIC MATH OLYMPIAD (APMO)


06 de março de 2010

XVI OLIMPÍADA DE MAIO


08 de maio de 2010

XXI OLIMPÍADA DE MATEMÁTICA DO CONE SUL


13 a 19 de junho de 2010
Águas de São Pedro, SP – Brasil

LI OLIMPÍADA INTERNACIONAL DE MATEMÁTICA


02 a 14 de julho de 2010
Astana, Cazaquistão

XVII OLIMPÍADA INTERNACIONAL DE MATEMÁTICA UNIVERSITÁRIA


24 a 30 de julho de 2010
Blagoevgrad, Bulgária

XXIV OLIMPÍADA IBEROAMERICANA DE MATEMÁTICA


17 a 27 de setembro de 2010
Paraguai

II COMPETIÇÃO IBEROAMERICANA INTERUNIVERSITÁRIA DE MATEMÁTICA


3 a 9 de outubro de 2010
Rio de Janeiro, Brasil

XIII OLIMPÍADA IBEROAMERICANA DE MATEMÁTICA UNIVERSITÁRIA

EUREKA! N°32, 2010

81
Sociedade Brasileira de Matemática

COORDENADORES REGIONAIS
Alberto Hassen Raad (UFJF) Juiz de Fora – MG
Américo López Gálvez (USP) Ribeirão Preto – SP
Andreia Goldani FACOS Osório – RS
Antonio Carlos Nogueira (UFU) Uberlândia – MG
Benedito Tadeu Vasconcelos Freire (UFRN) Natal – RN
Carmen Vieira Mathias (UNIFRA) Santa María – RS
Claus Haetinger (UNIVATES) Lajeado – RS
Cláudio de Lima Vidal (UNESP) S.J. do Rio Preto – SP
Denice Fontana Nisxota Menegais (UNIPAMPA) Bagé – RS
Disney Douglas Lima de Oliveira (UFAM) Manaus – AM
Edson Roberto Abe (Colégio Objetivo de Campinas) Campinas – SP
Edney Aparecido Santulo Jr. (UEM) Maringá – PR
Élio Mega (Grupo Educacional Etapa) São Paulo – SP
Eudes Antonio da Costa (Univ. Federal do Tocantins) Arraias – TO
Fábio Brochero Martínez (UFMG) Belo Horizonte – MG
Florêncio Ferreira Guimarães Filho (UFES) Vitória – ES
Francinildo Nobre Ferreira (UFSJ) São João del Rei – MG
Genildo Alves Marinho (Centro Educacional Leonardo Da Vinci) Taguatingua – DF
Graziela de Souza Sombrio (UNOCHAPECÓ) Chapecó – SC
Gilson Tumelero (UTFPR) Pato Branco – PR
Ivanilde Fernandes Saad (UC. Dom Bosco) Campo Grande – MS
João Benício de Melo Neto (UFPI) Teresina – PI
João Francisco Melo Libonati (Grupo Educacional Ideal) Belém – PA
Jose de Arimatéia Fernandes (UFPB) Campina Grande – PB
José Luiz Rosas Pinho (UFSC) Florianópolis – SC
José Vieira Alves (UFPB) Campina Grande – PB
José William Costa (Instituto Pueri Domus) Santo André – SP
Krerley Oliveira (UFAL) Maceió – AL
Licio Hernandes Bezerra (UFSC) Florianópolis – SC
Luciano G. Monteiro de Castro (Sistema Elite de Ensino) Rio de Janeiro – RJ
Luzinalva Miranda de Amorim (UFBA) Salvador – BA
Marcelo Rufino de Oliveira (Grupo Educacional Ideal) Belém – PA
Marcelo Mendes (Colégio Farias Brito, Pré-vestibular) Fortaleza – CE
Newman Simões (Cursinho CLQ Objetivo) Piracicaba – SP
Nivaldo Costa Muniz (UFMA) São Luis – MA
Nivaldo de Góes Grulha Jr. (USP – São Carlos) São Carlos – SP
Osnel Broche Cristo (UFLA) Lavras – MG
Uberlândio Batista Severo (UFPB)) João Pessoa – PB
Raul Cintra de Negreiros Ribeiro (Colégio Anglo) Atibaia – SP
Ronaldo Alves Garcia (UFGO) Goiânia – GO
Rogério da Silva Ignácio (Col. Aplic. da UFPE) Recife – PE
Reginaldo de Lima Pereira (Escola Técnica Federal de Roraima) Boa Vista – RR
Reinaldo Gen Ichiro Arakaki (UNIFESP) SJ dos Campos – SP
Ricardo Amorim (Centro Educacional Logos) Nova Iguaçu – RJ
Sérgio Cláudio Ramos (IM-UFRGS) Porto Alegre – RS
Seme Gebara Neto (UFMG) Belo Horizonte – MG
Tadeu Ferreira Gomes (UEBA) Juazeiro – BA
Tomás Menéndez Rodrigues (U. Federal de Rondônia) Porto Velho – RO
Valdenberg Araújo da Silva (U. Federal de Sergipe) São Cristovão – SE
Vânia Cristina Silva Rodrigues (U. Metodista de SP) S.B. do Campo – SP
Wagner Pereira Lopes (CEFET – GO) Jataí – GO

EUREKA! N°32, 2010

82
CONTEÚDO
XVI OLIMPÍADA DE MAIO
Enunciados e resultado brasileiro 2

XXI OLIMPÍADA DE MATEMÁTICA DO CONE SUL


Enunciados e resultado brasileiro 5

LI OLIMPÍADA INTERNACIONAL DE MATEMÁTICA (IMO)


Enunciados e resultado brasileiro 7

XXV OLIMPÍADA IBEROAMERICANA DE MATEMÁTICA


Enunciados e resultado brasileiro 9

ARTIGOS

ASSOCIANDO UM POLINÔMIO A EXPRESSÕES ALGÉBRICAS E


TRIGONOMÉTRICAS
Marcílio Miranda 11

SOMAS TRIGONOMÉTRICAS: DE PROSTAFÉRESE A FÓRMULA DE EULER


Rogério Possi Junior 18

UMA INTERESSANTE DEDUÇÃO PARA A FÓRMULA DE HERÃO


Flávio Antonio Alves 31

RAÍZES DA UNIDADE
Anderson Torres & Eduardo Tengan 33

COMO É QUE FAZ 42

SOLUÇÕES DE PROBLEMAS PROPOSTOS 45

PROBLEMAS PROPOSTOS 59

AGENDA OLÍMPICA 61

COORDENADORES REGIONAIS 62
Sociedade Brasileira de Matemática

XVI OLIMPÍADA DE MAIO


PRIMEIRO NÍVEL

PROBLEMA 1
Um recipiente fechado com formato de paralelepípedo retangular contém 1 litro de
água. Se o recipiente se apoia horizontalmente sobre três faces distintas, o nível da
água é de 2cm, 4cm e 5cm.
Calcule o volume do paralelepípedo.

PROBLEMA 2
Na etapa 0 escrevem-se os números 1, 1.
Na etapa 1 intercala-se a soma dos números 1, 2, 1.
Na etapa 2 entre cada par de números da etapa anterior intercala-se a soma deles:
1, 3, 2, 3, 1.
Uma etapa mais: 1, 4, 3, 5, 2, 5, 3, 4, 1.
Quantos números há na etapa 10?
Qual é a soma de todos os números que há na etapa 10?

PROBLEMA 3
É possível pintar os inteiros positivos com três cores de modo que, sempre que se
somam dois números de cores distintas, o resultado da soma seja da terceira cor?
(Há que usar as três cores.) Se a resposta é afirmativa, indique um possível modo
de pintar; se não é possível, explique o porquê.

PROBLEMA 4
Encontre todos os números naturais de 90 dígitos que são múltiplos de 13 e têm os
primeiros 43 dígitos iguais entre si e distintos de zero, os últimos 43 dígitos iguais
entre si, e os 4 dígitos do meio são 2, 0, 1, 0, nessa ordem.

PROBLEMA 5
Num tabuleiro de 2 × 7 quadriculado em casas de 1 × 1 se consideram os 24 pontos
que são vértices das casas. João e Matias jogam sobre este tabuleiro. João pinta de
vermelho uma quantidade igual de pontos em cada uma das três linhas horizontais.
Se Matias pode escolher três pontos vermelhos que sejam vértices de um triângulo
acutângulo, Matias vence o jogo. Qual é a máxima quantidade de pontos que João
pode pintar para ter certeza de que Matias não vencerá? (Para o número
encontrado, dê um exemplo de pintura que impeça que Matias vença e justifique
por quê Matias vence sempre se o número é maior.)

EUREKA! N°33, 2011

2
Sociedade Brasileira de Matemática

SEGUNDO NÍVEL
PROBLEMA 1
Determine o menor inteiro positivo que tenha todos seus dígitos iguais a 4, e que
seja múltiplo de 169.

PROBLEMA 2
Consideramos o retângulo ABCD e a circunferência de centro D e raio DA, que
corta o prolongamento do lado AD no ponto P. A reta PC corta a circunferência no
ponto Q e o prolongamento do lado AB no ponto R. Demonstre que QB = BR.

PROBLEMA 3
Encontre o menor k > 2 para o qual existem k números inteiros consecutivos, tais
que a soma dos seus quadrados é um quadrado.

PROBLEMA 4
Seja n um inteiro tal que 1 < n < 2010. Dado um polígono regular de 2010 lados e
n moedas, devemos pintar os vértices do polígono utilizando n cores dadas, e logo
colocar as n moedas em n vértices do polígono. Em seguida, a cada segundo, todas
as moedas se deslocam para o vértice vizinho, girando no sentido dos ponteiros do
relógio.
Determine os valores de n para os quais é possível pintar e escolher as posições
iniciais das moedas, de forma que em todo momento as n moedas estejam todas em
vértices de cores distintas.

PROBLEMA 5
Temos as seguintes peças: um retângulo de 4 × 1, dois retângulos de 3 × 1, três
retângulos de 2 × 1 e quatro quadrados de 1 × 1. Ariel e Bernardo jogam o seguinte
jogo num tabuleiro de n × n , onde n é um número escolhido por Ariel. A cada
movimento, Bernardo recebe de Ariel uma peça R. Em seguida Bernardo analisa se
poderá colocar R no tabuleiro de modo que não tenha pontos em comum com
nenhuma das peças colocadas anteriormente (nem sequer um vértice em comum).
Se existe uma tal colocação para R, Bernardo deve escolher uma delas e colocar R.

EUREKA! N°33, 2011

3
Sociedade Brasileira de Matemática

O jogo para se é impossível colocar R da forma explicada, e Bernardo vence. Ariel


vence somente se estão colocadas as 10 peças no tabuleiro.

a) Suponhamos que Ariel dá as peças a Bernardo em ordem decrescente de


tamanho. Qual é o menor n que garante a vitória do Ariel?
b) Para o n encontrado em a), se Bernardo recebe as peças em ordem crescente de
tamanho. Ariel tem garantida a vitória?

ESCLARECIMENTO: cada peça deve cobrir exatamente um número de quadrados


unitários do tabuleiro igual ao seu próprio tamanho. Os lados das peças podem
coincidir com partes da borda do tabuleiro.

RESULTADO BRASILEIRO

2010: Nível 1 (até 13 anos)

Nome Cidade – Estado Prêmio


Murilo Corato Zanarella Amparo – SP Medalha de Ouro
Daniel de Almeida Souza Brasília – DF Medalha de Prata
Viviane Silva Souza Freitas Salvador – BA Medalha de Prata
Carolina Lima Guimarães Vitória – ES Medalha de Bronze
Pedro Henrique Alencar Costa Fortaleza – CE Medalha de Bronze
Samuel Brasil de Albuquerque Fortaleza – CE Medalha de Bronze
Juliana Amoedo Plácido Salvador – BA Medalha de Bronze
Lucca Morais de Arruda Siaudjionis Fortaleza – CE Menção Honrosa
Antonio Wesley de Brito Vieira Cocal dos Alves – PI Menção Honrosa

2010: Nível 2 (até 15 anos)

Nome Cidade – Estado Prêmio


Rafael Kazuhiro Miyazaki São Paulo – SP Medalha de Ouro
Lucas Cauai Julião Pereira Caucaia – CE Medalha de Prata
Pedro Ivo Coêlho de Araújo Caucaia – CE Medalha de Prata
Francisco Markan Nobre de Souza Filho Fortaleza – CE Medalha de Bronze
Fellipe Sebastiam da Silva Paranhos Pereira Rio de Janeiro – RJ Medalha de Bronze
Tadeu Pires de Matos Belfort Neto Fortaleza – CE Medalha de Bronze
Henrique Gasparini Fiuza do Nascimento Brasília – DF Medalha de Bronze
Rafael Rodrigues Rocha de Melo Caucaia – CE Menção Honrosa
Mateus Henrique Ramos de Souza Pirapora – MG Menção Honrosa

EUREKA! N°33, 2011

4
Sociedade Brasileira de Matemática

XXI OLIMPÍADA DE MATEMÁTICA DO CONE SUL


Enunciados e resultado brasileiro

O Brasil, e particularmente o Estado de São Paulo teve a honra de sediar a


21ª Olimpíada de Matemática do Cone Sul, que aconteceu até o dia 19 de junho na
cidade de Águas de São Pedro, SP. A equipe foi liderada pelos professores
Francisco Bruno Holanda, de Fortaleza – CE e Tertuliano Franco Santos Franco, de
Rio de Janeiro – RJ.

RESULTADOS DA EQUIPE BRASILEIRA

BRA1 João Lucas Camelo Sá Medalha de Ouro


BRA2 Gabriel Militão Vinhas Lopes Medalha de Prata
BRA3 Maria Clara Mendes Silva Medalha de Prata
BRA4 Caíque Porto Lira Medalha de Bronze

PRIMEIRO DIA

PROBLEMA 1
Pedro tem que escolher duas frações irrredutíveis, cada uma com numerador e
denominador positivos, tais que:

• A soma das duas frações seja igual a 2.


• A soma dos numeradores das duas frações seja igual a 1000.

De quantas maneiras Pedro pode fazer isso?

PROBLEMA 2
Marcam-se em uma reta 44 pontos, numerados 1, 2, 3, ..., 44 da esquerda para a
direita. Vários grilos saltam na reta. Cada grilo parte do ponto 1, salta por pontos
marcados e termina no ponto 44. Além disso, cada grilo sempre salta de um ponto
marcado a outro marcado com um número maior.
Quando todos os grilos terminaram da saltar, notou-se que para cada par i, j, com
1 ≤ i ≤ j ≤ 44, há um grilo que saltou diretamente do ponto i para o ponto j, sem
pousar em nenhum dos pontos entre eles.
Determine a menor quantidade de grilos para que isso seja possível.

EUREKA! N°33, 2011

5
Sociedade Brasileira de Matemática

PROBLEMA 3
Recortar um polígono convexo de n lados significa escolher um par de lados
consecutivos AB, BC do polígono e substituí-los por três segmentos AM, MN, e
NC, sendo M o ponto médio de AB e N o ponto médio de BC. Em outras palavras,
corta-se o triângulo MBN e obtem-se um polígono convexo de n + 1 lados.
Seja P6 um hexágono regular de área 1. Recorta-se P6 e obtém-se o polígono P7.
Então recorta-se P7, de uma das sete maneiras possíveis, e obtém-se o polígono P8,
e assim sucessivamente. Prove que, independentemente de como sejam feitos os
recortes, a área de Pn é sempre maior do que 2 3.

SEGUNDO DIA

PROBLEMA 4
Pablo e Sílvia jogam em um tabuleiro 2010 × 2010. Primeiro Pablo escreve um
número inteiro em cada casa. Feito isso, Sílvia repete tantas vezes quanto quiser a
seguinte operação: escolher três casas que formem um L, como uma figura, e
somar 1 a cada número dessas três casas. Sílvia ganha se fizer com que todos os
números do tabuleiro sejam múltiplos de 10.
Demonstre que Sílvia sempre pode escolher uma sequência de operações com as
quais ela ganha o jogo.

PROBLEMA 5
O incírculo do triângulo ABC toca os lados BC, CA, e AB em D, E e F,
respectivamente. Sejam ωa , ωb e ωc os circuncírculos dos triângulos EAF, DBF e
DCE, respectivamente. As retas DE e DF cortam ωa em Ea ≠ E e
Fa ≠ F , respectivamente. Seja rA a reta Ea Fa . Defina rB e rC de modo análogo.
Prove que as retas rA , rB e rC determinam um triângulo cujos vértices pertencem
aos lados do triângulo ABC.

PROBLEMA 6
Determine se existe uma sequência infinita a0 , a1 , a2 , a3 ,... de inteiros não negativos
que satisfaz as seguintes condiciones:
(i) Todos os números inteiros não negativos aparecem na sequência uma única vez;
(ii) A sequência bn = an + n, n ≥ 0, é formada por todos os números primos, cada
um aparecendo uma única vez.

EUREKA! N°33, 2011

6
Sociedade Brasileira de Matemática

LI OLIMPÍADA INTERNACIONAL DE MATEMÁTICA (IMO)


Enunciados e resultado Brasileiro

A LI Olimpíada Internacional de Matemática (IMO) foi realizada na cidade


de Astana, Cazaquistão entre os dias 2 e 14 de julho de 2010. A equipe foi liderada
pelos professores Edmilson Luis Rodrigues Motta, de São Paulo – SP e Marcelo
Mendes de Oliveira, de Fortaleza – CE.

RESULTADOS DA EQUIPE BRASILEIRA

BRA1 Marcelo Tadeu de Sá Oliveira Sales Medalha de Prata


BRA2 Matheus Secco Torres da Silva Medalha de Prata
BRA3 Gustavo Lisbôa Empinotti Medalha de Bronze
BRA4 Deborah Barbosa Alves Menção Honrosa
BRA5 Hanon Lima Rossi Menção Honrosa
BRA6 João Lucas Camelo Sá Menção Honrosa

PRIMEIRO DIA

PROBLEMA 1
Determine todas as funções f : → tais que
f (  x  y ) = f ( x )  f ( y ) 
para os números x, y ∈ . (  z  designa o maior inteiro que é menor ou igual a z).

PROBLEMA 2
Seja ABC um triângulo, I o seu incentro e Γ a sua circunferência circunscrita. A
recta AI intersecta novamente Γ no ponto D. Sejam E um ponto do arco BDC e F
um ponto do lado BC tais que
1
B AF = C AE < B AC.
2
Seja G o ponto médio do segmento IF. Mostre que as rectas DG e EI se intersectam
sobre Γ .

PROBLEMA 3
Seja * o conjunto dos inteiros positivos. Determine todas as funções
g : * → * tais que

EUREKA! N°33, 2011

7
Sociedade Brasileira de Matemática

( g ( m) + n ) ( m + g ( n))
é um quadrado perfeito para todos m, n ∈ *.

SEGUNDO DIA

PROBLEMA 4
Seja Γ a circunferência circunscrita ao triângulo ABC e P um ponto no interior do
triângulo. As rectas AP, BP e CP intersectam novamente Γ nos pontos K, L, e M,
respectivamente.
A recta tangente a Γ em C intersecta a recta AB em S. Supondo que SC = SP,
mostre que MK = ML.

PROBLEMA 5
Em cada uma das seis caixas B1 , B2 , B3 , B4 , B5 , B6 há inicialmente só uma moeda.
Dois tipos de operações são possíveis:

Tipo 1: Escolher uma caixa não vazia B j , com 1 ≤ j ≤ 5. Retirar uma moeda da B j e
adicionar duas moedas a B j +1.

Tipo 2: Escolher uma caixa não vazia Bk , com 1 ≤ k ≤ 4. Retirar uma moeda da Bk e
trocar os conteúdos das caixas (possivelmente vazias) Bk +1 e Bk + 2 .
Determine se existe uma sucessão finita destas operações que deixa as caixas
2010
B1 , B2 , B3 , B4 , B5 vazias e a caixa B6 com exactamente 20102010 moedas. (Observe
c
que a b = a
(b ) . )
c

PROBLEMA 6
Seja a1 , a2 , a3 ,... uma sucessão de números reais positivos. Sabe-se que para algum
inteiro positivo s,
an = max {ak + an − k tal que 1 ≤ k ≤ n − 1}
para todo n > s. Mostre que existem inteiros positivos e N, com ≤ s, tais que
an = a + an − para todo n ≥ N .

EUREKA! N°33, 2011

8
Sociedade Brasileira de Matemática

XXV OLIMPÍADA IBEROAMERICANA DE MATEMÁTICA


Enunciados e resultado Brasileiro

A XXV Olimpíada Iberoamericana de Matemática foi realizada na cidade


de Assunção, Paraguai no período de 20 a 30 de setembro de 2010. A equipe
brasileria foi liderada pelos professores Onofre Campos, de Fortaleza – CE e
Luzinalva Miranda de Amorim, de Salvador – BA. A equipe brasileira ficou em
primeiro lugar na soma dos pontos dos participantes.

RESULTADOS DA EQUIPE BRASILEIRA


BRA1 Marcelo Tadeu de Sá Oliveira Sales Medalha de Ouro
BRA2 Deborah Barbosa Alves Medalha de Ouro
BRA3 Matheus Secco Torres da Silva Medalha de Ouro
BRA4 Gustavo Lisboa Empinotti Medalha de Prata

PRIMEIRO DIA

PROBLEMA 1
Numa fila de dez moedas indistinguíveis há duas delas que são falsas, ocupando
posições consecutivas. Para cada conjunto de posições, pode-se perguntar quantas
moedas falsas ele contém. É possível determinar quais são as moedas falsas
fazendo apenas duas destas perguntas? Não se sabe a resposta da primeira pergunta
antes de se formular a segunda.

PROBLEMA 2
Determinar se existem números inteiros positivos a e b tais que todos os termos da
sucesão definida por x1 = 2010, x2 = 2011,
xn + 2 = xn + xn +1 + a xn xn +1 + b , n ≥ 1,
sejam inteiros.

PROBLEMA 2
A circunferência Γ inscrita ao triângulo escaleno ABC é tangente aos lados BC,
CA e AB nos pontos D, E e F respectivamente. A recta EF corta a recta BC em G.
A circunferência de diâmetro GD corta Γ em R ( R ≠ D ) . Sejam P e Q
( P ≠ R, Q ≠ R ) as intersecções de BR e CR com Γ , respectivamente. As rectas BQ

EUREKA! N°33, 2011

9
Sociedade Brasileira de Matemática

e CP cortam-se em X. A circunferência circunscrita a CDE corta o segmento QR


em M e a circunferência circunscrita a BDF corta o segmento PR em N.
Demonstrar que as rectas PM, QN e RX são concorrentes.

SEGUNDO DIA

PROBLEMA 4
As médias aritmética, geométrica e harmônica de dois números inteiros positivos
distintos são números inteiros. Encontrar o menor valor possível para a média
aritmética.
Nota: Se a e b são números positivos, suas médias aritméticas, geométrica e
a+b 2
harmônica são respectivamente: , a ⋅b e .
2 1 1
+
a b

PROBLEMA 5
Seja ABCD um quadrilátero cíclico sujas diagonais AC e BD são perpendiculares.
Sejam O o circuncentro de ABDC, K a intersecção das diagonais, L ≠ O a
intersecção das circunferências circunscritas a OAC e OBD, e G a intersecção das
diagonais do quadrilátero cujos vértices são os pontos médios dos lados de ABCD.
Provar que O, K, L e G são colineares.

PROBLEMA 6
Ao redor de uma mesa circular sentam-se 12 pessoas e sobre a mesa há 28 vasos de
flores. Duas pessoas podem ver-se uma à outra se, e somente se, não há nenhum
vaso alinhado com elas. Provar que existem pelo menos duas pessoas que podem
ver-se.

EUREKA! N°33, 2011

10
Sociedade Brasileira de Matemática

ASSOCIANDO UM POLINÔMIO A EXPRESSÕES ALGÉBRICAS E


TRIGONOMÉTRICAS
Marcílio Miranda, IFRN (Caicó – RN)

♦ Nível Intermediário

O objetivo deste artigo é mostrar uma técnica que pode ser bastante útil na
hora de resolver problemas de olimpíadas de Matemática.Tal técnica consiste em
você associar um polinômio a uma determina expressão. Com isso você pode
calcular o valor de expressões trigonométricas, expressões algébricas e mostrar que
um determinado número é irracional.

Vejamos alguns exemplos disso:

I) EXPRESSÕES TRIGONOMÉTRICAS

Esse problema deixa bem clara a idéia de associarmos um polinômio a uma


expressão trigonométrica:

EXERCÍCIO RESOLVIDO 1 (BÉLGICA 2006):


a) Encontre todos os números reais α tais que cos ( 4α ) = cos ( 3α )
2π 4π 6π
b) Determine inteiros a, b, c, d tais que cos , cos , cos , são soluções da
7 7 7
equação ax 3 + bx 2 + cx + d = 0.

SOLUÇÃO:
a) cos ( 4α ) = cos ( 3α ) ⇔ 4α = 3α + 2kπ ou 4α = −3α + 2kπ ⇔ α = 2kπ ou
2kπ 2π 4π 6π
α= , logo 1,cos ,cos , cos são as raízes dessa equação.
7 7 7 7
Por outro lado temos que cos ( 4α ) = 8 ⋅ cos 4 α − 8 ⋅ cos 2 α + 1 e
cos ( 3α ) = 4 ⋅ cos3 α − 3 ⋅ cos α . Faça cos α = t. Daí temos que
cos ( 4α ) = cos ( 3α ) ⇔ ( t − 1) ⋅ ( 8t 3 + 4t 2 − 4t − 1) = 8 ⋅ t 4 − 4 ⋅ t 3 − 8 ⋅ t 2 + 3 ⋅ t + 1 = 0.
Assim, a equação (8t 3
+ 4t 2 − 4t − 1) = 0 tem como
2π 4π 6π
soluções cos ,cos ,cos .
7 7 7

EUREKA! N°33, 2011

11
Sociedade Brasileira de Matemática

EXERCÍCIO RESOLVIDO 2 (MOCP, JULHO DE 2003): Prove que


sec 400 + sec 800 + sec 1600 = 6.

SOLUÇÃO: Note que 400, 800 e 1600


satisfazem a equação
1
cos3α = − ⇒ 8cos3 α − 6cos α + 1 = 0, logo cos 400, cos 800, cos 1600 são as
2
raízes do polinômio 8cos3 α – 6 cos α +1, e assim temos que:
−6
cos 40° ⋅ cos80° + cos160° ⋅ cos80° + cos 40° ⋅ 160° =
8
−1
cos 40° ⋅ cos80° ⋅ cos160° =
8
1 1 1
sec 40° + sec80° + sec160° = + + =
cos 40° cos80° cos160°
cos 40° ⋅ cos80° + cos 40° ⋅ cos80° + cos 40° ⋅ cos80°
= 6.
cos160° cos80° ⋅ cos 40°

π 2π 3π 1
EXERCÍCIO RESOLVIDO 3 (IMO 1963): Prove que cos − cos + cos = .
7 7 7 2
SOLUÇÃO: Note que
π π 3π 3π 5π 5π π 3π 5π
3⋅ + 4⋅ = π ,3 ⋅
+ 4⋅ = 3π e 3 ⋅ + 4⋅ = 5π , logo , , são
7 7 7 7 7 7 7 7 7
soluções da equação cos 4 x = − cos3x.
7x x 7x
Essa equação equivale a cos 4 x + cos3 x = 0 ⇔ 2 ⋅ cos ⋅ cos = 0 ⇔ cos = 0 ou
2 2 2
x
cos = 0.
2
7x
PARTE 1: Resolver a equação cos =0
2
7x π π 2 kπ π 3π 5π 9π 11π 13π
= + kπ ⇒ x = + ⇒ x = , , ,π , , , , mas
2 2 7 7 7 7 7 7 7 7
π 13π 3π 11π 5π 9π
cos = cos ,cos = cos ,cos = cos , logo há 4 soluções distintas
7 7 7 7 7 7
π 3π 5π
entre 0 e 2π : , , , π .
7 7 7
x
PARTE 2: Resolver a equação cos = 0
2

EUREKA! N°33, 2011

12
Sociedade Brasileira de Matemática

x π
= + kπ ⇒ x = π + 2kπ , logo x = π é a única solução entre 0 e 2π .
2 2
Por outro lado temos que cos 4 x = 8cos 4 x − 8cos 2 x + 1 e cos3 x = 4cos3 x − 3cos x.
cos4 x = − cos3x ⇔ 8cos4 x + 4cos3 x − 8cos2 x − 3cos x + 1 = 0 ⇔ 8t 4 + 4t 3 − 8t 2 − 3t + 1 = 0,
onde t = cos x.
Claramente –1 é raiz desse polinômio, e temos 8t4 +4t3 – 8t2 – 3t + 1 =
(t +1) · (8t3 – 4t2 – 4t + 1), donde o polinômio 8t3 – 4t2 – 4t + 1 tem como raízes
π 3π 5π
cos ,cos ,cos . Logo temos pelas relações de Girard que:
7 7 7
π 3π 5π 4 1 π 2π 3π
cos + cos + cos = = = cos − cos − cos .
7 7 7 8 2 7 7 7

II) CALCULANDO O VALOR DE UMA EXPRESSÃO ALGÉBRICA:

EXERCÍCIO RESOLVIDO 4: Prove que 3


20 + 14 2 2 + 3 20 − 14 2 2 = 4.

SOLUÇÃO: Seja x = 3 20 + 14 2 2 + 3 20 − 14 2 2 . Temos x3 = 40 + 6x ⇒ x3 – 6x – 40


= 0. É fácil ver que 4 é raiz desse polinômio e x3 – 6x – 40 = (x – 4).(x2 + 4x + 10).
Note que as raízes de x2 + 4x + 10 não são reais e 3
20 + 14 2 2 + 3 20 − 14 2 2 é
real, logo 3
20 + 14 2 2 + 3 20 − 14 2 2 = 4.

EXERCÍCIO RESOLVIDO 5 (CROÁCIA 2001): Se a + b + c = 0, calcule o valor da


a 7 + b7 + c 7
expressão .
abc ⋅ ( a 4 + b 4 + c 4 )
SOLUÇÃO: Seja x3 + mx2 + px + q = 0. um polinômio de terceiro grau tal que suas
raízes são a, b, c. Daí temos que a + b + c = – m = 0, ab + ac + bc = p e abc = – q.
Assim temos que:
(a + b + c)2 = a2 + b2 + c2 + 2⋅(ab + ac + bc) ⇒ a2 + b2 + c2 = –2p
Por outro lado temos que:
a3 + pa + q = 0 ⇒ a3 = – pa – q (i)
3 3
b + pb + q = 0 ⇒ b = – pb – q (ii)
c3 + pc + q = 0 ⇒ a3 = – pc – q (iii)
somando (i) + (ii) + (iii), temos que a3 + b3 + c3 = – p.(a + b + c) – 3q = –3q

Da mesma forma temos que:


a4 + pa2 + qa = 0 ⇒ a4 = – pa2 – qa (iv)

EUREKA! N°33, 2011

13
Sociedade Brasileira de Matemática

b4 + pb2 + qb = 0 ⇒ b4 = – pb2 – qb (v)


4 2 4 2
c + pc + qc = 0 ⇒ c = – pc – qc (vi)
somando (iv) + (v) + (vi), temos que a4 + b4 + c4 = – p.(a2 + b2 + c2) – q.(a + b + c)
= 2 p2 .

Analogamente temos que:


a5 + pa3 + qa2 = 0 ⇒ a5 = – pa3 – qa2 (vii)
b5 + pb3 + qb2 = 0 ⇒ b5 = – pb3 – qb2 (viii)
c5+ pc3 + qc2 = 0 ⇒ c5 = – pc3 – qc2 (ix)
somando (vii) + (viii) + (ix), temos que a5 + b5 + c5 = – p · (a3 + b3 + c3) – q · (a2 +
b2 + c2) = 5pq.

Proseguindo do mesmo modo, temos que:


a7 + pa5 + qa4 = 0 ⇒ a7 = – pa5 – qa4 (x)
b7 + pb5 + qb4 = 0 ⇒ b7 = – pb5 – qb4 (xi)
c7 + pc5 + qc4 = 0 ⇒ c7 = – pc5 – qc4 (xii)
somando (x) + (xi) + (xii): a7 + b7 + c7 = – p · (a5 + b5 + c5) – q · (a4 + b4 +c4) = –
7 p 2q .
a 7 + b7 + c7 −7 p 2 q 7
Com isso temos que = = .
abc ⋅ ( a + b + c ) − q ⋅ ( 2 p ) 2
4 4 4 2

III) PROVANDO A IRRACIONALIDADE DE UM NÚMERO:

Antes do próximo problema vamos provar o seguinte teorema:


p
TEOREMA (TESTE DA RAIZ RACIONAL): Se o número , onde p e q são inteiros e
q
mdc(p, q) = 1, é uma raiz do polinômio com coeficientes inteiros
an ⋅ x n + an −1 ⋅ x n −1 + ... + a1 ⋅ x + a0 , então p é um divisor de a0 e q é um divisor de an.
p
PROVA: Como é raiz do polinômio temos que
q
n n−1
 p  p p
an   + an−1 ⋅   + ... + a1 ⋅ + a0 = 0 ⇒ an ⋅ pn + an−1 ⋅ pn−1 ⋅ q + ... + a1 ⋅ p ⋅ qn−1 + a0 ⋅ qn = 0,
q q q
logo temos que p é um divisor de a0 e q é um divisor de an.

EXERCÍCIO RESOLVIDO 6: Prove que 2 + 3 é irracional.

EUREKA! N°33, 2011

14
Sociedade Brasileira de Matemática

Solução: Seja x = 2 + 3 ⇒ x – 2 = 3 ⇒ x2 – 1 = 2 2 x ⇒ x4 – 2x2 + 1 = 8x2


⇒ x4 – 10x2 + 1 = 0. Logo pelo teorema acima as raízes racionais da equação só
podem ser 1 ou –1, que claramente não são soluções (em ambos os casos o valor
numérico do polinômio é –8). Logo esse polinômio só possui raízes irracionais,
portanto 2 + 3 é irracional.

EXERCÍCIOS PROPOSTOS:
π 2π 3π 7
1) (EUA) Prove que sen ⋅ sen
⋅ sen = .
7 7 7 8
2) (Vietnã 1982) Ache a, b, c inteiros tais que as raízes da equação ax2 + bx + c = 0
são cos 720 e cos 1440.

3) (Prova de Seleção da Romênia para a IMO 1970): Prove que para todo inteiro
positivo n:

tg
π
⋅ tg

⋅ tg

...tg
( n − 1) π ⋅ tg nπ = 2n + 1.
2n + 1 2n + 1 2n + 1 2n + 1 2n + 1

4) (Prova de Seleção da Suíça para a IMO 2004): Sejam a, b, c, d números reais


distintos satisfazendo as equações:
a = 45 − 21 − a , b = 45 − 21 − b , c = 45 − 21 − c , d = 45 − 21 − d
Prove que abcd = 2004.

5) (OBM 2003): Sejam a, b, c números reais não-nulos tais que a + b + c = 0.


(a + b3 + c 3 ) ⋅ ( a 4 + b 4 + c 4 )
3 2

Calcule os possíveis valores de .


(a + b5 + c5 )
5 2

6) (Bélgica 1978): Encontre um polinômio com coeficientes inteiros tal que


2 + 3 é raiz.

7) (Moldávia 2000): Os números a, b, c satisfazem a relação a + b + c = 0. Mostre


que o número 2a4 + 2b4 +2c4 é um quadrado perfeito.
8) Prove que 2 + 3 3 é irracional.
π
9) Prove que x = 2cos satisfaz a equação: x3 + x2 – 2x + 1 = 0.
7
π
Use este fato para provar que cos é irracional.
7
10) Prove que tg2 10 + tg2 30 +....+ tg2 870 + tg2 890 = 4005.

EUREKA! N°33, 2011

15
Sociedade Brasileira de Matemática

1
11) Prove que cos 200. cos 400.cos 800 = .
8
12) Prove que:
π 2π 3π
a) tg ⋅ tg
⋅ tg = 7.
7 7 7
π 2π 3π 4π 5π 6π
b) tg ⋅ tg ⋅ tg ⋅ tg ⋅ tg ⋅ tg = 13 .
13 13 13 13 13 13

13) Prove que cossec 6° + cossec 78° − cossec 42° − cossec 66° = 8.

14) Calcule as expressões:


π 2π 3π
a) tg 2 ⋅ tg 2
⋅ tg 2 .
7 7 7
π 2π 3π
b) tg 2 + tg 2 + tg 2 .
7 7 7
π 2π 3π π 2π 3π
c) tg 2 ⋅ tg 2 + tg 2 ⋅ tg 2 + tg 2 ⋅ tg 2 .
7 7 7 7 7 7
π 2π 4π 1
15) Prove que cos ⋅ cos ⋅ cos =− .
7 7 7 8
π 3π 5π
16) Ache uma equação do terceiro grau cujas raízes são cos ,cos ,cos .
7 7 7
17) Calcule as expressões:
π 3π 5π
a) cos ⋅ cos ⋅ cos .
7 7 7
π 3π 5π π 3π 5π
b) cos ⋅ cos + cos cos + cos ⋅ cos .
7 7 7 7 7 7
π 3π 5π
c) cos + cos + cos .
7 7 7
π 3 π 5π
d) cos 2 + cos 2 + cos 2 .
7 7 7
1 1 1
e) + + .
π 3π 5π
cos cos cos
7 7 7
18) Prove que tg 810 – tg 630 + tg 90 – tg 270 = 4.

EUREKA! N°33, 2011

16
Sociedade Brasileira de Matemática

19) Sejam u, v, w as raízes do polinômio x3 – 10x + 11. Determine o valor de arctg u


+ arctg v + arctg w.
π 5π 13π
20) Prove que cossec + cossec + cossec = 6.
18 18 18
21) Prove que tg 200. tg400. tg 600. tg 800 = 3.

22) Sejam a, b, c números reais tais que a + b + c = 0, prove que:


a) a3 + b3 + c3 = 3abc.
a 2 + b 2 + c 2 a 5 + b5 + c 5 a 7 + b 7 + c 7
b) ⋅ = .
2 5 5
3
23) Prove que sen20° ⋅ sen40° ⋅ sen80° = .
8
π 2π 3π
24) Prove que cot g 2 + cot g 2 + cot g 2 = 5.
7 7 7
π 4π 7π
25) Calcule o valor da expressão tg + tg + tg .
9 9 9
REFERÊNCIAS
[1] MIRANDA, Marcílio. Problemas Selecionados de Matemática ITA-IME – Olimpíadas,
Volume 1, Fortaleza (CE), Editora Vestseller, 2010.
[2] ANDREESCU, Titu; FENG , Zuming. 103 Trigonometry Problems from the Training
of the USA IMO Team, Birkhauser, 2004.
[3] ANDREESCU, Titu; GELCA, Razvan. Putnam and Beyond. New York: Springer-
Verlag, 2006.
[4] DOMINGUES, Hygino. Fundamentos de Aritmética, São Paulo, Atual Editora, 1991.

SITES ACESSADOS
[1] The IMO Compendium, Disponível em
<http://www.imomath.com/index.php?options=oth|other&p=0>, Acesso em: 10/08/2009.
[2] Treinamento do Cone Sul. Disponível em: < http://treinamentoconesul.blogspot.com/>,
Acesso em: 12/08/2009.
[3]Notas de Aula de Kin Yin Li. Disponível em:
<http://www.math.ust.hk/~makyli/190_2003Fa/lect-notes_03fa.pdf>, Acesso em:
15/08/2009.
[4] Página de Olimpíada da Sociedade Canadense de Matemática. Disponível em:
< http://www.cms.math.ca/Olympiads/ >, Acesso em: 20/07/2009.
[5] Matemática Nick Puzzles. Disponível em: < http://www.qbyte.org/puzzles/>, Acesso
em : 15/11/2009.
[6] Olimpíada Brasileira de Matemática. Disponível em: <http://www.obm.org.br >, Acesso
em: 20 /11/2009.

EUREKA! N°33, 2011

17
Sociedade Brasileira de Matemática

SOMAS TRIGONOMÉTRICAS: DE PROSTAFÉRESE À FÓRMULA


DE EULER
Rogério Possi Junior

♦ Nível Intermediário

INTRODUÇÃO

São apresentados fundamentos básicos da matemática elementar, cujos


conceitos somados podem auxiliar na resolução de problemas mais elaborados,
como os que podem aparecer quando se depara com o início do estudo das
Variáveis Complexas e o uso dos teoremas de De Moivre.

Seja através das fórmulas de Transformação de Soma em Produto, conhecidas


como Fórmulas de Prostaférese, ou através da Relação de Euler, são calculados
alguns exemplos de somas de funções trigonométricas aparentemente complexas.

AS FÓRMULAS DE TRANSFORMAÇÃO TRIGONOMÉTRICAS.

Admitamos conhecidas as fórmulas da soma e diferença de arcos para as funções


“seno” e “cosseno”, isto é

sen(a + b) = sen a cos b + sen b cos a (a)


sen(a − b) = sen a cos b − sen b cos a (b)
cos(a + b) = cos a cos b − sen a sen b (c)
cos(a − b) = cos a cos b + sen a sen b (d)

Somando-se (a) e (b) tem-se


sen(a + b) + sen(a − b) = 2 sen a cos b (e)

Subtraindo-se (a) de (b) tem-se


sen(a + b) − sen(a − b) = 2 sen b cos a (f)

Somando-se (c) e (d) teremos


cos(a + b) + cos(a − b) = 2 cos a cos b (g)

E por fim, subtraindo-se (c) de (d)

EUREKA! N°33, 2011

18
Sociedade Brasileira de Matemática

cos(a + b) − cos(a − b) = −2 sen a sen b (h)

α+β α −β
Fazendo a + b = α e a − b = β teremos que a = e b= , cujos
2 2
valores substituídos nas relações (e), (f), (g) e (h) fornecerão as seguintes relações

α + β  α − β 
sen α + sen β = 2 sen  cos 
 2   2 
α − β  α + β 
sen α − sen β = 2 sen  cos 
 2   2 
α + β  α − β 
cos α + cos β = 2 cos  cos 
 2   2 
α + β  α − β 
cos α − cos β = −2 sen  sen  , que são as conhecidas
 2   2 
Fórmulas de Transformação de soma em produto ou Fórmulas de Prostaférese.

A FÓRMULA DE EULER

Segundo GUIDORIZZI (1987), seja f ( x) uma função derivável até a ordem n em


um intervalo aberto I e seja x0 ∈ I . Define-se o polinômio P( x ) a seguir como o
polinômio de Taylor, de ordem n , de f ( x) em torno do ponto x0 , isto é
( x − x0 ) ( x − x0 )
n k
n
P ( x) = f ( x0 ) + f ′ ( x0 )( x − x0 ) + …+ f ( ( x0 ) =∑f ( ( x0 )
n) k)
(i)
n! k =1 k!

que, se fixado em torno de x0 = 0 , também pode ser chamado de polinômio de


Mac-Laurin. Tomando-se (i) f ( x) = ex e x0 = 0, pode-se demonstrar que

 x 2 x3 xn  x 2 x3
e x = lim1 + x + + + … +  = 1 + x + + + … (j)
n →∞
 2 3! n!  2 3!
A expressão da direita pode ser usada para definir ex para x para x complexo.
Analogamente demonstra-se que

EUREKA! N°33, 2011

19
Sociedade Brasileira de Matemática

 x3 x5
n −1
xn  x3 x5
sen x = lim x − + − … + (−1) 2 = x− + −… (k)
n →∞
 3! 5! n!  3! 5!
e que
 x2 x4 n x
2n
 x2 x4

cos x = lim1 − + + … + (−1)  = 1− + −… (l)
n →∞
 2 4! (2n)!  2! 4!

Para x = Z = iY , Y ∈ R e observando-se (j), (k) e (l) teremos que

 Y2 Y4   Y3 Y5 
e iY = 1 − + − … + i Y − + − … = cos Y + i sen Y (m)
 2 4!   3! 5! 

que é a conhecida fórmula de Euler.


Não obstante, também se demonstra que se e Z = e X +iY , onde X ≠ 0 , então
e Z = e X (cos Y + i sen Y ) (n)
Z
Se, alternativamente, adotássemos a expressão de (n) como definição de e , não é
difícil mostrar que e Z +W = e Z ⋅ eW ,∀Z ,W ∈ de fato, se Z1 = X 1 + iY1 e
Z 2 = X 2 + iY2 , eZ1 +Z2 = ex1 + x2 ( cos (Y1 + Y2 ) + isen (Y1 + Y2 ) ) = ex1 + x2

( cosY cosY
1 2 − senY1senY2 + i ( senY1cosY2 + senY2 cosY1 ) = )
=e x1
( cosY1 + isen Y1 ) ⋅ e ( cosY2 + isen Y2 ) = e
x2 Z1
⋅ e Z2 .

PROBLEMAS DE APLICAÇÃO

PROBLEMA 1: Começaremos com um exemplo de problema análogo ao proposto em


um exame de admissão ao Instituto Militar de Engenharia (IME). O problema pede
que se calcule as somas a seguir.

S1 = sen x + sen 2 x + sen 3 x + … + sen nx (1)


S 2 = cos x + cos 2 x + ... + cos nx (2)

Utilizaremos a transformação de somas de funções trigonométricas em produto,


conhecidas como “Fórmulas de Prostaférese”. Observamos que

EUREKA! N°33, 2011

20
Sociedade Brasileira de Matemática

3x x x
sen − sen = 2 sen cos x
2 2 2
5x 3x x
sen − sen = 2 sen cos 2 x
2 2 2
7x 5x x (3)
sen − sen = 2 sen cos 3 x
2 2 2

(2n − 1) x (2n − 3) x x
sen − sen = 2 sen cos(n − 1) x
2 2 2
(2n + 1) x (2n − 1) x x
sen − sen = 2 sen cos nx
2 2 2

Somando-se as linhas acima encontraremos uma “Soma Telescópica”, cujo valor


será dado por
(2n + 1) x x x n
sen − sen = 2 sen ⋅ ∑ cos jx
2 2 2 j =1
nx (n + 1) x
n sen ⋅ cos
⇒ S 2 = ∑ cos jx = 2 2 (4)
x
j =1
sen
2
Analogamente, para a soma das funções “seno” S1 pode-se escrever que:
3x x x
cos − cos = −2 sen sen x
2 2 2
5x 3x x
cos − cos = −2 sen sen 2 x
2 2 2
7x 5x x (5)
cos − cos = −2 sen sen 3 x
2 2 2

(2n − 1) x (2n − 3) x x
cos − cos = −2 sen sen(n − 1) x
2 2 2
(2n + 1) x (2n − 1) x x
cos − cos = −2 sen sen nx
2 2 2

Somando-se as linhas acima encontraremos outra “Soma Telescópica”, cujo valor é

EUREKA! N°33, 2011

21
Sociedade Brasileira de Matemática

(2n + 1) x x x n
cos − cos = −2 sen ⋅ ∑ sen jx
2 2 2 j =1

nx (n + 1) x
n sen ⋅ sen
⇒ S1 = ∑ sen jx = 2 2 (6),
x
j =1
sen
2
que é a soma procurada.

Não obstante, este problema também pode ser resolvido utilizando-se a conhecida
ix
x x
Relação de Euler. Seja e 2 = C = cos + i sen , onde i 2 = −1 ; assim tem-se que
2 2
2
 x x
C =  cos + i sen  = cos x + i sen x
2

 2 2
4
 x x
C 4 =  cos + i sen  = cos 2 x + i sen 2 x
 2 2 (7)

2n
 x x
C 2n
=  cos + i sen  = cos nx + i sen nx
 2 2

C 2 (C 2 n − 1) (
C n C n − C −n n
) (cos jx + i sen jx )
⇒ C + C +…+ C =
2 4 2n

(C 2 − 1)
= C ⋅
C − C −1( =
)∑j =1

nx
sen
n
 x x  nx nx 
⇒ ∑ (cos jx + i sen jx ) =  cos + i sen  cos + i sen  2 = S + iS
2 1
j =1  2 2  2 2  sen x
2
(n + 1) x nx (n + 1) x nx
cos ⋅ sen sen ⋅ sen
⇒ S 2 + iS1 = 2 2 +i 2 2 (8)
x x
sen sen
2 2
de onde tiramos os valores de interesse S1 e S 2 igualando-se as partes reais e
imaginárias da igualdade acima respectivamente.

EUREKA! N°33, 2011

22
Sociedade Brasileira de Matemática

PROBLEMA 2: Considere agora o problema de se determinar as somas dadas por


n n
S1 = ∑ sen 2 jx e S 2 = ∑ cos 2 jx . Para tal, observa-se inicialmente, da
j =1 j =1

1 1 1 1
Trigonometria que sen 2 x = − cos 2 x e cos 2 x = + cos 2 x , assim pode-
2 2 2 2
se reescrever S1 e S 2 como sendo

n
1 1 1 1
S1 = ∑ sen 2 jx =  + + …+  − (cos 2 x + cos 4 x + … + cos 2nx ) (9)
j =1 2 2 "n " 2 2

onde S´= cos 2 x + cos 4 x + … + cos 2nx .

Com o auxílio da Relação de Euler, seja C = cos x + i sen x , assim

(
C n C n − C −n )n
(cos 2 jx + i sen 2 jx )
C 2 + C 4 + … + C 2 n = C.
(C − C −1 ) = ∑
j =1
n
cos(n +1)x.sennx sen(n +1)x.sennx
⇒ ∑(cos2 jx + i sen2 jx) = +i = S´+iS´´ (10)
j =1 senx sen x
n
n 1
onde S´´= ∑ sen 2 jx . Sendo S1 = − S´
j =1 2 2
1  cos(n + 1) x. sen nx 
∴ S1 = n −  (11)
2 sen x

Outra solução para o cálculo da soma S´ consiste em transformá-la segundo as


fórmulas de Prostaférese. Para este caso tem-se que,

sen 3x − sen x = 2 sen x cos 2 x


sen 5 x − sen 3 x = 2 sen x cos 4 x
(12)
sen(2n − 1) x − sen(2n − 3) x = 2 sen x cos(2n − 2) x
sen(2n + 1) x − sen(2n − 1) x = 2 sen x cos 2nx

EUREKA! N°33, 2011

23
Sociedade Brasileira de Matemática

Somando-se todas as linhas acima tem-se que sen(2n + 1) x − sen x = 2 sen x ⋅ S´

sen nx. cos(n + 1) x


∴ S´= , que é exatamente o valor encontrado da parte real do
sen x
somatório dado por (10).
1 1
Observando-se (9) e que cos 2 x =+ cos 2 x
2 2
n
1 1  cos(n + 1) x. sen nx 
⇒ S 2 = ∑ cos 2 jx = [n + S´] = n +  (13)
j =1 2 2 sen x

que resolve o problema do cálculo de S 2 .

PROBLEMA 3: Considere a seguir o problema do cálculo das somas dadas por


n n
S1 = ∑ sen 3 kx e S 2 = ∑ cos 3 kx .
k =1 k =1

Seja C = cosx + isenx. Sendo C ± K = cos kx ± i sen kx pode-se escrever que


C k + C −k
cos kx = (14)
2
C − C −k
k
sen kx = (15)
2i
Elevando-se a relação (15) ao cubo tem-se que

sen 3 kx = 
C −C 
k −k
 =
3
C −C ( −3 k
−3 C −C
3k
) ( k −k
)
 2i  − 8i
3 sen kx − sen 3kx
⇒ sen 3 kx = (16)
4
n
1 n n

∴ S1 = ∑ sen 3 kx =  3∑ sen kx − ∑ sen 3kx  (17)
k =1 4  k =1 k =1 
nx (n + 1) x
n sen ⋅ sen
Por (6) tem-se que ∑ sen kx = 2 2 e observando-se que se
k =1
x
sen
2
D = cos 3 x + i sen 3 x teremos que:

EUREKA! N°33, 2011

24
Sociedade Brasileira de Matemática

D 2 = cos 6 x + i sen 6 x
D 3 = cos 9 x + i sen 9 x

D n = cos 3nx + i sen 3nx


1 n
 n −n

D 2 .D 2  D 2 − D 2 
⇒ S D = D + D 2 + D3 + … + D n =  
 12 −1

D − D 2 
 
 
 (3n + 1) x (3n + 1) x 
cos + i sen 
2 2 3nx
⇒ SD = ⋅ sen (18)
3x 2
sen
2
Tomando-se a parte imaginária da relação (18) tem-se que

 (3n + 1) x 3nx 
 sen ⋅ sen
2 
n

∑ sen 3kx =  2
3x  (19)
k =1  sen 
 2 
Logo, por (6) e (19) teremos que

 nx (n + 1) x (3n + 1) x 3nx 
 3 sen ⋅ sen sen ⋅ sen
2 
n
1
S1 = ∑ sen 3 kx =  2 2 − 2
 (20)
4 x 3x
k =1
sen sen 
 2 2 
n
Vale lembrar que a soma S = ∑ sen 3kx
k =1
também poderá ser calculada

observando-se as igualdades a seguir, isto é

EUREKA! N°33, 2011

25
Sociedade Brasileira de Matemática

9x 3x 3x
cos − cos = −2 sen ⋅ sen 3x
2 2 2
15 x 9x 3x
cos − cos = −2 sen ⋅ sen 6 x
2 2 2

3(2n − 1) x 3(2n − 3) x 3x
cos − cos = −2 sen ⋅ sen(3n − 1) x
2 2 2
3(2n + 1) x 3(2n − 1) x 3x
cos − cos = −2 sen ⋅ sen 3nx
2 2 2

cuja soma resultará em

3(2n + 1) x 3x 3x n
cos − cos = −2 sen ⋅ ∑ sen 3kx .
2 2 2 k =1
3nx 3(n + 1) x
n sen ⋅ sen
∴ ∑ sen 3kx = 2 2 , que é exatamente a expressão (19).
3 x
k =1
sen
2
n
Para o cálculo de S 2 = ∑ cos
k =1
3
kx elevando-se a expressão (14) ao cubo teremos

 C k + C −k 
que cos kx = 
3
 =
3
( ) (
C 3 k + C −3 k + 3 C k + C − k )
 2  8
3 cos kx + cos 3kx
⇒ cos 3 kx = (21)
4
n
1 n n

∴ S 2 = ∑ cos 3 kx =  3∑ cos kx + ∑ cos 3kx  (22)
k =1 4  k =1 k =1 
Utilizando-se a relação (4) e a parte real da relação (18) e substituindo-as em (22)
tem-se que
 nx (n + 1) x (3n + 1) x 3nx 
3 sen ⋅ cos cos ⋅ sen
n
1 2 
S 2 = ∑ cos kx = 
3 2 2 + 2
 (23)
4 x 3x
k =1
sen sen 
 2 2 

EUREKA! N°33, 2011

26
Sociedade Brasileira de Matemática

n
Ressaltamos que a soma ∑ cos 3kx
k =1
também pode ser calculada através das

fórmulas de Prostaférese, ou seja, fazendo


9x 3x 3x
sen − sen = 2 sen ⋅ cos 3x
2 2 2
15 x 9x 3x
sen − sen = 2 sen ⋅ cos 6 x
2 2 2

3(2n − 1) x 3(2n − 3) x 3x
sen − sen = 2 sen ⋅ cos(3n − 1) x
2 2 2
3(2n + 1) x 3(2n − 1) x 3x
sen − sen = 2 sen ⋅ cos 3nx
2 2 2
e somando-se as linhas teremos uma “Soma Telescópica”, cujo valor será
3(2n + 1) x 3x 3x n
sen − sen = 2 sen ⋅ ∑ cos 3kx
2 2 2 k =1
3nx 3(n + 1) x
n sen ⋅ cos
∴ ∑ cos 3kx = 2 2 , que é exatamente a parte real da
3x
k =1
sen
2
expressão (18).

PROBLEMA 4 (IMO-62): Aqui é proposto resolvermos a equação a seguir (observamos


que o segundo problema resolvido trata desta questão de forma generalizada).

cos 2 x + cos 2 2 x + cos 2 3x = 1 (A)


1 1
Notando que cos 2 x = + cos 2 x segue que
2 2
3 1
cos 2 x + cos 2 2 x + cos 2 3x = + (cos 2 x + cos 4 x + cos 6 x )
2 2
Sendo Z = cos x + i sen x
⇒ Z 2 = cos 2 x + i sen 2 x
⇒ Z 4 = cos 4 x + i sen 4 x
⇒ Z 6 = cos 6 x + i sen 6 x

EUREKA! N°33, 2011

27
Sociedade Brasileira de Matemática

Z 2 ( Z 6 − 1) sen 3 x
⇒Z +Z +Z =
2 4 6
= (cos 4 x + i sen 4 x ) (B)
( Z − 1)
2
sen x
Tomando-se a parte real de (B) tem-se que
(cos 2 x + cos 4 x + cos 6 x ) = cos 4 x ⋅ sen 3x (C)
sen x
1
Como (sen 7 x − sen x) = sen 3x. cos 4 x , então teremos que a equação (A)
2
reduz-se a sen 7 x + sen x = 0 .

⇒ 2 sen 4 x. cos 3 x = 0

∴ sen 4 x = 0 ∨ cos 3 x = 0

Logo, a solução da equação proposta será dada pelo conjunto

 kπ ( 2k + 1 )π ( 2k + 1 )π 
S = x ∈ x= ∨x= ∨x= ,k ∈ 
 2 4 6 

PROBLEMA 5: Determinaremos agora o valor das somas

a) cos x + 2 cos 2 x + 3 cos 3 x + … + n cos nx e


b) senx + 2sen2 x + 3sen3 x + … + nsennx
Sejam S1 = cos x + 2 cos 2 x + 3 cos 3 x + … + n cos nx e
S 2 = senx + 2sen2 x + 3sen3 x + … + nsennx
⇒ S1 + iS 2 = ( cosx + isenx ) + 2 ( cos2 x + isen2 x ) + … + n ( cosnx + isennx )
Sendo Z = cosx + isenx ⇒ S1 + iS 2 = Z + 2 Z 2 + 3Z 3 + … + nZ n . Multiplicando-
se ambos os termos por (1 − Z ) teremos
2 n +1
n +1
Z + Z + Z + … Z − nZ
2 3 n
nZ 2
( Z n − 1)
S1 + iS 2 = = −
(1 − Z ) 1

1 1

1
(Z − Z
2 2
) (Z − Z
2 2 2
)
  2n + 1   2n + 1  
n cos   x + isen   x
  2   2   ( cosnx + isennx − 1)
⇒ S1 + iS 2 = − (1)
x x
2isen 4i 2 sen 2
2 2

EUREKA! N°33, 2011

28
Sociedade Brasileira de Matemática

Observando-se que a parte real de (1) nos dará o valor de S1 e que a parte
imaginária nos dará o valor de S 2 tem-se, após alguma manipulação algébrica que
n
S1 = ∑ jcosjx =
( n + 1) cosnx − ncos ( n + 1) x − 1 ,e
x
j =1
4sen 2
2

S2 = ∑
n
jsenjx =
( n + 1) sennx − nsen ( n + 1) x
x
j =1
4sen 2
2
PROBLEMAS PROPOSTOS:

1) (URSS) Calcule o valor das somas


a) cos x + Cn1 cos 2 x + … + Cnn cos(n + 1) x
b) senx + Cn1sen2 x + … + Cnn sen ( n + 1) x
n
Obs: Cnk =   denota o binomial “n escolhe k”.
k 
2) (URSS) Mostre que
2π 4π 6π 2 nπ 1
cos + cos + cos + … + cos =− .
2n + 1 2n + 1 2n + 1 2n + 1 2

3) (URSS) Prove que

sen
( n + 1) α sen  ϕ + nα 
 
2  2 
a) senϕ + sen ( ϕ + α) + sen ( ϕ + 2α) + … + sen ( ϕ + nα) =
α
sen
2
sen
( n + 1) α cos  ϕ+ nα 
 
b) cosϕ + cos ( ϕ + α) + cos ( ϕ + 2α) + …+ cos ( ϕ+ nα) =
2  2 
α
sen
2
π 2π nπ
cos cos cos
4) Calcule o valor da soma S = 4+ 4 +…+ 4 .
2 n
2 2 2

5) Mostre que

EUREKA! N°33, 2011

29
Sociedade Brasileira de Matemática

ak +2 coskϕ − ak +1 cos(k +1)ϕ − a cosϕ +1


a) 1+ a cosϕ + a cos2ϕ + …+ a coskϕ =
2 k

a2 − 2a cosϕ +1
senϕ + asen ( ϕ + h) + … + ak sen ( ϕ + kh) =
b) ak +2sen ( ϕ + kh) − ak +1sen ϕ + ( k + 1) h − asen ( ϕ − h) + senϕ
 
a2 − 2acos h+ 1
0
6) Mostre que 72 é o menor ângulo positivo que satisfaz simultaneamente às
equações:
1 + cosx + cos2 x + cos3 x + cos4 x = 0

senx + sen2 x + sen3 x + sen4 x = 0

sen
( 2n + 1) x
1 2
7) (IME-92) Mostre que + cosx + cos2 x + … + cosnx = .
2 x
2sen
2

REFERÊNCIAS

[1] FADDEEV, D.; SOMINSKY, I. Problems in Higher Algebra, Moscou: Ed. MIR, 1968.
[2] GREITZER, S.L. International Mathematical Olympiads 1959-1977, Fifth Printing,
Washington D.C.: The Mathematical Association of America, 1978.
[3] GUIDORIZZI, H.L. Um curso de cálculo – Vol. 1, 2a Edição, São Paulo: Ed. Livros
Técnicos e Científicos, 1987.
[4] IEZZI, G. Fundamentos de Matemática Elementar, Vol. 3 (Trigonometria), 6a Edição,
São Paulo: Editora Moderna, 1985.
[5] IEZZI, G. Fundamentos de Matemática Elementar, Vol. 6 (Complexos – Polinômios -
Equações), 4a Edição, São Paulo: Editora Moderna, 1983.
[6] LIDSKI, V. B.; OVSIANIKOV, L. V.; TULAIKOV, A. N.; SHABUNIN M. I.
Problemas de Matematicas Elementales, Moscou: Ed. MIR, 1972.
[7] MORGADO, A. C; WAGNER, E.; DO CARMO, M. P., Trigonometria e Números
Complexos, 4a Edição, Rio de Janeiro: Publicação da Sociedade Brasileira de Matemática,
2001.
[8] SHKLARSKY, D.O., CHENTZOV, N.N., YAGLOM, I.M. The USSR Olympiad
Problem Book, New York, Dover Publications, Inc., 1994.

EUREKA! N°33, 2011

30
Sociedade Brasileira de Matemática

UMA INTERESSANTE DEDUÇÃO PARA A


FÓRMULA DE HERÃO
Flávio Antonio Alves, Amparo – SP

♦ Nível Intermediário

Nesta nota sugerimos uma dedução para a fascinante fórmula de Herão por meio
de aplicações dos números complexos à geometria.

Sejam z1 = a + bi e z 2 = c + di dois números complexos não nulos e distintos.


Vamos considerar o triângulo de vértices o, z1 e z2 (veja a figura abaixo).

Im

z2

Ө2 z1
Ө1
o Re

A área S do triângulo acima é dada por:


1 1
S= z1 z2 sen ( θ2 − θ1 ) = Im { z2 z1} .
2 2
Vamos multiplicar essa expressão, membro a membro, por 2 e elevar ao quadrado
ambos os termos da igualdade. Assim,

( z2 z1 − z1 z2 )
2

4 S 2 =  Im { z2 z1} =
2

( 2i )
2
=
1
4
(2 z2
2 2
z1 − z22 z12 − z12 z22 )

EUREKA! N°33, 2011

31
Sociedade Brasileira de Matemática

=
1
4
[ 2 2 2

4
]
4 z2 z1 − (z2 z1 + z1 z2 ) = [2 z2 z1 − (z2 z1 + z1 z2 )] [2 z2 z1 + (z2 z1 + z1 z2 )]
1

=
1
4
[
z1 − z 2 − ( z1 − z 2
2
) ] [( z
2
1 + z2 ) 2
− z1 − z 2
2
].
Notemos que:

− ( z1 − z 2 ) = (z − z 2 + z 2 − z1 )( z1 − z 2 + z1 − z 2 ),
2 2
i) z1 − z 2 1

E, do mesmo modo, temos que:

(
ii) z1 + z 2 )2
− z1 − z 2
2
= ( z1 + z 2 − z1 − z 2 )( z 1 + z 2 + z1 − z 2 ) .
Substituindo (i) e (ii) na expressão acima, vem:
1
= ( z1 − z2 + z2 − z1 )( z1 − z2 + z1 − z2 )( z1 + z2 − z1 − z2 )( z1 + z2 + z1 − z2 )
4
( z1 + z 2 + z1 − z 2 )
Nesse caso, pondo-se p = , onde p é o semi-perímetro,
2
concluímos que:

4S 2 =
1
(2 p − 2 z1 )(2 p − 2 z 2 )(2 p − 2 z 1 − z2 ) (2 p ) ⇒
4
4 S 2 = 4( p − z1 )( p − z 2 )( p − z 1 − z2 ) ( p) ⇒
S 2 = ( p − z1 )( p − z 2 )( p − z 1 − z2 ) ( p) ⇒
S= ( p ) ( p − z1 )( p − z 2 )( p − z1 − z 2 ) , que é a fórmula de Herão.

EUREKA! N°33, 2011

32
Sociedade Brasileira de Matemática

RAÍZES DA UNIDADE
Anderson Torres & Eduardo Tengan

♦ Nível Intermediário

Para θ∈ a Fórmula de Euler nos permite escrever ei⋅θ = cosθ + i ⋅ senθ.


Ela nos fornece uma maneira prática de multiplicar números complexos. Por
exemplo, o Teorema de De Moivre, normalmente escrito
( cosθ + i ⋅ senθ )
n
= cosnθ + i ⋅ sennθ , na notação exponencial fica bem mais
conciso: ( e )
iθ n
= ei ( nθ ) . Mas, e as raízes da unidade? Elas são os complexos que
zeram o polinômio P ( z ) = z n − 1. Por De Moivre, sabemos que ζ k = e 2 k πi n são
raízes deste polinômio (com 0 ≤ k < n ), e, como são n no total, elas são todas as
raízes.
E assim temos o primeiro resultado do artigo:

zn −1 = ∏ (z − ζ ),
0≤ k < n
k

em que ζ = e 2k πi n .

Raízes da unidade têm um monte de aplicações. Uma das mais imediatas é


simplificar contas com funções trigonométricas, usando estas fórmulas aqui:

e i θ + e − iθ e i θ − e − iθ
cosθ = ; senθ =
2 2i

PROBLEMA 1: calcule a soma tenebrosa



∑ sen
0≤ k < n n

SOLUÇÃO: Usando a nossa recente descoberta, esta soma se transforma numa


progressão geométrica! Sendo ζ = eiπ n , temos

kπ ζk − ζ−k 1  k 
∑ sen = ∑ =  ∑ ζ k − ∑ ( ζ −1 ) 
0≤ k < n n 0≤ k < n 2i 2i  0≤ k < n 0≤ k < n 

EUREKA! N°33, 2011

33
Sociedade Brasileira de Matemática

kπ 1  ζn − 1 (ζ ) −1 
 −1 n 

0≤ k < n
sen =
n 2i  ζ − 1
− −1
ζ −1 
 

Talvez você deva estar pensando: “uma diferença de complexos dando um


real? Mas como??” Simples: ζ −1 = ζ , logo a soma acima é uma diferença de
conjugados dividida por 2i. É por isso que o resultado é real...

k π 1  −2 2  ζ1 2 + ζ −1 2 π

0≤ k < n
sen =  + −1  = i ⋅ 1 2
n 2i  ζ − 1 ζ − 1  ζ +ζ −1 2
= cotg
2n

Agora, uma aplicação da fatoração de z n − 1 :

PROBLEMA 2: Prove que, para todo inteiro positivo n existem polinômios


f n ,g n ∈ [ x ] tais que

(
f n ( x )( x + 1) + g n ( x ) x 2 + 1 = 2
2r n

)
SOLUÇÃO: Primeiro, testar alguns casos pequenos: n = 1

f1 ( x )( x + 1) + g1 ( x ) ( x 2 + 1) = 2
2

Para eliminar g1 , podemos aplicar x = i, o que nos dá

2
f1 ( i )( i + 1) = 2 ⇔ f1 ( i ) =
2
= −i
(1 + i )
2

Podemos tomar f1 ( x ) = − x. Mas e quanto a g1 ( x ) ? Calma, coisas são feitas para


funcionar! Veja que
2 − f1 ( x )( x + 1) = 2 + x ( x + 1)
2 2

tem i com zero, e automaticamente –i (conjugados, a-há!). Portanto o polinômio


acima é múltiplo de x 2 + 1 e basta efetuar a divisão com Briot-Ruffini para achar
g1 .

EUREKA! N°33, 2011

34
Sociedade Brasileira de Matemática

n
Para o caso geral, vamos considerar os zeros de x 2 + 1. Mas os zeros de
n +1

2n x2 − 1
x +1 = n
são justamente as raízes 2n +1 -ésimas da unidade que não são
x2 − 1
n +1
raízes 2n -ésimas da unidade. Logo, se escolhermos ζ = e 2 πi 2 uma raiz 2n+1 -ésima
primitiva da unidade (isto é, que não é raiz t-ésima da unidade para nenhum t
menor que 2n+1 ), temos
∏ (x − ζ )
n
x2 + 1 = k

1≤ k ≤ 2n +1
k ≡1 ( mod 2 )

Escrevendo x = –1,

( −1) ∏ ( −1 − ζ ) ⇔ 2 = ∏ ( −1 − ζ )
2n
+1 = k k

1≤ k ≤ 2n +1 1≤ k ≤ 2n +1
k ≡1 ( mod 2 ) k ≡1 ( mod 2 )

Basta demonstrar que cada 1 + ζ k “é múltiplo” de 1+ ζ . Moleza:

1 + ζ k = (1 + ζ ) (1 − ζ + ζ 2 − ζ 3 + ... − ζ k − 2 + ζ k −1 )

Portanto, podemos escolher f n tal que 2 − f n ( x )(1 + x ) admite raízes ζ k ,k


2n

n
ímpar. Portanto, é divisível por x 2 + 1, o que acaba a demonstração.
Agora, um problema de Geometria:

PROBLEMA 3: ABCDE é um pentágono cíclico de circuncentro O. Os ângulos


internos do pentágono são ∠A = 70°,∠B = 120°,∠C = 120°,∠D = 130° ,∠E = 100°.
Demonstre que as diagonais BD e CE encontram-se em um ponto pertencente à
reta AO.

SOLUÇÃO: Como em qualquer problema de geometria, um bom arrastão para


começar. Inicialmente, vamos ligar o centro aos vértices do pentágono.
Esta é a melhor maneira de aproveitar a conciclicidade dos pontos.
Assim sendo, ∠AOB = 80°,∠BOC = 40°,∠COD = 80°,∠DOE = 20°,∠EOA = 140°.
Mas MDC ( 80,40,20,140) = 20 e portanto os vértices do pentágono estão entre os
360
vértices de um 18-ágono regular (afinal, = 18 )! Agora, vamos colocar as
20
coisas nos eixos: inicialmente, O = 0 , A = 1 (podemos fazer isto por homotetia: se

EUREKA! N°33, 2011

35
Sociedade Brasileira de Matemática

OA ≠ 1, aplicamos uma homotetia de centro O e razão 1 OA ). Seja ω = ei 2 π 18 uma


raiz 18-ésima (primitiva, por sinal) da unidade. Com isto, os vértices estão
determinados. Vamos usar minúsculas para os números complexos associados aos
pontos.

a = 1,b = ω4 ,c = ω6 ,d = ω10 ,e = ω11

Temos que provar que AO,BD,CE são concorrentes. Dada a escolha


esperta que fizemos, basta demonstrar que as retas BD e CE se intersectam em um
ponto real puro. Ou, em outras palavras, que se z é o complexo comum a BD e CE
então z = z .
Bem, para calcular equações de retas, vamos a uma técnica, ou melhor, um
teorema, bastante útil (e que fica como exercício para o leitor, haha!): Dados os
complexos p, q do círculo unitário, a reta pq tem equação dada por

z + pqz = p + q
Temos então:
AO : z =z
BD : z + bd z = b + d
CE : z + cez = c + e
que equivale a
AO : z =z
BD : z + ω14 z = ω4 + ω1ω10
CE : z + ω17 z = ω6 + ω11

ω4 + ω10 ω6 + ω11
Basta provar que AO ∩ BD : z = ; AO ∩ CE : z =
1 + ω14 1 + ω17

Antes de começar a calculeira, vamos estudar algumas propriedades interessantes


de ω. Bem, sabemos que ele é zero do polinômio x18 − 1, e 18 = 2 ⋅ 32. A ideia será
( )( )
fatorar este polinômio até a exaustão... x18 − 1 = x9 − 1 x9 + 1 . Como ω é raiz 18-
ésima primitiva da unidade, o primeiro fator não contém ω como raiz. Assim
( ) + 1 = ( x3 + 1)( x6 − x3 + 1) . Pode-
3
sendo, vamos pensar no outro fator: x9 + 1 = x3

EUREKA! N°33, 2011

36
Sociedade Brasileira de Matemática

se demonstrar (mas não será necessário) que este último fator é irredutível. Então
ω6 − ω3 + 1 = 0, e de quebra ω9 = −1.
Depois dessa volta toda, vamos ao que interessa: comparar as duas expressões de z:
ω4 + ω10 ω6 + ω11
=
1 + ω14 1 + ω17
( ω + ω )(1 + ω ) = ( ω
4 10 17 6
+ ω11 )(1 + ω14 )

( ω − ω )(1 − ω ) = ( ω
4 1 8 6
− ω2 )(1 − ω5 )
ω4 − ω1 − ω12 + ω9 = ω6 − ω2 − ω11 + ω7
ω4 − ω1 + ω3 − 1 = ω6 − ω2 + ω2 + ω7
ω4 − ω1 + ω3 − 1 = ω3 −1 + ω7
ω4 − ω1 = ω7
ω3 −1 = ω6
0=0
E fim!
Outra aplicação interessante das raízes da unidade é como “marcadores”. Veja este
problema:

PROBLEMA 4: Determine uma fórmula fechada para

n
∑ k 
3k  

SOLUÇÃO: Bem, alguém aí conhece algo parecido? Que tal o Binômio de Newton?

  n
∑  k z = (1 + z )
k n

3k  

Agora, já tem alguma ideia do que se pode fazer? Temos que filtrar os múltiplos de
3 desta expansão, e nada melhor que usar uma raiz cúbica da unidade ω = e2 πi 3 .
Substituindo z por 1, ω e ω2 , temos

EUREKA! N°33, 2011

37
Sociedade Brasileira de Matemática

 n
∑ k   = (1 + 1)
n

 k 
 n k n
∑ k  ω = (1 + ω) ⇒ ∑ k   (1 + ω + ω ) = 2 + (1 + ω) + (1 + ω )
n k 2k n n 2 n

 k  k
 n
∑  ω2 k = (1 + ω2 )
n

 k
k 

Agora, se k é múltiplo de 3,1 + ωk + ω2 k = 3; caso contrário, temos uma progressão


ω3k − 1
geométrica de razão ωk ≠ 1, e portanto 1 + ωk + ω2 k = k = 0.
ω −1
Ou seja, matamos todos os não múltiplos de 3!

n n ω +ω
n −n
3∑   = 2n + (1 + ω ) + (1 + ω2 ) = 2n + ( −ω2 ) + ( −ω ) = 2n + 2 ( −1)
n n n n

3k  k  2
 2 nπ 
2n + 2 ( −1) cos 
n

n 
 3 
∑  
3k  k 
=
3

Esta última técnica tem um nome chique: multisecção. Vamos usá-la em um


problema de, adivinha só, Combinatória Enumerativa!

PROBLEMA 5: (IMO 1995, Canadá) Seja p um primo ímpar, e seja S = {1,2,3...2 p} .


Determine o total de subconjuntos A ∈ S que satisfazem as condições a seguir:

• A = p;
• p ∑ x∈A x.

SOLUÇÃO: Este foi o problema 6 da Olimpíada Internacional de 1995, em Montreal,


Canadá. Ela foi tida como uma das mais interessantes pela riqueza de problemas
“legais e divertidos” daquele ano, algo comparável apenas à IMO da Argentina,
que aconteceria dois anos depois.

A solução aqui apresentada é uma pequena modificação daquela dada por Nikolai
Nikolov, ganhador de um Special Prize (prêmio especial, dado pela originalidade).

EUREKA! N°33, 2011

38
Sociedade Brasileira de Matemática

Vamos pensar em uma raiz p-ésima da unidade, primitiva por sinal: ε = e2 πi p . Veja
que ω = εk também é uma raiz p-ésima da unidade, para k ∈{1,2,3,..., p − 1} .
Excluímos o 1 propositalmente, pois ele não terá propriedades tão interessantes
quanto as outras raízes (logo verás o porquê).
Os complexos {ω ,ω ,ω ,...,ω } = {1,ε ,ε
0 1 2 p −1 k 2k
}
,...,εk ( p −1) são raízes p-ésimas da
unidade. Elas são distintas: de fato, se ε = ε para 0 ≤ i ≤ j < p, temos ik jk

e j −i k = e0 = 1 ⇔ p ( j − i ) k e, como 0 < k < p, p ( j − i ) ⇔ j − i = 0.


Agora vamos ao bom e velho polinômio f ( z) = z p −1= ∏0≤ j≤p−1 z −εj =∏1≤ j≤p z −ωj . ( ) ( )
Pensando em Séries Formais, conseguimos trabalhar com este polinômio os
elementos de 1 a p. Como podemos “alcançar” 2p? Oras, eleva ao quadrado!

( f ( z )) = ( z − 1) = ∏ (z −ω )⋅ ∏ (z −ω ) = ∏ (z −ω )⋅ ∏ (z −ω ) =
2 p 2 j j j j

0≤ j ≤ p −1 0≤ j ≤ p −1 1≤ j ≤ p p +1≤ j ≤ 2 p

= ∏ (z − ω )
1≤ j ≤ 2 p
j

Vamos abrir ( f ( z ) ) : ( f ( z ) ) = a0 + a1 z + a2 z 2 + ... + ap z p + ... + a2 p−1 z 2 p−1 + a2 p z 2 p


2 2

Agora, vamos observar como o ap é produzido de uma maneira combinatória.


Primeiramente, escolhemos arbitrariamente p fatores, e coletamos o termo z deles;
isto nos dará o expoente 2p. Já dos outros p fatores, escolhemos o termo −ω j . O ( )
resultado será então

ap = ∏ ( −ω )( −ω ) ...( −ω ) = − ∑
j1 j2 jp
cr ωr
1≤ j1 ≤ j2 <...< j p ≤2 p 0≤r ≤ p −1

em que cr é o total de p-tuplas j1 < j2 < ... < j p tais que


j1 + j2 + ... + j p ≡ r ( mod p ) . A nossa tarefa é achar c0 !
Mas ( f ( z ) ) = z 2 p − 2 z p + 1 ⇒ a p = −2. Assim,
2

c0 + c1ω + c2 ω + ... + c p −1ω p −1 = 2

Em outras palavras, ω é zero do polinômio

EUREKA! N°33, 2011

39
Sociedade Brasileira de Matemática

g ( z ) = ( c0 − 2 ) + c1 z + c2 z 2 + ... + c p −1ω p −1

Lembre-se que todo o raciocínio usado até aqui foi puramente combinatório, e é
válido para qualquer ω que seja raiz p-ésima da unidade (exceto o 1). Logo, todas
as raízes p-ésimas primitivas da unidade são raízes de g. Mas g tem grau p – 1,
portanto:
f ( z)
g ( z ) = c p −1 ⋅ = c p −1 (1 + z + z 2 + ... + z p −1 )
z −1

Igualando os coeficientes, c0 − 2 = c1 = c2 = ... = c p −1 .


2p
Mas c0 + c1 + c2 + ... + c p −1 =   . Contagem dupla: cada p-subconjunto de S é
p 
contado em exatamente um dos ci , justamente aquele correspondente à soma de
seus elementos módulo p.
Resolvendo as equações acima, concluímos que

1 2p 
c0 = 2 +   − 2
p  p  
E fim!
Bem, que tal uns exercícios?

EXERCÍCIOS PROPOSTOS:

1) Determine o valor numérico da série



∑ ∏ cos
n ≥1 1≤ j ≤ n n
A+ B +C
2) Sejam x, y, z, A, B, C reais tais que é inteiro.
π
Defina K r = x r sen ( rA ) + y r sen ( rB ) + z r sen ( rC ) .
Prove que se K1 = K 2 = 0 então K n = 0 para todo n > 0.

3) Fixe um dos vértices de um n-ágono regular inscrito numa circunferência de raio


1, e considere os segmentos que ligam este vértice a todos os outros. Prove que o
produto das medidas de todos estes n – 1 segmentos é n.

EUREKA! N°33, 2011

40
Sociedade Brasileira de Matemática

2π 4π 8π
4) Calcule sen + sen + sen .
7 7 7
2 iπ
Dica: sejam ζ = e 7 , p = ζ + ζ 2 + ζ 4 ,q = ζ 3 + ζ 5 + ζ 6 . O que queremos é calcular a
parte real de p. Calcule p + q e p · q e seja feliz!

5) Se P ,Q , R , S são polinômios tais que


P ( x5 ) + xQ ( x 5 ) + x 2 R ( x 5 ) = ( x 4 + x 3 + x 2 + x + 1) S ( x ) , prove que P (1) = 0.

6) Fórmula de Multisecção: Sendo p ( x ) = a0 + a1 x + a2 x 2 + ... + an x n , e l,m ∈ ,

∑ ω−lk p ( ωk ) 2i π
com 0 ≤ l ≤ m, temos ∑
k ≡ l ( mod m )
ak = 0≤ k ≤ m

m
em que ω = e m
.

2
n  θ  nθ 
7) Mostre que ∑   cos ( k θ ) =  2 cos   cos 
0≤ k ≤ n  k   2  2 

8) (Irlanda) Sabe-se que a, b, c são complexos tais que as raízes da equação


x 3 + ax 2 + bx + c = 0 têm módulo 1. Prove que as raízes de x 3 + a x 2 + b x + c = 0
também têm módulo 1.

( )
496
9) Seja 1 + x + x 2 + x 3 + x 4 = a0 + a1 x + a2 x 2 + ... + a1984 x1984 .

• Determine MDC ( a3 ,a8 ,...,a1983 )


• Prove que 10340 < a992 < 10347

10) Determine todos os polinômios P tais que P x 2 = P ( x ) P ( x − 1) . ( )


11) Determine o número de polinômios de grau 5 com coeficientes entre 1 e 9
inclusive e que sejam divisíveis por x 2 − x + 1.

 2n + 1  3 k
12) Prove que o número ∑  2 não é múltiplo de 5 para qualquer
 2k + 1
0≤ k ≤ n

n ≥ 0.

EUREKA! N°33, 2011

41
Sociedade Brasileira de Matemática

COMO É QUE FAZ?


Resolvermos aqui, a pedidos, três problemas propostos na seção “Olimpíadas ao redor do
mundo”.

1) (Problema 109 – Suíça, 2000, proposto na Eureka! 11) Seja q ( n ) a soma dos

((
algarismos de n. Calcule q q q ( 20002000 ) )) (Proposto por Cícero Soares Furtado,
de Reriutaba – CE).

SOLUÇÃO: Como 20002000 = 22000 ⋅ 106000 , sua representação decimal é a


representação decimal de 22000 seguida de 6000 zeros, e logo
q ( 20002000 ) = q ( 22000 ) . Como 23 < 10, 22000 < 22001 = ( 23 )
667
< 10667 , donde 22000
tem no máximo 667 dígitos. Como cada dígito é no máximo 9,
q ( 20002000 ) = q ( 22000 ) ≤ 9 ⋅ 667 = 6003.

((
Portanto, q q 20002000 ) ) ≤ 6 + 9 + 9 + 9 = 33, e logo q ( q ( q ( 2000 ) ) ) ≤ 3 + 9 = 12.
2000

Por outro lado, como n e q(n) sempre deixam o mesmo resto na divisão por 9, o
(( )) ( (
resto da divisão de q q q ( 20002000 ) = q q q ( 20002000 ) )) por 9 é igual ao resto
da divisão de 22000 por 9. Mas, como 26 = 64 deixa resto 1 quando dividido por 9,
22000 = 26⋅333+ 2 = ( 26 )
333
⋅ 22 = 4 quando dividido por 9. Como

(( ))
q q q ( 20002000 ) ≤ 12 e 4 + 9 = 13 > 12, concluímos que necessariamente

q ( q ( q ( 2000 ) ) ) = 4.
2000

2) (Problema 110 – Grécia, 2000, proposto na Eureka! 11) Determine os números


primo p para os quais o número 1 + p + p 2 + p 3 + p 4 é um quadrado perfeito.
(proposto por Cícero Soares Furtado, de Rariutaba – CE).
Vamos encontrar todos os naturais n tais que 1 + n + n 2 + n3 + n 4 é quadrado
2
 n +1 5n 2 n 1
perfeito. Note que  n 2 +  =n +n +
4 3
+ + > n 4 + n3 + n 2 + n + 1 para
 2  4 2 4

EUREKA! N°33, 2011

42
Sociedade Brasileira de Matemática

n2 n 3
todo n > 3 (pois − − > 0 para todo n > 3). Por outro lado, para todo
4 2 4
2
 n n2
n ∈ ,  n 2 +  = n 4 + n3 + < n 4 + n3 + n 2 + n + 1.
 4 4
n n +1
Como, para todo k ∈ , temos k ≤ n 2 + ou k ≥ n 2 + , se n > 3 temos
2 2
k 2 < n 4 + n3 + n 2 + n + 1 ou k 2 > n 4 + n 3 + n 2 + n + 1 Assim, basta olhar os
casos n ∈ {0,1, 2, 3} . Para n = 0, n 4 + n3 + n 2 + n + 1 = 1 = 12. Para n = 1,
n 4 + n3 + n 2 + n + 1 = 5, que não é quadrado perfeito. Para n = 2,
n + n + n + n + 1 = 31, que não é quadrado perfeito, e,
4 3 2
para n = 3,
n + n + n + n + 1 = 121 = 11 .
4 3 2 2
Assim, o único primo p tal que 1 + p + p 2 + p3 + p4
é quadrado perfeito é p = 3.

3) (Problema 188 – Rússia, 2002, proposto na Eureka! 15) No intervalo ( 22 n ,32 n )


são escolhidos 22 n−1 + 1 números ímpares. Mostre que podemos encontrar entre
estes números dois números tais que o quadrado de cada um deles não é divisível
pelo outro. (Proposto por Anderson Torres, de Santana de Parnaíba – SP).
2
 y−x
SOLUÇÃO: Se x < y são ímpares e y divide x 2 , então y   . Em particular,
 2 
2
 y−x
( )
2
  ≥ y , donde y − x ≥ 2 y , e logo y − 1 = y − 2 y + 1 > x, donde
 2 
y > x + 1. Assim, se 22 n < x0 < x1 < ... < x22 n−1 < 32 n são os números em questão,
temos x j +1 > x j + 1, para todo j ≥ 0, e logo x j > 2n + j , ∀j ≥ 0.

32 n > x22 n−1 > ( 2n + 22 n −1 ) ,


2
Em particular, x22 n−1 > 2n + 22 n −1 , donde e logo
3n > 2n + 22 n−1 , mas isso é falso para todo n ≥ 1 (para n = 1, 3 < 4, para
n = 2, 9 < 4 + 8 e, para n ≥ 3,3n < 22 n −1 : com efeito, 27 = 33 < 25 = 32 e, se
3n < 22 n −1 ,3n +1 = 3 ⋅ 3n < 3 ⋅ 22 n −1 < 4 ⋅ 22 n −1 = 22 n +1 = 22( n +1)−1 ), absurdo.

4) (Problema 113 – Polônia, 2000, proposto na Eureka! 11) Uma sequência p1 , p2,...
de números primos satisfaz à seguinte condição: para n ≥ 3, pn é o maior divisor

EUREKA! N°33, 2011

43
Sociedade Brasileira de Matemática

primo de pn −1 + pn − 2 + 2000. Mostre que a sequência ( pn ) é limitada. (Proposto


por Anderson Torres, de Santana de Parnaíba – SP).

SOLUÇÃO: Vamos mostrar a seguinte afirmação, que implica o resultado:


Para todo k ≥0 existe j com 1 ≤ j ≤ 40 tal que
2
max ( pk + j , pk + j +1 ) ≤ max ( pk , pk +1 ) + 40000 (de fato, a afirmação implica que
3
existe n0 ∈ tal que pn ≤ 160000, para todo n ≥ n0 ; note que
pn + 2 ≤ max ( pn , pn +1 ) + 1000, ∀n ≥ 1).
Suponhamos inicialmente que, para todo r, com 0 ≤ r ≤ 35, pk + r é um primo ímpar.
p + pk + r − 2 + 2000
Então, para todo r, com 2 ≤ r ≤ 36, pk + r ≤ k + r −1 . Definindo
2
q j −1 + q j − 2 + 2000
q0 = pk , q1 = pk +1 e q j = para 2 ≤ j ≤ 36, temos pk + r ≤ qr , para
2
0 ≤ r ≤ 36. Se, para algum r ≤ 36, pk + r ≠ qr , tomando um tal r mínimo temos
p + pk + r − 2 + 2000
pk + r ≤ k + r −1 , e a afirmação vale para j = r. Temos ainda que q j é
6
j
 p + 2pk+1 4000  2pk −2pk+1 4000  1  2000 j
dado pela expressão qj = k − + + ⋅  +
− , para
 3 9   3 9   2 3
0 ≤ j ≤ 42. Assim,
6j
 1
9q6 j = ( 3 pk + 6 pk +1 − 4000) + ( 6 pk − 6 pk +1 + 4000) ⋅  −  + 6000 j ≡ 9 pk + 6000 j ( mod7) .
 2
Portanto, existe s com 0 ≤ s ≤ 6 tal que 9q6 s (e logo q6 s ) é múltiplo de 7. Se s = 0,
a afirmação já vale para j = 1. Se tivéssemos pk + r = qr para 0 ≤ r ≤ 36, tomamos s
q
com 1 ≤ s ≤ 6 tal que q6 s é múltiplo de 7, e teríamos pk + 6 j ≤ 6 s , absurdo.
7
Se pk = 2, a afirmação já vale para j = 1. Finalmente, se pk + r = 2 para algum r
com 1 ≤ r ≤ 35, teremos pk + r +1 ≤ pk + r −1 + 2002 e pk + 2 + 2 ≤ pk +1−1 + 4004, mas um
desses números ( pk + r −1 , pk + r −1 +2002 e pk + r −1 + 4004 ) é múltiplo de 3, logo a
afirmação vale para j = r + 2.

EUREKA! N°33, 2011

44
Sociedade Brasileira de Matemática

SOLUÇÕES DE PROBLEMAS PROPOSTOS


 Publicamos aqui algumas das respostas enviadas por nossos leitores.

131) a) Considere o seguinte jogo: no início um jogador A entrega um número


k ≥ 2 ao jogador B . Quando A entrega um número m ≥ 2 a B, B pode devolver
m – 1 ou m + 1 a A. Quando A recebe um número n ≥ 2 deve, se n for ímpar
n
devolver 3n a B; se n for par mas não múltiplo de 4, pode devolver ou 3n a B, e,
2
n n
se n for múltiplo de 4, pode devolver , ou 3n a B. Qualquer jogador ganha o
4 2
jogo se devolver 1 ao adversário. Caso algum jogador devolva ao adversário um
número maior que 1000k, o jogo empata. Determine, para cada valor de k ≥ 2 , se
algum dos jogadores tem estratégia vencedora, e, nesses casos, qual deles.

b) Resolva o item anterior supondo que A, ao receber um número n ≥ 2, deve


n
devolver 3n a B se n for ímpar, deve devolver a B se n for par mas não múltiplo
2
n
de 4 e deve devolver a B se n for múltiplo de 4.
4

SOLUÇÃO DE JOSÉ DE ALMEIDA PANTERA (RIO DE JANEIRO – RJ)


a) Naturalmente A perde se entregar o número k = 2 ao jogador B, pois B poderá
devolver 1 imediatamente a A.
2 ⋅ 4n + 1
Vamos mostrar que A ganha se entregar a B um número de k da forma ou
3
4n − 1
da forma , para algum n ≥ 1 , e se k > 2 não for de nenhuma dessas formas
3
nenhum dos jogadores tem estratégia vencedora.
2 ⋅ 41 + 1
Para isso, note que, se A entrega 3 = a B, B pode devolver 2 ou 4 a A, e,
3
41+1 − 1
em qualquer caso, A pode devolver 1 a B e ganhar o jogo. Se A entrega 5 =
3
a B, B pode devolver 4 ou 6 a A.
Se devolve 4, A pode devolver 1 a B e ganhar. Se devolve 6, A pode devolver 3 a B,
e ganhar a seguir, como vimos antes.

EUREKA! N°33, 2011

45
Sociedade Brasileira de Matemática

2 ⋅ 4n +1 + 1
Em geral, podemos argumentar por indução: se A entrega a B, com
3
2 ⋅ 4n+1 − 2 4n+1 − 1
n ≥ 1, B pode devolver a A, caso em que A pode devolver a B,
3 3
2 ⋅ 4n+1 + 4
ganhando o jogo, ou B pode devolver a A, caso em que A pode devolver
3
2 ⋅ 4n + 1 4n+ 2 − 1
a B, ganhando o jogo. De modo similar, se A entrega a B, com
3 3
4n+ 2 − 4 4n+1 − 1
n ≥ 1, B pode devolver a A, caso em que A pode devolver a B,
3 3
4n+ 2 + 2
ganhando o jogo, ou B pode devolver a A , caso em que A pode devolver
3
2 ⋅ 4n+1 + 1
a B, ganahndo o jogo.
3
Notemos que A sempre pode no mínimo empatar o jogo se k ≥ 3. De fato, se em
algum momento do jogo A entrega m ≥ 3 a B, B devolve no mínimo m – 1, e A
pode devolver o triplo, que é no mínimo 3 ( m − 1) > m.
Assim, A pode devolver números cada vez maiores, que em algum momento
ultrapassarão 1000k, empatando o jogo.
Veremos agora que B pode garantir o empate se A entrega um número que não é
das formas descritas anteriormente. Mais precisamente, veremos que, se A envia
um número que não pertence ao conjunto
2 ⋅ 4 +1
n
  4 −1
n +1

X :=  , n ≥ 0 ∪  , n ≥ 1 , então B pode devolver um número a
 3   3 
partir do qual A não pode devolver nenhum número pertencente a X (note que
1∈ X ). Temos X = {1,3,5,11, 21,...} . Se A envia a B um número par m, B pode
devolver m – 1 ou m + 1, que são ímpares, a A, que deve devolver o triplo a B.
Como não é possível que 3 ( m − 1) e 3 ( m + 1) pertençam ambos a X, isso mostra
nossa afirmação no caso m par. Se A envia a B um número ímpar m, que não
pertence a X, não é difícil ver que m − 1∉ {2k , k ∈ X } ∪ {4k , k ∈ X } ou
m + 1∉ {2k , k ∈ X } ∪ {4k , k ∈ X } . Isso implica a afirmação no caso m par.

b) No meio do jogo, B só recebe um número par se A tiver acabado de dividir um


número (necessariamente múltiplo de 8) por 4. E, se B recebe um ímpar, devolverá
um par, o que forçará A a dividi-lo por 2 ou por 4. Assim, os números tendem a

EUREKA! N°33, 2011

46
Sociedade Brasileira de Matemática

decrescer, e , nos casos que empatavam, B ganha o jogo. Por outro lado, A ganha o
jogo, com a mesma estratégia, nos mesmos casos que no item A, pois nesses casos
sempre devolve números ímpares.

132) a) Considere uma família ℑ de 2000 círculos de raio 1 no plano tal que dois
círculos de ℑ nunca são tangentes e cada círculo de ℑ intersecta pelo menos dois
outros círculos de ℑ . Determine o número mínimo possível de pontos do plano
que pertencem a pelo menos dois círculos de ℑ .

SOLUÇÃO DE ZOROASTRO AZAMBUJA NETO (RIO DE JANEIRO – RJ)


Mostraremos que esse número mínimo é igual a 2000.
Para isso, consideramos um triângulo equilátero de lado 3 e os seguintes quatro
círculos: o círculo circunscrito ao triângulo e os três círculos que contêm o
circuncentro do triângulo e dois de seus vértices. Esses quatro círculos têm raio 1, e
cada um deles intersecta os outros três.
Considerando 500 cópias disjuntas dessa configuração de círculos, obtemos 2000
círculos como no enunciado tais que há 2000 pontos que pertencem a pelo menos
dois deles.

Para concluir, vamos mostrar que, numa configuração de n círculos de raio 1 no


plano ( n ≥ 2 ) em que cada círculo intersecta pelo menos outro círculo e não há
dois círculos tangentes, há sempre pelo menos n pontos que pertencem a pelo
menos dois dos círculos. Vamos mostrar, por indução em n, que, na situação acima,
não apenas há pelo menos n pontos que pertencem a pelo menos dois dos círculos,
mas também que existe uma função injetiva do conjunto dos n círculos no conjunto
dos pontos que pertencem a pelo menos dois dos círculos tal que a imagem de cada
círculo pertence a ele.

EUREKA! N°33, 2011

47
Sociedade Brasileira de Matemática

Para isso, note que se n = 2 isso é claramente verdadeiro. Suponha agora que
m ≥ 2 e que isso vale para todo n com 2 ≤ n ≤ m, e considere uma configuração
de m + 1 círculos como antes.
Suponha inicialmente que algum desses círculos, digamos C1 , intersecta só um dos
outros círculos, digamos C2 , seja C1 ∩ C2 = { p, q}.
Temos dois casos: no primeiro, C2 só intersecta C1. Então associamos p a C1 , q a
C2 e usamos a hipótese de indução para os m – 1 círculos restantes. No segundo
caso, C2 intersecta algum dos outros círculos. Então associamos P a C1 e usamos a
hipótese de indução para os m círculos C2 , C3 ,..., Cm +1.
Se, por outro lado, cada um desses m + 1 círculos intersecta pelo menos dois dos
outros, temos de novo dois casos:
Se há no total pelo menos m + 1 pontos que pertencem a pelo menos dois dos
círculos, podemos separar um dos círculos, digamos Cm +1 , e fixar uma injecção de
{C1 , C2 ,..., Cm } no conjunto dos pontos que pertencem a pelo menos dois dos
círculos C j , j ≤ m tal que a imagem de cada Ci , que chamaremos de Pi , pertence a
Ci .
Para cada X ⊂ {C1 ,..., Cm } , o conjunto dos pontos que pertencem a pelo menos
dois círculos de X ∪ {Cm +1} tem pelo menos X + 1 elementos. Se algum ponto
P ∉ { P1 ,..., Pm } pertence a Cm +1 e a algum dos outros círculos, simplesmente
estendemos a injeção associando P a Cm +1 . Senão, construímos uma sequência de
conjuntos A1 , A2 ,... do seguinte modo: A1 = {i ≤ m Pi ∈ Cm +1}. Se A1 , A2 ,..., Ar já
estão definidos, se o conjunto (de pelo menos Ar + 1 ) pontos que pertencem a pelo
menos dois círculos de {Cm +1} ∪ {C j , j ∈ Ar } está contido em {P1 ,..., Pm } ,
definimos {
Ar +1 = Ar ∪ j ≤ m Pj pertence a pelo menos dois círculos de

{Cm +1} ∪ {C j , j ∈ Ar }}. Note que Ar +1 > Ar . Em algum momento, haverá um


ponto P fora de { P1 ,..., Pm } que pertence a pelo menos dois círculos de
Cm +1 ∪ {C j , j ∈ Ar } , e logo a dois círculos C j e C j , com j , j ∈ Ar , j ∉ Ar −1 .
Podemos então alterar a injeção associando C j a P; como existe algum
jr −1 ∈ Ar −1 \ Ar − 2 tal que Pj pertence a C jr −1 , associamos C jr −1 a Pj , e, em geral,
para cada s com 1 < s ≤ r − 1, se já definimos js ∈ As \ As −1 , existe js −1 ∈ As −1 \ As − 2

EUREKA! N°33, 2011

48
Sociedade Brasileira de Matemática

tal que Pjs pertence a C js−1 ; associamos então C js−1 a Pjs . Fazemos isso até associar
C j1 a Pj2 . Como j1 ∈ A1 , podemos associar Cm +1 a Pj1 , estendendo nossa injeção a
{C1 , C2 ,..., Cm , Cm+1} ,
o que prova nossa afirmação.
Finalmente, suponhamos que há apenas m pontos que pertencem a pelo menos dois
dos círculos. Observamos que, como os círculos têm raio 1, se um par de pontos
está contido em dois círculos de família, não estará contido em nenhum outro
círculo de família, e portanto, se um ponto pertence a r ≥ 2 círculos da família,
cada um desses r círculos intersecta os outros r – 1 em outros r – 1 pontos distintos,
e distintos do ponto comum aos r círculos. Assim, cada um desses r círculos
contém pelo menos r pontos que pertecem a pelo menos dois círculos da família.
Podemos considerar uma injeção que leva {C1 , C2 ,..., Cm } no conjunto desses
pontos, a qual será uma bijeção. Sendo Pi a imagem de Ci , podemos considerar a
matriz ( aij ) ,1 ≤ i ≤ m + 1,1 ≤ j ≤ m, onde aij = 1 se Pj pertence a Ci e aij = 0, caso

contrário. Se, para {


j ≤ m, n j = i Pj ∈ Ci = i aij = 1 } { } e, para

{ } {
i ≤ m + 1, si := j Pj ∈ Ci = j aij = 1 , temos, } pelo que observamos acima,
m +1

∑ s = ∑ n = {( i, j ) a }
m m m
si ≥ ni , ∀i ≤ m, donde ∑s ≥ ∑n ,
i =1
i
i =1
i mas
i =1
i
j =1
j ij = 1 , e sm +1 > 0,

pois Cm +1 intersecta outros círculos, absurdo.

133) Considere um n–ágono regular inscrito em um círculo unitário, fixe um


vértice i e denote por dj a distância entre este vértice i e o vértice j. Prove que
n −1

∏ (5 − d ) = F
j ≠i
2
j n
2
onde F1 = 0, F1 = 1 e Fn = Fn −1 , Fn − 2 se n ≥ 2.
j =0

SOLUÇÃO DE ASDRUBAL PAFÚNCIO SANTOS (BOTUCATU – SP)


Podemos supor sem perda de generalidade que i = 0 e que o vértice j é e 2 jπ i n , para
n −1
  2 jπ 
0 ≤ j ≤ n − 1. Queremos então provar que ∏  3 + 2cos 
j =1 n
  = Fn .

2

 2 jπ 
e 2 jπ i n − 1 = ( e 2 jπ i n − 1)( e − 2 jπ i n − 1) = 2 − e
2 2 jπ i n
Temos − e −2 jπ i n = 2 − 2cos  ;
 n 
queremos provar portanto que

EUREKA! N°33, 2011

49
Sociedade Brasileira de Matemática

n −1
  2 jπ  
∏  3 + 2cos 
j =1 n 
  = Fn . Considere agora a sequência de polinômios
2

( f ( x ))
n n≥0
dada por f 0 ( x ) = 2, f1 ( x ) = x e f n +1 ( x ) = xf n ( x ) − f n −1 ( x ) , ∀n ≥ 1.
Temos, para todo n ≥ 0 e todo θ ∈ , f n ( 2cos θ ) = 2cos ( nθ ) . De fato isso vale
para n = 0 e n = 1 e, por indução,
f n+1 ( 2cosθ ) = 2cosθ f n ( 2cosθ ) − f n−1 ( 2cosθ ) = 4cosθ cos ( nθ ) − 2cos ( ( n − 1)θ ) =
= 2cos ( ( n + 1)θ ) . Além disso, para todo n ≥ 1, f n ( x ) é um polinômio mônico de
2 kπ
grau n. Como as solução de 2cos ( nθ ) = 2 são dadas por θ = , k ∈ , temos
n
n −1
  2 kπ   n −1
  2 jπ   f n ( x ) − 2
f n ( x ) − 2 = ∏  x − 2cos  , ∀n ≥ 1, donde ∏  x − 2cos   = .
k =0   n  j =1   n  x−2
O que queremos provar equivale a
n −1
 2 jπ   f n ( −3) − 2
( −1) Fn2 = ∏  −3 − 2cos 
1
= − ( f n ( −3) − 2 ) , o que é
n −1
 =
j =1   n  − 3 − 2 5
equivalente a f n ( −3) = 2 + ( −1) ⋅ 5 Fn2 . Como
n

1   1 + 5     1 − 5  
n n

Fn =   −   , ∀n ≥ 0, temos
5   2     2  
 
n n
 −3 + 5   −3 − 5 
2 + ( −1) ⋅ 5F = 
n
 +
2
 . Por outro lado, a sequência
2   2 
n

xn = f n ( −3) satisfaz x0 = 2, x1 = −3 e xn +1 = −3xn − xn −1 , ∀n ≥ 1, e logo (usando o
n n
−3± 5  −3+ 5   −3− 5 
fato de as raízes de x 2 + 3 x + 1 = 0 serem ), xn =   +  ,∀n ≥ 0,
2  2   2 
o que prova a igualdade desejada.

136) Sejam R, r1 , r2 e r3 os raios dos círculos de centro O, O1 , O2 e O3 ,


respectivamente, conforme a figura abaixo. Prove que: R = r1r2 + r1r3 + r2 r3 .

EUREKA! N°33, 2011

50
Sociedade Brasileira de Matemática

O1

R
O
R2
O2
O3

SOLUÇÃO DE ANDERSON TORRES (SANTANA DE PARNAÍBA – SP)


A boa e velha trigonometria...
A

X
OA
ra

TA

Y
r
O

TB OC
OB
B C

Como sempre, ∠A = α , ∠B = β e ∠C = γ , α +β +γ =π .
Vamos calcular ra , para começar:
AOA + OATA + TAO = AO ⇔ AOA + OATA = AO − OTA
r r
∆AOA X : AOA = a ; ∆AOY ; AO = . Substituindo:
α α
sen sen
2 2

EUREKA! N°33, 2011

51
Sociedade Brasileira de Matemática

 α α
ra r  1 − sen 2  ra
1 − sen
2
+r = − r ⇔ ra = r ⋅   , ou =
α a α α
 1 + sen  r α
sen sen 1 + sen
2 2  2 2
Como precisaremos de um “quadrado”, vamos aplicar um truque: a tangente do
meio arco.
α
2 tan
1− 4 2
α α  α α α
1 − sen 1 + tan 2
1 − 2 tan 1 − tan + tan 2
2 = 4 = 4 4 = 4 .
α α α α  α 
1 + sen 2 tan 1 + 2 tan + tan 2  1 + tan 
2 1+ 4 4 4  4

1 + tan
4
Com isto, já podemos substituir na igualdade que queremos demonstrar:

α β α γ β γ
1 − tan 1 − tan 1 − tan 1 − tan 1 − tan 1 − tan
4⋅ 4 + 4⋅ 4+ 4⋅ 4 =1
α β α γ β γ
1 + tan 1 + tan 1 + tan 1 + tan 1 + tan 1 + tan
4 4 4 4 4 4
α β γ
Para escrever menos, seja a = tan , b = tan , c = tan . Abrindo os
4 4 4
denominadores,
(1 − a )(1 − b )(1 + c ) + (1 − a )(1 + b )(1 − c ) + (1 + a )(1 − b )(1 − c ) = (1 + a )(1 + b )(1 + c )
3 − ( a + b + c ) − ( ab + ac + bc ) + 3 ( abc ) = 1 + ( a + b + c ) + ( ab + ac + bc ) + ( abc )
1 − ( a + b + c ) − ( ab + ac + bc ) + ( abc ) = 0 .
Mas isto é fácil?

β γ  α
+ tan  +  tan
π α β γ  4  4 4 =
1 = tan = tan  + +  =
4  4 4 4  1 − tan tan  β + γ 
α
 
4  4 4

EUREKA! N°33, 2011

52
Sociedade Brasileira de Matemática

β γ
tan + tan
α 4 4
tan +
4 β γ α β γ α β γ
1 − tan tan tan + tan + tan − tan tan tan
= 4 4 = 4 4 4 4 4 4
β γ α β α γ β γ
α tan 4 + tan 4 1 − tan
4
tan
2
− tan
4
tan
4
− tan
4
tan
4
1 − tan
4 β γ
1 − tan tan
4 4

⇒ a + b + c − abc = 1 − ( ab + ac + bc ) , como esperado.

137) Seja A um conjunto de quinze pontos de 2 tal que a distância de cada ponto
à origem é positiva e menor do que 1 e que quaisquer dois deles nunca sejam
colineares com a origem. Mostre que existe um triângulo com dois vértices em A e
1
um na origem cuja área é menor que .
4

SOLUÇÃO DE ITAMAR SALES DE OLIVEIRA FILHO (CEDRO – CE)

y
α3
α2
α1
0 α15 1
–1

–1

Distribuímos aleatoriamente os 15 pontos. Como a distância à origem é sempre


menor do que 1, com certeza todos esses pontos são interiores à circunferência de
raio 1 e centro na origem.

EUREKA! N°33, 2011

53
Sociedade Brasileira de Matemática

Pelo fato de não existerem dois colineares com o centro, temos os 15 ângulos
representados na figura (α1 ,α 2 ,...,α15 ) . Obviamente:
α1 ,α 2 ,...,α15 = 360°.
Vamos provar que existe pelo menos um ângulo menor do que ou igual a 24° . Para
isso, suponha o contrário, ou seja, α n > 24, para todo n Então:
α1 + α 2 + ... + α15 > 15 × 24° = 360° → absurdo.
Então realmente existe pelo menos um ângulo α ≤ 24°. Suponha α i = ∠XOY .
Olhando para o triângulo XOY:
1
Área XOY = ⋅ OX ⋅ OY ⋅ senα1 . Contudo, OX e OY são menores do que 1 e
2
senα i ≤ sen24° < sen30°, substituindo:
1 1 1 1
A < ⋅ 1 ⋅ 1 ⋅ sen30° → A < ⋅ → A < .
2 2 2 4
1
Então existe um triângulo como no enunciado cuja área é menor do que .
4

138) Calcule o máximo divisor comum entre todos os números da forma x ⋅ y ⋅ z ,


onde ( x, y, z ) percorre todas as soluções inteiras da equação x 2 + y 2 = z 2 com
x ⋅ y ⋅ z ≠ 0.

SOLUÇÃO DE MARCÍLIO MIRANDA DE CARVALHO (TERESINA – PI)


Seja d mdc entre todos os inteiros da forma x ⋅ y ⋅ z onde (x, y, z) percorre todas as
soluções inteiras da equação x 2 + y 2 = z 2 com x ⋅ y ⋅ z ≠ 0.
Note que (3, 4, 5) é solução, logo temos que d ≤ 60.

AFIRMAÇÃO 1: Se uma tripla ( x, y , z ) é solução da equação x 2 + y 2 = z 2 então


x ⋅ y ⋅ z é múltiplo de 3.

PROVA: Suponhamos, por absurdo, que x e y não são múltiplos de 3. Então


z 2 = x 2 + y 2 ≡ 2 ( mod 3) , absurdo!. Logo x ou y tem que ser múltiplo de 3. Assim
temos que x ⋅ y ⋅ z é múltiplo de 3.

AFIRMAÇÃO 2: Se uma tripla ( x, y, z ) é solução da equação x 2 + y 2 = z 2 então


x ⋅ y ⋅ z é múltiplo de 5.

EUREKA! N°33, 2011

54
Sociedade Brasileira de Matemática

PROVA: Suponhamos, por absurdo, que x e y não são múltiplos de 5. Então


x 2 + y 2 ≡ 0 ou 2 ou 3 (mod 5). No primeiro caso temos que z é múltiplo de 5,
portanto x ⋅ y ⋅ z é múltiplo de 5. No segundo e terceiro casos temos um absurdo,
pois um número quadrado perfeito só pode deixar restos 0, 1 ou 4 módulo 5.
Assim, temos que se ( x, y, z ) é solução da equação x 2 + y 2 = z 2 então x ⋅ y ⋅ z é
múltiplo de 5.

AFIRMAÇÃO 3: Se uma tripla ( x, y , z ) é solução da equação x 2 + y 2 = z 2 então


x ⋅ y ⋅ z é múltiplo de 4.

PROVA: Suponhamos, por absurdo, que x e y são ímpares. Então


x 2 + y 2 ≡ 2 ( mod 4 ) , absurdo. Portanto x ou y tem que ser par. Se x for par, mas não
for múltiplo de 4, então x 2 ≡ 4 ( mod8 ) ⇒ x 2 + y 2 ≡ 4 , 5 ou 0 (mod 8). No primeiro
e terceiro casos temos que y é par, portanto x ⋅ y ⋅ z é múltiplo de 4. No segundo
caso temos um absurdo, pois um número quadrado perfeito só pode deixar restos 0,
1 ou 4 módulo 8. E se y for par é análogo. Assim, temos que x ⋅ y ⋅ z é múltiplo de
4.
Portanto d é múltiplo de 3 ⋅ 4 ⋅ 5 = 60, logo d = 60.

139) Determine todos os inteiros positivos x, y, z satisfazendo x 3 − y 3 = z 2 , onde y


é primo, z não é divisível por 3 e z não é divisível por y.

SOLUÇÃO DE ADRIANO CARNEIRO TAVARES (CAUCAIA – CE)


Suponha que exista uma solução.
Então z 2 = x 3 − y 3 = ( x − y ) ( x 2 + xy + y 2 ) = ( x − y )(( x − y ) 2 + 3 xy ) (I)
Como z não é divisível por 3 e nem por y, e y é um número primo, teremos por (I)
mdc( x, y ) = 1 e mdc( x − y, 3) = 1.
Então mdc ( x 2 + xy + y 2 , x − y ) = mdc (3 xy, x − y ) = 1 (II)
Agora (I) e (II) implicam que x − y = m , x + xy + y = n e z = mn , para certos
2 2 2 2

inteiros positivos m e n.
Temos 4n 2 = 4 x 2 + 4 xy + 4 y 2 = (2 x + y ) 2 + 3 y 2 .
Então 3 y 2 = (2n + 2 x + y )(2n − 2 x − y ). Sendo y um primo, então existem três
possibilidades:
a) 2n + 2 x + y = 3 y 2 , 2n − 2 x − y = 1

EUREKA! N°33, 2011

55
Sociedade Brasileira de Matemática

b) 2n + 2 x + y = 3 y, 2n − 2 x − y = y
c) 2n + 2 x + y = y 2 , 2n − 2 x − y = 3
Em (a), após a subtração das equações temos:
3 y 2 − 1 = 2(2 x + y ) = 2(2m 2 + 3 y ).
Daí, m 2 + 1 = 3 y 2 − 6 y − 3m 2 ≡ 0 (mod 3).
Por outro lado, temos sempre m 2 + 1 ≡ 1 ou 2 (mod 3). Nós chegamos a uma
contradição.
Em (b), subtraindo as equações chegamos x = 0, o que é absurdo!
Subtraindo as equações em (c), chegamos em y 2 − 3 = 2(2 x + y ) = 2(2m 2 + 3 y ),
que pode ser escrito assim:
( y − 3) 2 − 4m 2 = 12, ou seja, ( y − 3 + 2m)( y − 3 − 2m) = 12.
Da equação chegamos a y = 7 e m = 1, pois devemos ter y − 3 + 2m = 6 e
y − 3 − 2m = 2 . Segue que x = y + m2 = 8 e z = 13 mn = m x + xy + y 2 .
Veja que de fato 83 − 73 = 132. Esta é a única solução.

140) Mostre que 2903n − 803n − 464n + 261n é divisível por 1897, para todo n ∈ .

SOLUÇÃO DE MARCELO RIBEIRO DE SOUZA (RIO DE JANEIRO – RJ)


LEMA: Sejam p1 , p2 dois números inteiros primos entre si. Então, se p1 a e
p2 a , ter-se-á p1 p2 a .

DEMONSTRAÇÃO: p1 a ⇔ ∃k ∈ a = kp1. No entanto, deve-se ter p2 a = kp1 , ora,


( p1 , p2 ) = 1, conclui-se
k
como que p2 k ⇔ ∃k1 ∈ = k1 p2 . Finalmente,
a = k1 p1 p2 ⇒ p1 p2 a .
Note-se, inicialmente, que 1897 = 7 × 271. Escreva-se, então:
2903n − 464n − 803n + 261n ≡ ( −78) − ( −78) − ( −10) + ( −10) ≡ 0 ( mod271) , ∀n ∈
n n n n
(i)
2903 − 464 − 803 + 261 ≡ 5 − 2 − 5 + 2 ≡ 0 ( mod 7 ) , ∀n ∈
n n n n n n n n
(ii)

Assim, temos, pelo Lema, que 2903n − 464n − 803n + 261n ≡ 0 ( mod1897 ) , ∀n ∈ ,
como se quis demonstrar.

EUREKA! N°33, 2011

56
Sociedade Brasileira de Matemática

141) Dado a ∈ {0,1, 2,3, 4,5,6,7,8,9} , seja X ≠ ∅ um conjunto finito de inteiros


positivos, tal que nenhum dos seus elementos possui o algarismo a em sua
1
representação decimal. Prove que ∑ < 80.
n∈ X n

SOLUÇÃO DE FABRÍCIO VASCONCELLOS PUPPI (SÃO PAULO – SP)


Por um simples raciocínio combinatório, nota-se que a quantidade de inteiros
positivos com k algarismos que não apresentam algum dígito a em sua
representação decimal é 8 ⋅ 9k −1 , caso a ≠ 0, e 9k , caso a = 0.
Seja N = max( X ) e d o número de dígitos de N. Seja S(T) a operação definida
sobre um subconjunto finito T qualquer de * , tal que:

1
S (T ) = ∑
n∈T n

Pela definição de S, como cada elemento de T tem contribuição positiva no valor


da soma que caracteriza a operação, claramente S é monótona em relação ao seu
argumento, de tal modo que T ⊆ Q ⇒ S (T ) ≤ S ( Q ) . Assim, para
X = {`1, 2,3,..., N ,...,10d − 1} ∩ {n a não é dígito de n} , X ⊆ X . Para todo inteiro
k −1 k −1
positivo n de k dígitos, n ≥ 10 ∴1 n ≤ 1 10 , sendo que a desigualdade estrita
vale para todos os n de k dígitos exceto para um deles. Considerando inicialmente o
caso em que a ≠ 0, tem-se:


1 1   1   1  d 8 ⋅ 9k −1
∑ n < (8 ⋅ 9 ) ⋅ 10
1−1
1−1 
+
 
 ( 8 ⋅ 9 2 −1
) ⋅
102−1 
+ ... + 

( 8 ⋅ 9 d −1
) ⋅ =∑
10d −1  k =1 10k −1
n∈ X

9k −1
( )
d
∴ S ( X ) ≤ S X < 8∑ k −1
k =1 10

Como a somatória à direita é uma série geométrica, trivialmente tem-se que:

1 − ( 9 /10 )
d

S ( X ) < 8⋅ = 80 ⋅ 1 − ( 9 10 )  < 80
d

1 − ( 9 10 )  

EUREKA! N°33, 2011

57
Sociedade Brasileira de Matemática

Para o caso a = 0 é necessário um refinamento de análise, pois o uso de processo


idêntico ao acima só permitiria afirmar que S ( X ) < 90. Pra este caso, separou-se
cada conjunto dos números de k algarismos em dois subconjuntos disjuntos: um
dos 4 ⋅ 9k −1 inteiros que satisfazem 10k −1 ≤ n < 5 ⋅ 10k −1 e outro dos 5 ⋅ 9k −1 inteiros
que satisfazem 5 ⋅ 10k −1 ≤ n < 10k . Assim, por um raciocionio análogo ao do caso
anterior:

9 k −1 9k −1 9 k −1
( )
d d d
S ( X ) ≤ S X < 4∑ k −1
+ 5∑ k −1
= 5∑ k −1
k =1 10 k =1 5 ⋅ 10 k =1 10

1 − ( 9 10 )
d

∴S ( X ) < 5⋅ = 50 ⋅ 1 − ( 9 10 )  < 50 < 80 .


d

1 − ( 9 10 )  

Agradecemos o envio de soluções e a colaboração de:

Adriano Carneiro Tavares (Caucaia – CE) Prob. 140


Anderson Torres (Santana de Parnaíba – SP) Prob. 133, 137, 138, 139, 140, 141
Douglas Oliveira de Lima (Brasília – DF) Prob. 140
Fabrício Vasconcellos Puppy (São Paulo – SP) Prob. 137, 138, 140.
Flávio Antonio Alves (Amparo – SP) Prob. 136
Itamar Sales de Oliveira Fiolho (Cedro – CE) Prob. 136
Jean Pierre Youyoute (Rio de Janeiro – RJ) Prob. 138
Lucas Alves, Douglas Oliveira de Lima, Danillo Leal, Gustavo Prob. 136
Campelo, Júlio Castro (Brasília – DF)
Lucas Colucci Prob. 138 e 140
Marcelo Ribeiro de Souza (Rio de Janeiro – RJ) Prob. 136
Marcílio Miranda de Carvalho (Teresina – PI) Prob. 140
Marcos Martinelli (Brasília – DF) Prob. 133 e 136
Matheus Henrique Alves Moura (Fortaleza – CE) Prob. 136
Renato Carneiro (Belo Horizonte – MG) Prob. 140

Continuamos aguardando soluções para os problemas 134 e 135.

EUREKA! N°33, 2011

58
Sociedade Brasileira de Matemática

PROBLEMAS PROPOSTOS
Convidamos o leitor a enviar soluções dos problemas propostos e sugestões de novos
problemas para próximos números.

142) Seja A = {4,8,9,16, 25, 27,36,64,...} o conjunto das potências não triviais
(números da forma a b , com a ≥ 2, b ≥ 2 naturais). Prove que, para todo natural
n ≥ 1, existe um natural k tal que todos os termos da progressão aritmética
k , 2k ,3k ,..., nk pertencem a A.

143) Determime todas as funções f , g,h : → tais que


f ( xy ) = g ( x + y ) + h ( x + y ) , ∀x, y ∈ .
3 3

144) Seja x ≥ 1 um número racional tal que existe uma constante c ≠ 0 e uma
sequência ( an )n≥1 de inteiros tal que lim ( cx n − an ) = 0. Prove que x é inteiro.
n →∞

145) Encontretodos os números racionais p, q, r de modo que


π 2π 3π
p cos + q cos + r cos = 1.
7 7 7

146) Determine todos os subconjuntos não-vazios A, B, C de de modo que:

a) A ∩ B = B ∩ C = C ∩ A = ∅.
b) A ∪ B ∪ C = .
c) para quaisquer a ∈ A, b ∈ B e c ∈ C , temos: a + c ∈ A, b + c ∈ B e a + b ∈ C.

( −1) ( −1)
k n
n−2 −1
147) Demonstre que ∑
k =0  ( 2k + 1) π 
=n+
2
, para todo inteiro n ≥ 2.
sen  
 4n − 2 
n −1
n − k 
148) Sejam m e n inteiros positivos. Calcule ∑ 
k =0 m 
.

149) a) Deseja-se organizar um torneio de futebol com n times ( n ≥ 2 ) em que


cada time joga uma vez contra cada um dos outros, dividido em um certo número
de rodadas. Em cada rodada cada time joga no máximo uma partida.

EUREKA! N°33, 2011

59
Sociedade Brasileira de Matemática

Prove que, se n é ímpar, é possível organizar um tal torneio com n rodadas e, se n é


par, é possível organizar um tal torneio com n – 1 rodadas.

b) Uma matriz n × n é preenchida com elementos do conjunto


S = {1, 2,3,..., 2n − 1} . Sabe-se que, para todo i ∈ {1, 2,..., n} , a i-ésima linha e a i-
ésima coluna contêm juntas todos os elementos de S.
Quais os possíveis valores de n?

150) Sejam a, b e c números reais tais que ( a − b ) + ( b − c ) + ( c − a ) = 9.


3 3 3

1 1 1
Prove que + + ≥ 3 3.
(a − b) ( a − b) (c − a)
2 2 2

Problema 142 adaptado de um problema proposto por Anderson Torres (Santana de Parnaíba
– SP); 143 proposto por Anderson Torres (Santana de Parnaíba – SP); 144, 145 e 146
propostos por Carlos da Silva Ramos (Belém – PA); 147 e 148 propostos por Marcos
Martinelli; 149 adapatado de um problema proposto por Anderson Torres (Santana de
Parnaíba – SP); 150 adaptado de um problema proposto por Adriano Carneiro (Caucaia – CE).

EUREKA! N°33, 2011

60
Sociedade Brasileira de Matemática

AGENDA OLÍMPICA
XXXIII OLIMPÍADA BRASILEIRA DE MATEMÁTICA

NÍVEIS 1, 2 e 3
Primeira Fase – sábado, 18 de junho de 2011
Segunda Fase – sábado, 3 de setembro de 2011
Terceira Fase – sábado, 15 de outubro de 2011 (níveis 1, 2 e 3)
domingo, 16 de outubro de 2011 (níveis 2 e 3 - segundo dia de prova)

NÍVEL UNIVERSITÁRIO
Primeira Fase – sábado, 3 de setembro de 2011
Segunda Fase – sábado, 15 e domingo, 16 de outubro de 2011

IV ROMANIAN MASTER OF MATHEMATICS (RMM)


23 a 28 de fevereiro de 2011(Bucareste, Romênia)

ASIAN PACIFIC MATH OLYMPIAD (APMO)


12 de março de 2011

XVII OLIMPÍADA DE MAIO


7 de maio de 2011

XXII OLIMPÍADA DE MATEMÁTICA DO CONE SUL


14 a 20 de agosto de 2011(La Paz, Bolívia)

LII OLIMPÍADA INTERNACIONAL DE MATEMÁTICA


13 a 24 de julho de 2011(Amsterdam, Holanda)

I OLIMPÍADA DE MATEMÁTICA DA LUSOFONIA


20 a 31 de julho de 2011(Coimbra, Portugal)

XVII OLIMPÍADA INTERNACIONAL DE MATEMÁTICA UNIVERSITÁRIA (IMC)


24 a 30 de julho de 2011(Blagoevgrad, Bulgária)

XXV OLIMPÍADA IBEROAMERICANA DE MATEMÁTICA


23 de setembro a 1 de outubro de 2011(São José, Costa Rica)

II COMPETIÇÃO IBEROAMERICANA INTERUNIVERSITÁRIA DE MATEMÁTICA


2 a 8 de outubro de 2011(Quito, Equador)

XIII OLIMPÍADA IBEROAMERICANA DE MATEMÁTICA UNIVERSITÁRIA

EUREKA! N°33, 2011

61
Sociedade Brasileira de Matemática

COORDENADORES REGIONAIS
Alberto Hassen Raad (UFJF) Juiz de Fora – MG
Américo López Gálvez (USP) Ribeirão Preto – SP
Antonio Carlos Nogueira (UFU) Uberlândia – MG
Benedito Tadeu Vasconcelos Freire (UFRN) Natal – RN
Bruno Holanda (CAEN – UFC) Fortaleza – CE
Carmen Vieira Mathias (UNIFRA) Santa María – RS
Claus Haetinger (UNIVATES) Lajeado – RS
Cláudio de Lima Vidal (UNESP) S.J. do Rio Preto – SP
Denice Fontana Nisxota Menegais (UNIPAMPA) Bagé – RS
Disney Douglas Lima de Oliveira (UFAM) Manaus – AM
Edson Roberto Abe (Colégio Objetivo de Campinas) Campinas – SP
Edney Aparecido Santulo Jr. (UEM) Maringá – PR
Fábio Brochero Martínez (UFMG) Belo Horizonte – MG
Florêncio Ferreira Guimarães Filho (UFES) Vitória – ES
Francinildo Nobre Ferreira (UFSJ) São João del Rei – MG
Genildo Alves Marinho (Centro Educacional Leonardo Da Vinci) Taguatingua – DF
Herivelto Martins (USP – São Carlos) São Carlos – SP
Gilson Tumelero (UTFPR) Pato Branco – PR
Ivanilde Fernandes Saad (UC. Dom Bosco) Campo Grande – MS
João Benício de Melo Neto (UFPI) Teresina – PI
João Francisco Melo Libonati (Grupo Educacional Ideal) Belém – PA
Diogo Diniz (UFPB) Campina Grande – PB
José Luiz Rosas Pinho (UFSC) Florianópolis – SC
José Vieira Alves (UFPB) Campina Grande – PB
José William Costa (Instituto Pueri Domus) Santo André – SP
Krerley Oliveira (UFAL) Maceió – AL
Licio Hernandes Bezerra (UFSC) Florianópolis – SC
Luciano G. Monteiro de Castro (Sistema Elite de Ensino) Rio de Janeiro – RJ
Luzinalva Miranda de Amorim (UFBA) Salvador – BA
Marcelo Dias (Grupo Educacional Etapa) São Paulo – SP
Marcelo Antonio dos Santos FACOS Osório – RS
Marcelo Rufino de Oliveira (Grupo Educacional Ideal) Belém – PA
Newman Simões (Cursinho CLQ Objetivo) Piracicaba – SP
Nivaldo Costa Muniz (UFMA) São Luis – MA
Osnel Broche Cristo (UFLA) Lavras – MG
Uberlândio Batista Severo (UFPB) João Pessoa – PB
Raul Cintra de Negreiros Ribeiro (Colégio Anglo) Atibaia – SP
Reginaldo de Lima Pereira (Escola Técnica Federal de Roraima) Boa Vista – RR
Reinaldo Gen Ichiro Arakaki (UNIFESP) SJ dos Campos – SP
Ricardo Amorim (Centro Educacional Logos) Nova Iguaçu – RJ
Ronaldo Alves Garcia (UFGO) Goiânia – GO
Rogério da Silva Ignácio (Col. Aplic. da UFPE) Recife – PE
Rosangela Ramon (UNOCHAPECÓ) Chapecó – SC
Sérgio Cláudio Ramos (IM-UFRGS) Porto Alegre – RS
Seme Gebara Neto (UFMG) Belo Horizonte – MG
Tadeu Ferreira Gomes (UEBA) Juazeiro – BA
Tomás Menéndez Rodrigues (U. Federal de Rondônia) Porto Velho – RO
Valdenberg Araújo da Silva (U. Federal de Sergipe) São Cristóvão – SE
Vânia Cristina Silva Rodrigues (U. Metodista de SP) S.B. do Campo – SP
Wagner Pereira Lopes (CEFET – GO) Jataí – GO
Wanderson Breder (CEFET – RJ) Nova Friburgo – RJ
William Serafim dos Reis (UFT – TO) Arraias – TO

EUREKA! N°33, 2011

62
CONTEÚDO

XXXII OLIMPÍADA BRASILEIRA DE MATEMÁTICA 3


Problemas e soluções da Primeira Fase

XXXII OLIMPÍADA BRASILEIRA DE MATEMÁTICA 15


Problemas e soluções da Segunda Fase

XXXII OLIMPÍADA BRASILEIRA DE MATEMÁTICA 34


Problemas e soluções da Terceira Fase

XXXII OLIMPÍADA BRASILEIRA DE MATEMÁTICA 59


Problemas e soluções da Primeira Fase Nível Universitário

XXXII OLIMPÍADA BRASILEIRA DE MATEMÁTICA 67


Problemas e soluções da Segunda Fase Nível Universitário

XXXII OLIMPÍADA BRASILEIRA DE MATEMÁTICA 78


Premiados

AGENDA OLÍMPICA 85

COORDENADORES REGIONAIS 86
Sociedade Brasileira de Matemática

Esta edição é dedicada à memória do professor Sergio Plaza Salinas da


Universidad de Santiago de Chile, que colaborou como membro do comitê editorial da
revista Eureka! desde 1998, e que nos deixou neste ano de 2011.

Os editores

EUREKA! N°34, 2011

2
Sociedade Brasileira de Matemática

XXXII OLIMPÍADA BRASILEIRA DE MATEMÁTICA


Problemas e soluções da Primeira Fase

PROBLEMAS – NÍVEL 1

1. Qual dos números a seguir não é múltiplo de 15?


A) 135 B) 315 C) 555 D) 785 E) 915

2. Ana, Esmeralda e Lúcia têm, juntas, 33 reais. Ana e Esmeralda, juntas, têm 19
reais e Esmeralda e Lúcia, juntas, têm 21 reais. Quantos reais tem Esmeralda?
A) 6 B) 7 C) 10 D) 12 E) 14

3. Aumentando 2% o valor um número inteiro positivo, obtemos o seu sucessor.


Qual é a soma desses dois números?

A) 43 B) 53 C) 97 D) 101 E) 115

4. Qual é o maior número de fichas que podemos colocar em um tabuleiro 5 × 5 ,


no máximo uma em cada casa, de modo que o número de fichas em cada linha e
cada coluna seja múltiplo de 3?
A) 6 B) 9 C) 12 D) 15 E) 24

5. Carlos tem 2010 blocos iguais de 10 cm de


largura por 20 cm de comprimento e 1,5 cm de
espessura e resolveu empilhá-los formando uma
coluna de 20 cm de largura por 40 cm de
comprimento, como na figura. Qual dos valores a
seguir, em metros, é o mais próximo da altura dessa
coluna?

A) 7 B) 7,5 C) 8 D) 8,5 E) 9

6. Qual das alternativas apresenta um divisor de 35 ⋅ 44 ⋅ 53 ?


A) 42 B) 45 C) 52 D) 85 E) 105

(4 )
2

7. Dividindo-se o número 4 por 44 obtemos o número:


A) 2 B) 43 C) 44 D) 48 E) 412

EUREKA! N°34, 2011

3
Sociedade Brasileira de Matemática

8. As quatro faces de um dado são triângulos equiláteros,


numerados de 1 a 4, como no desenho. Colando-se dois
dados iguais, fazemos coincidir duas faces, com o mesmo
número ou não. Qual dos números a seguir não pode ser a
soma dos números das faces visíveis?
A) 12 B) 14 C) 17 D) 18
E) 19

9. Quantos divisores positivos de 120 são múltiplos de 6?


A) 4 B) 5 C) 6 D) 8 E) 12

10. O desenho mostra dois quadrados de papel sobrepostos,


um de lado 5 cm e outro de lado 6 cm. Qual é o perímetro
da figura formada (linha grossa no contorno do desenho),
em centímetros?
A) 31 B) 34 C) 36 D) 38
E) 41

11. O horário indicado pelo relógio ao lado está correto. A


partir desse momento, porém, o relógio começa a atrasar
exatamente 5 minutos a cada hora real. Depois de quantos
dias o relógio voltará a apresentar um horário correto?
A) 1 B) 2 C) 4 D) 6
E) 12

12. No reticulado a seguir, pontos vizinhos na vertical ou na horizontal estão a 1


cm de distância.
1cm

1cm

Qual é a área da região sombreada?


A) 7 B) 8 C) 8,5 D) 9 E) 9,5

EUREKA! N°34, 2011

4
Sociedade Brasileira de Matemática

13. Um jornal publicou a tabela de um campeonato de futebol formado por quatro


times, apresentando os gols marcados e os gols sofridos por cada time. Por uma
falha de impressão, a tabela saiu com dois números borrados, conforme reprodução
a seguir.

Gols marcados Gols sofridos


Craques do Momento 8 4
Independentes 1 6
EC Boleiros 4 ***
Esmeralda FC 5 ***

Sabe-se que o time Esmeralda FC sofreu dois gols a mais que o time EC Boleiros.
Quantos gols sofreu o time Esmeralda FC?
A) 2 B) 3 C) 4 D) 5 E) 6

14. Ana começou a descer uma escada no mesmo instante em que Beatriz começou
3
a subi-la. Ana tinha descido da escada quando cruzou com Beatriz. No
4
momento em que Ana terminar de descer, que fração da escada Beatriz ainda terá
que subir?
1 1 1 5 2
A) B) C) D) E)
4 3 12 12 3

15. Alguns números inteiros positivos, não necessariamente distintos, estão escritos
na lousa. A soma deles é 83 e o produto é 1024. O menor número é igual a:
A) 1 B) 2 C) 4 D) 8 E) 16

16. Numa sala do 6º ano, todos gostam de pelo menos uma das duas matérias:
3 5
Matemática ou Português. Sabe-se que dos alunos gostam de Matemática e
4 7
dos alunos gostam de Português. A sala tem 56 alunos. Quantos alunos gostam
dessas duas matérias ao mesmo tempo?
A) 4 B) 8 C) 13 D) 24 E) 26

EUREKA! N°34, 2011

5
Sociedade Brasileira de Matemática

17. O desenho representa um canto de um tabuleiro


retangular convencional, formado por quadradinhos de
lado 1 cm. Nesse tabuleiro, 17 quadradinhos são
brancos. Qual é a área do tabuleiro, em centímetros
quadrados?

A) 29 B) 34 C) 35 D) 40
E) 150

18. A figura representa uma barra de chocolate que


tem um amendoim apenas num pedaço. Elias e Fábio
querem repartir o chocolate, mas nenhum deles gosta
de amendoim. Então combinam dividir o chocolate
quebrando-o ao longo das linhas verticais ou
horizontais da barra, um depois do outro e retirando o
pedaço escolhido, até que alguém tenha que ficar com
o pedaço do amendoim. Por sorteio, coube a Elias
começar a divisão, sendo proibido ficar com mais da
metade do chocolate logo no começo. Qual deve ser a
primeira divisão de Elias para garantir que Fábio fique
com o amendoim ao final?
A) Escolher a primeira coluna à esquerda.
B) Escolher as duas primeiras colunas à esquerda.
C) Escolher a terceira linha, de cima para baixo.
D) Escolher as duas últimas linhas, de cima para baixo.
E) Qualquer uma, já que Fábio forçosamente ficará com o amendoim.

19. Quatro amigos, Arnaldo, Bernaldo, Cernaldo e Dernaldo estão jogando cartas.
São 20 cartas diferentes, cada carta tem uma entre 4 cores (azul, amarelo, verde,
vermelho) e um número de 1 a 5. Cada amigo recebe cinco cartas, de modo que
todas as cartas são distribuídas. Eles fazem as seguintes afirmações:
Arnaldo: “Eu tenho quatro cartas com o mesmo número.”
Bernaldo: “Eu tenho as cinco cartas vermelhas.”
Cernaldo: “As minhas cinco cartas são de cores que começam com a letra V.”
Dernaldo: “Eu tenho três cartas de um número e duas cartas de outro número.”
Sabe-se que somente uma das afirmações é falsa. Quem fez essa afirmação?
A) Arnaldo B) Bernaldo C) Cernaldo D) Dernaldo
E) Não é possível definir.

EUREKA! N°34, 2011

6
Sociedade Brasileira de Matemática

20. A figura a seguir foi recortada em cartolina e depois dobrada para formar um
icosaedro. As faces em branco foram numeradas de modo que ao redor de cada
vértice (pontas do sólido) apareçam os números de 1 a 5. Qual número está na face
com a interrogação?

ICOSAEDRO
A) 1 B) 2 C) 3 D) 4 E) 5

PROBLEMAS – NÍVEL 2

1. Veja o problema No. 6 do Nível 1.

2. Aumentando em 2% o valor do menor de dois números consecutivos, obtém-se o


maior deles. Qual é a soma desses números?
A) 43 B) 53 C) 97 D) 101 E) 115

3. Veja o problema No. 7 do Nível 1

4. Cecília pegou uma cartolina e


recortou as 8 peças à direita,
formadas por quadradinhos de
mesmo tamanho.
De quantas maneiras
diferentes ela pode
escolher 3 dessas peças
para montar o quadrado
3× 3 à esquerda?
A) 3 B) 4 C) 5 D) 6 E) 7

1 1
5. Os números x e y são distintos e satisfazem x − = y − . Então xy é igual a
x y
A) 4 B) 1 C) –1 D) –4
E) é preciso de mais dados.

EUREKA! N°34, 2011

7
Sociedade Brasileira de Matemática

6. Sônia calculou a média aritmética de dois diferentes números de dois dígitos e


obteve 98. Qual é a diferença entre esses números?
A) 1 B) 2 C) 3 D) 4
E) um número maior que 4

7. Veja o problema No. 17 do Nível 1.

8. Quantos inteiros da lista 100, 101, 102, ..., 999 não possuem algarismos iguais a
2, 5, 7 ou 8?
A) 160 B) 170 C) 180 D) 190 E) 200

9. No triângulo ABC, m(BÂC) = 140o. Sendo M o ponto médio de BC, N o ponto


médio de AB e P o ponto sobre o lado AC tal que MP é perpendicular a AC, qual é
a medida do ângulo NMˆ P ?
A) 40o B) 50o C) 70o D) 90o E) 100o

10. Veja o problema No. 4 do Nível 1

n
11. Para quantos inteiros n o número é também inteiro?
100 − n
A) 1 B) 6 C) 10 D) 18 E) 100

12. Ana começou a descer uma escada de 24 degraus no mesmo instante em que
3
Beatriz começou a subi-la. Ana tinha descido da escada quando cruzou com
4
Beatriz. No momento em que Ana terminar de descer, quantos degraus Beatriz
ainda terá que subir?
A) 2 B) 6 C) 8 D) 10 E) 16

13. Veja o problema 19 do Nível 1.

14. No desenho, o retângulo cinza tem seus vértices sobre os lados do triângulo
equilátero de área 40 cm2. O menor lado do retângulo é um quarto do lado do
triângulo. A área do retângulo em cm2 é:

EUREKA! N°34, 2011

8
Sociedade Brasileira de Matemática

A) 5 B) 10 C) 15 D) 18 E) 22

15. Veja o problema No. 15 do Nível 1.

16. De quantas maneiras é possível desenhar a figura a seguir sem tirar o lápis do
papel (ou qualquer outro utensílio, se você preferir!) começando de P e sem passar
sobre o mesmo ponto mais de uma vez, com exceção do ponto comum aos três
triângulos?

A) 48 B) 24 C) 16 D) 108 E) 27

17. Os pontos P, Q, R, S e T são vértices de um polígono regular. Os lados PQ e TS


são prolongados até se encontrarem em X, como mostra a figura, e QXˆS mede
140o. Quantos lados o polígono tem?

A) 9 B) 18 C) 24 D) 27 E) 40

18. Veja o Problema No. 20 do Nível 1.

19. O professor Piraldo tem dois relógios, ambos digitais de 24 horas. Nenhum dos
dois funciona: um muda de horário com o dobro da velocidade normal e o outro vai

EUREKA! N°34, 2011

9
Sociedade Brasileira de Matemática

de trás para frente, na velocidade normal. Ambos mostram corretamente 13:00.


Qual é a hora certa na próxima vem em que os dois relógios mostrarem o mesmo
horário?
A) 05:00 B) 09:00 C) 13:00 D) 17:00 E) 21:00

20. Uma figura no formato de cruz, formada por quadrados de lado 1, está inscrita
em um quadrado maior, cujos lados são paralelos aos lados do quadrado tracejado,
cujos vértices são vértices da cruz. Qual é a área do quadrado maior?

49 81 32
A) 9 B) C) 10 D) E)
5 8 3

21. Quantos são os pares (x, y) de inteiros positivos tais que x2 – y2 = 22010?
A) 1000 B) 1001 C) 1002 D) 1003 E) 1004

22. Quatro números inteiros positivos a < b < c < d são tais que o mdc entre
quaisquer dois deles é maior do que 1, mas mdc(a, b, c, d) = 1. Qual é o menor
valor possível para d?
A) 10 B) 12 C) 15 D) 30 E) 105

23. Veja o problema No. 8 do Nível 1.

24. Na figura, BC = 2BH.


B

x
80o

H
30o
A C

A) 10o B) 15o C) 16o D) 20o E) 25o

EUREKA! N°34, 2011

10
Sociedade Brasileira de Matemática

25. Os números a e b são reais não negativos tais que a3 + a < b – b3. Então
A) b < a < 1 B) a = b = 1 C) a < 1 < b
D) a < b < 1 E) 1 < a < b

PROBLEMAS – NÍVEL 3

1. Dividindo-se o número 4
( 4 ) por 44 obtemos o número:
2

A) 2 B) 43 C) 44 D) 48 E) 412

2. Qual dos seguintes números é um divisor de 35 ⋅ 4 4 ⋅ 53 ?


A) 42 B) 45 C) 52 D) 85 E) 105

3. Veja o Problema No. 8 do Nível 1.

4. Veja o Problema No. 14 do Nível 1.

5. Um quadrado PQRS tem lados medindo x. T é o ponto médio de QR e U é o pé


da perpendicular a QS que passa por T. Qual é a medida de TU?

x x x x x
A) B) C) D) E)
2 3 2 2 2 4

1 1
6. Os números x e y são distintos e satisfazem x − = y − . Então xy é igual a
x y
A) 4 B) 1 C) –1 D) –4
E) é preciso de mais dados

7. Considere todos os números de três algarismos distintos, cada um igual a 0, 1, 2,


3 ou 5. Quantos desses números são múltiplos de 6?
A) 4 B) 7 C) 10 D) 15 E) 20

EUREKA! N°34, 2011

11
Sociedade Brasileira de Matemática

8. O máximo divisor comum de todos os números que são o produto de cinco


ímpares positivos consecutivos é
A) 1 B) 3 C) 5 D) 15 E) 105

9. Veja o problema 17 do Nível 2.

10. Veja o problema 19 do Nível 1.

11. Esmeralda ia desenhar o gráfico de y = 2x + 6 mas trocou os eixos de lugar.


Como fica o desenho dessa relação com os eixos trocados de lugar?

12. Qual das seguintes frações é mais próxima de 7 ?


3 5 8 13 18
A) B) C) D) E)
1 2 3 5 7

13. No triângulo ABC, m(BÂC) = 140º. Sendo M o ponto médio de BC, N o ponto
médio de AB e P o ponto sobre o lado AC tal que MP é perpendicular a AC, qual é
a medida do ângulo NMP ˆ ?
A) 40º B) 50º C) 70º D) 90º E) 100º

14. Veja o problema 16 do Nível 2.

15. Veja o problema No. 20 do Nível 1.

16. Os números a e b são reais não negativos tais que a3 + a < b – b3. Então
A) b < a < 1 B) a = b = 1 C) a < 1 < b
D) a < b < 1 E) 1 < a < b

17. Quantos são os pares (x, y) de inteiros positivos tais que x2 – y2 = 22010?
A) 1000 B) 1001 C) 1002 D) 1003 E) 1004

EUREKA! N°34, 2011

12
Sociedade Brasileira de Matemática

18. Veja o problema No. 8 do Nível 1.

19. Seja ABC um triângulo e X, Y e Z pontos sobre os lados BC, CA, AB tais que
CX AY BZ
= = =2.
XB YC ZA
A

Z
Y

B X C

A razão entre as áreas do triângulo XYZ e do triângulo cujos lados são congruentes
às medianas de ABC é:
Obs.: as medianas de um triângulo são os segmentos que ligam os vértices do
triângulo aos pontos médios dos lados opostos.
2 1 4 1 1
A) B) C) D) E)
3 2 9 3 4

20. Para cada subconjunto A de {1;2;3;4;5;6;7;8;9;10}, seja p(A) o produto de seus


elementos. Por exemplo, p({1;2;4;5}) = 40 e p(A) = 10! = 1 ⋅ 2 ⋅ 3 ⋅…⋅ 10 . Por
convenção, adote p ( ∅ ) = 1. A soma de todos os 210 produtos p(A) é igual a:
A) 211 B) 11! C) 1111 D) 211! E) 112!

21. Sendo n = 20102010 e log n é igual ao número m tal que 10m = n, então
A) n! < nlog n < (log n)n
B) nlog n < n! < (log n)n
C) (log n)n < nlog n < n!
D) (log n)n < n! < nlog n
E) nlog n < (log n)n < n!

22. Quatro números inteiros positivos a < b < c < d são tais que o mdc entre
quaisquer dois deles é maior do que 1, mas mdc(a, b, c, d) = 1. Qual é o menor
valor possível para d?
A) 10 B) 12 C) 15 D) 30 E) 105

23. Qual é o maior valor de xy2 se x e y são reais positivos cuja soma é 3?
A) 3 B) 4 C) 5 D) 6 E) 7

EUREKA! N°34, 2011

13
Sociedade Brasileira de Matemática

24. Um ponto P é escolhido ao acaso no interior de um quadrado QRST. Qual é a


ˆ ser agudo?
probabilidade do ângulo RPQ

3 1 π π
A) B) 2 −1 C) D) E) 1 −
4 2 4 8

25. Qual é o menor valor positivo de 21m2 – n2 para m e n inteiros positivos?


A) 1 B) 2 C) 3 D) 5 E) 7

GABARITO
NÍVEL 1 (6º. ou 7º. Anos do Ensino Fundamental)
1) D 6) B 11) D 16) E
2) B 7) E 12) B 17) C
3) D 8) E 13) D 18) A
4) D 9) C 14) E 19) B
5) B 10) D 15) A 20) D

NÍVEL 2 (8º. ou 9º. Anos do Ensino Fundamental)


1) B 6) B 11) D 16) A 21) E
2) D 7) C 12) E 17) D 22) C
3) E 8) C 13) B 18) D 23) E
4) E 9) D 14) C 19) E 24) Anulada
5) C 10) D 15) A 20) B 25) D

NÍVEL 3 (Ensino Médio)


1) E 6) C 11) E 16) D 21) E
2) B 7) B 12) C 17) E 22) C
3) E 8) D 13) D 18) A 23) B
4) E 9) D 14) A 19) C 24) E
5) D 10) B 15) D 20) B 25) C

EUREKA! N°34, 2011

14
Sociedade Brasileira de Matemática

XXXII OLIMPÍADA BRASILEIRA DE MATEMÁTICA


Problemas e soluções da Segunda Fase

PROBLEMAS – NÍVEL 1 – PARTE A


(Cada problema vale 5 pontos)
1
01. Uma jarra contém de sua capacidade em água. Despejando um copo cheio de
4
água na jarra, o volume de água atinge 1 da sua capacidade. Quantos copos cheios
3
mais ainda serão necessários para acabar de encher a jarra?

02. Joãozinho tem que fazer uma multiplicação como lição de casa, mas a chuva
molhou o caderno dele, borrando alguns algarismos, que estão representados por
(cada algarismo borrado pode ser diferente dos outros).

1
× 2 3
4
4 2 +
0
1 0 0 2

Qual é a soma dos algarismos que foram borrados?

03. Soninha pintou as seis faces de um cubo da seguinte


maneira: uma face preta e a face oposta vermelha, uma face
amarela e a face oposta azul, uma face branca e a oposta
verde. Ao olhar para o cubo, de modo a ver três faces,
como na figura, e considerando apenas o conjunto das cores
das três faces visíveis, de quantas maneiras diferentes pode
ser visto esse cubo?

04. Esmeralda foi escrevendo os quadrados dos números inteiros positivos um em


seguida ao outro formando o número 149162536... e parou quando chegou no
centésimo algarismo. Qual foi o último algarismo que ela escreveu?

EUREKA! N°34, 2011

15
Sociedade Brasileira de Matemática

05. Carlinhos escreve números inteiros positivos diferentes e menores do que 1000
em várias bolas e coloca-as numa caixa, de modo que Mariazinha possa pegar ao
acaso duas dessas bolas. Quantas bolas no máximo Carlinhos irá colocar na caixa
se os números das duas bolas deverão ter um divisor comum maior do que 1?

06. Num concurso com 10 questões, cada resposta correta valia 3 pontos, cada
resposta errada valia 1 ponto negativo e cada questão não respondida valia 0 ponto.
Não houve dois candidatos que apresentassem a mesma nota, feitas as correções.
Quantos candidatos no máximo fizeram essa prova?

PROBLEMAS – NÍVEL 1 – PARTE B


(Cada problema vale 10 pontos)

PROBLEMA 1
Com cinco quadrados com lados de 27 cm, formamos uma sequência de figuras,
das quais as quatro primeiras são:

a) Na 4ª figura, qual é a área do quadrado cinza?


b) Na 5ª figura, qual é a área do quadrado cinza?

PROBLEMA 2
Maria tem 90 cartões. Ela numerou os cartões de 10 a 99 numa das faces e, para
cada número escrito, escreveu a soma dos seus algarismos na outra face. Por
exemplo, o cartão de número 43 tem o número 7 escrito no verso. Em quais cartões
um número de uma face é o dobro do número escrito na outra face?

EUREKA! N°34, 2011

16
Sociedade Brasileira de Matemática

PROBLEMA 3
Fazendo três cortes num quadrado 3 × 3 e
juntando as quatro partes resultantes a um
quadrado 4 × 4 , obtemos um quadrado
5 × 5 , conforme indicado na figura. Os
cortes devem ser paralelos aos lados dos
quadrados e não pode haver sobreposição
de figuras para a realização dos cortes.
a) Transforme um quadrado de lado 8 cm e um quadrado de lado 15 cm num único
quadrado de lado 17 cm, fazendo quatro cortes apenas no quadrado de 8 cm.

b) Qual é o menor número de cortes para transformar três quadrados, de áreas


respectivamente iguais a 4 cm2, 9 cm2 e 36 cm2, num único quadrado?

PROBLEMAS – NÍVEL 2 – PARTE A


(Cada problema vale 5 pontos)

01. Seja N o menor número inteiro positivo que multiplicado por 33 resulta em um
número cujos algarismos são todos iguais a 7. Determine a soma dos algarismos de
N.

02. Na figura seguinte, os triângulos ABC e ABD são retângulos em A e D,


respectivamente. Sabendo que AC = 15 cm, AD = 16 cm e BD = 12 cm, determine,
em cm2, a área do triângulo ABE.
C

D
E

A B
03. Sejam p, q números reais satisfazendo as relações 2p – 3p – 1 = 0, q2 + 3q – 2 =
2

pq + p + 1
0 e pq ≠ 1. Ache o valor de .
q

EUREKA! N°34, 2011

17
Sociedade Brasileira de Matemática

04. Em uma cidade arbitrária o prefeito organizou uma rifa com bilhetes numerados
de 100 a 999. O prêmio de cada bilhete é determinado pela soma dos algarismos do
número do bilhete. Para que ninguém leve três prêmios iguais, estabeleceu-se que
quem retirar três bilhetes com as três somas iguais tem direito a um superprêmio.
Qual é o menor número de bilhetes que um cidadão deve comprar para ter a certeza
de que vai receber um superprêmio?

05. Sejam r e s números inteiros. Sabe-se que a equação do segundo grau

x2 – (r + s)x + rs + 2010 = 0

tem as duas soluções inteiras. Quantos são os possíveis valores de |r – s|?

PROBLEMAS – NÍVEL 2 – PARTE B


(Cada problema vale 10 pontos)

PROBLEMA 1
Joãozinho deseja colorir um tabuleiro 2 × 2010 com duas cores A e B. Uma
coloração é dita legal se não é possível encontrar um L-triminó, como na figura
abaixo, com todos os seus quadradinhos de mesma cor. Determine o número de
colorações legais.

L – Triminó

Veja abaixo duas colorações que não são legais:

PROBLEMA 2
Determine todos os números primos m e n tais que 0 < m < n e os três números
2m + n, m + 2n e m + n – 18
sejam também primos.]

PROBLEMA 3
Chamaremos de imagem de um número natural de dois algarismos o número que se

EUREKA! N°34, 2011

18
Sociedade Brasileira de Matemática

obtém trocando a ordem de seus algarismos. Por exemplo, a imagem de 34 é 43.


Quais são os números de dois algarismos que somados com sua imagem resultam
em um quadrado perfeito?

PROBLEMA 4
As bissetrizes internas dos ângulos  e Ĉ do triângulo ABC cortam-se no ponto I.
Sabe-se que AI = BC e que m( ICˆ A) = 2m( IAˆ C ) . Determine a medida do ângulo
ABˆ C .

PROBLEMAS – NÍVEL 3 – PARTE A


(Cada problema vale 4 pontos)

01. Seja N o menor número inteiro positivo que multiplicado por 33 resulta em um
número cujos algarismos são todos iguais a 7. Determine a soma dos algarismos de
N.

02. Sejam r e s números inteiros. Sabe-se que a equação do segundo grau

x2 – (r + s)x + rs + 2010 = 0

tem as duas soluções inteiras. Quantos são os possíveis valores de |r – s|?

03. Na figura a seguir, as três circunferências em traço contínuo são tangentes às


retas r e s e a circunferência tracejada passa pelos pontos A, B, C e D. Além disso,
a circunferência menor é tangente também a AD e a circunferência maior é também
tangente a BC. Se os raios das circunferências externas ao quadrilátero ABCD são 8
e 18, calcule o raio R da circunferência inscrita em ABCD.

r
B
A 18

8 R

D
C s

EUREKA! N°34, 2011

19
Sociedade Brasileira de Matemática

04. Cada uma das oito casas de um retângulo de duas linhas e quatro colunas é
pintada de uma entre três cores. Uma coluna é chamada de corte se as suas duas
casas são da mesma cor. De quantas maneiras é possível pintar o retângulo de
modo que haja exatamente um corte?

05. Calcule
(2 4
)( )( ) (
+ 2 2 + 1 4 4 + 4 2 + 1 6 4 + 6 2 + 1 … 32 4 + 32 2 + 1 )
(1 4
)( )( ) (
+ 12 + 1 3 4 + 3 2 + 1 5 4 + 5 2 + 1 … 314 + 312 + 1 )

PROBLEMAS – NÍVEL 3 – PARTE B


(Cada problema vale 10 pontos)

PROBLEMA 1
As bissetrizes internas dos ângulos  e Ĉ do triângulo ABC cortam-se no ponto I.
Sabe-se que AI = BC e que m( ICˆ A) = 2m( IAˆ C ) . Determine a medida do ângulo
ABˆ C .

PROBLEMA 2
Diamantino gosta de jogar futebol, mas se jogar dois dias seguidos ele fica com
dores musculares. De quantas maneiras Diamantino pode escolher em quais de dez
dias seguidos ele vai jogar bola sem ter dores musculares? Uma maneira é não
jogar futebol em nenhum dos dias.

PROBLEMA 3
Resolva o sistema
 x + y + z = 77

 xy + yz + zx + xyz = 946

sendo x ≤ y ≤ z inteiros não negativos.

PROBLEMA 4
Uma mesa de bilhar tem o formato de um quadrado ABCD. SuperPablo tem uma
missão especial: ele deve dar uma tacada em uma bola de bilhar, inicialmente
colocada no vértice A, de modo que, após bater exatamente 2010 vezes nos lados
do quadrado, a bola chegue, pela primeira vez, a um vértice do quadrado.

EUREKA! N°34, 2011

20
Sociedade Brasileira de Matemática

Quantos são os possíveis valores do ângulo formado pelo lado AB com a trajetória
inicial da bola?

Observação: ao bater nos lados do quadrado, a bola sofre reflexão perfeita, ou seja,
o ângulo de incidência é igual ao ângulo de reflexão. Suponha também que a bola
seja um ponto.

α α

SOLUÇÕES NÍVEL 1 – SEGUNDA FASE – PARTE A

Problema 01 02 03 04 05 06
Resposta 8 60 8 9 499 38

1 1 4−3 1
01. Volume de um copo de água é igual a − = = do volume da jarra.
3 4 12 12
2
1 2 2
Falta encher 1 − = da jarra. Para isso são necessários 3 = × 12 = 8 copos.
3 3 1 3
12

02. Como o algarismo das unidades do produto é 2, o algarismo das unidades do


multiplicando é 4. Assim, obtemos o algarismo da direita da 3ª linha do algoritmo e
1 4 também os dois últimos 1 4
× 2 9 3 algarismos da 5ª linha, conforme × 2 3
4 2 figura à direita. Como o algarismo 4 2
4 2 6 + das dezenas do produto é 0, o 4 2 +
0 2 8 algarismo da direita na 4ª linha do 0 2 8
1 0 0 2 algoritmo deve ser 6. Logo o 1 0 0 2
algarismo das dezenas do
multiplicador é 9, conforme figura à esquerda. Como o 2º algarismo à direita 5ª
linha é 0, o algarismo das centenas

EUREKA! N°34, 2011

21
Sociedade Brasileira de Matemática

do multiplicando é 5. A partir do algoritmo completo, 5 1 4


mostrado à direita, concluímos que a soma dos algarismos × 2 9 3
que foram borrados é 1 5 4 2
4 6 2 6 +
5 + 4 + 9 + 1 + 5 + 2 + 6 + 6 + 1 + 2 + 8 + 5 + 6 = 60 1 0 2 8
1 5 0 6 0 2
03. Cada 3 faces que podem ser vistas ao mesmo tempo
compartilham um vértice. Como o cubo tem 8 vértices, o número de composições
de cores percebidas visualmente é 8.

04. Os números 12, 22, 32 possuem um algarismo. Os números 42, 52, ..., 92 possuem
dois algarismos. Os números 102, 112, ..., 312 possuem três algarismos. Assim, ao
escrever o quadrado do número 31, o número de algarismos escritos é
1 × 3 + 2 × 6 + 3 × 22 = 81 , faltando escrever 19 algarismos. Com os quadrados de
32, 33, 34 e 35, temos mais 4 × 4 = 16 algarismos, faltando ainda escrever apenas
três algarismos. Como o quadrado de 36 é 1296, concluímos que o último
algarismo escrito foi o 9, o centésimo algarismo escrito por Esmeralda.

05. Não podemos colocar o número 1 em nenhuma bola, pois o MDC entre 1 e
qualquer outro número é 1, assim temos 998 números disponíveis. Além disso, se
forem usadas 500 bolas ou mais, haverá duas com números consecutivos, sempre
primos entre si, então não podemos colocar mais que 499 bolas. Mas existe uma
forma de colocar 499 bolas, usando os números pares de 2 a 998.

06. Quem acerta a questões e erra b obtém 3a – b pontos, com a + b ≤ 10 . Obtemos


os números de 0 a –10 com a = 0, ao todo 11 inteiros. Obtemos os números de 1 a
30 usando os valores 0, 1 ou 2 para b, não obtendo apenas 3.9 – 2 = 25, 3.10 – 1 =
29 e 3.10 – 2 = 28, pois nesses casos ficamos com a + b > 10, ao todo 30 – 3 = 27
inteiros. Logo, o número máximo de candidatos nas condições apresentadas é 11 +
27 = 38.

SOLUÇÕES NÍVEL 1 – SEGUNDA FASE – PARTE B

SOLUÇÃO DO PROBLEMA 1:
Em cada figura, a área do quadrado cinza é uma fração da área do quadrado
original. Nas figuras apresentadas, a partir da segunda, as áreas são iguais,

EUREKA! N°34, 2011

22
Sociedade Brasileira de Matemática

4
× 27 × 27
9
4 4
respectivamente, a × × 27 × 27
9 9
4 4 4
× × × 27 × 27 = 64
9 9 9

a) Na 4ª figura, a área do quadrado cinza é igual a 64, segundo os produtos acima.

b) Na 5ª figura, admitindo que a obtenção do quadrado cinza seja feita da mesma


4
maneira, a sua área é igual a da área do quadrado cinza da 4ª figura, ou seja, é
9
4 256 2
igual a × 64 = cm .
9 9

SOLUÇÃO DO PROBLEMA 2:
Para um número cujo algarismo das dezenas é a e cujo algarismo das unidades é b,
temos 10a + b = 2 ( a + b ) ou a + b = 2 (10a + b ) . A segunda equação não tem
soluções positivas, e na primeira equação temos
10a + b = 2 ( a + b ) ⇔ 10a + b = 2a + 2b ⇔ 8a = b . Necessariamente temos a = 1 e b
= 8. De fato, no cartão de número 18 a soma dos algarismos é 9.

SOLUÇÃO DO PROBLEMA 3:
a) Bastam 4 cortes no quadrado de lado 8
cm, conforme ilustrado nos desenhos à
direita.
4ºcorte

1º, 2º e 3º
cortes

Ou ainda, como a figura a seguir.

EUREKA! N°34, 2011

23
Sociedade Brasileira de Matemática

4ºcorte

1º, 2º e 3º
cortes

b) Uma possibilidade (exemplo 1) é juntar ao quadrado maior pedaços dos


quadrados menores, obtendo-se um quadrado de área 4 + 9 + 36 = 49 cm2. Para
isso, dividimos o quadrado de lado 3 em três tiras 3 × 1 com dois cortes e o
quadrado de lado 2 em duas tiras 2 × 1 com um corte, num total de 3 cortes,
conforme desenho à esquerda. Menos que 3 cortes não formam peças que se
encaixam na região sombreada.

(exemplo 1)
Outras maneiras (exemplos 2 e 3) demontar o quadrado também com três cortes
são apresentadas ao lado.

(exemplo 2)

(exemplo 3)

EUREKA! N°34, 2011

24
Sociedade Brasileira de Matemática

SOLUÇÕES NÍVEL 2 – SEGUNDA FASE – PARTE A

Problema 01 02 03 04 05
Resposta 25 75 1 53 8

01. O critério de divisibilidade por 11 nos diz que se o número 33N possui todos os
seus algarismos iguais e é divisível por 11, então ele deve possuir um número par
da algarismos. O critério de divisibilidade por 3 também nos diz que a soma dos
algarismos deve ser múltipla de 3 e isso obriga que a quantidade de algarismos 7
seja divisível por 3. O menor número que cumpre essas condições é 777777, ou
seja, N = 777777/33 = 23569.

02. Pelo teorema de Pitágoras, temos que AB = AD 2 + BD 2 = 20 e que


CB = AC 2 + AB 2 = 25 . Os triângulos ABC e ADB são semelhantes pois os seus
lados são proporcionais e consequentemente EAB = EBA e
temos
ACB = 90 − EBA = 90 − EAB = CAE. . Concluímos assim que E é o ponto médio de
15 ⋅ 20
CB e a área procurada é metade da área do triângulo CAB, ou seja, = 75.
4

03. Como p não pode ser zero, podemos dividir a primeira equação por − p 2 e
1 1
obter 2
+ 3 − 2. Isto nos diz que as raízes da primeira equação são os inversos
p p
das raízes da segunda equação. Como pq ≠ 1. , p é igual ao inverso da outra raíz da
1
segunda equação que é diferente de q, ou seja, p = pois a soma das raízes
−3 − q
da segunda equação é igual a –3. Substituindo na expressão procurada:
pq + p + 1 − q − 1 + 3 + q 2 2
= = = =1
q 3q + q 2
3q + q 2
2

04. A soma dos dígitos dos bilhetes é no mínimo 1 e no máximo 27. Para as somas
1 e 27 existem apenas dois bilhetes, enquanto que para qualquer outro valor
existem pelo menos três bilhetes. Então retirando 1 + 1 + 2 × ( 27 − 2 ) + 1 = 53 iremos
escolher pelo menos três números com mesma soma.

EUREKA! N°34, 2011

25
Sociedade Brasileira de Matemática

05. Para que as soluções sejam inteiras, o discriminante da equação do segundo


grau deve ser o quadrado de um inteiro positivo, digamos t 2 . Assim
(r + s)
2
− 4rs − 4 × 2010 = t 2

( r − s ) − t 2 = 4 × 2010
2

(( r − s ) + t ) (( r − s ) − t )
× = 2010
2 2
Como os números ( ( r − s ) + t ) e ( ( r + s ) + t ) possuem a mesma paridade e 2010 é
inteiro, concluímos que os termos no produto anterior são inteiros. A cada para de
 2010 
divisores do tipo  d ,  do número 2010, temos uma solução para t e r − s
 d 
na última equação. Como 2010 possui 16 divisores, o número de soluções é 8.

SOLUÇÕES NÍVEL 2 – SEGUNDA FASE – PARTE B

SOLUÇÃO DO PROBLEMA 1:
A pintura da primeira coluna 2 x 1 do tabuleiro limita o número de maneiras de
pintarmos o restante do tabuleiro. Temos dois casos a considerar:

Primeiro caso: As casas desta coluna são pintadas com a mesma cor.
Necessariamente a próxima coluna terá ambas casa da cor oposta à aquela da
primeira coluna e. Pela mesma razão, teremos que as cores das colunas do tabuleiro
devem ser alternadas. Assim, neste caso, temos apenas 2 pinturas diferentes.

Figura 1

Segundo caso: As casas desta coluna são pintadas com cores diferentes.
Necessariamente a próxima coluna é igual à primeira ou tem as cores opostas. O
mesmo se passará com as próximas colunas. Como para cada coluna sempre
2010
teremos duas escolhas a fazer, incluindo a coluna inicial, temos 2 pinturas
diferentes.

EUREKA! N°34, 2011

26
Sociedade Brasileira de Matemática

Figura 2

O total de pinturas é: 2 + 22010

SOLUÇÃO DO PROBLEMA 2:
Como os primos 2m + n e , m + 2n são maiores que dois, temos que ambos são
ímpares e consequentemente 2m + n + m + 2n = 3m + 3n é um número par. Assim
m + n é par e m + n − 18 é um primo par, ou seja, dois. O único par de primos
(m, n) que cumpre m + n = 20 e satisfaz o enunciado é (m, n) = (3,17)

SOLUÇÃO DO PROBLEMA 3:
A soma de um número de dois algarismos com a sua imagem é da forma
(10a + b) + (10b + a) = 11( a + b ) , onde a e b são seus algarismos. Se 11( a + b ) é
um quadrado perfeito, devemos ter outro fator primo 11 na soma a + b. Além
disso, como a e b são menores que 10, concluímos que a + b é um múltiplo de 11
menor que 20 e maior que 0, ou seja, é igual à 11. Os pares de dígitos ( a, b ) que
verificam a + b = 11 são: ( 2,9 ) , ( 3,8 ) , ( 4, 7 ) , ( 5, 6 ) , ( 6,5 ) , ( 7, 4 ) , ( 8,3) e ( 9, 2 ) .

Portanto, existem 8 números de dois algarismos que cumprem o enunciado.

PRIMEIRA SOLUÇÃO DO PROBLEMA 4:

Seja N o ponto de encontro da bissetriz do ângulo ∠ACD com o lado AB . Pelo


caso A.L. A , os triângulos NCA e ADC são congruentes. Consequentemente
NC = AD = BC .
Pelo teorema do ângulo externo, ∠BNC = NAC + ACN = ∠NCB .
portanto BN = BC = NC e BNC é equilátero. Daí ∠ABC = 60, ∠BCA = 80 e
∠BAC = 40

SEGUNDA SOLUÇÃO DO PROBLEMA 4:

Lema 1: Se ABC é um triângulo com ∠ACB = 2∠BAC então a (a + b) = c 2 .

EUREKA! N°34, 2011

27
Sociedade Brasileira de Matemática

Lema 2: A bissetriz do vértice C do triângulo ABC tem comprimento


∠ACB
2ab cos
2
a+b

Sejam α = ∠BAD e P o ponto de encontro da bissetriz do ângulo ∠C com o


cb 2ab cos 2α
lado AB . Pelo segundo lema temos = e daí c = 2a cos 2α .
a+b a+b
Pelo lema 1 temos a (a + b) = c = 4a cos 2α e daí b = a (4 cos 2α − 1) .
2 2 2 2

Como AD = BC temos que p − a = AD cos α = a cos α e daí


a + b + c = 2a(cos α + 1) . Substituindo os valores encontrados anteriormente
temos
a + a (4 cos 2 2α − 1) + 2a cos 2α = 2a (cos α + 1) ⇒
1 + 2 cos 4α + 1 + 2 cos 2α = 2 cos α + 2 ⇒
cos 4α + cos 2α = cos α ⇒
1
cos α (1 − 2 cos 3α ) = 0 ⇒ cos 3α =
2
E consequentemente ∠ABC = 3α = 60 .

SOLUÇÕES NÍVEL 3 – SEGUNDA FASE – PARTE A

Problema 01 02 03 04 05
Resposta 25 8 12 2592 1057

01. Queremos o menor múltiplo de 33 formado apenas por algarismos 7. Teremos

33⋅N = 7777...77, com k algarismos 7.

Para ser múltiplo de 33, deve ser múltiplo de 11 e de 3. Assim, k deve ser par (pelo
critério de divisibilidade por 11) e, também, k deve ser múltiplo de 3, pois a soma
dos algarismos de 33N é 7k. Logo, o menor N procurado satisfaz 33⋅N = 777.777, o
que nos dá N = 23.569. A soma dos algarismos de N é 2 + 3 + 5 + 6 + 9 = 25.

02. A expressão x2 – (r + s)x + rs pode ser escrita como (x – r)(x – s). Logo,
devemos ter (r – x)(x – s) = 2010.

EUREKA! N°34, 2011

28
Sociedade Brasileira de Matemática

Fazendo r – x = a e x – s = b, a e b com o mesmo sinal, devemos encontrar a + b =


r – s sabendo que a e b são inteiros positivos tais que a⋅b = 2010. O número de
pares {a, b} que satisfazem esta equação é igual a oito, sendo {a, b} = {1, 2010},
{2, 1005}, {3, 670}, {5, 402}, {6, 335}, {10, 201}, {15, 134}, {30, 67}.

03. Seja O o ponto de interseção entre as retas AB e CD. Veja que os triângulos
ODA e OBC são semelhantes, pois ∠OAD = 180° − ∠DAB = ∠BCA. Logo,
podemos igualar a razão de semelhança à razão entre os raios das circunferências
inscritas, bem como das ex-inscritas, obtendo:
8 R
= ⇔ R 2 = 144 ⇔ R = 12.
R 18
04. Em primeiro lugar, escolhemos a coluna que conterá o corte. Isso pode ser feito
de 4 modos. Em seguida, escolhemos a cor das casas do corte, o que pode ser feito
de 3 modos. Ficamos, então, com três colunas restantes para preencher.
Preencheremos primeiramente as casas da primeira linha. Temos 3 modos de
colorirmos cada casa da primeira linha, ou seja, 33 modos. Finalmente, resta-nos
colorir as casas da segunda linha, o que pode ser feito de 23 modos, já que as cores
das casas dessas colunas devem ser diferentes das cores das casas imediatamente
superiores. O total de colorações é 4⋅3⋅33⋅23 = 2592.

05. Em primeiro lugar, veja que cada termo do produto é do tipo


(k + 1) 4 + (k + 1) 2 + 1
. Além disso, podemos escrever
k 4 + k 2 +1
x 4 + x 2 + 1 = ( x 4 + 2 x 2 + 1) − x 2 = ( x 2 + 1) 2 − x 2 = ( x 2 − x + 1)( x 2 + x + 1).
Assim, ficamos com
(k + 1) 4 + ( k + 1) 2 + 1 [(k + 1) 2 − (k + 1) + 1].[(k + 1) 2 + ( k + 1) + 1]
= . Agora, veja que
k 4 + k 2 +1 ( k 2 − k + 1)(k 2 + k + 1)
(k + 1)2 – (k + 1) + 1 = k2 + k + 1 e k2 – k + 1 = (k – 1)2 + (k – 1) + 1. Logo, a última
expressão fica
(k + 1) 4 + (k + 1) 2 + 1 (k + 1) 2 + ( k + 1) + 1
= .
k 4 + k 2 +1 ( k − 1) 2 + ( k − 1) + 1
Logo, o produto pedido é igual a
22 + 2 + 1 42 + 4 + 1 62 + 6 + 1 32 2 + 32 + 1
⋅ ⋅ ⋅ … ⋅ = 32 2 + 32 + 1 = 1057 .
02 + 0 + 1 22 + 2 + 1 42 + 4 + 1 30 2 + 30 + 1

EUREKA! N°34, 2011

29
Sociedade Brasileira de Matemática

SOLUÇÕES NÍVEL 3 – SEGUNDA FASE – PARTE B

PROBLEMA 1:

Seja α = m(IÂC). Então m( ICˆ A) = 2α . Prolongue a reta CI até encontrar o lado AB


em D. Como m(CÂD) = 2m(IÂC) = 2α, o triângulo ACD é isósceles e, portanto,
suas bissetrizes AI e CE são congruentes.

α
α 2α

α
α
A B
E D

Logo, sendo m(CÊB) = α + 2α = 3α = m( ECˆ B ) e CE = AI = BC, o triângulo BCE


é equilátero. Assim, m( ABˆ C ) = 60 .

Outra solução: Considere a mesma figura acima. Aplicando a lei dos senos nos
triângulos ACI e ABC, obtemos

AC AI AC sen 3α
= ⇔ =
sen(180 − α − 2α ) sen 2α AI sen 2α

AC BC AC sen 6α
e = ⇔ =
sen(180 − 2α − 4α ) sen 2α BC sen 2α

Como AI = BC e 0 < 3α < 6α < 180º,

sen 3α = sen 6α ⇔ 3α + 6α = 180 ⇔ α = 20 . Logo m( ABˆ C ) = 180 − 6α = 60 .

EUREKA! N°34, 2011

30
Sociedade Brasileira de Matemática

PROBLEMA 2:
Note que Diamantino pode jogar futebol no máximo 5 vezes; caso contrário ele
necessariamente joga dois dias seguidos. Suponha que ele joga k dias. Então os k
dias em que ele joga devem ser imediatamente seguidos por dias em que ele não
joga. Assim, acrescentando um dia ao período, podemos dividir os 11 dias em k
blocos de dois dias e 11 – 2k blocos de um dia. Podemos permutar os k + 11 – 2k =
(11 − k )! 11 − k 
11 – k blocos de =   maneiras.
k!(11 − 2k )!  k 

Assim, o total de maneiras de Diamantino escolher os dias em que vai jogar é


11 − 0  11 − 1 11 − 2  11 − 3 11 − 4  11 − 5 
  +   +   +   +   +   = 1 + 10 + 36 + 56 + 35 + 6 = 144
 0   1   2   3   4   5 
.
Outra solução:
Seja an o número de maneiras de Diamantino escolher os dias em que vai jogar
entre n dias. Se ele jogar no dia n ele não pode ter jogado no dia n – 1, mas não há
restrições aos demais n – 2 dias; assim, nesse caso há an–2 maneiras de escolher os
dias em que vai jogar; se ele não jogar no dia n não há restrições aos demais n – 1
dias, então nesse caso há an–1 maneiras de escolher os dias.

Assim, an = an–1 + an–2, com a0 = 1 (a única opção é não jogar) e a1 = 2 (ele joga ou
não no único dia). Dessa forma, podemos encontrar os valores de an a partir dos
anteriores:

n 0 1 2 3 4 5 6 7 8 9 10
an 1 2 3 5 8 13 21 34 55 89 144

Logo Diamantino pode escolher os dias de 144 maneiras.

Comentários:
• Temos que an = Fn+2, em que Fn é a famosa sequência de Fibonacci (clique
no link para saber algumas de suas muitas propriedades!)
• Comparando e generalizando as duas soluções você pode obter a
identidade
 n   n − 1  n − 2   n − 3 
Fn +1 =   +   +   +   +
0  1   2   3 
que soma as outras diagonais do triângulo de Pascal.

EUREKA! N°34, 2011

31
Sociedade Brasileira de Matemática

PROBLEMA 3:
Observando que (1 + x)(1 + y)(1 + z) = 1 + x + y + z + xy + yz + zx + xyz,

 x + y + z = 77  x + y + z = 77
 ⇔
 xy + yz + zx + xyz = 946 1 + x + y + z + xy + yz + zx + xyz = 1024
 (1 + x) + (1 + y ) + (1 + z ) = 80
⇔ 10
(1 + x)(1 + y )(1 + z ) = 1024 = 2
Como x, y e z são inteiros não negativos, 1 + x, 1 + y e 1 + z são potências de 2.

Considerando que 80 = 26 + 24 > 3⋅24, 80 < 27 e x ≤ y ≤ z, temos 24 < 1 + z < 27, ou


seja, 1 + z = 25 = 32 ou 1 + z = 26 = 64.
Se 1 + z = 32, temos 1 + x + 1 + y = 48 e (1 + x)(1 + y) = 25 = 32. Mas, sendo 1 + x
e 1 + y potências de 2 com soma par, temos 1 + x ≥ 2 e, portanto, 1 + y ≤ 16.

Então 1 + x ≤ 16 e 1 + x + 1 + y ≤ 32 < 48, e não há soluções nesse caso. Se 1 + z =


64, temos 1 + x + 1 + y = 16 e (1 + x)(1 + y) = 24 = 16.

Desse modo, 1 + x e 1 + y são soluções da equação do segundo grau t2 – 16t + 16 =


0, que não tem soluções inteiras. Logo não há soluções.

PROBLEMA 4:
Como a bola sofre reflexão perfeita, ao refletir a mesa em relação a cada lado em
que a bola bate obtém-se uma linha reta. Repetindo as reflexões obtemos a seguinte
figura, em que a trajetória da bola é reta:
A

Assim, o problema é equivalente a encontrar uma trajetória em um retângulo de


dimensões inteiras m e n, dividido em mn quadradinhos unitários, que começa em
um vértice, termina no vértice oposto e corte os lados dos quadradinhos unitários
2010 vezes, sem passar por nenhum dos vértices internos dos quadrados unitários
(pois se passasse, a bola chegaria a um vértice do quadrado antes de 2010 rebatidas
nos lados).

EUREKA! N°34, 2011

32
Sociedade Brasileira de Matemática

Como a bola deve atravessar m – 1 quadrados em um sentido e n – 1 no outro, m –


1 + n – 1 = 2010 ⇔ m + n = 2012; como a bola não passa por vértices do quadrado
unitário, mdc(m, n) = 1 ⇔ mdc(m, m + n) = 1 ⇔ mdc(m, 2012) = 1. Assim, o
número pedido é a quantidade de números coprimos com 2012 menores do que
 1  1 
2012, que é φ(2012) = φ(22 ⋅ 503) = 20121 − 1 −  = 1004 .
 2  503 

EUREKA! N°34, 2011

33
Sociedade Brasileira de Matemática

XXXII OLIMPÍADA BRASILEIRA DE MATEMÁTICA


Problemas e soluções da Terceira Fase

TERCEIRA FASE – NÍVEL 1

PROBLEMA 1
Esmeralda tem muitos triângulos retângulos iguais aos da figura.

5 cm
3 cm

4 cm

a) Fazendo coincidir partes dos lados, sem sobrepor triângulos, Esmeralda montou
a figura a seguir. Qual é a área e qual é o perímetro dessa figura?

b) Usando o mesmo processo, Esmeralda montou o menor quadrado possível com


lado de medida inteira. Mostre, através de uma figura, como Esmeralda pode fazer
isso.

PROBLEMA 2
As casas de um tabuleiro 3 × 3 são numeradas de 1 a 9, cada número sendo
utilizado exatamente uma vez. Em cada linha horizontal, pintamos de vermelho a
casa com o maior número e, de verde, a casa com o menor número. Seja A o menor
dos números das casas vermelhas e B o maior dos números das casas verdes.

EUREKA! N°34, 2011

34
Sociedade Brasileira de Matemática

a) Mostre uma maneira de preencher o tabuleiro de forma que A – B = 4.


b) Mostre uma maneira de preencher o tabuleiro de forma que A – B = –3.
c) É possível obter A = 4 e B = 3? Não se esqueça de justificar a sua resposta.

PROBLEMA 3
Dado um sólido formado por cubos de 1 cm de aresta, como mostra a figura 1,
podemos indicar a quantidade de cubos em cada direção, como mostra a figura 2.

3
3 2
2 1 1
1 0
2 1
1
0 1
3 0 0 2
1 1
3 0 1 3
3 2
2 3

Esmeraldino montou um sólido com cubos de 1 cm de aresta e fez uma figura


similar à figura 2.
1
3 c
a b f
1 e
2 2
d
2 3
2 2 1 1
x 3
1 2 3 2
2 2
1 m

Encontre os valores de a, b, c, d, e, f, x e m.

PROBLEMA 4
Dizemos que um número inteiro positivo n é abestado se ao lermos da direita para
esquerda obtivermos um inteiro maior que n. Por exemplo, 2009 é abestado porque
9002 é maior que 2009, por outro lado, 2010 não é abestado pois 0102, que é o

EUREKA! N°34, 2011

35
Sociedade Brasileira de Matemática

número 102, é menor que 2010 e 3443 não é abestado pois quando lido da direita
para esquerda é exatamente igual ao original. Quantos inteiros positivos de quatro
algarismos são abestados?

PROBLEMA 5
a) Exiba um número inteiro positivo menor ou igual a 1000 com pelo menos 20
divisores positivos.
b) Existe um número inteiro positivo menor ou igual a 11000 com pelo menos 200
divisores positivos? Não se esqueça de justificar a sua resposta.

TERCEIRA FASE – NÍVEL 2

PRIMEIRO DIA

PROBLEMA 1
Dizemos que um número inteiro positivo n é abestado se ao lermos da direita para
esquerda obtivermos um inteiro maior que n. Por exemplo, 2009 é abestado porque
9002 é maior que 2009, por outro lado, 2010 não é abestado pois 0102, que é o
número 102, é menor que 2010 e 3443 não é abestado pois quando lido da direita
para esquerda é exatamente igual ao original. Quantos inteiros positivos de quatro
algarismos são abestados?

PROBLEMA 2
Seja ABCD um paralelogramo e Γ a circunferência circunscrita ao triângulo ABD.
Se E e F são as interseções de Γ com as retas BC e CD respectivamente, prove que
o circuncentro do triângulo CEF está sobre Γ.

PROBLEMA 3
Arnaldo e Bernaldo participam do seguinte jogo em um tabuleiro m × n, m, n ≥ 2.
Arnaldo começa escolhendo uma casinha e colocando um cavalo na casinha
escolhida; em seguida, Bernaldo e Arnaldo movem alternadamente o cavalo,
começando por Bernaldo, com a restrição de que o cavalo não pode cair em
casinhas que já foram visitadas. Perde quem não poder mover o cavalo.
Determinar, em função de m e n, qual jogador tem uma estratégia para ganhar o
jogo, não importando os movimentos do outro jogador e mostrar como ele deve
jogar para ganhar.
Observação: Cada movimento de um cavalo consiste em ir duas casas na vertical
ou na horizontal e, em seguida, uma casa na direção perpendicular.

EUREKA! N°34, 2011

36
Sociedade Brasileira de Matemática

SEGUNDO DIA

PROBLEMA 4
Sejam a, b e c reais tais que a ≠ b e a2(b + c) = b2(c + a) = 2010. Calcule c2(a + b).

PROBLEMA 5
As diagonais de um quadrilátero inscritível ABCD se intersectam em O. Os
círculos circunscritos aos triângulos AOB e COD intersectam as retas BC e AD,
pela segunda vez, nos pontos M, N, P e Q. Prove que o quadrilátero MNPQ está
inscrito em um círculo de centro O.

PROBLEMA 6
Os três lados e a área de um triângulo são números inteiros. Qual é o menor valor
da área desse triângulo?

TERCEIRA FASE – NÍVEL 3

PRIMEIRO DIA

PROBLEMA 1
Encontre todas as funções f do conjunto dos reais nos conjuntos dos reais tais que

f(a + b) = f(ab)

para todos a, b irracionais.

PROBLEMA 2
Seja P(x) um polinômio com coeficientes reais. Prove que existem inteiros
positivos n e k tais que k tem n dígitos e mais de P(n) divisores positivos.

PROBLEMA 3
Qual é a maior sombra que um cubo sólido de aresta 1 pode ter, no sol a pino?
Observação: Entende-se “maior sombra de uma figura no sol a pino” como a maior
área possível para a projeção ortogonal da figura sobre um plano.

SEGUNDO DIA

PROBLEMA 4
Seja ABCD um quadrilátero convexo e M e N os pontos médios dos lados CD e
AD, respectivamente. As retas perpendiculares a AB passando por M e a BC

EUREKA! N°34, 2011

37
Sociedade Brasileira de Matemática

passando por N cortam-se no ponto P. Prove que P pertence à diagonal BD se, e


somente se, as diagonais AC e BD são perpendiculares.

PROBLEMA 5
Determine todos os valores de n para os quais existe um conjunto S de n pontos,
sem que haja três deles colineares, com a seguinte propriedade: é possível pintar
todos os pontos de S de modo que todos os ângulos determinados por três pontos
de S, todos da mesma cor ou de três cores diferentes, não sejam obtusos. A
quantidade de cores disponíveis é ilimitada.

PROBLEMA 6
Encontre todos os pares (a, b) de inteiros positivos tais que 3a = 2b2 + 1.

SOLUÇÕES DA TERCEIRA FASE – NÍVEL 1

PROBLEMA 1: SOLUÇÃO DE HELENA VERONIQUE RIOS (SÃO CARLOS – SP)


a) Perímetro

3 + 3 + 4 + 4 + 5 + 5 + ( 4 − 3) + ( 4 − 3) = 26.
O perímetro da figura é 26cm.

Área de um triângulo:
b⋅h 4 ⋅ 3 12
→ = = 6cm 2
2 2 2
Cada triângulo tem 6cm2 de área.
Se na figura temos 4 desses triângulos, a área da figura é 4 ⋅ 6, ou seja, 24cm2.

b) 6cm2 – área de cada triângulo


Qual o menor múltiplo de 6 que é um quadrado perfeito? 36 ( 6 ⋅ 6 )
O quadrado deverá ter 36cm 2 e 6cm de lado, se possível.
Este quadrado, porém, é impossível de ser formado por causa da forma do
triângulo. Teriam de ter dois lados medindo 3cm em cada lado do quadrado, o que
seria impossível já que precisariam de 8 lados de 3cm sendo que só tem 6.
( 4 + 3 ≠ 6; 4 + 5 ≠ 6; 5 + 3 ≠ 6 )
O próximo menor quadrado possível de ser feito com formas de 6cm 2 é o de lado
12, cuja área é 12 × 12 (144cm 2 ) .
36cm 2 − lado 6 ,

EUREKA! N°34, 2011

38
Sociedade Brasileira de Matemática

49cm 2 − lado 7 ,
64cm 2 − lado 8 ,
81cm 2 − lado 9 ,
100cm 2 − lado 10 ,
121cm 2 − lado 11 ,
144cm 2 − lado 12 ; 49, 64, 81, 100 e 121 não são divisíveis por 6 (área do
triângulo). 144 é divisível por 6 ( 6 ⋅ 24 = 144 ) .

Quadrado de lado 12cm, área 144cm2 com 24


triângulos retângulos de lados 3, 4 e 5 cm.

PROBLEMA 2: SOLUÇÃO DE LUCCA MORAIS DE ARRUDA SIAUDZIONIS (FORTALEZA – CE)

a)

A=7
Verde
5
Vermelho B=3
1 7 A–B=4

Verde Vermelho
2 4 8

Verde Vermelho
3 6 9

EUREKA! N°34, 2011

39
Sociedade Brasileira de Matemática

b)

A=3
Verde
2
Vermelho B=6
1 3 A – B = –3

Verde Vermelho
4 5 7

Verde Vermelho
6 8 9

c) Para a ser igual a 4 os dois números que estarão juntos com ele na fileira devem
ser (1, 2 ) ; ( 2 ,3) ou (1,3) . Porém o 3 não pode estar junto com ele na fileira, senão
ele não seria pintado de verde. Então uma fileira horizontal é: 1, 2, 4. Porém, para
que o 3 seja o B, as outras duas casas verdes teriam que ser 1 e 2. Porém, 1 e 2
estão na mesma fileira, então casas verdes são (1,3) , já que o 2 não é verde, a
terceira casa verde é um número ≥ 4, ocasionando o fato de B ≠ 3. Portanto, não é
possível.

PROBLEMA 3: SOLUÇÃO DE VICTÓRIA MOREIRA REIS COGO (TERESINA – PI)


Nesse cubo, podemos formar expressões a partir das placas 3 × 3 × 1 e a partir delas,
encontramos o resultado. Veja:

1 c
3 a+3+1=2+2+1 b 1+b+c =2+x+2
a 1
2 a=5–4 2 1+b+c =2+3+2
a=1 b + c = 7– 1
2 x
b+c=6
1 2

EUREKA! N°34, 2011

40
Sociedade Brasileira de Matemática

2+2+x=3+3+1 Como os resultados só podem ser de 0 a


x=7–4 3, e a única soma que dá 6 é 3 + 3, então:
x=3
2 1 b= 3 e c = 3
x 3
2 3

3
1+2+1=m+2+2 b 3+b+e =3+3+2
e
m=4–4 3+3+e =3+3+2
m=0 3
1 2 e =8–6
2 2 3 e=2
1 m
2

a 1
1 a+1+d=1+3+m c 1+c+f =2+1+2
d f
1+1+d=1+3+0 1+3+f =5
1 d=4–2 2
f =5–4
3 d=2 1 f =1
m 2

PROBLEMA 4: SOLUÇÃO DE LUCCA MORAIS DE ARRUDA SIAUDZIONIS (FORTALEZA – CE)


Chamamos o primeiro algarismo de A, o segundo de B o terceiro de C e o quarto de
D.
Testamos os casos:
1º. Caso: o último algarismo é maior que o primeiro.
Se A = 1, temos: 1 ⋅10 ⋅10 ⋅ 8 = 800
Se A = 2, temos: 1 ⋅ 10 ⋅10 ⋅ 7 = 700
Se A = 3, temos: 1 ⋅10 ⋅10 ⋅ 6 = 600
Se A = 4, temos: 1 ⋅10 ⋅10 ⋅ 5 = 500
Se A = 5, temos: 1 ⋅ 10 ⋅10 ⋅ 4 = 400
Se A = 6, temos: 1 ⋅10 ⋅10 ⋅ 3 = 300
Se A = 7, temos: 1 ⋅ 10 ⋅10 ⋅ 2 = 200
Se A = 8, temos: 1 ⋅ 10 ⋅ 10 ⋅1 = 100

Total de 3600 casos.

EUREKA! N°34, 2011

41
Sociedade Brasileira de Matemática

2º. Caso: A = D, C > B.


Se B = 0, Temos: 9 ⋅ 1 ⋅ 9 ⋅ 1 = 81
Se B = 1, Temos: 9 ⋅ 1 ⋅ 8 ⋅ 1 = 72
Se B = 2, Temos: 9 ⋅ 1 ⋅ 7 ⋅ 1 = 63
Se B = 3, Temos: 9 ⋅ 1 ⋅ 6 ⋅ 1 = 54
Se B = 4, Temos: 9 ⋅ 1 ⋅ 5 ⋅ 1 = 45
Se B = 5, Temos: 9 ⋅ 1 ⋅ 4 ⋅ 1 = 36
Se B = 6, Temos: 9 ⋅ 1 ⋅ 3 ⋅ 1 = 27
Se B = 7, Temos: 9 ⋅ 1 ⋅ 2 ⋅1 = 18
Se B = 8, Temos: 9 ⋅1 ⋅1 ⋅1 = 9

Total de 405 casos.


Resposta final: 4005 números.

PROBLEMA 5: SOLUÇÃO DA BANCA


a) Por exemplo, 900 = 22 ⋅ 32 ⋅ 52 , que tem ( 2 + 1) ⋅ ( 2 + 1) ⋅ ( 2 + 1) = 27 divisores
positivos.

b) Não, não existe. Seja n um número com pelo menos 200 divisores. Se o i-ésimo
n
menor divisor é d, então o i-ésimo maior divisor é . Seja m o centésimo menor
d
n
divisor. Temos m ≥ 100 e > m, donde n > m 2 ≥ 10000. Chegamos perto, mas
m
isso ainda não resolve o problema. Consideremos o 98º., o 99º. e o 100º. menores
divisores de n, que chamaremos de k, l, e m. Note que, se m ≥ 105, teremos como
n
antes > m, donde n > m 2 ≥ 1052 = 11025 > 11000.
m
Podemos supor então que 98 ≤ k < l < m ≤ 104. Como para quaisquer inteiros
positivos distintos a, b temos mdc ( a,b ) ≤ b − a , e
a ⋅b
mmc ( a,b ) = , concluímos que n ≥ mmc ( k ,l,m ) = mmc ( k ,mmc ( l,m ) ) =
mdc ( a,b )
k ⋅ mmc ( l,m ) k ⋅ mmc ( l,m )
= ≥ =
mdc ( k ,mmc ( l,m ) ) mdc ( k ,l ) ⋅ mdc ( k ,m )
klm klm 98 ⋅ 99 ⋅ 100
= ≥ ≥ .
mdc ( l,m ) mdc ( k ,l ) mdc ( k ,m ) ( m − l )( l − k )( m − k ) ( m − l )( l − k )( m − k )

EUREKA! N°34, 2011

42
Sociedade Brasileira de Matemática

Como ( m − l ) + ( l − k ) = m − k ≤ 104 − 98 = 6, temos ( m − l )( l − k ) ≤ 3 ⋅ 3 = 9 e


98 ⋅ 99 ⋅ 100
( m − l )( l − k )( m − k ) ≤ 9 ⋅ 6 = 54, donde n≥ > 11000.
54

SOLUÇÕES DA TERCEIRA FASE – NÍVEL 2

PROBLEMA 1: SOLUÇÃO DE LIARA GUINSBERG (SÃO PAULO – SP)


Considere o número da forma ABCD. Temos 3 possibilidades:
• A > D ⇒ o número não é abestado.
• A = D ⇒ o número é abestado somente se C > B : calculando o número de
casos temos:

A = D : 9 possibilidades, pois A = D ≠ 0 , senão o número teria somente 3


algarismos.
C > D : 45 possibilidades, pois é o resultado do somatório 9 + 8 + 7 + ... + 1, já que
C pode assumir o valor zero.

Totalizando 9 × 45 = 405 números abestados.


• A < D ⇒ o número é abestado, independentemente dos valores B e C.
Novamente calculando o número de casos:

A < D : 36 possibilidades, pois é o somatório de 8 + 7 + 6 + ... + 1, já que A não


pode assumir o valor zero.
B e C : 100 possibilidades, já que B pode assumir 10 valores diferentes, assim
como C.
Total: 36 × 100 = 3600 números abestados.
Finalizando, teremos 405 + 3600 = 4005 números abestados de quatro algarismos.

PROBLEMA 2: SOLUÇÃO DE PEDRO MORAIS DE ARRUDA SIAUDZIONIS (FORTALEZA – CE)


Seja B AD = α , logo BCD = α , pois em um paralelogramo os ângulos opostos são
iguais.
Seja BDA = β. Assim ABD = 180° − α − β. Veja que o arco AB = 2 BDA = 2β e
AD = 2 ABD ⇒ AD = 360° − 2α − 2β.

EUREKA! N°34, 2011

43
Sociedade Brasileira de Matemática

A B
α 180° – α – β

360° – 2α – 2β
E

β 2α
α
D F C

BAD − EF
Note que BCD = ⇒ 2α = 360° − 2α − EF ⇒ EF = 360° − 4α
2
Com isso EBAF = 4α .
Seja O o circuncentro do ∆FCE. Sabemos que FOE = 2 FCE ⇒ FOE = 2α .
EBAF EBAF
Como = 2α ⇒ FOE = ⇒ O ∈ Γ , pois FOE é ângulo inscrito.
2 2

PROBLEMA 3: SOLUÇÃO DA BANCA


Chamaremos Arnaldo de A e Bernaldo de B. Suponha, sem perda de generalidade,
m ≤ n. Vamos mostrar que para m = 2, A tem estratégia vencedora se e somente se
n não é múltiplo de 4; para m ≥ 3, A tem estratégia vencedora se e somente se m e n
são ímpares.

Estudemos o caso m = 2. Se 4 não divide n, A pode vencer colocando o cavalo na


primeira coluna se n = 4k + 1 e na segunda coluna se n = 4k + r, para r = 2 ou r = 3.
O cavalo deve ser movido sempre duas colunas à direita em cada jogada,
permitindo 2k jogadas a mais.

Considere agora o caso n = 4k. Divida o tabuleiro em tabuleiros 2 × 4, e forme


pares de casas de modo que é possível mover o cavalo entre casas do mesmo par:

EUREKA! N°34, 2011

44
Sociedade Brasileira de Matemática

1 2 3 4
3 4 1 2

Como todo o tabuleiro está dividido em pares, B consegue jogar, não importando
onde A coloque o cavalo: basta mover o cavalo para a outra casa do par. Quando A
jogar, colocará o cavalo numa casa de outro par, e B repete a estratégia. Deste
modo, se n = 4k o jogador B tem estratégia ganadora.

Isto termina o caso m = 2. O caso m ≥ 3 segue de modo semelhante, dividindo o


tabuleiro em vários tabuleiros menores. Considere as seguintes maneiras de se
formar pares, além da descrita anteriormente:

1 2 3 4 1 2 3 4 5 6 1 2 3 1 2 3 4 5
3 6 1 5 3 4 1 7 8 9 4 A 1 3 4 6 A 7
2 5 4 6 2 7 8 9 6 5 2 3 4 2 1 7 5 6

Juntando esses tabuleiros se prova que B tem estratégia vencedora para tabuleiros 3
× n, n par e A tem estratégia vencedora para tabuleiros 3 × n, n ímpar: divida o
tabuleiro num 3 × 3 ou 3 × 5 e tabuleiros 3 × 4; basta colocar o cavalo na casa
marcada com A e seguir a mesma estratégia de B.

Para verificar o caso m = 4, basta juntar tabuleiros 4 × 2 se n é par e um tabuleiro 4


× 3 e vários tabuleiros 4 × 2 se n é ímpar. Isso também prova que se A ou B tem
estratégia vencedora para um tabuleiro m × n então tem estratégia vencedora
também para tabuleiros (m + 4) × n, m ≥ 3. Então basta resolver o problema para m
∈ {3,4,5,6}.
Esses tabuleiros resolvem o caso m = 5:

1 2 3 4 5 6 1 2 3 4 5
3 4 1 7 8 9 3 4 5 6 7
2 10 11 9 6 5 2 1 10 8 9
12 13 14 15 7 8 11 A 12 7 6
10 11 12 13 14 15 12 10 11 9 8

EUREKA! N°34, 2011

45
Sociedade Brasileira de Matemática

Note que B tem estratégia vencedora para o tabuleiro 5 × 4 e A tem estratégia


vencedora para o tabuleiro 5 × 3. Então, para o caso 5 × n, n par, juntamos
tabuleiros 5 × 4 se n é múltiplo de 4 e um tabuleiro 5 × 6 e tabuleiros 5 × 4 se n é
na forma 4k + 2; se n é ímpar, juntamos vários tabuleiros 5 × 4 ao tabuleiro 5 × 3
ou 5 × 5, se n = 4k + 3 ou n = 4k + 1, respectivamente. O caso 6 × n segue
diretamente do caso 3 × n se n é par (basta juntar dois tabuleiros 3 × n) e juntando
tabuleiros 6 × 4 a um tabuleiro 6 × 3 ou 6 × 5, se n = 4k + 3 ou n = 4k + 1,
respectivamente. Com isso, todos os casos estão cobertos.

PROBLEMA 4: SOLUÇÃO DE PAULO HENRIQUE OMENA DE FREITAS (SÃO PAULO – SP)

Vamos trabalhar com a equação:


a 2 ( b + c ) = 2010
a≠b
b 2 ( c + a ) = 2010

Usando o método da subtração:


a 2 ( b + c ) − b2 ( c + a ) = 0
⇔ a 2b + a 2 c − b 2 c − b 2 a = 0
⇔ ab ( a − b ) + c ( a 2 − b 2 ) = 0
⇔ ab ( a − b ) + c ( a + b )( a − b ) = 0
⇔ ( a − b )( ab + ac + bc ) = 0
⇔ a = b ou ab + ac + bc = 0.
Já que, do enunciado, a ≠ b, ab + ac + bc = 0.
Colocando c em evidência:
c ( a + b ) = − ab
⇔ c 2 ( a + b ) = − abc .
Colocando a em evidência:
a ( b + c ) = −bc
⇔ a 2 ( b + c ) = − abc .
Assim, temos a igualdade:
c 2 ( a + b ) = − abc = a 2 ( b + c ) = 2010 .
Finalmente, c 2 ( a + b ) = 2010.

EUREKA! N°34, 2011

46
Sociedade Brasileira de Matemática

PROBLEMA 5: SOLUÇÃO DE FELLIPE SEBASTIAM S. P. PEREIRA (RIO DE JANEIRO – RJ)

A
B
D
θ

N
M
N
M
O
P Q

α α
θ
D
B C

Observe que como ABCD é um quadrilátero inscritível temos que os ângulos ACB
e BDA são iguais. Temos também que OQCDP é inscritível, logo, como ACB ˆ e
BDA são iguais, segue que os arcos OQ e OP são iguais. Podemos concluir que
os segmentos OQ e OP são iguais também. Analogamente, fazendo tudo com o
quadrilátero ANOMB, chegamos à conclusão que OM e ON são iguais. Agora
para provar que O é o centro da circunferência que passa pelos vértices do
quadrilátero MNQP (não sabemos ainda se ela existe), basta provarmos que
OP = ON , pois assim teremos OM = ON = OP = OQ, concluindo assim que existe
uma circunferência de centro O que passa pelos vértices do quadrilátero MNQP.
Para provarmos isto façamos o seguinte: chamemos o ângulo ACD de θ . Como o
quadrilátero ABCD é inscritível segue que DBA = θ . Temos também que
OPN = θ , pois OPD = 180° − θ (pois o quadrilátero OPDC é inscritível).
Temos também que PNO = θ , pois NOBA é inscritível. Logo os ângulos O NP e
OPN são iguais, donde segue que OM = OP , c.q.d.

PROBLEMA 6: SOLUÇÃO DE TADEU PIRES DE MATOS BELFORT NETO (FORTALEZA – CE)


Sabemos pela fórmula de Heron que a área de um triângulo é:
S= p ( p − a )( p − b )( p − c )

EUREKA! N°34, 2011

47
Sociedade Brasileira de Matemática

a+b+c
p=
2
 a + b + c  a + b + c  a + b + c  a + b + c 
S=   − a  − b  − c
 2  2  2  2 
 a + b + c  b + c − a  a + c − b  a + b − c 
S=     
 2  2  2  2 
16S 2 = ( a + b + c )( b + c − a )( a + c − b )( a + b − c )
sabemos que a,b,c , S ∈ + e a,b e c são lados de um triângulo.
Podemos ver que a+b+c é par. Caso contrário
a + b + c,( a + b + c ) − 2a,( a + b + c ) − 2c e ( a + b + c ) − 2b seriam ímpares e
teriam o produto par, o que é claramente um absurdo.
O valor mínimo para essa soma é 4, mas no caso a única tripla de interiros
positivos que têm essa soma é (1,1, 2 ) . Mas desobedecem a desigualdade triangular:
1 + 1 não é maior que 2.
Essa soma também não pode ser 6. Porque nesse caso o produto teria um fator 3,
mas como é um quadrado perfeito teria que ter dois ou mais fatores 3, Assim,
( b + c − a ) ,( a + c − b ) ou ( a + b − c ) teriam esse fator 3. Mas eles são pares e
menores que 6, logo não há como isso acontecer (absurdo!).
Se a + b + c = 8, nenhum dos outros fatores poderia ser 6, caso contrário teria que
haver dois fatores iguais a 6. Sem perda de generalidade supomos que
b+c−a=6
a+c−b = 6
2c = 12 ⇒ c = 6 → a + b = 2 → c > a + b. Absurdo.
Assim, temos alguns casos a analisar, pois cada fator só pode ser 2 ou 4.
a+b−c = 4
I) a+c−b = 4 a + b + c = 12 . Absurdo!
b+c−a =4
a+b−c = 4
II) a +c−b = 4 a + b + c = 10 . Absurdo!
b+c−a =2

EUREKA! N°34, 2011

48
Sociedade Brasileira de Matemática

a+b+c =8
a+b−c = 4
III) ( a + b + c )( a + b − c )( a + c − b )( b + c − a ) = 128 .
a +c−b = 2
b+c−a =2
+
128 não é da forma 16S2 com S ∈ , Absurdo!

a+b−c = 2
IV) a +c−b = 2 a + b + c = 6 . Absurdo!
b+c−a =2
Podemos ver que a soma não pode ser 10, por um argumento análogo ao do 6. Pois
teria que haver outro fator 5, o que faria com que um dos fatores fosse 10, o que é
um absurdo!
Já que para a + b + c = 12 é possível, basta tornar a= 5, b= 4, c = 3.
Vale a desigualdade triangular e a área é dada por
S= p ( p − a )( p − b )( p − c )
p = 6 , a = 5, b = 4 , c = 3
S = 6 ⋅1 ⋅ 2 ⋅ 3
S = 62 = 6.
Vamos provar que o valor mínimo de ( p − a )( p − b )( p − c ) é 6, caso o produto
p ( p − a )( p − b )( p − c ) fosse menor que 36.
Supondo que não fosse, teríamos as seguintes possibilidades:
b+c−a a+b−c a+c−b
( p − a )( p − b )( p − c ) = 5 ⇒ = 5, = 1, =1⇒
2 2 2
p = 5 + 1 + 1 = 7 e p ( p − a )( p − b )( p − c ) = 7 ⋅ 5 = 35, que não é quadrado perfeito.
Absurdo!
( p − a )( p − b )( p − c ) = 4 . Para o produto disso com p ser menor que 36 e
quadrado perfeito, p seria 4 e já vimos anteriormente que isso é um absurdo.
( p − a )( p − b )( p − c ) = 3 . Nesse caso p seria 3, para o produto ser menor que 36 e
quadrado perfeito, mas nesse caso a + b + c = 6 e já analisamos esse caso.
a+b−c a+c−b b+c−a
( p − a )( p − b )( p − c ) = 2 =1 =1 =2
2 2 2
p = 1 + 1 + 2 = 4 , S = 8 . Absurdo!

EUREKA! N°34, 2011

49
Sociedade Brasileira de Matemática

Logo os valores de ( p − a )( p − b )( p − c ) e p são mínimos e portanto o produto é


mínimo. Assim, a área mínima é 6.

SOLUÇÕES DA TERCEIRA FASE – NÍVEL 3

PROBLEMA 1: SOLUÇÃO DE DAVI COELHO AMORIM (FORTALEZA – CE)


Temos que f : → e f ( a + b) = f ( a ⋅ b) (1)
,∀a,b irracionais.
Logo, temos:
f (a + b) = f (a ⋅ b) = f ( ( − a ) ⋅ ( −b ) ) = f ( − a − b ) ⇒ f ( a + b ) = f ( − a − b ) .
Lema: Todo número real pode ser representado como a soma de dois números
irracionais.
1º. Caso: Número irracional.
Seja x um racional e α um irracional. Logo, sendo β = x − α , β é um irracional,
pois se β for racional, α = x − β e α seria racional. Absurdo!
Logo, como x = α + ( x − α ) = α + β , todo racional pode ser escrito como a soma de
dois irracionais.
2º. Caso: Número irracional
Seja x esse irracional. Vamos supor que para todo 0 < α < 1 irracional, x − α = β ,
onde β é racional. Logo temos:
 x − α = ϕ p r
 , onde ϕ = e θ = , p,q,r,s ∈
 x − (1 − α ) = θ q s
p r ps + rq + sq
Somando obtemos 2 x − 1 = + ⇒ x = ⇒ x é racional. Absurdo!
q s 2qs
Logo, todo irracional pode ser escrito como a soma de dois irracionais.
Com o lema provado, temos que f ( x ) = f ( − x ) ,∀x ∈ (2)
Em (1), fazendo a = −b, temos:
( 2)
f ( 0 ) = f ( −b 2 ) = f ( b 2 ) . Seja f ( 0 ) = k ⇒ f ( b 2 ) = k ,∀b irracional.

Logo, provamos que ∀x ∈ +


tal que x ∉ , f ( x ) = k. Basta provarmos agora
que os números y ∈ +
tais que y∈ também satisfazem f ( y ) = k. Porém,
isso não é difícil de provar: seja y tal que y∈ . Temos um 0 < θ < 1 irracional
tal que y = ( y − θ ) + θ ⇒ f ( y ) = f ( ( y − θ ) ⋅ θ ) e também
EUREKA! N°34, 2011

50
Sociedade Brasileira de Matemática

y = ( y + θ ) + ( −θ ) ⇒ f ( y ) = f ( ( y + θ )( −θ ) ) . Como, para todo x irracional,


f ( x ) = r, vamos provar que um dos números ( y − θ ) ⋅ θ e ( y + θ ) ⋅ ( −θ ) é
irracional, fazendo assim com que todo número tenha imagem r. Vamos supor o
contrário, ou seja, que os dois são racionais:
 p  p
( y − θ ) θ = q  yθ − θ = q
2

 ⇒
( y + θ )( −θ ) = r − yθ − θ2 = r , onde p,q,r,s ∈
 s  s
p r
2 yθ = −
q s
t
Como y∈ ⇒ y∈ ⇒ y=
, onde t,u ∈
u
2t ps − rq psu − rqu
⇒ ⋅θ = ⇒θ= ⇒ θ é racional. Absurdo!
u qs 2qst
Com isso, provamos que f ( x ) = k ,∀x ∈ , onde k é uma constante qualquer. Para
qualquer k, essa função serve, de acordo com a questão, pois
f ( a + b ) = f ( a ⋅ b ) ⇔ k = k . OK!

PROBLEMA 2: SOLUÇÃO DE CARLOS HENRIQUE DE ANDRADE SILVA (FORTALEZA – CE)


Seja d = grau do polinômio p.
Agora vamos numerar os primos em ordem crescente, Logo
P1 = 2 , P2 = 3, P3 = 5, P4 = 7 , ... Então seja b inteiro positivo tal que

(
10b −1 < P2 d + 2 < 10b . Vamos definir k como sendo k = P1a ⋅ P2a ...P2ad + 2 < (10b ) ).
2d +2 a

Logo n ≤ ab ⋅ ( 2d + 2 ) . Podemos supor que para x suficientemente grande


p ( x + 1) > p ( x ) . Se isso não ocorre então p é constante ou lim p ( n ) = −∞ , e então
n →∞

claramente p ( n ) < d ( k ) se a é suficientemente grande, onde d ( k ) = número de


divisores positivos de k.
Então basta provarmos que d ( k ) > p ( ab ( 2d + 2 ) ) para a suficientemente grande
já que teremos p ( ab ( 2d + 2 ) ) ≥ p ( n ) .
Então vamos às contas: d ( k ) = ( a + 1)
2d +2
; e como o polinômio tem grau d então
p ( x) < x d +1
para x suficientemente grande. Como no nosso problema

EUREKA! N°34, 2011

51
Sociedade Brasileira de Matemática

x = ab ⋅ ( 2d + 2 ) com “a” variável então nosso “x” pode ser tão grande quanto
quisermos. Então basta provar que:
> ( ab ⋅ ( 2d + 2 ) )
d +1
( a + 1) > p ( n ) ↔ ( a + 1) > ab ( 2d + 2 ) . Como b, d são
2d +2 2

constantes então a única variável é “a” e como no lado direito ”a” tem menor grau,
então para “a” suficientemente grande a desigualdade é válida, mostrando que
existem k, n que satisfazem a condição do enunciado.

PROBLEMA 3: SOLUÇÃO DA BANCA


Sejam ABCD e EFGH duas faces opostas, AE, BF, CG e DH sendo lados do cubo.
Denotaremos por X´ a projeção ortogonal do ponto X no plano. Note que
{ A,G} ,{B,H } ,{C ,E} e {D,F } são pares de vértices opostos. Suponha, sem perda
de generalidade, que A´ pertence à fronteira da projeção do cubo. Então,
considerando a simetria do cubo em relação ao seu centro, o simétrico G´ de A´
também pertence à fronteira. Dois dos três vértices vizinhos de A serão projetados
em vértices vizinhos de A´ na fronteira (a menos que, digamos, a face AEHD seja
projetada em um segmento, mas nesse caso podemos considerar um vértice
degenerado nesse segmento).
Suponha, sem perda de generalidade que esses vizinhos são B´ e D´ . Então E´ é
interior à projeção. Novamente pela simetria, H´ e F´ pertencem à fronteira da
projeção e C´ pertence ao interior da projeção. Finalmente, como
AE = BF = CG = DH , a projeção do cubo é A´ D´ H´G´ F´ B´.

As faces ABCD, BCGF e CDHG são projetadas sobre paralelogramos (ou


segmentos) A´ B´C´ D´, B´C´G´ F´ e C´ D´ H´G´. Trace diagonais
B´ D´,B´G´ e D´G´. A área da projeção é portanto o dobro da área do triângulo
BDG. Esse triângulo é equilátero de lado 2 , e logo o máximo desejado é

EUREKA! N°34, 2011

52
Sociedade Brasileira de Matemática

( 2)
2
⋅ 3
2⋅ = 3 . Uma projeção ortogonal num plano paralelo ao plano BDG
4
realiza a igualdade.

PROBLEMA 4: SOLUÇÃO DE GUSTAVO LISBÔA EMPINOTTI (FLORIANÓPOLIS – SC)


Na verdade a recíproca é verdadeira, mas a implicação direta nem sempre vale,
como veremos a seguir.
( ⇐ ) Suponha AC ⊥ BD.
Então podemos tornar um sistema de coordenadas em que AC é o eixo x e BD o
eixo y. Sejam A = (a, 0), B = (0, b), C = (c, 0) e D = (0, d). Claramente abcd ≠ 0.
a c d
O coeficiente angular de PM é (pois AC ⊥ BD). Como M =  ,  , a equação
b 2 2
d a c
da reta PM é y − =  x −  . Analogamente, a equação de PN é
2 b 2
d c a
y− =  x −  . Como P ( x p , y p ) = PM ∩ PN , temos
2 b 2
 c  a
a  xp −  = c  xp −  → xp (c − a ) = 0 e a ≠ c
 2   2
→ x p = 0 → P ∈ BD (eixo y).

(⇒) Suponha P ∈ BD.


Então podemos tomar um sistema de coordenadas em que
P = ( 0 ,0 ) , B = ( 0 ,b ) , D = ( 0,d ) . Sejam A = ( xa , ya ) e C = ( xc , yc )
 x y +d   x y +d 
→M = c , c  , N = a , a .
 2 2   2 2 
y +d y −b
O coeficiente angular de PM é c (claramente xc ≠ 0 ) e o de AB, a
xc xa
(também temos xa ≠ 0 ).
 y + d   ya − b 
PM ⊥ AB ⇒  c ⋅  = −1 → ( yc + d )( ya − b ) = − xa ⋅ xc (*)
 xc   xa 
Analogamente, PN ⊥ BC → ( ya + d )( yc − b ) = − xa ⋅ xc

EUREKA! N°34, 2011

53
Sociedade Brasileira de Matemática

→ ( ya + d )( yc − b) = ( yc + d )( ya − b) ↔ ya ⋅ yc − b ⋅ ya + d ⋅ yc − bd = ya ⋅ yc − byc + d ⋅ ya − bd
↔ ya ( b + d ) = yc ( b + d ) ↔ ya = yc ou b + d = 0.
Se ya = yc , AC é paralelo ao eixo x e, portanto, perpendicular a BD, que é o eixo y.
 y − b   ya − b  yc − b
Se b + d = 0, (*) vira  c ⋅  = −1, o que implica BC ⊥ AB (pois
 xc   xa  xc
ya − b
e são, respectivamente, os coeficientes angulares de BC e AB), ou seja,
xa
∠ABC = 90°. Isso caracteriza todos os contra exemplos:

A partir de um triângulo ∆PNM retângulo em P, tome um ponto D dentro do


ângulo ∠NPM mas fora do triângulo ∆NPM, e tal que ∠( DP,NM ) ≠ 90°. Aplique
uma homotetia de centro em D e razão 2. Fazemos N´ = A, M´ = C e P´ = B. Temos
um quadrilátero ABCD convexo no qual M e o ponto médio de CD; N , o de AD; e
P, o de BD. Como PN // AB e PM ⊥ PN, temos PM ⊥ AB e da mesma forma
PN ⊥ BC. ∠ABC = ∠NPM = 90°. Esse é o contraexemplo, pois pela escolha de
D, P ∈ BD mas BD não é perpendicular a AC.

P´= B

N´= A P M´= C

N M

PROBLEMA 5: SOLUÇÃO DE ANDRÉ SARAIVA NOBRE DOS SANTOS (FORTALEZA – CE)


Se tivermos 4 pontos, todos da mesma cor ou todos de cores diferentes, teremos o
seguinte:

1º. Caso: Eles formam um quadrilátero convexo:

EUREKA! N°34, 2011

54
Sociedade Brasileira de Matemática

a + b+ c + d = 360° logo, se eles forem diferentes,


a
haverá um deles maior que 90° (pois se todos forem
menores, a soma não daria 360°), logo, todos têm que
b d
ser de 90°, ou seja, eles têm que formar um retângulo.

2º. Caso: Eles formam um quadrilátero côncavo:

Como a + b > 180°, garanto que um deles é maior que


a 90°, logo, isso não pode acontecer.

Então, como 4 pontos de cores diferentes têm que formar um retângulo, não temos
mais como ter 5 pontos de 5 cores diferentes nesse conjunto. Sendo assim,
podemos ter 1, 2, 3 ou 4 cores:

1 cor: só podemos ter até 4 pontos, pois 3 deles só definem a posição do próximo,
só que os 4 têm que formar um retângulo:
ex:
A A

A A

2 cores: podemos ter até 8 pontos, de cada cor 4:


ex:
A A

V V

A A

V V

EUREKA! N°34, 2011

55
Sociedade Brasileira de Matemática

3 cores: conseguimos uma configuração com até 12 pontos, basta eles ficarem
muito afastados, cada retângulo acutângulo, pois assim, ao escolhermos 3 cores
diferentes, os ângulos do triângulo vão ser aproximadamente os do triângulo
acutângulo:
ex:
A A

A A

60°

V V 60° 60° L L

V V L L

4 cores: já mostramos que só conseguimos 4 pontos: de fato, 3 pontos de cores


diferentes determinam uma única posição possível para os pontos da outra cor, e
logo há apenas um ponto de cada cor.
Logo, como checamos todos os casos, vimos que não conseguimos mais de 12
pontos e achamos um exemplo com 12, qualquer n menor também satisfaz, pois
tirar pontos de uma configuração faz com que a restante também satisfaça.
Logo, n de 0 até 12 é solução, não existindo mais nenhuma.

PROBLEMA 6: SOLUÇÃO DA BANCA

As soluções são (1,1), (2,2) e (5,11).

Se a é par e maior que 2, a equação equivale a (3a/2 – 1) ⋅ (3a/2 + 1) = 2b2. Porém


mcd(3a/2 – 1, 3a/2 + 1) = mcd(3a/2 – 1, 2) = 2 e se conclui que 3a/2 + 1 = 4u2 e 3a/2 – 1
= 2v2 ou 3a/2 + 1 = 2u2 e 3a/2 – 1 = 4v2.

No primeiro caso, 3a/2 = (2v – 1)(2v + 1), e como mcd(2v – 1, 2v + 1) = mcd(2v – 1,


2) = 1, 2v – 1 = 1⇔ v = 1 e a/2 = 1 ⇔ a = 2 e portanto b = 2.

EUREKA! N°34, 2011

56
Sociedade Brasileira de Matemática

No segundo caso, 3a/2 = 4v2 + 1 ⇒ 0 ≡ v2 + 1 (mod 3) ⇔ v2 ≡ –1 (mod 3), o que é


impossível.

Se a é ímpar, a equação é equivalente a 3 ⋅ (3(a–1)/2)2 – 2b2 = 1. Seja c = 3(a–1)/2.


Encontremos as soluções de 3c2 – 2b2 = 1 (*).
Como ( 3+ 2 )( 3 − 2 =1⇒ ) ( 3+ 2 ) (
2 k +1
3− 2 )2 k +1
=1
e
( 3+ 2 )2 k +1
= ck 3 + bk 2 y ( 3− 2 )2 k +1
= ck 3 − bk 2 (**)
(ck, bk) são soluções de (*), para k ≥ 0 inteiro. Suponha que existe uma solução
(α, β) distinta com α , β ∈ . Então existe k ∈ tal que
( 3+ 2 )
2 k −1
<α 3 + β 2 < ( 3+ 2 )
2 k +1

α 3+β 2
⇔ 3+ 2< < 9 3 + 11 2
(5 + 2 6 ) k −1

2 < (α 3 + β 2 )(5 − 2 6 )
k −1
⇔ 3+ < 9 3 + 11 2

É possível provar por indução que α 3 + β 2 5 − 2 6 ( )( ) k −1


=θ 3 +φ 2 ,
θ e φ ambos inteiros satisfazendo 3θ 2 − 2φ 2 = 1 . Além disso,
1
θ 3 +φ 2 >1> =θ 3 −φ 2 > 0, e portanto θ 3 >φ 2 > 0.
θ 3 +φ 2
Portanto (θ , φ ) é solução de (*), com 1 < θ < 9. Porém é possível verificar
(testando) que não há soluções com 1 < θ < 9, contradição.

( )
2 k +1
Para k = 0 temos 3+ 2 = 3 + 2 , o que nos dá a solução ( c,b ) = (1,1) de

( )
2 k +1
(*), e, para k = 1, temos 3+ 2 = 9 3 + 11 2 , o que nos dá a solução

(c, b) = (9, 11) de (*).

Suponha que c > 9, ou seja, k > 1. Por (**) e pelo teorema do binômio temos
k
 2k + 1  m k − m  2k + 1

c = 
m=0  2 m + 1
 ⋅ 3 ⋅ 2

= ( 2k + 1) ⋅ 2 k + 
 3
 ⋅ 3 ⋅ 2 k −1 +

(***)

EUREKA! N°34, 2011

57
Sociedade Brasileira de Matemática

Logo 3 divide 2k + 1. Seja 3t a maior potência de 3 que divide 2k + 1. Como


 2k + 1
  ⋅ 3 ⋅ 2 k −1 = ( 2k + 1) ⋅ k ⋅ ( 2k − 1) ⋅ 2 k −1 , a maior potência de 3 que divide o
 3 
segundo termo de (***) é também 3t . Para m > 1, sendo 3s a maior potência de 3
 2k + 1  m k − m 2k + 1  2k  m k − m
que divide 2m + 1, o m-ésimo termo,   ⋅ 3 ⋅ 2 =  ⋅3 ⋅2
 2m + 1 2m + 1  2m 
tem pelo menos t + m – s fatores 3 (t de 2k + 1, m de 3m, subtraindo s de 2m + 1).

Temos m − s ≥ 2 para todo m > 1. De fato, para m = 2 e m = 3 temos s = 0 e, para


m ≥ 4 ,m − s ≥ m − log 3 ( 2m + 1)  ≥ 2 (isso segue da desigualdade
3r − 1
≥ r + 2 ,∀r ≥ 2 , que pode ser facilmente provada por indução).
2

Então, como k > 1, todos os termos a partir do terceiro tem pelo menos t + 2 fatores
3, e
c = (2k + 1) ⋅ 2 k + (2k + 1) ⋅ k ⋅ (2k − 1) ⋅ 2 k −1 + 3t + 2 ⋅ N
⇔ 3( a −1) / 2 = (2k + 1) ⋅ 2 k −1 ⋅ [(2k + 1)(k − 1) + 3] + 3t + 2 ⋅ N

Note que como 3 divide 2k + 1 então também divide (2k + 1)(k – 1) + 3; além
disso, 9 divide (2k + 1)(k – 1), e portanto a maior potência de 3 que divide
(2k + 1)(k – 1) + 3 é 3. Portanto a maior potência de 3 que divide
( 2 k + 1) ⋅ 2 k −1 ⋅ [( 2 k + 1)( k − 1) + 3 ] é 3t+1 (t fatores de 2k + 1 e 1 de
(2k + 1)(k – 1) + 3). Finalmente, 3(a–1)/2 = c = 3t+1 (1 + 3N), que não é possível pois
N > 0.

Dessa forma, não há mais soluções.

EUREKA! N°34, 2011

58
Sociedade Brasileira de Matemática

XXXII OLIMPÍADA BRASILEIRA DE MATEMÁTICA


Problemas e soluções da Primeira Fase – Nível Universitário

PROBLEMA 1:
Há muito tempo em uma galáxia muito distante, utilizavam-se como referência
para viagens espaciais os pontos A, B, C, D, E, F, G, H, vértices de um cubo de
ares igual a um ano-luz tendo os quadrados ABCD e EFGH como faces e tendo os
segmentos AE, BF, CG e DH como arestas. Uma nave espacial viaja com
velocidade constante em trajetória retilínea de B para C. Outra nave viaja com
velocidade constante igual ao triplo da velocidade da primeira, em trajetória
retilínea de A para G. Sabendo que a primeira atinge o ponto C no mesmo instante
em que a segunda atinge o ponto G, determine a menor distãncia entre as naves
durante esse deslocamento.

PROBLEMA 2:
Quantos são os pares ordenados (x, y) com x, y ∈ {0 ,1,2 ,...,142} tais que
5 x 2 + 7 y 2 − 1 é múltiplo de 143?

PROBLEMA 3:
Dados os polinômios com coeficientes complexos em uma variável f ( x ) e h ( x ) ,
prove que existe um polinômio g ( x ) tal que f ( x ) = g ( h ( x ) ) se, e somente se,
existe um polinômio com coeficientes complexos em duas variáveis q ( x, y ) tal
que f ( x ) − f ( y ) = q ( x, y ) ( h ( x ) − h ( y ) ) .

PROBLEMA 4:
Seja n um inteiro positivo.
Seja An o subconjunto do plano definido por 1 ≤ x ≤ n,0 ≤ y ≤ ln ( x ) . Seja Bn o
polígono convexo de vértices (1,0) = (1,ln (1) ) ,( 2,ln ( 2) ) ,( 3,ln ( 3) ) ,...,( n,ln ( n) ) ,( n,0) .
Seja Cn = An − Bn , o complemento de Bn em relação a An .

a) Calcule as áeas de An ,Bn e Cn . Simplifique sua resposta.


b) Mostre que a área de Cn é menor que 1, para qualquer inteiro positivo n.

Obs: ln representa o logaritmo na base e.

EUREKA! N°34, 2011

59
Sociedade Brasileira de Matemática

PROBLEMA 5:
Suponha que temos um grafo com n + 1 ≥ 4 vértices e queremos pintar suas arestas
com duas cores de forma que não haja duas arestas disjuntas da mesma cor. Mostre
que há no máximo 2n tais colorações.

Observações: Um grafo é formado por um conjunto de vértices e um conjunto de


arestas, cada aresta unindo dois vértices distintos e cada para de vértices sendo
unido por no máximo uma aresta. Arestas disjuntas são arestas que não têm
vértices em comum.

PROBLEMA 6:
Cada um dos itens a seguir apresenta um valor diferente para a matriz B. Para cada
um desses valores, determine quantas matrizes reais A existem tais que
A3 − 3 A = B.
0 1
a) B =  
1 0
4 0
b) B =  
0 4
4 1
c) B =  
0 4

SOLUÇÕES NÍVEL UNIVERSITÁRIO – PRIMEIRA FASE

PROBLEMA 1
Dando coordenadas, suponha sem perda de generalidade que
A = ( 0 ,0 ,0 ) , B = (1,0 ,0 ) , C = (1,1,0 ) , D = ( 0 ,1,0 ) ,
E = ( 0,0 ,1) , F = (1,0,1) , G = (1,1,1) , H = ( 0 ,1,1) .
Se as posições (em função do tempo) das duas naves são α (t ) e
β ( t ) , respectivamente, se t = 0 é o instante em que α ( t ) = C e β ( t ) = G e t = −1 é
o instante em que α ( t ) = B temos
α ( t ) = (1,1,0 ) + t ( 0,1,0 ) ,β ( t ) = (1,1,1) + 3 t (1,1,1) .
Assim o quadrado da distância em função do tempo é

( ( )) + ((1 + t ) − (1 + 3t )) + ( 0 − (1 + 3t ))
2 2 2
h ( t ) = (1) − 1 + 3t

= 3t + ( 3 − 1) t + 1 + 2 3t + 3t = (10 − 2 3 ) t + 2 3t + 1.
2
2 2 2 2

EUREKA! N°34, 2011

60
Sociedade Brasileira de Matemática

Temos
( )
h´ ( t ) = 20 − 4 3 t + 2 3 .

Para ( ) ( )
t0 = −2 3 20 − 4 3 = − 3 + 5 3 44 ≈ −0 , 265 temos h´ ( t0 ) = 0; para
t < t0 temos h´ ( t ) < 0 e para t > t0 temos h´ ( t ) > 0.
Assim o mínimo do quadrado da distância é
(
h ( t0 ) = 29 − 3 3 ) 44 ≈ 0 ,541
e a distância mínima é
( 29 − 3 3 ) 44 ≈ 0,7355

PROBLEMA 2
Note que 143 = 11× 13. Se N p é o número de pares ordenados (x, y) com
x, y ∈ {0 ,1,..., p − 1} tais que 5 x 2 + 7 y 2 − 1 é múltiplo de p, então a resposta do
problema será N11 ⋅ N13 . de fato, 5 x 2 + 7 y 2 − 1 é múltiplo de 143 se, e somente se, é
múltiplo de 11 e 13. Por outro lado, pelo teorema chinês dos restos, dados pares
ordenados ( x´, y´ ) com x´, y´∈ {` 0,1,...,10} e ( x´´, y´´ ) com
x´´, y´´∈ {0,1,...,12} , existe um único par ordenado ( x, y ) com x, y ∈ {` 0,1,...,142} tal
que x ≡ x´ ( mod 11) ,x ≡ x" ( mod 13) , y ≡ y´ ( mod 11) e y ≡ y" ( mod 13) .
Vamos agora calcular N11 e N13 . Os possíveis valores de x 2 ( mod 11) são 0, 1, 4,
9, 5, 3, sendo cada valor não nulo atingido para duas classes de congruência
módulo 11. Assim, 5 é quadrado módulo 11 mas 7 não é, e portanto 5 x 2 ( mod 11)
assume os valores 0, 1, 3, 4, 5, 9, enquanto 7 x 2 ( mod 11) assume os valores 0,
2,6,7,8,10 (nos dois casos os valores não nulos são assumidos duas vezes). Temos
que 1 é o resultado módulo 11 da soma de números dessas duas listas nas formas 1
+ 0,4 + 8 e 5 + 7, o que dá 2 + 4 + 4 = 10 soluções módulo 11 de 5 x 2 + 7 y 2 = 1, e
portanto N11 = 10. Analogamente, os possíveis valores de x 2 ( mod 13) são 0, 1, 4,
9, 3, 12, 10, sendo cada valor não nulo atingido para duas classes de congruência
módulo 13. Assim, 5 e 7 não são quadrados módulo 13, e portanto 5 x 2 ( mod 13) e
7 x 2 ( mod 13) assumen os valores 0, 2, 5, 6, 7, 8, 11 (os valores não nulos são
assumidos duas vezes). Temos que 1 é o resultado módulo 13 da soma de dois
números dessa lista nas formas 6 + 8,8 + 6 e 7 + 7, o que dá 4 + 4 + 4 = 12
soluções módulo 13 de 5 x 2 + 7 y 2 = 1, e portanto N13 = 12.

EUREKA! N°34, 2011

61
Sociedade Brasileira de Matemática

Assim, a resposta do problema é N11 ⋅ N13 = 10 ⋅ 12 = 120.

PROBLEMA 3
Note que, se g ( x ) = an x n + ... + a1x + a0 , então
g ( u ) − g ( v ) = an ( u n − v n ) + ... + a1 ( u − v ) = ( u − v ) ( an u n −1 + u n − 2 v + ... + v n −1 ) + ... + a1 )
= R ( u,v ) ⋅ ( u − v ) , para um certo polinômio em duas variáveis R ( x, y ) , e logo, se
f ( x ) = g ( h ( x ) ) , então

( )
f ( x ) − f ( y ) = g ( h ( x ) ) − g ( h ( y ) ) = R ( h ( x ) ,h ( y ) ) ⋅ h ( x ) − h ( y ) = q ( x, y ) ( h ( x ) − h ( y ) ) ,
com q ( x, y ) : = R ( h ( x ) ,h ( y ) ) , o que mostra a primeira implicação.
Vamos provar a volta por indução no grau de f. Se o grau de f for 0, as duas
afirmações são verdadeiras. Suponha agora que f não é constante e que
f ( x ) − f ( y ) = q ( x, y ) ( h ( x ) − h ( y ) ) (daí segue que h não é constante). Fazendo y
= 0, obtemos f ( x ) − f ( 0 ) = q ( x,0 ) ( h ( x ) − h ( 0 ) ) , para todo x, e portanto,
f ( y ) − f ( 0 ) = q ( y,0 ) ( h ( y ) − h ( 0 ) ) , para todo y. Substraindo, obtemos
q ( x, y ) ( h ( x ) − h ( y ) ) = f ( x ) − f ( y ) = q ( x,0 ) ( h ( x ) − h ( 0 ) ) − q ( y,0 ) ( h ( y ) − h ( 0 ) ) =
Assim, h ( x ) − h ( y ) divide o polinômio ( q ( x,0 ) − q ( y,0 ) ) ( h ( y ) − h ( 0 ) ) . Como
h não é constante e h ( y ) − h ( 0 ) é um polinômio só na variável y, segue que
h ( x ) − h ( y ) não tem nenhum fator comum (não constante) com h ( y ) − h ( 0 ) , e
portanto h( x) − h( y) divide o polinômio q ( x,0 ) − q ( y,0 ) . Seja
q ( x ) : = q ( x ) − q ( 0) . Temos f ( x ) − f ( 0) = q ( x,0) ( h ( x ) − h ( 0) ) = q ( x ) ( h ( x ) − h ( 0) ) ,
donde o grau de q é menor que o grau de f. Por outro lado, h ( x ) − h ( y ) divide o
polinômio q ( x,0 ) − q ( y,0 ) = q ( x ) − q ( y ) , e portanto, por hipótese de indução,
existe um polinômio g ( x) tal que q ( x ) = g ( h ( x )) , donde
f ( x ) = f ( 0) + q ( x ) ( h ( x ) − h ( 0)) = f ( 0) + g ( h ( x )) ( h ( x ) − h ( 0)) = g ( h ( x )) ,
onde g ( x ) : = f ( 0 ) + g ( x ) ( x − h ( 0 ) ) , o que completa a demonstração.

EUREKA! N°34, 2011

62
Sociedade Brasileira de Matemática

PROBLEMA 4

A figura mostra as regiões A5 (abaixo da curva vermelha), B5 (abaixo da poligonal


azul), e C5 (entre a poligonal azul e a curva vermelha).

Temos
n
Área ( An ) = ∫ ln ( t ) dt = nln ( n ) − n + 1 ;
1

1 1
Área ( Bn ) = ln ( 2 ) + ln ( 3) + ... + ln ( n − 1) + ln ( n ) = ln ( n! ) − ln ( n ) ;
2 2
1
Área ( Cn ) = nln ( n ) − n + ln ( n ) + 1 − ln ( n! ) .
2
Para estimar Área Área ( Cn ) escreva

ln ( t ) − ln ( k ) − ( t − k ) ( ln ( k + 1) − ln ( k ) ) dt ;
k +1
ak = ∫
k

note que ak é a área da k-ésima ‘bochechinha” entre a poligonal azul e a curva


vermelha.
Assim,
Área ( Cn ) = a1 + a2 + ... + an −1 ;
Queremos estimar ak para mostrar que a série abaixo converge para S < 1:
0 < S = a1 + a2 + ... + ck + ... < 1.
Seja uk ( t ) = ln ( t ) − ln ( k ) − ( t − k ) ( ln ( k + 1) − ln ( k ) ) ; temos uk ( k ) = uk ( k + 1) = 0.
Note que u"k ( t ) = −t −2 . Integrando por partes temos
k +1
ak = ∫ uk ( t ) dt
k

k +1  1 '
= −∫  t − k −  uk ( t ) dt
k
 2

EUREKA! N°34, 2011

63
Sociedade Brasileira de Matemática

k +1
1  1
2
 k +1 1 
2
1 "
= −   t − k −  u'k ( t )  + ∫  t − k −  uk ( t ) dt
 2  2  k 2 2
k

1 1 1 
2

 −  t − k −   ( −uk ( t ) ) dt
k +1
=∫ "
k
 8 2  2  
Para k ≤ t ≤ k + 1 temos ( −u"k ( t ) ) dt donde
1 1
k +1 1 
2
1
ak ≤ k −2 ∫  −  t − k −   dt = 2
.
k
 8 2  2   12 k
Temos portanto
1
S ≤ (1 + 1 22 + ... + 1 k 2 + ...)
12

1
12 ( +∞
1 + ∫ t −2 dt ≤ ;
1 ) 1
6
Completando a demonstração.

Observação: Este problema mostra como obter estimativas como a de Stirling:


temos 0 ≤ Área ( Cn ) ≤ 1 6 donde
1 1
0 ≤ nln ( n ) − n + ln ( n ) + 1 − ln ( n! ) ≤
12 6
1 5 1
nln ( n ) − n + ln ( n ) + ≤ ln ( n! ) ≤ nln ( n ) − n + ln ( n ) + 1
12 6 2
n n e − n e5 3 n ≤ n! ≤ n n e − n e 2 n
Sabemos por Stirling que a melhor aproximação é
n! ≈ n n e − n 2πn ;
note que e5 3 < 2π < e 2 .

PROBLEMA 5
Suponha que algum vértice do grafo esteja contido em todas as arestas do grafo.
Então o grafo é uma estrela com n pontas, e o resultado segue (há exatamente 2n
colorações para este exemplo).
Suponha que o grafo tenha um vértice x de grau ≥ 3 (i.e., que pertença a pelo
menos 3 arestas) e que exista uma aresta disjunta de x, digamos e. Devido à
hipótese sobre o grau de x, para aqualquer aresta e, há uma aresta f = f ( e ) que
incide em x que é disjunta de e. Então, em qualquer coloração das arestas que

EUREKA! N°34, 2011

64
Sociedade Brasileira de Matemática

incidem em x, a cor de f define a cor de e (a cor de f é a oposta de e). Assim, há no


máximo 2 ( ) ≤ 2n colorações.
grau x

Se o grafo tem um vértice x de grau 2, ligado a dois outros vértices y e z, então para
toda aresta e disjunta de x que não seja a (possível) aresta yz, há uma aresta
f = f ( e ) que incide em x que é disjunta de e, cuja cor determina a cor de e.
Assim, as cores de xy, xz e de yz (se existir) determinam todas as outras. Assim, há
no máximo 23 ≤ 2n colorações.
Finalmente, se todo vértice tem grau no máximo 1, todas as arestas são disjuntas, e
nesse caso, pelas hipótese do problema, o grafo pode máximo duas arestas e há no
máximo 22 < 2 n colorações.

PROBLEMA 6
Antes de mais nada vamos esboçar o gráfico de f ( x ) = x 3 − 3 x.

Vemos que para −2 < y < 2 a equação f ( x ) = y admite três soluções reais
enquanto para y < −2 ou y > 2 ela admite uma solução real e duas complexas
conjugadas.

a) Os autovalores de B são 1 e – 1 donde podemos escrever B = XDX −1 para X


inversível e

EUREKA! N°34, 2011

65
Sociedade Brasileira de Matemática

1 0 
D= .
 0 −1
Sejam c1 ,c2 ,c3 (resp. d1 ,d 2 ,d3 ) as soluções reais de f ( x ) = 1( resp. f ( x ) = −1) . Se
f ( A) = B temos f ( X −1 AX ) = D donde X −1 AX é da forma
 c1 0 
X −1 AX =  
 0 dj 
Para i, j escolhidos independentemente. Há portanto 9 matrizes reais A que
satisfazem f ( A ) = B.

b) Sejam z,z as raízes complexas de f ( x ) = 4. Seja v = ( w1 ,w2 ) ∈ 2


um vetor

( )
linearmente independente com v = w1 ,w2 e considere A unicamente definida por
Av = zv, Av = zv. Em outras palavras,
−1
 w w1   z 0   w1 w1 
A= 1    .
 w w2   0 z   w w2 
 2   2 
Para qualquer tal A temos f ( A ) = 4 I = B. Temos além disso A real: há portanto
infinitas matrizes reais A que satisfazem f ( A ) = B.

c) Se M é diagonalmente então f ( M ) também o é. Como B não é diagonalmente


então A também não o é. Assim A deve ter autovalor com multiplicidade algébrica
igual a 2 logo o único autovalor de A é o único real c com f ( c ) = 4. Além disso
qualquer autovetor de A é autovetor de B; como e1 é (a menos de múltiplo escalar)
o único autovalor de B então e1 deve ser autovetor de A. Já e2 deve ser autovetor
generalizado, isto é, devemos ter Ae2 = ce2 + ye1 (para algum real y). Assim
c y
A= 
0 c 
 4 ( 3c 2 − 3) y 
f ( A ) = A3 − 3 A =  
0 4 
 
e devemos ter y = 1 ( 3c 2 − 3) . Há portanto uma única solução.

EUREKA! N°34, 2011

66
Sociedade Brasileira de Matemática

XXXII OLIMPÍADA BRASILEIRA DE MATEMÁTICA


Problemas e soluções da Segunda Fase – Nível Universitário

PRIMEIRO DIA

PROBLEMA 1:

π/ 4
x
Calcule ∫ ( sen x + cos x ) cos x dx.
0

PROBLEMA 2:
Qual a maior área possível para a sombra de um cubo de aresta 1?

(Obs.: supomos que o sol está a pino, isto é, a sombra é uma projeção ortogonal; o
cubo pode estar em qualquer posição).

PROBLEMA 3:
Sejam n1 e n2 inteiros positivos e n = n1n2 .
Considere a matriz real simétrica n × n, A = ai , j ( ) 1≤ i , j ≤ n
, tal que para todo i,

ai ,i = 4,
ai ,i +1 = ai +1,i = −1 para 1 ≤ i ≤ n − 1 tal que ( i + 1) não é múltiplo de n1 ,
ai ,i + n1 = ai + n1 ,i = −1,

e as demais entradas ai , j são iguais a 0.

Prove que A é invertível e todas as entradas de A−1 são positivas.

EUREKA! N°34, 2011

67
Sociedade Brasileira de Matemática

SEGUNDO DIA

PROBLEMA 4:

 x  x ( x − 1)( x − 2 ) ...( x − j + 1)
Definimos os polinômios   = para todo j natural,
 j j!
 x
com   = 1.
0

a) Prove que todo polinômio não identicamente nulo pode ser escrito como uma
 x
combinação linear desses   de forma única;
 j

n  x
b) Seja o coeficiente de   no desenvolvimento de x n (como no item a)).
k k 
Calcule
n n
+
k k +1
.
n +1
k +1

PROBLEMA 5:
Se F é um subconjunto finito de 3 , denotamos por Vr ( F ) a vizinhança de raio r
de F (i.e., a união das bolas abertas de raio r com centros pertencentes a F).
Prove que, se 0 < r < R, vol (VR ( F ) ) ≤ ( R / r ) . vol (Vr ( F ) ) .
3

PROBLEMA 6:
Prove que se 102 n + 8.10n + 1 tem um fator primo da forma 60k + 7 então n e k são
pares.

EUREKA! N°34, 2011

68
Sociedade Brasileira de Matemática

PROBLEMA 1: SOLUÇÃO DE BRUNO DA SILVA SANTOS (BELFORD ROXO – RJ)


π4 x π4 sec 2 x
I =∫ dx = ∫ x ⋅ dx.
0
( senx + cosx ) cosx 0 tgx + 1
sec 2 x
Fazendo: x = u;dv = dx → du = dx e v = ln (1 + tgx )
1 + tgx
como ∫ u dv = u ⋅ v − ∫ v du :
π
π4 π π4
I = x ⋅ ln (1 + tgx ) 4
−∫ ln (1 + tgx ) dx = ln2 − ∫ ln (1 + tgx ) dx
0
0 4 0

π4
Seja I 2 = ∫ ln (1 + tgx ) dx. Fazendo u = π 4 − x e du = − dx :
0

 1 − tgu 
ln (1 + tg ( π 4 − u ) ) du = ∫
π4 π4
I2 = ∫ ln 1 +  du
0 0
 1 + tgu 

( ln2 − ln (1 + tgu ) ) du = π4 ln2 − ∫


π4 π4
I2 = ∫ ln (1 + tgu ) du
0 0

π π
I2 = ln2 − I 2 → I 2 = ln2.
4 8
π π4 π π π
Portanto I = ln2 − ∫ ln (1 + tgx ) dx = ln2 − ln2 = ln2.
4 0 4 8 8
π
I = ln2.
4

PROBLEMA 2: SOLUÇÃO DE RAFAELTUPYNAMBÁ DUTRA (BELO HORIZONTE – MG)


Sendo A o vértice do cubo que está mais em baixo (um vértice com altura mínima),
as três faces que contêm A estão no escuro. Colocamos os eixos de forma que os
vetores unitários normais a essas três faces sejam i = (1,0,0); j = (0,1,0);
k = (0,0,1), e consideramos o vetor unitário N = ( N x ,N y ,N z ) , com
N + N + N = 1, paralelo à direção dos raios solares.
2
x
2
y
2
z

Projetando as faces escuras do cubo sobre o plano horizontal, vemos que a área da
sombra do cubo é igual à soma das áreas das projeções das três faces escuras. Mas
a área da projeção de uma face é igual à área da face original (que é 1) multiplicada
pelo módulo do produto escalar entre o vetor normal à face e o vetor normal ao

EUREKA! N°34, 2011

69
Sociedade Brasileira de Matemática

plano de projeção. Assim, a área da sombra é


N ⋅ i + N ⋅ j + N ⋅ k = Nx + Ny + Nz .
Como a média aritmética é menor ou igual à média quadrática, temos

Nx + Ny + Nz N x2 + N y2 + N z2 1
≤ =
3 3 3
Nx + N y + Nz ≤ 3 .

Logo, a maior área possível é 3 , que ocorre quando a direção N dos raios solares
é paralela à reta HA que liga dois vértices opostos do cubo.

PROBLEMA 3: SOLUÇÃO DA BANCA


Precisamos supor que n2 > 1. De fato, se n1 = 2 e n2 = 1, nem todas as entradas de
A−1 são positivas. Pedimos desculpas...
Vamos encontrar o inverso de A = I − X , onde todas as entradas de X são 0 ou
4
( I − X ) = I + X + X 2 + X 3 + ... (note que
−1
1 . Vamos usar a série
4

( I − X ) ( I + X + X 2 + ... + X n ) = I − X n+1 ). Vamos mostrar inicialmente que essa


série converge. Para isso, vamos mostrar (ao final da solução) que o menor M ∈ R
tal que Xv ≤ M v para todo vetor v em R n (aqui ||.|| denota a norma euclidiana
usual) é menor que 1. Daí segue que X k v ≤ M k v para todo vetor v em R n e
todo inteiro positivo k, e logo a série claramente converge.
Considere o grafo cujos vértices são v1 ,v2 ,...,vn e conectamos vi e v j se e somente
se a entrada X ij em X é 1
4 . Pela definição da matriz A, esse grafo pode ser
decomposto em diversos caminhos, como os seguintes: ( v1 ,v2 , ...,vn1 −1 ),
( vkn1 ,vkn1 +1, ...,v( k +1) n1 −1 ),1 ≤ k ≤ n2 − 1 e ( vr ,vr + n1 ,vr + 2 n1 ,...vr + n − n1 ),1 ≤ r ≤ n1 .
Claramente esse grafo é conexo. Como a entrada mij em X k é não nula se e
somente se existe um caminho de i a j com k lados, para quaisquer i, j existe k tal
que a entrada correspondente mij em X k é não nula (e logo positiva). Isso prova
que todas as entradas de A−1 são positivas.
Finalmente, para mostrar que o menor M ∈ R tal que Xv ≤ M v para todo vetor
v em R n é menor que 1, consideremos um vetor v = ( a1 ,a2 ,...,an ) tal que

EUREKA! N°34, 2011

70
Sociedade Brasileira de Matemática

Xv = M v . Note que, como X é simétrica e não-negativa, podemos tomar um tal


v com Xv=Mv. Temos que, para cada j ≤ n , a j-ésima coordenada de Xv é da
ar + arj 2 + ... + arjs( j )
forma j1 , onde s( j ) ≤ 4 , cujo quadrado é menor ou igual a
4
arj1 2 + arj 2 2 + ... + arjs( j ) 2 arj1 2 + arj 2 2 + ... + arjs( j ) 2
s( j) ≤ , e, se vale a igualdade, todos
16 4
arj1 2 + arj 2 2 + ... + arjs ( j ) 2
os arji devem ser iguais. Somando os termos para todos os
4
j ≤ n , o resultado é menor ou igual a a12 + a2 2 + ... + an 2 , pois em cada coluna de X
2 2
há no máximo 4 elementos não nulos. Daí segue que Xv ≤ v e portanto M ≤ 1 .
Suponha por absurdo que M = 1, isto é, que valha a igualdade. Para cada j com
a j ≠ 0 , todos os arji devem ser iguais a a j (e portanto não nulos). Pela conexidade
do grafo definido acima , deveríamos ter então todos os a j iguais, mas nesse caso
não vale a igualdade, pois nem todas as colunas têm 4 entradas não nulas.

PROBLEMA 4: SOLUÇÃO DE MATEUS OLIVEIRA DE FIGUEIREDO (FORTALEZA – CE)


a)
 x
i) O polinômio   possui grau j já que é o produto de j polinômios de grau 1.
 j
ii) Dado n ∈ + provemos que todo polinômio de grau n pode ser escrito de
 x
maneira única como combinação linear dos   .
 j
Seja Pn o espaço vetorial de todos os polinômios de grau ≤ n.
Uma base trivial para esse espaço é:
{x0 ,x1 ,x 2 ,...,x n } , já que a0 x0 + a1 x1 + ... + an x n ≡ 0 ⇒ a0 = a1 = a2 = ... = an = 0 e
qualquer polinômio de grau menor ou igual a n pode ser escrito como combinação
linear deles. Assim, a dimensão de Pn é n + 1.

 x
iii) Se escrevermos os   nessa base temos:
 j

EUREKA! N°34, 2011

71
Sociedade Brasileira de Matemática

 x 1 j j −1 j −2
  = x + b j −1 x + b j − 2 x + ... + b0 x
0

j
  j!
 x
Assim, note que para escrever   só precisamos dos vetores { x 0 ,x1 ,x 2 ,...,x j }
 j
1
pois por i) ele possui grau j e o coeficiente de x j é .
j!
Escrevendo matricialmente
  x 
   
  n 
 1 n! b j −1 b j −2 …  xn    x  
0 c j −3   n −1    n − 1 
 x   

1
( n −1)! c j −2
0 0 1 d j −3   x n − 2  =  x  
  
n − 2!
   
     n − 2  
 0 1    x0   
    
 
  x 
M

  0  
Como abaixo da diagonal principal só temos zeros, utilizando Laplace é fácil ver
que o determinante da matriz é:
1 1 1 1
det M = ⋅ ⋅ ... ≠ 0
n! ( n − 1)! ( n − 2 )! 1!
Como Det M ≠ 0 a matriz possui inversa, ou seja, os x j podem ser escritos como
 x  x   x   x  
combinação linear dos   . Logo o conjunto S =   ,  ,...,   gera Pn e
 j  0   1   n  
como possui n + 1 elementos é base. Portanto, todo polinômio de Pn pode ser
escrito como combinação linear dos elementos de S.
 x  
Podemos estender a propriedade para S' =   j ≥ 0  já que, para j > n,
 j  
 x  
deg    > n e portanto não aparecerá na combinação linear para escrever um
 j  
polinômio de Pn .

EUREKA! N°34, 2011

72
Sociedade Brasileira de Matemática

Mas como n foi pego genérico, qualquer polinômio pode ser escrito de manera
 x
única como combinação dos   .
 j
b)
 x
i) Vamos escrever x   na base S´.
i 
 x  x ⋅ x ( x − 1)( x − 2 ) ...( x − i − 1) ( x − i + i ) x ( x − 1)( x − 2 ) ...( x − i + 1)
x  = = =
i  i! i!
(ix )
x ( x − 1)( x − 2 )( x − 3) ...( x − i + 1)( x − i ) x ( x − 1)( x − 2 ) ...( x − i + 1)
= +i =
i! i!
x ( x − 1)( x − 2 ) ...( x − i + 1)( i + 1)  x  x   x
= + i   = ( i + 1)   + i 
( i + 1)! i   i + 1  i 
(i +x1)

 x  x   x
⇒ x   = ( i + 1)   + i  .
i   i + 1  i 

ii) Sabemos que:


n n  x
xn = ∑   . Multiplicando por x temos:
i =0 i  i 

n n  x  ¨n  n   x   x  
x n +1 = ∑ x  = ∑  ( i + 1)   + i     (I)
i =0 i  i  i = 0  i   i + 1  i   
n +1 n + 1
 x
Por outro lado, x n +1 = ∑   (II)
i =0 i i 
Como todo polinômio não nulo pode ser escrito de maneira única por S´, (I) e (II)
devem ter os mesmos coeficientes.
 x   n n 
Olhando para o coeficiente de   em (I) temos ( k + 1)  + .
 k + 1  k k +i 
n +1
E em II o coeficiente é .
k +1

EUREKA! N°34, 2011

73
Sociedade Brasileira de Matemática

n n
+
 n n  n +1 k k +1 1
Logo ( k + 1)  + = ⇒ = .
 k k +1  k +1 n +1 k +1
k +1

PROBLEMA 5: SOLUÇÃO DE RAMON MOREIRA NUNES (FORTALEZA – CE)


Vamos proceder por indução em #F. No caso inicial F tem um elemento.
Trivial:
3
9 R
Vol (Vr ( F ) ) = πr 3 ,∀r ∈ ⇒ Vol (Vr ( F ) ) =   ⋅ Vol (Vr ( F ) ) ,∀0 < r < R.
3 r
Agora, suponha que se F possui k pontos então vale o resultado. Se F é um
conjunto de ( k + 1) pontos, escreva F = { x1 ,...,xk ,xk +1} .

Se F = { x1 ,...,xk } , vale o resultado para F i.e.

( ( )) é função decrescente de r.
Vol Vr F
r3

Como
Vol (Vr ( F ) )
=
( ( )) + Vol ( B ( x
Vol Vr F r k +1 ))

( )
Vol (Vr F ∩ Br ( xk +1 )
3 3 3
r r r r3
decrescente constante

4
E a primeira função é decrescente, a segunda é constante igual a π , basta provar
3
 Vol V F ∩ B ( x ) 
que a última é crescente 
r r ( ( )
k +1
 . Para fazer isso suponha sem)
 r 3 
 
perda de generalidade que xk +1 é a origem de 3 i.e. xk +1 = ( 0,0 ,0 ) = 0.

1
Considere a homotetia Tr de centro O e razão . Temos
r
 x
r
( ( )) ( )
Tr ( Br ( x ) ) = B1   ,Tr Vr F = V1 F r , onde
x x 
F r =  1 ,..., k  .
r r 
Como

1
Vol (Tr A ) = 3 Vol ( A ) para qualquer A pois
r
EUREKA! N°34, 2011

74
Sociedade Brasileira de Matemática

1 1
Vol (Tr ( A ) ) = ∫ dx = ∫ det Tr dx = ∫ 3
dx = 3 Vol ( A ) , temos
Tr A A A r r
( ( )
Vol Vr F ∩ Br ( 0 ) ) = Vol V F ∩ B ( 0) . Portanto,
r 3 ( ( ) ) 1 r 1 basta mostrar que

r → Vol (V ( F ) ∩ B ( 0 ) ) é crescente. Basta para


1 r 1 isso mostrar que

( ) ( )
V1 F r ∩ B1 ( 0 ) ⊂ V1 F R ∩ B1 ( 0 ) se r < R. Para isso, usaremos o seguinte lema:
Lema: Dado y ∈ 3
fixado, a função x → x − y é convexa, isto é,
( tx + (1 − t ) x ) − y ≤ t x − y + (1 − t ) x
1 2 2 − y ,t ∈ [ 0,1] .
Prova: Dados x1 ,x2 ∈ 3
, temos
( tx + (1 − t ) x ) − y = tx + (1 − t ) x
1 2 1 2 − ty − (1 − t ) y = t ( x1 − y ) + (1 − t )( x2 − y ) .
Pela desigualdade triangular, isso é menor ou igual a t ( x1 − y ) + (1 − t )( x2 − y ) .
Como a norma é homogênea, ( tx + (1 + t ) x ) − y ≤ t x
1 2 1 − y + (1 − t ) x2 − y .

Agora voltemos à prova de que V1 F r ∩ B1 ( 0 ) ⊂ V1 F R ∩ B1 ( 0 ) . Tome ( ) ( )


( )
a ∈V1 Fr ∩ B1 ( 0 ) . Então, ∃ i ≤ k tal que a −
xi
r
< 1 e a < 1.

xi  x
Como R > r, está no intervalo  0 , i  . Vamos usar a convexidade de
R  r
r x x
x → x − a ; escolha t = ∈ [ 0 ,1] tal que i = t i + (1 − t ) ⋅ 0. Temos
R R r
x 
xi xi
a − ≤ t a − + (1 − t ) a − 0 < t ⋅ 1 + (1 − t ) ⋅ 1 = 1. Ou seja, a ∈ B1  i  ⊂ V1 F R .
R r R
( )
( )
Como a ∈ B1 ( 0 ) ,a ∈V1 F R ∩ B1 ( 0 ) .

( )
Ou seja, acabamos de provar que V1 F r ∩ B1 ( 0 ) ⊂ V1 FR ∩ B1 ( 0 ) . Como já ( )
Vol (V ( F ) ∩ B ( x ) ) r r k +1
vimos, isso implica que r → 3
é crescente, e como também
r
Vol (Vr ( F ) )
já vimos isso implica que r → é decrescente. Concluímos.
r3

EUREKA! N°34, 2011

75
Sociedade Brasileira de Matemática

PROBLEMA 6: SOLUÇÃO DE RÉGIS PRADO BARBOSA (FORTALEZA – CE)


Temos n,k ∈ + e p primo com p = 60k + 7 tal que p 102 n + 8 ⋅ 10n + 1 . Queremos
provar que n,k são pares.
Temos
10 2 n + 8 ⋅ 10 n + 1 = (10 2 n + 2 ⋅ 10 n + 1) + 6 ⋅ 10 n = (10 n + 1) + 6 ⋅ 10 n ⇒ (10 n + 1) ≡
2 2

 −6 ⋅ 10n 
≡ −6 ⋅ 10n ( mod p ) ⇒   = 1 onde
 p 
0 , se p a
a 
  = 1, se a é resíduo quadrático módulo p.
 p  −1, se a não é resíduo quadrático módulo p.

Veja que claramente p \/ −6 ⋅ 10n = −2n +1 ⋅ 3 ⋅ 5n pois p = 60k + 7 → 2 60 ,3 60 ,5 60
e 2 \/ 7 ,3 \/ 7 ,5 \/ 7 → p ≠ 2,3 e 5.
Sabemos que o símbolo de Legendre possui a seguinte propriedade:
 a ⋅b   a   b 
  =   ⋅ .
 p   p  p
n +1 n
 −6 ⋅ 10n   −1   2   3  5 
Logo:   =   ⋅      = 1 (simplesmente separei os fatores).
 p   p   p   p  p 
Calculemos cada um deles:
 −1  p −1 60 k + 7 −1
 −1   −1 
→   = ( −1) 2 = ( −1) 2 →   = ( −1) = ( −1) ⇒   = ( −1) (I)
30 k + 3

 p  p  p

2
2
p −1
→   = ( −1) 8 . Temos p = 60k + 7 ⇒ p 2 = 3600k 2 + 2 ⋅ 60k ⋅ 7 + 49 ⇒
 p
p − 1 3600k 2 + 840k + 48
2
= .
8 8
p2 − 1  
⇒ = 450k 2 + 105k + 6 =  450k 2 + 104k + 6  + k. Assim,
8  
 par 
p 2 −1 p2 −1 2 
( −1) 8 = ( −1)(
450 k +104 k + 6 ) + k
⇒ ( −1) 8 = ( −1)(
450 k +104 k + 6)
⋅ ( −1) ⇒   = ( −1) (II)
2 2
k k

 p

EUREKA! N°34, 2011

76
Sociedade Brasileira de Matemática

3 5
Para calcular   e   precisaremos da lei da Reciprocidade Quadrática: dados
 p  p
 p  q  p −1 q −1
p, q primos ímpares:    = ( −1) 2 2

 q  p 
 3  p 3 −1 p −1
 3  p 1⋅( 30 k + 3)
Assim: com q = 3,   ⋅   = ( −1) 2 2 ⇒   ⋅   = ( −1) = ( −1) .

 p  3  p  3
 p   60k + 7   1   p
Assim precisamos de calcular   =   =   = 1 →   = 1 (usando que
 3   3   3 3
60k + 7 ≡ 1 ≡ 1 ( mod 3) ).
2

3 3
Substituindo acima tem-se:   ⋅1 = ( −1) ⇒   = ( −1) (III)
 p  p
 5  p 5 −1 p −1
 5  p 2⋅( 30 k + 3)
Com q = 5,   ⋅   = ( −1) 2 2 ⇒   ⋅   = ( −1)

= 1. Assim
 p  5  p  5
 p   60k + 7   2   p
precisamos calcular o valor de   =   =   = ( −1) →   = ( −1) (pois
 5   5  5 5
60k + 7 ≡ 2 ( mod 5 ) e os resíduos quadráticos módulo 5 são 0, 1, 4 já que
x ≡ 0,1,2,3,4 ( mod 5 ) → x 2 ≡ 0 ,1,4 ,4,1( mod 5 ) ).
 5  p 5 5 
Substituindo acima   ⋅   = 1 →   ⋅ ( −1) = 1 →   = ( −1) (IV)
 p  5  p  p
Juntam-se (I), (II), (III) e (IV) em (*):
n +1 n
 −1  2   3  5  k ( n +1) k ( n +1) + n
  ⋅   ⋅    = 1 ⇒ ( −1) ⋅ ( −1) ⋅ ( −1) ⋅ ( −1) = 1 ⇒ ( −1)
n
=1
 p   p  p  p 
⇒ k ( n + 1) + n é par.
⇒ se n é ímpar → ( n + 1) par → k ( n + 1) par ⇒ k ( n + 1) + n ímpar. Absurdo!
Logo n é par → ( n + 1) ímpar.
⇒ se k é ímpar → k ( n + 1) ímpar → k ( n + 1) + n ímpar. Absurdo!
Logo k é par .
Assim, dados n,k ∈ + com p = 60k + 7 primo tal que p 102 n + 8 ⋅ 10n + 1 então n e
k são pares.

EUREKA! N°34, 2011

77
Sociedade Brasileira de Matemática

XXXII OLIMPÍADA BRASILEIRA DE MATEMÁTICA


Premiados

NÍVEL 1 (6º. e 7º. Anos)

NOME CIDADE – ESTADO PRÊMIO


Ana Emília Hernandes Dib S.J. do Rio Preto - SP Ouro
Pedro Henrique Alencar Costa Fortaleza - CE Ouro
Ryunosuke Watanabe Tagami Rio Claro - SP Ouro
Helena Veronique Rios São Carlos - SP Ouro
Italo Lesione de Paiva Rocha Fortaleza - CE Ouro
José Henrique Carvalho Curitiba - PR Ouro
Juliana Bacelar de Freitas Brasília - DF Prata
Daniel Lima Braga Eusébio - CE Prata
Hermes Lins e Nascimento Fortaleza - CE Prata
Laís Monteiro Pinto Rio de Janeiro - RJ Prata
Lucca Morais de Arruda Siaudzionis Fortaleza - CE Prata
Leandro Alves Cordeiro Ribeirão Pires - SP Prata
Henrique Gontijo Chiari Belo Horizonte - MG Prata
André Akinaga Benites São Paulo - SP Prata
Gabriel Diniz Vieira e Sousa Fortaleza - CE Prata
Rafael Seiji Uezu Higa São Paulo - SP Prata
Adriana de Sousa Figueiredo Porto Alegre - RS Prata
Gustavo Figueiredo Serra São Paulo - SP Prata
Matheus Uchôa Constante Goiânia - GO Bronze
Kristian Holanda Nogueira Manaus - AM Bronze
Fábio Itikama São Paulo - SP Bronze
Loic Dominguez Fortaleza - CE Bronze
Jiang Zhi São Paulo - SP Bronze
Ricardo Ken Wang Tsuzuki São Paulo - SP Bronze
Ana Caroline Obana da Cruz Curitiba - PR Bronze
Ana Paula Lopes Schuch Porto Alegre - RS Bronze
José Marcio Machado de Brito Cocal dos Alves - PI Bronze
Lucas Bastos Germano Fortaleza - CE Bronze
Victória Moreira Reis Cogo Teresina - PI Bronze
Thiago Araujo Oliveira Jaboatão dos Guararapes - PE Bronze
Gabriel Toneatti Vercelli Osasco - SP Bronze
Nathan Bonetti Teodoro Curitiba - PR Bronze
Jefferson Daxian Hong São Paulo - SP Bronze
Cristóbal Sciutto Rodriguez São Paulo - SP Bronze
Aruana Almeida Correa Porto Alegre - RS Bronze
Cynthia Lacroix Herkenhoff Vitória - ES Bronze
Kaíque Maestrini Sacchi São Paulo - SP Menção Honrosa
Igor de Lacerda Curitiba - PR Menção Honrosa

EUREKA! N°34, 2011

78
Sociedade Brasileira de Matemática

Rafael Reple Geromee São Paulo - SP Menção Honrosa


Leonardo de Matos Felippetti Mariano Curitiba - PR Menção Honrosa
Gabriel Passamani Correa Vitória - ES Menção Honrosa
Daniel de Almeida Souza Brasília - DF Menção Honrosa
Diego Teixeira Nogueira Fidalgo Salvador - BA Menção Honrosa
Natan Novellu Tu São Paulo - SP Menção Honrosa
Ricardo Borsari Brinati São Paulo - SP Menção Honrosa
Rafael Neves Vieira Brasília - DF Menção Honrosa
Juliano Pecica Negri Piracicaba - SP Menção Honrosa
Gustavo Rodrigues Machado Sorocaba - SP Menção Honrosa
Zoltan Flamarion Glueck Carvalho Belo Horizonte - MG Menção Honrosa
Gabriel Ribeiro Barbosa Fortaleza - CE Menção Honrosa
Pedro Henrique Rocha de Freitas Brasília - DF Menção Honrosa
Pedro Henrique Sacramento de Oliveira Loureira - SP Menção Honrosa
Guilherme Goulart Kowalczuk Porto Alegre - RS Menção Honrosa
Pedro de Vasconcellos Oporto Nova Lima - MG Menção Honrosa
Aryssa Victoria Shitara São Paulo - SP Menção Honrosa
Ives Vaz Caldeira Lopes São Paulo - SP Menção Honrosa
Marcos Vinícius de Oliveira Soares Rio de Janeiro - RJ Menção Honrosa
Jéssica Carolina Zilio Piracicaba - SP Menção Honrosa
João Pedro Graça Melo Vieira Rio de Janeiro - RJ Menção Honrosa
Henrique Medici Pontieri Campo Grande - MS Menção Honrosa
Gabriel Caino Castilho Rodrigues Salvador - BA Menção Honrosa
Tamara P. de A. Moraes Feira de Santana - BA Menção Honrosa
Karine Quaresma Lima Taguatinga - DF Menção Honrosa
Natália Brasileiro Lins Barbosa Jaboatão dos Guararapes - PE Menção Honrosa
Lucki Li São Paulo - SP Menção Honrosa
Heloísa Antunes de Medeiros Itamogi - MG Menção Honrosa
Iuri Grangeiro Carvalho Fortaleza - CE Menção Honrosa
Lara Sampaio Pinheiro de Freitas Olinda - PE Menção Honrosa
Maria Júlia Costa Medeiros Fortaleza - CE Menção Honrosa
Kevin Korpasch Guarapuana - PR Menção Honrosa
Sofía Leite Correia Lima Fortaleza - CE Menção Honrosa
João Baptista de Paula e Silva Belo Horizonte - MG Menção Honrosa
Bernardo Puetter Schaeffer Rio de Janeiro - RJ Menção Honrosa
Júlia Bertelli Joinville - SC Menção Honrosa
Rafael Purim de Azevedo Pirassununga - SP Menção Honrosa
Pedro Henrique da Silva Dias Porto Alegre - RS Menção Honrosa
Marcelo Bandeira de Melo Boavista Teresina - PI Menção Honrosa
Gabriel Branco Frizzo Curitiba - PR Menção Honrosa
Maria Eduarda Müller Eyng Porto Alegre - RS Menção Honrosa
Henrique Martínez Rocamora São Bernardo do Campo - SP Menção Honrosa
Felipe Roz Barscevicius Sorocaba - SP Menção Honrosa
João Vitor Vaz Oliveira Recife - PE Menção Honrosa
Mateus Siqueira Thimoteo Mogi das Cruzes - SP Menção Honrosa

EUREKA! N°34, 2011

79
Sociedade Brasileira de Matemática

Ebenezeer Pinto Banoeira Neto Fortaleza - CE Menção Honrosa


Maria Clara Vasconcelos Andrade Brasília - DF Menção Honrosa
Rafael Beck Salvador - BA Menção Honrosa
Arthur Monteiro Dos Santos Salvador - BA Menção Honrosa
Júlia Wotzasek Pereira São Paulo - SP Menção Honrosa
Gabriel Oliveira Rigo Cotia - SP Menção Honrosa
Leonardo Galante Barco São Paulo - SP Menção Honrosa
Bruno Scatolini São Paulo - SP Menção Honrosa
Lucas Pereira Galvão de Barros São Paulo - SP Menção Honrosa
Vítor Ossamu Rodrigues Okamura Brasília DF Menção Honrosa

NÍVEL 2 (8º. e 9º. Anos)

NOME CIDADE – ESTADO PRÊMIO


Rafael Rodrigues Rocha de Melo Caucaia - CE Ouro
Vinícius Canto Costa Rio de Janeiro - RJ Ouro
Henrique Vieira G. Vaz São Paulo - SP Ouro
Fellipe Sebastiam da Silva P. Pereira Rio de Janeiro - RJ Ouro
Roberto Tadeu Abrantes de Araújo Rio de Janeiro - RJ Ouro
Pedro Victor Falci de Rezende Santo Antonio - MG Ouro
Alessandro A. de Oliveira Pacanowski Rio de Janeiro - RJ Prata
Lincoln de Queiroz Vieira Fortaleza - CE Prata
Tadeu Pires de Matos Belford Neto Fortaleza - CE Prata
Vitor Ramos de Paula Belo Horizonte - MG Prata
Francisco Markan N. de Souza Filho Fortaleza - CE Prata
Jair Gomes Soares Júnior Montes Claros - MG Prata
Breno Soares da Costa Vieira J. dos Guararapes - PE Prata
Gabriel José Moreira da Costa Silva Maceió - AL Prata
Pedro Morais de Arruda Siaudzionis Fortaleza - CE Prata
Gabriel Sena Galvão Guará - DF Prata
Fabio da Silva Soares Planaltina - DF Prata
Michel Rozenberg Zelazny São Paulo - SP Prata
Bruno Eidi Nishimoto Jales - SP Prata
Franco Matheus de Alencar Severo Rio de Janeiro - RJ Prata
Aimê Parente de Sousa Fortaleza - CE Prata
Marcos Paulo Nunes de Lima Silva Rio de Janeiro - RJ Bronze
Gabriel N. Coelho de Togni de Souza Rio de Janeiro - RJ Bronze
Rafael T. Eugênio Pontes Barone Aracatuba - SP Bronze
Murilo Corato Zanarella Amparo - SP Bronze
Rodrigo Sanches Angelo São Paulo - SP Bronze
Alexandre Perozim de Faveri Neves Paulista - SP Bronze
Luíze Mello D'urso Vianna Rio de Janeiro - RJ Bronze
Maria Clara Cardoso São Paulo - SP Bronze
Liara Guinsberg São Paulo - SP Bronze
Lucas Cawai Julião Pereira Caucaia - CE Bronze

EUREKA! N°34, 2011

80
Sociedade Brasileira de Matemática

Luis Guilherme Gomes Aguiar Rio de Janeiro - RJ Bronze


Carlos Adriano Vieira Igarapé - MG Bronze
Daniel Santana Rocha Rio de Janeiro - RJ Bronze
Raphael Mendes de Oliveira Rio de Janeiro - RJ Bronze
Samuel Brasil de Albuquerque Fortaleza - CE Bronze
Gustavo Souto Henriques Campelo João Pessoa - PB Bronze
Lucas de Moura Herlin Rio de Janeiro - RJ Menção Honrosa
Vitor Dias Gomes Barrios Marin Presidente Prudente - SP Menção Honrosa
João Pedro Sedeu Godoi Rio de Janeiro - RJ Menção Honrosa
Suzane Eberhart Ribeiro da Silva Campo Grande - MS Menção Honrosa
Ícaro Sampaio Viana Fortaleza - CE Menção Honrosa
Pedro Henrique Bortolozo Maria Colombo - PR Menção Honrosa
Fábio Kenji Arai São Paulo - SP Menção Honrosa
Guilherme de Oliveira Rodrigues Fortaleza - CE Menção Honrosa
Alexandre Mendonça Cardoso Salvador - BA Menção Honrosa
Leyberson Pereira Assunção Fortaleza - CE Menção Honrosa
Rubens Martins Bezerra Farias Sobral - CE Menção Honrosa
João Vítor Fernandes Paiva Rio de Janeiro - RJ Menção Honrosa
Bruno Almeida Costa Fortaleza - CE Menção Honrosa
Daniel Lima Santanelli Rio de Janeiro - RJ Menção Honrosa
Marília Nascimento Monteiro Recife - PE Menção Honrosa
Igor Albuquerque Araujo Rio de Janeiro - RJ Menção Honrosa
Josué Knorst Picada Café - RS Menção Honrosa
Ricardo Vieira Marques Brasília - DF Menção Honrosa
Júlio César de Barros Santo André - SP Menção Honrosa
Thomas Akio Ikeda Valvassori Mogi das Cruzes - SP Menção Honrosa
Gabriel Fazoli Domingos Urupês - SP Menção Honrosa
Henrique Luan Gomes Pereira Braga Belem - PA Menção Honrosa
Beatriz Yumi Ota São Paulo - SP Menção Honrosa
Kiane Sassaki Menezes Rio de Janeiro - RJ Menção Honrosa
Eric Gripa Marques Rio de Janeiro - RJ Menção Honrosa
Samuel Kuo Chen Shao São Paulo - SP Menção Honrosa
Pedro Henrique Jagosenit Vilaça Santa Branca - SP Menção Honrosa
Caio de Souza Câmara Manaus - AM Menção Honrosa
Lucas David Noveline Belem - PA Menção Honrosa
Lucas Rebelo Vieira da Silva Recife - PE Menção Honrosa
Elias Brito Oliveira Brasília - DF Menção Honrosa
Guilherme Ryu Odaguiri Kobori São Paulo - SP Menção Honrosa
Mariana Souza de Araújo Recife - PE Menção Honrosa
Francisco Cláudio Coelho Rio de Janeiro - RJ Menção Honrosa
Murilo Leão Pereira Belem - PA Menção Honrosa
Jadi Diniz Guimarães de Queiroz Recife - PE Menção Honrosa
Caio Lima Albuquerque São Paulo - SP Menção Honrosa
Carolina Lima Guimarães Vitória - ES Menção Honrosa

EUREKA! N°34, 2011

81
Sociedade Brasileira de Matemática

Nível 3 (Ensino Médio)


NOME CIDADE – ESTADO PRÊMIO
Gustavo Lisbôa Empinotti Florianópolis - SC Ouro
Marcelo Tadeu de Sá Oliveira Sales Salvador - BA Ouro
João Lucas Camelo Sá Fortaleza - CE Ouro
Hanon Guy Lima Rossi São Paulo - SP Ouro
Maria Clara Mendes Silva Pirajuba - MG Ouro
Matheus Secco Torres da Silva Rio de Janeiro - RJ Prata
Lucas Lourenço Hernandes São Paulo - SP Prata
Deborah Barbosa Alves São Paulo - SP Prata
Henrique G. Fiuza do Nascimento Brasília - DF Prata
Luiz Filipe Martins Ramos Niterói - RJ Prata
André Macieira Braga Costa Belo Horizonte - MG Prata
Thiago Saksanian Hallak São Paulo - SP Prata
Victor Juca Martins Fortaleza - CE Prata
Caíque Porto Lira Fortaleza - CE Prata
Gustavo H. F. e Sampaio Braga São José dos Campos-SP Prata
Alvaro Lopes Pedroso Santa Isabel - SP Prata
André Amaral de Sousa Diadema - SP Prata
Marcos Massayuki Kawakami São Paulo - SP Bronze
Carlos Henrique de Andrade Silva Fortaleza - CE Bronze
Rafael Kazuhiro Miyazaki São Paulo - SP Bronze
André Saraiva Nobre dos Santos Fortaleza - CE Bronze
Daniel Eiti Nishida Kawai Atibaia - SP Bronze
Lucas de Freitas Smaira Guaxupé - MG Bronze
Cássio dos Santos Sousa Osasco - SP Bronze
Alessandro Macêdo de Araújo Fortaleza - CE Bronze
Breno Vieira da Silva Passos Aracaju - SE Bronze
Iago Dalmaso Brasil Dias Rio de Janeiro - RJ Bronze
Isabella Amorim Gonçalez Marília - SP Bronze
Daniel dos Santos Bossle Porto Alegre - SP Bronze
Davi Coelho Amorim Fortaleza - CE Bronze
Lucas Mestres Mendes Fortaleza - CE Bronze
Vinícius Gomes Pereira Rio de Janeiro - RJ Bronze
Renan Pablo da Cruz Fortaleza - CE Bronze
Jonas Rocha Lima Amaro Fortaleza - CE Bronze
Iuri Rezende Souza Mineiros - GO Bronze
Matheus Araújo Marins São Gonçalo - RJ Menção Honrosa
Felipe Vieira de Paula Fortaleza - CE Menção Honrosa
Rafael Farias Marinheiro Recife - PE Menção Honrosa
Elvis Falcao de Araujo Fortaleza - CE Menção Honrosa
Pablo Almeida Gomes Santana de Pirapama - MG Menção Honrosa
Paulo Gabriel Ramos Monteiro Rio de Janeiro - RJ Menção Honrosa
Victor de Oliveira Bitarães Betim - MG Menção Honrosa

EUREKA! N°34, 2011

82
Sociedade Brasileira de Matemática

Daniel Caueh Dunaiski Figueira Leal Curitiba - PR Menção Honrosa


Raphael Julio Barcelos Taguatinga - DF Menção Honrosa
Fernando Fonseca Andrade Oliveira Belo Horizonte - MG Menção Honrosa
Felipe Mendes dos Santos Gama - DF Menção Honrosa
Felipe Abella C. Mendonça de Souza João Pessoa - PB Menção Honrosa
Francisco Raul Lobo Rodrigues Fortaleza - CE Menção Honrosa
Gabriel Leite de Carvalho Fortaleza - CE Menção Honrosa
André Austregesilo Scussel Fortaleza - CE Menção Honrosa
Victorio Takahashi Chu São Paulo - SP Menção Honrosa
Victor José Tiburtius Franco Recife - PE Menção Honrosa
Matheus Cavalcante Lima Fortaleza - CE Menção Honrosa
Cleberton de Santana Oliveira São Miguel do Aleixo - SE Menção Honrosa
Mauro Brito Júnior Fortaleza - CE Menção Honrosa
Gabriel José Guimarães Barbosa Pequi - MG Menção Honrosa
Lucas Colucci Cavalcante de Souza São Paulo - SP Menção Honrosa
Sarah Villanova Borges Juiz de Fora - MG Menção Honrosa
Ivan Tadeu Ferreira Antunes Filho Lins - SP Menção Honrosa
Dalton Felipe de Menezes São José dos Campos-SP Menção Honrosa
Thiago de Paula Vasconcelos Fortaleza - CE Menção Honrosa
Jardiel Freitas Cunha Recife - PE Menção Honrosa
Ana Beatriz Prudêncio de A. Rebouças Fortaleza - CE Menção Honrosa
Rafael Sussumu Yamaguti Miada Valinhos - SP Menção Honrosa
Davi Sampaio de Alencar Fortaleza - CE Menção Honrosa
Bruno Ferri de Moraes São Paulo - SP Menção Honrosa

Nível Universitário
NOME CIDADE – ESTADO PREMIO
Rafael Tupynambá Dutra Belo Horizonte - MG Ouro
Renan Henrique Finder Rio de Janeiro - RJ Ouro
Regis Prado Barbosa Fortaleza - CE Ouro
Ramon Moreira Nunes Fortaleza - CE Ouro
Thomás Yoiti Sasaki Hoshina Rio de Janeiro - RJ Ouro
Guilherme Rodrigues N. de Souza S.J. dos Campos - SP Prata
Jorge Henrique Craveiro de Andrade Rio de Janeiro - RJ Prata
Rafael Assato Ando Campinas - SP Prata
Gabriel Luís Mello Dalalio S.J. dos Campos - SP Prata
Charles Barbosa de Macedo Brito Rio de Janeiro - RJ Prata
Leonardo Ribeiro de Castro Carvalho S.J. dos Campos - SP Prata
Marcelo Matheus Gauy São José do Rio Preto-SP Prata
Leandro Farias Maia Rio de Janeiro - RJ Prata
Adenilson Arcajo de Moura Júnior Fortaleza - CE Bronze
Paulo André Carvalho de Melo Piedade - RJ Bronze
Joas Elias dos Santos Rocha Muribeca - SE Bronze

EUREKA! N°34, 2011

83
Sociedade Brasileira de Matemática

Guilherme Lourenço Mejia S.J. dos Campos - SP Bronze


Reinan Ribeiro Souza Santos Lagarto - SE Bronze
Rafael Alves da Ponte Fortaleza - CE Bronze
Davi Lopes Alves de Medeiros Fortaleza - CE Bronze
Luca Mattos Moller Nova Friburgo - RJ Bronze
Renato Rebouças de Medeiros Fortaleza - CE Bronze
Danilo Furlan Kaio São Paulo - SP Bronze
Rafael Endlich Pimentel Vitória - ES Bronze
Paulo Sérgio de Castro Moreira Fortaleza - CE Bronze
Carlos Coelho Lechner Rio de Janeiro - RJ Menção Honrosa
Thiago Ribeiro Ramos Varginha - MG Menção Honrosa
Hugo Fonseca Araújo Rio de Janeiro - RJ Menção Honrosa
Alysson Espíndola de Sá Silveira Fortaleza - CE Menção Honrosa
Jordan Freitas Piva Rio de Janeiro - RJ Menção Honrosa
Érik Fernando de Amorim Araraquara - SP Menção Honrosa
Daniel Ungaretti Borges Belo Horizonte - MG Menção Honrosa
Antônio Deromir Neves Silva Júnior Fortaleza - CE Menção Honrosa
Rafael Parpinel Cavina São Paulo - SP Menção Honrosa
Isaque Santa Brigida Pimentel Barcarena - PA Menção Honrosa
Mateus Oliveira de Figueiredo Fortaleza - CE Menção Honrosa
Davi Dos Santos Lima Maceió - AL Menção Honrosa
Bruno da Silva Santos Belford Roxo - RJ Menção Honrosa
Francisco Osman Pontes Neto Fortaleza - CE Menção Honrosa
Breno Vieira de Aguiar Rio de Janeiro - RJ Menção Honrosa
Ricardo Turolla Bortolotti Rio de Janeiro - RJ Menção Honrosa
Guilherme Philippe Figueiredo São Paulo - SP Menção Honrosa
Daniel de Barros Soares Rio de Janeiro - RJ Menção Honrosa
Hudson do Nascimento Lima Fortaleza - CE Menção Honrosa
Eduardo Fischer Encantado - RS Menção Honrosa
Luty Rodrigues Ribeiro Fortaleza - CE Menção Honrosa
José Leandro Pinheiro Fortaleza - CE Menção Honrosa
Caio Ishizaka Costa S.J. dos Campos - SP Menção Honrosa
Gabriel Caser Brito Rio de Janeiro - RJ Menção Honrosa
Leonardo Donisete da Silva Campinas - SP Menção Honrosa
Alan Anderson da Silva Pereira União dos Palmares - AL Menção Honrosa
Diego Andrés de Barros Lima Barbosa Rio de Janeiro - RJ Menção Honrosa
Renato Dias Costa Rio de Janeiro - RJ Menção Honrosa
Ivan Guilhon Mitoso Rocha Fortaleza - CE Menção Honrosa
Willy George do Amaral Petrenko Rio de Janeiro - RJ Menção Honrosa
Leonardo Borges Avelino Rio de Janeiro - RJ Menção Honrosa
Jose Armando Barbosa Filho Fortaleza - CE Menção Honrosa

EUREKA! N°34, 2011

84
Sociedade Brasileira de Matemática

AGENDA OLÍMPICA
XXXIII OLIMPÍADA BRASILEIRA DE MATEMÁTICA

NÍVEIS 1, 2 e 3
Primeira Fase – sábado, 18 de junho de 2011
Segunda Fase – sábado, 3 de setembro de 2011
Terceira Fase – sábado, 15 de outubro de 2011 (níveis 1, 2 e 3)
domingo, 16 de outubro de 2011 (níveis 2 e 3 - segundo dia de prova)

NÍVEL UNIVERSITÁRIO
Primeira Fase – sábado, 3 de setembro de 2011
Segunda Fase – sábado, 15 e domingo, 16 de outubro de 2011

IV ROMANIAN MASTER OF MATHEMATICS (RMM)


23 a 28 de fevereiro de 2011(Bucareste, Romênia)

ASIAN PACIFIC MATH OLYMPIAD (APMO)


12 de março de 2011

XVII OLIMPÍADA DE MAIO


7 de maio de 2011

XXII OLIMPÍADA DE MATEMÁTICA DO CONE SUL


14 a 20 de agosto de 2011(La Paz, Bolívia)

LII OLIMPÍADA INTERNACIONAL DE MATEMÁTICA


13 a 24 de julho de 2011(Amsterdam, Holanda)

I OLIMPÍADA DE MATEMÁTICA DA LUSOFONIA


20 a 31 de julho de 2011(Coimbra, Portugal)

XVII OLIMPÍADA INTERNACIONAL DE MATEMÁTICA UNIVERSITÁRIA (IMC)


24 a 30 de julho de 2011(Blagoevgrad, Bulgária)

XXV OLIMPÍADA IBEROAMERICANA DE MATEMÁTICA


23 de setembro a 1 de outubro de 2011(São José, Costa Rica)

III COMPETIÇÃO IBEROAMERICANA INTERUNIVERSITÁRIA DE MATEMÁTICA


2 a 8 de outubro de 2011(Quito, Equador)

XIII OLIMPÍADA IBEROAMERICANA DE MATEMÁTICA UNIVERSITÁRIA


26 de novembro de 2011

EUREKA! N°34, 2011

85
Sociedade Brasileira de Matemática

COORDENADORES REGIONAIS
Alberto Hassen Raad (UFJF) Juiz de Fora – MG
Américo López Gálvez (USP) Ribeirão Preto – SP
Antonio Carlos Nogueira (UFU) Uberlândia – MG
Benedito Tadeu Vasconcelos Freire (UFRN) Natal – RN
Bruno Holanda (CAEN – UFC) Fortaleza – CE
Carmen Vieira Mathias (UNIFRA) Santa María – RS
Claus Haetinger (UNIVATES) Lajeado – RS
Cláudio de Lima Vidal (UNESP) S.J. do Rio Preto – SP
Denice Fontana Nisxota Menegais (UNIPAMPA) Bagé – RS
Disney Douglas Lima de Oliveira (UFAM) Manaus – AM
Edson Roberto Abe (Colégio Objetivo de Campinas) Campinas – SP
Edney Aparecido Santulo Jr. (UEM) Maringá – PR
Fábio Brochero Martínez (UFMG) Belo Horizonte – MG
Florêncio Ferreira Guimarães Filho (UFES) Vitória – ES
Francinildo Nobre Ferreira (UFSJ) São João del Rei – MG
Genildo Alves Marinho (Centro Educacional Leonardo Da Vinci) Taguatingua – DF
Herivelto Martins (USP – São Carlos) São Carlos – SP
Gilson Tumelero (UTFPR) Pato Branco – PR
Ivanilde Fernandes Saad (UC. Dom Bosco) Campo Grande – MS
João Benício de Melo Neto (UFPI) Teresina – PI
João Francisco Melo Libonati (Grupo Educacional Ideal) Belém – PA
Diogo Diniz (UFPB) Campina Grande – PB
José Luiz Rosas Pinho (UFSC) Florianópolis – SC
José Vieira Alves (UFPB) Campina Grande – PB
José William Costa (Instituto Pueri Domus) Santo André – SP
Krerley Oliveira (UFAL) Maceió – AL
Licio Hernandes Bezerra (UFSC) Florianópolis – SC
Luciano G. Monteiro de Castro (Sistema Elite de Ensino) Rio de Janeiro – RJ
Luzinalva Miranda de Amorim (UFBA) Salvador – BA
Marcelo Dias (Grupo Educacional Etapa) São Paulo – SP
Marcelo Antonio dos Santos FACOS Osório – RS
Marcelo Rufino de Oliveira (Grupo Educacional Ideal) Belém – PA
Newman Simões (Cursinho CLQ Objetivo) Piracicaba – SP
Nivaldo Costa Muniz (UFMA) São Luis – MA
Osnel Broche Cristo (UFLA) Lavras – MG
Uberlândio Batista Severo (UFPB) João Pessoa – PB
Raul Cintra de Negreiros Ribeiro (Colégio Anglo) Atibaia – SP
Reginaldo de Lima Pereira (Escola Técnica Federal de Roraima) Boa Vista – RR
Reinaldo Gen Ichiro Arakaki (UNIFESP) SJ dos Campos – SP
Ricardo Amorim (Centro Educacional Logos) Nova Iguaçu – RJ
Ronaldo Alves Garcia (UFGO) Goiânia – GO
Rogério da Silva Ignácio (Col. Aplic. da UFPE) Recife – PE
Rosangela Ramon (UNOCHAPECÓ) Chapecó – SC
Sérgio Cláudio Ramos (IM-UFRGS) Porto Alegre – RS
Seme Gebara Neto (UFMG) Belo Horizonte – MG
Tadeu Ferreira Gomes (UEBA) Juazeiro – BA
Tomás Menéndez Rodrigues (U. Federal de Rondônia) Porto Velho – RO
Valdenberg Araújo da Silva (U. Federal de Sergipe) São Cristóvão – SE
Wagner Pereira Lopes (CEFET – GO) Jataí – GO
Wanderson Breder (CEFET – RJ) Nova Friburgo – RJ
William Serafim dos Reis (UFT – TO) Arraias – TO

EUREKA! N°34, 2011

86
CONTEÚDO
XVII OLIMPÍADA DE MAIO
Enunciados e resultado brasileiro 03

XXII OLIMPÍADA DE MATEMÁTICA DO CONE SUL


Enunciados e resultado brasileiro 06

LII OLIMPÍADA INTERNACIONAL DE MATEMÁTICA (IMO)


Enunciados e resultado brasileiro 08

LIII OLIMPÍADA INTERNACIONAL DE MATEMÁTICA (IMO)


Enunciados e resultado brasileiro 10

II OLIMPÍADA DE MATEMÁTICA DA COMUNIDADE DOS PAÍSES DE LÍNGUA


PORTUGUESA 13
Enunciados e resultado brasileiro

XXVI OLIMPÍADA IBEROAMERICANA DE MATEMÁTICA


Enunciados e resultado brasileiro
16
ARTIGOS

PARIDADE DO NÚMERO BINOMIAL MÉDIO


J.C.S. de Miranda (USP) e Iesus C. Diniz (FRN) 18

PONTO MÉDIO LEMBRA? OUTRO PONTO MÉDIO! DOIS PONTOS MÉDIOS


LEMBRAM? BASE MÉDIA! 29
Cícero Thiago Magalhães

CAMPEONATOS 36
José Armando Barbosa Filho

SOLUÇÕES DE PROBLEMAS PROPOSTOS 47

PROBLEMAS PROPOSTOS 59

AGENDA OLÍMPICA 61

COORDENADORES REGIONAIS 62
Sociedade Brasileira de Matemática

Esta edição da Revista Eureka! é dedicada à memoria do professor Sérgio Cláudio Ramos
da Universidade Federal do Rio Grande do Sul (UFRGS), que colaborou como coordenador
regional da Olimpíada Brasileira de Matemática (OBM) desde 1998, e que nos deixou neste
ano de 2012.

A comunidade da Olimpíada Brasileira de Matemática (OBM)

EUREKA! N°35, 2012

2
Sociedade Brasileira de Matemática

XVII OLIMPÍADA DE MAIO


PRIMEIRO NÍVEL

PROBLEMA 1
As quatro palavras codificadas
*⊗ ⊕#• * • ⊗♦⊕
são em alguma ordem
AMO SUR REO MAS

Decifrar ⊗♦ *⊕# •⊗.

PROBLEMA 2
Utilizando apenas uma vez cada um dos dígitos 1, 2, 3, 4, 5, 6, 7 e 8 se escrevem o
quadrado e o cubo de um número inteiro positivo. Determine quanto pode valer
este número.

PROBLEMA 3
1
No retângulo ABCD, BC = 5, EC = CD e F é o ponto onde se cortam AE e BD.
3
O triângulo DFE tem área 12 e o triângulo ABF tem área 27. Encontre a área do
quadrilátero BCEF.
D E C

A B

PROBLEMA 4
Utilizando vários cubinhos brancos de aresta 1, Guille monta um cubo grande. Em
seguida, escolhe quatro faces do cubo grande e as pinta de vermelho. Finalmente
desmonta o cubo grande e observa que os cubinhos com pelo menos uma face
pintada de vermelho são 431. Encontre a quantidade de cubinhos que Guille
utilizou para montar o cubo grande.

Analise todas as possibilidades.

EUREKA! N°35, 2012

3
Sociedade Brasileira de Matemática

SEGUNDO NÍVEL

PROBLEMA 1
Encontre um número inteiro positivo x tal que a soma dos dígitos de x seja maior
que 2011 vezes a soma dos dígitos do número 3x (3 vezes x).

PROBLEMA 2
Dizemos que um número de quatro dígitos abcd (a ≠ 0) é porá se valem as
seguintes condições:
a ≥ b;
ab − cd = cd − ba.
por exemplo, 2011 é porá porque 20 – 11 = 11 – 02.

Encontre todos os números porá.

PROBLEMA 3
Num triângulo retângulo ABC tal que AB = AC, M é o ponto médio de BC. Seja P
um ponto da mediatriz de AC que pertence ao semiplano determinado por BC que
não contém A. As retas CP e AM se cortam em Q. Calcule o ângulo que formam
AP e BQ.

PROBLEMA 4
Dados n pontos em uma circunferência se escreve ao lado de um deles um 1 e ao
lado de cada um dos outros um 0. A operação permitida consiste em escolher um
ponto que tenha um 1 e trocar o número desse ponto e também os números dos
seus dois vizinhos, o da esquerda e o da direita (onde há 1 se escreve 0 e onde há 0
se escreve 1).

a) Se n = 101, mostre que se pode conseguir, mediante uma sucessão de operações


permitidas, que cada um dos n pontos tenha escrito 0.

b) Se n = 102, mostre que é impossível obter todos 0.

PROBLEMA 5
Determine para quais números naturais n é possível cobrir completamente um
tabuleiro de n × n dividido em casas de 1 × 1 com peças como a da figura, sem

EUREKA! N°35, 2012

4
Sociedade Brasileira de Matemática

buracos nem superposições e sem sair do tabuleiro. Cada uma das peças cobre
exatamente seis casas.
Nota: As peças podem girar.

RESULTADO BRASILEIRO

2011: Nível 1 (até 13 anos)

Nome Cidade – Estado Prêmio


Nathan Bonetti Teodoro Curitiba - PR Medalha de Ouro
Júlia Bertelli Joinville - SC Medalha de Prata
Lucas Pereira Galvão de Barros São Paulo - SP Medalha de Prata
Pedro Henrique Sacramento de Oliveira Vinhedo - SP Medalha de Bronze
Pedro Henrique da Silva Dias Porto Alegre - RS Medalha de Bronze
Jiang Zhi São Paulo - SP Medalha de Bronze
Lucki Li São Paulo - SP Medalha de Bronze
Lucca Morais de Arruda Siaudzionis Fortaleza - CE Menção Honrosa
Helena Veronique Rios São Carlos - SP Menção Honrosa
Cristóbal Sciutto São Paulo - SP Menção Honrosa

2011: Nível 2 (até 15 anos)

Nome Cidade – Estado Prêmio


Murilo Corato Zanarella Amparo - SP Medalha de Ouro
Dimas Macedo de Albuquerque Fortaleza - CE Medalha de Prata
Rodrigo Sanches Angelo São Paulo - SP Medalha de Prata
Rafael Rodrigues Rocha de Melo Fortaleza - CE Medalha de Bronze
Liara Guinsberg São Paulo - SP Medalha de Bronze
Luize Mello D'Urso Viana Rio de Janeiro - RJ Medalha de Bronze
Lincoln de Queiroz Vieira Fortaleza - CE Medalha de Bronze
Tadeu Pires de Matos Belfort Neto Caucaia - CE Menção Honrosa
Pedro Henrique Alencar Costa Fortaleza - CE Menção Honrosa
Michel Rozenberg Zelazny São Paulo - SP Menção Honrosa

EUREKA! N°35, 2012

5
Sociedade Brasileira de Matemática

XXII OLIMPÍADA DE MATEMÁTICA DO CONE SUL


Enunciados e resultado brasileiro

Os estudantes brasileiros tiveram uma participação destacada na Olimpíada


de Matemática do Cone Sul que foi realizada entre os dias 14 e 20 de agosto na
cidade de La Paz, Bolívia. A equipe foi liderada pelos professores Francisco Bruno
Holanda e Régis Barbosa Feitosa, ambos de Fortaleza (CE).

RESULTADOS DA EQUIPE BRASILEIRA

BRA1 Rafael Kazuhiro Miyazaki Medalha de Prata


BRA2 Henrique Gasparini Fiuza do Nascimento Medalha de Prata
BRA3 Rafael Rodrigues Rocha de Melo Medalha de Prata
BRA4 Vinícius Canto Costa Medalha de Prata

PRIMEIRO DIA

PROBLEMA 1
Encontrar todas as ternas de inteiros positivos (x, y, z) tais que
x 2 + y 2 + z 2 = 2011
PROBLEMA 2
Em uma lousa estão escritos os números inteiros positivos de 1 até 4n inclusive.
Em cada momento, Pedro apaga dois números da lousa, a e b, e escreve o número
ab
. Pedro repete o procedimento até que sobre apenas um número.
2a 2 + 2b 2
1
Demostrar que este número será menor que , sem importar quais números Pedro
n
escolha em cada momento.

PROBLEMA 3
Seja ABC um triângulo equilátero. P é um ponto interior tal que a raiz quadrada da
distancia de P até um dos lados seja igual à soma das raízes quadradas das
distancias de P até os outros dois lados. Encontre o lugar geométrico do ponto P.

EUREKA! N°35, 2012

6
Sociedade Brasileira de Matemática

SEGUNDO DIA

PROBLEMA 4
Dizemos que um número de 4 dígitos abcd é equilibrado se a + b = c + d.
Encontre todos os números equilibrados de quatro dígitos que podem ser expresos
como a soma de dois números palíndromos.

PROBLEMA 5
Seja ABC um triângulo e D um ponto sobre o lado AC. Se ∠CBD − ∠ABD = 60° ,
∠BDC = 30° e, além disso AB· BC = BD 2 , encontre as medidas dos ângulos do
triângulo ABC.

PROBLEMA 6
Algumas casas de um tabuleiro Q (2n + 1) × (2n + 1) são pintadas de preto, de
modo que qualquer quadrado 2 × 2 de Q contenha, no máximo, 2 casas pretas.
Achar o número máximo de casas pretas que o tabuleiro pode ter.

EUREKA! N°35, 2012

7
Sociedade Brasileira de Matemática

LII OLIMPÍADA INTERNACIONAL DE MATEMÁTICA (IMO)


Enunciados e resultado brasileiro

O Brasil obteve um ótimo resultado na 52a Olimpíada Internacional de


Matemática (IMO), que aconteceu até o dia 24 de julho na cidade de Amsterdã na
Holanda, conquistando três medalhas de prata e três de bronze. A equipe foi
liderada pelos professores Nicolau Corção Saldanha do Rio de Janeiro (RJ) e
Eduardo Tengan de São Carlos (SP).

RESULTADOS DA EQUIPE BRASILEIRA

BRA1 João Lucas Camelo Sá Medalha de Prata


BRA2 André Macieira Braga Costa Medalha de Prata
BRA3 Henrique Gasparini Fiúza do Nascimento Medalha de Prata
BRA4 Deborah Barbosa Alves Medalha de Bronze
BRA5 Gustavo Lisbôa Empinotti Medalha de Bronze
BRA6 Maria Clara Mendes Silva Medalha de Bronze

PRIMEIRO DIA

PROBLEMA 1
Para qualquer conjunto A = {a1 , a2 , a3 , a4 } de quatro inteiros positivos distintos, a
soma a1 + a2 + a3 + a4 é denotada por s A . Seja nA o número de pares de índices
( i, j ) , com 1 ≤ i < j ≤ 4, para os quais ai + a j divide s A .
Encontre todos os conjuntos A de quatro inteiros positivos para os quais nA alcança
o seu valor máximo.

PROBLEMA 2
Seja S um conjunto finito de dois ou mais pontos do plano. Em S não há três pontos
colineares. Um moinho de vento é um processo que começa com uma reta  que
passa por um único ponto P ∈ S . Roda-se  no sentido dos ponteiros do relógio ao
redor do pivot P até que a reta encontre pela primeira vez um outro ponto de S, que
denotaremos por Q. Com Q como novo pivot, a reta continua sem parar, sendo
sempre o pivot algum ponto de S.
Demonstre que se pode escolher um ponto P ∈ S e uma reta  que passa por P
tais que o moinho de vento resultante usa cada ponto de S como pivot infinitas
vezes.

EUREKA! N°35, 2012

8
Sociedade Brasileira de Matemática

PROBLEMA 3
Seja f :  →  uma função real definida no conjunto dos números reais que
satisfaz
f ( x + y ) ≤ yf ( x ) + f ( f ( x ) )
para quaisquer números reais x e y.
Demonstre que f ( x ) = 0 para todo x ≤ 0.

SEGUNDO DIA

PROBLEMA 4
Seja n um inteiro positivo. Temos uma balança de dois pratos e n pesos cujas
massas são 20 , 21 ,..., 2n−1. Devemos colocar os pesos na balança, um por um, de tal
forma que o prato direito nunca seja mais pesado do que o prato esquerdo. A cada
passo, devemos escolher um dos pesos que ainda não estejam na balança e colocá-
lo sobre o prato esquerdo ou sobre o prato direito, procedendo assim até que todos
os pesos tenham sido colocados nela.
Determine o número de maneiras em que isso pode ser feito.

PROBLEMA 5
Seja f :  → * uma função do conjunto dos inteiros para o conjunto dos inteiros
positivos. Supomos que para quaisquer inteiros m e n, a diferença f ( m ) − f ( n ) é
divisível por f ( m − n ) .
Demonstre que, para todos os inteiros m e n com f ( m ) ≤ f ( n ) , o número f ( n )
é divisível por f ( m ) .

PROBLEMA 6
Seja ABC um triângulo acutângulo cuja circunferência circunscrita é Γ . Seja 
uma reta tangente a Γ e sejam  a ,  b e  c as retas obtidas ao refletir  em relação
às retas BC, CA e AB, respectivamente.
Demonstre que a circunferência circunscrita ao triângulo determinado pelas retas
 a ,  b e  c é tangente à circunferência Γ .

EUREKA! N°35, 2012

9
Sociedade Brasileira de Matemática

LIII OLIMPÍADA INTERNACIONAL DE MATEMÁTICA (IMO)


Enunciados e resultado brasileiro

O estudante Rodrigo Sanches Ângelo (SP) conquistou medalha de ouro na


53ª Olimpíada Internacional de Matemática (IMO). O evento foi realizado entre os
dias 4 e 16 de julho na cidade de Mar del Plata, na Argentina reunindo 551
estudantes de 100 países. Além da medalha de ouro o Brasil conquistou uma
medalha de prata e três de bronze. Com este resultado o Brasil se classificou em
19º lugar entre os países participantes.

A equipe foi liderada pelos professores Luciano Guimarães Castro, do Rio de


Janeiro (RJ) e Carlos Yuzo Shine, de São Paulo (SP).

RESULTADOS DA EQUIPE BRASILEIRA

BRA1 Rodrigo Sanches Angelo Medalha de Ouro


BRA2 João Lucas Camelo Sá Medalha de Prata
BRA3 Franco Matheus de Alencar Severo Medalha de Bronze
BRA4 Henrique Gasparini Fiúza do Nascimento Medalha de Bronze
BRA5 Rafael Kazuhiro Miyazaki Medalha de Bronze
BRA6 Maria Clara Mendes Silva Menção Honrosa

PRIMEIRO DIA

PROBLEMA 1
Dado um triângulo ABC, o ponto J é o centro da circunferência ex-inscrita oposta
ao vértice A. Esta circunferência ex-inscrita é tangente ao lado BC em M, e às retas
AB e AC em K e L, respectivamente. As retas LM e BJ intersectam-se em F, e as
retas KM e CJ intersectam-se em G. Seja S o ponto de intersecção das retas AF e
BC, e seja T o ponto de intersecção das retas AG e BC.
Prove que M é o ponto médio de ST.
(A circunsferência ex-inscrita de ABC oposta ao vértice A é a circunferência
tangente ao segmento BC, ao prolongamento do segmento AB no sentido de A para
B e ao prolongamento do segmento AC no sentido de A para C.)

PROBLEMA 2
Seja n ≥ 3 um inteiro e sejam a2 , a3 ,..., an números reais positivos tais que
a2 a3 ...an = 1. Prove que

EUREKA! N°35, 2012

10
Sociedade Brasileira de Matemática

2 3 n
(1 + a2 ) (1 + a3 ) ...(1 + an ) > nn .

PROBLEMA 3
O desafio do mentiroso é um jogo para dois jogadores A e B. As regras do jogo
dependem de dois inteiros positivos k e n conhecidos por ambos os jogadores.

No início do jogo, o jogador A escolhe inteiros x e N com 1 ≤ x ≤ N . O jogador A


mantém x em segredo, e diz a B o verdadeiro valor de N. Em seguida, o jogador B
tenta obter informação sobre x fazendo perguntas a A da seguinte maneira: em cada
pergunta, B especifica um conjunto arbitrário S de inteiros positivos (que pode ser
um dos especificados em alguma pergunta anterior), e pergunta a A se x pertence a
S. O jogador B pode fazer tantas perguntas desse tipo como deseje. Depois de cada
pergunta, o jogador A deve responder imediatamente com sim ou não, mas pode
mentir tantas vezes como queira. A única restrição é que dadas quaisquer k + 1
respostas consecutivas, pelo menos uma deve ser verdadeira.

Quando B tenha feito tantas perguntas como pretenda, deve especificar um


conjunto X com no máximo n inteiros positivos. Se x pertencer a X então ganha B;
caso contrário, B perde. Prove que:

1. Se n ≥ 2 k , então B pode garantir a sua vitória.


2. Para todo k suficientemente grande, existe um inteiro n ≥ 1,99k tal que B
não pode garantir a sua vitória.

SEGUNDO DIA

PROBLEMA 4
Determine todas funções f :  →  tais que, para todos os inteiros a, b, c que
satisfazem a + b + c = 0, a seguinte igualdade é verdadeira:

2 2 2
f ( a ) + f (b ) + f (c ) = 2 f ( a ) f (b ) + 2 f (b ) f ( c ) + 2 f (c ) f ( a ).

(  designa o conjunto dos números inteiros.)

PROBLEMA 5
Seja ABC um triângulo tal que ∠BCA = 90° , e seja D o pé da altura relativa a C.
Seja X um ponto no interior do segmento CD. Seja K o ponto do segmento AX tal

EUREKA! N°35, 2012

11
Sociedade Brasileira de Matemática

que BK = BC. Analogamente, seja L o ponto do segmento BX tal que AL = AC.


Seja M o ponto de intersecção de AL com BK.
Prove que MK = ML.

PROBLEMA 6
Determine todos os inteiros positivos n para os quais existem inteiros não negativos
a1 , a2 ,..., an tais que

1 1 1 1 2 n
+ a2 + ... + an = a1 + a2 + ... + an = 1.
2 a1
2 2 3 3 2

EUREKA! N°35, 2012

12
Sociedade Brasileira de Matemática

II OLIMPÍADA DE MATEMÁTICA DA COMUNIDADE DOS PAÍSES DE


LÍNGUA PORTUGUESA
Enunciados e resultado brasileiro

O Brasil conquistou duas medalhas de ouro e duas de prata na 2ª


Olimpíada de Matemática da Comunidade dos Países de Língua Portuguesa,
realizada de 20 a 28 de julho, na cidade de Salvador (BA). Com este resultado o
país ficou pelo segundo ano consecutivo com a primeira posição na classificação
geral, com 160 pontos, seguido pela equipe de Portugal, que obteve 149 pontos. A
equipe foi liderada pelos professores Marcelo Mendes de Oliveira, de Fortaleza
(CE) e Guilherme Philippe Figueiredo, de São Paulo (SP).

RESULTADOS DA EQUIPE BRASILEIRA


BRA1 Daniel Santana Rocha Medalha de Ouro
BRA2 Murilo Corato Zanarella Medalha de Ouro
BRA3 Daniel Lima Braga Medalha de Prata
BRA4 Victor Oliveira Reis Medalha de Prata

PRIMEIRO DIA

PROBLEMA 1
Arnaldo e Bernaldo treinam para uma maratona ao longo de uma pista circular, a
qual possui em seu centro um mastro com uma bandeira hasteada. Arnaldo corre
mais rápido que Bernaldo, de modo que a cada 30 minutos de corrida, enquanto
Arnaldo dá 15 voltas na pista, Bernaldo só consegue dar 10 voltas completas.
Arnaldo e Bernaldo partiram no mesmo instante da linha e correram com
velocidades constantes, ambos no mesmo sentido. Entre o minuto 1 o o minuto 61
da corrida, quantas vezes Arnaldo, Bernaldo e o mastro ficaram colineares?

PROBLEMA 2
Maria dispõe de um tabuleiro de tamanho n × n , inicialmente com todas as casas
pintadas de cor branca. Maria decide pintar algumas casas do tabuleiro de preto,
formando um mosaico, como mostra a figura abaixo, da seguinte maneira: ela pinta
todas as casas do bordo do tabuleiro de preto, e em seguida deixa pintadas de
branco as casas do bordo do tabuleiro que ainda não foi pintado. Então, pinta
novamente de preto as casas do bordo do próximo tabuleiro restante, e assim
sucessivamente.

EUREKA! N°35, 2012

13
Sociedade Brasileira de Matemática

a) Determine um valor de n para que o número de casas pretas seja igual a


200.
b) Determine o menor valor de n para que o número de casas pretas seja maior que
2012.

PROBLEMA 3
Seja n um inteiro positivo. Abigail e Berenice disputam o seguinte jogo, que utiliza
n bolas numeradas de 1 até n. Elas dispõem de duas caixas, rotuladas com os
símbolos ∏ e ∑ , respectivamente.
Na sua vez, cada jogador escolhe uma bola e a coloca em uma das caixas. Ao final,
os números das bolas que estão na caixa ∏ são multiplicados, obtendo-se um
valor P e os números das bolas que estão na caixa ∑ são somados, obtendo-se um
valor S (se a caixa ∏ estiver vazia, então adotamos P = 1; se a caixa
∑ estiver vazia, adotamos S = 0). Elas jogam alternadamente, iniciando por
Abigail, até que não haja mais bolas fora das caixas.
Se o valor de P + S for par, Abigail ganha. Caso contrário, Berenice ganha.

a) Qual jogador possui estratégia vencedora para n = 6?


b) Qual jogador possui estratégia vencedora para n = 2012?

SEGUNDO DIA

PROBLEMA 4
Uma formiga decide passear sobre o perímetro de um triângulo ABC. A formiga
pode começar em qualquer vértice. Sempre que a formiga está num vertice, ela
escolhe um dos vértices adjacentes e caminha diretamente (em linha reta) até o
vértice escolhido.

a) De quantos modos a formiga pode passear visitando cada vértice


exatamente duas vezes?

EUREKA! N°35, 2012

14
Sociedade Brasileira de Matemática

b) De quantos modos a formiga pode passear visitando cada vérice


exatamente três vezes?

Observação: Em cada item, considere que o vértice inicial é visitado.

PROBLEMA 5
Arnaldo e Bernaldo estão brincando no quadro da sala de aula da seguinte maneira:
eles escrevem inicialmente no quadro um número inteiro positivo n. Então,
alternadamente, começando com Arnaldo, apagam o número que está no quadro e
escrevem um novo número que pode ser:

• o que acabou de ser apagado menos a maior potência de 2 (com expoente


inteiro não-negativo) menor do que ou igual ao número apagado;
• o que acabou de ser apagado dividido por 2, caso o número apagado seja
par.

Vence a brincadeira quem obtiver primeiro o número zero.

a) Determine qual dos jogadores possui uma estratégia vencedora para n = 40


e descreva-a.
b) Determine qual dos jogadores possui uma estratégia vencedora para n =
2012 e descreva-a.

PROBLEMA 6
Um quadrilátero ABCD está inscrito numa circunferência de centro O. Sabe-se que
as diagonais AC e BD são perpendiculares. Sobre cada um dos lados construímos
semicírculos, externamente, como mostra a figura.

a) Mostre que os triângulos AOB e COD têm a mesma área.


b) Se AC = 8 cm e BD = 6 cm, determine a área da região sombreada.

EUREKA! N°35, 2012

15
Sociedade Brasileira de Matemática

XXVI OLIMPÍADA IBERO–AMERICANA DE MATEMÁTICA


Enunciados e resultado brasileiro

A equipe brasileira formada por quatro estudantes do ensino médio


conquistou três medalhas de prata e uma de bronze na 26ª Olimpíada Ibero-
americana de Matemática (OIM), realizada entre os dias 23 e 30 de setembro na
cidade de San José, Costa Rica. A equipe foi liderada pelos professores Onofre
Campos, de Fortaleza (CE) e Carlos Moreira, do Rio de Janeiro (RJ).

RESULTADOS DA EQUIPE BRASILEIRA


BRA1 Henrique Gasparini Fiúza do Nascimento Medalha de Prata
BRA2 Maria Clara Mendes Silva Medalha de Prata
BRA3 João Lucas Camelo Sá Medalha de Prata
BRA4 André Macieira Braga Costa Medalha de Bronze

PRIMEIRO DIA

PROBLEMA 1
No quadro está escrito o número 2. Ana e Bruno jogam alternadamente,
começando por Ana, da seguinte maneira: cada um na sua vez substitui o número
escrito pelo que se obtém multiplicando-o por 2, ou por 3, ou somando-lhe 1. O
primeiro que obtenha um resultado maior ou igual a 2011 ganha. Mostre que um
dos dois tem uma estratégia vencedora e descreva-a.

PROBLEMA 2
Encontrar todos os inteiros positivos n para os quais existem três números inteiros
não nulos x, y, z tais que
1 1 1 1
x+ y+z =0 e + + = .
x y z n

PROBLEMA 3
Seja ABC um triângulo acutângulo e X, Y, Z os pontos da tangência de sua
circunferência inscrita com os lados BC, CA, AB, respectivamente. Sejam
C1 , C2 , C3 circunferências com cordas YZ, ZX, XY, respectivamente, de maneira que
C1 e C2 se intersectem sobre a reta CZ e que C1 e C3 se intersectem sobre a reta
BY. Suponha que C1 intersecta XY em J e intersecta ZX em M; que C2 intersecta YZ

EUREKA! N°35, 2012

16
Sociedade Brasileira de Matemática

em L e intersecta XY em I; e que C3 intersecta YZ em K e intersecta ZX em N.


Demonstrar que I, J, K, L, M, N estão sobre uma mesma circunferência.

SEGUNDO DIA

PROBLEMA 4
Seja ABC um triângulo acutângulo e O o seu circuncentro. Sejam P e Q pontos tais
que BOAP e COPQ são paralelogramos. Demonstrar que Q é o ortocentro de ABC.

PROBLEMA 5
Sejam x1 ,..., xn números reais positivos. Demonstrar que existem a1 ,..., an ∈ {−1,1}
tais que
2
a1 x12 + ... + an xn2 ≥ ( a1 x1 + ... + an xn ) .

PROBLEMA 6
Sejam k ≥ 2 e n inteiros positivos. Temos kn caixas em linha reta e em cada caixa
coloca-se uma pedra de uma de k cores diferentes de tal forma que haja n pedras de
cada cor. Uma troca consiste em trocar de caixa duas pedras que se encontrem em
caixas adjacentes. Encontrar o menor inteiro positivo m para o qual sempre é
possível conseguir mediante m trocas que as n pedras de cada cor fiquem em caixas
consecutivas se:
a) n é par.
b) n é ímpar e k = 3.

EUREKA! N°35, 2012

17
Sociedade Brasileira de Matemática

PARIDADE DO NÚMERO BINOMIAL MÉDIO


J.C.S. de Miranda, USP/SP
Iesus C. Diniz, UFRN

♦ Nível Avançado

1. INTRODUÇÃO

Apresentaremos duas soluções ao problema da paridade do número binomial


 2n 
médio, isto é, mostrar que 2   para todo n ∈ * . Uma delas é baseada em um
 n
argumento combinatório, enquanto que a outra será obtida de maneira aritmética.
Outras soluções deste problema são dadas em [6] e [8]. Ademais, como
consequência dos métodos de solução dados nas seções 2 e 3, respectivamente,
resolveremos os problemas olímpicos internacionais dados em [5] e [4].
O número binomial médio aparece em várias áreas da matemática, por
exemplo: teoria dos números [1], análise [2] e combinatória [7].
A segunda solução mostra quão interessantes podem ser as resoluções de
determinados problemas quando utilizamos argumentos combinatórios para
estabelecê-las. A primeira é obtida a partir de uma conveniente fatoração do
número binomial médio observando-se o expoente do fator 2.
Por fim, para exemplificar o uso de argumentos combinatórios na solução de
problemas apresentamos uma série de exercícios na seção 5; ver também em [3] e
[8]. Além disso, calculamos de maneira aproximada a série cujos termos são os
recíprocos dos números binomiais médios.

2. SOLUÇÃO ARITMÉTICA

Indicaremos por f ( n ) , para todo n ∈ * , o expoente do fator 2 na decomposição


 2n 
de   em fatores primos, isto é, f ( n ) é tal que:
 n
 2n   2n 
2 f ( n )   e 2 f ( n ) +1 |   (1)
 n  n

Note que

EUREKA! N°35, 2012

18
Sociedade Brasileira de Matemática

 2n   2 n  n
  =   ∏ ( 2i − 1) . (2)
 n   n!  i =1
Pois,

 2n  ( 2n )( 2n − 1)( 2n − 2 ) ...2.1 ( 2n )( 2n − 2 ) ...2  ( 2n − 1)( 2n − 3) ...1


 = = =
 n n !n ! n !n !

 2 ( n ) 2 ( n − 1) ...2 (1)  ( 2n − 1)( 2n − 3) ...1 =  2 n ! ( 2n − 1)( 2n − 3) ...1 =


n

n !n ! n !n !
 2n   2n  n
  ( 2n − 1)( 2n − 3) ...1 =   ∏ ( 2i − 1) .
 n!   n!  i =1
Seja k ∈  o máximo expoente de 2 tal que 2k ≤ n, isto é, 2k ≤ n < 2k +1.
Assim,
k ≤ log 2 n < k + 1, isto é, k = log 2 n  .
n
Note que em n! há k =  j  fatores que são múltiplos de 2 j , para todo
2 
j ∈ {1,..., k } . Logo o expoente do fator 2 em n! é dado por,
 log n
n  n  k  n   2  n 
2 + ... +  2k  ∑  2 j  ∑  2 j . (3)
= =
    j =1   j =1  
Portanto, da definição dada em (1) e das equações (2) e (3) temos:
log 2 n 
n
f ( n ) = n − ∑  j  (4)
j =1  2 

e podemos escrever
 2n  f (n)
  = 2 c, em que 2 | c. (5)
  n
Agora,
 log 2 n  log n
 2 
n ∞ n n
∑ j =1
2j  ∑ 2j
<
  j =1
= n ⇒ ∑  j  ≤ n − 1,
j =1  2 

donde, da equação (4), resulta que


f ( n ) ≥ n − ( n − 1) = 1.
 2n 
Logo, da equação (5), 2   pois f ( n ) ≥ 1.
 n

EUREKA! N°35, 2012

19
Sociedade Brasileira de Matemática

3. SOLUÇÃO POR ARGUMENTO COMBINATÓRIO

 2n 
Teorema 1 Para todo n ∈ * ,2  .
 n
Prova: Para todo n ∈ * , o número de partições não-ordenadas de um conjunto de 2n
 2n  n 
  
elementos em dois subconjuntos de n elementos cada, é dado por    .
n n
2!
Claramente, o referido número de partições é um inteiro positivo, donde segue que
 2n  n 
  
 n  n  ∈ * , e portanto, 2  2n  .
 
2!  n

4. COMENTÁRIOS

A solução por argumento combinatório é claramente mais objetiva; por outro lado,
da solução aritmética podem ser obtidos os seguintes resultados:
O expoente da maior potência de um primo p que divide n! é

 log p n 
 
 n  ∞  n 
∑  p j  = ∑  p j , (6)
j =1   j =1  

pois o argumento do qual se obtém a equação (3) pode ser repetido para um primo
p qualquer. Este resultado foi provado por Legendre em 1808.
k
O expoente r da maior potência, de um número não nulo m = ∏ piαi que divide n!,
i =1

isto é, r tal que m r n ! e mr +1 / n !, é o maior valor de r que satisfaz às


desigualdades rα i ≤ ri para todo i ∈ {1,..., k} , nas quais ri é o expoente da potência
máxima de pi em n!. Assim,
r   ∞  n  
r = min  i  = min ∑  j  / α i  . (7)
1≤ i ≤ k α
 i  1≤i ≤ k  j =1  pi  

EUREKA! N°35, 2012

20
Sociedade Brasileira de Matemática

Na prova da paridade do número binomial médio dada em [8], exercício 39 página


235, é usado um argumento combinatório juntamente com propriedades aritméticas
da fatoração do número binomial médio. Ei-la:

 2n  ( 2n )! 2n ⋅ ( 2n − 1)! ( 2n − 1)! = 2  2n − 1 é par.


 = = =2
 n  n!n! n ⋅ ( n − 1)!n! ( n − 1)!n!  n 
5. EXERCÍCIOS

Prove via argumentos combinatórios que:

 n + m   n  m   n  m   n  m 
a)   =    +    + ... +   
 r   0  r  1  r − 1  r  0 

Solução: Consideremos um grupo de n + m pessoas em que há n homens e m


mulheres.
n + m
O número de subconjuntos de tamanho r do grupo das n + m pessoas é  .
 r 
Por outro lado, particionando os subconjuntos em relação à quantidade de homens
 n  m 
e mulheres neles presentes, temos um total de    deles em que não há
 0  r 
 n  m 
nenhum homem presente e r mulheres,    em que há 1 homem presente e
 1  r − 1
 n  m 
r − 1 mulheres ...    em que há r homens presentes e nenhuma mulher.
 r  0 

n   n 
b)   =  
 p n − p
2
 2n  n  n 
c)   = ∑   (Fórmula de Lagrange)
 n  k =0  k 

Solução: Do item a com m e r iguais a n e do uso da identidade dada em b, segue-se


o resultado, pois:

EUREKA! N°35, 2012

21
Sociedade Brasileira de Matemática

 n + n   n  n   n  n   n  n 
  =    +    + ... +   
 n   0  n  1  n − 1  n  0 

2
 n  n   n  n   n  n  n  n 
=    +    + ... +    = ∑  
 0  0   1 1   n  n  k =0  k 

n  n   n − 1
d) k   = ( n − k + 1)   = n 
k  k − 1  k − 1

Solução: Considere um grupo de n pessoas pertencentes a uma empresa, dentre as


quais k constituem uma diretoria em que há um único diretor-presidente e todos os
demais membros têm iguais poderes. Podemos configurar a diretoria das três
seguintes formas, em que o total de possibilidades de cada uma delas representa,
respectivamente, cada um dos termos das igualdades dadas no item d.

1. Escolher a diretoria e depois o diretor-presidente;

2. Escolher os k – 1 membros de iguais poderes pertencentes a diretoria e depois o


diretor-presidente.

3. Escolher o diretor-presidente entre as n pessoas pertencentes a empresa e depois


os k – 1 indivíduos que completarão a diretoria.

 n  n  i −1 
e)   = ∑   (Identidade Combinatória de Fermat)
 k  i = k  k − 1

Solução: O lado esquerdo da igualdade é o número de amostras não ordenadas de


tamanho k, extraídas sem reposição de uma população de tamanho n, que sem
perda de generalidade suporemos numerada de 1 a n. Por outro lado, se
considerarmos a partição de todas essas amostras com relação à presença de seu
 k − 1  k   n − 1 
maior elemento, temos que existem  ,  ,...,   de tais amostras
 k − 1  k − 1  k − 1
cujo maior elemento é respectivamente k, k + 1,...,n.

n
n
f) ∑ k  k  = n2 n −1

k =1  

EUREKA! N°35, 2012

22
Sociedade Brasileira de Matemática

Solução: Considere uma empresa formada por n funcionários dentre os quais será
formada uma diretoria de no máximo n pessoas, a qual é constituída de um único
diretor-presidente escolhido entre os membros pertencentes à diretoria.
O lado esquerdo da igualdade representa o total de configurações das diretorias
possíveis, obtidas primeiramente a partir da determinação dos k membros
pertencentes à diretoria, seguindo-se da determinação de seu diretor-presidente
entre os k membros da diretoria.
O lado direito da igualdade representa o total de configurações das diretorias
possíveis, obtidas primeiramente escolhendo-se o diretor-presidente e em seguida
determinando-se para cada um dos demais n – 1 membros se pertencerão ou não à
diretoria.

n
n
g) ∑k 2 n −2
  = 2 n ( n + 1)
k =1  
k
n
n
h) ∑ k 3   = 2n −3 n 2 ( n + 3)
k =1 k
n
 n  j   n 
i) ∑  j  i  =  i  2 n −i

j =i     

j) (“British Mathematical Olympiad” – [4]) Para todo ( m, n ) ∈  + ×  +


n +1 m +1 2
( m!) ( n!) {( mn )!}
Solução: Sejam P1 e P2 , respectivamente, o número de maneiras de dividir mn
pessoas em n grupos de m pessoas e m grupos de n pessoas. Tem-se que:

P1 =
( mn )! ∈ + , P = ( mn )! ∈ + e, portanto,
n 2 m
( m!) n! ( n!) m!
2

P1 ⋅ P2 =
{( mn )!} n +1 m +1
∈  + , isto é, ( m!) ( n !) {( mn )!}
2
n +1 m +1
( m!) ( n!)
k) (XIV Olimpíada Internacional de Matemática – [5]) Quaisquer que sejam os
inteiros naturais m e n, temos:
( 2m )!( 2n )!
∈ .
m!n!( m + n )!

EUREKA! N°35, 2012

23
Sociedade Brasileira de Matemática

Solução: Seja p um primo arbitrário. A maior potência de p que divide o numerador


da fração acima é p a onde
∞ 
 2m   2n  
a = ∑  i  +  i  
i =1   p   p 
ao passo que a maior potência de p que divide seu denominador é p b com
∞ 
 m   n  m + n
b = ∑   i  +  i  +  i  .
i =1   p   p   p 
Basta mostrar que a ≥ b.
Para qualquer k inteiro maior ou igual a 1 podemos escrever m = m1k + r e
n = n1k + s em que 0 ≤ r ≤ k − 1,0 ≤ s ≤ k − 1, m1 e n1 são inteiros.
Assim,
 2m   2 n   2r   2 s 
 k  +  k  = 2m1 + 2n1 +  k  +  k  (8)
       
e
m n m + n r   s  r + s
 k  +  k  +  k  = m1 + n1 + ( m1 + n1 ) +  k  +  k  +  k  (9)
           
r + s
= 2m1 + 2n1 +  .
 k 
Como
 r + s   2 max {r , s}   2r   2s 
 k ≤  ≤   +  ,
   k  k  k 
por (8) e (9) temos
 2m   2 n   m   n   m + n 
 k  +  k  ≥  k  + k  +  k 
         

para todo k ≥ 1 e, em particualr, para k da forma k = pi , donde segue


imediatamente que a ≥ b.


1
l) Dada a série S = ∑ :
k =1  2k 
 
 k 
l.1) mostre que a série é convergente, isto é, S < ∞;

EUREKA! N°35, 2012

24
Sociedade Brasileira de Matemática

Solução: A partir da Fórmula de Lagrange, apresentada no item c) desta seção, tem-


se que
2 2
 2n  n  n   n 
  = ∑   >   = n , (10)
2

  j =0    
n j 1

e portanto, segue de (10) que



1 ∞
1 π2
∑  2n  < ∑ n
n =1 n =1
2
=
6
.
 
 n
Ou ainda, da equação (2) tem-se que, para n > 1,
 2n  1 1 1 1
  > 2 , donde segue que 2n < n , e se n = 1, resulta que 2n = n .
n

 n   2   2
   
 n  n
Portanto

1 ∞
1
∑n =1  2n 
< ∑
n =1 2
n
= 1.
 
 n
2 8
l.2) prove também que <S< ;
3 9
Solução: A partir da equação (2) tem-se que
n  2n 
∏ ( 2i − 1)  n 
i =1
=  n  , logo, para n > 1,
n! 2
 2n 
2 n −1
1.2.4... ( 2n − 2 )  n  1.4.6... ( 2n )
= < n < = 2n −1. (11)
n 1.2.3...n 2 1.2.3...n

De (11) resulta que


1 1 n
2 n −1
< < 2 n −1 ,
2  2n  2
 
 n
e portanto,

EUREKA! N°35, 2012

25
Sociedade Brasileira de Matemática


1 ∞
1 n ∞

n =1
2 n −1 ∑2
n =1  2n 
< ∑
n =1 2
2 n −1
<∑
. (12)
 
n
Os limitantes são obtidos a partir do cálculo das somas dadas em (12).

1 ∞
1  14  2
∑2 2 n −1
= 2∑
n =1 2
2n
= 2  1 
= e
n =1 1− 4  3

n ∞
n  1   1 1 1 1 1 
∑2 2 n −1
= 2∑
n =1 2
2n
= 2   2  +  4 + 4  +  6 + 6 + 6  + ... =
n =1  2   2 2  2 2 2  
 1 1 1  1 1  1 1 
2   2 + 4 + 6 + ... +  4 + 6 + ... +  6 + 8 + ...  =
 2 2 2  2 2  2 2 
 2  8  1
1 1 1
4 6 1 1 
2  2 1 + 2 1 + 2 1 + ... =  2 + 4 + 6 + ... =
1
 22− 1 − 22
1 − 22  3  2 2 2 

8  22  8
1

 = .
3 1 − 212  9

l.3) Finalmente, prove que 0,73639 < S < 0,73641.

Solução: Das desigualdades estabelecidas em (12), resulta que ∀k ∈ *


k
1 ∞
1 ∞
1 k
1 ∞
n
∑  2n  + ∑
n =1 n = k +1 2 2 n −1
<∑
n =1  2n 
<∑
n =1  2n 
+ ∑ 2 n −1 . (13)
n = k +1 2
     
 n  n  n

Analogamente ao que foi feito no item 12, temos que:


1 1 1 1 1 

n = k +1 2 2 n −1
=
2 2 k +1
+
2 2 k +3
+ ... =  2 k −1  (14)
3 2 
e também


n  k 4  1 

n = k +1 2 2 n −1
=  +   2 k −1  . (15)
 3 9 2 
Pois,

EUREKA! N°35, 2012

26
Sociedade Brasileira de Matemática


n k +1 k + 2 k + 3

n = k +1 2 2 n −1
= + +
22 k +1 22 k + 3 22 k + 5
... =

1 1   1 1   1 1
( k + 1)  2 k +1
+ 2 k +3

+ ... +  2 k + 3 + 2 k + 5 + ... +  2 k + 5 + 2 k + 7 + ... + ... =
2 2  2 2  2 2 
 1   1   1 
 2 k +1   2 k +3   2 k +5 
( k + 1)  2 1  +  2 1  +  2 1  + ... =
1−   1−   1− 
 4  4  4
4 1 4 1 1
( k + 1)  2k +1  +  2 k +3 + 2 k +5 + ... =
3 2  3 2 2 
 1 
1  1  4  22 k + 3 
( k + 1)  2 k −1  +  1  =
3 2  3 1− 
 4
1 1 1 1
( k + 1)  2 k −1  +  2k −1  =
3 2  9 2 
 k 4  1 
 +   2 k −1  .
 3 9 2 
De (14) e (15) resulta que
k
1 1 1  ∞ 1 k
1  k 4  1 

n =1  2 n 
+  2 k −1  < ∑
3  2  n =1  2 n 
<∑
n =1  2 n 
+  +   2 k −1 . (16)
 3 9 2 
     
 n  n  n
Para estabelecermos o resultado no item l.3, é suficiente tomarmos k = 10 em (16).


1
Observação: A série ∑  2n 
n =1
pode ser estimada por
 
 n
k
1  3k + 7  1   3k + 1  1 
∑n =1  2n 
+ 
 9  2 
2k 
com um erro de no máximo   2 k  .
 9  2 
 
 n
Assim, se quisermos um erro não superior a e em nossa estimativa da série, é
  3m + 1  1  
suficiente escolhermos k = min  m ∈  :   2 m  ≤ ε  .
  9  2  

EUREKA! N°35, 2012

27
Sociedade Brasileira de Matemática

Terminamos com um problema proposto.

Problema (Harvey Abbott e Murray Klamkin)

Sabe-se que
( 3m )!( 3n )! ( 4m )!( 4n )!
, e
( 5m )!( 5n )!
m!n!( m + n )!( n + m )! m!n!( 2m + n )!( 2n + m )! m!n!( 3m + n )!( 3n + m )!
são todos inteiros para quaisquer inteiros positivos m, n.

a) Encontre inteiros positivos m, n tais que I ( m, n ) =


( 6m )!( 6n )! não
m!n!( 4m + n )!( 4n + m )!
seja inteiro.

b) Seja A o conjunto dos pares (m, n) com m ≤ n, para os quais I ( m, n ) não é


inteiro, e seja A( x) o número de pares em A para os quais 1 ≤ m ≤ n ≤ x.
Prove que A contém uma proporção positiva dos pares de naturais, no sentido de
que existem uma constante c > 0 e um inteiro positivo n0 tal que, para todo
x ≥ n0 , A ( x ) ≥ c ⋅ x 2 .

Obs.: Para o item b) pode-se usar o chamado Postulado de Bertrand, um teorema


segundo o qual, para todo n ≥ 2, existe um primo p com n < p < 2n.

Referências

[1] Berend, D.; Harmse, J.E. On some arithmetical properties of middle binomial coefficients. Acta
arith. 84 (1998), no. 1, 31-41.
[2] Bloom, D. M.; Swanson, C.N; Schilling, K. A. Convolution of Middle Binomial Coefficients;
10921 The American Mathematical Monthly Vol 110 (Dec., 2003), no. 10, 958-959.
[3] Ross, S. M. A first course in probability.8th. ed. Upper Saddle River, N.J.: Prentice Hall, 2008.
[4] British Mathematical Olympiad -16/02/2005 – Exercice 4 – item b-
http://www.bmoc.maths.org/home/bmolot.pdf
[5] XIV Olimpíada Internacional de Matemática. Cracóvia, Polônia, 1972 – Exercice 3 –
http://imo.math.ca/Exams/1972imo.html
[6] Gardiner, A. Four Problems on Prime Power Divisibility. The American Mathematical Monthly
Vol 95 (Dec., 1998), no. 10, 926-931.
[7] Knuth D. E.; Vardi I.; Richberg R. The asymptotic expansion of the middle binomial coefficient.
The Amer. Math. Monthly Vol 97 (1990), 626-630.
[8] Morgado, A. C. O.; Carvalho, J. B. P.; Carvalho, P. C. P.; Fernandez, P. Análise combinatória e
probabilidade. 9. ed. Rio de Janeiro: Sociedade Brasileira de Matemática, 2006. (Coleção do
Professor de Matemática).

EUREKA! N°35, 2012

28
Sociedade Brasileira de Matemática

PONTO MÉDIO LEMBRA? OUTRO PONTO MÉDIO! DOIS PONTOS


MÉDIOS LEMBRAM? BASE MÉDIA!
Cícero Thiago Magalhães

♦ Nível Iniciante

Propriedade 1 Num triângulo retângulo ABC, a mediana BM relativa à hipotenusa


mede metade da hipotenusa AC.

A D

B C

Prova
Seja D o ponto sobre o prolongamento da mediana BM tal que BM = MD. Os
triângulos AMB e CMD são congruentes, pelo caso LAL. Daí, AB = CD e
∠BAM = ∠DCM , ou seja, AB e CD são segmentos iguais e paralelos e portanto

∠ABC = ∠DCB = 90°.

Assim, os triângulos ABC e DCB são congruentes, pelo caso LAL, e portanto
AC
BD = AC ⇒ 2 ⋅ BM = AC ⇒ BM = .
2
Definição 1 Uma base média de um triângulo é um segmento que une os pontos
médios de dois de seus lados.

Assim, todo triângulo possui exatamente três bases médias.

Propriedade 2 Sejam ABC um triângulo e M, N os pontos médios dos lados AB, AC,
respectivamente. Então
BC
MN BC e MN = .
2

EUREKA! N°35, 2012

29
Sociedade Brasileira de Matemática

M N P

B C

Prova
Inicialmente, prolonguemos a base média MN até um ponto P tal que MN = NP.
Em seguida, construímos o triângulo CNP. Note que os triângulos ANM e CNP são
congruentes, pelo caso LAL. Daí, CP = AM e ∠MAN = ∠PCN e portanto

CP AM ⇒ CP BM .
Assim, MBCP é um paralelogramo, pois CP e BM são segmentos paralelos e
iguais. Mas então MP BC e

BC
MP = BC ⇒ 2 MN = BC ⇒ MN = .
2
Definição 2 A base média de um trapézio é o segmento que une os pontos médios de
seus lados não paralelos.

Propriedade 3 Sejam ABCD um trapézio de bases AB e CD, e sejam M e N os pontos


médios dos lados BC e AD, respectivamente. Então,

AB + CD
MN AB, MN CD e MN = .
2

Prova
Inicialmente, prolonguemos AM até encontrar DC no ponto E. É fácil ver que
∆ABM ≡ ∆CME ( ALA) ⇒ AM = ME.
Portanto, MN é base média do triângulo ADE. Assim,

EUREKA! N°35, 2012

30
Sociedade Brasileira de Matemática

DE
MN DE ⇒ MN DC , e MN = .
2
DC + CE DC + AB
Finalmente, MN = = .
2 2

Problema 1 (OBM)
Considere um triângulo acutângulo ABC com ∠BAC = 30°. Sejam B1 , C1 os pés
das alturas relativas aos lados AC, AB, respectivamente, e B2 , C2 os pontos médios
dos lados AC, AB, respectivamente. Mostre que os segmentos B1C2 e B2 C1 são
perpendiculares.

Solução
Seja O a interseção entre B1C2 e B2 C1 . O segmento B1C2 é uma mediana do
triângulo retângulo AB1 B e portanto

AC2 = B1C2 e ∠C2 B1 A = ∠BAB1 = 30°.


Analogamente, AC1 B2 = 30°. Daí,

∠BC2 B1 = ∠C2 B1 A + ∠BAB1 = 60°


e portanto
∠C1OC2 = 180° − ∠BC2 B1 − ∠AC1 B2 = 90°.

Problema 2 Sejam ABC um triângulo e M o ponto médio do lado BC. Se D, E são os


pés das alturas relativas aos lados AC, AB, respectivamente, prove que ME = MD.

EUREKA! N°35, 2012

31
Sociedade Brasileira de Matemática

Solução

Note que ME é mediana relativa à hipotenusa do triângulo BEC. Daí,

ME = BM = CM
e, analogamente,
MD = BM = CM .
Assim, ME = MD.

Comentários M é o centro da circunferência circunscrita ao quadrilátero inscritível


BCDE.

Problema 3 Dado um quadrilátero ABCD, prove que os pontos médios M, N, P, Q


dos lados AB, BC, CD, DA formam um paralelogramo.

Solução

Temos
• Triângulo ABC : MN AC e MN = AC 2.
• Triângulo DAC : PQ AC e PQ = AC 2.
Assim, MN PQ e MN = PQ, isto é, MNPQ é paralelogramo.

Problema 4 Sejam ABC um triângulo e M o ponto médio de BC. Se AM = BM = CM,


prove que ∠BAC = 90°.

EUREKA! N°35, 2012

32
Sociedade Brasileira de Matemática

Problema 5 (Torneio das Cidades) Sejam ABCD um paralelogramo, M o ponto


médio de CD e H o pé da perpendicular baixada de B a AM. Prove que BCH é um
triângulo isósceles.

Problema 6 Em um triângulo ABC, retângulo em A e isósceles, sejam D um ponto no


lado AC ( A ≠ D ≠ C ) e E o ponto no prolongamento de BA tal que o triângulo
ADE é isósceles. Se P é o ponto médio de BD, R o ponto médio de CE e Q a
intersecção entre ED e BC, prove que o quadrilátero ARQP é um quadrado.

Problema 7 No triângulo acutângulo ABC, CF é altura e BM é mediana. Sabendo que


BM = CF e ∠MBC = ∠FCA , prove que o triângulo ABC é equilátero.

Problema 8 Seja ABCD um quadrilátero convexo tal que ∠ABC = ∠CDA = 90° e
∠BCD > ∠BAD. Prove que AC > BD.

Problema 9 Seja ABC um triângulo acutângulo tal que ∠B = 2∠C , AD é


perpendicular a BC, com D sobre BC, e E o ponto médio de BC. Prove que
AB = 2 DE.

Problema 10 Seja ABC um triângulo e D um ponto sobre o lado AC tal que


AB = CD. Sejam E e F os pontos médios de AD e BC, respectivamente. Se a reta
BA intersecta a reta FE em M, prove que AM = ME.

Problema 11 Uma reta r passa pelo baricentro de um triângulo ABC. As projeções de


A, B e C sobre a reta r são M, N e P, respectivamente. Prove que AM = BN + CP.

Problema 12 (OBM) Seja ABCD um quadrilátero convexo, onde N é o ponto médio


de DC, M é o ponto médio de BC e O é a interseção entre as diagonais AC e BD.
Mostre que O é o baricentro do triângulo AMN se, e somente se, ABCD é um
paralelogramo.

Problema 13 (China) Seja ABCD um trapézio, AB // BC , ∠B = 30°, ∠B = 60°, E, M,


F, N os pontos médios de AB, BC, CD, DA respectivamente. Se BC = 7, MN = 3,
determine a medida de EF.

Problema 14 (China) Seja ABCD um trapézio, AB // CD, ∠DAB = ∠ADC = 90°, e o


3
triângulo ABC é equilátero. Se a base média do trapézio EF = a, determine o
4
comprimento da menor base AB, em função de a.

EUREKA! N°35, 2012

33
Sociedade Brasileira de Matemática

Problema 15 (Moscou) Seja ABCD um quadrilátero convexo, O um ponto em seu


interior tal que ∠AOB = ∠COD = 120°, AO = OB, CO = OD. Sejam K, L, M os
pontos médios de AB, BC, CD respectivamente, prove que ∆KLM é equilátero.

Problema 16 (China) Seja ABCD um quadrilátero tal que AD // BC. Se a bissetriz do


ângulo ∠DAB intersecta CD em E, e BE bissecta o ângulo ∠ABC , prove que
AB = AD + BC.

Problema 17 (China) Seja ABCD um quadrilátero tal que AD > BC . Sejam E e F os


pontos médios de AB e CD respectivamente. Se as retas AD e BC intersectam FE
em H e G respectivamente, prove que ∠AHE < ∠BGE.

Problema 18 Seja ABC um triângulo e sejam D e E pontos sobre os lados AB e AC ,


respectivamente, tais que AD = DB , AE = 2 EC e BE intersecta CD em F. Prove
que 4 EF = BE.

Problema 19 (OBM) Num quadrilátero convexo, a reta que passa pelos pontos
médios de dois lados opostos forma ângulos iguais com ambas as diagonais.
Mostre que as duas diagonais têm o mesmo comprimento.

Problema 20 Se um segmento paralelo a um lado de um triângulo tem uma


extremidade no ponto médio de um lado e a outra extremidade no terceiro lado,
prove que esta extremidade é ponto médio do terceiro lado.

Problema 21 (OBM) No triângulo ABC, D é ponto médio de AB e E é um ponto


sobre o lado BC tal que BE = 2 ⋅ EC . Sabendo que ∠ADC = ∠BAE , calcule o
valor de ∠BAC .

Problema 22 (Austrália) Sejam ABC um triângulo e P um ponto em seu interior de


modo que ∠PAC = ∠PBC . Se L, M são os pés das perpendiculares por P aos lados
BC, AC, respectivamente, e D é o ponto médio de AB, prove que DL = DM.

Problema 23 (Romênia) Sejam ABC um triângulo isósceles com AB = AC, D o ponto


médio de BC, M o ponto médio de AD e N a projeção de D sobre BM. Prove que
∠ANC = 90°.

Problema 24 (Eslovênia) Seja ABCD um trapézio, com AB paralelo a CD. Sabendo


que a distância entre os pontos médios das bases é igual à distância entre os pontos
médios das diagonais, prove que ∠DAC e ∠DBC são ângulos obtusos.

EUREKA! N°35, 2012

34
Sociedade Brasileira de Matemática

Problema 25 Em um triângulo isósceles ABC, com AB = BC, sejam K, L pontos


sobre AB, BC, respectivamente, tais que AK + LC = KL. A reta paralela a BC
passando pelo ponto médio M de KL intersecta AC em N. Ache a medida de
∠KNL.

Problema 26 Sejam ABC um triângulo e D, E, F os pontos médios de BC, CA, AB,


respectivamente. Prove que
∠DAC = ∠ABE ⇔ ∠AFC = ∠ADB.

Problema 27 Seja ABCD um trapézio com bases AB = a e CD = b. Sejam também


M, N os pontos médios dos lados AB, CD, respectivamente. Sabendo que
∠DAB + ∠ABC = 90°, determine o comprimento de MN.

Problema 28 (OBM) Sejam ABCD um quadrilátero convexo, N o ponto médio de


DC, M o ponto médio de BC e O a interseção entre as diagonais AC e BD. Mostre
que O é o baricentro do triângulo AMN se e somente se ABCD é um paralelogramo.

Problema 29 (Cone Sul) Seja ABC um triângulo acutângulo e sejam NA, BM e CP as


alturas relativas aos lados BC, CA e AB, respectivamente. Sejam R, S as projeções
de N sobre os lados AB, CA, respectivamente, e Q, W as projeções de N sobre as
alturas BM, CP, respectivamente.

a) Mostre que R, Q, W e S são colineares.


b) Mostre que MP = RS – QW.

Problema 30 (TST Brasil) Sejam Q o ponto médio de lado AB de um quadrilátero


inscritível ABCD e S a interseção das diagonais AC e BD. Sejam P, R as projeções
ortogonais de S sobre AD, BC, respectivamente. Prove que PQ = QR.

Bibliografia

[1]. Lecture Notes on Mathematical Olympiad Courses, for section vol. 1.


Xu Jiagu, World Scientific.
[2]. Problems and solutions in euclidean geometry.
M.N. Aref e William Wernick, Dover.
[3]. Challenging problems in geometry.
Alfred Pasamentier e Charles Salkind, Dover.

EUREKA! N°35, 2012

35
Sociedade Brasileira de Matemática

CAMPEONATOS
José Armando Barbosa Filho

♦ Nível Iniciante

Há uma grande variedade de problemas de olimpíadas que envolvem campeonatos.


A principio, para simplificar o problema, vamos analisar casos onde cada partida
acontece entre somente 2 entidades (pessoas, times,...). Por exemplo, há situações
em que:

1. O vencedor ganha 3 pontos e, em caso de empate, cada time ganha 1


ponto.
2. O vencedor ganha 1 ponto e, em caso de empate, cada time ganha 0,5
ponto.
3. Não há empates. Nesse caso, o vencedor pode ganhar x pontos, onde x é
uma variável a ser definida pelo problema.

Em cada caso, expressões diferentes podem ser obtidas. Por exemplo, no caso 2,
pode-se deduzir que, em qualquer momento:

Número de jogos = soma dos pontos de todos os jogadores.

No entanto, essa relação não é válida para o caso 1. Porém, nem tudo é tão
específico. Existem conclusões que podem ser facilmente deduzidas. Por exemplo,
em qualquer instante:

Número de vitórias = número de derrotas (pois, sempre que um time ganha, um


time perde) (1)
Número de empates sempre é par (afinal cada empate é contado duas vezes: uma
para cada time envolvido) (2)
Número de vitórias + número de empates + números de derrotas = 2 × (número de
jogos) (pois para cada partida em que houve vencedor, o número de vitórias e
derrotas aumenta um e para cada partida que terminou empate, o número de
empates aumenta dois) (3)
Em situações que envolvam futebol, de forma análoga, podemos concluir as
mesmas equações acima para números de gols, sendo que em vez de números de
vitórias seriam números de gols a favor e, em vez de números de derrotas, seriam
números de gols contra. No caso da contagem de gols, falar em empates fica sem
sentido. Portanto, poderíamos ter as equações abaixo:

EUREKA! N°35, 2012

36
Sociedade Brasileira de Matemática

Número de gols a favor = número de gols contra (4)


Número de gols a favor + número de gols contra = 2 × (número total de gols) (5)

Muito mais pode ser comentado sobre o assunto. No entanto, a abordagem por
meio de exemplos pode ser bem mais interessante.

A primeira questão foi adaptada para adequar-se melhor à realidade.

Problema 1: (Rioplatense – Nível A - 2003)

Os cinco melhores times de futebol da América participaram de um torneio, onde


todos jogam contra todos, uma única vez. De forma usual, em caso de vitória, o
time vencedor ganha 3 pontos, em caso de empate, cada time ganha 1 ponto e, em
caso de derrota, o time perdedor ganha 0 pontos. No final do torneio, faz-se uma
tabela que mostra o total de pontos obtidos (Ptos), o número de vitórias (V), a
quantidade de gols a favor (GF) e a quantidade de gols contra (GC). Porém, faltam
alguns dados:

Time Ptos V GF GC

Ceará 10 3 4 0

Palmeiras 8 2 7 1

Boca Juniors 5 1 2 ?

América 3 1 1 4

Alianza Lima ? ? 0 6

a) Indique o total de pontos, o número de vitórias do time Alianza Lima e a


quantidade de gols recebidos pelo Boca Juniors. Justifique.

b) Determine o resultado de cada partida indicando o número de gols de cada


equipe. Justifique.

Solução:
a) Para saber o número de pontos do time Alianza Lima, precisamos saber quantas
vitórias e quantos empates ele teve. Desse modo, o nosso ponto de partida será
encontrar quantas vitórias e quantos empates ele teve.
Se soubermos quantos jogos aconteceram, poderemos aplicar as equações (1), (2) e

EUREKA! N°35, 2012

37
Sociedade Brasileira de Matemática

(3) do começo.

Então, tentemos, inicialmente, descobrir quantos jogos aconteceram. Sendo 5


times, jogando todos contra todos, uma única vez, teremos que cada time joga 4
vezes, pois um time não joga contra si mesmo. No entanto, nessa contagem, cada
jogo é contado duas vezes: uma para cada time. Por isso, temos que dividir por 2.
Logo, a quantidade de jogos é igual a:

5× 4
número de jogos = = 10.
2

Agora, busquemos tentar descobrir quantos empates os outros times tiveram. Como
cada vitória vale 3 pontos e cada empate vale 1 ponto, descobrir a quantidade de
empates fica fácil. Veja por que:

3 × (vitórias) + empates = pontos


Empates = pontos – 3 × (vitórias) (6)

Daí, aplicando a equação (6) para os times, exceto o Alianza Lima, temos que:

Ceará: 10 – 3 × 3 = 1 empate
Palmeiras: 8 – 3 × 2 = 2 empates
Boca Juniors: 5 – 3 × 1 = 2 empates
America = 3 – 3 × 1 = 0 empates.

Os times, exceto o Alianza Lima, possuem: 3 + 2 + 1 + 1 = 7 vitórias.


Daí, sendo x e y, respectivamente, a quantidade de vitórias e empates do Alianza
Lima, temos, pelas equações anteriores, que:

Pela equação (1), derrotas = vitórias = x + 7


Pela equação (2), empates = y + 1 + 2 + 2 = y + 5 é par, y é ímpar
Pela equação (3), (x + 7) + (y + 5) + (x + 7) = 2 × 10 = 20
2x + y + 19 = 20
2x + y = 1

Sabemos que x e y são números positivos. Logo, x = 0 e y = 1.


Em outras palavras, o Alianza Lima venceu 0 jogos e empatou 1 jogo. Logo, ele
fez 1 ponto.

Para saber quantos gols o Boca Juniors fez, basta aplicar a equação (4) do começo.

EUREKA! N°35, 2012

38
Sociedade Brasileira de Matemática

Sendo z a quantidade de gols que o Boca Juniors fez, temos que:

Pela equação (4), 4 + 7 + 2 + 1 + 0 = 0 + 1 + z + 4 + 6 ⇒ 14 = z + 11 ⇒ z = 3

b) Para começar, calculemos o número de derrotas de cada time, sabendo que cada
um jogou 4 jogos. Para facilitar, façamos a tabela abaixo, sabendo que já
calculamos o número de empates de cada time no item anterior:

Time Ptos Jogos V E D GF GC

Ceará 10 4 3 1 4 – 3 –-1 = 0 4 0

Palmeiras 8 4 2 2 4–2–2=0 7 1

Boca Juniors 5 4 1 2 4–1–2=1 2 3

América 3 4 1 0 4–1–0=3 1 4

Alianza Lima 1 4 0 1 4–0–1=3 0 6

Daí, como Ceará e Palmeiras não perderam e, além disso, o Ceará não tomou gol,
logo teremos que o jogo entre Ceará e Palmeiras terminou empate e, além disso:

Ceará 0 × 0 Palmeiras

Além do empate com o Ceará, o Palmeiras empatou outra partida. Além desse
empate, houve mais 2 empates em jogos do Boca Juniors e outro empate do
Alianza Lima. Se Palmeiras e Alianza Lima empataram, o Boca Juniors terá
empatado 2 vezes contra não se sabe quem, o que é absurdo. Logo, houve empates
entre Palmeiras e Boca Juniors e entre Boca Juniors e Alianza Lima. Como o
Alianza Lima não fez gol, logo temos que:

Boca Juniors 0 × 0 Alianza Lima

Além do empate com o Boca Juniors, o Alianza Lima perdeu todas as outras
partidas. Inclusive o jogo contra o América. O América só fez um gol, logo só há
um resultado possível para o jogo entre América e Alianza Lima:

América 1 × 0 Alianza Lima

EUREKA! N°35, 2012

39
Sociedade Brasileira de Matemática

Logo, nos jogos entre Alianza Lima ou América contra Ceará, Palmeiras ou Boca
Juniros, os times Alianza Lima e América não fizeram gol. Portanto, o gol que o
Palmeiras levou foi contra o Boca Juniors em jogo que terminou empate. Logo,
temos mais um resultado:

Palmeiras 1 × 1 Boca Juniors

O Boca Juniors fez 2 gols e levou 3. Considerando que o Boca Juniors teve 1 gol a
favor e 1 gol contra, no jogo contra o Palmeiras e um empate de 0 a 0 contra o
Alianza Lima, falta considerar a derrota contra o Ceará e a vitória contra o
América. A única forma disso acontecer é:

Ceará 2 × 0 Boca Juniors


Boca Juniors 1 × 0 América

O Ceará fez 4 gols e levou 0. Sendo que ele venceu Alianza Lima e América e fez
2 gols contra o Boca Juniors, logo só há uma possibilidade:

Ceará 1 × 0 Alianza Lima


Ceará 1 × 0 América

Faltam apenas os jogos do Palmeiras contra o Alianza Lima e o América. Como


sabemos os placares de Alianza Lima e de América contra dos demais e o resultado
do jogo entre eles, logo temos que:

Palmeiras 4 × 0 Alianza Lima


Palmeiras 2 × 0 América

Portanto, os 10 jogos estão com os resultados especificados.

A questão 1 é um exemplo de uma idéia importante: dividir o problema em casos e


resolvê-los. Algumas outras idéias podem ser aplicadas para resolver propriamente
o problema ou apenas para conseguir novas equações para resolver o problema.
Outras idéias importantes podem ser necessárias para resolução dos problemas.
Vejamos mais 2 problemas cujas soluções dependiam um pouco de algum
conhecimento sobre campeonatos e mais algumas outras idéias.

EUREKA! N°35, 2012

40
Sociedade Brasileira de Matemática

Problema 2: (OBM – Nível 1 – 2ª fase – 2009)


Um campeonato de xadrez de 7 rodadas, com 4 jogos por rodada, tem 8
participantes, cujas pontuações por jogo são as usuais: um ponto por vitória, meio
ponto por empate e nenhum ponto por derrota. Cada par de jogadores se enfrenta
exatamente uma vez.

a) Ao término da terceira rodada, é possível que um grupo de jogadores esteja em


primeiro lugar e o restante dos jogadores esteja em segundo lugar? Explique por
meio de um exemplo.

b) Ao término da terceira rodada, é possível que todos os jogadores tenham


pontuações diferentes? Explique.

Solução:
a) Sim, basta na primeira rodada 4 jogadores vencerem e nas seguintes rodadas ser
tudo empate.

b) Cada jogador fez no máximo 3 × 1 = 3 pontos (vencendo todas) e no mínimo 0


pontos.

As possibilidades de pontuação de cada jogador são: 0; 0,5; 1; 1,5; 2; 2,5; 3. São 7


possibilidades para 8 jogadores, logo haverá, pelo menos, 2 jogadores com a
mesma quantidade de pontos, pelo Princípio das Casas dos Pombos.

Na questão acima, vemos que uma forma interessante é pensar nos casos de todos
empatarem todas as vezes ou, pelo menos, em uma determinada rodada. Isso pode
nos ajudar a entender ou resolver uma questão.

Na próxima questão, note que não importa a pontuação atribuída a uma vitória.
Importa apenas que não há empates, ou seja, há apenas vitórias e derrotas.

Problema 3: (OBM – Nível 2 – 3ª fase – 2006)


Em um torneio de tênis de mesa (no qual nenhum jogo termina empatado), cada
um dos n participantes jogou uma única vez contra cada um dos outros. Sabe-se
que, para todo k > 2, não existem k jogadores J1, J2, …, Jk tais que J1 ganhou de
J2, J2 ganhou de J3, J3 ganhou de J4, …, J(k – 1) ganhou de Jk, Jk ganhou de J1.

Prove que existe um jogador que ganhou de todos os outros e existe um jogador
que perdeu de todos os outros.

EUREKA! N°35, 2012

41
Sociedade Brasileira de Matemática

Solução: Seja Jv o jogador que mais venceu. Suponhamos que o jogador Jv não
venceu todas. Então, existe um jogador Jx que o venceu. Note que se existe um
jogador Ji que perdeu para Jv e venceu Jx, logo teremos que Jv ganhou de Ji que
ganhou de Jx que ganha de Jv, fato que não é permitido pelo enunciado.
Logo todos os jogadores que Jv venceu, Jx venceu também. Mas, Jx venceu todos
os jogadores vencidos por Jv e o Jv, ou seja, ele venceu mais jogadores do que o
Jv. Absurdo! Portanto, se Jv é o jogador que mais venceu, ele venceu todas.
A idéia para mostrar que o jogador que mais perdeu, perdeu todas é totalmente
análoga e fica como exercício para o leitor.

Na questão acima, a ideia crucial para resolução do problema é olhar para os casos
de máximo e mínimo. Em questões além desse assunto, essa idéia, também, pode
fazer sentido e, por isso, ela é de extrema importância.

Em outras questões parecidas, podemos ter situações onde pessoas conseguem


pontuações em determinadas avaliações. Geralmente, nesses casos, deve-se tentar
achar relações, por meio de equações. Nem sempre é necessário achar o valor das
variáveis. Para exemplificar, veja o exemplo abaixo:

Problema 4: (Rioplatense – Nível A – 2000)


As possíveis pontuações para um exame são 0, 1, 2, 3, 4. Depois da correção,
observou-se que o número de estudantes que obtiveram 3 pontos foi igual ao
número de estudantes que obtiveram 2 pontos. Além disso, todos conseguiram,
pelo menos, 1 ponto. A soma de todos os pontos obtidos por todos os estudantes no
exame foi igual ao número de estudantes aumentado em 30. Encontre o número de
estudantes que conseguiram, pelo menos, 3 pontos.

Solução:
Lembre-se que todo estudante fez, ao menos, 1 ponto.
Seja x o número de estudantes que conseguiram 1 ponto.
Seja y o número de estudantes que conseguiram 2 pontos.
Seja z o número de estudantes que conseguiram 3 pontos.
Seja w o número de estudantes que conseguiram 4 pontos.

Pelo enunciado, temos que:

y=z
x + 2y + 3z + 4w = x + y + z + w + 30

Substituindo a primeira equação acima na segunda acima, temos que:

EUREKA! N°35, 2012

42
Sociedade Brasileira de Matemática

x + 2z + 3z + 4w = x + z + z + w + 30 ⇒ 5z + 4w = 2z + w + 30 ⇒ 3z + 3w =
30 ⇒ z + w = 10

Logo, foram, pelo menos, 10 estudantes que conseguiram, pelo menos, 3 pontos.

Por último, façamos mais uma questão que envolve achar relações e equações.
Note que na questão abaixo o resultado em gols de cada equipe não importa.

Problema 5: (Rioplatense – Nível A – 2000)


Quatro equipes A, B, C e D disputam um torneio de futebol. Todos jogam contra
todos, uma única vez. Em caso de vitória, a equipe vencedora obtém 3 pontos e o
perdedor 0 pontos. Em caso de empate, cada equipe obtém 1 ponto.
Ao final do torneio, as pontuações das equipes A, B, C e D foram a, b, c e d.
Observou-se que a ≥ b ≥ c ≥ d e que a – b = b – c = c – d

Encontre todas as possíveis pontuações finais das 4 equipes. Para cada uma dessas
possibilidades, construa uma tabela com os resultados dos jogos e justifique porque
estas são as únicas possíveis pontuações.

Por exemplo, uma pontuação final possível é: a = 3, b = 3, c = 3 e d = 3 e a tabela


é:
A empata com B B empata com C
A empata com C B empata com D
A empata com D C empata com D

Solução: Note que temos exatamente 6 partidas. Seja S = a + b + c + d; em outras


palavras, S é igual à soma total de pontos. Note que cada empate diminui o valor de
S, pois ao invés de somar 3 pontos, pontuação em caso de vitória, soma-se 2
pontos, 1 para cada time que empatou. Daí, temos que S máximo acontece quando
não há empates e S mínimo acontece quando há somente empates. Ajeitando tudo
numa equação, temos que:

6 × 2 = 12 ≤ a + b + c + d ≤ 18 = 6 × 3 (7)

Além disso, temos que:

a – b = b – c ⇒ a = 2b – c
b – c = c – d ⇒ d = 2c – b

EUREKA! N°35, 2012

43
Sociedade Brasileira de Matemática

Substituindo as equações acima na equação (7) e fazendo as contas, tem-se que:

12 < 2 × (b + c) < 18
6<b+c<9

Agora, temos 4 casos:

1) b + c = 6. É o caso onde tudo é empate, pois corresponde à minimalidade na


equação (7).
O caso que dá certo aqui é do enunciado.

2) b + c = 7. Como b > c, temos só 1 possibilidade:

2.1) b = 4, c = 3. Nesse caso, como a – b = b – c = c – d = 1, logo temos que a = 5 e


d = 2. Como há, no máximo, 3 empates para cada equipe, logo, nesse caso,
podemos perceber que A tem 2 empates, B tem 1 empate, D tem 2 empates e há
uma quantidade par de empates (equação (2) lá no começo), logo C tem 3 empates.
Portanto, A tem 1 vitória, B tem 1 vitória e 1 derrota e D tem 1 derrota. O caso A
vence D é absurdo, pois B não pode vencer a si mesmo. Logo A vence B e B vence
D. O resto das partidas termina empatado.

2.2) b=5, c=2. Nesse caso, d= -1. Absurdo. Logo, não temos mais casos.

3) b + c = 8. Como b > c, temos 2 possibilidades.

3.1) b = 4, c = 4. Nesse caso, como a – b = b – c = c – d = 0, logo temos a = 4 e d =


4. Nesse caso, há 1 vitória, 1 empate e 1 derrota para cada equipe. Nesse caso,
podemos fazer, por exemplo, A vence B, que vence C, que vence D, que vence A e
empates entre A e C e entre B e D.

3.2) b = 5, c = 3. Nesse caso, como a – b = b – c = c – d = 2, logo temos a = 7 e d =


1. Nesse caso, A tem 1 empate, B tem 2 empates e D tem 1 empate. C ter 3 empates
geraria uma quantidade ímpar de empates, o que é absurdo com a paridade de
empates (equação (2) do começo). Logo, C não tem empates. Se A empata com D,
B fica sem ter com quem empatar, o que geraria um absurdo. Logo, A empata com
B e vence C e D, pois A faz 7 pontos. Além disso, B vence C e empata com A e D,
pois B faz 5 pontos. Por último, C vence D.

3.3) b = 6, c = 2. Nesse caso, d = – 2. Absurdo. Logo, não temos mais casos.

EUREKA! N°35, 2012

44
Sociedade Brasileira de Matemática

4) b + c = 9. É o caso onde tudo é vitória e derrota, pois corresponde a


maximalidade da equação (7).
Sabendo que a > b > c > d, logo temos que A vence todas, B vence C e D e, por
último, C vence D.

Mais problemas:

Problema 6: Na questão anterior, seria possível A fazer 8 pontos? Justifique sua


resposta.

Problema 7: Considere um campeonato brasileiro de futebol com 20 times. Pelas


regras de futebol, sabe-se que, em caso de vitória, o time vencedor soma 3 pontos e
o time perdedor soma 0 pontos. Em caso de empate, os times que empataram
somam 1 ponto cada um. No começo do campeonato, todos começam com nenhum
ponto ganho. Cada rodada é composta por 10 jogos, com cada time jogando uma
única vez por rodada. Sabe-se que ao final de 7 rodadas, nenhum time venceu todas
e nenhum time perdeu todas. Prove que, após 7 rodadas, haverá, ao menos, dois
times com a mesma pontuação.

Problema 8: Um campeonato de basquete acontece no estilo todos jogam contra


todos, uma única vez. Prove que acontece, pelo menos, uma das situações abaixo:
1. Um time vence todas.
2. Um time perde todas.
3. Dois times vencem uma quantidade igual de partidas.

Problema 9: (Olímpiada de Maio – Nível 1 – 1999)


Ana, Beatriz, Carlos, Diego e Emilia participam de um torneio de xadrez.
Cada jogador enfrenta só uma vez cada um dos outros quatro jogadores.
Cada jogador soma 2 pontos se vence a partida, 1 ponto se empata e 0 ponto se
perde.
No final do torneio, nota-se que as pontuações dos 5 jogadores são todas distintas.
Ache o número máximo de empates que pode haver acontecido no torneio e
justifique por que não pode haver um número maior de empates.

Problema 10: (Olímpiada de Maio – Nível 1 – 2008)


No colégio Olímpico, as notas dos exames são, sempre, números inteiros, sendo a
menor nota igual a 0 e a maior nota igual a 10. Na classe de aritmética, o professor
aplica dois exames na sua turma de 15 alunos. Quando um de seus alunos obtém
menos que 3 no primeiro exame e mais que 7 no segundo, ele é chamado de aluno
superado. O professor ao terminar de corrigir os exames, fez a média das 30 notas e

EUREKA! N°35, 2012

45
Sociedade Brasileira de Matemática

obteve a nota 8 de média. Qual é a maior quantidade de alunos superados que pode
ter havido nessa classe?

Problema 11: (OBM – Nivel 1 – 2ª fase – 2002)


No jogo pega-varetas, as varetas verdes valem 5 pontos cada uma, as azuis valem
10 pontos, as amarelas valem 15, as vermelhas, 20 e a preta, 50. Existem 5 varetas
verdes, 5 azuis, 10 amarelas, 10 vermelhas e 1 preta. Carlinhos conseguiu fazer 40
pontos numa jogada. Levando em conta apenas a quantidade de varetas e suas
cores, de quantas maneiras diferentes ele poderia ter conseguido essa pontuação,
supondo que em cada caso fosse possível pegar as varetas necessárias?

Problema 12: (Seletiva de Fortaleza para a Olímpiada Rioplatense de Matemática –


2001)
Quatro problemas são propostos em uma Olímpiada de Matemática. Cada
problema vale 4 pontos. Após todas as provas terem sido corrigidas, notou-se que
quaisquer dois participantes não obtiveram mesma pontuação em mais de um
problema. (Cada estudante pode obter 0, 1, 2, 3 ou 4 pontos em cada problema).

a) Mostre que o número de participantes não pode ser igual a 26.


b) Dê um exemplo mostrando que podemos ter 25 estudantes.

EUREKA! N°35, 2012

46
Sociedade Brasileira de Matemática

SOLUÇÕES DE PROBLEMAS PROPOSTOS


 Publicamos aqui algumas das respostas enviadas por nossos leitores.

134) Considere a operação ⋅ entre dois vetores do 3 definida por:


( x, y, z ) ⋅ ( u, v, w) = ( xu + yw + zv, xw + zu + yv, xv + yu + zw) .
k
Prove que, para todo k ≥ 1, se ( x, y, z ) = ( 0,0,0 ) então x = y = z = 0.
1
Obs.: Para qualquer ( x , y , z ) ∈  3 , ( x, y , z ) = ( x, y , z ) e, para todo k > 1,
k k −1
( x, y , z ) = ( x, y , z ) ⋅ ( x, y , z ) .
SOLUÇÃO DE ZOROASTRO AZAMBUJA NETO (RIO DE JANEIRO – RJ)
Se s ( x, y, z ) = x + y + z, temos
s ( ( x, y, z ) ⋅ ( u, v, w) ) = xu + yw + zv + xw + zu + yv + xv + yu + zw = ( x + y + z )( u + v + w) =
k
= s ( x, y, z ) ⋅ s ( u, v, w ) . Assim, se ( x, y, z ) = ( 0,0,0 ) , temos
k
0 = s ( 0,0,0 ) = s ( x, y, z ) , donde s ( x, y, z ) = 0.
Se X = {( x, y, z ) ∈  3
}
s ( x, y, z ) = 0 = {( x, y, − x − y ) , x, y ∈ } , temos
( x , y , z ) , ( u , v , w ) ∈ X ⇒ ( x , y , z ) ⋅ ( u , v, w ) ∈ X .
De fato, ( x, y, − x − y ) ⋅ ( u, v, −u − v ) = ( a, b, − a − b ) , onde
a = xu − y ( u + v ) − v ( x + y ) e b = − x ( u + v ) − u ( x + y ) + yv. Considere a
transformação linear T :  2 →  2 dada por
T ( u, v ) = ( xu − y ( u + v ) − v ( x + y ) , − x ( u + v ) − u ( x + y ) + yv ) .
Se as duas primeiras coordenadas de um elemento α ∈ X são ( u, v ) , as duas
primeiras coordenadas de ( x, y, − x − y ) ⋅ α são dadas por T ( u, v ) . Assim, para
k
k ≥ 1, temos ( x, y, z ) = ( 0,0,0 ) se e somente se z = − x − y e T k −1 ( x, y ) = ( 0,0 ) .
 x− y −( x + 2y)
A matriz de T é   , cujo determinante é
 − ( 2x + y ) y−x 

2  y  3 y2 
2

− ( x − y ) − ( 2 x + y )( x + 2 y ) = −3 ( x + xy + y ) = −3   x +  +
2 2
 , que só se
 2 4 

anula quando x = y = 0. Assim, se ( x, y ) ≠ ( 0,0 ) , temos det T ≠ 0 e logo

EUREKA! N°35, 2012

47
Sociedade Brasileira de Matemática

k −1
det ( T k −1 ) = ( det T ) ≠ 0, para todo k ≥ 1, donde T k −1 ( x, y ) ≠ ( 0,0 ) (pois
k
( x, y ) ≠ ( 0,0 ) ), e logo ( x, y, − x − y ) ≠ ( 0,0,0 ) , o que resolve o problema.

135) Considere um hemisfério cuja base é um círculo ( C1 ) . Um círculo ( C2 ) do


hemisfério é paralelo a ( C1 ) , de forma que existem n círculos do hemisfério,
congruentes, tangentes entre si, a ( C1 ) e a ( C2 ) . Mostre que a razão K(n) entre os
cos 2 π n
raios de ( C2 ) e ( C1 ) é igual a: K ( n ) = .
1 + sen 2 π n

SOLUÇÃO DE JOSÉ DE ALMEIDA PANTERA (RIO DE JANEIRO – RJ)


 ,0 ≤ j ≤ n − 1, os n círculos tangentes a C e a C . Podemos supor sem
Sejam C j 1 2

perda de generalidade que o hemisfério é


H= {( x, y, z ) ∈  3 2 2 2
}
z ≥ 0 e x + y + z = 1 , C1 = {( cosθ , senθ ,0 ) ,θ ∈ [ 0,2π ]} e
 π
C2 = {( cos α cos θ ,cos α senθ , senα ) ,θ ∈ [ 0, 2π ]} , para um certo α ∈ 0,  (e logo
 2
o raio de C2 é cos α ). Podemos também supor que C  j é tangente a C em
1

 2 jπ 2 jπ   2 jπ 2 jπ 
 cos , sen ,0  , e (logo) a C2 em  cos α cos ,cos α sen , senα  .
 n n   n n 

Assim, o centro de C  j é   1 + cos α  cos 2 jπ ,  1 + cos α  sen 2 jπ , senα  .


    
 2  n  2  n 2 
Considere os círculos C 0 e C
 1.
2 2
1−cosα senα  1−cosα   senα  1− cosα α 
Seus raios são iguais a  ,0, =   +  = = sen 
 2 2   2   2  2  2
(que é a distância entre o centro de C  0 e o ponto de tangência entre C
 0 e C ), e as
1

distâncias entre o centro 0 = ( 0,0,0 ) da esfera e os centros de C0 e C 1 são iguais a


2 2
 1 + cos α   senα  1 + cos α α 
  +  = = cos   .
 2   2  2 2
Considerando a reta r, interseção dos planos que contêm os círculos C 0 e C1 ,
temos que r é tangente à esfera e aos círculos C 0 e C
 1 . O plano perpendicular a r
 0 e de C
passando por P contém os centros Q1 e Q2 de C  1 e o centro O da esfera.

EUREKA! N°35, 2012

48
Sociedade Brasileira de Matemática

Os triângulos OQ1P e OQ2 P são retângulos com hipotenusa OP , e logo Q1Q2 é o


α 
dobro da altura do triângulo retângulo OQ1 P , que, como OP = 1, OQ1 = cos   e
2
α  α  α  α  α 
Q1 P = sen   , é igual a cos   ⋅ sen   1 . Assim, Q1Q2 = 2cos   sen   ,
2
  2
  2
  2
  2
 1 + cos α senα 
mas Q1Q2 é a distância entre os pontos  ,0,  e
 2 2 
  1 + cos α  2π  1 + cos α  2π senα 
  cos ,  sen , , a qual é igual a
 2  n  2  n 2 
2
 1 + cosα   2π  2 2π  1 + cosα   2π  2 α  π 
  1 − cos  + sen =  2 − 2cos   = cos   ⋅ 2sen   ,
 2   n  n  2   n  2 n
α 
sen  
α  α  α  π  α   2  = sen  π  .
e logo 2cos   sen   = 2cos2   sen   , donde tan   =  
2 2  2 n  2  cos  α  n
 
2
π  π 
1 − sen 2   cos 2  
1 − tan 2 (α 2 ) n=  n  , cqd.
Assim, o raio de C2 é cos α = =
1 + tan 2 (α 2 )  π  π 
1 + sen 2   1 + sen 2  
n n

142) Seja A = {4,8,9,16, 25, 27,32,36, 49,64,...} o conjunto das potências não
triviais (números da forma a b , com a ≥ 2, b ≥ 2 naturais). Prove que, para todo
natural n ≥ 1, existe um natural k tal que todos os termos da progressão aritmética
k , 2k ,3k ,..., nk pertencem a A.

SOLUÇÃO DE MARCELO RIBEIRO (RIO DE JANEIRO – RJ)


Lema: ( ∃b ∈  ab ≡ 1( mod n ) ) ⇔ mdc ( a, n ) = 1.
Demonstração: Artigo “Divisibilidade, Congruências e Aritmética Módulo n”,
Eureka Nº2, ou qualquer texto sobre Teoria dos Números.
Mostraremos que, dado n ≥ 2, n ∈ , podemos escolher (n – 1) números naturais,
digamos α1 ,..., α n −1 , de forma a termos k = 2α1 × 3α 2 × ... × nα n−1 satisfazendo o
enunciado.

EUREKA! N°35, 2012

49
Sociedade Brasileira de Matemática

Tomemos, para isso, p1 , p2 ,..., pn números primos, distintos. Basta, agora, mostrar
que o sistema de equações
 α1 ≡ α 2 ≡ ... ≡ α n −1 ≡ 0 ( mod p1 )

α1 + 1 ≡ α 2 ≡ ... ≡ α n −1 ≡ 0 ( mod p2 )

 
α ≡ α ≡ ... ≡ α + 1 ≡ 0 ( mod p )
 1 2 n −1 n

Apresenta solução em inteiros.


De fato, ∀i ∈ {1, 2,..., n − 1} , tomemos
 
n
αi =  p j  × k i , ki ∈  + .

 j =1, j ≠ i +1 
Este valor satisfará a todas as equações, a menos talvez de
α i + 1 ≡ 0 ( mod pi +1 ) .
Contudo, pelo lema, podemos escolher ki , de forma a termos
 n 
αi =  ∏ p j  × ki ≡ −1( mod pi +1 ) ,
 j =1, j ≠i +1 
já que mdc ((∏ n
j =1, j ≠ i ) )
p j , pi +1 = 1. (Podemos supor k i naturais senão somamos
um múltiplo conveniente de pi +1 ).

145) Encontre todos


os números racionais p, q, r de modo que
π 2π 3π
p cos + q cos + r cos = 1.
7 7 7

SOLUÇÃO DE MICHEL FALEIROS MARTINS (CAMPINAS – SP)


Considere os polinômios pn tais que Pn ( 2cosθ ) = 2cos ( nθ ) , n ≥ 0.
Como cos ( n + 1)θ + cos ( n − 1)θ = 2cosθ ⋅ cos ( nθ ) , temos que
Pn +1 ( x ) = x ⋅ Pn ( x ) − Pn −1 ( x ) , onde x = 2 cosθ e n ≥ 1.
Agora, P0 ( x ) = 2 e P1 ( x ) = x ⇒
P2 ( x ) = x 2 − 2, P3 ( x ) = x 3 − 3x, P4 ( x ) = x 4 − 4 x 2 + 2,...
π
Afirmamos que, sendo α = 2cos ,
7
(I) α é raiz da equação x3 − x 2 − 2 x + 1 = 0;

EUREKA! N°35, 2012

50
Sociedade Brasileira de Matemática

(II) α é irracional;
π 2π 3π
(III) 2cos − 2cos + 2cos = 1.
7 7 7
Prova das afirmações:

3π 4π 3π 4π
(I) =π − ⇒ cos + cos = 0 ⇒ P3 (α ) + P4 (α ) = 0
7 7 7 7
⇒ (α 3 − 3α ) + (α 4 − 4α 2 + 2 ) = 0 ⇒ α 4 + α 3 − 4α 2 − 3α + 2 = 0
⇒ (α + 2 ) ⋅ (α 3 − α 2 − 2α + 1) = 0 ⇒ α 3 − α 2 − 2α + 1 = 0.
(II) Pelo Teorema da Raiz Racional, as possíveis raízes racionais não nulas de
x3 − x 2 − 2 x + 1 = 0 são ±1. No entanto, nenhum desses dois valores é, de fato, raiz.

(III) Como 2senφ ⋅ cos θ = sen (φ + θ ) + sen (φ − θ ) , temos que


π  π 2π 3π 
2sen ⋅  cos − cos + cos 
7  7 7 7 
 2π   3π  π   4π  2π   π
=  sen + sen 0  −  sen + sen  −   +  sen + sen  −   = sen
 7   7  7   7  7  7
π 2π 3π
⇒ 2cos − 2cos + 2cos = 1.
7 7 7
Lema: (0, 0, 0) é o único terno de racionais ( p´, q´, r´) que satisfaz
π 2π 3π
2 p´cos − 2q´cos + 2r´cos =0 (1).
7 7 7

Prova: 2cos = P2 (α ) = α 2 − 2
7

2cos = P3 (α ) = α 3 − 3α = α 2 + 2α − 1 − 3α = α 2 − α − 1
7
(1) ⇔ α p´− (α 2 − 2 ) q´+ (α 2 − α − 1) r´= 0
⇔ α 2 ⋅ ( r´− q´) + α ( p´− r´) + ( 2q´− r´) = 0
Temos os dois casos:
(i ) r´− q´= 0.
r´−2q´
• Se p´− r´≠ 0, então α = ∈ , absurdo.
p´−r´

EUREKA! N°35, 2012

51
Sociedade Brasileira de Matemática

• Se p´− r´= 0, então 2q´− r´= 0 ⇒ p´= q´= r´= 0 e obtemos a solução trivial de
(1).
(ii ) r´− q´≠ 0.

{
Resolvendo a equação de segundo grau temos que α ∈ s + t , s − t com s, t ∈ . }
• Se t < 0, α é complexo, absurdo.
• Se t = 0 ou t > 0 e t ∈ , α é racional, absurdo.
Portanto, t > 0 e t ∉ . Mas

3 2
(s ± t ) − (s ± t ) ( )
− 2 s ± t +1=

(s 3
− s 2 + 3st − 2s − t + 1) ± t ( 3s 2 − 2s − 2 + t )
Como α 3 − α 2 − 2α + 1 = 0 e t ∉ * ,
3s 2 − 2s − 2 + t = 0 (2) e s 3 − s 2 + 3st − 2s − t + 1 = 0 (3).
( 2 ) ⇒ t = −3s 2 + 2s + 2. Substituindo em (3),
s 3 − s 2 + 3s ( −3s 2 + 2 s + 2 ) − 2s − ( −3s 2 + 2 s + 2 ) + 1 = 0
3 2
⇔ ( 2 s ) − 2 ( 2 s ) − ( 2s ) + 1 = 0.
Pelo Teorema da Raiz Racional, 2s = ±1 são as únicas possibilidades de raiz
racional não nula, mas nenhum desses dois valores é, de fato, raiz.
Portanto, p´= q´= r´= 0 é a única solução racional de (1).
π 2π 3π
Voltando ao problema original, se p cos + q cos + r cos = 1 então, de (III),
7 7 7
obtemos
π 2π 3π
( p − 2 ) cos + ( q + 2 ) cos + ( r − 2 ) cos = 0.
7 7 7
Pelo Lema, p − 2 = q + 2 = r − 2 = 0 ⇔ p = 2, q = −2 e r = 2.
Portanto, a equação dada tem uma única solução racional: ( p, q, r ) = ( 2, −2, 2 ) .

146) Determine todos os subconjuntos não-vazios A, B, C de  de modo que:

a) A ∩ B = B ∩ C = C ∩ A = ∅.
b) A ∪ B ∪ C = .
c) para quaisquer a ∈ A, b ∈ B e c ∈ C , temos: a + c ∈ A, b + c ∈ B e a + b ∈ C.

EUREKA! N°35, 2012

52
Sociedade Brasileira de Matemática

SOLUÇÃO DE MICHEL FALEIROS MARTINS (CAMPINAS – SP)


A = {1, 4,7,...} , B = {2,5,8,...} e C = {0,3,6,...}
ou
A = {2,5,8,...} , B = {1, 4,7,...} e C = {0,3,6,...} .
Lema: Se S e T são dois subconjuntos não vazios dos naturais com a propriedade
que s + t ∈ S se s ∈ S e t ∈ T , então s + k ⋅ t ∈ S para todo k ∈  = {0,1, 2,...} , se
s ∈ S e t ∈T .
Prova: Por indução sobre k : para k = 0, s ∈ S é verdadeiro. Supondo que
s + k ⋅ t ∈ S para algum k ∈ , então s + ( k + 1) ⋅ t = ( s + k ⋅ t ) + t ∈ S , o que completa
a indução.
Considere a ∈ A, b ∈ B e c ∈ C elementos quaisquer. Sejam α , β e γ os menores
elementos de A, B e C, respectivamente. Se α = 0,0 + b ∈ C ⇒ b ∈ C , absurdo.
Analogamente se β = 0. Logo, γ = 0. Como α + β ∈ C e α + β ≥ 3 então C
admite um elemento mínimo m ≥ 1.
Se m = 1, pelo lema, α + k ⋅ m = α + k ∈ A e β + l ∈ B para quaisquer
k , l ∈  ⇒ α + β ∈ A e β + α ∈ B, absurdo.
Pela simetria do problema em relação a A e B, suponha α = 1.
Se m = 2, pelo lema, A é o conjunto dos naturais ímpares e então β é par. Mas aí
α + β ∈ C e como α + β = 1 + β é ímpar ⇒ α + β ∈ A, absurdo.
Se m = 3, temos dois casos:
(i) 2 ∈ A ⇒ 3k + 1,3k + 2 ∈ A para todo k ∈ , pelo lema. Então, β = 3l para algum
l ≥ 1 ⇒ α + β = 1 + 3l ∈ A, mas α + β ∈ C , absurdo.
(ii) 2 ∈ B ⇒ 3k + 2 ∈ B para todo k ∈ , pelo lema; e 1∈ A ⇒ 3l + 1∈ A para todo
l ∈ , pelo lema. Então, a + b = ( 3k + 2 ) + ( 3l + 1) = 3 ( k + l + 1) ∈ C para quaisquer
k , l ∈ . Assim, A = {1, 4,7,...} , B = {2,5,8...} e C = {0,3,6,...} . Esses conjuntos
satisfazem as condições dadas e, pela simetria, A = {2,5,8,...} , B = {1, 4,7,...} e
C = {0,3,6,...} também cumprem as condições do problema.
Se m ≥ 4, α + β ≥ m ⇒ β ≥ m − 1 ≥ 3 ⇒ {1, 2,3,...m − 2} ⊂ A. Temos dois casos:
(i) m − 1∈ A, pelo lema, todo natural n = r + k ⋅ m com 1 ≤ r ≤ m − 1 e k ∈ 
pertence a A. Assim, β = l ⋅ m para algum l ≥ 2 ⇒ α + β = 1 + m ⋅ ( k + l ) ∈ A, mas
α + β ∈ C , absurdo.
(ii) m − 1∈ B. Então β = m −1 ≥ 3 e 2 ∈ A ⇒ 2 + ( m − 1) ∈ C , mas
2 + ( m − 1) = 1 + m ∈ A, absurdo.

EUREKA! N°35, 2012

53
Sociedade Brasileira de Matemática

k n
n−2
( −1) ( −1) −1
147) Demonstre que ∑
k =0  ( 2k + 1) π 
=n+
2
, para todo inteiro n ≥ 2.
sen  
 4n − 2 

SOLUÇÃO DE MICHEL FALEIROS MARTINS (CAMPINAS – SP)


k

Seja Sn = ∑
n−2
( −1) , n ≥ 2. Então,
k =0  ( 2k + 1) π 
sen  
 4n − 2 
 
k  0 n−3 n−2 
2 n −2
( −1) =
( −1) ( −1) ( −1) +
∑k =0  ( 2k + 1) π    π 
+ ... +
 ( 2n − 5 ) π 
+
 ( 2n − 3 ) π  
sen    sen  4n − 2  sen   sen   
 4n − 2      4n − 2   4n − 2  
 
n −1 n n +1 
2 n−2

+
( −1) +
( −1) +
( −1) + ... +
( −1) 
 ( 2n − 1) π    ( 2n + 1) π   ( 2n + 3) π   ( 4n − 3) π  
sen    sen   sen   sen   
 4n − 2    4n − 2   4n − 2   4n − 2  
k

⇒∑
2n−2
( −1) n −1
= 2 ⋅ Sn + ( −1) .
k =0  ( 2k + 1) π 
sen  
 4n − 2 

pois
( 2n + 1) π + ( 2n − 3) π = π , ( 2n + 3) π + ( 2n − 5) = π ,..., ( 4n − 3) π +
π
=
4n − 2 4n − 2 4n − 2 4n − 2 4n − 2 4n − 2
n+a n−2−a
= π , ( −1) = ( −1) para 0 ≤ a ≤ n − 2, e sen (π − θ ) = sen (θ ) .
Façamos m = 4n – 2. Então,

1 1 1 1
− + ... − +
π   3π   ( m − 3) π   ( m − 1) π 
sen   sen   sen   sen  
 
m m   m   m 
1 1 1 1
=− + − ... + −
 ( m + 1) π   ( m + 3) π   ( 2m − 3 ) π   ( 2m − 1) π 
sen   sen   sen   sen  
 m   m   m   m 

EUREKA! N°35, 2012

54
Sociedade Brasileira de Matemática

k k

⇒∑
m −1
( −1) 2 n−2
= 2⋅ ∑
( −1) n −1
= 4 Sn + 2 ( −1) .
k =0  ( 2k + 1) π  k =0  ( 2k + 1) π 
sen  sen   
 m   4n − 2 
Já que

( m +1) π ( m −1) π ( m + 3) π ( m − 3) π ( 2m −1) π π


+ = 2π , + = 2π ,..., + = 2π , sen ( 2π − θ ) =
m m m m m m
sen ( −θ ) = −sen (θ ) e m é par.
 kπ 
i  eiθ − e−iθ
Seja ε k = e  m  com k ≥ 1. Como sen (θ ) = , temos que
2i
( ) 2k +1 = i ⋅ m−1  ( −1) + ( −1) .
k k k k
m−1
( −1) m−1 −1 ε

∑  ( 2k + 1) π 
= 2i ⋅ ∑ 2
k =0 ε 2k +1 −1
∑  
k =0  ε 2k +1 −1 ε 2k +1 + 1
k =0
sen    
 m 
 ( 2k +1+m)π   ( 2k +1)π 
i   i  +π i
= −ε2k +1 ⇒ ε 2k +1 = −ε
m  m 
Mas, ε 2k +1+m = e  
=e m 
.
2⋅ +k +1
2 

ε2⋅0+1 = −ε m 
2⋅ +0 +1
2 

ε2⋅1+1 = −ε m 
2⋅ +1+1
Assim, 2 

 
ε m 
= −ε m m 
2⋅ −1+1 2⋅ + −1+1
2  2 2 

m m
−1 k k +1 k −1 k +1
2 ( −1) m−1
( −1) m−1
( −1) 2 ( −1)
⇒∑ =∑ e ∑ε =∑ .
k =0 ε 2k +1 −1 k=
m −ε 2k +1 −1 k=
m 2k +1 −1 k =0 −ε 2k +1 −1
2 2

m
Como = 2n −1 é ímpar, o expoente k teve de ser trocado por k + 1 em ambas as
2
igualdades, pois os índices dos somatórios começam em paridades diferentes.

Logo,

EUREKA! N°35, 2012

55
Sociedade Brasileira de Matemática

k k k
m−1
( −1) m−1
( −1) m−1
( −1)
∑ε =∑ = 2i∑
n−1
⇒ 4Sn + 2( −1)
k =0 2k +1 −1 k =0 ε 2k +1 + 1 k =0 ε2k +1 −1
π 
k k −i   k +1
π 
m−1
( −1) m−1
( −1) ⋅e  m −i   m−1 ( −1)
e ∑ε
k =0 −1
=∑
k =0
 2kπ 
i 
π 
−i 
=e ∑
 m

k =0
π 
−i 
 2kπ 
i 
2k +1  m   m  m  m 
e −e e −e
k +1  2kπ 

Sejam S + ( x ) = ∑
1 m −1
e S− ( x ) = ∑
( −1) , m −1
onde
i
ωk = e  m 

com
k = 0 x − ωk k = 0 x − ωk

0 ≤ k ≤ m − 1 são as m–ésimas raízes da unidade.


Observe que, para um polinômio P ( x ) = ( x − r0 )( x − r1 ) ... ( x − rj ) ,
1
j j
 j
 P´( x )
∑ x − r = ∑ ( ln x − r )´=  ∑ ( ln x − r ) ´= ( ln P ( x ) )´= P ( x ) .
k k
k =0 k k =0  k =0 
 m  m   m 
Agora, sendo P ( x ) = x m − 1 =  x 2 − 1 x 2 + 1 =  x 2 − 1 ⋅ Q ( x ) , as raízes de
    
m
P ( x ) m
x 2 −1 = são ω2 k ,0 ≤ k ≤ − 1, e as raízes de Q ( x ) são
Q ( x) 2
m
ω2 k +1 ,0 ≤ k ≤ − 1, pois
2
m

m  i  2( 2 k +1)π   2 m


(ω2k +1 ) = e  
= ei ( 2 k +1)π = −1,0 ≤ k ≤ − 1.
m
2
  2
 
m
−1
2
1
Então, S + ( x ) + S − ( x ) = 2 ⋅ ∑
k =0 x − ω2 k +1
m
m 2 −1 m
2⋅ ⋅x −1
Q´( x) P´( x) 2 m ⋅ x m −1 m ⋅ x 2
⇒ S− ( x ) = 2 ⋅ − = − m =
Q ( x ) P ( x) m
x −1 1 − xm
x +1
2

m
−1
 − iπ  2
m⋅e m 
n −1 −

 − iπ   − iπ   
⇒ 4 ⋅ Sn + 2 ( −1) = 2i ⋅ e m
⋅ S−  e m  = 2i ⋅  e m ⋅ = m = 4n − 2.
iπ m
     −m 
1− e 
 

EUREKA! N°35, 2012

56
Sociedade Brasileira de Matemática

Portanto, S n = n +
( −1) −1
para todo n ≥ 2.
2

n −1
n − k 
148) Sejam m e n inteiros positivos. Calcule ∑ 
k =0 m 
.

SOLUÇÃO DE CARLOS ALBERTO DA SILVA VICTOR (NILÓPOLIS – RJ)


n −1
n − k 
s = ∑  onde m e n são inteiros positivos:
k =0  m 

i) Seja n ≤ m

 n   n − 1 1
s= +  + ...  
m  m  m

Tomando:  x  = menor inteiro maior do que ou igual a x, teremos:


s = 1 + 1 + ... + 1 = n ⋅ 1 = n.

ii) Seja n > m, logo n = q ⋅ m + r com r ∈ {0,1, 2,..., m − 1}


Logo
 n   n − 1  m   m − 1 1
s= +  + ... +   +   + ... +  
m  m  m  m  m
s = r ⋅ ( q + 1) + m ( q + q − 1 + ... + 2 + 1)

s = r ( q + 1) + m ⋅ q ⋅
( q + 1)
2
 m⋅q 
s = ( q + 1)  + r .
 2 
Se quisermos escrever esta expressão em termos apenas de m e n, obtemos
  n   m  n   n     n   m  n 
s =    + 1 ⋅   + n − m    =    + 1  n −    . Note que essa
   
m 2  
m       
m m 2 m
fórmula também vale se n ≤ m.

3 3 3
150) Sejam a, b e c números reais tais que ( a − b ) + ( b − c ) + ( c − a ) = 9.

EUREKA! N°35, 2012

57
Sociedade Brasileira de Matemática

1 1 1
Prove que 2
+ 2
+ 2
≥ 3 3.
(a − b) (a − b) (c − a)
SOLUÇÃO DE RENATO CARNEIRO (BELO HORIZONTE – MG)
Fazendo a − b = x, b − c = y e z = c − a, então teremos que provar que
1 1 1
2
+ 2 + 2 ≥ 3 3.
x y z
 x3 + y 3 + z 3 = 9
Para isso, temos que  , então vamos lá:
x + y + z = 0
Fazendo z = − ( x + y ) , teremos
3
x3 + y 3 + z 3 = x3 + y 3 − ( x + y ) = −3x 2 y − 3xy 2 = −3xy ( x + y ) = 3 xyz = 9 ⇒ xyz = 3.

Agora basta usar a famosa desigualdade das médias:


1 1 1
+ + 2
x2 y 2 z 2 1 1 1 1 1 1  1 
≥ 3 2 ⋅ 2 ⋅ 2 ⇒ 2 + 2 + 2 ≥ 3⋅ 3  
3 x y z x y z  xyz 
2
1 1 1 1 1 1 1
⇒ + + ≥ 3 ⋅ 3   ⇒ 2 + 2 + 2 ≥ 3 3.
x2 y 2 z 2 3
  x y z

Agradecemos o envio de soluções e a colaboração de:

Marcos Ferreira (Eunápolis – BA) Prob. 134


Besaleel Júnior (Teresina – PI) Prob. 144
Renato Carneiro (Belo Horizonte – MG) Prob. 145, 146
Marcelo Ribeiro (Rio de Janeiro – RJ) Prob. 150
Jean-Pierre Youyoute (Rio de Janeiro – RJ) Prob. 150
Itamar Sales (Fortaleza – CE) Prob. 150
Samuel Abdalla (Sorocaba – SP) Prob. 150
Douglas Oliveira de Lima (Brasília – DF) Prob. 150
Lucas Justo de Freitas Neto (Mossoró – RN) Prob. 149
José Armando Barbosa Filho (Fortaleza – CE) Prob. 134
Carlos Alberto da Silva Victor (Nilópolis – RJ) Prob. 146, 150

Continuamos aguardando soluções para os problemas 143, 144, 149.

EUREKA! N°35, 2012

58
Sociedade Brasileira de Matemática

PROBLEMAS PROPOSTOS
 Convidamos o leitor a enviar soluções dos problemas propostos e sugestões de novos
problemas para próximos números.

12 1
151) Encontre todas as soluções reais positivas de x x = .
2
152) Sejam a, b, c números reais positivos tais que a + b + c = 1.
a − bc b − ca c − ab 3
Prove que + + ≤ .
a + bc b + ca c + ab 2
153) Quatro pontos P, Q, R, S pertencem a um círculo de tal forma que o ângulo

PSR é reto. Sejam H e K as projeções de Q nos segmentos PR e PS,
respectivamente. Prove que a reta HK divide o segmento QS ao meio.

154) Determine todas as funções f :  →  ∩ [ 0, +∞ ) tais que


f ( xy ) = f ( x ) f ( y ) e f ( x + y ) ≤ f ( x ) + f ( y ) , ∀x, y ∈ .

155) Sejam a, b e c inteiros positivos, tais que existe um triângulo T de lados


a , b e c . Prove que são equivalentes:
i) Existe um triângulo congruente a T cujos vértices têm coordenadas inteiras em
2 .
ii) T tem área racional e existem x, y inteiros com a = x 2 + y 2 .
iii) T tem área racional e existem u, v inteiros com mdc ( a, b, c ) = u 2 + v 2 .

156) Denominamos Máquina de Conway o par C = ( e; S ) formado por uma entrada


e = 2α1 , a1 ∈ , e uma sequência finita S = ( si )1≤i ≤ n de racionais não nulos, com
sn ∈ .
Dada uma máquina de Conway C, construímos a sequência pi definida por:
(1) p1 = e;
(2) pk = pk −1 ⋅ si , onde i é o menor número inteiro positivo tal que pk −1 ⋅ si ∈ .
Dizemos que a ∈  é uma saída de C se, e somente se, existe um inteiro positivo k
tal que pk = 2a.
O conjunto de todas as saídas é denominado conjunto gerado por C.
Exemplo: A máquina de Conway formada pela entrada 22 e pela sequência

EUREKA! N°35, 2012

59
Sociedade Brasileira de Matemática

17 78 19 23 29 77 95 77 1 11 13 15 15
, , , , , , , , , , , , ,55
91 85 51 38 33 29 23 19 17 13 11 14 2

gera os números primos.


Mostre que é possível construir uma Máquina de Conway que gere o conjunto dos
quadrados perfeitos.

Problema 151 proposto por Douglas Oliveira de Lima, de Brasília (DF). Problemas 152 e 153
propostos por Adriano Carneiro Tavares, de Caucaia (CE). Problema 156 proposto em uma
prova de seleção para a Olimpíada Internacional de Matemática (IMO) de 1996.

EUREKA! N°35, 2012

60
Sociedade Brasileira de Matemática

AGENDA OLÍMPICA
XXXIV OLIMPÍADA BRASILEIRA DE MATEMÁTICA

NÍVEIS 1, 2 e 3
Primeira Fase – sábado, 16 de junho de 2012
Segunda Fase – sábado, 22 de setembro de 2012
Terceira Fase – sábado, 27 de outubro de 2011 (níveis 1, 2 e 3)
domingo, 28 de outubro de 2012 (níveis 2 e 3 - segundo dia de prova)

NÍVEL UNIVERSITÁRIO
Primeira Fase – sábado, 22 de setembro de 2012
Segunda Fase – sábado, 27 e domingo, 28 de outubro de 2012

V ROMANIAN MASTER OF MATHEMATICS (RMM)


29 de fevereiro a 4 de março de 2012 (Bucareste, Romênia)

ASIAN PACIFIC MATH OLYMPIAD (APMO)


12 de março de 2012

XVIII OLIMPÍADA DE MAIO


12 de maio de 2012

XXIII OLIMPÍADA DE MATEMÁTICA DO CONE SUL


27 de outubro a 03 de novembro (Lima, Peru)

LIII OLIMPÍADA INTERNACIONAL DE MATEMÁTICA


4 a 16 de julho de 2012 (Mar del Plata, Argentina)

II OLIMPÍADA DE MATEMÁTICA DA LUSOFONIA


20 a 28 de julho de 2012 (Salvador, Bahia)

XVIII OLIMPÍADA INTERNACIONAL DE MATEMÁTICA UNIVERSITÁRIA (IMC)


26 de julho a 1º de agosto de 2012 (Blagoevgrad, Bulgária)

XXVI OLIMPÍADA IBERO-AMERICANA DE MATEMÁTICA


29 de setembro a 6 de outubro de 2012 (Cochabamba, Bolívia)

IV COMPETIÇÃO IBERO-AMERICANA INTERUNIVERSITÁRIA DE MATEMÁTICA


1º a 5 de outubro de 2012 (Guanajuato, México)

XIV OLIMPÍADA IBEROAMERICANA DE MATEMÁTICA UNIVERSITÁRIA


A confirmar

EUREKA! N°35, 2012

61
Sociedade Brasileira de Matemática

COORDENADORES REGIONAIS
Alberto Hassen Raad (UFJF) Juiz de Fora – MG
Américo López Gálvez (USP) Ribeirão Preto – SP
Antonio Carlos Nogueira (UFU) Uberlândia – MG
Benedito Tadeu Vasconcelos Freire (UFRN) Natal – RN
Bruno Holanda (CAEN – UFC) Fortaleza – CE
Carmen Vieira Mathias (UNIFRA) Santa María – RS
Claus Haetinger (UNIVATES) Lajeado – RS
Cláudio de Lima Vidal (UNESP) S.J. do Rio Preto – SP
Denice Fontana Nisxota Menegais (UNIPAMPA) Bagé – RS
Disney Douglas Lima de Oliveira (UFAM) Manaus – AM
Edson Roberto Abe (Colégio Objetivo de Campinas) Campinas – SP
Edney Aparecido Santulo Jr. (UEM) Maringá – PR
Fábio Brochero Martínez (UFMG) Belo Horizonte – MG
Florêncio Ferreira Guimarães Filho (UFES) Vitória – ES
Francinildo Nobre Ferreira (UFSJ) São João del Rei – MG
Genildo Alves Marinho (Centro Educacional Leonardo Da Vinci) Taguatingua – DF
Herivelto Martins (USP – São Carlos) São Carlos – SP
Gilson Tumelero (UTFPR) Pato Branco – PR
Ivanilde Fernandes Saad (UC. Dom Bosco) Campo Grande – MS
João Benício de Melo Neto (UFPI) Teresina – PI
João Francisco Melo Libonati (Grupo Educacional Ideal) Belém – PA
Diogo Diniz (UFPB) Campina Grande – PB
José Luiz Rosas Pinho (UFSC) Florianópolis – SC
José Vieira Alves (UFPB) Campina Grande – PB
José William Costa (Instituto Pueri Domus) Santo André – SP
Krerley Oliveira (UFAL) Maceió – AL
Licio Hernandes Bezerra (UFSC) Florianópolis – SC
Luciano G. Monteiro de Castro (Sistema Elite de Ensino) Rio de Janeiro – RJ
Luzinalva Miranda de Amorim (UFBA) Salvador – BA
Marcelo Dias (Grupo Educacional Etapa) São Paulo – SP
Marcelo Antonio dos Santos FACOS Osório – RS
Marcelo Rufino de Oliveira (Grupo Educacional Ideal) Belém – PA
Newman Simões (Cursinho CLQ Objetivo) Piracicaba – SP
Nivaldo Costa Muniz (UFMA) São Luis – MA
Osnel Broche Cristo (UFLA) Lavras – MG
Uberlândio Batista Severo (UFPB) João Pessoa – PB
Raul Cintra de Negreiros Ribeiro (Colégio Anglo) Atibaia – SP
Reginaldo de Lima Pereira (Escola Técnica Federal de Roraima) Boa Vista – RR
Reinaldo Gen Ichiro Arakaki (UNIFESP) SJ dos Campos – SP
Ricardo Amorim (Centro Educacional Logos) Nova Iguaçu – RJ
Ronaldo Alves Garcia (UFGO) Goiânia – GO
Rogério da Silva Ignácio (Col. Aplic. da UFPE) Recife – PE
Rosangela Ramon (UNOCHAPECÓ) Chapecó – SC
Seme Gebara Neto (UFMG) Belo Horizonte – MG
Tadeu Ferreira Gomes (UEBA) Juazeiro – BA
Tomás Menéndez Rodrigues (U. Federal de Rondônia) Porto Velho – RO
Valdenberg Araújo da Silva (U. Federal de Sergipe) São Cristóvão – SE
Wagner Pereira Lopes (CEFET – GO) Jataí – GO
Wanderson Breder (CEFET – RJ) Nova Friburgo – RJ
William Serafim dos Reis (UFT – TO) Arraias – TO

EUREKA! N°35, 2012

62
CONTEÚDO

XXXIII OLIMPÍADA BRASILEIRA DE MATEMÁTICA 2


Problemas e soluções da Primeira Fase

XXXIII OLIMPÍADA BRASILEIRA DE MATEMÁTICA 15


Problemas e soluções da Segunda Fase

XXXIII OLIMPÍADA BRASILEIRA DE MATEMÁTICA 32


Problemas e soluções da Terceira Fase

XXXIII OLIMPÍADA BRASILEIRA DE MATEMÁTICA 55


Problemas e soluções da Primeira Fase – Nível Universitário

XXXIII OLIMPÍADA BRASILEIRA DE MATEMÁTICA 63


Problemas e soluções da Segunda Fase – Nível Universitário

XXXIII OLIMPÍADA BRASILEIRA DE MATEMÁTICA 79


Premiados

COORDENADORES REGIONAIS 87
Sociedade Brasileira de Matemática

XXXIII OLIMPÍADA BRASILEIRA DE MATEMÁTICA


Problemas e soluções da Primeira Fase

PROBLEMAS – NÍVEL 1

1. Em maio, o valor total da conta de telefone celular de Esmeralda foi R$119,76,


sem os impostos. Esse valor corresponde aos itens: chamadas, acesso à internet,
envio de mensagens. Se ela gastou R$29,90 com acesso à Internet e R$15,50 com o
serviço de envio de mensagens, quanto foi que ela gastou com chamadas?
A) R$74,36 B) R$74,46 C) R$84,36 D) R$89,86 E) R$104,26

2. Numa padaria, uma lata de 200g de achocolatado em pó CHOCOBM custa


R$3,00, uma lata de 400g custa R$5,00 e a de 800g custa R$9,00. Lara precisa de
1,2kg de CHOCOBM para fazer um enorme bolo. Qual das opções a seguir é a
maneira mais econômica de comprar 1,2kg de CHOCOBM nessa padaria?
A) 6 latas de 200g
B) 1 lata de 400g e 1 lata de 800g
C) 4 latas de 200g e 1 lata de 400g
D) 2 latas de 200g e 1 lata de 800g
E) 2 latas de 200g e 2 latas de 400g

3. O gráfico mostra a variação dos preços de alguns produtos alimentícios no


primeiro semestre em uma certa região. Com base no gráfico é possível afirmar
com certeza que

A) o milho sempre foi mais barato que o arroz e o feijão


B) o preço do arroz foi o mais estável no período
C) o feijão sempre custou mais caro que o milho
D) nunca houve dois produtos com o mesmo preço
E) o produto com menor variação de preços foi o feijão

EUREKA! N°36, 2012

2
Sociedade Brasileira de Matemática

4. Uma data curiosa neste ano é o dia 11/11/11, pois o dia, mês e dois últimos
dígitos do ano são iguais. No ano passado, esse padrão aconteceu em 10/10/10.
Quantos dias há desde 10/10/10 até 11/11/11, incluindo o dia 10 e o dia 11?
A) 396 B) 398 C) 400 D) 402 E) 404

5. Luana colou com fita adesiva 6 triângulos equiláteros nos lados de um


hexágono, conforme a figura, obtendo um polígono de 12 lados.

Se ela trocar 3 triângulos por 2 quadrados e 1 pentágono regular, todos com lado de
mesmo tamanho do lado do hexágono, ela vai obter um polígono com quantos
lados?
A) 14 B) 16 C) 17 D) 18 E) 25

6. Por conta de uma erupção de um vulcão, 10% dos voos de um aeroporto foram
cancelados. Dos voos restantes, 20% foram cancelados pela chuva. Que
porcentagem do total de voos deste aeroporto foram cancelados?
A) 28% B) 30% C) 35% D) 38% E) 70%

7. O produto de três números naturais é 105 e a sua soma é a maior possível. Qual é
essa soma?
A) 15 B) 23 C) 27 D) 39 E) 107

8. O retângulo da figura abaixo está dividido em 10 quadrados. As medidas dos


lados de todos os quadrados são números inteiros positivos e são os menores
valores possíveis.

A área desse retângulo é:


A) 180 B) 240 C) 300 D) 360 E) 450

EUREKA! N°36, 2012

3
Sociedade Brasileira de Matemática

9. Numa classe de 36 alunos, todos têm alturas diferentes. O mais baixo dos
meninos é mais alto do que cinco meninas, o segundo menino mais baixo é mais
alto do que seis meninas, o terceiro menino mais baixo é mais alto do que sete
meninas e assim por diante, observando-se que o mais alto dos meninos é mais alto
do que todas as meninas. Quantas meninas há nessa classe?
A) 12 B) 14 C) 16 D) 18 E) 20

10. Esmeralda escolheu quatro números e, ao somar cada um deles à média


aritmética dos outros três, achou os números 60, 64, 68 e 72. Qual é a média
aritmética dos quatro números que ela escolheu no início?
A) 30 B) 31 C) 32 D) 33 E) 66

11. Luca comprou uma revista por R$9,63 e deu uma nota de R$10,00 para pagar.
De quantas maneiras ele pode receber o troco de 37 centavos em moedas, se as
moedas disponíveis no caixa são as de 1, 5, 10 e 25 centavos? Suponha que há
muitas moedas de cada tipo.
A) 10 B) 12 C) 15 D) 24 E) 30

12. Dois triângulos equiláteros de perímetro 36 cm


cada um são sobrepostos de modo que sua interseção
forme um hexágono com pares de lados paralelos,
conforme ilustrado no desenho. Qual é o perímetro
desse hexágono?
A) 12 cm B) 16 cm C) 18 cm
D) 24 cm E) 36 cm

13. Numa corrida com 2011 participantes, Dido chegou à frente do quádruplo do
número de pessoas que chegaram à sua frente. Em que lugar chegou o Dido?
A) 20º B) 42º C) 105º D) 403º E) 1005º

14. Quantos números inteiros positivos menores do que 30 têm exatamente quatro
divisores positivos?
A) 9 B) 10 C) 11 D) 12 E) 13

EUREKA! N°36, 2012

4
Sociedade Brasileira de Matemática

15. Um cubo de madeira, pintado de


vermelho, foi serrado em 27 cubos
menores iguais e as faces desses cubos
ainda não pintadas o foram de branco.
Qual é a razão entre a área da superfície
total pintada em vermelho e a área da
superfície total pintada de branco?

A) 1:2 B) 1:1 C) 2:1 D) 1:3 E) 2:3

16. Esmeralda rasgou uma folha de papel em n pedaços e, em seguida, pegou uma
dessas partes e rasgou-a também em n pedaços. Não satisfeita, pegou uma destas
últimas partes e também a rasgou em n partes. Qual dos números a seguir poderia
ser a quantidade total de pedaços obtida por Esmeralda?
A) 15 B) 18 C) 24 D) 26 E) 28

17. O número n  999999 tem 2011 algarismos e todos iguais a 9. Quantos


algarismos 9 tem o número n 2 ?
A) nenhum B) 11 C) 2010 D) 2011 E) 4022

18. No desenho, três cubos iguais apoiados


sobre uma mesa têm suas faces pintadas com
os números 0, 1, 3, 4, 5 e 9. Qual é a soma
dos números de todas as faces em contacto
com a mesa?
A) 6 B) 8 C) 9 D) 10 E) 12

19. Representamos por n! o produto de todos os inteiros positivos de 1 a n. Por


exemplo, 5! 1  2  3  4  5 . Calculando a soma 1!2!3!4!   2010!2011! ,
qual é o algarismo das unidades do resultado obtido?
A) 1 B) 3 C) 4 D) 7 E) 9

20. Esmeralda tem 11 notas de dois reais, Rosa tem 7 notas de cinco reais e Nelly
tem 3 notas de dez reais. Qual é o menor número possível do total de notas que
devem mudar de mãos de forma que todas as moças fiquem com a mesma quantia?
A) 5 B) 6 C) 7 D) 8 E) 9

EUREKA! N°36, 2012

5
Sociedade Brasileira de Matemática

PROBLEMAS – NÍVEL 2

1. A figura ao lado representa um mapa de


estradas. Os números escritos nas setas
6
indicam quanto de pedágio um viajante 7
1
deve pagar ao passar pela estrada. Todas as
B
estradas são de mão única, como indicam 4 3 5
as setas. Qual o valor mínimo de pedágio A

pago por um viajante que sai da cidade A e 9


8 4
chega na cidade B? 1

A) 11 B) 14 C) 12 D) 10 E) 15

2. O pai de Esmeralda comprou um terreno


retangular de 120 metros de comprimento
por 80 metros de largura. Devido a leis
ambientais, ele deve plantar árvores em
20% do terreno. Ele faz isso plantando-as
em duas faixas de mesma largura nas
laterais do terreno, conforme mostra a
figura. Qual é essa largura?

A) 6m B) 8m C) 10m D) 16m E) 24m

3. Veja o problema Nº 18 do Nível 1

4. Qual é o valor da expressão 201120112  201120032  16  20112007?


A) 2  201120072 B) 2 201120032 C) 2 20112007
2
D) 2 20112003 E) 2 20112011

5. Quantos triângulos não congruentes de perímetro 7 têm todos os lados com


comprimentos inteiros?
A) 1 B) 2 C) 3 D) 4 E) 5

6) Veja o problema Nº 12 do Nível 1

EUREKA! N°36, 2012

6
Sociedade Brasileira de Matemática

7. Qual é o produto da quantidade de vogais pela quantidade de consoantes na


alternativa correta? (Não considere as letras A, B, C, D, E das alternativas na
contagem.)
A) Vinte e quatro. B) Trinta e seis. C) Quarenta e dois.
D) Quarenta e oito. E) Cinquenta e seis.

8. Luca comprou um gibi por R$4,63 e pagou com uma nota de R$5,00. De
quantas maneiras ele pode receber seu troco de 37 centavos, com moedas de 1, 5,
10 e 25 centavos? Suponha que há muitas moedas de cada tipo.
A) 10 B) 12 C) 15 D) 24 E) 25

9. Quantos números inteiros positivos menores que 30 têm exatamente quatro


divisores positivos?
A) 6 B) 7 C) 8 D) 9 E) 10

10. Em um triângulo ABC com m( ABˆ C )  m( BAˆ C )  50 , a bissetriz do ângulo


ACˆ B intersecta o lado AB em D. Seja E o ponto do lado AC tal que
m(CDˆ E )  90 . A medida do ângulo ADˆ E é:
A) 25o B) 30o C) 40o D) 45o E) 50o

13
11. Subtraindo um mesmo número do numerador e do denominador da fração ,
14
14
obtemos a fração . A soma dos algarismos desse número é:
13
A) 1 B) 3 C) 5 D) 7 E) 9

12. Se multiplicarmos todos os inteiros positivos menores que 2011 que não são
múltiplos de 5, qual será o algarismo das unidades do número obtido?
A) 2 B) 4 C) 6 D) 7 E) 8

13. Seja XOY um triângulo retângulo com XOˆ Y  90 . Sejam M e N os pontos
médios de OX e OY, respectivamente. Dado que XN = 19 e YM = 22, determine a
medida do segmento XY.
A) 24 B) 26 C) 28 D) 30 E) 32

EUREKA! N°36, 2012

7
Sociedade Brasileira de Matemática

14. Safira rasgou uma folha de papel em n pedaços e, em seguida, pegou um desses
pedaços e rasgou também em n pedaços. Não satisfeita, ela pegou um desses
últimos pedaços e também rasgou em n pedaços. Qual dos números a seguir pode
representar a quantidade final de pedaços em que Safira rasgou a folha?
A) 15 B) 26 C) 28 D) 33 E) 36

15. Qual é a maior quantidade de números do conjunto 1,2,3,,20 que podemos


escolher de modo que nenhum deles seja o dobro do outro?
A) 10 B) 11 C) 12 D) 13 E) 14

16. Topázio desenhou cada figura a seguir, exceto uma, tirando o lápis do papel
exatamente uma vez e nunca passando pela mesma linha duas vezes. Qual das
figuras abaixo ela não desenhou?

A) B) C) D) E)

17. No triângulo ABC, os pontos D e E


pertencem ao lado BC e são tais que BD = A
BA e CE = CA. Dado que m( DAˆ E )  40 ,
quanto mede, em graus, o ângulo BAˆ C ?
A) 80 B) 90 C) 100
D) 110 E) 120 B E D C

18. Em um teste de múltipla escolha com 24 problemas, cada um pode receber uma
das seguintes pontuações: 4 pontos se a resposta é correta, menos 1 ponto se a
resposta é incorreta e 0 ponto se a resposta está em branco. Sabendo que um
estudante recebeu exatamente 52 pontos, qual o número máximo de respostas
corretas que ele pode ter obtido?
A) 14 B) 15 C) 16 D) 17 E) 18

19. A calculadora de Esmeralda está quebrada: quando ela aperta o botão ,a


calculadora faz, ao acaso, uma das duas seguintes operações: tirar a raiz quadrada
(como deveria fazer) ou dividir o número por 100 (como não deveria fazer).
Esmeralda digitou o número 201120112011 na calculadora e começou a apertar o

EUREKA! N°36, 2012

8
Sociedade Brasileira de Matemática

botão repetidamente. Quantas vezes, no máximo, Esmeralda aperta o botão até


aparecer pela primeira vez um número menor que 2?
A) 2 B) 4 C) 5 D) 8 E) 9

20. Renan quer pintar os quadradinhos da figura ao lado,


usando até três cores diferentes, de modo que
quadradinhos que compartilham um lado em comum
possuam cores diferentes. Quantas pinturas distintas
Renan poderá fazer?
A) 39 B) 246 C) 178 D) 150
E) 120

21. No Planeta Nérdia, existem três espécies de nerds: ET-nerds, UFO-nerds e


OVNI-nerds. A primeira mente quando chove e diz a verdade quando não chove; a
segunda sempre mente; a terceira sempre diz a verdade. Certo dia Bruberson, um
nerd muito camarada, se encontra com quatro nerds. E eles falam:
X: "Hoje está chovendo."
Y: "O nerd que acabou de falar está mentindo."
Z: "Hoje não está chovendo."
W: "O primeiro nerd mentiu ou eu sou um ET-nerds."
Com quantos ET-nerds Bruberson falou no máximo?
A) 0 B) 1 C) 2 D) 3 E) 4

22. Qual é o primeiro dígito não nulo após a vírgula na representação decimal da
1
fração 12 ?
5
A) 1 B) 2 C) 4 D) 5 E) 7

23. Esmeralda tem 2011 balas e quer colocá-las em fileiras, cada fileira com a
mesma quantidade de balas. Ela estabelece que tanto a quantidade de fileiras como
a quantidade de balas em cada fileira devem ser maiores do que 32. Ela sabe que
não consegue fazer isso com 2011 balas, pois 2011 é primo, então faz isso com a
maior quantidade de balas que puder usar e dá as balas que sobrarem para Jade.
Quantas balas Jade ganhou?
A) 1 B) 2 C) 5 D) 7 E) 11

EUREKA! N°36, 2012

9
Sociedade Brasileira de Matemática

24. Uma circunferência passando pelos vértices B, A, D do paralelogramo ABCD


encontra o segmento CD em Q. Sabendo que m( BAˆ D)  60 e AD  10, o
tamanho do segmento CQ é:
5 3
A) 10 B) 20 C) D) 10 3 E) 15
2
25. Rosa escreveu os números de 1 a 6 nos vértices do
hexágono ao lado. Em seguida, para cada lado do
hexágono, ela multiplicou os números escritos nas
suas extremidades, obtendo seis números. Qual o valor
mínimo da soma dos seis números obtidos?
A) 69 B) 58 C) 59 D) 61
E) 57

PROBLEMAS – NÍVEL 3

1. Veja o problema Nº 1 do Nível 2

2. Um cubo de madeira, pintado de vermelho, foi


serrado em 27 cubos menores iguais e as faces desses
cubos ainda não pintadas o foram de branco. Qual é a
razão entre a área da superfície total pintada em
vermelho e a área da superfície total pintada de
branco?
A) 1:2 B) 1:1 C) 2:1
D) 1:3 E) 2:3
3. Numa padaria, uma lata de 200g de achocolatado em pó CHOCOBM custa
R$3,00, uma lata de 400g custa R$5,00 e a de 800g custa R$9,00. Lara precisa de
1,2kg de CHOCOBM para fazer um enorme bolo. Qual das opções a seguir é a
maneira mais econômica de comprar 1,2kg de CHOCOBM nessa padaria?
A) 6 latas de 200g
B) 1 lata de 400g e 1 lata de 800g
C) 4 latas de 200g e 1 lata de 400g
D) 2 latas de 200g e 1 lata de 800g
E) 2 latas de 200g e 2 latas de 400g

4. Os inteiros positivos 30, 72 e N possuem a propriedade de que o produto de


quaisquer dois é divisível pelo terceiro. Qual o menor valor possível de N?
A) 60 B) 30 C) 30  72 D) 360 E) 6

EUREKA! N°36, 2012

10
Sociedade Brasileira de Matemática

5. Numa classe de 36 alunos, todos têm alturas diferentes. O mais baixo dos
meninos é mais alto do que cinco meninas, o segundo menino mais baixo é mais
alto do que seis meninas, o terceiro menino mais baixo é mais alto do que sete
meninas e assim por diante, observando-se que o mais alto dos meninos é mais alto
do que todas as meninas. Quantas meninas há nessa classe?
A) 12 B) 14 C) 16 D) 18 E) 20

6. Qual é o produto da quantidade de vogais pela quantidade de consoantes na


alternativa correta? (Não considere as letras A, B, C, D, E das alternativas na
contagem.)
A) Vinte e quatro. B) Trinta e seis. C) Quarenta e dois.
D) Quarenta e oito. E) Cinquenta e seis.

ab
7. Sendo a e b reais tais que 0 < a  1 e 0 < b  1, o maior valor que pode
ab
assumir é:
1 1 1
A) 0 B) C) D) E) 1
4 3 2

8. Por conta de uma erupção de um vulcão, 10% dos voos de um aeroporto foram
cancelados. Dos voos restantes, 20% foram cancelados pela chuva. Que
porcentagem do total de voos deste aeroporto foram cancelados?
A) 28% B) 30% C) 35% D) 38% E) 70%

9. Qual é o valor da expressão 201120112  201120032  16  20112007?


A) 2  201120072 B) 2 201120032 C) 2 20112007
2
D) 2 20112003 E) 2 20112011

10. Luca comprou uma revista por R$9,63 e deu uma nota de R$10,00 para pagar.
De quantas maneiras ele pode receber o troco de 37 centavos em moedas, se as
moedas disponíveis no caixa são as de 1, 5, 10 e 25 centavos? Suponha que há
muitas moedas de cada tipo.
A) 10 B) 12 C) 15 D) 24 E) 30

11. Quantos números inteiros positivos menores que 30 têm exatamente quatro
divisores positivos?
A) 6 B) 7 C) 8 D) 9 E) 10

EUREKA! N°36, 2012

11
Sociedade Brasileira de Matemática

12. Em um triângulo ABC com m( ABˆ C )  m( BAˆ C )  50 , a bissetriz do ângulo


ACˆ B intersecta o lado AB em D. Seja E o ponto do lado AC tal que
m(CDˆ E )  90 . A medida do ângulo ADˆ E é:
A) 25o B) 30o C) 40o D) 45o E) 50o

13. Esmeralda tem 11 notas de dois reais, Rosa tem 7 notas de cinco reais e Nelly
tem 3 notas de dez reais. Qual é o menor número possível do total de notas que
devem mudar de mãos de forma que todas as moças fiquem com a mesma quantia?
A) 5 B) 6 C) 7 D) 8 E) 9

14. Qual é o primeiro dígito não nulo após a vírgula na representação decimal da
1
fração 12 ?
5
A) 1 B) 2 C) 4 D) 5 E) 7

15. No Planeta Nérdia, existem três espécies de nerds: ET-nerds, UFO-nerds e


OVNI-nerds. A primeira mente quando chove e diz a verdade quando não chove; a
segunda sempre mente; a terceira sempre diz a verdade. Certo dia Bruberson, um
nerd muito camarada, se encontra com quatro nerds. E eles falam:
X: "Hoje está chovendo."
Y: "O nerd que acabou de falar está mentindo."
Z: "Hoje não está chovendo."
W: "O primeiro nerd mentiu ou eu sou um ET-nerds."
Com quantos ET-nerds Bruberson falou no máximo?
A) 0 B) 1 C) 2 D) 3 E) 4

16. Um peso de papel tem a forma de um C


triângulo de lados BC = 6 cm e AB = AC = 5 A
cm e está parcialmente preenchido com água. 5 5 6 5
Quando o peso de papel se apoia sobre o lado 3 h
BC, a água tem uma altura de 3 cm. Qual é a
B 6 C B 5 A
altura da água, em cm, quando o peso de papel
se apoia sobre o lado AB?
4 3 8 18 24
A) B) C) D) E)
3 2 5 5 5

17. O maior inteiro positivo n tal que (2011!)! é divisível por ((n!)!)! é:
A) 3 B) 4 C) 5 D) 6 E) 7

EUREKA! N°36, 2012

12
Sociedade Brasileira de Matemática

18. A calculadora de Esmeralda está quebrada: quando ela aperta o botão ,a


calculadora faz, ao acaso, uma das duas seguintes operações: tirar a raiz quadrada
(como deveria fazer) ou dividir o número por 100 (como não deveria fazer).
Esmeralda digitou o número 201120112011 na calculadora e começou a apertar o
botão repetidamente. Quantas vezes, no máximo, Esmeralda aperta o botão até
aparecer pela primeira vez um número menor que 2?
A) 2 B) 4 C) 5 D) 8 E) 9

19. Existem 3 valores inteiros positivos de n > 1 tais que 10 pode ser escrito como
soma de n inteiros positivos e distintos:
n = 2: 10 = 3 + 7
n = 3: 10 = 2 + 3 + 5
n = 4: 10 = 1 + 2 + 3 + 4
Quantos valores inteiros e positivos de n > 1 existem para os quais é possível
expressar 2011 como soma de n inteiros positivos e distintos?
A) 59 B) 60 C) 61 D) 62 E) 63

20. Qual é a maior quantidade de números do conjunto 1,2,3,,20 que podemos


escolher de modo que nenhum deles seja o dobro do outro?
A) 10 B) 11 C) 12 D) 13 E) 14

21. Seja ABC um triângulo retângulo em A. O B


ponto D pertence ao lado AC e é tal que BD =
CD. Sejam M o ponto médio de BC e N a M
interseção de AM e BD. Sendo N o ponto N
médio de AM, qual a medida, em graus, do
ângulo BCˆ A ? A D C
A) 15 B) 22,5 C) 30 D) 37,5
E) 45


22. Sendo a e b inteiros tais que 1  2 
2011

 a  b 2 , 1 2 
2010
é igual a
A) a  2b  (a  b) 2 B) a  2b  (a  b) 2
C) a  2b  (b  a) 2 D) 2b  a  (b  a) 2
E) a  2b  (a  b) 2

EUREKA! N°36, 2012

13
Sociedade Brasileira de Matemática

1 1 1 1
23. Se a, b e c são inteiros positivos tais que a  b  c e    , qual é
a b c 2011
o menor valor possível de a?
A) 2011 B) 2012 C) 2013 D) 2014 E) 2011 2012

24. Três polígonos regulares, de 8, 12 e 18 lados respectivamente, estão inscritos


em uma mesma circunferência e têm um vértice em comum. Os vértices dos três
polígonos são marcados na circunferência. Quantos vértices distintos foram
marcados?
A) 20 B) 24 C) 26 D) 28 E) 30

25. Seja ABCD um quadrilátero inscritível (ou seja, cujos vértices pertencem a uma
circunferência) com AB = 4, BC = 8 3 , AC = 4 13 e AD = 2 13 . Sendo E a
interseção das diagonais AC e BD, o comprimento do segmento BE é:
12 3 13 3 15 3 16 3
A) B) C) 2 3 D) E)
7 7 7 7

GABARITO
NÍVEL 1 (6º ou 7º anos do Ensino Fundamental)
1) A 6) A 11) D 16) E
2) B 7) E 12) D 17) C
3) E 8) C 13) D 18) D
4) B 9) E 14) A 19) B
5) B 10) D 15) A 20) C

NÍVEL 2 (8º ou 9º anos do Ensino Fundamental)


1) B 6) D 11) B 16) C 21) A
2) E 7) E 12) B 17) D 22) D
3) B 8) B 13) D 18) C 23) D
4) B 9) E 14) D 19) C 24) C
5) C 10) D 15) C 20) C 25) B

NÍVEL 3 (Ensino Médio)


1) E 6) C 11) E 16) B 21) D
2) C 7) D 12) C 17) A 22) A
3) B 8) B 13) B 18) A 23) B
4) D 9) D 14) A 19) B 24) E
5) D 10) D 15) A 20) C 25) B

EUREKA! N°36, 2012

14
Sociedade Brasileira de Matemática

XXXIII OLIMPÍADA BRASILEIRA DE MATEMÁTICA


Problemas e soluções da Segunda Fase

PROBLEMAS – NÍVEL 1 – PARTE A


(Cada problema vale 5 pontos)

01. Na figura, os vértices do retângulo PQRS


pertencem aos lados do retângulo ABCD. Sendo AP
= 3cm, AS = 4 cm, SC = 6 cm e CR = 8 cm, qual é a
área do retângulo PQRS, em cm2?

02. Em cada vértice de um cubo foi escrito um número. Esmeralda calcula a soma
dos números escritos nos vértices de cada face e encontra os números 8, 10, 11, 12,
13 e x. Se a face de soma 8 é oposta à face de soma x, qual é o valor de x?

03. Duas tribos vivem numa ilha. Os da tribo azul só dizem a verdade e os da
vermelha, só mentira. Um dia, 100 pessoas da ilha se reuniram num círculo e um
repórter se dirigiu a cada uma delas, com a pergunta: “O seu vizinho à direita é um
mentiroso?”. Terminada a pesquisa, verificou-se que 48 pessoas responderam
“sim”. No máximo, quantas pessoas da tribo vermelha poderiam estar no círculo?

04. Com cubinhos de mesmo tamanho construiu-se um cubo 4  4  4 . Os cubinhos


são feitos de madeiras diferentes e foram colados assim: cubinhos com três cubos
vizinhos (cubos com faces comuns) pesam 10 gramas, com quatro vizinhos pesam
8 gramas, com cinco vizinhos pesam 6 gramas e com seis vizinhos pesam 4
gramas. Qual é a massa do cubo, em gramas?

05. Quantos números de três algarismos diferentes de zero têm pelo menos dois
algarismos iguais?

06. Dizemos que dois ou mais números são irmãos quando têm exatamente os
mesmos fatores primos. Por exemplo, os números 10  2  5 e 20  22  5 são
irmãos, pois têm 2 e 5 como seus únicos fatores primos. O número 60 tem quantos
irmãos menores do que 1000?

EUREKA! N°36, 2012

15
Sociedade Brasileira de Matemática

PROBLEMAS – NÍVEL 1 – PARTE B


(Cada problema vale 10 pontos)

PROBLEMA 1
A sequência 1, 5, 4, 0, 5, ... é formada pelos algarismos das unidades das somas a
seguir
12  1
12  22  5
12  22  32  14
12  22  32  42  30
12  22  32  42  52  45

12  22  32  42  52   ??
a) Escreva a sequência formada pelos algarismos das unidades das dez primeiras
somas obtidas da forma descrita acima.
b) Qual é o algarismo das unidades da soma 12  22   20112 ?

PROBLEMA 2
Vamos chamar de selo de um número inteiro positivo o par  x; y  no qual x é o
número de divisores positivos desse número menores do que ele e y é a soma
desses divisores. Por exemplo, o selo do número 10 é  3;8  pois o número 10 tem
como divisores menores do que ele os números 1, 2 e 5, cuja soma é 8. Já o selo do
número primo 13 é 1;1 .
a) Qual é o selo do número 9?
b) Qual número tem o selo  2;3 ?
c) Há números cujo selo é  6;m  . Qual é o menor valor possível para m?

PROBLEMA 3
Amarrando um pedaço de barbante em um dos
pregos do seu geoplano, Diamantino consegue
formar quadrados, sem passar o barbante duas
vezes pelo mesmo lado desses quadrados. A figura
ao lado mostra um quadrado obtido desta maneira.

EUREKA! N°36, 2012

16
Sociedade Brasileira de Matemática

A figura abaixo representa de forma simplificada uma parte do geoplano em que


foram obtidos dois quadrados da maneira descrita acima, partindo-se de qualquer
um dos pregos.

a) Desenhe, na parte do geoplano representada ao lado, a


maior quantidade de quadrados iguais que Diamantino
pode obter com um único pedaço de barbante. Coloque
as flechinhas como no exemplo para indicar como foi
colocado o barbante.

b) Diamantino garante que pode obter 11 quadrados no seu geoplano. Mostre que
você também pode obter a mesma quantidade na figura abaixo. Não se esqueça
das flechinhas no desenho.

PROBLEMAS – NÍVEL 2 – PARTE A


(Cada problema vale 5 pontos)

2
01. Sejam a e b números reais não nulos tais que a equação x + ax + b = 0 possui
soluções a e b. Determine a – b.

02. Quantos números compostos de dois algarismos distintos podem ser formados
usando os algarismos 2, 3, 4, 5 e 6?

EUREKA! N°36, 2012

17
Sociedade Brasileira de Matemática

03. O triângulo ABC é retângulo em B. As bissetrizes interna e externa do ângulo


BÂC cortam a reta BC em D e E, respectivamente. Dado que AD = 360 e AE = 480,
determine a medida do lado AB.
4
04. O número 7, quando elevado à quarta potência, termina com 01: 7 = 2401.
Quantos são os números de dois algarismos que, quando elevados à quarta
potência, terminam com 01?

ˆ C mede 30º e o
05. Na figura a seguir, o triângulo ABC é equilátero, o ângulo BD
ângulo ACˆ D mede 70º. Determine, em graus, a medida do ângulo BÂD.
A

D
30º
70º
B C

PROBLEMAS – NÍVEL 2 – PARTE B


(Cada problema vale 10 pontos)

PROBLEMA 1
Inicialmente o número 5 está escrito na tela de um computador. Em qualquer
momento, o número n escrito na tela do computador pode ser trocado por qualquer
número da forma a  b sendo a e b inteiros positivos tais que a + b = n.

a) Mostre como obter o número 19 realizando tais operações.


b) É possível obter o número 2011? Não se esqueça de justificar sua resposta.

PROBLEMA 2
Sejam a, b e c números reais positivos tais que a(b + c) = 152, b(c + a) = 162 e c(a
+ b) = 170. Determine o valor de abc.

EUREKA! N°36, 2012

18
Sociedade Brasileira de Matemática

PROBLEMA 3
Quantos são os pares ordenados (a,b), com a, b inteiros positivos, tais que

a  b  mdc(a, b)  33 ?

PROBLEMA 4
No quadrilátero convexo ABCD, os pontos X e Y dividem o lado AB em três
segmentos iguais enquanto que os pontos Z e T dividem o lado DC em três
segmentos iguais (veja a figura abaixo). Se a área do quadrilátero ABCD é 60,
mostre que a área do quadrilátero XYZT não depende do formato do quadrilátero
ABCD e calcule tal área.
B
Y
X
A

D T Z C

PROBLEMAS – NÍVEL 3 – PARTE A


(Cada problema vale 4 pontos)

01. A equação do segundo grau x – 5x + m = 2011 tem pelo menos uma solução
2

inteira. Qual é o menor valor inteiro positivo possível de m?

02. Uma sequência de letras, com ou sem sentido, é dita alternada quando é
formada alternadamente por consoantes e vogais. Por exemplo, EZEQAF,
MATEMÁTICA, LEGAL e ANIMADA são palavras alternadas, mas DSOIUF,
DINHEIRO e ORDINÁRIO não são. Quantos anagramas da palavra
FELICIDADE (incluindo a palavra FELICIDADE) são sequências alternadas?

03. O ângulo interno do vértice A de um triângulo acutângulo ABC mede 75 graus.


A altura relativa ao vértice A toca o lado BC no ponto D. As distâncias de D ao
vértice B e ao ortocentro do triângulo são ambas iguais a 10 cm. Qual é a área do
triângulo ABC, aproximada para o inteiro mais próximo? Se necessário, use
3  1,732 .

EUREKA! N°36, 2012

19
Sociedade Brasileira de Matemática

04. Qual é o maior valor possível do mdc de dois números distintos pertencentes ao
conjunto {1,2,3,…,2011}?

05. Seja f uma função dos reais não nulos nos reais não nulos tal que
 ( f ( x)  f ( y)  f ( z)) 2  ( f ( x)) 2  ( f ( y)) 2  ( f ( z)) 2 para todos x, y, z
reais não nulos tais que x + y + z = 0;
 f ( x)   f ( x) para todo x real não nulo;
 f(2011) = 1.
Encontre o inteiro mais próximo de f(33).

PROBLEMAS – NÍVEL 3 – PARTE B


(Cada problema vale 10 pontos)

PROBLEMA 1
No triângulo ABC, o ângulo BÂC mede 45º. O círculo de diâmetro BC corta os
lados AB e AC em D e E, respectivamente. Dado que DE = 10, encontre a distância
do ponto médio M de BC à reta DE.

PROBLEMA 2
Encontre todas as soluções reais (x, y, z) do sistema

1
2y  x 
x
1
2z  y 
y
1
2x  z 
z

PROBLEMA 3
Seja P(x) um polinômio de coeficientes inteiros. Sabe-se que P(x) = 2011 tem pelo
menos duas raízes inteiras distintas iguais a 1 e t, e que P(x) = 0 tem pelo menos
uma raiz inteira. Determine todos os possíveis valores de t.

PROBLEMA 4
Esmeralda tem um círculo de cartolina dividido em n setores circulares, numerados
de 1 a n, no sentido horário. De quantas maneiras Esmeralda pode pintar a
cartolina, pintando cada setor com uma cor, tendo disponíveis k cores e de modo
que quaisquer dois setores circulares vizinhos (isto é, que têm um segmento em

EUREKA! N°36, 2012

20
Sociedade Brasileira de Matemática

comum como fronteira) tenham cores diferentes? Note que isso implica que os
setores de números 1 e n devem ter cores diferentes.

SOLUÇÕES NÍVEL 1 – SEGUNDA FASE – PARTE A

Problema 01 02 03 04 05 06
Resposta 0050 0015 0076 0448 0225 0018

01. Por simetria, os triângulos APS e DRQ são congruentes, assim como os
triângulos SCR e QBP. Assim, os lados do retângulo ABDC são AS + SC = 4 + 6 =
10 cm e CR + RD = CR + AP = 8 + 3 = 11 cm. Deste modo, a área do retângulo
PQRS é obtida subtraindo as áreas dos triângulos APS, DRQ, SCR e QBP da área
3 4 68
do retângulo ABDC, ou seja, é 8  11  2   2  50 cm2.
2 2
02. Se em cada face estiver escrita a soma dos números dos vértices
correspondentes a face, então a soma dos números em duas faces opostas é igual a
soma dos números de todos os vértices do cubo. Logo se 8 e x é um par de faces
opostas, então outro par de faces opostas é 10 e 13 e o terceiro par é 11 e 12, para
que 10 + 13 = 11 + 12 = soma dos números em todos os vértices. Portanto
8  x  23  x  15 .

03. Observe que se uma pessoa responde “sim”, então esta pessoa e a da direita não
são da mesma tribo, mas se responder “não”, então ela e a pessoa à sua direita são
da mesma tribo. Assim, se 48 pessoas responderam “sim”, então ao percorrer o
círculo no sentido horário, observaremos 48 trocas de cor da tribo. Para que haja 48
trocas, devem haver pelo menos 24 pessoas da tribo azul e 24 da tribo vermelha
dispostas alternadamente. Como queremos o máximo de pessoas da tribo vermelha,
então podemos colocar as 100 – 24 – 24 = 52 pessoas restantes juntas num mesmo
bloco vermelho, como indicado a seguir:
 
AVAVA VAV
VV VV

24 AV 's 52V 's

Logo há no máximo 100 – 24 = 76 pessoas da tribo vermelha.

04. No cubo 4  4  4 , há 8 cubinhos nos vértices (que tem 3 vizinhos), 2 12  24


cubinhos nas arestas (que tem 4 vizinhos), 4  6  24 cubinhos nas faces (que tem
5 vizinhos) e 8 cubinhos no interior do cubo maior (que tem 6 vizinhos). Assim, o
cubo maior pesa 8 10  24 8  24 6  8  4  448g.

EUREKA! N°36, 2012

21
Sociedade Brasileira de Matemática

05. Há 9  9  9  729 números de três algarismos não nulos. Destes,


9  8  7  504 tem os três algarismos distintos. Portanto, há 729  504  225
números com pelo menos dois algarismos iguais.

06. 60  2 2  3  5 tem os fatores 2, 3 e 5, logo os irmãos de 60 são múltiplos de


1000
2  3  5  30 . Como há    33 múltiplos de 30 menores que 1000, então 60
 30 
tem no máximo 32 irmãos. Destes múltiplos, os que tem outros fatores além de 2, 3
e 5 são 7  30 , 11 30 , 13 30 , 14 30 , 17 30 , 19 30 , 21 30 , 22 30 , 23 30 ,
26 30 , 28 30 , 29 30 , 31 30 e 33 30 . Logo, 60 tem 32  14  18 irmãos.

SOLUÇÕES NÍVEL 1 – SEGUNDA FASE – PARTE B

PROBLEMA 1
a) Para calcular os termos, basta considerar os dígitos das unidades na soma e no
resultado. Assim, como os dígitos das unidades de 12 ,2 2 ,,102 são 1, 4, 9, 6, 5, 6,
9, 4, 1 e 0, então começando por 1, temos: 1, 1 + 4 = 5, 5 + 9 = 14, 4 + 6 = 10, 0 +
5 = 5, 5 + 6 = 11, 1 + 9 = 0, 0 + 4 = 4, 4 + 1 = 5 e 5 + 0 = 5, logo os 10 primeiros
termos da seqüência são 1, 5, 4, 0, 5, 1, 0, 4, 5 e 5.
b) Observe que a partir do 11º termo, vamos começar a somar novamente os dígitos
1, 4, 9, 6, 5, 6, 9, 4, 1 e 0, já que os dígitos das unidades de 112 ,122 ,,202 , são os
mesmos dígitos das unidades de 12 ,22 , ,102 . Assim, na soma
12  2 2    20112 , faremos as somas dos dígitos das unidades de 12 a 102
 2011
 10   201vezes e adicionaremos 1 de 2011 . Assim, o algarismo das unidades
2

 
da soma 12  2 2    20112 é o mesmo algarismo das unidades de
201 1  4  9  6  5  6  9  4  1  0  1  201 25  1  5026, que é 6.

PROBLEMA 2
a) Os divisores positivos de 9 menores que 9 são 1 e 3, logo o selo do número 9 é o
par (2, 4).
b) Observe que todo número inteiro positivo tem 1 como divisor. Como o número
que estamos procurando tem apenas dois divisores menores que ele, 1 terá que ser
um desses divisores e como a soma dos dois divisores é 3, então o outro divisor
deve ser 2. Como não há outros divisores, então o número que procuramos é uma

EUREKA! N°36, 2012

22
Sociedade Brasileira de Matemática

potência de 2, e para ter apenas dois divisores menores que ele próprio, então o
número deve ser 4.
c) Seja n um número com selo (6, m). n possui 7 divisores contando com ele
próprio, logo a única possibilidade é que ele seja da forma p6, com p primo, e m é
igual a 1  p  p 2    p 5 . Para que m seja mínimo, p terá que ser mínimo, logo p
= 2 e m  1  2  2 2    25  63 .

PROBLEMA 3
a) Observe que para cada prego do geoplano deve entrar e sair o mesmo número de
flechas (o barbante ao passar por um prego deve entrar em uma direção e sair em
outra), com exceção de onde começa e termina o barbante. Logo nos pregos onde
não começa ou termina o barbante temos um número par de flechas, metade
entrando e metade saindo. Mas no geoplano 4 4 , há 8 pregos com 3 arestas cada
(os da borda do geoplano), logo em 6 deles haverá pelo menos uma aresta por onde
o barbante não pode passar. No melhor caso, conseguimos fazer com que um
quadrado contenha 2 dessas arestas, assim não poderemos completar 3 quadrados.
Na figura abaixo temos um exemplo onde 9 – 3 = 6 quadrados são formados, em
que o barbante começa no vértice S e termina no vértice T:

b) Uma maneira de construir 11 quadrados com o barbante está descrita abaixo:

EUREKA! N°36, 2012

23
Sociedade Brasileira de Matemática

SOLUÇÕES NÍVEL 2 – SEGUNDA FASE – PARTE A

Problema 01 02 03 04 05 06
Resposta 3 17 ou 20 288 6 0225 100

01. Utilizando o produto das raízes, obtemos:


ab  b
a 1
pois b  0. Utilizando a soma das raízes, obtemos:
a  b  a
1  b  1
a b 3

02. Todos os números terminados em 2, 4, 5 e 6 são compostos. Existem 4  4  16


tais números. Dos números terminados em 3, apenas 63 é composto.
O enunciado apresenta uma ambiguidade. Outra interpretação seria considerar os
números de dois algarismos constituídos por dois números distintos do conjunto
{2,3,4,5,6}. Nesse caso, a resposta correta é 5  4 = 20.
Ambas as respostas devem ser consideradas corretas.

03.
A

90 –   

90 – 
E B y D C

Da semelhança dos triângulos ABD e EAD obtermos:


360 y 3
 y x
480 x 4
Pelo Teorema de Pitágoras no triângulo ABD , temos:
x2  y 2  3602
9 2
x2  x   360 
2

16
x  288

EUREKA! N°36, 2012

24
Sociedade Brasileira de Matemática

04. Um número de dois dígitos (ab) elevado à quarta potência possui a seguinte
forma:
10a  b   104 a 4  4 103 a3b  6 102 a 2b2  4 10ab3  b4
4

Assim, os últimos dois dígitos são determinados por 4 10ab3  b4 . Qualquer


número ímpar diferente de 5 elevado à quarta potência termina em 1. Logo, temos
quatro possibilidades para b:
Se b  1 , para o número 40a  1 terminar em 01 devemos ter a  5.
Se b  3 , para o número 1080a  81 terminar em 01devemos ter a  4 ou a  9 .
Se b  7 , para o número 13720a  2401 terminar em 01 devemos ter a  5 .
Se b  9 , para o número 29160a  6561 terminar em 01 devemos ter a  4 ou a  9.

05. Como BDC mede 30 e B AC é 60, o ponto D está no círculo de centro A e
raio AB. Como o triângulo ACD é isósceles com ângulo da base igual a 70, temos
C AD  40 e BAD  100

SOLUÇÕES NÍVEL 2 – SEGUNDA FASE – PARTE B

SOLUÇÃO DO PROBLEMA 1
a) Uma possível sequência de operações é:
23 3 3 4 5 19 1
5  6  9  20  19

b) Perceba que se é possível obtermos o número também é possível obtermos o


 n 1 1
número n  1 com a operação n  n  1 e consequentemente poderemos obter
todos os inteiros positivos menores que n  1 repetindo essa operação. Então é
suficiente obtermos um inteiro maior que 2011 começando em 5. Uma possível
sequência de operações para isso seria:
23 4 2 4 4 8 8 32 32 1022  2
5  6  8 16  64  1024  2044.

SOLUÇÃO DO PROBLEMA 2
Somando as três equações obtemos: 2ab  2bc  2ca  484
Daí,
ab  242  c  a  b   72,
bc  242  a  b  c   90,
ca  242  b  a  c   80.

EUREKA! N°36, 2012

25
Sociedade Brasileira de Matemática

Logo,  abc   72  90  80 e abc  720.


2

SOLUÇÃO DO PROBLEMA 3
Seja d  mdc  a, b  . Podemos reescrever a equação como:

a b 33
 1  .
d d d

O lado esquerdo é uma soma de números inteiros logo, divide 33. Agora
temos que mdc  a / d , b / d   mdc  a / d ,33 / d  1  mdc b / d ,33 / d  1  1.
Fixado d , é suficiente encontrarmos os pares de inteiros positivos  x, y 
com mdc  x,33 / d  1  1 tais que x  y  33 / d  1 pois daí obteremos
também que mdc  y,33 / d  1  1 e que  a, b    dx, dy  é solução. Vejamos
então as possibilidades para d :

a b 33
Para d  1 e  1  , temos 16 soluções, pois basta escolhermos x
d d d
ímpar.
Para d  3 e x  y  10, temos 4 soluções pois x não pode ser par nem
múltiplo de 5.
Para d  11 e x  y  2, temos 1 solução apenas.
Não podemos ter d  33 , pois a e b são positivos.
Logo, existem 21 pares de soluções.

SOLUÇÃO DO PROBLEMA 4
B
Y
X
A

D T Z C

EUREKA! N°36, 2012

26
Sociedade Brasileira de Matemática

Temos as seguintes relações de áreas:


 ADT    BYC    ADC    ABC    ABCD  20.
1 1
3 3
Portanto a área do quadrilátero  ATCY  é igual à 40. Além disso,

 ATX   YZC    ATY   YTC    ATCY   20.


1 1
2 2
Consequentemente,  XTZY    ATCY    ATX   YZC   20.

Segunda Solução
Sejam P a interseção de AB e DC, a  PA, b  PD, x  AX e y  DT .
Temos:

2 ABCD / senAPD   a  3x  b  3 y   ab  9 xy  3bx  3ay


2 XYZT  / senAPD   a  2 x  b  2 y    a  x  b  y   bx  ay  3xy

Logo a área do quadrilátero  XYZT  é um terço da área do quadrilátero  ABCD.

SOLUÇÕES NÍVEL 3 – SEGUNDA FASE – PARTE A

Problema 01 02 03 04 05
Resposta 0037 3600 0237 1005 0061

01. Como m é inteiro positivo, temos x – 5x < 2011. Sendo x inteiro e 47 – 5  47 <
2 2

2011 < 482 – 5  48, devemos ter x2 – 5x < 2011  x ≤ 47. Assim, o menor valor de
m é 2011 – (472 – 5  47) = 37.

02. As consoantes de FELICIDADE são F, L, C, D, D e as vogais são E, I, I, A, E.


As posições das vogais são as pares ou as ímpares, as consoantes podem se
5!
permutar entre si de  60 maneiras e as vogais podem se permutar de
1!1!1!2!
5!
 30 maneiras. Assim, o total de anagramas alternados de FELICIDADE é
2!1!2!
2  60  30  3600 .

03. Seja H o ortocentro do triângulo ABC. Então BD = HD = 10 cm. Então o


triângulo retângulo BDH é isósceles e, portanto, m( HBˆ D)  45 . Logo,

EUREKA! N°36, 2012

27
Sociedade Brasileira de Matemática

considerando o triângulo retângulo de hipotenusa BC, temos


ˆ  ˆ
m(C )  90  m( HBD)  90  45  45 .
 

Assim, m( DAˆ C )  90  m(Cˆ )  90  45  45 e


m( BAˆ D)  m( Aˆ )  m( DAˆ C )  75  45  30 .
Enfim, m( Bˆ )  90  m( BAˆ D)  90  30  60 .
A

B D C

Assim, AD  BD tg60  10 3 cm e CD = AD = 10 3 cm. Portanto a área de ABC


BC  AD (10  10 3 )10 3
é   150  50 3  150  50 1,732  236,6 cm2, cujo
2 2
valor inteiro mais próximo é 237 cm2.

04. O mdc de dois números é divisor de cada um dos dois números, ou seja, cada
um dos dois números é múltiplo de seu mdc. Logo queremos o maior valor de d
que tem dois múltiplos positivos menores ou iguais a 2011. O maior dos dois
múltiplos de d é maior ou igual a 2d, logo 2d ≤ 2011  d ≤ 1005. Como 1005 e 2 
1005 = 2010 são ambos menores do que 2011, o valor procurado é 1005.

05. A condição ( f ( x)  f ( y)  f ( z )) 2  ( f ( x)) 2  ( f ( y)) 2  ( f ( z )) 2 para x + y +


1 1 1
z = 0 é equivalente a    0 . Como f(–x) = –f(x), sendo
f ( x) f ( y ) f (  x  y )
1 1 1 1
g ( x)  temos    0  g ( x  y )  g ( x)  g ( y ) .
f ( x) f ( x) f ( y ) f ( x  y )
Fazendo y = x, obtemos g(2x) = 2g(x). Fazendo y = 2x, obtemos g(3x) = g(2x) +
g(x) = 2g(x) + g(x) = 3g(x) e, indutivamente, prova-se que g(nx) = ng(x) para n
inteiro positivo. Fazendo x = 1 e n = 2011, obtemos g(2011) = 2011g(1). Como
1 g (2011) 1
g (2011)   1 , temos g (1)   . Enfim, fazendo x = 1 e
f (2011) 2011 2011

EUREKA! N°36, 2012

28
Sociedade Brasileira de Matemática

1 33 1 2011
n = 33, temos g (33)  33 g (1)  33   . Logo f (33)   .
2011 2011 g (33) 33
Como 2011 = 3360 + 31, o inteiro mais próximo de f(33) é 61.

SOLUÇÕES NÍVEL 3 – SEGUNDA FASE – PARTE B


PROBLEMA 1
A

45º
E

D
d

B C
M

 
m(CB )  m( DE )
Temos m( BAˆ C )   2  45  180  m( DMˆ E )  m( DMˆ E )  90 .
2
Logo o triângulo DME é retângulo e, sendo M o centro do círculo, isósceles. Então,
sendo a projeção de M sobre DE o ponto médio de DE e, portanto, circuncentro de
DME. Logo DE  2d  10  2d  d  5 .

PROBLEMA 2
Somando as três equações, obtemos
1 1 1 1 1 1
2x  2 y  2z  x  y  z     x  y  z    .
x y z x y z

1 1
Note que x, y e z têm o mesmo sinal. De fato, se x > 0 então y   x    0 e
2 x
1 1
z   y    0 . Analogamente, se x < 0 então y < 0 e z < 0.
2 y

EUREKA! N°36, 2012

29
Sociedade Brasileira de Matemática

1 1 
  x  1  1 e,
Agora, veja que, pela desigualdade das médias, y   x 

2 x  x

1 1 1
analogamente, x  1 e z  1 . Mas isso implicaria 1, 1 e  1,
x y z
1 1 1 1 1 1
x y z  x  y  z 3 e      3.
x y z x y z
1 1 1
Mas x  y  z    , logo todas as desigualdades anteriores são igualdades,
x y z
ou seja, x  y  z  1 . Lembrando que x, y e z têm o mesmo sinal, as únicas
possibilidades são x = y = z = 1 e x = y = z = –1. Verifica-se facilmente que as duas
possibilidades são realmente soluções.

PROBLEMA 3
Seja Q(x) = P(x) – 2011. Então Q(x) = 0 tem coeficientes inteiros e duas de suas
raízes são 1 e r. Logo Q(x) = (x – 1)(x – r)R(x), sendo R(x) um polinômio de
coeficientes inteiros e, portanto, P(x) = (x – 1)(x – r)R(x) + 2011.
Como P(x) = 0  (x – 1)(x – r)R(x) = –2011 tem soluções inteiras, e R(x) é inteiro
para x inteiro, x – 1 e x – r são dois divisores distintos (não necessariamente
positivos) de 2011. Sendo 2011 primo, cada um desses dois fatores pode ser –2011,
–1, 1 ou 2011, com a única restrição sendo que eles não podem ser –2011 e 2011
simultaneamente. Assim, (x – 1) – (x – r) = r – 1 pode ser igual a 2010, –2010,
2012, –2012, 2 ou –2, ou seja, r pode ser igual a 2011, –2009, 2013, –2011, 3 ou –
1.
PROBLEMA 4
Para n = 1, temos k possibilidades (basta escolher a cor da região 1); para n = 2, há
k(k – 1) possibilidades (k escolhas para a região 1 e k – 1 para a região 2, que deve
ter cor diferente da região 1). Suponha n ≥ 3 e seja an a quantidade desejada de
maneiras de pintar um círculo dividido em n setores, sem que haja setores vizinhos
de mesma cor.

Pintemos a região de qualquer uma das k cores e cada uma das regiões 2, 3, …, n
de qualquer uma das k – 1 cores diferentes da cor da região anterior. Observe a cor
da região n: se a cor é diferente da cor da região 1, obtemos uma pintura válida
com n setores; se a cor é igual à cor da região 1, se juntarmos a região 1 e a região
n obtemos uma pintura válida com n – 1 setores. Note que qualquer pintura com n

EUREKA! N°36, 2012

30
Sociedade Brasileira de Matemática

setores e qualquer pintura com n – 1 setores é obtida de maneira única com esse
procedimento. Assim, an + an–1 = k(k – 1)n–1 para n ≥ 3.

Agora aplique a igualdade repetidas vezes:

an + an–1 = k(k – 1)n–1


–an–1 – an–2 = –k(k – 1)n–2
an–2 + an–3 = k(k – 1)n–3
–an–3 – an–4 = –k(k – 1)n–4

(–1) a3 + (–1) a2 = (–1)n+1k(k – 1)2
n+1 n+1

Somando as n – 2 igualdades, obtemos

an + (–1)n+1a2 = k(k – 1)2((k – 1)n–3 – (k – 1)n–4 + … + (–1)n+1) =


(1) n 1 ((k  1) n 2  1)
k (k  1) 2
 k 1 1
an  (1) n k (k  1)  (k  1) 2 (1) n ((1) n (k  1) n2  1)  (k  1) n  (k  1)(1) n

Logo, co

Você também pode gostar